13 Surgery

Download as pdf or txt
Download as pdf or txt
You are on page 1of 2106

Index

Breast Part 1 - Page 1


Breast Part 2 - Page 33
Thyroid Part 1 - Page 53
Thyroid Part 2 - Page 85
Parathyroid and Adrenals - Page 127
Previous Year Questions (Endocrine Surgery) - Page 181
Liver - Page 258
Portal Hypertension - Page 311
Gallbladder - Page 330
Bile Duct - Page 382
Pancreas Part 1 - Page 413
Pancreas Part 2 - Page 453
Previous Year Questions (Hepatobiliary Pancreatic Surgery) - Page 490
Esophagus - Page 544
Stomach and Duodenum Part 1 - Page 587
Stomach and Duodenum Part 2 - Page 621
Peritoneum - Page 641
Intestinal Obstruction - Page 651
Small Intestine - Page 679
Large Intestine Part 1 - Page 711
Large Intestine Part 2 - Page 729
Inflammatory Bowel Disease - Page 757
Appendix - Page 770
Rectum & Anal Canal - Page 789
Hernia and Abdominal Wall - Page 823
Spleen - Page 859
Previous Year Questions (Gastrointestinal Surgery) - Page 872
Kidney and Ureter - Page 1027
Urinary Bladder - Page 1073
Prostate and Seminal Vesicle - Page 1097
Urethra and Penis - Page 1111
Testis and Scrotum - Page 1140
Previous Year Questions (Urology) - Page 1169
Arterial Disorders - Page 1251
Venous Disorders - Page 1293
Lymphatic System - Page 1321
Index
Thorax and Mediastinum - Page 1334
Previous Year Questions (Cardiothoracic Vascular Surgery) - Page 1390
Burns - Page 1435
Plastic Surgery and Skin Lesions - Page 1458
Wound Healing, Tissue Repair and Scar - Page 1519
Previous Year Questions (Plastic Surgery) - Page 1536
Cerebrovascular Diseases - Page 1557
CNS Tumours - Page 1581
Previous Year Questions (Neurosurgery) - Page 1606
Oral Cavity - Page 1611
Salivary Glands - Page 1654
Neck - Page 1686
Facial Injuries and Abnormalities - Page 1711
Previous Year Questions (Head And Neck) - Page 1732
Oncology - Page 1749
Previous Year Questions (Oncology) - Page 1773
Trauma Part 1 - Page 1780
Trauma Part 2 - Page 1798
Instruments - Page 1819
General Surgery. - Page 1839
Paediatric Surgery - Page 1844
Fluid,Electrolyte,Nutrition - Page 1867
Transplantation - Page 1877
Robotics, Laparoscopy and Bariatric Surgery - Page 1909
Sutures and Anastomosis - Page 1916
Shock - Page 1931
Tubes ,Catheters, Drains - Page 1937
Previous Year Questions (Others) - Page 1969
Previous Year Questions (Clinical Essentials ) - Page 2074
Previous Year Questions (General Surgery) - Page 2078
Previous Year Questions (General) - Page 2096
Breast Part 1
1. What is the most likely diagnosis for a female with a non-tender, firm, and mobile breast mass, along
with tightly arranged ductal epithelial cells and discohesive bare nuclei on FNAC?
(or)
During an OPD visit, a 17-year-old female complained of discomfort and a palpable mass on her left
breast. Upon physical examination, a non-tender, firm and mobile mass of around 3*1.5 cm was found
in the upper outer quadrant of her left breast. She underwent FNAC for a lump and histopathology
revealed tightly arranged ductal epithelial cells with discohesive bare nuclei. What is the most likely
diagnosis?
A. Ductal carcinoma in situ (DCIS)
B. Fibroadenoma
C. Phyllodes tumor
D. Lobular carcinoma in situ (LCIS)
----------------------------------------
2. A 49-year-old post-menopausal female presents to the surgical OPD with complaints of breast
heaviness and purulent discharge from her left breast for the past 20 days. On physical examination, a
mass is felt in the left breast that is hard and immobile on contracting the ipsilateral pectoralis major.
The breast does not fall forward on leaning forward. No skin changes were noted. What is the T stage
of this patient's cancer?
(or)
What is the T stage of breast cancer in a 49-year-old post-menopausal female with complaints of breast
heaviness, purulent discharge, and hard immobile mass on contracting the ipsilateral pectoralis major?
A. T3
B. T4a
C. T4b
D. T4c
----------------------------------------
3. Which structure is preserved during Patey's modified radical mastectomy for invasive ductal
carcinoma in a 65-year-old female?
(or)
A 65-year-old woman attended the outpatient department with a lump measuring 5x4cm in the upper
outer quadrant of her right breast. A fine needle aspiration cytology of the lump revealed invasive ductal
carcinoma. As a result, she underwent a Patey's modified radical mastectomy. During the procedure,
which structure is preserved?
A. Intercostobrachial nerve
B. Pectoralis major
C. Pectoralis minor
D. Axillary fascia
----------------------------------------

1
4. A known case of papillary carcinoma of the right breast had come to your hospital for surgery. A
modified radical mastectomy procedure was done, and on the 2nd postoperative day, the patient
presents with the following finding, as shown in the figure below. Which nerve would have been
damaged during the operative procedure?
(or)
Which nerve, if damaged during a modified radical mastectomy, presents with the findings shown in the
figure below?

A. Musculocutaneous nerve
B. The long thoracic nerve of Bell
C. Intercostobrachial nerve
D. Thoracodorsal nerve
----------------------------------------
5. A 55-year-old postmenopausal woman with stage II breast carcinoma is scheduled for a modified
radical mastectomy. Explain how the Level II axillary lymph node will be identified with respect to the
pectoralis muscle?
(or)
In relation to the pectoralis muscle, where are the Level II axillary lymph nodes identified during a
modified radical mastectomy for stage II breast carcinoma?
A. Lateral to the pectoralis major and minor
B. Behind pectoralis minor
C. Medial to pectoralis minor
D. Superomedial to the pectoralis major
----------------------------------------
6. A 55-year-old female presented with complaints of a lump in her right breast that has been present
for 6 months. The lump gradually increased in size to reach its present size. Physical examination
revealed a mobile lump of size 4x3 cm in the right breast upper outer quadrant. The image below
shows the findings observed during the examination. Which of the following is responsible for this
finding?
(or)
What is the cause for the observation (in the given image) when an old female with a breast lump in the
upper outer quadrant raises her arms?

Page 2

2
A. Edema
B. Contraction of Cooper’s ligaments
C. Subdermal lymphangitis
D. Cancer en-cuirasse
----------------------------------------
7. Which statement accurately describes inflammatory breast carcinoma, diagnosed in a 37-year-old
female with a non-tender, hard, and fixed breast mass in the upper outer quadrant of left the breast?
(or)
A 37-year-old female in OPD complained of discomfort and a palpable mass in her left breast. Physical
examination revealed a non-tender, hard, and fixed mass of around 5 to 2.5 cm in the upper outer
quadrant of her left breast. Histopathology shows dermal lymphatic invasion by the tumor cells. Which
of the following statements is accurate about the patient's condition?
A. It is the most common breast cancer
B. It is common after the age of 55
C. It has the worst prognosis
D. It is non-invasive
----------------------------------------
8. A 57-year-old female presents in OPD with a lump of size 5x4cm in the upper outer quadrant of her
right breast . Physical examination revealed palpable axillary lymph nodes. FNAC of the breast lump
revealed invasion through the basement membrane of the breast duct. She subsequently underwent
Patey’s modified radical mastectomy and chemotherapy. Which of the following is the most important
prognostic factor for breast carcinoma?
(or)
What is the most important prognostic factor for breast carcinoma in a 57-year-old female with invasive
ductal carcinoma, palpable axillary lymph nodes, treated with Patey's modified radical mastectomy and
chemotherapy?
A. Stage
B. DNA content of tumor
C. Histologic subtype
D. Tumor grade

Page 3

3
----------------------------------------
9. A 28-year-old female presents with lump in her right breast. She had no family history of breast or
ovarian cancer. She underwent an ultrasound and a fine needle biopsy that revealed an invasive ductal
carcinoma. Three days following the biopsy results, she found out she was pregnant. She underwent a
lumpectomy and axillary node dissection, placing her at stage IIB (T2N1MX). It was estrogen
receptor-positive (ER1), and HER2 negative (HER2−). Which of the following statements is true about
breast cancer during pregnancy?
(or)
What is a true statement about breast cancer in pregnancy for a 28-year-old female diagnosed with
invasive ductal carcinoma shortly after receiving biopsy results?
A. Most malignant type of breast cancer having worst prognosis
B. Second most common non-gynecologic malignancy associated with pregnancy
C. Ductal carcinoma is MC type, accounting for 75-90% of breast cancer in pregnancy
D. Chemotherapy is contra-indicated during pregnancy
----------------------------------------
10. What is the best management approach for the underlying breast condition of a 54-year-old female
with greenish nipple discharge, bilateral breast pain, nipple retraction, and dilated ducts on
histopathology, with history of bipolar disorder treatment?
(or)
A 54-year-old female diagnosed with bipolar disorder and on treatment for 6 years came to your clinic
with complaints of greenish nipple discharge associated with pain in both breasts. A physical
examination revealed bilateral nipple retraction with little nipple discharge. Histopathology revealed
dilated ducts in both breasts. Which is the best step in managing this patient’s underlying breast
condition?
A. Simple mastectomy
B. Microdochectomy
C. Lobectomy
D. Hadfield's operation
----------------------------------------
11. A 51-year-old postmenopausal female with infiltrating ductal carcinoma was treated with 20mg of
Tamoxifen daily and radiotherapy (as adjuvant therapy). Chronic treatment with Tamoxifen can
increase the risk of which of the following conditions?
(or)
Long-term use of tamoxifen in a 51-year-old postmenopausal female can increase the risk of which of
the following conditions?
A. Epithelial Ovarian carcinoma
B. Endometrial carcinoma
C. Cervical carcinoma
D. Vulvar carcinoma
----------------------------------------

Page 4

4
12. A 44-year-old premenopausal woman recently diagnosed with breast cancer presents with a lump
in her breast and further investigation established a diagnosis of HER-2 negative, ER-negative, and
PR-negative invasive early breast cancer. Partial mastectomy was done to remove the tumor and
advised to receive 6 cycles of chemotherapy. Which of the following is the best chemotherapy regimen
for this patient?
(or)
What is the most appropriate chemotherapy regimen for a 44-year-old premenopausal woman with
HER-2-negative, ER-negative, and PR-negative invasive early breast cancer who underwent a partial
mastectomy?
A. Cyclophosphamide, Adriamycin, 5-Fluorouracil
B. Methotrexate, Cisplatin
C. Cisplatin, Adriamycin, Steroid
D. Methotrexate, Adriamycin, Steroid
----------------------------------------
13. What formula is used to calculate the Nottingham Prognostic Index for breast cancer in a
47-year-old female with invasive ductal carcinoma who underwent a partial mastectomy?
(or)
A 47-year-old female in OPD complained of discomfort and a palpable mass in her left breast. Physical
examination revealed a non-tender, hard, and fixed mass of around 5*2.5 cm in the upper outer
quadrant of her left breast. She underwent FNAC for a lump. Histopathology revealed invasion through
the basement membrane. A partial mastectomy was done, and the plan was to start her on adjuvant
therapy. Which formula is used to calculate the Nottingham Prognostic Index for CA breast?
A. (0.2 x size ) + grade + nodes
B. (0.4 x size ) + grade + nodes
C. (0.6 x size ) + grade + nodes
D. (0.8 x size ) + grade + nodes
----------------------------------------
14. A 64-year-old female presents to the OPD for a regular check-up. A physical examination revealed
a hard mass of almost 2 cm in diameter, but there was no evidence of abnormal nipple discharge, skin
changes, or axillary lymphadenopathy. A mammogram demonstrated a high-density mass with a
microlobulated contour, partially irregular margins, and microcalcifications. Histological examination
revealed an Indian file pattern. Which of the following is the most likely diagnosis?
(or)
What is the most likely diagnosis for a 64-year-old female with a hard breast mass, mammographic
findings of a high-density mass with microlobulated contour, irregular margins, microcalcifications, and
histological examination showing an Indian file pattern?
A. Phyllodes tumor
B. Inflammatory breast carcinoma
C. Invasive lobular carcinoma
D. Fibroadenoma
----------------------------------------

Page 5

5
15. What is the absolute contraindication for breast-conserving surgery?
A. Pregnancy
B. Location of the lump
C. Size of the lump
D. Age of the patient
----------------------------------------
16. What is the most common complication of sentinel lymph node biopsy using a gamma probe in a
64-year-old female with ductal carcinoma in situ?
(or)
A 64-year-old female presents to the OPD for a regular check-up. A physical examination revealed a
hard mass of almost 2 cm in diameter. Histopathology is suggestive of ductal carcinoma in situ. A plan
is made to get her sentinel lymph node biopsied using a gamma probe. What is the most common
complication of this technique?
A. Intercostobrachial nerve palsy
B. Anaphylaxis
C. Necrosis
D. Skin tattooing
----------------------------------------
17. A 29-year-old male presented with complaints of a progressively enlarging left breast lump.
Physical examination revealed a 2.5 cm round mass in the lower inner quadrant of the left breast. USG
showed a suspiciously enlarged lymph node at the left axilla, measuring 1.31 cm in diameter. The
irregular subareolar mass was classified as BIRADS category 5 and diagnosed as invasive ductal
carcinoma with a fine needle aspiration biopsy. Which of the following is true about male breast
cancer?
(or)
What is true about male breast cancer?
A. It accounts for greater than 20% of all cases of breast cancer.
B. Most commonly, it is infiltrating ductal carcinoma.
C. Klinefelter's syndrome is not a risk factor.
D. Exocrine or endocrine testosterone exposure can predispose to it.
----------------------------------------
18. What is the adequate tumor-free margin in breast conservation surgery for a 40-year-old female
with a 2 cm lump in the upper outer quadrant confirmed as an early breast cancer?
(or)
What is the adequate tumor-free margin in breast conservation surgery for a 40-year-old female with a
2 cm lump in the upper outer quadrant confirmed as an early breast cancer?
A. 1 cm
B. 2 cm
C. 3 cm

Page 6

6
D. 1 mm
----------------------------------------
19. What is the most likely diagnosis for a 37-year-old female smoker, with no history of pregnancy
presenting with unilateral greenish breast discharge on palpating the breast and feeling of a mass
beneath the areola, nipple retraction, tender to touch, no induration or edema of the breast seen.
(or)
A 37-year-old female patient visited the OPD clinic with a complaint of breast discharge on one side.
She reported having greenish discharge from the left side of her breast for several months,
accompanied by a feeling of a mass beneath her areola. Physical examination revealed a tender left
breast with greenish discharge and a refracted nipple. The patient has been smoking one pack of
cigarettes a day for the last 15 years and has no H/O pregnancy. What is the most likely diagnosis?
A. Intraductal papilloma
B. Duct ectasia/ periductal mastitis
C. Inflammatory breast cancer
D. Galactorrhea
----------------------------------------
20. A 56-year-old postmenopausal female presented with complaints of a lump in her right breast with
bloody discharge for the last 2 weeks. FNAC is done, which reveals invasive ductal carcinoma with
metastasis to thoracic vertebrae. Which of the following is the most likely explanation for her cancer
spread to vertebrae?
(or)
What is true regarding the metastasis of invasive ductal carcinoma to the thoracic vertebrae in a
56-year-old postmenopausal female?
A. Least common site of occurrence is Lower inner quadrant
B. Batson plexus involvement by posterior intercostal vein
C. Most common histological subtype is Invasive ductal Carcinoma
D. Most common site of metastasis is bone
----------------------------------------
21. A 31-year-old woman who has never given birth visits the breast clinic with a lump in her left breast.
She discovered the lump three months ago during a self-breast examination. The lump grows larger
before her menstrual cycle and decreases in size after it. During the examination, a single, non-tender,
round, rubbery lump of 2 cm with distinct soft margins in her left upper outer quadrant breast is noted.
Which type of calcification is most likely to be seen in a mammogram in this patient?
(or)
Which type of calcification is most likely to be seen in a mammogram of a 31-year-old nulliparous
female diagnosed with fibroadenoma?
A. Popcorn calcification
B. Powdery calcification
C. Pleiomorphic calcification
D. Curvilinear calcification

Page 7

7
----------------------------------------
22. A 45-year-old female presents to the OPD with a lump in her left breast. On examination, a 4 cm
mass is noted in her left breast, with the left axillary lymph nodes fixed and matted. Overlying breast
skin is normal, with no edema, swelling, or ulceration. No metastasis is noted clinically or on imaging.
Which of the following is the most appropriate management method?
(or)
What is the most appropriate management for a 45-year-old female presenting with a 4 cm mass in her
left breast, fixed and matted left axillary lymph nodes, and no evidence of metastasis clinically or on
imaging?
A. Breast conservation surgery + Axillary Lymph Node sampling + Radiotherapy
B. Simple mastectomy + Axillary Lymph Node sampling
C. Neoadjuvant chemotherapy + Modified radical mastectomy + Radiotherapy
D. Halsted radical mastectomy followed by Radiotherapy
----------------------------------------
23. What is the recommended follow-up for a 36-year-old female with (ER+) breast cancer, who
underwent breast conservation surgery and is on tamoxifen chemotherapy?
(or)
A 36-year-old female patient with breast cancer underwent breast conservation surgery and
chemotherapy for a 1.5 × 1.2 cm estrogen receptor-positive (ER+) breast cancer with one positive
axillary lymph node. She is now on the chemotherapy drug Tamoxifen. What is the best recommended
follow-up for this patient?
A. Annual bone scan
B. Assessment of tumor markers 6 monthly
C. Routine clinical examination 3 monthly in 1st year with an annual mammogram
D. Routine clinical examination of 3 monthly and 6 monthly liver function tests
----------------------------------------

Correct Answers
Question Correct Answer

Question 1 2
Question 2 2
Question 3 2
Question 4 2
Question 5 2
Question 6 2
Question 7 3
Question 8 1
Question 9 3

Page 8

8
Question 10 4
Question 11 2
Question 12 1
Question 13 1
Question 14 3
Question 15 1
Question 16 4
Question 17 2
Question 18 1
Question 19 2
Question 20 2
Question 21 1
Question 22 3
Question 23 3

Solution for Question 1:


Option B: Fibroadenoma
• The patient described in the clinical vignette is a classical case of fibroadenoma.
• Fibroadenoma commonly presents at a young age (15-25 years ) and is mostly located in the upper
outer quadrant of the breast.
• Fibroadenomas are a marble-like mass comprising both epithelial and stromal tissues located under
the skin of the breast.
• These are painless, unilateral, firm, rubbery, benign masses with regular borders that are often
variable in size
• Referred to as a breast mouse due to their high mobility
• IOC- FNAC, revealing tightly arranged epithelial cells with discohesive bare nuclei and slit-like ducts.

Page 9

9
Option A: Ductal carcinoma in situ (DCIS)
• It is a malignant mass
• Histopathology may show cribriform, comedo, or papillary growth patterns.
Option C: Phyllodes tumor
• It is usually present in postmenopausal women.
• Leaf-like appearance on cut section. Dual origin of mammary epithelium and connective tissue
(stroma)
Option D: Lobular carcinoma in situ (LCIS)
• It is a malignant mass
• Histopathology will reveal the Indian File Pattern.

Solution for Question 2:


Option B: T4a
• The breast does not fall forward on leaning forward, which is suggestive of chest wall fixity. Hence the
T stage is T4a
Stage
Size of tumor
T1
up to 2 cm
T2
2-5 cm

Page 10

10
T3
>5 cm
T4a
Extension to the chest wall (chest wall is formed by Ribs, Intercostal muscles, and Serratus Anterior)
T4b
• Ulceration
• edema including Peau d'orange
• Satellite nodules
T4c
T4a + T4b
T4d
Inflammatory breast cancer
Not included in T4
• Involvement of the dermis alone
• Nipple retraction/ deviation
• Involvement of Pectoralis major/ Pectoralis minor

Solution for Question 3:


Option B: Pectoralis major
• In Patey’s modified mastectomy, the pectoralis major is preserved.
• En-bloc removal of nipple, areola, skin, breast tissue, breast mass + removal of Level 1, 2, & 3 lymph
nodes + excision of pectoralis minor
• Medial and lateral pectoral nerves are preserved.
• Nerve to Serratus Anterior, Nerve to Latissimus dorsi, Axillary vein, and Cephalic vein are preserved.
Options A: Intercostobrachial nerve
• Intercostobrachial nerve is not preserved during the modified radical mastectomy.
Option C: Pectoralis minor
• Pectoralis minor is not preserved during the modified radical mastectomy.
Option D: Axillary fascia
• Axillary fascia is not preserved during the modified radical mastectomy.

Solution for Question 4:

Page 11

11
Option B: The long thoracic nerve of Bell
• The finding in the image given above is winging of the scapula.
• Winging of the scapula may be seen after mastectomy due to long thoracic nerve injury.
• The long thoracic nerve of the Bell: Course - Close to the chest wall on the axial side of the axilla were
encountered during axillary dissection. Also known as the external respiratory nerve of Bell. Innervates
serratus anterior - It fixes the scapula to the chest wall during shoulder adduction and arm extension.
• Course - Close to the chest wall on the axial side of the axilla were encountered during axillary
dissection.
• Also known as the external respiratory nerve of Bell.
• Innervates serratus anterior - It fixes the scapula to the chest wall during shoulder adduction and arm
extension.
• Course - Close to the chest wall on the axial side of the axilla were encountered during axillary
dissection.
• Also known as the external respiratory nerve of Bell.
• Innervates serratus anterior - It fixes the scapula to the chest wall during shoulder adduction and arm
extension.
Option A: Musculocutaneous nerve
• Supplies the biceps brachii and brachialis muscles
• Its injury can cause weakness in the elbow or shoulder flexion, atrophy of the biceps brachii, and pain
or paresthesia at the lateral forearm.
Option C: Intercostobrachial nerve
• Intercostobrachial nerve supplies sensation to the undersurface of the upper arm and skin of the chest
wall along the posterior axilla. Injury to this nerve results in cutaneous anesthesia in these areas.
Option D: Thoracodorsal nerve
• Encountered during axillary dissection
• Innervates latissimus dorsi
• Injury can cause weakness of internal rotation and abduction of the shoulder.

Solution for Question 5:


Option B: Behind pectoralis minor
• In a modified radical mastectomy, axillary lymph nodes are removed because they are one of the
common sites for the spread of breast cancer.
• Pectoralis minor muscle is used as a surgical landmark to locate these lymph nodes during surgery.
• Level II axillary lymph node is located behind the pectoralis minor.
• lnterpectoral (Rotter's) nodes can be cleared by sweeping finger underneath the pectoralis major
muscle

Page 12

12
Page 13

13
Option A: Lateral to the pectoralis major and minor
• Level II axillary lymph node is located behind pectoralis minor, not lateral to pectoralis major and
minor.
Option C: Medial to pectoralis minor
• The axillary artery is present medial to pectoralis minor.
Option D: Superomedial to the pectoralis major
• Level II axillary lymph node is located behind the pectoralis minor, not superomedial to the pectoralis
major.

Solution for Question 6:


Option B: Contraction of Cooper’s ligaments
• Skin dimpling can be a presentation of inflammatory breast cancer, especially if the lump is growing.
• Cooper ligaments provide shape and structure to the breast as they course from the overlying skin to
the underlying deep fascia. Because these ligaments are anchored into the skin, infiltration of these
ligaments by carcinoma commonly produces tethering, which can cause dimpling.

Page 14

14
Option A: Edema
• Edema of the skin due to lymphatic obstruction by tumor emboli produces a peau d'orange
appearance, not the skin dimpling.

Option C: Subdermal lymphangitis


• Subdermal lymphangitis causes swelling and pain in breast cancer
• It does not cause skin dimpling
Option D: Cancer en-cuirasse
• Infiltration of the breast skin and chest wall with multiple nodules and ulceration by the carcinoma.

Page 15

15
Solution for Question 7:
Option C: It has the worst prognosis
• The signs and symptoms, and the histopathologic findings in the question stem lead to the diagnosis
of Inflammatory breast cancer.
• Inflammatory breast cancer (Mastitis carcinomatosa) is rare, accounting for approximately 1% to 5%
of all breast tumors, it is the most aggressive subtype of breast cancer.
• The hallmark of this cancer is diffuse tumor involvement of the dermal lymphatic channels within the
breast and overlying skin, often without a discrete underlying tumor mass.
• It manifests clinically as erythema, edema, and warmth of the breast as a result of lymphatic
obstruction. Brawny induration and early onset of Peau d' orange noted.
• It is the most malignant and has the worst prognosis of all breast cancers.
• Current treatment approaches emphasize aggressive use of combined-modality treatment, including
neoadjuvant chemotherapy, mastectomy, and radiation therapy, with endocrine therapy for ER-positive
tumors and trastuzumab for HER-2–positive tumors

Page 16

16
Option A: It is the most common breast cancer
• Inflammatory breast cancer is a rare type of breast cancer.
• Most common breast cancer is ductal or lobular carcinoma.
Option B: It is common after the age of 55
• Inflammatory breast cancer occurs in younger women (younger than 40).
Option D: It is non-invasive
• Inflammatory breast carcinoma is very invasive.
• It develops rapidly and spreads to nearby structures

Solution for Question 8:


Option A: Stage
• The histopathologic finding in the question stem confirms the diagnosis of Invasive ductal carcinoma.
• The prognosis of breast cancer depends on various factors, including the number of positive axillary
nodes, tumor size, tumor grade (histologic or nuclear), lymphatic and vascular invasion, and
estrogen-receptor (ER) and progesterone-receptor (PR) positivity.
• The most important prognostic factor for carcinoma breast is the tumor stage.
• The most commonly used is the TNM staging. TNM stands for T= Size of the primary tumor (T) N=
Extent of spread to the lymph nodes (N) M= Presence of distant metastasis (M)
• T= Size of the primary tumor (T)
• N= Extent of spread to the lymph nodes (N)
• M= Presence of distant metastasis (M)
• 8th AJCC TNM classification for breast cancer
• T= Size of the primary tumor (T)

Page 17

17
• N= Extent of spread to the lymph nodes (N)
• M= Presence of distant metastasis (M)
T1
When the size of tumor is up to 2 cm
T2
When the size of tumor is > 2 to 5 cm.
T3
When The size of tumor is > 5 cm.
T4a
Extension to chest wal
T4b
• The skin changes are involved (All the changes should be confined to the same breast): Ulceration
Edema including Peau-D orange Satellite nodules
• Ulceration
• Edema including Peau-D orange
• Satellite nodules
• Ulceration
• Edema including Peau-D orange
• Satellite nodules
T4c
T4a + T4b
T4d
Inflammatory breast cancer (It is the most malignant type of breast cancer).
• Lymph node staging:
N1
Metastasis to ipsilateral axillary lymph node level I and II (That should be mobile
N2a
Is almost similar to N1 with only slight changes. It is metastasis to ipsilateral axillary lymph node level I
and II. It should be fixed/matted instead of mobile.
N2b
Clinically apparent internal mammary nodes.
N3a
Metastasis to ipsilateral infra-clavicular lymph nodes.
N3b
Metastasis to ipsilateral axillary + internal mammary lymph nodes
N3c

Page 18

18
Metastasis to ipsilateral supra clavicular lymph nodes.
M0
Means there is no distant metastasis.
M1
There is distant metastasis.

• Staging
Staging

Stage I
Stage IIA
T0-1 N1 (It means T0 N1 and T1 N1)/T2
Stage IIB
T2 N1 /T3
Stage IIIA
T0-2 N2 (It means T0 N2 , T1 N2 and T2 N2)/T3 N1-2
Stage IIIB
T4 N0-2
Stage IIIC
TANY N3
Stage IV
TANY NANY M
Option B: DNA content of tumor
• DNA content of tumors has prognostic significance in patients with colorectal carcinoma but not in
breast cancer.
Option C: Histologic subtype
• The histologic subtype is the most important prognostic factor for Wilm's tumor, not for breast cancer.
Option D: Tumor grade
• Tumor grade is the most important prognostic factor for soft tissue sarcoma, not for breast cancer.

Solution for Question 9:


Option C: Ductal carcinoma is MC type, accounting for 75-90% of breast cancer in pregnancy
Breast cancer during pregnancy:
• Ductal carcinoma is MC type, accounting for 75–90% of breast cancer in pregnancy.
• It presents as a painless palpable mass with or without nipple discharge

Page 19

19
• Pregnancy induced changes can mask the signs and symptoms, thus leads to a delayed presentation
• Diagnosis: USG and needle biopsy (FNAC) are used for diagnosis
• Treatment: Stage I and II: Mastectomy with axillary dissection Locally advanced breast cancer: NEO
ADJUVANT CHEMOTHERAPY after 1st trimester + MODIFIED RADICAL MASTECTOMY in
2nd trimester + RADIOTHERAPY after delivery
• Stage I and II: Mastectomy with axillary dissection
• Locally advanced breast cancer: NEO ADJUVANT CHEMOTHERAPY after 1st trimester +
MODIFIED RADICAL MASTECTOMY in 2nd trimester + RADIOTHERAPY after delivery
• Stage I and II: Mastectomy with axillary dissection
• Locally advanced breast cancer: NEO ADJUVANT CHEMOTHERAPY after 1st trimester +
MODIFIED RADICAL MASTECTOMY in 2nd trimester + RADIOTHERAPY after delivery

Solution for Question 10:


Option D: Hadfield's operation
• The above-given clinical scenario is a classical presentation of Duct ectasia.
• Mammary duct ectasia is a benign (non-cancerous) breast disorder that develops when the walls of a
milk duct in the breast thicken and widen. This may result in fluid accumulation and duct blockage.
• In duct ectasia, discharge is from multiple ducts.
• The treatment of choice for duct ectasia is Hadfield's operation. HADFIELD's OPERATION: A
peri-areolar incision is made, and a cone of tissue is removed with its apex just deep to the surface of
the nipple and its base on the pectoral fascia.
• HADFIELD's OPERATION: A peri-areolar incision is made, and a cone of tissue is removed with its
apex just deep to the surface of the nipple and its base on the pectoral fascia.
• HADFIELD's OPERATION: A peri-areolar incision is made, and a cone of tissue is removed with its
apex just deep to the surface of the nipple and its base on the pectoral fascia.
Option A: Simple mastectomy
• It is used to remove breast tissue in breast cancer.
Option B: Microdochectomy
• A microdochectomy is a surgical procedure in which a single duct is removed from the nipple-areolar
complex. It is usually undertaken when a patient has nipple discharge from a single duct.
Option C: Lobectomy
• It is a type of conservative breast surgery. A lobectomy removes only the cancerous part of the breast
while leaving the rest of the breast intact.

Solution for Question 11:

Page 20

20
Option B: Endometrial carcinoma
• Tamoxifen belongs to a class of selective estrogen receptor modulators (SERMs)
• Tamoxifen is effective in Estrogen Receptor (ER) positive breast carcinoma, but some ER-negative
tumors also respond to tamoxifen.
• It is a potent antagonist of estrogen in breast cells and has partial agonist activity in the bone and the
uterus.
• It induces pro-estrogenic changes in the endometrium resulting in abnormal growth of the
endometrium, and therefore, it increases the incidence of endometrial cancer.

Option A: Epithelial Ovarian carcinoma


• This is not associated with chronic use of tamoxifen therapy.
Options C: Cervical carcinoma
• Cervical carcinoma is associated with Human papillomavirus (HPV) infection, not tamoxifen therapy.
Options D: Vulvar carcinoma
• This is associated with Human papillomavirus (HPV) infection, not tamoxifen therapy.

Solution for Question 12:


Option A: Cyclophosphamide, Adriamycin, 5-fluorouracil
• For breast carcinoma, the best chemotherapy regimen is Cyclophosphamide, Adriamycin, and
5-Fluorouracil.
• These drugs are used now along with anthracyclines or taxanes as different combination regimes
• For HER-2-positive breast cancer, adding Trastuzumab to polychemotherapy is approved.
• Used for anthracycline and taxane-resistant breast cancer
• Inhibitor of Her-2-neu and EGFR tyrosine kinase
• Second-line Her-2-neu therapy
• Approved for advanced renal cancer and refractory metastatic breast cancer
Option B: Methotrexate, Cisplatin
• This combination has very low efficacy.
• This is not a recommended therapy for breast carcinoma.
Options C: Cisplatin, Adriamycin, Steroid
• Steroid-based chemotherapies are only helpful if the tumor is estrogen or
progesterone-receptor-positive.
• Tumor in the above clinical scenario is negative for both estrogen and progesterone receptors.
Options D: Methotrexate, Adriamycin, Steroid
• Only tumors with estrogen or progesterone receptors will benefit from steroid-based chemotherapy

Page 21

21
• In the clinical condition mentioned above, the tumor tests negative for both estrogen and
progesterone receptors.

Solution for Question 13:


Option A: (0.2 x size ) + grade + nodes
Nottingham Prognostic Index (NPI)
• The Nottingham Prognostic Index (NPI) is used to assess the prognosis of breast cancer patients after
surgery. Three pathological factors: tumor grade, number of affected lymph nodes, and tumor size are
used to compute its value.
• NPI =(0.2 x tumor size in cm ) + Tumor grade + lymph node stage
• NPI less than 3.4 is considered good, and a score >5.4 is considered poor in terms of prognosis
• Used to select patients for adjuvant treatment
Nottingham prognostic Index
Group
Score
15 yr BCSS
Excellent
< 2.4
15%
Good
£ 3.4
20%
Moderate
3.41-5.4
58%
Poor
> 5.4
87%
For NN patients = Score from 2.0 – 5.0
NN 2.0 cm Grade 2 = 3.4 (Good)

Solution for Question 14:


Option C: Invasive lobular carcinoma

Page 22

22
• Invasive lobular carcinoma originates from the terminal duct lobular units and develops only in the
female breast.
• It is mostly multifocal and bilateral
• Increased risk of development of cancer in both breasts
• Clinical presentation: Most present as palpable masses or mammographic densities, a significant
subgroup may have a diffusely invasive pattern without a desmoplastic response and may be clinically
occult.
• Calcifications associated with Invasive lobular carcinoma typically occur in adjacent tissues
(neighborhood calcification)
• The histologic hallmark of Invasive lobular carcinoma is the tendency of tumor cells to invade in linear
strands (Indian file pattern)

Option A: Phyllodes tumor


• It is mostly a benign tumor.
• It is characterized by Leaf-like (phyllode) epithelial pattern formed by an exaggerated intracanalicular
pattern.
Option B: Inflammatory breast carcinoma
• It causes skin changes in the involved breast.
• It presents as a red and swollen breast, mimics mastitis.
• It is characterized by dermal lymphatic invasion by tumor cells on a skin punch biopsy of the affected
breast.
Option D: Fibroadenoma
• It usually occurs in young women and regresses after menopause.
• The tumor consists of proliferating epithelial and mesenchymal components.
• Stroma proliferates around tubular ducts (pericanalicular pattern) or compressed cleft-like ducts
(Intracanalicular pattern).

Page 23

23
Solution for Question 15:
Option A: Pregnancy
• Breast-conserving surgery removes cancer while keeping as much of the breast tissue and shape as
possible. Performed for early invasive breast cancer (Stage I, IIA, IIB)
• Types of breast-conserving surgeries are Excision of lump – Lumpectomy Wide local excision with a 1
cm margin The Whole quadrant excision (Quadrantectomy)
• Excision of lump – Lumpectomy
• Wide local excision with a 1 cm margin
• The Whole quadrant excision (Quadrantectomy)
• Excision of lump – Lumpectomy
• Wide local excision with a 1 cm margin
• The Whole quadrant excision (Quadrantectomy)
• Pregnancy is an absolute contraindication to conservative breast surgery.
• If there is a contraindication for breast conservative surgery, Simple / Oblique / Total mastectomy is
preferred.
Contraindications of Breast conservation surgery:

Option B: Location of the lump


• A centrally located tumor (Tumor around the nipple and areola) is a contraindication to conservative
breast surgery.
• The tumor in this patient is located in the upper outer quadrant of the breast.
Option C: Size of the lump
• The size of the lump is not a contraindication to breast-conserving surgery.

Page 24

24
Option D: Age of the patient
• Age is not a contraindication to breast-conserving surgery
• It can be done at any age.

Solution for Question 16:


Option D: Skin tattooing
• Sentinel lymph node biopsy is a reliable method for evaluating the axillary lymph node status in
early-stage breast cancer patients with non-palpable lymph nodes.
• Indication of SLN biopsy in breast cancer: Clinically non-palpable axillary LN
• A gamma probe is used in the given image during a sentinel lymph node biopsy.
• Skin tattooing is the most common complication of sentinel lymph node biopsy using a gamma probe.
• Other complications of SLN biopsy: Necrosis, Urine discoloration, Anaphylaxis, Injury to
Intercostobranchial nerve

Option A: Intercostobrachial nerve palsy


• Intercostobrachial nerve palsy can occur during sentinel node biopsy, but it is uncommon.
Option B: Anaphylaxis
• It occurs due to the contrast agent used in SLN biopsy
• It can occur during SLN biopsy, but it is still less common than skin tattooing
Option C: Necrosis
• It is a feared complication of SLN biopsy using a gamma probe, but it is less common than tattooing.

Page 25

25
Solution for Question 17:
Option B: Most commonly, it is infiltrating ductal carcinoma
• Breast cancer can occur in men just like women. However, the incidence is low compared to women.
It accounts for less than 2% of cases in men.
• The most common variety is infiltrating ductal carcinoma.
• Lobular carcinoma (both in situ and invasive) is rarely seen due to the absence of lobules in males.
• Risk factors: Increased estrogen levels, Klinefelter's syndrome, BRCA 2 mutation
• In 20% cases gynecomastia precedes male breast cancer but it is not considered a risk factor

Option A: It accounts for greater than 20% of all cases of breast cancer
• The incidence of breast cancer in men is less than 2% of cases of all breast cancer.
Option C: Klinefelter's syndrome is not a risk factor
• Klinefelter syndrome can increase the risk of male breast cancer because it can cause altered
hormone levels (especially the ratio of estrogens to androgens).
Option D: Exocrine or endocrine testosterone exposure can predispose to it
• Excessive estrogen, not testosterone, can predispose men to breast cancer.
• Causes of excess estrogen include testicular disease, infertility, obesity, and cirrhosis.

Solution for Question 18:


Option A: 1 cm
Breast Conservation Surgery:
• Breast conservation surgery (BCS) may be used as part of a treatment plan for breast cancer.
• It is sometimes called partial mastectomy or lumpectomy.

Page 26

26
• 1 cm of tumor-free margin is normally removed with the tumor in BCS.
• It is performed for early invasive breast cancer (Stage I, IIA, IIB).
• Breast conservation involves resection of the primary breast cancer with a margin of
normal-appearing breast tissue, adjuvant radiation therapy(4500 cGy to entire breast), and regional
lymph node status assessment.
• ‘No ink on tumor' is used as a standard for an adequate margin for invasive breast cancer in Breast
Conservation Surgery.

Solution for Question 19:


Option B: Duct ectasia/ periductal mastitis
• The benign (non-cancerous) breast condition occurs due to duct swelling and thickening walls.
• This can cause the breast duct to become blocked and lead to fluid build-up.
• It can be asymptomatic or cause thick, sticky nipple discharge, often greenish, associated with pain
and tenderness.
• It usually causes the nipple to become retracted.
• Warm compresses and antibiotics are used for mild symptoms; however, for severe or persistent
cases, surgery is recommended for the removal of the affected breast.
• It is associated with advancing age, smoking, and a newly inverted nipple.
• It can be diagnosed with an ultrasound(USG) of the breast; however, mammography provides a more
detailed view.

Nipple Discharge
Bloody nipple discharge

Page 27

27
Serous nipple discharge
Greenish, Blackish, Grumous or Pultaceous discharge
• Duct papilloma (MC) Carcinoma Duct ectasia
• Carcinoma
• Duct ectasia
• Carcinoma
• Duct ectasia
• Fibrocystic disease (MC) Carcinoma Duct ectasia
• Carcinoma
• Duct ectasia
• Carcinoma
• Duct ectasia
• Duct ectasia Any kind of nipple discharge is possible for duct ectasia
• Any kind of nipple discharge is possible for duct ectasia
• Any kind of nipple discharge is possible for duct ectasia
Option A: Intraductal papilloma
• Intraductal papilloma is a small, non-cancerous (benign) tumor that grows in the breast's milk duct.
• It presents as a bloody discharge from the affected breast if present close to the nipple.
Option C: Inflammatory breast cancer
• It is a rare and very aggressive form of breast cancer in which cancerous cells block the lymph
vessels of the affected breast and overlying skin via invasion.
• It leads to fluid accumulation in the breast, leading to the edematous pitting appearance of the breast
called Peau d'orange.
Option D: Galactorrhea
• Galactorrhea is milk discharge from the breast outside the normal lactation period.
• It is bilateral and can be due to pituitary tumors or sexual stimulation or as a side effect of various
medications such as antidepressants, antipsychotics, and opioids, among others.

Solution for Question 20:


Option B: Batson plexus involvement by posterior intercostal vein
• Batson venous plexus is a valveless system of paravertebral veins that connect deep pelvic and
thoracic veins to the internal vertebral venous plexus.
• It enters the vertebrae and extends from the base of the skull to the sacrum.
• Posterior intercostal vein →azygous vein/hemizygous system of veins alongside bodies of vertebrae
→Batson vertebral venous plexus →Internal vertebral venous plexus surrounding the spinal cord

Page 28

28
• It provides a route for the spread of breast cancer metastasis to vertebrae, skull, pelvic bones, and
brain.

MC metastasis to
Bone > Lung> Pleura > Soft tissues > Liver
Type of metastatic lesions
Osteolytic and osteoclastic (osteolytic more common)
Batson plexus involvement via
Posterior intercostal vein

Option A: Least common site: Lower inner quadrant


• The most common site for breast cancer is the upper outer quadrant of the breast due to the high
density of glandular tissue present in the upper outer quadrant of the breast.
• The lower inner quadrant is the least common site for breast cancer. However, when present, it
signifies the worst outcome.
Option C: Most common histological subtype is Invasive ductal Carcinoma
• Invasive ductal carcinoma, also known as infiltrativeductal carcinoma (IDC), is breast cancer's most
common histological subtype.
• It roughly accounts for 80% of all breast cancers.
Option D: Most common site of metastasis is bone
• Bone is the most common site of breast cancer metastasis, followed by lung, pleura, soft tissues, and
liver.
• It can spread by the lymphatic system, hematologic system, or by direct invasion.

Page 29

29
Solution for Question 21:
Option A: Popcorn calcification
• Fibroadenoma is a benign solid breast lump most commonly in women aged 15-35.
• It often causes no pain and can be unilateral or bilateral.
• It can feel firm, smooth, and rubbery.
• It has a round shape and, when touched, easily moves with the breast.
• It grows slowly, and over time, it can get bigger with time. However, some adenomas shrink with time
and disappear.
• It may be tender or cause soreness a few days before menses.
• Mammography shows a popcorn appearance of fibroadenoma, especially with involuting
fibroadenoma.

• A fibroadenoma, in the long run, may degenerate and calcify. Initially, there are a few punctuate
peripheral microcalcifications. These, with time, become denser and coarser, finally giving rise to the
classical popcorn appearance.
Option B: Powdery calcification
• Powdery calcification of the breast is seen with fibrocystic disease of the breast on mammography.
• Fibrocystic breast changes develop fluid-filled round or oval sacs (cysts) and more prominent scar-like
(fibrous) tissue, making the breast feel tender, lumpy, or ropy.
Option C: Pleiomorphic calcification
• Pleiomorphic calcification of the breast is seen in carcinoma of the breast.
Option D: Curvilinear calcification
• Fat necrosis leads to curvilinear calcification of affected breast tissue in mammography.
• Fat necrosis is a benign breast condition that most commonly develops after trauma or injury to breast
tissue.

Page 30

30
Solution for Question 22:
Option C: Neoadjuvant chemotherapy + Modified radical mastectomy + Radiotherapy
• The patient has a 4 cm mass in her left breast with clinically palpable axillary lymph nodes and no
metastasis.
• She has locally advanced breast cancer.
• As per TMN staging, she has stage IIIC breast cancer.

• Treatment options for various stages of breast cancer:


• Breast conservation surgery (BCS), If BCS contraindicated: Simple mastectomy + ALNS
• Axillary Lymph Node sampling (ALNS)
• Radiotherapy
• Neoadjuvant chemotherapy
• Modified radical mastectomy
• Radiotherapy
• Hormone therapy preferred over chemotherapy

Solution for Question 23:


Option C: Routine clinical examination 3 monthly in 1st year with an annual mammogram
Breast cancer follow-up
• History and physical examination Every 3-6 months for first 3 years Every 6-12 months for 4th & 5th
year Annually thereafter
• Every 3-6 months for first 3 years
• Every 6-12 months for 4th & 5th year
• Annually thereafter
• Mammography Done annually If radiotherapy is given: Beginning no earlier than 6 months of
radiotherapy
• Done annually
• If radiotherapy is given: Beginning no earlier than 6 months of radiotherapy
• Self Breast examination: Monthly
• Pelvic examination: Annually
• Every 3-6 months for first 3 years
• Every 6-12 months for 4th & 5th year
• Annually thereafter

Page 31

31
• Done annually
• If radiotherapy is given: Beginning no earlier than 6 months of radiotherapy

Option A: Annual bone scan


• An annual bone scan is not recommended for routine surveillance for breast cancer recurrence.
Option B: Assessment of tumor markers 6 monthly
• Six monthly assessments of tumor markers have no proven benefit in leading to early diagnosis of
recurrence.
• Regular assessment of tumor markers is used as a screening tool in postmenopausal ovarian cancer
patients and also for prostate cancer patients
Option D: Routine clinical examination of 3 monthly and 6 monthly liver function tests
• Routine LFTs have no role in the surveillance of breast cancer recurrence.

Page 32

32
Breast Part 2
1. What HER-2/neu immunohistochemistry staining score in invasive ductal carcinoma is likely to
require further confirmatory testing?
(or)
A 63-year-old female comes to the physician complaining of a lump in her right breast that she noticed
4 months ago. After a complete workup, the diagnosis of invasive ductal carcinoma is established.
Before starting treatment, the physician suggests HER-2/neu immunohistochemistry staining. Which of
the following scores on this patient’s test results will most likely need further confirmatory testing?
A. 0
B. 1+
C. 2+
D. 3+
----------------------------------------
2. A 61-year-old smoker presents to his family physician with a palpable mass in his right breast. On
examination, the palpable mass of 3cm in the right breast is noted with no associated nipple discharge
or retraction. Further investigation confirms Ductal carcinoma in situ (DCIS) of the right breast. The
patient is referred for surgical extraction of the tumor. Van Nuys prognostic index (VNPI) determines
whether the patient requires radiotherapy after tumor resection. Which of the following is a component
of Van Nuys prognostic index (NPI)?
(or)
What is a component of Van Nuys prognostic index (NPI) for a 61-year-old female with right-sided
breast cancer referred to the surgeon for tumor removal?
A. Age of patient
B. Smoking status
C. Gender
D. Her-2-neu receptor status
----------------------------------------
3. A 67-year-old female presents in the surgical outpatient department due to the progressively
increasing size of her left breast for the last 2 months. She also complains of itching, heaviness, and an
increased temperature in her left breast. Physical examination reveals widespread erythema, intense
edema, and areolar erosion of the breast, with several enlarged axillary lymph nodes on the left side.
Which of the following is the best treatment for this patient?
(or)
What is the best treatment for a 67-year-old female presenting with increasing size of the left breast,
erythema, edema, areolar erosion, and enlarged axillary lymph nodes?
A. Neoadjuvant chemotherapy followed by modified radical mastectomy and radiotherapy
B. Radical mastectomy followed by chemotherapy
C. Radical mastectomy followed by radiotherapy
D. Chemotherapy followed by hormone therapy
----------------------------------------

33
4. A 50-year-old female presents to the surgical outpatient department with a massively enlarged left
breast, as shown below: On biopsy, the tumor has malignant with cellular atypia and hypercellular
stroma. Which of the following is the best treatment option for this patient?
(or)
What is the best treatment option for a 50-year-old female with a massively enlarged left breast and a
biopsy showing a malignant tumor with cellular atypia and hypercellular stroma?

A. Breast conservation surgery


B. Simple mastectomy
C. Wide local excision
D. Modified radical mastectomy
----------------------------------------
5. A 37-year-old woman presents to the outpatient department with a lump in her left breast that she
noticed one month ago. She has no family history of breast cancer. The mass is rubbery, mobile, and
non-tender to palpation. There are no overlying skin changes or axillary lymphadenopathy. Ultrasound
shows a 2-cm round cyst with smooth margins and no internal septations or solid components. Which
of the following is the most appropriate management of this patient’s condition?
(or)
What is the most appropriate management for a 37-year-old woman with a 2 cm round cyst with
smooth margins and no internal septations on breast ultrasound?
A. Reassurance and re-examination
B. Fluoroscopically guided needle biopsy
C. Aspiration of the cyst with cytologic analysis
D. Immediate excisional biopsy
----------------------------------------
6. A 53-year-old female is admitted to the post-operative floor after a mastectomy for a large malignant
phyllodes tumor. On the 2nd post-operative day, the output from the suction drain is 80ml for 24 hours.
The intern accidentally removed the drain on 3rd postoperative day. On examination, the patient has
oozing serosanguinous fluid from the wound with swelling around the wound. Which of the following is
the definitive management for this patient’s condition?
(or)
What is the definitive management for a 53-year-old female who developed swelling around the wound
with oozing serosanguinous fluid after accidental removal of the drain on the 3rd post-operative day

Page 2

34
following mastectomy?
A. Open incision and saline gauze placement
B. Aspiration under aseptic conditions and pressure dressing
C. Re-insertion of the drain
D. Wait and watch
----------------------------------------
7. A 28-year-old female presents to the surgical outpatient department due to pain in her left breast that
has persisted for 1 week. It is associated with fever and chills. She delivered a healthy baby boy via
simple vaginal delivery (SVD) 3 weeks ago and is currently breastfeeding the baby. Physical
examination reveals engorgement and tenderness with mild fluctuance in the left breast. Which of the
following is the most appropriate next step for managing this patient’s condition?
(or)
What is the next best management step for a 28-year-old lactating mother presenting with pain, fever,
chills, engorgement, tenderness, and mild fluctuance in the left breast?
A. Stop lactation
B. Analgesics and continued breastfeeding
C. Antipyretics
D. Incision and drainage
----------------------------------------
8. Which of the following statements is most accurate regarding breast cancer risk assessment in a
52-year-old female with a positive family history?
(or)
A 52-year-old female presents to the outpatient department for her routine annual evaluation. She has
no active complaints. The patient is concerned about her risk of developing breast cancer as her
58-year-old sister was recently diagnosed with invasive ductal carcinoma. Which of the following
statements is most accurate regarding the breast cancer risk assessment?
A. Claus is the most widely used risk assessment model
B. Claus and Gail are risk assessment models for breast cancer
C. The Gail model gives more information about radiation exposure
D. The Gail model accounts for the risk associated with BRCA1 and BRCA2 gene mutations
----------------------------------------
9. What is the most likely diagnosis for a 57-year-old female presenting with a lump in her left breast
and histopathology showing invasion of the basement membrane by malignant cells?
(or)
A 57-year-old female attends the clinic for an annual physical examination. Her temperature is 37.2°C
(99.0°F), her blood pressure is 130/80 mmHg, her pulse is 84/min, and her respiratory rate is 16/min.
Examination of the breasts reveals a palpable lump in the left breast. The histological examination of a
biopsy specimen shows the invasion of the basement membrane by the tumor cells. Which of the
following is the most likely diagnosis in this patient?
A. Ductal carcinoma in situ

Page 3

35
B. Paget disease of the nipple
C. Lobular carcinoma in situ
D. Medullary carcinoma
----------------------------------------
10. A 52-year-old postmenopausal woman presents to the OPD clinic complaining of occasional bloody
discharge in her left breast for the past 6 months. She reports no sensation of a lump on
self-examination or a history of trauma to the affected breast. The examination is non-specific except
for a bloody crust on her left nipple. Mammography shows casting type microcalcifications. FNAC
shows enlarged ducts filled with necrotic debris surrounded by zero to a few layers of highly atypical
epithelial cells and numerous mitotic figures with no invasion of ducts. Which of the following is the
most likely diagnosis?
(or)
What is the most likely diagnosis for a 52-year-old woman presenting with bloody nipple discharge,
microcalcifications on mammography, and enlarged ducts filled with necrotic debris on FNAC?
A. Ductal carcinoma-in-situ Comedo type
B. Tubular carcinoma of the breast
C. Infiltrating ductal carcinoma of the breast
D. Medullary carcinoma, including atypical medullary lesions
----------------------------------------
11. Ixabepilone (Ixempra), a semi-synthetic epothilone B analogue, is a chemotherapeutic drug given
intravenously (IV). It is a microtubule inhibitor and is usually used with capecitabine or alone as a
second-line drug for locally advanced or metastatic cancer. Its use is indicated in which of the following
cancers?
(or)
In which type of cancer is Ixabepilone (Ixempra) a chemotherapeutic drug indicated?
A. Melanoma
B. Breast carcinoma
C. Oat cell carcinoma
D. Prostate cancer
----------------------------------------
12. What is an indication for adjuvant chemotherapy after mastectomy in a 43-year-old woman with a
history of removal of a 1.5 cm breast tumor and 5 lymph nodes?
(or)
During regular self-examination, a 43-year-old woman felt a small, hard, painless lump in her left
breast. The lump did not disappear during the next few days, so she went to her physician for an
examination. Tests followed, and a biopsy confirmed the presence of malignant cells. The 1.5 cm tumor
and five lymph nodes, two of which contained malignant cells, were removed. No other metastases
were detected. She has a family history of breast cancer on her maternal side. Which of the following is
an indication for adjuvant chemotherapy after mastectomy?
A. Presence of malignant cells in biopsy
B. Size > 1cm & lymph node-positive

Page 4

36
C. Family history of breast cancer
D. Adjuvant chemotherapy is not advisable for this patient
----------------------------------------
13. A 57-year-old female comes to the outpatient department with complaints of redness, warmth, and
a progressive increase in the size of her right breast for the last 4 weeks. After thorough investigations,
the diagnosis of invasive ductal carcinoma with metastases is established. Further testing reveals
estrogen and progesterone receptor positivity as well as HER-2-neu negativity. The patient has been
prescribed a recently approved drug as the initial treatment. Which of the following drugs was most
likely given to this patient?
(or)
Which drug is most likely to be given as the initial treatment to a 57-year-old female with metastatic
invasive ductal carcinoma that is estrogen and progesterone receptor positive and
HER2/neu-negative?
A. Palbociclib
B. Ipatasertib
C. Herceptin
D. Buparlisib
----------------------------------------
14. What is the most likely cause of breast cancer in a 53-year-old menopausal woman with no prior
personal or family history of cancer, presenting with locally advanced breast cancer?
(or)
A 53-year-old menopausal woman presents to the emergency department with complaints of a breast
lump in her left breast that has gradually grown and has been associated with mild pain and itching for
the last 6 months. On examination, a 3 cm breast mass is found in her left breast with axillary lymph
node involvement. Her left breast is mildly swollen and edematous. She is diagnosed with locally
advanced breast cancer. She has no prior personal history of any cancer or family history of breast
cancer or any other cancer in her family. Which of the following is the most likely cause of this patient's
breast cancer?
A. PTEN gene mutation(Cowden disease)
B. Sporadic breast cancer
C. p53 gene mutation
D. BRCA1/ BRCA2 gene mutation
E. Familial breast cancer
----------------------------------------

Correct Answers
Question Correct Answer

Question 1 3
Question 2 1
Question 3 1

Page 5

37
Question 4 2
Question 5 1
Question 6 3
Question 7 4
Question 8 2
Question 9 4
Question 10 1
Question 11 2
Question 12 2
Question 13 1
Question 14 2

Solution for Question 1:


Option C: 2+
• This patient, who has been diagnosed with invasive ductal carcinoma, should have additional testing
for molecular markers.
• Various molecular markers, including those involved in the steroid hormone receptor pathways
(estrogen and progesterone receptors), and molecules involved in the human epidermal growth factor
receptor pathway (HER-2), are routinely tested for targeted treatment.
• HER-2 protein is the product of the ERB-B2 gene and is amplified in almost 20% of breast cancer
cases, therefore its testing is included in the routine pathologic reporting of breast cancer.
• The HER-2 receptor has an extracellular domain and intracellular tyrosine kinase.
• Overexpression of the HER-2 gene product results in the proliferation of tumor cells which
is measured by immunohistochemistry.
• The results are reported on a scale of 0 to 3+.
• A score of 0 or 1+ indicates a negative, whereas a score of 3+ indicates a positive HER-2/neu
amplification status.
• If the test yields a score of 2+, it is considered equivocal, and further testing with fluorescent in situ
hybridization (FISH) is done to confirm a positive or negative status.
• HER-2-positive breast cancer can be treated with trastuzumab, a humanized monoclonal antibody
targeted against the HER-2 receptor.
• Negative HER-2 has worse prognosis.

Page 6

38
Solution for Question 2:
Answer
Option A: Age of patient
• Ductal carcinoma-in-situ is a non-invasive (benign) tumor with the presence of cancer cells inside the
milk duct of breast tissue.
• DCIS is considered the earliest form of breast cancer and confers greater risk of bilateral breast
cancer

• Van Nuy's prognostic index (VNPI) classifies patients with DCIS to guide decisions on the best
treatment option. Size of tumor Resection margin width; the amount of healthy tissue surrounding the

Page 7

39
tumor after removal Age of patient Grade
• Size of tumor
• Resection margin width; the amount of healthy tissue surrounding the tumor after removal
• Age of patient
• Grade
• Size of tumor
• Resection margin width; the amount of healthy tissue surrounding the tumor after removal
• Age of patient
• Grade
Grade and necrosis

Solution for Question 3:


Option A: Neoadjuvant chemotherapy followed by modified radical mastectomy and radiotherapy
• This elderly female with skin thickening, erythema, the warmth of her breast, and palpable axillary
lymph nodes most likely has locally invasive inflammatory breast cancer.
• Inflammatory breast carcinoma accounts for <3% of breast cancers.
• Inflammatory breast cancer is a subtype of invasive ductal carcinoma that does not commonly present
with a palpable lump.
• It usually presents with an inflamed, swollen breast with thickening of the overlying skin (peau
d’orange appearance) due to blockage of dermal lymphatics.

• Inflammatory breast cancer can sometimes be misdiagnosed as mastitis due to its presentation.
• More than 75% of women have palpable axillary lymphadenopathy and distant metastases.

Page 8

40
• This patient most likely has stage IIIB breast cancer (tumor spread to the chest wall and axillary lymph
nodes with no involvement of other organs).
• The treatment of inflammatory breast cancer includes neoadjuvant chemotherapy (given before the
surgery) to reduce tumor size.
• Neoadjuvant chemotherapy is followed by modified radical mastectomy (removal of the entire breast
including skin, nipple, areola, and axillary lymph nodes with sparing of the pectoralis major and minor
muscles).
• Modified radical mastectomy is followed by radiotherapy to remove any residual tumor cells.
Option B: Radical mastectomy followed by chemotherapy
• Radical mastectomy involves the removal of the entire breast, nipple, areola, overlying skin, axillary
lymph nodes, and pectoralis major and minor muscles.
• Patients with inflammatory breast cancer are treated with modified radical mastectomy as it has better
outcomes than radical mastectomy.
Option C: Radical mastectomy followed by radiotherapy
• The treatment of inflammatory breast cancer is initiated with neoadjuvant chemotherapy.
• In addition, modified radical mastectomy is preferred over radical mastectomy for treating
inflammatory breast cancer.
Option D: Chemotherapy followed by hormone therapy
• Chemotherapy and hormonal therapy are the mainstays of treatment for metastatic breast cancer
(that has spread to other body organs).
• This patient has locally invasive inflammatory breast cancer that can be treated with chemotherapy,
surgery, and radiotherapy.
• Surgical treatment is the definitive treatment for stage III breast cancer to remove residual cancer from
the chest wall and axilla.

Solution for Question 4:


Option B: Simple mastectomy
• This patient, with a large, firm breast mass with biopsy findings of cellular atypia and increased
stromal cellularity, most likely has a phyllodes tumor.
• Phyllodes tumor is also known as cystosarcoma phyllodes or serocystic disease of Brodie
(Phyllodes-leaf-like appearance on the cut section).
• It has a dual origin, i.e., mammary epithelium and connective tissue (stroma)
• It can be benign, borderline, or malignant; however, most tumors are benign.
• The cut section shows a leaf-like appearance and cystic areas because of hemorrhage and necrosis.
• Cystosarcoma phyllodes are unique because it does not involve the structures like nipple, chest wall
and breast skin.
• Clinical features of Phyllodes tumor are as follows: It can present with a mobile mass and can mimic
fibroadenoma Affected breasts can be massively enlarged Pressure atrophy/necrosis of skin No skin
fixity (because the tumor does not involve the overlying skin) The skin may appear shiny with visible

Page 9

41
breast veins No fixation of a tumor to the chest wall as the chest wall is not involved. (tumor is mobile
over the chest wall) No nipple retraction/ulceration/deviation
• It can present with a mobile mass and can mimic fibroadenoma
• Affected breasts can be massively enlarged
• Pressure atrophy/necrosis of skin
• No skin fixity (because the tumor does not involve the overlying skin)
• The skin may appear shiny with visible breast veins
• No fixation of a tumor to the chest wall as the chest wall is not involved. (tumor is mobile over the
chest wall)
• No nipple retraction/ulceration/deviation
• The most common route of spread is hematogenous (lymphatic route of spread is not seen)
• The most common site of metastasis is the lungs, although it rarely metastasizes.
• Benign-appearing cystosarcoma phyllodes are treated with wide local excision, including a 2cm
margin of normal tissue.
• Malignant tumors are treated with simple mastectomy with or without reconstruction.
• Modified radical mastectomy is not performed because the tumor is not associated with lymph node
metastasis.
• It can present with a mobile mass and can mimic fibroadenoma
• Affected breasts can be massively enlarged
• Pressure atrophy/necrosis of skin
• No skin fixity (because the tumor does not involve the overlying skin)
• The skin may appear shiny with visible breast veins
• No fixation of a tumor to the chest wall as the chest wall is not involved. (tumor is mobile over the
chest wall)
• No nipple retraction/ulceration/deviation
Option A: Breast conservation surgery
• Breast conservation surgery with wide local excision is indicated for small phyllodes tumors.
• This patient has quite a large tumor which requires a mastectomy.
Option C: Wide local excision
• Wide local excision, including a rim of normal tissue, can be attempted with small tumors having a
small tumor-to-breast ratio.
• This patient’s large phyllodes tumor should be treated with a complete mastectomy.
Option D: Modified radical mastectomy
• Modified radical mastectomy is done for locally invasive breast cancer with axillary lymph node
spread.
• Phyllodes tumors rarely spreads to axillary lymph nodes or other organs and can be treated with
simple mastectomy.

Page 10

42
Solution for Question 5:
Option A: Reassurance and re-examination
• This patient with a palpable mass and ultrasound features of a small cyst with smooth margins and no
solid components or internal septations most likely has a simple cyst.
• A simple cyst does not need immediate intervention, and reassurance and re-examination are
recommended.
• If the ultrasound features of the cyst suggest a low risk of malignancy, fine needle aspiration is done.
• If the cyst disappears with aspiration and the contents are not grossly bloody → fluid does not need to
be sent for cytologic analysis.
• If the lesion does not completely disappear or recurs multiple times (more than twice is a reasonable
rule) after aspiration or the fluid is bloody→ fluid should be sent for cytology to evaluate for solid
components.
• Cysts appear to arise from the destruction and dilation of lobules and terminal ductules in the last
decade of reproductive life.
• Multiple and bilateral cysts can mimic malignancy.
• Direct aspiration or ultrasonography can confirm a palpable mass as a cyst.
• Ultrasound findings of a single, small cyst with smooth borders and no solid components or internal
septations suggest that the cyst is most likely benign.
• On the other hand, multiple cysts may be multilocular, with irregular margins, internal septations, and
solid components.
Option B: Fluoroscopically guided needle biopsy
• Fluoroscopically guided needle biopsy is indicated for evaluating suspicious masses or cysts with an
increased risk of malignancy.
• This patient has a simple cyst that poses a small risk of malignancy and does not require further
evaluation.
Option C: Aspiration of the cyst with cytologic analysis
• If the ultrasound features of a breast cyst suggest a complex cyst with an increased risk of malignancy
or the cyst is large, causing discomfort, aspiration of the cyst can be done.
• If the fluid is bloody or the cyst is complicated, the fluid can be sent for cytologic analysis.
• This patient has a small, simple cyst, and fluid cytologic analysis is unnecessary.
Option D: Immediate excisional biopsy
• Excision of a cyst is indicated if the cytology reveals findings of atypia or if the cysts recur multiple
times.
• This patient most likely has a benign simple cyst that does not need an excisional biopsy.

Solution for Question 6:

Page 11

43
Option C: Re-insertion of the drain
• Seroma is one of the most common and earliest complications of mastectomy or modified radical
mastectomy.
• This patient with wound swelling and serosanguinous discharge after mastectomy has likely
developed a seroma.
• It develops in the dead space under the skin flap created by removing the breast tissue or lymph node
dissection.
• This complication can be prevented using appropriate surgical techniques to minimize the dead space
beneath the skin flaps.
• It can also be prevented by placing suction drains under the skin flap to introduce negative pressure
and reduce the dead space.
• Suction drains are removed when output is <30cc/day for 2 consecutive days to minimize the risk of
seroma formation.
• Accidental removal of the drain results in the formation of seroma and discharge of serous or
serosanguinous fluid.
• Management of seroma involves fluid aspiration; however serial aspirations increase the risk of
infection.
• The definitive management of high-output seroma (as seen in this patient) is the re-insertion of the
suction drain until the daily output is <30ml for 2 days.
Option A: Open incision and saline gauze placement
• Open incision and removal of the seroma may be required for cases that persist for weeks and recur
after serial aspirations.
• This patient does not need an open incision or surgical exploration on her 3rd postoperative day.
Option B: Aspiration under aseptic conditions and pressure dressing
• Aspiration of the seroma and pressure dressing are temporary measures to manage seroma
formation.
• These techniques do not help prevent the recurrence of seroma.
Option D: Wait and watch
• Seromas increase the risk of infections and lymphoedema; hence, the wait-and-watch policy is not a
management option for these patients.
• In addition, seromas do not tend to resolve independently and require intervention.

Solution for Question 7:


Option D: Incision and drainage
• This lactating female patient presenting with fever, breast pain, and localized fluctuation has likely
developed a breast abscess.
• A breast abscess is a common complication of lactational mastitis if left untreated.
• Lactational mastitis is commonly seen in first-time mothers mostly because of a faulty technique of
breastfeeding leading to nipple injury, providing a route for penetration of Staphylococcus aureus from

Page 12

44
the overlying skin.
• Staphylococcus aureus initially causes mastitis presenting with erythema and pain of the affected
breast without fever or fluctuance.
• Mastitis caused by Staphylococcus aureus may often progress to a breast abscess, as seen in this
patient
• Breast abscess presents with: Pain, erythema, and tenderness over the affected breast Fever with
chills and rigors Breast is engorged with the presence of localized fluctuance
• Pain, erythema, and tenderness over the affected breast
• Fever with chills and rigors
• Breast is engorged with the presence of localized fluctuance
• Diagnosis is based on clinical examination and breast ultrasound.
• Treatment of breast abscess includes: Incision and drainage + antibiotics 1st line antibiotic for breast
abscess is cloxacillin/dicloxacillin for 10 -14 days
• Incision and drainage + antibiotics
• 1st line antibiotic for breast abscess is cloxacillin/dicloxacillin for 10 -14 days
• Pain, erythema, and tenderness over the affected breast
• Fever with chills and rigors
• Breast is engorged with the presence of localized fluctuance
• Incision and drainage + antibiotics
• 1st line antibiotic for breast abscess is cloxacillin/dicloxacillin for 10 -14 days
Option A: Stop Lactation
• Lactation is generally continued during the treatment of lactational mastitis or breast abscess and is
safe for the infant.
• Discontinuing lactation can further exacerbate breast engorgement and the resulting pain and
tenderness.
Option B: Analgesics and continued breastfeeding
• Although breastfeeding is continued, analgesia alone is insufficient to treat lactational mastitis or
breast abscess.
• The definitive management of breast abscess includes incision and drainage followed by antibiotics.
Option C: Antipyretics
• Antipyretics are given for fever to patients with breast abscesses.
• It is not the definitive treatment for breast abscesses as it does not treat the cause.
• However, they should be given in combination with antibiotics.
• Incision and drainage are required to remove the infected fluid from the breast.

Solution for Question 8:

Page 13

45
Option B: Claus and Gail are risk assessment models for breast cancer
Cancer and Steroid Hormone Study
• FDR or SDR with breast cancer
• Age of onset in relatives
• Current age
• Age at menarche
• Age at first live birth
• Number of breast biopsies
• Atypical hyperplasia in biopsies
• Race
Incorporates
• Paternal and maternal history
• Age of onset
• Familial history of ovarian carcinoma
• Gail is the most commonly used risk assessment model for evaluating breast cancer risk.
• Claus is another risk assessment model that is less commonly used. It includes more information
about family history and the age of first- and second-degree relatives at the time of diagnosis.
• Claus model provides a breast cancer risk assessment according to the decade of life.
• Gail model does not incorporate radiation exposure in the risk assessment for breast cancer.
• Neither Gail nor Claus model incorporates the less consistently associated risk factors for breast
cancer.
• BRCA1 and BRCA2 gene mutations are not included in the Gail and Claus model.

Solution for Question 9:


Option D: Medullary Carcinoma
• This female patient with a palpable breast mass and a biopsy specimen showing features of
basement membrane invasion most likely has medullary carcinoma of the breast.
• Medullary carcinoma is a histological subtype of invasive ductal carcinoma.
• Invasive ductal carcinoma, also known as infiltrating ductal carcinoma, is the most common type of
breast cancer and accounts for 50% to 70% of invasive breast cancers.
• Invasive lobular carcinoma and mixed lobular and ductal invasive carcinomas account for the rest of
breast cancer cases.
• Medullary carcinoma is characterized by an overall lack of architecture, atypical cells, high-grade
nuclear features, and increased mitotic activity.
• The tumor typically forms sheets of cells arranged in a syncytial pattern surrounded by small
mononuclear infiltrates (lymphocytes and plasma cells).

Page 14

46
• The hallmark of invasive ductal carcinoma is the invasion of the underlying basement membrane by
the tumor cells.
• The incidence of medullary carcinoma is increased in patients with BRCA1 gene mutation.
Option A: Ductal carcinoma in situ
• Ductal carcinoma in situ are premalignant lesions characterized by the growth of malignant cells
within the ducts.
• The growth of malignant cells may cause central necrosis and calcification; however, the tumor cells
have no invasion of the basement membrane.
Option B: Paget Disease of the Nipple
• Paget's disease of the nipple is characterized by ulceration and erythema of the nipple.
• It is the ductal carcinoma in situ (DCIS) that extends up the ducts and involves the skin of the nipple.
• Histological features of DCIS show no invasion of the basement membrane, as seen in this patient.
Option C: Lobular Carcinoma in situ
• Lobular carcinoma in situ is caused by the growth of atypical cells within one or more normal lobules.
• The risk of invasive breast cancer may increase in patients with lobular carcinoma in situ; however,
the tumor cells are confined to a lobule and do not invade the basement membrane.

Solution for Question 10:


Option A: Ductal carcinoma-in-situ Comedo type
• Intraductal carcinoma of the comedo type or comedo carcinoma-in-situ is the high-grade subtype of
ductal carcinoma-in-situ (DCIS).
• It is the most aggressive of intraductal carcinomas.
• Intraductal carcinoma refers to a malignancy of ductal origin that remains enclosed within duct
structures. This non-invasive proliferation can undergo central necrosis, which frequently calcifies to
form the microcalcifications seen on mammography.
• It usually does not have any symptoms and is diagnosed during routine mammography. However, it
can occasionally present with a breast lump or bloody discharge.
• The term comedo refers to a yellowish creamy necrotic material oozing from the ducts that resemble
comedones.
• The abundant necrotic material calcifies and appears as large linear branching calcifications on the
mammogram.
• Tubular, mucinous, and medullary carcinomas are histologic variants of infiltrating ductal cancer and
are all invasive malignancies.
• Infiltrating lobular cancer is a particular histologic variant of invasive breast cancer characterized by
permeation of the stroma with small cells that resemble those found in the breast lobule or acinus.

Page 15

47
Option B: Tubular carcinoma of the breast
• Tubular carcinoma of the breast is a relatively rare low-grade malignant lesion.
• It is composed of well-differentiated tubular structures with open lamina, typically one layer thick,
surrounded by abundant stroma.
Option C: Infiltrating ductal carcinoma of the breast
• It is the most common type of invasive breast cancer.
• It begins in the lining of milk ducts and spreads outside them to surrounding normal tissue.
Option D: Medullary carcinoma, including atypical medullary lesions
• It is a rare subtype of invasive ductal carcinoma.
• It is associated with germline mutations in the BRCA1 gene.

Page 16

48
• It is composed of poorly differentiated cells arranged in sheets without gland formation. Collagenous
stroma is scant, and there are prominent lymphoplasmacytic infiltrates.

Solution for Question 11:


Option B: Breast carcinoma
• Ixabepilone (Ixempra) is a microtubule inhibitor most commonly used for locally advanced or
metastatic breast cancer.
• It is a second-line drug in patients resistant to Adriamycin (doxorubicin) or taxanes.

Chemotherapy regimens used in carcinoma Breast


• Mnemonic- CMF C: Cyclophosphamide M: Methotrexate F: 5-fluorouracil (5FU)
• C: Cyclophosphamide
• M: Methotrexate
• F: 5-fluorouracil (5FU)
• Mnemonic-CAF C: Cyclophosphamide A: Adriamycin (doxorubicin) F: 5FU
• C: Cyclophosphamide
• A: Adriamycin (doxorubicin)
• F: 5FU
• The preferred form of Chemotherapy for CA breast is CAF, Adriamycin, or Anthracycline-based
Chemotherapy (Usually, 6 cycles are given).
• For Adriamycin-resistant breast cancer - Taxanes are given (Docetaxel, Paclitaxel).
• Chemotherapy agent is given in Adriamycin & Taxane resistant breast cancer Ixabepilone.
• Lapatinib is a 2nd line agent for HER-2-NEU-positive breast cancer
• Herceptin /Trastuzumab – 1st line agent for HER-2-NEU-positive breast cancer

Page 17

49
• Sunitinib, a tyrosine kinase inhibitor, is a second-line chemotherapeutic drug in metastatic and
refractory breast cancer.
• C: Cyclophosphamide
• M: Methotrexate
• F: 5-fluorouracil (5FU)
• C: Cyclophosphamide
• A: Adriamycin (doxorubicin)
• F: 5FU
Option A: Melanoma
• Pembrolizumab(Keytruda) is the first-line drug for metastatic or unresectable melanoma
• Ipilimumab, either alone or in combination with nivolumab, is used as a second-line drug for
unresectable or metastatic melanoma.
Option C: Oat cell carcinoma
• Small cell carcinoma of the lung is also called oat cell carcinoma.
• Combination therapy with cisplatin, etoposide, and irinotecan or topotecan alone is used as
second-line therapy for oat cell carcinoma.
Option D: Prostate cancer
• Cabazitaxel, abiraterone acetate, and enzalutamide are second-line drugs for metastatic
Castration-resistant prostate cancer(mCRPC).

Solution for Question 12:


Option B: Size > 1cm & lymph node positive
• Chemotherapy given after mastectomy is called adjuvant Chemotherapy.
• Not all patients with breast cancer need Chemotherapy, but several factors play a role in deciding
Chemotherapy.
• Indications of Adjuvant therapy after MRM: Size > 1cm Lymph node positive Size > 0.5cm and lymph
node negative with adverse prognostic factor like high histological grade, lymphovascular
and HER-2-NEU positive.
• Size > 1cm
• Lymph node positive
• Size > 0.5cm and lymph node negative with adverse prognostic factor like high histological grade,
lymphovascular and HER-2-NEU positive.
• Size > 1cm
• Lymph node positive
• Size > 0.5cm and lymph node negative with adverse prognostic factor like high histological grade,
lymphovascular and HER-2-NEU positive.
Option A: Presence of malignant cells in biopsy

Page 18

50
• The presence of malignant cells in biopsy is indicates neoplasm.
• It gives no clue whether the patient will benefit from adjuvant Chemotherapy.
Option C: Family history of breast cancer
• Multiple family members with a history of breast cancer indicate familial breast cancer.
Option D: Adjuvant Chemotherapy is not advisable for this patient
• Adjuvant Chemotherapy is indicated in patients with positive lymph nodes post-surgery.
• It is also indicated in patients who are at high risk for recurrence.

Solution for Question 13:


Option A: Palbociclib
• This postmenopausal patient who has been diagnosed with metastatic ductal carcinoma, with
estrogen and progesterone receptor-positive and HER-2 receptor-negative status, was most likely
prescribed Palbociclib as the initial treatment.
• Palbociclib is a third-generation non-steroidal aromatase inhibitor that has recently been approved for
adjuvant treatment of metastatic hormone receptor (estrogen and progesterone) positive and HER-2
negative breast cancer.
• The endocrine therapy for estrogen receptor (ER) positive and HER-2 negative breast cancer in
postmenopausal females includes aromatase inhibitors.
• The aromatase inhibitors can be non-steroidal (anastrozole and letrozole) or steroidal (exemestane).
• Palbociclib has recently been approved as adjuvant therapy in combination with letrozole because the
combination has better outcomes than letrozole alone.
• Palbociclib works by inhibiting cyclin-dependent kinase 4/6 (CDK4/CDK6).
• Activation of the CDK4/CDK6 signaling pathway is associated with mediating endocrine resistance.
Blockage of these pathways by Palbociclib results in a better prognosis.
• Other examples of this class of drugs include Ribociclib and Abemaciclib.
• These drugs and fulvestrant (selective estrogen-receptor down regulator) are used.
Option B: Ipatasertib
• Ipatasertib is an AKT (Protein kinase B) inhibitor, a key component of the PI3K/AKT pathway.
• It can be used for the treatment of metastatic triple-negative breast cancer.
Option C: Herceptin
• Herceptin/Trastuzumab is a humanized monoclonal antibody directed against the extracellular
component of the HER-2 receptor.
• It is used for the treatment of HER-2-neu-positive breast cancer.
Option D: Buparlisib
• Buparlisib is a Phosphatidyl-inositol 3 kinase (PI3k) inhibitor that is still in trial stages for treating
metastatic breast cancer.

Page 19

51
Solution for Question 14:
Option B: Sporadic breast cancer
• A sporadic mutation is usually the cause of breast cancer in patients who do not have a family history
of breast cancer or familial cancer syndromes such as Li-Fraumeni syndrome.
• The table given below depicts the most common causes of breast cancer based on their incidence in
the general population
Option A: PTEN gene mutation (Cowden disease)
• Mutations in the PTEN gene cause Cowden disease, a rare inherited disorder characterized by the
presence of many benign growths called hamartomas and an increased risk of cancer.
• Hamartomas form in many body parts, especially the skin, mouth, and gastrointestinal tract. Other
benign tumors may also occur in the breast, thyroid, uterus, soft tissues, and brain.
Option C: p53 gene mutation
• p53 is a tumor suppressor gene.
• Mutations in the p53 gene cause a genetic condition called as Li-Fraumeni syndrome.
• It can cause ovarian, esophageal, colorectal, head and neck, larynx, lung cancers, malignant
melanomas, invasive cervical cancer, and leukemias.
Option D: BRCA1/ BRCA2 gene mutation
• These genes are most commonly affected in hereditary breast and ovarian cancers.
• They usually run in families, and more than one family member is affected by breast or ovarian
cancer.

Page 20

52
Thyroid Part 1
1. A 45 Year-old woman presented to the emergency department complaining of excessive drowsiness
and decreased oral intake for the past 3 days. In the last three years, she has also had difficulty getting
up from the squatting position, combing her hair, and dressing. These difficulties were gradually
progressive, with no diurnal variations or periodic fluctuations. She complained of weight gain in the last
few months, increased sensitivity to cold, skin dryness, hair loss, hoarseness of voice, and easy
fatigability. She had no history of consuming any prescribed or over-the-counter medications. She also
had no family history or any major illnesses. On physical examination, periorbital puffiness and dry,
coarse skin were noted. No tremors or fasciculations were elicited. Which of the following is the most
likely diagnosis?
(or)
What is the most likely diagnosis for a 45-year-old woman presenting with drowsiness, weight gain,
cold intolerance, easy fatigability, periorbital puffiness, and dry coarse skin?
A. Hypothyroidism
B. Hyperthyroidism
C. Anemia of unknown cause
D. Underlying malignancy
----------------------------------------
2. A 75-year-old woman presented to the outpatient clinic complaining of fever for 1 month and
generalized weakness. She also presents with an anterior neck swelling for 4 years which rapidly
progressed in the past 3-4 months, accompanied by pain over the swelling, and hoarseness of voice.
The patient further reported a history of recent weight loss and decreased appetite. She denied
dysphagia, dyspnea, and any history of cancer in the family. Prior medical history was described as
non-significant. Which of the following is the most likely diagnosis?
(or)
What is the most likely diagnosis for a 75-year-old woman presenting with chronic fever, general
weakness, painful anterior neck swelling, hoarseness of voice, and weight loss?
A. Papillary thyroid carcinoma
B. Follicular thyroid carcinoma
C. Medullary thyroid carcinoma
D. Anaplastic thyroid carcinoma
----------------------------------------
3. Which statement is incorrect regarding the diagnosis of a 19-year-old female presenting with midline
neck swelling, raised serum calcitonin, and two hypoechoic lesions in the right lobe of the thyroid with
macrocalcifications on neck ultrasound?
(or)
A 19-year-old woman presented with progressively increasing midline swelling for 2 months. On
physical examination, there was a 3 cm × 2 cm × 2 cm swelling on the right side of the neck, extending
from the midline to the anterior border of the sternocleidomastoid. On palpation, it was firm and mobile.
She also had enlarged lymph nodes in levels II and III on the right , measuring 1 cm × 1 cm each .
Serum calcitonin was 100 times above normal. USG of the neck revealed the presence of two
hypoechoic lesions in the right lobe of the thyroid with macrocalcifications. Which of the following is an

53
incorrect statement regarding the diagnosis?
A. It arises from Parafollicular ‘C’ cells
B. Both hematogenous and lymphatic spread is seen
C. It is dependent on TSH
D. Most cases are sporadic
----------------------------------------
4. A 30-year-old woman was admitted to the post-op unit following a total thyroidectomy for medullary
carcinoma 3 days ago. During the morning round, a junior resident in the surgical department noticed
the following finding, shown in the image: Which of the following is the most probable cause for this
condition?
(or)
What is the most probable cause for the post-operative finding shown in the image of a 30-year-old
woman who underwent total thyroidectomy on the 3rd post-operative day?

A. Hypothyroidism
B. Parathyroid insufficiency
C. Nerve damage during surgery
D. Infection
----------------------------------------
5. A 22-year-old man presented to the casualty complaining of swelling in the right side of the neck
region. After reading some information online, he is worried that he has anaplastic thyroid carcinoma. A
small, solitary nodule in the right lobe of the thyroid was noted on physical examination FNAC shows
follicular adenoma. Which of the following is the best surgery for this condition?
(or)
What is the best surgical option for a 22-year-old man presenting with swelling in the right side of his
neck and a diagnosis of follicular adenoma on FNAC?
A. Enucleation
B. Sub-total thyroidectomy
C. Right Hemithyroidectomy
D. Near-total thyroidectomy
----------------------------------------

Page 2

54
6. A 50-year-old man presented to the physician's clinic complaining of neck swelling for the past few
months alongwith difficulty in swallowing for the past few weeks and wondered if it was due to neck
swelling. On physical examination, the neck revealed a 5 cm thyroid nodule that was mobile and had no
visible skin changes. Which of the following is not true about the thyroid nodule?
(or)
A 50-year-old man presented to the physician due to neck swelling and difficulty swallowing.
Examination revealed a 5 cm, mobile thyroid nodule. Which of the following is not true about the thyroid
nodule?
A. FNAC is the investigation of choice
B. FNAC cannot distinguish follicular adenoma from carcinoma
C. Managed by total thyroidectomy
D. Cold nodules are diagnostic of malignancy
----------------------------------------
7. Which of the following conditions does not show Psammoma bodies?
(or)
A 35-year-old woman presented to the physician's clinic complaining of neck swelling for the past 2
months. On physical examination, the swelling was mobile and moved with swallowing. Further
investigations included a histopathological examination, which revealed concentric calcifications called
Psammoma bodies. Which of the following does not show Psammoma bodies?
A. Follicular carcinoma of the thyroid
B. Papillary carcinoma of the thyroid
C. Meningioma
D. Serous cystadenocarcinoma of the ovary
----------------------------------------
8. What is the best treatment option for a 48-year-old man presenting with a painless anterior neck
mass, FNAC of which showed papillary projection, optically clear nuclei, and dystrophic calcification?
(or)
A 48-year-old man presented with an anterior neck mass. On physical examination, the mass was
well-demarcated, fixed, and hard, and it measured about 3.5 cm in its largest diameter with other
smaller nodules. The results of thyroid function tests were normal. FNAC showed the presence of
papillary projection, optically clear nuclei, and dystrophic calcification. Which of the following would be
the best treatment for this patient?
A. Hemithyroidectomy
B. Subtotal thyroidectomy with modified neck dissection
C. Total thyroidectomy + removal of an enlarged central group of lymph node ± Ipsilateral modified
radical neck dissection MRND (if any LN is positive)
D. Hemithyroidectomy with modified neck dissection
----------------------------------------
9. A 9-year-old boy with no significant past medical history presented with weight loss, and heat
intolerance for the past month. Family history is significant for thyroid disease in both grandmothers.
The clinician ordered thyroid function tests, including free T4, free T3, free TSH, anti-TSH receptor

Page 3

55
antibodies, anti-thyroglobulin, and anti-thyroid peroxidase antibodies. The laboratory findings confirmed
the clinical impression and a diagnosis of Graves' disease was made. Which of the following
characteristics will not be seen in this patient?
(or)
Which of the following characteristics will not be seen in a 9-year-old male diagnosed with Graves'
disease presenting with weight loss and heat intolerance?
A. Enophthalmos
B. Gynecomastia
C. Subperiosteal new bone formation in metacarpals
D. Pretibial myxedema
----------------------------------------
10. Which tracheal rings will be found underneath the isthmus of the thyroid gland during a total
thyroidectomy ?
(or)
A 40-year-old woman with multinodular goiter was taken up for total thyroidectomy. During the
procedure, after ligation of the vessels (superior and inferior pedicles and middle thyroid vein) and
preservation of the nerves (external and recurrent laryngeal nerves), the thyroid gland was removed.
Which of the following tracheal rings was found underneath the isthmus of the thyroid gland?
A. 1st tracheal cartilage
B. 1st and 2nd tracheal cartilages
C. 2nd, 3rd and 4th tracheal cartilage
D. 3rd and 4th tracheal cartilages
----------------------------------------
11. A 38-year-old woman was transferred to your hospital for further evaluation of fever and anterior
neck pain that had persisted for more than one month. Four weeks before admission, she had a sore
throat and otalgia, symptoms that had improved after medication at private clinics. Two days later, fever
and anterior neck pain developed. On physical examination, the swelling was non-tender, and no signs
of discharge/suppuration. The patient was treated with antipyretics, but her fever persisted.Lab findings
indicate low TSH, elevated free T4 and T3 .FNAC showed follicular destruction and giant cell formation.
What is the most likely diagnosis?
(or)
A 38-year-old woman presented with an upper respiratory tract infection four weeks ago followed by
fever and anterior neck painful swelling.Lab findings are low TSH, elevated free T4 and T3 .FNAC
showed follicular destruction and giant cell formation. What is the most likely diagnosis?
A. Acute thyroiditis
B. Subacute thyroiditis
C. Tubercular lymphadenitis
D. Hashimoto’s thyroiditis
----------------------------------------
12. What is the most appropriate management for a 35-year-old woman with thyrotoxicosis who
developed tachycardia, hypotension, difficulty in breathing, and tense neck swelling beneath the

Page 4

56
stitches two hours after total thyroidectomy?
(or)
A 35-year-old woman with thyrotoxicosis underwent total thyroidectomy . Two hours after the total
thyroidectomy, she rapidly became agitated and complained of increasing difficulty breathing. Her pulse
rate increased, and her BP dropped to 90/60 mm Hg. Also, her neck was found to be tensely swollen
beneath the stitches. Which of the following is the most appropriate management strategy in this case?
A. Intranasal oxygen
B. Passing an endotracheal tube in the ward
C. Removing sutures from all layers in the ward and evacuation of hematoma
D. Immediate transfer of the patient to the operation theatre for tracheostomy
----------------------------------------
13. What is the most likely diagnosis for a 30-year-old female presenting with neck swelling, decreased
T4, and elevated TSH levels?
(or)
A 30-year-old woman presented to the physician's clinic complaining of swelling in front of her neck for
the past month. The laboratory investigation of the patient showed decreased T4 and elevated TSH
levels. Which of the following is the most likely diagnosis?
A. Graves' disease
B. Hashimoto's disease
C. Pituitary failure
D. Hypothalamic failure
----------------------------------------
14. A 59-year-old woman presented to the physician's office complaining of swelling in the neck for 5
years that suddenly increased in size. Clinical examination showed a midline neck swelling of about 3
cm × 2 cm × 2 cm moving with deglutition. No evidence of compression of the trachea, esophagus, or
RLN. The cervical lymph nodes were not palpable. A biopsy was taken and sent to you. How will you
differentiate follicular carcinoma of the thyroid from a follicular adenoma?
(or)
A 59-year-old woman presented to the surgeon for neck swelling. Clinical examination revealed a
midline neck swelling, that moves with deglutition. How will you differentiate follicular carcinoma of the
thyroid from a follicular adenoma?
A. Hurthle cell change
B. Lining of tall columnar and cuboidal cells
C. Vascular invasion
D. Nuclear features
----------------------------------------
15. A 59-year-old woman presented to the physician's office complaining of a feeling of swollen and
enlarged neck for the past month. The patient had no previous history of such swelling and no family
history of thyroid-related disorders or other neoplasms. An endocrine examination showed that the
patient was euthyroid, and there were no thyroid autoantibodies.On ultrasound, a diffusely enlarged left
thyroid tumor showing a heterogenous pattern without clear capsule formation is seen in the cervical

Page 5

57
region. Which of the following is the best investigation for diagnosing this disease?
(or)
What is the best investigation for diagnosing a malignant growth on the left side of the thyroid in a
59-year-old woman with neck swelling, euthyroid status, and no thyroid autoantibodies?
A. Biopsy
B. FNAC
C. CT scan
D. MRI
----------------------------------------
16. A 45-year-old man presented with a headache and flushing. He had a brother who had died of
medullary carcinoma of the thyroid. Which of the following investigations would be required to diagnose
this patient?
(or)
What investigation would be required to diagnose a 45-year-old man presenting with headache,
diaphoresis, flushing, palpitations, and a positive family history of medullary carcinoma of the thyroid?
A. Chest X-ray
B. Measurement of 5-HIAA
C. Fractionated plasma metanephrines
D. Intravenous pyelography
----------------------------------------
17. A 55-year-old male with a family history of thyroid cancer came with complaints of swelling in the
front of the neck. During examination, two nodules were discovered in the right lobe of the thyroid, as
well as multiple cervical nodes of size 1x1 cm in levels II, III, and IV. Laboratory evaluation revealed
increased calcitonin and CEA levels. Which of the following is the most appropriate treatment option for
this patient?
(or)
What is the most appropriate treatment option for a 55-year-old male with a family history of thyroid
cancer, two nodules in the right lobe of the thyroid,multiple lymph nodes and increased calcitonin and
CEA levels?
A. Surgery and Radiotherapy
B. Radiotherapy and Chemotherapy
C. Surgery only
D. Radioactive ablation
----------------------------------------
18. A 30-year-old man presented to the physician's clinic complaining of a nodule in his neck for the
past 2 weeks. On physical examination, there was a neck nodule < 3 cm in size with two lymph nodes
palpable in the neck along with lung micro-metastasis as observed in a CT scan. After the biopsy, he
was later diagnosed with papillary thyroid cancer. Based on these findings, which of the following is the
stage of the disease according to AJCC 8th edition?
(or)

Page 6

58
What is the staging according to AJCC 8th edition for a 30-year-old man with papillary thyroid cancer
presenting with a neck nodule < 3 cm in size, two palpable lymph nodes, and lung micro-metastasis on
CT?
A. Stage I
B. Stage II
C. Stage III
D. Stage IV
----------------------------------------

Correct Answers
Question Correct Answer

Question 1 1
Question 2 4
Question 3 3
Question 4 2
Question 5 3
Question 6 4
Question 7 1
Question 8 3
Question 9 1
Question 10 3
Question 11 2
Question 12 3
Question 13 2
Question 14 3
Question 15 1
Question 16 3
Question 17 3
Question 18 2

Solution for Question 1:


Option A: Hypothyroidism
• These symptoms are suggestive of hypothyroidism. A deficiency in circulating levels of thyroid
hormone leads to hypothyroidism.
• Hypothyroidism signs and symptoms may include: Easy fatigability Increased sensitivity to cold (cold
intolerance) Constipation Dry skin ( due to reduced conversion of carotene to vitamin A) Weight gain
Patients with severe hypothyroidism or myxedema develop characteristic facial features due to the

Page 7

59
deposition of glycosaminoglycans in the subcutaneous tissues, leading to facial and periorbital
puffiness. Hoarseness of voice Muscle weakness (thyroid myopathy), Muscle aches, tenderness, and
stiffness Elevated blood cholesterol level Heavier than normal or irregular menstrual periods Infertility
Thinning hair and loss of outer two-thirds of eyebrow Enlarged thyroid gland (goiter) Cardiovascular
changes include bradycardia, cardiomegaly, pericardial effusion, reduced cardiac output and
pulmonary effusion. Hypothyroidism in children leads to cretinism and characteristic facies
• Easy fatigability
• Increased sensitivity to cold (cold intolerance)
• Constipation
• Dry skin ( due to reduced conversion of carotene to vitamin A)
• Weight gain
• Patients with severe hypothyroidism or myxedema develop characteristic facial features due to the
deposition of glycosaminoglycans in the subcutaneous tissues, leading to facial and periorbital
puffiness.
• Hoarseness of voice
• Muscle weakness (thyroid myopathy), Muscle aches, tenderness, and stiffness
• Elevated blood cholesterol level
• Heavier than normal or irregular menstrual periods
• Infertility
• Thinning hair and loss of outer two-thirds of eyebrow
• Enlarged thyroid gland (goiter)
• Cardiovascular changes include bradycardia, cardiomegaly, pericardial effusion, reduced cardiac
output and pulmonary effusion.
• Hypothyroidism in children leads to cretinism and characteristic facies
• Easy fatigability
• Increased sensitivity to cold (cold intolerance)
• Constipation
• Dry skin ( due to reduced conversion of carotene to vitamin A)
• Weight gain
• Patients with severe hypothyroidism or myxedema develop characteristic facial features due to the
deposition of glycosaminoglycans in the subcutaneous tissues, leading to facial and periorbital
puffiness.
• Hoarseness of voice
• Muscle weakness (thyroid myopathy), Muscle aches, tenderness, and stiffness
• Elevated blood cholesterol level
• Heavier than normal or irregular menstrual periods
• Infertility
• Thinning hair and loss of outer two-thirds of eyebrow
• Enlarged thyroid gland (goiter)

Page 8

60
• Cardiovascular changes include bradycardia, cardiomegaly, pericardial effusion, reduced cardiac
output and pulmonary effusion.
• Hypothyroidism in children leads to cretinism and characteristic facies
Option B: Hyperthyroidism
• Hyperthyroidism presents with weight loss despite increased appetite, heat
intolerance, tremors, gastrointestinal symptoms (diarrhea), and warm moist palms.
Option C: Anemia of unknown cause
• A patient with anemia presents easy fatigability, excessive drowsiness, dry skin, and dry/damaged
hair; however, patients with anemia do not show periorbital puffiness, cold intolerance, and hoarseness
of voice. Such symptoms are suggestive of hypothyroidism.
Option D: Underlying malignancy
• A patient with malignancy will present with weight loss, decreased appetite, and easy fatigability
gradually progressively. However, periorbital puffiness, dry, coarse skin, weight gain, and cold
intolerance are signs of hypothyroidism.

Solution for Question 2:


Option D: Anaplastic thyroid carcinoma
• This patient has had neck swelling for the past 4 years with a rapid progression in size since the last
3-4 months associated with hoarseness of voice and pain over the swelling likely has anaplastic thyroid
carcinoma.
• Anaplastic thyroid carcinoma arises from follicular cells and is most commonly seen in life's 7th to 8th
decade of life.
• It presents more commonly in women.
• The patients present with a history of neck swelling and a sudden increase in the size of the swelling
associated with severe pain over the swelling
• The most common route of spread is by direct invasion causing compression of the surrounding
structures: Dyspnea due to tracheal compression Dysphagia due to esophageal compression
Hoarseness of voice due to recurrent laryngeal nerve involvement
• Dyspnea due to tracheal compression
• Dysphagia due to esophageal compression
• Hoarseness of voice due to recurrent laryngeal nerve involvement
• The most common site of metastasis is Lungs
• On FNAC, it shows multinucleated giant cells
• Has a poor prognosis due to the aggressive nature of the tumor
• Dyspnea due to tracheal compression
• Dysphagia due to esophageal compression
• Hoarseness of voice due to recurrent laryngeal nerve involvement
Option A: Papillary thyroid carcinoma

Page 9

61
• Papillary thyroid carcinoma presents in young patients with a history of radiation exposure.
• It presents a painless thyroid mass, hard and fixed with irregular borders and regional
lymphadenopathy.
Option B: Follicular thyroid carcinoma
• Most commonly present in the fifth decade of life.
• Present with an enlargement of the thyroid gland, most commonly asymptomatic, and occasionally
show hypo or hyperthyroid symptoms.
Option C: Medullary thyroid carcinoma
• It is a rare calcitonin-producing tumor originating from the parafollicular C-cells of the thyroid gland.
• Present with symptoms of compression of surrounding organs i.e. dyspnea, dysphagia, and
hoarseness
• It has a history of similar presentation in the family due to RET mutation (the RET proto-oncogene
codes the RET receptor tyrosine kinase)

Solution for Question 3:


Option C: It is dependent on TSH
• The diagnosis is medullary thyroid carcinoma (MTC). MTC is TSH independent, so it does not take
radioactive iodine (hence, radioactive iodine ablation is ineffective for MTC)
• Thyroid gland parafollicular 'C' cells are the source of medullary cancer. C cells are
calcitonin-secreting structure which originate from the ultimobranchial bodies and are situated at the
superolateral region of the thyroid lobes.
• Calcium levels are normal, calcitonin levels are elevated in medullary thyroid cancer.
• Most cases are sporadic; however, occasionally, it is part of the MEN 2A and MEN 2B syndrome,
wherein the patients have a positive family history.
Sporadic
Familial
• 80% of cases
• 20% of cases

• Associated with MEN/ Non-MEN


• If associated with MEN, it is MEN 2A/2B
• Seen in 6th decade
• Young patients
• Single
• Multiple
• Unilateral
• Bilateral

Page 10

62
→ The similarity between sporadic and familial MTC - Both have RET Proto-oncogene mutation
Option A: It arises from Parafollicular ‘C’ cells
• Medullary thyroid carcinoma arises from the parafollicular 'C' cells of the thyroid gland.
Option B: Both hematogenous and lymphatic spread is seen
• It spreads by local invasion, hematogenous and lymphatic routes causing disease propagation;
distant metastasis is most frequently seen in the liver, bone, and lungs.
Option D: Most cases are sporadic
• Medullary thyroid cancers are mostly sporadic, as up to 80% of cases are sporadic and just 20% are
familial. The sporadic cases arose in the sixth decade and were most present as unilateral masses.

Solution for Question 4:


Option B: Parathyroid insufficiency
• The image above demonstrates a Carpopedal spasm by inflating the sphygmomanometer above
systolic blood pressure for 3 minutes (Trousseau's sign). It is due to hypocalcemia caused by
parathyroid insufficiency in a patient with total thyroidectomy.
• Parathyroid glands are the two sets of glands located at the upper and lower poles of the thyroid
gland. Total thyroidectomy can also lead to loss of parathyroid glands. This results in decreased effects
of parathyroid hormone leading to Hypocalcemia as a complication of the surgery.
• Hypocalcemia most commonly occurs on the second to the fifth day of surgery.
• The incidence of permanent hypoparathyroidism is less than 1%
• Classical findings include : Carpopedal spasm Tingling/numbness over the perioral region
• Carpopedal spasm
• Tingling/numbness over the perioral region
• Carpopedal spasms are frequent and involuntary muscle contractions in the hands and feet with
associated pain. It involves the adduction of the thumb, flexion of the metacarpophalangeal joint,
extension of the interphalangeal joint, and wrist flexion. It occurs due to underlying hypocalcemia,
which causes nerve and muscular hyperexcitability causing these involuntary muscle contractions.
• Chvostek’s sign : is the contraction of ipsilateral facial muscles elicited by tapping the facial nerve
just anterior to the ear.
• Carpopedal spasm
• Tingling/numbness over the perioral region
Option A: Hypothyroidism
• Total thyroidectomy leads to hypothyroidism, and the patients are to take thyroxine for the rest of their
lives. However, these patients do not show features of hypocalcemia; instead, hypothyroidism presents
as follows: Bradycardia Cold intolerance Constipation Periorbital edema Weight gain Myopathy
• Bradycardia
• Cold intolerance
• Constipation

Page 11

63
• Periorbital edema
• Weight gain
• Myopathy
• Bradycardia
• Cold intolerance
• Constipation
• Periorbital edema
• Weight gain
• Myopathy
Option C: Nerve damage during surgery
• During total thyroidectomy, the nerve at risk of injury is the recurrent laryngeal nerve. As a result, the
patient develops hoarseness of voice or total loss of speech. However, nerve damage does not cause
carpopedal spasms; such findings are due to hypocalcemia from the underlying hypoparathyroidism.
Option D: Infection
• Post-surgery infection occurs within 3 days of surgery, and the patients present with fever,
tachycardia, and signs of infection/inflammation at the incision site. This patient is afebrile and has no
worth mentioning signs at the surgery site

Solution for Question 5:


Option C: Right hemithyroidectomy
As FNAC can not differentiate between follicular adenoma and follicular carcinoma.Therefore histopath
ological examination of solitary nodule thyroid is used to make the diagnosis.
Follicular carcinoma of the thyroid:
• Seen in Iodine deficient areas
• Most common - malignancy in long-standing goiter
• Common among the fifth to the sixth decade and in females
• Involves the mutation of: PAX – 8/ PPAR -1, PTEN, P-53, RAS
• The patient presents with the following findings: A sudden increase in the size of swelling Minimal/no
pain over the swelling unless a hemorrhage into the nodule has occurred
• A sudden increase in the size of swelling
• Minimal/no pain over the swelling unless a hemorrhage into the nodule has occurred
• No evidence of compression of the trachea, esophagus or RLN
• In <1% of cases, follicular cancers may be hyperfunctioning leading patients to present with signs and
symptoms of thyrotoxicosis.
• Lymphatic spread is not seen (So lymph nodes need not be removed during surgery). It is most
commonly spread hematogenously. The most common sites of metastasis are the Bones (Thoracic
vertebra > Ribs > Pelvis > skull) and lungs.

Page 12

64
• Follicular carcinoma thyroid leads to pulsatile secondary and Renal cell carcinoma (osteolytic
and hypervascular)
• FNAC cannot differentiate follicular adenoma from follicular carcinoma. Hence hemithyroidectomy is
done initially for solitary nodule thyroid diagnosed as follicular adenoma, and the specimen is sent for
histopathological examination. Complete thyroidectomy is done when there is evidence of vascular
invasion / capsular invasion in a biopsy.
• A sudden increase in the size of swelling
• Minimal/no pain over the swelling unless a hemorrhage into the nodule has occurred
Option A: Enucleation
• The removal of the nodule alone by enucleation is associated with a greater risk of bleeding and/or
spillage and spread of the tumor. Therefore, the most effective removal is done by thyroidectomy
(hemithyroidectomy to remove the specimen and diagnosis)
Option B: Sub-total thyroidectomy
• Removal of majority of thyroid gland , leaving behind 4 - 8 g of thyroid.
Option D: Near-total thyroidectomy
Removal of entire thyroid gland except < 1 gm of thyroid tissue

Solution for Question 6:


Option D: Cold nodules are diagnostic of malignancy
Thyroid nodule:
• In case of thyroid nodule investigation of choice is FNAC, but it does not distinguish between
Adenoma and Carcinoma.
• Diagnosis of follicular carcinoma thyroid is based on vascular invasion / capsular invasion. Therefore,
the investigation of choice is a biopsy (which identifies capsular and vascular invasion).
• Solitary thyroid nodule diagnosed on FNAC as follicular adenoma in elderly patients; if the size is >4
cm, because of the high risk of cancer, total thyroidectomy is done.
• In hot or warm lesions, the risk of malignancy is <5%.
• Risk of malignancy in cold nodules is high (17%-20%), but the cold nodule is not diagnostic of
malignancy. Therefore, cold nodules are managed with observation, and an increase in size or
compressive symptoms (dysphagia, dyspnea, hoarseness of voice) require a repeat FNAC with biopsy
followed by thyroidectomy.
Option A: FNAC is the investigation of choice
• FNAC is the most important test in evaluating thyroid masses, as most nodules can be classified into
several categories that determine further management.
Option B: FNAC cannot distinguish follicular adenoma from carcinoma
• Even though it is an important initial test to classify the thyroid nodule into different categories, the
FNAC cannot distinguish between adenoma and carcinoma as the carcinoma is characterized by
capsular invasion, which requires excisional tissue biopsy to locate the invasion.
Option C: Managed by total thyroidectomy

Page 13

65
• Based on the results of the FNAC, if the sample is suspicious for/ or has signs of malignancy (i.e.
cellular atypia, high nuclear to cytoplasmic ratio, undifferentiated tissue), it proceeded with total
thyroidectomy.
• Furthermore, lobectomy is recommended for nodules greater than 4 cm, causing compressive
symptoms as they have a higher incidence of malignancy.

Solution for Question 7:


Correct Option
Option A: Follicular carcinoma of the thyroid
• Psammoma bodies: Psammoma bodies are round concentric lamellated microscopic calcified lesions.
It occurs due to dystrophic calcification in the cancer cells. The necrotic cells form the focus for
surrounding calcific deposition leading to the formation of a lamellated concentric calcified structure.
The ultrastructural study of the formation occurs after the thickening of the base lamina in the vascular
stalk of neoplastic papillae, followed by thrombosis, calcification, and tumor cell necrosis. In other
cases, it is formed by collagen production and subsequent calcification. It is suggested that psammoma
bodies may represent an active biologic process ultimately leading to degeneration/death of tumor cells
and retardation of growth of the neoplasm (i.e. acting as a barrier against the spread of neoplasm).
• Psammoma bodies are round concentric lamellated microscopic calcified lesions.
• It occurs due to dystrophic calcification in the cancer cells. The necrotic cells form the focus for
surrounding calcific deposition leading to the formation of a lamellated concentric calcified structure.
• The ultrastructural study of the formation occurs after the thickening of the base lamina in the vascular
stalk of neoplastic papillae, followed by thrombosis, calcification, and tumor cell necrosis. In other
cases, it is formed by collagen production and subsequent calcification.
• It is suggested that psammoma bodies may represent an active biologic process ultimately leading to
degeneration/death of tumor cells and retardation of growth of the neoplasm (i.e. acting as a barrier
against the spread of neoplasm).
• Follicular carcinoma shows follicles with a lumen devoid of colloid. The hallmark is the presence of
capsular and vascular invasion. The image below shows the capsular invasion.
• Psammoma bodies are round concentric lamellated microscopic calcified lesions.
• It occurs due to dystrophic calcification in the cancer cells. The necrotic cells form the focus for
surrounding calcific deposition leading to the formation of a lamellated concentric calcified structure.
• The ultrastructural study of the formation occurs after the thickening of the base lamina in the vascular
stalk of neoplastic papillae, followed by thrombosis, calcification, and tumor cell necrosis. In other
cases, it is formed by collagen production and subsequent calcification.
• It is suggested that psammoma bodies may represent an active biologic process ultimately leading to
degeneration/death of tumor cells and retardation of growth of the neoplasm (i.e. acting as a barrier
against the spread of neoplasm).

Page 14

66
Other Options
Option B: Papillary carcinoma of the thyroid
• Papillary thyroid cancer shows psammoma bodies formed due to thrombosis, calcification, and tumor
cell necrosis.They are formed due to sloughing off of papillary projections and are diagnostic of PTC .
Option C: Meningioma
• Meningioma show psammoma bodies and are visible as lamellated concentric calcified structures.
Option D: Serous cystadenocarcinoma of the ovary
• Psammoma bodies are an important microscopic finding in ovarian serous cystadenocarcinoma.

Solution for Question 8:


Option C: Total thyroidectomy + removal of an enlarged central group of lymph node ±
Ipsilateral modified radical neck dissection MRND (if any LN is positive)
• The clinical scenario describes papillary carcinoma thyroid
• The best treatment option in this case, is Total thyroidectomy + removal of an enlarged central group
of lymph node ± Ipsilateral modified radical neck dissection( MRND)
• Papillary thyroid carcinoma: Manifests as a painless lateral neck mass that is clinically detected
before detecting the primary thyroid lesion called lateral aberrant thyroid Pathology: Papillary
projections Optically clear nuclei aka ORPHAN ANNIE EYE NUCLEI Pseudo-inclusion bodies
Dystrophic calcification – PSAMMOMA BODIES The primary treatment is surgery. Other mainstays of
therapy are radioactive iodine ablation and TSH suppression.
• Manifests as a painless lateral neck mass that is clinically detected before detecting the primary
thyroid lesion called lateral aberrant thyroid
• Pathology: Papillary projections Optically clear nuclei aka ORPHAN ANNIE EYE NUCLEI
Pseudo-inclusion bodies Dystrophic calcification – PSAMMOMA BODIES
• Papillary projections

Page 15

67
• Optically clear nuclei aka ORPHAN ANNIE EYE NUCLEI
• Pseudo-inclusion bodies
• Dystrophic calcification – PSAMMOMA BODIES
• The primary treatment is surgery. Other mainstays of therapy are radioactive iodine ablation and TSH
suppression.

• Option A: Hemithyroidectomy
• Hemithyroidectomy or lobectomy is indicated for indeterminate nodules, and small intrathyroidal
cancer.
• For papillary cancer > 1cm in size, hemithyroidectomy leaves the residual disease and, thus risk of
recurrence of cancer.
• Manifests as a painless lateral neck mass that is clinically detected before detecting the primary
thyroid lesion called lateral aberrant thyroid
• Pathology: Papillary projections Optically clear nuclei aka ORPHAN ANNIE EYE NUCLEI
Pseudo-inclusion bodies Dystrophic calcification – PSAMMOMA BODIES
• Papillary projections
• Optically clear nuclei aka ORPHAN ANNIE EYE NUCLEI
• Pseudo-inclusion bodies
• Dystrophic calcification – PSAMMOMA BODIES
• The primary treatment is surgery. Other mainstays of therapy are radioactive iodine ablation and TSH
suppression.

• Papillary projections
• Optically clear nuclei aka ORPHAN ANNIE EYE NUCLEI
• Pseudo-inclusion bodies
• Dystrophic calcification – PSAMMOMA BODIES
Option B: Subtotal thyroidectomy with modified neck dissection
• Subtotal thyroidectomy is not recommended for thyroid malignancy due to the risk of recurrence of
cancer from the residual thyroid tissue.
• Furthermore, it is used mainly for benign diseases, small nodules on the side of lesions, and
hyperthyroidism.
Option D: Hemithyroidectomy with modified neck dissection
• Hemithyroidectomy is for indeterminate nodules and small thyroid cancer. A papillary thyroid cancer
>1 cm requires total thyroidectomy for improved survival.
• With hemithyroidectomy for papillary cancer, the residual disease will pose the risk of recurrence and
therefore is not done.

Page 16

68
Solution for Question 9:
Option A: Enophthalmos
• Enophthalmos is defined as the posterior displacement of the globe of the eye in an anterioposterior
plane within the orbit.
• This is caused by one of the following factors: Enlargement of the orbit Reduction of the orbital
contents Shrinkage or contracture of orbital contents
• Enlargement of the orbit
• Reduction of the orbital contents
• Shrinkage or contracture of orbital contents
• However, the ophthalmopathy seen in Graves' disease is called exophthalmos.
• Enlargement of the orbit
• Reduction of the orbital contents
• Shrinkage or contracture of orbital contents
Graves disease (diffuse toxic goitre):
• It is an autoimmune disorder caused by thyroid-stimulating auto-antibody against the TSH receptors.
• Associated with HLA B-8, DR-3.
• Has a strong familial predisposition with female preponderance (5:1).
• Peak incidence is seen among people aged 40 to 60 years.
• The patient presents with thyrotoxicosis, diffuse goitre, and some symptoms of hyperthyroidism, as
mentioned:
• Weight loss
• Increased appetite
• Heat intolerance
• Increased sweating and thirst
• Ophthalmopathy seen in Graves' disease is called exophthalmos which occurs due to the collection of
loose areolar area tissue behind the eyeball leading to the protrusion of the eye in front of the orbit.
• Other findings include dermopathy, gynecomastia, myopathy, and increased deep tendon reflexes.
• Characteristic features of Grave's disease - Thyrotoxicosis , opthalmopathy, exopthalmos ,
dermopathy, acropachy
Option B: Gynecomastia
• Most commonly seen in young boys.
• Gynecomastia occurs in hyperthyroidism due to increased thyroid hormone-mediated effects:
Increased aromatase activity in the peripheral tissue causes the conversion of androgens to estrogen.
Increased sex hormone binding globulin, which decreases the free testosterone levels causing
gynecomastia.
• Increased aromatase activity in the peripheral tissue causes the conversion of androgens to estrogen.
• Increased sex hormone binding globulin, which decreases the free testosterone levels causing
gynecomastia.

Page 17

69
• Increased aromatase activity in the peripheral tissue causes the conversion of androgens to estrogen.
• Increased sex hormone binding globulin, which decreases the free testosterone levels causing
gynecomastia.
Option C: Subperiosteal new bone formation in metacarpals
• Acropachy is defined as the thickening of extremities manifested by a triad of the following:
Subperiosteal new bone formation in metacarpals Digital clubbing Soft tissue swelling of hand and feet
• Subperiosteal new bone formation in metacarpals
• Digital clubbing
• Soft tissue swelling of hand and feet
• Subperiosteal new bone formation in metacarpals
• Digital clubbing
• Soft tissue swelling of hand and feet
Option D: Pretibial myxedema
• Pretibial myxedema is caused by the deposition of glycosaminoglycans leading to the thickness of the
skin in the pretibial region and dorsum of the foot.

Solution for Question 10:


Option C: 2nd, 3rd and 4th tracheal cartilage
THYROID GLAND

• It is a gland consisting of two lobes, the right and left, joined together by the isthmus.
• The two lobes are conical and have three lateral, medial, and posterolateral surfaces, with two
borders, anterior and posterior.

Page 18

70
• It has a fibrous/fibromuscular band, i.e. levator glandular thyroid, from the body of the hyoid to the
isthmus. The lobes are 5 x 2.5 x 2.5 cm in dimensions and weigh about 20-25 g.
• The isthmus, just inferior to the cricoid cartilage, is usually anterior to the 2nd and 3rd (mainly) 4th
tracheal rings, as shown in the image above.
• A pyramidal lobe is present in about 50% of patients.
• Anteriorly they are related to skin, superficial and deep fascia, and platysma; Posteriorly, the lobes are
associated with laminae of the thyroid cartilage and tracheal rings and laterally to the external carotid
artery and internal jugular vein.
• The thyroid capsule is condensed into the posterior suspensory or Berry's ligament near the cricoid
cartilage and upper tracheal rings.
• The external laryngeal nerve runs close to the superior thyroid artery, and the recurrent laryngeal
nerve runs close to the inferior thyroid artery.

Solution for Question 11:


Option B: Subacute thyroiditis
• The most likely diagnosis is subacute thyroiditis, as the swelling is painful and preceded by a viral
upper respiratory illness and the microscopy shows follicular destruction and giant cell formation.
• Subacute thyroiditis: Subacute thyroiditis is a clinical diagnosis. Most commonly occurs in 30 to
40-year-old women Neck pain with an exquisitely tender and firm thyroid gland on the exam is sufficient
enough to establish the diagnosis. Sudden or gradual onset of neck pain radiating to jaw or ear 4
phases: initial hyperthyroid, then euthyroid phase, hypothyroid, resolution, and return to the euthyroid
state The diagnosis is confirmed with low TSH, elevated free T4, T3, ESR, C-reactive protein, and low
radioiodine uptake ESR is raised FNAC is diagnostic - demonstrates giant cells of an epitheloid foreign
body type, which characterize the lesion Microscopic pathology shows large follicles infiltrated by
mononuclear cells, neutrophils and lymphocytes. Self-limited and treatment is primarily symptomatic
• Subacute thyroiditis is a clinical diagnosis.
• Most commonly occurs in 30 to 40-year-old women
• Neck pain with an exquisitely tender and firm thyroid gland on the exam is sufficient enough to
establish the diagnosis.
• Sudden or gradual onset of neck pain radiating to jaw or ear
• 4 phases: initial hyperthyroid, then euthyroid phase, hypothyroid, resolution, and return to the
euthyroid state
• The diagnosis is confirmed with low TSH, elevated free T4, T3, ESR, C-reactive protein, and low
radioiodine uptake
• ESR is raised
• FNAC is diagnostic - demonstrates giant cells of an epitheloid foreign body type, which characterize
the lesion
• Microscopic pathology shows large follicles infiltrated by mononuclear cells, neutrophils and
lymphocytes.

Page 19

71
• Self-limited and treatment is primarily symptomatic
The most likely diagnosis is subacute thyroiditis, as the swelling is painful and preceded by a
viral upper respiratory illness and the microscopy shows follicular destruction and giant cell formation.
Subacute thyroiditis:
• Subacute thyroiditis is a clinical diagnosis.
• Most commonly occurs in 30 to 40-year-old women
• Neck pain with an exquisitely tender and firm thyroid gland on the exam is sufficient enough to
establish the diagnosis.
• Sudden or gradual onset of neck pain radiating to jaw or ear
• 4 phases: initial hyperthyroid, then euthyroid phase, hypothyroid, resolution, and return to the
euthyroid state
• The diagnosis is confirmed with low TSH, elevated free T4, T3, ESR, C-reactive protein, and low
radioiodine uptake
• ESR is raised
• FNAC is diagnostic - demonstrates giant cells of an epitheloid foreign body type, which characterize
the lesion
• Microscopic pathology shows large follicles infiltrated by mononuclear cells, neutrophils and
lymphocytes.
• Self-limited and treatment is primarily symptomatic
Subacute thyroiditis is a clinical diagnosis.
Most commonly occurs in 30 to 40-year-old women
Neck pain with an exquisitely tender and firm thyroid gland on the exam is sufficient enough to establis
h the diagnosis.
Sudden or gradual onset of neck pain radiating to jaw or ear
4 phases: initial hyperthyroid, then euthyroid phase, hypothyroid, resolution, and return to the euthyroi
d state
The diagnosis is confirmed with low TSH, elevated free T4, T3, ESR, C-reactive protein, and low radioi
odine uptake
ESR is raised
FNAC is diagnostic -
demonstrates giant cells of an epitheloid foreign body type, which characterize the lesion
Microscopic pathology shows large follicles infiltrated by mononuclear cells, neutrophils and lymphocyt
es.
Self-limited and treatment is primarily symptomatic
Other options
Option A: Acute thyroiditis
• Acute suppurative thyroiditis is an infection of the thyroid gland by Streptococcus and anaerobes.
• More common in children and preceded by an upper respiratory tract infection or otitis media. It
usually occurs in those with immunosuppression and results from seeding an infectious agent via
hematogenous, direct spread from the pyriform sinus fistulae/thyroglossal duct cyst or penetrating

Page 20

72
trauma.
• Presents with severe neck pain radiating to the jaws or ear, fever, chills, odynophagia, and dysphonia.
Acute suppurative thyroiditis is an infection of the thyroid gland by Streptococcus and anaerobes.
More common in children and preceded by an upper respiratory tract infection or otitis media. It usually
occurs in those with immunosuppression and results from seeding an infectious agent via hematogeno
us, direct spread from the pyriform sinus fistulae/thyroglossal duct cyst or penetrating trauma.
Presents with severe neck pain radiating to the jaws or ear, fever, chills, odynophagia, and dysphonia.
Option C: Tubercular lymphadenitis
• May present with neck swelling, hoarseness of voice, dyspnea, dysphagia
• On gross examination, show cystic areas with foci of necrosis
• Microscopy shows caseous necrosis, multinucleated giant cells, and epitheloid cell granulomas.
May present with neck swelling, hoarseness of voice, dyspnea, dysphagia
On gross examination, show cystic areas with foci of necrosis
Microscopy shows caseous necrosis, multinucleated giant cells, and epitheloid cell granulomas.
Option D: Hashimoto's thyroiditis
• Hashimoto's thyroiditis is the most common inflammatory thyroid disorder due to an autoimmune
process resulting in lymphocyte-mediated damage of thyroid follicular cells.
• The patient presents with signs of hypothyroidism. The gland is enlarged, nodular and painless.
• On microscopy, it diffusely infiltrated by small lymphocytes and plasma cells, showing well developed
germinal center.
Hashimoto's thyroiditis is the most common inflammatory thyroid disorder due to an autoimmune proce
ss resulting in lymphocyte-mediated damage of thyroid follicular cells.
The patient presents with signs of hypothyroidism. The gland is enlarged, nodular and painless.
On microscopy, it diffusely infiltrated by small lymphocytes and plasma cells, showing well developed g
erminal center.

Solution for Question 12:


Option C: Removing sutures from all layers in the ward and evacuation of hematoma
• In this case, the patient has developed a hematoma beneath the stitches, which has led to
dyspnea. Evacuating the hematoma immediately at the bedside by removing the sutures is the first line
of management. Then the patient can be shifted to the operation theatre and explored.
• The post-thyroidectomy hematoma most frequently manifests within 24 hours and is the most
common life-threatening thyroidectomy side effect.
• The patient's growing neck edema, neck pain, and/or indications of an airway blockage are evident
(dyspnea, stridor or hypoxia). Although the prevalence of hematoma is modest, if it goes unnoticed, it
can quickly spread, compromising the airways and leading to asphyxiation.
• The examination will be performed to take off all bandages and check for neck swelling. Imaging will
then be used to evaluate edema, and finally, a laryngoscopy will be performed to check for airway

Page 21

73
problems and vocal cord function.
• Immediate suture removal, hematoma evacuation, and wound examination are required. It can be
prevented by not stressing the thyroid tissue during the procedure, ensuring good intraoperative
hemostasis, and avoiding neck coverings because they may conceal the formation of hematomas.
• Tracheostomy and endotracheal intubation are methods employed for respiratory distress due to
vocal cord paralysis (nerve injury) and not external compression from the hematoma.
Option A: Intranasal oxygen
• This patient experiences respiratory distress or symptoms/signs of hypoxia. Even though she is
agitated, the cause of agitation is the neck hematoma compressing the trachea beneath; therefore,
providing intranasal oxygen without first addressing the underlying cause will not be helpful.
Option B: Passing an endotracheal tube in the ward
• In cases of recurrent laryngeal nerve palsy, which results in bilateral vocal cord paralysis, the patient
presents with biphasic stridor or respiratory distress. If possible, endotracheal intubation is to be
performed, followed by cordotomy and adenoidectomy to enlarge the airway and permit decannulation
of a tracheostomy which may be required in the event of an emergency.
Option D: Immediate transfer of the patient to the operation theatre for tracheostomy
• In cases of bilateral RLN palsy post thyroidectomy, leading to bilateral vocal cord paralysis,
emergency tracheostomy may be necessary to prevent airway compromise.

Solution for Question 13:


Option B: Hashimoto’s disease
• Hashimoto's disease aka Hashimoto's thyroiditis/lymphocytic thyroiditis, is the most common
inflammatory disorder of the thyroid, causing hypothyroidism.
• It is an autoimmune process initiated by the CD4+ T-helper lymphocytes specific for the thyroid
antigens. As a result, the thyroid follicles are damaged by the recruited CD8 T cells. Hypothyroidism
results not only from the destruction of the thyrocytes by cytotoxic cells but also from the
auto-antibodies leading to the complement fixation and killing of the cells.
• The patient is more commonly women aged 30 to 50 years and present initially with signs of
hyperthyroidism(due to the release of preformed thyroid hormones) followed later by hypothyroidism.
• On gross examination, the thyroid gland is painless, mildly enlarged, granular, and nodular with a firm
consistency.
• Hashimoto disease is characterized by increased TSH and decreased T4 due to loss of thyroid
hormone synthesis, and the TSH is increased from the loss of negative feedback from thyroid
hormones.
Option A: Graves' disease
• Graves' disease causes elevated T3, T4 and decreased TSH. This is because, in Graves' disease, the
TSH receptor antibodies cause increased activation of the TSH receptor leading to elevated production
of thyroid hormones and, in turn, inhibition of the TSH levels (via negative feedback)
• Furthermore, Graves' disease presents symptoms of hyperthyroidism and signs of Graves' disease,
i.e. thyroid acropachy, exophthalmos and pretibial myxedema.
Option C: Pituitary failure

Page 22

74
• Pituitary failure decreases all hormones produced by the pituitary, including the TSH, which leads to
decreased TSH levels.
• Furthermore, effects of the decrease in other hormones are also seen, i.e. loss of ADH causes
increased thirst, loss of ACTH decreases cortisol levels resulting in hypoglycemia and low blood
pressure, decreased gonadotropins causes irregular or absent menses and loss of prolactin results in
absent milk production.
Option D: Hypothalamic failure
• The hypothalamic failures cause decreased TSH and decreased T3 and T4 levels. However, in
Hashimoto's disease, the disease is intrinsic to the thyroid gland causing inhibition of thyroid hormone
synthesis and loss of feedback inhibition resulting in elevated TSH levels.

Solution for Question 14:


Option C: Vascular invasion
Follicular carcinoma
• Follicular carcinoma of the thyroid is seen in iodine-deficient areas, the most common malignancy in
long-standing goiter among females in the 5th to 6th decade. Caused by a mutation in pAX-8/PPAR-1,
P-53 and RAS.
• The patient presents with a sudden increase in the size of neck swelling with minimal or no pain over
the swelling. The most common route of spread is hematogenous, as the lymphatic spread is not seen.
• The most common site of metastasis for follicular thyroid carcinoma are bones (Thoracic vertebra >
Ribs > Pelvis > skull) and lung causing follicular carcinoma leading to pulsatile secondaries and renal
cell carcinoma (osteolytic lesions and hypervascularity)
• Diagnosis of follicular carcinoma thyroid is based on vascular invasion / capsular invasion.
• Histological diagnosis of follicular thyroid carcinoma depends on demonstrating follicular cells
occupying abnormal positions, including capsular and vascular invasion.
• FNAC cannot differentiate follicular adenoma from follicular carcinoma. Therefore, the investigation of
choice is a biopsy (identifies capsular and vascular invasion)
Follicular carcinoma of the thyroid is seen in iodine-deficient areas, the most common malignancy in lo
ng-standing goiter among females in the 5th to 6th decade. Caused by a
mutation in pAX-8/PPAR-1, P-53 and RAS.
The patient presents with a sudden increase in the size of neck swelling with minimal or no pain over th
e swelling. The most common route of spread is hematogenous, as the lymphatic spread is not seen.
The most common site of metastasis for follicular thyroid carcinoma are bones (Thoracic vertebra >
Ribs > Pelvis > skull) and lung causing follicular carcinoma
leading to pulsatile secondaries and renal cell carcinoma (osteolytic lesions and hypervascularity)
Diagnosis of follicular carcinoma thyroid is based on vascular invasion / capsular invasion.
Histological diagnosis of follicular thyroid carcinoma depends on demonstrating follicular cells occupyin
g abnormal positions, including capsular and vascular invasion.
FNAC cannot differentiate follicular adenoma from follicular carcinoma. Therefore, the investigation of c
hoice is a biopsy (identifies capsular and vascular invasion)
Option A: Hurthle cell change

Page 23

75
• Hurthle cell change is associated with benign and malignant thyroid lesions e.g. Hashimotos
thyroiditis, multinodular goiter, and Hurthle cell adenoma and carcinoma.
• It does not account for the differentiation between follicular adenoma and carcinoma. That is done by
identifying the capsular and vascular invasion seen in the follicular carcinoma.
Hurthle cell change is associated with benign and malignant thyroid lesions e.g. Hashimotos thyroiditis,
multinodular goiter, and Hurthle cell adenoma and carcinoma.
It does not account for the differentiation between follicular adenoma and carcinoma. That is done by id
entifying the capsular and vascular invasion seen in the follicular carcinoma.
Option B: Lining of tall columnar and cuboidal cells
• The lining of follicular adenoma is cuboidal to low columnar cells with pale staining in a follicular
architecture and an intact capsule.
• The key differentiating factors are the capsular and vascular invasion which are more likely to be seen
in follicular carcinoma.
The lining of follicular adenoma is cuboidal to low columnar cells with pale staining in a
follicular architecture and an intact capsule.
The key differentiating factors are the capsular and vascular invasion which are more likely to be seen i
n follicular carcinoma.
Option D: Nuclear features
• Follicular carcinoma may have nuclear atypia, focal spindled areas and mitotic figures.
• The invasion of the adjacent parenchyma, capsule and blood vessels beyond the capsule.
• In Follicular adenoma, mitoses are uncommon
Follicular carcinoma may have nuclear atypia, focal spindled areas and mitotic figures.
The invasion of the adjacent parenchyma, capsule and blood vessels beyond the capsule.
In Follicular adenoma, mitoses are uncommon

Solution for Question 15:


Option A: Biopsy
Follicular adenoma
• It is a common benign neoplasm that usually presents as a solitary thyroid nodule.
• Follicular adenomas are mostly sporadic; multiple other causes include iodine deficiency and genetic
alterations, e.g. PTEN inactivation, BRAF, NRAS, RET, and KRAS mutations; in addition, prior iodine
(I-131) radiation exposure increases the risk of malignant lesions of the thyroid.
• Patients present with solitary thyroid nodules that may be associated with thyroiditis or nodular
hyperplasia. There is a solitary, slowly growing mass in the neck with a pressure sensation. Pain occurs
with a spontaneous hemorrhage or cystic degeneration in the nodule.
• After history and examination, serum TSH is done, followed by an ultrasound scan. However, the
sonographic features of both adenoma and carcinoma are similar and thus not helpful.
• Diagnosis is made by doing a biopsy of the sample. This is because, on biopsy, malignancy can be
defined based on the invasion of the tumor cells into the vascular endothelium or across the thyroid

Page 24

76
gland capsule. Based on this, a biopsy of the thyroid tissue is the diagnostic test.
It is a common benign neoplasm that usually presents as a solitary thyroid nodule.
Follicular adenomas are mostly sporadic; multiple other causes include iodine deficiency and genetic al
terations, e.g. PTEN inactivation, BRAF, NRAS, RET, and KRAS mutations; in addition, prior iodine (I-
131) radiation exposure increases the risk of malignant lesions of the thyroid.
Patients present with solitary thyroid nodules that may be associated with thyroiditis or nodular hyperpl
asia. There is a solitary, slowly growing mass in the neck with a pressure sensation. Pain occurs with a
spontaneous hemorrhage or cystic degeneration in the nodule.
After history and examination, serum TSH is done, followed by an ultrasound scan. However, the sono
graphic features of both adenoma and carcinoma are similar and thus not helpful.
Diagnosis is made by doing a biopsy of the sample. This is because, on biopsy, malignancy can be def
ined based on the invasion of the tumor cells into the vascular endothelium or across the thyroid gland
capsule. Based on this, a biopsy of the thyroid tissue is the diagnostic test.
Option B: FNAC
• FNAC is cytology sampling done to examine the cells from adenoma and carcinoma. However, based
on the histology of the cells, the decision cannot be made to differentiate between adenoma and
carcinoma. Rather, malignancy is defined by the capsular and vascular invasion of the thyroid follicles.
FNAC is cytology sampling done to examine the cells from adenoma and carcinoma. However, based
on the histology of the cells, the decision cannot be made to differentiate between adenoma and carcin
oma. Rather, malignancy is defined by the capsular and vascular invasion of the thyroid follicles.
Option C: CT scan
• CT scan has a limited role in the evaluation of solitary thyroid nodules. However, they can be
employed in detecting external compression of the surrounding structures and invasion of or
compression of surrounding tissue in the case of follicular carcinoma.
CT scan has a limited role in the evaluation of solitary thyroid nodules. However, they can be employed
in detecting external compression of the surrounding structures and invasion of or compression of surr
ounding tissue in the case of follicular carcinoma.
Option D: MRI
• MRI has a limited role in the initial evaluation of solitary thyroid nodules; however, it is helpful to
differentiate between follicular adenoma and carcinoma based on the invasion and extension of the
tissue into the surrounding mediastinum.
MRI has a limited role in the initial evaluation of solitary thyroid nodules; however, it is helpful to differe
ntiate between follicular adenoma and carcinoma based on the invasion and extension of the tissue int
o the surrounding mediastinum.

Solution for Question 16:


Option C: Fractionated plasma metanephrines
• MEN-2 syndrome has been suspected, as the inheritance of this condition is autosomal dominant and
the patient had a first degree relative suffering from medullary carcinoma of thyroid.
• Components of MEN2 : Medullary carcinoma of thyroid, pheochromocytomas, Parathyroid carcinoma.

Page 25

77
• The patient has symptoms of pheochromocytoma, so levels of fractionated plasma metanephrine
should be evaluated.
• Pheochromocytoma: It is a tumor of the enterochromaffin cells of the adrenal medulla. It affects men
and women equally, and the incidence is in the age 40 to 50 years. The patient presents with a classic
triad of headache, diaphoresis, and palpitations. Hypertension is present in 90% of cases. The main
hormone secreted by adrenal pheochromocytomas - Epinephrine. The main hormone secreted by
extra-adrenal pheochromocytomas - Norepinephrine ( as they lack phenylethanolamine
N-methyltransferase )
• It is a tumor of the enterochromaffin cells of the adrenal medulla.
• It affects men and women equally, and the incidence is in the age 40 to 50 years.
• The patient presents with a classic triad of headache, diaphoresis, and palpitations.
• Hypertension is present in 90% of cases.
• The main hormone secreted by adrenal pheochromocytomas - Epinephrine. The main hormone
secreted by extra-adrenal pheochromocytomas - Norepinephrine ( as they lack phenylethanolamine
N-methyltransferase )
• It is a tumor of the enterochromaffin cells of the adrenal medulla.
• It affects men and women equally, and the incidence is in the age 40 to 50 years.
• The patient presents with a classic triad of headache, diaphoresis, and palpitations.
• Hypertension is present in 90% of cases.
• The main hormone secreted by adrenal pheochromocytomas - Epinephrine. The main hormone
secreted by extra-adrenal pheochromocytomas - Norepinephrine ( as they lack phenylethanolamine
N-methyltransferase )
Option A: Chest X-ray
• Pheochromocytoma is a disease of the adrenal gland where excessive catecholamine synthesis due
to the tumor activity causes symptoms of hypertension, headache, palpitations, and diaphoresis. As
part of the MEN syndrome, where pheochromocytoma occurs alongside medullary thyroid cancer, a
chest X-ray is not needed for diagnosis.
Option B: Measurement of 5-HIAA
• The excessive serotonin synthesis causes serotonin syndrome. Such patients present with flushing
skin, diarrhea, vomiting, sweating, shivering, muscle twitching, involuntary contractions, muscle
spasms, and rigidity. This patient has no such symptoms.
• The end product of serotonin metabolism is 5-HIAA, which is used to diagnose serotonin syndrome.
Option D: Intravenous pyelography
• Intravenous pyelography is an x-ray exam that uses an injection of contrast material to evaluate the
kidneys, ureter, and urinary bladder to help diagnose blood in the urine or pain in the kidneys
• This patient has symptoms of pheochromocytoma and requires evaluation of the adrenal glands and
not of the kidneys.

Solution for Question 17:


Option C: Surgery only

Page 26

78
• Medullary carcinoma of the thyroid arises from Parafollicular ‘C’ Cells
• Despite raised calcitonin, the level of calcium is normal in these patients (Normocalcemia)
• Clinical Features: Midline swelling in the anterior part of the neck with palpable cervical
lymphadenopathy Female to male ratio is 5:1 Most patients present between 50 and 60 years old
History of diarrhea due to excessive secreation of calcitonin and calcitonin gene related peptide by the
tumor
• Midline swelling in the anterior part of the neck with palpable cervical lymphadenopathy
• Female to male ratio is 5:1
• Most patients present between 50 and 60 years old
• History of diarrhea due to excessive secreation of calcitonin and calcitonin gene related peptide by the
tumor
• immunohistochemistry Immunohistochemistry for calcitonin is more commonly used as a diagnostic
marker. These tumors also stain positively for CEA and calcitonin gene-related peptides.
• Immunohistochemistry for calcitonin is more commonly used as a diagnostic marker.
• These tumors also stain positively for CEA and calcitonin gene-related peptides.
• Positive family history of Pheochromocytoma and Hyperparathyroidism (due to association with MEN
2A & MEN 2B)
• Route of spread: Local invasion produces dyspnea, dysphagia, and dysphonia. Both hematogenous
and lymphatic spread is seen. Distant blood-borne metastases to the liver, bone (frequently
osteoblastic), and lungs occur later in the disease.
• Local invasion produces dyspnea, dysphagia, and dysphonia.
• Both hematogenous and lymphatic spread is seen.
• Distant blood-borne metastases to the liver, bone (frequently osteoblastic), and lungs occur later in
the disease.
• Treatment is by Total thyroidectomy + Routine central lymph node dissection + ipsilateral modified
radical node dissection (Tumor > 1cm) ± Bilateral modified Radical neck dissection (if any lymph node
is positive) It has a Poor prognosis (because it does not respond to radioactive iodine ablation)
Vandetanib - Only FDA-approved drug in advanced progressive medullary thyroid cancer.
• Total thyroidectomy + Routine central lymph node dissection + ipsilateral modified radical node
dissection (Tumor > 1cm) ± Bilateral modified Radical neck dissection (if any lymph node is positive)
• It has a Poor prognosis (because it does not respond to radioactive iodine ablation)
• Vandetanib - Only FDA-approved drug in advanced progressive medullary thyroid cancer.
• Midline swelling in the anterior part of the neck with palpable cervical lymphadenopathy
• Female to male ratio is 5:1
• Most patients present between 50 and 60 years old
• History of diarrhea due to excessive secreation of calcitonin and calcitonin gene related peptide by the
tumor
• Immunohistochemistry for calcitonin is more commonly used as a diagnostic marker.
• These tumors also stain positively for CEA and calcitonin gene-related peptides.
• Local invasion produces dyspnea, dysphagia, and dysphonia.

Page 27

79
• Both hematogenous and lymphatic spread is seen.
• Distant blood-borne metastases to the liver, bone (frequently osteoblastic), and lungs occur later in
the disease.
• Total thyroidectomy + Routine central lymph node dissection + ipsilateral modified radical node
dissection (Tumor > 1cm) ± Bilateral modified Radical neck dissection (if any lymph node is positive)
• It has a Poor prognosis (because it does not respond to radioactive iodine ablation)
• Vandetanib - Only FDA-approved drug in advanced progressive medullary thyroid cancer.
Option A: Surgery and Radiotherapy
• Total thyroidectomy is the treatment of choice for medullary thyroid cancer due to the high incidence
of multicentricity and aggressive course of the disease. This is done with bilateral prophylactic central
lymph node dissection (as they are frequently involved early in the disease).
• The role of external beam radiotherapy is controversial but is considered for patients with resected T4
disease and patients with unresectable residual or recurrent Tumors and symptomatic bony
metastases. Liver metastases tend to be multiple and typically not amenable to resection,
percutaneous ethanol ablation, or radiofrequency ablation. However, chemoembolization may be
helpful.
Option B: Radiotherapy and Chemotherapy
• Radiotherapy use is controversial; however, it is considered for patients with resected T4 disease and
those with unresectable residual or recurrent Tumors and symptomatic bony metastases.
• There is no effective chemotherapy regimen.
• Various targeted therapies are directed against the RET kinase and may inhibit the VEGF receptor.
• Sorafenib and sunitinib are multikinase inhibitors.
• Vandetanib also inhibits an EGF receptor inhibitor, and cabozantinib targets c-MET in addition to RET
and VEGF receptors. The FDA and EMA approve both drugs for treating advanced and progressive
medullary thyroid cancer and are recommended as first-line systemic therapy in symptomatic patients
with advanced medullary thyroid cancer.
Option D: Radioactive ablation
• Medullary thyroid cancer has a high incidence of multicentricity and a more aggressive course of
disease; therefore, radioactive iodine (I-131) ablation therapy is usually ineffective.

Solution for Question 18:


Option B: Stage II
• This patient has micro lung metastasis in addition to two lymph nodes and a size of < 3 cm nodule.
• Based on his age, i.e. < 55 years, any T stage and any N stage with metastases are considered stage
II.
AJCC Thyroid cancer staging:
• Primary tumour (T) Tx = Primary Tumor cannot be assessed T0 = No evidence of primary Tumor T1 =
Tumor ≤ 2cm in greatest dimension limited to the thyroid T1a = Tumor ≤ 1 cm in greatest dimension
limited to the thyroid T1b = Tumor >1cm but ≤ 2 cm in greatest dimension limited to the thyroid T2 =
Tumor >2cm but ≤ 4 cm in greatest dimension limited to the thyroid T3 = Tumor >4 cm limited to the

Page 28

80
thyroid or gross extrathyroidal extension invading only strap muscles T3a = Tumor > 4cm limited to the
thyroid T3b = Gross extrathyroidal extension invading only strap muscles (sternohyoid, sternothyroid,
thyrohyoid, or omohyoid muscles) from a tumour of any size T4 = Includes gross extrathyroidal
extension into major neck structures T4a = Gross extrathyroidal extension invading subcutaneous soft
tissues, larynx, Trachea, esophagus, or recurrent laryngeal nerve from a tumour of any size T4b =
Gross extrathyroidal extension invading prevertebral or encasing the artery or mediastinal Vessels from
a tumour of any size
• Tx = Primary Tumor cannot be assessed
• T0 = No evidence of primary Tumor
• T1 = Tumor ≤ 2cm in greatest dimension limited to the thyroid T1a = Tumor ≤ 1 cm in greatest
dimension limited to the thyroid T1b = Tumor >1cm but ≤ 2 cm in greatest dimension limited to the
thyroid
• T1a = Tumor ≤ 1 cm in greatest dimension limited to the thyroid
• T1b = Tumor >1cm but ≤ 2 cm in greatest dimension limited to the thyroid
• T2 = Tumor >2cm but ≤ 4 cm in greatest dimension limited to the thyroid
• T3 = Tumor >4 cm limited to the thyroid or gross extrathyroidal extension invading only strap muscles
T3a = Tumor > 4cm limited to the thyroid T3b = Gross extrathyroidal extension invading only strap
muscles (sternohyoid, sternothyroid, thyrohyoid, or omohyoid muscles) from a tumour of any size
• T3a = Tumor > 4cm limited to the thyroid
• T3b = Gross extrathyroidal extension invading only strap muscles (sternohyoid, sternothyroid,
thyrohyoid, or omohyoid muscles) from a tumour of any size
• T4 = Includes gross extrathyroidal extension into major neck structures T4a = Gross extrathyroidal
extension invading subcutaneous soft tissues, larynx, Trachea, esophagus, or recurrent laryngeal
nerve from a tumour of any size T4b = Gross extrathyroidal extension invading prevertebral or encasing
the artery or mediastinal Vessels from a tumour of any size
• T4a = Gross extrathyroidal extension invading subcutaneous soft tissues, larynx, Trachea,
esophagus, or recurrent laryngeal nerve from a tumour of any size
• T4b = Gross extrathyroidal extension invading prevertebral or encasing the artery or mediastinal
Vessels from a tumour of any size
• Regional lymph nodes (N): NX = Regional lymph nodes cannot be assessed N0 = No evidence of
locoregional lymph node metastasis N0a = One or more cytologically or histologically confirmed benign
lymph nodes N0b = No radiologic or clinical evidence of locoregional lymph node metastasis N1=
Metastasis to regional nodes N1a = metastasis to level VI or VII (pretracheal, paratracheal, or
prelaryngeal/ Delphian, or upper Mediastinal) lymph nodes. This can be a unilateral or bilateral disease.
N1b = metastasis to unilateral, bilateral, or contralateral lateral neck lymph (levels I, II, III, IV, or V) or
retropharyngeal lymph nodes
• NX = Regional lymph nodes cannot be assessed
• N0 = No evidence of locoregional lymph node metastasis N0a = One or more cytologically or
histologically confirmed benign lymph nodes N0b = No radiologic or clinical evidence of locoregional
lymph node metastasis
• N0a = One or more cytologically or histologically confirmed benign lymph nodes
• N0b = No radiologic or clinical evidence of locoregional lymph node metastasis
• N1= Metastasis to regional nodes N1a = metastasis to level VI or VII (pretracheal, paratracheal, or
prelaryngeal/ Delphian, or upper Mediastinal) lymph nodes. This can be a unilateral or bilateral disease.

Page 29

81
N1b = metastasis to unilateral, bilateral, or contralateral lateral neck lymph (levels I, II, III, IV, or V) or
retropharyngeal lymph nodes
• N1a = metastasis to level VI or VII (pretracheal, paratracheal, or prelaryngeal/ Delphian, or upper
Mediastinal) lymph nodes. This can be a unilateral or bilateral disease.
• N1b = metastasis to unilateral, bilateral, or contralateral lateral neck lymph (levels I, II, III, IV, or V) or
retropharyngeal lymph nodes
• Distant Metastasis (M) M0 = No distant metastasis M1 = Distant metastasis
• M0 = No distant metastasis
• M1 = Distant metastasis
• Tx = Primary Tumor cannot be assessed
• T0 = No evidence of primary Tumor
• T1 = Tumor ≤ 2cm in greatest dimension limited to the thyroid T1a = Tumor ≤ 1 cm in greatest
dimension limited to the thyroid T1b = Tumor >1cm but ≤ 2 cm in greatest dimension limited to the
thyroid
• T1a = Tumor ≤ 1 cm in greatest dimension limited to the thyroid
• T1b = Tumor >1cm but ≤ 2 cm in greatest dimension limited to the thyroid
• T2 = Tumor >2cm but ≤ 4 cm in greatest dimension limited to the thyroid
• T3 = Tumor >4 cm limited to the thyroid or gross extrathyroidal extension invading only strap muscles
T3a = Tumor > 4cm limited to the thyroid T3b = Gross extrathyroidal extension invading only strap
muscles (sternohyoid, sternothyroid, thyrohyoid, or omohyoid muscles) from a tumour of any size
• T3a = Tumor > 4cm limited to the thyroid
• T3b = Gross extrathyroidal extension invading only strap muscles (sternohyoid, sternothyroid,
thyrohyoid, or omohyoid muscles) from a tumour of any size
• T4 = Includes gross extrathyroidal extension into major neck structures T4a = Gross extrathyroidal
extension invading subcutaneous soft tissues, larynx, Trachea, esophagus, or recurrent laryngeal
nerve from a tumour of any size T4b = Gross extrathyroidal extension invading prevertebral or encasing
the artery or mediastinal Vessels from a tumour of any size
• T4a = Gross extrathyroidal extension invading subcutaneous soft tissues, larynx, Trachea,
esophagus, or recurrent laryngeal nerve from a tumour of any size
• T4b = Gross extrathyroidal extension invading prevertebral or encasing the artery or mediastinal
Vessels from a tumour of any size
• T1a = Tumor ≤ 1 cm in greatest dimension limited to the thyroid
• T1b = Tumor >1cm but ≤ 2 cm in greatest dimension limited to the thyroid
• T3a = Tumor > 4cm limited to the thyroid
• T3b = Gross extrathyroidal extension invading only strap muscles (sternohyoid, sternothyroid,
thyrohyoid, or omohyoid muscles) from a tumour of any size
• T4a = Gross extrathyroidal extension invading subcutaneous soft tissues, larynx, Trachea,
esophagus, or recurrent laryngeal nerve from a tumour of any size
• T4b = Gross extrathyroidal extension invading prevertebral or encasing the artery or mediastinal
Vessels from a tumour of any size
• NX = Regional lymph nodes cannot be assessed

Page 30

82
• N0 = No evidence of locoregional lymph node metastasis N0a = One or more cytologically or
histologically confirmed benign lymph nodes N0b = No radiologic or clinical evidence of locoregional
lymph node metastasis
• N0a = One or more cytologically or histologically confirmed benign lymph nodes
• N0b = No radiologic or clinical evidence of locoregional lymph node metastasis
• N1= Metastasis to regional nodes N1a = metastasis to level VI or VII (pretracheal, paratracheal, or
prelaryngeal/ Delphian, or upper Mediastinal) lymph nodes. This can be a unilateral or bilateral disease.
N1b = metastasis to unilateral, bilateral, or contralateral lateral neck lymph (levels I, II, III, IV, or V) or
retropharyngeal lymph nodes
• N1a = metastasis to level VI or VII (pretracheal, paratracheal, or prelaryngeal/ Delphian, or upper
Mediastinal) lymph nodes. This can be a unilateral or bilateral disease.
• N1b = metastasis to unilateral, bilateral, or contralateral lateral neck lymph (levels I, II, III, IV, or V) or
retropharyngeal lymph nodes
• N0a = One or more cytologically or histologically confirmed benign lymph nodes
• N0b = No radiologic or clinical evidence of locoregional lymph node metastasis
• N1a = metastasis to level VI or VII (pretracheal, paratracheal, or prelaryngeal/ Delphian, or upper
Mediastinal) lymph nodes. This can be a unilateral or bilateral disease.
• N1b = metastasis to unilateral, bilateral, or contralateral lateral neck lymph (levels I, II, III, IV, or V) or
retropharyngeal lymph nodes
• M0 = No distant metastasis
• M1 = Distant metastasis
• Staging according to age: Age <55 years
• Any T, any N, M0 – stage 1
• Any T, any N, M 1 – stage 2
• Age ≥55 years
• Stage 1 - T1/T2, N0, M0
• Stage 2 - T1/T2, N1, M0 and T3a/T3b, Any N, M0
• Stage 3 - T4a, Any N, M0
• Stage 4a - T4b, Any N, M0
• Stage 4b - Any T, Any N, M1
Option A: Stage I
• For individuals aged < 55 years, stage I is defined as having any T, N, and M0.
Option C: Stage III
• Stage III is usually seen in those > 55 years of age.
Option D: Stage IV
• Stage IV is usually seen in those > 55 years of age.
• It demarcates T4b, any N and M0. (i.e. T4 = gross extrathyroidal extension beyond strap muscles;
T4b = Gross extrathyroidal extension invading prevertebral or encasing the artery or mediastinal
vessels from a tumor of any size.

Page 31

83
Page 32

84
Thyroid Part 2
1. A 13-year-old male presents to the surgery OPD with complaints of soft swelling in front of his neck
associated with difficulty swallowing for the last 5 months. He also reports pain in the skin around the
swelling. His neck examination reveals a soft midline swelling in front of the neck, which moves with
deglutition and protrusion of the tongue. Which of the following statements is true about the given
condition?
(or)
Which of the following statements is true for a 13-year-old male presenting with a soft midline neck
swelling that moves with deglutition and protrusion of the tongue?

A. The most common location is sub hyoid


B. Occurs due to Trauma
C. Follicular carcinoma is seen in 1% of patients
D. Treated by Total Thyroidectomy
----------------------------------------
2. A 35-year-old male presents to the surgery OPD with complaints of swelling in the neck for the last 4
months. He is afebrile, has a blood pressure of 140/80mmHg, a pulse rate of 96 beats per minute, and
a respiratory rate 16 breaths per minute . His neck examination reveals a mass in the neck, which is a
mobile node at level III. An FNAC of the nodule shows thyroid follicular cells in the node, suggestive of
a lateral aberrant thyroid. The prognosis for this patient is assessed by the AMES score. Which of the
following is included in the AMES score?
(or)
Which of the following components is included in the AMES score for a 35-year-old male with neck
swelling and FNAC suggestive of a lateral aberrant thyroid?
A. Age
B. Grade of lymphadenopathy
C. Gender
D. Histological grade of the tumor
----------------------------------------
3. Which genetic defect is associated with a presentation of neck swelling and severe hearing loss
since early childhood in a 22-year-old female with a large goiter?
(or)

85
A 22-year-old female presents to the surgery OPD with complaints of increased swelling in her neck
over the previous 2 years. She has had a medical history of severe hearing loss since early childhood.
Her main mode of communication is signing and lip-reading. The patient is unaware of any family
history of thyroid disease, goiter, or hearing loss. Her partner also has hearing loss due to an unknown
cause. On examination, she has a large goiter extending retrosternally with small irregular nodules, no
audible bruit, and a negative Pemberton's sign. Which of the following genetic defects causes this
presentation?
A. Werner syndrome
B. Turcot syndrome
C. Cowden syndrome
D. Pendred syndrome
----------------------------------------
4. Which nerve is most commonly injured during total thyroidectomy?
(or)
A 50-year-old female presents to the surgery OPD with complaints of swelling in front of her neck for
the last 2 months. She is afebrile, has a blood pressure of 140/90 mmHg, a pulse rate of 90 beats per
minute, and a respiratory rate of 17 breaths per minute. Her examination reveals a swelling with
multiple nodules in front of the neck, which moves with deglutition. Her TSH is also low. Her features
and laboratory test results are suggestive of a toxic nodular goiter. She is taken in for a total
thyroidectomy after achieving a euthyroid state. Postoperatively, she complains of a change in voice.
Which one of the following is the most common nerve injury during thyroidectomy?
A. External laryngeal nerve
B. Recurrent laryngeal nerve
C. Vagus nerve
D. Facial nerve
----------------------------------------
5. A 40-year-old female presents to the surgery OPD with complaints of swelling in front of her neck for
the last 7 months. Her examination reveals a swelling with multiple nodules in front of the neck. Her
TSH is also low. Her facial features and laboratory test results are suggestive of a toxic nodular goiter.
After achieving a euthyroid state, she is taken in for surgery. Name the procedure shown below that
could be done for this patient.
(or)
Identify the procedure shown below that can be performed for a 40-year-old female with toxic nodular
goiter?

Page 2

86
A. Near-total thyroidectomy
B. Hartley Dunhill procedure
C. Total Thyroidectomy
D. Subtotal Thyroidectomy
----------------------------------------
6. A 35-year-old male is brought in by his mother with a complaint of swelling in the neck region for the
last 2 months. He is a known case of psychiatric illness. A nodule in the left lobe of the thyroid is noted
on his neck examination. The ultrasound of the thyroid gland shows a non-cystic solid mass. He is
further evaluated with a radionuclide scan, which reveals a cold nodule. What would be the
management of this patient?
(or)
What will management be for a 35-year-old male with neck swelling, a non-cystic solid mass on USG,
and a cold nodule on radionuclide scan?
A. Lobectomy
B. Hemithyroidectomy
C. Propylthiouracil
D. Radioactive iodine ablation
----------------------------------------
7. Which statement is correct for a female pediatric patient presenting with painful neck swelling and
difficulty swallowing with a history of upper respiratory tract infection one week ago?
(or)
A 4-year-old girl is brought in by her mother with a complaint of left-sided anterior neck swelling lasting
about a week. Her mother states that the swelling is painful and has progressively increased in size
over the past week, which is associated with difficulty swallowing. She has a fever of 100° F, a pulse
rate of 100 beats per minute, and a respiratory rate of 20 breaths per minute. She has a history of being
treated for an upper respiratory tract infection (URTI) a week ago. On examination, there is no
hoarseness, difficulty breathing, or features suggestive of hyperthyroidism or hypothyroidism. Which
one of the following statements is correct about the given condition?
A. More common in adults
B. The most common cause is the presence of pyriform sinus
C. Most common organism: legionella

Page 3

87
D. Most patients are managed conservatively
----------------------------------------
8. A 40-year-old female presents to the Surgery OPD with complaints of hard swelling in front of her
neck for the last 1 month. Her neck examination reveals a hard woody thyroid gland with fixation to
surrounding tissues. Her clinical features and examination findings are suggestive of Riedel’s
thyroiditis. Which of the following is true about Riedel’s thyroiditis?
(or)
Which of the following is true about Riedel's thyroiditis ?
A. It is associated with sclerosing cholangitis
B. The investigation preferred is FNAC
C. Histopathology shows complete replacement of thyroid and parathyroid glands by non-fibrous tissue
D. Vitamin D and Levothyroxine supplementation are needed for at least 2 years.
----------------------------------------
9. A 45-year-old female presented to the surgery OPD with a complaint of weight gain and decreased
appetite associated with lethargy and weakness for the last 6 months. She has a BMI of 33. She is
afebrile, has a blood pressure of 140/100 mmHg, a pulse rate of 96 beats per minute, and a respiratory
rate of 18 breaths per minute. On laboratory investigations, her anti-TPO is positive. Which of the
following cancer risks is elevated in this patient?
(or)
In a 45-year-old female presenting with weight gain, lethargy, weakness, decreased appetite, and
positive anti-TPO, which of the following malignancy risks is increased?
A. Papillary carcinoma
B. Both Papillary thyroid carcinoma and lymphoma
C. Lymphoma
D. None of the above
----------------------------------------
10. A 30-year-old female presented to the surgery OPD with complaints of swelling in front of her neck,
hoarseness of voice, trouble swallowing and feels like food is getting stuck in her throat, and constant
cough for the previous 4 months. Her neck examination reveals a hard mass in front of her neck. A fine
needle aspiration and cytology (FNAC) of the thyroid lump is done, which confirms a thyroid carcinoma
with no lymph node involvement. Her biopsy picture is given below. What is the most likely diagnosis?
(or)
A 30-year-old female presented with neck swelling, hoarseness of voice, and dysphagia. FNAC
confirms a thyroid carcinoma with no lymph node involvement. Her biopsy picture is given below. What
is the most likely diagnosis?

Page 4

88
A. Papillary carcinoma
B. Follicular carcinoma
C. Medullary carcinoma
D. None of the above
----------------------------------------
11. Which is the correct statement about thyroidectomy in a 50-year-old female presenting with midline
multinodular neck swelling?
(or)
A 50-year-old female presents to surgery OPD with a complaint of swelling in front of the neck for the
last 3 months. Her neck examination reveals a swelling in the midline of the neck with multiple nodules,
which moves with deglutition. Her laboratory investigation is suggestive of an over-reactive thyroid
gland. She is planned for a total thyroidectomy. Which of the following statements is correct about
thyroidectomy?
A. Superior thyroid vessels are ligated close to the thyroid
B. Inferior thyroid vessels are ligated away from the thyroid
C. The vagus nerve is most vulnerable to injury in the vicinity of the ligament of the Berry
D. Incidentally removed parathyroid gland cannot be auto-transplanted in the sternocleidomastoid
muscle
----------------------------------------
12. A 30-year-old female presents to the surgery OPD for an elective thyroidectomy. She is a known
case of Grave's disease and is admitted for a total thyroidectomy. All preoperative preparation orders
for the surgery are advised, such as administering I.V Lugol's iodine and prescribed drugs β blockers &
Methimazole. Although she didn't receive any drugs due to the unavailability of the drugs in the
pharmacy and was operated on. What could be an expected complication during surgery?
(or)
What complications may be expected during Total Thyroidectomy for a 30-year-old female with Grave's
disease, who did not receive preoperative I.V Lugol's iodine, β blockers, and Methimazole?
A. Parathyroid insufficiency
B. Perioperative hemorrhage
C. Carpopedal spasm
D. None of the above

Page 5

89
----------------------------------------
13. Which of the following eye sign are seen in graves ophthalmopathy?
(or)
A 35-year-old lady presents to the medical outpatient department with a complaint of neck swelling for
the last 1 month. She states that the swelling is gradually increasing, and she also feels a racing
heartbeat and extreme weight loss despite a good appetite. Her pulse rate is 109/min, BP is 130/90
mmHg, respiratory rate is 13/min, and temperature is 37°C (98.6°F). Examination reveals fine hand
tremors and central neck swelling that moves with swallowing. Which of the following is an eye sign of
the above presentation ?
A. Trousseau'ssign
B. Boston sign
C. Woltman sign
D. Chvostek sign
----------------------------------------
14. A 54-year-old female presents to the surgical outpatient department with the complaint of rapidly
increasing central neck swelling over the last 4 weeks. It is associated with weight loss, anorexia, and a
change in voice character but not with dysphagia. Her mother had a similar neck swelling and was
diagnosed with thyroid cancer. Her pulse rate is 100/min, BP is 120/80, respiratory rate is 14/min, and
temperature is 98.6° F. Examination reveals a hard nodule over the neck that moves with swallowing;
however, there were no palpable lymph nodes. Based on her history and clinical examination, which
type of thyroid cancer she might be suffering from?
(or)
Which type of thyroid cancer is most likely in a 54-year-old female with increasing neck swelling, weight
loss, change in voice, and a hard nodule moving with swallowing?
A. Follicular carcinoma
B. Papillary carcinoma
C. Anaplastic carcinoma
D. Medullary carcinoma
----------------------------------------
15. A 35-year-old female presents to the surgical outpatient department with complaints of palpitations,
weight loss, and menstrual irregularities from the last 1 month. Her pulse rate is 110/min, BP is 130/85
mmHg, respiratory rate is 16/min, and temperature is 99° F. Examination reveals a previous surgical
scar over the neck and fine tremors in the hands. She underwent a near-total thyroidectomy for diffuse
toxic goiter in the past and remained well for a year until similar symptoms started again. Her recent
thyroid function tests are consistent with hyperthyroidism. She was subsequently diagnosed with
recurrent thyrotoxicosis. What would be the treatment of choice for this patient?
(or)
What is the treatment of choice for a 35-year-old female with a history of near-total thyroidectomy and
recurrent thyrotoxicosis presenting with palpitations, weight loss, and menstrual irregularities?
A. Further surgery
B. Radioiodine followed by surgery
C. Radioiodine

Page 6

90
D. Observation and follow-up
----------------------------------------
16. A 29-year-old male presents to the surgery OPD complaining of swelling on the right side of the
neck for the last 6 months. He is afebrile, has a blood pressure of 130/70 mmHg, a pulse rate of 86
beats per minute, and a respiratory rate of 17 breaths per minute. His neck examination reveals a solid
mass on the right side of his neck. A thyroid scan reveals the following finding, shown in the image.
What is the most likely diagnosis in this case?
(or)
What is the most probable diagnosis for a 29-year-old male presenting with a solid mass on the right
side of his neck, and the following thyroid scan image?

A. Papillary carcinoma thyroid


B. Lateral aberrant thyroid
C. Hypersecreting adenoma
D. Graves’ disease
----------------------------------------
17. Which of the following is the true statement about complications of total thyroidectomy
postoperatively?
(or)
A 45-year-old female presents to surgery OPD with a complaint of swelling in the front of the neck for
the last 7 months. Her neck examination reveals a swelling in the midline of the neck with multiple
nodules, which moves with deglutition. Her clinical features and laboratory investigation suggest
multinodular goiter. She has been scheduled for a total thyroidectomy. Postoperatively, she developed
hoarseness of voice. Which of the following statements about the complications of thyroidectomy is
true?
A. A thyrotoxic crisis occurs because of inadequate preoperative preparation
B. The most common cause of respiratory obstruction is nerve injury
C. The recurrent laryngeal nerve is more commonly injured than the external branch of the superior
laryngeal nerve
D. Parathyroid insufficiency characteristically occurs on days 5-10 after Surgery due to vascular
infarction
----------------------------------------

Page 7

91
18. A 42-year-old female presents in the surgical outpatient department with a complaint of a central,
discrete neck swelling from the last 1 month, gradually increasing in size. She has a family history of
thyroid cancer. Examination reveals a 3-cm-long, hard, irregular, solitary nodule in the front of the neck
with multiple cervical and axillary lymph nodes. Histologic examination of the biopsied specimen
reveals psammoma bodies and Orphan Annie eye nuclei consistent with thyroid papillary carcinoma.
She subsequently undergoes a total thyroidectomy. Which of the following is an indication for
radioactive iodine therapy in this patient?
(or)
What is an indication for radioactive iodine therapy in a 42-year-old female who underwent total
thyroidectomy for thyroid papillary carcinoma ?
A. 2cm-sized metastatic lymph node
B. Absence of nodal or distant metastasis
C. Vascular invasion
D. Absence of extrathyroidal extension
----------------------------------------
19. A 42-year-old woman presents to the surgical outpatient department with a distinct right-sided neck
swelling that has been gradually increasing in size for the last 1 month. There is no relevant family
history. The patient is clinically euthyroid, and the rest of the physical examination is unremarkable
except for a 3 cm smooth, firm mass on the right side of the thyroid that moves with swallowing. Thyroid
function tests are normal. A fine-needle aspiration biopsy shows an amyloid-rich stroma on a thyroid
nodule's H and E stain. What is the diagnosis?
(or)
42-year-old female presents with neck swelling. FNAC shows an amyloid-rich stroma on a thyroid
nodule's H & E stain. What is the diagnosis?

A. Medullary thyroid carcinoma


B. Systemic amyloidosis
C. Thyroid lymphoma
D. Papillary thyroid carcinoma
----------------------------------------
20. A 51-year-old woman presents to the surgical outpatient department with a visible mass on the
neck that has been present for the last 6 weeks. It is associated with a change in voice quality and
difficulty swallowing. Her pulse rate is 99/min, BP is 130/90, respiratory rate is 17/min, and temperature
is 98.6° F. Physical examination reveals an enlarged and hard right lobe of the thyroid gland. A thyroid

Page 8

92
ultrasound reveals a 4 cm hypoechoic nodule on the right lobe with cystic degeneration and a normal
isthmus and theleft lobe. Her thyroid profile shows euthyroid state. Thyroid scintigraphy shows an
enlarged right lobe with a cold nodule in the lower part. A fine-needle aspiration biopsy reveals
neoplasia with hurthle cells. What would be the best management plan for this patient?
(or)
What is the recommended management plan for a 51-year-old woman with a neck mass, hoarseness
of voice, dysphagia, and FNAC showing neoplasia with Hürthle cells?
A. Total thyroidectomy + Central neck dissection
B. Hemithyroidectomy
C. Total thyroidectomy + Modified radical neck dissection
D. Near-Total Thyroidectomy with cervical LN I and II dissection
----------------------------------------
21. Which type of thyroid malignancy is most likely to be suspected in a 45-year-old female with a
history of exposure to ionizing radiation in childhood?
(or)
Which type of thyroid malignancy is most likely to be suspected in a 45-year-old female with a history of
exposure to ionizing radiation in childhood?
A. Papillary thyroid cancer
B. Medullary thyroid cancer
C. Anaplastic thyroid cancer
D. Thyroid lymphoma
----------------------------------------
22. A 26-year-old male presents to the surgical outpatient department with the complaint of localized
neck swelling for the last month. He denies any tremors, weight loss, or palpitations. He states that his
grandfather had a similar neck swelling with multiple other small neck swellings and was later
diagnosed with thyroid cancer. His pulse rate is 105/min, BP is 135/90, respiratory rate is 17/min, and
temperature is 99°F. Examination revealed a discrete, soft, non-tender thyroid swelling but no lymph
nodes. Ultrasound of the neck reveals an isolated cystic swelling of the gland. He is concerned that he
might have thyroid cancer, just like his grandfather. How much risk of malignancy does the man carry?
(or)
What is the estimated risk of malignancy in a 26-year-old male with a family history of thyroid cancer
and on USG isolated cystic swelling of the thyroid gland?
A. 48%
B. 24%
C. 12%
D. 3%
----------------------------------------
23. A 30-year-old female presents to the surgical outpatient department with complaints of swelling in
front of her neck, hoarseness of voice, and constant cough for the last 4 months. She also reports
trouble swallowing and feels food getting stuck in her throat. She is afebrile, has a blood pressure of
120/70 mmHg, a pulse rate of 80 beats per minute, and a respiratory rate of 17 breaths per minute. Her

Page 9

93
neck examination reveals a hard mass in front of her neck. Fine needle aspiration and cytology (FNAC)
of the thyroid mass are done, which reveal a papillary thyroid carcinoma. Which of the following
statements regarding the patient’s current condition is true?
(or)
Which of the following statement is true for a 30-year-old female presents with neck swelling,
hoarseness of voice, and chronic cough and FNAC reveals papillary thyroid carcinoma?
A. Papillary thyroid carcinomas (PTC) more than 1 cm in diameter are microcarcinomas
B. Only 1% of thyroid carcinomas diagnosed are papillary thyroid carcinoma
C. Microscopic evidence of multi-centric disease is present in 70% of cases
D. The most important risk factor for PTC is a family history
----------------------------------------

Correct Answers
Question Correct Answer

Question 1 1
Question 2 1
Question 3 4
Question 4 1
Question 5 3
Question 6 2
Question 7 2
Question 8 1
Question 9 2
Question 10 1
Question 11 1
Question 12 2
Question 13 2
Question 14 1
Question 15 3
Question 16 3
Question 17 1
Question 18 3
Question 19 1
Question 20 2
Question 21 1
Question 22 2

Page 10

94
Question 23 3

Solution for Question 1:


Option A: The most common location is sub hyoid
• The patient's clinical features of soft midline swelling that moves with deglutition and protrusion of the
tongue suggest a thyroglossal cyst. Its most common location is the sub hyoid region.
• It is a congenital cystic remnant of the thyroglossal duct.
• Most commonly found in the sub hyoid region, always close to the midline. Mostly asymptomatic
unless infected.
• It is congenital, but the usual presentation age is 15 to 30.
• There is an increased risk of malignancy (1%), usually papillary carcinoma (85%). Squamous cells,
Hurthle cells, and anaplastic cancers have also been reported.
• The cyst moves with deglutition, and it is the only swelling that moves up on tongue protrusion.
• The treatment for a thyroglossal duct cyst is sistrunk operation : En bloc excision of the central part of
the hyoid bone and the cyst to minimize recurrence.
• Infected thyroglossal duct cysts require immediate antibiotic treatment before surgery.
Option B: Occurs due to Trauma
• A thyroglossal cyst is a remnant of the thyroglossal duct.
• It is the most commonly encountered congenital cervical anomaly.
Option C: Follicular carcinoma is seen in 1% of patients
• Follicular cancer is frequently present in the long standing goitrous thyroid.
• Papillary carcinoma is seen in 1% of patients with thyroglossal cysts.
Option D: Treated by Total Thyroidectomy
• Total thyroidectomy (thyroid gland removal) is indicated in thyroid cancer, toxic multinodular goiter,
toxic adenomas, goiter with compressive symptoms, and Graves’ disease.

Solution for Question 2:


Option A: Age
• The patient mentioned above has the clinical features of neck swelling, and FNAC results suggest
lateral aberrant thyroid.
• The AMES score is used for the predictive assessment of lateral aberrant thyroid, and age is one of
the criteria for assessing this score.
• AMES score:
• A – Age
• M – Metastasis
• E – Extrathyroidal spread

Page 11

95
• S – Size of the tumor
• Lateral aberrant thyroid is a misnomer.
• It is the metastatic node from papillary thyroid carcinoma and hence usually shows diminished uptake
(cold nodule)

Option B: Grade of lymphadenopathy
• The grade of lymph node involvement can assess lateral aberrant thyroid.
• AMES score does not include the grade of lymphadenopathy as a factor for the prognostic
assessment.
Option C: Gender
• Lateral aberrant thyroid occurs irrespective of gender.
• AMES score does not include gender as a factor.
Option D: Histological grade of the tumor
• The histological grade of the tumor is one of the factors used in the AGES score to assess the
prognosis of the disease.
• AMES score does not include the Histology grade of the tumor as a factor.

Solution for Question 3:


ANSWER
Option D: Pendred syndrome
• The patient mentioned above has clinical features of sensorineural hearing loss associated with
goiter. This presentation is typical of Pendred syndrome.
• Presentation of Pendred syndrome:
• Hearing loss or deafness Speech impairment Electrolyte/homeostasis imbalance Goitre Enlarged
vestibular aqueduct Hypothyroidism
• Hearing loss or deafness
• Speech impairment
• Electrolyte/homeostasis imbalance
• Goitre
• Enlarged vestibular aqueduct
• Hypothyroidism
• Hearing loss or deafness
• Speech impairment
• Electrolyte/homeostasis imbalance
• Goitre
• Enlarged vestibular aqueduct

Page 12

96
• Hypothyroidism
• Pendred Syndrome is the combination of sensorineural hearing loss with dyshormonogenetic goitre.
• It is autosomal recessive. The affected gene is located on chromosome 7q.
• There is a mutation of the PDS gene, which encodes for pendrin, the protein that transports iodide to
the luminal surface of the follicular cell.
Other options
Option A: Werner syndrome
• Werner syndrome (MEN1) occurs due to germline mutation in the MEN1 gene located in chromosome
11 that encodes menin. It is autosomal dominant hereditary cancer syndrome.
• Symptoms and signs are shorter than average height, thin arms and legs, voice changes, thinning and
greying hair, skin changes, and unusual facial features.
Option B: Turcot syndrome
• Turcot syndrome is characterized by the following:
• Familial adenomatous polyposis (FAP). Primary CNS tumors (astrocytoma or medulloblastoma).
• Familial adenomatous polyposis (FAP).
• Primary CNS tumors (astrocytoma or medulloblastoma).
• Familial adenomatous polyposis (FAP).
• Primary CNS tumors (astrocytoma or medulloblastoma).
Option C: Cowden syndrome
• Cowden's syndrome is an autosomal dominant disorder due to mutations in PTEN.
• Associated cancers include:
• Breast carcinoma Thyroid carcinoma Endometrial carcinoma
• Breast carcinoma
• Thyroid carcinoma
• Endometrial carcinoma
• Breast carcinoma
• Thyroid carcinoma
• Endometrial carcinoma

Solution for Question 4:


Option A: External laryngeal nerve
• Postoperatively, the most common nerve injured after total thyroidectomy would be the external
laryngeal nerve.
• Total thyroidectomy is a procedure in which the whole of the thyroid gland is removed due to certain
thyroid conditions such as cancer or hyperactive thyroid gland, e.g., Graves’ disease or multinodular
toxic goiter.

Page 13

97
• Complications of thyroidectomy: Nerve injuries The most common injured nerve is the External branch
of the superior laryngeal nerve ( External laryngeal nerve) due to its proximity to the superior thyroid
artery. Leads to a loss of tension in the vocal cord with diminished power and range in the voice.
Recurrent laryngeal nerve - may be unilateral or bilateral, transient or permanent. Cervical sympathetic
trunk.
• Nerve injuries The most common injured nerve is the External branch of the superior laryngeal nerve
( External laryngeal nerve) due to its proximity to the superior thyroid artery. Leads to a loss of tension
in the vocal cord with diminished power and range in the voice. Recurrent laryngeal nerve - may be
unilateral or bilateral, transient or permanent. Cervical sympathetic trunk.
• The most common injured nerve is the External branch of the superior laryngeal nerve ( External
laryngeal nerve) due to its proximity to the superior thyroid artery.
• Leads to a loss of tension in the vocal cord with diminished power and range in the voice.
• Recurrent laryngeal nerve - may be unilateral or bilateral, transient or permanent.
• Cervical sympathetic trunk.
• Nerve injuries The most common injured nerve is the External branch of the superior laryngeal nerve
( External laryngeal nerve) due to its proximity to the superior thyroid artery. Leads to a loss of tension
in the vocal cord with diminished power and range in the voice. Recurrent laryngeal nerve - may be
unilateral or bilateral, transient or permanent. Cervical sympathetic trunk.
• The most common injured nerve is the External branch of the superior laryngeal nerve ( External
laryngeal nerve) due to its proximity to the superior thyroid artery.
• Leads to a loss of tension in the vocal cord with diminished power and range in the voice.
• Recurrent laryngeal nerve - may be unilateral or bilateral, transient or permanent.
• Cervical sympathetic trunk.
• The most common injured nerve is the External branch of the superior laryngeal nerve ( External
laryngeal nerve) due to its proximity to the superior thyroid artery.
• Leads to a loss of tension in the vocal cord with diminished power and range in the voice.
• Recurrent laryngeal nerve - may be unilateral or bilateral, transient or permanent.
• Cervical sympathetic trunk.
Option B: Recurrent laryngeal nerve
• Recurrent laryngeal nerve (RLN) can be damaged during thyroidectomy, but it may be unilateral or
bilateral.
• It could be transient or permanent damage of RLN, but the most common nerve injured during
thyroidectomy is the external laryngeal nerve.
Option C: Vagus nerve
• The Vagus nerve is damaged in gastroparesis.
• It is unlikely to get damaged during thyroidectomy.
Option D: Facial Nerve
• The facial nerve is damaged in the case of Bell's palsy.
• It is unlikely to get damaged during thyroidectomy as the external laryngeal nerve is the most common
nerve to get injured during the procedure.

Page 14

98
Solution for Question 5:
Option C: Total Thyroidectomy
Total Thyroidectomy – Removal of all visible thyroid tissue.
• The case mentioned above underwent a total thyroidectomy shown in the picture.
• There are various thyroidectomy indications, including thyroid cancer, multinodular toxic goiter, and
hyperthyroidism.
• Removal of the whole thyroid gland or a part of the thyroid gland depends on the underlying condition.

Option A: Near-total Thyroidectomy

Page 15

99
• Complete resection on one side while leaving a remnant of thyroid tissue (< 1g) on the contralateral
side adjacent to the recurrent laryngeal nerve at the ligament of the berry.
Option B: Hartley Dunhill Procedure
• Leaving 4-6 grams of thyroid in one lobe with the removal of the rest of the thyroid gland is done in
Hartley Dunhill Procedure.

Option D: Subtotal Thyroidectomy


• Leaving 3-4 grams of thyroid tissue in each lobe with the removal of the rest of the thyroid gland. The
typical reason to leave a remnant at the berry's ligament is to preserve the recurrent laryngeal nerve
and blood supply to the parathyroid.

Solution for Question 6:

Page 16

100
Option B: Hemithyroidectomy
• In this case, the chances of malignancy are high as the patient's ultrasound shows a solid mass, and
a radioactive scan shows a cold nodule. This should be confirmed with FNAC.
• Thus, management of the above case would be hemithyroidectomy.
• Hemithyroidectomy is the removal of one lobe with the isthmus.

• Areas that trap less radioactivity than the surrounding gland are termed COLD, whereas areas that
demonstrate increased activity are called HOT.
• Risk of malignancy:
• Hot - < 5% Cold - >20 %
• Hot - < 5%
• Cold - >20 %

• Hot - < 5%
• Cold - >20 %
Option A: Lobectomy
• Lobectomy is done to remove one or two lobes of the thyroid gland.
• Lobectomy is indicated if one or two lobes are hyperactive and produce hyperthyroidism symptoms.
Option C: Propylthiouracil
• Drug used in the case of hyperthyroidism.
• Mechanism of action ; inhibits thyroid hormone by inhibiting TPO, iodination of tyrosine and by
blocking the peripheral conversion of T4 to T3
• In pregnant females: Preferred treatment in first trimester
• So, in this case, propylthiouracil has no role in the management.
Option D: Radioactive iodine ablation

Page 17

101
• It is not indicated in a solitary cold nodule.
• It is useful in managing occult multifocal disease, facilitating thyroglobulin as a tumor marker.

Solution for Question 7:


Option B: The most common cause is the presence of pyriform sinus
• In the case discussed above, the child has features of neck swelling following an upper respiratory
tract infection one week ago. In this case, the diagnosis is suppurative thyroiditis.
• It is more common in children and is marked by the presence of pyriform sinus.
• Acute suppurative thyroiditis: It is a suppurative thyroid infection. The main organisms are
staphylococcus aureus and streptococci. It is more common in children because of the persistent
pyriform sinus. It is preceded by URTI and Otitis media. Normally,the thyroid is resistant to infection
because of its: High iodide content Rich vascular and lymphatic supply Fibrous capsule But in children
because of the persistent pyriform sinus infection occurs. Clinical features include: Pain and tenderness
over the thyroid. Fever, chills, and rigor. Severe neck pain radiating to the jaw or ear. Lab investigation
includes high ESR and raised WBC count. Diagnosis: FNAC shows the presence of neutrophils.
Culture and sensitivity can reveal the causative organism and antibiotic sensitivity. Treatment includes
incision and drainage and antibiotics.
• It is a suppurative thyroid infection.
• The main organisms are staphylococcus aureus and streptococci.
• It is more common in children because of the persistent pyriform sinus.
• It is preceded by URTI and Otitis media.
• Normally,the thyroid is resistant to infection because of its: High iodide content Rich vascular and
lymphatic supply Fibrous capsule But in children because of the persistent pyriform sinus infection
occurs.
• High iodide content
• Rich vascular and lymphatic supply
• Fibrous capsule
• But in children because of the persistent pyriform sinus infection occurs.
• Clinical features include: Pain and tenderness over the thyroid. Fever, chills, and rigor. Severe neck
pain radiating to the jaw or ear. Lab investigation includes high ESR and raised WBC count.
• Pain and tenderness over the thyroid.
• Fever, chills, and rigor.
• Severe neck pain radiating to the jaw or ear.
• Lab investigation includes high ESR and raised WBC count.
• Diagnosis: FNAC shows the presence of neutrophils. Culture and sensitivity can reveal the causative
organism and antibiotic sensitivity. Treatment includes incision and drainage and antibiotics.
• FNAC shows the presence of neutrophils.
• Culture and sensitivity can reveal the causative organism and antibiotic sensitivity.

Page 18

102
• Treatment includes incision and drainage and antibiotics.
• It is a suppurative thyroid infection.
• The main organisms are staphylococcus aureus and streptococci.
• It is more common in children because of the persistent pyriform sinus.
• It is preceded by URTI and Otitis media.
• Normally,the thyroid is resistant to infection because of its: High iodide content Rich vascular and
lymphatic supply Fibrous capsule But in children because of the persistent pyriform sinus infection
occurs.
• High iodide content
• Rich vascular and lymphatic supply
• Fibrous capsule
• But in children because of the persistent pyriform sinus infection occurs.
• Clinical features include: Pain and tenderness over the thyroid. Fever, chills, and rigor. Severe neck
pain radiating to the jaw or ear. Lab investigation includes high ESR and raised WBC count.
• Pain and tenderness over the thyroid.
• Fever, chills, and rigor.
• Severe neck pain radiating to the jaw or ear.
• Lab investigation includes high ESR and raised WBC count.
• Diagnosis: FNAC shows the presence of neutrophils. Culture and sensitivity can reveal the causative
organism and antibiotic sensitivity. Treatment includes incision and drainage and antibiotics.
• FNAC shows the presence of neutrophils.
• Culture and sensitivity can reveal the causative organism and antibiotic sensitivity.
• Treatment includes incision and drainage and antibiotics.
• High iodide content
• Rich vascular and lymphatic supply
• Fibrous capsule
• But in children because of the persistent pyriform sinus infection occurs.
• Pain and tenderness over the thyroid.
• Fever, chills, and rigor.
• Severe neck pain radiating to the jaw or ear.
• Lab investigation includes high ESR and raised WBC count.
• FNAC shows the presence of neutrophils.
• Culture and sensitivity can reveal the causative organism and antibiotic sensitivity.
• Treatment includes incision and drainage and antibiotics.
Option A: More common in adults
• Acute suppurative thyroiditis is often preceded by any upper respiratory infection or otitis media.

Page 19

103
• It is more common in children than adults because of the persistent pyriform sinus (the connection
between the oropharynx and thyroid).
Option C: Most common organism: legionella
• The most common organism that causes acute suppurative thyroiditis is either Staphylococcus aureus
or streptococci.
• Usually, cultural sensitivity is done to determine the type of organism so that a suitable antibiotic can
be given to resolve the infection.
Option D: Most patients are managed conservatively
• Patients with acute suppurative thyroiditis are not managed conservatively.
• The treatment includes incision, drainage, and a suitable antibiotic sensitive to the causative
organism.

Solution for Question 8:


Option A: It is associated with sclerosing cholangitis
• Riedel’s Thyroiditis: Also known as Invasive Fibrous thyroiditis or Riedel’s stroma. Characterized by
the complete replacement of thyroid and parathyroid glands by fibrous tissue.
• Also known as Invasive Fibrous thyroiditis or Riedel’s stroma.
• Characterized by the complete replacement of thyroid and parathyroid glands by fibrous tissue.
• Also known as Invasive Fibrous thyroiditis or Riedel’s stroma.
• Characterized by the complete replacement of thyroid and parathyroid glands by fibrous tissue.
Primary autoimmune etiology associated with retro-orbital fibrosis:
• Periorbital fibrosis
• Retroperitoneal fibrosis
• Mediastinal fibrosis

Option B: The investigation preferred is FNAC
• Fine needle aspiration and cytology (FNAC) is indicated in the thyroid diseases such as cancer.
• Diagnosis of Reidel's thyroiditis is confirmed by tru cut biopsy because the firm and fibrous nature of
the gland renders FNAC inadequate.
Option C: Histopathology shows complete replacement of thyroid and parathyroid glands by non-fibrou
s tissue
• Riedel's thyroiditis is also called Invasive Fibrous thyroiditis.
• It is characterized by the complete replacement of thyroid and parathyroid glands by fibrous tissue.
Option D: Vitamin D and Levothyroxine supplementation are needed for at least 2 years
• Lifelong calcium and Vitamin D supplementation due to permanent hypoparathyroidism.
• Corticosteroids, tamoxifen, mycophenolate mofetil, and rituximab can also be used.

Page 20

104
Solution for Question 9:
Option B: Both papillary thyroid cancer and lymphoma
• The patient mentioned above has symptoms of hypothyroidism with anti -TPO positive, which is
suggestive of Hashimoto's Thyroiditis. She is at risk of developing Lymphoma of the thyroid gland and
papillary thyroid cancer
• Hashimoto's Thyroiditis / Lymphocytic thyroiditis/struma lymphomatosis.
• Most common inflammatory disorder of the thyroid and the leading cause of hypothyroidism .
• Autoimmune disorder
• Antibodies are directed against three main antigens
• Thyroglobulin (60%) Thyroid peroxidase (95%) TSH-R (60%) Sodium/iodide symporter (25%): Less
common
• Thyroglobulin (60%)
• Thyroid peroxidase (95%)
• TSH-R (60%)
• Sodium/iodide symporter (25%): Less common
• The gland is diffusely infiltrated by small lymphocytes and plasma cells on microscopic examination.
• Thyroid follicles are lined by Hurthle or Askanazy cells, characterized by abundant eosinophilic,
granular cytoplasm.
• More common in women between the ages of 30 and 50 years old.
• Most common presentation- Hypothyroidism.
• Elevated TSH and the presence of anti TPO antibodies usually confirm the diagnosis.
• Thyroid lymphoma- well-recognized ominous complication.
• Treatment -Lifelong Levothyroxine should be given to these patients.
• Thyroglobulin (60%)
• Thyroid peroxidase (95%)
• TSH-R (60%)
• Sodium/iodide symporter (25%): Less common
Option A: Papillary carcinoma
• Papillary thyroid carcinoma (PTC) is the most common thyroid malignancy and is usually associated
with an excellent prognosis, particularly in young patients.
• It occurs as a complication of Hashimoto's Thyroiditis.
Option C: Lymphoma
• Lymphoma can develop as a complication of Hashimoto's Thyroiditis due to inflammatory disorder of
the thyroid and hypothyroidism.
Option D: None of the above

Page 21

105
• Lymphoma can occur due to autoimmune thyroid diseases as antibodies can destroy antigens like
Thyroglobulin, Thyroid peroxidase (95%), or TSH-R.

Solution for Question 10:


Option A: Papillary carcinoma
• In the case mentioned above, the patient's clinical features of swelling in the front of the neck
associated with hoarseness of voice and biopsy findings suggest Papillary thyroid carcinoma (PTC).
• Pathology of papillary thyroid carcinoma: Papillary projections Optically clear nuclei aka Orphan Annie
Eye Nuclei Pseudo-inclusion bodies Dystrophic calcification- Psammoma bodies
• Papillary projections
• Optically clear nuclei aka Orphan Annie Eye Nuclei
• Pseudo-inclusion bodies
• Dystrophic calcification- Psammoma bodies
• Papillary projections
• Optically clear nuclei aka Orphan Annie Eye Nuclei
• Pseudo-inclusion bodies
• Dystrophic calcification- Psammoma bodies

• Papillary thyroid carcinoma (PTC) is the most common thyroid malignancy and usually is associated
with an excellent prognosis, particularly in young patients.
• Occasionally, a metastatic PTC manifests as a painless lateral neck mass that is clinically detected
before detecting the primary thyroid lesion called lateral aberrant thyroid
• Prognostic risk classification for PTC includes the AGES and AMES scoring systems.
• The absence of distant metastasis at initial treatment and size less than 4cm are other important
positive predictors.

Page 22

106
• Total Thyroidectomy is recommended for all PTCs 1cm or larger
• PTCs less than 1 cm in diameter are defined as microcarcinomas.
Option B: Follicular carcinoma
• Follicular thyroid cancer is the second most common type, making up about 10 to 15% of all thyroid
cancers.

Option C: Medullary carcinoma


• Medullary carcinoma is a rare type of thyroid cancer.
• Medullary thyroid cancer (MTC) arises from parafollicular C cells and is a part of MEN syndrome
• The tumor secrets various tumor markers, including calcitonin and carcinoembryonic antigen (CEA).

Option D: None of the above


• The clinical case and biopsy picture above suggest Papillary carcinoma of the thyroid.

Page 23

107
Solution for Question 11:
Option A: Superior thyroid vessels are ligated close to the thyroid
• The patient mentioned above has features of hyperthyroidism;
• The examination reveals multinodular goiter. In this case, total thyroidectomy is indicated. During
thyroidectomy, Superior Thyroid vessels are ligated close to the thyroid gland.
• A total thyroidectomy is an operation that involves the surgical removal of all of the thyroid gland.
• An indication of thyroidectomy includes; cancer a noncancerous enlargement of the thyroid (goiter)
Overactive thyroid (hyperthyroidism).
• cancer
• a noncancerous enlargement of the thyroid (goiter)
• Overactive thyroid (hyperthyroidism).
• In a thyroidectomy, the superior thyroid artery, and vein are ligated separately and close to the thyroid
to prevent injury of the external branch of the superior laryngeal nerve.
• cancer
• a noncancerous enlargement of the thyroid (goiter)
• Overactive thyroid (hyperthyroidism).
Option B: Inferior thyroid vessels are ligated away from the thyroid
• The inferior thyroid artery is considered the principal blood supplier of the thyroid gland and supplies 4
parathyroid glands.
• These vessels are usually ligated close to the thyroid to prevent vascular infarction of the parathyroid
gland or injury to the recurrent laryngeal nerve.
Option C: The vagus nerve is most vulnerable to injury in the vicinity of the ligament of the Berry
• The recurrent laryngeal nerve is the most vulnerable to injury in the vicinity of the ligament of Berry.
• Bleeding is controlled with gentle pressure (use of electrocautery should be avoided).
Option D: Incidentally removed parathyroid gland cannot be auto-transplanted in the sternocleidomasto
id muscle
• The parathyroid gland can usually be identified within 1 cm of crossing the inferior thyroid artery and
recurrent laryngeal nerve.
• Inadvertently removed parathyroid is divided into 1 mm fragments and can be re-implanted into
individual pockets in the sternocleidomastoid muscle.

Solution for Question 12:


Option B: Perioperative hemorrhage
• The patient mentioned above is a known case of Graves' disease, and inadequate perioperative
preparation can lead to thyroid toxicosis and increased vascularity, which leads to perioperative

Page 24

108
bleeding.
• Preoperative preparation of a patient with thyrotoxicosis: Thyrotoxicosis must be managed medically
before thyroidectomy. If the patient is not properly treated preoperatively, a thyroid storm can be
life-threatening. As the gland will be highly vascular, the risk of perioperative bleeding is higher.
• Thyrotoxicosis must be managed medically before thyroidectomy.
• If the patient is not properly treated preoperatively, a thyroid storm can be life-threatening.
• As the gland will be highly vascular, the risk of perioperative bleeding is higher.
• Thyrotoxicosis must be managed medically before thyroidectomy.
• If the patient is not properly treated preoperatively, a thyroid storm can be life-threatening.
• As the gland will be highly vascular, the risk of perioperative bleeding is higher.
• Inhibits peripheral adrenergic action (tachycardia, palpitations, tremors, anxiety) of thyrotoxicosis.
• Do not directly inhibit thyroid hormone synthesis.
• But the hypermetabolic state can remain or progress with beta-blocker treatment alone.
• Blocks thyroid hormone synthesis by inhibiting thyroid peroxidase.
• Propylthiouracil also inhibits peripheral conversion of T4 to T3.
• They effectively suppress the pituitary thyroid axis.
• Inhibits peripheral conversion of T4 to T3.
• Inhibits thyroid hormone release.
• Reduces vascularity of the gland.
• Decreases the risk of precipitating thyroid storm.
Option A: Parathyroid insufficiency
• Parathyroid insufficiency occurs due to vascular infarction of the parathyroid gland. It most commonly
presents on the 2nd – 5th day of surgery, and its manifestations are carpopedal spasm, tingling, and
numbness over the perioral region.
• Treatment: Mild to moderate symptoms (calcium level >8gm/dl): oral calcium supplementation.
Severe symptoms (<8gm /dl): IV calcium gluconate.
• Mild to moderate symptoms (calcium level >8gm/dl): oral calcium supplementation.
• Severe symptoms (<8gm /dl): IV calcium gluconate.
• Mild to moderate symptoms (calcium level >8gm/dl): oral calcium supplementation.
• Severe symptoms (<8gm /dl): IV calcium gluconate.
Option C: Carpopedal spasm
• It occurs due to parathyroid insufficiency, which most commonly presents on the 2nd – 5th day of
Surgery.
• Treatment includes oral or intravenous calcium gluconate.
Option D: None of the above
• The patient is a known case of Graves’ disease.
• Inadequate perioperative preparation can leadto thyroid toxicosis, and increased vascularity leads to
perioperative bleeding.

Page 25

109
Solution for Question 13:
Option B: Boston sign
• Graves’s ophthalmopathy is an autoimmune condition affecting retro-orbital tissue and producing a
set of characteristic orbital and ocular features in individuals suffering from Grave’s disease.
• Various Eye signs of graves ophthalmopathy include: Von Graefe's sign: Lid lag on down gaze
Dalrymple's sign: Lid retraction Stellwag's sign: Incomplete & infrequent blinking Boston sign ( very
rare): Jerky movements of the lid on down gaze Jellinek's sign: Abnormal pigmentation of the upper lid
Gifford's sign: Difficulty in everting the upper lid
• Von Graefe's sign: Lid lag on down gaze
• Dalrymple's sign: Lid retraction
• Stellwag's sign: Incomplete & infrequent blinking
• Boston sign ( very rare): Jerky movements of the lid on down gaze
• Jellinek's sign: Abnormal pigmentation of the upper lid
• Gifford's sign: Difficulty in everting the upper lid
• Management of this orbitopathy involves treatment of underlying hyperthyroidism, high-dose IV
steroids for severe orbitopathy, and surgical interventions for resistant cases.
• Von Graefe's sign: Lid lag on down gaze
• Dalrymple's sign: Lid retraction
• Stellwag's sign: Incomplete & infrequent blinking
• Boston sign ( very rare): Jerky movements of the lid on down gaze
• Jellinek's sign: Abnormal pigmentation of the upper lid
• Gifford's sign: Difficulty in everting the upper lid
Option A: Trousseau’s sign
• It is not an eye sign of graves ophthalmopathy.
• This sign is seen in conditions like hypoparathyroidism or any condition leading to subclinical
hypocalcemia.
• Trousseau's sign is characterized by carpopedal spasm of the wrist and hand when a blood pressure
cuff is inflated over the systolic blood pressure of the affected individual for 2-3 minutes.
Option C: Woltman sign
• It is not an eye sign of graves ophthalmopathy.
• This sign is commonly associated with hypothyroidism.
• It is characterized by delayed relaxation of the deep tendon reflexes.
Option D: Chvostek sign
• It is not an eye sign of graves ophthalmopathy.
• Chvostek sign is also associated with subclinical hypocalcemia.

Page 26

110
• A positive sign is labeled when tapping the facial nerve in front of the ear produces contraction of the
facial muscles on the same side of the face.

Solution for Question 14:


Option A: Follicular carcinoma
• Follicular thyroid cancer is a tumor of the follicular cells, which are comprised of cuboidal epithelial
cells and have the potential for vascular and capsular invasion properties.
• Follicular thyroid carcinoma (FTC) is the second most common after papillary thyroid cancer and
constitutes 10-15% of all thyroid cancer.
• Genes involved in the pathogenesis of follicular thyroid cancer include PTEN, PAX8-PPARγ and RAS.
• Follicular carcinoma has the potential for lung and bone metastasis via the bloodstream.
• The diagnostic approach initially comprises thyroid function tests, thyroid ultrasound and thyroid
scintigraphy. Confirmation is by histology after fine needle aspiration cytology (FNAC).
• Thyroid tumor markers aid in estimating the prognosis and assessing the response to therapy.
• Treatment is total thyroidectomy with neck dissection as needed, followed by radioactive iodine
ablation and TSH suppression therapy.
• The prognosis depends on the stage of cancer. The overall 5-year survival rate is 50–85%.
Option B: Papillary carcinoma
• Papillary carcinoma is characterized by nodal metastasis, and lymph nodes are palpable in most
cases.
• The peak age of onset in papillary carcinoma is 30 to 50 years old.
• Distant metastasis to bones or lungs is uncommon in papillary carcinoma.
• It is the most common thyroid malignancy.
• History of radiation exposure predisposes to papillary carcinoma thyroid
Option C: Anaplastic carcinoma
• Anaplastic thyroid cancer has a peak incidence in ages >60 years.
• These present with rapid local growth, producing pressure effects like dysphagia or dyspnea.
• These metastasize to distant areas through lymphatics as well as the bloodstream.
• Its prognosis is very poor.
Option D: Medullary carcinoma
• Medullary carcinoma arises from parafollicular 'C' cells of the thyroid gland.
• These occur as part of multiple endocrine neoplasia type 2 (MEN2) syndromes.
• It produces calcitonin and also presents as facial flushing and diarrhea.

Solution for Question 15:

Page 27

111
Option C: Radioiodine
• The thyrotoxicosis recurrence following a subtotal thyroidectomy may occur due to persistent and
latent disease. In addition, Graves’ disease may recur due to thyroid tissue present in aberrant and
ectopic sites, including the thyroglossal duct remnants, anterior neck, mediastinum, and lateral neck.
• Indications of Radioactive iodine include Individuals who have relapsed after medical or surgical
treatment. Individuals in whom surgery of anti-thyroid drugs is contraindicated. Elderly patients with
small or medium-sized goiter. Radioactive iodine results in an acute decline of thyroid hormone release
and inhibits further synthesis. It inhibits the organification of iodine in the thyroid gland, inhibiting
hormone biosynthesis. This phenomenon is called the Wolff-Chaikoff effect. Radioactive iodine therapy
is the best line of management in this case.
• Individuals who have relapsed after medical or surgical treatment.
• Individuals in whom surgery of anti-thyroid drugs is contraindicated.
• Elderly patients with small or medium-sized goiter.
• Radioactive iodine results in an acute decline of thyroid hormone release and inhibits further
synthesis. It inhibits the organification of iodine in the thyroid gland, inhibiting hormone biosynthesis.
This phenomenon is called the Wolff-Chaikoff effect.
• Radioactive iodine therapy is the best line of management in this case.
• Individuals who have relapsed after medical or surgical treatment.
• Individuals in whom surgery of anti-thyroid drugs is contraindicated.
• Elderly patients with small or medium-sized goiter.
• Radioactive iodine results in an acute decline of thyroid hormone release and inhibits further
synthesis. It inhibits the organification of iodine in the thyroid gland, inhibiting hormone biosynthesis.
This phenomenon is called the Wolff-Chaikoff effect.
• Radioactive iodine therapy is the best line of management in this case.
Option A: Further surgery
• A redo thyroidectomy may result in injury to the recurrent laryngeal nerve.
• In a patient with recurrent thyrotoxicosis after thyroid surgery, nerve injury occurs due to fibrosis and
loss of tissue planes.
• Therefore, repeating surgery will not be an appropriate next step in this case.
Option B: Radioiodine followed by surgery
• Although radioiodine is the best treatment for this patient, doing a follow-up surgery is not the right
plan.
• A repeat surgery will result in injury to the recurrent laryngeal nerve
• Nerve injury occurs due to fibrosis and loss of tissue planes.
Option D: Observation and follow-up
• It is not a good strategy not to do anything when the patient is symptomatic.
• Observation and follow-up will result in the worsening of symptoms and patients.
• The patient should be managed immediately and put on a treatment plan.

Page 28

112
Solution for Question 16:
Option C: Hypersecreting adenoma
• In the case mentioned above, the patient has a swelling in the right region of the neck with no
associated symptoms, and the RAIU thyroid scan suggests hypersecreting adenoma.
• The given thyroid scan shows increased focal uptake in the right upper lobe of the thyroid
and decreased uptake in the rest of the thyroid gland; this is suggestive of hypersecreting thyroid
adenoma.
• There is no ectopic thyroid tissue, so lateral aberrant thyroid and lateral extension are ruled out.
• Malignancy is ruled out as there is no cold nodule.

• Hyperfunctioning adenomas become independent of stimulating thyroid hormone (TSH) control and
secrete thyroid hormone autonomously, resulting in clinical hyperthyroidism.
• The elevated thyroid hormone levels ultimately diminish TSH levels and thus depress the function of
the remaining normal thyroid gland.
• The sole focus of increased uptake on a thyroid scan points to the diagnosis of hyperfunctioning
adenoma.
• Most hyperfunctioning or autonomous thyroid nodules have attained a size of at least 3 cm before
hyperthyroidism occurs. These nodules are rarely malignant.
Option A: Papillary carcinoma thyroid
• Papillary carcinoma thyroid manifests as either a palpable thyroid nodule or an incidental imaging
finding.
• Carcinomas usually display diminished uptake and are called "cold" nodules, whereas a sole focus of
increased uptake on a thyroid scan is virtually diagnostic of a hyperfunctioning adenoma.
Option B: Lateral aberrant thyroid
• Lateral aberrant thyroid is a misnomer.
• It is the metastatic node from papillary carcinoma thyroid and usually shows diminished uptake (cold
nodule).

Page 29

113
Option D: Graves’ disease
• Graves’ disease presents a swelling of the neck and exopthalmos resulting from an overactive thyroid
gland.
• Graves’ disease on radioisotope scan shows elevated uptake with a diffusely enlarged gland without
nodularity.

Solution for Question 17:


Option A: A thyrotoxic crisis occurs because of inadequate preoperative preparation.
• Thyroid complications, including thyroid storm, occur due to inadequate preoperative preparation.
• Complications of Thyroidectomy include:
• Thyroid insufficiency
• Thyrotoxic crisis (storm) Inadequate preoperative preparation of a thyrotoxic patient
• Inadequate preoperative preparation of a thyrotoxic patient
• Acute exacerbation of hyperthyroidism: Hemorrhage It is the most frequent life-threatening
complication of Thyroidectomy Most often develops within 24 hours Arterial bleed — Central
compartment pressure exceeds venous pressure —Venous edema of the larynx — Airway obstruction
Immediate release of sutures and evacuation of the hematoma and wound exploration to be done
because unlike most hemorrhages , management of thyroid hemorrhage with compression can lead to
asphyxia. Respiratory obstruction Laryngeal edema (due to intubation injury) Other causes- recurrent
laryngeal nerve palsy, tension hematoma Nerve injuries Most commonly injured nerve : External
branch of a superior laryngeal nerve ( External laryngeal nerve) due to its proximity to the superior
thyroid artery. This leads to a loss of tension in the vocal cord with diminished power and range in the
voice. Recurrent laryngeal nerve - may be unilateral or bilateral, transient or permanent. Cervical
sympathetic trunk Parathyroid insufficiency MC cause – Vascular infarction of parathyroid gland ,if
inferior thyroid artery is ligated away from parathyroid gland during Surgery. Parathyroid insufficiency
most commonly presents on the 2nd – 5th day of Surgery. Manifestations are carpopedal spasms,
tingling, and numbness over the perioral region. Treatment: Mild to moderate symptoms (calcium level
>8gm/dl): oral calcium supplementation Severe symptoms (<8gm /dl): IV calcium gluconate Wound
infection Hypertrophic or keloid scar
• Hemorrhage It is the most frequent life-threatening complication of Thyroidectomy Most often
develops within 24 hours Arterial bleed — Central compartment pressure exceeds venous pressure
—Venous edema of the larynx — Airway obstruction Immediate release of sutures and evacuation of
the hematoma and wound exploration to be done because unlike most hemorrhages , management of
thyroid hemorrhage with compression can lead to asphyxia.
• It is the most frequent life-threatening complication of Thyroidectomy
• Most often develops within 24 hours
• Arterial bleed — Central compartment pressure exceeds venous pressure —Venous edema of the
larynx — Airway obstruction
• Immediate release of sutures and evacuation of the hematoma and wound exploration to be done
because unlike most hemorrhages , management of thyroid hemorrhage with compression can lead to
asphyxia.

Page 30

114
• Respiratory obstruction Laryngeal edema (due to intubation injury) Other causes- recurrent laryngeal
nerve palsy, tension hematoma
• Laryngeal edema (due to intubation injury)
• Other causes- recurrent laryngeal nerve palsy, tension hematoma
• Nerve injuries Most commonly injured nerve : External branch of a superior laryngeal nerve ( External
laryngeal nerve) due to its proximity to the superior thyroid artery. This leads to a loss of tension in the
vocal cord with diminished power and range in the voice. Recurrent laryngeal nerve - may be unilateral
or bilateral, transient or permanent. Cervical sympathetic trunk
• Most commonly injured nerve : External branch of a superior laryngeal nerve ( External laryngeal
nerve) due to its proximity to the superior thyroid artery.
• This leads to a loss of tension in the vocal cord with diminished power and range in the voice.
• Recurrent laryngeal nerve - may be unilateral or bilateral, transient or permanent.
• Cervical sympathetic trunk
• Parathyroid insufficiency MC cause – Vascular infarction of parathyroid gland ,if inferior thyroid artery
is ligated away from parathyroid gland during Surgery. Parathyroid insufficiency most commonly
presents on the 2nd – 5th day of Surgery. Manifestations are carpopedal spasms, tingling, and
numbness over the perioral region. Treatment: Mild to moderate symptoms (calcium level >8gm/dl):
oral calcium supplementation Severe symptoms (<8gm /dl): IV calcium gluconate
• MC cause – Vascular infarction of parathyroid gland ,if inferior thyroid artery is ligated away from
parathyroid gland during Surgery.
• Parathyroid insufficiency most commonly presents on the 2nd – 5th day of Surgery.
• Manifestations are carpopedal spasms, tingling, and numbness over the perioral region.
• Treatment: Mild to moderate symptoms (calcium level >8gm/dl): oral calcium supplementation Severe
symptoms (<8gm /dl): IV calcium gluconate
• Mild to moderate symptoms (calcium level >8gm/dl): oral calcium supplementation
• Severe symptoms (<8gm /dl): IV calcium gluconate
• Wound infection
• Hypertrophic or keloid scar
• Inadequate preoperative preparation of a thyrotoxic patient
• Hemorrhage It is the most frequent life-threatening complication of Thyroidectomy Most often
develops within 24 hours Arterial bleed — Central compartment pressure exceeds venous pressure
—Venous edema of the larynx — Airway obstruction Immediate release of sutures and evacuation of
the hematoma and wound exploration to be done because unlike most hemorrhages , management of
thyroid hemorrhage with compression can lead to asphyxia.
• It is the most frequent life-threatening complication of Thyroidectomy
• Most often develops within 24 hours
• Arterial bleed — Central compartment pressure exceeds venous pressure —Venous edema of the
larynx — Airway obstruction
• Immediate release of sutures and evacuation of the hematoma and wound exploration to be done
because unlike most hemorrhages , management of thyroid hemorrhage with compression can lead to
asphyxia.

Page 31

115
• Respiratory obstruction Laryngeal edema (due to intubation injury) Other causes- recurrent laryngeal
nerve palsy, tension hematoma
• Laryngeal edema (due to intubation injury)
• Other causes- recurrent laryngeal nerve palsy, tension hematoma
• Nerve injuries Most commonly injured nerve : External branch of a superior laryngeal nerve ( External
laryngeal nerve) due to its proximity to the superior thyroid artery. This leads to a loss of tension in the
vocal cord with diminished power and range in the voice. Recurrent laryngeal nerve - may be unilateral
or bilateral, transient or permanent. Cervical sympathetic trunk
• Most commonly injured nerve : External branch of a superior laryngeal nerve ( External laryngeal
nerve) due to its proximity to the superior thyroid artery.
• This leads to a loss of tension in the vocal cord with diminished power and range in the voice.
• Recurrent laryngeal nerve - may be unilateral or bilateral, transient or permanent.
• Cervical sympathetic trunk
• Parathyroid insufficiency MC cause – Vascular infarction of parathyroid gland ,if inferior thyroid artery
is ligated away from parathyroid gland during Surgery. Parathyroid insufficiency most commonly
presents on the 2nd – 5th day of Surgery. Manifestations are carpopedal spasms, tingling, and
numbness over the perioral region. Treatment: Mild to moderate symptoms (calcium level >8gm/dl):
oral calcium supplementation Severe symptoms (<8gm /dl): IV calcium gluconate
• MC cause – Vascular infarction of parathyroid gland ,if inferior thyroid artery is ligated away from
parathyroid gland during Surgery.
• Parathyroid insufficiency most commonly presents on the 2nd – 5th day of Surgery.
• Manifestations are carpopedal spasms, tingling, and numbness over the perioral region.
• Treatment: Mild to moderate symptoms (calcium level >8gm/dl): oral calcium supplementation Severe
symptoms (<8gm /dl): IV calcium gluconate
• Mild to moderate symptoms (calcium level >8gm/dl): oral calcium supplementation
• Severe symptoms (<8gm /dl): IV calcium gluconate
• Wound infection
• Hypertrophic or keloid scar
• It is the most frequent life-threatening complication of Thyroidectomy
• Most often develops within 24 hours
• Arterial bleed — Central compartment pressure exceeds venous pressure —Venous edema of the
larynx — Airway obstruction
• Immediate release of sutures and evacuation of the hematoma and wound exploration to be done
because unlike most hemorrhages , management of thyroid hemorrhage with compression can lead to
asphyxia.
• Laryngeal edema (due to intubation injury)
• Other causes- recurrent laryngeal nerve palsy, tension hematoma
• Most commonly injured nerve : External branch of a superior laryngeal nerve ( External laryngeal
nerve) due to its proximity to the superior thyroid artery.
• This leads to a loss of tension in the vocal cord with diminished power and range in the voice.

Page 32

116
• Recurrent laryngeal nerve - may be unilateral or bilateral, transient or permanent.
• Cervical sympathetic trunk
• MC cause – Vascular infarction of parathyroid gland ,if inferior thyroid artery is ligated away from
parathyroid gland during Surgery.
• Parathyroid insufficiency most commonly presents on the 2nd – 5th day of Surgery.
• Manifestations are carpopedal spasms, tingling, and numbness over the perioral region.
• Treatment: Mild to moderate symptoms (calcium level >8gm/dl): oral calcium supplementation Severe
symptoms (<8gm /dl): IV calcium gluconate
• Mild to moderate symptoms (calcium level >8gm/dl): oral calcium supplementation
• Severe symptoms (<8gm /dl): IV calcium gluconate
• Mild to moderate symptoms (calcium level >8gm/dl): oral calcium supplementation
• Severe symptoms (<8gm /dl): IV calcium gluconate
Option B: The most common cause of respiratory obstruction is nerve injury.
• The most common cause of respiratory obstruction is arterial bleeding leading to the formation of
tension hematoma.
• Arterial bleeding leads to increased central compartment pressure compared to venous pressure.
• It causes venous edema of the larynx, obstructing the airway.
• Immediate release of sutures and evacuation of the hematoma and wound exploration will be done.
Option C: The recurrent laryngeal nerve is more commonly injured than the external branch of the supe
rior laryngeal nerve
• During Thyroidectomy, the most commonly injured nerve is the External branch of the superior
laryngeal nerve (also known as the External laryngeal nerve) due to its proximity to the superior thyroid
artery.
Option D: Parathyroid insufficiency characteristically occurs on days 2-5 after Surgery due to vascular i
nfarction
• Parathyroid insufficiency occurs due to vascular infarction of the parathyroid gland.
• It most commonly presents on the 2nd – 5th day of surgery, and its manifestations are carpopedal
spasm, tingling, and numbness over the perioral region.

Solution for Question 18:


ANSWER
Option C: Vascular invasion
• Papillary thyroid carcinoma(PTC) is the most common thyroid carcinoma comprising 70-80% of all
thyroid cancer.
• It carries an excellent prognosis, especially in young patients.
• The most important risk factor for PTC is childhood radiation.

Page 33

117
• Spread of PTC, beyond thyroid gland capsule,to the nearby soft tissues is an indication for
radioactive iodine therapy
• Female to male ratio of 2.5:1 occurs, and the peak incidence is 30-50 years of age.
• A typical manifestation of PTC is a palpable thyroid nodule.
• Infrequently, a metastatic PTC presents as a painless lateral neck mass detected clinically before the
primary thyroid lesion's detection and is termed lateral aberrant thyroid.
• The high-risk disease is characterized by the following: Gross extrathyroidal invasion Incomplete
tumor resection Presence of distant metastases Pathologic N1 with any metastatic lymph node ≥3 cm
in the largest dimension
• Gross extrathyroidal invasion
• Incomplete tumor resection
• Presence of distant metastases
• Pathologic N1 with any metastatic lymph node ≥3 cm in the largest dimension
• Gross extrathyroidal invasion
• Incomplete tumor resection
• Presence of distant metastases
• Pathologic N1 with any metastatic lymph node ≥3 cm in the largest dimension
Other Options
Option A: 2cm-sized metastatic lymph node
• Lymph nodes of this size are seen in mild diseases and are surgically removed.
• These do not need radioactive iodine therapy.
• Use of radioactive iodine therapy is limited to high-risk diseases with metastatic lymph nodes sized
>3cm.
Option B: Absence of nodal or distant metastasis
• The absence of nodal or distant metastasis represents low-risk disease
• Radioactive iodine is not recommended in individuals with low-risk disease
Option D: Absence of extrathyroidal extension
• The absence of extrathyroidal tumor indicates a low-risk disease
• Radioactive iodine is not recommended to be used in low-risk diseases.
• Therefore, no radioactive iodine is required for treatment if there is no extrathyroidal tumor extension,
as it is low- risk.

Solution for Question 19:


Option A: Medullary thyroid carcinoma
• Medullary Carcinoma of the thyroid (MCT) arises from parafollicular 'C' Cells of the thyroid gland,
located supero laterally in the gland, producing calcitonin, which is responsible for decreasing plasma
calcium levels in the body.

Page 34

118
• The calcitonin level is markedly raised, but these patients remain normocalcemic.
• The presence of amyloid-rich stroma is suggestive of medullary carcinoma thyroid as in this case.
• Diagnosis is made on typical histopathological findings of biopsied thyroid tissue by FNAC.
• Management includes total thyroidectomy and routine central LN dissection with ipsilateral Modified
Radical Neck Dissection if the tumor size is > 1cm and Bilateral Modified Radical Neck Dissection if any
lymph node is positive.
• It carries a poor prognosis because it lacks response to radioactive iodine ablation.
Option B: Systemic amyloidosis
• Systemic amyloidosis is a multisystem disease that commonly presents with heart failure, nephrotic
syndrome, and hepatomegaly.
• It rarely presents in the thyroid gland producing neck swelling.
• This patient has developed only neck swelling over one month with no systemic symptoms, so
systemic amyloidosis chances are negligible.
Option C: Thyroid lymphoma
• Thyroid lymphoma is not an appropriate choice in this case as there are no obstructive symptoms like
dysphagia or dyspnea, which is the most common presentation of rapidly enlarging thyroid lymphoma.
• The presence of amyloid stroma is not associated with thyroid lymphoma.
Option D: Papillary thyroid carcinoma
• Although it is the most common thyroid carcinoma, it usually presents with palpable lymph nodes.
• In addition, histopathological findings of psammoma bodies and orphan Annie eye nuclei are seen in
papillary thyroid cancer.
• Amyloid stroma is not seen in papillary thyroid cancer.

Solution for Question 20:


Option B: Hemithyroidectomy
• The biopsy is consistent with the diagnosis of Hurthle cell adenoma or carcinoma due to the presence
of Hurthle cells. To distinguish between adenoma or carcinoma, a histologic examination is required.
Due to capsular and vascular invasion, it cannot be diagnosed with FNAC and requires excision .
Hemithyroidectomy should be done as the disease is limited to one lobe (unilateral disease), and the
specimen should be sent for histopathological examination.
• Hurthle cell adenomas are an oncocytic variant of follicular carcinoma.
• It arises from oxyphilic cells of the thyroid gland.
• FNAC cannot diagnose it as it has vascular and capsular invasion.
• These usually do not take up radioactive iodine.
• These carry the potential to metastasize to local lymph nodes and distant tissues.
• The mortality rate is high in patients suffering from hurthle cell carcinoma.
• Treatment Hemithyroidectomy is done for unilateral Hurthle cell adenomas. Total thyroidectomy is
done when the capsular invasion is found on a histopathological specimen. Prophylactic central lymph

Page 35

119
node dissection is also done. If lateral neck nodes are involved, modified radical neck dissection should
be done.
• Hemithyroidectomy is done for unilateral Hurthle cell adenomas.
• Total thyroidectomy is done when the capsular invasion is found on a histopathological specimen.
• Prophylactic central lymph node dissection is also done.
• If lateral neck nodes are involved, modified radical neck dissection should be done.
• Hemithyroidectomy is done for unilateral Hurthle cell adenomas.
• Total thyroidectomy is done when the capsular invasion is found on a histopathological specimen.
• Prophylactic central lymph node dissection is also done.
• If lateral neck nodes are involved, modified radical neck dissection should be done.
Option A: Total thyroidectomy + Central neck dissection
• Total thyroidectomy is done when the capsular invasion is found on a histopathological specimen
• As the disease is unilateral in this case, a hemithyroidectomy should be done with excision and
histopathology.
• Normal thyroid lobes need not be removed.
Option C: Total thyroidectomy + Modified radical neck dissection
• Total thyroidectomy is done when the capsular invasion is found on a histopathological specimen
• As the disease is unilateral in this case, a hemithyroidectomy should be done with excision and
histopathology.
• Normal thyroid lobes need not be removed.
Option D: Near-Total Thyroidectomy with cervical LN I and II dissection
• As the disease is unilateral in this case, a hemithyroidectomy should be done for excision and
histopathology.
• Normal thyroid lobes need not be removed.

Solution for Question 21:


Option A: Papillary thyroid cancer
• Based on the history most likely diagnosis is papillary carcinoma of the thyroid because of the
exposure to radiation, which is a predominant risk factor for developing papillary carcinoma of the
thyroid.
• Papillary thyroid carcinoma (PTC) is the most common thyroid carcinoma comprising 70-80% of all
thyroid cancer.
• It carries an excellent prognosis, especially in young patients.
• The most important risk factor for PTC is childhood radiation.
• Female to male ratio of 2.5:1 occurs, and the peak incidence is 30-50 years of age.
Other options

Page 36

120
Option B: Medullary thyroid cancer
• It is not an appropriate diagnosis as exposure to radiation is not a risk factor for medullary thyroid
cancer.
• Medullary carcinoma arises from parafollicular 'C' thyroid gland cells.
• These occur as part of multiple endocrine neoplasia type 2 (MEN2) syndromes.
• It produces calcitonin and also presents as facial flushing and diarrhea.
Option C: Anaplastic thyroid cancer
• It is not an appropriate diagnosis as exposure to radiation is not a risk factor for medullary thyroid
cancer.
• Anaplastic thyroid cancer has a peak incidence in ages >60 years.
• These present with rapid local growth, producing pressure effects like dysphagia or dyspnea.
• These metastasize to distant areas through lymphatics as well as the bloodstream.
Option D: Thyroid lymphoma
• Thyroid lymphoma is not an appropriate choice in this case as there are no obstructive symptoms like
dysphagia or dyspnea, which is the most common presentation of rapidly enlarging thyroid lymphoma.
• Additionally, her treatment of primary disease (Hodgkin's lymphoma) went well, and her symptoms
improved, minimizing the risk of secondary thyroid lymphoma.

Solution for Question 22:


Option B: 24%
• Fluid-filled cavities in thyroid or thyroid cysts most commonly form due to degenerating thyroid
adenomas. It often contains solid and liquid components. These are generally non-cancerous but can
occasionally contain solid cancerous components.
• The primary diagnostic and therapeutic procedure is the aspiration of the cyst. About half of these will
re-accumulate fluid, which requires surgical excision.
• If the nodule does not completely disappear after aspiration, it may be a complex cyst associated with
higher malignant potential.
• Fluid cytology typically reveals nonspecific results.
• Suppressive therapy with levothyroxine reduces the risk of relapse, especially when followed by
aspiration.
• Cyst sclerotherapy by ethanol injection into the nodule after aspiration of cystic fluid is an emerging
alternative treatment.
• In the case of an isolated cystic swelling in a male patient, the risk of malignancy is 24%
• The risk of malignancies in thyroid swelling can be expressed as a factor of 12, known as the ‘rule of
twelve.' Isolated Thyroid swelling:
• Isolated Thyroid swelling:
• Solid 24% [Male-48%(i.e. 24*2) % Female-12%(i.e 24/2)] Cystic 12% [Male-24%(i.e. 12*2) %
Female-6%(i.e 12/2)] Dominant Thyroid swelling:

Page 37

121
• Solid 24% [Male-48%(i.e. 24*2) % Female-12%(i.e 24/2)] Cystic 12% [Male-24%(i.e. 12*2) %
Female-6%(i.e 12/2)]
• Solid 24% [Male-48%(i.e. 24*2) % Female-12%(i.e 24/2)] Cystic 12% [Male-24%(i.e. 12*2) %
Female-6%(i.e 12/2)]
• Solid 24% [Male-48%(i.e. 24*2) % Female-12%(i.e 24/2)] Cystic 12% [Male-24%(i.e. 12*2) %
Female-6%(i.e 12/2)]
• Solid 24% [Male-48%(i.e. 24*2) % Female-12%(i.e 24/2)] Cystic 12% [Male-24%(i.e. 12*2) %
Female-6%(i.e 12/2)]
• Solid 24% [Male-48%(i.e. 24*2) % Female-12%(i.e 24/2)] Cystic 12% [Male-24%(i.e. 12*2) %
Female-6%(i.e 12/2)]
• Solid 24% [Male-48%(i.e. 24*2) % Female-12%(i.e 24/2)]
• Cystic 12% [Male-24%(i.e. 12*2) % Female-6%(i.e 12/2)]
• Dominant Thyroid swelling:
• Solid 12% [Male-24%(i.e. 12*2) % Female-6%(i.e 12/2)] Cystic 6% [Male-12%(i.e. 6*2) %
Female-3%(i.e 6/2)] Generalized Thyroid swelling: 3%
• Solid 12% [Male-24%(i.e. 12*2) % Female-6%(i.e 12/2)] Cystic 6% [Male-12%(i.e. 6*2) %
Female-3%(i.e 6/2)]
• Solid 12% [Male-24%(i.e. 12*2) % Female-6%(i.e 12/2)] Cystic 6% [Male-12%(i.e. 6*2) %
Female-3%(i.e 6/2)]
• Solid 12% [Male-24%(i.e. 12*2) % Female-6%(i.e 12/2)] Cystic 6% [Male-12%(i.e. 6*2) %
Female-3%(i.e 6/2)]
• Solid 12% [Male-24%(i.e. 12*2) % Female-6%(i.e 12/2)] Cystic 6% [Male-12%(i.e. 6*2) %
Female-3%(i.e 6/2)]
• Solid 12% [Male-24%(i.e. 12*2) % Female-6%(i.e 12/2)] Cystic 6% [Male-12%(i.e. 6*2) %
Female-3%(i.e 6/2)]
• Solid 12% [Male-24%(i.e. 12*2) % Female-6%(i.e 12/2)]
• Cystic 6% [Male-12%(i.e. 6*2) % Female-3%(i.e 6/2)]
• Generalized Thyroid swelling: 3%
• Isolated Thyroid swelling:
• Solid 24% [Male-48%(i.e. 24*2) % Female-12%(i.e 24/2)] Cystic 12% [Male-24%(i.e. 12*2) %
Female-6%(i.e 12/2)]
• Solid 24% [Male-48%(i.e. 24*2) % Female-12%(i.e 24/2)] Cystic 12% [Male-24%(i.e. 12*2) %
Female-6%(i.e 12/2)]
• Solid 24% [Male-48%(i.e. 24*2) % Female-12%(i.e 24/2)] Cystic 12% [Male-24%(i.e. 12*2) %
Female-6%(i.e 12/2)]
• Solid 24% [Male-48%(i.e. 24*2) % Female-12%(i.e 24/2)] Cystic 12% [Male-24%(i.e. 12*2) %
Female-6%(i.e 12/2)]
• Solid 24% [Male-48%(i.e. 24*2) % Female-12%(i.e 24/2)] Cystic 12% [Male-24%(i.e. 12*2) %
Female-6%(i.e 12/2)]
• Solid 24% [Male-48%(i.e. 24*2) % Female-12%(i.e 24/2)]
• Cystic 12% [Male-24%(i.e. 12*2) % Female-6%(i.e 12/2)]

Page 38

122
• Dominant Thyroid swelling:
• Solid 24% [Male-48%(i.e. 24*2) % Female-12%(i.e 24/2)] Cystic 12% [Male-24%(i.e. 12*2) %
Female-6%(i.e 12/2)]
• Solid 24% [Male-48%(i.e. 24*2) % Female-12%(i.e 24/2)] Cystic 12% [Male-24%(i.e. 12*2) %
Female-6%(i.e 12/2)]
• Solid 24% [Male-48%(i.e. 24*2) % Female-12%(i.e 24/2)] Cystic 12% [Male-24%(i.e. 12*2) %
Female-6%(i.e 12/2)]
• Solid 24% [Male-48%(i.e. 24*2) % Female-12%(i.e 24/2)] Cystic 12% [Male-24%(i.e. 12*2) %
Female-6%(i.e 12/2)]
• Solid 24% [Male-48%(i.e. 24*2) % Female-12%(i.e 24/2)]
• Cystic 12% [Male-24%(i.e. 12*2) % Female-6%(i.e 12/2)]
• Solid 24% [Male-48%(i.e. 24*2) % Female-12%(i.e 24/2)] Cystic 12% [Male-24%(i.e. 12*2) %
Female-6%(i.e 12/2)]
• Solid 24% [Male-48%(i.e. 24*2) % Female-12%(i.e 24/2)] Cystic 12% [Male-24%(i.e. 12*2) %
Female-6%(i.e 12/2)]
• Solid 24% [Male-48%(i.e. 24*2) % Female-12%(i.e 24/2)] Cystic 12% [Male-24%(i.e. 12*2) %
Female-6%(i.e 12/2)]
• Solid 24% [Male-48%(i.e. 24*2) % Female-12%(i.e 24/2)]
• Cystic 12% [Male-24%(i.e. 12*2) % Female-6%(i.e 12/2)]
• Solid 24% [Male-48%(i.e. 24*2) % Female-12%(i.e 24/2)] Cystic 12% [Male-24%(i.e. 12*2) %
Female-6%(i.e 12/2)]
• Solid 24% [Male-48%(i.e. 24*2) % Female-12%(i.e 24/2)] Cystic 12% [Male-24%(i.e. 12*2) %
Female-6%(i.e 12/2)]
• Solid 24% [Male-48%(i.e. 24*2) % Female-12%(i.e 24/2)]
• Cystic 12% [Male-24%(i.e. 12*2) % Female-6%(i.e 12/2)]
• Solid 24% [Male-48%(i.e. 24*2) % Female-12%(i.e 24/2)] Cystic 12% [Male-24%(i.e. 12*2) %
Female-6%(i.e 12/2)]
• Solid 24% [Male-48%(i.e. 24*2) % Female-12%(i.e 24/2)]
• Cystic 12% [Male-24%(i.e. 12*2) % Female-6%(i.e 12/2)]
• Solid 24% [Male-48%(i.e. 24*2) % Female-12%(i.e 24/2)]
• Cystic 12% [Male-24%(i.e. 12*2) % Female-6%(i.e 12/2)]
• Solid 12% [Male-24%(i.e. 12*2) % Female-6%(i.e 12/2)] Cystic 6% [Male-12%(i.e. 6*2) %
Female-3%(i.e 6/2)]
• Solid 12% [Male-24%(i.e. 12*2) % Female-6%(i.e 12/2)] Cystic 6% [Male-12%(i.e. 6*2) %
Female-3%(i.e 6/2)]
• Solid 12% [Male-24%(i.e. 12*2) % Female-6%(i.e 12/2)] Cystic 6% [Male-12%(i.e. 6*2) %
Female-3%(i.e 6/2)]
• Solid 12% [Male-24%(i.e. 12*2) % Female-6%(i.e 12/2)] Cystic 6% [Male-12%(i.e. 6*2) %
Female-3%(i.e 6/2)]

Page 39

123
• Solid 12% [Male-24%(i.e. 12*2) % Female-6%(i.e 12/2)] Cystic 6% [Male-12%(i.e. 6*2) %
Female-3%(i.e 6/2)]
• Solid 12% [Male-24%(i.e. 12*2) % Female-6%(i.e 12/2)]
• Cystic 6% [Male-12%(i.e. 6*2) % Female-3%(i.e 6/2)]
• Generalized Thyroid swelling: 3%
• Solid 12% [Male-24%(i.e. 12*2) % Female-6%(i.e 12/2)] Cystic 6% [Male-12%(i.e. 6*2) %
Female-3%(i.e 6/2)]
• Solid 12% [Male-24%(i.e. 12*2) % Female-6%(i.e 12/2)] Cystic 6% [Male-12%(i.e. 6*2) %
Female-3%(i.e 6/2)]
• Solid 12% [Male-24%(i.e. 12*2) % Female-6%(i.e 12/2)] Cystic 6% [Male-12%(i.e. 6*2) %
Female-3%(i.e 6/2)]
• Solid 12% [Male-24%(i.e. 12*2) % Female-6%(i.e 12/2)] Cystic 6% [Male-12%(i.e. 6*2) %
Female-3%(i.e 6/2)]
• Solid 12% [Male-24%(i.e. 12*2) % Female-6%(i.e 12/2)]
• Cystic 6% [Male-12%(i.e. 6*2) % Female-3%(i.e 6/2)]
• Solid 12% [Male-24%(i.e. 12*2) % Female-6%(i.e 12/2)] Cystic 6% [Male-12%(i.e. 6*2) %
Female-3%(i.e 6/2)]
• Solid 12% [Male-24%(i.e. 12*2) % Female-6%(i.e 12/2)] Cystic 6% [Male-12%(i.e. 6*2) %
Female-3%(i.e 6/2)]
• Solid 12% [Male-24%(i.e. 12*2) % Female-6%(i.e 12/2)] Cystic 6% [Male-12%(i.e. 6*2) %
Female-3%(i.e 6/2)]
• Solid 12% [Male-24%(i.e. 12*2) % Female-6%(i.e 12/2)]
• Cystic 6% [Male-12%(i.e. 6*2) % Female-3%(i.e 6/2)]
• Solid 12% [Male-24%(i.e. 12*2) % Female-6%(i.e 12/2)] Cystic 6% [Male-12%(i.e. 6*2) %
Female-3%(i.e 6/2)]
• Solid 12% [Male-24%(i.e. 12*2) % Female-6%(i.e 12/2)] Cystic 6% [Male-12%(i.e. 6*2) %
Female-3%(i.e 6/2)]
• Solid 12% [Male-24%(i.e. 12*2) % Female-6%(i.e 12/2)]
• Cystic 6% [Male-12%(i.e. 6*2) % Female-3%(i.e 6/2)]
• Solid 12% [Male-24%(i.e. 12*2) % Female-6%(i.e 12/2)] Cystic 6% [Male-12%(i.e. 6*2) %
Female-3%(i.e 6/2)]
• Solid 12% [Male-24%(i.e. 12*2) % Female-6%(i.e 12/2)]
• Cystic 6% [Male-12%(i.e. 6*2) % Female-3%(i.e 6/2)]
• Solid 12% [Male-24%(i.e. 12*2) % Female-6%(i.e 12/2)]
• Cystic 6% [Male-12%(i.e. 6*2) % Female-3%(i.e 6/2)]
Option A: 48%
• In the case of an isolated cystic swelling in a male patient, the risk of malignancy is 24%, not 48%
• A 48% risk of malignancy occurs in a male with solid thyroid swelling, not a cystic one.
Option C: 12%

Page 40

124
• In the case of an isolated cystic swelling in a male patient, the risk of malignancy is 24%, not 12%
• A 12% risk of malignancy occurs in a female with a solid neck swelling and will not occur in a male
with cystic thyroid swelling.
Option D: 3%
• In the case of an isolated cystic swelling in a male patient, the risk of malignancy is 24%, not 3%
• A 3% risk of malignancy will occur in a female with dominant and cystic thyroid swelling or in any
individual with generalized thyroid swelling.

Solution for Question 23:


Option C: Microscopic evidence of multi-centric disease is present in 70% of cases
• The patient is likely suffering from papillary thyroid carcinoma (PTC).
• PTC is a multi-centric disease, as it is not limited to one lobe.Multicentricity can be anticipated in 70%
of patients with PTC and may represent intraglandular metastasis or multiple primary tumors.
• It is the most common thyroid malignancy and usually is associated with an excellent prognosis,
particularly in young patients.
• PTC manifests as either a palpable thyroid nodule or an incidental imaging finding.
• Occasionally, a metastatic PTC manifests as a painless lateral neck mass.
• Additionally, cervical lymph node metastasis is common, particularly in children, who may have a 50%
incidence of clinically detectable nodal disease at presentation.
• The primary treatment is surgery. Other mainstays of therapy are radioactive iodine ablation and TSH
suppression.
• There are several rationales for considering total thyroidectomy for a patient with unilateral thyroid
malignancy and no evidence of metastasis.
• Radioactive iodine ablation is less effective and requires a larger dosage if residual thyroid exists.
Occult multifocal disease. Facilitating the use of thyroglobulin as a tumor marker.
• Radioactive iodine ablation is less effective and requires a larger dosage if residual thyroid exists.
• Occult multifocal disease.
• Facilitating the use of thyroglobulin as a tumor marker.
• Total thyroidectomy is recommended for all PTCs 1cm or larger.
• If any lymph nodes is positive - total thyroidectomy+ removal of enlarged central LN+ ipsilateral
MRND
• Radioactive iodine ablation is less effective and requires a larger dosage if residual thyroid exists.
• Occult multifocal disease.
• Facilitating the use of thyroglobulin as a tumor marker.
Option A: Papillary thyroid carcinomas (PTC) more than 1 cm in diameter are microcarcinomas
• PTCs less than 1 cm in diameter are defined as microcarcinomas.
• Surgery is not required for these patients:

Page 41

125
• If there are no clinically involved nodes. No extrathyroidal extension. No history of head and neck
irradiation. Each patient needs an individualized approach
• If there are no clinically involved nodes.
• No extrathyroidal extension.
• No history of head and neck irradiation.
• Each patient needs an individualized approach
• If there are no clinically involved nodes.
• No extrathyroidal extension.
• No history of head and neck irradiation.
• Each patient needs an individualized approach
Option B: Only 1% of thyroid carcinomas diagnosed are papillary thyroid carcinoma
• Papillary thyroid carcinoma is a common thyroid carcinoma with a good prognosis.
• 70 to 80% of thyroid carcinomas diagnosed are papillary thyroid carcinoma.
Option D: The most important risk factor for PTC is a family history
• The most important risk factor for papillary thyroid carcinoma is not family history.
• Childhood radiation is the most important risk factor for developing papillary thyroid cancer.

Page 42

126
Parathyroid and Adrenals
1. A 6-month-old infant is brought to the genetic counselor due to the family history of MEN-2B. His
father died of medullary thyroid carcinoma. His genetic testing was positive for the RET protooncogene
mutation. Which of the following will be the best line of management that will improve the prognosis?
(or)
What is the best management approach to improve the prognosis of a 6-month-old infant with a family
history of MEN-2B and a father who died of medullary thyroid carcinoma at 21 years of age?
A. Prophylactic thyroidectomy
B. Clinical observation and follow-up
C. Regular FNAC
D. All of the above
----------------------------------------
2. Which statement is false regarding the condition of a 50-year-old female presenting with heartburn,
nausea, bone pain, elevated PTH levels, and subperiosteal resorption in the middle phalanx of the
middle finger on X-ray?
(or)
A 50-year-old female presented with complaints of heartburn, nausea, and bone pain. An x-ray of the
hand shows subperiosteal resorption in the middle phalanx of the middle finger. Laboratory studies
show elevated PTH levels. Which one of the following is not a feature of the above condition?
A. Generalized osteoporosis
B. Renal calculi
C. Hypercalcemia
D. Osteosclerosis
----------------------------------------
3. A 40-year-old female patient presented to the outpatient department with complaints of muscle
weakness, new growth of facial hair, and coarseness of voice. Physical examination revealed purple
striae on the abdominal wall and a palpable abdominal mass. A CT scan of the abdomen revealed a
heterogeneous adrenal mass of 7cm with indistinct borders and central necrosis. Which of the following
is true about the given condition?
(or)
Which statement is true about the condition of a 40-year-old female presenting with muscle weakness,
facial hair growth, coarseness of voice, purple striae, and a palpable abdominal mass, CT scan findings
areshown below.
A. Rare tumor
B. More than half are functional
C. Most commonly associated with Cushing syndrome
D. All of the above
----------------------------------------

127
4. A 50-year-old non-adherent male known case of chronic renal failure presented with complaints of
fatigue, weakness, decreased urine output, bone and loin pain, raised PTH and normal calcium levels.
Which statement is true for his condition?
(or)
A 50-year-old male patient presents with fatigue, weakness, decreased urine output, bone pain, and
loin pain. He is a known case of chronic renal failure. He is poorly adherent to his medications. Serum
calcium levels were found to be normal. Serum PTH levels were elevated. Which of the following is true
about this condition?
A. Commonly occurs in Chronic renal failure
B. Related to hyperphosphatemia
C. Patients are generally hypocalcemic
D. All of the above
----------------------------------------
5. An 18-month-old baby was brought in with complaints of abdominal mass, blueberry muffin spots on
the skin, proptosis, and periorbital ecchymosis. Upon further evaluation, urinary catecholamines and
their metabolites were elevated. Which of the following is incorrect about the described condition?
(or)
An 18-month-old baby presents with abdominal mass, blueberry muffin spots on the skin, proptosis and
periorbital ecchymosis, and elevated levels of urinary catecholamines and its metabolites. Which
statement is false for this condition?
A. Homer Wright’s pseudorosettes are characteristic.
B. Most common Intracranial solid tumor of childhood
C. May undergo spontaneous regression
D. It can originate anywhere along the sympathetic chain
----------------------------------------
6. A 48-year-old female patient presented with complaints of sudden repeated episodes of headache
and palpitations lasting for 30 minutes and resolve spontaneously for the past two weeks. Vitals- BP:
168/100 mmHg, Temperature: 37°C, pulse: 110/min, and respiratory rate: 18/min. A CT scan of the
abdomen revealed a mass in the adrenal gland. Which of the following is the most common clinical sign
of this condition?
(or)
A 48-year-old female presents with sudden, repeated episodes of headache and palpitations, which
lasted for 30 minutes and resolved spontaneously. CT showed a mass in the adrenal gland. What is the
most common sign of this condition?
A. Tachycardia
B. Hypertension
C. Sweating
D. Dyspnea
----------------------------------------
7. A 62-year-old female presented to the clinic with nausea, vomiting, muscle weakness, fatigue,
confusion, oliguria, and weight loss. She also complained of back pain at night. Her mammogram five

Page 2

128
years ago was abnormal, but she did not follow up with her doctor regarding the results. Her blood
pressure is 126/82 mmHg, her temperature is 37ºC (98.6º F), his pulse is 87/min, and her respirations
are 18/min. Her serum chemistry report: Sodium 138 mEq/L Potassium 4.5 mEq/L Chloride 104 mEq/L
Calcium 15 mg/dL Bicarbonate 22 mEq/L Blood urea nitrogen 20 mg/dL Creatinine 1.1 mg/dL Which of
the following is the most likely cause of this electrolyte abnormality?
(or)
A 62-year-old female had a weight loss of 10 kg in the last 4 months despite having a healthy diet. She
also complained of back pain at night. Her mammogram five years ago was abnormal. Na: 138 mEq/L,
K: 4.5 mEq/L, Cl: 104 mEq/L, Ca: 15 mg/dL What is the most likely diagnosis?=
A. Parathyroid adenoma
B. Parathyroid hyperplasia
C. Hypercalcemia of malignancy
D. Paget’s disease
----------------------------------------
8. Which of the following features is not suggestive of adrenal adenoma in a 46-year-old male
presenting with abdominal pain after meals, greasy loose stools, weight loss, pancreatic calcifications,
and a right adrenal mass on abdominal CT?
(or)
A 46-year-old male patient presented with complaints of abdominal pain following meals for the last 6
months. He has lost 6 kg of weight in the last two months. He has also passed greasy loose stools for
the last 6 months. Chronic pancreatitis is suspected, and an abdominal CT scan is ordered. In addition
to the calcifications in pancreatic tissue, a right adrenal mass is seen incidentally on the CT scan.
Which of the following radiologic features is not suggestive of adrenal adenoma?
A. Low attenuation
B. Homogeneous density and well-defined borders
C. Enhances rapidly; contrast stays in it for a relatively long time and washes out late
D. Calcification is rare
----------------------------------------
9. What is an indication for adrenalectomy in a 40-year-old male present with complaints of early
satiety, fullness, abdominal pain radiating to the back, and adrenal mass on abdominal CT scan?
(or)
A 40-year-old male patient presented to your clinic with complaints of abdominal pain radiating to the
back, early satiety, and fullness. During his workup, an incidental adrenal mass was found on the
abdominal CT scan. Which of the following is an indication for adrenalectomy in this patient?
A. Size >5 cm
B. Bilateral adrenal metastasis
C. Functional tumor
D. All of the above
----------------------------------------
10. Which organ is affected first in a 25-year-old male with a family history of hyperparathyroidism,
presenting with flank pain due to bilateral renal stones and elevated serum calcium, PTH, prolactin?

Page 3

129
(or)
A 25-year-old male patient with a family history of hyperparathyroidism was brought to the OPD with
complaints of loin pain. On evaluation, multiple renal calculi were noted in both kidneys. Serum calcium,
PTH and prolactin. Which of the following organs was first affected in this patient?
A. Parathyroid
B. Thyroid
C. Adrenal
D. Testis
----------------------------------------
11. A 58-year-old female presents to the clinic complaining of sweating, weight loss, and palpitations. A
Tc99m Sestamibi scan revealed increased right superior parathyroid gland uptake. The patient
underwent an exploration of neck and parathyroidectomy. All of the following are true about the
parathyroid gland except?
(or)
All of the following are true about the parathyroid gland except?
A. Upper parathyroid glands are superior to the junction of the inferior thyroid artery and RLN (recurrent
laryngeal nerve)
B. Most common location of ectopic parathyroid glands is the paraesophageal
C. Lower parathyroid glands are inferior to the junction of the inferior thyroid artery and RLN (recurrent
laryngeal nerve)
D. The superior parathyroid glands are located anterior to RLN (recurrent laryngeal nerve)
----------------------------------------
12. What is the commonest cause of hirsutism, menstrual irregularities, and elevated serum cortisol
levels that don't decrease with a low-dose dexamethasone suppression test in a 32-year-old obese
female taking medication for multiple sclerosis?
(or)
A 32-year-old obese female patient presented to the OPD with features of hirsutism, purple striae,
acne, and menstrual irregularities. On further evaluation, serum cortisol levels were elevated and did
not decrease with the low-dose dexamethasone suppression test. The abdominal CT scan is normal.
She is currently taking medications for multiple sclerosis. Which of the following is the commonest
cause of the above-described syndrome?
A. Adrenal adenoma
B. Pituitary adenoma
C. Adrenal carcinoma
D. Exogenous steroid
----------------------------------------
13. Hypoparathyroidism occurs most commonly in?
(or)
Hypoparathyroidism Occurs Most commonly in?
A. After thyroid surgery

Page 4

130
B. Down syndrome
C. After high fever
D. MEN 1
----------------------------------------
14. A 40-year-old male patient underwent bilateral adrenalectomy for Cushing’s disease, after which
the patient developed gradual loss of vision, hyperpigmentation of the skin, and headache. The
underlying reason is suspected to be the loss of feedback inhibition. Which of the following is the most
likely cause?
(or)
What is the most likely cause of gradual loss of vision, hyperpigmentation of the skin, and headache,
loss of feedback inhibition in a 40-year-old male patient who underwent bilateral adrenalectomy for
Cushing's disease?
A. Addison’s disease
B. Nelson’s syndrome
C. Cushing’s disease
D. Hypopituitarism
----------------------------------------
15. Which of the following statements is incorrect about Females presenting with headache,
palpitations, BP 180/100mmhg, and elevated urinary catecholamines and their metabolites?
(or)
A 40-year-old female patient presents with complaints of headache, palpitations, and increased
sweating. The blood pressure was found to be 180/100 mm Hg. On examination Urinary
catecholamines and their metabolites were found to be elevated. Which of the following statements is
incorrect?
A. 90% are malignant
B. 10% occur in the abdomen
C. Pheochromocytoma releases catecholamines
D. They can arise from sympathetic ganglions
----------------------------------------
16. A 43-year-old female presents with weakness, fatigue, loin pain, and bone pain. On further
evaluation, serum calcium and PTH levels are elevated. Sestamibi scan revealed increased uptake in
all four parathyroid glands. Treatment for the above condition is?
(or)
What is the treatment for a 43-year-old female with loin pain, bone pain, elevated serum calcium and
PTH levels, and increased uptake in all four parathyroid glands on a sestamibi scan?
A. Phosphate binders
B. Calcitonin
C. Removal of 3 ½ glands
D. Enlarged glands to be removed

Page 5

131
----------------------------------------
17. A 32-year -old male presents to the ER with severe epigastric pain. He has a history of epigastric
pain following food intake. The patient is stabilized and treated for peptic ulcer disease. In the ward, the
house officer takes a detailed history, revealing recurrent renal stones and eyesight problems affecting
both temporal sides. Based on this history, the house officer informs the consultant during rounds that
the patient may have a syndrome. Which of the following syndromes is the house officer suspecting in
this patient?
(or)
A 32-year-old male presenting with severe epigastric pain with food intake, recurrent renal stones, and
temporal visual field defects is likely suffering from which syndrome?
A. Zollinger Ellison Syndrome
B. MEN 2A
C. MEN 1
D. MEN 2B
----------------------------------------
18. A 20-year-old patient underwent prophylactic thyroidectomy at 4 years of age as his father has a
history of medullary carcinoma of the thyroid and parathyroid adenoma. Which of the following
statements about genetic syndrome is correct
(or)
The patient underwent prophylactic thyroidectomy at 4 years of age due to a family history of medullary
carcinoma thyroid and parathyroid adenoma, which statement is correct about this genetic syndrome?
A. Also known as Sipple syndrome
B. Associated with cutaneous lichen amyloidosis
C. Caused by germline missense mutations in extracellular cysteine codons of RET
D. Associated with Medullary thyroid carcinoma, pheochromocytoma and parathyroid adenoma
----------------------------------------
19. A 32-year-old male presents to the ER with severe epigastric pain. He has a previous history of
pain in the epigastric region with food intake. The patient is stabilized and treated along the lines of
peptic ulcer disease. History revealed that he has recurrent renal stones and eyesight problems
affecting both temporal sides. Genetic testing performed revealed heterozygous mutations of CDNK1B
on chromosome 12. What is the possible diagnosis?
(or)
Which genetic syndrome is associated with peptic ulcer disease, recurrent renal stones, eyesight
problems affecting both temporal sides, and a heterozygous mutation of CDKN1B on chromosome 12?
A. MEN 1
B. MEN 2A
C. MEN 4
D. MEN 2B
----------------------------------------

Page 6

132
20. A 48-year-old female with a known case of gout presents with loin pain, nausea, weakness, fatigue,
and depression. Physical examination revealed a 4x3 cm swelling in the neck, palpable hard-fixed
lymph nodes. Elevated PTH and serum calcium levels were found. Which of the following statements
about the given clinical condition is correct?
(or)
Which of the following statements is incorrect about a 48-year-old female, a known case of gout,
presented with loin pain and swelling of 4x3 cm in the neck with palpable hard fixed lymph nodes,
elevated PTH, and serum calcium levels?
A. Cinacalcet is used in controlling hypocalcemia
B. The mostcommon route of spread is local invasion
C. Diagnosed by biopsy showing vascular or capsular invasion
D. Can be suspected preoperatively by serum calcium >14 mg/dL, elevated PTH, and palpable
parathyroid glands
----------------------------------------
21. A 50-year-old male patient presented to the OPD with complaints of paroxysms of headache,
nausea, abdominal pain, vomiting, and palpitations. Urinary catecholamines and plasma metanephrine
levels were found to be elevated. Which of the following is used preoperatively to control blood
pressure during surgery?
(or)
What is used preoperatively to control blood pressure during surgery in a 50-year-old male presenting
with paroxysms of headache, vomiting, and palpitations with elevated urinary catecholamines?
A. Beta-blocker
B. Diuretic
C. Irreversible alpha blocker
D. None of the above
----------------------------------------
22. A 42-year-old male patient has come to the OPD with complaints of loin pain. On the CT scan,
multiple renal calculi were noted. The patient also had 3 cm mass in the adrenal gland. In, most
patients, this adrenal mass is most likely to be?
(or)
What is the most likely diagnosis for a 42-year-old male presenting with loin pain, multiple renal calculi
on CT, and a 3cm adrenal gland mass?
A. Cushing's adenoma
B. Pheochromocytoma
C. Adrenocortical carcinoma
D. Non-functioning adenoma
----------------------------------------
23. A 36-year-old female, asymptomatic but with raised 24-hour urinary cortisol level and raised ACTH
with suppression in high dose dexamethasone suppression test. MRI of the head reveals no
enlargement of the pituitary gland. What should be the next step of management in this case?

Page 7

133
(or)
What is the next step of management in an asymptomatic female with raised 24-hour urinary cortisol
level, raised ACTH, and suppression in high dose dexamethasone suppression test, and normal
pituitary gland on MRI of the head?
A. MRI to assess the adrenals
B. CT chest
C. Inferior petrosal sinus venous sampling
D. Ultrasound abdomen
----------------------------------------
24. Which statement is correct regarding genetic syndromes in a 30-year-old male presenting with
fatigue, bone pain, abnormal gait, and elevated serum calcium and PTH levels?
(or)
A 30-year-old male patient was brought to the OPD with fatigue, bone pain, and abnormal gait. Serum
calcium and PTH levels were elevated. Which of the following statements regarding the genetic
syndromes associated with the condition is correct?
A. The most common endocrine abnormality in MEN 1 is pancreatic neuroendocrine tumors
B. Renal and gonadal tumors are seen in MEN 3
C. Marfanoid features and neuromas are seen in Sipple syndrome
D. MEN 2A is characterized by cutaneous lichen amyloidosis
E. Age of prophylactic thyroidectomy in MEN 2A is before 1-year
----------------------------------------
25. A 52-year-old female patient was found to have an adrenal mass on a CT scan. Her blood pressure
was found to be 200/100 mmHg. On further evaluation, urinary vanillyl mandelic levels were found to be
elevated. Which of the following is correct about the given clinical condition?
(or)
Which of the following is correct about a 52-year-old hypertensive female patient with an adrenal mass
on CT scan and elevated urinary vanillyl mandelic acid levels?
A. Pheochromocytoma is the most commonly diagnosed adrenal incidentaloma
B. Most common manifestation is diaphoresis
C. Best test for diagnosis is urinary catecholamines
D. Extraadrenal pheochromocytoma secretes adrenaline exclusively
E. Phenoxybenzamine is the most preferred drug
----------------------------------------

Correct Answers
Question Correct Answer

Question 1 1
Question 2 4

Page 8

134
Question 3 4
Question 4 4
Question 5 2
Question 6 2
Question 7 3
Question 8 3
Question 9 4
Question 10 1
Question 11 4
Question 12 4
Question 13 1
Question 14 2
Question 15 1
Question 16 3
Question 17 3
Question 18 1
Question 19 3
Question 20 1
Question 21 3
Question 22 4
Question 23 3
Question 24 F,F,F,T,F
Question 25 F,F,F,F,T

Solution for Question 1:


Correct Option A: Prophylactic thyroidectomy
MEN2:
• Due to mutation in RET proto-oncogene located on chromosome 10
• Hyperparathyroidism develops in 20% of patients and is less severe
MEN2A (Sipple’s syndrome)
MEN2B (also known as MEN3)
Features of Medullary thyroid carcinoma
Multifocal, Bilateral
Multifocal, Bilateral, Occurs earlier, even within the first year of life.
Inheritance pattern

Page 9

135
Autosomal dominant
Associated abnormalities
Pheochromocytomas, hyperparathyroidism
Pheochromocytomas, multiple mucosal neuromas, megacolon, skeletal abnormalities (craniosynostosi
s), Marfanoid habitus, abnormal dental enamel
Genetic defect
RET proto-oncogene mutation
• Any patient suspected to develop MEN 2A or MEN 2B or with a positive family history should be
followed and screened annually for Pheochromocytoma with urine VMA (vanillylmandelic acid)
Hyperparathyroidism with serum calcium Medullary carcinoma with serum calcitonin
• Pheochromocytoma with urine VMA (vanillylmandelic acid)
• Hyperparathyroidism with serum calcium
• Medullary carcinoma with serum calcitonin
• RET mutation testing should be performed routinely for at-risk members of MEN2 families
• Prophylactic thyroidectomy is advised for patients with MEN2A syndrome before 5 to 6 years of age
and with MEN2B syndrome before the age of 1-year
• Pheochromocytoma with urine VMA (vanillylmandelic acid)
• Hyperparathyroidism with serum calcium
• Medullary carcinoma with serum calcitonin
Option B: Clinical observation and follow up
• Clinical observation and follow-up are not recommended in MEN-2B patients.
Option C: Regular FNAC
• Regular FNAC is not recommended in MEN-2B patients because virtually all these patients will
develop medullary thyroid carcinoma. Therefore, prophylactic thyroidectomy is the preferred approach.
Option D: All of the above
• We only perform prophylactic thyroidectomy in MEN-2B patients to prevent the development of
medullary thyroid carcinoma.

Solution for Question 2:


Option D: Osteosclerosis
• The above clinical case is suggestive of primary hyperparathyroidism (HPT). Osteosclerosis is not a
feature of primary HPT.
• Primary hyperparathyroidism (HPT): The abnormality in the parathyroid gland leads to the increased
secretion of parathyroid hormone. Common among females. Causes: Parathyroid adenoma,
Parathyroid hyperplasia, Parathyroid carcinoma Most are asymptomatic; if symptomatic, they have the
following symptoms: Pentad of symptoms Kidney stones are composed of calcium phosphate or
oxalate. Painful bones (bone and joint pain) Osteopenia Osteoporosis Both occur with a reduction in
bone mineral density Osteitis fibrosa cystica (elevated alkaline phosphatase) Pathological fractures

Page 10

136
Abdominal groans Constipation and decreased appetite, nausea, heartburn, pruritus Peptic ulcer
disease Pancreatitis Cholelithiasis (due to the formation of calcium bilirubinate stones) Psychic moans
(depression and memory loss): florid psychosis, obtundation, or coma Fatigue overtones (weakness,
fatigue): Prominent in proximal muscle groups Other features Chondrocalcinosis, gout, and pseudogout
Calcification at ectopic sites (blood vessels, cardiac valves, and skin) Band keratopathy (deposition of
calcium in Bowman’s membrane just inside the iris of the eye) Diagnosis:
• The abnormality in the parathyroid gland leads to the increased secretion of parathyroid hormone.
Common among females.
• Causes: Parathyroid adenoma, Parathyroid hyperplasia, Parathyroid carcinoma
• Most are asymptomatic; if symptomatic, they have the following symptoms:
• Pentad of symptoms Kidney stones are composed of calcium phosphate or oxalate. Painful bones
(bone and joint pain) Osteopenia Osteoporosis Both occur with a reduction in bone mineral density
Osteitis fibrosa cystica (elevated alkaline phosphatase) Pathological fractures Abdominal groans
Constipation and decreased appetite, nausea, heartburn, pruritus Peptic ulcer disease Pancreatitis
Cholelithiasis (due to the formation of calcium bilirubinate stones) Psychic moans (depression and
memory loss): florid psychosis, obtundation, or coma Fatigue overtones (weakness, fatigue): Prominent
in proximal muscle groups Other features Chondrocalcinosis, gout, and pseudogout Calcification at
ectopic sites (blood vessels, cardiac valves, and skin) Band keratopathy (deposition of calcium in
Bowman’s membrane just inside the iris of the eye)
• Kidney stones are composed of calcium phosphate or oxalate.
• Painful bones (bone and joint pain)
• Osteopenia
• Osteoporosis
• Both occur with a reduction in bone mineral density
• Osteitis fibrosa cystica (elevated alkaline phosphatase)
• Pathological fractures
• Abdominal groans
• Constipation and decreased appetite, nausea, heartburn, pruritus
• Peptic ulcer disease
• Pancreatitis
• Cholelithiasis (due to the formation of calcium bilirubinate stones)
• Psychic moans (depression and memory loss): florid psychosis, obtundation, or coma
• Fatigue overtones (weakness, fatigue): Prominent in proximal muscle groups
• Other features
• Chondrocalcinosis, gout, and pseudogout
• Calcification at ectopic sites (blood vessels, cardiac valves, and skin)
• Band keratopathy (deposition of calcium in Bowman’s membrane just inside the iris of the eye)
• Diagnosis:
• The abnormality in the parathyroid gland leads to the increased secretion of parathyroid hormone.
Common among females.

Page 11

137
• Causes: Parathyroid adenoma, Parathyroid hyperplasia, Parathyroid carcinoma
• Most are asymptomatic; if symptomatic, they have the following symptoms:
• Pentad of symptoms Kidney stones are composed of calcium phosphate or oxalate. Painful bones
(bone and joint pain) Osteopenia Osteoporosis Both occur with a reduction in bone mineral density
Osteitis fibrosa cystica (elevated alkaline phosphatase) Pathological fractures Abdominal groans
Constipation and decreased appetite, nausea, heartburn, pruritus Peptic ulcer disease Pancreatitis
Cholelithiasis (due to the formation of calcium bilirubinate stones) Psychic moans (depression and
memory loss): florid psychosis, obtundation, or coma Fatigue overtones (weakness, fatigue): Prominent
in proximal muscle groups Other features Chondrocalcinosis, gout, and pseudogout Calcification at
ectopic sites (blood vessels, cardiac valves, and skin) Band keratopathy (deposition of calcium in
Bowman’s membrane just inside the iris of the eye)
• Kidney stones are composed of calcium phosphate or oxalate.
• Painful bones (bone and joint pain)
• Osteopenia
• Osteoporosis
• Both occur with a reduction in bone mineral density
• Osteitis fibrosa cystica (elevated alkaline phosphatase)
• Pathological fractures
• Abdominal groans
• Constipation and decreased appetite, nausea, heartburn, pruritus
• Peptic ulcer disease
• Pancreatitis
• Cholelithiasis (due to the formation of calcium bilirubinate stones)
• Psychic moans (depression and memory loss): florid psychosis, obtundation, or coma
• Fatigue overtones (weakness, fatigue): Prominent in proximal muscle groups
• Other features
• Chondrocalcinosis, gout, and pseudogout
• Calcification at ectopic sites (blood vessels, cardiac valves, and skin)
• Band keratopathy (deposition of calcium in Bowman’s membrane just inside the iris of the eye)
• Diagnosis:
• Kidney stones are composed of calcium phosphate or oxalate.
• Painful bones (bone and joint pain)
• Osteopenia
• Osteoporosis
• Both occur with a reduction in bone mineral density
• Osteitis fibrosa cystica (elevated alkaline phosphatase)
• Pathological fractures
• Abdominal groans

Page 12

138
• Constipation and decreased appetite, nausea, heartburn, pruritus
• Peptic ulcer disease
• Pancreatitis
• Cholelithiasis (due to the formation of calcium bilirubinate stones)
• Psychic moans (depression and memory loss): florid psychosis, obtundation, or coma
• Fatigue overtones (weakness, fatigue): Prominent in proximal muscle groups
• Other features
• Chondrocalcinosis, gout, and pseudogout
• Calcification at ectopic sites (blood vessels, cardiac valves, and skin)
• Band keratopathy (deposition of calcium in Bowman’s membrane just inside the iris of the eye)
Serum tests
Alteration
Calcium
Increased, except in normocalcemic primary HPT
Intact PTH
Increased
Chloride
Increased or high normal
Phosphate
Decreased or low normal
Chloride to phosphate ratio
Increased ( usually >33)
Magnesium
Unchanged or decreased (osteitis fibrosa cystica)
Uric acid
Normal or increased
Alkaline phosphatase
Acid-base status
Mild hyperchloremic metabolic acidosis
Calcium to creatinine clearance ratio
>0.02
Urine tests
24-hour urinary calcium
Preoperative localization studies
Preoperative Non-invasive

Page 13

139
Preoperative Invasive
Sestamibi-technetium 99m scan
FNAB (Fine needle aspiration biopsy)
Ultrasound
Angiogram
CT scan
Venous sampling
MRI scan
Intraoperative
Four-dimensional CT scan
PTH assay (to determine the adequacy of parathyroid resection)
• Sestamibi-technetium 99m scan Investigation of choice: Most widely used and accurate modality with
a sensitivity >80% Sestamibi concentrates in mitochondrial-rich tissue Initially was introduced for
cardiac imaging
• Investigation of choice: Most widely used and accurate modality with a sensitivity >80%
• Sestamibi concentrates in mitochondrial-rich tissue
• Initially was introduced for cardiac imaging
• Treatment: Proper treatment results in the resolution of osteitis fibrosa cystica and decreased
formation of renal stones, improving bone mineral density. Medical Bisphosphonates Hormone
replacement therapy Selective estrogen receptor modulators Surgery: parathyroidectomy - Treatment
of choice
• Medical Bisphosphonates Hormone replacement therapy Selective estrogen receptor modulators
• Bisphosphonates
• Hormone replacement therapy
• Selective estrogen receptor modulators
• Surgery: parathyroidectomy - Treatment of choice
• Investigation of choice: Most widely used and accurate modality with a sensitivity >80%
• Sestamibi concentrates in mitochondrial-rich tissue
• Initially was introduced for cardiac imaging
• Medical Bisphosphonates Hormone replacement therapy Selective estrogen receptor modulators
• Bisphosphonates
• Hormone replacement therapy
• Selective estrogen receptor modulators
• Surgery: parathyroidectomy - Treatment of choice
• Bisphosphonates
• Hormone replacement therapy
• Selective estrogen receptor modulators

Page 14

140
Other options: A, B and C
• Generalized osteoporosis, Renal calculi, and Hypercalcemia are commonly seen in primary
hyperparathyroidism.
.

Solution for Question 3:


Option D: All of the above
• This female presenting with the signs of virilization (new facial hair and coarseness of voice), purple
striae, and a heterogenous adrenal mass on abdominal CT scan most likely have adrenocortical
carcinoma.
• ADRENOCORTICAL CARCINOMA (ACC) Rare tumor – incidence is 1 or 2 per million population
only. Bimodal age distribution with disease peaks before the age of five and in the fourth to fifth decade
of life More aggressive in adults compared to children Women are more likely to develop ACC than
men - (female-to-male ratio ~ 1.5 to 2.5:1) >50% are functional Cushing’s syndrome is most commonly
seen, followed by virilization Radiographic evaluation is done primarily with CT, which typically reveals
a heterogeneous mass with irregular or indistinct borders, central necrosis, and invasion of adjacent
structures Size of the adrenal mass is the single most important criterion to help diagnose malignancy
Metastases to the liver, lung, and lymph nodes may be found Treatment requires radical resection,
which is achieved by an open approach Primary chemotherapeutic agent of choice – mitotane
(adrenolytic drug that directly suppresses the adrenal cortex) Prognosis is usually poor, even in
patients with early-stage adrenocortical tumors.
• Rare tumor – incidence is 1 or 2 per million population only.
• Bimodal age distribution with disease peaks before the age of five and in the fourth to fifth decade of
life
• More aggressive in adults compared to children
• Women are more likely to develop ACC than men - (female-to-male ratio ~ 1.5 to 2.5:1)
• >50% are functional
• Cushing’s syndrome is most commonly seen, followed by virilization
• Radiographic evaluation is done primarily with CT, which typically reveals a heterogeneous mass with
irregular or indistinct borders, central necrosis, and invasion of adjacent structures
• Size of the adrenal mass is the single most important criterion to help diagnose malignancy
• Metastases to the liver, lung, and lymph nodes may be found
• Treatment requires radical resection, which is achieved by an open approach
• Primary chemotherapeutic agent of choice – mitotane (adrenolytic drug that directly suppresses the
adrenal cortex)
• Prognosis is usually poor, even in patients with early-stage adrenocortical tumors.
• Rare tumor – incidence is 1 or 2 per million population only.
• Bimodal age distribution with disease peaks before the age of five and in the fourth to fifth decade of
life
• More aggressive in adults compared to children

Page 15

141
• Women are more likely to develop ACC than men - (female-to-male ratio ~ 1.5 to 2.5:1)
• >50% are functional
• Cushing’s syndrome is most commonly seen, followed by virilization
• Radiographic evaluation is done primarily with CT, which typically reveals a heterogeneous mass with
irregular or indistinct borders, central necrosis, and invasion of adjacent structures
• Size of the adrenal mass is the single most important criterion to help diagnose malignancy
• Metastases to the liver, lung, and lymph nodes may be found
• Treatment requires radical resection, which is achieved by an open approach
• Primary chemotherapeutic agent of choice – mitotane (adrenolytic drug that directly suppresses the
adrenal cortex)
• Prognosis is usually poor, even in patients with early-stage adrenocortical tumors.

(Red arrow indicates adrenocortical tumor)


Option A: Rare tumor
• The incidence of adrenocortical cancer is only 1 or 2 per million.
Option B: More than half are functional
• Approximately 60% of adrenocortical cancers secrete sufficient hormones to produce symptoms.
Option C: Most commonly associated with Cushing syndrome
• Approximately 45% of adrenocortical cancers present with Cushing syndrome.

Solution for Question 4:


Option D: All of the above
• In the clinical scenario above, patients have secondary hyperparathyroidism due to chronic renal
failure.

Page 16

142
• Secondary hyperparathyroidism is defined as adaptive parathyroid gland hyperplasia and increased
production of PTH
• Causes of secondary hyperparathyroidism: Chronic renal failure 25(OH) vitamin D deficiency
Inadequate calcium or vitamin D intake Malabsorption syndromes Celiac disease Cystic fibrosis Short
gut syndrome Bariatric procedures Medications Lithium Diuretics Metabolic abnormalities
Hypermagnesemia Hyperphosphatemia Congenital disorders Transient neonatal hyperparathyroidism
DiGeorge syndrome
• Chronic renal failure
• 25(OH) vitamin D deficiency
• Inadequate calcium or vitamin D intake
• Malabsorption syndromes Celiac disease Cystic fibrosis Short gut syndrome Bariatric procedures
• Celiac disease
• Cystic fibrosis
• Short gut syndrome
• Bariatric procedures
• Medications Lithium Diuretics
• Lithium
• Diuretics
• Metabolic abnormalities Hypermagnesemia Hyperphosphatemia
• Hypermagnesemia
• Hyperphosphatemia
• Congenital disorders Transient neonatal hyperparathyroidism DiGeorge syndrome
• Transient neonatal hyperparathyroidism
• DiGeorge syndrome
• Chronic renal failure
• 25(OH) vitamin D deficiency
• Inadequate calcium or vitamin D intake
• Malabsorption syndromes Celiac disease Cystic fibrosis Short gut syndrome Bariatric procedures
• Celiac disease
• Cystic fibrosis
• Short gut syndrome
• Bariatric procedures
• Medications Lithium Diuretics
• Lithium
• Diuretics
• Metabolic abnormalities Hypermagnesemia Hyperphosphatemia
• Hypermagnesemia

Page 17

143
• Hyperphosphatemia
• Congenital disorders Transient neonatal hyperparathyroidism DiGeorge syndrome
• Transient neonatal hyperparathyroidism
• DiGeorge syndrome
• Celiac disease
• Cystic fibrosis
• Short gut syndrome
• Bariatric procedures
• Lithium
• Diuretics
• Hypermagnesemia
• Hyperphosphatemia
• Transient neonatal hyperparathyroidism
• DiGeorge syndrome

Option A: Commonly occurs in Chronic renal failure


• Most chronic kidney disease patients develop secondary hyperparathyroidism due to increased serum
phosphate levels and decreased serum calcium levels, stimulating parathyroid glands to release PTH.
Option B: Related to hyperphosphatemia
• Most chronic kidney disease patients develop hyperphosphatemia due to decreased renal clearance
of phosphate.
Option C: Patients are generally hypocalcemic
• Most chronic kidney disease patients develop hypocalcemia due to decreased vitamin D levels and
phosphorus retention leading to binding with calcium.

Page 18

144
Solution for Question 5:
Correct Option B: Most common Intracranial solid tumor of childhood
• This is incorrect statement.
• The above-described clinical condition is neuroblastoma
NEUROBLASTOMA
• Most common extracranial solid tumor in infants and children, arises from neural crest cells
• Malignant neoplasm of the sympathetic nervous system
• Over 80% of cases are present before the age of 4, and the peak incidence is 2 years
• It can originate anywhere along the sympathetic chain
• Most common site: 65% occur in the abdomen, with 50% localized to the adrenal medulla, neck (5%),
chest (20%), or pelvis (5%)
• Exhibit spontaneous regression completely, especially in stage 4S with a spontaneous cure
• Mostly occur sporadically, but 1% to 2% are familial
• Homer Wright pseudorosettes are characteristic of neuroblastoma
• Metastasizes to the liver, lungs, cortices of long bones, bone marrow, regional or distant lymph nodes,
skull, eyes, and skin
• Clinical features Asymptomatic Constitutional symptoms Enlarging mass, Pain, Abdominal distension
Lymphadenopathy Respiratory distress Bladder or bowel disturbance (diarrhea) - Tumor metabolites
(VIP) Horner syndrome - Stellate ganglion involvement Periorbital swelling and proptosis (raccoon
eyes) - ocular metastases Long bone pain and tenderness- bony metastasis Rubbery lymph nodes in
the neck or axilla - Lymph node metastases Limping, paralysis or weakness, and failure to thrive - bone
marrow metastases Palpable skull nodule - skull metastases Blueberry muffin syndrome (blue
subcutaneous nodules) - skin metastases
• Asymptomatic
• Constitutional symptoms
• Enlarging mass, Pain, Abdominal distension
• Lymphadenopathy
• Respiratory distress
• Bladder or bowel disturbance (diarrhea) - Tumor metabolites (VIP)
• Horner syndrome - Stellate ganglion involvement
• Periorbital swelling and proptosis (raccoon eyes) - ocular metastases
• Long bone pain and tenderness- bony metastasis
• Rubbery lymph nodes in the neck or axilla - Lymph node metastases
• Limping, paralysis or weakness, and failure to thrive - bone marrow metastases
• Palpable skull nodule - skull metastases
• Blueberry muffin syndrome (blue subcutaneous nodules) - skin metastases

Page 19

145
• Asymptomatic
• Constitutional symptoms
• Enlarging mass, Pain, Abdominal distension
• Lymphadenopathy
• Respiratory distress
• Bladder or bowel disturbance (diarrhea) - Tumor metabolites (VIP)
• Horner syndrome - Stellate ganglion involvement
• Periorbital swelling and proptosis (raccoon eyes) - ocular metastases
• Long bone pain and tenderness- bony metastasis
• Rubbery lymph nodes in the neck or axilla - Lymph node metastases
• Limping, paralysis or weakness, and failure to thrive - bone marrow metastases
• Palpable skull nodule - skull metastases
• Blueberry muffin syndrome (blue subcutaneous nodules) - skin metastases

Page 20

146
• Paraneoplastic syndromes (intractable diarrhea)
• Opsoclonus myoclonus syndrome
• Diagnosis Elevated levels of Urinary catecholamines and their metabolites (vanillylmandelic acid or
homovanillic acid) Lactate dehydrogenase (>1500 U/ml) Ferritin >142 ng/mL Neuron-specific enolase
(>100 ng/mL) Histology - Undifferentiated small round blue cells Modified Shimda Classification - Helps
to predict prognosis The Shimada classification describes tumors as either favorable or unfavorable
histology based on the degree of differentiation, the mitosis-karyorrhexis index, and the presence or
absence of schwannian stroma.
• Elevated levels of Urinary catecholamines and their metabolites (vanillylmandelic acid or homovanillic
acid) Lactate dehydrogenase (>1500 U/ml) Ferritin >142 ng/mL Neuron-specific enolase (>100 ng/mL)
• Urinary catecholamines and their metabolites (vanillylmandelic acid or homovanillic acid)
• Lactate dehydrogenase (>1500 U/ml)
• Ferritin >142 ng/mL
• Neuron-specific enolase (>100 ng/mL)
• Histology - Undifferentiated small round blue cells
• Modified Shimda Classification - Helps to predict prognosis
• The Shimada classification describes tumors as either favorable or unfavorable histology based on
the degree of differentiation, the mitosis-karyorrhexis index, and the presence or absence of
schwannian stroma.
• Treatment Localized neuroblastoma: Excision Unresectable tumor: Biopsy, initially treated by
chemotherapy and radiotherapy, followed by surgical resection of residual tumor Disseminated
disease: Chemotherapy (Cyclophosphamide, vincristine, dacarbazine, doxorubicin, Cisplatin)
• Localized neuroblastoma: Excision
• Unresectable tumor: Biopsy, initially treated by chemotherapy and radiotherapy, followed by surgical
resection of residual tumor
• Disseminated disease: Chemotherapy (Cyclophosphamide, vincristine, dacarbazine, doxorubicin,
Cisplatin)

Page 21

147
• Elevated levels of Urinary catecholamines and their metabolites (vanillylmandelic acid or homovanillic
acid) Lactate dehydrogenase (>1500 U/ml) Ferritin >142 ng/mL Neuron-specific enolase (>100 ng/mL)
• Urinary catecholamines and their metabolites (vanillylmandelic acid or homovanillic acid)
• Lactate dehydrogenase (>1500 U/ml)
• Ferritin >142 ng/mL
• Neuron-specific enolase (>100 ng/mL)
• Histology - Undifferentiated small round blue cells
• Modified Shimda Classification - Helps to predict prognosis
• The Shimada classification describes tumors as either favorable or unfavorable histology based on
the degree of differentiation, the mitosis-karyorrhexis index, and the presence or absence of
schwannian stroma.
• Urinary catecholamines and their metabolites (vanillylmandelic acid or homovanillic acid)
• Lactate dehydrogenase (>1500 U/ml)
• Ferritin >142 ng/mL
• Neuron-specific enolase (>100 ng/mL)
• Localized neuroblastoma: Excision
• Unresectable tumor: Biopsy, initially treated by chemotherapy and radiotherapy, followed by surgical
resection of residual tumor
• Disseminated disease: Chemotherapy (Cyclophosphamide, vincristine, dacarbazine, doxorubicin,
Cisplatin)
Other Options
Option A: Homer Wright pseudorosettes are characteristic
• Homer Wright’s pseudorosettes are classically seen on the histological examination of the tumor.
Option C: May undergo spontaneous regression
• Some neuroblastoma variants also undergo spontaneous regression.
Option D: It can originate anywhere along the sympathetic chain
• Neuroblastoma can originate anywhere along the sympathetic chain from the cervical sympathetic
ganglia to the thoracic, abdominal, and pelvic ganglia.

Solution for Question 6:


Option B: Hypertension
• The above-described condition (repeated short spells of palpitation, headaches, and sweating) is
pheochromocytoma, and the most common clinical sign is hypertension.
• Pheochromocytoma: Symptoms and signs: Classic triad Headache Palpitations Diaphoresis
• Symptoms and signs: Classic triad Headache Palpitations Diaphoresis
• Headache

Page 22

148
• Palpitations
• Diaphoresis
• Pheochromocytoma is one of the curable causes of hypertension
• Hypertension related to this tumor is paroxysmal with intervening normotension, sustained with
paroxysms, or sustained hypertension alone
• Diagnosis 24-hour urinary catecholamines and their metabolites Plasma metanephrine levels (98%
sensitive and specific) Fractionated urinary catecholamines (norepinephrine, epinephrine, and
dopamine)
• 24-hour urinary catecholamines and their metabolites
• Plasma metanephrine levels (98% sensitive and specific)
• Fractionated urinary catecholamines (norepinephrine, epinephrine, and dopamine)
• Main hormone secreted by adrenal pheochromocytomas - Epinephrine
• Main hormone secreted by extra-adrenal pheochromocytomas - Norepinephrine (as they lack
phenylethanolamine N-methyltransferase)
• CT scan - 85% to 95% sensitive and 70% to 100% specific
• MRI scan 95% sensitive and 100% specific (as they have a characteristic appearance on T2-weighted
images) It is also the study of choice in pregnant women
• 95% sensitive and 100% specific (as they have a characteristic appearance on T2-weighted images)
• It is also the study of choice in pregnant women
• Treatment Medical Phenoxybenzamine - irreversible long acting α blocker Propanolol - β blocker
(should be initiated only after adequate α blockade to avoid the effects of unopposed α-stimulation)
Nicardipine- the most commonly used calcium channel blocker Metyrosine- catecholamine synthesis
inhibitor Surgery: Adrenalectomy is the treatment of choice
• Medical Phenoxybenzamine - irreversible long acting α blocker Propanolol - β blocker (should be
initiated only after adequate α blockade to avoid the effects of unopposed α-stimulation) Nicardipine-
the most commonly used calcium channel blocker Metyrosine- catecholamine synthesis inhibitor
• Phenoxybenzamine - irreversible long acting α blocker
• Propanolol - β blocker (should be initiated only after adequate α blockade to avoid the effects of
unopposed α-stimulation)
• Nicardipine- the most commonly used calcium channel blocker
• Metyrosine- catecholamine synthesis inhibitor
• Surgery: Adrenalectomy is the treatment of choice
• Syndromes associated with pheochromocytoma:
• MEN 2A
• MEN 2B
• VHL (Von Hippel Lindau)
• Neurofibromatosis type 1 (NF1 gene)
• Sturge Weber syndrome
• Tuberous sclerosis

Page 23

149
• Carney’s syndrome
• Symptoms and signs: Classic triad Headache Palpitations Diaphoresis
• Headache
• Palpitations
• Diaphoresis
• Headache
• Palpitations
• Diaphoresis
• 24-hour urinary catecholamines and their metabolites
• Plasma metanephrine levels (98% sensitive and specific)
• Fractionated urinary catecholamines (norepinephrine, epinephrine, and dopamine)
• 95% sensitive and 100% specific (as they have a characteristic appearance on T2-weighted images)
• It is also the study of choice in pregnant women
• Medical Phenoxybenzamine - irreversible long acting α blocker Propanolol - β blocker (should be
initiated only after adequate α blockade to avoid the effects of unopposed α-stimulation) Nicardipine-
the most commonly used calcium channel blocker Metyrosine- catecholamine synthesis inhibitor
• Phenoxybenzamine - irreversible long acting α blocker
• Propanolol - β blocker (should be initiated only after adequate α blockade to avoid the effects of
unopposed α-stimulation)
• Nicardipine- the most commonly used calcium channel blocker
• Metyrosine- catecholamine synthesis inhibitor
• Surgery: Adrenalectomy is the treatment of choice
• Phenoxybenzamine - irreversible long acting α blocker
• Propanolol - β blocker (should be initiated only after adequate α blockade to avoid the effects of
unopposed α-stimulation)
• Nicardipine- the most commonly used calcium channel blocker
• Metyrosine- catecholamine synthesis inhibitor
Syndromes associated with pheochromocytoma:
Option A: Tachycardia
• Although tachycardia and palpitations are commonly seen in pheochromocytoma, these are not as
common as hypertension.
Option C: Sweating
• Although diaphoresis is commonly seen in pheochromocytoma, it is not as common as hypertension.
Option D: Dyspnea
• Dyspnea is not a common feature of pheochromocytoma.

Page 24

150
Solution for Question 7:
Option C: Hypercalcemia of malignancy
• The patient has presented with a hypercalcemic crisis due to significantly elevated serum calcium
levels. She has lost 10 kg despite a healthy diet.
• The most likely underlying cause of the hypercalcemic crisis is the hypercalcemia of malignancy. Her
history of abnormal mammogram results, back pain at night, and weight loss suggest she has breast
cancer.
• Hypercalcemic crisis: It is a medical and surgical emergency due to severe hypercalcemia from
uncontrolled PTH secretion or other conditions causing hypercalcemia in association with end-organ
dysfunction Patients with parathyroid cancer or familial hyperparathyroidism are more likely to present
with a hypercalcemic crisis Calcium levels are markedly elevated and may be as high as 16 to 20
mg/dL. It generally occurs when serum calcium exceeds 14 mg/dL Present acutely with nausea,
vomiting, fatigue, muscle weakness, confusion, obtundation, decreased level of consciousness,
oliguria/anuria, and hyperkalemia
• It is a medical and surgical emergency due to severe hypercalcemia from uncontrolled PTH secretion
or other conditions causing hypercalcemia in association with end-organ dysfunction
• Patients with parathyroid cancer or familial hyperparathyroidism are more likely to present with a
hypercalcemic crisis
• Calcium levels are markedly elevated and may be as high as 16 to 20 mg/dL. It generally occurs when
serum calcium exceeds 14 mg/dL
• Present acutely with nausea, vomiting, fatigue, muscle weakness, confusion, obtundation, decreased
level of consciousness, oliguria/anuria, and hyperkalemia
• It is a medical and surgical emergency due to severe hypercalcemia from uncontrolled PTH secretion
or other conditions causing hypercalcemia in association with end-organ dysfunction
• Patients with parathyroid cancer or familial hyperparathyroidism are more likely to present with a
hypercalcemic crisis
• Calcium levels are markedly elevated and may be as high as 16 to 20 mg/dL. It generally occurs when
serum calcium exceeds 14 mg/dL
• Present acutely with nausea, vomiting, fatigue, muscle weakness, confusion, obtundation, decreased
level of consciousness, oliguria/anuria, and hyperkalemia
• Stepwise approach for management of Hypercalcemic crisis Step 1: Confirm the diagnosis. Step
2: Restore normal hydration - Through IV fluids (0.9% normal saline at a rate of at least 200mL/hr) to
maintain a urine output of >100 ml/hr. Step 3: Use of loop diuretics (but the patient's blood pressure
must be stable) Calcitonin or bisphosphonates- Preferred drugs for hypercalcemic management after
restoring normal hydration with IV fluids Other drugs - Glucocorticoids, cinacalcet, mithramycin, and
gallium nitrate
• Step 1: Confirm the diagnosis.
• Step 2: Restore normal hydration - Through IV fluids (0.9% normal saline at a rate of at least
200mL/hr) to maintain a urine output of >100 ml/hr.
• Step 3: Use of loop diuretics (but the patient's blood pressure must be stable) Calcitonin or
bisphosphonates- Preferred drugs for hypercalcemic management after restoring normal hydration with
IV fluids Other drugs - Glucocorticoids, cinacalcet, mithramycin, and gallium nitrate
• Use of loop diuretics (but the patient's blood pressure must be stable)

Page 25

151
• Calcitonin or bisphosphonates- Preferred drugs for hypercalcemic management after restoring normal
hydration with IV fluids
• Other drugs - Glucocorticoids, cinacalcet, mithramycin, and gallium nitrate
• Step 1: Confirm the diagnosis.
• Step 2: Restore normal hydration - Through IV fluids (0.9% normal saline at a rate of at least
200mL/hr) to maintain a urine output of >100 ml/hr.
• Step 3: Use of loop diuretics (but the patient's blood pressure must be stable) Calcitonin or
bisphosphonates- Preferred drugs for hypercalcemic management after restoring normal hydration with
IV fluids Other drugs - Glucocorticoids, cinacalcet, mithramycin, and gallium nitrate
• Use of loop diuretics (but the patient's blood pressure must be stable)
• Calcitonin or bisphosphonates- Preferred drugs for hypercalcemic management after restoring normal
hydration with IV fluids
• Other drugs - Glucocorticoids, cinacalcet, mithramycin, and gallium nitrate
• Use of loop diuretics (but the patient's blood pressure must be stable)
• Calcitonin or bisphosphonates- Preferred drugs for hypercalcemic management after restoring normal
hydration with IV fluids
• Other drugs - Glucocorticoids, cinacalcet, mithramycin, and gallium nitrate

• The various mechanisms for hypercalcemia in malignancy are PTHrP - squamous cell lung cancer
releases PTHrP, which causes hypercalcemia by mimicking actions of PTH Excessive activation of
vitamin D as seen in lymphomas Bone invasion by tumor metastases, as seen in metastatic bone
cancer.
• PTHrP - squamous cell lung cancer releases PTHrP, which causes hypercalcemia by mimicking
actions of PTH
• Excessive activation of vitamin D as seen in lymphomas
• Bone invasion by tumor metastases, as seen in metastatic bone cancer.
• PTHrP - squamous cell lung cancer releases PTHrP, which causes hypercalcemia by mimicking
actions of PTH

Page 26

152
• Excessive activation of vitamin D as seen in lymphomas
• Bone invasion by tumor metastases, as seen in metastatic bone cancer.
Incorrect Options A, and B: Parathyroid adenoma, Parathyroid hyperlasia
• Although parathyroid adenoma and parathyroid hyperplasia does cause hypercalcemia, this patient’s
clinical presentation suggests hypercalcemia of malignancy.
Option D: Paget’s disease
• This option is incorrect because serum calcium levels are usually normal in Paget’s disease.

Solution for Question 8:


Option C: Enhances rapidly, contrast stays in it for a relatively long time and washes out late
• Adrenal adenoma is a benign lesion, and the following are radiologic (CT) features distinguishing
benign from malignant adrenal masses.
Benign
Malignant
• Homogenous
• Well encapsulated
• Smooth, regular, and well-defined margins
• Hypoattenuating lesions
• High lipid content
• Rapid washout of contrast material
• Low degree of vascularity
• Non-homogeneous
• Evidence of local invasion or adjacent lymphadenopathy
• Irregular borders, necrosis
• Internal calcifications or hemorrhage
• High vascularity

Page 27

153
• Arrow in the above picture points toward benign adrenal adenoma.
• Adrenal adenomas are benign neoplasms that originate from the adrenal cortex
• Cause: Mutation in CTNNB1 gene, PRKACA gene.
• Clinical manifestation: obesity, hypertension, hyperglycemia, fatigue, depression, menstrual
irregularities, proximal muscle weakness, acne, facial plethora, purple striae, fractures and osteopenia
• Treatment: Unilateral adrenalectomy
Option A: Low attenuation
• Adrenal adenomas appear as hypoattenuating lesions on the abdominal CT scan, so this statement is
also true.
• The reason for low attenuation in adrenal adenomas is intracytoplasmic fat.
Option B: Homogeneous density and well-defined borders
• This is a characteristic feature of adrenal adenomas.
Option D: Calcification is rare
• This statement is true regarding benign adrenal adenomas, as calcification is rarely seen in
adenomas.

Solution for Question 9:


Option D: All of the above
Indications of surgery in Incidentaloma:
• Suspicious imaging characteristics (irregular margins)
• Size >5 cm
• Bilateral adrenal metastasis
• Functional tumor

Page 28

154
• Young age
• Interval tumor growth
• Patient preference
• Patients with worsening hypertension, abnormal glucose tolerance, or osteoporosis
Adrenal incidentalomas:
• Adrenal lesions discovered during imaging performed for unrelated reasons are referred to as
incidentalomas
• The incidence of these lesions identified by CT scans ranges from 0.4 to 4.4%

• Arrow points towards non-functioning adrenal adenocarcinoma.


Differential diagnosis of adrenal incidentaloma:

Benign
• Cortisol producing adenoma
• Aldosteronoma
• Sex steroid-producing adenoma
• Pheochromocytoma
• Myelolipoma
• Cortical adenoma
• Cyst
• Ganglioneuroma
• Hemorrhage
Malignant
• Malignant pheochromocytoma

Page 29

155
• Adrenocortical cancer
• Adrenocortical cancer
• Metastasis
Option A: Size >5 cm
• Incidental adrenal masses with a size > 5 cm have a high risk of being malignant. Therefore,
adrenalectomy is indicated in such masses.
Option B: Bilateral adrenal metastasis
• Bilateral adrenal metastasis indicates malignancy; therefore, adrenalectomy is indicated in such
cases.
Option C: Functional tumor
• Functional incidental adrenal tumors produce troublesome symptoms; therefore, adrenalectomy is
indicated in such cases.

Solution for Question 10:


Option A: Parathyroid
• In the clinical scenario mentioned above, the patient has MEN1 syndrome, and the parathyroid gland
is the first organ affected by this syndrome.
• MEN1 (Wermer’s syndrome): Multiple endocrine neoplasia type 1 (MEN1) is a rare disorder that is
classically characterized by a predisposition to tumors of the parathyroid glands, pancreatic islet cells,
and anterior pituitary gland. Due to germline mutation in the MEN1 gene located in chromosome 11 that
encodes menin Autosomal dominant hereditary cancer syndrome Primary hyperparathyroidism Earliest
and most common manifestation (develops in 80% to 100% of patients by 40 years) Four-gland
hyperplasia with asymmetrical enlargement of the parathyroid gland is usually found It is usually benign
Hypercalcemia is usually the first biochemical abnormality detected in patients with MEN1 Treatment -
Total parathyroidectomy with heterotopic intramuscular autotransplantation of parathyroid tissue grafts
into skeletal muscle or subtotal parathyroidectomy, leaving a vascularized remnant of one parathyroid
gland in situ in the neck Pancreatic neuroendocrine tumors Second most frequent of MEN1 Associated
with risk of malignant progression PETs are the most common cause of syndrome-associated death
Most common functional neuroendocrine tumors associated with MEN1 are Gastrinomas (50%)
(multiple and metastatic at diagnosis) followed by Insulinomas (10% to 15%) Treatment involves
resection to prevent liver metastasis and progression of the disease Pituitary adenomas Most common
- Prolactinomas (10% to 50%)
• Multiple endocrine neoplasia type 1 (MEN1) is a rare disorder that is classically characterized by a
predisposition to tumors of the parathyroid glands, pancreatic islet cells, and anterior pituitary gland.
• Due to germline mutation in the MEN1 gene located in chromosome 11 that encodes menin
• Autosomal dominant hereditary cancer syndrome
• Primary hyperparathyroidism Earliest and most common manifestation (develops in 80% to 100% of
patients by 40 years) Four-gland hyperplasia with asymmetrical enlargement of the parathyroid gland is
usually found It is usually benign Hypercalcemia is usually the first biochemical abnormality detected in
patients with MEN1 Treatment - Total parathyroidectomy with heterotopic intramuscular
autotransplantation of parathyroid tissue grafts into skeletal muscle or subtotal parathyroidectomy,
leaving a vascularized remnant of one parathyroid gland in situ in the neck

Page 30

156
• Earliest and most common manifestation (develops in 80% to 100% of patients by 40 years)
• Four-gland hyperplasia with asymmetrical enlargement of the parathyroid gland is usually found
• It is usually benign
• Hypercalcemia is usually the first biochemical abnormality detected in patients with MEN1
• Treatment - Total parathyroidectomy with heterotopic intramuscular autotransplantation of parathyroid
tissue grafts into skeletal muscle or subtotal parathyroidectomy, leaving a vascularized remnant of one
parathyroid gland in situ in the neck
• Pancreatic neuroendocrine tumors Second most frequent of MEN1 Associated with risk of malignant
progression PETs are the most common cause of syndrome-associated death Most common functional
neuroendocrine tumors associated with MEN1 are Gastrinomas (50%) (multiple and metastatic at
diagnosis) followed by Insulinomas (10% to 15%) Treatment involves resection to prevent liver
metastasis and progression of the disease
• Second most frequent of MEN1
• Associated with risk of malignant progression
• PETs are the most common cause of syndrome-associated death
• Most common functional neuroendocrine tumors associated with MEN1 are Gastrinomas (50%)
(multiple and metastatic at diagnosis) followed by Insulinomas (10% to 15%)
• Treatment involves resection to prevent liver metastasis and progression of the disease
• Pituitary adenomas Most common - Prolactinomas (10% to 50%)
• Most common - Prolactinomas (10% to 50%)
• Multiple endocrine neoplasia type 1 (MEN1) is a rare disorder that is classically characterized by a
predisposition to tumors of the parathyroid glands, pancreatic islet cells, and anterior pituitary gland.
• Due to germline mutation in the MEN1 gene located in chromosome 11 that encodes menin
• Autosomal dominant hereditary cancer syndrome
• Primary hyperparathyroidism Earliest and most common manifestation (develops in 80% to 100% of
patients by 40 years) Four-gland hyperplasia with asymmetrical enlargement of the parathyroid gland is
usually found It is usually benign Hypercalcemia is usually the first biochemical abnormality detected in
patients with MEN1 Treatment - Total parathyroidectomy with heterotopic intramuscular
autotransplantation of parathyroid tissue grafts into skeletal muscle or subtotal parathyroidectomy,
leaving a vascularized remnant of one parathyroid gland in situ in the neck
• Earliest and most common manifestation (develops in 80% to 100% of patients by 40 years)
• Four-gland hyperplasia with asymmetrical enlargement of the parathyroid gland is usually found
• It is usually benign
• Hypercalcemia is usually the first biochemical abnormality detected in patients with MEN1
• Treatment - Total parathyroidectomy with heterotopic intramuscular autotransplantation of parathyroid
tissue grafts into skeletal muscle or subtotal parathyroidectomy, leaving a vascularized remnant of one
parathyroid gland in situ in the neck
• Pancreatic neuroendocrine tumors Second most frequent of MEN1 Associated with risk of malignant
progression PETs are the most common cause of syndrome-associated death Most common functional
neuroendocrine tumors associated with MEN1 are Gastrinomas (50%) (multiple and metastatic at
diagnosis) followed by Insulinomas (10% to 15%) Treatment involves resection to prevent liver
metastasis and progression of the disease

Page 31

157
• Second most frequent of MEN1
• Associated with risk of malignant progression
• PETs are the most common cause of syndrome-associated death
• Most common functional neuroendocrine tumors associated with MEN1 are Gastrinomas (50%)
(multiple and metastatic at diagnosis) followed by Insulinomas (10% to 15%)
• Treatment involves resection to prevent liver metastasis and progression of the disease
• Pituitary adenomas Most common - Prolactinomas (10% to 50%)
• Most common - Prolactinomas (10% to 50%)
• Earliest and most common manifestation (develops in 80% to 100% of patients by 40 years)
• Four-gland hyperplasia with asymmetrical enlargement of the parathyroid gland is usually found
• It is usually benign
• Hypercalcemia is usually the first biochemical abnormality detected in patients with MEN1
• Treatment - Total parathyroidectomy with heterotopic intramuscular autotransplantation of parathyroid
tissue grafts into skeletal muscle or subtotal parathyroidectomy, leaving a vascularized remnant of one
parathyroid gland in situ in the neck
• Second most frequent of MEN1
• Associated with risk of malignant progression
• PETs are the most common cause of syndrome-associated death
• Most common functional neuroendocrine tumors associated with MEN1 are Gastrinomas (50%)
(multiple and metastatic at diagnosis) followed by Insulinomas (10% to 15%)
• Treatment involves resection to prevent liver metastasis and progression of the disease
• Most common - Prolactinomas (10% to 50%)

Option B: Thyroid
• The Thyroid gland remains unaffected in MEN1 syndrome.

Page 32

158
Option C: Adrenal gland
• Adrenal glands are unaffected in MEN1 syndrome. Pheochromocytomas are seen in MEN2.
Option D: Testis
• Testes are not affected in patients with MEN1 syndrome.

Solution for Question 11:


Option D: The superior parathyroid glands are located anterior to RLN (recurrent laryngeal nerve)
IDENTIFICATION OF PARATHYROIDS:
• Approximately 85% of the parathyroid glands are found within 1 cm of the junction of the inferior
thyroid artery and Recurrent laryngeal nerve (RLN)
• Upper parathyroid glands Superior to the junction of the inferior thyroid artery and RLN Dorsal
(posterior) to RLN
• Superior to the junction of the inferior thyroid artery and RLN
• Dorsal (posterior) to RLN
• Lower parathyroid glands Inferior to the junction of the inferior thyroid artery and RLN Ventral
(anterior) to RLN
• Inferior to the junction of the inferior thyroid artery and RLN
• Ventral (anterior) to RLN
• The normal size of the parathyroid gland is equal to the size of a rice grain.
• Normal parathyroid glands weigh 35-50 mg and are 5 by 4 by 2 mm.
• An enlarged parathyroid gland can weigh 50 mg to 20 grams.
• Most common location of ectopic parathyroid gland in descending order: Paraesophageal (28%)
Mediastinal (26%) Intrathymic (24%) Carotid sheath (9%) High cervical or undescended (2%)
• Paraesophageal (28%)
• Mediastinal (26%)
• Intrathymic (24%)
• Carotid sheath (9%)
• High cervical or undescended (2%)
• Superior to the junction of the inferior thyroid artery and RLN
• Dorsal (posterior) to RLN
• Inferior to the junction of the inferior thyroid artery and RLN
• Ventral (anterior) to RLN
• Paraesophageal (28%)
• Mediastinal (26%)
• Intrathymic (24%)

Page 33

159
• Carotid sheath (9%)
• High cervical or undescended (2%)

Option A: Upper parathyroid glands are superior to the junction of the inferior thyroid artery and RLN (r
ecurrent laryngeal nerve)
• Upper parathyroid glands are located 1-2 cm superior to the junction of the inferior thyroid artery and
RLN.
Option B: Most common location of ectopic parathyroid glands is para esophageal
• The underlying reason for the ectopic parathyroid gland is the failure of complete migration of the
parathyroid gland. The most common location for the ectopic parathyroid gland is the para esophageal.
Option C: Lower parathyroid glands are inferior to the junction of the inferior thyroid artery and RLN (re
current laryngeal nerve)
• Lower parathyroid glands are located inferior to the junction of the inferior thyroid artery and RLN
(recurrent laryngeal nerve)

Solution for Question 12:


Option D: Exogenous steroid
• This young female is presenting with features of hirsutism, purple striae, acne, and menstrual
irregularities. The positive dexamethasone suppression test points toward Cushing syndrome.
• The most common cause of Cushing’s syndrome is iatrogenic- exogenous administration of steroids
• Most common endogenous cause of Cushing’s syndrome is ACTH-producing pituitary tumor
Pituitary adenoma or Cushing’s disease (70%)
Ectopic ACTH production (10%)
Ectopic CRH production (<1%)
Adrenal adenoma (10-15%)

Page 34

160
Adrenal carcinoma (5-10%)
Adrenal hyperplasia (5%)
Other:
Pseudo-Cushing’s syndrome
Iatrogenic - exogenous administration of steroids
Features of Cushing’s syndrome:
• Excessive levels of cortisol affect all the organ systems in the body and result in the following
manifestations:

Option A: Adrenal adenoma


• Adrenal adenomas may secrete cortisol and cause Cushing syndrome, but they are a relatively rare
cause of Cushing syndrome. This patient had a normal abdominal CT scan which rules out this option.
Option B: Pituitary adenoma
• Pituitary adenomas secrete ACTH and cause Cushing disease. However this patient is taking
medication for multiple sclerosis (most likely steroids), which is the most likely reason for her
presentation. Moreover, iatrogenic Cushing syndrome is more common than Cushing’s disease,
caused by ACTH-producing pituitary adenomas.
Option C: Adrenal carcinoma
• Adrenal carcinomas may secrete cortisol and cause Cushing syndrome, but they are a relatively rare
cause of Cushing syndrome. This patient had a normal abdominal CT scan which rules out this option.

Solution for Question 13:


Option A : After thyroid surgery -
• Parathyroid hormone deficiency also called hypoparathyroidism.

Page 35

161
• By far the most common cause of hypoparathyroidism is thyroid surgery, particularly total
thyroidectomy with a concomitant central neck dissection is most common cause.
• Can result in hypocalcemia, hyperphosphatemia and increased neuromuscular irritability. Can present
with myalgias, muscle spasms and tetany.
• Diagnosis: lab evaluation of total calcium, albumin
• Treatment: Supplemental calcium and active Vitamin D metabolites.

Other options
Option B : Down syndrome
• Down syndrome is not associated with hypoparathyroidism, DiGeorge syndrome is associated with it.
Option C : After high fever
• This option is incorrect
Option D: MEN 1
• This option is incorrect, MEN 1 causes hyperparathyroidism

Solution for Question 14:


Option B: Nelson’s syndrome
• 30% of patients who undergo bilateral adrenalectomy for Cushing’s disease are at risk of developing
Nelson’s syndrome (corticotroph tumor progression)
• Nelson syndrome is defined as an enlarging pituitary tumor (new mass or an increase of at least 2 mm
in diameter) on MRI due to the progressive growth of pre-existing pituitary tumors due to loss of
feedback inhibition from cortisol.
• Characterized by Increased ACTH levels Hyperpigmentation Visual field defects Headaches
Extraocular muscle palsies

Page 36

162
• Increased ACTH levels
• Hyperpigmentation
• Visual field defects
• Headaches
• Extraocular muscle palsies
• Initial mode of therapy: Transsphenoidal pituitary resection
• This surgery can be performed endoscopically or microscopically
• The prognosis is good if surgery is performed.
• If left untreated, it can prove deadly.
• External beam radiotherapy is used in patients with residual tumors or extracellular invasion
• Increased ACTH levels
• Hyperpigmentation
• Visual field defects
• Headaches
• Extraocular muscle palsies
Option A: Addison’s disease
• Addison’s disease is characterized by adrenal insufficiency.
• The patient can develop hyperpigmentation of the skin, but headaches and the gradual loss of vision
suggest the growth of the pituitary gland in the context of bilateral adrenalectomy. These features are
not seen in Addison’s disease.
Option C: Cushing’s disease
• Cushing disease is characterized by signs and symptoms of hypercortisolism.
• This patient had Cushing syndrome and underwent bilateral adrenalectomy.
• His current symptoms are due to excessive ACTH and growth of the pituitary gland.
Option D: Hypopituitarism
• Instead of hypopituitarism, this patient's pituitary gland is secreting excessive ACTH.

Solution for Question 15:


Option A: 90% are malignant
• The given condition is pheochromocytoma. They arise from sympathetic ganglia and secrete
catecholamines
• According to the rule of 10, pheochromocytomas are: 10%- bilateral 10% - malignant 10% - occur in
pediatric patients 10% - extra-adrenal 10% - familial
• 10%- bilateral
• 10% - malignant

Page 37

163
• 10% - occur in pediatric patients
• 10% - extra-adrenal
• 10% - familial
• Extra adrenal tumors, also called functional paragangliomas, may be found at sites of sympathetic
ganglia in the Organ of Zuckerkandl Neck Mediastinum Abdomen and pelvis
• Organ of Zuckerkandl
• Neck
• Mediastinum
• Abdomen and pelvis
• 10%- bilateral
• 10% - malignant
• 10% - occur in pediatric patients
• 10% - extra-adrenal
• 10% - familial
• Organ of Zuckerkandl
• Neck
• Mediastinum
• Abdomen and pelvis

Option B: 10% occur in the abdomen


• Most pheochromocytomas arise in the adrenal medulla. Almost 10% of pheochromocytomas are
extra-adrenal.
Option C: Pheochromocytoma releases catecholamines

Page 38

164
• Pheochromocytomas are catecholamine-secreting tumors. They release epinephrine and
norepinephrine.
Option D: They arise from sympathetic ganglions
• Pheochromocytomas originate from the sympathetic ganglia.

Solution for Question 16:


Option C: Removal of 3 ½ glands
• Elevated calcium and PTH levels indicate the patient has primary hyperparathyroidism. As the
Sestamibi scan reveals increased uptake in all four glands, the increased PTH levels are due to
parathyroid hyperplasia
TREATMENT OF PRIMARY HYPERPARATHYROIDISM
• Initial correction of hypercalcemia (Rapid IV normal saline followed by forced diuresis with
furosemide)
• Neck exploration is done, and treatment is done accordingly
• In this case, all four glands are hyperplastic, so treatment is done by resection and three and a half
gland or all glands resection and autotransplantation as explained below in table
A single parathyroid adenoma (85%)
• Resection
Two adenomas (5%)
• Resection
Hyperplasia of all four glands (10-15%)
• Resection of 3 ½ glands or
• Resection of all four glands with Parathyroid autotransplantation - into Brachioradialis of the
non-dominant arm
PARATHYROID AUTOTRANSPLANTATION
• Whenever multiple parathyroids are resected, it is preferable to cryopreserve tissue so that it may be
auto transplanted
• Approximately 12-14 pieces of 1 mm are transplanted into the nondominant forearm in pockets made
in the belly of the brachioradialis muscle
Option A: Phosphate Binders
• As too much parathyroid hormone is produced from overgrowth and over-excessive function of one or
more of the parathyroid glands. Parathyroid hormone stimulates the kidney to remove phosphate,
reducing phosphate levels in the bloodstream.
• So there is no point in using phosphate binder in primary hyperparathyroidism as phosphate is already
low
• They are used in secondary hyperparathyroidism, which is due to CKD causing excessive parathyroid
hormone secretion from low Calcium and high phosphate due to deficiency of active form of vitamin D
and positive phosphate balance due to CKD

Page 39

165
• Phosphate binders are used to reduce positive phosphate balance and to lower serum phosphate
levels for people with chronic kidney disease (CKD) to prevent the progression of chronic kidney
disease-mineral and bone disorder

Option B: Calcitonin
• Calcitonin lowers calcium levels and works opposite to parathyroid
• But in this case, we should remove the actual cause, which is causing hypercalcemia and
hyperparathyroidism
• Primary hyperparathyroidism may only be cured by removing the adenoma or overactive parathyroid
glands
Option D: Enlarged glands to be removed
• As all four glands are enlarged, and if all of them are removed, then there will be hypoparathyroidism

Solution for Question 17:


Option C: MEN 1
• The patient has MEN 1 syndrome and has a history of recurrent stones (Primary
hyperparathyroidism), bitemporal hemianopia (Prolactinoma) and a history of epigastric pain after food
intake (Gastrinoma)
• Due to germline mutation in the MEN 1 gene located in chromosome 11 that encodes menin
• Autosomal dominant hereditary cancer syndrome
• Primary hyperparathyroidism Earliest and most common manifestation (develops in 80% to 100% of
patients by 40 years) Four-gland hyperplasia with asymmetrical enlargement of the parathyroid gland is
usually found It is usually benign Hypercalcemia is usually the first biochemical abnormality detected in
patients with MEN 1 Treatment - Total parathyroidectomy with heterotopic intramuscular
auto-transplantation of parathyroid tissue grafts into skeletal muscle or subtotal parathyroidectomy,
leaving a vascularized remnant of one parathyroid gland in situ in the neck
• Earliest and most common manifestation (develops in 80% to 100% of patients by 40 years)
• Four-gland hyperplasia with asymmetrical enlargement of the parathyroid gland is usually found
• It is usually benign
• Hypercalcemia is usually the first biochemical abnormality detected in patients with MEN 1
• Treatment - Total parathyroidectomy with heterotopic intramuscular auto-transplantation of
parathyroid tissue grafts into skeletal muscle or subtotal parathyroidectomy, leaving a vascularized
remnant of one parathyroid gland in situ in the neck
• Pancreatic neuroendocrine tumours Second most frequent of MEN 1 Associated with risk of malignant
progression PNETs are the most common cause of syndrome-associated death Most common
functional neuroendocrine tumours associated with MEN 1 are gastrinomas (50%) (multiple and
metastatic at diagnosis) followed by insulinomas (10% to 15%) Treatment involves resection to prevent
liver metastasis and progression of the disease
• Second most frequent of MEN 1
• Associated with risk of malignant progression

Page 40

166
• PNETs are the most common cause of syndrome-associated death
• Most common functional neuroendocrine tumours associated with MEN 1 are gastrinomas (50%)
(multiple and metastatic at diagnosis) followed by insulinomas (10% to 15%)
• Treatment involves resection to prevent liver metastasis and progression of the disease
• Pituitary adenomas - Most common - prolactinomas (10% to 50%)
• Bronchial and thymic carcinoid
• Adrenocortical nodular hyperplasia
• Cutaneous lesions(lipomas, collagenomas, and angiofibromas)
• Men and women are affected equally
• Endocrine tumours associated with MEN 1 have an earlier age at onset
• Earliest and most common manifestation (develops in 80% to 100% of patients by 40 years)
• Four-gland hyperplasia with asymmetrical enlargement of the parathyroid gland is usually found
• It is usually benign
• Hypercalcemia is usually the first biochemical abnormality detected in patients with MEN 1
• Treatment - Total parathyroidectomy with heterotopic intramuscular auto-transplantation of
parathyroid tissue grafts into skeletal muscle or subtotal parathyroidectomy, leaving a vascularized
remnant of one parathyroid gland in situ in the neck
• Second most frequent of MEN 1
• Associated with risk of malignant progression
• PNETs are the most common cause of syndrome-associated death
• Most common functional neuroendocrine tumours associated with MEN 1 are gastrinomas (50%)
(multiple and metastatic at diagnosis) followed by insulinomas (10% to 15%)
• Treatment involves resection to prevent liver metastasis and progression of the disease
Option A: Zollinger-Ellison syndrome
• Zollinger-Ellison syndrome is a rare condition in which one or more tumors form in pancreas or the
upper part of your small intestine. These tumor are called gastrinomas, and secrete large amounts of
the hormone gastrin, which causes stomach to produce too much acid
• It does not involve any pituitary or parathyroid pathology
Option B: MEN 2A
• Multiple endocrine neoplasia type 2A (MEN 2A) is an inherited disorder caused by genetic changes in
the RET gene.
• Individuals with MEN 2A are at high risk of developing medullary carcinoma of the thyroid. About 50%
will develop pheochromocytoma, a tumor of the adrenal glands which may increase blood pressure.
• Individuals with MEN 2A are also at increased risk for parathyroid adenoma or hyperplasia
(overgrowth of the parathyroid gland).
• Occasionally an itchy skin condition called cutaneous lichen amyloidosis also occurs in people with
type 2A disease. The condition is inherited in an autosomal dominant manner.
Option D: MEN 2B

Page 41

167
• Multiple endocrine neoplasias, type 2B (MEN 2B) has a mutation of the same gene as in MEN 2A and
manifests similarly except for the absence of hyperparathyroidism, the presence of more aggressive
medullary thyroid carcinoma and the presence of multiple mucosal neuromas and a marfanoid habitus.
• Patients should have genetic testing for RET proto-oncogene mutations and blood or urine tests for
pheochromocytoma.
• Often, mucosal neuromas are the earliest sign, occurring in most or all patients. Neuromas appear as
small glistening bumps on the lips, tongue, and buccal mucosae.
• The eyelids, conjunctivae, and corneas also commonly develop neuromas; infants often cannot make
tears. Thickened eyelids and everted, diffusely hypertrophied lips are characteristic.

Solution for Question 18:


Option A: Also known as Sipple syndrome
• Medullary carcinoma thyroid and parathyroid adenoma are seen in patients with MEN 2A (Sipple's)
syndrome. Patients with a family history of MEN 2A syndrome should undergo prophylactic
thyroidectomy before 5 years of age
• MEN 2A (SIPPLE SYNDROME): Medullary thyroid carcinoma Hyperparathyroidism
Pheochromocytoma Hirschsprung disease
• Medullary thyroid carcinoma
• Hyperparathyroidism
• Pheochromocytoma
• Hirschsprung disease
• Over 95% of cases of MEN 2A are associated with mutations in the ret proto-oncogene (encodes
tyrosine kinase receptor involved in the regulation of cell growth and differentiation).
• A Triad of medullary carcinoma thyroid, parathyroid adenoma, and pheochromocytomas are called
MEN 2A
Medullary carcinoma thyroid and parathyroid adenoma are seen in patients with MEN 2A (Sipple's) syn
drome. Patients with a
family history of MEN 2A syndrome should undergo prophylactic thyroidectomy before 5 years of age
MEN 2A (SIPPLE SYNDROME):
• Medullary thyroid carcinoma
• Hyperparathyroidism
• Pheochromocytoma
• Hirschsprung disease
Medullary thyroid carcinoma
Hyperparathyroidism
Pheochromocytoma
Hirschsprung disease

Page 42

168
Over 95% of cases of MEN 2A are associated with mutations in the ret proto-oncogene (encodes tyrosi
ne kinase receptor involved in the regulation of cell growth and differentiation).
A Triad of medullary carcinoma thyroid, parathyroid adenoma, and pheochromocytomas are called ME
N 2A
Option B: Associated with cutaneous lichen amyloidosis
• Cutaneous lichen amyloidosis: Amyloid deposition in the papillary dermis - pruritic cutaneous plaques
interscapular region or extensor surfaces of the extremities.
• An itchy skin condition called cutaneous lichen amyloidosis also occurs in people with Multiple
Endocrine Neoplasia type 2A disease.
• Hirschsprung's disease has been associated with MEN 2A
Cutaneous lichen amyloidosis: Amyloid deposition in the papillary dermis -
pruritic cutaneous plaques interscapular region or extensor surfaces of the extremities.
An itchy skin condition called cutaneous lichen amyloidosis also occurs in people with Multiple Endocri
ne Neoplasia type 2A disease.
Hirschsprung's disease has been associated with MEN 2A

Option C: Caused by germline missense mutations in extracellular cysteine codons of RET


• MEN 2A is caused by germline missense mutations in extracellular cysteine codons of RET
• While MEN 2B is caused by germline missense mutations in the tyrosine kinase domain of RET
MEN 2A is caused by germline missense mutations in extracellular cysteine codons of RET
While MEN 2B is caused by germline missense mutations in the tyrosine kinase domain of RET
Option D: Associated with Medullary thyroid carcinoma, pheochromocytoma, and parathyroid adenoma
• A Triad of medullary thyroid carcinoma, parathyroid adenoma, and pheochromocytoma constitutes
MEN 2A or Sipple syndrome
A Triad of medullary thyroid carcinoma, parathyroid adenoma, and pheochromocytoma constitutes ME
N 2A or Sipple syndrome

Page 43

169
Solution for Question 19:
Option C: MEN 4
• Multiple endocrine neoplasia type 4 (MEN 4) is the latest member of MEN syndrome. MEN 4 is rare
and shares a similar phenotype spectrum to MEN 1. The difference between MEN 1 and MEN 4 is the
germline gene mutation. In MEN 4, there is a mutation in the cyclin-dependent kinase inhibitor 1b gene
(CDKN1B)
• Characterized by Primary hyperparathyroidism
• Possible tumors in the adrenals, reproductive organs, kidneys
• Possible pancreatic, gastric, bronchial and cervical neuroendocrine tumors.
• Autosomal dominant

Option A: MEN 1
• Multiple endocrine neoplasia type 1 (MEN 1) is the most common syndrome.
• MEN 1 is characterized by primary hyperparathyroidism secondary to parathyroid gland hyperplasia,
pituitary adenoma, and pancreatic neuroendocrine tumors.
Option B: MEN 2A
• Multiple endocrine neoplasia type 2A (MEN 2A) is an inherited disorder caused by genetic changes in
the RET gene.
• Individuals with MEN 2A are at high risk of developing medullary carcinoma of the thyroid. About 50%
will develop pheochromocytoma, a tumors of the adrenal glands which may increase blood pressure.
• Individuals with MEN 2A are also at increased risk for parathyroid adenoma or hyperplasia
(overgrowth of the parathyroid gland).
Option D: MEN 2B

Page 44

170
• Multiple endocrine neoplasias, type 2B (MEN 2B) has a mutation of the same gene as in MEN 2A and
manifests similarly except for the absence of hyperparathyroidism, the presence of more aggressive
medullary thyroid carcinoma and the presence of multiple mucosal neuromas and a marfanoid habitus.
• Patients should have genetic testing for RET proto-oncogene mutations and blood or urine tests for
pheochromocytoma.
• Often, mucosal neuromas are the earliest sign

Solution for Question 20:


A. Option A: Cinacalcet is used in controlling hypocalcemia
• Cinacalcet controlling hypercalcemia in refractory parathyroid carcinoma
• Cinacalcet acts as a calcimimetic (i.e., it mimics the action of calcium on tissues) by allosteric
activation of the calcium-sensing receptor that is expressed in various human organ tissues
• Cinacalcet directly lowers parathyroid hormone levels by increasing the sensitivity of the
calcium-sensing receptors to activation by extracellular calcium, resulting in the inhibition of PTH
secretion. The reduction in PTH is associated with a concomitant decrease in serum calcium levels.
• Cinacalcet is indicated for treating secondary hyperparathyroidism in people with chronic kidney
disease on dialysis and hypercalcemia in people with parathyroid carcinoma.
• Cinacalcet can also be used to treat severe hypercalcemia in patients with primary
hyperparathyroidism who cannot undergo parathyroidectomy.
D. Option D: Can be suspected preoperatively byserumcalcium>14mg/dl, elevated PTH,
andpalpableparathyroidglands
• The given clinical condition is parathyroid carcinoma which spreads by local invasion commonly
• Very rare, occurring in 1% of cases of PHPT
• Can be suspected pre-operatively by Severe symptoms Serum Ca2+ >14mg/dl PTH (> 5 times of
normal value) Palpable parathyroid gland

Page 45

171
• Severe symptoms
• Serum Ca2+ >14mg/dl
• PTH (> 5 times of normal value)
• Palpable parathyroid gland
• Severe symptoms
• Serum Ca2+ >14mg/dl
• PTH (> 5 times of normal value)
• Palpable parathyroid gland
B. Option B: Most common route of spread is local invasion
• The most common route of spreading Local invasion
• Lymph node metastasis in 15% of cases
• Distant metastasis 33% of cases

C. Option C: Diagnosed bybiopsyshowingvascularor capsular invasion


• Accurate diagnosis is by histological examination
• The major diagnostic criteria include Vascular or capsular invasion Trabecular or fibrous stroma
Frequent mitosis
• Vascular or capsular invasion
• Trabecular or fibrous stroma
• Frequent mitosis
• Treatment: B/L neck exploration + en bloc excision of the tumour with ipsilateral thyroid lobe ±
Modified Radical Neck Dissection if the lymph node is positive Radiation therapy: Patients with a high
risk for local recurrence Primary therapy for unresectable disease Palliation for bone metastases
Bisphosphonates to treat hypercalcemia associated with parathyroid carcinoma
• B/L neck exploration + en bloc excision of the tumour with ipsilateral thyroid lobe ± Modified Radical
Neck Dissection if the lymph node is positive
• Radiation therapy: Patients with a high risk for local recurrence Primary therapy for unresectable
disease Palliation for bone metastases Bisphosphonates to treat hypercalcemia associated with
parathyroid carcinoma
• Patients with a high risk for local recurrence
• Primary therapy for unresectable disease
• Palliation for bone metastases
• Bisphosphonates to treat hypercalcemia associated with parathyroid carcinoma
• Vascular or capsular invasion
• Trabecular or fibrous stroma
• Frequent mitosis
• B/L neck exploration + en bloc excision of the tumour with ipsilateral thyroid lobe ± Modified Radical
Neck Dissection if the lymph node is positive

Page 46

172
• Radiation therapy: Patients with a high risk for local recurrence Primary therapy for unresectable
disease Palliation for bone metastases Bisphosphonates to treat hypercalcemia associated with
parathyroid carcinoma
• Patients with a high risk for local recurrence
• Primary therapy for unresectable disease
• Palliation for bone metastases
• Bisphosphonates to treat hypercalcemia associated with parathyroid carcinoma
• Patients with a high risk for local recurrence
• Primary therapy for unresectable disease
• Palliation for bone metastases
• Bisphosphonates to treat hypercalcemia associated with parathyroid carcinoma

Solution for Question 21:


Option C: Irreversible alpha blocker
• Complaints of paroxysmal headache, nausea, vomiting, abdominal pain, and palpitations, along with
elevated levels of urinary and plasma metanephrines levels lead to a probable diagnosis of
Pheochromocytoma.
• Pheochromocytoma is characterized by hypertension and low blood volume, resulting from an
excessive concentration of catecholamines in the plasma. With insufficient preoperative
antihypertensive management or untreated hypovolemia, the hemodynamic instability during surgical
treatment of pheochromocytoma may be lethal.
• The main goal of preoperative management of pheochromocytoma is to normalize blood pressure and
heart rate, restore effective circulating blood volume, improve the metabolic condition, and prevent a
patient from catecholamines storm and hemodynamic instability during surgery.
• Phenoxybenzamine is a non-selective, non-competitive α-AR antagonist, which binds irreversibly with
both α1 and α2-AR causes vasodilation, and decreases blood pressure. Phenoxybenzamine is
long-acting, and its effects persist long after it has been discontinued since the effect diminishes only
after α-AR resynthesis.
• The advantage of phenoxybenzamine is that its effect is profound and long-acting, even when
excessive amounts of catecholamines reach the circulation.
Option A: Beta Blocker
• Beta receptors normally cause vasodilation.
• Since catecholamines have both alpha and beta receptor activity, they also cause some degree of
vasodilation through beta receptors.
• When beta blockers are used, then the vasodilatory effect of catecholamines through beta receptors
is antagonized, and there is alpha receptors stimulation which can cause severe hypertension and
end-organ damage.
• So beta blockers are avoided.
Option B: Diuretic

Page 47

173
• Pheochromocytoma patients are already hypovolemic.
• Using a diuretic to control hypertension would be detrimental to the patient.
• A state of hypovolemia is classic for pheochromocytoma due to prolonged vasoconstriction by
circulating catecholamines; further volume depletion can precipitate a crisis.
Option D: None of the above
• Phenoxybenzamine (irreversible long-acting α blocker) controls blood pressure in
pheochromocytoma.

Solution for Question 22:


Option D: Non-Functioning adenoma
• In patients with adrenal masses diagnosed incidentally, the majority of them are found to have
nonfunctioning adenomas.
• Adrenal incidentalomas: Adrenal lesions discovered during imaging performed for unrelated reasons
are referred to as incidentalomas The incidence of these lesions identified by CT scans ranges from 0.4
to 4.4% Non-functioning cortical adenomas account for the majority(36-94%) of adrenal incidentalomas
in patients without a history of cancer
• Adrenal lesions discovered during imaging performed for unrelated reasons are referred to as
incidentalomas
• The incidence of these lesions identified by CT scans ranges from 0.4 to 4.4%
• Non-functioning cortical adenomas account for the majority(36-94%) of adrenal incidentalomas in
patients without a history of cancer
• Adrenal lesions discovered during imaging performed for unrelated reasons are referred to as
incidentalomas
• The incidence of these lesions identified by CT scans ranges from 0.4 to 4.4%
• Non-functioning cortical adenomas account for the majority(36-94%) of adrenal incidentalomas in
patients without a history of cancer

Differential diagnosis of adrenal incidentaloma:


Functioning lesions
Non-functioning lesions
Benign
• Aldosteronoma
• Cortisol producing adenoma
• Sex steroid-producing adenoma
• Pheochromocytoma
• Cortical adenoma
• Myelolipoma

Page 48

174
• Cyst
• Ganglioneuroma
• Hemorrhage
Malignant
• Adrenocortical cancer
• Malignant pheochromocytoma
• Metastasis
• Adrenocortical cancer
Option A: Cushing's Adenoma
• They may present with subclinical Cushing's syndrome, characterized by Weight gain Skin atrophy
Facial fullness Diabetes Hypertension Loss of normal diurnal variation in cortisol secretion Autonomous
cortisol secretion Resistance to suppression by dexamethasone Total cortisol produced and 24-hour
urinary cortisol levels may be normal
• Weight gain
• Skin atrophy
• Facial fullness
• Diabetes
• Hypertension
• Loss of normal diurnal variation in cortisol secretion
• Autonomous cortisol secretion
• Resistance to suppression by dexamethasone
• Total cortisol produced and 24-hour urinary cortisol levels may be normal
• Weight gain
• Skin atrophy
• Facial fullness
• Diabetes
• Hypertension
• Loss of normal diurnal variation in cortisol secretion
• Autonomous cortisol secretion
• Resistance to suppression by dexamethasone
• Total cortisol produced and 24-hour urinary cortisol levels may be normal
Option B: Pheochromocytoma
• These neuroendocrine tumours are capable of producing and releasing massive amounts of
catecholamines, metanephrines, or methoxytyramine, which result in the most common symptoms,
including hypertension (high blood pressure), tachycardia (fast heart rate), and diaphoresis (sweating)
• 5 P's of pheochromocytoma Paroxysmal hypertension Perspiration Pain in the head Pallor
Palpitations

Page 49

175
• Paroxysmal hypertension
• Perspiration
• Pain in the head
• Pallor
• Palpitations
• Paroxysmal hypertension
• Perspiration
• Pain in the head
• Pallor
• Palpitations
Option C: Adrenocortical Carcinoma
• Adrenocortical carcinoma is aggressive cancer originating in the cortex (steroid hormone-producing
tissue) of the adrenal gland.
• In adults presenting with hormonal syndromes, Cushing's syndrome alone is most common, followed
by mixed Cushing's and virilization (glucocorticoid and androgen overproduction).
• Adrenocortical carcinoma is a rare tumor with an incidence of one to two per million population
annually.

Solution for Question 23:


Option C: Inferior petrosal sinus venous sampling
• This patient most likely has Cushing Syndrome because of a positive screening test of 24-hr urinary
cortisol level.

• A high dose dexamethasone suppression test is positive, which indicates a pituitary cause

Page 50

176
• MRI head would reveal any adenoma of the pituitary gland, but as it is inconclusive, it should be
followed by a sampling of inferior petrosal sinus
• Inferior petrosal sinus sampling (IPSS) is an invasive procedure in which adrenocorticotropic hormone
(ACTH) levels are sampled from the veins that drain the pituitary gland.
• These levels are then compared with the ACTH levels in the peripheral blood to determine whether a
pituitary tumor (as opposed to an ectopic source of ACTH) is responsible for ACTH-dependent Cushing
syndrome. IPSS can also be used to establish on which side of the pituitary gland the tumor is located.
• The procedure is usually reserved for patients with consistent ACTH-dependent Cushing's syndrome
without a clear-cut lesion on the pituitary MRI
• In patients with true hypercortisolism, a central-to-peripheral ACTH gradient of ≥2 before, or ≥3 after
CRH administration, is considered diagnostic for a pituitary source of ACTH. Some studies have shown
that if catheters were appropriately placed, a gradient of ≥1.4 for ACTH concentrations between the two
sinuses predicted the side of the tumor with ~70% accuracy
Option A: MRI to assess the adrenals
• Increased ACTH with suppression with high dose dexamethasone test suggests a pituitary cause, so
there is no point for MRI to assess the adrenals
• If ACTH is low and cortisol levels are high, then we should suspect adrenal adenoma and go for MRI.
Option B: CT chest
• As the ACTH is high, it can be due to lung cancer
• But elevated ACTH due to lung cancer will not be suppressed by high dose dexamethasone test
• As ACTH is suppressed with high dose dexamethasone test, so pituitary is the source of elevated
ACTH and Cushing syndrome, and there is no indication of CT chest
Option D: Ultrasound abdomen
• An abdominal ultrasound is used to identify an adrenal tumormay be recommended if suspicious of
adrenal-dependent Cushing's disease.

Solution for Question 24:


Option D: MEN 2A is characterized by cutaneous lichen amyloidosis
• The diagnosis of the above clinical condition is a parathyroid adenoma, and the syndromes
associated with it are multiple endocrine neoplasia syndromes which are MEN 1, MEN 2A, MEN 2B
and MEN 4
• MEN 2A-sipple's syndrome Cutaneous lichen amyloidosis - amyloid deposition in the papillary dermis
- pruritic cutaneous plaques interscapular region or extensor surfaces of the extremities. An itchy skin
condition called cutaneous lichen amyloidosis also occurs in people with Multiple Endocrine Neoplasia
type 2A disease. Hirschsprung's disease has been associated with MEN 2A
• Cutaneous lichen amyloidosis - amyloid deposition in the papillary dermis - pruritic cutaneous plaques
interscapular region or extensor surfaces of the extremities.
• An itchy skin condition called cutaneous lichen amyloidosis also occurs in people with Multiple
Endocrine Neoplasia type 2A disease.
• Hirschsprung's disease has been associated with MEN 2A

Page 51

177
The diagnosis of the above clinical condition is a parathyroid adenoma, and the syndromes associated
with it are multiple endocrine neoplasia syndromes which are MEN 1, MEN 2A, MEN 2B and MEN 4
MEN 2A-sipple's syndrome
• Cutaneous lichen amyloidosis - amyloid deposition in the papillary dermis - pruritic cutaneous plaques
interscapular region or extensor surfaces of the extremities.
• An itchy skin condition called cutaneous lichen amyloidosis also occurs in people with Multiple
Endocrine Neoplasia type 2A disease.
• Hirschsprung's disease has been associated with MEN 2A
Cutaneous lichen amyloidosis - amyloid deposition in the papillary dermis -
pruritic cutaneous plaques interscapular region or extensor surfaces of the extremities.
An itchy skin condition called cutaneous lichen amyloidosis also occurs in people with Multiple Endocri
ne Neoplasia type 2A disease.
Hirschsprung's disease has been associated with MEN 2A

Option A: Most common endocrine abnormality in MEN 1 is pancreatic neuroendocrine tumours


• The most common endocrine abnormality in MEN1 is a multiglandular parathyroid disease
• Due to germline mutation in the MEN1 gene located in chromosome 11 that encodes menin
• Autosomal dominant hereditary cancer syndrome
The most common endocrine abnormality in MEN1 is a multiglandular parathyroid disease
Due to germline mutation in the MEN1 gene located in chromosome 11 that encodes menin
Autosomal dominant hereditary cancer syndrome
Option B: Renal and gonadal tumorsare seen in MEN 3
• Renal and gonadal tumors are seen in MEN 4
• Multiple endocrine neoplasia type 4 (MEN 4) is the latest member of MEN syndrome.
• MEN 4 is rare and shares a similar phenotype spectrum to MEN1.

Page 52

178
• The difference between MEN 1 and MEN 4 is the germline gene mutation.
• In MEN 1, there is a mutation of the MEN1 gene.
• In MEN 4, there is a mutation in the cyclin-dependent kinase inhibitor 1b gene (CDKN1B).
• In humans, the CDKN1B gene is located on chromosome 12p13.
• The CDKN1B gene encodes a protein called p27, a cyclin-dependent kinase inhibitor (CDKI), a
well-known cell cycle regulator.
Renal and gonadal tumors are seen in MEN 4
Multiple endocrine neoplasia type 4 (MEN 4) is the latest member of MEN syndrome.
MEN 4 is rare and shares a similar phenotype spectrum to MEN1.
The difference between MEN 1 and MEN 4 is the germline gene mutation.
In MEN 1, there is a mutation of the MEN1 gene.
In MEN 4, there is a mutation in the cyclin-dependent kinase inhibitor 1b gene (CDKN1B).
In humans, the CDKN1B gene is located on chromosome 12p13.
The CDKN1B gene encodes a protein called p27, a cyclin-dependent kinase inhibitor (CDKI), a
well-known cell cycle regulator.
Option C: Marfanoid features and neuromas are seen in Sipple syndrome
• Marfanoid features and neuromas are seen in MEN 2B/MEN-3
• Multiple endocrine neoplasias, type 2B (MEN 2B) has a mutation of the same gene as in MEN 2A and
manifests similarly except for the absence of hyperparathyroidism, the presence of more aggressive
medullary thyroid carcinoma and the presence of multiple mucosal neuromas and a marfanoid habitus.
• Patients should have genetic testing for RET proto-oncogene mutations and blood or urine tests for
pheochromocytoma.
Marfanoid features and neuromas are seen in MEN 2B/MEN-3
Multiple endocrine neoplasias, type 2B (MEN 2B) has a mutation of the same gene as in MEN 2A and
manifests similarly except for the absence of hyperparathyroidism, the presence of more aggressive m
edullary thyroid carcinoma and the presence of multiple mucosal neuromas and a marfanoid habitus.
Patients should have genetic testing for RET proto-oncogene mutations and blood or urine tests for ph
eochromocytoma.
Option E: The age of prophylactic thyroidectomy in MEN 2A is before 1 year
• The age of prophylactic thyroidectomy in MEN 2A is before 5 years
• While in MEN 2B before 1 year
The age of prophylactic thyroidectomy in MEN 2A is before 5 years
While in MEN 2B before 1 year

Solution for Question 25:


Option E: Phenoxybenzamine is the most preferred drug

Page 53

179
• Adrenal Mass on CT, along with hypertension and elevated urinary vanillyl mandelic acid, leads to a
probable diagnosis of pheochromocytoma
• Pheochromocytoma is characterized by hypertension and low blood volume, resulting from an
excessive concentration of catecholamines in the plasma. With insufficient preoperative
antihypertensive management or untreated hypovolemia, the hemodynamic instability during surgical
treatment of pheochromocytoma may be lethal
• The main goal of preoperative management of pheochromocytoma is to normalize blood pressure and
heart rate, restore effective circulating blood volume, improve the metabolic condition, and prevent a
patient from catecholamines storm and hemodynamic instability during surgery
• Phenoxybenzamine is a non-selective, non-competitive α-AR antagonist, which binds irreversibly with
both α1 and α2-AR and causes vasodilation and decreases blood pressure. Phenoxybenzamine is
long-acting, and its effects persist long after it has been discontinued since the effect diminishes only
after α-AR resynthesis
• The advantage of phenoxybenzamine is that its effect is profound and long-acting, even when
excessive amounts of catecholamines reach the circulation
Option A: Pheochromocytoma is the most commonly diagnosed adrenal incidentaloma
• The most common adrenal incidentaloma is a Non-functioning adenoma
Option B: Most common manifestation is diaphoresis
• The most common manifestation of pheochromocytoma is hypertension
Option C: Best test for diagnosis is urinary catecholamines
• The best test for diagnosis is Fractionated plasma metanephrines
Option D: Extraadrenal pheochromocytoma secretes adrenaline exclusively
• Extraadrenal pheochromocytoma secretes noradrenaline exclusively

Page 54

180
Previous Year Questions
1. In the surgical procedure for breast carcinoma, a frozen section was performed. This technique has
benefits for all individuals, with the exception of:
A. To know sentinel lymph node
B. To know margin positivity
C. To know metastasis
D. To confirm definitive diagnosis immediately in all cases
----------------------------------------
2. Which of the following statements is accurate when it comes to obtaining a sufficient sample for
fine-needle aspiration cytology (FNAC) of the thyroid?
A. 6 follicular clusters each with 10 cells each
B. 3 follicular clusters with 10-15 cells each
C. 10 follicular cell clusters with 6-8 cells each
D. 12 follicular cell clusters with 10-12 cells each
----------------------------------------
3. A 45-year-old female patient underwent a thyroidectomy. Three days after the surgery, she
developed perioral numbness. Which of the following investigations need to be done for her?
A. Free T3, T4
B. T3, T4, TSH
C. Radioiodine scan
D. Calcium, phosphate, and parathormone levels
----------------------------------------
4. A 59-year-old lady presents with a progressive, painless lump in the breast. What is the cause for the
following skin change?

A. Infiltration of subdermal lymphatics


B. Infiltration of the lactiferous duct
C. Involvement of Cooper’s ligament
D. Spread of the tumor to the anterior chest wall

181
----------------------------------------
5. A male patient presented with midline neck swelling. He later developed cervical node enlargement.
The histopathology of the lesion is shown below. Which of the following statements is false about this
condition?

A. Excellent prognosis is associated with this condition


B. It spreads quickly via lymphatics
C. Nuclear characteristics are used for the identification
D. Fine needle aspiration cytology (FNAC) is not diagnostic
----------------------------------------
6. What is the term used to describe the signs and symptoms caused by hormones released from
tumor tissue, which cannot be explained by the tumor's anatomical distribution?
A. Pretumor syndrome
B. Paraneoplastic syndrome
C. Paratumour syndrome
D. Autocrine phenomenon
----------------------------------------
7. Which of the following is the class of shock where urine output is first decreased?
A. Class I
B. Class II
C. Class III
D. Class IV
----------------------------------------
8. What is the probable diagnosis for a 73-year-old male smoker with a previous history of coronary
artery disease who is experiencing blackening of the toes as shown in the image?

Page 2

182
A. Buerger's disease
B. Atherosclerosis
C. Raynaud's disease
D. Giant cell arteritis
----------------------------------------
9. A 55-year-old male patient presents with hematuria and a neovascular mass in the left kidney on a
CT scan, as shown below. What is the diagnosis?

A. Renal cell carcinoma


B. Renal oncocytoma
C. Renal angiomyolipoma
D. Renal cyst
----------------------------------------
10. Which of the following is the best imaging modality to diagnose neuroendocrine tumors?
A. USG
B. CT
C. MRI
D. PET
----------------------------------------
11. Genetic testing for BRCA 1/BRCA 2 is indicated for all of the following except:

Page 3

183
A. Breast cancer diagnosed in a postmenopausal female >50 years of age
B. Male breast cancer
C. Bilateral breast cancer
D. Breast and ovarian cancer
----------------------------------------
12. A woman presented with a BIRADS-5 breast lesion. Which of the following is a good prognostic
factor for this lesion?
A. BRCA-1 positive
B. p53 positive
C. ER positive
D. High Ki-67
----------------------------------------
13. What is the probable diagnosis for a male patient, aged 23, who is experiencing a swelling in the
midline of his neck that shifts when he swallows or sticks out his tongue?
A. Brachial cyst
B. Thyroglossal cyst
C. Plunging ranula
D. Dermoid cyst
----------------------------------------
14. What preoperative intervention can be administered to minimize intraoperative bleeding in a
32-year-old female patient diagnosed with Grave's disease, presenting with eye symptoms and an
enlarged thyroid, who is scheduled to undergo a total thyroidectomy?
A. Potassium iodide
B. Propranolol
C. Propylthiouracil
D. Betamethasone
----------------------------------------
15. A 49 year old female presents with a breast lump. Which of the following findings is in accordance
with basal – like breast cancer?
A. ER+, PR+, HER2-
B. ER+, PR-, HER2-
C. ER-, PR-, HER2-
D. ER-, PR-, HER2+
----------------------------------------
16. Which of the following is not a contraindication for breast conservative surgery?
A. Macromastia
B. Extensive Tumor in medial regions

Page 4

184
C. Prior chest irradiation
D. Pregnancy
----------------------------------------
17. What is the appropriate next step in managing a 37-year-old woman who is unmarried, has never
given birth, is sexually active, and currently taking oral contraceptive pills, considering that her mother
was diagnosed with breast cancer at the age of 60 and her older sister was diagnosed with ovarian
cancer at 40?
A. Genetic counseling and screening for BRCA
B. Routine mammography
C. Stop taking oral contraceptive pills
D. Prophylactic surgery
----------------------------------------
18. A 25-year-old woman presents with a breast lump, no skin changes, and no indurations. On
histology, there was a fibroadenoma appearance. Which of the following categories does it fall under?
A. BIRADS 1 and 2
B. BIRADS 2 or 4
C. BIRADS 2,3,4
D. BIRADS 2
----------------------------------------
19. A lump in the breast is observed in a female patient. Among the options listed below, all but one can
be employed to exclude the presence of cancer. Which one is not applicable for this purpose?
A. Radiological investigations
B. PET scan
C. Clinical examination
D. Histology and cytology
----------------------------------------
20. A 52-year-old female patient presents with HER-2 positive breast cancer that has become resistant
to trastuzumab treatment. The oncologist is considering the next line of treatment for the patient. Which
of the following options would be the most appropriate choice?
A. Lapatinib
B. Sorafenib
C. Vemurafenib
D. Erlotinib
----------------------------------------
21. A 45-year-old woman with early-stage breast cancer is discussing treatment options with her
surgeon. Which of the following statements regarding breast conservation surgery is NOT true?
A. Wide local excision
B. Post-operative radiotherapy

Page 5

185
C. Lymph node dissection
D. Axillary dissection
----------------------------------------
22. A 55-year-old female presents with a breast lump and is seeking medical evaluation. On
examination, a palpable mass is detected in the breast. Which of the following conditions is not
associated with the development of breast cancer?
A. Alcohol intake
B. Late menopause
C. Hormone replacement therapy:
D. Multiparity
----------------------------------------
23. A 45-year-old patient presents with complaints of dysuria and intermittent back pain. On
examination, there are no palpable abdominal masses or tenderness. An X-ray image of the abdomen
is shown. What is the most likely diagnosis?

A. PTH adenoma
B. Wilm's tumor
C. Renal cell carcinoma
D. Lipoma
----------------------------------------
24. What is the probable diagnosis for a patient who previously had surgery for breast carcinoma and is
currently experiencing painless swelling in the arm, as depicted in the image provided?

Page 6

186
A. Lymphedema
B. Lymphangiosarcoma
C. Venous thrombosis
D. Cellulitis
----------------------------------------
25. Gynecomastia is seen in all, Except
(or)
A 68-year-old male presents with breast enlargement and tenderness without a substance abuse
history. Examination shows visibly enlarged breasts with palpable tissue beneath the nipples. What is
the least likely cause of his gynecomastia?
A. Klinefelter syndrome
B. Liver cirrhosis
C. Hypothyroidism
D. Prostate cancer
----------------------------------------
26. Which of the following is true about the flap given in the image?

A. Pedicled TRAM based on superior epigastric artery


B. Free TRAM based on superior and inferior epigastric artery
C. DIEP based on deep inferior epigastric artery
D. Lower Abdominal Fat based inferior epigastric vessels
----------------------------------------
27. In MEN 1 syndrome, where is the most frequent occurrence of gastrinoma observed?
A. Jejunum
B. Ileum
C. Duodenum
D. Stomach
----------------------------------------
28. Which statement regarding retrosternal goitre is accurate?

Page 7

187
A. Operated in all patients regardless of symptoms
B. Sternal incision is always required to operate
C. It receives blood supply from the thoracodorsal artery
D. Most of the retrosternal goiters can be removed by a neck incision
----------------------------------------
29. What could be the reason for lethargy and fatigue in a 15-year-old boy who has undergone bilateral
adrenalectomy for pheochromocytoma, with no signs of bleeding, and blood pressure measuring 80/60
mm Hg on the third day after surgery?
A. Addisonian crisis
B. SIADH
C. Diabetes insipidus
D. Cerebral salt wasting disease
----------------------------------------
30. What could be the most likely reason for a patient presenting with chronic non-pitting edema in the
lower extremities?
A. Lymphedema
B. Arterial Disorders
C. Venous Disorders
D. Coronary heart disease
----------------------------------------
31. What is the definition of the Miami criteria regarding intraoperative parathyroid hormone (PTH)
measurement among the options listed below?
A. Quarter fall in 5 min
B. Quarter fall in 10 min
C. Half fall in 5 min
D. Half fall in 10 min
----------------------------------------
32. A 30-year-old lactating mother presents with pain in her left breast and a fever. On examination of
the affected side, there is localized erythema, warmth, and point tenderness, the fluctuation test is
positive. There is mild axillary lymphadenopathy on the affected side and the contralateral breast is
normal. Repeated aspirations under antibiotic cover were unsuccessful. A USG done showed fluid
collection. What is the management plan for this patient?

Page 8

188
A. Incision and drainage
B. Broad spectrum antibiotics only
C. Breast conservation surgery
D. Punch biopsy
----------------------------------------
33. A 23-year-old woman presents to the surgery out-patient with a left-sided breast lump, which she
noticed on a self-breast examination 3 weeks ago. She has no other complaints. On palpation, the lump
seems to be mobile and firm. FNAC is characterized by stromal and epithelial elements with no signs of
malignancy. What is the best next step in management?
A. Mammography
B. Reassurance and follow-up
C. Surgical excision
D. Start Tamoxifen
----------------------------------------
34. Best prognosis among the following breast cancer subtypes is for:
A. Luminal A
B. Luminal B
C. ER negative
D. Basal like
----------------------------------------
35. The following drain is placed after mastectomy. Identify the type of drain.

Page 9

189
A. Open
B. Closed
C. Semi open
D. Semi closed
----------------------------------------
36. What would be the appropriate next step in the management of a 55-year-old female patient who
has a 4x3 cm lump in the right upper outer quadrant, with no involvement of axillary lymph nodes, and a
BIRADS 4b staging on mammography? The patient underwent breast conservation surgery, and the
final histopathology examination (HPE) report indicated high nuclear-grade ductal carcinoma in situ
(DCIS) with necrosis and a 10mm margin clearance.
A. Adjuvant radiotherapy
B. Adjuvant chemotherapy
C. Follow up 6 monthly for 2 years and then yearly follow up
D. Trastuzumab therapy
----------------------------------------
37. Fluorescence in-situ hybridization (FISH) is required in which of the following interpretations of
HER2/NEU?
A. 1+
B. 2+
C. 3+
D. All of the above
----------------------------------------
38. Which of the following statements are true? Due to increasing mammography there occurs over
diagnosis of breast carcinoma Colon cancer screening is done by digital rectal examination Oral cancer
screening is done by visual inspection Cervix cancer screening is done by a pap smear
A. 2, 3, 4
B. 1, 2, 3, 4
C. 4 only
D. 1, 3, 4

Page 10

190
----------------------------------------
39. Parathyroid gland post-surgery is commonly transferred to?
A. Triceps
B. Biceps
C. Sartorius
D. Brachioradialis
----------------------------------------
40. A patient presents with a painless neck swelling. The histology of the tissue is as given in the image
below. What is the diagnosis?

A. Medullary carcinoma
B. Non Hodgkin’s lymphoma
C. Grave’s disease
D. Hashimoto’s thyroiditis
----------------------------------------
41. What is the name of the procedure in which a probe is inserted into the duct from which the
discharge is emerging, followed by making a tennis racquet incision and dissecting the flap to reach the
duct, and finally excising the duct?
A. Macrodochectomy
B. Microdochectomy
C. Cone excision
D. Wide excision of the lactiferous duct
----------------------------------------
42. Extra mammary Paget’s disease is commonly seen in the:
A. Vulva
B. Vagina
C. Uterus
D. Cervix
----------------------------------------

Page 11

191
43. What risks are associated with the initiation of tamoxifen in a patient diagnosed with unilateral
breast cancer?
A. Ovarian cancer
B. Endometrial cancer
C. Breast cancer on other side
D. Cervical cancer
----------------------------------------
44. Which of the following is not included in the Van Nuys grading system for ductal carcinoma in situ?
A. Age of the patient
B. Size of the tumor
C. Hormone receptor staining
D. Comedo-type necrosis
----------------------------------------
45. What is the drug used for estrogen dependant breast cancer?
A. Lapatinib
B. Sunitinib
C. Tamoxifen
D. Adriamycin
----------------------------------------
46. A 50-year-old female patient presents with a 4 x 3 cm irregular mass in the upper outer quadrant of
the right breast, which is not fixed to the skin or the chest wall. A 2 cm lymph node is palpated in the
right axilla. No mass is palpated in the opposite breast. Which of the following investigations would you
perform in this patient? Bilateral Mammography PET Scan Biopsy of the breast mass USG guided
biopsy of the lymph node
A. 1, 2, 3 & 4
B. 1, 2 & 3
C. 1, 3 & 4
D. 1 & 3
----------------------------------------
47. Which of the following is the most common surgical approach for a retrosternal goiter?
A. Mediastinal incision
B. Subxiphoid incision
C. Transaxillary incision
D. Cervical incision
----------------------------------------
48. Which of the following is the most common surgical approach for a retrosternal goiter?
A. Mediastinal incision

Page 12

192
B. Subxiphoid incision
C. Transaxillary incision
D. Cervical incision
----------------------------------------
49. A female patient presents with a breast abscess. Under antibiotic cover, an aspiration was
attempted which failed to reveal any aspirate. What is the next step in management?
A. Incision and Drainage
B. Continue antibiotics only
C. Incision and drainage + Antibiotics
D. Stop antibiotics as no aspirate was collected
----------------------------------------
50. A 30-year-old lactating mother presents with pain in her left breast and a fever. On examination of
the affected side, there is localized erythema, warmth, and point tenderness, and fluctuation test is
positive. Mild axillary lymphadenopathy is on the affected side, and the contralateral breast is normal.
What is the most likely diagnosis for this patient?

A. Lactational mastitis
B. Breast carcinoma
C. Breast abscess
D. Fibroadenoma
----------------------------------------
51. What is the stage of breast cancer shown in the image?

Page 13

193
A. T2c
B. T4a
C. T2a
D. T4b
----------------------------------------
52. Which of the following tests would a surgeon advise first in a patient with a large thyroid swelling?
A. Ultrasound of the neck
B. FNAC
C. Thyroid function tests
D. Radioiodine uptake
----------------------------------------
53. A 45 year old lady presented with a swelling in neck. She had an exposure to radiation in the past.
Identify the type of disease , image of FNAC is given below:

A. Papillary carcinoma
B. Follicular carcinoma
C. Medullary carcinoma
D. Anaplastic carcinoma
----------------------------------------
54. Which is the most conspicuous sign of breast cancer ?
A. Nipple retraction
B. Peau d’orange
C. Puckering
D. Both nipple retraction and puckering
----------------------------------------
55. What is the most effective treatment for a 27-year-old woman who is 26 weeks pregnant and has
been diagnosed with papillary carcinoma of the thyroid?
A. Thyroid ablation using radioactive Iodine
B. Total thyroidectomy

Page 14

194
C. Observation
D. Hemi-thyroidectomy
----------------------------------------
56. What is the most effective method for diagnosing pheochromocytoma among the following options?
A. 24-hour urinary Vanillylmandelic acid
B. 24-hour urinary Fractionated Metanephrine
C. 24-hour Urinary Hydroxy indole acetic acid
D. 24-hour Urinary Hydroxy tryptamine
----------------------------------------
57. The image shows methylene blue dye being injected into the peritumoral region. What is this
procedure useful for?

A. Localizing the tumor


B. Identifying the margins
C. Sentinel Lymph node biopsy
D. Identifying metastasis
----------------------------------------
58. The probable diagnosis based on the below clinical image of the breast is?

A. Paget's disease
B. Mondor's disease
C. Zuska's disease

Page 15

195
D. Tietze syndrome
----------------------------------------
59. Which of the following does not classify as locally advanced breast cancer?
A. Tumor less than 4 cm
B. Inflammatory breast cancer
C. Chest wall involvement
D. Skin involvement
----------------------------------------
60. A patient with a nodule of the neck presents to the surgery OPD. A Junior resident performs the
following method to palpate it. Identify the procedure shown in the video below.

A. Lahey's method
B. Crile's method
C. Kocher's method
D. Pemberton's method
----------------------------------------
61. Match the correct answer related to the nerve injuries in various surgeries: A. Breast surgery 1.
External Laryngeal nerve B. Thyroid surgery 2. Hypoglossal nerve C. Submandibular excision 3. Vagus
nerve D. Thymectomy 4. Long Thoracic Nerve
A. Breast surgery 1. External Laryngeal nerve
B. Thyroid surgery 2. Hypoglossal nerve
C. Submandibular excision 3. Vagus nerve
D. Thymectomy 4. Long Thoracic Nerve

A. A-1, B-2, C-3, D-4


B. A-2, B-3, C-4, D-1
C. A-3, B-4, C-1, D-2
D. A-4, B-1, C-2, D-3
----------------------------------------
62. During thyroid surgery, which nerve is the least likely to sustain injury among the following options?

Page 16

196
A. Recurrent laryngeal nerve
B. Cervical sympathetic trunk
C. External laryngeal nerve
D. Marginal mandibular nerve
----------------------------------------
63. What is the condition most commonly associated with elevated TSH levels and decreased T4
levels?
A. Hypoparathyroidism
B. Pituitary Adenoma
C. Grave's disease
D. Hashimoto's thyroiditis
----------------------------------------
64. A 40-year-old woman presents with complaints of swelling of the right breast over the past 15 days.
On examination, there was local rise of temperature, and the lump was mobile over the chest wall.
What is the likely diagnosis?

A. Inflammatory carcinoma
B. Giant fibroadenoma
C. Phyllodes tumor
D. Galactocele
----------------------------------------
65. Choose from the following the typical trigger for thyroid storm following thyroidectomy:
A. Rough handling of thyroid during surgery
B. Due to parathyroid gland
C. Inadequate patient preparation
D. Infections
----------------------------------------
66. Best way to localize extra-adrenal pheochromocytoma:
A. X-ray

Page 17

197
B. Nucleotide scan
C. VMA excretion
D. Clinical examination
----------------------------------------
67. Which type of thyroid cancer is often associated with a history of radiation exposure and Increased
RET/ PTC rearrangements and BRAF mutations?
(or)
What kind of thyroid cancer is frequently linked to prior radiation exposure, elevated RET/PTC
rearrangements, and BRAF mutations?
A. Follicular carcinoma
B. Papillary carcinoma
C. Anaplastic carcinoma
D. Medullary carcinoma
----------------------------------------
68.
(or)
A 45-year-old woman comes to the OPD with a history of a lump in the left breast. On clinical
examination, the size of the lump in the left breast is 9x10 cm with the following appearance and no
axillary lymphadenopathy. The histopathology of the biopsy from the lump shows increased stromal
cellularity, and typical leaflike architecture. What is your likely diagnosis?
A. Phyllodes tumour
B. Giant fibroadenoma
C. Inflammatory carcinoma
D. Ductal carcinoma
----------------------------------------
69. Which of the following is the most common cause of thyroid storm following thyroidectomy?
A. Rough handling of thyroid during surgery
B. Due to parathyroid gland
C. Inadequate patient preparation
D. Infections
----------------------------------------
70. In which of the following locations is the dye injected for Sentinel Lymph Node Biopsy?
A. Axilla
B. Tail of spence
C. Nipple
D. Areola
----------------------------------------

Page 18

198
71. What is the accurate portrayal of Paget's disease of the breast?
A. Eczema of skin of nipple
B. Eczema of skin of areola
C. Mastitis carcinomatosis
D. Atrophic scirrhous carcinoma
----------------------------------------
72. Most common benign breast tumour:
A. Fibroadenoma
B. Fibroadenosis
C. DCIS
D. Phyllodes tumour
----------------------------------------
73. Which of the following is not included in the breast triple assessment?
A. Clinical examination
B. Axillary sampling
C. USG
D. FNAC and biopsy
----------------------------------------
74. A 35-year-old female presents with a mobile, rubbery mass in her left breast. A histological image is
provided. What is the most likely diagnosis?

A. Invasive ductal carcinoma


B. Fibroadenoma
C. Phyllodes tumor
D. Lobular carcinoma in-situ
----------------------------------------
75. A 35-year-old woman, 10 years after mastectomy, notices gradual swelling and heaviness in her
right upper limb without pain. On examination, bluish nodules on the skin with no infections are noted.
What is the most likely cause of her symptoms?

Page 19

199
A. Lymphangiosarcoma
B. Thoracic outlet syndrome
C. Chronic venous insufficiency
D. Recurrent breast cancer
----------------------------------------
76. What is the preferred treatment for a benign tumor of the superficial parathyroid gland?
A. Enucleation
B. Superficial parathyroidectomy
C. Chemotherapy
D. Radiation therapy
----------------------------------------

Correct Answers
Question Correct Answer

Question 1 4
Question 2 1
Question 3 4
Question 4 1
Question 5 4
Question 6 2
Question 7 2
Question 8 2
Question 9 3
Question 10 4
Question 11 1
Question 12 3
Question 13 2
Question 14 1
Question 15 3
Question 16 1
Question 17 1
Question 18 4
Question 19 2
Question 20 1
Question 21 4

Page 20

200
Question 22 4
Question 23 1
Question 24 1
Question 25 4
Question 26 3
Question 27 3
Question 28 4
Question 29 1
Question 30 1
Question 31 4
Question 32 1
Question 33 2
Question 34 1
Question 35 2
Question 36 1
Question 37 2
Question 38 4
Question 39 4
Question 40 4
Question 41 2
Question 42 1
Question 43 2
Question 44 3
Question 45 3
Question 46 3
Question 47 4
Question 48 4
Question 49 3
Question 50 3
Question 51 4
Question 52 1
Question 53 1
Question 54 2
Question 55 2
Question 56 2

Page 21

201
Question 57 3
Question 58 2
Question 59 1
Question 60 1
Question 61 4
Question 62 4
Question 63 4
Question 64 1
Question 65 3
Question 66 2
Question 67 2
Question 68 1
Question 69 3
Question 70 4
Question 71 1
Question 72 1
Question 73 2
Question 74 2
Question 75 1
Question 76 2

Solution for Question 1:


Correct Option D - To confirm definitive diagnosis immediately in all cases:
• Frozen section analysis is a procedure performed during surgery to obtain a rapid preliminary
diagnosis of tissue samples.
• However, it is not suitable for confirming a definitive diagnosis immediately in all cases.
• Frozen sections are primarily used for intraoperative decision-making and guiding surgical
management.
• They are particularly helpful in evaluating certain aspects of the surgical procedure, such as margin
positivity and sentinel lymph node involvement, as well as assessing for the presence of metastasis.
Incorrect Options:
Option A - To know sentinel lymph node: Frozen section analysis can be useful for evaluating sentinel l
ymph nodes during surgery. Sentinel lymph node biopsy is a procedure used to determine if cancer ha
s spread to the nearby lymph nodes, and frozen section analysis can provide a
preliminary assessment of lymph node involvement during the surgery.
Option B - To know margin positivity: Frozen section analysis is valuable in assessing the status of sur
gical margins. It helps determine if cancer cells are present at the edges of the excised tissue, indicatin
g positive or close margins. This information is crucial for guiding the surgeon to ensure complete tumo

Page 22

202
ur removal or plan for additional surgical interventions.
Option C - To know metastasis: Frozen section analysis can be used to detect the presence of metasta
sis during surgery. It allows for a rapid assessment of tissue samples from areas suspected of being in
volved in metastatic disease, such as lymph nodes or distant organ sites. This information can help gui
de the surgeon's decision-making and potentially influence the extent of surgical intervention

Solution for Question 2:


Correct Option A - 6 follicular clusters each with 10 cells each:
• Thyroid FNAC is a procedure for obtaining cells from the thyroid gland for cytological examination.
Adequate sampling is crucial to obtaining accurate and reliable results. The Bethesda System for
Reporting Thyroid Cytopathology provides guidelines for evaluating thyroid FNAC samples.
• According to the Bethesda System, an adequate thyroid FNAC sample should contain at least 6
follicular clusters. Each cluster should have a minimum of 10 well-preserved and well-visualized cells.
This ensures sufficient material for cytological evaluation and minimizes the chances of false-negative
or inconclusive results.
Incorrect Options
Option B, C
& D: These options do not meet the recommended criteria for an adequate thyroid FNAC sample.

Solution for Question 3:


Correct Option D - Calcium, phosphate, and parathormone levels:
• Calcium, phosphate, and parathormone levels should be investigated.
• Perioral numbness following thyroidectomy suggests the possibility of hypocalcemia, which can occur
due to parathyroid insufficiency.
• Parathyroid glands are at risk during thyroidectomy, especially if the inferior thyroid artery is ligated
away from the thyroid. This can lead to vascular infarction of the parathyroid glands.
• Hypocalcemia typically manifests 2nd to 5th days post-surgery.
• Symptoms of hypocalcemia include perioral tingling, numbness, and carpopedal spasm.
• Management depends on the severity of symptoms and calcium levels: Mild to moderate symptoms
(Ca level > 8g/dL): Oral calcium supplementation. Severe symptoms like seizures (Ca level < 8g/dL): IV
calcium gluconate administration.
• Mild to moderate symptoms (Ca level > 8g/dL): Oral calcium supplementation.
• Severe symptoms like seizures (Ca level < 8g/dL): IV calcium gluconate administration.
• Mild to moderate symptoms (Ca level > 8g/dL): Oral calcium supplementation.
• Severe symptoms like seizures (Ca level < 8g/dL): IV calcium gluconate administration.
Incorrect Options:
Option A - Free T3, T4:

Page 23

203
• Free T3 and T4 are wrong because they are tests for thyroid function, which are not directly related to
the symptom of perioral numbness.
Option B - T3, T4, TSH:
• Free T3, T4, and thyroid-stimulating hormones are used to assess the levels of thyroid hormones in
the blood, which are important for regulating the body's metabolism. However, perioral numbness is
more likely associated with low calcium levels due to hypoparathyroidism, which can occur as a
complication of thyroidectomy.
Option C - Radioiodine scan:
• It is also incorrect as a radioiodine scan is a diagnostic imaging test that uses radioactive iodine to
evaluate the structure and function of the thyroid gland. This test is not relevant for the evaluation of
perioral numbness following thyroidectomy.

Solution for Question 4:


Correct Option A - Infiltration of subdermal lymphatics:
• Peau d'orange refers to the dimpled, pitted appearance of the breast skin resembling the texture of an
orange peel, as shown in the image. This occurs due to lymphatic obstruction and subsequent edema.
• In breast cancer, infiltration of subdermal lymphatics is a common occurrence. This infiltration leads to
lymphatic obstruction, which manifests clinically as cutaneous edema.
• The lymphatic obstruction caused by tumor cells entering the lymphatics of the skin is responsible for
the characteristic appearance known as Peau d'orange (French for "orange peel skin").
• Infiltration of subdermal lymphatics compromises the normal drainage of lymph fluid from the breast
tissue, leading to the accumulation of fluid in the interstitial spaces and resulting in the characteristic
skin changes.
Incorrect Options:
Option B - Infiltration of lactiferous duct:
• Infiltration of the lactiferous duct is not directly associated with the characteristic skin changes seen in
Peau d'orange.
• Lactiferous duct infiltration may contribute to other symptoms such as nipple discharge or distortion
but does not specifically cause the dimpled appearance of the skin.
Option C - Involvement of Cooper’s ligament:
• Involvement of Cooper's ligaments, which provide structural support to the breast tissue, may lead to
distortion of breast shape but is not directly responsible for the characteristic skin changes of Peau
d'orange.
• Cooper's ligament involvement may cause breast dimpling or retraction, but the primary cause of
Peau d'orange is lymphatic obstruction due to tumor infiltration.

Page 24

204
Option D - Spread of the tumor to the anterior chest wall:
• Spread to the anterior chest wall can cause skin nodules or ulceration, but not skin changes such as
thickening or peau d'orange appearance.

Solution for Question 5:


Correct Option D - Fine needle aspiration cytology (FNAC) is not diagnostic:
• The histopathology shows empty-appearing nuclei with central clearing (“Orphan Annie” eyes), which
is a characteristic finding of papillary carcinoma of the thyroid.
• Fine needle aspiration cytology (FNAC) is a minimally invasive and highly accurate diagnostic tool for
papillary carcinoma of the thyroid.

Page 25

205
• It is often used as the first-line investigation to diagnose thyroid nodules and can differentiate between
benign and malignant lesions with high accuracy.
Incorrect Options:
Option A - Excellent prognosis is associated with this condition: Papillary carcinoma of the thyroid has
an excellent prognosis with a 10-year survival rate of over 90%.
Option B - It spreads quickly via lymphatics: Papillary carcinoma of the thyroid typically spreads throug
h the lymphatic system to the cervical lymph nodes, which is why the patient presented with cervical no
de enlargement.
Option C - Nuclear characteristics are used for identification: Nuclear characteristics, such as the prese
nce of optically clear nuclei (Orphan Annie eye nuclei), pseudo-inclusion bodies, and dystrophic calcific
ation (Psammoma Bodies), are distinctive features used for the identification of papillary carcinoma of t
he thyroid. These histopathological features are key in diagnosing this type of thyroid cancer.

Solution for Question 6:


Correct Option B - Paraneoplastic syndrome:
• Paraneoplastic syndrome refers to a group of signs and symptoms that occur because of hormones or
other substances produced by tumor cells but are not related to the primary tumor's location or direct
invasion of surrounding tissues. These symptoms may be the first indication of an underlying
malignancy and may even precede the diagnosis of cancer.
• Characteristics: Paraneoplastic syndromes can affect various organ systems. They often manifest as
neurologic, endocrine, dermatologic, rheumatologic, or hematologic abnormalities. These syndromes
may precede the diagnosis of cancer, occur simultaneously, or follow the diagnosis.
• Paraneoplastic syndromes can affect various organ systems.
• They often manifest as neurologic, endocrine, dermatologic, rheumatologic, or hematologic
abnormalities.
• These syndromes may precede the diagnosis of cancer, occur simultaneously, or follow the
diagnosis.
• Examples: Lambert-Eaton myasthenic syndrome (LEMS) Cushing syndrome due to ectopic ACTH
production Hypercalcemia due to ectopic parathyroid hormone production Syndrome of inappropriate
antidiuretic hormone (SIADH) secretion
• Lambert-Eaton myasthenic syndrome (LEMS)
• Cushing syndrome due to ectopic ACTH production
• Hypercalcemia due to ectopic parathyroid hormone production
• Syndrome of inappropriate antidiuretic hormone (SIADH) secretion
• Paraneoplastic syndromes can affect various organ systems.
• They often manifest as neurologic, endocrine, dermatologic, rheumatologic, or hematologic
abnormalities.
• These syndromes may precede the diagnosis of cancer, occur simultaneously, or follow the
diagnosis.
• Lambert-Eaton myasthenic syndrome (LEMS)

Page 26

206
• Cushing syndrome due to ectopic ACTH production
• Hypercalcemia due to ectopic parathyroid hormone production
• Syndrome of inappropriate antidiuretic hormone (SIADH) secretion
Incorrect Options:
Option A - Pretumor syndrome: Pretumor syndrome is not a
recognized medical term, and there is no established definition or diagnostic criteria.
Option C - Para tumor syndrome: Para tumor syndrome is also not a
recognized medical term and has no defined meaning.
Option D - Autocrine phenomenon: Autocrine phenomenon refers to the production and secretion of a
hormone or other signaling molecule by a cell, which then acts on the same cell or nearby cells through
specific receptors. This term is not specific to cancer and can occur in various physiological and pathol
ogical contexts.

Solution for Question 7:


Correct Option B - Class II:
• Hypovolemic Shock Class II Involves a blood loss of 15-30%. Characterized by hypoxemia,
hypotension, and generalized vasoconstriction. Urine output typically first decreases to around 20-30
ml/hour. The body initiates compensatory mechanisms to maintain cardiac output.
• Involves a blood loss of 15-30%.
• Characterized by hypoxemia, hypotension, and generalized vasoconstriction.
• Urine output typically first decreases to around 20-30 ml/hour.
• The body initiates compensatory mechanisms to maintain cardiac output.
• In Class II hypovolemic shock, the initial decrease in urine output is a result of the body's
compensatory mechanisms attempting to maintain adequate perfusion to vital organs despite the loss
of blood volume. As the body senses decreased blood volume, it activates mechanisms such as
vasoconstriction to redirect blood flow to essential organs, resulting in decreased blood flow to
non-essential organs like the kidneys. This decreased renal blood flow leads to decreased urine output.
• Involves a blood loss of 15-30%.
• Characterized by hypoxemia, hypotension, and generalized vasoconstriction.
• Urine output typically first decreases to around 20-30 ml/hour.
• The body initiates compensatory mechanisms to maintain cardiac output.
Incorrect Options:
Option A - Class I:
• Class I hypovolemic shock involves a blood loss of up to 15%.
• This stage is often referred to as compensated hypovolemic shock.
• In Class I, compensatory mechanisms effectively maintain cardiac output, and urine output remains
relatively normal.
• Therefore, urine output is not significantly decreased in Class I hypovolemic shock.

Page 27

207
Option C - Class III:
• Class III hypovolemic shock involves a blood loss of 30-40%.
• This stage is known as decompensated shock.
• While urine output decreases further compared to Class II, it typically remains around 5-15 ml/hour.
• The decrease in urine output becomes more pronounced as shock progresses, but it is not the first
sign of decreased urine output in hypovolemic shock.
Option D - Class IV:
• Class IV hypovolemic shock involves a blood loss of more than 40%.
• This stage is referred to as refractory shock.
• In Class IV, there is marked hypotension and complete failure of compensatory mechanisms.
• While urine output significantly decreases or may cease altogether in Class IV shock, it is not the
initial sign of decreased urine output in hypovolemic shock.

Solution for Question 8:


Correct Option B - Atherosclerosis:
• Age of patient is 73 yr ,so correct option will be B
• In the given image the blackening of the toes is likely due to compromised blood flow caused by the
buildup of plaques within the arteries.
• Atherosclerosis is a condition characterized by the accumulation of fatty deposits, cholesterol, and
other substances on the inner walls of the arteries, leading to reduced blood flow to the affected area.
• In this case, the patient's history of coronary artery disease, which is commonly associated with
atherosclerosis, further supports this diagnosis.
Incorrect Options:
Option A - Buerger's disease: Buerger's disease, also known as thromboangiitis obliterans, primarily th
e arteries and veins in the extremities, particularly the hands and feet. It is commonly seen in young ad
ult smokers and is characterized by the formation of blood clots and inflammation in the affected vessel
s, leading to reduced blood flow. Symptoms include pain, ulcers, and gangrene in the affected areas.

Page 28

208
Option C - Raynaud's disease: Raynaud's disease is a condition characterized by episodic narrowing o
f the blood vessels in response to cold temperatures or emotional stress. It typically affects the fingers
and toes, leading to color changes (pale, blue, or red) and sometimes pain or numbness.
Option D - Giant cell arteritis: Giant cell arteritis is an inflammatory condition that primarily affects the m
edium and large arteries, particularly those in the head and neck. While it can cause a
wide range of symptoms, including headache, vision changes, and jaw pain.

Solution for Question 9:


Correct Option C - Renal angiomyolipoma:
• Renal angiomyolipoma is a benign tumor that consists of blood vessels, smooth muscle cells, and fat
cells.
• It can cause hematuria, and on imaging studies like a CT scan, it often appears as a neovascular
mass due to the presence of blood vessels.

Page 29

209
• Renal angiomyolipoma is commonly associated with a genetic condition called tuberous sclerosis
complex (TSC).
Incorrect Options:
Option A - Renal cell carcinoma: Renal cell carcinoma is a
malignant tumor of the kidney that can also present with hematuria and a mass on imaging. However, t
he presence of neovascularization is more characteristic of renal angiomyolipoma in this case.
Option B - Renal oncocytoma: Renal oncocytoma is a benign kidney tumor composed of specialized ce
lls called oncocytic cells. It does not typically cause hematuria or neovascular masses.
Option D - Renal cysts: Renal cysts are fluid-filled sacs that are usually noncancerous and asymptomat
ic. They are unlikely to present with hematuria or neovascular masses on imaging.

Solution for Question 10:


Correct Option D - PET
• PET (Positron Emission Tomography) imaging with specific radiotracers, such as gallium-68, is
considered highly effective in diagnosing neuroendocrine tumors.
• It provides functional information about the metabolic activity of the tumors, aiding in accurate
localization and characterization.
• PET imaging with gallium-68 is particularly useful when other negative imaging tests and minimally
invasive surgical approaches are being considered.
• This modality helps detect even small lesions and provides valuable information for treatment
planning.
Incorrect Options:
Option A - USG (Ultrasound): While ultrasonography can be used as an initial screening tool, it may no
t provide detailed information necessary for the diagnosis and characterization of neuroendocrine tumo
rs, especially smaller lesions or those in challenging locations.

Page 30

210
Option B - CT (Computed Tomography): CT scanning is commonly used in the evaluation of neuroend
ocrine tumors, especially for tumors larger than 2 cm. However, it may not provide as precise functiona
l information as PET imaging, and it may have limitations in detecting smaller lesions or those with low
somatostatin receptor density.
Option C - MRI (Magnetic Resonance Imaging): MRI is valuable for visualizing neuroendocrine tumors,
particularly in soft tissue contrast and delineation of anatomical structures. However, similar to CT, it m
ay have limitations in detecting smaller lesions or those with low somatostatin receptor density compar
ed to PET imaging.

Solution for Question 11:


Correct Option A - Breast cancer diagnosed in a postmenopausal female >50 years of age:
• BRCA1/BRCA2 genetic testing is generally not indicated for breast cancer diagnosed in a
postmenopausal female over 50 years of age because the likelihood of having a pathogenic mutation in
these genes decreases with increasing age, especially after menopause.
• However, if there are other risk factors present, such as a strong family history of breast or ovarian
cancer, then genetic testing may still be considered on an individual basis.
Incorrect Options:
Option B - Male breast cancer:
• BRCA1/BRCA2 mutations can increase the risk of breast cancer in males as well.
• Testing may be indicated in males with breast cancer, particularly if there is a family history of breast
or ovarian cancer.
Option C - Bilateral breast cancer:
• The occurrence of bilateral breast cancer may suggest a genetic predisposition, making
BRCA1/BRCA2 testing relevant.
• Individuals with bilateral breast cancer, especially at a young age, may benefit from genetic testing to
guide treatment and management decisions.
Option D - Breast and ovarian cancer:
• BRCA1/BRCA2 mutations significantly increase the risk of both breast and ovarian cancer.
• Therefore, individuals with a personal or family history of both breast and ovarian cancer are strong
candidates for genetic testing to assess their risk and guide medical management decisions.

Solution for Question 12:


Correct Option C - ER positive
• A BIRADS-5 breast lesion indicates a high likelihood of malignancy. Among the options provided, ER
positivity (Estrogen receptor positivity) is a good prognostic factor in breast cancer. ER-positive tumors
are typically less aggressive and respond well to hormonal therapy.
• The primary prognostic factor in many malignancies is stage, typically assessed using TNM
components.

Page 31

211
• Exceptions include Wilm's tumor, where histology precedes stage in significance, and Soft Tissue
Sarcoma (STS), where tumor grade holds primary importance.
• In breast cancer, the foremost prognostic factor is axillary lymph node status within the TNM staging
system.
• For metastatic breast cancer, the ER/PR status determines responsiveness to hormone therapy,
indicating its crucial prognostic significance.
Incorrect Options:
Option A - BRCA-1 positive: BRCA-1 gene mutations are associated with a
higher risk of developing breast and ovarian cancers, and thus are not considered a
good prognostic factor.
Option B - p53 positive: Mutations in the p53 gene are often found in various types of cancers, includin
g breast cancer. p53 positivity indicates an aggressive form of the disease and is therefore not a
good prognostic factor.
Option D - High Ki-67: Ki-67 is a protein found in proliferating cells. A high Ki-67 index suggests a
high rate of cell division, which is commonly associated with more aggressive tumors and thus is not a
good prognostic factor.

Solution for Question 13:


Correct Option B - Thyroglossal cyst:
• A thyroglossal cyst is a fibrous cyst that forms from a persistent thyroglossal duct. They are the most
common type of congenital anomaly in the neck region. They typically present as a midline neck mass
(in the region of the hyoid bone) that moves upward with swallowing or protrusion of the tongue.

• Typically congenital, presenting between 15-30 years.


• Predominantly found subhyoid, but may also occur suprahyoid, near thyroid cartilage, close to
foramen cecum, or in the floor of the mouth.
• Manifests as a midline swelling in the anterior neck, moves with swallowing and tongue protrusion.

Page 32

212
Incorrect Options:
Option A - Brachial cyst: A brachial cyst, also known as a branchial cleft cyst, typically presents as a lu
mp in the neck, but it's usually located along the anterior border of the sternocleidomastoid muscle, not
in the midline.
Option C - Plunging ranula: A plunging ranula is a type of mucous cyst that occurs in the floor of the m
outh, often lateral to the midline, and typically does not move with swallowing or tongue protrusion.
Option D - Dermoid cyst: A dermoid cyst is a type of teratoma that is often present at birth. They can oc
cur in the midline neck but typically do not move with swallowing or tongue protrusion.

Solution for Question 14:


Correct Option A - Potassium iodide:
• In the preoperative period of a patient with Graves' disease who is scheduled for a total
thyroidectomy, the administration of potassium iodide can help reduce intraoperative bleeding.
Potassium iodide is given to inhibit thyroid hormone synthesis and decrease the thyroid gland's
vascularity. Reducing the blood supply to the thyroid gland can help minimize intraoperative bleeding
during the surgical procedure.
Incorrect Options:
Option B - Propranolol: Propranolol is a beta-blocker that can be used to manage the symptoms of hyp
erthyroidism, such as tremors, palpitations, and anxiety. While it can help control the symptoms associ
ated with Graves' disease, it does not directly affect intraoperative bleeding.
Option C - Propylthiouracil: Propylthiouracil is an anti-thyroid medication used in the management of hy
perthyroidism. It inhibits the production of thyroid hormones. However, it does not have a
direct effect on reducing intraoperative bleeding.
Option D - Betamethasone: Betamethasone is a corticosteroid that has anti-inflammatory and immunos
uppressive properties. While corticosteroids can be used in some surgical settings to reduce inflammati
on and allergic reactions, they are not specifically indicated for reducing intraoperative bleeding in patie
nts with Graves' disease.

Solution for Question 15:


Correct Option C - ER-, PR-, HER2+:
• Basal-like breast cancer refers to a subtype of breast cancer that has specific molecular
characteristics.
• It is typically characterized by the absence of estrogen receptor (ER), progesterone receptor (PR),
and human epidermal growth factor receptor 2 (HER2) expression, which is denoted as ER-, PR-,
HER2-.
Incorrect Options:
Option A - (ER+, PR+, HER2-): (ER+, PR+, HER2-) is incorrect because basal-like breast cancer is def
ined by the absence (negative status) of ER, PR, and HER2 expression. If a
breast cancer is positive for ER and PR, it would not be classified as basal-like.

Page 33

213
Option B - (ER+, PR-, HER2-): (ER+, PR-, HER2-) is incorrect because basal-like breast cancer is char
acterized by the absence of both ER and PR expression. A
tumor that is positive for ER but negative for PR does not fit the profile of basal-like breast cancer.
Option D - (ER-, PR-, HER2+): (ER-, PR-, HER2+) is incorrect because basal-like breast cancer is char
acterized by the absence of HER2 expression. In this option, the tumor is positive for HER2, which is n
ot in accordance with basal-like breast cancer.

Solution for Question 16:


Correct Option A - Macromastia:
• Macromastia is not a contraindication for breast conservative surgery.
• It may be an indication for a more extensive surgery.
Incorrect Options:
Option B - Extensive Tumor in medial regions:
• This can be a relative contraindication, depending on the extent and location of the tumor. While
extensive tumors in the medial regions may pose challenges for breast conservative surgery.
Option C - Prior chest irradiation:
• Prior chest irradiation is typically considered an absolute contraindication for breast conservative
surgery due to the increased risk of complications such as poor wound healing and
radiation-induced malignancy or neoplasm.
Option D - Pregnancy:
• Pregnancy itself is a absolute contraindication for breast conservative surgery.

Solution for Question 17:


Correct Option A - Genetic counseling and screening for BRCA:
• Genetic counseling and screening for BRCA.
• Given the patient's family history of breast and ovarian cancer, genetic counseling and screening for
BRCA mutations would be the next line of management.
Incorrect Options:
Option B - Routine mammography: Routine mammography is typically recommended for women aged
40 and above or those with certain risk factors, such as a strong family history of breast cancer. Howev
er, in this case, the patient is only 37 years old and does not meet the age criteria for routine mammogr
aphy. Additionally, mammography is primarily a screening tool for breast cancer detection and does no
t address the genetic risk associated with BRCA mutations.
Option C
- Stop taking oral contraceptive pills: Oral contraceptive pills (OCPs) have been associated with a slight
ly increased risk of breast cancer. However, the patient's current use of OCPs alone is not sufficient to
warrant stopping their use. The decision to continue or discontinue OCPs should be made in consultati
on with a healthcare provider, considering the patient's individual risk factors and overall health.

Page 34

214
Option D - Prophylactic surgery: Prophylactic surgery, such as prophylactic mastectomy or oophorecto
my (removal of breasts or ovaries), is an option for individuals with a significantly increased risk of brea
st or ovarian cancer due to BRCA mutations. However, it is not the next line of management in this cas
e. Genetic counselling and screening for BRCA mutations would be the initial step to assess the patien
t's genetic risk and determine the appropriate management plan.

Solution for Question 18:


Correct Option D - BIRADS 2:
• A fibroadenoma, with its characteristic appearance and benign nature, falls under BIRADS 2,
representing benign findings on imaging.
0
• Incomplete assessment
• Other assessment is required
NA
1
• Negative
• Annual mammography is recommended
0%
2
• Benign
• Annual mammography is recommended
3
• Probably benign
• Short-term follow up is recommended
> 0-2%
4
• Suspicious
• Biopsy is recommended
• It is further divided into: 4A - Low suspicion 4B - Moderate suspicion 4C - High suspicion
• 4A - Low suspicion
• 4B - Moderate suspicion
• 4C - High suspicion
• 4A - Low suspicion
• 4B - Moderate suspicion
• 4C - High suspicion

Page 35

215
4A: > 2-10%
4B: > 10-50%
4C: > 50-90%
5
• Highly suggestive of malignancy
• Intervention is recommended
> 95%
6
• Biopsy proven malignancy
Incorrect Options:
Options A, B, and C
are incorrect. Refer to the explanation of the correct answer and the learning objective.

Solution for Question 19:


Correct Option B - PET scan:
• While a PET (positron emission tomography) scan is a useful tool in cancer diagnosis and staging, it
is not typically employed as a first-line investigation for evaluating breast lumps. It may be more
commonly used in other cancers, such as lung or colorectal cancer, where it can detect metastases or
assess the extent of the disease.
Incorrect Options:
Option A - Radiological investigations: Radiological investigations such as mammography, ultrasound,
and MRI can provide valuable information about the characteristics of the breast lump, such as its size,
shape, and presence of calcifications. These investigations can help identify features suggestive of ma
lignancy, such as irregular margins or suspicious microcalcifications.
Option C - Clinical examination: A thorough clinical breast examination performed by a
healthcare professional is an important initial step in evaluating a breast lump. The examination involve
s palpation of the lump and assessment of its characteristics, such as size, mobility, and consistency.
Clinical examination can help identify suspicious features, such as fixed or irregular lumps, which may
warrant further investigation.
Option D - Histology and cytology: Histology refers to the microscopic examination of tissue samples o
btained through a
biopsy, while cytology involves the examination of individual cells. These tests provide a definitive diag
nosis and are considered the gold standard for determining the presence of malignancy. They can help
differentiate between benign and malignant breast lesions.

Solution for Question 20:


Correct Option A - Lapatinib:

Page 36

216
• Lapatinib is a targeted therapy that inhibits both HER-2 and EGFR (epidermal growth factor receptor)
pathways. It is considered an effective treatment option for patients with HER-2 positive breast cancer
who have developed resistance to trastuzumab, a monoclonal antibody targeting HER-2.
Incorrect Options:
Option B - Sorafenib: Sorafenib is a multikinase inhibitor that primarily targets angiogenesis pathways i
nvolved in tumor growth. However, it is not specifically indicated for HER-2 positive breast cancer or tra
stuzumab resistance. Sorafenib has shown limited efficacy in HER-2 positive breast cancer and is not
considered a standard treatment option.
Option C - Vemurafenib: Vemurafenib is primarily used in the treatment of melanoma with a specific m
utation (BRAF V600E). It is not indicated for breast cancer, particularly HER-2 positive breast cancer. T
herefore, it is not the best choice for this patient.
Option D - Erlotinib: Erlotinib is primarily used in the treatment of non-small cell lung cancer with specifi
c EGFR mutations. While EGFR pathways are involved in breast cancer, erlotinib has limited activity in
HER-2 positive breast cancer and is not considered the most appropriate treatment option in this case.

Solution for Question 21:


Correct Option D - Axillary dissection:
In breast conservation surgery, axillary dissection is not routinely performed unless there is clinical or r
adiological evidence of lymph node involvement.
• Breast conservation surgery aims to remove the tumor while preserving as much of the breast tissue
as possible. It typically involves wide local excision, which is the surgical removal of the tumor along
with a margin of healthy tissue. This helps to ensure complete tumor removal while maintaining the
cosmetic appearance of the breast.
• However, the extent of lymph node involvement plays a crucial role in determining the further
management and prognosis of breast cancer. If there is suspicion or evidence of lymph node
involvement, further assessment is necessary to determine the extent of the disease and guide
treatment decisions.
• Axillary dissection involves the removal of lymph nodes in the axilla for pathological examination. It
provides information about the presence or absence of cancer cells in the lymph nodes and helps in
staging the disease.
• However, axillary dissection is associated with potential complications such as lymphedema and
decreased shoulder mobility.
• To minimize the morbidity associated with axillary dissection, the concept of sentinel lymph node
biopsy has emerged.
• It involves the identification and removal of the first few lymph nodes that receive drainage from the
breast tumor.
• If the sentinel lymph nodes are negative for cancer cells, it indicates a low likelihood of axillary lymph
node involvement, and further axillary dissection may be avoided.
Incorrect Options:
Option A - Wide local excision: This option is incorrect because wide local excision is actually a true co
mponent of breast conservation surgery. It involves the surgical removal of the tumor along with a mar
gin of healthy tissue to ensure complete tumor removal while preserving as much of the breast tissue a

Page 37

217
s possible.
Option B - Post-op radiotherapy: This option is incorrect because post-operative radiotherapy is a stan
dard part of breast conservation surgery. After the tumor is surgically removed, radiation therapy is typi
cally recommended to the remaining breast tissue to reduce the risk of local recurrence.
Option C - Lymph node dissection: This option is incorrect because lymph node dissection may be perf
ormed as part of breast cancer management. It involves the removal of lymph nodes in the axilla (armp
it) to assess the presence or absence of cancer cells in the lymph nodes and aid in staging the disease
. However, the decision to perform lymph node dissection depends on factors such as clinical and radi
ological findings, and it is not universally done in all cases of breast conservation surgery.

Solution for Question 22:


Correct Option D - Multiparity:
• Multiparity refers to the condition of having given birth to multiple children.
• Several studies have indicated that women who have given birth to multiple children have a reduced
risk of breast cancer compared to nulliparous women (women who have never given birth).
• The protective effect of multiparity is thought to be due to hormonal changes that occur during
pregnancy and breastfeeding, which may have a protective effect against the development of breast
cancer.
Incorrect Options:
Option A - Alcohol intake:
• Alcohol consumption is associated with an increased risk of breast cancer.
Option B - Late menopause:
• Late menopause (the cessation of menstruation at an older age) is associated with an increased risk
of breast cancer.
• Prolonged exposure to estrogen due to late menopause contributes to the increased risk.
Option C - Hormone replacement therapy (HRT):
• Hormone replacement therapy, particularly estrogen-progestin therapy, is associated with an
increased risk of breast cancer in postmenopausal women.
• HRT involves taking estrogen and often progestin to replace the hormones that the body stops
producing during menopause.

Solution for Question 23:


Correct Option A - PTH adenoma:
• The correct diagnosis based on the X-ray image provided is a PTH adenoma.
• PTH adenoma, also known as parathyroid adenoma, is a benign tumor of the parathyroid glands.
• It is a common cause of primary hyperparathyroidism, leading to increased secretion of parathyroid
hormone (PTH) and resulting in hypercalcemia.

Page 38

218
• Patients with PTH adenoma may present with symptoms such as dysuria and back pain, which are
consistent with the clinical scenario mentioned in the question.
Incorrect Options:
Option B - Wilms' tumor: Wilms' tumor is a
type of kidney cancer that primarily affects children. It typically presents as a renal mass on imaging st
udies, but it would not be the appropriate diagnosis based on the X-ray image provided.
Option C - Renal cell carcinoma: Renal cell carcinoma is the most common type of kidney cancer in ad
ults. It may present with various imaging findings, but an X-ray alone is not sufficient to make a
definitive diagnosis of renal cell carcinoma.
Option D - Lipoma: A lipoma is a benign tumor composed of fatty tissue. It typically appears as a
well-defined, soft tissue mass on imaging. However, based on the X-ray image provided, a
lipoma would not be the most appropriate diagnosis.

Solution for Question 24:


Correct Option A - Lymphedema:
• The given image is of Lymphedema
• It is a common complication that can occur after procedures such as mastectomy (partial or total
removal of the breast) or axillary lymph node dissection (removal of the lymph nodes in the armpit).
• During breast cancer surgery, the removal of lymph nodes or damage to the lymphatic vessels can
disrupt the normal flow of lymphatic fluid, leading to the accumulation of fluid in the affected area. This
can result in swelling, discomfort, heaviness, and limited range of motion in the arm, chest, or breast
region on the side of the surgery.
• The risk of developing lymphedema after breast cancer surgery can vary depending on factors such
as the extent of surgery, radiation therapy, obesity, infection, and pre-existing lymphatic abnormalities.
Not all individuals who undergo breast cancer surgery will develop lymphedema, but it is important to
be aware of the risk and take preventive measures.
Incorrect Options:
Option B - Lymphangiosarcoma: The given image is not of Lymphangiosarcoma

Page 39

219
Option C - Venous thrombosis: The given image is not of Venous thrombosis

Option D - Cellulitis: The given image is not of Cellulitis

Page 40

220
Solution for Question 25:
Correct Option D - Prostate cancer:
• Prostate cancer: Prostate cancer is a malignant tumor that develops in the prostate gland. Although
hormonal imbalances can occur in advanced prostate cancer, gynecomastia is not a common
manifestation of this condition.
• The growth of prostate cancer is primarily driven by testosterone, and gynecomastia is more
commonly associated with an imbalance between estrogen and testosterone levels. Therefore,
gynecomastia is not typically seen in prostate cancer.
Incorrect Options:
Option A - Klinefelter syndrome: Klinefelter syndrome is a
genetic disorder in which males are born with an extra X chromosome (XXY instead of XY). This condit
ion often leads to lower testosterone production and an imbalance between estrogen and testosterone,
resulting in gynecomastia. Therefore, gynecomastia is commonly seen in individuals with Klinefelter sy
ndrome.
Option B - Liver cirrhosis: Liver cirrhosis refers to advanced liver disease characterized by scarring and
impaired liver function. In cirrhosis, the liver's ability to metabolize hormones, including estrogen, is aff
ected. This can lead to an imbalance between estrogen and testosterone, resulting in gynecomastia. T
hus, gynecomastia is frequently observed in individuals with liver cirrhosis.
Option C - Hypothyroidism: Hypothyroidism might be associated with gynecomastia

Solution for Question 26:


Correct Option C - DIEP based on deep inferior epigastric artery:

Page 41

221
• The DIEP flap procedure, which stands for Deep Inferior Epigastric Perforator, involves the transfer of
skin and fat from the lower abdomen to reconstruct the breast.
• The blood supply for the flap is derived solely from the deep inferior epigastric artery, preserving the
rectus abdominis muscle.
• This option accurately describes the flap shown in the image.

Incorrect Options:
Option A - Pedicled TRAM based on superior epigastric artery: The pedicled TRAM flap procedure invo
lves using a portion of the rectus abdominis muscle along with the overlying skin and fat to reconstruct
the breast. The flap is typically supplied by the superior epigastric artery, which is a
branch of the internal mammary artery. However, the image actually shows a DIEP flap, not a
pedicled TRAM flap.
Option B - Free TRAM based on superior and inferior epigastric artery: The image does not depict a
free TRAM flap, image actually shows a DIEP flap.

Page 42

222
Option D - Lower Abdominal Fat based inferior epigastric vessels: The image does not depict lower ab
dominal fat based on inferior epigastric vessels, image actually shows a DIEP flap.

Solution for Question 27:


Correct Option C - Duodenum:
• The duodenum is the most common site of gastrinoma in MEN 1 syndrome.
• Gastrinomas arising in the duodenum can cause a condition called Zollinger-Ellison syndrome,
characterized by excessive gastric acid production and the development of peptic ulcers.
• MEN 1 syndrome, also known as Multiple Endocrine Neoplasia type 1, is a hereditary condition
characterized by the development of tumors in multiple endocrine glands.
• Gastrinomas are one type of tumor associated with MEN 1 syndrome.
• Gastrinomas are neuroendocrine tumors that produce excessive amounts of gastrin, a hormone that
stimulates the secretion of gastric acid.
Incorrect Options:
Option A - Jejunum: While gastrinomas can occur in the small intestine, the most common site of gastri
noma in MEN 1 syndrome is the duodenum, not the jejunum.
Option B - Ileum: Although gastrinomas can occasionally occur in the ileum, it is not the most common
site in MEN 1 syndrome. The duodenum is the primary site of gastrinoma in this condition.
Option D - Stomach: While gastrinomas can rarely occur in the stomach, they are much more commonl
y found in the duodenum in the context of MEN 1 syndrome.

Solution for Question 28:


Correct Option D - Most of the retrosternal goiters can be removed by a neck incision:
• Retrosternal goiter refers to an enlarged thyroid gland that extends into the chest, behind the sternum
(breastbone).
• A neck incision (cervical approach) is the most common method of removing retrosternal goiters.
• This approach provides access to the lower part of the goiter and allows for its safe removal without a
sternal incision.
Incorrect Options:
Option A - Operated in all patients regardless of symptoms: The decision to operate on a retrosternal g
oiter is not solely based on its presence but rather on the presence of symptoms or complications caus
ed by the goiter. Asymptomatic retrosternal goiters that are not causing any problems may not necessa
rily require surgical intervention.
Option B - Sternal incision is always required to operate: While some large and complex retrosternal go
iters may require a sternal incision (median sternotomy) to gain access and remove the gland, it is not
always necessary. In most cases, a
neck incision (cervical approach) is sufficient to remove the retrosternal goiter.

Page 43

223
Option C - It receives blood supply from the thoracodorsal artery: The blood supply to the retrosternal g
oiter primarily comes from the thyroid arteries, which are branches of the external carotid artery and its
branches. The thoracodorsal artery is not directly involved in the blood supply to the thyroid gland.

Solution for Question 29:


Correct Option A - Addisonian crisis:
• The most likely cause of the patient's symptoms of lethargy and hypotension on postoperative day 3 is
an Addisonian crisis (also known as an adrenal crisis).
• An Addisonian crisis refers to acute adrenal insufficiency that occurs due to the sudden withdrawal or
absence of glucocorticoids and mineralocorticoids, essential hormones produced by the adrenal
glands.
• In this case, the bilateral adrenalectomy has removed both adrenal glands, leading to a complete
deficiency of adrenal hormones.
• The symptoms of an Addisonian crisis include profound fatigue, hypotension, dizziness, weakness,
nausea, vomiting, abdominal pain, and electrolyte imbalances.
Incorrect Options:
Option B - SIADH: SIADH(syndrome of inappropriate antidiuretic hormone secretion) typically presents
with hyponatremia and fluid overload, not hypotension.
Option C - Diabetes insipidus: Diabetes insipidus is characterized by excessive thirst and urination due
to either inadequate production or response to antidiuretic hormone (ADH), which does not typically ca
use hypotension.
Option D - Cerebral salt-wasting disease: Cerebral salt-wasting disease is a condition where there is e
xcessive renal sodium loss leading to hyponatremia and dehydration, but it does not typically cause hy
potension.

Solution for Question 30:


Correct Option A - Lymphedema:
Lymphedema is the most probable cause of long-standing non-pitting edema in the legs.
• Lymphedema is a condition characterized by swelling due to the accumulation of lymph fluid. It occurs
when there is damage or obstruction to the lymphatic system, leading to impaired drainage of lymph
fluid from tissues.
• Clinical Presentation: Patients with lymphedema typically present with chronic, progressive swelling in
one or more limbs, such as the legs. The edema in lymphedema is usually non-pitting, meaning that
pressure applied to the swollen area does not leave an indentation.
• Etiology: Lymphedema can be primary or secondary. Primary lymphedema is due to developmental
abnormalities or genetic factors affecting the lymphatic system. Secondary lymphedema is more
common and often results from surgery, radiation therapy, infection, trauma, or tumors that obstruct
lymphatic flow.

Page 44

224
• Complications: Chronic lymphedema can lead to various complications, including discomfort, pain,
recurrent infections (bacterial or fungal), cellulitis, lymphangiosarcoma (a rare form of cancer), and skin
changes such as thickening and discoloration.
• Diagnosis: Lymphedema is primarily diagnosed clinically, based on history and physical examination.
However, in cases of atypical presentation or severe lymphedema with complications, further
investigation, such as lymphangiography, may be required to assess lymphatic function and identify
underlying causes.
Incorrect Options:
Option B - Arterial Disorders:
• Arterial disorders typically present with symptoms of arterial insufficiency, such as intermittent
claudication, rest pain, cool extremities, diminished pulses, and skin changes like pallor or mottling.
Edema is not a prominent feature of arterial disorders.
• Conditions such as peripheral arterial disease (PAD) can cause arterial insufficiency but are not
associated with non-pitting edema.
Option C - Venous Disorders:
• Venous disorders, such as chronic venous insufficiency (CVI) or deep vein thrombosis (DVT), can
lead to edema in the lower extremities. However, venous edema typically exhibits pitting upon
pressure, whereas lymphedema presents with non-pitting edema.
• Other signs of venous disorders include varicose veins, venous ulcers, and hemosiderin staining.
Option D - Coronary Heart Disease:
• Coronary heart disease primarily affects the heart and blood vessels supplying the heart muscle,
leading to symptoms such as chest pain (angina), shortness of breath, and heart attacks. It is not
directly related to the development of lower extremity edema.
• Edema in the legs is not a characteristic feature of coronary heart disease and is more indicative of
conditions affecting the lymphatic or venous systems.

Solution for Question 31:


Correct Option D - Half fall in 10 min:
• The Miami criteria for intraoperative parathyroid hormone (PTH) measurement is used to assess the
success of parathyroid surgery and predict the long-term outcome.
• Half fall in 10 min: The Miami criteria state that there should be a "half fall" in PTH levels within 10
minutes after the excision of a parathyroid adenoma. This is considered an indicator of successful
surgery and predicts the long-term outcome of normocalcemia.
Incorrect Options:
Option A - Quarter fall in 5 min: The Miami criteria do not involve a "quarter fall" in PTH levels within a
specific time frame.
Option B - Quarter fall in 10 min: This option is incorrect for the same reason as option A. The Miami cr
iteria do not utilize a "quarter fall" in PTH levels within a specific time frame.
Option C - Half fall in 5 min: The Miami criteria require a
"half fall" in PTH levels, but the time frame for measurement is not specified as 5 minutes.

Page 45

225
Solution for Question 32:
Correct Option A - Incision and drainage:
• The affected breast appears engorged.
• Skin erythema is evident in the provided image.
• Breast abscess commonly affects lactating females, primarily those experiencing breastfeeding
difficulties.
• Staphylococcus aureus is the predominant organism causing breast abscesses.
• Pain and tenderness typically accompany pus accumulation in the body.
• Clinical Presentation: Symptoms include pain, tenderness, engorgement, and skin redness in the
affected breast. Fever accompanied by chills and rigor may also manifest.
• Symptoms include pain, tenderness, engorgement, and skin redness in the affected breast.
• Fever accompanied by chills and rigor may also manifest.
• Treatment: Treatment involves incision and drainage coupled with antibiotic therapy. It is necessary to
perform incision and drainage with failure of antibiotic aspirations. Cloxacillin or Dicloxacillin is the
preferred first-line antibiotic, administered for 10-14 days.
• Treatment involves incision and drainage coupled with antibiotic therapy. It is necessary to perform
incision and drainage with failure of antibiotic aspirations.
• Cloxacillin or Dicloxacillin is the preferred first-line antibiotic, administered for 10-14 days.
• Symptoms include pain, tenderness, engorgement, and skin redness in the affected breast.
• Fever accompanied by chills and rigor may also manifest.
• Treatment involves incision and drainage coupled with antibiotic therapy. It is necessary to perform
incision and drainage with failure of antibiotic aspirations.
• Cloxacillin or Dicloxacillin is the preferred first-line antibiotic, administered for 10-14 days.
Incorrect Options:
Option B, C, D: Broad spectrum antibiotics only, Breast conservation surgery, and Punch biopsy are no
t the ideal treatment modalities for this condition. With the fluid collection, erythema, warmth, mild axilla
ry lymphadenopathy, incision and drainage are the management plan for this patient.

Solution for Question 33:


Correct Option B - Reassurance and follow-up:
• Since the patient is a young woman presenting with a mobile and firm breast lump, and FNAC results
do not indicate malignancy, the most appropriate next step in management is reassurance and regular
follow-up. This approach is based on the understanding that the most common cause of breast lumps
in this age group is fibroadenoma, which is benign in nature.
• Aberrations in Normal Development and Involution: The most common types of ANDI in different age
groups: 15-25 years- Fibroadenoma 25-40 years- Fibro-adenosis > 40 years- Fibro-adenosis followed

Page 46

226
by duct ectasia.
• The most common types of ANDI in different age groups: 15-25 years- Fibroadenoma 25-40 years-
Fibro-adenosis > 40 years- Fibro-adenosis followed by duct ectasia.
• 15-25 years- Fibroadenoma
• 25-40 years- Fibro-adenosis
• > 40 years- Fibro-adenosis followed by duct ectasia.
• The most common types of ANDI in different age groups: 15-25 years- Fibroadenoma 25-40 years-
Fibro-adenosis > 40 years- Fibro-adenosis followed by duct ectasia.
• 15-25 years- Fibroadenoma
• 25-40 years- Fibro-adenosis
• > 40 years- Fibro-adenosis followed by duct ectasia.
• 15-25 years- Fibroadenoma
• 25-40 years- Fibro-adenosis
• > 40 years- Fibro-adenosis followed by duct ectasia.
Incorrect Options : Option A , C and D
- Fibroadenomas are benign, non-cancerous lesions. So it doesn't need a
mammography, surgical excision, or tamoxifen.

Solution for Question 34:


Correct Option A - Luminal A:
• Most common type is Liminal-A/Lum-A.
• Lum-A has the best prognosis.
• HER-2-NEU type has the worst prognosis.
• Luminal A subtype is the subtype with good prognosis and is only 50 % invasive. Luminal-A subtype is
defined as ER-positive and/or PR-positive tumors with negative HER2 and low Ki67 (proliferating cell
nuclear antigen) index by immunohistochemistry.
Incorrect Options:
Option B - Luminal B: Luminal B breast cancer is a
subtype of breast cancer named for the genes the cancer cells express. It's also called group 2
or HR+/HER2+ breast cancer. About 10 percent of breast cancer cases are luminal B.
Option C - ER negative : Estrogen receptor (ER)-negative breast cancers are tumors with poor progno
sis and fewer cancer prevention and treatment strategies.
Option D - Basal like: Basal-like breast cancer demonstrates typical metastasis. It often spreads to the
brain and lungs .

Solution for Question 35:

Page 47

227
Correct Option B - Closed:
• The image of Romovac drain is a type of closed drain system commonly used after mastectomy.
• It consists of a plastic drain tube connected to a collection bulb or container.
• The drain tube is inserted into the surgical site to facilitate the removal of fluid or blood.
• The collection bulb is designed to create negative pressure or suction, which helps in the drainage
process.
• The closed system helps prevent air from entering the drain, reducing the risk of infection and
maintaining a sterile environment.
Incorrect Options:
Option A - Open: An open drain system would not have a
collection bulb or container attached and typically relies on gravity for drainage.

Option C - Semi-open: A semi-open drain system typically has a


small opening that allows for some drainage but may not provide a controlled suction mechanism like a
closed drain.
Option D - Semi-closed: This term is not commonly used to describe a
specific type of drain system. It may refer to a drain that has some elements of both open and closed s
ystems, but without further context, it is difficult to provide a specific explanation for this option.

Solution for Question 36:


Correct Option (A): Adjuvant Radiotherapy
• Adjuvant radiotherapy is the appropriate management option following breast conservation surgery for
high nuclear-grade DCIS with necrosis and margin clearance.
• DCIS (Ductal Carcinoma In-situ) is a pre-invasive form of breast cancer where abnormal cells are
found in the lining of a breast duct but have not spread outside the duct to invade surrounding breast
tissue.

Page 48

228
• High nuclear grade DCIS with necrosis indicates a more aggressive form of the disease.
• Margin clearance of 10mm suggests that the surgical margins around the tumor are clear of cancer
cells, reducing the risk of local recurrence.
• Adjuvant radiotherapy helps to further reduce the risk of recurrence by targeting any remaining cancer
cells in the breast tissue.
• It is important to note that adjuvant radiotherapy following breast conservation surgery has been
shown to significantly decrease the risk of ipsilateral (same side) breast cancer recurrence.
Incorrect Options:
B. Adjuvant Chemotherapy:
• Chemotherapy is not typically indicated for DCIS because it is a non-invasive form of breast cancer.
• Chemotherapy is usually reserved for invasive breast cancer where cancer cells have spread beyond
the ducts into the surrounding breast tissue or to other parts of the body.
C. Follow up 6 monthly for 2 years and then yearly follow up:
• While follow-up appointments are important for monitoring patients after treatment for breast cancer,
in this case, adjuvant radiotherapy is necessary to reduce the risk of recurrence.
• Follow-up appointments alone would not address the risk of recurrence adequately.
D. Trastuzumab Therapy:
• Trastuzumab is a targeted therapy used in the treatment of HER2-positive breast cancer, which is a
subtype of invasive breast cancer.
• DCIS is typically HER2-negative, and targeted therapies like trastuzumab are not indicated for DCIS.
• Trastuzumab therapy is usually reserved for cases of invasive breast cancer where HER2
overexpression is present.

Solution for Question 37:


Correct Option B - 2+:
• Fluorescence in-situ hybridization (FISH) is required for the interpretation of HER2/NEU 2+.
• HER2/NEU scoring system typically includes 0, 1+, 2+, and 3+.
• When the immunohistochemistry (IHC) staining for HER2/NEU yields a score of 2+, it falls into the
equivocal range, meaning it's neither definitively negative nor positive.
• In such cases, FISH is performed to further analyze the HER2/NEU gene amplification status.
• FISH determines whether there is amplification of the HER2/NEU gene, which would confirm the
presence of HER2/NEU overexpression and thus render the test positive.
• Therefore, FISH is essential for accurately determining the status of HER2/NEU in cases where the
IHC result is 2+.
Incorrect Options:
Option A - 1+:

Page 49

229
• For HER2/NEU interpretation, 1+ IHC score usually indicates low levels of HER2/NEU expression,
which is considered negative.
• In such cases, FISH is not typically required because the IHC result itself indicates negativity.
Option C - 3+:
• A score of 3+ in HER2/NEU IHC indicates strong overexpression of HER2/NEU protein, which is
considered positive.
• In this scenario, FISH is not usually necessary as the IHC result alone is indicative of HER2/NEU
positivity, and subsequent treatment with trastuzumab may be initiated.
Option D - Option All of the above:
• This option suggests that FISH is required for all interpretations of HER2/NEU, which is incorrect.
• FISH is only necessary for cases with equivocal (2+) IHC results to confirm HER2/NEU gene
amplification and determine positivity.
• For IHC scores of 1+ and 3+, FISH is not routinely performed as the results are typically clear-cut
negative or positive, respectively.

Solution for Question 38:


Correct Option D - 1, 3, 4:
1. Due to increasing mammography, there occurs over diagnosis of breast carcinoma: This statement i
s true. Mammography is a screening tool used for the early detection of breast cancer. While it has bee
n effective in detecting early-stage breast cancer and reducing mortality, it can also lead to overdiagno
sis. Overdiagnosis refers to the detection of cancers that would not have caused symptoms or harm du
ring a person's lifetime.
3. Oral cancer screening is done by visual inspection: This statement is true. Visual inspection of the or
al cavity is a common method used for screening and detecting oral cancers. Dentists and healthcare p
rofessionals often perform oral cancer screenings by visually examining the oral tissues for abnormaliti
es, such as ulcers, lesions, or discoloration.
4. Cervix cancer screening is done by a Pap smear: This statement is true, with a
minor correction. Cervix cancer screening is commonly done by a Pap smear (not a "gap smear"). A
Pap smear, also known as a
Pap test, involves collecting cells from the cervix and examining them under a
microscope to detect any abnormal changes that may indicate precancerous or cancerous conditions.
Thus, Option D is correct.
Incorrect Options:
Option A - 2, 3, 4 is incorrect because it includes statement 2, which is false.
Option B - 1, 2, 3, 4 is incorrect because it includes all the statements, but statement 2 is a
false statement.
Option C - 4 only is incorrect because it excludes statements 1 and 3, which are true.
2. Colon cancer screening is done by digital rectal examination: This statement is false. As for colon ca
ncer screening, it is done by colonoscopy.

Page 50

230
Solution for Question 39:
Correct Option D - Brachioradialis:
• In cases of parathyroid surgery, particularly in hyperparathyroidism caused by parathyroid
hyperplasia, if a gland needs to be removed, it's common practice to transplant a portion of the
remaining gland to ensure adequate parathyroid function.
• The parathyroid tissue is typically divided into small pieces and transplanted into the brachioradialis
muscle of the patient's non-dominant arm.
• This transplantation allows the parathyroid tissue to continue its function of regulating calcium levels
in the body.
Incorrect Options:
Option A, B
and C: The triceps, biceps, and sartorius are not commonly used for parathyroid gland transplantation.

Solution for Question 40:


Correct Option D - Hashimoto’s thyroiditis:
• Based on the histology picture showing lymphocytic infiltration and follicular cell changes with a
history of painless progression, the most likely diagnosis is Hashimoto's thyroiditis.
• The hallmark of Hashimoto's thyroiditis includes Hurthle cells, characterized by abundant
mitochondria giving a pink color, although they are not specific and can occur in other thyroid disorders.

Incorrect Options:
Option A - Medullary carcinoma: Medullary carcinoma of the thyroid is a
rare type of thyroid cancer that arises from the parafollicular or C cells of the thyroid gland. Characterist

Page 51

231
ic features of medullary thyroid carcinoma include elevated calcitonin levels, a
history of diarrhea, the presence of amyloid stroma, and a positive family history of pheochromocytoma
or hyperparathyroidism, indicating an association with MEN 2A and MEN 2B.

Option B - Non-Hodgkin's lymphoma: Non-Hodgkin's lymphoma can involve the thyroid gland, but the h
istological features would typically show malignant lymphoid cells infiltrating the thyroid tissue. The hist
ology picture provided does not show the characteristic features of lymphoma.

Option C - Graves' disease: Graves' disease is another autoimmune disorder that affects the thyroid gl
and. It is characterized by hyperthyroidism, goiter (enlargement of the thyroid gland), and the presence
of thyroid-stimulating immunoglobulins (TSI) that stimulate the thyroid gland to produce excess thyroid
hormones. In Graves' disease microscopy, pseudopapillae resembling true papillae seen in papillary c
arcinoma, distinguished by a
fibrovascular core, with colloid scalloping indicating colloid consumption are seen.

Page 52

232
Solution for Question 41:
Correct Option B: Microdochectomy
• It refers to the surgical removal of a single lactiferous duct or a small segment of the duct in the
breast.
• This procedure is typically performed when there is a persistent nipple discharge or abnormality in a
specific duct in duct ectasia and duct papilloma.
Incorrect Options:
Option A: Macrodochectomy- Macrodochectomy refers to the surgical removal of a
major lactiferous duct in the breast. It involves excising a larger duct within the breast tissue.

Page 53

233
Option C: Cone excision- Cone excision, also known as conization or cone biopsy, is a
procedure performed on the cervix to remove a cone-shaped portion of abnormal or precancerous tiss
ue. It is not directly related to the treatment of nipple discharge or lactiferous ducts.
Option D: Wide excision of the lactiferous duct- Wide excision of the lactiferous duct is not a
specific procedure or term used in breast surgery.

Solution for Question 42:


Correct Option A - Vulva:
• Extra mammary Paget's disease refers to the presence of Paget's disease of the skin occurring
outside the mammary (breast) region.
• It commonly affects the vulva, which is the external female genitalia.
• Extra mammary Paget's disease of the vulva is a rare condition characterized by the presence of
Paget cells in the skin of the vulva, causing symptoms such as itching, redness, and skin changes.
Incorrect Options
Option B , C
& D: Extra mammary Paget's disease is not commonly seen in the vagina, uterus and cervix.

Solution for Question 43:


Correct Option B - Endometrial cancer:
• Tamoxifen is a selective estrogen receptor modulator (SERM) commonly used in the treatment of
hormone receptor-positive breast cancer. While tamoxifen provides benefits in reducing the risk of
breast cancer recurrence and mortality, it is associated with certain side effects, including an increased
risk of endometrial cancer.
• Tamoxifen has both estrogen agonist and antagonist effects depending on the target tissue. In breast
tissue, it acts as an antagonist, blocking the estrogen receptors and inhibiting the growth-promoting
effects of estrogen. However, in the endometrium (lining of the uterus), tamoxifen acts as an estrogen
agonist, stimulating the growth of the endometrial cells.
• Prolonged tamoxifen use can lead to endometrial hyperplasia, a condition characterized by excessive
growth of the endometrium. Over time, this can progress to endometrial cancer in some cases. The risk
of endometrial cancer is higher in postmenopausal women, especially those receiving tamoxifen for
more extended periods.
Tamoxifen is a selective estrogen receptor modulator (SERM) commonly used in the treatment of horm
one receptor-positive breast cancer. While tamoxifen provides benefits in reducing the risk of breast ca
ncer recurrence and mortality, it is associated with certain side effects, including an increased risk of e
ndometrial cancer.
Tamoxifen has both estrogen agonist and antagonist effects depending on the target tissue. In breast ti
ssue, it acts as an antagonist, blocking the estrogen receptors and inhibiting the growth-promoting effe
cts of estrogen. However, in the endometrium (lining of the uterus), tamoxifen acts as an estrogen ago
nist, stimulating the growth of the endometrial cells.

Page 54

234
Prolonged tamoxifen use can lead to endometrial hyperplasia, a condition characterized by excessive
growth of the endometrium. Over time, this can progress to endometrial cancer in some cases. The ris
k of endometrial cancer is higher in postmenopausal women, especially those receiving tamoxifen for
more extended periods.
Incorrect Options:
Option A - Ovarian cancer: Tamoxifen use is not associated with an increased risk of ovarian cancer. I
n fact, tamoxifen may have a
protective effect against ovarian cancer, particularly in premenopausal women.
Option C - Breast cancer on the other side: Tamoxifen use is actually beneficial in reducing the risk of
developing contralateral (on the other side) breast cancer. It is used as an adjuvant therapy to decreas
e the risk of breast cancer recurrence in the same or contralateral breast.
Option D - Cervical cancer: Tamoxifen use is not directly linked to an increased risk of cervical cancer.
However, regular cervical cancer screening, including Pap smears, should still be maintained as recom
mended for all women.

Solution for Question 44:


Correct Option C: Hormone receptor staining
• The Van Nuys grading system for DCIS does not consider hormone receptor staining as one of the
parameters to assess the tumor.
• Hormone receptor status, including estrogen receptor (ER) and progesterone receptor (PR) staining,
is relevant in determining the treatment options for DCIS.
Scoring system for University of Southern California/Van Nuys Prognostic Index
Incorrect Options: A, B & D - These are included in the grading system.

Solution for Question 45:


Correct Option C - Tamoxifen:
• Estrogen has positive effects on breast cancer cells.
• In pre-menopausal patients, the drug of choice (DOC) for hormonal therapy is Tamoxifen.
• Tamoxifen works as a selective estrogen receptor modulator (SERM).
• It has potent antagonistic action on breast cancer cells and blood vessels.
• The recommended dose is 10 mg twice daily for 5 years.
• Tamoxifen also has a partial agonistic effect on certain tissues such as the pituitary, bone, uterus, and
liver.
• It is avoided in post-menopausal patients due to its increased risk of endometrial cancer.
• Tamoxifen helps prevent estrogen from binding to estrogen receptors on breast cancer cells, thereby
inhibiting their growth.

Page 55

235
Incorrect Options:
Option A - Lapatinib:
• Lapatinib is a tyrosine kinase inhibitor used in the treatment of HER2-positive breast cancer, not
estrogen-dependent breast cancer.
• It works by targeting the HER2 protein to inhibit cancer cell growth.
Option B - Sunitinib:
• Sunitinib is a multi-targeted receptor tyrosine kinase inhibitor primarily used in the treatment of
advanced renal cell carcinoma and gastrointestinal stromal tumors.
• It is not indicated for the treatment of estrogen-dependent breast cancer.
Option D - Adriamycin:
• Adriamycin (doxorubicin) is a chemotherapy drug used to treat various types of cancer, including
breast cancer.
• However, it is not specifically indicated for estrogen-dependent breast cancer but rather used in
combination with other chemotherapy agents for breast cancer treatment.
• Adriamycin works by intercalating DNA strands, inhibiting topoisomerase II activity, and generating
free radicals, resulting in cancer cell death.

Solution for Question 46:


Correct Option C - 1, 3 & 4:
In a patient presenting with a
breast mass and palpable axillary lymph nodes, the following investigations would be appropriate:
1. Bilateral Mammography: Mammography is a crucial initial investigation in suspected cases of breast
cancer. It helps visualize any abnormal masses or calcifications within the breast tissue. In this patient,
it is important to perform bilateral mammography to assess the entire breast tissue for any additional le
sions.
3. Biopsy of the breast mass: Given the presence of an irregular mass in the upper outer quadrant of th
e right breast, a biopsy of this mass is essential for a definitive diagnosis. Tissue sampling through biop
sy allows for histopathological examination, which is the gold standard for diagnosing breast cancer.
4. USG-guided biopsy of the lymph node: The palpable lymph node in the right axilla raises concern for
potential metastasis from breast cancer. Performing an ultrasound-guided biopsy of the lymph node h
elps confirm the presence of metastatic disease and aids in staging the cancer.
Incorrect statement:
2. PET Scan: While PET (Positron Emission Tomography) scan is a valuable tool in cancer staging an
d detecting distant metastases, it is not typically performed as an initial investigation in patients with su
spected breast cancer. Its role is more prominent in later stages for staging purposes rather than initial
diagnosis.
Incorrect Options - A, B & D: These are incorrect.

Page 56

236
Solution for Question 47:
Correct Option: D) Cervical incision
• Cervical incision is the most common surgical approach for retrosternal goiter because the vascular
supply of the thyroid primarily comes from vessels in the cervical region.
• Thyroidectomy via cervical incision allows direct access to the retrosternal goiter while minimizing the
risk of damaging important structures in the mediastinum.
Incorrect Options:
A) Mediastinal incision
• Mediastinal incision is not commonly preferred for retrosternal goiter because it involves accessing
the thoracic cavity, which carries higher risks compared to cervical incision.
• Direct access to the thyroid gland can be challenging via a mediastinal incision, and there's a higher
risk of complications due to the proximity to vital structures in the chest.
B) Subxiphoid incision
• Subxiphoid incision is not typically used for retrosternal goiter as it involves accessing the thyroid
gland through an incision below the sternum.
• This approach is more commonly utilized for surgeries involving the heart or other structures in the
mediastinum, not specifically for thyroidectomy.
C) Transaxillary incision
• Transaxillary incision involves accessing the thyroid gland through an incision in the axilla (armpit).
• While this approach may be suitable for certain thyroid surgeries, it is not ideal for retrosternal goiter
because it does not provide direct access to the thyroid gland in the cervical region, where the main
vascular supply originates.

Solution for Question 48:


Correct Option D - Cervical incision:
• Cervical incision is the most common surgical approach for retrosternal goiter because the vascular
supply of the thyroid primarily comes from vessels in the cervical region.
• Thyroidectomy via cervical incision allows direct access to the retrosternal goiter while minimizing the
risk of damaging important structures in the mediastinum.
Cervical incision is the most common surgical approach for retrosternal goiter because the vascular su
pply of the thyroid primarily comes from vessels in the cervical region.
Thyroidectomy via cervical incision allows direct access to the retrosternal goiter while minimizing the ri
sk of damaging important structures in the mediastinum.
Incorrect Options:
Option A - Mediastinal incision:
• Mediastinal incision is not commonly preferred for retrosternal goiter because it involves accessing
the thoracic cavity, which carries higher risks compared to cervical incision.

Page 57

237
• Direct access to the thyroid gland can be challenging via a mediastinal incision, and there's a higher
risk of complications due to the proximity to vital structures in the chest.
Mediastinal incision is not commonly preferred for retrosternal goiter because it involves accessing the
thoracic cavity, which carries higher risks compared to cervical incision.
Direct access to the thyroid gland can be challenging via a mediastinal incision, and there's a
higher risk of complications due to the proximity to vital structures in the chest.
Option B - Subxiphoid incision:
• Subxiphoid incision is not typically used for retrosternal goiter as it involves accessing the thyroid
gland through an incision below the sternum.
• This approach is more commonly utilized for surgeries involving the heart or other structures in the
mediastinum, not specifically for thyroidectomy.
Subxiphoid incision is not typically used for retrosternal goiter as it involves accessing the thyroid gland
through an incision below the sternum.
This approach is more commonly utilized for surgeries involving the heart or other structures in the me
diastinum, not specifically for thyroidectomy.
Option C - Transaxillary incision:
• Transaxillary incision involves accessing the thyroid gland through an incision in the axilla (armpit).
• While this approach may be suitable for certain thyroid surgeries, it is not ideal for retrosternal goiter
because it does not provide direct access to the thyroid gland in the cervical region, where the main
vascular supply originates.
Transaxillary incision involves accessing the thyroid gland through an incision in the axilla (armpit).
While this approach may be suitable for certain thyroid surgeries, it is not ideal for retrosternal goiter be
cause it does not provide direct access to the thyroid gland in the cervical region, where the main vasc
ular supply originates.

Solution for Question 49:


Correct Option C - Incision and drainage + Antibiotics :
• In some cases, the first attempt at aspiration may not successfully collect any fluid due to various
factors such as the location or depth of the abscess, the presence of loculations, or the consistency of
the fluid. A second attempt should be made with an incision and drainage + antibiotics.
Incorrect Options:
Option A - Incision and Drainage:
• If the second attempt at aspiration is also unsuccessful in obtaining any aspirate, alternative
management options such as incision and drainage or other interventions may be considered.
Option B - Continuing antibiotics only:
• Continuing antibiotics alone may not be sufficient to treat the abscess and achieve resolution
adequately.
Option D - Stop antibiotics as no aspirate was collected:

Page 58

238
• Stopping antibiotics without draining the abscess can lead to persistent infection and potential
complications.

Solution for Question 50:


Correct Option C - Breast abscess:
• The clinical presentation of a lactating mother with pain in the left breast, fever, localized erythema,
warmth, and point tenderness, along with a positive fluctuation test, indicates the presence of a breast
abscess.
• Breast abscess commonly occurs in lactating females, especially primiparous women, and is often a
consequence of improper breastfeeding technique, leading to milk stasis and subsequent infection.
• Staphylococcus aureus is the most common causative organism responsible for breast abscesses.
• The presence of mild axillary lymphadenopathy on the affected side further supports the diagnosis.
• Treatment involves incision and drainage of the abscess along with appropriate antibiotic therapy.
First-line agents are typically Cloxacillin or Dicloxacillin administered for 10-14 days.
Incorrect Options:
Option A - Lactational mastitis: In Lactational mastitis cracked nipple is a predisposing factor. It present
s with engorgement of lactational ducts, and the presence of clumps of milk. It does not cause fluctuan
ce.
Option B - Breast carcinoma: In the case of carcinoma of the breast we see a
fixed lump, ulcerated nipple and peu d
orange. The swelling is not fluctuant. Hence this choice is incorrect.
Option D - Fibroadenoma: Fibroadenoma is a benign, painless well defined firm mobile mass which do
es not show fluctuance. Hence this is the incorrect choice.

Solution for Question 51:


Correct Option D - T4b:
• The stage shown in the image is the T4b stage of breast carcinoma. It is characterised by extension
into the skin, peau d' orange, ulceration, and satellite lesions, and no puckering/ dimpling is seen at this
stage.
Incorrect Options:
Option A - T2c: In T2c the tumour is more than 2 cm but not exceeding 5cms. It does not show involve
ment of the chest wall. There is no involvement of the chest wall or skin seen in this stage. The image
shows skin involvement hence this choice is incorrect.
Option B - T4a: In the T4a stage we see an extension into the chest wall but no extension into the skin.
Hence this stage is not the T4a stage since peau d’ orange is seen in the image.
Option C - T2a: There is no involvement of skin seen at this stage as seen in the image. Hence, this is
an incorrect choice.

Page 59

239
Solution for Question 52:
Correct Option A - Ultrasound of the neck:
• Ultrasound of the neck is the initial investigation of choice for evaluating thyroid swellings.
• It provides valuable information about the size, characteristics, and composition of the swelling.
• Ultrasound can help differentiate between benign and malignant nodules based on various features
such as microcalcifications, increased vascularity, nodal involvement, and capsular breach.
• It serves as the gold standard investigation for assessing thyroid nodules.
• Ultrasound can guide further management, such as whether fine-needle aspiration cytology (FNAC) is
necessary.
Incorrect Options:
Option B - FNAC:
• While FNAC is a crucial diagnostic tool for thyroid nodules, it is not typically the first investigation
advised.
• FNAC is recommended if a nodule does not fulfill a fully benign classification on ultrasound or if
malignancy is suspected based on ultrasound findings.
• It helps distinguish between benign and malignant nodules and can reliably identify papillary thyroid
cancer.
• FNAC is highly specific and sensitive, especially when performed under ultrasound guidance.
Option C - Thyroid function tests:
• Thyroid function tests (TFTs) are essential but are not typically the first investigation for a large thyroid
swelling.
• TFTs are performed to assess thyroid hormone levels, especially if hyperthyroidism is suspected
clinically.
• They help in diagnosing conditions like toxic adenoma, toxic multinodular goiter, or thyroiditis.
• TFTs guide further management and treatment options.
Option D - Radioiodine uptake:
• Radioiodine uptake is not typically performed as an initial investigation for a thyroid swelling.
• It is used in the evaluation of hyperthyroidism to determine the cause and localization of excessive
thyroid hormone production.
• Radioiodine uptake is particularly useful in cases of toxic adenoma or toxic multinodular goiter to
identify areas of hyperfunction.
• This investigation is not typically indicated for the initial evaluation of a large thyroid swelling but may
be considered based on clinical suspicion and other findings.

Solution for Question 53:

Page 60

240
Correct Option A - Papillary Carcinoma:
• The radiation risk and characteristic features, such as optically clear nuclei (Orphan Annie eye nuclei),
suggest a diagnosis of Papillary thyroid Carcinoma.
• Papillary carcinoma is the most common type of thyroid malignancy.
• It is often found in iodine-sufficient areas.
• Low-dose radiation exposure during childhood is a significant risk factor.
• Other risk factors include thyroglossal cyst and Hashimoto's thyroiditis.
• Pathologically, papillary projections can be observed along with characteristic features such as
optically clear nuclei (Orphan Annie eye nuclei) , pseudo inclusion bodies, and dystrophic calcification
(Psammoma bodies).
• Psammoma bodies are also found in conditions like papillary carcinoma of the kidney (RCC), serous
cystadenoma of the ovary, and meningioma.
Incorrect Options:
Option B, C & D - Follicular carcinoma, Medullary carcinoma & Anaplastic carcinoma:
• Unlike papillary carcinoma, low-dose radiation exposure during childhood is not a significant risk
factor for any of these carcinomas.
• Pathologically, these carcinomas do not typically exhibit the characteristic features seen in papillary
carcinoma, such as orphan Annie eye nuclei or Psammoma bodies.

Solution for Question 54:


Correct Option B - Peau d’orange:

• Peau d'orange refers to the appearance of the breast skin resembling the texture of an orange peel.
• It occurs due to the buildup of fluid in the breast tissue and is often caused by blocked lymphatic
channels.

Page 61

241
• Peau d'orange can be a significant sign of inflammatory breast cancer, which is an aggressive form of
breast cancer.
• It is characterized by rapid changes in the breast, including redness, warmth, swelling, and the peau
d'orange appearance.
• Therefore, it is considered a more conspicuous sign of breast cancer compared to nipple retraction or
puckering.
Incorrect Options:
Option A - Nipple retraction:

• Nipple retraction refers to a change in the position or appearance of the nipple, causing it to turn
inward or flatten.
• While nipple retraction can be a potential sign of breast cancer, it is not the most conspicuous sign.
• Other conditions, such as benign breast changes or infections, can also cause nipple retraction.
Therefore, it is important to consider other symptoms and undergo further evaluation if nipple retraction
is observed.
Option C - Puckering:

Page 62

242
• Puckering or dimpling of the breast refers to an area of skin that appears pulled inward or has an
uneven texture. It can be caused by the presence of a tumor that tethers the underlying tissue, leading
to changes in the skin's appearance.
• Puckering can be a sign of breast cancer, but it is not as noticeable or specific as peau d'orange in
indicating the presence of breast cancer.
Option D - Both nipple retraction and puckering:
• While nipple retraction and puckering can be signs of breast cancer, they are not considered the most
conspicuous sign.

Solution for Question 55:


Correct Option B - Total thyroidectomy:
• Papillary carcinoma of the thyroid is the most common type of thyroid cancer and typically has an
excellent prognosis.
• The treatment of choice for papillary carcinoma of the thyroid, especially in a young patient with a
significant tumor burden, is a total thyroidectomy.
• This surgical intervention ensures complete removal of the thyroid gland, which is the primary site of
the cancer.
Incorrect Options:
Options A - Thyroid ablation using radioactive Iodine: Radioactive iodine ablation is a treatment option f
or certain cases of thyroid cancer, but it is typically not the initial treatment of choice for papillary carcin
oma. Total thyroidectomy is preferred in this scenario.
Options C - Observation: Observation alone is not an appropriate treatment for papillary carcinoma of t
he thyroid. Active intervention is necessary to remove the cancerous tissue and prevent further progres
sion.
Options D - Hemi-thyroidectomy: Hemi-thyroidectomy involves removing only one lobe of the thyroid gl
and. However, in papillary carcinoma, a

Page 63

243
total thyroidectomy is usually recommended to ensure complete removal of the cancerous tissue.

Solution for Question 56:


Correct Option B - 24-hour urinary Fractionated Metanephrine:
• Pheochromocytoma is a rare neuroendocrine tumor arising from chromaffin cells of the adrenal
medulla or sympathetic ganglia. It secretes catecholamines, including epinephrine, norepinephrine, and
dopamine, leading to characteristic symptoms like hypertension, headaches, palpitations, and
sweating.
• The diagnosis of pheochromocytoma involves biochemical testing to measure levels of
catecholamines and their metabolites.
• Fractionated metanephrines, including metanephrine and normetanephrine, are the most sensitive
and specific tests for diagnosing pheochromocytoma.
• Fractionated metanephrines are measured in a 24-hour urine collection to provide an accurate
reflection of catecholamine production over time.
• Elevated levels of fractionated metanephrines indicate excess catecholamine secretion, supporting
the diagnosis of pheochromocytoma.
Incorrect Options:
Option A - 24-hour urinary Vanillylmandelic acid:
• Vanillylmandelic acid (VMA) is a metabolite of epinephrine and norepinephrine.
• While elevated VMA levels can be seen in patients with pheochromocytoma, fractionated
metanephrines are more sensitive and specific for diagnosis.
• VMA measurement was historically used but has been largely replaced by fractionated metanephrines
due to their higher sensitivity and specificity.
Option C - 24-hour Urinary Hydroxy indole acetic acid:
• Hydroxyindoleacetic acid (HIAA) is a metabolite of serotonin, not catecholamines.
• While serotonin-producing tumors can cause symptoms similar to pheochromocytoma, such as
flushing and diarrhea, measuring HIAA is not the primary diagnostic test for pheochromocytoma.
Option D - 24-hour Urinary Hydroxy tryptamine:
• Hydroxytryptamine, also known as serotonin, is not directly related to the pathophysiology of
pheochromocytoma.
• Elevated levels of serotonin may be seen in certain carcinoid tumors, which can present with
symptoms similar to pheochromocytoma, but this test is not specific for pheochromocytoma diagnosis.

Solution for Question 57:


Correct Option C - Sentinel Lymph node biopsy:
• This procedure involves identifying and sampling the sentinel lymph node(s), which are the first lymph
nodes that receive drainage from the tumor site. It is commonly performed in certain cancers, such as

Page 64

244
breast cancer and melanoma, to assess whether cancer cells have spread to the lymph nodes. It helps
determine the extent of lymph node involvement and guides further treatment decisions.
Incorrect Options:
Option A - Localizing the tumor: This refers to procedures or techniques used to precisely locate the tu
mor within the body. It can involve imaging techniques such as ultrasound, MRI, or CT scans, or it can i
nvolve the use of markers or tracers to highlight the tumor. The goal is to accurately identify the tumor's
location for further diagnostic or therapeutic interventions.
Option B - Identifying the margins: This refers to the assessment of surgical margins during tumor rese
ction. Surgeons aim to remove the tumor along with a margin of healthy tissue to ensure complete rem
oval and reduce the risk of cancer recurrence. Techniques such as intraoperative frozen section analys
is or imaging-guided localization can be used to evaluate the margins and ensure adequate tumor clea
rance.
Option D - Identifying metastasis: Metastasis refers to the spread of cancer cells from the primary tumo
r to distant sites in the body. Identifying metastasis involves imaging techniques, such as CT scans, M
RI, or PET scans, to detect the presence of cancer cells in other organs or tissues. It helps determine t
he stage of the cancer and guides treatment planning.

Solution for Question 58:


Correct Option B - Mondor's disease:
• Mondor's disease, also known as Mondor's phlebitis, is a rare condition characterized by inflammation
and thrombosis of the superficial veins of the breast or chest wall involving the lateral thoracic vein.
• It presents with a palpable cord-like structure under the skin, often accompanied by pain and redness.
It typically resolves spontaneously within a few weeks.
Incorrect Options:
Option A - Paget's disease: Paget's disease of the breast is a rare form of breast cancer that affects th
e skin and nipple. It typically presents with symptoms such as itching, redness, flaking, and crusting of t
he nipple and surrounding areola. It is less likely to be the probable diagnosis based solely on the infor
mation that a clinical image of the breast is provided.
Option C - Zuska's disease: Zuska's disease is not a
recognized medical condition related to breast disorders. Therefore, it is an incorrect option.
Option D - Tietze syndrome: Tietze syndrome is a rare inflammatory condition characterized by swellin
g and tenderness of the cartilage that connects the ribs to the breastbone (sternum). It typically causes
localized chest pain and swelling. While it involves the chest area, it is not directly related to breast dis
orders, making it an unlikely diagnosis.

Solution for Question 59:


Correct Option A - Tumor less than 4 cm:
• Locally advanced breast cancer (LABC) typically refers to breast cancer that has spread beyond the
breast to nearby tissues but has not yet metastasized to distant organs. Locally advanced breast
cancer (LABC) encompasses stages IIIA, IIIB, and IIIC.

Page 65

245
• In the staging of breast cancer, tumor size is an important factor. Generally, tumors larger than 5 cm
(T3 or T4 tumors) are considered locally advanced.
Incorrect Options:
Option B - Inflammatory breast cancer:
• The inflammatory breast cancer is classified as stage T4d, equivalent to stage IIIb, indicating its
resemblance to locally advanced breast cancer (LABC).
Option C - Chest wall involvement:
• Involvement of the chest wall by breast cancer (T4) is a criterion for classifying breast cancer as
locally advanced.
• T4 tumors are specifically mentioned as part of Stage III breast cancer.
Option D - Skin involvement:
• Skin involvement by breast cancer, such as ulceration, edema (including Peau d'orange), or satellite
nodules, is another criterion for classifying breast cancer as locally advanced (T4b).
• T4b tumors are explicitly mentioned as part of Stage III breast cancer.

Solution for Question 60:


Correct Option A - Lahey's method:
• Lahey's method, also known as the Lahey maneuver, is a technique used to palpate a neck nodule. It
involves the examiner placing their thumb and index finger on either side of the neck nodule and gently
applying pressure while rolling the nodule between their fingers. This maneuver helps to assess the
size, shape, consistency, and mobility of the nodule.
Incorrect Options:
Option B - Crile's method: Crile's method, also known as the Crile maneuver, is a technique used in thy
roid surgery to mobilize the thyroid gland. It involves the surgeon placing their fingers around the thyroi
d gland and applying gentle upward and downward traction to free it from surrounding structures. This
method is not specifically related to palpation of neck nodules.
Option C - Kocher's test: Used to rule out tracheal narrowing
Option D - Pemberton's method: Pemberton's method is a technique used to assess the presence of s
uperior vena cava obstruction. It involves the patient raising their arms above their head for a prolonge
d period, which leads to facial congestion and respiratory distress if there is an obstruction. This metho
d is not relevant to the palpation of neck nodules.

Solution for Question 61:


Correct Option D - A-4, B-1, C-2, D-3:
• A. Breast surgery - 4. Long Thoracic Nerve: The long thoracic nerve innervates the serratus anterior
muscle, which stabilizes the scapula. Injury to the long thoracic nerve can result in winging of the
scapula, causing weakness in shoulder abduction and upward rotation.

Page 66

246
• B. Thyroid surgery - 1. External Laryngeal nerve: The external laryngeal nerve supplies the
cricothyroid muscle responsible for vocal cord tension. Damage to the external laryngeal nerve can
lead to hoarseness and difficulty in pitch modulation.
• C. Submandibular excision - 2. Hypoglossal nerve: The hypoglossal nerve controls the movement of
the tongue. Injury to the hypoglossal nerve can result in tongue weakness, deviation towards the
affected side, and difficulty speaking and swallowing.
• D. Thymectomy - 3. Vagus nerve: The vagus nerve has various branches and functions, including
innervating the vocal cords. Injury to the vagus nerve can lead to hoarseness, vocal cord paralysis, and
swallowing difficulties.
Incorrect Options:
Options A, B
& C: These options are incorrect as they do not match nerve injuries and surgeries correctly.

Solution for Question 62:


Correct Option D - Marginal mandibular nerve:
• Thyroid surgery involves the removal or partial removal of the thyroid gland. During the procedure,
there is a risk of injury to nearby nerves, which can result in various complications.
• The marginal mandibular nerve is the nerve branch of the facial nerve that supplies the muscles of the
lower lip and chin. It is not directly involved or at risk during thyroid surgery, as it is not in close
proximity to the thyroid gland.
Thyroid surgery involves the removal or partial removal of the thyroid gland. During the procedure, ther
e is a risk of injury to nearby nerves, which can result in various complications.
The marginal mandibular nerve is the nerve branch of the facial nerve that supplies the muscles of the l
ower lip and chin. It is not directly involved or at risk during thyroid surgery, as it is not in close proximit
y to the thyroid gland.
Incorrect Options:
Option A - Recurrent laryngeal nerve:
• The recurrent laryngeal nerve is one of the most commonly injured nerves during thyroid surgery.
• It is crucial for the innervation of the muscles controlling the vocal cords. Injury to this nerve can lead
to vocal cord paralysis or paresis, resulting in hoarseness of voice or even airway compromise in
severe cases.
The recurrent laryngeal nerve is one of the most commonly injured nerves during thyroid surgery.
It is crucial for the innervation of the muscles controlling the vocal cords. Injury to this nerve can lead to
vocal cord paralysis or paresis, resulting in hoarseness of voice or even airway compromise in severe
cases.
Option B - Cervical sympathetic trunk:
• Injury to the cervical sympathetic trunk during thyroid surgery can result in Horner's syndrome.
• Horner's syndrome presents with a combination of symptoms, including ptosis (drooping of the upper
eyelid), miosis (constriction of the pupil), anhidrosis (lack of sweating), and sometimes enophthalmos
(sunken appearance of the eyeball).

Page 67

247
• The cervical sympathetic trunk carries sympathetic fibers involved in the control of pupil size, eyelid
position, and facial sweating.
Injury to the cervical sympathetic trunk during thyroid surgery can result in Horner's syndrome.
Horner's syndrome presents with a combination of symptoms, including ptosis (drooping of the upper e
yelid), miosis (constriction of the pupil), anhidrosis (lack of sweating), and sometimes enophthalmos (s
unken appearance of the eyeball).
The cervical sympathetic trunk carries sympathetic fibers involved in the control of pupil size, eyelid po
sition, and facial sweating.
Option C - External laryngeal nerve:
• While the external laryngeal nerve can be injured during thyroid surgery, it is not the least injured
nerve among the options provided.
• Injury to the external laryngeal nerve can lead to weakness or paralysis of the cricothyroid muscle,
which may result in changes in voice pitch and quality.
While the external laryngeal nerve can be injured during thyroid surgery, it is not the least injured nerve
among the options provided.
Injury to the external laryngeal nerve can lead to weakness or paralysis of the cricothyroid muscle, whi
ch may result in changes in voice pitch and quality.

Solution for Question 63:


Correct Option D - Hashimoto's thyroiditis:
• Hashimoto's thyroiditis is an autoimmune disorder where the body's immune system mistakenly
attacks the thyroid gland, leading to chronic inflammation and destruction of thyroid tissue.
• This results in decreased production of thyroid hormones, including T4. As a compensatory
mechanism, the pituitary gland releases more TSH to stimulate the thyroid gland, causing increased
TSH levels.
Incorrect Options:
Option A - Hypoparathyroidism: Hypoparathyroidism is a condition characterized by inadequate produc
tion or secretion of parathyroid hormone (PTH) by the parathyroid glands. It is not directly associated w
ith changes in TSH and T4 levels.
Option B - Pituitary Adenoma: A pituitary adenoma is a noncancerous tumor that develops in the pituita
ry gland. While it can cause various hormonal imbalances, such as excessive production of prolactin (l
eading to hyperprolactinemia), it does not typically result in increased TSH and low T4 levels.
Option C - Grave's disease: Grave's disease is an autoimmune disorder characterized by overactivity o
f the thyroid gland, leading to excessive production of thyroid hormones. In Grave's disease, TSH level
s are typically decreased due to the negative feedback mechanism resulting from high levels of thyroid
hormones. Therefore, it does not match the scenario of increased TSH and low T4 levels described in t
he question.

Solution for Question 64:

Page 68

248
Correct Option A - Inflammatory carcinoma:
• The clinical presentation includes a rapid onset of swelling, redness, warmth, and sometimes pain in
the affected breast, making it a case of Inflammatory breast cancer.
• Inflammatory breast cancer (IBC), also known as mastitis carcinomatosa, is an aggressive form of
breast cancer characterized by inflammatory changes in the breast tissue.
• Unlike other types of breast cancer, IBC may not always present with a palpable lump unlike in this
case. Instead, the breast may appear swollen and red and have an "orange peel" texture due to skin
involvement.
• In IBC, more than one-third of the breast skin or more than 33% of the breast is involved by
inflammatory changes.
• The stage of IBC is usually T4d, which corresponds to stage III breast cancer. This indicates locally
advanced disease with a poor prognosis.
Incorrect Options:
Option B - Giant Fibroadenoma:
• Giant fibroadenoma is a benign breast tumor that typically presents as a large, well-defined, mobile
lump in the breast.
• Unlike inflammatory breast cancer, it does not cause inflammatory changes in the breast tissue or skin
involvement.
• Giant fibroadenomas are usually painless and do not cause local rise of temperature or skin changes.
Option C - Phyllodes Tumor:
• Phyllodes tumors are rare fibroepithelial tumors of the breast that can be benign, borderline, or
malignant.
• While they can present as a palpable breast lump, they do not typically cause inflammatory changes
or skin involvement as seen in inflammatory breast cancer.
• Phyllodes tumors are usually well-circumscribed and may not cause a local rise of temperature.
Option D - Galactocele:
• Galactocele is a benign breast lump that develops due to milk retention within the breast ducts.
• It is commonly seen in breastfeeding women and presents as a painless, cystic lump.
• Galactoceles do not cause inflammatory changes, skin involvement, or a local rise of temperature as
seen in inflammatory breast cancer.

Solution for Question 65:


Correct Option C - Inadequate patient preparation:
• Inadequate patient preparation, such as insufficient control of thyroid hormone levels before surgery,
can contribute to the occurrence of thyroid storm. Patients with poorly controlled hyperthyroidism or
Graves' disease are at a higher risk of developing thyroid storm during or after thyroidectomy.
Incorrect Options:
Option A - Rough handling of thyroid during surgery: While rough handling of the thyroid during surgery
can potentially cause complications such as bleeding or damage to surrounding structures, it is not the

Page 69

249
usual cause of thyroid storm. Thyroid storm refers to a life-threatening condition characterized by an e
xaggerated hypermetabolic state, and it is typically related to the release of excessive thyroid hormone
s into the bloodstream.
Option B - Due to parathyroid gland: The parathyroid glands are small glands located near the thyroid
gland. Their primary function is to regulate calcium levels in the body. While complications related to th
e parathyroid glands can occur during thyroid surgery, they are not the usual cause of thyroid storm. T
hyroid storm is specifically associated with the overproduction of thyroid hormones, not parathyroid hor
mone.
Option D - Infections: Infections, such as surgical site infections, can be a complication of thyroid surge
ry. However, they are not typically associated with the development of thyroid storm. Infections may ca
use local symptoms and complications but are not the primary cause of the systemic hypermetabolic st
ate seen in thyroid storm.

Solution for Question 66:


Correct Option B - Nucleotide scan:
• Nucleotide scans, specifically metaiodobenzylguanidine (MIBG) scintigraphy, are highly effective in
localizing pheochromocytomas. MIBG is a radiopharmaceutical that is selectively taken up by
neuroendocrine tumors, including pheochromocytomas. By administering a small amount of radioactive
iodine-labeled MIBG intravenously, and then performing a whole-body scan using a gamma camera,
the location of extra-adrenal pheochromocytomas can be accurately identified.
Incorrect Options:
Option A - X-ray: X-ray imaging is not the best way to localize extra-adrenal pheochromocytoma. X-ray
s are not sensitive enough to detect small tumors or provide precise anatomical information about their
location. Therefore, X-rays are not commonly used for localizing pheochromocytomas.
Option C - VMA excretion: Vanillyl mandelic acid (VMA) is a metabolite of catecholamines, which are p
roduced by pheochromocytomas. VMA excretion refers to the measurement of VMA levels in a 24-hour
urine sample. While elevated VMA levels can suggest the presence of pheochromocytoma, they do no
t provide precise localization information. VMA excretion is useful as a screening tool for detecting exc
ess catecholamine production but is not the best method for localizing the tumor.
Option D - Clinical examination: Clinical examination, including physical examination and medical histo
ry assessment, can provide valuable information in the evaluation of patients suspected of having pheo
chromocytoma. However, clinical examination alone is insufficient for accurate localization of the tumor
. Imaging studies, such as nucleotide scans or other radiological modalities, are necessary for precise l
ocalization.

Solution for Question 67:


Correct Option B - Papillary Carcinoma:
• Frequently Linked Factors: Prior Radiation Exposure: Low-dose radiation exposure during childhood
is a significant risk factor. Elevated RET/PTC Rearrangements: This genetic rearrangement is
commonly found in papillary thyroid carcinoma. BRAF Mutations: BRAF mutations are also frequently
associated with papillary thyroid carcinoma.

Page 70

250
• Prior Radiation Exposure: Low-dose radiation exposure during childhood is a significant risk factor.
• Elevated RET/PTC Rearrangements: This genetic rearrangement is commonly found in papillary
thyroid carcinoma.
• BRAF Mutations: BRAF mutations are also frequently associated with papillary thyroid carcinoma.
• Pathology Features: Papillary projections are characteristic. Optically clear nuclei, also known as
"Orphan Annie eye nuclei," are observed. Pseudo inclusion bodies and dystrophic calcification
(Psammoma Bodies) are present.
• Papillary projections are characteristic.
• Optically clear nuclei, also known as "Orphan Annie eye nuclei," are observed.
• Pseudo inclusion bodies and dystrophic calcification (Psammoma Bodies) are present.
• Clinical Features: Midline swelling in the anterior part of the neck is typical. Lateral aberrant thyroid
may be present, leading to palpable cervical lymph nodes. Metastatic deposits from papillary carcinoma
thyroid can be identified via FNAC.
• Midline swelling in the anterior part of the neck is typical.
• Lateral aberrant thyroid may be present, leading to palpable cervical lymph nodes.
• Metastatic deposits from papillary carcinoma thyroid can be identified via FNAC.
• Prior Radiation Exposure: Low-dose radiation exposure during childhood is a significant risk factor.
• Elevated RET/PTC Rearrangements: This genetic rearrangement is commonly found in papillary
thyroid carcinoma.
• BRAF Mutations: BRAF mutations are also frequently associated with papillary thyroid carcinoma.
• Papillary projections are characteristic.
• Optically clear nuclei, also known as "Orphan Annie eye nuclei," are observed.
• Pseudo inclusion bodies and dystrophic calcification (Psammoma Bodies) are present.
• Midline swelling in the anterior part of the neck is typical.
• Lateral aberrant thyroid may be present, leading to palpable cervical lymph nodes.
• Metastatic deposits from papillary carcinoma thyroid can be identified via FNAC.
Incorrect Options:
Option A - Follicular Carcinoma:
• Follicular carcinoma is not typically associated with prior radiation exposure.
• It is characterized by follicular patterns rather than papillary projections.
Option C - Anaplastic Carcinoma:
• Anaplastic carcinoma is not commonly associated with prior radiation exposure during childhood.
• It is characterized by rapid growth and aggressive behavior.
Option D - Medullary Carcinoma:
• Medullary carcinoma is not typically associated with prior radiation exposure.
• It arises from parafollicular C cells rather than follicular or papillary cells.
• It is associated with multiple endocrine neoplasia type 2 (MEN 2) syndromes.

Page 71

251
• Calcitonin levels are elevated in this type of carcinoma.

Solution for Question 68:


Correct Option A - Phyllodes tumor:
• Phyllodes tumors, also known as cystosarcoma phyllodes, are rare breast tumors that arise from the
stromal tissue of the breast.
• They can present as a palpable lump in the breast, and on clinical examination, they may exhibit large
size, as in this case (9x10 cm).
• Phyllodes tumors are characterized by increased stromal cellularity and a leaflike architecture on
histopathology.
• They are typically benign but can have malignant features in some cases.
Incorrect Options:
Option B - Giant fibroadenoma: Giant fibroadenomas are benign breast tumors that arise from the glan
dular tissue. They can also present as a palpable lump in the breast and can grow to a large size. How
ever, the histopathology findings of increased stromal cellularity and leaflike architecture are not charac
teristic of fibroadenomas.
Option C - Inflammatory carcinoma: Inflammatory carcinoma refers to an aggressive and rare form of b
reast cancer characterized by the presence of redness, warmth, and swelling of the breast, resembling
an infection. The absence of axillary lymphadenopathy and the histopathology findings described are n
ot consistent with inflammatory carcinoma.
Option D - Ductal carcinoma: Ductal carcinoma is the most common form of breast cancer and arises fr
om the cells lining the milk ducts. It typically presents as a palpable lump and can exhibit a variety of hi
stopathological features. However, the histopathology findings of increased stromal cellularity and leafli
ke architecture are not indicative of ductal carcinoma.

Solution for Question 69:


Correct Option C - Inadequate patient preparation:
• Inadequate patient preparation, such as insufficient control of thyroid hormone levels before surgery,
can contribute to the occurrence of thyroid storm. Patients with poorly controlled hyperthyroidism or
Graves' disease are at a higher risk of developing thyroid storm during or after thyroidectomy.
Inadequate patient preparation, such as insufficient control of thyroid hormone levels before surgery, c
an contribute to the occurrence of thyroid storm. Patients with poorly controlled hyperthyroidism or Gra
ves' disease are at a higher risk of developing thyroid storm during or after thyroidectomy.
Incorrect Option:
Option A - Rough handling of thyroid during surgery: While rough handling of the thyroid during surgery
can potentially cause complications such as bleeding or damage to surrounding structures, it is not the
usual cause of thyroid storm. Thyroid storm refers to a life-threatening condition characterized by an e
xaggerated hypermetabolic state, and it is typically related to the release of excessive thyroid hormone
s into the bloodstream.

Page 72

252
Option B - Due to parathyroid gland: The parathyroid glands are small glands located near the thyroid
gland. Their primary function is to regulate calcium levels in the body. While complications related to th
e parathyroid glands can occur during thyroid surgery, they are not the usual cause of thyroid storm. T
hyroid storm is specifically associated with the overproduction of thyroid hormones, not parathyroid hor
mone.
Option D - Infections: Infections, such as surgical site infections, can be a complication of thyroid surge
ry. However, they are not typically associated with the development of thyroid storm. Infections may ca
use local symptoms and complications but are not the primary cause of the systemic hypermetabolic st
ate seen in thyroid storm.

Solution for Question 70:


Correct Option D - Areola:
• In sentinel lymph node biopsy (SLNB), a dye is injected near the site of the primary tumor to identify
the sentinel lymph node(s) draining that area. The sentinel lymph node is the first lymph node in the
lymphatic system that receives drainage from the tumor site, and it is the most likely node to contain
metastatic cancer cells if they have spread.
• In breast cancer, the dye for SLNB is commonly injected in or around the areola. The areola is the
pigmented area surrounding the nipple. By injecting the dye in this location, it can flow through the
lymphatic channels and reach the sentinel lymph node(s) in the axilla (armpit).
Incorrect Options:
Option A - Axilla:
• Injecting the dye directly into the axilla is not the common practice for sentinel lymph node biopsy in
breast cancer. While the axillary lymph nodes are the primary drainage site for the breast, the dye is
typically injected near the areola to allow it to flow through the lymphatic channels and reach the
sentinel lymph node(s) in the axilla.
Option B - Tail of Spence:
• The tail of Spence is an extension of breast tissue into the axilla. However, the dye is not typically
injected specifically into the tail of Spence. It is more common to inject the dye near the areola to
ensure proper drainage to the sentinel lymph node(s).
Option C - Nipple:
• Injecting the dye directly into the nipple is not the standard practice for sentinel lymph node biopsy.
While the nipple is part of the breast tissue, it is not the typical injection site for the dye. The areola,
which surrounds the nipple, is a more common site for the injection.

Solution for Question 71:


Correct Option A - Eczema of skin of nipple:

Page 73

253
• Paget's disease of the breast is a rare form of breast cancer that primarily affects the skin of the nipple
and areola. It is characterized by eczema-like changes in the appearance and texture of the skin, such
as redness, itching, flaking, and crusting. The affected skin may also become thickened, scaly, or
ulcerated.
• The disease usually starts in the nipple and then spreads to the areola. It is often associated with an
underlying ductal carcinoma in situ (DCIS), which is a non-invasive form of breast cancer. In some
cases, invasive breast cancer may also be present.
• Paget's disease of the breast is typically diagnosed through a combination of clinical examination,
imaging studies, such as mammography or breast ultrasound, and biopsy of the affected skin and
underlying breast tissue.
Incorrect Options:
Options B - Eczema of the skin of the areola: This option is incorrect because Paget's disease of the br
east primarily affects the skin of the nipple, not the areola. Eczema-like changes are seen in the nipple
skin, such as redness, scaling, itching, and crusting. The areola may be involved to a
lesser extent, but it is not the primary site of involvement.
Options C
- Mastitis carcinomatosis: This option is incorrect because mastitis carcinomatosis refers to a condition
where breast cancer cells invade the breast tissue, causing inflammation and infection-like symptoms.
Paget's disease of the breast is a distinct entity and is not synonymous with mastitis carcinomatosis. W
hile Paget's disease can be associated with underlying breast cancer, it does not represent a
direct inflammation of the breast tissue.
Options D
- Atrophic scirrhous carcinoma: This option is incorrect because atrophic scirrhous carcinoma is a subt
ype of invasive breast carcinoma characterized by dense fibrous tissue and marked stromal reaction. It
is not specific to Paget's disease of the breast, which primarily involves the skin of the nipple. Atrophic
scirrhous carcinoma does not manifest as eczema-like changes on the nipple.

Solution for Question 72:

Page 74

254
Correct Option A - Fibroadenoma:
• Fibroadenoma is the most common benign breast tumor.
• It is a non-cancerous growth that typically occurs in young women.
• Fibroadenomas are composed of glandular and fibrous tissue and usually feel like a firm, rubbery
mass that is smooth and movable.
• They are usually painless and do not increase the risk of developing breast cancer.
Incorrect Options:
Options B - Fibroadenosis:
• Fibroadenosis, also known as fibrocystic changes or benign breast changes, refers to the presence of
lumps, pain, and breast swelling. It is not a tumor.
Options C - DCIS (Ductal Carcinoma In Situ):
• DCIS is not a benign tumor but rather a non-invasive form of breast cancer. It is characterized by
abnormal cells confined to the milk ducts of the breast.
Options D - Phyllodes tumor:
• Phyllodes tumors are rare fibroepithelial tumors of the breast. While they are benign in most cases,
they have the potential to be malignant. However, fibroadenomas are more commonly encountered in
clinical practice compared to phyllodes tumors.

Solution for Question 73:


Correct Option B - Axillary sampling:
• Axillary sampling, is not part of the breast triple assessment. Axillary sampling refers to the removal of
lymph nodes from the armpit (axilla) for evaluation, particularly in cases of known or suspected breast
cancer to determine if the cancer has spread to the lymph nodes. While axillary sampling is an
important procedure in breast cancer staging, it is not considered part of the initial diagnostic
assessment known as the breast triple assessment.
Axillary sampling, is not part of the breast triple assessment. Axillary sampling refers to the removal of l
ymph nodes from the armpit (axilla) for evaluation, particularly in cases of known or suspected breast c
ancer to determine if the cancer has spread to the lymph nodes. While axillary sampling is an important
procedure in breast cancer staging, it is not considered part of the initial diagnostic assessment known
as the breast triple assessment.
Incorrect Option:
Option A - Clinical examination: This involves a
thorough examination of the breasts and surrounding areas by a healthcare professional to assess any
palpable masses, changes in breast shape or texture, or other clinical findings.
Option C - USG: This may include a mammogram and/or ultrasound (USG) of the breasts. These imagi
ng tests help visualize the internal structures of the breast and can detect abnormalities such as masse
s or calcifications.
Option D - FNAC and biopsy: Fine needle aspiration cytology (FNAC) or biopsy involves obtaining a
tissue sample from a breast abnormality for pathological examination. FNAC uses a
fine needle to extract cells for analysis, while biopsy involves the removal of a
small piece of tissue. These samples are then examined under a

Page 75

255
microscope to determine if they are benign or malignant.
• Breast triple assessment is a diagnostic process used to evaluate breast abnormalities or suspected
breast cancer. It typically involves three components:

Solution for Question 74:


Correct Option B - Fibroadenoma:
• Fibroadenoma is a benign encapsulated tumor occurring commonly in young females of 15-25 years
of age.
• Microscopic findings of fibroadenoma: Intracanalicular- Large and soft (mainly cellular). Stroma with
distorted duct. Pericanalicular- Small and hard (mainly fibrous). Stroma with normal duct.
• Intracanalicular- Large and soft (mainly cellular). Stroma with distorted duct.
• Pericanalicular- Small and hard (mainly fibrous). Stroma with normal duct.
• Investigation of choice- FNAC
• Popcorn calcification is seen in mammography although mammography has a reduced sensitivity due
to dense and glandular breasts.
• Intracanalicular- Large and soft (mainly cellular). Stroma with distorted duct.
• Pericanalicular- Small and hard (mainly fibrous). Stroma with normal duct.
Incorrect Options:
Option A - Invasive ductal carcinoma:
• Invasive ductal carcinoma shows irregular infiltrative borders without a well-defined capsule and
intense stromal reaction
Option C - Phyllodes tumor:
• Phyllodes tumor on microscopy shows cystic spaces with leaf-like projections and cells show
hypercellularity and pleomorphism.
Option D - Lobular carcinoma in-situ:
• Lobular carcinoma in-situ consists of a uniform population of cells with oval or round nuclei and small
nucleoli involving ducts and lobules

Solution for Question 75:


Correct Option A - Lymphangiosarcoma:
• Patients develop edema, and violet/purple/brownish nodules in the affected limbs.
• This is called Stewart Treves syndrome/Lymphangiosarcoma.
• Skin/nodule biopsy is confirmatory and it carries a poor prognosis.
Incorrect Options:
Option B - Thoracic outlet syndrome:

Page 76

256
• Thoracic outlet syndrome presents with symptoms of pain, paraesthesia, edema, cyanosis, cold, and
weakness of the hands
Option C - Chronic venous insufficiency:
• Chronic venous insufficiency presents with postural discomfort, varicose veins, edema, pigmentation,
induration, dermatitis, lipodermatosclerosis, and ulceration.
Option D - Recurrent breast cancer:
• Recurrent breast cancer presents with a lump, nipple retraction, lymph node swelling, and signs of
metastasis.

Solution for Question 76:


Correct Option B - Superficial parathyroidectomy:
• Parathyroid adenoma is the most common cause of primary hyperparathyroidism.
• Superficial parathyroidectomy is the treatment of choice.
• It is preferred in patients with classical pentad and patients below 50 years of age. Kidney stones
Painful bones Abdominal groans Psychic moans Fatigue overtones
• Kidney stones
• Painful bones
• Abdominal groans
• Psychic moans
• Fatigue overtones
• Kidney stones
• Painful bones
• Abdominal groans
• Psychic moans
• Fatigue overtones
Incorrect Options:
Option A - Enucleation:
• This is not used in the treatment of parathyroid adenoma
Option C - Chemotherapy:
• This is not used in the treatment of parathyroid adenoma
Option D - Radiation therapy:
• This is not used in the treatment of parathyroid adenoma

Page 77

257
Liver
1. An 18-year-old boy came to your hospital with complaints of severe abdominal pain with no other
associated symptoms. On examination, you noted several new bruises all over the body. After
establishing a rapport with the patient, he gives a history of being bullied and punched in the abdomen
multiple times forcefully by his classmate. His blood pressure is 140/100 mmHg, and his pulse rate is
102 per minute. The respiratory rate is 18 per minute. The patient is afebrile. Which of the following
should be the next step in managing this patient?
(or)
An 18-year-old boy came with several new bruises all over his body. He has a history of being punched
in the abdomen repeatedly by his classmate. Which of the following should be the next step in
managing this patient?
A. E-FAST
B. Abdominal ultrasound
C. CT scan of abdomen
D. Peritoneal lavage
----------------------------------------
2. A 37-year-old Indian man who had recently moved to Taiwan presented with right upper quadrant
pain and intermittent low-grade fever. Both sonography and computed tomography demonstrated a
cystic lesion with vesicles at its periphery in segments 6 and 7 of the liver. What is your probable
diagnosis?
(or)
What is the diagnosis in a patient with cystic lesion with vesicles at its periphery in segments 6 and 7 of
the liver who recently moved to Taiwan and presents with upper quadrant pain and intermittent low
grade fever?
A. Polycystic liver disease
B. Amoebic liver abscess
C. Hydatid cyst
D. Pyogenic liver abscess
----------------------------------------
3. For a 47-year-old male liver recipient with hepatitis B virus (HBV) cirrhosis and hepatocellular
carcinoma (two tumors, each ≤ 3 cm without extrahepatic and major vessel involvement), a BMI of 19.8
kg/m2, MELD score of 18, and CPT classification B7, what criteria are employed to assess eligibility for
liver transplantation in cases of hepatocellular carcinoma?
(or)
The criteria used to determine whether patients with HCC can proceed with liver transplantation is?
A. Ranson criteria
B. Milan's criteria
C. Eagle's criteria
D. Rockall criteria
----------------------------------------

258
4. A 35-year-old male patient comes with an altered state of consciousness. He has a history of chronic
liver disease. On examination, there are Slight ascites and tremors. Laboratory values show: Total
Bilirubin - 2.5 mg/dL Albumin - 3g/dL INR=2.0 According to the given data, the patient belongs to which
class of Child Turcotte Pugh score?
(or)
A 35-year-old male patient is brought to the hospital with an altered state of consciousness. He has a
history of chronic liver disease. On examination, there are mild ascites and tremors. Laboratory values
show: Total Bilirubin - 2.5 mg/dL Albumin - 3 g/dL INR=2.0 According to the given data, the patient
belongs to which class of Child Turcotte Pugh score?
A. Class A
B. Class B
C. Class C
D. Class D
----------------------------------------
5. A 60-year-old man is brought to the emergency department with complaints of right upper quadrant
pain, high fever, yellowish discolouration of the skin, and altered mental status. Temperature is 40° C
(104° F), blood pressure is 90/50 mmHg, pulse is 120 bpm, and respiration rate is 22 bpm. The patient
is sluggish and difficult to arouse. Scleral icterus is present. He is a known case of ulcerative colitis and
complaints of hematochezia. The cardiopulmonary examination is unremarkable. The abdomen is
tender to palpation in the right upper quadrant. Laboratory evaluation reveals the following: Liver
function studies Total bilirubin 6.2g/dl Alkaline phosphatase 800 U/L Aspartate aminotransferase
(SGOT) 108 U/L Alanine aminotransferase (SGPT) 142 U/L Which of the following is the most likely
diagnosis?
A. Annular pancreas
B. Cholangiocarcinoma
C. Chronic pancreatitis
D. Primary sclerosing cholangitis
----------------------------------------
6. A 61-year-old female with a history of Autosomal Dominant Polycystic Kidney Disease since 2010
was admitted with two months history of abdominal discomfort with rapidly progressing jaundice
following nausea and weight loss. Liver function tests showed a picture of obstructive jaundice. The
Hepatitis virus test came out negative. MR Cholangiopancreatography revealed a bilobar polycystic
liver with cysts at the porta hepatis that caused a marked narrowing at the bifurcation of the common
hepatic duct. Mild dilation of intrahepatic right bile ducts was seen. Which of the following options for
the above patient is correct?
(or)
In a 61-year-old female, MR Cholangiopancreatography revealed a bilobar polycystic liver with cysts at
the porta hepatis that caused a marked narrowing at the bifurcation of the common hepatic duct. Mild
dilation of intrahepatic right bile ducts was seen. Which of the following options for the above patient is
correct?
A. Somatostatin as an initial treatment for this condition
B. It is not associated with hypertension
C. Serum calcium is low

Page 2

259
D. Liver transplantation for progressive disease
----------------------------------------
7. In a 54-year-old truck driver with a 2-month history of right hypochondriac pain, fever, decreased
appetite, weight loss, and occasional vomiting, a computed tomography reveals multiple hypodense
hepatic lesions with ring enhancement. The patient reports multiple unprotected sexual contacts during
work travels. What tumor markers are likely to be elevated if malignancy is suspected?
(or)
Which tumour markers will be raised if you suspect malignancy of liver?
A. CEA
B. PIVKA-2
C. ALP
D. CA 125
----------------------------------------
8. A 59-year-old male reached the OPD with complaints of abdominal pain and fever for the previous 3
months. Examination indicated a hard, enlarged liver. On further investigation, AFP levels were found
to be elevated. A triple-phase CT was done, which verified the diagnosis of hepatocellular carcinoma.
Which of the following is responsible for hepatocellular carcinoma?
(or)
Which of the following is risk factor for hepatocellular carcinoma?
A. Fair skin(skin type I and II)
B. Barrett's Esophagus
C. Viral hepatitis B
D. Human papillomavirus (HPV)
----------------------------------------
9. A 33-year-old housewife presented with a six-week history of upper abdominal pain of sudden onset
with progressive abdominal distention, and edema of the legs accompanied by mild Jaundice. She has
been on oral contraceptives for two years. An abdominal ultrasound scan confirmed the presence of
hepatomegaly and ascites, with evidence of splenic enlargement. The liver appearance was
nonspecific. The jugular venous pulse is also normal. Which of the following is the most likely
diagnosis?
(or)
A 33-year-old housewife presented a six-week history of upper abdominal pain of sudden onset,
progressive abdominal distention, and oedema of the legs accompanied by mild jaundice. She has
been on Oral contraceptives for two years. The Jugular Venous Pulse is also normal. Which of the
following is most likely like diagnosis
A. Budd Chiari Syndrome
B. Hepatic hemangioma
C. Hepatic adenomas
D. Hepatocellular carcinoma
----------------------------------------

Page 3

260
10. Which of the following is not a part of the capsular plate of the liver?
(or)
The Fusion of the Glisson capsule with the connective tissue sheaths surrounding the biliary and
vascular elements at the inferior aspect of the liver constitutes the plate system of the liver. Which of
the following is not a part of the capsular plate?
A. Portal plate
B. Hilar plate
C. Umbilical plate
D. Cystic plate
----------------------------------------
11. A 29-year-old civil engineer came to your hospital for a yearly general check-up. The patient was
admitted, and all the routine tests were taken which were all within the normal range. Ultrasonography
of the abdomen was taken which revealed a solitary hypoechoic lesion of the liver without septa or
debris. Which of the following is the most probable diagnosis?
(or)
A 29-year-old civil engineer had an Ultrasonography of the abdomen which revealed a Solitary
hypoechoic lesion of the liver without septa or debris. Which of the following is the most probable
diagnosis?

A. Hydatid cyst
B. Caroli's disease
C. Liver abscess
D. Simple cyst
----------------------------------------
12. The Oncological outcome in this case of hepatocellular carcinoma is best described by which of the
following?
(or)
A 65 years male patient from northeast India presented with persistent vomiting and weight loss of 6 kg
in 3 months. His vomiting was intermittent once in 2-3 days in the beginning but progressively
increased to 2-3 times daily in the last month. He was a known case of hepatitis B virus carrier for the
last 3 years. On examination, the patient was dehydrated; there was fullness in the upper abdomen
with visible peristalsis and non-tender hepatomegaly 3 cm below the right costal margin. Liver function
tests showed mild elevation of liver enzymes. CT scan of the abdomen showed a large tumour in the

Page 4

261
right lobe (segments VI and VII) of the liver, of size 10.5 × 8.4 × 7.2 cm, with an area of central necrosis
and early washout in the arterial phase. The oncological outcome in this case of hepatocellular
carcinoma is best described by which of the following?
A. MELD
B. METAVIR
C. ERCP
D. OKUDA
----------------------------------------
13. A 60-year-old male comes with weight loss. Liver span is 14 cm with no tenderness to palpation.
The prostate gland is slightly enlarged and nontender to palpation. A CT scan of the abdomen with
contrast is given below. Which of the following should be the next best step?
(or)
A 60-year-old male comes to thospital due to an 8kg weight loss in the past 6 months. He is a chronic
smoker. Vital signs are normal. Lungs auscultation is clear, but abdominal examination shows a liver
span of 14 cm with no tenderness to palpation. Rectal examination shows normal sphincter tone, but
the prostate gland is slightly enlarged and nontender to palpation. A CT scan of the abdomen with
contrast is given below. Which of the following should be the next best step?
A. Alpha-fetoprotein measurement
B. Aspiration and culture followed by antibiotic therapy
C. Prostate-specific antigen
D. Colonoscopy
----------------------------------------
14. A 30-year-old woman comes for frequent acute pain attacks in the upper abdominal region. She
had been taking contraceptive pills and steroids. AFP was normal. Ultrasound examination (US)
revealed a neoformation of about 8 cm in the left hepatic lobe, with clear margins; the mass presented
a Heterogeneous echostructure, with hypoechogenic areas alternating with hyperechogenic zones.
CECT scan reveals enhancement with centripetal progression. What is the most likely diagnosis for this
patient?
(or)
A 30-year-old woman was referred to your hospital for frequent attacks of acute pain in the upper
abdominal region; she also complained of nausea and vomiting after eating on several occasions. The
patient had no history of abdominal disease and reported that she had been taking contraceptive pill for
the last ten years and has been using steroids as she is an athlete. Clinical examination of the
abdomen revealed a painful, palpable mass located in the upper quadrants, between the epigastric
region and the left hypochondrium. Ultrasound examination (US) revealed a neoformation of about 8
cm in the left hepatic lobe, with clear margins; the mass presented a Heterogeneous echostructure,
with hypoechogenic areas alternating with hyperechogenic zones. CECT scan reveals enhancement
with centripetal progression. What is the most likely diagnosis for this patient?
A. Hepatic adenoma
B. Hepatocellular carcinoma
C. Focal nodular hyperplasia
D. Metastatic colorectal cancer

Page 5

262
----------------------------------------
15. A 30-year-old man comes to the clinic complaining of yellowish discolouration of skin for the past 15
days. On physical examination, he has icteric sclera. A CT scan of the abdomen reveals a
hypervascular mass in the liver. You order an AFP tumour marker test. Which of the following is not a
feature of the tumour marker AFP?
(or)
Which of the following is not a feature of the tumour marker AFP?
A. Produced by maternal hepatocytes during Pregnancy
B. Conditions with elevated levels of AFP are HCC, Non-seminomatous testicular tumour, Pregnancy
and Gastric cancer
C. Levels >400 ng/mL with arterial enhancement on CT are suggestive of HCC
D. Normal adult serum AFP level is <20 ng/mL
----------------------------------------
16. A 45-year-old male came to the OPD with complaints of pain in the right upper quadrant for the past
two months. Examination revealed a palpable mass in the right hypochondrium, which moves with
respiration, suggesting a hepatic lesion. Which of the following liver tumours always merits Surgery?
(or)
Which of the following liver tumour always merits Surgery?
A. Hemangioma
B. Hepatic adenoma
C. Focal nodular hyperplasia
D. Hepatic cyst
----------------------------------------
17. A 14 months old male child, born of consanguineous marriage. His mother brought him with
presenting complaints of abdominal swelling for 20 days and poor feeding for 10 days. The child was
apparently alright 20 days back. On examination, the liver was palpable 8 cm below the right costal
margin in the midclavicular line, and the spleen was not palpable. CBC showed thrombocytosis (8
lakhs/mm), with other tests normal in range. Had normal liver function tests. USG's abdomen showed a
solid echogenic mass in the liver's right lobe, with a well-defined capsule. Serum AFP level was >1000
ng/ml. Hepatitis B and C and HIV tests were negative. A liver biopsy was performed, which showed
medium-sized cells arranged in solid sheets and trabecular patterns. Which of the following is the most
likely diagnosis?
A. Hepatoblastoma
B. Hepatic adenoma
C. Hepatocellular carcinoma
D. Hepatic cyst
----------------------------------------
18. A 45-year-old male presents to the clinic who is a known case of the Hepatitis B virus. The patient
had developed cirrhosis due to the virus over time and has now come for routine screening for
hepatocellular carcinoma. Which of the following are the screening modalities for HCC in a chronic liver
disease patient.?

Page 6

263
(or)
A 45-year-old male presents to the clinic. He is a known case of the hepatitis B virus. The screening
modalities for hepatocellular carcinoma in chronic liver disease include?
A. Serial Ultrasound + Alpha Fetoprotein
B. Serial Liver function tests + Alpha Fetoprotein
C. Serial Liver function tests + CT scan
D. Serial Ultrasound + Serial Liver function tests
----------------------------------------
19. A 42-year-old male patient was admitted with complaints of severe abdominal pain, fever, jaundice,
and weakness. Plain abdominal radiography showed free gas under the right hemidiaphragm and
heterogeneous liver shade with small gas-fluid levels. The rupture of a liver abscess was suspected.
Laparotomy with adhesiolysis, debridement of the liver abscess cavity, and abdominal drainage were
performed. If this case of the liver abscess had been diagnosed earlier, the rupture could have been
predicted with which of the following factor?
(or)
The rupture of a liver abscess could have been predicted with the following factors?
A. Abscess with a diameter of 2 to 5 cm
B. Age of the patient
C. Gradual increase in the size of the abscess
D. Location of abscess in the right lobe
----------------------------------------
20. A 35-year-old US citizen presented to OPD with abdominal pain and fever. He gives a history of his
visit to India 3 months ago. Ultrasonography showed a 5.5 cm x 3.0 cm x 4.0 cm cavity in the left lobe
of the liver. USG's finding is given below. Which of the following is the most appropriate treatment?
(or)
Which of the following is the most appropriate treatment based on USG findings of liver?

A. Oral metronidazole
B. Emetine hydrochloride
C. Pigtail catheter insertion
D. Laparotomy and surgical drainage

Page 7

264
----------------------------------------
21. A 38-year-old male presents to the clinic with complaints of dull right upper quadrant pain for the
past month. He has been on a visit to South Africa, and the pain has occurred after his visit. An
ultrasound was performed, which revealed the following findings. He was diagnosed with a hydatid
cyst. What is the Gharbi classification for this patient’s cyst?
(or)
What is the Gharbi classification for this patient’s hydatid cyst?

A. Type IV
B. Type III
C. Type I
D. Type II
----------------------------------------
22. A 40-year-old female comes to the clinic complaining of abdominal discomfort and pain. He is a
resident of Australia. Which of the following statements is correct regarding the treatment of hydatid
cysts?
(or)
Which of the following statement is correct regarding the treatment of hydatid cysts?
A. Drug of choice for hydatid cyst is mebendazole
B. PAIR is the most preferred treatment, optionally
C. Surgery is the most effective treatment for small cysts
D. 3% hypertonic saline is most commonly used for PAIR
----------------------------------------
23. A 5-week-old child is brought to the hospital due to two-day history of increasing jaundice. His urine
is dark, and his stools are pale. He was born at 37 weeks of gestation after an uncomplicated
pregnancy through vaginal delivery. The vital signs are normal. The baby is irritable. The jaundice is
present from the face to the lower abdomen. A sub hepatic mass is palpable on abdominal
examination. Which of the following is most likely the diagnosis in this patient?
(or)
A 5-week-old child is brought due to two-day history of increasing jaundice. His urine is dark, and his
stools are pale. The jaundice is present. A Sub hepatic mass is palpable on abdominal examination.
Which of the following is most likely the diagnosis in this patient?

Page 8

265
A. Benign neonatal hyperbilirubinemia
B. Biliary atresia
C. Biliary cyst
D. Congenital infection
----------------------------------------
24. A 45-year-old female presented to the outpatient department with dull aching pain in the right upper
quadrant. Nodular enhancement was seen in the mass lesion on CT. Which of the following is true
about the given condition?
A. Most common malignant tumour of the liver
B. FNAC / Biopsy is contraindicated
C. Commonly associated with Kasabach-Merritt syndrome
D. The treatment of choice is Radical Excision
----------------------------------------
25. A 45-year-old male came to the OPD complaining of right upper quadrant pain. On examination,
hepatomegaly was noted. A triphasic CT scan revealed a hyperdense lesion in the arterial phase,
which becomes hypodense in the delayed phase. AFP levels were found to be elevated. Which of the
following statements is correct regarding this tumour?
(or)
A 45-year-old male came complaining of right upper quadrant pain. Hepatomegaly was noted. A
triphasic CT scan revealed a hyperdense lesion in the arterial phase, which becomes hypodense in the
delayed phase. AFP levels were found to be elevated. Which of the following statements is correct
regarding this tumour?
A. Most common malignancy of the liver
B. Paraneoplastic manifestation includes hypoglycemia, hypercalcemia, and erythrocytosis
C. Surgery is a palliative method that has a survival advantage
D. 50% of tumours are resectable at the time of presentation
----------------------------------------
26. A 23-year-old man complained of abdominal pain for 15 days. An ultrasound of the liver revealed a
hypoechoic, heterogenous mass and enlarging portal vein in the left lobe of the liver. The Hepatitis B
antigen was negative, and serum alpha-fetoprotein was not increased. The results of a CT scan
revealed the following discovery. What is the likely diagnosis?
(or)
A 23-year-old man complained of abdominal pain for 15 days. An ultrasound of the liver revealed a
hypoechoic, heterogenous mass and enlarging portal vein in the left lobe of the liver. The results of a
CT scan revealed the following discovery. What is the likely diagnosis?

Page 9

266
A. Hepatocellular carcinoma
B. Hepatic adenoma
C. Fibrolamellar variant of HCC
D. Hepatoblastoma
----------------------------------------
27. A 48-year-old male is admitted to the ward. He is a chronic alcoholic and has been diagnosed with
liver cirrhosis. He is scheduled for removal of his 9th segment of the liver. Which of the following
marked structure is known as the 9th segment of the liver?
(or)
Which of the following marked structure is known as the 9th segment of the liver?

A. A
B. B
C. C
D. None of the above
----------------------------------------
28. A 48-year-old male is admitted to the ward. He is scheduled for the removal of the segments with
blue shades. What is the name of this procedure?
(or)
A 48-year-old male is admitted to the ward. He is a chronic alcoholic and has been diagnosed with liver
cirrhosis. He is scheduled for the removal of the segments with blue shades. What is the name of this
procedure?

Page 10

267
A. Left hepatectomy
B. Left trisectionectomy
C. Right trisectionectomy
D. Extended right hepatectomy
----------------------------------------
29. A 40-year-old chronic alcoholic reported right upper quadrant discomfort and audible bruit on
examination. AFP was more than 20 mg/mL, with arterial enhancement on CT. Which of the following
statement is most appropriate regarding treatment options for the diagnosis?
(or)
Which of the following statements is most appropriate regarding treatment options for the diagnosis of a
patient with right upper quadrant discomfort with AFP>20 mg/dl and arterial enhancement on CT?
A. Complete excision by partial or total hepatectomy & transplantation is the only treatment modality
with curative potential
B. Sorafenib is given for metastatic and unresectable cases
C. Liver transplantation is the definitive treatment
D. Only 15-20 % of cases have an unresectable tumour
----------------------------------------
30. A 38-year-old male is admitted to the ward. He is a chronic alcoholic and has been diagnosed with
liver cirrhosis. He is scheduled for surgery. During the surgery, the following manoeuvre is performed.
Which of the following statements is most appropriate regarding the manoeuvre shown in the image?
(or)
A 38-year-old male is admitted with liver cirrhosis. Which of the following statements is most
appropriate regarding the manoeuvre shown in the image?

Page 11

268
A. Clamping in Foramen of Winslow occludes Portal triad and Inferior vena cava
B. The hepatoduodenal ligament is clamped to occlude hepatic blood flow from the portal vein and
hepatic vein
C. Allows identification of an accessory or replaced right hepatic artery
D. It can control bleeding from the Inferior vena cava and Hepatic Vein
----------------------------------------
31. A 35-year-old male presents to the emergency department with complaints of the right-sided upper
back and flank pain, which he attributes to a "cupping" procedure he had the previous day. On further
history, he reported having had approximately 6 weeks of intermittent fevers, cough, anorexia, and
general malaise. On physical examination, diffuse tenderness in RUQ was noted along with icteric
sclera. USG showed multiple hypoechoic, loculated fluid collections within the parenchyma of the liver,
consistent with hepatic abscesses. All of the following statements about the diagnosis of the given case
are true, except?
(or)
A 35-year-old male presents with right-sided upper back and flank pain. He reported having had
approximately 6 weeks of intermittent fevers, cough, anorexia, and general malaise. USG showed
multiple hypoechoic, loculated fluid collections within the parenchyma of the liver, consistent with
hepatic abscesses. The following statements about the diagnosis of the given case are correct, except?
A. Most common route of infection is bile duct
B. Most common organism responsible is E. coli
C. Most common symptom is jaundice
D. Most common LFT abnormality is raised ALP
----------------------------------------
32. A 2-year-old child is admitted to the surgical department for liver transplantation. What is the most
common indication of a pediatric liver transplant?
(or)
What is the most common indication of a pediatric liver transplant?
A. Hepatoblastoma
B. Biliary atresia
C. Viral hepatitis
D. HCV-induced cirrhosis

Page 12

269
----------------------------------------
33. Which of the following explanations accounts for the fact that hepatitis C is the most common cause
of posttransfusion hepatitis?
(or)
Which of the following explanations accounts for the fact that hepatitis C is the most common cause of
post-transfusion hepatitis?
A. Because HCV is transmitted efficiently through occupational exposures to blood
B. Blood infected with the hepatitis C virus (HCV) cannot be eliminated through routine testing
C. Current serologic tests for HCV antigens do not exclude carriers
D. The risk for transmission is approximately 1 per 1 million donations
----------------------------------------
34. The liver is divided into right and left lobes and contains 8 Couinaud segments; concerning hepatic
anatomy, the falciform ligament divides which of the following two components of the liver?
A. Caudate lobe, quadrate lobe
B. Right lobe, left lobe
C. Left medial segment, left lateral segment
D. Left medial segment, right lobe
----------------------------------------
35. A 35-year-old female came to the OPD complaining of generalized pruritus, fatigue, and jaundice.
Her liver function tests revealed elevated bilirubin and alkaline phosphatase levels. Upon further
evaluation, the patient was diagnosed with primary biliary cirrhosis. What is the most common symptom
of primary biliary cirrhosis?
(or)
The most common symptom in primary biliary cirrhosis is which of the following?
A. Jaundice
B. Pruritus
C. Melanosis
D. Vomiting
----------------------------------------
36. Given below are two statements, one labelled as Assertion (A) and the other labelled as Reason
(R): Assertion (A): In blunt trauma Liver, CT is the investigation of choice in a stable patient. Reason
(R): Pringle’s manoeuvre is used to manage liver trauma. In the context of the above two statements,
which one of the following is correct?
A. Both A and R are true, and R is the correct explanation of A
B. Both A and R are true, but R is not a correct explanation of A
C. A is true, but R is false
D. A is false, but R is true
----------------------------------------

Page 13

270
Correct Answers
Question Correct Answer

Question 1 1
Question 2 3
Question 3 2
Question 4 3
Question 5 4
Question 6 4
Question 7 2
Question 8 3
Question 9 1
Question 10 1
Question 11 4
Question 12 4
Question 13 4
Question 14 1
Question 15 1
Question 16 2
Question 17 1
Question 18 1
Question 19 3
Question 20 3
Question 21 2
Question 22 2
Question 23 3
Question 24 2
Question 25 2
Question 26 3
Question 27 2
Question 28 2
Question 29 3
Question 30 3
Question 31 3
Question 32 2

Page 14

271
Question 33 4
Question 34 3
Question 35 2
Question 36 2

Solution for Question 1:


Option A: E-FAST
• This patient, with multiple new bruises on the abdomen due to multiple forceful punches, is a case of
blunt abdominal injury
• Extended Focused Assessment with Sonography in Trauma (E-FAST)
• It is an ultrasound type used as a next step investigation for trauma cases where CT is not practical
due to time constrain
• It is very useful in viewing the following areas: the right upper quadrant, perisplenic space in the left
lower quadrant, pelvis, subcostal or cardiac view, and the parietal or visceral pleural movements.
Option C: CT scan of abdomen.
• It is the most commonly used and better investigation for abdominal trauma.
• It is useful in blunt, penetrating trauma, pancreatic, spleen, liver, duodenal, and retroperitoneal
injuries.
• Smaller injuries and early hemoperitoneum are detected easily with CT scan.
• It is noninvasive, highly specific, and highly accurate, with low false positive/ low false negative
results. It is done for more detailed view.
Option B: Abdominal ultrasound
• In blunt abdominal injury, the CT scan is the most sensitive and specific test than abdominal
ultrasound.
Option D: Peritoneal lavage
• Intraperitoneal lavage is performed if the patient is unstable (Systolic BP less than 90 mmHg) and the
EFAST is not available.
• Peritoneal lavage can be done when there is a high suspicion of intraperitoneal bleeding.

Solution for Question 2:


Option C: Hydatid cyst
• Hydatid cyst, an infection is caused by the tapeworm Echinococcus granulosus in its larval or cyst
stage.
• Life cycle : Dog intestine (definitive host) → Ova ingested by humans (accidental intermediate host) →
Portal vein → Liver
• The Common lobe affected is the Right lobe (anterior inferior of inferior posterior segments).
• Symptoms are Dull right upper quadrant pain and Abdominal distension.

Page 15

272
• Eosinophilia is present in 30% of patients.
• The Complications are Infection that may involve other organs, and rupture of the cyst (allergic or
anaphylactic reaction).
• Diagnosis includes Serology (ELISA test for echinococcal antigens).
• ELISA test for echinococcal antigens is positive in 85% of patients.
• The test will be negative if the cyst has not leaked or does not contain scolices or if the parasite is no
longer viable.
• Ultrasound and CT scan show the following: Well-defined hypodense lesions with a distinct wall
Budding signs of the cyst membrane contain free-floating hyperechoic hydatid sand Multiloculated cyst
Floating membrane Rosette's appearance (when daughter cysts are present) Ring-like calcification of
the pericytes A calcified cyst ( dead or inactive cyst)
• Well-defined hypodense lesions with a distinct wall
• Budding signs of the cyst membrane contain free-floating hyperechoic hydatid sand
• Multiloculated cyst
• Floating membrane
• Rosette's appearance (when daughter cysts are present)
• Ring-like calcification of the pericytes
• A calcified cyst ( dead or inactive cyst)
• Well-defined hypodense lesions with a distinct wall
• Budding signs of the cyst membrane contain free-floating hyperechoic hydatid sand
• Multiloculated cyst
• Floating membrane
• Rosette's appearance (when daughter cysts are present)
• Ring-like calcification of the pericytes
• A calcified cyst ( dead or inactive cyst)

Option A: Polycystic liver disease


• Ultrasound imaging will show hyperechoic areas in a subcapsular portion of the liver.
• CT scan and MRI shows hyperdense well-circumscribed lesions.
Option B: Amoebic liver abscess
• The amoebic liver abscess typically has high serum transaminase, Leukocytosis, and elevated
Alkaline phosphatase.
• On imaging, the amoebic liver abscess will be found in the right lobe.
• Ultrasound shows hypoechoic mass.
• CT Scan shows low-density mass with the peripheral enhancing rim.
Option D: Pyogenic liver abscess
• Liver abscess presents with a high fever.

Page 16

273
• On imaging, the pyogenic liver disease shows a Centrally non-enhancing abscess cavity with an
enhancing capsule.
• This is usually a contrast enhancement pattern.

Solution for Question 3:


Option B: Milan’s criteria
• Milan's criteria are used for Liver transplantation
• To assess if a patient with HCC can proceed to Liver transplantation
• This criteria is used to predict the risk of recurrent disease after transplantation Patients who meet the
criteria- the risk of recurrence <20% Patients outside the criteria - the risk of recurrence- 60%
• Patients who meet the criteria- the risk of recurrence <20%
• Patients outside the criteria - the risk of recurrence- 60%
• Currently used to define acceptable candidates for transplantation in the United States
• After the Liver transplant, the patient has a 4-year survival of 80%.
• Patients who meet the criteria- the risk of recurrence <20%
• Patients outside the criteria - the risk of recurrence- 60%

Option A: Ranson criteria


• Ranson criteria are used to predict the severity and mortality of acute pancreatitis.
• Five parameters are assessed on admission, and the other six are assessed 48 hours after
admission.
Option C: Eagle's criteria
• The eagle score is a 5 points scoring system used for the risk of vascular events in cardiac patients.
• The Eagle criteria include age over 70 years, angina, diabetes, Q wave on ECG, and history of
congestive heart failure to predict adverse cardiac events following major vascular surgery.
Option D: Rockall criteria
• Rockall scores are designed to combine information such as the patient's age, the occurrence of
shock assessed from systolic blood pressure records and pulse rate, and the presence and severity of
co-morbid conditions to assess UGI bleeding.

Solution for Question 4:


Option C: Class C
• The total score for the given patient amounts to 2+2+2+2+3=11 points, and hence he belongs to CTP
Class C
• Child-Turcotte-Pugh (CTP) Scoring System and this patient have Grade 3 encephalopathy.

Page 17

274
• Child-Turcotte-Pugh (CTP) Score:
• Class A 5-6 points
• Class B 7-9 points
• Class C 10-15 points
Option A: Class A
• Class A has points 5-6. This patient's point is 11, so the class should be C.
Option B: Class B
• Class B has points 7-9. This patient's point is 11, so the class should be C.
Option D: Class D
• There are a total of three classes (Class A, B, and C) in the child Pugh scoring system.

Solution for Question 5:


Option D: Primary sclerosing cholangitis
• This patient with right upper quadrant pain, fever, jaundice, hypotension, and altered mental status
(i.e., Reynolds pentad) has the typical features of acute cholangitis.
• Acute cholangitis is an infection of the extrahepatic biliary system that usually occurs due to biliary
obstruction (e.g., bile stone, malignancy, stricture), which predisposes patients to bacterial invasion of
the normally sterile biliary tree.
• In a young adult male with a history of hematochezia, the most likely etiology of recurrent acute
cholangitis (e.g., similar symptoms in the past) is a biliary stricture due to primary sclerosing cholangitis
(PSC).
• Primary Sclerosing Cholangitis is a chronic disease characterized by fibrosis and stricture of the
medium and large intra- and extrahepatic bile ducts.
• This promotes cholestasis (elevated alkaline phosphatase, hyperbilirubinemia) and obstruction. PSC
usually affects men and is associated with ulcerative colitis, which explains this patient's hematochezia.
Patients may have normal ultrasonography because intrahepatic bile ducts are not easily visible.
Magnetic resonance cholangiopancreatography overcomes this limitation and is the preferred test to
confirm the diagnosis in these patients.
• Management of PSC includes endoscopic interventions for strictures, therapy for coexisting ulcerative
colitis, antibiotics for cholangitis (with some patients maintained on long-term antibiotic therapy), and, in
some cases, ursodeoxycholic acid.
Option A: Annular pancreas
• Annular pancreas is a congenital disorder that results in a ring-like pancreas that encircles the
duodenum. It can cause duodenal obstruction (e.g., vomiting, pain, early satiety).
• But acute cholangitis is atypical.
Option B: Cholangiocarcinoma
• It usually develops in older patients (e.g., age >50), and unexpected weight loss is typical.
Option C: Chronic pancreatitis

Page 18

275
• Chronic pancreatitis presents with recurrent abdominal pain, vomiting, and steatorrhea. It typically
occurs in patients with risk factors (e.g. Smoking, alcohol use).

Solution for Question 6:


Option D: Liver transplantation for progressive disease
• These clinical features are suggestive of Polycystic liver disease.
• It is common in patients with inherited autosomal dominant adult polycystic kidney disease.
• Treatmentfor severe symptoms related to large cysts and complications includes: Percutaneous
aspiration with or without sclerotherapy Cyst fenestration or surgical unroofing of the cyst Hepatic
resection Orthotopic liver transplantation (for progressive disease)
• Percutaneous aspiration with or without sclerotherapy
• Cyst fenestration or surgical unroofing of the cyst
• Hepatic resection
• Orthotopic liver transplantation (for progressive disease)
• Percutaneous aspiration with or without sclerotherapy
• Cyst fenestration or surgical unroofing of the cyst
• Hepatic resection
• Orthotopic liver transplantation (for progressive disease)
Option A: Somatostatin as an initial treatment for this condition
• Somatostatin is given for severe diseases with large cysts and complications.
Option B: It is not associated with hypertension
• These liver cysts are associated with Polycystic kidney disease.
• In Polycystic kidney disease, there is increased secretion of renin which is associated with high blood
pressure due to vasoconstriction.
Option C: Serum calcium is low
• Serum calcium is normal in Polycystic kidney and liver disease.

Solution for Question 7:


Option B: PIVKA-2
• Tumour markers most commonly used for the detection and management of hepatocellular
carcinoma: AFP will be elevated (seen in 70% of HCC) Increase in PIVKA-2 (Protein Induced by
Vitamin-K Absence 2 ) Immunohistochemical markers such as Hepatocyte 1, Glypican 3, Arginase 1,
and Heat shock protein (HSP 70) can also be used to diagnose HCC.
• AFP will be elevated (seen in 70% of HCC)
• Increase in PIVKA-2 (Protein Induced by Vitamin-K Absence 2 )

Page 19

276
• Immunohistochemical markers such as Hepatocyte 1, Glypican 3, Arginase 1, and Heat shock protein
(HSP 70) can also be used to diagnose HCC.
• AFP will be elevated (seen in 70% of HCC)
• Increase in PIVKA-2 (Protein Induced by Vitamin-K Absence 2 )
• Immunohistochemical markers such as Hepatocyte 1, Glypican 3, Arginase 1, and Heat shock protein
(HSP 70) can also be used to diagnose HCC.
Option A: CEA
• Carcinoembryonic antigen is a protein found in very low levels in the blood; it is increased in colorectal
cancer.
Option C: ALP
• The alkaline phosphate test measures the amount of ALP in the blood; it is commonly used in
diagnosing liver and Blood disorders.
Option D: CA 125
• CA 125 is commonly raised in ovarian carcinoma.

Solution for Question 8:


Option C: Viral hepatitis B
• Risk factors for hepatocellular carcinoma (order of frequency from greatest to smallest): Viral hepatitis
B Viral hepatitis C Alcoholic cirrhosis Smoking Aflatoxin produced by Aspergillus species Vinyl chloride
Oral contraceptive pill use NASH (Non-alcoholic steatohepatitis) Hemochromatosis α1 Antitrypsin
deficiency disease Copper deficiency disease
• Viral hepatitis B
• Viral hepatitis C
• Alcoholic cirrhosis
• Smoking
• Aflatoxin produced by Aspergillus species
• Vinyl chloride
• Oral contraceptive pill use
• NASH (Non-alcoholic steatohepatitis)
• Hemochromatosis
• α1 Antitrypsin deficiency disease
• Copper deficiency disease
• Viral hepatitis B
• Viral hepatitis C
• Alcoholic cirrhosis
• Smoking

Page 20

277
• Aflatoxin produced by Aspergillus species
• Vinyl chloride
• Oral contraceptive pill use
• NASH (Non-alcoholic steatohepatitis)
• Hemochromatosis
• α1 Antitrypsin deficiency disease
• Copper deficiency disease
Option A: Fair skin(skin type I and II)
• It is a risk factor for Basal cell carcinoma
• Other factors include Intermittent UV exposure Personal history of BCC Prior treatment with ionizing
radiation
• Intermittent UV exposure
• Personal history of BCC
• Prior treatment with ionizing radiation
• Intermittent UV exposure
• Personal history of BCC
• Prior treatment with ionizing radiation
Option B: Barrett's Esophagus
• It is a risk factor for Esophageal adenocarcinoma
• Other factors include Acid-peptic disorders Motor disorders of the esophagus Other malignancies
Medications, environmental exposures Diet Nutrition
• Acid-peptic disorders
• Motor disorders of the esophagus
• Other malignancies
• Medications, environmental exposures
• Diet
• Nutrition
• Acid-peptic disorders
• Motor disorders of the esophagus
• Other malignancies
• Medications, environmental exposures
• Diet
• Nutrition
Option D: Human papillomavirus(HPV)
• It is a risk factor for Cervical cancer

Page 21

278
• Other factor includes Smoking Weak Immune system Having Multiple Sexual partners Long term Use
of birth control pill
• Smoking
• Weak Immune system
• Having Multiple Sexual partners
• Long term Use of birth control pill
• Smoking
• Weak Immune system
• Having Multiple Sexual partners
• Long term Use of birth control pill

Solution for Question 9:


Option A: Budd Chiari Syndrome
• The etiology is Hepatic venous outflow obstruction.
• It is usually due to Myeloproliferative disorder (e.g., Polycythemia Vera), Malignancy (e.g.,
hepatocellular carcinoma), and Oral contraception use/pregnancy.
• The acute symptoms are Jaundice, variceal bleeding.
• The manifestations are Subacute/chronic Vague, progressive abdominal pain, hepatomegaly,
splenomegaly, ascites, Mild/moderate elevation in bilirubin, and transaminases.
• Abdominal Doppler ultrasound shows decreased hepatic vein flow.
Option B: Hepatic hemangioma
• Hepatic hemangioma, the most common benign liver tumour, is often found incidentally during
imaging for other conditions.
Option C: Hepatic adenomas
• Hepatic adenomas are benign liver tumours that often arise in the right lobe of the liver in women on
oral contraception (possibly due to the effects of estrogen on hepatocytes, although studies have been
inconclusive). However, centripetal enhancement is usually seen on a triphasic CT scan, and no central
scar would be present. Scanning images ruled out Hepatic adenomas with normal liver texture.
Option D: Hepatocellular carcinoma
• Hepatocellular carcinoma is a malignant liver tumour that usually arises in the setting of chronic liver
disease (e.g., cirrhosis, viral hepatitis).
• Patients are often asymptomatic but may develop weight loss or early satiety.

Solution for Question 10:


Option A: Portal plate

Page 22

279
• Portal plate is not a part of the capsular plate.
• Fusion of the Glisson capsule with the connective tissue sheaths surrounding the biliary and vascular
elements at the inferior aspect of the liver constitutes the plate system.
Option B: Hilar plate
• The hilar plate is also called a hatched area.
• It covers the elements of the hilum and is in continuity with a gallbladder plate on the right and an
umbilical plate on the left.
Option C: Umbilical plate
• The umbilical plate is situated above the umbilical portion of the left portal vein.
Option D: Cystic plate
• The cystic plate is related to the gallbladder.

Solution for Question 11:


Option D: Simple cyst

• Hepatic cysts are typically discovered incidentally and are almost always asymptomatic. They can
demonstrate slow growth over time, although rapid size increases may be caused by internal
hemorrhage.
• Simple hepatic cyst contains serous fluid, they do not communicate with the biliary tree and do not
have septations.
• Most common complication is Intracystic bleeding.
Option A: Hydatid cyst
• Hydatid cysts result from infection by the Echinococcus tapeworm species and can result in cyst
formation anywhere in the body.

Page 23

280
• The cysts usually have three components. Pericyst: composed of inflammatory tissue of host origin
Exocyst Endocyst: Scolices (the larval stage of the parasite) and the laminated membrane are
produced here
• Pericyst: composed of inflammatory tissue of host origin
• Exocyst
• Endocyst: Scolices (the larval stage of the parasite) and the laminated membrane are produced here
• Pericyst: composed of inflammatory tissue of host origin
• Exocyst
• Endocyst: Scolices (the larval stage of the parasite) and the laminated membrane are produced here

Option B: Caroli’s disease


• Caroli disease and Caroli syndrome are congenital disorders comprising of multifocal cystic dilatation
of segmental intrahepatic bile ducts.

Page 24

281
Option C: Liver abscess
• The typical presentation is one of right upper quadrant pain, fever, and jaundice, anorexia, malaise,
and weight loss are also frequently seen.

Solution for Question 12:


Option D: OKUDA
• OKUDA staging system is a hepatocellular staging classification that describes both cancer-related
variables and liver function variables.
• OKUDA Classification:
• Stages of OKUDA: Stage I: 0 points Stage II: 1-2 points Stage III: 3-4 points
• Stage I: 0 points
• Stage II: 1-2 points
• Stage III: 3-4 points
• Stage I: 0 points
• Stage II: 1-2 points
• Stage III: 3-4 points
Option A: MELD
• MELD Score (The Model of End-stage Liver Disease) is used to estimate the relative disease severity
and likely survival of patients awaiting liver transplantation. The score is based on a formula that
considers bilirubin and creatinine levels and the international normalized ratio (INR).
Option B: METAVIR
• METAVIR(Meta-analysis of histological data in viral hepatitis).
• It is used to evaluate the severity of fibrosis caused by hepatitis C.

Page 25

282
• The grade indicates the amount of inflammation while stages describes the amount of scarring and
fibrosis.

Option C: ERCP
• ERCP(Endoscopic retrograde cholangiopancreatography) is used to diagnose and treat biliary and
pancreatic ductal system, such as gallstones, malignant strictures
• This technique uses endoscopy and fluoroscopy.

Solution for Question 13:


Option D: Colonoscopy
• The patient with a history of weight loss, and his imaging shows multiple liver lesions on the CT scan,
suggesting metastatic disease to the liver rather than primary liver cancer.
• Colorectal cancer is the most common source of liver metastasis because blood from the Colon
moves through portal circulation directly to the liver, so this pathology should be excluded in the patient
even in the absence of specific symptoms, alteration in bowel habits, or gastrointestinal bleeding.
• Colonoscopy is the next best Diagnostic step as it both localizes that tumour and provides the tissue
diagnosis.
• Lung and breast cancer also metastasize to the liver, although they are less common than
Gastrointestinal malignancies.
• This patient is a smoker, so a chest X-ray should also be done, although he has no respiratory
symptoms like cough.
Option A: Alpha-fetoprotein measurement
• Alpha-fetoprotein measurement is used along with imaging studies in the diagnosis of hepatocellular
carcinoma, which most commonly occurs in the setting of liver chronic liver disease.
• It presents as a solitary mass.
Option B: Aspiration and culture followed by antibiotic therapy
• In Pyogenic liver abscess, aspiration and culture followed by antibiotic therapy should be done.
• Pyogenic liver abscesses present with fevers, right upper quadrant pain, and weight loss.
Option C: Prostate-specific antigen
• Prostate-specific antigen measurement is a Tumor Marker for prostate cancer. Prostate cancer tends
to be an indolent malignancy that commonly metastasizes to the pelvic lymph nodes and bones rather
than the liver.

Solution for Question 14:


Option A: Hepatic adenoma
• This woman, who had been using OCP'S regularly and is using steroids for athletic performance, has
a hyperechoic mass with the left lobe of the liver with no irregular margins indicates a hepatic adenoma.

Page 26

283
• Hepatic adenoma: Adenomas are benign solid liver tumours seen most commonly in premenopausal
women aged older than 30 years. Benign proliferation of hepatocytes is seen Associated with OCP
usage. The microscopic exam usually reveals no sign of malignancy, but the cells contain abundant
glycogen or fat. The liver architecture is absent, and one may not visualize central veins, bile ducts, or
portal tracts. As bile ductules are not observed, the normal architecture of the liver is absent in these
lesions Mostly symptomatic- abdominal pain CT - sharply defined borders and hence confused with
metastatic tumours Venous phase - hypodense or isodense Arterial phase - hypervascular
enhancement CECT - peripheral enhancement with centripetal progression Tumour markers are
normal Complication - spontaneous rupture with intraperitoneal bleeding, risk of malignant
transformation Large adenomas (>4-5 cm) can be surgically resected
• Adenomas are benign solid liver tumours seen most commonly in premenopausal women aged older
than 30 years.
• Benign proliferation of hepatocytes is seen
• Associated with OCP usage.
• The microscopic exam usually reveals no sign of malignancy, but the cells contain abundant glycogen
or fat. The liver architecture is absent, and one may not visualize central veins, bile ducts, or portal
tracts.
• As bile ductules are not observed, the normal architecture of the liver is absent in these lesions
• Mostly symptomatic- abdominal pain
• CT - sharply defined borders and hence confused with metastatic tumours Venous phase -
hypodense or isodense Arterial phase - hypervascular enhancement CECT - peripheral enhancement
with centripetal progression
• Venous phase - hypodense or isodense
• Arterial phase - hypervascular enhancement
• CECT - peripheral enhancement with centripetal progression
• Tumour markers are normal
• Complication - spontaneous rupture with intraperitoneal bleeding, risk of malignant transformation
• Large adenomas (>4-5 cm) can be surgically resected
• Adenomas are benign solid liver tumours seen most commonly in premenopausal women aged older
than 30 years.
• Benign proliferation of hepatocytes is seen
• Associated with OCP usage.
• The microscopic exam usually reveals no sign of malignancy, but the cells contain abundant glycogen
or fat. The liver architecture is absent, and one may not visualize central veins, bile ducts, or portal
tracts.
• As bile ductules are not observed, the normal architecture of the liver is absent in these lesions
• Mostly symptomatic- abdominal pain
• CT - sharply defined borders and hence confused with metastatic tumours Venous phase -
hypodense or isodense Arterial phase - hypervascular enhancement CECT - peripheral enhancement
with centripetal progression
• Venous phase - hypodense or isodense
• Arterial phase - hypervascular enhancement

Page 27

284
• CECT - peripheral enhancement with centripetal progression
• Tumour markers are normal
• Complication - spontaneous rupture with intraperitoneal bleeding, risk of malignant transformation
• Large adenomas (>4-5 cm) can be surgically resected
• Venous phase - hypodense or isodense
• Arterial phase - hypervascular enhancement
• CECT - peripheral enhancement with centripetal progression
Option B: Hepatocellular carcinoma
• In Hepatocellular carcinoma, the tumour marker AFP is raised, and the CT scan will demonstrate
hypervascularization, and the mass is not well differentiated like in hepatic adenoma. The patient also
has risk factors for Hepatic adenoma.
Option C: Focal nodular hyperplasia
• FNH on CECT will demonstrate Centrifugal enhancement, not a centripetal enhancement, and a
hypoattenuating scar in the middle of the tumour is visible.
Option D: Metastatic colorectal cancer
• Metastatic colorectal cancer would have demonstrated masses that are many, and the patient would
have some features of CRC, like blood in the stool or obstruction due to the mass.

Solution for Question 15:


Option A: Produced by maternal hepatocytes during Pregnancy
• AFP is produced by fetal hepatocytes and yolk sac, not maternal hepatocytes, during Pregnancy.
• Alpha-fetoprotein (AFP) Tumor marker most commonly used for the detection and management of
hepatocellular carcinoma The oncofetal protein is produced by fetal hepatocytes and yolk-sac Elevated
in utero, falls to low levels after birth and again elevates during Pregnancy Conditions with elevated
levels of AFP: Hepatocellular carcinoma Tumors of gonadal origin (Nonseminomatous testicular tumor)
Gastric cancer Pregnancy Acute or chronic hepatitis Inflammatory bowel disease
• Tumor marker most commonly used for the detection and management of hepatocellular carcinoma
• The oncofetal protein is produced by fetal hepatocytes and yolk-sac
• Elevated in utero, falls to low levels after birth and again elevates during Pregnancy
• Conditions with elevated levels of AFP: Hepatocellular carcinoma Tumors of gonadal origin
(Nonseminomatous testicular tumor) Gastric cancer Pregnancy Acute or chronic hepatitis Inflammatory
bowel disease
• Hepatocellular carcinoma
• Tumors of gonadal origin (Nonseminomatous testicular tumor)
• Gastric cancer
• Pregnancy
• Acute or chronic hepatitis

Page 28

285
• Inflammatory bowel disease
• Tumor marker most commonly used for the detection and management of hepatocellular carcinoma
• The oncofetal protein is produced by fetal hepatocytes and yolk-sac
• Elevated in utero, falls to low levels after birth and again elevates during Pregnancy
• Conditions with elevated levels of AFP: Hepatocellular carcinoma Tumors of gonadal origin
(Nonseminomatous testicular tumor) Gastric cancer Pregnancy Acute or chronic hepatitis Inflammatory
bowel disease
• Hepatocellular carcinoma
• Tumors of gonadal origin (Nonseminomatous testicular tumor)
• Gastric cancer
• Pregnancy
• Acute or chronic hepatitis
• Inflammatory bowel disease
• Hepatocellular carcinoma
• Tumors of gonadal origin (Nonseminomatous testicular tumor)
• Gastric cancer
• Pregnancy
• Acute or chronic hepatitis
• Inflammatory bowel disease
Option B: Conditions with elevated levels of AFP are HCC, Non-seminomatous testicular tumour, Preg
nancy, and Gastric cancer
• The conditions with elevated AFP are Gastric cancer, HCC, non-seminomatous testicular cancer,
Pregnancy, hepatitis, and cirrhosis
Option C: Levels >400 ng/mL with arterial enhancement on CT are suggestive of HCC
• Levels >400ng/ml of AFP are suggestive of HCC. On CT scan, HCC demonstrates Arterial
enhancement and a quick wash-out phase
Option D: Normal adult serum AFP level is <20 ng/mL
• The normal adult level of the AFP is <20 ng/ml, and in HCC, it gets raised greater than 400, which
helps it to diagnose HCC in most cases.

Solution for Question 16:


Option B: Hepatic adenoma
• There are no characteristic radiological features to differentiate hepatic adenomas from malignant
tumours. These tumours are also thought to have malignant potential, and hence resection is,
therefore, the treatment of choice for hepatic adenomas.
• Hepatic adenoma: Adenomas are benign solid liver tumours seen most commonly in premenopausal
women aged older than 30 years. Benign proliferation of hepatocytes It is typically solitary and

Page 29

286
associated with OCP use Histology: The microscopic exam usually reveals no sign of malignancy, but
the cells contain abundant glycogen or fat. The liver architecture is absent, and one may not visualize
central veins, bile ducts, or portal tracts Mostly symptomatic- abdominal pain CT - sharply defined
borders and hence confused with metastatic tumours Venous phase - hypodense or isodense Arterial
phase - hypervascular enhancement CECT - peripheral enhancement with centripetal progression
Tumour markers are normal Complication - spontaneous rupture with intraperitoneal bleeding, risk of
malignant transformation Large adenomas (>4-5 cm) are surgically resected
• Adenomas are benign solid liver tumours seen most commonly in premenopausal women aged older
than 30 years.
• Benign proliferation of hepatocytes
• It is typically solitary and associated with OCP use
• Histology: The microscopic exam usually reveals no sign of malignancy, but the cells contain
abundant glycogen or fat. The liver architecture is absent, and one may not visualize central veins, bile
ducts, or portal tracts
• The microscopic exam usually reveals no sign of malignancy, but the cells contain abundant glycogen
or fat. The liver architecture is absent, and one may not visualize central veins, bile ducts, or portal
tracts
• Mostly symptomatic- abdominal pain
• CT - sharply defined borders and hence confused with metastatic tumours Venous phase -
hypodense or isodense Arterial phase - hypervascular enhancement
• Venous phase - hypodense or isodense
• Arterial phase - hypervascular enhancement
• CECT - peripheral enhancement with centripetal progression
• Tumour markers are normal
• Complication - spontaneous rupture with intraperitoneal bleeding, risk of malignant transformation
• Large adenomas (>4-5 cm) are surgically resected
• Adenomas are benign solid liver tumours seen most commonly in premenopausal women aged older
than 30 years.
• Benign proliferation of hepatocytes
• It is typically solitary and associated with OCP use
• Histology: The microscopic exam usually reveals no sign of malignancy, but the cells contain
abundant glycogen or fat. The liver architecture is absent, and one may not visualize central veins, bile
ducts, or portal tracts
• The microscopic exam usually reveals no sign of malignancy, but the cells contain abundant glycogen
or fat. The liver architecture is absent, and one may not visualize central veins, bile ducts, or portal
tracts
• Mostly symptomatic- abdominal pain
• CT - sharply defined borders and hence confused with metastatic tumours Venous phase -
hypodense or isodense Arterial phase - hypervascular enhancement
• Venous phase - hypodense or isodense
• Arterial phase - hypervascular enhancement

Page 30

287
• CECT - peripheral enhancement with centripetal progression
• Tumour markers are normal
• Complication - spontaneous rupture with intraperitoneal bleeding, risk of malignant transformation
• Large adenomas (>4-5 cm) are surgically resected
• The microscopic exam usually reveals no sign of malignancy, but the cells contain abundant glycogen
or fat. The liver architecture is absent, and one may not visualize central veins, bile ducts, or portal
tracts
• Venous phase - hypodense or isodense
• Arterial phase - hypervascular enhancement

Option A: Hemangioma
• Hemangioma is the most common benign tumour of the liver.
• Cavernous hemangiomas are associated with focal nodular hyperplasia.
• Usually solitary and <5 cm in diameter
• Most patients remain asymptomatic (if tumour size < 5cm size)
• If size >5cm → Giant hemangioma
• Spontaneous rupture is exceedingly rare
• Liver function tests and tumour markers are usually normal
• CT shows a typical peripheral nodular enhancement pattern
• Isotope labelled red blood cell scan is an accurate test
• Malignant transformation is rare
• FNAC and Biopsy→ Contraindicated (because it is a vascular tumour)
• Asymptomatic patients- observation
Option C: Focal nodular hyperplasia
• Second most common benign tumour of the liver in young women
• This may be due to a hyperplastic response to an anomalous artery
• Characterized by central fibrous scar with radiating septa, although no central scar is seen in
approximately 15% of cases
• The central scar often contains a large artery that branches out into multiple smaller arteries in a
spoke wheel pattern
• IOC– CECT (strong hypervascularity in the arterial phase with central non-enhancing scar)
• Asymptomatic patients with typical radiologic features do not require treatment
• Treatment for patients with persistent symptomatic FNH or an enlarging mass or in case of diagnostic
uncertainty - Resection
Option D: Hepatic cyst
• Simple hepatic cysts are one of the commonest liver lesions, occurring in ~5% (range 2-7%) of the
population

Page 31

288
• Hepatic cysts are typically discovered incidentally and are almost always asymptomatic
• On CT, a hepatic cyst demonstrates homogeneous hypoattenuation
• The treatment options for small cysts are percutaneous aspiration
• The cysts that can't be treated with percutaneous aspiration because of their size are surgically
removed

Solution for Question 17:


Option A: Hepatoblastoma
• This 14-month-old age patient with a liver mass and biopsy showing medium-sized cells in a
trabecular pattern resembling the fetal hepatocytes indicates that the tumour is Hepatoblastoma
• Hepatoblastoma Most common primary malignancy of the liver in children Low birth weight increases
the risk The median age at presentation is 18 months, and all patients <3yrs of age Manifests as an
asymptomatic mass Mild anemia and thrombocytosis are commonly found at presentation Serum AFP
elevated in 85% to 90% of cases and can serve as a useful marker for therapeutic response
Histologically consists predominantly of immature liver cells resembling fetal or embryonal hepatocytes
Management- Neo Adjuvant Chemotherapy (NACT) + Resection is done 50% of patients with
pulmonary metastasis can be cured with resection of the hepatic tumour and chemotherapy or
resection of the pulmonary metastasis
• Most common primary malignancy of the liver in children
• Low birth weight increases the risk
• The median age at presentation is 18 months, and all patients <3yrs of age
• Manifests as an asymptomatic mass
• Mild anemia and thrombocytosis are commonly found at presentation
• Serum AFP elevated in 85% to 90% of cases and can serve as a useful marker for therapeutic
response
• Histologically consists predominantly of immature liver cells resembling fetal or embryonal
hepatocytes
• Management- Neo Adjuvant Chemotherapy (NACT) + Resection is done 50% of patients with
pulmonary metastasis can be cured with resection of the hepatic tumour and chemotherapy or
resection of the pulmonary metastasis
• 50% of patients with pulmonary metastasis can be cured with resection of the hepatic tumour and
chemotherapy or resection of the pulmonary metastasis
• Most common primary malignancy of the liver in children
• Low birth weight increases the risk
• The median age at presentation is 18 months, and all patients <3yrs of age
• Manifests as an asymptomatic mass
• Mild anemia and thrombocytosis are commonly found at presentation
• Serum AFP elevated in 85% to 90% of cases and can serve as a useful marker for therapeutic
response

Page 32

289
• Histologically consists predominantly of immature liver cells resembling fetal or embryonal
hepatocytes
• Management- Neo Adjuvant Chemotherapy (NACT) + Resection is done 50% of patients with
pulmonary metastasis can be cured with resection of the hepatic tumour and chemotherapy or
resection of the pulmonary metastasis
• 50% of patients with pulmonary metastasis can be cured with resection of the hepatic tumour and
chemotherapy or resection of the pulmonary metastasis
• 50% of patients with pulmonary metastasis can be cured with resection of the hepatic tumour and
chemotherapy or resection of the pulmonary metastasis
Option B: Hepatic adenoma
• Hepatic adenoma is common in middle-aged women who use OCPs or anabolic steroids. The biopsy
is avoided for Hepatic adenoma due to the vascularity of the tumour as it can cause massive bleeding
Option C: Hepatocellular carcinoma
• Although AFP is raised in Hepatocellular carcinoma, HCC only presents in children with some
congenital disorders like Ataxia telangiectasia; otherwise, it mostly presents in older age. The main risk
factors are Alcoholic cirrhosis, non-alcoholic fatty liver disease, and alfa toxin. A biopsy would show a
trabecular pattern with high vascularity
Option D: Hepatic cyst
• Hepatic cysts are benign lesions in the liver that can present in childhood, but USG will show a
homogenous, unilocular lesion in the liver. Biopsy is not done for simple hepatic cysts

Solution for Question 18:


Option A: Serial Ultrasound + Alpha-fetoprotein
• Screening in high-risk cases is done with Serial ultrasound + Alpha-fetoprotein every 6 months.
• It has been found to reduce mortality due to hepatocellular cancer(HCC) by close to 40%
Option B: Serial Liver function tests + Alpha-fetoprotein
• Alpha-fetoproteinis a part of routine screening tests for HCC in high-risk individuals, but liver function
tests are not a part of the screening program.
Option C: Serial Liver function tests + CT scan
• Screening in high-risk cases is done with Serial ultrasound + Alpha-fetoprotein every 6 months.
Option D: Serial Ultrasound + Serial Liver function tests
• Serial ultrasounds are a part of routine screening tests for HCC in high-risk individuals, but liver
function tests are not a part of the screening program.

Solution for Question 19:


Option C: Gradual increase in the size of the abscess

Page 33

290
• The most common consequence of a liver abscess is an abscess rupture into neighbouring organs or
anatomic spaces.
• The most common type of rupture is in the thorax or abdomen.
• Factors prone to rupture: 5 to 10 cm in diameter Abscess gradually increasing in size The location of
the abscess in the left lobe
• 5 to 10 cm in diameter
• Abscess gradually increasing in size
• The location of the abscess in the left lobe
• 5 to 10 cm in diameter
• Abscess gradually increasing in size
• The location of the abscess in the left lobe
Option A: Abscess with a diameter of 2 to 5 cm
• Abscess with a diameter of 5 to 10 cm is prone to rupture.
Option B: Age of the patient
• The patient's age can not predict whether the abscess is prone to rupture.
Option D: Location of abscess in the right lobe
• The location of the abscess in the left lobe is more prone to rupture.

Solution for Question 20:


Option C: Pigtail catheter insertion
• A young patient with a history of travel to an endemic location (India) complains of
abdominal discomfort and fever.
• A big cavity is visible on USG in the liver's left lobe. These point to an amoebic liver abscess.
• Because it is in the left lobe of the liver, the possibilities of rupture are higher; thus, pigtail catheter
placement is the suitable therapy.
• Indications of Pigtail insertion: -Abscess >5cm -Pregnancy -Left lobe abscess
Option A: Oral metronidazole
• Although oral metronidazole 750mg TDS for 10-14 days is the mainstay of treatment and is curative in
more than 90% of patients, this patient's abscess is prone to rupture; therefore, a pigtail catheter is
warranted.
• Luminal agents such as Iodoquinol, Paromomycin, and diloxanide furoate are also used.
Option B: Emetine hydrochloride
• Emetine hydrochloride is indicated in invasive amoebiasis.
Option D: Laparotomy and surgical drainage
• Indications for aspiration in Amoebic Liver Abscess Failure to respond to metronidazole within 3-5
days High risk for rupture, i.e. Left Lobe Abscess and size >5 cm Diagnostic uncertainty Bacterial
superinfection Pregnancy

Page 34

291
• Failure to respond to metronidazole within 3-5 days
• High risk for rupture, i.e. Left Lobe Abscess and size >5 cm
• Diagnostic uncertainty
• Bacterial superinfection
• Pregnancy
• This patient does not have any indication of surgical drainage.
• Failure to respond to metronidazole within 3-5 days
• High risk for rupture, i.e. Left Lobe Abscess and size >5 cm
• Diagnostic uncertainty
• Bacterial superinfection
• Pregnancy

Solution for Question 21:


Option B: Type III
• The ultrasound image shows honeycomb septa which are suggestive of Gharbi classification 3
• Gharbi classification of hydatid cyst:

Option A: Type IV

Page 35

292
• The above image shows that Gharbi classification type IV has a heterogenous echographic pattern.
Option C: Type I

• As shown in the above image, Gharbi classification type I only has a pure fluid collection.

Option D: Type II

Page 36

293
• As shown in the above image, Gharbi classification type II has a Fluid collection with a split wall
(floating membrane).

Solution for Question 22:


Option B: PAIR is the most preferred treatment, optionally
• Surgery is the first choice for complicated hydatid cysts
• Percutaneous and medical treatments represent alternatives to surgery
• Surgery is not done if cysts are too small or the patient is not a suitable candidate for surgery
• For most cysts, surgical resection involving laparoscopic or open complete cyst removal with the
instillation of a scolicidal agent is preferred and is usually curative.
• Puncture aspiration injection reaspiration (PAIR): Most preferred technique for anatomically and
surgically appropriate lesions PAIR - initial course of albendazole → puncture of the cyst → aspiration
of the cyst contents → instillation of hypertonic saline in the cyst cavity → reaspiration
• PAIR - initial course of albendazole → puncture of the cyst → aspiration of the cyst contents →
instillation of hypertonic saline in the cyst cavity → reaspiration
• PAIR - initial course of albendazole → puncture of the cyst → aspiration of the cyst contents →
instillation of hypertonic saline in the cyst cavity → reaspiration
Option A: Drug of choice for hydatid cyst is mebendazole
• Albendazole is the drug of choice for hydatid cysts.
• Mebendazole has poor reabsorption.
Option C: Surgery is the most effective treatment for small cysts
• Pericystectomy is the most effective treatment.
• Pericystectomy / radical cystectomy involves the complete removal of the hydatid cyst.
• Surgery is a good choice for large cysts; if the cysts are too small, the surgery is not an indication.

Page 37

294
Option D: 3% hypertonic saline is most commonly used for PAIR
• The scolicidal agents are 15% to 20% hypertonic saline solution, 0.5% cetrimide solution, 10%
povidone-iodine.
• At present, 20% hypertonic saline is recommended.

Solution for Question 23:


Option C: Biliary cyst
• The patient with dark urine, pale stools, and jaundice has neonatal cholestasis.
• Neonatal cholestasis is due to impaired biliary excretion.
• Biliary atresia is the most common cause of neonatal jaundice, but subhepatic mass makes biliary
cyst the most likely etiology.
• On abdominal examination, the patient presents with abdominal pain, jaundice, and subhepatic mass.
• A biliary cyst is a rare congenital abnormality characterized by intra or extrahepatic cystic dilation of
the biliary tree.
• Management includes complete removal of the cyst to decrease the risk of cholangiocarcinoma.
Option A: Benign neonatal hyperbilirubinemia
• Benign neonatal hyperbilirubinemia is characterized by unconjugated hyperbilirubinemia that peaks in
the first few days, not a week of life; this patient has increased conjugated bilirubin.
Option B: Biliary atresia
• Biliary atresia is the most common cause of neonatal jaundice (increased conjugated bilirubin), but
patients do not present with subhepatic mass, unlike this patient.
Option D: Congenital infection
• Congenital infection, for example, cytomegalovirus, can present with neonatal cholestasis due to
hepatitis, but abdominal mass would not be seen.

Solution for Question 24:


Option B: FNAC / Biopsy is contraindicated
• 45 year female, with nodular enhancement on MRI, is suggestive of hemangioma
• Hemangioma Most common benign tumour of the liver Occurs more commonly in women (3:1) It is of
two types: Cavernous Capillary Cavernous hemangiomas are associated with focal nodular hyperplasia
Usually solitary and <5 cm in diameter Most patients remain asymptomatic (if tumour size < 5cm size) If
size >5cm → Giant hemangioma Malignant transformation is rare FNAC and Biopsy → Contraindicated
(because it is a vascular tumour) Asymptomatic patients- observation TOC→ Enucleation with arterial
inflow control
• Most common benign tumour of the liver
• Occurs more commonly in women (3:1)

Page 38

295
• It is of two types: Cavernous Capillary
• Cavernous
• Capillary
• Cavernous hemangiomas are associated with focal nodular hyperplasia
• Usually solitary and <5 cm in diameter
• Most patients remain asymptomatic (if tumour size < 5cm size)
• If size >5cm → Giant hemangioma
• Malignant transformation is rare
• FNAC and Biopsy → Contraindicated (because it is a vascular tumour)
• Asymptomatic patients- observation
• TOC→ Enucleation with arterial inflow control
• Most common benign tumour of the liver
• Occurs more commonly in women (3:1)
• It is of two types: Cavernous Capillary
• Cavernous
• Capillary
• Cavernous hemangiomas are associated with focal nodular hyperplasia
• Usually solitary and <5 cm in diameter
• Most patients remain asymptomatic (if tumour size < 5cm size)
• If size >5cm → Giant hemangioma
• Malignant transformation is rare
• FNAC and Biopsy → Contraindicated (because it is a vascular tumour)
• Asymptomatic patients- observation
• TOC→ Enucleation with arterial inflow control
• Cavernous
• Capillary
Option A: Most common malignant tumour of the liver
• Hemangioma is the most common benign tumour of the liver
Option C: Commonly associated with Kasabach-Merritt syndrome
• Kasabach-Merritt syndrome is rarely( not commonly) associated with hepatic hemangioma usually
giant hemangioma.
Option D: The treatment of choice is Radical Excision
• The treatment of choice for hemangioma is an enucleation

Page 39

296
Solution for Question 25:
Option B: Paraneoplastic manifestation includes hypoglycemia, hypercalcemia, and erythrocytosis
• According to the clinical scenario, the most likely diagnosis is Hepatocellular carcinoma
• Hepatocellular carcinoma: Most common primary malignant neoplasm of the liver It is the fifth most
common malignancy worldwide More common in men Paraneoplastic syndromes
Hypercholesterolemia (M/C) Hypoglycemia (2nd M/C) Hypercalcemia Erythrocytosis
• Most common primary malignant neoplasm of the liver
• It is the fifth most common malignancy worldwide
• More common in men
• Paraneoplastic syndromes Hypercholesterolemia (M/C) Hypoglycemia (2nd M/C) Hypercalcemia
Erythrocytosis
• Hypercholesterolemia (M/C)
• Hypoglycemia (2nd M/C)
• Hypercalcemia
• Erythrocytosis
• Most common primary malignant neoplasm of the liver
• It is the fifth most common malignancy worldwide
• More common in men
• Paraneoplastic syndromes Hypercholesterolemia (M/C) Hypoglycemia (2nd M/C) Hypercalcemia
Erythrocytosis
• Hypercholesterolemia (M/C)
• Hypoglycemia (2nd M/C)
• Hypercalcemia
• Erythrocytosis
• Hypercholesterolemia (M/C)
• Hypoglycemia (2nd M/C)
• Hypercalcemia
• Erythrocytosis
Option A: Most common malignancy of the liver
• Hepatocellular carcinoma is the most common primary malignant tumour of the liver.
• The most common malignancy of the liver is metastasis from cancers of other organs like colorectal
carcinoma.
Option C: Surgery is a palliative method that has a survival advantage
• TACE (Trans-arterial chemoembolization) is only a palliative method that has a survival advantage
Option D: 50% of tumours are resectable at the time of presentation
• Only 15-20% of tumours are resectable at the presentation time.

Page 40

297
Solution for Question 26:
Option C: Fibrolamellar variant of HCC
• The given computerized tomography scan image shows a central stellate scar appearance.
• Serum alpha-fetoprotein levels are usually normal in fibrolamellar hepatocellular carcinoma.
• Fibrolamellar HCC is well demarcated and encapsulated tumour and has a central fibrotic area (scar)
• It is associated with elevated neurotensin levels.
• It has a better prognosis
• The recurrence is common, and it occurs in 80% of patients
• The presence of metastases to lymph nodes predicts a bad outcome.
Option A: Hepatocellular carcinoma
• Hepatocellular carcinoma (HCC) is a kind of primary liver tumour that typically arises in the context of
chronic liver disease, notably in people with cirrhosis caused by alcohol, chronic hepatitis B or C virus
infections, or non alcohol-associated steatohepatitis (NASH)
Option B: Hepatic adenoma
• Hepatocellular adenoma (HCA), also known as hepatic adenoma is a rare solid, benign liver tumour.
• HCA is typically detected in young women in connection with the use of estrogen-containing
medicines.
• Furthermore, people with glycogen storage disorder or metabolic syndrome are more likely to acquire
HCA.
Option D: Hepatoblastoma
• Although it is a relatively uncommon pediatric solid tumour, hepatoblastoma is children's most
prevalent kind of liver cancer.
• Children under the age of three are typically affected by the condition.
• AFP levels in hepatoblastoma are frequently between 100,000 and 300,000 mcg/mL.

Solution for Question 27:


Option B: B
• In the given image, the marked structure ''B'' depicts Segment IX
• Segment IX is the paracaval portion of the caudate lobe.
• According to the latest study, the liver has nine segments.
• The dorsal sector is made up of two segments: Segment I to the left and in front of the inferior vena
cava and segment IX to the right and front of the cava; are joined inferiorly by the caudate process.
Segment IX is located on the right liver's posterior side. Small dorsal pedicles protrude from the
posterior edge of the major portal components and climb upward. Segment IX is divided into three
subsegments. IXb is located between the middle and right superior hepatic veins. IXc is located

Page 41

298
underneath the right hepatic vein. IXd is to the right of a vertical plane that passes through the right
superior vein. Hepatic veins, particularly the middle and left hepatic veins, enter the cava directly.
• Segment I to the left and in front of the inferior vena cava and segment IX to the right and front of the
cava; are joined inferiorly by the caudate process.
• Segment IX is located on the right liver's posterior side. Small dorsal pedicles protrude from the
posterior edge of the major portal components and climb upward.
• Segment IX is divided into three subsegments. IXb is located between the middle and right superior
hepatic veins. IXc is located underneath the right hepatic vein. IXd is to the right of a vertical plane that
passes through the right superior vein.
• IXb is located between the middle and right superior hepatic veins.
• IXc is located underneath the right hepatic vein.
• IXd is to the right of a vertical plane that passes through the right superior vein.
• Hepatic veins, particularly the middle and left hepatic veins, enter the cava directly.
• Segment I to the left and in front of the inferior vena cava and segment IX to the right and front of the
cava; are joined inferiorly by the caudate process.
• Segment IX is located on the right liver's posterior side. Small dorsal pedicles protrude from the
posterior edge of the major portal components and climb upward.
• Segment IX is divided into three subsegments. IXb is located between the middle and right superior
hepatic veins. IXc is located underneath the right hepatic vein. IXd is to the right of a vertical plane that
passes through the right superior vein.
• IXb is located between the middle and right superior hepatic veins.
• IXc is located underneath the right hepatic vein.
• IXd is to the right of a vertical plane that passes through the right superior vein.
• Hepatic veins, particularly the middle and left hepatic veins, enter the cava directly.
• IXb is located between the middle and right superior hepatic veins.
• IXc is located underneath the right hepatic vein.
• IXd is to the right of a vertical plane that passes through the right superior vein.
Option A: A
• The part of the liver marked as ‘A’ is the caudate lobe.
• The liver parenchyma enclosed by the main portal vein trunk and the right portal vein (ventral border),
inferior vena cava (dorsal border), Spiegel lobe (left border), and right posterior region is designated as
the caudate lobe(right border).
Option C: C
• The part of the liver marked as ‘C’ is the spiegel lobe, also known as segment I.
• Spiegel's lobe (papillary process) is distinguished by a sac-like protrusion of the liver's core section
between two grooves, one for the inferior vena cava and the other for the ductus venosus or Arantius
duct.
Option D: None of the above
• The part of the liver marked as ‘B’ is the paracaval portion of the caudate lobe, also known as
segment IX.

Page 42

299
Solution for Question 28:
Option B: Left trisectionectomy
• The resection of three segments (typically of the liver) is trisectionectomy.
• Left trisectionectomy, also known as an extended left hepatectomy, involves the removal of the
Couinaud’s Segments 2,3,4,5, and 8.
• Segments 2, 3, 4, 5, and 8 are removed in the illustration; hence the treatment is known as a left
trisectionectomy or extended left hepatectomy.
• Liver segment anatomy: Cantlie's line separates the liver into the right and left lobes by forming a
plane between the gallbladder fossa and the inferior vena cava (IVC). The right lobe contributes 60 to
70% of the liver mass, with the left lobe (including the caudate lobe) accounting for the rest. Couinaud
divided the liver into eight segments, beginning with the caudate lobe as segment I and working
clockwise. The left lateral segment is made up of segments II and III, and the left medial segment is
made up of segment IV. As a result, the left lobe is composed of left lateral and left medial portions.
Segment IV is further broken into IVA and IVB. Segment IV is also known as the quadrate lobe.
Segments V, VI, VII, and VIII, make up the right lobe, with segments V and VIII forming the right
anterior lobe and segments VI and VII forming the right posterior lobe.
• Cantlie's line separates the liver into the right and left lobes by forming a plane between the
gallbladder fossa and the inferior vena cava (IVC). The right lobe contributes 60 to 70% of the liver
mass, with the left lobe (including the caudate lobe) accounting for the rest.
• Couinaud divided the liver into eight segments, beginning with the caudate lobe as segment I and
working clockwise.
• The left lateral segment is made up of segments II and III, and the left medial segment is made up of
segment IV. As a result, the left lobe is composed of left lateral and left medial portions. Segment IV is
further broken into IVA and IVB. Segment IV is also known as the quadrate lobe.
• Segments V, VI, VII, and VIII, make up the right lobe, with segments V and VIII forming the right
anterior lobe and segments VI and VII forming the right posterior lobe.
• Cantlie's line separates the liver into the right and left lobes by forming a plane between the
gallbladder fossa and the inferior vena cava (IVC). The right lobe contributes 60 to 70% of the liver
mass, with the left lobe (including the caudate lobe) accounting for the rest.
• Couinaud divided the liver into eight segments, beginning with the caudate lobe as segment I and
working clockwise.
• The left lateral segment is made up of segments II and III, and the left medial segment is made up of
segment IV. As a result, the left lobe is composed of left lateral and left medial portions. Segment IV is
further broken into IVA and IVB. Segment IV is also known as the quadrate lobe.
• Segments V, VI, VII, and VIII, make up the right lobe, with segments V and VIII forming the right
anterior lobe and segments VI and VII forming the right posterior lobe.

Page 43

300
• The falciform ligament splits the left lateral segment from the left medial segment rather than
separating the right and left lobes.

Option A: Left hepatectomy


• A left hepatectomy is performed to treat hilar cholangiocarcinoma, which primarily affects the left side
of the hepatic hilum.
• It involves the removal of the Couinaud’s Segments labeled as 2,3,4 in the image above.
Option C: Right trisectionectomy
• Right trisectionectomy also known as an extended right hepatectomy, involves the removal of the
Couinaud’s Segments 4,5,6,7, and 8.
Option D: Extended right hepatectomy
• Extended right hepatectomy is the other name for right trisectionectomy.

Page 44

301
Solution for Question 29:
The above clinical picture suggests Hepatocellular carcinoma (Raised AFP with arterial enhancement
on CT).
Hepatocellular carcinoma
• Most common primary malignant neoplasm of the liver
• It is the fifth most common malignancy worldwide
• More common in men
Risk factors
• Viral hepatitis (B > C)
• Alcoholic cirrhosis
• Hemochromatosis
• NASH
• Aflatoxin produced by Aspergillus species
• Hydrocarbons, Vinyl chloride, Thorotrast
Clinical features
• Right upper quadrant pain and weight loss
• Presents at an advanced stage
• May rupture and cause hypovolemic shock
• Can cause hepatic vein occlusion (Budd Chiari syndrome)
• Obstructive jaundice
• Hemobilia
• Fever of unknown origin
• Paraneoplastic syndromes
• Hypercalcemia
• Hypoglycemia
• Erythrocytosis
Diagnosis
• Hypervascular - Blood supplied predominantly from the hepatic artery
• CT - Appears hypervascular during the arterial phase and relatively hypodense during the delayed
phase due to early washout of the contrast medium by the arterial blood
• MRI - Variable on T1 weighted images and usually hyperintense on T2 weighted images
• Portal vein invasion common - The presence of an enhancing portal vein thrombus is highly
suggestive of hepatocellular carcinoma.

Page 45

302
• Alpha-fetoprotein (AFP) levels are elevated. A level greater than 400ng/mL is diagnostic for a liver
mass greater than 2cm with arterial enhancement. AFP can be <20 ng/mL in more than 40% of patients
with HCC
Treatment
• Only 10% to 20% of patients are considered to have a resectable disease
• Liver resection and transplantation is the only treatment option with curative potential.
• Of this, transplantation is preferred because most HCCs are cirrhotic and hence do not have enough
reserve
• HCCs are commonly multifocal
• Recurrence rates are high at 5 years after resection
Treatment options
• Surgical
• Liver resection ( treatment of choice)
• Patients without cirrhosis with HCC
• Child’s class A cirrhosis with preserved liver function and no portal hypertension
• Liver transplantation
• Child’s class C
• Sorafenib, molecular targeted therapy - For advanced, unresectable HCC
Option A: Complete excision by partial or total hepatectomy &
transplantation is the only treatment modality with curative potential
• Although complete excision by partial or total hepatectomy & transplantation is the only treatment
modality with curative potential, option B, which states that sorafenib is given for metastatic and
unresectable cases, is also correct.
Option B: Sorafenib is given for metastatic and unresectable cases
• Although sorafenib is given for metastatic and unresectable cases, option A, which states that
complete excision by partial or total hepatectomy & transplantation is the only treatment modality with
curative potential, is also correct.
Option D: Only 15-20 % of cases have an unresectable tumour
• Only 10% to 20% of patients are considered to have a resectable disease; the rest have unresectable
tumours.

Solution for Question 30:


Option C: Allows identification of an accessory or replaced right hepatic artery

Page 46

303
• Pringle’s manoeuvre is also known as total inflow occlusion Allows identification of an accessory or
replaced right hepatic artery The hepatoduodenal ligament is clamped to occlude hepatic blood flow
from the Portal Vein and Hepatic Artery It doesn't control bleeding from the Inferior vena cava and
Hepatic Vein Distinguishes hepatic arterial and portal venous bleeding from hepatic vein bleeding,
which will persist when the hepatoduodenal ligament is clamped To prevent ischemia of the liver,
intermittent vascular occlusion with cycles of approximately 15 minutes on and 5 minutes off may be
done
• Allows identification of an accessory or replaced right hepatic artery
• The hepatoduodenal ligament is clamped to occlude hepatic blood flow from the Portal Vein and
Hepatic Artery
• It doesn't control bleeding from the Inferior vena cava and Hepatic Vein
• Distinguishes hepatic arterial and portal venous bleeding from hepatic vein bleeding, which will persist
when the hepatoduodenal ligament is clamped
• To prevent ischemia of the liver, intermittent vascular occlusion with cycles of approximately 15
minutes on and 5 minutes off may be done
• Allows identification of an accessory or replaced right hepatic artery
• The hepatoduodenal ligament is clamped to occlude hepatic blood flow from the Portal Vein and
Hepatic Artery
• It doesn't control bleeding from the Inferior vena cava and Hepatic Vein
• Distinguishes hepatic arterial and portal venous bleeding from hepatic vein bleeding, which will persist
when the hepatoduodenal ligament is clamped
• To prevent ischemia of the liver, intermittent vascular occlusion with cycles of approximately 15
minutes on and 5 minutes off may be done
Option A: Clamping in Foramen of Winslow occludes Portal triad and Inferior vena cava
• Clamping in the Foramen of Winslow occludes the Portal triad (Hepatic artery, Portal vein, Common
bile duct)
Option B: The hepatoduodenal ligament is clamped to occlude hepatic blood flow from the portal vein a
nd hepatic vein

Page 47

304
• The hepatoduodenal ligament is clamped to occlude hepatic blood flow from the Portal Vein and
Hepatic Artery
Option D: It can control bleeding from the Inferior vena cava and Hepatic Vein
• It doesn't control bleeding from the Inferior vena cava and Hepatic Vein

Solution for Question 31:


Option C: Most common symptom is jaundice
• According to the given clinical scenario, the most likely diagnosis is Pyogenic liver abscess.
• Pyogenic liver abscess is the most common type of liver abscess
• Clinical features of liver abscess: Fever, chills, and abdominal pain are the most common presenting
symptoms
• Fever, chills, and abdominal pain are the most common presenting symptoms
• Ultrasound and CT are the mainstays of diagnostic modalities for hepatic abscess
• Broad-spectrum intravenous antibiotic therapy followed by percutaneous catheter drainage has
become the treatment of choice for most patients. After the advent of this technique, there has been a
significant reduction in the mortality rate, and currently, it is less than 10%
• Fever, chills, and abdominal pain are the most common presenting symptoms
Option A: Most common route of infection is bile duct
• Along with cryptogenic infections, infections from the biliary tree (acute suppurative cholangitis) are
the most common identifiable cause of the hepatic abscess
Option B: Most common organism responsible is E. coli
• Pyogenic abscesses are polymicrobial. The most common organisms cultured are Escherichia coli
and Klebsiella pneumoniae
Option D: Most common LFT abnormality is raised ALP
• The alkaline phosphatase is mildly elevated in 80% of patients, whereas total bilirubin is elevated 20%
to 50% of the time

Solution for Question 32:


Option B: Biliary atresia
• Biliary atresia is a rare obstructive cholangiopathy that affects both the intrahepatic and extrahepatic
bile ducts.
• It manifests as chronic jaundice, clay-colored feces, and hepatomegaly in the newborn era.
• If left untreated, it is fatal, with a reported survival rate of less than 10% at 3 years of age.
• It is the most common indication of a pediatric liver transplant.
Option A: Hepatoblastoma

Page 48

305
• Although it is a relatively uncommon pediatric solid tumour, hepatoblastoma is children's most
prevalent kind of liver cancer.
• Children under the age of three are typically affected by the condition.
• AFP levels in hepatoblastoma are frequently between 100,000 and 300,000 mcg/mL.
Option C: Viral hepatitis
• Viral hepatitis is an indication of a liver transplant.
Option D: HCV-induced cirrhosis
• HCV-induced cirrhosis is the most common indication for liver transplant in adults.

Solution for Question 33:


Option D: The risk for transmission is approximately 1 per 1 million donations
• The ability to identify persons infected with HCV has become available only available recently.
• It is true, however, that the successful elimination of most HCV carriers occurs through routine testing.
• Although serologic tests are available for HCV, they only test antigens and not antibodies. Therefore,
this test alone may not screen out persons who are infected but have not yet developed antibodies.
• Transmission of Hepatitis C (HCV) has been dramatically minimized by improved donor selection and
antibody screening techniques. The residual risk among allergenic donations is now estimated to be
less than 1 per 1,000,000 donations. The residual risk of hepatitis B is approximately 1 per 3,000,000
donations.
• HCV, previously known as non-A, non-B hepatitis, is an RNA flavivirus. The number of new infections
per year has declined since the 1980s due to routine testing of blood donors for the virus. In the past,
blood transfusion was the major cause of HCV infection, but currently, only <2% of acute infections are
caused by transfusions. Fortunately, HCV is not transmitted efficiently through occupational exposures
to blood, with the seroconversion rate after accidental needlestick approximately 1.8%. But, healthcare
workers have a higher carrier state than the general public. Transmission among healthcare workers
has usually related to needle stick incidents, and the risk of transmission is higher than that of HBV and
HIV.
• Acute HCV infection typically develops 2 to 26 weeks after exposure to the virus.
• Diagnosis is confirmed by the presence of HCV RNA and anti-HCV antibodies in the serum.
• Hepatitis C is first detectable in the serum by polymerase chain reaction within days to weeks
following the exposure, whereas antibodies will not appear until 2 to 6 months after.
• The HCV antibody remains positive for life after exposure, but do notconfer immunity to the virus.
• No specific HCV antigen tests exist.
• Treatment involves ribavirin and pegylated gamma interferon.
Incorrect option:
Option A: Because HCV is transmitted efficiently through occupational exposures to blood
• Hepatitis C is transmitted through needle pricks sharing drug injection equipment, and needle
syringes for any equipment used to prepare and inject drugs.
Option B: Blood infected with the hepatitis C virus (HCV) cannot be eliminated through routine testing

Page 49

306
• Hepatitis C infection can be detected through anti-HCV antibodies, the antibody can be detected in
blood against Hepatitis C antigen, so this option is eliminated.
Option C: Current serologic tests for HCV antigens do not exclude carriers
• Elimination of most HCV carriers occurs through routine testing, so this option is eliminated.

Solution for Question 34:


Option C: Left medial segment, left lateral segment
• The falciform ligament is a Sickle-shaped ligament that divides the liver's left lobe into the left medial
and left lateral segments.
• The liver is the largest organ after the skin.
• It weighs about 1500 grams.
• It lies under the diaphragm in the right upper abdomen and mid abdomen and extends to the left
upper abdomen.
Other options
Option A: Caudate lobe, quadrate lobe
• It divides the left lob into the left lateral and medial segments, so this option is wrong.
Option B: Right lobe, left lobe
• It divides the left lobe into the left lateral and medial segments, so this option is wrong.
• Cantlie's line divides liver into left and right lobe.
Option D: Left medial segment, right lobe
• It divides the left lobe into the left lateral and medial segments, so this option is wrong.

Solution for Question 35:


Option B: Pruritus
• Primary biliary cholangitis or primary biliary cirrhosis is an autoimmune condition with obliteration of
bile ducts and most commonly presenting with pruritus that is worse at night
• Primary biliary cirrhosis It is an autoimmune condition with obliteration of the bile ducts, causing a
string of beads appearance on the bile duct on ERCP Most patients are asymptomatic The most
common symptom- fatigue (40% to 80%) > generalized pruritus > jaundice (occurs late) Pruritus
precedes jaundice
• It is an autoimmune condition with obliteration of the bile ducts, causing a string of beads appearance
on the bile duct on ERCP
• Most patients are asymptomatic
• The most common symptom- fatigue (40% to 80%) > generalized pruritus > jaundice (occurs late)
• Pruritus precedes jaundice

Page 50

307
• It is an autoimmune condition with obliteration of the bile ducts, causing a string of beads appearance
on the bile duct on ERCP
• Most patients are asymptomatic
• The most common symptom- fatigue (40% to 80%) > generalized pruritus > jaundice (occurs late)
• Pruritus precedes jaundice
Option A: Jaundice
• Obstructive jaundice is caused by obliteration of the bile ducts in primary biliary cirrhosis, but since the
disease is progressive and in the early stages, the liver cirrhosis has not happened extensively,
jaundice and yellowing of the skin and sclera are not very pronounced, pruritis is present in the early
stages because of cholestasis that causes the pruritogens to accumulate and cause pruritis
Option C: Melanosis
• Melanosis is not a symptom of Primary biliary cirrhosis. It can present in conditions like Lentiginosis
Option D: Vomiting
• Vomiting is not a feature of Primary biliary cirrhosis as it is an autoimmune condition that does not
cause any obstruction of the duodenum like biliary cholangiocarcinoma or pancreatic head carcinomas

Solution for Question 36:


Option B: Both A and R are true, but R is not a correct explanation of A
• Blunt trauma is caused by direct compression.
• Penetrating injuries to the upper abdomen or lower thorax might cause liver injury.
• In the hemodynamically stable patient, CT scanning is the investigation of choice.
• Most blunt trauma patients who are hemodynamically stable can be handled conservatively
• The development of hemodynamic instability is an indication of terminating cautious therapy.
• Evidence of continued blood loss in the absence of treatment of any underlying coagulopathy
• The emergence of symptoms of generalised peritonitis
• Hemodynamically unstable patients will necessitate an urgent laparotomy.
• Surgical treatment includes the following procedures: push, Pringle, plug, and pack.
• The portal vein cannot be closed up, but the hepatic artery can (which should be stented)
• Closed suction drainage should be employed at all times.

Page 51

308
• Total Inflow Occlusion, commonly known as Pringle's Maneuver
• The Pringle manoeuvre is a surgical technique used in some abdominal operations and in liver
trauma. The hepatoduodenal ligament is clamped either with a surgical tool called a haemostat, an
umbilical tape or by hand.
• The Winslow foramen clamping obstructs the portal triad (hepatic artery, portal vein, common bile
duct)
• The hepatoduodenal ligament is constricted to prevent hepatic blood flow from the Portal Vein and
Hepatic Artery.
• It does not stop bleeding from the inferior vena cava and the hepatic vein.
• Differentiates hepatic arterial and portal venous bleeding from hepatic vein haemorrhage that persists
after the hepatoduodenal ligament is clamped.
• Intermittent vascular occlusion with cycles of around 15 minutes on and 5 minutes off may be
performed to prevent liver ischemia.

Page 52

309
Option A: Both A and R are true, and R is the correct explanation of A
• Both A and R are true, but R is not a correct explanation of A
Option C: A is true, but R is false
• Both A and R are true, but R is not a correct explanation of A
Option D: A is false, but R is true
• Both A and R are true, but R is not a correct explanation of A

Page 53

310
Portal Hypertension
1. A 60-year-old male patient is brought to the emergency department with recurrent episodes of bloody
vomiting. The patient has previously tested positive for hepatitis C. His temperature is 98° F (38° C), his
blood pressure is 100/70 mmHg, his pulse rate is 98 per minute, and his respiratory rate is 18 per
minute. After giving the initial supportive treatment, the patient is stable. The upper GI endoscopy is
done, which shows esophageal varices. Which of the following is the Normal portal venous pressure?
(or)
Which of the following is the Normal portal venous pressure?
A. <3 mmHg
B. 3-5 mmHg
C. 5-10 mmHg
D. 10-12 mmHg
----------------------------------------
2. A 63-year-old male comes with bleeding per rectum. There is a nontender abdomen with positive
shifting dullness and 2+ peripheral pitting oedema. The Blood urea nitrogen and serum creatinine are
raised. Urine sodium is 5 mEq/L: Which of the following is the cause?
(or)
A 63-year-old male with known liver cirrhosis comes to the emergency department with bleeding per
rectum for 4 days. He also has increasing abdominal distension. He had variceal banding after an
episode of esophageal bleeding 2 months ago. His vitals are stable. There is a non-tender abdomen
with positive shifting dullness and normal bowel sounds. There is grade 2+ peripheral pitting edema.
The blood urea nitrogen and serum creatinine are raised. Urine dipstick is negative for protein and
blood. Urine sodium is 5 mEq/L. Which of the following is the most likely cause of this patient?
A. Nephrotic syndrome
B. Acute kidney injury
C. Hepatorenal syndrome
D. Acute interstitial nephritis
----------------------------------------
3. A 22-year-old male presented with repeated episodes of hematemesis. There is no history of
jaundice or liver decompensation. On examination, the significant findings include massive
splenomegaly and the presence of esophageal varices. There are no ascites. The liver function tests
are normal. Which of the following is likely the cause of this patient's diagnosis?
(or)
A 22-year-old male presents with repeated episodes of hematemesis. There are no ascites or peptic
ulceration. The liver function tests are normal. Which of the following is likely the cause?
A. Hepatitis C
B. Non-Cirrhotic portal fibrosis
C. Cirrhosis
D. Hepatic venous outflow tract obstruction
----------------------------------------

311
4. A 45-year-old man presented for haematemesis and haematochezia. An urgent upper GI endoscopy
showed multiple oesophageal varices, mild portal hypertensive gastropathy, and no gastric varices.
Which of the following agents is recommended for variceal bleeding?
(or)
A 45-year-old man with a history of active alcoholism and chronic hepatitis C presented to the
emergency department with hematemesis and hematochezia started 5 days before. The patient was
alert, tachycardic, and hypotensive. Apart from an enlarged palpable spleen, physical examination was
unremarkable for other stigmata of chronic liver disease. After adequate hemodynamic resuscitation,
the patient underwent an urgent upper GI endoscopy, which showed multiple esophageal varices, mild
portal hypertensive gastropathy, and no gastric varices. Which of the following agents is recommended
for the medical treatment of variceal bleeding?
A. Octreotide
B. TIPS
C. Antibiotics
D. Nitroglycerine
----------------------------------------
5. A 57-year-old male comes to the physician's clinic for a follow-up. The patient has been Hepatitis B
positive for the last 10 years. The patient is currently asymptomatic, and vital signs are normal. The
patient never had an endoscopic investigation. This time the physician advises the patient for upper GI
endoscopy. During the procedure, the esophageal varices are seen, which are spread to the lesser
curvature of the stomach. Which of the following is most likely the classification of esophageal varices
in the patient?
(or)
hat is the likely type of varices in a patient with esophageal varices that also involve the LC of the
stomach?
A. Type 1
B. Type 2
C. Type 3
D. Type 4
----------------------------------------
6. A 65-year-old male is brought to emergency department due to five episodes of Bloody vomiting at
home. After giving iv Fluids and appropriate medications, the surgeon thinks about surgical shunting.
Which of the following is the correct option?
(or)
A 65-year-old male is brought to the emergency department due to five episodes of bloody vomiting at
home. During the past 3 months, this is the third time the patient has been brought to the emergency
department with the same presenting complaint. The patient is hemodynamically unstable due to
hypotension. After giving IV fluids and appropriate medications, the gastroenterologist thinks about
surgical shunting. Which of the following is the correct option?
A. Mesocaval shunt is a selective shunt
B. ECK Fistula is a selective shunt
C. ECK Fistula is a non-selective shunt

Page 2

312
D. Distal splenorenal shunt is a non-selective shunt
----------------------------------------
7. A 51-year-old male patient comes to the clinic with complaints of weight gain over the last three
months. The patient lives a healthy lifestyle and eats a balanced diet. Regardless, he gained 4 kg in the
last three months. The patient has been swimming in a freshwater pool for the last 6 months. He
notices an increase in abdominal circumference. On examination, the abdomen is tender, and there is
shifting dullness. Which of the following is most likely the cause of intrahepatic presinusoidal portal
hypertension in this case?
(or)
Which of the following is the cause of intrahepatic presinusoidal portal hypertension
A. Budd-Chiari syndrome
B. Veno occlusive disease
C. Abdominal trauma
D. Schistosomiasis
----------------------------------------
8. A 67-year-old man with acute pancreatitis came to your hospital due to recurrent episodes of upper
gastrointestinal bleeding and hemoglobin of 5.8 gm/dL, requiring transfusions of red cell units.
Gastroscopy showed diffuse fundal gastric varices but no esophageal varices. A computerized
tomography scan showed liquefactive necrosis of pancreatic parenchyma and splenomegaly with
diffuse varices around the gastrosplenic and gastrohepatic ligament. With the diagnosis of left-sided
portal hypertension due to acute pancreatitis, which of the following would be the appropriate
treatment?
(or)
A 67-year-old man comes with recurrent episodes of upper gastrointestinal bleeding. With the
diagnosis of left-sided portal hypertension due to acute pancreatitis, which of the following would be the
appropriate treatment?
A. Splenectomy
B. Portocaval shunt
C. Splenorenal shunt
D. None of the above
----------------------------------------
9. A 40-year-old man comes due to exertional dyspnea and Abdominal discomfort. Examination shows
a distended abdomen with shifting dullness and ankle oedema. His ascitic fluid is taken and shows the
following laboratory values? Albumin 2.5 g/dL Lactate dehydrogenase 38 units/L Blood investigations
show: Albumin 3.8 g/dL Lactate dehydrogenase 100 units/L Which of the following is the most likely
cause of this condition?
(or)
A 40-year-old man comes to the hospital due to exertional dyspnea and Abdominal discomfort. He has
a positive history of alcohol abuse, peptic ulcer disease, and acute pancreatitis. On examination, the
patient shows a distended abdomen with shifting dullness and ankle edema. His ascitic fluid is taken
and shows the following laboratory values: Albumin 2.5 g/dL Neutrophils 204/µL Lactate
dehydrogenase 38 units/L Glucose 83 mg/dL Blood investigations show: Albumin 3.8 g/dL Potassium
3.2 mEq/L Sodium 130 mEq/L Glucose 120 mg/dL Lactate dehydrogenase 100 units/L Which of the

Page 3

313
following is the most likely cause of this condition?
A. Decrease in liver function
B. Increased hydrostatic capillary pressure
C. Increased capillary permeability
D. Increase Glomerular filtration pressure
----------------------------------------
10. A 62-year-old male who is alcoholic comes to the clinic due to decreased libido and inability to
maintain erections for the past four months. He has anorexia and 4 kg weight loss over the same
duration. The vital signs are normal. Physical examination shows bilateral gynecomastia and small
testes. Laboratory values show normal TSH with decreased levels of total T3 and thyroxine T4. Which
of the following is most likely the diagnosis in this case?
(or)
A 62-year-old male comes due to decreased libido, inability to maintain erections, 4 kg weight loss,
gynecomastia and small testes with normal TSH with decreased levels of total T3 and thyroxine T4.
Which of the following is most likely the diagnosis in this case?
A. Adrenal insufficiency
B. Chronic liver disease
C. Hashimoto thyroiditis
D. Hyperthyroidism
----------------------------------------
11. A 39-year-old man referred from a local hospital where he was being followed for cirrhosis of the
liver due to 4 years of chronic alcoholism. The patient has had seven episodes of vomiting in the last
seven hours. The blood pressure is 100/70 mmHg, the pulse rate is 110 beats per minute, and the
respiratory rate is 18 per minute. The patient is afebrile. His condition was complicated by
alcohol-induced encephalopathy, delirium tremens, portal hypertension, and arrhythmia. The
examination revealed abdominal ascites, pedal edema, moist skin, and esophageal varices during the
endoscopic examination. What is the next likely treatment?
(or)
A 39-year-old man is brought for cirrhosis of the liver due to 4 years of chronic alcoholism. The blood
pressure is 100/70 mmHg. The examination revealed abdominal ascites, pedal oedema, moist skin,
and oesophagal varices. What is the likely treatment?
A. Administer octreotide infusion
B. Obtain second intravenous access
C. Perform-esophagogastroduodenoscopy
D. Perform mesenteric arteriography
----------------------------------------
12. A 61-year-old male was brought to the emergency department due to confusion, fatigue, lethargy,
and anorexia for one day. He also complained of right upper quadrant abdominal discomfort. The
patient has pruritus and jaundice. Hand tremors are also present. Laboratory values show blood
glucose 80 mg/dL and an INR of 1.7. Which of the following values of liver enzymes would be seen in
this patient?

Page 4

314
(or)
A 61-year-old male comes with confusion, lethargy, anorexia , right abdominal pain, pruritus and
jaundice. Hand tremors are also present. Laboratory values show blood glucose of 80 mg/dl and INR
1.7. Which of the following values would present?
A. AST 200: ALT 300
B. AST 850: ALT 900
C. AST 500: ALT 250
D. AST 1100: ALT 1230
----------------------------------------
13. A 37-year-old female was admitted with a complaint of escalating abdominal pain, increased
abdominal circumference, and recurrent diarrhea. The symptoms started 2 months ago, mild at first, but
now become more obvious and bothersome. The patient gave a history of consumption of bush teas for
5 months to reduce weight. Physical examination revealed mild jaundice, distended abdomen, and
tenderness on RUQ. The sign of ascites was positive. Laboratory results showed WBC 6.82 × 109/L,
Hb 122 g/L, HbsAg (-), ALT 66 U/L, AST 81 U/L, serum total bilirubin 39.7 µmol/L, direct bilirubin 10.6
µmol/L, and ALB 32.8 g/L. CT scan of the abdomen revealed moderate ascites, hepatomegaly, delayed
enhancement of parenchyma, and obscure hepatic veins. Which of the following drugs is effective in
improving survival in this patient?
(or)
A 37-year-old female was admitted with a complaint of escalating abdominal pain, increased abdominal
circumference, and recurrent diarrhoea. CT scan of the abdomen revealed moderate ascites,
hepatomegaly, delayed enhancement of parenchyma, and obscure hepatic veins. Which of the
following drugs is effective?
A. Amiodarone
B. Tenofovir
C. Defibrotide
D. Ibutilide
----------------------------------------
14. A 50-year-old male who is a chronic alcoholic is brought to the emergency department with a poor
appetite for the last two weeks. His vitals are normal. His sclera appears icteric. On examination, the
liver is enlarged and tender. There is no spider angioma, tremors, or abdominal distention. Which of the
following Lab findings are most likely to be present in this patient in later stages?
(or)
In a chronic alcoholic patient, sclera appears icteric. On examination, the liver is enlarged and tender.
Which of the following Laboratories is most likely present in this patient in later stages?
A. ALT -220;AST -111;GGT -increased; ferritin- decreased
B. ALT -2500;AST -1050;GGT -increased; ferritin- increased
C. ALT -2640;AST - 2355;GGT -increased; ferritin - increased
D. ALT -266;AST -105;GGT -normal; ferritin - normal
----------------------------------------

Page 5

315
15. A 39-year-old male was referred to your hospital from a local hospital, where he was followed up for
liver cirrhosis induced by 4 years of chronic alcoholism. His condition was complicated by
alcohol-induced encephalopathy, delirium tremens, portal hypertension, and arrhythmia. On
endoscopic examination, the existence of abdominal ascites, pedal edema, clammy skin, and
esophageal varices. Child-Turcotte-Pugh (CTP) Scoring was done, which placed him in class C. What
of the following probable treatment?
(or)
Child-Turcotte-Pugh (CTP) Scoring placed a patient in class C. What of the following probable
treatment?
A. Liver transplantation
B. Major surgery
C. Minor surgery
D. TIPS
----------------------------------------
16. A 12-year-old boy presents with hematemesis, melena, and mild splenomegaly. There is no
obvious jaundice or ascites. Which of the following is the most likely diagnosis?
(or)
A 12-year-old boy presents with hematemesis, melena, and mild splenomegaly. There is no obvious
jaundice or ascites. Which of the following is the most likely diagnosis?
A. Extrahepatic portal venous obstruction
B. Non-cirrhotic portal fibrosis
C. Cirrhosis
D. Hepatic venous outflow tract obstruction
----------------------------------------

Correct Answers
Question Correct Answer

Question 1 3
Question 2 3
Question 3 2
Question 4 1
Question 5 1
Question 6 3
Question 7 4
Question 8 1
Question 9 2
Question 10 2
Question 11 2

Page 6

316
Question 12 4
Question 13 3
Question 14 1
Question 15 1
Question 16 1

Solution for Question 1:


Option C: 5-10 mmHg
• Portal hypertension is increased pressure within the portal venous system. It is determined by
increased portal pressure gradient (the difference in pressures between the portal venous pressure and
the pressure within the inferior vena cava or the hepatic vein.
• The patient with hepatitis C-positive variceal bleeding shown on upper GI endoscopy is most likely
diagnosed with liver cirrhosis.
• Bloody vomiting is due to the rupture of varices caused by portal hypertension.
• Portal Pressure: Normal portal venous pressure is 5-10 mmHg. At this pressure, very little blood is
shunted from the portal system into the systemic circulation. As portal venous pressure increases, the
collateral communication with the systemic circulation dilates, and blood is shunted into the systemic
circulation. Variceal bleeding occurs when portal pressure is >12 mmHg.
• Normal portal venous pressure is 5-10 mmHg.
• At this pressure, very little blood is shunted from the portal system into the systemic circulation.
• As portal venous pressure increases, the collateral communication with the systemic circulation
dilates, and blood is shunted into the systemic circulation.
• Variceal bleeding occurs when portal pressure is >12 mmHg.
• Normal portal venous pressure is 5-10 mmHg.
• At this pressure, very little blood is shunted from the portal system into the systemic circulation.
• As portal venous pressure increases, the collateral communication with the systemic circulation
dilates, and blood is shunted into the systemic circulation.
• Variceal bleeding occurs when portal pressure is >12 mmHg.
Option A: <3 mmHg
• Portal hypertension occurs when portal pressure exceeds 8 mmHg, so this option is wrong.
Option B: 3-5 mmHg
• This option is wrong because Portal hypertension occurs when portal pressure exceeds 8 mmHg.
Option D: 10-12 mmHg
• This value shows portal hypertension.

Solution for Question 2:


Option C: Hepatorenal syndrome

Page 7

317
• The patient with a history of liver cirrhosis presents with an abnormal renal function test and most
likely a hepatorenal syndrome diagnosis.
• The risk factor is advanced cirrhosis.
• Hepatorenal syndrome (HRS) is development of acute renal failure due to severe hepatic or biliary
disease often with jaundice. It is defined as a reversible functional renal impairment in the absence of
other causes of renal failure, tubular dysfunction, proteinuria, or morphological alterations in histological
studies.
• The Precipitating factors are Reduced renal perfusion, GI bleeding, vomiting, sepsis, excessive
diuretic use, reduced glomerular pressure, and NSAID use (constricts afferent arterioles).
• The diagnosis is made by Renal hypoperfusion, FeNa <1% (or urine Na <10 mEq/L), absence of
tubular injury, no RBC, protein, or granular casts in the urine, and no improvement in renal function with
fluids.
• Diagnostic criteria for HRS: (1) Chronic or acute liver disease with advanced liver failure and portal
hypertension (2) Plasma creatinine concentration> 1.5 mg/dl (3) The absence of other apparent cause.
(4) Lack of improvement in renal function after volume expansion with intravenous albumin (1 g/kg of
body weight per day up to 100 g/day) for at least two days and withdrawal of diuretics (5) Absence of
parenchymal kidney disease as indicated by proteinuria >500 mg/day, microhematuria (>50 red blood
cells per high power field) or ultrasonographic evidence of obstructive uropathy or renal parenchymal
disease
• The management includes addressing precipitating factors (e.g., hypovolemia, anemia, infection),
Splanchnic vasoconstrictors (midodrine, octreotide, norepinephrine), and Liver transplantation.
Option A: Nephrotic syndrome
• Nephrotic syndrome is characterized by the loss of proteins in urine (more than 3 grams per day).
• The nephrotic syndrome patient does not present with ascites and abdominal distention, which is the
Hallmark of liver cirrhosis.
Option B: Acute kidney injury
• Acute kidney injuries are caused by ingesting nephrotoxic drugs like aminoglycosides.
• Granular casts are seen in the urine.
Option D: Acute interstitial nephritis
• In acute interstitial nephritis, the inflammation of the kidney interstitium occurs.
• WBC casts, and eosinophils are seen in the urine.
• Proteinuria can also occur.

Solution for Question 3:


Option B: Non-Cirrhotic portal fibrosis
• Non-Cirrhotic Portal HTN is elevated portal venous pressure(>8 mmHg )without liver cirrhosis (without
liver Pathology).
• It is an autosomal recessive hereditary condition.
• It is also called " obliterative portal venopathy."

Page 8

318
• About 85% of people with NCPF have multiple episodes of recurrent bloody vomiting.
• The features of NCPF (Non-Cirrhotic Portal Fibrosis) are: No ascites No jaundice No encephalopathy
No hepatic failure Varices present Splenomegaly more common (Moderate to Massive) In this
condition, there is sclerosis and thrombosis of portal vein branches.
• No ascites
• No jaundice
• No encephalopathy
• No hepatic failure
• Varices present
• Splenomegaly more common (Moderate to Massive)
• In this condition, there is sclerosis and thrombosis of portal vein branches.
• No ascites
• No jaundice
• No encephalopathy
• No hepatic failure
• Varices present
• Splenomegaly more common (Moderate to Massive)
• In this condition, there is sclerosis and thrombosis of portal vein branches.
Option A: Hepatitis C
• The patients with Hepatitis C mostly have liver cirrhosis, which presents with bloody vomiting due to
esophageal varices, splenomegaly, and ascites which is absent in the case.
Option C: Cirrhosis
• Cirrhosis patient presents with abdominal distension called ascites which is absent in this patient, so
this option is wrong.
Option D: Hepatic venous outflow tract obstruction
• It is called Budd-Chiari syndrome.
• The causes of this condition are a hypercoagulable state, anti-thrombin 3 deficiency, nephrotic
syndrome, or malignancy.

Solution for Question 4:


Option A: Octreotide
• For managing variceal bleeding, somatostatin analogs, octreotide, are the agents of choice.
• It should be initiated at the onset of variceal bleeding.
• A combination of octreotide and endoscopic therapy is more effective in controlling bleeding and is the
preferred treatment for most patients.
• octreotide dose: 50 µg as loading dose IV, 50 µg 1 hour in 5% dextrose as maintenance dose

Page 9

319
• Although Vasopressin is the most potent available vasoconstrictor, it causes hypertension, myocardial
ischemia, arrhythmia, ischemic abdominal pain, and limb gangrene, so it is not used.
Option B: TIPS
• TIPS (Transjugular intrahepatic portosystemic shunt).
• It is a second-line therapy if the other medications fail.
• TIPS decreases the portal pressure by channeling between the hepatic vein and the portal vein
branch.
Option C: Antibiotics
• This patient is hemodynamically unstable, so the first target is maintaining blood pressure to make the
patient hemodynamically stable, which is life-saving.
• So the first step is to give IV Fluids and octreotide to decrease the bleeding by causing
vasoconstriction of splanchnic vessels.
Option D: Nitroglycerine
• Nitroglycerine is an important vasodilator. Instead of decreasing bleeding, it will exacerbate this
condition.
• This drug is popular to use in angina.

Solution for Question 5:


Option A: Type 1
• Sarin's classification is the most commonly used classification for esophageal varices.
• Classification is classified into four types: Type 1 is also called Gastroesophageal varices 1. It extends
from the esophagus to the lesser curvature of the stomach. Type 2 is also called Gastroesophageal
varices 2. It extends from the esophagus to the greater curvature of the stomach. Type 3 is Isolated
gastric varices. In this, gastric varices occur in the fundus, and there is an absence of esophageal
varices. Type 4 is Isolated gastric varices in which any part of the stomach is involved, like the
stomach's antrum, body, or pylorus, but esophageal varices are absent.
• Type 1 is also called Gastroesophageal varices 1. It extends from the esophagus to the lesser
curvature of the stomach.
• Type 2 is also called Gastroesophageal varices 2. It extends from the esophagus to the greater
curvature of the stomach.
• Type 3 is Isolated gastric varices. In this, gastric varices occur in the fundus, and there is an absence
of esophageal varices.
• Type 4 is Isolated gastric varices in which any part of the stomach is involved, like the stomach's
antrum, body, or pylorus, but esophageal varices are absent.
• Type 1 is also called Gastroesophageal varices 1. It extends from the esophagus to the lesser
curvature of the stomach.
• Type 2 is also called Gastroesophageal varices 2. It extends from the esophagus to the greater
curvature of the stomach.
• Type 3 is Isolated gastric varices. In this, gastric varices occur in the fundus, and there is an absence
of esophageal varices.

Page 10

320
• Type 4 is Isolated gastric varices in which any part of the stomach is involved, like the stomach's
antrum, body, or pylorus, but esophageal varices are absent.
Option B: Type 2
• Type 2 extends from the esophagus to the greater curvature of the stomach.
Option C: Type 3
• In this, gastric varices occur in the fundus, and there is an absence of esophageal varices.
Option D: Type 4
• Type 4 is isolated gastric varices in which any part of the stomach is involved, like the stomach's
antrum, body, or pylorus, but esophageal varices are absent.

Solution for Question 6:


Option C: ECK Fistula is a non-selective shunt
• Surgical shunts are an effective method of preventing rebleeding from esophageal or gastric varices
• They reduce the portal venous pressure by diverting the blood into the low-pressure systemic
circulation
• Shunts may be selective, non-selective, or partial shunts. ECK Fistula is a type of non-selective shunt
Non-selective shunts control and prevent further bleeding as well as reduce the ascites. But since it
diverts the blood from liver there is high incidence of encephalopathy
• ECK Fistula is a type of non-selective shunt
• Non-selective shunts control and prevent further bleeding as well as reduce the ascites. But since it
diverts the blood from liver there is high incidence of encephalopathy
• ECK Fistula is a type of non-selective shunt
• Non-selective shunts control and prevent further bleeding as well as reduce the ascites. But since it
diverts the blood from liver there is high incidence of encephalopathy
Option A: Mesocaval shunt is a selective shunt
• The mesocaval shunt is non-selective, so this option is wrong.
Option B: ECK Fistula is a selective shunt
• ECK Fistula is a non-selective shunt, so this option is wrong.
Option D: Distal splenorenal shunt is a non-selective shunt
• The distal splenorenal shunt is selective, so this option is wrong

Solution for Question 7:


Option D: Schistosomiasis
• The most common cause of intrahepatic presinusoidal portal hypertension → Schistosomiasis.

Page 11

321
• Schistosomiasis: The risk factor is swimming or bathing in contaminated freshwater. It causes an
enlargement of the liver and spleen. In the liver, it causes inflammation, Fibrosis, and Portal
hypertension. Snails are the intermediate hosts. It can also cause pulmonary hypertension and bladder
cancer.
• The risk factor is swimming or bathing in contaminated freshwater.
• It causes an enlargement of the liver and spleen. In the liver, it causes inflammation, Fibrosis, and
Portal hypertension.
• Snails are the intermediate hosts.
• It can also cause pulmonary hypertension and bladder cancer.
• The risk factor is swimming or bathing in contaminated freshwater.
• It causes an enlargement of the liver and spleen. In the liver, it causes inflammation, Fibrosis, and
Portal hypertension.
• Snails are the intermediate hosts.
• It can also cause pulmonary hypertension and bladder cancer.
Option A: Budd-Chiari syndrome
• Budd-Chiari syndrome causes post-hepatic post-sinusoidal portal hypertension, so this option is
wrong.
Option B: Veno occlusive disease
• Veno occlusive disease causes intrahepatic post-sinusoidal portal hypertension.
Option C: Abdominal trauma
• Abdominal trauma causes prehepatic presinusoidal portal hypertension, so this option is wrong.

Solution for Question 8:


Option A: Splenectomy
• Acute pancreatitis can cause splenic vein thrombosis and pseudo-aneurysm formation in the splenic
artery.
• This aneurysm causes splenomegaly.
• Splenomegaly causes gastric varices, mainly along the greater curvature of the stomach and fundus.
• Left-sided portal hypertension is also called SINISTRAL hypertension.
• The treatment of choice is Splenectomy.
Option B: Portocaval shunt
• The portocaval shunt is an abnormal vein connecting the blood supply from the intestine to the vein
that returns blood to the heart, bypassing the liver which is called shunting.
• In this clinical scenario, the patient has a problem with the spleen, so Splenectomy should be done.
Option C: Splenorenal shunt

Page 12

322
• This is the surgical process in which the splenic vein is detached from the portal vein and reattached
to the left kidney vein, called the renal vein, to relieve pressure, causing a portal system shunt.
• The first line of treatment is surgical splenectomy to decompress the left portal venous system in this
patient.
Option D: None of the above
• Splenic vein thrombosis and pseudo-aneurysm formation in the splenic artery cause splenomegaly
due to acute pancreatitis, so splenectomy should be done.

Solution for Question 9:


Option B: Increased hydrostatic capillary pressure
• This patient's SAAG (Serum ascites albumin gradient) of 1.3 g/dL (3.8-2.5) is consistent with portal
hypertension, e.g., cirrhosis with increased hydrostatic pressure in hepatic capillary beds.
• Ascitic fluid helps to rule out the cause of ascites.
• The following colours and their cause are described below: If Bloody: trauma, malignancy, TB (rarely)
If Milky: chylous If Turbid: possible infection If Straw colour: likely more benign causes If neutrophils are
≥250/mm³, then peritonitis may be due to secondary or spontaneous bacteria. If ascitic protein is more
than 2.5 g/dL (high-protein ascites), the causes are CHF, constrictive pericarditis, peritoneal
carcinomatosis, TB, Budd-Chiari syndrome, and fungal. If the protein is less than 2.5 g/dL (low-protein
ascites), the causes are cirrhosis and nephrotic syndrome. If protein is more than 1.1 g/dL (indicates
portal hypertension), the causes are Cardiac ascites, cirrhosis, and Budd-Chiari syndrome. If the
protein is <1.1 g/dL (absence of portal hypertension), the causes are TB, peritoneal carcinomatosis,
pancreatic ascites, and nephrotic syndrome.
• If Bloody: trauma, malignancy, TB (rarely)
• If Milky: chylous
• If Turbid: possible infection
• If Straw colour: likely more benign causes
• If neutrophils are ≥250/mm³, then peritonitis may be due to secondary or spontaneous bacteria.
• If ascitic protein is more than 2.5 g/dL (high-protein ascites), the causes are CHF, constrictive
pericarditis, peritoneal carcinomatosis, TB, Budd-Chiari syndrome, and fungal.
• If the protein is less than 2.5 g/dL (low-protein ascites), the causes are cirrhosis and nephrotic
syndrome.
• If protein is more than 1.1 g/dL (indicates portal hypertension), the causes are Cardiac ascites,
cirrhosis, and Budd-Chiari syndrome.
• If the protein is <1.1 g/dL (absence of portal hypertension), the causes are TB, peritoneal
carcinomatosis, pancreatic ascites, and nephrotic syndrome.
• If Bloody: trauma, malignancy, TB (rarely)
• If Milky: chylous
• If Turbid: possible infection
• If Straw colour: likely more benign causes

Page 13

323
• If neutrophils are ≥250/mm³, then peritonitis may be due to secondary or spontaneous bacteria.
• If ascitic protein is more than 2.5 g/dL (high-protein ascites), the causes are CHF, constrictive
pericarditis, peritoneal carcinomatosis, TB, Budd-Chiari syndrome, and fungal.
• If the protein is less than 2.5 g/dL (low-protein ascites), the causes are cirrhosis and nephrotic
syndrome.
• If protein is more than 1.1 g/dL (indicates portal hypertension), the causes are Cardiac ascites,
cirrhosis, and Budd-Chiari syndrome.
• If the protein is <1.1 g/dL (absence of portal hypertension), the causes are TB, peritoneal
carcinomatosis, pancreatic ascites, and nephrotic syndrome.
Option A: Decrease in liver function
• Decreased liver function causes decreased serum albumin concentration and increased Prothrombin
time.
• Due to decreased proteins, edema occurs in end-stage liver disease, but this patient's normal serum
albumin makes this option less likely.
Option C: Increased capillary permeability
• Increased capillary permeability causes non-portal hypertension with SAAG less than 1.1g/dL, for
example, malignant ascites.
Option D: Increase Glomerular filtration pressure
• Increased glomerular filtration pressure usually increases the Glomerular filtration rate.
• It does not cause ascites.

Solution for Question 10:


Option B: Chronic liver disease
• The patient presents with erectile dysfunction and testicular atrophy due to hypogonadism, a very bad
complication of chronic liver disease, such as cirrhosis.
• Chronic liver disease causes hypogonadism due to primary gonadal injury or hypothalamic-pituitary
dysfunction. It causes elevated circulating levels of estrogen due to increased conversions from
androgens. Cirrhosis leads to decreased synthesis of serum binding proteins for thyroid hormones,
which lowers the T3 and T4 in the blood, but free T3 and T4 levels are unchanged, but TSH will be
normal, and the patient will be euthyroid.
• It causes elevated circulating levels of estrogen due to increased conversions from androgens.
• Cirrhosis leads to decreased synthesis of serum binding proteins for thyroid hormones, which lowers
the T3 and T4 in the blood, but free T3 and T4 levels are unchanged, but TSH will be normal, and the
patient will be euthyroid.
• It causes elevated circulating levels of estrogen due to increased conversions from androgens.
• Cirrhosis leads to decreased synthesis of serum binding proteins for thyroid hormones, which lowers
the T3 and T4 in the blood, but free T3 and T4 levels are unchanged, but TSH will be normal, and the
patient will be euthyroid.
Option A: Adrenal insufficiency

Page 14

324
• Adrenal insufficiency can cause weakness, weight loss, and anorexia.
• The patient also presents with loss of libido due to decreased production of androgens in females.
• But this does not occur in males because androgens are formed in the testis in males.
• But gynecomastia, testicular trophy, and thyroid hormone deficiency are not seen in this condition, so
this option is ruled out.
Option C: Hashimoto thyroiditis
• Hashimoto thyroiditis is commonly seen in middle-aged females.
• The patient presents with thyroid enlargement called a goiter and Hypothyroid symptoms, such as
weight gain, dry skin, depression, hypersomnia, constipation, etc.
• Laboratory values also show increased TSH levels.
Option D: Hyperthyroidism
• Hyperthyroidism is just the opposite of Hashimoto thyroiditis.
• Patient presents with weight loss, tremors, proptosis, and goiter such as Graves disease, etc.
• Lab value also shows decreased TSH level and increased T3 T4, unlike in this patient.

Solution for Question 11:


Option B: Obtain second intravenous access
• The patient has a decompensated-liver disease and presents with bloody vomiting and
life-threatening hypotension.
• If the patient is suspected of variceal bleeding, always place two large-bore IV catheters, and then
volume resuscitation should be done.
• Octreotide should be given intravenously, followed by antibiotics.
• Octreotide dose: 50 µg as loading dose IV, 50 µg 1 hour in 5% dextrose as maintenance dose.
• After the patient gets stable, urgent endoscopic therapy for variceal bleeding should be done.
• If there is no further bleeding, initiate secondary prophylaxis like beta blockers plus endoscopy band
ligation two weeks later.
• If continued bleeding, then balloon tamponade (temporary) should be placed.
• If early rebleeding is seen, then repeat endoscopic therapy should be indicated.
Option A: Administer octreotide infusion
• The patient is hemodynamically unstable, so volume resuscitation should be considered first.
• Two IV lines should be taken in such patients to give enough fluids to resuscitate.
• This should be followed by IV octreotide and antibiotics.
Option C: Perform-esophagogastroduodenoscopy
• Upper GI endoscopy is indicated after the patient gets stable hemodynamically, so this option is
wrong.
Option D: Perform mesenteric arteriography

Page 15

325
• Mesenteric arteriography is not indicated in this condition, so this option is easily eliminated.

Solution for Question 12:


Option D: AST 1100: ALT 1230
• The patient presents with fatigue, lethargy, anorexia, right upper quadrant pain, hypoglycemia, and
Jaundice due to hyperbilirubinemia, the most likely diagnosis is acute liver failure.
• There are three diagnostic requirements for acute liver failure: Elevated liver enzymes, especially
more than 1000 U/L Signs of hepatic encephalopathy, confusion, asterixis Synthetic liver dysfunction,
e.g., INR more than 1.5 Renal insufficiency and thrombocytopenia can also be seen.
• Elevated liver enzymes, especially more than 1000 U/L
• Signs of hepatic encephalopathy, confusion, asterixis
• Synthetic liver dysfunction, e.g., INR more than 1.5
• Renal insufficiency and thrombocytopenia can also be seen.
• Elevated liver enzymes, especially more than 1000 U/L
• Signs of hepatic encephalopathy, confusion, asterixis
• Synthetic liver dysfunction, e.g., INR more than 1.5
• Renal insufficiency and thrombocytopenia can also be seen.
Option A: AST 200: ALT 300
• There is a marked elevation in AST, and ALT in acute liver failure, usually more than 1000 U/L.
• A slight elevation in liver enzymes is seen in fatty liver.
Option B: AST 850: ALT 900
• There is a marked elevation in AST, and ALT in acute liver failure. Usually, more than 1000 U/L, show
this option is wrong.
Option C: AST 500: ALT 250
• A marked elevation in AST, and ALT in acute liver failure, usually more than 1000 U/L, shows this
option is wrong.

Solution for Question 13:


Option C: Defibrotide
Veno occlusive disease:
• In western countries, Bone marrow transplantation is the commonest risk factor.
• Bush teas and cytosine arabinoside are risk factors.
• There is no difference between Veno occlusive disease and Budd-Chiari syndrome except for the site
of occlusion.
• The investigation of choice for diagnosis – Biopsy.

Page 16

326
• Treatment of choice: If hepatic failure is present – Liver transplantation.
• If there is no hepatic failure – side-to-side portocaval shunt.
• One drug is effective in improving survival – DEFIBROTIDE.
Option A: Feature
• Amiodarone is an antiarrhythmic medication used to treat ventricular tachycardia or ventricular
fibrillation.
Option B: Tenofovir
• Tenofovir is commonly used to treat hepatitis B.
• It is also used in the treatment and prevention of AIDS.
Option D: Ibutilide
• Ibutilide is a class 3 anti-arrhythmic drug.
• It is used in the treatment of atrial fibrillation and atrial flutter.

Solution for Question 14:


Option A: ALT - 220; AST - 111; GGT - increased; FERRITIN - decreased
• This patient with a history of alcohol use likely has alcoholic hepatitis with anorexia and tender
hepatomegaly.
• There is no evidence of ascites or cirrhosis, and he has no weight loss history to suggest malignancy.
• There are no signs of infection as well.
• Alcoholic liver disease is generally characterized by modest elevations of liver enzymes, usually less
than 300 but almost always less than 500 IU/L, but the ratio of AST to ALT is more than 2. There is no
correlation between the degree of elevation of enzymes and liver disease severity.
• Gamma-glutamyltransferase (GGT) is an enzyme in the liver and other cells.
• FERRITIN is an acute phase reactant seen in alcoholic liver disease.
Option B: ALT - 2500; AST - 1050; GGT - increased; FERRITIN - increased
• Marked increases in liver enzymes are seen in toxic-induced liver injuries like paracetamol poisoning
or ischemic or viral hepatitis.
Option C: ALT - 2640; AST - 2355; GGT - increased; FERRITIN - increased
• Marked increases in liver enzymes are seen in toxic-induced liver injuries like paracetamol poisoning
or ischemic or viral hepatitis.
Option D: ALT - 266; AST - 105; GGT - normal; FERRITIN - normal
• Normal liver enzymes and normal GGT are rarely seen in alcoholic hepatitis.

Solution for Question 15:


Option A: Liver transplantation

Page 17

327
• This patient has cirrhosis; the complication is portal hypertension manifested in variceal bleeding with
ascites.
• The child's class 'C' score means no surgical intervention should be done (severe decompensation)
→ Best treatment is liver transplantation.
• Patients with variceal bleeding who are transplantation candidates include Alcoholic cirrhotic patients
with limited hepatic functional reserve (Child-Pugh class B and C). There is poor quality of life
secondary to the disease (encephalopathy, fatigue, bone pain ), and intervention should be done.
• Alcoholic cirrhotic patients with limited hepatic functional reserve (Child-Pugh class B and C).
• There is poor quality of life secondary to the disease (encephalopathy, fatigue, bone pain ), and
intervention should be done.
• Child-Pugh classification:
• Alcoholic cirrhotic patients with limited hepatic functional reserve (Child-Pugh class B and C).
• There is poor quality of life secondary to the disease (encephalopathy, fatigue, bone pain ), and
intervention should be done.
Option B: Major surgery
• Major surgery is indicated in Class B, Child-Pugh Classification.
Option C: Minor surgery
• Minor surgery is indicated in Class A, Child-Pugh Classification.
Option D: TIPS
• TIPS is a transjugular intrahepatic portal-systemic shunt.
• This procedure is used to relieve portal hypertension.
• Hepatic encephalopathy is a notorious complication of TIPS.

Solution for Question 16:


ANSWER
Option A: Extrahepatic portal venous obstruction
• The most likely diagnosis, in this case, is extrahepatic portal vein obstruction.
• Differences between EHPVO and NCPF: EHPVO (Extrahepatic portal venous obstruction) - Usually
seen in 1st to 2nd decade; splenomegaly is less common and if its present, it is only mild/moderate.
NCPF (Non-Cirrhotic portal fibrosis) - In the 2nd to 4th decade; Splenomegaly is most common and if
present, it is moderate to severe; hematemesis present, no obvious jaundice, and ascites.
• EHPVO (Extrahepatic portal venous obstruction) - Usually seen in 1st to 2nd decade; splenomegaly is
less common and if its present, it is only mild/moderate.
• NCPF (Non-Cirrhotic portal fibrosis) - In the 2nd to 4th decade; Splenomegaly is most common and if
present, it is moderate to severe; hematemesis present, no obvious jaundice, and ascites.
• The liver is small but histologically normal in extrahepatic portal vein obstruction resulting from portal
vein malformation or obstruction.

Page 18

328
• EHPVO (Extrahepatic portal venous obstruction) - Usually seen in 1st to 2nd decade; splenomegaly is
less common and if its present, it is only mild/moderate.
• NCPF (Non-Cirrhotic portal fibrosis) - In the 2nd to 4th decade; Splenomegaly is most common and if
present, it is moderate to severe; hematemesis present, no obvious jaundice, and ascites.
Other options
Option B: Non-cirrhotic portal fibrosis
• Non-cirrhotic portal fibrosis, also called obliterative portal venopathy, causes portal hypertension.
• It causes massive splenomegaly, unlike in this patient.
• The patient also presents with variceal bleeding in young adults from low social backgrounds.
Option C: Cirrhosis
• Patients with liver cirrhosis must have some history of hepatitis B/C or alcohol. This patient has no
such history, so liver cirrhosis is less likely.
Option D: Hepatic venous outflow tract obstruction
• Hepatic venous outflow tract obstruction is also called Budd-Chiari syndrome.
• This condition is mostly seen in myeloproliferative disorders in which abnormal growth of cells
occursin the bone marrow.
• Hepatomegaly is also seen, which is absent in the clinical scenario.

Page 19

329
Gallbladder
1. In a patient with diagnosis of choledocholithiasis, the stones were removed (shown in the image).
What is the composition of the stone?
(or)
A 45-year old female presents with complaints of abdominal pain for the past 3 days. She localizes the
pain to her epigastric area and states that it radiates to her right upper quadrant. She notes that it
became markedly worse after eating dinner last night. She recalls a past history of similar pain but has
never had any diagnostic workup. She gave a history of total abdominal hysterectomy 1 year ago. Her
abdominal exam revealed tenderness over her epigastric and right upper quadrants without rebound
tenderness. Bowel sounds are normal. Based on USG and CT, a diagnosis of choledocholithiasis was
made. The patient was operated on, and the stones were removed (shown in the image). What is the
composition of the stone?

A. Calcium oxalate + calcium phosphate + calcium stearate


B. Calcium bilirubinate + calcium palmitate + calcium stearate
C. Insoluble bilirubin polymer + calcium phosphate + calcium bicarbonate
D. Insoluble bilirubin polymer + calcium phosphate + calcium carbonate
----------------------------------------
2. A 42-year old female came to the OPD complaining of pain in the right upper quadrant of the
abdomen since yesterday. On palpation, tenderness in the right hypochondrium was elicited. The
patient was suspected of having gallstones. What is the investigation of choice in this case?
(or)
What is the investigation of choice in patients suspected of having gallstones?
A. Ultrasound
B. X-ray
C. Barium study
D. Oral cholecystography
----------------------------------------
3. A 39-year old female came to the OPD complaining of pain in the right upper quadrant of the
abdomen for the past 5 days. The patient was further evaluated, and CT abdomen was done. What is
your diagnosis based on the sign shown in the CT image?
(or)

330
What is your diagnosis based on the sign shown in the CT image?

A. Gallstones
B. Liver abscess
C. Common bile duct stones
D. Hydatid cyst
----------------------------------------
4. A 46-year old woman was admitted to your hospital due to appetite loss, nausea, and back pain. A
physical examination disclosed right hypochondriac tenderness and obvious jaundice. Magnetic
resonance cholangiography revealed that the intrahepatic and common hepatic bile ducts were dilated
and that the gallbladder was distended. Endoscopic retrograde cholangiography demonstrated a
round-shaped filling defect at the confluence of the bile duct. Based on the findings, the patient was
diagnosed with Mirizzi syndrome Type II according to the Csendes classification. What is the best next
step in the management of this case?
(or)
A 46-year-old woman was diagnosed with Mirizzi syndrome Type II according to the Csendes
classification. What is the best next step?
A. Partial cholecystectomy + T- tube drainage
B. Conservative management
C. Partial cholecystectomy alone
D. Partial cholecystectomy + Bilioenteric anastomosis
----------------------------------------
5. A 46-year-old female came to your OPD and gave a vague history of occasional mild abdominal pain
with no other complaints. She further requests you order a CT scan, as her relatives told her to do a
general body check-up just to be sure. You examined the patient and ordered for CT abdomen as she
kept insisting on it. The next day patient came with a CT film which revealed the following finding as
shown in the image. How will you manage this case?
(or)
A 46-year-old female came to your OPD and gave a vague history of occasional mild abdominal pain.
The next day patient came with a CT film which revealed the following finding as shown in the image.
How will you manage this case?

Page 2

331
A. Reassurance and conservative management
B. Cholecystectomy
C. Left Hepatectomy
D. Extended left Hepatectomy
----------------------------------------
6. A 62-year-old female with a history of hypertension presented with diffuse abdominal pain and
decreased appetite for 1 day. On examination, she had right upper quadrant tenderness, rebound, and
a positive Murphy’s sign. Ultrasound showed stones, fluid around the gallbladder, and wall thickening;
CT showed wall thickening. The patient was taken for laparoscopic cholecystectomy. The specimen
appeared normal and was retrieved in a specimen bag. Pathology showed carcinoma gallbladder with
the invasion of perimuscular connective tissue without serosal involvement. Which of the following is
the most appropriate treatment option?
(or)
A 62-year-old female comes with diffuse abdominal pain and decreased appetite for 1 day. Pathology
showed carcinoma gallbladder with the invasion of perimuscular connective tissue without serosal
involvement. Which of the following is a treatment option?
A. Resection of segments IVb and V of liver
B. Resection of segments IVb and V of the liver with nodal clearance
C. Palliative management with duodenal wall stent
D. Wedge excision of the liver with lymphadenectomy and port site excision
----------------------------------------
7. A 45-year-old female came to the OPD complaining of pain and tenderness in the right
hypochondrium. USG revealed an inflamed gallbladder with multiple stones, the largest stone
measuring 1 cm in diameter. The patient was planned for laparoscopic cholecystectomy and taken up
for surgery. Which of the following is an absolute contraindication to laparoscopic cholecystectomy?
(or)
Which of the following is an absolute contraindication to laparoscopic cholecystectomy?
A. Gallbladder carcinoma
B. Cholangitis
C. Morbid obesity
D. Diffuse peritonitis

Page 3

332
----------------------------------------
8. A 60-year-old previously well-female was admitted to our hospital with generalized abdominal pain,
fever, and vomiting. She also gave a history of being operated on for total hip replacement surgery
recently. On examination, the patient was afebrile and her abdomen was soft with generalized
tenderness and guarding. Laboratory tests showed mild elevation of white cell count and her
biochemical studies, including liver function, renal function, and electrolyte levels, were normal. CT
scan of the abdomen showed evidence of pericholecystic fluid with no gallstones and no gallbladder
wall thickening. What is the best next step in the treatment for this patient?
(or)
A 60-year-old comes with generalized abdominal pain, fever & vomiting and went total hip replacement
surgery recently. CT scan of the abdomen showed evidence of pericholecystic fluid with no gallstones
or gallbladder wall thickening. The best next step in the treatment for this patient would be?
A. Cholecystectomy
B. Conservative management
C. Percutaneous cholecystostomy
D. Endoscopic cholangiography + Drainage
----------------------------------------
9. A 38-year-old female came to the OPD complaining of abdominal pain and jaundice. The patient
gives similar episodes of abdominal pain in the past. The pain gets aggravated by eating fatty meals.
Liver function tests revealed elevated levels of total and direct bilirubin. Gamma-glutamyl transferase
and alkaline phosphatase levels were also found to be elevated. These findings suggest a biliary
obstruction causing obstructive jaundice. Which of the following is the initial investigation of choice for
biliary obstruction?
(or)
Which of the following is the initial investigation of choice for biliary obstruction?
A. CT Abdomen
B. Magnetic resonance cholangiopancreatography(MRCP)
C. Endoscopic retrograde cholangiopancreatography (ERCP)
D. Ultrasound (USG)
----------------------------------------
10. A 42-year-old woman presented to the clinic with a complaint of right upper quadrant pain last
week. The pain worsens after meals. Last week she had two episodes of pain which subsided after
some time. The physician ordered an ultrasound abdomen which revealed gallstones. The patient
inquired about the reason for the gallstones. Which of the following is an important factor in the
formation of cholesterol gallstones?
(or)
Which is an important factor in the formation of cholesterol gallstones?
A. An increase in bile salt concentration leads to increased solubility of cholesterol in bile, leading to
stone formation
B. Progesterone-containing oral contraceptives can predispose to gallstone formation
C. Primary event in the formation of cholesterol stones is increased stasis in the gallbladder, allowing
more time for solutes to precipitate in the gallbladder

Page 4

333
D. Age, sex, genetics, obesity, and diet play a role in the supersaturation of bile
----------------------------------------
11. Below is an image of the specimen retrieved during laparoscopic cholecystectomy in a 36-year-old
female who came to the OPD complaining of swelling in the right hypochondrium. Which of the
following statement is related to the condition shown in the image?
(or)
Given below is an image of the specimen retrieved during laparoscopic cholecystectomy in a
36-year-old female who came to the OPD with complaints of swelling in the right hypochondrium.
Which of the following statements is related to the condition shown in the image?

A. Gall stones are not related to it


B. Produces the classic studded macroscopic appearance of a strawberry gallbladder
C. Obstruction at the fundus of gallbladder
D. Gallbladder is never palpable
----------------------------------------
12. A 35-year-old male came to the OPD complaining of abdominal pain in the right upper quadrant.
Examination revealed tenderness in the right hypochondrium. An X-ray of the abdomen revealed
multiple gallbladder stones. What is the percentage of radiopaque gallstones?
(or)
What percentage of gallstones are radiopaque?
A. 10%
B. 20%
C. 30%
D. 40%
----------------------------------------
13. Which of the following risk factors given below is associated with gallbladder carcinoma?
(or)
Physician explains the various risk factors for gallbladder carcinoma to a patient. Which of the following
risk factors given below is associated with gallbladder carcinoma?
A. Oral contraceptive pills
B. Gallstones

Page 5

334
C. Acute cholecystitis
D. Mucocele
----------------------------------------
14. The technique of laparoscopic cholecystectomy was first described by?
(or)
Removal of the gallbladder in case of gallstones by laparoscopic cholecystectomy has become popular
nowadays. The technique of laparoscopic cholecystectomy was first described by?
A. Erich Muhe
B. Philippe Mouret
C. Kurt Semm
D. Eddie Reddick
----------------------------------------
15. A 43-year-old woman is admitted with a complaint of right upper quadrant pain. Ultrasound shows
stones and stricture in the common bile duct. Her surgical history is significant for recurrent common
bile duct stones and ERCP procedures for removal. A procedure is performed to fix this recurrent stone
formation. Which of the following complication of the procedure is shown in the below image?
(or)
Which of the following complication of a procedure is shown in the below image?

A. Short bowel syndrome


B. Sump syndrome
C. Mirizzi’s syndrome
D. Anastomosis leak
----------------------------------------
16. A 57-year-old woman presented with right upper quadrant pain associated with 2 episodes of
vomiting. On examination, tenderness was noted in the right upper quadrant. On USG, a large normal
functioning, and thin-walled gall bladder with multiple small radiopaque stones of varying size were
noted. The patient was advised to have surgery for stone removal. Which of the following is a
contraindication for the medical management of gallstones in this patient?
(or)
Which of the following is a contraindication for medically managing gallstones in a patient?

Page 6

335
A. Radiopaque stones
B. Female gender
C. Normal functioning gallbladder
D. Small stones
----------------------------------------
17. A 71-year-old male was referred to the surgery department by his family physician for an incidental
finding of a gallstone. The patient denied any abdominal pain, nausea, vomiting, or jaundice. What is
the treatment of choice for this patient?
(or)
What is the treatment of choice for patients with incidental finding of gall stone?
A. Observation
B. Ursodeoxycholic acid
C. Cholecystectomy
D. Lithotripsy
----------------------------------------
18. A 46-year-old woman was admitted to your hospital with recurrent upper abdominal pain for one
year. The patient also had presented with diabetes/glucose intolerance, steatorrhea, gallstones, and
achlorhydria.. Computed tomography (CT) indicated an isodense mass of 8.4 cm×3.7 cm in the body
and tail of the pancreas with notable enhancement. What is your probable diagnosis?
A. Insulinoma
B. VIPoma
C. Somatostatinoma
D. Glucagonoma
----------------------------------------
19. A 65-year-old male is admitted to the surgical department. His history is significant for yellowing of
the eyes, pruritus, and weight loss. He has 15 years of smoking history. On examination, the patient’s
gall bladder is palpable. Which of the following statements describes Courvoisier’s law?
(or)
Which of the following statement describes Courvoisier’s law?
A. In obstruction of the common bile duct due to a stone, the organ usually is distended owing to the
obstructing of biliary flow
B. In a jaundiced patient, if the gallbladder is palpable, then it is due to stones
C. In obstructions caused due to cancer of the ampulla, pancreas, or bile duct, the gall bladder will not
get distended
D. Mucocele due to stone in the cystic duct is an exception to this law
----------------------------------------
20. A 36-year-old female came to the OPD with complaints of mild right upper quadrant pain. The
patient was further evaluated and diagnosed as having carcinoma of the gallbladder with invasion into
perimuscular connective tissue. An extended cholecystectomy was done in this patient that included

Page 7

336
the removal of which of the following structures?
(or)
Extended cholecystectomy was done in a patient, includes removing which of the following structures?
A. Segment IV b and V of the liver, gallbladder, peri-choledochal lymph node
B. Gallbladder, right hepatic bile duct, peri-choledochal lymph node
C. Peri-choledochal lymph node, segment IV b and V of the liver, right hepatic bile duct
D. Right hepatic bile duct, gallbladder, segment IV b and V of liver
----------------------------------------
21. Given below is a specimen of gallbladder removed during cholecystectomy in a 40-year-old female
patient diagnosed with cholelithiasis. The student asks the consultant about the composition of stones.
Which of the following is the best statement about gallstone composition?
(or)
Which of the following is the best statement about gallstone composition?

A. Most common type of gallstone is mixed


B. Black stones are hard and radiolucent
C. Brownstones are most common in the gallbladder
D. Most common type of gallstone in the Asian population is cholesterol stones
----------------------------------------
22. A 38-year-old female presents to the emergency room with a complaint of high-grade fever, chills,
and abdominal pain. The temperature is 102°F, and the blood pressure is 90/60 mm Hg. Her medical
history is significant for diabetes mellitus. An X-ray abdomen is taken. Identify the type of cholecystitis
given in the image below. What is the best treatment option for this?
(or)
A 38-year-old female presents with a complaint of high-grade fever, chills and abdominal pain. Her
medical history is significant for diabetes mellitus. What is the best treatment option for the type of
cholecystitis?

Page 8

337
A. Emergency cholecystectomy under antibiotic coverage
B. Conservative management followed by interval cholecystectomy
C. Conservative treatment and treat primary causes like diabetes mellitus
D. Urgent endoscopic retrograde cholangiopancreatography
----------------------------------------
23. A 55-year-old woman has been admitted to the high-dependency unit for the last three weeks. She
sustained a road traffic accident three weeks ago. Her GCS is 9. Her medical history is significant for
diabetes mellitus. On routine monitoring, her temperature has been 101°F for the last two days. An
ultrasound abdomen is performed, and a procedure given in the image below is performed in which of
the following conditions?
(or)
A procedure given in the image below is performed in which of the following conditions?

A. Mucocele
B. Mirizzi’s syndrome
C. Xanthogranulomatous cholecystitis
D. Acalculous cholecystitis
----------------------------------------
24. A 32-year-old female underwent elective cholecystectomy. The gallbladder is sent for histological
examination. Which of the following statements is related to the condition in the image below?
(or)

Page 9

338
A 32-year-old female underwent elective cholecystectomy. The gallbladder is sent for histological
examination. Which of the following statements is related to the condition given in the image below?

A. It is an inherited condition
B. These are only cholesterol gallstones
C. Associated with cholesterol polyp
D. Cholesterol deposits in the submucosa of the gallbladder
----------------------------------------
25. Given below is a specimen of the gallbladder. What is your likely diagnosis?
(or)
A 43-year-old female underwent elective laparoscopic cholecystectomy. The gallbladder is sent for
histopathological diagnosis. Given below is a specimen of the gallbladder. What is your likely
diagnosis?

A. Diverticulosis of gallbladder
B. Porcelain gallbladder
C. Strawberry gallbladder
D. Polyps in the gallbladder
----------------------------------------
26. A 44-year-old female presents to the clinic for a comprehensive medical evaluation. Given below is
an ultrasound image of the gallbladder done in a patient. What is the most probable diagnosis?
(or)
A 44-year-old female had an ultrasound of the gallbladder. What is the most probable diagnosis?

Page 10

339
A. Gallbladder stone
B. Gallbladder polyp
C. Adenomyomatosis
D. Xanthogranulomatous cholecystitis
----------------------------------------
27. A 79-year-old woman was admitted to your hospital for suspected acute cholecystitis and had
Murphy’s sign on admission. The ultrasound images revealed many small stones and sludge in the
gallbladder. CT images similarly revealed a thick-walled gallbladder containing multiple stones; Multiple
3-cm-sized round nodular lesions with heterogeneous enhancement were also noted along the cystic
duct and common bile duct, which were suggestive of 1-3 metastatic lymph nodes. Our surgical
procedure included radical cholecystectomy with partial liver resection together with lymph node
dissection. Pathology of the resected specimen showed that the entire gallbladder was tumorous.
Additionally, the tumours invaded the liver. Histopathological examination of the tumours revealed
neuroendocrine carcinoma, a small cell type. Identify the stage of tumour in this patient.
A. T3N1M0
B. T2aN2M0
C. T3N2M1
D. T4N1M0
----------------------------------------
28. A 32-year-old female presents to the clinic for a comprehensive medical evaluation. An ultrasound
abdomen is performed, and an unusual finding is appreciated. What is the name of the congenital
anomaly seen on the ultrasound image below?
(or)
What is the name of the congenital anomaly seen on the ultrasound in 32 year old female given below?

Page 11

340
A. Moynihan’s hump
B. Phrygian cap
C. Duplication
D. None of the above
----------------------------------------
29. Which of the following statements are best about the anatomy and physiology of the gallbladder?
A. Concentration of bile occurs by active absorption of water, sodium chloride, and bicarbonate via the
mucous membrane of the gallbladder
B. Gallbladder is 5-7 cm long, with a normal capacity of about 15-20 ml
C. Gall bladder lacks a serosa and submucosa
D. Bile is concentrated 10-15 times in the gallbladder
E. The sphincter of Lutkens is present in the submucosa of the cystic duct
----------------------------------------
30. A 30-year-old woman presented with a one-day history of colicky epigastric pain and vomiting. She
also reported two days of constipation and not passing flatus. She had no previous medical history,
denied any previous biliary symptoms, had no history of cholelithiasis and had no previous abdominal
surgery. She was hemodynamically stable and afebrile on presentation. Examination revealed a soft
abdomen with moderate distension and epigastric tenderness; however, no rebound tenderness or
guarding was noted. Routine blood tests were unremarkable. CT of the abdomen demonstrated the
findings shown in the image. Which of the following is the best statement about the given condition?
(or)
A 30-year-old woman presented with colicky epigastric pain and vomiting. She also reported two days
of constipation and not passing flatus. She was hemodynamically stable and afebrile. CT of the
abdomen is shown in the image. Which of the following is the best statement?

Page 12

341
A. The most common site of the fistula is cholecystocolic
B. Stone moves from the gallbladder and drops into GIT through the fistula causing obstruction in the
jejunal part of the small intestine
C. Rigler’s triad: Large bowel obstruction + Pneumobilia + Ectopic gallstone
D. Most commonly seen in elderly
----------------------------------------
31. A patient is admitted for hepatocellular carcinoma. Couinaud divided the liver into eight segments.
Cholecystocaval line separates which of the following structures?
(or)
A patient is admitted to the surgical department for hepatocellular carcinoma. On the clinical rounds,
the professor decides on the surgical resection of the tumour without damaging the liver structures.
Based on the distribution of the portal vein and hepatic vein, Couinaud divided the liver into eight
segments. Cholecystocaval line separates which of the following structures?
A. Right anterior and right posterior sectors
B. Gallbladder from the portal vein
C. Right and left hepatic lobes
D. Left medial and left lateral sectors
----------------------------------------
32. The image given below indicates which of the following procedures?
(or)
A 43-year-old female presents to the clinic with a complaint of recurrent right upper quadrant pain. They
exacerbate, especially after eating. An ultrasound abdomen is performed, which comes out to be
non-conclusive. The image given below indicates which of the following procedures?

Page 13

342
A. Percutaneous transhepatic cholangiogram
B. T-tube cholangiogram
C. Endoscopic retrograde cholangiopancreatography (ERCP)
D. Hepatobiliary iminodiacetic acid (HIDA) scan
----------------------------------------
33. What is the diagnosis based on the HIDA scan given below in a patient with fever, right upper
quadrant pain, nausea and loss of appetite and Murphy's sign positive?
(or)
A 36-year-old female presents to the clinic with a complaint of fever, right upper quadrant pain, nausea,
and loss of appetite. On examination, murphy’s sign is positive. An ultrasound result is non-conclusive.
To get a final diagnosis, a HIDA scan is ordered. What is the diagnosis based on the hepatobiliary
iminodiacetic acid (HIDA) scan given below?

A. Chronic cholecystitis
B. Mucocele
C. Acute cholecystitis
D. Normal scan
----------------------------------------
34. A 42-year-old male presents to the clinic complaining of intermittent abdominal pain, nausea, and
bloating. These symptoms are associated with occasional diarrhea and constipation. The physician
orders a stool microscopy examination. Which of the following is the most serious and dreaded
complication of infestation caused by the parasite shown in the image?

Page 14

343
(or)
Which of the following is the most severe and dreaded infestation complication caused by the parasite
shown in the image?

A. Urinary bladder carcinoma


B. Cholangiocarcinoma
C. Hepatocellular carcinoma
D. Neurocysticercosis
----------------------------------------
35. A 60-year-old man presented with four days of right upper abdominal pain with nausea, vomiting,
anorexia, flatulence, and bloating. The patient is a known case of Crohn’s disease with a history of
small bowel obstruction (SBO) and multiple surgeries. On examination, the patient was tender to light
palpation over the right upper quadrant and epigastric region. No rigidity, rebound tenderness, or
guarding was noted. His blood report showed elevated WBC (>21000 /mm3 ) with neutrophilic shift and
C-reactive protein > 250 with normal lipase, liver enzymes, and renal function. CT was taken, which
showed enlargement and thickening of the gallbladder wall, gallstones in the gallbladder neck, and
pericholecystic fat stranding. With the given information, a diagnosis of acute cholecystitis was made.
According to the Tokyo Consensus Guidelines, which of the following is the grading for this case?
(or)
A 60-year-old gentleman presented with four days of right upper abdominal pain with nausea, vomiting,
anorexia, flatulence, and bloating. His bloodwork showed elevated WBC (>21000 /mm3 )with the
neutrophilic shift. CT showed enlargement & thickening of the gallbladder wall, gallstones in the
gallbladder neck, and pericholecystic fat stranding. A diagnosis of acute cholecystitis was made.
According to the Tokyo Consensus Guidelines, what is this case's grading?
A. Grade I
B. Grade II
C. Grade III
D. Grade I
----------------------------------------

Correct Answers
Question Correct Answer

Page 15

344
Question 1 2
Question 2 1
Question 3 1
Question 4 1
Question 5 2
Question 6 2
Question 7 1
Question 8 1
Question 9 4
Question 10 4
Question 11 2
Question 12 1
Question 13 2
Question 14 1
Question 15 2
Question 16 1
Question 17 1
Question 18 3
Question 19 4
Question 20 1
Question 21 1
Question 22 1
Question 23 4
Question 24 3
Question 25 4
Question 26 3
Question 27 1
Question 28 2
Question 29 1
Question 30 4
Question 31 3
Question 32 3
Question 33 3
Question 34 2
Question 35 2

Page 16

345
Solution for Question 1:
Option B: Calcium bilirubinate + calcium palmitate + calcium stearate
• The stones shown in the image are brownstones.
• Gallstones: The major organic solutes in the bile are bilirubin, bile salts, phospholipids, and
cholesterol. Gallstones are classified by their cholesterol content as either cholesterol stones or
pigment stones. Pigment stones can be further classified as either black or brown. Pigment stones
contain <30% cholesterol.Overall, 20–30% of pigment stones are black Black stones - Insoluble
bilirubin polymer + calcium phosphate + calcium bicarbonate. These stones are associated with
haemolysis, as in hereditary spherocytosis and sickle cell disease Brown pigment stones- calcium
bilirubinate + calcium palmitate + calcium stearate+ cholesterol Brownstones are brownish-yellow, soft,
and mushy. They may form either in the gallbladder or in the bile ducts, secondary to bacterial infection
and bile stasis.
• The major organic solutes in the bile are bilirubin, bile salts, phospholipids, and cholesterol.
• Gallstones are classified by their cholesterol content as either cholesterol stones or pigment stones.
• Pigment stones can be further classified as either black or brown.
• Pigment stones contain <30% cholesterol.Overall, 20–30% of pigment stones are black Black stones -
Insoluble bilirubin polymer + calcium phosphate + calcium bicarbonate. These stones are associated
with haemolysis, as in hereditary spherocytosis and sickle cell disease Brown pigment stones- calcium
bilirubinate + calcium palmitate + calcium stearate+ cholesterol Brownstones are brownish-yellow, soft,
and mushy. They may form either in the gallbladder or in the bile ducts, secondary to bacterial infection
and bile stasis.
• Black stones - Insoluble bilirubin polymer + calcium phosphate + calcium bicarbonate. These stones
are associated with haemolysis, as in hereditary spherocytosis and sickle cell disease
• Brown pigment stones- calcium bilirubinate + calcium palmitate + calcium stearate+ cholesterol
Brownstones are brownish-yellow, soft, and mushy. They may form either in the gallbladder or in the
bile ducts, secondary to bacterial infection and bile stasis.
• Brownstones are brownish-yellow, soft, and mushy. They may form either in the gallbladder or in the
bile ducts, secondary to bacterial infection and bile stasis.
• The major organic solutes in the bile are bilirubin, bile salts, phospholipids, and cholesterol.
• Gallstones are classified by their cholesterol content as either cholesterol stones or pigment stones.
• Pigment stones can be further classified as either black or brown.
• Pigment stones contain <30% cholesterol.Overall, 20–30% of pigment stones are black Black stones -
Insoluble bilirubin polymer + calcium phosphate + calcium bicarbonate. These stones are associated
with haemolysis, as in hereditary spherocytosis and sickle cell disease Brown pigment stones- calcium
bilirubinate + calcium palmitate + calcium stearate+ cholesterol Brownstones are brownish-yellow, soft,
and mushy. They may form either in the gallbladder or in the bile ducts, secondary to bacterial infection
and bile stasis.
• Black stones - Insoluble bilirubin polymer + calcium phosphate + calcium bicarbonate. These stones
are associated with haemolysis, as in hereditary spherocytosis and sickle cell disease
• Brown pigment stones- calcium bilirubinate + calcium palmitate + calcium stearate+ cholesterol
Brownstones are brownish-yellow, soft, and mushy. They may form either in the gallbladder or in the
bile ducts, secondary to bacterial infection and bile stasis.
• Brownstones are brownish-yellow, soft, and mushy. They may form either in the gallbladder or in the
bile ducts, secondary to bacterial infection and bile stasis.

Page 17

346
• Black stones - Insoluble bilirubin polymer + calcium phosphate + calcium bicarbonate. These stones
are associated with haemolysis, as in hereditary spherocytosis and sickle cell disease
• Brown pigment stones- calcium bilirubinate + calcium palmitate + calcium stearate+ cholesterol
Brownstones are brownish-yellow, soft, and mushy. They may form either in the gallbladder or in the
bile ducts, secondary to bacterial infection and bile stasis.
• Brownstones are brownish-yellow, soft, and mushy. They may form either in the gallbladder or in the
bile ducts, secondary to bacterial infection and bile stasis.
• Brownstones are brownish-yellow, soft, and mushy. They may form either in the gallbladder or in the
bile ducts, secondary to bacterial infection and bile stasis.
Option A: Calcium oxalate + calcium phosphate + calcium stearate
• Calcium oxalate and calcium phosphate stones are more commonly seen in the kidney.
• Therefore, this option is incorrect.
Option C: Insoluble bilirubin polymer + calcium phosphate + calcium bicarbonate
• This composition is of black pigmented stones.
Option D: Insoluble bilirubin polymer + calcium phosphate + calcium carbonate
• This composition is not correct for the above-given brownstones.

Solution for Question 2:


Answer
Option A: Ultrasound
• Ultrasound is considered the most useful initial radiological test of choice for diagnosing cholelithiasis
and acute cholecystitis, with a sensitivity and specificity of 84% to 99%
• Ultrasound is effective in documenting the presence or absence of stones, and it can show gallbladder
wall thickening and pericholecystic fluid, both of which are highly suggestive of acute cholecystitis.
• Most of the gallstones are radiolucent and hence cannot be detected in X-rays
Other Options
Option B: X-ray
• Because only 10% of stones are calcified, this imaging study has limited usefulness.
• However, porcelain gallbladder can also be seen in plain X-rays.
Option C: Barium study
• Barium study is used to highlight the gastrointestinal tract.
• It is not useful to study gallbladder.
Option D: Oral cholecystography
• Oral cholecystography is a procedure used to image the gallbladder.
• Although rarely performed now, following oral administration, the contrast agent gets absorbed by the
bowel and concentrated in the gall bladder.
• Radiolucent gallstones may then be outlined as filling defects on fluoroscopic imaging.

Page 18

347
Solution for Question 3:
Option A: Gallstones
• The CT image shows the Mercedes-Benz sign; this sign is seen in Gallstones. The center of the
gallstone may contain radiolucent gas in a triradiate or biradiate fissure; this gives rise to characteristic
dark shapes on the radiograph suggestive of the triradiate - Mercedes-Benz sign, biradiate - Seagull
sign. A plain radiograph of the gallbladder will show radiopaque gallstones in 10% of patients.
• The center of the gallstone may contain radiolucent gas in a triradiate or biradiate fissure; this gives
rise to characteristic dark shapes on the radiograph suggestive of the triradiate - Mercedes-Benz sign,
biradiate - Seagull sign.
• A plain radiograph of the gallbladder will show radiopaque gallstones in 10% of patients.
• The center of the gallstone may contain radiolucent gas in a triradiate or biradiate fissure; this gives
rise to characteristic dark shapes on the radiograph suggestive of the triradiate - Mercedes-Benz sign,
biradiate - Seagull sign.
• A plain radiograph of the gallbladder will show radiopaque gallstones in 10% of patients.

Option B: Liver abscess


• In general, they appear as peripherally enhancing, centrally hypoattenuating lesions.
• Occasionally they appear solid or contain gas in the form of bubbles or air-fluid levels.
• The “double target sign” is a characteristic imaging feature of hepatic abscess, in which a central
low-attenuation lesion (fluid-filled) is surrounded by a high-attenuation inner rim and a low-attenuation
outer rim.

Page 19

348
Option C: Common bile duct stones
• The sensitivity of CT to detect stones in the common bile duct is 65%-88%.
• Target sign: Central round density of the stone and surrounding lower attenuating bile or mucosa.
• Rim sign: Stone is outlined by a thin shell of density.
• Crescent sign: Bile eccentrically outlines luminal stone, creating a low attenuation crescent.

Option D: Hydatid cyst


• Fluid-filled cyst, with frequent peripheral focus areas of calcification, usually indicates no active
infection if completely circumferential.
• The “water-Lily sign” indicates a cyst with a floating, undulating membrane caused by a detached
endocyst.
• Hyperdense internal septa within a cyst showing a spoke wheel pattern may also be seen.

Page 20

349
Solution for Question 4:
ANSWER
Option A: Partial cholecystectomy + T- tube drainage
• The best treatment for Mirizzi syndrome type II is partial cholecystectomy + T- tube drainage
• Mirizzi’s syndrome Obstruction of the bile ducts due to a severe pericholecystic inflammation
secondary to the impaction of a stone in the infundibulum of the gallbladder is known as Mirizzi’s
syndrome. The prerequisite for this syndrome, characterized by gallbladder disease causing
obstructive jaundice, include: A cystic duct that courses parallel to the common hepatic duct An
impacted stone in the gallbladder neck or cystic duct Obstruction of the common hepatic duct is caused
by the stone or inflammatory response. The resultant inflammation can cause a cholecystocholedochal
fistula. It usually presents as jaundice or recurrent cholangitis. The presence of Mirizzi syndrome
obliterates the triangle of Calot.
• Obstruction of the bile ducts due to a severe pericholecystic inflammation secondary to the impaction
of a stone in the infundibulum of the gallbladder is known as Mirizzi’s syndrome.
• The prerequisite for this syndrome, characterized by gallbladder disease causing obstructive jaundice,
include: A cystic duct that courses parallel to the common hepatic duct An impacted stone in the
gallbladder neck or cystic duct Obstruction of the common hepatic duct is caused by the stone or
inflammatory response. The resultant inflammation can cause a cholecystocholedochal fistula. It
usually presents as jaundice or recurrent cholangitis. The presence of Mirizzi syndrome obliterates the
triangle of Calot.
• A cystic duct that courses parallel to the common hepatic duct
• An impacted stone in the gallbladder neck or cystic duct
• Obstruction of the common hepatic duct is caused by the stone or inflammatory response. The
resultant inflammation can cause a cholecystocholedochal fistula.
• It usually presents as jaundice or recurrent cholangitis. The presence of Mirizzi syndrome obliterates
the triangle of Calot.

Page 21

350
• Obstruction of the bile ducts due to a severe pericholecystic inflammation secondary to the impaction
of a stone in the infundibulum of the gallbladder is known as Mirizzi’s syndrome.
• The prerequisite for this syndrome, characterized by gallbladder disease causing obstructive jaundice,
include: A cystic duct that courses parallel to the common hepatic duct An impacted stone in the
gallbladder neck or cystic duct Obstruction of the common hepatic duct is caused by the stone or
inflammatory response. The resultant inflammation can cause a cholecystocholedochal fistula. It
usually presents as jaundice or recurrent cholangitis. The presence of Mirizzi syndrome obliterates the
triangle of Calot.
• A cystic duct that courses parallel to the common hepatic duct
• An impacted stone in the gallbladder neck or cystic duct
• Obstruction of the common hepatic duct is caused by the stone or inflammatory response. The
resultant inflammation can cause a cholecystocholedochal fistula.
• It usually presents as jaundice or recurrent cholangitis. The presence of Mirizzi syndrome obliterates
the triangle of Calot.
• A cystic duct that courses parallel to the common hepatic duct
• An impacted stone in the gallbladder neck or cystic duct
• Obstruction of the common hepatic duct is caused by the stone or inflammatory response. The
resultant inflammation can cause a cholecystocholedochal fistula.
• It usually presents as jaundice or recurrent cholangitis. The presence of Mirizzi syndrome obliterates
the triangle of Calot.
Other Options
Option B: Conservative management
• The most common complication of Mirizzi syndrome is cholecystobiliary or cholecysto-enteric fistula
formation due to prolonged inflammation.
• Therefore, surgical management is required.
Option C: Partial cholecystectomy alone
• Partial cholecystectomy alone is the treatment of choice for type I choledochal cysts.
Option D: Partial cholecystectomy + Bilioenteric anastomosis
• The treatment of choice for type IV and V Partial cholecystectomy + Bilioenteric anastomosis.

Solution for Question 5:


Option B: Cholecystectomy

Page 22

351
• The given image is of Porcelain Gallbladder Porcelain gallbladder is characterized by dense
circumferential calcification of the gallbladder wall Clinical Features: Most porcelain gallbladders (90%)
are associated with gallstones It is a rare premalignant condition that is associated with an
approximately 10% risk of gallbladder carcinoma Diagnosis: Generally diagnosed incidentally on X-Ray
or CT but is more commonly seen on CT. In ultrasound, it will appear as a curvilinear echogenic focus
along the entire gallbladder wall with posterior shadowing. Treatment: Cholecystectomy is the
treatment of choice in patients with porcelain gallbladder, even in asymptomatic patients (due to the
high incidence of development of carcinoma gallbladder)
• Porcelain gallbladder is characterized by dense circumferential calcification of the gallbladder wall
• Clinical Features: Most porcelain gallbladders (90%) are associated with gallstones It is a rare
premalignant condition that is associated with an approximately 10% risk of gallbladder carcinoma
• Most porcelain gallbladders (90%) are associated with gallstones
• It is a rare premalignant condition that is associated with an approximately 10% risk of gallbladder
carcinoma
• Diagnosis: Generally diagnosed incidentally on X-Ray or CT but is more commonly seen on CT. In
ultrasound, it will appear as a curvilinear echogenic focus along the entire gallbladder wall with
posterior shadowing.
• Generally diagnosed incidentally on X-Ray or CT but is more commonly seen on CT. In ultrasound, it
will appear as a curvilinear echogenic focus along the entire gallbladder wall with posterior shadowing.
• Treatment: Cholecystectomy is the treatment of choice in patients with porcelain gallbladder, even in
asymptomatic patients (due to the high incidence of development of carcinoma gallbladder)
• Cholecystectomy is the treatment of choice in patients with porcelain gallbladder, even in
asymptomatic patients (due to the high incidence of development of carcinoma gallbladder)
• Porcelain gallbladder is characterized by dense circumferential calcification of the gallbladder wall
• Clinical Features: Most porcelain gallbladders (90%) are associated with gallstones It is a rare
premalignant condition that is associated with an approximately 10% risk of gallbladder carcinoma
• Most porcelain gallbladders (90%) are associated with gallstones

Page 23

352
• It is a rare premalignant condition that is associated with an approximately 10% risk of gallbladder
carcinoma
• Diagnosis: Generally diagnosed incidentally on X-Ray or CT but is more commonly seen on CT. In
ultrasound, it will appear as a curvilinear echogenic focus along the entire gallbladder wall with
posterior shadowing.
• Generally diagnosed incidentally on X-Ray or CT but is more commonly seen on CT. In ultrasound, it
will appear as a curvilinear echogenic focus along the entire gallbladder wall with posterior shadowing.
• Treatment: Cholecystectomy is the treatment of choice in patients with porcelain gallbladder, even in
asymptomatic patients (due to the high incidence of development of carcinoma gallbladder)
• Cholecystectomy is the treatment of choice in patients with porcelain gallbladder, even in
asymptomatic patients (due to the high incidence of development of carcinoma gallbladder)
• Most porcelain gallbladders (90%) are associated with gallstones
• It is a rare premalignant condition that is associated with an approximately 10% risk of gallbladder
carcinoma
• Generally diagnosed incidentally on X-Ray or CT but is more commonly seen on CT. In ultrasound, it
will appear as a curvilinear echogenic focus along the entire gallbladder wall with posterior shadowing.
• Cholecystectomy is the treatment of choice in patients with porcelain gallbladder, even in
asymptomatic patients (due to the high incidence of development of carcinoma gallbladder)
Option A: Reassurance and conservative management
• Porcelain gallbladder is associated with a 10% risk of gallbladder carcinoma
• Therefore, cholecystectomy is the best next step.
Option C: Left Hepatectomy
• Left hepatectomy is not required because it is not a malignant condition.
Option D: Extended left hepatectomy
• Extended left hepatectomy is not required because it is not a malignant condition.

Solution for Question 6:


Option B: Resection of segments IVb and V of the liver with nodal clearance
• Carcinoma gallbladder with an invasion of perimuscular connective tissue without serosal involvement
is stage T2.
• In T2 lesions, radical(extended) cholecystectomy is indicated because more than 40% of these
patients have lymph node metastases, and up to 25% have positive margins when treated with
standard cholecystectomy alone.
• Treatment For T1a lesions, laparoscopic cholecystectomy should suffice for therapy. The likelihood of
nodal disease in this setting is less than 3%. For T1b lesions, cholecystectomy is sufficient as long as
the margins are negative. With T1b lesions and perineural, lymphatic or vascular invasion, the
likelihood of nodal disease increases significantly, and therefore an extended cholecystectomy is
indicated.
• For T1a lesions, laparoscopic cholecystectomy should suffice for therapy. The likelihood of nodal
disease in this setting is less than 3%.

Page 24

353
• For T1b lesions, cholecystectomy is sufficient as long as the margins are negative. With T1b lesions
and perineural, lymphatic or vascular invasion, the likelihood of nodal disease increases significantly,
and therefore an extended cholecystectomy is indicated.
• Extended cholecystectomy involves en bloc removal of segment IVb and V of liver + gall bladder +
lymph node along with: Cystic duct CBD (peri-choledochal lymph node) Periportal lymph nodes Right
celiac lymph nodes Retro-pancreatic lymph nodes
• Cystic duct
• CBD (peri-choledochal lymph node)
• Periportal lymph nodes
• Right celiac lymph nodes
• Retro-pancreatic lymph nodes
• For T1a lesions, laparoscopic cholecystectomy should suffice for therapy. The likelihood of nodal
disease in this setting is less than 3%.
• For T1b lesions, cholecystectomy is sufficient as long as the margins are negative. With T1b lesions
and perineural, lymphatic or vascular invasion, the likelihood of nodal disease increases significantly,
and therefore an extended cholecystectomy is indicated.
• Cystic duct
• CBD (peri-choledochal lymph node)
• Periportal lymph nodes
• Right celiac lymph nodes
• Retro-pancreatic lymph nodes
Option A: Resection of segments IVb and V of liver
• With T1b lesions and perineural, lymphatic or vascular invasion, the likelihood of nodal disease
increases significantly, and therefore an extended cholecystectomy is indicated.
Option C: Palliative management with duodenal wall stent
• T4 tumours are typically considered unresectable, and therefore the goal of therapy is the palliation of
symptoms.
• Jaundice can be managed by endoscopic biliary stenting.
• Pain is treated with analgesics or percutaneous neurolysis of the celiac ganglion.
• Intestinal obstruction is usually due to gastric outlet obstruction from the local extension of the tumour
and is generally managed by an endoscopic duodenal wall stent.
Option D: Wedge excision of the liver with lymphadenectomy and port site excision
• For T3 lesions, there is a higher likelihood of intraperitoneal or distant spread. However, if no
peritoneal or nodal involvement is found, complete tumour excision with extended right hepatectomy
and possible caudate lobectomy with lymphadenectomy must be performed for adequate tumour
clearance.

Solution for Question 7:

Page 25

354
Option A: Gallbladder carcinoma
• Gallbladder carcinoma is an absolute contraindication to laparoscopic cholecystectomy.
• Laparotomy is the procedure of choice for gallbladder carcinoma.
Option B: Cholangitis
• Cholangitis is a relative contraindication to laparoscopic cholecystectomy.
Option C: Morbid obesity
• Morbid obesity is a relative contraindication of laparoscopic cholecystectomy.
Option D: Diffuse peritonitis
• Diffuse peritonitis is a relative contraindication to laparoscopic cholecystectomy.

Solution for Question 8:


Option A: Cholecystectomy
• Any blockage of the cystic duct is known as acute cholecystitis. When this occurs in the absence of
stones, the diagnosis is acute acalculous cholecystitis
• Risk factors include old age, burns and trauma, prolonged use of total parenteral nutrition, critical
illness, immunosuppression, and diabetes.
• The disease process is generally more fulminant than that of calculous cholecystitis and may progress
to gangrene and perforation of the gallbladder
• Ultrasound reveals a thickened gallbladder wall with pericholecystic fluid. HIDA scans make the
diagnosis but can have a false positive rate of up to 40%
• Treatment of choice is cholecystectomy.
Option B: Conservative management
• Patients of acalculous cholecystitis are at risk of developing gangrene and perforation of the
gallbladder.
• Therefore, cholecystectomy is the procedure of choice.
Option C: Percutaneous cholecystostomy
• Percutaneous cholecystostomy is done for unstable patients.
Option D: Endoscopic cholangiography + Drainage
• Acalculous cholecystitis is a fulminant condition; patients are at risk of developing gangrene and
perforation of the gallbladder.
• Therefore, the procedure of choice is cholecystectomy.

Solution for Question 9:


Option D: Ultrasound (USG)

Page 26

355
• Transabdominal ultrasonography is the initial imaging modality of choice as it is accurate, readily
available, inexpensive, and quick to perform.
• It has sensitivity and specificity of 84% and 99%, respectively
• For patients with biliary obstruction and obstructive jaundice, ultrasonography is the first investigation
because it can identify intrahepatic and extrahepatic biliary dilatation and often the level of obstruction.
• In addition, the cause of obstruction may also be determined.
• Disadvantages are it is operator dependent and may be compromised by excessive body fat and
intraluminal bowel gas.
Option A: CT Abdomen
• Unlike USG, CT is less affected by body habits and is not operator-dependent.
• It is the modality of choice in the staging of cancers of the liver, gallbladder, bile ducts, and pancreas.
• However, as only 75% of gallstones are identified by CT, it is not used as a screening modality for
uncomplicated gallstones.
Option B: Magnetic resonance cholangiopancreatography (MRCP)
• It is a non-invasive modality that provides excellent imaging of the gallbladder and biliary system.
• Images can be obtained of the biliary tree demonstrating obstruction, strictures, or other intraductal
abnormalities.
Option C: Endoscopic retrograde cholangiopancreatography (ERCP)
• This technique remains widely used as both a diagnostic and a therapeutic modality.

Solution for Question 10:


Option D: Age, sex, genetics, obesity, and diet play a role in the supersaturation of bile
• Supersaturation of secreted bile (lithogenic bile) - It is the primary event in the formation of cholesterol
stones.
• Cholesterol is highly nonpolar, and its solubility in water and bile depends on the relative
concentration of cholesterol, bile salts, and lecithin (the main phospholipid in bile).
• When cholesterol hypersecretion is present, supersaturation occurs, and cholesterol concentrations
exceed the ability of the bile salts and phospholipids to maintain solubility; the cholesterol precipitates
out of the solution into crystals.
• Age, sex, genetics, obesity, and diet play a role in the supersaturation of bile.
Option A: An increase in bile salt concentration leads to increased solubility of cholesterol in bile, leadin
g to stone formation
• A decrease in bile salt concentration leads to decreased solubility of cholesterol in bile, leading to
stone formation
• Therefore, this option is incorrect.
Option B: Progesterone-containing oral contraceptives can predispose to gallstone formation
• Progesterone-containing oral contraceptives do not predispose to gallstone formation.

Page 27

356
Option C: Primary event in the formation of cholesterol stones is increased stasis in the gallbladder, all
owing more time for solutes to precipitate in the gallbladder
• Gallbladder dysmotility - Increases stasis in the gallbladder, allowing more time for solutes to
precipitate in the gallbladder.
• However, the primary event in the formation of cholesterol stones is the supersaturation of bile with
cholesterol.

Solution for Question 11:


Option B: Produces the classic studded macroscopic appearance of a strawberry gallbladder
• The given image shows mucocele or hydrops of the gallbladder.
• Mucocele: An impacted stone without cholecystitis will result in what is called hydrops of the
gallbladder. It is a complication of gallstones. The gallbladder epithelium continues to secrete mucus
due to the obstructed cystic duct, and hence the gallbladder will become distended. The gallbladder
may be palpable but usually is non-tender. It produces the classic studded macroscopic appearance of
a strawberry gallbladder. Treatment is cholecystectomy
• An impacted stone without cholecystitis will result in what is called hydrops of the gallbladder. It is a
complication of gallstones.
• The gallbladder epithelium continues to secrete mucus due to the obstructed cystic duct, and hence
the gallbladder will become distended.
• The gallbladder may be palpable but usually is non-tender. It produces the classic studded
macroscopic appearance of a strawberry gallbladder.
• Treatment is cholecystectomy
• An impacted stone without cholecystitis will result in what is called hydrops of the gallbladder. It is a
complication of gallstones.
• The gallbladder epithelium continues to secrete mucus due to the obstructed cystic duct, and hence
the gallbladder will become distended.
• The gallbladder may be palpable but usually is non-tender. It produces the classic studded
macroscopic appearance of a strawberry gallbladder.
• Treatment is cholecystectomy
Option A: Gallstones are not related to it
• Mucocele of the gallbladder is a condition caused by the prolonged blockage of the cystic duct,
usually by an impacted gallstone.
Option C: Obstruction at the fundus of gallbladder
• Obstruction is at the level of the neck or cystic duct.
• Therefore, this option is incorrect.
Option D: Gallbladder is never palpable
• The gallbladder becomes distended due to continuous mucus secretion due to obstructed cystic duct.

Page 28

357
Solution for Question 12:
Option A: 10%
• Gallstones are classified by their cholesterol content as either cholesterol stones or pigment stones.
Pigment stones can be further classified as either black or brown.
• Most cholesterol stones (>90%) are radiolucent.
• Only 10% of the cholesterol stones are radiopaque.
Option B: 20%, Option C: 30%, Option D: 40%
• Only 10% of stones are radiopaque.
• Therefore, these options are incorrect.

Solution for Question 13:


Option B: Gallstones
• Cholelithiasis is the most important risk factor for gallbladder carcinoma; up to 85% of patients with
gallbladder carcinoma have gallstones.
• However, <3% of patients with gallstones have gallbladder cancer.
• Larger stones (>3cm) are associated with a 10-fold increased risk of carcinoma.
• The risk of developing cancer of the gallbladder is higher in patients with symptomatic than
asymptomatic gallstones.
Other options
Option A: Oral contraceptive pills
• Oral contraceptive pills are not associated with gallbladder carcinoma.
Option C: Acute cholecystitis
• Acute cholecystitis is not associated with gallbladder carcinoma.
• However, chronic cholecystitis is associated with gallbladder carcinoma.
Option D: Mucocele
• Mucocele is not associated with gallbladder carcinoma.
• Therefore, this option is not correct.

Solution for Question 14:


Option A: Erich Muhe
• Cholecystectomy is one of the most common surgical procedures in the United States.
• Open cholecystectomy, first performed by Carl Langenbuch in 1882, had been the primary treatment
of gallbladder disease through the early 1990s.
• In 1985, the first laparoscopic cholecystectomy was performed by Erich Muhe in Germany.

Page 29

358
Option B: Philippe Mouret
• Philippe Mouret, in Lyon, France, performed the first video-assisted laparoscopic cholecystectomy.
Option C: Kurt Semm
• Kurt Semm is the pioneer of laparoscopic appendicectomy.
Option D: Eddie Reddick
• Eddie Reddick is not the inventor of laparoscopic cholecystectomy

Solution for Question 15:


Option B: Sump syndrome
• Sump syndrome is seen after choledochoduodenostomy
• To remove an impacted stone in the setting of a dilated biliary tree, drainage of the biliary tree through
a separate choledochoenterostomy is done.
• The two options for drainage are a choledochoduodenostomy and Roux-en-Y choledochojejunostomy
• Anastomosis to the duodenum can be performed rapidly with a single anastomosis. This anatomic
arrangement continues to allow endoscopic access to the entire biliary tree. The downside of this
approach is that the bile duct distal to the anastomosis does not drain well and may collect debris that
obstructs the anastomoses or the pancreatic duct, a process known as sump syndrome.
• Anastomosis to the jejunum in a Roux-en-Y arrangement provides excellent drainage of the biliary
tree without the risk of sump syndrome but does not allow future endoscopic evaluation of the biliary
tree.
Option A: Short bowel syndrome
• Short bowel syndrome is a condition in which the body cannot absorb enough nutrients from the food
because of extensive surgical resection or congenital intestinal diseases.
• Conditions that require surgical removal of large portions of the small intestine include Crohn’s
disease, cancer, traumatic injuries, radiation enteritis and blood clots in the arteries that provide blood
to the intestines.
Option C: Mirizzi’s syndrome
• Obstruction of the bile ducts due to a severe pericholecystic inflammation secondary to the impaction
of a stone in the infundibulum of the gallbladder is known as Mirizzi’s syndrome.
Option D: Anastomosis leak
• An anastomosis leak occurs when a surgical anastomosis fails and the contents of a reconnected
body channel leak from the surgical connection.
• It is one of the most serious complications of bowel resection surgery.

Solution for Question 16:


Option A: Radiopaque stones

Page 30

359
• Medical Therapy for Gallstones: In carefully selected patients with a functioning gallbladder and with
radiolucent stones <10 mm in diameter, complete dissolution can be achieved in 50% of patients within
6 months to 2 years. The dose of ursodeoxycholic acid (UDCA) should be 10-15 mg/kg daily. Pigment
stones (radiopaque) are not responsive to UDCA therapy.
• In carefully selected patients with a functioning gallbladder and with radiolucent stones <10 mm in
diameter, complete dissolution can be achieved in 50% of patients within 6 months to 2 years.
• The dose of ursodeoxycholic acid (UDCA) should be 10-15 mg/kg daily.
• Pigment stones (radiopaque) are not responsive to UDCA therapy.
• In carefully selected patients with a functioning gallbladder and with radiolucent stones <10 mm in
diameter, complete dissolution can be achieved in 50% of patients within 6 months to 2 years.
• The dose of ursodeoxycholic acid (UDCA) should be 10-15 mg/kg daily.
• Pigment stones (radiopaque) are not responsive to UDCA therapy.
Option B: Female gender
• Female gender is not a contraindication for medical therapy of radiolucent stones.
Option C: Normal functioning gallbladder
• Normal functioning gallbladder and with radiolucent stones <10 mm in diameter, complete dissolution
can be achieved in 50% of patients within 6 months to 2 years.
Option D: Small stones
• Complete dissolution can be achieved in 50% of patients for radiolucent stones <10 mm diameter
within 6 months to 2 years.
• Stones >10mm in size rarely dissolve.

Solution for Question 17:


Option A: Observation
• Since only 1% of patients with asymptomatic stones develop complications, prophylactic
cholecystectomy is not warranted in asymptomatic patients.
• Most patients with gallstones are asymptomatic, often being identified at the time of abdominal
imaging for other reasons or during laparotomy.
• Symptomatic patients present with typical symptoms of postprandial right upper quadrant pain (biliary
colic) caused by a stone obstructing the cystic duct
• Medical treatment of gallstones is generally unsuccessful and is rarely used. Options include
dissolution with oral bile salt therapy, contact dissolution, which requires cannulation of the gallbladder,
and infusion of organic solvent, and extracorporeal shock wave lithotripsy.
• Hence treatment of silent gallstones is observation.
• Because patients with mild symptoms have a low rate of complications from gallstones
(1%–3%/year),observation and dietary and lifestyle changes are appropriate in this population. Patients
with more severe or recurrent symptoms have a higher rate of complications of the disease (7%/year),
so elective laparoscopic cholecystectomy is warranted.

Page 31

360
Option B: Ursodeoxycholic acid
• Ursodeoxycholic acid can be used as a medical therapy in carefully selected patients with functional
gallbladder radiolucent stones <10 mm in diameter.
• Stones are dissolved in 50% of patients within 6 months to 2 years.
• However, stones >10mm in size rarely dissolve.
• This patient is asymptomatic.
• Therefore, she is not a candidate for any surgical or medical management.
Option C: Cholecystectomy
• Cholecystectomy is not indicated in asymptomatic patients.
• Therefore, this option is incorrect.
Option D: Lithotripsy
• Lithotripsy of bile duct stones is highly effective and can be considered in patients in whom primary
endoscopic or surgical stone removal fails.
• This patient is asymptomatic, lithotripsy is not done.

Solution for Question 18:


Option C: Somatostatinoma
• Gallstones are associated with somatostatinoma.
• Somatostatinoma: Somatostatinoma syndrome is a triad of diabetes mellitus, steatorrhea, and
gallstones, but it also includes weight loss and hypochlorhydria. Because somatostatin inhibits
pancreatic and biliary secretions, patients with a somatostatinoma present with gallstones due to bile
stasis, diabetes due to inhibition of insulin secretion, and steatorrhea due to inhibition of pancreatic
exocrine secretion and bile secretion Most somatostatinomas originate in the proximal pancreas or the
pancreatoduodenal groove, with the ampulla and periampullary area as the most common site (60%)
The most common presentations are abdominal pain, jaundice, and cholelithiasis. Of tumours, >70%
are malignant, with metastasis to the liver or lymph nodes commonly noted at the time of diagnosis.
They are rarely associated with MEN1 syndrome but are associated with Von Recklinghausen disease
and pheochromocytoma. Diagnosis is confirmed by elevated serum somatostatin levels. Treatment
involves complete excision of the tumour and cholecystectomy if there is no metastatic disease
• Somatostatinoma syndrome is a triad of diabetes mellitus, steatorrhea, and gallstones, but it also
includes weight loss and hypochlorhydria.
• Because somatostatin inhibits pancreatic and biliary secretions, patients with a somatostatinoma
present with gallstones due to bile stasis, diabetes due to inhibition of insulin secretion, and steatorrhea
due to inhibition of pancreatic exocrine secretion and bile secretion
• Most somatostatinomas originate in the proximal pancreas or the pancreatoduodenal groove, with the
ampulla and periampullary area as the most common site (60%)
• The most common presentations are abdominal pain, jaundice, and cholelithiasis. Of tumours, >70%
are malignant, with metastasis to the liver or lymph nodes commonly noted at the time of diagnosis.
They are rarely associated with MEN1 syndrome but are associated with Von Recklinghausen disease

Page 32

361
and pheochromocytoma.
• Diagnosis is confirmed by elevated serum somatostatin levels.
• Treatment involves complete excision of the tumour and cholecystectomy if there is no metastatic
disease
• Somatostatinoma syndrome is a triad of diabetes mellitus, steatorrhea, and gallstones, but it also
includes weight loss and hypochlorhydria.
• Because somatostatin inhibits pancreatic and biliary secretions, patients with a somatostatinoma
present with gallstones due to bile stasis, diabetes due to inhibition of insulin secretion, and steatorrhea
due to inhibition of pancreatic exocrine secretion and bile secretion
• Most somatostatinomas originate in the proximal pancreas or the pancreatoduodenal groove, with the
ampulla and periampullary area as the most common site (60%)
• The most common presentations are abdominal pain, jaundice, and cholelithiasis. Of tumours, >70%
are malignant, with metastasis to the liver or lymph nodes commonly noted at the time of diagnosis.
They are rarely associated with MEN1 syndrome but are associated with Von Recklinghausen disease
and pheochromocytoma.
• Diagnosis is confirmed by elevated serum somatostatin levels.
• Treatment involves complete excision of the tumour and cholecystectomy if there is no metastatic
disease
Other options
Option A: Insulinoma
• Insulinomas are the most common functional pancreatic endocrine neoplasms and present with a
typical clinical syndrome known as Whipple’s triad.
• Whipple's triad: fasting hypoglycemia (<50 mg/dL or <2.7 mmol/L) symptoms of hypoglycemia.
immediate relief of symptoms after the administration of intravenous glucose.
• The majority of insulinomas are benign and solitary, and 10% are malignant. They are typically cured
by simple enucleation.
• Approximately 90% of insulinomas are sporadic, and 10% are associated with MEN1 syndrome.
Option B: VIPoma
• Vasoactive Intestinal Peptide Secreting Tumour (VIPoma) syndrome, also called WDHA (Watery
diarrhea, hypokalemia, achlorhydria) syndrome. This syndrome was described by Verner and Morrison.
They are most commonly located in the distal pancreas.
Option D: Glucagonoma
• Diabetes in association with dermatitis should raise the suspicion of a glucagonoma. The classic
necrolytic migratory erythema (due to low levels of amino acids) manifests as cyclic migrations of
lesions with spreading margins and healing centres typically on the lower abdomen, perineum, perioral
area and feet. Patients also complain of an enlarged sensitive tongue.
• The diagnosis is confirmed by measuring serum glucagon levels. They are more often in the body and
tail of the pancreas.

Solution for Question 19:

Page 33

362
Option D: Mucocele due to stone in the cystic duct is an exception to this law

• Courvoisier’s law states that: In a jaundiced patient, if the gallbladder is palpable, then it is not due to
stones. In obstruction of the common bile duct due to a stone, distension of the gall bladder seldom
occurs; the organ usually is shriveled owing to the inflammatory process.
• In a jaundiced patient, if the gallbladder is palpable, then it is not due to stones.
• In obstruction of the common bile duct due to a stone, distension of the gall bladder seldom occurs;
the organ usually is shriveled owing to the inflammatory process.
• In a jaundiced patient, if the gallbladder is palpable, then it is not due to stones.
• In obstruction of the common bile duct due to a stone, distension of the gall bladder seldom occurs;
the organ usually is shriveled owing to the inflammatory process.
• Mucocele or hydrops of the gallbladder An impacted stone without cholecystitis will result in what is
called hydrops of the gallbladder. It is a complication of gallstones. The gallbladder epithelium
continues to secrete mucus due to the obstructed cystic duct, and hence the gallbladder will become
distended. The gallbladder may be palpable but usually is non-tender. It produces the classic studded
macroscopic appearance of a strawberry gallbladder.
• An impacted stone without cholecystitis will result in what is called hydrops of the gallbladder. It is a
complication of gallstones.
• The gallbladder epithelium continues to secrete mucus due to the obstructed cystic duct, and hence
the gallbladder will become distended.
• The gallbladder may be palpable but usually is non-tender. It produces the classic studded
macroscopic appearance of a strawberry gallbladder.
• Mucocele is an exception to Courvoisier’s law.
• An impacted stone without cholecystitis will result in what is called hydrops of the gallbladder. It is a
complication of gallstones.
• The gallbladder epithelium continues to secrete mucus due to the obstructed cystic duct, and hence
the gallbladder will become distended.
• The gallbladder may be palpable but usually is non-tender. It produces the classic studded
macroscopic appearance of a strawberry gallbladder.
Option A: In obstruction of the common bile duct due to a
stone, the organ usually is distended owing to the obstructing of biliary flow
• option is incorrect
Option B: In a jaundiced patient, if the gallbladder is palpable, then it is due to stones
• Option is incorrect.
Option C: In obstructions caused due to cancer of the ampulla, pancreas, or bile duct, the gall bladder
will not get distended
• If there is no disease in the gall bladder and the obstruction is due to cancer of the ampulla, pancreas
or bile duct, then the gall bladder will be distended.

Solution for Question 20:

Page 34

363
Option A: Segment IV b and V of the liver, gallbladder, peri-choledochal lymph node
• For T1a lesions, laparoscopic cholecystectomy should suffice for therapy. The likelihood of nodal
disease in this setting is less than 3%
• For T1b lesions, cholecystectomy is sufficient as long as the margins are negative. With T1b lesions
and perineural, lymphatic or vascular invasion, the likelihood of nodal disease increases significantly,
and therefore an extended cholecystectomy is indicated.
• Extended cholecystectomy involves en bloc removal of segment IV b and V of liver + gall bladder +
lymph node along: Cystic duct Common bile duct (peri-choledochal lymph node) Periportal lymph
nodes Retro-pancreatic lymph nodes
• Cystic duct
• Common bile duct (peri-choledochal lymph node)
• Periportal lymph nodes
• Retro-pancreatic lymph nodes
• In T2 lesions, radical(extended) cholecystectomy is indicated because more than 40% of these
patients have lymph node metastases, and up to 25% have positive margins when treated with
standard cholecystectomy alone.
• For T3 lesions, there is a higher likelihood of intraperitoneal or distant spread. However, if no
peritoneal or nodal involvement is found, complete tumour excision with extended right hepatectomy
and possible caudate lobectomy with lymphadenectomy must be performed for adequate tumour
clearance.
• T4 tumours are typically considered unresectable, and therefore the goal of therapy is the palliation of
symptoms.
• Cystic duct
• Common bile duct (peri-choledochal lymph node)
• Periportal lymph nodes
• Retro-pancreatic lymph nodes
Option B: Gall bladder, right hepatic bile duct, peri-choledochal lymph node
• In extended cholecystectomy, the right hepatic bile duct is not removed.
Option C: Peri-choledochal lymph node, segment IV b and V of the liver, right hepatic bile duct
• In extended cholecystectomy, the right hepatic bile duct is not removed.
Option D: Right hepatic bile duct, gall bladder, segment IV b and V of liver
• In extended cholecystectomy, the right hepatic bile duct is not removed.

Solution for Question 21:


Option A: Most common type of gallstone is mixed
• Most common gallstone worldwide is mixed stone > cholesterol stones.
• Gallstones are classified by their cholesterol content as either cholesterol stones or pigment stones.
Pigment stones can be further classified as either black or brown.

Page 35

364
• Most cholesterol stones (>90%) are radiolucent.
• Only 10% of the cholesterol stones are radiopaque.
• In the USA and Europe, 80% are cholesterol or mixed stones, whereas, in Asia, 80% are pigment
stones.
• Pigmented stones are radiopaque and soft
Option B: Black stones are hard and radiolucent
• Black stones are hard and radiopaque.
Option C: Brownstones are most common in the gallbladder
• Brownstones most commonly occur in the biliary tree and are rare in the gallbladder.
Option D: Most common type of gallstone in the Asian population is cholesterol stones
• The most common gallstones in Asia / India are pigmented stones.

Solution for Question 22:


Option A: Emergent cholecystectomy under antibiotic coverage

• In the X-Ray, the gallbladder shows the presence of air which is suggestive of emphysematous
cholecystitis.
• The most common organism responsible for emphysematous cholecystitis is clostridium perfringens
(anaerobe).
• Emphysematous cholecystitis is a rare entity that results from the presence of gas-forming bacteria in
the bile. It may be seen in association with acute or gangrenous cholecystitis, and it is more common in
men and patients with diabetes.
• The diagnosis can occasionally be made by simple abdominal radiographs, but more often, it is
diagnosed on ultrasound or CT scan (shows gas in the gallbladder lumen and in the wall of the
gallbladder.

Page 36

365
• The patient should receive IV antibiotics followed by emergent cholecystectomy.
Option B: Conservative management followed by interval cholecystectomy
• If a patient is discharged from the hospital without surgery after an episode of acute cholecystitis or
pancreatitis, an interval cholecystectomy is indicated.
• Although, there are increased risks associated with delayed Surgery.
Option C: Conservative treatment and treat primary causes like diabetes mellitus
• Emphysematous cholecystitis is a surgical emergency; therefore, emergency cholecystectomy is the
best next step.
Option D: Urgent endoscopic retrograde cholangiopancreatography
• Indications for urgent (< 24 hours) ERCP include severe acute cholangitis, acute biliary pancreatitis
with cholangitis, biliary or pancreatic leaks in the absence of percutaneous drainage, and severe acute
cholecystitis in patients who are unfit for Surgery and do not respond to conservative management.
• Emphysematous cholecystitis is a surgical emergency; therefore, emergent cholecystectomy is the
best next step.

Solution for Question 23:


Option D: Acalculous cholecystitis
• The procedure given in the image above is percutaneous cholecystostomy.
• Percutaneous cholecystostomy is most commonly indicated for the treatment of acute acalculous
cholecystitis in severely ill hospitalized patients but may also be used to treat calculous cholecystitis in
patients who are too sick or in whom comorbidities preclude definitive cholecystectomy.
• It may also be used to provide access to the gallbladder or biliary tree for stone removal or to provide
biliary drainage in patients with common bile duct obstruction distal to the insertion of the cystic duct.
• For acalculous cholecystitis, the treatment of choice is cholecystectomy. In unstable cases,
percutaneous cholecystostomy is done.
Option A: Mucocele
• Mucocele or hydrops of the gallbladder An impacted stone without cholecystitis will result in what is
called hydrops of the gallbladder. It is a complication of gallstones.
• An impacted stone without cholecystitis will result in what is called hydrops of the gallbladder. It is a
complication of gallstones.
• An impacted stone without cholecystitis will result in what is called hydrops of the gallbladder. It is a
complication of gallstones.
Option B: Mirizzi’s syndrome
• Obstruction of the bile ducts due to a severe pericholecystic inflammation secondary to the impaction
of a stone in the infundibulum of the gallbladder is known as Mirizzi’s syndrome.
• The prerequisites for this syndrome, characterized by gallbladder disease causing obstructive
jaundice, include: A cystic duct that courses parallel to the common hepatic duct An impacted stone in
the gallbladder neck or cystic duct Obstruction of the common hepatic duct is caused by the stone or

Page 37

366
inflammatory response. The resultant inflammation can cause a cholecystocholedochal fistula. It
usually presents as jaundice or recurrent cholangitis. The presence of Mirizzi syndrome obliterates the
triangle of Calot.
• A cystic duct that courses parallel to the common hepatic duct
• An impacted stone in the gallbladder neck or cystic duct
• Obstruction of the common hepatic duct is caused by the stone or inflammatory response. The
resultant inflammation can cause a cholecystocholedochal fistula.
• It usually presents as jaundice or recurrent cholangitis. The presence of Mirizzi syndrome obliterates
the triangle of Calot.
• A cystic duct that courses parallel to the common hepatic duct
• An impacted stone in the gallbladder neck or cystic duct
• Obstruction of the common hepatic duct is caused by the stone or inflammatory response. The
resultant inflammation can cause a cholecystocholedochal fistula.
• It usually presents as jaundice or recurrent cholangitis. The presence of Mirizzi syndrome obliterates
the triangle of Calot.
Option C: Xanthogranulomatous cholecystitis
• Xanthogranulomatous cholecystitis (XGC) is an uncommon inflammatory disease of the gallbladder
that may be difficult to differentiate from malignancy, both on imaging and pathologically. It is
characterized by the presence of multiple intramural nodules.

Solution for Question 24:


Option C: Associated with cholesterol polyp
• The image given above shows the inner wall of the gallbladder studded with yellow flecks, which is
suggestive of the strawberry gallbladder or Cholesterolosis

Page 38

367
• Cholesterolosis is due to the deposition of cholesterol ester and triglycerides in macrophages in the
gall bladder lamina propria, either locally or as polyps. It is an acquired condition. Risk factor includes
hypercholesterolemia. It is associated with the presence of cholesterol polyps and cholesterol
gallstones.
• It is an acquired condition.
• Risk factor includes hypercholesterolemia. It is associated with the presence of cholesterol polyps
and cholesterol gallstones.
• It is an acquired condition.
• Risk factor includes hypercholesterolemia. It is associated with the presence of cholesterol polyps
and cholesterol gallstones.
Option A: It is an inherited condition
• It is an acquired condition.
Option B: These are only pigmented gallstones
• It is associated with the presence of cholesterol polyps and cholesterol gallstones.
Option D: Cholesterol deposits in the submucosa of the gallbladder
• Cholesterolosis due to the deposition of cholesterol ester and triglycerides in macrophages in the gall
bladder lamina propria.

Solution for Question 25:


Option D: Polyps in the gallbladder
• The image given above is suggestive of a polypoid lesion of the gallbladder
• Gallbladder polyps may be found in approximately 5% of patients who undergo ultrasonography and
consists of pseudo tumours and adenomas.
• They can be divided into pseudopolyps and true polyps.
• Pseudopolyps are further divided into cholesterol polyps, focal adenomyomatosis, hyperplastic
polyps, and inflammatory polyps.
• True polyps are benign growths in the wall of the gallbladder and consist of only 5% of all polypoid
disorders of the gallbladder. True polyps may be difficult to differentiate from adenocarcinoma
preoperatively due to imaging limitations in detecting mural invasion.
• Cholesterol polyps appear as pedunculated echogenic lesions of the gallbladder, are usually smaller
than 1 cm, and are frequently multiple.
• Adenomyomatosis is a sessile lesion commonly in the fundus with characteristic microcysts within the
lesion and is frequently larger than 1 cm.
Option A: Diverticulosis of gallbladder
• Gallbladder diverticulum is an unusual congenital disease that has the appearance of a hernia-like
protrusion of the normal gallbladder wall.
• This disorder may not be diagnosed until surgically resected because it has no clinical significance
unless there are associated diseases.
Option B: Porcelain gallbladder

Page 39

368
• It is characterized by dense circumferential calcification of the gallbladder wall.
• 90% are associated with gallstones
• It is a rare premalignant condition that is associated with an approximately 10% risk of gallbladder
carcinoma.
• Diagnosed incidentally on X-Ray or CT but is more commonly seen on CT. In ultrasound, it will appear
as a curvilinear echogenic focus along the entire gallbladder wall with posterior shadowing.
• Cholecystectomy is the treatment of choice in patients with porcelain gallbladder, even if
asymptomatic (due to the high incidence of development of carcinoma gallbladder)
Option C: Strawberry gallbladder
• Deposition of cholesterol ester and triglycerides in macrophages within the gall bladder lamina
propria, either locally or as polyps. It gives the classic studded macroscopic appearance of a strawberry
gallbladder.

Solution for Question 26:


Answer
Option C: Adenomyomatosis

• The ultrasound image above shows a comet tail artifact in the gallbladder, which is suggestive of
adenomyomatosis.
• Adenomyomatosis is a benign hyperplastic cholecystitis with no malignant potential that results from
hyperplasia of both the mucosa and muscularis propria of the gallbladder wall.
• Intramural diverticula are called Rokitansky–Aschoff sinuses, which trap bile that accumulates
cholesterol crystals appearing as cystic spaces in a thickened gallbladder wall with a characteristic
comet tail artifact.
• The gallbladder wall thickening in adenomyomatosis may be focal or diffuse. Focal wall thickening is
most common in the fundus.

Page 40

369
• It is seen as a sessile lesion commonly in the fundus with characteristic microcysts within the lesion
and is frequently larger than 1 cm.
Other Options
Option A: Gallbladder stone
• Ultrasound is considered the gold standard for detecting gallstones.
• Greyscale ultrasound Highly reflective echogenic focus within the gallbladder lumen, normally with
prominent posterior acoustic shadowing regardless of pathological type (acoustic shadowing is
independent of the composition and calcium content) Gravity-dependent movement is often seen with a
change of patient position (the rolling stone sign)
• Highly reflective echogenic focus within the gallbladder lumen, normally with prominent posterior
acoustic shadowing regardless of pathological type (acoustic shadowing is independent of the
composition and calcium content)
• Gravity-dependent movement is often seen with a change of patient position (the rolling stone sign)
• Highly reflective echogenic focus within the gallbladder lumen, normally with prominent posterior
acoustic shadowing regardless of pathological type (acoustic shadowing is independent of the
composition and calcium content)
• Gravity-dependent movement is often seen with a change of patient position (the rolling stone sign)

Option B: Gallbladder polyp


• Gallbladder polyps may be found in approximately 5% of patients who undergo ultrasonography and
consists of pseudo tumours and adenomas.
• Pseudotumors are further divided into adenomyomatosis and cholesterol polyps and have no
malignant potential.
• True adenomatous polyps occur in 0.3-0.5% of the population. The risk of malignant transformation
increases with the increasing size of the polyp.
• Cholesterol polyps appear as pedunculated echogenic lesions of the gallbladder, are usually smaller
than 1cm, and are frequently multiple.

Page 41

370
Option D: Xanthogranulomatous cholecystitis
• Xanthogranulomatous cholecystitis (XGC) is an uncommon inflammatory disease of the gallbladder
that may be difficult to differentiate from malignancy, both on imaging and pathologically. It is
characterized by the presence of multiple intramural nodules.

Solution for Question 27:


Option A: T3N1M0
• T3 indicates that the tumour has a serosal perforation or direct hepatic invasion or involvement of a
single extrahepatic organ
• N1 means that the tumour is metastasized to cystic duct, common bile duct, hepatic artery and/or
portal vein; : metastasis to 1-3 lymph nodes
• M0 means no distant metastasis:
Option B: T2aN2M0
• T2a means invasion of perimuscular connective tissue towards the peritoneal side without extension
to the serosa
• N2 is the metastasis to periaortic, pericaval, superior mesenteric artery, and/or celiac artery lymph
nodes; metastases to 4-6 lymph nodes
• M0 shows no distant metastasis
Option C: T3N2M1
• T3 indicates that the tumour has serosal perforation or direct hepatic invasion or involvement of a
single extrahepatic organ
• N2 means metastasis to periaortic, pericaval, superior mesenteric artery, and/or celiac artery lymph
• M1 that tumour has distant metastasis
Option D: T4N1M0

Page 42

371
• T4 means involvement of portal vein or hepatic artery or two or more extrahepatic organ
• N1 indicates metastasis to nodes along the cystic duct, common bile duct, hepatic artery and/or portal
vein; metastases to 1-3 lymph nodes
• M0 has no distant metastasis

Solution for Question 28:


Option B: Phrygian cap
• The ultrasound image shows the infoldings of the septum between the fundus and body of the
gallbladder, which looks like the Phrygian cap.

• The Phrygian cap is the most common congenital anomaly of the gallbladder.
• It is present in 5% of cases and may be mistaken for pathological deformity of the organ.
Option A: Moynihan’s hump
• The caterpillar or Moynihan's hump configuration is characterized by a tortuous right hepatic artery
(RHA) running proximal and/or parallel to the cystic duct and predisposes to a small and/or short cystic
artery (CA).
Option C: Duplication
• Gallbladder duplication is a rare congenital malformation that occurs in about one in 4000 births.
• Congenital anomalies of the gallbladder and anatomical variations of their positions are associated
with an increased risk of complications after laparoscopic cholecystectomy.
Option D: None of the above
• The picture shown above is the Phrygian cap.
• Therefore, this option is incorrect.

Page 43

372
Solution for Question 29:
Option A: Concentration of bile occurs by active absorption of water, sodium chloride, and bicarbonate
via the mucous membrane of the gallbladder
• Bile is produced by the liver and stored in the gallbladder.
• In the gallbladder, the concentration of bile occurs by active absorption of water, sodium chloride, and
bicarbonate via the mucous membrane of the gallbladder.
Option B: Gallbladder is 5-7cm long, with a normal capacity of about 15-20 ml
• It is a pear-shaped structure, 7.5 -12 cm long, with a normal capacity of about 25-30 ml
Option C: Gallbladder lacks a serosa and submucosa
• Gallbladder lacks a muscularis mucosa and submucosa

Option D: Bile is concentrated 10-15 times in the gallbladder


• The hepatic bile which enters the gallbladder becomes concentrated 5-10 times with a corresponding
increase in the proportion of bile salts, bile pigments, cholesterol, and calcium.
Option E: The sphincter of Lutkens is present in the submucosa of the cystic duct
• The mucosa of the cystic duct is arranged in spiral folds known as the valves of Heister, and the wall
is surrounded by a sphincteric structure called the sphincter of Lutkens.

Solution for Question 30:


Option D: Most commonly seen in elderly

Page 44

373
• Obstruction of the intestinal lumen from a gall stone carries the misnomer gall stone ileus, which is, in
fact, a mechanical blockage. It occurs when a large gallstone erodes through the wall of the gallbladder
directly into the intestine via a choledochoenteric fistula, which results in the air entering the biliary tree
(pneumobilia).
• Most of these fistulas occur in older patients and may be caused by inflammation in the gallbladder or
simply pressure necrosis.
• The patient presents with clinical evidence of mechanical small bowel obstruction.
• Plain radiographs demonstrate air-fluid levels consistent with small bowel obstruction.
• Proximal stones can become impacted in the pylorus or proximal duodenum, causing gastric outlet
obstruction (Bouveret syndrome).
• Pneumobilia, which may be identified only by CT scan, is a ubiquitous finding because the fistula that
permits a stone to pass into the duodenum allows air into the gallbladder and biliary tree.
• Management of gallstone ileus focuses on relieving the intestinal obstruction and removing the stone.
In cases of very proximal obstructions in the stomach or duodenum, endoscopic retrieval can be
effective. For more distal stones, surgical enterolithotomy can be accomplished either laparoscopically
or open.
Option A: The most common site of the fistula is cholecystocolic
• Most common is choledochoduodenal fistula.
Option B: Stone moves from the gallbladder and drops into GIT through the fistula causing obstruction i
n the jejunal part of the small intestine
• It occurs when a large gallstone erodes through the wall of the gallbladder directly into the intestine
via a choledochoenteric fistula, which results in the air entering the biliary tree
• The blockage can occur anywhere in the small intestine but occurs most commonly in the distal ileum,
about 60 cm proximal to the ileocecal valve.
Option C: Rigler’s triad: Large bowel obstruction+ Pneumobilia + Ectopic gallstone
• Rigler’s triad for gallstone ileus: Small bowel obstruction Pneumobilia Ectopic gallstone
• Small bowel obstruction

Page 45

374
• Pneumobilia
• Ectopic gallstone
• Therefore, this option is incorrect.
• Small bowel obstruction
• Pneumobilia
• Ectopic gallstone

Solution for Question 31:


Option C: Right and left hepatic lobes
• Cholecystocaval line/Cantlie’s line: It is an imaginary line that passes through the groove for the
inferior vena cava (IVC) to the gallbladder fossa and separates the right and left hepatic lobes
functionally.
Option A: Right anterior and right posterior sectors
• The cholecystocaval line is an imaginary line that passes through the groove for the inferior vena cava
(IVC) to the gallbladder fossa and separates the right and left hepatic lobe functionally.
• Therefore, this option is incorrect.
Option B: Gallbladder from the portal vein
• The cholecystocaval line does not separate the gallbladder from the portal vein.
Option D: Left medial and left lateral sectors
• The left hepatic vein, located in the left intersegmental fissure, divides the left lobe into the left medial
and left lateral sectors.

Solution for Question 32:


Option C: Endoscopic retrograde cholangiopancreatography (ERCP)

Page 46

375
• The image given above is endoscopic retrograde cholangiopancreatography (ERCP)
• ERCP is used both as a diagnostic and therapeutic procedure.
• Using a side viewing scope, the image is identified and cannulated. Injection of water-soluble contrast
directly into the bile duct provides excellent images of the ductal anatomy.
• It helps in determining the cause and level of obstruction.
• During ERCP, bile aspirates can be sent for cytological and microbiological examination, and
endoluminal brushings can be taken from strictures for cytological examination.
• The advantages of ERCP include direct visualization of the ampullary region and direct access to the
distal common bile duct for cholangiography or choledochoscopy. The test is rarely needed for
uncomplicated gallstone disease. However, for cases of choledocholithiasis, obstructive jaundice,
biliary strictures, or cholangitis, ERCP has the advantage of being both diagnostic and therapeutic.
• Therapeutic interventions such as stent placement or stone removal to relieve the obstruction can be
performed.
• complications of diagnostic ERCP include pancreatitis, as well as rare occurrences of bleeding,
perforation, or infection (cholangitis).
Option A: Percutaneous transhepatic cholangiogram
• The best method to visualize the proximal duct is PTC (Percutaneous Transhepatic Cholangiography)
• It is an invasive procedure used to evaluate the biliary tree. A needle is passed directly into the liver to
access one of the biliary radicals, and the tract is then used for the insertion of transhepatic catheters.
Useful for patients with the intrahepatic biliary disease (proximal bile ducts) or in whom ERCP is not
technically feasible
• PTC can decompress proximal biliary obstruction and stent obstructions non-operatively and provide
anatomic information for biliary reconstruction
Option B: T - tube cholangiogram
• T-tube cholangiogram is a special X-ray investigation carried out using X-ray dye to visualize the bile
ducts after cholecystectomy or after common bile duct exploration with supra-duodenal
choledochotomy.

Page 47

376
• It provides external bile drainage into a controlled route while the healing process of cholecystectomy
matures and the original pathology resolves.
• Common bile duct T-tubes should remain in for 10 days.
• However, once the T-tube cholangiogram has shown that there is a free flow of bile into the
duodenum and that there are no retained stones, some surgeons like to clamp the T-tube prior to
removal.
• The 10-day period is required to minimize the risk of biliary peritonitis after removal.
• T-tubes are traditionally and intentionally made of latex to stimulate fibrosis, which results in the
formation of a tract to allow the drainage of bile if required.
Option D: Hepatobiliary iminodiacetic acid (HIDA) scan
• HIDA scan can be used to evaluate the physiologic secretion of bile. Technetium 99m labeled
derivatives of iminodiacetic acid are injected intravenously, taken up by the kupffer cells in the liver, and
excreted in the bile.
• Uptake by the liver is usually detected within 10 minutes, and the gallbladder, bile ducts, and
duodenum are typically visualized within 60 minutes in fasting subjects.

Solution for Question 33:


Option C: Acute cholecystitis
• The given scenario with HIDA image is diagnostic of Acute cholecystitis.
• Acute cholecystitischaracterized by a triad of: Right upper quadrant pain Fever Leukocytosis
• Right upper quadrant pain
• Fever
• Leukocytosis

Page 48

377
• When the right 9th costal cartilage is pressed there is a catch in inspiration and this is known as
Murphy's sign / Naunyn sign
• Hyperesthesia at the inferior angle of the right scapula is Boas sign
• Diagnosis: IOC is ultrasound (>4mm thickness of GB wall) Gold standard investigation: HIDA scan
• IOC is ultrasound (>4mm thickness of GB wall)
• Gold standard investigation: HIDA scan
• Treatment: Cholecystectomy
• Right upper quadrant pain
• Fever
• Leukocytosis
• IOC is ultrasound (>4mm thickness of GB wall)
• Gold standard investigation: HIDA scan

Option A: Chronic cholecystitis


• HIDA scan can be of particular benefit in cases where the diagnosis is uncertain and for differentiation
from acute cholecystitis.
• Delayed visualization of the gallbladder between 1-4 hours is a reliable sign of chronic cholecystitis.
• Chronic cholecystitis may be diagnosed by calculating the percentage of isotope excreted (ejection
fraction) from the gallbladder following cholecystokinin or after a fatty meal.
Option B: Mucocele
• Mucocele or hydrops of the gallbladder: An impacted stone without cholecystitis will result in what is
called hydrops of the gallbladder. It is a complication of gallstones. The gallbladder epithelium
continues to secrete mucus due to the obstructed cystic duct, and hence the gallbladder will become
distended.
• An impacted stone without cholecystitis will result in what is called hydrops of the gallbladder. It is a
complication of gallstones.

Page 49

378
• The gallbladder epithelium continues to secrete mucus due to the obstructed cystic duct, and hence
the gallbladder will become distended.
• An impacted stone without cholecystitis will result in what is called hydrops of the gallbladder. It is a
complication of gallstones.
• The gallbladder epithelium continues to secrete mucus due to the obstructed cystic duct, and hence
the gallbladder will become distended.

Option D: Normal scan


• In a normal HIDA scan, the radiotracer moved freely from the liver into the gallbladder and small
intestine.

Solution for Question 34:


Option B: Cholangiocarcinoma
• Clonorchis Sinensis is a liver fluke in snails and fish that act as intermediate hosts. Ingestion of
infected fish and snails, when eaten raw or improperly cooked, causes the infection in humans and
other fish-eating mammals, who are the definitive hosts.
• It has a high incidence in the tropical regions of Southeast Asia, particularly amongst those living in
the major seaports and near river estuaries.
• In humans, the parasite matures into the adult worm in the intrahepatic biliary radicals, where they
may reside for many years (dormant).
• The intrahepatic biliary radicals are dilated with epithelial hyperplasia and periductal fibrosis. These
changes may lead to dysplasia, causing cholangiocarcinoma - the most serious and dreaded
complication of this parasitic infestation.
• It also causes an inflammatory reaction in the bile ducts leading to oriental cholangiohepatitis and
biliary obstruction.

Page 50

379
Option A: Urinary bladder carcinoma
• Urinary bladder carcinoma is a complication caused by an infestation of Schistosoma haematobium.
• Without treatment, the infestation can persist for years, and chronic infection can also lead to an
increased risk of liver fibrosis or bladder cancer.
Option C: Hepatocellular carcinoma
• Risk factors for hepatocellular carcinoma include chronic hepatitis B virus, chronic hepatitis C virus
infection and Schistosoma mansoni.
Option D: Neurocysticercosis
• Neurocysticercosis is a preventable parasite infection caused by larval cysts (enclosed sacs
containing the immature stage of a parasite) of tapeworm (Taenia solium).

Solution for Question 35:


Option B: Grade II
• According to the Tokyo Consensus Guidelines, grade II is moderate cholecystitis associated with any
one of the following: Elevated WBC count (>18000 /mm3 ) Palpable tender mass in the right upper
abdominal quadrant Duration of complaints >72 hours Marked local inflammation (gangrenous
cholecystitis, pericholecystic abscess, hepatic abscess, biliary peritonitis, emphysematous
cholecystitis)
• Elevated WBC count (>18000 /mm3 )
• Palpable tender mass in the right upper abdominal quadrant
• Duration of complaints >72 hours
• Marked local inflammation (gangrenous cholecystitis, pericholecystic abscess, hepatic abscess,
biliary peritonitis, emphysematous cholecystitis)
• Elevated WBC count (>18000 /mm3 )
• Palpable tender mass in the right upper abdominal quadrant
• Duration of complaints >72 hours
• Marked local inflammation (gangrenous cholecystitis, pericholecystic abscess, hepatic abscess,
biliary peritonitis, emphysematous cholecystitis)
Option A: Grade I
• According to the Tokyo Consensus Guidelines, grade I is mild cholecystitis which comprises mild
inflammatory changes in the gallbladder in a healthy person with no organ dysfunction.
Option C: Grade III
• According to the Tokyo Consensus Guidelines, grade III is severe cholecystitis which comprises any
one of the following: Cardiovascular dysfunction- Hypertension requiring treatment with
dopamine/norepinephrine Neurological dysfunction- Decreased level of consciousness. Respiratory
dysfunction- PaO2 /FiO2 ratio <300 Renal dysfunction- Oliguria; creatinine >2.0mg/dL Hepatic
dysfunction- PT-INR >1.5 Hematological dysfunction- Platelet count <100,000/mm3
• Cardiovascular dysfunction- Hypertension requiring treatment with dopamine/norepinephrine
• Neurological dysfunction- Decreased level of consciousness.

Page 51

380
• Respiratory dysfunction- PaO2 /FiO2 ratio <300
• Renal dysfunction- Oliguria; creatinine >2.0mg/dL
• Hepatic dysfunction- PT-INR >1.5
• Hematological dysfunction- Platelet count <100,000/mm3
• Cardiovascular dysfunction- Hypertension requiring treatment with dopamine/norepinephrine
• Neurological dysfunction- Decreased level of consciousness.
• Respiratory dysfunction- PaO2 /FiO2 ratio <300
• Renal dysfunction- Oliguria; creatinine >2.0mg/dL
• Hepatic dysfunction- PT-INR >1.5
• Hematological dysfunction- Platelet count <100,000/mm3
Option D: Grade IV
• There are only three grades for Tokyo Consensus Guidelines.
• Therefore, this option is incorrect.

Page 52

381
Bile Duct
1. To minimize the chance of pancreatic duct injury during Surgery, you should make the incision at
which of the following position in the ampulla?
(or)
You are a third-year resident in the surgery department. During a sphincterotomy procedure for
sphincter of Oddi dysfunction, the chief surgeon passes on the surgical blade to you and tells you to
continue the procedure. To minimize the chance of pancreatic duct injury during surgery, you should
make the incision at which of the following positions in the ampulla?
A. 3 o’clock position
B. 6 o’clock position
C. 9 o’clock position
D. 11 o’clock position
----------------------------------------
2. During the examination, a resident encounters the following images; according to Bismuth
classification, the two images come under which type?
(or)
In the intermediate module of gastroenterology residency, the residents are examined by an observed
structure performance evaluation (OSPE) examination. During the examination, a resident encounters
the following images; according to Bismuth classification, the given two images come under which
type? 1 2

A. Type IIIa; Type IIIb

382
B. Type IIIb; Type V
C. Type II; Type IIIa
D. Type IIIb; Type IV
----------------------------------------
3. A 46-year old Indian woman presents to emergency with a complaint of right upper abdominal pain,
fever and yellowing of eyes. An ultrasound is performed, which reveals a stone in the common bile
duct. Which of the following statements is best regarding choledocholithiasis?
(or)
Which of the following statements is best regarding choledocholithiasis?
A. Alkaline phosphatase is sensitive and specific for common bile duct stones
B. Primary common bile duct stones are more common in the Asian population
C. Laboratory values are always abnormal in patients with choledocholithiasis
D. ERCP is the investigation of choice for the diagnosis of common bile duct stones
----------------------------------------
4. A 55-year-old woman was admitted with cholangitis. She had undergone Surgery 11 years earlier for
cholelithiasis. Her postoperative course was complicated by biliary peritonitis. Biliary strictures were
noted, with dilated bile ducts proximal to the stricture. What is the probable cause of biliary stricture?
(or)
A 55-year-old woman was admitted to your hospital with cholangitis. She had undergone surgery 11
years ago for cholelithiasis when a laparoscopic procedure was converted to an open one. Her
postoperative course was complicated by biliary peritonitis, for which she underwent reoperation.
Eleven years later, the patient presented with fever as well as pain over her right upper quadrant with
jaundice for the past 4 weeks. Initially, she was admitted and managed conservatively with
symptomatic relief. On ultrasound, biliary strictures were noted with dilated bile ducts proximal to the
stricture. What is the probable cause of biliary stricture in the case given above?
A. Diet-induced
B. Iatrogenic trauma
C. Mirizzi’s syndrome
D. Cholangiohepatitis
----------------------------------------
5. A 52-year old lady presented with a 5-day history of jaundice and abdominal pain nausea and dark
urine. She was nauseated and had dark urine. On initial assessment, she was deeply icteric, and her
temperature was 38.2°C, but she was hemodynamically stable. Systemic examination did not reveal
any other abnormalities. No organs or lymph nodes were palpable, and the abdomen was soft and
non-tender. Biochemical analyses demonstrated leukocytosis. Intrahepatic duct dilatation was also
noted. What is the best next step in treatment for this condition?
(or)
A 52-year-old lady presented with jaundice and abdominal pain. Her temperature was 38.2°C, but she
was haemodynamically stable. An ultrasound demonstrates dilatation of the common bile duct to 1.5
cm. Intrahepatic duct dilatation was also noted. What is the best next step?
A. Conservative management with stone removal via ERCP

Page 2

383
B. Adequate hydration and IV antibiotics and then elective cholecystectomy
C. Emergency cholecystectomy
D. ERCP with sphincterotomy
----------------------------------------
6. A 74-year-old man presents to the clinic with a complaint of yellowing of eyes, pruritus and weight
loss. An ultrasound reveals a mass of the bile duct. Magnetic resonance cholangiopancreatography
(MRCP) provides detailed information about mass suggestive of hilar cholangiocarcinoma. Which of the
following are the non-resectability criteria in hilar cholangiocarcinoma?
(or)
Which of the following are the non-resectability criteria in hilar cholangiocarcinoma?
A. Hepatic duct involvement up to secondary biliary radicals bilaterally
B. Atrophy of one lobe without encasement of the contralateral portal vein branch
C. Hypertrophy of one lobe with ipsilateral involvement of secondary biliary radicals
D. Biopsy proved metastasis to lymph node stations inside the hepatoduodenal ligament
----------------------------------------
7. A 35-year old female patient, an occasional drinker, complains of severe pruritus that becomes more
bothersome in the evening. On examination, the patient was noted with icteric sclera. Laboratory tests
revealed anti-mitochondrial antibodies. What is the possible diagnosis?
(or)
A 35-year old female with severe pruritus, lab tests revealing presence of anti-mitochondrial antibodies.
What is the possible diagnosis?
A. Primary sclerosing cholangitis
B. Xanthogranulomatous cholecystitis
C. Primary biliary cirrhosis
D. Alcoholic cirrhosis
----------------------------------------
8. A 36-year old woman presents to the emergency department with complaint of fever, right upper
quadrant pain, and yellowish discolouration of eyes. The patient is recently diagnosed with gallbladder
stones. The resident is suspecting cholangitis. A fourth-year medical student asks the resident on duty
about the etiology of cholangitis. Which of the following is the best statement regarding the etiology of
cholangitis?
(or)
Which of the following is the best statement regarding the etiology of cholangitis?
A. Characterized by a triad of vomiting, jaundice, and fever
B. Most common organism responsible is Klebsiella
C. Most common cause of cholangitis is cholangiocarcinoma
D. Mandatory principles→ Obstruction+Infection
----------------------------------------

Page 3

384
9. In a patient with cholangitis, surgical intervention was performed. On the 10th day of the
postoperative period, the investigation given below was conducted. What is the name of this
investigation?
(or)
In a patient with cholangitis, surgical intervention was performed. On the 10th day, the investigation
given below was performed in the postoperative period. What is the name of this investigation?

A. Endoscopic retrograde cholangiopancreatography (ERCP)


B. Magnetic resonance cholangiopancreatography (MRCP)
C. T-tube cholangiogram
D. Percutaneous transhepatic cholangiogram (PTC)
----------------------------------------
10. A patient affected by more than 20 years of symptomatic cholelithiasis, with only obesity as risk
factor, underwent laparoscopic cholecystectomy. The sudden bleeding of the cystic artery, poor
visibility, and probably the long history of symptoms, producing a phlogistic alteration of the anatomy,
caused a misidentification of the cystic duct and the common bile duct leading to partial lateral wall
injury of the common hepatic duct. The error was unrecognized intra-operatively, but after progressive
jaundice, postoperative cholangiography was done, which showed the iatrogenic injury to CBD.
According to Strasberg's classification for bile duct injuries, the image given below comes under which
type?
(or)
According to Strasberg's Classification for bile duct injuries, this comes under which type?

A. Type B
B. Type C

Page 4

385
C. Type D
D. Type E2
----------------------------------------
11. A 45-year-old male presented with jaundice, weight loss, pruritus, and recurrent attacks of
cholangitis and a known case of ulcerative colitis. Endoscopic retrograde cholangiopancreatography
(ERCP) findings are given below. What is the treatment of choice for this condition?
(or)
Endoscopic retrograde cholangiopancreatography (ERCP) findings of a patient with recurrent
cholangitis is given below. What is the treatment of choice for this condition?

A. ERCP with stenting


B. High dose ursodeoxycholic acid
C. High dose prednisolone
D. Liver transplantation
----------------------------------------
12. Which of the statements is best related to haemobilia?
(or)
A 40-year old female known case of proximal cholangiocarcinoma underwent percutaneous
transhepatic biliary drainage as the patient’s bilirubin levels were very high. After the procedure, the
patient developed abdominal pain. On further evaluation, the patient was found to have hemobilia due
to an iatrogenic trauma to biliary tree. Which of the following statements is the best related to
hemobilia?
A. Tumors are the most common cause
B. The primary treatment of severe hemobilia is surgery
C. The classic triad is upper abdominal pain, upper gastrointestinal haemorrhage, jaundice
D. Ultrasonography usually reveals a specific diagnosis
----------------------------------------
13. A 5-year old female child, born of a non-consanguinous marriage, presented with a history of fever
and loose stools for 3 days and a lump in the abdomen noticed for 6 months. The lump was in the left
upper abdomen and has gradually increased in size since then. There was no history of icterus or any
bladder bowel complaints. A general examination revealed mild pallor. On systemic examination, the
child had splenomegaly (6 cm, firm) and hepatomegaly (3 cm, hard). Contrast-enhanced CT( CECT)

Page 5

386
was done; which of the following is the most likely diagnosis on the basis of CECT?
(or)
Which of the following is the diagnosis based on CECT?

A. Type 1 choledochal cyst


B. Type 3 choledochal cyst
C. Type 4 choledochal cyst
D. Type 5 choledochal cyst
----------------------------------------
14. You are an intern posted in the medicine department. Your chief resident gave you a CT film and
ordered you to get the radiologist's opinion as soon as possible. Being a hyperactive intern, barge into
the radiology department in the middle of a procedure without knocking. The irritated interventional
radiologist shows you the following image from the monitor and asks you what procedure is being
performed. What would be your answer?
(or)
The interventional radiologist shows you the following image from the monitor and asks you what
procedure is being performed. What would be your answer?

A. Percutaneous Transhepatic Cholangiography (PTC)


B. Endoscopic ultrasound
C. Endoscopic retrograde cholangiopancreatography (ERCP)
D. Magnetic resonance cholangiopancreatography(MRCP)
----------------------------------------

Page 6

387
15. During the viva voce of the final surgery examination, the professor handed over a printed image to
a student and asked to identify the structure. Which of the following structures form the complex shown
in the image?
(or)
Which of the following structures form the complex shown in the image?

A. Sphincter pancreaticus, Sphincter ampullae, Superior choledochal sphincter


B. Sphincter ampullae, Superior choledochal sphincter, Posterior choledochal sphincter
C. Superior choledochal sphincter, Posterior choledochal sphincter, Sphincter pancreaticus
D. Posterior choledochal sphincter, Sphincter ampullae, Superior choledochal sphincter
----------------------------------------
16. A 14-year old boy was involved in a bike accident, receiving blunt trauma to his upper abdomen.
After immediate admission to the local hospital, an intraperitoneal hemorrhage was demonstrated by
peritoneal lavage. Surgical laparotomy showed a large hepatic rupture 1.5 cm long on the upper
surface of the right lobe. Laceration repair was done, and adequate drainage was performed.
Hemostasis was completely successful at the end of the operation. On postoperative day 2,
asymptomatic clinical jaundice occurred. The total bilirubin level increased very rapidly to 18.7 mg/dl
and to 40.7 mg/dl on postoperative day 5. The other laboratory findings showed anemia, a slight
decrease in plasma proteins, and leukocytosis. On postoperative day 6, ERCP was done, which
showed the following finding: Which of the following is the most likely diagnosis?
(or)
What is the most likely diagnosis in a patient in which laparotomy showed large hepatic rupture 1.5 cm
long on the upper surface of the right lobe. Laceration repair was done. On postoperative day 2,
asymptomatic clinical jaundice occurred. The total bilirubin level increased very rapidly on
postoperative day 5. On day 6 ERCP was performed which revealed the finding given below. .

Page 7

388
A. Haemobilia
B. Bilhemia
C. Biliary stricture
D. Iatrogenic injury during the procedure
----------------------------------------
17. A 30-year-old female came to the OPD with complaints of pruritus and jaundice. On further
evaluation patient was found to have an elevated bilirubin level (direct > indirect), alkaline phosphatase
levels, and gamma-glutamyl transferase levels. On further evaluation, the patient was found to have a
choledochal cyst in the common bile duct. A choledochal cyst is characterised by which of the
following?
(or)
A choledochal cyst is characterized by which of the following?
A. Most commonly used classification system is the Todani modification of the Alonso-Lej classification
B. Most common type of choledochal cyst is Type III
C. Treatment of choice is conservative monitoring
D. Characterised by a triad of abdominal pain, mass, and continuous jaundice
----------------------------------------
18. A 42-year old woman presents to the emergency room with a complaint of severe epigastric pain
which radiates to the back and also has nausea. Her medical history is significant for recurrent
pancreatitis. Serum chemistry shows elevated lipase and normal liver function tests. A magnetic
resonance cholangiopancreatography (MRCP) reveals dilated common bile duct. ERCP is then
performed, which diagnoses the patient with sphincter of Oddi dysfunction. Which of the following
statement is clinically related to the sphincter of Oddi dysfunction?
(or)
Which of the following statement is clinically related to the sphincter of Oddi dysfunction?
A. Milwaukee classification is used to classify the sphincter of Oddi dysfunction
B. Males are affected more than females
C. Sphincter pressure >20 mmHg is diagnostic for sphincter of Oddi dysfunction
D. The sphincter of Oddi is 3-5 mm long and lies at the duodenal wall
----------------------------------------
19. A 7-week-old infant presented with jaundice and direct hyperbilirubinaemia, transaminitis and
elevated alkaline phosphatase and gamma-glutamyl transferase (GGT). The direct bilirubin level was
3.2. Ultrasound revealed the following finding. Which of the following is the treatment?
(or)
A 7-week old female infant presented with jaundice and abnormal laboratory findings significant for
direct hyperbilirubinemia, transaminitis and elevated alkaline phosphatase and gamma-glutamyl
transferase (GGT). The total bilirubin level was 6.4, and the direct bilirubin level was 3.2. The baby was
exclusively breastfed and otherwise healthy. Ultrasound was done, which revealed the following
finding. Which of the following is the most appropriate treatment?

Page 8

389
A. Billroth I
B. Billroth II
C. Kasai
D. Mayo
----------------------------------------
20. According to the Bismuth-Corlette classification system, perihilar cholangiocarcinoma extending
into the right secondary intrahepatic ducts is classified as?
(or)
A 38-year old female came to the OPD complaining of generalised pruritus, and the patient was found
to be deeply icteric. Ultrasonography revealed dilated intrahepatic biliary radicals. The patient was
further evaluated and diagnosed with a case of perihilar cholangiocarcinoma. According to the
Bismuth-Corlette classification system, perihilar cholangiocarcinoma extending into the right secondary
intrahepatic ducts is classified as?
A. Type II
B. Type III b
C. Type III a
D. Type IV
----------------------------------------
21. For the gastroenterology fellowship examination, Identify the lesion and appropriate treatment.
(or)
For the gastroenterology fellowship examination, the professor wants to examine candidates by relating
the radiological findings and the treatment plan so that candidates are competent enough to apply that
knowledge clinically. Identify the lesion and appropriate treatment?

Page 9

390
A. Type 1a and excision of the cystic dilatations in the extrahepatic biliary tree + cholecystectomy +
Roux-en-Y hepaticojejunostomy
B. Type 3 and endoscopic sphincterotomy
C. Type 1b and excision of the cystic dilatations in the extrahepatic biliary tree + cholecystectomy +
Roux-en-Y hepaticojejunostomy
D. Type 4b and excision and Roux-en-Y hepaticojejunostomy
----------------------------------------
22. While attending an outpatient rotation of gastroenterology, a patient brought his investigation
records, and the professors handed over a film to the resident. Identify the technique given in the image
below.
(or)
Identify the technique given in the image below.

A. Endoscopic retrograde cholangiopancreatography (ERCP)


B. Magnetic resonance cholangiopancreatography (MRCP)
C. CT colonoscopy
D. Sigmoid colonoscopy
----------------------------------------

Correct Answers
Question Correct Answer

Page 10

391
Question 1 4
Question 2 4
Question 3 2
Question 4 2
Question 5 2
Question 6 1
Question 7 3
Question 8 4
Question 9 3
Question 10 3
Question 11 4
Question 12 3
Question 13 4
Question 14 1
Question 15 1
Question 16 2
Question 17 1
Question 18 1
Question 19 3
Question 20 3
Question 21 3
Question 22 1

Solution for Question 1:


Option D: 11 o’clock position
• Sphincterotomy is performed from 10 to 11 o’clock position.
• Placing the incision at this location in the ampulla minimizes the chance of pancreatic duct injury,
which is generally located opposite the planned sphincterotomy site.
Option A: 3 o’clock position
• Incision at the 3 o’clock position increases the risk of pancreatic duct injury.
Option B: 6 o’clock position
• Incision at the 6 o’clock position increases the risk of pancreatic duct injury.
Option C: 9 o’clock position
• Incision at the 9 o’clock position increases the risk of pancreatic duct injury.

Page 11

392
Solution for Question 2:
Option D: Type IIIb; Type IV
• Perihilar cholangiocarcinoma also referred to as Klatskin tumours, are further classified based on
anatomic location by the Bismuth-Corlette classification. Type IIIb tumours extend into the left
secondary intrahepatic ducts. Type IV tumours involve both the right and left secondary intrahepatic
ducts.
• Type IIIb tumours extend into the left secondary intrahepatic ducts.
• Type IV tumours involve both the right and left secondary intrahepatic ducts.
• Type IIIb tumours extend into the left secondary intrahepatic ducts.
• Type IV tumours involve both the right and left secondary intrahepatic ducts.

Option A: Type IIIa; Type IIIb


• Type IIIa tumours extend into the right secondary intrahepatic ducts
• Type III b tumours extend into the left secondary intrahepatic ducts
Option B: Type IIIb; Type V
• Type III b tumours extend into the left secondary intrahepatic ducts
• Type V tumours involve the junction of the cystic duct and bile duct
Option C: Type II; Type IIIa
• Type II tumours involve the bifurcation without the involvement of the secondary intrahepatic ducts
• Type IIIa tumours extend into the right secondary intrahepatic ducts

Solution for Question 3:


Option B: Primary common bile duct stones are more common in the Asian population

Page 12

393
• Choledocholithiasis, or common bile duct stones, is classified by the point of origin.
• Primary common duct stones arise de novo in the bile duct. They are generally from brown pigment
stones, a combination of precipitated bile pigments and cholesterol. Brownstones are more common in
the Asian population and are associated with biliary stasis and bacterial infection of the bile duct. Biliary
stasis leading to the development of primary common bile duct (CBD) stones can be caused by biliary
strictures, papillary stenosis, tumours, or other (secondary) stones.
• Secondary stones pass from the gallbladder into the bile duct and are usually cholesterol stones.
They are common in the United States and are termed retained common duct stones when they are
found within 2 years after cholecystectomy.
Option A: Alkaline phosphatase is sensitive and specific for common bile duct stones
• Elevations of serum bilirubin, alkaline phosphatase and transaminases are commonly seen. Alkaline
phosphatase is not specific because it is produced by bone and the placenta.
Option C: Laboratory values are always abnormal in patients with choledocholithiasis
• In about one-third of patients with common bile duct stones, the liver chemistries are normal,
particularly if the obstruction is incomplete or intermittent.
Option D: ERCP is the investigation of choice for the diagnosis of common bile duct stones
• Ultrasound should be routinely used for evaluation of the gallbladder and biliary tree. A dilated
common bile duct (>8mm in diameter) on ultrasonography in a patient with gallstones, jaundice and
biliary pain is highly suggestive of common bile duct stones.
• Alternatively, MRCP is highly sensitive (>90%) with an almost 100% specificity for diagnosing
common duct stones. It is a non-invasive test and cannot be used for therapeutic purposes.
• Gold standard investigation – ERCP (Endoscopic Retrograde Cholangiopancreatography) but
investigation of choice – MRCP (Magnetic Resonance Cholangiopancreatography) for diagnosis of
common bile duct stones

Solution for Question 4:


Option B: Trauma (Iatrogenic)
• The most common cause of benign biliary stricture is related to operative injury, most often after
laparoscopic cholecystectomy (operative trauma).
• Bile duct strictures most commonly result in recurrent attacks of cholangitis but may present with
isolated jaundice without infection.
• Liver function tests show elevated bilirubin and alkaline phosphatase. Ultrasound or CT shows dilated
bile ducts proximal to the stricture.
• Treatment involves percutaneous or endoscopic dilatation and/or stent placement. For persistent or
complex strictures, surgical resection and reconstruction with Roux-en-Y choledochojejunostomy or
hepaticojejunostomy may be necessary.
Other options
Option A: Diet-induced
• This option is incorrect.
Option C: Mirizzi’s syndrome

Page 13

394
• Mirizzi’s syndrome is defined as common hepatic duct obstruction caused by extrinsic compression
from an impacted stone in the cystic duct or infundibulum of the gall bladder.

Option D: Cholangiohepatitis
• Recurrent pyogenic cholangiohepatitis, previously known as oriental cholangiohepatitis, is a condition
most commonly found in patients residing in or immigrated from Southeast Asia and is characterized by
intra and extrahepatic bile duct strictures and dilatation with an intraductal pigmented stone formation.
• Diagnosis is made after the exclusion of more common conditions, such as biliary stricture of a known
cause, such as previous surgery, trauma, primary or secondary sclerosing cholangitis, and
cholangiocarcinoma.

Solution for Question 5:


Option B: Adequate hydration and IV antibiotics and then elective cholecystectomy
• The diagnosis for the given clinical scenario is Acute cholangitis.
• Acute cholangitis, also known as ascending cholangitis, is a life-threatening condition caused by an
ascending bacterial infection of the biliary tree. Choledocholithiasis is the most common cause, with
infection-causing stones in the common bile duct leading to partial or complete obstruction of the biliary
system.
• Acute cholangitis is a severe medical condition that can progress quickly to septic shock and death.
• Adequate hydration and IV antibiotics should be started immediately.
• Patients who have suffered an episode of acute cholangitis related to gallstone disease should be
recommended to undergo elective cholecystectomy approximately 6 weeks after the resolution of
cholangitis.
Option A: Conservative management with stone removal via ERCP
• If the patient does not respond to medical therapy and progresses to shock, the patient requires
urgent stone removal by ERCP

Page 14

395
Option C: Emergency cholecystectomy
• Emergency cholecystectomy is indicated for gall bladder necrosis and emphysematous gall bladder.
Option D: ERCP with sphincterotomy
• Emergent decompression of the biliary tree is usually accomplished by ERCP and sphincterotomy.
• Indications for emergent sphincterotomy are: The common bile duct is dilated and contains multiple
stones and biliary sludge. When papilla is fibrosed and stenosed secondary to the passage of stones
through it.
• The common bile duct is dilated and contains multiple stones and biliary sludge.
• When papilla is fibrosed and stenosed secondary to the passage of stones through it.
• The common bile duct is dilated and contains multiple stones and biliary sludge.
• When papilla is fibrosed and stenosed secondary to the passage of stones through it.

Solution for Question 6:


Option A: Hepatic duct involvement up to secondary biliary radicals bilaterally
• This option is correct because of hepatic duct involvement up to the secondary biliary radicals
bilaterally.
Option B: Atrophy of one lobe without encasement of the contralateral portal vein branch
• Cholangiocarcinoma is non-resectable if there is an atrophy of one lobe with encasement of the
contralateral portal vein branch.
Option C: Hypertrophy of one lobe with ipsilateral involvement of secondary biliary radicals
• Cholangiocarcinoma is non-resectable if there is atrophy of one lobe with contralateral involvement of
secondary biliary radicals.
Option D: Biopsy proved metastasis to lymph node stations inside the hepatoduodenal ligament
• Cholangiocarcinoma is non-resectable if there is biopsy-proven metastasis to lymph node stations
outside the hepatoduodenal ligament.

Solution for Question 7:


Option C: Primary biliary cirrhosis
• The possible diagnosis is primary biliary cirrhosis or chronic non-suppurative destructive cholangitis.
• It is an autoimmune disorder that leads to the destruction of intrahepatic bile ducts and subsequent
cholestasis and cirrhosis. Biliary epithelial cells are the primary target of injury in PBC. Most common in
middle-aged women
• Pathology Progressive destruction of intrahepatic bile ducts only Diagnostic appearance – florid duct
lesion with lymphocytic infiltration and granulomatous inflammation.
• Progressive destruction of intrahepatic bile ducts only

Page 15

396
• Diagnostic appearance – florid duct lesion with lymphocytic infiltration and granulomatous
inflammation.
• Clinical features Pruritus precedes jaundice. The earliest symptom is usually unrelenting pruritus,
even before clinical jaundice is apparent, but later the patient may experience covert jaundice. Pruritus
and fatigue (most common symptom) – characteristic symptoms The patient may later present with
hepatosplenomegaly, xanthelasma, especially in the palm and around the eyes, vitiligo, and arthritis.
PBC is often found in association with other connective tissue diseases
• Pruritus precedes jaundice. The earliest symptom is usually unrelenting pruritus, even before clinical
jaundice is apparent, but later the patient may experience covert jaundice.
• Pruritus and fatigue (most common symptom) – characteristic symptoms
• The patient may later present with hepatosplenomegaly, xanthelasma, especially in the palm and
around the eyes, vitiligo, and arthritis. PBC is often found in association with other connective tissue
diseases
• Diagnosis Anti-mitochondrial antibodies and antinuclear antibodies directed against self-antigen can
confirm the diagnosis. An antimitochondrial antibody is the hallmark for diagnosis of PBC and is
detected in 95% of patients. Investigation of choice - Biopsy
• Anti-mitochondrial antibodies and antinuclear antibodies directed against self-antigen can confirm the
diagnosis. An antimitochondrial antibody is the hallmark for diagnosis of PBC and is detected in 95% of
patients.
• Investigation of choice - Biopsy
• Treatment is liver transplantation
• Progressive destruction of intrahepatic bile ducts only
• Diagnostic appearance – florid duct lesion with lymphocytic infiltration and granulomatous
inflammation.
• Pruritus precedes jaundice. The earliest symptom is usually unrelenting pruritus, even before clinical
jaundice is apparent, but later the patient may experience covert jaundice.
• Pruritus and fatigue (most common symptom) – characteristic symptoms
• The patient may later present with hepatosplenomegaly, xanthelasma, especially in the palm and
around the eyes, vitiligo, and arthritis. PBC is often found in association with other connective tissue
diseases
• Anti-mitochondrial antibodies and antinuclear antibodies directed against self-antigen can confirm the
diagnosis. An antimitochondrial antibody is the hallmark for diagnosis of PBC and is detected in 95% of
patients.
• Investigation of choice - Biopsy
Option A: Primary sclerosing cholangitis
• It is an uncommon disease characterized by inflammatory strictures involving the intrahepatic and
extrahepatic biliary trees. It is a progressive disease that eventually results in secondary biliary
cirrhosis. PSC is commonly associated with other autoimmune diseases, including ulcerative colitis in
about two-thirds of patients, Riedel’s thyroiditis, and retroperitoneal fibrosis.
• It is associated with P-ANCA antibodies
Option B: Xanthogranulomatous cholecystitis
• Xanthogranulomatous cholecystitis (XGC) is an uncommon inflammatory disease of the gall bladder
that may be difficult to differentiate from malignancy, both on imaging and pathologically.

Page 16

397
• It is characterized by multiple intramural nodules.
Option D: Alcoholic cirrhosis
• Cirrhosis is a late-stage liver disease in which healthy liver tissue is replaced with scar tissue, and the
liver is permanently damaged.

Solution for Question 8:


Option D: Mandatory principles→ Obstruction+Infection
• CHOLANGITIS Acute cholangitis is due to an acute, ascending bacterial infection of the biliary tree
caused by an obstruction. The two absolute requirements for the development of acute cholangitis are:
Bacteria in the biliary tree (infection) Obstruction of flow with increased intraluminal pressure
• Acute cholangitis is due to an acute, ascending bacterial infection of the biliary tree caused by an
obstruction.
• The two absolute requirements for the development of acute cholangitis are: Bacteria in the biliary
tree (infection) Obstruction of flow with increased intraluminal pressure
• Bacteria in the biliary tree (infection)
• Obstruction of flow with increased intraluminal pressure
• Acute cholangitis is due to an acute, ascending bacterial infection of the biliary tree caused by an
obstruction.
• The two absolute requirements for the development of acute cholangitis are: Bacteria in the biliary
tree (infection) Obstruction of flow with increased intraluminal pressure
• Bacteria in the biliary tree (infection)
• Obstruction of flow with increased intraluminal pressure
• Bacteria in the biliary tree (infection)
• Obstruction of flow with increased intraluminal pressure
Option A: Characterized by a triad of vomiting, jaundice, and fever
• The classic presentation of acute cholangitis is Charcot triad with fever, jaundice, and right upper
quadrant pain.
• Reynolds pentad is Charcot triad plus shock and altered mental status.
Option B: Most common organism responsible is Klebsiella
• The source of bacteria is unknown because the culture of most bile is sterile. The most common
pathogens include E.coli, Klebsiella, Enterobacter, Pseudomonas and Citrobacter.
Option C: Most common cause of cholangitis is cholangiocarcinoma
• Gallstones are the most common cause of obstruction in cholangitis. Other causes include primary
sclerosing cholangitis, benign and malignant stricture, parasites, instrumentation of the ducts and
indwelling stents, and partially obstructed biliary enteric anastomosis.

Page 17

398
Solution for Question 9:
Option C: T-tube cholangiogram
• The name of the investigation given is T-tube cholangiogram.
• T-tube cholangiogram is a special X-ray investigation carried out using X-ray dye to visualize the bile
ducts after cholecystectomy or after common bile duct exploration with supra-duodenal
choledochotomy.
• It provides external bile drainage into a controlled route while the healing process of cholecystectomy
matures and the original pathology resolves.
• Common bile duct T-tubes should remain in for 10 days.
• However, once the T-tube cholangiogram has shown that there is a free flow of bile into the
duodenum and that there are no retained stones, some surgeons like to clamp the T-tube prior to
removal.
• The 10-day period is required to minimize the risk of biliary peritonitis after removal.
Option A: Endoscopic retrograde cholangiopancreatography (ERCP)
• ERCP is used both as a diagnostic and therapeutic procedure.
• It helps in determining the cause and level of obstruction.
• During ERCP, bile aspirates can be sent for cytological and microbiological examination, and
endoluminal brushings can be taken from strictures for cytological examination
• Therapeutic interventions such as stent placement or stone removal to relieve the obstruction can be
performed.
Option B: Magnetic resonance cholangiopancreatography (MRCP)
• MRCP is a non-invasive imaging technique to visualize the intra and extrahepatic biliary tree and
pancreatic ductal system.
• It can provide diagnostically-equivalent images to ERCP and is a useful technique in high-risk patients
to avoid significant morbidity.

Option D: Percutaneous transhepatic cholangiogram (PTC)

Page 18

399
• PTC demonstrates the intrahepatic biliary tree, identifies the location of the injury, provides drainage
of bile, and possibly even allows the leak to close.

Solution for Question 10:


Option C: Type D
• Bile duct injury The most devastating complication of any right upper quadrant operation occurring
with any significant frequency is iatrogenic bile duct injury. More than 80% of all iatrogenic bile duct
injury occurs during cholecystectomy. Visual misperception accounts for 97% of iatrogenic biliary
injuries. In experienced hands, the rate of iatrogenic biliary injury is likely 0.3% or less in both open and
laparoscopic cholecystectomy. The critical view of safety remains the current standard technique to
reduce the incidence of bile duct injury during cholecystectomy. Strasberg classification for bile duct
injuries (Type E is subdivided into E1 to E5 according to Bismuth classification). Type A encompasses
the most common cause of biliary leak after cholecystectomy. However, the above injury is a lateral
injury to a major bile duct, which comes under type D. Strasberg classification
• The most devastating complication of any right upper quadrant operation occurring with any
significant frequency is iatrogenic bile duct injury.
• More than 80% of all iatrogenic bile duct injury occurs during cholecystectomy. Visual misperception
accounts for 97% of iatrogenic biliary injuries.
• In experienced hands, the rate of iatrogenic biliary injury is likely 0.3% or less in both open and
laparoscopic cholecystectomy. The critical view of safety remains the current standard technique to
reduce the incidence of bile duct injury during cholecystectomy.
• Strasberg classification for bile duct injuries (Type E is subdivided into E1 to E5 according to Bismuth
classification). Type A encompasses the most common cause of biliary leak after cholecystectomy.
• However, the above injury is a lateral injury to a major bile duct, which comes under type D.
• Strasberg classification

Page 19

400
• The most devastating complication of any right upper quadrant operation occurring with any
significant frequency is iatrogenic bile duct injury.
• More than 80% of all iatrogenic bile duct injury occurs during cholecystectomy. Visual misperception
accounts for 97% of iatrogenic biliary injuries.
• In experienced hands, the rate of iatrogenic biliary injury is likely 0.3% or less in both open and
laparoscopic cholecystectomy. The critical view of safety remains the current standard technique to
reduce the incidence of bile duct injury during cholecystectomy.
• Strasberg classification for bile duct injuries (Type E is subdivided into E1 to E5 according to Bismuth
classification). Type A encompasses the most common cause of biliary leak after cholecystectomy.
• However, the above injury is a lateral injury to a major bile duct, which comes under type D.
• Strasberg classification

Option A: Type B
• Type B injury is the occlusion of the biliary tree, commonly aberrant right hepatic duct(s).
Option B: Type C
• Type C injury is the transaction without ligation of aberrant right hepatic duct(s).
Option D: Type E2
• Type E2 injury is the injury to the common hepatic duct, which is less than 2cm from the confluence.

Solution for Question 11:


Option D: Liver transplantation
• The image given above shows pruned tree appearance, which is suggestive of primary sclerosing
cholangitis.

Page 20

401
• Primary sclerosing cholangitis (PSC) It is an uncommon disease characterized by inflammatory
strictures involving the intrahepatic and extrahepatic biliary trees. It is a progressive disease that
eventually results in secondary biliary cirrhosis. PSC is commonly associated with other autoimmune
diseases, including ulcerative colitis in about two-thirds of patients, Riedel’s thyroiditis, and
retroperitoneal fibrosis. There is no curative treatment, and medical management is largely supportive.
In patients with PSC and advanced liver disease, liver transplantation is the only option.
• It is an uncommon disease characterized by inflammatory strictures involving the intrahepatic and
extrahepatic biliary trees. It is a progressive disease that eventually results in secondary biliary
cirrhosis. PSC is commonly associated with other autoimmune diseases, including ulcerative colitis in
about two-thirds of patients, Riedel’s thyroiditis, and retroperitoneal fibrosis.
• There is no curative treatment, and medical management is largely supportive.
• In patients with PSC and advanced liver disease, liver transplantation is the only option.
• It is an uncommon disease characterized by inflammatory strictures involving the intrahepatic and
extrahepatic biliary trees. It is a progressive disease that eventually results in secondary biliary
cirrhosis. PSC is commonly associated with other autoimmune diseases, including ulcerative colitis in
about two-thirds of patients, Riedel’s thyroiditis, and retroperitoneal fibrosis.
• There is no curative treatment, and medical management is largely supportive.
• In patients with PSC and advanced liver disease, liver transplantation is the only option.
Option A: ERCP with stenting
• Therapeutic ERCP is useful in primary sclerosing cholangitis to dilate or to stent dominant strictures
and to perform surveillance for cholangiocarcinoma by way of brush cytology and biopsy.
• However, in advanced liver disease, liver transplantation is the only option.
Option B: High dose ursodeoxycholic acid
• Long-term high-dose ursodeoxycholic acid therapy is associated with improvement in serum liver
tests in PSC.
• However, it does not improve survival and is associated with higher rates of serious adverse events.
Option C: High-dose prednisolone
• Corticosteroids have been used to treat PSC; however, it is not recommended because patients with
cholestatic liver disease are prone to prednisone side effects such as osteoporosis.

Solution for Question 12:


Option C: The classic triad is upper abdominal pain, upper gastrointestinal hemorrhage, jaundice
• Hemobilia is bleeding into the biliary tree from abnormal communication between a blood vessel and
the bile duct. Causes of hemobilia are iatrogenic trauma, accidental trauma, gall stones, tumours,
inflammatory disorders, and vascular disorders.
• The symptoms and signs of major haemobilia are melena, hematemesis, biliary colic and jaundice.
• embolization is the mainstay of therapy, other options include thrombin injection, stent placement,
and/or placement of a percutaneous biliary drain.
Other options
Option A: Tumors are the most common cause

Page 21

402
• The most common cause of hemobilia is iatrogenic trauma to the liver and biliary tree. Percutaneous
liver biopsy results in hemobilia in less than 1% of cases, but percutaneous transhepatic biliary
drainage procedures have an incidence of 2% to 10%
Option B: The primary treatment of severe hemobilia is surgery
• Most cases of minor bleeding can be managed conservatively.
• Surgery is indicated when conservative therapy and transarterial embolization have failed.
Option D: Ultrasonography usually reveals a specific diagnosis
• Arterial angiography is now recognized as the test of choice when significant hemobilia is suspected
and will reveal the source of bleeding in approximately 90% of cases.
• Once hemobilia is suspected, the first workup should be upper gastrointestinal endoscopy, but it is
diagnostic of hemobilia in only 10% of cases. Ultrasound and CT are other modalities used.

Solution for Question 13:


Option D: Type 5 choledochal cyst
• The CECT shows multiple cystic dilatations of the intrahepatic bile ducts.
• The arrow indicates the central dot sign (enhancing dots within the dilated intrahepatic bile ducts
representing portal radicals), which is seen in type 5 choledochal cyst/ Caroli’s disease (multiple cysts
involving only intrahepatic bile duct)
• Choledochal cyst: Choledochal cysts are congenital dilatations of the intra and/or extrahepatic biliary
system. Anomalous pancreaticobiliary junction (long common channel >1.5 cm) may play a role in
pathogenesis. They affect females more than males. Patients may present at any age with jaundice,
abdominal pain, and a right upper quadrant mass on examination. They also have an increased risk of
developing cholangiocarcinoma, with the risk varying directly with age at diagnosis. Ultrasonography
will confirm the presence of an abnormal cyst, and magnetic resonance imaging (MRCP/MRI) will
reveal the anatomy. Radical excision of the cyst is the treatment of choice with the reconstruction of the
biliary tract using a Roux-en-Y loop of the jejunum.
• Choledochal cysts are congenital dilatations of the intra and/or extrahepatic biliary system.
• Anomalous pancreaticobiliary junction (long common channel >1.5 cm) may play a role in
pathogenesis.
• They affect females more than males.
• Patients may present at any age with jaundice, abdominal pain, and a right upper quadrant mass on
examination. They also have an increased risk of developing cholangiocarcinoma, with the risk varying
directly with age at diagnosis.
• Ultrasonography will confirm the presence of an abnormal cyst, and magnetic resonance imaging
(MRCP/MRI) will reveal the anatomy.
• Radical excision of the cyst is the treatment of choice with the reconstruction of the biliary tract using a
Roux-en-Y loop of the jejunum.
• Choledochal cysts are congenital dilatations of the intra and/or extrahepatic biliary system.
• Anomalous pancreaticobiliary junction (long common channel >1.5 cm) may play a role in
pathogenesis.

Page 22

403
• They affect females more than males.
• Patients may present at any age with jaundice, abdominal pain, and a right upper quadrant mass on
examination. They also have an increased risk of developing cholangiocarcinoma, with the risk varying
directly with age at diagnosis.
• Ultrasonography will confirm the presence of an abnormal cyst, and magnetic resonance imaging
(MRCP/MRI) will reveal the anatomy.
• Radical excision of the cyst is the treatment of choice with the reconstruction of the biliary tract using a
Roux-en-Y loop of the jejunum.
Option A: Type 1 choledochal cyst
• Type 1 cysts are the most common and have the highest risk of malignancy (>60%)
• These are dilatation of the extrahepatic bile duct only. 1a: Cystic 1b: Saccular 1c: Fusiform
• 1a: Cystic
• 1b: Saccular
• 1c: Fusiform
• 1a: Cystic
• 1b: Saccular
• 1c: Fusiform
Option B: Type 3 choledochal cyst
• Type 3 is a choledochocoele involving the intraduodenal portion of the common bile duct (CBD)
Option C: Type 4 choledochal cyst
• Intra and extrahepatic duct dilatation 4a: Intro and extrahepatic cysts 4b: Multiple extrahepatic cysts
• 4a: Intro and extrahepatic cysts
• 4b: Multiple extrahepatic cysts
• 4a: Intro and extrahepatic cysts
• 4b: Multiple extrahepatic cysts

Solution for Question 14:


Option A: Percutaneous Transhepatic Cholangiography (PTC)
• The procedure performed is Percutaneous Transhepatic Cholangiography
• The best method to visualize the proximal duct is PTC.
• Percutaneous Transhepatic Cholangiography It is an invasive procedure used to evaluate the biliary
tree. A needle is passed directly into the liver to access one of the biliary radicals, and the tract is then
used for the insertion of transhepatic catheters. PTC can decompress proximal biliary obstruction and
stent obstructions non-operatively and provide anatomic information for biliary reconstruction
• It is an invasive procedure used to evaluate the biliary tree. A needle is passed directly into the liver to
access one of the biliary radicals, and the tract is then used for the insertion of transhepatic catheters.

Page 23

404
• PTC can decompress proximal biliary obstruction and stent obstructions non-operatively and provide
anatomic information for biliary reconstruction
• It is an invasive procedure used to evaluate the biliary tree. A needle is passed directly into the liver to
access one of the biliary radicals, and the tract is then used for the insertion of transhepatic catheters.
• PTC can decompress proximal biliary obstruction and stent obstructions non-operatively and provide
anatomic information for biliary reconstruction
Option B: Endoscopic ultrasound
• It has limited use in the evaluation of gallbladder disease or intrahepatic disease of the biliary tree. It is
used to identify gall stones and cholecystitis
• It is valuable in the assessment of the distal common bile duct and ampulla.
Option C: Endoscopic retrograde cholangiopancreatography (ERCP)
• It is an invasive test using endoscopy and fluoroscopy to inject contrast material through the ampulla
to image the biliary tree.
• The distal bile duct is best visualized by ERCP
Option D: Magnetic resonance cholangiopancreatography(MRCP)
• The image shown is not MRCP.

Solution for Question 15:


Option A: Sphincter pancreaticus, Sphincter ampullae, Superior choledochal sphincter
• The image shown above is the sphincter of the Oddi complex.
• It is formed by the following structures: Sphincter ampullae Sphincter pancreaticus Superior
choledochal sphincter Inferior choledochal sphincter
• Sphincter ampullae
• Sphincter pancreaticus
• Superior choledochal sphincter
• Inferior choledochal sphincter
• Sphincter ampullae
• Sphincter pancreaticus
• Superior choledochal sphincter
• Inferior choledochal sphincter
Option B: Sphincter ampullae, Superior choledochal sphincter, Posterior choledochal sphincter
• The image shown above is the sphincter of the Oddi complex.
• Posterior choledochal sphincter is not a part of sphincter of Oddi complex.
• Therefore, this option is incorrect.
Option C: Superior choledochal sphincter, Posterior choledochal sphincter, Sphincter pancreaticus
• Posterior choledochal sphincter is not a part of sphincter of Oddi complex.

Page 24

405
Option D: Posterior choledochal sphincter, Sphincter ampullae, Superior choledochal sphincter
• The image shown above is the sphincter of the Oddi complex.
• Posterior choledochal sphincter is not a part of sphincter of Oddi complex.
• Therefore, this option is incorrect.

Solution for Question 16:


Option B: Bilhemia
• Fig-A, Endoscopic retrograde cholangiogram. Bile ducts injected by catheter (C) positioned in the
common bile duct. Opacification of fistula (arrow) and segmental branch of right hepatic vein (RHV).
• Fig-B, Selective opacification of the segmental branch of the hepatic vein, fistula (arrow), and bile duct
• BILHEMIA: It is the reverse of haemobilia, with bile entering the bloodstream through communication
with the hepatic veins or with branches of the portal vein. The reversed flow is caused by the inversion
of the pressure gradient due to high intraductal biliary pressure, generally resulting from peripheral
obstruction or low venous pressure. The symptoms are rapidly increasing jaundice with elevated direct
bilirubin without increased liver enzymes. Treatment involves endoscopic sphincterotomy or
percutaneous transhepatic drain or endostenting.
• It is the reverse of haemobilia, with bile entering the bloodstream through communication with the
hepatic veins or with branches of the portal vein.
• The reversed flow is caused by the inversion of the pressure gradient due to high intraductal biliary
pressure, generally resulting from peripheral obstruction or low venous pressure.
• The symptoms are rapidly increasing jaundice with elevated direct bilirubin without increased liver
enzymes.
• Treatment involves endoscopic sphincterotomy or percutaneous transhepatic drain or endostenting.
• It is the reverse of haemobilia, with bile entering the bloodstream through communication with the
hepatic veins or with branches of the portal vein.
• The reversed flow is caused by the inversion of the pressure gradient due to high intraductal biliary
pressure, generally resulting from peripheral obstruction or low venous pressure.
• The symptoms are rapidly increasing jaundice with elevated direct bilirubin without increased liver
enzymes.
• Treatment involves endoscopic sphincterotomy or percutaneous transhepatic drain or endostenting.
Option A: Haemobilia
• An abnormal communication between a blood vessel and the bile duct causes bleeding into the biliary
tree.
• The classic triad is upper abdominal pain, upper gastrointestinal hemorrhage, and jaundice.
• The most common cause of haemobilia is iatrogenic trauma, mostly due to percutaneous transhepatic
biliary drainage from 2% to 10%
• The test of choice is arterial angiography.
• Most bleeds are managed conservatively; however, angiography with transarterial embolization is
indicated for major haemobilia requiring blood transfusion.

Page 25

406
Option C: Biliary stricture
• In biliary strictures, hepatic enzymes are deranged with an elevation of direct bilirubin.
Option D: Iatrogenic injury during the procedure
• Iatrogenic injury during the procedure causes haemobilia.
• Bilhemia is an abnormal reversed flow between hepatic veins and the bloodstream.

Solution for Question 17:


Option A: Most commonly used classification system is the Todani modification of the Alonso-Lej classi
fication.
• Choledochal cyst Choledochal cysts are congenital dilatations of the intra and/or extrahepatic biliary
system. Anomalous pancreaticobiliary junction (long common channel >1.5 cm) may play a role in
pathogenesis. They affect females more than males. The most commonly used classification system is
the Todani modification of the Alonso-Lej classification. Clinical features: Abdominal Mass +Abdominal
Pain + Obstructive Jaundice
• Choledochal cysts are congenital dilatations of the intra and/or extrahepatic biliary system.
• Anomalous pancreaticobiliary junction (long common channel >1.5 cm) may play a role in
pathogenesis.
• They affect females more than males.
• The most commonly used classification system is the Todani modification of the Alonso-Lej
classification.
• Clinical features: Abdominal Mass +Abdominal Pain + Obstructive Jaundice
• Choledochal cysts are congenital dilatations of the intra and/or extrahepatic biliary system.
• Anomalous pancreaticobiliary junction (long common channel >1.5 cm) may play a role in
pathogenesis.
• They affect females more than males.
• The most commonly used classification system is the Todani modification of the Alonso-Lej
classification.
• Clinical features: Abdominal Mass +Abdominal Pain + Obstructive Jaundice
Option B: Most common type of choledochal cyst is Type III
• Most common type of choledochal cyst is Type I > Type IV> Type III
Option C: Treatment of choice is conservative monitoring
• They also have an increased risk of developing cholangiocarcinoma, with the risk varying directly with
age at diagnosis.
• Ultrasonography will confirm the presence of an abnormal cyst, and magnetic resonance imaging
(MRCP/MRI) will reveal the anatomy.
• Radical excision of the cyst is the treatment of choice with a reconstruction of the biliary tract using a
Roux-en-Y loop of the jejunum.
Option D: Characterised by a triad of abdominal pain, mass, and continuous jaundice

Page 26

407
• Patients may present at any age with the triad of intermittent jaundice, abdominal pain and a right
upper quadrant mass on examination.

Solution for Question 18:


Option A: Milwaukee classification is used to classify the sphincter of Oddi dysfunction
Sphincter of Oddi dysfunction(SOD)
• It is caused by a structurally abnormal sphincter or a histologically normal but functionally abnormal
one.
• It manifests as biliary tract pain with normal liver function tests and recurrent pancreatitis. Scarring or
stenosis of the sphincter can result from the passage of stones, pancreatitis, or prior endoscopic
sphincterotomies.
• Noninvasive options include calcium channel blockers, tricyclic antidepressants, glyceryl trinitrate, and
somatostatin.
• Invasive interventions for the treatment of SOD include ERCP with sphincterotomy
• Two clinical types Biliary type SOD Pancreatic type SOD
• Biliary type SOD
• Pancreatic type SOD
• Milwaukee classification of the sphincter of Oddi (SOD) dysfunction:
• Biliary type SOD
• Pancreatic type SOD
Type I:
• Typical biliary-type pain
• Liver enzymes (AST, ALT or ALP) >2 times normal limit documented on at least 2 occasions during
episodes of pain
• Dilated CBD >12 mm in diameter
• Prolonged biliary drainage time (>45 min)
• Pancreatic-type pain
• Amylase and/or lipase >2 times upper normal limit on at least 2 occasions during episodes of pain
• Dilated pancreatic duct (head >6 mm, body >5 mm)
• Prolonged pancreatic drainage time (>9 min)
Type II:
• Biliary type pain, and
• One or two of the above criteria
• Pancreatic type pain, and one or two of the above criteria
Type III:
Option B: Males are affected more than females

Page 27

408
• Females are affected more than males.
Option C: Sphincter pressure >20 mmHg is diagnostic for sphincter of Oddi dysfunction
• Sphincter pressure higher than 40 mmHg is the manometric criterion used to diagnose SOD
Option D: The sphincter of Oddi is 3-5 mm long and lies at the duodenal wall
• The sphincter of Oddi is 6-10 mm long and lies within the duodenal wall

Solution for Question 19:


Option C: Kasai
• The given ultrasound image shows a Triangular cord sign which is seen in Biliary atresia
• Biliary atresia The extrahepatic bile ducts are progressively destroyed by an inflammatory process
that starts around the time of birth. Intrahepatic changes eventually occur, causing biliary cirrhosis and
portal hypertension. Untreated, death from the consequences of liver failure occurs before the age of 3
years. Treatment of choice for biliary atresia: Kasai portoenterostomy
• The extrahepatic bile ducts are progressively destroyed by an inflammatory process that starts around
the time of birth.
• Intrahepatic changes eventually occur, causing biliary cirrhosis and portal hypertension. Untreated,
death from the consequences of liver failure occurs before the age of 3 years.
• Treatment of choice for biliary atresia: Kasai portoenterostomy
• The extrahepatic bile ducts are progressively destroyed by an inflammatory process that starts around
the time of birth.
• Intrahepatic changes eventually occur, causing biliary cirrhosis and portal hypertension. Untreated,
death from the consequences of liver failure occurs before the age of 3 years.
• Treatment of choice for biliary atresia: Kasai portoenterostomy
Option A: Billroth I
• Billroth I is the gastroduodenal anastomosis; this procedure is performed for gastric ulcers
Option B: Billroth II
• Billroth II is the gastrojejunostomy; this procedure is done for refractory peptic ulcers and gastric
adenocarcinoma.
Option D: Mayo
• Mayo repair is used for umbilical hernia repair

Solution for Question 20:


Option C: Type III a
• Perihilar cholangiocarcinoma, also referred to as Klatskin tumours, is further classified based on
anatomic location of the tumor, by the Bismuth-Corlette classification.
• Type III a: Tumours extend into the right secondary intrahepatic duct.

Page 28

409
Option A: Type II
• Type II tumours involve the bifurcation without the involvement of the secondary intrahepatic ducts
Option B: Type III b
• Type III b tumours extend into the left secondary intrahepatic ducts.
Option D: Type IV
• Type IV tumours involve both the right and left secondary intrahepatic ducts.

Solution for Question 21:


Option C: Type 1b and excision of the cystic dilatations in the extrahepatic biliary tree +
cholecystectomy + Roux-en-Y hepaticojejunostomy
The image shown here is choledochal cyst.
• Type 1 cysts are the most common and have the highest risk of malignancy (>60%)
• Patients may present mostly before 10 years of age with the triad of intermittent jaundice, abdominal
pain, and a right upper quadrant mass on examination. They also have an increased risk of developing
cholangiocarcinoma, with the risk varying directly with age at diagnosis.
• Ultrasonography will confirm the presence of an abnormal cyst, and magnetic resonance imaging
(MRCP/MRI) will reveal the anatomy.
• The cyst shown in the image above is type 1b saccular dilatation extrahepatic bile duct only.
• Radical excision of the cyst is the treatment of choice with the reconstruction of the biliary tract using a
Roux-en-Y loop of the jejunum.

Option A: Type 1a and excision of the cystic dilatations in the extrahepatic biliary tree +
cholecystectomy + Roux-en-Y hepaticojejunostomy
• Type 1a is the cystic dilatation of the extrahepatic bile duct only.

Page 29

410
• Treatment is excision of the cystic dilatations in the extrahepatic biliary tree + cholecystectomy +
Roux-en-Y hepaticojejunostomy.
Option B: Type 3 and endoscopic sphincterotomy
• Type 3 is choledochocoele involving the intraduodenal portion of the common bile duct (CBD), and its
treatment is endoscopic sphincterotomy.
Option D: Type 4b and excision and Roux-en-Y hepaticojejunostomy
• Multiple extrahepatic cysts are classified as type 4b, and the treatment is Roux-en-Y
hepaticojejunostomy

Solution for Question 22:


Option A: Endoscopic retrograde cholangiopancreatography (ERCP)
• The technique given above is endoscopic retrograde cholangiopancreatography (ERCP)

• ERCP is used both as a diagnostic and therapeutic procedure.


• ERCP helps to identify the cause and obstruction level.
• Using a side viewing scope, the ampulla of Vater is identified and cannulated.
• Injection of water-soluble contrast directly into the bile duct provides excellent images of the ductal
anatomy.
• It helps in determining the cause and level of obstruction.
Option B: Magnetic resonance cholangiopancreatography (MRCP)
• MRCP is a non-invasive imaging technique to visualize the intra and extrahepatic biliary tree and
pancreatic ductal system.
• It can provide diagnostically-equivalent images to ERCP and is a useful technique in high-risk patients
to avoid significant morbidity.

Page 30

411
Option C: CT colonoscopy
• Computed tomographic (CT) colonography, also called virtual colonoscopy (VC) or CT pneumocolon,
is a powerful, minimally invasive technique for colorectal screening.

Option D: Sigmoidoscopy
• A sigmoidoscopy is a diagnostic test used to check the sigmoid colon, which is the lower part of the
large intestine.
• This section of the colon is close to the rectum and anus

Page 31

412
Pancreas Part 1
1. A 60-year-old chronic smoker presented with painless jaundice, pruritus, and clay-colored stools for
2 months. History of waxing and waning of jaundice was present. A CT scan reveals a dilated main
pancreatic duct and common bile duct. What is the likely diagnosis?
A. Carcinoma of the head of the pancreas
B. Periampullary carcinoma
C. Chronic pancreatitis
D. Hilar cholangiocarcinoma
----------------------------------------
2. A 30-year-old patient who is a chronic smoker comes to the clinic complaining of tiredness and
yellowing of skin for the past 1 month. CT scan reveals a carcinoma of the head of the pancreas. What
is the median survival in carcinoma pancreas after surgery and adjuvant therapy?
(or)
What is the median survival rate in a patient with carcinoma head of pancreas?
A. 12 months
B. 22 months
C. 32 months
D. 44 months
----------------------------------------
3. A 32y.o male came to your clinic with a headache. On inspection of the patient, you see the following
finding shown in the image and diagnose a genetic disease. What type of cancer would you expect the
patient to have?
(or)
A 32-year-old male came to your clinic with a complaint of headache with no other associated
symptoms. On inspection of the patient, you see the following finding shown in the image and diagnose
a genetic disease. What type of cancer would you expect the patient to have?

A. Pancreatic cancer
B. Ganglioma
C. Testicular cancer

413
D. Esophageal cancer
----------------------------------------
4. A 30-year-old woman comes to the emergency department complaining of epigastric pain for the last
5 hours. She has consumed 14 units of alcohol per week for the last 5 years. Her amylase and lipase
are elevated, and a diagnosis of acute pancreatitis is made. Which one of the following does not
indicate a bad prognosis according to Ranson's criteria for Non-gallstone pancreatitis?
(or)
Ranson criteria are used to evaluate the severity of pancreatitis. Which one of the following does not
indicate a bad prognosis according to Ranson's criteria for Non-gallstone pancreatitis?
A. TLC >16,000/mm3
B. Calcium <8 mg/dL
C. Blood Urea Nitrogen elevation > 5 mg/dL
D. Base deficit < 4 mEq/L
----------------------------------------
5. A 33-year-old chronic alcoholic female presented with complaints of pain in the epigastric region and
multiple episodes of vomiting. The patient was admitted and diagnosed with chronic pancreatitis after
an extensive evaluation. The patient had severe abdominal pain, which did not resolve with medical
therapy. Which of the following Nerve block helps relieve pain in this patient?
(or)
To get better pain relief in the case of chronic pancreatitis, which nerve should be blocked?
A. Celiac ganglia
B. Vagus nerve
C. Anterolateral column of spinal cord
D. None of the above
----------------------------------------
6. A 30-year-old man is diagnosed with carcinoma of the head of the pancreas. The doctors decide to
resect the tumour by the procedure shown in the image below. What is the most common complication
of the procedure shown in the image?
(or)
What is the most common complication of the procedure shown in the image used to resect carcinoma
of the head of the pancreas in a 30-year-old man?

Page 2

414
A. Delayed gastric emptying
B. Bleeding
C. Exocrine insufficiency
D. Anastomotic leak
----------------------------------------
7. A 60-year-old man comes to the clinic complaining of epigastric pain and stools that have had
difficulty flushing for the last 1 month. He has a history of alcohol abuse and 50 pack-year smoking
history. Upon diagnosis the doctors decide to use the TIGAR-O Classification for the case. For which of
the following conditions is TIGAR-O Classification used?
(or)
For which of the following conditions is TIGAR-O Classification used?
A. Acute pancreatitis
B. Chronic pancreatitis
C. Pancreatic cancer
D. Pancreatic injuries
----------------------------------------
8. A 23-year-old patient comes to the emergency department due to pain in the epigastric region for the
past 3 hours. The amylase and lipase levels of the patient are increased. He is diagnosed with acute
pancreatitis, and medical management is started. When his condition improves, an MRCP is done to
evaluate the cause of acute pancreatitis. The MRCP image is given below. Which of the following is the
most likely diagnosis?
(or)
The MRCP image of a patient is given below. Which of the following is the most likely diagnosis?

A. Pancreatic divisum
B. Annular pancreas
C. Ectopic pancreas
D. Duplication anamoly
----------------------------------------
9. Which is the most common primary cancer leading to pancreatic metastasis?
(or)

Page 3

415
A 35-year-old patient comes to the clinic because of pain in the abdomen for the last 1 month. A CT
scan of the patient reveals metastatic cancer that has reached the pancreas. Which is the most
common primary cancer leading to pancreatic metastasis?
A. Kidney
B. Breast
C. Ovary
D. Stomach
----------------------------------------
10. Which part of pancreatic cancer has best prognosis?
(or)
A 72-year-old man presents to his primary care physician with generalized fatigue and mild abdominal
pain. He has had a 25-pound weight loss over 2 months. He has a past medical history of alcohol use
disorder complicated by chronic pancreatitis. He also has been smoking one pack of cigarettes per day
for the past 40 years. A CT scan reveals pancreatic cancer with no indication of its location within the
pancreas; which location has the best prognosis?
A. Head
B. Tail
C. Body
D. Periampullary
----------------------------------------
11. Which of the following is the most common complication after ERCP?
(or)
A 36-year-old woman comes to the clinic complaining of pain in the right upper quadrant of abdomen
for the last 5 days. The pain especially increases after meals. Ultrasound reveals gallstones in the bile
duct. ERCP done and the gallstones are removed. Which of the following is the most common
complication after ERCP?
A. Acute pancreatitis
B. Acute cholangitis
C. Acute cholecystitis
D. Duodenal perforation
----------------------------------------
12. Diagnostic investigation of choice in carcinoma pancreas is :
(or)
Diagnostic investigation of choice in carcinoma pancreas is:
A. MDCT
B. PET scan
C. ERCP
D. MRCP

Page 4

416
----------------------------------------
13. A 4-year-old girl falls on the playground and is rushed to the emergency room. Her workup reveals
a fracture in her right tibia with multiple old fractures in her bilateral arms and legs at various stages of
healing. On examination, her sclera is blue. She is diagnosed with osteogenesis imperfecta type 1.
Which vitamin is required for the synthesis of type I collagen?
(or)
Synthesis of type I collagen requires which vitamin?
A. Vitamin A
B. Vitamin C
C. Vitamin D
D. Vitamin K
----------------------------------------
14. A 54-year-old woman presents to her primary care physician complaining of recurrent diaphoresis,
palpitations, and blurry vision episodes. The patient has another episode while in the clinic. The blood
pressure is 148/90 mmHg, the pulse rate is 110/min, and the respiration rate is 20/min. Her serum
glucose level is 50 mg/dL. Her symptoms are relieved after the administration of glucose. A week later,
the remainder of the lab results returned, revealing a high insulin level and high C-peptide. Which of the
following is the most likely diagnosis?
(or)
A 54-year-old woman presents complaining of diaphoresis, palpitations, and blurry vision episodes. Her
symptoms are relieved after the administration of glucose. A week later, the labs revealed a high insulin
level and high C-peptide. Which of the following is the diagnosis?
A. Insulinoma
B. Glucagonoma
C. Exogenous use of insulin
D. Somatostatinoma
----------------------------------------
15. A 48-year-old man with a history of stage IV pancreatic adenocarcinoma was referred for a second
opinion for a worsening dry cough in the setting of cavitary ground-glass opacities and non-cavitating
nodules found on chest CT. The previous workup was non-diagnostic and his CT findings were atypical
for pulmonary metastasis. Due to his extensive alternative therapy use, he was diagnosed with
interstitial lung disease (ILD). Flexible bronchoscopy with transbronchial cryo biopsies revealed
adenocarcinoma with intestinal/pancreaticobiliary differentiation consistent with metastatic pancreatic
cancer. The most common gene mutation that you would expect in a case of pancreatic
adenocarcinoma is :
(or)
The most common gene mutation that you would expect in a case of pancreatic adenocarcinoma is
A. K-Ras
B. p16
C. p53
D. BRAF

Page 5

417
----------------------------------------
16. A healthy 18-year-old girl presented to a local emergency room with 48hr of abdominal pain and
vomiting. On physical examination. there is tenderness over the left lower abdominal quadrant with
guarding. Based on the radiological and biochemical reports, a diagnosis of severe acute pancreatitis
was made. All the following can be used to predict severe acute pancreatitis except :
(or)
All the following can be used to predict severe acute pancreatitis except :
A. Glasgow score ≥ 3
B. APACHE II score ≥ 8
C. CT severity score ≥ 7
D. C-reactive protein <100
----------------------------------------
17. The patient was diagnosed with Unresectable Locally Advanced Pancreatic cancer. Which of the
following drug can be prescribed to the patient to increase his survival rate?
(or)
A 52-year-old man presented to us with c/o epigastric pain and weight loss. He also gave a history of
dyspepsia and progressively increasing jaundice with clay-colored stools with no history of fever or
pruritus. On physical examination, there was evidence for icterus without any palpable neck
lymphadenopathy. Abdominal and chest CT scan confirmed the presence of a tumor measuring 50 mm
in the head of the pancreas. Endoscopic ultrasonography (EUS) also indicated that the tumor was in
contact with both the SMA and the SMV, and fine-needle aspiration biopsy revealed adenocarcinoma.
On the basis of these clinical findings, the patient was diagnosed with Unresectable Locally Advanced
Pancreatic cancer. Which of the following drug can be prescribed to the patient to increase his survival
rate?
A. Doxorubicin
B. Streptozocin
C. Gemcitabine
D. Paclitaxel
----------------------------------------
18. A 30-year-old man came to the emergency department after a domestic violence episode an hour
ago. He has signs of penetrating knife injuries to the epigastrium and chest. The FAST scan reveals
fluid in the Morrison pouch. His BP is 110/70 mmHg, and his pulse is 90/min. A CT scan is performed,
and it reveals a superficial laceration at the pancreatic head without duct injury. Which of the following
is the next best step in management?
(or)
What is the next best step in a patient with superficial laceration at the pancreatic head without duct
injury?
A. Observation
B. Drainage and repair
C. Explorative laparotomy
D. Distal pancreatectomy

Page 6

418
----------------------------------------
19. A 42-year-old male presented with recurrent episodes of pain and bloating of the upper abdomen
for 1½ years and stools that were difficult to flush for 6 months. The patient had suffered from an attack
of acute pancreatitis 5 years before. The abdominal examination revealed mild tenderness in the
epigastrium with no rebound tenderness, palpable lump, or swelling. His fasting blood glucose is 180
mg/dL. Serum amylase and lipase levels were high. Which of the following is the most likely diagnosis?
A. Chronic pancreatitis
B. Carcinoma of the head of the pancreas
C. Recurrent acute pancreatitis
D. Myocardial infarction
----------------------------------------
20. What is the preferred route for supplementary nutrition after Puestow's procedure?
(or)
A 42-year-old male presented with recurrent episodes of severe pain and bloating of the upper
abdomen for 1½ years and pain in the left loin for 1 week. The patient had suffered from an attack of
acute pancreatitis 5 years before. Serum amylase and lipase levels were high. A CECT abdomen was
suggestive of features of chronic calcific pancreatitis. The patient underwent Puestow's procedure as
medical treatment failed. What is the most preferred route for supplementary nutrition in this patient?
A. Total parenteral nutrition
B. Feeding gastrostomy
C. Feeding jejunostomy
D. Oral feeding
----------------------------------------
21. A 10-year-old girl presented with a 9-year history of early satiety and recurrent vomiting after a
meal. Her birth history was unremarkable, and she had been treated symptomatically with the clinical
diagnosis of chronic gastroenteritis for her recurrent vomiting. The abdominal ultrasound scan was
normal, but the upper gastrointestinal (UGI) study revealed a distended duodenal bulb accompanied by
a diffusely narrowed second portion of the duodenum. Abdominal CT demonstrated the following
finding shown in the image. What is your probable diagnosis?
(or)
The upper gastrointestinal (UGI) study in a patient revealed a distended duodenal bulb accompanied
by a diffusely narrowed second portion of the duodenum. Abdominal CT demonstrated the following
finding shown in the image. What is your probable diagnosis?

Page 7

419
A. Duodenal stenosis
B. Annular pancreas
C. Midgut volvulus
D. Pyloric stenosis
----------------------------------------
22. A 33-year-old woman comes to the emergency department complaining of pain in her abdomen for
the last 2 hours. On physical examination, she has guarding and tenderness in the epigastrium. Tests
reveal increased amylase and lipase. Her history reveals a drug that likely induced pancreatitis. Which
of the following drugs is not a cause of acute pancreatitis?
(or)
Which of the following drugs is not a cause of acute pancreatitis?
A. Azathioprine
B. Gentamicin
C. Metronidazole
D. L-asparaginase
----------------------------------------
23. A 60-year-old man presented to the outpatient department with abdominal pain for the past 3 hours.
He consumes 20 drinks of alcohol per week. Presently, he has severe vomiting. He also complains of
dizziness when standing. On examination revealed tenderness in the epigastrium radiating to the back.
A reddish discolouration was noted in the flanks. Which of the following is the investigation of choice?
(or)
A 60-year-old man presented with severe vomiting. Examination revealed tenderness in the
epigastrium radiating to the back. A reddish discolouration was noted in the flanks. Which of the
following is the investigation of choice?

Page 8

420
A. CT abdomen for appendicitis
B. USG
C. CT scan of the abdomen for necrotising pancreatitis
D. X-ray abdomen
----------------------------------------

Correct Answers
Question Correct Answer

Question 1 2
Question 2 2
Question 3 1
Question 4 4
Question 5 1
Question 6 1
Question 7 2
Question 8 1
Question 9 1
Question 10 4
Question 11 1
Question 12 1
Question 13 2
Question 14 1
Question 15 1
Question 16 4
Question 17 3
Question 18 1
Question 19 1

Page 9

421
Question 20 3
Question 21 2
Question 22 2
Question 23 3

Solution for Question 1:


Option B: Periampullary carcinoma
• This patient with obstructive jaundice symptoms and being a chronic smoker indicates a carcinoma of
the biliary pancreatic region.
• The patient has symptoms suggestive of obstructive jaundice which includes pruritus due to
deposition of bile salts in the skin which releases histamine, and silver-colored stools.
• Causes of intermittent jaundice include periampullary carcinoma and common bile duct stones.
• As the jaundice is painless and the CT scan shows dilated main pancreatic duct and common bile
duct, findings favour periampullary carcinoma. Jaundice - painful and intermittent- common bile duct
stones Painless and intermittent jaundice - periampullary carcinoma Painless and progressive jaundice-
carcinoma head of the pancreas
• Jaundice - painful and intermittent- common bile duct stones
• Painless and intermittent jaundice - periampullary carcinoma
• Painless and progressive jaundice- carcinoma head of the pancreas
• Periampullary cancers include tumours arising from the Distal bile duct Duodenal mucosa Pancreas
just adjacent to the ampulla
• Distal bile duct
• Duodenal mucosa
• Pancreas just adjacent to the ampulla
• Jaundice - painful and intermittent- common bile duct stones
• Painless and intermittent jaundice - periampullary carcinoma
• Painless and progressive jaundice- carcinoma head of the pancreas
• Distal bile duct
• Duodenal mucosa
• Pancreas just adjacent to the ampulla
Option A: Carcinoma of the head of the pancreas
• Carcinoma of the head of the pancreas will present with progressive jaundice that increases with time,
but this patient is experiencing intermittent jaundice, a feature of periampullary carcinoma. On CT scan,
mostly a mass will be seen in the head of the pancreas with metastasis toward the liver at the time of
the diagnosis.
Option C: Chronic pancreatitis
• Chronic pancreatitis will present with epigastric pain and steatorrhea, mostly in alcoholics. Chronic
pancreatitis cannot cause obstructive jaundice.
Option D: Hilar cholangiocarcinoma

Page 10

422
• Hilar cholangiocarcinoma presents with intermittent jaundice but will not present on CT with
pancreatic and common bile duct dilation.

Solution for Question 2:


Option B: 22 months
• Carcinoma of the head of the pancreas or in the tail does not have a good prognosis even after
surgery and adjuvant therapy.
• The symptoms of carcinoma of the head of the pancreas are progressive, painless jaundice, and
weight loss.
• The biggest risk factors are smoking and chronic pancreatitis.
• After surgical resection and adjuvant therapy for pancreatic cancer, the median survival rate is
approximately 22 months, with a 5-year survival of 15% to 20%.
• Most patients experience a relapse of the disease in the form of metastatic disease (85%) and, less
commonly, local recurrence (40%).
• In the absence of surgical resection, those with locally advanced disease who receive palliative
chemotherapy may survive 10 to 12 months, whereas those with metastases rarely survive beyond 6
months.
Option A: 12 months
• In the absence of surgical resection and the patient is only on palliative treatment, the median survival
time is 12 months.
Option C: 32 months
• The median survival time of carcinoma of the pancreas is 22 months as most patients, after even
surgical resection, can have a relapse of the disease in the form of metastasis.
Option D: 44 months
• The median survival time of pancreatic cancer, even after surgical resection and adjuvant therapy,
is 22 months.

Solution for Question 3:


Option A: Pancreatic cancer
• This patient with hyperpigmented macules on his lips points towards a diagnosis of a genetic disease,
the Peutz-Jeghers syndrome.
• Among the cancers given above, Peutz-Jeghers syndrome(STK 11 gene mutation has a maximum
risk of developing pancreatic ductal adenocarcinoma.
• Pancreatic cancer Pancreatic cancer is more common in older adults, with most patients being 75 to
84 years old. It is slightly more common in men than women. Risk factors include family history,
smoking, alcohol and caffeine consumption, high fat diet, and low in fibre, fruits and vegetables,
diabetes mellitus, chronic pancreatitis. About two-thirds of pancreatic adenocarcinomas arise within the
head or uncinate process of the pancreas; 15% are in the body, and 10% are in the tail, with the

Page 11

423
remaining tumours demonstrating diffuse involvement of the gland. Ductal adenocarcinoma makes
up 75% of nonendocrine cancers of the pancreas. Other less common types include adenosquamous
carcinoma, acinic cell carcinoma Genetics of pancreatic cancer The K-Ras oncogene is currently
thought to be the most commonly mutated gene in pancreatic cancer, with approximately 90% of
tumours having a mutation. The Her2/neu oncogene is overexpressed in pancreatic cancer. Multiple
tumour suppressor genes are deleted and mutated in pancreatic cancer, including p53, p16 and DPC 4
(Smad 4) and in a minority of cases, BRCA 2 Rare familial cancer syndromes that are associated with
an increased risk of pancreatic cancer include BRCA2, the familial atypical multiple mole melanoma
syndrome, hereditary pancreatitis, familial adenomatous polyposis (FAP), hereditary non-polyposis
colorectal cancer, Peutz-Jeghers syndrome and ataxia-telangiectasia
• Pancreatic cancer is more common in older adults, with most patients being 75 to 84 years old. It is
slightly more common in men than women.
• Risk factors include family history, smoking, alcohol and caffeine consumption, high fat diet, and low
in fibre, fruits and vegetables, diabetes mellitus, chronic pancreatitis.
• About two-thirds of pancreatic adenocarcinomas arise within the head or uncinate process of the
pancreas; 15% are in the body, and 10% are in the tail, with the remaining tumours demonstrating
diffuse involvement of the gland. Ductal adenocarcinoma makes up 75% of nonendocrine cancers of
the pancreas. Other less common types include adenosquamous carcinoma, acinic cell carcinoma
Genetics of pancreatic cancer The K-Ras oncogene is currently thought to be the most commonly
mutated gene in pancreatic cancer, with approximately 90% of tumours having a mutation. The
Her2/neu oncogene is overexpressed in pancreatic cancer. Multiple tumour suppressor genes are
deleted and mutated in pancreatic cancer, including p53, p16 and DPC 4 (Smad 4) and in a minority of
cases, BRCA 2 Rare familial cancer syndromes that are associated with an increased risk of pancreatic
cancer include BRCA2, the familial atypical multiple mole melanoma syndrome, hereditary pancreatitis,
familial adenomatous polyposis (FAP), hereditary non-polyposis colorectal cancer, Peutz-Jeghers
syndrome and ataxia-telangiectasia
• Ductal adenocarcinoma makes up 75% of nonendocrine cancers of the pancreas. Other less common
types include adenosquamous carcinoma, acinic cell carcinoma
• Genetics of pancreatic cancer The K-Ras oncogene is currently thought to be the most commonly
mutated gene in pancreatic cancer, with approximately 90% of tumours having a mutation. The
Her2/neu oncogene is overexpressed in pancreatic cancer. Multiple tumour suppressor genes are
deleted and mutated in pancreatic cancer, including p53, p16 and DPC 4 (Smad 4) and in a minority of
cases, BRCA 2 Rare familial cancer syndromes that are associated with an increased risk of pancreatic
cancer include BRCA2, the familial atypical multiple mole melanoma syndrome, hereditary pancreatitis,
familial adenomatous polyposis (FAP), hereditary non-polyposis colorectal cancer, Peutz-Jeghers
syndrome and ataxia-telangiectasia
• The K-Ras oncogene is currently thought to be the most commonly mutated gene in pancreatic
cancer, with approximately 90% of tumours having a mutation. The Her2/neu oncogene is
overexpressed in pancreatic cancer. Multiple tumour suppressor genes are deleted and mutated in
pancreatic cancer, including p53, p16 and DPC 4 (Smad 4) and in a minority of cases, BRCA 2
• Rare familial cancer syndromes that are associated with an increased risk of pancreatic cancer
include BRCA2, the familial atypical multiple mole melanoma syndrome, hereditary pancreatitis, familial
adenomatous polyposis (FAP), hereditary non-polyposis colorectal cancer, Peutz-Jeghers syndrome
and ataxia-telangiectasia
• Pancreatic cancer is more common in older adults, with most patients being 75 to 84 years old. It is
slightly more common in men than women.
• Risk factors include family history, smoking, alcohol and caffeine consumption, high fat diet, and low
in fibre, fruits and vegetables, diabetes mellitus, chronic pancreatitis.

Page 12

424
• About two-thirds of pancreatic adenocarcinomas arise within the head or uncinate process of the
pancreas; 15% are in the body, and 10% are in the tail, with the remaining tumours demonstrating
diffuse involvement of the gland. Ductal adenocarcinoma makes up 75% of nonendocrine cancers of
the pancreas. Other less common types include adenosquamous carcinoma, acinic cell carcinoma
Genetics of pancreatic cancer The K-Ras oncogene is currently thought to be the most commonly
mutated gene in pancreatic cancer, with approximately 90% of tumours having a mutation. The
Her2/neu oncogene is overexpressed in pancreatic cancer. Multiple tumour suppressor genes are
deleted and mutated in pancreatic cancer, including p53, p16 and DPC 4 (Smad 4) and in a minority of
cases, BRCA 2 Rare familial cancer syndromes that are associated with an increased risk of pancreatic
cancer include BRCA2, the familial atypical multiple mole melanoma syndrome, hereditary pancreatitis,
familial adenomatous polyposis (FAP), hereditary non-polyposis colorectal cancer, Peutz-Jeghers
syndrome and ataxia-telangiectasia
• Ductal adenocarcinoma makes up 75% of nonendocrine cancers of the pancreas. Other less common
types include adenosquamous carcinoma, acinic cell carcinoma
• Genetics of pancreatic cancer The K-Ras oncogene is currently thought to be the most commonly
mutated gene in pancreatic cancer, with approximately 90% of tumours having a mutation. The
Her2/neu oncogene is overexpressed in pancreatic cancer. Multiple tumour suppressor genes are
deleted and mutated in pancreatic cancer, including p53, p16 and DPC 4 (Smad 4) and in a minority of
cases, BRCA 2 Rare familial cancer syndromes that are associated with an increased risk of pancreatic
cancer include BRCA2, the familial atypical multiple mole melanoma syndrome, hereditary pancreatitis,
familial adenomatous polyposis (FAP), hereditary non-polyposis colorectal cancer, Peutz-Jeghers
syndrome and ataxia-telangiectasia
• The K-Ras oncogene is currently thought to be the most commonly mutated gene in pancreatic
cancer, with approximately 90% of tumours having a mutation. The Her2/neu oncogene is
overexpressed in pancreatic cancer. Multiple tumour suppressor genes are deleted and mutated in
pancreatic cancer, including p53, p16 and DPC 4 (Smad 4) and in a minority of cases, BRCA 2
• Rare familial cancer syndromes that are associated with an increased risk of pancreatic cancer
include BRCA2, the familial atypical multiple mole melanoma syndrome, hereditary pancreatitis, familial
adenomatous polyposis (FAP), hereditary non-polyposis colorectal cancer, Peutz-Jeghers syndrome
and ataxia-telangiectasia
• Ductal adenocarcinoma makes up 75% of nonendocrine cancers of the pancreas. Other less common
types include adenosquamous carcinoma, acinic cell carcinoma
• Genetics of pancreatic cancer The K-Ras oncogene is currently thought to be the most commonly
mutated gene in pancreatic cancer, with approximately 90% of tumours having a mutation. The
Her2/neu oncogene is overexpressed in pancreatic cancer. Multiple tumour suppressor genes are
deleted and mutated in pancreatic cancer, including p53, p16 and DPC 4 (Smad 4) and in a minority of
cases, BRCA 2 Rare familial cancer syndromes that are associated with an increased risk of pancreatic
cancer include BRCA2, the familial atypical multiple mole melanoma syndrome, hereditary pancreatitis,
familial adenomatous polyposis (FAP), hereditary non-polyposis colorectal cancer, Peutz-Jeghers
syndrome and ataxia-telangiectasia
• The K-Ras oncogene is currently thought to be the most commonly mutated gene in pancreatic
cancer, with approximately 90% of tumours having a mutation. The Her2/neu oncogene is
overexpressed in pancreatic cancer. Multiple tumour suppressor genes are deleted and mutated in
pancreatic cancer, including p53, p16 and DPC 4 (Smad 4) and in a minority of cases, BRCA 2
• Rare familial cancer syndromes that are associated with an increased risk of pancreatic cancer
include BRCA2, the familial atypical multiple mole melanoma syndrome, hereditary pancreatitis, familial
adenomatous polyposis (FAP), hereditary non-polyposis colorectal cancer, Peutz-Jeghers syndrome
and ataxia-telangiectasia

Page 13

425
• The K-Ras oncogene is currently thought to be the most commonly mutated gene in pancreatic
cancer, with approximately 90% of tumours having a mutation. The Her2/neu oncogene is
overexpressed in pancreatic cancer. Multiple tumour suppressor genes are deleted and mutated in
pancreatic cancer, including p53, p16 and DPC 4 (Smad 4) and in a minority of cases, BRCA 2
• Rare familial cancer syndromes that are associated with an increased risk of pancreatic cancer
include BRCA2, the familial atypical multiple mole melanoma syndrome, hereditary pancreatitis, familial
adenomatous polyposis (FAP), hereditary non-polyposis colorectal cancer, Peutz-Jeghers syndrome
and ataxia-telangiectasia
Option B: Ganglioma
• Ganglioma are associated with Multiple endocrine neoplasias 2B (MEN 2B), which consist of
Gangliomas, Pheochromocytomas, and medullary thyroid carcinoma.
Option C: Testicular cancer
• Testicular cancer risk can increase in Klinefelter syndrome.
Option D: Esophageal cancer
• Esophageal cancer risk is not increased with Peutz-jeghers syndrome. It is increased by smoking,
Barrett's esophagus, acid reflux, and alcohol.

Solution for Question 4:


Option D: Base deficit < 4 mEq/L
• The original Ranson criteria is a scoring system that uses 11 parameters to assess the severity of
acute pancreatitis.
• Five parameters are assessed on admission, and the other six are assessed 48 hours
post-admission. One point is given for each positive parameter for a maximum score of 11.
• A base deficit of >4mEq/L will indicate a bad prognosis
• RANSON'S Prognostic Criteria (for Non-Gall Stone Induced Pancreatitis)

Option A: TLC >16,000/mm3


• TLC >16,000/mm3 is indicative of a bad prognosis at presentation, and it gives a score of 1 in the
Ranson's criteria
Option B: Calcium <8 mg/dL
• Calcium <8 mg/dL indicates a bad prognosis according to Ranson's criteria. The calcium levels drop
due to the pericalcification of the pancreas after the episode of pancreatitis
Option C: Blood Urea Nitrogen elevation > 5 mg/dL
• BUN elevation > 5 mg/dL after 48 hours is a criterion of bad prognosis according to Ranson's criteria

Solution for Question 5:


Option A: Celiac ganglia

Page 14

426
• Celiac plexus blockade in chronic pancreatitis is used for pain relief
• Cause of pain in chronic pancreatitis: Inflammatory changes of pancreatic parenchyma with
intrapancreatic and peripancreatic neural alterations Ductal and intraparenchymal hypertension Altered
nociception of pain
• Inflammatory changes of pancreatic parenchyma with intrapancreatic and peripancreatic neural
alterations
• Ductal and intraparenchymal hypertension
• Altered nociception of pain
• Medical treatment options for pain Alcohol abstinence and diet Enzyme therapy Antioxidant therapy
Analgesics Upto 70% of patients seem to benefit from medical treatment
• Alcohol abstinence and diet
• Enzyme therapy
• Antioxidant therapy
• Analgesics Upto 70% of patients seem to benefit from medical treatment
• Upto 70% of patients seem to benefit from medical treatment
• Inflammatory changes of pancreatic parenchyma with intrapancreatic and peripancreatic neural
alterations
• Ductal and intraparenchymal hypertension
• Altered nociception of pain
• Alcohol abstinence and diet
• Enzyme therapy
• Antioxidant therapy
• Analgesics Upto 70% of patients seem to benefit from medical treatment
• Upto 70% of patients seem to benefit from medical treatment
• Upto 70% of patients seem to benefit from medical treatment
Option B: Vagus nerve
• Vagus nerve block can be used to block pain in vagal neuralgia and after thoracic surgeries to block
the cough.
Option C: Anterolateral column of spinal cord
• The anterolateral column of the spinal cord is where the spinothalamic tracts travel, responsible for
pain, temperature, and crude touch. We want a local neural block for pain relief in pancreatitis.
Option D: None of the above
• Celiac plexus block is used for the pain relief of chronic pancreatitis

Solution for Question 6:


Option A: Delayed gastric emptying

Page 15

427
• The most common complication of Whipple's procedure is Delayed gastric emptying.
• Whipple procedure — also known as a pancreaticoduodenectomy — is a complex operation to
remove the head of the pancreas, the first part of the duodenum, the gallbladder and the bile duct. The
Whipple procedure treats tumours and other disorders of the pancreas, intestine and bile duct.
• This procedure is associated with significant postoperative morbidity, rates of which range from 20%
to 50%
• Indications for Whipple procedure Cancer located at the head of the pancreas The pancreatic
neuroendocrine tumours (PNETs) The gastrointestinal stromal tumour (GIST) The intraductal papillary
mucinous neoplasms (IPMN) Periampullary cancer which includes distal bile duct cholangiocarcinoma
(DBDC) The Adenocarcinoma of the ampulla of Vater
• Cancer located at the head of the pancreas
• The pancreatic neuroendocrine tumours (PNETs)
• The gastrointestinal stromal tumour (GIST)
• The intraductal papillary mucinous neoplasms (IPMN)
• Periampullary cancer which includes distal bile duct cholangiocarcinoma (DBDC)
• The Adenocarcinoma of the ampulla of Vater
• Cancer located at the head of the pancreas
• The pancreatic neuroendocrine tumours (PNETs)
• The gastrointestinal stromal tumour (GIST)
• The intraductal papillary mucinous neoplasms (IPMN)
• Periampullary cancer which includes distal bile duct cholangiocarcinoma (DBDC)
• The Adenocarcinoma of the ampulla of Vater
complication:
Complications are-
• delayed gastric emptying (19%)
• pancreatic fistula (14%)
• infection (intra-abdominal abscess wound infection, cholangitis, pancreatitis , pneumonia)
• bile leak
• Mortality is 3%
Option B: Bleeding
• Bleeding mainly occurs from the stump of the gastroduodenal artery after a Whipple procedure, but it
is not the most common complication, as it occurs in only 10% of patients.
Option C: Exocrine insufficiency
• Exocrine insufficiency is common after the Whipple procedure, but its incidence is less than that of
delayed gastric emptying as the pancreatic tissue is mostly preserved except the head of the pancreas.
Option D: Anastomotic leak
• The anastomotic leak may present as a bile leak with increasing pain in the abdomen with LFTs
bilirubin increasing, but its incidence is less than that of gastric emptying.

Page 16

428
Solution for Question 7:
Option B: Chronic pancreatitis
• TIGAR-O Classification is used for chronic pancreatitis
• The Classification is useful for determining the etiology of chronic pancreatitis: T → Toxic –
Metabolic: Alcoholic Tobacco smoking Hypercalcemia Hyperlipidemia (rare and controversial) Chronic
renal failure Medications (Phenacetin abuse) Toxins (Organotin compounds) I → Idiopathic Early onset
Late-onset Tropical (Tropical calcific and fibrocalculous pancreatic diabetes) G → Genetic Cationic
trypsinogen (Codon 29 and 122 mutations) Autosomal recessive/modifier genes: CFTR
mutations, SPINK1 mutations, Cationic trypsinogen (codon 16, 22, 23 mutations), α1-Antitrypsin
deficiency (possible) A → Auto Immune Sjogren syndrome–associated chronic pancreatitis
Inflammatory bowel disease–associated chronic pancreatitis Primary biliary cirrhosis–associated
chronic pancreatitis R → Recurrent and severe Postnecrotic (severe acute pancreatitis) Recurrent
acute pancreatitis Vascular diseases/ischemic Radiation injury O → Obstructive Pancreatic divisum
Sphincter of Oddi disorders (controversial) Duct obstruction (e.g., tumour) Periampullary duodenal wall
cysts Post-traumatic pancreatic duct scars stenotic papilla choledochocele
• T → Toxic – Metabolic: Alcoholic Tobacco smoking Hypercalcemia Hyperlipidemia (rare and
controversial) Chronic renal failure Medications (Phenacetin abuse) Toxins (Organotin compounds)
• Alcoholic
• Tobacco smoking
• Hypercalcemia
• Hyperlipidemia (rare and controversial)
• Chronic renal failure
• Medications (Phenacetin abuse)
• Toxins (Organotin compounds)
• I → Idiopathic Early onset Late-onset Tropical (Tropical calcific and fibrocalculous pancreatic
diabetes)
• Early onset
• Late-onset
• Tropical (Tropical calcific and fibrocalculous pancreatic diabetes)
• G → Genetic Cationic trypsinogen (Codon 29 and 122 mutations) Autosomal recessive/modifier
genes: CFTR mutations, SPINK1 mutations, Cationic trypsinogen (codon 16, 22, 23
mutations), α1-Antitrypsin deficiency (possible)
• Cationic trypsinogen (Codon 29 and 122 mutations)
• Autosomal recessive/modifier genes: CFTR mutations, SPINK1 mutations, Cationic trypsinogen
(codon 16, 22, 23 mutations), α1-Antitrypsin deficiency (possible)
• A → Auto Immune Sjogren syndrome–associated chronic pancreatitis Inflammatory bowel
disease–associated chronic pancreatitis Primary biliary cirrhosis–associated chronic pancreatitis
• Sjogren syndrome–associated chronic pancreatitis
• Inflammatory bowel disease–associated chronic pancreatitis

Page 17

429
• Primary biliary cirrhosis–associated chronic pancreatitis
• R → Recurrent and severe Postnecrotic (severe acute pancreatitis) Recurrent acute pancreatitis
Vascular diseases/ischemic Radiation injury
• Postnecrotic (severe acute pancreatitis)
• Recurrent acute pancreatitis
• Vascular diseases/ischemic
• Radiation injury
• O → Obstructive Pancreatic divisum Sphincter of Oddi disorders (controversial) Duct obstruction (e.g.,
tumour) Periampullary duodenal wall cysts Post-traumatic pancreatic duct scars stenotic papilla
choledochocele
• Pancreatic divisum
• Sphincter of Oddi disorders (controversial)
• Duct obstruction (e.g., tumour)
• Periampullary duodenal wall cysts
• Post-traumatic pancreatic duct scars
• stenotic papilla
• choledochocele
• T → Toxic – Metabolic: Alcoholic Tobacco smoking Hypercalcemia Hyperlipidemia (rare and
controversial) Chronic renal failure Medications (Phenacetin abuse) Toxins (Organotin compounds)
• Alcoholic
• Tobacco smoking
• Hypercalcemia
• Hyperlipidemia (rare and controversial)
• Chronic renal failure
• Medications (Phenacetin abuse)
• Toxins (Organotin compounds)
• I → Idiopathic Early onset Late-onset Tropical (Tropical calcific and fibrocalculous pancreatic
diabetes)
• Early onset
• Late-onset
• Tropical (Tropical calcific and fibrocalculous pancreatic diabetes)
• G → Genetic Cationic trypsinogen (Codon 29 and 122 mutations) Autosomal recessive/modifier
genes: CFTR mutations, SPINK1 mutations, Cationic trypsinogen (codon 16, 22, 23
mutations), α1-Antitrypsin deficiency (possible)
• Cationic trypsinogen (Codon 29 and 122 mutations)
• Autosomal recessive/modifier genes: CFTR mutations, SPINK1 mutations, Cationic trypsinogen
(codon 16, 22, 23 mutations), α1-Antitrypsin deficiency (possible)

Page 18

430
• A → Auto Immune Sjogren syndrome–associated chronic pancreatitis Inflammatory bowel
disease–associated chronic pancreatitis Primary biliary cirrhosis–associated chronic pancreatitis
• Sjogren syndrome–associated chronic pancreatitis
• Inflammatory bowel disease–associated chronic pancreatitis
• Primary biliary cirrhosis–associated chronic pancreatitis
• R → Recurrent and severe Postnecrotic (severe acute pancreatitis) Recurrent acute pancreatitis
Vascular diseases/ischemic Radiation injury
• Postnecrotic (severe acute pancreatitis)
• Recurrent acute pancreatitis
• Vascular diseases/ischemic
• Radiation injury
• O → Obstructive Pancreatic divisum Sphincter of Oddi disorders (controversial) Duct obstruction (e.g.,
tumour) Periampullary duodenal wall cysts Post-traumatic pancreatic duct scars stenotic papilla
choledochocele
• Pancreatic divisum
• Sphincter of Oddi disorders (controversial)
• Duct obstruction (e.g., tumour)
• Periampullary duodenal wall cysts
• Post-traumatic pancreatic duct scars
• stenotic papilla
• choledochocele
• Alcoholic
• Tobacco smoking
• Hypercalcemia
• Hyperlipidemia (rare and controversial)
• Chronic renal failure
• Medications (Phenacetin abuse)
• Toxins (Organotin compounds)
• Early onset
• Late-onset
• Tropical (Tropical calcific and fibrocalculous pancreatic diabetes)
• Cationic trypsinogen (Codon 29 and 122 mutations)
• Autosomal recessive/modifier genes: CFTR mutations, SPINK1 mutations, Cationic trypsinogen
(codon 16, 22, 23 mutations), α1-Antitrypsin deficiency (possible)
• Sjogren syndrome–associated chronic pancreatitis
• Inflammatory bowel disease–associated chronic pancreatitis
• Primary biliary cirrhosis–associated chronic pancreatitis

Page 19

431
• Postnecrotic (severe acute pancreatitis)
• Recurrent acute pancreatitis
• Vascular diseases/ischemic
• Radiation injury
• Pancreatic divisum
• Sphincter of Oddi disorders (controversial)
• Duct obstruction (e.g., tumour)
• Periampullary duodenal wall cysts
• Post-traumatic pancreatic duct scars
• stenotic papilla
• choledochocele
Option A: Acute pancreatitis
• GET SMASHED is the mnemonic used to classify the etiology of acute pancreatitis
Option C: Pancreatic cancer
• The etiology of pancreatic cancer is smoking (most common), chronic pancreatitis, hereditary causes,
and genetics.
Option D: Pancreatic injuries
• Pancreatic injuries are mainly due to blunt injuries and penetrating injuries to the epigastric region of
the abdomen.

Solution for Question 8:


Option A: Pancreatic divisum
• The above image is an MRCP picture of pancreas divisum

Page 20

432
• The pancreas is formed by the fusion of the dorsal and ventral bud
• Failure of the dorsal and ventral ducts to fuse during embryogenesis leads to the pancreatic divisum,
a condition identified by a ventral pancreatic duct and common bile duct that enter the duodenum
through a major papilla, whereas a dorsal pancreatic duct enters through a minor papilla that is slightly
proximal.
• Most pancreatic exocrine secretions exit through the dorsal duct, and pancreas divisum can lead to a
condition of partial obstruction caused by a small minor papilla, leading to chronic back pressure in the
duct.
• It predisposes the pancreas to recurrent acute and chronic pancreatitis (as a large volume of
secretions flow through a narrow papilla)
• Diagnosis: IOC for diagnosis: MRCP Gold standard investigation: ERCP
• IOC for diagnosis: MRCP
• Gold standard investigation: ERCP
• Treatment - Endoscopic sphincterotomy and stenting through the lesser papilla results in temporary
relief of symptoms. Surgical options include sphincteroplasty, pancreaticojejunostomy, or pancreatic
head resection
• IOC for diagnosis: MRCP
• Gold standard investigation: ERCP

Page 21

433
(A) Indicates the dorsal pancreatic duct (the most central duct in the pancreatic divisum) as it crosses o
ver the common bile duct. There are obvious dilatations of this dorsal pancreatic duct, as shown in this
MRCP, consistent with chronic pancreatitis
(B) Indicates part of the dorsal duct, where it drains directly into the duodenum through the minor papill
a. The inherently small diameter of the minor papilla causes increased pressure in the dorsal pancreati
c duct
(C) Indicates the common bile duct, which is joined by the ventral pancreatic duct before draining into t
he duodenum through the major papilla.
(D) Indicates the short remnant or filamentous communication between the ventral and the dorsal duct
Option B: Annular pancreas
• The annular pancreas is an abnormal ring or collar of pancreatic tissue that encircles the duodenum

Option C: Ectopic pancreas

Page 22

434
• The ectopic pancreas is the pancreas found at an abnormal location. But in the MRCP image, you can
locate the gallbladder and pancreatic duct, so this pancreas is at the normal location.
Option D: Duplication anomaly
• The duplicated pancreas is a rare anomaly of the pancreas where a small tissue (body or tail) of the
pancreas is duplicated and situated near the original location of the pancreas. But in this MRCP image,
pancreatic divisum is clear, and there is no pancreatic duplication.

Solution for Question 9:


Option A: Kidney
• The most common primary cancer site resulting in an isolated pancreatic metastasis is Renal cell
carcinoma (RCC)
• Pancreatic metastases are asymptomatic in more than 50% of cases: they are often detected during
follow-up investigations after surgery for a primary lesion or as an incidental finding on imaging studies
performed for an unrelated condition.
• At a CT scan, pancreatic metastases may appear as hypervascular lesions, like in renal cell cancer
(RCC) metastases, or, as in the case of colon and melanoma metastases, as hypodense masses
Option B: Breast
• Breast carcinoma can metastasize to the pancreas, but the percentage of Breast carcinoma is 4% Out
of the total percentage of metastasis to the pancreas
Option C: Ovary
• Ovarian cancer is responsible for 4% of metastasis of primary cancer to the pancreas
Option D: Stomach
• Gastric cancers do not metastasize to the pancreas

Solution for Question 10:


Option D: Periampullary
• Periampullary carcinomas can be detected early and hence have the best prognosis
• Periampullary carcinoma includes: Adenocarcinoma of the head of the pancreas, Adenocarcinoma of
the ampulla of Vater, Distal bile duct adenocarcinoma, Duodenal Adenocarcinoma
• The most common periampullary carcinoma is pancreatic ductal Adenocarcinoma (has the worst
prognosis), followed by ampullary Adenocarcinoma, distal cholangiocarcinoma, and duodenal
Adenocarcinoma
• When compared to other periampullary malignancies, ampullary cancers tend to become symptomatic
at an earlier stage, and thus as many as 80% of these tumours are resectable.
• The 5-year survival rate after resection was 15% for pancreatic Adenocarcinoma, 27% for distal bile
duct tumours, 39% for ampullary Adenocarcinoma, and 59% for duodenal Adenocarcinoma. Median
and overall survival at 5 years is higher for patients with right-sided lesions in the pancreatic head when
compared to left-sided pancreatic cancers

Page 23

435
• Prognosis: Duodenal Adenocarcinoma > Ampullary carcinoma > Distal bile duct adenocarcinoma >
Head of pancreas > Body and tail of the pancreas
Option A: Head
• The head of the pancreas carcinoma has a 15 to 20% 5-year survival rate compared to 39% 5-year
survival of ampullary Adenocarcinoma and 59% for duodenal carcinoma, both periampullary
carcinomas
Option B: Tail
• Patients with pancreas Adenocarcinoma involving the body or tail of the gland are more likely to have
weight loss and abdominal pain as their initial complaints. These lesions can grow to a large size before
producing symptoms and are often diagnosed later with a poorprognosis. Most body and tail cancers
have already metastasized to distal sites or extended locally to involve nodes, nerves or major vessels
by the time of diagnosis
Option C: Body
• Pancreatic cancers involving the body of the pancreas become symptomatic at a very later stage
when they have already metastasized to different locations, making them unresectable at the time of
diagnosis

Solution for Question 11:


Option A: Acute pancreatitis
• The most common complication after ERCP is Acute pancreatitis
• ERCP (Endoscopic Retrograde Cholangiopancreatography)
• indication: 1. Malignancy-irregular filling defect. 2. Chronic pancreatitis-'chain-of-lakes'
appearance. 3. Congenital anomalies-Stones. 4. Stricture of biliary tree-Choledochal cyst. 5. For
sampling of biliary and pancreatic juices for analysis and cytology. 6. Brush biopsy from tumor site.
• 1. Malignancy-irregular filling defect.
• 2. Chronic pancreatitis-'chain-of-lakes' appearance. 3. Congenital anomalies-Stones. 4.
Stricture of biliary tree-Choledochal cyst. 5. For sampling of biliary and pancreatic juices for analysis
and cytology. 6. Brush biopsy from tumor site.
• 3. Congenital anomalies-Stones. 4. Stricture of biliary tree-Choledochal cyst. 5. For sampling of biliary
and pancreatic juices for analysis and cytology. 6. Brush biopsy from tumor site.
• Uses (diagnostic and therapeutic) Biopsy Brush cytology Placement of stents to relieve obstruction
Drain a pseudocyst Stone removal Diagnosis and staging of chronic pancreatitis
• Biopsy
• Brush cytology
• Placement of stents to relieve obstruction
• Drain a pseudocyst
• Stone removal
• Diagnosis and staging of chronic pancreatitis
• 1. Malignancy-irregular filling defect.

Page 24

436
• 2. Chronic pancreatitis-'chain-of-lakes' appearance. 3. Congenital anomalies-Stones. 4.
Stricture of biliary tree-Choledochal cyst. 5. For sampling of biliary and pancreatic juices for analysis
and cytology. 6. Brush biopsy from tumor site.
• 3. Congenital anomalies-Stones. 4. Stricture of biliary tree-Choledochal cyst. 5. For sampling of biliary
and pancreatic juices for analysis and cytology. 6. Brush biopsy from tumor site.
• 3. Congenital anomalies-Stones. 4. Stricture of biliary tree-Choledochal cyst. 5. For sampling of biliary
and pancreatic juices for analysis and cytology. 6. Brush biopsy from tumor site.
• Biopsy
• Brush cytology
• Placement of stents to relieve obstruction
• Drain a pseudocyst
• Stone removal
• Diagnosis and staging of chronic pancreatitis
Complications of ERCP:
• Pancreatitis (5%) - Most common complication post ERCP
Risk factors for post-ERCP pancreatitis
• Suspected Sphincter of Oddi dysfunction
• Young age
• Normal bilirubin
• Prior ERCP-related pancreatitis
• Difficult cannulation
• Pancreatic duct contrast injection
• Pancreatic sphincterotomy
• Balloon dilatation of the biliary sphincter
Hemorrhage
• Rare complication
• Most commonly seen with sphincterotomy
Infection
• A serious complication of ERCP
• Most common - Cholangitis, cholecystitis
• Cholangitis most commonly occurs when there is failed or incomplete biliary drainage
Perforation (0.5% to 2.1%)
• The most common type- is retroperitoneal perforation
• Associated with sphincterotomy
Option A: Acute cholangitis
• Acute Cholangitis most commonly occurs when there is incomplete biliary drainage during ERCP, but
acute pancreatitis is more common as a complication of ERCP

Page 25

437
Option C: Acute cholecystitis
• Acute cholecystitis is a rare complication of ERCP due to the seeding of enteric bacteria that enter by
a retrograde route to the gallbladder, causing acute cholecystitis
Option D: Duodenal perforation
ERCP-related perforations can occur during handling or manipulation of the endoscope, endoscopic sp
hincterotomy, or insertion of the guidewire. But it is a rare complication nowadays.

Solution for Question 12:


Multidetector computed tomography(MDCT)
Pancreatic cancer
• Pancreatic cancer is more common in older adults with most patients being 75 to 84 years old. It is
slightly more common in men than women.
Risk factors:
• family history, smoking, alcohol and coffee consumption, diet high in fat and low in fibre, fruits and
vegetables, diabetes mellitus, chronic pancreatitis
Region:
• About two thirds of pancreatic adenocarcinomas arise within the head or uncinate process of the
pancreas; 15% are in the body and 10% are in the tail with the remaining tumours demonstrating
diffuse involvement of the gland
• Ductal adenocarcinoma makes upto 75% of nonendocrine cancers of the pancreas. Other less
common types include adenosquamous carcinoma , acinic cell carcinoma
Genetics:
• The K-Ras oncogene is currently thought to be the most commonly mutated gene in pancreatic
cancer with approximately 90% of tumours having a mutation
CA 19-9 is a mucin associated carbohydrate antigen that can be detected in the serum of patients with
pancreatic cancer. Ultrasound is the first diagnostic imaging modality. The current diagnostic and stagi
ng test of choice for pancreatic cancer is a multidetector , dynamic , contrast enhanced CT scan .
The accuracy of CT scan to predict unresectable disease is about 90% to 95%
CT finding that indicate a
tumour is unresectable include involvement of more than or equal to 180° of the celiac axis , hepatic or
superior mesenteric artery, enlarged lymph nodes outside the boundary of resection, ascites and dista
nt metastasis. Invasion of the superior mesenteric vein or portal vein is not in itself a
contraindication to resection as long as the veins are patent

Solution for Question 13:


Option B: Vitamin C
• Basically, for collagen type I or II, vitamin C is required.

Page 26

438
• Collagen Synthesis requirements: Hydroxylation reaction for proline and lysine is done, and for both
these reactions, Vitamin C, Fe, Prolyl hydroxylase, and Lysyl hydroxylase are required Glycosylation
reaction (adding carbohydrate to it) Here, the enzyme required is Lysyl oxidase, and this oxidase
requires copper. So, for the synthesis of collagen, the following are required: 2 Metals - Fe, Cu 1
Vitamin -Vitamin C 3 Enzymes
• Hydroxylation reaction for proline and lysine is done, and for both these reactions, Vitamin C, Fe,
Prolyl hydroxylase, and Lysyl hydroxylase are required
• Glycosylation reaction (adding carbohydrate to it) Here, the enzyme required is Lysyl oxidase, and
this oxidase requires copper. So, for the synthesis of collagen, the following are required: 2 Metals - Fe,
Cu 1 Vitamin -Vitamin C 3 Enzymes
• So, for the synthesis of collagen, the following are required: 2 Metals - Fe, Cu 1 Vitamin -Vitamin C 3
Enzymes
• 2 Metals - Fe, Cu
• 1 Vitamin -Vitamin C
• 3 Enzymes
• Hydroxylation reaction for proline and lysine is done, and for both these reactions, Vitamin C, Fe,
Prolyl hydroxylase, and Lysyl hydroxylase are required
• Glycosylation reaction (adding carbohydrate to it) Here, the enzyme required is Lysyl oxidase, and
this oxidase requires copper. So, for the synthesis of collagen, the following are required: 2 Metals - Fe,
Cu 1 Vitamin -Vitamin C 3 Enzymes
• So, for the synthesis of collagen, the following are required: 2 Metals - Fe, Cu 1 Vitamin -Vitamin C 3
Enzymes
• 2 Metals - Fe, Cu
• 1 Vitamin -Vitamin C
• 3 Enzymes
• So, for the synthesis of collagen, the following are required: 2 Metals - Fe, Cu 1 Vitamin -Vitamin C 3
Enzymes
• 2 Metals - Fe, Cu
• 1 Vitamin -Vitamin C
• 3 Enzymes
• 2 Metals - Fe, Cu
• 1 Vitamin -Vitamin C
• 3 Enzymes
Option A: Vitamin A
• Vitamin A affects cell proliferation, differentiation, and functioning due to its influence on genome
expression at the level of regulation of the transcription of certain genes, post transcriptional
modification of mRNA levels.
Option C: Vitamin D
• Vitamin D is required for the absorption of Calcium into the blood from the duodenum. So it is required
for bone growth and bone cells differentiation
Option D: Vitamin K

Page 27

439
• Vitamin K is a cofactor for the synthesis of blood coagulation Factors II, VII, IX and X, and inhibitors
such as Protein C and S

Solution for Question 14:


Option A: Insulinoma
• According to the clinical scenario, the diagnosis is Insulinoma.
• Insulinomas are the most common functional pancreatic endocrine neoplasms arising from beta cells
of pancreas
• Clinical features: Characterized by Whipple's triad: Symptomatic fasting hypoglycemia Serum glucose
level <50 mg/dL Relief of symptoms with the administration of glucose Neuroglycopenic symptoms:
Dizziness, headache, confusion, coma Sympathetic overactivity leads to tachycardia, palpitation,
excessive sweating, and tremors Painless condition associated with weight gain.
• Characterized by Whipple's triad: Symptomatic fasting hypoglycemia Serum glucose level <50 mg/dL
Relief of symptoms with the administration of glucose
• Symptomatic fasting hypoglycemia
• Serum glucose level <50 mg/dL
• Relief of symptoms with the administration of glucose
• Neuroglycopenic symptoms: Dizziness, headache, confusion, coma
• Sympathetic overactivity leads to tachycardia, palpitation, excessive sweating, and tremors
• Painless condition associated with weight gain.
• Serum insulin and C peptide levels are elevated. C peptide level > 1.2 µg/ml with glucose level <40
mg% suggests insulinoma.
• Characterized by Whipple's triad: Symptomatic fasting hypoglycemia Serum glucose level <50 mg/dL
Relief of symptoms with the administration of glucose
• Symptomatic fasting hypoglycemia
• Serum glucose level <50 mg/dL
• Relief of symptoms with the administration of glucose
• Neuroglycopenic symptoms: Dizziness, headache, confusion, coma
• Sympathetic overactivity leads to tachycardia, palpitation, excessive sweating, and tremors
• Painless condition associated with weight gain.
• Symptomatic fasting hypoglycemia
• Serum glucose level <50 mg/dL
• Relief of symptoms with the administration of glucose
Option B: Glucagonoma
• Glucagonoma will present with necrolytic migratory erythema with new-onset diabetes in patients.
This patient is suffering from hyperinsulinemia and hypoglycemia
Option C: Exogenous use of insulin

Page 28

440
• In exogenous use of insulin, the patient will have increased insulin but decreased C peptide. As C
peptide is released along with insulin from the pancreas and is not present in exogenous insulin
Option D: Somatostatinoma
• Somatostatin inhibits pancreatic and biliary secretions, patients with a somatostatinoma present with
gallstones due to bile stasis, diabetes due to inhibition of insulin secretion, and steatorrhea due to
inhibition of pancreatic exocrine secretion and bile secretion.

Solution for Question 15:


Pancreatic cancer
• Pancreatic cancer is more common in older adults with most patients being 75 to 84 years old. It is
slightly more common in men than women. Risk factors include family history, smoking, alcohol and
coffee consumption, diet high in fat and low in fibre, fruits and vegetables, diabetes mellitus, chronic
pancreatitis.
• About two-thirds of pancreatic adenocarcinomas arise within the head or uncinate process of the
pancreas; 15% are in the body and 10% are in the tail with the remaining tumours demonstrating
diffuse involvement of the gland
• Ductal adenocarcinoma makes upto 75% of nonendocrine cancers of the pancreas. Other less
common types include adenosquamous carcinoma, acinic cell carcinoma
• Genetics of pancreatic cancer The K-Ras oncogene is currently thought to be the most commonly
mutated gene in pancreatic cancer with approximately 85% of tumours having a mutation. The
Her2/neu oncogene is overexpressed in pancreatic cancer. Multiple tumour suppressor genes are
deleted and/or mutated in pancreatic cancer, including p53, p16 and DPC 4(Smad 4) and in a minority
of cases , BRCA2 Rare familial cancer syndromes that are associated with an increased risk of
pancreatic cancer include BRCA2, Familial atypical multiple mole melanoma syndrome , Hereditary
pancreatitis, Familial adenomatous polyposis (FAP), Hereditary non polyposis colorectal cancer, Peutz
Jeghers syndrome and Ataxia telangiectasia.
• The K-Ras oncogene is currently thought to be the most commonly mutated gene in pancreatic
cancer with approximately 85% of tumours having a mutation. The Her2/neu oncogene is
overexpressed in pancreatic cancer. Multiple tumour suppressor genes are deleted and/or mutated in
pancreatic cancer, including p53, p16 and DPC 4(Smad 4) and in a minority of cases , BRCA2
• Rare familial cancer syndromes that are associated with an increased risk of pancreatic cancer
include BRCA2, Familial atypical multiple mole melanoma syndrome , Hereditary pancreatitis, Familial
adenomatous polyposis (FAP), Hereditary non polyposis colorectal cancer, Peutz Jeghers syndrome
and Ataxia telangiectasia.
• The K-Ras oncogene is currently thought to be the most commonly mutated gene in pancreatic
cancer with approximately 85% of tumours having a mutation. The Her2/neu oncogene is
overexpressed in pancreatic cancer. Multiple tumour suppressor genes are deleted and/or mutated in
pancreatic cancer, including p53, p16 and DPC 4(Smad 4) and in a minority of cases , BRCA2
• Rare familial cancer syndromes that are associated with an increased risk of pancreatic cancer
include BRCA2, Familial atypical multiple mole melanoma syndrome , Hereditary pancreatitis, Familial
adenomatous polyposis (FAP), Hereditary non polyposis colorectal cancer, Peutz Jeghers syndrome
and Ataxia telangiectasia.

Page 29

441
Solution for Question 16:
Tools for predicting the severity of Acute pancreatitis
At 48 hours
• Ranson scoring a system-A score of 3 or more indicates a severe attack. It predicts the severity of the
disease on the basis of 11 parameters obtained at the time of admission or 48 hours later. It has a low
positive predictive value of 50% and a high negative predictive value (90%)
• Glasgow scoring system-A score of 3 or more indicates the severe attack
• Serum C- Reactive protein levels - >150 mg/dL is an indicator of severity
At admission
• APACHE II - Based on the patient’s age, previous health status and 12 routine physiologic
measurements, APACHE II provides a general measure of the severity of disease. An APACHE II
score of 8 or higher defines severe pancreatitis. The main advantage is that it can be used upon
admission and repeated at any time. APACHE II has a positive predictive value of 43% and a negative
predictive value of 89%.
• BISAP (Bedside Index for Severity of Acute Pancreatitis )- Blood urea nitrogen (>25mg/dL), impaired
mental status(GCS <15), presence of systemic inflammatory response syndrome, age >60 years and
pleural effusion. A score of more than or equal to 3 is a predictor of severe acute pancreatitis
• Urea at admission - >60 mmoL/L
Balthazar’s CT severity index -A score of greater than or equal to 7 is a predictor of severe pancreatitis

Solution for Question 17:


• Gemcitabine is currently the standard of care for patients with metastatic pancreatic cancer
• Gemcitabine was approved by the U.S. Food and Drug Administration for use in pancreatic cancer in
1996.
• In patients with unresectable pancreatic cancer, gemcitabine results in symptomatic improvement,
improved pain control and performance status, and weight gain.
• Prior to gemcitabine, 5-fluorouracil was used as the standard treatment for unresectable pancreatic
cancer.

Solution for Question 18:


Option A: Observation
• This patient with a penetrating injury has a superficial laceration of the pancreatic head without duct
injury, which makes it a grade I pancreatic injury which is managed conservatively with strict
observation.
Hematoma
Laceration
Mild contusion without duct injury

Page 30

442
Superficial laceration without duct injury
Major contusion without duct injury
Major laceration without duct injury or tissue loss

Management:
• I, II - Non-operative management I – Observation II – Debridement, drainage, possible repair III –
Distal pancreatectomy, Roux-en-Y drainage ( Ductal injuries may be proximal or distal based on
whether it is to the right or left of superior mesenteric vessels (SMV)) Extended distal pancreatectomy-
if laceration is to the right of SMV
• I – Observation
• II – Debridement, drainage, possible repair
• III – Distal pancreatectomy, Roux-en-Y drainage ( Ductal injuries may be proximal or distal based on
whether it is to the right or left of superior mesenteric vessels (SMV)) Extended distal pancreatectomy-
if laceration is to the right of SMV
• Extended distal pancreatectomy- if laceration is to the right of SMV
• IV and V - Damage control surgery Resection + Roux-en-Y drainage Pyloric exclusion ( pancreas
resected, pylorus sutured, gastrointestinal continuity established by gastrojejunostomy)
Pancreaticoduodenectomy ( For severe pancreaticoduodenal trauma). A staged Whipple procedure is
better
• Resection + Roux-en-Y drainage
• Pyloric exclusion ( pancreas resected, pylorus sutured, gastrointestinal continuity established by
gastrojejunostomy)
• Pancreaticoduodenectomy ( For severe pancreaticoduodenal trauma). A staged Whipple procedure is
better
• Complications of pancreatic injury include hemorrhage, abscess, fistula, pseudocyst, or false
aneurysm.
• I – Observation
• II – Debridement, drainage, possible repair
• III – Distal pancreatectomy, Roux-en-Y drainage ( Ductal injuries may be proximal or distal based on
whether it is to the right or left of superior mesenteric vessels (SMV)) Extended distal pancreatectomy-
if laceration is to the right of SMV
• Extended distal pancreatectomy- if laceration is to the right of SMV
• Extended distal pancreatectomy- if laceration is to the right of SMV
• Resection + Roux-en-Y drainage
• Pyloric exclusion ( pancreas resected, pylorus sutured, gastrointestinal continuity established by
gastrojejunostomy)
• Pancreaticoduodenectomy ( For severe pancreaticoduodenal trauma). A staged Whipple procedure is
better
Option B: Drainage and repair
• Drainage and repair, although non-surgical procedures are useful in Grade II injuries where there is a
major contusion or laceration without duct injury. This patient has Grade I with only a superficial

Page 31

443
laceration
Option C: Explorative laparotomy
• Explorative laparotomy is necessary when the vitals of the patient are decreasing and the patient is
unstable; this patient had normal vitals, so observation is the best possible step here
Option D: Distal pancreatectomy
• Distal pancreatectomy is necessary for grade III pancreatic injury in which there is an injury to the
distal part of the pancreatic duct, but in this patient, the duct was intact

Solution for Question 19:


Option A: Chronic pancreatitis
• This patient, with episodes of epigastric pain for the last 1 year and steatorrhea for the last 6 months,
with fasting blood glucose raised points towards chronic pancreatitis with exocrine and endocrine
insufficiency
• Chronic pancreatitis: The most common cause of chronic pancreatitis: Alcohol The histologic hallmark
of chronic pancreatitis - persistent inflammation and irreversible fibrosis associated with atrophy of the
pancreatic parenchyma Associated with irreversible exocrine and endocrine insufficiency Primary
manifestation - Pain At least 90% of the gland needs to be dysfunctional for malabsorption symptoms to
occur Exocrine insufficiency occurs in 80% to 90% of patients with long-standing chronic pancreatitis
Diabetes mellitus - is seen in 40% to 80% of patients Jaundice or cholangitis - due to fibrosis of the
common bile duct Exocrine dysfunction: Diarrhoea, asthenia, loss of weightand appetite, steatorrhoea
(signifies severe pancreatic insufficiency) (90%), malabsorption. Endocrine dysfunction: Diabetes
mellitus. Pancreatic diabetesmay often be typically brittle because of concomitantglucagon deficiency
and requires insulin. Diagnosis Findings in C.T. abdomen Dilated pancreatic duct (68%) Parenchymal
atrophy (54%) Pancreatic calcification (50%) ERCP - Gold standard for the diagnosis of chronic
pancreatitis. But now, it is used only for therapeutic purposes Shows chain of lakes appearance of the
main pancreatic duct with intermittent points of obstruction in a dilated pancreatic duct Rosemont
criteria - Criteria to diagnose chronic pancreatitis by endoscopic ultrasound Investigation of choice -
MRCP Treatment Lifestyle modification Medical management To palliate symptoms as the disease is
irreversible Analgesics ( NSAIDs are the first line of treatment) Enzyme replacement therapy Celiac
neurolysis Primary modality for treating symptomatic pancreatic duct obstruction - ERCP- dilation and
polyethene stent placement Patient with pain and pancreatic duct dilation secondary to stones -
Endoscopic stone extraction Large impacted stones - Extracorporeal shock wave lithotripsy followed by
therapeutic ERCP Surgery Reserved for otherwise unmanageable symptomatic chronic pancreatitis
• The most common cause of chronic pancreatitis: Alcohol
• The histologic hallmark of chronic pancreatitis - persistent inflammation and irreversible fibrosis
associated with atrophy of the pancreatic parenchyma
• Associated with irreversible exocrine and endocrine insufficiency
• Primary manifestation - Pain
• At least 90% of the gland needs to be dysfunctional for malabsorption symptoms to occur
• Exocrine insufficiency occurs in 80% to 90% of patients with long-standing chronic pancreatitis
• Diabetes mellitus - is seen in 40% to 80% of patients
• Jaundice or cholangitis - due to fibrosis of the common bile duct

Page 32

444
• Exocrine dysfunction: Diarrhoea, asthenia, loss of weightand appetite, steatorrhoea (signifies severe
pancreatic insufficiency) (90%), malabsorption.
• Endocrine dysfunction: Diabetes mellitus. Pancreatic diabetesmay often be typically brittle because of
concomitantglucagon deficiency and requires insulin.
• Diagnosis Findings in C.T. abdomen Dilated pancreatic duct (68%) Parenchymal atrophy (54%)
Pancreatic calcification (50%) ERCP - Gold standard for the diagnosis of chronic pancreatitis. But now,
it is used only for therapeutic purposes Shows chain of lakes appearance of the main pancreatic duct
with intermittent points of obstruction in a dilated pancreatic duct Rosemont criteria - Criteria to
diagnose chronic pancreatitis by endoscopic ultrasound Investigation of choice - MRCP
• Findings in C.T. abdomen Dilated pancreatic duct (68%) Parenchymal atrophy (54%) Pancreatic
calcification (50%)
• Dilated pancreatic duct (68%)
• Parenchymal atrophy (54%)
• Pancreatic calcification (50%)
• ERCP - Gold standard for the diagnosis of chronic pancreatitis. But now, it is used only for therapeutic
purposes
• Shows chain of lakes appearance of the main pancreatic duct with intermittent points of obstruction in
a dilated pancreatic duct
• Rosemont criteria - Criteria to diagnose chronic pancreatitis by endoscopic ultrasound
• Investigation of choice - MRCP
• Treatment Lifestyle modification Medical management To palliate symptoms as the disease is
irreversible Analgesics ( NSAIDs are the first line of treatment) Enzyme replacement therapy Celiac
neurolysis Primary modality for treating symptomatic pancreatic duct obstruction - ERCP- dilation and
polyethene stent placement Patient with pain and pancreatic duct dilation secondary to stones -
Endoscopic stone extraction Large impacted stones - Extracorporeal shock wave lithotripsy followed by
therapeutic ERCP Surgery Reserved for otherwise unmanageable symptomatic chronic pancreatitis
• Lifestyle modification
• Medical management To palliate symptoms as the disease is irreversible Analgesics ( NSAIDs are the
first line of treatment) Enzyme replacement therapy Celiac neurolysis Primary modality for treating
symptomatic pancreatic duct obstruction - ERCP- dilation and polyethene stent placement Patient with
pain and pancreatic duct dilation secondary to stones - Endoscopic stone extraction Large impacted
stones - Extracorporeal shock wave lithotripsy followed by therapeutic ERCP Surgery Reserved for
otherwise unmanageable symptomatic chronic pancreatitis
• To palliate symptoms as the disease is irreversible
• Analgesics ( NSAIDs are the first line of treatment)
• Enzyme replacement therapy
• Celiac neurolysis
• Primary modality for treating symptomatic pancreatic duct obstruction - ERCP- dilation and
polyethene stent placement
• Patient with pain and pancreatic duct dilation secondary to stones - Endoscopic stone extraction
• Large impacted stones - Extracorporeal shock wave lithotripsy followed by therapeutic ERCP
• Surgery Reserved for otherwise unmanageable symptomatic chronic pancreatitis

Page 33

445
• The most common cause of chronic pancreatitis: Alcohol
• The histologic hallmark of chronic pancreatitis - persistent inflammation and irreversible fibrosis
associated with atrophy of the pancreatic parenchyma
• Associated with irreversible exocrine and endocrine insufficiency
• Primary manifestation - Pain
• At least 90% of the gland needs to be dysfunctional for malabsorption symptoms to occur
• Exocrine insufficiency occurs in 80% to 90% of patients with long-standing chronic pancreatitis
• Diabetes mellitus - is seen in 40% to 80% of patients
• Jaundice or cholangitis - due to fibrosis of the common bile duct
• Exocrine dysfunction: Diarrhoea, asthenia, loss of weightand appetite, steatorrhoea (signifies severe
pancreatic insufficiency) (90%), malabsorption.
• Endocrine dysfunction: Diabetes mellitus. Pancreatic diabetesmay often be typically brittle because of
concomitantglucagon deficiency and requires insulin.
• Diagnosis Findings in C.T. abdomen Dilated pancreatic duct (68%) Parenchymal atrophy (54%)
Pancreatic calcification (50%) ERCP - Gold standard for the diagnosis of chronic pancreatitis. But now,
it is used only for therapeutic purposes Shows chain of lakes appearance of the main pancreatic duct
with intermittent points of obstruction in a dilated pancreatic duct Rosemont criteria - Criteria to
diagnose chronic pancreatitis by endoscopic ultrasound Investigation of choice - MRCP
• Findings in C.T. abdomen Dilated pancreatic duct (68%) Parenchymal atrophy (54%) Pancreatic
calcification (50%)
• Dilated pancreatic duct (68%)
• Parenchymal atrophy (54%)
• Pancreatic calcification (50%)
• ERCP - Gold standard for the diagnosis of chronic pancreatitis. But now, it is used only for therapeutic
purposes
• Shows chain of lakes appearance of the main pancreatic duct with intermittent points of obstruction in
a dilated pancreatic duct
• Rosemont criteria - Criteria to diagnose chronic pancreatitis by endoscopic ultrasound
• Investigation of choice - MRCP
• Treatment Lifestyle modification Medical management To palliate symptoms as the disease is
irreversible Analgesics ( NSAIDs are the first line of treatment) Enzyme replacement therapy Celiac
neurolysis Primary modality for treating symptomatic pancreatic duct obstruction - ERCP- dilation and
polyethene stent placement Patient with pain and pancreatic duct dilation secondary to stones -
Endoscopic stone extraction Large impacted stones - Extracorporeal shock wave lithotripsy followed by
therapeutic ERCP Surgery Reserved for otherwise unmanageable symptomatic chronic pancreatitis
• Lifestyle modification
• Medical management To palliate symptoms as the disease is irreversible Analgesics ( NSAIDs are the
first line of treatment) Enzyme replacement therapy Celiac neurolysis Primary modality for treating
symptomatic pancreatic duct obstruction - ERCP- dilation and polyethene stent placement Patient with
pain and pancreatic duct dilation secondary to stones - Endoscopic stone extraction Large impacted
stones - Extracorporeal shock wave lithotripsy followed by therapeutic ERCP Surgery Reserved for
otherwise unmanageable symptomatic chronic pancreatitis

Page 34

446
• To palliate symptoms as the disease is irreversible
• Analgesics ( NSAIDs are the first line of treatment)
• Enzyme replacement therapy
• Celiac neurolysis
• Primary modality for treating symptomatic pancreatic duct obstruction - ERCP- dilation and
polyethene stent placement
• Patient with pain and pancreatic duct dilation secondary to stones - Endoscopic stone extraction
• Large impacted stones - Extracorporeal shock wave lithotripsy followed by therapeutic ERCP
• Surgery Reserved for otherwise unmanageable symptomatic chronic pancreatitis
• Findings in C.T. abdomen Dilated pancreatic duct (68%) Parenchymal atrophy (54%) Pancreatic
calcification (50%)
• Dilated pancreatic duct (68%)
• Parenchymal atrophy (54%)
• Pancreatic calcification (50%)
• ERCP - Gold standard for the diagnosis of chronic pancreatitis. But now, it is used only for therapeutic
purposes
• Shows chain of lakes appearance of the main pancreatic duct with intermittent points of obstruction in
a dilated pancreatic duct
• Rosemont criteria - Criteria to diagnose chronic pancreatitis by endoscopic ultrasound
• Investigation of choice - MRCP
• Dilated pancreatic duct (68%)
• Parenchymal atrophy (54%)
• Pancreatic calcification (50%)
• Lifestyle modification
• Medical management To palliate symptoms as the disease is irreversible Analgesics ( NSAIDs are the
first line of treatment) Enzyme replacement therapy Celiac neurolysis Primary modality for treating
symptomatic pancreatic duct obstruction - ERCP- dilation and polyethene stent placement Patient with
pain and pancreatic duct dilation secondary to stones - Endoscopic stone extraction Large impacted
stones - Extracorporeal shock wave lithotripsy followed by therapeutic ERCP Surgery Reserved for
otherwise unmanageable symptomatic chronic pancreatitis
• To palliate symptoms as the disease is irreversible
• Analgesics ( NSAIDs are the first line of treatment)
• Enzyme replacement therapy
• Celiac neurolysis
• Primary modality for treating symptomatic pancreatic duct obstruction - ERCP- dilation and
polyethene stent placement
• Patient with pain and pancreatic duct dilation secondary to stones - Endoscopic stone extraction
• Large impacted stones - Extracorporeal shock wave lithotripsy followed by therapeutic ERCP
• Surgery Reserved for otherwise unmanageable symptomatic chronic pancreatitis

Page 35

447
• To palliate symptoms as the disease is irreversible
• Analgesics ( NSAIDs are the first line of treatment)
• Enzyme replacement therapy
• Celiac neurolysis
• Primary modality for treating symptomatic pancreatic duct obstruction - ERCP- dilation and
polyethene stent placement
• Patient with pain and pancreatic duct dilation secondary to stones - Endoscopic stone extraction
• Large impacted stones - Extracorporeal shock wave lithotripsy followed by therapeutic ERCP
• Surgery Reserved for otherwise unmanageable symptomatic chronic pancreatitis
Option B: Carcinoma of the head of the pancreas
• Carcinoma of the head of the pancreas does not present with epigastric pain. It will present late in its
course when it starts to compress the bile duct and cause progressive, painless jaundice(obstructive
jaundice)
Option C: Recurrent acute pancreatitis
• If this had been recurrent acute pancreatitis, the patient would have had severe pain in the epigastric,
unlike in this patient, who only had mild tenderness. So many episodes of recurrent acute pancreatitis
would have caused serious complications in this patient, like perforation and pseudocyst by now which
are not present currently in this patient with no rebound tenderness
Option D: Myocardial infarction
• Myocardial infarction can present with epigastric pain in patients, but this patient with endocrine and
exocrine insufficiency and pain for the last 1 year with amylase and lipase increase will suggest chronic
pancreatitis

Solution for Question 20:


Option C: Feeding jejunostomy

Page 36

448
• This feeding tube passes through the anterior abdominal wall into the jejunum.
• A jejunostomy tube can be placed surgically or radiologically by extending through the pylorus into the
jejunum. Endoscopically a percutaneous endoscopic gastrojejunostomy (PEGJ) can be placed
• The benefits of a jejunostomy tube are that it decreases the risk of food and fluids passing into the
lungs; allows for early postoperative feeding
• Enteral nutrition is preferred over Total parenteral nutrition as it Maintains gut barrier integrity Limits
bacterial translocation Less expensive Fewer complications
• Maintains gut barrier integrity
• Limits bacterial translocation
• Less expensive
• Fewer complications
• Maintains gut barrier integrity
• Limits bacterial translocation
• Less expensive
• Fewer complications
Option A: Total parenteral nutrition
• Feeding jejunostomy is preferred over total parenteral nutrition because it has fewer complications
and is less expensive than total parenteral nutrition
Option B: Feeding gastrostomy
• Feeding jejunostomy has a lower incidence of complications, especially pulmonary aspiration, than
gastrostomy. Stamm jejunostomy should be used for enteral feeding in older patients and patients with
short life expectancies. In younger patients requiring lifelong enteral feeding, Roux-en-Y jejunostomy
should be used.
Option D: Oral feeding
• Oral feeding is not started right after surgery as it can cause serious complications like aspiration. You
need a feeding tube for decompression, and oral feeding is started after a certain time when the

Page 37

449
physicians evaluate that the patient is fit and the feeding tube can be removed

Solution for Question 21:


Option B: Annular pancreas
• The given image shows pancreatic parenchyma encircling the second part of the
duodenum(arrowheads); the most likely diagnosis is the Annular pancreas.
• Annular pancreas Failure of a complete rotation of the ventral pancreatic bud during the development
Around 2nd part of the duodenum, there is a ring formed by the ventral pancreas Vomiting is
non-bilious, as the obstruction is proximal to the ampulla
• Failure of a complete rotation of the ventral pancreatic bud during the development
• Around 2nd part of the duodenum, there is a ring formed by the ventral pancreas
• Vomiting is non-bilious, as the obstruction is proximal to the ampulla
• Failure of a complete rotation of the ventral pancreatic bud during the development
• Around 2nd part of the duodenum, there is a ring formed by the ventral pancreas
• Vomiting is non-bilious, as the obstruction is proximal to the ampulla

Option A: Duodenal stenosis


• Duodenal stenosis is the failure of canalization of the duodenum in the fetus and will present with
bilious vomiting and obstruction signs within a few weeks of birth
Option C: Midgut volvulus
• Midgut volvulus will cause extreme pain and bilious vomiting in a patient
Option D: Pyloric stenosis

Page 38

450
• Pyloric stenosis within a few weeks of birth in a patient with non-bilious vomiting and an olive-shaped
mass in the abdomen

Solution for Question 22:


Option B: Gentamicin
• Gentamicin belongs to the aminoglycosides family of antibiotics, and it does not cause Drug-induced
acute pancreatitis
• Medications cause up to 2% of drug induced acute pancreatitis
• The potential mechanism includes pancreatic duct constriction and accumulation of a metabolite of
the drug which causes it.
Option A:Azathioprine
• Azathioprine is defined as a definitive cause of drug-induced pancreatitis. It is used for many
conditions like inflammatory bowel disease and vasculitis
Option C: Metronidazole
• Metronidazole is one of the three antibiotics(the others are tetracycline and erythromycin) that have
many reports of drug-induced pancreatitis after a few days of treatment
Option D: L-asparaginase
• Acute pancreatitis is one of the most common reasons clinicians don't prescribe L-asparaginase
anymore. Acute pancreatitis can be caused within a few days of treatment in patients, and chronic
pancreatitis and diabetes are the long-term complications

Solution for Question 23:


Option C: CT scan of the abdomen for necrotising pancreatitis
• The history and clinical features of the patient (Abdominal tenderness, hypotension due to blood loss,
Grey Turner's sign) are suggestive of severe pancreatitis.
• Hence a contrast CT scan of the abdomen will reveal severe necrotising pancreatitis.
• The cardinal symptom of acute pancreatitis is epigastric or periumbilical pain (the nature of pain is
constant) that radiates to the back. Up to 90% of patients have nausea or vomiting that typically does
not relieve the pain
• Dehydration, poor skin turgor, tachycardia, hypotension and dry mucous membrane are commonly
seen in patients with acute pancreatitis. Severely dehydrated and older patients may also develop
mental status changes
• Rare physical examination findings include Reddish/bluish discolouration of the flanks is known as
Grey Turner's sign Another feature of pancreatic necrosis is a bluish discolouration around the
umbilicus, known as Cullen's sign Both are indicative of retroperitoneal bleeding associated with severe
pancreatitis. But they are not pathognomonic of pancreatitis
• Reddish/bluish discolouration of the flanks is known as Grey Turner's sign

Page 39

451
• Another feature of pancreatic necrosis is a bluish discolouration around the umbilicus, known as
Cullen's sign
• Both are indicative of retroperitoneal bleeding associated with severe pancreatitis.
• But they are not pathognomonic of pancreatitis
• Reddish/bluish discolouration of the flanks is known as Grey Turner's sign
• Another feature of pancreatic necrosis is a bluish discolouration around the umbilicus, known as
Cullen's sign
• Both are indicative of retroperitoneal bleeding associated with severe pancreatitis.
• But they are not pathognomonic of pancreatitis
Option A: CT abdomen for appendicitis
• Appendicitis patients will initially present with generalised abdominal pain, which will localise into the
right iliac fossa pain after a few hours. This patient had epigastric pain and reddish-blue discolouration
of the flanks suggestive of necrotising pancreatitis
Option B: USG
• Ultrasound of the abdomen does not give physicians a good picture of acute pancreatitis, which is
why it is not recommended. Ultrasound of the abdomen for acute pancreatitis is only done in pregnant
patients due to the threat of radiation exposure from a CT scan
Option D: X-ray abdomen
• X-ray of the abdomen is of no use in acute pancreatitis as the changes will not be visible on the x-ray.
Chronic pancreatitis calcifications can be visible on x-ray.

Page 40

452
Pancreas Part 2
1. A 43-year-old woman presents with complaints of abdominal discomfort and abdominal mass at the
epigastric region for the past 3 weeks. She had 1 episode of gallstone pancreatitis 5 weeks ago. Which
of the following is the next best step for the patient?
(or)
A 43-year-old woman presents to the clinic with complaints of abdominal discomfort for the past 3
weeks. She reports some dull pain around the epigastric region, especially after she eats. Her previous
medical history is notable for one episode of gallstone pancreatitis 5 weeks ago. A physical
examination demonstrates an abdominal mass and tenderness in the epigastric region. The X-ray was
unremarkable. Which of the following is the next best step in managing this patient?
A. Percutaneous drainage
B. Laparotomy
C. Cystogastrostomy
D. Conservative management
----------------------------------------
2. A 13-year-old boy is brought to the emergency department for recurrent severe abdominal pain. He
reports 10/10 stabbing, epigastric pain that has been intermittently happening for the past 2 months. He
also has constipation and stool that is difficult to flush. A laboratory study demonstrates mildly elevated
levels of amylase and lipase. His family history is unremarkable. The doctors diagnose it as a case of
Idiopathic chronic pancreatitis. Which of the following gene is involved in this condition?
(or)
Which of the following gene is involved in Idiopathic chronic pancreatitis?
A. CFTR
B. PRSS 1 gene
C. SPINK1 gene
D. p53
----------------------------------------
3. A chronic pancreatitis patient who is a chronic alcoholic presents with abdominal pain radiating to the
back that does not respond to analgesics. On evaluation, the pancreatic duct was dilated, and multiple
stones were noted in the pancreatic duct. ERCP was performed, but the patient remained symptomatic.
Which is the best treatment option for this patient?
A. Pancreatic tail resection
B. Pancreaticojejunostomy
C. Percutaneous removal of stone
D. Medical management
----------------------------------------
4. A 45-year-old patient diagnosed with chronic pancreatitis has come to the clinic with pain
unresponsive to analgesics for the past 1 month. On imaging, it is seen that the pancreatic head is >4
mm, and he is referred for surgery. What is the surgical procedure shown in the image given below?
(or)

453
Name the surgical procedure given below.

A. Beger's procedure
B. Frey's procedure
C. Puestow's procedure
D. Duval procedure
----------------------------------------
5. A 60-year-old female presents to the clinic with dull aching pain in the central abdomen. On
laboratory evaluation, amylase and carcinoembryonic antigen levels were decreased. The image of the
CECT abdomen and tumour cut section after surgery is given below. Which of the following statements
is most appropriate about this tumour?

A. Most commonly seen in males


B. Mostly malignant
C. Macrocystic
D. CECT shows peripheral calcified central scar
----------------------------------------
6. A 45-year-old female presented with diffuse abdominal pain. On laboratory investigation, amylase
was normal, but CEA was raised. CT scan showed a hypodense lesion in the pancreas, which had
internal septation and wall calcification. What is the most likely diagnosis?

Page 2

454
A. Serous cystadenoma
B. Mucinous cystadenoma
C. Intraductal papillary mucinous neoplasm
D. Acinic cell tumour
----------------------------------------
7. A planned diagnostic ERCP of a patient with abnormality in pancreas revealed the following
endoscopic finding in the image. Which of the following is most appropriate?
(or)
A 36-year-old male presents to the clinic with complaints of abdominal pain in the epigastric region. An
ultrasound revealed increased echogenicity in the pancreas. A planned diagnostic ERCP revealed the
following endoscopic finding in the image. Which of the following statements is most appropriate
regarding the diagnosis?

A. Seen in only small ducts


B. Higher incidence in males
C. Most common in the tail of the pancreas
D. Usually diagnosed by ERCP
----------------------------------------
8. A 45-year-old female is admitted to the ward for pancreatic cancer. Whipple’s procedure is planned
for her. During the surgery, what order of anastomosis will the surgeon follow?
(or)
In whipple's procedure, what is the order of the anastomosis followed?

Page 3

455
A. Pancreaticojejunostomy, gastrojejunostomy, hepaticojejunostomy
B. Hepaticojejunostomy, pancreaticojejunostomy, gastrojejunostomy
C. Gastrojejunostomy, pancreaticojejunostomy, hepaticojejunostomy
D. Pancreaticojejunostomy, hepaticojejunostomy, gastrojejunostomy
----------------------------------------
9. A 52-year-old man presented to the clinic complaining of abdominal pain, secretory diarrhoea, and
reflux. After further investigations, he was diagnosed with Zollinger-Ellison syndrome. Which of the
following statements is most appropriate regarding his diagnosis?
(or)
Which of the following statements is most appropriate regarding Zollinger-Ellison syndrome?
A. Mostly found in the Hesselbach triangle
B. Basal acid output >5 mEq/hour is diagnostic
C. Most common site is the duodenum
D. Lymphadenectomy improves survival
----------------------------------------
10. A 70-year-old woman was referred to your hospital with persistent subacute eczema affecting her
lower extremities and groin area associated with weight loss, glossitis, and angular stomatitis that had
been present for 12 months. She was treated with topical and oral steroids with no improvement. Her
medical history revealed a long-standing type 2 diabetes mellitus and recurrent episodes of deep-vein
thrombosis in her right leg despite anticoagulant therapy. An abdominal (CT) scan showed
hypervascularized tumour measuring 5 to 7 cm in the tail of her pancreas without evidence of
metastatic disease. What is your probable diagnosis with the given data?

A. Insulinoma
B. Glucagonoma
C. Gastrinoma
D. Somatostatinoma
----------------------------------------
11. A 37-year-old man with a history of pulmonary tuberculosis was referred to your hospital with
persistent watery diarrhoea, weight loss, and hypokalemia for 2 years. Diarrhoea did not stop with
fasting, and serum potassium had remained at 2.5 mmol/L despite multiple cycles of intravenous
potassium replacement. An abdominal CT showed a 5.9 × 3.4 cm lesion in the pancreatic tail,

Page 4

456
confirmed by MRI. The patient was scheduled for Surgery after fluid resuscitation and electrolyte
correction. Which drug should be prescribed for this patient preoperatively to reduce diarrhoea?
(or)
A 37-year-old man with a history of pulmonary tuberculosis was referred to your hospital with persistent
watery diarrhoea, weight loss, and hypokalemia of 2 years. An abdominal CT showed a 5·9 × 3·4 cm
lesion in the pancreatic tail. Which drug should be prescribed for this patient preoperatively to reduce
diarrhoea?
A. Loperamide
B. Octreotide
C. Bismuth subsalicylate
D. Diamode
----------------------------------------
12. A 22-year-old male presented to the emergency room with worsening abdominal pain after
sustaining a strong blow from an opponent's knee during a soccer game the day before. He deferred
medical attention at the time but developed severe pain over the next 24 hours and the inability to
tolerate food or water. On presentation, the patient was afebrile with normal vital signs but appeared in
moderate distress. His abdomen was soft and mildly distended with diffuse tenderness, most prominent
over the epigastrium, and localized rebound and guarding. Computed tomography of the abdomen and
pelvis demonstrated a transection at the pancreatic head with large amounts of high attenuation-free
fluid and no other injuries. What should be the procedure of choice for this patient?
(or)
A 22-year-old student presented with worsening abdominal pain after sustaining a strong blow.
Computed tomography of the abdomen and pelvis demonstrated a transection at the pancreatic head
with large amounts of high attenuation-free fluid and no other injuries. The procedure of choice for this
patient should be?
A. Roux-en-Y anastomosis
B. Pancreaticoduodenectomy
C. Lateral tube jejunostomy
D. Retrograde jejunostomy
----------------------------------------
13. A 73-year-old female with a history of atrial fibrillation and hypertension presented with persistent
nausea and vomiting associated with epigastric discomfort for the past 3 months. Laboratory
investigation revealed a slightly elevated AST and GGT. CEA was elevated at 11.16 ng/mL (0–3.0).
Abdominal ultrasound revealed a mixed solid and cystic mass (5.8 cm) possibly arising from the
pancreas. Subsequent CECT abdomen demonstrated a large pancreatic mass in the head with a
dilated distal pancreatic duct and two peritoneal nodes. In this patient, the cause for persistent nausea
and vomiting is probably due to which of the following?
(or)
A 73-year-old female presented with persistent nausea and vomiting associated with epigastric
discomfort. CT demonstrated a large pancreatic mass in the head with a dilated distal pancreatic duct
and two peritoneal nodes. Persistent nausea and vomiting are probably due to which of the following?
A. External compression of the duodenum
B. Portal vein infiltration

Page 5

457
C. Proliferative infiltration of tumour into the duodenum
D. Activation of chemoreceptors in the brain
----------------------------------------
14. A 70-year-old man who was a non-smoker presented to your hospital with a 3-month history of 15
kg weight loss and recent generalized itching and jaundice. Serum biochemistry and ultrasound of the
abdomen confirmed obstructive jaundice due to a mass in the head of the Pancreas. He underwent
ERCP with a 5 cm 10F plastic stent inserted into the common bile duct, followed by pylorus-sparing
pancreaticoduodenectomy two months after diagnosis, and recovered uneventfully. Macroscopically,
there was a 5.5 cm diameter, lobulated tumour within the head of the Pancreas extending to the
superior mesenteric vessel margin. According to AJCC 8th edition, the staging of pancreatic cancer, in
this case, would be?
(or)
A 70-year-old man presented with weight loss. Ultrasound confirmed obstructive jaundice due to a
mass in the pancreas. Macroscopically, there was a 5.5cm diameter, lobulated tumour within the head
of the pancreas extending to the superior mesenteric vessel margin. According to AJCC 8th edition, the
staging of pancreatic cancer, in this case, would be?
A. T1
B. T2
C. T3
D. T4
----------------------------------------
15. A 53-year-old man, a chronic alcoholic, presented with epigastric pain and abdominal distension.
He had made several visits to a local hospital within the past 6 months for a similar presentation. An
abdominal examination revealed marked generalized abdominal tenderness. CT scan of the abdomen
and pelvis showed decreased pancreatic bulk and small cystic area in uncinate process with adjacent
peripancreatic and retroperitoneal collection extending to bilateral pararenal space. Serum alkaline
phosphatase 248 IU/L, serum amylase 1301 IU/L, and lipase 1311 IU/L were elevated, while serum
calcium was decreased (1.5 mmol/l). Ascitic fluid amylase was elevated (2801 IU/L). Usually, ascites
can be controlled medically. Surgery is not done in which of the following conditions of pancreatic
ascites?
(or)
A 53-year-old man, a chronic alcoholic, presented with epigastric pain and abdominal distension.
Ascitic fluid amylase was elevated (2801 IU/L). Usually, ascites can be controlled medically. Surgery is
not done in which of the following conditions ofpancreatic ascites?
A. Symptomatic
B. Completely blocked duct in MRCP
C. Not responding to medical therapy
D. Leak from the stented duct
----------------------------------------
16. On admission to ICU, a 30-year-old patient with acute pancreatitis suddenly develops tachypnea
and respiratory distress for the last 30 minutes. An image of his x-ray is shown below. What is the most
likely diagnosis?

Page 6

458
A. ARDS
B. Pleural effusion
C. Pulmonary embolism
D. Fat embolism
----------------------------------------
17. Which of the following is the most common cause of death in acute pancreatitis within 2 weeks of
hospitalization?
(or)
A 50-year-old man presents to the emergency room with acute onset epigastric pain. His medical
history includes hyperlipidemia, hypertension, diabetes, and alcohol abuse. The physical exam is
notable for tenderness on palpation of the epigastrium. There is no muscle spasm with the percussion
of the cheeks. Laboratory evaluation shows significantly elevated amylase and lipase. He was
diagnosed with acute pancreatitis and started on aggressive fluid resuscitation. Which of the following
is the most common cause of death in acute pancreatitis within 2 weeks of hospitalization?
A. Sepsis
B. Shock
C. Multiorgan dysfunction syndrome
D. Massive bleeding
----------------------------------------
18. How will you manage the patient with past history of acute pancreatitis with the current CT image
given below?
(or)
A 35-year-old female came with epigastric pain and fever. She reported that she was admitted to the
hospital 4 weeks back for acute pancreatitis. CECT abdomen is performed, which is shown in the
image below. How will you manage the patient?

Page 7

459
A. Cystogastrostomy
B. Cystojejunostomy
C. Percutaneous catheter drainage
D. Antibiotics alone
----------------------------------------
19. A 19-year-old girl from Kerala presented with a 5-month history of recurrent abdominal pain. She
was a known diabetic with a past medical history significant for vitamin B12 deficiency, iron deficiency
anemia, and acute pancreatitis. History was negative for smoking, alcohol, drug use, gallstone disease,
constitutional symptoms, and medication use. Lab work was unremarkable. Abdominal CT showed
extensive calcification within the pancreas head, body, and tail and dilated pancreatic ducts. ERCP
showed multiple calcified stones with a large stone in the head of the pancreas causing an obstruction.
What is the gene mutation that you can expect the patient to have?
(or)
What is the gene mutation associated with Tropical Acute Pancreatitis?
A. ALK-4 mutation
B. K-RAS mutation
C. P16 gene mutation
D. SPINK1 mutations
----------------------------------------
20. A 30-year-old patient comes to the emergency department with abdominal pain that has persisted
for the last 5 hours. On physical examination, she has epigastric pain. Amylase and lipase levels were
elevated, and she was diagnosed with acute pancreatitis and started on treatment. She is discharged
when her condition improves, but returns after 18 days of the initial presentation with abdominal pain
and fever. A CECT scan demonstrates extraluminal air in the peripancreatic fat collection. Which of the
following is the most likely diagnosis?
(or)
A 30-year-old patient is diagnosed with acute pancreatitis. After 18 days of the initial presentation with
abdominal pain and fever. A CECT scan demonstrates extraluminal air in the peripancreatic fat
collection. Which is the most likely diagnosis?
A. Recurrent acute pancreatitis
B. Infected pancreatic necrosis

Page 8

460
C. Pseudocyst
D. Systemic inflammatory response syndrome
----------------------------------------
21. A 30-year-old man comes to the clinic complaining of foul-smelling diarrhoea, increased urination,
and right upper quadrant pain. His fasting blood glucose is 140 mg/dl, and the ultrasound reveals
gallstones in the bile duct. Which of the following is the most likely diagnosis?
A. Glucagonoma
B. Somatostatinoma
C. VIPoma
D. Insulinoma
----------------------------------------
22. A 37-year-old is hospitalized for 2 weeks with epigastric pain radiating to his back, nausea, and
vomiting. CT scan demonstrates a 6 cm x 6 cm rim-enhancing fluid collection in the body of the
pancreas. Which of the following is the most definitive management of fluid collection?
(or)
A 37-year-old is hospitalized for 2 weeks with epigastric pain radiating to his back, nausea, and
vomiting. Initial laboratory values revealed an elevated amylase level consistent with acute pancreatitis.
4 weeks later, he complained of early satiety, epigastric pain, and fever. His white blood cell (WBC)
count is 23,000/mm3. CT scan demonstrates a 6 cm x 6 cm rim-enhancing fluid collection in the body
of the Pancreas. Which of the following is the most definitive management of fluid collection?
A. Antibiotics and percutaneous catheter drainage
B. CT-guided aspiration with repeat imaging in 2 to 3 days
C. Antibiotics and CT-guided aspiration with repeat imaging in 2 to 3 days
D. Antibiotic therapy alone
----------------------------------------
23. Which of the following statements regarding Annular Pancreas is incorrect?
A. The ring of the ventral pancreas is present around the 2nd part of the Duodenum
B. There are bilobed ventral pancreatic buds
C. The annular pancreas is located proximal to the ampulla
D. It is associated with Down Syndrome and Duodenal Atresia.
----------------------------------------
24. A 45-year-old male presents with severe epigastric pain radiating to the left side of the back. He
experiences partial relief when sitting or bending forward. On examination, there is nausea, vomiting,
and abdominal distention. X-ray abdomen reveals a gasless abdomen with a ground glass appearance,
and the presence of a sentinel loop sign when involving the jejunum and colon cut-off sign when
involving the colon. Which of the following statements regarding this condition is incorrect? A. It is
associated with left-sided pleural effusion. B. There is a retroperitoneal bleed into the facial planes. C.
Cullen sign, ecchymosis around the umbilicus, is a characteristic finding. D. Grey Turner sign,
ecchymosis around the flank, is observed in this condition. E. Fox sign, ecchymosis around the inguinal
region, is a common manifestation.
(or)

Page 9

461
In a 45-year-old male with severe epigastric pain, nausea, vomiting, and a gasless abdomen on X-ray,
which statement is incorrect? A. It is associated with left-sided pleural effusion. B. There is a
retroperitoneal bleed into the facial planes. C. Cullen sign, ecchymosis around the umbilicus, is a
characteristic finding. D. Grey Turner sign, ecchymosis around the flank, is observed in this condition.
E. Fox sign, ecchymosis around the inguinal region, is a common manifestation.

A. A, B, C
B. B, C
C. A only
D. B only
----------------------------------------
25. A 65-year-old male presents with severe abdominal pain, nausea, and vomiting. Laboratory
investigations reveal a BUN level of 30 mg/dL, an elevated heart rate of 100 beats/min, a respiratory
rate of 22 breaths/min, and a WBC count of 14,500 cells/mm3 with 8% band cells. The patient does not
show impaired mental status or pleural effusion. What is the BISAP score for this patient, and what
does it suggest about the severity of acute pancreatitis?
(or)
What is the BISAP score for a 65-year-old male with severe abdominal pain, a BUN level of 30 mg/dL,
heart rate 100 beats/min, respiratory rate 22 breaths/min, and WBC count 14,500 cells/mm3 (8% band
cells)? No impaired mental status or pleural effusion is observed. What does the BISAP score suggest
about the severity of acute pancreatitis?
A. BISAP score of 3, indicative of severe acute pancreatitis with >15% mortality
B. BISAP score of 2, suggesting a mild form of acute pancreatitis with <2% mortality
C. BISAP score of 4, indicative of severe acute pancreatitis with >15% mortality.
D. BISAP score of 1, suggesting a moderate form of acute pancreatitis with 2-15% mortality.
----------------------------------------
26. A 40-year-old male presents with recurrent abdominal pain, steatorrhea, and weight loss.
Suspecting chronic pancreatitis, which of the following statements regarding diagnostic investigations is
accurate?
A. MRCP is the Gold Standard investigation for chronic pancreatitis
B. ERCP is both diagnostic and therapeutic and shows a String of Pearls or Chains of Lakes
Appearance on imaging
C. Endoscopic ultrasound is not useful in diagnosing minimal changes in chronic pancreatitis

Page 10

462
D. ROSEMONT criteria is based on MRCP findings for the diagnosis of chronic pancreatitis.
----------------------------------------
27. A 45-year-old male presents with persistent abdominal pain, weight loss, and jaundice. On further
evaluation, imaging studies reveal a mass in the pancreatic head. Which of the following established
risk factors is most strongly associated with the development of pancreatic carcinoma in this patient?
(or)
Which of the following established risk factors is most strongly associated with the development of
pancreatic carcinoma in this patient?
A. Hereditary Breast and Ovarian Cancer
B. Ataxia Telangiectasia
C. Familial atypical multiple mole melanoma
D. Peutz-Jeghers Syndrome
----------------------------------------

Correct Answers
Question Correct Answer

Question 1 4
Question 2 1
Question 3 2
Question 4 1
Question 5 4
Question 6 2
Question 7 4
Question 8 4
Question 9 3
Question 10 2
Question 11 2
Question 12 2
Question 13 1
Question 14 4
Question 15 1
Question 16 1
Question 17 3
Question 18 3
Question 19 4
Question 20 2

Page 11

463
Question 21 2
Question 22 1
Question 23 3
Question 24 4
Question 25 1
Question 26 2
Question 27 4

Solution for Question 1:


Option D: Conservative management
• The patient had an episode of pancreatitis 5 weeks ago and now presents with epigastric pain for the
last 3 weeks, and an abdominal mass on the epigastric region points toward the formation of a
pseudocyst of the pancreas, which is a complication of post-acute pancreatitis
• Pseudocyst is a chronic collection of pancreatic amylase-rich fluid surrounded by a non-epithelialized
wall of granulation tissue and fibrosis
• They comprise the most common complication of chronic pancreatitis. They also arise following a mild
attack of acute pancreatitis and lie outside but close to the pancreas, especially in the lesser sac but
can be found in the pelvis, scrotum, mediastinum, or thorax
• Formation of a pseudocyst requires 4 weeks or more from the onset of acute pancreatitis.
Pseudocysts usually cause symptoms of pain, fullness, or early satiety. The fluid from a pseudocyst
typically has a low CEA level, high amylase level, and absence of mucin
• Most cases are managed conservatively. If the pseudocyst fails to resolve with conservative therapy
and symptoms persist, internal drainage is usually preferred to external drainage to avoid the
complication of a pancreatico-cutaneous fistula. Internal drainage may be performed with either
endoscopic methods ( transgastric or transduodenal puncture and multiple stent placements) or
surgical methods
• Pseudocysts that are thick-walled or large (over 6 cm in diameter), has lasted for a long time (over 12
weeks), or have arisen in the context of chronic pancreatitis are less likely to resolve spontaneously
Option A: Percutaneous drainage
• Percutaneous drainage through endoscopy or surgical method would be necessary if the patient does
not recover from conservative management, which is the first line of treatment
Option B: Laparotomy
• Laparotomy would have been necessary if the pseudocyst had ruptured and there was hemodynamic
instability. There are no signs of peritonitis in this patient, which presents with guarding and rebound
tenderness and pneumoperitoneum(air under the diaphragm) on x-ray
Option C: Cystogastrostomy
• Cystogastrostomy is a surgery to create an opening between a pancreatic pseudocyst and the
stomach when the cyst is in a suitable position to be drained into the stomach. This conserves
pancreatic juices that would otherwise be lost. But the first line of management is conservative
management in the case of pseudocyst

Page 12

464
Solution for Question 2:
Option A: CFTR
• The patient has had intermittent epigastric pain for the last 2 months and is suffering from steatorrhea,
and his amylase and lipase levels are high. Since this is a teenage boy with no history of alcohol abuse
and gallstones which are the most common reasons for chronic pancreatitis. Then there must be some
internal genetic predisposition to chronic pancreatitis in this patient
• This patient is diagnosed with idiopathic chronic pancreatitis, which is due to an abnormal CFTR gene
• The risk of idiopathic chronic pancreatitis (ICP) increases in individuals with CFTR genotypes
containing a CF-causing mutation plus a second pathogenic allele.
• The CFTR gene codes for an ABC transporter-class ion channel protein that conducts chloride ions
across epithelial cell membranes. Mutations of the CFTR gene affecting chloride ion channel function
lead to dysregulation of epithelial lining fluid (mucus) transport in the lung, pancreas, and other
organs across epithelial cell membranes.
• The CFTR mutation can cause cystic fibrosis, which is an obstructive respiratory disease along with
infertility and chronic pancreatitis in children, but rarely it can cause only idiopathic chronic
pancreatitis like in this patient
Option B: PRSS 1 gene
• PRSS 1 gene is involved in Hereditary pancreatitis, an autosomal condition with early-onset
pancreatitis in the family.
Option C: SPINK1 gene
• SPINK 1 gene is involved in Tropical calcific pancreatitis. SPINK1 is an inflammation-induced trypsin
inhibitor secreted in acinar cells. With the mutation of the SPINK gene, the trypsin is no longer inhibited,
and it causes autoactivation of the trypsin, causing Tropical calcific pancreatitis in patients. This
condition is mostly seen in developing countries of the tropical world
Option D: p53
• p53 is the tumour suppressor gene, and its mutation can cause Li-Fraumeni syndrome, which
consists of sarcoma, breast carcinoma, leukemia, and adrenal gland tumor.

Solution for Question 3:


Option B: Pancreaticojejunostomy
• In the case history above, the patient has intractable pain with pancreatic dilatation. The ERCP was
performed, but still, the patient was symptomatic. The best treatment option for this case is
Pancreaticojejunostomy through Puestow's procedure
• The surgery involves creating a longitudinal incision along the pancreas while the main pancreatic
duct is filleted open longitudinally from the head of the organ to its tail. The duct and pancreas are then
attached to the jejunum (Pancreaticojejunostomy), which is overseen to the exposed pancreatic duct to
allow drainage. When used in the appropriate setting, pain from chronic pancreatitis can improve
Option A: Pancreatic tail resection
• Pancreatic tail resection, which is the distal pancreatectomy, is done in patients with a non dilated
pancreatic duct, but it is done when drainage with Pancreaticojejunostomy with frey's procedure is not
possible as it is a complicated surgery and can have many complications

Page 13

465
Option C: Percutaneous removal of stone
• Percutaneous removal of stone through endoscopy is a new procedure done to give relief to
symptomatic patients, but in this patient, ERCP is already done, and it failed to give results, so
permanent drainage through Pancreaticojejunostomy is the better option in this case
Option D: Medical management
• Medical management through pain relief medications and enzyme replacement therapy is the first line
of treatment for patients with chronic pancreatitis, but this patient is not responsive to analgesics and
needs further treatment

Solution for Question 4:


Option A: Beger's procedure
• The duodenum-preserving pancreatic head resection was first described by Beger and developed to
avoid some of the adverse sequelae of pancreaticoduodenectomy.
• A pancreatic head resection is performed, and only a small part of pancreatic tissue is left on the
duodenum to preserve the duodenum. In this procedure, the stomach and common bile duct are not
divided.
• In 25% of the cases, it is still necessary to perform biliary anastomosis on the intrapancreatic part of
the common bile duct. A two-sided pancreaticojejunostomy is performed on the small part of pancreatic
tissue on the duodenum and the pancreas remnant.
• Compared to the pancreaticoduodenectomy, an advantage of the Beger procedure is that the
duodenum is preserved and that only an anastomosis should be made on both sides of the pancreas. A
disadvantage is that it can be a technically demanding operation with a difficult technical anastomosis.
Option B: Frey's procedure

• Frey and Smith described the local Resection of the pancreatic head with longitudinal
Pancreaticojejunostomy (LR-LPJ)

Page 14

466
• In this procedure, the pancreatic head is not fully resected. An advantage of the Frey procedure
compared to pancreatic head resection is that it also can treat duct disease in the pancreatic remnant
and that coring out the pancreatic head appears to be safer than a pancreatic head resection when
portal hypertension or thrombosis is present.
• It provides thorough decompression of the pancreatic head as well as the body and tail of the gland
Option C: Puestow's procedure

• Puestow aka Longitudinal pancreaticojejunostomy (LPJ): Originally described as an invaginating


anastomosis that drained the entire body and tail. The anastomoses were created after amputating the
tail of the gland and opening the duct along the long axis of the gland. This procedure allows
good postoperative pain relief of patients and maintains pancreatic function by providing wide ductal
drainage without pancreatic resection.
• The pancreatic head is not resected in this procedure
Option D: Duval procedure
• Duval procedure is the Resection of the tail of the pancreas with retrograde pancreaticojejunostomy.
The head of the pancreas is not resected as it is a drainage procedure and not a resection procedure

Solution for Question 5:


Option D: CECT shows peripheral calcified central scar
• The images above reveals a large tumour central scar with calcifications, indicating a serous
cystadenoma. A contrast-enhanced CT scan reveals a calcified central scar.
• Serous cystadenomas are classified as benign tumours with no malignant potential. They are most
commonly encountered in older women. About half of all cystadenomas are asymptomatic and
discovered by chance. Mild upper stomach discomfort, epigastric fullness, or modest weight loss
characterize the majority of symptomatic individuals.
• These lesions have thin serous fluid that does not stain positive for mucin and has a low CEA
concentration (200 ng/mL).

Page 15

467
• A well-circumscribed mass, tiny septations, fluid near water density, and, occasionally, a central scar
with calcification are typical imaging findings.
• Endoscopic ultrasound-FNA should produce nonviscous fluid with low CEA and amylase levels; if
cells are recovered, uncommonly, they should be cuboidal with transparent cytoplasm.
Option A: Most commonly seen in males
• Serous cystadenomas are most commonly seen in females, especially in the 5th-6th decade.
Option B: Mostly malignant
• Serous cystadenomas are classified as benign tumours with no malignant potential.
Option C: Macrocystic
• They have a spongy look and are more likely to contain several tiny cysts (microcystic) than bigger
cysts ( macrocystic or oligocystic)

Solution for Question 6:


Option B: Mucinous cystadenoma
• CECT showing peripheral egg-shell calcification suggesting the diagnosis of mucinous cystadenoma
• Mucinous cystadenoma: Mucinous cystadenoma is the most common pancreatic cystic neoplasm
Females are more prone to it (perimenopausal women). Columnar epithelial lining The body and tail of
the pancreas are the most common locations. It is premalignant The mucin-producing epithelium is
present, and mucin-rich cells and ovarian-like stroma distinguish them histologically. In most cases,
oestrogen and progesterone staining are positive. Linked to an increase in CEA Normal amylase levels
The cyst has strong walls and does not connect with the main pancreatic duct on imaging. The cyst wall
may contain nodules or calcifications. CECT shows egg-shell calcification of the periphery ( solitary
cyst with fine septations surrounded by a calcification rim). Malignancy is indicated by peripheral
egg-shell calcification, and greater tumour size. FNA reveals mucin-rich aspirate and high CEA values
(>192 ng/mL). Amylase levels in cyst fluid are low. Distal pancreatectomy is the treatment.
• Mucinous cystadenoma is the most common pancreatic cystic neoplasm
• Females are more prone to it (perimenopausal women). Columnar epithelial lining
• The body and tail of the pancreas are the most common locations.
• It is premalignant
• The mucin-producing epithelium is present, and mucin-rich cells and ovarian-like stroma distinguish
them histologically.
• In most cases, oestrogen and progesterone staining are positive.
• Linked to an increase in CEA
• Normal amylase levels
• The cyst has strong walls and does not connect with the main pancreatic duct on imaging. The cyst
wall may contain nodules or calcifications.
• CECT shows egg-shell calcification of the periphery ( solitary cyst with fine septations surrounded by
a calcification rim).
• Malignancy is indicated by peripheral egg-shell calcification, and greater tumour size.

Page 16

468
• FNA reveals mucin-rich aspirate and high CEA values (>192 ng/mL).
• Amylase levels in cyst fluid are low.
• Distal pancreatectomy is the treatment.
• Mucinous cystadenoma is the most common pancreatic cystic neoplasm
• Females are more prone to it (perimenopausal women). Columnar epithelial lining
• The body and tail of the pancreas are the most common locations.
• It is premalignant
• The mucin-producing epithelium is present, and mucin-rich cells and ovarian-like stroma distinguish
them histologically.
• In most cases, oestrogen and progesterone staining are positive.
• Linked to an increase in CEA
• Normal amylase levels
• The cyst has strong walls and does not connect with the main pancreatic duct on imaging. The cyst
wall may contain nodules or calcifications.
• CECT shows egg-shell calcification of the periphery ( solitary cyst with fine septations surrounded by
a calcification rim).
• Malignancy is indicated by peripheral egg-shell calcification, and greater tumour size.
• FNA reveals mucin-rich aspirate and high CEA values (>192 ng/mL).
• Amylase levels in cyst fluid are low.
• Distal pancreatectomy is the treatment.
Option A: Serous cystadenoma
• Pancreatic serous cystadenoma is a benign pancreatic tumour.
• CECT findings Central Stellate scar Central Sunburst Calcification
• Central Stellate scar
• Central Sunburst Calcification
• Central Stellate scar
• Central Sunburst Calcification
Option C: Intraductal papillary mucinous neoplasm
• Intraductal papillary mucinous neoplasm (IPMN) is a form of tumour that can develop within
pancreatic duct cells. IPMN tumours produce mucus, that can develop pancreatic cysts. Although
intraductal papillary mucinous neoplasms are benign tumours, they can develop into pancreatic cancer.
• They appear as single or multiple pancreatic cystic hypodense lesions on CT imaging. For the primary
type IPMN, dilatation of the main duct greater than 5 mm is cause for worry. On CT, connection with the
pancreatic ducts, particularly side branch lesions, may be difficult to see. They are not calcified.
Option D: Acinic cell tumour
• Acinar cell carcinoma is a rare malignant exocrine pancreatic tumour. It accounts for 5% of all
pancreatic exocrine tumours, making it the second most frequent kind of pancreatic cancer.

Page 17

469
• On contrast-enhanced CT, the tumour is often shown as an enhancing, ovoid, solid pancreatic
tumour. Variable hypo/hyperattenuating mass relative to the pancreas, with distinct borders, can be
seen along with dilation of non-biliary ducts.

Solution for Question 7:


Option D: Usually diagnosed by ERCP
• The mucin extrusion found on ERCP is frequently used to diagnose intraductal papillary mucinous
neoplasm (IPMN).
• They are more common in the gland's head and within the pancreatic ducts. They are more common
in senior males (initial reports showed male predominance, but more recent reports indicate an equal
distribution).
• Patients appear with stomach discomfort or recurrent pancreatitis, which is considered to be caused
by thick mucus obstructing the pancreatic duct. Some people get steatorrhea, hyperglycemia, and
weight loss due to pancreatic insufficiency.
• The mucin generated may exude from the ampulla of Vater, resulting in a fish-eye sign, a finding
found on ERCP that is almost always indicative of an IPMN. In addition, radiographic evidence of ductal
dilatation with abnormalities is frequently diagnostic.
• Some IPMNs affect the primary pancreatic duct, whilst others affect the branch ducts.
Option A: Seen in only small ducts
• It is seen in both small and large ducts
Option B: Higher incidence in males
• IPMN has an equal incidence in males and females
Option C: Most common in the tail of the pancreas
• It occurs most commonly in the head of the pancreas

Solution for Question 8:


Option D: Pancreaticojejunostomy, hepaticojejunostomy, gastrojejunostomy
• After ensuring negative margins with a frozen slice, a pancreaticojejunostomy is made in two layers,
anterior and posterior, with a duct to mucosa anastomosis. For ducts less than 5 mm in diameter, an
internal pancreatic stent can be left in place.
• Following the completion of the pancreatic anastomoses, the choledochojejunostomy is done around
10 cm down the jejunal limb from the pancreatic anastomosis. It is normally done from end to finish with
one layer of interrupted sutures.
• The ante colic duodenojejunostomy or gastrojejunostomy is done 10 to 15 cm downstream from the
biliary anastomoses using a two-layer approach.

Option A: Pancreaticojejunostomy, gastrojejunostomy, hepaticojejunostomy

Page 18

470
• After pancreaticoduodenectomy, Whipple's procedure restores the pancreas, then the bile duct, and
finally, the duodenum or stomach.
Option B: Hepaticojejunostomy, pancreaticojejunostomy, gastrojejunostomy
• Whipple's approach involves restoring the pancreas, then the bile duct, and finally, the duodenum or
stomach following pancreaticoduodenectomy.
Option C: Gastrojejunostomy, pancreaticojejunostomy, hepaticojejunostomy
• Following pancreaticoduodenectomy, Whipple's procedure restores the pancreas, then the bile duct,
and finally, the duodenum or stomach.

Solution for Question 9:


Option C: Most common site is the duodenum

• Gastrinoma is the second most frequent functional pancreatic endocrine tumour, causing
Zollinger-Ellison syndrome (ZES), which is characterised by acid hypersecretion and peptic ulceration.
• Although the majority of the ulcers are solitary, several ulcers in unusual sites that do not react to
antacids should raise the possibility of ZES. The most frequent type of ulcer is a duodenal ulcer.
• Gastrin levels of more than 1000 pg/mL are indicative of gastrinoma. However, this is not the case
because gastrin levels are high in other illnesses such as pernicious anaemia, atrophic gastritis, and so
on. A secretin stimulation test is useful in ambiguous instances.
• Somatostatin receptors SSTR (octreotide) scintigraphy in conjunction with CT is the preferred test.
The octreotide scan is more sensitive than CT, identifying tumours as small as 1 cm and finding around
85% of gastrinomas.
• Endoscopic ultrasonography (EUS) is useful in locating tumours in the pancreatic head or duodenum
wall, where gastrinomas are typically 1cm in size.
• The combination of an octreotide scan with a EUS detects more than 90% of gastrinomas.

Page 19

471
• One-fourth of gastrinomas are associated with the MEN1 syndrome. Approximately 50% of
gastrinomas spread to lymph nodes or the liver and are thus classified as malignant.
Option A: Mostly found in the Hesselbach triangle
• Primary gastrinoma is detected in 70% to 90% of patients in Passaro's triangle, a region described by
a triangle with points at the junction of the cystic duct and common bile duct, the second and third
section of the duodenum, and the neck and body of the pancreas.
Option B: Basal acid output >5 mEq/hour is diagnostic
• A basal acid output of more than 15 mEq/hour is diagnostic
Option D: Lymphadenectomy improves survival
• Treatment includes full tumour removal. Lymphadenectomy is unnecessary because there is no
benefit in survival.

Solution for Question 10:


Option B: Glucagonoma
• The image shows Central necrosis and Peripheral Erythema, "Necrolytic migratory erythema". It is the
most common manifestation of Glucagonoma
• Glucagonoma: Diabetes in association with dermatitis should raise the suspicion of a glucagonoma.
The classic necrolytic migratory erythema (due to low levels of amino acids) manifests as cyclic
migrations of lesions with spreading margins and healing centres, typically on the lower abdomen,
perineum, perioral area, and feet. Patients also complain of an enlarged sensitive tongue The diagnosis
is confirmed by measuring serum glucagon levels, usually >500 pg/mL They are more often in the body
and tail of the pancreas. Treatment involves complete resection for resectable disease
• Diabetes in association with dermatitis should raise the suspicion of a glucagonoma. The classic
necrolytic migratory erythema (due to low levels of amino acids) manifests as cyclic migrations of
lesions with spreading margins and healing centres, typically on the lower abdomen, perineum, perioral
area, and feet.
• Patients also complain of an enlarged sensitive tongue
• The diagnosis is confirmed by measuring serum glucagon levels, usually >500 pg/mL
• They are more often in the body and tail of the pancreas.
• Treatment involves complete resection for resectable disease
• Diabetes in association with dermatitis should raise the suspicion of a glucagonoma. The classic
necrolytic migratory erythema (due to low levels of amino acids) manifests as cyclic migrations of
lesions with spreading margins and healing centres, typically on the lower abdomen, perineum, perioral
area, and feet.
• Patients also complain of an enlarged sensitive tongue
• The diagnosis is confirmed by measuring serum glucagon levels, usually >500 pg/mL
• They are more often in the body and tail of the pancreas.
• Treatment involves complete resection for resectable disease
Option A: Insulinoma

Page 20

472
• Insulinoma is a pancreatic tumour in which the symptoms present as a Whipple triad Symptoms of
hypoglycemia (palpitations, dizziness) Fasting hypoglycemia Correction of symptoms with
administration of glucose
• Symptoms of hypoglycemia (palpitations, dizziness)
• Fasting hypoglycemia
• Correction of symptoms with administration of glucose
• Symptoms of hypoglycemia (palpitations, dizziness)
• Fasting hypoglycemia
• Correction of symptoms with administration of glucose
Option C: Gastrinoma
• Gastrinoma is a pancreatic tumour with an overproduction of gastrin which causes hypersecretion of
acid and distal duodenum ulcers with gastrin levels >1000 pg/mL.
Option D: Somatostatinoma
• Somatostatinoma is a somatostatin-producing pancreatic tumour. Somatostatin inhibits the pancreatic
enzymes, both endocrine and exocrine, so it causes diabetes and steatorrhea.

Solution for Question 11:


Option B: Octreotide
• The tumour in the pancreatic tail and hypokalemia with watery diarrhoea suggests VIPoma, which is
treated with somatostatin analogue octreotide
• Vasoactive Intestinal Peptide Secreting Tumour (VIPoma): It usually arise from pancreatic islet D2
cells and release high levels of VIP It also called as WDHA syndrome or pancreatic cholera. Verner and
Morrison described this syndrome. They are most commonly located in the distal pancreas. More than
two-thirds of tumours are malignant, and greater than 70% of patients have metastatic disease at the
time of presentation. Weight loss, cramps, abdominal pain, dehydration, electrolyte abnormalities and
metabolic acidosis are common Serum VIP levels must be measured multiple times because the
excess secretion of VIP is episodic, and single measurements might be normal and misleading.
VIPoma have serum VIP levels ranging from 225 to 2000 pg/mL A CT scan localizes most VIPomas,
but endoscopic ultrasound is the most sensitive imaging method Treatment begins with aggressive
preoperative hydration and correction of electrolyte abnormalities and acid-base
disturbances. Somatostatin analogues (octreotide) are commonly used preoperatively to reduce
diarrhoea volume and facilitate fluid and electrolyte replacement. Resection with negative margins and
lymphadenectomy should be done in resectable cases
• It usually arise from pancreatic islet D2 cells and release high levels of VIP
• It also called as WDHA syndrome or pancreatic cholera.
• Verner and Morrison described this syndrome. They are most commonly located in the distal
pancreas. More than two-thirds of tumours are malignant, and greater than 70% of patients have
metastatic disease at the time of presentation.
• Weight loss, cramps, abdominal pain, dehydration, electrolyte abnormalities and metabolic acidosis
are common

Page 21

473
• Serum VIP levels must be measured multiple times because the excess secretion of VIP is episodic,
and single measurements might be normal and misleading. VIPoma have serum VIP levels ranging
from 225 to 2000 pg/mL
• A CT scan localizes most VIPomas, but endoscopic ultrasound is the most sensitive imaging method
• Treatment begins with aggressive preoperative hydration and correction of electrolyte abnormalities
and acid-base disturbances. Somatostatin analogues (octreotide) are commonly used preoperatively to
reduce diarrhoea volume and facilitate fluid and electrolyte replacement. Resection with negative
margins and lymphadenectomy should be done in resectable cases
• It usually arise from pancreatic islet D2 cells and release high levels of VIP
• It also called as WDHA syndrome or pancreatic cholera.
• Verner and Morrison described this syndrome. They are most commonly located in the distal
pancreas. More than two-thirds of tumours are malignant, and greater than 70% of patients have
metastatic disease at the time of presentation.
• Weight loss, cramps, abdominal pain, dehydration, electrolyte abnormalities and metabolic acidosis
are common
• Serum VIP levels must be measured multiple times because the excess secretion of VIP is episodic,
and single measurements might be normal and misleading. VIPoma have serum VIP levels ranging
from 225 to 2000 pg/mL
• A CT scan localizes most VIPomas, but endoscopic ultrasound is the most sensitive imaging method
• Treatment begins with aggressive preoperative hydration and correction of electrolyte abnormalities
and acid-base disturbances. Somatostatin analogues (octreotide) are commonly used preoperatively to
reduce diarrhoea volume and facilitate fluid and electrolyte replacement. Resection with negative
margins and lymphadenectomy should be done in resectable cases
Option A: Loperamide
• Loperamide acts as a motility blocker in the gut by binding to the opiate receptors. It is used for
travelers' diarrhoea. But it will not be beneficial in VIPoma as the watery diarrhoea is due to increased
Vasoactive intestinal peptide produced from the tumour
Option C: Bismuth sub-salicylate
• Bismuth sub-salicylate acts as a cytoprotective used in peptic ulcer disease as it forms a coat around
the ulcers to make them protected to further damage from the acid
Option D: Diamode
• Diamode is a brand name for loperamide which is used in travelers' diarrhoea.
• Diamode acts by blocking the motility of the gut

Solution for Question 12:


Option B: Pancreaticoduodenectomy
• According to the given situation, the injury is a Grade V injury. Pancreaticoduodenectomy is to be
done to manage this patient.
• Whipple's procedure/Pancreaticoduodenectomy- order of anastomoses 1st - Pancreaticojejunostomy
2nd -Hepaticojejunostomy 3rd -Gastrojejunostomy

Page 22

474
• 1st - Pancreaticojejunostomy
• 2nd -Hepaticojejunostomy
• 3rd -Gastrojejunostomy
• 1st - Pancreaticojejunostomy
• 2nd -Hepaticojejunostomy
• 3rd -Gastrojejunostomy
Grading
Injury
Description
Grade I
Hematoma
Laceration
Mild contusion without duct injury
Superficial laceration without duct injury
Grade II
Major contusion without duct injury
Major laceration without duct injury or tissue loss
Grade III
Distal transection or parenchymal injury with duct injury
Grade IV
Proximal transection or parenchymal injury involving the ampulla
Grade V
Massive disruption of the pancreatic head
Management:
• Grade I, II - Non-operative management I – Observation II – Debridement, drainage, possible repair
• I – Observation
• II – Debridement, drainage, possible repair
• Grade III: Distal pancreatectomy, Roux-en-Y drainage
• Grade IV and V: Damage control surgery Resection + Roux-en-Y drainage Pyloric exclusion (
pancreas resected, pylorus sutured, gastrointestinal continuity established by gastrojejunostomy
Pancreaticoduodenectomy
• Resection + Roux-en-Y drainage
• Pyloric exclusion ( pancreas resected, pylorus sutured, gastrointestinal continuity established by
gastrojejunostomy
• Pancreaticoduodenectomy
• I – Observation

Page 23

475
• II – Debridement, drainage, possible repair
• Resection + Roux-en-Y drainage
• Pyloric exclusion ( pancreas resected, pylorus sutured, gastrointestinal continuity established by
gastrojejunostomy
• Pancreaticoduodenectomy
Option A: Roux-en-Y anastomosis
• Roux-en-Y anastomosis surgery with pancreaticojejunostomy is done in Grade III and Grade IV
injuries in which the ampulla or the duct is injured
Option C: Lateral tube jejunostomy
• Lateral tube jejunostomy is a feeding jejunostomy tube that is used for feeding mostly after pancreatic
surgeries like the Whipple procedure and other pancreaticoduodenectomy
Option D: Retrograde jejunostomy
• A retrograde jejunostomy is used for decompression after esophagectomy and is not used for treating
pancreatic injury grade V

Solution for Question 13:


Option A: External compression of the duodenum
• Nausea and vomiting are due to external compression of the duodenum by pancreatic cancer
• The majority of resectable pancreatic cancers occur in the right side of the pancreas, in the head or
uncinate process of the gland
• Classic symptoms from these right-sided tumours include jaundice(painless) from biliary obstruction,
often accompanied by pruritus and epigastric pain radiating to the back due to involvement of the celiac
plexus. Less commonly, persistent nausea or vomiting as a result of gastric outlet obstruction from
tumour narrowing at the duodenal C-loop is the initial symptom
• Tumours in the left side of the pancreas typically cause few symptoms until they are locally advanced
or metastatic. For such left-sided tumours, pain is the most common presenting symptom, occasionally
steatorrhea may be seen, and jaundice is rare
Option B: Portal vein infiltration
• Portal vein infiltration will cause enlarged varices and GI bleeding and can cause Budd-Chiari
syndrome. But it mostly occurs in HCC or late-stage metastasis of pancreatic carcinoma
Option C: Proliferative infiltration of tumour into the duodenum
• It occurs with pancreatic tumour metastasis that will cause complete obstruction of the duodenum,
which will cause extreme pain and zero tolerance towards foods and drinks, and the patient will not be
able to survive for 3 months. Here the obstruction due to external compression is partial, which causes
nausea and vomiting in the patient
Option D: Activation of chemoreceptors in the brain
• Activation of chemoreceptors causing nausea and vomiting occurs due to chemotherapy and after
anesthesia

Page 24

476
Solution for Question 14:
Option D: T4
• This patient has a tumour in the head of the Pancreas that is extending to the superior mesenteric
vessel, which suggests T4
• TNM staging of pancreatic adenocarcinoma
• In this patient, there is involvement of superior mesenteric artery, so it is T4,
Option A: T1
• T1 means that the tumour is limited to the pancreas and is less than 2 cm in the greatest dimension.
Here in this patient tumour involves the SMA and is not limited only to the pancreas
Option B: T2
• If the tumour is >2 cm and <4 cm in greatest dimension but is still limited to the Pancreas, that would
be T2
Option C: T3
• The tumour is >4 cm and is limited to Pancreas, which would mean that it is in the T3 stage

Solution for Question 15:


Option A: Symptomatic
• Symptomatic patients and many cases of pancreatic ascites respond to medical therapy with bowel
rest and parenteral nutrition
• PANCREATIC ASCITES: Pancreatic ascites occurs when a pancreatic duct is disrupted(leak), leading
to pancreatic fluid extravasation that does not become sequestrated as a pseudocyst but drains freely
into the peritoneal cavity and causes pancreatic ascites Also called an Internal pancreatic fistula Seen
more often in patients with chronic pancreatitis Paracentesis - non-infected fluid with protein level >25
g/L and a markedly elevated amylase level Serum amylase levels may also be elevated ERCP -
delineates the leak's location and elucidates the underlying pancreatic ductal anatomy. Pancreatic duct
stenting may be considered at the time of ERCP Antisecretory therapy with somatostatin analogue
octreotide, together with bowel rest and the nasojejunostomy feeding tube, is successful in more than
half of the patients Reappositon of serosal surfaces to facilitate closure of the leak is considered part of
therapy, and this is accomplished by complete paracentesis
• Pancreatic ascites occurs when a pancreatic duct is disrupted(leak), leading to pancreatic fluid
extravasation that does not become sequestrated as a pseudocyst but drains freely into the peritoneal
cavity and causes pancreatic ascites
• Also called an Internal pancreatic fistula
• Seen more often in patients with chronic pancreatitis
• Paracentesis - non-infected fluid with protein level >25 g/L and a markedly elevated amylase level
• Serum amylase levels may also be elevated

Page 25

477
• ERCP - delineates the leak's location and elucidates the underlying pancreatic ductal anatomy.
Pancreatic duct stenting may be considered at the time of ERCP
• Antisecretory therapy with somatostatin analogue octreotide, together with bowel rest and the
nasojejunostomy feeding tube, is successful in more than half of the patients
• Reappositon of serosal surfaces to facilitate closure of the leak is considered part of therapy, and this
is accomplished by complete paracentesis
• Pancreatic ascites occurs when a pancreatic duct is disrupted(leak), leading to pancreatic fluid
extravasation that does not become sequestrated as a pseudocyst but drains freely into the peritoneal
cavity and causes pancreatic ascites
• Also called an Internal pancreatic fistula
• Seen more often in patients with chronic pancreatitis
• Paracentesis - non-infected fluid with protein level >25 g/L and a markedly elevated amylase level
• Serum amylase levels may also be elevated
• ERCP - delineates the leak's location and elucidates the underlying pancreatic ductal anatomy.
Pancreatic duct stenting may be considered at the time of ERCP
• Antisecretory therapy with somatostatin analogue octreotide, together with bowel rest and the
nasojejunostomy feeding tube, is successful in more than half of the patients
• Reappositon of serosal surfaces to facilitate closure of the leak is considered part of therapy, and this
is accomplished by complete paracentesis
Option B: Completely blocked duct in MRCP
• If there is a complete blockage of the pancreatic duct observed on MRCP, it suggests a severe
obstruction in the pancreatic duct system. In such cases, surgical therapy is often considered the
definitive treatment. This is because complete ductal blockages can impede the effectiveness of
endoscopic procedures like ERCP, making it challenging to place stents or perform interventions to
restore normal pancreatic ductal flow.
• Prompt surgical intervention becomes crucial to address the blockage and prevent further
complications. It's important to note that while medical therapy may be used in conjunction, surgery is
often necessary for cases with a complete blockage visible on MRCP.
Option C: Not responding to medical therapy
• Patients, not responding to medical therapy, nasojejunostomy feeding tube and octreotide will have to
be treated with surgery
Option D: Leak from the stented duct
• ERCP can place a stent in the pancreatic duct at the place of the leak in patients with pancreatic
ascites, but if the stent placement becomes defective and there is a leak from the stented duct, then
surgery is the only treatment.

Solution for Question 16:


Option A: ARDS
• At admission, a patient with acute pancreatitis may present with systemic complications leading to
Multi Organ dysfunction Syndrome (MODS).

Page 26

478
• A respiratory complication associated with acute pancreatitis is ARDS which presents with tachypnea
and respiratory distress. Also, in the given x-ray, widespread bilateral "Ground glass" opacity of lungs
can be noticed, which is suggestive of ARDS
• ARDS can be differentiated from heart failure, which can also present with bilateral opacities on lung
X-ray by looking the PaO2/FiO2 ratio, which is less than 300 and Pcpw is <18
Option B: Pleural effusion
• Pleural effusion mostly presents with unilateral opacities on the lung x-ray, but it can also present
bilaterally, but here ARDS is a complication of acute pancreatitis, so ARDS is the best diagnosis
according to the history and presentation
Option C: Pulmonary embolism
• Pulmonary embolism will cause severe pleuritic pain and tachypnea in the patient. X-ray will not show
any findings in the case of pulmonary embolism
Option D: Fat embolism
• Fat embolism will present with shortness of breath, petechiae, and confusion, which this patient
doesn't have. He only has shortness of breath

Solution for Question 17:


Option C: Multiorgan dysfunction syndrome
• The most common cause of death in acute pancreatitis within 2 weeks of hospitalization is MODS
(Multiorgan Dysfunction Syndrome)
• Acute pancreatitis: It is the inflammation of the pancreas and surrounding tissue, often by
autodigestion with pancreatic enzyme leakage Risk factors are gallstones (more common), heavy
alcohol use (more common), electrolyte abnormalities, increased serum calcium,
increased triglycerides, trauma, and drugs The symptoms of acute pancreatitis are Sudden onset
epigastric pain radiating to the back Nausea and vomiting Systemic inflammation Fever Chills
Complications of acute pancreatitis Pancreatic pseudocyst Fistula formation Pancreatic abscess
Hemorrhagic pancreatitis Pleural effusions (often on the left) Chronic pancreatitis Disseminated
intravascular coagulation (DIC)
• It is the inflammation of the pancreas and surrounding tissue, often by autodigestion with pancreatic
enzyme leakage
• Risk factors are gallstones (more common), heavy alcohol use (more common), electrolyte
abnormalities, increased serum calcium, increased triglycerides, trauma, and drugs
• The symptoms of acute pancreatitis are Sudden onset epigastric pain radiating to the back Nausea
and vomiting Systemic inflammation Fever Chills
• Sudden onset epigastric pain radiating to the back
• Nausea and vomiting
• Systemic inflammation
• Fever
• Chills

Page 27

479
• Complications of acute pancreatitis Pancreatic pseudocyst Fistula formation Pancreatic abscess
Hemorrhagic pancreatitis Pleural effusions (often on the left) Chronic pancreatitis Disseminated
intravascular coagulation (DIC)
• Pancreatic pseudocyst
• Fistula formation
• Pancreatic abscess
• Hemorrhagic pancreatitis
• Pleural effusions (often on the left)
• Chronic pancreatitis
• Disseminated intravascular coagulation (DIC)
• It is the inflammation of the pancreas and surrounding tissue, often by autodigestion with pancreatic
enzyme leakage
• Risk factors are gallstones (more common), heavy alcohol use (more common), electrolyte
abnormalities, increased serum calcium, increased triglycerides, trauma, and drugs
• The symptoms of acute pancreatitis are Sudden onset epigastric pain radiating to the back Nausea
and vomiting Systemic inflammation Fever Chills
• Sudden onset epigastric pain radiating to the back
• Nausea and vomiting
• Systemic inflammation
• Fever
• Chills
• Complications of acute pancreatitis Pancreatic pseudocyst Fistula formation Pancreatic abscess
Hemorrhagic pancreatitis Pleural effusions (often on the left) Chronic pancreatitis Disseminated
intravascular coagulation (DIC)
• Pancreatic pseudocyst
• Fistula formation
• Pancreatic abscess
• Hemorrhagic pancreatitis
• Pleural effusions (often on the left)
• Chronic pancreatitis
• Disseminated intravascular coagulation (DIC)
• Sudden onset epigastric pain radiating to the back
• Nausea and vomiting
• Systemic inflammation
• Fever
• Chills
• Pancreatic pseudocyst
• Fistula formation

Page 28

480
• Pancreatic abscess
• Hemorrhagic pancreatitis
• Pleural effusions (often on the left)
• Chronic pancreatitis
• Disseminated intravascular coagulation (DIC)
Option A: Sepsis
• Sepsis is the most common cause of death after 2 weeks of acute pancreatitis initial episode. Sepsis
can cause hemodynamic shock causing death in a patient
Option B: Shock
• Sepsis and multiorgan failure are the main cause of death after acute pancreatitis. Shock can
manifest as a result of sepsis and multiorgan failure, but it is a secondary cause, not the primary one
Option D: Massive bleeding
• Massive internal bleeding is a cause of death in liver failure in which the coagulation factors are not
formed.

Solution for Question 18:


Option C: Percutaneous catheter drainage
• In the image given above, peripancreatic fluid collection with air-fluid levels is seen, which is
suggestive of pancreatic abscess
• Pancreatic abscess can form as a complication of acute pancreatitis, and it normally occurs after 4
weeks of the initial episode of acute pancreatitis
• A pancreatic abscess is a circumscribed intra-abdominal collection of pus, usually in proximity to the
Pancreas.
• It may be an acute necrotic collection or walled-off necrosis that has become infected.
• Percutaneous drainage with the widest possible drains placed under imaging guidance is the
treatment, along with appropriate antibiotics, and supportive care.
Option A: cystogastrostomy
• Cystogastrostomy is an endoscopic outpatient surgery performed to drain a
pancreatic pseudocyst that develops after acute or chronic pancreatitis. It creates an opening between
the pseudocyst and the stomach to drain the pseudocyst. But in this patient with air-fluid levels, this is
an abscess that needs to be percutaneously drained
Option B: Cystojejunostomy
• Cystojejunostomy is a procedure for pancreatic pseudocysts where an opening is created between
the pseudocyst and the jejunum. This patient has a pancreatic abscess
Option D: Antibiotics alone
• Antibiotics alone will not be able to treat the abscess; the pancreatic abscess needs to be drained with
percutaneous drainage and then started on antibiotics

Page 29

481
Solution for Question 19:
Option D: SPINK1 mutations
• This patient from Kerala has calcification of the Pancreas and pancreatic calculi and is a diabetic,
which indicates that she has tropical chronic pancreatitis and SPINK1 mutation is involved in the
pathology of tropical chronic pancreatitis
• Tropical chronic pancreatitis (TCP) is a juvenile form of chronic calcific non-alcoholic pancreatitis,
seen almost exclusively in the developing countries of the tropical world.
• Tropical chronic pancreatitis: Form of idiopathic pancreatitis Begins at a young age Associated with a
high incidence of diabetes mellitus and stone formation This has been described in Kerala, and
resource-poor countries in Asia, Africa, and Central America Up to 45% to 50% of patients with tropical
pancreatitis have SPINK1 mutations Mechanism: Malnutrition Ingestion of cyanogenic glycosides in
cassava Exposure to hydrocarbons released by kerosene or paraffin lamps Early diagnosis and better
control of the endocrine and exocrine dysfunction could help to ensure better survival and improve the
prognosis and quality of life of TCP patients.
• Form of idiopathic pancreatitis
• Begins at a young age
• Associated with a high incidence of diabetes mellitus and stone formation
• This has been described in Kerala, and resource-poor countries in Asia, Africa, and Central America
• Up to 45% to 50% of patients with tropical pancreatitis have SPINK1 mutations
• Mechanism: Malnutrition Ingestion of cyanogenic glycosides in cassava Exposure to hydrocarbons
released by kerosene or paraffin lamps
• Malnutrition
• Ingestion of cyanogenic glycosides in cassava
• Exposure to hydrocarbons released by kerosene or paraffin lamps
• Early diagnosis and better control of the endocrine and exocrine dysfunction could help to ensure
better survival and improve the prognosis and quality of life of TCP patients.
• Form of idiopathic pancreatitis
• Begins at a young age
• Associated with a high incidence of diabetes mellitus and stone formation
• This has been described in Kerala, and resource-poor countries in Asia, Africa, and Central America
• Up to 45% to 50% of patients with tropical pancreatitis have SPINK1 mutations
• Mechanism: Malnutrition Ingestion of cyanogenic glycosides in cassava Exposure to hydrocarbons
released by kerosene or paraffin lamps
• Malnutrition
• Ingestion of cyanogenic glycosides in cassava
• Exposure to hydrocarbons released by kerosene or paraffin lamps

Page 30

482
• Early diagnosis and better control of the endocrine and exocrine dysfunction could help to ensure
better survival and improve the prognosis and quality of life of TCP patients.
• Malnutrition
• Ingestion of cyanogenic glycosides in cassava
• Exposure to hydrocarbons released by kerosene or paraffin lamps
Option A: ALK-4 mutation
• ALK-4 mutations are described in lung cancers and neuroblastoma, but it is not being reported to be
involved in Tropical chronic pancreatitis
Option B: K-RAS mutation
• K-RAS mutation is described in pancreatic adenocarcinoma, and most patients with pancreatic
adenocarcinoma are reported to have K-RAS mutations
Option C: P16 gene mutation
• The P16 gene is a tumour suppressor gene which, if mutated, can cause many carcinomas like
familial melanoma

Solution for Question 20:


Option B: Infected pancreatic necrosis
• The patient presented 18 days after an initial episode of acute pancreatitis with fever and Epigastric
pain, and CT showing air present within the peri pancreatic fat collection
• Infected necrosis of the Pancreas starts almost after 2 weeks after an acute pancreatitis episode,
which differentiates from SIRS, which occurs after <2 weeks
• Infected pancreatic necrosis Collections associated with necrotizing pancreatitis are sterile, to begin
with, but often become subsequently infected, probably due to translocation of gut bacteria. Infected
necrosis is associated with a mortality rate of up to 50% Infected necrosis should be suspected in
patients with prolonged fever, elevated white blood cell count, or progressive clinical deterioration after
an initial episode of acute pancreatitis. Evidence of air within the peri-pancreatic fat collection seen on a
CT scan confirms the diagnosis Percutaneous drainage of the infected fluid should be carried out with
appropriate antibiotics and supportive care. If the sepsis worsens despite this, then a pancreatic
necrosectomy should be considered
• Collections associated with necrotizing pancreatitis are sterile, to begin with, but often become
subsequently infected, probably due to translocation of gut bacteria. Infected necrosis is associated
with a mortality rate of up to 50%
• Infected necrosis should be suspected in patients with prolonged fever, elevated white blood cell
count, or progressive clinical deterioration after an initial episode of acute pancreatitis. Evidence of air
within the peri-pancreatic fat collection seen on a CT scan confirms the diagnosis
• Percutaneous drainage of the infected fluid should be carried out with appropriate antibiotics and
supportive care. If the sepsis worsens despite this, then a pancreatic necrosectomy should be
considered
• Collections associated with necrotizing pancreatitis are sterile, to begin with, but often become
subsequently infected, probably due to translocation of gut bacteria. Infected necrosis is associated
with a mortality rate of up to 50%

Page 31

483
• Infected necrosis should be suspected in patients with prolonged fever, elevated white blood cell
count, or progressive clinical deterioration after an initial episode of acute pancreatitis. Evidence of air
within the peri-pancreatic fat collection seen on a CT scan confirms the diagnosis
• Percutaneous drainage of the infected fluid should be carried out with appropriate antibiotics and
supportive care. If the sepsis worsens despite this, then a pancreatic necrosectomy should be
considered
Option A: Recurrent acute pancreatitis
• In recurrent acute pancreatitis, focal or diffuse enlargement of the Pancreas, heterogeneous
enhancement of the gland, and blurring of peripancreatic fat. This patient's CT showed air in peri
pancreatic fat
Option C: Pseudocyst
• Pseudocysts are fluid-filled oval or round collections with relatively thick walls. Pseudocysts are easily
identified on CT scans due to these features.
Option D: Systemic inflammatory response syndrome
• SIRS can happen after acute pancreatitis in patients, but it occurs < 2 weeks after an initial episode of
acute pancreatitis. The patient will experience fever and epigastric pain, but extraluminal air in the
peripancreatic fat collection is a feature of infected necrotic Pancreas

Solution for Question 21:


Option B: Somatostatinoma
• The clinical scenario suggestive of somatostatinoma
• Somatostatinoma syndrome is a triad of diabetes mellitus, steatorrhoea, and gallstones, but it also
includes weight loss and hypochlorhydria
• Because somatostatin inhibits pancreatic and biliary secretions, patients with a somatostatinoma
present with gallstones due to bile stasis, diabetes due to inhibition of insulin secretion, and
steatorrhoea due to inhibition of pancreatic exocrine secretion and bile secretion
• Most somatostatinomas originate in the proximal pancreas or the pancreaticoduodenal groove, with
the ampulla and periampullary area as the most common site (60%)
• The most common presentations are abdominal pain, jaundice, and cholelithiasis. Of tumours, 90%
are malignant, with metastasis to the liver or lymph nodes commonly noted at the time of diagnosis.
• They are rarely associated with MEN1 but are associated with Von Recklinghausen disease and
pheochromocytoma
Option A: Glucagonoma

• Glucagonomas are neuroendocrine tumors of the pancreatic islets that secrete glucagon secreted by
alpha cells
• Diabetes in association with dermatitis should raise the suspicion of a glucagonoma.
• The classic necrolytic migratory erythema (due to low levels of amino acids) manifests as cyclic
migrations of lesions with spreading margins and healing centres typically on the lower abdomen,
perineum, perioral area, and feet. Patients also complain of an enlarged sensitive tongue

Page 32

484
• The diagnosis is confirmed by measuring serum glucagon levels, usually >500pg/mL. They are more
often in the body and tail of the pancreas
Option C: VIPoma
• Vasoactive intestinal peptide tumour (VIPoma) syndrome, also called WDHA syndrome, is due to
watery diarrhoea, hypokalemia and achlorhydria. Verner and Morrison described this syndrome. They
are most commonly located in the distal pancreas
Option D: Insulinoma
• Insulinomas are the most common functional pancreatic endocrine neoplasms and present with a
typical clinical syndrome known as Whipple's triad
• The triad consists of
• Symptomatic fasting hypoglycemia
• Serum glucose level <50 mg/dL
• Relief of symptoms with the administration of glucose
• Serum insulin and C peptide levels are elevated. Most insulinomas are benign and solitary, and 10%
are malignant. They are typically cured by simple enucleation

Solution for Question 22:


Option A: Antibiotics and percutaneous catheter drainage
• The clinical case suggests that the pancreatic pseudocyst is infected. Treatment of this involves
percutaneous drainage and antibiotics
• Pseudocyst of Pancreas A pseudocyst is a chronic collection of pancreatic amylase-rich fluid
surrounded by a non-epithelialized wall of granulation tissue and fibrosis. They comprise the most
common complication of chronic pancreatitis Formation of a pseudocyst requires 4 weeks or more from
the onset of acute pancreatitis. Pseudocysts usually cause symptoms of pain, fullness, or early satiety.
The fluid from a pseudocyst typically has a low CEA level, high amylase level, and absence of mucin. A
pseudocyst is usually identified on ultrasound or a CT scan Infected pseudocyst should be suspected in
patients with prolonged fever, elevated white blood cell count, or progressive clinical deterioration
Percutaneous drainage with the widest possible drains placed under imaging guidance is the treatment,
along with appropriate antibiotics, and supportive care. Occasionally, open drainage may be necessary
• A pseudocyst is a chronic collection of pancreatic amylase-rich fluid surrounded by a
non-epithelialized wall of granulation tissue and fibrosis. They comprise the most common complication
of chronic pancreatitis
• Formation of a pseudocyst requires 4 weeks or more from the onset of acute pancreatitis.
Pseudocysts usually cause symptoms of pain, fullness, or early satiety. The fluid from a pseudocyst
typically has a low CEA level, high amylase level, and absence of mucin. A pseudocyst is usually
identified on ultrasound or a CT scan
• Infected pseudocyst should be suspected in patients with prolonged fever, elevated white blood cell
count, or progressive clinical deterioration
• Percutaneous drainage with the widest possible drains placed under imaging guidance is the
treatment, along with appropriate antibiotics, and supportive care. Occasionally, open drainage may be
necessary

Page 33

485
• A pseudocyst is a chronic collection of pancreatic amylase-rich fluid surrounded by a
non-epithelialized wall of granulation tissue and fibrosis. They comprise the most common complication
of chronic pancreatitis
• Formation of a pseudocyst requires 4 weeks or more from the onset of acute pancreatitis.
Pseudocysts usually cause symptoms of pain, fullness, or early satiety. The fluid from a pseudocyst
typically has a low CEA level, high amylase level, and absence of mucin. A pseudocyst is usually
identified on ultrasound or a CT scan
• Infected pseudocyst should be suspected in patients with prolonged fever, elevated white blood cell
count, or progressive clinical deterioration
• Percutaneous drainage with the widest possible drains placed under imaging guidance is the
treatment, along with appropriate antibiotics, and supportive care. Occasionally, open drainage may be
necessary
Option B: CT-guided aspiration with repeat imaging in 2 to 3 days
• CT- guided aspiration is not done in the treatment of pseudocyst, and rather drainage with a
CT-guided or ultrasound-guided placement of a catheter to drain the pseudocyst
Option C: Antibiotics and CT-guided aspiration with repeat imaging in 2 to 3 days
• For a pseudocyst or pancreatic abscess, we need to drain it with a catheter. CT aspiration is not done
in the treatment of pseudocyst as it will not be able to drain the fluid properly and there can be fluid
extravasation which can cause further complications
Option D: Antibiotic therapy alone
• Antibiotic therapy alone will not be beneficial as antibiotics cannot pass the thick walls of the
pseudocyst, and it has to be drained first and then only antibiotics can perform their function

Solution for Question 23:


Correct Option C - The annular pancreas is located proximal to the ampulla:
• The annular pancreas is a congenital anomaly where a ring of pancreatic tissue surrounds the second
part of the duodenum. However, it is not located proximal to the ampulla but rather encircles the
duodenum.

Incorrect Options:
Option A - The ring of the ventral pancreas is present around the 2nd part of the Duodenum:
The annular pancreas involves a ring of pancreatic tissue around the second part of the duodenum.
Option B - There are bilobed ventral pancreatic buds:
In the case of the annular pancreas, there are bilobed ventral pancreatic buds.
Option D - It is associated with Down Syndrome and Duodenal Atresia:
Annular pancreas is associated with Down Syndrome and Duodenal Atresia.

Solution for Question 24:

Page 34

486
Correct Option D - B only:
• The statement "There is a retroperitoneal bleed into the facial planes" is incorrect. In the described
clinical scenario, the presence of a retroperitoneal bleed into the facial planes is not a characteristic
feature of the condition. Instead, it suggests an alternative pathology such as retroperitoneal
hemorrhage.
Explanation of correct statements:
A. It is associated with left-sided pleural effusion (Correct): This statement is correct. Severe cases of a
cute pancreatitis can lead to complications like left-sided pleural effusion.
C. Cullen sign, ecchymosis around the umbilicus, is a characteristic finding (Correct): This statement is
correct. Cullen sign is associated with hemorrhagic pancreatitis.
D. Grey Turner sign, ecchymosis around the flank, is observed in this condition (Correct): This stateme
nt is correct. Grey Turner sign is also associated with hemorrhagic pancreatitis.
E. Fox sign, ecchymosis around the inguinal region, is a common manifestation (Correct): This stateme
nt is correct. Fox sign is another manifestation of hemorrhagic pancreatitis.
Top of Form

Incorrect Options:
Option A – A, B, C: This option is incorrect as it contain two correct statements as statement A and C.
Option B - B, C: This option is incorrect as it contain correct statement as statement C.
Option C - A only: This option is incorrect as it contain correct statement as statement A.

Solution for Question 25:


Correct Option A -BISAP score of 3, indicative of severe acute pancreatitis with >15% mortality:
• The BISAP score is calculated based on the parameters provided (BUN, impaired mental status,
SIRS criteria, age, and pleural effusion). In this case, the patient has a BUN > 25 mg/dL, elevated heart
rate, increased respiratory rate, and an elevated WBC count, resulting in a BISAP score of 3. According
to the information given, a BISAP score of 3 is suggestive of severe acute pancreatitis with >15%
mortality.

Incorrect Options:
Option B - BISAP score of 2, suggesting a
mild form of acute pancreatitis with <2% mortality: Incorrect because the given patient has a
BISAP score of 3, indicating severe acute pancreatitis with >15% mortality.
Option C - BISAP score of 4, indicative of severe acute pancreatitis with >15% mortality: The correct BI
SAP score is 3, not 4, and it suggests severe acute pancreatitis with >15% mortality.
Option D - BISAP score of 1, suggesting a moderate form of acute pancreatitis with 2-15% mortality: T
he patient's BISAP score is 3, indicating severe acute pancreatitis, not a moderate form.

Page 35

487
Solution for Question 26:
Correct Option B - ERCP is both diagnostic and therapeutic and shows a
String of Pearls or Chains of Lakes Appearance on imaging:
• ERCP (Endoscopic Retrograde Cholangiopancreatography) is considered both diagnostic and
therapeutic in chronic pancreatitis. It involves the injection of contrast directly into the pancreatic ducts,
allowing visualization of the ductal system. The characteristic findings of chronic pancreatitis on ERCP
include alternate stricture and dilatation of the pancreatic duct, along with the presence of pancreatic
stones. This appearance is often described as a String of Pearls or Chains of Lakes Appearance or
Beaded Appearance.

Incorrect Options:
Option A - MRCP is the Gold Standard investigation for chronic pancreatitis: MRCP (Magnetic Resona
nce Cholangiopancreatography) is a valuable non-invasive imaging modality for visualizing the pancre
atic ducts, but it is not considered the Gold Standard for the diagnosis of chronic pancreatitis. The Gold
Standard is ERCP.
Option C - Endoscopic ultrasound is not useful in diagnosing minimal changes in chronic pancreatitis:
Endoscopic ultrasound is, in fact, highly useful in diagnosing minimal changes in chronic pancreatitis. It
is considered the most accurate investigation for detecting subtle abnormalities in the pancreatic pare
nchyma and ducts.
Option D - ROSEMONT criteria is based on MRCP findings for the diagnosis of chronic pancreatitis: Th
e ROSEMONT criteria are based on endoscopic ultrasound findings, not MRCP. The ROSEMONT crit
eria are used to standardize the interpretation of endoscopic ultrasound findings for the diagnosis of ch
ronic pancreatitis.

Solution for Question 27:


Correct Option D - Peutz-Jeghers Syndrome:
• Peutz-Jeghers Syndrome is an autosomal dominant genetic disorder characterized by the presence of
mucocutaneous pigmentation and gastrointestinal hamartomatous polyps.
• Individuals with Peutz-Jeghers Syndrome are at an increased risk of developing various malignancies,
including pancreatic carcinoma.
• The hamartomatous polyps in the gastrointestinal tract, along with other factors, contribute to the
elevated risk of malignancy in these patients.

Incorrect Options:
Option A - Hereditary Breast and Ovarian Cancer (associated with BRCA-2) – Autosomal dominant:
• While BRCA-2 mutations are associated with an increased risk of breast and ovarian cancers, they
are not specifically linked to pancreatic carcinoma.
• The primary focus of BRCA-2 mutations is on breast and ovarian tissues, making this option incorrect
for pancreatic carcinoma.
Option B - Ataxia Telangiectasia – Autosomal recessive:

Page 36

488
• Ataxia Telangiectasia is an autosomal recessive disorder characterized by cerebellar degeneration,
immunodeficiency, and an increased risk of malignancies, particularly lymphoid tumors.
• However, it is not strongly associated with pancreatic carcinoma, making it an incorrect option in this
context.
Option C - Familial atypical multiple mole melanoma (FAMMM) syndrome - Autosomal dominant:
• FAMMM syndrome, associated with CDKN2A mutations, is primarily linked to an increased risk of
melanoma and pancreatic carcinoma.
• However, it is not strongly associated with pancreatic carcinoma, making it an incorrect option in this
context.

Page 37

489
Previous Year Questions
1. A 40 year old woman presented with jaundice, fever, and abdominal pain. She is suspected to have
cholangitis. Which of the following apart from abdominal pain makes up Reynold pentad? Fever
Jaundice Hypotension Altered sensorium Hematemesis Melena
A. 1, 2, 3, 4
B. 2, 3, 4, 5
C. 1, 3, 4, 5
D. 1, 2, 4, 5
----------------------------------------
2. What is the optimal management approach for a patient presenting with fever and abdominal pain
five days after experiencing a common bile duct injury resulting in a leak?
A. Ultrasound–guided aspiration
B. ERCP and stenting
C. Re-exploration and hepaticojejunostomy
D. Re-exploration and primary repair
----------------------------------------
3. Which of the following represents a type III choledochal cyst from the given image?

A. A
B. B
C. C
D. D
----------------------------------------
4. Which neuroendocrine tumor of the pancreas is commonly linked with necrolytic erythema migrans?
A. Insulinoma
B. Glucagonoma
C. Somatostatinoma
D. VIPoma
----------------------------------------

490
5. Which of the following is not true about Infantile hypertrophic pyloric stenosis ?
A. Presents with non-bilious vomiting
B. Caused by hypertrophy of longitudinal muscles
C. USG is very sensitive in diagnosing this condition
D. Hypokalemic metabolic alkalosis seen
----------------------------------------
6. A patient presented with fever and abdominal pain with jaundice. Investigations showed the given
findings. Which of the following statements is true regarding this condition?

A. Surgery is mandatory
B. Fine-needle aspiration cytology is diagnostic
C. Angioembolization is the treatment of choice
D. Echinococcus species is involved in etiology
----------------------------------------
7. A patient presented with features of chronic pancreatitis with recurrent attacks and has a 10 mm
dilatation of the pancreatic duct with intraductal calculi present. Which of the following is the surgery of
choice?
A. Pancreaticoduodenectomy
B. Longitudinal pancreaticojejunostomy
C. ERCP and sphincterotomy
D. Coring of pancreas head
----------------------------------------
8. A patient arrives with pain in the right upper abdomen area. One week ago, the patient experienced
a bout of diarrhea. Abdominal CT scan results indicate the presence of a liver abscess measuring
approximately 25 cubic centimeters. What should be the subsequent course of action in treating this
patient?
A. PAIR
B. Surgery
C. Percutaneous Catheter drainage + Systemic Antibiotics
D. Percutaneous drainage

Page 2

491
----------------------------------------
9. Which of the following statements is false regarding acute cholecystitis?
A. Gall bladder thickness >4 mm on USG
B. Murphy’s sign positive
C. Leukocytosis
D. Gold standard investigation is CT scan
----------------------------------------
10. What is the most likely diagnosis for a 50-year-old patient who is an alcoholic and has been
experiencing intermittent episodes of jaundice for the past 2 months, with CT scan findings showing an
enlargement of both the common bile duct and pancreatic duct?
A. Cholangiocarcinoma
B. Periampullary carcinoma
C. Common bile duct stone
D. Head of the pancreas carcinoma
----------------------------------------
11. The following procedure is performed for the management of?

A. Advanced gastric carcinoma


B. Gallbladder carcinoma
C. Distal cholangiocarcinoma
D. Chronic calcific pancreatitis
----------------------------------------
12. What is true about gall bladder carcinoma?
A. Palpable abdominal lump is the earliest sign
B. Earliest sign is jaundice
C. Clinical presentation of gallstones can be distinguished from gallbladder cancer
D. Median survival is 6 months
----------------------------------------

Page 3

492
13. What are the components of the treatment for a patient who has presented with a substantial
buildup of fluid in the abdomen and has been diagnosed with pancreatic necrosis? Antibiotics IV IV
fluids Culture MRI Assisted drainage Laparotomy
A. 1, 2, 3, 4 and 5
B. 2, 4, 5 and 6
C. 1, 2, 3 and 5
D. 1, 2, 3, 4, 5 and 6
----------------------------------------
14. Which condition shown in the MRCP image?

A. Stricture
B. Choledocholithiasis
C. Cholecystitis
D. Cholangiocarcinoma
----------------------------------------
15. What is the sign of the gall bladder seen in this USG?

A. Phrygian cap sign


B. Murphy sign
C. Claw sign
D. Omega sign
----------------------------------------

Page 4

493
16. After lap cholecystectomy what should be the urine output of the patient if the renal function of the
patient is normal?
A. 0.1 ml/hr
B. 0.5 ml/min
C. 0.5-1.5 ml/kg/min
D. 1 ml/kg/min
----------------------------------------
17. A 35-year-old patient with a history of dysentery for 6 weeks complains of pain in the right upper
abdomen. On ultrasound, a hypoechoic hyperdense lesion is seen in the liver. Identify the condition
based on the given image?

A. Amoebic liver abscess


B. Pyogenic liver abscess
C. Hydatid cyst
D. Hemangioma
----------------------------------------
18. A patient with a history of alcoholism presented with intense pain in the abdomen. The medical
team suspected pancreatitis and conducted a CT scan, which showed abnormal areas with fluid
accumulation near the pancreas. Among the following enzymes, which one is expected to show
increased levels in this scenario? GGT - Gamma-glutamyltransferase CEA - Carcinoembryonic antigen
A. Serum GGT
B. CEA
C. Serum lipase
D. Serum bilirubin
----------------------------------------
19. In a 32-year-old male, who complained of fever and pain in the upper right abdomen following
meals, the recommended diagnostic test is
A. CECT
B. USG
C. MRI
D. HRCT

Page 5

494
----------------------------------------
20. What triad is characterized by abdominal pain, fever, and jaundice?
A. Saint triad
B. Charcot’s triad
C. Virchow triad
D. Reynold's Pentad
----------------------------------------
21. What should be the subsequent course of action in managing an 80-year-old female patient who
has reported fatigue and has been diagnosed with a scleroatrophic gallbladder containing impacted
stones? Additionally, a large stone is observed in the common bile duct, resulting in dilation of the
biliary tree.
A. Immediately take up patient for cholecystectomy
B. Endoscopic retrograde cholangiopancreatography
C. Wait and watch
D. CT scan
----------------------------------------
22. A 35-year-old man presented with recurrent episodes of fever with chills. The MRCP image is
shown below. What is the most likely diagnosis?

A. Primary sclerosing cholangitis


B. Caroli disease
C. Oriental cholangitis
D. Primary biliary carcinoma
----------------------------------------
23. A 5-year-old child with Wilson's disease is being considered for liver transplantation. Which of the
following is not a parameter in the selection criteria for transplantation?
A. AST
B. PT
C. Bilirubin
D. ALT

Page 6

495
----------------------------------------
24. What is the most frequently encountered neuroendocrine tumor in the pancreas?
A. Gastrinoma
B. Somatostatinoma
C. Insulinoma
D. VIPoma
----------------------------------------
25. A patient underwent laparoscopic cholecystectomy and was discharged after removing the drain
tube on the first postoperative day. On day two, the patient came back to the hospital with complaints of
increasing abdominal pain with tachycardia and was febrile on examination . On ultrasound
examination, there was a 5 x 5 cm collection in the subhepatic space. What is the most likely cause of
this condition?
A. Infection
B. Surgical site pain
C. Bile leak
D. Retained gallstones
----------------------------------------
26. What would be your approach in managing a patient who is a 40-year-old woman with a 5 mm
gallbladder polyp detected on abdominal ultrasound, but no gallbladder stones? The patient is overall
healthy and shows no symptoms.
A. Regular follow-up every 2 months
B. Polypectomy
C. Laparoscopic cholecystectomy
D. Open cholecystectomy
----------------------------------------
27. Which of the following is the most common side effect of cholecystectomy?
A. Acute kidney injury
B. Osteoporosis
C. Steatorrhea
D. Colorectal cancer
----------------------------------------
28. What is the probable diagnosis for a young man who visits the emergency department due to
intense abdominal pain, accompanied by three incidents of vomiting within the past nine hours? The
patient reports that the pain extends to the back and is partly alleviated by bending forward. Attached
below is an abdominal radiograph of the patient displaying a 'colon cut-off sign'.

Page 7

496
A. Acute hepatitis
B. Acute pancreatitis
C. Acute colitis
D. Intestinal obstruction
----------------------------------------
29. What is the most appropriate next step in the management of a female patient who has abdominal
pain, multiple gallstones with no thickening of the gallbladder wall as seen on ultrasound, a common
bile duct diameter of 12mm, serum bilirubin level of 0.8, alkaline phosphatase level of 380, and a 5-fold
increase in GGT?
A. MRCP
B. ERCP
C. Endoscopic ultrasound
D. Laparoscopic cholecystectomy
----------------------------------------
30. Which of the following statements about hepatocellular carcinoma (HCC) is/are correct?
TACE(Transarterial chemoembolization) is used for multinodular lesions Lenvatinib is used for lesions
less than 3 cm The incidence of HCC is increasing NASH and NAFLD are risk factors for HCC
A. 1, 2, 3
B. 2, 3, 4
C. 1, 3, 4
D. 1 and 2
----------------------------------------
31. What is the most helpful test to identify the underlying cause in a 56-year-old patient with
obstructive jaundice, who presents with a distended gall bladder, dilated common bile duct, and
enlarged intrahepatic biliary radicles on ultrasound?
A. Magnetic resonance imaging
B. Endoscopic ultrasound
C. Percutaneous transhepatic cholangiography
D. Endoscopic retrograde cholangiopancreatography

Page 8

497
----------------------------------------
32. What should be the subsequent course of action in the management of a female patient who
experienced abdominal pain and increased heart rate three days after cholecystectomy? Her white
blood cell count is 11000 cells / mm^3, and an ultrasound examination revealed a 5x5 cm accumulation
of fluid in the sub-hepatic space on the right side.
A. Antibiotics and send home
B. USG-guided pigtail catheter drainage
C. Laparoscopy and re-exploration
D. ERCP and T-tube placement
----------------------------------------
33. The instrument shown in the image is used to insufflate the abdomen for laparoscopic surgery.
What is its name?

A. Veress needle
B. Lind’s needle
C. Jacob needle
D. Mathew needle
----------------------------------------
34. What is the primary etiology of chronic pancreatitis, commonly observed in clinical practice?
A. Gallstones
B. Alcohol
C. Autoimmune
D. Drugs
----------------------------------------
35. What is the most effective technique for procuring tissue for the purpose of diagnosing pancreatic
carcinoma?
A. Transgastric FNAC guided by endoscopic ultrasound
B. MRI - guided biopsy
C. Laparoscopic - guided biopsy
D. Percutaneous transperitoneal biopsy

Page 9

498
----------------------------------------
36. ERCP is least useful in the diagnosis of
A. Recurrent pancreatitis
B. Unexplained jaundice
C. Periampullary mass
D. Acute Pancreatitis without choledocholithiasis
----------------------------------------
37. A 40-year-old man is brought to the casualty with severe abdominal pain radiating to the back for
the past 1 day and has had 3 episodes of vomiting. The pain radiates to the back and is slightly relieved
in the sitting forward position. The following signs are seen on examination and lab investigations
reveal elevated lipase and amylase. Which of the following is the most likely diagnosis?

A. Acute appendicitis
B. Acute cholecystitis
C. Acute pancreatitis
D. Acute ischemic colitis
----------------------------------------
38. Identify the procedure shown in the image:

A. Beger’s procedure
B. Whipple’s procedure
C. Frey’s procedure

Page 10

499
D. Pancreatic redivision
----------------------------------------
39. What is the procedure being depicted in the image provided?

A. Gastrojejunostomy
B. Pancreaticojejunostomy
C. Pancreatic Ileostomy
D. Duodenojejunostomy
----------------------------------------
40. A 36-year-old man presents with dull aching epigastric pain. He gives a history of acute abdominal
pain following a binge of alcohol 5 weeks back, which was managed conservatively. On examination,
he appeared icteric, and a firm mass was palpable in the epigastrium. His CT scan is given below.
Which of the following is the most likely diagnosis?

A. Pancreatic pseudocyst
B. Large choledochal cyst
C. Gastric duplication cyst
D. Mesenteric cyst
----------------------------------------
41. What is the most probable diagnosis for a 42-year-old woman who came with symptoms of fever,
vomiting, right hypochondriac pain, which radiates to the neck? She has experienced similar episodes
in the past and a positive Murphy's sign was observed during examination.
A. Acute cholecystitis

Page 11

500
B. Carcinoma gallbladder
C. Peptic ulcer
D. Acute appendicitis
----------------------------------------
42. Which of the following is tumor marker of pancreatic cancer?
A. NSE
B. Alpha-Fetoprotein
C. Ca 15-3
D. CA 19-9
----------------------------------------
43. A young sheep farmer presents to the OPD with pain in the right upper part of the abdomen. Upon
examination, there was tenderness and enlargement of the liver. Blood investigations show
eosinophilia, elevated liver enzymes, and abdominal ultrasound, as shown below. Which of the
following is the most likely diagnosis?

A. Amoebic liver abscess


B. Hydatid cyst
C. Pyogenic liver abscess
D. Hepatocellular carcinoma
----------------------------------------
44. A factory worker working in a plastic pipe factory presented to the OPD with weight low and right
upper quadrant abdominal pain of 6 months duration. He had hepatomegaly. Further investigation
revealed that most of the liver was replaced by a hemorrhagic tumor with extensive necrosis. What can
be the likely diagnosis in this case?
A. Cavernous hemangioma
B. Metastatic carcinoma of the liver
C. Angiosarcoma of the liver
D. Primary hepatocellular carcinoma
----------------------------------------
45. A patient underwent a laparoscopic cholecystectomy for cholelithiasis. A picture of the resected gall
bladder is shown below. What is the diagnosis?

Page 12

501
A. Gall bladder carcinoma
B. Mixed stone in gall bladder
C. Cholecystitis
D. Choledocholithiasis
----------------------------------------
46. Which of the following is the least helpful in diagnosing biliary dyskinesia in a patient presenting
with recurring pain in the right hypochondrium, along with symptoms of nausea and vomiting?
A. ERCP
B. HIDA scan
C. USG
D. Post surgery follow-up
----------------------------------------
47. A chronic alcoholic patient came to the emergency with pain in the epigastrium radiating to the back
and recurrent vomiting. On examination, guarding was present in the upper epigastrium. The erect
chest X-ray was normal. What is the next step?
A. Upper Gl endoscopy
B. Alcohol breath test
C. Serum lipase
D. CECT
----------------------------------------
48. The most common neuroendocrine tumor of the pancreas is:
A. Gastrinoma
B. Somatostatinoma
C. Insulinoma
D. VIPoma
----------------------------------------
49. In accordance with Strasberg's classification of bile duct injuries, what is thestage?

Page 13

502
A. Type A
B. Type B
C. Type C
D. Type D
----------------------------------------
50. What is the most appropriate test to diagnose a suspected bile leak in a patient who presents with
pain in the right upper quadrant on the second day after undergoing a cholecystectomy, and has a
collection in the subhepatic space?
A. MRCP
B. ERCP
C. HIDA
D. CECT
----------------------------------------
51. What is the term for a stone obstructing the cystic duct, causing dilation of the common hepatic
duct, resulting in CBD obstruction and jaundice?
A. Acute cholecystitis
B. Porcelain gallbladder
C. Cholesterolosis
D. Mirizzi’s syndrome
----------------------------------------
52. What is the preferred management of the condition in the image below?

Page 14

503
A. Surgical excision
B. Radiotherapy
C. Antibiotics
D. Intralesional steroids
----------------------------------------
53. For which of the following conditions is AFP a tumor marker?
A. HCC
B. RCC
C. Oncocytoma
D. Chordoma
----------------------------------------
54. A 43-year-old female presents to the clinic with a complaint of recurrent right upper quadrant pain
complaint. They exacerbate, especially after eating. An ultrasound of the abdomen is performed, which
comes out to be non-conclusive. Which of the following procedures is indicated in the image below?

A. Percutaneous transhepatic cholangiogram


B. T-tube cholangiogram
C. Endoscopic retrograde cholangiopancreatography (ERCP)
D. Hepatobiliary iminodiacetic acid (HIDA) scan
----------------------------------------

Page 15

504
Correct Answers
Question Correct Answer

Question 1 1
Question 2 2
Question 3 3
Question 4 2
Question 5 2
Question 6 4
Question 7 2
Question 8 3
Question 9 4
Question 10 2
Question 11 3
Question 12 4
Question 13 3
Question 14 2
Question 15 1
Question 16 3
Question 17 1
Question 18 3
Question 19 2
Question 20 2
Question 21 2
Question 22 1
Question 23 4
Question 24 3
Question 25 3
Question 26 1
Question 27 3
Question 28 2
Question 29 1
Question 30 3
Question 31 1
Question 32 2

Page 16

505
Question 33 1
Question 34 2
Question 35 1
Question 36 4
Question 37 3
Question 38 2
Question 39 2
Question 40 1
Question 41 1
Question 42 4
Question 43 2
Question 44 3
Question 45 1
Question 46 3
Question 47 3
Question 48 3
Question 49 3
Question 50 2
Question 51 4
Question 52 1
Question 53 1
Question 54 3

Solution for Question 1:


Correct Option A - 1, 2, 3, 4:
Reynold pentad is a clinical syndrome associated with severe cholangitis, which is an inflammation and
infection of the bile ducts. It consists of the following five components:
Incorrect Options
Options B, C, and D
are incorrect because they include additional symptoms that are not part of Reynold pentad

Solution for Question 2:


Correct Option: B - ERCP and stenting:
• ERCP (Endoscopic Retrograde Cholangiopancreatography) with stenting is the optimal management
approach for a patient presenting with fever and abdominal pain five days after experiencing a common

Page 17

506
bile duct injury resulting in a leak. ERCP: This procedure allows direct visualization of the bile ducts
and enables interventions such as stent placement to address the bile duct injury. ERCP is a minimally
invasive procedure performed by passing an endoscope through the mouth, esophagus, and stomach
into the duodenum to access the bile ducts. Stenting: Stents are inserted during ERCP to facilitate bile
drainage and promote healing of the bile duct injury. They also help maintain the patency of the duct
and prevent further complications such as bile leakage and bile peritonitis.
• ERCP: This procedure allows direct visualization of the bile ducts and enables interventions such as
stent placement to address the bile duct injury. ERCP is a minimally invasive procedure performed by
passing an endoscope through the mouth, esophagus, and stomach into the duodenum to access the
bile ducts.
• Stenting: Stents are inserted during ERCP to facilitate bile drainage and promote healing of the bile
duct injury. They also help maintain the patency of the duct and prevent further complications such as
bile leakage and bile peritonitis.
• ERCP: This procedure allows direct visualization of the bile ducts and enables interventions such as
stent placement to address the bile duct injury. ERCP is a minimally invasive procedure performed by
passing an endoscope through the mouth, esophagus, and stomach into the duodenum to access the
bile ducts.
• Stenting: Stents are inserted during ERCP to facilitate bile drainage and promote healing of the bile
duct injury. They also help maintain the patency of the duct and prevent further complications such as
bile leakage and bile peritonitis.
Incorrect Options:
Option A -
Ultrasound-guided aspiration: While ultrasound-guided aspiration may be useful for draining a biloma (
collection of bile outside the biliary system), it does not address the underlying bile duct injury. Therefor
e, it is not the definitive management approach for this condition.
Option C - Re-exploration and hepaticojejunostomy: Re-exploration and hepaticojejunostomy involve s
urgical intervention to repair the bile duct injury by creating a new connection between the bile duct and
the jejunum. While this approach may be necessary in some cases, it is usually reserved for more co
mplex or severe injuries. In the presented case, less invasive options such as ERCP with stenting are
preferred before resorting to surgical intervention.
Option D - Re-exploration and primary repair: Similar to option C, re-exploration and primary repair inv
olve surgical intervention to directly repair the bile duct injury. This approach may be considered if less
invasive options fail or if the injury is severe. However, it is not the initial management approach for a
patient presenting with fever and abdominal pain after a common bile duct injury.

Solution for Question 3:


Correct Option C - C:
• Choledochal cysts are rare congenital anomalies that involve dilation of the biliary tree. These cysts
can occur anywhere along the biliary tree, from the intrahepatic bile ducts to the common bile duct.
Based on the location and extent of the dilation, choledochal cysts are classified into five types.
• Image C shows a Dilatation of the intraduodenal part of the bile duct (choledochocele); it represents a
type III choledochal cyst.
Choledochal cysts are rare congenital anomalies that involve dilation of the biliary tree. These cysts ca
n occur anywhere along the biliary tree, from the intrahepatic bile ducts to the common bile duct. Base

Page 18

507
d on the location and extent of the dilation, choledochal cysts are classified into five types.
Image C shows a Dilatation of the intraduodenal part of the bile duct (choledochocele); it represents a
type III choledochal cyst.
Incorrect Options:
Option A - A: Type II cyst is Diverticular dilatation of the extrahepatic bile duct.
Option B - B: Type I cyst is Cystic dilatation of extrahepatic bile duct (most common)
Option D - D: Type V cyst involves multiple dilations of the intrahepatic and/or extrahepatic bile ducts.

Solution for Question 4:


Correct Option B - Glucagonoma:
• Necrolytic Erythema Migrans (NEM): This is a characteristic skin manifestation associated with
glucagonoma, also known as the hyperglycemic cutaneous syndrome. NEM presents as a rash with
erythema and necrosis, typically affecting the lower extremities and perineum. It's considered the most
common symptom of glucagonoma.
Necrolytic Erythema Migrans (NEM): This is a characteristic skin manifestation associated with glucago
noma, also known as the hyperglycemic cutaneous syndrome. NEM presents as a rash with erythema
and necrosis, typically affecting the lower extremities and perineum. It's considered the most common
symptom of glucagonoma.

• "4D" Syndrome: Glucagonoma is associated with a set of symptoms known as the "4D" syndrome:
Diabetes Mellitus: Glucagonoma can lead to elevated blood glucose levels due to the excess release of
glucagon, which opposes insulin action, contributing to the development of diabetes mellitus.
Dermatitis: NEM, as described earlier, is the dermatological manifestation characterized by erythema
and necrosis. Deep Vein Thrombosis (DVT): Glucagonoma can increase the risk of DVT formation due
to various factors such as hypercoagulability. Depression: Patients with glucagonoma may experience
depression, which could be attributed to various physiological and psychological factors associated with
the disease.

Page 19

508
• Diabetes Mellitus: Glucagonoma can lead to elevated blood glucose levels due to the excess release
of glucagon, which opposes insulin action, contributing to the development of diabetes mellitus.
• Dermatitis: NEM, as described earlier, is the dermatological manifestation characterized by erythema
and necrosis.
• Deep Vein Thrombosis (DVT): Glucagonoma can increase the risk of DVT formation due to various
factors such as hypercoagulability.
• Depression: Patients with glucagonoma may experience depression, which could be attributed to
various physiological and psychological factors associated with the disease.
"4D" Syndrome: Glucagonoma is associated with a set of symptoms known as the "4D" syndrome:
• Diabetes Mellitus: Glucagonoma can lead to elevated blood glucose levels due to the excess release
of glucagon, which opposes insulin action, contributing to the development of diabetes mellitus.
• Dermatitis: NEM, as described earlier, is the dermatological manifestation characterized by erythema
and necrosis.
• Deep Vein Thrombosis (DVT): Glucagonoma can increase the risk of DVT formation due to various
factors such as hypercoagulability.
• Depression: Patients with glucagonoma may experience depression, which could be attributed to
various physiological and psychological factors associated with the disease.
Diabetes Mellitus: Glucagonoma can lead to elevated blood glucose levels due to the excess release o
f glucagon, which opposes insulin action, contributing to the development of diabetes mellitus.
Dermatitis: NEM, as described earlier, is the dermatological manifestation characterized by erythema a
nd necrosis.
Deep Vein Thrombosis (DVT): Glucagonoma can increase the risk of DVT formation due to various fac
tors such as hypercoagulability.
Depression: Patients with glucagonoma may experience depression, which could be attributed to vario
us physiological and psychological factors associated with the disease.
Incorrect Options:
Option A - Insulinoma:
• Insulinomas are neuroendocrine tumors of the pancreas that primarily secrete insulin.
• While hypoglycemia is a hallmark symptom of insulinoma due to excess insulin secretion, it is not
associated with necrolytic erythema migrans.
Insulinomas are neuroendocrine tumors of the pancreas that primarily secrete insulin.
While hypoglycemia is a hallmark symptom of insulinoma due to excess insulin secretion, it is not asso
ciated with necrolytic erythema migrans.
Option C - Somatostatinoma:
• Somatostatinomas are rare neuroendocrine tumors that overproduce somatostatin.
• Dermatological manifestations associated with somatostatinoma typically include acanthosis nigricans
and diabetes mellitus, but not necrolytic erythema migrans.
Somatostatinomas are rare neuroendocrine tumors that overproduce somatostatin.
Dermatological manifestations associated with somatostatinoma typically include acanthosis nigricans
and diabetes mellitus, but not necrolytic erythema migrans.
Option D - VIPoma:

Page 20

509
• VIPomas are neuroendocrine tumors that secrete vasoactive intestinal peptide (VIP), leading to
profuse watery diarrhea, hypokalemia, and achlorhydria (WDHA syndrome).
• While VIPomas can lead to skin flushing, diarrhea is the prominent symptom, and necrolytic erythema
migrans is not associated with VIPomas.
VIPomas are neuroendocrine tumors that secrete vasoactive intestinal peptide (VIP), leading to profus
e watery diarrhea, hypokalemia, and achlorhydria (WDHA syndrome).
While VIPomas can lead to skin flushing, diarrhea is the prominent symptom, and necrolytic erythema
migrans is not associated with VIPomas.

Solution for Question 5:


Correct Option B - Caused by hypertrophy of longitudinal muscles:
• Infantile hypertrophic pyloric stenosis (IHPS) is characterized by hypertrophy of the circular muscle
fibers at the pylorus, which causes constriction.
• Contrary to option B, hypertrophy primarily affects the circular muscles rather than the longitudinal
muscles.
• The hypertrophy of the circular muscle fibers leads to the narrowing of the pyloric canal, resulting in
the classical clinical features of IHPS.
Incorrect Options:
Option A - Presents with non-bilious vomiting: This statement is true. Non-bilious vomiting is a hallmark
symptom of IHPS. The vomitus is typically non-bilious because the obstruction occurs proximal to the
duodenum, preventing the bile from entering the stomach.
Option C - USG is very sensitive in diagnosing this condition: This statement is true. Ultrasonography (
USG) is the investigation of choice for diagnosing IHPS. It offers high sensitivity and specificity in visual
izing the hypertrophied pyloric muscle. The diagnostic criteria include pyloric muscle thickness >4mm,
channel length >16mm, and transverse diameter >13mm.
Option D - Hypokalemic metabolic alkalosis seen: This statement is true. IHPS leads to electrolyte imb
alances due to recurrent vomiting, including hypokalemia, hypochloremia, and metabolic alkalosis. Los
s of potassium (K+) and chloride (Cl-) ions through vomiting results in hypokalemia and hypochloremia,
while loss of hydrogen ions (H+) leads to metabolic alkalosis. Paradoxical aciduria may also be observ
ed in severe cases, where the body excretes H+ in the urine despite metabolic alkalosis.

Solution for Question 6:


Correct Option D - Echinococcus species is involved in etiology:
• The patient's symptoms of fever, abdominal pain, and jaundice, combined with the provided findings
of the serpentine linear membrane, which is a wavy membrane within the cyst, suggest a diagnosis of
hydatid disease or echinococcosis . This disease is caused by infection with the Echinococcus species
of tapeworm, often through contact with dogs or livestock.
Incorrect Choices:

Page 21

510
Option A - Surgery is mandatory: While surgical removal of hydatid cysts can be an effective treatment,
it is not always mandatory. In some cases, non-surgical treatments such as antiparasitic medications o
r percutaneous drainage may be appropriate.
Option B - Fine-needle aspiration cytology is diagnostic: This method is usually avoided in diagnosing
echinococcosis due to the risk of anaphylaxis and spreading the infection.
Option C - Angioembolization is the treatment of choice: This is not generally the first-line treatment for
echinococcosis.

Solution for Question 7:


Option B - Longitudinal pancreaticojejunostomy:
• In a patient with chronic pancreatitis, recurrent attacks, and significant dilatation of the pancreatic duct
with intraductal calculi, the surgery of choice is a longitudinal pancreaticojejunostomy, also known as
the Puestow procedure. This procedure decompresses the duct and helps alleviate pain, making it the
most appropriate choice in this scenario.
Incorrect Options:
Option A - Pancreaticoduodenectomy: This is a major surgical procedure, also known as the Whipple p
rocedure, usually reserved for pancreatic head tumors. It is not typically the first choice for chronic pan
creatitis with intraductal calculi.
Option C - ERCP and Sphincterotomy: This endoscopic procedure can treat certain cases of chronic p
ancreatitis, particularly those with a
dominant stricture or obstruction in the pancreatic duct. However, in the presence of a
large duct dilatation and stones, it is less effective than surgical interventions.
Option D - Coring of pancreas head: Also known as the Frey's procedure, this is typically used in patie
nts with chronic pancreatitis where the disease is predominantly in the head of the pancreas. It is not th
e first choice in this scenario with a dilated duct and calculi.

Solution for Question 8:


Correct Option C - Percutaneous Catheter drainage + Systemic Antibiotics:
• The clinical presentation suggests a liver abscess, confirmed by the CT scan. Medical therapy is
usually the first-line treatment for a solitary liver abscess of this size (around 25 cubic cm). This typically
involves percutaneous drainage via catheter antimicrobial therapy directed against the likely organisms.
Incorrect Options:
Option A - PAIR: PAIR (Puncture, Aspiration, Injection, Re-aspiration) is primarily used for cystic echin
ococcal liver disease and not for pyogenic liver abscesses.
Option B - Surgery: Surgical intervention is typically reserved for abscesses that are not responsive to
medical therapy or percutaneous drainage or for those with complications like rupture.
Option D - Percutaneous drainage: Percutaneous drainage alone without concomitant systemic antibiot
ics may not adequately treat the underlying infection.

Page 22

511
Solution for Question 9:
Correct Option D - Gold standard investigation is CT scan:
• Gold standard investigation is a CT scan. This statement is incorrect.
• In acute cholecystitis, the Gold standard investigation is the HIDA scan.
• Diagnostic indicator of acute cholecystitis in a HIDA scan: Absence of gallbladder visualization.
• Absence of gallbladder visualization.
• Absence of gallbladder visualization.

Incorrect Options:
Option A - Gall bladder thickness >4 mm on USG: This statement is true. Thickening of the gall bladder
wall (>4 mm) is a characteristic finding in acute cholecystitis. It is often seen on ultrasound imaging.
Option B - Murphy's sign positive: This statement is true. Murphy's sign is elicited during physical exam
ination by palpating the right upper quadrant of the abdomen while the patient takes a
deep breath. If there is tenderness and the patient stops breathing due to pain, it is considered a
positive Murphy's sign and is suggestive of acute cholecystitis.
Option C - Leukocytosis: This statement is true. Acute cholecystitis is often associated with an inflamm
atory response, resulting in an increase in white blood cell count (leukocytosis).

Solution for Question 10:


Correct Option B - Periampullary Carcinoma:
• Periampullary carcinomas typically involve the ampulla of Vater, which is the region where the
common bile duct and pancreatic duct converge and enter the duodenum.

Page 23

512
• These tumors can cause obstruction of both the common bile duct and pancreatic duct, resulting in
waxing and waning jaundice and dilatation of both ducts on imaging studies.
Periampullary carcinomas typically involve the ampulla of Vater, which is the region where the common
bile duct and pancreatic duct converge and enter the duodenum.
These tumors can cause obstruction of both the common bile duct and pancreatic duct, resulting in wa
xing and waning jaundice and dilatation of both ducts on imaging studies.
Incorrect Options:
Option A - Cholangiocarcinoma: Cholangiocarcinoma refers to cancer that arises from the bile ducts.
While dilatation of the common bile duct is present in both periampullary carcinoma and cholangiocarci
noma, the history of waxing and waning jaundice and the presence of dilatation of the pancreatic duct i
n this case point more towards periampullary carcinoma.
Option C - Common bile duct stone: Common bile duct stones can cause dilatation of the common bile
duct. However, the history of waxing and waning jaundice and the involvement of the pancreatic duct s
een on CT examination make periampullary carcinoma a more likely diagnosis than a
common bile duct stone.
Option D - Head of the pancreas carcinoma: Carcinoma of the head of the pancreas can also cause dil
atation of the common bile duct and pancreatic duct. However, the term "periampullary carcinoma" enc
ompasses malignancies arising from the ampulla of Vater, distal common bile duct, duodenum, and pa
ncreas. Given the involvement of both the common bile duct and pancreatic duct, periampullary carcin
oma is a more specific and likely diagnosis than carcinoma of the head of the pancreas.

Solution for Question 11:


Correct Option C - Distal cholangiocarcinoma:
• In Distal Cholangiocarcinoma, the surgery of choice is PPPD or Pylorus Preserving Whipple’s
procedure.
• In the pylorus-preserving Whipple's procedure, the surgical technique is modified to preserve the
pylorus.
• By preserving the pylorus, the natural flow of food from the stomach into the small intestine is
maintained, potentially reducing the risk of complications such as delayed gastric emptying
(postoperative gastroparesis).
Incorrect Options:
Option A - Advanced gastric carcinoma: Treatment for Gastric Carcinoma varies based on tumor locati
on. For malignancies in the proximal third, Extended gastrectomy with D2 Lymphadenectomy is perfor
med. In cases affecting the middle third, Total gastrectomy with D2 Lymphadenectomy is preferred. Fo
r tumors in the distal third, Subtotal Gastrectomy with D2 Lymphadenectomy is the recommended appr
oach.
Option B - Gallbladder carcinoma: For Gallbladder Carcinoma, the procedure is called an Extended ch
olecystectomy, which is an en bloc removal of segment IV b & V of the liver, gallbladder, and lymph no
de along with the Cystic duct, CBD (peri-choledochal lymph node), Periportal, and Retro-pancreatic por
tion.
Option D - Chronic calcific pancreatitis: Managing Chronic Calcific Pancreatitis involves exocrine insuffi
ciency treated with enteric-coated pancreatic enzyme supplementation, while endocrine insufficiency re
quires insulin. Pain management includes analgesics such as NSAIDs (e.g., Metamizole) and opioids (

Page 24

513
e.g., Buprenorphine). If pain persists despite analgesics, options include ERCP with stenting/stone rem
oval, followed by celiac ganglion blockade if pain persists, and finally surgery (Puestow's procedure for
drainage or BEGER’S and FREY’S procedure for resection) if other interventions fail.

Solution for Question 12:


Correct Option D - Median survival is 6 months:
• Early detection and diagnosis of gallbladder carcinoma can improve the prognosis. However, due to
the aggressive nature of the disease and its often late presentation, the overall survival rates remain
relatively low.
• Treatment options for gallbladder cancer may include surgery, chemotherapy, and radiation therapy,
depending on the stage and extent of the disease.
Incorrect Options:
Option A - Palpable abdominal lump is the earliest sign: While a palpable abdominal lump can be a sig
n of advanced gall bladder carcinoma, it is not typically the earliest sign. Early signs usually include no
nspecific symptoms like abdominal pain, nausea, and weight loss.
Option B - Earliest sign is jaundice: Jaundice typically occurs in later stages of gall bladder carcinoma
when the tumor obstructs the bile ducts. It is not usually the earliest sign. Early symptoms are often no
nspecific and can include abdominal pain, bloating, and changes in appetite.
Option C - Clinical presentation of gallstones can be distinguished from gallbladder cancer: While gallst
ones can cause symptoms similar to gall bladder carcinoma (such as biliary colic), distinguishing betwe
en the two based solely on symptoms can be challenging.

Solution for Question 13:


Correct Option C - 1, 2, 3 and 5:
• The management of pancreatic necrosis typically involves the following:
• Antibiotics IV: Intravenous antibiotics are administered to prevent or treat infection, which is a
common complication of pancreatic necrosis.
• IV fluids: Intravenous fluids are given to maintain hydration and correct any electrolyte imbalances.
• Culture: Culturing the fluid or tissue is important to identify the causative organisms and guide
antibiotic selection.
• MRI: Magnetic resonance imaging (MRI) is not typically used in the initial management of pancreatic
necrosis. It may be used later in the course of the disease to assess the extent of necrosis or
complications. Usually CT scan is preferred.
• Assisted drainage: Drainage of the intra-abdominal fluid collection is an important aspect of managing
pancreatic necrosis. This can be done using various techniques, such as percutaneous catheter
drainage or endoscopic necrosectomy.
• Laparotomy: Surgical intervention with laparotomy may be required in some cases of severe
pancreatic necrosis or if there are complications that cannot be managed with less invasive methods.
However, it is not always necessary in the initial management.

Page 25

514
The management of pancreatic necrosis typically involves the following:
Antibiotics IV: Intravenous antibiotics are administered to prevent or treat infection, which is a
common complication of pancreatic necrosis.
IV fluids: Intravenous fluids are given to maintain hydration and correct any electrolyte imbalances.
Culture: Culturing the fluid or tissue is important to identify the causative organisms and guide antibiotic
selection.
MRI: Magnetic resonance imaging (MRI) is not typically used in the initial management of pancreatic n
ecrosis. It may be used later in the course of the disease to assess the extent of necrosis or complicati
ons. Usually CT scan is preferred.

Assisted drainage: Drainage of the intra-abdominal fluid collection is an important aspect of managing
pancreatic necrosis. This can be done using various techniques, such as percutaneous catheter draina
ge or endoscopic necrosectomy.
Laparotomy: Surgical intervention with laparotomy may be required in some cases of severe pancreati
c necrosis or if there are complications that cannot be managed with less invasive methods. However, i
t is not always necessary in the initial management.
Incorrect Options:
Option A - 1, 2, 3, 4, and 5: It Include MRI and laparotomy, which are not typically part of the initial man
agement of pancreatic necrosis.
Option B - 2, 4, 5, and 6: It includes MRI and laparotomy, which are not usually part of the initial manag
ement, and it does not include antibiotic administration.
Option D - 1, 2, 3, 4, 5, and 6: It include MRI and laparotomy, which are not typically part of the initial m
anagement of pancreatic necrosis.

Solution for Question 14:


Correct Option B - Choledocholithiasis:

Page 26

515
• The MRCP (Magnetic Resonance Cholangiopancreatography) image shows dilatation of the CBD
along with the presence of filling defects, representing the gallstones within the duct.
• Filling defects on MRCP can indicate the presence of stones (choledocholithiasis) or other obstructive
lesions within the bile ducts. These defects appear as areas of non-filling or decreased signal intensity
within the bile ducts.
• Symptoms of choledocholithiasis may include jaundice, abdominal pain, nausea, and vomiting.
• Treatment usually involves endoscopic retrograde cholangiopancreatography (ERCP) with
sphincterotomy and stone extraction.
Incorrect Options:
Option A - Stricture:
• Bile duct strictures may appear as narrowing or constriction of the bile duct on imaging.
• Causes of strictures include inflammation, scarring, or tumors obstructing the bile duct.
• Strictures may present similarly to choledocholithiasis on imaging, but the absence of filling defects
differentiates them from choledocholithiasis.
Option C - Cholecystitis:
• Cholecystitis refers to inflammation of the gallbladder, typically due to gallstones obstructing the cystic
duct.
• MRCP in cholecystitis may show gallbladder wall thickening, pericholecystic fluid, and gallstones
within the gallbladder.
• It does not typically involve the bile ducts directly, so dilatation of the CBD and filling defects would not
be characteristic of cholecystitis.
Option D - Cholangiocarcinoma:
• Cholangiocarcinoma is a malignant tumor arising from the bile duct epithelium.
• MRCP in cholangiocarcinoma may show biliary ductal dilatation, strictures, and intraductal masses or
nodules.
• While it may cause obstruction of the bile duct similar to choledocholithiasis, the cause would be a
tumor rather than gallstones.
• The appearance of the lesion would differ from the filling defects seen in choledocholithiasis.

Solution for Question 15:


Correct Option A - Phrygian cap sign:
• The image on the USG shows the Phrygian cap sign, a common gallbladder anatomical variation.
• The Phrygian cap sign is the most common congenital anomaly of the gallbladder.
• It is characterized by an infolding of the septum between the fundus and body of the gallbladder,
giving it a hat-like appearance resembling the cap worn by the ancient Phrygians.
• Importantly, despite this anatomical variation, the functions of the gallbladder remain normal.
• This has no pathological significance.

Page 27

516
Incorrect Options:
Option B - Murphy sign: Murphy sign is a clinical finding associated with acute cholecystitis, characteriz
ed by pain and inspiratory arrest upon palpation of the gallbladder during deep inspiration. It is not relat
ed to the anatomical variation seen in the USG image.
Option C - Claw sign: The Claw sign is a radiological finding observed in intussusception

Option D - Omega sign: The Omega sign is a


radiological finding observed in choledochal cysts, where the dilated common bile duct has a
characteristic "omega" or "figure of 8" shape. It is not related to the anomaly shown in the USG image.

Solution for Question 16:


Correct Option C - 0.5-1.5 ml/kg/min:

Page 28

517
• After laparoscopic cholecystectomy, the expected urine output in a patient with normal renal function
is approximately 0.5-1.5 ml/kg/min.
• This range ensures that the kidneys are adequately perfused and functioning properly.
• Adequate urine output is important to maintain renal function, eliminate waste products, and prevent
complications such as acute kidney injury.
• Monitoring urine output is a crucial aspect of postoperative care, as it provides valuable information
about the patient's fluid status and renal function. If the urine output falls below the expected range, it
may indicate decreased renal perfusion or impaired kidney function, which should be promptly
assessed and managed.
After laparoscopic cholecystectomy, the expected urine output in a
patient with normal renal function is approximately 0.5-1.5 ml/kg/min.
This range ensures that the kidneys are adequately perfused and functioning properly.
Adequate urine output is important to maintain renal function, eliminate waste products, and prevent co
mplications such as acute kidney injury.
Monitoring urine output is a crucial aspect of postoperative care, as it provides valuable information ab
out the patient's fluid status and renal function. If the urine output falls below the expected range, it ma
y indicate decreased renal perfusion or impaired kidney function, which should be promptly assessed a
nd managed.

Solution for Question 17:


Correct Option A - Amoebic liver abscess:
• The given ultrasound image shows a hypoechoic hyperdense lesion in the liver. Based on the clinical
history of dysentery, the most likely diagnosis is an amoebic liver abscess.
• Amoebic liver abscess is caused by the protozoan parasite Entamoeba histolytica. It typically occurs
following the ingestion of contaminated food or water. The abscess appears as a well-defined,
hypoechoic lesion with variable internal echoes on ultrasound. The surrounding liver parenchyma may
show mild to moderate edema.
• Therefore, based on the given image and clinical history, the most likely condition is an amoebic liver
abscess.
The given ultrasound image shows a hypoechoic hyperdense lesion in the liver. Based on the clinical h
istory of dysentery, the most likely diagnosis is an amoebic liver abscess.
Amoebic liver abscess is caused by the protozoan parasite Entamoeba histolytica. It typically occurs fol
lowing the ingestion of contaminated food or water. The abscess appears as a well-defined, hypoechoi
c lesion with variable internal echoes on ultrasound. The surrounding liver parenchyma may show mild
to moderate edema.
Therefore, based on the given image and clinical history, the most likely condition is an amoebic liver a
bscess.
Incorrect Options:
Option B - Pyogenic liver abscess: Pyogenic liver abscess is a bacterial infection of the liver, usually ca
used by organisms such as Escherichia coli or Klebsiella pneumoniae. On ultrasound, pyogenic liver a
bscess typically presents as multiple hypoechoic lesions rather than a single lesion seen in the provide
d image. The surrounding liver parenchyma may also show signs of inflammation.

Page 29

518
Option C - Hydatid cyst: Hydatid cysts are caused by the larval stage of the Echinococcus granulosus
parasite. On ultrasound, hydatid cysts usually appear as well-defined, anechoic or hypoechoic lesions
with multiple daughter cysts within. The provided image shows a
single hypoechoic hyperdense lesion, which is not consistent with a hydatid cyst.
Option D - Hemangioma: Hemangiomas are benign vascular tumors that commonly occur in the liver.
On ultrasound, hemangiomas typically appear as hyperechoic lesions with posterior acoustic enhance
ment. The provided image shows a hypoechoic hyperdense lesion, which is not consistent with a
hemangioma.

Solution for Question 18:


Correct Option C - Serum lipase:
• In the given scenario, the patient's presentation with severe abdominal pain and the CT scan findings
suggestive of pancreatitis indicate an inflammatory process in the pancreas.
• Pancreatitis is often associated with elevated levels of pancreatic enzymes, such as serum lipase and
serum amylase.
Incorrect Options:
Option A - Serum GGT: It is not a
specific marker for pancreatitis and is not directly related to pancreatic inflammation.
Option B
- CEA: It is not specific marker for pancreatitis and are not directly related to pancreatic inflammation
Option D - Serum bilirubin: Serum bilirubin may be elevated in certain cases of pancreatitis, especially i
f there is associated biliary obstruction, but it is not the most specific enzyme for diagnosing pancreatiti
s.

Solution for Question 19:


Correct Option B - USG:
• Based on the symptoms of fever and pain in the right upper hypochondrium after food intake, the
probable diagnosis is cholecystitis or gallstones.
• Cholecystitis refers to inflammation of the gallbladder, which is commonly caused by gallstones
obstructing the cystic duct.
• The symptoms are often precipitated or worsened by food intake, particularly fatty meals.
• USG is the initial investigation of choice for evaluating the right upper quadrant of the abdomen,
including the liver, gallbladder, and biliary system.
• USG is non-invasive, widely available, cost-effective, and does not involve exposure to ionizing
radiation.
Based on the symptoms of fever and pain in the right upper hypochondrium after food intake, the proba
ble diagnosis is cholecystitis or gallstones.
Cholecystitis refers to inflammation of the gallbladder, which is commonly caused by gallstones obstruc
ting the cystic duct.

Page 30

519
The symptoms are often precipitated or worsened by food intake, particularly fatty meals.
USG is the initial investigation of choice for evaluating the right upper quadrant of the abdomen, includi
ng the liver, gallbladder, and biliary system.
USG is non-invasive, widely available, cost-effective, and does not involve exposure to ionizing radiatio
n.
Incorrect Options:
Option A - CECT (Contrast-Enhanced Computed Tomography): While CECT can provide detailed imag
ing of the abdominal organs, it is not the investigation of choice as the initial step in this scenario. USG
is preferred as it is a
safer and more readily available option for evaluating gallbladder and biliary pathology.
Option C - MRI (Magnetic Resonance Imaging): MRI can provide detailed imaging of the abdominal or
gans without radiation exposure, but it is usually not the initial investigation for suspected cholecystitis
or gallstones. MRI may be reserved for specific indications or if further characterization of the findings f
rom USG is required.
Option D - HRCT (High-Resolution Computed Tomography): HRCT is primarily used for evaluating lun
g parenchymal and interstitial lung diseases. It is not the investigation of choice for assessing the gallbl
adder or biliary system in this scenario.

Solution for Question 20:


Correct Option B - Charcot’s triad:
• In the given scenario of a patient presenting with abdominal pain, fever, and jaundice, the triad
commonly associated with these symptoms is Charcot's triad.
• Charcot's triad is indicative of a condition affecting the biliary system, such as obstruction of the bile
ducts due to gallstones or other causes.
• Charcot's Triad: Charcot's triad is a clinical term used to describe a combination of symptoms
associated with acute cholangitis, which is inflammation of the bile ducts. The classic triad consists of:
Abdominal pain: Typically located in the right upper quadrant of the abdomen, it can be severe and
colicky in nature. Fever: Often accompanied by chills and rigors. Jaundice: Yellowing of the skin and
eyes due to elevated levels of bilirubin in the bloodstream.
• Abdominal pain: Typically located in the right upper quadrant of the abdomen, it can be severe and
colicky in nature.
• Fever: Often accompanied by chills and rigors.
• Jaundice: Yellowing of the skin and eyes due to elevated levels of bilirubin in the bloodstream.
• Abdominal pain: Typically located in the right upper quadrant of the abdomen, it can be severe and
colicky in nature.
• Fever: Often accompanied by chills and rigors.
• Jaundice: Yellowing of the skin and eyes due to elevated levels of bilirubin in the bloodstream.
Incorrect Options:
Option A - Saint Triad: Saint's triad comprises the simultaneous occurrence of the following medical co
nditions: gallstones (cholelithiasis), hiatal hernia, and diverticular disease (diverticulosis of the colon).

Page 31

520
Option C
- Virchow Triad: The Virchow triad consists of blood stasis, endothelial injury, and hypercoagulability.
Option D - Reynold's Pentad: Reynold's pentad is characterized by Charcot's triad alongside shock and
alterations in mental status.

Solution for Question 21:


Correct Option B - Endoscopic retrograde cholangiopancreatography:
• In this scenario, the presence of a large stone in the common bile duct along with biliary tree dilation
indicates an obstruction of the bile flow.
• ERCP is a diagnostic and therapeutic procedure commonly used for the management of common bile
duct stones.
• ERCP is a procedure that combines endoscopy and fluoroscopy to visualize and treat conditions
affecting the bile ducts and pancreatic duct.
• It allows for the removal of common bile duct stones, dilation of strictures, and placement of stents to
relieve biliary obstruction.
Incorrect Options:
Option A - Immediately take up the patient for cholecystectomy: Cholecystectomy is the surgical remov
al of the gallbladder. While cholecystectomy may be necessary for the management of gallstones and
a scleroatrophic gallbladder, it does not address the obstruction in the common bile duct caused by the
large stone. Therefore, it is not the immediate next step in this situation.
Option C - Wait and watch: Waiting and observing without intervention is not recommended in this sce
nario because the patient is experiencing symptoms (fatigue) and has evidence of biliary obstruction. D
elaying management could lead to complications such as cholangitis or pancreatitis.
Option D - CT scan: A CT scan may provide additional information about the extent of the biliary tree di
latation and the presence of other abnormalities, but it does not address the immediate need for interve
ntion to relieve the biliary obstruction.

Solution for Question 22:


Correct Option A - Primary sclerosing cholangitis:
• The MRCP image showing a beaded appearance is a classic finding in primary sclerosing cholangitis.
• This beaded appearance is due to alternating areas of stenosis and dilatation along the bile ducts,
giving them a segmented or beaded appearance.
• Primary sclerosing cholangitis (PSC) is a chronic inflammatory disease of the bile ducts, characterized
by progressive inflammation, fibrosis, and stricturing of the intrahepatic and extrahepatic bile ducts.
• It commonly affects young to middle-aged adults, with a slight male predominance.
Incorrect Options:
Option B - Caroli disease: Caroli disease is a congenital disorder characterized by saccular or fusiform
dilatation of the intrahepatic bile ducts. It typically presents with recurrent episodes of cholangitis and c

Page 32

521
an be associated with intrahepatic biliary stones. The MRCP image in Caroli disease shows focal or se
gmental cystic dilatation of the intrahepatic bile ducts, rather than the beaded appearance seen in the p
resented case.
Option C - Oriental cholangitis: Oriental cholangitis is a term used to describe a specific form of cholan
gitis seen predominantly in Asian populations, associated with hepatolithiasis (biliary stones) and recur
rent cholangitis. However, there is no specific imaging finding or mention of the beaded appearance in
Oriental cholangitis. Therefore, this is an incorrect answer choice.
Option D
- Primary biliary carcinoma: Primary biliary carcinoma, also known as cholangiocarcinoma, is a maligna
ncy arising from the epithelial cells of the bile ducts. It can cause obstructive jaundice, but it typically do
es not present with recurrent episodes of fever and chills. Additionally, the MRCP image in primary bilia
ry carcinoma would typically show focal or diffuse strictures of the bile ducts, rather than the beaded ap
pearance seen in the presented case.

Solution for Question 23:


Correct Option D - ALT:
• It is because the Nazer prognostic index for Wilson's disease does not include serum alanine
aminotransferase (ALT) levels as a parameter for liver transplantation selection criteria
Nazer prognostic index for Wilson's disease:
• Parameters included: Serum bilirubin levels. Serum aspartate aminotransferase (AST) levels.
Prolongation of prothrombin time (PT).
• Serum bilirubin levels.
• Serum aspartate aminotransferase (AST) levels.
• Prolongation of prothrombin time (PT).
• Evaluate hepatic decompensation patients by determining disease severity using Nazer prognostic
index.
• Scores <7 can be managed with medical therapy.
• Scores >9 necessitate immediate consideration for liver transplantation.
• For scores between 7 and 9, the decision between transplantation and medical therapy requires
clinical judgment.
• Combination of trientine and zinc can treat patients with Nazer scores up to 9.
• Close monitoring is crucial for patients with Nazer scores nearing 9 for signs of hepatic deterioration,
indicating transplantation necessity.
• Serum bilirubin levels.
• Serum aspartate aminotransferase (AST) levels.
• Prolongation of prothrombin time (PT).
Incorrect Options
Option A, B & C - AST, PT & Bilirubin:

Page 33

522
• Options A, B, and C are not suitable because they align with the parameters included in the Nazer
prognostic index for Wilson's disease, which are: Serum bilirubin levels. Serum aspartate
aminotransferase (AST) levels. Prolongation of prothrombin time (PT).
• Serum bilirubin levels.
• Serum aspartate aminotransferase (AST) levels.
• Prolongation of prothrombin time (PT).
• Serum bilirubin levels.
• Serum aspartate aminotransferase (AST) levels.
• Prolongation of prothrombin time (PT).

Solution for Question 24:


Correct Option C - Insulinoma:
• Insulinoma is the most common neuroendocrine tumor of the pancreas.
• Insulinomas arise from the cells that produce insulin, known as beta cells.
• Insulinomas are typically small, benign tumors that can cause excessive production of insulin, leading
to hypoglycemia.
• The classic clinical presentation includes recurrent episodes of fasting hypoglycemia,
neuroglycopenic symptoms (such as confusion and altered consciousness), and relief of symptoms
after glucose administration.
Insulinoma is the most common neuroendocrine tumor of the pancreas.
Insulinomas arise from the cells that produce insulin, known as beta cells.
Insulinomas are typically small, benign tumors that can cause excessive production of insulin, leading t
o hypoglycemia.
The classic clinical presentation includes recurrent episodes of fasting hypoglycemia, neuroglycopenic
symptoms (such as confusion and altered consciousness), and relief of symptoms after glucose admini
stration.
Incorrect Options:
Option A - Gastrinoma: Gastrinomas are neuroendocrine tumors that arise from the cells producing ga
strin in the pancreas. While gastrinomas are commonly associated with a condition called Zollinger-Elli
son syndrome, which involves excessive gastrin production and gastric acid hypersecretion, they are le
ss common than insulinomas.
Option B - Somatostatinoma: Somatostatinomas are rare neuroendocrine tumors that arise from the ce
lls producing somatostatin in the pancreas. These tumors can cause various symptoms related to exce
ss somatostatin, such as diabetes mellitus, cholelithiasis, steatorrhea, and weight loss. However, soma
tostatinomas are less common than insulinomas.
Option D - VIPoma: VIPomas are neuroendocrine tumors that produce vasoactive intestinal peptide (VI
P). These tumors are associated with a rare condition known as Verner-Morrison syndrome or VIPoma
syndrome, characterized by profuse watery diarrhea, hypokalemia, and achlorhydria. While VIPomas
can occur in the pancreas, they are less common than insulinomas.

Page 34

523
Solution for Question 25:
Correct Option C - Bile leak:
This given clinical scenario is suggestive of bile leak following bile duct injury after laparoscopic cholec
ystectomy
• Incidence of Bile duct injury in laparoscopic cholecystectomy (0.3 - 0.85%)
• Incidence of Bile duct injury in open cholecystectomy (0.1 - 0.2%)
Most common injured part of the bile duct in laparoscopic cholecystectomy is the lateral wall of commo
n hepatic duct
Incorrect Options:
Option A & B - Infection & Surgical site pain: Infection and Surgical site pain do not lead to a
collection of such amounts in subhepatic space.
Option D - Retained gallstones: Retained gallstones/dropped/slipped gallstones are common during la
paroscopic cholecystectomy. It occurs in 0.1–20% of the cases. It occurs when gallstones are inadvert
ently spilled into the peritoneal cavity

Solution for Question 26:


Correct Option A - Regular follow-up every 2 months:
• Regular follow-up every two months allows for close monitoring of the polyp's size and any changes
over time. If the size increases or concerning features develop, further intervention can be considered.
• Factors Which Increase the Risk of Invasive Cancer in Adenomatous Polyp Age more than 60 years
Presence of gallstone Size of polyp more than 1 cm Documented increase in size on repeated scan
• Age more than 60 years
• Presence of gallstone
• Size of polyp more than 1 cm
• Documented increase in size on repeated scan
Regular follow-up every two months allows for close monitoring of the polyp's size and any changes ov
er time. If the size increases or concerning features develop, further intervention can be considered.
Factors Which Increase the Risk of Invasive Cancer in Adenomatous Polyp
• Age more than 60 years
• Presence of gallstone
• Size of polyp more than 1 cm
• Documented increase in size on repeated scan
Age more than 60 years
Presence of gallstone

Page 35

524
Size of polyp more than 1 cm
Documented increase in size on repeated scan
Incorrect Options:
Option B - Polypectomy: A polypectomy is the surgical removal of a polyp. While polypectomy is a viabl
e option for larger or symptomatic polyps, it's not usually recommended for small, asymptomatic polyps
like the one in this case. Moreover, polypectomy might not completely eliminate the risk of malignancy
if the polyp has already demonstrated concerning features like increased size or dysplasia.
Option C - Laparoscopic cholecystectomy: Cholecystectomy (removal of the gallbladder) is generally re
served for cases where there's a high suspicion of malignancy or if the polyp is large (>1 cm) and/or as
sociated with gallstones. In this case, the polyp is small, the patient is asymptomatic, and there are no
associated gallstones, making cholecystectomy an overly aggressive option at this stage.
Option D - Open cholecystectomy: Extensive inflammation, adhesions, anatomical variances, bile duct
injury, retained bile duct stones, and uncontrolled bleeding are indications to convert a
cholecystectomy to an open procedure.

Solution for Question 27:


Correct Option C - Steatorrhea:
• Absence of the Gallbladder: The gallbladder plays a crucial role in storing and releasing bile, which
aids in the digestion of fats. After cholecystectomy, the continuous flow of bile into the intestines can
disrupt the normal process of fat digestion and absorption.
• Bile Flow Changes: Without the gallbladder, bile is continuously released into the small intestine,
which can overwhelm the capacity of the intestine to emulsify and absorb fats effectively.
• Steatorrhea Symptoms: Patients with steatorrhea often experience bulky, greasy, and foul-smelling
stools due to the presence of undigested fat.
Incorrect Options:
Option A - Acute kidney injury: Acute kidney injury is not a common side effect of cholecystectomy. Th
e surgery primarily affects the digestive system, not the kidneys.
Option B - Osteoporosis: While there might be some association between gallbladder removal and cha
nges in bone metabolism, osteoporosis is not a direct or common side effect of cholecystectomy.
Option D - Colorectal cancer: Cholecystectomy does not directly increase the risk of colorectal cancer.
The procedure is aimed at addressing gallbladder-related issues and does not affect the colon or rectu
m, where colorectal cancer typically originates.

Solution for Question 28:


Correct Option B - Acute pancreatitis:
• The colon cutoff sign describes the abrupt termination of gas within the proximal colon caused by a
functional spasm of infiltrating phrenicocolic ligament. The presence of the colon cutoff sign on
radiography is usually related to acute pancreatitis.
Incorrect Options:

Page 36

525
Option A - Acute hepatitis: Symptoms of acute hepatitis are fever, fatigue, loss of appetite, nausea, vo
miting, abdominal pain, dark urine, light-colored stools, joint pain, and jaundice
Option C - Acute colitis: In colitis, the mucosal lining of the colon becomes acutely or chronically inflam
ed. Symptoms of patients with colitis include watery diarrhea, abdominal pain, tenesmus, urgency, feve
r, subjective fatigue, or blood in the stool
Option D - Intestinal obstruction: Intestinal obstructions presents with dilated bowel loops with multiple
air fluid levels on abdominal xray.

Solution for Question 29:


Correct Option A - MRCP:
• MRCP is a non-invasive imaging modality that can provide detailed visualisation of the biliary system,
including the common bile duct (CBD) and the presence of any stones or obstruction.
• In this case, the patient has multiple gallstones, and the CBD diameter is increased at 12mm, which
suggests the possibility of choledocholithiasis (stones in the common bile duct).
• The elevated alkaline phosphatase and significantly raised GGT levels also indicate some degree of
biliary obstruction.
• MRCP can accurately assess the CBD and help identify the presence, location, and size of any
stones within the duct.
• It can also evaluate the pancreatic duct for any associated pathology. By obtaining this information,
MRCP can guide further management decisions and determine the most appropriate intervention,
whether it's endoscopic or surgical.
Incorrect Options:
Option B - ERCP (Endoscopic Retrograde Cholangiopancreatography):
• ERCP is an invasive procedure that combines endoscopy and fluoroscopy to diagnose and treat
conditions of the bile ducts and pancreas. While ERCP can be therapeutic and used for stone removal,
it is typically reserved for cases where there is a clear indication for intervention, such as symptomatic
choledocholithiasis or evidence of biliary obstruction with significant symptoms or abnormal liver
function tests.
• In this case, the patient is asymptomatic for biliary obstruction, and the serum bilirubin level is within
the normal range (0.8). The elevated alkaline phosphatase and GGT levels suggest some degree of
biliary obstruction, but they are not significantly elevated to warrant immediate intervention without
further evaluation. Therefore, ERCP may not be the next best step in this scenario.
Option C - Endoscopic ultrasound:
• Endoscopic ultrasound (EUS) is a procedure that combines endoscopy and ultrasound to obtain
detailed imaging of the gastrointestinal tract and surrounding structures. While EUS can be useful for
evaluating certain conditions, such as pancreatic tumours or staging of gastrointestinal malignancies, it
is not the primary modality for assessing gallstones or the biliary system. In this case, the patient's main
concern is gallstones and possible choledocholithiasis, which are better evaluated using MRCP.
Option D - Laparoscopic cholecystectomy:
• Laparoscopic cholecystectomy is a surgical procedure performed to remove the gallbladder and is the
definitive treatment for symptomatic gallstones. However, in this case, the patient's symptoms are not
mentioned, and the focus is primarily on the evaluation of the biliary system. Therefore, performing a

Page 37

526
cholecystectomy without further assessment and identification of possible choledocholithiasis would be
premature

Solution for Question 30:


Correct Option C - 1, 3, 4:
Incorrect Options
Option A - 1, 2, 3: It includes the incorrect statement that Lenvatinib is used for lesions less than 3
cm. Lenvatinib is an approved systemic therapy for advanced HCC
Option B - 2, 3, 4: It includes the incorrect statement that Lenvatinib is used for lesions less than 3 cm.
Option D - 1
and 2: It does not include the correct statement that NASH and NAFLD are risk factors for HCC.

Solution for Question 31:


Correct Option A - Magnetic resonance imaging:
• In a patient with obstructive jaundice and the described findings of a distended gallbladder, dilated
common bile duct, and intrahepatic biliary radicles on ultrasound and CT, magnetic resonance imaging
(MRI) would be the most useful modality to localise the cause.
• MRI is particularly valuable for visualising the biliary system and can provide detailed images of the
liver, gallbladder, bile ducts, and surrounding structures.
• It can help identify the site and cause of the obstruction in cases of obstructive jaundice.
Incorrect Options
Option B - Endoscopic ultrasound (EUS): While EUS can provide detailed imaging of the biliary system
, it is more commonly used for evaluating the gastrointestinal tract, including the pancreas and nearby l
ymph nodes. In this case, the main concern is localising the cause of the obstruction, which is better ac
hieved with other modalities.
Option C - Percutaneous transhepatic cholangiography (PTC): PTC involves the insertion of a needle i
nto the liver to inject contrast material directly into the biliary system. It can be useful in cases of biliary
obstruction, but it is an invasive procedure and is typically reserved for cases where other imaging mod
alities have been inconclusive or when therapeutic interventions, such as stent placement, are needed.
Option D - Endoscopic retrograde cholangiopancreatography (ERCP): ERCP is an invasive procedure
that combines endoscopy and fluoroscopy to visualise the biliary and pancreatic ducts. While it is usefu
l for diagnosing and treating certain biliary conditions, it is generally reserved for cases where interventi
on or therapeutic procedures are required, rather than for initial localization of the cause of obstructive j
aundice.

Solution for Question 32:


Option B - USG-guided pigtail catheter drainage:

Page 38

527
• The next step in managing a woman who presented with abdominal pain and tachycardia, with a fluid
collection in the right sub-hepatic space seen on ultrasound, is USG-guided pigtail catheter drainage as
this is a case of bile duct injury following cholecystectomy.
• Ultrasound-guided pigtail catheter drainage offers several advantages, including improved accuracy in
locating the fluid collection and minimizing the risk of complications.
Incorrect Options:
Option A - Antibiotics and send home: The presence of a fluid collection and symptoms suggestive of p
ostoperative complications necessitates intervention rather than sending the patient home with antibioti
cs alone.
Option C - Laparoscopy and re-exploration: Laparoscopy and re-exploration are not the first-line mana
gement options for a fluid collection in the subhepatic space. Less invasive measures like drainage sho
uld be attempted first.
Option D - ERCP and T-tube placement: ERCP and T-tube placement are indicated in biliary tract-relat
ed issues, which may not be the primary cause of the fluid collection in this scenario.

Solution for Question 33:


Correct Option A - Veress needle:
• The instrument shown in the image is a Veress needle.
• During laparoscopic surgery, it is used to insufflate the abdomen with carbon dioxide gas. The Veress
needle is inserted into the abdominal cavity, and the gas is delivered to create pneumoperitoneum,
which provides a clear working space for laparoscopic procedures.

Incorrect Options:
Option B, C & D are incorrect as they are not used in laparoscopic surgeries.

Page 39

528
Solution for Question 34:
Correct Option B - Alcohol:
• Alcohol is indeed the most common cause of chronic pancreatitis.
TIGARO Classification of Etiology:
• T - Toxic - Metabolic
• I - Idiopathic
• G - Genetic
• A - Autoimmune
• R - Recurrent severe
• O - Obstructive
Incorrect Options:
Option A - Gallstones: Gallstones can cause acute pancreatitis when they obstruct the pancreatic duct,
leading to inflammation. However, while gallstones can contribute to recurrent acute pancreatitis, they
are not the most common cause of chronic pancreatitis.
Option C - Autoimmune: Autoimmune pancreatitis is a relatively rare condition characterized by chronic
inflammation of the pancreas due to an autoimmune reaction. It is an important cause of chronic pancr
eatitis, but it is less common than alcohol-induced pancreatitis.
Option D - Drugs: Certain medications, such as certain antiretroviral drugs used in HIV/AIDS treatment
or certain immunosuppressive drugs used in organ transplant recipients, can rarely cause chronic panc
reatitis. However, drug-induced pancreatitis accounts for a
small percentage of cases and is not the most common cause.

Solution for Question 35:


Correct Option A - Transgastric FNAC guided by endoscopic ultrasound:
• Transgastric fine-needle aspiration cytology (FNAC) guided by endoscopic ultrasound (EUS) is
considered the best method to obtain tissue for the diagnosis of pancreatic carcinoma. E
• US allows for precise visualization and targeting of the pancreatic lesion, while FNAC involves using a
thin needle to extract cells for examination under a microscope.
• This method is minimally invasive, has high diagnostic accuracy, and can provide valuable information
for staging and treatment planning.
Incorrect Options:
Option B - MRI-guided biopsy: While MRI can provide detailed images of pancreatic tumors, it is not ty
pically used for biopsy procedures. MRI-guided biopsies are not commonly performed for pancreatic ca
rcinoma diagnosis.
Option C - Laparoscopic-guided biopsy: Laparoscopic-guided biopsies may be considered in some cas
es, but they are generally not the first-line approach for obtaining tissue for the diagnosis of pancreatic
carcinoma. Laparoscopic procedures involve making small incisions and using a camera and surgical i
nstruments to access and visualize the pancreas. However, transgastric FNAC guided by EUS is prefe
rred over laparoscopic-guided biopsies due to its minimally invasive nature.

Page 40

529
Option D
- Percutaneous transperitoneal biopsy: Percutaneous transperitoneal biopsies involve inserting a needl
e through the abdominal wall to reach the pancreas and obtain tissue samples. While this method may
be used in some cases, it is generally less preferred compared to transgastric FNAC guided by EUS, w
hich provides better visualization and targeting of the pancreatic lesion.

Solution for Question 36:


Correct Option D - Acute Pancreatitis without choledocholithiasis:
• ERCP is an invasive diagnostic and therapeutic procedure used to examine and treat disorders of the
bile ducts, pancreatic ducts, and gallbladder. It involves inserting an endoscope through the mouth,
esophagus, and stomach to reach the duodenum, where the ampulla of Vater (the opening that
connects the bile ducts and pancreatic duct to the duodenum) is visualized. ERCP allows for the
visualization and imaging of the biliary and pancreatic ductal systems.
• ERCP is the least useful diagnostic procedure for acute pancreatitis without choledocholithiasis. If no
suspected biliary duct stones are causing the pancreatitis and the patient's condition is stable, ERCP
may not be the initial procedure of choice. Other diagnostic modalities, such as abdominal ultrasound,
computed tomography (CT), or magnetic resonance imaging (MRI), may be employed to assess the
cause of pancreatitis.
Incorrect Options:
Option A - Recurrent pancreatitis: ERCP can be useful in the diagnosis of recurrent pancreatitis, especi
ally when it is suspected to be caused by abnormalities in the pancreatic or biliary ducts. It enables the
assessment of ductal anatomy, identification of strictures, and evaluation for the presence of stones or
other obstructions.
Option B - Unexplained jaundice: ERCP is helpful in diagnosing unexplained jaundice. It allows for visu
alization of the bile ducts and helps identify the cause of jaundice, such as stones, strictures, or tumors
obstructing the bile flow.
Option C - Periampullary mass: ERCP is particularly useful in the diagnosis of periampullary masses. It
provides detailed imaging of the ampulla of Vater and helps identify the location, extent, and nature of
the mass, aiding in further management decisions.

Solution for Question 37:


Correct Option C - Acute pancreatitis:
• The image shows Cullen’s sign, which is superficial edema with bruising in the subcutaneous fatty
tissue around the peri-umbilical region, as well as the Grey Turner sign, which is flank ecchymosis.
These signs are indicative of acute pancreatitis.
• Acute pancreatitis is characterized by pain in the abdomen that radiates to the back and is relieved in
the sitting forward position.
• Elevated lipase and amylase are seen in acute pancreatitis.

Page 41

530
Incorrect Options:
Option A, B and D are incorrect. Refer to the explanation of correct answer.

Solution for Question 38:


Correct Option B - Whipple’s procedure:
• The procedure shown in the image is Whipple’s procedure or pancreaticoduodenectomy. It is the
primary surgical treatment for pancreatic cancer. The head of the pancreas, most of the duodenum, a
portion of the bile duct, the gallbladder, and associated lymph nodes are removed.
• Whipple's pancreaticoduodenectomy involves resecting the tumor along with the 'C' loop of duodenum
up to DJ flexure, proximal jejunum, head of the pancreas up to the neck, and partial gastrectomy,
followed by three anastomoses: Pancreaticojejunostomy, Hepaticojejunostomy, and
Gastrojejunostomy, typically performed for a mobile growth without metastasis.

Page 42

531
Incorrect Options:
Option A - Berger’s procedure: In Beger’s procedure all of the head of the pancreas is removed but the
bile duct, stomach and duodenum are preserved.
Option C - Frey’s procedure: This procedure includes head coring (a portion of the head of the pancrea
s is incised). The Duodenum is spared.
Option D - Pancreatic division: The image shown indicates Pancreaticoduodenectomy and not pancrea
tic division.

Solution for Question 39:


Correct choice: B
• The procedure shown in the above image is pancreaticojejunostomy. Puestow's operation is indicated
for diffuse chronic pancreatitis and dilated pancreatic duct. The duct is opened wide, strictures are cut
open, stones are removed and it is anastomosed to one of the loops of jejunum. This is a by-pass
procedure. It preserves both endocrine and exocrine functions.
Incorrect choices:
Option A. Gastrojejunostomy: it is the anastomosis of the body of the stomach to the first loop of the jej
unum bypassing the duodenum. This procedure does not include the pancreas.
Option C. Pancreatic Ileostomy: Done in case of pancreatic duct obstruction and the pancreas is anast
omosed to the ileum. This procedure is not indicated in the image.
Option D. Duodenojejunostomy: It is a procedure done to anastomose a part of the duodenum to the je
junum to bypass an obstruction in the duodenum. This procedure is not indicated in the image.

Solution for Question 40:

Page 43

532
Correct Option A - Pancreatic pseudocyst:
• Pancreatic pseudocyst is a complication seen after 2 weeks of an acute pancreatitis attack. It is seen
commonly in alcoholism. The cyst is usually located between the stomach and transverse colon,
between the stomach and liver or Behind or below the transverse colon. Clinically a Tensely cystic
mass in the epigastrium, umbilical region or left hypochondrium can be seen.
Incorrect Options:
Option B - Large choledochal cyst: Choledochal cyst is a
congenital cyst and is not caused due to alcoholism. It is not seen as a tense mass in the epigastrium.
Option C - Gastric duplication cyst: It is a congenital cyst and not caused due to alcoholism. It is seen u
sually in relation to the distal region of the greater curvature. There is no clinically evident tense mass n
ear the epigastrium.
Option D - Mesenteric cyst: This cyst is congenital. The swelling is seen in the umbilical region and not
at the epigastric region.

Solution for Question 41:


Correct Option A - Acute cholecystitis:
• Murphy's sign: when we place the fingers on the right hypochondrium and ask the patient to take deep
inspiration. At the height of inspiration, there is a sudden catch. This catch is due to an inflamed
gallbladder coming in contact with the abdominal wall. This produces pain.
• In the case of acute cholecystitis, there is inflammation of the gallbladder and Murphy's sign is
positive.
Incorrect Options:
Option B - Carcinoma gallbladder: In carcinoma of the gall bladder Murphy’s sign is not positive.
Option C - Peptic ulcer: In peptic ulcer, Murphy’s sign is not positive.
Option D - Acute appendicitis: In acute appendicitis Murphy’s sign is not positive.

Solution for Question 42:


Correct Option D - CA 19-9:
• CA 19-9 (carbohydrate antigen 19-9) is a tumor marker that is commonly associated with
gastrointestinal cancers, particularly pancreatic cancer.
• Normally produced by various cells in the gastrointestinal tract, including the pancreas, gallbladder,
and stomach.
Incorrect Options:
Option A - NSE (Neuron-Specific Enolase): NSE is a tumor marker primarily associated with neuroend
ocrine tumors, such as small-cell lung cancer and neuroblastoma. It is not specific to pancreatic cancer
.
Option B - Alpha-Fetoprotein (AFP): AFP is a tumor marker commonly associated with liver cancer (he
patocellular carcinoma) and certain types of germ cell tumors. It is not specific to pancreatic cancer.

Page 44

533
Option C - Ca 15-3: Ca 15-3 is a
tumor marker often used in the management of breast cancer. It is not specific to pancreatic cancer.

Solution for Question 43:


Correct Option B - Hydatid cyst:
• Based on the given information, the most likely diagnosis in this case is Hydatid cyst.
• Hydatid cyst is a parasitic infection caused by the larval stage of the Echinococcus granulosus
tapeworm. The infection commonly affects the liver and can lead to the formation of cysts within the
organ.
• In the case described, the patient is a sheep farmer, which increases the risk of exposure to the
Echinococcus tapeworm. The presence of pain in the right upper part of the abdomen, tenderness, and
enlargement of the liver are consistent with liver involvement.
• Blood investigations showing eosinophilia (elevated eosinophil count) and elevated liver enzymes
further support the diagnosis of a parasitic infection. Eosinophilia is commonly seen in parasitic
infections, including hydatid cysts.
• The abdominal ultrasound image provided shows a well-defined cystic lesion in the liver, which is
typical of a hydatid cyst. The presence of a cyst with daughter cysts or a characteristic "water lily sign"
(a detached endocyst floating within the cyst) on imaging can help confirm the diagnosis.

Incorrect Options:
Option A - Amoebic liver abscess: Amoebic liver abscess is another possible differential diagnosis in p
atients with liver involvement and a history of travel to or residence in endemic areas, but it is less likel
y in this case as there is no mention of travel history.
Option C & D - Pyogenic liver abscess & hepatocellular carcinoma: Pyogenic liver abscess and hepato
cellular carcinoma are less likely in this case as they usually present with different clinical features and
may not be associated with eosinophilia or a history of exposure to the Echinococcus tapeworm.

Page 45

534
Solution for Question 44:
Correct Option C - Angiosarcoma of the liver:
• Based on the given information, the likely diagnosis in this case is Angiosarcoma of the liver.
• Angiosarcoma of the liver is a rare malignant tumor that arises from the cells lining the blood vessels
of the liver. It is associated with exposure to certain chemicals and environmental factors, including
vinyl chloride, which is used in the production of plastic pipes.
• The patient in this case is a factory worker in a plastic pipe factory, which suggests a potential
exposure to vinyl chloride or other chemical agents. The presentation of weight loss, right upper
quadrant abdominal pain, and hepatomegaly is consistent with liver involvement.
• The description of the liver being replaced by a hemorrhagic tumor with extensive necrosis is typical of
an aggressive tumor such as angiosarcoma. The extensive necrosis may contribute to weight loss and
other systemic symptoms.
Incorrect Options:
Option A - Cavernous hemangioma: Cavernous hemangioma is a
benign vascular tumor of the liver and usually presents as a well-circumscribed, non-hemorrhagic mas
s. It is not typically associated with weight loss or extensive necrosis.
Option B - Metastatic carcinoma of the liver: Metastatic carcinoma of the liver refers to the spread of ca
ncer from other sites to the liver. While it can cause hepatomegaly and liver involvement, the descriptio
n of extensive necrosis and hemorrhage is more suggestive of a primary malignant tumor.
Option D - Primary hepatocellular carcinoma: Primary hepatocellular carcinoma (HCC) is the most com
mon primary malignancy of the liver. While it can also present with hepatomegaly and weight loss, the
description of extensive necrosis and hemorrhage is more suggestive of angiosarcoma.

Solution for Question 45:


Correct Option A - Gall bladder carcinoma:
• Gross appearance: The image provided shows a resected gallbladder with features suggestive of
carcinoma. Gallbladder carcinomas often present as a firm, poorly circumscribed mass or may infiltrate
the wall diffusely, mimicking chronic cholecystitis.
• The image provided shows a resected gallbladder with features suggestive of carcinoma.
• Gallbladder carcinomas often present as a firm, poorly circumscribed mass or may infiltrate the wall
diffusely, mimicking chronic cholecystitis.
• Precursor lesions: Gallbladder carcinoma typically arises from precursor lesions, such as flat in situ
lesions with varying degrees of dysplasia, intracholecystic papillary tubular neoplasms, and intestinal
metaplasia. This indicates a stepwise process leading to the development of full-blown cancer.
• Gallbladder carcinoma typically arises from precursor lesions, such as flat in situ lesions with varying
degrees of dysplasia, intracholecystic papillary tubular neoplasms, and intestinal metaplasia.
• This indicates a stepwise process leading to the development of full-blown cancer.
• Microscopic characteristics: Microscopically, gallbladder carcinomas are usually adenocarcinomas
characterized by glands embedded in the desmoplastic stroma. However, in some cases, cytologic
atypia and stromal responses may be minimal.

Page 46

535
• Microscopically, gallbladder carcinomas are usually adenocarcinomas characterized by glands
embedded in the desmoplastic stroma.
• However, in some cases, cytologic atypia and stromal responses may be minimal.
• The image provided shows a resected gallbladder with features suggestive of carcinoma.
• Gallbladder carcinomas often present as a firm, poorly circumscribed mass or may infiltrate the wall
diffusely, mimicking chronic cholecystitis.
• Gallbladder carcinoma typically arises from precursor lesions, such as flat in situ lesions with varying
degrees of dysplasia, intracholecystic papillary tubular neoplasms, and intestinal metaplasia.
• This indicates a stepwise process leading to the development of full-blown cancer.
• Microscopically, gallbladder carcinomas are usually adenocarcinomas characterized by glands
embedded in the desmoplastic stroma.
• However, in some cases, cytologic atypia and stromal responses may be minimal.
Incorrect Options:
Option B - Mixed stone in gall bladder:
• Mixed stones in the gallbladder typically consist of cholesterol, bilirubin, and calcium salts.
• They are usually detected through imaging techniques like ultrasound and may not manifest grossly
as a carcinoma.
Option C - Cholecystitis:
• Cholecystitis refers to inflammation of the gallbladder, often due to obstruction of the cystic duct by
gallstones.
Option D - Choledocholithiasis:
• Choledocholithiasis refers to the presence of stones in the common bile duct.

Solution for Question 46:


Correct Option C - USG:
• Biliary dyskinesia is a disorder characterized by abnormal gallbladder motility. It results in symptoms
such as recurrent pain in the right hypochondrium (upper right abdomen), nausea, and vomiting. The
condition is often associated with a malfunctioning gallbladder, which fails to contract and empty
properly.
• Ultrasound is a non-invasive imaging modality commonly used to evaluate the biliary system.
However, in cases of biliary dyskinesia, ultrasound findings may appear normal or nonspecific. Since
biliary dyskinesia primarily involves abnormal gallbladder motility rather than structural abnormalities,
ultrasound may not provide conclusive evidence for the diagnosis. Therefore, while ultrasound can be
helpful in ruling out other conditions or identifying other structural abnormalities, it is considered the
least useful in diagnosing biliary dyskinesia specifically.
Incorrect Options:
Option A - ERCP (Endoscopic Retrograde Cholangiopancreatography): ERCP is an invasive procedur
e that involves both endoscopy and radiography. While ERCP can be useful in diagnosing various bilia
ry and pancreatic disorders, including biliary dyskinesia, it is not the least useful option in this scenario.
ERCP allows direct visualization of the biliary system and can also provide an opportunity for therapeu

Page 47

536
tic interventions if needed.
Option B - HIDA scan (Hepatobiliary Iminodiacetic Acid Scan): HIDA scan is a nuclear medicine imagin
g test that evaluates the functioning of the gallbladder and biliary system. It involves the injection of a r
adioactive tracer, which is taken up by the liver and excreted into the biliary system. The tracer's move
ment is tracked to assess gallbladder function and bile flow. HIDA scan is commonly used in the diagn
osis of biliary dyskinesia and can help determine the gallbladder's ability to contract and empty properly
. It is a valuable diagnostic tool in this scenario.
Option D - Post-surgery follow-up: Post-surgery follow-up is not directly related to the initial diagnosis o
f biliary dyskinesia. However, it is important for patients who undergo surgical interventions for biliary d
yskinesia or related conditions to have appropriate follow-up care to monitor their recovery and addres
s any potential complications. Post-surgery follow-up is not used to diagnose biliary dyskinesia but rath
er to manage and assess the patient's condition after surgical intervention.

Solution for Question 47:


Correct Option C - Serum lipase:
• Serum lipase is an enzyme that is elevated in acute pancreatitis. Measuring serum lipase levels can
help confirm the diagnosis and guide further management. Elevated lipase levels, along with the
patient's clinical presentation, would support the diagnosis of acute pancreatitis.
Serum lipase is an enzyme that is elevated in acute pancreatitis. Measuring serum lipase levels can he
lp confirm the diagnosis and guide further management. Elevated lipase levels, along with the patient's
clinical presentation, would support the diagnosis of acute pancreatitis.
Incorrect Option:
Option A - Upper GI endoscopy: Upper gastrointestinal endoscopy is not the immediate next step in thi
s scenario because the patient's symptoms and examination findings suggest pancreatitis rather than a
n upper gastrointestinal pathology. Pancreatitis should be considered and investigated first.
Option B - Alcohol breath test: An alcohol breath test may be useful in determining recent alcohol cons
umption. While chronic alcoholism is a risk factor for pancreatitis, a breath test alone would not confirm
the diagnosis or guide immediate management. Therefore, the next step in this situation is not the mo
st appropriate one.
Option D - CECT: Contrast-enhanced computed tomography (CECT) scan is a valuable diagnostic tool
for assessing pancreatic pathology, including pancreatitis. However, in this scenario, the patient's clini
cal presentation and examination findings strongly suggest acute pancreatitis. Given the typical clinical
picture, a serum lipase test would be the more appropriate and efficient next step before considering m
ore invasive imaging studies.

Solution for Question 48:


Correct Option C - Insulinomas:
• Insulinomas are the most common neuroendocrine tumors of the pancreas. They arise from
insulin-producing cells in the pancreas called beta cells. Insulinomas typically produce excessive
amounts of insulin, leading to recurrent episodes of hypoglycemia (low blood sugar). They are usually
benign and solitary tumors.

Page 48

537
Insulinomas are the most common neuroendocrine tumors of the pancreas. They arise from insulin-pro
ducing cells in the pancreas called beta cells. Insulinomas typically produce excessive amounts of insul
in, leading to recurrent episodes of hypoglycemia (low blood sugar). They are usually benign and solita
ry tumors.
Incorrect Options:
Option A - Gastrinoma: Gastrinomas are neuroendocrine tumors that occur in the duodenum or pancre
as. They produce excessive amounts of gastrin, leading to a condition called Zollinger-Ellison syndrom
e. Gastrinomas are typically associated with peptic ulcers and acid hypersecretion rather than being th
e most common neuroendocrine tumor of the pancreas.
Option B - Somatostatinoma: Somatostatinomas are rare neuroendocrine tumors that can arise in vario
us organs, including the pancreas. These tumors produce excessive amounts of somatostatin, which in
hibits the release of multiple hormones. While somatostatinomas can occur in the pancreas, they are r
elatively rare compared to other neuroendocrine tumors.
Option D - VIPoma: VIPomas, also known as Verner-Morrison syndrome or pancreatic cholera, are ne
uroendocrine tumors that secrete vasoactive intestinal peptide (VIP). These tumors are rare and can o
ccur in the pancreas or other organs. VIPoma is associated with a condition called VIPoma syndrome,
which includes symptoms such as severe diarrhea, dehydration, electrolyte imbalances, and flushing.

Solution for Question 49:


Correct Option C - Type C
• Strasberg's classification is a widely used classification system for bile duct injuries that occur during
cholecystectomy or other biliary surgeries. It categorizes the injuries based on the anatomical location
and severity of the injury.

• In the given scenario, the injury is specifically described as "injury to the right aberrant posterior
sectoral duct with bile leak." This corresponds to a Type C injury according to Strasberg's classification.
Incorrect Options:

Page 49

538
Option A - Type A: Type A
injuries involve the common bile duct, which is not described in the given scenario.
Option B - Type B: Type B injuries involve a
major bile duct, such as the right or left hepatic duct, but the description does not match this category.
Option D - Type D: Type D
injuries involve both the right and left hepatic ducts, which is not the case in the given scenario.

Solution for Question 50:


Correct Option B - ERCP:
• ERCP is the best investigation for this patient for the following reasons: Direct Visualization: ERCP
allows for direct visualization of the biliary tree, enabling the identification of the site and extent of the
bile leak. Therapeutic Intervention: ERCP not only diagnoses bile leaks but also provides an
opportunity for therapeutic intervention. Stenting can be performed during ERCP to bridge the defect
and allow for bile drainage, promoting healing. Minimally Invasive: ERCP is minimally invasive and can
often be performed without the need for surgery, reducing the risk of complications associated with
additional surgical procedures. Precise Localization: ERCP provides precise localization of the bile
leak, aiding in the planning of further management strategies.
• Direct Visualization: ERCP allows for direct visualization of the biliary tree, enabling the identification
of the site and extent of the bile leak.
• Therapeutic Intervention: ERCP not only diagnoses bile leaks but also provides an opportunity for
therapeutic intervention. Stenting can be performed during ERCP to bridge the defect and allow for bile
drainage, promoting healing.
• Minimally Invasive: ERCP is minimally invasive and can often be performed without the need for
surgery, reducing the risk of complications associated with additional surgical procedures.
• Precise Localization: ERCP provides precise localization of the bile leak, aiding in the planning of
further management strategies.
ERCP is the best investigation for this patient for the following reasons:
• Direct Visualization: ERCP allows for direct visualization of the biliary tree, enabling the identification
of the site and extent of the bile leak.
• Therapeutic Intervention: ERCP not only diagnoses bile leaks but also provides an opportunity for
therapeutic intervention. Stenting can be performed during ERCP to bridge the defect and allow for bile
drainage, promoting healing.
• Minimally Invasive: ERCP is minimally invasive and can often be performed without the need for
surgery, reducing the risk of complications associated with additional surgical procedures.
• Precise Localization: ERCP provides precise localization of the bile leak, aiding in the planning of
further management strategies.
Direct Visualization: ERCP allows for direct visualization of the biliary tree, enabling the identification of
the site and extent of the bile leak.
Therapeutic Intervention: ERCP not only diagnoses bile leaks but also provides an opportunity for ther
apeutic intervention. Stenting can be performed during ERCP to bridge the defect and allow for bile dra
inage, promoting healing.

Page 50

539
Minimally Invasive: ERCP is minimally invasive and can often be performed without the need for surger
y, reducing the risk of complications associated with additional surgical procedures.
Precise Localization: ERCP provides precise localization of the bile leak, aiding in the planning of furth
er management strategies.
Incorrect Options:
Option A - MRCP (Magnetic Resonance Cholangiopancreatography): MRCP can be used to visualize t
he biliary tree non-invasively. However, in this case, where a bile leak is already suspected and therap
eutic intervention is required, MRCP may not be the best option as it does not allow for direct therapeut
ic intervention.
Option C - HIDA (Hepatobiliary Iminodiacetic Acid) Scan: HIDA scan is useful for evaluating biliary pate
ncy and assessing bile flow. However, it may not be the best investigation for diagnosing and managin
g a bile leak, as it lacks the capability for direct visualization and therapeutic intervention.
Option D - CECT (Contrast-enhanced Computed Tomography): CECT can detect fluid collections and
may suggest the presence of a bile leak. However, it may not provide the detailed information required
for precise localization and therapeutic intervention that ERCP offers. Additionally, CECT involves radi
ation exposure and may not be the optimal choice when a
more specific investigation like ERCP is available.

Solution for Question 51:


Correct Option D - Mirizzi's syndrome:
• Mirizzi's syndrome is a rare condition characterized by an impacted gallstone in the cystic duct or neck
of the gallbladder, leading to compression or erosion of the common hepatic duct.

• This can result in obstruction of the CBD, leading to jaundice.


• The stone in the cystic duct causes mechanical obstruction and bile flow disruption, leading to the
dilation of the common hepatic duct.

Page 51

540
• Mirizzi's syndrome can mimic other conditions such as choledocholithiasis (stones in the common bile
duct), but the key feature is the impaction of the stone at the cystic duct and subsequent CBD
obstruction.
Incorrect Options:
Option A - Acute cholecystitis: Acute cholecystitis refers to inflammation of the gallbladder, usually due
to gallstone obstruction in the cystic duct. While gallstones can cause cystic duct obstruction, leading t
o dilatation of the common hepatic duct, the specific feature of stone impacting on the cystic duct is not
characteristic of acute cholecystitis.
Option B - Porcelain gallbladder: Porcelain gallbladder refers to the calcification of the gallbladder wall.
It is typically associated with chronic inflammation and scarring. While gallstones can be present in por
celain gallbladder, the specific feature of stone impacting on the cystic duct with CBD obstruction is not
indicative of porcelain gallbladder.
Option C - Cholesterolosis: Cholesterolosis, also known as strawberry gallbladder, refers to the deposit
ion of cholesterol and triglycerides in the gallbladder mucosa, causing it to appear speckled or polypoid
. While cholesterolosis does not typically cause stone impaction on the cystic duct with CBD obstructio
n and jaundice, it can be associated with the formation of gallstones.

Solution for Question 52:


Correct Option A - Surgical excision:
• Dermoid cysts are benign cystic tumors that can contain various tissues, such as hair follicles,
sebaceous glands, and sweat glands.
• Surgical excision, which involves the complete removal of the cyst and its contents, is the preferred
treatment for dermoid cysts.
• Surgical excision is necessary to prevent complications such as infection, rupture, or recurrence.
Incorrect Options:
Option B - Radiotherapy: Radiotherapy is not the preferred treatment for dermoid cysts. Radiotherapy i
s typically reserved for the management of malignant tumors and is not commonly used for benign cyst
s.
Option C - Antibiotics: Antibiotics are not effective in treating dermoid cysts. Dermoid cysts are non-infe
ctious and do not respond to antibiotic treatment.
Option D - Intralesional steroids: Intralesional steroids are not a standard treatment for dermoid cysts.
Steroid injections are usually used in the management of certain inflammatory or autoimmune conditio
ns and are not effective in the treatment of cystic tumors.

Solution for Question 53:


Correct Option A - HCC:
• AFP (Alpha-fetoprotein) is a Hepatocellular Carcinoma (HCC) tumor marker.
• Hepatocellular Carcinoma is the most common type of primary liver cancer, and AFP is one of the key
tumor markers associated with this disease. AFP is a protein the liver produces during fetal

Page 52

541
development, and its levels decrease after birth. However, in certain conditions, such as HCC, the
production of AFP can be reactivated, leading to elevated levels in the blood.
• Measuring AFP levels in the blood can be helpful in screening, diagnosing, and monitoring the
progression of HCC. High levels of AFP are often seen in patients with HCC, although it is important to
note that not all individuals with HCC will have elevated AFP levels. Elevated AFP levels can also be
seen in other conditions such as cirrhosis, hepatitis, and certain germ cell tumors.
Incorrect Options:
Option B - RCC: ESR, CEA, haptoglobin, fibrinogen, and C3 to estimate the staging of RCC and ESR,
CEA, haptoglobin, and C3 to estimate the prognosis.
Option C - Oncocytoma: Oncocytomas are benign tumors commonly found in the kidneys but can also
occur in other organs, such as the salivary glands, thyroid, and parathyroid glands. Unlike malignant tu
mors, oncocytomas do not typically produce specific tumor markers commonly associated with cancer.
Option D - Chordoma: Chordomas are rare, slow-growing malignant tumors that arise from remnants of
the notochord, a structure present during early embryonic development. They usually occur in the skull
base, spine, or sacrum bones. Unlike many other cancers, chordomas do not typically produce specifi
c tumor markers widely used for diagnostic or monitoring purposes.

Solution for Question 54:


Correct Option C - Endoscopic retrograde cholangiopancreatography (ERCP):
• The image provided suggests the procedure of endoscopic retrograde cholangiopancreatography
(ERCP).
• ERCP is a diagnostic and therapeutic procedure used to evaluate and treat disorders of the bile ducts
and pancreatic ducts. It involves the use of an endoscope, a flexible tube with a light and camera,
which is inserted through the mouth and guided down to the duodenum. Contrast dye is then injected
into the bile ducts or pancreatic ducts, and X-ray images are taken to visualize the anatomy and detect
any abnormalities.
• In the given clinical scenario, the patient is experiencing recurrent right upper quadrant pain, which
can be indicative of biliary system pathology. The non-conclusive ultrasound abdomen suggests the
need for further evaluation of the bile ducts. ERCP allows direct visualization of the bile ducts and can
help identify any obstructions, strictures, or other abnormalities that may be causing the patient's
symptoms. It can also be used for therapeutic interventions, such as removing gallstones or placing
stents to relieve obstructions.
Incorrect Options:
Option A - Percutaneous transhepatic cholangiogram: This involves the injection of contrast dye directl
y into the bile ducts through a
needle inserted into the liver. It is typically used when ERCP is not possible or unsuccessful.
Option B - T-tube cholangiogram: This is a radiographic study performed after the placement of a T-tub
e in the common bile duct following surgery. It allows visualization of the bile ducts and assessment of
bile flow.
Option D - Hepatobiliary iminodiacetic acid (HIDA) scan: This nuclear medicine scan is used to evaluat
e the function of the gallbladder and bile ducts. It involves the injection of a
radioactive tracer that is taken up by the liver and excreted into the bile ducts.

Page 53

542
Page 54

543
Esophagus
1. Which of the following is the investigation of choice for a patient with esophageal rupture?
A. Dynamic MRI
B. Rigid esophagoscopy
C. Barium swallow
D. Water soluble low molecular weight contrast swallow
----------------------------------------
2. A 54-year-old male presents with complaints of dysphagia and hematemesis. He has a long history
of smoking and alcohol consumption. He is a known case of advanced carcinoma esophagus and is on
palliative chemotherapy. What is the common site of the CA esophagus for this patient?
(or)
What is the most common site of carcinoma of the esophagus in a patient with a history of smoking and
alcohol consumption?
A. Middle 1/3rd
B. Upper 1/3rd
C. Lower 1/3rd
D. The lower end of the esophagus
----------------------------------------
3. Which of the following is the most common diaphragmatic hernia in a newborn?
(or)
A newborn exhibits respiratory distress and cyanosis, accompanied by X-ray evidence of multiple
bowel loops within the thoracic cavity and a mediastinal shift, what is the most frequently encountered
type of defect associated with these symptoms?
A. Bochdalek
B. Morgagni
C. Paraesophageal type I
D. Paraesophageal type III
----------------------------------------
4. Which of the following statements is true regarding Schatzki's ring?
A. The ring represents a pan mural fibrotic stricture resulting from gastroesophageal reflux
B. Schatzki's ring indicates reflux esophagitis
C. Dysphagia is invariable when the ring diameter is 13 mm or less
D. Schatzki's ring signifies the need for an antireflux operation
----------------------------------------
5. A 50-year-old female presents with complaints of heartburn and regurgitation. She also says that she
feels post-prandial fullness. A barium swallow reveals an upward dislocation of both the cardia and
gastric fundus. Which type of hiatus hernia does this belong to?

544
(or)
What type of hiatus hernia is characterized by an upward dislocation of the cardia and gastric fundus on
barium swallow in a 50-year-old female?
A. Type I
B. Type II
C. Type III
D. Type IV
----------------------------------------
6. Which of the following does a successful treatment of a Zenker's diverticulum involve?
A. Diverticulopexy
B. Resection of the diverticulum
C. Either diverticulopexy or resection with cricopharyngeal myotomy
D. Observation
----------------------------------------
7. Which of the following is a feature of Mackler's triad?
A. Tachypnea
B. Dyspnea
C. Abdominal rigidity
D. Thoracic pain
----------------------------------------
8. In a 30-year-old male presenting to the emergency with throat pain, alleging accidental ingestion of a
fishbone, subsequently identified by endoscopy as lodged 25 cm from the incisors within the
esophagus, at which anatomical level is the foreign body likely located?
(or)
At which approximate anatomical level within the esophagus would a foreign body be situated if it is
lodged 25 cm from the incisors?
A. Left mainstem bronchus
B. Diaphragm
C. Cricopharyngeal constriction
D. Gastroesophageal junction
----------------------------------------
9. Where is Killian's dehiscence located?
A. Below Superior constrictor
B. Between Inferior constrictor
C. Above the cricopharyngeal muscle
D. Below the upper 1/3rd of the smooth muscle of the esophagus
----------------------------------------

Page 2

545
10. What is the most common complication of Ryle tube insertion in a Newborn?
(or)
In newborns presenting with respiratory distress, what is the primary complication associated with
Ryle's tube insertion, prompting chest X-ray assessment by pediatricians?
A. Aspiration pneumonia
B. Dyspnea
C. Pulmonary hypertension
D. Excessive lung fluid
----------------------------------------
11. A 5-year-old male child presented to the OPD with complaints of dyspnea. The X-ray reveals bowel
loops in the thoracic cavity. The X-ray is given below. What is the diagnosis
(or)
What type of diaphragmatic hernia is indicated by the X-ray findings of bowel loops in the thoracic
cavity?

A. Bochdalek hernia
B. Morgagni hernia
C. Hiatus hernia
D. Agenesis of diaphragm
----------------------------------------
12. Which of the following is the most preferred treatment option for Gastroesophageal reflux disease
(GERD) that is irresponsive to medical treatment?
A. Lifestyle modification
B. Double dose of PPI
C. Laparoscopic Nissen’s fundoplication
D. Dietary modification
----------------------------------------
13. A 40-year-old male presented with squeezing chest pain, mimicking angina, and difficulty
swallowing solids and liquids equally. He also complained of having an object stuck in his throat and
having food and liquids return up his esophagus. The barium swallow finding is given below. Which of
the following options is particularly related to this disease condition?

Page 3

546
(or)
What is the treatment of the condition based on the x-ray given below?

A. Caused by secondary contraction of the Esophagus


B. Symptoms are pronounced during times of heightened physical stress
C. Gold standard for diagnosis is Endoscopy
D. The mainstay of treatment is nonsurgical by nitrates/calcium channel blockers or endoscopic
intervention (Bougie dilation)
----------------------------------------
14. A 38-year-old female presented with complaints of dysphagia, regurgitation, and chest pain. She
also described recurring bouts of pneumonia and heartburn. Manometry revealed an abnormality in the
peristaltic movement of the esophagus and LES relaxation. Which of the following statements
delineates the motility disorders of the esophagus?
(or)
What statement describes the motility disorders of the esophagus and LES relaxation in a 40-year-old
female with abnormal peristaltic movement?
A. Best understood motility disorder of the Esophagus is Achalasia
B. Most common hypermotility disorder of the Esophagus is diffuse esophageal spasm
C. Components of Triple A-syndrome or Allgrove disease are Achalasia, alopecia and ACTH- Adrenal
resistant insufficiency
D. For hypertensive LES, esophagectomy is the operation of choice
----------------------------------------
15. Which of the following is not a risk factor for squamous cell carcinoma of the esophagus?
A. Alcohol
B. Caustic injuries
C. HPV infection
D. Obesity
----------------------------------------
16. A 60-year-old male chronic smoker came with dysphagia and weight loss symptoms. Dysphagia
was initially for solids, but it progressed to liquids, too. He now complains of heartburn, chest pain, and
repeated attacks of pneumonia. The barium swallow findings are given below. Which of the following

Page 4

547
statements is true regarding his diagnosis?
(or)
Which of the following statements is true regarding CA esophagus?

A. Overall most common site is the lower third


B. The chemotherapyregimen given is Cisplatin + epirubicin
C. Endoscopic Ultrasound is the investigation of choice for the staging of carcinoma esophagus
D. The most common type of CA Esophagus worldwide is Adenocarcinoma
----------------------------------------
17. A 58-year-old male chronic smoker with a history of progressive dysphagia from solids to liquids
over the last 6 months and weight loss had an endoscopic biopsy that revealed esophageal carcinoma.
The patient underwent a radical esophagectomy. Which of the following features are the replacement
conduits based on after the procedure?
(or)
What is the basis for selecting replacement conduits after a radical esophagectomy in a patient with
carcinoma of the esophagus?
A. The ideal replacement for the Esophagus after esophagectomy is the only oesophagus
B. Gastric conduit is based on the left gastric and left gastroepiploic artery
C. Colonic conduit is based on the right colic artery
D. Gastric conduit is based on the right gastric and right gastroepiploic artery
----------------------------------------
18. What is the most likely diagnosis in a patient with dysphagia and postprandial chest pain whose
barium swallow reveals a ringed esophagus?
(or)
A 43-year-old male with a history of multiple episodes of dysphagia and postprandial chest pain
showed a ringed esophagus on a barium meal X-ray. Which of the following is the most likely
diagnosis?
A. Eosinophilic esophagitis
B. Radiation esophagitis
C. GERD
D. Carcinoma esophagus

Page 5

548
----------------------------------------
19. A 48-year-old male came complaining of odynophagia. On endoscopy, shallow punched-out ulcers
with a few vesicles are seen on the lower 1/3rd of the esophagus. Which of the following is the likely
diagnosis :

A. CMV esophagitis
B. Candida esophagitis
C. Corrosive esophagitis
D. Herpes simplex esophagitis
----------------------------------------
20. Which of the following is a feature of CREST syndrome?
A. Esophageal hypermotility
B. Raynaud’s phenomenon
C. Erythematous malar rash
D. All of these
----------------------------------------
21. Which of the following causes an increase in lower esophageal sphincter (LES) pressure?
A. Alcohol
B. Theophylline
C. Chocolate
D. Prostaglandin F2
----------------------------------------
22. What is the gold standard of investigation for diagnosing gastroesophageal reflux disease (GERD)?
A. Barium swallow
B. Endoscopy
C. 24-hour pH monitoring
D. Esophageal manometry
----------------------------------------

Page 6

549
23. A 25-year-old female came to the OPD with complaints of difficulty swallowing and chest pain.
Signs of ptosis and miosis are also present. The difficulty swallowing was discovered to be caused by
an esophageal anomaly. What is the most accurate diagnosis?
(or)
What is the most accurate diagnosis in patients who have difficulty swallowing and chest pain due to an
anomaly compressing the esophagus?
A. Lou Gehrig's disease
B. Horner's syndrome
C. Kommerell’s diverticulum
D. Dysphagia lusoria
----------------------------------------
24. What is the name of the fundoplication procedure shown in the image?

A. Nissen's fundoplication
B. Watson fundoplication
C. Dor fundoplication
D. Toupet fundoplication
----------------------------------------
25. In Nutcracker's Esophagus, which of the following statements regarding diagnostic criteria is
accurate?
A. Amplitude of contraction > 120 mmHg
B. Duration of contraction > 8 sec
C. Amplitude of contraction > 180 mm Hg
D. Duration of contraction > 4 sec
----------------------------------------
26. Which of the following statements regarding Barrett's esophagus is correct?
A. The squamous mucosa in the upper part of the esophagus is replaced by columnar mucosa
B. The most common columnar epithelium seen in Barrett's esophagus is gastric epithelium
C. The investigation of choice for diagnosis is barium swallow

Page 7

550
D. Endoscopy with biopsy should be performed every 1-2 years to detect dysplasia and
adenocarcinoma.
----------------------------------------
27. Which of the following statements regarding esophageal leiomyoma is incorrect?
A. It is the most common benign tumor of the esophagus
B. It is predominantly found in the upper 1/3rd of the esophagus
C. It occurs more frequently in males
D. The most common symptom is dysphagia and chest pain.
----------------------------------------
28. A 42-year-old female presents with a history of progressively worsening dysphagia for solid foods,
accompanied by fatigue and pallor. On examination, she is noted to have koilonychia and atrophic oral
mucosa. Laboratory investigations reveal chronic iron deficiency anemia. What syndrome is most likely
associated with her presentation?
A. Plummer-Vinson Syndrome
B. Barrett's Esophagus
C. Zenker's Diverticulum
D. Mallory-Weiss Syndrome
----------------------------------------

Correct Answers
Question Correct Answer

Question 1 4
Question 2 1
Question 3 1
Question 4 3
Question 5 3
Question 6 3
Question 7 4
Question 8 1
Question 9 2
Question 10 1
Question 11 2
Question 12 3
Question 13 4
Question 14 1
Question 15 4

Page 8

551
Question 16 3
Question 17 4
Question 18 1
Question 19 4
Question 20 2
Question 21 4
Question 22 3
Question 23 4
Question 24 1
Question 25 3
Question 26 4
Question 27 2
Question 28 1

Solution for Question 1:


Option D: Water soluble low molecular weight contrast swallow
• The investigation of choice for esophageal rupture is water soluble low molecular weight contrast
swallow.
• The use of a water-soluble medium such as Gastrografin is preferred due to the concern of
exacerbating contamination if barium were used and leaked.
• Although barium is superior in demonstrating small perforations as compared with Gastrografin, it
causes an inflammatory response in mediastinal or pleural cavities and is therefore not used as the
initial diagnostic study
• Esophageal rupture: Esophageal rupture is strongly suggested by pain after instrumentation of the
esophagus or substernal in a vomiting patient, particularly in the cervical area. Diagnosis is almost
certain if subcutaneous emphysema is present. Within 8 to 24 hours of injury, extensive contamination
can cause frank sepsis. Investigations: Initial screening evaluation is with a plain upright chest
radiograph which may reveal a pleural effusion, pneumothorax, pneumoperitoneum, subcutaneous or
mediastinal emphysema, or mediastinal widening suggestive of perforation. However normal X-ray
does not rule out the possibility of a leak. The diagnosis is confirmed with a contrast esophagram which
will demonstrate extravasation in 90% of patients. A 10% False negative rate is noted. CT is another
diagnostic modality which can be used.
• Esophageal rupture is strongly suggested by pain after instrumentation of the esophagus or
substernal in a vomiting patient, particularly in the cervical area. Diagnosis is almost certain if
subcutaneous emphysema is present. Within 8 to 24 hours of injury, extensive contamination can
cause frank sepsis.
• Investigations: Initial screening evaluation is with a plain upright chest radiograph which may reveal a
pleural effusion, pneumothorax, pneumoperitoneum, subcutaneous or mediastinal emphysema, or
mediastinal widening suggestive of perforation. However normal X-ray does not rule out the possibility
of a leak.
• Initial screening evaluation is with a plain upright chest radiograph which may reveal a pleural
effusion, pneumothorax, pneumoperitoneum, subcutaneous or mediastinal emphysema, or mediastinal

Page 9

552
widening suggestive of perforation. However normal X-ray does not rule out the possibility of a leak.
• The diagnosis is confirmed with a contrast esophagram which will demonstrate extravasation in 90%
of patients.
• A 10% False negative rate is noted.
• CT is another diagnostic modality which can be used.
• Esophageal rupture is strongly suggested by pain after instrumentation of the esophagus or
substernal in a vomiting patient, particularly in the cervical area. Diagnosis is almost certain if
subcutaneous emphysema is present. Within 8 to 24 hours of injury, extensive contamination can
cause frank sepsis.
• Investigations: Initial screening evaluation is with a plain upright chest radiograph which may reveal a
pleural effusion, pneumothorax, pneumoperitoneum, subcutaneous or mediastinal emphysema, or
mediastinal widening suggestive of perforation. However normal X-ray does not rule out the possibility
of a leak.
• Initial screening evaluation is with a plain upright chest radiograph which may reveal a pleural
effusion, pneumothorax, pneumoperitoneum, subcutaneous or mediastinal emphysema, or mediastinal
widening suggestive of perforation. However normal X-ray does not rule out the possibility of a leak.
• The diagnosis is confirmed with a contrast esophagram which will demonstrate extravasation in 90%
of patients.
• A 10% False negative rate is noted.
• CT is another diagnostic modality which can be used.
• Initial screening evaluation is with a plain upright chest radiograph which may reveal a pleural
effusion, pneumothorax, pneumoperitoneum, subcutaneous or mediastinal emphysema, or mediastinal
widening suggestive of perforation. However normal X-ray does not rule out the possibility of a leak.
Option A: Dynamic MRI
• While MRI can provide detailed images of the esophagus, it is not usually the initial investigation for
esophageal rupture due to the need for rapid diagnosis and the availability of other more immediate
imaging modalities..
Option B: Rigid esophagoscopy
• Rigid esophagoscopy is an invasive procedure that may be used in the management of esophageal
rupture but is not typically the primary investigation for diagnosis.
Option C: Barium swallow
• A barium swallow is not preferred due to its limited sensitivity in identifying small perforations and
potential risks associated with introducing barium into the mediastinum in cases of perforation.
• Barium swallow is the investigation of choice for Zenker’s diverticulum.

Solution for Question 2:


Option A: Middle 1/3rd
• The middle part of the esophagus is the most common location for Squamous cell carcinoma of the
esophagus(SCCE), which is the most common histology found all around the world.

Page 10

553
• Intraepithelial neoplasia, such as dysplasia and carcinoma in situ, can lead to the development of
invasive SCCE if it progresses far enough to invade the lamina propria and spreads beyond the
submucosa.
• Because symptoms appear more commonly in advanced disease, an early diagnosis of SCCE must
not be based on symptoms, and as a result, screening approaches must be employed in asymptomatic
persons who are exposed to risk factors.
• Although esophageal squamous cell carcinoma can develop in the proximal and middle portions of
the esophagus.
Option B: Upper 1/3rd
• The Upper 1/3rd of the esophagus is not the most common site for SCCE.
Option C: Lower 1/3rd
• The lower 1/3rd of the esophagus is not the most common site for SCCE.
Option D: The lower end of the esophagus
• The lower end of the esophagus is not the most common site for SCCE.

Solution for Question 3:


Option A: Bochdalek
• Bochdalek hernia is the most common congenital diaphragmatic hernia.
• Congenital Diaphragmatic hernia: The overall incidence of Bochdalek hernia is 1:2000 to 1:5000. It
occurs due to the failure of closure of the pleuroperitoneal canal in the developing fetus. Abdominal
contents herniate into the thoracic cavity compressing the ipsilateral developing lung. Patients have
respiratory distress and cyanosis at birth. Clinical features: Includes grunting respiration, chest
retractions, dyspnea, and cyanosis with the scaphoid abdomen. Decreased breath sounds, along with
bowel sounds, are heard on auscultating the chest. Shifting of heart sounds to the right. Diagnosis:
Diagnosis made prenatally at 15 weeks of gestation with ultrasound. Ultrasound findings include
herniated abdominal viscera in the chest, changes in liver position, and a mediastinal shift away from
the herniated viscera. Chest radiograph demonstrates multiple bowel loops in the thoracic cavity along
with mediastinal shift. Management: Immediate securing of the airway with endotracheal intubation is
critical. To minimize the degree of pulmonary hypertension, inhaled nitric oxide may be administered.
After achieving hemodynamic stability,a diaphragmmatic repair is done. Fetal tracheal occlusion is an
experimental prenatal therapy for the treatment of severe congenital diaphragmatic hernia that reverses
lung hypoplasia.
• The overall incidence of Bochdalek hernia is 1:2000 to 1:5000.
• It occurs due to the failure of closure of the pleuroperitoneal canal in the developing fetus.
• Abdominal contents herniate into the thoracic cavity compressing the ipsilateral developing lung.
• Patients have respiratory distress and cyanosis at birth.
• Clinical features: Includes grunting respiration, chest retractions, dyspnea, and cyanosis with the
scaphoid abdomen. Decreased breath sounds, along with bowel sounds, are heard on auscultating
the chest. Shifting of heart sounds to the right.
• Includes grunting respiration, chest retractions, dyspnea, and cyanosis with the scaphoid abdomen.
• Decreased breath sounds, along with bowel sounds, are heard on auscultating the chest.

Page 11

554
• Shifting of heart sounds to the right.
• Diagnosis: Diagnosis made prenatally at 15 weeks of gestation with ultrasound. Ultrasound findings
include herniated abdominal viscera in the chest, changes in liver position, and a mediastinal shift away
from the herniated viscera. Chest radiograph demonstrates multiple bowel loops in the thoracic cavity
along with mediastinal shift.
• Diagnosis made prenatally at 15 weeks of gestation with ultrasound.
• Ultrasound findings include herniated abdominal viscera in the chest, changes in liver position, and a
mediastinal shift away from the herniated viscera.
• Chest radiograph demonstrates multiple bowel loops in the thoracic cavity along with mediastinal
shift.
• Management: Immediate securing of the airway with endotracheal intubation is critical. To minimize
the degree of pulmonary hypertension, inhaled nitric oxide may be administered. After achieving
hemodynamic stability,a diaphragmmatic repair is done. Fetal tracheal occlusion is an experimental
prenatal therapy for the treatment of severe congenital diaphragmatic hernia that reverses lung
hypoplasia.
• Immediate securing of the airway with endotracheal intubation is critical.
• To minimize the degree of pulmonary hypertension, inhaled nitric oxide may be administered.
• After achieving hemodynamic stability,a diaphragmmatic repair is done.
• Fetal tracheal occlusion is an experimental prenatal therapy for the treatment of severe congenital
diaphragmatic hernia that reverses lung hypoplasia.
• The overall incidence of Bochdalek hernia is 1:2000 to 1:5000.
• It occurs due to the failure of closure of the pleuroperitoneal canal in the developing fetus.
• Abdominal contents herniate into the thoracic cavity compressing the ipsilateral developing lung.
• Patients have respiratory distress and cyanosis at birth.
• Clinical features: Includes grunting respiration, chest retractions, dyspnea, and cyanosis with the
scaphoid abdomen. Decreased breath sounds, along with bowel sounds, are heard on auscultating
the chest. Shifting of heart sounds to the right.
• Includes grunting respiration, chest retractions, dyspnea, and cyanosis with the scaphoid abdomen.
• Decreased breath sounds, along with bowel sounds, are heard on auscultating the chest.
• Shifting of heart sounds to the right.
• Diagnosis: Diagnosis made prenatally at 15 weeks of gestation with ultrasound. Ultrasound findings
include herniated abdominal viscera in the chest, changes in liver position, and a mediastinal shift away
from the herniated viscera. Chest radiograph demonstrates multiple bowel loops in the thoracic cavity
along with mediastinal shift.
• Diagnosis made prenatally at 15 weeks of gestation with ultrasound.
• Ultrasound findings include herniated abdominal viscera in the chest, changes in liver position, and a
mediastinal shift away from the herniated viscera.
• Chest radiograph demonstrates multiple bowel loops in the thoracic cavity along with mediastinal
shift.
• Management: Immediate securing of the airway with endotracheal intubation is critical. To minimize
the degree of pulmonary hypertension, inhaled nitric oxide may be administered. After achieving
hemodynamic stability,a diaphragmmatic repair is done. Fetal tracheal occlusion is an experimental

Page 12

555
prenatal therapy for the treatment of severe congenital diaphragmatic hernia that reverses lung
hypoplasia.
• Immediate securing of the airway with endotracheal intubation is critical.
• To minimize the degree of pulmonary hypertension, inhaled nitric oxide may be administered.
• After achieving hemodynamic stability,a diaphragmmatic repair is done.
• Fetal tracheal occlusion is an experimental prenatal therapy for the treatment of severe congenital
diaphragmatic hernia that reverses lung hypoplasia.
• Includes grunting respiration, chest retractions, dyspnea, and cyanosis with the scaphoid abdomen.
• Decreased breath sounds, along with bowel sounds, are heard on auscultating the chest.
• Shifting of heart sounds to the right.
• Diagnosis made prenatally at 15 weeks of gestation with ultrasound.
• Ultrasound findings include herniated abdominal viscera in the chest, changes in liver position, and a
mediastinal shift away from the herniated viscera.
• Chest radiograph demonstrates multiple bowel loops in the thoracic cavity along with mediastinal
shift.
• Immediate securing of the airway with endotracheal intubation is critical.
• To minimize the degree of pulmonary hypertension, inhaled nitric oxide may be administered.
• After achieving hemodynamic stability,a diaphragmmatic repair is done.
• Fetal tracheal occlusion is an experimental prenatal therapy for the treatment of severe congenital
diaphragmatic hernia that reverses lung hypoplasia.
Option B: Morgagni
• It is not the most commonly occurring diaphragmatic hernia in newborns.
Option C: Paraesophageal type I
• Paraesophageal type I is also called a Sliding hernia.
• It is not the most commonly occurring diaphragmatic hernia in newborns.
Option D: Paraesophageal type III
• Paraesophageal type III is also called a Mixed paraesophageal hernia
• Herniation of both GE junctions of the fundus of thestomach.

Solution for Question 4:


Option C: Dysphagia is invariable when the ring diameter is 13 mm or less
Schatzki's ring
• At the confluence of the esophagus and the stomach, a radiographic examination will reveal
Schatzki's ring as an annular web-like constriction.
• It is a non-malignant fibrous thickening and constriction of the GEJ that is concentric in nature.
Squamous epithelium is found above it, while columnar cells are found below it.

Page 13

556
• It is characterized by the prominence of the esophagogastric junction and mild submucosal fibrosis.
However, it is not a genuine pan mural fibrotic reflux stricture.
• They are idiopathic and there is no evidence to suggest that they are linked in any way to reflux
esophagitis.
• When the ring size is 20 mm or less, dysphagia may occur occasionally, but when the ring measures
13 mm or less, dysphagia almost always occurs.
• At the precise location of the squamocolumnar epithelial junction, Schatzki's ring can be found.
• The vast majority of rings are incidental discoveries.
• Some of them are accompanied by dysphagia, and in those circumstances, a procedure called
dilatation is performed in conjunction with anti-reflux medical medication.
• Patients with refractory severe reflux symptoms after dilation therapy are candidates for anti-reflux
surgery.

Option A: The ring represents a pan mural fibrotic stricture resulting from gastroesophageal reflux
• It is not a genuine pan mural fibrotic reflux stricture.
Option B: Schatzki's ring indicates reflux esophagitis
• Schatzki's ring does not indicate reflux esophagitis.
Option D: Schatzki's ring signifies the need for an anti-reflux operation
• Schatzki's ring does not signify the need for an anti-reflux operation.
• Incidentally detected asymptomatic patients require no treatment
• For symptomatic patients esophageal dilatation is done
• Surgical excisions are not recommended (because it leads to stricture formation)

Page 14

557
Solution for Question 5:
Option C: Type III

Option A: Type I
• Upward dislocation of only the cardia in the posterior mediastinum occurs in Type I.
Option B: Type II
• Upward dislocation of the gastric fundus anteriorly alongside a normally positioned cardia occurs in
Type II.
Option D: Type IV
• The clinical presentation of the patient is not suggestive of Type IV hiatal hernia.

Solution for Question 6:


Option C: Either diverticulopexy or resection with cricopharyngeal myotomy

Page 15

558
• Surgical or endoscopic repair is the gold standard of treatment.
• Cricopharyngeal Myotomy and Diverticuloplexy is the preferred treatment modality for this patient.
• Open repair involves Diverticulectomy (resection) and Diverticulopexy ( surgical fixation of the
diverticulum).
• In all cases, a myotomy of the proximal and distal thyropharyngeus and cricopharyngeus muscle is
performed. For small diverticula (<2 cm ), a myotomy is sufficient alone. For a large sac (>5 cm), sac
excision is indicated.
• For small diverticula (<2 cm ), a myotomy is sufficient alone.
• For a large sac (>5 cm), sac excision is indicated.
• Endoscopic repair is by Dohlman’s procedure (Diverticulo-esophagostomy).
• For small diverticula (<2 cm ), a myotomy is sufficient alone.
• For a large sac (>5 cm), sac excision is indicated.

Option A: Diverticulopexy
• Diverticulopexy alone is not recommended. This is because diverticulopexy involves attaching the
diverticulum to the surrounding tissue, which may not adequately address the underlying muscle
dysfunction that causes Zenker's diverticulum.

• Along with Diverticulopexy, resection of the diverticulum with cricopharyngeal myotomy can be
performed.
Option B: Resection of the diverticulum
• Resection of the diverticulum, without cricopharyngeal myotomy, may not fully address the underlying
muscle dysfunction that contributes to Zenker's diverticulum.
Option D: Observation
• The patient needs immediate surgical intervention.

Page 16

559
Solution for Question 7:
Option D: Thoracic pain
• Boerhaave's syndrome also known as Spontaneous esophageal perforation.
• Boerhaave's syndrome is characterized by the presence of Mackler's triad. Thoracic pain Vomiting
Cervical subcutaneous emphysema
• Thoracic pain
• Vomiting
• Cervical subcutaneous emphysema
• Thoracic pain
• Vomiting
• Cervical subcutaneous emphysema
INCORRECT OPTIONS:
Options A and C are a part of the Anderson triad, seen in esophageal rupture (Abdominal rigidity, tachy
pnea, and subcutaneous emphysema)
Option B: Dyspnea
• Dyspnea is not a feature of Mackler’s triad.

Solution for Question 8:


Option A: Left mainstem bronchus
• At 25cm, a foreign body can get lodged at the left main stem bronchus or aortic arch.

Page 17

560
• Endoscopy reveals the presence of three different narrowings in the esophagus: The first narrowing
is seen at 15 centimeters down from the upper incisors, at the beginning of the cricopharynx. It has a
diameter of 1.5 centimeters and is the spot where the esophagus is at its narrowest. The second
narrowing is seen at 25 centimeters from the upper incisors and at the arch of the aorta. The third
narrowing is seen at the bronchus at the left main stem, 25 centimeters from the upper incisors. At this
juncture, the diameter of the visible light source is 1.6 cm. The fourth narrowing is seen at 40
centimeters away from the upper incisors and at the diaphragm. Resulting from a malfunction in the
mechanism of the gastroesophageal sphincter. Luminal diameter varies from 1.6 to 1.9 cm. These
normal constrictions have a tendency to hold up foreign objects that have been swallowed, and the
overlying mucosa is susceptible to injury from corrosive liquids that have been swallowed due to the
slow passage of these liquids through these areas.
• The first narrowing is seen at 15 centimeters down from the upper incisors, at the beginning of the
cricopharynx. It has a diameter of 1.5 centimeters and is the spot where the esophagus is at its
narrowest.
• The second narrowing is seen at 25 centimeters from the upper incisors and at the arch of the aorta.
• The third narrowing is seen at the bronchus at the left main stem, 25 centimeters from the upper
incisors. At this juncture, the diameter of the visible light source is 1.6 cm.
• The fourth narrowing is seen at 40 centimeters away from the upper incisors and at the diaphragm.
Resulting from a malfunction in the mechanism of the gastroesophageal sphincter. Luminal diameter
varies from 1.6 to 1.9 cm. These normal constrictions have a tendency to hold up foreign objects that
have been swallowed, and the overlying mucosa is susceptible to injury from corrosive liquids that have
been swallowed due to the slow passage of these liquids through these areas.
• These normal constrictions have a tendency to hold up foreign objects that have been swallowed, and
the overlying mucosa is susceptible to injury from corrosive liquids that have been swallowed due to the
slow passage of these liquids through these areas.
• The first narrowing is seen at 15 centimeters down from the upper incisors, at the beginning of the
cricopharynx. It has a diameter of 1.5 centimeters and is the spot where the esophagus is at its
narrowest.
• The second narrowing is seen at 25 centimeters from the upper incisors and at the arch of the aorta.

Page 18

561
• The third narrowing is seen at the bronchus at the left main stem, 25 centimeters from the upper
incisors. At this juncture, the diameter of the visible light source is 1.6 cm.
• The fourth narrowing is seen at 40 centimeters away from the upper incisors and at the diaphragm.
Resulting from a malfunction in the mechanism of the gastroesophageal sphincter. Luminal diameter
varies from 1.6 to 1.9 cm. These normal constrictions have a tendency to hold up foreign objects that
have been swallowed, and the overlying mucosa is susceptible to injury from corrosive liquids that have
been swallowed due to the slow passage of these liquids through these areas.
• These normal constrictions have a tendency to hold up foreign objects that have been swallowed, and
the overlying mucosa is susceptible to injury from corrosive liquids that have been swallowed due to the
slow passage of these liquids through these areas.
• These normal constrictions have a tendency to hold up foreign objects that have been swallowed, and
the overlying mucosa is susceptible to injury from corrosive liquids that have been swallowed due to the
slow passage of these liquids through these areas.
Option B: Diaphragm
• The fourth narrowing is seen at 40 centimetres away from the upper incisors and at the diaphragm.
Option C: Cricopharyngeal constriction
• The first narrowing is seen at 15 centimetres down from the upper incisors, at the beginning of the
cricopharynx.
Option D: Gastroesophageal junction
• It is not present 25 cm from the incisors.

Solution for Question 9:


Option B: Between Inferior constrictor
• Killian’s dehiscence is a weakness located between the inferior constrictor muscle.
• The dehiscence area of muscular weakness is a triangle zone formed between the pharyngeal and
esophageal muscles, the inferior pharyngeal constrictor, and the cricopharyngeus. This zone is located
in the middle of the pharynx. The surrounding muscles in this area are weak, which gives this region its
distinctive appearance. In medical terminology, the term "dehiscence" refers to an opening that has
gaped open, burst open, or split apart along natural or sutured lines. This type of hole can also be
called a "burst open."
• Between the muscle fibers of the thyropharyngeus and the cricopharyngeus, there is a dehiscence
that is known as Killian's dehiscence. This dehiscence may be found between the two muscles. The
protrusion of mucosa and submucosa that arises as a result of Killian's dehiscence is referred to as
Zenker's diverticulum.
• One can view the dehiscence of Laimer's in the region below the cricopharyngeus.

Page 19

562
Option A: Below Superior constrictor
• Killian's dehiscence is located between the inferior constrictor muscle.
Option C: Above the cricopharyngeal muscle
• It is located below and lateral to the cricopharyngeal muscle.
Option D: Below the upper 1/3rd of the smooth muscle of the esophagus
• Killian’s dehiscence is an area of muscular weakness in the neck.

Solution for Question 10:


Option A: Aspiration pneumonia
• Aspiration is one of the most common complications in enterally-fed patients. The aspiration source is
due to the accumulation of secretions in the pharynx of gastric reflux contents from the stomach into the
pharynx.
• Reviewed literature mentioned the following as complications: Aspiration pneumonia Fatal
hematemesis due to erosion of retro esophageal right subclavian artery Esophageal-aortic fistula and
congenital anomaly of the thoracic aorta Intracranial placement of a nasogastric tube in a patient with
severe head
• Aspiration pneumonia
• Fatal hematemesis due to erosion of retro esophageal right subclavian artery
• Esophageal-aortic fistula and congenital anomaly of the thoracic aorta
• Intracranial placement of a nasogastric tube in a patient with severe head
• Pneumonia is inflammation (swelling) and infection of the lungs or large airways. Aspiration
pneumonia occurs when food or liquid is breathed into the airways or lungs instead of being swallowed.
• Aspiration pneumonia is caused by inhaling foreign materials into the lungs. These materials can be
Bacteria from saliva and secretions from the mouth and nose. Stomach contents such as digestive

Page 20

563
juices or vomit.
• As many as 40% of patients receiving enteral tube feedings aspirate the feedings into their lower
respiratory tract, resulting in pneumonia.
• Dislodged or misplaced enteral feeding tubes, high gastric residual volume (GRV), dysphagia, and
poor oral hygiene are all possible causes of aspiration pneumonia.
• Aspiration pneumonia
• Fatal hematemesis due to erosion of retro esophageal right subclavian artery
• Esophageal-aortic fistula and congenital anomaly of the thoracic aorta
• Intracranial placement of a nasogastric tube in a patient with severe head
Option B: Dyspnea
• The shortness of breath known medically as dyspnea is often described as an intense tightening in
the chest, air hunger, difficulty breathing, breathlessness, or a feeling of suffocation.
• The most common causes of dyspnea are asthma, heart failure, chronic obstructive pulmonary
disease (COPD), interstitial lung disease, pneumonia, and psychogenic problems that are usually
linked to anxiety. If shortness of breath starts suddenly, it is acute dyspnea.
• Dyspnea occurs with GERD because stomach acid that creeps into the esophagus can enter the
lungs, particularly during sleep, and cause swelling of the airways. This can lead to asthma reactions or
cause aspiration pneumonia.
Option C: Pulmonary hypertension
• Pulmonary hypertension appears to be a ubiquitous complication of acute respiratory distress
syndrome.
• Pulmonary hypertension contributes to impaired right ventricular performance and reduced cardiac
output and is associated with increased mortality.
• Pulmonary hypertension is high blood pressure in pulmonary arteries, which carry oxygen-poor blood
from the heart to the lungs. The earliest symptom is shortness of breath during the usual routine. The
most common causes are heart disease, lung disease, and hypoxia.
• Persistent pulmonary hypertension of the newborn (PPHN) is a dangerous condition that may cause a
baby not to get enough oxygen after birth. During pregnancy, babies get all of the oxygen from the
mother through the placenta. The placenta is an organ in the mother's womb connected to the umbilical
cord.
Option D: Excessive lung fluid
• Transient tachypnea of the newborn (TTN) is a condition that causes breathing problems in newborn
babies. Babies have fluid in their lungs before birth.
• The fluid normally goes away when a baby is born. In some babies, the fluid does not go away as
quickly as it should.
• When extra fluid remains in the lungs, a newborn shows respiratory distress. We will provide
breathing support and treat the underlying cause of wet lungs.
• Although it can be difficult to see babies struggle to breathe, their lungs should recover within a few
days to weeks.
• Before birth, a developing fetus does not use the lungs for breathing, all oxygen comes from the
placenta's blood vessels. During this time, the baby's lungs are filled with fluid. As the baby's due date
nears, the lungs begin to absorb the fluid.

Page 21

564
Solution for Question 11:
Option B: Morgagni hernia
• Most commonly, herniated viscera in Morgagni hernia is the transverse colon.
• Morgagni hernias are rare congenital diaphragmatic defects that can lead to bowel obstruction and
incarceration if not repaired. Surgery can be performed through either an open incision or minimally
invasive techniques through the chest or the abdomen.
• Morgagni hernias develop through a defect between the diaphragm's attachments to the sternum and
costal cartilage. They are usually right-sided.
• They constitute 3% of diaphragmatic hernias. Most are asymptomatic, but such patients should still be
considered for surgical repair.

Option A: Bochdalek hernia


• Bochdalek hernia is a developmental defect in the posterolateral diaphragm, allowing herniation of
abdominal contents into the thorax, causing mechanical compression of the developing lung
parenchyma and sometimes causing lung hypoplasia.
• Symptomatic Bochdalek hernias in adults are rarer but may lead to fatal complications. Patients with
acute gastric volvulus on diaphragmatic hernia are a diagnostic and therapeutic emergency.
• It is generally recommended that adults with BH undergo surgical repair to prevent life-threatening
complications due to the incarceration of hernia, obstruction, strangulation, and perforation.
• Surgical reduction of the prolapsed organs and the closure of the hernia orifice is required.
Option C: Hiatus hernia
• A hiatus hernia, or hiatal hernia, is when part of the stomach squeezes up into the chest through an
opening ("hiatus") in the diaphragm.
• The diaphragm is a large, thin sheet of muscle between the chest and the abdomen.
• It is not clear what causes a hiatus hernia. It is more common if a patient is over 50, pregnant or
overweight.
• Hiatal Hernia Symptoms Heartburn from gastroesophageal reflux disease (GERD) Chest pain.
Bloating. Burping. Trouble swallowing.
• Heartburn from gastroesophageal reflux disease (GERD)
• Chest pain.
• Bloating.
• Burping.
• Trouble swallowing.
• Heartburn from gastroesophageal reflux disease (GERD)
• Chest pain.
• Bloating.
• Burping.

Page 22

565
• Trouble swallowing.
Option D: Agenesis of diaphragm
• The diaphragm is a thin layer of muscle and tissue that separates the chest and abdominal cavity. It is
the major muscle that the body uses to breathe.
• Agenesis of the diaphragm is a congenital diaphragmatic developmental anomaly where all or part of
the diaphragm fails to form. It can sometimes be considered an extreme form of congenital
diaphragmatic herniation.
• In the extreme form of diaphragmatic maldevelopment, there might be complete agenesis of the
diaphragm. Neonates with CDH present postnatally with respiratory distress and a characteristic
absence of breath sounds in the ipsilateral chest.

Solution for Question 12:


Option C: Laparoscopic Nissen’s fundoplication
• Gold standard treatment of GERD: Laparoscopic Nissen’s fundoplication
• Laparoscopic Nissen's fundoplication is a minimally invasive procedure to restore the function of the
lower esophageal sphincter (the valve between the esophagus and the stomach) by wrapping the
stomach around the esophagus.
Option A: Lifestyle modification
• Lifestyle modification helps reduce the symptoms of GERD. Following are some of the recommended
modifications. Avoid lying down for at least two hours after a meal or after drinking acidic beverages,
like soda or other caffeinated beverages. Eat smaller and more frequent meals daily instead of a few
large meals. Cessation of smoking Decrease caffeine intake
• Avoid lying down for at least two hours after a meal or after drinking acidic beverages, like soda or
other caffeinated beverages.
• Eat smaller and more frequent meals daily instead of a few large meals.
• Cessation of smoking
• Decrease caffeine intake
• Avoid lying down for at least two hours after a meal or after drinking acidic beverages, like soda or
other caffeinated beverages.
• Eat smaller and more frequent meals daily instead of a few large meals.
• Cessation of smoking
• Decrease caffeine intake

Option B: Double dose of PPI


• Medical management: Double dose of PPI is the initial approach
• Proton-pump inhibitors are a class of medications that cause a profound and prolonged reduction of
stomach acid production
• They irreversibly inhibit the stomach's H■/K■ ATPase proton pump. They are the most potent
inhibitors of acid secretion available.

Page 23

566
• PPIs are generally believed to have few adverse effects and are generally well tolerated. Patients
have experienced minor side effects of short-term PPI use, such as headache, rash, dizziness, and
gastrointestinal symptoms, including nausea, abdominal pain, flatulence, constipation, and diarrhea.
Option D: Dietary modification
• Dietary modification helps reduce the symptoms of GERD.
• Recommendations for the dietary management of GERD include: Limit high-fat foods, e.g. fried foods,
high-fat baked goods, cream, ice cream, high-fat cheeses, sausages, bacon, potato chips, etc. Ensure
adequate protein intake. Limit chocolate and coffee. Foods to eat:
Vegetables, Ginger, Oatmeal, Non-citrus fruits.
• Limit high-fat foods, e.g. fried foods, high-fat baked goods, cream, ice cream, high-fat cheeses,
sausages, bacon, potato chips, etc.
• Ensure adequate protein intake.
• Limit chocolate and coffee.
• Foods to eat: Vegetables, Ginger, Oatmeal, Non-citrus fruits.
• Vegetables, Ginger, Oatmeal, Non-citrus fruits.
• Limit high-fat foods, e.g. fried foods, high-fat baked goods, cream, ice cream, high-fat cheeses,
sausages, bacon, potato chips, etc.
• Ensure adequate protein intake.
• Limit chocolate and coffee.
• Foods to eat: Vegetables, Ginger, Oatmeal, Non-citrus fruits.
• Vegetables, Ginger, Oatmeal, Non-citrus fruits.
• Vegetables, Ginger, Oatmeal, Non-citrus fruits.

Solution for Question 13:


Option D: The mainstay of treatment is nonsurgical by nitrates/calcium channel blockers or endoscopic
intervention (Bougie dilation)
The likely diagnosis of this patient is Diffuse Esophageal Spasm. (the barium swallow shows a
Corkscrew appearance)
• Esophageal contractions are repetitive, simultaneous, and of high amplitude.
• Basic pathology is related to a motor abnormality of the esophageal body that is most notable in the
lower two-thirds of the Esophagus.
• More common in women and is often found in patients with multiple complaints.
• Clinical feature: Chest pain and dysphagia (may be related to eating or exertion and may mimic
angina) Complaints of squeezing pressure in the chest that may radiate to the jaw, arms, and upper
back The symptoms are often pronounced during times of heightened emotional stress Regurgitation of
esophageal contents and saliva is common, but acid reflux is not
• Chest pain and dysphagia (may be related to eating or exertion and may mimic angina)
• Complaints of squeezing pressure in the chest that may radiate to the jaw, arms, and upper back

Page 24

567
• The symptoms are often pronounced during times of heightened emotional stress
• Regurgitation of esophageal contents and saliva is common, but acid reflux is not
• Diagnosis: Barium swallow: Corkscrew or rosary-bead esophagus, segmental spasm, or pseudo
diverticulosis appearance Due to the presence of tertiary contractions Indicative of advanced disease
Manometry is the gold standard test for diagnosis Simultaneous multipeaked contractions of high
amplitude (>120 mmHg) or long duration (>2.5 seconds)
• Barium swallow: Corkscrew or rosary-bead esophagus, segmental spasm, or pseudo diverticulosis
appearance Due to the presence of tertiary contractions Indicative of advanced disease
• Corkscrew or rosary-bead esophagus, segmental spasm, or pseudo diverticulosis appearance
• Due to the presence of tertiary contractions
• Indicative of advanced disease
• Manometry is the gold standard test for diagnosis Simultaneous multipeaked contractions of high
amplitude (>120 mmHg) or long duration (>2.5 seconds)
• Simultaneous multipeaked contractions of high amplitude (>120 mmHg) or long duration (>2.5
seconds)
• Treatment: The mainstay of treatment for DES is nonsurgical and pharmacologic (Nitrates, calcium
channel blockers), or endoscopic intervention (Bougie dilation) is preferred Indications of surgery (long
esophagostomy): Incapacitating chest pain or dysphagia who have failed medical and endoscopic
therapy Presence of pulsion diverticulum of the thoracic Esophagus
• The mainstay of treatment for DES is nonsurgical and pharmacologic (Nitrates, calcium channel
blockers), or endoscopic intervention (Bougie dilation) is preferred
• Indications of surgery (long esophagostomy): Incapacitating chest pain or dysphagia who have failed
medical and endoscopic therapy Presence of pulsion diverticulum of the thoracic Esophagus
• Incapacitating chest pain or dysphagia who have failed medical and endoscopic therapy
• Presence of pulsion diverticulum of the thoracic Esophagus
• Chest pain and dysphagia (may be related to eating or exertion and may mimic angina)
• Complaints of squeezing pressure in the chest that may radiate to the jaw, arms, and upper back
• The symptoms are often pronounced during times of heightened emotional stress
• Regurgitation of esophageal contents and saliva is common, but acid reflux is not
• Barium swallow: Corkscrew or rosary-bead esophagus, segmental spasm, or pseudo diverticulosis
appearance Due to the presence of tertiary contractions Indicative of advanced disease
• Corkscrew or rosary-bead esophagus, segmental spasm, or pseudo diverticulosis appearance
• Due to the presence of tertiary contractions
• Indicative of advanced disease
• Manometry is the gold standard test for diagnosis Simultaneous multipeaked contractions of high
amplitude (>120 mmHg) or long duration (>2.5 seconds)
• Simultaneous multipeaked contractions of high amplitude (>120 mmHg) or long duration (>2.5
seconds)
• Corkscrew or rosary-bead esophagus, segmental spasm, or pseudo diverticulosis appearance
• Due to the presence of tertiary contractions

Page 25

568
• Indicative of advanced disease
• Simultaneous multipeaked contractions of high amplitude (>120 mmHg) or long duration (>2.5
seconds)
• The mainstay of treatment for DES is nonsurgical and pharmacologic (Nitrates, calcium channel
blockers), or endoscopic intervention (Bougie dilation) is preferred
• Indications of surgery (long esophagostomy): Incapacitating chest pain or dysphagia who have failed
medical and endoscopic therapy Presence of pulsion diverticulum of the thoracic Esophagus
• Incapacitating chest pain or dysphagia who have failed medical and endoscopic therapy
• Presence of pulsion diverticulum of the thoracic Esophagus
• Incapacitating chest pain or dysphagia who have failed medical and endoscopic therapy
• Presence of pulsion diverticulum of the thoracic Esophagus
Treatment options include:
• Botulinum toxin (Botox). Botulinum toxin is a poison produced by the bacteria that causes botulism.
During an upper GI endoscopy, a small amount of the Botulinum toxin can be injected into the muscle.
The Botulinum toxin inhibits acetylcholine release from nerve endings in the muscle, which blocks the
function of nerves that make the muscle contract. This procedure may need to be repeated over time.
• Medications to relax the muscles: While medications can help some patients, they are ineffective
overall.
• Peppermint oil: A small amount of water makes the esophagus muscles contract normally again.
• Surgery to cut the muscles along the lower Esophagus. This procedure is usually performed only in
serious cases that do not respond to other therapy.
Option A: Caused by secondary contraction of the Esophagus
• It is due to tertiary contractions of the esophagus.
• The diffuse esophageal spasm occurs due to defective propagation of peristaltic waves through the
esophageal wall. Several segments of the esophagus contract independently simultaneously, thus
causing improper propagation of the food bolus in DES.
Option B: Symptoms are pronounced during times of heightened physical stress
• Symptoms are pronounced during times of heightened emotional stress.
• Esophageal Spasms, such as sudden chest pain or "flutter" that can radiate to the back, neck, jaw,
throat, and arms, are not as common as other anxiety symptoms, but they can occur for some anxious
and stressed people.
Option C: Gold standard for diagnosis is Endoscopy
• The gold standard for diagnosis is Manometry.
• The major manometric criteria for DES are simultaneous contractions on more than 10% of wet
swallows and intermittently normal primary peristalsis.
• Associated findings include repetitive or prolonged-duration contractions, high-amplitude contractions,
and frequent spontaneous contractions. Some patients with DES have normal LES function, but others
have incomplete LES relaxation during swallowing.

Page 26

569
Solution for Question 14:
Option A: Best understood motility disorder of the Esophagus is Achalasia
• Most common motility disorder of the esophagus is Achalasia cardia
• Achalasia is a primary esophageal motility disorder characterized by the absence of esophageal
peristalsis and impaired relaxation of the lower esophageal sphincter (LES) in response to swallowing.
• The LES is hypertensive in about 50% of patients. These abnormalities cause a functional obstruction
at the gastroesophageal junction (GEJ).
Option B: Most common hypermotility disorder of the Esophagus is diffuse esophageal spasm
• The most common hypermotility disorder of the Esophagus is the Nutcracker esophagus.
• The nutcracker esophagus is an abnormality in which swallowing contractions are too powerful. In
up to half of patients, this condition is caused by gastroesophageal reflux.
Option C: Components of Triple A-syndrome or Allgrove disease are Achalasia, alopecia, and ACTH-
Adrenal resistant insufficiency
• Allgrove syndrome or 3A syndrome is characterized by a triad of adrenocorticotrophic hormone
(ACTH) resistant adrenal insufficiency, alacrima, and Achalasia, along with progressive neurological
impairment with or without mild mental retardation.
Option D: For hypertensive LES, esophagectomy is the operation of choice
• For hypertensive LES, laparoscopic Heller's cardiomyotomy is the operation of choice.
• Laparoscopic Heller myotomy is a minimally invasive procedure that opens the tight lower esophageal
sphincter (the valve between the Esophagus and the stomach) by performing a myotomy to relieve the
dysphagia.

Solution for Question 15:


Option D: Obesity
• Obesity is associated with an increased risk of Esophageal Adenocarcinoma (EAC) and a decreased
risk of esophageal squamous cell carcinoma (ESCC). However, little is known about the risk of EAC
and ESCC related to other metabolic risk factors.
• Risk Factors for SCC And Adenocarcinoma SCC (squamous cell carcinoma) ADENOCARCINOMA
Alcohol Smoking Zenker’s diverticula Achalasia cardia Intake of Nitrites Nitrates Nitrosamine Fungal
toxins GERD (gastroesophageal reflux disease) High fat diet Scleroderma - Esophageal dysmotility and
smooth muscle atrophy cause GERD (gastroesophageal reflux disease) Barret’s esophagus HPV
(human papilloma virus) infections (16, 18, 31, 33) Plummer- Vinson syndrome Hot liquid ingestion
Tylosis Palmaris & Plantaris Deficiency of MO -Molybdenum Z -Zinc A -VITAMIN-A Bulimia nervosa
• Alcohol
• Smoking
• Zenker’s diverticula
• Achalasia cardia
• Intake of
• Nitrites

Page 27

570
• Nitrates
• Nitrosamine
• Fungal toxins
• GERD (gastroesophageal reflux disease)
• High fat diet
• Scleroderma - Esophageal dysmotility and smooth muscle atrophy cause GERD (gastroesophageal
reflux disease)
• Barret’s esophagus
• HPV (human papilloma virus) infections (16, 18, 31, 33)
• Plummer- Vinson syndrome
• Hot liquid ingestion
• Tylosis Palmaris & Plantaris
• Deficiency of
• MO -Molybdenum
• Z -Zinc
• A -VITAMIN-A
• Bulimia nervosa
Risk Factors for SCC And Adenocarcinoma
SCC (squamous cell carcinoma)
ADENOCARCINOMA
• Alcohol
• Smoking
• Zenker’s diverticula
• Achalasia cardia
• Intake of
• Nitrites
• Nitrates
• Nitrosamine
• Fungal toxins
• GERD (gastroesophageal reflux disease)
• High fat diet
• Scleroderma - Esophageal dysmotility and smooth muscle atrophy cause GERD (gastroesophageal
reflux disease)
• Barret’s esophagus
• HPV (human papilloma virus) infections (16, 18, 31, 33)
• Plummer- Vinson syndrome

Page 28

571
• Hot liquid ingestion
• Tylosis Palmaris & Plantaris
• Deficiency of
• MO -Molybdenum
• Z -Zinc
• A -VITAMIN-A
• Bulimia nervosa
Option A: Alcohol
• Alcohol over a long period of time increases the risk of squamous cell carcinoma of the Esophagus,
especially when combined with tobacco use.
• Using any form of tobacco, such as cigarettes, cigars, pipes, chewing tobacco, and snuff, raises the
risk of esophageal cancer, especially squamous cell carcinoma.
Option B: Caustic injuries
• Caustic ingestion of acidic or alkaline agents or strong irritants can induce severe esophageal
corrosive injury and increase esophageal cancer risk.
Option C: HPV infection
• Infection with high-risk human papillomavirus (HPV) has been identified as a causal agent in cancers
of some sites, including the cervix, anogenital region, head, and neck. As reported previously, the
high-risk HPV prevalence was 89.7% in cervical cancer, 29.5% in head and neck cancer, and 22.2% in
esophageal cancer.

Solution for Question 16:


Option C: Endoscopic Ultrasound is the investigation of choice for the staging of carcinoma esophagus
• EUS is the most accurate modality for locoregional staging of esophageal cancer. It is a minimally
invasive procedure that uses high-frequency sound waves to visualize the layers of the esophageal
wall and surrounding tissues, thereby evaluating the primary tumor and locoregional adenopathy.
Option A: Overall most common site is the lower third
• The middle third of the Esophagus is the most common site for squamous cell carcinoma.
Option B: Chemotherapy regimen given is Cisplatin + epirubicin
• Common chemotherapy drugs for esophageal cancer are fluorouracil, capecitabine, cisplatin,
epirubicin, and docetaxel. Usually, a combination of 2 or 3 of these drugs is used.
• Chemo drugs for esophageal cancer are typically given into a vein (IV), either as an injection over a
few minutes or as an infusion over a longer period of time. Some drugs you take by mouth. All these
drugs enter your bloodstream and reach most areas of your body.
Option D: The most common type of CA Esophagus worldwide is Adenocarcinoma
• Squamous cell carcinoma occurs most often in the upper and middle portions of the Esophagus.
Squamous cell carcinoma is the most prevalent esophageal cancer worldwide.

Page 29

572
Solution for Question 17:
Option D: Gastric conduit is based on the right gastric and right gastroepiploic artery
• The Esophagus is replaced using another organ, most commonly the stomach but occasionally the
small or large intestine.
• In most circumstances, esophagectomy can be done with minimally invasive surgery, either by
laparoscopy, robot-assisted, or a combination of these approaches.
Option A: Ideal replacement for the Esophagus after esophagectomy is only the esophagus
• The most preferred conduit for esophageal replacement is the stomach. Because of its adequate
length, robust and reliable vascularity, and single anastomosis, the stomach is considered a
time-honored and reliable conduit.
Option B: Gastric conduit is based on the left gastric and left gastroepiploic artery
• A gastric conduit is created using either a whole stomach or a gastric tube, with or without a pyloric
drainage procedure. The gastric conduit with the right gastroepiploic blood supply is brought to the neck
through the left cervical incision.
Option C: Colonic conduit is based on the right colic artery
• The standard colon interposition is based on the ascending branch of the left colic artery from the
inferior mesenteric artery. The middle colic vessels are divided, and the region of the hepatic flexure is
brought up for anastomosis to the residual Esophagus in an isoperistaltic fashion.

Solution for Question 18:


Option A: Eosinophilic esophagitis
• This scenario indicates that the patient has eosinophilic esophagitis.
• Eosinophilic esophagitis is a chronic disease of the immune system.
• The presentation of eosinophilic esophagitis is chest pain (often postprandial) and dysphagia.
Dysphagia may occur with solids or liquids, but solid-food dysphagia is the most common.
• In this disease, eosinophils build up in the esophagus lining, causing inflammation and damage.

• It does not respond to proton pump inhibitors. A barium swallow should be the first test obtained in the
patient with dysphagia. EE has a characterized finding often called the "ringed esophagus" or the
"feline esophagus," as the esophagus rings are felt to look like the strips on a housecat.

Page 30

573
(Feline Esophagus-BARIUM SWALLOW - Normal Esophagus)
• Food allergies mainly trigger it, so dietary avoidance of allergic foods is advised. The second-line
therapy includes inhaled or ingested corticosteroids.
Option B: Radiation esophagitis
• Radiation esophagitis is a consequence of radiotherapy toxicity which causes inflammation or
damage to the esophagus. It can be acute or chronic, depending on the radiographic exposure
duration.
• It is most common in patients receiving radiation cancer therapy for breast, lung, and other
lymphomas. Treatment is most supportive with proper hydration and nutrition intake.
Option C: GERD
• Gastro-esophageal reflux disease develops when the mucosa is exposed to gastric contents for a
long time, which results in esophagitis and heartburn.

Page 31

574
• It can be diagnosed with manometry, barium swallow, and esophagoscopy. Mostly antacids and
proton pump inhibitors are given for the treatment of this condition. But in some cases, surgical
treatment is also required.
Option D: Carcinoma esophagus
• It is esophageal cancer in which malignant cells are formed in the tissues of the esophagus. This
carcinoma is most common in men and presents in old age.
• Important risk factors include smoking, alcohol intake, and nitroso compounds. They can result in
pain, dysphagia, weight loss, and fistula formation. Mostly they are managed surgically, but if
widespread, radiotherapy and chemotherapy are also used.

Solution for Question 19:


Option D: Herpes simplex esophagitis
• Herpes simplex virus (HSV) infection of the esophagus is usually observed in immunocompromised
patients but can occasionally be seen in immunocompetent patients. Most infections are related to HSV
type 1, although HSV-2 has occasionally been reported.
• Dysphagia and odynophagia are the typical symptoms of herpes esophagitis (HE).
• Endoscopic findings reveal shallow erosions or ulcers(punched-out lesions) and vesicles, typically
involving the lower third of the esophagus.
• The three main antiviral drugs for treating herpes esophagitis are acyclovir, famciclovir, and
valacyclovir. Medical or dental professionals may also recommend taking pain medication and eating a
soft, nutrient-rich diet. In contrast, the infection takes time to clear over a few days or weeks.
Option A: CMV esophagitis
• CMV esophagitis is characteristically seen in immunocompromised; endoscopy would show deep
ulcers, most commonly in the middle third of the esophagus.
• The treatment of CMV esophagitis involves induction therapy for 3 to 6 weeks, but the optimal therapy
duration is unclear. Maintenance treatment is controversial. In general, intravenous ganciclovir 5 mg/kg
or foscarnet 90 mg/kg is recommended for induction therapy.
Option B: Candida esophagitis
• Esophageal candidiasis is also commonly seen in people with HIV/AIDS. Patients are usually
asymptomatic. Endoscopy would show white plaques on the esophageal mucosa. A biopsy would show
the presence of yeasts and pseudohyphae invading mucosal cells.
• The most commonly used medication for esophageal candidiasis is the systemic antifungal with oral
fluconazole 200 to 400 mg daily for 14 to 21 days. The alternative for patients who may not tolerate oral
medication is 400 mg of fluconazole intravenously daily.
Option C: Corrosive esophagitis
• Corrosive esophagitis is characterized by caustic injury due to the ingestion of chemical agents,
mainly alkaline substances such as detergents. Esophageal bleeding, perforation, or stricture can be
worsened by high-degree corrosive esophagitis.
• Treatment:
• Medicines may be given to decrease inflammation or irritation from stomach acids. They may
help increase the protective lining of the esophagus to help it heal. Dilatation is a procedure used to

Page 32

575
widen the esophagus. Surgery may be needed to remove an area of the esophagus.
• Medicines may be given to decrease inflammation or irritation from stomach acids. They may
help increase the protective lining of the esophagus to help it heal.
• Dilatation is a procedure used to widen the esophagus.
• Surgery may be needed to remove an area of the esophagus.
• Medicines may be given to decrease inflammation or irritation from stomach acids. They may
help increase the protective lining of the esophagus to help it heal.
• Dilatation is a procedure used to widen the esophagus.
• Surgery may be needed to remove an area of the esophagus.

Solution for Question 20:


Option B: Raynaud’s phenomenon
• CREST syndrome is an autoimmune condition that is associated with atrophy and fibrosis of the
oesophageal musculature, resulting in dysphagia and reflux-type symptoms.
• Manifestations of the CREST syndrome: Calcinosis Raynaud phenomenon The contractions and
spasms of the esophagus Sclerodactyly Telangiectasia
• Calcinosis
• Raynaud phenomenon
• The contractions and spasms of the esophagus
• Sclerodactyly
• Telangiectasia
• Raynaud phenomenon is episodic vasoconstriction of arteries and arterioles, causing numbness and
tingling of the fingers and toes.
• Calcinosis
• Raynaud phenomenon
• The contractions and spasms of the esophagus
• Sclerodactyly
• Telangiectasia
Option A: Esophageal hypermotility
• In CREST syndrome, there is esophageal dysmotility
Option C: Erythematous malar rash
• Erythematous malar rash is not a feature of CREST syndrome
Option D: All of these
• Esophageal hypermobility and erythematous malar rash are not features of CREST syndrome, while
the Raynaud phenomenon is a feature of CREST syndrome.

Page 33

576
Solution for Question 21:
Option D: Prostaglandin F2
• Prostaglandin F2 has an innate effect of increasing the tone of smooth muscles.
• In the lower esophageal sphincter, the smooth muscle ring controls the contraction and relaxation of
LES. When there is relaxation of smooth muscles in LES, the pressure of LES decreases.
• When there is a contraction of the lower esophageal sphincter smooth muscles, then the pressure of
LES increases.
• Prostaglandin F2 increases the tone of the smooth muscles of the LES, causing them to contract,
which increases LES pressure.
Factors influencing LES pressure:
• LES pressure is increased by Neural such as Alpha-adrenergic agonists, beta blocker Hormones such
as gastrin, motilin Peptides such as Bombesin, l-enkephalins, substance P Drugs such as antacids,
domperidone, metoclopramide, prostaglandin F2
• Neural such as Alpha-adrenergic agonists, beta blocker
• Hormones such as gastrin, motilin
• Peptides such as Bombesin, l-enkephalins, substance P
• Drugs such as antacids, domperidone, metoclopramide, prostaglandin F2
• LES pressure is decreased by: Drugs: Prostaglandin E1, E2, Progesterone, estrogen, Peptides
(calcitonin gene-related peptide, gastric inhibitory peptide, neuropeptide Y, Vasoactive intestinal
peptide) Meperidine Theophylline Barbiturates, beta-agonists Diazepam, Dopamine Calcium channel
blockers, cholecystokinin Atropine, alpha-blockers, anticholinergics Nitrates Somatostatin, Secretin
Foods: Chocolate, coffee, caffeine Alcohol Peppermint Fat
• Drugs: Prostaglandin E1, E2, Progesterone, estrogen, Peptides (calcitonin gene-related peptide,
gastric inhibitory peptide, neuropeptide Y, Vasoactive intestinal peptide) Meperidine Theophylline
Barbiturates, beta-agonists Diazepam, Dopamine Calcium channel blockers, cholecystokinin Atropine,
alpha-blockers, anticholinergics Nitrates Somatostatin, Secretin
• Prostaglandin E1, E2, Progesterone, estrogen, Peptides (calcitonin gene-related peptide, gastric
inhibitory peptide, neuropeptide Y, Vasoactive intestinal peptide)
• Meperidine
• Theophylline
• Barbiturates, beta-agonists
• Diazepam, Dopamine
• Calcium channel blockers, cholecystokinin
• Atropine, alpha-blockers, anticholinergics
• Nitrates
• Somatostatin, Secretin
• Foods: Chocolate, coffee, caffeine Alcohol Peppermint Fat

Page 34

577
• Chocolate, coffee, caffeine
• Alcohol
• Peppermint
• Fat
• Neural such as Alpha-adrenergic agonists, beta blocker
• Hormones such as gastrin, motilin
• Peptides such as Bombesin, l-enkephalins, substance P
• Drugs such as antacids, domperidone, metoclopramide, prostaglandin F2
• Drugs: Prostaglandin E1, E2, Progesterone, estrogen, Peptides (calcitonin gene-related peptide,
gastric inhibitory peptide, neuropeptide Y, Vasoactive intestinal peptide) Meperidine Theophylline
Barbiturates, beta-agonists Diazepam, Dopamine Calcium channel blockers, cholecystokinin Atropine,
alpha-blockers, anticholinergics Nitrates Somatostatin, Secretin
• Prostaglandin E1, E2, Progesterone, estrogen, Peptides (calcitonin gene-related peptide, gastric
inhibitory peptide, neuropeptide Y, Vasoactive intestinal peptide)
• Meperidine
• Theophylline
• Barbiturates, beta-agonists
• Diazepam, Dopamine
• Calcium channel blockers, cholecystokinin
• Atropine, alpha-blockers, anticholinergics
• Nitrates
• Somatostatin, Secretin
• Foods: Chocolate, coffee, caffeine Alcohol Peppermint Fat
• Chocolate, coffee, caffeine
• Alcohol
• Peppermint
• Fat
• Prostaglandin E1, E2, Progesterone, estrogen, Peptides (calcitonin gene-related peptide, gastric
inhibitory peptide, neuropeptide Y, Vasoactive intestinal peptide)
• Meperidine
• Theophylline
• Barbiturates, beta-agonists
• Diazepam, Dopamine
• Calcium channel blockers, cholecystokinin
• Atropine, alpha-blockers, anticholinergics
• Nitrates
• Somatostatin, Secretin

Page 35

578
• Chocolate, coffee, caffeine
• Alcohol
• Peppermint
• Fat
Option A: Alcohol
• Alcohol decreases the LES pressure.
Option B: Theophylline
• Theophylline decreases the LES pressure.
Option C: Chocolate
• Chocolate decreases the LES pressure.

Solution for Question 22:


Option C: 24-hour pH monitoring
• Gold standard Investigation for GERD
• Esophageal pH monitoring is used to evaluate gastroesophageal reflux disease and determine the
effectiveness of medications that prevent acid reflux.
• This test measures the amount of acid refluxing or backing up from the stomach into the esophagus.
• The test involves placing a catheter (a thin tube), or a special device called a pH probe into the
esophagus. The catheter or device will measure the acid level (known as pH level) for 24 to 96 hours.
Option A: Barium swallow
• IOC for diagnosis of Hiatus hernia and Zenker’s diverticula
• A barium swallow, also called an esophagogram, is an imaging test that checks for problems in the
upper GI tract.
• The test uses a special type of x-ray called fluoroscopy.
• A chalky white substance is swallowed. It is often mixed with water to make a thick drink that looks like
a milkshake. When swallowed, this liquid coats the inside of the upper GI. Barium absorbs X-rays and
looks white on X-ray film.
Option B: Endoscopy
• IOC for diagnosis of Barrett's esophagus
• An endoscopy is a test to look inside the body.
• A long, thin tube with a small camera inside, called an endoscope, is passed into the body through a
natural opening such as the mouth.
• Endoscopy allows for collecting tissue samples (biopsy) to test for diseases and conditions causing
anemia, bleeding, inflammation, or diarrhea. It can also detect some cancers of the upper digestive
system.
Option D: Esophageal manometry

Page 36

579
• IOC for diagnosis of motility disorders of the esophagus like Achalasia cardia, Diffuse esophageal
spasm, and Nut cracker's esophagus.
• The esophagus is a long, muscular tube that connects the throat to the stomach.
• After swallowing, esophagus contracts and pushes food into the stomach; esophageal manometry
measures the contractions.

Solution for Question 23:


Option D: Dysphagia lusoria
• Disorder of swallowing (Dysphagia) caused by vascular abnormalities because of either vascular rings
(or) pulmonary slings, which causes compression of the esophagus leading to dysphagia.
• Compression of the trachea leads to recurrent respiratory infections and Dyspnea
• Treatment: Both vascular rings and Pulmonary artery slings are Repaired
• Most patients with dysphagia lusoria are asymptomatic; when symptomatic, solid food dysphagia,
coughing, chest pain, and Horner syndrome are the most common symptoms.
Option A: Lou Gehrig's disease
• Amyotrophic Lateral Sclerosis, or ALS, is a progressive nervous system disease that affects nerve
cells in the brain and spinal cord, causing loss of muscle control.
• ALS is often called Lou Gehrig's disease after the baseball player is diagnosed.
• Early symptoms:
• Muscle cramps in the arm, leg, shoulder, or tongue.
• Muscle cramps.
• Tight and stiff muscles (spasticity)
• Causes include gene mutations, an overabundance of the neurotransmitter glutamate (which can be
toxic to nerve cells), and an autoimmune response.
Option B: Horner's syndrome
• Horner syndrome is a relatively rare disorder characterized by a constricted pupil (miosis), drooping of
the upper eyelid (ptosis), absence of sweating of the face (anhidrosis), and sinking of the eyeball into
the bony cavity that protects the eye (enophthalmos).
• The most common causes of Horner syndrome in children include injury to the neck or shoulders
during delivery. Defect of the aorta present at birth. Tumour of the hormonal and nervous systems
(neuroblastoma)
Option C: Kommerell’s diverticulum
• Kommerell's diverticulum is a rare condition that occurs in association with a left aortic arch (LAA) with
an aberrant right subclavian artery or a right aortic arch (RAA) with an aberrant left subclavian artery
(ALSA)
• Kommerell diverticulum, a remnant of the left dorsal arch, is located where the left aberrant subclavian
artery originates.
• It causes external posterior compression of the esophagus.

Page 37

580
• Patients become symptomatic in later decades as the arteries become tortuous and aneurysmal.

Solution for Question 24:


Option A: Nissen’s fundoplication
• A Nissen fundoplication is a surgery to treat gastroesophageal reflux disease (GERD). During the
procedure, a surgeon creates a sphincter (tightening muscle) at the bottom of the esophagus to prevent
acid reflux.
• Most people notice a significant decrease in acid reflux symptoms after the Surgery.

Type of fundoplication
Degree of Wrap
Watson
90 ° anterior fundoplication
Dor
180 ° anterior fundoplication
Toupet
180° posterior fundoplication subsequently modified to 270 °
Belsey Mark IV
270 ° anterior fundoplication
Nissen’s
360 ° fundoplication (most commonly performed)

Option B: Watson fundoplication

Page 38

581
• Traditionally, the anti-reflux surgical procedure of choice has been Nissen's operation. However, this
technique has a significant incidence of mechanical complications and has a reoperation rate of
approximately 7%, leading to the development of alternative approaches.
• Watson's technique of partial anterior fundoplication has been shown to achieve long-lasting reflux
control in adults with few mechanical complications, but there is limited data in the pediatric population.
• Nevertheless, a high incidence of mechanical side effects has been reported, such as dysphagia, an
inability to belch or vomit, and gas bloating, all of which are likely due to the establishment of a
super-competent gastroesophageal sphincter by the 360-degree wrap.

Option C: Dor fundoplication


• Dor fundoplication (a partial wrapping of the stomach around the esophagus to make a low-pressure
valve) is performed to prevent reflux from the stomach into the esophagus following the myotomy.
• The Dor fundoplication procedure consists of a 180-degree anterior fundoplication. This type of
fundoplication is particularly indicated in patients with severe impairment of esophageal body motility.
• It is used to treat achalasia. One of the side effects of Heller myotomy is the development of reflux. In
order to prevent reflux, Dor fundoplication is performed at the time of Surgery.
• Patients with a failed fundoplication due to structural laxity will often present with recurrence of or
persistent GERD symptoms, including heartburn, regurgitation, and erosive esophageal disease such
as peptic stricture formation or reflux esophagitis.

Page 39

582
Option D: Toupet fundoplication
• Laparoscopic Toupet fundoplication is a surgical procedure performed to treat GERD. The procedure
strengthens the valve between the esophagus and the stomach, preventing the backflow of acid.
• The Toupet procedure is the better operation because it has a lower rate of dysphagia and is as
effective as the Nissen fundoplication in controlling reflux.
• Laparoscopic Toupet fundoplication achieved a 5-year healing rate of GERD of 85%. Of all operated
patients, 3.5% had to be reinstalled on a regular PPI treatment because of postoperative GERD
reappearance.
• The Toupet fundoplication or 270-degree wrap is equally effective in controlling GERD but tends to fall
apart with time due to a lack of strong serosal adhesions between the two edges of the wrapped gastric
fundus.

Page 40

583
Solution for Question 25:
Correct Option C - Amplitude of contraction > 180 mmHg:
• Nutcracker's Esophagus is diagnosed based on certain criteria related to esophageal contractions.
The correct statement is option C, which mentions "Amplitude of contraction > 180 mm Hg." This
means that in Nutcracker's Esophagus, the force or strength of esophageal contractions should be
greater than 180 mm Hg. This criterion helps distinguish abnormal, hypermotile contractions in
Nutcracker's Esophagus from normal esophageal motility.

Incorrect Options:
Option A - Amplitude of contraction > 120 mm Hg: This option suggests a lower amplitude threshold of
120 mm Hg. However, the accurate criterion for Nutcracker's Esophagus is an amplitude of contraction
greater than 180 mm Hg. Selecting this option would lead to a
misinterpretation of the diagnostic criteria.
Option B - Duration of contraction > 8 sec: This option focuses on the duration of contraction, setting a
threshold of greater than 8 seconds. In Nutcracker's Esophagus, the correct criterion is a
duration of contraction exceeding 6 seconds. Opting for this choice might lead to confusion, as it does
not align with the established diagnostic criteria.
Option D - Duration of contraction > 4 sec: This option proposes a
lower duration threshold of greater than 4
seconds for the diagnosis of Nutcracker's Esophagus. However, the accurate criterion is a
duration of contraction exceeding 6 seconds. Choosing this option could result in an incorrect interpret
ation of the diagnostic criteria, as it sets a threshold below the established standard.

Solution for Question 26:


Correct Option D - Endoscopy with biopsy should be performed every 1-2 years to detect dysplasia an
d adenocarcinoma:
• Regular surveillance with endoscopy and biopsy is recommended in Barrett's esophagus to detect
dysplasia and early signs of adenocarcinoma. This is because individuals with Barrett's esophagus are
at an increased risk of developing esophageal adenocarcinoma, and periodic monitoring allows for the
early detection and intervention to prevent progression to cancer.

Incorrect Option:
Option A - The squamous mucosa in the upper part of the esophagus is replaced by columnar mucosa:
The squamous mucosa in the upper part of the esophagus is not replaced by columnar mucosa in Bar
rett's esophagus. It is the lower part or distal esophagus where this metaplastic change occurs.
Option B - The most common columnar epithelium seen in Barrett's esophagus is gastric epithelium: T
he most common columnar epithelium seen in Barrett's esophagus is intestinal epithelium, not gastric e
pithelium. The presence of intestinal metaplasia is a key characteristic of Barrett's esophagus.
Option C - The investigation of choice for diagnosis is barium swallow: The investigation of choice for t
he diagnosis of Barrett's esophagus is endoscopy with biopsy, not barium swallow. Endoscopy allows f
or direct visualization of the esophageal mucosa and the collection of biopsy samples for histological e
xamination, providing a more accurate diagnosis.

Page 41

584
Solution for Question 27:
Correct Option B - It is predominantly found in the upper 1/3rd of the esophagus:
• Esophageal leiomyoma is not predominantly found in the upper 1/3rd of the esophagus; instead, it is
more commonly present in the lower 2/3rd of the esophagus. This distinction is important for accurate
clinical evaluation and management.

Incorrect Options:
Option A - It is the most common benign tumor of the esophagus: This statement is correct. Esophage
al leiomyoma is indeed the most common benign tumor of the esophagus.
Option C - It occurs more frequently in males: This statement is correct. Esophageal leiomyoma is actu
ally more common in males.
Option D - The most common symptom is dysphagia and chest pain: This statement is correct. Dyspha
gia and chest pain are the most common symptoms associated with esophageal leiomyoma.

Solution for Question 28:


Correct Option A - Plummer-Vinson Syndrome:
• Plummer-Vinson Syndrome, also known as Paterson Brown Kelly Syndrome or Sideropenic
Dysphagia, is characterized by a triad of dysphagia due to an upper esophageal web, chronic iron
deficiency anemia, and atrophic oral mucosa. Koilonychia (spoon-shaped nails) is another
characteristic feature. Additionally, this syndrome is considered premalignant, increasing the risk of
squamous cell carcinoma (SCC) of the oral cavity, hypopharynx, or esophagus. The achlorhydria and
splenomegaly are associated findings. The patient's presentation with dysphagia, chronic iron
deficiency anemia, atrophic oral mucosa, and koilonychia aligns with the classic features of
Plummer-Vinson Syndrome.

Incorrect Options:
Option B - Barrett's Esophagus: Barrett's esophagus is a condition characterized by metaplastic chang
es in the lower esophageal mucosa due to chronic gastroesophageal reflux disease (GERD). It is asso
ciated with an increased risk of esophageal adenocarcinoma. However, Barrett's esophagus does not t
ypically present with upper esophageal web, atrophic oral mucosa, or koilonychia.
Option C - Zenker's Diverticulum: Zenker's diverticulum is a pulsion diverticulum that occurs in the post
erior hypopharyngeal wall. It is characterized by symptoms such as dysphagia, regurgitation of undiges
ted food, halitosis, and aspiration. While dysphagia is a common feature, the presence of atrophic oral
mucosa, koilonychia, and chronic iron deficiency anemia is not consistent with Zenker's diverticulum.
Option D - Mallory-Weiss Syndrome: Mallory-Weiss Syndrome is characterized by mucosal lacerations
at the gastroesophageal junction, often associated with severe vomiting, such as in cases of alcohol in
toxication or prolonged retching. While it may lead to upper gastrointestinal bleeding, the constellation
of symptoms including atrophic oral mucosa, koilonychia, and chronic iron deficiency anemia is not indi
cative of Mallory-Weiss Syndrome.

Page 42

585
Page 43

586
Stomach and Duodenum Part 1
1. A 62-year-old male presents with dysphagia, vomiting, and pallor. Diagnosed with Stage III gastric
carcinoma involving the body of the stomach extending into the cardia. which of the following is true
about gastric carcinoma?
(or)
What are the potential complications of peptic ulcer surgery in apatient who underwent distal
gastrectomy with reconstruction for an intractable peptic ulcer?
A. More in the low socioeconomic group
B. More common in young females
C. H. pylori infection increases the risks
D. Vitamin C has a protective effect
----------------------------------------
2. A 60-year-old male comes to the outpatient department complaining of loss of appetite and early
satiety. There is no history of allergies or drug addiction. He has a 40-pack-year history. On endoscopic
evaluation, an ulcero-proliferative lesion in the stomach is noted. A biopsy of the lesion is suggestive of
gastric adenocarcinoma. According to Bormann's classification, which of the following most likely
represents Type 1 gastric cancer lesions?
(or)
Which type of gastric cancer, according to Bormann's classification, most likely represents type 1 in a
male with gastric adenocarcinoma?
A. Protruding lesions
B. Ulcerated lesions
C. Flat lesions
D. Excavating lesions
----------------------------------------
3. A 53-year-old male comes to the outpatient department complaining of dysphagia and hematemesis.
Dysphagia affects more solids than liquids. An endoscopic evaluation reveals a mass lesion in the
gastroesophageal junction. Which of the following options is most likely associated with tumorsof the
gastroesophageal junction?
(or)
What is a common association with tumors of the gastroesophageal junction ?
A. Siewert type I
B. Siewert type II
C. Siewert type III
D. Siewert type IV
----------------------------------------
4. A 40-year-old female comes to the outpatient department with complaints of epigastric pain. The pain
is burning in nature, with no associated vomiting or diarrhea. She had multiple similar episodes in the
past. There is no recent history of travel or unsafe intercourse. Vital signs are within normal range. The

587
patient has an endoscopy, which reveals a type III gastric ulcer. Which of the following portions of the
stomach is most likely associated with a gastric ulcer type III?
(or)
Which of the following portions of the stomach is most likely associated with Gastric ulcer type III?
A. Lesser curvature
B. Body
C. Prepyloric region
D. Gastroesophageal junction
----------------------------------------
5. What is the most likely diagnosis in a 2-day-old boy who has persistent vomiting and abdominal
distension with X-ray abdomen showsing a typical double bubble sign.
(or)
A 2-day-old boy is brought to the emergency department due to persistent vomiting and abdominal
distension. The vomit is green and contains food particles. Vital signs are within the normal range. On
physical examination, he seems pale, dehydrated and constantly crying. An X-ray abdomen shows a
typical double bubble. Other investigationshave revealed that this child has trisomy 21. Which of the
following is most likely the diagnosis in this infant?
A. Pyloric stenosis
B. Duodenal atresia
C. Ileal atresia
D. Esophageal atresia
----------------------------------------
6. A 58-year-old male comes to the outpatient department complaining of hematemesis and melena.
He has been suffering from abdominal discomfort and vomiting for many months. His blood pressure is
130/90 mmHg, and his pulse is 60/min. On physical examination, he is pale and thin. There are also
swollen lymph nodes in the left supraclavicular region . On further evaluation with endoscopy and
imaging modalities, the patient is diagnosed with a case of locally advanced gastric carcinoma. Which
of the following is an investigation of choice to know the depth of cancer?
(or)
A 58-year-old male is diagnosed with a locally advanced gastric carcinoma. Which of the following is an
investigation of choice to know the depth of cancer?
A. CECT
B. MRI
C. Barium
D. EUS
----------------------------------------
7. A 32-year-old male comes to the emergency department with complaints of persistent vomiting. The
vomiting is brownish and is associated with abdominal pain. The patient has a history of dyspepsia.
There is no history of smoking, alcohol, or drug abuse. His blood pressure is 100/70 mm Hg, and his
pulse is 80/min. Endoscopic findings are suggestive of upper GI bleed. Which is the most common
cause of upper gastrointestinal tract bleeding?

Page 2

588
(or)
A 32-year-old male has an upper GI bleed. Which is the most common cause of upper gastrointestinal
tract bleeding?
A. Esophageal varices
B. Peptic ulcer
C. Gastritis
D. Mallory-Weiss tear
----------------------------------------
8. A 34-year-old female comes to the emergency department with massive hematemesis. The patient is
hypotensive and tachycardic, with hemoglobinlevels of 9gm/dL. There is no history of allergies, fall, or
accidents. On physical examination, the abdomen is tender to touch. The patient is resuscitated, and
an endoscopy is done. The patient is diagnosed with Dieulafoy’s gastric lesion. Which of the following
statements is most likely associated with Dieulafoy’s gastric lesion?
(or)
A 34-year-old female is diagnosed with Dieulafoy’s gastric lesion. Which of the following statement is
most likely associated with Dieulafoy’s gastric lesion?
A. Longitudinal partial tear due to severe vomiting
B. Gastric antral vascular ectasia
C. Gastric hemorrhagic telangiectasia
D. Aberrant vessel in the submucosa that bleeds from a mucosal defect
----------------------------------------
9. A 45-year-old male is brought to the emergency room in a state of shock. He has a history of chronic
duodenal ulcers. After resuscitation, the patient is sent for investigation. A chest X-ray is given below.
What is the preferred treatment option?
(or)
What is the preferred treatment option for a male in a state of shock, with findings as shown in the given
Chest X-ray?

A. Modified Graham’s repair


B. Truncal vagotomy with antrectomy
C. Truncal vagotomy with gastrojejunostomy

Page 3

589
D. Billroth II gastrectomy
----------------------------------------
10. A 56-year-old male is diagnosed with gastric adenocarcinoma. Which of the following anemiais a
risk factor for the development of gastric carcinoma?
(or)
A 56-year-old male comes to the outpatient department complaining of ball-rolling movements and
projectile vomiting of undigested food particles. There is no history of allergy or drug abuse. Vital signs
are within normal range. Physical examination reveals visible gastric peristalsis from left to right and a
positive succussion splash test. Endoscopy reveals a gastric mass obstructing the pylorus. A biopsy of
the lesion revealed gastric adenocarcinoma. Which of the following anemia is a risk factor for
developing gastric carcinoma?
A. Pernicious anemia
B. Megaloblastic anemia
C. Aplastic anemia
D. Hemolytic anemia
----------------------------------------
11. A 60-year-old male comes to the outpatient department to evaluate weight loss. The patient is
diabetic and hypertensive. His blood pressure is 140/95 mmHg, and his pulse is 70/min. After different
examinations and lab tests, he was diagnosed with gastric adenocarcinoma in the antrum of the
stomach. The patient underwent a D2 subtotal gastrectomy. During D2 dissection, levels 1-11 lymph
nodes are removed. Which of the following includes the Level 9 lymph node station?
(or)
Which lymph node station is included in Level 9 during a D2 subtotal gastrectomy for gastric
adenocarcinoma in the antrum of the stomach?
A. Celiac nodes
B. Splenic hilum
C. Splenic artery
D. Hepatoduodenal
----------------------------------------
12. A newborn with bilious vomiting and signs of dehydration is diagnosed with duodenal atresia from a
double bubble appearance on a plain abdominal radiograph. What is the recommended treatment?
(or)
What is the treatment of choice for duodenal atresia, characterized by a double bubble appearance in a
newborn infant?
A. Duodenoduodenostomy
B. Duodenojejunostomy
C. Bishop-koop procedure
D. Gastroduodenostomy
----------------------------------------

Page 4

590
13. A 47-year-old male presents to the casualty department with a GCS of 8/15 following a road traffic
accident. The patient sustained a head injury, and a CT scan showed a subarachnoid hemorrhage. The
patient is managed with antiepileptics and diuretics. On the second day, blood-stained fluid is seen
coming out through Ryle's tube, which can be due to stress-induced gastritis. Which of the following
statements represents stress ulcers?
(or)
Which statement best represents stress ulcers ?
A. Mucosal resistance is increased in curling ulcers
B. Blood flow to the stomach increases in curling ulcers
C. The most common sites are the fundus and the body of the stomach
D. In curling ulcers, acid secretion in the stomach is increased
----------------------------------------
14. A 45-year-old female with chronic NSAID abuse presents with massive hematemesis, hypotension,
and tachycardia. Physical examination shows abdominal tenderness. IV proton pump inhibitors are
administered, and endoscopy reveals a bleeding peptic ulcer in the proximal duodenum. Which of the
following vessels needs to be ligated in this patient to stop the bleeding?
(or)
Which vessel requires ligation to control bleeding in a bleeding peptic ulcer in the proximal duodenum?
A. Gastroduodenal artery
B. Superior pancreaticoduodenal artery
C. Left gastric artery
D. Left gastroepiploic artery
----------------------------------------
15. A 4-week-old male baby is brought to the casualty by his parents with complaints of recurrent
episodes of non-bilious vomiting. The child also refuses to take adequate feeds. The child is
hypotensive and tachypneic. Ultrasound abdomen reveals features consistent with hypertrophic pyloric
stenosis. Which of the following is most likely associated with this patient?
(or)
Which condition is commonly associated with hypertrophic pyloric stenosis in a 4-week-old male baby?
A. Commonly seen after 13 weeks
B. The second-born male is commonly affected
C. Ramstedt operation is done
D. Visible peristalsis is always seen
----------------------------------------
16. A 56-year-old man with early satiety and a palpable mass in the abdomen was diagnosed with
diffuse gastric cancer through endoscopy and biopsy. According to Lauren's classification, what is most
likely associated with diffuse gastric cancer?
(or)
According to Lauren's classification, which of the following is most likely associated with diffuse gastric
cancer?

Page 5

591
A. Associated with APC Gene mutation
B. More common in males
C. affects the body of the stomach
D. More common in the distal part
----------------------------------------
17. What is the most common metabolic complication following partial gastrectomy?
(or)
A 40-year-old known alcoholic male presents to the emergency department with complaints of
abdominal pain, abdominal distension, and vomiting. The patient is tachycardic and hypotensive. An
abdominal X-ray reveals gas under the diaphragm. The patient is immediately resuscitated and taken
up for emergency laparotomy. Intraoperatively, a large gastric perforation of 3x3 cm is noted; hence,
partial gastrectomy with reconstruction is done. Postoperatively, the patient most commonly develops
which of the following metabolic complications?
A. Iron deficiency anemia
B. Megaloblastic anemia
C. Hypocalcemia
D. Osteoporosis
----------------------------------------
18. Which category in Forrest's classification is associated with an intermediate risk of rebleeding after
electrocoagulation for bleeding peptic ulcer ?
(or)
A 37-year-old male presents to the OPD with complaints of hematemesis. The patient is a known case
of peptic ulcer disease taking irregular treatment. Endoscopy reveals the site of bleeding, and
electrocoagulation is done. According to Forrest’s classification, which of the following is associated
with an intermediate risk of rebleeding?
A. Visible vessel
B. Visible pulsatile bleeding
C. Adherent clot
D. Visible oozing from the vessel
----------------------------------------
19. A 34-year-old male presents to the OPD with complaints of recurrent episodes of forceful vomiting
followed by hematemesis. Endoscopy reveals a longitudinal tear in the esophagogastric junction,
suggestive of Mallory-Weiss syndrome. The doctor noticed that the patient had presented with a
feature rarely seen in Mallory-Weiss syndrome. Which of the following is that feature?
(or)
What is the rare feature observed in a with Mallory-Weiss syndrome?
A. Massive hemorrhage
B. Forceful contraction of the abdominal wall
C. Most tears occur along the lesser curvature

Page 6

592
D. Most of the bleeding episodes are self-limited
----------------------------------------
20. A 55-year-old male presents to the OPD with complaints of weight loss and appetite. He also
complains of swelling around the umbilical region. On further examination, the swelling was diagnosed
as Sister Joseph's nodule. This nodule indicates cancer of which of the following structures?
(or)
In a male with swelling around the umbilical region diagnosed as Sister Joseph's nodule, which
structure is indicated to have cancer?
A. Rectum
B. Gall bladder
C. Stomach
D. Fallopian tubes
----------------------------------------
21. A 60-year-old male is diagnosed with a carcinoma stomach. A CT scan of the abdomen shows a
mass measuring 4x4 cm in the antrum. The mass indicates celiac node and right gastric node
involvement. Which of the following is the preferred management option?
(or)
What is the management of choice for with carcinoma of the stomach involving the antrum, celiac
nodes, and right gastric nodes, as seen on CT scan?
A. Total gastrectomy
B. Subtotal gastrectomy
C. Palliative
D. Chemotherapy
----------------------------------------
22. What is the likely diagnosis in a female with alopecia with exclamation mark hair and crepitus in the
epigastrium?
(or)
A 23-year-old female presents to the emergency department with complaints of abdominal pain,
abdominal distention, and vomiting. Upon examination, she is found to have alopecia with exclamation
mark hair. She also has crepitus in the epigastrium. What is the diagnosis?
A. Trichobezoar
B. Carcinoma pyloric antrum
C. Intestinal tuberculosis
D. Rectus sheath hematoma
----------------------------------------
23. What is the endoscopic finding for a male with a traumatic esophageal perforation and active
bleeding filling the NG tube?
(or)

Page 7

593
A 34-year-old male is admitted to the ICU following a thoracotomy for a traumatic esophageal
perforation. He has a high TLC count and an abnormal coagulation profile, as well as being febrile and
hypotensive. An intern puts in an NG tube, but it quickly fills with blood and continues to bleed. What
would be the endoscopic finding for this case of stress gastritis?
A. Single deep ulcer in the fundus
B. Multiple, shallow ulcers with areas of erythema and focal hemorrhage in the antrum
C. Multiple, shallow ulcers with areas of erythema and focal hemorrhage in the fundus
D. Multiple deep ulcers in the antrum
----------------------------------------

Correct Answers
Question Correct Answer

Question 1 2
Question 2 1
Question 3 2
Question 4 3
Question 5 2
Question 6 4
Question 7 2
Question 8 4
Question 9 1
Question 10 1
Question 11 1
Question 12 1
Question 13 3
Question 14 1
Question 15 3
Question 16 3
Question 17 1
Question 18 3
Question 19 1
Question 20 3
Question 21 2
Question 22 1
Question 23 3

Page 8

594
Solution for Question 1:
Option B: More common in young females
• This patient is suffering from stage III gastric carcinoma in the body of the stomach extending into
cardia. It is more common in older men rather than young females.
Option A: More in the low socioeconomic group
• Stage III gastric carcinoma is more common in the low socioeconomic group due to low food quality or
a higher risk of infections.
Option C: H. pylori infection increases the risks
• H. pylori infection increases the risks of gastric carcinoma.
• Other diseases that increase the risk of gastric carcinoma include: Epstein-Barr virus Gastric atrophy
and gastritis Adenomatous polyps
• Epstein-Barr virus
• Gastric atrophy and gastritis
• Adenomatous polyps
• Epstein-Barr virus
• Gastric atrophy and gastritis
• Adenomatous polyps
Option D: Vitamin C has a protective effect
• Vitamin C has a protective effect against stage III gastric carcinoma.
• The following substances also decrease the risk: Aspirin High fresh fruit and vegetable intake
• Aspirin
• High fresh fruit and vegetable intake
• Aspirin
• High fresh fruit and vegetable intake

Solution for Question 2:


Option A: Protruding lesions
• These fungating tumors typically contain friable and ulcerated masses that can bleed easily.
• They usually arise from the broad base in the gastric lumen.
• The most common location is greater curvature of the stomach.
Option B: Ulcerated lesions
• Ulcerated tumors consist of irregular margins and raised borders.
• The base of the ulcer is necrotic and nodular.
• The surrounding mucosa is indurated and thickened.
Option C: Flat lesions

Page 9

595
• The infiltrating tumors are superficially spreading in mucosa and submucosa.
• It often gives a flat or plaque-like appearance on biopsy.
Option D: Excavating lesions
• Excavating tumors occur when infiltrations spread to the whole of the stomach wall.
• It gives the stomach a stiff consistency.
• These are limited to a specific area and give a "linitis plastica” or “leather-stomach” appearance.
• These tumors are not visible on the surface.

Solution for Question 3:


Option B: Siewert type II
• Siewert classification is a system of anatomical classification used for adenocarcinoma of the
esophagogastric junction.
• In type Siewert type II, Adenocarcinoma is present in the real cardia.
• Its exact location is 1 cm above and 2 cm below the gastroesophageal junction add often called
junctional carcinoma.
Option A: Siewert type I
• In Siewert type I, adenocarcinoma is present in the distal esophagus.
• Usually arises from an area with specialized intestinal metaplasia of the esophagus (ie, Barrett's
esophagus) and that may infiltrate the EGJ from above.
• It is 1-5 cm above the gastroesophageal (GE) junction.
Option C: Siewert type III
• In Siewert type III, Adenocarcinoma is present in the Sub-Cardinal Stomach.
• Its location is 2-5 cm below the gastroesophageal (GE) junction.
Option D: Siewert type IV
• This classification has only three types.

Solution for Question 4:


Option C: Prepyloric region
• According to Modified Johnson's Classification of Gastric Ulcer, Type III gastric ulcer is associated
with the pre pyloric region of the stomach that is located within 2-3 cm of the pylorus.
• In this type of ulcer, acid secretion will be high.
• Its rate of occurrence is 20%.
• It has the following complications: Bleeding Perforation Obstruction Modified Johnson's Classification
of Gastric Ulcer: Type Location Acid Secretion I Lesser curvature near incisura angularis (Most
common,60%) Low to normal II Body of the stomach and duodenum (15%) High III Prepyloric (Within

Page 10

596
2-3 cm of the pylorus) (20%) High IV High on the lesser curve, near GE junction (<10%) Normal V
Anywhere, induced by medication (NSAIDs) Normal
• Bleeding
• Perforation
• Obstruction
• Modified Johnson's Classification of Gastric Ulcer:
• Type Location Acid Secretion I Lesser curvature near incisura angularis (Most common,60%) Low to
normal II Body of the stomach and duodenum (15%) High III Prepyloric (Within 2-3 cm of the pylorus)
(20%) High IV High on the lesser curve, near GE junction (<10%) Normal V Anywhere, induced by
medication (NSAIDs) Normal
• Bleeding
• Perforation
• Obstruction
• Modified Johnson's Classification of Gastric Ulcer:
• Type Location Acid Secretion I Lesser curvature near incisura angularis (Most common,60%) Low to
normal II Body of the stomach and duodenum (15%) High III Prepyloric (Within 2-3 cm of the pylorus)
(20%) High IV High on the lesser curve, near GE junction (<10%) Normal V Anywhere, induced by
medication (NSAIDs) Normal
Option A: Lesser curvature
• According to Modified Johnson's Classification of Gastric Ulcer, Type I gastric ulcer is associated with
lesser curvature near incisura angularis.
• It is the most common type of gastric ulcer.
• Its rate of occurrence is 60%.
• The acid secretion will be low or normal in this ulcer.
Option B: Body
• According to Modified Johnson's Classification of Gastric Ulcer, Type II gastric ulcer is associated with
the body of the stomach and duodenum.
• Its rate of occurrence is 15%.
• In this type of ulcer, acid secretion will be high.
Option D: Gastroesophageal junction
• According to Modified Johnson's Classification of Gastric Ulcer, Type IV gastric ulcer is associated
with Gastroesophageal junction (GE junction).
• Its rate of occurrence is less than 10%.
• In this type of ulcer, acid secretion will be normal.

Solution for Question 5:


Option B: Duodenal atresia

Page 11

597
• This infant with vomiting, abdominal distension, and an x-ray showing a double-bubble sign suffers
from duodenal atresia.
• Duodenal atresia occurs due to the failure of the recanalization of the lumen.
• It causes : Mild epigastric pain Abdominal distension Emesis.
• Mild epigastric pain
• Abdominal distension
• Emesis.
• It is most commonly associated with Down syndrome (trisomy 21).
• The x-ray shows the two dilations, gastric and proximal duodenal dilation, as given in the image.
• Mild epigastric pain
• Abdominal distension
• Emesis.

Option A: Pyloric stenosis


• Pyloric stenosis occurs due to the pylorus thickening, resulting in gastric outlet obstruction.
• Thickening occurs due to the hypertrophy of the circular layer of smooth muscles surrounding the
pylorus of the stomach.
• It has the following signs/symptoms: Projectile non-bilious vomiting. Sunken fontanelle (signs of
dehydration) Palpable Olive shaped epigastric mass.
• Projectile non-bilious vomiting.
• Sunken fontanelle (signs of dehydration)
• Palpable Olive shaped epigastric mass.
• It has the following risk factors: Firstborn male Positive family history Macrolide antibiotic exposure
• Firstborn male
• Positive family history

Page 12

598
• Macrolide antibiotic exposure
• It mainly develops during 3 to 6 weeks of age, and a delayed diagnosis can result in hypochloremic
metabolic alkalosis during vomiting.
• Unconjugated hyperbilirubinemia is also seen due to dehydration.
• On the x-ray, it shows a single-bubble sign.
• Projectile non-bilious vomiting.
• Sunken fontanelle (signs of dehydration)
• Palpable Olive shaped epigastric mass.
• Firstborn male
• Positive family history
• Macrolide antibiotic exposure
Option C: Ileal atresia
• It occurs due to vascular occlusion.
• It is characterized by marked abdominal distension and bilious emesis.
• X-ray shows dilated loops of small bowel filled with air.
• It is commonly associated with gastroschisis and volvulus.
• Maternal use of nicotine is one of the risk factors.
Option D: Esophageal atresia
• Esophageal atresia and trachea-esophageal fistula (TEF) occur mostly due to abnormal separation of
the trachea-bronchial diverticulum.
• It is presented as polyhydramnios.
• Neonate shows the following signs with the first feed: Drooling Choking Vomiting Cyanosis
• Drooling
• Choking
• Vomiting
• Cyanosis
• X-ray shows constricted esophageal tube and gastric dilation with TEF due to air moving from the
trachea to the stomach.
• Drooling
• Choking
• Vomiting
• Cyanosis

Solution for Question 6:


Option D: EUS

Page 13

599
• EUS (endoscopic ultrasound) is a minimally invasive procedure to assess the digestive tract and
nearby organs.
• It uses a thin, flexible tube inserted into the gastrointestinal tract and a device that uses sound waves
to create images.
• This endoscopic tube also has a small needle to remove fluid or tissue samples for biopsy.
• The best way to stage the tumor locally is via EUS, which gives fairly accurate (80%) information
about the depth of tumor penetration into the gastric wall and can show enlarged (>5mm) perigastric
and celiac lymph nodes.
Option A: CECT
• CECT is Contrast-enhanced Computed tomography.
• It is used for imaging the different structures, and the contrast helps to improve the visualization.
• In this procedure, a beam of x-ray moves all around the body and different images are taken to form a
dimensional image.
• During surgeries, it is used to place a needle at a specific point for biopsies or aspiration.
• It helps to visualize the tumors or other diseases before or after treatment.
Option B: MRI
• MRI (magnetic resonance) is an imaging test that uses powerful magnets and radio waves.
• These waves create images of the inside of the abdomen and do not use radiation.
• Single MRI images are called slices stored on a computer and viewed on a monitor.
• MRI is mainly used for brain and spinal cord injuries (stroke, tumour, aneurysms)
• MRI of the abdomen is mainly used for the following: Blood flow in the abdomen Blood vessels in the
abdomen Lymph nodes Masses in the liver, kidney, pancreas, and spleen.
• Blood flow in the abdomen
• Blood vessels in the abdomen
• Lymph nodes
• Masses in the liver, kidney, pancreas, and spleen.
• Blood flow in the abdomen
• Blood vessels in the abdomen
• Lymph nodes
• Masses in the liver, kidney, pancreas, and spleen.
Option C: Barium
• Different types of barium x-ray examinations are used to determine different parts.
• A barium swallow is used to study the pharynx and esophagus.
• A mixture of barium is swallowed in the supine position, and images are taken.
• A barium meal is used to study the lower esophagus and stomach.
• Barium follow-through is used to study the small intestine.
• A barium enema is used to study the large intestine and rectum.

Page 14

600
• It is used for the following purposes: Diagnosis of tumor Location of tumor Ulceration Strictures
• Diagnosis of tumor
• Location of tumor
• Ulceration
• Strictures
• Diagnosis of tumor
• Location of tumor
• Ulceration
• Strictures

Solution for Question 7:


Option B: Peptic ulcer
• Upper gastrointestinal bleeding originates proximal to the ligament of Treitz (suspensory ligament of
the duodenum).
• It usually presents with hematemesis and melena.
• Peptic ulcers are open sores in the stomach (gastric ulcer) and the upper part of the duodenum
(duodenal ulcer).
• A peptic ulcer is the most common cause of upper GI bleeding, present in one-half to two-thirds of
patients with upper GI bleeding.
• It has the following two types:

Option A: Esophageal varices


• These are abnormally enlarged veins in the esophagus, mostly in the lower portion.
• It mostly occurs in people suffering from liver disease (cirrhosis).

Page 15

601
• The main causes include the following:
• Cirrhosis
• Portal vein thrombosis
• Budd-Chiari syndrome
• Schistosomiasis infection.
• It is presented by the following:
• Vomiting large amounts of blood
• Tarry stools
Option C: Gastritis
• Inflammation of the stomach wall is called gastritis.
• Erosions cause acute gastritis due to the following:
• NSAIDs.
• Hypovolemia
• Increase acid production (in vagal stimulation due to brain injury)
• Chronic gastritis occurs due to mucosal inflammation, often leading to atrophy and intestinal
metaplasia.
• The most common causes include the following:
• H. pylori infection (affects antrum first)
• Autoimmune (antibodies against the H+/K+ ATPase, affects body/fundus).
• Acute gastritis is most common in alcoholics using NSAIDs (rheumatoid arthritis).
• It is usually presented by the following signs/symptoms:
• Abdominal pain
• Nausea
• Vomiting (with blood)
• Abdominal fullness after eating.
Option D: Mallory-Weiss tear
• Longitudinal partial tears characterize Mallory-Weiss Syndrome due to severe vomiting.
• It is only confined to mucosa and submucosa involved.
• The most common site is cardia (just below GE junction.)
• More common in alcoholic males and bulimia nervosa girls.
• Usually present with hematemesis and abdominal/back pain.

Solution for Question 8:


Option D: Aberrant vessel in the submucosa that bleeds from a mucosal defect

Page 16

602
• Dieulafoy’s gastric lesion is an aberrant vessel in the submucosa that bleeds from a mucosal defect.
• It is presented as hematemesis, melena, anemia and abdominal pain.
• Unlike peptic ulcer disease, there is no such history of alcohol and NSAIDs.
• It most commonly occurs in the upper part of the stomach, near the gastroesophageal junction.
Option A: Longitudinal partial tear due to severe vomiting
• Longitudinal partial tears characterize Mallory-Weiss Syndrome due to severe vomiting.
• It is only confined to mucosa and submucosa involved.
• The most common site is cardia (just below GE junction.)
• More common in alcoholic males and bulimia nervosa girls.
• Usually present with hematemesis and abdominal/back pain.
Option B: Gastric antral vascular ectasia
• Gastric antral vascular ectasia (GAVE) is a condition in which blood vessels of the stomach become
fragile and prone to rupture and bleeding.
• The stomach shows a watermelon sign on endoscopy.
• It is presented by the following:
• Acute bleeding
• Bloody vomitus
• Tarry stools
• Anemia
Option C: Gastric hemorrhagic telangiectasia
• It is a hereditary disorder in which blood vessels do not develop properly.
• The patient is usually present by the following:
• Epistaxis
• Gastrointestinal bleeding
• Telangiectasia
• Arteriovenous malformations
• Aneurysms

Solution for Question 9:


Option A: Modified Graham’s repair
• X-ray shows gas under the right dome of the diaphragm, which represents the pneumoperitoneum.
• Pneumoperitoneum: Anteriorly located ulcers present with perforations. Posteriorly located ulcer
present with bleeding due to gastroduodenal artery. These ulcers result in massive bleeding. Surgery
of choice is Omental patch repair or modified graham repair.
• Anteriorly located ulcers present with perforations.

Page 17

603
• Posteriorly located ulcer present with bleeding due to gastroduodenal artery. These ulcers result in
massive bleeding.
• Surgery of choice is Omental patch repair or modified graham repair.
• A perforated peptic ulcer is the second most common complication of peptic ulcer.
• Surgery is almost always indicated for ulcer perforation.
• The options for surgical treatment are: Simple patch closure (procedure of choice) Patch closure and
highly selective vagotomy Patch closure and vagotomy, and drainage
• Simple patch closure (procedure of choice)
• Patch closure and highly selective vagotomy
• Patch closure and vagotomy, and drainage
• Anteriorly located ulcers present with perforations.
• Posteriorly located ulcer present with bleeding due to gastroduodenal artery. These ulcers result in
massive bleeding.
• Surgery of choice is Omental patch repair or modified graham repair.
• Simple patch closure (procedure of choice)
• Patch closure and highly selective vagotomy
• Patch closure and vagotomy, and drainage

Option B: Truncal vagotomy with antrectomy


• Truncal vagotomy with antrectomy is used in the management of type II and III gastric ulcers.
Option C: Truncal vagotomy with gastrojejunostomy
• Truncal vagotomy with gastrojejunostomy is one of the treatment methods for ulcers.
Option D: Billroth II gastrectomy
• Billroth II gastrectomy is not the preferred treatment for perforation peritonitis.
• Dumping syndrome is treated with a Billroth II gastrectomy.

Solution for Question 10:


Option A: Pernicious anemia
• It is an autoimmune disease due to the CD4+ cell-mediated immune response or antibodies against
the stomach's parietal cells.
• In pernicious anemia, there is progressive destruction of parietal cells, and it results in:
• Decrease hydrochloric acid secretion, which increases gastric pH.
• Decrease intrinsic factor secretion impairs vitamin B■■ absorption.
Option B: Megaloblastic anemia
• It is a type of megaloblastic anemia that occurs due to vitamin B■■ or folate deficiency.

Page 18

604
• In this anemia, the bone marrow produces stem cells that produce enlarged red blood cells.
• Fatigue
• Pallor
• Shortness of breath.
• Vitamin B■■ deficiency occurs due to the following:
• Pernicious anemia
• Gastrectomy
• Zollinger-Ellison syndrome
• Pancreatic insufficiency
• Folate/vitamin B9 deficiency occurs due to the following:
• Decrease intake of green vegetables, fish, and meat.
• Cohn’s or ulcerative colitis
• Hemolytic anemia
• Increase alcohol intake.
Option C: Aplastic anemia
• It is a type of anemia in which bone marrow does not produce new blood cells.
• It is an autoimmune disease in which T-cell-mediated autoimmune destruction of bone marrow
occurs.
• It mostly occurs due to the following:
• Chemotherapy
• Toxins (benzene)
• Certain drugs (like chloramphenicol, carbamazepine, and phenytoin)
• Autoimmune disorder.
• Parvovirus (short-lived)
• It manifests the following:
• Fatigue
• Pale skin
• Shortness of breath
• Severe bruises
• Increased risk of infection
Option D: Hemolytic anemia
• It is a type of anemia that occurs due to the early breakdown of red blood cells.
• Red blood cells have a 120-day life span; if its breakdown occurs, bone marrow has less time to form
new RBCs.
• There are many types of hemolytic anemia:
• Sickle cell anemia

Page 19

605
• Thalassemia
• G6PD deficiency
• It causes the following signs/symptoms:
• Jaundice
• Tachycardia
• Dyspnea
• Fatigue
• Enlarged spleen or liver

Solution for Question 11:


Option A: Celiac nodes
• According to the Japanese classification for Gastric Carcinoma (JCGC), Level 9 lymph nodes are the
celiac trunk lymph nodes.
• These lymph nodes are located near the origin of the left gastric artery, the common hepatic artery,
and the splenic artery.
• These lymph nodes are deep in a tunnel-shaped retroperitoneal space high in the epigastrium.
• These lymph nodes are associated with the celiac artery and are divided into the following types:
Gastric lymph nodes Hepatic lymph nodes Splenic lymph nodes
• Gastric lymph nodes
• Hepatic lymph nodes
• Splenic lymph nodes
• Gastric lymph nodes
• Hepatic lymph nodes
• Splenic lymph nodes

Option B: Splenic hilum


• According to the Japanese classification for Gastric Carcinoma, Level 10 lymph nodes are present
near the splenic hilum, distal to the end of the pancreatic tail.
Option C: Splenic artery
• According to the Japanese classification for Gastric Carcinoma, the Level 11 lymph nodes are near
the splenic artery.
Option D: Hepatoduodenal
• According to the Japanese classification for Gastric Carcinoma, the Level 12 lymph node is near the
hepatoduodenal ligament.

Page 20

606
Solution for Question 12:
Option A: Duodenoduodenostomy
• Duodenoduodenostomy is an operative repair used for duodenal atresia. It is a surgical connection
between two segments of the duodenum. It is usually done to bypass the obstructed segment in
duodenal atresia.
• It is a surgical connection between two segments of the duodenum.
• It is usually done to bypass the obstructed segment in duodenal atresia.
• It is done for the following anomalies: Duodenal atresia Intestinal atresia Annular pancreas
• Duodenal atresia
• Intestinal atresia
• Annular pancreas
• Management (Duodenal atresia) - Surgical bypass of the duodenal obstruction as a side-to-side or
proximal transverse to distal longitudinal (diamond-shaped) duodenoduodenostomy as given below:
• It is a surgical connection between two segments of the duodenum.
• It is usually done to bypass the obstructed segment in duodenal atresia.
• Duodenal atresia
• Intestinal atresia
• Annular pancreas

Option B: Duodenojejunostomy
• It is a surgical operation that connects the duodenum and jejunum.
• It is mostly used as a safe treatment plan for the superior mesenteric syndrome (duodenal
compression).
• It is preferred in adults over duodenoduodenostomy due to less mobility of the duodenum.

Page 21

607
Option C: Bishop-koop procedure
• Bishop Koop procedure is done for meconium ileus.
• It is an anastomosis in which the end of the proximal loop is an end-stoma, given in the image.

Option D: Gastroduodenostomy
• It is an anastomosis between the stomach remnant and the proximal loop of the duodenum.
• It is done in the following cases: Gastric cancer Gastric ulcer Pyloric malfunctioning
• Gastric cancer
• Gastric ulcer
• Pyloric malfunctioning
• Gastric cancer
• Gastric ulcer
• Pyloric malfunctioning

Page 22

608
Solution for Question 13:
Option C: Most common sites are the fundus and the body of the stomach
• Stress gastritis occurs after physical trauma, shock, sepsis, hemorrhage, or respiratory failure and
may lead to life-threatening gastric bleeding.
• A decrease in the mucosa's tolerance to the harmful radicals produced as a result of stress is the
initial stage in the development of stress-induced gastritis.
• The body's response to stress causes a reduction in the process of gastric renewal, which ultimately
results in the atrophy of the mucosa lining the stomach.
• The amount of blood that flows to the stomach reduces, which increases the risk of acid-pepsin
ulceration and hyperacid secretion in the stomach.
• The hormone angiotensin II is secreted in response to stress, and this action has the effect of
reducing the amount of blood that flows to the mucosa.
• This leads to the creation of reactive oxygen species, which then damage DNA, ultimately forming
8-hydroxydeoxyguanosine (8-OHdG). Because of this, an oxidative mutagenesis byproduct is
produced, and as a consequence, the mucosa is subjected to oxidative stress.
• On the other hand, due to its ability to induce vasodilation, nitric oxide that is produced spontaneously
is thought to be protective against stress gastritis.
• The most common site of curling ulcers is the fundus and the body of the stomach.
Option A: Mucosal resistance is increased in curling ulcers
• Mucosal resistance is decreased in curling ulcers.
Option B: Blood flow to the stomach increases in curling ulcers
• Blood flow to the stomach decreases in curling ulcers
Option D: In curling ulcers, acid secretion in the stomach is increased

Page 23

609
• In curling ulcers, acid secretion in the stomach is normal.

Solution for Question 14:


Option A: Gastroduodenal artery
• Peptic ulcers are open sores in the stomach (gastric ulcer) and the upper part of the duodenum
(duodenal ulcer).
• It usually presents with hematemesis and melena.
• A peptic ulcer is the most common cause of upper GI bleeding, present in one-half to two-thirds of
patients with upper GI bleeding.
• Duodenal ulcer bleeds from the gastroduodenal artery.
• Gastric ulcers mostly bleed from the left gastric artery.
• Vessel Branch of Left gastric artery (largest artery to the stomach) Celiac artery Right gastric artery
Hepatic artery (or gastroduodenal artery) Left gastroepiploic artery Splenic artery Right gastroepiploic
artery Gastroduodenal artery (duodenal ulcers) is the most common cause of upper GI bleeding
Vessel
Branch of
Left gastric artery (largest artery to the stomach)
Celiac artery
Right gastric artery
Hepatic artery (or gastroduodenal artery)
Left gastroepiploic artery
Splenic artery
Right gastroepiploic artery
Gastroduodenal artery (duodenal ulcers) is the most common cause of upper GI bleeding

Page 24

610
Option B: Superior pancreaticoduodenal artery
• It arises from the gastroduodenal artery.
• It divides into anterior and posterior branches.
• It mostly supplies the pancreas and duodenum.
Option C: Left gastric artery
• The left gastric artery is a branch of the celiac artery.
• The lesser curve is supplied by the right and left gastric vessels, and the greater curve is by the right
and left gastroepiploic vessels.
• Gastric ulcers mostly bleed from the left gastric artery.
Option D: Left gastroepiploic artery
• The left gastroepiploic artery arises from the splenic artery and runs within the gastrosplenic ligament.
• The greater curve is supplied by the right and left gastroepiploic vessels.

Solution for Question 15:


Option C: Ramstedt operation is done
• Pyloric stenosis occurs due to the pylorus's thickening, resulting in gastric outlet obstruction.
• Thickening occurs due to the hypertrophy of the circular layer of smooth muscles surrounding the
pylorus of the stomach.
• In Hypertrophic pyloric stenosis, Infants are normal at birth, symptomatic between the ages of 3-6
weeks.
• Firstborn males are commonly affected.
• Infants with HPS typically present with projectile non-bilious vomiting.

Page 25

611
• Treatment of choice: Fredet-Ramstedt pyloromyotomy (The hypertrophied muscle is split, leaving the
mucosa intact).

Other Options
Option A: Commonly seen after 13 weeks
• Infantile hypertrophic pyloric stenosis (IHPS) presents with non-bilious projectile vomiting between 2
and 8 weeks of age and is only rarely seen after 13 weeks. It is easily distinguished from many other
serious causes of vomiting, such as infections because the baby is particularly hungry.
Infantile hypertrophic pyloric stenosis (IHPS) presents with non-bilious projectile vomiting between 2
and 8 weeks of age and is only rarely seen after 13 weeks. It is easily distinguished from many other s
erious causes of vomiting, such as infections because the baby is particularly hungry.
Option B: The second-born male is commonly affected
• In Hypertrophic pyloric stenosis, first-born males are commonly affected.
Option D: Visible peristalsis is always seen
• Visible gastric peristalsis may be seen (not always) as a wave of contraction from the left upper
quadrant to the epigastrium In a relaxed baby, the pyloric tumor is palpable as an olive in the right
upper quadrant.
• In a relaxed baby, the pyloric tumor is palpable as an olive in the right upper quadrant.
• In a relaxed baby, the pyloric tumor is palpable as an olive in the right upper quadrant.

Solution for Question 16:


Option C: Undifferentiated
• According to Lauren Classification, gastric cancer is divided into two types:

Page 26

612
• In diffuse gastric cancer, tumor cells lack adhesion, and cells are present individually or in small
subgroups resulting in non-cohesive, scattered tumor cells.
• Intracellular mucus is present, which pushes the nucleus to one side and forms signet-ring cell
carcinoma, as given in the image.
• These tumors show a more equal male:female ratio and are more frequent in younger individuals.
• The stomach wall becomes grossly thickened and leathery (linitis plastica)
• It usually affects the body of the stomach, has a short duration, and has the worst prognosis.

Option A: Associated with APC Gene mutation


• Diffuse gastric cancer has Decreased E- CADHERIN.
• Associated with APC Gene mutation is characteristic of the intestinal type of gastric cancer.
Option B: More common in males
• Diffuse gastric cancer is more common in Women.
• Intestinal type of gastric cancer is more common in males.
Option D: More common in the distal part
• Diffuse gastric cancer is more common in the proximal part.
• The distal part of the stomach is associated with the intestinal type of gastric cancer.

Solution for Question 17:


Option A: Iron deficiency anemia
• Gastric surgery results in numerous disorders collectively called post-gastrectomy syndromes.
• Metabolic disturbances (Iron deficiency and megaloblastic anemia): The most common metabolic
defect appearing after gastrectomy is anemia. Anemia is related to iron deficiency (more common) or
impairment in vitamin B12 metabolism. More than 30% of patients undergoing gastrectomy have iron

Page 27

613
deficiency anemia, for which the exact mechanism is not known. Patients undergoing subtotal
gastrectomy require lifelong vitamin B12 supplementation. Osteoporosis and osteomalacia occur due to
deficiencies in calcium.
• The most common metabolic defect appearing after gastrectomy is anemia.
• Anemia is related to iron deficiency (more common) or impairment in vitamin B12 metabolism.
• More than 30% of patients undergoing gastrectomy have iron deficiency anemia, for which the exact
mechanism is not known.
• Patients undergoing subtotal gastrectomy require lifelong vitamin B12 supplementation.
• Osteoporosis and osteomalacia occur due to deficiencies in calcium.
• The most common metabolic defect appearing after gastrectomy is anemia.
• Anemia is related to iron deficiency (more common) or impairment in vitamin B12 metabolism.
• More than 30% of patients undergoing gastrectomy have iron deficiency anemia, for which the exact
mechanism is not known.
• Patients undergoing subtotal gastrectomy require lifelong vitamin B12 supplementation.
• Osteoporosis and osteomalacia occur due to deficiencies in calcium.
Option B: Megaloblastic anemia
• Megaloblastic anemia is not the most common postoperative complication.
• Megaloblastic anemia from vitamin B12 deficiency only rarely develops after partial gastrectomy but is
dependent on the amount of stomach removed. Vitamin deficiency occurs secondary to poor
absorption of dietary vitamin B12 because of the lack of intrinsic factors. Patients undergoing subtotal
gastrectomy should be placed on lifelong vitamin B12 supplementation. If a patient develops
macrocytic anemia, serum vitamin B12 levels should be obtained and, if abnormal, treated with
long-term vitamin B12 therapy.
Option C: Hypocalcemia
• Hypocalcemia is not the most common postoperative complication.
Option D: Osteoporosis
• Osteoporosis is not the most common postoperative complication.

Solution for Question 18:


Option C: Adherent clot
• Rebleeding risk is high in all of the above except in the case of adherent clots
• The Forrest Classification for Endoscopic findings and Rebleeding Risks in Peptic Ulcer Disease:
Classification

Rebleeding risk
Grade Ia
Active pulsatile bleeding

Page 28

614
High
Grade Ib
Active non-pulsatile bleeding
Grade IIa
Nonbleeding visible vessel
Grade IIb
Adherent clot
Intermediate
Grade IIC
Ulcer with black spot
Low
Grade III
Clean nonbleeding ulcer
• Management: Endoscopic therapy is recommended in cases of active bleeding as well as for a visible
vessel (Forrest I-IIa) For Forrest IIb, the clot is removed, and the underlying lesion evaluated Ulcers
with a clean base or a black spot, secondary to hematin deposition, are generally not treated
endoscopically
• Endoscopic therapy is recommended in cases of active bleeding as well as for a visible vessel
(Forrest I-IIa)
• For Forrest IIb, the clot is removed, and the underlying lesion evaluated
• Ulcers with a clean base or a black spot, secondary to hematin deposition, are generally not treated
endoscopically
• Endoscopic therapy is recommended in cases of active bleeding as well as for a visible vessel
(Forrest I-IIa)
• For Forrest IIb, the clot is removed, and the underlying lesion evaluated
• Ulcers with a clean base or a black spot, secondary to hematin deposition, are generally not treated
endoscopically
Option A: Visible vessel
• A nonbleeding visible vessel falls in Grade IIa and carries a high risk of rebleeding.
Option B: Visible pulsatile bleeding
• Active pulsatile bleeding falls in Grade Ia and carries a high risk of rebleeding.
Option D: Visible oozing from the vessel
• A Visible oozing carries a high risk of rebleeding.

Solution for Question 19:


Option A: Massive hemorrhage

Page 29

615
• Massive hemorrhage is a rare feature of Mallory-Weiss Syndrome.
• Mallory-Weiss tears: They are mucosal and submucosal tears that occur near the gastroesophageal
junction. The tears classically occur in alcoholic patients after a period of intense retching and vomiting
following binge drinking. The tears can occur in patients with a history of repeated emesis. Forceful
contraction of the abdominal wall occurs against an unrelaxed cardia resulting in mucosal laceration of
the cardia as a result of the increased intragastric pressure. Most tears occur along the lesser
curvature. Clinical features are Minor bleeding in the Majority of the cases Massive bleeding in only
10-20% of cases. It is a Painless condition The Most common vessel responsible for bleeding is the
Left gastric artery The Largest vessel supplying the stomach is the Left gastric artery Diagnosis
Investigation of choice for diagnosis is Endoscopy Management Conservative management (because
of minor bleeding) If not controlled, then: Endoscopy with Electrocoagulation If not available or failed:
Angiography with embolization is done If not available or failed: Open surgical ligation of the bleeding
vessel is done Sengstaken-Blakemore tube It is indicated in variceal bleeding (venous bleeding) It is
contraindicated in Mallory Weiss syndrome (arterial bleeding)
• They are mucosal and submucosal tears that occur near the gastroesophageal junction.
• The tears classically occur in alcoholic patients after a period of intense retching and vomiting
following binge drinking.
• The tears can occur in patients with a history of repeated emesis.
• Forceful contraction of the abdominal wall occurs against an unrelaxed cardia resulting in mucosal
laceration of the cardia as a result of the increased intragastric pressure.
• Most tears occur along the lesser curvature.
• Clinical features are
• Minor bleeding in the Majority of the cases
• Massive bleeding in only 10-20% of cases.
• It is a Painless condition The Most common vessel responsible for bleeding is the Left gastric artery
The Largest vessel supplying the stomach is the Left gastric artery
• The Most common vessel responsible for bleeding is the Left gastric artery
• The Largest vessel supplying the stomach is the Left gastric artery
• Diagnosis
• Investigation of choice for diagnosis is Endoscopy
• Management
• Conservative management (because of minor bleeding)
• If not controlled, then: Endoscopy with Electrocoagulation
• If not available or failed: Angiography with embolization is done
• If not available or failed: Open surgical ligation of the bleeding vessel is done
• Sengstaken-Blakemore tube
• It is indicated in variceal bleeding (venous bleeding)
• It is contraindicated in Mallory Weiss syndrome (arterial bleeding)
• They are mucosal and submucosal tears that occur near the gastroesophageal junction.
• The tears classically occur in alcoholic patients after a period of intense retching and vomiting
following binge drinking.

Page 30

616
• The tears can occur in patients with a history of repeated emesis.
• Forceful contraction of the abdominal wall occurs against an unrelaxed cardia resulting in mucosal
laceration of the cardia as a result of the increased intragastric pressure.
• Most tears occur along the lesser curvature.
• Clinical features are
• Minor bleeding in the Majority of the cases
• Massive bleeding in only 10-20% of cases.
• It is a Painless condition The Most common vessel responsible for bleeding is the Left gastric artery
The Largest vessel supplying the stomach is the Left gastric artery
• The Most common vessel responsible for bleeding is the Left gastric artery
• The Largest vessel supplying the stomach is the Left gastric artery
• Diagnosis
• Investigation of choice for diagnosis is Endoscopy
• Management
• Conservative management (because of minor bleeding)
• If not controlled, then: Endoscopy with Electrocoagulation
• If not available or failed: Angiography with embolization is done
• If not available or failed: Open surgical ligation of the bleeding vessel is done
• Sengstaken-Blakemore tube
• It is indicated in variceal bleeding (venous bleeding)
• It is contraindicated in Mallory Weiss syndrome (arterial bleeding)
Clinical features are
• The Most common vessel responsible for bleeding is the Left gastric artery
• The Largest vessel supplying the stomach is the Left gastric artery
Diagnosis
Management
Sengstaken-Blakemore tube
Option B: Forceful contraction of the abdominal wall
• Forceful contraction of the abdominal wall is a common feature of Mallory Weiss syndrome.
Option C: Most tears occur along the lesser curvature
• In Mallory Weiss syndrome, most tears occur along the lesser curvature.
Option D: Most of the bleeding episodes are self-limited
• In Mallory Weiss syndrome, most of the bleeding episodes are self-limited.

Solution for Question 20:

Page 31

617
Option C: Stomach
Sister Mary Joseph’s nodule
Gastrointestinal malignancies
Gynecological malignancies
Gastric cancer (MC)
Colonic cancer
Pancreatic cancer (mostly body and tail)
Ovarian cancer
Uterine cancer
• A malignant umbilical nodule that has spread to other parts of the body is known by its eponymous
name, Sister Mary Joseph's nodule. Because it is a marker of an advanced stage of cancer, it is a
finding that is not very common but significant.
• The etiology of the nodule found on Sister Mary Joseph is still unknown, and the hypotheses that have
been put forward include direct expansion of the tumor to the umbilicus, lymphatic spread, and
hematogenous spread.
• A tissue sample can be easily obtained with a biopsy of the umbilical nodule, which allows for a quick
histological diagnosis of the condition.
• The severity of the cancer is determined through imaging with a CT scan or an MRI. Because Sister
Mary Joseph's nodule is a symptom of advanced cancer, the outlook for patients with this condition is
typically not favorable.
Option A: Rectum
• Sister Joseph’s nodule does not indicate cancer of the rectum.
Option B: Gallbladder
• Sister Joseph’s nodule does not indicate cancer of the Gallbladder.
Option D: Fallopian tubes
• Sister Joseph’s nodule does not indicate cancer of the Fallopian tubes.

Solution for Question 21:


Option B: Subtotal gastrectomy
• The treatment varies according to the stage and location of cancer. If the tumor is located in the
proximal one-third of the stomach, the recommended treatment is a total gastrectomy with D2
lymphadenectomy. If the tumor is located in the body as well as the antrum, the recommended
treatment is a subtotal gastrectomy in conjunction with D2 lymphadenectomy.
• If the tumor is located in the proximal one-third of the stomach, the recommended treatment is a total
gastrectomy with D2 lymphadenectomy.
• If the tumor is located in the body as well as the antrum, the recommended treatment is a subtotal
gastrectomy in conjunction with D2 lymphadenectomy.

Page 32

618
• The treatment for the intestinal variant of stomach carcinoma, as well as the diffuse type of stomach
carcinoma, is the same.
• With one notable exception, a total gastrectomy and D2 lymphadenectomy will be performed in cases
where the cancer is diffuse and affects the distal one-third of the stomach.
• If the tumor is located in the proximal one-third of the stomach, the recommended treatment is a total
gastrectomy with D2 lymphadenectomy.
• If the tumor is located in the body as well as the antrum, the recommended treatment is a subtotal
gastrectomy in conjunction with D2 lymphadenectomy.

Option A: Total gastrectomy


• Total gastrectomy is not the treatment of choice for this case.
Option C: Palliative
• Palliative surgery is not the treatment of choice for this case.
Option D: Chemotherapy
• Chemotherapy is not the treatment of choice for this case.

Solution for Question 22:


Option A: Trichobezoar
• Concretions of hair, which are commonly observed in long-haired girls or women who deny eating
their own hair and are discovered in a significant number of these individuals.
• Symptoms may include occasional gastric perforation and obstruction of the small bowel, in addition
to the discomfort caused by stomach ulcers and the feeling of fullness induced by obstruction of the
gastric outlet.
• It is more likely that surgical operations will be required for the management of trichobezoars.

Page 33

619
• This disorder is referred to as "Rapunzel Syndrome" in the medical field, and it is characterized by the
presence of stomach trichobezoars along with a lengthy extension of hairs that extends into the
duodenum.
Option B: Carcinoma pyloric antrum
• The alopecia and other clinical findings are not suggestive of carcinoma pyloric antrum.
Option C: Intestinal tuberculosis
• The alopecia and other clinical findings are not suggestive of intestinal tuberculosis.
Option D: Rectus sheath hematoma
• The alopecia and other clinical findings are not suggestive of rectus sheath hematoma.

Solution for Question 23:


Option C: Multiple, shallow ulcers with areas of erythema and focal hemorrhage in the fundus
• Stress ulcers are caused by acute gastric or duodenal erosive lesions that can develop as a result of
shock, sepsis, major surgery, trauma, or burns. Resulting in life-threatening bleeding from the
stomach.
• The condition is characterized by repeated, superficial erosions that do not ulcerate and start in the
proximal stomach, which is the region of the stomach that secretes acid (the fundus) and advances
distally.
• The lesions are often many and superficial, and they are characterized by distinct patches of
erythema, as well as focal hemorrhage or an adhering clot in certain cases.
• In most cases, these involve the fundus and the body.
• Ischemic injury to the mucosa as a result of decreased splanchnic blood flow is the primary pathology,
and increased gastric secretion is not the cause of this condition.
• In the presence of a disease affecting the central nervous system, a Cushing ulcer may develop.
• A curling ulcer develops as a consequence of thermal burn damage that affects more than thirty
percent of the body's surface area.
• Endoscopic findings include multiple shallow ulcers with areas of erythema and focal hemorrhage in
the fundus.
Option A: Single deep ulcer in the fundus
• A single deep ulcer in the fundus cannot be the endoscopic finding in stress-induced ulcers.
Option B: Multiple, shallow ulcers with areas of erythema and focal hemorrhage in the antrum
• Multiple, shallow ulcers with areas of erythema and focal hemorrhage in the fundus are seen on
endoscopy.
Option D: Multiple deep ulcers in the antrum
• Multiple deep ulcers in the antrum cannot be the endoscopic finding in stress-induced ulcers.

Page 34

620
Stomach and Duodenum Part 2
1. A 40-year-old male, a known case of peptic ulcer disease under medical management for the past 3
months, presents to the OPD with complaints of vomiting and abdominal pain. An endoscopy reveals
an ulcer in the duodenum. As the ulcer failed to respond to medical therapy, the patient was taken up
for surgery. Which of the following represents the complications after gastric surgery?
(or)
What are the potential complications following gastric surgery in a 40-year-old male with a
non-responsive to medical therapy for duodenal ulcer?
A. Recurrent ulceration is common after gastric surgery for peptic ulcer
B. Early satiety cannot occur in the absence of gastric resection
C. Early dumping syndrome is due to a sudden high osmotic load to the large bowel
D. Late dumping syndrome is due to reactive hyperglycemia
----------------------------------------
2. Which of the following conditions represent gastric outlet obstruction (GOO)?
(or)
A 39-year-old male presents to the OPD with complaints of vomiting undigested food particles one hour
after a meal. The doctor orders an endoscopic evaluation for this patient. The endoscopy reveals a
chronically scarred duodenal ulcer that is causing gastric outlet obstruction. Which of the following
conditions represents gastric outlet obstruction (GOO)?
A. It is most commonly associated with long-standing peptic ulcer disease and gastric cancer
B. The usual metabolic abnormality is hypochloremic alkalosis
C. Surgery is the mainstay of treatment
D. All of the above
----------------------------------------
3. A 62-year-old male arrives at the emergency department complaining of a mass in the epigastric
region for the past two months. A history of weight loss and loss of appetite is present. Endoscopy and
biopsy revealed a Stage 3 gastric adenocarcinoma in the antrum. A subtotal gastrectomy with D2
dissection is performed. The procedure includes which of the following?
(or)
Subtotal gastrectomy with D2 dissection includeswhich of the following in a 62-year-old male has a
Stage 3 gastric adenocarcinoma in the antrum?
A. 11 lymph nodes removed
B. Grossly negative margins of 2 cm
C. Distal pancreatectomy
D. Splenectomy
----------------------------------------
4. What is the best surgical management for a young male with a 2 × 2 cm duodenal ulcer visualized on
upper GI endoscopy?
(or)

621
A 30 year old male presents to the emergency department with massive hematemesis. A 2 × 2 cm ulcer
is visualized on the upper GI endoscopy on the posterior aspect of the first part of the duodenum. The
bleeding vessel is visualized, but the bleeding is not being controlled endoscopically. A blood
transfusion is done, and the patient is planned for surgery. Which of the following would be the best
surgical management?
A. Antrectomy with ligation of left gastric artery
B. Duodenotomy with ligation of bleeding vessels with postoperative PPI
C. Duodenotomy with ligation of bleeding vessels, truncal vagotomy, and pyloroplasty
D. Duodenotomy with ligation of bleeding vessels, highly selective vagotomy
----------------------------------------
5. A 35-year-old male presents to the OPD complaining of projectile vomiting and ball-rolling
movements. The patient is evaluated, and endoscopy reveals a cicatrizing benign duodenal ulcer
causing gastric outlet obstruction. The patient is prepared preoperatively, and a truncal vagotomy with
gastrojejunostomy is performed. Which of the following statements represents T-vagotomy?
(or)
Which statement represents the surgical procedure of truncal vagotomy in a middle aged male with a
cicatrising benign duodenal ulcer causing gastric outlet obstruction?
A. Highly selective vagotomy is associated with the lowest recurrence rate
B. Truncal vagotomy is associated with minimum morbidity
C. Nerves of Latarjet supplying the pylorus are preserved in highly selective vagotomy
D. Selective vagotomy is performed by dividing the vagus above the celiac and hepatic branches
----------------------------------------
6. A 55-year-old male presents to the OPD with complaints of hematemesis and melena. The patient
has a history of chronic alcohol consumption and smoking. The clinical features are suggestive of an
upper GI bleed. Which of the following is a component of the Blatchford score for risk stratification
systems for upper GI bleeding?
(or)
What is a component of the Blatchford score for risk stratification systems in a middle aged male
presenting with hematemesis and melena, and a history of chronic alcohol consumption and smoking?
A. Age
B. Endoscopic Stigmata of recent bleeding
C. Shock
D. Blood urea nitrogen
----------------------------------------
7. Which of the following statements represents the condition in a middle aged female with a
hemoglobin level of 7.5 gm/dL, endoscopic findings are given below.
(or)
A 50-year-old female presents to medicine OPD with complaints of fatigue, weakness, and chest pain.
Lab reports show a Hb of 7.5 gm/dL. She says that she is anemic. The endoscopic finding is given
below. Which of the following statements represents this condition?

Page 2

622
A. Dilated mucosal blood vessels in the lamina propria
B. Bleeding is the most common presentation
C. Diagnosis is based on typical endoscopic and biopsy appearance
D. All of these
----------------------------------------
8. Which of the following statements is incorrect in a patient with with a history of epigastric pain, weight
loss, anorexia, and peripheral edema, endoscopic findings shown below?
(or)
A 43-year-old male presented to the medicine OPD with epigastric pain, weight loss, anorexia, and
peripheral edema for the last 2 months. The endoscopic findings are given below in the image. Which
of the following statementsis incorrect about this condition?

A. Acquired and premalignant condition


B. Associated with CMV infection in children and H. pylori infection in adults
C. Characterized by foveolar hyperplasia with absent parietal cells
D. Associated with hyperchlorhydria
----------------------------------------
9. A 56-year-old male came to the OPD with complaints of hematemesis and melena. The patient also
complained of a loss of appetite and a feeling of fullness in the stomach. The endoscopic and further
evaluation was done, and the patient was diagnosed with a Gastrointestinal stromal tumor( GIST). All of
the following statements are correct about GIST, except?
(or)

Page 3

623
All of the following statements are correct about GIST except.
A. Most GISTs are positive for CD-117
B. Carney triad components are extra-adrenal paragangliomas, pulmonary chondromas, and multifocal
GIST
C. The investigation of choice for diagnosis is endoscopy with Biopsy
D. Regorafenib is third-line therapy in patients failing imatinib and sunitinib therapy
----------------------------------------
10. A 45-year-old male came to the OPD complaining of acute abdominal pain, abdominal distension,
vomiting, and frequent retching. On further evaluation with USG and CECT, the patient was diagnosed
with gastric volvulus, a surgical emergency. All of the following statements are correct about gastric
volvulus, except?
(or)
Which of the following statements about gastric volvulus is incorrect in a middle aged male presenting
with acute abdominal pain, distension, vomiting, and retching?
A. The most common type is organo-axial
B. Mesenteroaxial volvulus is partial and recurrent
C. Borchardt’s triad components are epigastric pain + Vomiting + Inability to pass a nasogastric tube
D. Secondary gastric volvulus is most commonly associated with diaphragmatic hernia
----------------------------------------
11. A 40-year-old male underwent distal gastrectomy with Billroth II reconstruction for peptic ulcer
disease. Now he has abdominal pain and steatorrhoea. A possible diagnosis of ''blind loop syndrome''
is being considered. Which of the following is characterized by the ‘’blind loop syndrome’’
(or)
What are the characteristic features of blind loop syndrome in a middle aged male post distal
gastrectomy with Billroth II reconstruction, presenting with abdominal pain and steatorrhea?
A. Diarrhea
B. Deficiency of fat-soluble vitamins
C. Diagnosed by culture obtained through an intestinal tube
D. All of the above
----------------------------------------
12. Which statement regarding superior mesenteric artery [SMA] syndrome is incorrect?
(or)
A 16-year-old female came to the OPD complaining of recurrent abdominal pain and vomiting
episodes. A CECT abdomen and CT angiogram revealed that the superior mesenteric artery was
obstructing the duodenum. Superior mesenteric artery syndrome is diagnosed. Which statement
regarding superior mesenteric artery [SMA] syndrome is incorrect?
A. The third part of the duodenum compresses the Superior mesenteric artery at the ligament of the
Treitz attachment
B. Superior mesenteric artery has a normal angle between 38-65 degrees with the duodenum

Page 4

624
C. The Strong's procedure is corrective surgery in which the ligament of Treitz is divided
D. It is characterized by an aortomesenteric angle of fewer than 22 degrees
----------------------------------------
13. A 52-year-old male presents with recurrent epigastric pain, foul-smelling, floating diarrhea, weight
loss, and positive stool tests for occult blood and fat. Gastrointestinal endoscopy reveals multiple ulcers
in the duodenum and jejunum. Which of the following statements is correct regarding Zollinger-Ellison
syndrome?
(or)
Which of the following statementsis correct regarding Zollinger-Ellison syndrome?
A. Associated with MEN1
B. The most common site of ulceration is the stomach
C. Diagnosis is confirmed by pentagastrin test
D. Fasting gastrin levels >200pg/mL is strongly suggestive of gastrinoma
----------------------------------------
14. A 68-year-old woman has been diagnosed with a benign ulcer on the greater curvature of her
stomach, 5 cm proximal to the antrum. After 3 months of standard medical therapy, she continues to
have a guaiac-positive stool, anemia, and abdominal pain with the failure of the ulcer to heal in spite of
medical therapy for weeks. Biopsies of the gastric ulcer have not identified a malignancy. Which of the
following is the next step in management?
(or)
What is the next step in management for a 68-year-old woman with a benign ulcer on the greater
curvature of her stomach, continued guaiac-positive stool, anemia, and abdominal pain despite
negative biopsy results for malignancy?
A. Treatment of the anemia and repeat all studies in 6 weeks
B. Endoscopy and bipolar electrocautery or laser photocoagulation of the gastric ulcer
C. Admission of the patient for total parenteral nutrition (TPN), treatment of anemia, and endoscopic
therapy
D. Surgical intervention, including partial gastric resection
----------------------------------------
15. A 43-year-old male presents to the emergency department complaining of nausea, vomiting, and
abdominal pain. The patient also complains of early satiety. The doctor suspects an ulcer with an
associated malignancy. On endoscopy, he confirms the malignancy. What is the diagnosis?
(or)
What is the likely diagnosis for a 43-year-old male presenting with nausea, vomiting, abdominal pain,
early satiety, and confirmed malignancy on endoscopy, with initial suspicion of an ulcer?
A. Gastric ulcer
B. Duodenal ulcer
C. Esophageal carcinoma
D. GERD
----------------------------------------

Page 5

625
16. An infant is brought to the emergency department with complaints of recurrent episodes of vomiting
and failure to consume adequate food.. The child's barium meal reveals a string sign. Which of the
following contains the string sign?
(or)
What condition is indicated by a "string sign" seen on a barium meal in an infant with recurrent vomiting
and poor food intake?
A. Duodenal atresia
B. Intestinal obstruction
C. Jejunal atresia
D. Congenital hypertrophic pyloric stenosis
----------------------------------------

Correct Answers
Question Correct Answer

Question 1 1
Question 2 4
Question 3 1
Question 4 3
Question 5 3
Question 6 4
Question 7 4
Question 8 4
Question 9 3
Question 10 3
Question 11 4
Question 12 1
Question 13 1
Question 14 4
Question 15 1
Question 16 4

Solution for Question 1:


Option A: Recurrent ulceration is common after gastric surgery for peptic ulcer
• Complications of gastric surgeries: Gastric atony Recurrent ulceration Alkaline reflux gastritis Small
stomach syndrome Post vagotomy diarrhoea Malignant Transformation Metabolic disturbances
Afferent and efferent loop obstruction Early and late dumping
• Gastric atony

Page 6

626
• Recurrent ulceration
• Alkaline reflux gastritis
• Small stomach syndrome
• Post vagotomy diarrhoea
• Malignant Transformation
• Metabolic disturbances
• Afferent and efferent loop obstruction
• Early and late dumping
Complications of gastric surgeries:
• Gastric atony
• Recurrent ulceration
• Alkaline reflux gastritis
• Small stomach syndrome
• Post vagotomy diarrhoea
• Malignant Transformation
• Metabolic disturbances
• Afferent and efferent loop obstruction
• Early and late dumping
Gastric atony
Recurrent ulceration
Alkaline reflux gastritis
Small stomach syndrome
Post vagotomy diarrhoea
Malignant Transformation
Metabolic disturbances
Afferent and efferent loop obstruction
Early and late dumping
Option B: Early satiety cannot occur in the absence of gastric resection
• Early satiety can occur even in the absence of gastric resection.
• Small stomach syndrome (Early satiety): This occurs after highly selective vagotomy. Though there is
no anatomical disturbance, there is a loss of receptive relaxation. Hence, it can occur even in the
absence of gastric resection.
Option C: Early dumping syndrome is due to a sudden high osmotic load to the large bowel
• Early dumping syndrome is due to a sudden high osmotic load to the small bowel.
Option D: Late dumping syndrome is due to reactive hyperglycemia
• Late dumping syndrome is due to reactive hypoglycemia.

Page 7

627
Solution for Question 2:
Option D: All of the above
• Gastric cancer and peptic ulcer disease are the two most common causes of obstruction of the gastric
outlet.
• Peptic ulceration leads to pyloric stenosis as a subsequent condition.
• On examination, one can notice a bloated stomach and a positive succession splash.
• Surgery is still the primary treatment option for most patients.
Option A: It is most commonly associated with long-standing peptic ulcer disease and gastric cancer
• Along with its association with long-standing peptic ulcer disease and gastric cancer, there is also
hypochloremic alkalosis.
Option B: The usual metabolic abnormality is hypochloremic alkalosis
• Along with hypochloremic alkalosis, it is associated with long-standing peptic ulcer disease and
gastric cancer.
• Urine contains low chloride and high bicarbonate.
• Due to dehydration, the body retains salt and excretes potassium and hydrogen.
• Urine becomes acidic
Option C: Surgery is the mainstay of treatment
•Surgery is the main treatment option for gastric cancer and peptic ulcer disease.

Solution for Question 3:


Answer Option A: 11 lymph nodes removed
• Generally, in the radical subtotal gastrectomy procedure, the surgeon strives for a grossly negative
margin of at least 5 cm.
• In the absence of involvement by direct extension, the spleen and pancreatic tail are not removed.
• D1 resection involves the removal of the primary tumor with perigastric nodes (stations 1,2, 3,4,5,6,
and 7)
• The operation provided that tumor-free margins are obtained, more than 15 lymph nodes are
removed, and all gross tumors are resected.
• Reconstruction is usually by Billroth II gastro-jejunostomy or Roux-en-Y reconstruction.
• D2 dissection includes resection of stations 8 to 12a.
Other Options
Option B: Grossly negative margins of 2 cm
• Generally, in this procedure, the surgeon strives for a grossly negative margin of at least 5 cm.
Option C: Distal pancreatectomy

Page 8

628
• In the absence of involvement by direct extension, the spleen and pancreatic tail are not removed.
Option D: Splenectomy
• In the absence of involvement by direct extension, the spleen and pancreatic tail are not removed.

Solution for Question 4:


Option C: Duodenotomy with ligation of bleeding vessels, truncal vagotomy, and pyloroplasty
• The following are the components of the optimal surgical treatment for bleeding duodenal
ulcer patient: Duodenotomy Surgical ligation of the bleeding vessel Pyloroplasty with a truncal
vagotomy.
• Duodenotomy
• Surgical ligation of the bleeding vessel
• Pyloroplasty with a truncal vagotomy.
• Despite its rarity, bleeding is the ulcer-related condition that most frequently results in the death of the
patient.
• Initial treatment for the patient should involve providing appropriate cardiopulmonary resuscitation and
initiating intravenous administration of proton pump inhibitors, to which the vast majority of patients will
react favorably.
• 25% of individuals don't respond; thus, endoscopic hemostatic therapy (such as clipping,
cauterization, or epinephrine injection) is recommended for these high-risk patients.
• the surgery is taken up if endoscopic hemostasis fails.
• Duodenotomy
• Surgical ligation of the bleeding vessel
• Pyloroplasty with a truncal vagotomy.
Option A: Antrectomy with ligation of left gastric artery
• Antrectomy with ligation of the left gastric artery is not a surgical treatment of choice for this patient.
Option B: Duodenotomy with ligation of bleeding vessels with postoperative PPI
• Duodenotomy with ligation of bleeding vessels with postoperative PPI is not a surgical treatment of
choice for this patient.
Option D: Duodenotomy with ligation of bleeding vessels, highly selective vagotomy
• Duodenotomy with ligation of bleeding vessels and highly selective vagotomy is not a surgical
treatment of choice for this patient.

Solution for Question 5:


Option C: Nerves of Latarjet supplying the pylorus are preserved in highly selective vagotomy
• Highly selective vagotomy, also known as parietal cell vagotomy and proximal gastric vagotomy,
involves the following procedures: Seromuscular branches division. Denervation is only performed on

Page 9

629
the parietal cell mass of the stomach. The nerve of the latarjet and the nerve that supplies the pylorus
are both intact. Not necessary to do the drainage procedure Linked to the lowest possible mortality but
the highest possible recurrence Preferred treatment for persistent as well as intractable duodenal
ulcers
• Seromuscular branches division.
• Denervation is only performed on the parietal cell mass of the stomach.
• The nerve of the latarjet and the nerve that supplies the pylorus are both intact.
• Not necessary to do the drainage procedure
• Linked to the lowest possible mortality but the highest possible recurrence
• Preferred treatment for persistent as well as intractable duodenal ulcers
• Seromuscular branches division.
• Denervation is only performed on the parietal cell mass of the stomach.
• The nerve of the latarjet and the nerve that supplies the pylorus are both intact.
• Not necessary to do the drainage procedure
• Linked to the lowest possible mortality but the highest possible recurrence
• Preferred treatment for persistent as well as intractable duodenal ulcers

Option A: Highly selective vagotomy is associated with the lowest recurrence rate
• Highly selective vagotomy is associated with the highest recurrence rate.
Option B: Truncal vagotomy is associated with minimum morbidity
• Truncal vagotomy is associated with maximum morbidity.
Option D: Selective vagotomy is performed by dividing the vagus above the celiac and hepatic branche
s
• Selective vagotomy is performed by dividing the vagus below the celiac and hepatic branches.

Page 10

630
Solution for Question 6:
Option D: Blood urea nitrogen
• There are different scoring systems that have been developed to aid in the process of risk
classification for upper gastrointestinal hemorrhages. Among these are the Glasgow Blatchford Score,
sometimes known as the GBS, and the Rockall and Baylor Score (RBS).
• The Blatchford is a formal risk assessment tool for upper GI hemorrhages. It takes into account the
patient's blood results, blood pressure, known history, and presentation characteristics to determine the
degree to which patients require endoscopic therapy as quickly as possible.
• The RBS was developed to forecast the probability of death based on the observations obtained
before and after an endoscopic procedure.
• Components of Blatchford score: Pulse Blood Urea nitrogen Systolic BP Hemoglobin Presence of
Malena, Syncope, Hepatic or Cardiac dysfunction
• Pulse
• Blood Urea nitrogen
• Systolic BP
• Hemoglobin
• Presence of Malena, Syncope, Hepatic or Cardiac dysfunction
• Pulse
• Blood Urea nitrogen
• Systolic BP
• Hemoglobin
• Presence of Malena, Syncope, Hepatic or Cardiac dysfunction
Option A: Age
• Age is a component of Rockall and Baylor score.
Option B: Endoscopic Stigmata of recent bleeding
• Endoscopic Stigmata of recent bleeding is a component of Rockall and Baylor score.
Option C: Shock
• Shock is a component of Rockall and Baylor’sscore.

Solution for Question 7:


Option D: All of these
the given clinical scenario suggests Watermelon stomach:
• Also known as Gastric Antral Vascular Ectasia (GAVE).
• It appears like there are red strips running parallel across the mucosal folds of the distal stomach.

Page 11

631
• It is distinguished by dilated mucosal blood vessels in the lamina propria that frequently include
thrombosis and converges on the antrum in a longitudinal pattern, giving it the appearance of a
watermelon.
• The antrum is the most typical location for the site (helps to distinguish from Menetrier disease).
• This condition typically strikes older women, who experience persistent blood loss from the digestive
tract, which can develop into iron deficiency anemia and may necessitate blood transfusions.
• Extremely sudden bleeding is uncommon.
• The symptoms of anemia include feelings of exhaustion.
• Endoscopy is the preferred method of investigation in this case.
• Endoscopy followed by electrocoagulation of the vessel using either an argon plasma coagulator or a
YAG laser.
• Patients who do not react to endoscopic treatments should have an antrectomy evaluated as a
treatment option.
Option A: Dilated mucosal blood vessels in the lamina propria
• Dilated mucosal blood vessels in the lamina propria are present.
Option B: Bleeding is the most common presentation
• Bleeding is the most common presentation.
Option C: Diagnosis is based on typical endoscopic and biopsy appearance
• Diagnosis is based on typical endoscopic and biopsy appearance.

Solution for Question 8:


Option D: Associated with hyperchlorhydria
• The above picture shows Menetrier's disease, characterized by finger-like projections due to foveolar
hyperplasia (expansion of surface mucus cells) and the parietal cells' absence.
• It is a rare condition characterized by large mucosal folds in the proximal part of the stomach,
decreased acid secretion, and a protein-losing state.
• There is an increase in the production of TGF-alpha, which causes the increased signaling of the
epidermal growth factor receptor (EGFR), leading to the excessive proliferation of mucosa epithelial
cells.
• TGF-alpha also causes a decrease in the production of acid. The decrease in acid is mainly due to the
decrease or absence of parietal cells and indirectly due to stimulation of somatostatin release.
• Hypersecretion of mucus leads to malabsorption of electrolytes, nutrients, and vitamins in the small
intestine. It leads to a syndrome called protein-losing gastropathy.
• It does not cause excessive acid production in the stomach, called hyperchlorhydria.
Option A: Acquired and premalignant condition
• Menetrier's disease is an acquired and premalignant condition. It has been seen to be associated with
CMV infection in children and H. pylori infection in adults.

Page 12

632
• It can lead to gastric carcinoma. Therefore, partial or total gastrectomy is done in cases not
responding to the medical therapy.
Option B: Associated with CMV infection in children and H. pylori infection in adults
• It is associated with CMV infection in children and H. pylori infections in adults.
Option C: Characterized by foveolar hyperplasia with absent parietal cells
• On histopathological examination, there are foveolar hyperplasia, oxyntic glands atrophy, reduction in
chief and cells, and cystic dilation of pits. Edema and hyperplasia of smooth muscle in the lamina
propria have also been seen.

Solution for Question 9:


Option C: The investigation of choice for diagnosis is endoscopy with Biopsy
• Gastrointestinal stromal tumors make up less than 1% of GI tumors, but they are the most common
mesenchymal neoplasms of the gastrointestinal tract. GISTs are commonly found in the stomach or
small intestine, but they can occur anywhere along the GI tract and rarely involve extra GI.
• Contrast-enhanced abdomen and pelvis CT is the investigation of choice for diagnosing GIST.
• Endoscopy with Biopsy is not an investigation of choice for GIST.
• Upper endoscopy with endoscopic ultrasound (EUS) is the preferred diagnostic procedure to
characterize further upper gastrointestinal (GI) tumors suspicious for GIST involving the stomach, small
intestine, or esophagus.
• On endoscopy, GISTs and leiomyomas may appear as submucosal masses with smooth margins,
normal overlying mucosa, and bulging into the gastric lumen.
• The addition of EUS to endoscopy can distinguish intramural from extramural tumors by identifying
the layer of origin.
Option A: Most GISTs are positive for CD-117

Page 13

633
• GISTs represent a distinct entity from other mesenchymal tumors of the GI tract. GISTs were initially
thought to be derived from smooth muscle based on histologic assessment. However, their
immunohistochemistry profile differs from leiomyomas and leiomyosarcomas from other sites.
• Nearly all the GISTs have an expression of the CD117 antigen, whereas true leiomyosarcomas,
leiomyomas, and other spindle cell tumors of the gastrointestinal tract are typically CD117 negative.
Option B: Carney triad components are extra-adrenal paragangliomas, pulmonary chondromas, and m
ultifocal GIST
• Carney triad is an extremely rare syndrome consisting of GIST, paraganglioma, and pulmonary
chondromas.
• It predominantly affects young women and is generally considered a nonhereditary disorder caused
by hypermethylation of the SDHC promoter. This results in epigenetic inactivation of the SDHC gene
locus with functional impairment of the SDH complex.
Option D: Regorafenib is third-line therapy in patients failing imatinib and sunitinib therapy
• Treatment of GIST includes the Wedge-shaped resection / segmental resection with a 2 cm margin.
• If the surgery is not possible, the following drugs can be used for downgrading or unresectable GIST:
1st line → IMATINIB 2nd line → SUNITINIB 3rd line → REGORAFENIB
• 1st line → IMATINIB
• 2nd line → SUNITINIB
• 3rd line → REGORAFENIB
• 1st line → IMATINIB
• 2nd line → SUNITINIB
• 3rd line → REGORAFENIB

Solution for Question 10:


Option C: Borchardt's triad components are epigastric pain + Vomiting + Inability to pass a
nasogastric tube
• Acute symptoms include pain in the upper abdomen or lower chest associated with severe vomiting
(which may become unproductive). The combination of pain, recurrent retching with the inability to
vomit, and an inability to pass a nasogastric tube, known as Borchardt's triad, is present in as many as
70 percent of patients with acute gastric volvulus.
• So the statement given about gastric volvulus is incorrect.
• Hematemesis can occur due to mucosal ischemia or mucosal tears from vomiting. Also reported is
pain related to pancreatic or omental ischemia, omental avulsion, or splenic rupture due to traction on
these structures.
Option A: The most common type is organo-axial
• Organo-axial rotation refers to the rotation of the stomach along its long axis through a line that
connects the gastroesophageal junction and the pylorus. The antrum rotates anterosuperiorly, and the
fundus rotates posteroinferiorly.
• The greater curvature of the stomach comes to rest superior to the lesser curvature of the stomach in
an inverted position (upside-down stomach).

Page 14

634
• Organo-axial volvulus is the most common type of abnormal rotation, occurring in 60 percent of cases
of gastric volvulus. It is associated with secondary etiologies (e.g., paraesophageal hernias,
diaphragmatic hernia, and diaphragmatic eventration).
• Strangulation of the stomach is more common with this type of volvulus, occurring in up to 30 percent
of cases.

Option B: Mesentero-axial volvulus is partial and recurrent


• With mesentero-axial volvulus, the stomach rotates around its short axis through a perpendicular line
connecting the greater and lesser curvatures of the stomach.
• The antrum becomes displaced above the gastroesophageal junction. With mesentero-axial volvulus,
rotation is usually partial (<180°) and is not generally associated with a secondary anatomic defect.
Option D: Secondary gastric volvulus is most commonly associated with diaphragmatic hernia
• Secondary gastric volvulus occurs in two-thirds of patients with gastric volvulus. It is defined as
volvulus due to other anatomic abnormalities (i.e., not primarily the gastric attachments), such as para
esophagealhernia, diaphragmatic hernia (Morgagni hernia, Bochdalek hernia, traumatic hernia),
diaphragmatic eventration, or phrenic nerve paralysis, as well as anatomic abnormalities of other
organs (e.g., stomach, spleen).

Solution for Question 11:


Option D: All of the above
• All of the above statements about blind loop syndrome are correct.
Option A: Diarrhea
• Blind loop syndrome occurs due to an overgrowth of bacteria in the intestine. It causes food to route
around it. Because of the blind loop, the small intestine becomes shorter than normal. Therefore, the
intestines can not absorb nutrients properly and are passed unabsorbed in feces.
Option B: Deficiency of fat-soluble vitamins

Page 15

635
• Obstructing the normal flow of food through the affected segment of the small intestine leads to
ineffective digestion of fats and fat-soluble vitamins by bile salts.
• Visual impairment may occur due to Vitamin A deficiency and osteoporosis because of Vitamin D
deficiency. Vitamin K deficiency can lead to bleeding disorders.
Option C: Diagnosed by culture obtained through an intestinal tube
• A jejunal aspirate culture can diagnose the overgrowth of bacteria in the small intestine for the
bacterial count.

Solution for Question 12:


Option A: The third part of the duodenum compresses the Superior mesenteric artery at the ligament of
the Treitz attachment
• Superior mesenteric artery syndrome is a rare cause of proximal intestinal obstruction. It is also
known as Cast syndrome, Wilkie syndrome, arterio-mesenteric duodenal obstruction, and chronic
duodenal ileus.
• The third portion of the duodenum lies between the aorta and the superior mesenteric artery. The
duodenum crosses anterior to the aorta at the level of the 3rd lumbar vertebral body and is suspended
by the ligament of Treitz.
• The aorta gives rise to the superior mesenteric artery at the level of the L1 vertebral body. It is
covered by lymphatic and fatty tissue and extends in a caudal direction.

Option B: The superior mesenteric artery has a


normal angle between 38-65 degrees with the duodenum
• In most patients, the angle between the aorta and the superior mesenteric artery ranges from 38 to
65º due to the mesenteric fat pad. This angle correlates with body mass index. The aortomesenteric
distance is usually 10 to 28 mm.
Option C: The strong procedure is corrective surgery in which the ligament of Treitz is divided

Page 16

636
• Strong's procedure involves the mobilization of the duodenum by dividing the ligament of Treitz. After
the mobilization of the duodenal-jejunal junction, the duodenum is placed on the right side of the SMA.
Therefore, it does not lie within the space between the aorta and the superior mesenteric artery.
• The integrity of the bowel is maintained in this procedure; however, the short branches of the inferior
pancreaticoduodenal artery may decrease the duodenum's ability to fall away from the aorta.
Option D: It is characterized by an aortomesenteric angle of fewer than 22 degrees
• In superior mesenteric artery syndrome, the aortomesenteric angle can be reduced to as low as 6º
and the aortomesenteric distances as low as 2 mm. It minimizes the space between the superior
mesenteric artery and the aorta, potentially leading to duodenal compression. The left renal vein can
also be compressed, resulting in nutcracker syndrome.

Solution for Question 13:


Option A: Associated with MEN1
The inherited or familial form of Zollinger-Ellison syndrome (ZES) is associated with MEN I syndrome.
• The inactivating mutations of the MEN1 suppressor gene can cause an endocrine tumor syndrome
called Multiple Endocrine Neoplasia type 1. Although it is usually inherited as an autosomal dominant
disorder, MEN1 can also occur sporadically (without a family history) due to new mutations.
• MEN1 is characterized by parathyroid tumors, pancreatic islet cell tumors, and anterior pituitary
tumors.

Option B: The most common site of ulceration is the stomach


• The stomach is not the most common site for ulceration in ZES.
• Proximal duodenum is the most common form of ulcer, but atypical locations like distal duodenum,
jejunum or the presence of multiple ulcers suggest high suspicion of ZES.
Option C: Diagnosis is confirmed by pentagastrin test
• The pentagastrin test is a test of basal and stimulated acid production by the parietal cells of the
stomach.
• The basal acid output of more than 15 mEq/h or more than 5 mEq/h in patients with a prior vagotomy
and partial gastrectomy suggests ZES.
• But it is not a confirmatory test for ZES.
• The secretin stimulation test confirms the diagnosis of Zollinger - Ellison syndrome (ZES).
Option D: Fasting gastrin levels >200pg/mL is strongly suggestive of gastrinoma
• The fasting serum gastrin test is the best single screening test for ZES.
• It is important to note that patients should not be taking gastric anti-secretory medications at the time
of the test.
• Serial measurements on different days should be performed because fasting gastrin levels can
fluctuate from day to day and can appear to be normal.
• Fasting gastrin levels of more than 1000 pg/ml are strongly suggestive of gastrinoma

Page 17

637
Solution for Question 14:
Option D: Surgical intervention, including partial gastric resection
• Peptic ulcers are referred to as refractory ulcers when they do not heal completely despite undergoing
the standard anti-secretory drug treatment for eight to twelve weeks.
• Patients with peptic ulcers who do not respond to treatment are typically assumed to have chronic H.
pylori infections or strains that are resistant to treatment.
• Nonsteroidal anti-inflammatory medicines (NSAIDs), a large size, malignancy, a refractory response
to drug administration, or other acid hypersecretory states are often the causes of these ulcers;
however, they can also be caused by other conditions.
• Treatment of a non-healing gastric ulcer: Partial gastrectomy with a gastro-duodenal anastomosis:
Billroth I.
Option A: Treatment of the anemia and repeat all studies in 6 weeks
• Treatment of the anemia and repeating all studies in 6 weeks is not done in case of refractory gastric
ulcer.
Option B: Endoscopy and bipolar electrocautery or laser photocoagulation of the gastric ulcer
• Endoscopy and bipolar electrocautery or laser photocoagulation of the gastric ulcer are not done in
case of refractory gastric ulcer.
Option C: Admission of the patient for total parenteral nutrition (TPN), treatment of anemia, and endosc
opic therapy
• Admission of the patient for total parenteral nutrition (TPN), treatment of anemia, and endoscopic
therapy is not made in case of refractory gastric ulcer.

Solution for Question 15:


Option A: Gastric ulcer
• Along with smoking, other risk factors for gastric ulcers and duodenal ulcers include H. pylori and
nonsteroidal anti-inflammatory drugs (NSAIDs).
• Ulcers of the duodenum that are chronic are not connected with a higher risk of cancer. However,
stomach ulcers are associated with a higher risk of cancer.
• Gastric ulcers: Ulcers of the stomach are quite less common. Gastric ulcers are characterized by a
reduction in mucosal resistance. The generation of acid might be enhanced or kept at the regunormalel.
Gastric ulcers are caused by an infection with H. pylori bacteria and by the use of nonsteroidal
anti-inflammatory drugs (NSAIDs). Because eating makes the symptoms of stomach ulcers worse, the
patient often complains of feeling full too quickly. Ulcers of the stomach have the potential to develop
into cancerous tumors . The lesser curvature of the stomach is the location where gastric ulcers are
most frequently found.
• Ulcers of the stomach are quite less common. Gastric ulcers are characterized by a reduction in
mucosal resistance. The generation of acid might be enhanced or kept at the regunormalel.

Page 18

638
• Gastric ulcers are caused by an infection with H. pylori bacteria and by the use of nonsteroidal
anti-inflammatory drugs (NSAIDs). Because eating makes the symptoms of stomach ulcers worse, the
patient often complains of feeling full too quickly. Ulcers of the stomach have the potential to develop
into cancerous tumors . The lesser curvature of the stomach is the location where gastric ulcers are
most frequently found.
• Ulcers of the stomach are quite less common. Gastric ulcers are characterized by a reduction in
mucosal resistance. The generation of acid might be enhanced or kept at the regunormalel.
• Gastric ulcers are caused by an infection with H. pylori bacteria and by the use of nonsteroidal
anti-inflammatory drugs (NSAIDs). Because eating makes the symptoms of stomach ulcers worse, the
patient often complains of feeling full too quickly. Ulcers of the stomach have the potential to develop
into cancerous tumors . The lesser curvature of the stomach is the location where gastric ulcers are
most frequently found.
Option B: Duodenal ulcer
• Duodenal ulcers are non-malignant.
• Ulcers of the duodenum are more prevalent and are associated with increased acid production. The
consumption of food helps to alleviate the discomfort caused by duodenal ulcers. These ulcers do not
carry the potential to develop into cancerous tumors. The first portion of the duodenum is typically
where patients experience problems with duodenal ulcers.
• Clinical features of duodenal ulcer: Epigastric pain 2-5 hours after eating Malena
• Epigastric pain 2-5 hours after eating
• Malena
• Epigastric pain 2-5 hours after eating
• Malena
Option C: Esophageal carcinoma
• The clinical features of the patient are not suggestive of esophageal carcinoma.
• Most common symptom of esophageal carcinoma: Dysphagia > Weight loss.
Option D: GERD
• The clinical features of the patient are not suggestive of GERD.
• Clinical features of GERD (Triad): Retrosternal heartburn Epigastric pain Regurgitation
• Retrosternal heartburn
• Epigastric pain
• Regurgitation
• Retrosternal heartburn
• Epigastric pain
• Regurgitation

Solution for Question 16:


Option D: Congenital hypertrophic pyloric stenosis

Page 19

639
• Infants are normal at birth; symptoms appear between 3-6 weeks after birth.
• First-born male child is commonly affected.
• Infants with HPS typically present with projectile non-bilious vomiting.
• Palpation of the pyloric tumor or olive-shaped mass in the epigastrium or right upper quadrant by a
skilled examiner is pathognomonic in the diagnosis of HPS. If the olive mass is palpated, no additional
diagnostic testing is necessary.
• If the olive mass is palpated, no additional diagnostic testing is necessary.
• Barium Meal: A string Sign indicating a narrowed elongated pyloric canal that does not relax is seen.
The shoulder Sign is caused by the hypertrophied muscle indenting the antrum. The double-track sign
is Caused by the redundant mucosa
• A string Sign indicating a narrowed elongated pyloric canal that does not relax is seen.
• The shoulder Sign is caused by the hypertrophied muscle indenting the antrum.
• The double-track sign is Caused by the redundant mucosa
• If the olive mass is palpated, no additional diagnostic testing is necessary.
• A string Sign indicating a narrowed elongated pyloric canal that does not relax is seen.
• The shoulder Sign is caused by the hypertrophied muscle indenting the antrum.
• The double-track sign is Caused by the redundant mucosa

Option A: Duodenal atresia


• String sign is not seen with duodenal atresia.
• On abdominal X-ray, a double bubble appearance is seen
Option B: Intestinal obstruction
• string sign is not seen with Intestinal obstruction.
Option C: Jejunal atresia
• On abdominal X-ray, a triple bubble appearance is seen in jejunal atresia

Page 20

640
Peritoneum
1. Which of the following best associates with the clinical condition that includes abdominal distension,
diffuse abdominal pain, and growth of E. coli on the ascitic fluid culture medium of a patient with
cirrhosis?
A. Ascitic fluid neutrophil count is < 250/cumm
B. Bowel perforation should be present
C. Multiple organisms are isolated from ascitic fluid
D. 3rd Generation cephalosporin is the antibiotic of choice
----------------------------------------
2. A 30-year-old woman has painless abdominal swelling for a year. Physical examination shows
uniform distension without clear delineation. The mass has limited mobility along the mesentery root
attachment and can be moved freely at a right angle to the mesentery plane. A USG abdomen reveals
a cystic-looking mass measuring 5 x 7 cm in the abdominal cavity. Which of the following statements
does not correlate with this patient's clinical presentation?
A. Chylolymphatic is the most common type
B. Enterogenous is the second most common type
C. Seen most commonly in males
D. Treatment of choice for enterogenous is resection + anastomosis
----------------------------------------
3. A 53-year-old man with fever and worsening abdominal pain in the left lower quadrant for 3 days has
localized peritonitis in the same area, along with early signs of sepsis. His CBC shows a WBC count of
19.7× 10³ per u/L. A plain CT of the abdomen and pelvis reveals a fluid-filled collection (abscess) in the
left lower quadrant associated with complicated diverticulitis and adjacent linear air. What is the
commonest site of this abscess formation?
A. Lesser Sac
B. Greater Sac
C. Pelvis
D. Paracolic Gutter
----------------------------------------
4. Which of the following statements is less likely to be descriptive of retroperitoneal fibrosis?
A. Most patients present with dull, non-colicky, lower back, flank, or abdominal pain.
B. Evidence of impaired renal function with an elevated blood urea nitrogen is common.
C. CT or MRI scan can be used to diagnose the disease.
D. The prognosis for nonmalignant retroperitoneal fibrosis is grim, with the progression of the disease
until death occurring in most patients.
----------------------------------------
5. Mucinous cystic tumor of which of the following organs is most likely to be associated with
pseudomyxoma peritonei?
A. Pancreas

641
B. Kidney
C. Ovary
D. Abdominal Testis
----------------------------------------
6. Which of the following organisms is responsible for developing peritonitis in an adult with ascites?
A. Escherichia coli
B. Klebsiella pneumoniae
C. Streptococcus pneumoniae
D. Bacteroides Fragilis
----------------------------------------
7. Which bacteria is responsible for peritonitis in patients undergoing dialysis?
A. Staphylococcus Aureus
B. Staphylococcus Epidermidis
C. Escherichia coli
D. Bacteroides
----------------------------------------
8. A 45-year-old male presents with a complaint of non-specific dull aching back pain. On further
evaluation, imaging reveals retroperitoneal fibrosis (RPF), and the patient is diagnosed with secondary
RPF. Which of the following is NOT a cause of secondary RPF?
A. Chronic inflammation
B. Methysergide
C. Carcinoma stomach
D. Ormond's disease
----------------------------------------
9. A 32-year-old female presents with a gradually increasing abdominal mass and recurrent episodes of
abdominal pain. On examination, a non-tender, smooth, cystic mass is palpable in the left upper
quadrant. Imaging reveals a chylolymphatic mesenteric cyst with congenital misplaced lymphatics in
the ileum, characterized by the absence of efferent lymphatics and a progressive increase in cyst size
due to chyle accumulation. What is the most appropriate treatment for this condition?
(or)
What is the most appropriate treatment for a 32-year-old female with a non-tender, left upper quadrant
cystic mass, identified as a chylolymphatic mesenteric cyst with congenital misplaced lymphatics in the
ileum, lacking efferent lymphatics, and exhibiting progressive enlargement due to chyle accumulation?
A. Percutaneous drainage
B. Sclerotherapy
C. Enucleation
D. Total cyst excision with bowel resection
----------------------------------------

Page 2

642
Correct Answers
Question Correct Answer

Question 1 4
Question 2 3
Question 3 3
Question 4 4
Question 5 3
Question 6 1
Question 7 2
Question 8 4
Question 9 3

Solution for Question 1:


Option D: 3rd generation cephalosporin is the antibiotic of choice
• The given scenario is a classic case of Spontaneous bacterial peritonitis (SBP).
• Spontaneous bacterial peritonitis characterized by fever and abdominal pain in ascites. The common
culprit microorganisms include E. coli, Klebsiella pneumoniae, and Streptococcus pneumoniae.
• The common culprit microorganisms include E. coli, Klebsiella pneumoniae, and Streptococcus
pneumoniae.
• Antibiotics must be administered just after an ascitic fluid sample has been taken for the culture and
sensitivity. Antibiotics for SBP are indicated only when the patients with ascites have one of the
following. Temperature greater than 37.8°C (100°F) Abdominal pain and tenderness Altered mental
status Ascitic fluid polymorphonuclear leukocyte (PMN) count ≥250 cells/microL Third-generation
cephalosporins are the preferred antibiotics for patients having a CLID-SOFA score < 7 (for acute
decompensation of cirrhosis) For patients with CLIF-SOFA scores equal to or greater than 7, IV
Carbepenems are the preferred antibiotics.
• Antibiotics for SBP are indicated only when the patients with ascites have one of the following.
Temperature greater than 37.8°C (100°F) Abdominal pain and tenderness Altered mental status Ascitic
fluid polymorphonuclear leukocyte (PMN) count ≥250 cells/microL
• Temperature greater than 37.8°C (100°F)
• Abdominal pain and tenderness
• Altered mental status
• Ascitic fluid polymorphonuclear leukocyte (PMN) count ≥250 cells/microL
• Third-generation cephalosporins are the preferred antibiotics for patients having a CLID-SOFA score
< 7 (for acute decompensation of cirrhosis) For patients with CLIF-SOFA scores equal to or greater
than 7, IV Carbepenems are the preferred antibiotics.
• For patients with CLIF-SOFA scores equal to or greater than 7, IV Carbepenems are the preferred
antibiotics.

Page 3

643
• The common culprit microorganisms include E. coli, Klebsiella pneumoniae, and Streptococcus
pneumoniae.
• Antibiotics for SBP are indicated only when the patients with ascites have one of the following.
Temperature greater than 37.8°C (100°F) Abdominal pain and tenderness Altered mental status Ascitic
fluid polymorphonuclear leukocyte (PMN) count ≥250 cells/microL
• Temperature greater than 37.8°C (100°F)
• Abdominal pain and tenderness
• Altered mental status
• Ascitic fluid polymorphonuclear leukocyte (PMN) count ≥250 cells/microL
• Third-generation cephalosporins are the preferred antibiotics for patients having a CLID-SOFA score
< 7 (for acute decompensation of cirrhosis) For patients with CLIF-SOFA scores equal to or greater
than 7, IV Carbepenems are the preferred antibiotics.
• For patients with CLIF-SOFA scores equal to or greater than 7, IV Carbepenems are the preferred
antibiotics.
• Temperature greater than 37.8°C (100°F)
• Abdominal pain and tenderness
• Altered mental status
• Ascitic fluid polymorphonuclear leukocyte (PMN) count ≥250 cells/microL
• For patients with CLIF-SOFA scores equal to or greater than 7, IV Carbepenems are the preferred
antibiotics.

Option A: Ascitic fluid neutrophil count is < 250/mm3


• The diagnostic criterion is an ascitic fluid neutrophil count of > 250/cubic millimeter.
Option B: Bowel perforation should be present
• Bowel perforation is part of the diagnostic criterion of the acute abdomen, and it causes moderate to
severe abdominal pain, not a dull one. It is often associated with hemodynamic instability and can
cause secondary peritonitis.
Option C: Multiple organisms are isolated from ascitic fluid
• In SBP, a single organism is commonly isolated from ascitic fluid cultures (Escherichia coli in
adults, Streptococcus pneumoniae in children) . While it is possible to have polymicrobial infections in
the ascitic fluid, it is less common in SBP.

Solution for Question 2:


Option C: Seen most commonly in males
• The given scenario is a classic case of the mesenteric cyst.
• The characteristic features of a mesenteric cyst are a cystic-looking benign abdominal mass that
moves freely at right angles to the plane of mesentery but shows restricted movement along the
mesentery.

Page 4

644

• Mesenteric cysts are twice as common in females than in men. The mean age for adults is 45 years,
whereas, for children, the mean age is 4.9 years.
• The mean age for adults is 45 years, whereas, for children, the mean age is 4.9 years.
• The most common presentation is painless abdominal swelling with characteristic physical signs.

• The mean age for adults is 45 years, whereas, for children, the mean age is 4.9 years.
Option A: Chylolymphatic is the most common type
• Among many mesenteric cysts, chylolymphatic cyst is the most common type, followed by
enterogenous cysts and urogenital remnant and dermoid cysts.
Option B: Enterogenous is the second most common type
• Enterogenous mesenteric cysts are the second most common type, only second to chylolymphatic
cysts.
Option D: The treatment of choice for enterogenous is resection + anastomosis
• The Enterogenous type of mesenteric cyst is formed by the failure of the mesenteric leaflets to fuse
and the collection of mucinous content between them. It shares the blood supply with the adjacent
intestinal wall. Therefore the treatment of choice for this type of mesenteric cyst is resection of the
affected part of the intestine and anastomosis.

Solution for Question 3:


Option C: Pelvis
• The given scenario is a classic case of intraperitoneal abscess formation owing to the rupture of
diverticulitis, as is evident by the presence of air in the peritoneal cavity in association with the
diverticulitis and fluid-filled collection (abscess). The commonest site of abscess formation is the pelvis
owing to being the dependent part of the body while standing erect. Fluid accumulates in the pelvis due
to the effect of gravity.

Page 5

645
• The commonest site of abscess formation is the pelvis owing to being the dependent part of the body
while standing erect. Fluid accumulates in the pelvis due to the effect of gravity.
• The commonest site of abscess formation is the pelvis owing to being the dependent part of the body
while standing erect. Fluid accumulates in the pelvis due to the effect of gravity.
Option A: Lesser Sac
• It’s very unlikely for an intraperitoneal abscess to accumulate in the lesser sac except for the one
resulting from perforation of the posterior wall of the stomach.
Option B: Greater Sac
• The greater sac is not the dependent part of the body while sitting or standing; therefore, it's less likely
for the purulent fluid to accumulate in the greater sac.
Option D: Paracolic Gutter
• Purulent fluid accumulates in the right and left paracolic gutters only when supine. Even lying supine,
the hepatorenal pouch is relatively deeper than the paracolic gutters, thus raising the probability of fluid
accumulating and abscess formation in that pouch in bed-ridden patients.

Solution for Question 4:


Option D: The prognosis for nonmalignant retroperitoneal fibrosis is grim, with the progression of the di
sease until death occurring in most patients
• Retroperitoneal fibrosis is a benign condition characterized by developing grey/white plaques in the
low lumbar region. However, it spreads laterally and encases the common iliac vessels, ureters, and
aorta. It is not associated with malignant progression or a grim prognosis. It is usually a static or very
slowly progressive disease and has a good prognosis if removed through surgery.
• It is not associated with malignant progression or a grim prognosis.
• It is usually a static or very slowly progressive disease and has a good prognosis if removed through
surgery.
• It is not associated with malignant progression or a grim prognosis.
• It is usually a static or very slowly progressive disease and has a good prognosis if removed through
surgery.
Option A: Most patients present with dull, non-colicky, lower back, flank, or abdominal pain
• Characteristic clinical features of retroperitoneal fibrosis include dull, aching lower back, flank, or
abdominal pain, relieved by NSAIDs and not by opiates.
Option B: Evidence of impaired renal function with an elevated blood urea nitrogen is common
• Encasement and obstruction of ureters may lead to obstructive uropathy and deranged renal function
tests, resulting in elevated blood urea nitrogen.
Option C: CT or MRI scan can be used to diagnose the disease
• Imaging studies, including a simple CT scan or an MRI, can help differentiate it from other conditions
and reach a diagnosis.

Page 6

646
Solution for Question 5:
Option C: Ovary
• Pseudomyxoma peritonei (PMP) is characterized by large quantities of encysted yellow-jelly material
filling the abdomen and peritoneal cavity. It is associated with mucinous cystic tumors of the ovary and
appendix. It may also arise from a primary appendiceal tumor with secondary implantation to one or
both ovaries.
• It is associated with mucinous cystic tumors of the ovary and appendix.
• It may also arise from a primary appendiceal tumor with secondary implantation to one or both
ovaries.
• As mucinous cystic tumor of the appendix or ovary rupture, it leads to seeding of the peritoneum with
mucus-producing cells, which continue to proliferate and produce mucus. The progressive
accumulation of copious amounts of mucinous fluid gradually fills the peritoneal cavity, resulting in the
characteristic "jelly belly".
• The progressive accumulation of copious amounts of mucinous fluid gradually fills the peritoneal
cavity, resulting in the characteristic "jelly belly".
• It is associated with mucinous cystic tumors of the ovary and appendix.
• It may also arise from a primary appendiceal tumor with secondary implantation to one or both
ovaries.
• The progressive accumulation of copious amounts of mucinous fluid gradually fills the peritoneal
cavity, resulting in the characteristic "jelly belly".
Option A: Pancreas
• Mucinous cystic tumor of the pancreas are not associated with developing pseudomyxoma peritonei.
Option B: Kidney
• Mucinous cystadenocarcinoma is a very rare epithelial tumor originating from the urothelium of the
renal pelvis. It does not contribute to the risk factor for developing pseudomyxoma peritonei.
Option D: Abdominal Testis
• Mucinous cystic tumor of the abdominal testis are extremely rare and are not associated with the
pathophysiology of pseudomyxoma peritonei development.

Solution for Question 6:


Option A: Escherichia Coli
• In adults with ascites, Gram-negative bacteria like E. coli are the most common cause of peritonitis.
• The presence of ascites provides a natural culture medium for bacterial growth.
Option C: Streptococcus pneumoniae
• In pediatric nephrotic patients, spontaneous bacterial peritonitis (SBP) commonly involves bacterial
species Streptococcus pneumoniae.
Option B: Klebsiella pneumoniae

Page 7

647
• Klebsiella pneumoniae is the second most common causative pathogen for spontaneous bacterial
peritonitis in adults.
Option D: Bacteroides Fragilis
• It is very unlikely for Bacteroides fragilis to cause spontaneous bacterial peritonitis.

Solution for Question 7:


Option B: Staphylococcus Epidermidis
• Continuous ambulatory peritoneal dialysis (CAPD) involves multiple exchanges (3-4) daily, followed
by an overnight dwell.
• In the patient undergoing CAPD, peritonitis is caused by the introduction of microbes to the sterile
peritoneum (intraluminal contamination, touch contamination and periluminal contamination) in the
setting of compromised host defenses (The continuous presence of a large amount of fluid in the
peritoneal cavity can impair host defenses) This results in contamination of the peritoneum with
pathogenic skin bacteria, the most common of which is staphylococcus epidermidis. Contamination
with skin bacteria occurs when the patient or the caretaker is performing exchanges or
connecting/disconnecting from the transfer set.
• This results in contamination of the peritoneum with pathogenic skin bacteria, the most common of
which is staphylococcus epidermidis.
• Contamination with skin bacteria occurs when the patient or the caretaker is performing exchanges or
connecting/disconnecting from the transfer set.
• These infections can be prevented by Switching the patient to hemodialysis Ensuring an aseptic
environment while performing CAPD Trained team Single-use disconnect or Y-system rather than the
older spike system.
• Switching the patient to hemodialysis
• Ensuring an aseptic environment while performing CAPD
• Trained team
• Single-use disconnect or Y-system rather than the older spike system.
• This results in contamination of the peritoneum with pathogenic skin bacteria, the most common of
which is staphylococcus epidermidis.
• Contamination with skin bacteria occurs when the patient or the caretaker is performing exchanges or
connecting/disconnecting from the transfer set.
• Switching the patient to hemodialysis
• Ensuring an aseptic environment while performing CAPD
• Trained team
• Single-use disconnect or Y-system rather than the older spike system.
Option A: Staphylococcus Aureus
• Staphylococcus is also a skin pathogen and a source of contamination for the causation of peritonitis,
but it is not as prevalent as staphylococcus epidermidis.
Option C: Escherichia coli

Page 8

648
• Escherichia coli is the commonest causative pathogen of spontaneous bacterial peritonitis but not in
peritonitis in the settings of CAPD as it is not a part of the flora.
Option D: Bacteroides
• Bacteroides is one of the causative organisms for peritonitis, but not in the setting of CAPD, as it is not
a skin pathogen but rather a part of gut flora.

Solution for Question 8:


Correct Option D - Ormond's disease:
• Ormond's disease refers to Primary (Idiopathic) Retroperitoneal Fibrosis (RPF), which accounts for
approximately 70% of RPF cases. It is not considered a cause of secondary RPF. Primary RPF
typically involves fibrotic proliferation in the retroperitoneum without an identifiable secondary cause.
Therefore, in the context of secondary RPF, Ormond's disease is the correct answer as it is not
associated with the secondary form of the condition.

Incorrect Options:
Option A - Chronic inflammation: Chronic inflammation is a known cause of secondary RPF. Inflammat
ory conditions can lead to fibrotic changes in the retroperitoneum, contributing to the development of R
PF.
Option B - Methysergide: Methysergide is listed as one of the drugs associated with secondary RPF. T
he mnemonic "MAHE BP" can be used to remember drug-related causes: Methysergide, Amphetamine
s, Hydralazine, Entacapone, Beta-blockers, Bromocriptine, Phenacetin.
Option C - Carcinoma stomach: Carcinoma stomach is a malignancy associated with secondary RPF.
Tumors, including gastric carcinoma, can induce fibrotic changes in the retroperitoneum, contributing to
the development of RPF.

Solution for Question 9:


Correct Option C - Enucleation:
• Enucleation is the most appropriate treatment for chylolymphatic mesenteric cysts. Enucleation
involves carefully removing the cyst without disturbing the surrounding structures. Since these cysts
have independent vascular supply and no shared wall, enucleation allows for complete removal while
minimizing the risk of recurrence. This approach is preferred over other interventions, as it addresses
the underlying problem without unnecessary disruption to adjacent tissues.

Incorrect Options:
Option A - Percutaneous drainage: Percutaneous drainage involves inserting a needle or catheter thro
ugh the skin into the cyst to aspirate its contents. This option is inappropriate for chylolymphatic mesen
teric cysts because the absence of efferent lymphatics means that drainage alone may not effectively a
ddress the underlying problem. Additionally, the risk of recurrence is high with this approach.

Page 9

649
Option B - Sclerotherapy: Sclerotherapy involves injecting a sclerosing agent into the cyst to induce sc
arring and collapse. However, in chylolymphatic mesenteric cysts, where the cyst has an independent
vascular supply, sclerotherapy is not the preferred treatment. The cyst may not respond well to this inte
rvention, and there is an increased risk of recurrence.
Option D - Total cyst excision with bowel resection: Total cyst excision with bowel resection is an unne
cessary and overly aggressive approach for chylolymphatic mesenteric cysts. Since these cysts lack ef
ferent lymphatics and do not share a wall with surrounding structures, a less invasive option like enucle
ation is preferred. Total cyst excision with bowel resection may lead to unnecessary complications and
is not the treatment of choice for this condition.

Page 10

650
Intestinal Obstruction
1. Which of the following is the investigation of choice for intermittent low-grade intestinal obstruction?
A. X-ray
B. USG
C. Enteroclysis
D. Barium small-bowel follow-through
----------------------------------------
2. Which of the following is the most common electrolyte imbalance that causes paralytic ileus?
A. Hyponatremia
B. Hypernatremia
C. Hypokalemia
D. Hyperkalemia
----------------------------------------
3. Which of the following is the first symptom manifesting in acute small intestinal obstruction?
A. Constipation
B. Colicky pain
C. Vomiting
D. Distension
----------------------------------------
4. Which of the following is the most common cause of colonic obstruction?
A. Volvulus
B. Hernia
C. Adhesions
D. Neoplasm
----------------------------------------
5. Which of the following is the most common cause of dynamic intestinal obstruction?
A. Adhesions
B. Fecal impaction
C. Obstructed hernia
D. Carcinoma
----------------------------------------
6. What is the most common type of tumor associated with intussusception in children?
(or)
A 3-year-old child has a diagnosis of intussusception. Which of the following is the most likely tumor
associated with this patient's intussusception?

651
A. Carcinoid
B. Villous adenoma
C. Lymphoma
D. Smooth muscle tumor
----------------------------------------
7. Which of the following is the most common site of the volvulus?
A. Sigmoid colon
B. Cecum
C. Transverse colon
D. Stomach
----------------------------------------
8. A 50-year-old female presents with complaints of abdominal distension, vomiting, and pain. Imaging
reveals a kidney bean sign in the left upper quadrant, suggestive of the cecal volvulus. Which of the
following is least likely to be associated with cecal volvulus?
A. Present with small bowel obstruction
B. Present with air-fluid levels in the right upper quadrant and convexity towards left
C. Endoscopic derotation is not effective like sigmoid volvulus
D. Cecopexy is the procedure of choice
----------------------------------------
9. Bird of prey sign is present in which of the following disorders?
A. Gastric volvulus
B. Intussusception
C. Sigmoid volvulus
D. Cecal volvulus
----------------------------------------
10. A 40-year-old male with a history of colicky pain, multiple episodes of bilious vomiting, no passage
of flatus, and constipation shows a distended abdomen with a tympanic note. The X-ray of the
abdomen is done as shown in the image: Which of the following is the most likely diagnosis?

A. Duodenal obstruction

Page 2

652
B. Jejunal obstruction
C. Intussusception
D. Colonic obstruction
----------------------------------------
11. A preterm baby has abdominal distention. After a water-soluble contrast enema, the baby passed
something, which is given in the image below. What is the likely condition associated with the finding?

A. Hirschsprung'sdisease
B. Hypothyroidism
C. Cystic fibrosis
D. All of the above
----------------------------------------
12. A 70-year-old male is brought to the emergency room with colicky pain in the lower abdomen. He
also complains of absolute constipation. After performing a physical examination, an X-ray is ordered.
The abdominal X-ray findings are given below: Which of the following is the preferred treatment option
for this patient?
(or)
A X-ray shown below suspects a condition. Which of the following is the preferred treatment option for
this patient?

A. Barium enemaC
B. Observation
C. Air enema

Page 3

653
D. Colonoscopic detorsion
----------------------------------------
13. A 1-year-old male baby is brought by his parents with a complaint of abdominal distention. The
parents also complain of blood in the stool. A barium enema X-ray is ordered after the physical
examination, which shows the following findings: Which of the following findings are indicated in the
Abdominal X-ray?

A. Meniscus sign
B. Pseudo kidney sign
C. Coiled spring sign
D. Target sign
----------------------------------------
14. Which of the following is the best treatment for a 9-month-old child diagnosed with intussusception?
A. IV Fluids – antibiotics - NG tube
B. IV Fluids – antibiotics - air enema
C. IV Fluids – antibiotics – barium enema
D. IV Fluids – antibiotics – warm saline enema
----------------------------------------
15. Which of the following is the radiological feature of small bowel obstruction?
A. Air under the diaphragm
B. Apple core appearance of colon
C. Multiple air-fluid levels
D. Fluid in the peritoneum
----------------------------------------
16. Which of the following statements is most likely related to postoperative ileus?
A. The use of intravenous patient-controlled analgesia does not affect the return of small bowel motor
activity
B. The presence of peritonitis at the time of the original operation delays the return of normal bowel
function
C. The routine use of metoclopramide will hasten the return of small intestinal motor activity

Page 4

654
D. Contrast radiographic studies have no role in distinguishing early postoperative bowel obstruction
from normal ileus
----------------------------------------
17. A 65-year-old male presents with abdominal distension and discomfort. Bowel sounds are sluggish
on auscultation. Enema reveals colonic distension. The patient is diagnosed with colonic
pseudo-obstruction. Which of the following is the primary treatment of choice?
(or)
A 65-year-old male with abdominal distension, discomfort, and sluggish bowel sounds is diagnosed
with colonic pseudo-obstruction. What is the primary treatment of choice?
A. Opiates
B. Neostigmine (IV)
C. Flatus tube insertion
D. Retroperitoneal hematoma evaluation
----------------------------------------
18. A 65-year-old male presents with colicky pain in the lower abdomen, abdominal distension, and
absolute constipation. Abdominal X-ray reveals large bowel obstruction due to sigmoid malignancy.
Which of the following statements regarding the management of this patient is correct?
(or)
In managing a 65-year-old male with large bowel obstruction due to sigmoid malignancy, which
procedure is recommended, especially in elderly individuals?
A. Resection of the sigmoid with colorectal anastomosis is the preferred approach
B. Total colectomy with ileorectal anastomosis is the standard treatment for sigmoid malignancy
C. Hartman’s procedure is the recommended choice for managing sigmoid malignancy in elderly
individuals
D. Conservative management with close observation is appropriate for this patient
----------------------------------------
19. A 45-year-old male presents with intermittent colicky pain in the right lower quadrant of the
abdomen. On examination, there is tenderness in the cecal region. The pain resolves spontaneously
between episodes. What is the most likely cause of his symptoms?
(or)
The most likely cause of the patient's intermittent colicky pain in the right lower quadrant with
spontaneous resolution is cecal bascule, where the cecum folds anteriorly over the ascending colon,
causing intermittent isolated cecal obstruction.
A. Appendicitis
B. Cecal volvulus
C. Cecal bascule
D. Crohn's disease
----------------------------------------

Correct Answers

Page 5

655
Question Correct Answer

Question 1 3
Question 2 3
Question 3 2
Question 4 4
Question 5 1
Question 6 3
Question 7 1
Question 8 4
Question 9 3
Question 10 2
Question 11 4
Question 12 4
Question 13 3
Question 14 2
Question 15 3
Question 16 2
Question 17 2
Question 18 3
Question 19 3

Solution for Question 1:


Option C: Enteroclysis
• Enteroclysis is the investigation of choice for diagnosis of low-grade intermittent small bowel
obstruction.
• IOC for diagnosis of Small bowel obstruction is CT(Computed tomography)
• Enteroclysis: Enteroclysis involves the installation of contrast into the intestine using a nasoenteric
tube. For identifying low-grade or intermittent obstruction, CT enteroclysis has a reported sensitivity and
specificity of approximately 88% and 82%, respectively.
• Enteroclysis involves the installation of contrast into the intestine using a nasoenteric tube. For
identifying low-grade or intermittent obstruction, CT enteroclysis has a reported sensitivity and
specificity of approximately 88% and 82%, respectively.
• Enteroclysis involves the installation of contrast into the intestine using a nasoenteric tube. For
identifying low-grade or intermittent obstruction, CT enteroclysis has a reported sensitivity and
specificity of approximately 88% and 82%, respectively.
Option A: X-ray

Page 6

656
• Erect abdominal films are no longer routinely obtained, and the radiological diagnosis is based on a
passive abdominal film
• An erect film may subsequently be requested when further doubt exists.
Option B: USG
• USG is used to diagnose intestinal hepatobiliary pathologies.
• For intermittent intestinal obstruction, enteroclysis is preferred.
Option D: Barium meal follow-through
• Barium small-bowel follow-through is used to diagnose small intestine disorders, such as ulcers,
tumours, and inflammatory bowel disease.

Solution for Question 2:


Option C: Hypokalemia
• Paralytic ileus is associated with hypokalemia.
• Hypokalemia can cause changes in the coordinated reflexes and patterns of GI motility by reducing
neural conduction to and within the enteric nervous system (ENS).
• Paralytic ileus: This is a condition where peristaltic waves fail due to neuromuscular failure in the
myenteric (Auerbach's) and submucous (Meissner's) plexuses. The resultant stasis leads to the
accumulation of fluid and gas within the bowel, with associated distension, vomiting, absence of bowel
sounds, and absolute constipation. Clinical features: Paralytic ileus takes on a clinical significance if,
72 hours after laparotomy: There has been no return of bowel sounds on auscultation; There has been
no passage of flatus. Abdominal distension becomes more marked and tympanitic. Colicky pain is
not a feature. Distension increases pain from the abdominal wound. In the absence of gastric
aspiration, effortless vomiting may occur. Radiologically, the abdomen shows gas-filled loops of the
intestine with multiple fluid levels (if an erect film is felt necessary).
• This is a condition where peristaltic waves fail due to neuromuscular failure in the myenteric
(Auerbach's) and submucous (Meissner's) plexuses.
• The resultant stasis leads to the accumulation of fluid and gas within the bowel, with associated
distension, vomiting, absence of bowel sounds, and absolute constipation.
• Clinical features: Paralytic ileus takes on a clinical significance if, 72 hours after laparotomy: There
has been no return of bowel sounds on auscultation; There has been no passage of flatus. Abdominal
distension becomes more marked and tympanitic. Colicky pain is not a feature. Distension increases
pain from the abdominal wound. In the absence of gastric aspiration, effortless vomiting may occur.
Radiologically, the abdomen shows gas-filled loops of the intestine with multiple fluid levels (if an erect
film is felt necessary).
• Paralytic ileus takes on a clinical significance if, 72 hours after laparotomy: There has been no return
of bowel sounds on auscultation; There has been no passage of flatus.
• Abdominal distension becomes more marked and tympanitic.
• Colicky pain is not a feature.
• Distension increases pain from the abdominal wound.
• In the absence of gastric aspiration, effortless vomiting may occur.

Page 7

657
• Radiologically, the abdomen shows gas-filled loops of the intestine with multiple fluid levels (if an erect
film is felt necessary).
• This is a condition where peristaltic waves fail due to neuromuscular failure in the myenteric
(Auerbach's) and submucous (Meissner's) plexuses.
• The resultant stasis leads to the accumulation of fluid and gas within the bowel, with associated
distension, vomiting, absence of bowel sounds, and absolute constipation.
• Clinical features: Paralytic ileus takes on a clinical significance if, 72 hours after laparotomy: There
has been no return of bowel sounds on auscultation; There has been no passage of flatus. Abdominal
distension becomes more marked and tympanitic. Colicky pain is not a feature. Distension increases
pain from the abdominal wound. In the absence of gastric aspiration, effortless vomiting may occur.
Radiologically, the abdomen shows gas-filled loops of the intestine with multiple fluid levels (if an erect
film is felt necessary).
• Paralytic ileus takes on a clinical significance if, 72 hours after laparotomy: There has been no return
of bowel sounds on auscultation; There has been no passage of flatus.
• Abdominal distension becomes more marked and tympanitic.
• Colicky pain is not a feature.
• Distension increases pain from the abdominal wound.
• In the absence of gastric aspiration, effortless vomiting may occur.
• Radiologically, the abdomen shows gas-filled loops of the intestine with multiple fluid levels (if an erect
film is felt necessary).
• Paralytic ileus takes on a clinical significance if, 72 hours after laparotomy: There has been no return
of bowel sounds on auscultation; There has been no passage of flatus.
• Abdominal distension becomes more marked and tympanitic.
• Colicky pain is not a feature.
• Distension increases pain from the abdominal wound.
• In the absence of gastric aspiration, effortless vomiting may occur.
• Radiologically, the abdomen shows gas-filled loops of the intestine with multiple fluid levels (if an erect
film is felt necessary).
Option A: Hyponatremia
• Hyponatremia is associated with neurologic symptoms.
• It does not cause ileus.
Option B: Hypernatremia
• Hypernatremia usually presents with neurologic symptoms.
• Ileus is not seen in hypernatremia.
Option D: Hyperkalemia
• Hyperkalemia leads to life-threatening arrhythmias.
• Ileus is not seen

Page 8

658
Solution for Question 3:
Option B: Colicky pain
• Colicky pain is often the primary symptom of a low small bowel obstruction.
• The pain is sudden and severe, and it typically centers around the umbilicus (small bowel) or lower
abdomen (large bowel).
• The pain coincides with increased peristaltic activity. If the pain becomes severe, it may indicate the
presence of strangulation.
• In postoperative simple mechanical obstruction, pain may not be a significant feature, and it does not
usually occur in paralytic ileus.
In high small bowel obstruction
In low small bowel obstruction
In large bowel obstruction
Vomiting occurs early, profuse
Vomiting is delayed
Vomiting is late
Rapid dehydration
Pain is predominant
Dehydration is late, Pain is mild
Distention is minimal
Central distention
Distention is early and pronounced
Little evidence of fluid levels on X-ray
Multiple central fluid levels in X-ray

Option A: Constipation
• Absolute constipation is a cardinal feature of complete intestinal obstruction.
• However, this is a relatively late manifestation of the disease.
Option C: Vomiting
• The more distal the obstruction, the longer the interval between the onset of symptoms and the
appearance of nausea and vomiting
• As obstruction progresses, the character of the vomitus alters from digested food to feculent material
• It is not an early symptom.
Option D: Distension
• In the small bowel, the degree of distension is dependent on the site of the obstruction and is greater
the more distal the lesion
• Distension is a late feature in colonic obstruction and may be minimal or absent in the presence of
mesenteric vascular occlusion

Page 9

659
Solution for Question 4:
Correct Option-D: Neoplasm
• Neoplasms, including colorectal cancer, are the most common cause of colonic obstruction.
Colorectal cancer can lead to luminal narrowing and obstruction as the tumor grows, impeding the
passage of feces through the colon.
Incorrect Options:
Option A: Volvulus
• Volvulus involves the twisting of a segment of the colon around its mesentery, leading to obstruction.
While volvulus can cause colonic obstruction, it is not as common as neoplasms.
Option B: Hernia
• Hernias, such as inguinal or femoral hernias, can rarely involve the colon and potentially lead to
obstruction. However, hernias are not as common as neoplasms as a cause of colonic obstruction.
Option C: Adhesions
• Adhesions are fibrous bands that form between tissues and organs following surgery or inflammation.
While adhesions can cause mechanical obstruction of the colon, particularly after abdominal surgery,
they are less common than neoplasms as a cause of colonic obstruction.

Solution for Question 5:


Option A: Adhesions
• Adhesions are fibrous bands that form between tissues and organs, typically after abdominal surgery
or inflammation. These bands can cause segments of the intestine to become tethered or constricted,
leading to dynamic intestinal obstruction. Adhesions are the most common cause of dynamic intestinal
obstruction (accounting for 40% of cases).
• Treatment of adhesive obstruction Initially treat conservatively provided there are no signs of
strangulation; should rarely continue conservative treatment for longer than 72 hours At operation,
divide only the causative adhesion(s) and limit dissection Repair serosal tears; invaginate (or resect)
areas of doubtful viability Laparoscopic adhesiolysis in the hands of advanced laparoscopic
practitioners
• Initially treat conservatively provided there are no signs of strangulation; should rarely continue
conservative treatment for longer than 72 hours
• At operation, divide only the causative adhesion(s) and limit dissection
• Repair serosal tears; invaginate (or resect) areas of doubtful viability
• Laparoscopic adhesiolysis in the hands of advanced laparoscopic practitioners
• Initially treat conservatively provided there are no signs of strangulation; should rarely continue
conservative treatment for longer than 72 hours
• At operation, divide only the causative adhesion(s) and limit dissection
• Repair serosal tears; invaginate (or resect) areas of doubtful viability

Page 10

660
• Laparoscopic adhesiolysis in the hands of advanced laparoscopic practitioners
Other Options
Option B: Fecal impaction
• About 8% of cases of dynamic obstruction are attributed to the fecal impaction
• This is common in old age and must be ruled out when suspected.
Option C: Obstructed hernia
• About 12% of cases of dynamic intestine obstruction are due to obstructed hernia
• These are less common causes than adhesions.
Option D: Carcinoma
• About 15% of cases of dynamic obstruction are caused by carcinoma.
• Moreover, carcinoma is the most common cause of large bowel obstruction accounting for 30% of all
the causes.

Solution for Question 6:


Option C: Lymphoma
Intussusceptions:
• Most intussusceptions in children are seen from two months to two years of age.
• Lymphoma is a common cause in children.
• Intussusception typically causes a strangulating bowel obstruction, which can progress to gangrene
and perforation.
• In the majority, the cause is hyperplasia of Peyer's patches (lymphoid tissue), which may be
secondary to a viral infection.
• In 10 percent of children, intussusception is secondary to a pathological lead point, such as a
Meckel's diverticulum, enteric duplication cyst, or even a small bowel lymphoma.
• Such cases are more likely in children over the age of two years and in those with recurrent
intussusception.
• Clinical signs include dehydration, abdominal distension, and a palpable sausage-shaped mass in the
right upper quadrant.
Option A: Carcinoid
• Carcinoid syndrome rarely occurs in children.
• This can lead to intussusception
Option B: Villous adenoma
• A villous adenoma is common in adults.
• The prevalence is less than leiomyoma.
• However, in children with intussusception, lymphoma is a more common cause than villous adenoma.
Option D: Smooth muscle tumor

Page 11

661
• Smooth muscle tumors are common in old age and unlikely to be seen in children.
• Lymphoma is more commonly associated with intussusception.

Solution for Question 7:


Option A: Sigmoid colon
• Sigmoid colon is the most common site of the volvulus
• Sigmoid volvulus is uncommon in Europe and the United States but more common in Eastern Europe
and Africa.
• Indeed, it is the most common cause of large bowel obstruction in the indigenous Black African
population.
• Rotation nearly always occurs in the anticlockwise direction.
• Other predisposing factors include a high-residue diet and constipation.
• In Western populations, the condition is seen most often in elderly patients with chronic constipation;
comorbidities are common, and chronic psychotropic drug use is associated with this condition.
• Younger patients present earlier, and the prognosis is inversely related to the duration of symptoms.
• The presentation can be classified as follows: Fulminant: sudden onset, severe pain, early vomiting,
rapidly deteriorating clinical course Indolent: insidious onset, slowly progressive course, less pain, late
vomiting.
• Fulminant: sudden onset, severe pain, early vomiting, rapidly deteriorating clinical course
• Indolent: insidious onset, slowly progressive course, less pain, late vomiting.
• Fulminant: sudden onset, severe pain, early vomiting, rapidly deteriorating clinical course
• Indolent: insidious onset, slowly progressive course, less pain, late vomiting.
Option B: Cecum
• The cecum is a less common site of volvulus..
• Moreover, the coffee bean sign is seen in the sigmoid volvulus.
Option C: Transverse colon
• The volvulus originating from the transverse colon is the least likely site.
• Most commonly, volvulus originates from the sigmoid colon.
Option D: Stomach
• Gastric volvulus is a rare cause of intestinal obstruction.
• However, the coffee bean sign is not seen.

Solution for Question 8:


Option D: Cecopexy is the procedure of choice

Page 12

662
• Cecopexy is not the treatment for cecal volvulus because of a high recurrence rate with this
procedure.
• Volvulus is mainly clockwise
• The rotation occurs around the ileocolic blood vessels
• It consists of an axial rotation of the terminal ileum, cecum, and ascending colon with concomitant
twisting of the associated mesentery
• Investigation of choice: Plain x-ray · KIDNEY BEAN sign: Characteristic kidney-shaped air-filled
structure in the left upper quadrant ( opposite the site of obstruction) Comma-shaped cecum with the
concavity facing inferiorly and to the right
• KIDNEY BEAN sign: Characteristic kidney-shaped air-filled structure in the left upper quadrant (
opposite the site of obstruction)
• Comma-shaped cecum with the concavity facing inferiorly and to the right

• Treatment includes Most cases require the operation to correct the volvulus and prevent ischemia. If
ischemia has already occurred, an immediate operation is required. Right colectomy with primary
anastomosis is the procedure of choice In frankly gangrenous bowel, resection of the gangrenous
bowel with ileostomy is a safer approach Unlike sigmoid volvulus, cecal volvulus can seldom be
detorsed endoscopically.
• Most cases require the operation to correct the volvulus and prevent ischemia.
• If ischemia has already occurred, an immediate operation is required.
• Right colectomy with primary anastomosis is the procedure of choice
• In frankly gangrenous bowel, resection of the gangrenous bowel with ileostomy is a safer approach
• Unlike sigmoid volvulus, cecal volvulus can seldom be detorsed endoscopically.
• KIDNEY BEAN sign: Characteristic kidney-shaped air-filled structure in the left upper quadrant (
opposite the site of obstruction)
• Comma-shaped cecum with the concavity facing inferiorly and to the right

Page 13

663
• Most cases require the operation to correct the volvulus and prevent ischemia.
• If ischemia has already occurred, an immediate operation is required.
• Right colectomy with primary anastomosis is the procedure of choice
• In frankly gangrenous bowel, resection of the gangrenous bowel with ileostomy is a safer approach
• Unlike sigmoid volvulus, cecal volvulus can seldom be detorsed endoscopically.
Option A: Present with small bowel obstruction
• The twisting of the cecum usually involves the ileum also.
• This leads to symptoms of intestinal obstruction
Option B: Present with air-fluid levels in the right upper quadrant and convexity towards left
• The cecal volvulus presents with multiple air-fluid levels, mainly in the right upper quadrant.
• Moreover, the convexity of these air-fluid levels is towards the left.
Option C: Endoscopic derotation is not effective like sigmoid volvulus
• Cecal volvulus should not be derotated using an endoscope due to the high risk of perforation.
• However, sigmoid volvulus can be endoscopically derotated.

Solution for Question 9:


Option C: Sigmoid volvulus
• Radiological (X-ray) Characteristics of the sigmoid colon Markedly dilated sigmoid colon with a bent
inner tube or coffee bean appearance. The dilated loop's inferior convergence points towards the
pelvis's left side. Whorl sign CT scan. Contrast enema demonstrates the point of obstruction with the
pathognomic 'bird beak' or 'bird of prey' or 'ace of spades' sign.
• Markedly dilated sigmoid colon with a bent inner tube or coffee bean appearance.
• The dilated loop's inferior convergence points towards the pelvis's left side. Whorl sign CT scan.
• Contrast enema demonstrates the point of obstruction with the pathognomic 'bird beak' or 'bird of prey'
or 'ace of spades' sign.
• Markedly dilated sigmoid colon with a bent inner tube or coffee bean appearance.
• The dilated loop's inferior convergence points towards the pelvis's left side. Whorl sign CT scan.
• Contrast enema demonstrates the point of obstruction with the pathognomic 'bird beak' or 'bird of prey'
or 'ace of spades' sign.

Page 14

664
• Volvulus: Volvulus is defined as the twisting of a loop of the bowel on its mesentery and is one of the
most common causes of intestinal obstruction. The sigmoid colon and, less frequently, the cecum are
the common sites of volvulus in adults. Patients typically show features of bowel obstruction (abdominal
pain, distension, bilious vomiting) or bowel ischemia and gangrene (tachycardia, hypotension,
hematochezia, peritonitis) in severe cases. The whirl sign and a grossly dilated loop of the bowel on an
abdominal CT scan establish the diagnosis of volvulus in adults. Sigmoid volvulus without peritonitis is
initially managed with endoscopic detorsion, followed by a semi-elective surgery (sigmoid colectomy).
Sigmoid volvulus with peritonitis, and all cases of cecal volvulus, require emergency surgery.
Endoscopic detorsion should not be attempted in a patient with cecal volvulus because of the high risk
of perforation.
• Volvulus is defined as the twisting of a loop of the bowel on its mesentery and is one of the most
common causes of intestinal obstruction.
• The sigmoid colon and, less frequently, the cecum are the common sites of volvulus in adults.
• Patients typically show features of bowel obstruction (abdominal pain, distension, bilious vomiting) or
bowel ischemia and gangrene (tachycardia, hypotension, hematochezia, peritonitis) in severe cases.
• The whirl sign and a grossly dilated loop of the bowel on an abdominal CT scan establish the
diagnosis of volvulus in adults.
• Sigmoid volvulus without peritonitis is initially managed with endoscopic detorsion, followed by a
semi-elective surgery (sigmoid colectomy).
• Sigmoid volvulus with peritonitis, and all cases of cecal volvulus, require emergency surgery.
• Endoscopic detorsion should not be attempted in a patient with cecal volvulus because of the high risk
of perforation.
• Volvulus is defined as the twisting of a loop of the bowel on its mesentery and is one of the most
common causes of intestinal obstruction.
• The sigmoid colon and, less frequently, the cecum are the common sites of volvulus in adults.
• Patients typically show features of bowel obstruction (abdominal pain, distension, bilious vomiting) or
bowel ischemia and gangrene (tachycardia, hypotension, hematochezia, peritonitis) in severe cases.
• The whirl sign and a grossly dilated loop of the bowel on an abdominal CT scan establish the
diagnosis of volvulus in adults.

Page 15

665
• Sigmoid volvulus without peritonitis is initially managed with endoscopic detorsion, followed by a
semi-elective surgery (sigmoid colectomy).
• Sigmoid volvulus with peritonitis, and all cases of cecal volvulus, require emergency surgery.
• Endoscopic detorsion should not be attempted in a patient with cecal volvulus because of the high risk
of perforation.

Option A: Gastric volvulus


• Plain radiograph of gastric volvulus: Chest radiograph intrathoracic: upside-down stomach mediastinal
or retrocardiac air-fluid level Abdominal radiograph: when performed with the patient upright
unexpected location of the gastric bubble double air-fluid level large, distended stomach collapsed
small bowel
• Chest radiograph intrathoracic: upside-down stomach mediastinal or retrocardiac air-fluid level
• Abdominal radiograph: when performed with the patient upright unexpected location of the gastric
bubble double air-fluid level large, distended stomach collapsed small bowel
• Chest radiograph intrathoracic: upside-down stomach mediastinal or retrocardiac air-fluid level
• Abdominal radiograph: when performed with the patient upright unexpected location of the gastric
bubble double air-fluid level large, distended stomach collapsed small bowel
Option B: Intussusception
• Abdominal x-rays in the cause of intussusception may demonstrate an elongated soft tissue mass
(typically in the upper right quadrant in children) with bowel obstruction (and therefore air-fluid levels
and bowel dilation) proximal to it.
• There may be an absence of gas in the distal collapsed bowel.
• On barium enema: Claw sign, Coiled spring sign
• On X-ray: Target sign, Meniscus sign
Option D: Cecal volvulus
• KIDNEY BEAN sign is a characteristic kidney-shaped air-filled structure in the left upper quadrant (
opposite the site of obstruction) seen in patients with cecal volvulus.

Solution for Question 10:


Option B: Jejunal obstruction
• No passage of flatus suggests small bowel obstruction.
• X-ray of the abdomen shows 1. Multiple dilated bowel loops 2. Circumferential ring-like pattern
indicating → JEJUNAL OBSTRUCTION
• VALVULAE CONNIVENTES Small Bowel Obstruction: Valvulae conniventes are straight segments
that cross the bowel width and appear like a ladder. They are mostly visible in the jejunum. In SBO,
colon gas may be minimal or absent due to the obstruction. Large bowel Obstruction: Haustral folds
are irregular folds in the colon that lack opposite indentations and do not cross the entire diameter of
the bowel. They are commonly seen in the colon and cecum. In LBO, a distended cecum may appear
as a round gas shadow in the right iliac fossa.

Page 16

666
• Small Bowel Obstruction: Valvulae conniventes are straight segments that cross the bowel width and
appear like a ladder. They are mostly visible in the jejunum. In SBO, colon gas may be minimal or
absent due to the obstruction.
• Valvulae conniventes are straight segments that cross the bowel width and appear like a ladder. They
are mostly visible in the jejunum. In SBO, colon gas may be minimal or absent due to the obstruction.
• Large bowel Obstruction: Haustral folds are irregular folds in the colon that lack opposite indentations
and do not cross the entire diameter of the bowel. They are commonly seen in the colon and cecum. In
LBO, a distended cecum may appear as a round gas shadow in the right iliac fossa.
• Haustral folds are irregular folds in the colon that lack opposite indentations and do not cross the
entire diameter of the bowel. They are commonly seen in the colon and cecum. In LBO, a distended
cecum may appear as a round gas shadow in the right iliac fossa.
• Small Bowel Obstruction: Valvulae conniventes are straight segments that cross the bowel width and
appear like a ladder. They are mostly visible in the jejunum. In SBO, colon gas may be minimal or
absent due to the obstruction.
• Valvulae conniventes are straight segments that cross the bowel width and appear like a ladder. They
are mostly visible in the jejunum. In SBO, colon gas may be minimal or absent due to the obstruction.
• Large bowel Obstruction: Haustral folds are irregular folds in the colon that lack opposite indentations
and do not cross the entire diameter of the bowel. They are commonly seen in the colon and cecum. In
LBO, a distended cecum may appear as a round gas shadow in the right iliac fossa.
• Haustral folds are irregular folds in the colon that lack opposite indentations and do not cross the
entire diameter of the bowel. They are commonly seen in the colon and cecum. In LBO, a distended
cecum may appear as a round gas shadow in the right iliac fossa.
• Valvulae conniventes are straight segments that cross the bowel width and appear like a ladder. They
are mostly visible in the jejunum. In SBO, colon gas may be minimal or absent due to the obstruction.
• Haustral folds are irregular folds in the colon that lack opposite indentations and do not cross the
entire diameter of the bowel. They are commonly seen in the colon and cecum. In LBO, a distended
cecum may appear as a round gas shadow in the right iliac fossa.
Option A: Duodenal obstruction
• Abdominal radiographs may classically show a double bubble sign with a gas-filled distended
stomach and duodenum without distal gas.
• A similar appearance (either filled with fluid or gas) can be seen in other modalities. A double-bubble
sign on an abdominal x-ray is a reliable indicator of duodenal atresia with an even higher positive
predictive value in patients with Down syndrome. The presence of distal bowel gas, although more
classically associated with duodenal stenosis, can be seen in duodenal atresia via anomalous bile duct
anatomy
• A double-bubble sign on an abdominal x-ray is a reliable indicator of duodenal atresia with an even
higher positive predictive value in patients with Down syndrome.
• The presence of distal bowel gas, although more classically associated with duodenal stenosis, can
be seen in duodenal atresia via anomalous bile duct anatomy
• A double-bubble sign on an abdominal x-ray is a reliable indicator of duodenal atresia with an even
higher positive predictive value in patients with Down syndrome.
• The presence of distal bowel gas, although more classically associated with duodenal stenosis, can
be seen in duodenal atresia via anomalous bile duct anatomy
Option C: Intussusception

Page 17

667
• Common in children.
• A target sign is usually seen.
Option D: Colonic obstruction
• Haustra are seen in abdominal X-rays and barium enemas.
• The distribution of loops of the intestine is peripheral

Solution for Question 11:


Option D: All of the above
• The image of the "Meconium plug" is shown in the question.
• Plug of meconium in the distal colon
• Signs and Symptoms of distal intestinal obstruction / large bowel obstruction are associated with:
Hirschsprung’s disease Hypothyroidism Cystic fibrosis Maternal diabetes mellitus
• Hirschsprung’s disease
• Hypothyroidism
• Cystic fibrosis
• Maternal diabetes mellitus
• Hirschsprung’s disease
• Hypothyroidism
• Cystic fibrosis
• Maternal diabetes mellitus
Meconium plug syndrome:
• Meconium plug syndrome refers to obstruction of the descending colon and rectosigmoid by a plug of
inspissated meconium
• Meconium plug syndrome is a condition that predominantly affects preterm neonates of low birth
weight.
• Infants present with progressive abdominal distension, vomiting, and failure to pass meconium in the
first 24 hours after birth
• The most commonly accepted hypothesis is that meconium plug syndrome is a transient functional
colonic immaturity, specifically of the myenteric nervous system.
• Consequently, peristalsis is ineffectual, and the meconium remains in the colon for a longer period,
leading to higher water absorption and the development of meconium plugs that cause obstruction.
• The Contrast Enema is the investigation of choice and is often diagnostic and therapeutic. It
demonstrates the classical visualization of the meconium against the colonic wall, creating a
double-contrast impression.
• Water-soluble contrast is the agent of choice, as barium-based contrast agents risk causing peritonitis
if a perforation occurs during the study.

Page 18

668
• Treatment is predominantly non-surgical.
• The primary motive is stimulating the rectum or colon using a water-soluble contrast enema. Some
patients might require serial enemas, possibly using acetylcysteine.
• Most cases respond immediately to the enema, with the instant passage of stools and cessation of
abdominal distension.
Option A: Hirschsprung's disease
• Hirschsprung disease is associated with Down syndrome
• It causes the delayed passage of meconium in newborns.
• It is one of the causes of meconium plug syndrome
Option B: Hypothyroidism
• There is sluggish movement of the intestine due to hypothyroidism
• This leads to the plugging of meconium in the intestine, leading to meconium plug syndrome.
Option C: Cystic fibrosis
• Cystic fibrosis makes the secretion thick, making the passage through the intestine difficult
• It is one of the causes of meconium plug syndrome.

Solution for Question 12:


ption D: Colonoscopic detorsion
• Given Abdominal X-ray: Coffee bean sign / Omega sign / Bent inner tube suggestive of Sigmoid
volvulus.
• Gastrograffin enema shows a narrowing at the site of volvulus and a pathognomonic bird's beak
• Treatment of sigmoid volvulus:
• Colonoscopic detorsion
• After 3 days of sigmoid colectomy (as the risk of recurrence is high)
Option A: barium enema
Option B: Elective sigmoid colectomy
• Elective sigmoid colectomy is done only after the colonoscopy detorsion
Option C: Air enema
• Air enema is helpful in intussusception cases but not in the case of volvulus

Solution for Question 13:


Option C: Coiled spring sign
• The above-shown picture depicts a coiled spring sign, and the most likely cause is intussusception.

Page 19

669
• Children with classic findings of intussusception need to be investigated with an enema, which is both
diagnostic (the gold standard in the diagnosis of intussusception) and therapeutic.
• On Barium enema: Claw sign Coiled spring sign
• Claw sign
• Coiled spring sign
• Claw sign
• Coiled spring sign
Option A: Meniscus sign
• A highly specific X-ray finding for intussusception is called the meniscus sign (or crescent sign).
• This is crescent shaped-gas in the bowel lumen (intussuscipiens) that surrounds the apex of the
intussusceptum (invaginated segment), creating a well-demarcated soft tissue/gas interface

Option B: Pseudo kidney sign


• The pseudo kidney sign is a characteristic ultrasound appearance in some cases of intestinal
intussusception.
• It refers to the longitudinal ultrasound appearance of the intussuscepted bowel segment, mimicking a
kidney.

Page 20

670
Option D: Target sign
• A target sign is classically defined as a CT image associated with the thickening of the intestinal wall.
A lower attenuated middle layer is surrounded on each side by layers of higher attenuation.
• The diagnosis of a small bowel target sign most commonly correlates with intussusception.

Solution for Question 14:


Option B: IV Fluids - antibiotics - air enema
• IV fluids, antibiotics, and air enema are treatment options for intussusception.
• Management of intussusception: Radiological reduction is indicated in most pediatric cases
Resuscitate → start IV Fluids → antibiotics → nonoperative reduction using an air enema for
spontaneous resolution (air is preferred over barium) Surgery is required when radiological reduction

Page 21

671
has failed or is contraindicated
• Radiological reduction is indicated in most pediatric cases
• Resuscitate → start IV Fluids → antibiotics → nonoperative reduction using an air enema for
spontaneous resolution (air is preferred over barium)
• Surgery is required when radiological reduction has failed or is contraindicated
• Radiological reduction is indicated in most pediatric cases
• Resuscitate → start IV Fluids → antibiotics → nonoperative reduction using an air enema for
spontaneous resolution (air is preferred over barium)
• Surgery is required when radiological reduction has failed or is contraindicated
Option A: IV Fluids - antibiotics - NG tube
• NG decompression is not required in the case of intussusception.
• Enema such as air enema is the preferred treatment option.
Option C: IV Fluids - antibiotics - barium enema
• Air enema is considered more effective than barium enema in achieving a reduction of
intussusception.
• Barium enema has been associated with potential complications such as barium peritonitis if
perforation occurs during the procedure.
Option D: IV Fluids - antibiotics - warm saline enema
• A warm saline enema is not as effective as air enema in achieving reduction of the intussusception.

Solution for Question 15:


Option C: Multiple air-fluid levels
• In intestinal obstruction, fluid levels appear later than gas shadows as it takes time for gas and fluid to
separate.

• These are most prominent on an erect film. In adults, two in constant fluid levels – one at the duodenal
cap and the other at the terminal ileum – may be considered normal. In infants (less than one-year-old),
a few fluid levels in the small bowel may be physiological.
• In this age group, it is difficult to distinguish large from small bowel in the presence of obstruction
because the characteristic features seen in adults are not present or are unreliable.
• During the obstructive process, fluid levels become more conspicuous and more numerous when
paralysis occurs.
• When fluid levels are pronounced, the obstruction is advanced.
• In the small bowel, the number of fluid levels is directly proportional to the degree of obstruction and
its site, the number increasing the more distal the lesion
• Ultrasound has 90% sensitivity and 96% specificity in diagnosing small bowel obstruction, and looking
for large bowel obstruction is as good as CT.
• However, a CT scan is better at finding the cause of the obstruction

Page 22

672
Option A: Air under the diaphragm
• Air under the diaphragm is a feature of intestinal perforation
Option B: Apple core appearance of colon
• Apple core appearance is seen in colon cancer.
Option D: Fluid in the peritoneum
• Fluid in the peritoneum is a finding in ascites

Solution for Question 16:


Option B: The presence of peritonitis at the time of the original operation delays the return of normal bo
wel function
• When there is peritonitis along with ileus, the recovery is delayed.
• An ileus is defined as intestinal distension and the slowing or absence of passage of luminal contents
without a demonstrable mechanical obstruction
• Causes of ileus After laparotomy Metabolic and electrolyte derangements (Hypokalemia,
Hyponatremia, Hypomagnesemia, Uremia, Diabetic coma) Drugs (Opiates, Psychotropic drugs,
Anticholinergics, Calcium channel blockers, Tricyclic antidepressants) Infection (Intra abdominal
inflammation, Systemic sepsis, Peritonitis, Pneumonia) Retroperitoneal inflammation or hemorrhage
Mesenteric ischemia Hypothyroidism Ureteral colic Spinal cord injury Myocardial infarction
• After laparotomy
• Metabolic and electrolyte derangements (Hypokalemia, Hyponatremia, Hypomagnesemia, Uremia,
Diabetic coma)
• Drugs (Opiates, Psychotropic drugs, Anticholinergics, Calcium channel blockers, Tricyclic
antidepressants)

Page 23

673
• Infection (Intra abdominal inflammation, Systemic sepsis, Peritonitis, Pneumonia)
• Retroperitoneal inflammation or hemorrhage
• Mesenteric ischemia
• Hypothyroidism
• Ureteral colic
• Spinal cord injury
• Myocardial infarction
• The proposed mechanisms responsible for inhibition of intestinal motility in postoperative ileus are
Surgical stress-induced sympathetic reflexes, Inflammatory response mediator release, and
Anesthetic/analgesic side effects.
• The use of intravenous patient-controlled analgesia may delay the recovery of postoperative ileus
compared to the IM route of narcotic administration.
• Normal motility returns generally follow a characteristic temporal sequence, with small intestinal
motility returning to normal within the first 24 hours after laparotomy and gastric and colonic motility
returning to normal by 48 hours and 3 to 5 days, respectively. Because small bowel motility returns
before colonic and gastric motility, listening for bowel sounds is not a reliable indicator that the ileus has
fully resolved
• The typical and most notable clinical feature is abdominal distension without the usual colicky
abdominal pain. Nausea and vomiting may or may not occur. Patients with ileus continue to pass flatus
and diarrhea, which may help distinguish these patients from those with a mechanical small bowel
obstruction
• Plain radiographs may reveal distended small bowel and large bowel loops. In the postoperative
setting, CT scanning is the test of choice because it can demonstrate the presence of an
intraabdominal abscess or other evidence of peritoneal sepsis that may be causing ileus and can
exclude the presence of complete mechanical obstruction
• The treatment of ileus is entirely supportive, with nasogastric decompression and IV fluids. Prokinetic
agents have been used with little success to shorten recovery times after lower abdominal procedures.
• Measures to reduce postoperative ileus Intraoperative measures Minimize handling of the bowel
Laparoscopic approach, if possible Avoid excessive intraoperative fluid administration Postoperative
measures Correct electrolyte abnormalities Early enteral feeding Epidural anesthesia, if indicated Avoid
excessive intravenous fluid administration Consider mu-opioid antagonists
• Intraoperative measures Minimize handling of the bowel Laparoscopic approach, if possible Avoid
excessive intraoperative fluid administration
• Minimize handling of the bowel
• Laparoscopic approach, if possible
• Avoid excessive intraoperative fluid administration
• Postoperative measures Correct electrolyte abnormalities Early enteral feeding Epidural anesthesia, if
indicated Avoid excessive intravenous fluid administration Consider mu-opioid antagonists
• Correct electrolyte abnormalities
• Early enteral feeding
• Epidural anesthesia, if indicated
• Avoid excessive intravenous fluid administration

Page 24

674
• Consider mu-opioid antagonists
• After laparotomy
• Metabolic and electrolyte derangements (Hypokalemia, Hyponatremia, Hypomagnesemia, Uremia,
Diabetic coma)
• Drugs (Opiates, Psychotropic drugs, Anticholinergics, Calcium channel blockers, Tricyclic
antidepressants)
• Infection (Intra abdominal inflammation, Systemic sepsis, Peritonitis, Pneumonia)
• Retroperitoneal inflammation or hemorrhage
• Mesenteric ischemia
• Hypothyroidism
• Ureteral colic
• Spinal cord injury
• Myocardial infarction
• Intraoperative measures Minimize handling of the bowel Laparoscopic approach, if possible Avoid
excessive intraoperative fluid administration
• Minimize handling of the bowel
• Laparoscopic approach, if possible
• Avoid excessive intraoperative fluid administration
• Postoperative measures Correct electrolyte abnormalities Early enteral feeding Epidural anesthesia, if
indicated Avoid excessive intravenous fluid administration Consider mu-opioid antagonists
• Correct electrolyte abnormalities
• Early enteral feeding
• Epidural anesthesia, if indicated
• Avoid excessive intravenous fluid administration
• Consider mu-opioid antagonists
• Minimize handling of the bowel
• Laparoscopic approach, if possible
• Avoid excessive intraoperative fluid administration
• Correct electrolyte abnormalities
• Early enteral feeding
• Epidural anesthesia, if indicated
• Avoid excessive intravenous fluid administration
• Consider mu-opioid antagonists
Option A: The use of intravenous patient-controlled analgesia does not affect the return of small bowel
motor activity
• Controlled analgesia with opioids delays the recovery of postoperative ileus.
Option C: The routine use of metoclopramide will hasten the return of small intestinal motor activity

Page 25

675
• Routine use of metoclopramide is not recommended.
• Although it is a promotility drug.
• There are several adverse effects, such as movement disorders which are associated with the regular
use of metoclopramide
Option D: Contrast radiographic studies have no role in distinguishing early postoperative bowel obstru
ction from normal ileus
• Contrast CT is the best test for differentiating early postoperative bowel obstruction from normal ileus.

Solution for Question 17:


Correct Option B - Neostigmine (IV):
• Colonic pseudo-obstruction, or Ogilvie's syndrome, is characterized by signs and symptoms of colonic
obstruction without a mechanical cause. Neostigmine, a parasympathomimetic drug, is the primary
treatment of choice. The parasympathetic system promotes peristalsis, and in colonic
pseudo-obstruction, there is an overactivity of the sympathetic system, leading to impaired peristalsis
and sluggish bowel sounds.
• Neostigmine acts by enhancing the parasympathetic activity, helping to relieve the obstruction and
allowing the passage of flatus. When administered intravenously, Neostigmine typically results in
prompt relief, making it the preferred initial treatment.

Incorrect Options:
Option A - Opiates: Opiates are contraindicated in colonic pseudo-obstruction. They can further inhibit
bowel motility and exacerbate the symptoms, making them an inappropriate choice for the primary trea
tment.
Option C - Flatus Tube Insertion: While flatus tube insertion may be considered in some cases, it is not
the primary treatment of choice. Neostigmine is preferred due to its ability to enhance peristalsis rapidl
y. Flatus tube insertion is typically reserved for situations where Neostigmine is contraindicated or not p
referred.
Option D - Retroperitoneal Hematoma Evaluation: Retroperitoneal hematoma evaluation is not the prim
ary intervention for colonic pseudo-obstruction. Secondary pseudo-obstruction may be associated with
conditions like retroperitoneal hematoma, but the immediate focus in treating colonic pseudo-obstructio
n lies in addressing the functional obstruction. Retroperitoneal hematoma evaluation would be relevant
if there is a
specific indication for its presence but not as the initial treatment for colonic pseudo-obstruction.

Solution for Question 18:


Correct Option C - Hartman's procedure is the recommended choice for managing sigmoid malignancy
in elderly individuals:

Page 26

676
• Hartman's procedure involves three components: sigmoidectomy, descending colostomy, and closure
of the rectal stump. In elderly patients, anastomosis is generally not preferred during emergencies,
making Hartman's procedure a suitable option for managing sigmoid malignancy. This approach allows
for the removal of the affected sigmoid colon while diverting the stool through a colostomy, avoiding the
complications associated with anastomosis in elderly individuals.

Incorrect Options:
Optin A - Resection of the sigmoid with colorectal anastomosis is the preferred approach: Resection of
the sigmoid with colorectal anastomosis is not the preferred approach for sigmoid malignancy. The info
rmation provided suggests that for sigmoid malignancy, there are three options: sigmoidectomy with co
lorectal anastomosis, total colectomy with ileorectal anastomosis (in specific circumstances), and Hart
man's procedure (especially in elderly individuals).
Option B - Total colectomy with ileorectal anastomosis is not the standard treatment for sigmoid malign
ancy: It is mentioned in the information that total colectomy is considered in cases where the patient pr
esents after 24 or 48 hours of obstruction, leading to ischemia of the entire colon. This is not the primar
y approach for sigmoid malignancy.
Option D - Conservative management with close observation is appropriate for this patient: Conservati
ve management with close observation is not appropriate for a patient with large bowel obstruction due
to malignancy, as mentioned in the information provided. Large bowel obstruction is considered a real
emergency, and surgery is recommended as soon as possible. Conservative management without inte
rvention is not suitable in this scenario.

Solution for Question 19:


Correct Option C - Cecal bascule:
• Cecal bascule is the most likely cause of the patient's symptoms. In cecal bascule, the cecum folds in
a cephalad direction anteriorly over the ascending colon, leading to intermittent isolated cecal
obstruction. This obstruction results in intermittent colicky pain. Importantly, cecal bascule typically
resolves spontaneously without the need for active treatment.

Incorrect Options:
Option A - Appendicitis: Appendicitis is characterized by inflammation of the appendix, leading to const
ant pain in the right lower quadrant. The pain in appendicitis is usually more severe and persistent than
the intermittent colicky pain seen in cecal bascule. Surgical intervention is often required to treat appe
ndicitis.
Option B - Cecal volvulus: Cecal volvulus involves a twisting of the cecum, causing acute obstruction.
Unlike cecal bascule, cecal volvulus typically presents with severe and constant abdominal pain. It is c
onsidered a
surgical emergency, and urgent intervention is required to untwist the bowel and prevent ischemia.
Option D - Crohn's disease: Crohn's disease is a chronic inflammatory condition that can affect any par
t of the gastrointestinal tract. While it may cause abdominal pain, it is usually associated with other sym
ptoms such as diarrhea, weight loss, and inflammatory changes seen on imaging studies. The pain in
Crohn's disease is not typically colicky, and the condition requires long-term management, often includi
ng medications.

Page 27

677
Page 28

678
Small Intestine
1. A 2-week-old male child has abdominal distension, incessant crying, and failure to take feeds. What
does the marked white arrow in the x-ray show?
(or)
A 2-week-old male child is brought to the emergency department with complaints of abdominal
distension, incessant crying, and failure to take feeds. An X-ray of the abdomen is ordered, which is
given below. What does the marked white arrow in the x-ray show?

A. Ileal perforation
B. Necrotizing enterocolitis
C. Meconium ileus
D. Colonic aganglionosis
----------------------------------------
2. A 42-year-old man comes to your hospital with a complaint of abdominal pain, abdominal distension,
and constipation associated with a poor appetite for the past 1 month. The patient underwent a barium
enema, and it showed a characteristic sign, which is shown in the image given below. Which of the
following is not true regarding the pathology involved?
(or)
A 42-year-old man had pain, distension, and poor appetite for 1 month. The barium enema is shown in
the image given below. Which of the following is not true regarding the diagnosis of this case?

A. Has Transverse ulcer


B. Common site - ileocecal region

679
C. Most common laboratory abnormality seen - increase in ESR
D. Surgical management is the treatment of choice
----------------------------------------
3. What is the most common indication of surgery in gastrointestinal tuberculsosis?
(or)
A 27-year-old female is admitted to the hospital with a complaint of abdominal swelling associated with
a history of fever for 2 weeks. She is anemic with a Hb of 10.0 g/dl; liver functions were within normal
limit; ESR = 70 mm/hr (Normal = <20). Based on her CT findings, she was diagnosed with GI
tuberculosis. What is the most common indication for surgery in this patient?
A. Acute Obstruction
B. Mucosal ulcerations
C. Mass in the abdomen
D. GI symptoms
----------------------------------------
4. A 56-year-old woman is admitted for asymptomatic renal stones. She received extensive resection of
the small intestine due to strangulating obstructive ileus 7 years ago. She has a strong family history of
renal stones. A plain film of the kidney, ureter, and bladder and a CT scan revealed bilateral renal
stones (right 4 mm, left 10 mm). Which of the following could have predisposed the patient to develop
urolithiasis?
(or)
Which of the following predisposed a patient who received extensive small intestinal resection and has
a strong family history of renal stones, to develop urolithiasis?
A. Family history of renal stones
B. Short bowel syndrome
C. Strangulating obstructive ileus
D. Prolonged hospitalization and bedridden
----------------------------------------
5. A 27-year-old female patient is brought to the emergency department with colicky abdominal pain for
5 hours. An abdominal CT scan revealed large ileo-ileal intussusceptions. During a subsequent
exploratory laparotomy, a markedly distended small bowel proximal to the intussuscepted segment was
found. A small bowel resection was performed using a Gastrointestinal Anastomosis (GIA) stapler.
Which technique is used for anastomosis of the small bowel?
(or)
Which technique is used for the anastomosis of the small bowel?
A. Suture in 1 layer by absorbable suture
B. Suture in 2 layer by non-absorbable suture
C. Suture in 2 layer by absorbable sutures
D. None of the above
----------------------------------------

Page 2

680
6. A 61-year-old man presented to the emergency department with abdominal pain and fever since
yesterday. A CT scan with oral contrast revealed a fistulous tract extending from the gastrojejunostomy
into an abscess cavity in the adjacent anterior abdominal wall. He underwent open wound drainage
with debridement followed by excision of the fistula one month later. Which of the following could be the
reason for the non-healing of entero cutaneous fistula?
(or)
Which of the following could be the factor for the non-healing of entero-cutaneous fistula?
A. Track length >3 cm
B. Proximal obstruction
C. Greater than 10% of bowel circumference disruption
D. Epithelialization of track
----------------------------------------
7. A three-day-old infant was brought to the emergency department by his mother, who complained of
poor feeding and incessant crying. The child had not passed meconium even after 48 hours. On
examination, the abdomen was distended. Abdominal x-rays were ordered, which showed a
double-bubble sign. What part of the gastrointestinal tract is most commonly involved in this disease?
(or)
What part of the gastrointestinal tract is most commonly involved when the abdominal x-ray shows
double bubble sign?
A. Duodenum
B. Jejunum
C. Ileum
D. Colon
----------------------------------------
8. A 3-day-old infant was brought to the emergency department with vomiting and abdominal distention.
Prenatal history revealed polyhydramnios and pregnancy-induced hypertension. The baby was born at
term by emergency LSCS. Abdominal examination revealed distension and dilated bowel loops. After
per rectal wash, pale jelly stools were expelled. An emergency laparotomy was performed. The feature
shown in the image is seen in which type of ileal atresia?
(or)
The feature shown in the image is seen in which type of ileal atresia?

Page 3

681
A. Type 1
B. Type 2
C. Type 3
D. Type 4
----------------------------------------
9. Which of the following statements is correct about small bowel tumors in a patient with metastases in
the liver?
(or)
A 55-year-old male presented to the outpatient department with non-specific recurrent upper abdominal
pain, weight loss of around 8 kg, and anorexia for the past six months. On further evaluation with
ultrasound and CT scan, the patient was diagnosed with a small bowel tumor. PET scan showed
numerous metastases in the liver. Which of the following statement is correct about small bowel
tumors?
A. The most common small bowel tumorsis adenoma
B. Small bowel tumors are the most common gastrointestinal tumors
C. The preferred chemotherapy regimen for metastatic small bowel malignancies is FOLFOX
D. The most common gastrointestinal tumor in children is adenocarcinoma
----------------------------------------
10. A 60-year-old man came to the ER with severe abdominal pain with vomiting and constipation for 5
hours. The patient has a history of Crohn’s disease, and abdominal imaging showed features of
intussusception. The patient was taken for an emergency laparotomy intraoperatively to reduce the
intussusception; a mass was noted in the terminal ileum causing ileocolic intussusception. A biopsy
confirmed malignancy; what is the most probable tumor in this case?
(or)
What is the most likely malignant tumor in a patient with a history of Crohn’s disease with a mass in the
terminal ileum causing ileocolic intussusception?
A. Carcinoid tumor
B. Gastrointestinal stromal tumor(GIST)
C. Adenocarcinoma
D. Lymphoma
----------------------------------------
11. What is the most probable cause of intestinal obstruction in a patient with an intraluminal mass in
the ileocolic junction causing intestinal obstruction?
(or)
A 41-year-old female presented to the ER with severe colicky abdominal pain, nausea, vomiting, and
constipation for one day. She said that she could pass flatus but not stool; she also had an episode of
acute choledocholithiasis 1 week ago. On examination, her abdomen is distended. A CT scan shows a
small intraluminal mass in the ileocolic junction, causing intestinal obstruction. What is the most
probable cause of intestinal obstruction in this patient?
A. Adenocarcinoma

Page 4

682
B. Gallstones
C. Adhesions
D. Hernia
----------------------------------------
12. A 32-year-old male presented to the clinic with the complaint of painless fresh rectal bleeding for 2
weeks. He denies nausea, vomiting, diarrhea, tenderness, etc. The doctor suspects the bleeding is
coming from Meckel's diverticulum. What would be the best diagnostic modality for this disease?
(or)
What would be the best diagnostic modality for Meckel diverticulum?
A. Upper GI endoscopy
B. Abdominal CT contrast
C. Radioisotope scanning with technetium-99m
D. Stool DR
----------------------------------------
13. A 55-year-old man with Parkinson's is admitted with abdominal pain, irregular bowel movements,
and weight loss, all associated with occasional flushing episodes for the past ten months. Physical
examination revealed spider-like facial veins, and hepatomegaly. Urinary 5-HIAA level is 320 mg/24 h
(range<10), and chromogranin A is >1200 ng/ml (range<98 ng/ml). A barium enema revealed a filling
defect in the ileocecal valve. Which of the following is correct regarding this syndrome?
(or)
What is correct regarding a syndrome that presents with high urinary HIAA and chromogranin A levels
and a filling defect in the ileocecal valve?
A. Occurs in <10% of these tumours
B. Omeprazole injection reduces symptoms of flushing and diarrhea
C. It occurs when the tumours metastasize to the adrenal gland
D. Pale-looking skin is a typical feature
----------------------------------------
14. What is the diagnosis for a 12-year-old boy with melena and small black macules on the lips and
oral mucosa.
(or)
A 12-year-old boy presented with black stools for two days. On physical examination, small black
macules are on the lips, oral mucosa, extremities, and perianal skin. No anal fissures or hemorrhoids
were seen on the rectal examination. He has no other medical conditions. The mother states that his
older sister also had similar lesions that faded with time. What is the most likely diagnosis?
A. Carcinoid tumor
B. Melanoma
C. Villous adenoma
D. Peutz-Jegher syndrome
----------------------------------------

Page 5

683
15. A 5-month-old boy is brought to the emergency department with colicky abdominal pain that began
6 hours ago. He had episodes of non-bilious, non-projectile vomiting and loose stools with currant
jelly-like appearance. On examination, there was a sausage-shaped mass in the right lumbar region.
Which of the following can be used as a diagnostic and therapeutic modality in this case?
(or)
What can be used as a diagnostic and therapeutic modality in an infant with a sausage-shaped mass in
the right lumbar region?
A. Contrast enema
B. Flexible sigmoidoscopy
C. Surgical intervention
D. Computed tomography (CT) scan
----------------------------------------
16. A 47-year-old female working as a staff nurse in a local primary health center was referred to a
hospital with a one-month history of continuous abdominal pain and distension, fever, night sweats, and
significant weight loss. An abdominal examination revealed no organomegaly. Barium meal
follow-through was done. Which of the following radiological signs is seen in the image?
(or)
Which of the following radiological signs is seen in the barium follow-through of a woman with
abdominal pain and distension, fever, night sweats, and significant weight loss?

A. String sign of Kantor


B. Gooseneck appearance
C. Fleischner's sign
D. Umbrella sign
----------------------------------------
17. Which of the following is the likely finding from an autopsy in an adolescent who died from acute
abdominal events, that developed a few days after she suffered from splenomegaly, leukopenia, and
rose spots on abdomen?
(or)
A 14-year-old girl presented to the outpatient department with a history of prolonged fever and
abdominal discomfort for three weeks. Her general physical examination is unremarkable. Her
abdominal examination reveals splenomegaly and rose spots. Her complete blood count shows
leukopenia. A few days later, she is admitted and develops acute abdominal events, and dies. Which of

Page 6

684
the following is the likely finding on autopsy?
A. Transverse ulcers
B. Longitudinal ulcers
C. Pinpoint ulcers
D. Pseudopolyps
----------------------------------------
18. For gastric cancer, a 45-year-old male underwent subtotal gastrectomy with Roux-en-Y
gastrojejunostomy. Postoperatively, the patient developed an enterocutaneous fistula. The patient was
treated conservatively with total parenteral nutrition and electrolyte abnormalities were corrected.
Which of the following fistula types has the highest potential for causing electrolyte abnormalities?
(or)
Which entero-cutaneous fistula type has the highest potential to cause electrolyte abnormalities?
A. Gastric
B. Duodenal
C. Sigmoid
D. Rectal
----------------------------------------
19. A 40-year-old man presents with diarrhea for the past three days; his stools are bulky, malodorous,
and difficult to flush. He has been lethargic and anorexic. His medical history is notable for Crohn’s
disease, for which he has undergone three laparotomies over the past five years, leaving only 40 cm of
small bowel behind. On ultrasound, gallstones in the abdomen are visualized. Which of the following
can be a treatment option?
(or)
Which of the following is best recommended for a patient with gallstones with a history of multiple
laparotomies for Crohn's disease leaving behind only 40 cm of small bowel behind?
A. Serial transverse enteroplasty procedure (STEP)
B. Delorme’s operation
C. Abdominoperineal resection
D. Ripstein operation
----------------------------------------
20. What could be the most probable diagnosis for a one-year-old with rectal bleeding and an
abdominal CT scan revealing an ectopic tubular mass near the mesenteric border of the ileum?
(or)
A one-year-old male child was brought by his parents to the pediatric emergency department with
complaints of vomiting and abdominal pain for three days. He had an episode of fresh rectal bleeding
mixed with stool. On examination, the abdomen was distended, rigid, and tender. An abdominal
computed tomography (CT) scan revealed an ectopic tubular mass near the mesenteric border of the
ileum. What could be the most probable diagnosis?
A. Duplication of Alimentary tract
B. Meckel’s Diverticulum

Page 7

685
C. Mesenteric Cyst
D. Torsion of the omentum
----------------------------------------
21. A 46-year-old male presented to the clinic with complaints of abdominal pain associated with
nausea, vomiting, weight loss, and melena. Endoscopy and abdominal CT scan contrast showed a
mass in the region of the ampulla of Vater; a biopsy confirmed the diagnosis to be adenocarcinoma.
What would be the best surgical treatment for adenocarcinoma in this region?
(or)
What would be the best surgical treatment for adenocarcinoma at the ampulla of Vater?
A. Pancreaticoduodenectomy
B. Segmental intestinal resection with lymphadenectomy
C. Wide local resection
D. Right hemicolectomy
----------------------------------------
22. A 34-year-old male patient presents with recurrent episodes of abdominal pain and gastrointestinal
bleeding. Diagnostic investigations reveal the presence of Meckel's diverticulum with an unhealthy
base, showing clear signs of ulceration. The patient is otherwise healthy. What is the most appropriate
treatment?
(or)
What is the best treatment approach for a woman with Meckel's diverticulum with an unhealthy base
and ulceration?
A. Diverticulectomy alone
B. Resection of Meckel's diverticulum with anastomosis
C. Observation without surgical intervention
D. Symptomatic management with proton pump inhibitors
----------------------------------------
23. Which type of carcinoid tumor is primarily associated with the best prognosis?
A. Laryngeal carcinoid
B. Carcinoid of the appendix
C. Pancreatic carcinoid
D. Colon carcinoid
----------------------------------------
24. In a patient with carcinoid heart disease, which valve is primarily affected due to increased
serotonin or 5-HTA production, leading to valvular abnormalities?
A. Aortic valve
B. Mitral valve
C. Tricuspid valve
D. Pulmonary valve

Page 8

686
----------------------------------------
25. Which of the following statements regarding small bowel carcinoid is correct?
A. Small bowel carcinoid originates from the G cells located in the crypts of Lieberkühn
B. Small bowel carcinoid is not argentaffinic, unlike midgut carcinoids
C. Carcinoid syndrome is commonly associated with small bowel carcinoid due to its midgut origin
D. Carcinoid tumors are generally large in size initially.
----------------------------------------
26. Which variety of intestinal tuberculosis is characterized by the formation of transverse ulcers in the
ileum, often associated with decreased immunity and high virulence of the organism?
A. Sclerotic / Fibrotic variety
B. Hyperplastic variety
C. Ulcerative variety
D. None of the above
----------------------------------------

Correct Answers
Question Correct Answer

Question 1 2
Question 2 4
Question 3 1
Question 4 2
Question 5 1
Question 6 4
Question 7 1
Question 8 3
Question 9 3
Question 10 3
Question 11 2
Question 12 3
Question 13 1
Question 14 4
Question 15 1
Question 16 1
Question 17 2
Question 18 2

Page 9

687
Question 19 1
Question 20 1
Question 21 1
Question 22 2
Question 23 1
Question 24 3
Question 25 3
Question 26 3

Solution for Question 1:


Option B: Necrotizing enterocolitis
• This patient's symptoms of abdominal pain, reduced oral intake, and abdominal distension, together
with the radiological findings of air in the bowel wall, are indicative of necrotizing enterocolitis.
• Pneumatosis intestinalis is the specific term for the presence of gas within the wall of the small or
large intestine.
• It involves subserosa, submucosa, and rarely, muscularis layer.
• Most common site: Jejunum > ileocecal region > colon
• Pneumatosis in neonates is usually associated with necrotizing enterocolitis In gross appearance, the
cysts resemble cystic lymphangiomas or hydatid cysts On the histologic section, the involved portion
has a honeycomb appearance
• In gross appearance, the cysts resemble cystic lymphangiomas or hydatid cysts
• On the histologic section, the involved portion has a honeycomb appearance
• On plain films, pneumatosis intestinalis appears as radiolucent areas within the bowel wall, as shown
by the arrow.
• The radiolucency may be linear or curvilinear or appear as grape-like clusters or tiny bubbles.
• In gross appearance, the cysts resemble cystic lymphangiomas or hydatid cysts
• On the histologic section, the involved portion has a honeycomb appearance
Option A: Ileal perforation
• Small bowel (SB) perforation is an acute pathological condition resulting from a discontinuity of the
small bowel wall secondary to trauma, typhoid fever, and congenital conditions in this age group with
subsequent leakage of intestinal gas and contents into the peritoneal cavity.
• Abdominal x-ray shows free air under the diaphragm.
• The x-ray shown in the question shows air in the bowel wall and not under the diaphragm.

Page 10

688
Option C: Meconium ileus
• Meconium ileus refers to a neonatal bowel obstruction of the distal ileum due to an abnormally thick
and impacted meconium.
• It is usually a manifestation of cystic fibrosis.
• Plain radiographs are non-specific and may show dilated bowel loops proximal to the impaction.
Classically, there is a paucity or absence of air-fluid levels and a "bubbly" appearance of the distended
intestinal loops on radiographs.

Option D: Colonic aganglionosis


• Hirschsprung disease is characterized by aganglionosis (absence of ganglion cells) in the distal colon
and rectum.
• X-ray findings are primarily those of a bowel obstruction. The affected bowel is of smaller caliber; thus,
depending on the length of the segment affected, a variable amount of colonic distension is present.

Page 11

689
Solution for Question 2:
Option D: Surgical management is the treatment of choice
• The given image shows Goose Neck Deformity, which is a classical sign of Abdominal TB.
• The deformity is characterized by fibrosis of the cecum (red arrow) and dilated ileum (blue arrow) with
a loss of normal ileocecal angle.
• Gastrointestinal tuberculosis: Caused by Mycobacterium tuberculosis Two types - Ulcerative and
Hyperplastic
• Caused by Mycobacterium tuberculosis
• Two types - Ulcerative and Hyperplastic
• CT scan is the modality of choice in evaluating the extent and type of GI TB.
• Medical therapy with antituberculous drugs is the treatment of choice, especially in uncomplicated
cases.
• Surgery is not the treatment of choice and is indicated only in complicated cases.
• Caused by Mycobacterium tuberculosis
• Two types - Ulcerative and Hyperplastic
Option A: Has Transverse ulcer
• In most cases of abdominal TB, the ileum has transverse ulcers on gross appearance.
• Longitudinal ulcers are present on enteric fever (typhoid).
• Hence this finding is not unusual in abdominal TB.
Option B: Common site - ileocecal region
• Extrapulmonary TB accounts for 12% of TB cases and among them, gastrointestinal TB accounts for
16% of cases, with the ileocecal region being the most common one.

Page 12

690
• Hence this is a true statement and corresponds with the diagnosis.
Option C: Most common laboratory abnormality seen - increase in ESR
• Tuberculosis is a chronic mycobacterial infection and leads to an inflammatory response in the body
by virtue of Interferon-gamma interleukins.
• This inflammatory response results in markedly increased ESR.
• Hence this is a true statement and corresponds with the diagnosis.

Solution for Question 3:


Option A: Acute Obstruction
• Patients with abdominal tuberculosis have a wide range of clinical presentations due to complex
pathological processes.
• The diagnosis is usually based on radiological imaging including ultrasound and CT scan.
• Common acute presentations include intestinal obstruction, perforation, and rarely gastrointestinal
bleeding.
• The other clinical presentation may be in the form of recurrent intestinal obstruction, ileocecal mass
with an uncertain diagnosis, fistulizing tuberculosis, and incidentally detected peritoneal tuberculosis.
Management includes a judicious combination of anti-tubercular treatment and intervention (surgery
and endoscopic).
• Surgery is indicated in the following conditions: Intestinal obstruction secondary to a stricture (most
common) Free perforation Severe GI hemorrhage Intra-abdominal abscess Internal or external fistula
• Intestinal obstruction secondary to a stricture (most common)
• Free perforation
• Severe GI hemorrhage
• Intra-abdominal abscess
• Internal or external fistula
• Intestinal obstruction secondary to a stricture (most common)
• Free perforation
• Severe GI hemorrhage
• Intra-abdominal abscess
• Internal or external fistula
Option B: Mucosal ulcerations
• Gastrointestinal tuberculosis presents with different symptoms and mucosal ulcerations are one of the
most common symptoms.
• Surgery is only indicated in acute and emergency conditions. Since mucosal ulcerations in TB is not a
surgical emergency, therefore it is not an indication for surgical exploration.
Option C: Mass in the abdomen

Page 13

691
• Mass in the abdomen is a common physical exam finding of tuberculosis. Surgery is only indicated in
acute and emergency conditions.
• Since abdominal mass in TB is not a surgical emergency, therefore it is not an indication for surgical
exploration.
• However, if the mass remains undiagnosed even after multiple biopsies, then it requires surgical
exploration.
Option D: GI symptoms
• Non-acute GI symptoms in TB are not a surgical emergency.
• Surgery is indicated only if there is evidence of obstruction, perforation, or severe GI hemorrhage

Solution for Question 4:


Option B: Short bowel syndrome
• Short bowel syndrome is a malabsorptive condition that arises secondary to the removal of significant
segments of the small intestine. Net secretors: <100 cm bowel Net absorbers : >100 cm bowel
• Net secretors: <100 cm bowel Net absorbers : >100 cm bowel
• The most common causes are small mesenteric artery infarction, Crohn's disease, tumors, and
radiation enteritis.
• It usually appears when there is less than 200 cm of small bowel.
• Net secretors: <100 cm bowel Net absorbers : >100 cm bowel
• Unabsorbed fatty acids bind with calcium.
• Increased concentration of free oxalates (oxalates bind with calcium normally and therefore escape
without intestinal absorption) --> free oxalates are absorbed --> oxalate kidney stones
• Vitamin B12 Malabsorption- Megaloblastic anemia.
• Bile salts Malabsorption-watery diarrhea
• Decreased bile salts in the bile leads to: Cholesterol gall stones Steatorrhea and Malabsorption of
fat-soluble vitamins
• Cholesterol gall stones
• Steatorrhea and Malabsorption of fat-soluble vitamins
• Cholesterol gall stones
• Steatorrhea and Malabsorption of fat-soluble vitamins
Option A: Family history of renal stones
• Kidney stones develop more frequently in individuals with a family history of kidney stones than in
those without a family history; however, little information is available regarding whether the increased
risk is attributable to genetic factors, environmental exposures, or some combination
• This patient has a history of small bowel resection, which predisposes him to kidney stones
Option C: Strangulating obstructive ileus

Page 14

692
• Strangulated bowel obstruction is defined as ischemia and infarction of an obstructed loop of the
bowel because of the compromised blood supply.
• The disease itself has no association with the development of kidney stones. However, bowel
resection performed as a result of it predisposes to kidney stones.
Option D: Prolonged hospitalization and bedridden
• The risk factors for kidney stones include metabolic diseases, small bowel resection, recurrent urinary
transactions, dehydration, and poor dietary habits.
• Prolonged hospitalization is not a risk factor for kidney stones.

Solution for Question 5:


Option A: Suture in 1 layer by absorbable suture
• Anastomosis of the small bowel is done by one-layer anastomoses.
• It is relatively quick and has similar efficacy to double-layer suturing.
• It can be done using interrupted sutures or continuous sutures.
• It involves the full thickness of the intestinal wall.
Incorrect Options:
Option B: Suture in 2-layer by non-absorbable suture
• Two-layer anastomosis is, by comparison, more time-consuming than single-layer anastomosis.
• The inner layer is stitched with an absorbable suture, and the outer layer can be stitched with a
non-absorbable suture.
Option C: Suture in 2 layers by absorbable sutures
• Absorbable sutures can be used to stitch the inner and outer layers of a two-layer full-thickness stitch.
• The efficacy of a two-layer anastomosis is similar to that of a one-layer anastomosis.
Option D: None of the above
• Anastomosis of the small bowel is done by one-layer anastomoses.

Solution for Question 6:


Option D: Epithelialization of track
• An enterocutaneous fistula is a pathological communication between the small or large intestines and
the skin.
• 75% or more of enterocutaneous fistula arise as postoperative complications, whereas 15-25% result
from trauma or spontaneously due to cancer, inflammatory bowel disease (IBD), irradiation, or ischemic
or infective diseases.
• Several factors can prevent spontaneous fistula closure, including: High output fistulas (>500ml/24
hours) Severe disruption of intestinal continuity (>50% of bowel circumference) Active inflammatory
disease of bowel segment Cancer Radiation enteritis Distal obstruction Undrained abscess cavity

Page 15

693
Foreign body in the fistula tract Fistula tract < 2.5 cm long Epithelialization of the fistula tract
• High output fistulas (>500ml/24 hours)
• Severe disruption of intestinal continuity (>50% of bowel circumference)
• Active inflammatory disease of bowel segment
• Cancer
• Radiation enteritis
• Distal obstruction
• Undrained abscess cavity
• Foreign body in the fistula tract
• Fistula tract < 2.5 cm long
• Epithelialization of the fistula tract
• High output fistulas (>500ml/24 hours)
• Severe disruption of intestinal continuity (>50% of bowel circumference)
• Active inflammatory disease of bowel segment
• Cancer
• Radiation enteritis
• Distal obstruction
• Undrained abscess cavity
• Foreign body in the fistula tract
• Fistula tract < 2.5 cm long
• Epithelialization of the fistula tract
Option A: Track length >3 cm
• If the tract length of the fistula is less, chances for spontaneous closure will be low.
• This is because of decreased resistance to flow.
• A track length of less than 2.5cm prevents the spontaneous closure of the fistula.
Option B: Proximal obstruction
• Proximal obstructions have a better outcome than distal obstructions.
• Proximal fistulas are more common, closure is spontaneous.
• Distal obstruction may, despite the presence of other favorable factors, prevent the closure of an ECF.
• If present, surgical intervention is needed.
Option C: Greater than 10% of bowel circumference disruption
• Post-operatively, disruption of the anastomosis can be due to inadequate blood flow, especially when
extensive mesenteric vessels are ligated.
• Generally, severe disruption (around 50% of bowel circumference or more) is shown to prevent
spontaneous closure of the fistula.

Page 16

694
Solution for Question 7:
Option A: Duodenum
• The finding of a double bubble appearance on abdominal X-rays points to the diagnosis of
duodenal atresia.
• Intestinal atresia is a congenital disability resulting in intestinal obstruction in neonates. It develops in
late pregnancy due to inadequate blood flow to the developing bowel.
• Bilious vomiting is the dominant feature in jejunal atresia, while abdominal distension is the prominent
feature in ileal atresia.
• In descending frequency, intestinal atresia sites are the duodenum, followed by the jejunum, ileum,
and colon.
Option B: Jejunum
• The signs and symptoms may be similar to duodenal atresia.
• The radiographic sign of jejunal atresia is a triple bubble appearance. It is similar to the double bubble
sign of duodenal appearance plus a third bubble caused by enlargement of the jejunum by air.
Option C: Ileum
• An abdominal radiograph will show dilated loops of the proximal bowel.
• It is often incidentally diagnosed on prenatal ultrasounds or during the evaluation of polyhydramnios.
On ultrasound, there is a proximal dilated intestinal segment.
Option D: Colon
• Colonic atresia is very rare.
• Radiological findings are distended bowel loops.

Solution for Question 8:


Option C: Type 3
• In this case, the neonate presented with vomiting and abdominal distention, suggesting intestinal
obstruction. On abdominal examination, dilated bowel loops are also found. The mother also had
polyhydramnios and PIH during pregnancy, which suggests apple-peel atresia.
• Type 3b atresia, also known as apple-peel intestinal atresia or Christmas tree intestinal atresia, is
rare. It is characterized by the duodenum or proximal jejunum ending in a blind pouch. The distal small
bowel wraps around its vascular supply in a spiral resembling an apple peel.
• Often a vast segment of the small bowel is absent.
• Surgical intervention is the definitive treatment.
Incorrect Options:
Option A: Type 1

Page 17

695
• In this type, partial or complete atresia is caused by a membrane that forms inside the intestine.
• The intestine usually grows to normal length.
Option B: Type 2
• In this type of atresia, one or more segments of the intestine form a "blind end.”
• A fibrous cord connects the blind ends.
• The length of the intestine may be normal.
Option D: Type 4
• This involves several obstructions, resulting in a “string-of-sausages” appearance.
• Various combinations of types I, II, and III atresia may be involved.
• The intestine is shorter than normal.

Solution for Question 9:


Option C: The preferred chemotherapy regimen for metastatic small bowel malignancies is FOLFOX
• Small bowel tumors are rare gastrointestinal tumors with vague presentations, e.g., nonspecific upper
abdominal pain and anorexia. They are difficult to diagnose due to the low index of suspicion and are
usually diagnosed when they've metastasized.
• FOLFOX is the chemotherapy regimen including Folinic Acid (Leucovorin), 5-Fluorouracil and
Oxaliplatin. It is used as the first-line treatment in patients with metastasized disease.
• It has the best response, survival rates, and the least toxicity.
• Surgical resection is the best mode of treatment for locoregional disease.
Incorrect Options:
Option A: The most common small bowel tumor is adenoma
• Adenocarcinoma is the most common tumor of the small intestines. 98% of all adenocarcinomas
occur in the large intestine.
• Adenomas are benign but premalignant intraepithelial neoplasms that are the presumed precursor
lesions of adenocarcinoma.
• Adenomas greater than 1cm in diameter with villous histology and high-grade dysplasia are more
likely to progress to malignancy.
Option B: Small bowel tumors are the most common gastrointestinal tumors
• Even though the small intestine is the longest part of the GIT and has the largest surface area, tumors
of the small intestine are the rarest, accounting for less than 2% of all gastrointestinal neoplasms
• Colorectal cancers are the most common gastrointestinal tumors. Their incidence is 50-fold higher
than small intestine tum.
Option D: The most common gastrointestinal tumor in children is adenocarcinoma
• Primary GI tumors are rare in the pediatric age group, accounting for less than 1% of all tumors in
children.

Page 18

696
• Non-Hodgkin lymphoma is the most common small bowel neoplasm in children. The most common
histologic subtype is Burkitt's lymphoma.

Solution for Question 10:


Option C: Adenocarcinoma
• Patients with Crohn’s disease have a predilection to develop adenocarcinoma.
• It is the most common malignant tumor to cause ileocolic intussusception, the cause of intestinal
obstruction in this patient.
• It occurs most commonly in the duodenum; an exception is in people with Crohn's disease, whose
most common location is the terminal ileum.
• Adenocarcinomas in the duodenum are usually diagnosed early compared to the ileum, which is
usually diagnosed at an advanced stage with metastatic processes having started.
• If occurring in the periampullary region can cause obstructive jaundice and pancreatitis by causing the
obstruction.
Incorrect Options:
Option A: Carcinoid tumor
• These are found in five typical sites in decreasing order of frequency: Appendix (38%) > small
intestine (29%) > colon > stomach > rectum.
• It is usually diagnosed later after metastasis; the most common site of metastasis is the liver.
Option B: Gastrointestinal stromal tumour (GIST)
• The most common site of their occurrence is the stomach, and the small intestine is the second most
common site.
• They are more likely to be associated with overt hemorrhage than other malignancies.
Option D: Lymphoma
• The ileum is the most common site of their presentation due to the many lymphoid follicles in that
region.
• It can present with features of bowel obstruction as well as bowel perforation.
• It has no association with Crohn’s disease.

Solution for Question 11:


Option B: Gallstones
• Gallstones cause intraluminal intestinal obstruction, and their most common location is the ileocolic
junction.
• Severe colicky abdominal pain along with nausea, vomiting, and constipation suggests intestinal
obstruction, and the patient can pass flatus but not stool which means there is partial obstruction. The
patient had an episode of choledocholithiasis, which means she had a stone in her bile duct and the
mass on the CT scan was also intraluminal, causing intestinal obstruction in the ileocolic junction.

Page 19

697
• The ileocolic junction is a narrow opening; thus, even small objects like gallstones can impact this
region and cause intestinal obstruction, thus stopping the movement of gastrointestinal content further
in the bowel.
• Gallstones are the most common cause of intraluminal intestinal obstruction.
Incorrect Options:
Option A: Adenocarcinoma
• Adenocarcinoma is the most common cause of malignancy in the gastrointestinal tract and presents
with intestinal obstruction.
• It is an intramural cause of intestinal obstruction.
Option C: Adhesions
• Adhesions occur most commonly in patients with a history of abdominal surgery like laparotomy,
C-section, etc.
• They are a cause of extrinsic intestinal obstruction.
Option D: Hernia
• A hernia is caused by a defect in any layer of the abdominal wall, most commonly abdominal
musculature, leading to herniation of any bowel content.
• Herniation of the intestine is a cause of extrinsic intestinal obstruction.

Solution for Question 12:


Option C: Radioisotope scanning with technetium-99m
• Complaint of fresh rectal bleeding that is painless without any other symptoms is usually suggestive of
Meckel’s diverticulum, and the best diagnostic modality is radioisotope scanning with technetium-99m.
• It is a persistent remnant of the vitellointestinal duct and is present in about 2% of the population; it is
found on the antimesenteric side of the ileum, usually approximately 60cm from the ileocaecal valve,
and is classically 5cm long.
• It contains all three coats of the bowel wall with its blood supply and is congenital.
• It usually presents with hemorrhage when gastric mucosa is inside and presents as painless fresh or
dark rectal bleeding.
• If not removed, it can lead to diverticulitis, chronic ulceration, intussusception, intestinal obstruction,
and perforation.
Incorrect Options:
Option A: Upper GI endoscopy
• Upper GI endoscopy excludes pathologies in the upper GI system like the esophagus, duodenum,
and jejunum.
• It is useful in evaluating causes of melena but doesn’t show Meckel’s diverticulum.
Option B: Abdominal CT contrast
• Abdominal CT contrast is mostly useful for ruling out GI tract malignancies.
• It is usually unsuccessful in showing Meckel’s diverticulum.

Page 20

698
Option D: Stool DR
• Stool DR is useful for evaluating infective causes of GI disorders like H. pylori and different infections.
• Since Meckel's diverticulum is not an infective cause, it doesn't show up on stool DR.

Solution for Question 13:


Option A: Occurs in <10% of these tumors
• The given condition with elevated 5-HIAA and chromogranin A indicates carcinoid syndrome.
• Carcinoid syndrome occurs in <10% of patients with carcinoid tumors.
• Intestinal carcinoid tumors / neuroendocrine tumors (NET) arise from enterochromaffin cells located at
the base of the crypts of Lieberkuhn.
• They occur most commonly in the appendix, ileum, and rectum.
• They produce several vasoactive peptides, most commonly serotonin but also histamine,
prostaglandin, and kallikrein.
• When they metastasize to the liver, carcinoid syndrome becomes evident, manifesting with episodic
flushing, tachycardia, diarrhea and bronchospasm.
Incorrect Options:
Option B: Omeprazole injection reduces symptoms of flushing and diarrhea
• Omeprazole is a proton pump inhibitor and has no role in reducing the symptoms of flushing and
diarrhea.
• These symptoms can be reduced by octreotide injection in carcinoid syndrome.
• Octreotide is a somatostatin analog. It reduces GI motility, inhibits gastric acid secretion, and
decreases IG mucosal blood flow.
Option C: It occurs when carcinoid tumors metastasize to the adrenal gland
• When carcinoid tumors metastasize to the liver, carcinoid syndrome becomes evident.
• In patients with no liver metastases, serotonin released by carcinoid tumors will become inactive in the
liver.
• In a patient with liver metastases, serotonin bypasses the first-pass metabolism in the liver and
directly enters the bloodstream, causing symptoms.
Option D: Pale-looking skin is a typical feature
• The most common presenting symptom of carcinoid syndrome is skin flushing.
• The skin color of the face and upper chest changes from red to purple, and it feels hot.
• Bright-red patchy flushing is typically seen in Gastric carcinoids.

Solution for Question 14:


Option D: Peutz-Jegher syndrome

Page 21

699
• Hyperpigmented mucocutaneous macules on the lips, oral mucosa and extremities with a positive
family history of such pigmentation suggest Peutz-Jegher syndrome (PJS).
• Gastrointestinal bleeding is a common complication in the first three decades of life, usually seen
when a polyp ulcerates.
• PJS is a rare autosomal dominant condition characterized by benign hamartomatous polyps in the
small bowel, colon and stomach.
• It is caused by a mutation in the tumor suppressor STK11 gene. This gene is also expressed in other
tissues of the body, and a mutation increases the risk of breast, pancreatic, testicular, ovarian, and lung
cancers.
• Other complications include anemia, intussusception, and intestinal obstruction.
• Diagnostic criteria include a patient having one of the following: More than two PJ polyps confirmed
histologically More than one PJ polyp, along with a positive family history PJS-associated
mucocutaneous lesions
• More than two PJ polyps confirmed histologically
• More than one PJ polyp, along with a positive family history
• PJS-associated mucocutaneous lesions
• More than two PJ polyps confirmed histologically
• More than one PJ polyp, along with a positive family history
• PJS-associated mucocutaneous lesions

Incorrect Options:
Option A: Carcinoid tumor
• Carcinoid tumors are slow-growing neuroendocrine tumors that commonly arise in the GIT from the
enterochromaffin cells.
• They are usually asymptomatic and benign in children till they metastasize to the liver, cause liver
dysfunction and produce flushing, bronchospasm, abdominal pain, and diarrhea due to a decrease in
the metabolism of hormones, e.g. serotonin, histamine, and bradykinin.

Page 22

700
Option B: Melanoma
• Melanoma is an aggressive form of skin cancer that may develop at any age but is usually seen in
adults in the 5th to 6th decade. It is rare in childhood.
• Melanomas can be differentiated from benign moles and hyperpigmentation by the ABCDEs, as
follows: Asymmetry Border irregularity Color variation within the lesion Diameter greater than 6mm
Evolves rapidly and has skin elevation
• Asymmetry
• Border irregularity
• Color variation within the lesion
• Diameter greater than 6mm
• Evolves rapidly and has skin elevation
• Asymmetry
• Border irregularity
• Color variation within the lesion
• Diameter greater than 6mm
• Evolves rapidly and has skin elevation
Option C: Villous adenoma
• Adenomas are intraepithelial neoplasms that range from small pedunculated polyps to large sessile
lesions.
• They are premalignant lesions that arise due to gene mutations in APC, KRAS and p53 genes.
• Villous polyps are slender, sessile finger-like projections usually in the recto-sigmoid region seen in
old age > 50 years.

Solution for Question 15:


Option A: Contrast enema
• Vomiting, currant jelly stools with abdominal pain, and an abdominal mass in an infant are signs and
symptoms suggestive of intussusception.
• A barium contrast enema can be used to diagnose ileocolic intussusception, which shows the ‘claw
sign.’ However, it cannot diagnose small-small bowel intussusception.
• Reduction via enema is also the mainstay treatment of intussusception in children.
• After administering IV fluids and antibiotics, decompression may be attempted via an enema with
barium or water-soluble contrast or pneumatic techniques. Contrast enema is contraindicated in cases
presenting with perforation, shock, or a pathological lead point.
• Air enema is the treatment of choice in many places as it is faster, cheaper, and safer (less risk of
peritoneal contamination if perforation occurs)
• It is successful if the patient becomes asymptomatic or the reducing agent is seen to reflux back into
the terminal ileum.
Incorrect Options:

Page 23

701
Option B: Flexible sigmoidoscopy
• It is a minimally invasive diagnostic technique that images the distal 60 cm of the large intestine from
the sigmoid colon to the rectum.
• It Is of little use in intussusception as most cases occur at the ileocolic junction and would not be
visualised by sigmoidoscopy.
Option C: Surgical intervention
• Surgical intervention is not the first choice in hemodynamically stable patients. It is indicated if the
non-operative reduction is unsuccessful after two consecutive attempts or if an obvious perforation is
seen.
• The reduction can be done by delivering the intestine into the wound and milking the intussusceptum
out of the intussuscipiens. If perforation is present, a resection with end-to-end anastomosis may be
done.
Option D: Computed tomography (CT) scan
• CT scan is only a diagnostic modality for intussusception. It is not therapeutic.
• A 'target' sign Is seen on CT; however, the findings are unreliable, and with the risk of radiation,
intravenous contrast and sedation in children, it is not the diagnostic procedure of choice in
intussusception.

Solution for Question 16:


Option A: String sign of Kantor
• Barium study showed persistent narrowing due to stricture formation, also called the string sign of
Kantor. This sign is seen in gastrointestinal tuberculosis as well as Crohn’s disease. Considering the
likelihood of an infectious cause and the radiographic sign, the diagnosis is gastrointestinal
tuberculosis.
• Barium meal follow-through is a diagnostic procedure used for the diagnosis of many diseases of the
gastrointestinal tract. On barium meal follow-through In tuberculosis, the following findings can be seen:
• Contraction of the mesocolon Pulled-up cecum Obtuse ileocecal angle Gooseneck deformity Stierlin
sign: The rapid uptake of barium into the colon due to the formation of transverse ulcers in the ileum
causes hyper-irritability. This results in a narrowed appearance of the ileum. Persistent narrowing due
to strictures results in a string sign of Kantor.
• Contraction of the mesocolon
• Pulled-up cecum
• Obtuse ileocecal angle
• Gooseneck deformity
• Stierlin sign: The rapid uptake of barium into the colon due to the formation of transverse ulcers in the
ileum causes hyper-irritability. This results in a narrowed appearance of the ileum.
• Persistent narrowing due to strictures results in a string sign of Kantor.
• Contraction of the mesocolon
• Pulled-up cecum

Page 24

702
• Obtuse ileocecal angle
• Gooseneck deformity
• Stierlin sign: The rapid uptake of barium into the colon due to the formation of transverse ulcers in the
ileum causes hyper-irritability. This results in a narrowed appearance of the ileum.
• Persistent narrowing due to strictures results in a string sign of Kantor.
Option B: Gooseneck appearance
• This is a feature of gastrointestinal tuberculosis affecting the ileum.
• The dilated terminal ileum appears to be hanging from a retracted cecum, resulting in a gooseneck
appearance.
Option C: Fleischner's sign
• This is a sign of ileocecal tuberculosis.
• Thickening of ileocecal valve lips and wide gaping of the valve with narrowing of the terminal ileum
results in an inverted umbrella sign or Fleischner's sign.
Option D: Umbrella sign
• The umbrella sign is found in long-standing cases of gastrointestinal tuberculosis.
• The thickened ileocecal valve, rigid cecum, and narrowed ileum are umbrella signs.

Solution for Question 17:


Option B: Longitudinal ulcers
• The clinical presentation and lab abnormalitues suggests typhoid or enteric fever.
• Enteric fever is caused by Salmonella typhi or paratyphi.
• Presents with fever and abdominal pain after an incubation period of 10-20 days.
• Over the next week, the patient can develop distension, diarrhea, splenomegaly, and usual rose spots
on the abdomen caused by vasculitis.
• Ulceration and necrosis of Ileocecal Peyer’s patches are pathological findings associated with enteric
fever.
• The ulcer is parallel to the long axis of the gut and is usually situated in the distal ileum.
• A complication of enteric fever is perforation of the intestines, which is fatal. Perforation of a typhoid
ulcer characteristically occurs during the third week of the illness.
Option A: Transverse ulcers
• Transverse ulcers are a feature of inflammatory bowel disease.
• Crohn’s disease is characterized by trans-luminal inflammation, which results in skin lesions, fistulae,
transverse ulcers, and anal fissures.
Option C: Pinpoint ulcers
• Colon erosions are small, shallow sores or ulcers on the colon's lining.
• They're often surrounded by a ring of erythematous tissue with irregular shapes.

Page 25

703
• They are a feature of many colon diseases, like diverticulitis and inflammatory bowel disease and can
be postoperative.
Option D: Pseudopolyps
• They are due to severe inflammation in the bowel and result from repair due to a chronic inflammatory
process.
• They are commonly seen as a local complication of ulcerative colitis.

Solution for Question 18:


Option B: Duodenal
• A fistula is an abnormal connection between two epithelized surfaces. A fistula that is formed between
any part of the gut and the skin is called an enterocutaneous fistula.
• In general, the more proximal the fistula in the intestine, the more serious the problem, with greater
fluid and electrolyte loss.
• Proximal fistulas also have a higher output and greater loss of digestive capacity.
• Distal fistulas tend to have lower output, making them easier to manage and more likely to close
spontaneously.
• Hence Fistula leading to the highest electrolyte imbalance is the Duodenal fistula.
Incorrect Options:
Option A: Gastric
• A gastrocutaneous fistula represents a fistula connecting the stomach with the skin
• Gastrocutaneous fistulae are a seemingly rare occurrence, and literature in this respect is lacking.
• It is a post-operative complication and also occurs secondary to inflammatory conditions.
• It is not a common cause of high-output fistula.
Option C: Sigmoid
• Sigmoid fistulas are generally low-output fistulas and do not cause much electrolyte imbalance since
these fistulas are located distally.
• Moreover, Roux-en-y bypass surgery would not involve manipulation of the sigmoid colon.
Option D: Rectal
• Rectal fistulas are generally low-output fistulas and do not cause much electrolyte imbalance since
these fistulas are located distally.
• Moreover, Roux-en-y bypass surgery would not involve manipulation of the rectum.

Solution for Question 19:


Option A: Serial transverse enteroplasty procedure (STEP)

Page 26

704
• Diarrhea and steatorrhea after bowel resection for Crohn's are suggestive of short bowel syndrome.
The length of the small intestines is around 600cm. Short bowel syndrome is a malabsorptive condition
that arises secondary to removing significant small intestine segments (50% or more).
• Gallstones are formed due to the malabsorption of bile salts in the terminal ileum.
• Most patients are managed initially by total parenteral nutrition and anti-secretory and anti-motility
agents.
• STEP is an intestinal lengthening procedure that creates a longer and narrower intestine by making
cuts in the intestine and creating a zig-zag pattern increasing the surface area available for absorption.
Incorrect Options:
Option B: Delorme’s operation
• Delorme’s procedure is a modified perineal rectosigmoidectomy to treat complete rectal prolapse. It
has no role in the management of short bowel syndrome.
Option C: Abdominoperineal resection
• Abdominoperineal resection is a surgical intervention performed to treat anal or rectal carcinoma. It
removes the distal colon, rectum, and anal sphincter.
Option D: Ripstein operation
• It is one of the most common procedures for treating complete rectal prolapse. It uses a mesh to fix
the rectum to the pre-sacral fascia. It has no role in the treatment of short bowel syndrome.

Solution for Question 20:


Option A: Duplication of Alimentary tract
• These are very rare conditions, mostly located on the mesenteric border of the intestine.
• Typically, they are lined by the alimentary tract mucosa.
• They mostly share a common smooth muscle wall and blood supply with the adjacent bowel.
• The most common site for duplication is the ileum.
• Duplications can contain heterotopic gastric mucosa.
• It can be tubular or cystic.
• It may be associated with spinal or vertebral anomalies.
• Clinical presentation: recurrent abdominal pain, vomiting, constipation, abdominal distension, per
rectal bleeding, and abdominal mass.
• Radiological investigations: ultrasonography, plain abdominal X-ray, abdominal CT scan, and barium
studies can be helpful.
• Mostly present in infancy or early childhood with intestinal obstruction, bleeding, intussusception, or
perforation.
• Rarely present in adults with malignancy.
• Surgical management includes resection, anastomoses and multiple enterotomies with mucosal
stripping in the duplicated segment.
Incorrect Options:

Page 27

705
Option B: Meckel’s Diverticulum
• In most cases, Meckel's diverticulum is clinically present in children with painless gastrointestinal
bleeding.
• Its diagnosis can be confirmed with Technetium-99m scintigraphy, also known as Meckel's scan.
Option C: Mesenteric Cyst
• It occurs mostly in adults; one-third of cases occur in children younger than the age of 15 years.
• The most common presentation is a painless fluctuant abdominal swelling near the umbilicus.
• Other clinical features: recurrent abdominal pain, with or without vomiting.
Option D: Torsion of the omentum
• It is a rare emergency. It can be primary or secondary to adhesions of the omentum.
• Torsion of the omentum most frequently occurs in middle-aged obese men.
• A tender lump can be found on abdominal examination.

Solution for Question 21:


Option A: Pancreaticoduodenectomy
• The best surgical treatment option for adenocarcinoma at the ampulla of Vater is
pancreaticoduodenectomy.
• It is the best treatment because adenocarcinomas are usually multiple and sessile masses that tend
to recur if a conservation approach is used by going only for a duodenectomy. Recurrence is even
possible after this radical procedure.
• Surveillance is necessary even after a pancreaticoduodenectomy to diagnose the recurrence before it
has metastasized.
Incorrect Options:
Option B: Segmental intestinal resection with lymphadenectomy
• Localized small intestinal carcinoid tumorsare treated surgically with this approach.
• Lymph nodes are resected because of the high chances of metastasis.
Option C: Wide local resection
• It is the surgical procedure of choice in jejunal and ileal malignancies.
• Wide resection of the mesentery and associated lymph nodes are removed down to the superior
mesenteric artery.
Option D: Right hemicolectomy
• It is the surgical procedure of choice for distal ileal carcinomas.
• Lymph nodes along the ileocolic artery usually drain the distal ileal carcinomas.

Solution for Question 22:

Page 28

706
Correct Option B - Resection of Meckel's diverticulum with anastomosis:
• The patient has Meckel's diverticulum with an unhealthy base and signs of ulceration. In such cases,
the appropriate treatment is resection of the diverticulum and a part of the bowel, followed by
anastomosis.
• This approach is necessary to address the unhealthy base and prevent complications associated with
Meckel's diverticulum.
Treatment of Meckel’s diverticulum
• There are two situations: The patient has a Meckel’s diverticulum with healthy base (means, there is
no ulceration around). The patient has Meckel’s diverticulum with unhealthy base (There is ulceration
present).
• The patient has a Meckel’s diverticulum with healthy base (means, there is no ulceration around).
• The patient has Meckel’s diverticulum with unhealthy base (There is ulceration present).
• MD with healthy base: Diverticulectomy (Excision of only diverticulum) + Anastomosis.
• MD with unhealthy base: Resection (The diverticulum and a part of bowel are removed) +
Anastomosis.
• Incidentally detected Meckel’s diverticulum during elective surgery should be removed in all the
patients up to 80 years of age (as there is no added morbidity in performing diverticulectomy). If it is left
untreated, there is increased risk of complication in 12 % of the patients.
• If it is left untreated, there is increased risk of complication in 12 % of the patients.
• The patient has a Meckel’s diverticulum with healthy base (means, there is no ulceration around).
• The patient has Meckel’s diverticulum with unhealthy base (There is ulceration present).
• If it is left untreated, there is increased risk of complication in 12 % of the patients.
Incorrect Options:
Option A - Diverticulectomy alone: This option is unsuitable for a Meckel's diverticulum with an unhealt
hy base and ulceration. Diverticulectomy alone (excision of only the diverticulum) is reserved for cases
with a healthy base.
Option C - Observation without surgical intervention: Given the presence of an unhealthy base and sig
ns of ulceration, observation without surgical intervention is not appropriate. Meckel's diverticulum with
these features carries an increased risk of complications, making surgical resection necessary.
Option D - Symptomatic management with proton pump inhibitors: Symptomatic management with prot
on pump inhibitors may help in controlling gastric acid secretion but does not address the underlying is
sue of an unhealthy Meckel's diverticulum. Surgical resection is the definitive treatment in cases with a
n unhealthy base and ulceration.

Solution for Question 23:


Correct Option A - Laryngeal carcinoid:
• Laryngeal carcinoid is a localized carcinoid tumor, and it is specifically mentioned to have the best
prognosis among localized carcinoids.
• Localized carcinoid tumors are confined to the organ, and laryngeal carcinoid is associated with a
favorable outcome.

Page 29

707
Incorrect Options:
Option B - Carcinoid of the appendix: Carcinoid of the appendix is also a
localized carcinoid tumor and generally has a good prognosis. However, laryngeal carcinoid is specific
ally stated to have the best prognosis, making option B incorrect.
Option C. - Pancreatic carcinoid: Pancreatic carcinoid falls under non-localized carcinoid tumors, which
are associated with metastasis and worse prognosis. Laryngeal carcinoid, being localized, is expected
to have a better prognosis than pancreatic carcinoid, making option C incorrect.
Option D - Colon carcinoid: Similar to pancreatic carcinoid, colon carcinoid is a non-localized carcinoid
tumor, and it is not specified to have the best prognosis. Laryngeal carcinoid, being localized, is expect
ed to have a better prognosis than colon carcinoid, making option D incorrect.

Solution for Question 24:


Correct Option C - Tricuspid valve:
• Carcinoid heart disease primarily affects the right side of the heart.
• Whenever there is increased production of serotonin or 5-HTA, the valve that first opens is the
tricuspid valve.
• After that, the pulmonary valve opens.
• Catabolism of serotonin occurs in the pulmonary capillaries.
• Pathogenesis: The right side of the heart is mainly affected by pulmonary heart disease. In the right
heart, the valve that is affected first is the tricuspid valve, followed by the pulmonary valve.
Regurgitation is more common than stenosis. The most common valvular abnormality: Tricuspid
regurgitation > Pulmonary regurgitation > Tricuspid stenosis > Pulmonary stenosis.
• The right side of the heart is mainly affected by pulmonary heart disease.
• In the right heart, the valve that is affected first is the tricuspid valve, followed by the pulmonary valve.
• Regurgitation is more common than stenosis.
• The most common valvular abnormality: Tricuspid regurgitation > Pulmonary regurgitation > Tricuspid
stenosis > Pulmonary stenosis.
• The right side of the heart is mainly affected by pulmonary heart disease.
• In the right heart, the valve that is affected first is the tricuspid valve, followed by the pulmonary valve.
• Regurgitation is more common than stenosis.
• The most common valvular abnormality: Tricuspid regurgitation > Pulmonary regurgitation > Tricuspid
stenosis > Pulmonary stenosis.
Incorrect Options:
Option A - Aortic valve: The aortic valve is not the primary valve affected in Carcinoid heart disease. Th
e involvement is mainly on the right side of the heart, and the aortic valve is not in the direct pathway of
blood flow affected by serotonin or 5-HTA.
Option B - Mitral valve: Similarly, the mitral valve is not the primary valve affected in Carcinoid heart dis
ease. The left side of the heart is not primarily involved in this condition, and the mitral valve is not in th
e direct pathway affected by increased serotonin or 5-HTA production.

Page 30

708
Option D - Pulmonary valve: While the pulmonary valve is affected in Carcinoid heart disease, it is not t
he first valve to be impacted. The tricuspid valve is the initial valve affected, followed by the pulmonary
valve. Therefore, option D
is incorrect as it suggests that the pulmonary valve is the primary valve affected.

Solution for Question 25:


Correct Option C
- Carcinoid syndrome is commonly associated with small bowel carcinoid due to its midgut origin:
• Small bowel carcinoid is a type of midgut carcinoid, and carcinoid syndrome is commonly associated
with midgut carcinoids. Carcinoid syndrome results from the release of serotonin and other vasoactive
substances produced by the tumor, causing symptoms such as flushing, diarrhea, and abdominal pain.

Incorrect Options:
Option A - Small bowel carcinoid originates from the G cells located in the crypts of Lieberkühn:
Small bowel carcinoid arises from the Enterochromaffin-like (ECL) cells, not G
cells. ECL cells are neuroendocrine cells found in the gastrointestinal tract.
Option B - Small bowel carcinoid is not argentaffinic, unlike midgut carcinoids: Small bowel carcinoid is
argentaffinic, just like midgut carcinoids. Argentaffin staining is commonly positive in midgut carcinoids,
including those in the small bowel.
Option D - Carcinoid tumors are generally large in size initially: Carcinoid tumors are generally small ini
tially, not large. They often grow slowly, and even though they may cause symptoms, their size is typic
ally small.

Solution for Question 26:


Correct Option C - Ulcerative variety:
• The ulcerative variety of intestinal tuberculosis is characterized by the formation of transverse ulcers
in the ileum.
• This particular type is observed when the patient's immunity is decreased, and the virulence of the
tuberculosis-causing organism is high.
• The transverse ulcers in the ileum are a hallmark of the ulcerative variety, making option (C) the
correct answer.
Incorrect Options:
Option A - Sclerotic / Fibrotic variety: The sclerotic or fibrotic variety of tuberculosis is not associated wi
th the formation of transverse ulcers in the ileum. Instead, this type is characterized by fibrosis and sca
rring, often leading to stricture formation in chronic cases. Therefore, option (A) is incorrect.
Option B - Hyperplastic variety: The hyperplastic variety of tuberculosis is not specifically associated wi
th the formation of transverse ulcers in the ileum. Instead, it is characterized by a strong inflammatory r
esponse, mesenteric lymphadenopathy, caseous necrosis, and the formation of matted bowel loops. T

Page 31

709
herefore, option (B) is incorrect.
Option D - None of the above: Given that the ulcerative variety is indeed associated with the formation
of transverse ulcers in the ileum, option (D) "None of the above" is incorrect in this context. There is a
specific type of tuberculosis (ulcerative variety) mentioned in the information provided, making it a
valid choice among the given options.

Page 32

710
Large Intestine Part 1
1. What is the diagnosis of an elderly patient with aortic stenosis and lower gastrointestinal bleeding
due to the presence of small, flat cherry-red fern-like projecting vessels?
(or)
A 65-year-old man presented with an episode of syncope. He reported feeling dizzy during defecation
and seeing gross blood in the toilet for a month. Further evaluations are done based on examination,
and an aortic stenosis is visualized on echocardiography. Colonoscopy reveals an appearance of
small, flat cherry-red fern-like projecting vessels originating from a central artery. What is the
diagnosis?
A. Familial adenomatous polyposis
B. Heyde syndrome
C. Ulcerative colitis
D. Colorectal carcinoma
----------------------------------------
2. A 10-day-old newborn presented to the emergency department with abdominal distension,
constipation, and green-colored vomiting the previous day. He was suspected to be suffering from large
bowel obstruction Contrast enema revealed a distal, contracted, and proximal, dilated segment of the
large bowel with a transition zone. A rectal biopsy showed aganglionosis, hypertrophied nerve trunks,
and staining for acetylcholine esterase. What is the diagnosis?
(or)
What is the diagnosis in a newborn with rectal biopsy showing aganglionosis, hypertrophied nerve
trunks, and staining for acetylcholine esterase?
A. Ulcerative colitis
B. Heyde syndrome
C. Hirschsprung’s disease
D. Familial adenomatous polyps
----------------------------------------
3. What is the diagnosis in an adult male with rectal bleeding, multiple colon polyps, and a family history
colon carcinoma?
(or)
A 30-year-old male presented to the outpatient department with rectal bleeding occasionally for 1
month. On his rectal examination, there is no hemorrhoidal mass or growth. The colonoscopic
examination showed multiple polyps in the colon. On further probing of his history, the patient revealed
that his grandfather died of colon carcinoma. What is the diagnosis in this case?
A. Hamartomatous polyps
B. Ulcerative colitis
C. Familial adenomatous polyposis
D. Juvenile polyps
----------------------------------------

711
4. A 60-year-old male presents to the clinic with a change in bowel habits, occasional bleeding while
defecating, flatulence, urinary urgency, and mild abdominal pain with heaviness in the left lower
quadrant. He has an allergy to fibrous foods and does not use them routinely. His CT scan shows the
thickening and outpouching of the colonic wall, and his enema contrast shows a saw-tooth appearance
of the colon. What is the diagnosis?
(or)
A 60-year-old male has a thickened colonic wall with outpouching, and his enema contrast history
shows a saw-tooth appearance of the colon. What is the diagnosis?
A. Diverticulosis
B. Ulcerative colitis
C. Hamartomatous polys
D. Angiodysplasia
----------------------------------------
5. What operation is done to correct a condition in newborn whose rectal biopsy showed aganglionosis
and hypertrophied nerve trunks?
(or)
A newborn infant presented to emergency with complaints of abdominal distension and failure to pass
meconium in the first 24 hours. His diagnosis was confirmed by a rectal biopsy. The patient was
diagnosed with a distal large bowel. The contrast enema showed distal, contracted, and proximal,
dilated bowel segments. The rectal biopsy showed aganglionosis and hypertrophied nerve trunks. What
operation is done to correct this condition?
A. Delorme’s operation
B. Hartmann’s procedure
C. Duhamel’s operation
D. Koch’s ileostomy
----------------------------------------
6. A 70-year-old man presents to the emergency department with sudden onset left lower abdominal
pain associated with fever and leukocytosis. On examination, he has localized tenderness in the left
lower quadrant with guarding and rebound tenderness. His past medical history is significant for
hypertension and constipation. Which of the following is the most likely diagnosis for this patient?
(or)
Which condition leads to fever, acute abdominal pain with left lower quadrant tenderness, and
leukocytosis in an elderly patient with a history of constipation?
A. Heyde syndrome
B. Colorectal carcinoma
C. Ulcerative colitis
D. Colonic diverticulitis
----------------------------------------
7. What is the most likely diagnosis for a patient with rectal bleeding, weight loss, adenomatous colon
polyps, and a family history of ovarian cancer in the mother and gastric adenocarcinoma cancer in the
sister?

Page 2

712
(or)
A 56-year-old male presented to the clinic with complaints of bleeding per rectum for the last 15 days.
The patient had a history of 10 kg of weight loss and decreased appetite over the last six months.
Family history reveals ovarian cancer in the mother and gastric adenocarcinoma cancer in the sister. A
colonoscopy reveals an adenomatous polyp in the descending colon. A barium enema x-ray result is
pending. What is the most likely diagnosis?
A. Familial adenomatous polyposis
B. Lynch syndrome
C. Diverticulitis
D. Peutz-jeghers syndrome
----------------------------------------
8. A 49-year-old male presented to the clinic with hematochezia and abdominal pain for three months.
He reports a change in bowel habits, weight loss, and fatigue. The patient has a history of familial
adenomatous polyposis. On a complete blood count, hemoglobin is 11.5g/dL. Which of the following is
the best next step in managing this patient?
(or)
What is the best next step in managing a patient with a history of familial adenomatous polyposis
presenting with hematochezia, change in bowel habits, weight loss, and anemia?
A. Colonoscopy with biopsy
B. Computed tomography (CT) scan of the abdomen
C. X-ray of the abdomen
D. Carcinoembryogenic antigen
----------------------------------------
9. A 55-year-old textile worker presents with complaints of chronic constipation associated with per
rectal bleeding for three weeks. In addition, he has lost 6 kg in the last two months. He has a
20-pack-year smoking history, and his father died of cancer at the age of 70. He is afebrile and vitally
stable. A digital rectal examination reveals a friable mass that bleeds on the touch. Which of the
following is the gene mutation likely to be present in this case?
(or)
Which gene mutation is likely present in a patient with chronic constipation, rectal bleeding, and a
friable rectal mass that bleeds on the touch?
A. APC
B. EGFR
C. NF1 gene
D. BRCA 1
----------------------------------------
10. A 45-year-old male presents with complaints of vomiting, constipation, and colicky abdominal pain
associated with difficulty passing flatus for the last 2 hours. He had presented with bloody diarrhea 6
months ago, and an x-ray revealed a thumbprint sign suggestive of ischemic colitis. He is now treated
with broad-spectrum antibiotics. A colonoscopy reveals a stricture in the colon. What is the most
common site of post-ischemic stricture?

Page 3

713
(or)
What is the most common site of post-ischemic colonic strictures?
A. Ascending colon
B. Hepatic flexure
C. Splenic flexure
D. Descending colon
----------------------------------------
11. A 45-year-old woman presents due to diarrhea and abdominal cramping for the past three days.
She has been self-medicating with broad-spectrum antibiotics for allergic rhinitis for the past two years.
She has a temperature of 100 °F, a blood pressure of 130/80 mmHg, and a pulse rate of 100 beats per
minute. Laboratory investigations reveal an elevated erythrocyte sedimentation rate (ESR) and
leukocytosis. What is the drug of choice for this condition?
(or)
What is the drug of choice for a condition causing diarrhea, abdominal cramps, fever, tachycardia,
elevated ESR, and leukocytosis in patient who has been using broad-spectrum antibiotics for two
years?
A. Gentamicin
B. Metronidazole
C. Vancomycin
D. Linezolid
----------------------------------------
12. A 27-year-old male from India presents due to per rectal bleeding associated with pain on passing
stools for the past three weeks. He is very anxious as his father died of colorectal cancer. He is afebrile,
with a blood pressure of 120/80 mmHg, and a regular pulse rate of 78 beats per minute. His abdominal
is soft and non-tender. What is the most common cause of lower gastrointestinal bleeding in this
patient?
(or)
What is the most common cause of lower gastrointestinal bleeding in a young patient with pain on
passing stools and a family history of colorectal cancer in the father?
A. Benign tumour
B. Non-specific ulcer
C. Cancer rectosigmoid
D. Hemorrhoids
----------------------------------------
13. A 45-year-old female presents a gradually expanding painless abdominal swelling for six months.
She underwent a total abdominal colectomy for familial adenomatous polyposis 5 years ago.
Abdominal examination reveals a non-tender, regular, and well-defined mass. Abdominal CT image is
given belwo.The biopsy of this mass shows a benign lesion with no sign of malignancy. What will be the
appropriate management for this patient?
(or)

Page 4

714
What is the best next step in managing a patient who has undergone total abdominal colectomy for
familial adenomatous polyposis in the past and is currently presenting with a benign abdominal mass
that is shown in the CT scan image?

A. Observation
B. Enucleation
C. Wide local excision
D. A course of dacarbazine, or carboplatin
----------------------------------------

Correct Answers
Question Correct Answer

Question 1 2
Question 2 3
Question 3 3
Question 4 1
Question 5 3
Question 6 4
Question 7 2
Question 8 1
Question 9 1
Question 10 3
Question 11 3
Question 12 4
Question 13 3

Solution for Question 1:


Option B: Heyde syndrome

Page 5

715
• Heyde syndrome is a multisystemic disorder characterized by the triad of aortic stenosis (AS),
gastrointestinal bleeding, and acquired von Willebrand syndrome.
• The primary mechanism involves the degradation of von Willebrand factor (vWF) in the presence of
shear stress across the stenotic aortic valve, leading to gastrointestinal bleeding, often from
angiodysplastic lesions.
• These are abnormal, tortuous, dilated small blood vessels in the mucosal and submucosal layers of
the GI tract.
• Angiodysplasia on direct visualization from colonoscopy looks like 5 to 10 mm flat cherry-red fern-like
projecting vessels originating from a central artery.

Incorrect Options:
Option A: Familial adenomatous polyposis
• Symptoms include abdominal pain, diarrhea/constipation, weight loss, and lethargy.
• Colonoscopy of FAP shows >100 polyps in the intestinal mucosa.
Option C: Ulcerative colitis
• Patients present with severe diarrhea, weight loss, and nonspecific systemic symptoms.
• The typical colonoscopic findings in patients with UC include edematous mucosa, erythema, loss of
vascular markings, and mucosal friability.
Option D: Colorectal carcinoma
• The diagnosis of colorectal carcinoma involves various diagnostic modalities, including imaging
studies, colonoscopy, and biopsy.
• Colonoscopy shows a tumor with ill-defined edges and changes like increased friability and/or
superficial bleeding.

Solution for Question 2:

Page 6

716
Option C: Hirschsprung’s disease
• The presenting features of the baby correlate with Hirschsprung’s disease: abdominal distension,
constipation, and green-colored vomiting.
• These symptoms are due to functional obstruction because of muscular spasms of the distal colon
and internal anal sphincter.
• Diagnosis is made by contrast enema and rectal biopsy.
• The contrast enema shows distally contracted and proximally dilated areas with the transition zone in
between.
• Histopathologic examination of the rectal biopsy confirms the diagnosis of the disease by highlighting
the association of the absence of ganglion cells in the submucosal and myenteric plexus with
hypertrophy of nerve fibers in the aganglionic segment. At least two biopsies, each with a minimum
diameter of 3mm, should be required. The biopsy should contain as much mucosa as the submucosa.
• At least two biopsies, each with a minimum diameter of 3mm, should be required. The biopsy should
contain as much mucosa as the submucosa.
• Acetylcholinesterase (AChE) staining is an ancillary method for identifying the increased activity of
parasympathetic nerve fibers in the lamina propria and the muscularis mucosa. Thus, it helps make the
diagnosis, especially in difficult cases. The normally innervated intestine does not stain with AChE.
• Thus, it helps make the diagnosis, especially in difficult cases.
• The normally innervated intestine does not stain with AChE.
• At least two biopsies, each with a minimum diameter of 3mm, should be required. The biopsy should
contain as much mucosa as the submucosa.
• Thus, it helps make the diagnosis, especially in difficult cases.
• The normally innervated intestine does not stain with AChE.
Incorrect Options:
Option A: Ulcerative colitis
• It is an inflammatory bowel disease that peaks at ages 15 to 30 and then again at ages 50 to 70.
• Lead pipe colon appearance is seen on contrast enema.
Option B: Heyde syndrome
• It is most common in the > 60 years age group.
• Characterized by: Angiodysplasia Aortic stenosis Von Willebrand disease
• Angiodysplasia
• Aortic stenosis
• Von Willebrand disease
• Angiodysplasia
• Aortic stenosis
• Von Willebrand disease
Option D: Familial adenomatous polyps
• The colonoscopy shows hundreds to thousands of polyps.
• The patients will present with rectal bleeding.

Page 7

717
Solution for Question 3:
Option C: Familial adenomatous polyposis
• Familial adenomatous polyposis is an autosomal dominant neoplastic tumor due to an APC gene
mutation located on chromosome 5. It is associated with a 100% risk of developing colon cancer.
• This tumor of the large intestine genetically runs in the families, as the patient's grandfather had
colonic carcinoma, suggesting this disease.
• The colonoscopic examination shows multiple polyps, usually >100, as in the patient's colonoscopy.
• The treatment options are: Restorative proctocolectomy + Ileal pouch-anal anastomosis (J-shaped
pouch created) Total proctocolectomy with an end (Brooke) ileostomy Total abdominal colectomy with
ileorectal anastomosis
• Restorative proctocolectomy + Ileal pouch-anal anastomosis (J-shaped pouch created)
• Total proctocolectomy with an end (Brooke) ileostomy
• Total abdominal colectomy with ileorectal anastomosis
• Restorative proctocolectomy + Ileal pouch-anal anastomosis (J-shaped pouch created)
• Total proctocolectomy with an end (Brooke) ileostomy
• Total abdominal colectomy with ileorectal anastomosis
Incorrect Options:
Option A: Hamartomatous polyps
• Hamartomatous polyps (HPs) in the gastrointestinal (GI) tract are rare compared to other types of GI
polyps, yet they are the most common type of polyp in children.
• The symptoms are usually rectal bleeding, abdominal pain, obstipation, anemia, and/or small bowel
obstruction.
• HPs can be classified as juvenile polyps or Peutz-Jeghers polyps based on their histopathological
appearance.
• Family history is often present, especially in PJ polyps. However, these polyps are not as numerous
as FAP.
Option B: Ulcerative colitis
• The colonoscopic findings in ulcerative colitis shown here include edema, loss of vascularity, friable
mucosa, and ulcers.
• The presence of numerous polyps is not a feature of UC.
Option D: Juvenile polyps
• They are the non-neoplastic polyps that usually occur in the juvenile age group - most have polyps by
the age of 20 years.
• There is no family history of colon cancer in affected individuals.

Page 8

718
Solution for Question 4:
Option A: Diverticulosis
• The patient with rectal bleeding occasionally, altered bowel habits, and pain in the left lower quadrant
most likely has diverticulosis.
• The etiology is a low-fiber diet because a lack of fibers causes small-volume stools requiring high
intra-luminal pressure and tension for propulsion, resulting in muscular hypertrophy and pulsion
diverticula.
• Diverticular bleeding usually causes painless bleeding from the rectum. In approximately 50 percent
of cases, the person will see maroon or bright red blood with bowel movements.
• The pain and heaviness in the left lower quadrant are due to inflammation of the abnormal sac of
colons, usually of the sigmoid colon.
• The investigation of choice for diverticulosis is a barium enema, which shows a saw-tooth appearance
of the colon because of the outpouching of colonic mucosa.

Incorrect Options:
Option B: Ulcerative colitis
• The colonoscopic findings in ulcerative colitis shown here include edema, loss of vascularity, friable
mucosa, and ulcers.
• It is an inflammatory disease that is often associated with autoimmunity.
Option C: Hamartomatous polys
• Hamartomatous polyps (HPs) in the gastrointestinal (GI) tract are rare compared to other types of GI
polyps, yet they are the most common type of polyp in children.
• Polyps are visualized on colonoscopy, not outpouchings.
Option D: Angiodysplasia
• These are abnormal, tortuous, dilated small blood vessels in the mucosal and submucosal layers of
the GI tract.

Page 9

719
• Angiodysplasias characteristically appear as small (2 to 10 mm), flat, cherry-red lesions with a
fern-like pattern of arborizing, ectatic blood vessels radiating from a central vessel

Solution for Question 5:


Option C: Duhamel’s operation
• The clinical features of this patient correlate with Hirschsprung’s disease, which presents with
abdominal distension and failure to pass meconium in the first 24 hours.
• The rectal biopsy is the investigation of choice showing aganglionosis and hypertrophied nerve trunks
that are the same as the biopsy findings in this patient.
• Surgery is the ultimate treatment, and the aim is to remove the aganglionic segment and bring the
ganglionic part to the anus.
• Duhamel’s operation is one of the most commonly used operations. The characteristic of the modified
Duhamel operation is to resect the proximal dilated colon, retain part of the aganglionic rectum, and
divide the spur between the aganglionic rectum and ganglionic proximal colon. At the same time, the
anastomosis is low enough to prevent residual symptoms of megacolon.
• The characteristic of the modified Duhamel operation is to resect the proximal dilated colon, retain
part of the aganglionic rectum, and divide the spur between the aganglionic rectum and ganglionic
proximal colon.
• At the same time, the anastomosis is low enough to prevent residual symptoms of megacolon.
• The characteristic of the modified Duhamel operation is to resect the proximal dilated colon, retain
part of the aganglionic rectum, and divide the spur between the aganglionic rectum and ganglionic
proximal colon.
• At the same time, the anastomosis is low enough to prevent residual symptoms of megacolon.
Incorrect Options:
Option A: Delorme’s operation
• Delorme’s operation is performed to correct a rectal prolapse in adults.
• Rectal prolapse refers specifically to the prolapse of some or all of the rectal mucosa through the
external anal sphincter.
• In pediatric populations between infancy and age 4, rectal prolapse is usually a self-limiting condition,
responding to conservative management.
Option B: Hartmann’s procedure
• Hartmann's procedure is performed for colonic diverticula.
• Diverticular disease does not occur in neonates.
• In the colonic diverticula, the barium enema shows a saw-tooth appearance of the colon.
Option D: Koch’s ileostomy
• Koch’s ileostomy is not a standard procedure for ulcerative colitis.
• Ulcerative colitis can occur at any age but is extremely uncommon in neonates.

Page 10

720
Solution for Question 6:
Option D: Colonic diverticulitis
• Colonic diverticulitis is characterized by inflammation and infection of the diverticula, which are
outpouchings of the colonic mucosa.
• It commonly presents with sudden onset left lower abdominal pain, fever, leukocytosis, and localized
tenderness, particularly in the left lower quadrant.
• This patient's age, symptoms, and physical examination findings are consistent with colonic
diverticulitis, making it the most likely diagnosis.
• The radiological test of choice for acute diverticulitis is CT of the abdomen and pelvis, preferably with
water-soluble oral or rectal (if significant nausea and vomiting) contrast and intravenous contrast,
provided there are no contraindications.
• Endoscopy should be avoided in suspected acute diverticulitis due to an increased risk of perforation.
• It is recommended that a colonoscopy is performed approximately six to eight weeks after symptoms
have resolved to rule out malignancy, inflammatory bowel disease, or possibly colitis if the patient has
not had a recent colonoscopy.
Option A: Heyde syndrome
• It is the angiodysplasia of the colon associated with aortic stenosis and Von Willebrand disease.
• It is not associated with fever and leukocytosis,
Option B: Colorectal carcinoma
• The triggers of a diagnostic colonoscopy are blood per rectum, abdominal pain, and anemia.
• The tumor location on the clinical presentation can be separated on the left side, with more changes in
bowel habits and hematochezia, and on the right side, with obscured anemia impacting the late stage
at diagnosis.
Option C: Ulcerative colitis
• Inflammatory bowel disease is a more chronic condition presenting with severe diarrhea, weight loss
and malabsorption.

Solution for Question 7:


Option B: Lynch syndrome
• Lynch syndrome is the most common form of hereditary colorectal cancer, also known as hereditary
nonpolyposis colorectal cancer syndrome. The mechanism underlying this disease involves inherited
mutations in DNA mismatch repair genes, leading to microsatellite instability and inadequate
expression and function of their proteins.
• Lynch syndrome should be suspected in patients with synchronous or metachronous colorectal
cancer, CRC or endometrial cancer prior to 50 years of age, multiple Lynch-associated cancers (eg,
CRC and endometrial, ovarian, stomach, small intestine, or renal pelvis/ureter), and in cases of familial
clustering of Lynch-associated cancers.

Page 11

721
• It should also be suspected in any tumor found to have deficient mismatch repair (MMR) on
microsatellite instability (MSI) or immunohistochemistry (IHC) testing.
• Genetic testing should be performed in patients with a strong family history of colon cancer (e.g., >3
relatives involving multiple generations).
• Types of Lynch Syndrome: Lynch I: Colorectal cancer Lynch II: Colorectal cancer + Extraintestinal
malignancy (MC: Endometrium > Gastric > Ovarian)
• Lynch I: Colorectal cancer
• Lynch II: Colorectal cancer + Extraintestinal malignancy (MC: Endometrium > Gastric > Ovarian)
• Lynch I: Colorectal cancer
• Lynch II: Colorectal cancer + Extraintestinal malignancy (MC: Endometrium > Gastric > Ovarian)
Incorrect Options:
Option A: Familial adenomatous polyposis
• It involves adenomatous polyposis coli (APC) gene mutation on chromosome 5q22.
• It is characterized by 100s to 1000s of adenomatous polyps.
• Polyps often develop in the early teenage years and result in a nearly 100 percent lifetime risk of
colorectal cancer by age forty if untreated.
Option C: Diverticulitis
• It is an infection of the diverticula, which leads to the inflammation of the diverticula with wall
thickening.
• It presents as lower left quadrant (LLQ) pain, fever, and leukocytosis.
• Complications are colovesical fistula, abscess, perforation, and obstruction.
• There is no association with development of malignancies.
Option D: Peutz-Jeghers syndrome
• It is an autosomal dominant syndrome characterized by numerous hamartomas throughout the gut.
• It also forms hyperpigmented macules on lips, mouth, hands, and genitalia.
• It increases the risk of breast, colorectal, stomach, and pancreas cancers.

Solution for Question 8:


Option A: Colonoscopy with biopsy
• According to the clinical features, this is a case of colorectal cancer.
• Colorectal cancer presents with hematochezia, abdominal pain, weight loss, change in bowel habits,
and microcytic anemia.
• Risk factors: Smoking, high-fat diet, alcohol, familial adenomatous polyposis, HNPCC, adenomatous
polyps, acromegaly, ulcerative colitis, and Crohn's disease.
• Smoking, high-fat diet, alcohol, familial adenomatous polyposis, HNPCC, adenomatous polyps,
acromegaly, ulcerative colitis, and Crohn's disease.

Page 12

722
• Diagnostic tests for colorectal cancer: Colonoscopy with biopsy: It is the gold standard for the
diagnosis of colorectal cancer On barium enema x-ray, an Apple core lesion is seen.
• Colonoscopy with biopsy: It is the gold standard for the diagnosis of colorectal cancer
• On barium enema x-ray, an Apple core lesion is seen.
• CT colonography, FOBT, FIT, and flexible sigmoidoscopy are other diagnostic tests.
• Smoking, high-fat diet, alcohol, familial adenomatous polyposis, HNPCC, adenomatous polyps,
acromegaly, ulcerative colitis, and Crohn's disease.
• Colonoscopy with biopsy: It is the gold standard for the diagnosis of colorectal cancer
• On barium enema x-ray, an Apple core lesion is seen.
Incorrect options:
Option B: Computed tomography (CT) scan of the abdomen
• The gold standard test for colorectal cancer diagnosis is colonoscopy with biopsy.
• While colonoscopy with biopsy is the primary method for diagnosing colorectal cancer, a CT scan of
the abdomen may be used for staging the cancer, assessing the extent of spread, and identifying any
metastasis.
Option C: X-ray of the abdomen
• An abdominal X-ray is performed in case of an abdominal aortic aneurysm or pancreatitis.
• It is also performed in case of gut perforation, ureter injury, and intussusception.
• It is not used for the diagnosis of colorectal cancer.
Option D: Carcinoembryonic antigen
• The serum level of carcinoembryonic antigen (CEA) increases in colorectal cancer.
• It is used to monitor colorectal cancer recurrence.
• It should not be used for diagnosis due to poor sensitivity and specificity.

Solution for Question 9:


Option A: APC
• In the above-mentioned clinical scenario, all of these factors are indicative of colorectal cancer.
• The cardinal genomic alteration that has been found universally present in CRC is a mutation in the
adenomatous polyposis coli gene (APC).
• APC mutation causes the unrestricted action of the Wnt signaling pathway, which subsequently
enhances the intracellular accumulation of a protein called beta-catenin, which is responsible for cell
proliferation, differentiation, and enhanced survival of colorectal epithelial cells.
• Following is an illustration demonstrating the importance of APC in the pathogenesis of colorectal CA:

Page 13

723
Incorrect Options:
Option B: EGFR
• Epidermal growth factor receptor is a trans-membrane glycoprotein with an extracellular epidermal
growth factor binding domain and an intracellular tyrosine kinase domain that regulates signaling
pathways to control cellular proliferation.
• EGFR mutations are often implicated in non-small cell lung cancer.
Option C: NF1 gene
• It is present in neurofibromatosis 1.
• Its mutations cause abnormality in neurofibromin protein, leading to the manifestations of the disease.
Option D: BRCA 1
• BRCA 1 is a gene mutated in the case of breast cancer.
• Its mutations also manifest in ovarian cancer.

Solution for Question 10:


Option C: Splenic flexure
• In the above-mentioned clinical scenario, the patient complained of melena and abdominal pain
suggestive of Ischemic colitis and was treated. Now, he has developed a stricture due to ischemic
colitis.
• The most common sites of stricture in the colon are splenic flexure (Griffiths point) and rectosigmoid
junction (Sudek's point). These two areas are most prone to ischemia. These are also known as the
'watershed' areas, which mean the regions in the colon between 2 major arteries that supply the colon.
Splenic flexure is the area between SMA and IMA supplies, and the rectosigmoid junction is the region
between the IMA and the superior rectal artery supplies. These areas are mostly supplied by the
marginal artery.
• These two areas are most prone to ischemia.

Page 14

724
• These are also known as the 'watershed' areas, which mean the regions in the colon between 2 major
arteries that supply the colon.
• Splenic flexure is the area between SMA and IMA supplies, and the rectosigmoid junction is the
region between the IMA and the superior rectal artery supplies.
• These areas are mostly supplied by the marginal artery.
• The most common causes of ischemia are atherosclerosis, hypotension, and bowel obstruction.
• These two areas are most prone to ischemia.
• These are also known as the 'watershed' areas, which mean the regions in the colon between 2 major
arteries that supply the colon.
• Splenic flexure is the area between SMA and IMA supplies, and the rectosigmoid junction is the
region between the IMA and the superior rectal artery supplies.
• These areas are mostly supplied by the marginal artery.
Incorrect Options:
Option A: Ascending colon
• Crohn's disease is more common in ascending colon.
• Ascending colon has an adequate blood supply and is less likely to be involved in ischemia.
Option B: Hepatic flexure
• The hepatic flexure is the right upper part of the colon.
• It has a good blood supply and is less likely to be involved in ischemia.
Option D: Descending colon
• Ulcerative colitis is more common in descending colon.
• With a good blood supply, there is a minimal risk of ischemia in the descending colon.

Solution for Question 11:


Option C: Vancomycin
• The patient presented with diarrhea and abdominal pain in the clinical scenario. Another significant
finding is the association of his symptoms with antibiotics suggestive of Clostridium difficile-induced
pseudomembranous colitis.
• The judicious use of certain antibiotics, such as clindamycin and gentamicin, decreases the
population of other gut flora, leading to an increase in the Clostridium difficile population.
• The pathogenic changes are due to toxin A and toxin B.
• The drugs of choice to treat pseudomembranous enterocolitis are oral vancomycin and fidaxomicin
(not commonly used because of price).
• IV vancomycin is not recommended because only the oral form stays in the GIT and acts on the
bacteria, whereas the IV drug does not attain sufficient concentrations in the GIT.
• If the patient cannot tolerate oral or in complicated cases - metronidazole is indicated.

Page 15

725
Incorrect Options:
Option A: Gentamicin
• Gentamicin use itself causes pseudomembranous colitis.
• Its use for its management is counter-productive.
Option B: Metronidazole
• It is the 2nd line drug in the management of Clostridium difficile colitis.
• It is used only when oral vancomycin is unavailable or in complicated/refractory cases in combination
with vancomycin.
Option D: Linezolid
• Linezolid is yet under trial as an effective drug against Clostridium difficile.
• Its efficacy is not yet established.

Solution for Question 12:


Option D: Hemorrhoids
• Hemorrhoid disease is a common pathology that can yield symptoms ranging from minimal discomfort
or inconvenience to excruciating pain with defecation.
• Hemorrhoids are rich in vascular supply and have a tendency to engorge and prolapse. Symptoms
can vary from mild itching and bleeding to severe pain.
• They are rare under 20 years of age, and incidence peaks between the ages of 45 and 65 years of
age.
• Lower gastrointestinal bleeding is defined as bleeding from a site distal to the ligament of Treitz.
• The most common site of lower GI bleeding is the colon (95%).
• The most common cause of lower GI bleeding in India is hemorrhoids (rarely massive bleeding).
• Diverticular disease is the overall most common cause of lower GI bleed because of dietary
habits around the world- Indian food has good fiber and hence less incidence.
Incorrect Options:
Option A: Benign tumor
• A benign tumor is a mass that is least likely to metastasize.
• Benign tumors are rarely vascular.
Option B: Non-specific ulcer
• An ulcer is defined as the discontinuity in the surface epithelium over a mucosal surface.
• They neither bleed as much as a hemorrhoid nor are they more common than a hemorrhoid.
Option C: Cancer rectosigmoid
• CA rectosigmoid is much less common than hemorrhoids

Page 16

726
Solution for Question 13:
Option C: Wide local excision
• This patient with a history of familial adenomatous polyposis and a benign abdominal mass most likely
has a desmoid tumor that is visualized in the CT image as a well-circumscribed, relatively homogenous
mass.
• Desmoid tumors: Desmoid tumors are benign, non-inflammatory fibroblastic tumors with a tendency
for local invasion and recurrence but without metastasis Occur in 10-20% of patients with FAP.
Histologically, they are characterized by spindle-shaped cells. Management: Surgical excision with a
wide local margin is required to prevent a recurrence. Medical treatment with an antineoplastic agent
such as doxorubicin, dacarbazine, or carboplatin can produce remission, but the prognosis of advanced
desmoids is poor.
• Desmoid tumors are benign, non-inflammatory fibroblastic tumors with a tendency for local invasion
and recurrence but without metastasis
• Occur in 10-20% of patients with FAP.
• Histologically, they are characterized by spindle-shaped cells.
• Management: Surgical excision with a wide local margin is required to prevent a recurrence. Medical
treatment with an antineoplastic agent such as doxorubicin, dacarbazine, or carboplatin can produce
remission, but the prognosis of advanced desmoids is poor.
• Surgical excision with a wide local margin is required to prevent a recurrence.
• Medical treatment with an antineoplastic agent such as doxorubicin, dacarbazine, or carboplatin can
produce remission, but the prognosis of advanced desmoids is poor.
• Desmoid tumors are benign, non-inflammatory fibroblastic tumors with a tendency for local invasion
and recurrence but without metastasis
• Occur in 10-20% of patients with FAP.
• Histologically, they are characterized by spindle-shaped cells.
• Management: Surgical excision with a wide local margin is required to prevent a recurrence. Medical
treatment with an antineoplastic agent such as doxorubicin, dacarbazine, or carboplatin can produce
remission, but the prognosis of advanced desmoids is poor.
• Surgical excision with a wide local margin is required to prevent a recurrence.
• Medical treatment with an antineoplastic agent such as doxorubicin, dacarbazine, or carboplatin can
produce remission, but the prognosis of advanced desmoids is poor.
• Surgical excision with a wide local margin is required to prevent a recurrence.
• Medical treatment with an antineoplastic agent such as doxorubicin, dacarbazine, or carboplatin can
produce remission, but the prognosis of advanced desmoids is poor.
Incorrect Options:
Option A: Observation
• Observation means monitoring any growth without any intervention.
• It is suitable for very slow-growing tumours, not desmoid tumours.

Page 17

727
Option B: Enucleation
• It means excising the demised tissue.
• Desmoid tissues need to be excised widely.
Option D: A course of dacarbazine, or carboplatin
• Wide local excision is a better treatment option for a desmoid tumour than chemotherapy.
• Medical treatment with an antineoplastic agent such as doxorubicin, dacarbazine, or carboplatin can
produce remission, but the prognosis of advanced desmoids is poor.

Page 18

728
Large Intestine Part 2
1. What is the underlying cause for recent-onset bloody diarrhea in a patient with chronic diarrhea that
started after consuming unhygienic food and a biopsy of the caecum showing the following findings?
(or)
A 40-year-old male presents with bloody diarrhea for two days. The patient is otherwise healthy but
reports experiencing diarrhea for three months, apparently started after consuming street food and did
not resolve. There are no abnormalities on physical examination. A colonoscopy with biopsy is
performed, and a specimen from the caecum is shown in the image. What is the underlying cause?

A. Giardia lamblia
B. Entamoeba histolytica
C. Helicobacter pylori
D. Enterobius vermicularis
----------------------------------------
2. A 38-year-old female is brought to the emergency with complaints of fever, generalized abdominal
pain, and distension for five days. She mentioned that she had not passed stool and flatus for two days.
She has no history of any chronic illness. She is vitally stable. Abdominal examination reveals
generalized tenderness. An X-ray of the abdomen is shown in the image. What is the diagnosis?
(or)
What is the diagnosis for a 38-year-old woman with fever, generalized abdominal pain, and distension
whose abdomen X-ray is shown in the image?

A. Adhesive small bowel obstruction

729
B. Crohn’s disease
C. Obstructed hernia
D. Cecal volvulus
----------------------------------------
3. A newborn is evaluated for 2 episodes of projectile vomiting, which started 5 hours after the delivery.
Vomitus is green in color and half a cup in quantity. He was born at 35 weeks of gestation via caesarian
section due to placental insufficiency causing fetal distress. The abdomen is distended. An X-ray
abdomen image is shown below. What is the probable diagnosis?
(or)
Establish the diagnosis for a newborn who developed 2 episodes of projectile vomiting after birth with
the help of the X-ray image.

A. Duodenal atresia
B. Jejunal atresia
C. Midgut Malrotation
D. Hypertrophic pyloric stenosis
----------------------------------------
4. A newborn is brought due to episodes of projectile vomiting since yesterday. He has not passed
meconium since birth. The patient is further evaluated and diagnosed with intestinal atresia. The patient
is urgently posted for surgery and an intraoperative image of the pathology is given below. What is the
type of intestinal atresia?
(or)
What type of intestinal atresia is shown in the image?

Page 2

730
A. Type I
B. Type II
C. Type III
D. Type IV
----------------------------------------
5. A 55-year-old female presents with complaints of cramping abdominal pain associated with nausea,
vomiting, and diarrhea for 6 hours. The patient is on chemotherapy and radiotherapy for cervical
carcinoma. Upon abdominal examination, generalized abdominal tenderness with positive bowel
sounds is observed. Radiation enteritis is suspected. Which of the following statements regarding this
condition is correct?
(or)
Select the correct statement regarding radiation enteitis?
A. Endoscopy should always be done to find the extent of the injury
B. Late effects are thrombosis and vascular insufficiency, necrosis, perforation, or stricture formation
C. Radiotherapy should be immediately stopped in such cases until symptoms subside
D. General mucosal architecture is intact in chronic cases
----------------------------------------
6. A 50-year-old male presented to the clinic with constipation and fresh blood in his stools for 2
months. After a thprpigh evaluation, a colonoscopy was done, and it confirmed the diagnosis of
colorectal polyps. Which of the following statements best correlates with the diagnosis?
(or)
Which of the following statements best correlates with the diagnosis of colorectal polyps?
A. A hamartomatous polyp is the most common colorectal polyp
B. An adenomatous polyp is the most common neoplastic colorectal polyp
C. Cowden’s disease decreases the risk of GI malignancies
D. Cronkhite-Canada syndrome is not an association
----------------------------------------
7. A 60-year-old obese male presents with complaints of abdominal pain and alteration of bowel habits
for six months. Further evaluations are performed. Double-contrast barium enema was done, and the
finding is given below in the image. Which of the following statements is correct about this condition?
(or)
Which of the following statements is correct about a pathology presenting with alteration of bowel habits
in an elderly male whose double-contrast barium enema is shown in the image?

Page 3

731
A. The most common site of metastasis is the lungs
B. The preferred chemotherapy regimen is FOLFOX
C. Mucin production improves the prognosis since mucin blocks tumor extension
D. The investigation of choice for staging is colonoscopy with biopsy
----------------------------------------
8. According to the modified Astler-Collar classification for colorectal cancer, the findings of a tumor
penetrating the entire bowel wall with lymph node invasion, best correlates with which stage?
(or)
A 60-year-old male presents to the outpatient department with complaints of recurrent mild abdominal
pain and anal bleeding. He has a history of inflammatory bowel disease. A colonoscopy revealed an
ulceroproliferative lesion in the descending colon. CECT abdomen and pelvis revealed a tumor
penetrating the entire bowel wall with lymph node invasion. According to the modified Astler-Coller
classification for colorectal cancer, the above findings best correlate with which stage?
A. B1
B. B2
C. C1
D. C2
----------------------------------------
9. A 50-year-old male has been diagnosed with stage III colorectal carcinoma after coming to the clinic
with excessive fatigue, weight loss, altered bowel movements, and vomiting. The patient is planned for
elective surgery and adjuvant chemotherapy. Which of the following drugs can be used in the adjuvant
settings in this case?
(or)
Which of the following drugs can be used in the adjuvant settings for a patient with stage III colorectal
carcinoma?
A. Bevacizumab
B. Cetuximab
C. Irinotecan
D. Oxaliplatin
----------------------------------------

Page 4

732
10. A 40-year-old male is brought to the emergency department with a fever and watery diarrhea for
three days. The diagnosis was made by stool cytotoxin assay. On colonoscopy, ulcers, plaques, and
pseudomembranes were visualized. Which of the following signs is seen on the CECT of the patient in
this case?
(or)
A 40-year-old male has ulcers, plaques, and pseudomembranes visualised on colonoscopy. The
diagnosis is made by using stool cytotoxin assay. Which of the following signs is seen on the CECT?

A. Accordion sign
B. Central stellate scar
C. Whirl sign
D. Honeycombing
----------------------------------------
11. A 56-year-old male presents to the surgical department with complaints of painless, intermittent
bleeding during defecation for one month. The colonoscopy reveals a tumor in the ascending colon.
The patient was posted for an extended right colectomy. According to the image given below, which of
the following structures is removed in the procedure?
(or)
Which of the following structures is removed in extended right colectomy for colon cancer?

A. A, B, C, D, E & F
B. D, E, F, G & H
C. A, B, C, D, E, F, G
D. J & K

Page 5

733
----------------------------------------
12. A 34-year-old male presents due to abdominal pain and bleeding per rectum for one week. On
further evaluation, the patient was found to have multiple polyps in the intestine and was diagnosed
with Gardner syndrome. Which of the following is also found in this condition?
(or)
Which of the following abnormalities can also be found in a patient with Gardner syndrome?
A. Glioma
B. Hyperpigmented macules
C. Multiple osteomas
D. Hamartomatous polyps
----------------------------------------
13. Which of the following statements is correct regarding a condition that involves abnormal
positioning of the ligament of Treitz in a neonate?
(or)
A 3-week-old infant is brought to the emergency department by his mother with complaints of 2
episodes of bilious vomiting and inconsolable crying since last night. He is vitally stable, and a physical
examination revealed no abnormalities. Further investigations revealed abnormal positioning of the
ligament of Treitz. Which of the following statements regarding the patient’s condition is correct?
A. Mesenteric base is wide
B. Duodenum is posterior to the colon in reverse rotation
C. Ladd procedure corrects the condition
D. The gold standard test for diagnosis is an upper gastrointestinal contrast study
----------------------------------------
14. Which of the following statements regarding the anatomy and physiology of the large intestine is
incorrect?
A. The normal length of the large intestine is 150 cm
B. The widest part of the large intestine is the sigmoid
C. Griffith's point is located in the splenic flexure
D. The arc of Riolan connects the superior mesenteric artery with the inferior mesenteric artery.
----------------------------------------
15. A 58-year-old male presents to the emergency department with severe lower abdominal pain, fever,
and an elevated white blood cell count. The patient has a history of recurrent left lower quadrant pain
and constipation. On physical examination, there is localized tenderness and guarding in the left lower
abdomen. A contrast-enhanced CT scan reveals a collection of fluid in the pericolonic region. What is
the most appropriate Hinchey classification for this patient's condition?
(or)
What is the appropriate Hinchey classification for a patient with recurrent left lower quadrant pain and
localized tenderness with a pericolonic fluid collection on CT?
A. Stage I

Page 6

734
B. Stage II
C. Stage III
D. Stage IV
----------------------------------------
16. A 55-year-old male presents with abdominal pain and altered bowel habits. A colonoscopy reveals
the presence of a colorectal polyp. Histopathological examination identifies it as a neoplastic polyp.
Which of the following is the most likely subtype of neoplastic polyp in this patient?
A. Tubulovillous polyp
B. Villous polyp
C. Tubular polyp
D. Hyperplastic polyp
----------------------------------------
17. A 28-year-old male presents with a history of intermittent abdominal pain, rectal bleeding, and
diarrhea. On colonoscopy, multiple polyps are identified in the colon and rectum. Biopsy reveals
characteristic features of juvenile polyposis syndrome. The patient has a family history of colorectal
cancer. Which of the following statements regarding this sydnrome is true?
(or)
Which of the following statements regarding juvenile polyps is true?
A. Both juvenile polyposis syndrome and juvenile polyps have a similar risk of malignancy
B. The risk of malignancy is higher in juvenile polyps compared to juvenile polyposis syndrome
C. Juvenile polyposis syndrome has a higher risk of malignancy than juvenile polyps
D. Juvenile polyposis syndrome and juvenile polyps have no association with colorectal cancer.
----------------------------------------
18. A 45-year-old male presents with a history of intermittent abdominal pain and changes in bowel
habits. Colonoscopy reveals the presence of a polyp in the sigmoid colon. Biopsy confirms it to be an
adenomatous polyp. The patient has no family history of colorectal cancer. Which of the following
factors increases the risk of malignancy in adenomatous polyps?
(or)
Which histological characteristic is associated with an increased risk of malignancy in the adenomatous
polyp?
A. Ureterosigmoidostomy
B. Streptococcus bovis Bacteremia
C. Acromegaly
D. Villous histology
----------------------------------------
19. A 40-year-old male presents with a history of familial adenomatous polyposis (FAP) in his family.
The patient is concerned about the risk of developing FAP and wishes to undergo screening. Which of
the following is the most appropriate initial step in screening his family members?
A. Colonoscopy every 5 years

Page 7

735
B. APC gene testing
C. Total proctocolectomy
D. Genetic counseling without further testing
----------------------------------------
20. A 38-year-old female presents with multiple hamartomas in the skin, breast, and mucous
membranes. The most common feature of this condition is multiple trichilemmomas, and the affected
gene is PTEN on chromosome 10. Ectodermal and endodermal polyps are observed. Which condition
is most likely to be associated with the described clinical scenario?
(or)
Which of the following is associated with multiple hamartomas, trichilemmomas, and PTEN gene
involvement on chromosome 10?
A. Cronkhite-Canada Syndrome
B. Peutz-Jeghers Syndrome
C. Cowden's Disease
D. Hereditary Hemorrhagic Telangiectasia (HHT)
----------------------------------------
21. In the screening of Colorectal cancer, which investigation is recommended for individuals aged 45
years and above, who are asymptomatic?
A. FOBT (Fecal Occult Blood Test)
B. Flexible Sigmoidoscopy every 5 years
C. Double Contrast Barium Enema every 10 years
D. Colonoscopy every 10 years
----------------------------------------
22. A 60-year-old male presents to the emergency department with sudden-onset severe abdominal
pain and bloody diarrhea. The pain is localized to the left upper quadrant of the abdomen. On
examination, there is tenderness, and the patient has a fever. Laboratory investigations reveal
leukocytosis. What is the most likely diagnosis?
A. Acute Appendicitis
B. Ischemic Colitis
C. Infectious Colitis
D. Diverticulitis
----------------------------------------

Correct Answers
Question Correct Answer

Question 1 2
Question 2 4
Question 3 2

Page 8

736
Question 4 3
Question 5 2
Question 6 2
Question 7 2
Question 8 4
Question 9 4
Question 10 1
Question 11 3
Question 12 3
Question 13 4
Question 14 2
Question 15 2
Question 16 3
Question 17 3
Question 18 4
Question 19 2
Question 20 3
Question 21 4
Question 22 2

Solution for Question 1:


Option B: Entamoeba histolytica
• The colonoscopy with biopsy in this otherwise healthy patient with chronic diarrhea shows multiple
flask-shaped ulcers in the caecum, which is suggestive of amoebiasis, caused by entamoeba
histolytica.
• It may be asymptomatic or present with abdominal pain, diarrhea, or dysentery.
• Severe complications may include inflammation and perforation, resulting in peritonitis.
• Pathology: It usually reveals discrete ulcers, mucosal thickening, and edematous mucosa. Sometimes
flask-shaped ulcers may be seen in the submucosal layers.
• It usually reveals discrete ulcers, mucosal thickening, and edematous mucosa.
• Sometimes flask-shaped ulcers may be seen in the submucosal layers.
• Amebiasis can be diagnosed by demonstrating the organism using direct microscopy of stools or
rectal swabs.
• Antigen detection using an ELISA or PCR technique is often done.
• A colonoscopy is done to obtain scrapings of the mucosal surface. It is appropriate when the stool
studies are negative for amebiasis.

Page 9

737
• The primary therapy for symptomatic amebiasis requires hydration and the use of metronidazole
and/or tinidazole.
• It usually reveals discrete ulcers, mucosal thickening, and edematous mucosa.
• Sometimes flask-shaped ulcers may be seen in the submucosal layers.
Incorrect Options:
Option A: Giardia lamblia
• Giardia lamblia causes giardiasis.
• Clinical presentation ranges from asymptomatic shedding of giardia cysts to symptomatic giardiasis
(i.e., malabsorption and foul-smelling chronic diarrhea).
• Pathology: Villous blunting and expansion Increased intraepithelial lymphocytes Increased
inflammatory cells and prominent lymphoid aggregates in the lamina propria Organisms are
pear-shaped, with 2 nuclei, mostly pale eosinophilic to translucent in crescent-shaped trophozoites
• Villous blunting and expansion
• Increased intraepithelial lymphocytes
• Increased inflammatory cells and prominent lymphoid aggregates in the lamina propria
• Organisms are pear-shaped, with 2 nuclei, mostly pale eosinophilic to translucent in crescent-shaped
trophozoites
• Villous blunting and expansion
• Increased intraepithelial lymphocytes
• Increased inflammatory cells and prominent lymphoid aggregates in the lamina propria
• Organisms are pear-shaped, with 2 nuclei, mostly pale eosinophilic to translucent in crescent-shaped
trophozoites
Option C: Helicobacter pylori
• Helicobacter pylori gastritis is the most frequent and treatable form of gastritis.
• Most patients are asymptomatic or have mild self-limited dyspeptic symptoms, but some present with
abdominal pain with or without peptic ulcer disease.
• Pathology: Chronic antral gastritis: infiltration of lamina propria by plasma cells, lymphocytes, and a
small number of eosinophils seen as a superficial band of inflammation Active chronic antral gastritis:
when the above is associated with neutrophils; this should prompt a search for organisms Lymphoid
follicles in antral mucosa are common but nonspecific H. pylori organisms are Gram-negative, helical or
spiral-shaped, and flagellate
• Chronic antral gastritis: infiltration of lamina propria by plasma cells, lymphocytes, and a small number
of eosinophils seen as a superficial band of inflammation
• Active chronic antral gastritis: when the above is associated with neutrophils; this should prompt a
search for organisms
• Lymphoid follicles in antral mucosa are common but nonspecific
• H. pylori organisms are Gram-negative, helical or spiral-shaped, and flagellate
• Chronic antral gastritis: infiltration of lamina propria by plasma cells, lymphocytes, and a small number
of eosinophils seen as a superficial band of inflammation

Page 10

738
• Active chronic antral gastritis: when the above is associated with neutrophils; this should prompt a
search for organisms
• Lymphoid follicles in antral mucosa are common but nonspecific
• H. pylori organisms are Gram-negative, helical or spiral-shaped, and flagellate
Option D: Enterobius vermicularis
• Enterobius vermicularis is a pinworm.
• The most common symptom associated with pinworm infestation is perianal itching. Perianal
erythema may be seen due to itching and scratching.
• Enterobius can be diagnosed through a cellophane tape test or pinworm paddle test where
an adhesive tape-like material is applied to the perianal area and then examined under a microscope.
• The examination might reveal characteristic ova which are 50 by 30 microns in size and have a
flattened surface on one side or may reveal the worms.

Solution for Question 2:


Option D: Cecal volvulus
• The x-ray in this patient with acute abdomen shows a dilation of bowel extending from the right lower
quadrant moving upwards to the left upper quadrant of the abdomen.
• Cecal volvulus describes torsion of the caecum around its mesentery which often results in
obstruction. If unrecognized, it can result in bowel perforation and fecal peritonitis.
• Patients presents with clinical features of proximal large bowel obstruction. This is usually with colicky
abdominal pain, vomiting, and abdominal distension.
• The colonic haustral pattern is generally maintained in contradistinction to a sigmoid volvulus.
• The cecal volvulus may have one air-fluid level compared to the sigmoid volvulus, which has several.
• When shorter segments of the colon and cecum are involved, the distended cecum may be found in
the normal location. In most patients, obstruction is almost complete, and the distal colon is usually
empty, and the small bowel is frequently distended.
• Management: Colonoscopic decompression may be appropriate if the patient is unfit for surgery.
However, laparotomy is normally required. Where there is colonic ischemia, a right hemicolectomy is
performed; in some cases, primary anastomosis is not possible, and stoma formation at both ends is
the safest option.
• Colonoscopic decompression may be appropriate if the patient is unfit for surgery.
• However, laparotomy is normally required.
• Where there is colonic ischemia, a right hemicolectomy is performed; in some cases, primary
anastomosis is not possible, and stoma formation at both ends is the safest option.
• Colonoscopic decompression may be appropriate if the patient is unfit for surgery.
• However, laparotomy is normally required.
• Where there is colonic ischemia, a right hemicolectomy is performed; in some cases, primary
anastomosis is not possible, and stoma formation at both ends is the safest option.

Page 11

739
Incorrect Options:
Option A: Adhesive small bowel obstruction
• Adhesive small bowel obstruction is a type of small intestine obstruction that occurs due to an
adhesion caused by previous abdominal surgery.
• X-ray findings: dilated loops of small bowel proximal to the obstruction predominantly central dilated
loops three instances of dilatation > 2.5 - 3 cm valvulae conniventes are visible gas-fluid levels if the
study is erect
• dilated loops of small bowel proximal to the obstruction
• predominantly central dilated loops
• three instances of dilatation > 2.5 - 3 cm
• valvulae conniventes are visible
• gas-fluid levels if the study is erect
• dilated loops of small bowel proximal to the obstruction
• predominantly central dilated loops
• three instances of dilatation > 2.5 - 3 cm
• valvulae conniventes are visible
• gas-fluid levels if the study is erect
Option B: Crohn’s disease
• The terminal ileum and proximal colon are most often affected.
• CT is commonly the first imaging assessment of those patients in the setting of an acute abdomen.
mural hyperenhancement fat halo sign: submucosal fat deposition bowel wall thickening (1-2 cm),
which is most frequently seen in the terminal ileum comb sign: engorgement of the vasa recta
perienteric fat stranding
• mural hyperenhancement
• fat halo sign: submucosal fat deposition
• bowel wall thickening (1-2 cm), which is most frequently seen in the terminal ileum
• comb sign: engorgement of the vasa recta
• perienteric fat stranding
• mural hyperenhancement
• fat halo sign: submucosal fat deposition
• bowel wall thickening (1-2 cm), which is most frequently seen in the terminal ileum
• comb sign: engorgement of the vasa recta
• perienteric fat stranding
Option C: Obstructed hernia
• An obstructive hernia is a mechanical bowel obstruction.
• X-ray shows dilated loops of the affected bowel.

Page 12

740
Solution for Question 3:
Option B: Jejunal atresia
• This is a case of jejunal atresia, a type of congenital visceral malformation caused by any vascular
accident in the uterus. It results in ischemia, atresia, and obstruction of the small bowel.
• Risk factors for developing intestinal atresia are not well established.
• Babies present with signs and symptoms of intestinal obstruction, such as abdominal distension,
emesis, and, in some cases, delayed passage of meconium. Normal-appearing meconium may be
seen, but most often, light-colored plugs are passed from the rectum.
• Radiographic evaluation with a plain abdominal X-ray using swallowed air as a contrast is a useful
diagnostic tool. The X-ray image shows a triple-bubble sign (dilated loops of the small intestine due to
distention of the stomach, duodenum, and jejunum)
• The X-ray image shows a triple-bubble sign (dilated loops of the small intestine due to distention of
the stomach, duodenum, and jejunum)
• Surgical correction should be performed urgently. The most common technique is resection of the
proximal dilated and atretic bowel with primary end-to-end anastomosis with or without tapering
enteroplasty of the proximal bowel.
• The most common technique is resection of the proximal dilated and atretic bowel with primary
end-to-end anastomosis with or without tapering enteroplasty of the proximal bowel.
• The X-ray image shows a triple-bubble sign (dilated loops of the small intestine due to distention of
the stomach, duodenum, and jejunum)
• The most common technique is resection of the proximal dilated and atretic bowel with primary
end-to-end anastomosis with or without tapering enteroplasty of the proximal bowel.
Incorrect Options:
Option A: Duodenal atresia
• Duodenal atresia is the complete/incomplete obstruction of the duodenal lumen with similar symptoms
during the clinical presentation.
• However, on X-ray a double-bubble sign is usually seen and is usually diagnostic.
Option C: Midgut Malrotation
• Presentation is very similar, but malrotation can be differentiated based on a radiographic examination
of the abdomen.
• X-ray: The GI tract might be distended with/without radiological signs of pneumatosis intestinalis. It
has limited use for diagnosing intestinal obstruction.
Option D: Hypertrophic pyloric stenosis
• In Hypertrophic pyloric stenosis, the symptoms usually develop after 2nd week of age. Moreover, the
vomiting is usually non-bilious.
• Barium contrast studies show a single-bubble appearance along with string and beak signs.

Solution for Question 4:


Option C: Type III

Page 13

741
• Grosfeld classification systems have described four types of intestinal atresia.
• The intraoperative image shows type IIIb atresia with apple peel deformity which describes proximal
jejunal atresia and a short ileal segment coiled around the ileocolic artery.
• The types of atresia hugely impact treatments and prognosis. Bowel loss is more common in type IIIb
and type IV atresia.
• Type IIIb (Apple peel) atresia is the least common atresia.
• Type IIIb atresias are more likely associated with volvulus and increased risk of distal bowel vascular
compromise.
• Ileal atresias are rare compared to jejunal atresia, and as the atresia becomes more distal, the less
the frequency of occurrence.
Option A: Type I
• In type I ileal atresia, there is a mucosal atresia with intact muscularis.
• Therefore, it is described as an internal membrane with serosa continuity and no mesenteric defect
Option B: Type II
• Type II involves a proximal and distal blind pouch connected by a fibrous cord with serosal
discontinuity.
Option D: Type IV
• In type IV atresia, type I, II, or III atresia can be present simultaneously with any number of
combinations
• A characteristic "string of sausages" appearance is seen.

Solution for Question 5:


Option B: Late effects are thrombosis and vascular insufficiency, necrosis, perforation, or stricture form
ation
• This is the case of acute radiation enteritis caused by direct radiation effects on the mucosa of the
bowel, which results in injury to the intestinal mucosa, ulceration, and necrosis.
• Predisposing factors in the above scenario include advancing age and hypertension.
• Acute effects of radiation enteritis are abdominal pain, generalized abdominal tenderness, diarrhea,
nausea, and vomiting, and are usually self-limiting.
• Chronic effects include progressive occlusive vasculitis, which leads to chronic ischemia, and fibrosis,
which affects all layers of the intestinal wall rather than the mucosa alone. The terminal ileum is most
frequently affected in such cases and becomes evident 2 years after radiotherapy.
Incorrect Options:
Option A: Endoscopy should always be done to find the extent of the injury
• Endoscopy should usually be avoided in acute cases of radiation-related enteritis as it can increase
the risk of perforation.
• In endoscopy, thin, friable mucosa is seen with multiple submucosal telangiectasias in chronic
radiation enteropathy.

Page 14

742
Option C: Radiotherapy should be immediately stopped in such cases until symptoms subside
• Radiotherapy should not be stopped in cases of acute radiation enteritis.
• However, the delivery of radiotherapy should be modified to decrease injury to the bowel.
Option D: General mucosal architecture is intact in chronic case
• Chronic radiation enteritis is due to obliterative arteritis that leads to intestinal ischemia, which can
result in stricture, ulceration, fibrosis, and occasionally fistula formation.
• Therefore, mucosa can also be affected.

Solution for Question 6:


Option B: An adenomatous polyp is the most common neoplastic colorectal polyp
• This is the case of colorectal polyps, which are slow-growing abnormal overgrowths of the mucosa of
the colon that can become malignant in a small number of cases.
• Histologically, adenomatous polyps are the most common neoplastic polyps, while hyperplastic polyps
are the most common non-neoplastic polyps.
• Adenomatous polyps have high malignant potential, with its villous subtype being the most malignant.
These polyps usually remain asymptomatic until colon cancer starts developing.
• The risk of progression to malignancy depends upon the size and histology of adenomatous polyps.
• Surveillance colonoscopy at 3 years after the baseline is a necessary measure in patients with
high-risk adenomas.
Incorrect Options:
Option A: Hamartomatous polyp is the most common colorectal polyp
• Hyperplastic polyps are the most common polyps.
• The most common syndromes associated with hamartomatous polyps include Peutz–Jeghers
syndromes (PJS), Cowden syndrome (CS), and juvenile polyposis syndrome (JPS)
Option C: Cowden’s disease decreases the risk of GI malignancies
• Cowden’s disease is the most common PTEN hamartoma syndrome. It is usually asymptomatic but
increases the risk of colorectal, breast, endometrial, and thyroid carcinoma.
• It does not affect the incidence of GI malignancies as it causes PTEN gene mutation, which is usually
associated with pelvis-related malignancies.
Option D: Cronkhite-Canada syndrome is not an association
• It is a rare form of non-hereditary, often immune-mediated hamartomatous polyposis that causes
inflammatory changes within the polyps resulting in GI hemorrhage and sepsis.

Solution for Question 7:


Option B: The preferred chemotherapy regimen Is FOLFOX-IV

Page 15

743
• This patient with abdominal pain and altered bowel habits with an apple core lesion (well-defined
circumferential constriction of the lumen) on barium enema is suggestive of carcinoma of the colon.
• Surgical management is the first step in patients with resectable tumors, followed by chemotherapy.
• FOLFOX intravenous (5-fluorouracil, leucovorin, and oxaliplatin) is the preferred chemotherapy
regimen for treating colonic carcinoma.
Incorrect Options:
Option A: The most common site of metastasis is the lungs
• The liver is the most common site for the metastasis of colonic carcinoma and should be suspected in
patients with suspicious clinical features such as altered bowel habits, rectal bleeding, weight loss,
and/or iron deficiency anemia in an elderly patient.
• If liver metastasis is suspected, consider MRI or CT angiography of the patient.
Option C: Mucin production improves the prognosis since mucin blocks tumor extension
• Alteration in mucin expression, a major part of mucus, has usually been linked with a worse prognosis
in benign and malignant pathologies.
• Patients with high MUC1 expression have a higher risk of metastasis.
Option D: The investigation of choice for staging is colonoscopy with biopsy
• CT abdomen/pelvis, MRI, and PET scanning are usually appropriate for staging and assessment of
the extent of colonic carcinoma.
• Colonoscopy with biopsy is usually the investigation of choice for confirming the diagnosis of colonic
carcinoma.

Solution for Question 8:


Option D: C2
• This is the case of colorectal carcinoma (CRC) based on a history of abdominal pain, anal bleeding,
and past inflammatory bowel disease.
• A colonoscopy reveals an ulceroproliferative lesion in the descending colon.
• Staging of CRC can be done in various ways, one of which is the Astler-Coller modification of the
Duke's classification.
• According to this classification, the above CECT finding can be best classified as the C2 stage.

Page 16

744
Incorrect Options:
Option A: B1
• CRC partially penetrating the muscularis propria, a muscle layer adjacent to the submucosa, is
classified as stage B1.
• There is no lymph node invasion at this stage.
Option B: B2
• In the B2 stage, carcinoma has fully penetrated the mucosa, submucosa, and muscularis propria into
the serosa.
• There is still no lymph node invasion at this stage.
Option C: C1
• In stage C1, the tumor has fully invaded the bowel wall.
• Lymph node invasion has started in this stage with the involvement of 1-4 nodes.

Solution for Question 9:


Option D: Oxaliplatin
• A history of unexplained iron deficiency anemia is also suggestive of colorectal carcinoma in this case.
• Curative treatment for colorectal carcinoma includes radiation therapy, adjuvant chemotherapy,
biological therapy, and surgery in some cases.
• FOLFOX-IV is the most appropriate and commonly used chemotherapeutic treatment regimen for
metastatic carcinoma. It includes folinic acid, 5-fluorouracil, and oxaliplatin.
• Oxaliplatin-based chemotherapy is now considered the standard adjuvant therapy for stage III and
high-risk stage II colon cancer.
Incorrect Options:

Page 17

745
Option A: Bevacizumab
• It is a monoclonal antibody that blocks vascular endothelial growth factors (VEGF), thus resulting in
the inhibition of angiogenesis. It can be used as a biological therapy for the treatment of colorectal
carcinoma.
• It is indicated as a second-line treatment of metastatic carcinoma in combination with
fluoropyrimidine-based chemotherapy.
Option B: Cetuximab
• It is a monoclonal antibody indicated for treating EGFR-expressing KRAS mutation-negative
colorectal malignancy.
• It can be only used as a second-line treatment in combination with other chemotherapy regimens.
However, it can be used as a single agent in those patients who are intolerant of irinotecan or
oxaliplatin.
Option C: Irinotecan
• It binds to the topoisomerase I-DNA complex and inhibits DNA replication.
• It is used as a standard therapeutic agent for metastatic cancer. However, due to its worse overall
survival rate and higher incidence of associated side effects, oxaliplatin is preferred.

Solution for Question 10:


Option A: Accordion sign
• This is a case of pseudomembranous colitis in which the patient presents with complaints of fever and
watery diarrhea for three days.
• Pseudomembranous colitis is caused by clostridiums difficile, a gram-positive anaerobic bacillus.
• Diagnosis is made after the detection of one or both toxins (Toxin A and B) by: Gold standard
investigation: Stool cytotoxin assay. Colonoscopy: Characteristic ulcers, plaques, and pseudo
membranes. On CECT: Accordion sign is seen.
• Gold standard investigation: Stool cytotoxin assay.
• Colonoscopy: Characteristic ulcers, plaques, and pseudo membranes.
• On CECT: Accordion sign is seen.
• The "accordion sign" is highly suggestive of pseudomembranous colitis; it consists of mucosal edema
and inflammation involving the large bowel and is seen when orally administered contrast material
becomes trapped between thickened haustral folds, giving the appearance of alternating bands.
• Gold standard investigation: Stool cytotoxin assay.
• Colonoscopy: Characteristic ulcers, plaques, and pseudo membranes.
• On CECT: Accordion sign is seen.

Page 18

746
Option B: Central stellate scar
• The central stellate scar is a centrally placed, well-defined, low-density area that is visible on the CT
abdomen in cases of renal oncocytoma.
Option C: Whirl sign
• The whirl sign is a whorled appearance formed by twisting of the intestine, which is usually visible on
the CT abdomen.
• It is usually seen in cases of intestinal volvulus.
Option D: Honeycombing
• Honeycombing is a pathological finding of the lung parenchyma commonly visible in the CT scan of
the lungs.
• It is composed of diffused cysts of varying sizes.
• It is commonly seen in cases of pulmonary fibrosis.

Solution for Question 11:


Option C: A, B, C, D, E, F, G
• This patient with colon adenocarcinoma in the ascending colon required extended right colectomy.
• In extended right colectomy following structures are removed Ileocolic, right colic vessels, and middle
colic vessels at their base. The right colon until the distal transverse colon is resected.
• Ileocolic, right colic vessels, and middle colic vessels at their base.
• The right colon until the distal transverse colon is resected.
• Ileocolic, right colic vessels, and middle colic vessels at their base.
• The right colon until the distal transverse colon is resected.

Page 19

747
Incorrect Options:
Options A, B, and D:
These options do not reflect the correct of locations involved in extended right colectomy.

Solution for Question 12:


Option C: Multiple osteomas
• Gardner syndrome is a phenotypic variant of familial adenomatous polyposis. It is an autosomal
dominant disease characterized by numerous adenomatous polyps lining the intestinal mucosal
surface, which has a high potential for malignancy.
• The extracolonic manifestations include osteomas of the mandible and skull intestinal polyposis
desmoids epidermoid cysts fibromatosis supernumerary teeth (extra teeth)
• osteomas of the mandible and skull
• intestinal polyposis
• desmoids
• epidermoid cysts
• fibromatosis
• supernumerary teeth (extra teeth)
• A characteristic finding is bilateral, multiple, pigmented, ocular fundus lesions, known as congenital
hypertrophy of the retinal epithelium.
• The development of intestinal polyposis and colorectal adenocarcinoma are key features of Gardner
syndrome.
• osteomas of the mandible and skull

Page 20

748
• intestinal polyposis
• desmoids
• epidermoid cysts
• fibromatosis
• supernumerary teeth (extra teeth)
Incorrect Options:
Option A: Glioma
• Gliomas are more commonly associated with Turcot syndrome.
• Turcot syndrome is the association of primary brain tumors with colorectal cancer.
Option B: Hyperpigmented macules
• Hyperpigmented macules on the mouth, lips, hands, and genitalia are seen in patients with
Peutz-Jeghers syndrome.
• It is an autosomal dominant condition featuring numerous hamartomas through the GI tract.
Option D: Hamartomatous polyps
• A hamartomatous polyp is a growth of normal colonic tissue with distorted architecture.
• These polyps are associated with Peutz-Jeghers syndrome and juvenile polyposis.

Solution for Question 13:


Option D: The gold standard test for diagnosis is an upper gastrointestinal contrast study
• Midgut malrotation is a developmental rotational anomaly of the embryonic bowel. Malrotation can
present either acutely, intermittently, or asymptotically.
• Bilious emesis in a child younger than one year should prompt immediate suspicion and workup for an
obstructive lesion, including malrotation with volvulus.
• Over hours, hemodynamic instability (from hypovolemia or shock), peritonitis (indicating volvulus with
inflammation), and hematochezia (indicating bowel ischemia with possible necrosis due to volvulus)
can result.
• The gold standard test for diagnosis of malrotation is an upper gastrointestinal contrast study.
• However, upper GI series should only be obtained in hemodynamically stable patients, such as this
neonate.
• Findings of malrotation include ligament of Trietz on the right side of the abdomen instead of the left, a
corkscrew duodenum, a duodenal obstruction with a beak-like appearance if a volvulus is present.
• ligament of Trietz on the right side of the abdomen instead of the left,
• a corkscrew duodenum,
• a duodenal obstruction with a beak-like appearance if a volvulus is present.
• ligament of Trietz on the right side of the abdomen instead of the left,
• a corkscrew duodenum,

Page 21

749
• a duodenal obstruction with a beak-like appearance if a volvulus is present.

Incorrect Options:
Option A: Mesenteric base is wide
• The anatomic features that lead to bowel obstruction and volvulus result from a narrow mesenteric
base that allows for mobility of the small bowel.
• The small bowel twists around the superior mesenteric artery and leads to vascular compromise of the
small bowel.
Option B: Duodenum is posterior to the colon in reverse rotation
• In reverse rotation, part of the rotation occurs in a clockwise direction around the superior mesenteric
artery.
• The duodenum assumes an anterior position, and the colon lies posterior to the duodenum and the
superior mesenteric artery.
Option C: Ladd procedure corrects the condition
• The Ladd procedure is performed by reducing the volvulus (if present) by rotating the bowel in a
counterclockwise direction, dividing the mesenteric (Ladd) bands, placing the small bowel on the right
abdomen and the large bowel on the left of the abdomen, and performing an appendectomy.
• Surgery does not correct malrotation but rather widens the mesenteric pedicle to prevent future
volvulus.

Solution for Question 14:


Correct Option B – The widest part of the large intestine is the sigmoid:
• The correct statement is that the widest part of the large intestine is the cecum, not the sigmoid. The
cecum is the pouch-like structure that marks the beginning of the large intestine, and it is wider than the
sigmoid colon, which is a relatively narrower part located towards the end of the large intestine.

Page 22

750
Incorrect Options:
Option A - The normal length of the large intestine is 150 cm: The normal length of the large intestine is
indeed 150 cm (1.5 m), as stated. This information reflects the average length of the large intestine in
adults.
Option C - Griffith's point is located in the splenic flexure: This statement is correct. Griffith's point is ind
eed located in the splenic flexure, which is the bend between the transverse colon and the descending
colon. This point is particularly vulnerable to ischemia.
Option D. - The arc of Riolan connects the superior mesenteric artery with the inferior mesenteric arter
y: This statement is correct. The arc of Riolan, also known as the marginal artery of Drummond, does c
onnect the superior mesenteric artery with the inferior mesenteric artery. It serves as a collateral branc
h connecting the proximal part of the middle colic artery to the left colic artery, providing an anastomoti
c link between the two major arteries supplying the large intestine.

Solution for Question 15:


Correct Option B - Stage II:
• In this clinical scenario, the patient's presentation of severe lower abdominal pain, fever, and elevated
white blood cell count, along with the findings on a contrast-enhanced CT scan revealing a collection of
fluid in the pericolonic region, align with Hinchey Stage II.
• Stage II is characterized by a large abscess extending into the pelvis, forming a walled-off pelvic
abscess.
• The localized tenderness and guarding in the left lower abdomen are consistent with an abscess that
is contained within the pelvis.

Incorrect Options:
Option A - Stage I: Stage I is characterized by small or confined pericolic or mesenteric abscess. This
option is not suitable for the presented case, as the CT scan revealed a
larger abscess in the pericolonic region.

Page 23

751
Option C - Stage III: Stage III involves generalized purulent peritonitis with liquid discharge. This is not t
he most appropriate classification for this patient, as the scenario describes a
localized abscess rather than generalized peritonitis.
Option D - Stage IV: Stage IV is associated with generalized fecal peritonitis with fecal discharge. The
presented case does not align with Stage IV, as there is no mention of fecal discharge, and the CT find
ings indicate a contained fluid collection rather than a diffuse spread.

Solution for Question 16:


Correct Option C - Tubular polyp:
• Tubular polyps are the most common subtype of neoplastic (adenomatous) colorectal polyps.
• Neoplastic polyps are characterized by dysplastic changes and are considered precursors to
colorectal cancer.
• Tubular polyps are typically composed of glandular structures and are associated with a lower risk of
malignancy compared to villous polyps.
• In this clinical scenario, the patient with abdominal pain and altered bowel habits is found to have a
neoplastic polyp, making the most likely subtype a tubular polyp.
Incorrect Options:
Option A - Tubulovillous polyp:
• Tubulovillous polyps are a subtype of neoplastic colorectal polyps that have both tubular and villous
features.
• While tubulovillous polyps are less common than tubular polyps, they are associated with an
intermediate risk of malignancy.
Option B - Villous polyp:
• Villous polyps are the most malignant subtype of neoplastic colorectal polyps.
• Although associated with a higher risk of malignancy, they are less common overall compared to
tubular polyps.
Option D - Hyperplastic polyp:
• Hyperplastic polyps are the most common type of colorectal polyps overall but are non-neoplastic.
• They are not associated with dysplastic changes and are generally considered benign.
• In this case, where the histopathological examination identifies the polyp as neoplastic, a hyperplastic
polyp is less likely.

Solution for Question 17:


Correct Option C - Juvenile polyposis syndrome has a higher risk of malignancy than juvenile polyps:
• However, despite both juvenile polyps and juvenile polyposis syndrome having a lower risk of
malignancy compared to other types of polyps, the risk of malignancy is higher in juvenile polyposis
syndrome.

Page 24

752
• This is because juvenile polyposis syndrome involves the presence of multiple juvenile polyps,
increasing the likelihood of malignant transformation.
Incorrect Options:
Option A - Both juvenile polyposis syndrome and juvenile polyps have a similar risk of malignancy: This
option is incorrect because, as mentioned, juvenile polyposis syndrome has a higher risk of malignanc
y compared to juvenile polyps due to the presence of multiple polyps in the former.
Option B - The risk of malignancy is higher in juvenile polyps compared to juvenile polyposis syndrome
: This option is incorrect as it presents the opposite scenario. The correct relationship is that juvenile po
lyposis syndrome has a higher risk of malignancy compared to juvenile polyps.
Option D - Juvenile polyposis syndrome and juvenile polyps have no association with colorectal cancer
: This option is incorrect as both juvenile polyposis syndrome and juvenile polyps are associated with a
n increased risk of colorectal cancer. The comparison of the risks between the two conditions is crucial
for understanding their malignant potential.

Solution for Question 18:


Correct Option D - Villous Histology:
• Adenomatous polyps are precursors to colorectal cancer, and the risk of malignancy is influenced by
various factors.
• Among these, the histological characteristics play a crucial role. Villous histology refers to the
presence of finger-like projections in the polyp, and this histological subtype is associated with an
increased risk of malignancy.
• Therefore, in this scenario, the presence of villous histology in the adenomatous polyp in the sigmoid
colon is the factor that significantly raises the risk of malignancy.
Incorrect Options:
Option A - Ureterosigmoidostomy: While ureterosigmoidostomy is associated with an increased risk of
adenomatous polyps, it is not directly linked to an elevated risk of malignancy. It is a condition that may
predispose individuals to the development of adenomatous polyps, but it is not the specific factor influ
encing the risk of cancer in this context.
Option B - Streptococcus bovis Bacteremia: Streptococcus bovis bacteremia has been linked to an incr
eased risk of adenomatous polyps, particularly in the colon. However, like ureterosigmoidostomy, it is n
ot a direct factor influencing the risk of malignancy in adenomatous polyps. It is a
risk factor for the development of polyps but does not specifically indicate an elevated risk of cancer.
Option C - Acromegaly: Acromegaly is another condition mentioned in the provided information as a
risk factor for adenomatous polyps. However, similar to options A and B, acromegaly does not directly
correlate with an increased risk of malignancy. It is associated with the development of polyps, but it is
not the primary factor influencing the risk of cancer in adenomatous polyps.

Solution for Question 19:


Correct Option B - APC gene testing:

Page 25

753
• Familial Adenomatous Polyposis (FAP) is an autosomal dominant condition caused by mutations in
the APC (adenomatous polyposis coli) gene. Since FAP has a strong genetic component, screening
family members is crucial.
• APC gene testing is the most appropriate initial step for screening individuals at risk for FAP.
Identifying the pathogenic mutation in the APC gene allows for precise determination of familial risk and
enables targeted surveillance.
• If the mutation is positive, appropriate surveillance measures, such as regular colonoscopy, can be
initiated to detect and manage polyps at an early stage. This approach ensures a more personalized
and effective screening strategy for family members at risk.
Incorrect Options:
Option A - Colonoscopy every 5 years: This option suggests a surveillance approach without identifyin
g the specific genetic mutation first. While colonoscopy is an important tool for surveillance in FAP, it is
not the most appropriate initial step. Performing colonoscopy without genetic testing may lead to unnec
essary procedures for individuals who do not carry the pathogenic mutation.
Option C - Total proctocolectomy: Total proctocolectomy is a surgical intervention and is not the initial s
tep in screening for FAP. It is typically recommended when polyps are detected during surveillance and
other conservative measures are inadequate. This option is overly aggressive and not indicated for ro
utine screening without evidence of polyps.
Option D - Genetic counseling without further testing: While genetic counseling is an important compon
ent of managing familial conditions, it is insufficient as the sole step in screening for FAP. Genetic coun
seling should be complemented by genetic testing, such as APC gene testing, to identify the specific m
utation and guide appropriate surveillance and management strategies. Choosing genetic counseling a
lone may delay the identification of at-risk individuals and appropriate interventions.

Solution for Question 20:


Correct Option C - Cowden's Disease:
• Cowden's disease, also known as multiple hamartoma syndrome, is the most likely diagnosis. The
key features supporting this choice include:

Incorrect Options:
Option A - Cronkhite-Canada Syndrome:
• Skin Pigmentary Changes: Cronkhite-Canada syndrome is associated with skin pigmentary changes,
hair loss, and onychodystrophy, which are not features of Cowden's disease.
• Diarrhea and Fluid Loss: The most common symptom in Cronkhite-Canada syndrome is diarrhea,
leading to massive fluid loss and dehydration. This is not a characteristic feature of Cowden's disease.
• Colorectal Malignancy: Unlike Cowden's disease, Cronkhite-Canada syndrome is not associated with
an increased risk of colorectal malignancy.
Option B - Peutz-Jeghers Syndrome:
• Hamartomatous Polyps: Peutz-Jeghers syndrome is characterized by hamartomatous polyps in the
gastrointestinal tract, which is not a feature of Cowden's disease.
• Genetic Mutation: The genetic mutation in Peutz-Jeghers syndrome involves the STK11 gene.

Page 26

754
• Increased Risk of Malignancies: While Peutz-Jeghers syndrome is associated with an increased risk
of various cancers, the specific pattern involving breast, uterus, and thyroid is not typical.
Option D - Hereditary Hemorrhagic Telangiectasia (HHT):
• Vascular Abnormalities: Hereditary Hemorrhagic Telangiectasia primarily involves vascular
abnormalities, including telangiectasias and arteriovenous malformations. This is different from the
hamartomas seen in Cowden's disease.
• Genetic Mutation: HHT is associated with mutations in genes such as ENG and ACVRL1.
• Malignancy Risk: Unlike Cowden's disease, HHT is not known for an increased risk of breast, uterus,
or thyroid malignancies.

Solution for Question 21:


Correct Option D - Colonoscopy every 10 years:
• Colonoscopy is the recommended investigation for individuals aged 45 years and above who are
asymptomatic in the screening of Colorectal cancer.
• It should be performed every 10 years. Colonoscopy is considered the most preferred and effective
method among the options listed for detecting colorectal abnormalities, including polyps and cancers.
• It allows for both visualization and removal of precancerous lesions during the same procedure,
making it a comprehensive screening tool.
Incorrect Options:
Option A - FOBT (Fecal Occult Blood Test): This option suggests performing FOBT annually after 40 y
ears of age, which is incorrect. While FOBT is a screening option, the recommended age for starting sc
reening is 50 years, and it should be performed annually.
Option B - Flexible Sigmoidoscopy every 5
years: This option suggests performing flexible sigmoidoscopy every 5
years, which is incorrect. While flexible sigmoidoscopy is a
screening option, the recommended frequency is every 10 years, not every 5 years.
Option C - Double Contrast Barium Enema every 10 years: This option suggests performing double-co
ntrast barium enema every 10 years, which is incorrect. Although it is mentioned as a screening option,
colonoscopy is the most preferred investigation among the four. Double-contrast barium enema is con
sidered an alternative only if colonoscopy is contraindicated in certain patients. Therefore, it is not the p
rimary recommendation for individuals aged 50 years and above who are asymptomatic.

Solution for Question 22:


Correct Option B - Ischemic Colitis:
• Ischemic colitis is characterized by colitis caused due to ischemia.
• The most common site of ischemic colitis is the splenic flexure of the colon.
• Clinical features include sudden-onset severe abdominal pain, bloody diarrhea, tenderness, fever,
and leukocytosis.

Page 27

755
• Diagnosis can be confirmed through imaging studies such as barium enema or plain X-ray, showing
the characteristic "thumb-print sign." Colonoscopy may reveal dark hemorrhagic mucosa due to
ischemia.
• Colonoscopy may reveal dark hemorrhagic mucosa due to ischemia.
• Management involves bowel rest and the administration of broad-spectrum antibiotics.
• Stricture, commonly occurring in the sigmoid colon, is a major complication of ischemic colitis.
• Colonoscopy may reveal dark hemorrhagic mucosa due to ischemia.

Incorrect Options:
Option A - Acute Appendicitis:
• Acute appendicitis typically presents with right lower quadrant abdominal pain, not left upper quadrant
pain.
• The description of sudden-onset severe abdominal pain in the left upper quadrant is not characteristic
of acute appendicitis.
Option C - Infectious Colitis:
• While infectious colitis can present with abdominal pain and diarrhea, the sudden-onset severe
abdominal pain localized to the left upper quadrant, along with bloody diarrhea, is more indicative of
ischemic colitis.
• Infectious colitis may not necessarily involve the left upper quadrant specifically.
Option D - Diverticulitis:
• Diverticulitis usually presents with left lower quadrant pain, not left upper quadrant pain.
• The patient's symptoms, including the location of pain and tenderness, are not typical for diverticulitis.
• Ischemic colitis is a more likely diagnosis based on the presented clinical scenario.

Page 28

756
Inflammatory Bowel Disease
1. Which of the following is true about the disease that presents with chronic diarrhea, recurrent
aphthous ulcers, and multiple superficial ulcers involving the entire colon?
A. First-line therapy is Aminosalicylates
B. Infliximab was not the first anti-TNF α antibody used in the treatment of ulcerative colitis
C. Steroid-dependent cases need surgery
D. Surgery is palliative in ulcerative colitis
----------------------------------------
2. A 25-year-old male presents with complaints of abdominal pain and diarrhea with rectal bleeding for
1.5 months. On colonoscopy, hyperemic granular mucosa with polyps is seen, which is continuous from
the rectum to the caecum. Which of the following is true about the given condition?
A. Pre-malignant condition
B. Cobblestone mucosa is characteristic
C. Pseudo polyps are only seen in this condition
D. Rarely affects the rectum
----------------------------------------
3. A 35-year-old female presents with complaints of intermittent and colicky lower abdominal pains and
weight loss over the previous three months. Further evaluation with colonoscopy revealed multiple
aphthous ulcers with interspersed normal colon areas in between. A biopsy and a histological
examination show granulomas in all layers. Where can the given condition be seen?
(or)
A 35-year-old female has aphthous ulcers with interspersed normal colon areas in between. A biopsy
and a histological examination show granulomas in all layers. Where can the given condition be seen?
A. Jejunum only
B. Colon only
C. Terminal ileum and right side
D. Mouth to anus
----------------------------------------
4. A 37-year-old male presents with complaints of abdominal pain and diarrhea over the past three
months. The patient has a 10-pack-year smoking history. His barium studies show a string sign of
Kantor. This sign is associated with which disease?
(or)
In which condition string sign of Kantor seen?
A. Crohn’s disease
B. Ulcerative colitis
C. Hirschsprung’s disease
D. None of the above
----------------------------------------

757
5. A 31-year-old male presents with complaints of lower abdominal pain from the last 3 months. On
presentation, the patient is lethargic and afebrile Upon further evaluation, colonoscopy reveals focal
aphthous ulcerations with polyploid mucosal changes and adjacent areas of normal mucosa. Which of
the following is associated with this disease?
(or)
Which of the following is associated with the disease that presents with blood in stools with focal
ulcerations?
A. NOD-2/CARD-15 gene
B. p53 suppressor gene
C. Philadelphia chromosome
D. BRCA-1 gene
----------------------------------------
6. What is the primary action of sulphasalazine when used in the treatment of ulcerative colitis?
A. Folic acid synthesis
B. Formation of prostaglandins (PG)
C. Inhibition of NF-KB Activation
D. Formation of interleukins
----------------------------------------
7. What will be the definitive treatment for the remission of acute exacerbation of ulcerative colitis?
A. Sulfasalazine
B. Prednisolone
C. Aminosalicylic acid
D. NSAIDs
----------------------------------------
8. A 24-year-old male presents with complaints of lower abdominal pain and bleeding per rectum from
the last 20 days. On further evaluation, the patient has multiple ulcers involving the mucosa and
submucosa of the rectum extending upwards to involve the entire colon. What is the risk of the patient
developing cancer with this clinical condition?
(or)
What is the likelihood of developing cancer in a patient with extensive ulceration affecting the rectum
and entire colon?
A. 20% after 20 years
B. 35% after 30 years
C. 30% after 30 years
D. 50% after 30 years
----------------------------------------
9. A patient presents with fever and acute abdominal pain who has a history of chronic ulcerative colitis,
and an erect abdominal x-ray is presented below. What is the likely diagnosis?

Page 2

758
A. Pseudomembranous colitis
B. Adenocarcinoma colon
C. Pneumatosis intestinalis
D. Toxic megacolon
----------------------------------------
10. Which of the following distinguishes ulcerative colitis from Crohn's disease histologically in this
case?
A. Crypt abscess
B. Diffuse distribution of pseudo polyps
C. Mucosal edema
D. Lymphoid aggregates in the mucosa
----------------------------------------

Correct Answers
Question Correct Answer

Question 1 1
Question 2 1
Question 3 4
Question 4 1
Question 5 1
Question 6 3
Question 7 2
Question 8 2
Question 9 4
Question 10 1

Solution for Question 1:

Page 3

759
Option A: First-line therapy is Aminosalicylates
• Based on the symptoms of chronic diarrhea, abdominal pain, and recurrent aphthous ulcers, and
colonoscopic findings of multiple superficial ulcers in the rectum, the most likely diagnosis is ulcerative
colitis.
• The first line of therapy for ulcerative colitis is Aminosalicylates, which involve sulfasalazine, 5-acetyl
salicylic acid (5-ASA), and related compounds. These are the medications used to induce and maintain
remission in patients with mild to moderately active ulcerative colitis.
• They act by inhibiting cyclooxygenase and 5-lipoxygenase in the gut mucosa and have a minor role in
inhibiting cytokines. At the same time, PAF, TNF α, and nuclear activation factors play a major
role. 5-ASAs downregulate PPAR-gamma, induce PPAR-gamma gene expression, suppress NFκB
and TLR activation, inhibit proinflammatory cytokines (IL-1, TNFα, IL-2, IL-8), cyclooxygenase,
lipoxygenase enzymes, prevent formation of proinflammatory prostaglandins and leukotrienes, and
exhibit antioxidant activity, contributing to their effectiveness in treating IBD.
• These drugs require direct contact with affected mucosa for efficacy.
• Tacrolimus and Cyclosporine induce clinical remission in patients with moderate to severe ulcerative
colitis who are unresponsive to steroids and hence used as second-line therapy.
Option B: Infliximab was not the first anti-TNF α antibody used in the treatment of ulcerative colitis
• The first Food and Drug Administration (FDA) approved anti-TNFα antibody used to treat ulcerative
colitis was infliximab.
• Adalimumab is used in patients who are intolerant to infliximab.
Option C: Steroid-dependent cases need surgery
• The steroid-dependent cases of ulcerative colitis do not need surgery.
• Corticosteroids are considered the most effective therapy in inducing remission in patients who do not
respond to 5 amino-salicylic acids.
Option D: Surgery is palliative in ulcerative colitis
• The surgical treatment is not entirely palliative.
• In ulcerative colitis the surgical treatment is colectomy
• Chronic ulcerative colitis is a diffuse inflammatory disease of the colon and rectal mucosa. Complete
removal of the colon and rectum ensures complete healing.

Solution for Question 2:


ANSWER
Option A: Pre-malignant condition
• Based on the clinical features, the most likely diagnosis is ulcerative colitis.
• Hyperemic granular mucosa is a premalignant stage and can become cancerous over time
• Before adenocarcinoma develops, the intestinal epithelium progresses through a premalignant phase
of dysplasia.

Page 4

760
• Surveillance colonoscopy with biopsies looking for dysplasia is now standard practice in managing the
cancer problem in ulcerative colitis. Colonoscopy should be done and follow up ( at diagnosis and 8-10
years after)
Other options
Option B: Cobblestone mucosa is characteristic
• Cobblestone mucosa is not seen in ulcerative colitis, but is the hallmark of Crohn's disease
• Inflammation in Crohn's disease often leads to the formation of ulcers. Sometimes these ulcers
appear close to each other in the intestine and can look like cobblestones.
• Deep cuts with longitudinal edges create a cobblestone-like appearance.
Option C: Pseudo polyps are only seen in this condition
• Pseudo polyps are also seen in Crohn’s but are less common in Crohn's disease than in ulcerative
colitis.
• Ulcerative colitis (UC) polyps are indicators of severe inflammation and previous episodes of mucosal
ulceration.
• Polyps project clusters of scar tissue that develop into granulation tissue during the healing phase of
recurrent ulcers (especially inflammatory bowel disease).
Option D: Rarely affects the rectum
• Crohn's disease has rectal sparing
• The involvement of the rectum is the hallmark of ulcerative colitis.
• In ulcerative colitis, inflammation can start in the rectum and spread to the colon.
• Rectal lesions are characteristic of ulcerative colitis.
• Symptoms of the disease are damage to the rectum (proctitis).

Solution for Question 3:


Option D: Mouth to anus
• The given condition is Crohn's disease. It can occur anywhere from the mouth to the anus.
• Most common site involved ileum > ileocecal valve > cecum
• In Crohn's disease, the involvement of Small intestine and Large intestine is seen in 55% of cases
Only small intestine in 30% of cases Only large intestine in 15% of cases
• Small intestine and Large intestine is seen in 55% of cases
• Only small intestine in 30% of cases
• Only large intestine in 15% of cases
• Rectal sparing, cobblestone appearance, skip lesions, Kantor sign, and extensive fat wrapping are
important features.
• Small intestine and Large intestine is seen in 55% of cases
• Only small intestine in 30% of cases

Page 5

761
• Only large intestine in 15% of cases
Crohn’s disease:
• Chronic transmural inflammatory disease of the gastrointestinal tract.
• Although it can infect any organ, including the mouth and the anus
• The most typical small bowel primary surgical condition.
• Clinical features: Symptomatic periods of abdominal pain and diarrhea interspersed with an
asymptomatic period. The most common symptom is intermittent and colicky abdominal pain, most
common in the lower abdomen. Vienna can classify Crohn's disease based on age at onset, behavior,
and site of origin.
• Symptomatic periods of abdominal pain and diarrhea interspersed with an asymptomatic period.
• The most common symptom is intermittent and colicky abdominal pain, most common in the lower
abdomen.
• Vienna can classify Crohn's disease based on age at onset, behavior, and site of origin.
• Pathology: The disease process is discontinuous and segmental. Rectal sparing - characteristic of
Crohn’s Perirectal and perianal involvement occurs in one-third of patients. Areas of diseased bowel
separated by areas of grossly appearing normal bowel called skip lesions are seen. Extensive fat
wrapping caused by the circumferential growth of the mesenteric fat around the bowel wall known as
creeping fat. The bowel wall becomes increasingly thickened, firm, rubbery, and almost incompressible.
The uninvolved proximal bowel is dilated due to obstruction of the diseased segment. Earliest gross
pathologic lesion - superficial aphthous ulcer in the mucosa. Ulcers are characteristically linear and
merge to produce transverse sinuses with islands of normal mucosa in between/
• The disease process is discontinuous and segmental.
• Rectal sparing - characteristic of Crohn’s
• Perirectal and perianal involvement occurs in one-third of patients.
• Areas of diseased bowel separated by areas of grossly appearing normal bowel called skip lesions
are seen.
• Extensive fat wrapping caused by the circumferential growth of the mesenteric fat around the bowel
wall known as creeping fat.
• The bowel wall becomes increasingly thickened, firm, rubbery, and almost incompressible. The
uninvolved proximal bowel is dilated due to obstruction of the diseased segment.
• Earliest gross pathologic lesion - superficial aphthous ulcer in the mucosa.
• Ulcers are characteristically linear and merge to produce transverse sinuses with islands of normal
mucosa in between/
• Diagnosis: Barium contrast studies: Cobblestone appearance of the mucosa composed of linear
ulcers, transverse sinuses, and notches. Kantor sign - long lengths of the narrowed terminal ileum.
• Barium contrast studies: Cobblestone appearance of the mucosa composed of linear ulcers,
transverse sinuses, and notches. Kantor sign - long lengths of the narrowed terminal ileum.
• Cobblestone appearance of the mucosa composed of linear ulcers, transverse sinuses, and notches.
• Kantor sign - long lengths of the narrowed terminal ileum.
• Symptomatic periods of abdominal pain and diarrhea interspersed with an asymptomatic period.

Page 6

762
• The most common symptom is intermittent and colicky abdominal pain, most common in the lower
abdomen.
• Vienna can classify Crohn's disease based on age at onset, behavior, and site of origin.
• The disease process is discontinuous and segmental.
• Rectal sparing - characteristic of Crohn’s
• Perirectal and perianal involvement occurs in one-third of patients.
• Areas of diseased bowel separated by areas of grossly appearing normal bowel called skip lesions
are seen.
• Extensive fat wrapping caused by the circumferential growth of the mesenteric fat around the bowel
wall known as creeping fat.
• The bowel wall becomes increasingly thickened, firm, rubbery, and almost incompressible. The
uninvolved proximal bowel is dilated due to obstruction of the diseased segment.
• Earliest gross pathologic lesion - superficial aphthous ulcer in the mucosa.
• Ulcers are characteristically linear and merge to produce transverse sinuses with islands of normal
mucosa in between/
• Barium contrast studies: Cobblestone appearance of the mucosa composed of linear ulcers,
transverse sinuses, and notches. Kantor sign - long lengths of the narrowed terminal ileum.
• Cobblestone appearance of the mucosa composed of linear ulcers, transverse sinuses, and notches.
• Kantor sign - long lengths of the narrowed terminal ileum.
• Cobblestone appearance of the mucosa composed of linear ulcers, transverse sinuses, and notches.
• Kantor sign - long lengths of the narrowed terminal ileum.
Other options
Options A, B and C are incorrect.

Solution for Question 4:


Option A: Crohn’s disease
• This Kantor sign is suggestive of Crohn's disease.
• Crohn’s disease: Chronic transmural inflammatory disease of the gastrointestinal tract. It can involve
any part from the mouth to the anus but most commonly affects the small intestine and colon. The most
common primary surgical disease of the small bowel. Occurs in the second and third decades of life.
Barium contrast studies-- Cobblestone appearance of the mucosa and Kantor sign are seen.
• Chronic transmural inflammatory disease of the gastrointestinal tract.
• It can involve any part from the mouth to the anus but most commonly affects the small intestine and
colon.
• The most common primary surgical disease of the small bowel.
• Occurs in the second and third decades of life.
• Barium contrast studies-- Cobblestone appearance of the mucosa and Kantor sign are seen.

Page 7

763
• Chronic transmural inflammatory disease of the gastrointestinal tract.
• It can involve any part from the mouth to the anus but most commonly affects the small intestine and
colon.
• The most common primary surgical disease of the small bowel.
• Occurs in the second and third decades of life.
• Barium contrast studies-- Cobblestone appearance of the mucosa and Kantor sign are seen.
Option B: Ulcerative colitis
• Ulcerative colitis is a type of inflammatory bowel disease that involves the colon and rectum. The
symptoms include bloody diarrhea and abdominal pain along with iron deficiency anemia.
• Ulcerative colitis presents with a colon that has a smooth, rigid, and featureless appearance,
resembling a lead pipe.
Option C: Hirschsprung’s disease
• Hirschsprung's disease is characterized by the absence of ganglionic cells in the distal colon and
rectum.
• It occurs due to the failure of migration of neural crest cells to distal bowel segments and affects cells
in both myenteric and submucosal plexuses.
Option D: None of the above
• These clinical features and Kantor signs are suggestive of Crohn's disease.

Solution for Question 5:


Option A: NOD-2/CARD-15 gene
• These clinical features are suggestive of Crohn's disease. It is associated with NOD 2/CARD 15 gene
mutations.
• Risk factors of Crohn's disease Smoking Familial predisposition Infectious agents Mycobacterium
paratuberculosis Enteroadherent Escherichia coli Immunological factors Genetic factors NOD 2/CARD
15, IL23R, ATG16L
• Smoking
• Familial predisposition
• Infectious agents Mycobacterium paratuberculosis Enteroadherent Escherichia coli
• Mycobacterium paratuberculosis
• Enteroadherent Escherichia coli
• Immunological factors
• Genetic factors NOD 2/CARD 15, IL23R, ATG16L
• NOD 2/CARD 15, IL23R, ATG16L
• Smoking
• Familial predisposition
• Infectious agents Mycobacterium paratuberculosis Enteroadherent Escherichia coli

Page 8

764
• Mycobacterium paratuberculosis
• Enteroadherent Escherichia coli
• Immunological factors
• Genetic factors NOD 2/CARD 15, IL23R, ATG16L
• NOD 2/CARD 15, IL23R, ATG16L
• Mycobacterium paratuberculosis
• Enteroadherent Escherichia coli
• NOD 2/CARD 15, IL23R, ATG16L
Option B: p53 suppressor gene
• The p53 gene, like the Rb gene, is a tumor suppressor gene, meaning that its activity prevents the
growth of malignancies.
• Mutations in the p53 suppressor gene are associated with various types of cancer, but they are not
specifically linked to Crohn's disease.
Option C: Philadelphia chromosome
• The Philadelphia chromosome is associated with chronic myeloid leukemia (CML), a type of blood
cancer, and is not related to Crohn's disease.
Option D: BRCA-1 gene
• Mutations in the BRCA-1 gene are associated with an increased risk of breast and ovarian cancer but
are not linked to Crohn's disease.

Solution for Question 6:


Option C: Inhibition of NF-KB Activation
• These clinical features are suggestive of ulcerative colitis.
• Its mainstay therapy is sulfasalazine.
• Sulfasalazine exerts its anti-inflammatory action by inhibition of NF-KB and inhibition of prostaglandin
synthesis.
Option A: Folic acid synthesis
• All cells require folic acid for development, produced by bacteria from the substrate
para-amino-benzoic acid (PABA).
• Sulfasalazine inhibits bacterial folic acid synthesis by targeting dihydropteroate synthase. Mammalian
cells, which do not synthesize folic acid de novo, acquire it from dietary sources through diffusion or
active transport. Sulfasalazine indirectly affects mammalian cells by potentially reducing overall folic
acid availability due to its impact on bacterial synthesis.
• Mammalian cells are diffused or transported with folic acid (a vitamin in meals). By diffusion or active
transport, folic acid cannot penetrate bacterial cell walls.
Option B: Formation of prostaglandins (PG)

Page 9

765
• After sequentially oxygenating arachidonic acid, DGLA, or EPA, cyclooxygenases (COX-1 and
COX-2) and terminal prostaglandin synthases create prostaglandins. The traditional tenet is as follows:
The basal prostaglandin levels are caused by COX-1. COX-2 produces prostaglandins through
stimulation.
• The basal prostaglandin levels are caused by COX-1.
• COX-2 produces prostaglandins through stimulation.
• The basal prostaglandin levels are caused by COX-1.
• COX-2 produces prostaglandins through stimulation.
Option D: Formation of interleukins
• Monocytes, macrophages, endothelial cells, and CD4 helper T lymphocytes are responsible for most
interleukins synthesis.
• They support the growth and differentiation of hematopoietic cells, including T and B lymphocytes.

Solution for Question 7:


Option B: Prednisolone
• The patient had an acute exacerbation of ulcerative colitis, in which case corticosteroids are the
mainstay of treatment.
• Corticosteroids such as Prednisolone: In cases of acute exacerbation. Most effective therapy in
inducing remission in patients who do not respond to 5-aminosalicylic acid.
• In cases of acute exacerbation.
• Most effective therapy in inducing remission in patients who do not respond to 5-aminosalicylic acid.
• In cases of acute exacerbation.
• Most effective therapy in inducing remission in patients who do not respond to 5-aminosalicylic acid.

Option A and C
While other medications like sulfasalazine and aminosalicylic acid are commonly used for maintenance
therapy or in milder cases of ulcerative colitis
Option D: NSAIDs
• NSAIDs are generally avoided in ulcerative colitis as they can worsen symptoms and increase the risk
of complications.

Solution for Question 8:


Option B: 35% after 30 years
Given the lesions are limited to the colon, the patient is likely suffering from ulcerative colitis.

Page 10

766
• Clinical signs: bleeding per rectum and lower abdominal pain suggest ulcerative colitis, and the risk of
colorectal carcinoma in ulcerative colitis is 35% at 30 years.
• Most serious sequela of ulcerative colitis - colorectal carcinoma.
• Most important risk factors: Prolonged duration of the disease Pancolonic disease Continuously active
disease Severity of inflammation
• Prolonged duration of the disease
• Pancolonic disease
• Continuously active disease
• Severity of inflammation
• The cumulative risk of cancer increases with the duration of ulcerative colitis, reaching 5% at 25 years
35% at 30 years 45% at 35 years 65% at 40 years
• 5% at 25 years
• 35% at 30 years
• 45% at 35 years
• 65% at 40 years
• Patients with the left-sided colonic disease have a lower risk of developing cancer than those with
entire colonic disease
• The risk of malignancy in Crohn's pancolitis is similar to UC pancolitis 2% after ten years 8% after 20
years 18% after 30 years
• 2% after ten years
• 8% after 20 years
• 18% after 30 years
• Prolonged duration of the disease
• Pancolonic disease
• Continuously active disease
• Severity of inflammation
• 5% at 25 years
• 35% at 30 years
• 45% at 35 years
• 65% at 40 years
• 2% after ten years
• 8% after 20 years
• 18% after 30 years

INCORRECT OPTIONS:
Options B, C and D are incorrect.

Page 11

767
Solution for Question 9:
Option D: Toxic megacolon
• X-ray shows a massively distended transverse colon, a haustral pattern of the left colon
• This is a patient with a history of fever and known ulcerative colitis who has now developed fulminant
colitis or toxic megacolon.
• Toxic Megacolon: Toxic megacolon is a serious life-threatening condition that can occur in patients
with Ulcerative colitis Crohn's colitis Infectious colitis (Clostridium difficile)
• Toxic megacolon is a serious life-threatening condition that can occur in patients with Ulcerative colitis
Crohn's colitis Infectious colitis (Clostridium difficile)
• Ulcerative colitis
• Crohn's colitis
• Infectious colitis (Clostridium difficile)
• This results in a necrotic thin-walled bowel in which pneumatosis can often be seen radiographically.
• The patient usually has a high fever, tachycardia, severe abdominal pain and tenderness, and
leukocytosis, which are signs of toxicity.
• Toxic megacolon is a serious life-threatening condition that can occur in patients with Ulcerative colitis
Crohn's colitis Infectious colitis (Clostridium difficile)
• Ulcerative colitis
• Crohn's colitis
• Infectious colitis (Clostridium difficile)
• Ulcerative colitis
• Crohn's colitis
• Infectious colitis (Clostridium difficile)
Option A: Pseudomembranous colitis
• Pseudomembranous colitis is a severe form of colitis characterized by the formation of
pseudomembranes on the surface of the colon, primarily caused by the overgrowth of Clostridium
difficile bacteria in the colon.
• Diagnosis is typically made through stool tests to detect C. difficile toxins or through visualization of
pseudomembranes during colonoscopy.
Option B: Adenocarcinoma colon
• Adenocarcinoma typically presents with more insidious symptoms, such as changes in bowel habits,
weight loss, and occult bleeding.
• While patients with chronic ulcerative colitis are at an increased risk of developing colorectal cancer,
the acute presentation with fever, abdominal pain, and characteristic X-ray findings (including colonic
dilation and free intraperitoneal air) is more consistent with toxic megacolon than with adenocarcinoma.
Option C: Pneumatosis intestinalis

Page 12

768
• Pneumatosis intestinalis is a known radiographic finding in severe cases of NEC. It appears as linear
or cystic collections of gas within the wall of the intestine, particularly in the submucosal layer.

Solution for Question 10:


Option A: Crypt abscess
• The most characteristic lesion of ulcerative colitis- crypt abscess (collections of neutrophils filling and
expanding the lumina of individual crypts of Lieberkuhn) and crypt branching.
• Inflammatory bowel disease:
Ileal disease
Anal fissure, fistula, abscess
Common
Rare
Total proctocolectomy
Combined disease: colon and rectum
Curative
INCORRECT OPTIONS:
Options B, C and D are common to both the conditions.

Page 13

769
Appendix
1. In the case of acute appendicitis, pain is illustrated in the right hip on the extension of the right thigh.
What is the term used to refer to this examination finding?
A. Dunphy’s sign
B. Obturator sign
C. Psoas sign
D. Rovsing’s sign
----------------------------------------
2. Alvarado's scoring in appendicitis includes which of the following?
A. Migratory right iliac fossa pain
B. Flank pain
C. Normal temperature
D. Leukopenia
----------------------------------------
3. A 28-year-old female presents to the Gynecology emergency department with complaints of
right-sided abdominal pain associated with vomiting for the last 2 hours. She is G2P1+0 and has
gestational amenorrhea of 8 weeks. She has no previous significant gynecology history. Her
examination reveals a tender iliac fossa associated with guarding and rebound tenderness. Her urine
analysis shows leukocytosis, and an ultrasound abdomen suggests acute appendicitis. Which of the
following is true regarding appendicitis in pregnancy?
(or)
A 28-year-old pregnant female had an ultrasound abdomen which suggests acute appendicitis. Which
of the following is true regarding appendicitis in pregnancy?
A. The most common cause is acute abdomen in the first trimester
B. Pregnancy increases the risk
C. Conservative management by antibiotics should be tried
D. After rupture, fetal mortality is around 1%
----------------------------------------
4. In which of the following conditions Oschner-Sherren's regimen is used as management?
A. Appendicular abscess
B. Chronic appendicitis
C. Appendicular mass
D. Acute appendicitis
----------------------------------------
5. A 30-year-old man with abdominal pain, fever, and vomiting underwent an appendectomy after a
diagnosis of a neoplastic lesion in his appendix was made. What would be the most common neoplasm
of the appendix in this clinical scenario?
A. Lymphoma

770
B. Adenocarcinoma
C. Leiomyosarcoma
D. Argentaffinoma
----------------------------------------
6. What is the most common incision used in open appendectomy?
A. McBurney’s incision
B. Lanz Incision
C. Lower midline incision
D. Battle incision
----------------------------------------
7. An 18-year-old female presents to the surgical emergency room with a complaint of abdominal pain
for the last 7 hours. She states that it initially started around the umbilicus and later shifted to the right
lower abdomen. There is associated loss of appetite, nausea, and vomiting. Her previous medical and
surgical history is insignificant. Her pulse rate is 96/min, her blood pressure is 125/90 mmHg, her
respiratory rate is 17/min, and her temperature is 37°C (98°F). Examination reveals tender right iliac
fossa with rebound tenderness. A diagnosis of acute uncomplicated appendicitis is made, and she is
planned for emergency open appendectomy under general anesthesia with Gridiron incision. Which
nerve is commonly injured with this incision?
(or)
An 18-year-old female diagnosed with acute uncomplicated appendicitis is made, and she is planned
for emergency open appendectomy under general anaesthesia with Gridiron incision. Which nerve is
commonly injured with this incision?
A. Subcostal nerve
B. Iliohypogastric nerve
C. Femoral nerve
D. Ilioinguinalnerve
----------------------------------------
8. The appendicular artery arises from which of the following arteries?
A. Right colic artery
B. Ileocolic artery
C. Posterior cecal artery
D. Superior mesenteric artery
----------------------------------------
9. Removal of the appendix has been associated with a protective effect of which of the following?
A. Crohn's disease
B. Ulcerative colitis
C. Clostridium difficile
D. Colonic carcinoma

Page 2

771
----------------------------------------
10. A 22-year-old female presents to a surgical emergency with abdominal pain for the last 5 hours.
She states that it initially started around the umbilicus and later shifted to the right lower abdomen and
is gradually progressive. It is associated with nausea, vomiting, and loss of appetite. Examination
reveals tender right iliac fossa with rebound tenderness. Investigations show only leukocytosis. A
diagnosis is made, and she is planned for an emergency open appendectomy. She is concerned about
the scar post-surgery. Which of the following incisions would be preferable for appendectomy for a
better cosmetic approach in this patient?
(or)
A 22-year-old female is planned for an emergency open appendectomy. She is concerned about the
scar post-surgery. Which of the following incisions would be preferable for appendectomy for a better
cosmetic approach in this patient?
A. Rutherford Morison incision
B. Lanz incision
C. Lower midline incision
D. Gridiron incision
----------------------------------------
11. In a patient with an appendectomy specimen revealing a carcinoid tumor measuring 2.5 cm at the
tip of the appendix, what would be the next step in management?
A. Segmental resection
B. Appendectomy
C. Right hemicolectomy
D. Do yearly 5-HIAA assay
----------------------------------------
12. Which of the following is the most common complication of appendectomy in the early
postoperative period?
A. Wound infection
B. Ileus
C. Adhesive intestinal obstruction
D. Portal pyemia
----------------------------------------
13. A 23-year-old female with tenderness in the right iliac fossa is being prepared for an exploratory
laparotomy. The specimen from the surgery is given below. What gold standard investigation is used to
diagnose this condition before surgery in doubtful cases?

Page 3

772
A. Contrast-enhanced CT scan abdomen
B. Ultrasound
C. MRI
D. Laparotomy
----------------------------------------
14. Which of the following is true about appendectomy?
A. Lanz incision is also known as Battle incision
B. Purse string suture in cases of edematous and inflamed caecal wall
C. Crushing is done in a gangrenous base of the appendix
D. The base is not crushed in cases of an inflamed base of the appendix
----------------------------------------
15. A 23-year-old primigravida with 5 weeks of amenorrhea presented with right iliac fossa tenderness,
guarding, and rebound tenderness. Urine analysis showed leukocytosis. An abdominal ultrasound was
performed but the appendix was not visualized. What would be the diagnostic tool to proceed further in
this case?
A. Diagnostic laparoscopy
B. Abdominal CT scan
C. Abdominal Magnetic resonance imaging (MRI) scan
D. Serial clinical observations
----------------------------------------

Correct Answers
Question Correct Answer

Question 1 3
Question 2 1
Question 3 1
Question 4 3
Question 5 4

Page 4

773
Question 6 1
Question 7 2
Question 8 2
Question 9 2
Question 10 2
Question 11 3
Question 12 1
Question 13 1
Question 14 4
Question 15 3

Solution for Question 1:


Option C: Psoas sign
• Psoas sign: this sign is present when an inflamed appendix lies over the psoas muscle. The psoas
sign is associated with a retrocecal appendix, which manifests as right lower quadrant pain with passive
right hip extension. The inflamed appendix may lie against the right psoas muscle, causing the patient
to shorten the muscle by drawing up the right knee. Passive extension of the iliopsoas muscle with hip
extension causes right lower quadrant pain.
• The psoas sign is associated with a retrocecal appendix, which manifests as right lower quadrant pain
with passive right hip extension. The inflamed appendix may lie against the right psoas muscle, causing
the patient to shorten the muscle by drawing up the right knee. Passive extension of the iliopsoas
muscle with hip extension causes right lower quadrant pain.
• The psoas sign is associated with a retrocecal appendix, which manifests as right lower quadrant pain
with passive right hip extension. The inflamed appendix may lie against the right psoas muscle, causing
the patient to shorten the muscle by drawing up the right knee. Passive extension of the iliopsoas
muscle with hip extension causes right lower quadrant pain.
Other options
Option A: Dunphy’s sign
• In appendicitis, any abdominal wall movement may elicit right lower abdominal pain.
• Increased right iliac fossa pain with coughing is called Dunphy’s sign.
• This happens because a cough jostles the inflamed peritoneum.
Option B: Obturator sign
• The Obturator sign is positive when the internal rotation of the right hip with a flexed knee and hip
produces right lower quadrant pain.
• The obturator sign is associated with a pelvic appendix.
• The given clinical finding is inconsistent with the obturator sign
Option D: Rovsing’s sign
• Rovsing’s sign is the pain in the right iliac fossa during deep palpation of the left iliac fossa.

Page 5

774
• This maneuver causes air within the colon to flow retrograde in response to compression, resulting in
distension of the inflamed appendix and activation of a viscerosensory segmental reflex because the
ileocaecal valve is competent, pain is produced in the right iliac fossa with inflammation of the appendix
and caecum.

Solution for Question 2:


Option A: Migratory right iliac fossa pain
• Migratory right iliac fossa pain starts in the periumbilical region and then migrates to the right iliac
fossa.
• Initially, pain near the umbilicus is due to inflamed visceral peritoneum. As inflammation progresses,
the inflamed appendix affects the parietal peritoneum, leading to sharp pain in the right iliac fossa.
• Alvarado (MANTRELS) score is used to predict acute appendicitis.
• Migratory right iliac fossa pain is included in the Alvarado (MANTRELS) score to predict acute
appendicitis.
• Migratory RIF pain
• Anorexia
• Nausea and vomiting
1
• Tenderness (RIF)
• Rebound tenderness
• Elevated temperature
2
• Leukocytosis
• Shift to left

Option B: Flank pain


• Flank pain occurs in conditions such as renal colic or urinary tract infections.
• The pain of acute appendicitis is migratory and starts from the umbilical region and then migrates to
the right iliac fossa.
Option C: Normal temperature
• Mild pyrexia is common. Temperature is usually 37.5° C - 38° C.
Option D: Leukopenia
• Leukopenia is defined as decreased white blood cells in the blood and occurs in viral as well as
enteric fever.
• A neutrophil-predominant leukocytosis is seen in 80-90% of acute appendicitis.

Page 6

775
Solution for Question 3:
Option A: The most common cause is acute abdomen in the first trimester
• The patient is most likely suffering from acute appendicitis. It is the most common cause of acute
abdomen in the first trimester.
• Appendicitis in a pregnant patient remains the most common non-obstetric emergency.
• It occurs in 1 in 800 to 1 in 1000 pregnancies, mostly in the first and second trimesters.
• Pregnancy may alter the disease's presentation and laboratory values like WBC count (leukocytosis
as high as 16000 cells/L is common in pregnancy), making diagnosis difficult.
• Its symptoms typically consist of right lateral abdominal pain, nausea, and anorexia, yet typical
presentation accounts for only 50% to 60% of cases.
• The point of maximum tenderness is usually displaced on physical examination.
• Ultrasonography is the preferred imaging modality, although non-visualization can occur.
• In pregnant patients, plain MRI can be used alternatively for diagnosis.
• For suspects at any stage of pregnancy, an early appendectomy is the best option.
Option B: Pregnancy increases the risk
• Pregnancy does not increase the risk of getting acute appendicitis.
• Risk during pregnancy is the same as in non-pregnant women of the same age. However, pregnancy
may alter the disease's presentation and laboratory values.
Option C: Conservative management by antibiotics should be tried
• In the case of non-operative management, treatment failure rates have been reported to be as high as
25%.
• Hence, early appendectomy is the appropriate therapy for suspected during all stages of pregnancy.
Option D: After rupture, fetal mortality is around 1%
• The risk of fetal loss is up to 36% if appendiceal perforation occurs.

Solution for Question 4:


Option C: Appendicular mass
• Oschner-Sherren is indicated in appendicular mass.
• If an appendix mass is present and the patient's condition is satisfactory, the standard treatment is the
conservative Oschner-Sherren regime. Temperature and pulse rate should be recorded 4 hourly. The
fluid balance record is maintained. The extent of mass should be marked and examined regularly.
• Temperature and pulse rate should be recorded 4 hourly.
• The fluid balance record is maintained.
• The extent of mass should be marked and examined regularly.
• Criteria for stopping conservative management of an appendix mass A rising pulse rate Increased or
spreading abdominal pain The increasing size of the mass

Page 7

776
• A rising pulse rate
• Increased or spreading abdominal pain
• The increasing size of the mass
• Temperature and pulse rate should be recorded 4 hourly.
• The fluid balance record is maintained.
• The extent of mass should be marked and examined regularly.
• A rising pulse rate
• Increased or spreading abdominal pain
• The increasing size of the mass
Option A: Appendicular abscess
• The appendicular abscess is a collection of pus resulting from necrosis of the tissue superimposed
with infection in an inflamed appendix.
• Treatment of appendicular abscesses usually involves drainage either by percutaneous or surgical
methods, along with antibiotic therapy.
Option B: Chronic appendicitis
• Chronic appendicitis is a long-standing inflammation or fibrosis of the appendix that presents clinically
as prolonged or intermittent abdominal pain.
• The preferred treatment for chronic appendicitis is surgical removal of the appendix (appendectomy)
rather than conservative management.
Option D: Acute appendicitis
• Acute appendicitis is an inflammatory condition of the appendix characterized by acute severe right
iliac fossa pain and leukocytosis and requires urgent appendectomy.

Solution for Question 5:


Option D: Argentaffinoma
• Argentaffinoma may be encountered unexpectedly during any surgical procedure involving the
appendix.
Argentaffinoma:
• The most common neoplasm of the appendix is Argentaffinoma.
• They are often asymptomatic and may be discovered incidentally during imaging or surgery for other
conditions. When symptoms do occur, they may include abdominal pain and appendicitis-like
symptoms
• Most common site is the tip of the appendix
• Measurement of serum Chromogranin A is recommended as the best predictor of malignant behavior,
metastatic potential, and lymphovascular invasion.
• Treatment:
• Up to 1cm- Negative margin Appendectomy

Page 8

777
• >1cm to 2cm - Right colectomy if mesentery involved, enlarged lymph nodes, positive or unclear
margins.
• > 2cm - Right hemicolectomy
• Indications of Right hemicolectomy: Involvement of mesoappendix, cecal base, and lymph node.
Option A: Lymphoma
• Primary lymphomas of the appendix are extremely rare tumours.
• The incidence is 0.015% out of all gastrointestinal lymphomas.
Option B: Adenocarcinoma
• Adenocarcinoma appendix grows from cells that make up the appendix’s lining.
• It is extremely rare, with an estimated incidence of 0.15-0.9 per 100,000 people.
• It may also be called Epithelial appendiceal cancer. These cells help make a jelly-like substance
called mucin.
Option C: Leiomyosarcoma
• Mesenchymal tumours of the appendix are most often of the smooth muscle type, usually a
leiomyoma.
• Leiomyosarcoma is a very rare carcinoma.

Solution for Question 6:


Option A: McBurney’s incision
• The treatment of choice for acute uncomplicated appendicitis is emergency appendectomy.
• McBurney’s or Gridiron incision is the most common incision used for open appendectomy.
• The McArthur gridiron incision is centered on McBurney's point and made at a right angle to a line
between the anterior superior iliac spine and the umbilicus.
• It comprises an arc incision over the right lower abdomen through the skin, subcutaneous fat and
fascia, external and internal oblique, transversus abdominis, and transversalis fascia.

• The incision may be altered to follow a horizontal Langer's line, which improves cosmetic results when
the appendix has become sufficiently inflamed to cause localized peritonitis.
• If it is converted into a muscle-cutting incision, it is called Rutherford Morison incision.

Page 9

778
Other options
Option B: Lanz incision
• Lanz incision is a horizontal incision, while the gridiron incision is on an oblique angle.
• Injury to ilioinguinal and iliohypogastric nerves is more common with this incision.
Option C: Lower Midline incision
• Lower Midline incision is not the most common incision for open appendectomy.
• This incision is made when the diagnosis of a ruptured appendix or peritonitis is suspected. The
incision is made approximately 2 cm below the umbilicus centered on the midclavicular–mid-inguinal
line.
• It allows more exposure to the abdominal cavity
Option D: Battle incision
• It is also not a common incision in open appendectomy.
• Battle incision is a lower right paramedian incision placed more laterally than other standard
paramedian incisions.

Solution for Question 7:


Option B: Iliohypogastric nerve
• Gridiron incision is the most common incision used for open appendectomy.
• It comprises an arc incision over the right lower abdomen through the skin, subcutaneous fat and
fascia, external and internal oblique, transversus abdominis, and transversalis fascia.
• Injury to the iliohypogastric nerve is the most common nerve injury following a McBurney’s incision.
• Trauma to the iliohypogastric nerve causes paresthesia in the inguinal and suprapubic regions.
• It is treated with a local anesthetic injection, anti-inflammatory medicines, and cryotherapy.

Page 10

779
• Its injury also causes the weakening of the posterior wall of the inguinal canal and predisposes the
development of a direct hernia.
• This injury later leads to the development of a right inguinal hernia
Option A: Subcostal nerve
• This is not a common nerve injured following a gridiron incision for appendectomy.
• This nerve lies a bit above the iliohypogastric nerve.
• Therefore, less likely to be injured with this incision
Option C: Femoral nerve
• This is not a common nerve injured following a Gridiron incision for appendectomy.
• Femoral nerve is mostly injured in orthopedic and gynecologic surgeries.
Option D: Ilioinguinal nerve
• Although the ilioinguinal nerve is also at risk of injury in Gridiron incision, it is less common compared
to iliohypogastric nerve injury.
• It can also result in an increased risk of inguinal hernia

Solution for Question 8:


Option B: Ileocolic artery
• The appendicular artery is a branch of the lower division of the ileocolic artery.
• It traverses behind the terminal ileum and enters the mesoappendix, just proximal to the base of the
appendix.
• It traverses further to lie in the free border of the mesoappendix.
• An accessory appendicular artery may be present.
• The appendicular artery is an end artery that may easily become thrombosed during acute
inflammation, leading to necrosis and perforation of the distal appendix.

Page 11

780
Option A: Right colic artery
• The right colic artery is a branch of the superior mesenteric artery.
• It supplies the ascending colon.
Option C: Posterior cecal artery
• Posterior cecal artery directly supplies the cecum and ileocecal valve.
• It arises from the ileocolic artery.
Option D: Superior mesenteric artery
• Superior mesenteric artery arises directly from the aorta.
• It divides into the right colic, middle colic, and ileocolic artery (this artery, in turn, gives appendicular
artery).

Solution for Question 9:


Option B: Ulcerative colitis
• Early appendicectomy modifies the intestinal immune response to protect against the development of
ulcerative colitis.
• Multiple case-control and epidemiological studies have proven a strong inverse relationship between
prior appendectomy and the development of ulcerative colitis.
Option A: Crohn’s disease
• Appendectomy does not confer protection against the development of Crohn's disease.
• Studies have shown an increased risk of Crohn's disease in patients with appendectomy.
• Young adults are at risk of developing Crohn's disease, particularly during the first years after surgery.
Option C: Clostridium difficile

Page 12

781
• While appendectomy removes the appendix, which can serve as a reservoir for gut bacteria, there is
no evidence to suggest that it provides a protective effect against Clostridium difficile infection.
Option D: Colonic carcinoma
• Appendectomy does not confer protection against colonic carcinoma.

Solution for Question 10:


Option B: Lanz incision
• The treatment of choice for acute uncomplicated appendicitis is emergency appendectomy.
• The Lanz incision gives additional cosmetic benefits because it is hidden below the bikini line.
• It also gives better exposure and easier extension if needed.
• It is a transverse skin crease incision of appropriate length made approximately 2 cm below the
umbilicus centered on the midclavicular and mid-inguinal lines. The aponeurosis of the internal oblique
and external oblique is split in the direction of the fibers, and the peritoneum is opened.
• If necessary, the incision may be extended medially after suitable division or retraction of the rectus
abdominis muscle.

Option A: Rutherford Morison incision


• Rutherford Morison’s incision is not ideal in this case as it does not give the cosmetic benefit required
by the patient.
• It is beneficial when the appendix is retrocecal and is fixed.
• It is an oblique muscle-cutting incision having a lower end over McBurney’s point and extending
upwards and laterally obliquely as needed.
• All layers are divided in the line of the incision.
Option C: Lower midline incision

Page 13

782
• A lower midline incision is not ideal in this case as it does not give the cosmetic benefit required by the
patient
• It is preferred only when the diagnosis is in doubt, particularly in the presence of peritonitis and
incision.
Option D: Gridiron incision
• McBurney’s or Gridiron incision is the most common incision used for open appendectomy.
• It also lacks additional cosmetic benefits and is not an ideal option in this case.

Solution for Question 11:


Option C: Right hemicolectomy
• Carcinoid tumors originate in argentaffin or Kulchitsky cells of the crypts of Lieberkuhn of the
appendix.
• Carcinoid tumor of the appendix rarely gives rise to metastases.
• The treatment is based on their grade, size, and the extent of tumor invasion.
• Tumor size best predicts malignant behavior, metastatic potential, and lymphovascular invasion.
Tumour size up to 1cm- Negative margin appendectomy Tumour size >1cm to 2cm - Right colectomy if
mesentery involved, enlarged lymph nodes, positive or unclear margins Tumour size > 2cm - Right
hemicolectomy
• Tumour size up to 1cm- Negative margin appendectomy
• Tumour size >1cm to 2cm - Right colectomy if mesentery involved, enlarged lymph nodes, positive or
unclear margins
• Tumour size > 2cm - Right hemicolectomy
• Indications for right hemicolectomy: Involvement of cecal wall Tumour of 2cm or more in size
Involvement of lymph nodes
• Involvement of cecal wall
• Tumour of 2cm or more in size
• Involvement of lymph nodes
• Tumour size up to 1cm- Negative margin appendectomy
• Tumour size >1cm to 2cm - Right colectomy if mesentery involved, enlarged lymph nodes, positive or
unclear margins
• Tumour size > 2cm - Right hemicolectomy
• Involvement of cecal wall
• Tumour of 2cm or more in size
• Involvement of lymph nodes
Option A: Segmental resection
• Segmental resection is not an ideal option for a carcinoid tumor of the appendix.
• Carcinoid tumor rarely involves the cecal wall and rarely metastasizes.

Page 14

783
• This is the surgical option that would be suitable for colonic cancer.
Segmental resection is not an ideal option for a carcinoid tumor of the appendix.
Carcinoid tumor rarely involves the cecal wall and rarely metastasizes.
This is the surgical option that would be suitable for colonic cancer.
Option B: Appendectomy
• Limiting treatment to appendectomy only is not sufficient.
• Most carcinoid tumors of the appendix are incidentally diagnosed on biopsy after appendectomy.
• For carcinoid tumors of 2.5 cms, appendectomy should be followed by right hemicolectomy.
Option D: Do yearly 5-HIAA assay
• 5-HIAA (5-hydroxy-indole-acetic acid) is a metabolic product of serotonin excreted in the urine
• Measurement of this in a 24-hour urine collection is useful for the diagnosis and follow-up of carcinoid
tumors
• This assay is useful for monitoring disease after debulking surgery; therefore, its yearly measurement
without hemicolectomy is not recommended

Solution for Question 12:


Option A: Wound infection
• The treatment of choice for acute uncomplicated appendicitis is emergency appendectomy.
• Surgical site infection is the most common complication in 5-10 percent of all cases.
• This commonly presents with erythema and pain of the wound on the 4th or 5th postoperative day and
often soon after hospital discharge.
• Treatment includes wound drainage and antibiotics as required.
• A mixture of organisms, including anaerobes, gram-negative bacilli, Bacteroides species, and
anaerobic streptococci is responsible for causing wound infection.
Option B: Ileus
• A period of ileus is expected after appendectomy and may last for a few days after removing a
gangrenous appendix.
• Persistent ileus for more than 4 to 5 days, particularly with fever, indicates continuing intra-abdominal
sepsis and should prompt additional investigations.
• This complication is not as common as wound infection.
Option C: Adhesive intestinal obstruction
• It is the most common late complication of appendectomy.
• Often, a single-band adhesion is responsible for producing intestinal obstruction.
• Sometimes, chronic pain in the right iliac fossa is due to adhesion formation after an appendectomy,
where laparoscopy is used to confirm the presence of adhesions and also to divide those adhesions.
• This complication is not as common as wound infection.

Page 15

784
Option D: Portal pyemia
• Portal pyemia is a rare condition associated with infection and thrombosis within the portal vein.
• This complication is not as common as wound infection.

Solution for Question 13:


Option A: Contrast-enhanced CT scan abdomen
• The specimen in the given image is the appendix and the surgery performed is appendectomy
through a lower midline incision.
• The gold standard for diagnosis of acute appendicitis is a contrast-enhanced CT scan of the
abdomen.

The Arrowhead sign demonstrates a dilated and fluid-filled appendix with thickened appendiceal walls.
• Findings on CT scan: Inflamed and thickened appendix with surrounding stranding indicating
inflammation. The appendix is classically enlarged more than 7 mm in diameter with an inflamed,
thickened (>2mm thick) and mural enhancement or target sign. Perforation of the appendix is
suggested by perpendicular fluid or air. Other CT findings Periappendiceal fat stranding Appendiceal
wall enhancement Appendicolith (seen in approximately 25 percent of patients)
• Inflamed and thickened appendix with surrounding stranding indicating inflammation.
• The appendix is classically enlarged more than 7 mm in diameter with an inflamed, thickened (>2mm
thick) and mural enhancement or target sign.
• Perforation of the appendix is suggested by perpendicular fluid or air.
• Other CT findings Periappendiceal fat stranding Appendiceal wall enhancement Appendicolith (seen
in approximately 25 percent of patients)
• Periappendiceal fat stranding

Page 16

785
• Appendiceal wall enhancement
• Appendicolith (seen in approximately 25 percent of patients)
• Findings on Ultrasound The appendix is found immobile, enlarged (>7mm), and non-compressible.
• The appendix is found immobile, enlarged (>7mm), and non-compressible.
• Inflamed and thickened appendix with surrounding stranding indicating inflammation.
• The appendix is classically enlarged more than 7 mm in diameter with an inflamed, thickened (>2mm
thick) and mural enhancement or target sign.
• Perforation of the appendix is suggested by perpendicular fluid or air.
• Other CT findings Periappendiceal fat stranding Appendiceal wall enhancement Appendicolith (seen
in approximately 25 percent of patients)
• Periappendiceal fat stranding
• Appendiceal wall enhancement
• Appendicolith (seen in approximately 25 percent of patients)
• Periappendiceal fat stranding
• Appendiceal wall enhancement
• Appendicolith (seen in approximately 25 percent of patients)
• The appendix is found immobile, enlarged (>7mm), and non-compressible.
Option B: Ultrasound
• It is not the gold standard investigation for diagnosing acute appendicitis.
• It can demonstrate an immobile, enlarged and non-compressible appendix
• Sensitivity is very low and operator dependent
• Not a common investigation for diagnosing acute appendicitis
Option C: MRI
• It is also not a gold standard investigation for acute appendicitis.
• MRI is reserved for radiosensitive populations such as children and pregnant females.
• Widespread availability and claustrophobia are important limitations to its use.
Option D: Laparotomy
• Laparotomy is not a preoperative diagnostic tool.
• It is diagnostic as well as therapeutic for acute appendicitis.
• In doubtful cases and the non-availability of imaging modalities, laparotomy is done.

Solution for Question 14:


Option D: The base is not crushed in cases of an inflamed base of the appendix
• Treatment of choice for acute uncomplicated appendicitis - emergency appendectomy.

Page 17

786
• Many types of incision have been introduced depending upon clinical findings about the site of
maximum tenderness, confirmation of diagnosis and patient preference.
• Upon entering the peritoneum, the appendix is identified, and adhesions are released.
• The next step is clamping and ligation of the base of the mesoappendix
• The base of the appendix is crushed near its junction with the caecum.
• An absorbable 2/0 or 3/0 purse-string or ‘Z’ suture may then be inserted into the caecum about 1.25
cm from the base. The stump of the appendix is invaginated while the purse-string or ‘Z’ suture is tied,
thus burying the appendix stump.

Methods to be adopted in special circumstances


The inflamed base of the appendix
• The base is not crushed as it may spread infection through the bloodstream and lymphatics.
• The base of the appendix is ligated close to the caecal wall tightly enough to occlude the lumen, after
which the appendix is removed and the stump invaginated.
The gangrenous base of the appendix
• Neither ligation nor crushing should be done.
• Two stitches are placed through the wall of the caecum near the base of the gangrenous appendix. It
is followed by the removal of the appendix, which is amputated flush with the wall of the caecum, after
which those two stitches are tied.
Edematous and inflamed caecal wall
• Purse string suture is not applied.
• The stump is not invaginated.
Option A: Lanz incision is also known as Battle incision
• Lanz incision is also known as Davis incision, not battle incision.
• It is a transverse skin crease incision of appropriate length made approximately 2 cm below the
umbilicus centered on the midclavicular and mid-inguinal lines.
• The Lanz incision gives additional cosmetic benefits
Option B: Purse string suture is applied in cases of edematous and inflamed caecal wall
• Purse string suture is not applied in cases of an edematous and inflamed caecal wall.
• The purse-string suture is in danger of cutting out when applied to the edematous caecal wall.
• If the caecal edema is limited, this can be overcome by putting the purse-string suture into a healthier
part of the caecal wall farther from the base of the appendix.
Option C: Crushing is done in a gangrenous base of the appendix
• Neither ligation nor crushing should be done.
• Two stitches are placed through the wall of the caecum near the base of the gangrenous appendix. It
is followed by the removal of the appendix, which is amputated flush with the wall of the caecum, after
which those two stitches are tied.
• Further closure is done through the second layer of interrupted seromuscular sutures.

Page 18

787
Solution for Question 15:
Option C: Abdominal Magnetic resonance imaging (MRI) scan
• The patient is most likely suffering from acute appendicitis based on the features of right-side
abdominal pain associated with vomiting. The investigation of choice is ultrasonography, but the
appendix could not be visualized in this case. So, Magnetic resonance imaging is indicated.
• Pregnancy may alter the presentation of the disease and laboratory values like WBC count
(leukocytosis as high as 16000 cells/µL is common in pregnancy), making diagnosis difficult.
• Its symptoms typically consist of right lateral abdominal pain, nausea, and anorexia, yet typical
presentation accounts for only 50% to 60% of cases.
• Signs include: Right lower quadrant tenderness, Rebound tenderness Abdominal guarding Rectal
tenderness Right upper quadrant tenderness
• Right lower quadrant tenderness, Rebound tenderness
• Abdominal guarding
• Rectal tenderness
• Right upper quadrant tenderness
• Ultrasonography is the preferred imaging modality, although non-visualization can occur.
• MRI is an alternating imaging modality in case of non-visualization of the appendix in ultrasound, with
a sensitivity of 94% and specificity of 97%.
• Right lower quadrant tenderness, Rebound tenderness
• Abdominal guarding
• Rectal tenderness
• Right upper quadrant tenderness
Option A: Diagnostic laparoscopy
• The next best step is not to directly do surgery or diagnostic lap as it poses a very high risk to both the
mother and the fetus- hence, the diagnosis should be first confirmed with ultrasound or MRI
if ultrasound examination is inconclusive.
Option B: Abdominal CT scan
• Plain abdominal films and CT scans can be done, but there is a risk of radiation exposure to the
developing fetus.
Option D: Serial clinical observations
• The risk of fetal loss is up to 36% if appendicular perforation occurs.
• Serial clinical observations are not indicated in a pregnant lady.

Page 19

788
Rectum & Anal Canal
1. Which of the following is the most common type of anorectal fistula?
A. Intersphincteric
B. Trans-sphincteric
C. Suprasphincteric
D. Extrasphincteric
----------------------------------------
2. A 43-year-old woman with a history of constipation and pelvic surgery presented with lower
abdominal pain, fecal incontinence, and mucus for the past two weeks. She has delivered four healthy
children vaginally.On anal examination, a red mass extended through the anus with concentric rings of
the rectum. What is the most likely diagnosis for this patient?
A. Anal fissure
B. Rectal prolapse
C. Colonic diverticula
D. Irritable bowel syndrome
----------------------------------------
3. What is the ideal pre-surgical investigation for a patient with an anal abscess with associated fistula
formation?
A. Endoanal ultrasound
B. MRI
C. Sinogram
D. Digital rectal examination
----------------------------------------
4. A 42-year-old man has severe pain during defecation. His mother is diagnosed with inflammatory
bowel syndrome. On examination there is perianal pruritus. What is the most appropriate step in the
diagnosis of this patient?
A. Anoscopy
B. History and clinical examination
C. PR examination
D. Ultrasound
----------------------------------------
5. Which of the following setons is used in the surgical treatment of fistula-in-ano?
A. Dissecting seton and draining seton
B. Cutting seton and dissolving seton
C. Cutting seton and draining seton
D. Permanent seton
----------------------------------------

789
6. A 3-year-old child has had blood from the rectum for the last week. Clinical examination revealed
anemia. Detailed history revealed the death of the child's paternal uncle due to colorectal cancer. What
could be the probable diagnosis?
A. Adenomatous Polyposis Coli
B. Piles
C. Juvenile rectal polyp
D. Dysentery
----------------------------------------
7. What is the disadvantage of performing total mesorectal excision?
A. Decrease in the local recurrence
B. Decrease the incidence of impotence
C. Decrease the incidence of bladder dysfunction
D. Higher rate of leakage and stoma formation
----------------------------------------
8. What is the recommended distal resection margin for the surgical removal of rectal carcinoma stage
1?
A. 2 cm
B. 3 cm
C. 4 cm
D. 5 cm
----------------------------------------
9. A 60-year-old female with a history of five spontaneous vaginal delivery, uterine prolapse, mild
chronic constipation, and occasional fecal incontinence is found to have 8 cm of mucosa protruding
from the anus. What is the preferred treatment for this case?
A. Frykman-Goldberg procedure
B. Well's procedure
C. Delorme procedure
D. Ripstein procedure
----------------------------------------
10. A 45-year-old female diabetic with a known case of Crohn's disease presented with the complaint of
painful swelling near the anus for the last 2 days with intermittent fever. A 1cm* 1.5 cm mass was noted
at the 9 o'clock position to the anus. The mass was fluctuant, erythematous, tender, and warmer than
the surroundings. Which of the following statements is true for this disease?
A. The most common type is ischiorectal
B. May be caused by an underlying neoplasm
C. An elliptical-shaped incision is given for drainage under anesthesia
D. Commonly seen more in the female population
----------------------------------------

Page 2

790
11. A 45-year-old male patient with perianal discharge, flatus, and stool coming out from an opening.
Medical history of diabetes, hypertension, and tuberculosis but non-compliant with medications. He had
an anorectal abscess two years ago, which was surgically removed. On examination, the external
opening was found at the 4 o'clock position to the anus. What is the location of the internal opening
according to Goodsall's rule ?
A. 3'o clock
B. 4'o clock
C. 5'o clock
D. 6'o clock
----------------------------------------
12. A 40-year-old male army officer has a 1 cm swelling at the top of the buttock at the natal cleft with
erythema and discharge. He was noted to be more hairy than usual. Which of the following are used as
a treatment option for this condition?
A. Wait for spontaneous rupture
B. Local antibiotics
C. IV antibiotics
D. Bascom's procedure
----------------------------------------
13. A 60-year-old male has an ulcer-proliferative growth located at 3 cm from the anal verge. After
chemoradiation, what is the next best step?
A. Observation and follow-up
B. Low anterior resection
C. Abdominoperineal excision of the rectum
D. Hartmann's procedure
----------------------------------------
14. Which of the following is the investigation of choice for assessing depth of penetration and
perirectal nodes in rectal cancer?
A. Transrectal ultrasound
B. CT scan of the pelvis
C. MRI scan
D. Double-contrast Barium enema
----------------------------------------
15. Which one of the following is the treatment option for stage II carcinoma anal canal?
A. Abdominoperineal resection (APR)
B. APR followed by chemoradiation
C. Concurrent chemoradiation
D. Neoadjuvant chemotherapy followed by Surgery
----------------------------------------

Page 3

791
16. A 60-year-old female patient has a carcinoma extending to the extra rectal tissues, but no lymph
node involvement was seen. What would be the stage of this carcinoma according to Duke's
classification?
A. Stage A
B. Stage B
C. Stage C
D. Stage D
----------------------------------------
17. A 50-year-old patient has rectal carcinoma invading through the muscularis propria into the
perirectal tissue with 2 enlarged lymph nodes. What would be the best treatment route for this patient?
A. Preoperative chemoradiation followed by surgical resection
B. Segmental resection
C. Polypectomy
D. Local transanal excision
----------------------------------------
18. Which of the following procedures used to treat full-thickness rectal prolapse has become obsolete
due to the high risk of anal stenosis?
A. Delorme's procedure
B. Thiersch procedure
C. Altemeier’s procedure
D. Hartmann’s procedure
----------------------------------------
19. During a rectosigmoid resection of rectal carcinoma, surgeon encounters a layer separating the
rectum from the coccyx during the rectosigmoid resection. What is this structure?
A. Denonvilliers’ fascia
B. Scarpa’s fascia
C. Buck’s fascia
D. Waldeyer’s fascia
----------------------------------------
20. A 60-year-old male has a growth at the tip of the index finger around 5 cm from the anal verge.
Which one of the following is the next best investigation tool for this patient?
A. Sigmoidoscopy
B. Proctoscopy
C. Colonoscopy
D. Colposcopy
----------------------------------------
21. A 65-year-old male patient presented with bleeding in the rectum, pruritus at the anus, and
discharge from the anus for the last two months. He also had a history of human papillomavirus

Page 4

792
infection and a 30-pack-year smoking history, lost 10 kg over the last six months. On physical
examination, the lymph nodes in the groin are enlarged. Which of the following is the best next step in
treating this patient?
A. Combined radiation and chemotherapy
B. Reassurance and follow-up after six weeks
C. Abdominal perineal resection
D. Sitz bath and high-fiber diet
----------------------------------------
22. A 35-year-old patient presents with intermittent and painless rectal bleeding. On examination,
first-degree hemorrhoids are observed above the dentate line. The patient is concerned about the
bleeding and seeks appropriate management. What is the most suitable initial management for this
patient?
(or)
What is the recommended initial management for a 35-year-old patient with painless rectal bleeding
and first-degree hemorrhoids observed above the dentate line?
A. Milligan-Morgan open Hemorrhoidectomy
B. Sclerotherapy with 5% phenol in almond oil
C. High-fiber diet and sitz bath
D. Longo’s Stapler hemorrhoidectomy
----------------------------------------
23. A 45-year-old patient presents with recurrent episodes of rectal prolapse. On examination, it is
observed that the prolapse involves the full thickness of the rectum. The patient reports difficulty
tolerating long-duration surgeries. What is the most appropriate management option for this patient?
A. Rectopexy with mesh fixation to the sacrum
B. Resection rectopexy
C. Anal wiring
D. Conservative management with lifestyle modifications
----------------------------------------
24. A 5-year-old child from a rural area presents with rectal prolapse. The parents report a history of
protein energy malnutrition, worm infestation, and recent episodes of diarrhea. What is the most likely
cause of rectal prolapse in this child?
(or)
What is the most likely cause of rectal prolapse in this child?
A. Sacral agenesis
B. Meningomyelocele
C. Wells Rectopexy
D. Protein energy malnutrition, worm infestation, and diarrhea
----------------------------------------
25. Which of the following options about anorectal abscess is incorrect?

Page 5

793
A. Perianal abscess is the most common type of anorectal abscess
B. Ischiorectal abscess is the second most common type of anorectal abscess
C. Infection entering the gland via anal crypts leads to the formation of anorectal abscess
D. Pelvi-rectal abscess is the most common type of anorectal abscess.
----------------------------------------
26. What is the characteristic triad seen in chronic fissures-in-ano?
A. Canoe-shaped Ulcer, Sentinel Pile, and Hypertrophied Papilla
B. Circular Ulcer, Fissure Tag, and Atrophied Papilla
C. Linear Ulcer, Sentinel Pile, and Hypertrophied Papilla
D. Horse-shoe Ulcer, Sentinel Tag, and Atrophied Papilla
----------------------------------------
27. A 35-year-old patient presents with chronic anal fissures and seeks appropriate management.
Which of the following interventions is NOT typically performed in the initial conservative or medical
treatment of chronic anal fissures?
A. Sitz bath with 1% potassium permanganate
B. NOTARA’S lateral sphincterotomy
C. Botox injection into sphincters
D. Lord’s procedure – manual dilatation of sphincter
----------------------------------------
28. What is the characteristic feature of pilonidal sinus, and which flap is commonly used for closure
during its surgical management?
A. Multiple openings along the natal cleft; Karydaki's flap
B. Single opening in the natal cleft; Limberg’s Flap
C. Presence of hair collection in the perianal region; Rhomboid Flap
D. Intermittent serous discharge in the axillary region; BASCOM procedure
----------------------------------------

Correct Answers
Question Correct Answer

Question 1 1
Question 2 2
Question 3 2
Question 4 2
Question 5 3
Question 6 3
Question 7 4

Page 6

794
Question 8 1
Question 9 3
Question 10 2
Question 11 4
Question 12 4
Question 13 2
Question 14 3
Question 15 3
Question 16 2
Question 17 1
Question 18 2
Question 19 4
Question 20 1
Question 21 1
Question 22 3
Question 23 3
Question 24 4
Question 25 4
Question 26 1
Question 27 4
Question 28 2

Solution for Question 1:


Correct option A - Intersphincteric:
• Based on Park's classification, the most common type of anal fistula is intersphincteric.
• Park's classification: There are 4 types of anal fistulas according to Park’s classification.

Page 7

795
Incorrect options:
Option B - Trans-sphincteric:
• Trans-sphincteric is the second most common type of anal fistula.
• The primary track crosses both the internal and external sphincter and then passes through the
ischiorectal fossa to reach the skin of the buttock.
Option C - Suprasphincteric:
• They can be iatrogenic.
• These are difficult to distinguish from high-level trans-sphincteric anal fistula.
Option D - Extrasphincteric:
• They run without specific relation to the sphincters.
• They usually result from pelvic disease or trauma.

Solution for Question 2:


Option B: Rectal prolapse
• This is a case of rectal prolapse. It is circumferential descent of rectum (bowel) through the anal canal.
• Rectal prolapse occurs due to a defect in the pelvic floor or rectal tissue fascia.
• Signs and symptoms Lower abdominal discomfort/pain, straining on defecation, incomplete bowel
evacuation, fecal incontinence with mucus, and erythematous mass extending through the anus with
concentric rings of the rectum.
• Lower abdominal discomfort/pain, straining on defecation, incomplete bowel evacuation, fecal
incontinence with mucus, and erythematous mass extending through the anus with concentric rings of
the rectum.
• Risk factor Women aged >40 Vaginal deliveries or multiparty Pelvic floor dysfunction Chronic
constipation and straining on defecation Prior pelvic surgery Stroke

Page 8

796
• Women aged >40
• Vaginal deliveries or multiparty
• Pelvic floor dysfunction
• Chronic constipation and straining on defecation
• Prior pelvic surgery
• Stroke
• Treatment Medical: Preferred in case of non–full-thickness prolapse (mucosal) Patients are advised to
take adequate fluids and fibres. Patients are advised to perform pelvic floor muscle exercises. Possible
bio-feedback for fecal incontinence. Surgical: Rectopexy is the procedure of choice. It is performed if
full-thickness or symptomatic prolapse presents with fecal incontinence, constipation, or mass
sensation.
• Medical: Preferred in case of non–full-thickness prolapse (mucosal) Patients are advised to take
adequate fluids and fibres. Patients are advised to perform pelvic floor muscle exercises. Possible
bio-feedback for fecal incontinence.
• Preferred in case of non–full-thickness prolapse (mucosal)
• Patients are advised to take adequate fluids and fibres.
• Patients are advised to perform pelvic floor muscle exercises.
• Possible bio-feedback for fecal incontinence.
• Surgical: Rectopexy is the procedure of choice. It is performed if full-thickness or symptomatic
prolapse presents with fecal incontinence, constipation, or mass sensation.
• Rectopexy is the procedure of choice.
• It is performed if full-thickness or symptomatic prolapse presents with fecal incontinence, constipation,
or mass sensation.
• Lower abdominal discomfort/pain, straining on defecation, incomplete bowel evacuation, fecal
incontinence with mucus, and erythematous mass extending through the anus with concentric rings of
the rectum.
• Women aged >40
• Vaginal deliveries or multiparty
• Pelvic floor dysfunction
• Chronic constipation and straining on defecation
• Prior pelvic surgery
• Stroke
• Medical: Preferred in case of non–full-thickness prolapse (mucosal) Patients are advised to take
adequate fluids and fibres. Patients are advised to perform pelvic floor muscle exercises. Possible
bio-feedback for fecal incontinence.
• Preferred in case of non–full-thickness prolapse (mucosal)
• Patients are advised to take adequate fluids and fibres.
• Patients are advised to perform pelvic floor muscle exercises.
• Possible bio-feedback for fecal incontinence.

Page 9

797
• Surgical: Rectopexy is the procedure of choice. It is performed if full-thickness or symptomatic
prolapse presents with fecal incontinence, constipation, or mass sensation.
• Rectopexy is the procedure of choice.
• It is performed if full-thickness or symptomatic prolapse presents with fecal incontinence, constipation,
or mass sensation.
• Preferred in case of non–full-thickness prolapse (mucosal)
• Patients are advised to take adequate fluids and fibres.
• Patients are advised to perform pelvic floor muscle exercises.
• Possible bio-feedback for fecal incontinence.
• Rectopexy is the procedure of choice.
• It is performed if full-thickness or symptomatic prolapse presents with fecal incontinence, constipation,
or mass sensation.
Option A: Anal fissure
• It occurs due to a tear in the anal mucosa (generally appearing in the posterior midline) below the
dentate line.
• Risk factors: a diet low in fibers, chronic constipation, receptive anal intercourse, and vaginal birth.
• Treatment: sitz bath, topical lidocaine, or nitroglycerine
Option C: Colonic diverticula
• Colonic diverticula are due to outpouchings of mucosa and submucosa through the muscularis
propria.
• It is associated with constipation, the strain on defecation, and a low-fiber diet.
• Complications are hematochezia, colovesicular fistula, and diverticulitis.
Option D: Irritable bowel syndrome
• Most commonly affects middle-aged females and presents as relapsing abdomen pain associated
with flatulence, bloating, and change in bowel habits (diarrhea or constipation)
• IBS is due to disturbed intestinal motility with no identifiable pathologic changes.
• IBS symptoms improve with defecation

Solution for Question 3:


Option B: MRI
Magnetic Resonance Imaging (MRI) is particularly well-suited for evaluating perianal abscesses, fistula
s, and associated complications. It provides detailed soft tissue contrast and multiplanar imaging capab
ilities, allowing for accurate assessment of the extent and location of inflammation, identification of abs
cesses and fistulous tracts, and evaluation of the involvement of adjacent structures.
Anal Abscess:
• Symptoms: The patient presents with dull perianal pain and pruritus and difficulty with sitting on hard
surfaces and defecating. The perianal area also presents as erythematous, with swelling and warmth
on examination.

Page 10

798
• The patient presents with dull perianal pain and pruritus and difficulty with sitting on hard surfaces and
defecating.
• The perianal area also presents as erythematous, with swelling and warmth on examination.
• Etiology: It involves flow obstruction, but less common causes include inflammatory bowel disease,
Crohn's disease, diverticulitis, appendicitis, and some foreign bodies that cause obstruction.
• It involves flow obstruction, but less common causes include inflammatory bowel disease, Crohn's
disease, diverticulitis, appendicitis, and some foreign bodies that cause obstruction.
• Pathophysiology: It involves the obstruction of anal glands (anal crypt glands) by hard debris that
leads to bacterial overgrowth, abscess formation, and fistula formation. As seen in this patient, pus with
some fecal material draining through the perianal region.
• It involves the obstruction of anal glands (anal crypt glands) by hard debris that leads to bacterial
overgrowth, abscess formation, and fistula formation. As seen in this patient, pus with some fecal
material draining through the perianal region.
• CLINICAL ASSESSMENT Full examination under anesthesia should be repeated before surgical
intervention. MRI is the gold standard for fistula imaging Usually reserved for difficult recurrent cases.
Advantage of MRI: Its ability to demonstrate secondary extensions Fistulography and CT: Useful
techniques if an extra sphincteric fistula is suspected.
• Full examination under anesthesia should be repeated before surgical intervention.
• MRI is the gold standard for fistula imaging
• Usually reserved for difficult recurrent cases.
• Advantage of MRI: Its ability to demonstrate secondary extensions
• Fistulography and CT: Useful techniques if an extra sphincteric fistula is suspected.
• TREATMENT Treatment options: Fistulotomy, Fistulectomy, Setons, Advancement flaps, and Glues.
Laying open is the surest eradication method, but sphincter division may result in incontinence.
• Treatment options: Fistulotomy, Fistulectomy, Setons, Advancement flaps, and Glues.
• Laying open is the surest eradication method, but sphincter division may result in incontinence.
• The patient presents with dull perianal pain and pruritus and difficulty with sitting on hard surfaces and
defecating.
• The perianal area also presents as erythematous, with swelling and warmth on examination.
• It involves flow obstruction, but less common causes include inflammatory bowel disease, Crohn's
disease, diverticulitis, appendicitis, and some foreign bodies that cause obstruction.
• It involves the obstruction of anal glands (anal crypt glands) by hard debris that leads to bacterial
overgrowth, abscess formation, and fistula formation. As seen in this patient, pus with some fecal
material draining through the perianal region.
• Full examination under anesthesia should be repeated before surgical intervention.
• MRI is the gold standard for fistula imaging
• Usually reserved for difficult recurrent cases.
• Advantage of MRI: Its ability to demonstrate secondary extensions
• Fistulography and CT: Useful techniques if an extra sphincteric fistula is suspected.
• Treatment options: Fistulotomy, Fistulectomy, Setons, Advancement flaps, and Glues.

Page 11

799
• Laying open is the surest eradication method, but sphincter division may result in incontinence.
Option A: Endoanal ultrasound
• Endoanal ultrasound is used as an assessment tool for anal anatomy.
• Indications for endoanal ultrasound are anal carcinoma, fistulas, defective sphincters and abscesses.
• MRI is still the gold standard for perianal fistula and abscesses and is commonly recommended.
Option C: Sinogram
• A sinogram is used with contrast material to look for any sinus, which is the blind pouch.
• Fistulogram instead of sinogram is most commonly used in anal fistulas to view more clearly the
abnormal two-end passage.
Option D: Digital rectal examination
• Digital rectal examination is not recommended in anal abscesses, fistulas and sinuses.
• Patients with fistula already have throbbing pain in the perianal region, so DRE is a painful procedure
in these conditions and is generally not recommended in fistulas.

Solution for Question 4:


Option B: History and clinical examination
• This is the case of an anal fissure, and the patient has a history of pain and bleeding during
defecation. It is an ulcer in the longitudinal axis of the lower anal canal. Commonly it occurs in the
midline posteriorly (more common in males), but can also occur in the midline anteriorly (more common
in females)
• Anal fissure, a tear in the posterior midline, is commonly presented with pain during or after bowel
movements; constipation, a low dietary fiber diet, opioids, and smoking increases the risk for anal
fissure.
• An anal fissure is diagnosed based on history and clinical examination.
• Management includes a high-fiber diet and commonly consuming water, sitz bath, and stool softeners.
Option A: Anoscopy
• Anoscopy is indicated when a clear cause of anal fissure is unknown, or symptoms persist after
adequate treatment for anal fissure.
• Anoscopy is indicated in hemorrhoids, or causes of perianal pain are unknown.
Option C: PR examination
• PR examination is not recommended for anal fissures as it causes more pain during the examination
and may increase the tear of the anal fissure.
• PR examination is recommended for benign prostatic hyperplasia, internal hemorrhoids, anal
condyloma, and fecal incontinence.
Option D: Ultrasound
• There is no association of ultrasound findings with anal fissures.
• However, video-assisted sigmoidoscopy can be used for anal fissure patients younger than 50.

Page 12

800
• Ultrasound is used for genital and prostate problems, breast lumps, gallstones etc.

Solution for Question 5:


Option C: Cutting seton and draining seton
• Frequent anal abscesses, and pus discharge from a site near the anal opening suggest a
fistula-in-ano.
• Setons are used in the surgical treatment of anal fistulas, significantly cutting and draining setons.
• A seton is a ligature of silk, nylon, silastic, or linen. It is used in making, draining, cutting, or staging
the fistula.
• A high fistula is converted into a low fistula with the help of setons.
• Setons are used in managing complex anorectal fistulas, fistulas in Crohn's disease,
immunocompromised and incontinent patients, patients with chronic diarrhea, and anterior fistulas in
women.
• Cutting and draining seton are widely used in treating highly complex anal fistula.
• Cutting seton can damage an anorectal ring; however, draining seton can better protect the sphincter.
Option A: Dissecting seton and draining seton
• A dissecting seton is used in a new technique for creating a filtering pathway two weeks after
trabeculectomy (glaucoma surgery).
Option B: Cutting seton and dissolving seton
• Low anal fistulas can safely be treated by fistulotomy. However, high anal fistulas need surgical repair
by using setons, mainly cutting and draining setons.
• These setons remain in their place for about six weeks and are then removed after the desired result
is obtained. However, these setons don't dissolve on their own.
Option D: Permanent seton
• Cutting and draining setons are usually placed in anal fistulas temporarily for about six weeks and
removed after the drainage.

Solution for Question 6:


Option C: Juvenile rectal polyp
• An afebrile anemic toddler with blood drops coming out of the rectum and the death of a close relative
due to gastrointestinal tract cancer are suggestive of a Juvenile rectal polyp.
• A genetic mutation in BMPR1A and SMAD4 genes causes juvenile Rectal Polyposis Syndrome.
• Patients develop symptoms of rectal bleeding along with abdominal pain and diarrhea.
• JPS is inherited in an autosomal dominant pattern and can potentially turn malignant. People with JPS
are at increased risk of suffering from gastrointestinal tract cancer, most commonly colorectal cancer.

Page 13

801
• JPS is diagnosed with colonoscopy and upper endoscopy having any one of the following: More than
five juvenile polyps in the colon or rectum Juvenile polyps in the GI tract anywhere One or more family
members with juvenile polyps
• More than five juvenile polyps in the colon or rectum
• Juvenile polyps in the GI tract anywhere
• One or more family members with juvenile polyps
• There is no proper treatment for JPS; however, endoscopic polypectomy can be done if the polyps are
small
• More than five juvenile polyps in the colon or rectum
• Juvenile polyps in the GI tract anywhere
• One or more family members with juvenile polyps
Other options
Option A: Adenomatous Polyposis Coli
• Adenomatous polyposis coli is caused by a mutation in the tumor-suppressing APC gene and mainly
affects people in their second or third decade of life.
• Familial Adenomatous Polyposis Coli syndrome is mainly asymptomatic until colorectal cancer
occurs. It is associated with a 100% risk of malignancy if left untreated.
• Sigmoidoscopy/colonoscopy shows more significant than 100 (usually hundreds to thousands)
colorectal polyps.
• In FAP, the colon is surgically removed with ileorectal anastomosis to prevent malignant
transformation.
Option B: Piles
• Piles or hemorrhoids are swollen and enlarged veins within the anal canal that might protrude out,
resulting in bleeding per rectum.
• Hemorrhoids are uncommon in children. They are most common in pregnant women due to increased
intraabdominal pressure and obese and old people.
Option D: Dysentery
• Dysentery is an infective intestinal condition resulting in bloody diarrhea and is common in children.
• The child usually suffers from sudden onset of high-grade fever along with hematochezia. It is caused
by either a bacteria (Shigella) or a parasite (Entamoeba).

Solution for Question 7:


Option D: Higher rate of leakage and stoma formation
• Total mesorectal excision is the complete removal of the rectum, mesorectum, regional lymph nodes,
and vessels with the preservation of pelvic nerves and anal sphincter.
• Total mesorectal excision is the gold standard for treating rectal cancer, especially when it involves
the middle and lower third of the rectum.

Page 14

802
• In TME for the middle and lower rectum, the entire mesorectum should be removed. For upper rectal
tumors, TME is done 5-6 cm below the lower margin of the tumor. TME improves the quality of life in
relation to impotence, retrograde ejaculation and urinary incontinence
• This is a complex procedure with difficult low-lying colorectal or coloanal anastomosis. Moreover, it is
associated with an increased risk of anastomotic dehiscence.
• A temporary stoma prevents anastomotic dehiscence episodes until the anastomosis is completely
healed.
• Leakage of the anastomosis after total mesorectal excision often leads to sepsis, increased morbidity,
and permanent stoma formation (end colostomy).
Option A: Decrease in the local recurrence
• Total mesorectal excision is becoming the widely accepted and performed surgery for rectal cancer
excision.
• As this procedure involves the en bloc excision of the rectum, mesorectum, and pararectal lymph
nodal dissection, it is associated with a meager local recurrence rate, especially when combined with
preoperative radiotherapy.
Option B: Decrease the incidence of impotence
• Total mesorectal excision involves sparing pelvic and perineal nerves, preserving the sexual and
bladder function.
• However, as the pelvic organs and the autonomic nerves involved in sexual function are related to the
rectum, they are at slight risk of injury resulting in ejaculatory dysfunction and erectile disorder.
Option C: Decrease the incidence of bladder dysfunction
• Total mesorectal excision is a pelvic and perineal autonomic nerve-sparing technique, resulting in
decreased incidence of bladder dysfunction after surgery.
• If the superior hypogastric plexus is injured, this will result in urinary problems.
• The incidence of urinary and sexual complications following total mesorectal excision has reduced.
However, it is still a recognized complication.

Solution for Question 8:


Option A: 2 cm
• In rectal carcinoma, the distal resection margin is 2 cm to preserve the anal sphincter.
• The distal resection margin is the distal margin of non-cancerous tissue around the tumor that has to
be surgically resected along with the tumor.
• Distal resection margin plays a significant role in rectal surgical oncology because the aim tends to
preserve the anal sphincter and tumor excision.
• With the emerging total mesorectal excision surgery and preoperative chemoradiotherapy, the distal
resection margin for rectal carcinoma is reduced to 2cm, focusing mainly on anal sphincter
preservation.
Other Options

Page 15

803
Options B, C and D are incorrect.

Solution for Question 9:


C. Option C: Delorme procedure
• Case of rectal prolapse, female gender, weak pelvic floor due to multiparity, history of another pelvic
organ prolapse, and constant straining due to constipation are risk factors in the case and since old
age, the Delorme procedure is done
• Delorme procedure: The mucosa is separated from the muscular wall, which is then plicated with
sutures to reduce the prolapse. Excess mucosa is excised while the remaining is anastomosed. This
procedure has a high recurrence rate.
• The mucosa is separated from the muscular wall, which is then plicated with sutures to reduce the
prolapse. Excess mucosa is excised while the remaining is anastomosed.
• This procedure has a high recurrence rate.
• Generally, the abdominal approach has a low recurrence, but higher morbidity than the perineal
approach and is preferred in younger patients.
Case of rectal prolapse, female gender, weak pelvic floor due to multiparity, history of another pelvic or
gan prolapse, and constant straining due to constipation are risk factors in the case and since old age,
the Delorme procedure is done
• The mucosa is separated from the muscular wall, which is then plicated with sutures to reduce the
prolapse. Excess mucosa is excised while the remaining is anastomosed.
• This procedure has a high recurrence rate.

A. Option A: Frykman-Goldberg procedure


• Anterior resection is done along with Marlex rectopexy.
• Good for patients with severe constipation
• The disadvantage of this procedure is the risk of anastomosis leakage.

Page 16

804
Anterior resection is done along with Marlex rectopexy.
Good for patients with severe constipation
B. Option B: Well's procedure
• In Well's procedure or posterior sling rectopexy, the posterior rectal wall is mobilized
• Mesh placed on the lateral aspect of the anterior rectal wall.
• It has a lower rate of stricture formation.
In Well's procedure or posterior sling rectopexy, the posterior rectal wall is mobilized
Mesh placed on the lateral aspect of the anterior rectal wall.
D. Option D: Ripstein procedure
• In the Ripstein procedure or anterior rectopexy, the posterior rectal wall is mobilized and loose mesh
is placed around the anterior wall sutured to the sacrum.
• It has a low recurrence rate, but sling complications are seen.
In the Ripstein procedure or anterior rectopexy, the posterior rectal wall is mobilized and loose mesh is
placed around the anterior wall sutured to the sacrum.
It has a low recurrence rate, but sling complications are seen.

Solution for Question 10:


Option B: May be caused by an underlying neoplasm
• The case described is of an anorectal abscess; the patient is predisposed to the condition due to his
gender, diabetes, and Crohn's disease; lastly, the mass is fluctuant, erythematous, tender, hot, and
located near the anus.
• Anorectal abscess is more common in males, especially those with diabetes, suppressed immune
system, AIDS, and fistula-in-ano.
• It is presented with a short history of perianal pain and tender swelling.
• Underlying neoplasm or Crohn's disease may be the cause.
• Cryptoglandular theory is the cause of infection, the pus moves along the path of least resistance
which may spread downwards to form an anorectal abscess.
• The most common causative organisms include E. coli and Bacteroides.
• Commonly associated with fistula-in-ano, which is suspected when culture yields enteric bacteria.
• Treatment includes drainage of pus by a cruciate-shaped incision along with antibiotics.
• Sites of abscess formation:

Page 17

805
Option A: The most common type is ischiorectal
• The most common type of anorectal abscess is a perianal abscess.
• Ischiorectal is the second most common type.
Option C: An elliptical-shaped incision is given for drainage under anesthesia
• The type of incision most commonly given is cruciate.
• Skin margins are then excised.
Option D: Commonly seen more in the female population
• It is more common in the male population.
• Especially in dark-haired males.

Solution for Question 11:


Option D: 6'o clock
• This is the case of fistula-in-ano presented with discharge, the passage of flatus, and feces from an
opening near the anal orifice, indicating a connection with the rectum/anus.
• The risk factors in this case: male gender, tuberculosis, and a history of anorectal abscess.
• The location of the internal opening is 6 o’clock according to Goodsall's law.
• Goodsall's rule is applied to predict the location of the internal opening.

Page 18

806
• Anterior openings usually have straight tracks hence the location of external and internal openings are
in the same position. For example, the location of the external opening is at 1'o clock position the
location of the internal opening is also at 1'o clock position.
• For example, the location of the external opening is at 1'o clock position the location of the internal
opening is also at 1'o clock position.
• Posterior openings usually follow a Curved track, so the location of external and internal openings are
not the same. For example, if an external opening is at 4,5,6,7,8,'o clock, Internal opening may be at
the 6'o clock position.
• For example, if an external opening is at 4,5,6,7,8,'o clock, Internal opening may be at the 6'o clock
position.
• Exceptions to this rule include multiple external openings and external openings more than 3cm away
from the anal verge; in these cases, the Posterior rule is applied.
• For example, the location of the external opening is at 1'o clock position the location of the internal
opening is also at 1'o clock position.
• For example, if an external opening is at 4,5,6,7,8,'o clock, Internal opening may be at the 6'o clock
position.
Incorrect Options:
Options A, B and C are incorrect.

Solution for Question 12:


Option D: Bascom's procedure
• The case described is of pilonidal sinus and risk factors in this patient are male, hairy, and army
personnel; hence Bascom's procedure is done.
• Pilonidal sinus usually presents with chronic episodic pain, swelling, and discharge in the natal cleft
area overlying the coccyx.

Page 19

807
• It is formed due to tract blockage by dead hair and hair follicles not being found inside the sinus.
• Spontaneous regression is possible with rest, strict hygiene, and broad-spectrum antibiotics.
• Surgery is done in recurrent, symptomatic or chronic cases.
• Pilonidal sinus surgery includes the excision of all the tracts of the pilonidal sinus and the repair of the
wound by a flap or left open. To prevent a recurrence, ensure the entire tract, along with all its
openings, is excised. All flaps are sutured and designed in a way to prevent midline wound.
• Surgical methods:
• Bascom's procedure: Incision made lateral to the midline, sinus cavity accessed to clean out hair and
granulation tissue, Entire tract with all its opening excised, and midline pits are sutured. The lateral
wound is left open.
• Ppost-operative care mostly includes continuous hair removal and hygiene maintenance.
• Recurrence may be due to a part of the tract still being left inside.
Option A: Wait for spontaneous rupture
• Do not usually resolve to own it.
• May spread further or get complicated.
Option B: Local antibiotics
• Local antibiotics may not reach the bacteria due to its walling-off effect in the cases of an abscess.
• It may not reach in concentrations required to kill the infection fully.
Option C: IV antibiotics
• It is not fully effective until the source of infection is addressed.
• Abscess must be drained.

Solution for Question 13:


Option B: Low anterior resection
• In the scenario mentioned above, the tumor growth is more than 2 cm above the anal canal, so low
anterior resection is the definitive surgical procedure.
• Colorectal carcinoma is the fourth most common cause of cancer death. Risk factors: diet, obesity,
smoking, and lack of physical exercise. Around 60% of the affected patients are aged 70 years or older.
The rectum is the most commonly involved site. Early symptoms: bleeding per rectum, tenesmus, and
early morning diarrhea. Surgical excision of the tumor is the mainstay of treatment. The length of a
normal surgical canal is about 4cm. Low anterior resection (LAR) of the rectum involves the radical
excision of the neoplastic growth along with all the surrounding mesorectum, which contains lymph
nodes and blood vessels. Continuity is restored via end-to-end anastomosis. LAR is also known as a
sphincter-saving procedure. Low anterior resection is indicated in most tumors of the middle third of the
rectum and many in the lower third. Any tumor located at or above 2 cm above the anal canal is usually
treated with LAR.
• Risk factors: diet, obesity, smoking, and lack of physical exercise.
• Around 60% of the affected patients are aged 70 years or older.
• The rectum is the most commonly involved site.

Page 20

808
• Early symptoms: bleeding per rectum, tenesmus, and early morning diarrhea.
• Surgical excision of the tumor is the mainstay of treatment. The length of a normal surgical canal is
about 4cm. Low anterior resection (LAR) of the rectum involves the radical excision of the neoplastic
growth along with all the surrounding mesorectum, which contains lymph nodes and blood vessels.
Continuity is restored via end-to-end anastomosis. LAR is also known as a sphincter-saving procedure.
Low anterior resection is indicated in most tumors of the middle third of the rectum and many in the
lower third. Any tumor located at or above 2 cm above the anal canal is usually treated with LAR.
• The length of a normal surgical canal is about 4cm.
• Low anterior resection (LAR) of the rectum involves the radical excision of the neoplastic growth along
with all the surrounding mesorectum, which contains lymph nodes and blood vessels.
• Continuity is restored via end-to-end anastomosis.
• LAR is also known as a sphincter-saving procedure.
• Low anterior resection is indicated in most tumors of the middle third of the rectum and many in the
lower third.
• Any tumor located at or above 2 cm above the anal canal is usually treated with LAR.
• Risk factors: diet, obesity, smoking, and lack of physical exercise.
• Around 60% of the affected patients are aged 70 years or older.
• The rectum is the most commonly involved site.
• Early symptoms: bleeding per rectum, tenesmus, and early morning diarrhea.
• Surgical excision of the tumor is the mainstay of treatment. The length of a normal surgical canal is
about 4cm. Low anterior resection (LAR) of the rectum involves the radical excision of the neoplastic
growth along with all the surrounding mesorectum, which contains lymph nodes and blood vessels.
Continuity is restored via end-to-end anastomosis. LAR is also known as a sphincter-saving procedure.
Low anterior resection is indicated in most tumors of the middle third of the rectum and many in the
lower third. Any tumor located at or above 2 cm above the anal canal is usually treated with LAR.
• The length of a normal surgical canal is about 4cm.
• Low anterior resection (LAR) of the rectum involves the radical excision of the neoplastic growth along
with all the surrounding mesorectum, which contains lymph nodes and blood vessels.
• Continuity is restored via end-to-end anastomosis.
• LAR is also known as a sphincter-saving procedure.
• Low anterior resection is indicated in most tumors of the middle third of the rectum and many in the
lower third.
• Any tumor located at or above 2 cm above the anal canal is usually treated with LAR.
• The length of a normal surgical canal is about 4cm.
• Low anterior resection (LAR) of the rectum involves the radical excision of the neoplastic growth along
with all the surrounding mesorectum, which contains lymph nodes and blood vessels.
• Continuity is restored via end-to-end anastomosis.
• LAR is also known as a sphincter-saving procedure.
• Low anterior resection is indicated in most tumors of the middle third of the rectum and many in the
lower third.
• Any tumor located at or above 2 cm above the anal canal is usually treated with LAR.

Page 21

809
Option A: Observation and follow-up
• Surgical excision will always be the mainstay of colorectal carcinoma treatment.
• Observation and follow-up are a part of management, but in the scenario mentioned above, the best
option is Surgery after a period of chemoradiation.
Option C: Abdominoperineal excision of the rectum
• It is indicated in some tumors of the lower third of the rectum, which are not suitable for
sphincter-saving procedures (LAR).
• It involves the removal of the rectum and anus with a permanent colostomy.
Option D: Hartmann's procedure
• Hartmann's procedure is done for elderly or frail patients with concerns about anal sphincter function
or the viability of anastomosis.
• It is also indicated in emergency situations in the elderly when the patient is in an unstable condition
or has sepsis.
• The rectum is excised via an abdominal incision, and an end colostomy is made.

Solution for Question 14:


Option C: MRI scan
• MRI scan is the investigation of choice for staging in the head, neck, and pelvic malignancies.
• MRI is the investigation of choice in rectal cancer, helping to guide both surgical and oncological
management.
• MRI is the best radiological option to visualize images of soft tissues and organs.
• Hence, the investigation of choice for lymph node staging is an Endorectal MRI scan.
Investigations for CA RECTUM
• Length of proctoscope – 10 cm
• Rigid sigmoidoscope - 25 cm (Can be performed in minor OT by surgeons).
• Flexible sigmoidoscope - 60 cm
• Colonoscope – 160 cm.
• Investigation of choice for diagnosis - Rigid sigmoidoscopy + biopsy.
• Colonoscopy is mandatory for adequate evaluation of the whole colon and to rule out any
synchronous polyp or synchronous malignancy.
• Synchronous (If tumor is detected within 6 months of the diagnosis of the primary tumor) -
simultaneously.
• Metachronous (If the tumor is detected after 6 months of diagnosis of the primary tumor)- later after
surgery.
• Virtual colonoscopy (3D reconstruction colonoscopy using CT): Advantage: can visualize outside of
lumen also Disadvantage: Biopsy of colon cannot be done
• Advantage: can visualize outside of lumen also

Page 22

810
• Disadvantage: Biopsy of colon cannot be done
• In head & neck malignancies & pelvic malignancies, overcrowding of nerves, blood vessels & soft
tissues. So, IOC for the staging of most of head and neck malignancy – MRI
• For T- staging, investigation of choice- TRUS (Transrectal ultrasound).
• Sensitivity of ultrasound is maximum when the distant between organ and probe is minimum.
• Distantly lying nerve, lymph node, and the vessel cannot be differentiated as Sensitivity of ultrasound
decreases if the distance between probe & organ increases.
• For lymph node staging, the investigation of choice is – Endorectal MRI.
• Overall Best investigation for staging- MRI.
• Advantage: can visualize outside of lumen also
• Disadvantage: Biopsy of colon cannot be done
Option A: Transrectal ultrasound
• For T-staging, the investigation of choice is Transrectal ultrasound (TRUS).
• It is also performed for ultrasound-guided needle biopsy of the prostate gland.
Option B: CT scan of the pelvis
• CT scan can also help trace rectal cancer's nodal and organ metastases.
• For soft tissue imaging, MRI is always the best choice.
Option D: Double-contrast Barium enema
• When a full colonoscopy is not possible, a double-contrast barium enema can be performed to rule
out any synchronous polyp or synchronous malignancy.
• Rectal contrast is given, and structures are visualized using X-rays.

Solution for Question 15:


Option C: Concurrent chemoradiation
• In this procedure, chemotherapy and radiation are given simultaneously, or one followed by the other.
• Stage-II anal cancer refers to locally metastasized anal canal cancer with no nodal or other organ
involvement; concurrent chemoradiation is the best management option for patients of anal canal
cancer with local spread.
• In carcinoma anal canal, the primary treatment is chemoradiotherapy, which involves the NIGRO
regimen (CMT). Nigro regimen: Initial radiotherapy(RT) for 3 weeks 3000 rads (30 Gy total) to
perineum and pelvis. Then chemotherapy-5 FU, for 4-5 days; is a radiosensitizer, started on 1st day of
RT as 1000 mg/m2 continuous infusion. Mitomycin C is 15 mg/m2 as a single dose on 1st day of RT.
Later after 3 weeks abdominoperineal resection (APR)
• Nigro regimen: Initial radiotherapy(RT) for 3 weeks 3000 rads (30 Gy total) to perineum and pelvis.
Then chemotherapy-5 FU, for 4-5 days; is a radiosensitizer, started on 1st day of RT as 1000 mg/m2
continuous infusion. Mitomycin C is 15 mg/m2 as a single dose on 1st day of RT. Later after 3 weeks
abdominoperineal resection (APR)
• Initial radiotherapy(RT) for 3 weeks 3000 rads (30 Gy total) to perineum and pelvis.

Page 23

811
• Then chemotherapy-5 FU, for 4-5 days; is a radiosensitizer, started on 1st day of RT as 1000 mg/m2
continuous infusion.
• Mitomycin C is 15 mg/m2 as a single dose on 1st day of RT. Later after 3 weeks abdominoperineal
resection (APR)
• Small marginal tumors are best treated by local excision; radical Surgery is indicated in those with
persistent or recurrent disease following CMT.
• Despite good results with chemotherapy, 20-25 % of patients will have local disease relapse.
• After a thorough assessment, these patients may require radical abdominoperineal resection.
• Nigro regimen: Initial radiotherapy(RT) for 3 weeks 3000 rads (30 Gy total) to perineum and pelvis.
Then chemotherapy-5 FU, for 4-5 days; is a radiosensitizer, started on 1st day of RT as 1000 mg/m2
continuous infusion. Mitomycin C is 15 mg/m2 as a single dose on 1st day of RT. Later after 3 weeks
abdominoperineal resection (APR)
• Initial radiotherapy(RT) for 3 weeks 3000 rads (30 Gy total) to perineum and pelvis.
• Then chemotherapy-5 FU, for 4-5 days; is a radiosensitizer, started on 1st day of RT as 1000 mg/m2
continuous infusion.
• Mitomycin C is 15 mg/m2 as a single dose on 1st day of RT. Later after 3 weeks abdominoperineal
resection (APR)
• Initial radiotherapy(RT) for 3 weeks 3000 rads (30 Gy total) to perineum and pelvis.
• Then chemotherapy-5 FU, for 4-5 days; is a radiosensitizer, started on 1st day of RT as 1000 mg/m2
continuous infusion.
• Mitomycin C is 15 mg/m2 as a single dose on 1st day of RT. Later after 3 weeks abdominoperineal
resection (APR)
Option A: Abdominoperineal resection (APR)
• Patients with local disease relapse may require radical abdominoperineal resection.
• It involves the removal of the rectum and anus with a permanent colostomy.
• In females, it also includes excision of the posterior wall of the vagina in 70% of women and
reconstruction of the perineum using mucocutaneous flaps.
Option B: APR followed by chemoradiation
• Chemoradiation is the primary treatment in patients with stage-II anal canal cancer.
• Abdominoperineal resection (APR) is done in patients with local relapse after chemoradiation.
Option D: Neoadjuvant chemotherapy followed by Surgery
• The best initial treatment for locally advanced disease is chemoradiotherapy.
• Surgery is required if chemoradiation fails in these patients.

Solution for Question 16:


Option B: Stage B
• On MRI, a rectal carcinoma extending to the extra rectal tissue but has no lymph node involvement
belongs to stage B

Page 24

812
• According to Duke's classification, Stage B tumor is of moderate intensity.
• It is associated with around 70% 5-year survival rate.

Duke's classification is a grading system used for rectal carcinomas.


• It consists of 4 grades, A, B, C, and D. C is further subdivided into C1 and C2.
• Grades A and B are associated with a good prognosis, while the rest are associated with a poor
disease prognosis. Grade A: Tumour confined to mucosa Grade B1: Partial penetration of muscularis
propria Grade B2: Full penetration of muscularis propria Grade C1: Partial penetration of muscularis
propria + LN involvement Grade C2: Full penetration of muscularis propria + LN involvement Grade D:
Distant metastasis
• Grade A: Tumour confined to mucosa
• Grade B1: Partial penetration of muscularis propria
• Grade B2: Full penetration of muscularis propria
• Grade C1: Partial penetration of muscularis propria + LN involvement
• Grade C2: Full penetration of muscularis propria + LN involvement
• Grade D: Distant metastasis
• Grade A: Tumour confined to mucosa
• Grade B1: Partial penetration of muscularis propria
• Grade B2: Full penetration of muscularis propria
• Grade C1: Partial penetration of muscularis propria + LN involvement
• Grade C2: Full penetration of muscularis propria + LN involvement
• Grade D: Distant metastasis
Incorrect Options:
Options A, C and D are incorrect.

Solution for Question 17:


Option A: Preoperative chemoradiation followed by surgical resection
• The clinical features are suggestive of a rectal malignancy, which is usually staged using MRI.
The invasion of malignancy through the muscularis propria into the perirectal tissue with 2 enlarged
lymph nodes indicates the use of preoperative chemoradiation followed by surgical resection of the
tumor.
• Preoperative chemoradiation is used to decrease the size of the tumor before it is excised.
• The use of chemoradiation before surgical excision is indicated in the T2-T3 stage in which cancer
has invaded into and beyond the muscularis propria.
Principle of treatment in CA Rectum:
• Stage I malignancy: Surgical resection of malignancy.

Page 25

813
• State II & III malignancy: Neoadjuvant chemoradiation followed by surgical resection (downstage of
tumor & then surgery)
• Stage IV malignancy: Neoadjuvant chemoradiation followed by palliation ± surgical excision.
• TOC- TOTAL MESORECTAL EXCISION. It was given by Bill Heald Significant length of bowel
removal around the tumor. Removal of Surrounding tissues up to the plane between the Mesorectum &
Presacral Fascia known as HEALD’S HOLYPLANE.
• It was given by Bill Heald
• Significant length of bowel removal around the tumor.
• Removal of Surrounding tissues up to the plane between the Mesorectum & Presacral Fascia known
as HEALD’S HOLYPLANE.
• If CA rectum is located > 5 cm above Anal verge – Low Anterior Resection (LAR) (As we have enough
margin)
• If CA rectum is located at or below 5 cm from anal verge - APR (Abdomino Perineal resection aka
Mile’s Procedure)(As there is no sufficient margin)
• It was given by Bill Heald
• Significant length of bowel removal around the tumor.
• Removal of Surrounding tissues up to the plane between the Mesorectum & Presacral Fascia known
as HEALD’S HOLYPLANE.
Option B: Segmental resection
• Segmental resection is indicated in the treatment of tumors with the T1 stage.
• T1 tumors are tumors that have invaded the submucosa.
Option C: Polypectomy
• Polypectomy is indicated in tumors with Tis grade.
• Tis, also known as a tumor in situ, is a tumor grade when the tumor is intraepithelial or has only
invaded the lamina propria.
Option D: Local transanal excision
• It is used in early-stage cancers of T1-T2 grade.
• Local transanal excision is a minimally invasive procedure and doesn't involve an open abdomen
approach.

Solution for Question 18:


Option B: Thiersch procedure
• The complaint of something coming out of the anus is associated with pain, bleeding, and fecal
incontinence in a multiparous woman with a history of perineal trauma (which is a risk factor for rectal
prolapse) is suggestive of rectal prolapse; this is further confirmed by demonstration of full thickness
rectum prolapsed out of the anus.
• Thiersch procedure is an obsolete surgical technique because of anal stenosis.

Page 26

814
• In this procedure, a steel wire, or silastic or nylon tape, is placed around the anal canal to treat
full-thickness rectal prolapse.
• It is done under local anesthesia. Two small incisions are made at the lateral parts of the anal canal.
Silver wire is passed around deeper to these incisions and tied after placing the index finger into the
anal canal. Wire can be removed after 12 months. Polypropylene and nylon are the other materials that
can be used.
• Other complications include chronic perineal sepsis, anal stenosis, and obstructed defecation. Pain
due to wire erosion, incarceration, high recurrence of 50%
Option A: Delorme's procedure
• This procedure consisted of stripping rectal mucosa circumferentially from the rectum over the length
of prolapse and then plication of underlying muscles with a series of sutures.
• It is preferred in patients with short-segment full rectal prolapse, with high recurrence rates of 30%
over 5 years.
Option C: Altemeier’s procedure
• This procedure involves prolapse of the rectum through the anal canal and then performing a
full-thickness resection incorporating any associated colonic prolapse.
• It is usually considered an alternative perineal procedure to Delorme's procedure following
recurrence.
• Altemeier's procedure is associated with poor bowel control with fecal soiling secondary to loss of
rectal reservoir. Recurrence rates are between 0-20%.
Option D: Hartmann’s procedure
• Hartmann’s procedure is used to treat rectal carcinoma.
• It is used in elderly patients in whom there is concern about poor anal sphincter and postoperative
incontinence.

Solution for Question 19:


Option D: Waldeyer’s fascia
• The treatment of rectal carcinoma is usually a rectosigmoid resection, and the layer of fascia
encountered that separates the rectum from the coccyx is Waldeyer's fascia.
• It is associated with the rectum's lower third and separates it from the prostate/vagina anteriorly.
• Waldeyer’s fascia lines the sacrum's anterior aspect, encloses sacral vessels and nerves and
continues anteriorly as parietal fascia covering the entire pelvic cavity.
• It divides the retro rectal space into superior and inferior compartments.
• Recto-sacral fascia / Waldeyer's fascia: Both fascia act as a barrier for rectal metastasis and separate
the rectum from the sacrum.

Page 27

815
Option A: Denonvilliers’ fascia
• It is a surgical landmark used by urologists in intrapelvic surgery.
• It consists of a single layer arising from the fusion of the two walls of the embryological peritoneal
cul-de-sac under embryological and anatomical review.
Option B: Scarpa’s fascia
• It is a membranous layer of the anterior abdominal wall.
• Scarpa's fascia lies below the Camper's fascia and above the external oblique muscle. It is connected
laterally to the aponeurosis of the external oblique muscle. Medially it fades into the linea alba and
pubic symphysis.
Option C: Buck’s fascia
• It is a layer of deep fascia covering the three erectile bodies of the penis.
• It is continuous with the external spermatic fascia in the scrotum and the suspensory ligament of the
penis.

Solution for Question 20:


Option A: Sigmoidoscopy
• In this scenario, sigmoidoscopy is needed to complete the visualization of the anal canal and rectum
to assess the growth found on DRE.
• The usual length of a flexible sigmoidoscope is about 60 cm.
• In the past, the sigmoidoscope was a rigid stainless steel instrument of variable diameter and normally
25 cm in length.
• Disposable plastic instruments have replaced rigid sigmoidoscopes.
• For sigmoidoscopy to be done, the rectum must be empty for proper inspection.
• Sigmoidoscopy can easily be done in an outpatient setting.

Page 28

816
• Flexible sigmoidoscopy is used when it is required to inspect the region proximal to the rectum.
• It is indicated to investigate underlying causes of fresh rectal bleeding or other bowel symptoms when
full visualization of the colon by colonoscopy is not required.
Option B: Proctoscopy
• The length of the proctoscope is about 10 cm.
• This procedure can inspect the anus, anorectal junction, and lower rectum.
• This patient needs complete visualization of the rectum, so sigmoidoscopy is the best answer.
Option C: Colonoscopy
• In this procedure, a flexible tube of about 160 cm is inserted into the anal canal.
• It is indicated for complete views of the anal canal, rectum, and whole colon.
• The laxative is given before the procedure to evacuate the bowel completely and for better
visualization.
Option D: Colposcopy
• Colposcopy involves the examination of the cervix.
• It is used to visualize the cervix, vagina, and vulva, not the vulva, vagina, and cervix as mentioned in
the explanation.

Solution for Question 21:


Option A: Combined radiation and chemotherapy
• This is a case of squamous cell cancer (SCC) of the anal canal present with bleeding per rectum,
pruritus at the anus, discharge from the anus, weight loss, and enlarged groin lymph node. Squamous
cell carcinoma is the most common type. Predisposing causes: Papilloma, irradiation, dermatitis,
long-standing fistula-in-ano.
• Risk factors of SCC: Age more than 50 Human papillomavirus infection (HPV) Human
immunodeficiency virus (HIV) Smoking Immunosuppressed patients (patients on immunosuppressant
for organ transplant) Anal intercourse
• Age more than 50
• Human papillomavirus infection (HPV)
• Human immunodeficiency virus (HIV)
• Smoking
• Immunosuppressed patients (patients on immunosuppressant for organ transplant)
• Anal intercourse
• Treatment: The first-line treatment for squamous cell cancer of the anal canal is combined
chemotherapy and radiation. The current protocol includes fusional 5-FU with mitomycin 'C' or cisplatin
and external beam radiation to the pelvis. The inguinal nodes, pelvis, anus, and perineum should be
included in the radiation fields.
• The first-line treatment for squamous cell cancer of the anal canal is combined chemotherapy and
radiation.

Page 29

817
• The current protocol includes fusional 5-FU with mitomycin 'C' or cisplatin and external beam radiation
to the pelvis.
• The inguinal nodes, pelvis, anus, and perineum should be included in the radiation fields.
• Age more than 50
• Human papillomavirus infection (HPV)
• Human immunodeficiency virus (HIV)
• Smoking
• Immunosuppressed patients (patients on immunosuppressant for organ transplant)
• Anal intercourse
• The first-line treatment for squamous cell cancer of the anal canal is combined chemotherapy and
radiation.
• The current protocol includes fusional 5-FU with mitomycin 'C' or cisplatin and external beam radiation
to the pelvis.
• The inguinal nodes, pelvis, anus, and perineum should be included in the radiation fields.
Other options
Option B: Reassurance and follow-up after six weeks
• The patient is a diagnosed case of squamous cell cancer of the anal canal, which needs proper
treatment with combined chemotherapy and radiation
• Delay in treatment increase the risk of serious complication and mortality
• Follow-ups are advised after treatment to assess for response to treatment.
Option C: Abdominal perineal resection
• Abdominal perineal resection can be performed in case of persistent disease.
• Especially those who have persistent symptoms up to 6 months after treatment, and those who have
local recurrence are recommended abdominoperineal resection (APR)
Option D: Sitz bath and high-fiber diet
• Sitz baths and high-fiber diets are advised for anal fissure patients.
• An anal fissure risk factor is chronic constipation, a diet low in fibers, straining on defecation, and
receptive anal intercourse.

Solution for Question 22:


Correct Option C - High-fiber diet and sitz bath:
• First-degree hemorrhoids, as described in the scenario, are characterized by bleeding without
prolapse. Conservative management is the initial approach for such cases. High-fiber diet helps soften
stools, reducing straining during bowel movements, and sitz baths provide relief by promoting
cleanliness and reducing inflammation in the anal area.

Incorrect Options:

Page 30

818
Option A - Milligan-Morgan open Hemorrhoidectomy: Milligan-Morgan open hemorrhoidectomy involve
s surgical excision and is typically reserved for more severe cases, such as third-degree and fourth-de
gree hemorrhoids with prolapse. It is not the initial management for first-degree hemorrhoids.
Option B - Sclerotherapy with 5% phenol in almond oil: While sclerotherapy is a treatment option, it is g
enerally considered for second-degree and selected patients with third-degree hemorrhoids. It involves
injecting a sclerosing agent to shrink the hemorrhoidal tissue. However, it is not the primary managem
ent for first-degree hemorrhoids.
Option D - Longo’s Stapler hemorrhoidectomy: Longo's Stapler hemorrhoidectomy is associated with le
sser post-operative pain and is suitable for selected patients with third-degree and fourth-degree hemo
rrhoids presenting with prolapse. It is not the initial management for first-degree hemorrhoids and is us
ually considered in more advanced cases.

Solution for Question 23:


Correct Option C - Anal wiring:
• Anal wiring is a management option for rectal prolapse in patients who cannot tolerate long-duration
surgeries. In anal wiring, the anal canal is narrowed to prevent the rectum from prolapsing. This is a
less invasive procedure compared to rectopexy or resection rectopexy and may be suitable for patients
with difficulty tolerating extensive surgeries. It provides a conservative approach to manage rectal
prolapse and can be a viable option in selected cases.

Incorrect Options:
Option A - Rectopexy with mesh fixation to the sacrum: Rectopexy involves fixing the rectum to the sac
rum with mesh. While it is a
valid option for rectal prolapse, it may not be the most appropriate choice for a
patient reporting difficulty tolerating long-duration surgeries.
Option B - Resection rectopexy: Resection rectopexy involves excising the excessive length of the rect
um before fixing it. This is a more extensive surgical procedure and may not be suitable for a
patient with difficulty tolerating long surgeries.
Option D - Conservative management with lifestyle modifications: Conservative management, including
lifestyle modifications, is not the most appropriate option for a patient with recurrent episodes of rectal
prolapse involving the full thickness of the rectum. Conservative measures are generally considered for
milder cases or as part of preoperative and postoperative care rather than as a
primary management strategy for extensive rectal prolapse.

Solution for Question 24:


Correct Option D - Protein energy malnutrition, worm infestation, and diarrhea:
• Rectal prolapse in the given scenario is most likely caused by a combination of protein energy
malnutrition, worm infestation, and recent episodes of diarrhea. Children in rural areas may develop
rectal prolapse due to the straining associated with these conditions. The information provided
highlights the importance of considering the underlying causes, and in this case, the correct answer
reflects the comprehensive understanding of the clinical context.

Page 31

819
Incorrect Options:
Option A - Sacral agenesis: Sacral agenesis is a rare cause of rectal prolapse but is not related to the c
ommon factors mentioned in the patient's history. It involves the absence of the sacrum, leading to insu
fficient support for the rectum.
Option B) - Meningomyelocele: Meningomyelocele is another rare cause and is not directly associated
with protein energy malnutrition, worm infestation, or diarrhea. It involves a
neural tube defect and is unlikely to be the primary cause in this scenario.
Option C - Wells Rectopexy: Wells Rectopexy is a
type of surgical procedure for rectal prolapse and is not a cause of rectal prolapse. It is important to diff
erentiate between causes and management strategies when interpreting clinical information.

Solution for Question 25:


Correct Option D - Pelvi-rectal abscess is the most common type of anorectal abscess:
• Anorectal abscesses are commonly classified based on their location, and the most common types
are perianal and ischiorectal abscesses. Pelvi-rectal abscesses are less common compared to perianal
and ischiorectal abscesses.

Incorrect Options:
Option A - Perianal abscess is the most common type of anorectal abscess: Correct.
Option B - Ischiorectal abscess is the second most common type of anorectal abscess: Correct.
Option C - Infection entering the gland via anal crypts leads to the formation of anorectal abscess (Cry
ptoglandular Theory): Correct.

Solution for Question 26:


Correct Option A - Canoe-shaped Ulcer, Sentinel Pile, and Hypertrophied Papilla:
The characteristic triad seen in chronic fissures-in-ano includes:
• Canoe-shaped Ulcer: Describing the shape of the ulceration.
• Sentinel Pile (Sentinel Tag): Refers to a skin tag located near the fissure.
• Hypertrophied Papilla: Indicates an enlargement of a specific structure.

Incorrect Options:
Option B - Circular Ulcer, Fissure Tag, and Atrophied Papilla: The term "Circular Ulcer" and "Fissure T
ag" do not align with the characteristics of chronic fissures. Additionally, the term "Atrophied Papilla" co
ntradicts the hypertrophied nature seen in chronic fissures.
Option C - Linear Ulcer, Sentinel Pile, and Hypertrophied Papilla: The term "Linear Ulcer" does not mat
ch the characteristics of chronic fissures, which are typically longitudinal. The other components are co

Page 32

820
rrect.
Option D - Horse-shoe Ulcer, Sentinel Tag, and Atrophied Papilla: The term "Horse-shoe Ulcer" is not
consistent with the typical appearance of fissures. Additionally, the use of "Atrophied Papilla" is inconsi
stent with the hypertrophied nature seen in chronic fissures.

Solution for Question 27:


Correct Option D - Lord’s procedure – manual dilatation of sphincter:
• Lord’s procedure involves manual dilatation of the anal sphincter and is usually considered a surgical
intervention for chronic anal fissures. It is not typically performed in the initial conservative or medical
treatment.

Incorrect Options:
Option A - Sitz bath with 1% potassium permanganate: Sitz bath with 1% potassium permanganate is
part of the initial conservative management for chronic anal fissures. It helps in promoting hygiene and
reducing inflammation.
Option B - NOTARA’S lateral sphincterotomy: NOTARA’S lateral sphincterotomy is a surgical procedur
e and is not considered part of the initial conservative or medical treatment. It is usually reserved for ca
ses where conservative and medical measures have failed.
Option C - Botox injection into sphincters: Botox injection into sphincters is a medical treatment option f
or chronic anal fissures. It involves the injection of botulinum toxin to relax the anal sphincter, promotin
g healing of the fissure.

Solution for Question 28:


Correct Option B - Single opening in the natal cleft; Limberg’s Flap:
• Pilonidal sinus is characterized by a single opening in the natal cleft, usually over the coccyx.
Limberg’s Flap is commonly used for closure during the surgical management of pilonidal sinus.

Incorrect Options:
Option A - Multiple openings along the natal cleft; Karydaki's flap: Pilonidal sinus is characterized by a
single opening, not multiple openings along the natal cleft. Karydaki's Flap is not commonly used for cl
osure in the context of pilonidal sinus.
Option C - Presence of hair collection in the perianal region; Rhomboid Flap: Pilonidal sinus is associat
ed with the collection of dead hair in the natal cleft overlying the coccyx, not in the perianal region. Rho
mboid Flap is used for closure in pilonidal sinus surgery.
Option D - Intermittent serous discharge in the axillary region; BASCOM procedure:
Intermittent serous discharge is not a characteristic feature of pilonidal sinus, and it is not associated wi
th the axillary region. The BASCOM procedure is a
surgical treatment for pilonidal sinus but is not related to intermittent serous discharge.

Page 33

821
Page 34

822
Hernia and Abdominal Wall
1. What would be the management of a middle-aged female with three finger test revealing positive
cough impulse in the femoral region?
A. Bassini repair
B. Hunters repair
C. Shouldice repair
D. McVay repair
----------------------------------------
2. What is the management of hydrocele of the hernial sac in 3-year-old child?
A. Herniotomy
B. Herniorrhaphy
C. Observation only
D. Operate after 5 years of age
----------------------------------------
3. What type of hernia is a funicular hernia?
A. Direct inguinal hernia
B. Indirect inguinal hernia
C. Femoral hernia
D. Umbilical hernia
----------------------------------------
4. Which among the following is true regarding the examination of patient with funicular type of indirect
inguinal hernia?
A. A cough impulse is felt at the region of the deep inguinal ring
B. The internal abdominal ring lies 1.25 cm below the midpoint of the poupart's ligament
C. The external abdominal ring lies 1.25 cm above and medial to ASIS
D. None of the above
----------------------------------------
5. Which technique of hernia repair uses Pascal's law in an elderly female with an incisional hernia who
is being posted for hernioplasty?
A. Lichtenstein mesh repair
B. Stoppa preperitoneal repair
C. Bassini’s repair
D. Darning repair
----------------------------------------
6. Which of the following hernia is associated with hydrocele?
A. Gibbon’s hernia

823
B. Berger’s hernia
C. Beclard’s hernia
D. Stammer’s hernia
----------------------------------------
7. Which method of reduction can be employed in an individual with an irreducible indirect inguinal
hernia?
A. Kugel procedure
B. Taxis procedure
C. McVay procedure
D. Stoppa technique
----------------------------------------
8. What type of hernia is most common in females?
A. Femoral hernia
B. Indirect inguinal hernia
C. Direct Inguinal Hernia
D. Sliding hernia
----------------------------------------
9. Where is the defect and opening in a laugier's hernia?
A. Lacunar ligament
B. Conjoint tendon
C. External oblique
D. None of the above
----------------------------------------
10. Which of the following statements is correct regarding indirect inguinal hernia repair?
A. Shouldice repair is a four-layered repair
B. Most commonly injured nerve in open hernia repair is the femoral nerve
C. Most commonly injured nerve in laparoscopic hernia repair is the ilioinguinal nerve
D. Shouldice repair has a high recurrence rate
----------------------------------------
11. Which of the following statements is correct regarding landmarks of laparoscopic repair of indirect
inguinal hernia?
A. Triangle of Doom: External iliac vessels are content
B. Triangle of pain: Electric-friendly zone
C. Crown of death: Aberrant inferior epigastric artery
D. Quadrangle of Doom and pain: Triangle of Doom plus triangle of death
----------------------------------------

Page 2

824
12. Which of the following approaches is preferred in a middle-aged man with unilateral indirect inguinal
hernia with the bowel as a content?
A. Bassini’s
B. Lotheissen
C. McVay procedure
D. Mayo repair
----------------------------------------
13. Which of the following hernias is related to the abdominal wall?
A. Richter’s hernia
B. Hiatus Hernia
C. Petersen hernia
D. Serafini's hernia
----------------------------------------
14. What is type IIIA in the Nyhus classification of hernias?
A. Direct inguinal hernia
B. Indirect inguinal hernia
C. Femoral hernia
D. Femoral recurrent hernia
----------------------------------------
15. Which of the following is the risk factor for direct inguinal hernia among the options given below?
A. Chronic Diarrhea
B. Underweight
C. Cigarette smoking
D. None of the above
----------------------------------------
16. Which of the following statements is correct about the content in a direct inguinal hernia, with
doughy consistency, with the first part easy to reduce and the latter part difficult during surgery?
A. Content is the omentum
B. It is an enterocele
C. It is a rectocele
D. It is a cystocele
----------------------------------------
17. Which of the following statements is correct about femoral hernia?
A. More common in males
B. Lockwood operation is the treatment of choice
C. In Cloquet’s hernia, the sac lies down the pectineus fascia

Page 3

825
D. Should be kept for observation without surgery
----------------------------------------
18. Which of the following is a true statement regarding Spigelian hernia?
A. It occurs exclusively in males
B. It involves part of the circumference of the bowel wall
C. It is best repaired by the classical Bassini technique of inguinal ligament repair
D. It occurs at the lateral edge of the linea semilunaris
----------------------------------------
19. Which of the following types of indirect inguinal hernia has caecum as a part of the wall of the
hernia sac?
A. Richter’s hernia
B. Spigelian hernia
C. Sliding hernia
D. Epigastric hernia
----------------------------------------
20. Which of the following statements is correct about the lumbar hernia given in the picture below?

A. Is always congenital
B. Hernia through Grynfeltt triangle is less common
C. Petit’s triangle is bounded by the iliac crest, latissimus dorsi, and external oblique muscle
D. Grynfeltt triangle is bounded by the 11th rib, paraspinal, and external oblique muscle
----------------------------------------
21. Which of the following hernias is responsible for causing pain in the epigastric region and positive
cough impulse in a person with no history of abdominal surgery?
A. Umbilical hernia
B. Fatty hernia of the linea alba
C. Incisional hernia
D. Inguinal hernia
----------------------------------------

Page 4

826
22. Which technique of hernial repair is done at point A in the given image?

A. Onlay
B. Retromuscular
C. Inlay
D. Preperitoneal
----------------------------------------
23. Which of the following is characteristic of omphalocele disorder found in a newborn?
A. Not covered by a sac
B. A defect in the abdominal fascia
C. Associated with Intestinal atresia
D. Associated with congenital malformations
----------------------------------------
24. Which of the following structures is found in the inguinal canal?
A. Spermatic cord
B. Femoral nerve
C. Obturator nerve
D. Pudendal nerve
----------------------------------------
25. A 49-year-old female was scheduled for herniorrhaphy. During the surgery, the surgeon observed
that the shape of the hernia was in a “W” shape, as shown in the image below. What is the name of the
hernia?

Page 5

827
A. Richter's hernia
B. Littre's hernia
C. Pantaloon hernia
D. Maydl's hernia
----------------------------------------

Correct Answers
Question Correct Answer

Question 1 4
Question 2 1
Question 3 2
Question 4 1
Question 5 2
Question 6 1
Question 7 2
Question 8 2
Question 9 1
Question 10 1
Question 11 1
Question 12 1
Question 13 1
Question 14 1
Question 15 3
Question 16 1
Question 17 3
Question 18 4
Question 19 3

Page 6

828
Question 20 3
Question 21 2
Question 22 1
Question 23 4
Question 24 1
Question 25 4

Solution for Question 1:


Option D: McVay repair
• Given the clinical scenario and examination findings, the patient most likely suffers from an inguinal
hernia and femoral ring defects.
• In such cases, McVay repair is indicated.
• McVay repair addresses both inguinal and femoral ring defects.
• The McVay repair uses autologous tissue to close the inguinal canal floor.
• McVay repair is shown in the picture below:

• It is indicated for femoral hernia and where prosthetic material is contraindicated.


• After reaching the preperitoneal space, the superior transversalis flap is fastened to the Cooper’s
ligament to occlude the femoral ring.
Option A: Bassini repair
• This procedure is done for inguinal hernia only. It is a triple-layer repair.
• Internal oblique, transversus abdominis, and transversalis fascia are fixed to the inguinal ligament and
pubic periosteum with interrupted suture.
Option B: Hunter’srepair
• Hunter's repair does flexor tendon reconstruction.

Page 7

829
• It is not indicated in treating inguinal and femoral ring defects.
Option C: Shouldice repair
• This is indicated for inguinal hernia repair.
• It is a four-layer repair for an inguinal hernia.

Solution for Question 2:


Option A: Herniotomy
• Herniotomy is the procedure of choice for the congenital hydrocele.
• A hydrocele describes the accumulation of fluid within the tunica vaginalis.
Congenital hydrocele:
• The processus vaginalis is patent in congenital hydrocele and connects with the peritoneal cavity.
• The communication is usually too small to allow herniation to intra-abdominal contents.
• Spontaneous obliteration occurs at 2 years of age
• If not obliterated, then the management of hydrocele of the hernia sac (congenital hydrocele) is
herniotomy.
Steps of herniotomy:
• Inguinal skin crease incision
• Division of Subcutaneous fat
• Division of Camper's fascia, Scarpa's fascia
• Incision along direction of fibres n external oblique aponeurosis
• Isolate the Sac from cord
• Open the Sac at fundus
• Invert the contents back to the peritoneal cavity
• Apply PURSE- STRING suture over the sac
• Excise the Redundant Sac
Option B: Herniorrhaphy
• Herniorrhaphy is the removal of the hernial sac plus mesh repair of the posterior wall of the inguinal
canal.
• Mesh is not used because there will be growth in the body compared to mesh.
Option C: Observation only
• If the hydrocele persists beyond the age of 1 year, or if it becomes symptomatic or significantly
enlarges, surgical intervention may be considered.
• With time, the communication may become large to allow herniation of intra-abdominal contents; it
needs to be managed.
Option D: Operate after 5 years of age

Page 8

830
• Infantile hydroceles are generally repaired if they do not resolve spontaneously by 1-2 years of age.
• The management of congenital hydrocele of the hernia sac is herniotomy.

Solution for Question 3:


Correct option B - Indirect inguinal hernia:
• The funicular is a type of indirect inguinal hernia.
• An indirect inguinal hernia occurs as a failure of the processus vaginalis to close.
• It passes lateral to the inferior epigastric artery.
• Types of indirect inguinal hernia: Bubonocele A hernia is limited to the inguinal canal. Funicular The
processus vaginalis is closed just above the epididymis. Contents of the sac can be felt separately from
the testis, which lies below the hernia. Complete or scrotal The processus vaginalis is patent
throughout. The hernia descends down to the bottom of the scrotum and it is difficult to differentiate the
testis from hernia.
• Bubonocele A hernia is limited to the inguinal canal.
• A hernia is limited to the inguinal canal.
• Funicular The processus vaginalis is closed just above the epididymis. Contents of the sac can be felt
separately from the testis, which lies below the hernia.
• The processus vaginalis is closed just above the epididymis.
• Contents of the sac can be felt separately from the testis, which lies below the hernia.
• Complete or scrotal The processus vaginalis is patent throughout. The hernia descends down to the
bottom of the scrotum and it is difficult to differentiate the testis from hernia.
• The processus vaginalis is patent throughout.
• The hernia descends down to the bottom of the scrotum and it is difficult to differentiate the testis from
hernia.
• Bubonocele A hernia is limited to the inguinal canal.
• A hernia is limited to the inguinal canal.
• Funicular The processus vaginalis is closed just above the epididymis. Contents of the sac can be felt
separately from the testis, which lies below the hernia.
• The processus vaginalis is closed just above the epididymis.
• Contents of the sac can be felt separately from the testis, which lies below the hernia.
• Complete or scrotal The processus vaginalis is patent throughout. The hernia descends down to the
bottom of the scrotum and it is difficult to differentiate the testis from hernia.
• The processus vaginalis is patent throughout.
• The hernia descends down to the bottom of the scrotum and it is difficult to differentiate the testis from
hernia.
• A hernia is limited to the inguinal canal.
• The processus vaginalis is closed just above the epididymis.

Page 9

831
• Contents of the sac can be felt separately from the testis, which lies below the hernia.
• The processus vaginalis is patent throughout.
• The hernia descends down to the bottom of the scrotum and it is difficult to differentiate the testis from
hernia.
Incorrect options:
Option A - Direct inguinal hernia:
• Direct inguinal hernia occurs due to a defect in the transversalis fascia.
• It protrudes through the Hesselbach triangle and passes medially to the inferior epigastric artery.
Option C - Femoral hernia:
• It protrudes below the inguinal ligament, lateral to the pubic tubercle.
• It is more common in females and has a high risk of strangulation.
Option D - Umbilical hernia:
• An umbilical hernia can occur when fatty tissue or a part of the bowel protrudes into an area near the
umbilicus.
• In adults, being overweight can contribute to developing an umbilical hernia.

Solution for Question 4:


Option A: A cough impulse is felt at the region of the deep inguinal ring
• The classical examination feature of Indirect inguinal hernia is, on examination, a cough impulse is felt
at the region of the deep inguinal ring.
Indirect Inguinal hernia:
• Indirect Inguinal hernia occurs as a failure of the processus vaginalis to close and passes lateral to the
inferior epigastric artery.
• The processus vaginalis is closed just above the epididymis.
• Contents of the sac can be felt separately from the testis, which lies below the hernia.
• Clinical features: Males are 20 times more commonly affected than females. The patient complains of
pain in the groin and referred pain in the testicle when performing heavy work or strenuous exercise.
When asked to cough, a small transient bulging may be seen and felt with an expansile impulse. For an
indirect hernia, the impulse is felt at the deep inguinal ring, whereas for a direct inguinal hernia, the
impulse is felt at the superficial inguinal ring.
• Males are 20 times more commonly affected than females.
• The patient complains of pain in the groin and referred pain in the testicle when performing heavy
work or strenuous exercise.
• When asked to cough, a small transient bulging may be seen and felt with an expansile impulse.
• For an indirect hernia, the impulse is felt at the deep inguinal ring, whereas for a direct inguinal hernia,
the impulse is felt at the superficial inguinal ring.
• Males are 20 times more commonly affected than females.

Page 10

832
• The patient complains of pain in the groin and referred pain in the testicle when performing heavy
work or strenuous exercise.
• When asked to cough, a small transient bulging may be seen and felt with an expansile impulse.
• For an indirect hernia, the impulse is felt at the deep inguinal ring, whereas for a direct inguinal hernia,
the impulse is felt at the superficial inguinal ring.

Option B: The internal abdominal ring lies 1.25 cm below the midpoint of the poupart's ligament
• In the case of an indirect inguinal hernia, the internal abdominal ring lies 1.25 cm above the midpoint
of the poupart’s ligament.
Option C: The external abdominal ring lies 1.25 cm above and medial to ASIS
• The external abdominal ring lies 1.25 cm above and medial to anterior superior iliac spine (ASIS) in
case of direct inguinal hernia.
• The external abdominal ring lies 1.25 cm above and medial to the pubic tubercle in an indirect inguinal
hernia.
Option D: None of the above
• For an indirect hernia, the impulse is felt at the deep inguinal ring
• For a direct inguinal hernia, the impulse is felt at the superficial inguinal ring.

Solution for Question 5:


Option B: Stoppa preperitoneal repair
• Stoppa preperitoneal repair is shown in the figure below:

• This technique involves placing prosthetic mesh in the extraperitoneal position in the preperitoneal
space or retro rectus position.

Page 11

833
• In Stoppa’s preperitoneal repair, the expanding intra-abdominal pressure holds the mesh in place
without suture fixation.
• According to Pascal’s law, any additional pressure exerted upon an enclosed fluid mass is transmitted
equally in all directions.
• Using a large mesh extending 3-5 cm beyond the edge of a defect on all sides utilizes Pascal's law.
Option A: Lichtenstein mesh repair
• It is a tension-free mesh repair, an example of hernioplasty and is currently one of the most common
open inguinal hernia repair techniques.
• This procedure leads to the opening of the subcutaneous fat along the incision line.
Option C: Bassini’s repair
• Bassini’s technique is used for inguinal hernia repair.
• It involves suturing the transversalis fascia and the conjoined tendon to the inguinal ligament.
Option D: Darning repair
• A darning inguinal hernia repair is a pure-tissue tensionless technique.
• It is performed by placing a continuous suture between the conjoined tendon and the inguinal ligament
without approximation structures.

Solution for Question 6:


Option A: Gibbon’s hernia
• Hernia with hydrocele is called Gibbon's hernia.
• Gibbon’s hernia is a hernia with hydrocele. Indirect Inguinal hernia occurs as a failure of the
processus vaginalis to close, and it protrudes through the inguinal ring. The opening of processus
vaginalis should normally close on its own during development, but it remains open, and parts of the
abdomen (bowel and fat) can enter the sac. If the opening is only wide enough to let in fluid, this
creates a hydrocele. Presentation of hernia along with hydrocele gives rise to Gibbon's hernia.
• Indirect Inguinal hernia occurs as a failure of the processus vaginalis to close, and it protrudes through
the inguinal ring.
• The opening of processus vaginalis should normally close on its own during development, but it
remains open, and parts of the abdomen (bowel and fat) can enter the sac.
• If the opening is only wide enough to let in fluid, this creates a hydrocele.
• Presentation of hernia along with hydrocele gives rise to Gibbon's hernia.
• Indirect Inguinal hernia occurs as a failure of the processus vaginalis to close, and it protrudes through
the inguinal ring.
• The opening of processus vaginalis should normally close on its own during development, but it
remains open, and parts of the abdomen (bowel and fat) can enter the sac.
• If the opening is only wide enough to let in fluid, this creates a hydrocele.
• Presentation of hernia along with hydrocele gives rise to Gibbon's hernia.
Option B: Berger’s hernia

Page 12

834
• It is a hernia in the pouch of Douglas.
• It does not present with hydrocele.
Option C: Beclard’s hernia
• It refers to a femoral hernia through the opening of the saphenous vein.
• It does not lead to the formation of a hydrocele.
Option D: Stammer’s hernia
• It refers to an internal hernia occurring through a window in the transverse mesocolon after retro colic
gastrojejunostomy.
• Hydrocele is not a feature of Stammer's hernia.

Solution for Question 7:


Option B: Taxis procedure
• In the case mentioned above, the patient has inguinoscrotal swelling consistent with an irreducible
inguinal hernia.
• The management of reduction of irreducible inguinal hernia is called taxis procedure
• This is shown in the figure below:

• Taxis should be attempted for incarcerated hernias without sequelae of strangulation.


• It should not be performed when strangulation is suspected as reduction of potentially gangrenous
tissue into the abdomen may result in an intraabdominal catastrophe.
Option A: Kugel procedure
• Kugel procedure is a classic preperitoneal herniorrhaphy via the posterior approach for inguinal
hernias.
• It has a steep learning curve and is not indicated in reducing irreducible hernia.

Page 13

835
Option C: McVay procedure
• This procedure addresses both inguinal and femoral ring defects. It is indicated for femoral hernia and
where prosthetic material is contraindicated.
• It is also contraindicated in an irreducible hernia.
Option D: Stoppa technique
• This technique involves placing prosthetic mesh in the extraperitoneal position in the preperitoneal
space or retro rectus position.
• In Stoppa’s preperitoneal repair, the expanding intra-abdominal pressure holds the mesh in place
without suture fixation and is not indicated in the irreducible hernia.

Solution for Question 8:


Option B: Indirect inguinal hernia
• The most common type of hernia in females is an indirect inguinal hernia. In females, the ovaries and
the parts of the reproductive system may slide through the opening and cause an indirect inguinal
hernia.
• An indirect Inguinal hernia occurs as a failure of the processus vaginalis to close, and it protrudes
through the inguinal ring. It passes lateral to the inferior epigastric artery.
• It is shown below:

Option A: Femoral hernia


• The femoral hernia protrudes below the inguinal ligament, lateral to the pubic tubercle.
• It has a high risk of strangulation. Femoral hernia is more common in females than males, but the
most common is an indirect inguinal hernia in females.
Option C: Direct inguinal hernia

Page 14

836
• Direct inguinal hernia occurs as a defect or weakness in the transversalis fascia area of the
Hesselbach triangle and protrudes through the Hesselbach triangle and passes medial to the inferior
epigastric artery.
• It is more common in the elderly.
Option D: Sliding hernia
• More common in males.
• Posterior wall of the sac is formed by viscera
• Increased risk of bowel injury in surgery during ligation of the sac.

Solution for Question 9:


Option A: Lacunar ligament
• Laugier’s hernia is a hernia that occurs through a gap in the lacunar (Gimbernat's) ligament.
• Diagnosis is based on the unusual medial position of a small femoral hernia sac.
• It is always strangulated.
• The treatment is Herniorrhaphy.
• Incision of the ligament is necessary for the release of strangulation.
• The repair is required to close the gap in the lacunar ligament.
• The femoral ring is also closed as a precautionary measure and to facilitate the repair of the lacunar
ligament.
Option B: Conjoint tendon
• Direct inguinal hernia protrudes through the Hesselbach triangle and presents medial to the inferior
epigastric artery.
• There is a defect or weakness in the transversalis fascia (Conjoint tendon) area of the Hesselbach
triangle. It has a low risk of strangulation and is more common in males.
Option C: External oblique
• An indirect inguinal hernia protrudes through the inguinal ring and passes lateral to the inferior
epigastric artery.
• There is the failure of the processus vaginalis to close. It has a low risk of strangulation.
Option D: None of the above
• The case mentioned above is Laugier's hernia, which occurs through a gap in the lacunar
(Gimbernat's) ligament.

Solution for Question 10:


Option A: Shouldice repair is a four-layered repair

Page 15

837
• Clinical signs: swelling in the inguinoscrotal region and negative cough impulse suggest the indirect
inguinal hernia, which is not an emergency until it is obstructive.
• Shouldice repair: Four-layered repairs: Double breasting of fascia transversalis, transversus
abdominis, and internal oblique. The transversalis fascia is incised by a central incision from the
deep inguinal ring to the pubic tubercle and then closed to create a double thick two-layered posterior
wall (double breasting). Less tension means less risk of recurrence. This repair is done under local
anesthesia. Upper and lower flaps are made by cutting the transversalis fascia from the internal ring
laterally to the pubic tubercle medially.
• Four-layered repairs: Double breasting of fascia transversalis, transversus abdominis, and internal
oblique. The transversalis fascia is incised by a central incision from the deep inguinal ring to the pubic
tubercle and then closed to create a double thick two-layered posterior wall (double breasting).
• Double breasting of fascia transversalis, transversus abdominis, and internal oblique.
• The transversalis fascia is incised by a central incision from the deep inguinal ring to the pubic
tubercle and then closed to create a double thick two-layered posterior wall (double breasting).
• Less tension means less risk of recurrence.
• This repair is done under local anesthesia.
• Upper and lower flaps are made by cutting the transversalis fascia from the internal ring laterally to the
pubic tubercle medially.
• Four-layered repairs: Double breasting of fascia transversalis, transversus abdominis, and internal
oblique. The transversalis fascia is incised by a central incision from the deep inguinal ring to the pubic
tubercle and then closed to create a double thick two-layered posterior wall (double breasting).
• Double breasting of fascia transversalis, transversus abdominis, and internal oblique.
• The transversalis fascia is incised by a central incision from the deep inguinal ring to the pubic
tubercle and then closed to create a double thick two-layered posterior wall (double breasting).
• Less tension means less risk of recurrence.
• This repair is done under local anesthesia.
• Upper and lower flaps are made by cutting the transversalis fascia from the internal ring laterally to the
pubic tubercle medially.
• Double breasting of fascia transversalis, transversus abdominis, and internal oblique.
• The transversalis fascia is incised by a central incision from the deep inguinal ring to the pubic
tubercle and then closed to create a double thick two-layered posterior wall (double breasting).
Option B: Most commonly injured nerve in open hernia repair is the femoral nerve
• The most commonly injured nerves in open hernia repair are the iliohypogastric, ilioinguinal, and
genitofemoral nerve branches.
• The femoral nerve is far from this hernia repair.
Option C: Most commonly injured nerve in laparoscopic hernia repair is the ilioinguinal nerve
• The most commonly injured nerve in laparoscopic hernia repair - lateral cutaneous nerve and
genitofemoral nerve
Option D: Shouldice repair has a high recurrence rate
• Shouldice repair has a low recurrence rate due to relatively low tension in tissues

Page 16

838
Solution for Question 11:
Option A: Triangle of Doom: External iliac vessels are content
• Clinical signs: left inguinal bulge and negative cough impulse indicate an indirect inguinal hernia,
which is repaired either openly or laparoscopically.
• Triangle of Doom: Boundaries: Medially: Vas deferens Laterally: Gonadal vessels Base: Fold of
peritoneum Apex: Deep ring Contents: External iliac vessels
• Boundaries: Medially: Vas deferens Laterally: Gonadal vessels Base: Fold of peritoneum Apex: Deep
ring
• Medially: Vas deferens
• Laterally: Gonadal vessels
• Base: Fold of peritoneum
• Apex: Deep ring
• Contents: External iliac vessels
• Boundaries: Medially: Vas deferens Laterally: Gonadal vessels Base: Fold of peritoneum Apex: Deep
ring
• Medially: Vas deferens
• Laterally: Gonadal vessels
• Base: Fold of peritoneum
• Apex: Deep ring
• Contents: External iliac vessels
• Medially: Vas deferens
• Laterally: Gonadal vessels
• Base: Fold of peritoneum
• Apex: Deep ring

Page 17

839
Page 18

840
Option B: Triangle of pain: Electric-friendly zone
• It is a hazardous electric zone.
• This triangle holds the lateral femoral cutaneous nerve, the femoral branch of the genitofemoral nerve,
and the femoral nerve.
Option C: Crown of death: Aberrant inferior epigastric artery
• Corona Mortis is a variant vascular anastomosis between the external iliac artery or inferior epigastric
artery and obturator artery; located posterior to the superior pubic ramus, close to the quadrilateral
plate of the acetabulum.
Option D: Quadrangle of Doom and pain: Triangle of Doom plus triangle of death
• Quadrangle of Doom and pain: Triangle of Doom plus triangle of pain.

Solution for Question 12:


Option A: Bassini’s
• The patient mentioned above has an indirect inguinal hernia, and Bassini's technique is most suitable
in this case.
• Bassini’s technique for inguinal hernia repair involves suturing the transversalis fascia and the
conjoined tendon to the inguinal ligament.
• It is shown below:

Page 19

841
Bassini’s
TEP (Totally extraperitoneal)
Option B: Lotheissen
• It is the inguinal approach to femoral hernia repair.
• Types of femoral hernia repair: Low inguinal operation (Lockwood) [simplest but suitable only when
there is no risk of bowel resection]. Inguinal operation (Lotheissen) High inguinal operation (McEvedy)
[ideal in the emergency where the risk of bowel strangulation is high].
• Low inguinal operation (Lockwood) [simplest but suitable only when there is no risk of bowel
resection].
• Inguinal operation (Lotheissen)
• High inguinal operation (McEvedy) [ideal in the emergency where the risk of bowel strangulation is
high].
• Low inguinal operation (Lockwood) [simplest but suitable only when there is no risk of bowel
resection].
• Inguinal operation (Lotheissen)
• High inguinal operation (McEvedy) [ideal in the emergency where the risk of bowel strangulation is
high].
Option C: McVay procedure
• This procedure addresses both inguinal and femoral ring defects.
• It is indicated for femoral hernia repair and in cases where the use of prosthetic material is
contraindicated.
Option D: Mayo repair
• Mayo repair technique is used in Umbilical hernia.
• This technique is not usually performed because of increased tension associated with increased
recurrence.

Page 20

842
Solution for Question 13:
Option A: Richter’s hernia
• In a Richter hernia, a small portion or circumference of the anti-mesenteric wall of the intestine is
trapped within the hernia, and strangulation can occur without intestinal obstruction.
• Herniation of part of the circumference of the bowel mimics acute gastroenteritis.
• They are most commonly diagnosed in 60 to 80 years of age.
• They comprise up to 10 percent of all strangulated hernias.
• Clinical features: Colicky pain Diarrhea Delay in diagnosis leads to a risk of strangulation
• Colicky pain
• Diarrhea
• Delay in diagnosis leads to a risk of strangulation
• Midline laparotomy is typically appropriate for ventral incisional hernias.
• Colicky pain
• Diarrhea
• Delay in diagnosis leads to a risk of strangulation

Option B: Hiatus Hernia


• Unlike ventral hernias, which protrude through the abdominal wall, a hiatal hernia occurs when the
upper part of the stomach pushes up into the chest through a so, all opening in the diaphragm.
Option C: Petersen hernia
• Petersen hernias are indeed a type of internal hernia that can occur after certain types of gastric
surgery, particularly after a Roux-en-Y gastric bypass procedure. These hernias develop in the
potential space created between the transverse mesocolon and the Roux limb after a

Page 21

843
gastrojejunostomy. This space is called the Petersen space, hence the name "Petersen hernia."
Option D: Serafini's hernia
• Aka Retro vascular hernia
• Sac is located behind femoral vessels

Solution for Question 14:


Option A: Direct inguinal hernias
• According to the Nyhus classification system, Type IIIA hernia encompasses direct inguinal hernia.
• Nyhus classification categorizes hernia defects by size, location, and type.
Option B: Indirect inguinal hernia
• Indirect inguinal hernia is Type II.
• The internal ring is enlarged without impingement on the inguinal canal floor: It does not extend to the
scrotum.
• Type IIIB Indirect hernias are hernias that have enlarged enough to infringe upon the posterior
inguinal wall, indirect sliding or scrotal hernias, and pantaloon hernias.
Option C: Femoral hernia
• Femoral hernia is classified as type IIIC
Option D: Femoral recurrent hernia
• A recurrent femoral hernia is of type IV C.

Solution for Question 15:


Option C: Cigarette smoking
• Cigarette smoking is not directly involved in causing a direct inguinal hernia. It can cause chronic
obstructive pulmonary disease and chronic cough (Bronchitis, tuberculosis), leading to increased
intra-abdominal pressure, thus being the culprit for direct inguinal hernia.
• Factors that can cause increased intra-abdominal pressure Chronic cough (Bronchitis, tuberculosis)
Chronic obstructive pulmonary disease (COPD) (because of increased abdominal pressure while
coughing) Obesity Chronic constipation with straining Enlarged prostate with straining at micturition
Pregnancy Cirrhosis with ascites Heavy weight lifting Chronic ambulatory peritoneal dialysis
Intra-abdominal tumors Chronically enlarged pelvic organs Previous right lower quadrant incision
• Chronic cough (Bronchitis, tuberculosis)
• Chronic obstructive pulmonary disease (COPD) (because of increased abdominal pressure while
coughing)
• Obesity
• Chronic constipation with straining
• Enlarged prostate with straining at micturition

Page 22

844
• Pregnancy
• Cirrhosis with ascites
• Heavy weight lifting
• Chronic ambulatory peritoneal dialysis
• Intra-abdominal tumors
• Chronically enlarged pelvic organs
• Previous right lower quadrant incision
• Chronic cough (Bronchitis, tuberculosis)
• Chronic obstructive pulmonary disease (COPD) (because of increased abdominal pressure while
coughing)
• Obesity
• Chronic constipation with straining
• Enlarged prostate with straining at micturition
• Pregnancy
• Cirrhosis with ascites
• Heavy weight lifting
• Chronic ambulatory peritoneal dialysis
• Intra-abdominal tumors
• Chronically enlarged pelvic organs
• Previous right lower quadrant incision
Option A: Chronic Diarrhea
• Chronic diarrhea is not a risk factor for developing an inguinal hernia.
• Constipation is one of the risk factors, as the straining associated with constipation severely increases
intra-abdominal pressure, which could increase the risk of hernia formation.
• Therefore, a low-fiber diet that could result in constipation is also associated with a higher risk of
hernia formation.
Option B: Underweight
• Being underweight is not associated with developing an inguinal hernia.
• Obesity is a risk factor for developing a hernia.
• Obesity increases the strain and pressure on the abdominal muscles, making them weaker and more
prone to a hernia.
Option D: None of the above
• Only Smoking is the mentioned risk factor for developing an inguinal hernia.

Solution for Question 16:

Page 23

845
Option A: Content is the omentum
• The given description of the content is consistent with that of omentum.
• When the inguinal hernia sac contains omentum, it is called omentocele.
• Omentum is doughy in consistency. The last portion is more difficult to reduce than the first in a
omentocele.
Option B: It is an enterocele
• The inguinal hernia sac contains a large intestine called an enterocele.
• It gurgles on reduction.
• During reduction, the first portion is more difficult to reduce than the last.
Option C: It is a rectocele
• A rectocele occurs when the end of the large intestine (rectum) pushes against and moves posterior
wall of the vagina into the hernial sac
Option D: It is a cystocele
• A cystocele occurs when the urinary bladder may be the content

Solution for Question 17:


Option C: In Cloquet’s hernia, the sac lies down the pectineus fascia
• Variants of femoral hernia: Laugier's hernia: Hernia through a gap in the lacunar ligament Cloquet
hernia: Hernia located behind pectineus fascia Narath hernia: Seen in congenital dislocation of hip
Lateral displacement of sac Sac is located behind femoral vessels Hesselbach's hernia: Lateral to the
femoral artery Serafini's hernia: Behind the femoral vessels
• Laugier's hernia: Hernia through a gap in the lacunar ligament
• Cloquet hernia: Hernia located behind pectineus fascia
• Narath hernia: Seen in congenital dislocation of hip Lateral displacement of sac Sac is located behind
femoral vessels
• Seen in congenital dislocation of hip
• Lateral displacement of sac
• Sac is located behind femoral vessels
• Hesselbach's hernia: Lateral to the femoral artery
• Serafini's hernia: Behind the femoral vessels
• Laugier's hernia: Hernia through a gap in the lacunar ligament
• Cloquet hernia: Hernia located behind pectineus fascia
• Narath hernia: Seen in congenital dislocation of hip Lateral displacement of sac Sac is located behind
femoral vessels
• Seen in congenital dislocation of hip
• Lateral displacement of sac

Page 24

846
• Sac is located behind femoral vessels
• Hesselbach's hernia: Lateral to the femoral artery
• Serafini's hernia: Behind the femoral vessels
• Seen in congenital dislocation of hip
• Lateral displacement of sac
• Sac is located behind femoral vessels
Option A: More common in male
• Indirect inguinal hernia is more common in males.
• But most common type of hernia in females is indirect inguinal hernia.
Option B: Lockwood operation is the treatment of choice
• Low inguinal operation (Lockwood) is simplest but suitable only when there is no risk of bowel
resection.
• In a femoral hernia, there is a high risk of strangulation.
• Midline abdominal extraperitoneal femoral hernioplasty (Henry procedure) is now considered the
surgery of choice for a femoral hernia.
Option D: Should be kept for observation without surgery
• The femoral hernia has a high risk of strangulation, so it should be operated on as soon as possible.

Solution for Question 18:


Option D: It occurs at the lateral edge of the linea semilunaris
• A Spigelian hernia occurs at linea semilunaris, which extends along the lateral border of each rectus
abdominis muscle.
• The posterior rectus sheath is deficient at the level of the arcuate line (semicircular line), about
one-third of the distance between the umbilicus and the pubic symphysis; this is the most common site
for Spigelian hernia to occur through the linea semilunaris.
Spigelian hernia:
• Hernia through Spigelian fascia (Spigelian fascia: a thin aponeurosis located between rectus
abdominis muscle medially and semilunar line laterally)
• Located at infra umbilical region due to absence of posterior rectus sheath.
• It is also called as Inter parietal hernia
• Clinical features: Swelling is neither visible nor palpable leading to delayed presentation Abdominal
pain Delay in presentation leads to delay in diagnosis, which causes an increased risk of strangulation.
• Swelling is neither visible nor palpable leading to delayed presentation
• Abdominal pain
• Delay in presentation leads to delay in diagnosis, which causes an increased risk of strangulation.
• Investigation: USG/CT

Page 25

847
• Management: Reduce the herniated content and close the defect.
• Swelling is neither visible nor palpable leading to delayed presentation
• Abdominal pain
• Delay in presentation leads to delay in diagnosis, which causes an increased risk of strangulation.
Option A: It occurs exclusively in males
• Spigelian hernia usually affects men and women equally.
• It can occur at any age but is most common in older adults.
Option B: It involves part of the circumference of the bowel wall
• A Spigelian hernia occurs at linea semilunaris and does not involve the circumference of the bowel
wall.
Option C: It is best repaired by the classical Bassini technique of inguinal ligament repair
• The Bassini technique is used for the repair of inguinal hernias only.
• Spigelian hernias have to be repaired because of their relatively narrow neck and the risk of
incarceration.
• Spigelian hernias are approached through conventional or laparoscopic methods for reducing the
herniated content back and closing the defect.

Solution for Question 19:


Option C: Sliding hernia
• The question describes the cecum as constituting a part of the wall of the sac rather than as content,
and the cecum is the most common content in right-side sliding hernia.
• Sliding hernia, also known as Hernia-en-glissade, is an acquired hernia due to weakening in the
abdominal wall. It occurs at the deep inguinal ring lateral to the inferior epigastric vessels.

Page 26

848
• Retroperitoneal fatty tissue is pushed downwards along the inguinal canal.
• As more tissue enters the hernia, the peritoneum is pulled with it, thus creating a sac.
• However, the sac has formed secondarily, distinguishing it from a classic indirect hernia.
• On the left side, the sigmoid colon may be pulled into a sliding hernia, and on the right side, the
cecum.
Option A: Richter’s hernia
• In a Richter hernia, a small portion or circumference of the anti-mesenteric wall of the intestine is
trapped within the hernia.
• Strangulation can occur without the presence of intestinal obstruction.
Option B: Spigelian hernia
• A Spigelian hernia occurs the linea semilunaris, which extends along the lateral border of each rectus
abdominis muscle.
• The posterior rectus sheath is deficient at the arcuate line (semicircular line) level, about one-third of
the distance between the umbilicus and the pubic symphysis; this is the most common site for Spigelian
hernia to occur through the linea semilunaris.
Option D: Epigastric hernia
• Any hernia from the xiphoid process till 2cm of umbilicus
• AKA Epigastric lipoma
• Pain referred to epigastric region. It mimics peptic ulcer.

Solution for Question 20:


Option C: Petit’s triangle is bounded by the iliac crest, latissimus dorsi, and external oblique muscle
• Lumbar hernia: Herniation in the lumbar region of the posterior abdominal wall.

Page 27

849
• Boundaries of the Inferior triangle of Petit Posteriorly: Latissmus dorsi Anteriorly: External oblique
muscle Inferiorly: Iliac crest
• Posteriorly: Latissmus dorsi
• Anteriorly: External oblique muscle
• Inferiorly: Iliac crest
• Posteriorly: Latissmus dorsi
• Anteriorly: External oblique muscle
• Inferiorly: Iliac crest

Option A: Is always congenital


• Infantile inguinal hernia is always congenital.
• Lumbar hernia can be acquired as well as congenital.
Option B: Hernia through superior Grynfeltt triangle is less common
• Most lumbar hernias occur through the superior triangle of Grynfeltt.
• Some lumbar hernias occur through the Inferior triangle of Petit.
Option D: Grynfeltt triangle is bounded by the 11th rib, paraspinal, and external oblique muscle
• Boundaries of the Superior triangle of Grynfeltt. Posteriorly - Para spinal muscle. Superiorly - 12th rib
Anteriorly - Internal oblique muscle
• Posteriorly - Para spinal muscle.
• Superiorly - 12th rib
• Anteriorly - Internal oblique muscle
• Posteriorly - Para spinal muscle.
• Superiorly - 12th rib
• Anteriorly - Internal oblique muscle

Page 28

850
Solution for Question 21:
Option B: Fatty hernia of the linea alba
• Epigastric pain with positive cough impulse indicates fatty hernia of the linea alba/ epigastric hernia.
• Epigastric hernia It is also known as epigastric lipoma/fatty hernia of the linea alba. Herniation of
preperitoneal fat due to a defect in the linea alba. They are usually asymptomatic but can become
symptomatic depending on size, location, and if there is ulceration of the lesion They frequently affect
people between 20 and 50 years of age and men more frequently than women. Pain, referred to as the
epigastrium, mimics the pain of a peptic ulcer.
• It is also known as epigastric lipoma/fatty hernia of the linea alba.
• Herniation of preperitoneal fat due to a defect in the linea alba.
• They are usually asymptomatic but can become symptomatic depending on size, location, and if there
is ulceration of the lesion
• They frequently affect people between 20 and 50 years of age and men more frequently than women.
• Pain, referred to as the epigastrium, mimics the pain of a peptic ulcer.
• Treatment: Excision of fat + closure Size > 4 cm; the mesh repair is indicated.
• Excision of fat + closure
• Size > 4 cm; the mesh repair is indicated.
• It is also known as epigastric lipoma/fatty hernia of the linea alba.
• Herniation of preperitoneal fat due to a defect in the linea alba.
• They are usually asymptomatic but can become symptomatic depending on size, location, and if there
is ulceration of the lesion
• They frequently affect people between 20 and 50 years of age and men more frequently than women.
• Pain, referred to as the epigastrium, mimics the pain of a peptic ulcer.
• Excision of fat + closure
• Size > 4 cm; the mesh repair is indicated.
Option A: Umbilical hernia
• It is the hernia occurring directly through the umbilicus.
• Umbilical hernias are frequently benign and common.
• Infants are most frequently affected by umbilical hernias, but adults can also be diagnosed with it.
Option C: Incisional hernia
• A tissue protrusion that develops at the site of a healing surgical scar is known as an incisional hernia.
• About 15%–20% of abdominal hernias are incisional hernias.
Option D: Inguinal hernia
• Inguinal hernias occur in the groin area, not the epigastric region.

Page 29

851
Solution for Question 22:
Option A: Onlay
• Onlay mesh placement is a technique for surgical hernia repair. The mesh is used to strengthen,
repair, and reinforce the abdominal wall. The mesh is fixed to the fascia using sutures to close the
defect.

• Different types of mesh repair are shown below:


Mesh placement abdominal wall reconstruction in case of incisional hernia
Onlay (Overlay)
Underlay (Sublay)
Inlay
• Placement of mesh over anterior fascia after the closure of the fascial defect
• Mesh is placed outside the abdominal cavity avoiding direct contact with abdominal viscera in the
subcutaneous space
• Placement of mesh below the fascial components (retro rectus, preperitoneal, or intraperitoneal
position)
• Mesh placed immediately extraperitoneal against muscle or fascia
• Intra-abdominal pressure acts to hold the mesh and prevent migration due to the wide overlap of
mesh and fascia
• Mesh is sutured to the fascial edge without overlap
• Mesh placed within the defect
• Associated with high recurrence rate because high intra-abdominal pressure pulls away the mesh
from fascial edges.

Page 30

852
Option B: Retromuscular
• Placement of mesh below the fascial components (retro rectus, preperitoneal, or intraperitoneal
position).
• This is also called retro-muscular Sublay repair.
Option C: Inlay
• This mesh is sutured to the fascial edge without overlap. Mesh is placed within the defect.
• It is associated with a high recurrence rate because high intra-abdominal pressure pulls away the
mesh from the fascial edges.
Option D: Preperitoneal
• In this mesh, it is placed immediately extraperitoneal against a muscle or fascia.
• It is also a Sublay repairing technique.

Solution for Question 23:


Option D: Associated with congenital malformations
• Given the clinical picture and examination findings of organ protrusion through the umbilicus, the
patient is most likely suffering from Omphalocele, which is associated with congenital malformations.
• Infants with an omphalocele have an approximately 50% incidence of associated congenital
anomalies. Associated with Trisomy 13, 18, 21. Associated with congenital malformations: CVS >
Musculoskeletal system > Gastrointestinal system > Genito-urinary system Associated with Beckwith –
Wiedemann syndrome (a variant of Wilms tumor).
• Associated with Trisomy 13, 18, 21.
• Associated with congenital malformations: CVS > Musculoskeletal system > Gastrointestinal system >
Genito-urinary system
• Associated with Beckwith – Wiedemann syndrome (a variant of Wilms tumor).
• Associated with Trisomy 13, 18, 21.
• Associated with congenital malformations: CVS > Musculoskeletal system > Gastrointestinal system >
Genito-urinary system
• Associated with Beckwith – Wiedemann syndrome (a variant of Wilms tumor).

Page 31

853
Option A: Not covered by a sac
• A sac covers the Omphalocele.
• The sac is an intact membrane composed of an outer layer of amnion and an inner layer of
peritoneum.
• Gastroschisis is a congenital malformation of the abdominal wall that causes the viscera to protrude
without a covering membrane.
Option B: A defect in the abdominal fascia
• Omphalocele is caused by failure of the abdominal wall musculature to unite in the midline during fetal
development but not a defect in the abdominal fascia.
Option C: Associated with Intestinal atresia
• Intestinal atresia is present in up to 15% of cases with Gastroschisis.
• The intestine has not undergone complete mesenteric rotation and fixation; hence the infant is at risk
for mesenteric volvulus with resultant intestinal ischemia and necrosis.
• It is not a characteristic feature of Omphalocele.

Solution for Question 24:


Option A: Spermatic cord
• The inguinal canal is 4cm long and extends between the deep and superficial inguinal rings.
• Contents of the inguinal canal: Spermatic cord- in males Round ligament- in females Ilioinguinal nerve
Iliohypogastric nerve
• Spermatic cord- in males
• Round ligament- in females
• Ilioinguinal nerve

Page 32

854
• Iliohypogastric nerve
• Spermatic cord contains: Cremaster muscle fibres Testicular vessels Genital branch of genitofemoral
nerve Vas deferens Cremasteric vessels Lymphatics Processus vaginalis
• Cremaster muscle fibres
• Testicular vessels
• Genital branch of genitofemoral nerve
• Vas deferens
• Cremasteric vessels
• Lymphatics
• Processus vaginalis
• Spermatic cord- in males
• Round ligament- in females
• Ilioinguinal nerve
• Iliohypogastric nerve
• Cremaster muscle fibres
• Testicular vessels
• Genital branch of genitofemoral nerve
• Vas deferens
• Cremasteric vessels
• Lymphatics
• Processus vaginalis
Option B: Femoral nerve
• The most lateral content of the femoral triangle is the femoral nerve.
• It supplies the pectineus muscle in the medial compartment of the thigh and the sartorius, vastus
lateralis, rectus femoris, vastus intermedius, and vastus medialis muscles in the anterior compartment.
Option C: Obturator nerve
• The adductor longus, adductor brevis, adductor magnus, gracilis, obturator externus, and pectineus
muscles, which are responsible for adducting the thigh, are innervated by the obturator nerve.
Option D: Pudendal nerve
• The inferior rectal nerve, located below the pectinate line, sends somatic motor fibres to the external
anal sphincter and returns sensations from the anal canal.

Solution for Question 25:


Correct Option D - Maydl's hernia :

Page 33

855
• The picture shown above is that of a Maydl hernia.
• Maydl's hernia is an uncommon kind that can be fatal if left untreated.
• The hernial sac has two loops of the bowel, one of which is intra-abdominal. The hernial sac contains
a loop of the bowel in the shape of a 'W', and the centre section of the 'W' loop may become
strangulated, either alone or in conjunction with the bowel in the hernial sac.
Incorrect Options:
Option A - Richter’s hernia:

• A Richter's hernia arises when the intestine's antimesenteric wall protrudes through a hole in the
abdominal wall.
• It is unique from other forms of abdominal hernias in that only one intestinal wall protrudes through the
opening, resulting in an incompletely contained intestine lumen in the defect, with the remainder
remaining in the peritoneal cavity.
Option B - Littre's hernia:

Page 34

856
• The protrusion of a Meckel diverticulum through a possible abdominal orifice is referred to as a Littre
hernia.
• The ileal loop to which it is linked may accompany the Meckel diverticulum in the sac; on rare
occasions, it may endure imprisonment or strangulation, necrosis, and perforation.
Option C - Pantaloon hernia:

• A pantaloon hernia, also known as a "Saddlebag" hernia, is defined as any combination of two
contiguous femoral or inguinal hernia sacs on the same side (direct or indirect inguinal hernias).
• Femoral with direct hernias, femoral with indirect hernias, and indirect with direct hernias are all
examples.
• The Romberg hernia is a pantaloon hernia that exclusively consists of ipsilateral indirect and direct
hernias. Hernial sacs can be found on both sides of the inferior epigastric vessels, separated by the
posterior wall of the inguinal canal, which has been pushed down by the direct hernia.

Page 35

857
Page 36

858
Spleen
1. Positive Kehr sign is seen in which of the following?
A. Splenic injury
B. Liver injury
C. Renal injury
D. Mesenteric hematoma
----------------------------------------
2. A 50-year-old male patient with generalized malaise and weakness was found to have a palpable
spleen during physical examination. Further evaluation revealed severe neutropenia, monocytopenia,
and the presence of hairy cells in the blood and bone marrow. The patient underwent an elective
splenectomy. Splenectomy can also be curative in which of the following cases?
(or)
A 50-year-old male has a palpable spleen, severe neutropenia, monocytopenia, and the characteristic
hairy cells in the blood and bone marrow. The patient underwent an elective splenectomy. Splenectomy
can also be curative in which of the following?
A. Budd-Chiari syndrome
B. Sickle cell disease
C. Asymptomatic splenic cysts
D. Stomach CA
----------------------------------------
3. A 12-year-old female has had purple spots on her body for two weeks. She also has gum bleeding
and nasal bleeding episodes. An elective splenectomy is planned. In the postoperative period, the child
is more prone to infection by?
A. Uncapsulated bacteria
B. Capsulated bacteria
C. Gram-positive bacteria
D. Gram-negative bacteria
----------------------------------------
4. Which of the following is the most common malignant tumor of the spleen?
A. Lymphoma
B. Sarcoma
C. Hemangioma
D. Metastasis
----------------------------------------
5. Which of the following organisms is least likely to cause splenic abscess?
A. Streptococcus
B. Toxoplasmosis

859
C. Staphylococcus
D. Salmonella
----------------------------------------
6. A 20-year-old male has an injury to the hilum of the spleen, with devascularisation of the spleen
noted. The patient underwent an emergency splenectomy. Which of the following statements is true
regarding the anatomy of the specimen?
A. The spleen lies in front of the left 9th, 10th, and 11th ribs
B. The splenic artery arises from the SMA
C. The inferior mesenteric vein empties into the right gastric vein
D. The tail of the pancreas lies in the gastrosplenic ligament
----------------------------------------
7. Which of the following is the common cause of splenic cyst ?
A. Bacterial infection
B. Pseudocyst
C. Parasitic infection
D. Congenital anomaly
----------------------------------------
8. A patient with steroid-resistant ITP underwent laparoscopic splenectomy. On day 2 of the
postoperative period, the patient had fever episodes. Which of the following scenarios is most likely?
A. Left lower lobe consolidation
B. Port site infection
C. Intra-abdominal collection
D. Urinary tract infection
----------------------------------------
9. A 12-year-old female child with idiopathic thrombocytopenic purpura is planning for laparoscopic
splenectomy. What is the best time for platelet infusion in this patient?
A. 2 hours before surgery
B. At the time of the skin incision
C. After ligating the splenic artery
D. Immediately after the removal of the spleen
----------------------------------------
10. Which among the following is true regarding the wandering spleen?
A. Chronic torsion leads to splenomegaly
B. The treatment of choice is splenectomy only
C. Infarction and torsion are uncommon
D. It is encapsulated with a short vascular pedicle
----------------------------------------

Page 2

860
11. Which of the following primary tumors cause metastases to the spleen?
A. Carcinoma prostate
B. Malignant Melanoma
C. Carcinoma liver
D. Carcinoma pancreas
----------------------------------------
12. A patient is found to have sustained grade I blunt injury to the spleen. The downward displacement
of the spleen during such trauma is prevented by which of the following structures?
A. Phrenicocolic ligament
B. Lienorenal ligament
C. Gastrosplenic ligament
D. Splenocolic ligament
----------------------------------------
13. A 35-year-old male came to the emergency department after a road traffic accident. He has severe
pain in his left upper abdomen, radiating to his left shoulder. On examination, the patient is stable, with
a tender left hypochondrium with a bruise over it. What is the investigation of choice for this patient?
A. Diagnostic peritoneal lavage
B. CECT
C. Ultrasonography
D. Isotope scan
----------------------------------------

Correct Answers
Question Correct Answer

Question 1 1
Question 2 2
Question 3 2
Question 4 1
Question 5 2
Question 6 1
Question 7 3
Question 8 1
Question 9 3
Question 10 1
Question 11 2
Question 12 1

Page 3

861
Question 13 2

Solution for Question 1:


Option A: Splenic injury
• The patient met a road traffic accident and has presented with abdominal pain, tenderness, and a
positive Kehr’s sign.
• Kehr’s sign is seen in patients with splenic trauma
• Left shoulder pain caused by irritation of the left hemidiaphragm by subphrenic blood
• There may be hyperaesthesia in the area of the left shoulder
• It can be elicited by elevating the foot of the bed for 15 minutes, by which time blood will accumulate
below the left cupula of the diaphragm
• Ballance’s sign is persistent dullness on the left side of the abdomen due to early coagulation of
splenic blood. Shifting dullness is present on the right side.
Option B: Liver injury
• The physical findings associated with the liver injury include right upper quadrant or generalized
abdominal tenderness, abdominal wall contusion or hematoma (seatbelt sign), right lower chest wall
tenderness, contusion, or instability due to rib fractures.
Option C: Renal injury
• Physical exam findings such as flank tenderness and ecchymosis or displaced lower rib fractures
should alert for a possible renal injury.
Option D: Mesenteric hematoma
• Mesenteric hematoma occurs as a direct result of abdominal trauma
• On examination, a mass can be palpable.

Solution for Question 2:


Option B: Sickle cell disease
• The patient has a low white cell count with characteristic hairy cells; thus, he suffers from Hairy cell
leukemia (HCL). The patient accumulates small mature B-cell lymphoid cells. This accumulation
resulted in splenomegaly, indicating elective splenectomy.
• Splenectomy can improve symptoms of sickle cell anemia by reducing the destruction of sickled red
blood cells within the spleen
• Sickle cells are more fragile and prone to rupture, leading to anemia and spleen enlargement.
Removing the spleen reduces the breakdown of sickled cells, decreases anemia, and helps manage
complications such as acute chest syndrome and vaso-occlusive crises in some patients with sickle cell
disease
Other options
Option A: Budd-Chiari syndrome

Page 4

862
• Management of Budd-Chiari syndrome is done by correcting underlying disorders predisposed to the
development of Budd-Chiari syndrome, initiating anticoagulation, and treating complications of portal
hypertension.
• liver transplantation may be considered in decompensated cirrhotic patients, not splenectomy
Option C: Asymptomatic splenic cysts
• Symptomatic splenic cysts are an indication of splenectomy.
• Asymptomatic splenic cysts of 3-5cm are serially followed with ultrasound.
Option D: Stomach CA
• It is not an indication for splenectomy

Solution for Question 3:


Option B: Capsulated bacteria
• The girl has purpura all over the body with isolated thrombocytopenia, which confirms the diagnosis of
idiopathic thrombocytopenic purpura.
• Overwhelming post-splenectomy infections are the most common and fatal late complication of
splenectomy.
• Mortality associated with OPSI is around 40-50%.
• OPSI is caused by polysaccharide-encapsulated bacteria which are as follows: Streptococcus
pneumonia (most common) Neisseria meningitidis Hemophilus influenza (in particular subtype B)
• Streptococcus pneumonia (most common)
• Neisseria meningitidis
• Hemophilus influenza (in particular subtype B)
• Infection may occur at any time after splenectomy. Most infections occur within the first two years after
splenectomy. Risk is greatest in Thalassemia major and Sickle cell anemia.
• In elective splenectomy, the patient should be vaccinated two weeks before surgery, while in
emergency splenectomy, it is recommended to vaccinate postoperatively.
• Streptococcus pneumonia (most common)
• Neisseria meningitidis
• Hemophilus influenza (in particular subtype B)
Incorrect Options:
Option A: Uncapsulated bacteria
• Uncapsulated bacteria are mainly cleared by phagocytosis of white blood cells and macrophages.
Option C and D:
Post splenectomy patients are not prone to all gram-positive and negative bacterial infections, they are
at risk of contracting only the polysaccharide - encapsulated bacteria

Page 5

863
Solution for Question 4:
Option A: Lymphoma
• The most common malignant neoplasm of the spleen is lymphoma (non-Hodgkin’s lymphoma).
• Tumors of the spleen may be lymphoid or nonlymphoid.
• Lymphoid tumors of the spleen are mainly Hodgkin disease or non-Hodgkin lymphoma.
• Non-lymphoid tumors may be either primary or secondary (metastatic).
• The most common nonlymphoid primary tumors are vascular tumors (angiosarcoma.
Option B: Sarcoma
• The most common primary nonlymphoid malignant spleen tumor is angiosarcoma
(Hemangiosarcoma).
Option C: Hemangioma
• The most common benign tumor of the spleen is the hemangioma.
Option D: Metastasis
• Secondary or metastatic tumors are most commonly from melanoma, lung, and breast cancers
• Isolated splenic metastases are extremely unusual but may occur in the setting of a concomitant
carcinoma
• Lung cancer is the tumor that most commonly spreads to the spleen, although colorectal, ovarian and
melanoma may also metastasize to spleen

Solution for Question 5:


Option B: Toxoplasmosis
• Toxoplasma is a parasite and has no role in splenic abscess formation.
• Splenic abscess is a rare condition. In healthy patients with unilocular abscess, the mortality rate is 15
- 20%, while in immunocompromised, it is multilocular, and the mortality rate is 80%.
• Most common organism responsible – Staph aureus.
• Fungal abscess is typically seen in HIV positive and Immunocompromised patients - caused by
candida.
• Immunocompromised patients - caused by candida.
• Other organisms responsible: Streptococcus Salmonella Gram negative enteric bacilli
• Streptococcus
• Salmonella
• Gram negative enteric bacilli
• Immunocompromised patients - caused by candida.
• Streptococcus
• Salmonella

Page 6

864
• Gram negative enteric bacilli
Incorrect Options:
Options A, C and D are associated with splenic abscess.

Solution for Question 6:


Option A: The spleen lies in front of the left 9th, 10th, and 11th ribs
• Splenic trauma is one of the most common indications of emergency splenectomy.
• The weight of a normal adult spleen is 75-250 g and measures 7 to 11cm in length.
• It lies in the left hypochondrium, with its long axis along the 10th rib.
• The spleen lies in front of the left 9th, 10th, and 11th ribs. Therefore, the splenic injury should be
suspected in a fracture of these ribs.
• It is suspended by the following peritoneal reflections and ligaments: splenocolic ligament and
gastrosplenic ligament.
• It lies between the fundus of the stomach and the diaphragm
Anatomy of spleen:
• Spleen is the largest lymphatic organ
• It is known as blood bank of body
• Stores 2% of blood
• Average weight 75-100 grams
• Usually, the spleen is not palpable If enlarged >2.5 times, it becomes palpable
• If enlarged >2.5 times, it becomes palpable
• Relations of spleen: Spleen in relation to 9th, 10th, and 11th ribs Long axis of spleen is along the 10th
rib
• Spleen in relation to 9th, 10th, and 11th ribs
• Long axis of spleen is along the 10th rib
• Develops from cephalic part of dorsal mesogastrium
• If enlarged >2.5 times, it becomes palpable
• Spleen in relation to 9th, 10th, and 11th ribs
• Long axis of spleen is along the 10th rib

Page 7

865
Option B: The splenic artery arises from the SMA
• The tortuous splenic artery arises from the coeliac axis and runs along the upper border of the
pancreas body and tail, giving it small branches.
• The main splenic artery divides into superior and inferior branches into segmental branches.
Option C: The inferior mesenteric vein empties into the right gastric vein
• Inferior mesenteric vein drains into splenic vein which joins with superior mesenteric vein and finally
drains into portal vein.
Option D: The tail of the pancreas lies in the gastrosplenic ligament
• The hilum sits at the angle between the stomach and kidney and is in contact with the tail of the
pancreas, which lies in the lienorenal ligament
• The inner (visceral) surface of the spleen has gastric, colic, renal, and pancreatic impressions

Solution for Question 7:


Option C: Parasitic infection
• Parasitic infection is the most common cause of splenic cysts worldwide, and the majority are due to
Echinococcus species.
• Splenic cysts can be primary or secondary. The primary splenic cyst may be parasitic or non-parasitic.
Secondary cysts occur as a result of any ongoing pathology like pseudocysts.
• The primary splenic cyst may be parasitic or non-parasitic.
• Secondary cysts occur as a result of any ongoing pathology like pseudocysts.
• Cysts resulting from trauma are pseudocysts due to their lack of an epithelial cell lining.
• Examples of non-parasitic cysts are dermoid, epidermoid, and epithelial cysts.
• Symptomatic splenic cysts are treated with splenectomy

Page 8

866
• The primary splenic cyst may be parasitic or non-parasitic.
• Secondary cysts occur as a result of any ongoing pathology like pseudocysts.
Option A: Bacterial infection
• Splenic abscesses are relatively uncommon conditions.
• It presents with the triad of fever, left upper quadrant tenderness, and leukocytosis.
Option B: Pseudocyst
• Pseudocysts lack epithelial cell lining and are usually asymptomatic.
• The etiology behind these cysts is mainly trauma.
Option D: Congenital anomaly
• A congenital splenic cyst (CSC) is seen in late childhood when the rupture or other complications lead
to its diagnosis.
• It is not a common splenic pathology.

Solution for Question 8:


Option A: Left lower lobe consolidation
• The patient underwent laparoscopic splenectomy on account of her steroid-resistant ITP. She
develops a fever on the third postoperative day, suggesting some infectious complications.
• Fever on postoperative day (POD) two after laparoscopic splenectomy suggests left lower lobe
consolidation
• The causes of postoperative fever include ‘W’s and are as follows: Wind - POD 1 to 3 - Atelectasis
and pneumonia Water - POD 3 to 5 - Urinary tract infection Walking - POD 4 to 8 - Deep vein
thrombosis and pulmonary embolism Wound - POD 5 to 7 - Surgical site infection Wonder drugs -
anytime - Drug fever
• Wind - POD 1 to 3 - Atelectasis and pneumonia
• Water - POD 3 to 5 - Urinary tract infection
• Walking - POD 4 to 8 - Deep vein thrombosis and pulmonary embolism
• Wound - POD 5 to 7 - Surgical site infection
• Wonder drugs - anytime - Drug fever
• Wind - POD 1 to 3 - Atelectasis and pneumonia
• Water - POD 3 to 5 - Urinary tract infection
• Walking - POD 4 to 8 - Deep vein thrombosis and pulmonary embolism
• Wound - POD 5 to 7 - Surgical site infection
• Wonder drugs - anytime - Drug fever
Other options
Option B: Port site infection

Page 9

867
• Surgical site infection occurs in people with poor wound healing, infectious etiology, and
immunocompromised patients.
• It manifests on the 5th to 7th postoperative day
Option C: Intra-abdominal collection
• Intra-abdominal collection can result from anastomotic leakage or poor hemostatic control.
• It also manifests on the 5th to 7th postoperative day.
Option D: Urinary tract infection
• UTI manifests on the 3rd to 5th postoperative day.
• It presents with fever, dysuria, and burning micturition.

Solution for Question 9:


Option C: After ligating the splenic artery
• The best time for platelet transfusion in ITP is after ligating the splenic artery.
• In the acute setting of ITP, severe bleeding not controlled by steroid therapy, fresh blood transfusion,
or transfusion with platelet concentrates after controlling the splenic vessels is necessary.
• A low platelet count characterizes ITP despite normal bone marrow and the absence of other causes
of thrombocytopenia.
• Low platelet count results from developing antibodies to specific platelet membrane glycoproteins.
• ITP is predominantly a disease of young women
Option A: 2 hours before surgery
• Preoperative transfusion of platelets is contraindicated in patients undergoing splenectomy.
• All platelets will be destroyed by an immune response against them, resulting in no net benefit as the
spleen will destroy the opsonized platelets.
• Splenic congestion will also occur.
Option B: At the time of the skin incision
• Transfusion of platelets at the time of skin incision will cause an immune response against the
platelets and result in spleen destruction until the specimen is removed.
Option D: Immediately after the removal of the spleen
• Transfusion of platelets in an ITP patient after the removal of the spleen is unduly late.
• It may pose some hemostatic problems while performing direction due to low platelet count.

Solution for Question 10:


Option A: Chronic torsion leads to splenomegaly
• Wandering spleen is when the position of the spleen is not fixed due to laxity of ligaments or previous
surgery, which results in twisting of the vascular pedicle and splenic congestion.

Page 10

868
• It is a rare finding seen in children and women between the ages of 20 and 40 years.
• Clinical Features include intermittent abdominal pain, splenomegaly resulting from venous
congestion, and severe persistent pain.
• A mobile mass may be palpable on clinical examination
• The investigation of choice is a CECT abdomen which shows a spleen located outside its usual
position
• Chronic torsion typically causes venous congestion and splenomegaly
Option B: The treatment of choice is splenectomy only
• Splenectomy or splenopexy is the treatment of choice for the wandering spleen.
Option C: Infarction and torsion are uncommon
• Splenic torsion and infarction are common in wandering spleens
• It occurs due to laxity and torsion of ligaments.
Option D: It is encapsulated with a short vascular pedicle
• Wandering spleen has a long pedicle that renders the spleen hypermobile.
• They have a visible capsule.

Solution for Question 11:


Answer
Option B: Malignant Melanoma
• Secondary or metastatic tumors in the spleen are most commonly from melanoma, lung cancer, and
breast cancer.
• Isolated splenic metastases are extremely unusual, but these may occur in the setting of a
concomitant carcinoma.
Splenic Tumors
• Most Common neoplasm or tumor of spleen-NHL (non-Hodgkin's lymphoma).
• Most Common primary benign tumor of spleen-Hemangioma.
• Most Common primary malignant tumor of spleen-angiosarcoma.
• Most Common primary for splenic metastasis - Malignant melanoma.
• Most Common primary leading to isolated secondaries to spleen-CA ovary.
Other
Option A: Carcinoma prostate
• Prostatic carcinoma metastasizes to lymph nodes, mostly obturator, and internal iliac lymph nodes.
• It also spreads to the vertebrae and bones.
Option C: Carcinoma liver
• Hepatocellular carcinoma metastasizes to the lungs, lymph nodes, adrenal gland, and bones,
including the skull.

Page 11

869
Option D: Carcinoma pancreas
• Pancreatic cancer most commonly spreads to the liver, peritoneum, and lungs.

Solution for Question 12:


Option A: Phrenicocolic ligament
• Grade 1 splenic trauma has subcapsular hematoma < 10% surface area and a capsular tear of < 1 cm
parenchymal depth.
• The phrenicocolic ligament prevents downward displacement of the spleen
• It is a peritoneal ligament extending from the splenic flexure of the colon to the diaphragm

Option B: Lienorenal ligament


• The lienorenal ligament contains the pancreas splenic artery, vein, and tail.
• It represents the dorsal most of the dorsal mesentery and forms part of the lateral border of the lesser
sac.
Option C: Gastrosplenic ligament
• The gastrosplenic ligament is part of the greater omentum.
• It connects the hilum of the spleen to the greater curvature of the fundus and body of the stomach.
Option D: Splenocolic ligament
• The splenocolic ligament is a peritoneal ligament connecting the splenic capsule to the transverse
colon.
• It is made of the visceral peritoneum and is a component of the greater omentum

Page 12

870
Solution for Question 13:
Option B: CECT
• Most likely, the patient has suffered a splenic trauma.
• In a hemodynamically stable patient with splenic trauma, the diagnostic modality of choice is a CECT
scan to grade the severity of the splenic injury.
• Splenic trauma includes:
• left upper quadrant or flank ecchymosis and abrasions
• left shoulder pain caused by irritation of the left hemidiaphragm by subphrenic blood (Kehr’s sign)
• Evaluation of hematocrit, hemoglobin, platelet, and coagulation profile should be done, followed by
focused abdominal sonography for trauma (FAST). FAST offers a rapid and non-invasive approach to
detecting intraperitoneal bleeding.
Option A: Diagnostic peritoneal lavage
• Diagnostic peritoneal lavage is indicated to aid in the evaluation of the hemodynamically unstable
patient, to detect blunt bowel injury in hemodynamically stable patients under observation, and to aid in
diagnosing diaphragmatic injury in select patients where chest leak is suspected.
• DPL is used when FAST is inconclusive and when there is no CECT available.
• Diagnostic peritoneal lavage (DPL) is a highly accurate test for evaluating intraperitoneal hemorrhage
or a ruptured hollow viscus, but it has been largely replaced with eFAST in modern-day practices.
Option C: Ultrasonography
• A focused assessment with sonography for trauma (FAST) is one of the initial modalities of
investigations in trauma patients.
• For those with normal hemodynamics, a primary survey followed by a focused abdominal examination
(ecchymosis, abrasions, flank pain, and distension) is done.
• The primary purpose of the FAST examination is to determine the presence of pathologic pericardial,
intrathoracic, or intraperitoneal free fluid.
• In any blunt trauma, the first investigation is FAST and the IOC is CECT
Option D: Isotope scan
• Isotopic scans are not a part of investigations of trauma patients.
• They are sometimes used in thyroid cancer scan.

Page 13

871
Previous Year Questions part 2
1. Which of the following cancers are correctly matched with the criteria for the minimum number of
lymph nodes required for pathological staging? A. CA stomach: 10 B. CA colon: 12 C. CA gall bladder:
6 D. CA breast:15
A. B, C
B. A, B, C
C. A, C, D
D. A, B, C, D
----------------------------------------
2. Match the following: A. Nylon 3-0 1. Bowel anastomosis B. Polyglactin 2. Suturing linea alba in the
midline C. Polypropylene 1-0 3. Midline laparotomy skin wound closure D. Polypropylene 2-0 4. Fixing
mesh in hernia surgery
A. Nylon 3-0 1. Bowel anastomosis
B. Polyglactin 2. Suturing linea alba in the midline
C. Polypropylene 1-0 3. Midline laparotomy skin wound closure
D. Polypropylene 2-0 4. Fixing mesh in hernia surgery

A. A-4, B-1, C-4, D-2


B. A-3, B-1, C-4, D-2
C. A-3, B-1, C-2, D-4
D. A-1, B-4, C-3, D-4
----------------------------------------
3. Which of the following is the most frequently observed gallstone chemistry?
A. Crystalline cholesterol monohydrate
B. Crystalline cholesterol dihydrate
C. Amorphous cholesterol dihydrate
D. Amorphous cholesterol monohydrate
----------------------------------------
4. Which of the following statements accurately describes McBurney point in cases of acute
appendicitis?
A. McBurney point is located in the left lower quadrant of the abdomen.
B. McBurney point corresponds to the site of maximal tenderness in acute appendicitis.
C. McBurney point is used to assess rebound tenderness in acute appendicitis.
D. McBurney point is located at the umbilicus in acute appendicitis.
----------------------------------------
5. A 3-month-old child presented with umbilical discharge. The abdominal examination was normal
except for the finding below. The next step of management is?

872
A. MRI Abdomen
B. USG Abdomen
C. CECT
D. Exploratory laparotomy
----------------------------------------
6. In the classification system of the European Hernia Society, what does M3 incisional hernia signify?
A. Epigastric
B. Infraumbilical
C. Umbilical
D. Suprapubic
----------------------------------------
7. Which of the subsequent options are used in case of bleeding from esophageal varices?

A. A, C and D
B. A and B only
C. A, B and C
D. A, B and D
----------------------------------------
8. A neonate presented with mild intermittent upper abdominal pain. On examination, the infant has a
scaphoid abdomen and breath sounds are decreased on both sides. X-ray is given below. What is the
diagnosis?

Page 2

873
A. Bochdalek hernia
B. Pneumonia
C. Gastric volvulus
D. Eventration of diaphragm
----------------------------------------
9. A patient presents with acute–onset, severe abdominal pain. He is hemodynamically stable. A chest
X-ray is shown below. What is the next step in management?

A. Gastric lavage
B. Chest tube insertion
C. Tracheostomy
D. Resuscitation and laparotomy
----------------------------------------
10. What is the most frequently occurring location for an intraperitoneal abscess to develop?
A. Suprahepatic
B. Subhepatic
C. Left lobe of liver
D. Left side below diaphragm
----------------------------------------
11. A 45 years old man complained of recurrent discharge and pain due to lesions around the anus for
3 years. What is the diagnosis?

Page 3

874
A. Pilonidal sinus
B. Carbuncle
C. Boil
D. Fistula in ano
----------------------------------------
12. A 23-year-old male had acute appendicitis 3 days ago and presented to the surgery department
with abdominal pain and tenderness. Which of the following incisions would be appropriate for this
condition?

A. 1
B. 2
C. 3
D. 5
----------------------------------------
13. Which of the following CT findings is not an indication to perform angiography in a splenic trauma
patient under conservative management?
A. Subcapsular hematoma covering 40% surface area.
B. Significant hemoperitoneum
C. Presence of pseudoaneurysm
D. Contrast extravasation
----------------------------------------

Page 4

875
14. A 45-year-old alcoholic with a history of binge drinking presents with an episode of vomiting bright
red blood. His vitals are stable. An endoscopy was performed, an image of which is given below; what
is the probable diagnosis here?

A. Tension pneumothorax
B. Boerhaave syndrome
C. Mallory Weiss Tear
D. Gastric ulcer perforation
----------------------------------------
15. What is the most probable cause of massive, sudden onset hematemesis in a 50-year-old patient
with a history of chronic liver disease and chronic alcoholism, who has arrived at the emergency
department?
A. Mallory Weiss tear
B. Bleeding esophageal varices
C. Boerhaave’s syndrome
D. Perforated ulcer
----------------------------------------
16. A 3-day old neonate presents abdominal distension and bilious vomiting. Among the following,
which is the most appropriate investigation to diagnose the condition?
A. Barium swallow
B. Barium meal
C. Barium meal follow through
D. CECT abdomen
----------------------------------------
17.
(or)
Identify the pathology in the given radiograph of a newborn.:
A. Meningocele
B. Imperforate anus
C. Congenital megacolon

Page 5

876
D. Pyloric stenosis
----------------------------------------
18. Identify the investigation and the lesion depicted in the image given below.

A. Retrograde cystourethrogram with membranous urethra stricture


B. Retrograde cystourethrogram with penile urethral stricture
C. Micturating cystourethrography with bulbar stricture
D. Micturating cystourethrography with psoriatic urethra stricture
----------------------------------------
19. Which of the following is the best investigation to establish a diagnosis of GERD?
A. 24 hours ambulatory pH monitoring
B. Upper GI endoscopy
C. CECT
D. MRI
----------------------------------------
20. A 42-year-old alcoholic and a chronic smoker presented with bouts of vomiting and stabbing pain in
the epigastrium. He was binge drinking shortly before the symptoms started. He is febrile and
tachycardic. The given findings were seen on bedside radiological imaging. Which among the following
is a likely cause of this?

A. Tension pneumothorax
B. Peptic ulcer perforation

Page 6

877
C. Boerhaave syndrome
D. Mallory Weiss tear
----------------------------------------
21. Which of the following is the most common deficiency observed after gastric bypass /bariatric
surgery?
A. Iron
B. Copper
C. Zinc
D. Calcium
----------------------------------------
22. An elderly man underwent Roux-en-Y gastric bypass surgery for obesity. He now presents to his
physician with complaints of dizziness, fatigue, and tremors 2 hours after the ingestion of a
carbohydrate-rich meal. His symptoms occur mainly due to Hypoglycemia Small volume of food intake
High osmotic load of food Increased release of insulin Increased release of glucagon
A. 1, 2 and 5
B. 1, 3 and 5
C. 1 and 4
D. 3 and 5
----------------------------------------
23. A young adult with gastric outlet obstruction presents with vomiting. Which of the following fluids is
preferred in this patient for initial resuscitation?
A. Normal saline
B. Ringer lactate
C. 5% dextrose in water
D. 20% dextrose in water
----------------------------------------
24. A 34 years old female with a history of alcohol intake was taking NSAIDs for abdominal pain. She
developed severe pain in the whole abdomen. A radiograph is given below. What is the diagnosis?

A. Rupture of ectopic pregnancy

Page 7

878
B. Acute pancreatitis
C. Perforated liver abscess
D. Hollow viscus perforation
----------------------------------------
25. What is the diagnosis of a patient who had a bronchospasm history, and was found to have a tumor
in their ileum along with elevated levels of 5-hydroxy indoleacetic acid in their urine?
A. Carcinoid tumor
B. Neuroblastoma
C. Leiomyoma
D. Chromaffinoma
----------------------------------------
26. A 28-year-old mother presents with pain in her left breast. Which of the following is the most likely
cause of acute mastitis?
A. Breast engorgement
B. Hormonal influence
C. Absence of lactation
D. Crack in the nipple
----------------------------------------
27. Which lymph node is most commonly involved in prostate cancer?
A. Deep inguinal
B. Pelvic
C. Superficial inguinal
D. Internal iliac
----------------------------------------
28. A man came with complaints of recurrent discharge and pain due to lesions around the anus for 3
years. A diagnosis of fistula-in-ano is made. What is the gold standard investigation for this condition?
A. Fistulogram
B. USG
C. CECT
D. MRI
----------------------------------------
29. A 32-year-old female undergoes emergency laparotomy for perforation peritonitis. Which of the
following is the surest sign of wound dehiscence?
A. Copious sero-sanguinous discharge
B. Rebound tenderness
C. Hemorrhage
D. Hypotension

Page 8

879
----------------------------------------
30. A homosexual man presents with painful defecation and a protruding mass from the anal canal.
Biopsy confirms squamous cell carcinoma of the anus. What is the appropriate course of action for
managing this patient?
A. Chemotherapy
B. Abdominoperineal repair
C. Combined chemoradiation
D. Wide local excision
----------------------------------------
31. A 25-year-old patient presents to the surgical OPD with a painless left inguinal reducible mass. On
examination cough impulse is positive. After further investigations, the patient is diagnosed with an
inguinal hernia. What is the surgical management of this patient?
A. Herniotomy
B. Hernioplasty
C. Wait and watch
D. Emergency laparotomy
----------------------------------------
32. During an endoscopy, when a constriction is detected at the esophageal junction approximately 25
cm from the incisor, what is the likely cause?
A. Inferior vena cava
B. Arch of aorta
C. Right bronchus
D. Diaphragmatic opening
----------------------------------------
33. Which of the following is the most common complication following ligation of the first vessel during
abdominoperineal resection for rectal carcinoma?
A. Parasympathetic - bladder dysfunction and retrograde ejaculation.
B. Sympathetic - bladder dysfunction and impotence.
C. Sympathetic - retrograde ejaculation and bladder dysfunction.
D. Sympathetic - Impotence and loss of cutaneous sensation in perineal region
----------------------------------------
34. Which nerve entrapment is responsible for the symptom of right thigh pain experienced by a
45-year-old male after undergoing bilateral laparoscopic hernia repair for inguinal hernia?
A. Ilioinguinal nerve
B. Iliohypogastric nerve
C. Femoral nerve
D. Lateral cutaneous nerve of thigh
----------------------------------------

Page 9

880
35. A patient presents with a swelling in the groin region and it is diagnosed as a direct inguinal hernia.
Weakness in which of the following structures is most likely to cause this condition?
A. Pectineal ligament
B. Conjoint tendon
C. Reflected part of inguinal ligament
D. Lacunar ligament
----------------------------------------
36. A 5-year-old child was admitted to the hospital for prolapsing rectal mass and painless rectal
bleeding. Histopathological examination reveals enlarged and inflamed glands filled with mucin. What is
the likely diagnosis?
A. Adenoma
B. Choristoma
C. Hamartoma
D. Carcinoma
----------------------------------------
37. A 20-year-old female patient consumed a cleaning product containing 90% sodium hydroxide. She
is having complete dysphagia. What is the best step in management?
A. Esophagojejunostomy
B. Stent placement
C. Feeding jejunostomy
D. Gastrojejunostomy
----------------------------------------
38. What is the probable diagnosis for a patient who experiences a sudden occurrence of vomiting
blood (hematemesis), takes aspirin for arthritis, consumes alcohol occasionally, and reports sporadic
abdominal pain?
A. Mallory-Weiss tear
B. Esophagitis
C. Peptic ulcer
D. Esophageal varices
----------------------------------------
39. An elderly woman underwent surgical resection of her bowel; after a few months, she presents with
easy fatiguability and tiredness. Blood smear evaluation shows macrocytic anemia. Resection of all the
following can cause this, except:
A. Jejunum
B. Duodenum
C. Ileum
D. Stomach
----------------------------------------

Page 10

881
40. Which of the following is not a perineal approach for the condition shown in the image?

A. Ripstein’s procedure
B. Delorme’s operation
C. Thiersch operation
D. Altemeier’s procedure
----------------------------------------
41. Which of the following statements about psoas abscess are true? Fluctuant swelling in the hip Cold
abscess can develop from Pott’s abscess Pain on passive extension of hip joint is seen It develops
more commonly through hematogenous route in immunocompromised patients and elderly
A. 1 and 2
B. 1, 3 and 4
C. 1, 2, 3 and 4
D. 2, 3 and 4
----------------------------------------
42. Which of the following would be the most commonly encountered ectopic tissue in Meckel’s
diverticulum?
A. Gastric and pancreatic
B. Colonic and pancreatic
C. Gastric and colonic
D. Thyroid and lung
----------------------------------------
43. A patient who had undergone appendicectomy 2 days ago presents with severe abdominal pain.
On exploration, there is a collection of blood in the abdomen. Which of the following arteries should
have been ligated during surgery?
A. Ileocolic artery
B. Right colic artery
C. Superior rectal artery
D. Superior mesenteric artery
----------------------------------------

Page 11

882
44. A 10-year old child with hereditary spherocytosis and hypersplenism presents to the hospital. A
splenectomy is planned for this child. All the following vaccines are recommended before splenectomy
except……..:
A. Pneumococcal vaccine
B. Typhoid vaccine
C. Haemophilus inflenzae B vaccine
D. Meningococcal vaccine
----------------------------------------
45. Identify the correct statement regarding gastric carcinoma A. It is mostly hereditary B. E-cadherin
adhesion defect is seen in diffuse type of gastric CA. C. Wnt signaling pathway defect is seen in
intestinal – type gastric CA. D. Depth of cancer has no role in prognosis.
A. A, C, D
B. B, C
C. B, C, D
D. A, D
----------------------------------------
46. What should be the subsequent step in managing a 32-year-old male patient who comes to the
surgical emergency department with a 2-day history of peptic ulcer disease, experiencing abdominal
pain, bloating, and difficulty passing stools? A. IV fluids B. CECT abdomen C. Exploratory laparotomy
D. Upper GI endoscopy Below is the provided x-ray film image.

A. A and B
B. A and C
C. A and D
D. A, B and C
----------------------------------------
47. Match the location of the appendix with the given descriptions in red colour:

Page 12

883
A. Retrocecal
B. Preileal
C. Subcecal
D. Pelvic
----------------------------------------
48. A surgeon examined a case of hernia, he was able to retract the hernial sac on examination and not
the content. Identify the type of hernia with the image given?

A. Incarcerated hernia
B. Reduction en masse
C. Sliding hernia
D. Maydl'shernia
----------------------------------------
49. A 53-year-old patient presents with a complaint of per rectal bleeding. The surgeon suspects a
possible colorectal pathology and wants to determine the most appropriate investigation for this patient.
Which of the following options would be the best choice?
A. Colonoscopy
B. CEA levels
C. CA-19-9 levels
D. CECT
----------------------------------------

Page 13

884
50. The image depicted below shows the outcome following a surgical intervention performed on a
patient presenting with rectal prolapse. Kindly identify the specific procedure employed.

A. Stapled hemorrhoidopexy
B. Well's procedure
C. Thiersch wiring
D. Altemeier repair
----------------------------------------
51. A patient presents with a hernia medial to the inferior epigastric artery. What is the most likely
diagnosis and appropriate management for the same?
A. Direct hernia and Bassini repair
B. Direct hernia and Lichtenstein mesh repair
C. Indirect hernia and Bassini repair
D. Indirect hernia and Lichtenstein mesh repair
----------------------------------------
52. A 54-year-old female patient presents with abdominal pain and bilious vomiting. She had a history
of abdominal hysterectomy 2 years back. On examination, abdomen auscultation revealed high-pitched
bowel sounds. Investigations showed air within the biliary tree. What is the most likely diagnosis?
A. Gallstone ileus
B. Adhesive intestinal obstruction
C. Ischemic enterocolitis
D. Diverticulitis
----------------------------------------
53. A patient was found to have a mass arising from the antrum of the stomach, which involves the
head of the pancreas and metastasis to the right lobe of the liver. How will you manage this patient?
A. Palliative gastrojejunostomy followed by chemotherapy
B. Radical gastrectomy
C. Whipple's procedure
D. Gastrectomy with right hepatectomy
----------------------------------------

Page 14

885
54. A baby playing unsupervised was brought with complaints of difficulty in swallowing for the last few
hours. The X-ray image is given below. What is the diagnosis?

A. Foreign body in the trachea


B. Foreign body in the esophagus
C. Artifact
D. Soft tissue calcification in the neck
----------------------------------------
55.
(or)
What is the most probable diagnosis for a 25-year-old patient who has been experiencing abdominal
pain and fever for the past three days, based on the provided chest X-ray taken in an upright position?
A. Hollow viscus perforation
B. Gastric volvulus
C. Liver abscess
D. Empyema thoracis
----------------------------------------
56. What is the most probable diagnosis for a 28-year-old patient who is an alcoholic and presents to
the hospital with symptoms of frequent vomiting, chest pain, fever, and hydropneumothorax?
A. Boerrhave syndrome
B. Tension pneumothorax
C. PUD perforation
D. Mallory Weiss tear
----------------------------------------
57. What could be the possible cause of sweating and diarrhea occurring within 20 minutes of eating in
a patient who has undergone gastrectomy?
A. Early dumping syndrome
B. Late dumbing syndrome
C. Hyperglycemia
D. Hypoglycemia

Page 15

886
----------------------------------------
58. A 6 months child presented with the following presentation. What is the management protocol?

A. Indication of surgery if not resolved by 2 years


B. Surgery after 5 year
C. Immediate surgery
D. Hernioplasty
----------------------------------------
59. The distal ileum was removed from a 20-year-old girl. Which absorption deficiency will be seen?
A. Iron
B. Folic acid
C. Copper
D. Bile salts
----------------------------------------
60. Which of the following is NOT a risk factor for Esophageal carcinoma?
A. GERD
B. Betel chewing
C. Oxalate food
D. Spicy food
----------------------------------------
61. Alvardo score of 2 is given for:
A. Nausea
B. Temperature
C. Tenderness in right lower quadrant
D. Migratory pain
----------------------------------------
62. 20 year old female previously diagnosed with Adenocarcinoma of the stomach, on examination
following image was seen. What is the probable diagnosis ?

Page 16

887
A. Virchow’s node
B. Irish node
C. Sister mary joseph nodule
D. Infected umbilical hernia
----------------------------------------
63. What stage of Hinchey's classification is indicated when a patient with clinical features suggestive
of peritonitis presents with a diverticular perforation and fecal peritonitis during surgery?
A. Stage 1
B. Stage 2
C. Stage 3
D. Stage 4
----------------------------------------
64. What is the likely diagnosis for a middle-aged man who comes to the emergency department with
upper abdominal pain following a large meal? During clinical examination, there is tenderness in the
upper abdomen, and a chest X-ray reveals widening of the mediastinum and pneumomediastinum.
A. Spontaneous perforation of esophagus
B. Perforation of peptic ulcer
C. Foreign body in esophagus
D. Rupture of emphysematous bulla of lung
----------------------------------------
65. What type of achalasia does a 32-year-old male with a diagnosis of achalasia cardia have, based
on the findings of high-resolution manometry which showed simultaneous, high-pressure contractions
of the distal esophagus?
A. Type II
B. Type I
C. Type IV
D. Type III
----------------------------------------

Page 17

888
66. What is the most prevalent condition that leads to the occurrence of heartburn and chest pain in a
patient?
A. Barrett's esophagus
B. Pneumonia
C. Myocardial infarction
D. Esophagitis
----------------------------------------
67. A baby playing unsupervised was brought with complaints of difficulty in swallowing for the last few
hours. The x-ray image is given below. What is the diagnosis?

A. Foreign body in the trachea


B. Foreign body in the esophagus
C. Foreign body in lungs
D. Foreign body in pleura
----------------------------------------
68. What is the diagnosis of a patient who presents with chronic small bowel diarrhea, with duodenal
biopsy showing villous atrophy, and positive results for anti-endomysial antibodies and IgA TTG
antibodies?
A. Short bowel syndrome
B. Whipple’s disease
C. Tropical sprue
D. Celiac sprue
----------------------------------------
69. What is the most appropriate next course of action in managing a male patient, aged 40, who
presents with widespread abdominal pain and has a confirmed history of Crohn's disease, as well as
rigidity and guarding observed during a physical examination, with ileal perforation being detected on
imaging?
A. Antibiotics and NPO
B. Upper GI endoscopy
C. Urgent surgical exploration with ileostomy
D. CT abdomen

Page 18

889
----------------------------------------
70. An elderly male patient presented with purulent discharge from an opening located in the perianal
region. The patient is put in a lithotomy position, and the opening is located at the 11 o’clock position, 2
cm from the anal margin. What will be the likely position of the internal opening, and where does the
internal opening start?
A. 12 o’clock and at external anal sphincter
B. 11 o’clock and at internal anal sphincter
C. 9 o’clock and at anal verge
D. 6 o’clock and crypt of the dentate line
----------------------------------------
71. Pressure greater than __ mm of Hg causes pressure sores.:
A. 33
B. 45
C. 54
D. 67
----------------------------------------
72. Which of the following statements are true about colon cancer? Right and left colon cancers can
behave differently Colon cancers can be hereditary Colon cancer risk can be reduced by metformin
Colon cancers frequently metastasize to the liver
A. 1, 2 and 3 only
B. 2, 3 and 4 only
C. 2 and 4 only
D. 1, 2, 3 and 4 only
----------------------------------------
73. Match the following: 1. Hirschsprung’s disease 2. Posterior urethral valve 3. Choledochal cyst 4.
Intussusception A. MRCP B. CT scan C. Rectal biopsy D. Micturating cystourethrogram
rung’s disease2. Posterior urethral valve3. Choledochal cyst4. Intussusception A. MRCPB. CT scanC. Rectal biopsyD. Micturating cystou

A. 1-C, 2-D, 3-B, 4-A


B. 1-A, 2-D, 3-B, 4-C
C. 1-C, 2-D, 3-A, 4-B
D. 1-D, 2-C, 3-A, 4-B
----------------------------------------
74. What is the diagnosis for a patient who exhibits symptoms of dysphagia, experiencing pain behind
the breastbone, and regurgitation accompanied by coughing following meals?
A. Brown Patterson Kelly syndrome
B. Achalasia
C. Esophageal diverticula
D. Reflux esophagitis

Page 19

890
----------------------------------------
75. What is the likely diagnosis for a 40-year-old patient who has experienced the passage of mucus
and blood in their stool, with inflammation predominantly observed in the rectum during sigmoidoscopy,
and the presence of crypt abscess detected during biopsy?
A. Ulcerative colitis
B. Irritable bowel syndrome
C. Crohn disease
D. Pseudomembranous colitis
----------------------------------------
76. Which of the following structures are incised by a surgeon during an appendectomy performed
initially with a gridiron incision? External oblique Internal oblique Transversus abdominis Rectus
abdominis
A. 1, 3, 4
B. 2, 3, 4
C. 1, 2, 3
D. 1, 2, 4
----------------------------------------
77. A patient presented with retrosternal chest pain radiating to the jaw and dysphagia. A barium
swallow was performed which showed the following image. Which of the following is the most likely
diagnosis?

A. Infectious esophagitis
B. Diffuse esophageal spasm
C. Barrette’s oesophagus
D. Esophageal stricture
----------------------------------------
78. The provided barium swallow indicates

Page 20

891
A. Achalasia cardia
B. Diffuse esophageal spasm
C. Nutcracker oesophagus
D. Hypertensive lower esophageal sphincter
----------------------------------------
79. Which of the following conditions has a barium enema as shown in the picture below?

A. Multiple colonic polyps


B. Ulcerative colitis
C. Diverticulosis
D. Irritable bowel syndrome
----------------------------------------
80. Dohlman's procedure is for:
A. Meckel's diverticulum
B. Zenker's diverticulum
C. Bochadlek hernia
D. Menetrier's disease
----------------------------------------
81. Which of the following statements is false regarding rectal carcinoma?
A. Hartmann's operation is done in elderly debilitated patients

Page 21

892
B. Adenocarcinoma is a rare variant of rectal carcinoma
C. Early morning spurious diarrhoea and tenesmus can occur
D. Growth confined to the rectal mucosa is stage A of modified Duke's staging
----------------------------------------
82. A gasless abdomen is seen in?
A. Ulcerative colitis
B. Intussusception
C. Acute pancreatitis
D. Crohn's disease
----------------------------------------
83. What does the presence of serpiginous ulcers during esophageal endoscopy in a patient indicate in
terms of diagnosis?
A. CMV esophagitis
B. Candida esophagitis
C. Herpetic esophagitis
D. Radiation esophagitis
----------------------------------------
84. In the OPD, a 35-year-old male presented with concerns of passing dark-colored stools. Following
an endoscopy, the patient was diagnosed with an upper gastrointestinal (GI) bleed. Which anatomical
structure serves as a distinguishing point to differentiate it from a lower GI bleed?
A. Ampulla of Vater
B. Ligament of Treitz
C. Superior duodenal flexure
D. IIeocaecal junction
----------------------------------------
85. An abdominal mass in a patient with congenital hypertrophic pyloric stenosis is best
seen_________:
A. During feeding
B. Soon after birth
C. During palpation, over the left hypochondirum
D. During palpation, over the epigastrium
----------------------------------------
86. What is the accurate statement regarding Crohn's disease?
A. For effective treatment of a Crohn’s mass, only antibiotics are prescribed
B. Symptomatic relapse after surgery is not related to the presence of disease during resection
C. The proximal caecum is most commonly involved in Crohn’s disease

Page 22

893
D. On endoscopic examination continuous lesions are visualised, which are diagnostic of Crohn’s
disease
----------------------------------------
87. A 65-year-old man presents with abdominal pain for the past 6 hours, BP- 89/50 mmHg, RR-
24/min, saturation 92% with 6 L of oxygen. The radiograph shows extraluminal air in the abdomen.
Which of the following must be done before shifting this patient into the OT?
A. Intubate the patient
B. Insert central venous catheter as initial IV access of choice
C. 2 litres of crystalloid infusion
D. Confirm the diagnosis with CECT
----------------------------------------
88. What is the earliest indication of Crohn's disease among the options provided?
A. Cobblestone appearance
B. Deep fissured ulcers
C. Rose-thorn appearance
D. Aphthous ulcers
----------------------------------------
89. Which of the following statements accurately describes the Tillaux sign?
A. Mass is felt in the periumbilical region
B. Mass is brilliantly transilluminant
C. Mass is mobile perpendicular to mesentery attachment line
D. Fluctuation is positive
----------------------------------------
90. A child is diagnosed with congenital hypertrophic pyloric stenosis, which among the following can
be seen in this child? A. Bilious vomiting B. Visible peristalsis from right to left C. Hypochloremia and
hypokalemia D. Metabolic alkalosis
A. B, C and D
B. A and D only
C. A, B and D
D. A, B, C and D
----------------------------------------
91. Heller’s operation is done for _______:
A. Achalasia cardia
B. Carcinoma esophagus
C. Esophageal stricture
D. Hypertrophic pyloric stenosis
----------------------------------------

Page 23

894
92. Which of the following statements about Barrett's esophagus is incorrect?
A. It is the intestinal metaplasia of esophagus
B. Mucus secreting foveolar cells are characteristic
C. Alician blue is used in the staining of biopsy specimen
D. Adenocarcinoma of O-G junction is a complication
----------------------------------------
93. A gross specimen of the colon obtained from a 14-year-old boy is shown below. Which of the
following is the most likely diagnosis?

A. Familial adenomatous polyposis


B. Colon cancer
C. Juvenile polyp
D. Ulcerative colitis
----------------------------------------
94. During a routine check-up, an elderly male was discovered to have a rectal mass during a digital
rectal examination, indicating the possibility of rectal carcinoma. Despite normal bowel habits, the next
course of action for managing this patient should be:
A. CT scan of the abdomen
B. Rigid sigmoidoscopy and biopsy
C. Full colonoscopy and biopsy
D. Barium enema
----------------------------------------
95. A 30-year-old female presented with a low-grade fever of 1 week duration associated with loss of
appetite and weight loss. Barium meal follow-through image of the patient is given below. What is the
probable diagnosis?

Page 24

895
A. Ileocaecal TB
B. Ulcerative colitis
C. Crohn’s disease
D. Carcinoma caecum
----------------------------------------
96. Which of the following statements is false about carcinoid tumors of appendix?
A. Size appears to be the best predictor of malignant behavior and metastatic potential
B. Arise from Kulchitsky cells of the crypts of Leiberkuhn
C. Most common location is the base of the appendix
D. Most common tumor of the appendix
----------------------------------------
97. Identify the incision marked as B.

A. Gridiron incision
B. Lanz incision
C. Rutherford – Morison incision
D. Rockey-Davis incison
----------------------------------------
98. Which of the following do not require instruments to examine?
A. Acute anal fissure

Page 25

896
B. Carcinoma rectum
C. Hemorrhoids
D. Rectal polyp
----------------------------------------
99. What is the name of this triangle?

A. Gastrinoma triangle
B. Calot’s triangle
C. Doom’s triangle
D. Cholecystohepatic triangle
----------------------------------------
100. A patient who underwent Nissen’s fundoplication was seen in the ward during evening rounds. He
is asymptomatic at present with stable vitals. What is the next line of management for this patient?
A. Nasogastric tube to prevent gas bloat syndrome
B. Insert chest tube to prevent pneumothorax
C. Monitor vitals
D. Imaging to rule out perforation
----------------------------------------
101. Mandatory exploration is required in the injuries of which zones of retroperitoneum:

A. 1
B. 2

Page 26

897
C. 3
D. 1 & 3
----------------------------------------
102. A 35-year-old man arrives at the emergency department with a concern of vomiting blood. He
mentions a recent pattern of excessive alcohol consumption. Upon examination, his vital signs are
steady, and he has a pale appearance. There are no indications of ongoing bleeding. The results of his
liver function tests are within the normal range. To further assess the situation, an upper
gastrointestinal endoscopy is scheduled. What is the most probable diagnosis?
A. Esophageal varices
B. Gastric ulcer
C. Mallory-Weiss tear
D. Alcoholic gastritis
----------------------------------------
103. Which of the following accurately represents Bormann's classification for gastric carcinoma?
A. Type I: Polypoid tumor with a small ulcerated component
B. Type II: Ulcerated tumor with infiltrative margins
C. Type III: Diffusely infiltrative tumor with linitis plastica appearance
D. Type IV: Mucinous carcinoma with extensive peritoneal metastases
----------------------------------------
104. A 60-year-old male presents to the emergency department with abdominal pain and vomiting. He
reports a 2-day history of constipation and bloating. On examination, he appears uncomfortable, and
his abdomen is distended with high-pitched bowel sounds. X-ray reveals dilated loops of the small
bowel. What is the most likely cause of his symptoms?

A. Colonic malignancy
B. Small bowel obstruction
C. Acute pancreatitis
D. Gastric ulcer
----------------------------------------
105. Which of the following findings of colonoscopy is highly malignant?
A. Multiple flat polyp about 100

Page 27

898
B. Single pedunculatedpolyp
C. Multiple pedunculated polyp
D. Solitary flat polyp
----------------------------------------
106. What is the likely diagnosis for a patient who has been experiencing fever and pain in the lower
right abdomen for the past 5 days, with the presence of a tender mass in the right iliac fossa and an
elevated leukocyte count?
A. Appendicitis
B. Hydronephrosis
C. Tuberculosis
D. Carcinoma colon
----------------------------------------
107. What structures are contained within the anterior pararenal space?
A. Head, neck, and body of the pancreas
B. Adrenal gland, kidney, ureters
C. Aorta, inferior vena cava
D. Ascending and descending colon
----------------------------------------
108. Which of the following is not a boundary of the triangle of doom?
A. Vas deferens
B. Peritoneal Fold
C. Iliopubic tract
D. Spermatic vessels
----------------------------------------
109. An abdominal aortic aneurysm is operated when its size is greater than____?
A. 5 cm
B. 6 cm
C. 5.5 cm
D. 6.5 cm
----------------------------------------
110. What is the initial approach to managing a 50-year-old male who arrives at the emergency
department with a blood pressure reading of 160/100 mmHg, a heart rate of 120, and a CECT scan
indicating a Stanford A type aortic dissection?
A. Surgery
B. LMWH
C. Surgical repair
D. Vitamin K inhibitors

Page 28

899
----------------------------------------
111. A condition requires the surgery depicted in the image shown below. All the following are usually
seen associated with this condition except?

A. Absent kidney
B. Hepatic artery anomalies
C. Malrotation of gut
D. Polysplenia
----------------------------------------
112. What is the best position for a conscious patient to be in during the insertion of a Ryle's tube?
A. Supine with neck flexion
B. Supine with neck extension
C. Sitting with neck flexion
D. Sitting with neck extension
----------------------------------------
113. Where is the location of Killian's dehiscence, which causes Zenker's diverticulum, in a 40-year-old
male presenting with dysphagia, regurgitation, and a foul-smelling mouth, and diagnosed after further
evaluation in the emergency department?
A. Below Superior constrictor
B. Between Inferior constrictor
C. Above the cricopharyngeal muscle
D. Below the upper 1/3rd of the smooth muscle of the esophagus
----------------------------------------
114. Which of the following cancers rarely metastasizes to the liver?
A. Colon
B. Breast
C. Prostate
D. Melanoma
----------------------------------------

Page 29

900
115. A 3-day-old neonate presented with recurrent bilious vomiting. An X-ray the erect abdomen is
given below. What is the next best step in managing this condition?

A. CT abdomen
B. Gastrograffin follow-through
C. USG abdomen
D. Explorative laparotomy
----------------------------------------
116. In the retroperitoneal space, which tumor is most frequently observed?
A. Liposarcoma
B. Rhabdomyosarcoma
C. Synovial sarcoma
D. Neurofibroma
----------------------------------------
117. A man in his middle age arrives with a gradual onset of challenges while swallowing both solid and
liquid substances over the course of 8 months. The results of a barium swallow test conducted are
provided below. What additional examinations should be conducted for this individual?

A. Upper Gl endoscopy & Manometry


B. Upper Gl endoscopy & PET scan
C. Upper Gl endoscopy & CECT
D. Upper Gl endoscopy and 24 hour PH monitoring

Page 30

901
----------------------------------------
118. Identify the following?

A. CA bladder
B. Right inguinal hernia
C. Left inguinal hernia
D. None of the above
----------------------------------------
119.
(or)
A 40-year-old man presents to the ER with severe abdominal pain. On examination, his HR is 110 bpm,
BP-88/60 mmHg. Guarding is present on per abdomen examination. Erect abdominal X-ray is shown
below. Which of the following would you proceed with, for the management of this case? A. USG
abdomen B. IV fluids C. Exploratory laparotomy D. Elective surgery E. Conservative management
A. B & C
B. B & D
C. A, B & C
D. A, B & E
----------------------------------------
120. What could be a possible diagnosis for a 55-year-old man with ischemic heart disease who is
experiencing sudden, intense, widespread abdominal pain, along with repeated episodes of vomiting?
During examination, the patient has a pulse rate of 118 beats per minute, displaying an irregularly
irregular rhythm, and a blood pressure reading of 90/60 mmHg. The abdomen appears soft, not
swollen, and there are no bowel sounds.
A. Acute severe pancreatitis
B. Ruptured aortic aneurysm
C. Acute mesenteric ischemia
D. Peptic ulcer perforation
----------------------------------------
121. Which mesh is the most suitable for hernioplasty?
A. Heavyweight with small pores

Page 31

902
B. Heavyweight mesh with large pores
C. Lightweight mesh with small pores
D. Lightweight mesh with large pores
----------------------------------------
122. Which gene mutations are implicated in colon carcinoma?
A. APC, BRCA, k-ras
B. APC, k-ras, p53
C. MYh, K-Ras, APC
D. MYH, p53, DCC
----------------------------------------
123. Which of the following is the probable source of origin for this metastasis, considering the woman's
diagnosis of metastatic liver adenocarcinoma with CK 7 (-), CK20 (+), and CDX-1 (+) IHC markers?
A. Colon carcinoma
B. Breast Carcinoma
C. Pancreatic carcinoma
D. Ovarian carcinoma
----------------------------------------
124. Which of the following statements is correct regarding Peutz-Jeghers syndrome? Autosomal
dominant disease Defect in the STK11 gene Melanin spots on digits and perianal skin Autosomal
recessive Congenital hypertrophy of retinal pigment epithelium is seen
A. 2, 4 and 5
B. 1, 2 and 3
C. 1, 3 and 4
D. 2, 3 and 5
----------------------------------------
125. You are doing an emergency laparotomy for a case of intestinal obstruction. Which organ will you
first visualize to know if this is a case of small bowel or large bowel obstruction?
A. Ileum
B. Caecum
C. Jejunum
D. Sigmoid colon
----------------------------------------
126. A patient comes with active variceal bleeding. Which of the following is not done?
A. Platelet transfusion
B. Arrange pRBC
C. IV Octreotide
D. Urgent Endoscopy

Page 32

903
----------------------------------------
127. Grey-Turner's sign in seen in:
A. Acute appendicitis
B. Acute pancreatitis
C. Acute cholecystitis
D. Acute hepatitis
----------------------------------------
128. All of the following are features of Zollinger-Ellison syndrome except:
A. Intractable peptic ulcers
B. Severe diarrhea
C. Arise from beta cell tumors of the pancreas
D. Very high acid output
----------------------------------------
129. A most common type of gastric sarcoma:
A. Lipoma
B. Glomus tumor
C. Leiomyosarcoma
D. Leioblastoma
----------------------------------------
130. Most common type of intussusception:
A. Colocolic
B. Ileoileal
C. Ileocolic
D. Ileal
----------------------------------------
131. A patient after heavy drinking of alcohol presents with multiple episodes of vomiting and
hematemesis. The most likely diagnosis could be:
A. Mallory-Weiss syndrome
B. Esophageal carcinoma
C. Achalasia cardia
D. Boerhaave's syndrome
----------------------------------------
132. The treatment of choice for inguinal hernia in infants is:
A. Herniotomy
B. Herniorrhaphy
C. TRUSS

Page 33

904
D. Hernioplasty
----------------------------------------
133. Serum acid phosphate is raised in:
A. Osteosarcoma
B. Prostatic carcinoma
C. Paget's disease
D. Hyperparathyroidism
----------------------------------------
134. What is the probable diagnosis for a 5-year-old child with abdominal swelling, mild discomfort, no
pain or vomiting, and a fluctuant soft swelling that moves freely in a plane perpendicular to the
mesentery attachment, as seen in the outpatient department?
A. Peritoneal cyst
B. Lipoma
C. Chylolymphatic cyst
D. Pseudocyst
----------------------------------------
135. Which of the following is the most probable diagnosis for a 38-year-old male patient who visited
the surgery OPD with a history of occasional discharge of pus with a slight presence of blood from an
external opening in the perianal region, despite multiple attempts at drainage? The examination of the
perianal region is depicted below.

A. Pilonidal sinus
B. Perianal abscess
C. Hemorrhoids
D. Fistula-in-ano
----------------------------------------
136. A infant of 12 weeks, presented with the following defect in the umbilical area. Identify the
condition.

Page 34

905
A. Omphalocele
B. Gastroschisis
C. Umbilical hernia
D. Paraumbilical hernia
----------------------------------------
137. What is the diagnosis from the given abdominal x-ray?

A. Ureterocoele
B. Pelvic bone fracture
C. Rectal foreign body
D. Bladder stone
----------------------------------------
138. A 40-year-old male presents with diffuse abdominal pain. He has a past history of peptic ulcer
disease. Physical examination reveals rigidity. X-ray of the abdomen is shown below. What is the next
best step in the management of this patient?

Page 35

906
A. Antibiotics and NPO(nothing by mouth)
B. Upper GI endoscopy
C. Urgent surgical exploration
D. CT abdomen
----------------------------------------
139. What is the diagnosis of a neonate who is brought to the hospital with complaints of forceful
vomiting that is not accompanied by bile after feeding? The child appears lethargic and dehydrated,
and abdominal examination shows visible movements of the stomach. A barium meal was conducted
and the provided image is shown below.

A. Hirschsprung’s disease
B. Congenital hypertrophic pyloric stenosis
C. Jejunal atresia
D. Duodenal atresia
----------------------------------------
140. What electrolyte imbalance is commonly observed in patients with this postoperative condition, as
depicted in the provided x-ray?

Page 36

907
A. Hypocalcemia
B. Hypokalemia
C. Hyperkalemia
D. Hypercalcemia
----------------------------------------
141. A 70-year-old woman had her routine follow up and her USG of the liver is shown below. She is
asymptomatic at present but has a history of mild right hypochondriac pain which is relieved by taking
analgesics. What is the most likely diagnosis?

A. Gallstones
B. Amoebic liver abscess
C. Hydatid cyst
D. Acute cholecystitis
----------------------------------------
142. All are true about intestinal transplantation except?
A. Most of the grafts are multi-visceral grafts.
B. It is not widely used due to vigorous rejection reactions.
C. The most dangerous complication is sepsis.
D. The most common indication is the ischemia
----------------------------------------
143. Which of the following statements regarding superior mesenteric artery syndrome is false?

Page 37

908
A. Third part of the duodenum is compressed
B. Angle between SMA and aorta is more than 70 degrees
C. Surgical management is by duodenojejunostomy
D. Also known as Wilkie's Syndrome
----------------------------------------
144. Most common immediate complication of splenectomy:
A. Hemorrhage
B. Fistula
C. Bleeding from gastric mucosa
D. Pancreatitis
----------------------------------------
145. A 6-year-old child is brought with a high fever with rigors for 5 days with pain in the right
hypochondrium. On examination, the patient is anicteric, and tenderness is noted in the
■■■■■■■right upper quadrant. What is the best investigation for this case?
A. USG
B. Serology
C. SGOT/LFT
D. CECT
----------------------------------------
146. What is the most effective treatment approach for anal canal carcinoma?
A. Chemoradiation
B. Surgery
C. Radiation Therapy
D. Chemotherapy
----------------------------------------
147. A 32-year-old female presented to the OPD with complaints of abdominal pain, diarrhea, and
weight loss. On further evaluation, the patient was found to have multiple strictures of small bowel with
interspersed areas of normal mucosa in between. Which of the following is false regarding the given
condition?
A. No recurrence after surgery
B. Aphthous ulcer
C. Skip lesions
D. Fistula formation
----------------------------------------
148. What is the urgent indication for performing laparotomy in a 30-year-old man who presents with
right iliac fossa pain lasting for four days, as depicted in the provided USG image?

Page 38

909
A. Rising pulse rate
B. Increasing abdominal pain
C. Increase in the size of mass
D. All of the above
----------------------------------------
149. Provide your opinion on the diagnosis based on a radiographic image displayed of a 65-year-old
male experiencing acute abdominal pain.

A. Ileocolic intussusception
B. Sigmoid volvulus
C. Toxic megacolon
D. Colocolic intussusception
----------------------------------------
150. A 30-year-old man visits the outpatient department with concerns of pain in the lower abdomen
and rectal bleeding. After conducting additional tests, it is found that the patient has several shallow
ulcers in the rectum that extend upwards to involve the entire colon. Upon examination of the tissue
samples, it is discovered that only the mucosa and submucosa are affected. Which of the following
complications associated with the aforementioned conditions is not harmful?
A. Toxic megacolon
B. Colonic polyp
C. Colonic obstruction
D. Massive colonic bleeding

Page 39

910
----------------------------------------
151. What is the most common complication of ERCP?
A. Duodenal perforation
B. Cholangitis
C. Hemorrhage
D. Pancreatitis
----------------------------------------
152. A 45-year-old male presents to the emergency department with severe abdominal pain radiating to
the back. Laboratory investigations reveal elevated serum amylase and lipase levels. Which of the
following should be avoided in the management of the given condition?
A. Nil per oral to avoid pancreatic stimulation
B. Administration of 5% dextrose
C. Parenteral nutrition is preferred for 2-3 days over the nil per oral (NPO) approach
D. Early initiation of oral feedings
----------------------------------------
153. A 55-year-old female patient with chronic pancreatitis undergoes a puestow procedure. What is
the most preferred route for supplementary nutrition in this patient?
A. TPN
B. Oral feeding
C. Feeding gastrostomy
D. Feeding jejunostomy
----------------------------------------

Correct Answers
Question Correct Answer

Question 1 1
Question 2 3
Question 3 1
Question 4 2
Question 5 2
Question 6 3
Question 7 1
Question 8 1
Question 9 4
Question 10 2
Question 11 4

Page 40

911
Question 12 3
Question 13 1
Question 14 3
Question 15 2
Question 16 4
Question 17 2
Question 18 2
Question 19 1
Question 20 2
Question 21 1
Question 22 3
Question 23 1
Question 24 4
Question 25 1
Question 26 4
Question 27 2
Question 28 4
Question 29 1
Question 30 3
Question 31 2
Question 32 2
Question 33 4
Question 34 4
Question 35 2
Question 36 3
Question 37 3
Question 38 3
Question 39 2
Question 40 1
Question 41 3
Question 42 1
Question 43 1
Question 44 2
Question 45 2
Question 46 2

Page 41

912
Question 47 1
Question 48 2
Question 49 1
Question 50 3
Question 51 2
Question 52 1
Question 53 1
Question 54 2
Question 55 1
Question 56 1
Question 57 1
Question 58 2
Question 59 4
Question 60 3
Question 61 3
Question 62 3
Question 63 4
Question 64 1
Question 65 4
Question 66 4
Question 67 2
Question 68 4
Question 69 3
Question 70 2
Question 71 1
Question 72 4
Question 73 3
Question 74 3
Question 75 1
Question 76 3
Question 77 2
Question 78 1
Question 79 3
Question 80 2
Question 81 2

Page 42

913
Question 82 3
Question 83 1
Question 84 2
Question 85 1
Question 86 2
Question 87 3
Question 88 4
Question 89 3
Question 90 1
Question 91 1
Question 92 2
Question 93 1
Question 94 3
Question 95 1
Question 96 1
Question 97 2
Question 98 1
Question 99 1
Question 100 3
Question 101 1
Question 102 3
Question 103 1
Question 104 2
Question 105 1
Question 106 1
Question 107 1
Question 108 3
Question 109 3
Question 110 1
Question 111 1
Question 112 3
Question 113 2
Question 114 3
Question 115 2
Question 116 1

Page 43

914
Question 117 1
Question 118 3
Question 119 1
Question 120 3
Question 121 4
Question 122 2
Question 123 1
Question 124 2
Question 125 2
Question 126 1
Question 127 2
Question 128 3
Question 129 3
Question 130 3
Question 131 1
Question 132 1
Question 133 2
Question 134 3
Question 135 4
Question 136 3
Question 137 3
Question 138 3
Question 139 2
Question 140 2
Question 141 1
Question 142 1
Question 143 2
Question 144 1
Question 145 2
Question 146 1
Question 147 1
Question 148 4
Question 149 4
Question 150 2
Question 151 4

Page 44

915
Question 152 2
Question 153 1

Solution for Question 1:


Correct Option A - B, C:
• CA colon: It is recommended to examine a minimum of 12 lymph nodes for pathological staging of
colon cancer.
• CA gall bladder: It is recommended to examine a minimum of 6 lymph nodes for pathological staging
of stomach cancer.
CA colon: It is recommended to examine a
minimum of 12 lymph nodes for pathological staging of colon cancer.
CA gall bladder: It is recommended to examine a minimum of 6
lymph nodes for pathological staging of stomach cancer.
Incorrect Options:
• The minimum number of lymph nodes to be examined for effective staging are: Stomach Carcinoma:
16 Lymphnodes Colon Carcinoma: 12 Lymphnodes Gall Bladder Carcinoma: 6 Lymph Nodes
Breast Carcinoma: 10 Lymph Nodes
• Stomach Carcinoma: 16 Lymphnodes
• Colon Carcinoma: 12 Lymphnodes
• Gall Bladder Carcinoma: 6 Lymph Nodes
• Breast Carcinoma: 10 Lymph Nodes
• So, Options B, C, D are incorrect choices
The minimum number of lymph nodes to be examined for effective staging are:
• Stomach Carcinoma: 16 Lymphnodes
• Colon Carcinoma: 12 Lymphnodes
• Gall Bladder Carcinoma: 6 Lymph Nodes
• Breast Carcinoma: 10 Lymph Nodes
Stomach Carcinoma: 16 Lymphnodes
Colon Carcinoma: 12 Lymphnodes
Gall Bladder Carcinoma: 6 Lymph Nodes
Breast Carcinoma: 10 Lymph Nodes
So, Options B, C, D are incorrect choices

Solution for Question 2:


Correct Option C - A-3, B-1, C-2, D-4 :
Let's match the sutures with their respective applications:

Page 45

916
• Nylon 3-0: Midline laparotomy skin wound closure.
• Polyglactin: Bowel anastomosis.
• Polypropylene 1-0: Suturing linea alba in the midline.
• Polypropylene 2-0: Fixing mesh in hernia surgery.
Nylon 3-0: Midline laparotomy skin wound closure.
Polyglactin: Bowel anastomosis.
Polypropylene 1-0: Suturing linea alba in the midline.
Polypropylene 2-0: Fixing mesh in hernia surgery.

Solution for Question 3:


Correct Option A Crystalline cholesterol monohydrate:
• Gallstones are solid deposits that form in the gallbladder, a small organ located beneath the liver.
They can be composed of various substances, but the most common type of gallstone seen is made up
of crystalline cholesterol monohydrate.
• Crystalline cholesterol monohydrate stones are yellowish in color and make up approximately 80% of
all gallstones. They form when there is an imbalance in the components of bile, a fluid produced by the
liver and stored in the gallbladder. In this case, there is an excess of cholesterol compared to the
amount of bile salts and other substances needed to keep cholesterol dissolved.
Incorrect Options
Option B Crystalline cholesterol dihydrate:
• While cholesterol dihydrate crystals can be present in gallstones, they are not the most common type.
Cholesterol dihydrate stones are less common and often coexist with cholesterol monohydrate stones.
Option C Amorphous cholesterol dihydrate:
• Amorphous cholesterol dihydrate is a non-crystalline form of cholesterol that does not commonly
contribute to the formation of gallstones. It is less frequently seen as a major component of gallstones.
Option D Amorphous cholesterol monohydrate:
• Amorphous cholesterol monohydrate is another non-crystalline form of cholesterol that is not typically
associated with the formation of gallstones. It is less commonly seen as a major component of
gallstones.

Solution for Question 4:


Correct Option B
- McBurney point corresponds to the site of maximal tenderness in acute appendicitis.:
• McBurney point is approximately one-third of the distance from the anterior superior iliac spine to the
umbilicus in the right lower quadrant. It is the site where the appendix is most likely to be inflamed and
where maximal tenderness is commonly elicited during physical examination in cases of acute

Page 46

917
appendicitis.
McBurney point is approximately one-third of the distance from the anterior superior iliac spine to the u
mbilicus in the right lower quadrant. It is the site where the appendix is most likely to be inflamed and w
here maximal tenderness is commonly elicited during physical examination in cases of acute appendici
tis.
Incorrect Options:
Option A - McBurney point is located in the left lower quadrant of the abdomen: McBurney point is not i
n the left lower quadrant but rather in the right lower quadrant of the abdomen. It is a
specific anatomical landmark used to help locate the appendix.
Option C - McBurney point is used to assess rebound tenderness in acute appendicitis: Rebound tend
erness is assessed by applying pressure at a different location, typically in the left lower quadrant, and
observing if there is increased pain when the pressure is released. McBurney's point is not specifically
used to assess rebound tenderness.
Option D - McBurney point is located at the umbilicus in acute appendicitis: McBurney point is not loca
ted at the umbilicus but rather in the right lower quadrant.

Solution for Question 5:


Correct Option B - USG Abdomen:
• Ultrasound (USG) of the abdomen is a noninvasive imaging technique that uses sound waves to
produce real-time images of the abdominal organs. It is commonly used in pediatric patients to evaluate
various abdominal conditions. In the case of umbilical discharge, it can help identify any underlying
abnormalities in the umbilicus, such as an umbilical hernia, patent urachus, or infection.
Ultrasound (USG) of the abdomen is a noninvasive imaging technique that uses sound waves to produ
ce real-time images of the abdominal organs. It is commonly used in pediatric patients to evaluate vari
ous abdominal conditions. In the case of umbilical discharge, it can help identify any underlying abnor
malities in the umbilicus, such as an umbilical hernia, patent urachus, or infection.
Incorrect Options:
Option A - MRI Abdomen: Magnetic Resonance Imaging (MRI) of the abdomen is a detailed imaging te
chnique that uses strong magnetic fields and radio waves to generate images of the abdominal organs.
While it can provide valuable information in certain cases, it is not typically the first-line investigation fo
r evaluating umbilical discharge in a 3-month-old child.
Option C - CECT: Contrast-enhanced Computed Tomography (CECT) is a
specialized imaging technique that involves injecting a
contrast material to enhance the visualization of blood vessels and organs during a computed tomogra
phy (CT) scan. While it can provide detailed information, it is usually reserved for specific indications a
nd is not typically the initial investigation for umbilical discharge in a pediatric patient.
Option D - Exploratory laparotomy: Exploratory laparotomy is a surgical procedure in which the abdom
en is opened to directly visualize and assess the abdominal organs. It is a more invasive and definitive
step in the management process and is generally not the first-line approach for umbilical discharge in a
3-month-old child.

Page 47

918
Solution for Question 6:
Correct Option C - Umbilical:
• The European Hernia Society (EHS) classification is a widely used system for categorizing different
types of hernias. The M3 classification specifically refers to incisional hernias occurring at the
umbilicus.
The European Hernia Society (EHS) classification is a widely used system for categorizing different typ
es of hernias. The M3 classification specifically refers to incisional hernias occurring at the umbilicus.
EHS classification (European Hernia Society Classification)
Midline (M1 - M5)
Location
M1: Sub xiphoidal
3 cm from xiphoid process
M2: Epigastric
3 cm below xiphoid to 3 cm above umbilicus
M3: Umbilical
3 cm above and below umbilicus
M4: Infra umbilical
3 cm below umbilicus and till 3 cm above pubis
M5: Suprapubic
3 cm cranial to the pubis
Incorrect Options:
Option A - Epigastric: Epigastric hernias are classified separately in the EHS classification under the M
2 category.
Option B - Infraumbilical: Infraumbilical hernias are also classified separately under the M4 category.
Option D - Suprapubic: Suprapubic hernias are also classified separately under the M5 category.

Solution for Question 7:


Correct Option A - A, C and D:
• (A) Sengstaken-Blakemore tube: The Sengstaken-Blakemore tube is a device used to stop
esophageal variceal bleeding. It consists of a gastric balloon and an esophageal balloon that can be
inflated to apply pressure and tamponade the bleeding varices. This device is effective in providing
temporary control of bleeding.
• (B) Minnesota tube: The Minnesota tube is a variation of the Sengstaken-Blakemore tube. The
Minnesota tube has a fourth port that pulls out air and fluid from the esophagus. This helps stop you
from breathing fluids into your lungs during this procedure.
• (C) Upper GI endoscope: Upper GI endoscopy is a crucial tool in managing esophageal variceal
bleeding. While it does not directly stop the bleeding, it helps visualize the varices and allows for

Page 48

919
interventions such as band ligation or sclerotherapy. These procedures aim to prevent re-bleeding in
the future. So, option C is correct.
• (D) Upper GI endoscope (repeated): Since upper GI endoscopy is already mentioned in option C,
repeating it in option D doesn't provide any additional information. Therefore, option D is correct.
(A) Sengstaken-Blakemore tube: The Sengstaken-Blakemore tube is a
device used to stop esophageal variceal bleeding. It consists of a gastric balloon and an esophageal b
alloon that can be inflated to apply pressure and tamponade the bleeding varices. This device is effecti
ve in providing temporary control of bleeding.
(B) Minnesota tube: The Minnesota tube is a
variation of the Sengstaken-Blakemore tube. The Minnesota tube has a fourth port that pulls out air an
d fluid from the esophagus. This helps stop you from breathing fluids into your lungs during this proced
ure.
(C) Upper GI endoscope: Upper GI endoscopy is a crucial tool in managing esophageal variceal bleedi
ng. While it does not directly stop the bleeding, it helps visualize the varices and allows for intervention
s such as band ligation or sclerotherapy. These procedures aim to prevent re-bleeding in the future. So
, option C is correct.
(D) Upper GI endoscope (repeated): Since upper GI endoscopy is already mentioned in option C, repe
ating it in option D doesn't provide any additional information. Therefore, option D is correct.
Incorrect Options:
Option B, C & D are incorrect

Solution for Question 8:


Correct Option A - Bochdalek hernia:
• The scaphoid abdomen and decreased breath sounds on both sides seen in the neonate, along with
the X-ray findings, are consistent with Bochdalek hernia.
• Bochdalek hernia: Bochdalek hernia is a congenital diaphragmatic hernia that occurs due to a defect
in the posterolateral aspect of the diaphragm.
• This allows abdominal organs, such as the stomach, intestines, or liver, to herniate into the chest
cavity, often compressing the lungs and impairing lung development.
The scaphoid abdomen and decreased breath sounds on both sides seen in the neonate, along with th
e X-ray findings, are consistent with Bochdalek hernia.
Bochdalek hernia: Bochdalek hernia is a congenital diaphragmatic hernia that occurs due to a
defect in the posterolateral aspect of the diaphragm.
This allows abdominal organs, such as the stomach, intestines, or liver, to herniate into the chest cavity
, often compressing the lungs and impairing lung development.
Incorrect Options:
Option B - Pneumonia: Pneumonia refers to an infection and inflammation of the lungs, typically cause
d by bacteria, viruses, or fungi. While pneumonia can present with decreased breath sounds on one si
de of the chest and respiratory symptoms, the scaphoid abdomen is not characteristic of pneumonia. T
he X-ray findings are also not consistent with pneumonia.

Page 49

920
Option C
- Gastric volvulus: Gastric volvulus occurs when the stomach twists or rotates abnormally, resulting in a
partial or complete obstruction of the stomach. It can cause upper abdominal pain and other symptoms
, but the scaphoid abdomen and decreased breath sounds on both sides seen in the neonate do not ali
gn with the typical presentation of gastric volvulus. The X-ray findings are also not consistent with gastr
ic volvulus.
Option D - Eventration of diaphragm: Eventration of the diaphragm refers to a condition where a part of
the diaphragm muscle is abnormally thin or weak, causing it to protrude upwards into the chest cavity.
This condition is usually asymptomatic and discovered incidentally. The symptoms described in the ne
onate, along with the X-ray findings, are not indicative of eventration of the diaphragm.

Solution for Question 9:


Correct Option D - Resuscitation and laparotomy:
• The chest X-ray shows gas under the diaphragm, which is indicative of hollow viscus perforation.
• In case of hollow viscus perforation, even if the patient is stable, surgery still needs to be carried.
• Prior to surgery, Resuscitation is important.
Incorrect Options:
Option A - Gastric lavage:
• Gastric lavage is a procedure used to remove ingested toxins or drugs from the stomach. It is not
indicated in this case, as the patient's symptoms and chest X-ray findings suggest a surgical
emergency.
Option B - Chest tube insertion:
• Chest tube insertion is typically used to manage pneumothorax or pleural effusion, but it is not
indicated in this case as the patient's symptoms and chest X-ray findings suggest a surgical emergency
in the abdomen.
Option C - Tracheostomy: Its indications are:
• Prolonged dependence on a ventilator for breathing.
• To bypass an obstructed upper airway.
• To clean and remove secretions from the airway.
• To deliver oxygen to the lungs more easily or safely.

Solution for Question 10:


Correct Option B - Subhepatic:
• An intraperitoneal abscess is a collection of pus within the abdominal cavity, often resulting from
localized infections in abdominal organs or tissues.
• The most frequently occurring location for an intraperitoneal abscess to develop is the subhepatic
region after pelvic abscess.

Page 50

921
• Morrison pouch, also known as the hepatorenal pouch, lies in the subhepatic space, making it a
common site for abscess formation.
• The presence of an intraperitoneal abscess can lead to symptoms such as fever, chills, rigors,
abdominal pain, nausea, vomiting, and a general feeling of illness.
• Diagnosis of an intraperitoneal abscess typically involves imaging studies like CT scans or
ultrasounds.
• Treatment often includes drainage of the abscess and administration of antibiotics to manage the
underlying infection.
• A pigtail catheter may be used for drainage, and in severe cases, surgery may be necessary to
remove infected tissue or organs.
An intraperitoneal abscess is a collection of pus within the abdominal cavity, often resulting from localiz
ed infections in abdominal organs or tissues.
The most frequently occurring location for an intraperitoneal abscess to develop is the subhepatic regio
n after pelvic abscess.
Morrison pouch, also known as the hepatorenal pouch, lies in the subhepatic space, making it a
common site for abscess formation.
The presence of an intraperitoneal abscess can lead to symptoms such as fever, chills, rigors, abdomi
nal pain, nausea, vomiting, and a general feeling of illness.
Diagnosis of an intraperitoneal abscess typically involves imaging studies like CT scans or ultrasounds.
Treatment often includes drainage of the abscess and administration of antibiotics to manage the unde
rlying infection.
A pigtail catheter may be used for drainage, and in severe cases, surgery may be necessary to remove
infected tissue or organs.
Incorrect Options:
Option A - Suprahepatic: While the suprahepatic region (above the liver) can be a site for certain abdo
minal conditions, such as liver abscesses or subphrenic abscesses, it is not the most frequently occurri
ng location for intraperitoneal abscesses.
Option C - Left lobe of liver: While the left lobe of the liver can be involved in certain intra-abdominal pa
thologies, such as liver abscesses, it is not the most frequent location for intraperitoneal abscess forma
tion.
Option D - Left side below diaphragm: While infections below the diaphragm can lead to abscess forma
tion, such as subphrenic abscesses, it is not the most frequently occurring location for intraperitoneal a
bscesses overall.

Solution for Question 11:


Correct Option D - Fistula-in-ano:
Fistula in ano is the correct diagnosis for the presented case based on the clinical features described:
• Recurrent discharge and pain around the anus: Fistula in ano commonly presents with symptoms
such as intermittent purulent discharge and pain around the anal region due to the presence of an
abnormal tract or tunnel connecting the anal canal to the skin surface.

Page 51

922
• More common in males: Fistula in ano is indeed more prevalent in males, aligning with the
demographics of the patient in question.
• Typically seen in the 3rd to 5th decade: The onset of fistula in ano typically occurs in adulthood,
commonly in the age range mentioned.
• Presence of external opening: The discharge and pain are often accompanied by the presence of an
external opening or fistulous tract near the anus, through which the purulent material is expelled.
• Internal opening likely in rectum: If the patient passes feces and flatus through the external opening, it
suggests that the internal opening of the fistula is likely within the rectum, establishing the
communication between the anal canal and the skin.
Recurrent discharge and pain around the anus: Fistula in ano commonly presents with symptoms such
as intermittent purulent discharge and pain around the anal region due to the presence of an abnormal
tract or tunnel connecting the anal canal to the skin surface.
More common in males: Fistula in ano is indeed more prevalent in males, aligning with the demographi
cs of the patient in question.
Typically seen in the 3rd to 5th decade: The onset of fistula in ano typically occurs in adulthood, comm
only in the age range mentioned.
Presence of external opening: The discharge and pain are often accompanied by the presence of an e
xternal opening or fistulous tract near the anus, through which the purulent material is expelled.
Internal opening likely in rectum: If the patient passes feces and flatus through the external opening, it
suggests that the internal opening of the fistula is likely within the rectum, establishing the communicati
on between the anal canal and the skin.
Incorrect Options:
Option A - Pilonidal sinus:
• Pilonidal sinus typically presents as a cyst or sinus tract in the sacrococcygeal region rather than
around the anus.
• It commonly occurs in the younger age group, often in individuals in their late teens or early twenties,
unlike the 3rd to 5th decade as seen in the patient.
• Symptoms include pain, swelling, and sometimes discharge of pus or blood from the sinus opening in
the sacrococcygeal area.
Pilonidal sinus typically presents as a
cyst or sinus tract in the sacrococcygeal region rather than around the anus.
It commonly occurs in the younger age group, often in individuals in their late teens or early twenties, u
nlike the 3rd to 5th decade as seen in the patient.
Symptoms include pain, swelling, and sometimes discharge of pus or blood from the sinus opening in t
he sacrococcygeal area.
Option B - Carbuncle:
• A carbuncle is a collection of infected hair follicles and surrounding tissue, typically occurring on the
back of the neck, shoulders, or thighs, rather than around the anus.
• It presents as a cluster of boils or abscesses with multiple openings oozing pus.
• Carbuncles are not specifically associated with the anal region and do not involve the characteristic
features of fistula in ano.

Page 52

923
A carbuncle is a collection of infected hair follicles and surrounding tissue, typically occurring on the ba
ck of the neck, shoulders, or thighs, rather than around the anus.
It presents as a cluster of boils or abscesses with multiple openings oozing pus.
Carbuncles are not specifically associated with the anal region and do not involve the characteristic fea
tures of fistula in ano.
Option C - Boil:
• A boil, also known as a furuncle, is a localized infection of a hair follicle or sweat gland, typically
appearing as a tender, swollen area with a central core of pus.
• Boils can occur anywhere on the body, including the buttocks, but they do not typically manifest as
recurrent lesions specifically around the anus.
• Unlike fistula in ano, boils do not involve the formation of abnormal tracts or tunnels connecting the
anal canal to the skin surface.
A boil, also known as a furuncle, is a localized infection of a
hair follicle or sweat gland, typically appearing as a tender, swollen area with a central core of pus.
Boils can occur anywhere on the body, including the buttocks, but they do not typically manifest as rec
urrent lesions specifically around the anus.
Unlike fistula in ano, boils do not involve the formation of abnormal tracts or tunnels connecting the ana
l canal to the skin surface.

Solution for Question 12:


Correct Option C - 3:
• The appropriate incision for acute appendicitis is McBurney's incision.
• Acute appendicitis is a medical emergency that requires urgent surgical intervention to prevent
complications such as perforation, abscess formation, and peritonitis. The goal of the surgery is to
remove the inflamed appendix while minimizing complications and promoting rapid recovery.
• McBurney's incision is the standard for appendectomy (the surgical removal of the appendix). It is a
muscle-splitting incision made in the right lower quadrant of the abdomen, approximately one-third of
the distance from the anterior superior iliac spine (ASIS) to the umbilicus. The incision is usually 4-5 cm
long and is oriented obliquely with the upper end towards the ASIS.
The appropriate incision for acute appendicitis is McBurney's incision.
Acute appendicitis is a medical emergency that requires urgent surgical intervention to prevent complic
ations such as perforation, abscess formation, and peritonitis. The goal of the surgery is to remove the
inflamed appendix while minimizing complications and promoting rapid recovery.
McBurney's incision is the standard for appendectomy (the surgical removal of the appendix). It is a mu
scle-splitting incision made in the right lower quadrant of the abdomen, approximately one-third of the d
istance from the anterior superior iliac spine (ASIS) to the umbilicus. The incision is usually 4-5 cm long
and is oriented obliquely with the upper end towards the ASIS.
Incorrect Options:
Option A - 1: Kocher's incision is a
subcoastal incision to gain access for gall bladder and biliary tree pathology.

Page 53

924
Option B - 2: Midline incision is a vertical incision made in the midline of the abdomen. This type of inci
sion is typically used for surgeries involving multiple organs or structures, such as bowel resections, an
d is not the optimal incision for appendectomy.
Option D - 5: Lanz incision is a muscle-splitting incision made in the left lower quadrant of the abdomen
. It is typically used for surgeries involving the descending colon and rectum. This type of incision is not
appropriate for appendectomy.

Solution for Question 13:


Correct Option A - Subcapsular hematoma covering 40% surface area.:
• Subcapsular hematoma covering 40% surface area alone is not an indication for performing
angiography in a splenic trauma patient under conservative management. This is because, according
to the AAST grading system provided, a subcapsular hematoma covering 40% of the surface area falls
under Grade II, which is considered stable for conservative management.
Subcapsular hematoma covering 40% surface area alone is not an indication for performing angiograp
hy in a splenic trauma patient under conservative management. This is because, according to the AAS
T grading system provided, a subcapsular hematoma covering 40% of the surface area falls under Gra
de II, which is considered stable for conservative management.

Incorrect Options:
Option B - Significant hemoperitoneum:
• Hemoperitoneum indicates bleeding into the peritoneal cavity, which can result from splenic trauma,
among other causes.
• Angiography is considered if the patient's hemodynamic status deteriorates despite conservative
management or if other imaging findings, such as contrast extravasation or pseudoaneurysm, are
present.
Hemoperitoneum indicates bleeding into the peritoneal cavity, which can result from splenic trauma, a
mong other causes.

Page 54

925
Angiography is considered if the patient's hemodynamic status deteriorates despite conservative mana
gement or if other imaging findings, such as contrast extravasation or pseudoaneurysm, are present.
Option C - Presence of pseudoaneurysm:
• A pseudoaneurysm refers to an abnormal communication between an artery and a surrounding
hematoma or tissue, forming a sac-like structure.
• In the context of splenic trauma, the presence of a pseudoaneurysm indicates ongoing arterial
bleeding and is a potential indication for angiography.
• Angiography may be performed to identify the bleeding vessel and subsequently embolize it to control
bleeding and preserve splenic function.
A pseudoaneurysm refers to an abnormal communication between an artery and a
surrounding hematoma or tissue, forming a sac-like structure.
In the context of splenic trauma, the presence of a
pseudoaneurysm indicates ongoing arterial bleeding and is a potential indication for angiography.
Angiography may be performed to identify the bleeding vessel and subsequently embolize it to control
bleeding and preserve splenic function.
Option D - Contrast extravasation:
• Contrast extravasation refers to the leakage of contrast material outside of normal blood vessels,
indicating active bleeding.
• In the context of splenic trauma, the presence of contrast extravasation on CT imaging suggests
ongoing arterial bleeding and is an indication for further evaluation and intervention.
• Angiography may be performed to identify the bleeding vessel and perform embolization to stop the
bleeding and potentially avoid the need for surgical intervention.
Contrast extravasation refers to the leakage of contrast material outside of normal blood vessels, indic
ating active bleeding.
In the context of splenic trauma, the presence of contrast extravasation on CT imaging suggests ongoi
ng arterial bleeding and is an indication for further evaluation and intervention.
Angiography may be performed to identify the bleeding vessel and perform embolization to stop the ble
eding and potentially avoid the need for surgical intervention.

Solution for Question 14:


Correct Option C - Mallory Weiss Tear:
• The most probable diagnosis for a 45-year-old alcoholic with a history of binge drinking who presents
with an episode of vomiting bright red blood is Mallory Weiss Tear.
• Mallory Weiss Tear (MWT) is a condition characterized by a tear between the esophagus and the
stomach, known as the gastroesophageal junction.
• It is commonly associated with alcoholism and binge drinking, as well as with excessive vomiting and
retching.
• The tear occurs due to the forceful contraction of the abdominal muscles during vomiting, which can
lead to a partial tear of the mucosal lining of the gastroesophageal junction, resulting in bleeding.

Page 55

926
The most probable diagnosis for a 45-year-old alcoholic with a history of binge drinking who presents w
ith an episode of vomiting bright red blood is Mallory Weiss Tear.
Mallory Weiss Tear (MWT) is a condition characterized by a
tear between the esophagus and the stomach, known as the gastroesophageal junction.
It is commonly associated with alcoholism and binge drinking, as well as with excessive vomiting and r
etching.
The tear occurs due to the forceful contraction of the abdominal muscles during vomiting, which can le
ad to a partial tear of the mucosal lining of the gastroesophageal junction, resulting in bleeding.
Incorrect Options:
Correct Option A - Tension pneumothorax: Tension pneumothorax is when air accumulates in the pleur
al space between the lungs and the chest wall, resulting in increased pressure that can cause the lung
to collapse. It typically presents with sudden onset shortness of breath, chest pain, and tachycardia an
d is a medical emergency that requires immediate intervention.
Option B - Boerhaave syndrome: Boerhaave syndrome, also known as Spontaneous Esophageal Perf
oration, typically occurs due to forceful or repeated vomiting, predominantly affecting males. The prima
ry perforation site is the lower 1/3 of the esophagus, commonly along the left posterolateral direction. T
his condition involves a full-thickness perforation, enabling gastric contents to enter the mediastinum, r
esulting in severe mediastinitis.
Option D - Gastric ulcer perforation: Gastric ulcer perforation is when an ulcer in the stomach lining ero
des through the stomach wall, leading to a hole or perforation. It can present with sudden onset severe
abdominal pain, nausea, and vomiting and is a
medical emergency that requires immediate intervention.

Solution for Question 15:


Correct Option B - Bleeding esophageal varices:
• The most likely cause of massive, sudden onset hematemesis in a 50-year-old chronic alcoholic with
a history of chronic liver disease is bleeding esophageal varices. Esophageal varices are dilated veins
in the lower esophagus that develop as a consequence of portal hypertension, which is often caused by
chronic liver disease such as cirrhosis.
• As the portal hypertension worsens, pressure in the veins surrounding the liver increases, leading to
the formation of collaterals or new blood vessels. These collaterals are fragile and can rupture easily,
leading to massive bleeding.
The most likely cause of massive, sudden onset hematemesis in a 50-year-old chronic alcoholic with a
history of chronic liver disease is bleeding esophageal varices. Esophageal varices are dilated veins in
the lower esophagus that develop as a
consequence of portal hypertension, which is often caused by chronic liver disease such as cirrhosis.
As the portal hypertension worsens, pressure in the veins surrounding the liver increases, leading to th
e formation of collaterals or new blood vessels. These collaterals are fragile and can rupture easily, lea
ding to massive bleeding.
Incorrect Options:
Option A - Mallory Weiss tear: Mallory Weiss tear refers to a tear in the mucous membrane at the juncti
on of the esophagus and stomach, often caused by forceful vomiting or retching. Although this can lea
d to bleeding, it is usually self-limited and does not typically cause massive hematemesis.

Page 56

927
Option C - Boer-haave’s syndrome: Boer-haave’s syndrome refers to a spontaneous rupture of the eso
phagus, typically caused by forceful vomiting or retching. This can lead to massive hematemesis, but it
is rare and usually occurs in patients without a history of chronic liver disease or portal hypertension.
Option D - Perforated ulcer: Perforated ulcer refers to a hole in the wall of the stomach or duodenum, t
ypically caused by an ulcer. Although this can lead to bleeding, it usually presents with severe abdomin
al pain, fever, and other signs of peritonitis rather than sudden onset hematemesis.

Solution for Question 16:


Correct Option D - CECT abdomen:
• The most appropriate investigation to diagnose the condition in a 3-day-old neonate with abdominal
distension and bilious vomiting is a CECT (Contrast-enhanced computed tomography) abdomen.
The most appropriate investigation to diagnose the condition in a
3-day-old neonate with abdominal distension and bilious vomiting is a
CECT (Contrast-enhanced computed tomography) abdomen.
Incorrect Options:
Option A, B & C - Barium swallow, Barium meal & Barium meal follow through:
• Barium swallow, barium meal, and barium meal follow-through are radiological studies and are not
appropriate for a neonate with abdominal distension and bilious vomiting because they require the
ingestion of a significant amount of barium, which can be harmful to a newborn's delicate digestive
system.
Barium swallow, barium meal, and barium meal follow-through are radiological studies and are not app
ropriate for a
neonate with abdominal distension and bilious vomiting because they require the ingestion of a
significant amount of barium, which can be harmful to a newborn's delicate digestive system.

Solution for Question 17:


Correct Option B - Imperforate anus:
• The given radiograph of a newborn indicates the presence of an imperforate anus, which is a
congenital condition in which the anus does not develop properly, leading to a blockage in the passage
of stool.
• Imperforate anus is a rare condition that occurs in approximately 1 in 5,000 live births. It can range
from a minor defect to a complete obstruction of the anus, which can lead to life-threatening
complications if not treated promptly.
• The radiograph shows the absence of a normal gas pattern in the rectum and sigmoid colon, which is
indicative of an obstruction. This is further supported by the presence of a distended abdomen, which is
a common symptom of imperforate anus.
The given radiograph of a newborn indicates the presence of an imperforate anus, which is a
congenital condition in which the anus does not develop properly, leading to a
blockage in the passage of stool.

Page 57

928
Imperforate anus is a rare condition that occurs in approximately 1
in 5,000 live births. It can range from a minor defect to a complete obstruction of the anus, which can le
ad to life-threatening complications if not treated promptly.
The radiograph shows the absence of a normal gas pattern in the rectum and sigmoid colon, which is i
ndicative of an obstruction. This is further supported by the presence of a
distended abdomen, which is a common symptom of imperforate anus.
Incorrect Options:
Option A - Meningocele: Meningocele is a congenital disorder in which the spinal cord protrudes throug
h an opening in the spinal column, resulting in a
sac-like protrusion filled with cerebrospinal fluid. This condition is not visible on a
radiograph of the abdomen.
Option C - Congenital megacolon: Congenital megacolon, also known as Hirschsprung's disease, is a
condition in which there is a lack of nerve cells in the colon, leading to an obstruction of stool passage.
This condition can cause abdominal distension, but it does not involve the anus and would not show an
absence of gas in the rectum and sigmoid colon.
Option D - Pyloric stenosis: Pyloric stenosis is a
condition in which the muscle at the base of the stomach becomes thickened, leading to a narrowing of
the opening between the stomach and the small intestine. This condition can cause vomiting and dehy
dration, but it would not affect the anus and would not cause the absence of gas in the rectum and sig
moid colon.

Solution for Question 18:


Correct Option B - Retrograde cystourethrogram with penile urethral stricture:
• The image is showing a radiographic view of the male urethra, which is being filled with contrast
material from a retrograde cystourethrogram. This is a diagnostic test used to evaluate the urethra,
bladder, and sometimes the kidneys. During the test, a catheter is inserted into the urethra and contrast
material is injected, followed by X-ray imaging.
• The stricture is a narrowing of the urethral lumen, which can occur at any point along the urethra, from
the bladder neck to the external urethral meatus. In this case, the stricture is located in the penile
urethra, which is the portion of the urethra that runs through the penis. Thus the image shows
retrograde cystourethrogram with penile urethral stricture.
The image is showing a
radiographic view of the male urethra, which is being filled with contrast material from a
retrograde cystourethrogram. This is a
diagnostic test used to evaluate the urethra, bladder, and sometimes the kidneys. During the test, a
catheter is inserted into the urethra and contrast material is injected, followed by X-ray imaging.
The stricture is a narrowing of the urethral lumen, which can occur at any point along the urethra, from
the bladder neck to the external urethral meatus. In this case, the stricture is located in the penile ureth
ra, which is the portion of the urethra that runs through the penis. Thus the image shows retrograde cy
stourethrogram with penile urethral stricture.
Incorrect Options:
• Option A - Retrograde cystourethrogram with membranous urethral stricture: The stricture is not
located in the membranous portion of the urethra, which is located between the prostate gland and the
bulbous urethra.

Page 58

929
• Option C - Micturating cystourethrography with bulbar stricture: A micturating cystourethrogram
(MCU) is a different type of test used to evaluate the bladder and urethra during urination. A bulbar
stricture refers to a narrowing in the bulbar portion of the urethra, which is located between the bulbous
urethra and the membranous urethra.
• Option D - Micturating cystourethrography with psoriatic urethral stricture: Psoriasis is a skin condition
and does not directly affect the urethra.
Option A - Retrograde cystourethrogram with membranous urethral stricture: The stricture is not locate
d in the membranous portion of the urethra, which is located between the prostate gland and the bulbo
us urethra.
Option C - Micturating cystourethrography with bulbar stricture: A
micturating cystourethrogram (MCU) is a
different type of test used to evaluate the bladder and urethra during urination. A
bulbar stricture refers to a narrowing in the bulbar portion of the urethra, which is located between the b
ulbous urethra and the membranous urethra.
Option D - Micturating cystourethrography with psoriatic urethral stricture: Psoriasis is a
skin condition and does not directly affect the urethra.

Solution for Question 19:


Correct Option A - 24 hours ambulatory pH monitoring:
• Gastroesophageal reflux disease (GERD) is a condition in which stomach acid flows back up into the
esophagus, causing symptoms such as heartburn, regurgitation, and chest pain. The diagnosis of
GERD is usually made based on a combination of symptoms and diagnostic tests.
• 24-hour ambulatory pH monitoring is the best investigation to establish a diagnosis of GERD. It
involves the insertion of a small probe through the nose into the esophagus to monitor acid levels in the
esophagus over a 24-hour period. This test can help to identify episodes of acid reflux and assess the
severity of the condition.
Gastroesophageal reflux disease (GERD) is a condition in which stomach acid flows back up into the e
sophagus, causing symptoms such as heartburn, regurgitation, and chest pain. The diagnosis of GER
D is usually made based on a combination of symptoms and diagnostic tests.
24-hour ambulatory pH monitoring is the best investigation to establish a
diagnosis of GERD. It involves the insertion of a
small probe through the nose into the esophagus to monitor acid levels in the esophagus over a 24-ho
ur period. This test can help to identify episodes of acid reflux and assess the severity of the condition.
Incorrect Options:
Option B - Upper GI endoscopy: An upper GI endoscopy involves the insertion of a flexible tube with a
camera on the end into the esophagus, stomach, and duodenum to visualize the inside of these organs
. It can help to identify esophageal inflammation, ulcers, or other structural abnormalities that may be c
ontributing to symptoms, but it may not always detect the presence of acid reflux.
Option C - CECT: CECT (contrast-enhanced computed tomography) is not typically used in the diagno
sis of GERD. This imaging modality is more useful in assessing the structure and function of organs su
ch as the liver, pancreas, and kidneys.
Option D - MRI: MRI (magnetic resonance imaging) is also not typically used in the diagnosis of GERD
. Like CCECT, it more useful in assessing the structure and function of organs such as the liver, pancre

Page 59

930
as, and kidneys.

Solution for Question 20:


Correct Option B - Peptic ulcer perforation:
• The likely cause of the patient's symptoms and findings on the bedside radiological imaging is Peptic
ulcer perforation.
• Peptic ulcer disease (PUD) is a condition in which there is erosion or ulceration of the mucosal lining
of the stomach or duodenum. A perforated peptic ulcer is a complication of PUD in which the ulcer
penetrates through the mucosa and extends into the peritoneal cavity, causing peritonitis.
• The patient's history of alcohol abuse and smoking puts him at an increased risk of developing PUD.
The sudden onset of vomiting and stabbing pain in the epigastrium suggests a perforated peptic ulcer,
which can cause severe abdominal pain and vomiting.
• Bedside radiological imaging may show the presence of free air in the peritoneal cavity, a classic
finding in patients with a perforated peptic ulcer. The patient's fever and tachycardia may be due to the
systemic inflammatory response caused by peritonitis.
The likely cause of the patient's symptoms and findings on the bedside radiological imaging is Peptic ul
cer perforation.
Peptic ulcer disease (PUD) is a
condition in which there is erosion or ulceration of the mucosal lining of the stomach or duodenum. A
perforated peptic ulcer is a complication of PUD in which the ulcer penetrates through the mucosa and
extends into the peritoneal cavity, causing peritonitis.
The patient's history of alcohol abuse and smoking puts him at an increased risk of developing PUD. T
he sudden onset of vomiting and stabbing pain in the epigastrium suggests a
perforated peptic ulcer, which can cause severe abdominal pain and vomiting.
Bedside radiological imaging may show the presence of free air in the peritoneal cavity, a
classic finding in patients with a perforated peptic ulcer. The patient's fever and tachycardia may be du
e to the systemic inflammatory response caused by peritonitis.
Incorrect Options:
Option A - Tension pneumothorax: Tension pneumothorax is characterized by the presence of air in th
e pleural cavity, causing compression of the lung and mediastinal shift.
Option C - Boerhaave syndrome: Boerhaave syndrome is a
rare but life-threatening condition characterized by a
spontaneous rupture of the esophagus, usually because of prolonged vomiting.
Option D - Mallory Weiss tear: Mallory Weiss tear is a condition in which there is a tear in the mucosa a
t the junction of the esophagus and stomach, usually caused by excessive vomiting or retching.

Solution for Question 21:


Correct Option A - Iron:

Page 60

931
• Gastric bypass surgery and other bariatric surgeries involve the surgical alteration of the
gastrointestinal tract to help people with obesity lose weight. These surgeries result in a reduction in the
absorption of certain vitamins and minerals from food, leading to potential nutritional deficiencies.
• Iron is the most common mineral deficiency seen after gastric bypass surgery. The stomach produces
hydrochloric acid, which helps to break down food and release iron from it. After gastric bypass surgery,
the amount of stomach acid produced is reduced, leading to decreased iron absorption. This can result
in iron deficiency anemia, which can cause symptoms such as fatigue, weakness, and shortness of
breath.
Gastric bypass surgery and other bariatric surgeries involve the surgical alteration of the gastrointestin
al tract to help people with obesity lose weight. These surgeries result in a reduction in the absorption o
f certain vitamins and minerals from food, leading to potential nutritional deficiencies.
Iron is the most common mineral deficiency seen after gastric bypass surgery. The stomach produces
hydrochloric acid, which helps to break down food and release iron from it. After gastric bypass surgery
, the amount of stomach acid produced is reduced, leading to decreased iron absorption. This can resu
lt in iron deficiency anemia, which can cause symptoms such as fatigue, weakness, and shortness of b
reath.
Incorrect Options:
Option B - Copper: Copper deficiency is also relatively common after bariatric surgery, particularly in p
eople who have undergone malabsorptive procedures. Symptoms of copper deficiency include anemia,
neutropenia (low white blood cell count), and peripheral neuropathy.
Option C - Zinc: Zinc deficiency can occur after bariatric surgery, especially in people who have underg
one malabsorptive procedures. Symptoms of zinc deficiency include hair loss, skin lesions, and impaire
d immune function.
Option D - Calcium: Calcium deficiency is less common than iron, copper, or zinc deficiency after bariat
ric surgery, but it can still occur. Symptoms of calcium deficiency include osteoporosis, muscle spasms
, and numbness or tingling in the fingers and toes.

Solution for Question 22:


Correct Option C - 1 and 4:
(1) Hypoglycemia
• Dumping syndrome, particularly late dumping syndrome, can lead to reactive hypoglycemia. This
occurs due to the rapid emptying of hyperosmolar and hyperglycemic contents into the jejunum after a
meal, triggering an excessive insulin release.
• In the case of the elderly man who underwent Roux-en-Y gastric bypass surgery, the rapid transit of
carbohydrates into the small intestine leads to an exaggerated insulin response, causing a drop in
blood glucose levels, resulting in symptoms such as dizziness, fatigue, and tremors.
Dumping syndrome, particularly late dumping syndrome, can lead to reactive hypoglycemia. This occur
s due to the rapid emptying of hyperosmolar and hyperglycemic contents into the jejunum after a
meal, triggering an excessive insulin release.
In the case of the elderly man who underwent Roux-en-Y gastric bypass surgery, the rapid transit of ca
rbohydrates into the small intestine leads to an exaggerated insulin response, causing a
drop in blood glucose levels, resulting in symptoms such as dizziness, fatigue, and tremors.
(4) Increased release of insulin

Page 61

932
• Dumping syndrome triggers an increased insulin release due to the rapid influx of glucose into the
small intestine, leading to hypoglycemia.
• This excessive insulin release exacerbates the drop in blood glucose levels, contributing to the
symptoms experienced by the patient.
Dumping syndrome triggers an increased insulin release due to the rapid influx of glucose into the smal
l intestine, leading to hypoglycemia.
This excessive insulin release exacerbates the drop in blood glucose levels, contributing to the sympto
ms experienced by the patient.
Incorrect Statements
(2) (Small volume of food intake) is incorrect because dumping syndrome is characterized by the rapid
emptying of the stomach contents into the small intestine, typically after consuming a
regular or large meal.
(3) (High osmotic load of food) is incorrect because dumping syndrome does involve hyperosmolar con
tents being rapidly emptied into the jejunum. However, this contributes to the symptoms through mech
anisms such as increased insulin release, not directly causing dizziness, fatigue, and tremors.
(5) (Increased release of glucagon) is incorrect because dumping syndrome leads to an increased rele
ase of insulin, not glucagon. Glucagon typically acts to increase blood glucose levels, whereas insulin
decreases blood glucose levels.
Incorrect Options:
Option A, B & D: Refer to the explanation of the correct option.

Solution for Question 23:


Correct Option A - Normal saline:
• Gastric outlet obstruction is a condition where there is a blockage at the point where the stomach
empties into the small intestine, leading to vomiting, abdominal distention, and pain. Fluid management
is crucial in patients with gastric outlet obstruction, and the preferred fluid in this condition for initial
resuscitation is normal saline.
• Normal saline is an isotonic solution that is similar in composition to extracellular fluid. It is the
preferred fluid in this condition as it helps to maintain fluid and electrolyte balance, does not affect
gastric pH, and does not stimulate gastric secretions.
Gastric outlet obstruction is a condition where there is a blockage at the point where the stomach empti
es into the small intestine, leading to vomiting, abdominal distention, and pain. Fluid management is cr
ucial in patients with gastric outlet obstruction, and the preferred fluid in this condition for initial resuscit
ation is normal saline.
Normal saline is an isotonic solution that is similar in composition to extracellular fluid. It is the preferre
d fluid in this condition as it helps to maintain fluid and electrolyte balance, does not affect gastric pH, a
nd does not stimulate gastric secretions.
Incorrect Options:
Option B - Ringer lactate: Ringer lactate solution is also not recommended in this condition as it contai
ns lactate, which can increase gastric acid production and worsen vomiting.

Page 62

933
Option C - 5% dextrose in water: 5% dextrose in water are hypertonic solution that can worsen fluid ov
erload and hyponatremia in patients with gastric outlet obstruction.
Option D - 20% dextrose in water: 20% dextrose in water are hypertonic solution that can worsen fluid
overload and hyponatremia in patients with gastric outlet obstruction.

Solution for Question 24:


Correct Option D - Hollow viscus perforation:
• The radiograph in question shows air under the diaphragm, which indicates pneumoperitoneum. This
finding is consistent with the diagnosis of "hollow viscus perforation."
• Hollow viscus perforation is a critical medical situation involving a tear in a gastrointestinal organ like
the stomach or intestines, causing leakage of digestive fluids and waste into the abdomen, leading to
inflammation and potential infection.
• Symptoms include sudden and severe abdominal pain, nausea, vomiting, abdominal tenderness,
chills, fever, reduced urine output, rapid heartbeat, shortness of breath, and bloating.
• Diagnosis involves a physical examination to assess abdominal tenderness, along with imaging
studies such as X-rays, CT scans, or ultrasounds to pinpoint the perforation's location and severity.
Blood tests aid in identifying inflammation and infection.
The radiograph in question shows air under the diaphragm, which indicates pneumoperitoneum. This fi
nding is consistent with the diagnosis of "hollow viscus perforation."
Hollow viscus perforation is a critical medical situation involving a tear in a gastrointestinal organ like th
e stomach or intestines, causing leakage of digestive fluids and waste into the abdomen, leading to infl
ammation and potential infection.
Symptoms include sudden and severe abdominal pain, nausea, vomiting, abdominal tenderness, chills,
fever, reduced urine output, rapid heartbeat, shortness of breath, and bloating.
Diagnosis involves a physical examination to assess abdominal tenderness, along with imaging studies
such as X-rays, CT scans, or ultrasounds to pinpoint the perforation's location and severity. Blood test
s aid in identifying inflammation and infection.
Incorrect Options:
Option A - Rupture of ectopic pregnancy: Ectopic pregnancy is a pregnancy that occurs outside the ute
rus, most commonly in the fallopian tube. Rupture of an ectopic pregnancy can cause internal bleeding
and abdominal pain, but it typically does not cause pneumoperitoneum, as seen in the radiograph.
Option B - Acute pancreatitis: Acute pancreatitis is pancreas inflammation, which can cause severe ab
dominal pain. It can be caused by alcohol consumption and the use of NSAIDs. However, it does not u
sually cause pneumoperitoneum, as seen in the radiograph.
Option C - Perforated liver abscess: A liver abscess is a pus-filled sac in the liver. Perforation of a liver
abscess can cause infection and abdominal pain, but it typically does not cause pneumoperitoneum, a
s seen in the radiograph.

Solution for Question 25:

Page 63

934
Correct Option A - Carcinoid tumor:
• Carcinoid tumors are a type of neuroendocrine tumor that can arise in various parts of the body,
including the gastrointestinal tract (especially the ileum), lungs, and pancreas.
• They are usually slow-growing but can metastasize to other organs over time.
• One of the hallmark features of carcinoid tumors is the secretion of serotonin, which can lead to
elevated levels of urinary 5-hydroxy indole acetic acid (5-HIAA). This is because serotonin is converted
to 5-HIAA in the liver and then excreted in the urine.
Carcinoid tumors are a type of neuroendocrine tumor that can arise in various parts of the body, includi
ng the gastrointestinal tract (especially the ileum), lungs, and pancreas.
They are usually slow-growing but can metastasize to other organs over time.
One of the hallmark features of carcinoid tumors is the secretion of serotonin, which can lead to elevat
ed levels of urinary 5-hydroxy indole acetic acid (5-HIAA). This is because serotonin is converted to 5-
HIAA in the liver and then excreted in the urine.
Incorrect Options:
Option B - Neuroblastoma: Neuroblastoma is a type of cancer that arises in immature nerve cells, usua
lly in the adrenal glands (located above the kidneys). It is most commonly diagnosed in infants and you
ng children and can cause symptoms such as abdominal swelling or pain, fever, and weight loss.
Option C - Leiomyoma: Leiomyoma is a benign tumor arising from smooth muscle tissue and can occu
r in various parts of the body (including the uterus, gastrointestinal tract, and skin).
Option D - Chromaffinoma: Chromaffinoma (also known as pheochromocytoma) is a type of tumor that
arises from chromaffin cells located in the adrenal glands and sympathetic ganglia. Chromaffinomas ca
n cause symptoms such as high blood pressure, palpitations, sweating, and headache but do not typic
ally secrete serotonin or cause elevated 5-HIAA levels (although they can secrete other hormones suc
h as adrenaline and noradrenaline).

Solution for Question 26:


Correct Option D - Crack in the nipple:
• Cracked nipple during lactation can lead to acute mastitis due to the introduction of bacteria into the
breast tissue through the crack. When the nipple is cracked, it becomes a potential entry point for
bacteria present on the skin or in the baby's mouth during breastfeeding. This can result in
inflammation and infection of the breast tissue, causing acute mastitis.
• Management Rest: The affected nipple should be rested for 24-48 hours to allow healing. Breast
emptying: The breast should be emptied regularly, either through breastfeeding or using a breast
pump, to prevent milk stasis and further complications. Resuming feeding: Breastfeeding should be
resumed as soon as possible, ensuring proper latch and positioning to prevent further trauma to the
nipple.
• Rest: The affected nipple should be rested for 24-48 hours to allow healing.
• Breast emptying: The breast should be emptied regularly, either through breastfeeding or using a
breast pump, to prevent milk stasis and further complications.
• Resuming feeding: Breastfeeding should be resumed as soon as possible, ensuring proper latch and
positioning to prevent further trauma to the nipple.

Page 64

935
Cracked nipple during lactation can lead to acute mastitis due to the introduction of bacteria into the br
east tissue through the crack. When the nipple is cracked, it becomes a potential entry point for bacteri
a present on the skin or in the baby's mouth during breastfeeding. This can result in inflammation and i
nfection of the breast tissue, causing acute mastitis.
Management
• Rest: The affected nipple should be rested for 24-48 hours to allow healing.
• Breast emptying: The breast should be emptied regularly, either through breastfeeding or using a
breast pump, to prevent milk stasis and further complications.
• Resuming feeding: Breastfeeding should be resumed as soon as possible, ensuring proper latch and
positioning to prevent further trauma to the nipple.
Rest: The affected nipple should be rested for 24-48 hours to allow healing.
Breast emptying: The breast should be emptied regularly, either through breastfeeding or using a
breast pump, to prevent milk stasis and further complications.
Resuming feeding: Breastfeeding should be resumed as soon as possible, ensuring proper latch and p
ositioning to prevent further trauma to the nipple.
Incorrect Options:
Option A - Breast engorgement: Breast engorgement refers to the swelling and overfilling of the breast
with milk. While breast engorgement can cause discomfort and pain, it is not typically associated with a
cute mastitis unless it leads to milk stasis and subsequent infection.
Option B - Hormonal influence: Hormonal influences, such as fluctuations in hormone levels during me
nstruation or pregnancy, can contribute to breast tenderness or changes in breast tissue. However, the
y are not direct causes of acute mastitis. Hormonal influences may predispose individuals to conditions
like breast engorgement, but they do not directly lead to cracked nipples or subsequent mastitis.
Option C - Absence of lactation: The absence of lactation, known as lactation failure or insufficient milk
supply, is not typically associated with acute metastasis. While it can lead to breast discomfort or pain
due to milk stasis or engorgement, it does not directly relate to the spread of cancer cells.

Solution for Question 27:


Correct Option B - Pelvic:
• Prostate cancer commonly spreads to the nearby lymph nodes, and the pelvic lymph nodes are the
most frequently involved in this process.
• The pelvic lymph nodes, which are more proximal to the prostate gland, are typically the primary sites
of metastasis in prostate cancer
Prostate cancer commonly spreads to the nearby lymph nodes, and the pelvic lymph nodes are the mo
st frequently involved in this process.
The pelvic lymph nodes, which are more proximal to the prostate gland, are typically the primary sites
of metastasis in prostate cancer
Incorrect Options:
Option A, C & D: Deep inguinal, Superficial inguinal & Internal iliac:
• These are not involved in prostate cancer.

Page 65

936
These are not involved in prostate cancer.

Solution for Question 28:


Correct Option D - MRI:
• MRI is the gold standard investigation for Fistula in ano.
• Investigations for Anal Fistula: DRE reveals localized induration around the internal opening.
Proctoscopy localizes internal opening within 10cm of the anal verge. The first investigation to be done
is a fistulogram
• DRE reveals localized induration around the internal opening.
• Proctoscopy localizes internal opening within 10cm of the anal verge.
• The first investigation to be done is a fistulogram
• Goodsall’s rule is used to indicate the position of the internal opening on the basis of the
external opening
MRI is the gold standard investigation for Fistula in ano.
Investigations for Anal Fistula:
• DRE reveals localized induration around the internal opening.
• Proctoscopy localizes internal opening within 10cm of the anal verge.
• The first investigation to be done is a fistulogram
DRE reveals localized induration around the internal opening.
Proctoscopy localizes internal opening within 10cm of the anal verge.
The first investigation to be done is a fistulogram
Goodsall’s rule is used to indicate the position of the internal opening on the basis of the external openi
ng

Page 66

937
Incorrect Options:
Option A - Fistulogram: Fistulogram involves injecting contrast material into the fistulous tract followed
by imaging with X-rays. While it can provide some information about the anatomy of the fistula, it is les
s accurate compared to MRI. Fistulogram may fail to demonstrate the entire course of the fistula tract a
nd can miss associated abscesses or branching tracts.
Option B - USG: USG (Ultrasonography) may be used as an initial screening tool for fistula-in-ano due
to its accessibility and non-invasive nature. However, it lacks the accuracy and detail provided by MRI.
USG may not adequately visualize deep or complex fistulas, and it may not reliably identify internal ope
nings or associated abscesses.
Option C - CECT: While CECT (Contrast-enhanced Computed Tomography) can provide detailed imag
es of the pelvic anatomy, including the presence of abscesses and inflammatory changes, it is not the
preferred imaging modality for evaluating fistula-in-ano. CECT involves radiation exposure and may not
offer the same level of detail in soft tissue resolution as MRI.

Solution for Question 29:


Correct Option A - Copious sero-sanguinous discharge:
• The surest sign of wound dehiscence following emergency laparotomy for perforation peritonitis is the
presence of copious serosanguinous discharge, also known as salmon fluid or serous fluid, indicating
the opening of the rectus sheath. This discharge typically occurs around day 6 post-operatively and
signifies the development of the Burst abdomen, where the rectus sheath has opened up, confirming
wound dehiscence. Management of this condition involves Urobag laparotomy.
The surest sign of wound dehiscence following emergency laparotomy for perforation peritonitis is the
presence of copious serosanguinous discharge, also known as salmon fluid or serous fluid, indicating t
he opening of the rectus sheath. This discharge typically occurs around day 6 post-operatively and sig
nifies the development of the Burst abdomen, where the rectus sheath has opened up, confirming wou
nd dehiscence. Management of this condition involves Urobag laparotomy.
Incorrect Options:
Option B - Rebound tenderness: Rebound tenderness occurs when the tissue that layers the abdomina
l cavity or the peritoneum becomes irritated, infected, or inflamed. This is also called peritonitis.
Option C - Hemorrhage: Hemorrhage occurs when blood is clotted in an injured blood vessel. It can be
caused inside or outside the body for various reasons.
Option D - Hypotension: Hypotension causes a sudden drop in blood pressure that occurs when standi
ng after the sitting position or after lying down. It can be caused due to long term bed rest, dehydration,
pregnancy, some medications, and certain medical conditions. Older adults commonly experience suc
h type of low pressure.

• But these are not the surest signs of dehiscence.


But these are not the surest signs of dehiscence.

Solution for Question 30:

Page 67

938
Correct Option C - Combined chemoradiation:
• The correct treatment for a homosexual man with painful defecation and a mass protruding from the
anal canal, with biopsy revealing squamous cell carcinoma of the anus, is Combined chemoradiation.
• The recommended treatment for squamous cell carcinoma of the anus is combined chemoradiation.
This approach has been established to improve local control of the disease and overall survival rates. It
spares the anal sphincter, hence preserves the anal function. A typical regimen includes fluorouracil
plus mitomycin C, with concurrent radiation therapy.
The correct treatment for a homosexual man with painful defecation and a mass protruding from the an
al canal, with biopsy revealing squamous cell carcinoma of the anus, is Combined chemoradiation.
The recommended treatment for squamous cell carcinoma of the anus is combined chemoradiation. Th
is approach has been established to improve local control of the disease and overall survival rates. It s
pares the anal sphincter, hence preserves the anal function. A
typical regimen includes fluorouracil plus mitomycin C, with concurrent radiation therapy.
Incorrect Options:
Option A - Chemotherapy: While chemotherapy may be a component of the treatment for anal squamo
us cell carcinoma, it is typically used in combination with radiation therapy (chemoradiation). Chemothe
rapy alone is not considered the primary treatment for this type of cancer.
Option B - Abdominoperineal repair: Abdominoperineal resection was once the standard treatment for
anal cancer. But it's now reserved for those who do not respond to chemoradiation or have recurrent di
sease after initial treatment. This procedure involves removing the anus, rectum, and part of the colon,
which results in a permanent colostomy.
Option D - Wide local excision: Wide local excision is not typically used as a
primary treatment for anal cancer.

Solution for Question 31:


Correct Option B - Hernioplasty:
• Hernia repair surgery (herniorrhaphy) involves the return of displaced tissues to their proper position.
• Hernioplasty is a hernia repair surgery where a patch of net is sewn over the tissue’s weakened
region.
Hernia repair surgery (herniorrhaphy) involves the return of displaced tissues to their proper position.
Hernioplasty is a hernia repair surgery where a
patch of net is sewn over the tissue’s weakened region.
Incorrect Options:
Option A - Herniotomy: Herniotomy involves the surgical removal of the hernia sac without repairing th
e weakened abdominal wall. It may be suitable for certain cases, such as small hernias in pediatric pati
ents or high-risk individuals who cannot tolerate more extensive surgery. However, in the case describ
ed, the patient is 25 years old, and hernioplasty is the standard procedure for inguinal hernia repair in a
dults.
Option C - Wait and watch: Inguinal hernias do not resolve on their own and have the potential to beco
me incarcerated or strangulated, leading to complications. Therefore, waiting and watching without sur
gical intervention is generally not recommended. Surgery is typically the preferred approach to prevent
complications and provide a long-term solution.

Page 68

939
Option D - Emergency laparotomy: Emergency laparotomy is performed in cases of incarcerated or str
angulated hernias where there is compromised blood supply to the herniated contents or signs of bowe
l obstruction. It is considered a more invasive procedure and is not the first-line treatment for a reducibl
e inguinal hernia. In the case described, the hernia is reducible, which means the herniated contents c
an be easily pushed back into the abdominal cavity. Therefore, emergency laparotomy is not necessar
y in this scenario.

Solution for Question 32:


Correct Option B - Arch of aorta:
The constriction detected at the esophageal junction approximately 25 cm from the incisor during endo
scopy suggests involvement of the arch of the aorta.
• The esophagus is a muscular tube that carries food and liquids from the mouth to the stomach.
• At approximately 25 cm from the upper incisors, the esophagus encounters the arch of the aorta and
the left main bronchus, leading to a potential area of narrowing.
• The close proximity of the arch of the aorta to the esophagus can result in compression or constriction
of the esophageal lumen, leading to difficulty in swallowing or other symptoms.
• Therefore, when a constriction is detected at this location during endoscopy, the arch of the aorta is
the likely cause.
The esophagus is a muscular tube that carries food and liquids from the mouth to the stomach.
At approximately 25 cm from the upper incisors, the esophagus encounters the arch of the aorta and th
e left main bronchus, leading to a potential area of narrowing.
The close proximity of the arch of the aorta to the esophagus can result in compression or constriction
of the esophageal lumen, leading to difficulty in swallowing or other symptoms.
Therefore, when a
constriction is detected at this location during endoscopy, the arch of the aorta is the likely cause.
Incorrect Options:
Option A - Inferior vena cava:
• The inferior vena cava (IVC) is a large vein that carries deoxygenated blood from the lower half of the
body to the right atrium of the heart.
• It runs parallel to the abdominal aorta but is not typically in direct contact with the esophagus in the
thoracic cavity.
• Therefore, constriction at the esophageal junction 25 cm from the incisor is unlikely to be caused by
the inferior vena cava.
The inferior vena cava (IVC) is a large vein that carries deoxygenated blood from the lower half of the b
ody to the right atrium of the heart.
It runs parallel to the abdominal aorta but is not typically in direct contact with the esophagus in the thor
acic cavity.
Therefore, constriction at the esophageal junction 25 cm from the incisor is unlikely to be caused by th
e inferior vena cava.
Option C - Right bronchus:

Page 69

940
• The right bronchus is one of the two main branches of the trachea, leading into the right lung.
• While the right bronchus does cross posterior to the esophagus, it does so lower down in the thoracic
cavity compared to the arch of the aorta.
• Constriction at the esophageal junction 25 cm from the incisor is more likely to be due to the arch of
the aorta than the right bronchus.
The right bronchus is one of the two main branches of the trachea, leading into the right lung.
While the right bronchus does cross posterior to the esophagus, it does so lower down in the thoracic c
avity compared to the arch of the aorta.
Constriction at the esophageal junction 25 cm from the incisor is more likely to be due to the arch of th
e aorta than the right bronchus.
Option D - Diaphragmatic opening:
• The diaphragmatic opening, also known as the esophageal hiatus, is the opening in the diaphragm
through which the esophagus passes as it enters the abdominal cavity.
• It is located lower down the esophagus, approximately 40 cm from the upper incisors.
• Constriction at the esophageal junction 25 cm from the incisor is too high in the thoracic cavity to be
attributed to the diaphragmatic opening.
The diaphragmatic opening, also known as the esophageal hiatus, is the opening in the diaphragm thro
ugh which the esophagus passes as it enters the abdominal cavity.
It is located lower down the esophagus, approximately 40 cm from the upper incisors.
Constriction at the esophageal junction 25 cm from the incisor is too high in the thoracic cavity to be att
ributed to the diaphragmatic opening.

Solution for Question 33:


Correct Option D - Sympathetic - Impotence and loss of cutaneous sensation in perineal region:
Ligation of the inferior mesenteric artery (first vessel) during abdominoperineal resection (APR) for rect
al carcinoma can result in complications due to injury to:
• Sympathetic system: Injury to the sympathetic system causes retrograde ejaculation.
• Parasympathetic system: Injury to the parasympathetic system can cause bladder dysfunction and
impotence.
Sympathetic system: Injury to the sympathetic system causes retrograde ejaculation.
Parasympathetic system: Injury to the parasympathetic system can cause bladder dysfunction and imp
otence.
Among the given options, the only option with a correct sequence is sympathetic injury leading to impot
ence and loss of cutaneous sensation in the perineal region.
Incorrect Options:
• Options A, B, and C are incorrect. Refer to Option D for an explanation.
Options A, B, and C are incorrect. Refer to Option D for an explanation.

Page 70

941
Solution for Question 34:
Correct Option D - Lateral cutaneous nerve of thigh:
• The nerves of triangle of pain are commonly damaged during laparoscopic surgeries are: Femoral
nerve. Lateral cutaneous nerve of thigh Femoral branch of genitofemoral nerve.
• Femoral nerve.
• Lateral cutaneous nerve of thigh
• Femoral branch of genitofemoral nerve.
• The patient's pain in the right thigh following bilateral laparoscopic hernia repair suggests entrapment
of the lateral cutaneous nerve of the thigh. This nerve provides sensation to the skin on the outer
(lateral) part of the thigh. It can be injured during abdominal or pelvic surgery, leading to pain and
paresthesia in its distribution.
The nerves of triangle of pain are commonly damaged during laparoscopic surgeries are:
• Femoral nerve.
• Lateral cutaneous nerve of thigh
• Femoral branch of genitofemoral nerve.
Femoral nerve.
Lateral cutaneous nerve of thigh
Femoral branch of genitofemoral nerve.
The patient's pain in the right thigh following bilateral laparoscopic hernia repair suggests entrapment o
f the lateral cutaneous nerve of the thigh. This nerve provides sensation to the skin on the outer (lateral
) part of the thigh. It can be injured during abdominal or pelvic surgery, leading to pain and paresthesia
in its distribution.
Incorrect Options:
Option A - Ilioinguinal nerve: Damage to this nerve is typically seen in open surgeries.
Option B - Iliohypogastric nerve: Damage to this nerve is typically seen in open surgeries.
Option C - Femoral nerve: Entrapment of the femoral nerve would cause weakness in leg extension an
d decreased sensation over the anterior thigh and medial leg but not typically isolated lateral thigh pain
.

Solution for Question 35:


Correct Option B - Conjoint tendon:
• The patient is diagnosed with a direct inguinal hernia, which is likely to be caused by weakness in the
conjoint tendon. The conjoint tendon (formed by the union of the internal oblique and transversus
abdominis muscles) forms the posterior wall of the inguinal canal medially and contributes to the
posterior wall of the superficial inguinal ring. Weakness in this structure can lead to a direct inguinal
hernia.

Page 71

942
The patient is diagnosed with a direct inguinal hernia, which is likely to be caused by weakness in the c
onjoint tendon. The conjoint tendon (formed by the union of the internal oblique and transversus abdo
minis muscles) forms the posterior wall of the inguinal canal medially and contributes to the posterior w
all of the superficial inguinal ring. Weakness in this structure can lead to a direct inguinal hernia.
Incorrect Options:
Option A - Pectineal ligament: This ligament, also known as Cooper's ligament, is not directly related to
the formation of inguinal hernias.
Option C - Reflected part of inguinal ligament: The lacunar ligament and the inferior crus of the subcuta
neous inguinal ring expand to form the reflected inguinal ligament, which is a
layer of tendinous fibers with a
triangle shape. This is not associated with the formation of inguinal hernia.
Option D - Lacunar ligament: The lacunar ligament is the medial extension of the inguinal ligament. It c
ontributes to the floor of the inguinal canal but its weakness is not directly associated with the formation
of direct inguinal hernias.

Solution for Question 36:


Correct Option C - Hamartoma:
• The child's symptoms of a prolapsing rectal mass and painless rectal bleeding, along with
histopathological findings of enlarged and inflamed glands filled with mucin, are suggestive of a juvenile
polyp, which is a type of hamartoma. Hamartomas are benign, tumor-like malformations characterized
by an overgrowth of mature cells and tissues normally present in the affected part of the body, although
the cells themselves are normal. Juvenile polyps are the most common type of polyp in children and
are typically found in the rectum.
The child's symptoms of a prolapsing rectal mass and painless rectal bleeding, along with histopatholo
gical findings of enlarged and inflamed glands filled with mucin, are suggestive of a
juvenile polyp, which is a type of hamartoma. Hamartomas are benign, tumor-like malformations chara
cterized by an overgrowth of mature cells and tissues normally present in the affected part of the body,
although the cells themselves are normal. Juvenile polyps are the most common type of polyp in childr
en and are typically found in the rectum.
Incorrect Options:
Option A - Adenoma: This is a type of benign tumor that originates in glandular tissue. Although adeno
mas can occur in the rectum, they are less common in children.
Option B - Choristoma: Choristomas are ectopic rests of normal tissues. In other words, they are norm
al tissues located in abnormal locations. They are rare in the rectum.
Option D - Carcinoma: This refers to malignant tumors derived from epithelial cells. It's highly unlikely i
n a 5-year-old child.

Solution for Question 37:


Correct Option C - Feeding jejunostomy:

Page 72

943
• Ingestion of strong alkalis, such as sodium hydroxide, can result in severe esophageal damage and
cause a condition known as caustic esophagitis.
• The immediate concern in these cases is to ensure adequate nutrition and hydration while avoiding
further injury to the esophagus.
• A feeding jejunostomy allows nutrition to bypass the esophagus entirely, giving it time to heal.
Ingestion of strong alkalis, such as sodium hydroxide, can result in severe esophageal damage and ca
use a condition known as caustic esophagitis.
The immediate concern in these cases is to ensure adequate nutrition and hydration while avoiding furt
her injury to the esophagus.
A feeding jejunostomy allows nutrition to bypass the esophagus entirely, giving it time to heal.
Incorrect Options:
Option A - Esophagojejunostomy: Esophagojejunostomy, a
procedure where the esophagus is connected directly to the jejunum, is a
significant surgical procedure and is typically not the first step in managing caustic esophagitis.
Option B - Stent placement: Stent placement in the esophagus will not be best option for this scenario
Option D - Gastrojejunostomy: Gastrojejunostomy, a procedure that connects the stomach to the jejun
um, would still require food through the injured esophagus, so it is not the best management option.

Solution for Question 38:


Correct Option C - Peptic ulcer:
• The given case involves a patient presenting with sudden onset of hematemesis, a history of aspirin
use, and occasional abdominal pain, which are all suggestive of a peptic ulcer.
• Peptic ulcers, which may occur in the stomach or the first part of the small intestine (duodenum), can
cause upper gastrointestinal bleeding, presenting as hematemesis.
• The patient's use of aspirin, a nonsteroidal anti-inflammatory drug (NSAID), increases the risk of
peptic ulcer disease due to its inhibitory effect on prostaglandin synthesis, impairing the stomach's
protective mechanisms.
The given case involves a patient presenting with sudden onset of hematemesis, a
history of aspirin use, and occasional abdominal pain, which are all suggestive of a peptic ulcer.
Peptic ulcers, which may occur in the stomach or the first part of the small intestine (duodenum), can c
ause upper gastrointestinal bleeding, presenting as hematemesis.
The patient's use of aspirin, a nonsteroidal anti-inflammatory drug (NSAID), increases the risk of peptic
ulcer disease due to its inhibitory effect on prostaglandin synthesis, impairing the stomach's protective
mechanisms.
Incorrect Options:
Option A - Mallory-Weiss tear: This condition typically presents with hematemesis following forceful or
prolonged vomiting, not reported in this case.
Option B - Esophagitis: While esophagitis can cause upper gastrointestinal bleeding, it's often associat
ed with symptoms such as heartburn, dysphagia, and odynophagia.

Page 73

944
Option D - Esophageal varices: These are often seen in patients with a
history of liver disease, particularly cirrhosis, which is not reported in this case.

Solution for Question 39:


Correct Option B - Duodenum:
• In the given scenario, an elderly woman who underwent surgical resection of her bowel presents with
easy fatiguability and tiredness along with macrocytic anemia; the likely cause is vitamin B12 and
Vitamin B9 deficiency. Absorption of B12 occurs mainly in the stomach and ileum, whereas folic acid
absorption occurs mainly in the jejunum. So, resection of the duodenum is least likely to cause
macrocytic anemia.
Incorrect Options:
Option A - Jejunum: Folic acid absorption occurs mainly in the jejunum. Therefore, it can contribute to
malabsorption and is likely to cause macrocytic anemia.
Option C - Ileum: As mentioned earlier, the ileum is the primary site of vitamin B12 absorption. Resecti
on of the ileum can lead to impaired absorption and subsequent deficiency, resulting in macrocytic ane
mia.
Option D - Stomach: The stomach plays a
role in the absorption of vitamin B12 through the release of intrinsic factor, a protein required for vitami
n B12 absorption in the ileum. So the resection of the stomach is expected to cause macrocytic anemia
.

Solution for Question 40:


Correct Option A - Ripstein’s Procedure:
• The image shown is rectal prolapse, which manifests in two forms: mucosal prolapse, primarily seen
in children, and rectal prolapse, involving full-thickness prolapses, predominantly in adults.
• Ripstein’s procedure even though it is a surgical approach for rectal prolapse, is, however, not a
perineal approach.
• Ripstein's procedure, also known as Ripstein rectopexy, is a surgical technique used to treat rectal
prolapse and prevent its recurrence.
• The procedure involves providing support to the rectum by anchoring it to the sacrum using a
synthetic mesh or a strip of fascia.
The image shown is rectal prolapse, which manifests in two forms: mucosal prolapse, primarily seen in
children, and rectal prolapse, involving full-thickness prolapses, predominantly in adults.
Ripstein’s procedure even though it is a surgical approach for rectal prolapse, is, however, not a
perineal approach.
Ripstein's procedure, also known as Ripstein rectopexy, is a
surgical technique used to treat rectal prolapse and prevent its recurrence.
The procedure involves providing support to the rectum by anchoring it to the sacrum using a
synthetic mesh or a strip of fascia.

Page 74

945
Incorrect Options:
Option B - Delorme's Operation: Delorme's operation, also known as perineal proctectomy or perineal r
ectosigmoidectomy, is a surgical procedure primarily used to treat rectal prolapse. In this procedure, th
e surgeon makes an incision in the perineum to access the rectum. Delorme's operation is often perfor
med as an alternative to abdominal approaches (such as abdominal rectopexy) for selected cases of re
ctal prolapse.
Option C - Thiersch Operation: Thiersch operation, also known as anal encirclement, is a surgical tech
nique used to treat anal incontinence. The surgical approach is through the perineum. It involves placin
g a circumferential strip of synthetic material, such as polypropylene mesh, around the anal canal to pr
ovide support and improve sphincter function. The mesh is positioned in a
submucosal tunnel created around the anus.
Option D - Altemeier's Procedure: Altemeier's procedure, also referred to as perineal rectosigmoidecto
my with rectal mucosectomy, is a surgical approach used to treat rectal prolapse, particularly in elderly
or debilitated patients. The procedure involves removing the prolapsed portion of the rectum and suturi
ng the remaining rectal tissue to the anus, similar to Delorme's operation. Additionally, in Altemeier's pr
ocedure, the rectal mucosa (inner lining of the rectum) is also partially or completely excised to prevent
recurrence. The surgery is performed through a perineal incision.

Solution for Question 41:


Correct Option C - 1, 2, 3 and 4:
• Fluctuant swelling in the hip: Psoas abscess can present as a fluctuant swelling in the hip region. The
abscess may cause localized swelling and inflammation, leading to pain and discomfort in the hip area.
• Cold abscess can develop from Pott's abscess: Pott's abscess is a type of tuberculous abscess that
affects the spine. It can extend into the psoas muscle, leading to the formation of a psoas abscess. This
type of abscess is often referred to as a "cold abscess" due to its chronic nature and lack of significant
acute inflammation or pus formation.
• Pain on passive extension of the hip joint is seen: Psoas abscess can cause pain on passive
extension of the hip joint. The psoas muscle is involved in hip flexion, and when an abscess is present,
movement or stretching of the muscle can cause pain and discomfort.
• It develops more commonly through the hematogenous route in immunocompromised patients and
the elderly: Psoas abscess can develop through various routes, including hematogenous spread from a
distant infection, direct extension from nearby structures, or rarely, trauma. In immunocompromised
patients and the elderly, the risk of hematogenous spread of infection is increased, making this route
more common in these populations.
• Therefore, all the given options are correct and represent possible characteristics of Psoas abscess.
Fluctuant swelling in the hip: Psoas abscess can present as a fluctuant swelling in the hip region. The
abscess may cause localized swelling and inflammation, leading to pain and discomfort in the hip area.
Cold abscess can develop from Pott's abscess: Pott's abscess is a type of tuberculous abscess that aff
ects the spine. It can extend into the psoas muscle, leading to the formation of a
psoas abscess. This type of abscess is often referred to as a
"cold abscess" due to its chronic nature and lack of significant acute inflammation or pus formation.
Pain on passive extension of the hip joint is seen: Psoas abscess can cause pain on passive extension
of the hip joint. The psoas muscle is involved in hip flexion, and when an abscess is present, moveme
nt or stretching of the muscle can cause pain and discomfort.

Page 75

946
It develops more commonly through the hematogenous route in immunocompromised patients and the
elderly: Psoas abscess can develop through various routes, including hematogenous spread from a dis
tant infection, direct extension from nearby structures, or rarely, trauma. In immunocompromised patie
nts and the elderly, the risk of hematogenous spread of infection is increased, making this route more c
ommon in these populations.
Therefore, all the given options are correct and represent possible characteristics of Psoas abscess.

Solution for Question 42:


Correct Option A - Gastric and pancreatic:
• In Meckel's diverticulum, the most commonly encountered ectopic tissue is gastric and pancreatic
tissue.
• This occurs because, during embryonic development, the diverticulum arises from the same
embryonic structure as the stomach and pancreas.
• As a result, gastric mucosa and pancreatic tissue may be present in the diverticulum.
In Meckel's diverticulum, the most commonly encountered ectopic tissue is gastric and pancreatic tissu
e.
This occurs because, during embryonic development, the diverticulum arises from the same embryonic
structure as the stomach and pancreas.
As a result, gastric mucosa and pancreatic tissue may be present in the diverticulum.
Incorrect Options:
Options B, C and D are not commonly associated with Meckel’s diverticulum.

Solution for Question 43:


Correct Option A - Ileocolic artery:
• This is the correct answer because the appendiceal artery, which supplies blood to the appendix, is a
branch of the ileocolic artery. Ligating the appendiceal artery during the appendicectomy is important to
prevent postoperative bleeding.
This is the correct answer because the appendiceal artery, which supplies blood to the appendix, is a b
ranch of the ileocolic artery. Ligating the appendiceal artery during the appendicectomy is important to
prevent postoperative bleeding.
Incorrect Options:
Option B - Right colic artery: The right colic artery supplies blood to the ascending colon and is not dire
ctly related to the appendix. Ligating the right colic artery is not necessary during an appendicectomy.
Option C - Superior rectal artery: The superior rectal artery supplies blood to the rectum and is not dire
ctly involved in the blood supply of the appendix. Ligating the superior rectal artery is not necessary dur
ing an appendicectomy.
Option D - Superior mesenteric artery: The superior mesenteric artery is a major artery that supplies bl
ood to the small intestine, cecum, and ascending colon. Ligating the superior mesenteric artery would r

Page 76

947
esult in ischemia of a large portion of the intestines and is not performed during an appendicectomy.

Solution for Question 44:


Correct Option B - Typhoid vaccine:
• Typhoid vaccine: This option is incorrect. Typhoid vaccine is not typically included in the routine
pre-splenectomy vaccination recommendations. Typhoid fever is caused by Salmonella typhi, which is
not an encapsulated organism. Therefore, it is not directly related to the increased risk of infection
following splenectomy.
Incorrect Options:
Option A - Pneumococcal vaccine: This option is correct. The Pneumococcal vaccine is recommended
before splenectomy to protect against Streptococcus pneumoniae, a
common cause of severe infections in patients without a functioning spleen.
Option C - Haemophilus influenzae B vaccine: This option is correct. The Haemophilus influenzae B va
ccine is recommended before splenectomy to protect against Haemophilus influenzae type B, an enca
psulated organism that can cause severe infections in individuals without a functioning spleen.
Option D - Meningococcal vaccine: This option is correct. The Meningococcal vaccine is recommended
before splenectomy to protect against Neisseria meningitidis, an encapsulated organism known to cau
se meningitis and septicemia. It is an important vaccine to prevent meningococcal infections in individu
als without a spleen.

Solution for Question 45:


Correct Option B - B, C:
A - It is mostly hereditary: This statement is incorrect. Gastric carcinoma is not predominantly hereditar
y. While there are certain genetic factors that can increase the risk of developing gastric cancer, the m
ajority of cases are not directly linked to hereditary factors.
B - E-cadherin adhesion defect is seen in the diffuse type of gastric carcinoma: This statement is corre
ct. E-cadherin is a cell adhesion molecule, and its defect is associated with the diffuse type of gastric c
arcinoma, which is characterized by the loss of cell adhesion and infiltration of individual cancer cells.
C - Wnt signaling pathway defect is seen in intestinal-type gastric carcinoma: This statement is correct.
The Wnt signaling pathway plays a crucial role in the development of the intestinal-type gastric carcino
ma. Dysregulation of this pathway can lead to uncontrolled cell growth and tumor formation.
D - Depth of cancer has no role in prognosis: This statement is incorrect. The depth of cancer invasion,
known as tumor staging, is an important prognostic factor in gastric carcinoma. It helps determine the
extent of tumor spread and guides treatment decisions.

Solution for Question 46:


Correct Option B - A and C:

Page 77

948
• The given presentation and radiograph showing air under the diaphragm is suggesting intestinal
perforation .
• (A) IV fluids: This is the initial step in the management of a patient presenting with symptoms
suggestive of obstruction or possible surgical pathology. IV fluids help correct dehydration and maintain
hemodynamic stability.
• (C) Exploratory laparotomy: In the case of suspected bowel obstruction and signs of peritonitis or
worsening clinical condition, exploratory laparotomy may be necessary to identify the cause of the
obstruction and provide appropriate surgical intervention.
The given presentation and radiograph showing air under the diaphragm is suggesting intestinal perfor
ation .
(A) IV fluids: This is the initial step in the management of a patient presenting with symptoms suggestiv
e of obstruction or possible surgical pathology. IV fluids help correct dehydration and maintain hemody
namic stability.
(C) Exploratory laparotomy: In the case of suspected bowel obstruction and signs of peritonitis or wors
ening clinical condition, exploratory laparotomy may be necessary to identify the cause of the obstructi
on and provide appropriate surgical intervention.
Incorrect Options:
Option A - A and B: CECT abdomen (B) is not the immediate next step in this scenario. While it may pr
ovide useful information, such as the location and extent of the obstruction, it is not the initial step in th
e management of a patient with symptoms of obstruction
Option C - A and D: Upper GI endoscopy (D) is not the next line of management for this patient. While
upper GI endoscopy is a useful diagnostic tool for peptic ulcer disease, it is not the immediate priority i
n this case, with symptoms suggestive of obstruction and possible surgical pathology. Exploratory lapa
rotomy (C) would be considered before upper GI endoscopy.
Option D - A, B and C: CECT abdomen (B) is not the immediate next step in this scenario. While it may
provide useful information, such as the location and extent of the obstruction, it is not the initial step in
the management of a patient with symptoms of obstruction. Exploratory laparotomy (C) would be consi
dered before ordering a CECT abdomen.

Solution for Question 47:


Correct Option A - Retrocecal:
This option refers to the appendix located behind the cecum, which is the initial part of the large intestin
e. The retrocecal position is one of the most common variations in the anatomical location of the appen
dix.
• In this position, the appendix lies posterior to the cecum, typically in close proximity to the ileocecal
valve.
• In a retrocecal appendix, the symptoms of appendicitis may vary slightly from the typical presentation.
• Patients may experience pain and tenderness in the right lower quadrant, but the pain can also be
located more toward the flank or the back.
In this position, the appendix lies posterior to the cecum, typically in close proximity to the ileocecal val
ve.
In a retrocecal appendix, the symptoms of appendicitis may vary slightly from the typical presentation.

Page 78

949
Patients may experience pain and tenderness in the right lower quadrant, but the pain can also be loca
ted more toward the flank or the back.

Incorrect Options:
Option B - Preileal: "Preileal," it refers to the location of the appendix in front of the ileum, which is the
distal part of the small intestine.
Option C - Subcecal: This option suggests the appendix located beneath the cecum. While the subcec
al position is a variation, it is less common than the retrocecal position. In this position, the appendix is
positioned below the cecum.
Option D - Pelvic: This option implies the appendix located within the pelvic cavity. While the appendix
can descend into the pelvis, particularly in cases of a long appendix or a
mobile cecum, it is not the typical or most common location.

Solution for Question 48:


Correct Option B - Reduction en masse:
• In the context of the question, the surgeon examined a case of hernia and was able to retract the
hernial sac on examination, but not the contents. This suggests that the hernia sac and its contents
have been unintentionally pushed into a deeper anatomical space during the examination, which is
characteristic of reduction en masse.
• Reduction en masse refers to the unintentional reduction or displacement of a hernia sac and its
contents into a deeper anatomical space, such as the preperitoneal or retroperitoneal space. This can
occur during the attempted reduction of a hernia.
• Reduction en masse is a relatively uncommon occurrence and is considered a surgical emergency.
• It can lead to complications such as bowel obstruction, ischemia, or perforation if not promptly
recognized and managed.
In the context of the question, the surgeon examined a case of hernia and was able to retract the herni
al sac on examination, but not the contents. This suggests that the hernia sac and its contents have be

Page 79

950
en unintentionally pushed into a
deeper anatomical space during the examination, which is characteristic of reduction en masse.
Reduction en masse refers to the unintentional reduction or displacement of a
hernia sac and its contents into a deeper anatomical space, such as the preperitoneal or retroperitonea
l space. This can occur during the attempted reduction of a hernia.
Reduction en masse is a relatively uncommon occurrence and is considered a surgical emergency.
It can lead to complications such as bowel obstruction, ischemia, or perforation if not promptly recogniz
ed and managed.
Incorrect Options:
Option A - Incarcerated hernia: An incarcerated hernia occurs when the hernia contents, such as bowel
loops or omentum, become trapped within the hernia sac and cannot be easily reduced or pushed bac
k into the abdominal cavity. In this scenario, the description states that the hernial sac was retracted on
examination, indicating that the contents were not trapped. Therefore, an incarcerated hernia is not th
e correct option.
Option C - Sliding hernia: A sliding hernia is a type of hernia in which a portion of an organ, usually the
stomach or colon, forms part of the hernia sac. The hernia sac is usually composed of both the periton
eum and the serosa of the organ involved. The description does not provide any indication that an orga
n is part of the hernia sac. Therefore, a sliding hernia is not the correct option.
Option D - Maydl's hernia: Maydl hernia is a
rare type of hernia characterized by multiple loops of bowel within the hernia sac, with a constricted se
gment of bowel between the two hernia orifices. The description does not mention any specific charact
eristics of multiple loops of bowel or a
constricted segment. Therefore, Maydl hernia is not the correct option.

Solution for Question 49:


Correct Option A - Colonoscopy:
• Colonoscopy is the initial investigation of choice for evaluating patients who are stable with per rectal
bleeding and suspected colorectal pathology.
• It allows direct visualization of the entire colon and rectum, providing the opportunity for biopsy and
intervention if necessary. A colonoscopy can help identify the source and nature of bleeding, such as
polyps, tumors, or inflammatory conditions.
Colonoscopy is the initial investigation of choice for evaluating patients who are stable with per rectal bl
eeding and suspected colorectal pathology.
It allows direct visualization of the entire colon and rectum, providing the opportunity for biopsy and inte
rvention if necessary. A colonoscopy can help identify the source and nature of bleeding, such as polyp
s, tumors, or inflammatory conditions.
Incorrect Options:
Option B - CEA levels:
• Carcinoembryonic Antigen (CEA) is a tumor marker that can be elevated in certain cancers, including
colorectal cancer. However, CEA levels alone are insufficient to diagnose or evaluate the cause of
per-rectal bleeding.

Page 80

951
• While CEA levels may help monitor treatment response or detect recurrence in known colorectal
cancer cases, it is not the primary investigation for initial evaluation.
Carcinoembryonic Antigen (CEA) is a tumor marker that can be elevated in certain cancers, including c
olorectal cancer. However, CEA levels alone are insufficient to diagnose or evaluate the cause of per-r
ectal bleeding.
While CEA levels may help monitor treatment response or detect recurrence in known colorectal cance
r cases, it is not the primary investigation for initial evaluation.
Option C - CA-19-9 levels:
• CA-19-9 is another tumor marker used primarily in pancreatic and hepatobiliary malignancies. Its
utility in evaluating rectal bleeding is limited, and it is not recommended for this scenario. CA-19-9
levels are not specific to colorectal pathology and may not provide accurate diagnostic or prognostic
information in this context.
CA-19-9 is another tumor marker used primarily in pancreatic and hepatobiliary malignancies. Its utility
in evaluating rectal bleeding is limited, and it is not recommended for this scenario. CA-19-9 levels are
not specific to colorectal pathology and may not provide accurate diagnostic or prognostic information i
n this context.
Option D - CECT (Contrast-enhanced CT scan):
• While a CECT scan can help evaluate various abdominal and pelvic conditions, it is not the initial
investigation of choice for per rectal bleeding. Colonoscopy provides direct visualization and biopsy
capabilities, allowing for a more comprehensive evaluation of the colon and rectum. CT scans may be
used as an adjunct investigation in specific situations or to assess for extra-colonic findings.
While a CECT scan can help evaluate various abdominal and pelvic conditions, it is not the initial inves
tigation of choice for per rectal bleeding. Colonoscopy provides direct visualization and biopsy capabiliti
es, allowing for a more comprehensive evaluation of the colon and rectum. CT scans may be used as
an adjunct investigation in specific situations or to assess for extra-colonic findings.

Solution for Question 50:


Correct Option C - Thiersch wiring:
• Thiersch wiring is a surgical technique for managing certain types of rectal prolapse. It involves
applying wires or sutures to the rectal mucosa to create a supportive framework and provide
mechanical support to the rectum.
Thiersch wiring is a surgical technique for managing certain types of rectal prolapse. It involves applyin
g wires or sutures to the rectal mucosa to create a
supportive framework and provide mechanical support to the rectum.
Incorrect Options:
Option A - Stapled hemorrhoidopexy: Also known as stapled hemorrhoidectomy or procedure for prola
pse and hemorrhoids (PPH), is a surgical technique used for the treatment of hemorrhoids. It aims to r
emove or reduce prolapsed hemorrhoidal tissue and restore normal anatomical position.
Option B - Well's procedure: Well procedure The given image is not of the well procedure.
Option D - Altemeier repair: The Altemeier repair, also known as the perineal rectosigmoidectomy or th
e perineal proctosigmoidectomy, is a
surgical procedure used to treat rectal prolapse. It involves the removal of a

Page 81

952
segment of the sigmoid colon and rectum through an incision in the perineal area

Solution for Question 51:


Correct Option B - Direct hernia and Lichtenstein mesh repair:
• The most likely diagnosis in this case is a direct inguinal hernia. Direct hernias occur medially to the
inferior epigastric artery, whereas indirect hernias occur lateral to the artery.
• The appropriate management for a direct inguinal hernia is typically surgical repair using a
tension-free mesh technique. The two options given for mesh repair in the question are the Bassini
repair and the Lichtenstein mesh repair.
• The Lichtenstein mesh repair is the preferred approach for most inguinal hernias, including direct
hernias. It involves placing a mesh patch over the hernia defect to provide support and prevent
recurrence. The Lichtenstein mesh repair has been shown to have lower recurrence rates and a faster
recovery compared to tissue-based repairs.
The most likely diagnosis in this case is a direct inguinal hernia. Direct hernias occur medially to the inf
erior epigastric artery, whereas indirect hernias occur lateral to the artery.
The appropriate management for a direct inguinal hernia is typically surgical repair using a tension-free
mesh technique. The two options given for mesh repair in the question are the Bassini repair and the L
ichtenstein mesh repair.
The Lichtenstein mesh repair is the preferred approach for most inguinal hernias, including direct herni
as. It involves placing a mesh patch over the hernia defect to provide support and prevent recurrence.
The Lichtenstein mesh repair has been shown to have lower recurrence rates and a
faster recovery compared to tissue-based repairs.
Incorrect Options:
Option A - Direct hernia and Bassini repair: The Bassini repair is a traditional tissue-based repair that in
volves suturing the hernia defect and reinforcing the weakened area with adjacent tissue. However Lic
htenstein repairs are preferred over Bassini repair.
Option C - Indirect hernia and Bassini repair: This is not a case of indirect hernia repair.
Option D - Indirect hernia and Lichtenstein mesh repair: This is not a case of indirect hernia repair

Solution for Question 52:


Correct Option A - Gallstone ileus:
• The most likely diagnosis in this case is Gallstone ileus.
• The presentation of gallstone ileus includes abdominal pain, bilious vomiting, and signs of intestinal
obstruction.
• The history of abdominal hysterectomy may be relevant if it suggests previous surgical interventions
or conditions that increase the risk of developing a cholecystoenteric fistula.
The most likely diagnosis in this case is Gallstone ileus.

Page 82

953
The presentation of gallstone ileus includes abdominal pain, bilious vomiting, and signs of intestinal ob
struction.
The history of abdominal hysterectomy may be relevant if it suggests previous surgical interventions or
conditions that increase the risk of developing a cholecystoenteric fistula.
Incorrect Options:
Option B - Adhesive intestinal obstruction:
• It occurs when adhesions, which are bands of scar tissue, develop within the abdomen following
previous abdominal surgeries or abdominal inflammation. These adhesions can cause the intestines to
become twisted, kinked, or trapped, resulting in mechanical obstruction.
• The typical presentation of adhesive intestinal obstruction includes abdominal pain, distention, and
vomiting. The onset of symptoms may occur years after the initial surgery or inflammation. On
examination, high-pitched bowel sounds.
It occurs when adhesions, which are bands of scar tissue, develop within the abdomen following previo
us abdominal surgeries or abdominal inflammation. These adhesions can cause the intestines to beco
me twisted, kinked, or trapped, resulting in mechanical obstruction.
The typical presentation of adhesive intestinal obstruction includes abdominal pain, distention, and vo
miting. The onset of symptoms may occur years after the initial surgery or inflammation. On examinatio
n, high-pitched bowel sounds.
Option C - Ischemic enterocolitis:
• Ischemic enterocolitis refers to inflammation and injury of the large intestine (colon) due to inadequate
blood supply. It can be caused by various factors, such as arterial blockage, low blood pressure, or
vasoconstriction. Common symptoms include abdominal pain, bloody stools, and diarrhea. However,
the presence of air within the biliary tree is not typically associated with ischemic enterocolitis.
Ischemic enterocolitis refers to inflammation and injury of the large intestine (colon) due to inadequate
blood supply. It can be caused by various factors, such as arterial blockage, low blood pressure, or vas
oconstriction. Common symptoms include abdominal pain, bloody stools, and diarrhea. However, the p
resence of air within the biliary tree is not typically associated with ischemic enterocolitis.
Option D - Diverticulitis:
• It is an inflammatory condition that occurs when the diverticula, which are small pouches that develop
in the colon, become infected or inflamed. It commonly presents with abdominal pain, often in the left
lower quadrant, along with other symptoms such as fever, nausea, and changes in bowel habits. In
severe cases, complications such as abscess formation or bowel obstruction may occur.
It is an inflammatory condition that occurs when the diverticula, which are small pouches that develop i
n the colon, become infected or inflamed. It commonly presents with abdominal pain, often in the left lo
wer quadrant, along with other symptoms such as fever, nausea, and changes in bowel habits. In seve
re cases, complications such as abscess formation or bowel obstruction may occur.

Solution for Question 53:


Correct Option A - Palliative gastrojejunostomy followed by chemotherapy:
• Given the advanced stage of the disease with involvement of the pancreas and liver metastasis,
curative surgical options such as radical gastrectomy or Whipple's procedure may not be feasible or
effective in achieving a complete cure.

Page 83

954
• Palliative gastrojejunostomy can be performed to alleviate symptoms related to gastric outlet
obstruction, such as persistent vomiting and abdominal pain. This procedure creates an alternative
pathway for food to bypass the obstruction, improving the patient's quality of life.
Given the advanced stage of the disease with involvement of the pancreas and liver metastasis, curativ
e surgical options such as radical gastrectomy or Whipple's procedure may not be feasible or effective i
n achieving a complete cure.
Palliative gastrojejunostomy can be performed to alleviate symptoms related to gastric outlet obstructio
n, such as persistent vomiting and abdominal pain. This procedure creates an alternative pathway for f
ood to bypass the obstruction, improving the patient's quality of life.
Incorrect Options:
Option B - Radical gastrectomy:
• It is not feasible in this condition. Radical gastrectomy involves the surgical removal of the entire
stomach along with the surrounding lymph nodes and any involved adjacent organs, such as the head
of the pancreas in this case.
It is not feasible in this condition. Radical gastrectomy involves the surgical removal of the entire stoma
ch along with the surrounding lymph nodes and any involved adjacent organs, such as the head of the
pancreas in this case.
Option C - Whipple's procedure:
• It involves the removal of the head of the pancreas, the first part of the small intestine (duodenum),
the gallbladder, and sometimes a portion of the stomach. In some cases, the surgery may also include
removal of nearby lymph nodes and parts of the bile duct.
• Whipple's procedure is considered the treatment of choice for tumors located in the periampullary
region, which includes the ampulla of Vater, the duodenum, and the head of the pancreas.
It involves the removal of the head of the pancreas, the first part of the small intestine (duodenum), the
gallbladder, and sometimes a portion of the stomach. In some cases, the surgery may also include rem
oval of nearby lymph nodes and parts of the bile duct.
Whipple's procedure is considered the treatment of choice for tumors located in the periampullary regio
n, which includes the ampulla of Vater, the duodenum, and the head of the pancreas.
Option D - Gastrectomy with right hepatectomy:
• The procedure involves the removal of the affected portion of the stomach (gastrectomy) and the right
lobe of the liver (right hepatectomy) to ensure complete removal of the tumor and any associated
metastasis.
• Gastrectomy with right hepatectomy is a complex surgical procedure performed in cases of advanced
gastric cancer with liver metastasis
The procedure involves the removal of the affected portion of the stomach (gastrectomy) and the right l
obe of the liver (right hepatectomy) to ensure complete removal of the tumor and any associated metas
tasis.
Gastrectomy with right hepatectomy is a
complex surgical procedure performed in cases of advanced gastric cancer with liver metastasis

Solution for Question 54:

Page 84

955
Correct Option B - Foreign body in the esophagus:
The given image is of a Foreign body in the esophagus.
• Location Esophagus: Located behind the trachea. Trachea: Main airway allowing lung airflow.
• Esophagus: Located behind the trachea.
• Trachea: Main airway allowing lung airflow.
• Symptoms Esophagus: Difficulty swallowing, chest pain, regurgitation, sensation of throat obstruction.
Trachea: Choking, coughing, wheezing, breathing difficulty, stridor.
• Esophagus: Difficulty swallowing, chest pain, regurgitation, sensation of throat obstruction.
• Trachea: Choking, coughing, wheezing, breathing difficulty, stridor.
• Complications Esophagus: Risks include esophageal tear, infection, bleeding, and fistula formation.
Trachea: Potential for complete airway blockage, severe respiratory distress, oxygen deprivation, and
life-threatening consequences.
• Esophagus: Risks include esophageal tear, infection, bleeding, and fistula formation.
• Trachea: Potential for complete airway blockage, severe respiratory distress, oxygen deprivation, and
life-threatening consequences.
• Management Esophagus: Endoscopic removal primarily. Trachea: Immediate interventions such as
Heimlich maneuver, back blows, chest thrusts, emergency tracheostomy, or bronchoscopy.
• Esophagus: Endoscopic removal primarily.
• Trachea: Immediate interventions such as Heimlich maneuver, back blows, chest thrusts, emergency
tracheostomy, or bronchoscopy.
Location
• Esophagus: Located behind the trachea.
• Trachea: Main airway allowing lung airflow.
Esophagus: Located behind the trachea.
Trachea: Main airway allowing lung airflow.
Symptoms
• Esophagus: Difficulty swallowing, chest pain, regurgitation, sensation of throat obstruction.
• Trachea: Choking, coughing, wheezing, breathing difficulty, stridor.
Esophagus: Difficulty swallowing, chest pain, regurgitation, sensation of throat obstruction.
Trachea: Choking, coughing, wheezing, breathing difficulty, stridor.
Complications
• Esophagus: Risks include esophageal tear, infection, bleeding, and fistula formation.
• Trachea: Potential for complete airway blockage, severe respiratory distress, oxygen deprivation, and
life-threatening consequences.
Esophagus: Risks include esophageal tear, infection, bleeding, and fistula formation.
Trachea: Potential for complete airway blockage, severe respiratory distress, oxygen deprivation, and li
fe-threatening consequences.
Management

Page 85

956
• Esophagus: Endoscopic removal primarily.
• Trachea: Immediate interventions such as Heimlich maneuver, back blows, chest thrusts, emergency
tracheostomy, or bronchoscopy.
Esophagus: Endoscopic removal primarily.
Trachea: Immediate interventions such as Heimlich maneuver, back blows, chest thrusts, emergency tr
acheostomy, or bronchoscopy.
Incorrect Options:
Option A , C & D: They are incorrect as the above given image is of a foreign body in the esophagus

Solution for Question 55:


Correct Option A - Hollow viscus perforation:
• When a hollow viscus perforation occurs, the contents of the organ, including digestive juices and
bacteria, can leak into the surrounding tissues or abdominal cavity, leading to a serious condition
known as peritonitis. Peritonitis is characterized by severe abdominal pain, tenderness, rigidity, and
signs of infection such as fever and increased heart rate.
• Immediate surgical intervention is usually required to repair the perforation and prevent further
complications.
When a hollow viscus perforation occurs, the contents of the organ, including digestive juices and bact
eria, can leak into the surrounding tissues or abdominal cavity, leading to a serious condition known as
peritonitis. Peritonitis is characterized by severe abdominal pain, tenderness, rigidity, and signs of infe
ction such as fever and increased heart rate.
Immediate surgical intervention is usually required to repair the perforation and prevent further complic
ations.
Incorrect Options:
Option: B - Gastric volvulus:
• Gastric volvulus is a condition characterized by the abnormal twisting or rotation of the stomach
around its axis. It can lead to obstruction of the stomach and impede the passage of food and fluids.
There are two main types of gastric volvulus: Organo-axial volvulus Mesentero-axial volvulus
• Organo-axial volvulus
• Mesentero-axial volvulus
Gastric volvulus is a condition characterized by the abnormal twisting or rotation of the stomach around
its axis. It can lead to obstruction of the stomach and impede the passage of food and fluids. There ar
e two main types of gastric volvulus:
• Organo-axial volvulus
• Mesentero-axial volvulus
Organo-axial volvulus
Mesentero-axial volvulus
Option C - Liver abscess:

Page 86

957
• A liver abscess is a localized collection of pus within the liver tissue. It is usually caused by a bacterial
infection, with the most common organisms being Escherichia coli and Klebsiella pneumoniae. Liver
abscesses can occur due to various factors, including biliary tract infections, ascending cholangitis,
bloodstream infections, or the spread of infection from other nearby organs.
A liver abscess is a localized collection of pus within the liver tissue. It is usually caused by a bacterial i
nfection, with the most common organisms being Escherichia coli and Klebsiella pneumoniae. Liver ab
scesses can occur due to various factors, including biliary tract infections, ascending cholangitis, blood
stream infections, or the spread of infection from other nearby organs.
Option D - Empyema thoracis:
• Also known as pyothorax, refers to the presence of infected fluid (pus) in the pleural space, which is
the space between the lungs and the chest wall. It is typically caused by a bacterial infection, most
commonly due to the spread of an underlying lung infection such as pneumonia.
Also known as pyothorax, refers to the presence of infected fluid (pus) in the pleural space, which is th
e space between the lungs and the chest wall. It is typically caused by a bacterial infection, most comm
only due to the spread of an underlying lung infection such as pneumonia.

Solution for Question 56:


Correct Option A - Boerrhave syndrome:
• In the given scenario of a 28-year-old alcoholic patient presenting with binge vomiting, chest pain,
fever, and hydropneumothorax, the probable diagnosis is Boerrhave syndrome.
• Boerrhave Syndrome also known as spontaneous esophageal rupture, is a rare but life-threatening
condition characterized by a full-thickness tear or rupture of the esophagus.
• It typically occurs due to a sudden increase in intraesophageal pressure, often as a result of severe
and forceful vomiting.
• The classic triad of Boerrhave syndrome includes severe chest pain, vomiting, and subcutaneous
emphysema.
In the given scenario of a 28-year-old alcoholic patient presenting with binge vomiting, chest pain, fever
, and hydropneumothorax, the probable diagnosis is Boerrhave syndrome.
Boerrhave Syndrome also known as spontaneous esophageal rupture, is a
rare but life-threatening condition characterized by a full-thickness tear or rupture of the esophagus.
It typically occurs due to a sudden increase in intraesophageal pressure, often as a
result of severe and forceful vomiting.
The classic triad of Boerrhave syndrome includes severe chest pain, vomiting, and subcutaneous emp
hysema.
Incorrect Options:
Option B - Tension Pneumothorax: Tension pneumothorax occurs when air accumulates in the pleural
space and cannot escape, leading to increased pressure on the affected lung and mediastinal shift. Wh
ile tension pneumothorax can cause respiratory distress, chest pain, and other symptoms, it is not dire
ctly related to the binge vomiting and hydropneumothorax in this scenario.
Option C - PUD Perforation: PUD (Peptic Ulcer Disease) perforation refers to a complication of peptic u
lcer disease in which an ulcer in the stomach or duodenum perforates through the gastric or duodenal
wall. This can lead to the leakage of gastric or duodenal contents into the peritoneal cavity, resulting in

Page 87

958
peritonitis. While PUD perforation can cause abdominal pain and fever, it is not directly associated with
the symptoms of binge vomiting, chest pain, and hydropneumothorax described in the scenario.
Option D - Mallory Weiss Tear: Mallory Weiss tear is a condition characterized by a mucosal tear at the
gastroesophageal junction, often caused by severe or prolonged vomiting or retching. While it can cau
se upper gastrointestinal bleeding, it is not typically associated with chest pain, fever, or hydropneumot
horax.

Solution for Question 57:


Correct Option A - Early Dumping syndrome:
• Dumping syndrome is a condition that can occur after gastrectomy, a surgical procedure where a part
or the entire stomach is removed.
• It is characterized by a rapid emptying of the stomach contents into the small intestine, leading to
various symptoms.
• In this scenario, the patient experiences symptoms such as sweating and diarrhea within 20 minutes
after eating.
• These symptoms are characteristic of early dumping syndrome.
Dumping syndrome is a condition that can occur after gastrectomy, a surgical procedure where a
part or the entire stomach is removed.
It is characterized by a
rapid emptying of the stomach contents into the small intestine, leading to various symptoms.
In this scenario, the patient experiences symptoms such as sweating and diarrhea within 20 minutes af
ter eating.
These symptoms are characteristic of early dumping syndrome.
Incorrect Options:
Option B - Late dumbing syndrome: Late dumping syndrome occurs 1 to 3 hours after a meal and is ch
aracterized by symptoms like weakness, lightheadedness, sweating, and palpitations due to reactive h
ypoglycemia. The symptoms in this scenario occur within 20 minutes after eating, which is more consis
tent with early dumping syndrome.
Option C - Hyperglycemia: Hyperglycemia refers to high blood glucose levels. While it can cause symp
toms such as increased thirst, frequent urination, and fatigue, it is not typically associated with sweatin
g and diarrhea after eating.
Option D - Hypoglycemia: Hypoglycemia refers to low blood glucose levels. While it can cause sympto
ms such as sweating, dizziness, confusion, and hunger, it is not typically associated with diarrhea after
eating. Hypoglycemia may occur as a result of excessive insulin release during the early dumping synd
rome, but the predominant symptoms in this case are sweating and diarrhea.

Solution for Question 58:


Correct Option B - Surgery after 5 year:

Page 88

959
• In the given clinical scenario a 6-month-old child presents with an image suggestive of umbilical
hernia.
• Umbilical hernias occur when a part of the intestine or abdominal tissue pushes through the
abdominal muscles near the belly button.
• In the case, the management approach typically involves surgery after 5 years.
• Umbilical hernias in infants often resolve on their own without any intervention by the age of 2 years
but persistence beyond 5 years is an indication of surgery.
In the given clinical scenario a 6-month-old child presents with an image suggestive of umbilical hernia.
Umbilical hernias occur when a
part of the intestine or abdominal tissue pushes through the abdominal muscles near the belly button.
In the case, the management approach typically involves surgery after 5 years.
Umbilical hernias in infants often resolve on their own without any intervention by the age of 2
years but persistence beyond 5 years is an indication of surgery.
Incorrect Options:
Option A - Indication of surgery if not resolved by 2 years:
• Indication of surgery if not resolved by 2 years is not right as surgery is indicated after 5 yr age
Indication of surgery if not resolved by 2 years is not right as surgery is indicated after 5 yr age
Option C - Immediate surgery:
• Immediate surgery, is incorrect for uncomplicated umbilical hernias in infants.
Immediate surgery, is incorrect for uncomplicated umbilical hernias in infants.
Option D - Hernioplasty:
• Hernioplasty is a surgical technique used to repair hernias.
Hernioplasty is a surgical technique used to repair hernias.

Solution for Question 59:


Correct Option D - Bile salts:
• The distal ileum plays a crucial role in the absorption of bile salts, which are essential for the digestion
and absorption of dietary fats.
• Without the distal ileum, there can be impaired absorption of bile salts, leading to difficulties in fat
digestion and subsequent deficiencies of fat-soluble vitamins, such as vitamins A, D, E, and K.
Incorrect Options:
Option A - Iron: Iron absorption primarily occurs in the duodenum.
Option B - Folic acid: Folic acid, or folate, is absorbed in the upper small intestine, particularly in the du
odenum and jejunum.
Option C - Copper: Copper absorption occurs primarily in the stomach and upper small intestine.

Page 89

960
Solution for Question 60:
Correct Option C - Oxalate food:
• Oxalate-rich foods are not considered a risk factor for esophageal carcinoma.
• Oxalate-rich foods, such as spinach, rhubarb, and beets, are not considered risk factors for
esophageal carcinoma.
• While oxalate-rich foods can contribute to the formation of kidney stones in some individuals, they are
not directly linked to the development of esophageal cancer.
Oxalate-rich foods are not considered a risk factor for esophageal carcinoma.
Oxalate-rich foods, such as spinach, rhubarb, and beets, are not considered risk factors for esophagea
l carcinoma.
While oxalate-rich foods can contribute to the formation of kidney stones in some individuals, they are
not directly linked to the development of esophageal cancer.
Incorrect Options:
Option A - GERD (Gastroesophageal reflux disease): GERD is a risk factor for esophageal carcinoma.
Chronic reflux of stomach acid into the esophagus can lead to chronic inflammation, cellular damage, a
nd an increased risk of developing esophageal cancer.
Option B - Betel chewing: Betel chewing is associated with an increased risk of developing esophageal
cancer. Betel nuts contain carcinogenic substances, and the habit of chewing betel nuts is prevalent in
certain regions, especially in South Asia.
Option D - Spicy food: The relationship between spicy food and esophageal carcinoma is not well-esta
blished. While spicy food can potentially irritate the esophagus and worsen symptoms in individuals wit
h pre-existing conditions like GERD, it is not considered a
direct risk factor for developing esophageal cancer.

Solution for Question 61:


Correct Option C - Tenderness in right lower quadrant:
• The Alvarado score is a scoring system used in the assessment of suspected acute appendicitis.
• It takes into account various clinical signs, symptoms, and laboratory findings to help evaluate the
likelihood of appendicitis.
The Alvarado score is a scoring system used in the assessment of suspected acute appendicitis.
It takes into account various clinical signs, symptoms, and laboratory findings to help evaluate the likeli
hood of appendicitis.
Incorrect Options:
Option A - Nausea: Only 1 point is given for nausea.
Option B - Temperature: The Alvarado score assigns 1 point for a elevated temperature.
Option D - Migratory pain: Only 1 point is given for Migratory pain

Page 90

961
Solution for Question 62:
Correct Option C - Sister mary joseph nodule:
• The given clinical image in a patient with adenocarcinoma stomach shows Sister Mary Joseph nodes.
• The presence of cutaneous metastatic deposits around the umbilicus is referred to as Sister Mary
Joseph nodes, which are not lymph nodes.
The given clinical image in a patient with adenocarcinoma stomach shows Sister Mary Joseph nodes.
The presence of cutaneous metastatic deposits around the umbilicus is referred to as Sister Mary Jose
ph nodes, which are not lymph nodes.
Incorrect Options:
Option A - Virchow's node: Virchow's node, also known as left supraclavicular lymph node enlargement
, is often associated with metastatic cancer, particularly from the gastrointestinal tract or pelvic region.
While it is a sign of potential malignancy, it is not specifically associated with the umbilicus, as seen in t
he image provided.
Option B - Irish node: Irish node is a palpable left anterior axillary lymph node. It is thought to arise fro
m intrathoracic metastatic spread of an intra-abdominal malignancy, particularly gastric cancer
Option D - Infected umbilical hernia: An infected hernia would typically present with signs of inflammati
on, such as redness, warmth, and tenderness, rather than a discrete nodule.

Solution for Question 63:


Correct Option D - Stage 4:
• The correct Hinchey's stage for a patient presenting with a diverticular perforation and fecal peritonitis
is Stage 4.
• Hinchey's classification is a system used to classify the severity of diverticulitis based on the
intraoperative findings. It helps in determining the appropriate management and prognosis.
• It indicates the most severe form of diverticulitis with diffuse fecal contamination of the peritoneal
cavity.
• Timely surgical intervention is crucial in these cases to control the infection, remove the source of
contamination, and prevent further complications.
• Stage 4: This stage represents diverticulitis with generalized fecal peritonitis. It signifies a perforation
of the diverticulum with the spread of fecal material throughout the peritoneal cavity, leading to
widespread inflammation and contamination.
The correct Hinchey's stage for a patient presenting with a
diverticular perforation and fecal peritonitis is Stage 4.
Hinchey's classification is a system used to classify the severity of diverticulitis based on the intraopera
tive findings. It helps in determining the appropriate management and prognosis.
It indicates the most severe form of diverticulitis with diffuse fecal contamination of the peritoneal cavity
.
Timely surgical intervention is crucial in these cases to control the infection, remove the source of cont
amination, and prevent further complications.

Page 91

962
Stage 4: This stage represents diverticulitis with generalized fecal peritonitis. It signifies a perforation of
the diverticulum with the spread of fecal material throughout the peritoneal cavity, leading to widespre
ad inflammation and contamination.
Incorrect Options:
Option A - Stage 1: This stage refers to localized diverticulitis without evidence of perforation or absces
s formation. It typically presents as localized inflammation or a phlegmon.
Option B - Stage 2: This stage involves diverticulitis with a pericolic or mesocolic abscess. It is charact
erized by the presence of an abscess adjacent to the affected colonic segment.
Option C - Stage 3: This stage includes diverticulitis with a
distant abscess or generalized purulent peritonitis. It typically indicates the presence of a
distant abscess in the abdominal cavity or diffuse contamination of the peritoneal cavity with pus.

Solution for Question 64:


Correct Option A - Spontaneous perforation of esophagus:
• Spontaneous perforation of the esophagus, also known as Boerhaave syndrome, is a condition
characterized by a rupture or tear in the esophagus.
• It typically occurs due to a sudden increase in intraesophageal pressure, often caused by forceful
vomiting or retching.
• The history of heavy meal and upper abdominal tenderness, along with the findings of
pneumomediastinum and widening of the mediastinum on the chest radiography, strongly suggest
spontaneous perforation of the esophagus.
Spontaneous perforation of the esophagus, also known as Boerhaave syndrome, is a
condition characterized by a rupture or tear in the esophagus.
It typically occurs due to a
sudden increase in intraesophageal pressure, often caused by forceful vomiting or retching.
The history of heavy meal and upper abdominal tenderness, along with the findings of pneumomediasti
num and widening of the mediastinum on the chest radiography, strongly suggest spontaneous perfora
tion of the esophagus.
Incorrect Options:
Option B - Perforation of peptic ulcer: Perforation of a peptic ulcer typically causes localized abdominal
pain and tenderness. While it can lead to pneumoperitoneum (air in the abdominal cavity), it is less likel
y to cause pneumomediastinum or widening of the mediastinum on chest radiography. Therefore, this
option is less likely in the given scenario.
Option C - Foreign body in the esophagus: This option is unlikely based on the presented symptoms a
nd findings. Foreign body ingestion may cause symptoms such as dysphagia or odynophagia, but it wo
uld not typically present with upper abdominal pain or pneumomediastinum. The imaging findings of m
ediastinal widening are not consistent with a foreign body in the esophagus.
Option D - Rupture of emphysematous bulla of the lung: This option is also less likely in this case. Rupt
ure of an emphysematous bulla is characterized by sudden-onset pleuritic chest pain and may cause p
neumothorax. However, it would not typically present with upper abdominal pain or mediastinal widenin
g.

Page 92

963
Solution for Question 65:
Correct Option D - Type III achalasia:
• Type III achalasia is characterized by elevated median integrated relaxation pressure (IRP) (>15
mmHg), absence of normal peristalsis, and premature (spastic) contractions with distal contractile
integral (DCI) values >450 mmHg·s·cm in at least 20% of swallows.
• In this case, the patient exhibits massive, simultaneous, high-pressure contractions of the distal
esophagus, indicating spastic contractions.
• These findings align with the criteria for Type III achalasia as per the Chicago Classification.
Type III achalasia is characterized by elevated median integrated relaxation pressure (IRP) (>15 mmH
g), absence of normal peristalsis, and premature (spastic) contractions with distal contractile integral (D
CI) values >450 mmHg·s·cm in at least 20% of swallows.
In this case, the patient exhibits massive, simultaneous, high-pressure contractions of the distal esoph
agus, indicating spastic contractions.
These findings align with the criteria for Type III achalasia as per the Chicago Classification.
Incorrect Options:
Option A - Type II achalasia:
• Type II achalasia is characterized by elevated median IRP (>15 mmHg), 100% failed peristalsis, and
panoesophageal pressurization with ≥20% of swallows.
• The patient in the scenario does not exhibit panoesophageal pressurization, but rather massive,
simultaneous, high-pressure contractions, indicating spastic contractions rather than oesophageal
compression.
Type II achalasia is characterized by elevated median IRP (>15 mmHg), 100% failed peristalsis, and p
anoesophageal pressurization with ≥20% of swallows.
The patient in the scenario does not exhibit panoesophageal pressurization, but rather massive, simult
aneous, high-pressure contractions, indicating spastic contractions rather than oesophageal compressi
on.
Option B - Type I achalasia:
• Type I achalasia is characterized by elevated median IRP (>15 mmHg) and 100% failed peristalsis
with DCI <100 mmHg·s·cm.
• The patient in the scenario exhibits massive, simultaneous, high-pressure contractions with DCI >450
mmHg·s·cm, which does not align with the criteria for Type I achalasia.
Type I achalasia is characterized by elevated median IRP (>15 mmHg) and 100% failed peristalsis with
DCI <100 mmHg·s·cm.
The patient in the scenario exhibits massive, simultaneous, high-pressure contractions with DCI >450
mmHg·s·cm, which does not align with the criteria for Type I achalasia.
Option C - Type IV achalasia:
• Type IV achalasia is not a recognized subtype according to the Chicago Classification.
• The correct option for the patient's presentation is Type III achalasia based on the provided
information and the criteria outlined in the Chicago Classification.

Page 93

964
Type IV achalasia is not a recognized subtype according to the Chicago Classification.
The correct option for the patient's presentation is Type III achalasia based on the provided information
and the criteria outlined in the Chicago Classification.

Solution for Question 66:


Correct Option D - Esophagitis:
• Esophagitis, characterized by inflammation of the esophagus, presents symptoms such as chest pain,
heartburn, acid regurgitation, and difficulty swallowing. Treatment typically involves Proton Pump
Inhibitors (PPIs). Esophagitis is categorized based on its underlying causes: Reflux esophagitis
Eosinophilic esophagitis Lymphocytic esophagitis Drug-induced esophagitis Infectious esophagitis
• Reflux esophagitis
• Eosinophilic esophagitis
• Lymphocytic esophagitis
• Drug-induced esophagitis
• Infectious esophagitis
Esophagitis, characterized by inflammation of the esophagus, presents symptoms such as chest pain,
heartburn, acid regurgitation, and difficulty swallowing. Treatment typically involves Proton Pump Inhibi
tors (PPIs). Esophagitis is categorized based on its underlying causes:
• Reflux esophagitis
• Eosinophilic esophagitis
• Lymphocytic esophagitis
• Drug-induced esophagitis
• Infectious esophagitis
Reflux esophagitis
Eosinophilic esophagitis
Lymphocytic esophagitis
Drug-induced esophagitis
Infectious esophagitis
Incorrect Options:
Option A - Barrett's esophagus: In lower part or distal esophagus, the Squamous mucosa is replaced b
y columnar mucosa, there is a long history of heartburn.
Option B - Pneumonia: Pneumonia is an infection that affects one or both lungs. The air sacs/alveoli of
the lungs are filled up with fluid or pus.
Option C - Myocardial infarction: Caused by decreased supply of blood flow to a
portion of the myocardium.

Page 94

965
Solution for Question 67:
Correct Option B - Foreign body in the esophagus:
• The narrowest part of the esophagus is the cricopharynx, with a diameter of approximately 1.5 cm
(15mm). Any foreign object small enough to pass through this point can potentially be ingested,
absorbed, and later expelled through bowel movements. Exceptions include sharp objects and button
batteries, which require intervention for removal. On a chest X-ray, the position of a coin can be
determined: in the antero-posterior view, the edge of the coin is visible, indicating its location; in the
lateral view, the front of the coin indicates whether it is in the trachea or the esophagus.
The narrowest part of the esophagus is the cricopharynx, with a diameter of approximately 1.5 cm (15
mm). Any foreign object small enough to pass through this point can potentially be ingested, absorbed,
and later expelled through bowel movements. Exceptions include sharp objects and button batteries,
which require intervention for removal. On a chest X-ray, the position of a coin can be determined: in th
e antero-posterior view, the edge of the coin is visible, indicating its location; in the lateral view, the fron
t of the coin indicates whether it is in the trachea or the esophagus.

Incorrect Options:
Option A - Foreign body in the trachea: These are foreign bodies most commonly caused by aspiration.
Option C - Foreign body in lungs: Bronchoscopy is the standard method for the removal of an aspirated
foreign body in the lung
Option D - Foreign body in pleura: Foreign bodies in the pleura and chest are mostly removed by surgi
cal means. Complications can lead to pneumonia, lung abscess, pulmonary gangrene, and empyema.

Solution for Question 68:


Correct Option D - Celiac sprue:
• In Celiac sprue, antibodies play a crucial role, with the most sensitive being anti-TTG antibody, and
anti-endomysial antibody being the most specific. Notably, symptoms are often remembered through
the "Four D's": Diarrhea triggered by gluten ingestion Duodenum primarily affected, often leading to iron

Page 95

966
deficiency anemia Dermatitis herpetiformis, a skin disorder linked to IgA, typically treated with Dapsone
• Diarrhea triggered by gluten ingestion
• Duodenum primarily affected, often leading to iron deficiency anemia
• Dermatitis herpetiformis, a skin disorder linked to IgA, typically treated with Dapsone
• In adults, symptoms include diarrhea, fatigue, weight loss, bloating, gas, abdominal pain, nausea,
vomiting, and constipation. Children typically experience nausea, vomiting, chronic diarrhea, a swollen
belly, constipation, gas, and pale, foul-smelling stools as primary symptoms.
In Celiac sprue, antibodies play a crucial role, with the most sensitive being anti-TTG antibody, and anti
-endomysial antibody being the most specific. Notably, symptoms are often remembered through the "
Four D's":
• Diarrhea triggered by gluten ingestion
• Duodenum primarily affected, often leading to iron deficiency anemia
• Dermatitis herpetiformis, a skin disorder linked to IgA, typically treated with Dapsone
Diarrhea triggered by gluten ingestion
Duodenum primarily affected, often leading to iron deficiency anemia
Dermatitis herpetiformis, a skin disorder linked to IgA, typically treated with Dapsone
In adults, symptoms include diarrhea, fatigue, weight loss, bloating, gas, abdominal pain, nausea, vomi
ting, and constipation. Children typically experience nausea, vomiting, chronic diarrhea, a
swollen belly, constipation, gas, and pale, foul-smelling stools as primary symptoms.
Incorrect Options:
Option A - Short bowel syndrome: Short bowel syndrome is a condition in which your body is unable to
absorb enough nutrients because of the inadequate small intestine. The diagnosis is based on detailed
patient history, clinical evaluation, and a
variety of specialized tests including laboratory tests(CBC) and X-rays.
Option B - Whipple’s disease: The diagnosis of Whipple's disease is made by biopsy of the intestine an
d identification of the organism. Current diagnostic criteria require positive results for PAS-positive foa
my macrophages in the small bowel biopsy.
Option C - Tropical sprue: The investigations include CBC, quantitative fecal fat assay, D-xylose test, a
nd serum folate.

Solution for Question 69:


Correct Option C - Urgent surgical exploration with ileostomy:
• Usually Crohns disease is treated using anti-inflammatory drugs, immune suppressors, antibiotics,
and nutrition therapy. Since in this case there is ileal perforation, there is a need for Urgent surgical
exploration with ileostomy to contain the bleeding.
Usually Crohns disease is treated using anti-inflammatory drugs, immune suppressors, antibiotics, and
nutrition therapy. Since in this case there is ileal perforation, there is a
need for Urgent surgical exploration with ileostomy to contain the bleeding.
Incorrect Options:

Page 96

967
Option A - Antibiotics and NPO: In ileal perforation, surgery is indicated.
Option B - Upper GI endoscopy: In ileal perforation, surgery is indicated.
Option D - CT abdomen: A CT scan is used to look for signs of Crohn's disease, such as inflammation,
ulcers, and thickening intestinal walls. Surgery is indicated for ileal perforation.

Solution for Question 70:


Correct Option B - 11 o’clock and at internal anal sphincter:
• According to Good Sall's Rule, an anteriorly located external opening corresponds to a short straight
track internally.
• Since the external opening is located at the 11 o'clock position, the internal opening is also expected
to be at the 11 o'clock position.
• The internal opening is situated within the internal anal sphincter.
According to Good Sall's Rule, an anteriorly located external opening corresponds to a
short straight track internally.
Since the external opening is located at the 11 o'clock position, the internal opening is also expected to
be at the 11 o'clock position.
The internal opening is situated within the internal anal sphincter.
Incorrect Options:
Option A - 12 o'clock and at the external anal sphincter:
• The external opening is at the 11 o'clock position, not the 12 o'clock position.
• Additionally, the internal opening is expected to be within the internal anal sphincter, not the external
anal sphincter.
The external opening is at the 11 o'clock position, not the 12 o'clock position.
Additionally, the internal opening is expected to be within the internal anal sphincter, not the external a
nal sphincter.
Option C - 9 o'clock and at the anal verge:
• The external opening is at the 11 o'clock position, not the 9 o'clock position.
• Also, Good Sall's Rule states that anteriorly located external openings correspond to the internal
opening at the same position, not at the anal verge.
The external opening is at the 11 o'clock position, not the 9 o'clock position.
Also, Good Sall's Rule states that anteriorly located external openings correspond to the internal openi
ng at the same position, not at the anal verge.
Option D - 6 o'clock and crypt of the dentate line:
• It suggests that the internal opening is always at the 6 o'clock position, which is an exception to Good
Sall's Rule.
• Additionally, the external opening is not posteriorly located, so it wouldn't follow the rule of the internal
opening being at the 6 o'clock position in such cases.

Page 97

968
It suggests that the internal opening is always at the 6
o'clock position, which is an exception to Good Sall's Rule.
Additionally, the external opening is not posteriorly located, so it wouldn't follow the rule of the internal
opening being at the 6 o'clock position in such cases.

Solution for Question 71:


Correct Option A - 33:
• Pressure sores are chronic wounds caused by tissue necrosis from excessive pressure on bony
areas.
• When external pressure surpasses capillary perfusion pressure (16 to 33 mm Hg), it leads to
ischaemic necrosis.
• Pressures exceeding 33 mm Hg block blood vessels, causing tissue oxygen deprivation and, if
prolonged, cell death and ulceration.
• Risk factors: immobility, incontinence, prolonged exposure to urine/stool, sensory impairment, poor
nutrition/hydration, and medical conditions affecting blood flow.
Pressure sores are chronic wounds caused by tissue necrosis from excessive pressure on bony areas.
When external pressure surpasses capillary perfusion pressure (16 to 33 mm Hg), it leads to ischaemic
necrosis.
Pressures exceeding 33 mm Hg block blood vessels, causing tissue oxygen deprivation and, if prolong
ed, cell death and ulceration.
Risk factors: immobility, incontinence, prolonged exposure to urine/stool, sensory impairment, poor nut
rition/hydration, and medical conditions affecting blood flow.
Incorrect Options
Option B, C, D: Any external pressure of more than 33 mm Hg occludes the blood vessel. As a conseq
uence, the underlying and surrounding tissues become anoxic.

Solution for Question 72:


Correct Option D - 1, 2, 3, and 4 only:
(1) Right and left colon cancers can behave differently
• True. Right-sided colon cancer tends to have a different clinical presentation, molecular
characteristics, and prognosis compared to left-sided colon cancer. Right-sided tumors are more likely
to present with vague symptoms and have a poorer prognosis compared to left-sided tumors.
True. Right-sided colon cancer tends to have a different clinical presentation, molecular characteristics,
and prognosis compared to left-sided colon cancer. Right-sided tumors are more likely to present with
vague symptoms and have a poorer prognosis compared to left-sided tumors.
(2) Colon cancers can be hereditary
• True. Some cases of colon cancer have a hereditary component. Hereditary conditions such as
familial adenomatous polyposis (FAP) and Lynch syndrome (hereditary nonpolyposis colorectal cancer)

Page 98

969
significantly increase the risk of developing colon cancer.
True. Some cases of colon cancer have a hereditary component. Hereditary conditions such as familial
adenomatous polyposis (FAP) and Lynch syndrome (hereditary nonpolyposis colorectal cancer) signifi
cantly increase the risk of developing colon cancer.
(3) Colon cancer risk can be reduced by metformin
• True. Metformin, a medication commonly used to treat type 2 diabetes, has been studied for its
potential protective effects against various cancers, including colon cancer. Some research suggests
that metformin may reduce the risk of developing colon cancer or improve outcomes in individuals with
existing colon cancer.
True. Metformin, a medication commonly used to treat type 2 diabetes, has been studied for its potenti
al protective effects against various cancers, including colon cancer. Some research suggests that met
formin may reduce the risk of developing colon cancer or improve outcomes in individuals with existing
colon cancer.
(4) Colon cancers frequently metastasize to the liver
• True. Liver metastases are common in colon cancer. The liver is a frequent site of metastasis due to
the anatomical proximity and the pattern of blood flow from the colon to the liver via the portal vein.
Liver metastases significantly affect the prognosis and treatment approach in colon cancer patients.
True. Liver metastases are common in colon cancer. The liver is a frequent site of metastasis due to th
e anatomical proximity and the pattern of blood flow from the colon to the liver via the portal vein. Liver
metastases significantly affect the prognosis and treatment approach in colon cancer patients.
Incorrect Options:
Option A - 1, 2 and 3 only:
• This option is incorrect because it excludes the statement "Colon cancers frequently metastasize to
the liver," which is true.
This option is incorrect because it excludes the statement "Colon cancers frequently metastasize to the
liver," which is true.
Option B - 2, 3 and 4 only:
• This option is incorrect because it excludes the statement "Right and left colon cancers can behave
differently," which is true.
This option is incorrect because it excludes the statement "Right and left colon cancers can behave diff
erently," which is true.
Option C - 2 and 4 only:
• This option is incorrect because it excludes the statements "Right and left colon cancers can behave
differently" and "Colon cancer risk can be reduced by metformin," both of which are true.
This option is incorrect because it excludes the statements "Right and left colon cancers can behave di
fferently" and "Colon cancer risk can be reduced by metformin," both of which are true.

Solution for Question 73:


Correct Option C - 1-C, 2-D, 3-A, 4-B:
The correct matching of the conditions with the corresponding investigations is:

Page 99

970
• 1-C (Hirschsprung's disease - Rectal biopsy)
• 2-D (Posterior urethral valve - Micturating cystourethrogram)
• 3-A (Choledochal cyst - MRCP)
• 4-B (Intussusception - CT scan)
1-C (Hirschsprung's disease - Rectal biopsy)
2-D (Posterior urethral valve - Micturating cystourethrogram)
3-A (Choledochal cyst - MRCP)
4-B (Intussusception - CT scan)
Incorrect Options:
• Option A - 1-C, 2-D, 3-B, 4-A: It mismatches the investigations for Hirschsprung's disease, Posterior
urethral valve, and Intussusception.
• Option B - 1-A, 2-D, 3-B, 4-C: It mismatches the investigation for Choledochal cyst.
• Option D - 1-D, 2-C, 3-A, 4-B: It mismatches the investigations for Hirschsprung's disease and
Choledochal cyst.
Option A - 1-C, 2-D, 3-B, 4-A: It mismatches the investigations for Hirschsprung's disease, Posterior ur
ethral valve, and Intussusception.
Option B - 1-A, 2-D, 3-B, 4-C: It mismatches the investigation for Choledochal cyst.
Option D
- 1-D, 2-C, 3-A, 4-B: It mismatches the investigations for Hirschsprung's disease and Choledochal cyst.

Solution for Question 74:


Correct Option C - Esophageal diverticula:
• The patient's symptoms of dysphagia, retrosternal pain, and regurgitation with coughing after meals
are suggestive of esophageal diverticula. Esophageal diverticula are outpouchings or pockets that
develop in the wall of the esophagus. These diverticula can interfere with the normal passage of food,
leading to symptoms such as dysphagia (difficulty swallowing) and regurgitation.
The patient's symptoms of dysphagia, retrosternal pain, and regurgitation with coughing after meals ar
e suggestive of esophageal diverticula. Esophageal diverticula are outpouchings or pockets that develo
p in the wall of the esophagus. These diverticula can interfere with the normal passage of food, leading
to symptoms such as dysphagia (difficulty swallowing) and regurgitation.
Incorrect Options:
Option A - Brown Patterson Kelly syndrome: Brown Patterson Kelly syndrome, also known as Plummer
-Vinson syndrome, is characterized by a triad of dysphagia, iron deficiency anemia, and esophageal w
ebs. It does not typically present with retrosternal pain, regurgitation, or coughing after meals, so it is n
ot the most likely diagnosis based on the given symptoms.
Option B - Achalasia: Achalasia is a condition characterized by the impaired relaxation of the lower eso
phageal sphincter and absence of peristalsis in the esophagus. It commonly presents with dysphagia t
o both solids and liquids, chest pain, and regurgitation. However, the symptom of retrosternal pain is n
ot typically associated with achalasia, so it is not the most likely diagnosis based on the given symptom
s.

Page 100

971
Option D - Reflux esophagitis: Reflux esophagitis refers to inflammation of the esophagus due to the re
flux of gastric acid from the stomach. It commonly presents with symptoms such as heartburn, regurgit
ation, and chest pain. While reflux esophagitis can cause dysphagia in some cases, the presence of ret
rosternal pain and regurgitation with coughing after meals is less typical of reflux esophagitis, making it
less likely as the primary diagnosis.

Solution for Question 75:


Correct Option A - Ulcerative colitis:
• The patient's presentation of passage of mucus and blood in the stool, along with inflammation mainly
in the rectum and the presence of crypt abscess on biopsy, is consistent with the diagnosis of ulcerative
colitis.
• Ulcerative colitis is a type of inflammatory bowel disease (IBD) characterized by chronic inflammation
of the colon and rectum. It typically presents with symptoms such as bloody diarrhea, mucus in the
stool, abdominal pain, and urgency to have bowel movements. Inflammation in ulcerative colitis is
continuous and primarily affects the mucosal layer of the colon and rectum. The presence of crypt
abscesses, as seen on biopsy, is a characteristic finding in ulcerative colitis.
The patient's presentation of passage of mucus and blood in the stool, along with inflammation mainly i
n the rectum and the presence of crypt abscess on biopsy, is consistent with the diagnosis of ulcerative
colitis.
Ulcerative colitis is a type of inflammatory bowel disease (IBD) characterized by chronic inflammation o
f the colon and rectum. It typically presents with symptoms such as bloody diarrhea, mucus in the stool
, abdominal pain, and urgency to have bowel movements. Inflammation in ulcerative colitis is continuou
s and primarily affects the mucosal layer of the colon and rectum. The presence of crypt abscesses, as
seen on biopsy, is a characteristic finding in ulcerative colitis.
Incorrect Options:
Option B - Irritable bowel syndrome: Irritable bowel syndrome (IBS) is a functional gastrointestinal disor
der characterized by symptoms such as abdominal pain, bloating, and changes in bowel habits. Howev
er, IBS does not typically cause the passage of mucus and blood in the stool, which are more suggesti
ve of an inflammatory condition like ulcerative colitis.
Option C - Crohn's disease: Crohn's disease is another type of inflammatory bowel disease that can aff
ect any part of the digestive tract from the mouth to the anus. Unlike ulcerative colitis, Crohn's disease
often involves patchy areas of inflammation that can affect multiple layers of the bowel wall. Rectal invo
lvement alone, as described in the case, is less typical of Crohn's disease, making it less likely compar
ed to ulcerative colitis.
Option D - Pseudomembranous colitis: Pseudomembranous colitis is an infection of the colon caused b
y the bacterium Clostridium difficile (C. difficile). It typically occurs as a result of antibiotic use, which di
srupts the normal gut flora and allows C. difficile to overgrow. While it can cause inflammation and pas
sage of blood in the stool, the presence of crypt abscesses on biopsy is not a
characteristic feature of pseudomembranous colitis.

Solution for Question 76:


Option C - 1, 2, 3:

Page 101

972
• In the given scenario, the surgeon initially performs an appendectomy with a grid iron incision, aka Mc
Burney's incision.
• Grid iron incision: A grid iron incision is a transverse incision made in the right lower quadrant of the
abdomen for an appendectomy. It involves cutting through the layers of the abdominal wall, including
the skin, subcutaneous tissue, and muscle layers.
• Based on these descriptions, the muscles cut during the conversion from a gridiron are: A gridiron
incision involves an arcing incision through the skin, subcutaneous fat and fascia, external and internal
obliques, transversus abdominis and transversalis fascia. It is commonly used for open
appendicectomies.
• A gridiron incision involves an arcing incision through the skin, subcutaneous fat and fascia, external
and internal obliques, transversus abdominis and transversalis fascia. It is commonly used for open
appendicectomies.
In the given scenario, the surgeon initially performs an appendectomy with a
grid iron incision, aka Mc Burney's incision.
Grid iron incision: A grid iron incision is a transverse incision made in the right lower quadrant of the ab
domen for an appendectomy. It involves cutting through the layers of the abdominal wall, including the
skin, subcutaneous tissue, and muscle layers.
Based on these descriptions, the muscles cut during the conversion from a gridiron are:
• A gridiron incision involves an arcing incision through the skin, subcutaneous fat and fascia, external
and internal obliques, transversus abdominis and transversalis fascia. It is commonly used for open
appendicectomies.
A gridiron incision involves an arcing incision through the skin, subcutaneous fat and fascia, external a
nd internal obliques, transversus abdominis and transversalis fascia. It is commonly used for open app
endicectomies.
Incorrect Options:
Option A, B & D: In a Rutherford Morrison incision, the rectus abdominis muscle is retracted but not us
ually cut. The incisions are made above and below the muscle to access the abdominal contents while
preserving the rectus abdominis muscle.

Solution for Question 77:


Correct Option B - Diffuse esophageal spasm:
• Diffuse esophageal spasm is a motility disorder of the esophagus. In barium swallow, we can observe
a “corkscrew esophagus' ' Clinically, chest pain and dysphagia, predominantly while eating spicy food,
are the symptoms. Sacculations and pseudo diverticula can also be seen.
Diffuse esophageal spasm is a
motility disorder of the esophagus. In barium swallow, we can observe a “corkscrew esophagus' ' Clinic
ally, chest pain and dysphagia, predominantly while eating spicy food, are the symptoms. Sacculations
and pseudo diverticula can also be seen.

Page 102

973
Incorrect Options:
Option A - Infectious esophagitis: In infectious esophagitis, linear, regular plaque-like lesions can be se
en on the contrast X-ray, which is not seen in the image.
Option C - Barette’s esophagus: In Barette's esophagus, X-ray does not show any specific findings.
Option D - Esophageal stricture: Barium meal shows a narrow esophagus with a
shoulder-like appearance, which is not consistent with the given image.

Solution for Question 78:


Correct Option A - Achalasia cardia:
• Achalasia cardia is a massively dilated esophagus. It has a narrow distal end. Radiologically we can
observe a bird beak sign as seen in the given image. A patient suffering from achalasia cardia has
more difficulty swallowing liquids than solids.
Achalasia cardia is a massively dilated esophagus. It has a
narrow distal end. Radiologically we can observe a bird beak sign as seen in the given image. A
patient suffering from achalasia cardia has more difficulty swallowing liquids than solids.
Incorrect Options:
Option B - Diffuse esophageal spasm: A
corkscrew sign is seen in the diffuse esophagus, but this is not seen in the image given.
Option C - Nutcracker oesophagus: The nutcracker esophagus also shows a
corkscrew sign similar to the diffuse esophageal spasm.
Option D - Hypertensive lower esophageal sphincter: It does not show a
bird beak sign, and hence, it is an incorrect choice.

Page 103

974
Solution for Question 79:
Correct Option C - Diverticulosis:
• Diverticulosis is the outpouching of the colon wall. In a barium swallow, the contrast x-ray shows an
outpouching filled with barium as seen in the image it presents as a “Saw tooth appearance” It is most
commonly seen in the sigmoid colon.
Diverticulosis is the outpouching of the colon wall. In a barium swallow, the contrast x-ray shows an out
pouching filled with barium as seen in the image it presents as a
“Saw tooth appearance” It is most commonly seen in the sigmoid colon.

Incorrect Options:
Option A - Multiple colonic polyps: Colonic polyps appear as single polypoid lesions in the radiograph. I
t does not present a Sawtooth appearance.
Option B - Ulcerative colitis: A
smooth colon with no haustrations “Lead pipe colon” is seen in the radiograph.
Option D - Irritable bowel Syndrome: Saw tooth appearance is not seen in irritable bowel syndrome.

Solution for Question 80:


Correct Option B - Zenker's diverticulum:
• Dohlman's procedure is a surgical technique used in the treatment of Zenker's diverticulum. Zenker's
diverticulum is a condition characterized by the formation of a pouch or outpouching in the lining of the
esophagus, specifically in the area called the pharyngoesophageal junction. It can cause symptoms
such as difficulty swallowing, regurgitation of undigested food, halitosis (bad breath), and chronic
cough.
• Dohlman's procedure, also known as pharyngoesophageal myotomy, is a surgical approach used to
treat Zenker's diverticulum. It involves making a small incision or myotomy in the muscular wall
between the esophagus and the diverticulum. This allows for the release of the pressure and improves
the emptying of the diverticulum, relieving the associated symptoms.

Page 104

975
Dohlman's procedure is a
surgical technique used in the treatment of Zenker's diverticulum. Zenker's diverticulum is a
condition characterized by the formation of a pouch or outpouching in the lining of the esophagus, spec
ifically in the area called the pharyngoesophageal junction. It can cause symptoms such as difficulty sw
allowing, regurgitation of undigested food, halitosis (bad breath), and chronic cough.
Dohlman's procedure, also known as pharyngoesophageal myotomy, is a
surgical approach used to treat Zenker's diverticulum. It involves making a small incision or myotomy in
the muscular wall between the esophagus and the diverticulum. This allows for the release of the pres
sure and improves the emptying of the diverticulum, relieving the associated symptoms.
Incorrect Options:
Option A - Meckel's diverticulum: Meckel's diverticulum is a congenital anomaly of the small intestine.
Option C - Bochdalek hernia: Bochdalek hernia is a type of congenital diaphragmatic hernia.
Option D - Menetrier's disease: Menetrier's disease is a
rare stomach disorder characterized by enlarged gastric folds and excessive mucus production.

Solution for Question 81:


Correct Option B - Adenocarcinoma is a rare variant of rectal carcinoma:
• Rectal carcinoma is predominantly composed of adenocarcinoma, which is the most common
histological type of rectal cancer. Adenocarcinoma arises from the glandular cells lining the inner
surface of the rectum. Therefore, adenocarcinoma is not a rare variant of rectal carcinoma; it is the
most common type.
Rectal carcinoma is predominantly composed of adenocarcinoma, which is the most common histologi
cal type of rectal cancer. Adenocarcinoma arises from the glandular cells lining the inner surface of the
rectum. Therefore, adenocarcinoma is not a
rare variant of rectal carcinoma; it is the most common type.
Incorrect Options:
Option A - Hartmann's operation is done in elderly debilitated patients: This statement is true. Hartman
n's operation is a surgical procedure performed in cases of advanced rectal cancer or emergency situat
ions. It involves the removal of the rectal tumor, closure of the rectal stump, and creation of an end colo
stomy. This operation is commonly considered in elderly, debilitated patients who may not tolerate a
more extensive procedure such as a low anterior resection with primary anastomosis.
Option C - Early morning spurious diarrhoea and tenesmus can occur: This statement is true. Rectal ca
rcinoma can cause various symptoms, including changes in bowel habits. Early morning spurious diarr
hea and tenesmus (a feeling of incomplete bowel emptying with straining) can be seen in rectal carcino
ma due to the obstructive nature of the tumor.
Option D - Growth confined to the rectal mucosa is stage A of modified Duke's staging: This statement
is true. The modified Duke's staging system for colorectal cancer categorizes stage A as carcinoma co
nfined to the mucosa (inner lining) of the rectum or colon, without invasion into deeper layers or spread
to lymph nodes or distant sites.

Solution for Question 82:

Page 105

976
Correct Option C - Acute pancreatitis:
• A gasless abdomen refers to a condition in which there is a lack of free gas or air within the abdominal
cavity. It can be seen in imaging studies, such as X-rays or CT scans.
• In the context of the given options, acute pancreatitis is the condition associated with a gasless
abdomen. Acute pancreatitis is an inflammatory condition of the pancreas that can be caused by
various factors, such as gallstones, alcohol consumption, or certain medications. In severe cases of
acute pancreatitis, the inflammation and associated fluid collections can lead to a loss of normal gas
patterns in the abdomen, resulting in a gasless appearance on imaging.
A gasless abdomen refers to a condition in which there is a lack of free gas or air within the abdominal
cavity. It can be seen in imaging studies, such as X-rays or CT scans.
In the context of the given options, acute pancreatitis is the condition associated with a gasless abdom
en. Acute pancreatitis is an inflammatory condition of the pancreas that can be caused by various facto
rs, such as gallstones, alcohol consumption, or certain medications. In severe cases of acute pancreati
tis, the inflammation and associated fluid collections can lead to a
loss of normal gas patterns in the abdomen, resulting in a gasless appearance on imaging.
Incorrect Options:
Option A - Ulcerative colitis: Ulcerative colitis is a type of inflammatory bowel disease that primarily affe
cts the colon and rectum. It typically does not cause a gasless abdomen.
Option B - Intussusception: Intussusception refers to the telescoping or invagination of one segment of
the intestine into another, often leading to bowel obstruction. While intussusception may cause abdomi
nal distention, it does not typically result in a gasless abdomen.
Option D - Crohn's disease: Crohn's disease is another type of inflammatory bowel disease that can aff
ect any part of the digestive tract. It is characterized by inflammation that can extend through the layers
of the intestinal wall. While Crohn's disease can cause abdominal symptoms and complications, it doe
s not commonly present with a gasless abdomen.

Solution for Question 83:


Correct Option A. CMV esophagitis
• When esophageal endoscopy reveals the presence of serpiginous ulcers, it is indicative of CMV
(Cytomegalovirus) esophagitis.
• CMV esophagitis is a viral infection caused by the cytomegalovirus. It is commonly seen in individuals
with weakened immune systems, such as those with HIV/AIDS, organ transplant recipients, or
individuals receiving immunosuppressive therapy.
• The characteristic finding of serpiginous ulcers refers to the appearance of shallow, linear ulcers that
follow a winding or serpentine pattern in the lining of the esophagus. These ulcers can be seen during
endoscopy and are highly suggestive of CMV esophagitis.
Incorrect Options
Option B Candida esophagitis:
• Candida esophagitis is caused by an overgrowth of Candida species, typically Candida albicans, in
the esophagus. It may present with white patches or plaques but not serpiginous ulcers.
Optioin C. Herpetic esophagitis:

Page 106

977
• Herpetic esophagitis is caused by the herpes simplex virus (HSV), specifically HSV-1. It can lead to
small, shallow ulcers, but they are not typically described as serpiginous.
Option D. Radiation esophagitis:
• Radiation esophagitis is a condition characterized by inflammation and damage to the esophageal
lining due to radiation therapy. It can present with mucosal changes, ulcers, or strictures, but
serpiginous ulcers are not a typical feature.

Solution for Question 84:


Correct Option B - Ligament of Treitz:
• Ligament of Treitz, is the correct demarcation point used to differentiate an upper gastrointestinal (GI)
bleed from a lower GI bleed.
• The Ligament of Treitz, also known as the suspensory muscle of the duodenum, is a fibrous band
located at the junction between the duodenum (the first part of the small intestine) and the jejunum (the
second part of the small intestine).
• This anatomical landmark serves as a useful reference point for distinguishing between upper and
lower GI bleeding.
Ligament of Treitz, is the correct demarcation point used to differentiate an upper gastrointestinal (GI)
bleed from a lower GI bleed.
The Ligament of Treitz, also known as the suspensory muscle of the duodenum, is a fibrous band locat
ed at the junction between the duodenum (the first part of the small intestine) and the jejunum (the sec
ond part of the small intestine).
This anatomical landmark serves as a
useful reference point for distinguishing between upper and lower GI bleeding.
Incorrect Options:
Option A - Ampulla of Vater: The Ampulla of Vater is the site where the common bile duct and the panc
reatic duct join and enter the duodenum. While it is an important anatomical structure, it is not used as
a demarcation point to differentiate upper and lower GI bleeding.
Option C - Superior duodenal flexure: The superior duodenal flexure refers to the curved segment of th
e duodenum, where it transitions from the descending part to the horizontal part. Although it is a
notable anatomical feature, it is not used as a
demarcation point to differentiate upper and lower GI bleeding.
Option D - Ileocecal junction: The ileocecal junction is the point where the small intestine (ileum) conne
cts to the large intestine (cecum). A lower GI bleed typically involves bleeding from the large intestine b
eyond this junction. However, it is not the demarcation point used to differentiate upper and lower GI bl
eeding.

Solution for Question 85:


Correct Option A - During feeding:

Page 107

978
• An abdominal mass in a patient with congenital hypertrophic pyloric stenosis (CHPS) is best seen
during feeding.
• During feeding, the increased peristaltic activity and the passage of milk through the narrowed pylorus
can cause the abdominal mass to become more prominent and easily palpable.
An abdominal mass in a
patient with congenital hypertrophic pyloric stenosis (CHPS) is best seen during feeding.
During feeding, the increased peristaltic activity and the passage of milk through the narrowed pylorus
can cause the abdominal mass to become more prominent and easily palpable.
Incorrect Options:
Option B - Soon after birth: While congenital hypertrophic pyloric stenosis typically presents in the first f
ew weeks of life, it may not be immediately apparent after birth. The condition usually becomes evident
within a
few weeks as the baby starts to feed more frequently and the symptoms become more pronounced.
Option C - During palpation, over the left hypochondrium: It is usually located in the epigastric region,
which is the upper central part of the abdomen.
Option D - During palpation, over the epigastrium: The abdominal mass in congenital hypertrophic pylo
ric stenosis is best palpated over the epigastrium, which is the upper central part of the abdomen, just
below the sternum. During palpation, the thickened pylorus can be felt as a firm, olive-shaped mass in t
his area. However, the mass is more easily appreciated during feeding when it becomes more promine
nt.

Solution for Question 86:


Correct Option B
- Symptomatic relapse after surgery is not related to the presence of disease during resection:
• In Crohn's disease, symptomatic relapse after surgery is not necessarily related to the presence of
disease at the site of resection.
• Crohn's disease is known for its chronic and relapsing nature, and recurrence of symptoms can occur
even in areas where surgical resection was performed.
• This can happen due to the development of new areas of disease involvement or ongoing
inflammation in other parts of the gastrointestinal tract.
• Therefore, relapse of symptoms after surgery does not necessarily indicate that the disease was not
adequately removed during resection.
In Crohn's disease, symptomatic relapse after surgery is not necessarily related to the presence of dise
ase at the site of resection.
Crohn's disease is known for its chronic and relapsing nature, and recurrence of symptoms can occur e
ven in areas where surgical resection was performed.
This can happen due to the development of new areas of disease involvement or ongoing inflammation
in other parts of the gastrointestinal tract.
Therefore, relapse of symptoms after surgery does not necessarily indicate that the disease was not ad
equately removed during resection.
Incorrect Options:

Page 108

979
Option A - For effective treatment of a
Crohn’s mass, only antibiotics are prescribed: Antibiotics can be a part of the treatment for Crohn's dis
ease, especially in cases where there is an infection or abscess present. However, the treatment of Cr
ohn's disease is comprehensive and typically involves a combination of medications, including anti-infla
mmatory drugs, immunosuppressants, biologic agents, and sometimes surgical intervention. The speci
fic treatment approach depends on the severity and location of the disease, as well as individual patien
t factors.
Option C - The proximal caecum is most commonly involved in Crohn’s disease: While Crohn's disease
can affect any part of the gastrointestinal tract from the mouth to the anus, the most common site of in
volvement is the terminal ileum, which is the distal portion of the small intestine.
Option D - On endoscopic examination, continuous lesions are visualized, which are diagnostic of Croh
n’s disease: Crohn's disease is characterized by a patchy pattern of inflammation rather than continuou
s lesions. The inflammation can appear as skip lesions, meaning that there are areas of healthy tissue i
n between affected areas. Endoscopic examination can reveal findings such as ulcers, aphthous erosio
ns, cobblestone appearance, and areas of narrowing or strictures, which are suggestive of Crohn's dis
ease. However, the diagnosis of Crohn's disease is not solely based on endoscopic findings but requir
es a combination of clinical, radiological, and pathological assessments.

Solution for Question 87:


Correct Option C - 2 litres of crystalloid infusion:
• The patient's presentation with abdominal pain, hypotension (BP 89/50 mmHg), tachypnea (RR
24/min), and evidence of extraluminal air on an abdominal radiograph suggests a possible diagnosis of
abdominal perforation, such as a perforated viscus. This condition requires urgent surgical intervention.
Stabilizing the patient's hemodynamic status is crucial before shifting the patient to the OT.
• In a hypotensive patient with possible abdominal perforation, fluid resuscitation is a crucial step to
improve intravascular volume and stabilize the patient's hemodynamics. Crystalloid infusion, such as
normal saline or lactated Ringer's solution, is typically initiated as the first-line fluid therapy to restore
circulating volume.
The patient's presentation with abdominal pain, hypotension (BP 89/50 mmHg), tachypnea (RR 24/min
), and evidence of extraluminal air on an abdominal radiograph suggests a
possible diagnosis of abdominal perforation, such as a perforated viscus. This condition requires urgen
t surgical intervention. Stabilizing the patient's hemodynamic status is crucial before shifting the patient
to the OT.
In a hypotensive patient with possible abdominal perforation, fluid resuscitation is a crucial step to impr
ove intravascular volume and stabilize the patient's hemodynamics. Crystalloid infusion, such as norma
l saline or lactated Ringer's solution, is typically initiated as the first-line fluid therapy to restore circulati
ng volume.
Incorrect Options:
Option A - Intubate the patient: Although securing the patient's airway is important, it is not the immedi
ate priority in this scenario. If the patient is conscious and maintaining adequate oxygenation (saturatio
n 92% with 6 L of oxygen), intubation may not be necessary at this stage. However, the patient should
be closely monitored, and if there are signs of respiratory distress or worsening oxygenation, intubation
may be required.
Option B - Insert central venous catheter as initial IV access of choice: While establishing IV access is i
mportant, inserting a central venous catheter is not the immediate priority in this critical situation. Perip

Page 109

980
heral IV access can be rapidly obtained for initial resuscitation and administration of fluids and medicati
ons.
Option D - Confirm the diagnosis with CECT: While computed tomography (CT) scan (CECT) may be u
seful for confirming the diagnosis and assessing the extent of the abdominal pathology, it is not the im
mediate priority in a critically ill patient who requires urgent surgical intervention. CT scan may delay de
finitive treatment, and in this scenario, the patient's unstable condition warrants immediate resuscitatio
n and surgical evaluation.

Solution for Question 88:


Correct Option B - Aphthous ulcers:
• Aphthous ulcers, also known as aphthous stomatitis, are shallow, round or oval ulcers.
• They are considered one of the earliest signs of Crohn's disease and can precede the development of
other intestinal symptoms.
• Aphthous ulcers may be recurrent and are often painful.
Aphthous ulcers, also known as aphthous stomatitis, are shallow, round or oval ulcers.
They are considered one of the earliest signs of Crohn's disease and can precede the development of
other intestinal symptoms.
Aphthous ulcers may be recurrent and are often painful.
Incorrect Options:
Option A - Cobblestone appearance: The cobblestone appearance refers to the characteristic appeara
nce of the mucosa in Crohn's disease, where there are areas of normal or slightly erythematous mucos
a interspersed with raised, edematous, and ulcerated patches. While this is a common finding in Crohn'
s disease, it is not typically the earliest sign. It often develops as the disease progresses and affects a
larger area of the gastrointestinal tract.
Option B - Deep fissured ulcers: Deep fissured ulcers are a common feature of Crohn's disease, partic
ularly in the colon and rectum. However, similar to the cobblestone appearance, they tend to develop a
s the disease progresses rather than being the earliest sign.
Option C - Rose-thorn appearance: The rose-thorn appearance is associated with Crohn's disease. It i
s not an early sign of Crohn's disease.

Solution for Question 89:


Correct Option C - Mass is mobile perpendicular to mesentery attachment line:
• The Tillaux sign is a clinical finding associated with mesentric cyst.
• The swelling generally will be moving: Perpendicular to the attachment of mesentery Along the
attachment of mesentery.
• Perpendicular to the attachment of mesentery
• Along the attachment of mesentery.

Page 110

981
• Hence, the mobility of the cyst will be perpendicular to the attachment of mesentery. This is known as
the Tillaux sign.
The Tillaux sign is a clinical finding associated with mesentric cyst.
The swelling generally will be moving:
• Perpendicular to the attachment of mesentery
• Along the attachment of mesentery.
Perpendicular to the attachment of mesentery
Along the attachment of mesentery.
Hence, the mobility of the cyst will be perpendicular to the attachment of mesentery. This is known as t
he Tillaux sign.
Incorrect Options:
Option A - Mass is felt in the periumbilical region: This statement does not describe the Tillaux sign. Th
e periumbilical region is associated with another clinical finding called the Dance sign, which is a
manifestation of intussusception. It refers to the palpable mass in the periumbilical area.
Option B - Mass is brilliantly transilluminant: This statement does not apply to the Tillaux sign. Transillu
mination is a technique used to assess the presence of fluid or air within a
mass or body cavity. However, it is not a characteristic feature of the Tillaux sign.
Option D - Fluctuation is positive: This statement does not describe the Tillaux sign. Fluctuation refers t
o the palpable fluid wave within a
mass, typically seen in fluid-filled structures such as cysts. It is not specific to the Tillaux sign.

Solution for Question 90:


Correct Option A - B, C and D:
• Congenital hypertrophic pyloric stenosis (CHPS) is a condition characterized by the thickening and
narrowing of the pylorus, which leads to obstruction of the passage of food from the stomach to the
small intestine. It typically presents in infants between 2 and 8 weeks of age.
• (B) Visible peristalsis from left to right: This is a characteristic finding in infants with congenital
hypertrophic pyloric stenosis. Due to the obstruction at the pyloric region, the peristaltic waves of the
stomach can be visualized as they attempt to push the food through the narrowed pylorus.
• (C) Hypochloremia and hypokalemia: Infants with congenital hypertrophic pyloric stenosis often
experience prolonged vomiting, which leads to the loss of hydrochloric acid (HCl) and potassium (K+)
from the stomach. This can result in hypochloremic metabolic alkalosis, characterized by low chloride
(Cl-) and potassium levels in the blood.
• (D) Metabolic alkalosis: Prolonged vomiting and loss of stomach acid in congenital hypertrophic
pyloric stenosis can cause metabolic alkalosis, a condition characterized by an elevation in blood pH
and bicarbonate levels.
• (A) Bilious vomiting: Bilious vomiting is not typically seen in congenital hypertrophic pyloric stenosis.
Bilious vomiting refers to the vomiting of greenish-yellow fluid (bile) and is more commonly associated
with intestinal obstruction or other conditions affecting the small intestine. In congenital hypertrophic
pyloric stenosis, there is brown color vomiting
Incorrect Options:

Page 111

982
Option B - A and D only: is incorrect as bilious vomiting is not a
characteristic feature of congenital hypertrophic pyloric stenosis.
Option C - A, B and D: is incorrect as it does not include visible peristalsis from right to left, which is a
specific finding in this condition.
Option D - A, B, C and D: is incorrect as it includes all the listed features, which is not accurate. Bilious
vomiting is not associated with congenital hypertrophic pyloric stenosis.

Solution for Question 91:


Correct Option A - Achalasia cardia:
• Heller's operation, also known as Heller myotomy, is a surgical procedure performed to treat
achalasia cardia.
• Achalasia is a disorder of the esophagus characterized by the inability of the lower esophageal
sphincter (LES) to relax and allow food to pass into the stomach.
• Heller's operation involves cutting the muscles of the LES to relieve the obstruction and improve the
movement of food through the esophagus.
Heller's operation, also known as Heller myotomy, is a
surgical procedure performed to treat achalasia cardia.
Achalasia is a disorder of the esophagus characterized by the inability of the lower esophageal sphinct
er (LES) to relax and allow food to pass into the stomach.
Heller's operation involves cutting the muscles of the LES to relieve the obstruction and improve the m
ovement of food through the esophagus.
Incorrect Options:
Option B - Carcinoma esophagus: Heller's operation is not performed for carcinoma esophagus. In cas
es of esophageal cancer, the treatment approach typically involves a combination of chemotherapy, ra
diation therapy, and surgical resection, depending on the stage and extent of the disease.
Option C - Esophageal stricture: Heller's operation is not commonly performed for esophageal stricture
s. Esophageal strictures are often caused by chronic gastroesophageal reflux disease (GERD) or dam
age to the esophageal lining. The treatment of esophageal strictures usually involves nonsurgical meth
ods such as dilation (stretching) of the stricture using an endoscope or the placement of stents to wide
n the narrowed area.
Option D - Hypertrophic pyloric stenosis: Heller's operation is not indicated for hypertrophic pyloric sten
osis. Hypertrophic pyloric stenosis is a condition that affects infants and involves the thickening and nar
rowing of the pylorus (the opening between the stomach and the small intestine). The treatment for hyp
ertrophic pyloric stenosis is surgical pyloromyotomy, which involves making a
small incision in the pylorus to relieve the obstruction.

Solution for Question 92:


Correct Option B - Mucus-secreting foveolar cells are characteristic:

Page 112

983
• Barrett's esophagus is characterized by the presence of specialized columnar cells with goblet cells,
not mucus-secreting foveolar cells.
• Goblet cells are responsible for producing mucus, and their presence in the columnar epithelium is a
hallmark of Barrett's esophagus.
Barrett's esophagus is characterized by the presence of specialized columnar cells with goblet cells, no
t mucus-secreting foveolar cells.
Goblet cells are responsible for producing mucus, and their presence in the columnar epithelium is a
hallmark of Barrett's esophagus.
Incorrect Options:
Option A - It is the intestinal metaplasia of the esophagus: Barrett's esophagus refers to the condition
where the normal squamous epithelium of the lower esophagus undergoes metaplastic changes and tr
ansforms into columnar epithelium with intestinal characteristics. This change is known as intestinal me
taplasia.
Option C - Alcian blue is used in the staining of biopsy specimens: Alcian blue is a commonly used stai
n in histopathology to detect the presence of mucin-containing cells. In the case of Barrett's esophagus
, Alcian blue staining can be used to identify the presence of goblet cells in the columnar epithelium dur
ing histological examination of biopsy specimens.
Option D - Adenocarcinoma of O-G junction is a complication: This statement is correct

Solution for Question 93:


Correct Option A - Familial adenomatous polyposis:
• Familial adenomatous polyposis is a genetic disorder inherited as a Mendelian dominant. It presents
multiple polyps ( more than 100) of colorectal adenomas, as seen in the image. It has a 100% risk of
malignancy. In early adulthood, symptoms such as blood and mucus in the stool, loose stools, crampy
lower abdominal pain, anemia, weight loss, and protein malnutrition can be seen.
Familial adenomatous polyposis is a genetic disorder inherited as a
Mendelian dominant. It presents multiple polyps (
more than 100) of colorectal adenomas, as seen in the image. It has a 100% risk of malignancy. In earl
y adulthood, symptoms such as blood and mucus in the stool, loose stools, crampy lower abdominal p
ain, anemia, weight loss, and protein malnutrition can be seen.

Page 113

984
Incorrect Options:
Option B - Colon cancer: In the case of colon cancer, it is usually more common in old age and is not s
een in a patient aged 14, as seen in this case. Colon cancer is associated with colon obstruction, tumor
s, bleeding, pain in the abdomen, and many other symptoms.
Option C - Juvenile polyp: It is characterised by a
single polyp, not multiple polyps, as seen in the given image.
Option D - Ulcerative colitis: This does not include multiple polyps. Pinpoint ulcers, Pseudopolyposis, a
nd Pipe stem colon are observed, which are not seen in this case.

Solution for Question 94:


Correct Option C - Full colonoscopy and biopsy:
• The next step in the management of the patient is a full colonoscopy and biopsy. A full colonoscopy is
done to confirm the diagnosis of rectal carcinoma and to rule out the presence of any synchronous
lesions. If any lesions are identified a biopsy should be done to confirm the diagnosis and determine the
stage and the characteristics of the tumor. Hence it is necessary to get a full colonoscopy and biopsy in
the case of the discovery of a rectal mass.
The next step in the management of the patient is a full colonoscopy and biopsy. A full colonoscopy is
done to confirm the diagnosis of rectal carcinoma and to rule out the presence of any synchronous lesi
ons. If any lesions are identified a biopsy should be done to confirm the diagnosis and determine the st
age and the characteristics of the tumor. Hence it is necessary to get a
full colonoscopy and biopsy in the case of the discovery of a rectal mass.
Incorrect Options:
Option A - CT scan of the abdomen: CT scan of the abdomen is useful in determining the size, location
and staging of the tumour. It can also be used to detect any secondary metastatic lesion but it is not th
e primary line of management for the rectal carcinoma.
Option B - Rigid sigmoidoscopy and biopsy: This procedure can only be used to examine the distal port
ion of the rectum and sigmoid colon but not the entire colon. This may lead to compromise in the compl

Page 114

985
ete thorough screening of the colon.
Option D - Barium enema: This technique was used earlier but now is vastly replaced by colonoscopy
as it is a better diagnostic procedure.

Solution for Question 95:


Correct Option A - Ileocaecal TB:
• In the barium study of the ileum in the case of ileocaecal TB these are the signs that can be seen
narrowing of the terminal ileum. Thickening of the ileocecal valve, and the thickening of the cecal walls,
the cecum appears as a conical shape. In later stages, the ileum is fixed and rigid. These findings
correlate with the above image.
In the barium study of the ileum in the case of ileocaecal TB these are the signs that can be seen narro
wing of the terminal ileum. Thickening of the ileocecal valve, and the thickening of the cecal walls, the c
ecum appears as a conical shape. In later stages, the ileum is fixed and rigid. These findings correlate
with the above image.
Incorrect Options:
Option B - Ulcerative colitis: In ulcerative colitis radiologically we see a contracted pipe stem colon and
thickened edematous haustrae. There is usually no weight loss and low-grade fever associated with it.
Hence this is an incorrect choice.

Option C - Crohn’s disease: In Crohn’s disease radiologically we see a


cobblestone appearance and not a
conical ileum as shown in the image. Hence it is the incorrect choice.
Option D - Carcinoma caecum: In the case of carcinoma of the cecum you can see apple core deformit
y on the right side in the right iliac fossa and not at the terminal ileum.

Page 115

986
Solution for Question 96:
Correct Option A
- Size appears to be the best predictor of malignant behavior and metastatic potential:
• In the case of carcinoid tumors of the appendix, the size alone is not the best predictor of malignant
behavior and metastatic potential. Other factors, such as lymphovascular invasion, tumor grade, and
distant metastasis, are considered more reliable indicators of aggressive behavior.
In the case of carcinoid tumors of the appendix, the size alone is not the best predictor of malignant be
havior and metastatic potential. Other factors, such as lymphovascular invasion, tumor grade, and dist
ant metastasis, are considered more reliable indicators of aggressive behavior.
Incorrect Options:
Option B - Arise from Kulchitsky cells of the crypts of Leiberkuhn: Carcinoid tumors of the appendix aris
e from neuroendocrine cells, also known as Kulchitsky cells, which are present in the crypts of Lieberk
uhn. These cells are responsible for the production of various hormones and neurotransmitters.
Option C - Most common location is the base of the appendix: This statement is true. Carcinoid tumors
of the appendix are most commonly located at the base of the appendix. They can also be found in the
body and tip of the appendix, but the base is the most frequent site.
Option D - Most common tumor of the appendix: This statement is true. Carcinoid tumors are the most
common tumors of the appendix, accounting for the majority of all appendix neoplasms. They are typic
ally low-grade malignant tumors with a favorable prognosis, mainly when they are confined to the appe
ndix without lymph node involvement or distant metastasis.

Solution for Question 97:


Correct Option B - Lanz incision:
• The Lanz incision is another type of incision used for open appendectomy. It is a transverse or oblique
incision made in the right lower quadrant of the abdomen, just above the inguinal ligament. The incision
is placed parallel to and just above the line, joining the anterior superior iliac spine (ASIS) and the
umbilicus. The Lanz incision provides good exposure of the appendix and is commonly used when
there is suspicion of perforation or abscess formation.
The Lanz incision is another type of incision used for open appendectomy. It is a transverse or oblique
incision made in the right lower quadrant of the abdomen, just above the inguinal ligament. The incisio
n is placed parallel to and just above the line, joining the anterior superior iliac spine (ASIS) and the um
bilicus. The Lanz incision provides good exposure of the appendix and is commonly used when there i
s suspicion of perforation or abscess formation.
Incorrect Options:
Option A - Gridiron Incision: "A" is The Gridiron incision, also known as the McBurney incision, is a co
mmon incision used for open appendectomy. It is named after the anatomical landmark, the McBurney'
s point, which lies on the right lower quadrant of the abdomen. The incision is typically made along the
right semilunar line, starting at McBurney's point and extending towards the umbilicus. This incision pro
vides access to the appendix and allows for its removal.
Option C - Rutherford-Morison Incision: "C" is The Rutherford-Morison incision, also known as the mus
cle-splitting or muscle-cutting incision, is a variation of the Lanz incision. It involves splitting the externa
l oblique muscle along the direction of its fibers to access the appendix. This incision provides good ex
posure and minimizes muscle trauma. It is often used when there is suspicion of complicated appendici

Page 116

987
tis or the need for exploration of the surrounding structures.
Option D - Rockey-Davis Incision: The Rockey-Davis incision is a
vertical midline incision used for open appendectomy. It involves a
vertical cut made from the umbilicus to the pubic symphysis. This incision provides a
wide exposure of the abdominal cavity and is useful in cases where there is suspicion of a
complex or retrocecal appendix. It allows for exploration of the entire abdomen if necessary.

Solution for Question 98:


Correct Option A - Acute Anal fissures:
• In Acute Anal fissures the diagnosis and treatment of acute anal fissures typically do not require
specific instruments. However, a digital rectal examination (DRE) may be performed using a gloved
finger to assess the anal area and confirm the presence of a fissure. This examination helps to identify
the location and extent of the fissure.
In Acute Anal fissures the diagnosis and treatment of acute anal fissures typically do not require specifi
c instruments. However, a digital rectal examination (DRE) may be performed using a
gloved finger to assess the anal area and confirm the presence of a
fissure. This examination helps to identify the location and extent of the fissure.
Incorrect Options:
Option B - Carcinoma Rectum:
Diagnosing and treating rectal carcinoma often involves various instruments for evaluation, staging, an
d surgical intervention. Some commonly used instruments include:
• Proctoscope or Sigmoidoscope
• Endorectal Ultrasound (ERUS)
• Colonoscope
Proctoscope or Sigmoidoscope
Endorectal Ultrasound (ERUS)
Colonoscope
Option C - Hemorrhoids:
The diagnosis and treatment of hemorrhoids may involve the use of specialized instruments, including:
• Proctoscope or Anoscope
• Sclerotherapy or Banding Instruments
Proctoscope or Anoscope
Sclerotherapy or Banding Instruments
Option D - Rectal Polyp:
The diagnosis and treatment of rectal polyps may involve the use of the following instruments:
• Sigmoidoscope or Colonoscope
• Polypectomy Instruments
Sigmoidoscope or Colonoscope

Page 117

988
Polypectomy Instruments

Solution for Question 99:


Correct Option A - Gastrinoma triangle:
• The Gastrinoma triangle, also known as the Triangle of Kocher, is an anatomical landmark in the
upper abdomen that helps identify the location of a gastrinoma. A gastrinoma is a neuroendocrine
tumor that secretes excessive amounts of gastrin, leading to a condition called Zollinger-Ellison
syndrome.
• The gastrinoma triangle is defined as the confluence of the cystic and common bile duct superiorly,
the second and third portions of the duodenum inferiorly, and the neck and body of the pancreas
medially, both dorsally and ventrally
• The presence of a gastrinoma within this triangle can cause hypersecretion of gastric acid, leading to
symptoms such as recurrent peptic ulcers and diarrhea.
The Gastrinoma triangle, also known as the Triangle of Kocher, is an anatomical landmark in the upper
abdomen that helps identify the location of a gastrinoma. A gastrinoma is a
neuroendocrine tumor that secretes excessive amounts of gastrin, leading to a
condition called Zollinger-Ellison syndrome.
The gastrinoma triangle is defined as the confluence of the cystic and common bile duct superiorly, the
second and third portions of the duodenum inferiorly, and the neck and body of the pancreas medially,
both dorsally and ventrally
The presence of a gastrinoma within this triangle can cause hypersecretion of gastric acid, leading to s
ymptoms such as recurrent peptic ulcers and diarrhea.
Incorrect Options:
Option B - Calot's triangle: Calot's triangle is an anatomical triangle in the upper abdomen that is releva
nt to gallbladder surgery. It is formed by the cystic duct, common hepatic duct, and the inferior border o
f the liver. This triangle is important for identifying and protecting the cystic duct and the cystic artery du
ring cholecystectomy.
Option C - Doom's triangle: The dangerous triangle (the triangle of doom) refers to a triangular area bo
und by the vas deferens, the testicular vessels and the peritoneal fold. Within the boundaries of this are
a, the external iliac artery and vein are also present.
Option D - Cholecystohepatic triangle: It is another name for the calot’s triangle and it is an incorrect op
tion as also described above.

Solution for Question 100:


Correct Options C - Monitor vitals:
• The next line of management for a patient who underwent Nissen's fundoplication and is currently
asymptomatic with stable vitals would be to monitor vitals.
• Nissen's fundoplication is a surgical procedure performed to treat gastroesophageal reflux disease
(GERD) by reinforcing the lower esophageal sphincter (LES) to prevent the backward flow of stomach
acid into the esophagus. After the surgery, it is important to monitor the patient's vital signs to ensure

Page 118

989
stability and assess for any signs of complications.
The next line of management for a patient who underwent Nissen's fundoplication and is currently asy
mptomatic with stable vitals would be to monitor vitals.
Nissen's fundoplication is a surgical procedure performed to treat gastroesophageal reflux disease (GE
RD) by reinforcing the lower esophageal sphincter (LES) to prevent the backward flow of stomach acid
into the esophagus. After the surgery, it is important to monitor the patient's vital signs to ensure stabilit
y and assess for any signs of complications.
Incorrect Options:
Option A - Nasogastric tube to prevent gas bloat syndrome: Gas bloat syndrome refers to excessive ga
s accumulation in the stomach after Nissen's fundoplication. It can cause discomfort and bloating. How
ever, if the patient is asymptomatic, there is no need for nasogastric tube placement.
Option B - Insert chest tube to prevent pneumothorax: Pneumothorax (collapsed lung) is a potential co
mplication of any thoracic surgery. However, in an asymptomatic patient with stable vitals, there is no i
ndication for inserting a chest tube to prevent pneumothorax.
Option D - Imaging to rule out perforation: Perforation is a rare but serious complication of Nissen's fun
doplication. However, in the absence of symptoms or clinical signs suggesting perforation, routine imag
ing is not indicated.

Solution for Question 101:


Correct Option A - 1:
• From the image: Zone 1: Central Zone 2: Lateral Zone 3: Pelvic There is also zone 4 called Portal or
retro hepatic region.
• Zone 1: Central
• Zone 2: Lateral
• Zone 3: Pelvic
• There is also zone 4 called Portal or retro hepatic region.
• Zone 1 contains the IVC, Aorta, and proximal branches; exploration is necessary if there's a
hematoma.
• Hematoma in Zone 2 can be managed with angioembolization due to its relation to the kidney.
• Bleeding control is challenging in Zone 3.
• Zone 1 is the correct answer out of the three zones mentioned.
From the image:
• Zone 1: Central
• Zone 2: Lateral
• Zone 3: Pelvic
• There is also zone 4 called Portal or retro hepatic region.
Zone 1: Central
Zone 2: Lateral

Page 119

990
Zone 3: Pelvic
There is also zone 4 called Portal or retro hepatic region.
Zone 1 contains the IVC, Aorta, and proximal branches; exploration is necessary if there's a
hematoma.
Hematoma in Zone 2 can be managed with angioembolization due to its relation to the kidney.
Bleeding control is challenging in Zone 3.
Zone 1 is the correct answer out of the three zones mentioned.

Solution for Question 102:


Correct Option C - Mallory-Weiss tear:
• Mallory-Weiss tears are longitudinal mucosal lacerations that occur at the gastroesophageal junction,
usually due to severe vomiting or retching. Binge alcohol drinking can lead to forceful vomiting, which
may cause these tears. Mallory-Weiss tears are a common cause of upper gastrointestinal bleeding in
individuals with a history of alcohol abuse, and they can present with hematemesis. This option is the
most likely diagnosis based on the given history.
Mallory-Weiss tears are longitudinal mucosal lacerations that occur at the gastroesophageal junction, u
sually due to severe vomiting or retching. Binge alcohol drinking can lead to forceful vomiting, which m
ay cause these tears. Mallory-Weiss tears are a
common cause of upper gastrointestinal bleeding in individuals with a history of alcohol abuse, and the
y can present with hematemesis. This option is the most likely diagnosis based on the given history.
Incorrect Options:
Option A - Esophageal varices: Esophageal varices are dilated veins in the lower part of the esophagu
s that occur due to portal hypertension. They are often seen in patients with chronic liver disease. Whil
e alcohol abuse can contribute to liver disease, the absence of liver function abnormalities makes esop
hageal varices less likely.
Option B - Gastric ulcer: Gastric ulcers are open sores that develop on the stomach lining. They can ca
use upper gastrointestinal bleeding, including hematemesis. Although alcohol consumption can irritate
the stomach lining, gastric ulcers are less commonly associated with binge drinking alone. Risk factors
like Helicobacter pylori infection or non-steroidal anti-inflammatory drug (NSAID) use are usually more
significant in developing gastric ulcers.
Option D - Alcoholic gastritis: Alcohol abuse can cause inflammation of the stomach lining, known as al
coholic gastritis. However, in the absence of any liver function abnormalities and considering the prese
ntation with hematemesis, Mallory-Weiss tear is a
more probable diagnosis compared to alcoholic gastritis.

Solution for Question 103:


Correct Option A - Type I: Polypoid tumor with a small ulcerated component.:
• Borrman's classification is a pathologic classification system based on the macroscopic appearance
of gastric carcinomas.

Page 120

991
• Type I tumors are characterized by a diffuse polypoidal growth pattern.
• These tumors typically present as raised or elevated masses within the gastric lumen.
• Although they primarily exhibit a polypoid growth pattern, Type I tumors may also have a small
ulcerated component.
• This combination of a predominantly polypoid growth with a small ulcerated area distinguishes Type I
gastric carcinomas according to Borrman's classification.
Borrman's classification is a
pathologic classification system based on the macroscopic appearance of gastric carcinomas.
Type I tumors are characterized by a diffuse polypoidal growth pattern.
These tumors typically present as raised or elevated masses within the gastric lumen.
Although they primarily exhibit a polypoid growth pattern, Type I tumors may also have a
small ulcerated component.
This combination of a predominantly polypoid growth with a small ulcerated area distinguishes Type I
gastric carcinomas according to Borrman's classification.
Incorrect Options:
Option B - Type II: Ulcerated tumor with infiltrative margins: Type II tumors in Borrman's classification a
re described as ulcerated lesions with raised or elevated borders, without mention of infiltrative margins
.
Option C - Type III: Diffusely infiltrative tumor with linitis plastica appearance: Type III tumors are chara
cterized by ulcerated lesions that infiltrate the gastric wall. Linitis plastica appearance, which involves d
iffuse thickening and rigidity of the gastric wall, is typically associated with Type IV tumors, not Type III.
Option D - Type IV: Mucinous carcinoma with extensive peritoneal metastases: Type IV tumors, accord
ing to Borrman's classification, are identified by linitis plastica appearance, which refers to a diffuse thic
kening and rigidity of the gastric wall. There is no mention of mucinous histology or peritoneal metastas
es in the description of Type IV tumors.

Solution for Question 104:


Correct Option B - Small bowel obstruction:
• Small bowel obstruction is the most likely cause in this case. The patient's acute symptoms,
abdominal distension, high-pitched bowel sounds, and dilated loops of small bowel on X-ray are
consistent with a mechanical obstruction in the small bowel.
Small bowel obstruction is the most likely cause in this case. The patient's acute symptoms, abdominal
distension, high-pitched bowel sounds, and dilated loops of small bowel on X-ray are consistent with a
mechanical obstruction in the small bowel.
Incorrect Options:
Option A - Colonic malignancy: Colonic malignancy can lead to obstruction, but it typically causes a gra
dual onset of symptoms, such as changes in bowel habits, blood in stool, and weight loss. In this case,
the patient has acute symptoms and dilated small bowel loops, suggesting a
more proximal obstruction.
Option C - Acute pancreatitis: Acute pancreatitis can cause abdominal pain and vomiting, but it does n
ot typically lead to distended loops of small bowel on X-ray. The presentation of acute pancreatitis ofte

Page 121

992
n includes epigastric pain radiating to the back, elevated serum amylase or lipase levels, and may be a
ssociated with gallstones or alcohol consumption.
Option D - Gastric ulcer: Gastric ulcers can cause abdominal pain, but they are less likely to cause vo
miting and distended loops of small bowel. Gastric ulcers are typically associated with symptoms such
as epigastric pain, heartburn, and may be exacerbated by eating.

Solution for Question 105:


Correct Option A - Multiple flat polyp about 100:
• Among the options you provided, the finding of "Multiple flat polyps about 100" is concerning for a
higher risk of malignancy. This is because having a large number of flat polyps (approximately 100)
increases the likelihood of some of these polyps being dysplastic or even cancerous.
Among the options you provided, the finding of "Multiple flat polyps about 100" is concerning for a
higher risk of malignancy. This is because having a large number of flat polyps (approximately 100) inc
reases the likelihood of some of these polyps being dysplastic or even cancerous.
Incorrect Options:
Option B - Single pedunculated polyp: A pedunculated polyp is a
polyp that is attached to the intestinal wall by a stalk or a
narrow base. While any polyp has the potential to be cancerous, a single pedunculated polyp, especiall
y if small and without concerning features, is typically considered low risk for malignancy. However, it is
important to evaluate the histology of the polyp after removal to confirm its benign nature.
Option C - Multiple pedunculated polyps: The presence of multiple pedunculated polyps also raises the
possibility of an underlying genetic condition called familial adenomatous polyposis (FAP). FAP is ass
ociated with a higher risk of developing colorectal cancer. Therefore, in the context of multiple peduncu
lated polyps, further evaluation and genetic testing may be warranted to assess the risk of malignancy.
Option D - Solitary flat polyp: A solitary flat polyp, also known as a sessile polyp, is a
polyp that lies flat against the intestinal wall without a stalk. While flat polyps have a higher potential for
malignancy compared to pedunculated polyps, the risk still depends on other factors such as size, mor
phology, and histological characteristics. The malignancy risk of a
solitary flat polyp would need to be assessed based on these factors.

Solution for Question 106:


Correct Option A - Appendicitis:
• Based on the symptoms described, including fever, pain in the right lower abdomen, a painful lump in
the right iliac fossa, and an increased leukocyte count, the most probable diagnosis would be
appendicitis.
• Appendicitis is the inflammation of the appendix, a small pouch-like organ located in the lower right
abdomen. The classic symptoms include fever, abdominal pain (particularly in the right lower quadrant),
and an elevated white blood cell count (leukocytosis). The presence of a painful lump in the right iliac
fossa further supports the likelihood of appendicitis.
Based on the symptoms described, including fever, pain in the right lower abdomen, a painful lump in t
he right iliac fossa, and an increased leukocyte count, the most probable diagnosis would be appendici

Page 122

993
tis.
Appendicitis is the inflammation of the appendix, a small pouch-like organ located in the lower right abd
omen. The classic symptoms include fever, abdominal pain (particularly in the right lower quadrant), an
d an elevated white blood cell count (leukocytosis). The presence of a
painful lump in the right iliac fossa further supports the likelihood of appendicitis.
Incorrect Options:
Option B, C and D - Hydronephrosis, Tuberculosis & Carcinoma colon:
• While other conditions such as hydronephrosis, tuberculosis, and colon carcinoma can cause
abdominal pain and increased leukocyte count, the combination of symptoms you provided is more
consistent with appendicitis.
While other conditions such as hydronephrosis, tuberculosis, and colon carcinoma can cause abdomin
al pain and increased leukocyte count, the combination of symptoms you provided is more consistent
with appendicitis.

Solution for Question 107:


Correct Option A - Head, neck, and body of the pancreas:
• They are contained within the anterior pararenal space
They are contained within the anterior pararenal space
Incorrect Options:
Option B - Adrenal gland, kidney, ureters: These structures are contained within the perirenal space, n
ot the anterior pararenal space. The perirenal space is adjacent to the kidneys and contains the adrena
l glands, kidneys, ureters, and renal vessels.
Option C - Aorta, inferior vena cava: These major blood vessels are not contained within the anterior p
ararenal space. They are located within the fourth space mentioned in the paragraph, known as the gre
at vessel space, which lies anterior to the vertebral bodies and psoas muscles.
Option D - Ascending and descending colon: The ascending and descending colon are also not locate
d within the anterior pararenal space. They are part of the abdominal digestive tract and are not specifi
cally mentioned as being within the anterior pararenal space.

Solution for Question 108:


Correct Option C - Iliopubic tract:
The Triangle of Doom is an anatomical region in the inguinal area where sharp dissection should be av
oided due to the risk of injuring vital structures. Option C is not a boundary of the Triangle of Doom:
Medially - Vas Deferens: The Vas Deferens, also known as the ductus deferens, is a
muscular tube that carries sperm from the epididymis to the ejaculatory duct.
Laterally - Gonadal vessels: Gonadal vessels refer to the blood vessels (artery and vein) that supply th
e testes or ovaries. Injury to these vessels can lead to significant bleeding.

Page 123

994
Base - Fold of peritoneum: This fold of peritoneum creates the base of the Triangle of Doom. It is cruci
al because it serves as a
landmark for surgeons to avoid during dissection to prevent injury to underlying structures.
Apex - Deep ring: The deep inguinal ring, also known as the internal inguinal ring, is a natural opening i
n the transversalis fascia through which structures like the spermatic cord (in males) pass through.

Incorrect Options:
Option A - Vas deferens: This is correctly mentioned as one of the boundaries of the Triangle of Doom.
Injury to the Vas Deferens can affect sperm transport and fertility.
Option B - Peritoneal Fold: This is also correctly mentioned as the base of the Triangle of Doom. Injury
to this fold can result in complications related to the abdominal cavity.
Option D - Spermatic vessels: These are correctly referred to as part of the lateral boundary of the Tria
ngle of Doom. Injury to the spermatic vessels can lead to significant bleeding and other complications.

Solution for Question 109:


Correct Option C - 5.5 cm:
• This is the commonly accepted size threshold for elective surgical repair of an AAA. Once an
aneurysm reaches or exceeds this size, the risk of rupture significantly increases, and the potential
benefits of surgical intervention to prevent rupture outweigh the risks associated with the procedure.
This is the commonly accepted size threshold for elective surgical repair of an AAA. Once an aneurys
m reaches or exceeds this size, the risk of rupture significantly increases, and the potential benefits of
surgical intervention to prevent rupture outweigh the risks associated with the procedure.
Incorrect Options:
Option A - 5 cm: This is below the generally accepted size threshold for surgical intervention. AAA mea
suring less than 5
cm in diameter are often monitored through regular imaging studies to detect any significant growth.

Page 124

995
Option B - 6 cm: While an AAA measuring 6 cm in diameter is larger and carries a
higher risk of rupture, the threshold for elective repair is typically set at 5.5 cm.
Option D - 6.5 cm: An AAA measuring 6.5 cm is considered a large aneurysm that poses a
high risk of rupture.

Solution for Question 110:


Correct Option A: Surgery
• Aortic dissection is a potentially life-threatening condition characterized by a tear in the inner layer of
the aorta, leading to the formation of a false lumen and separation of the arterial wall layers. Stanford A
refers to a type of aortic dissection involving the ascending aorta.
• The initial management of Stanford A aortic dissection aims to reduce the shear forces acting on the
aortic wall, lower blood pressure, and decrease the risk of propagation and rupture. Here's an
explanation of the options:

Incorrect Options:
Option B. LMWH (Low Molecular Weight Heparin): LMWH is not the initial management for Stanford A
aortic dissection. Anticoagulation therapy is generally not recommended in the acute setting of aortic di
ssection due to the risk of increased bleeding complications.
Option C. Beta blockers : This option is incorrect, used in type B aortic dissection
Option D. Vitamin K inhibitors: Vitamin K
inhibitors, such as warfarin, are not indicated in the initial management of Stanford A aortic dissection.
They are commonly used for anticoagulation in conditions like atrial fibrillation or venous thromboembol
ism but are not the primary treatment for aortic dissection.

Page 125

996
Solution for Question 111:
Correct Option A - Absent kidney:
• Although renal anomalies can occur in Heterotaxy polysplenia syndrome, the absence of a kidney
(unilateral renal agenesis) is not typically associated with this condition.
Although renal anomalies can occur in Heterotaxy polysplenia syndrome, the absence of a
kidney (unilateral renal agenesis) is not typically associated with this condition.
Incorrect Options:
Option B - Hepatic artery anomalies: Anomalies or variations in the hepatic artery can be seen in Heter
otaxy polysplenia syndrome, as the condition affects the normal development and arrangement of abd
ominal organs.
Option C - Malrotation of the gut: Malrotation of the gut, where the intestines are not properly positione
d during development, is a common finding in Heterotaxy polysplenia syndrome. It can lead to complic
ations such as volvulus or intestinal obstruction.
Option D - Polysplenia: Polysplenia is commonly associated with the condition depicted in the image. T
he image shows multiple splenic arteries, indicating the presence of multiple spleens or splenic anomal
ies.

Solution for Question 112:


Correct Option C - Sitting with neck flexion:
• This is the correct position for the insertion of a Ryle's tube in conscious patients. The patient sits
upright with the neck slightly bent forward. This position aligns the airway and esophagus, facilitating
the passage of the tube into the stomach. It also reduces the risk of accidental tracheal intubation and
aspiration.
This is the correct position for the insertion of a Ryle's tube in conscious patients. The patient sits uprig
ht with the neck slightly bent forward. This position aligns the airway and esophagus, facilitating the pa
ssage of the tube into the stomach. It also reduces the risk of accidental tracheal intubation and aspirat
ion.
Incorrect Options:
Option A - Supine with neck flexion: In this position, the patient lies on their back with the neck bent for
ward. This position is not suitable for the insertion of a
Ryle's tube in conscious patients; it is used in comatose patients.
Option B - Supine with neck extension: In this position, the patient lies on their back with the neck exte
nded backward. Similar to option A, this position can obstruct the airway and hinder the passage of the
tube into the esophagus. It is not recommended for Ryle's tube insertion.
Option D - Sitting with neck extension: This position, where the patient sits upright with the neck extend
ed backward, is not appropriate for Ryle's tube insertion. It can compromise the airway and impede the
passage of the tube into the stomach.

Solution for Question 113:

Page 126

997
Correct Option B - Between Inferior constrictor:
• Killian's dehiscence is the anatomical defect or weakness in the posterior pharyngeal wall.
• In Zenker's diverticulum, the herniation of the esophagus occurs through this specific area, which is
located between the Inferior constrictor muscles cricopharyngeus and cricopharyngeus.
Killian's dehiscence is the anatomical defect or weakness in the posterior pharyngeal wall.
In Zenker's diverticulum, the herniation of the esophagus occurs through this specific area, which is loc
ated between the Inferior constrictor muscles cricopharyngeus and cricopharyngeus.
Incorrect Options:
Option A
- Below Superior constrictor: Killian's dehiscence is not located below the Superior constrictor muscle.
Option C - Above the cricopharyngeal muscle: Killian's dehiscence is not located above the cricophary
ngeal muscle.
Option D - Below the upper 1/3rd of the smooth muscle of the esophagus: Killian's dehiscence is not lo
cated below the upper 1/3rd of the smooth muscle of the esophagus.

Solution for Question 114:


Correct Option C -Prostate:
• Prostate cancer typically metastasizes to the bones, particularly the spine, pelvis, and long bones.
Liver metastasis is relatively rare in prostate cancer compared to other sites.
Prostate cancer typically metastasizes to the bones, particularly the spine, pelvis, and long bones. Live
r metastasis is relatively rare in prostate cancer compared to other sites.
Incorrect Options:
Option A - Colon: Colon cancer commonly metastasizes to the liver. The liver is a common site of meta
stasis for colorectal cancer, and liver metastases are often seen in advanced stages of the disease.
Option B - Breast: Breast cancer can metastasize to various organs, including the liver. Liver metastas
es from breast cancer are relatively common, especially in the advanced stages of the disease.
Option D - Melanoma: Melanoma, a type of skin cancer, has the potential to spread to various organs, i
ncluding the liver. Liver metastases from melanoma can occur, although they are less common than m
etastases to the lymph nodes, lungs, or brain.

Solution for Question 115:


Correct Option B - Gastrograffin follow-through:
• A Gastrograffin follow-through is a valuable diagnostic tool in a 3-day-old neonate who presents with
recurrent bilious vomiting. It can help determine if there is an obstruction or other abnormality in the
gastrointestinal tract causing the symptoms. Bilious vomiting in a neonate can be indicative of an
obstructed gastrointestinal tract, and a Gastrograffin follow-through can identify the location and nature
of the obstruction.

Page 127

998
• A Gastrograffin follow-through is a radiographic imaging procedure in which a contrast agent (such as
Gastrograffin) is ingested orally, and sequential X-rays are taken to visualize the passage of the
contrast through the gastrointestinal tract. This test is useful for evaluating the presence and location of
intestinal obstructions or anatomical abnormalities that may be causing the neonate's symptoms. It can
help identify the site of obstruction and guide further management decisions.
A Gastrograffin follow-through is a valuable diagnostic tool in a 3-day-old neonate who presents with re
current bilious vomiting. It can help determine if there is an obstruction or other abnormality in the gastr
ointestinal tract causing the symptoms. Bilious vomiting in a
neonate can be indicative of an obstructed gastrointestinal tract, and a
Gastrograffin follow-through can identify the location and nature of the obstruction.
A Gastrograffin follow-through is a radiographic imaging procedure in which a contrast agent (such as
Gastrograffin) is ingested orally, and sequential X-rays are taken to visualize the passage of the contra
st through the gastrointestinal tract. This test is useful for evaluating the presence and location of intest
inal obstructions or anatomical abnormalities that may be causing the neonate's symptoms. It can help
identify the site of obstruction and guide further management decisions.
Incorrect Options:
Option A - CT abdomen: Computed tomography (CT) abdomen is not typically the initial diagnostic test
of choice for evaluating a neonate with bilious vomiting. CT scans involve radiation exposure, and in th
e case of an infant, alternative imaging modalities are preferred.
Option C - USG abdomen: Ultrasonography (USG) abdomen is a non-invasive imaging modality that c
an provide valuable information in evaluating abdominal conditions. However, in the case of a
neonate with recurrent bilious vomiting, a Gastrograffin follow-through is more appropriate for assessin
g intestinal patency and identifying possible obstructions.
Option D - Explorative laparotomy: Explorative laparotomy, which is a surgical procedure involving abd
ominal exploration, is generally reserved for cases in which other diagnostic modalities have been inco
nclusive or when there is a high suspicion of a
surgical emergency. It is not the initial step in managing a neonate with recurrent bilious vomiting.

Solution for Question 116:


Correct Option A - Liposarcoma:
• Liposarcoma is the most commonly seen tumor in the retroperitoneal space. Liposarcomas are
malignant tumors that arise from adipose tissue. They can occur in various locations, but they are
frequently found in the retroperitoneal area.
Liposarcoma is the most commonly seen tumor in the retroperitoneal space. Liposarcomas are malign
ant tumors that arise from adipose tissue. They can occur in various locations, but they are frequently f
ound in the retroperitoneal area.
Incorrect Options:
Option B - Rhabdomyosarcoma: Rhabdomyosarcoma is a malignant tumor that arises from skeletal m
uscle. While it can occur in various sites, it is less commonly seen in the retroperitoneal space compar
ed to liposarcoma.
Option C - Synovial sarcoma: Synovial sarcoma is a malignant tumor that typically arises in the extremi
ties, particularly around the joints. It is rare in the retroperitoneal space.

Page 128

999
Option D - Neurofibroma: Neurofibroma is a benign tumor that arises from peripheral nerves. While ne
urofibromas can occur in the retroperitoneal space, they are less common than liposarcomas.

Solution for Question 117:


Correct Option A - Upper Gl endoscopy & Manometry:
• Achalasia cardia is a condition characterized by the impaired movement and relaxation of the lower
esophageal sphincter (LES) and absence of esophageal peristalsis, leading to difficulty in swallowing
and food getting stuck in the chest. The barium swallow image provided indicates the characteristic
findings of achalasia, such as a dilated esophagus with a narrowed LES ("bird's beak" appearance).
• To confirm the diagnosis of achalasia and further evaluate the condition, additional investigations are
required. Upper GI endoscopy (also known as esophagogastroduodenoscopy or EGD) is performed to
directly visualize the esophagus, LES, and rule out other possible causes of dysphagia such as
strictures, tumors, or inflammation.
• Manometry is an important test for diagnosing achalasia. It measures the pressure and motility of the
esophagus and LES, providing information about the impaired function and lack of coordinated
contractions.
Achalasia cardia is a condition characterized by the impaired movement and relaxation of the lower es
ophageal sphincter (LES) and absence of esophageal peristalsis, leading to difficulty in swallowing and
food getting stuck in the chest. The barium swallow image provided indicates the characteristic finding
s of achalasia, such as a dilated esophagus with a narrowed LES ("bird's beak" appearance).
To confirm the diagnosis of achalasia and further evaluate the condition, additional investigations are r
equired. Upper GI endoscopy (also known as esophagogastroduodenoscopy or EGD) is performed to
directly visualize the esophagus, LES, and rule out other possible causes of dysphagia such as strictur
es, tumors, or inflammation.
Manometry is an important test for diagnosing achalasia. It measures the pressure and motility of the e
sophagus and LES, providing information about the impaired function and lack of coordinated contracti
ons.
Incorrect Options :
Option B - Upper GI endoscopy & PET scan: A PET scan is not typically required for the initial evaluati
on of achalasia. It is more commonly used in cancer staging or to assess the metabolic activity of tumo
rs. Achalasia is a motility disorder rather than a malignant condition, so a PET scan is not necessary.
Option C - Upper GI endoscopy & CECT: Contrast-enhanced computed tomography (CECT) is not the
first-line investigation for achalasia. While it may provide additional information about structural abnorm
alities or complications associated with achalasia, such as esophageal dilation or diverticula, it is not ty
pically performed as the initial diagnostic step.
Option D - Upper GI endoscopy and 24-hour pH monitoring: 24-hour pH monitoring is primarily used to
assess gastroesophageal reflux disease (GERD) and the presence of abnormal acid reflux. Achalasia i
s not typically associated with excessive acid reflux, so 24-hour pH monitoring is not the primary investi
gation needed in this case.

Solution for Question 118:

Page 129

1000
Correct Option C - Left inguinal hernia:
• The given image is of Left inguinal hernia.

Incorrect Options:
• Option A, B & D are incorrect

Solution for Question 119:


Correct Option A - B & C:
• Gas under the diaphragm seen on an erect abdominal X-ray is indicative of a perforated viscus, most
commonly a perforated peptic ulcer or a gastrointestinal perforation. This condition is a surgical
emergency that requires immediate intervention. The patient's severe abdominal pain, tachycardia (HR
of 110 bpm), hypotension (BP of 88/60 mmHg), and guarding on abdominal examination further
support the need for urgent management.
Gas under the diaphragm seen on an erect abdominal X-ray is indicative of a
perforated viscus, most commonly a perforated peptic ulcer or a
gastrointestinal perforation. This condition is a surgical emergency that requires immediate intervention
. The patient's severe abdominal pain, tachycardia (HR of 110 bpm), hypotension (BP of 88/60 mmHg),
and guarding on abdominal examination further support the need for urgent management.
The correct approach in this case would be:
• (B) IV fluids: Intravenous fluid resuscitation is crucial to stabilize the patient's hemodynamic status and
address hypotension. This helps maintain organ perfusion and improve overall patient condition before
surgical intervention.
• (C) Exploratory laparotomy: This is the definitive management for a suspected perforated viscus.
Exploratory laparotomy allows direct visualization of the abdominal organs and identification of the
perforation site, which can then be repaired or managed accordingly.
(B) IV fluids: Intravenous fluid resuscitation is crucial to stabilize the patient's hemodynamic status and
address hypotension. This helps maintain organ perfusion and improve overall patient condition before

Page 130

1001
surgical intervention.
(C) Exploratory laparotomy: This is the definitive management for a suspected perforated viscus. Explo
ratory laparotomy allows direct visualization of the abdominal organs and identification of the perforatio
n site, which can then be repaired or managed accordingly.
Incorrect Options:
Option B - B & D:
• (D) Elective surgery: Elective surgery refers to planned procedures that are not urgent or emergent. In
this case, with the patient presenting with severe abdominal pain, hypotension, and gas under the
diaphragm on X-ray, elective surgery is not appropriate. The patient requires immediate intervention
and stabilization.
(D) Elective surgery: Elective surgery refers to planned procedures that are not urgent or emergent. In t
his case, with the patient presenting with severe abdominal pain, hypotension, and gas under the diaph
ragm on X-ray, elective surgery is not appropriate. The patient requires immediate intervention and sta
bilization.
Option C - A, B & C:
• (A) USG abdomen: While ultrasound (USG) abdomen can be a useful diagnostic tool in certain cases,
such as evaluating the gallbladder or identifying fluid collections, it is not the primary modality for
managing a suspected perforated viscus. In this acute scenario, immediate intervention with fluid
resuscitation and surgery takes priority over diagnostic imaging.
(A) USG abdomen: While ultrasound (USG) abdomen can be a useful diagnostic tool in certain cases,
such as evaluating the gallbladder or identifying fluid collections, it is not the primary modality for mana
ging a suspected perforated viscus. In this acute scenario, immediate intervention with fluid resuscitatio
n and surgery takes priority over diagnostic imaging.
Option D - A, B & E:
• (E) Conservative management: Conservative management, which involves non-surgical or
non-invasive approaches, is not suitable for a suspected perforated viscus. Conservative management
alone would not address the underlying problem and could potentially lead to serious complications or
deterioration of the patient's condition.
(E) Conservative management: Conservative management, which involves non-surgical or non-invasiv
e approaches, is not suitable for a suspected perforated viscus. Conservative management alone woul
d not address the underlying problem and could potentially lead to serious complications or deterioratio
n of the patient's condition.

Solution for Question 120:


Correct Option C - Acute mesenteric ischemia:
• Acute mesenteric ischemia refers to a sudden decrease in blood flow to the mesentery, which is the
tissue that supplies the small and large intestines. It is typically caused by a blood clot or embolus that
obstructs the mesenteric arteries, leading to inadequate blood supply to the intestines. The symptoms
of acute mesenteric ischemia can include severe abdominal pain, vomiting, and absent bowel sounds.
The pulse rate may be elevated, and the blood pressure can be low due to shock.
• Chronic mesenteric ischemia is usually caused by atherosclerosis, a condition characterized by the
buildup of fatty plaques in the arteries. Over time, these plaques can narrow the blood vessels that
supply the intestines, leading to reduced blood flow during periods of increased demand (e.g., after a

Page 131

1002
meal).
• Symptoms of mesenteric ischemia can vary depending on the severity and duration of blood flow
compromise. Common symptoms include severe abdominal pain, typically out of proportion to physical
examination findings, nausea, vomiting, diarrhea, and bloody stools in some cases. Chronic mesenteric
ischemia may present with postprandial abdominal pain, unintentional weight loss, and fear of eating
due to discomfort.
Acute mesenteric ischemia refers to a sudden decrease in blood flow to the mesentery, which is the tis
sue that supplies the small and large intestines. It is typically caused by a blood clot or embolus that ob
structs the mesenteric arteries, leading to inadequate blood supply to the intestines. The symptoms of
acute mesenteric ischemia can include severe abdominal pain, vomiting, and absent bowel sounds. Th
e pulse rate may be elevated, and the blood pressure can be low due to shock.
Chronic mesenteric ischemia is usually caused by atherosclerosis, a condition characterized by the buil
dup of fatty plaques in the arteries. Over time, these plaques can narrow the blood vessels that supply
the intestines, leading to reduced blood flow during periods of increased demand (e.g., after a meal).
Symptoms of mesenteric ischemia can vary depending on the severity and duration of blood flow comp
romise. Common symptoms include severe abdominal pain, typically out of proportion to physical exa
mination findings, nausea, vomiting, diarrhea, and bloody stools in some cases. Chronic mesenteric isc
hemia may present with postprandial abdominal pain, unintentional weight loss, and fear of eating due
to discomfort.
Incorrect Options:
Option A - Acute severe pancreatitis: While acute severe pancreatitis can cause severe abdominal pai
n and vomiting, it is not typically associated with absent bowel sounds or hemodynamic instability (elev
ated heart rate, low blood pressure) as seen in this case.
Option B - Ruptured aortic aneurysm: Ruptured aortic aneurysm is a life-threatening condition that can
present with sudden, severe abdominal pain. However, it is more commonly associated with signs of h
ypovolemic shock, such as a rapidly falling blood pressure and palpable abdominal mass. In this case,
the blood pressure is 90/60 mmHg, which is low, but there is no mention of an abdominal mass.
Option D - Peptic ulcer perforation: Peptic ulcer perforation can cause sudden, severe abdominal pain
and vomiting. However, in this case, the pulse rate is elevated and irregularly irregular, which may sug
gest atrial fibrillation or another cardiac arrhythmia. Peptic ulcer perforation is not typically associated w
ith cardiac arrhythmias.

Solution for Question 121:


Correct Option D - Lightweight mesh with large pores:
• Hernioplasty is a surgical procedure used to repair hernias by reinforcing the weakened abdominal
wall with a mesh implant. The choice of mesh is an important consideration as it affects the outcome of
the procedure and the long-term success of the hernia repair.
Hernioplasty is a
surgical procedure used to repair hernias by reinforcing the weakened abdominal wall with a mesh impl
ant. The choice of mesh is an important consideration as it affects the outcome of the procedure and th
e long-term success of the hernia repair.
The ideal mesh for hernioplasty should possess certain characteristics:

Page 132

1003
• Lightweight: A lightweight mesh is preferred as it provides sufficient strength and support while
minimizing the risk of complications, such as mesh-related pain or discomfort. Heavyweight meshes
may lead to a higher incidence of these complications.
• Large pores: Meshes with large pores allow for better tissue integration and facilitate the ingrowth of
the patient's tissue. This promotes better healing and reduces the risk of complications, such as
infection or seroma formation. Large-pore meshes also provide adequate mesh flexibility, which is
important for the natural movement and biomechanics of the abdominal wall.
Lightweight: A lightweight mesh is preferred as it provides sufficient strength and support while minimiz
ing the risk of complications, such as mesh-related pain or discomfort. Heavyweight meshes may lead t
o a higher incidence of these complications.
Large pores: Meshes with large pores allow for better tissue integration and facilitate the ingrowth of th
e patient's tissue. This promotes better healing and reduces the risk of complications, such as infection
or seroma formation. Large-pore meshes also provide adequate mesh flexibility, which is important for
the natural movement and biomechanics of the abdominal wall.
Incorrect Options:
Option A - Heavyweight mesh with small pores: Heavyweight meshes with small pores may offer incre
ased strength and durability, but they are associated with a higher risk of complications, such as chroni
c pain and mesh-related infections. The small pores restrict tissue ingrowth and can lead to a
higher incidence of complications.
Option B - Heavyweight mesh with large pores: Lightweight meshes are generally favored over heavyw
eight meshes due to their lower risk of complications. For most routine hernioplasty cases, a
lightweight mesh is preferred.
Option C - Lightweight mesh with small pores: Lightweight meshes are generally favored over heavywe
ight meshes due to their lower risk of complications. However, meshes with small pores may hinder tis
sue ingrowth and increase the risk of complications, similar to those described with Option A

Solution for Question 122:


Correct Options B - APC, k-ras, p53:
• Colorectal carcinoma, including colon cancer, is a complex disease involving multiple genetic
alterations. The most common gene mutations associated with colon cancer are APC (adenomatous
polyposis coli), k-ras (Kirsten rat sarcoma viral oncogene homolog), and p53 (tumor protein p53).
• APC mutation: The APC gene plays a crucial role in regulating cell growth and division. Mutations in
the APC gene are commonly found in colorectal cancer. These mutations can lead to the development
of precancerous polyps (adenomas) in the colon, which can progress to carcinoma over time.
• k-ras mutation: The k-ras gene is involved in cell signaling pathways that control cell growth and
differentiation. Mutations in the k-ras gene are found in approximately 40% to 50% of colon cancers.
These mutations lead to the activation of the K-Ras protein, which contributes to uncontrolled cell
growth and tumor formation.
• p53 mutation: The p53 gene is a tumor suppressor gene that helps regulate cell division and prevent
the formation of cancerous cells. Mutations in the p53 gene are observed in a significant proportion of
colon cancers. Loss of normal p53 function allows cells with damaged DNA to survive and proliferate,
increasing the risk of cancer development.

Page 133

1004
Colorectal carcinoma, including colon cancer, is a complex disease involving multiple genetic alteration
s. The most common gene mutations associated with colon cancer are APC (adenomatous polyposis c
oli), k-ras (Kirsten rat sarcoma viral oncogene homolog), and p53 (tumor protein p53).
APC mutation: The APC gene plays a crucial role in regulating cell growth and division. Mutations in th
e APC gene are commonly found in colorectal cancer. These mutations can lead to the development of
precancerous polyps (adenomas) in the colon, which can progress to carcinoma over time.
k-ras mutation: The k-ras gene is involved in cell signaling pathways that control cell growth and differe
ntiation. Mutations in the k-ras gene are found in approximately 40% to 50% of colon cancers. These m
utations lead to the activation of the K-Ras protein, which contributes to uncontrolled cell growth and tu
mor formation.
p53 mutation: The p53 gene is a tumor suppressor gene that helps regulate cell division and prevent th
e formation of cancerous cells. Mutations in the p53 gene are observed in a significant proportion of col
on cancers. Loss of normal p53 function allows cells with damaged DNA to survive and proliferate, incr
easing the risk of cancer development.
Incorrect Options:
Options A - APC, BRCA, k-ras: While APC and k-ras mutations are correctly mentioned in this option,
BRCA (Breast Cancer Susceptibility Gene) mutations are primarily associated with an increased risk of
breast and ovarian cancers, not colorectal carcinoma.
Options C - MYH, K-RAS, APC: This option includes the MYH (MutY Homolog) gene, which is associat
ed with an autosomal recessive condition called MUTYH-associated polyposis (MAP). MAP can increa
se the risk of colorectal cancer, but it is a relatively rare condition. The correct combination of gene mut
ations involved in the majority of colon cancers is APC, k-ras, and p53, as mentioned in option B.
Options D - MYH, p53, DCC: Similar to option C, this option includes the MYH gene mutation associate
d with MUTYH-associated polyposis. However, DCC (Deleted in Colorectal Cancer) is a
gene that was initially identified as a
potential tumor suppressor gene in colorectal cancer but is now known to play a
lesser role compared to APC, k-ras, and p53 mutations in colon cancer development.

Solution for Question 123:


Correct Option A - Colon carcinoma:
• Immunohistochemistry is a technique used to study the presence and distribution of specific proteins
in tissue samples. In this case, the tumor cells are negative for CK 7 (cytokeratin 7) but positive for
CK20 (cytokeratin 20) and CDX-1 (caudal type homeobox transcription factor 1). These markers are
commonly used to identify the origin of metastatic adenocarcinomas.
• The combination of CK 7 (-), CK20 (+), and CDX-1 (+) markers is characteristic of colorectal
adenocarcinoma, which originates in the colon or rectum. CK 7 is typically negative in colorectal
adenocarcinoma, while CK20 and CDX-1 are often positive. This immunophenotype is seen in the
majority of colorectal adenocarcinomas.
Immunohistochemistry is a technique used to study the presence and distribution of specific proteins in
tissue samples. In this case, the tumor cells are negative for CK 7 (cytokeratin 7) but positive for CK20
(cytokeratin 20) and CDX-1 (caudal type homeobox transcription factor 1). These markers are commo
nly used to identify the origin of metastatic adenocarcinomas.
The combination of CK 7 (-), CK20 (+), and CDX-1 (+) markers is characteristic of colorectal adenocarc
inoma, which originates in the colon or rectum. CK 7 is typically negative in colorectal adenocarcinoma,

Page 134

1005
while CK20 and CDX-1 are often positive. This immunophenotype is seen in the majority of colorectal
adenocarcinomas.
Incorrect Options:
Option B - Breast carcinoma: Breast carcinoma typically shows a
different immunophenotype. It is often CK 7 (+) and CK20 (-). Therefore, breast carcinoma is less likely
to be the primary site of origin for this metastatic liver adenocarcinoma.
Option C - Pancreatic carcinoma: Pancreatic carcinoma usually shows a CK 7 (+) and CK20 (-) immun
ophenotype. Therefore, it is less likely to be the primary site of origin in this case.
Option D - Ovarian carcinoma: Ovarian carcinoma can have variable immunophenotypes, but it is less l
ikely to present with a CK 7 (-) and CK20 (+) pattern. Therefore, ovarian carcinoma is less likely to be t
he primary site of origin for this metastatic liver adenocarcinoma.

Solution for Question 124:


Correct Option B - 1, 2 and 3:
Autosomal dominant disease - True
Defect in the STK11 gene - The gene STK11 on chromosome 19has been found in a
proportion of patients with this condition.
Melanin spots on digits and perianal skin - True
Autosomal recessive - False as it is autosomal dominant
Congenital hypertrophy of retinal pigment epithelium is seen - False
Incorrect Options:
Option A, C & D are incorrect.

Solution for Question 125:


Correct Option B - Caecum:
• The caecum is the first part of the large intestine or colon.
• During a laparotomy, when examining the caecum: If the caecum is distended, it indicates the
obstruction is beyond the caecum. If the caecum is collapsed, it suggests the patient is experiencing
small bowel obstruction.
• If the caecum is distended, it indicates the obstruction is beyond the caecum.
• If the caecum is collapsed, it suggests the patient is experiencing small bowel obstruction.
The caecum is the first part of the large intestine or colon.
During a laparotomy, when examining the caecum:
• If the caecum is distended, it indicates the obstruction is beyond the caecum.
• If the caecum is collapsed, it suggests the patient is experiencing small bowel obstruction.
If the caecum is distended, it indicates the obstruction is beyond the caecum.

Page 135

1006
If the caecum is collapsed, it suggests the patient is experiencing small bowel obstruction.
Incorrect Option:
Option A - Ileum: The ileum is a part of the small intestine, and its visualization can provide information
about the location of the obstruction within the small bowel. However, it does not specifically differentiat
e between small bowel and large bowel obstruction.
Option C - Jejunum: The jejunum is also a part of the small intestine. Like the ileum, visualization of the
jejunum provides information about the location of the obstruction within the small bowel but does not
specifically differentiate between small bowel and large bowel obstruction.
Option D - Sigmoid colon: The sigmoid colon is the S-shaped part of the large intestine located just bef
ore the rectum. While a sigmoid colon obstruction can cause symptoms of large bowel obstruction, it is
not the first organ to visualize for determining whether it is a
case of small bowel or large bowel obstruction.

Solution for Question 126:


Correct Option A - Platelet transfusion:
• Platelet transfusion is not typically done as a primary intervention for active variceal bleeding.
Platelets are not directly involved in the clotting mechanism of variceal bleeding, which is primarily
caused by increased portal pressure and vascular wall abnormalities. Platelet transfusion may be
considered in cases of concomitant thrombocytopenia or other specific indications but is not a routine
intervention for variceal bleeding.
Platelet transfusion is not typically done as a primary intervention for active variceal bleeding. Platelets
are not directly involved in the clotting mechanism of variceal bleeding, which is primarily caused by inc
reased portal pressure and vascular wall abnormalities. Platelet transfusion may be considered in case
s of concomitant thrombocytopenia or other specific indications but is not a
routine intervention for variceal bleeding.
Incorrect Option:
Option B - Arrange pRBC: Packed red blood cell transfusion is a crucial intervention in the managemen
t of active variceal bleeding. It aims to correct anemia and maintain hemodynamic stability by restoring
blood volume and oxygen-carrying capacity.
Option C - IV Octreotide: Octreotide is a
medication that is frequently used in the management of active variceal bleeding. It is a synthetic soma
tostatin analog that reduces splanchnic blood flow and inhibits the release of various vasoactive substa
nces, including vasodilatory peptides. IV administration of octreotide helps control bleeding and decrea
se portal pressure.
Option D - Urgent Endoscopy: Urgent endoscopy is a vital component of the management of active var
iceal bleeding. It allows for direct visualization of the bleeding site, identification of high-risk stigmata, a
nd the application of endoscopic therapies, such as band ligation or sclerotherapy, to achieve hemosta
sis.

Solution for Question 127:


Correct Option B - Acute pancreatitis:

Page 136

1007
• Grey-Turner's sign is a clinical finding that can be seen in cases of severe acute pancreatitis. It refers
to bruising or discoloration around the flanks, specifically the area between the lower ribs and the iliac
crest. Grey-Turner's sign is attributed to retroperitoneal hemorrhage or bleeding into the flank area,
resulting from pancreatic enzymes and inflammatory mediators damaging blood vessels. It is
considered a rare finding but can be observed in severe cases of pancreatitis.
Grey-Turner's sign is a clinical finding that can be seen in cases of severe acute pancreatitis. It refers t
o bruising or discoloration around the flanks, specifically the area between the lower ribs and the iliac c
rest. Grey-Turner's sign is attributed to retroperitoneal hemorrhage or bleeding into the flank area, resul
ting from pancreatic enzymes and inflammatory mediators damaging blood vessels. It is considered a
rare finding but can be observed in severe cases of pancreatitis.

Incorrect Options:
Option A - Acute appendicitis: Grey-Turner's sign is not typically associated with acute appendicitis. Ac
ute appendicitis refers to inflammation of the appendix, which is a small pouch-like structure connected
to the large intestine. The most common symptom of acute appendicitis is severe abdominal pain, typi
cally starting around the umbilicus and then shifting to the right lower quadrant of the abdomen. Grey-T
urner's sign, which refers to bruising or discoloration around the flanks, is not a
characteristic finding in acute appendicitis.
Option C - Acute cholecystitis: Grey-Turner's sign is not typically associated with acute cholecystitis. Ac
ute cholecystitis refers to inflammation of the gallbladder, usually caused by gallstones blocking the cys
tic duct. The main symptom of acute cholecystitis is right upper quadrant abdominal pain, which can be
severe and persistent. Grey-Turner's sign is not a characteristic finding in acute cholecystitis.
Option D - Acute hepatitis: Grey-Turner's sign is not typically associated with acute hepatitis. Acute hep
atitis refers to inflammation of the liver, often caused by viral infections (such as hepatitis A, B, or C) or
drug-induced liver injury. While acute hepatitis can present various signs and symptoms, Grey-Turner's
sign is not a common finding in this condition.

Solution for Question 128:


Correct Option C - Arise from beta cell tumors of the pancreas:

Page 137

1008
• Zollinger-Ellison syndrome is not caused by beta cell tumors of the pancreas. Instead, it is associated
with gastrinomas, which are neuroendocrine tumors derived from the cells that produce gastrin.
Gastrinomas can occur in various locations, including the pancreas, duodenum, and other areas of the
gastrointestinal tract.
Zollinger-Ellison syndrome is not caused by beta cell tumors of the pancreas. Instead, it is associated
with gastrinomas, which are neuroendocrine tumors derived from the cells that produce gastrin. Gastrin
omas can occur in various locations, including the pancreas, duodenum, and other areas of the gastroi
ntestinal tract.
Incorrect Options:
Option A - Intractable peptic ulcers: Intractable peptic ulcers are a characteristic feature of Zollinger-Elli
son syndrome. These ulcers are persistent and do not heal despite conventional treatment. They are u
sually located in the upper part of the small intestine (duodenum) but can also occur in the stomach an
d other parts of the gastrointestinal tract.
Option B - Severe diarrhea: Severe diarrhea is another hallmark feature of Zollinger-Ellison syndrome.
It is caused by the excessive production of gastrin, a hormone that stimulates gastric acid secretion. In
creased acid secretion can lead to malabsorption and stimulate the intestines, resulting in diarrhea.
Option D - Very high acid output: Zollinger-Ellison syndrome is characterized by very high acid output.
Gastrinomas in Zollinger-Ellison syndrome produce excessive amounts of gastrin, leading to the hyper
secretion of gastric acid. This high acid output contributes to the development of intractable peptic ulce
rs and other symptoms associated with the condition.

Solution for Question 129:


Correct Option C - Leiomyosarcoma:
• Leiomyosarcoma is the most common type of gastric sarcoma. It arises from the smooth muscle cells
of the stomach wall. Leiomyosarcomas can be locally invasive and have the potential to metastasize to
other organs. They are considered malignant tumors and require appropriate management, including
surgery and sometimes chemotherapy.
Leiomyosarcoma is the most common type of gastric sarcoma. It arises from the smooth muscle cells o
f the stomach wall. Leiomyosarcomas can be locally invasive and have the potential to metastasize to
other organs. They are considered malignant tumors and require appropriate management, including s
urgery and sometimes chemotherapy.
Incorrect Options:
Option A - Lipoma: A lipoma is a benign tumor composed of fat cells. It is not considered a
sarcoma, which is a malignant tumor arising from mesenchymal tissues. Lipomas can occur in various
parts of the body, including the gastrointestinal tract, but they are not the most common type of gastric
sarcoma.
Option B - Glomus tumor: Glomus tumors are rare neoplasms that arise from the glomus body, a therm
oregulatory organ located in the dermis of the skin. Glomus tumors are usually benign, and their occurr
ence in the gastrointestinal tract is extremely rare. Therefore, they are not the most common type of ga
stric sarcoma.
Option D - Leioblastoma: Leioblastoma is not a recognized medical term or a
well-established tumor entity. It is likely a misspelling or a term that is not commonly used in medical lit
erature. Therefore, it is not considered the most common type of gastric sarcoma.

Page 138

1009
Solution for Question 130:
Correct Option C - Ileocolic:
• Ileocolic intussusception is the most common type of intussusception. It involves the invagination of
the ileum (small intestine) into the colon (large intestine). This type of intussusception accounts for the
majority of cases, particularly in children.
Ileocolic intussusception is the most common type of intussusception. It involves the invagination of the
ileum (small intestine) into the colon (large intestine). This type of intussusception accounts for the maj
ority of cases, particularly in children.
Incorrect Options:
Option A - Colocolic: Colocolic intussusception refers to the telescoping or invagination of a
segment of the colon into an adjacent segment of the colon. It is a less common type of intussusceptio
n compared to other variations. Colocolic intussusception can occur but is not the most common type.
Option B - Ileoileal: Ileoileal intussusception involves the invagination of a
segment of the small intestine (ileum) into an adjacent segment of the small intestine. It is also a less c
ommon type of intussusception. Ileoileal intussusception can occur but is not the most common type.
Option D
- Ileal: Ileal intussusception refers to the invagination of the ileum (small intestine) into itself. It is a less
common type of intussusception compared to ileocolic intussusception. Ileal intussusception can occur
but is not the most common type.

Solution for Question 131:


Correct Option A - Mallory-Weiss syndrome:
• Mallory-Weiss syndrome is a condition characterized by a tear or laceration in the lining of the junction
between the esophagus and the stomach. It is typically caused by excessive vomiting or retching, such
as in cases of heavy alcohol consumption or bulimia. The main symptom is upper gastrointestinal
bleeding, which can manifest as hematemesis or melena. Given the patient's history of heavy alcohol
drinking, multiple episodes of vomiting, and hematemesis, Mallory-Weiss syndrome is a likely
diagnosis.
Mallory-Weiss syndrome is a condition characterized by a tear or laceration in the lining of the junction
between the esophagus and the stomach. It is typically caused by excessive vomiting or retching, such
as in cases of heavy alcohol consumption or bulimia. The main symptom is upper gastrointestinal blee
ding, which can manifest as hematemesis or melena. Given the patient's history of heavy alcohol drinki
ng, multiple episodes of vomiting, and hematemesis, Mallory-Weiss syndrome is a likely diagnosis.
Incorrect Options:
Option B - Esophageal carcinoma: Esophageal carcinoma refers to cancer of the esophagus, which ca
n present with symptoms such as dysphagia, weight loss, chest pain, and gastrointestinal bleeding. Wh
ile heavy alcohol consumption is a risk factor for esophageal cancer, the acute onset of vomiting and h
ematemesis in this patient is not typically associated with esophageal carcinoma.
Option C - Achalasia cardia: Achalasia cardia is a disorder of the esophagus characterized by the inabi
lity of the lower esophageal sphincter to relax and allow food to pass into the stomach. Symptoms inclu

Page 139

1010
de difficulty swallowing, regurgitation of undigested food, and chest pain. Achalasia cardia does not typ
ically present with episodes of vomiting and hematemesis.
Option D - Boerhaave's syndrome: Boerhaave's syndrome is a
life-threatening condition characterized by a
rupture or perforation of the esophagus, usually caused by severe vomiting or retching. It is a medical
emergency and can present with symptoms such as severe chest pain, difficulty breathing, and subcut
aneous emphysema (air trapped under the skin). While heavy alcohol consumption can increase the ri
sk of esophageal rupture, the multiple episodes of vomiting and hematemesis in this patient are more s
uggestive of Mallory-Weiss syndrome.

Solution for Question 132:


Correct Option A - Herniotomy:
• Herniotomy refers to the surgical procedure performed to repair an inguinal hernia by removing the
herniated sac and closing the defect. It is the treatment of choice for inguinal hernia in infants. In this
procedure, the surgeon makes an incision in the groin area, reduces the hernia back into the abdominal
cavity, and closes the opening or weak spot in the abdominal wall. Herniotomy is a commonly
performed procedure in infants with inguinal hernia and has a high success rate.
Herniotomy refers to the surgical procedure performed to repair an inguinal hernia by removing the her
niated sac and closing the defect. It is the treatment of choice for inguinal hernia in infants. In this proc
edure, the surgeon makes an incision in the groin area, reduces the hernia back into the abdominal ca
vity, and closes the opening or weak spot in the abdominal wall. Herniotomy is a
commonly performed procedure in infants with inguinal hernia and has a high success rate.
Incorrect Options:
Option B - Herniorrhaphy: Herniorrhaphy is a general term for the surgical repair of a hernia. It involves
closing the defect in the abdominal wall through suturing or mesh reinforcement. While herniorrhaphy
can be used to repair inguinal hernias in adults, it is not the preferred treatment for inguinal hernias in i
nfants.
Option C - TRUSS: A truss is a supportive device that is sometimes used to provide temporary relief an
d support for inguinal hernias in adults. It consists of a belt or bandage-like device that applies pressur
e to the hernia site to keep it reduced. However, truss is not recommended as the primary treatment for
inguinal hernia in infants, as it does not address the underlying anatomical defect and carries the risk
of complications.
Option D - Hernioplasty: Hernioplasty is a
surgical technique that involves reinforcing the weakened area or defect in the abdominal wall with a sy
nthetic mesh or other material. It is often used in adult inguinal hernia repair procedures. Hernioplasty
may be considered in specific cases in infants or older children with complex or recurrent inguinal herni
as, but it is not the treatment of choice for routine inguinal hernia repair in infants.

Solution for Question 133:


Correct Option B - Prostatic carcinoma:
• Prostatic carcinoma, or prostate cancer, is a malignant tumor that develops in the prostate gland.
Prostate-specific antigen (PSA) is the commonly used marker for prostate cancer, but serum acid

Page 140

1011
phosphate levels can also be elevated in some cases. Prostatic carcinoma is the correct answer choice
for the question.
Prostatic carcinoma, or prostate cancer, is a malignant tumor that develops in the prostate gland. Prost
ate-specific antigen (PSA) is the commonly used marker for prostate cancer, but serum acid phosphate
levels can also be elevated in some cases. Prostatic carcinoma is the correct answer choice for the qu
estion.
Incorrect Options:
Option A - Osteosarcoma: Osteosarcoma is a type of bone cancer that arises from the cells that form n
ew bone tissue. While osteosarcoma can lead to elevated levels of alkaline phosphatase, it is not typic
ally associated with raised levels of serum acid phosphate.
Option C - Paget's disease: Paget's disease of bone is a chronic disorder characterized by abnormal b
one remodeling. It can lead to increased activity of osteoclasts (cells that break down bone) and subse
quent bone resorption. While Paget's disease can cause elevated levels of alkaline phosphatase, it is n
ot typically associated with raised levels of serum acid phosphate.
Option D - Hyperparathyroidism: Hyperparathyroidism is a condition characterized by overactivity of th
e parathyroid glands, resulting in increased levels of parathyroid hormone (PTH). PTH regulates calciu
m and phosphate metabolism, and in hyperparathyroidism, it leads to elevated levels of serum calcium
and reduced levels of serum phosphate. While hyperparathyroidism can affect phosphate levels, it doe
s not typically cause raised levels of serum acid phosphate.

Solution for Question 134:


Correct Option C - Chylolymphatic cyst:
• A 5-year-old child presents with abdominal swelling, which is typically caused by the presence of a
chylolymphatic cyst.
• A chylolymphatic cyst is a benign cystic lesion that occurs in the abdomen, often in pediatric patients.
• It is characterized by the accumulation of lymphatic fluid and chyle within a cystic structure.
• The clinical features described in the question stem to support the diagnosis of a chylolymphatic cyst.
• The swelling is fluctuant, which means it feels like it contains fluid. It is soft in consistency, indicating
that it is not solid.
• The swelling's mobility in a plane perpendicular to the attachment of the mesentery further suggests a
cystic lesion.
A 5-year-old child presents with abdominal swelling, which is typically caused by the presence of a
chylolymphatic cyst.
A chylolymphatic cyst is a benign cystic lesion that occurs in the abdomen, often in pediatric patients.
It is characterized by the accumulation of lymphatic fluid and chyle within a cystic structure.
The clinical features described in the question stem to support the diagnosis of a chylolymphatic cyst.
The swelling is fluctuant, which means it feels like it contains fluid. It is soft in consistency, indicating th
at it is not solid.
The swelling's mobility in a plane perpendicular to the attachment of the mesentery further suggests a
cystic lesion.

Page 141

1012
Incorrect Options:
Option A - Peritoneal cyst: Peritoneal cysts are rare and usually do not present with fluctuant soft swelli
ng. They can be solid or cystic in nature.
Option B - Lipoma: Lipomas are benign tumors composed of fatty tissue. They typically feel soft and ar
e not usually associated with abdominal swelling.
Option D - Pseudocyst: Pseudocysts are fluid-filled sacs that can develop in various locations, includin
g the abdomen. However, they are more commonly associated with acute or chronic pancreatitis and w
ould typically present with symptoms such as abdominal pain or other signs of pancreatic pathology.

Solution for Question 135:


Correct Option D - Fistula-in-ano:
• A fistula-in-ano, or anal fistula, is a chronic abnormal communication, usually lined to some degree by
granulation tissue, which runs outwards from the anorectal lumen (the internal opening) to an external
opening on the skin of the perineum or buttock (or rarely, in women, to the vagina).
• It is usually caused by an infection close to the anus, which results in a collection of pus (abscess) in
the surrounding tissue.
• When pus drains, it can leave behind a tiny canal. Anal fistulas can cause unpleasant symptoms such
as soreness and skin irritation, and they seldom heal on their own.
• Surgery is recommended in most cases.
A fistula-in-ano, or anal fistula, is a chronic abnormal communication, usually lined to some degree by
granulation tissue, which runs outwards from the anorectal lumen (the internal opening) to an external
opening on the skin of the perineum or buttock (or rarely, in women, to the vagina).
It is usually caused by an infection close to the anus, which results in a
collection of pus (abscess) in the surrounding tissue.
When pus drains, it can leave behind a tiny canal. Anal fistulas can cause unpleasant symptoms such
as soreness and skin irritation, and they seldom heal on their own.
Surgery is recommended in most cases.
Incorrect Options:
Option A - Pilonidal sinus: Pilonidal sinus describes a condition found in the natal cleft overlying the co
ccyx consisting of one or more, usually non-infected, midline openings that communicate with a
fibrous track lined by granulation tissue and containing hair lying loosely within the lumen. A common a
ffliction amongst the military, it has been referred to as "Jeep disease’. The case presented here does
not signify a pilonidal sinus.
Option B - Perianal abscess: A perianal abscess is a pus-filled boil that develops around the anus, rect
um, or perineal area (the space between the genitals and the anus). It occurs when one of the anus gla
nds becomes clogged and infected. Pus and fluid accumulate in the anal gland, resulting in an excrucia
tingly painful abscess around the anus (the hole in your buttocks). Hence, the condition presented can
not be diagnosed as a perianal abscess.
Option C - Hemorrhoids: Hemorrhoids are hard lumps that occur inside or outside of your rectum. They
can be painful and cause bleeding. Hemorrhoids, unlike abscesses, are not infections. Hemorrhoids ar
e bulging veins that can be treated with over-the-counter medications. Abscesses are typically more pa
inful and larger than hemorrhoids.

Page 142

1013
Solution for Question 136:
Correct Option C - Umbilical hernias:
• Umbilical hernias are frequent in newborns and young children near the navel. They rarely require
treatment because the majority resolve naturally during childhood.
• Umbilical hernias occur directly through the umbilicus, while paraumbilical hernias are within 2cm of
the umbilicus but not through it. In infants, umbilical hernias are often congenital, more common in
Africans, and usually close spontaneously by age 2. In adults, they are typically acquired, more
prevalent in females, associated with increased intra-abdominal pressure, and commonly seen in
conditions like pregnancy, ascites, cirrhosis, and obesity. Due to stretching, the defects in umbilical
hernias enlarge over time, leading to fewer complications. Consequently, complications are rare in
umbilical hernias.
Umbilical hernias are frequent in newborns and young children near the navel. They rarely require treat
ment because the majority resolve naturally during childhood.
Umbilical hernias occur directly through the umbilicus, while paraumbilical hernias are within 2cm of th
e umbilicus but not through it. In infants, umbilical hernias are often congenital, more common in Africa
ns, and usually close spontaneously by age 2. In adults, they are typically acquired, more prevalent in f
emales, associated with increased intra-abdominal pressure, and commonly seen in conditions like pre
gnancy, ascites, cirrhosis, and obesity. Due to stretching, the defects in umbilical hernias enlarge over t
ime, leading to fewer complications. Consequently, complications are rare in umbilical hernias.

Incorrect Options:
Option A - Omphaloceles: Omphaloceles are typically diagnosed on prenatal ultrasound. The intestines
become longer and push out from the belly into the umbilical cord as the baby develops between week
s six and ten of pregnancy. The intestines usually return to the belly by the eleventh week of pregnancy
. If this does not occur, an omphalocele develops. The case being discussed here is not related to omp
halocele.
Option B - Gastroschisis: Gastroschisis develops early in pregnancy, during the fourth through eighth w
eeks. It is the most prevalent birth defect in newborns born to women under the age of 20. A defect to t

Page 143

1014
he right of the umbilicus allows the fetal gut to prolapse. The case presented here does not signify gast
roschisis.
Option D - Paraumbilical hernia: Paraumbilical hernias, a type of midline ventral abdominal hernia, occ
ur due to weakened abdominal wall near the navel, allowing abdominal contents to protrude through th
e weakened area. This weakening typically coincides with the location where the umbilical cord previou
sly passed, resulting in a
bulge primarily consisting of intestinal or fatty tissue. Patients often exhibit overweight status and a thin
ned midline raphe, with the protrusion frequently manifesting on one side of the umbilical depression, g
iving the umbilicus a crescent shape

Solution for Question 137:


Correct Option C - Rectal foreign body:
• The rectum or anal canal can be wounded in a variety of ways, none of which are common: Falling in
a seated position onto a pointed item Penetrating injuries (including bullets) to the buttocks Sexual
assault or sexual activity involving anal penetration By the fetal head during childbirth, particularly
forceps-assisted childbirth.
• Falling in a seated position onto a pointed item
• Penetrating injuries (including bullets) to the buttocks
• Sexual assault or sexual activity involving anal penetration
• By the fetal head during childbirth, particularly forceps-assisted childbirth.
• Rectal foreign bodies are a difficult and unique topic of colorectal trauma. The reasons for insertion
and types of foreign bodies vary greatly. The most common objects observed are household objects
such as bottles and glasses. Other items include toothbrushes, deodorant bottles, food items, knives,
sports equipment, cell phones, flashlights, wooden rods, broomsticks, sex toys such as dildos and
vibrators, light bulbs, nails or other construction tools, Christmas ornaments, aerosol canisters, cocaine
packets, and a variety of others.
• In decreasing order of frequency, the motives for insertion are autoeroticism, concealment,
attention-seeking behavior, accidental, assault, and constipation relief. Some patients present soon
after becoming aware of their incapacity to remove the device, whereas others may present up to two
weeks following insertion for examination.
The rectum or anal canal can be wounded in a variety of ways, none of which are common:
• Falling in a seated position onto a pointed item
• Penetrating injuries (including bullets) to the buttocks
• Sexual assault or sexual activity involving anal penetration
• By the fetal head during childbirth, particularly forceps-assisted childbirth.
Falling in a seated position onto a pointed item
Penetrating injuries (including bullets) to the buttocks
Sexual assault or sexual activity involving anal penetration
By the fetal head during childbirth, particularly forceps-assisted childbirth.

Page 144

1015
Rectal foreign bodies are a difficult and unique topic of colorectal trauma. The reasons for insertion and
types of foreign bodies vary greatly. The most common objects observed are household objects such
as bottles and glasses. Other items include toothbrushes, deodorant bottles, food items, knives, sports
equipment, cell phones, flashlights, wooden rods, broomsticks, sex toys such as dildos and vibrators, li
ght bulbs, nails or other construction tools, Christmas ornaments, aerosol canisters, cocaine packets, a
nd a variety of others.
In decreasing order of frequency, the motives for insertion are autoeroticism, concealment, attention-se
eking behavior, accidental, assault, and constipation relief. Some patients present soon after becoming
aware of their incapacity to remove the device, whereas others may present up to two weeks following
insertion for examination.
Incorrect Options:
Option A - Ureterocele: A ureterocele is a cystic structure that projects into the bladder, usually around
the vesicoureteric junction (VUJ). In the majority of cases, this is ectopic and not at the typical location
of the ureteric orifice. The accompanying ureter is typically visibly dilated. A ureterocoele is a cystic enl
argement of the intramural ureter, probably due to atresia of the ureteric orifice. Hence, the case prese
nted depicts a case of a foreign rectal body and not a ureterocele.
Option B - Pelvic bone fracture: The majority of pelvic fractures are caused by trauma like motor vehicl
e collisions, pedestrian versus motor vehicle collisions, falls from great heights, and others, such as sp
orts injury or a
low-energy tumble. The X-ray image does not include any such trauma. Hence, the case is not a
pelvic bone fracture.
Option D - Bladder stone: Bladder stone (calculi) occurs in a "hollow" viscus and has a layered calcifica
tion made up of alternating layers of strongly calcified and matrix components. The X-ray image does n
ot relate to bladder calculi.

Solution for Question 138:


Correct Option C - Urgent surgical exploration:
• The next best step in the management of a 40-year-old male presenting with diffuse abdominal pain,
rigidity on physical examination, and an X-ray abdomen showing signs of possible perforated peptic
ulcer is urgent surgical exploration
• Urgent exploration is crucial to identify the source of the perforation, repair it, and address any
associated complications.
The next best step in the management of a 40-year-old male presenting with diffuse abdominal pain, ri
gidity on physical examination, and an X-ray abdomen showing signs of possible perforated peptic ulce
r is urgent surgical exploration
Urgent exploration is crucial to identify the source of the perforation, repair it, and address any associat
ed complications.
Incorrect Options:
Option A - Antibiotics and NPO (nothing by mouth): This option is not the best next step in this scenario
. While antibiotics may be a part of the management for suspected perforation, they should not be the i
nitial step. Urgent surgical exploration takes precedence over starting antibiotics. NPO status (withholdi
ng oral intake) is generally recommended in cases of suspected bowel perforation, but it is not sufficien
t as the sole management step.

Page 145

1016
Option B - Upper GI endoscopy: Upper GI endoscopy is a diagnostic procedure used to visualize the u
pper digestive tract, including the esophagus, stomach, and duodenum. However, in this case, the pati
ent's physical examination findings of rigidity and X-ray findings suggestive of possible perforation indic
ate a need for immediate surgical intervention rather than diagnostic endoscopy. Therefore, urgent sur
gical exploration is more appropriate.
Option D - CT abdomen: While a CT scan of the abdomen can provide valuable information in many ca
ses of abdominal pain, including suspected perforation, it is not the best next step when there are signs
of peritonitis, such as rigidity on physical examination. In this case, immediate surgical exploration is w
arranted, as waiting for a CT scan may delay necessary treatment.

Solution for Question 139:


Correct Option B - Congenital hypertrophic pyloric stenosis:
• Congenital hypertrophic pyloric stenosis (CHPS) is a condition that occurs in infants and is
characterized by hypertrophy (thickening) of the pylorus, the muscular valve between the stomach and
the small intestine.
• This hypertrophy leads to narrowing of the pyloric canal, resulting in obstruction of the passage of
food from the stomach to the intestines.
• Infants with CHPS typically present with projectile nonbilious vomiting, which is forceful and occurs
shortly after feeding. Lethargy and dehydration may also be present.
• Abdominal examination often reveals visible gastric peristalsis, known as an "olive" due to the
hypertrophied pylorus.
• The barium meal image may show delayed passage of contrast material through the pylorus or an
"string sign," indicating the narrowed pyloric canal.
Congenital hypertrophic pyloric stenosis (CHPS) is a condition that occurs in infants and is characteriz
ed by hypertrophy (thickening) of the pylorus, the muscular valve between the stomach and the small i
ntestine.
This hypertrophy leads to narrowing of the pyloric canal, resulting in obstruction of the passage of food
from the stomach to the intestines.
Infants with CHPS typically present with projectile nonbilious vomiting, which is forceful and occurs sho
rtly after feeding. Lethargy and dehydration may also be present.
Abdominal examination often reveals visible gastric peristalsis, known as an "olive" due to the hypertro
phied pylorus.
The barium meal image may show delayed passage of contrast material through the pylorus or an "stri
ng sign," indicating the narrowed pyloric canal.
Incorrect Options:
Option A - Hirschsprung's disease: Hirschsprung's disease is a
congenital disorder characterized by the absence of ganglion cells in the rectum and a portion of the co
lon. It typically presents with symptoms of bowel obstruction such as constipation, distended abdomen,
and failure to pass meconium shortly after birth. It is not associated with projectile nonbilious vomiting
or visible gastric peristalsis.
Option C - Jejunal atresia: Jejunal atresia refers to a blockage or absence of a portion of the jejunum, a
part of the small intestine. It commonly presents with symptoms of intestinal obstruction, including bilio

Page 146

1017
us vomiting and abdominal distension. Visible gastric peristalsis is not typically seen in jejunal atresia.
Option D - Duodenal atresia: Duodenal atresia is a congenital condition characterized by the complete
or partial blockage of the duodenum, the first part of the small intestine. It often presents with symptom
s of intestinal obstruction, including bilious vomiting. Visible gastric peristalsis may or may not be prese
nt. The barium meal image for duodenal atresia typically shows a
"double bubble" sign, indicating the dilation of the stomach and proximal duodenum.

Solution for Question 140:


Correct Option B - Hypokalemia:
• The X-ray shows a typical appearance of a paralytic ileus, which is a condition characterized by the
failure of normal bowel peristalsis after surgery.
• This can lead to abdominal distension and the absence of bowel gas patterns, as seen in the X-ray.
• Hypokalemia refers to a low level of potassium in the blood. It is a common electrolyte abnormality
that can occur after surgery due to various factors, including inadequate potassium intake, increased
renal losses, or shifts of potassium into cells.
• Postoperative patients may experience electrolyte imbalances, including hypokalemia, due to factors
such as reduced oral intake, gastrointestinal losses, and use of diuretics.
The X-ray shows a typical appearance of a paralytic ileus, which is a
condition characterized by the failure of normal bowel peristalsis after surgery.
This can lead to abdominal distension and the absence of bowel gas patterns, as seen in the X-ray.
Hypokalemia refers to a low level of potassium in the blood. It is a common electrolyte abnormality that
can occur after surgery due to various factors, including inadequate potassium intake, increased renal
losses, or shifts of potassium into cells.
Postoperative patients may experience electrolyte imbalances, including hypokalemia, due to factors s
uch as reduced oral intake, gastrointestinal losses, and use of diuretics.
Incorrect Options:
Option A - Hypocalcemia: Hypocalcemia is another electrolyte abnormality that can occur after surgery,
but it is not the most common one associated with the condition shown in the X-ray. Hypocalcemia is
more commonly associated with thyroid or parathyroid surgery.
Option C - Hyperkalemia: Hyperkalemia refers to a
high level of potassium in the blood and is not typically associated with a postoperative paralytic ileus.
Option D - Hypercalcemia: Hypercalcemia refers to a
high level of calcium in the blood and is not directly related to the condition shown in the X-ray.

Solution for Question 141:


Correct Option A - Gallstones:
• Gallstones are small, solid deposits that form in the gallbladder. They can vary in size and
composition.

Page 147

1018
• The USG findings of a thickened gallbladder and gallbladder coating biliary sludge are consistent with
chronic cholecystitis, which is inflammation of the gallbladder often associated with gallstones. The
acoustic shadow produced by the gallstones is a characteristic finding in ultrasound imaging.
• The patient's history of mild right hypochondriac pain, which is relieved by taking analgesics, is
suggestive of biliary colic.
• Biliary colic occurs when a gallstone obstructs the cystic duct or the common bile duct, leading to
intermittent episodes of pain.
• The presence of gallstones on the ultrasound, along with the history of pain, supports the diagnosis of
gallstones.
Gallstones are small, solid deposits that form in the gallbladder. They can vary in size and composition.
The USG findings of a thickened gallbladder and gallbladder coating biliary sludge are consistent with
chronic cholecystitis, which is inflammation of the gallbladder often associated with gallstones. The aco
ustic shadow produced by the gallstones is a characteristic finding in ultrasound imaging.
The patient's history of mild right hypochondriac pain, which is relieved by taking analgesics, is suggest
ive of biliary colic.
Biliary colic occurs when a
gallstone obstructs the cystic duct or the common bile duct, leading to intermittent episodes of pain.
The presence of gallstones on the ultrasound, along with the history of pain, supports the diagnosis of
gallstones.
Incorrect Options:
Option B - Amoebic liver abscess: Amoebic liver abscess is caused by an infection with the parasite En
tamoeba histolytica. It typically presents with symptoms such as fever, right upper quadrant pain, and a
history of travel to endemic areas. The patient in this case does not exhibit symptoms consistent with a
n amoebic liver abscess.
Option C - Hydatid cyst: Hydatid cysts are caused by the larval stage of the tapeworm Echinococcus. T
hey usually present with symptoms such as abdominal pain, hepatomegaly (enlarged liver), and can be
associated with a
history of exposure to dogs or livestock. The patient's symptoms and history are not suggestive of a
hydatid cyst.
Option D - Acute cholecystitis: Acute cholecystitis is an inflammation of the gallbladder often caused by
gallstones. It typically presents with severe right upper quadrant pain, fever, and signs of inflammation.
The patient in this case is asymptomatic and does not exhibit signs consistent with acute cholecystitis.

Solution for Question 142:


Correct Option A - Most of the grafts are multi-visceral grafts:
• Intestinal transplantation involves the transplantation of the small intestine alone in most cases, rather
than multi-visceral grafts. Multi-visceral grafts may include the small intestine along with other organs
like the stomach, pancreas, and liver, but they are less common.
Intestinal transplantation involves the transplantation of the small intestine alone in most cases, rather t
han multi-visceral grafts. Multi-visceral grafts may include the small intestine along with other organs lik
e the stomach, pancreas, and liver, but they are less common.
Incorrect Options:

Page 148

1019
Option B - It is not widely used due to vigorous rejection reactions: Intestinal transplantation is a compl
ex and challenging procedure, and one reason it is not widely used is the high risk of rejection. The im
mune system's response to the transplanted intestine can be quite vigorous, leading to rejection of the
graft.
Option C - The most dangerous complication is sepsis: Sepsis, a severe infection that spreads through
out the body, is one of the most dangerous complications following intestinal transplantation. Infections
can occur due to the immunosuppressive medications used to prevent rejection and the disruption of i
ntestinal barrier function.
Option D - The most common indication is ischemia: While intestinal ischemia (reduced blood flow to th
e intestine) can cause intestinal failure and be a possible indication for transplantation, it is not the mos
t common indication. The most common indications for intestinal transplantation include short bowel sy
ndrome, which can result from surgical resection or congenital malformations, and severe functional ga
strointestinal disorders.

Solution for Question 143:


Correct Option B - Angle between SMA and aorta is more than 70 degrees:
• Superior mesenteric artery (SMA) syndrome, also known as Wilkie's syndrome, is a rare condition
characterized by compression of the third part of the duodenum (not the entire duodenum) between the
superior mesenteric artery and the abdominal aorta. However, the angle between the SMA and the
aorta is typically less than 20 degrees, not more than 70 degrees as mentioned in option B.
Superior mesenteric artery (SMA) syndrome, also known as Wilkie's syndrome, is a rare condition char
acterized by compression of the third part of the duodenum (not the entire duodenum) between the sup
erior mesenteric artery and the abdominal aorta. However, the angle between the SMA and the aorta is
typically less than 20 degrees, not more than 70 degrees as mentioned in option B.
Incorrect Options:
Option A - Third part of the duodenum is compressed: The duodenum, specifically the third part, is com
pressed between the SMA and the aorta, leading to symptoms such as abdominal pain, nausea, and v
omiting.
Option C - Surgical management is by duodenojejunostomy: Surgical intervention, specifically duodeno
jejunostomy, is often required to relieve the compression and establish a
bypass between the duodenum and the jejunum.
Option D - Also known as Wilkie's Syndrome: Superior mesenteric artery syndrome is also known as W
ilkie's syndrome, named after the doctor who first described it.

Solution for Question 144:


Correct Option A - Hemorrhage:
• The most common immediate complication of splenectomy (removal of the spleen) is hemorrhage,
which refers to excessive bleeding. The spleen is a highly vascular organ, and its removal can lead to
significant bleeding. Surgeons take precautions to control bleeding during the procedure, but it remains
a potential complication.

Page 149

1020
The most common immediate complication of splenectomy (removal of the spleen) is hemorrhage, whi
ch refers to excessive bleeding. The spleen is a highly vascular organ, and its removal can lead to sign
ificant bleeding. Surgeons take precautions to control bleeding during the procedure, but it remains a
potential complication.
Incorrect Options:
Option B - Fistula: While a fistula (an abnormal connection between two organs or tissues) can occur a
fter splenectomy, it is not the most common immediate complication.
Option C - Bleeding from gastric mucosa: Bleeding from the gastric mucosa can occur in some cases,
but it is not the most common immediate complication after splenectomy.
Option D - Pancreatitis: Pancreatitis, inflammation of the pancreas, is not a
common immediate complication of splenectomy.

Solution for Question 145:


Correct Option B - Serology:
• Given the clinical presentation of a 6-year-old child with high fever, rigors, pain in the right
hypochondrium, tenderness in the right upper quadrant, and anicteric appearance, the most
appropriate investigation would be serology. Serological tests can help identify the presence of specific
antibodies or antigens in the blood, which can aid in the diagnosis of various infectious diseases.
• Serological tests can be used to detect specific pathogens, such as viral or bacterial infections, that
may be causing the symptoms. These tests can include antibody tests, antigen tests, or tests for
specific biomarkers associated with certain infections.
Incorrect Options:
Option A - USG: Ultrasonography, may be helpful in evaluating the abdominal organs and detecting an
y abnormalities, but it may not provide specific information about the underlying cause of the symptoms
.
Option C - SGOT/LFT: Serum glutamic-oxaloacetic transaminase/liver function tests may be part of a
comprehensive evaluation, but they do not specifically target the cause of the symptoms described.
Option D - CECT: Contrast-enhanced computed tomography is a useful imaging modality for assessing
abdominal structures in detail, but it may not be the initial investigation of choice in this case.

Solution for Question 146:


Correct Option A - Chemoradiation:
• The best treatment strategy for carcinoma of the anal canal is chemoradiation.
• Carcinoma of the anal canal is typically treated with a combination of chemotherapy and radiation
therapy, known as chemoradiation. This approach has been shown to be highly effective in achieving
local tumor control and preserving anal function. The chemotherapy drugs commonly used in this
regimen include 5-fluorouracil (5-FU) and mitomycin C.
• The rationale behind using chemoradiation is that the anal canal is a highly radiosensitive area, and
the addition of chemotherapy enhances the radio sensitivity of the tumor cells, leading to improved

Page 150

1021
treatment outcomes. Chemoradiation is considered the primary treatment modality for anal canal
carcinoma, especially in early-stage tumors.
The best treatment strategy for carcinoma of the anal canal is chemoradiation.
Carcinoma of the anal canal is typically treated with a combination of chemotherapy and radiation thera
py, known as chemoradiation. This approach has been shown to be highly effective in achieving local t
umor control and preserving anal function. The chemotherapy drugs commonly used in this regimen in
clude 5-fluorouracil (5-FU) and mitomycin C.
The rationale behind using chemoradiation is that the anal canal is a highly radiosensitive area, and th
e addition of chemotherapy enhances the radio sensitivity of the tumor cells, leading to improved treat
ment outcomes. Chemoradiation is considered the primary treatment modality for anal canal carcinoma
, especially in early-stage tumors.
Incorrect Options:
Option B - Surgery: This may be considered in certain cases, such as locally advanced tumors that do
not respond adequately to chemoradiation or in cases where the tumor is not amenable to radiation the
rapy. Depending on the extent and location of the tumor, surgical options may include local excision, wi
de local excision, or abdominoperineal resection (APR).
Option C - Radiation therapy: It is a key component of the treatment for anal canal carcinoma, but it is t
ypically combined with chemotherapy (chemoradiation) for optimal outcomes. Radiation therapy alone
may be considered in select cases, such as in patients who are not candidates for chemotherapy or in
cases of localized tumors in patients with significant comorbidities.
Option D - Chemotherapy: It is an important component of the treatment for anal canal carcinoma, but i
t is typically used in conjunction with radiation therapy (chemoradiation). Chemotherapy alone is not th
e preferred treatment modality for this type of cancer.

Solution for Question 147:


Correct Option A - No recurrence after surgery:
• The description of the patient's symptoms and findings suggests a possible diagnosis of Crohn's
disease, which is a chronic inflammatory bowel disease. Crohn's disease can affect any part of the
gastrointestinal tract, but it most commonly involves the terminal ileum and colon.
• Crohn's disease is characterized by the presence of multiple strictures (narrowing) of the affected
bowel segments. These strictures can lead to symptoms such as abdominal pain, diarrhea, and weight
loss. The strictures are often interspersed with areas of normal mucosa, which is a characteristic
feature of the disease and can reoccur after surgery.
The description of the patient's symptoms and findings suggests a
possible diagnosis of Crohn's disease, which is a chronic inflammatory bowel disease. Crohn's disease
can affect any part of the gastrointestinal tract, but it most commonly involves the terminal ileum and c
olon.
Crohn's disease is characterized by the presence of multiple strictures (narrowing) of the affected bowe
l segments. These strictures can lead to symptoms such as abdominal pain, diarrhea, and weight loss.
The strictures are often interspersed with areas of normal mucosa, which is a
characteristic feature of the disease and can reoccur after surgery.
Incorrect Options:

Page 151

1022
Option B - Aphthous ulcers: Crohn's disease can cause aphthous ulcers, which are small, shallow, pai
nful sores that can develop in the mouth, esophagus, or gastrointestinal tract.
Option C - Skip lesions: Crohn's disease is known for its patchy distribution, with areas of inflammation
(affected by the disease) and areas of normal tissue (unaffected by the disease) interspersed. This patt
ern of skip lesions is a distinguishing feature of Crohn's disease.
Option D - Fistula formation: Crohn's disease can lead to the formation of abnormal connections (fistula
s) between different parts of the gastrointestinal tract or between the gastrointestinal tract and other or
gans. Fistulas can cause additional complications and symptoms.

Solution for Question 148:


Correct Option D:
• In a patient with a four-day history of right iliac fossa pain, the presence of the following signs and
symptoms would warrant urgent surgical intervention.
Incorrect Options:
Option A: Rising pulse rate: An increasing or persistently elevated pulse rate can be indicative of ongoi
ng inflammation, infection, or compromised blood flow to the affected area. It may suggest a
worsening condition that requires immediate surgical evaluation and intervention.
Option C: Increasing abdominal pain: If the abdominal pain is progressively worsening, it may indicate
a worsening or evolving pathology such as an inflamed appendix or an abscess. Increasing pain can b
e a sign of a worsening condition that may require surgical exploration.
Option D: Increase in the size of the mass: If there is a palpable mass in the right iliac fossa that is incr
easing in size, it could suggest an enlarging abscess or a rapidly progressing inflammatory process. Th
is may necessitate urgent surgical drainage or intervention.

Solution for Question 149:


Correct Option D:
• The X-ray image shows colocolonic intussusception, it means that a part of the colon has telescoped
into an adjacent part of the colon. Colocolonic intussusception is a relatively rare condition and can
occur in any age group.
• The diagnosis of colocolonic intussusception on an X-ray image would typically show signs of bowel
obstruction, such as a dilated loop of the colon or evidence of air-fluid levels. The telescoping of the
colon may appear as a target or "bull's eye" appearance on the X-ray.
• Colocolonic intussusception can cause symptoms such as abdominal pain, cramping, vomiting, and
blood in the stool. Treatment usually involves reducing the intussusception either by air or liquid enema
or through surgical intervention.
Incorrect Options:
Option A. Ileocolic intussusception: Ileocolic intussusception refers to the telescoping of the small intest
ine (ileum) into the large intestine (colon). It is a distinct condition from colocolonic intussusception. Th
erefore, if the X-ray image shows colocolonic intussusception, it cannot be diagnosed as ileocolic intus
susception.

Page 152

1023
Option B. Sigmoid volvulus: Sigmoid volvulus is a condition where the sigmoid colon twists upon itself, l
eading to bowel obstruction. It is characterized by a "coffee bean" or "omega" sign on imaging. The ter
m "colocolonic intussusception" does not describe sigmoid volvulus, so it is an incorrect option.
Option C. Toxic megacolon: Toxic megacolon refers to a severe complication of inflammatory bowel di
sease, usually ulcerative colitis, where the colon becomes dilated and at risk of perforation. It is not dir
ectly related to intussusception. Therefore, toxic megacolon is an incorrect option for the given X-ray i
mage of colocolonic intussusception.

Solution for Question 150:


Correct Option B - Colonic polyp:
• Colonic polyps are abnormal growths that develop on the inner lining of the colon or rectum. They can
vary in size and shape and may be noncancerous (benign) or have the potential to develop into colon
cancer (premalignant). In the given scenario, the presence of multiple superficial ulcers in the rectum
and colon suggests an inflammatory bowel disease (such as ulcerative colitis or Crohn's disease) as
the underlying condition.
• However, it is important to monitor and manage colonic polyps appropriately due to their potential to
progress to colon cancer over time.
Colonic polyps are abnormal growths that develop on the inner lining of the colon or rectum. They can
vary in size and shape and may be noncancerous (benign) or have the potential to develop into colon c
ancer (premalignant). In the given scenario, the presence of multiple superficial ulcers in the rectum an
d colon suggests an inflammatory bowel disease (such as ulcerative colitis or Crohn's disease) as the
underlying condition.
However, it is important to monitor and manage colonic polyps appropriately due to their potential to pr
ogress to colon cancer over time.
Incorrect Options:
Options A - Toxic megacolon: Toxic megacolon is a severe complication of inflammatory bowel diseas
e characterized by massive inflammation and dilation of the colon. It is considered a
medical emergency and can lead to life-threatening complications such as bowel perforation.
Options C - Colonic obstruction: Colonic obstruction refers to a blockage in the colon that prevents the
normal passage of stool. It can occur due to various causes, such as tumors, strictures, or adhesions.
Colonic obstruction typically presents with symptoms of abdominal pain, distention, and altered bowel
movements.
Options D - Massive colonic bleeding: Massive colonic bleeding refers to significant and potentially life-
threatening bleeding from the colon. It can occur due to various causes, including ulcers, tumors, divert
iculosis, or vascular abnormalities. Massive colonic bleeding may present with symptoms such as brigh
t red or dark stools, anemia, and hemodynamic instability.

Solution for Question 151:


Correct Option D - Pancreatitis:
• Pancreatitis is the most common complication of ERCP.

Page 153

1024
• Incidence- 4.3%
• Although mild, post-ERCP pancreatitis can be severe with extensive pancreatic necrosis.
• Risk factors for post-ERCP pancreatitis. Suspected sphincter of Oddi dysfunction Young age Normal
bilirubin Prior ERCP-related pancreatitis Difficult cannulation Pancreatic sphincterotomy Balloon
dilatation of biliary sphincter.
• Suspected sphincter of Oddi dysfunction
• Young age
• Normal bilirubin
• Prior ERCP-related pancreatitis
• Difficult cannulation
• Pancreatic sphincterotomy
• Balloon dilatation of biliary sphincter.
• Suspected sphincter of Oddi dysfunction
• Young age
• Normal bilirubin
• Prior ERCP-related pancreatitis
• Difficult cannulation
• Pancreatic sphincterotomy
• Balloon dilatation of biliary sphincter.
Incorrect Options:
Option A - Duodenal perforation:
• Duodenal perforation occurs in 1.3% of the cases
Option B - Cholangitis:
• Cholangitis is not an ERCP-specific complication
Option C - Hemorrhage:
• Hemorrhage occurs in 1.4% of the cases

Solution for Question 152:


Correct Option B - Administration of 5% dextrose:
• Severe abdominal pain radiating to the back with elevated serum lipase and amylase values indicates
a diagnosis of acute pancreatitis.
• Administration of 5% dextrose is not indicated in the treatment of acute pancreatitis.
• Rehydration is essential as there is a lot of fluid sequestration and 3rd space fluid loss.
• Aggressive early rehydration in the first 24 hours using 400 ml/hr crystalloids (Ringer lactate, normal
saline) to achieve rapid repletion of the severe volume depletion.

Page 154

1025
Incorrect Options:
Option A - Nil per oral to avoid pancreatic stimulation:
• Nil per oral to avoid pancreatic stimulation is indicated for a short duration but there is no physiological
justification for prolonged NPO.
Option C - Parenteral nutrition is preferred for 2-3 days over the nil per oral (NPO) approach:
• Enteral nutrition is preferred over TPN (Total parenteral nutrition) due to fewer complications
Option D - Early initiation of oral feedings:
• Early initiation of oral feedings once the patient's condition improves.

Solution for Question 153:


Correct Option A - TPN:
• Puestow procedure is longitudinal pancreaticojejunostomy.
• The most preferred route for supplementary nutrition in chronic pancreatitis following Puestow
procedure is TPN.
• Post-op care in chronic pancreatitis Nutrition-TPN Fluid and electrolyte management
Prevention/control of sepsis; Proper monitoring Octreotide on the table and postoperatively-regular
intervals or slow infusion-5 days
• Nutrition-TPN
• Fluid and electrolyte management
• Prevention/control of sepsis; Proper monitoring
• Octreotide on the table and postoperatively-regular intervals or slow infusion-5 days
• Nutrition-TPN
• Fluid and electrolyte management
• Prevention/control of sepsis; Proper monitoring
• Octreotide on the table and postoperatively-regular intervals or slow infusion-5 days
Incorrect Options:
Option B - Oral feeding:
• Oral feeding is not advised in post-op care of chronic pancreatitis patient
Option C - Feeding gastrostomy:
• Feeding gastrostomy is not advised in post-op care of chronic pancreatitis patient
Option D - Feeding jejunostomy:
• Feeding jejunostomy is not advised in post-op care of chronic pancreatitis patient

Page 155

1026
Kidney and Ureter
1. A 7-year-old with a history of bed wetting and recurrent urinary tract infections is brought with
complaints of left-sided abdominal pain that intensifies on urination. She has a fever of 101 F°. Left
costovertebral angle tenderness is present. IV pyelogram is conducted, and urinary tract infection is
suspected. How would this condition be treated?
(or)
How is duplication of ureter treated in a 7 year old girl ?

A. If the function of the upper pole ureter is normal, then no intervention is needed, and for the lower
pole ureter, ureterovesicoplasty can be done
B. Both the poles should be excised and reimplanted into the bladder as the patient may become
susceptible to UTI more frequently
C. For upper pole hydronephrosis with no function, an Upper pole nephrectomy should be performed.
D. The lower pole ureter is prone to develop Ureterocele, so it should be excised in any case
----------------------------------------
2. A young primigravida presents fever and right flank pain. Costovertebral angle tenderness is present
on her left. Laboratory studies show a leukocyte count of 14000/mm3 and Hb count of 15 g/dll. Urine
analysis reveals 3+ nitrates and numerous WBC casts. What is the next step in management?
(or)
A 28-year-old primigravida female at 30 weeks gestation comes to an emergency with a 2-day history
of fever and right flank pain. She has costovertebral angle tenderness on her left and a burning
sensation during urination. Laboratory studies show a leukocyte count of 14000/mm3 and Hb count of
15g/dl. Urine analysis shows blood along with, Nitrates 3+ Protein 2+ WBC 700/hpf RBC 10/hpf WBC
casts numerous What is the most appropriate next step in the management of this patient?
A. CT scan
B. Ultrasound
C. Third-generation cephalosporin
D. Fluoroquinolones
----------------------------------------
3. A 12-year-old girl has been experiencing painful micturition and increased frequency for a few
months, with recurrent episodes since childhood. Urinalysis reveals few pus cells and an abdominal
ultrasound shows a scarred kidney. What is the most common congenital disorder that predisposes to

1027
this condition?
(or)
What is the most common congenital disorder that predisposes to chronic pyelonephritis?
A. Diabetes mellitus
B. Renal stone
C. Posterior urethral valve
D. Vesicoureteric reflux
----------------------------------------
4. A female patient complains of continuous urine dribbling despite normal voiding. Tests show normal
kidney size, mild dilation of the left ureter, and pooling of contrast into the vagina. what is true regarding
diagnosis?
(or)
A 16-year-old female presents to the surgical OPD with complaints of continuous dribbling of urine
along with normal voiding since childhood. There is no evidence of lower urinary tract symptoms, stress
incontinence, or urinary tract infection. She is afebrile, with a blood pressure of 120/80 mmHg and a
regular pulse rate of 80 beats per minute. Her abdominal examination reveals no abnormal findings;
however, pelvic examination reveals urine leakage from vagina. Her lab reports, abdominal ultrasound,
and IVU revealed no abnormal findings except that the left lower ureter was dilated (7mm) and the
pooling of contrast into the vagina. Which of the following is true about the diagnosis of this case?
A. Around 30% is associated with a duplication collecting system
B. The most common presentation in males is urinary incontinence, not UTI
C. This condition is more common in females.
D. Around 100% is associated with a duplication collecting system
----------------------------------------
5. A 4-year-old boy who is the second child in his family is brought to the emergency department with
complaints of blood in urine with clots and a history of crying at each time of urination. The child's birth
and developmental history are uneventful. On examination, the abdomen was soft and not tender, but
at the right side of the abdominal wall, a mass is palpated, which is freely movable with smooth edges
and did not extend over the midline. CT scan showed a solid mass with cystic areas in the lower and
upper quadrants arising from the right kidney. The hematologic assessment showed moderate
microcytic anaemia. What is the most probable diagnosis?
(or)
A boy is brought with complaints of hematuria and a history of crying with urination. On abdominal
examination, a non-tender mass is palpable on the right side that did not extend over the midline. The
CT scan showed a solid mass with cystic areas in the lower and upper quadrants arising from the right
kidney. The hematologic assessment showed moderate microcytic anaemia. What is the diagnosis?
A. Neuroblastoma
B. Angiomyolipoma
C. Wilms tumor
D. Grawitz tumor
----------------------------------------

Page 2

1028
6. A 72-year-old male patient with a rectal abscess, polyuria, abnormal creatinine level, and glomerular
filtration rate, and a family history of polycystic kidney disease presents to the surgical outpatient
department. The patient's vital signs are normal, and there are no other pertinent symptoms. An
intravenous pyelogram image is attached below. What is the diagnosis?
(or)
An elderly male patient presents with complaints of polyuria, rectal pain, and a rectal abscess. He has
mild leukocytosis and an abnormal creatinine level and glomerular filtration rate. The patient has a
positive family history of polycystic kidney disease. The patient's intravenous pyelogram image is given
below. What is the diagnosis?

A. Urothelial carcinoma
B. Urolithiasis
C. Autosomal Dominant Polycystic Kidney Disease
D. Hydronephrosis
----------------------------------------
7. A 9-year-old girl is brought to the surgical outpatient department with complaints of burning
micturition and fever. She has a history of recurrent UTIs, which were successfully treated with
antibiotics on every occasion. The intravenous urogram shows renal calyces pointing towards the
vertebrae and the patient's IVU image is shown below. What is the probable diagnosis of the patient's
condition mentioned above?
(or)
A girl child with a history of recurrent UTI since childhood presents with burning micturition and fever.
The intravenous urogram shows renal calyces pointing toward the vertebrae. What is the diagnosis?

A. Renal cell carcinoma

Page 3

1029
B. Hydronephrosis
C. Horseshoe kidney
D. Pyonephrosis
----------------------------------------
8. An elderly woman complains of high-grade fever, rigors and pain in her right loin. Her blood pressure
is 160/70 mmHg, with a bounding pulse of 90/minute with tenderness in her right loin. What is the
diagnosis?
(or)
A 60-year-old woman presents to the tertiary care hospital emergency department with a complaint of
high-grade fever, rigorsand pain in her right loin from the last six days. On examination, her blood
pressure is 160/70 mmHg, a bounding pulse of 90/minute, and tenderness in her right loin. What is the
most probable diagnosis for the patient mentioned above?
A. Pyonephrosis
B. Hydronephrosis
C. Urolithiasis
D. Polyuria
----------------------------------------
9. A 20-year-old male presents to the surgical emergency departmentwith a complaint of blood in urine
for last one hour. On examination, his blood pressure is 110/60mmHg, pulse rate is 110/minute, and
fullness in his left loin. On history, he reports that on the same day in the morning, he played rugby.
After that, he passed frank blood in his urine. He also feels mild discomfort in his left loin. What is the
diagnosis related to the above-mentioned patient's condition?
(or)
A young male presents with a complaint of blood in urine from the last hour. Earlier, he passed frank
blood in his urine after playing rugby. On examination, there is fullness in his left loin and mild
discomfort. What is the diagnosis?
A. Renal cell cancer
B. Adenocarcinoma
C. Congenital Hydronephrosis
D. Pyonephrosis
----------------------------------------
10. A 70-year-old male presents to the surgical outpatient department with complaints of painless,
profuse, and intermittent hematuria for the last six weeks. He also complains about burning micturition
and mild retropubic discomfort. He is a known smoker with a 20-pack per year history. Urine analysis
reveals blood mixed uniformly with urine and atypical masses of urothelial cells. What is the most
probable diagnosis for the above-mentioned patient?
(or)
A 70-year-old male presents with complaints of hematuria, burning micturition and retropubic
discomfort. He has a 20-pack-year smoking history. Urine analysis reveals blood mixed uniformly with
urine and atypical masses of urothelial cells. What is the diagnosis?
A. Bladder carcinoma

Page 4

1030
B. Adenocarcinoma
C. Ductal carcinoma
D. Squamous cell carcinoma
----------------------------------------
11. A 4-year-old girl presents to the emergency department with a complaint of persistent abdominal
pain for a month which is aggravated. She is febrile when examined physically, and a sensitive mass
on palpation is observed on the right side of the abdomen. Bowel sounds are present. Laboratory
studies show hematuria without proteinuria. An abdominal CT scan shows a 12-cm, circumscribed,
solid mass in the right kidney. A right-sided nephrectomy is performed. The figure depicts the gross
appearance of the mass. Which diagnosis is more likely?
(or)
A young girl presents with persistent abdominal pain. A mass is palpable on the right side of the
abdomen. Laboratory studies show hematuria without proteinuria. CT scan reveals a 12-cm,
circumscribed, solid mass in the right kidney. A right-sided nephrectomy is performed. What is the
diagnosis?

A. Angiomyolipoma
B. Interstitial cell tumor
C. Renal cell carcinoma
D. Wilms tumor
----------------------------------------
12. A 66-year-old woman complains of experiencing five transient ischemic attacks within a week. A
physical examination reveals a blood pressure reading of 150/95 mm Hg and a urine study shows 1+
proteinuria but no blood, glucose, or ketones. Abdominal ultrasonography shows slightly smaller
kidneys and high-magnification microscopic appearance of the kidneys indicates a kidney lesion.
(or)
An elderly woman complains of experiencing five transient ischemic attacks within a week. Blood
pressure is 150/95 mmHg, and urine analysis reveals 1+ proteinuria and few oxalate crystals.
High-magnification microscopic appearance of the kidneys is shown. What is the diagnosis?

Page 5

1031
A. Acute tubular necrosis
B. Fibromuscular dysplasia
C. Hyaline arteriolosclerosis
D. Interstitial nephritis
----------------------------------------
13. A 55-year-old man presents to the emergency department with a complaint of passing dark-colored
urine and back pain for the past month. On physical examination, there is tenderness over the right
costovertebral angle. His urine analysis report is shown below. Urine microscopic analysis reveals
many RBCs, few WBCs, and no casts or crystals. The kidney lesion's typical gross appearance is
depicted in the figure. Which one of the following components in this man's blood is most likely to be
elevated Urine Analysis Report pH 6 Specific Gravity 1.015 Hematuria 2+ Protein Nill Glucose Nill
Ketones Nill
(or)
A patient presents with a complaint of passing dark-coloured urine and back pain. On examination,
there is tenderness over the right costovertebral angle. The kidney lesion's typical gross appearance is
depicted in the figure. Which components in this man's blood are most likely to be elevated?
pH 6
Specific Gravity 1.015
Hematuria 2+
Protein Nill
Glucose Nill
Ketones Nill

A. Cortisol
B. Globulins
C. Erythropoietin
D. Vasopressin
----------------------------------------
14. A 5-year-old girl presents to the hospital's emergency department complaining of cramping
abdominal pain, diarrhea, and decreased urine output for the last 4 days. She had chili and ice cream
at a home barbecue; on examination, there were petechial hemorrhages on the skin. Lab tests showed
positive occult blood in stool, schistocytes in peripheral blood smear, and elevated serum D-dimer

Page 6

1032
level. Urinalysis showed 2+ hematuria with no protein or glucose.This complication develops most
commonly after infection with which of the following organisms? Laboratory Findings Serum Creatinine
Level 2.2mg/dL Urea Nitrogen Level 20mg/dL Haemoglobin 10.8g/dL Hematocrit 32.4% Platelet Count
64,300/mm3 WBC Count 6480/mm3
(or)
A young girl presents cramping abdominal pain, diarrhoea, and decreased urine output for 4 days after
eating hamburgers. On physical examination, there are petechial haemorrhages on the skin. The
peripheral blood smear shows schistocytes, and the serum D-dimer level is elevated and low platelet
count. Her renal biopsy specimen shows small thrombi within glomerular capillary loops. This
complication develops after infection with which organisms?
Serum Creatinine Level 2.2mg/dL
Urea Nitrogen Level 20mg/dL
Haemoglobin 10.8g/dL
Hematocrit 32.4%
Platelet Count 64,300/mm3
WBC Count 6480/mm3

A. Candida albicans
B. Clostridium difficile
C. Escherichia coli
D. Proteus mirabilis
----------------------------------------
15. Renal cell carcinoma invading the IVC and renal vein. What is true in the management and
evaluation of a patient with renal cell carcinoma invading the IVC and renal vein?
(or)
A 38-year-old male has an eight-month history of progressive abdominal swelling, gross hematuria with
flank pain, and a weight loss of 20 kg over two years. Physical examination reveals a palpable mass in
the left upper quadrant of the abdomen. Laboratory examination shows low hemoglobin levels, high
platelets, and alkaline phosphate. The patient was diagnosed with renal cell carcinoma invading the
inferior vena cava and renal vein through an abdominal CT scan.which of the following is true in
management and evaluation of this patient?
A. A pre-op biopsy is necessary
B. IVC involvement indicates inoperability
C. Pre-op radiotherapy is essential.
D. Chest X-ray could be done to rule out pulmonary metastasis
----------------------------------------
16. A 6-year-old girl has urinary incontinence since birth. During examination, it was found that she has
three separate openings in her genitalia. Further evaluation revealed that she has a single urethral
meatus, a single vaginal opening, and an ectopic ureteral opening to the right of the vagina. The right
ureter is mildly dilated, coursing posterior to the bladder neck, and ending at the level of the urethra
with contrast also seen in the vagina. Which structure prevents these kidneys from ascending?
(or)

Page 7

1033
Which structure prevents Horse-shoe kidneys from ascending?
A. Superior mesenteric artery
B. Superior mesenteric vein
C. Inferior mesenteric artery
D. Inferior mesenteric vein
----------------------------------------
17. A patient presents with severe abdominal pain radiating to his groin. Ultrasound reveals mild
hydronephrosis bilaterally, and a CT scan reveals bilateral ureterolithiasis. Which nerve carries the
referred pain from the ureteric colic to the groin?
(or)
A 30-year old man presents to the emergency department with extreme abdominal pain radiating to his
groin for the last one hour. The ultrasound examination revealed mild hydronephrosis bilaterally.
Non-enhanced computer tomography is then performed and shows a 10-mm hyperdense picture in the
left ureter topography and a 9-mm hyperdense image in the right ureter topography, consistent with a
diagnosis of bilateral ureterolithiasis. Given the clinical feature, which nerve carries the referred pain
from the ureteric colic to the groin?
A. Subcostal nerve
B. Iliohypogastric nerve
C. Ilioinguinal nerve
D. Genitofemoral nerve
----------------------------------------
18. A patient presents with colicky, intermittent, bilateral flank pain. No fever, haematuria, weight loss,
palpable flank mass or tenderness exists. X-ray KUB shows multiple left renal and bilateral lower
ureteral stones. What is the correct order of stones based on images of crystals?
(or)
A 20-year old boy presents to the emergency department with colicky, intermittent, bilateral flank pain
for the past 2 hours. There is no history of fever, hematuria or weight loss. There is no family history of
any urological pathology. On clinical examination, there is no palpable flank mass or tenderness. His
serum creatinine is 1.1 mg, serum sodium -141 mEq/L and serum potassium- 4.1 mEq/L. The urine
culture is sterile. X-ray KUB shows multiple left renal and bilateral lower ureteral stones. What is the
correct order of stones based on images of crystals?

Page 8

1034
A. Calcium oxalate, struvite, uric acid, cystine
B. Calcium oxalate, uric acid, struvite, cystine
C. Uric acid, calcium oxalate, struvite, cystine
D. Struvite, uric acid, calcium oxalate cystine
----------------------------------------
19. A 55-year old male presents to the emergency department with acute right flank pain for the past 3
hours. He was prescribed painkillers and discharged. The next day, he presented again with severe
pain and was referred to the urology department. A ureteric calculus associated with hydronephrosis is
identified on the ultrasound abdomen. He is managed with a procedure shown in the following image.
Which of the following is correct about the given procedure?
(or)
A 55-year-old male presents with acute right flank pain. A ureteric calculus associated with
hydronephrosis is identified on the ultrasound abdomen. Which of the following is correct about the

Page 9

1035
given procedure done?

A. Gold standard lithotripter is Dornier unmodified HM-3


B. Steinstrasse is seen in cases of proximal ureteric obstruction
C. Pregnancy is an indication
D. The maximum size of the stone removed is 1.5 cm
----------------------------------------
20. A 20-year-old woman presents with fever, anaemia, vomiting, and right flank pain despite treatment
with Ciprofloxacin, Examination reveals a palpable 6cm firm mass in the right flank. CT shows a right
kidney with a staghorn calculus, thickening of the renal pelvis wall, and inflammatory changes in
perirenal and pararenal adipose tissue. Urine culture shows Proteus Mirabilis sensitive to Ciprofloxacin.
what is the diagnosis?
(or)
A 20-year-old woman presents a history of persistent right flank pain, fever, anaemia, vomiting and
weight loss. On examination, a right flank mass is palpable. CT shows a right kidney with a staghorn
calculus, thickening of the renal pelvis wall, obliteration of its lumen and several multilocular cystic
formations. Important inflammatory changes of perirenal and pararenal adipose tissue. Urine culture
reveals Proteus Mirabilis. What is the most diagnosis?
A. TB kidney
B. Xanthogranulomatous pyelonephritis
C. Chronic pyelonephritis
D. Emphysematous pyelonephritis
----------------------------------------
21. A 2-year-old girl with a history of β-thalassemia minor is brought with intermittent diffuse abdominal
pain. On examination, she is pale looking with abdominal tenderness and a palpable mass in the left
flank. Blood tests show leukocytosis with neutrophilia. Her haemoglobin is 8.5 g/dL, platelets (724
000/mm3), CRP 3.8 mg/dL and lactate dehydrogenase 950 U/L. What is the best treatment for this
condition?
(or)
What is the best treatment for a young girl presenting with intermittent diffuse abdominal pain, palpable
mass in the left flank, anemia, and leucocytosis with neutrophilia?
A. Percutaneous drainage under CT guidance
B. Open surgical drainage

Page 10

1036
C. Nephrectomy
D. Observation & follow up
----------------------------------------
22. A 40-year-old woman has come to the emergency department with a mass in her vulva that is
causing urinary retention. She has been experiencing mild left flank pain and lower urinary tract
symptoms, but has not seen a doctor about them. She has no other relevant medical history or
comorbid conditions. During a local examination, an interlabial mass is found emerging from the
meatus. The mass is firm in consistency and irreducible. On abdominal examination, a suprapubic
bulge is discovered. An intravenous pyelogram is performed, and the results are given below. What is
the best treatment option for this . What is the preferred treatment option for this case?
(or)
A 40-year-old woman presents with a vulval mass associated with urinary retention and mild left flank
pain. Examination reveals an irreducible interlabial mass emerging from the meatus and a suprapubic
bulge on abdominal examination. An intravenous pyelogram is performed. What is the treatment for this
condition?

A. Endoscopic incision
B. Nephrectomy with partial ureterectomy
C. STING operation
D. Lich-Gregoir technique
----------------------------------------
23. A 44-year-old female patient presents a progressive bloating sensation in the abdomen. She has a
family history of tuberous sclerosis. Ultrasound reveals a 10cm hyperechoic mass in the left abdominal
area, originating from the left kidney. CT examination reveals a mass with mixed density.
Immunohistochemistry is done after a mass biopsy, which is positive for HMB-45. What is the
diagnosis?
(or)
A middle aged female patient presents a progressive bloating sensation in the abdomen. She has a
family history of tuberous sclerosis. Ultrasound reveals a 10cm hyperechoic mass in the left abdominal
area, originating from the left kidney. CT examination reveals a mass with mixed density.
Immunohistochemistry is done after a mass biopsy, which is positive for HMB-45. What is the
diagnosis?
A. Secondary to malignant melanoma
B. Renal cell carcinoma

Page 11

1037
C. Angiomyolipoma
D. Carcinoma of the renal pelvis
----------------------------------------
24. A young child was brought to the hospital with complaints of abdominal mass, pink urine, and fever.
On CT, crescent-shaped, discrete & peripheral calcification was seen in the left kidney. Which of the
following chemotherapy regimens is given in this condition?
(or)
A 4-year-old child was brought to the hospital with complaints of abdominal mass, pink urine, and fever.
On CT, crescent-shaped, discrete & peripheral calcification was seen in the left kidney. Which of the
following chemotherapy regimens is given in this condition?
A. FOLFOX
B. ECF
C. CAF
D. VCD
----------------------------------------
25. A 25-year-old man presents to the surgical outpatient department with dull, intermittent right flank
pain of 1-year duration. Clinical examination and laboratory investigations are normal. Abdominal
ultrasound reveals right hydronephrosis. The following finding is seen on intravenous pyelogram. The
persistence of which of the following veins causes the development of the condition shown in the image
below?
(or)
A young man presenting with dull, intermittent right flank pain reveals right hydronephrosis on
ultrasound. The intravenous pyelogram image of the patient is given below. The persistence of which
vein causes the development of the condition?

A. Azygous vein
B. Hemiazygos
C. Anterior cardinal vein
D. Posterior cardinal vein
----------------------------------------
26. A 60-year-old male presents to the surgical outpatient department with fatigue, weight loss, and
persistent pain in the right side of the abdomen for the last six months. An Intravenous pyelogram

Page 12

1038
reveals a mass in the right kidney, which was later found to be renal cell carcinoma. Which of the
following is part of the Too Late triad associated with renal cell carcinoma?
(or)
Which of the following is part of the Too Late triad associated with renal cell carcinoma?
A. Polyuria
B. Pyrexia
C. Hematuria
D. Anaemia
----------------------------------------
27. A 45-year-old male presents to the surgical emergency department with burning micturition, urinary
frequency and blood in urine for the last four months. There are pus cells on examination. Urine culture
comes negative (sterile pyuria). After a few hours patient deteriorated suddenly and died due to renal
failure. Gross autopsy pictures of the kidneys are shown below. A family history of renal disease is
present. What is the most probable diagnosis associated with the above condition?
(or)
A 45-year-old male presents with burning micturition, urinary frequency and blood in the urine. Urine
culture is negative (sterile pyuria) with pus cells on examination. The patient died due to renal failure.
The gross autopsy picture of the kidney is shown below. A family history of renal disease is present.
What is the diagnosis?

A. Renal cell cancer


B. Renal tuberculosis
C. Cystic renal disease
D. Metastasis
----------------------------------------
28. A 35-year-old woman with hypertension and diabetes presented with fever and acute left lower
quadrant abdominal pain. CT scan showed acute diverticulitis. She was started on antibiotics. The next
day, her serum creatinine level was 2.0 mg/dl (increased from 0.8 mg/dl on the initial visit) and
urinalysis showed a muddy brown cast. What is the most likely diagnosis?
(or)
A 35-year-old woman presents with fever, and acute onset left lower quadrant abdominal pain. She is
started on broad-spectrum antibiotics. Serum creatinine (Cr) performed the next day shows a Cr level
of 2.0 mg/dl, which was 0.8 mg/dl on the initial visit. Urinalysis reveals a muddy brown cast. What is the

Page 13

1039
diagnosis?
A. Acute tubular necrosis
B. Acute interstitial nephritis
C. Acute pyelonephritis
D. Renal papillary necrosis
----------------------------------------
29. What is the diagnosis in a middle aged female presenting with intermittent massive bleeding from
urethra with Urine cytology revealing fresh red blood cells with masses of atypical urothelial cells ?
(or)
A 50-year-old female presents with massive bleeding from the urethra over the last two years. She has
a history of cesarean section and is a smoker. The patient is also experiencing fatigue and right
hydronephrosis up to the upper third of the right ureter in a nonfunctioning kidney is detected. Urine
cytology reveals fresh red blood cells with masses of atypical urothelial cells. What is the most accurate
diagnosis associated with the patient's condition?
A. Transitional cell carcinoma
B. Squamous cell carcinoma
C. Adenocarcinoma
D. Ductal carcinoma
----------------------------------------
30. A 40-year-old lady presents with 4th episode of urinary discomfort with flank pain. She has a history
of TB. Urinalysis shows hematuria, sterile pyuria and traces of protein. CT shows dilated ureters and
calcification around the bladder wall, and fibrosis. Cystoscopy reveals the abnormal characterization of
the ureteral orifice as shown in the exhibit What is the cause of this patient's symptoms?
(or)
A 40-year-old woman with a history of TB and benign cancer presents with recurrent urinary discomfort
and flank pain. Examination reveals a tender abdomen, and urinalysis indicates hematuria, sterile
pyuria, and protein traces. CT reveals dilated ureters, bladder wall calcification, and fibrosis. Urine
cultures are negative. Cystoscopy reveals the abnormal characterization of the ureteral orifice as
shown in the exhibit . What is the most likely cause of this patient's symptoms?

A. Genitourinary Tuberculosis
B. Ureterocele

Page 14

1040
C. Vesicoureteral reflux
D. Carcinoma of the Bladder
----------------------------------------

Correct Answers
Question Correct Answer

Question 1 3
Question 2 3
Question 3 4
Question 4 3
Question 5 3
Question 6 3
Question 7 3
Question 8 1
Question 9 3
Question 10 1
Question 11 4
Question 12 3
Question 13 3
Question 14 3
Question 15 4
Question 16 3
Question 17 2
Question 18 2
Question 19 1
Question 20 2
Question 21 1
Question 22 1
Question 23 3
Question 24 4
Question 25 4
Question 26 3
Question 27 2
Question 28 1
Question 29 1

Page 15

1041
Question 30 1

Solution for Question 1:


Option C
- For upper pole hydronephrosis with no function, an Upper pole nephrectomy should be performed:
• In the above-mentioned clinical scenario, the patient is a 7-year-old female child who has presented
with left-sided abdominal pain, fever and bed wetting. She has had a recurrent UTI since childhood,
and her examination and imaging results reveals duplication of her left ureter.
• Duplication of the ureter (AD) Most Common congenital anomaly of the upper urinary tract. The mode
of inheritance is autosomal dominant. More common in females. Unilateral duplication is more common
than bilateral. The left side is more commonly involved. Yo-Yo effect in the fused ureter (incomplete
duplication) is seen, Yo-Yo reflux is the reflux of contrast in both limbs of the ureter on retrograde
pyelogram in complete duplication. Complete duplication occurs when there are two separate ureteric
buds. In complete duplication, the upper pole Ureteral orifice in the bladder is caudal and medial to the
lower pole of the Ureteral orifice (Weigert Meyer rule). Upper pole ureter is associated with Ureterocele,
Lower pole ureter is associated with Short intravesical length of the ureter( risk of VUR).
• Most Common congenital anomaly of the upper urinary tract.
• The mode of inheritance is autosomal dominant.
• More common in females.
• Unilateral duplication is more common than bilateral.
• The left side is more commonly involved.
• Yo-Yo effect in the fused ureter (incomplete duplication) is seen, Yo-Yo reflux is the reflux of contrast
in both limbs of the ureter on retrograde pyelogram in complete duplication.
• Complete duplication occurs when there are two separate ureteric buds.
• In complete duplication, the upper pole Ureteral orifice in the bladder is caudal and medial to the lower
pole of the Ureteral orifice (Weigert Meyer rule).
• Upper pole ureter is associated with Ureterocele, Lower pole ureter is associated with Short
intravesical length of the ureter( risk of VUR).
• Diagnosis: IVP shows duplication in most cases. which is aka drooping lily sign.
• IVP shows duplication in most cases.
• which is aka drooping lily sign.
• Treatment: If the function of the Upper pole ureter is normal- Excision and reimplantation into the
bladder. If the function of the Upper pole ureter is impaired- an upper pole nephrectomy is done. Lower
pole ureter- Ureterovesicoplasty is done.
• If the function of the Upper pole ureter is normal- Excision and reimplantation into the bladder.
• If the function of the Upper pole ureter is impaired- an upper pole nephrectomy is done.
• Lower pole ureter- Ureterovesicoplasty is done.
• Most Common congenital anomaly of the upper urinary tract.
• The mode of inheritance is autosomal dominant.
• More common in females.

Page 16

1042
• Unilateral duplication is more common than bilateral.
• The left side is more commonly involved.
• Yo-Yo effect in the fused ureter (incomplete duplication) is seen, Yo-Yo reflux is the reflux of contrast
in both limbs of the ureter on retrograde pyelogram in complete duplication.
• Complete duplication occurs when there are two separate ureteric buds.
• In complete duplication, the upper pole Ureteral orifice in the bladder is caudal and medial to the lower
pole of the Ureteral orifice (Weigert Meyer rule).
• Upper pole ureter is associated with Ureterocele, Lower pole ureter is associated with Short
intravesical length of the ureter( risk of VUR).
• IVP shows duplication in most cases.
• which is aka drooping lily sign.
• If the function of the Upper pole ureter is normal- Excision and reimplantation into the bladder.
• If the function of the Upper pole ureter is impaired- an upper pole nephrectomy is done.
• Lower pole ureter- Ureterovesicoplasty is done.
Incorrect Options:
Option A - If the function of the upper pole ureter is normal, then no intervention is needed, and for the l
ower pole ureter, ureterovesicoplasty can be done:
• In case the function of the entire duplicate ureter is intact, with no symptoms, no intervention is
required.
Option B - Both the poles should be Excised and reimplanted into the bladder as the patient may beco
me susceptible to UTI more common:
• There is no need to excise both poles.
• Only the dysfunctional part needs to be excised and treated.
Option D - The lower pole ureter is prone to develop Ureterocele, so it should be excised:
• The lower pole should only be excised and treated if any pathology is present.

Solution for Question 2:


Option C: Third-generation cephalosporin
• This is a case of pyelonephritis, which presents with fever, rigors, flank pain, nausea and vomiting,
costovertebral angle tenderness, and some features like cystitis (dysuria and urinary frequency).
• Urine analysis will show numerous WBCs casts, WBCs and nitrates.
• Pregnant women are at increased risk of complications with pyelonephritis, so hospitalization and
treatment with IV third-generation cephalosporins are considered best.
• The next step is to switch to oral antibiotic therapy after accurate culture results are obtained. The
patient can be discharged with oral antibiotics if the patient is asymptomatic for 24-48 hours.
• The risk of pyelonephritis increases in pregnancy due to gestational diabetes and high glucose in
urine due to increased threshold and progesterone-related spacing in ureters and less commonly due
to obstruction.

Page 17

1043
Option A: CT scan
• CT scan can be performed in pyelonephritis patients with an increased risk of abscess and obstruction
due to casts or inpatients that do not respond to any treatment.
• CT scan can also be performed for a patient who has a structural abnormality in the ureter.
• However, CT scan is mostly avoided in pregnant women due to harm to the fetus.
Option B: Ultrasound
• Ultrasound is not the gold standard in pyelonephritis. Still, it can be performed in the same condition
where a CT scan is recommended, as in immunocompromised, structural abnormality in kidneys and in
a patient who does not respond to antibiotic therapy within 24-48 hours.
Option D: Fluoroquinolones
• Fluoroquinolones are recommended for acute pyelonephritis infection in non-pregnant patients.
• Pregnant patients should not be given Fluoroquinolones as they can cause serious harm to the fetus
due to side effects on cartilage.

Solution for Question 3:


Option D: Vesicoureteric reflux
• It indicates a chronic urinary tract infection. In the clinical scenario, the patient is a 12 –year-old
female child who has presented with dysuria and increased frequency. These episodes have been
recurrent since her childhood. The most common condition predisposing to UTI is Vesicoureteric reflux.
• Chronic pyelonephritis is often associated with vesicoureteric reflux, which some feel is better named
"Reflux nephropathy".
• Vesicoureteral reflux is backflowing of urine from the bladder to the ureters and in worst cases, up to
the kidneys.
• Symptoms: A persistent urge to urinate Burning micturition Cloudy urine Fever Pain in the side flank of
the abdomen
• A persistent urge to urinate
• Burning micturition
• Cloudy urine
• Fever
• Pain in the side flank of the abdomen
• Causes: Primary vesicoureteral reflux Congenital Secondary vesicoureteral reflux Acquired mostly
due to increasing obstruction to urinary outflow, such as Benign prostatic hyperplasia
• Primary vesicoureteral reflux Congenital
• Congenital
• Secondary vesicoureteral reflux Acquired mostly due to increasing obstruction to urinary outflow, such
as Benign prostatic hyperplasia
• Acquired mostly due to increasing obstruction to urinary outflow, such as Benign prostatic hyperplasia

Page 18

1044
• Management: Medical treatment includes antibiotics for recurrent UTIs Surgical therapy to correct the
defect
• Medical treatment includes antibiotics for recurrent UTIs
• Surgical therapy to correct the defect
• Complications: Renal scarring Hypertension Chronic UTI Kidney failure It is an important cause of
renal damage and death from end-stage renal failure.
• Renal scarring
• Hypertension
• Chronic UTI
• Kidney failure
• It is an important cause of renal damage and death from end-stage renal failure.
• A persistent urge to urinate
• Burning micturition
• Cloudy urine
• Fever
• Pain in the side flank of the abdomen
• Primary vesicoureteral reflux Congenital
• Congenital
• Secondary vesicoureteral reflux Acquired mostly due to increasing obstruction to urinary outflow, such
as Benign prostatic hyperplasia
• Acquired mostly due to increasing obstruction to urinary outflow, such as Benign prostatic hyperplasia
• Congenital
• Acquired mostly due to increasing obstruction to urinary outflow, such as Benign prostatic hyperplasia
• Medical treatment includes antibiotics for recurrent UTIs
• Surgical therapy to correct the defect
• Renal scarring
• Hypertension
• Chronic UTI
• Kidney failure
• It is an important cause of renal damage and death from end-stage renal failure.
Option A: Diabetes mellitus
• Chronic pyelonephritis in a diabetic patient is only seen rarely.
• It is seen in cases of chronically uncontrolled blood sugar levels.
Option B: Renal stone
• Chronic pyelonephritis in a patient with renal stones is only seen in the case of mismanaged renal
stones.
• Or in a patient with prolonged asymptomatic renal stones.

Page 19

1045
Option C: Posterior urethral valve
• It is mostly diagnosed in children.
• Chronic pyelonephritis in the posterior urethral valve is less common than in vesicoureteral reflux.

Solution for Question 4:


Option C: This condition is more common in females.
• Clinical signs: In the above-mentioned clinical scenario, the patient has presented with continuous
dribbling of urine along with normal voiding associated with urine leakage per vagina on examination.
• These signs indicate a left ectopic ureter, which is more common in females.
• Ectopic ureteral orifice Around 80% is associated with a duplication collecting system More common
in females
• Around 80% is associated with a duplication collecting system
• More common in females
• Location: Males: Prostatic or posterior urethra (MC), lateral in the bladder. Females: Anterior urethra
(MC), vestibule, vagina.
• Males: Prostatic or posterior urethra (MC), lateral in the bladder.
• Females: Anterior urethra (MC), vestibule, vagina.
• Around 80% is associated with a duplication collecting system
• More common in females
• Males: Prostatic or posterior urethra (MC), lateral in the bladder.
• Females: Anterior urethra (MC), vestibule, vagina.
Clinical Features:
• Females: Continuous incontinence with an otherwise normal voiding.
• Males: Most common presentation is UTI and never urinary incontinence, as the opening of the ureter
is above the external urethral sphincter.
• Diagnosis: IVP: Drooping Lily's sign
• IVP: Drooping Lily's sign
• Treatment: Mainly expectant, Ureteric reimplantation or pole nephrectomy, depending on moiety and
function.
• Mainly expectant, Ureteric reimplantation or pole nephrectomy, depending on moiety and function.
• IVP: Drooping Lily's sign
• Mainly expectant, Ureteric reimplantation or pole nephrectomy, depending on moiety and function.
Option A: Around 30% is associated with a duplication collecting system
• Around 80% of ectopic ureteral orifice is associated with a duplication collecting system and not 30%.
Option B: Most common presentation in males is urinary incontinence, not UTI.

Page 20

1046
• The most common presentation is UTI and never urinary incontinence, as the opening of the ureter is
above the external urethral sphincter.
Option D: Around 100% is associated with a duplication collecting system
• Around 80% of ectopic ureteral orifice is associated with a duplication collecting system and not
100%.

Solution for Question 5:


Option C: Wilms tumor
• Clinical signs: In the scenario mentioned above, a 4-year-old boy has complained of blood in his urine
with clots and a history of crying at each time of urination. Examination revealed a mass that is freely
movable and has smooth edges and a solid mass with cystic areas on a CT scan associated with
moderate microcytic anemia. This is a typical presentation of a Wilms tumor.
• Wilms tumor Wilms Tumor: Most common primary renal tumor of childhood (2-5 Years). Wilms Tumor:
2nd most common malignant abdominal tumor in children (Most common is neuroblastoma). It arises
from the kidney, which is composed of three elements: Blastema, Epithelium, and stroma (BESt).
Treatment involves surgery and chemotherapy.
• Wilms Tumor: Most common primary renal tumor of childhood (2-5 Years).
• Wilms Tumor: 2nd most common malignant abdominal tumor in children (Most common is
neuroblastoma).
• It arises from the kidney, which is composed of three elements: Blastema, Epithelium, and stroma
(BESt).
• Treatment involves surgery and chemotherapy.
• Wilms Tumor: Most common primary renal tumor of childhood (2-5 Years).
• Wilms Tumor: 2nd most common malignant abdominal tumor in children (Most common is
neuroblastoma).
• It arises from the kidney, which is composed of three elements: Blastema, Epithelium, and stroma
(BESt).
• Treatment involves surgery and chemotherapy.
Option A: Neuroblastoma
• It is also a tumor presented in childhood.
• However, it involves multiple sites and times of onset.
• Presenting complaints include lumps under tissues, back pain, and proptosis.
Option B: Angiomyolipoma
• It is a benign tumor of muscle or fat.
• It is usually presented in adults.
Option D: Grawitz tumor
• These are malignant adenocarcinomas of the kidney.
• They are presented in adults of about 50-70 years of age.

Page 21

1047
Solution for Question 6:
Correct Option C - Autosomal Dominant Kidney Disease:
• Clinical signs: Polyuria, rectal pain, a rectal abscess, abnormal creatinine, and GFR indicates
polycystic kidney disease.
• The image shows a spider leg/swiss cheese appearance, which is suggestive of autosomal dominant
polycystic kidney disease (ADPKD).
• ADPKD is an autosomal dominant genetic disease with the involvement of chromosome 16. The
disease occurs due to an abnormality in the polycystin protein.
• In ADPKD, both kidneys are replaced by multiple cysts. The kidney loses its function of the urine
concentration, due to which the patient passes a large amount of diluted urine (polyuria).
• Multiple cysts in the kidney cause compression of blood vessels that lead to the activation of the
renin-angiotensin-aldosterone system, or renin-angiotensin-system, which is a regulator of blood
pressure and cardiovascular function.
• Any compression in the pelvicalyceal system will lead to HIS (Hydronephrosis, Infection, Stone).
Dysregulated RAAS in ADPKD patients results in hypertension.
• Clinical features are usually presented on 3rd to 4th decade of life. Most commonly, patients present
with hypertension (75% adults & 25% children), while other presentations include polyuria, nocturia,
hematuria and nephrolithiasis.
• Diagnosis of ADPKD is made by intravenous pyelography (IVP), where spider leg, bell-shaped and
Bubble Swiss- Cheese appearance is observed.
Incorrect Options:
Option A - Urothelial carcinoma:
• The best diagnostic procedures for TCC are ureteroscopic brush cytology for malignant cells,
intravenous pyelogram showing radiolucent intraluminal filling defects and CT urography for evaluating
the upper urinary tract.
• Ureteral tumors are often characterized by ureteral dilation below the site of the lesion.
Option B - Urolithiasis:
• Intravenous pyelography can diagnose urinary tract diseases like kidney stones and bladder stones.
Imaging tests can detect kidney stones in the urinary tract.
• High-speed or dual-energy computed tomography (CT) can reveal even tiny stones. A typical
abdominal X-ray is less common as it can miss small kidney stones.
Option D - Hydronephrosis:
• Hydronephrosis is usually identified by imaging tests such as a kidney ultrasound, CT scan, or MRI.
• Imaging reveals the outline of an enlarged kidney which is the hallmark of hydronephrosis.

Page 22

1048
Solution for Question 7:
Option C - Horseshoe Kidney:
• The patient mentioned above had recurrent urinary tract infections and was successfully treated each
time with antibiotics. For recurrent attacks, IVU was performed, showing characteristic features of the
horseshoe kidney, with the calyx oriented toward the center of the spine and often "vase" in
appearance.
• A horseshoe kidney is a basal fusion of kidneys. When fused, it forms a "U" shape, which is why it is
called a "horseshoe."
• If it causes symptoms, it is because of infection and stone formation. The most common symptoms
associated with horseshoe kidney are renal calculi, flank pain, UTI, and nausea.
• The condition occurs as the pelvis and ureters have to pass through the isthmus, causing congestion.
This condition rarely requires surgery.
• The reason for surgical treatment may be for pelvic ureteric junction obstruction or abdominal aortic
aneurysm.
Incorrect Options:
Option A - Renal Cell Carcinoma:
• Renal cell cancer is one of the most common renal cancers in the adult age group.
• In the early stages, patients remain symptom-free, while after the progression of the disease, lump in
the abdomen, Hematuria, weight loss, fatigue, and persistent pain in the abdomen.
• this patient has no such symptoms.
Option B - Hydronephrosis:
• Hydronephrosis is kidney inflammation, usually due to blockage in the urinary tract.
• Imaging reveals the outline of an enlarged kidney which is the hallmark of hydronephrosis.
Option D - Pyonephrosis:
• Pyonephrosis is a pyogenic infection of the upper urinary tract due to ureteral obstruction.
• It is usually associated with suppurative lesions of the renal parenchyma and loss of renal function.
Patients are often symptomatic but remain asymptomatic in 15% of cases.

Solution for Question 8:


Option A: Pyonephrosis
• Clinical signs: The above mentioned patient has all the features of septicemia, including shock and
hyperdynamic circulation (large pulse pressure), and bounding pulse. Marked loin tenderness points to
a diagnosis of pyonephrosis.
• Pyonephrosis is an infection of the collective system of the kidneys. Pus collects in the renal pelvis,
and the kidney stretches. This can lead to kidney failure.
• It is a disease that causes purulent destruction of the renal parenchyma.

Page 23

1049
• The most common symptoms are fever, chills and flank pain. Patients are often symptomatic but
remain asymptomatic in 15% of cases.
• Pyonephrosis may be a complication of kidney stones that causes persistent infection. It can also
occur spontaneously.
• It may occur as a complication of hydronephrosis or pyelonephritis.
• It requires drainage, which is best achieved by placement of a ureteral stent or nephrectomy.
Option B: Hydronephrosis
• Hydronephrosis is kidney inflammation, usually due to blockage in the urinary tract.
• Imaging reveals the outline of an enlarged kidney which is the hallmark of hydronephrosis.
Option C: Urolithiasis
• Urolithiasis occurs when stones migrate from the renal pelvis to the rest of the urinary system,
including the ureters, bladder, and urethra.
Option D: Polyuria
• Polyuria is when the body urinates more frequently than usual, producing excessive or abnormally
large amounts of urine with each urination.
• Polyuria due to central diabetes insipidus is an early symptom of acute myeloid leukemia but has no
association with renal cell carcinoma.

Solution for Question 9:


Option C: Congenital Hydronephrosis
• Clinical signs: The above mentioned 20-year-old patient developed hematuria following minor trauma
while playing rugby. He was completely unaware of the trauma and surprised to have hematuria. He did
notice mild left loin discomfort. Hematuria following minor trauma typically occurs in a pathological
kidney.
• In a healthy young man, this would be a congenital hydronephrosis (sometimes called idiopathic
hydronephrosis) from a pelvic-ureteric junction (PUJ) obstruction that has ruptured with urinary
extravasation, the cause of fullness in his left loin.
• This patient needs resuscitation, although he is probably stable from the cardiovascular point of view
but for his tachycardia. He needs an IVU to make sure that the other kidney is normal.
• Congenital hydronephrosis (CH) is a clinically significant genitourinary congenital disability identified
by the demonstration of dilated renal pelvis and calyx.
• Symptoms of hydronephrosis usually depend on the cause. About 69% of affected infants
develop abnormal renal function or require corrective surgery.
• Often there are no symptoms,Symptoms may include-- sudden or severe flank pain, nausea, frequent
urination or painful urination, weakness, fever due to urinary tract infection and hematuria.
Option A: Renal Cell Cancer
• Renal cell cancer is one of the most common renal cancers in adults.

Page 24

1050
• In the early stages, patients remain symptoms free, while after the progression of the disease,patient
can present with lump in the abdomen, hematuria, weight loss, fatigue and persistent pain in the
abdomen.
Option B: Adenocarcinoma
• Adenocarcinoma is a type of cancer that starts in the glands that line the organs and can occur almost
anywhere in the body.
• Adenocarcinoma usually arises in mucous membranes and initially appears as thickened, white,
patchy mucous membranes.
Option D: Pyonephrosis

• Pyonephrosis is a pyogenic infection of the upper urinary tract due to ureteral obstruction.
• It is usually associated with suppurative lesions of the renal parenchyma and loss of renal function.
Patients are most often symptomatic but remain asymptomatic in 15% of cases.

Solution for Question 10:


Option A: Bladder Carcinoma
• Clinical signs: In the above-mentioned case, the patient's history of hematuria, micturition problems,
and the atypical masses of urothelial cells suggest carcinoma of the bladder. The patient is a smoker,
which is the principal cause of bladder carcinoma. It is the fourth most common non-
• Painless, progressive, profuse and periodic hematuria is the classical presentation, and physical signs
are few except for anaemia.
• The diagnosis is made by cystoscopy in a hematuria clinic.
• The patient's urine meta-analysis reveals fresh red blood cells with masses of atypical urothelial cells,
which are the definite signs of transitional cell carcinoma (TCC), also known as urothelial carcinoma.
• The best diagnostic procedures for bladder carcinoma are ureteroscopic brush cytology for malignant
cells, intravenous pyelogram showing radiolucent intraluminal filling defects, and CT urography for
evaluating the upper urinary tract.
• Transurethral resection of bladder tumor (TURBT) is the treatment; the other is radical cystectomy
and pelvic lymphadenectomy. Those undergoing TURBT are kept under regular cystoscopic
surveillance.
Option B: Adenocarcinoma
• Adenocarcinoma is a type of cancer that starts in the glands that line the organs and can occur almost
anywhere in the body.
• Adenocarcinoma usually arises in mucous membranes and initially appears as thickened, white,
patchy mucous membranes.
Option C: Ductal Carcinoma
• Ductal carcinoma in situ (DCIS) of the breast is a complex heterogeneous pathological condition in
which malignant epithelial cells are localized in the milk ducts without evidence of invasion.
• DCIS is in situ or pre-invasive breast cancer.

Page 25

1051
• Ductal carcinoma is a neoplastic growth of malignant cells that form glands.
Option D: Squamous cell carcinoma
• Squamous cell skin cancer (SCC) is the second most common skin cancer characterized by abnormal
proliferation of squamous cells.
• SCC is caused by abnormal changes in squamous epithelial cells due to DNA damage caused by
exposure to UV light and other harmful substances.

Solution for Question 11:


Correct Option D - Wilms tumor:
• According to the scenario mentioned above, the patient is suffering from Wilms tumor, one of the most
prevalent pediatric neoplasms and the most prevalent kidney neoplasm in children.
• The nephrogenic zone of the fetal kidney can be seen in Wilms tumor (nephroblastoma)
• A palpable, painless abdominal mass, appetite loss, stomach pain, fever, nausea, and hematuria are
common indicators of a Wilms tumor.
• A high cure rate is achieved using a comprehensive staging, grading, and molecular analysis
methodology together with surgery, chemotherapy, and radiation.
Incorrect Options:
Option A - Angiomyolipomas:
• Angiomyolipomas can be idiopathic or a complication of hereditary tuberous sclerosis syndrome.
• With well-differentiated muscle, adipose tissue, and vascular components, they may be bilateral and
multiple.
Option B - Interstitial cell tumors:
• Renomedullary interstitial cell tumors, also known as medullary fibromas, are typically less than 1 cm
in size and present incidentally.
• Malignant interstitial cell tumors occur exclusively in adults.
Option C - Renal cell carcinoma:
• Renal cell carcinoma is rare in pediatric age; most people with renal cell carcinoma are older, usually
between 50 and 70.
• Clear cell, papillary, and chromophobe patterns are the most common.

Solution for Question 12:


Correct Option C - Hyaline arteriolosclerosis:
• The patient mentioned above, microscopic appearance of kidneys demonstrate hyaline
atherosclerosis, a condition commonly seen in people with benign hypertension, and renal
parenchymal changes may be termed benign nephrosclerosis.

Page 26

1052
• In the absence of hypertension, similar modifications can be observed with age. Ischemic
abnormalities are brought on by vascular constriction and progress slowly over time. The patient's
kidneys have shrunk, and there is widespread scarring.
• Hyaline arteriolosclerosis is characterized by the thickening of the arteriolar wall followed by the
buildup of homogenous material that appears pink on slides stained with hematoxylin and eosin.
• Patients with diabetes or benign arterial hypertension frequently exhibit hyaline arteriolosclerosis in
their kidneys.
• An important technique is blood pressure screening, which can detect hypertensive patients before
serious organ damage occurs.
• Essential (benign) hypertension may evolve into malignant hypertension that causes distinctive renal
vascular lesions, including fibrinoid necrosis and hyperplastic arteriosclerosis.
Incorrect Options:
Option A - Acute tubular necrosis:
• Acute tubular necrosis results from anoxic or toxic injury to the renal tubules.
Option B - Fibromuscular dysplasia:
• The medium-sized arteries in the body develop stenosis and aneurysms as a result of the condition of
fibromuscular dysplasia. Blood flow can be reduced, and narrowed arteries may impact organs'
performance.
• The most typical site of fibromuscular dysplasia development is in the arteries to the kidneys and
brain.
• the histological features are not suggestive of FMD in this patient.
Option D - Interstitial nephritis
• Interstitial nephritis is most commonly caused by an allergic reaction to a medication or by some
bacterial infections like E. coli, viral infections like HIV or herpes, syphilis, parasites, and exposure to
fungi are all infections that are linked to interstitial nephritis.
• There would be more cells in the urine sediment in interstitial nephritis.

Solution for Question 13:


Correct Option C - Erythropoietin:
• The figure demonstrates renal cell carcinoma. In between 5% and 10% of these tumours,
erythropoietin is secreted, resulting in polycythemia.
• Erythropoietin production by renal cell carcinoma cells and cyst epithelial cells in the left kidney may
cause polycythemia.
• Serum urea nitrogen and creatinine levels are not raised, and renal function is not significantly lost
since renal cell carcinomas are frequently unilateral and do not completely kill a kidney.
• Erythropoietin is a glycoprotein produced mainly by the kidney and controls erythropoiesis. Increased
erythropoiesis production results in secondary erythrocytosis, which is observed in various tumours,
including renal cell carcinoma, hepatocellular carcinoma, and cerebellar hemangioblastoma.
• The lack of erythrocytosis in renal cell carcinoma patients with elevated erythropoietin levels may be
explained by the possibility that renal cell carcinoma cells create an inactive isoform of erythropoietin.

Page 27

1053
Incorrect Options:
Option A - Globulin:
• Globulin would be increased with multiple myeloma and lead to amyloid deposition, a diffuse process.
• Increased gamma globulin proteins may indicate any acute infection
Option B - Cortisol:
• Corticotropin (adrenocorticotropic hormone) results in hypercortisolism in Cushing syndrome, but
these cases are encountered less frequently than polycythemia.
• Cushing's disease can manifest as secondary polycythemia, and polycythemia can be the first sign of
Cushing's disease.
Option D - Vasopressin:
• A syndrome of inappropriate antidiuretic hormone (vasopressin) is more likely a paraneoplastic
syndrome associated with small-cell lung carcinomas.
• Vasopressin also has a potent constricting effect on arterioles throughout the body

Solution for Question 14:


Option C: Escherichia coli
• In the scenario mentioned above, the patient suffers from hemolytic uremic syndrome (HUS),which is
the most common cause of acute renal failure among children.
• HUS is a rare but severe condition that compromises a person's ability to clot blood and function
ofkidneys. Red blood cells are destroyed by HUS infection, which can lead to renal failure. As a result
of a diarrheal infection (E. coli O157:H7 infection), HUS can develop.
• It most commonly occurs after ingesting meat infected with verocytotoxin-producing Escherichia coli,
most often serotype O157:H7. This Shiga toxin damages endothelium, reducing nitric oxide, promoting
vasoconstriction and necrosis, and promoting platelet activation to form thrombi in small vessels.
• With supportive therapy, most patients recover in a few weeks, although perhaps one-fourth progress
to chronic renal failure.
• However, the hemolytic uremic syndrome may also occur in adults from Shiga toxin and drug
ingestion.
• Thrombotic thrombocytopenic purpura (TTP) in adults can lead to similar renal thrombotic
microangiopathy but is due to abnormal ADAMTS13 metalloproteinase clearance of von Willebrand
multimers.
Option A: Candida albicans
• Candidal urinary tract infections typically affect the urinary bladder.
• Candida cystitis, often known as a symptomatic lower UTI, can develop
Option B: Clostridium difficile
• Clostridium difficile is best known for causing pseudomembranous enterocolitis, not renal lesions.
• Colon inflammation caused by the bacterium Clostridium difficile overgrowth is known as C. difficile
colitis.

Page 28

1054
Option D: Proteus mirabilis
• Proteus is the main pathogen causing complicated urinary tract infections (UTIs), especially
catheter-associated urinary tract infections.
• The development of biofilms on catheter surfaces is primarily responsible for their ability to cause
such infections.

Solution for Question 15:


Option D: Chest X-ray could be done to rule out pulmonary metastasis
• Clinical signs: Hematuria, significant weight loss left flank pain and progressive abdominal distension
are the symptoms. The palpable mass, mildly tender in the left upper quadrant of the abdomen,
indicates renal cell carcinoma.
• Chest X-ray could be done to rule out pulmonary metastasis.
• Renal cell carcinoma: Five Histological subtypes are Clear cell carcinoma, Papillary carcinoma,
Chromophobe type, Bellini duct tumor, and Medullary carcinoma. Male: Female–2:1. Risk factors are
age, sex, hypertension, obesity, and tobacco smoking. Clinical presentation includes loin pain, mass,
and hematuria (the most common presentation). Indications of partial Nephrectomy: Size <4 cm, RCC
in a solitary kidney, bilateral RCC (familial syndromes), and RCC with diseased contra-lateral kidney. A
radical nephrectomy is done when there is a large-size tumor. The metastatic tumor is removed by
cytoreductive nephrectomy, adrenals are not mandatory, and ureters are removed until the pelvic brim.
Patients with renal vein or IVC involvement by renal cell carcinoma but without evidence of metastatic
disease are the surgical candidates.
• Five Histological subtypes are Clear cell carcinoma, Papillary carcinoma, Chromophobe type, Bellini
duct tumor, and Medullary carcinoma.
• Male: Female–2:1. Risk factors are age, sex, hypertension, obesity, and tobacco smoking.
• Clinical presentation includes loin pain, mass, and hematuria (the most common presentation).
• Indications of partial Nephrectomy: Size <4 cm, RCC in a solitary kidney, bilateral RCC (familial
syndromes), and RCC with diseased contra-lateral kidney.
• A radical nephrectomy is done when there is a large-size tumor.
• The metastatic tumor is removed by cytoreductive nephrectomy, adrenals are not mandatory, and
ureters are removed until the pelvic brim.
• Patients with renal vein or IVC involvement by renal cell carcinoma but without evidence of metastatic
disease are the surgical candidates.
• Five Histological subtypes are Clear cell carcinoma, Papillary carcinoma, Chromophobe type, Bellini
duct tumor, and Medullary carcinoma.
• Male: Female–2:1. Risk factors are age, sex, hypertension, obesity, and tobacco smoking.
• Clinical presentation includes loin pain, mass, and hematuria (the most common presentation).
• Indications of partial Nephrectomy: Size <4 cm, RCC in a solitary kidney, bilateral RCC (familial
syndromes), and RCC with diseased contra-lateral kidney.
• A radical nephrectomy is done when there is a large-size tumor.

Page 29

1055
• The metastatic tumor is removed by cytoreductive nephrectomy, adrenals are not mandatory, and
ureters are removed until the pelvic brim.
• Patients with renal vein or IVC involvement by renal cell carcinoma but without evidence of metastatic
disease are the surgical candidates.
• Occasionally, nephrectomy and removal of IVC tumor thrombus need to be done.
Option A: A Pre-op biopsy is necessary
• A pre-op biopsy is unnecessary as it is diagnosed with radiology and on a clinical basis.
Option B: IVC involvement indicates inoperability
• Patients with renal vein or IVC involvement by renal cell carcinoma but without evidence of metastatic
disease remain surgical candidates.
• Occasionally, nephrectomy and removal of IVC tumor thrombus need to be done.
Option C: Pre-op radiotherapy is essential
• Pre-op radiotherapy is not required as long as the candidate is still a candidate for surgical removal.
After that, radiotherapy may be needed as per demand.

Solution for Question 16:


Correct Option C - Inferior mesenteric artery:
• Clinical signs: Urinary incontinence with intermittent normal voiding since birth and fused moiety,
dilated ureters indicate the horseshoe kidney.
• The ascent of the horseshoe kidney remains incomplete due to inferior mesenteric artery preventing
further migration of the kidney.
• The normal ascent of the kidneys makes the organs take their place in the abdomen below the
adrenal glands.
• However, with horseshoe kidney, the ascent into the abdomen is prevented by the inferior mesenteric
artery, which hooks over the isthmus.
• Isthmus is usually located adjacent to L3-L4 vertebrae, just below the origin of IMA from the
abdominal aorta.

Page 30

1056
Incorrect Options:
Option A - Superior mesenteric artery:
• In the middle of the digestive tract is the superior mesenteric artery (midgut). It comes from the aorta,
which is situated between the renal and celiac arteries.
• It has no relationship with the ascent of the kidney.
Option B - Superior mesenteric vein:
• The Superior mesenteric vein (SMV), a significant venous tributary of the abdominal cavity, is the
superior mesenteric vein.
• It has no relationship with the ascent of the kidney.
Option D - Inferior mesenteric vein:
• It's the inferior mesenteric artery, not the vein, which prevents the ascent of the horseshoe kidney in
the abdomen.

Solution for Question 17:


Option B: Iliohypogastric nerve
• Clinical sign: Abdominal Pain radiating to the groin, hydronephrosis bilaterally, and hyperdense
area in CT images indicate ureterolithiasis.
• Stone in the middle one-third of the ureter→ pain is radiated to the iliac fossa, hypogastrium, and
groin via the Iliohypogastric nerve.
• The most common symptom of ureteric colic is pain.
• The severity of pain is not related to the size of the stone.
• Radiation of pain depends on the location of the stone.
Option A: Subcostal nerve

Page 31

1057
• Stones in the upper ureter or renal pelvis→ pain is referred to the testis in males and labia majora in
females via the subcostal nerve.
Option C: Ilioinguinal nerve
• Stone located in the distal one-third of the ureter→ pain is referred to in the inner aspect of the thigh,
scrotum, and perineum via the Ilioinguinal nerve.
• Stone located on the left side in the middle one-third of the ureter- Pain is radiated to the left iliac
fossa (mimics diverticulitis).
• Stone on the right side in the middle one-third of the ureter- Pain is radiated to the right iliac fossa
(mimics acute appendicitis)
Option D: Genitofemoral nerve
• Stone located in the distal one-third of ureter→ pain is referred to in the inner aspect of the thigh,
scrotum, and perineum via the genitofemoral nerve.

Solution for Question 18:


Option B: Calcium oxalate, uric acid, struvite, cystine
• Clinical signs: Colicky, intermittent, bilateral flank pain without hematuria and mildly raised creatinine
indicate renal stones.
• Calcium oxalate stone: Kidney stones can develop when there is dehydration and a high
concentration of calcium, oxalate, cystine, or phosphate. Kidney stones can be of various types. The
most common kidney stones are calcium oxalate stones.
• Kidney stones can develop when there is dehydration and a high concentration of calcium, oxalate,
cystine, or phosphate.
• Kidney stones can be of various types. The most common kidney stones are calcium oxalate stones.
• Uric acid stone: A kidney stone is generated from substances present in the urine. After forming, the
stone may remain in the kidney or pass through the ureter and into the lower urinary system.
• A kidney stone is generated from substances present in the urine.
• After forming, the stone may remain in the kidney or pass through the ureter and into the lower urinary
system.
• Struvite stone: Due to their close ties to urea-splitting urinary tract infections (UTIs), struvite
(magnesium ammonium phosphate) stones are frequently referred to as "infection stones.”
• Due to their close ties to urea-splitting urinary tract infections (UTIs), struvite (magnesium ammonium
phosphate) stones are frequently referred to as "infection stones.”
• Cystine stone: Cystine stones are formed due to inherited defect in the transport of the amino acid
cystine leading to excessive excretion causing cystinuria.
• Cystine stones are formed due to inherited defect in the transport of the amino acid cystine leading to
excessive excretion causing cystinuria.
• Kidney stones can develop when there is dehydration and a high concentration of calcium, oxalate,
cystine, or phosphate.
• Kidney stones can be of various types. The most common kidney stones are calcium oxalate stones.

Page 32

1058
• A kidney stone is generated from substances present in the urine.
• After forming, the stone may remain in the kidney or pass through the ureter and into the lower urinary
system.
• Due to their close ties to urea-splitting urinary tract infections (UTIs), struvite (magnesium ammonium
phosphate) stones are frequently referred to as "infection stones.”
• Cystine stones are formed due to inherited defect in the transport of the amino acid cystine leading to
excessive excretion causing cystinuria.
Option A: Calcium oxalate, struvite, uric acid, cysteine
• Based on the images of crystals, the correct order of stones is calcium oxalate, uric acid, struvite, and
cysteine.
Option C: Uric acid, calcium oxalate, struvite, cysteine
• The correct order of stones is calcium oxalate, uric acid, struvite, and cysteine, based on the image of
crystals above.
Option D: Struvite, uric acid, calcium oxalate cysteine
• Based on the images of crystals shown above, the correct order of stones is calcium oxalate, uric
acid, struvite, and cysteine.

Solution for Question 19:


Correct Option A - Gold standard lithotripter is Dornier unmodified HM-3:
• Clinical signs: Right flank pain, ureteric calculus, and hydronephrosis are indications of a renal stone.
• ESWL- Extracorporeal Shock wave Lithotripsy: Gold standard Lithotripter- Dornier Unmodified HM-3
Shock waves recognize the stone with the help of density changes in the body→ erosion and shattering
of stones → fragmentation of stones → excreted via the ureter. If there is an obstruction in the ureter
(distal ureter- 64%, proximal ureter - 29% and mid ureter -8%), stone fragments are collected in the
proximal part of the ureter leading to a complication known as Steinstrasse (also known as Street of
stones/ Columnation of Gravel). The maximum size of stone removed by ESWL is 2cm.
• Gold standard Lithotripter- Dornier Unmodified HM-3
• Shock waves recognize the stone with the help of density changes in the body→ erosion and
shattering of stones → fragmentation of stones → excreted via the ureter.
• If there is an obstruction in the ureter (distal ureter- 64%, proximal ureter - 29% and mid ureter -8%),
stone fragments are collected in the proximal part of the ureter leading to a complication known as
Steinstrasse (also known as Street of stones/ Columnation of Gravel).
• The maximum size of stone removed by ESWL is 2cm.
• Gold standard Lithotripter- Dornier Unmodified HM-3
• Shock waves recognize the stone with the help of density changes in the body→ erosion and
shattering of stones → fragmentation of stones → excreted via the ureter.
• If there is an obstruction in the ureter (distal ureter- 64%, proximal ureter - 29% and mid ureter -8%),
stone fragments are collected in the proximal part of the ureter leading to a complication known as
Steinstrasse (also known as Street of stones/ Columnation of Gravel).
• The maximum size of stone removed by ESWL is 2cm.

Page 33

1059
• Absolute contraindications of ESWL: Pregnancy Bleeding disorders
• Pregnancy
• Bleeding disorders
• Relative contraindications of ESWL: Pacemaker UTI Distal obstruction Obesity Orthopaedic
abnormalities like Scoliosis and Kyphoscoliosis Uncontrolled HTN Aneurysm Renal failure
• Pacemaker
• UTI
• Distal obstruction
• Obesity
• Orthopaedic abnormalities like Scoliosis and Kyphoscoliosis
• Uncontrolled HTN
• Aneurysm
• Renal failure
• Whenever ESWL cannot be used→, PCNL is preferred
• Pregnancy
• Bleeding disorders
• Pacemaker
• UTI
• Distal obstruction
• Obesity
• Orthopaedic abnormalities like Scoliosis and Kyphoscoliosis
• Uncontrolled HTN
• Aneurysm
• Renal failure
Incorrect Options:
Option B - Steinstrasse is seen in cases of proximal ureteric obstruction:
• Steinstrasse is a complication of ESWLseen in cases of distal ureteric obstruction.
• Distal obstruction is responsible for this.
Option C - Pregnancy is an Indication:
• In pregnancy, it is contraindicated.
Option D - Maximum size of stone removed is 1.5 cm:
• The maximum size of stone removed by ESWL is 2 cm.
• It is not 1.5cm; stones up to 2 cm could be removed.

Page 34

1060
Solution for Question 20:
Correct Option B - Xanthogranulomatous pyelonephritis:
• Clinical signs: Fever up to 38 °C, anemia and episodes of vomiting, increasing right flank pain and
weight loss, staghorn calculi, and recurrent infections. All the mentioned signs and symptoms are
suggestive of Xanthogranulomatous pyelonephritis.
• Signs and symptoms of the case mentioned above: Diabetic Middle-aged Female Flank pain Fever
Fat density lesion Associated with calculi
• Diabetic
• Middle-aged Female
• Flank pain
• Fever
• Fat density lesion
• Associated with calculi
• Xanthogranulomatous (due to the presence of fat) pyelonephritis (infection with kidney etiology) It is
more common in females as they are more prone to Struvite stone formation. Causative Organism -
Proteus
• It is more common in females as they are more prone to Struvite stone formation.
• Causative Organism - Proteus
• Diabetic
• Middle-aged Female
• Flank pain
• Fever
• Fat density lesion
• Associated with calculi
• It is more common in females as they are more prone to Struvite stone formation.
• Causative Organism - Proteus
• Emphysematous pyelonephritis Most common causative organism- E. coli Presence of a ring of air
around the kidney or within the kidney.
• Most common causative organism- E. coli
• Presence of a ring of air around the kidney or within the kidney.
• Most common causative organism- E. coli
• Presence of a ring of air around the kidney or within the kidney.

Page 35

1061
• Adults – Xanthogranulomatous pyelonephritis most typically arises in middle-aged women with a
history of recurrent urinary tract infections, weight loss and anorexia.
• Typical symptoms include flank pain, fever, and malaise.
• A unilateral renal mass can usually be palpated on physical examination.
Incorrect Options:
Option A - TB kidney:
• Caseous lesions that may necrose and destroy functional renal parenchyma are the hallmark of renal
TB. Scarring, contraction, and fibrosis all help lesions heal.
• Strictures, hydronephrosis, and obstructive uropathy can result from ureteral fibrosis. Another
symptom of renal TB is interstitial nephritis.
Option C - Chronic pyelonephritis:
• Chronic pyelonephritis, a persistent pyogenic infection of the kidney, nearly always affects people with
severe anatomical anomalies.
• Fever, malaise, and flank pain are possible symptoms. Urinalysis, culture, and imaging tests are used
to make the diagnosis.
Option D - Emphysematous pyelonephritis:
• A necrotizing infection of the renal parenchyma and its surrounding tissues that results in gas in the
renal parenchyma, collecting system, or perinephric tissue is called Emphysematous pyelonephritis
(EPN).

Solution for Question 21:


Option A: Percutaneous drainage under CT guidance

Page 36

1062
• Clinical signs: β-thalassemia minor, intermittent diffuse abdominal pain with two weeks of evolution,
paleness, abdominal tenderness and palpable mass in the left flank and neutrophilia and leukocytosis
all indicates the diagnosis to be perinephric abscess.
• The collection of fluid inside the kidney is called pyonephrosis.
• The collection of fluid(pus) around the kidney is called a perinephric abscess.
• Perinephric Abscess: Treatment of choice for is incision and drainage + Antibiotics. If there is no
improvement with antibiotics, then a deep-seated intraabdominal abscess is possible. Percutaneous
drainage under USG/CT guidance is preferred over Open surgical drainage. A nephrectomy is not done
in this patient because kidney function is not impaired. Observation and follow-up are not done because
the patient’s condition did not improve.
• Treatment of choice for is incision and drainage + Antibiotics.
• If there is no improvement with antibiotics, then a deep-seated intraabdominal abscess is possible.
Percutaneous drainage under USG/CT guidance is preferred over Open surgical drainage.
• A nephrectomy is not done in this patient because kidney function is not impaired.
• Observation and follow-up are not done because the patient’s condition did not improve.
• Treatment of choice for is incision and drainage + Antibiotics.
• If there is no improvement with antibiotics, then a deep-seated intraabdominal abscess is possible.
Percutaneous drainage under USG/CT guidance is preferred over Open surgical drainage.
• A nephrectomy is not done in this patient because kidney function is not impaired.
• Observation and follow-up are not done because the patient’s condition did not improve.
Option B: Open surgical drainage
• If there is no improvement with antibiotics, then a deep-seated intraabdominal abscess is possible.
Percutaneous drainage under USG/CT guidance preferred over Open surgical drainage.
Option C: Nephrectomy
• A nephrectomy is not done because kidney function is not impaired in this case.
• As serum BUN and creatinine are normal.
Option D: Observation & follow up
• Observation and follow-up were not done because the patient’s condition did not improve with this
treatment option.

Solution for Question 22:


Correct Option A - Endoscopic incision:
• Clinical signs: Vulval mass with urinary retention, mild left flank pain, lower urinary tract symptoms,
interlabial mass emerging from the meatus, which is irreducible, and suprapubic bulge, all these
indicates the ureterocele which is managed by endoscopic incision.
• In IVP, the bilateral terminal portion of the ureters are dilated (Ureterocele)→Adder head /Cobra head
appearance.

Page 37

1063
• Ureterocele causes outflow obstruction, which causes backflow pressure changes causing called
hydroureteronephrosis. So, the diagnosis is bilateral ureterocele with bilateral hydroureteronephrosis.
• Ureterocele Cystic dilatation of terminal ureter Most common in females Types Intra- Vesical (20%)
Ectopic (80%) - A/W duplication of the ureter In Intravesical ureterocele - The terminal part of the ureter
is cystic & dilated and is inside the bladder. In ectopic ureterocele - Duplication of the ureter and the
upper pole of the ureter has an ectopic ureteric orifice.
• Cystic dilatation of terminal ureter
• Most common in females
• Types Intra- Vesical (20%) Ectopic (80%) - A/W duplication of the ureter
• Intra- Vesical (20%)
• Ectopic (80%) - A/W duplication of the ureter
• In Intravesical ureterocele - The terminal part of the ureter is cystic & dilated and is inside the bladder.
• In ectopic ureterocele - Duplication of the ureter and the upper pole of the ureter has an ectopic
ureteric orifice.
• The most common congenital abnormality of the upper urinary tract is duplication of the ureter.
• The most common congenital abnormality of the urogenital tract is vesicoureteral reflux.
• Cystic dilatation of terminal ureter
• Most common in females
• Types Intra- Vesical (20%) Ectopic (80%) - A/W duplication of the ureter
• Intra- Vesical (20%)
• Ectopic (80%) - A/W duplication of the ureter
• In Intravesical ureterocele - The terminal part of the ureter is cystic & dilated and is inside the bladder.
• In ectopic ureterocele - Duplication of the ureter and the upper pole of the ureter has an ectopic
ureteric orifice.
• Intra- Vesical (20%)

Page 38

1064
• Ectopic (80%) - A/W duplication of the ureter
Incorrect Options:
Option B - Nephrectomy with partial ureterectomy:
• Nephrectomy is not indicated in the above condition.
• As the drainage of the cyst is the best treatment option, not the nephrectomy or partial ureterectomy.
Option C - STING operation:
• This operation is used to prevent urinary reflux, preventing recurrent urinary tract infections.
• It has no role in the ureterocele.
Option D - Lich-Gregoir technique:
• This treatment is also used for primary reflux of all grades if the ureter is not grossly dilated on the
excretory urogram.
• This is also not useful for ureterocele.

Solution for Question 23:


Option C: Angiomyolipoma
• Clinical sign: Progressive bloating sensation in the abdomen, family history of tuberous sclerosis,
hyperechoic mass with mixed density, and positive for HMB-45, indicating the diagnosis of
angiomyolipoma.
• Angiomyolipoma Angiomyolipoma is a benign disorder composed of matured blood vessels, muscles,
and fat. More common in females and is associated with tuberous sclerosis. Has positive
immunoreactivity for HMB-45. The presence of microaneurysm (detected on angiography) increased
the risk of retroperitoneal bleeding in 10 % of patients, known as Wunderlich syndrome. The
investigation of choice for this condition is CECT.
• Angiomyolipoma is a benign disorder composed of matured blood vessels, muscles, and fat.
• More common in females and is associated with tuberous sclerosis.
• Has positive immunoreactivity for HMB-45.
• The presence of microaneurysm (detected on angiography) increased the risk of retroperitoneal
bleeding in 10 % of patients, known as Wunderlich syndrome.
• The investigation of choice for this condition is CECT.
• CT findings The presence of fat and absence of calcification is highly suggestive of angiomyolipoma
(In RCC → presence of calcification; this feature is used to differentiate between RCC &
Angiomyolipoma).
• The presence of fat and absence of calcification is highly suggestive of angiomyolipoma (In RCC →
presence of calcification; this feature is used to differentiate between RCC & Angiomyolipoma).
• Asymptomatic tumors up to 4cm in size are kept under observation.
• Angiomyolipoma is a benign disorder composed of matured blood vessels, muscles, and fat.
• More common in females and is associated with tuberous sclerosis.
• Has positive immunoreactivity for HMB-45.

Page 39

1065
• The presence of microaneurysm (detected on angiography) increased the risk of retroperitoneal
bleeding in 10 % of patients, known as Wunderlich syndrome.
• The investigation of choice for this condition is CECT.
• The presence of fat and absence of calcification is highly suggestive of angiomyolipoma (In RCC →
presence of calcification; this feature is used to differentiate between RCC & Angiomyolipoma).
Option A: Secondary to malignant melanoma
• The kidney is a common location for micrometastasis in patients with malignant melanomas.
• These lesions are usually small, multiple, asymptomatic, and bilateral and associated with a known
primary lesion.
• Not associated with tuberous sclerosis and immunoreactivity for HMB-45 is negative.
Option B: Renal cell carcinoma
• Clinical presentation of renal cell carcinoma includes loin pain, loin mass, and hematuria (the most
common presentation).
• This patient had no hematuria; her family history and biopsy results point toward a diagnosis of
angiomyolipoma.
Option D: Carcinoma of the renal pelvis
• Cancer of the renal pelvis starts in the cells of the renal pelvis. It is a Urothelial carcinoma(also called
transitional cell carcinoma).
• Not associated with tuberous sclerosis and immunoreactivity for HMB-45 is negative.

Solution for Question 24:


Option D: VCD
• Clinical signs: Rapidly growing mass in the abdomen, pink-colored urine; on CT investigation, findings
of crescent-shaped, discrete & peripheral calcification and young age show that he has a case of Wilms
tumor or nephroblastoma.
• The chemotherapy regimen used in Wilms tumor is Vincristine, Cyclophosphamide,
Doxorubicin/Dactinomycin.
Staging
Pre-chemotherapy staging
Post-chemotherapy staging
• Preferred by National Wilms Tumor Staging System (NWTS system)
• Staging & surgery is done before chemotherapy
• Preferred by International society of Staging & Surgery. It is done after chemotherapy
• Pediatric Oncology (SIOP staging system)
Option A: FOLFOX

Page 40

1066
• FOLFOX-IV Folinic acid/Leucovorin + 5-FU+ Oxaliplatin) is the regimen used in colorectal carcinoma,
not in the case of the Wilms tumor.
Option B: ECF
• ECF (Epirubicin + Cisplatin + 5-FU) is the treatment regimen for carcinoma of the esophagus and
carcinoma of the stomach.
Option C: CAF
• CAF (Cyclophosphamide + Adriamycin + 5-FU) is used for breast carcinoma, not the Wilms tumor.

Solution for Question 25:


Correct Option D - Posterior Cardinal Vein:
• Clinical signs: Dull, intermittent right flank pain, hydronephrosis and the IVP imaging suggest the
diagnosis of the retrocaval ureter/circumcaval ureter.
• The image above represents a rare condition known as the retrocaval ureter. The posterior vena cava
or retrocaval ureter is a rare congenital anomaly caused by malformation of the inferior vena cava
(IVC), resulting in the passage of the right ureter behind and medial to the IVC.
• An unusual embryology behind the pre-ureteral vena cava is the right posterior cardinal vein retention
rather than the right subcardiac vein.
• The ureter effectively lies in a "retrocaval" or "circumcaval" position because the right posterior
cardinal vein lies ventral to the ureter, also known as the circumaural ureter. This can lead to
hydronephrosis, a rare cause of long-term, periodic abdominal pain.
• Abnormal development of IVC causes the ureter to pass behind IVC and to take a U-turn before
draining into the bladder. It happens only due to the abnormal persistence of the right posterior
subcardinal vein.
• The most common clinical feature is ureteric obstruction.
• MRI makes a diagnosis to delineate anatomy clearly and non-invasively, and on IVP, "Reverse J",
"Fish Hook", Or "Shepherd Crook" deformities are observed.
• Treatment for retrocaval ureter involves the procedure known as relocation ureteroureterostomy.
Incorrect Options:
Option A - Azygos vein:
• The azygos vein is a unilateral blood vessel that ascends to the thorax on the right side of the spine. It
carries deoxygenated blood from the posterior chest and abdominal walls.
• Azygos vein helps to drain most of the posterior intercostal veins on the right side of the body. It is part
of the azygous venous system and has no role in hydronephrosis.
Option B - Hemiazygos vein:
• The hemiazygos vein arises from the confluence of the lumbar and subcostal veins that rise from the
epigastric region to the left.
• The hemiazygos vein often connects with the left renal vein. It functions to drain the left mediastinum
and left lower oesophagus.
Option C - Anterior Cardinal Vein:

Page 41

1067
• The anterior cardinal vein (major anterior cardiac vein) contributes to forming the internal jugular vein,
which, together with the common cardinal vein, forms the superior vena cava.
• The anterior cardinal vein drains the cranial portion of the embryo and is interconnected by a wide
central anastomotic tract.

Solution for Question 26:


Option C: Hematuria
• Clinical signs: The above mentioned patient has fatigue, weight loss, persistent pain symptoms, and
the right flank mass has been diagnosed with renal cell carcinoma.
• "Too- Late" triad associated with renal cell carcinoma includes Abdominal mass, flank pain and
hematuria. Triad is seen in 10% of patients with advanced disease.
• The most common presentation of renal cell carcinoma is hematuria.
• 25-30% of patients are asymptomatic, and renal cell carcinoma is detected on random radiographs.
Renal cell carcinoma (RCC) is often not diagnosed until it reaches the advanced metastatic stage.
• Renal cell carcinoma is also considered an incidental finding on imaging and may rarely appear on
physical examination.
• This cancer is known to be asymptomatic in its early stages. However, some symptoms of RCC are
hematuria, flank pain, and a palpable mass in the abdomen.
• RCC is also diagnosed by signs of metastasis to distant organs such as the lungs, liver, lymph nodes,
and adrenal glands.
• In Advanced cases, a non-reducing varicocele is observed due to a tumour thrombus in the left
kidney.
• The treatment of choice for renal cell carcinoma is radical nephrectomy.
Option A: Polyuria
• Polyuria is when the body urinates more frequently than usual, producing excessive or abnormally
large amounts of urine with each urination.
• Polyuria due to central diabetes insipidus is an early symptom of acute myeloid leukemia but has no
association with renal cell carcinoma.
Option B: Pyrexia
• Clinical presentation of renal cell carcinoma includes pyrexia of unknown origin. RCC may cause a
fever as the natural result of the immune system fighting an infection.
• However, fever is not a component of the too-late triad associated with renal cell carcinoma.
Option D: Anemia
• Anaemia is a disease caused by various factors. This refers to low levels of oxygen-carrying red blood
cells. RCC can cause anaemia, which causes tiredness, lethargy, and exhaustion.
• However, anaemia is not considered a component of the Too Late triad associated with renal cell
carcinoma.

Page 42

1068
Solution for Question 27:
Correct Option B - Renal Tuberculosis:
• Clinical signs: In the case mentioned above, the patient's urine culture report came out negative
(sterile pyuria). Sterile pus in urine is characteristic of renal tuberculosis (TB).
• The patient died because of renal failure; auto-nephrectomy (calcified nonfunctioning kidney) is
observed in renal TB.
• Genitourinary organs are involved through the hematogenous spread.
• In genitourinary TB first organ involved is the lungs. The kidney and prostate are involved primarily.
The other organs are involved in the descent/ascent of infection.
• In renal TB, cortical granulomas result in tubercular abscess. It ruptures and spreads pus into the
pelvicalyceal system.
• Dysuria and pus cells are observed in urine, and no growth on culture in 48 hours, representing renal
tuberculosis.
• Stenosis of the calyx or Pelvic-ureteric Junction (PUJ) obstruction causes hydronephrosis and
pyonephrosis, resulting in a perinephric abscess.
• This causes the kidney to be replaced by caseous material (putty kidney) and calcification (cement
kidney) which ultimately leads to auto-nephrectomy (calcified-end-stage kidney).
Incorrect Options:
Option A - Renal Cell Cancer:
• Renal cell cancer is one of the most common renal carcinomas in adults.
• In the early stages, patients remain symptom-free, while after the progression of the disease, lump in
the abdomen, hematuria, weight loss, fatigue, and persistent pain in the abdomen.
Option C - Cystic Renal Disease:
• Cystic kidney disease (CKD) describes a series of conditions in which cysts (fluid-filled sacs) form in
or around the kidneys. Renal cysts prevent the kidneys from filtering water and waste products from the
blood.
• Headache, hematuria, hypertension, kidney infection, and kidney stones are seen; this patient has no
such symptoms.
Option D - Metastasis:
• During metastasis, cancer cells leave the primary tumor, travel through the blood or lymphatic system,
and form new tumors in other organs or tissues of the body. Initially, nearby lymph nodes are affected
early.
• When an organ receives metastatic disease, its lymph nodes shrink until they rupture or dissolve. The
lung, liver, brain, and bone are the most common sites of solid tumor metastasis. This patient has no
lymphadenopathy or signs of malignancy.

Page 43

1069
Solution for Question 28:
Option A: Acute tubular necrosis
• This is a case of acute tubular necrosis (ATN) presented with a sudden onset increase in serum
creatinine level (from 0.8 mg/dl to 2.0 mg/dl) after the use of broad-spectrum antibiotics for acute
diverticulitis and muddy brown cast on urinalysis.
• It is the most common cause of intrinsic renal failure (especially in hospitalized patients)
• Causes: Ischemia occurs due to hypotension, shock, and heart failure, which decreases the blood
supply to the kidneys. The proximal convoluted tubule and thick ascending limb of the loop of Henle are
most commonly affected by ischemia. Nephrotoxin: It occurs due to toxic substances (cisplatin, lead,
and aminoglycosides), myoglobinuria, and hemoglobinuria. Proximal convoluted tubules are most
commonly affected by nephrotoxins.
• Ischemia occurs due to hypotension, shock, and heart failure, which decreases the blood supply to
the kidneys. The proximal convoluted tubule and thick ascending limb of the loop of Henle are most
commonly affected by ischemia.
• Nephrotoxin: It occurs due to toxic substances (cisplatin, lead, and aminoglycosides), myoglobinuria,
and hemoglobinuria. Proximal convoluted tubules are most commonly affected by nephrotoxins.
• Ischemia occurs due to hypotension, shock, and heart failure, which decreases the blood supply to
the kidneys. The proximal convoluted tubule and thick ascending limb of the loop of Henle are most
commonly affected by ischemia.
• Nephrotoxin: It occurs due to toxic substances (cisplatin, lead, and aminoglycosides), myoglobinuria,
and hemoglobinuria. Proximal convoluted tubules are most commonly affected by nephrotoxins.
Option B: Acute interstitial nephritis (AIN)
• Acute interstitial nephritis (AIN) is also called tubulointerstitial nephritis.
• AIN is the inflammation of the kidney interstitium, which occurs secondary to the intake of rifampin,
diuretics, penicillin, and cephalosporins.
• It presents with signs/symptoms of rash, hematuria, pyuria, fever, and flank tenderness.
Option C: Acute pyelonephritis
• It is inflammation of the kidney (most commonly affects the cortex and relatively spare glomeruli and
vessels)
• Urinalysis shows WBCs in urine, with/without WBC casts, and computed tomography (CT) scan
shows parenchymal enhancement.
• It is caused by urinary tract infection and hematogenous spread to the kidney.
Option D: Renal papillary necrosis (RPN)
• RPN presents with hematuria (gross) and proteinuria due to sloughing of necrotic renal papillae
• It is caused by diabetes mellitus, sickle cell disease or sickle cell trait, pyelonephritis (acute)
• Gross examination reveals grey, white, or yellow necrosis of distal 2/3rd of the renal pyramid is seen.

Solution for Question 29:


Option A: Transitional cell carcinoma

Page 44

1070
• In the scenario mentioned above, the patient's urine meta-analysis reveals fresh red blood cells with
masses of atypical urothelial cells, which are definite signs of transitional cell carcinoma (TCC), also
known as urothelial carcinoma.
• Clinical features of urothelial carcinoma include, most commonly, painless gross hematuria, flank
pain, and irritative voiding symptoms.
• The best diagnostic procedures for transitional cell carcinoma are ureteroscopic brush cytology for
malignant cells, intravenous pyelogram showing radiolucent intraluminal filling defects, and CT
urography for evaluating the upper urinary tract.
• Ureteral tumors are often characterized by ureteral dilation below the site of the lesion.
• Bergman sign: A ureteral catheter passed up to the ureter may coil distal to the ureter to diagnose
transitional cell carcinoma.
Option B: Squamous cell carcinoma
• Squamous cell skin cancer is the 2nd most common skin cancer characterized by abnormal
proliferation of squamous cells.
• SCC is caused by abnormal changes in squamous epithelial cells due to DNA damage caused by
exposure to UV light and other harmful substances.
Option C: Adenocarcinoma
• Adenocarcinoma is a type of cancer that begins in the glands that line the organs and can occur
almost anywhere in the body.
• Adenocarcinoma usually arises in mucous membranes and initially appears as thickened, white,
patchy mucous membranes.
Option D: Ductal carcinoma
• Ductal carcinoma in situ of the breast is a complex heterogeneous pathological condition in which
malignant epithelial cells are localized in the milk ducts without evidence of invasion.
• DCIS is in situ or pre-invasive breast cancer. It is a neoplastic growth of malignant cells that form
glands.

Solution for Question 30:


Correct Option A - Genitourinary Tuberculosis:
• This is a case of genitourinary tuberculosis, which presents with recurrent episodes of urinary tract
infection unresponsive to antibiotics, hematuria, sterile pyuria, and flank pain.
• It is due to hematogenous spread to kidneys, ureters, and bladder and reactivated in the
immunocompromised state as in this patient compliant with immunosuppressant drugs.
• Urine cultures do not show growth of bacteria, urinalysis show hematuria, and sterile pyuria and
acid-fast staining are appropriate to identify the cause and organism.
• On CT, genitourinary tuberculosis presents with hydroureteronephrosis and calcification around the
urinary tract.
• Cystoscopy reveals fibrosis and ulcerations.
• The abnormal characterization of the ureteral orifice shown in the exhibit is the "Golf-hole ureter"
Which is a characteristic finding in genitourinary tuberculosis.

Page 45

1071
• It occurs because of the contraction caused by the fibrosis of the ureter at the ureteric orifice, which
leads to the widening of its internal opening, which gives the appearance of a golf hole.
Incorrect Options:
Option B - Ureterocele:
• A ureterocele is a congenital disability presenting as dilation at the distal part of the ureter. The ureter
does not properly enter the bladder, results in abnormal development, and does not show fibrosis and
calcification on CT.
• Ureterocele is caused by specific genes and environmental exposure to ACE inhibitors in fetal life.
Option C - Vesicoureteral reflux:
• Vesicoureteral reflux is abnormal urine flow from the bladder to the upper urinary tracts in the ureter
and the kidneys.
• It can be due to abnormal insertion of the ureter into the ureterovesical junction, and it also presents
with recurrent UTIs but does not show calcification and fibrosis on CT.
Option D - Carcinoma of the bladder:
• Carcinoma of the bladder, also called transitional cell carcinoma, occurs in the ureters, renal pelvis,
and bladder.
• Transitional cell carcinoma can present as painless hematuria and is associated with exposure to
smoking, phenacetin use, rubber and cyclophosphamide.

Page 46

1072
Urinary Bladder
1. A 29-year-old woman presents with severe abdominal and flank pain after a motor vehicle collision
where she was the seatbelt-restrained driver. Her abdomen was distended and showing guarding with
rebound tenderness. There is frank blood at the urethral orifice, and the introduction of a bladder
catheter yielded no urine. Retrograde cystography is shown below. What is the next step in
management?
(or)
A 29-year-old woman presents with severe abdominal and flank pain after a motor vehicle collision
where she was the seatbelt-restrained driver. Her abdomen was distended and showing guarding with
rebound tenderness. There is frank blood at the urethral orifice, and the introduction of a bladder
catheter yielded no urine. Retrograde cystography is shown below. What is the next step in
management?

A. Radical cystectomy has to be performed


B. Requires laparotomy
C. Simple catheter drainage + Broad-spectrum antibiotics as a precaution for peritonitis
D. Conservative management is enough, as the intra-abdominal fluid can be drained
----------------------------------------
2. A 58-year old male presents to the surgery department with complaints of painful urination and
difficulty passing urine for the last month. He also reports blood in his urine on several occasions. He
smokes 10 to 12 cigarettes per day. His ultrasound shows a mass in his urinary bladder. MRI and
further evaluation confirms a diagnosis of urothelial cell carcinoma. Which of the following is a true
statement regarding urothelial cell carcinoma of the bladder?
(or)
Which statement is true regarding urothelial cell carcinoma of the bladder?
A. Ileal conduit diversion is required after cystectomy
B. Intravesical chemotherapy and immunotherapy are beneficial in non-muscle invasive bladder cancer
(NMIBC)
C. Radical cystectomy following radiotherapy is of benefit in Muscle invasive bladder cancer
D. Strongly associated with smoking & Schistosoma crematorium
----------------------------------------

1073
3. A 61-year old male presents to the surgery department with complaints of frequent urination
associated with dribbling and loss of feeling that the bladder has been full over the previous two
months. He also reports increased frequency and urgency. He is a known case of multiple sclerosison
medication. His further evaluation reveals a disorder of the bladder shown in the image. The diagnosis
of the given image is:
(or)
A 61-year-old male known case of multiple sclerosis presents with complaints of frequent urination
associated with dribbling and loss of feeling that the bladder has been full over the previous two
months. Further evaluation reveals a disorder of the bladder shown in the image. What is the
diagnosis?

A. Schistosomiasis
B. Acute tuberculosis
C. Chronic tuberculosis
D. Neurogenic bladder
----------------------------------------
4. Cystoscopy reveals the following image. What is true about the management of this condition?
(or)
A 26-year-old woman has been experiencing lower abdominal pain and frequent urination for seven
months. She has no other symptoms, and tests have been inconclusive. Cystoscopy revealed a finding,
what is true about the management of this condition?

A. Conservative management; Has a good prognosis


B. Application of dimethyl-sulfoxide into the bladder

Page 2

1074
C. Surgery followed by chemotherapy
D. Bladder resection under anaesthesia
----------------------------------------
5. A male newborn is delivered vaginally at 38 weeks. Pregnancy is uneventful, and no fetal anomalies
are detected at prenatal ultrasound controls. Maternal serologies for HbsAg, HCV, HIV, Toxoplasma,
and Rubella are negative, along with insignificant vaginal swabs. Delivery is not complicated; Apgar
score is nine at 1st, 5th and 10th of life. The neonate presents at birth with a genitourinary defect, as
shown in the image. Which of the following feature is seen in this condition?
(or)
A male newborn is delivered vaginally at 38 weeks. Delivery is not complicated, and Apgar's score is
nine at 1st and 5th minute of life. What feature is seen in this condition?

A. Inguinal hernia
B. Visible ureterovesical efflux
C. Hypospadias
D. Normal gait
----------------------------------------
6. A 67-year old female has a five-year history of progressive urinary bladder retention and voiding
difficulties. MRI reveals exuberant concentric hypertrophy of the entire urethra. What is the diagnosis?
(or)
A 67-year old female presents to the surgery department with a five-year history of progressive urinary
bladder retention and voiding difficulties. She underwent a pelvic MRI for evaluation and treatment
planning. MRI reveals exuberant concentric hypertrophy of the entire urethra. This characteristic
imaging finding resembles a bulb of a sphygmomanometer on the sagittal and coronal planes and a
doughnut on the axial ones. The combination of the MRI findings together with the patient's symptoms
is consistent with the diagnosis of?

Page 3

1075
A. Benign prostatic hyperplasia
B. Detrusor-sphincter dyssynergia
C. Marion's disease
D. Urge incontinence
----------------------------------------
7. A 51-year old male presents to the surgery department with complaints of cloudy discharge
associated with burning micturition for the last week. He has a smoking history of 20-pack years. His is
posted for a specific test to rule out the cause. The test in the image is done in which shreds are
present only in the first glass. What is the most probable diagnosis?
(or)
A 51-year old male presents with complaints of cloudy discharge associated with burning micturition.
He has a smoking history of 20-pack years. He is scheduled for a specific test to rule out the cause.
The test in the image is done in which shreds are present only in the first glass. What is the diagnosis?

A. Prostatitis
B. Cystitis
C. Urethritis
D. Renal pathology
----------------------------------------
8. Which of the following statements about detrusor instability is true?
(or)

Page 4

1076
A 54-year old male presents to the surgery department with complaints of burning micturition for the
last three days. He is a known case of diabetes and has had multiple urinary tract infections over the
previous two years. Further evaluation confirms detrusor instability. Which of the following statements
about detrusor instability is true?
A. Only 1 % of men with bladder outflow obstruction have detrusor instability
B. Patients with neurogenic bladder may have a detrusor instability
C. Genuine stress incontinence (GSI) is indistinguishable from detrusor instability
D. Urodynamic studies are not used for the evaluation of detrusor instability
----------------------------------------
9. A 57-year old male presents to the surgery department with complaints of painful urination and
difficulty passing urine for the last month. He also reports blood in his urine on several occasions. He
smokes 10 to 12 cigarettes per day. His ultrasound shows a mass in his urinary bladder. MRI and
further evaluation confirm the diagnosis of transitional cell carcinoma. Which of the following is true
about bladder cancer?
(or)
Which of the following is true about bladder cancer?
A. For patients with carcinoma of bladder invading the bladder muscle (T2 lesion), immediate
radiotherapy followed by surgery offers the best chance of cure
B. Patients with limited lymph node involvement may be cured by surgery alone
C. Continent neo-bladders have yet to be successfully utilized in patients undergoing cystectomy
D. Intravesical chemotherapy before surgery is routinely used for bladder cancers invading the bladder
muscle (T2 lesion)
----------------------------------------
10. A 72-year old man presents with a complaint of painless, macroscopic hematuria. CT urography
reveals an enlarged prostate, and a 2.7 cm large, spiculated, and mobile calcification is seen in the
urinary bladder. What is the composition of this stone?
(or)
A 72-year-old man visited the Urology department with painless, macroscopic hematuria for three
weeks. A digital rectal examination showed an enlarged prostate, confirmed by ultrasound revealing a
large bladder stone and post-void residual urine. A CT urography showed an enlarged prostate and a
2.7 cm large, spiculated, and mobile calcification in the urinary bladder. What is the composition of this
stone?

Page 5

1077
A. Calcium oxalate monohydrate
B. Calcium oxalate dihydrate
C. Ammonium urate
D. Magnesium and ammonium
----------------------------------------
11. A 55-year female presents to the urology outpatient department with complaints of frequency,
urgency, dysuria, gross hematuria with the passage of clots, and right flank pain for one month. The
patient also had a history of recurrent urinary tract infections. She is a known diabetic and has been on
oral hypoglycemics for the last 16 years. A cystoscopy of the patient showed multiple nodular brush-like
lesions with easy bleeding in the bladder, especially around the right vesicoureteric junction. Which of
the following is the most likely diagnosis?
(or)
What is the most likely diagnosis in a patient with frequency, urgency, dysuria, gross hematuria and
Histopathological examination reveals basophilic inclusions, which are a feature of Michaelis- Gutmann
bodies?
A. Urothelial carcinoma
B. Polypoid cystitis
C. Squamous cell carcinoma of the bladder
D. Malakoplakia
----------------------------------------
12. A 35-year-old man from Goa who worked as an engineer in a water dam for the past 1 year
presented with abdominal pain, increased urine frequency, and hematuria. He was admitted to the
hospital and underwent cystoscopy with biopsies. Histological examination of the biopsies revealed a
granulomatous inflammatory reaction . These granulomas contained bilharzia eggs in the center. All of
the following statements are correct about the diagnosis of the given case except:
(or)
A 35-year-old man presents with complaints of abdominal pain, increased urine frequency, and
hematuria. He was admitted to the hospital and underwent a cystoscopy with biopsies. The granulomas
on HPE contained bilharzia eggs in the center. What statement is false about the diagnosis?

A. Man is both the definitive and intermediate host.


B. most common cause of bladder calcification
C. Swimmer’s itch is the first clinical sign

Page 6

1078
D. Increases the risk of both SCC and TCC
----------------------------------------
13. A 34-year-old male is brought to the emergency department after a road traffic accident. He
complained of suprapubic pain, difficulty in the ability to pass urine and hematuria. He gives no history
of any other abnormalities. He has no history of any urinary tract abnormalities. He is otherwise vitally
stable, and other general physical examination reveals no abnormalities. A pelvic x-ray of the patient
was done, which revealed a pelvic fracture. The cystography of the patient is done, which is given
below. Which of the following is the most likely diagnosis?
(or)
A 34-year-old male presents suprapubic pain, difficulty passing urine, and hematuria after a road traffic
accident. A pelvic x-ray of the patient was done, which revealed a pelvic fracture. The cystography of
the patient is done, which is given below. What is the most likely diagnosis?

A. Extraperitoneal bladder rupture


B. Intraperitoneal bladder rupture
C. Bladder contusion
D. Posterior urethral injury
----------------------------------------
14. A 60-year old male presents to the emergency department with complaints of painless gross
hematuria for one day. He gives no history of any other abnormalities. He has no history of any chronic
illness. He is a chronic smoker. The patient is otherwise vitally stable, and other general physical
examination reveals no abnormalities. On cystoscopy, there is a mass in the bladder. Biopsy revealed a
tumor of transitional epithelium. The bladder tumor antigen in the urine is positive. Which of the
following statements is true regarding the patient’s pathology?
(or)
A 60-year old male presents with complaints of painless gross hematuria. He is a chronic smoker.
Cystoscopy reveals a mass in the bladder. Biopsy revealed a tumour of transitional epithelium. The
bladder tumour antigen in the urine is positive. What statement is true regarding the patient’s
pathology?
A. The preferred systemic chemotherapy is cyclophosphamide.
B. The most common site of lymphatic metastases is the axillary lymph node
C. The most common site is the posterolateral wall of the trigone
D. The most effective intravesical chemotherapy is methotrexate

Page 7

1079
----------------------------------------
15. A 2-day old male infant is brought to the emergency department by her mother with complaints of
his organs of urinary tract bulging out of the abdomen since birth. His mother further states that the
child also has urinary incontinence. The child has no other abnormalities. He is otherwise vitally stable,
and other general physical examination reveals no abnormalities. The image of the child’s
abnormalities is shown below. Which of the following is the most likely diagnosis?
(or)
A 2-day old male infant is presented with complaints of his organs of urinary tract bulging out of the
abdomen since birth, along with urinary incontinence. The image of the child’s abnormalities is shown
below. What is the most likely diagnosis?

A. Cloacal exstrophy
B. Omphalocele minor
C. Omphalocele major
D. Gastroschisis
----------------------------------------

Correct Answers
Question Correct Answer

Question 1 2
Question 2 4
Question 3 4
Question 4 2
Question 5 2
Question 6 3
Question 7 3
Question 8 2
Question 9 2
Question 10 2
Question 11 4

Page 8

1080
Question 12 1
Question 13 1
Question 14 3
Question 15 1

Solution for Question 1:


Correct Option B - Requires laparotomy:

• A ruptured intraperitoneal bladder is depicted in the photograph. Note the extraluminal contrast (red
arrows) seeping into the peritoneal cavity from outside the normal bladder's boundaries. There is
contrast outside of the intestine, in the left paracolic gutter (yellow arrow) according to the above
mentioned clinical scenario and image findings, it is diagnosed to be a case of intraperitoneal bladder
injury.
• The intrarenal collecting system and ureter are visualized because the patient had a
contrast-enhanced CT done moments earlier.
• Intraperitoneal Rupture (20%): Blow, kick or fall on a fully distended bladder. Apart from the classic
triad suggestive of bladder rupture (Suprapubic pain and tenderness + Difficulty or inability to pass
urine + Hematuria), patients develop peritonitis and abdominal distention. Diagnosis is made by
retrograde cystography or CT cystography. Cystography demonstrates a typical sunburst appearance.
Laparotomy with peritoneal lavage and bladder repair with Suprapubic catheterization (SPC) should be
done.
• Blow, kick or fall on a fully distended bladder.
• Apart from the classic triad suggestive of bladder rupture (Suprapubic pain and tenderness + Difficulty
or inability to pass urine + Hematuria), patients develop peritonitis and abdominal distention.
• Diagnosis is made by retrograde cystography or CT cystography.
• Cystography demonstrates a typical sunburst appearance. Laparotomy with peritoneal lavage and
bladder repair with Suprapubic catheterization (SPC) should be done.
• Blow, kick or fall on a fully distended bladder.

Page 9

1081
• Apart from the classic triad suggestive of bladder rupture (Suprapubic pain and tenderness + Difficulty
or inability to pass urine + Hematuria), patients develop peritonitis and abdominal distention.
• Diagnosis is made by retrograde cystography or CT cystography.
• Cystography demonstrates a typical sunburst appearance. Laparotomy with peritoneal lavage and
bladder repair with Suprapubic catheterization (SPC) should be done.
Incorrect Options:
Option A - Radical cystectomy has to be performed:
• The patient has intraperitoneal bladder rupture, so radical cystectomy does not indicate this case.
• A radical cystectomy is done to treat cancer that has invaded muscle tissue of the bladder or recurrent
noninvasive bladder cancer.
Option C - Simple catheter drainage + Broad-spectrum antibiotics as a precaution for peritonitis:
• The patient discussed above has intraperitoneal rupture leading to peritonitis.
• A simple catheter cannot drain it. However, broad-spectrum antibiotics should be given to treat the
peritonitis.
Option D - Conservative management is enough, as the intra-abdominal fluid can be drained:
• Peritonitis is a fatal condition which requires immediate drainage by doing laparotomy and giving
broad-spectrum antibiotics to treat peritonitis.
• Conservative management is not enough in this case.

Solution for Question 2:


Option D: Strongly associated with smoking & Schistosoma crematorium
• The patient is suffering from urothelial cell carcinoma.
• It is strongly associated with smoking & Schistosoma crematorium.
• Risk factors for carcinoma urinary bladder.

Risk Factor for Carcinoma Urinary Bladder


Transitional Cell Carcinoma
Squamous Cell Carcinoma
• Cigarette smoking is the main etiological factor, accounting for about 50% of bladder cancer.
• Occupational exposure to chemicals: Naphthylamine, benzidine, acrolein, aniline dyes, hydrocarbons
• Schistosoma crematorium: A risk factor for both TCC and SCC, more for SCC
• Drugs: Phenacetin and Chlornaphazine. Cyclophosphamide.
• Phenacetin and Chlornaphazine.
• Cyclophosphamide.
• Pelvic Irradiation.

Page 10

1082
• Occupations associated with increased risk: Chemical, dye, rubber, petroleum. Leather and glinting
Industry workers.
• Phenacetin and Chlornaphazine.
• Cyclophosphamide.
• Schistosoma crematorium: A risk factor for both TCC and SCC, more for SCC
• Chronic irritation: urinary calculi, long-term indwelling catheters, chronic urinary tract infections.
• Bladder diverticula.
• Management Cystoscopy and TUR or biopsy: further management is based on stage, grade, size,
multiplicity and recurrence pattern. Drugs used for intravesical chemotherapy: BCG (Most Effective),
Mitomycin-C, Epirubicin.
• Cystoscopy and TUR or biopsy: further management is based on stage, grade, size, multiplicity and
recurrence pattern.
• Drugs used for intravesical chemotherapy: BCG (Most Effective), Mitomycin-C, Epirubicin.
• Cystoscopy and TUR or biopsy: further management is based on stage, grade, size, multiplicity and
recurrence pattern.
• Drugs used for intravesical chemotherapy: BCG (Most Effective), Mitomycin-C, Epirubicin.
Option A: Ileal conduit diversion is required after cystectomy
• Ileal conduit diversion is not required after cystectomy.
Option B: Intravesical chemotherapy and immunotherapy are beneficial in Non –
muscular invasive bladder cancer (NMIBC)
• Intravesical chemotherapy and immunotherapy are not beneficial in non-muscle invasive bladder
cancer (NMIBC)
• It is only given in early stages of bladder cancer
(NMIBC)
Option C: Radical cystectomy following radiotherapy is of benefit in Muscle invasive bladder cancer
• Radiotherapy has no role.

Solution for Question 3:


Correct Option D - Neurogenic bladder:
• The patient discussed above suffers from a neurogenic bladder, and the picture shows a Christmas
tree appearance. He is a known case of multiple sclerosis, and it is a risk factor for developing a
neurogenic bladder.
• Symptoms may include Urinary tract infection (UTI), Kidney stones, Urinary incontinence (unable to
control urine), and Small urine volume during voiding.
Incorrect Options:
Option A - Schistosomiasis:
• These worms deposit egg clusters (pseudo papilloma) in the bladder, causing inflammation. The
calcification seen on the x-ray is the calcification of the egg clusters, not the bladder itself.

Page 11

1083
• Depending on the site of these pseudo papilloma in the bladder, they can cause obstructive uropathy
and kidney damage.
Option B - Acute tuberculosis:
• Inflammation of bladder mucosa in early stages → Tubercle formation (seen endoscopically as white
or yellow raised nodules surrounded by a halo of hyperemia) → Mural fibrosis.
• (Thimble bladder)

Option C - Chronic tuberculosis:


• Tuberculosis is a chronic, progressive mycobacterial infection, often with an asymptomatic latent
period following the initial infection.

Solution for Question 4:


Correct Option B - Application of dimethyl-sulfoxide into the bladder:
This patient is suffering from interstitial cystitis, and the treatment is the instillation of dimethyl-sulfoxide
into the distended bladder under anaesthesia. Interstitial cystitis is a
chronic condition causing bladder pressure, bladder pain and pelvic pain.
Pathology-
• There is pancystitis with fibrosis of vesical musculature along with linear ulcers in the bladder mucosa.
Microscopically, severe inflammation of all layers of bladder with fibrosis is observed.
• Bladder eventually becomes thimble (systolic) bladder with decreased bladder capacity up to 30-60 ml
(less than 100 ml).
Features- Pain.
• Decreased bladder capacity.
• Pain increases with bladder distension.
• Frequency and often haematuria.

Page 12

1084
Investigation-
• Cystography and cystoscopy are diagnostic.
Treatment-
• Hydrostatic dilatation.
• Instillation of dimethyl sulphoxide (Rimso 50).
• Ranitidine instillation.
• lleocystoplasty or caecocystoplasty, to increase the bladder capacity.
• Urinary diversion.
Incorrect Options:
Option A - Conservative management; has a good prognosis:
• Interstitial cystitis is not managed conservatively. Its treatment is difficult and unsatisfactory, involving
the distention of the bladder and the instillation of dimethyl sulfoxide.
Option C - Surgery followed by chemotherapy:
• The treatment of hunger ulcers is the instillation of dimethyl sulfoxide into the bladder. The bladder is
distended under anaesthesia.
• No surgery followed by chemotherapy is needed.
Option D - Bladder resection under anesthesia:
• The bladder is distinct under anesthesia but not resected; dimethyl sulfoxide is instilled into the blood,
but the treatment is difficult and unsatisfactory.

Solution for Question 5:


Correct Option B - Visible ureterovesical efflux:
• The newborn discussed above has clinical features, and examination findings suggestive of Ectopia
vesicae. There is visible ureterovesical efflux.
• Exstrophy of the bladder (ectopia vesicae) is a rare congenital anomaly.
• The tubercle genitalia, the anterior wall of the bladder, and the inferior section of the anterior
abdominal wall are all formed by the imperfect fusing of the mesoderm. Epispadias is present, not the
hypospadias in Ectopia vesicae. Umbilical hernia, visible ureterovesical efflux, and waddling gait are
seen in Ectopia vesicae.
Incorrect Options:
Option A - Inguinal hernia:
• Rectus muscles which are inserted on the pubic rami, are also widely separated.
• An umbilical hernia though usually small is present along with the extrophic bladder.
• Inguinal hernia is not a feature.
Option C - Hypospadias:
• Epispadias is present, not the hypospadias in Ectopia vesicae.

Page 13

1085
• In males, complete epispadias with a wide & shallow scrotum. Undescended testis & inguinal hernia
are common.
• Females also have epispadias with bifid clitoris and wide separation of the labia.
Option D - Normal gait:
• Waddling gait is seen in Ectopia vesicae.
• Gait is not normal.

Solution for Question 6:


Correct Option C - Marion’s disease:

• Coronal (a), sagittal (b), and axial (c), T2-weighted MRI images showing exuberant concentric
hypertrophy of the muscular planes surrounding the urethra along its complete extension (arrows).
• This characteristic finding resembled the bulb of a sphygmomanometer in "a" and "b" and a doughnut
in “c”.
• Primary bladder neck obstruction / Marion's disease: It is the bladder outlet obstruction caused by an
enlargement of the muscle cells in the neck of the bladder. It is due to dyssynergic contraction of the
smooth muscle of the bladder neck (bladder neck dyssynergia).
• It is the bladder outlet obstruction caused by an enlargement of the muscle cells in the neck of the
bladder.
• It is due to dyssynergic contraction of the smooth muscle of the bladder neck (bladder neck
dyssynergia).
• It is the bladder outlet obstruction caused by an enlargement of the muscle cells in the neck of the
bladder.
• It is due to dyssynergic contraction of the smooth muscle of the bladder neck (bladder neck
dyssynergia).
Incorrect Options:

Page 14

1086
Option A - Benign prostatic hyperplasia:
• Benign prostatic hyperplasia (BPH) is common in older men.
• BPH typically presents with lower urinary tract symptoms, which may be categorized as voiding
(obstructive) and storage (irritative).
Option B - Detrusor-sphincter dyssynergia:
• Detrusor sphincter dyssynergia (DSD) is the urodynamic term for variable lower urinary tract
symptoms.
• It occurs due to detrusor muscle contraction with concomitant and inappropriate involuntary urethral
sphincter contraction.
Option D - Urge Incontinence:
• Overactive bladder (OAB)/urge incontinence is due to detrusor overactivity.
• An urge to urinate is quickly followed by uncontrollable leakage ranging from a few drops to complete
bladder emptying.

Solution for Question 7:


Correct Option C - Urethritis:
• The patient discussed above suffers from urethritis, and the image shown is three glass tests.
Urethritis is inflammation of the urethra.
• It is usually caused by an infection
• The term non-gonococcal urethritis (NGU) is used when the sexually transmitted infection is not
caused by N.gonorrhoea.

• The image shown in the question is a procedure of the three-glass test: A three-glass test is a
procedure to determine the location of an infection affecting the urinary system of a male. The bladder
is emptied by passing urine into a series of 3-ounce test tubes, and the contents of the first and the last
are examined; The first tube contains the washings from the anterior urethra, The second material is

Page 15

1087
from the bladder, and The last material is from the posterior urethra, prostate, and seminal vesicles. If
flakes or threads of pus are passed in the urine, their source is usually urethral.
• A three-glass test is a procedure to determine the location of an infection affecting the urinary system
of a male.
• The bladder is emptied by passing urine into a series of 3-ounce test tubes, and the contents of the
first and the last are examined;
• The first tube contains the washings from the anterior urethra,
• The second material is from the bladder, and
• The last material is from the posterior urethra, prostate, and seminal vesicles.
• If flakes or threads of pus are passed in the urine, their source is usually urethral.
• A three-glass test is a procedure to determine the location of an infection affecting the urinary system
of a male.
• The bladder is emptied by passing urine into a series of 3-ounce test tubes, and the contents of the
first and the last are examined;
• The first tube contains the washings from the anterior urethra,
• The second material is from the bladder, and
• The last material is from the posterior urethra, prostate, and seminal vesicles.
• If flakes or threads of pus are passed in the urine, their source is usually urethral.
Incorrect Options:
Option A - Prostatitis:
• Prostatitis is inflammation of the prostate gland.
• In three glass tests, material from the posterior urethra, prostate, and seminal vesicles would be
visible on the third glass.
Option B - Cystitis:
• Cystitis is the inflammation of the urinary bladder.
• In three glass tests, the material from the bladder could be seen on the second glass.
Option D - Renal pathology:
• Renal pathology does not appear on three glass tests.

Solution for Question 8:


Option B: Patients with neurogenic bladder may have a detrusor instability
• This patient suffers from detrusor instability. The patients with neurogenic bladder may have detrusor
instability.
• Detrusor instability (DI) Detrusor hyperreflexia refers to an overactive bladder condition triggered by
neurological disorders like cerebrovascular disease, multiple sclerosis, or spinal cord injury. Irritation of
the bladder by inflammation (such as urinary tract infection) or prior pelvic surgery can also cause
detrusor instability.

Page 16

1088
• Detrusor hyperreflexia refers to an overactive bladder condition triggered by neurological disorders
like cerebrovascular disease, multiple sclerosis, or spinal cord injury.
• Irritation of the bladder by inflammation (such as urinary tract infection) or prior pelvic surgery can also
cause detrusor instability.
• Motor Urge Incontinence Symptomatic presentation is similar to sensory urge incontinence.
Urodynamic findings of detrusor constrictions usually support the diagnosis. It can be idiopathic or
associated with bladder outlet obstruction in men from (BPH) or anatomic stress incontinence in
women. Also referred to as Detrusor instability
• Symptomatic presentation is similar to sensory urge incontinence.
• Urodynamic findings of detrusor constrictions usually support the diagnosis.
• It can be idiopathic or associated with bladder outlet obstruction in men from (BPH) or anatomic stress
incontinence in women.
• Also referred to as Detrusor instability
• Idiopathic detrusor instability can mimic GSI and may coexist.
• But they are different conditions that urodynamic studies can properly diagnose.
• Detrusor hyperreflexia refers to an overactive bladder condition triggered by neurological disorders
like cerebrovascular disease, multiple sclerosis, or spinal cord injury.
• Irritation of the bladder by inflammation (such as urinary tract infection) or prior pelvic surgery can also
cause detrusor instability.
• Symptomatic presentation is similar to sensory urge incontinence.
• Urodynamic findings of detrusor constrictions usually support the diagnosis.
• It can be idiopathic or associated with bladder outlet obstruction in men from (BPH) or anatomic stress
incontinence in women.
• Also referred to as Detrusor instability
Option A: Only 1 % of men with bladder outflow obstruction have detrusor instability
• About 50% of men with bladder outflow obstruction have detrusor instability.
Option C: Genuine stress incontinence (GSI) is indistinguishable from detrusor instability
• Genuine stress incontinence (GSI) is distinguishable from detrusor instability.
• Idiopathic detrusor instability can mimic GSI and may coexist.
• GSI occurs commonly in females with weak pelvic floor muscles and occurs on coughing, sneezing
which puts pressure on the bladder
Option D: Urodynamic studies are not used for the evaluation of detrusor instability
• Urodynamic studies are essential for the evaluation of detrusor instability.
• Urodynamic findings of detrusor constrictions usually support the diagnosis.

Solution for Question 9:


Option B: Patients with limited lymph node involvement may be cured by surgery alone

Page 17

1089
• The patient mentioned above is suffering from carcinoma bladder.

• Patients with limited lymph node involvement may be cured with surgery alone.
• Treatment of Bladder Cancer: Early-stage: Surgery (TUR or transurethral resection of the bladder)
followed by intravesical chemotherapy (i.e., Mitomycin) or immunotherapy (BCG). Muscle Invading
Cancer: Surgery (total or partial cystectomy) may be preceded by chemotherapy or followed by
chemotherapy and radiation, depending on the stage. Nd: YAG laser can also be used in the resection
of bladder cancer. Advanced: Chemotherapy or Radiation.
• Early-stage: Surgery (TUR or transurethral resection of the bladder) followed by intravesical
chemotherapy (i.e., Mitomycin) or immunotherapy (BCG).
• Muscle Invading Cancer: Surgery (total or partial cystectomy) may be preceded by chemotherapy or
followed by chemotherapy and radiation, depending on the stage.
• Nd: YAG laser can also be used in the resection of bladder cancer.
• Advanced: Chemotherapy or Radiation.
• Early-stage: Surgery (TUR or transurethral resection of the bladder) followed by intravesical
chemotherapy (i.e., Mitomycin) or immunotherapy (BCG).
• Muscle Invading Cancer: Surgery (total or partial cystectomy) may be preceded by chemotherapy or
followed by chemotherapy and radiation, depending on the stage.
• Nd: YAG laser can also be used in the resection of bladder cancer.
• Advanced: Chemotherapy or Radiation.
Option A: For patients with bladder cancer invading the bladder muscle (T2 lesion), immediate radiothe
rapy followed by surgery offers the best chance of cure
• For patients with disease invading into bladder muscle (T2), immediate (within three months of
diagnosis) cystectomy with extended lymph node dissection offers the best chance of survival.
Option C: Continent neo-bladders have yet to be successfully utilized in patients undergoing cystectom
y
• YAG laser can also be used in the resection of bladder cancer.
Option D: Intravesical chemotherapy before surgery is routinely used for bladder cancers invading the
bladder muscle (T2 lesion)
• Surgery (TUR or transurethral resection of the bladder) followed by intravesical chemotherapy (i.e.,
Mitomycin) or immunotherapy (BCG).

Solution for Question 10:


Correct Option B - Calcium oxalate dihydrate:
• The patient is suffering from a stone, visible as spiculated, and mobile calcification. The composition
of this stone is calcium oxalate dihydrate. Stone has spikes. This stone is known as the Jack stone. The
composition is calcium oxalate dihydrate.
• Stone has spikes. This stone is known as the Jack stone.
• The composition is calcium oxalate dihydrate.

Page 18

1090
• Stone has spikes. This stone is known as the Jack stone.
• The composition is calcium oxalate dihydrate.
Incorrect Options:
Option A - Calcium oxalate monohydrate:
• Stones are radio-opaque (though less than calcium phosphate stones).
• Hyperuricosuria may cause uric acid stones to which calcium oxalate binds.
Option C - Calcium Phosphate:
• It may occur in renal tubular acidosis; high urinary pH increases the supersaturation of urine with
calcium and phosphate.
• Renal tubular acidosis types 1 and 3 increase the risk of stone formation (types 2 and 4 do not).
• Radio-opaque stones (composition similar to bone).
Option D - Magnesium and ammonium:
• Stones formed from magnesium, ammonium and phosphate.
• Occur as a result of urease-producing bacteria (and are thus associated with chronic infections).
• Under the alkaline conditions produced, the crystals can precipitate.
• Slightly radio-opaque.

Solution for Question 11:


Option D: Malakoplakia
• This is a case of malakoplakia in which the patient presents with complaints of frequency, urgency,
dysuria, gross hematuria and right flank pain for one month. The patient is a known diabetic. The
cystoscopy showed multiple nodular brush-like lesions with easy bleeding in the bladder.
• All these findings are suggestive of malakoplakia.
• Malakoplakia is an infection of the bladder.
• It is believed to result from the inadequate killing of bacteria macrophages or monocytes that exhibit
defective phagolysosomal activity.
Option A: Urothelial carcinoma
• Urothelial carcinoma is a malignant tumor of the urinary tract originating from the mucosa of the
urinary tract.
• It is usually asymptomatic or is associated with painless hematuria.
Option B: Polypoid cystitis
• Polypoid cystitis is a reversible inflammatory exophytic lesion of the mucosa of the bladder.
• Histologically, it shows mildly hyperplastic urothelium.
Option C: Squamous cell carcinoma of the bladder
• Squamous cell carcinoma of the bladder can occur in multiple areas of the bladder, but the lateral wall
and trigone are the most common sites.

Page 19

1091
• On cystoscopy, it appears as nodular and has a plaque-like, irregular surface.

Solution for Question 12:


Correct Option A -Man is both the definitive and intermediate host.
this is incorrect as in Schistosomiasis, The organism responsible is S. Haematobium.
Man is definitive host and the intermediate host is “snail”.

• Histopathology of bladder mucosa shows the eggs of S.hematobium surrounded by intense


inflammatory infiltration in granuloma

Incorrect Options:
Option B - most common cause of bladder calcification:
• Schistosoma hematobium infestation (bilharziasis) is the most common cause of bladder wall
calcification
Option C - Swimmer’s itch is the first clinical sign:
• The site of penetration of Cercaria produces severe itching known as SWIMMER’S ITCH within 24
hours.
Option D - Increases the risk of both SCC and TCC:
• Schistosomiasis has an increased risk of Squamous cell carcinoma > Transitional cell carcinoma

Solution for Question 13:


Correct Option A - Extraperitoneal bladder rupture:

Page 20

1092
• This is a case of extraperitoneal bladder rupture in which the patient presents to the emergency
department after a road traffic accident with complaints of suprapubic pain, difficulty in the ability to
pass urine and hematuria.
• The x-ray of the pelvis reveals a pelvic fracture.
• The cystography of the patient is given in the case, which reveals a flame sign and teardrop-shaped
bladder, which is suggestive of extraperitoneal bladder rupture.
• In extraperitoneal bladder rupture, the bladder is clearly visible in the pelvis.
Incorrect Options:
Option B - Intraperitoneal bladder rupture:
• In intraperitoneal bladder rupture, there is history of an external blow to the bladder, a distended
bladder and peritonitis.
• cystrography shows intraperitoneal contrast material around bowel loops, mesentric folds and
paracolic gutters.
Option C - Bladder contusion:
• An incomplete or partial thickness tear in the bladder results in bladder contusion.
• It usually results due to blunt trauma to the bladder.
• Cystography in bladder contusion reveals hematoma in the bladder.
Option D - Posterior urethral injury:
• In posterior urethral injuries, there are significant bladder fractures due to trauma.
• The membranous bulb junction is most commonly injured in this case.

Solution for Question 14:


Option C: The most common site is the posterolateral wall of the trigone
• This is a case of transitional cell carcinoma of the bladder in which the patient presents with
complaints of painless gross hematuria for one day.
• On cystoscopy of the patient, there is a mass in the bladder. Biopsy revealed a tumor of transitional
epithelium. The bladder tumor antigen in the urine is positive. All these findings suggest transitional cell
carcinoma of the bladder.
• The most common site of transitional cell CA of the bladder is the posterolateral wall of the trigone.
Clinical Features
• The most common symptom is – Hematuria.
• Painless, gross intermittent hematuria
• Irritative symptoms
• Frequency
• Urgency
• Dysuria
Option A: Preferred systemic chemotherapy is cyclophosphamide

Page 21

1093
• Cyclophosphamide is not preferred as systemic chemotherapy in transitional cell carcinoma of the
bladder.
• Systemic chemotherapy agents which are used for transitional CA of the bladder include:
Methotrexate Vincristine Adriamycin Cisplatin
• Methotrexate
• Vincristine
• Adriamycin
• Cisplatin
• Methotrexate
• Vincristine
• Adriamycin
• Cisplatin
Option B: The most common site of lymphatic metastases is the axillary lymph node
• The axillary lymph node is not the most common site of lymphatic metastasis in this case.
• The most common group of lymph nodes involved in transitional cell carcinoma of the bladder are
pelvic lymph nodes, which are also known as obturator nodes.
Option D: Most effective intravesical chemotherapy is methotrexate
• BCG is the most effective intravesical chemotherapy in transitional cell carcinoma.
• Other intravesical chemotherapy agents include:
• Mitomycin-C
• Epirubicin
• Thiotepa
• BCG (most effective)
• Any T, N+, M+ → Neoadjuvant chemotherapy followed by surgery/palliation.

Solution for Question 15:


Correct Option A - Cloacal exstrophy:
• This is a case of cloacal exstrophy in which the patient presents with complaints of organs of the
urinary tract bulging out of the abdomen since birth.
• It is a severe birth defect in which most of the abdominal organs, such as the bladder & intestines, are
exposed.
• It causes the splitting of the bladder, genitalia & anus.
• It is caused by a defect of the ventral body wall: Mesodermal migration is inhibited & folding fails.

Page 22

1094
Incorrect Options:
Option B - Omphalocele minor:
• Omphalocele is a defect in the ventral abdominal wall that will result in herniation of abdominal
organs.
• In omphalocele minor, there is a protrusion of only a small portion of the intestine, and the size of the
defect is less than 5 cm.
Option C - Omphalocele major:
• Omphalocele is a defect in the ventral abdominal wall that will result in herniation of abdominal
organs.
• In omphalocele major, there is a protrusion of the intestines and liver, and the size of the defect is
more than 5 cm.

Option D - Gastroschisis:

Page 23

1095
• Gastroschisis is a defect in the ventral wall which results in paraumbilical herniation of the intestine
through the abdominal wall.
• In gastroschisis, there is no formation of the hernia sac.

Page 24

1096
Prostate and Seminal Vesicle
1. What is the suspected diagnosis of a patient with an enlarged prostate gland and a free total
concentration of PSA ratio of 17%?
(or)
A 68-year-old male complains of increased frequency of urination, urinary urgency, and hesitancy with
a weak stream. He also has reported hematuria and incontinence. The patient has an enlarged prostate
gland with several palpable nodules and a ratio of the free total concentration of PSA was 17%. A
biopsy through transrectal ultrasound was indicated. what is the suspected diagnosis in this patient?
A. Adenocarcinoma prostate stage 4
B. Benign prostate hyperplasia
C. Adenocarcinoma prostate stage 2
D. Acute prostatitis
----------------------------------------
2. A 16-year-old boy presented to the emergency department with suprapubic pain, difficulty urinating,
hematuria, and high-grade fever for 4 days after trauma to the lower abdomen. He had some
discomfort in the suprapubic region that seemed to be improving. Two days after the incident, the
patient developed a fever and nausea with vomiting. He also described difficulty voiding with blood in
his urine. A rectal exam showed a mildly enlarged smooth prostate with mild tenderness to palpation of
the right base and right lateral aspect. There was also a small area of ecchymosis in the suprapubic
region. The CBC report showed a WBC count of 12,000/microliter. What is the common diagnosis in
this patient?
(or)
A 16-year old boy presents with suprapubic pain, difficulty urinating, hematuria, and high-grade fever
for 4 days after trauma to the lower abdomen. A rectal exam showed a mildly enlarged smooth prostate
with mild tenderness to palpation and ecchymosis in the suprapubic region. The CBC report showed
WBCs count of 12,000/microliter. What is the diagnosis of this patient?
A. Carcinoma of prostate
B. Benign prostatic hyperplasia
C. Acute prostatitis
D. Prostatic abscess
----------------------------------------
3. A 66-year-old male presented to the local hospital with complaints of urinary retention with severe
pain and burning sensation in the lower urinary tract since yesterday. He also has a poor urinary
stream, nocturia, and post-void dribbling. The vital signs and Laboratory tests were normal. On digital
rectal examination, there is a palpable, hard, nodular prostate with an obliterated median sulcus.
Ultrasound reports showed an enlarged prostate of weight 93 grams. His ratio of free: total PSA is
estimated as 20%. The ultrasound of this patient is given below. What diagnosis does the doctor
suspect in this case?
(or)
A 66-year-old male presents with complaints of poor urinary stream, nocturia and postvoid dribbling. On
digital rectal examination, there is a palpable prostate with smooth muscle hypertrophy. Ultrasound
reports showed enlarged prostate . His ratio of free total PSA is estimated that is 20%. What is the

1097
diagnosis?

A. Tuberculous prostate
B. Benign prostatic hyperplasia
C. Acute prostatitis
D. Carcinoma of prostate
----------------------------------------
4. A 62-year-old male comes to the emergency department with blood in his urine, urinary incontinence,
urgency, and hesitancy. DRE found a hard, nodular, and fixed prostate. The PSA ratio is 15%. Biopsy
indicates carcinoma of the prostate, staged as stage 4 adenocarcinoma on MRI. What is the best
treatment?
(or)
A 62-year-old male presents with complaints of blood in the urine and constant low-grade back and leg
pain. A hard, nodular and fixed prostate is felt on digital rectal examination. The PSA ratio is 15%.
Diagnosis of stage 4 adenocarcinoma of the prostate is made. What is the best treatment in stage 4
adenocarcinoma?
A. Radical prostatectomy and medical castration
B. Transurethral resection of the prostate and hormonal ablation
C. Active surveillance and radiotherapy
D. Orchidectomy and radiotherapy only
----------------------------------------
5. Which statement about the treatment of patient with prostatic cancer correct?
(or)
A 65-year-old male has come to the hospital with urinary symptoms such as urinary incontinence,
hesitancy and urgency for 2 months. A hard, nodular and fixed palpable prostatic tumor is present on
digital rectal examination. The prostatic specific antigen (PSA) value is measured at 12ng/ml.He has no
clinical features of metastasis to any other organ. His transrectal biopsy is done, which confirms
prostatic cancer. The MRI and CT of pelvic areas show no extracapsular invasion of the tumor. Which
statement is correct about the treatment of this patient?
A. Orchidectomy and radiotherapy
B. Active surveillance
C. Radical prostatectomy is indicated

Page 2

1098
D. Hormonal ablation due to an old aged patient
----------------------------------------
6. A 68-year-old male with benign prostate hyperplasia presents with blood in urine and severe pain.
Recent tests showed multiple bladder stones. He's been taking alpha-blockers for 6 years and is
prepared for surgery. During surgery, he developed vomiting and altered sensorium. What's the most
probable cause?
(or)
what is the cause of postoperative vomiting and altered sensorium in a patient with BPH?
A. Dilutional hyponatremia
B. Dilutional hyperkalemia
C. Rupture of bladder
D. Hemorrhage
----------------------------------------
7. A 66-year-old male presented with symptoms of urinary hesitancy, poor urinary stream, post-void
dribbling, and urine retention. The prostate was hard and nodular on digital rectal examination. Imaging
showed extensive, calcified areas of the prostatic parenchyma. What is the correct statement about this
condition?
(or)
Which is the correct statement about the given condition?

A. Benign prostate hyperplasia is a cause of prostate stones


B. Usually present at the peripheral zone of the prostate
C. No treatment is needed in this patient
D. No lamellar bodies are formed in prostatic calculi
----------------------------------------
8. A 70-year old male underwent transurethral resection of the prostate for BPH under spinal
anaesthesia. The patient is positioned in a lithotomy posture, and the surgery is done successfully.
What is the most common complication that can occur because of the injury during surgery?
(or)
What is the most common complication that can occur because of the injury during TURP ?
A. Erectile dysfunction

Page 3

1099
B. TURP syndrome
C. Retrograde ejaculation
D. Urinary incontinence
----------------------------------------
9. A 70-year old male presented with blood in urine, urinary urgency, hesitancy with a weak stream,
and low-grade but constant back pain. Digital rectal examination revealed an enlarged prostate gland
with several nodules. Biopsy results showed adenocarcinoma and MRI report revealed stage 4
carcinoma. Surgery is advised for symptomatic relief. What type of surgery is indicated?
(or)
Which type of surgery is indicated for adenocarcinoma stage 4 in a 70 year old male?
A. TURP surgery
B. Millin prostatectomy
C. Freyer’s prostatectomy
D. Young’s prostatectomy
----------------------------------------
10. A couple presents with complaints of not having a child for 6 years after unprotected intercourse for
the same period. The female partner has regular menstruation. She is assessed for ovarian reserves,
which are 25 antral follicles. The male partner semen analysis is assessed. His sperm concentration is
100 M/ml, total sperm motility is 50%, and sperm morphology is 2%. What is the cause of infertility?
(or)
A couple (the male is 32 years old and the female is 30 years) presents in the gynecology outpatient
department with complaints of not having a child for 6 years after unprotected intercourse for the same
period. They married 7 years ago and trying to conceive after 1 year of marriage. The male partner has
no obvious disorders, and his semen analysis is assessed. The female partner has regular
menstruation and no other gynaecological disorders. She is assessed for ovarian reserves, which are
25 antral follicles. The semen analysis is given below: Semen volume 3 ml Sperms concentration 100
M/ml Total sperms No. per ejaculate 70 M/ml Total sperms motility 50% Sperms morphology 2% The
couple is advised to try intrauterine insemination. What is the cause of infertility in this case?
A. Sperms concentration
B. Total sperms motility
C. Ovarian reserves
D. Sperms morphology
----------------------------------------
11. A 35-year-old man presented with fever, difficulty in urination, and groin pain for 2 days. He also
complained of lower back and perineal pain for the past month. On examination, mild tenderness and
swelling of the prostate were noted. CBC and urine examination were done, revealing leukocytosis and
findings consistent with bacterial infection. What's the probable diagnosis?
(or)
A 35-year-old male presents with fever, difficulty in urination and pain in the groin. He also worried
about his lower back and perineal pain for 1 month. CBC reveals a WBCs count of 12000/microliter of
blood, and urine examination revealed leucocytosis and findings consistent with a bacterial infection.
What is the diagnosis?

Page 4

1100
A. Pyelonephritis
B. Acute prostatitis
C. Benign prostatic hyperplasia
D. Nephrolithiasis
----------------------------------------

Correct Answers
Question Correct Answer

Question 1 1
Question 2 3
Question 3 2
Question 4 2
Question 5 3
Question 6 1
Question 7 1
Question 8 3
Question 9 1
Question 10 4
Question 11 2

Solution for Question 1:


Option A: Adenocarcinoma prostate stage 4
Prostatic carcinoma is a type of cancer that develops in the prostate gland.
Signs and Symptoms:
Urinary frequency, Urgency, Hesitancy, Weak stream, Hematuria, Incontinence, Low-grade constant b
ack pain, Cough: Due to metastatic pulmonary deposits. Palpable hard, nodular prostate is a
feature of prostatic carcinoma.
Investigations:
•Digital rectal examination: Used to detect the presence of a palpable hard, nodular prostate
•Free total concentration of prostatic specific antigen (PSA) ratio: Estimated to differentiate between ca
rcinoma and benign prostatic hyperplasia .
normal values of PSA (4ng/ml).
• Biopsy through transrectal ultrasonography: Indicated if the PSA ratio is <18%
•Imaging tests: Used to detect the presence of metastasis
Treatment:
•Hormonal ablation: Through bilateral subcapsular orchidectomy or medical castration through luteinizi
ng hormone-releasing hormone agonists. •Radiotherapy: IV radiopharmaceutical agent strontium-89 is

Page 5

1101
given because of multiple metastases.
Option B: Benign prostate hyperplasia
• This patient's free total PSA ratio is < 18%, so it does not suggest benign prostate hyperplasia.
• Free PSA is usually high in non-cancerous conditions such as BPH.
Option C: Adenocarcinoma prostate stage 2
• In stage 2 carcinoma, there are no metastatic deposits to the other organs, such as bones, lungs and
liver.
• The patient has metastatic deposits in bone and lungs, so it can't be staged as carcinoma 2.
Option D: Acute prostatitis
• It is the inflammation of the prostate, usually due to any lower abdominal trauma or urinary tract
infection.
• On digital rectal examination, the prostate is warm and tender. Leukocytosis is the typical lab finding.

Solution for Question 2:


Option C: Acute prostatitis
• The patient had trauma to the lower abdomen after an accident. The presence of ecchymosis on the
suprapubic region also indicates the trauma due to which infection may be passed to the prostate
causing symptoms of urinary tract infection.
• After a trauma to the suprapubic region, the young patient with symptoms of high-grade fever, dysuria,
blood in urine, and suprapubic pain strongly suggests acute bacterial prostatitis.
• The most common causative bacteria are E-coli, streptococcus or staphylococcus.
• He has no family history of any prostatic disease, and his insignificant past medical and drug history
also supports acute prostatitis.
• A warm, tender, and swollen smooth prostate is present on digital rectal examination. In CA or BPH,
prostate is hard and nodular..
• The CBC report showed low Hb due to blood loss in urine and high WBC value showing the presence
of acute infection.
• The treatment in this patient is an antibiotic that penetrates the prostate, such as trimethoprim or
ciprofloxacin for 2 months.
Option A: Carcinoma of the prostate
• It usually occurs after 65 years of age.
• A hard, nodular and craggy prostate is present on digital rectal examination.
Option B: Benign prostatic hyperplasia
• The presenting age is above 60 years.
• The hard, nodular prostate with obliterated median sulcus is the finding on digital rectal examination.
Option D: Prostatic abscess

Page 6

1102
• It is the sequelae of untreated acute prostatitis. The prostate is extremely tender and fluctuant in this
case.
• Diagnosis is made by the presence of persistent threads in the urine.

Solution for Question 3:


Correct Option B - Benign prostatic hyperplasia:
It is a suspected case of benign prostatic hyperplasia due to urinary symptoms such as urinary retentio
n, poor urinary stream, nocturia, and postvoid dribbling in the old male.
Benign prostatic hyperplasia (BPH) is a benign enlargement of the prostate that occurs after 50 years,
usually between 60 and 70 years. BPH affects both glandular epithelium and connective tissue stroma.
It is involuntary hyperplasia due to disturbance of the ratio and quantity of circulating androgens and e
strogens.
•On digital rectal examination, a
palpable prostate with smooth muscle hypertrophy is the typical finding of benign prostatic hyperplasia.
•The free: total prostate-specific antigen ratio discriminates between prostate carcinoma and benign pr
ostate hyperplasia. If it is < 18%, it is suggestive of carcinoma; if it is > 18 %, it suggests benign prostat
e hyperplasia, as it is 20% in this patient, indicating benign prostate hyperplasia.
Incorrect Options:
Option A - Tuberculous prostate:
• As there is no history and symptoms of tuberculosis, this disease does not correlate with the patient's
condition.
• It is diagnosed by urine examination for ZN staining and culture of tubercle bacilli from the first
morning urine sample.
Option C - Acute prostatitis:
• It is the inflammation of the prostate due to any lower abdominal trauma or secondary to urinary tract
infection.
• On palpation, there is a warm and tender prostate. Leukocytosis is the typical lab finding.
Option D - Carcinoma of the prostate:
• The free: total ratio of PSA is the differentiating point between carcinoma and benign prostate
hyperplasia.
• If it is > 18%, it is suggestive of benign prostate hyperplasia as this patient has a 20% ratio, so it
excludes carcinoma of the prostate.

Solution for Question 4:


Option B: Hormonal ablation and radiotherapy
• It is the case of adenocarcinoma stage 4 because of urinary symptoms such as hematuria, urinary
incontinence, urgency and hesitancy.

Page 7

1103
• The metastatic deposits are present in vertebras, femur bone and bone marrow, causing low back
pain, leg pain and pancytopenia.
• The CBC report shows low Hb, low WBCs, and low platelets manifest metastatic deposits in the bone
marrow.
• The digital rectal examination shows a palpable, hard, fixed, and nodular prostate typically found in
carcinoma.
• His PSA ratio is < 18%, which is suggestive of carcinoma. The biopsy confirms the diagnosis of
carcinoma.
• If the urinary outflow obstruction is present, then palliative surgery, i.e. transurethral resection of the
prostate, is done. It is the procedure in which a 26Fr resectoscope with a 30-degree telescope is
inserted per-urethra into the bladder. Cysto-urethroscopy is done. Tissues are cut by a diathermy
loop mounted in front of the scoop. First median lobe and then lateral lobes are resected.
• Other treatment options are: Hormonal ablation by bilateral subcapsular orchidectomy and medical
castration by luteinising hormone-releasing hormone agonists such as goserelin. This option aims to
remove the testosterone source, as prostate carcinoma is a testosterone-dependent tumor.
Radiotherapy by IV radiopharmaceutical agents such as strontium-89. It delivers effective radiotherapy
to metastatic areas.
• Hormonal ablation by bilateral subcapsular orchidectomy and medical castration by luteinising
hormone-releasing hormone agonists such as goserelin. This option aims to remove the testosterone
source, as prostate carcinoma is a testosterone-dependent tumor.
• Radiotherapy by IV radiopharmaceutical agents such as strontium-89. It delivers effective
radiotherapy to metastatic areas.
• Hormonal ablation by bilateral subcapsular orchidectomy and medical castration by luteinising
hormone-releasing hormone agonists such as goserelin. This option aims to remove the testosterone
source, as prostate carcinoma is a testosterone-dependent tumor.
• Radiotherapy by IV radiopharmaceutical agents such as strontium-89. It delivers effective
radiotherapy to metastatic areas.
Option A: Radical prostatectomy and medical castration
• Radical prostatectomy is open or laparoscopic surgery in the case of T1, T2 or T3a stage of
carcinoma.
• It removes the entire prostate, surrounding tissues, seminal vesicles, and bilateral pelvic lymph
nodes.
Option C: Active surveillance and radiotherapy
• The localized T1a stage is managed by watchful wait and active surveillance with DRE and PSA
(every 3-6 months).
• Treatment is considered if there is a progression of the disease.
Option D: Orchidectomy and radiotherapy only
• If there are symptoms of urinary outflow obstruction, then only orchidectomy is not recommended, but
trans-urethral resection of the prostate is also needed.

Solution for Question 5:

Page 8

1104
Option C: Radical prostatectomy is indicated
• It is diagnosed to be a case of prostatic carcinoma that is most probably T2. The patient presents with
urinary symptoms, e.g., hesitancy, incontinence, and urgency.
• A hard, palpable tumor, PSA value of 12 ng/ml, biopsy showing cancerous cells, and MRI or CT scan
of the pelvis area all confirm the presence of carcinoma.
• The T1 stage tumor is not palpable by digital rectal examination compared to the hard palpable
nodular tumor in this case.
• No metastasis negates stage 4, where other organs are involved.
• Stage 3 carcinoma has extracapsular invasion, but MRI and CT scan of this patient show no
extracapsular invasion.
• The palpable, confined to the capsule, and no metastasis strongly suggest T2 prostatic carcinoma.
The treatment will be according to T2.
• Radical prostatectomy should be done in patients with a life expectancy of > 10 years and no lymph
node involvement or metastatic deposits.
• The complications associated with this surgery are: Impotence Urinary incontinence.
• Impotence
• Urinary incontinence.
• Radiotherapy can also be given to patients, but the chance of recurrence is high.
• Impotence
• Urinary incontinence.
Option A: Orchidectomy and radiotherapy
• The Orchidectomy can be done as hormonal ablation surgery in the management of stages 3 and
stage 4.
• It eliminates the major source of testosterone production.
Option B: Active surveillance
• In young patients with low-grade tumors and stage T1a, active surveillance is done with the routine
digital rectal examination and PSA measurements (3-6 monthly).
• Further treatment is considered if there is a progression of the disease.
Option D: Hormonal ablation due to an old aged patient
• Hormonal ablation occurs when multiple metastases and the tumor have progressed to high stages.
• Subcapsular orchidectomy and medical castration are done to eliminate testosterone production.

Solution for Question 6:


Option A: Dilutional hyponatremia
• Trans urethral resection of the prostate (TURP) is the surgical procedure for benign prostate
hyperplasia. The indications of surgery in this patient are : Failure of medical treatment Bladder stones
are a complication of BPH

Page 9

1105
• Failure of medical treatment
• Bladder stones are a complication of BPH
• Cysto-urethroscopy is done. It is the procedure in which a 26Fr resectoscope with a 30-degree
telescope is introduced per-urethra into the bladder. The prostate is visualized and assessed for size.
The bladder is seen for stones. Tissues are cut from the bladder neck by a diathermy loop mounted in
front of the scoop. First median lobe and then lateral lobes are resected. Chips of a prostate pass to the
bladder, and these pieces and stones are evacuated by the Ellik evacuator.
• For continuous irrigation, a three-way Foley catheter is passed. The irrigation is done with normal
saline to avoid clot retention.
• This continuous irrigation with fluid (glycine) may be absorbed into the circulation, leading to dilutional
hyponatremia. This condition is called TURP syndrome. It is the most common complication of TURP
surgery.
• The treatment is: Fluid restriction Diuretics Hypertonic saline 3% infusion
• Fluid restriction
• Diuretics
• Hypertonic saline 3% infusion
• Failure of medical treatment
• Bladder stones are a complication of BPH
• Fluid restriction
• Diuretics
• Hypertonic saline 3% infusion
• The risk of TURP syndrome increases with resection time > 90 minutes or gland size > 75 gm.
Option B: Dilutional hyperkalemia
• By irrigation of saline, hyponatremia occurs, which causes altered sensorium.
Option C: Rupture of bladder
• It can be a complication of surgery, but it is not associated with vomiting and altered sensorium.
Option D: Hemorrhage
• Hemorrhage can be a complication during this surgery, but to a less extent, that can be tolerated by
this patient.

Solution for Question 7:


Correct Option A - Benign prostate hyperplasia is a cause of prostate stones:
• The prostatic calculi are commonly present in old-aged males but are usually asymptomatic. But if
they cause any obstructive urinary symptoms, they must be treated.
• The above-given case is prostatic calculi diagnosed by the X-ray and the pelvis CT.
• Benign prostate hyperplasia is the inflammation of the prostate that causes the production of thick
secretions from the prostate. These thick secretions tend to bind with the proteinaceous substances in
the prostate. It causes fluid to become harder and tends to form into stones.

Page 10

1106
• The patient has urinary symptoms such as hesitancy, poor urinary stream and postvoid dribbling.
These obstructive urinary symptoms needs to be managed.
• Diagnosis may be made by: CT scan of the pelvis X-rays of the kidney and bladder Transrectal
ultrasound
• CT scan of the pelvis
• X-rays of the kidney and bladder
• Transrectal ultrasound
• CT scan of the pelvis
• X-rays of the kidney and bladder
• Transrectal ultrasound
• Treatment is usually unnecessary, but in complicated cases, antibiotics are required, or surgery is
done to remove the stones. The options are: Transurethral electro resection loop surgery Holmium
laser surgery
• Transurethral electro resection loop surgery
• Holmium laser surgery
• Transurethral electro resection loop surgery
• Holmium laser surgery
Incorrect Options:
Option B - Usually present at the peripheral zone of the prostate:
• These stones are usually present in the peripheral region of the transition zone of the prostate.
Option C - No treatment is needed for this patient:
• In asymptomatic patients, no treatment is needed for prostatic calculi.
• As this patient presents with obstructive urinary symptoms, treatment is needed.
Option D - No lamellar bodies are formed in prostatic calculi:
• The prostatic calculi usually tend to form in clusters (lamellar bodies)

Solution for Question 8:


Option C: Retrograde ejaculation
• The transurethral resection of the prostate (TURP) is the surgery done under spinal anesthesia.
• Retrograde ejaculation is the most common complication that occurs during surgery. It is due to injury
of the internal urinary sphincter.
• The internal urinary sphincter is under the control of the L1 sympathetic ganglion. And ejaculation is
under the sympathetic system.
Option A: Erectile dysfunction
• The erection is under the control of the parasympathetic system, so erectile dysfunction is not a
complication of TURP.

Page 11

1107
Option B: TURP syndrome
• The complication after surgery due to continuous irrigation leads to dilutional hyponatremia called
TURP syndrome.
Option D: Urinary incontinence
• It is the least common complication seen in < 1% of cases.

Solution for Question 9:


Option A: TURP surgery
• The old aged patient with stage T4 carcinoma who presents urinary symptoms is advised to undergo
least invasive surgery.
• T4 is usually managed with hormonal ablation and radiotherapy. Still, if there are lower urinary tract
symptoms, the patient must be treated with palliative surgery of transurethral resection of the prostate.
• The TURP is the surgical procedure in which a 26Fr resectoscope is inserted through the urethra into
the bladder. Cysto-urethroscopy is done. The prostate is assessed for size. Strips of tissues are cut
from the bladder neck using a diathermy loop. First median lobe and then lateral lobes are resected
afterwards. The Ellik evacuator evacuates pieces of the prostate to pass to the bladder.
• It is accompanied by other treatments such as medical treatment (LHRH agonists) and radiotherapy
to manage metastatic deposits, especially in bones.
Option B: Millin prostatectomy
• It is done using a retropubic approach that may cause disturbances in many patients.
• It is usually indicated in benign prostatic hyperplasia with very large adenoma ( > 75ml)
Option C: Freyer’s prostatectomy
• Freyer’s suprapubic prostatectomy uses the transvesical approach to remove the prostate gland.
• It is preferred for patients having bladder stones and bladder diverticulum.
Option D: Young’s prostatectomy
• Young’s prostatectomy uses a perineal approach. It is not preferred by urologists nowadays.

Solution for Question 10:


Option D: Sperms morphology
• The semen analysis of the male partner shows the above-given values. Normal morphology means
the presence of normal-shaped and sized sperms. Normal morphology has an oval head with a long
tail. Its normal range is 4-48%, as it is 2% in this patient, so it is the causative factor of infertility in this
case.
• Large or misshaped heads and crooked or double tails are abnormal shapes of sperms. These affect
the sperm's capability to penetrate the egg, causing infertility.
• Etiology:

Page 12

1108
• increased testicular temperature, exposure to toxic chemicals, genetic traits, and infections.
• Sperms concentration defines as the number of sperms per unit volume of semen. The normal range
is 15-259 M/ml as the patient has 100 m/ml, so it can not affect fertility.
• Semen volume is the total fluid that's ejaculated through the penis. The patient has a 3 ml volume that
is normal. The normal range is 1.5- 7.6 ml.
• Total sperms number per ejaculate is 39-928 M/ejaculate. The patient has its normal value.
• Total sperm motility is the percentage of sperm having any movement. Its normal range is 40-81 %.
The patient has its normal value.
• The ovarian reserve is the total no. of immature and healthy eggs in the ovaries. The normal value is
15-30 antral follicles. The female has 25 follicles, so it is not the cause of infertility.
Option A: Sperms concentration
• The normal values of sperm concentration are 15-259 M/ml.
• The male has sperm concentration within the normal range, i.e. 100M/ml, so it is not the cause of
infertility in this case.
Option B: Total sperms motility
• As the patient has total sperm motility within normal ranges, it may not affect fertility.
• The reference value is 40-81%, and the patient's value is 50%.
Option C: Ovarian reserves
• The normal ovarian reserves in normally menstruating women are 15- 30 antral follicles.
• The female has a normal ovarian reserve, i.e. 25 antral follicles, so it may not cause infertility.

Solution for Question 11:


ANSWER
Option B: Acute prostatitis
• A middle-aged male with complaints of fever, urinary symptoms and groin pain is the typical case of
acute prostatitis.
• It is also associated with low back and perineal pain.
• The digital rectal examination palpates a mild, tender, swollen prostate due to bacterial infection. The
digital rectal examination should be done cautiously to avoid the risk of septicemia.
• The leukocytosis in blood and urine reports is due to infection and inflammation in the body.
• The diagnosis is made by: Digital rectal examination CBC and urine reports show leukocytosis.
• Digital rectal examination
• CBC and urine reports show leukocytosis.
• The treatment is with the antibiotics such as trimethoprim and ciprofloxacin because of their
penetration ability into the prostate.
• Digital rectal examination
• CBC and urine reports show leukocytosis.

Page 13

1109
Other options
Option A: Pyelonephritis
• It is not associated with pain in the perineum.
• The swollen and tender prostate negates this disease.
Option C: Benign prostatic hyperplasia
• It is usually present above 60 years old male.
• The fever and the leukocytosis are not associated features of this disease.
Option D: Nephrolithiasis
• Nephrolithiasis is not associated with perineal pain.

Page 14

1110
Urethra and Penis
1. A 17-year-old male comes to the hospital with a congenital condition of a curved penis. Clinical
examination shows he has incomplete prepuce and distal meatus. He has a poor urinary stream and
has to sit to urinate. Medical history includes correction surgery at the age of 21 months. Again, the
surgery is considered to improve fertility and allow voiding while standing. Which of the following is
considered true regarding the condition shown in the image?.
(or)
A 17-year-old male presents with a congenital condition of a curved penis. On examination, he has an
incomplete prepuce, distal meatus, and a poor urinary stream. Correction surgery was done at the age
of 21 months. Again, the surgery is considered to improve fertility and allow proper voiding. What
statement is true regarding this condition?

A. It is attributed to the failure of complete urethral tabularization in the fetus.


B. The urethral opening is most commonlyseen in the perineum
C. The urethra opens proximally and dorsally
D. It is seen in 1 in 1500 boys
----------------------------------------
2. What surgery can be done in penoscrotal hypospadias with mild chordee?
(or)
Surya, an 8-month-old baby boy, was brought to the clinic with an abnormal looking penis and passage
of urine from the underside of the penis almost near the scrotum. On examination, he had a condition
called penoscrotal hypospadias with mild chordee (abnormality where the penis is bent downwards). In
this patient, which one of the following surgeries can be done?
A. Circumcision
B. Ombredann’sprocedure
C. Duckett technique
D. External beam radiation
----------------------------------------
3. A patient presents with rectal bleeding and purulent drainage associated with a perianal tumour,
pain, and weight loss. On examination, a large ulcerated, hard and brittle lesion is noted, occupying the
entire right Hemi-circumference. The patient underwent endovaginalus, which revealed a bulky mass to
the right of the anal canal measuring 4.1 cm×2.4 cm. What is the feature of this tumour?

1111
(or)
A 38-year old female presents with a picture of rectal bleeding, elimination of purulent drainage
associated with a perianal tumor, pain, and weight loss. On physical examination, a large ulcerated,
hard and brittle lesion was noted, which extended up to 2 cm from the anal margin, occupying the entire
right Hemi-circumference, with the presence of pus, blood, and cellulitis in the gluteal region. The
patient was hospitalized and underwent the endovaginal USG, which revealed a bulky mass to the right
of the anal canal measuring 4.1 cm×2.4 cm with no signs of an abscess. What is the feature of this
tumor?
A. Complicated perianal abscess(i.e ruptured).
B. Malignant transformation in anogenital wart
C. Malignant transformation in common wart
D. Malignant transformation in seborrheic wart
----------------------------------------
4. A 4-year-old male child is referred to the pediatric department in the with complaints of an distended
abdomen, dribbling of urine with overflow incontinence. On physical examination, there is only a
palpable and distended bladder up to the umbilicus with no other abnormalities. A urinary
catheterization was performed, which drained 350 mL of clear urine, with immediate relief of the
symptoms. This condition in a male child may be due to the following:
(or)
A 4-year-old male child is brought with a complaints of an disteded abdomen and dribbling of urine. On
examination, the bladder is palpable and distended up to the umbilicus. A urinary catheterization
drained 350 mL of clear urine, immediately relieving the symptoms. What is the cause of his
symptoms?
A. Prostatic radiotherapy
B. Urethral stricture
C. Hysteria
D. Meatal ulcer with scabbing
----------------------------------------
5. A 25-year old male presents with pelvic hematoma and fracture following a road traffic accident.
Examination reveals a palpable bladder and blood at the external urethral meatus. A digital rectal
examination shows impalpable prostate. What is true about the patient's diagnosis?
(or)
A 25-year old male presents to ER following a road traffic accident sustaining injuries to the lower
abdomen. On examination, there is a pelvic hematoma and fracture. Physical examination shows a
palpable bladder, and blood at the external urethral meatus. A digital rectal examination shows
impalpable prostate. On the basis of examination, which of the following is true about the patient's
diagnosis?
A. The anterior urethra is the most likely place of injury
B. Retrograde urethrography should be done immediately
C. A Foley catheter can be carefully passed if the RGU is normal
D. Rectal examination may not reveal a large pelvic hematoma
----------------------------------------

Page 2

1112
6. A 10-year old boy presents to the hospital with his mother.The primary complaint is urinary
incontinence from birth. " When he feels the urge to void, he has a little warning before urine starts to
drain through his urethra. An ultrasound is taken, which shows mild hydronephrosis bilaterally. The
patient undergoes a voiding cystourethrogram which shows a dilated posterior (proximal) urethra and
some flaps of tissue that obstruct urine flow (image). Which of the following should be the probable
management for this patient?
(or)
A 10-year old boy presents with a complaint of the life-long problem of day and night urine incontinence
and being unable to pee across the bathroom like the other boys. He has a little warning before urine
starts to drain through his urethra. Ultrasound reveals mild hydronephrosis bilaterally. The voiding
cystourethrogram shows a dilated posterior (proximal) urethra and some flaps of tissue obstructing
urine flow (image). What should be the management of this patient?

A. Conservative management is enough as it spontaneously resolves with ageing.


B. The patient should be scheduled for Endoscopic fulguration of valves immediately as B/L
hydronephrosis has developed.
C. Suprapubic cystostomy can be performed, and then a polyethylene feeding tube should be used to
allow secondary healing.
D. Insert an infant feeding tube into the bladder and leave it for several days; Further management
should be done according to the S.creatinine level.
----------------------------------------
7. A 48-year-old man arrived at the hospital complaining of hesitancy and a slow urine stream. He
experiences moderate urinary frequency with nocturia occurring three times per night and feels like his
bladder is not completely empty. He also experiences perineal discomfort, painful urination of small
volumes, and deep pelvic pain that lasts for several hours after urination. During the physical
examination, a slightly enlarged prostate that is palpable is observed. Which of the following statements
is true regarding this condition?
(or)
which of the following statements is true regarding male lower urinary tract symptoms (LUTS)
A. Charting patient’s' symptoms according to the International Prostate Symptom Score (IPSS) is rarely
used
B. Flow rate measurement studies and pressure-flow urodynamic studies are done
C. A thorough digestive system examination is mandatory
D. Cystourethroscopy is a good guide as an indication of surgery

Page 3

1113
----------------------------------------
8. A 17-years old male presents to the hospital with complaints of soreness and swelling of the penis.
The primary complaint includes the inability to retract the penis foreskin, pain and burning sensation
while urinating. He also says that he suffers from pain during erections. Physical examination shows
that the patient is uncircumcised. Which of the following statements is correct regarding the diagnosis?
(or)
A 17-year old male presents with complaints of inability to retract the penis foreskin. He complains
about pain, a burning sensation while urinating, and pain during erections. The patient is
uncircumcised. Based on the diagnosis, what statement is correct?

A. The most common cause is STI.


B. Congenital and acquired
C. Ballooning of prepuce during erection
D. Circumcision is done in all the cases
----------------------------------------
9. A 3-year-old boy is admitted to the hospital with painful swelling of the glans penis, including
retraction of the foreskin behind the glans penis (shown in the image). The patient's history revealed
that while playing outside, he unintentionally retracted the foreskin and could not reduce it for 5 hours
before coming to the hospital. Which of the following statement is correct about this condition?
(or)
A 3-year-old boy presents with painful swelling of the glans penis with foreskin retracted behind the
glans penis for 5 hours. What statement is true regarding this condition?

A. Circumcision is done in this condition.

Page 4

1114
B. Decreased flow of urine
C. Pain in the back or groin
D. Frequent micturition
----------------------------------------
10. A 10-year-old male child underwent retrograde urethrogram, which shows urethral stricture. What is
the location of the stricture?
(or)
A 10-year-old male child has presented with obstructive urinary symptoms since birth. His parents said
that he passes urine drop by drop, accompanied by severe abdominal straining and takes
approximately 30 minutes to complete one episode of micturition. He also has fecal incontinence during
voiding due to severe straining. On external genital examination, the child is uncircumcised. The patient
underwent retrograde urethrogram, which shows urethral stricture. What is the location of stricture in
the given urethrogram?

A. Membranous urethra
B. Bulbar urethra
C. Penile urethra
D. Prostatic urethra
----------------------------------------
11. A 40-year-old presents with a history of curvature of the erect penis and severe painful erection,
unable to perform sexual intercourse for the last month. On examination, he had a non-tender, rubbery
plaque on the dorsolateral aspect of the left side of the penile shaft in its distal half. What statement is
true about this condition?
(or)
A 40-year-old, married young man presented in the outpatient department with a history of painful
erection and curvature of the erect penis for the last month. On examination, he had a non-tender,
rubbery plaque on the dorsolateral aspect of the left side of the penile shaft in its distal half. The
erection was so painful that the patient could not perform sexual intercourse. Which of the following
statement is true about this condition?
A. Medical treatment is enough for spontaneous resolution
B. Chronic vasculitis involving tunica adventitia and presents with fibrous plaque over the dorsolateral
aspect of the penis
C. Galizia's Triad is Dupuytren’s contracture + Peyronie’s disease + Retroperitoneal fibrosis

Page 5

1115
D. Transurethral resection of the prostate is performed
----------------------------------------
12. A 38-year old male underwent a laparoscopic right-sided hemicolectomy because of colon
carcinoma. The surgery went successful. After one day of persistent painful penile erection, the patient
notified the treating physician. On close inspection, painful engorgement of the penis in the absence of
sexual arousal was observed. Which of the following statement is correct about this condition?
(or)
What statement is correct about priapism in a post operative adult male?
A. Low flow priapism occurs secondary to perineal trauma
B. Low flow priapism can lead to compartment syndrome
C. Diagnosed mainly by panoramic radiography
D. Whipple procedure is done
----------------------------------------
13. A 55-year-old man has an ulceroproliferative lesion on his glans penis with a foul-smelling
discharge. A biopsy revealed squamous cell carcinoma of the penis. A CT scan is recommended to rule
out metastasis. Which of the following statements is correct about carcinoma of the penis?
(or)
Which of the following statements is correct about carcinoma of the penis?
A. Neonatal circumcision is of no use.
B. Leukoplakia is treated by YAG laser
C. More than 50% of patients present with enlarged inguinal lymph nodes
D. Metastasis in the liver is the cause of immediate death.
----------------------------------------
14. A 59-year-old male has a non-healing ulcer in his penis with a smelly discharge under the foreskin.
He also reported unusual weight loss and a 1x1 cm lesion was noted in the prepuce. What's the best
treatment?
(or)
A 59-year old male patient presents with complaints of a non-healing ulcer in the penis with a smelly
discharge under the foreskin for 4 weeks. On examination, an ulcer-proliferative lesion of size 1x1 cm is
noted in the prepuce. What is the most appropriate treatment for this condition?
A. Total penectomy
B. Partial penectomy
C. Balanoplasty
D. Circumcision
----------------------------------------
15. A 68-year-old male patient with previous history of urinary tract infections presents with difficulty
passing urine, weak flow, a lump on the penis, and blood in urine. Lymph nodes in the groin area are
enlarged. What is the most appropriate statement based on his symptoms?
(or)

Page 6

1116
A 68-year old male patient presents with complaints of dysuria, hematuria, weak urine flow, and a lump
on the penis for one month. On examination, lymph nodes in the groin area are enlarged. Given the
likely underlying condition, which statement is the most appropriate?
A. The most common type is TCC > SCC.
B. The most common site is the bulbomembranous urethra
C. The investigation of choice for soft tissue evaluation is a CT scan.
D. Radiotherapy is the mainstay of treatment.
----------------------------------------
16. A 5 months old baby boy of a Muslim family is brought by his parents to the urology outpatient
department. They wanted to go for a surgical procedure on the penis. The doctor advised complete
blood count, clotting time and bleeding time. All investigations are within normal range. Parents are
very concerned about the procedure and asked for details. Which of the following is true regarding this
procedure?
(or)
A 5 months old Muslim baby boy is brought by his parents for a surgical procedure on the penis. Which
of the following is true regarding this procedure?
A. Risk of Hemorrhage due to bleeding from a frenular artery
B. Increases sexual drive
C. Decreased sexual drive
D. Increased sexually transmitted infections
----------------------------------------
17. A 10 months old baby boy is brought by her parents to the urology outpatient department with
complaints of an abnormal-looking penis and passage of urine from the underside of the penis almost
near the scrotum. On examination, the opening of the urethra is located where the penis meets the
scrotum with mild chordee (abnormality where the penis is bent downwards). Surgery was planned for
the patient, identify and the correct order of correction of the above-given condition in this patient?
(or)
The order of correction of the penoscrotal hypospadias should be?
A. Straightening of the penis – Balanoplasty – Urethroplasty
B. Urethroplasty – Balanoplasty – Straightening
C. Straightening of the penis – Urethroplasty – Balanoplasty
D. Balanoplasty – Urethroplasty – Straightening
----------------------------------------
18. A 40-year-old male presents to the urology outpatient department complaining of a painful erection
without sexual drive. He complains of severe pain and discomfort associated with severe nausea. The
patient was previously diagnosed with depression and is taking antipsychotic medication. The patient is
also addicted to alcohol and some drugs. The cut-off duration for the diagnosis is:
(or)
A 40-year old male presents complaining of a painful erection without sexual drive associated with
severe nausea. The patient is taking antipsychotic medication and is addicted to alcohol and some
drugs. Cut off duration for the diagnosis is?

Page 7

1117
A. 1 hour
B. 2 hours
C. 3 hours
D. 4 hours
----------------------------------------
19. The Heineke-Mikulicz strictureplasty of longitudinal incision and transverse closure is used in which
surgical procedure?
(or)
The Heineke-Mikulicz strictureplasty is a widely recognized surgical technique that involves making a
longitudinal incision and then closing it transversely. This technique is commonly used to correct pyloric
stenosis or to widen narrowed segments of the small bowel, often due to Crohn's disease. It has also
been successfully applied in treating urethral strictures. In which surgical procedure this technique is
used?
A. Preputioplasty
B. Total penectomy
C. Circumcision
D. Balanoplasty
----------------------------------------
20. A 42-year-old male presents with complaints of weak stream, straining to urinate, incomplete
emptying associated with post-void dribbling, and a history of recurrent urinary tract infection.
Retrograde urethrography revealed >2 cm bulbar urethral stricture. What is the management of this
patient?
(or)
A 42-year-old male presents to the ER with complaints of weak stream, straining to urinate, incomplete
emptying associated with post-void dribbling, and a history of recurrent urinary tract infection. The
patient also gave a history of pelvic fracture 2 months before. The patient underwent retrograde
urethrography as a part of the evaluation, which revealed >2 cm bulbar urethral stricture. What is the
probable management for this patient?
A. Dilatation with a catheter
B. Internal Urethrotomy
C. Excision followed by end-to-end urethroplasty
D. Excision followed by reconstruction using buccal mucosa
----------------------------------------
21. A 35-year old patient was admitted to the hospital because of multiple injuries he sustained in a car
crash. He was conscious with GCS 15/15. Clinical examination suggested multiple abrasions all over
the body and severe tenderness in the pelvic region. Also, one failed urinary catheterization was
attempted, and blood appeared in the catheter during this procedure. A CT scan was performed, which
detected a fracture of the left pubic rami and the presence of the catheter balloon inside the urethra.
The patient underwent RGU, which revealed a membranous urethral rupture. In this patient, the
extravasated blood/contrast can be seen in which of the following? shorten the question. reframe
shorter version of question
(or)

Page 8

1118
In a 35-year-old patient with a car crash injury, CT scan and RGU revealed a membranous urethral
rupture. Where would extravasated blood/contrast be seen?
A. Ischiorectal fossa
B. Deep perineal pouch
C. Superficial inguinal region
D. Pelvic diaphragm
----------------------------------------
22. Which part of the male urethra is the widest and most distensible?
A. Prostatic
B. Membranous
C. Bulbous
D. Penile
----------------------------------------

Correct Answers
Question Correct Answer

Question 1 1
Question 2 3
Question 3 2
Question 4 4
Question 5 3
Question 6 4
Question 7 2
Question 8 2
Question 9 1
Question 10 2
Question 11 3
Question 12 2
Question 13 3
Question 14 4
Question 15 2
Question 16 1
Question 17 3
Question 18 4
Question 19 1

Page 9

1119
Question 20 4
Question 21 2
Question 22 1

Solution for Question 1:


Correct Option A - It is attributed to the failure of complete urethral tabularization in the fetus:
• In this case, as the patient has distal meatus, hypospadias is diagnosed.
• Hypospadias is a congenital condition in which the urethra folds are incomplete, and the urethral
meatus opens on the underside of the penis or perineum (ventral surface of the penis).

Incorrect Options:
Option B - Urethral opening is most commonly in the perineum:
• The perineal urethral opening is the posterior hypospadias.
• However, the most common urethral opening in the case of hypospadias is the anterior type, most
commonly glanular.
Option C - The urethra opens proximally and dorsally:
• It is a congenital abnormality of anterior urethral and penile development.
• Hypospadias is characterized by a urethral meatus present ventrally on the penis or scrotum, dorsal
winged prepuce (foreskin), and also the chordee, which is the ventral curvature of the penis.
• All the features can be present at different levels of severity in patients.
Option D - It is seen in 1 in 1500 boys:
• Hypospadias is the most common urethral malformation.
• It occurs in 1 in 250 male births.
• It is multifactorial in inheritance.

Page 10

1120
Solution for Question 2:
Option C: Duckett technique
Penoscrotal Hypospadias is the abnormality where the penis is bent downwards.
the surgical treatment is as described below:
• Dennis - brown technique (Two stages).
• MAGPI (Meatal advancement and glanduloplasty integrated) for coronal or subcoronal hypospadias.
• Mathieu repair (Perimeatal based flap, one stage) for distal and midshaft hypospadias.
• Asopa and Duckett technique using vascularized preputial island.
• Thiersch - Duplay or Bracka technique for proximal and midshaft penile hypospadias.
Option A: Circumcision
• Circumcision is contraindicated in children born with Penoscrotal hypospadias.
• The foreskin that is removed by circumcision is used for the reconstruction of hypospadias.
Option B: Ombredann’s procedure
• Ombredann’s procedure is not used for penoscrotal hypospadias.
Option D: External beam radiation
• External beam radiation is not recommended in Penoscrotal Hypospadias.
• It is used for the management of tumors.

Solution for Question 3:


Correct Option B - Malignant transformation in anogenital wart:
Buschke - Lowenstein tumor (verrucous carcinoma):
• In the above-given case, the presence of rectal bleeding, elimination of purulent drainage, large
ulcerated, hard and brittle lesion, and bulky mass is suggestive of the diagnosis of Buschke -
Lowenstein tumor (verrucous carcinoma).
• Tumors destroy adjacent tissues by compression.
• No metastasis usually (Locally malignant).
• Verrucous carcinoma is a locally aggressive form of condyloma acuminata.
Incorrect Options:
Option A - Malignant transformation in plantar wart:
• Buschke - Lowenstein tumor is a malignant transformation in the anogenital wart.
• It is not a malignant transformation in a plantar wart.
Option C - Malignant transformation in common wart:

Page 11

1121
• The malignant transformation in common warts is not the feature of Buschke - Lowenstein tumor.
• A biopsy can be done to evaluate the malignancy of warts.
Option D - Malignant transformation in seborrheic wart:
• Seborrheic wart can be caused by seborrheic keratosis.
• Seborrheic keratosis is benign skin growth.

Solution for Question 4:


Correct Option D: Meatal ulcer with scabbing
• In the above-given case, the presence of an enlarged bladder, dribbling urine, and distended bladder
up to the umbilicus is suggestive of the diagnosis of acute urinary retention.
• Acute urinary retention in a male child may be due to local inflammatory causes like a meatal ulcer
with scabbing.
Incorrect Options:
Option A: Prostatic radiotherapy
• Prostatic brachytherapy can lead to swelling of the urethra.
• The swelling may cause urethra blockage, which can lead to urinary retention.
• Prostatic radiotherapy may not lead to urinary retention in children.
Option B: Urethral stricture
• Urinary strictures are rare in children.
• Urinary strictures are associated with sudden urinary retention in adults.
Option C: Hysteria
• Hysteria is not the common cause of urinary retention in children.
• Urinary retention can be caused by mechanical obstruction in children.

Solution for Question 5:


Option C: Foley catheter can be carefully passed if the RGU is normal
• In this case, blood at the urethral meatus + pelvic hematoma and high-lying prostate are the
indications of injuries to the posterior urethra.
• Posterior urethra injuries occur with pelvic fractures, mostly in case of motor vehicle accidents.
• The presence of blood at the external urethral meatus indicates that Immediate urethrography should
be done. The urethra must be imaged first before any kind of urethral instrumentation.
• Once the patient has been stabilized, the correct placement of the Foley balloon catheter inside the
bladder must be checked.
• Retrograde urethrogram (RGU) is a diagnostic imaging test performed most commonly in male
patients to diagnose urethral injuries.

Page 12

1122
• If the blood at the urethral meatus is seen after urethral injury, the RGU test is considered to
determine the degree and nature of the injury and for operative planning for urethral reconstruction.
• If blunt trauma has injured the urethra of the patient and RGU is performed, urine leakage is often
seen.
• In the case of normal RGU, both the anterior and posterior urethra should be filled with contrast after
proper injection of radiopaque contrast, which is seen to jet into the bladder neck.
• A 16-F or 18-F Foley catheter is flushed into the meatus to remove air bubbles.
• It contains radiopaque contrast.
• When a Foley catheter is placed inside the meatus, the Foley catheter balloon reaches the fossa
navicularis.
Anterior urethral Injuries:
• The site of injury is the bulbar part of the urethra.
• External blow/kick, Straddle/manhole injuries are the common causes.
• Retrograde urethrogram (RGU) should always be the first investigation of suspected urethral injury.
• SPC (suprapubic cystostomy) is considered to be followed by delayed repair within three months.
Option A: Anterior urethra is the most likely place of injury
• Blunt or penetrating injuries with bulbar urethra result in Anterior urethral injuries.
• The most common site affected by anterior injuries is the bulbar urethra.
• It can be caused due to gunshot wounds or self-inflicted sexual misadventures.
Option B: Retrograde urethrography should be done immediately
• Retrograde urethrogram (RUG) is the diagnostic imaging test of choice in the case of suspected
urethral injury.
• It is recommended to do RUG after the stabilization of a patient.
Option D: Rectal examination may not reveal a large pelvic hematoma
• High-speed motor vehicle accidents and fall from height are the most common mechanism of injury
that results in pelvic fracture and urethral injuries.
• A digital rectal exam reveals pelvic hematoma and obscures the prostate to palpation.
• Pie-in-the-sky appearance is seen in this case on cystography, which is a floating bladder high in the
pelvis due to a large pelvic hematoma

Solution for Question 6:


Option D: Insert an infant feeding tube into the bladder and leave it for several days; Further managem
ent should be done according to the S.creatinine level.
• In this case, as the patient shows posterior urethra and some flaps of tissue that obstruct urine flow
along with urine incontinence, it makes the diagnosis of posterior urethral valves.
• Posterior urethral valves are congenital malformations in which obstructive membranes develop in the
urethra. present in male patients only.=

Page 13

1123
• It is associated with the following: Oligohydramnios Renal parenchymal dysplasia (the most important
factor in overall prognosis). Abnormal bladder function (25%) Pulmonary hypoplasia (the most common
cause of death).
• Oligohydramnios
• Renal parenchymal dysplasia (the most important factor in overall prognosis).
• Abnormal bladder function (25%)
• Pulmonary hypoplasia (the most common cause of death).
• Oligohydramnios
• Renal parenchymal dysplasia (the most important factor in overall prognosis).
• Abnormal bladder function (25%)
• Pulmonary hypoplasia (the most common cause of death).
Management:
• First of all, a small polyethylene feeding tube is inserted in the bladder and left for several days. Then
further management is done according to serum creatinine level.
• If the serum creatinine level is normal, Transurethral ablation (endoscopic fulguration) of the valves is
considered.
• If the serum creatinine level shows an increased concentration than the normal range, Vesicostomy
(Blockson technique is best) is considered to bypass the obstruction.
Option A: Conservative management is enough as it spontaneously resolves with aging.
• PUV patients require surgical interventions and medical management, and ongoing monitoring for the
rest of their lives.
• Endoscopic incision of the valves is the surgery which is considered for this cause, as it can not
resolves on its own withageing.
Option B: The patient should be scheduled for Endoscopic fulguration of valves immediately as B/L hyd
ronephrosis has developed.
• Before scheduling Endoscopic fulguration, serum creatinine level is measured.
• In the case of normal serum creatinine levels, endoscopic fulguration can be scheduled.
• But if there are abnormally high serum creatinine levels, vesicostomy, which is surgery to create a
small opening in the bladder made through the abdomen in order to help the flow of urine, is considered
Option C: Suprapubic cystostomy can be performed, and then a
polyethylene feeding tube should be used for secondary healing.
• If the child is stable, a vesicostomy can be performed.
• In rare cases in which hypertrophy of the bladder neck is seen in patients, a urethral catheter cannot
be placed. Such patients require cystoscopy for catheter placement, suprapubic tube placement, or
primary vesicostomy under anesthesia

Solution for Question 7:


Option B: Flow rate measurement studies and pressure-flow urodynamic studies are done.

Page 14

1124
• In this case, as the patient is showing enlarged prostate, stranguria and poor urine stream, it makes
the diagnosis of lower urinary tract symptoms.
• lower urinary tract symptoms describe various urinary problems related to the control or quantity of
micturition.
Below is a flowchart representing the approach to a patient presenting with LUTS:

• Bladder outlet obstruction BOO is assessing the pressure-flow relation during voiding urodynamically.
• It is a common cause of lower urinary tract symptoms (LUTS) in both sexes.
• Pressure-flow urodynamic studies are considered the gold standard for BOO diagnosis and LUTS
etiology.
• Urinary flow rate measurement is one of the simplest urodynamic tests, which is used as a general
indicator of normal or abnormal voiding.
• Urodynamic studies are used to assess the following: Flow rate Detrusor pressure Storage capacity
Urodynamic studies are the most useful studies, and particular treatments should be considered after
these studies' results
• Flow rate
• Detrusor pressure
• Storage capacity
• Urodynamic studies are the most useful studies, and particular treatments should be considered after
these studies' results
• Flow rate
• Detrusor pressure
• Storage capacity
• Urodynamic studies are the most useful studies, and particular treatments should be considered after
these studies' results
Option A: Charting a
patient’s symptoms according to the International Prostate Symptom Score (IPSS) is rarely used.

Page 15

1125
• IPSS is a questionnaire which is designed to be self-administered by the patient, with speed and ease
in mind and measure the degree of lower urinary tract symptoms.
• It is routinely used for charting patient's symptoms.
Option C: A thorough digestive system examination is mandatory
• A thorough nervous system examination is mandatory.
• Neurogenic Lower Urinary Tract Dysfunction is a condition when a person lacks bladder control due to
brain, spinal cord, or nerve problems.
• It results in either underactivity or overactivity of the sphincter or loss of sphincter coordination with
bladder function.
Option D: Cystourethroscopy is a good guide as an indication for surgery.
• It is a procedure of visually examining the urethra and bladder.
• In the context of lower urinary tract symptoms, there is no evidence that suggests any benefit, in terms
of outcome, related to performing cystoscopy in men.
• Cystourethroscopy does not give any guidance as to the degree of bladder outlet obstruction (BOO)
and the need for surgery.

Solution for Question 8:


Correct Option B - Congenital and acquired:
• In this case, as the patient has the inability to retract the penile foreskin and shows ballooning and
swelling of the penis, it makes the diagnosis of phimosis.
• Phimosis most commonly occurs in the uncircumcised penis.
• Phimosis is the formation of adhesion between the glans penis and prepuce.
• It is the inability to fully retract the penile foreskin.
• There are two types of phimosis.
Congenital:
• It is also called physiological or primary phimosis and is present at birth..
Acquired
• It is also called pathological or secondary phimosis and occurs as a result of scarring, infection, or
inflammation.
• Adult men can have secondary phimosis caused by tears or recurring infections, or STIs.
Incorrect Options:
Option A - Most common cause is STI:
• The most common cause of phimosis is a chronic infection due to poor hygiene.
• Not cleaning the penis can cause infection and result in phimosis.
• It is occasionally associated with sexually transmitted infections (STIs).
• STI is not a common cause.

Page 16

1126
Option C - Ballooning of prepuce during erection:
• Bulging or ballooning of prepuce while urinating is one of the most common symptoms of phimosis.
• This happens because, during micturition, urine accumulates in the foreskin before making its way
out.
Option D - Circumcision is done in all the cases:
• Circumcision for phimosis should be avoided in children requiring general anesthesia except in cases
with recurrent infections.
• The procedure should be postponed until the child reaches an age when local anesthesia can be
used.

Solution for Question 9:


Correct Option A - Circumcision is done in this condition:
• The symptoms manifested by the three-year old child point towards paraphimosis.
• Retraction of the foreskin causing swelling of the glans penis is a classical sign of paraphimosis until
proven otherwise.
Paraphimosis
• It is a condition that is common in uncircumcised males.
• Caused by the formation of a ring of prepuce around the glans because once prepuce is retracted, it
cannot be brought back to normal position.
• It is an acquired condition.
• Necrosis of the glans can occur in severe cases
• Treatment includes applying Ice bags to the area
• Gentle manual compression
• Injection of hyaluronidase that helps to reduce swelling
• It does not affect micturition.
Incorrect Options:
Option B - Decrease the flow of urine:
• Decreased urine flow is not a sign of paraphimosis.
• A decrease in Urine outflow is related to any obstruction in urine flow.
Option C - Pain in the back and groin:
• In paraphimosis, pain is localized to the glans penis
• Pain and swelling are localized to the glans only.
• Pain does not radiate to the back or groin.
Option D - Frequent micturition:
• Frequent micturition is not a symptom of paraphimosis.
• Frequent micturition is related to renal problems, urinary tract infections and many other pathologies.

Page 17

1127
Solution for Question 10:
Correct Option B - Bulbar urethra:
• The symptoms of urinary obstruction, drop-by-drop micturition, abdominal straining during urination,
and fecal incontinence while voiding point towards a stricture somewhere in the urethra.
• Urethral stricture is the obstruction of urine flow through the urethra, which obstructs the outflow of
urine.
• The retrograde urethrogram confirms the stricture of the bulbar part of the urethra.
• Bulbar urethral stricture might present with various levels of severity.
• In cases where the stricture is light, urine continues to flow out of the body.
• While in cases where the obstruction is severe or acute obstruction no passage of urine or anuria
occurs.
• The Investigation of choice for diagnosing posterior urethral stricture is micturating cystourethrogram.
• The Investigation of choice for diagnosing anterior urethral stricture-retrograde urethrogram
• For adequate evaluation of posterior and anterior urethral stricture, retrograde urethrogram and
Cystourethrogram are required.
Incorrect Options:
Option A - Membranous Urethra:
• The membranous urethral stricture is a very rare condition.
• The urethrogram confirms the stricture in the bulbar urethra.
Option C - Penile Urethra:
• The urethrogram confirms the presence of bulbar urethra in the given scenario.
• The penile urethra is not a congenital anomaly, and the scenario explains that obstructive symptoms
have been present since birth.
Option D - Prostatic urethra:
• The signs and symptoms of prostatic urethral stricture differ from those explained in the question.
• The urethrogram confirms the presence of the bulbar urethra.

Solution for Question 11:


Option C: Galizia's Triad is Dupuytren’s contracture + Peyronie’s disease + Retroperitoneal fibrosis
• Painful penile erection and other symptoms lead to penile fibromatosis.
• Peyronie’s disease is also known as penile fibromatosis.
• Chronic vasculitis involving tunica albuginea presents with fibrous plaques over the dorsolateral
aspect of the penis.

Page 18

1128
• Penis has characteristic curvature because of the formation of fibrous plaque due to contraction,
therefore upward turning & Peyronie’s disease is a component of Galizia’s triad [Dupuytren’s
contracture + Peyronie’s disease + RPF (retroperitoneal fibrosis)].
Treatment
• More than 50% of cases undergo spontaneous resolution by observation and emotional support.
• If it doesn't reduce, Nesbit procedure is used to correct the curvature ), surgery is done by placement
of Non–absorbable suture opposite to the plaque.
Option A: Medical treatment is enough for spontaneous resolution
• Medical treatment alone is not very effective in this case.
• Observation and emotional support play a very important role in treating this condition.
Option B: Chronic vasculitis involving tunica adventitia and presents with fibrous plaque over the dorsol
ateral aspect of the penis
• Chronic vasculitis does not involve tunica adventitia. Rather it involves tunica albuginea.
Option D: Transurethral resection of the prostate is performed
• Transurethral resection of the prostate is performed in the enlarged prostate gland.
• It is of no help in this case of penile erection.

Solution for Question 12:


Option B: Low flow priapism can lead to compartment syndrome
• The sign like persistent painful penile erections mentioned in the question point towards the diagnosis
of priapism.
• Priapism: Priapism is the persistent and painful penile erection in the absence of sexual excitement or
desire, and it may persist after sexual excitement. The minimum duration of a painful erection is 4
hours. It is a medical emergency, and delay in treatment leads to Cavernosal fibrosis and impotence. It
should be treated within 6 hours to prevent other complications. Priapism is of 2 types: High flow and
Low flow
• Priapism is the persistent and painful penile erection in the absence of sexual excitement or desire,
and it may persist after sexual excitement.
• The minimum duration of a painful erection is 4 hours.
• It is a medical emergency, and delay in treatment leads to Cavernosal fibrosis and impotence.
• It should be treated within 6 hours to prevent other complications.
• Priapism is of 2 types: High flow and Low flow
• Priapism is the persistent and painful penile erection in the absence of sexual excitement or desire,
and it may persist after sexual excitement.
• The minimum duration of a painful erection is 4 hours.
• It is a medical emergency, and delay in treatment leads to Cavernosal fibrosis and impotence.
• It should be treated within 6 hours to prevent other complications.
• Priapism is of 2 types: High flow and Low flow

Page 19

1129
• Non- Ischemic
Causes
• Penile Trauma
• Perineal trauma
• Due to the formation of arterial sinusoidal shunts.
• Ischemic
• Sickle cell anemia
• Leukemia
• Fat emboli
• Spinal cord lesion
• Malignant Penile inflammation
• Trazodone Injection
• Clinical presentation In children(5-10 years) with sickle cell disease- painful erection during nights In
adults-iatrogenic
• In children(5-10 years) with sickle cell disease- painful erection during nights
• In adults-iatrogenic
• Diagnosis Mainly clinical, but doppler is helpful in the diagnosis
• Mainly clinical, but doppler is helpful in the diagnosis
• Treatment If the patient comes within 4-6 hours, a ketamine injection is given (> 50% of cases
improve). If the condition does not improve after that, aspiration and saline irrigation is done. When the
condition is not improved even after saline irrigation, hyaluronidase injection is injected. For high-flow
priapism, selective internal pudendal angiography is followed by embolization of the feeding vessel.
• If the patient comes within 4-6 hours, a ketamine injection is given (> 50% of cases improve).
• If the condition does not improve after that, aspiration and saline irrigation is done.
• When the condition is not improved even after saline irrigation, hyaluronidase injection is injected.
• For high-flow priapism, selective internal pudendal angiography is followed by embolization of the
feeding vessel.
• In children(5-10 years) with sickle cell disease- painful erection during nights
• In adults-iatrogenic
• Mainly clinical, but doppler is helpful in the diagnosis
• If the patient comes within 4-6 hours, a ketamine injection is given (> 50% of cases improve).
• If the condition does not improve after that, aspiration and saline irrigation is done.
• When the condition is not improved even after saline irrigation, hyaluronidase injection is injected.
• For high-flow priapism, selective internal pudendal angiography is followed by embolization of the
feeding vessel.
Option A: Low flow priapism occurs secondary to perineal trauma

Page 20

1130
• Low flow priapism does not occur secondary to perineal trauma.
• High-flow Priapism occurs secondary to perineal or penile trauma.
Option C: Diagnosed mainly by panoramic radiography
• It is diagnosed mainly by color Doppler.
• Panoramic radiography is done for dental procedures.
Option D: Whipple procedure is done
• Whipple procedure is not done in priapism.
• Whipple procedure is a complex surgery done to remove the head of the pancreas.

Solution for Question 13:


Option C: more than 50% of patients present with enlarged inguinal lymph nodes
• Squamous cell carcinoma of the penis most commonly presents in the late stage of life.
• More than 50% of the patients present with enlarged inguinal lymph nodes.
Carcinoma of the penis:
• Malignant carcinoma is squamous cell carcinoma Bowen’s disease Erythroplasia of Queyrat: involves
glans penis
• Bowen’s disease
• Erythroplasia of Queyrat: involves glans penis
• Treatment includes 5% 5-FU Cream and ND-YAG laser
• Malignant carcinoma Location: Glans> Prepuce>Sulcus
• Hematogenous spread leads to priapism.
• The earliest clinical presentation of penile carcinoma is priapism.
• Metastatic carcinoma presentation Lesion Foul-smelling discharge >50% of cases have inguinal
lymphadenopathy
• Lesion
• Foul-smelling discharge
• >50% of cases have inguinal lymphadenopathy
• Metastasis → Metastatic carcinoma site is the lungs Enlarged inguinal lymph nodes lead to erosion of
femoral vessels This will lead to bleeding from the femoral vessel. A femoral vessel bleed is a
life-threatening emergency.
• Enlarged inguinal lymph nodes lead to erosion of femoral vessels
• This will lead to bleeding from the femoral vessel.
• A femoral vessel bleed is a life-threatening emergency.
• The Investigation of choice for diagnosis is a biopsy.
• The investigation of choice for staging is MRI- Magnetic resonance imaging.

Page 21

1131
• Bowen’s disease
• Erythroplasia of Queyrat: involves glans penis
• Lesion
• Foul-smelling discharge
• >50% of cases have inguinal lymphadenopathy
• Enlarged inguinal lymph nodes lead to erosion of femoral vessels
• This will lead to bleeding from the femoral vessel.
• A femoral vessel bleed is a life-threatening emergency.
Option A: Neonatal circumcision is of no use
• Neonatal circumcision is a protective measure to prevent penile carcinoma.
• Circumcision should be done prophylactically.
Option B: Leukoplakia is treated by YAG laser
• Leukoplakia is not treated by a YAG laser.
• Erythroplakia is treated by a YAG laser.
Option D: Metastasis to the liver is the cause of immediate death
• Liver Mets are not common in penile carcinomas.
• Immediate death is never because of liver Mets.
• It's a gradual process that will ultimately lead to the death of the patient

Solution for Question 14:


Option D: Circumcision
• The above-given case is CA penile prepuce, as it presents as a nonhealing ulcer in the penis and
sometimes smelly discharge under the foreskin.
• The best treatment in the above-given case is circumcision, which is best for small preputial lesions.
• Causes of CA penis include smoking, acquired immunodeficiency syndrome, and Human
papillomavirus 16 and age older than 50.
• Penile cancer can present as: Change in skin thickness or colour. A rash or small crusty bump on the
penis Bluish growth Lump on penis A sore, which may bleed Lumps under the skin of the groin.
• Change in skin thickness or colour.
• A rash or small crusty bump on the penis
• Bluish growth
• Lump on penis
• A sore, which may bleed
• Lumps under the skin of the groin.
• It is diagnosed with a biopsy.

Page 22

1132
• Treatment involves wide local excision with a 2 cm margin.
• Treatment options are: Circumcision for tumors confined to the foreskin. Mohs Surgery involves wide
excision, glossectomy or removal of part of the penis. Radiation therapy Chemotherapy: Bleomycin and
cisplatin.
• Circumcision for tumors confined to the foreskin.
• Mohs Surgery involves wide excision, glossectomy or removal of part of the penis.
• Radiation therapy
• Chemotherapy: Bleomycin and cisplatin.
• Change in skin thickness or colour.
• A rash or small crusty bump on the penis
• Bluish growth
• Lump on penis
• A sore, which may bleed
• Lumps under the skin of the groin.
• Circumcision for tumors confined to the foreskin.
• Mohs Surgery involves wide excision, glossectomy or removal of part of the penis.
• Radiation therapy
• Chemotherapy: Bleomycin and cisplatin.
Option A: Total penectomy
• Penectomy is done if the tumor size is large, it is invasive, or there are chances of recurrence.
• It is the treatment of choice if the lesion is in the proximal shaft with a perineal urethrostomy.
Option B: Partial penectomy
• Penectomy is done if the tumor size is large, it is invasive, or there are chances of recurrence.
• It is done if the lesion is on the distal shaft
Option C: Balanoplasty
• It is the procedure done for reconstruction of the glans penis.
• Balanoplasty is the treatment of choice for hypospadias.

Solution for Question 15:


Option B: Most common site is the bulbo-membranous urethra
• The above-given case is carcinoma of the male urethra, as obstructive voiding symptoms and chronic
infections are the strongest risk factor.
• The most common site is bulbo-membranous urethra> penile urethra > prostatic urethra.
• The most common type is SCC > TCC > adenocarcinoma.
• The most common type of carcinoma prostatic urethra is TCC > SC

Page 23

1133
• The most common type of carcinoma penile urethra is SCC > TCC
• Risk factors include a history of bladder cancer, recurrent UTI, STIs, and being older than 60.
• Most common presenting symptom: Palpable mass associated with obstructive voiding symptoms.
• Other symptoms include: frequent urination, interrupted urine flow, bleeding from the urethra, a
lump in the penis, or any discharge from the urethra
• MRI with gadolinium is the Investigation of choice for evaluating local soft tissue and LNs metastasis.
• Surgery is the mainstay of treatment. Radiotherapy is also used.
• Ilioinguinal node dissection only in the presence of palpable adenopathy
Option A: Most common type is TCC > SCC
• The most common type is SCC > TCC > adenocarcinoma.
• Transitional cell carcinoma is most common in the prostatic urethra.
Option C: The Investigation of choice for soft tissue evaluation is a CT scan.
• CT scan is not used for soft tissue evaluation.
• The Investigation of choice for soft tissue evaluation is MRI with gadolinium.
Option D: Radiotherapy is the mainstay of treatment
• Radiotherapy is not the mainstay of treatment.
• Surgery is the best treatment choice for carcinoma of the male urethra

Solution for Question 16:


Option A: Risk of Hemorrhage due to bleeding from a frenular artery
• The surgical procedure of the penis that would be done on this boy is circumcision, which is done as a
religious rite in Muslims.
• Circumcision is a procedure to remove the skin which covers the tip of the penis.
• Bleeding is the commonest complication of circumcision
• Bleeding may occur along the skin edges between sutures or from discrete blood.
• Circumcision lowers the risk of certain penile disorders, i.e. balanoposthitis, paraphimosis, phimosis,
ca penis, and sexually transmitted infections like HIV and UTIs.
• Risks of circumcision: bleeding, infection, reaction to anaesthesia, pain, mastitis, and cutting the
foreskin too short or too long.
• The three common methods used for circumcision are the Mogen clamp, the Gomco clamp, and the
plastic bell device.
• Contraindication for circumcision: In hypospadias patients, the meatus opens on the ventral aspect
associated with hooded prepuce on the dorsal aspect.
Option B: Increases sexual drive & Option C: Decreased sexual drive
• There are no significant differences in Sexual drive and erection between circumcised and
uncircumcised men.

Page 24

1134
• There is no difference in ejaculation and ejaculation latency time between circumcised and
uncircumcised men.
Option D: Increased sexually transmitted infections
• The risk for sexually transmitted infections is reduced.
• Circumcision also reduces the chance of a Ca penis.

Solution for Question 17:


Correct Option C - Straightening of the penis – Urethroplasty – Balanoplasty:
• The above-given case is penoscrotal hypospadias, as it presents with an abnormal looking penis, the
passage of urine from the underside of the penis and the opening of the urethra where the penis meets
the scrotum.
• The best time for Surgery for hypospadias: is 6 - 12 months of age.
• Hypospadias is a congenital disability in which the opening of the urethra is not present at the tip of
the penis
• Types of hypospadias: Sub coronal: the opening of the urethra is present somewhere near the head
of the penis. Mid-shaft: the opening of the urethra is present on the shaft of the penis. Peno-scrotal: the
opening of the urethra is where the penis and scrotum meet.
• Sub coronal: the opening of the urethra is present somewhere near the head of the penis.
• Mid-shaft: the opening of the urethra is present on the shaft of the penis.
• Peno-scrotal: the opening of the urethra is where the penis and scrotum meet.
• Sub coronal: the opening of the urethra is present somewhere near the head of the penis.
• Mid-shaft: the opening of the urethra is present on the shaft of the penis.
• Peno-scrotal: the opening of the urethra is where the penis and scrotum meet.

Page 25

1135
• The patient presents with an opening of the urethra other than the tip, downward curvature of the
penis, a hooded appearance of the penis and abnormal spraying during urination.
• Hypospadias is present congenitally.
• Diagnosed usually by physical examination.
Successful hypospadias surgery incorporates the following steps:
• Straightening of the penis (arthroplasty)
• Reconstruction of the urethra (urethroplasty)
• Meatus (meatoplasty)
• Glans (glanuloplasty)
• The skin of the penis & scrotum, whenever necessary.
• In hypospadias, after straightening the penis, urethroplasty is done. Balanoplasty is done after
urethroplasty.

Solution for Question 18:


Option D: 4 hours
• The above-given case is priapism, in which patients present with a painful erection without sexual
drive. Antipsychotic medications, alcoholism and drugs are important causes of priapism.
• Priapism is a persistent painful erection lasting longer than 4 hours, irrespective of any sexual
stimulation & beyond reaching orgasm.
• It is an emergency
• It is of two types: Ischemic priapism Non-ischemic priapism
• Ischemic priapism
• Non-ischemic priapism
• Ischemic priapism (veno-occlusive, low flow) occurs when blood can't enter or leave the penis. This
causes erection lasting more than 4 hours. The shaft of the penis is very hard, but the tip is soft. It
causes pain and discomfort. This is a nonsexual and persistent erection.
• Non-Ischemic priapism (arterial, high flow) occurs when a large amount of blood continuously flows
through the penis. The shaft and tip of the penis are hard. This condition is less painful.
• Causes of priapism: sickle cell anaemia, alcohol and drug use, injury to genitals; pelvis or area
between the penis and anus, tumors.
• Treatment: Non-Ischemic often goes without any treatment Ischemic priapism is an emergency, and
treatment options are aspiration to drain excess blood; Shunt is made to vent blood from the penis
• Non-Ischemic often goes without any treatment
• Ischemic priapism is an emergency, and treatment options are aspiration to drain excess blood; Shunt
is made to vent blood from the penis
• Ischemic priapism
• Non-ischemic priapism

Page 26

1136
• Non-Ischemic often goes without any treatment
• Ischemic priapism is an emergency, and treatment options are aspiration to drain excess blood; Shunt
is made to vent blood from the penis

Solution for Question 19:


Option A: Preputioplasty
• Heineke-Mikulicz principle refers to using a longitudinal incision followed by closure in a transverse
fashion.
• It is used in: Preputioplasty - to retract the foreskin Frenuloplasty - to lengthen the frenulum
• Preputioplasty - to retract the foreskin
• Frenuloplasty - to lengthen the frenulum
• Preputioplasty is a surgical procedure in which the foreskin of the penis is widened.
• Preputioplasty is performed because the foreskin does not pull back, causes recurrent infections, and
can be painful.
• Preputioplasty - to retract the foreskin
• Frenuloplasty - to lengthen the frenulum
Option B: Total penectomy
• Penectomy is done if the tumor size is large, it is invasive, or there are chances of recurrence.
• It is the treatment of choice if the lesion is in the proximal shaft with a perineal urethrostomy.
Option C: Circumcision
• It is a procedure in which the foreskin of the penis is removed.
• Circumcision lowers the risk of penile disorders, i.e. balanoposthitis, phimosis, paraphimosis, penile
cancer, STI and UTIs.
Option D: Balanoplasty
• It is the procedure done for reconstruction of the glans penis.
• Balanoplasty is the treatment of choice for hypospadias

Solution for Question 20:


Option D: Excision Followed by reconstruction using buccal mucosa
• The above condition is urethral stricture, as it presents with a weak stream, straining to urinate, and
incomplete emptying associated with post-void dribbling.
• Causes include: A medical procedure, inserting some instruments i.e. endoscope. Long-term insertion
of a catheter Trauma Injury to the urethra or pelvis Enlarged prostate Prostrate or urethral cancer
• A medical procedure, inserting some instruments i.e. endoscope.
• Long-term insertion of a catheter

Page 27

1137
• Trauma
• Injury to the urethra or pelvis
• Enlarged prostate
• Prostrate or urethral cancer
• Patients present with the following symptoms: decreased urine stream, incomplete bladder emptying,
spraying of urine stream, pain when urinating, frequent urination and urinary tract infections.
• Management of patients of urethral stricture:
• A medical procedure, inserting some instruments i.e. endoscope.
• Long-term insertion of a catheter
• Trauma
• Injury to the urethra or pelvis
• Enlarged prostate
• Prostrate or urethral cancer
Option A: Dilatation with a catheter
• It is a procedure to widen the urethra.
• Dilatation is done to resolve the symptoms.
Option B: Internal Urethrotomy
• This procedure is performed for short bulbar stricture.
• Internal urethrotomy is cutting the stricture through the scope.
Option C: Excision followed by end-to-end urethroplasty
• If the stricture length is up to 2 cm, excision followed by an end-to-end urethroplasty is done.
• It is also done in cases in which dilatation and internal urethrotomy fail

Solution for Question 21:


Option B: Deep perineal pouch
Injuries to the posterior urethra
• The part of the urethra most likely to be injured in a pelvic fracture is the membranous urethra.
• The membranous urethra is sheared at the bulbo-membranous or prostato-membranous junction.
• Bulbomembranous junction is more prone than prostate membranous junction during pelvic fracture
(posterior urethra densely adheres to pubis via urogenital diaphragm & puboprostatic ligaments)
Option A: Ischiorectal fossa
• It is the space present between the internal surface of the perineal skin and the plate of the levator ani
muscle.
• Blood supply to the ischiorectal fossa is supplied by the internal pudendal artery.
Option C: Superficial inguinal region

Page 28

1138
• The superficial inguinal region is the terminal end of the inguinal canal.
• The superficial inguinal region is located superior to the pubic tubercle.
• The blood supply to this region is derived from the branches of the common femoral artery.
Option D: Pelvic diaphragm
• The pelvic diaphragm forms the inferior border of the abdominopelvic cavity.
• The blood supply to the pelvic diaphragm is carried by the inferior phrenic, superior phrenic,
musculophrenic, pericardiacophrenic, and lower internal intercostal arteries.

Solution for Question 22:


Option A: Prostatic
• Male urethra parts- The most distensible is Prostatic
• The most distensible is Prostatic
• The most distensible is Prostatic
Option B: Membranous
• The membranous part of the urethra is the least dilatable part.
• The muscles in the membranous urethra help to hold the urine.
Option C: Bulbous
• The bulbous urethra is invested in the penile bulb.
• It extends from the urogenital diaphragm to the penile suspensory ligament.
Option D: Penile
• The penile is the longest part of the urethra.
• It is distal to the suspensory ligament.

Page 29

1139
Testis and Scrotum
1. A 46-year-old male came to the OPD complaining of painless swelling in the right side of the scrotum
for the past two years. On Examination, tests were stony hard and non-tender. No signs of
inflammation were evident. On asking the patient gives a history of weight loss and bone pains.
According to the patient, his father died of the same disease. What is the most common type of this
disease?
(or)
A 46-year-old male presents with painless swelling in the right side of the scrotum. On examination,
testes were stony hard and non-tender. He gives a history of weight loss, bone pains, and his father's
death due to the same disease. What is the most common type of this disease?
A. Seminoma
B. Teratoma
C. Choriocarcinoma
D. Embryonal carcinoma
----------------------------------------
2. A 6-month-old baby presented with an absence of left testes in the left scrotum. On examination, one
testis is absent from the left side, and a mass is palpable in the left inguinal region. Which statement is
true about the condition mentioned above?
(or)
A 6-month-old baby was brought to the surgical clinic by his mother, with a complaint of an absent left
testis in the left scrotum. The mother noticed this condition about three days ago. The doctor on duty
examined both scrotums and found that one testis was missing from the left side, and a mass was
palpable in the left inguinal region. Can you identify which statement is true about the condition?
A. Atrophy of Leydig cells & decrease in tubular diameter can occur by two years of age.
B. No effect on spermatogenesis in this condition
C. Histologic and morphologic changes can be seen in the testis as early as two months of age
D. Surgery should be done before 3yr of age for optimal result
----------------------------------------
3. A 65-year-old male patient with previously good health presented to the emergency department with
new testicular pain and swelling. Physical examination and ultrasonography revealed a large,
heterogeneous, vascularized mass in the right scrotum. The patient underwent high inguinal
orchidectomy, and histopathology showed that the mass had invaded the tunica albuginea and the
epididymis. What is the disease stage?
(or)
a patient with a testicular tumor undergone high inguinal orchidectomy and, and histopathology shows
a mass of the testis invading tunica albuginea with the involvement of epididymis. The stage of the
disease is?
A. T1
B. T2
C. T3

1140
D. T4
----------------------------------------
4. A 32-year-old complains of swelling in the left testes and inguinal region. On examination, a bag of
worms is felt in the left scrotum. Doppler scan showed abnormal dilatation of the internal spermatic
venous system. Which statement is true about this condition?
(or)
A 32-year-old man arrived at the surgical department with a complaint of swelling in his left testicle and
groin area. The swelling increased in size during walking and standing, but disappeared while lying
down. The patient reported feeling a small mass in the testicle. Upon examination, the doctor found a
"bag of worms" in the left scrotum. A Doppler scan revealed abnormal dilatation of the internal
spermatic venous system. Which statement is true about this condition?
A. Negative impulse on coughing
B. More common on the right side
C. The usual cause is the absence or incompetence of valves in the proximal testicular vein
D. Diagnosis can be made by histopathology.
----------------------------------------
5. A 3-month-old infant is brought to the outpatient department by his mother with a complaint of
swelling in the scrotum. The swelling has been present since birth but has been increasing in size with
no other associated symptoms. On performing the transillumination test, clear fluid is observed around
the testis. The infant has a history of fever on and off. What would be the probable management for this
case?
(or)
A 3-month-old baby is presented with the complaint of swelling in the scrotum, which was there since
birth but now increasing in size. On the transillumination test, clear fluid is noted around the testis. What
is the management for this case?

A. Eversion of sac
B. Excision of sac
C. Lord's procedure
D. Herniotomy
----------------------------------------
6. A 45-year-old man presents in the outpatient department with the complaint of painful swelling of the
scrotum and feeling of pressure at the base of the penis. The physical examination reveals that there is

Page 2

1141
a mass on one side of the testicle. The vitals are within normal limits. The sign is shown in the image
below. What is the most probable diagnosis?
(or)
A 45-year-old man presents with the complaint of painful swelling of the scrotum and feeling pressure
at the base of the penis. On examination, there is a mass on one side of the testicle. The sign is shown
in the image below. What is the most probable diagnosis?

A. Spermatocele
B. Epididymal cyst
C. Hydrocele of cord
D. Secondary hydrocele
----------------------------------------
7. A male newborn was found to have an undescended testicle on one side, but his external genitalia
appeared normal. An ultrasound was ordered, which revealed a mass on the left side of his lower
abdomen in the area of the superficial inguinal ring. Later, the mass was identified as the true
undescended testicle. The parents requested immediate surgery, as they were concerned that this
could potentially affect their child's future fertility. At what time would you recommend surgery for
undescended testicles in this patient?
(or)
The best time for surgery of undescended testes that is recommended is?

A. Just after birth


B. 6 months of age
C. 12 months of age

Page 3

1142
D. 24 months of age
----------------------------------------
8. A male child presents with left empty scrotum and swelling on the left flank since birth.
Ultrasonography reports a hernia on the left side; testis is not seen during the scan but is a palpable
soft tissue mass in the perineum on the same side of an empty hemi scrotum perineum. The
commonest position to look for the testis in this case is?
(or)
A nine-month-old male child is brought by his mother to the OPD with complaints of an empty left
scrotum and swelling on the left flank that has been present since birth. An ultrasound of the abdomen
was done at another center which reported a hernia on the left side. However, the testis was not seen
during the scan. A palpable soft tissue mass was noticed in the perineum on the same side as the
empty hemi-scrotum. In this case, what is the most common position to look for the testis?
A. At the root of the penis
B. Contralateral scrotum
C. The medial part of the femoral triangle
D. The upper part of the femoral triangle
----------------------------------------
9. A 31 year old male presents to the emergency room with a complaint of sudden onset and
progressive scrotal pain over the last five hours. The patient's past medical history is insignificant,
though upon questioning, he reports sleeping with several women in the past six-month period. On
physical examination, the testes appear to have a horizontal line, and the scrotum is neither swollen nor
discolored. Doppler ultrasonography shows decreased blood flow to the affected testicle. Which of the
following is the most likely diagnosis?
(or)
A 31-year-old male presents with a complaint of sudden onset scrotal pain. On examination, the testes
appear to have a horizontal line. Doppler ultrasonography shows decreased blood flow to the affected
testicle. He is sexually active with several women. What is the diagnosis?
A. Seminoma
B. Orchitis
C. Epididymitis
D. Testicular torsion
----------------------------------------
10. A 35-year-old man has come to the outpatient department, reporting a firm swelling in his right
scrotum and a dull ache in his lower groin. During the physical examination, a mass was found in his
testicles. Immunohistochemistry tests showed that the placental alkaline phosphatase marker was
positive. All of the vital signs are within normal range. Which of the following is associated with the
placental alkaline phosphatase marker?
(or)
A 35-year-old male complains of hard swelling in the right scrotum and a dull ache in the lower groin.
The physical examination shows a testicular mass. Immunohistochemistry tested positive for placental
alkaline phosphatase, which is associated with which of the following?
A. Theca cell tumor

Page 4

1143
B. Teratoma
C. Choriocarcinoma
D. Seminoma
----------------------------------------
11. During Orchiopexy for undescended testes, accidental ligation of which artery compromises the
blood supply to the testes?
(or)
Accidental ligation of which artery during Orchiopexy for undescended testes can compromise the
blood supply to the testes?
A. Hypogastric arteries
B. Pudendal arteries
C. Internal spermatic arteries
D. External spermatic arteries
----------------------------------------
12. what is the grade of diagnosis when the testes look distended and feel like like a ' bag of worms' at
rest
(or)
A 34-year-old man presents to the fertility clinic for evaluation of infertility. He and his wife have been
trying to have children for two years. His wife was recently evaluated and found to be normal and
healthy. The patient denies any pain in his testicular region. However, he reports occasional feelings of
heaviness in his scrotum. On physical exam, his scrotum looks distended. At rest, a "bag of worm"-like
finding is visible and palpable in the testicle. What is the grade of the given condition?
A. Grade I
B. Grade II
C. Grade III
D. Grade IV
----------------------------------------
13. A 26-year-old man presents to the outpatient department with left-sided testicular pain and swelling
from the last few days. He has a past medical history of multiple sexually transmitted infections. On
physical exam, his left scrotum and testicle are indurated, erythematous, and tender on palpation.
Ultrasonography reveals a thickened epididymis. He is sent to the lab for testing for sexually
transmitted diseases. What is the most likely diagnosis?
(or)
A patient presents with left-sided testicular pain and swelling. He has a history of multiple sexually
transmitted infections. On examination, his left scrotum and testicle are indurated, erythematous, and
tender on palpation. Ultrasound reveals a thickened epididymis. What is the diagnosis?
A. Seminoma
B. Orchitis
C. Epididymitis
D. Testicular torsion

Page 5

1144
----------------------------------------
14. A 5-month-old child is brought to the pediatrician's office for evaluation of a "lump" in his genital
area. His birth history is unremarkable, and he has been feeding well. His parents report no other
concerns except this lump. On physical exam, there is a tense and non-tender mass in the scrotum.
The transillumination test is positive. What is the appropriate diagnosis for this condition?
(or)
Based on the image given below of a transillumination test done on a 5-month-old. What is the
appropriate diagnosis?

A. Inguinal hernia
B. Hydrocele
C. Varicocele
D. Epididymal cyst
----------------------------------------
15. A 37-year-old man presents with left testicular pain associated with increased urinary frequency
and a burning sensation while urinating. On examination, a swollen, tender left testicle is noted and
sudden relief from pain on the elevation of the left testicle. What statement is true regarding the
diagnosis of the patient?
(or)
A 37-year-old man presents to the outpatient department with a 3-day history of left testicular pain
associated with increased urinary frequency and burning sensation while urinating. He has no chronic
illnesses and is not immunocompromised. He is sexually active but had no previous sexually
transmitted diseases and no urethral discharge. He has no history of trauma and no similar episodes
before. On examination, a swollen, tender left testicle was noted, and when you tried lifting the left
testicle, the patient felt a sudden relief from pain. Which of the following statement is true regarding the
diagnosis of the patient?
A. Most common causative organism in <35 years old and sexually active males is Neisseria
gonorrhoeae
B. Cremasteric reflexnegative
C. Most common cause of primary hydrocele
D. Prehn's sign is positive
----------------------------------------

Page 6

1145
16. A 60-year-old diabetic male presents to the emergency department with complaints of fever,
malaise, severe pain, swelling in the genital region, and smell of the affected region. The physical
examination shows the blackish discoloration of the scrotal skin, which is described in the image below.
There is a gas that can be seen on scrotal ultrasound. Which of the following should be excised during
surgery?
(or)
Based on the image given below, which of the following should be excised during surgery?

A. Skin
B. Skin & subcutaneous tissue
C. Skin, subcutaneous tissue and corpora cavernosa
D. Skin, subcutaneous tissue, corpora cavernosa & testis
----------------------------------------
17. A 34-year-old man has swelling in his left scrotum for two weeks, along with mild lower abdominal
discomfort for four weeks. Physical exam reveals an enlarged testis, gynecomastia, and elevated
serum markers. Imaging shows liver lesions and retroperitoneal lymph nodes. What are the risk factors
for testicular tumors?
(or)
What is a risk factor for testicular tumors?
A. Extratubular germ cell neoplasia
B. Klinefelter's syndrome
C. CA scrotum
D. Acute Epididymo-Orchitis (EDO)
----------------------------------------
18. A man presents with painless swelling in the right testis. An examination and tests reveal a tumor in
the right testis. A right-sided high inguinal radical orchidectomy is performed, and histopathological
examination shows tubules with thickened walls, edematous stroma, and numerous dilated blood
vessels.. What is your probable diagnosis?
(or)
A 43-year-old male presented with painless swelling in the right testis for 4 months. Clinical
examination revealed an enlarged, firm to hard right testis. Blood tests showed normal ranges of serum
tumor markers. Ultrasonography and CT scan revealed a tumor in the right testis with multiple cystic
areas and an ill-defined mass. A right-sided orchidectomy was performed. Histopathological

Page 7

1146
examination revealed thickened tubule walls filled with small and intermediate-sized cells. The stroma
in between the tubules contained numerous dilated blood vessels and edematous stroma. What is your
probable diagnosis?
A. Seminoma
B. Testicular teratoma
C. Intratubular germ cell neoplasia
D. CA Scrotum
----------------------------------------
19. A 39-year-old male presents in the outpatient department with complaints of scrotal ulcers, which
are painful, feeling heaviness in the scrotum, and sudden swelling in the right testicle. There is a scrotal
mass, and scrotal ulcers with everted edges are found on physical examination. The tumour markers
AFP, hCG, and LDH are elevated. Which of the following statements is correct about chimney sweeps
cancer?
(or)
Which statement is correct about chimney sweeps cancer?
A. It's a squamous cell carcinoma of the scrotum
B. Diagnosis is established by biopsy
C. Para-aortic lymph nodes are the last involved lymph nodes
D. Treated by wide local excision
----------------------------------------
20. A 29-year-old man underwent varicocele surgery with two young residents in attendance. During
the procedure, one of the residents got unsterile, and the chief surgeon asked him which vessel to use
as an alternative for venous drainage to avoid being scrubbed out of the procedure. The resident's
answer should be:
(or)
A 29-year-old man underwent varicocele surgery with two young residents in attendance. During the
procedure, one of the residents got unsterile, and the chief surgeon asked him which vessel to use as
an alternative for venous drainage to avoid being scrubbed out of the procedure. The resident's answer
should be:
A. IVC
B. Aorta
C. Superior epigastric vein
D. Cremasteric vein
----------------------------------------
21. A 15-year-old male presented to the emergency department with sudden and continuous pain in his
left testicle after an injury while playing sports. The pain was accompanied by radiating pain to the
abdomen and left inguinal area, nausea, and vomiting for more than 12 hours. The patient had no prior
history of this kind of pain. Upon examination, no testes were found in the left scrotum. What is the
appropriate surgical procedure for this condition?
(or)

Page 8

1147
A 5-year-old boy presents with pain in the left testicle radiatingto the abdomen and left inguinal area
after injury while playing sports. It is accompanied by nausea and vomiting. On examination, there were
no testes in the left scrotum. What is true about the surgical procedure of the above-given condition?
A. Can be delayed 24 hours and limited to the affected side
B. Can be delayed but should include the asymptomatic side
C. Should be immediate and limited to the affected side
D. Should be immediate and include the asymptomatic side
----------------------------------------

Correct Answers
Question Correct Answer

Question 1 1
Question 2 1
Question 3 1
Question 4 3
Question 5 4
Question 6 2
Question 7 2
Question 8 1
Question 9 4
Question 10 4
Question 11 3
Question 12 3
Question 13 3
Question 14 2
Question 15 4
Question 16 2
Question 17 2
Question 18 3
Question 19 1
Question 20 4
Question 21 4

Solution for Question 1:


Option A: Seminoma
• This is the case of testicular tumors due to the hard stony feel of the testes and bone metastasis.

Page 9

1148
• Testicular cancer is a common cancer in men aged 20 to 40.
• Testicular tumors may be germ cell tumors or those that arise from stromal cells (Leydig or Sertoli
cells).
• Germ cell tumors can be classified as either seminomatous or non-seminomatous.
• Seminoma constitutes more than 50% of all testicular tumors
Option B: Teratoma
• These tumors contain more than one cell type with components derived from ectoderm, endoderm,
and mesoderm.
• It accounts for only 3-5% of tumors.
Option C: Choriocarcinoma
• This is a highly malignant tumor that metastasizes early via both the lymphatics and the bloodstream.
• It is a rare disease.
Option D: Embryonal carcinoma
• It occasionally invades cord structures.
• It accounts for about 25% of tumors

Solution for Question 2:


Option A: Atrophy of Leydig cells & decrease in tubular diameter can occur by two years of age
• Cryptorchidism is a condition in which one testis or both testes fail to descend into the scrotum before
birth.
• Some undescended testes eventually descend by one year, but they are unlikely to descend after this
time.
• Atrophy of Leydig cells decreased tubular diameter, and impaired spermatogenesis can occur by two
years of age.
• It is now recommended that the undescended testicle be surgically repositioned by one year of age to
facilitate spermatogenesis.
• it is associated with testicular cancer and surgery doesn't reduce the risk.
• Nonseminomatous germ cell tumors are the most common tumor type with undescended testes.
Option B: No effect on spermatogenesis in this condition.
• The absence of one testis can affect spermatogenesis.
• Due to temperature changes, spermatogenesis is affected.
Option C: Histologic and morphologic changes can be seen in the testis as early as two months.
• Histologic changes cannot be seen in early neonatal life.
• Histological changes are evident after two years of age.
Option D: Surgery should be done before 3yr of age for optimal result
• Surgery should be done before one year of age.

Page 10

1149
• There is poor healing at a later age.

Solution for Question 3:


Option A: T1
• This is a case of teratoma, as testes are hard with moderate tenderness.

TNM staging for testicular tumours


M1a: Non-regional nodal or pulmonary metastases
M1b: Distant metastasis at a site other than non-regional lymph nodes or lung
S1 : LDH <1.5 x normal, hCG <5000 IU/L, AFP < 1000 ng/mL
S2 : LDH 1.5-10 x normal, hCG 5000 - 50000 IU/L, AFP 1000 - 10000 ng/mL
S3 : LDH >10 x normal, hCG >50000 IU/L, AFP >10000 ng/mL
Option B: T2
• In T2, there is the involvement of tunica vaginalis and lymphatic and vascular invasion.
Option C: T3
• In T3 spermatic cord is involved.
Option D: T4
• In T4 scrotum is involved

Solution for Question 4:


Option C: The usual cause is the absence or incompetence of valves in the proximal testicular vein
• Painless swelling in the testes with the feel of a bag of worms suggests varicocele.
• It is an abnormal dilatation of the pampiniform plexus.
• Its main cause is the incompetence of valves in the testicular vein.
• It can be asymptomatic or may present with a dragging sensation.
• Often bilateral but if present unilateral evaluate for carcinoma
• Most varicoceles are idiopathic; obstruction of the left testicular vein by a renal tumour or nephrectomy
is a cause of varicocele in later life
Option A: Negative impulse on coughing
• Varicocele cannot be palpated by lying down.
• Cough impulse is usually positive in varicocele.
Option B: More common on the right side
• The varicocele is less common on the right side.

Page 11

1150
• It is common on the left side.
Option D: Diagnosis can be made by histopathology.
• Histopathology cannot reveal the diagnosis of varicocele.
• Diagnosis is made on ultrasound and Doppler scan

Solution for Question 5:


Correct Option D - Herniotomy:
• In the above-given case, the signs and symptoms of scrotal swelling, which increases in size and the
presence of a clear fluid transillumination test, are suggestive of the diagnosis of congenital hydrocele.
• In congenital hydrocele, the processus vaginalis connects with the peritoneal cavity. This allows the
peritoneal fluid to be tracked down to form a hydrocele.
• The communication is mostly too small to allow herniation of intra-abdominal contents.
• Hydroceles are unilateral or bilateral, asymptomatic, non tender scrotal swellings.
• Hydroceles may be tense or lax but typically transilluminate.
• Pressure present on the hydrocele does not empty it, but the hydrocele fluid can drain into the
peritoneal cavity at the time when the child is lying down; thus, the hydrocele may be intermittent.
• Congenital hydrocele is treated by herniotomy in case they do not resolve spontaneously 2 years.
Incorrect Options:
Option A - Eversion of sac:
• Eversion of the sac is also known as Jaboulay's procedure.
• The sac is opened and everted behind the testis (Jaboulay's procedure).
• It is used for hydrocele of the testes.
• This procedure is used for acquired hydrocele, not for congenital hydrocele.
Option B - Excision of sac:
• Excision of the sac can be done with acquired hydrocele and not recommended for congenital
hydrocele
Option C - Lord's procedure:
• Lord's operation is suitable when the sac is reasonably thin-walled
• It is recommended for acquired hydrocele, not for congenital hydrocele.

Solution for Question 6:


Correct Option B - Epididymal cyst:
• In the above-given case, the presence of scrotal mass, painful swelling, and pressure on the penis are
suggestive of the diagnosis of the epididymal cyst.

Page 12

1151
• Cysts of the epididymis are the cystic transformation of the aberrant structures like paradidymis,
hydatid of Morgagni (appendix of the epididymis) etc.
• There are actually multiple cysts or multilocular cysts above and behind the body of the testis.
• In contrast to hydrocele, these can be felt apart from the testis.
• They are tense, fluctuant and brilliantly translucent. But due to the presence of multiple septa, brilliant
translucency is finely tessellated, giving the appearance of a Chinese lantern.
• This condition may be bilateral. The cyst lies behind the testis, whereas the hydrocele lies in front of
the testis.

Incorrect Options:
Option A - Spermatocele:
• Spermatocele is an abnormal sac that may develop in the spermatic cord.
• It is filled with clear or milky fluid that may contain sperm.
• It is generally painless and non-cancerous.
• the epidymal cyst which is greater than 2cms is spermatocele
Option C - Hydrocele of cord:
• Hydrocele of the cord can be caused due to unclosed processus vaginalis.
• There might be a passage of fluid between the abdomen and the spermatic cord.
• When the baby lies or cries, the area of the cord changes in size.
Option D - Secondary hydrocele:
• Secondary hydrocele can be partially palpated on physical examination.
• It can be of communicating and non-communicating hydrocele.

Solution for Question 7:

Page 13

1152
Correct Option B - 6 months of age:
Undescended testis
• An undescended testis requires orchidopexy between 6 and 12 months of age.
• Best time – 6 months (Values given closest to this according to question).
• Cryptorchidism, or undescended testis, is a condition in which one testis or both testes fail to descend
into the scrotum before birth.
• The testicle may reside in the retroperitoneum, in the internal inguinal ring, in the inguinal canal, or
even in the external ring. At birth, approximately 95% of infants have the testicle normally positioned in
the scrotum.
• Up to 30% of preterm infants and 3% of full-term infants can present with cryptorchidism. Some
undescended testes eventually descend by 1 year of age, but they are unlikely to descend after this
time.
• A retraction testis reaches the base of the scrotum but retracts.
• Examination reveals a hypoplastic or poorly rugated scrotum. Usually, a unilateral undescended testis
can be palpated in the inguinal canal or in the upper scrotum. If no testicle is palpated, examine with
legs crossed. This manoeuvre diminishes the cremasteric reflex and facilitates the identification of the
location of the testes.
• When the testicle is not present within the scrotum, it is subjected to a higher temperature resulting in
decreased spermatogenesis.
• The undescended testis is associated with histologic and morphologic changes as early as 6 months
of age; atrophy of Leydig cells, decrease in tubular diameter, and impaired spermatogenesis can occur
by 2 years of age.
• The undescended testis also demonstrates an increased predisposition to malignant degeneration.
Non-seminomatous germ cell tumors are usually associated with undescended testis.
• An undescended testis requires orchidopexy between 6 and 12 months of age. It is now
recommended that the "undescended testicle be surgically repositioned by 1 year of age" to optimise
spermatogenesis and improve fertility . Though this procedure improves fertility potential, it is never
normal, and the risk of malignant change is also present. However, the location of the testis in the
scrotum facilitates potentially earlier detection of testicular malignancy.
• The use of chorionic gonadotropin occasionally may be effective in patients with bilateral
undescended testis.
• When an undescended testis is not palpable in the inguinal canal, a diagnostic laparoscopy is useful.
If the testicular vessels are seen exiting the internal ring, an open inguinal orchidopexy is performed.
For an intra-abdominal testis, two-stage Fowler Stephens orchidopexy can be considered in which
testicular vessels are ligated as a first stage to allow collateral circulation to develop for 6 months
before orchidopexy is performed as a second stage.
Incorrect Options:
Option A - Just after birth:
• The best time for surgery for undescended testes is 6 months.
• Early surgical treatment can lower the complications.
Option C - 12 months of age:
• The surgical treatment of undescended testes should be done before the age of 12 months.

Page 14

1153
• Taking a longer time may increase the risk of infertility or testicular cancer.
Option D - 24 months of age:
• The optimal time for orchidopexy is 6 months.
• The early management of cryptorchidism may reduce the risk of complications such as infertility or
testicular cancer.

Solution for Question 8:


Correct Option A - At the root of the penis:
• In the given case, the palpable soft tissue mass in the perineum on the same side of an empty hemi
scrotum perineum is very suggestive of Ectopic testis, and the commonest position to look for would be
the superficial inguinal pouch, but since it's not in the option and ultrasonography already ruled out that
place so it should be looking at the root of the penis.
• An ectopic testis is a type that has passed through the external ring and then has deviated from its
usual path of descent.
• The scrotum is usually empty, and the testis is well-developed.
• The commonest position of the ectopic testis is at the superficial inguinal pouch, which lies just medial
and inferior to the superficial inguinal ring.

• Superficial inguinal pouch (most common location)


• At the root of the penis (pubic type)
• At the perineum (perineal type)
• Rarely at the upper and medial part of the femoral triangle (femoral type)
• The ectopic testis probably occurs due to the rupture of the main scrotal tail of the gubernaculum
testis.
• The most ectopic testis can be felt (are palpable).
• Surgical treatment to place an ectopic testis in its normal position by one year of age.
Incorrect Options:
Option B - Contralateral scrotum:
• The most common location of the ectopic testis is the superficial inguinal pouch.
• The contralateral scrotum is not the common location.
Option C - The medial part of the femoral triangle:
• Although ectopic testis can lie in the medial part of the femoral triangle, this is rare.
• The most common location of the ectopic testis is the superficial inguinal pouch.
Option D - The upper part of the femoral triangle:
• The ectopic testis is rarely found in the upper part of the femoral triangle.
• The most common location is either a superficial inguinal pouch or at the root of the penis.

Page 15

1154
Solution for Question 9:
Option D: Testicular torsion
• In the above case, the signs and symptoms such as scrotal pain of sudden onset, sexual history with
multiple partners, testes appear to have a horizontal lie on examination, and decreased blood flow to
the affected testicle on Doppler ultrasonography are suggestive of Testicular torsion.
• Testicular torsion is the most common genitourinary tract emergency in childhood. It is the most
common cause of acute scrotal pain. This usually occurs in neonates or adolescent males. The supply
of blood to the testicle is compromised due to the spermatic cord twisting within the tunica vaginalis,
resulting in ischemia to the epididymis and the testis.
• Extravaginal torsion is more common in neonates, in whom there can be torsion of the spermatic cord
along its course outside the tunica vaginalis.
• Intravaginal torsion is associated with bell clapper deformity, in which the suspended testis can twist
torsion.
• Twists of 720° cause more rapid ischemia than twists of 360° or less.
• For torsion of fewer than 6 hours, 100% chance of testicular salvage is possible.
• Less than 20% have good prognosis with more than 24 hours of symptoms.
• Clinical findings include sudden onset of intense unilateral scrotal pain unrelated to trauma that may
be associated with nausea and vomiting. The most consistent physical examination finding is loss of
the cremasteric reflex of the testicle. The testis that has undergone torsion may be high riding,
edematous and significantly tender.
• Prehn's sign is negative (On the elevation of the testis, pain is relieved in epi-didymo orchitis but not in
torsion testis).
• Diagnosis is usually made clinically. Ultrasound may be helpful in diagnosis. Radioisotope scanning is
the most specific diagnostic test. On diagnosis, a prompt surgical detorsion with testicular fixation
should be performed to relieve acute testicular ischemia. The diagnosis and surgical repair time directly
correlate with the testicular salvage rate.

Option A: Seminoma
• Seminoma is a malignant germ cell tumor that involves most commonly the testicle or, less frequently,
the mediastinum, the retroperitoneum, or other extra-gonadal sites.
• Seminoma and other malignant testicular tumors present as painless masses that do not trans
illuminate.
Option B: Orchitis
• The inflammation of the testicle, unilaterally or bilaterally usually caused by viruses and bacteria.
• Orchitis is often a complication of mumps and presents with fever, scrotal swelling, and acute scrotal
pain
Option C: Epididymitis
• It is an inflammation of the small, coiled tube at the back of the testicle (epididymis).

Page 16

1155
• Testicular torsion can be differentiated from epididymitis through Doppler ultrasonography, which
shows testicular ischemia in testicular torsion rather than increased blood flow as in epididymitis

Solution for Question 10:


Option D: Seminoma
• In the above-given case, the presence of placental alkaline phosphatase, hard scrotal swelling, and a
dull ache are suggestive of the diagnosis of seminoma.
• Testicular cancer is commonest in men aged 20 to 40 years. Testicular tumors may be germ cell
tumors or those that arise from stromal cells (Leydig or Sertoli cells)
• Germ cell tumors can be classified as either seminomatous or non-seminomatous.
• Seminoma constitutes more than 50% of all testicular tumors. Most seminomas do not produce tumor
markers, but the presence of syncytiotrophoblastic giant cells in a minority of cases accounts for
modest elevations of human chorionic gonadotropin (hCG). Seminomas never produce
alpha-fetoprotein (AFP). Immunohistochemistry is usually positive for placental alkaline
phosphatase(PLAP); negative for CD30, AFP and epithelial membrane antigen (EMA).
• Non-seminomatous germ cell tumors can be divided into the following histologic types: Embryonal
carcinoma: highly malignant tumors that occasionally involve cord structures. It is the most
undifferentiated type of germ cell tumor and is thought to be pluripotent. On immunohistochemical
staining, embryonal carcinoma can be positive for cytokeratin, CD30, PLAP, AFP and hCG. Lactate
dehydrogenase (LDH) concentration in the serum is an important prognostic factor in metastatic
embryonal carcinoma that is marker-negative. Yolk sac tumors are those tumors with this component
that secrete alpha-fetoprotein (AFP). Immunostaining is diffusely positive for AFP and may also be
positive for cytokeratin, SALL4, glypican-3, PLAP and CD117.
• Embryonal carcinoma: highly malignant tumors that occasionally involve cord structures. It is the most
undifferentiated type of germ cell tumor and is thought to be pluripotent. On immunohistochemical
staining, embryonal carcinoma can be positive for cytokeratin, CD30, PLAP, AFP and hCG. Lactate
dehydrogenase (LDH) concentration in the serum is an important prognostic factor in metastatic
embryonal carcinoma that is marker-negative.
• Yolk sac tumors are those tumors with this component that secrete alpha-fetoprotein (AFP).
Immunostaining is diffusely positive for AFP and may also be positive for cytokeratin, SALL4,
glypican-3, PLAP and CD117.
• Embryonal carcinoma: highly malignant tumors that occasionally involve cord structures. It is the most
undifferentiated type of germ cell tumor and is thought to be pluripotent. On immunohistochemical
staining, embryonal carcinoma can be positive for cytokeratin, CD30, PLAP, AFP and hCG. Lactate
dehydrogenase (LDH) concentration in the serum is an important prognostic factor in metastatic
embryonal carcinoma that is marker-negative.
• Yolk sac tumors are those tumors with this component that secrete alpha-fetoprotein (AFP).
Immunostaining is diffusely positive for AFP and may also be positive for cytokeratin, SALL4,
glypican-3, PLAP and CD117.
Option A: Theca cell tumor
• It is present as a painless mass in the testicle.
• It is positive for inhibin, calretinin, and vimentin, not for placental alkaline phosphatase.
Option B: Teratoma

Page 17

1156
• Teratomas contain more than one cell type with components which are derived from ectoderm,
endoderm and mesoderm.
• Teratoma does not produce elevated AFP or hCG.
• Patients with elevated serum AFP and /or hCG should be assumed to have a non-teratoma germ cell
tumor component unless the elevation can be explained otherwise.
Option C: Choriocarcinoma
• Choriocarcinoma strongly expresses human chorionic gonadotropin (hCG).
• This is a highly malignant tumor that metastasises early via both the bloodstream and the lymphatics.
• It is composed of both cytotrophoblasts and syncytiotrophoblasts.

Solution for Question 11:


Correct Option C - Internal spermatic arteries:
• The anatomy of the adult testis reflects its embryonic development.
• Testis arises from portions of the Wolffian bodies on the genital ridge below the kidneys at around the
10th thoracic level; therefore, the major blood vessels for the testes arise high up the retroperitoneum
from the aorta below the renal arteries and are called the internal spermatic arteries or testicular
arteries.

Secondary blood supply to the testes arises from the artery of the vas deferens and a small branch fro
m the epigastric artery termed the external spermatic artery or cremasteric artery, which forms during t
he descent of the testes from the abdomen to the scrotum.

Page 18

1157
• The surgical importance of this phenomenon is that operations involving the region of the renal
arteries may sacrifice the internal spermatic artery.
• The vascular and genital ductal structures leave the testis from the mediastinum in the
posterosuperior portion and travel through the scrotal neck into the inguinal canal.
• The venous drainage of the testicle initially begins as a pampiniform plexus coalescing into the
gonadal or testicular veins.
• On the right, the vein drains directly into the vena cava, and on the left, the vein drains into the left
renal vein.
Incorrect Options:
Option A - Hypogastric arteries:
• Hypogastric arteries are not the main blood supply to the testes.
• Hypogastric arteries supply the pelvic viscera.
Option B - Pudendal arteries:
• The internal pudendal artery supply to the posterior scrotal region.
• The main supply to the testes is via the testicular artery, which arises from the aorta.
Option D - External spermatic arteries:
• External spermatic arteries mainly supply to the spermatic cord.
• The external spermatic arteries are the 2nd blood supply of the testes.

Solution for Question 12:


Correct Option C - Grade III:
• In the above case, the heaviness and distension of the scrotum, with a "bag of worm" finding on
palpation, suggestive of a varicocele. As it is visible and palpable at rest, it is grade III.

Page 19

1158
• Varicocele is an abnormal dilatation of the pampiniform plexus of the internal spermatic venous
system.
• They are unusual in boys and typically develop during late childhood and adolescence
• Sometimes, the dilated vessels are cremasteric veins and not part of the pampiniform plexus. The
usual cause is the absence or incompetence of valves in the proximal testicular vein.
• Etiology:
• While most varicoceles are idiopathic, obstruction of the left testicular vein by a renal tumor or
nephrectomy is a cause of varicocele in later life; characteristically, in such cases, the varicocele does
not decompress in the supine position.
• Signs and symptoms:
• Varicoceles may be asymptomatic, or patients may experience an annoying dragging discomfort that
is worse on standing at the end of the day. This presumably reflects distension of the testicular veins.
• When examined in the erect position, the scrotum on the affected side hangs lower than normal; on
standing, the varicose plexus feels like a bag of worms. There may be a cough impulse.
• Diagnosis
• ultrasonography and color Doppler.
• Treatment:
• percutaneous embolization of the gonadal veins is the usual first-line intervention.
• If this is impossible or if the varicocele recurs, then surgical ligation of the testicular veins is the
appropriate treatment, although recurrence can occur even after surgery.

Varicocele Grading
Grade 1
Palpable during Valsalva maneuver, but not otherwise
Grade 2
Palpable at rest but not visible
Grade 3

Page 20

1159
Visible and palpable at rest
Incorrect Options:
Option A - Grade I:
• If a varicocele is palpable during the Valsalva manoeuvre but not otherwise, it will be grade 1.
• If it is visible and palpable at rest, it is under grade 3.
Option B - Grade II:
• If the varicocele is palpable at rest but not visible, it will fall in grade 2.
• If it is visible at rest and also palpable, then it will be grade 3.
Option D - Grade IV:
• There are usually three grades of varicocele.
• According to Sarteschi's classification of varicocele, grade 4 vessels appear to enlarge in the supine
position.
• Testicular hypotrophy is very common in this stage.

Solution for Question 13:


Answer
Option C: Epididymitis
• In the above mentioned case, the signs and symptoms such as testicular pain and swelling, positive
sexually transmitted infection history, tenderness, erythematous scrotum on palpation and thick
epididymis on ultrasound are suggestive of epididymitis
• Inflammation of epididymis with symptoms of < 6 weeks.
• Doppler Ultrasound should be done to rule out testicular torsion.
• Ultrasound may show enlarged and thick epididymis, increased blood flow, and may also find reactive
hydrocele.
• A urine test may show positive leukocyte esterase.
• Symptoms may include testicular pain and often unilateral lower abdominal pain, urinary frequency or
urgency, and urinary dysuria.
• Treatment should include antibiotics guided by sexual and urologic history.
• Complications can be in the form of sepsis and testicular atrophy.
Other Options
Option A: Seminoma
• Seminoma is a malignant germ cell tumor that involves most commonly the testicle or, less frequently,
the mediastinum, the retroperitoneum, or other extra-gonadal sites.
• Seminoma and other malignant testicular tumors present as painless masses that do not
Transilluminate.
Option B: Orchitis

Page 21

1160
• The inflammation of the testicle, unilaterally or bilaterally usually caused by viruses and bacteria.
• Orchitis is often a complication of mumps and presents with fever, scrotal swelling, and acute scrotal
pain.
Option D: Testicular torsion
• It is the most common genitourinary tract emergency in childhood and the most common cause of
acute scrotum. The supply of blood to the testicle is compromised due to the spermatic cord twisting
within the tunica vaginalis, resulting in ischemia to the epididymis and the testis.
• Testicular torsion is often mistaken for epididymitis due to the similarity in symptoms, including scrotal
pain and tenderness. However, there are key differences between the two conditions. While both may
present with scrotal pain, nausea, and vomiting, testicular torsion typically involves a sudden onset of
severe pain, whereas epididymitis pain tends to develop more gradually.

Solution for Question 14:


Correct Option B - Hydrocele:
• In the above-mentioned case, the signs and symptoms such as a "lump" in his genital area, tense
and non-tender mass in the scrotum, and a transillumination test positive are suggestive of hydrocele.
• A hydrocele is an abnormal collection of serous fluid in a part of the processus vaginalis, usually the
tunica vaginalis.
• Acquired hydroceles are primary or idiopathic or secondary to epididymal or testicular disease.
• A primary hydrocele is seen most commonly in the middle and later life but may also occur in older
children.
• In congenital hydrocele, the processus vaginalis is patent and connects with the peritoneal cavity.
• Funicular Hydrocele: Processus Vaginalis patent up to top of epididymis.
• Infantile Hydrocele: Tunica Vaginalis and processus Vaginalis patent up to Deep Ring.
• Encysted hydrocele: The center part of the cord is unobliterated and has fluid.
• Bilocular Hydrocele: 2 loculi – one above the neck of the scrotum and one below.
• Hydrocele of hernia sac: Neck of sac pulls omentum.
• Congenital hydroceles are treated by herniotomy if they do not resolve spontaneously.
Incorrect Options:
Option A - Inguinal hernia:
• An inguinal hernia results when abdominal organs protrude into the inguinal canal or scrotum.
• About 1-5% of children will have a hernia or hydrocele, including newborns.
Option C - Varicocele:
• A varicocele is a vein enlargement within the loose bag of skin that holds the testicles.
• Often described as feeling like a "bag of worms" due to the dilated and tortuous veins within the
scrotum. It may become more prominent when standing and tends not to transilluminate.
Option D - Epididymal cyst:

Page 22

1161
• These are filled with crystal-clear fluid. They are very common, usually multiple, and vary in size at
presentation. They represent cystic degeneration of the epididymis.
• The clusters of tense cysts feel like tiny bunches of grapes that lie posterior to, and quite separate
from, the testis.
• They Transilluminate brilliantly. The diagnosis can be confirmed by ultrasound.

Solution for Question 15:


Option D: Prehn's sign is positive
• In the above-given case, the sign and symptoms of testicular pain, urinary frequency, burning
sensation, and swollen tender testicles are suggestive of the diagnosis of Epididymo-orchitis.
• Epididymo-orchitis refers to inflammation of the testis and epididymis.
• Common etiologies include sexually transmitted infections in younger males or urinary tract infections
or infections secondary to an indwelling catheter in older males.
• The most common organism causing epididymo-orchitis in < 35 and sexually active males is
Chlamydia.
• The most common organism causing epididymo-orchitis in children, the elderly, and homosexuals is
E.coli.
• Symptoms include pain and swelling of the epididymis and testis. Some men may report nausea and
vomiting due to irritation of the spermatic cord. Urinary symptoms may be present. Physical
examination reveals a tender, swollen epididymis and testis.
• The infection reaches the epididymis via the vas from a primary infection of the urethra, prostate, or
seminal vesicles.
• Secondary hydrocele is associated with acute or chronic epididymo-orchitis. It is usually lax and is
typically small; the underlying testis is usually palpable. A secondary hydrocele is severe when the
primary lesion resolves.
• Prehn's sign: The affected hemiscrotum is elevated. This action relieves the pain of epididymitis while
the pain worsens in torsion (positive Prehn's sign).
• Investigations should include a urethral swab, a urine specimen for culture, nucleic acid amplification
testing (NAAT) and scrotal ultrasound.
• Treatment involves antibiotics, rest and scrotal elevation.
Option A: Most common causative organism in <35 years old and sexually active males is Neisseria go
norrhoeae
• The most common causative organism causing epididymo-orchitis in <35 years old and sexually
active males is Chlamydia.
Option B: Cremasteric reflex negative
• Cremasteric reflex is positive in epididymo-orchitis.
• A cremasteric reflex is performed to evaluate the acute scrotal pain for testicular torsion.
Option C: Most common cause of primary hydrocele
• It is the most common cause of secondary hydrocele, not primary.

Page 23

1162
• Secondary hydrocele is associated with acute or chronic epididymo-orchitis.

Solution for Question 16:


Correct Option B - Skin & subcutaneous tissue:
• The Image shows blackish discoloration of scrotal skin and some part of penile skin: Fournier's
gangrene.
• Aka idiopathic scrotal gangrene.
Fournier's gangrene
• Fournier's gangrene is a necrotizing fasciitis of the male genitalia and perineum (secondary to
infection by aerobic and anaerobic bacteria), affecting predominantly middle-aged and older men with
pre-existing comorbidities. It involves the skin and subcutaneous tissues. It is rare for the infection to
involve the testicles or deep tissues of the penis because of the tunica vaginalis and Buck fascia,
respectively, so these structures can be preserved. Corpora cavernosa lies under the Buck’s fascia.
• It is a urological emergency that often presents with swelling, pain, and erythema of the genital region,
which may progress to discoloration and frank necrosis accompanied by systemic features such as
fever and rigor.
• The presence of crepitus is virtually diagnostic.
• It is usually a polymicrobial infection with microaerobes, anaerobes and gram-positive and
gram-negative organisms. It spreads along Dartos, Scarpas and Colles' fascia.
• Risk factors for development include peripheral vascular disease, diabetes mellitus, malnutrition,
alcoholism, and other immunocompromised states.
• Given the potential for rapid spread and tissue destruction, early diagnosis and urgent surgical
drainage with aggressive debridement of the necrotic tissue is an essential component of management,
along with the broad-spectrum antibiotic cover.
• It is uncommon for the urethra to be involved, although a defined urinary tract source may be evident
such as urethral stricture with perforation and local infection.
• Mortality is high and related to the level of existing co-morbidity.
Incorrect Options:
Option A - Skin:
• There is necrosis of skin and subcutaneous tissue.
• Only the excision of skin is not useful for the management, so the subcutaneous tissue should also be
excised.
Option C - Skin, subcutaneous tissue, and corpora cavernosa:
• Fournier's gangrene involves the skin and subcutaneous tissues.
• It is rare to involve corpora cavernosa so there is no need to excise corpora cavernosa.
Option D - Skin, subcutaneous tissue, corpora cavernosa & testis:
• It is rare for the infection to involve the testicles or deep tissues of the penis because of the tunica
vaginalis and Buck fascia.
• So these structures can be preserved.

Page 24

1163
• Corpora cavernosa lies under the Buck’s fascia.

Solution for Question 17:


Option B: Klinefelter's syndrome
• Testicular cancer is the most common cancer in men aged 20 to 40 years.
• Testicular tumors may be germ cell tumors or those that arise from stromal cells (Leydig or Sertoli
cells).
• Germ cell tumors can be classified as either seminomatous or non-seminomatous. Seminoma
constitutes more than 50% of all testicular tumors.
• Non-seminomatous germ cell tumors can be divided into the following histologic types: Embryonal
carcinoma is a highly malignant tumor that occasionally invades cord structures. Yolk sac tumors are
tumors with this component that secrete alpha-fetoprotein (AFP). Choriocarcinoma: Often produces
human chorionic gonadotropin (hCG). This is a highly malignant tumor that metastasizes early by both
the lymphatics and the bloodstream. Teratoma: These tumors contain more than one cell type with
components derived from ectoderm, endoderm, and mesoderm.
• Embryonal carcinoma is a highly malignant tumor that occasionally invades cord structures.
• Yolk sac tumors are tumors with this component that secrete alpha-fetoprotein (AFP).
• Choriocarcinoma: Often produces human chorionic gonadotropin (hCG). This is a highly malignant
tumor that metastasizes early by both the lymphatics and the bloodstream.
• Teratoma: These tumors contain more than one cell type with components derived from ectoderm,
endoderm, and mesoderm.
• Embryonal carcinoma is a highly malignant tumor that occasionally invades cord structures.
• Yolk sac tumors are tumors with this component that secrete alpha-fetoprotein (AFP).
• Choriocarcinoma: Often produces human chorionic gonadotropin (hCG). This is a highly malignant
tumor that metastasizes early by both the lymphatics and the bloodstream.
• Teratoma: These tumors contain more than one cell type with components derived from ectoderm,
endoderm, and mesoderm.
There are five well-established risk factors for testis cancer:
• White race
• Cryptorchidism
• Family history of testicular tumor,
• Personal history of testicular tumor
• Intratubular germ cell neoplasia (ITGCN)
Klinefelter's syndrome is also a risk factor for testis cancer.
Testicular cancer typically presents as testicular pain or a testicular mass. A scrotal ultrasound and ser
um tumor markers are the standard initial workup. Radical inguinal orchiectomy is the gold standard tre
atment for excision of the primary tumor. Pure seminoma is radiosensitive and can be treated with exte
rnal beam radiation or a single dose of carboplatin. Non-seminomatous testis cancer can be cured with
retroperitoneal lymph node dissection or chemotherapy. Seminomatous germ cell tumors generally ha

Page 25

1164
ve a better prognosis than non-seminomatous tumors.
Option A: Extratubular germ cell neoplasia
Extratubular germ cell neoplasia is not a risk factor for testicular tumors.
Option C: CA scrotum
• It is a squamous cell carcinoma that may present as an ulcer with an everted edge.
• It was the first reported occupational cancer.
• It is not the risk factor for testicular tumors; instead, testicular tumors are the risk factor for the CA
scrotum.
Option D: Acute Epididymo-Orchitis (EDO)
• Acute Epididymo-Orchitis (EDO) is an acute inflammatory disease of the ipsilateral testes and
epididymis.
• It is the risk factor for infertility, not for testicular tumors.

Solution for Question 18:


Option C: Intratubular germ cell neoplasia
• In the above-given case, the sign and symptoms of painless swelling of the right testes, bulky right
testis measuring 5.8 cm × 4.7 cm × 5.0 cm, thickened tubular walls and stroma are suggestive of the
diagnosis of Intratubular germ cell neoplasia.
• With the exception of spermatocytic tumors, postpubertal invasive germ cell tumors arise from a
precursor lesion known as intratubular germ cell neoplasia (ITGCN) or germ cell neoplasia in situ
(GCNIS).
• Exceptions to this are prepubertal germ cell tumors (which can rarely occur after puberty), ovarian
cystic teratomas, dermoid cysts and spermatocytic tumors (previously referred to as spermatocytic
seminomas).
• GCNIS consists of undifferentiated germ cells that have the appearance of seminoma that are located
basally within the seminiferous tubules. The tubules usually show decreased or absent
spermatogenesis and normal constituents are replaced by GCNIS.
• GCNIS is diagnosed by a testicular biopsy that is performed for the investigation of infertility,
contralateral testis biopsy in a patient with germ cell tumor or within the affected testis in a patient
undergoing testis-sparing surgery.
• GCNIS is associated with a 50% risk of developing an invasive germ cell tumor within 5 years. Radical
orchiectomy or low dose (> 20 Gy) radiation therapy is an effective treatment option for GCNIS, of
which radical orchiectomy is the most definitive therapy.
• Patients with GCNIS show reduction (but do not eliminate) in the risk of developing germ cell tumors
on receiving cisplatin-based chemotherapy.
Option A: Seminoma
• Most seminomas do not produce tumor markers, but the presence of syncytiotrophoblastic giant cells
in a minority of cases accounts for modest elevations of human chorionic gonadotropin (hCG).
• Seminomas never produce alpha-fetoprotein (AFP).

Page 26

1165
• Immunohistochemistry is usually positive for placental alkaline phosphatase (PLAP); negative for
CD30, AFP and epithelial membrane antigen (EMA).
Option B: Testicular teratoma
• Teratomas contain more than one cell type with components derived from ectoderm, endoderm and
mesoderm.
• It presents with painless swelling of the testes.
Option D: CA Scrotum
• It is a squamous cell carcinoma that may present as an ulcer with an everted edge.
• The growth starts as a wart or ulcer, and as it grows, it may involve the testis.

Solution for Question 19:


Option A: It's a basal cell carcinoma of the scrotum
Chimney sweeps cancer: CA scrotum
• The above-given case is a squamous cell carcinoma that may present as an ulcer with an everted
edge.
• It was the first reported occupational cancer.
• The growth starts as a wart or ulcer, and as it grows, it may involve the testis.
• The tumor should be excised with a margin of healthy skin.
• Inguinal lymph nodes are the first involved nodes, scrotal skin is supplied by the superficial lymph
nodes. The tumor is diagnosed by scrotal ultrasound.
• The management of the inguinal nodes parallels the management of penile cancer, and if nodal
enlargement does not settle with antibiotics following treatment of the primary, then a bilateral groin
dissection is indicated.
Option B: Diagnosis is established by biopsy
• A biopsy is not recommended for the diagnosis of chimney sweep cancer or CA scrotum due to the
risk of proliferation.
• A scrotal USG is recommended in the diagnosis of the CA scrotum.
Option C: Para-aortic lymph nodes are the last involved lymph nodes
• Testicular lymphatic drain into the para-aortic lymph nodes.
• Since this is scrotal cancer, they are not the first lymph nodes to be involved in scrotal cancer.
Option D: Treated by wide local excision
• The tumor should be excised with a margin of healthy skin.
• The wide local excision is not recommended in the CA scrotum.

Solution for Question 20:

Page 27

1166
Option D: Cremasteric vein
Varicocele:
• Varicocele is an abnormal dilatation of the pampiniform plexus of the internal spermatic venous
system.
• In some cases, the dilated vessels are cremasteric veins and not part of the pampiniform plexus. The
usual cause is the absence or incompetence of valves in the proximal testicular vein.
• Varicoceles may be asymptomatic, or patients may experience an annoying dragging discomfort that
is worse on standing at the end of the day. This presumably reflects distension of the testicular veins.
• When examined in the erect position, the scrotum on the affected side hangs lower than normal; on
standing, the varicose plexus feels like a bag of worms. There may be a cough impulse.
The association of varicocele with subfertility is controversial
• Diagnosis can be made by ultrasonography and colour Doppler
• Treatment - Operation not indicated for asymptomatic patients When the discomfort is significant, then
percutaneous embolisation of the gonadal veins is the usual first-line intervention. If this is not possible
or if the varicocele recurs, then surgical ligation of the testicular veins is the appropriate treatment,
although recurrence can occur even after surgery.
• Operation not indicated for asymptomatic patients
• When the discomfort is significant, then percutaneous embolisation of the gonadal veins is the usual
first-line intervention.
• If this is not possible or if the varicocele recurs, then surgical ligation of the testicular veins is the
appropriate treatment, although recurrence can occur even after surgery.
• Operation not indicated for asymptomatic patients
• When the discomfort is significant, then percutaneous embolisation of the gonadal veins is the usual
first-line intervention.
• If this is not possible or if the varicocele recurs, then surgical ligation of the testicular veins is the
appropriate treatment, although recurrence can occur even after surgery.
Treatment options: -
• High inguinal ligation (near deep ring) of pampiniform plexus of veins.
• Microvascular sub-inguinal ligation (best results).
• Palomo's operation: ligation of Left Gonadal Vein in retroperitoneum.
• Alternative venous drainage via cremasteric veins.
Option A: IVC
• IVC is a large blood vessel which transports deoxygenated blood to the right atrium.
• It is a thin-walled blood vessel.
• It is not used as an alternative vessel for venous drainage in varicocele surgery.
Option B: Aorta
• Aorta is the main blood vessel which carries oxygenated blood from the heart and supply to the body.
• As it carries oxygenated blood so it cannot be used for venous drainage.
Option C: Superior epigastric vein

Page 28

1167
• The superior epigastric vein drains from the abdominal wall and some of the diaphragm.
• It cannot be used as an alternative vessel to drain from the scrotum.

Solution for Question 21:


Option D: Should be immediate and include the asymptomatic side
• This is the case of testicular torsion, as the patient presented after trauma with a painful scrotal and
inguinal area associated with nausea and vomiting.
• This occurs in neonates and adolescent male.
• The blood supply to the testicle is compromised due to the torsion of the spermatic cord within the
tunica vaginalis, resulting in ischemia to the epididymis and the testis.
• On diagnosis, a prompt surgical detorsion with testicular fixation should be performed to relieve acute
testicular ischemia.
• The contralateral side should be fixed, as that side can also be affected.
Option A: Can be delayed 24 hours and limited to the affected side
• It cannot be delayed as it can lead to ischemia of the testes.
• The contralateral side should also be explored as that can also be prone to testicular torsion.
Option B: Can be delayed but should include the asymptomatic side
• Surgery cannot be delayed due to avascular necrosis.
Option C: Should be immediate and limited to the affected side
• The contralateral side should be explored as this anatomical defect can also occur on that side.

Page 29

1168
Previous Year Questions
1. What is the diagnosis of a 6-year-old boy with a history of recurring urinary tract infections, based on
the provided imaging?

A. Vesicoureteric reflux
B. Urinary bladder diverticulum
C. Urinary bladder hernia
D. Vesicocolic fistula
----------------------------------------
2. What is the probable diagnosis based on the delayed intravenous urogram of a patient?

A. Pelviureteric junction obstruction


B. Putty kidney
C. Staghorn calculus
D. Cystic kidney
----------------------------------------
3. What is the likely diagnosis for a 40-year-old man with a history of hypertension, who is experiencing
recurring instances of blood in urine and pain in the lower back? It is worth mentioning that his older
sibling died from a stroke at the same age. Please refer to the provided abdominal ultrasound image for
further evaluation.

1169
A. Renal cell carcinoma
B. Autosomal dominant polycystic kidney disease
C. Tuberculosis of the kidney
D. Autosomal recessive polycystic kidney disease
----------------------------------------
4. In a patient with prostate carcinoma, which has metastasized to the vertebrae, what is the pathway
through which it spreads to the lumbar vertebra?
A. Prostatic venous plexus
B. Transcoelomic spread
C. Inferior vesical vein
D. Internal iliac vein
----------------------------------------
5. What is the probable diagnosis for an elderly male patient who experienced scrotal pain and
discharge, as indicated in the provided image?

A. Torsion of testis
B. Acute epididymo-orchitis
C. Scrotal carcinoma
D. Fournier’s gangrene
----------------------------------------

Page 2

1170
6. The surgery OPD received a referral for a 33-year-old male from the infertility clinic. Upon
examination, the scrotum displayed the appearance depicted in the image. Palpation revealed a
sensation similar to a bag of worms. What is the most probable diagnosis?

A. Hydrocele
B. Varicocele
C. Testicular torsion
D. Epididymo Orchitis
----------------------------------------
7. Which of the subsequent options are indicative of storage symptoms associated with benign
prostatic hyperplasia? Urgency Enuresis Hesitancy Frequency Dribbling
A. 1, 3, 4
B. 2 and 5
C. 1, 2, 4, 5
D. 1, 2, 4
----------------------------------------
8. A 10-month-old male baby was observed to have an empty scrotum. An ultrasound examination
showed that the right testis was located in the right inguinal canal, while the left testis was found in the
perineum. Please provide your comments on the diagnosis of this baby.
A. Bilateral undescended testis
B. Bilateral ectopic testis
C. Right side undescended testis and left sided ectopic testis
D. Right side ectopic testis and left side undescended testis
----------------------------------------
9. What is the likely diagnosis for a patient who came to the hospital with a complaint of more frequent
urination and was found to have a pale ureteric orifice during cystoscopy? The patient's KUB x-ray is
provided for reference.

Page 3

1171
A. Bladder carcinoma
B. Schistosomiasis
C. Bladder tuberculosis
D. Papilloma of urinary bladder
----------------------------------------
10. A 77-year-old man presented with worsening hip and back pain. He also reports weight loss and
loss of appetite. His PSA levels are elevated. What is the most likely diagnosis here?
(or)
What is the probable diagnosis for a 77-year-old male who visited the clinic due to deteriorating hip and
back pain, along with symptoms of weight loss, loss of appetite, and elevated PSA levels?

A. Ankylosing spondylitis
B. Osteosarcoma
C. Osteoclastoma
D. Prostate cancer
----------------------------------------
11. A 50-year-old male patient presents with palpitations, episodic headaches, and sweating episodes.
Workup shows a small left-sided suprarenal mass. Which of the following would be done for
confirmation?
A. Urinary vanillyl mandelic acid levels
B. Blood 5-HIAA levels

Page 4

1172
C. Urine myoglobin
D. Stool trypsin levels
----------------------------------------
12. If radiotherapy is administered along with chemotherapy for bladder cancer, which drug's dosage
should be decreased?
A. Cisplatin
B. Gemcitabine
C. Paclitaxel
D. Ifosfamide
----------------------------------------
13. Which of the subsequent options is employed in the treatment of prostate cancer?
A. Gleason score
B. Bethesda score
C. Alvarado score
D. Nottingham score
----------------------------------------
14. What is the most suitable course of action for the condition of a 44-year-old woman who arrives at
the hospital with a palpable mass on her flank? The mass measures 4cm according to CT imaging, and
urine examination reveals the presence of malignant cells.

A. Partial nephrectomy
B. Partial nephrectomy + neoadjuvant chemotherapy
C. Radical nephrectomy
D. Radical nephrectomy + postoperative radiotherapy
----------------------------------------
15. Which of the options listed below is most likely to present with unilateral hydronephrosis?
A. Phimosis
B. Posterior urethral valves
C. Urethral strictures

Page 5

1173
D. Retrocaval ureter
----------------------------------------
16. Which of the following statements regarding the image is false?
(or)
Which of the following statements is not true about the image shown below?

A. Staph. aureus is the causative organism


B. Common in diabetic patients
C. High mortality
D. The testes remain unaffected
----------------------------------------
17. Which structures' involvement is responsible for the clinical presentation depicted in the image?

A. Superficial lymphatics
B. Deep lymphatics
C. Superficial veins
D. Skin
----------------------------------------
18. What treatment options would you suggest for a 50-year-old female patient who has undergone
thyroidectomy and seeks a second opinion regarding the management of papillary thyroid cancer?
A. Radiotherapy
B. Chemotherapy

Page 6

1174
C. Radioiodine scan
D. Wait and watch.
----------------------------------------
19. A patient who was diagnosed with prostate cancer is being investigated. The bone scan is reported
as a super scan with increased uptake in the bones and reduced activity in the spleen. What is the
reason for this super scan appearance?
A. Increased uptake due to diffuse metastasis
B. Increased uptake by carcinoma prostate
C. Increased uptake by the bone
D. Decreased uptake by adrenal glands and kidney
----------------------------------------
20. A 51-year-old woman comes in with concerns about a lump in her right breast. Upon examination,
the lump measures over 5 cm in size and there are movable lymph nodes in the same side armpit.
What would be the appropriate stage for this patient, assuming the lump is cancerous?
A. CT3N1MO
B. ■T4N1M0
C. CT4N1MO
D. ■T3N1M0
----------------------------------------
21. The arrow in the image indicates a probable condition in a male patient who presented with
symptoms of hematuria and recurrent urinary tract infection.

A. Polycystic kidney disease


B. Horseshoe kidney
C. Hydronephrosis
D. Pancake kidney
----------------------------------------
22. Which of the statements below is false regarding nephroblastoma?
A. Treatment is chemotherapy followed by surgery
B. Early lung mets

Page 7

1175
C. Hematogenous spread
D. Most common intra abdominal malignancy in children
----------------------------------------
23. What is the initial management approach for urinary symptoms in a patient with benign prostatic
hyperplasia (BPH)?
A. TURP - Transurethral resection of prostate
B. Tamsulosin
C. Finasteride
D. Observation
----------------------------------------
24. Which of the following is the recommended treatment for the condition shown in the image, here
size of stone is 2mm?

A. Percutaneous nephrolithotomy
B. Ureteroscopy
C. ESWL
D. Wait and watch
----------------------------------------
25. A diabetic patient presents with sudden-onset perineal pain. On examination, foul-smelling
discharge, and necrotic tissue as seen in the image is noted. Which of the following is true about the
given condition?

A. Anti-gas gangrene serum indicated for all cases

Page 8

1176
B. Urinary diversion is the next step
C. Bilateral orchidectomy must be done
D. Mixed aerobic and anaerobic infection
----------------------------------------
26. What does the number(14Fr) marked on the Foley catheter correspond to?

A. Inner lumen diameter


B. Length of the tube
C. Outer tube diameter
D. Bulb size
----------------------------------------
27. After undergoing surgery to correct his left-sided varicocele, a 30-year-old male patient inquires
about the primary route through which venous drainage occurs following the surgery. How should the
surgeon correctly respond to the patient's question?
A. Cremasteric veins
B. Penile veins
C. Ectopic drainage in the iliac fossa
D. Testicular veins
----------------------------------------
28. What would be your most probable course of action for a 75-year-old male diagnosed with prostate
cancer, having a PSA level of 9 ng/mL and a small tumor focus, along with a Gleason score of 6?
A. Radical prostatectomy
B. External beam radiation
C. Brachytherapy
D. Active surveillance
----------------------------------------
29. What is the gross specimen obtained from a cystoprostatectomy?

Page 9

1177
A. Malakoplakia
B. Bladder carcinoma
C. Prostate carcinoma
D. Schistosomiasis
----------------------------------------
30. A patient underwent cholecystectomy for gallstones 2 years ago. She now presents with symptoms
of obstructive jaundice. Investigation reveals a stone in the common bile duct. What is the stone most
likely to be?
A. Primary
B. Secondary
C. Tertiary
D. Retained
----------------------------------------
31. Which of the following statements accurately describes the condition mentioned below?

A. A Patient must be started on anti-gas gangrene serum therapy


B. It is a mixed flora aerobic and anaerobic infection
C. Bilateral orchiectomy is generally performed
D. Compulsory urinary diversion is performed
----------------------------------------

Page 10

1178
32. What is the identified condition in the image provided for a patient presenting with abdominal pain
and sterile pyuria?

A. Putty kidney
B. Nephrocalcinosis
C. Staghorn calculus
D. Psoas calcification
----------------------------------------
33. Thimble bladder is seen in
A. Acute TB
B. B. Chronic TB
C. Neurogenic bladder
D. Schistosomiasis
----------------------------------------
34. What is the recommended initial investigation for genital tuberculosis?
A. Endometrial biopsy
B. Urine sample for acid-fast bacilli (AFB) culture
C. Genital swab for polymerase chain reaction (PCR) testing
D. Chest X-ray
----------------------------------------
35. Cellulitis is:
A. Nonsuppurative & Noninvasive
B. Nonsuppurative & invasive
C. Suprative & Noninvasive
D. Suprative & Invasive
----------------------------------------
36. In which zone is prostate cancer predominantly observed?
A. Central zone
B. Transition zone

Page 11

1179
C. Peripheral zone
D. Anterior fibromuscular stroma
----------------------------------------
37. The hospital received a patient with varicose veins, and an intern was on duty. Which test should
the intern conduct to exclude the possibility of deep vein thrombosis (DVT)?
A. Brodie Tredelenberg test
B. Perthe’s test
C. Thomas test
D. Ober test
----------------------------------------
38. Where will the left kidney transplantation be located?
A. Iliac fossa
B. Kidney fossa
C. Retroperitoneal space
D. Hypogastric region
----------------------------------------
39. Identify the investigation and the lesion depicted in the image given below.
(or)
Identify the investigation and the lesion depicted in the image given below.

A. Retrograde cystourethrogram with membranous urethra stricture


B. Retrograde cystourethrogram with penile urethral stricture
C. Micturating cystourethrography with bulbar stricture
D. Micturating cystourethrography with prostatic urethra stricture
----------------------------------------
40. What is the preferred treatment for renal cell carcinoma with a tumor size smaller than 4 cm?
A. Partial nephrectomy
B. Partial nephrectomy + neoadjuvant chemotherapy
C. Radical nephrectomy

Page 12

1180
D. Radical nephrectomy + postoperative radiotherapy
----------------------------------------
41. A man was brought to the emergency department after he fell into a manhole and injured his
perineum. He gets an urge to micturate but is unable to urinate. On examination, blood was seen at the
tip of the urethra, and swelling of the penis and scrotum was seen. What is the site of injury?
(or)
A male patient was brought to the emergency department following a fall into a manhole, resulting in
injury to his perineum. He experiences a strong urge to urinate but is unable to do so. During
examination, blood is observed at the opening of the urethra, and there is swelling of the penis and
scrotum. Which specific area is affected by the injury?
A. Membranous urethra rupture
B. Bulbar urethra injury
C. Prostatic urethra injury
D. Urinary bladder rupture
----------------------------------------
42. Please identify the abnormal condition outlined below.

A. Hydrocele
B. Varicocele
C. Testicular atrophy
D. Testicular Torsion
----------------------------------------
43. What is the most likely diagnosis for a young girl who presents with continuous dribbling of urine
despite voiding normally, as reported by her parents who mention her never being completely dry since
birth?
A. Ectopic ureter
B. Ureterocele
C. Congenital megaureter
D. Vesicoureteric reflux
----------------------------------------

Page 13

1181
44. A 1-month-old infant is brought to the outpatient department for a consultation regarding an empty
scrotal sac. During examination, bilateral inguinal masses are felt. What should be the subsequent
course of action for managing this child?
A. Perform karyotyping
B. Surgical exploration within the next 2 weeks
C. Perform orchidopexy now
D. Wait and watch for spontaneous descent within the next 6 months
----------------------------------------
45. The most common type of renal stones is
A. Calcium oxalate
B. Cystine
C. Triple phosphate
D. Uric acid
----------------------------------------
46. For which of the following conditions is the Whitaker test recommended?
A. Hydronephrosis
B. Posterior urethral valve
C. Urethral atresia
D. Urethral stricture
----------------------------------------
47. What is the probable diagnosis for a 34-year-old man who has a painless lump in the right groin
area? During examination, a cough impulse is felt when pressing on the index finger, and the swelling
extends into the scrotum. The swelling can be completely reduced by applying pressure through the
deep inguinal ring, accompanied by a gurgling sound.
A. Femoral hernia
B. Direct inguinal hernia
C. Indirect inguinal hernia
D. Umbilical hernia
----------------------------------------
48. A 50-year-old malecomes to the clinic with swelling in the scrotum. Upon examination, the testis
can be felt as distinct from the swelling in the scrotum, and the swelling exhibits transillumination. What
is the most likely diagnosis for this patient?
(or)
A 50-year-old male presents with scrotal swelling that he has noticed recently. On examination, the
testis can be palpated as a separate structure from the scrotal swelling. The swelling is also noted to
show transillumination when examined with a light source. The patient reports no pain or discomfort
associated with the swelling. What is the most likely diagnosis for this patient?
A. Spermatocele
B. Epididymal cyst

Page 14

1182
C. Secondary hydrocele
D. Primary hydrocele
----------------------------------------
49. What is the most probable diagnosis for a 33-year-old man who was referred to the surgery
outpatient department (OPD) from the infertility clinic, where a bag of worms sensation was detected on
palpation along with dilated veins?

A. Hydrocele
B. Varicocele
C. Testicular torsion
D. Epididymo Orchitis
----------------------------------------
50. What would be the best treatment option for a patient who has been diagnosed with renal cell
carcinoma and has a tumor measuring 10 cm in size, located at the lower part of the kidney, and
negative regional lymph nodes?
A. Radical nephrectomy
B. Patrial nephrectomy
C. Radiotherapy
D. Chemotherapy
----------------------------------------
51. What is the most likely diagnosis for a patient who has a scrotal mass on the left side that is hard in
texture, has uneven edges, does not decrease in size when lying down or manipulated, does not have
a cough impulse, but shows enlargement in the para-aortic lymph nodes?
A. Hydrocele
B. Direct inguinal hernia
C. Indirect inguinal hernia
D. Metastasising testicular tumor
----------------------------------------
52. A 26 year old male presented with a painless large round firm right testicular swelling, with a
dragging sensation. The left testis was normal. AFP was around 3000U. What is the next best step in
the management?

Page 15

1183
A. FNAC
B. Scrotal true cut biopsy
C. High inguinal orchidectomy
D. Repeat tumor markers
----------------------------------------
53. How would you approach the management of a 20-year-old patient who was admitted to the
emergency room following a bicycle accident? The patient's vital signs are stable, but there is a bruise
observed in the perineal area, and he has not been able to urinate. Additionally, blood is visible at the
meatus, and the bladder cannot be felt upon palpation.
A. Wait and send him home
B. Immediate insertion of foleys catheter
C. Suprapubic aspiration of urine
D. Wait for bladder to fill and urge to urinate
----------------------------------------
54. What is the primary reason for bladder outlet obstruction in male children?
A. Posterior urethral valve
B. Urethral atresia
C. Anterior urethral valve
D. Ureterocele
----------------------------------------
55. Which of the following is not considered a lower urinary tract symptom?
A. Sudden urge to urinate
B. Incontinence while asleep
C. Incontinence when patient lies down and coughs
D. Incontinence during intercourse
----------------------------------------
56. During the child's bath, the mother observed a swelling in the abdomen of her 3-year-old child. The
child had experienced hematuria two weeks prior, which had resolved on its own. Upon examination, a
reniform mass on the right side that could be moved was detected. What further tests will you conduct
to investigate this condition? CECT PET-CT 24 hour urinary VMA FNAC
A. 1 and 2
B. 2 and 4
C. 1, 2 and 3
D. 1,2,3 and 4
----------------------------------------
57. A 50-year-old male patient came to the outpatient department with complaints of hematuria. A 2x2
cm bladder mass is seen which is low grade transitional cell carcinoma. Which among the following is
the ideal management ?

Page 16

1184
A. Transurethral resection of the tumor
B. Resection with ileal conduit
C. Partial cystectomy with bladder reconstruction
D. Neoadjuvant chemotherapy
----------------------------------------
58. Which drugs are known to be associated with the development of bladder carcinoma?
A. Cyclophosphamide
B. Cisplatin
C. Taxane
D. Tamoxifen
----------------------------------------
59. What is the likely clinical diagnosis based on the retrograde ureteropyelogram image of a patient
experiencing loin pain?

A. Hydronephrosis
B. Duplex kidney
C. Renal carcinoma
D. Renal stone
----------------------------------------
60. Which of the subsequent Caused by infection of protease?

Page 17

1185
A. Uric acid
B. Ammoniummagnesium phosphate
C. Struvite
D. Calcium oxalate
----------------------------------------
61. What is the diagnosis of the patient, a 45-year-old male, who underwent orchidectomy due to
scrotal swelling, based on the histopathology finding depicted in the image provided?

A. Seminoma
B. Yolk-sac tumor
C. Embryonal cell carcinoma
D. Teratoma
----------------------------------------
62. What is the most frequently encountered pathogen responsible for acute bacterial prostatitis?
A. Enterococcus
B. Proteus
C. Streptococcus Agalacticia
D. E.coli
----------------------------------------
63. A male baby, aged 10 months, arrived with an absence of testes in the scrotum. An ultrasound
examination indicated that the right testis was located within the right inguinal canal, while the left testis
was found in the perineum. Please provide your analysis of this baby's diagnosis.
A. Bilateral undescended testis
B. Bilateral ectopic testis
C. Right-side undescended testis and left-sided ectopic testis
D. Right side ectopic testis and left side undescended testis
----------------------------------------
64. What is the most probable diagnosis for a 14-year-old boy who is experiencing sudden scrotal pain,
where physical examination reveals a swollen and red scrotum with the right testes being retracted,
absent cremasteric reflex, and negative Prehn's sign?

Page 18

1186
A. Right-sided hydrocele
B. Strangulated inguinal hernia
C. Right testicular torsion
D. Right epididymo orchitis
----------------------------------------
65. A 68-year-old man presents to OPD with low backache. His PSA level are 100ng/ml. Which of the
following drug is indicated for treatment?
(or)
Which drug is recommended for the treatment of a 68-year-old man who presents with low backache
and a PSA level of 100ng/ml in the outpatient department (OPD)?
A. Somatostatin
B. Terlipressin
C. Goserelin
D. Testosterone
----------------------------------------
66. What is the recommended treatment for a severely obese 25-year-old male patient who has
presented with severe abdominal pain and urinary disturbances? The patient's urine microscopy shows
the presence of specific crystals, and a kidney ultrasound has identified a 2.5 cm renal stone.

A. Extracorporeal shockwave lithotripsy (ESWL)


B. Percutaneous nephrolithotomy (PCNL)
C. Open surgery
D. Ureteroscopy assisted removal
----------------------------------------
67. As per the Weigert Meyer's principle of ureter duplication, the ureter originating from the lower pole
of the kidney inserts into the urinary bladder at which location?
A. Lateral and cephalad to the upper pole ureter
B. Lateral and caudal to the upper pole ureter
C. Medial and cephalad to the upper pole ureter
D. Medial and caudal to the upper pole ureter

Page 19

1187
----------------------------------------
68. What is true regarding distended bladder following which he developed sudden severe pain in
hypogastrium associated with syncope and has no desire to micturate?
(or)
What is correct about the condition of a 35-year-old man who experienced a blow to his swollen bladder
resulting in sudden severe pain in the lower abdomen, accompanied by fainting? As the pain
decreased, his abdomen started to swell, but he did not feel the urge to urinate.
A. This a typical case of extra peritoneal bladder rupture
B. MRI is the ideal investigation to diagnose bladder rupture
C. Repair is done by suturing edges with single-layer 2/0 absorbable suture
D. Laparoscopic methos is avoided as it can be done only on laparotomy
----------------------------------------
69. An 11-month-old boy had a recent urinary tract infection that resolved with antibiotics. Vital signs
are within normal limits. Physical examination revealed an empty scrotum on the right side with normal
left testis. However, a mass was palpable on the right inguinal canal. The USG revealed the right
testicle under the subcutaneous fast near the right inguinal canal. What is the next best step in
management?
(or)
What should be the next course of action in managing an 11-month-old boy who recently had a urinary
tract infection that was treated with antibiotics? The boy's vital signs are normal, but physical
examination reveals an empty scrotum on the right side with a normal left testis. However, a mass is
palpable in the right inguinal canal. Ultrasonography shows the right testicle located beneath the
subcutaneous fat near the right inguinal canal, as depicted in the image. What would be the most
appropriate next step in managing this condition?

A. Observation
B. Laparoscopy
C. Orchiodopexy
D. Surgical exploration
----------------------------------------
70. A 65-year-old male patient visited his primary care doctor complaining of a gradual increase in
urinary frequency over the past three months, without experiencing any burning sensation. The patient
also mentioned having to wake up four times during the night to urinate. Despite reducing his fluid

Page 20

1188
intake and avoiding caffeine in the evenings, he did not notice much improvement. Upon conducting a
prostate examination, the physician noticed a palpable swelling with a smooth surface. The texture felt
firm, non-tender, and rubbery. The doctor suspects benign prostate hypertrophy (BPH). Which of the
following features is not typically seen in BPH?
A. Increased trabeculations in bladder
B. J shaped/ fish hook proximal ureter
C. J shaped/ fish hook distal ureters
D. Bilateral hydronephrosis
----------------------------------------
71. What is the likely diagnosis for an elderly male patient who has come to the hospital with lower back
pain, a movable mass in the lower back that becomes more prominent when breathing in, and a
general feeling of weakness? The lab tests show the presence of red blood cells in the urine, with no
sign of pus cells.
A. Polycystic kidney disease
B. Hydronephrosis
C. Renal cell carcinoma
D. Chronic pyelonephritis
----------------------------------------
72. What is the likely diagnosis for a 60-year-old male who is experiencing lower back pain and has
sclerotic lesions in the vertebrae?
A. Carcinoma prostate
B. Tuberculosis
C. Renal cell carcinoma
D. Carcinoma colon
----------------------------------------
73. In a patient with a history of trauma, if the bulbar urethra is ruptured, where does urine accumulate?
A. Superficial pouch
B. Deep pouch Rectouterine pouch Uterovesical pouch
C. Rectouterine pouch
D. Uterovesical pouch
----------------------------------------
74. In a new born baby boy with an empty scrotum , testis was found to be lying close to the
peritoneum. Which of the following helps the testis to reach the scrotum?
(or)
In a newborn male with an undeveloped scrotum, the testis was observed to be located near the
peritoneum. Which of the following factors aids in the descent of the testis into the scrotum?
A. GnRH Analogues
B. Testosterone

Page 21

1189
C. Cold Compressions
D. Manually bring testis down to the bottom of scrotum
----------------------------------------
75. What is the most probable diagnosis for a 35-year-old man who presented with enlarged scrotum,
where LDH levels were notably elevated, while HCG and alpha fetoprotein levels were within normal
range, as indicated by the histopathology report?

A. Teratoma of testis
B. Seminoma of testid
C. Choriocarcinoma
D. Yolk sac tumor
----------------------------------------
76. What is the most frequently observed side effect following the removal of bilateral L1 ganglions
during lumbar sympathectomy in a young male?
A. Impotence
B. Retrograde ejaculation
C. Bladder dysfunction
D. Incontinence
----------------------------------------
77. What is the probable diagnosis for a 27-year-old male patient who presented at the clinic with
symptoms of headache, palpitations, and sweating? Subsequently, an abdominal ultrasound was
conducted due to a family history of renal issues in his brother and father, which revealed the existence
of multiple renal and pancreatic cysts. Additionally, his 24-hour urinary metanephrine levels were found
to be elevated.
A. Multiple endocrine neoplasia type
B. Von Hippel-Lindau disease
C. Birt-Hogg-Dube syndrome
D. Tuberous sclerosis
----------------------------------------
78. What is the most appropriate treatment for a 40-year-old man experiencing scrotal heaviness, with
the presence of a bag of worms observed during scrotal examination, and reduction in swelling

Page 22

1190
observed in a supine position?
A. Suction drainage
B. Varicocelectomy
C. Jaboulay procedure
D. Herniotomy
----------------------------------------
79. A patient presents with Renal calculus of > 4 cm of the staghorn type. What is the best
management?
A. ESWL
B. Percutaneous Nephrolithotomy
C. Ureteroscopic removal
D. Retro renal surgery
----------------------------------------
80. A child presents with pain and swelling in the right side of his scrotum and testicular torsion is
suspected. Which is the most appropriate management?
A. Immediate exploration on both symptomatic and asymptomatic side
B. Immediate exploration on symptomatic side only
C. Delayed exploration on both symptomatic and asymptomatic side
D. Delayed exploration on symptomatic side only
----------------------------------------
81. What should be the subsequent course of action after observing blind-ending testicular vessels on
the right side during a diagnostic laparoscopy in a child with a unilateral undescended testis?
A. Inguinal exploration
B. Abdominal exploration
C. Scrotal exporation Nothing to be done
D. Nothing to be done
----------------------------------------
82. In the case of a 60-year-old patient who has undergone TURP, if the patient experiences changes
in sensitivity and becomes drowsy after 3 days, the most likely diagnosis would be:
A. Hypernatremia
B. Hyponatremia
C. Stroke
D. Meningitis due to spinal anesthesia
----------------------------------------
83. What is the most probable diagnosis for a female patient who presents with persistent urinary
dribbling and has experienced bladder fullness that she has been voiding since birth?
A. Stress incontinence

Page 23

1191
B. Ureterocele
C. Ectopic Ureter
D. Vesico vaginal fistula
----------------------------------------
84. What is the clinical diagnosis of a 65-year-old male who is experiencing low backache, weight loss,
progressive difficulty in urination, and had an episode of urinary retention that was relieved by
catheterization? The patient's PSA level is 5.2 ng/ml, and an x-ray of the lumbar sacral spine revealed
osteoblastic changes.
A. Benign prostatic hypertrophy
B. Carcinoma bladder
C. Prostate cancer
D. Renal cell cancer
----------------------------------------
85. What should be the initial course of action for a patient who has presented with a pelvic fracture and
a rupture in the membranous urethra?
A. A suprapubic catheter should be inserted as soon as possible.
B. IVP should be done
C. Immediate urethroplasty
D. MCU should be done
----------------------------------------
86. What is the cause of the appearance observed in intravenous pyelography (IVP)?

A. Horseshoe kidney
B. Ureterocele
C. Bladder tumor
D. Posterior urethral valve
----------------------------------------
87. In a 30-year-old male who presented with abdominal pain and is currently undergoing evaluation,
the following observation is noted in his contrast-enhanced computed tomography (CECT) scan. Can
you identify the probable medical condition indicated by the arrow in the image?

Page 24

1192
A. Appendicitis
B. Horseshoe kidney
C. Wilms tumor
D. Abdominal aortic aneurysm
----------------------------------------
88. What is the likely diagnosis for a patient who visited the outpatient department (OPD) experiencing
intense pain during erection and a deviated penis for the past half-year?
A. Peyronie’s disease
B. Priapism
C. Marion’s disease
D. Erectile dysfunction
----------------------------------------
89. Which of the following is not a characteristic of polycystic kidney disease?
A. Hematuria
B. Hypertension
C. Renal failure
D. Erythrocytosis
----------------------------------------
90. What is the diagnosis based on the x-ray given below?

Page 25

1193
A. Renal calculi
B. Bladder calculi
C. Schistosomiasis
D. Ureteric calculi
----------------------------------------
91. The type of renal calculi which is resistant to Extracorporeal Shock-Wave Lithotripsy (ESWL) is?
A. Struvite
B. Cystine
C. Calcium oxalate dihydrate
D. Uric Acid
----------------------------------------
92. A 22-year-old male presented with a hard painless testicular swelling. There were inguinal,
intra-abdominal, or mediastinal lymph nodes on CT Scan. The blood investigations showed AFP levels
of > 3000. USG shows uniform echotexture and small areas of necrosis. The next line of management
is____.
A. High inguinal orchidectomy
B. Core needle tract biopsy
C. FNAC
D. Whole body PET/CT
----------------------------------------
93. What is the most appropriate approach for treating a 3 cm stone located in the renal pelvis when
there is no evidence of hydronephrosis?
A. ESWL
B. PCNL
C. Antegrae pyeloplasty
D. Retrograde pyeloplasty
----------------------------------------
94. Germ cell tumour not seen in males:
A. Choriocarcinoma
B. Seminoma
C. Sertoli cell tumour
D. Teratoma
----------------------------------------
95. A 40-year-old male presented to the outpatient department (OPD) with concerns regarding painful
erection in the absence of sexual desire. The patient also reported a history of using antipsychotic
medication. What is the defined timeframe for diagnosing Priapism?
A. 1 hour

Page 26

1194
B. 2 hours
C. 3 hours
D. 4 hours
----------------------------------------
96. Bell clapper testis predisposes to:
A. Torsion testis
B. Varicocele
C. Cancer of testis
D. Hydrocele
----------------------------------------
97. A 55-year-old male presented with verrucous carcinoma around the glans of the penis. Examination
reveals that the inguinal lymph nodes are not enlarged. What is the appropriate management for this
patient?
A. Total penectomy
B. CO2 laser excision
C. Topical 5-fluorouracil
D. Partial penectomy
----------------------------------------

Correct Answers
Question Correct Answer

Question 1 1
Question 2 1
Question 3 2
Question 4 1
Question 5 4
Question 6 2
Question 7 4
Question 8 3
Question 9 3
Question 10 4
Question 11 1
Question 12 2
Question 13 1
Question 14 1
Question 15 4

Page 27

1195
Question 16 1
Question 17 1
Question 18 3
Question 19 1
Question 20 1
Question 21 2
Question 22 4
Question 23 2
Question 24 2
Question 25 4
Question 26 3
Question 27 1
Question 28 4
Question 29 2
Question 30 4
Question 31 2
Question 32 1
Question 33 2
Question 34 3
Question 35 4
Question 36 3
Question 37 2
Question 38 1
Question 39 2
Question 40 1
Question 41 2
Question 42 4
Question 43 1
Question 44 4
Question 45 1
Question 46 1
Question 47 3
Question 48 1
Question 49 2
Question 50 1

Page 28

1196
Question 51 4
Question 52 3
Question 53 4
Question 54 1
Question 55 4
Question 56 3
Question 57 1
Question 58 1
Question 59 1
Question 60 2
Question 61 1
Question 62 4
Question 63 3
Question 64 3
Question 65 3
Question 66 2
Question 67 2
Question 68 1
Question 69 4
Question 70 1
Question 71 3
Question 72 1
Question 73 1
Question 74 1
Question 75 2
Question 76 2
Question 77 2
Question 78 2
Question 79 2
Question 80 1
Question 81 4
Question 82 2
Question 83 3
Question 84 3
Question 85 1

Page 29

1197
Question 86 2
Question 87 2
Question 88 1
Question 89 4
Question 90 2
Question 91 2
Question 92 1
Question 93 2
Question 94 3
Question 95 4
Question 96 1
Question 97 4

Solution for Question 1:


Correct option A
• Vesicoureteric reflux (VUR) is when urine flows backward from the bladder into the ureters or
kidneys.
• It is a common cause of recurrent urinary tract infections in children. The imaging shown in the
question is a voiding cystourethrogram (VCUG), which is a test that helps to diagnose VUR. In this test,
a catheter is placed in the bladder, and a special dye is inserted through the catheter. X-ray images are
taken while the child urinates, showing whether the dye flows backward into the ureters or kidneys.
Incorrect options:
Option B. Urinary bladder diverticulum is an outpouring of the bladder wall, which can sometimes be se
en on imaging but is not typically associated with recurrent UTIs.
Option C. Urinary bladder hernia: It is a rare condition in which the bladder protrudes through a
weakness in the abdominal wall and, again, would not typically be associated with recurrent UTIs.
Option D. Vesicocolic fistula is a connection between the bladder and the colon, which can cause stool
to pass through the bladder. This would not be consistent with the provided image or the patient's histo
ry of recurrent UTIs.

Solution for Question 2:


Correct option A:
• The image shows a delay in contrast dye uptake by the right kidney, which suggests an obstruction at
the pelvic ureteric junction. At this point, the renal pelvis narrows to become the ureter. This condition
can cause renal colic and hydronephrosis, dilation of the renal pelvis, and calyces due to the back-up of
urine. Various factors, including congenital abnormalities, scarring, and tumors, can cause PUJ
obstruction.
Incorrect options:

Page 30

1198
Option B. Putty kidney - "Putty kidney" is not a medical term nor a recognized diagnosis.
Option C. Staghorn calculus - A staghorn calculus is a large kidney stone that fills the renal pelvis and
calyces, causing obstruction and potentially leading to hydronephrosis. However, the image provided d
oes not show evidence of a staghorn calculus.
Option D. Cystic kidney - A cystic kidney is a condition in which the kidneys contain numerous fluid-fille
d sacs or cysts. This can cause enlargement of the kidneys and impair their function, but it does not ca
use obstruction or delay of contrast dye uptake, as seen in the image provided.

Solution for Question 3:


Correct option B:
• Autosomal dominant polycystic kidney disease: This genetic disorder is characterized by the
formation of multiple cysts in the kidneys, leading to kidney enlargement and dysfunction. It commonly
presents with hematuria, flank pain, and hypertension, all present in this patient.
Incorrect options:
Option A. Renal cell carcinoma: Renal cell carcinoma may present with hematuria and flank pain. How
ever, it usually appears as a
solid mass on imaging studies rather than multiple cysts, as the ultrasound image provided shows.
Option C. Tuberculosis of the kidney: While tuberculosis can affect the kidneys and is present with hem
aturia and loin pain, it would not be typically present with multiple cysts in the kidneys as seen on the ul
trasound image.
Option D. Autosomal recessive polycystic kidney disease: Autosomal recessive polycystic kidney disea
se is a rare genetic disorder that presents in infancy or childhood and leads to the formation of numero
us small cysts in the kidneys. It is unlikely to be the diagnosis in this case as the patient is 40 years old
and has a family history suggestive of ADPKD.

Solution for Question 4:


Correct option A:
• The prostatic venous plexus is a network of veins surrounding the prostate gland, communicating with
the vertebral venous plexus. Carcinoma of the prostate can spread to the lumbar vertebra through the
prostatic venous plexus. Options B, C, and D are incorrect as they do not involve the prostatic venous
plexus. Transcoelomic spread is the spread of cancer from one organ to another via the peritoneal
cavity. The inferior vesical vein and the internal iliac vein drain into the common iliac vein and not into
the vertebral venous plexus
Incorrect options:
Option B.
Transcoelomic spread refers to the spread of cancer through the peritoneal cavity, which is not a
common route for prostate cancer.
Option C. The inferior vesical vein drains into the internal iliac vein. It is involved in spreading bladder c
ancer to the prostate gland, but it is not a common route to spread to the lumbar vertebrae.

Page 31

1199
Option D. The internal iliac vein drains into the common iliac vein and is not directly involved in spreadi
ng prostate cancer to the lumbar vertebrae.

Solution for Question 5:


Correct option D:
• The given image and symptoms are confirmatory for Fournier’s gangrene
• Fournier’s gangrene is a rare but serious infection of the scrotum and genital area that affects the
skin, muscles, and underlying tissue.
• It is a polymicrobial infection (I.e. Both aerobe &Anaerobes; are responsible)
• Single most common organism is Group A B hemolytic streptococci
• Symptoms may include severe pain, swelling, redness, and fever and can progress quickly to
necrosis (death of tissue). Treatment typically involves surgical removal of the affected tissue and
antibiotics.
Incorrect options:
Option A. Torsion of the testis is a medical emergency when the spermatic cord twists and blocks bloo
d flow, leading to severe scrotal pain, swelling, and tenderness. If not promptly diagnosed and treated,
it can permanently damage the testis and cause infertility.
Option B. Acute epididymal-orchitis is an inflammation or infection of the epididymis (a coiled tube behi
nd the testis) and/or the testis itself. It typically presents with sudden onset of pain, swelling, and redne
ss of the scrotum, along with fever and other symptoms. Bacterial or viral infections can cause it and m
ay require antibiotics or other treatments.
Option C. Scrotal carcinoma is a
rare type of skin cancer that can occur on the scrotum or other genital areas. It usually presents as a p
ainless lump or lesion that grows over time and can become ulcerated. Treatment typically involves sur
gery to remove the tumor, along with other therapies.

Solution for Question 6:


Correct Option B:
• The most likely diagnosis for this presentation is Varicocele.
• A varicocele is an enlargement of the veins within the scrotum, similar to varicose veins that occur in
the legs. It is a common cause of male infertility.
• The typical presentation of a varicocele is an asymptomatic painless swelling of the scrotum, which
may be described as a "bag of worms" sensation on palpation. The swelling is typically more prominent
when the patient is standing and may decrease when the patient is lying down.
Incorrect Options:
Option A: Hydrocele is a collection of fluid within the tunica vaginalis (the membrane that covers the tes
tis) that leads to scrotal swelling. It is typically painless and feels like a smooth, round mass on palpatio
n. Unlike varicocele, hydrocele is not typically associated with male infertility.

Page 32

1200
Option C: Testicular torsion is a medical emergency that occurs when the spermatic cord (the bundle o
f blood vessels and nerves that supports the testis) becomes twisted, cutting off the blood supply to the
testis. If not treated promptly, this can lead to severe pain, swelling, and a
high risk of testicular loss. Unlike varicocele, testicular torsion is typically associated with severe pain.

Option D: Epididymo-orchitis is an inflammation of the epididymis (a coiled tube located at the back of t
he testis that stores and carries sperm) and/or the testis. It is typically associated with pain, swelling, a
nd tenderness of the scrotum, as well as fever and other signs of infection. Unlike varicocele, epididym
o-orchitis is typically associated with pain and tenderness.

Solution for Question 7:


• Benign prostatic hyperplasia (BPH) is a noncancerous enlargement of the prostate gland that occurs
commonly in men as they age. BPH can cause a range of lower urinary tract symptoms, including both

Page 33

1201
voiding and storage symptoms. Storage symptoms refer to the symptoms related to the bladder's
storage capacity and include the following: Urgency: A sudden and strong urge to urinate that may be
difficult to suppress. Enuresis: Involuntary leakage of urine that occurs during sleep. Frequency: The
need to urinate frequently, often more than 8 times a day. Nocturia - is the complaint that the individual
needs to wake at night at least once to void. Urge incontinence (UI) is involuntary urinary leakage, often
large volume, immediately preceded by the sensation of urgency. Urgency and episodes of urge
incontinence are often associated with an overactive bladder or bladder neuropathy.
• Urgency: A sudden and strong urge to urinate that may be difficult to suppress.
• Enuresis: Involuntary leakage of urine that occurs during sleep.
• Frequency: The need to urinate frequently, often more than 8 times a day.
• Nocturia - is the complaint that the individual needs to wake at night at least once to void.
• Urge incontinence (UI) is involuntary urinary leakage, often large volume, immediately preceded by
the sensation of urgency. Urgency and episodes of urge incontinence are often associated with an
overactive bladder or bladder neuropathy.
• Voiding symptoms refer to the symptoms related to the emptying of bladder and include the following:
Hesitancy: Difficulty initiating urination, often characterized by a delay in the start of the urinary stream
Dribbling: A continued leakage of urine after the urinary stream has stopped. Intermittency is the term
used when the individual describes urine flow which stops and starts, on one or more occasions, during
micturition. Poor flow Incomplete emptying is the sensation that at the end of micturition bladder
fullness persists.
• Hesitancy: Difficulty initiating urination, often characterized by a delay in the start of the urinary stream
• Dribbling: A continued leakage of urine after the urinary stream has stopped.
• Intermittency is the term used when the individual describes urine flow which stops and starts, on one
or more occasions, during micturition.
• Poor flow
• Incomplete emptying is the sensation that at the end of micturition bladder fullness persists.
• Therefore, the correct answer is option d, as it includes all the storage symptoms of BPH.
• Urgency: A sudden and strong urge to urinate that may be difficult to suppress.
• Enuresis: Involuntary leakage of urine that occurs during sleep.
• Frequency: The need to urinate frequently, often more than 8 times a day.
• Nocturia - is the complaint that the individual needs to wake at night at least once to void.
• Urge incontinence (UI) is involuntary urinary leakage, often large volume, immediately preceded by
the sensation of urgency. Urgency and episodes of urge incontinence are often associated with an
overactive bladder or bladder neuropathy.
• Hesitancy: Difficulty initiating urination, often characterized by a delay in the start of the urinary stream
• Dribbling: A continued leakage of urine after the urinary stream has stopped.
• Intermittency is the term used when the individual describes urine flow which stops and starts, on one
or more occasions, during micturition.
• Poor flow
• Incomplete emptying is the sensation that at the end of micturition bladder fullness persists.
Incorrect Choices:

Page 34

1202
• Option a: Urgency, hesitancy, and frequency are incorrect.
• Option b: Enuresis and dribbling are incorrect.
• Option c: All except dribbling is incorrect, as dribbling is not a storage symptom but a voiding
symptom.

Solution for Question 8:


• The diagnosis of this 10-month-old male baby is that he has one undescended testis on the right side
and one ectopic testis on the left side. An ectopic testis is a testis that has not followed the normal
pathway of descent and is found outside of the scrotum.
• During normal fetal development, the testes form in the abdomen and then descend into the scrotum
through the inguinal canal. This process should be complete by the time a male baby is born. However,
in some cases, one or both testes fail to descend into the scrotum and remain in the abdomen or along
the pathway of descent. This condition is known as undescended testis or cryptorchidism.
• In the case of this baby, the right testis has not descended into the scrotum and is located in the right
inguinal canal. This is classified as an undescended testis. The left testis, on the other hand, has not
only failed to descend into the scrotum but has also followed an abnormal pathway and is located in the
perineum. This is classified as an ectopic testis.
Incorrect Choices:
• Option a: Bilateral undescended testis refers to the situation where both testes have failed to descend
into the scrotum.
• Option b: Bilateral ectopic testis refers to the situation where both testes have followed abnormal
pathways and are located outside of the scrotum.
• Option c: Right side undescended testis and left-sided ectopic testis would be an incorrect diagnosis
for this baby since only one testis is undescended, and the other one is ectopic.

Solution for Question 9:


Answer - C Bladder tuberculosis
• The most probable diagnosis based on the given history, cystoscopic findings, and KUB x-ray is
"Bladder tuberculosis."
• Bladder tuberculosis is a chronic bacterial infection caused by Mycobacterium tuberculosis that affects
the urinary bladder.
• The most common symptoms of bladder tuberculosis are increased frequency of micturition, dysuria,
and hematuria. In advanced cases, patients may also experience suprapubic pain, urinary
incontinence, and bladder dysfunction.
• The pale ureteric orifice seen on cystoscopy is a typical finding in patients with bladder tuberculosis,
indicating that the mucosa around the orifice is inflamed and edematous. The KUB x-ray may show
calcification in the bladder wall, which is a characteristic finding in bladder tuberculosis.
Incorrect Choices:

Page 35

1203
• Option a. Bladder carcinoma is a malignant tumor that arises from the epithelial cells lining the urinary
bladder. It can also present with similar symptoms, such as hematuria and urinary frequency. However,
the pale ureteric orifice seen on cystoscopy is not a typical finding in these conditions.
• Option b. Papilloma of the urinary bladder is a benign tumor that arises from the urothelium lining the
bladder. It can also present with similar symptoms, such as hematuria and urinary frequency. However,
the pale ureteric orifice seen on cystoscopy is not a typical finding in these conditions.
• Option c. Schistosomiasis is a parasitic infection caused by Schistosoma haematobium that can affect
the urinary tract and cause bladder inflammation and fibrosis. It can also present with similar
symptoms, such as hematuria and urinary frequency. However, the pale ureteric orifice seen on
cystoscopy is not a typical finding in these conditions.

Solution for Question 10:


Option A:
• The most likely diagnosis in this case is prostate cancer. Prostate cancer typically presents with
symptoms such as urinary frequency, urgency, and difficulty urinating. However, in advanced cases, it
can cause hip and back pain, weight loss, and loss of appetite.
• Elevated levels of PSA (prostate-specific antigen) in the blood are also a common finding in prostate
cancer. Therefore, given the presentation of worsening hip and back pain, weight loss, loss of appetite,
and elevated PSA levels, prostate cancer is the most likely diagnosis in this case.
Incorrect Choices:
• Option a. Ankylosing spondylitis is a chronic inflammatory condition that affects the joints of the spine
and the sacroiliac joint. It typically presents with back pain and stiffness, especially in the morning or
after prolonged periods of inactivity. However, ankylosing spondylitis does not typically cause weight
loss or elevated PSA levels, so it is an unlikely diagnosis in this case.
• Option b. Osteosarcoma is a type of bone cancer that typically affects children and young adults. It
can occur in any bone, but it most commonly affects the long bones of the arms and legs.
Osteosarcoma typically presents with bone pain, swelling, and tenderness, but it is unlikely to cause
weight loss or elevated PSA levels. Therefore, osteosarcoma is an unlikely diagnosis in this case.
• Option c. Osteoclastoma, also known as giant cell tumor of bone, is a rare type of bone tumor that
typically affects the long bones of the arms and legs. It can cause bone pain, swelling, and tenderness,
but it is unlikely to cause weight loss or elevated PSA levels. Therefore, osteoclastoma is an unlikely
diagnosis in this case.

Solution for Question 11:


• The given symptoms of palpitations, episodic headache, sweating episodes, and a small left-sided
suprarenal mass suggest a possible diagnosis of pheochromocytoma.
• Pheochromocytoma is a rare tumor of the adrenal gland tumor that produces excessive amounts of
catecholamines (epinephrine, norepinephrine, and dopamine), which can cause symptoms such as
hypertension, headache, and sweating.
• Among the various blood and urine tests, measuring urinary vanillyl mandelic acid (VMA) levels is a
commonly used and highly sensitive test for pheochromocytoma.

Page 36

1204
Incorrect Choices:
• b. 5-HIAA (5-5-hydroxy indole acetic acid) is a breakdown product of serotonin produced by carcinoid
tumors. While carcinoid tumors can also produce catecholamines, they are much less common than
pheochromocytomas. Thus, measuring blood 5-HIAA levels is not useful for confirming the diagnosis of
pheochromocytoma.
• c. Myoglobin is a protein found in muscle tissue, and its presence in urine can indicate muscle
damage or injury. However, urine myoglobin levels are not specific for pheochromocytoma and do not
help confirm the diagnosis.
• d. Stool trypsin levels can be used to diagnose pancreatic insufficiency, a condition where the
pancreas does not produce enough enzymes to digest food properly. However, this test is not relevant
for diagnosing pheochromocytoma.

Solution for Question 12:


• Gemcitabine is a chemotherapy drug commonly used to treat bladder cancer.
• If a patient receives chemotherapy and radiation therapy, the dose of gemcitabine may need to be
reduced to avoid increased toxicity.
• This is because radiation therapy can affect the body's ability to metabolize and eliminate the
chemotherapy drug.
• Radiation can increase the levels of gemcitabine in the body, leading to increased side effects such as
low white blood cell counts, nausea, and fatigue. Therefore, the dose of gemcitabine may need to be
reduced with radiation therapy.
Incorrect Choices:
• a: Cisplatin is another chemotherapy drug used to treat bladder cancer. While it can also cause
toxicity when given radiation therapy, the question specifically asks about the drug for which the dose
should be reduced, and the correct answer is gemcitabine.
• c: Paclitaxel is a chemotherapy drug used to treat various cancers but not commonly used to treat
bladder cancer. Additionally, no evidence suggests that the paclitaxel dose should be reduced with
radiation therapy.
• d: Ifosfamide is another chemotherapy drug used to treat bladder cancer. While it can cause toxicity
when given radiation therapy, the question specifically asks about the drug for which the dose should
be reduced, and the correct answer is gemcitabine.

Solution for Question 13:


• The Gleason score is a grading system used to assess the aggressiveness of prostate cancer.
• It is determined by examining prostate tissue samples obtained through a biopsy.
• The pathologist assigns a grade to the two most common patterns of cancer cells observed in the
tissue sample.
• The grades are added together to create a total Gleason score ranging from 6 to 10.
• Higher scores indicate a more aggressive cancer.

Page 37

1205
Incorrect Choices:
• Option b. The Bethesda score is a classification system for interpreting thyroid biopsies. It is used to
assess the likelihood of a thyroid nodule being malignant.
• Option c. The Alvarado score is a scoring system used to diagnose acute appendicitis. It combines
clinical symptoms and laboratory values to assess the patient’s likelihood of acute appendicitis.
• Optiond. The Nottingham score is a grading system used to assess the aggressiveness of breast
cancer. It is determined by examining breast tissue samples obtained through a biopsy.

Solution for Question 14:


• The Given clinical scenario, is highly suggestive of Renal cell carcinoma (RCC)
• In renal cell carcinoma in which the tumor is less than 4 cm in size the preferred treatment is the
Partial nephrectomy (Nephron sparing surgery).
Incorrect Choices:
Option b. Neoadjuvant chemotherapy is a cancer therapy in which chemotherapy drugs are conducted
on the tumor before its surgical extraction. The doctor may recommend neoadjuvant chemotherapy to
shrink the breast cancer tumor to provide more surgical options for care.
Option c. Radical nephrectomy is such type of surgery in which the entire kidney, along with the sectio
n of the ureter (bladder) tube, glands present on the kidney, i.e., adrenal gland, and fatty acids that surr
ound the kidney, are removed. Hence, this is the incorrect answer.
Option d. The radiotherapy (RT) applied on post-surgical resection decreases the risk of local reappear
ance.

Solution for Question 15:


Bilateral hydronephrosis refers to the enlargement or swelling of both kidneys due to the restriction of t
he passage of urine from both ureters.
Retrocaval or circuncaval ureters are rare congenital anomaly, in which the ureter is located posterior t
o the inferior vena cava (IVC). This anatomical abnormality can lead to the compression or obstruction
of the ureter, resulting in urine flow obstruction and hydronephrosis. However, the retrocaval ureter typi
cally affects only one side (unilateral) rather than causing bilateral hydronephrosis.
Incorrect Choices:
Option a. Phimosis is a condition in males where the foreskin of the penis is tight and cannot be retract
ed. Phimosis itself does not cause bilateral hydronephrosis. It is unrelated to the urinary system and w
ould not obstruct urine flow from both ureters.
Option b. Posterior urethral valves are abnormal folds or flaps in the urethra, obstructing urine flow out
of the bladder. This condition is typically seen in male infants. It commonly leads to significant urinary tr
act obstruction, resulting in bilateral hydronephrosis.
Option c. Urethral strictures are narrowings or obstructions in the urethra, which can occur due to infla
mmation, trauma, or previous surgeries. Urethral strictures can cause obstructine flow and result in bila
teral hydronephrosis if the strictures are located at or above the level of the bladder.

Page 38

1206
Solution for Question 16:
Option A: Staph. aureus is the causative organism
• The Given image is of Fournier's gangrene which is necrotizing fasciitis involving perineum region
• It is a polymicrobial infection (I.e. Both aerobic and anaerobes are responsible) hence the given
statement is incorrect.
Incorrect Choices:
Option B: It is common in diabetic patients.
• It is correct and also there is high mortality in diabetic patients.
Option C: High mortality
• High mortality rates are found in this case. Hence, daily rounds are needed in this case.
Option D: The testes remain unaffected
• The testis are preserved in this condition as it have the dual blood supply

Solution for Question 17:


Lymphangitis is the inflammation of the lymphatic vessels, specifically the superficial lymphatics. It is ty
pically caused by an infection that enters the body through a break in the skin, such as a wound or an i
nsect bite. The infection can then spread to the lymphatic vessels, causing them to become inflamed a
nd visible as red streaks on the skin. Superficial lymphatics are a
network of lymphatic vessels that are close to the surface of the skin. They play a crucial role in carryin
g lymph fluid and immune cells from the tissues towards the lymph nodes for filtration and processing.
When an infection occurs in the surrounding tissues, the microorganisms can enter the lymphatic vess
els through the disrupted skin, leading to lymphangitis.
Incorrect options explanations:
Option b. Deep lymphatics: Deep lymphatics are located deeper within the body and are responsible fo
r draining lymph fluid from the organs and deeper tissues. While deep lymphatics play a crucial role in t
he lymphatic system, they are not directly involved in lymphangitis. Lymphangitis primarily affects the s
uperficial lymphatics, which are the lymphatic vessels closer to the skin's surface.
Option c. Superficial veins: Superficial veins are responsible for carrying deoxygenated blood back to t
he heart and are not directly involved in the lymphatic system. They do not play a
role in lymphangitis, which is an inflammation of the lymphatic vessels rather than the veins.
Option d. Skin: While the skin can be affected by lymphangitis, it is not the structure directly responsibl
e for causing lymphangitis. Lymphangitis occurs when an infection enters the body through a break in t
he skin and then spreads to the superficial lymphatic vessels, leading to their inflammation. So, while t
he skin can be a site of entry for the infection, it is the involvement of the superficial lymphatics that lea
ds to lymphangitis.

Page 39

1207
Solution for Question 18:
A radioiodine scan (I-131 scan) may be recommended in the given case of a
50-year-old woman after thyroidectomy for papillary thyroid cancer for several reasons: 1.
Detection of residual thyroid tissue: The primary goal of a radioiodine scan is to assess the presence o
f any residual thyroid tissue, including any remaining cancer cells, after thyroidectomy. The scan uses
radioactive iodine (I-131), which is taken up by thyroid tissue. By administering this radioactive iodine, t
he scan can identify any remaining thyroid tissue, helping to determine if further treatment is necessary
. 2. Evaluation of the extent of disease:. It can detect the presence of any metastases (spread) of the
cancer to other parts of the body, such as lymph nodes or distant organs. This information is crucial for
determining the appropriate treatment approach and to plan further management. 3.
Planning subsequent treatment strategies Incorrect Choices: Option a. Radiotherapy may be used as
an adjuvant therapy in some cases, it is generally not the primary treatment for papillary thyroid cancer.
Radiotherapy is more commonly considered when there is evidence of cancer spread beyond the thyr
oid bed or a high risk of cancer recurrence. Option b. Chemotherapy: papillary thyroid cancer is genera
lly less responsive to chemotherapy compared to other types of cancer. Chemotherapy is typically not t
he first-line treatment for papillary thyroid cancer. It may be considered in cases where the cancer has
spread to distant organs or if other treatments have not been effective. Option d. The "wait and watch"
approach, also known as active surveillance or observation, may be appropriate in certain cases where
the papillary thyroid cancer is low-risk and has been completely removed during surgery with no evide
nce of residual cancer cells. This approach involves regular monitoring through blood tests, ultrasound,
and periodic follow-up visits to ensure there is no recurrence or spread of the cancer.

Solution for Question 19:


Answer Option A- Increased uptake due to diffuse metastasis
• The reason for the super scan appearance described, with increased uptake in the bones and
reduced activity in the spleen, is typically due to option a: increased uptake due to diffuse metastasis.
• A super scan is a term used to describe an intense and diffuse uptake of a radiopharmaceutical in the
bones on a bone scan.
• It is commonly associated with extensive metastatic disease, such as in advanced prostate cancer.
• In this scenario, the increased uptake in the bones indicates widespread metastasis of the prostate
cancer to the skeletal system.
• Prostate cancer commonly spreads to the bones, and the increased uptake on the bone scan
suggests the presence of multiple bone lesions throughout the body.
Incorrect option explanations
Option b Increased uptake by carcinoma prostate- is not the correct answer because the bone scan is
used to detect metastatic involvement in the bones, not the primary tumor.
Option c Increased uptake by the bone- is not the correct answer because while there is increased upt
ake in the bones, it is due to the metastatic spread of prostate cancer rather than increased physiologic
al bone uptake.
Option d Decreased uptake by adrenal glands and kidney- is not the correct answer because the bone
scan does not typically assess adrenal gland or kidney function. It is primarily used to evaluate bone he
alth and detect metastatic involvement.

Page 40

1208
Solution for Question 20:
In this staging, "CT3" indicates that the tumor size is larger than 5 cm and cT4 involves the chest wall o
r skin. "N1" indicates the presence of metastasis to ipsilateral axillary lymph nodes. "M0" indicates no d
istant metastasis.
Incorrect option explanations: Option b. ■T4N1M0: This option uses the "■" prefix, indicating clinical st
aging. However, the "■T4" suggests a tumor larger than 5 cm or involving the chest wall or skin, which
is not mentioned in the scenario. The correct size mentioned in the scenario is larger than 2
cm but not specified to be larger than 5 cm. Option c. CT4N1MO: This option suggests a
tumor size of T3, which typically means a tumor size between 2 cm and 5
cm. However, the scenario states that the mass is larger than 2 cm, so a
T3 classification would not be appropriate.
Option d. ■T3N1M0: This option also uses the "■" prefix for clinical staging. However, it suggests a T3
tumor, which, as mentioned earlier, would not be accurate based on the scenario. The correct size me
ntioned is larger than 2 cm, but the T3 classification represents a tumor size between 2 cm and 5 cm.

Solution for Question 21:


Answer Option B - CECT stands for Contrast-Enhanced Computed Tomography, which is a
medical imaging technique used to visualize the internal structures of the body. Horseshoe kidney is a
congenital condition in which the two kidneys are fused at the lower ends, forming a
U-shaped structure. In CECT imaging, horseshoe kidney appears as a characteristic "U" or horseshoe-
shaped mass with the renal pelvis and collecting system located between the two fused kidneys. CECT
can provide detailed information about the anatomy, blood supply, and any associated abnormalities o
r complications.
Incorrect option explanation:
Option a. Polycystic kidney disease: In CECT imaging, polycystic kidney disease can be visualized as
multiple cysts of varying sizes within the kidneys. These cysts are filled with fluid and can be seen as r
ound or oval structures in the renal parenchyma. CECT can help in assessing the number, size, and lo
cation of cysts, as well as any associated complications such as hemorrhage or infection.
Option c. Hydronephrosis: Hydronephrosis refers to the swelling or enlargement of the kidney due to th
e buildup of urine. It can occur as a result of various underlying causes, such as kidney stones, urinary
tract obstruction, or anatomical abnormalities. In CECT imaging, hydronephrosis appears as dilated re
nal pelvis and calyces, indicating the presence of urine accumulation. CECT can help identify the caus
e of hydronephrosis and assess the severity and extent of the condition.
Option d. Pancake kidney: Pancake kidney, also known as discoid kidney, is a
rare congenital anomaly characterized by a flattened disc-like shape of the kidney. It results from fusio
n and malrotation of the kidneys during fetal development. In CECT imaging, pancake kidney may app
ear as a flattened or disc-shaped mass, often lower in the abdomen compared to the normal kidney po
sition. CECT can provide detailed anatomical information about the pancake kidney and its relationship
with surrounding structures.

Page 41

1209
Solution for Question 22:
Option-3
Neuroblastoma is the most common intrabdominal malignancy.
Nephroblastoma is also known as Wilms tumor. Nephroblastoma is a
malignant or cancerous tumor that originates in the kidney cells. It is a very common type of renal canc
er, but has been reported in 6% of all cancers in children. Prognosis and survival term can vary from ch
ild to child greatly. However, most children are able to cure neuroblastoma.
Incorrect options explanation:
Option a. Treatment of newly diagnosed low-risk neuroblastoma may include the following: Surgery foll
owed by observation. Chemotherapy and surgery, for children with symptoms or children whose tumor
has continued to grow and are unable to remove by surgery. Chemotherapy, for certain patients.
Option b. Nephroblastoma or Wilms’ tumor (WT) is the most common renal malignant tumor of childho
od. Approximately 12% of WT are metastatic at presentation, with 80% having pulmonary metastases.

Option c. Hematogenous spread is typical route of metastasis for sarcomas, but it is also the favored ro
ute for certain types of carcinomas, such as renal cell carcinoma originating in the kidney and follicular
carcinomas of the thyroid.

Solution for Question 23:


Correct option B - Tamsulosin:
• Tamsulosin is used to treat the symptoms of an enlarged prostate (benign prostatic hyperplasia or
BPH) in which which include difficulty urinating (hesitation, dribbling, weak stream, and incomplete
bladder emptying), painful urination, and urinary frequency and urgency.
Incorrect options:
Option A - TURP (Transurethral resection of prostate):
• TURP is a surgical procedure performed to treat moderate to severe urinary symptoms caused by
BPH.
• TURP is not typically used in the initial stage of BPH when the symptoms are mild. It is usually
considered when conservative measures and medication have not provided adequate relief or when
there are complications associated with BPH.
Option C - Finasteride:
• Finasteride works by inhibiting the conversion of testosterone to dihydrotestosterone (DHT).
• While finasteride is effective in managing BPH symptoms and can provide long-term benefits, it is not
typically the initial choice for managing urinary symptoms in the early stages of BPH.
• It may be considered for patients with larger prostate volumes or those at a higher risk of disease
progression.
Option D - Observation:
• Observation may be an option for patients with mild urinary symptoms that do not significantly affect
their quality of life.

Page 42

1210
• In such cases, regular monitoring, lifestyle modifications (such as limiting fluid intake before bedtime),
and periodic follow-up with a healthcare provider may be recommended.
• However, if the symptoms worsen or become bothersome, medical intervention with medication, such
as tamsulosin, is usually recommended.
• Observation alone is not considered the standard initial management strategy for urinary symptoms
caused by BPH.

Solution for Question 24:


Answer Option B - For stones located in the ureters near the bladder, the most commonly used treatm
ent option is ureteroscopy. Ureteroscopy is a minimally invasive procedure in which a
thin, flexible tube called a ureteroscope is inserted into the ureter through the urethra and bladder. The
ureteroscope allows the urologist to visualize the stone and perform various interventions to address it.

Incorrect Choices: Option a. Percutaneous Nephrolithotomy (PCNL): This is a


surgical procedure used to remove large or complex kidney stones. It involves making a
small incision in the back and inserting a nephroscope (a thin tube with a camera) into the kidney to loc
ate and remove the stones. PCNL is typically recommended for stones that are larger than 2
centimeters, or when other treatments have been unsuccessful.
Option c. Extracorporeal Shock Wave Lithotripsy (ESWL): ESWL is a procedure that uses shock wave
s to break kidney stones into smaller pieces, making them easier to pass through the urinary tract. It is
a non-invasive procedure. The procedure is performed externally, meaning the shock waves are gener
ated outside the body and focused on the stone using imaging techniques. ESWL is generally used for
smaller stones located in the kidney or upper ureter. Option d. Wait and Watch: In some cases, particul
arly for small kidney stones that are not causing significant symptoms or complications, a "wait and wat
ch" approach may be adopted. This means that the patient will be monitored for any changes in the sto
ne size or symptoms over time. If the stone does not cause any problems or becomes small enough to
pass naturally, no further intervention may be needed. However, if the stone grows or causes symptom
s, other treatment options may be considered.

Solution for Question 25:


Correct Option D - Mixed aerobic and anaerobic infection:
• The patient's symptoms, along with the presented clinical picture, suggest a diagnosis of Fournier's
gangrene, a severe and potentially life-threatening necrotizing fasciitis involving the skin, superficial
fascia, and deep fascia of the perineal region. The structures spared are corpora cavernosa, corpora
Spongiosa, urethra, testis, and cord structures.
• This condition is characterized by a mixed infection caused by both aerobic and anaerobic bacteria.
Escherichia coli, Proteus, Staphylococcus, Klebsiella, and Streptococcus are the most prevalent
aerobic microorganisms. Bacteroides, Clostridium, and Peptostreptococcus are the three most
prevalent anaerobic microorganisms.
• Management IV fluids + IV antibiotics. Extensive surgical debridement. After extensive debridement,
skin grafting is done over the testis.
• IV fluids + IV antibiotics.

Page 43

1211
• Extensive surgical debridement.
• After extensive debridement, skin grafting is done over the testis.
• IV fluids + IV antibiotics.
• Extensive surgical debridement.
• After extensive debridement, skin grafting is done over the testis.
Incorrect Options:
Options A, B, and C are incorrect. Refer to Option D for an explanation.

Solution for Question 26:


Correct Option C - Outer tube diameter:
• The number marked on the Foley catheter corresponds to the outer tube diameter. It indicates the
size or diameter of the catheter, typically measured in French (Fr) units. The French scale is a unit of
measurement commonly used for catheters, where 1 French (Fr) is equal to 0.33 mm in diameter.
Incorrect Options:
Option A - Inner lumen diameter: It is incorrect because the marked number does not correspond to th
e inner lumen diameter of the catheter. The inner lumen diameter is important for determining the flow r
ate and functionality of the catheter but is not indicated by the marked number.
Option B - Length of the tube: It is incorrect because the marked number does not represent the length
of the catheter tube. The length of the catheter can vary depending on the specific clinical situation an
d the intended use.
Option D - Bulb size: It is incorrect because the marked number does not correspond to the bulb size.
The bulb size refers to the size of the balloon at the tip of the catheter, which is inflated to secure the c
atheter in place within the bladder.

Solution for Question 27:


Correct Option A: Cremasteric veins
• Varicoceles, found in approximately 15–20% of adult males, predominantly affects the left side due to
venous anatomy. They are rare in boys, typically emerging during late childhood or adolescence. In
some instances, dilated vessels involve cremasteric veins rather than the pampiniform plexus. The
primary cause is often valve absence or incompetence in the proximal testicular vein. While most cases
are idiopathic, varicoceles can result from left testicular vein obstruction caused by renal tumors or
nephrectomy in later life.
• After varicocele surgery, venous drainage primarily occurs through the cremasteric veins. These small
veins originate from the pampiniform plexus, the network of veins surrounding the spermatic cord. They
receive blood from the testicles and scrotum.
• During varicocele surgery, the surgeon typically ligates or ties off the dilated internal cremasteric veins
within the spermatic cord to restore proper venous drainage. After the surgery, the blood is rerouted
through the remaining patent veins, including the external cremasteric veins.

Page 44

1212
Incorrect options:
Option B: Penile veins: Penile veins primarily drain blood from the penis and are not directly involved in
the venous drainage of the scrotum or the surgical correction of varicocele. They have a separate drai
nage pathway and are not significant in the context of post-varicocele surgery venous drainage.
Option C: Ectopic drainage in the iliac fossa: Ectopic drainage refers to the redirection of venous flow t
o an abnormal location. In the context of varicocele surgery, the aim is to restore proper venous draina
ge at the usual location, which involves redirecting blood flow through the remaining patent veins within
the spermatic cord. Ectopic drainage in the iliac fossa would be an abnormal scenario and is not the e
xpected route of venous drainage after varicocele surgery.
Option D: Testicular veins: While testicular veins are involved in the venous drainage of the testicles, v
aricocele surgery often aims to correct issues with the testicular veins, such as valve incompetence or
obstruction. Therefore, after surgery, the drainage route typically shifts away from the problematic testi
cular veins to alternative pathways like the cremasteric veins.

Solution for Question 28:


Correct Option D:
Active surveillance is the most appropriate management according to patient condition, as gleason sco
re is 6 .
Incorrect Options:
Option A: Radical prostatectomy Is not required in this condition as the score is low.
Option B: External beam radiation Is not required in this condition as the score is low.
Option C: Brachytherapy Is not required in this condition as the score is low.

Solution for Question 29:


Correct Option B:
• The Given specimen is of Bladder carcinoma.
• TCC > SCC > Adenocarcinoma
• The cheesy mass with hematogenous mass is finding of Bladder carcinoma
Incorrect Options:
Option A: Malakoplakia:
• Inflammatory disorder of bladder
• Also involve ureter and kidney
• Formation of nodules/plaques, composed of large histiocytes (Von Hansemann cells)
• Associated with laminar inclusion bodies – Michaelis-Guttman bodies
Pathophysiology

Page 45

1213
• Defective phago-lysosomal activity - Inadequate killing of bacteria by monocytes/macrophage.
Option C: Prostate carcinoma
• The given specimen is of Urinary bladder not prostate.
Option D: Schistosomiasis:
• Aka Bilharziasis
• Causative agent - S. haematobium
• Endemic in Middle east, Egypt, Africa MC in Males
• MC in Males
• Species of Schistosoma : S. Haematobium -Invades bladder
• MC in Males
Treatment
• Drug of choice – Praziquantel
Alternative drugs
• Metrifonate
• Oxamniquine

Solution for Question 30:


Correct Option D:
• The stone most likely to be found in the common bile duct after cholecystectomy for gallstones is a
retained stone.
• Retained stones are those that were not removed during the initial cholecystectomy procedure and
remain in the common bile duct. They can cause symptoms of obstructive jaundice by obstructing the
flow of bile from the liver to the small intestine.
• In this scenario, since the patient had a previous cholecystectomy for gallstones, it is more likely that
the stone in the common bile duct is a retained stone that was not removed during the initial surgery.
Incorrect Options:
Option A: Primary stones are formed within the bile ducts themselves, usually as a
result of an underlying condition such as primary sclerosing cholangitis or Caroli disease.
Option B: Secondary stones are stones that form in the gallbladder and subsequently migrate to the co
mmon bile duct.
Option C: Tertiary stones are formed within the bile ducts as a result of bacterial or parasitic infection.

Solution for Question 31:


Correct Option B:

Page 46

1214
The give image is of fournier gangrene which occurs due to mixed flora aerobic and anaerobic infection

Incorrect Options:
Option A. Patient must be started on anti-gas gangrene serum therapy :
This is an incorrect statement as it is not required in fournier gangrene.
Option C. Bilateral orchiectomy is generally performed : This is an incorrect statement as Bilateral orchi
ectomy is generally Is not required in this condition and testis are unaffected.
Option D . Compulsory urinary diversion is performed :
This is an incorrect statement as the urinary system is unaffected in this condition.

Solution for Question 32:


Correct Option A:
The give X ray is of Putty kidney
• The term "putty kidney" is a descriptive term and not a formal medical diagnosis. It highlights the
significant loss of renal parenchyma and the contracted appearance of the kidney on imaging.
Incorrect Options:
Option B Nephrocalcinosis: Nephrocalcinosis refers to the deposition of calcium salts, such as calcium
oxalate or calcium phosphate, in the renal parenchyma, including the renal tubules. Nephrocalcinosis r
efers to the deposition of calcium salts, such as calcium oxalate or calcium phosphate, in the renal par
enchyma, including the renal tubules. This condition can occur as a
result of various underlying causes and can lead to impaired kidney function.
Option C. Staghorn calculus: it is a large kidney stone that fills the renal pelvis and extends into the bra
nching calyces of the kidney.Staghorn calculi are composed of a mixture of minerals, such as calcium
oxalate, calcium phosphate, and magnesium ammonium phosphate (struvite).The formation of a stagh
orn calculus is often associated with recurrent urinary tract infections, particularly those caused by cert
ain bacteria that produce urease enzyme, such as Proteus and Klebsiella species.
Option D - Psoas calcification: refers to the abnormal deposition of calcium within the psoas muscle. T
he psoas muscle is a large muscle located in the lower back that plays a
role in hip flexion and stability of the lumbar spine.

Solution for Question 33:


Correct option:
Option B:
• Thimble bladder is a term used to describe a condition in which the bladder becomes small,
contracted, and thickened due to chronic inflammation and fibrosis.
• It is typically seen in chronic tuberculosis (TB) of the bladder.

Page 47

1215
• The chronic inflammation causes fibrosis and contraction of the bladder, resulting in its characteristic
appearance.

Incorrect options:
Options A. Acute TB: Acute TB refers to an active and progressive infection caused by Mycobacterium
tuberculosis. While TB can affect various organs, including the lungs, lymph nodes, and kidneys, it is le
ss commonly associated with acute bladder involvement. Therefore, acute TB is not typically associate
d with a thimble bladder.
Options C. Neurogenic bladder is defined as bladder dysfunction resulting from a central or peripheral
neurologic insult. Possible symptoms include urinary incontinence, urgency, nocturia, straining, incomp
lete voiding, and urinary retention.
Options D. Schistosomiasis: Schistosomiasis is a parasitic infection caused by Schistosoma species.
While schistosomiasis can affect the urinary system, causing inflammation and fibrosis, it does not com
monly result in the characteristic thimble bladder appearance. Schistosomiasis-associated bladder abn
ormalities usually manifest as thickening, calcifications, and bladder wall irregularities.

Solution for Question 34:


Correct option:
Option C: Genital swab for polymerase chain reaction (PCR) testing
Explanation of options:
Endometrial biopsy: While endometrial biopsy can be useful in diagnosing genital TB, it is an invasive p
rocedure and may not be the first-line investigation. Endometrial biopsy is typically considered when ot
her less invasive tests, such as PCR testing, have not provided a
conclusive diagnosis or when there is a strong clinical suspicion of genital TB.
Urine sample for acid-fast bacilli (AFB) culture: Urine sample for AFB culture is essential in diagnosing
genitourinary TB, but it may not be the early investigation of choice for genital TB. Genitourinary TB ca
n involve the kidneys, bladder, and ureters, leading to the presence of mycobacteria in the urine. Howe
ver, when specifically investigating for genital TB, other tests like PCR are more targeted and less inva
sive.
Genital swab for polymerase chain reaction (PCR) testing: This is the correct option. PCR testing is a s
ensitive and specific method to detect the presence of Mycobacterium tuberculosis DNA in genital sam
ples. PCR testing allows for the rapid identification of TB infection in the genital tract and can be an ear
ly investigation of choice. It is less invasive compared to procedures like endometrial biopsy and provid
es a targeted approach to diagnosing genital TB.
Chest X-ray: While a chest X-ray is useful for diagnosing pulmonary TB (TB affecting the lungs), it may
not be the primary investigation for genital TB. Genital TB is a specific form of extrapulmonary TB, and
although pulmonary TB can be associated with genital TB, direct investigation of the genital tract is ne
eded for a definitive diagnosis.
• Endometrial biopsy involves obtaining a tissue sample from the lining of the uterus (endometrium) for
pathological examination.
• In cases of suspected genital TB, endometrial biopsy can reveal granulomatous inflammation,
caseating necrosis, or acid-fast bacilli, which are indicative of TB infection.

Page 48

1216
• It provides direct evidence of TB involvement in the genital tract.

Solution for Question 35:


Correct option:
Option D: Suprative & Invasive
• Cellulitis is a bacterial skin infection characterized by inflammation of the skin and underlying tissues.
The correct option is option d: Suppurative and Invasive.
• Cellulitis is a bacterial skin infection that affects the deeper layers of the skin and underlying tissues.
• It is commonly caused by bacteria such as Staphylococcus and Streptococcus.
• Cellulitis usually occurs when bacteria enter the skin through a cut, scrape, insect bite, or other breaks
in the skin barrier.
• The infection typically leads to redness, swelling, warmth, and pain in the affected area.
• It may also be accompanied by symptoms like fever, chills, and swollen lymph nodes.
• The infection can spread rapidly, both horizontally across the skin and vertically into deeper tissues.
• Risk factors for cellulitis include compromised immune system, chronic skin conditions, obesity,
lymphedema, and previous episodes of cellulitis.
• Prompt treatment with antibiotics is essential to control the infection and prevent complications
Incorrect options:
Option A: Stating cellulitis as nonsuppurative and noninvasive, is incorrect. Cellulitis is typically associa
ted with the presence of pus (suppuration) due to bacterial infection, and it can involve deeper layers of
the skin and tissues (invasive).
Option B: Stating cellulitis as nonsuppurative and invasive, is also incorrect. While cellulitis can be inva
sive, it is commonly associated with the presence of pus (suppuration) as bacteria invade and cause a
n inflammatory response in the affected area.
Option C: Stating cellulitis as suppurative and noninvasive, is incorrect as well. Cellulitis often involves
the production of pus (suppuration) due to bacterial infection, and it can extend into deeper tissues (inv
asive).

Solution for Question 36:


Correct option:
Option C:
Prostate cancer most commonly originates in the peripheral zone of the prostate gland
The peripheral zone constitutes the outer part of the prostate and is where about 70-80% of prostate c
ancers arise.

Page 49

1217
Incorrect options:
Option A. Central zone: Prostate cancer is less frequently seen in the central zone of the prostate. The
central zone comprises about 5-10% of the prostate gland and is located near the ejaculatory ducts.
Option B. Transition zone: Prostate cancer is less commonly observed in the transition zone as well. T
he transition zone surrounds the urethra and accounts for approximately 10-20% of the prostate gland.
Benign prostatic hyperplasia (BPH), a non-cancerous condition, is more commonly found in this zone.
Option D. Anterior fibromuscular stroma: Prostate cancer does not typically arise in the anterior fibromu
scular stroma. The anterior fibromuscular stroma is a
non-glandular tissue in the prostate that provides structural support.

Solution for Question 37:


Correct Option:
Option B.
Perthes test can be used to rule out DVT
The Perthes test is a
clinical test for assessing the patency of the deep femoral vein prior to varicose vein surgery.
Perthes test – Elastic bandage is applied in lower limb causing superficial vein obstruction

Crampy pain after work or Exercise indicating deep vein obstruction
DVT

Incorrect options:
Options A. Brodie Tredelenberg test: The Trendelenburg test is used to determine the site of valvular in
competence in a patient with varicose veins.
Options C. Thomas test: The Thomas test is a
test performed to assess hip flexion contracture and is not specific to ruling out DVT in a
patient with varicose veins.
Options D. Ober test: The Ober test is used to assess tightness or contracture of the iliotibial band (ITB
) and is unrelated to the diagnosis of DVT in a patient with varicose veins.

Solution for Question 38:


Correct option:
Option A: Iliac fossa
• During a left kidney transplantation, the transplanted kidney is typically placed in the iliac fossa.
• The iliac fossa is a concave region located in the lower part of the abdomen.
• It is situated on the inner surface of the ilium, which is a bone in the pelvis.

Page 50

1218
• The iliac fossa provides a suitable anatomical space for accommodating the transplanted kidney and
connecting its blood vessels and ureter.
Incorrect options:
Option B: Kidney fossa: There is no specific anatomical location known as the "kidney fossa." The term
"fossa" generally refers to a shallow depression or hollow area in a
bone, but it does not accurately describe the placement of a transplanted kidney.
Option C: Retroperitoneal space: The retroperitoneal space is the area located behind the peritoneum,
which is the lining of the abdominal cavity. While the kidneys are naturally situated in the retroperitonea
l space, during transplantation, the transplanted kidney is typically placed in the iliac fossa rather than t
he retroperitoneal space.
Option D: Hypogastric region: The hypogastric region refers to the lower central part of the abdomen, b
elow the umbilical region. It is not the specific location where a
left kidney transplantation would be placed. The iliac fossa, as mentioned earlier, is a
more accurate and commonly used placement site for a transplanted kidney.

Solution for Question 39:


Correct Option B: Retrograde cystourethrogram with penile urethral stricture
• Based on the image, the correct answer is B.
• The image appears to be a retrograde cystourethrogram, which involves the injection of contrast
material into the urethra and imaging to evaluate the urinary tract.
• The narrowing or obstruction observed in the penile urethra indicates a penile urethral stricture.
Incorrect Option
Option A. Retrograde cystourethrogram with membranous urethral stricture: This option is incorrect. A r
etrograde cystourethrogram is an imaging procedure used to evaluate the urethra and urinary bladder.
However, the image provided does not show a membranous urethral stricture.
Option C. Micturating cystourethrography with bulbar stricture: This option is incorrect. Micturating cyst
ourethrography (MCU) is a dynamic imaging study used to assess the bladder and urethra during voidi
ng. However, the image provided does not depict a bulbar stricture.
Option D. Micturating cystourethrography with prostatic urethral stricture: This option is incorrect. The i
mage does not show a prostatic urethral stricture, which is a
narrowing or obstruction in the urethra at the level of the prostate gland.

Solution for Question 40:


Correct Option A:
• Partial nephrectomy: This option is correct. Partial nephrectomy, also known as nephron-sparing
surgery, is the preferred treatment for renal cell carcinoma (RCC) when the tumor is less than 4 cm in
size.
Incorrect Option:

Page 51

1219
Option B. Partial nephrectomy + neoadjuvant chemotherapy: This option is incorrect. Neoadjuvant che
motherapy, which is chemotherapy given before surgery, is not the standard treatment for RCC with a
tumor size less than 4 cm. It is not the recommended approach in this scenario.
Option C. Radical nephrectomy: This option is incorrect. Radical nephrectomy involves the complete re
moval of the affected kidney along with the surrounding tissues, including the adrenal gland and nearb
y lymph nodes. It is typically reserved for larger tumors or cases where preservation of kidney function i
s not a concern.
Option D. Radical nephrectomy + postoperative radiotherapy: This option is incorrect. Postoperative ra
diotherapy is not the standard treatment for RCC with a tumor size less than 4
cm. It is not routinely recommended in this scenario.

Solution for Question 41:


Correct Option B:
• Bulbar urethra injury: The presence of an urge to micturate but inability to urinate, along with blood at
the tip of the urethra, suggests an injury to the bulbar urethra. The bulbar urethra is the most common
site of injury in pelvic trauma and is prone to damage due to its fixed position and lack of mobility.
Incorrect Option;
Option A. Membranous urethra rupture:
• This option is incorrect. Membranous urethra rupture typically presents with a high-riding prostate on
digital rectal examination and is associated with an inability to void urine. However, in the given
scenario, the presence of blood at the tip of the urethra and swelling of the penis and scrotum suggests
a more distal injury, making membranous urethra rupture less likely.
Option C. Prostatic urethra injury:
• This option is incorrect. Prostatic urethra injury usually occurs in cases of severe pelvic trauma and is
associated with blood at the urethral meatus, an inability to void urine, and pelvic pain. However, the
symptoms described in the scenario, such as swelling of the penis and scrotum, indicate a more distal
injury, making prostatic urethra injury less likely.
Option D. Urinary bladder rupture:
• This option is incorrect. Urinary bladder rupture typically presents with gross hematuria, an inability to
pass urine, and abdominal pain. The presence of blood at the tip of the urethra and swelling of the
penis and scrotum in the given scenario suggests a urethral injury rather than a urinary bladder rupture.

Solution for Question 42:


Correct option:
Option D: Testicular Torsion
• The given image in question is suggestive of testicular torsion.
• Testicular torsion is a urological emergency characterized by the twisting of the spermatic cord, which
results in the twisting of the testicle.

Page 52

1220
• This condition leads to the disruption of blood supply to the testicle, causing severe pain, swelling, and
potential damage to the testicular tissue
• Testicular torsion requires immediate medical attention and surgical intervention to restore blood flow
and prevent testicular loss.
Incorrect options:
Option A: Hydrocele: Hydrocele refers to the accumulation of fluid within the sac that surrounds the test
icle. It typically causes painless swelling of the scrotum but is unrelated to testicular torsion.
Option B: Varicocele: Varicocele is the enlargement of the veins within the scrotum. It often presents a
s a mass or swelling and is commonly described as a "bag of worms" sensation. While varicocele can
cause discomfort or pain, it is not directly associated with testicular torsion.
Option C: Testicular atrophy: Testicular atrophy refers to the shrinking or reduction in size of the testicl
es. It can occur due to various reasons, such as hormonal imbalances, infections, or trauma. Testicular
atrophy is not specific to testicular torsion but may occur as a
consequence of prolonged or untreated torsion.

Solution for Question 43:


Correct Option A.
• In a female an ectopic ureter opens either into the urethra below the sphincter or into the vagina. The
diagnosis is made from history and confirmed by excretion orography.
• A girl or woman who has dribbled urine for a long time, despite the fact that she has the desire to void
and indeed voids normally, probably has an ectopic ureteral orifice.
• The orifice is often difficult to see because of the valve: An intravenous injection of a dye such as
indigo carmine can be used to color the urine leaking from it
Incorrect Options
Option B: Ureterocele: Ureterocele is a cystic enlargement of the intramural portion of the ureter thoug
ht to result from congenital atresia of the ureteric orifice.
Option C: Congenital megaureter: Uncommon condition which may be bilateral. This is a
functional obstruction at the lower end of the ureter that leads to progressive dilatation and a
tendency to infection.
Option D: Vesicoureteric reflux : Vesicoureteric reflux of urine is detectable in about 35 percent of childr
en with recurrent urinary infections. In some patients, the reflux is caused by high pressure in a
neuropathic bladder. It may be intermittent and is often more marked when there is active infection

Solution for Question 44:


Correct Option D.
Undescended testes are when one or both of the male testes have not passed down (descended) into t
he scrotal sac. This is a
condition seen in some newborn baby boys. Most cases involve only one testis.

Page 53

1221
Incorrect Options
Option A and B: It is not indicated since the baby is only one month old.
Option C: The undescended testicle may be moved into the scrotal sac by a
procedure called orchiopexy. It's often done between ages 6 and 18 months.

Solution for Question 45:


Correct Option A.
• Calcium oxalate stones : It is the most common type of the renal stone, It is radio-opaque
• If the patient is having increased calcium in the urine, it gets precipitated with the oxalate leading to
the formation of calcium oxalate stones.
Incorrect Options
Option B: Cystine:. There is a defect in the absorption of COLA amino acids (COLA – Cysteine, Ornithi
ne, Lysine, and Arginine) due to which the cysteine can precipitate out and lead to the formation of cyst
ine stones. The other amino acids do not form the stones.
Option C: Triple phosphate: It usually presents early due to colic renal abdominal pain. It grows along t
he pelvicalyceal system. Therefore, it does not cause obstruction to the urine flow and the patient will p
resent very late. The appearance of the stone resembles the antler's head and that is why it is known a
s staghorn. It is also known as Struvite stone and triple phosphate stone.
Option D: Uric acid: Any condition which leads to the increased proliferation of cells is one of the predis
posing factors for the formation of uric acid stones.Pure uric acid stones are radiolucent. It is rarely fou
nd.

Solution for Question 46:


Correct Option A.
Whitaker test
• A percutaneous puncture of the kidney is made through the loin and fluid is infused at a constant rate
with monitoring of intrapelvic pressure.
• An abnormal rise in pressure confirms obstruction of the ureter.
• Retrograde pyelography is rarely indicated but will confirm the site of obstruction immediately before
corrective surgery.
Procedure: Percutaneous puncture of the renal pelvis is performed or an indwelling nephrostomy tube i
s used. The upper urinary tract is then perfused at a
constant rate of 5--10 ml/min with saline or diluted contrast media, and a serial pressure recording is m
ade in the renal pelvis and bladder. The high flow rate used will be tolerated easily in a
nonobstructed system without a progressive rise in renal pelvic pressure. In obstructed systems abnor
mally high pressure above 12 cm of water or a constant rise in pressure will be recorded.
Incorrect Options

Page 54

1222
Option B: Posterior urethral valve : The gold standard imaging modality for the diagnosis of posterior ur
ethral valves is Voiding Cystourethrography (VCUG). The patient is catheterized, and a contrast agent
(sodium and meglumine diatrizoate) is injected into the bladder, fluoroscopy is then used to observe vo
iding and to identify reflux.
Option C: Urethral atresia: Diagnosis is by Ultrasonography.
Option D: Urethral stricture: Is diagnosed by cystoscopy procedure. This allows doctors to view the uret
hra to see if strictures are present. A narrow, flexible scope into the urethra uses ultrasound to determi
ne the location and length of strictures.

Solution for Question 47:


Correct Option C.
• An indirect inguinal hernia travels down the canal on the outer (lateral and anterior) side of the
spermatic cord. This is the most common of all forms of hernia. It is most common in the young,
whereas a direct hernia is most common in the old.
• As an indirect inguinal hernia increases in size, it becomes apparent when the patient coughs and
persists until reduced . Gradually the hernia comes down as soon as the patient stands up.
• In large hernias, there is a sensation of weight, and dragging on the mesentery may produce
epigastric pain.
• A direct inguinal hernia shows a bulge from the posterior wall of the inguinal canal, whereas an
indirect inguinal hernia passes through the inguinal canal or the groin.
Indirect Inguinal Hernia:
Sac enters via deep ring

Traverse inguinal canal
Goes out via superficial ring

Page 55

1223
Incorrect Options
Option A: Femoral hernia: Femoral hernia is the third most common type of primary hernia. It accounts
for about 20 percent of hernias in women and 5
percent in men. The importance of femoral hernia lies in the facts that it cannot be controlled by a truss
and that of all hernias it is the most liable to become strangulated, mainly because of the narrowness
of the neck of the sac and the rigidity of the femoral ring
Option B: Direct inguinal hernia: A direct inguinal hernia is always acquired. The sac passes through a
weakness or defect of the transversal fascia in the posterior wall of the inguinal canal. In some cases, t
he defect is small and is represented by a
discrete defect in the transversalis fascia, while in others there is a generalized bulge. Often the patient
has poor lower abdominal musculature, as shown by the presence of elongated bulgings (Malgaigne’s
bulges).
Option D: Umbilical hernia: It is due to failure of all or part of the midgut to return to the coelom during e
arly fetal life. Symptomatically, a large umbilical hernia causes a dragging pain by its weight. Gastrointe
stinal symptoms are common and are probably due to traction on the stomach or transverse colon.

Solution for Question 48:


Correct Option A.
Spermatocele
• Is a unilocular retention cyst derived from some portion of the sperm-conducting mechanism of the
epididymis.
Clinical features
• It lies in the head of the epididymis above and behind the upper pole of the testis, so one can feel the
testis separate from the scrotal swelling. It is usually softer and laxer than other cystic lesions in the
scrotum but like them it transilluminates.
• The fluid contains spermatozoa and resembles barley water in appearance.
• Spermatoceles are usually small and unobtrusive. Less frequently they are large enough to make the
patient think that he has three testicles.
Treatment:
Small spermatoceles can be ignored. Larger ones should be aspirated or excised through a
scrotal incision.
Incorrect Options:
Option B: Epididymal cyst : Epididymal cysts These are filled with crystal-clear fluid (contrary to a
spermatocele is filled with barley-water fluid or the amber fluid of a hydrocele).
Option C and D: Secondary hydrocele and primary hydrocele:
• Secondary hydrocele is most frequently associated with acute or chronic epididymo-orchitis. Primary
vaginal hydrocele is most common in middle and later life but can also occur in early childhood.

Solution for Question 49:

Page 56

1224
Correct Option B.
• A varicocele is a varicose dilatation of the veins draining the testis.
• Most varicoceles appear in adolescence or early adult-hood. The left side is affected mostly.
• There may be a vague and annoying dragging discomfort which is worse if the testis is unsupported
by underwear.
• The scro-tum on the affected side hangs lower than normal position .
• On palpation, with patient standing, the varicose plexus feels like a bag of worms (which was felt by
the examiner/physician on palpation hence the diagnosis)
Incorrect Options
Option A: Hydrocele: A hydrocele is an abnormal collection of serous fluid in some part of the processu
s vaginalis, usually the tunica.
Option C: Testicular torsion : Testicular torsion is seen in a teenager or young adult male. It is easily mi
ssed. Pain can be referred to the right iliac fossa, and will suspect appendicitis unless the scrotum is ex
amined.
Option D: Epididymo Orchitis: Epididymo Orchitis is intrascrotal swelling and is the most common amo
ng intra scrotal swelling. It is caused usually as a result of retrograde ascent of pathogens.

Solution for Question 50:


Correct Option A.
• In a radical nephrectomy, the surgeon removes the whole kidney, the fatty tissues surrounding the
kidney and a portion of ureter.It is indicated in multiple small tumours and when tumours extend into
vasculature.
• The management of renal cell carcinoma is based on the stage of the disease.Nephrectomy is now
rarely performed for benign disease. It may be necessary if the kidney is atrophic or dysplastic or the
cause of accelerated hypertension.
Incorrect choices:
Option B: Partial nephrectomy: If one moiety is severely diseased or atrophic, partial nephrectomy is us
ually simple and effective.
Option C: Radiotherapy: Radiotherapy (RT) can decrease the risk of local recurrence after surgery and
can lead to better outcomes in patients who are unfit for surgery
Option D: Chemotherapy: Chemotherapy isn't generally advised renal cell carcinoma.

Solution for Question 51:


Correct Option D:
• Metastasising testicular tumor
• Presenting clinical features and para aortic lymph node suggests towards testicular tumor.
• Primary landing site from left testicular tumor- Para-aortic LN (Overall MC)

Page 57

1225
Clinical features of Seminoma
• Painless swelling or mass in relation to testis
• 10% cases have secondary hydrocele
• 5% cases - Gynecomastia Most common in Leydig cell tumors Sertoli cell tumors Granulosa cell
tumor Gonadoblastoma
• Most common in Leydig cell tumors
• Sertoli cell tumors
• Granulosa cell tumor
• Gonadoblastoma
• Seminoma -2/3rd-localized tumor
• NSGCT-widespread metastasis
• Most common route of spread-Lymphatics
• Choriocarcinoma Early hematogenous spread Most common site of metastasis : lungs CannonBall
secondaries (Renal cell carcinoma > Choriocarcinoma)
• Early hematogenous spread
• Most common site of metastasis : lungs
• CannonBall secondaries (Renal cell carcinoma > Choriocarcinoma)
• Most common in Leydig cell tumors
• Sertoli cell tumors
• Granulosa cell tumor
• Gonadoblastoma
• Early hematogenous spread
• Most common site of metastasis : lungs
• CannonBall secondaries (Renal cell carcinoma > Choriocarcinoma)
Incorrect Options:
Option A: Hydrocele fluid is amber coloured and sterile, and contains albumin and fibrinogen.
Option B and C: An indirect inguinal hernia increases in size it becomes apparent when the patient cou
ghs and persists until reduced. Direct inguinal hernia some cases the defect is small and is represente
d by a discrete defect in the transversalis fascia, while in others there is a generalised bulge

Solution for Question 52:


Correct Option C.
• The next best step in the management of a painless large round firm right testicular swelling with an
elevated AFP level is high inguinal orchidectomy as these findings are suggestive of testicular tumor
probably non seminomatous germ cell tumor due to raised AFP levels.

Page 58

1226
• An inguinal orchidectomy, also known as an inguinal orchiectomy or radical orchiectomy, is a surgical
procedure to remove one or both testicles. It is commonly performed as a treatment for testicular
cancer or for other conditions such as testicular torsion or certain cases of transgender surgery.
Incorrect Options
Option A. "FNAC," is incorrect because FNAC is not recommended in the management of testicular ma
sses.
Option B. "Scrotal true-cut biopsy," is incorrect because a high inguinal orchidectomy is the preferred a
pproach for the management of suspected testicular malignancies.
Option D. "Repeat tumor markers," is incorrect because an elevated AFP level in the context of a testic
ular mass requires further evaluation and definitive management rather than repeating tumor markers.

Solution for Question 53:


Correct Option D.
• In a patient who fell off a bicycle, has blood at the meatus, perineal bruising, and no palpable bladder,
the management would be to wait for the bladder to fill and for the patient to have an urge to urinate.
• This is done to ensure the urethral integrity before attempting catheterization.
Incorrect Options
Option A. "Wait and send him home," is incorrect because in this scenario, there are indications of pote
ntial urethral injury, and therefore, the patient should be managed actively rather than being sent home
without intervention.
Option B. "Immediate insertion of Foley's catheter," is incorrect because there are signs of potential ure
thral injury, and inserting a
Foley catheter without assessing urethral integrity can lead to further complications.
Option C. "Suprapubic aspiration of urine," is incorrect because suprapubic aspiration is not the initial
management approach in this scenario. The bladder needs to be filled and the urge to urinate needs to
be assessed before considering invasive measures.

Solution for Question 54:


Correct Option A.
• Posterior urethral valve (PUV) is the most common cause of bladder outlet obstruction in male
children.
• PUV is an abnormal fold of tissue within the urethra that obstructs the flow of urine.
• It is a congenital condition that occurs during foetal development.
• PUV can cause various urinary symptoms, such as difficulty urinating, weak urinary stream, urinary
tract infections, and urinary retention. It is more commonly seen in male infants.
Incorrect Options:
Option B. Urethral atresia: Urethral atresia is a
rare condition in which the urethra is completely closed or absent. It is a congenital anomaly and can le

Page 59

1227
ad to urinary retention and other urinary complications. While urethral atresia can cause bladder outlet
obstruction, it is not as common as the posterior urethral valve in male children.
Option C. Anterior urethral valve: Anterior urethral valve is a less common condition compared to the p
osterior urethral valve. It is characterised by abnormal tissue folds or membranes in the anterior portion
of the urethra. Anterior urethral valve can cause urinary obstruction and other urinary symptoms, but it
is not the most common cause of bladder outlet obstruction in male children.
Option D. Ureterocele: A ureterocele is a bulging of the ureter into the bladder. It can obstruct the flow
of urine from the affected kidney. While ureterocele can lead to urinary obstruction, it primarily affects t
he upper urinary tract rather than the bladder outlet. Therefore, it is not the most common cause of bla
dder outlet obstruction in male children.

Solution for Question 55:


Correct Option D.
Incontinence during intercourse is not typically considered a lower urinary tract symptom. Lower urinar
y tract symptoms are related to problems with the bladder and urethra, and they commonly include the
following:
Incorrect options:
Option A. Sudden urge to urinate (A): This symptom, as mentioned earlier, is known as urgency. It is c
haracterized by a strong and immediate need to urinate, which may be difficult to delay.
Option B. Incontinence while asleep (B): Nocturnal enuresis, or bedwetting, is the involuntary loss of uri
ne during sleep. It can occur in children and adults and is considered a lower urinary tract symptom.
Option C. Incontinence when the patient lies down and coughs (C): This symptom is called stress urina
ry incontinence. It involves the involuntary loss of urine when pressure is exerted on the bladder, such
as during coughing, sneezing, laughing, or physical activity. It is a
common lower urinary tract symptom, particularly in women.

Solution for Question 56:


The given clinical scenario is Wilms tumour or Neuroblastoma
Incorrect Choices:
4. FNAC (Fine Needle Aspiration Cytology): FNAC is not included in the correct answer. FNAC is gene
rally not recommended as the initial investigation in the case of a suspected neuroblastoma. Neuroblas
tomas are highly vascular tumors, and FNAC can lead to bleeding and dissemination of tumor cells. Th
erefore, other investigations such as imaging (CECT, PET-CT) and specific tests (24-hour urinary VMA
) are preferred before considering a biopsy or FNAC.

Solution for Question 57:

Page 60

1228
Correct option A - Transurethral resection of the tumour:
• Transurethral resection of the tumour (TURBT) is the initial standard treatment for low-grade
transitional cell carcinoma (TCC) of the bladder.
• This procedure is performed using a cystoscope inserted through the urethra to remove the tumor
from the bladder.
• TURBT allows for both diagnostic and therapeutic purposes, as the tumor is removed and sent for
histopathological examination to confirm the diagnosis and assess the stage and grade of the tumor.
Incorrect options:
Option B - Resection with ileal conduit:
• Resection with ileal conduit is not the ideal management for low-grade transitional cell carcinoma of
the bladder.
• This procedure involves the removal of the entire bladder and the diversion of urine through a
surgically created ileal conduit (urinary diversion).
• It is typically reserved for cases of invasive or high-grade bladder cancer that cannot be adequately
treated with transurethral resection alone.
Option C - Partial cystectomy with bladder reconstruction:
• Partial cystectomy with bladder reconstruction may be considered for select cases of localised
low-grade bladder cancer, particularly if the tumour is in a favourable location and can be completely
excised while preserving bladder function.
• However, transurethral resection is generally the preferred initial treatment for most cases of
low-grade bladder cancer.
Option D - Neoadjuvant chemotherapy:
• Neoadjuvant chemotherapy refers to the administration of chemotherapy before the main treatment.
• It is typically used for muscle-invasive bladder cancer or cases with a high risk of progression.

Solution for Question 58:


Correct Option A
• Cyclophosphamide is an alkylating agent and an immunosuppressant that is used in the treatment of
various cancers, including bladder cancer. However, long-term or high-dose use of cyclophosphamide
has been associated with an increased risk of developing bladder carcinoma. It is important to note that
the risk of bladder carcinoma is a potential side effect of cyclophosphamide, but not everyone who
takes the drug will develop this condition.
Incorrect Options
Option B Cisplatin is a chemotherapy drug commonly used to treat various types of cancer, but it is not
typically associated with bladder carcinoma.
Option C.Taxane drugs, such as paclitaxel and docetaxel, are commonly used in the treatment of brea
st, lung, and ovarian cancers. They are not known to be causally associated with bladder carcinoma.
Option D. Tamoxifen is a selective estrogen receptor modulator (SERM) primarily used in the treatment
of breast cancer. It is not known to be associated with bladder carcinoma.

Page 61

1229
Solution for Question 59:
Correct Option A: Hydronephrosis
• In the provided retrograde ureteropyelogram image, if there is evidence of dilatation or swelling of the
renal pelvis and calyces, it indicates that there is an obstruction in the urinary tract leading to the
accumulation of urine in the kidney. This finding is consistent with the diagnosis of hydronephrosis.
Incorrect Options:
Option B: Duplex kidney- A duplex kidney refers to a
congenital condition where an individual has two separate ureters draining a
single kidney. This condition does not typically present with hydronephrosis as seen in the image.
Option C: Renal carcinoma- Renal carcinoma, or kidney cancer, is a malignant tumor that develops in t
he kidney. While renal carcinoma can cause obstruction and potentially lead to hydronephrosis, the retr
ograde ureteropyelogram image alone cannot provide a definitive diagnosis of renal carcinoma.
Option D: Renal stone- Renal stones, also known as kidney stones or renal calculi, are solid masses th
at form in the kidneys. While renal stones can cause obstruction and subsequent hydronephrosis, the r
etrograde ureteropyelogram image alone cannot confirm the presence of a renal stone.

Solution for Question 60:


Correct Option B: Ammonium magnesium phosphate
• Ammonium magnesium phosphate is Caused by infection of protease.
• Account for 15% of stones. Caused by infection with urease ⊕ bugs (eg, Proteus mirabilis,
Staphylococcus saprophyticus, Klebsiella) that hydrolyze urea to ammonia --> urine alkalinization.
• Commonly form staghorn calculi.
Account for 15% of stones. Caused by infection with urease ⊕ bugs (eg, Proteus mirabilis, Staphylococ
cus saprophyticus, Klebsiella) that hydrolyze urea to ammonia --> urine alkalinization.
Commonly form staghorn calculi.
Incorrect Options:
Option A: Uric acid stones- Uric acid stones form due to high levels of uric acid in the urine. They are c
ommonly associated with conditions such as gout or high purine intake in the diet. Uric acid stones are
not specifically related to infection of protease.
Option C: Struvite stones- Struvite stones are composed of magnesium ammonium phosphate and are
typically associated with urinary tract infections caused by urease-producing bacteria. These bacteria
convert urea into ammonia, which increases the urine pH and promotes the formation of struvite stones
. They are not related to infection of protease.
Option D: Calcium oxalate stones- Calcium oxalate stones are the most common type of kidney stones
. They form due to an excessive amount of oxalate in the urine or reduced urine volume, leading to hig
her concentrations of oxalate. They are not related to infection of protease.

Page 62

1230
Solution for Question 61:
Correct Option A: Seminoma
• The image provided in the question is consistent with the histopathological appearance of a seminom
• Seminoma: Seminoma is a type of testicular germ cell tumor. It typically presents as a painless
testicular mass or swelling.
• Histologically, seminomas consist of uniform, undifferentiated germ cells with clear cytoplasm and
prominent nuclei.
Incorrect Options:
Option B: Yolk-sac tumor- Yolk-sac tumor, also known as endodermal sinus tumor, is another type of t
esticular germ cell tumor. It primarily occurs in infants and young children. Histologically, yolk-sac tumo
rs exhibit characteristic structures resembling a primitive yolk sac, such as Schiller-Duval bodies. The p
rovided histopathology image does not show the features of a yolk-sac tumor.
Option C: Embryonal cell carcinoma- Embryonal cell carcinoma is another type of testicular germ cell t
umor. It is composed of primitive, undifferentiated germ cells. Histologically, embryonal cell carcinomas
often exhibit a mixture of different tissue types and can show features of glandular or tubular structure
s. The histopathology image provided does not demonstrate the characteristics of an embryonal cell ca
rcinoma.
Option D: Teratoma- Teratoma is a type of testicular germ cell tumor that contains a variety of tissues d
erived from three embryonic germ layers (ectoderm, mesoderm, and endoderm). Histologically, terato
mas exhibit a diverse mixture of tissues, such as epithelial, mesenchymal, and neural elements. The pr
ovided histopathology image does not resemble the features of a teratoma.

Solution for Question 62:


Correct Option D- E.coli:
• Acute bacterial prostatitis is an infection of the prostate gland that is primarily caused by bacteria. E.
coli (Escherichia coli) is the most frequently implicated organism in cases of acute bacterial prostatitis.
• E. coli is a type of bacteria commonly found in the gastrointestinal tract, and it can enter the prostate
gland through various routes, such as ascending infection from the urethra or reflux of infected urine
into the prostatic ducts.
Incorrect Options
Option A- Enterococcus: Enterococcus species are Gram-positive bacteria that can cause urinary tract
infections, including prostatitis. However, they are less commonly associated with acute bacterial prost
atitis compared to E. coli. Enterococcus is more frequently implicated in infections such as urinary tract
infections (UTIs), intra-abdominal infections, and infective endocarditis.
Option B- Proteus: Proteus species, particularly Proteus mirabilis, are known to cause urinary tract infe
ctions. While they can infect the prostate gland, they are not the most common organism associated wi
th acute bacterial prostatitis. Proteus species are more commonly associated with complicated UTIs, p
articularly those involving urinary tract calculi or urinary catheterization.
Option C- Streptococcus Agalactiae: (Group B
Streptococcus) Streptococcus agalactiae, also known as Group B Streptococcus (GBS), is a Gram-pos
itive bacterium that can cause various infections, including urinary tract infections. However, it is not a
common cause of acute bacterial prostatitis. GBS is more commonly associated with infections in preg

Page 63

1231
nant women, newborns, and immunocompromised individuals.

Solution for Question 63:


Correct choice: C Right-side undescended testis and left-sided ectopic testis
• In the case of undescended testis the testis is usually found in the route at which it descends such as
the inguinal canal. In the above patient, the right testis was found in the inguinal canal. In ectopic testis,
the testis is found in places other than the route of descent of the testis as in this case the left testis is
seen in the perineum. Hence the correct choice is Right-side undescended testis and left-sided ectopic
testis.
Incorrect choices:
Option A. Bilateral undescended tesis: The left testis is found in the perineum which is indicative of ect
opic testis. Hence this option is incorrect.
Option B. Bilateral ectopic testis: The right testis is found in the inguinal canal which is indicative of an
undescended testis. Hence this option is incorrect.
Option D. Right side ectopic testis and left side undescended testis: the right testis is found in the ingui
nal canal and is indicative of undescended testis whereas the left side is found in the perineum which i
s indicative of ectopic testis hence, this option is incorrect.

Solution for Question 64:


Correct choices: C. Right testicular torsion
• In the case of torsion of the testis, the testicular clinical features include an empty, swollen scrotum
which is oedematous on the side of the lesion. Tender lump at the external abdominal ring. Elevation of
the scrotum increases pain.
Incorrect choices:
Option A. Right-sided hydrocele: Hydrocoele presents with a painless swelling. There is no sudden pai
n seen in the right-sided hydrocele. Hence it is not the correct choice.
Option B. Strangulated inguinal hernia: In the case of strangulated inguinal hernia there will be ischemi
c changes seen in the scrotum. The scrotum will not be edematous.
Option D. Right epi didymo orchitis: It is an inflammation of the epididymis. It is caused due to a
bacterial infection and presents as a swelling on the posterior aspect of the epididymis.

Solution for Question 65:


Correct Answer: C
• In the context of a 68-year-old man presenting with low backache and an elevated PSA level of 100
ng/ml, the most appropriate drug for treatment would be "Goserelin."

Page 64

1232
• Goserelin: Goserelin is a gonadotropin-releasing hormone (GnRH) agonist. It is commonly used in the
treatment of prostate cancer, including in younger patients with advanced disease. Goserelin works by
reducing the production of testosterone, which can help slow down the growth of prostate cancer cells.
It is administered as a subcutaneous implant or injection and can effectively lower PSA levels.
Incorrect options:
Option A: Somatostatin: Somatostatin analogs such as octreotide and lanreotide are primarily used in t
he treatment of neuroendocrine tumors and certain hormonal disorders. They are not typically indicate
d for the treatment of prostate cancer or elevated PSA levels.
Option B: Terlipressin: Terlipressin is a synthetic analog of vasopressin and is mainly used in the mana
gement of conditions such as variceal bleeding and hepatorenal syndrome. It is not a
standard treatment for prostate cancer or elevated PSA levels.
Option D: Testosterone: Testosterone is a male hormone that can promote the growth of prostate canc
er cells. In the case of prostate cancer with elevated PSA levels, the goal of treatment is to reduce test
osterone levels rather than administer additional testosterone.
Considering the patient's age, symptoms, and elevated PSA level, goserelin would be the most approp
riate treatment option.

Solution for Question 66:


Correct Option: B
• The treatment of choice for a 2.5 cm renal stone in a 25-year-old morbidly obese male patient with
severe abdominal pain and urinary disturbances is Percutaneous Nephrolithotomy (PCNL).
• Percutaneous nephrolithotomy (PCNL): PCNL is a minimally invasive surgical procedure that involves
making a small incision in the back and inserting a nephroscope to directly visualize and remove the
kidney stone. It is considered the treatment of choice for larger stones (>2 cm) or stones that are not
amenable to ESWL. In this case, with a 2.5 cm stone and severe symptoms, PCNL is the most
appropriate option.
Incorrect options:
Option A: Extracorporeal shockwave lithotripsy (ESWL): ESWL uses shock waves to break down kidne
y stones into smaller pieces that can be passed through urine. It is more suitable for smaller stones (ty
pically less than 2 cm) and may not be as effective for larger stones like the one in this case (2.5 cm).
Option C: Open surgery: Open surgery, also known as open nephrolithotomy, involves making a larger
incision in the back to directly access and remove the kidney stone. It is now less commonly performed
due to the availability of less invasive techniques like PCNL.
Option D: Ureteroscopy-assisted removal: Ureteroscopy involves the insertion of a thin tube (ureterosc
ope) through the urethra and bladder to reach the stone in the ureter or kidney. While ureteroscopy can
be effective for smaller stones, a
2.5 cm stone in the kidney may be difficult to reach and remove solely with ureteroscopy.
Therefore, the correct answer is Percutaneous nephrolithotomy (PCNL). It allows for direct visualizatio
n and effective removal of the large renal stone in a
minimally invasive manner, making it the treatment of choice in this case.

Page 65

1233
Solution for Question 67:
Correct Option: B
• According to Weigert-Meyer's rule of duplication of the ureter, the correct statement regarding the
lower pole ureter in the urinary bladder is:
• Lateral and caudal to the upper pole ureter: This option is correct. Weigert-Meyer's rule describes the
anatomical relationship between the upper and lower pole ureters in a duplicated ureter. The lower pole
ureter is located on the lateral side and in a more caudal position compared to the upper pole ureter.
Incorrect options:
Option A: Lateral and cephalad to the upper pole ureter: This option is incorrect. Weigert-Meyer's rule s
tates that the lower pole ureter is located laterally and caudally to the upper pole ureter, not cephalad (t
owards the head).
Option C: Medial and cephalad to the upper pole ureter: This option is incorrect. Weigert-Meyer's rule d
oes not describe the lower pole ureter as being medial to the upper pole ureter. It is rather lateral in rel
ation to the upper pole ureter.
Option D: Medial and caudal to the upper pole ureter: This option is incorrect. Weigert-Meyer's rule stat
es that the lower pole ureter is located laterally and caudally to the upper pole ureter, not medial (towar
ds the midline).
Therefore, the correct answer is: The lower pole ureter in the urinary bladder is lateral and caudal to th
e upper pole ureter, according to Weigert-Meyer's rule of duplication of the ureter.

Solution for Question 68:


Correct Option: A
• This is a typical case of extraperitoneal bladder rupture.
• This is a typical case of extraperitoneal bladder rupture: This option is correct. Based on the given
information, the patient experienced a blow to the distended bladder, resulting in sudden severe pain in
the hypogastrium (lower abdomen) associated with syncope (fainting). Subsequently, the abdomen
began to distend, indicating leakage of urine into the extraperitoneal space. These findings are
consistent with extraperitoneal bladder rupture, which occurs when the bladder wall is disrupted but the
peritoneal lining remains intact.
Incorrect Options:
Option B: MRI is the ideal investigation to diagnose bladder rupture: This option is incorrect. While MRI
can provide detailed imaging of the bladder, it is not the ideal investigation for diagnosing bladder rupt
ure. The initial diagnostic modality of choice for suspected bladder rupture is usually a combination of c
linical assessment, history, physical examination, and imaging studies such as plain radiography (X-ray
) or computed tomography (CT) scan.
Option C: Repair is done by suturing edges with single-layer 2/0 absorbable suture: This option is incor
rect. The repair of bladder rupture typically involves suturing the edges of the bladder wall. However, th
e specific technique and suture material used may vary depending on the extent of the rupture and sur

Page 66

1234
geon's preference. Single-layer suturing with absorbable suture is commonly performed, but the specifi
c size of the suture (2/0 in this option) may not be applicable universally.
Option D: Laparoscopic method is avoided as it can be done only on laparotomy: This option is incorre
ct. Laparoscopic repair of bladder rupture can be a viable option in selected cases. Laparoscopic techn
iques have been utilized for bladder repair, especially in cases of intraperitoneal bladder rupture. The d
ecision to perform laparoscopic repair or open (laparotomy) repair depends on several factors, includin
g the expertise of the surgeon, availability of equipment, and patient-specific factors.
Therefore, the correct answer is: This is a typical case of extraperitoneal bladder rupture.

Solution for Question 69:


Correct Option: D
• The next best step in management for an 11-month-old boy with an empty scrotum on the right side, a
palpable mass in the right inguinal canal, and the right testicle visualized under the subcutaneous fat
near the right inguinal canal on ultrasound (USG) is surgical exploration.
• Surgical exploration: This is the correct answer. Given the clinical findings and the location of the
testicle on USG, surgical exploration is necessary to determine the precise location of the testicle and
its blood supply. Surgical exploration allows for appropriate management, which may include
orchidopexy or other interventions based on the intraoperative findings.
Incorrect options:
Option A: Observation: Observation alone is not appropriate in this case. The presence of an empty scr
otum on the right side and a palpable mass in the right inguinal canal indicate an abnormality in testicul
ar descent that requires further evaluation and intervention.
Option B: Laparoscopy: While laparoscopy can be used in the management of undescended testes, it i
s not the next best step in this case. Surgical exploration is the preferred initial approach when a
testicle is palpable in the inguinal canal.
Option C: Orchidopexy: Orchidopexy, which involves surgically securing the undescended testicle in th
e scrotum, is a treatment option for undescended testes. However, in this case, the location of the testi
cle under the subcutaneous fat near the right inguinal canal suggests an abnormal descent pattern that
requires further evaluation before deciding on the appropriate surgical intervention.
Therefore, the next best step in management for this patient is surgical exploration.

Solution for Question 70:


Correct Option is A: Increased trabeculations in bladder
• Explanation: increased trabeculations in the bladder are not characteristic features of BPH.
Trabeculations are irregular muscular bands or folds seen in the bladder wall, typically associated with
bladder muscle hypertrophy and bladder outlet obstruction. In BPH, the bladder wall may show
hypertrophy, but increased trabeculations are not typically observed.

Page 67

1235
Characteristic features of BPH include:
Option B: J-shaped or fish-hook-shaped proximal ureter: This occurs due to the mechanical obstruction
of the bladder outlet by an enlarged prostate. The ureter may curve or hook as it tries to overcome the
obstruction.
Option C: J-shaped or fish-hook-shaped distal ureters: Similar to the proximal ureter, the distal ureters
may also demonstrate a
curved or hook-shaped appearance due to the bladder outlet obstruction caused by BPH.
Option D: Bilateral hydronephrosis: This refers to the dilation or swelling of the kidneys due to the obstr
uction of urine flow from the bladder. BPH can cause bladder outlet obstruction, leading to backflow of
urine into the kidneys and subsequent hydronephrosis.

Solution for Question 71:


Correct Answer: C.
• Renal cell carcinoma: Renal cell carcinoma is a type of kidney cancer that originates in the cells of the
kidney tubules. It commonly presents with symptoms such as persistent back pain, a palpable mass in
the lumbar region, fatigue, and hematuria. The presence of RBCs in the urine without pus cells, along
with lumbar pain and a ballotable mass, is consistent with renal cell carcinoma. Therefore, renal cell
carcinoma is the most probable diagnosis based on the provided information.
Incorrect options:
Option A. Polycystic kidney disease: Polycystic kidney disease (PKD) is a genetic disorder characteris
ed by the growth of multiple cysts in the kidneys. It typically presents with symptoms such as abdomina
l or flank pain, hypertension, and enlarged kidneys. While PKD can cause hematuria (blood in the urine
), the presence of a ballotable mass and lumbar pain during inspiration is not commonly associated wit
h PKD. Therefore, PKD is less likely to be the most probable diagnosis in this case.
Option B. Hydronephrosis: Hydronephrosis refers to the swelling or enlargement of the kidney due to th
e buildup of urine caused by an obstruction in the urinary tract. It can result from conditions such as kid
ney stones, tumors, or structural abnormalities. While hydronephrosis can cause lumbar pain and a bal
lotable mass in the affected area, the presence of RBCs in the urine without pus cells suggests an alter
native diagnosis. Therefore, hydronephrosis is less likely to be the most probable diagnosis in this case
.
Option D. Chronic pyelonephritis: Chronic pyelonephritis is a long-term inflammation of the kidney resul
ting from recurrent urinary tract infections. It may cause symptoms such as flank pain, fever, frequent u
rination, and the presence of pus cells in the urine. However, the absence of pus cells and the presenc
e of RBCs in the urine, along with the presence of a ballotable mass, are not typical for chronic pyelone
phritis. Therefore, chronic pyelonephritis is less likely to be the most probable diagnosis in this case.

Solution for Question 72:


Correct Answer A:
• Based on the presented information, the probable diagnosis would be metastatic cancer to the
vertebrae. The most likely primary cancer site associated with sclerotic lesions in the vertebrae in a
60-year-old male is carcinoma prostate. Prostate cancer commonly metastasizes to the bones,

Page 68

1236
including the vertebrae, leading to sclerotic (hardened) lesions.
Incorrect Options:
Option B. Tuberculosis (TB) typically affects the lungs, but it can also involve other organs, including th
e spine. Spinal tuberculosis, known as Pott's disease, can cause vertebral destruction and collapse, le
ading to back pain. However, sclerotic lesions in the vertebrae are not characteristic of tuberculosis.
Option C. Renal cell carcinoma (RCC): Renal cell carcinoma is a type of kidney cancer. While it can m
etastasize to the bones, including the vertebrae, it typically presents with lytic (destructive) lesions rath
er than sclerotic lesions. Therefore, RCC is less likely to be the cause of the described presentation.
Option D. Carcinoma colon: Colorectal cancer can metastasize to the bones, including the vertebrae. H
owever, similar to RCC, it typically presents with lytic lesions rather than sclerotic lesions in the bones.

Solution for Question 73:


Correct Answer A :
• Superficial pouch (Superficial perineal space): The superficial pouch, also known as the superficial
perineal space or superficial perineal compartment, is an anatomical space located between the
perineal membrane (a fibrous layer) and the superficial fascia. It contains various structures, including
the root of the penis, bulb of the penis (which includes the bulbar urethra in males), and associated
muscles. If the bulbar urethra is ruptured, urine can potentially leak into the superficial pouch and
accumulate there. Therefore, option 1 can indeed be a correct answer for the location of urine
accumulation in a patient with a ruptured bulbar urethra.
Incorrect options:
Option B. Deep pouch (Space of Retzius): The deep pouch, also known as the space of Retzius, is an
extraperitoneal space located between the pubic symphysis and the anterior bladder wall. It is situated
in the lower abdomen and is not directly involved in the accumulation of urine in the case of a
ruptured bulbar urethra.
Option C. Rectouterine pouch (Pouch of Douglas): The rectouterine pouch, also referred to as the pouc
h of Douglas, is a peritoneal space located between the posterior wall of the uterus (in females) or the
bladder (in males) and the anterior wall of the rectum. It is not directly related to the accumulation of uri
ne in the context of a ruptured bulbar urethra.
Option D. Uterovesical pouch (Vesicouterine pouch): The uterovesical pouch, also called the vesicoute
rine pouch, is a peritoneal space located between the uterus and the bladder in females. Similarly to th
e rectouterine pouch, it is not directly involved in the accumulation of urine in the case of a
ruptured bulbar urethra.

Solution for Question 74:


Correct Option A - GnRH Analogue:
• GnRH analogue increases the level of gonadotropins which further increase the level of androgens
that help in testicular descent as it is a androgen dependent process.
Incorrect Options:

Page 69

1237
Option B - Testosterone: GnRH analogue is preferred over testosterone as neonatal gonadotropin ther
apy can increase the testosterone levels without fear of inducing spermatogenesis or negatively affecti
ng ultimate number of Sertoli cells (an important determinant of fertility).
Option C - Cold Compressions: Cold compresses or ice application is not a
recognized method to assist testicular descent in newborns. It is not a
recommended or established approach for this purpose.
Option D - Manually bring testis down to the bottom of the scrotum: This option describes a manual tec
hnique called orchidopexy or surgical orchiopexy. It involves surgically bringing the undescended testis
down to the scrotum. It is a
definitive treatment for undescended testis, but it is typically performed by a
healthcare professional and not manually by parents or caregivers.

Solution for Question 75:


Correct Answer : B.
• Based on the information provided, the most likely diagnosis based on the histopathology findings and
the normal levels of HCG (human chorionic gonadotropin) and alpha-fetoprotein (AFP) is: Seminoma of
testis.
Incorrect Options:
Option A, C & D. Teratoma of the testis, choriocarcinoma, and yolk sac tumor are other types of germ c
ell tumors that can affect the testes. However, their histopathology findings and tumor marker profiles d
iffer from those seen in this case.

Solution for Question 76:


Correct Answer: B.
• Lumbar sympathectomy can disrupt the sympathetic innervation responsible for controlling the closure
of the bladder neck during ejaculation.
• This disruption can lead to retrograde ejaculation, where semen enters the bladder instead of being
expelled through the penis during ejaculation.
• Retrograde ejaculation is a known and frequently reported side effect of lumbar sympathectomy.
Incorrect Options:
Option A. Impotence can also occur following lumbar sympathectomy. However, retrograde ejaculation
is considered the more common side effect in this scenario.
Option C. Bladder dysfunction, including issues with bladder control or urinary retention, is less commo
nly associated with lumbar sympathectomy.
Option D. Incontinence does not occur after this procedure.

Solution for Question 77:

Page 70

1238
Correct Option: B
• The correct answer based on the given information is Von Hippel-Lindau (VHL) disease.
• Von Hippel-Lindau disease is an autosomal dominant genetic disorder characterized by the
development of multiple tumors in various organs. It is caused by mutations in the VHL gene, which
plays a role in tumor suppression.
• In the case described, the presence of multiple renal and pancreatic cysts on abdominal ultrasound,
along with a family history of renal problems, is suggestive of Von Hippel-Lindau disease. Additionally,
elevated 24-hour urinary metanephrine levels are not specific to Multiple endocrine neoplasia type 2
(MEN2) but can be seen in VHL disease due to the possible presence of pheochromocytomas.
• VHL disease is associated with the development of various tumors, including hemangioblastomas in
the central nervous system and retina, renal cell carcinomas, pheochromocytomas, pancreatic cysts,
and other tumors in organs such as the adrenal glands, liver, and lungs.
Incorrect options:
Option A: Multiple Endocrine Neoplasia Type (MEN): MEN is a group of genetic disorders that affect m
ultiple endocrine glands, resulting in the development of tumors. There are several types of MEN, inclu
ding MEN1, MEN2A, and MEN2B. Each type is characterized by specific patterns of tumors involving v
arious endocrine glands, such as the parathyroid glands, pituitary gland, adrenal glands, and pancreas.
MEN syndromes are typically inherited in an autosomal dominant manner.
Option C: Birt-Hogg-Dubé Syndrome: Birt-Hogg-Dubé (BHD) syndrome is a rare genetic disorder caus
ed by mutations in the FLCN gene. It primarily affects the skin, lungs, and kidneys. People with BHD sy
ndrome often develop benign skin tumors called fibrofolliculomas, lung cysts, and an increased risk of
developing renal cell carcinoma (kidney cancer). BHD syndrome is inherited in an autosomal dominant
manner.
Option D: Tuberous Sclerosis: Tuberous sclerosis (TS), also known as tuberous sclerosis complex (TS
C), is a genetic disorder characterized by the growth of benign tumors in various organs, including the
brain, heart, kidneys, lungs, and skin. The tumors in TS are called hamartomas and can lead to a wide
range of symptoms and complications, depending on the affected organs. TS is caused by mutations in
the TSC1 or TSC2 genes and can be inherited or occur sporadically.

Solution for Question 78:


Correct option B.
• The patient's symptoms and physical findings are consistent with a varicocele, which is the
enlargement of the veins within the scrotum.
• A varicocele can cause discomfort, heaviness, and a "bag of worms" sensation in the scrotum. The
swelling may reduce in the supine position due to the effect of gravity.
• The most effective treatment for symptomatic varicoceles is varicocelectomy, which involves surgical
ligation or removal of the affected veins.
• Varicocelectomy helps alleviate symptoms, improve fertility in some cases, and prevent potential
complications associated with varicoceles, such as testicular atrophy or infertility.
Incorrect Options:
Option A. Suction drainage: Suction drainage is not the appropriate treatment for a varicocele. It is a
procedure used to remove fluid or pus from a cavity or abscess.

Page 71

1239
Option C. Jaboulay procedure: The Jaboulay procedure is a
surgical technique used for intestinal anastomosis, not for the treatment of varicoceles.
Option D. Herniotomy: Herniotomy refers to the surgical repair of a
hernia, which involves the reduction and closure of a
hernial sac. It is not the recommended treatment for a varicocele.

Solution for Question 79:


Correct Option B.
Percutaneous Nephrolithotomy (PNL): This procedure involves creating a
small incision in the back to access the kidney directly. A nephroscope is then inserted, allowing the su
rgeon to visualize and remove the kidney stone or stones. PNL is considered the gold standard for ma
naging large or complex renal calculi, including the Staghorn type, as it allows for complete clearance o
f the stone burden in a single procedure. The surgeon can use various techniques such as fragmentati
on, suction, and retrieval to remove the stones effectively.
Incorrect Option:
Option A. Extracorporeal Shock Wave Lithotripsy (ESWL): This treatment option uses shock waves to
break down kidney stones into smaller fragments, which can then pass out of the body through urine.
However, ESWL is typically more effective for smaller stones (less than 2 cm) and may not be suitable
for larger stones or complex cases like the Staghorn type, which can be challenging to fragment adequ
ately.
Option C. Ureteroscopic removal: Ureteroscopy involves the use of a thin tube (ureteroscope) that is p
assed through the urethra and bladder to reach the affected ureter or kidney. This procedure is suitable
for smaller stones in the ureter or kidney. However, in the case of a large Staghorn calculus (>4 cm), u
reteroscopic removal alone may not be effective, as the stone's size and complexity make it challengin
g to completely remove through the smaller instrument used in ureteroscopy.
Option D. Retrorenal surgery: Retrorenal surgery is not a commonly used term in the context of managi
ng kidney stones. It is unclear what specific procedure or technique this option refers to. Therefore, it c
annot be considered the best management option for a patient with a large Staghorn calculus.

Solution for Question 80:


Correct Option A.
Immediate exploration on both symptomatic and asymptomatic side: This is the correct option. Testicul
ar torsion is a surgical emergency that requires prompt intervention. Exploring both the symptomatic an
d asymptomatic side is recommended because torsion can occur bilaterally or can progress to involve t
he contralateral testicle. Exploratory surgery allows for immediate assessment of the torsion, detorsion
of the affected testicle, and fixation of both testicles to prevent future torsion episodes.
Incorrect options
Option B. Immediate exploration on the symptomatic side only: This option is incorrect. While immediat
e exploration of the symptomatic side is essential, it does not address the potential involvement of the
contralateral testicle. Failing to explore the asymptomatic side may lead to delayed identification and m
anagement of a potential torsion on that side, which can result in testicular loss.

Page 72

1240
Option C. Delayed exploration on both symptomatic and asymptomatic side: This option is incorrect. D
elaying exploration increases the risk of irreversible testicular damage due to compromised blood flow.
Testicular torsion is a
time-sensitive condition, and any delay in surgical intervention can lead to testicular necrosis and loss.
Option D. Delayed exploration on the symptomatic side only: This option is incorrect. Delaying explorati
on on the symptomatic side only can result in testicular ischemia and loss. As mentioned earlier, testic
ular torsion requires immediate surgical intervention to prevent further damage and preserve testicular
viability.

Solution for Question 81:


Correct option:
Option D.
• Nothing to be done: This is the correct option. If blind-ending testicular vessels are seen during
laparoscopy, it indicates the absence or atrophy of the testis on the right side. In such cases, there is no
further intervention required as the finding suggests that no viable testis is present. However, it is
important to document the absence or atrophy of the testis and discuss the findings with the patient and
their family.
Incorrect option:
Option A. Inguinal exploration: Inguinal exploration involves surgical exploration of the inguinal canal to
locate and bring down the undescended testis. However, in this scenario, if blind-ending testicular ves
sels are seen on the right side during laparoscopy, it indicates that the testis is absent or atrophic (non-
functional). Inguinal exploration would not be beneficial in such cases since there is no testis to bring d
own.
Option B. Abdominal exploration: Abdominal exploration refers to further surgical exploration within the
abdomen to locate the undescended testis. However, if blind-ending testicular vessels are visualized d
uring laparoscopy, it suggests the absence or atrophy of the testis. Additional abdominal exploration w
ould not change this finding and would not be necessary.
Option C. Scrotal exploration: Scrotal exploration involves surgical exploration of the scrotum to locate
the undescended testis. However, if blind-ending testicular vessels are observed during laparoscopy, it
suggests the absence or atrophy of the testis. Scrotal exploration would not yield a
different outcome since the testis is not present or non-functional.

Solution for Question 82:


Correct Option B: Hyponatremia
• Hyponatremia: Hyponatremia, on the other hand, refers to a low level of sodium in the blood. It is a
more probable diagnosis based on the symptoms described in the question, specifically altered
sensitivity and drowsiness. Severe hyponatremia can lead to neurological symptoms, including
confusion, lethargy, and even coma. Transurethral resection of the prostate (TURP) is a surgical
procedure that can disrupt fluid balance and electrolyte levels, potentially leading to hyponatremia.
Incorrect Options:

Page 73

1241
Option A: Hypernatremia: Hypernatremia refers to an elevated level of sodium in the blood. Symptoms
of hypernatremia typically include excessive thirst, dry mucous membranes, restlessness, and confusio
n. However, altered sensitivity and drowsiness are not commonly associated with hypernatremia. Ther
efore, hypernatremia is less likely to be the correct diagnosis in this case.
Option C: Stroke: Stroke is a medical emergency that occurs when the blood supply to the brain is inter
rupted, resulting in neurological deficits. While stroke can present with various symptoms, including alt
ered consciousness and neurological deficits, it is less likely to be the diagnosis in this case, as the sy
mptoms described are more suggestive of an electrolyte disturbance, such as hyponatremia.
Option D: Meningitis due to spinal anesthesia: Meningitis is an inflammation of the protective membran
es surrounding the brain and spinal cord. It can be caused by various infectious agents or, in rare case
s, by non-infectious causes such as drug reactions. While spinal anesthesia is a potential risk factor for
developing meningitis, it is less likely to be the correct diagnosis based solely on the symptoms descri
bed in the question.

Solution for Question 83:


Correct Option C.
Ectopic Ureter: Ectopic ureter is a congenital abnormality where the ureter, which is the tube that carrie
s urine from the kidney to the bladder, does not properly connect to the bladder but instead opens into
other structures such as the urethra, vagina, or urethral opening. In the case described, where the girl
presents with continuous urinary dribbling and fullness of the bladder that she voids, ectopic ureter is th
e most likely diagnosis. The continuous urinary dribbling is due to the abnormal connection of the urete
r, bypassing the sphincter mechanism that normally controls urine flow.
Incorrect Options:
Option A. Stress incontinence: Stress incontinence is the involuntary loss of urine during physical activi
ties or movements that put pressure on the bladder, such as coughing, sneezing, or exercising. It is not
the likely diagnosis in this case, as the continuous urinary dribbling and fullness of the bladder are pre
sent since birth and not associated with specific activities.
Option B. Ureterocele: Ureterocele is a
congenital abnormality characterized by the dilation of the distal end of the ureter, creating a bulge that
obstructs urine flow. While ureterocele can cause symptoms such as urinary frequency, urgency, and r
ecurrent urinary tract infections, it is less likely to present with continuous urinary dribbling and fullness
of the bladder that she voids.
Option D. Vesico vaginal fistula: A vesico vaginal fistula is an abnormal connection between the urinary
bladder and the vagina, resulting in urine leakage into the vagina. While vesico vaginal fistula can cau
se continuous urinary dribbling, it is unlikely to present with the associated symptom of fullness of the b
ladder that she voids.

Solution for Question 84:


Correct Option C: Prostate cancer
The clinical diagnosis based on the given information would be prostate cancer

Page 74

1242
• Prostate cancer: Prostate cancer is a malignant condition that arises from the prostate gland. The
patient's symptoms of low backache, weight loss, progressive difficulty in urination, and urinary
retention episode are consistent with the clinical presentation of prostate cancer. The elevated PSA
level is concerning and, in conjunction with the osteoblastic changes seen on X-ray, further supports
the diagnosis of prostate cancer.
Incorrect Options
Option A: Benign prostatic hypertrophy (BPH): BPH is a non-cancerous condition characterized by the
enlargement of the prostate gland, commonly seen in older men. While BPH can cause lower urinary tr
act symptoms, such as difficulty urinating, it is unlikely to present with weight loss and osteoblastic cha
nges on X-ray. Additionally, the elevated PSA level and urinary retention episode are more indicative of
prostate cancer rather than BPH.
Option B: Carcinoma bladder: Bladder carcinoma primarily affects the urinary bladder and may cause s
ymptoms such as hematuria (blood in urine) and urinary frequency. Although the patient's urinary symp
toms are suggestive of bladder involvement, the elevated PSA level and osteoblastic changes on X-ray
are more consistent with prostate cancer. Bladder carcinoma would typically require additional diagno
stic tests, such as cystoscopy or imaging studies, to confirm the diagnosis.
Option D: Renal cell cancer: Renal cell cancer, or renal cell carcinoma, originates in the cells of the kid
ney. It typically presents with symptoms such as hematuria, flank pain, and a palpable abdominal mass
. The symptoms described by the patient, along with the elevated PSA level and osteoblastic changes
on X-ray, are not characteristic of renal cell cancer.

Solution for Question 85:


Correct Option A: A suprapubic catheter should be inserted as soon as possible.
• The initial management for a patient presenting with a pelvic fracture and membranous urethral
rupture is to insert a suprapubic catheter as soon as possible (Option A).
• Pelvic fractures can often be associated with injuries to the urethra, including membranous urethral
rupture. It is crucial to manage this condition promptly to prevent further complications. The initial step
in management is the insertion of a suprapubic catheter.
• By placing a suprapubic catheter, urine can be diverted from the injured urethra, allowing for adequate
drainage of the bladder. This helps to prevent urinary retention, minimize the risk of urinary
extravasation, and reduce further damage to the urethra. Suprapubic catheterization is considered a
temporary measure to provide urine drainage until definitive treatment can be performed.
Incorrect Options
Option B: IVP should be done: Also called IVP/IVU (Intravenous Pylogram / Intravenous urogram ), this
exam allows the care team to see the parts of the urinary tract and how well it works. This test can hel
p with diagnosis of problems such as kidney stones, enlarged prostate, urinary tract tumors or problem
s present at birth. Access site of obstruction using Intravenous dye injected usually antecubital fossa. U
sed to access the proximal part of obstruction, hydronephrosis and renal function.
Option C: Immediate urethroplasty: Urethroplasty is the fixing of scar tissue in the urine channel from th
e bladder out the penis. Immediate urethroplasty in the setting of Pelvic-fracture associated urethral inj
uries (PFUI) are present in 1.5% –10% of injuries resulting in fracture of the pelvis results in an unacce
ptably high rate of stricture, erectile dysfunction (ED), and incontinence.

Page 75

1243
Option D: MCU should be done: It is an investigation of choice for posterior urethral valve.
MCU (Micturating cysto urethrogram) shows Posterior Urethral Stricture Vesicourethral Reflux (PUV).

Solution for Question 86:


Correct Option B: Ureterocele
• Ureterocele: A ureterocele is a congenital abnormality characterized by the ballooning of the distal
end of the ureter within the bladder. In an IVP, a ureterocele appears as a cystic or dilated structure
within the bladder, often described as a "cobra head" appearance. This appearance is due to the
contrast filling the dilated portion of the ureter within the bladder.
Incorrect Options
Option A: Horseshoe kidney: A horseshoe kidney is a
congenital anomaly where the lower poles of both kidneys are fused. It appears as a
U-shaped or horseshoe-shaped mass on imaging, typically located lower in the abdomen than a
normal kidney. The appearance of a
horseshoe kidney on an IVP would be characteristic of its fused and lower position, rather than a
specific contrast pattern.
Option C: Bladder tumor: A
bladder tumor refers to the abnormal growth of cells in the bladder lining. The appearance of a
bladder tumor on an IVP may vary depending on its size, location, and other factors. It can appear as a
filling defect or irregular mass within the bladder, causing distortion or obstruction of the normal contras
t flow.
Option D: Posterior urethral valve: Posterior urethral valves are a
congenital abnormality in males where there is a flap-like obstruction in the posterior urethra. This cond
ition is typically diagnosed in infancy or childhood. While an IVP may show associated findings, such a
s bladder distention or bilateral hydronephrosis, the specific appearance related to posterior urethral va
lves is not a distinct pattern.

Solution for Question 87:


Correct Option B: Horseshoe kidney
• Based on the information provided, the likely condition indicated by the arrow in the image is
"Horseshoe kidney" (Option B).
• A horseshoe kidney is a congenital anomaly where the lower poles of both kidneys are fused. It
appears as a U-shaped or horseshoe-shaped mass on imaging, typically located lower in the abdomen
than a normal kidney. The arrow in the image likely indicates the presence of a horseshoe kidney.
Incorrect Options:
Option A: Appendicitis: Appendicitis is inflammation of the appendix, typically presenting with right lowe
r abdominal pain. While CECT (contrast-enhanced computed tomography) can help diagnose appendi
citis by showing signs such as appendiceal wall thickening or inflammation, the arrow in the image is n
ot consistent with appendicitis.
Option C: Wilms tumor: Wilms tumor, also known as nephroblastoma, is a type of kidney cancer that pr
imarily affects children. It usually presents as an abdominal mass or swelling. While CECT can be helpf

Page 76

1244
ul in diagnosing and characterizing Wilms tumor, the arrow in the image does not correspond to the typ
ical appearance of this tumor.
Option D: Abdominal aortic aneurysm: An abdominal aortic aneurysm (AAA) is an abnormal dilation of t
he aorta in the abdominal region. It may present with abdominal pain or be incidentally detected on ima
ging studies. While CECT is a common imaging modality to evaluate AAA, the arrow in the image does
not correspond to the characteristic appearance of an AAA.

Solution for Question 88:


Correct Option: A: Peyronie’s disease
• Peyronie's disease: Peyronie's disease is a condition characterized by the development of fibrous
scar tissue, called plaques, within the penis.
• These plaques can cause penile curvature, pain during erection, and difficulty with sexual intercourse.
• The curvature may be upward, downward, or sideways. Peyronie's disease typically develops
gradually and can result in significant distress and difficulty in sexual activity.
Incorrect options:
Option B: Priapism: Priapism is a condition where a prolonged and often painful erection occurs unrelat
ed to sexual stimulation. It is typically unrelated to penile curvature. Priapism is caused by abnormal bl
ood flow in the penis, leading to stagnant blood in the erectile tissues. It is considered a
urological emergency and requires immediate medical attention.
Option C: Marion's disease: Marion's disease is not a
recognized medical condition or term related to the symptoms described. It may be a
typographical error or a misnomer.
Option D: Erectile dysfunction: Erectile dysfunction (ED) refers to the inability to achieve or maintain an
erection sufficient for satisfactory sexual performance. While ED can be associated with pain or disco
mfort, the primary symptom is the difficulty in achieving or sustaining an erection rather than abnormal
penile curvature.

Solution for Question 89:


Correct Option D:
• Erythrocytosis: Erythrocytosis refers to an elevated red blood cell count. While it is not a typical
feature of PKD, in some cases, PKD can lead to the development of renal cell carcinoma, which may
cause erythrocytosis as a paraneoplastic syndrome. However, this is not a common finding in most
cases of PKD.
Incorrect Options:
Option A: Hematuria: Hematuria, which is the presence of blood in the urine, can occur in PKD due to t
he rupture or bleeding within the cysts. It is a common symptom of PKD.

Page 77

1245
Option B: Hypertension: High blood pressure is a common feature of PKD. The cysts in the kidneys ca
n interfere with normal kidney function, leading to increased renin secretion and subsequent hypertensi
on.
Option C: Renal Failure: PKD is a progressive condition that can eventually lead to kidney failure. The
numerous cysts gradually replace the healthy kidney tissue, impairing kidney function over time.

Solution for Question 90:


Correct Option B: Bladder calculi
• The X-ray image shows radio-opaque shadows within the pelvis, which are consistent with bladder
calculi. Bladder calculi are stones that form in the bladder.
• They can cause symptoms such as urinary frequency, urgency, hematuria (blood in the urine), and
lower abdominal pain.
• In X-ray images, bladder calculi appear as radiopaque shadows due to their mineral composition,
which can be visualized within the pelvis.
Incorrect Options:
Option A: Renal calculi: Renal calculi, also known as kidney stones, are formed within the kidneys. Whi
le kidney stones can cause similar symptoms to bladder calculi, such as hematuria and abdominal pain
, they would typically appear higher up in the urinary tract on an X-ray image. The presence of radio-op
aque shadows in the bladder, as shown in the X-ray, suggests the location of the stones within the blad
der rather than the kidneys.
Option C: Schistosomiasis: Schistosomiasis is a parasitic infection caused by Schistosoma parasites. It
primarily affects the urinary and intestinal systems. While schistosomiasis can lead to the formation of
bladder stones, the X-ray image alone cannot confirm the presence of schistosomiasis. Diagnosis of sc
histosomiasis typically involves stool or urine tests to detect the parasite's eggs.
Option D: Ureteric calculi: Ureteric calculi, or ureteral stones, are stones that form within the ureter, the
tube connecting the kidneys to the bladder. They can cause severe flank pain and blood in the urine. H
owever, the X-ray image provided does not show the stones in the ureter. The radio-opaque shadows
are localized to the pelvis, indicating their presence within the bladder.

Solution for Question 91:


Correct Option B:
• Renal calculi, also known as kidney stones, can be composed of different types of minerals. Each
type of stone has its own characteristics and may respond differently to treatment methods such as
Extracorporeal Shock-Wave Lithotripsy (ESWL).
• Cystine: Cystine stones are formed due to a genetic disorder called cystinuria, which affects the
reabsorption of cystine in the kidneys. These stones are composed of cystine, an amino acid. Cystine
stones are typically resistant to ESWL due to their hard and dense nature.
Incorrect Options:
Option A. Struvite: Struvite stones, also called infection stones, are composed of magnesium ammoniu
m phosphate. These stones can grow rapidly and are often associated with urinary tract infections. Str

Page 78

1246
uvite stones are generally amenable to ESWL treatment.
Option C. Calcium oxalate dihydrate: Calcium oxalate stones are the most common type of kidney ston
es. They can occur in different forms, including calcium oxalate dihydrate. These stones can vary in the
ir response to ESWL, with some calcium oxalate stones being successfully fragmented with shock wav
es. However, the effectiveness of ESWL may depend on the size, location, and composition of the ston
e.
Option D. Uric Acid: Uric acid stones form due to high levels of uric acid in the urine. They can be disso
lved or fragmented using ESWL. Uric acid stones are generally responsive to ESWL treatment.
Based on the information provided, the correct answer is B. Cystine stones are known to be resistant t
o ESWL due to their hard and dense composition.

Solution for Question 92:


Correct Option A:
• High inguinal orchidectomy: This is the recommended initial step in the management of testicular
tumors suspected to be malignant. It involves the surgical removal of the affected testicle through a
high inguinal incision. The excised testicle is sent for histopathological examination to confirm the
diagnosis and determine the exact type of testicular cancer.
Incorrect Options:
Option B. Core needle tract biopsy: Core needle tract biopsy is not the preferred option in this scenario.
Testicular tumors are usually managed by surgical excision (orchidectomy) rather than biopsy due to t
he risk of tumor dissemination along the needle tract.
Option C. FNAC (Fine-needle aspiration cytology): FNAC is not the preferred option for testicular tumor
s. It is less commonly used for testicular masses as it may not provide sufficient tissue for accurate hist
opathological examination and diagnosis.
Option D. Whole-body PET/CT: While whole-body PET/CT can be useful in staging and detecting meta
stasis in testicular cancer, it is not the initial step in management. Orchidectomy is performed first for hi
stopathological diagnosis, and subsequent staging and imaging studies may be performed to assess th
e extent of the disease.

Solution for Question 93:


Correct Option B:
• The best management option for a 3 cm stone in the renal pelvis without evidence of hydronephrosis
is PCNL (Percutaneous Nephrolithotomy) (Option B).
• Percutaneous Nephrolithotomy (PCNL) is a minimally invasive surgical procedure used to remove
large kidney stones. It involves making a small incision in the back and creating a tract into the kidney
to access and remove the stone. PCNL is particularly effective for larger stones, typically those larger
than 2 cm.
• In the case of a 3 cm stone in the renal pelvis, PCNL is the preferred treatment option because it
allows for direct access to the stone and provides effective clearance. The procedure involves using

Page 79

1247
specialized instruments, such as a nephroscope and laser or ultrasound energy, to break up the stone
into smaller fragments and remove them. The surgeon can visualize the stone and surrounding
structures to ensure complete stone clearance.
Incorrect Options:
Option A: Extracorporeal Shock Wave Lithotripsy (ESWL): ESWL uses shock waves to break up kidne
y stones from outside the body. However, it is generally more suitable for smaller stones (less than 2
cm) and may be less effective for larger stones like the 3 cm stone in the renal pelvis mentioned.
Option C: Antegrade Pyeloplasty: Antegrade pyeloplasty is a
surgical procedure used to treat ureteropelvic junction (UPJ) obstruction, which is a blockage at the jun
ction between the renal pelvis and the ureter. It is not the primary treatment for kidney stones and is no
t specifically indicated for stone removal.
Option D: Retrograde Pyeloplasty: Retrograde pyeloplasty is another surgical procedure used to treat
UPJ obstruction. Similar to antegrade pyeloplasty, it is not the primary treatment for kidney stones and
is not specifically indicated for stone removal.

Solution for Question 94:


Correct Option C:
• Germ cell tumor not seen in males: c. Sertoli cell tumour
• Sertoli cell tumors are sex cord-stromal tumors and are typically seen in the testes. They arise from
the Sertoli cells, which are supportive cells in the seminiferous tubules of the testes. Sertoli cell tumors
are rare and are almost exclusively found in males.
• Choriocarcinoma, seminoma, and teratoma are all germ cell tumors that can occur in males.
Choriocarcinoma is rare in males but can occur in the testes or other sites. Seminoma is the most
common type of testicular germ cell tumor, and teratoma is another type that can occur in the testes.
Incorrect Options:
Options A. Choriocarcinoma: Choriocarcinoma is a type of germ cell tumor that can occur in both male
s and females. It arises from trophoblastic cells, which are normally found in the placenta. In males, ch
oriocarcinoma can occur in the testes as a part of testicular germ cell tumors.
Options B. Seminoma: Seminoma is a type of germ cell tumor that primarily occurs in males, specificall
y in the testes. It is one of the most common types of testicular cancers. Seminomas typically arise fro
m the germ cells in the testes.
Options D. Teratoma: Teratoma is another type of germ cell tumor that can occur in both males and fe
males. It can develop in various organs and tissues, including the testes. Testicular teratomas are mor
e commonly seen in infants and young children but can also occur in adults.

Solution for Question 95:


Correct Option is D. 4 hours.
Priapism is a prolonged and painful erection that is unrelated to sexual stimulation. It occurs when ther
e is persistent engorgement of the corpora cavernosa of the penis, leading to inadequate drainage of bl
ood. Priapism can be caused by various factors, including medications such as antipsychotics.

Page 80

1248
The duration of priapism is an important factor in its diagnosis and management. According to the guid
elines, priapism is typically classified based on the duration of the erection:
• Ischemic priapism: Lasts longer than 4 hours and is considered a urologic emergency. It is associated
with low-flow or veno-occlusive priapism, which can result in tissue ischemia and damage if not
promptly treated.
• Non-ischemic priapism: Lasts less than 4 hours and is usually not associated with tissue ischemia. It
is commonly seen in conditions such as high-flow or arterial priapism.
Therefore, in the given scenario, since the patient has a painful erection without sexual drive and is on
antipsychotic medication, if the duration of the erection exceeds 4
hours, it would be considered priapism and would require immediate medical attention.
Incorrect Options:
Priapism is a condition characterized by a prolonged and often painful erection of the penis that persist
s beyond or is unrelated to sexual stimulation. It is typically unrelated to sexual desire or arousal and is
considered a
medical emergency. Priapism can occur in males of all ages, including children and adults.
There are two main types of priapism:
• Ischemic priapism (low-flow priapism): This is the most common type of priapism, accounting for
about 95% of cases. It occurs when blood becomes trapped in the erectile tissues of the penis, leading
to reduced oxygen supply and ischemia. Ischemic priapism is often painful and can last for several
hours or more if left untreated. It is typically associated with conditions such as sickle cell disease,
leukemia, trauma to the penis, certain medications (e.g., antipsychotics), and drug use (e.g., cocaine).
• Non-ischemic priapism (high-flow priapism): This type of priapism is less common and usually not
associated with pain. It occurs due to abnormal blood flow into the erectile tissues, usually from an
arterial source. Non-ischemic priapism is typically caused by an injury or trauma to the penis, such as a
pelvic fracture.

Solution for Question 96:


Correct Option A:
• The Bell clapper deformity refers to a condition in which the testis lacks proper fixation within the
scrotum, allowing it to rotate freely on the spermatic cord. This anatomical abnormality predisposes the
individual to testicular torsion, which is the twisting of the testicle and its associated structures.
Testicular torsion can lead to ischemia (lack of blood flow) and if not promptly treated, it can result in
testicular damage or loss. Therefore, the correct answer is A. Torsion testis.
Incorrect Options:
Option B. Varicocele: Varicocele is a condition characterized by the enlargement of veins within the scr
otum. It is not directly related to the Bell clapper deformity. Varicocele is more commonly associated wi
th the impairment of venous drainage in the testicles.
Option C. Cancer of testis: The Bell clapper deformity itself does not predispose to testicular cancer. T
esticular cancer typically arises from abnormal growth of cells within the testicles and is influenced by v
arious factors such as genetic predisposition, previous testicular abnormalities, and environmental fact
ors.
Option D. Hydrocele: Hydrocele is a condition characterized by the accumulation of fluid around the tes
ticle, leading to scrotal swelling. It is not directly related to the Bell clapper deformity. Hydrocele can oc

Page 81

1249
cur due to various reasons, including trauma, infection, or underlying testicular abnormalities.

Solution for Question 97:


Correct Option D - Partial penectomy:
• Verrucous carcinoma of the penis, aka Buschke-Loewenstein tumor a premalignant condition of
squamous cell carcinoma penis, is a local malignant tumor destroying surrounding tissue by
compression, not invasion.
• It is best managed through partial penectomy, which aims to remove the cancerous portion while
preserving as much normal function as possible.

Incorrect Choices:
Option A
- Total penectomy: Total penectomy is generally reserved for carcinoma penis involving proximal part.
Option B - CO2 laser excision: A CO2 laser might is used in some penile intra-epithelial neoplasia but i
s not typically the primary therapeutic approach for this type of carcinoma.
Option C - Topical 5-fluorouracil: Topical 5-fluorouracil is a chemotherapy agent used in stage 0
or precancerous conditions of cancer. It is not the most appropriate management.

Page 82

1250
Arterial Disorders
1. What is the most likely diagnosis in a patient with new onset diffuse abdominal pain lasting for 1
hour, a history of non-compliant peptic ulcer disease, severe gastroesophageal reflux, and the CT
angiography finding shown in the image below?
(or)
A 35-year-old male patient presented to the emergency department with new onset diffuse abdominal
pain for one hour. The patient has had a history of peptic ulcer disease over the past ten years, which is
treated with a proton pump inhibitor and histamine receptor blocker. The patient is not compliant with
treatment and has had severe gastroesophageal reflux for the last year. On physical examination, the
patient had severe diffuse abdominal pain, tenderness, and an altered mental status. Vital signs reveal
a temperature of 37.9 C (100.2 F), blood pressure of 105/69 mmHg, pulse pressure of 100/min, and
respiratory rate of 12/min. A computed tomography (CT) angiography is ordered, which reveals the
following finding as shown in the image: What is the most likely diagnosis for this patient?

A. Peritonitis
B. Mesenteric ischemia
C. Diverticulitis
D. Barrett esophagus
----------------------------------------
2. A 72-year-old female came to the medical OPD with elevated blood pressure and type II diabetes
mellitus. She underwent angiography because of a painful, non-healing ulcer over the ankle of the left
leg. The angiography showed arterial occlusive disease. Which of the following is the best for treating
arterial occlusive diseases in this case?
(or)
What is the preferred treatment for arterial occlusive disease in an old female with a painful,
non-healing ulcer over the ankle of the left leg with hypertension, diabetes, and an arterial occlusive
disease?
A. Elevation of the affected extremity
B. Revascularize the area along with debridement
C. Oxygen therapy
D. Heart transplantation
----------------------------------------

1251
3. A 19-year-old female student presented in to the emergency, complaining of numbness in her left
toe. She had a 3-year smoking history. Angiography revealed peripheral arterial occlusion of both legs
and arms. A detailed laboratory examination excluded collagen disease, hypercoagulable state, and
juvenile atherosclerosis. A below knee amputation of the left leg was performed. Typical histologic
findings of Buerger’s disease were observed in the crural arteries and saphenous veins. Which of the
following structures is affected by the disease?
(or)
Which of the following structures is affected by the Buerger's disease?
A. Small vessels
B. Muscles
C. Bones
D. Skin
----------------------------------------
4. An 18-year-old college student presented to the outpatient department with a complaint of pain and
paraesthesia on the right arm. The hands were cold and clammy. She also complained of weakness in
her hand while writing. On examination, blood pressure was 120/80 mmHg, and the pulse was 82 beats
per minute. There was no history of cyanosis. She was fully vaccinated. Adson’s test was positive.
What is the probable diagnosis?
(or)
What is the probable diagnosis in an college student presenting with pain and paraesthesia on the right
side of the arm, cold and clammy hands, weakness in hand while writing, and a positive Adson's test?

A. Peripheral vascular disease


B. Varicose veins
C. Cervical rib
D. AV fistula
----------------------------------------
5. What is the most probable diagnosis in a male who underwent surgery for aortic aneurysm and now
presents with dryness of the face on the side of the incision and constricted pupils?
(or)
A 45-year-old male underwent surgery for an aortic aneurysm four days ago. During the surgery, a
structure passing close to the vertebrae was cut. The patient came for a follow-up. At the time of the
visit, the patient's blood pressure was 150/90 mmHg and his pulse was 89 beats per minute. The

Page 2

1252
patient complains of dryness on one side of his face. Pupils are constricted. What is the most probable
diagnosis?
A. Intermittent claudication
B. Horner syndrome
C. Wet Gangrene
D. Arterial ulcers
----------------------------------------
6. A 25-year-old male presented to the outpatient department complaining of pain and numbness on
the medial aspect of his right arm. On examination, wasting of the thenar and hypothenar eminence
was noted. The adduction and abduction of fingers were restricted. The hands were cold and
cyanosed. The radial pulse on the right side was diminished. The left hand appeared normal. A chest
X-ray showed a cervical rib. Which of the following is the commonest feature of thoracic outlet
syndrome?
(or)
What is the commonest feature of thoracic outlet syndrome?
A. Intermittent claudication
B. Pain in radial distribution
C. Pain in ulnar distribution
D. Gangrene
----------------------------------------
7. Which patient complaint in a man presenting with cramp-like pain, redness, and swelling after
walking, absent pedal pulses, and a positive sunset foot sign indicates the need for emergency
intervention to prevent limb amputation?
(or)
A 43-year-old man came to the OPD complaining of severe pain in his both legs for the past 20 days.
On history-taking, the patient complained of cramp-like pain felt in the muscles after walking along with
redness and swelling, but for the past one day, he's had severe pain in his foot even while he's resting,
which is exacerbated even by lying down and by touching his bed sheet. With a positive sunset foot
sign, there are no pedal pulses. Which of the following patient complaints will necessitate emergency
intervention to prevent limb amputation?
A. Intermittent claudication
B. Resting pain
C. Swelling
D. Redness
----------------------------------------
8. A patient presented to the ER with complaints of severe chest pain, breathlessness, fatigue, and
palpitations for 2 hours. On his ECG, Q waves were noted. There is also elevated troponin I levels. He
was prepared for coronary angiography, and on that, 60% stenosis of the left main stem was noted.
The doctors are planning a coronary artery bypass graft (CABG) for him and have decided to use a
radial artery for grafting. Which of the following tests should be performed before using a radial artery?
(or)

Page 3

1253
Which test should be performed before using a radial artery for grafting in a patient undergoing
coronary artery bypass graft (CABG)?
A. Tinel’s test
B. Allen’s test
C. Phalen’s test
D. Froment’s test
----------------------------------------
9. What is the best way to arrest the development of disease for a male, presenting with progressive
worsening ulcers in the fingertips of both hands and feet, intermittent claudication, a history of migratory
superficial thrombophlebitis, and a history of smoking 20 packs per year?
(or)
A 35-year-old male presented to the OPD with the complaint of progressively worsening ulcers in the
fingertips of both hands and feet for 4 years, along with intermittent claudication of the feet whenever
he is walking. After taking history, he told the doctor that he had "migratory superficial
thrombophlebitis." He is a 20-pack-year smoker who is not a diabetic; his cholesterol levels are normal
according to him, and his ECG findings were normal. What would be the best way for this patient to
arrest the development of his disease?
A. Aspirin
B. Good glycaemic control
C. Blood cholesterol control
D. Smoking cessation
----------------------------------------
10. A 12-year-old boy presented in the ER with a complete transaction midway through his right arm
due to a motor vehicle accident 2 hours back. The boy arrived at the emergency room with an
amputated extremity in a bag with no cooling measures. After initial resuscitation, he was immediately
transferred to the operating room. What would be the ischemic time for his limb in this condition, after
which the limb cannot be re-implanted?
(or)
What is the maximum ischemic time for successful re-implantation of a limb in a boy with a complete
transection of his right arm due to a motor vehicle accident 2 hours ago?
A. 8 hours
B. 4 hours
C. 8-12 hours
D. 24 hours
----------------------------------------
11. A 40-year-old male patient presented to the OPD with a complaint of pain in his right arm that was
relieved by elevation. On examination, a prominent, large, and swollen vein was noted in the arm with a
palpable thrill; the patient's blood pressure was 90/70 and his heart rate was 140 bpm. The doctor
performed a test in which he compressed the proximal part of the artery present before the prominent
vein, which resulted in a decrease in swelling of the vein, the disappearance of the thrill, an elevation of
blood pressure to 120/90, and a decrease in heart rate to 90. What is the name of this sign?

Page 4

1254
(or)
Name the sign in which compression of the proximal part of the artery present before the prominent
vein, results in a decrease in swelling of the vein, the disappearance of the thrill, elevation of blood
pressure and a decrease in heart rate ?
A. Murray sign
B. Bancroft’s sign
C. Darier sign
D. Branham sign
----------------------------------------
12. A 9-year-old female patient was brought to the clinic because of skin color changes from white to
blue to red in symmetrically in bilateral hands and feet with six months of progress. Symptoms occur
once or twice a week, sometimes fortnightly, and are triggered by cold. They usually last from 10 to 15
min with spontaneous resolution. What would be the most probable diagnosis?
(or)
What is the most probable diagnosis for a female child with vasospastic phenomena in hands and feet,
triggered by cold, and associated with skin colour changes from white to blue to red, lasting 10 to 15
minutes with spontaneous resolution?
A. Raynaud’s Disease
B. Acrocyanosis
C. Arteriovenous Fistula
D. Subclavian Steal Syndrome
----------------------------------------
13. What is the probable diagnosis for an old male with hypertension, sudden-onset excruciating
central chest pain radiating to the back, and unequal blood pressure in bilateral upper limbs, as shown
in the thoracic MRA image below?
(or)
A 62-year-old male who is a known case of hypertension for 30 years and ischemic heart disease
presented to the emergency department with complaints of sudden-onset excruciating central chest
pain radiating to the back, associated with shortness of breath and dizziness. He has unequal blood
pressure in the bilateral upper limbs. A thoracic MRA was performed as shown below. What could be
the probable diagnosis?

A. Subclavian steal syndrome

Page 5

1255
B. Thoracic aortic aneurysm
C. Atherosclerosis
D. Cardiac tamponade
----------------------------------------
14. A 77-year-old male with ischemic heart disease for the past 23 years presented to the OPD with
complaints of severe headache, fever, intensive perspiration, and scalp pain. On examination, he had
swollen eyelids and thickened nodular bilateral temporal arteries. He also gave a history of jaw
claudication, particularly during chewing, for which his molar was extracted, but the pain remained after
extraction. What is the most probable diagnosis?
(or)
What is the most probable diagnosis for an old male with severe headache, fever, intensive
perspiration, scalp pain, swollen eyelids, thickened nodular bilateral temporal arteries, with history of
jaw claudication?
A. Wegener's granulomatosis
B. Takayasu's arteritis
C. Giant cell arteritis
D. Polyarteritis nodosa
----------------------------------------
15. What is the pathophysiological effect of creating an arteriovenous fistula in an old man prepared for
dialysis?
(or)
A 74-year-old man with chronic kidney disease stage 3A who has developed a decline in glomerular
filtration rate (GFR) after the initiation of warfarin therapy due to atrial fibrillation is referred for dialysis.
The patient has been admitted and is prepared to initiate dialysis. What is the pathophysiological effect
of creating an AV fistula?
A. Sinus bradycardia
B. Decrease in preload
C. Cardiac arrhythmias
D. Increased cardiac output
----------------------------------------
16. A 50-year-old male with varicose veins in the lower limbs has developed itching and ulceration
around the ankle and calf region for a few days. Which type of ulcer is likely to develop in a
long-standing chronic venous ulcer?
(or)
Which type of ulcer is likely to develop in a long-standing chronic venous ulcer in a male with varicose
veins of the lower limb who has developed itching and ulceration around the ankle and calf region?

Page 6

1256
A. Marjolin’s ulcer
B. Aphthous ulcer
C. Pressure sores
D. Necrotizing fasciitis
----------------------------------------
17. A 60-year-old man presents to the surgical emergency department with the acute onset of right leg
numbness and pallor. He has a history of hypertension for 20 years and a 40-pack-year smoking
history. He also complains about long-standing burning and cramping in his hips and buttocks after
walking about 50 yards or climbing two to three flights of stairs and impotence and erectile dysfunction.
On examination, his blood pressure is 156/92 mmHg and his pulse is 89 per minute. His lower
extremities have no palpable pulses, including absent femoral pulses. Which of the following is the
likely cause of these symptoms?
(or)
What is the likely cause of acute onset right leg numbness, pallor, long-standing burning and cramping
in the hips and buttocks after walking, impotence, erectile dysfunction, and absence of palpable pulses
in the lower extremities, including the femoral arteries?
A. Aortoiliac occlusive disease
B. Common femoral occlusive disease
C. Superficial femoral occlusive disease
D. Popliteal artery occlusive disease
----------------------------------------
18. The catheter shown in the image is used for:
(or)
A 50-year-old female came into the emergency department with complaints of shortness of breath,
rapid breathing, and wheezing. She was coughing along with bloody sputum. She was also having
sharp chest pain and back pain. Upon taking her medical history, it was found out that she had gone
through a craniotomy 2 months ago, and now she is suspected of having a pulmonary embolism. The
catheter shown in the image is used for:

Page 7

1257
A. Stop variceal bleeding from blood vessels
B. Parenteral hyperalimentation
C. Removal of an embolus from blood vessels
D. Ureteric catheterization
----------------------------------------
19. A 35-year-old male has come to the clinic with breathing difficulty, hoarseness of voice, and
difficulty swallowing for two months. He has been diagnosed with Marfan’s syndrome since childhood.
His chest X-rays and CT scan are done. The X-rays show space-occupying lesions in the thorax. The
radiography shows the dilatation of the aorta, which is 7cm, compressing the nearby structures too.
What is the most probable diagnosis, and what is the treatment required in this case?
(or)
What is the most probable diagnosis and the required treatment for a young male diagnosed with
Marfan's syndrome presenting with breathing difficulty, hoarseness of voice, difficulty swallowing, and a
7cm aortic aneurysm compressing nearby structures on radiography?
A. Thoracic aorta aneurysms, no surgery required
B. Abdominal aorta aneurysm, no surgery required
C. Abdominal aorta aneurysm, graft surgery
D. Thoracic aorta aneurysms, graft surgery
----------------------------------------
20. What is the diagnosis and the preferred graft for repairing aortic dissection in a young female
presenting with chest pain, cough, difficulty breathing and swallowing, and a computed tomography
revealing a tear in the aorta?
(or)
A 35-year-old female was brought to the emergency department because of excruciating chest pain,
described as a sudden and stabbing retrosternal pain that also radiates to the back. She also
complained of a cough and difficulty breathing and swallowing. Two days earlier, the patient underwent
a cesarean delivery. Her pregnancy was complicated by moderate hypertension. Computed
tomography revealed an aortic tear in the diagnostic algorithm for acute thoracic pain. What is the
diagnosis, and what is the best graft for this patient to repair?
A. Type A aortic dissection, PET graft
B. Type B aortic dissection, PTFE graft

Page 8

1258
C. Type B aortic dissection, PET graft
D. Type A aortic dissection, PTFE graft
----------------------------------------
21. A 50-year-old male patient presented to the emergency department with sudden onset sharp
tearing chest pain that started 1 hour ago. The patient had a history of hypertension and a 40-pack-year
smoking history. Family history is also significant for Marfan syndrome. On physical examination, lungs
are clear to auscultation, and the heart sounds are normal. Vital signs reveal a temperature of 36.7 C
(98.1 F), blood pressure is 170/95 mmHg in both arms, pulse is 108/min and regular, and respiration
rate 18/min. A chest x-ray reveals widened mediastinum. The CT scan is shown below: Which of the
following is the most likely diagnosis in this patient?
(or)
What is the likely diagnosis in a middle aged male with sudden onset tearing chest pain, a history of
hypertension and 40 packs per year cigarette, significant family history of Marfan syndrome, clear lungs
on auscultation , and a widened mediastinum on chest x-ray?

A. Aortic dissection
B. Thoracic aorta aneurysm
C. Angina
D. Heart failure
----------------------------------------
22. A 50-year-old male patient presented with complaints of pain, fatigue, and paresthesia in his left
arm for the last nine months. The patient also had dizziness for the past six months, which worsen with
upper-extremity activity or exercise. The patient had a history of hypertension and diabetes mellitus. On
physical examination, there is a systolic bruit in the left supraclavicular fossa. Vital signs are include
blood pressure of 160/90 mm Hg, pulse pressure of 65/min and respiration rate of 18/min. Coronal
magnetic resonance is ordered, which reveals the following finding as shown in the image: What is the
most likely diagnosis for this patient?
(or)
What is the most likely diagnosis in a middle aged male patient with complaints of pain, fatigue,
paresthesia in the left arm, dizziness, worsened by upper extremity activity or exercise, and the coronal
magnetic resonance imaging finding shown in the image?

Page 9

1259
A. Angina
B. Subclavian steal syndrome
C. Aortic dissection
D. Heart failure
----------------------------------------
23. An 80-year-old female was admitted to a hospital for severe fatigue after minimal exertion for two
years. Her haemoglobin was 5.7 mg/dL and transfusion was initiated.The haematological profile
showed iron-deficiency anaemia and a normal bone marrow. Fecal occult blood tests were positive,
and an upper gastrointestinal endoscopy and a colonoscopy were performed. Non-hemorrhagic
angiodysplasia in the cecum was the found. Echocardiography demonstrated severe aortic stenosis
and a mean pressure gradient of 62 mm Hg (peak 140 mm Hg). What is your probable diagnosis?
(or)
What is the probable diagnosis in an old female presenting with severe fatigue on minimal exertion, 5.7
mg/dL hemoglobin, iron-deficiency anaemia, positive fecal occult blood tests, non-hemorrhagic
angiodysplasia in the cecum, and severe aortic stenosis with high pressure gradients on
echocardiography?
A. Char syndrome
B. Heyde syndrome
C. Williams-Beuren syndrome
D. Angelman syndrome
----------------------------------------
24. A 62-year-old male with hypertension for 20 years but not compliant with medications comes to the
emergency room with sharp chest pain radiating to the intra-scapular region. He also has back and
abdominal pain radiating to his leg. On examination, the lower limb shows pallor and weak pulses. His
CT and MRI of the abdomen show a tear in the wall of the abdomen. He has smoked 20 packs per
year. What is the most probable diagnosis in this case?
(or)
What is the most probable diagnosis in an old male with hypertension presenting with sharp chest pain
radiating to the intra-scapular region, pallor and weak pulses in the lower limb, and a CT and MRI
showing a tear in the wall of the abdomen?
A. Ascending aortic dissection
B. Abdominal aortic aneurysms

Page 10

1260
C. Descending aortic dissection
D. Thoracic aortic aneurysms
----------------------------------------
25. An 18 year old male, smoker, non-alcoholic, and non-diabetic, was admitted to the hospital with
complaints of progressive aching pain while walking in both legs, which was not relieved even by taking
rest for the last three years. After a thorough examination, the patient was diagnosed with
thromboangiitis obliterans, and a lumbar sympathectomy was recommended to relieve the constant
pain. In a lumbar sympathectomy, the sympathetic chain in its usual position is likely to be confused
with which of the following?
(or)
In lumbar sympathectomy, the sympathetic chain in its usual position is likely to be confused with which
of the following?
A. Ilioinguinal nerve
B. Abdominal aorta
C. Posterior root of brachial plexus
D. Psoas major and lymphatic chain
----------------------------------------
26. A 30-year-old male presents to the clinic with atrophy of the intrinsic hand muscles, ischemic pain,
and edema of the unilateral hand. The patient has hoarseness and ipsilateral miosis. The patient was
asked to abduct the shoulder to 90º with external rotation and to flex the elbows to 90º. Then he was
asked to slowly open and close his hand for 3 minutes. Which of the following tests would indicate
these manoeuvres?
(or)
The patient was asked to abduct the shoulder to 90º with external rotation and to flex elbows to 90º.
Then he was asked to slowly open and close his hand for 3 min. Which of the following test would
indicate these manoeuvres?

A. Adson test
B. Elvey test
C. Wright test
D. Roos test
----------------------------------------

Page 11

1261
27. Which of the following will help in determining the need for hospitalization in a newborn girl with
cyanosis of the hands and feet but pink skin elsewhere?
(or)
A newborn girl is undergoing an examination in the delivery room following her birth. She was born via
spontaneous vaginal delivery at 35 weeks' gestation. Her heart rate is 111 beats per minute and her
respiratory rate is 35 beats per minute; she cries vigorously when stimulated. The patient has cyanosis
of the hands and feet, but the remaining of the skin is pink, as shown in the exhibit. Which of the
following would help in determining if the patient needs hospitalization?

A. X-ray of the chest


B. ECG
C. Pulse oximetry
D. 100% oxygen
----------------------------------------
28. What is the most likely diagnosis in a middle aged man with complaints of pain in calf muscles while
walking, aggravated by exercise and relieved by rest, weak peripheral pulses, and the presence of an
audible bruit on examination?
(or)
A 55-year-old known diabetic and hypertensive man presents to the OPD with the complaint of pain in
the calf muscles while walking for the last few weeks. Pain is aggravated by exercise and climbing
stairs and relieved by rest, either standing or sitting. On examination, there are weak peripheral pulses
and an audible bruit is present. There are no skin changes. What is the most likely condition he is
suffering from?
A. Intermittent claudication
B. Buerger’s disease
C. Frostbite
D. Acrocyanosis
----------------------------------------

Correct Answers
Question Correct Answer

Question 1 2

Page 12

1262
Question 2 2
Question 3 1
Question 4 3
Question 5 2
Question 6 3
Question 7 2
Question 8 2
Question 9 4
Question 10 2
Question 11 4
Question 12 1
Question 13 2
Question 14 3
Question 15 4
Question 16 1
Question 17 1
Question 18 3
Question 19 4
Question 20 1
Question 21 1
Question 22 2
Question 23 2
Question 24 3
Question 25 4
Question 26 4
Question 27 3
Question 28 1

Solution for Question 1:


Correct Option B - Mesenteric ischemia:
• The symptoms of sudden sever abdominal pain with CT angiography showing findings of obstruction
in the mesenteric arterial supply is suggestive of mesenteric ischemia.
• It is small bowel necrosis due to blockage of the superior mesenteric artery (often due to embolism)
• It causes abdominal pain, which is out of proportion to physical finding.
• On bowel movements, the patient has currant jelly stools.

Page 13

1263
• On CT scan, string of sausages formed due to ischemia in the wall of the bowel connected through
mesentery.
Incorrect Options:
Option A - Peritonitis:
• It is an inflammation of the peritoneum, which lines the inner walls of the abdomen and provides
support to the abdominal organ.
• If left untreated, it can lead to sepsis.
• Obstruction and perforation of hollow viscous organs in the abdomen is a major cause.
• Signs and symptoms Fever > 37.8 C Chills Vomiting Abdominal pain/tenderness and distention Gut
fluid in the abdominal cavity Difficulty having bowel movements Passing less than usual or not passing
urine.
• Fever > 37.8 C
• Chills
• Vomiting
• Abdominal pain/tenderness and distention
• Gut fluid in the abdominal cavity
• Difficulty having bowel movements
• Passing less than usual or not passing urine.
• Diagnosis Blood and urine test X-ray and computed tomography (CT) scan Exploratory surgery
• Blood and urine test
• X-ray and computed tomography (CT) scan
• Exploratory surgery
• Fever > 37.8 C
• Chills
• Vomiting
• Abdominal pain/tenderness and distention
• Gut fluid in the abdominal cavity
• Difficulty having bowel movements
• Passing less than usual or not passing urine.
• Blood and urine test
• X-ray and computed tomography (CT) scan
• Exploratory surgery
Option C - Diverticulitis:
• Diverticulitis is inflammation of the diverticulum, which is a blind pouch protruding from the
gastrointestinal tract.
• Patient presents with left lower quadrant pain (LLQ), fever, and leukocytosis.
• It causes complications, i.e. abscess, obstruction, perforation, and colovesical fistula.

Page 14

1264
Option D - Barrett's esophagus:
• Barrett's esophagus is intestinal metaplasia that replaces the non-keratinised squamous epithelium of
the lower esophagus with non-ciliated columnar goblet cells.
• It occurs in patients with chronic gastroesophageal reflux disease (GERD).
• It increases the risk of esophageal adenocarcinoma.

Solution for Question 2:


Option B: Revascularize the area along with debridement
• Treatment of arterial ulcers includes vascular surgery to revascularize the area, debridement of
devitalized tissue, antibiotic administration and bed rest.
• The arterial occlusive disease occurs when the arteries throughout the body gradually narrow. It can
affect the arms and legs.
• Arterial insufficiency ulcers and ischemic ulcers mostly affect the lateral surface of the ankle or the
distal digits.
• Characteristic features: Punched-out appearance. Pulses are not palpable. Associated skin changes
(thin shiny skin, absence of hair, brittle nails)
• Punched-out appearance.
• Pulses are not palpable.
• Associated skin changes (thin shiny skin, absence of hair, brittle nails)
• Diagnosis: The lesion can be easily identified clinically. Arterial doppler and pulse volume recording
for baseline assessment of blood flow. Radiography may be necessary to rule out osteomyelitis.
• The lesion can be easily identified clinically.
• Arterial doppler and pulse volume recording for baseline assessment of blood flow.
• Radiography may be necessary to rule out osteomyelitis.
• Punched-out appearance.
• Pulses are not palpable.
• Associated skin changes (thin shiny skin, absence of hair, brittle nails)
• The lesion can be easily identified clinically.
• Arterial doppler and pulse volume recording for baseline assessment of blood flow.
• Radiography may be necessary to rule out osteomyelitis.

Option A: Elevation of the affected extremity


• Elevation of the affected extremity will decrease perfusion and cause further ischemia.
• Ulceration and gangrene may occur, resulting in the loss of either limb.
Option C: Oxygen therapy
• Oxygen therapy has not been found to be treating arterial occlusive diseases in the best possible way.

Page 15

1265
• Oxygen therapy is generally a safe option, but it can cause side effects, including dry or bloody nose,
tiredness, and morning headaches.
• Different types of devices can provide oxygen. Some use tanks of liquid or gas oxygen.
Option D: Heart transplantation
• A compromised heart is replaced with a healthier heart in heart transplantation surgery.
• It is done when the patient's condition does not improve with medications and other surgeries.
• Risk includes multi-organ failure, infection, and even cancer.
• A person has to take lifelong medications and immunosuppressants.

Solution for Question 3:


Option A: Small vessels
• Numbness and sensation of coldness in a young female with a smoking history are suggestive of
Buerger's disease. Angiography showing peripheral arterial occlusion further confirms the diagnosis.
• Thromboangiitis obliterans (Buerger's disease) is a non-atherosclerotic, segmental inflammatory, and
occlusive disease that affects small and medium-sized arteries and veins in the upper and lower
extremities.
• The cause remains unknown; however, the use of tobacco is central to the initiation and progression
of the disease.
• The most effective treatment for TOA is smoking cessation.
• Medical line of treatment with vasodilators, pentoxifylline (reduces viscosity, increases
microperfusion) and cilostazol may help improve pain-free walking but cannot prevent disease
progression.
• Surgical treatment: Lumbar Symphatectomtomy.
Option B: Muscles
• Buerger’s disease does not affect muscles.
Option C: Bones
• Bone is a specialized form of dense connective tissue consisting of osteocytes embedded in a
non-living matrix; osseous tissue. Bone matrix is made of calcium carbonate, calcium phosphate, and
collagen fibres.
• Bones form the skeleton of the body.
• The most common disease of bones in old age is osteoarthritis, affecting mainly females.
Option D: Skin
• Skin is the largest organ of the body.
• Layers of skin are; Basal cell layer Squamous cell layer Stratum granulosum and stratum lucidum
Stratum corneum Papillary layer Reticular layer
• Basal cell layer
• Squamous cell layer

Page 16

1266
• Stratum granulosum and stratum lucidum
• Stratum corneum
• Papillary layer
• Reticular layer
• Basal cell layer
• Squamous cell layer
• Stratum granulosum and stratum lucidum
• Stratum corneum
• Papillary layer
• Reticular layer

Solution for Question 4:


Correct Option C: Cervical rib
• Pain and paraesthesia of the arm in a young patient are suggestive of thoracic outlet syndrome.
Adson test further confirms the presence of a cervical rib.
• The cervical rib is a supernumerary rib that arises from the seventh cervical vertebrae. They are
present in <1% of the population (70% female.)
• Most cases are asymptomatic. However, in rare cases, they may cause problems contributing to
thoracic outlet syndrome because of pressure on the lower trunk of the brachial plexus or subclavian
artery.
• Compression of the brachial plexus may be identified by the weakness of the muscles of the hand.
Compression of the subclavian artery is often diagnosed by a positive Adson’s test.
• Adson’s Test: Locate the radial pulse with the patient sitting or standing. Feel the pulse as you abduct,
extend, and externally rotate the patient’s arm. In this position, instruct the patient to take a deep breath
and hold it and rotate the head towards the tested arm. The test is positive if the pulse is reduced or
lost. A positive test indicates a comp of the subclavian artery.
• Locate the radial pulse with the patient sitting or standing.
• Feel the pulse as you abduct, extend, and externally rotate the patient’s arm.
• In this position, instruct the patient to take a deep breath and hold it and rotate the head towards the
tested arm.
• The test is positive if the pulse is reduced or lost.
• A positive test indicates a comp of the subclavian artery.
• If symptoms are severe, then an excision of the cervical rib is done. Other measures include
physiotherapy and electrotherapy.
• Locate the radial pulse with the patient sitting or standing.
• Feel the pulse as you abduct, extend, and externally rotate the patient’s arm.
• In this position, instruct the patient to take a deep breath and hold it and rotate the head towards the
tested arm.

Page 17

1267
• The test is positive if the pulse is reduced or lost.
• A positive test indicates a comp of the subclavian artery.
Incorrect Options:
Option A: Peripheral vascular disease
• Peripheral arterial disease, also known as a peripheral vascular disease is caused by atheroma in the
walls of the arteries leading to insufficient blood flow to the muscles and other tissues.
• The spectrum of clinical presentation can be classified into three categories: Asymptomatic
Intermittent claudication Critical limb ischemia
• Asymptomatic
• Intermittent claudication
• Critical limb ischemia
• Asymptomatic
• Intermittent claudication
• Critical limb ischemia
Option B: Varicose veins
• Varicose veins are dilated, tortuous, subcutaneous veins, often palpable, >3mm in diameter,
measured in an upright position with demonstrable reflux reverse blood flow.
• Varicose veins present as swelling of the ankle or leg, unsightly appearance, itching and throbbing,
reticular threaded veins, and Atrophie Blanche.
• Varicose veins are managed by graduated compression stockings, sclerotherapy, and laser ablation.
Option D: AV fistula
• An AV fistula is an irregular connection between an artery and a vein. Blood flows directly from the
artery into the vein bypassing capillaries.
• The signs and symptoms include purplish, bulging veins, swelling, fatigue, and decreased blood
pressure.

Solution for Question 5:


Option B: Horner syndrome
• Dryness of the face and constriction of the pupil on the side of the cut indicates that the thoracic
sympathetic chain was cut and suggestive of Horner syndrome.
• The sympathetic trunk lies lateral to the vertebral bodies for the entire length of the vertebral column.
The sympathetic trunk permits preganglionic fibres of the sympathetic system.
• Horner syndrome occurs due to a Pancoast tumour, birth trauma with injury to the lower brachial
plexus, aortic aneurysm, and neuroblastoma.
• Horner syndrome comprises the constellation of clinical signs, including the classical triad
(ptosis, anhidrosis, and miosis). It results from the lesion to the sympathetic pathway that supplies the
head and neck region.
Option A: Intermittent claudication

Page 18

1268
• Intermittent claudication is pain or fatigue in the muscles of the lower limb caused by walking and
relieved by rest.
• The distance between taking the first step and the onset of pain is called claudication distance, which
is the parameter of the severity of the disease.
Option C: Wet Gangrene
• Gangrene is the macroscopic death of tissues. Clinically, there is a loss of pulsation along with an
alteration of colour, temperature and sensations.
• Wet gangrene is usually seen in a patient with diabetes, where tissue gets infected along with
gangrene.
Option D: Arterial ulcers
• Arterial ulcers result from reduced arterial blood flow and subsequent tissue perfusion.
• They usually occur over the toes, heels and bony prominences. They are painful and appear punched
out with well-demarcated edges.

Solution for Question 6:


Correct Option C: Pain in ulnar distribution
• Pain and numbness on the medial aspect of the arm, wasting of the thenar and hypothenar eminence
with diminished radial pulse indicate thoracic outlet syndrome.
• Thoracic outlet syndrome constitutes a group of diverse disorders that result in compression of the
neurovascular bundle exiting the thoracic outlet. The important structures include the brachial plexus,
subclavian artery and subclavian vein.
• Thoracic outlet syndrome is classified based on the pathophysiology of symptoms Neurogenic
thoracic outlet syndrome (nTOS) Venous thoracic outlet syndrome (vTOS) Arterial thoracic outlet
syndrome (aTOS)
• Neurogenic thoracic outlet syndrome (nTOS)
• Venous thoracic outlet syndrome (vTOS)
• Arterial thoracic outlet syndrome (aTOS)
• Etiology includes congenital, traumatic, and functionally acquired causes.
• Symptoms and signs: Slowly progressive unilateral atrophic weakness of the intrinsic hand muscles
and sensory abnormalities in the C8-T1 distribution in neurogenic type. Non-positional ischemia of the
fingers and hands. Thrombosis of subclavian and axillary veins, Paget von Schroetter syndrome, are
features of venous type. Pain in ulnar distribution and paraesthesia of the upper extremities are
common in all three types.
• Slowly progressive unilateral atrophic weakness of the intrinsic hand muscles and sensory
abnormalities in the C8-T1 distribution in neurogenic type.
• Non-positional ischemia of the fingers and hands.
• Thrombosis of subclavian and axillary veins, Paget von Schroetter syndrome, are features of venous
type.
• Pain in ulnar distribution and paraesthesia of the upper extremities are common in all three types.

Page 19

1269
• It is diagnosed by Adson’s test, Roos test, Wright's hyperabduction test.
• Neurogenic thoracic outlet syndrome (nTOS)
• Venous thoracic outlet syndrome (vTOS)
• Arterial thoracic outlet syndrome (aTOS)
• Slowly progressive unilateral atrophic weakness of the intrinsic hand muscles and sensory
abnormalities in the C8-T1 distribution in neurogenic type.
• Non-positional ischemia of the fingers and hands.
• Thrombosis of subclavian and axillary veins, Paget von Schroetter syndrome, are features of venous
type.
• Pain in ulnar distribution and paraesthesia of the upper extremities are common in all three types.
Other Option
Option A: Intermittent claudication
• Intermittent claudication occurs due to muscle ischemia during exercise caused by obstruction to
arterial flow.
• Only a small proportion of patients with intermittent claudication require revascularization.
• The main aim of treatment is to reduce the risk of mortality from cardiovascular events.
• Smoking cessation, control of hypertension and diabetes, statins, and anti-platelet drugs are used for
treating the condition.

Option B: Pain in radial distribution


• Radial distribution of the arm is supplied by the posterior cord of the brachial plexus (C5 to C8and T1).
• The radial nerve provides a sensory supply of the skin of the arm, the lateral aspect of the forearm,
and the hand. Here patients present with symptoms on the medial aspect of arm.
Option D: Gangrene
• Gangrene is macroscopic death of the tissues. It is due to insufficient blood supply and ensuing
bacterial invasion.
• It is most commonly seen in distal parts of limbs, the loop of the appendix, and the small intestine.

Solution for Question 7:


Option B: Resting pain
• Bilateral, severe pain in the legs associated with cramp-like pain felt in the muscles after walking
(intermittent claudication), redness and swelling with absent pedal pulses and positive sunset foot sign
indicate peripheral arterial stenosis and occlusion in the vessels of the limb. The dangerous sign in this
patient is resting pain, and it is an indication of emergency intervention to prevent limb amputation.
• Resting pain means that the blood supply to the organ is so compromised that the muscles cannot
even fulfill their metabolic demands at rest and have to switch to anaerobic respiration.
• This means that the blood supply is getting critical, and if necessary measures are not taken, it will
eventually lead to limb amputation.

Page 20

1270
Option A: Intermittent claudication
• Intermittent claudication means cramp-like pain in the foot while performing any movement like
walking or running.
• This indicates that the blood supply is sufficient during the resting period, but when the metabolic
demands are increased due to some activity, the blood supply is insufficient.
Option C: Swelling
• It is an indication that there is some obstruction and inflammation, which has led to the leaking of fluid
from the vessels to the extracellular compartment.
• It is an important sign but does not necessitate emergency amputation..
Option D: Redness
• Redness indicates that there is an inflammatory process going on in the organ.
• The presence of inflammatory mediators has led to vasodilation and increased blood supply, thus,
redness and warmth.
• It is not a dangerous sign of arterial stenosis.

Solution for Question 8:


Option B: Allen’s test
• Before using the radial artery, Allen's test should be performed.
• It assesses whether the ulnar artery can maintain collateral blood flow to the hand.
• It consists of the patient making a fist while the surgeon compresses both radial and ulnar arteries
digitally; this squeezes blood from the hand. The hand is then relaxed, and compression of the ulnar
artery is released; the speed of returning colour to the hand is assessed. If colour returns in 5-7s,
patency and collateral flow from the ulnar artery are confirmed.
Incorrect Options:
Option A: Tinel’s test
• It is used for the identification of compression of a peripheral nerve.
• It consists of tapping over the nerve of interest,and tingling sensation may indicate nerve
compression.
Option C: Phalen’s test
• It indicates compression of the median nerve which causes paraesthesia.
• Used to diagnose Carpal tunnel syndrome..
Option D: Froment’s test
• It suggests the weakness of the adductor pollicis muscle supplied by the ulnar nerve.
• A positive test is indicated by flexion of the thumb instead of adduction while holding a book or card.

Page 21

1271
Solution for Question 9:
Option D: Smoking cessation
• This diagnosis is Buerger’s disease and the only measure that can stop disease progression is the
cessation of smoking
• Buerger’s disease, also known as thromboangitis obliterans, is characterized by occlusive disease of
small and medium-sized arteries.
• It is associated with superficial thrombophlebitis and Raynaud’s phenomenon in a smoker of usually
less than 30 years of age.
• Amputation is indicated if the disease is not controlled.
Option A: Aspirin
• It is a platelet aggregation inhibitor, usually beneficial in patients with atheromatous disease.
• It has very little to no role in thromboangiitis obliterans.
Option B: Good glycaemic control
• It is useful in patients with a disease due to increased blood sugar levels.
• Because Buerger’s disease has no diabetic component, glycaemic control is useless in these
patients.
Option C: Blood cholesterol control
• It is useful in patients suffering from a disease due to the formation of atheromatous plaques in the
vessel walls, resulting from increased cholesterol levels.
• Cholesterol levels are normal in Buerger’s disease, so they don’t have a role in disease cessation.

Solution for Question 10:


Option B: 4 hours
• A completely transected “limb”, not the fingers, which are kept in normal conditions without any
cooling, needs to be re-implanted in under 4 hours, after which sufficient necrosis would occur.
• The severed limb needs to be put in a cool environment of at least 0-4°C to delay the process of
necrosis.
Option A: 8 hours
• It is the time before the fingers can be re-implanted if they’re kept without cooling.
• Severed fingers can survive more time outside as compared to the limb.
Option C: 8-12 hours
• Severed bone, tendons and skin can survive for 8-12 hours in the outside environment.
• After 8-12 hours, necrosis starts in bone, tendon and skin and then they're not able to be re-implanted.
Option D: 24 hours
• It is the time for which a severed limb can survive if kept in a cool environment.
• Cooling slows the metabolism and thus delays the process of necrosis.

Page 22

1272
• One hour of an organ kept outside equals 6 hours in a cold environment.

Solution for Question 11:


Correct Option D: Branham sign
• Nicoladoni-Branham (Branham's) sign, a decrease in pulse and increase in blood pressure that
immediately follows the sudden occlusion of an arteriovenous fistula.
Nicoladoni-Branham (Branham's) sign, a decrease in pulse and increase in blood pressure that immedi
ately follows the sudden occlusion of an arteriovenous fistula.
Incorrect Options:
Option A: Murray sign
• Murray’s sign indicates subcostal region tenderness during inspiration.
• Helps in diagnosing acute cholecystitis.
Option B: Bancroft’s (Moses') sign
• Pain elicited when the calf muscle is compressed forwards against the tibia, but not when the calf
muscle is compressed from side to side.
• A clinical sign that helps in diagnosing deep vein thrombosis.
Pain elicited when the calf muscle is compressed forwards against the tibia, but not when the calf musc
le is compressed from side to side.
A clinical sign that helps in diagnosing deep vein thrombosis.
Option C: Darier sign
• Darier sign is the development of an urticarial wheal and flare after firm stroking of lesional skin.
• Helps in diagnosing mastocytosis.

Solution for Question 12:


Option A: Raynaud’s disease
• Episodic vasospasm of extremities and triphasic color response makes Raynaud's disease the best
probable diagnosis.
• It is a medium-sized vessel vasculitis affecting digital vessels in fingers and toes.
• It is caused by episodic vasospasm triggered by cold or stress.
• It affects young women below the age of 30.
• It has a classic clinical presentation of triphasic color response.
• The triphasic color response includes three stages Blanching (stage of local syncope): With
exposure to cold, digital arterioles go into spasm. Decreased flow is evidenced by pallor, or blanching
digits may appear white. Cyanosis (stage of local asphyxia) Capillaries and venules dilate, and digits
appear blue. Red Engorgement (stage of recovery) With rewarming or passing of attack, the
digital vasospasm resolves. Blood flow into dilated arteries and capillaries increases dramatically.

Page 23

1273
Reactive hyperemia imparts bright red color to the digits.
• Blanching (stage of local syncope): With exposure to cold, digital arterioles go into spasm.
Decreased flow is evidenced by pallor, or blanching digits may appear white.
• With exposure to cold, digital arterioles go into spasm.
• Decreased flow is evidenced by pallor, or blanching digits may appear white.
• Cyanosis (stage of local asphyxia) Capillaries and venules dilate, and digits appear blue.
• Capillaries and venules dilate, and digits appear blue.
• Red Engorgement (stage of recovery) With rewarming or passing of attack, the digital
vasospasm resolves. Blood flow into dilated arteries and capillaries increases dramatically. Reactive
hyperemia imparts bright red color to the digits.
• With rewarming or passing of attack, the digital vasospasm resolves.
• Blood flow into dilated arteries and capillaries increases dramatically.
• Reactive hyperemia imparts bright red color to the digits.
• Blanching (stage of local syncope): With exposure to cold, digital arterioles go into spasm.
Decreased flow is evidenced by pallor, or blanching digits may appear white.
• With exposure to cold, digital arterioles go into spasm.
• Decreased flow is evidenced by pallor, or blanching digits may appear white.
• Cyanosis (stage of local asphyxia) Capillaries and venules dilate, and digits appear blue.
• Capillaries and venules dilate, and digits appear blue.
• Red Engorgement (stage of recovery) With rewarming or passing of attack, the digital
vasospasm resolves. Blood flow into dilated arteries and capillaries increases dramatically. Reactive
hyperemia imparts bright red color to the digits.
• With rewarming or passing of attack, the digital vasospasm resolves.
• Blood flow into dilated arteries and capillaries increases dramatically.
• Reactive hyperemia imparts bright red color to the digits.
• With exposure to cold, digital arterioles go into spasm.
• Decreased flow is evidenced by pallor, or blanching digits may appear white.
• Capillaries and venules dilate, and digits appear blue.
• With rewarming or passing of attack, the digital vasospasm resolves.
• Blood flow into dilated arteries and capillaries increases dramatically.
• Reactive hyperemia imparts bright red color to the digits.
Option B: Acrocyanosis
• It refers to peripheral arterial disease caused by vasospasm of small blood vessels.
• It presents as painless, non-episodic skin color changes in fingers or toes.
• Digits show bluish discoloration and may appear as mottled cyanosed fingers or toes.
• It is also triggered by cold or stress.
Option C: Arteriovenous Fistula

Page 24

1274
• It refers to abnormal communication between an artery and a vein.
• The vein becomes tortuous, dilated, and thick-walled.
• It can cause high-output cardiac failure.
• It will present as pulsatile swelling in the superficial lesion and may give machinery murmur on
palpation.
Option D: Subclavian Steal Syndrome
• It refers to the subclavian artery stenosis proximal to the origin of the vertebral artery leading to
hypoperfusion distal to the stenosis.
• Clinical presentation: Arm ischemia, pain, paresthesia, vertebrobasilar insufficiency (dizziness,
vertigo).
• Associated with arteriosclerosis, Takayasu's arteritis, and heart surgery.

Solution for Question 13:


Correct Option B: Thoracic aortic aneurysm
• The image of Thoracic magnetic resonance angiography reveals a clear saccular mid-descending
thoracic aortic aneurysm with a locally contained leak and periaortic inflammation.
• An aortic aneurysm is pathological dilatation of the aorta. It can be either an abdominal aortic
aneurysm or a thoracic aortic aneurysm.
• The abdominal aortic aneurysm will present as a palpable pulsatile abdominal mass.
• Thoracic aortic aneurysms are those occurring distal to the left subclavian vein.
• A thoracic aortic aneurysm is associated with cystic medial degeneration.
• Risk factors for thoracic aortic aneurysm: hypertension, bicuspid aortic valve, Marfan syndrome,
tertiary syphilis.
• When a patient presents with sudden onset chest or abdominal pain radiating to the back, it is a sign
of leaking, dissection, or imminent rupture.
• Unequal blood pressure in bilateral arms is another classic presentation in aortic aneurysm rupture.
• The risk for rupture is increased with the following: Increasing aneurysm size Advanced age History of
chronic obstructive pulmonary disease (COPD) Concurrent dissection. Note: Anticoagulation is not a
risk factor for rupture.
• Increasing aneurysm size
• Advanced age
• History of chronic obstructive pulmonary disease (COPD)
• Concurrent dissection. Note: Anticoagulation is not a risk factor for rupture.
• Note: Anticoagulation is not a risk factor for rupture.
• Increasing aneurysm size
• Advanced age
• History of chronic obstructive pulmonary disease (COPD)

Page 25

1275
• Concurrent dissection. Note: Anticoagulation is not a risk factor for rupture.
• Note: Anticoagulation is not a risk factor for rupture.
• Note: Anticoagulation is not a risk factor for rupture.
Incorrect Options:
Option A: Subclavian Steal Syndrome
• It refers to the subclavian artery stenosis proximal to the origin of the vertebral artery which leads to
hypoperfusion distal to the stenosis.
• Clinical presentation: arm ischemia, pain, paresthesia, vertebrobasilar insufficiency (dizziness,
vertigo).
• Associated with arteriosclerosis, Takayasu's arteritis, and heart surgery.
Option C: Atherosclerosis
• It is a disease of elastic arteries and large-and medium-sized muscular arteries.
• It is caused by a buildup of cholesterol plaques in the intimal layer.
• Most commonly occurs in the abdominal aorta.
• Symptoms: chest pain, claudication, or can be asymptomatic.
• It can lead to abdominal aortic aneurysms, ischemia, infarcts, thrombus, and emboli.
Option D: Cardiac tamponade
• Compression of the heart by fluid accumulation in pericardial space.
• It leads to decreased cardiac output.
• Clinical findings include pulsus paradoxus and Beck triad: Hypotension Distended neck veins
Muffled heart sounds
• Hypotension
• Distended neck veins
• Muffled heart sounds
• ECG will show low-voltage QRS and electrical alterations.
• Hypotension
• Distended neck veins
• Muffled heart sounds

Solution for Question 14:


Option C: Giant cell arteritis
• Clinical signs and symptoms like scalp tenderness, episodic headache, jaw claudication, and nodular
temporal arteries make ‘Giant cell arteritis’ the most probable diagnosis.
• Giant cell arteritis is also known as temporal arteritis.
• It is a disease in which localized infiltration with inflammatory and giant cells leads to arterial
occlusion, ischemic headache, and tender, palpable, pulseless arteries in the scalp.

Page 26

1276
• Other ischemic symptoms include claudication of facial or extremity muscles (jaw claudication) and
retinal ischemia ESR will be raised.
• Irreversible blindness can occur if the ophthalmic artery becomes occluded.
• Superficial temporal, vertebral, and major aortic arch branches may be involved.
• Signs and symptoms make the diagnosis, but a temporal artery biopsy can confirm the diagnosis.
• Treatment should not be delayed for a temporal artery biopsy.
• Treatment is immediate steroid therapy to arrest and reverse the process.
Option A: Wegener’s granulomatosis
• Also known as Granulomatosis with polyangiitis.
• It involves pulmonary and renal vessels.
• Clinical presentation: Perforation of the nasal septum, chronic sinusitis, otitis media, mastoiditis,
hemoptysis, cough, dyspnea, hematuria, red cells cast.
Option B: Takayasu’s arteritis
• It is also known as 'Pulseless disease' as the patient will have weak upper extremity pulses.
• Other symptoms: Fever, night sweats, arthritis, myalgias, skin nodules, and ocular disturbances.
• It is arthritis that obstructs major arteries, particularly the large vessels coming out of the aortic arch.
Option D: Polyarteritis nodosa
• It usually involves renal and visceral vessels and is associated with Hepatitis B.
• Clinical presentation: Fever, weight loss, malaise, headache, abdominal pain, melena, hypertension,
neurological dysfunction, cutaneous eruptions, and renal damage.

Solution for Question 15:


Option D: Increased cardiac output
• When an arteriovenous fistula is formed involving a major artery, it can lead to a large decrease in
peripheral resistance.
• This lowered peripheral resistance causes the heart to increase cardiac output to maintain proper
blood flow to all tissues.
• In this case, a patient with chronic kidney disease has been admitted for dialysis and an arteriovenous
fistula has been created for hemodialysis access.
• As a result of which, veins become dilated, tortuous, and become arterialized.
Option A: Sinus bradycardia
• Arteriovenous fistula leads to a decrease in total peripheral resistance, as a result of which cardiac
output and heart rate increase leading to sinus tachycardia.
Option B: Decrease in preload
• An arteriovenous fistula can increase preload. AV shunts also decrease the afterload of the heart.
• As the blood bypasses the arterioles, it results in a decrease in total peripheral resistance (TPR).

Page 27

1277
• AV shunts increase both the rate and volume of blood returning to the heart.
Option C: Cardiac arrhythmias
• If an arteriovenous fistula is superficial, it may present as a palpable mass with a bruit on auscultation.
• It increases preload and decreases afterload; as a result, heart rate increases.
• Cardiac arrhythmias don’t occur in AV fistula.

Solution for Question 16:


Correct Option A: Marjolin’s ulcer
• Chronic leg ulcers usually develop in varicose veins, venous hypertension, chronic infections, and
long-standing vasculitis.
• Marjolin’s ulcer arises from a long-standing ulcer or burn scar. It is a low-grade squamous cell
carcinoma.
• It destroys nerves and is painless, slow-growing, and radio-resistant.
• There is no distant metastasis.
• It is a diagnosis by skin biopsy and treated by wide local excision with flap closure.
Incorrect Options:
Option B: Aphthous ulcer
• Aphthous ulcer is the most common extraintestinal manifestation of Crohn's disease.
• It is characterized by an irregular, ulcerated, erythematous area with normal colonic mucosa.
Option C: Pressure sores
• Pressure sores are areas of tissue necrosis with ulceration under prolonged pressure along bony
prominences
• Sites of pressure sore are the occiput, sacrum, ischium, and greater trochanter.
Option D: Necrotizing fasciitis
• Necrotizing fasciitis is a polymicrobial infection of soft tissues with gram-positive aerobes,
gram-negative anaerobes, and beta-hemolytic.
• It is characterized by pain, edema, crepitus, blistering, fever, and greyish pus.
• It can lead to shock and multi-organ failure in the advanced stage.
• Treatment is broad-spectrum antibiotics and surgical excision.

Solution for Question 17:


Option A: Aortoiliac Occlusive Disease
• Based on the history, the most likely diagnosis is an aortoiliac occlusive disease.

Page 28

1278
• The aortoiliac occlusive disease is an aspect of peripheral arterial disease (PAD) in which occluding
plaques caused by atherosclerosis occlude the infrarenal aorta and iliac arteries, ultimately causing
partial or total vascular occlusion.
• The most common symptom of patients with aortoiliac disease is claudication.
• Claudication can occur in the calf, thigh, hip, and buttocks.
• Symptoms of buttock claudication can occur with erectile dysfunction in patients with absent femoral
pulses. This is termed Leriche syndrome.
• Risk Factors: Atherosclerosis Smoking Hypercholesterolemia Diabetes Hypertension
• Atherosclerosis
• Smoking
• Hypercholesterolemia
• Diabetes
• Hypertension
• Investigations: Serum lipid profile Computed tomography (CT) angiography Magnetic resonance
angiography (MRA) or arterial duplex mapping is the investigation of choice for planning surgery
• Serum lipid profile
• Computed tomography (CT) angiography
• Magnetic resonance angiography (MRA) or arterial duplex mapping is the investigation of choice for
planning surgery
• Management: Medical: The risk factors must be identified and treated Initiate a walking exercise
program Pentoxifylline and cilostazol may reduce the symptoms of claudication caused by
lower-extremity arterial occlusive disease. Surgical: Thromboendarterectomy Aortobifemoral bypass
Extra-anatomic bypass Angioplasty and stenting
• Medical: The risk factors must be identified and treated Initiate a walking exercise program
Pentoxifylline and cilostazol may reduce the symptoms of claudication caused by lower-extremity
arterial occlusive disease.
• The risk factors must be identified and treated
• Initiate a walking exercise program
• Pentoxifylline and cilostazol may reduce the symptoms of claudication caused by lower-extremity
arterial occlusive disease.
• Surgical: Thromboendarterectomy Aortobifemoral bypass Extra-anatomic bypass Angioplasty and
stenting
• Thromboendarterectomy
• Aortobifemoral bypass
• Extra-anatomic bypass
• Angioplasty and stenting
• Atherosclerosis
• Smoking
• Hypercholesterolemia

Page 29

1279
• Diabetes
• Hypertension
• Serum lipid profile
• Computed tomography (CT) angiography
• Magnetic resonance angiography (MRA) or arterial duplex mapping is the investigation of choice for
planning surgery
• Medical: The risk factors must be identified and treated Initiate a walking exercise program
Pentoxifylline and cilostazol may reduce the symptoms of claudication caused by lower-extremity
arterial occlusive disease.
• The risk factors must be identified and treated
• Initiate a walking exercise program
• Pentoxifylline and cilostazol may reduce the symptoms of claudication caused by lower-extremity
arterial occlusive disease.
• Surgical: Thromboendarterectomy Aortobifemoral bypass Extra-anatomic bypass Angioplasty and
stenting
• Thromboendarterectomy
• Aortobifemoral bypass
• Extra-anatomic bypass
• Angioplasty and stenting
• The risk factors must be identified and treated
• Initiate a walking exercise program
• Pentoxifylline and cilostazol may reduce the symptoms of claudication caused by lower-extremity
arterial occlusive disease.
• Thromboendarterectomy
• Aortobifemoral bypass
• Extra-anatomic bypass
• Angioplasty and stenting
Option B: Common femoral occlusive disease
• The most common symptoms of the common femoral occlusive disease are thigh and calf
claudication.
• Surgical endarterectomy is the standard treatment for the occlusive disease of the common femoral
artery.
Option C: Superficial femoral occlusive disease
• Patients with superficial femoral occlusive disease experience pain in the upper two-thirds of the calf.
About half of them complain about atypical leg pain, and about 40% remain asymptomatic.
• Acute limb ischemia is an uncommon presentation.
• Revascularization is a treatment option only for selected patients with this disease.
Option D: Popliteal artery occlusive disease

Page 30

1280
• Popliteal artery occlusive disease patients most commonly present with intermittent claudication.
Patients experience cramping pain distal to the level of obstruction.
• Surgical therapy for popliteal artery occlusion involves bypassing the occlusion, which can be
achieved with grafts, including great saphenous veins or prosthetic grafts.

Solution for Question 18:


Correct Option C - Removal of an embolus from blood vessels:
• Surgical thrombectomy is a medical procedure used to remove a blood clot from an artery or vein.
• A thin, flexible catheter is guided through a small incision in the groin or neck and into the artery in the
lungs.
• The catheter is placed near the clot to break it up or remove it.
• The blood vessel is repaired after the clot is removed.
• This increases the flow of blood. Sometimes, a device like a balloon may be placed in the blood
vessel to help keep it open.
• For patients with massive pulmonary embolism, surgical thrombectomy is an effective procedure with
a low risk of bleeding.

Incorrect Options:
Option A - Stop variceal bleeding from blood vessels:
• Variceal bleeding occurs when varices in the gastrointestinal tract, such as the esophagus, stomach,
and rectum, bleed.
• If the varices have bled, there is a high chance that they will bleed again.
• Band ligation of varices is the preferred endoscopic treatment for esophageal variceal bleeding.
• Injections of cyanoacrylate are used to treat bleeding from gastric varices.

Page 31

1281
• Treatment with vasoactive drugs and antibiotics is initiated before or concurrently with endoscopy.
• Bleeding from portal hypertensive gastropathy is less common, usually chronic, and can be treated
with beta-blocker therapy, injection therapy, or interventional radiology.
Option B - Parenteral hyperalimentation:
• Hyperalimentation, also known as total parenteral nutrition (TPN), is used for people with medical
conditions that make gastrointestinal absorption difficult or impossible.
• Parenteral nutrition is used by people whose digestive systems cannot absorb or tolerate adequate
oral food.
• When administered outside the hospital, intravenous feeding is called home parenteral nutrition.
Home parenteral nutrition may sometimes be required for weeks, months, or even lifetime in some
patients.
Option D - Ureteric catheterization:
• Urethral catheterization is a common medical procedure for direct urinary bladder drainage. It can be
used either for diagnostic (to help determine the etiology of various genitourinary conditions) or
therapeutic purposes (to relieve urinary retention, instill the medication, or provide irrigation).
• For long-term drainage, urethral catheters can be brought out of the urethra and abdominal wall.
• Because the upper J keeps the catheter within the renal pelvis and the lower J keeps the catheter
within the bladder, ureteral double-J stents are ideal for ureteral drainage.

Solution for Question 19:


Option D: Thoracic aorta aneurysms, graft surgery
• The patient presents with difficulty breathing, difficulty swallowing, and hoarseness of voice. These
symptoms are present in thoracic aortic aneurysms.
• Marfan’s syndrome is a connective tissue disorder affecting many structures, such as the blood
vessels, increasing the risk of aneurysms.
• The radiography shows a dilatation of 7cm, requiring surgery.
• The indications for surgery are: Marfan-related aneurysms Symptomatic aneurysms Diameter > 4.5–5
cm in ascending thoracic aneurysm Diameter > 6cm in descending thoracic aneurysm A growth of 1
cm/year or faster.
• Marfan-related aneurysms
• Symptomatic aneurysms
• Diameter > 4.5–5 cm in ascending thoracic aneurysm
• Diameter > 6cm in descending thoracic aneurysm
• A growth of 1 cm/year or faster.
• These symptoms are due to compression effects of thoracic aneurysms to other nearby structures:
Esophagus (dysphagia) Left main bronchus (respiratory symptoms) Left recurrent laryngeal nerve
(hoarseness of voice)
• Esophagus (dysphagia)
• Left main bronchus (respiratory symptoms)

Page 32

1282
• Left recurrent laryngeal nerve (hoarseness of voice)
• Marfan-related aneurysms
• Symptomatic aneurysms
• Diameter > 4.5–5 cm in ascending thoracic aneurysm
• Diameter > 6cm in descending thoracic aneurysm
• A growth of 1 cm/year or faster.
• Esophagus (dysphagia)
• Left main bronchus (respiratory symptoms)
• Left recurrent laryngeal nerve (hoarseness of voice)
Option A: Thoracic aorta aneurysms, no surgery required
• If the ascending thoracic aneurysm diameter is >4.5–5 cm or descending thoracic aneurysm diameter
is > 6.5cm, then surgery is required.
Option B: Abdominal aortic aneurysms, no surgery required
• Respiratory symptoms and dysphagia are not the typical features of abdominal aorta aneurysms
Option C: Abdominal aortic aneurysm, graft surgery
• Respiratory symptoms, hoarseness of voice, and dysphagia are not the typical features of abdominal
aorta aneurysms.

Solution for Question 20:


Option A: Type A aortic dissection, PET graft
• Type A aortic dissection is stated to involve the ascending abdominal aorta.
• The patient shows ascending abdominal aorta dissection features such as cough, breathing, and
swallowing. All these symptoms are due to the compression effect on nearby structures.
• Arteriography is the gold standard for diagnosis. Surgery is the mainstay of treatment in Type A
dissection. After localization of the tear, it is resected, and a synthetic graft is implanted.
• PET grafts are commonly used for Type A dissection.
Option B: Type B aortic dissection, PTFE graft
• PTFE(Polytetrafluoroethylene) material that is used in infra-inguinal bypass.
Option C: Type B aortic dissection, PET graft
• Type B classification does not involve ascending aortic dissection. The presenting symptoms of type
B are abdominal pain, radiation to the leg, and back pain.
Option D: Type A aortic dissection, PTFE graft
• PTFE is commonly used for the infra-inguinal bypass.

Page 33

1283
Solution for Question 21:
Correct Option A - Aortic dissection:
• This is a case of aortic dissection with sudden onset sharp tearing chest pain, increased blood
pressure, and mediastinum widening.
• Aortic dissection presents with >20 mmHg difference in SBP between the arms and ECG with
nonspecific ST and T wave changes.
• It is associated with hypertension, congenital bicuspid aortic valve, cocaine use, and Marfan
syndrome.
• Diagnosis Transesophageal echocardiography (TEE) Computed tomography (CT) angiography
• Transesophageal echocardiography (TEE)
• Computed tomography (CT) angiography
• Transesophageal echocardiography (TEE)
• Computed tomography (CT) angiography

Incorrect Options:
Option B - Thoracic aorta aneurysm:
• Involves ascending aorta and descending aorta above the diaphragm.
• It is associated with hypertension, Marfan syndrome, and Ehler-Danlos syndrome (EDS).
• It causes aortic root dilation, possibly leading to aortic valve regurgitation.
Option C - Angina:
• Angina is chest pain due to ischemic myocardium secondary to coronary artery narrowing or spasm.
• ECG findings are ST depression (stable angina), transient ST elevation (vasospastic angina), and ST
depression and T wave inversion (unstable angina).
Option D - Heart failure:

Page 34

1284
• Heart failure is a failure of the cardiac pump that presents with shortness of breath, orthopnea, fatigue,
crackles, S3 heart sound, jugular venous distention, and pitting edema.
• Left heart failure is caused by hypertension, myocardial infarction, ischemia, and dilated
cardiomyopathy.
• The right heart is caused by chronic lung disease (leads to cor pulmonale).

Solution for Question 22:


Correct Option B - Subclavian steal syndrome:
• This is a case of subclavian steal syndrome with pain, fatigue, paresthesia in the left arm, dizziness,
disequilibrium, and supraclavicular bruit.
• It occurs due to atherosclerosis of the subclavian artery proximal to the origin of the vertebral artery,
leading to a reversal of blood flow in the ipsilateral vertebral artery.
• Diagnosis Doppler ultrasound Magnetic resonance angiography (MRA)
• Doppler ultrasound
• Magnetic resonance angiography (MRA)
• Doppler ultrasound
• Magnetic resonance angiography (MRA)
• Treatment Lipid-lowering intervention Smoking cessation Stent placement in some cases
• Lipid-lowering intervention
• Smoking cessation
• Stent placement in some cases
• Lipid-lowering intervention
• Smoking cessation
• Stent placement in some cases
Incorrect Options:
Option A - Angina:
• Angina is chest pain due to ischemic myocardium secondary to coronary artery narrowing or spasm.
• Atherosclerosis is the most common risk factor and doesn't lead to cardiac myocyte necrosis.
• ECG findings are ST depression (stable angina), transient ST elevation (vasospastic angina) and ST
depression and T wave inversion (unstable angina)
Option C - Aortic dissection:
• Aortic dissection is an intima tear that occurs in the proximal 10 cm of the aorta with pre-existing
weakness of tunica media.
• It is associated with risk factors like a bicuspid aortic valve, hypertension, Marfan syndrome, and
cocaine use.

Page 35

1285
Option D - Heart failure:
• Heart failure is a failure of the cardiac pump that presents with shortness of breath, orthopnea, fatigue,
crackles, S3 heart sound, jugular venous distention, and pitting edema.
• Left heart failure is caused by hypertension, myocardial infarction, ischemia, and dilated
cardiomyopathy.
• A right heart failure is caused by left heart failure, chronic lung disease (leads to cor pulmonale)

Solution for Question 23:


Option B: Heyde syndrome
• Severe fatigue on minimal exertion, haemoglobin level at 5.7 mg/dL, along with iron deficiency
indicate Heyde syndrome.
• Endoscopy or colonoscopy shows angiodysplasia, which confirms the diagnosis of Heyde’s
syndrome.
• Heyde syndrome is a multisystem disorder that includes aortic stenosis (AS), gastrointestinal
bleeding, and acquired von Willebrand syndrome.
• It is a rare condition that, if not treated promptly, can result in severe morbidity and mortality.
• The gastrointestinal bleeding and aortic stenosis from angiodysplasia is a lack of
high-molecular-weight multimers of the von Willebrand factor.

Option A: Char syndrome


• Char syndrome is distinguished by three major characteristics: a distinct facial appearance, a heart
defect known as patent ductus arteriosus, and hand abnormalities.
• It all begins with a mutation in a gene called TFAP2B. This gene instructs the body to produce a
protein that provides normal development of the face, limbs, and heart.
• It is identified through molecular genetic testing.
Option C: Williams-Beuren syndrome
• It is a rare genetic disorder defined by prenatal and postnatal growth retardation, short stature, varying
degrees of mental deficiency, and distinct facial features that usually become more pronounced with
age.
• William syndrome is usually diagnosed in children before they reach age four.
Option D: Angelman syndrome
• It is defined as developmental delays, speech and balance issues, intellectual disability, and, in rare
cases, seizures.
• Patients frequently smile and laugh and have upbeat excitable personalities.
• Genetic testing can detect chromosomal abnormalities (Paternal disomy) in children that indicate
Angelman syndrome.
• A blood test almost always provides a definitive diagnosis.

Page 36

1286
Solution for Question 24:
Option C: Descending aortic dissection
• The patient with Descending aortic dissection presents with chest pain, abdominal pain radiating to
the leg, and back.
• The weak lower limb pulses are due to insufficient blood supply. The back pain is due to the
involvement of dissection to renal arteries. If mesenteric arteries are involved, abdominal pain will also
be the presenting feature of aortic dissection.
• The causative factors in this case are: Hypertension (causing wall tear of the aorta) Non-compliance
to antihypertensives Age > 60 years
• Hypertension (causing wall tear of the aorta)
• Non-compliance to antihypertensives
• Age > 60 years
• IOC: CT scan
• The management is to control blood pressure (permissive hypotension with esmolol), and if symptoms
persist, surgery is indicated, i.e., grafting the damaged part of the aorta.
• Hypertension (causing wall tear of the aorta)
• Non-compliance to antihypertensives
• Age > 60 years
Option A: Ascending aortic dissection
• The patient presents with cough, breathing difficulty, or stroke.
• The pain in the abdomen, leg, and back is usually not the presenting feature.
Option B: Abdominal aortic aneurysms
• IOC for screening of Abdominal aortic aneurysms is USG
• Most common symptom: Chronic vague abdominal pain/back(retroperitoneal)
• Most common and lethal complication of aneurysm: Rupture.
Option D: Thoracic aortic aneurysms
• The CT scan and MRI show the presence of a tear of the aorta, not the dilatation.

Solution for Question 25:


Option D: Psoas major and lymphatic chain
• The lumbar chain can be mistaken for the lymphatic chain, genitofemoral nerve, and psoas sheath.
• Genitofemoral nerve arises from the lumbar plexus, descends, pierces the psoas major muscle, and
lies in close vicinity of the sympathetic chain, which is why it can be confused with the lumbar
sympathetic chain.

Page 37

1287
• Lumbar lymph nodes are located on either side of vertebral columns between the diaphragm and
pelvis.
• A lumbar sympathectomy injection method provides pain relief by blocking the pain-inducing
sympathetic nerves in the lumbar region.
• Open sympathectomy is done, preferably through an extraperitoneal approach.
• Lumbar sympathectomy reduces foot and leg pain due to inadequate blood supply; it helps leg and
foot ulcers to heal properly and relieves leg and back pain caused by spinal canal stenosis.

Option A: Ilioinguinal nerve


• The ilioinguinal nerve is a mixed nerve that arises from the inferior branch of the anterior ramus of the
lumbar plexus.
• The function of the ilioinguinal nerve is to provide sensory innervation to the upper anteromedial thigh
and somewhat to the external genitalia.
Option B: Abdominal aorta
• The abdominal aorta is the continuation of descending thoracic aorta at the level of T12 to the median
arcuate ligament and diaphragmatic crura.
• It descends in the retroperitoneum and slightly to the left of the lumbar vertebral bodies.
• It eventually terminates at the L4 level by bifurcating in the left and right common iliac arteries. That is
also where the median sacral artery originates, which descends along the middle of the sacral
promontory.
Option C: Posterior root of brachial plexus
• The posterior cord of the brachial plexus is formed by the posterior divisions of the superior, middle
and inferior trunks.
• It gives three nerve branches: the upper subscapular nerve, the thoracodorsal nerve, and the lower
subscapular nerve which supplies the back side of the body.

Solution for Question 26:


Correct Option D - Roos test:
• This is the case of thoracic outlet syndrome due to a Pancoast tumour, also known as a sulcus
tumour, which occurs at the apex of the lungs or due to the cervical rib; the patient was instructed to do
a Roos test.
• Roos Test (Elevated Arm Stress Test). Manoeuvre: Arms are placed in surrender position with
shoulders abducted to 90º, and in external rotation, elbows flexed to 90º. The patient slowly opens and
closes their hand for 3 min.
• Manoeuvre: Arms are placed in surrender position with shoulders abducted to 90º, and in external
rotation, elbows flexed to 90º. The patient slowly opens and closes their hand for 3 min.
• Manoeuvre: Arms are placed in surrender position with shoulders abducted to 90º, and in external
rotation, elbows flexed to 90º. The patient slowly opens and closes their hand for 3 min.

Page 38

1288
• Results: Precipitates pain, paresthesia, heaviness, or weakness.
Incorrect Options:
Option A - Adson Test:
• Manoeuvre: Affected arm is abducted 30º at the shoulder while maximally extended. While extending
the neck and turning the head towards the ipsilateral shoulder, the patient inhales deeply.

• Results: Decrease or absence of an ipsilateral radial pulse.


• Indication: Thoracic outlet syndrome associated with compression of the subclavian artery by the
cervical rib or stretched anterior and middle scalene muscles.
Option B - Elvey test
• Manoeuvre: Upper Limb Tension Test (ULTT) Position 1: Arms abducted to 90º with elbows flexed.
Position 2: Active dorsiflexion of both wrists. Position 3: Head is tilted ear to shoulder in both directions.
• Position 1: Arms abducted to 90º with elbows flexed.

Page 39

1289
• Position 2: Active dorsiflexion of both wrists.
• Position 3: Head is tilted ear to shoulder in both directions.
• Position 1: Arms abducted to 90º with elbows flexed.
• Position 2: Active dorsiflexion of both wrists.
• Position 3: Head is tilted ear to shoulder in both directions.

• Results: Positions 1 and 2 elicit symptoms on the ipsilateral side, while Position 3 elicits symptoms on
the contralateral side.
Option C - Wright test:
• Manoeuvre: Hyperabduction test, head forward while arm is passively brought in the abduction and
with external rotation to 90º without tilting head and elbow is flexed no more than 1 minute.

• Results: A decrease in radial pulse or pulse disappearance indicates a positive test result for thoracic
outlet syndrome.

Page 40

1290
Solution for Question 27:
Correct Option C - Pulse oximetry:
• This is a case of acrocyanosis;
• Acrocyanosis is caused by the narrowing (constriction) of small arterioles (tiny arteries) toward the
end of the arms and legs.
• Acrocyanosis is the blueness of the extremities with a pink body. It is symmetrical.
• It is characterized by a mottled blue or red discoloration of the skin on the fingers and wrists and the
toes and ankles.
• Profuse sweating and coldness of the fingers and toes can also occur, exacerbating cold-mimicking
Raynaud disease.
• Routine newborn screening for congenital heart disease with pre- and post-ductal pulse oximetry is
recommended
• Pre-ductal pulse oximetry shows oxygen saturation in the right arm.
• Post-ductal pulse oximetry shows oxygen saturation in either leg.
Incorrect Options:
Option A - X-ray of the chest:
• X-ray of the chest can be indicated in newborns with neonatal respiratory distress syndrome,
congenital diaphragmatic hernia, foreign body, etc.
• The initial patient evaluation with mild respiratory distress, which significantly improved without any
intervention, does not indicate a need for a chest x-ray.
Option B - ECG:
• ECG screening in neonates can be done for early detection of congenital long QT syndrome (loss of
function mutation in K+ channels).
• Romano-Ward syndrome- autosomal dominant with no deafness.
• Jervell and Lange-Nielsen syndrome- autosomal recessive with sensorineural deafness
Option D - 100% oxygen:
• 100% oxygen administration can help differentiate between respiratory and cardiac causes.
• 100% oxygen therapy can improve most pulmonary causes but does not improve right-to-left shunting
cardiac causes, which mostly present as central cyanosis, not acrocyanosis.

Solution for Question 28:


Option A: Intermittent claudication

Page 41

1291
• In this case, the patient presents with pain in the calf muscles during walking. There are weak
peripheral pulses and an audible bruit. All these are suggestive of intermittent claudication.
• Intermittent claudication is a debilitating condition characterized by cramping pain usually felt in calf
muscles by walking. It is aggravated by exercise and relieved by rest, either sitting or standing.
• Claudication distance is the distance that a patient can walk without stopping due to pain. It is
decreased by general health conditions, e.g. anaemia and heart failure, and by increasing work
demands.
• Ankle-brachial pressure index is used to assess arterial occlusion. Its normal range is 0.9-1.3. A value
below 0.9 indicates intermittent claudication.
• Treatment includes smoking cessation and managing diabetes, hypertension, and dyslipidemia.

Option B: Buerger’s disease


• Buerger’s disease (Thromboangiitis obliterans) is an inflammatory disease of small and medium-sized
arteries in the upper and lower extremities.
• It is seen in male smokers. Treatment includes cessation of smoking. It cannot reverse the symptoms
but can stop further progression.
Option C: Frostbite
• Frostbite is caused by exposure to cold. It damages the vessel wall resulting in swelling and pain.
• Treatment is gradual rewarming and analgesics.
Option D: Acrocyanosis
• Acrocyanosis usually occurs in young females. It is characterized by mottled cyanosis of the toes and
fingers.
• It may be accompanied by paraesthesia and chilblains.

Page 42

1292
Venous Disorders
1. A 30-year-old professional with tetraplegia who had sustained a complete spinal cord injury,
functionally, at C-3 ± 4 ± 5 levels with partial sparing of the left biceps came to the clinic for a regular
checkup. On examination, the doctor noted a small ulcer on the left gluteal region of the patient. Which
of the following interventions should be included in the patient's plan of care to prevent the
development of pressure ulcers?
(or)
Which intervention should be included in the plan of care for a 30-year-old professional with a complete
spinal cord injury and a small ulcer on the left gluteal region to prevent the development of pressure
ulcers?
A. Vigorously massage lotion into bony prominences
B. Post a turning schedule at the patient’s bedside
C. Turn and reposition the patient at least once every 8 hours
D. Slide the patient, rather than lifting, when turning
----------------------------------------
2. If the two index fingers are placed firmly side by side on a vein and the finger nearer the heart is
moved so as to empty a short length of the vein, the release of the distal finger will allow the speed of
venous refilling to be observed (image). What is the sign called?
(or)
Identify the sign shown below ?

A. Macewen's sign
B. Harvey's sign
C. Homan's sign
D. Brown's sign
----------------------------------------
3. Which statement is true about the procedure shown in the image?
(or)
A 30-year-old man presents to the outpatient department with the complaint of dull aching, swelling,
and tingling sensation in the leg for 2 weeks. He further explains that the pain gets worse by standing
and relieving by putting his legs up. On physical examination, there is bilateral pedal edema and visible

1293
varicose veins on the surface. The physician used a procedure for the treatment of this disease. Which
statement is true about the procedure shown in the image?

A. Used for incompetent perforators


B. The procedure is done with the help of a microscope
C. The vascular clip is applied above the level of deep fascia
D. All of the above
----------------------------------------
4. What is the most common complication that can be expected after bilateral varicose vein stripping ?
(or)
A 54-year-male patient got admitted to a hospital with a chief complaint of bulging veins in both lower
limbs. On the Doppler test, abnormal blood flow in lower limbs was noted. A diagnosis of bilateral
varicose veins in the lower limbs was made. The patient was posted for surgery, and the varicose veins
were successfully stripped bilaterally from the ankle to the groin. What is the most common
complication that can be expected after varicose vein stripping?
A. Infection
B. Hemorrhage
C. Ecchymosis
D. Thromboembolism
----------------------------------------
5. A 40-year-old female patient presents with large bulging veins in her bilateral extremities along with
tiredness, heaviness, and throbbing pain in the lower legs. A duplex Ultrasound examination shows an
underlying feeding vein that was directly feeding the surface varicose veins in the thigh. Which of the
following drugs can be used for sclerotherapy of varicose veins?
(or)
Which drug can be used for sclerotherapy of varicose veins ?
A. Metandienone
B. Polidocanol
C. Ethanol
D. Oxandrolone
----------------------------------------

Page 2

1294
6. A 40-year-old female who has had a history of smoking for the past 15 years comes to the OPD with
complaints of dilated, tortuous veins in her right leg for the past five months. Doppler reveals a
saphenofemoral junction and perforator incompetence in the right lower limb. For her disease, the
doctor prescribed injection sclerotherapy. What is the size of the veins that can be treated by
sclerotherapy?
(or)
What is the size of veins that can be treated by sclerotherapy in a 40-year-old female diagnosed with
varicose veins?
A. 8 mm
B. <3 mm
C. 4 mm
D. 6 mm
----------------------------------------
7. A 35-year-old bus driver presents to the hospital with a complaint of pain in the legs. The patient also
complains that there is a continuous burning sensation in his left foot. On examination, the doctor
notices dilated, tortuous veins on the back side of the left leg. Doppler scan, revealed valve
incompetence. The doctor advises him to go for Cockett and Dodd's operation. Cockett and Dodd's
operation is for:
(or)
What is the indication for cockett and Dodd's operation in a 35-year-old bus driver diagnosed with
varicose veins?
A. Saphenofemoral flush ligation
B. Sub-fascial ligation
C. Deep vein thrombosis
D. Diabetic foot
----------------------------------------
8. A 10-year-old child is admitted to the paediatric ward for the treatment of pneumonia. He is on
intravenous medication. The on-duty staff calls the doctor to check the child, who is continuously
irritable and his arm is red, warm, and tender to the touch. The superficial veins are inflamed. What is
the main cause of this condition mentioned above?
(or)
What is the likely cause of red, warm, and tender arm with inflamed superficial veins in a 10-year-old
child on IV medication admitted for the treatment of pneumonia?
A. Intravenous catheters/infusion
B. DVT
C. Varicose veins
D. Trauma
----------------------------------------
9. A 50-year-old male presents with a complaint of pain in the right foot with a severe burning
sensation. On examination, veins are dilated and tortuous. There is an ulcer that is causing serous
discharge on the medial malleolus too. Doppler scan, revealed incompetent perforators. According to

Page 3

1295
the CEAP classification for chronic venous disorders, how would you classify the patient's disease?
(or)
What is the CEAP classification for chronic venous disorders in a 50-year-old male with right foot pain
with severe burning sensation ,dilated and tortuous veins, and medial malleolus ulcer with serous
discharge?
A. C1
B. C3
C. C4a
D. C6
----------------------------------------
10. A 54-year-old male who has been bedridden for the past six weeks complains of severe cramping
in his left leg for the past three days. He is unable to move his legs due to pain. The calf muscles are
warm and extremely tender. He undergoes a D-dimer test, which is markedly raised. Doppler
ultrasound shows clots in the deep veins of the leg. Which is the high-risk factor for the disease in the
patient?
(or)
What is the high-risk factor for deep venous thrombosis in a 54-year-old bedridden patient?
A. Mobile Patient
B. Trauma
C. Major orthopaedic surgery of lower limbs
D. Surgery lasting for ≥30 minutes
----------------------------------------
11. A 43-year-old male presents to the hospital complaining of pain in his right calf muscles. The patient
also complains about swelling and redness in the affected area. Examination reveals that the patient
has calf pain with knee flexion and dorsiflexion of the ankle. Calf compression against the tibia also
causes pain. Duplex ultrasound results are suggestive of DVT. Which of the following statements is true
about the diagnosis?
(or)
Which of the following statements is true about the diagnosis of deep vein thrombosis (DVT) in a
43-year-old male based on Duplex ultrasound results?
A. MC site of DVT leading to pulmonary embolism is the soleal vein
B. The earliest sign of DVT is a Calf pain
C. Homan's sign is the resistance of the calf muscles to forcible dorsiflexion
D. Filling defects inflow and lack of compressibility indicate the presence of an ulcer on duplex
ultrasound
----------------------------------------
12. Which one of the following statements is true regarding a 55-year-old male watchman who presents
with swelling and bulging of veins in the lower limbs, along with dilated tortuous veins in the right lower
limb?
(or)

Page 4

1296
A 55-year-old male, a watchman by occupation, comes to the OPD complaining of swelling and bulging
of veins in the lower limbs. He also describes feeling restlessness, occasional cramping bilaterally, and
tiredness at the end of the day. Examination reveals dilated, tortuous veins in the patient's right lower
limb, extending from the middle thigh to the leg in a standing position. Doppler test results show venous
stasis. On the basis of the patient's condition, which of the following is true?
A. Most common symptom: Dull aching pain in the veins after prolonged standing the whole day
B. Clinical investigation is the choice for the diagnosis of varicose veins
C. Bisgard's Regime is not used for venous ulcers
D. Trendelenberg operation is a gold standard treatment for varicose veins
----------------------------------------
13. An 81-year-old woman in good health arrives at the hospital with a traumatic wound on her right leg
that has not healed after two months of self-care. She has a medical history of osteoarthritis. Her leg is
edematous and has a pale appearance. The skin around the wound is discoloured, and the wound size
is 1.1 cm x 3.9 cm with foul-smelling discharge. The patient also complains of pain, itching, and
swelling. Which of the following statements is true about the patient's condition?
(or)
Which of the following statements is true about an 81-year-old lady diagnosed with a venous leg ulcer
and generally in good health?
A. Hypopigmentation of the skin is seen in long-standing cases due to lichenification
B. Inverted Champagne bottle leg is seen in varicose veins
C. Incidence of re-ulceration after management is rare
D. Ulcers characteristically develop in the bones of the gaiter region
----------------------------------------
14. What is the likely diagnosis for a 20-year-old patient presenting with varicose veins in the left leg
since birth, dilation increasing with walking and standing decrease while sitting, cutaneous
hemangioma involving the face, enlarged toes, and multiple varicosities all over the legs and lower
thigh region on colour Doppler results?
(or)
A 20-year-old patient presents to the hospital with a complaint of varicose vein in her left leg since birth.
He also complains that the dilation increases while walking and standing and decreases while sitting.
On examination, it is seen that the patient has cutaneous hemangiomas involving the face with
enlarged toes. Colour Doppler results also show multiple varicosities all over the leg and lower thigh
region. What is likely to be the diagnosis based on the patient's condition?
A. Klippel-Trenaunay syndrome
B. Tricuspid regurgitation
C. Deepvein thrombosis
D. Kasabach-Merritt syndrome
----------------------------------------
15. A 43-year-old male race-course farrier presents to his family doctor complaining of persistent pain
and numbness, fatigue, and frequent cramps extending bilaterally throughout the upper limbs
(especially on the right side). Colour flow Doppler USG is prescribed, which reveals bilateral functional
stenosis of the axillary-subclavian vein, which is more severe on the right side. A subsequent

Page 5

1297
angiography-assisted CT scan shows the presence of right subclavian venous thrombosis. What is
likely the management plan for this patient?
(or)
What is the likely management plan for a 43-year-old male with bilateral functional stenosis of the
axillary-subclavian vein, more severe on the right side, and symptoms of persistent pain, numbness,
fatigability, and frequent cramps in the upper limbs?
A. IV anticoagulants + Rest should be advised as the provocative factor is his occupation
B. Treated with IV anticoagulants alone
C. Anticoagulation therapy followed by catheter-directed Thrombolytic therapy
D. Ultrasound-guided embolectomy
----------------------------------------
16. A 46-year-old man presents to the outpatient department with complaints of facial plethora, neck
swelling, and purplish discoloration across his chest. He is overweight with small-cell lung cancer (on
treatment). On physical examination, a diffuse edema in the neck and dilated, engorged blood vessels
on the chest (image) were noted. Except for mild dyspnea when supine and cough, the patient was
asymptomatic and in no acute distress. Labs revealed a normal CBC and BMP. What is the probable
clinical diagnosis from the given data?
(or)
What is the probable clinical diagnosis for a 46-year-old overweight male with small-cell lung cancer
presenting with facial plethora, neck swelling, and purplish discoloration across the chest?

A. Pulmonary hypertension
B. Congestive heart failure secondary to lung cancer
C. Superior vena cava syndrome
D. Paraneoplastic syndrome of SCLC
----------------------------------------
17. A 29-year-old man presents with complaints of pain, heaviness, and swelling on the right leg. He
has had a history of varicose veins since the age of 12. The patient underwent below-knee stripping of
the long saphenous vein, after which the symptoms completely resolved, along with visible varicose
veins. Which of the following is the most common complication that can occur in this patient?
(or)
What is the most common complication following below-knee stripping of the long saphenous vein in
the varicose vein, which resolved after the procedure?

Page 6

1298
A. Hemorrhage
B. Thromboembolism
C. Neuralgia
D. Infection
----------------------------------------
18. Which of the following statements is true about a 29-year-old man presenting with painful swelling
of the right arm, shortness of breath, redness, and cyanosis on the right arm, and dilated visible veins
across the upper arm and shoulder?
(or)
A 29-year-old man presents to the outpatient department with complaints of painful swelling of the right
arm and shortness of breath for one week. On physical examination, there is redness and cyanosis on
the right arm and dilated, visible veins across the upper arm and shoulder. There is a history of
smoking one pack per day. Which of the following is true about this condition?
A. Also known as Effort thrombosis
B. Associated with Jugular-subclavian venous thrombosis
C. Patients usually have no history of performing prolonged activities
D. Results in damage to the femoral vein
----------------------------------------
19. A 60-year-old woman presents to the outpatient department with complaints of right calf pain and
swelling. She further complained of shortness of breath and sharp chest pain. She underwent a left
hemicolectomy for diverticular disease. The physical examination shows unilateral leg swelling. The
physician made the diagnosis based on signs and symptoms. What is the most appropriate
management in this case?
(or)
What is the most appropriate management of DVT in 60 year old women?
A. Pneumatic compression stockings
B. Heparin
C. Thrombolysis
D. Inferior vena cava filter
----------------------------------------

Correct Answers
Question Correct Answer

Question 1 2
Question 2 2
Question 3 1
Question 4 3
Question 5 2

Page 7

1299
Question 6 2
Question 7 2
Question 8 1
Question 9 4
Question 10 3
Question 11 3
Question 12 1
Question 13 2
Question 14 1
Question 15 3
Question 16 3
Question 17 3
Question 18 1
Question 19 2

Solution for Question 1:


Option B: Post a turning schedule at the patient’s bedside
• Pressure sore: tissue necrosis and ulceration due to prolonged pressure
• Incidence site: Ischium (m/c site), Greater trochanter, Sacrum, Heel, Malleolus (lateral > medial),
Occiput
• Staging of pressure ulcer
No blanchable erythema of skin without breach in epidermis; Early superficial ulcer

Partial thickness less including epidermis+ dermis; Late superficial ulcer

Page 8

1300
Full thickness skin loss involving subcutaneous tissues but not through underlying fascia; Early deep ul
cer

Full thickness skin loss involving subcutaneous tissue, muscle bone, joints or tendon; Late deep ulcer

Page 9

1301
• The pathogenesis of pressure sores is local tissue pressure above that of perfusion pressure
produces ischemic necrosis that is directly proportional to the duration and degree of pressure.
• Muscle and fat are more susceptible to pressure than skin.
• The patient on the bed should be turned at least every 2 hours(not every 8 hours), the
wheelchair-bound patient being taught to lift themselves off their seat for 10 seconds every 10 minutes.
• A turning schedule with a signing sheet will ensure that the patient gets turned and, thus, help prevent
pressure ulcers.
Option A: Vigorously massage lotion into bony prominences
• According to the most recent guidelines, massage should be avoided.
• Vigorous massaging of lotion into bony prominences has been used for the treatment of pressure
ulcers.
Option C: Turn and reposition the patient at least once every 8 hours
• Repositioning the patient can be used to prevent pressure ulcers.
• Repositioning and turning the patient every 2 hours is indicated, not 8 hours, to prevent pressure
ulcers.
Option D: Slide the patient, rather than lifting, when turning
• Frequent repositioning of the patient is recommended in patients with pressure ulcers.
• It is recommended to lift the patient, not slide.

Solution for Question 2:


Correct Option B: Harvey's sign
• Related to venous filling after emptying a length of the vein.
• Delayed venous refilling is called Harvey’s sign.

Page 10

1302
• According to Harvey’s theory, the blood is pumped throughout the body by the heart in ceaseless
motion.
• Through his ligature experiments, Harvey showed that the blood only flows in one direction from the
periphery to the heart.
• Finally, he concluded that the blood had to return to the heart.
• The two index fingers are placed side by side on a vein. The fingers are now pressed firmly and the
finger nearer the heart is moved proximally keeping the steady pressure on the vein so as to empty the
short length of the vein between the two fingers.
• The distal finger is now released. This will allow venous refilling to be observed.This is known as
Harvey's sign
• This is poor in ischaemic limb and is increased in arteriovenous fistula.
Incorrect options
Option A: Macewen's sign
• Macewen's sign is used to detect hydrocephalus and brain abscesses.
• The percussion on the skull near the junction of temporal, frontal, and parietal bones reveals an
unusually resonant sound.
Option C: Homan's sign
• Homan's sign is used for the diagnosis of Deep vein thrombosis.
• A positive Homan's sign indicates the calf at the level of dorsiflexion of the foot.
Option D: Brown's sign
• Brown’s sign is used for the diagnosis of an ear tumour which is known as glomus jugulare.
• The air pressure on the tympanic membrane may cause blanching of the tumour during pneumatic
otoscopy.

Solution for Question 3:


Correct Option A - Used for incompetent perforators:
• In the above-given case, the sign and symptoms of leg pain with swelling and visible varicose veins
on the surface are suggestive of chronic insufficiency.
• Chronic venous insufficiency is a common condition that typically involves lower extremity edema,
trophic skin changes, and discomfort secondary to venous hypertension.
• Classification of CVI C0: no obvious feature of venous disease C1: the presence of reticular or spider
veins C2: obvious varicose veins C3: presence of edema but no skin changes C4: skin discoloration,
pigmentation C5: ulcer that has healed C6: acute ulcer
• C0: no obvious feature of venous disease
• C1: the presence of reticular or spider veins
• C2: obvious varicose veins
• C3: presence of edema but no skin changes
• C4: skin discoloration, pigmentation

Page 11

1303
• C5: ulcer that has healed
• C6: acute ulcer
• Subfascial endoscopic perforator vein surgery (SEPS) is a new endoscopic technique for the
management of chronic insufficiency due to incompetent perforator veins.
• SEPS involves the insertion of a rigid endoscope through the skin and fascia to a plane below the
deep fascia, such that perforator veins are visible as they exit the muscles.
• These perforator veins are dissected free from surrounding tissue and closed with the help of metal
clips.
• Complications of surgery: Infection, Injury to the arterial system, Nerve injury, Poor cosmesis,
Scarring
• C0: no obvious feature of venous disease
• C1: the presence of reticular or spider veins
• C2: obvious varicose veins
• C3: presence of edema but no skin changes
• C4: skin discoloration, pigmentation
• C5: ulcer that has healed
• C6: acute ulcer
Incorrect Options:
Option B - The procedure is done with the help of a microscope:
• SEPS is a minimally invasive surgery.
• The procedure is done with the help of an endoscope, not a microscope.
Option C - The vascular clip is applied above the level of deep fascia:
• The vascular clip is applied below the level of the deep fascia, not above the deep fascia.
Option D - All of the above:
• SEPS is used for incompetent perforators.
• SEPS is a procedure used for chronic insufficiency.

Solution for Question 4:


Option C: Ecchymosis
• Complications of varicose vein surgery Recurrence is the most common incidence, which occurs after
surgery in up to 10% Hematoma or Ecchymosis is the most common cause of discomfort after varicose
vein surgery. Sensory nerve injury can also be caused after varicose veins surgery. The greater
saphenous vein should only be stripped to just below the knee to avoid damage to the accompanying
saphenous nerve. The sural nerve must be carefully dissected off the lesser saphenous vein at the
ankle.
• Recurrence is the most common incidence, which occurs after surgery in up to 10%
• Hematoma or Ecchymosis is the most common cause of discomfort after varicose vein surgery.

Page 12

1304
• Sensory nerve injury can also be caused after varicose veins surgery.
• The greater saphenous vein should only be stripped to just below the knee to avoid damage to the
accompanying saphenous nerve.
• The sural nerve must be carefully dissected off the lesser saphenous vein at the ankle.
• Recurrence is the most common incidence, which occurs after surgery in up to 10%
• Hematoma or Ecchymosis is the most common cause of discomfort after varicose vein surgery.
• Sensory nerve injury can also be caused after varicose veins surgery.
• The greater saphenous vein should only be stripped to just below the knee to avoid damage to the
accompanying saphenous nerve.
• The sural nerve must be carefully dissected off the lesser saphenous vein at the ankle.
Option A: Infection
• Infection is not associated with long saphenous vein stripping to the knee.
• There are rare chances of infection after long saphenous vein stripping to the knee, but sometimes it
can cause cellulitis.
Option B: Hemorrhage
• Hemorrhage is not the complication of below-knee stripping.
Option D: Thromboembolism
• The most common complication after varicose vein surgery is Ecchymosis.
• It can be caused by lower limb injury or fracture and surgery of the lower extremities.

Solution for Question 5:


Option B: Polidocanol
• Sclerosing agents (Polidocanol) are irritants that cause inflammation, coagulation, and, ultimately,
fibrosis when injected into haemorrhoids or varicose veins.
• These can be used only for local injection.
• Other sclerosing agents: 5% phenol in Almond oil, 5% Ethanolamine oleate, 3% sodium tetradecyl
sulphate, hypertonic solution
• Sclerosing agents are used in esophageal varices, hemorrhoids and varicose veins
Option A: Metandienone
• Metandienone is an androgen and anabolic steroid which is used for bulky cycles.
• It is contraindicated in varicose veins because it can cause a recurrence of the varicose veins.
Option C: Ethanol
• Ethanol is an alcohol.
• Ethanol worsens the symptoms of varicose veins, so it is contraindicated.
Option D: Oxandrolone

Page 13

1305
• Oxandrolone is an anabolic steroid which is used to gain weight.
• Varicose veins can also be caused by steroids, so Oxandrolone is contraindicated in varicose veins.

Solution for Question 6:


Option B:< 3 mm
• In this case, the patient has dilated tortuous veins, which are suggestive of varicose veins.
• Diameter 4-5 mm - varicose, 1-4 mm reticular vein, <1mm -telangiectasia
• The varicose veins are dilated tortuous veins. In this, there is a weakening of vessel valves.
• It is caused due to prolonged standing and walking, which causes increased pressure in veins.
• The gold standard treatment for the varicose vein is Endothermal ablation.
• Surgery done for the varicose vein is Trendelenburg operation.
• For intermediate sizes, sclerotherapy and laser therapy are done.
• Sclerotherapy is useful for dealing with minor varicosities (<3 mm) and recurrences in the calf and
lower leg.
• The amount of sclerosants used in the treatment of varicose veins is 0.5 ml.
Option A: 8 mm
• Sclerotherapy cannot be used for veins of 8mm.
• Surgery is usually done for large-sized veins.
Option C: 4 mm
• 4 mm is a larger size to treat with sclerotherapy.
• Endothermal ablation can be done.
Option D: 6 mm
• Sclerotherapy cannot be used for veins of 6mm.
• Surgery is usually done for large-sized veins.

Solution for Question 7:


Option B: Sub-fascial ligation
• Cockett and Dodd's operation is a Sub-fascial ligation of incompetent perforators.
• Dilated and tortuous veins with valve incompetence showed that it is a case of varicose veins.
• In Cockett and Dodd's operation, perforators are ligated deep to the deep fascia through incisions in
the anteromedial side of the leg.
• It avoids the recurrence of the disease.
• Various named perforators in the lower limb:

Page 14

1306
• Below the level of Medial malleolus: Inframalleolar/May or Kuster perforator
• In the medial calf: Cockett perforator
• Just below the knee: Boyd's perforator
• Just above knee: Dodd's perforator
• At the level of Abductor canal: Hunterian perforator
Option A: Saphenofemoral flush ligation
• It is the isolated ligation of the saphenofemoral junction.
• It is an effective treatment for a chronic leg ulcer.
Option C: Deep vein thrombosis
• In this, clot forms in the deep vein.
• For deep vein thrombosis, blood thinners are given.
Option D: Diabetic foot
• Diabetic foot occurs due to uncontrolled diabetes.
• Treatment of diabetic foot depends on the condition of the foot.

Solution for Question 8:


Option A: Intravenous catheters/infusion
• This patient has redness and tenderness around the area of the IV catheter, which is a suggestive
feature of superficial thrombophlebitis., most commonly caused by infusions of IV fluids.
• Other causes: Trauma, Buerger's disease, Malignancies
• Clinical features: affected veins are- cord like on palpation, tender, erythematous skin around the vein
• It is the inflammation of veins just below the skin.
• It is treated with non-steroidal anti-inflammatory drugs and wearing compression stockings.
Option B: DVT
• DVT is a disease of deep veins.
• The patient's signs and symptoms are suggestive of superficial thrombophlebitis rather than DVT.
Option C: Varicose veins
• Varicose veins are defects in the valves.
• The patient's signs and symptoms are suggestive of superficial thrombophlebitis rather than varicose
veins.
Option D: Trauma
• Trauma causes hemorrhagic instability.
• There is no trauma history in the question stem.

Page 15

1307
Solution for Question 9:
Option D: C6
• It is a case of a varicose vein due to dilated and tortuous veins with the presence of an active venous
ulcer.
• CEAP is a classification of chronic venous disease.
• According to CEAP(Clinical-Etiology-Anatomy-Pathophysiology), C6 is for active venous ulcer, which
is in the above-mentioned case.
• CEAP classification for the chronic venous disorder is as follows: C0: No signs of venous disease C1:
Telangiectasia or reticular veins C2: Varicose veins. C3: Edema C4a: Pigmentation or eczema C4b:
Lipodermatosclerosis or atrophie Blanche C5: Healed venous ulcer C6: Active venous ulcer
• C0: No signs of venous disease
• C1: Telangiectasia or reticular veins
• C2: Varicose veins.
• C3: Edema
• C4a: Pigmentation or eczema
• C4b: Lipodermatosclerosis or atrophie Blanche
• C5: Healed venous ulcer
• C6: Active venous ulcer
• C0: No signs of venous disease
• C1: Telangiectasia or reticular veins
• C2: Varicose veins.
• C3: Edema
• C4a: Pigmentation or eczema
• C4b: Lipodermatosclerosis or atrophie Blanche
• C5: Healed venous ulcer
• C6: Active venous ulcer

Solution for Question 10:


Option C: Major orthopaedic surgery of lower limbs
• It is a case of deep venous thrombosis as the patient is bedridden and has throbbing pain in the leg.
• Calf muscles are warm and tender to the touch.
• It is a clot in the deep veins of the leg.
• It is an emergency condition that presents severe pain in the legs with redness and tenderness of the
calf muscles.

Page 16

1308
• Major orthopaedic surgery of the lower limb is a high-risk case for DVT as the patient remains
immobilized for a longer time.
• Risk factors thrombosis: Defective inhibition of coagulation factors Impaired clot lysis Disease/
syndrome Physiological states: pregnancy, obesity, prolonged immobi
• Defective inhibition of coagulation factors
• Impaired clot lysis
• Disease/ syndrome
• Physiological states: pregnancy, obesity, prolonged immobi
• Defective inhibition of coagulation factors
• Impaired clot lysis
• Disease/ syndrome
• Physiological states: pregnancy, obesity, prolonged immobi
• General urological in patients > 40 years
• Extensive pelvic or abdominal surgery
• Major orthopaedic surgery of lower limbs
• General surgery in patients ≥ 40 years
• Surgery lasting for ≥ 30 minutes
• General surgery in patients < 40 years on OCPs
• Uncomplicated surgery in patients < 40 years without additional risk
• Minor surgery (<30 minutes) in patients < 40 years additional risk
Other Options
Option A: Mobile patient
• Ambulatory and mobile patients are unlikely to develop DVT due to the effective pumping of blood
through the veins.
Option B: Trauma
• Trauma is not a risk factor for deep venous thrombosis.
• It leads to the hemorrhagic issue.
Option D: Surgery lasting for ≥30 minutes
• Minor or short surgery is not a high-risk factor.
• The patient gets mobilized early; hence DVT chances are less.

Solution for Question 11:


Option C:Homan's sign is the resistance of the calf muscles to forcible dorsiflexion

Page 17

1309
• In this case, as the patient is showing pain in the calf with knee flexion and ankle dorsiflexion and pain
due to calf compression against the tibia suggests the diagnosis of Deep vein thrombosis (DVT).
• Pain due to calf compression against the tibia is the moses' sign.
• Homan's sign is characterised by discomfort in the calf muscles due to forced dorsiflexion of the foot
when the knee of the person is straightened.
• It has been a time-honoured sign of Deep vein thrombosis.
• It is also called the dorsiflexion sign.
• To elicit this sign, the patient's thigh and foot are supported by the examiner's hands. Slightly bend the
leg at the knee, followed by abrupt dorsiflexion of the ankle (force placed on the toes of the foot).
• If this results in deep calf pain or the patient involuntarily resists the flexion, Homan's sign is positive.
• Homan's sign is not very sensitive and specific. It is present in less than 30% of patients with
confirmed DVT and can be present in more than half of patients without DVT.
• Only on the basis of these signs, DVT cannot be diagnosed; hence duplex doppler ultrasound is done,
and its positive results, along with Homan's and Moses' signs, make the diagnosis of DVT.
Option A: MC site of DVT leading to pulmonary embolism is the soleal vein
• The most common site of DVT is Calf or soleal vein.
• But the most common site of DVT leading to pulmonary embolism is the Femoropopliteal vein.
Option B:The earliest sign of DVT is a Calf pain
• The most common characteristic feature of DVT is pain and swelling, especially in the calf of one
lower limb.
• However, the earliest sign of DVT is calf tenderness.
Option D:Filling defects inflow and lack of compressibility indicate the presence of an ulcer on duplex ul
trasound
• A duplex ultrasound is done to diagnose DVT.
• It is the investigation of choice for diagnosis of DVT, which is filling defects inflow and lack of
compressibility and indicates the presence of thrombosis.

Page 18

1310
Solution for Question 12:
Option A: Most common symptom: Dull aching pain in the veins after prolonged standing the whole day
• In this case, as the patient's occupation is watchman, he has to stand for a prolonged period. The
patient is also showing dilated tortuous veins. Both symptoms are indications of varicose veins.
• If a person has to sit or stand for a long time, the blood in the leg veins can pool, leading to increased
blood pressure.
• Deep and perforating veins can withstand short periods of increased pressure. But prolonged
standing can result in the stretching of veins. This stretching can result in the weakening of the walls of
veins and damage vein valves and resulting in an enlarged and swollen vein called varicose vein
formation.
• It is caused by blood reflux and stagnation due to the dysfunction of the superficial veins in the lower
extremities. Most people show no obvious discomfort, but some patients can face pain, soreness,
swelling and even bleeding.
• Symptoms of varicose vein include: Enlarged veins in purple-blue colour Aching, heavy and
uncomfortable legs Swollen feet and ankles Worsen pain while sitting and standing Muscle cramp in
legs Dry skin and colour changes in the lower leg
• Enlarged veins in purple-blue colour
• Aching, heavy and uncomfortable legs
• Swollen feet and ankles
• Worsen pain while sitting and standing
• Muscle cramp in legs
• Dry skin and colour changes in the lower leg
• Enlarged veins in purple-blue colour
• Aching, heavy and uncomfortable legs
• Swollen feet and ankles
• Worsen pain while sitting and standing
• Muscle cramp in legs
• Dry skin and colour changes in the lower leg

• Treatment: Endothermal Ablation (EVLA and RFA) is a gold standard treatment for varicose veins.
Surgery is done for Varicose veins: Trendelenburg operation. Sclerotherapy for veins <3 mm in
diameter
• Endothermal Ablation (EVLA and RFA) is a gold standard treatment for varicose veins.
• Surgery is done for Varicose veins: Trendelenburg operation.
• Sclerotherapy for veins <3 mm in diameter
• Endothermal Ablation (EVLA and RFA) is a gold standard treatment for varicose veins.
• Surgery is done for Varicose veins: Trendelenburg operation.

Page 19

1311
• Sclerotherapy for veins <3 mm in diameter
Option B: Clinical investigation is the choice for the diagnosis of varicose veins
• Varicose veins cannot be diagnosed only on the basis of clinical examination.
• Duplex ultrasound is considered the ideal investigation of choice for the diagnosis of varicose veins.
• It measures the speed of blood flow and the structure of the veins of the legs.
Option C: Bisgaard's Regime is not used for venous ulcers
• Bisgaard's Regime is Limb elevation.
• Compression stockings + Massage for varicose ulcers
• Bisgaard’s Regime is used for venous ulcers.
Option D: Trendelenberg operation is a gold standard treatment for varicose vein
• Endothermal Ablation (EVLA and RFA) is a gold standard treatment for varicose veins.
• It includes Endovenous laser ablation, which restores the normal blood flow of the legs.
• Radiofrequency ablation (RFA) seals off the varicose vein and redirects the blood flow to healthier
veins in the legs.

Solution for Question 13:


ANSWER
Option B: Inverted Champagne bottle leg is seen in varicose veins
• As the patient has a foul-smelling wound on the leg with pain, itching, swelling, and skin discoloration
around the ulcer, the diagnosis of a venous leg ulcer is made.
• Venous leg ulcer characteristically develops in the skin of the gaiter region, which is the area between
muscles of the calf and ankle.
• It is a skin ulcer on the leg due to low blood circulation in the limb.
• It is considered the most common type of leg ulcer.
• Lipodermatosclerosis is a chronic inflammatory condition that is characterized by subcutaneous
fibrosis. It results in the hardening of the skin on the lower legs.
• It is caused by:
• Incompetent venous valves Venous outflow obstruction Dysfunction of the calf muscle pump
• Incompetent venous valves
• Venous outflow obstruction
• Dysfunction of the calf muscle pump
• Lipodermatosclerosis presents with hyperpigmentation of the skin involving one or both of the lower
legs, which shows a characteristic appearance of an "inverted champagne bottle."
• Incompetent venous valves
• Venous outflow obstruction
• Dysfunction of the calf muscle pump

Page 20

1312
Incorrect Options:
Option A: Hypopigmentation of the skin is seen in long-standing cases due to lichenification
• A venous leg ulcer is characterised by hyperpigmentation.
• Almost all venous ulcers have surrounding lipodermatosclerosis, which is thickening, pigmentation,
inflammation and induration of calfskin.
Option C: Incidence of re-ulceration after management is rare
• Venous leg ulcer has high recurrence.
• Even after conservative management, there is a high recurrence rate of 20-30% in most cases.
Option D: Ulcers characteristically develop in the bones of the gaiter region
• A venous ulcer is formed on the skin.
• Most precisely, on the skin of the gaiter region.
• The Gaiter region is the area between the muscles of the calf and ankle.

Solution for Question 14:


Option A: Klippel-Trenaunay syndrome
• In this case, the patient shows pulsating varicose veins, cutaneous hemangioma, and enlarged toes; it
indicates AV Fistula.
• Klippel-Trenaunay syndrome is a rare congenital malformation that involves blood and lymph vessels
and abnormal development of soft tissue such as skin and bone tissue.
• Congenital AV fistula and varicose veins are the characteristics of Klippel-Trenaunay syndrome.
• It is a genetic disorder due to mutations, most commonly in the PIK3CAgene
• It is characterised by the following: Congenital AV (arteriovenous) fistula. Cutaneous hemangioma
Varicose veins Hypertrophy of involved extremity Absence of deep venous system

Page 21

1313
• Congenital AV (arteriovenous) fistula.
• Cutaneous hemangioma
• Varicose veins
• Hypertrophy of involved extremity
• Absence of deep venous system
• Symptoms include: Port-wine stain: The pink to reddish-purple port wine stain by birth, which is
caused by capillaries in the top layer of the skin, mostly covers part of one leg. Vein
malformations: These are varicose and twisted veins on the leg's surface. It may also result in deeper
malformed veins in the arms, legs, abdomen, and pelvic region. These varicose veins become more
obvious with age. Overgrowth of bones and soft tissue: This symptom usually remains on one leg or
arm and becomes obvious during infancy. It results in larger and longer extremities. Lymphatic system
malformations: This malformation results in extra lymphatic vessels that don't even work properly,
leading to fluid leakage into the tissues and swelling.
• Port-wine stain: The pink to reddish-purple port wine stain by birth, which is caused by capillaries in
the top layer of the skin, mostly covers part of one leg.
• Vein malformations: These are varicose and twisted veins on the leg's surface. It may also result in
deeper malformed veins in the arms, legs, abdomen, and pelvic region. These varicose veins become
more obvious with age.
• Overgrowth of bones and soft tissue: This symptom usually remains on one leg or arm and becomes
obvious during infancy. It results in larger and longer extremities.
• Lymphatic system malformations: This malformation results in extra lymphatic vessels that don't even
work properly, leading to fluid leakage into the tissues and swelling.
• ■■■■■■■The investigation of choice for evaluation of peripheral AV malformation is MRI.
• Conservative management by compressed stocking.
• Congenital AV (arteriovenous) fistula.
• Cutaneous hemangioma
• Varicose veins
• Hypertrophy of involved extremity
• Absence of deep venous system
• Port-wine stain: The pink to reddish-purple port wine stain by birth, which is caused by capillaries in
the top layer of the skin, mostly covers part of one leg.
• Vein malformations: These are varicose and twisted veins on the leg's surface. It may also result in
deeper malformed veins in the arms, legs, abdomen, and pelvic region. These varicose veins become
more obvious with age.
• Overgrowth of bones and soft tissue: This symptom usually remains on one leg or arm and becomes
obvious during infancy. It results in larger and longer extremities.
• Lymphatic system malformations: This malformation results in extra lymphatic vessels that don't even
work properly, leading to fluid leakage into the tissues and swelling.
Option B: Tricuspid regurgitation
• It is a condition when the flaps of the tricuspid valve present between the right atrium and right
ventricle do not close properly and results in a backward flow of blood.

Page 22

1314
• Symptoms include:
• Pulsing in the neck region Abnormal heart rhythm
• Pulsing in the neck region
• Abnormal heart rhythm
• Pulsing in the neck region
• Abnormal heart rhythm
Option C: Deep vein thrombosis
• Deep vein thrombosis (DVT) occurs due to a blood clot formation in a deep vein. These clots mostly
form in the lower leg, arm, pelvis and thighs.
• Symptoms include the following in the affected area of the body:
• Swelling Pain Tenderness Redness of the skin
• Swelling
• Pain
• Tenderness
• Redness of the skin
• Swelling
• Pain
• Tenderness
• Redness of the skin
Option D: Kasabach-Merritt syndrome
• It is characterised by a combination of enlarging vascular tumours (Hemangioma or AV malformation)
along with the following:
• Thrombocytopenia Microangiopathic hemolytic anaemia Consumptive coagulopathy Enlarging
vascular lesion
• Thrombocytopenia
• Microangiopathic hemolytic anaemia
• Consumptive coagulopathy
• Enlarging vascular lesion
• Thrombocytopenia
• Microangiopathic hemolytic anaemia
• Consumptive coagulopathy
• Enlarging vascular lesion

Solution for Question 15:


Correct options

Page 23

1315
Option C: Anticoagulation therapy followed by catheter-directed Thrombolytic therapy
• In this case, as the patient is farrier, he overuses his arm. History of overuse of the arm and functional
stenosis of the axillary-subclavian vein suggests the diagnosis of Axillary-subclavian venous
thrombosis (ASVT).
• It is a condition in which the blood vessel of the upper arm is compressed by nearby rib or muscle and
results in blood clots.
• ASVT may develop in the subclavian vein or axillary vein.
• ASVT is a form of DVT that usually results from sudden injury or overuse.
• High risk: sports requiring repetitive arm and shoulder motions
• Primary ASVT: Less common. History of performing repetitive and prolonged motion activities, leading
to damage to the subclavian vein, usually where it passes between the first rib and head of the clavicle.
• Less common.
• History of performing repetitive and prolonged motion activities, leading to damage to the subclavian
vein, usually where it passes between the first rib and head of the clavicle.
• Secondary ASVT: More common. Associated with an easily identifiable cause such as an indwelling
catheter or a hypercoagulable state.
• More common.
• Associated with an easily identifiable cause such as an indwelling catheter or a hypercoagulable
state.
• Treatment: Once ASVT is diagnosed, anticoagulation therapy should be initiated in order to prevent
pulmonary embolism (PE) and decrease symptoms. Patients diagnosed with acute symptomatic
primary ASVT are candidates for catheter-directed thrombolytic therapy.
• Once ASVT is diagnosed, anticoagulation therapy should be initiated in order to prevent pulmonary
embolism (PE) and decrease symptoms.
• Patients diagnosed with acute symptomatic primary ASVT are candidates for catheter-directed
thrombolytic therapy.
• Less common.
• History of performing repetitive and prolonged motion activities, leading to damage to the subclavian
vein, usually where it passes between the first rib and head of the clavicle.
• More common.
• Associated with an easily identifiable cause such as an indwelling catheter or a hypercoagulable
state.
• Once ASVT is diagnosed, anticoagulation therapy should be initiated in order to prevent pulmonary
embolism (PE) and decrease symptoms.
• Patients diagnosed with acute symptomatic primary ASVT are candidates for catheter-directed
thrombolytic therapy.
Other Options
Option A: IV anticoagulants + Rest should be advised as the provocative factor is his occupation
• Rest and elevation of the limb are the initial management for ASVT.
• As in this case, the patient's symptoms are severe; only rest cannot cure the damage.

Page 24

1316
Option B:Treated with IV anticoagulants alone
• Anticoagulants are blood thinners and prevent blot clot formation. But already present blood clots are
treated by Catheter-directed thrombolytic therapy.
• Catheter-directed thrombolytic therapy is used to treat acute ASVT. It dissolves the blood clot that can
travel to the lungs and can cause shortness of breath and chest pain and lead to death.
Option D:Ultrasound-guided embolectomy
• Ultrasound-guided embolectomy is performed when the blood clot has reached lungs and causes
pulmonary embolism.
• As the patient has not reached this stage, catheter-directed thrombolysis will be used for the
treatment.

Solution for Question 16:


Option C: Superior vena cava syndrome
• In the above-given case, neck swelling, skin discoloration, obesity, edema, and dilated engorged
blood vessels are suggestive of superior vena cava syndrome.
• The most common cause is lung cancer (small cell and squamous cell carcinoma), along with
lymphoma and metastatic tumours responsible for more than 90% of all SVC syndrome.
• In young adults, malignant lymphoma is the leading cause of SVC syndrome.
• Characteristic physical findings are dilated neck veins, an increased number of collateral veins
covering the anterior chest wall, cyanosis, and edema of the face, arms, and chest.
• The investigation of choice is CT scan.
• The most significant chest radiographic finding is the widening of the superior mediastinum (MC right
side)
• Treatment: Depends on the type of tumor Life threatening complication: tracheal obstruction →
tracheostomy in order to remove obstruction Radiotherapy for Non-small cell lung cancer, Metastatic
solid tumor Chemotherapy for SCLC, Lymphoma
• Depends on the type of tumor
• Life threatening complication: tracheal obstruction → tracheostomy in order to remove obstruction
• Radiotherapy for Non-small cell lung cancer, Metastatic solid tumor
• Chemotherapy for SCLC, Lymphoma
• Depends on the type of tumor
• Life threatening complication: tracheal obstruction → tracheostomy in order to remove obstruction
• Radiotherapy for Non-small cell lung cancer, Metastatic solid tumor
• Chemotherapy for SCLC, Lymphoma
Option A: Pulmonary hypertension
• Pulmonary hypertension is the rise of blood pressure in the pulmonary artery.
• It causes cyanosis, syncope, chest pain, palpitations, and shortness of breath.

Page 25

1317
Option B: Congestive heart failure secondary to lung cancer
• The treatment of lung cancer can cause cardiomyopathy and congestive heart failure.
• Congestive heart failure causes shortness of breath, pedal edema, palpitations, and persistent cough.
Option D: Paraneoplastic syndrome of SCLC
• Paraneoplastic syndrome of SCLC is caused by ectopic hormone production caused by neural
antigen expression from a cancer cell.
• SIADH is the common paraneoplastic syndrome associated with small-cell lung carcinoma.
• It causes increased peripheral pain, muscular pain, dementia, difficulty in swallowing, slurred speech,
loss of motor coordination, and seizures.

Solution for Question 17:


Option C: Neuralgia
• Ideally, the long saphenous vein should not be stripped below the knee as it causes injury to the
saphenous nerve, which is the most common serious complication.
• The incidence of saphenous nerve neuralgia is up to 7% following long saphenous vein stripping to
the knee.
• The most commonly injured nerve while stripping the short saphenous vein is the sural nerve, whose
incidence of injury may be as high as 20%. The vein of Giacomini connects the small saphenous vein
with the great saphenous vein.
• Other complications: Bruising or scarring, recurrence of varicose vein
Option A: Haemorrhage
• Haemorrhage is not a side effect of below-knee stripping.
Option B: Thromboembolism
• The risk factors of thromboembolism are a history of DVT, clotting disorder, obesity, and a catheter
located in a central vein.
• It can be caused by lower limb injury or fracture and surgery of the lower extremities.
Option D: Infection
• Infection is not associated with long saphenous vein stripping to the knee.
• There are rare chances of infection after long saphenous vein stripping to the knee, but sometimes it
can cause cellulitis.

Solution for Question 18:


Option A: Also known as Effort thrombosis
• In the above-given case, features such as painful swelling of the arm, shortness of breath, and dilated
visible veins are suggestive of the diagnosis of Paget-Schroetter syndrome.
• Paget-Schroetter syndrome is an acute thrombosis of the subclavian vein.

Page 26

1318
• Paget-Schroetter syndrome is also known as venous thoracic outlet syndrome or effort thrombosis.
• Clinical findings: History of performing prolonged activities, and repetitive motion activities. pain,
warmth, redness and swelling in the arm. Results in damage to the subclavian vein, where it passes
between the head of the clavicle and the first rib in association with the subclavius muscle.
• History of performing prolonged activities, and repetitive motion activities.
• pain, warmth, redness and swelling in the arm.
• Results in damage to the subclavian vein, where it passes between the head of the clavicle and the
first rib in association with the subclavius muscle.
• Treatment Anticoagulant Followed by Catheter directed thrombolytic therapy
• Anticoagulant
• Followed by Catheter directed thrombolytic therapy
• History of performing prolonged activities, and repetitive motion activities.
• pain, warmth, redness and swelling in the arm.
• Results in damage to the subclavian vein, where it passes between the head of the clavicle and the
first rib in association with the subclavius muscle.
• Anticoagulant
• Followed by Catheter directed thrombolytic therapy
Other Options
Option B: Associated with Jugular-subclavian venous thrombosis
• Paget-Schroetter syndrome is not associated with jugular-subclavian venous thrombosis.
• Paget-Schroetter syndrome is associated with axillary-subclavian venous thrombosis (ASVT).
Option C: Patients usually have no history of performing prolonged activities
• Patients with Paget-Schroetter syndrome usually have a history of performing prolonged activities.
Option D: Results in damage to the femoral vein
• Paget-Schroetter syndrome does not cause femoral vein damage.
• It causes subclavian vein damage.

Solution for Question 19:


Option B: Heparin
• In the above-given case, right calf pain and swelling, shortness of breath, sharp chest pain, and
unilateral leg swelling indicates the diagnosis of Deep venous thrombosis (DVT).
• Anticoagulants can be used for the management of DVT.
• The drug of choice is IV Heparin for 5 days.
• Warfarin can also be started, which acts after 72 hours (from the 3rd day onwards, dose adjustment is
required).

Page 27

1319
• Bridge therapy: use of short acting anticoagulants for a period of time during interruption of warfarin
therapy
Option A: Pneumatic compression stockings
• Pneumatic compression stockings are used to prevent blood clotting.
• These stockings are not recommended after DVT.
Option C: Thrombolysis
• Thrombolysis is not recommended in DVT due to the low risk of post-thrombotic syndrome.
• For the management of DVT, the drugs such as urokinase, streptokinase, and tissue plasminogen
activator are injected directly at the site of the clot.
Option D: Inferior vena cava filter
• An inferior vena cava filter is used for the prevention of DVT, not for the management of DVT.
• DVT is the delayed complication of the inferior vena cava filter.

Page 28

1320
Lymphatic System
1. According to Brunner's clinical classification of lymphedema, under which category does a patient
with slowly progressive non-pitting bilateral leg edema that does not significantly reduce upon elevation
?
(or)
A 32-year-old woman presents to the surgical outpatient department complaining of slowly progressive
leg edema for three days. She gives no history of any other abnormalities. She has no history of any
chronic illnesses. She is vitally stable. The patient's physical examination revealed non-pitting bilateral
leg edema that does not improve with elevation. According to Brunner’s clinical classification of
lymphedema, the patient’s pathology falls under which of the following?
A. Grade I
B. Grade II
C. Grade III
D. Grade IV
----------------------------------------
2. A 5-year-old child is presented to the physician by his mother with complaints of right foot swelling for
three days. He has no history of any other abnormalities. There is no family history of lymphedema.
The patient if febrile. On physical examination, the right foot had pitting edema and a positive
Stemmer’s sign. A lymphoscintigram was performed to confirm the diagnosis. Which of the following
will be the most common bacterial infection that can occur in this patient?
(or)
What is the most common bacterial infection that can occur in a child with a 3-month history of right foot
swelling, pitting edema, and a positive Stemmer's sign?
A. Staphylococcus
B. Streptococcus
C. E. coli
D. Pseudomonas
----------------------------------------
3. A 47-year-old male presents to the physician complaining of swelling in both feet. He had a low
anterior resection for colon cancer 15 days ago. Examination of the lower extremities reveals
non-pitting edema extending till the lower thighs. One of the interns made a differential diagnosis of
primary lymphedema after a microscopic examination of the interstitial space in this patient's lower
extremities. Which of the following statements about the patient's current pathology is correct?
(or)
In a female diagnosed with primary lymphedema after microscopic examination of the interstitial space
in the lower extremities, which statement is correct about the patient's current pathology?
A. Familial version of lymphedema praecox is known as Milroy disease
B. Lymphedema tarda is seen after 35 years
C. In congenital lymphedema, the swelling is limited to the foot and calf
D. Familial version of congenital lymphedema is known as Meig’s disease

1321
----------------------------------------
4. Which sign shown in the image below is associated with cardiac failure?
(or)
A 67-year-old male presents to the physician complaining of fatigue and shortness of breath on exertion
for six months with bilateral ankle swelling for a month. He is a known case of hypertension and type 2
diabetes. On examination, the lower limbs show bilateral pitting edema. An ECG shows a right bundle
branch block. Which of the following signs is shown in the image below about the patient's pathology?

A. Ballance's sign
B. Dahl's sign
C. Stemmer’s sign
D. Ladin's sign
----------------------------------------
5. Which of the following is true about malignant tumors of lymphatic vessels?
(or)
A 71-year-old woman presents to the physician with complaints of swelling of the left proximal arm for
three years. She has a history of hypertension and is a known case of breast cancer. She had
undergone a modified radical mastectomy 13 years ago for infiltrative ductal carcinoma of the right
breast. The swelling in the upper arm grew gradually with non-radiating, mild pain and the appearance
of nodular lesions on the upper arm. On examination, the patient was afebrile with lymphedema of the
entire left arm. A mass biopsy confirmed a malignant tumor of lymphatic vessels. Which of the following
is true about the patient’s current pathology?
A. Surgical excision is avoided for treatment of cavernous lymphangioma
B. Cystic hygroma is a type of lymphangiosarcoma
C. Cavernous lymphangiomas are mainly in the neck or axilla
D. Lymphangiosarcoma is associated with a relatively poor prognosis
----------------------------------------
6. A 20-year-old boy presents to the physician with complaints of diarrhea and generalized swelling for
two months. His parents reported that he does not participate in any college activities. A general
physical examination is unremarkable. A double-balloon enteroscopy and multi-dot biopsy revealed that
he had protein-losing enteropathy caused by intestinal lymphangiectasia. After treatment with total
parenteral nutrition, he was relieved of his symptoms. A procedure was performed in this case, shown
in the image below. Which of the following is true about the procedure involved in this case?

Page 2

1322
(or)
Which of the following statements is true about the procedure shown below performed in a boy with
protein-losing enteropathy caused by intestinal lymphangiectasia?

A. The visualised vein is exposed through a small incision and cannulated with a 16- to 22-G needle
B. Sodium diatrizoate injected retrogradely under pressure into a small vein on the dorsum of the foot
C. Used in patients with Lymphangiectasia and Lymphatic fistulas
D. Lymphatic channels and nodes are then seen with ultrasonography
----------------------------------------
7. What is the appropriate management procedure for persistent accumulation of chyle in the pleural
space after initial management of chylothorax?
A. Neck exploration and ligation of the thoracic duct
B. VATS Guided Thoracic duct ligation
C. Thoracotomy and repair of the thoracic duct
D. Thoracotomy and ligation of the thoracic duct
----------------------------------------
8. A 40-year-old male visits the doctor complaining of swelling in his right leg and fever for two weeks.
He has no history of any chronic illnesses. He is otherwise vitally stable, and a general physical
examination reveals no abnormalities. On examination, the right leg has pedal edema. The patient's
serology revealed elevated IgE levels. Which of the following is the most likely diagnosis?
(or)
What is the most likely diagnosis in a male presenting with unilateral pedal edema, fever, and elevated
serum IgE ?
A. Lymphatic filariasis
B. Post-traumatic lymphedema
C. Post-irradiation lymphedema
D. Milroy’s disease
----------------------------------------

Correct Answers

Page 3

1323
Question Correct Answer

Question 1 2
Question 2 2
Question 3 2
Question 4 3
Question 5 4
Question 6 3
Question 7 2
Question 8 1

Solution for Question 1:


Option B: Grade II
• This is a grade II lymphedema in which the patient complains of slowly progressive leg edema for
three days. She is vitally stable.
• The patient's physical examination revealed non-pitting bilateral leg edema that does not significantly
reduce upon elevation.
• According to Brunner’s clinical classification of lymphedema, the patient’s pathology comes under
grade II.
• The grade II lymphedema doesn’t significantly reduce upon elevation of the legs.

Option A: Grade I
• The grade I lymphedema is pitting edema.
• It is a type of edema that completely resolves on elevation. It also resolves after bed rest.
Option C: Grade III

Page 4

1324
• Grade III lymphedema is a type of edema with irreversible skin changes.
• It results in fibrosis papillae.
Option D: Grade IV
• Grade IV lymphedema is a type of edema in which ulceration of the affected area occurs.
• Cerebral edema is also included in this category.

Solution for Question 2:


Option B: Streptococcus
• This is a case of erysipelas in which the patient is brought to the physician complaining of right foot
swelling for three months.
• A lymphoscintigram was performed to confirm the diagnosis of erysipelas.
• It is the most common complication of both primary and secondary lymphedema.
• Erysipelas is caused by streptococcus infection.
• Cellulitis may occur concurrently with lymphedema because the pooling of protein-rich lymph fluid
makes it easier for the patient to develop an infection.
• Patients with erysipelas generally have an acute onset of symptoms with systemic manifestations,
including fever, chills, severe malaise, and headache; these can precede onset of local inflammatory
signs and symptoms by minutes to hours.
• In erysipelas, there is a clear demarcation between involved and uninvolved tissue. There may be a
raised, advancing border or erythema with central clearing.
• Additional manifestations of erysipelas include lymphangitis and enlargement of regional lymph
nodes. Edema surrounding the hair follicles may lead to dimpling in the skin, creating an appearance
reminiscent of an orange peel texture ("peau d'orange"). Vesicles, bullae, and ecchymoses or
petechiae may be observed. Cutaneous hemorrhage can occur in the setting of significant inflammation
in the skin.
Option A: Staphylococcus
• Erysipelas is usually not caused by staphylococcus infections.
• Staphylococcus bacteria commonly cause cellulitis.
Option C: E. coli
• Escherichia is a gram-negative bacterium.
• It can cause:
• Diarrhea Urinary tract infection.
• Diarrhea
• Urinary tract infection.
• E. coli shows lactose fermenting and non-mucoid colonies on MacConkey agar.
• Diarrhea
• Urinary tract infection.

Page 5

1325
Option D: Pseudomonas
• Pseudomonas aeruginosa is an aerobic gram-negative bacterium.
• It is a non-spore-forming rod.
• It usually causes infections in immunocompromised patients.
• It can cause UTI.
• It doesn’t cause erysipelas.

Solution for Question 3:


Option B: Lymphedema tarda is seen after 35 years
• This is a case of lymphedema in which the patient complains of swelling in both feet.
Congenital Lymphedema
Lymphedema Praecox
Lymphedema Tarda
• Onset before 1st year of life
• Edema is typically present at birth
• It can involve a single lower extremity, multiple limbs, genitalia, or face
• More likely to be bilateral and involve the whole leg
• A familial version of congenital lymphedema is known as Milroy disease
• Onset between 1-35 years
• MC type, seen in 90% of cases
• More common in women
• Mostly unilateral
• Limited to foot and calf
• A familial version of lymphedema praecox is known as Meig’s disease
• Onset after 35 years
• Relatively rare

Option A: The familial version of lymphedema praecox is known as Milroy disease


• A familial version of lymphedema praecox is known as Meig’s disease
• Milroy disease is a different pathology
• Lymphedema praecox is the most common type of lymphedema.
Option C: In congenital lymphedema, the swelling is limited to the foot and calf
• In congenital lymphedema, swelling is not limited to the foot and calf.
• Congenital lymphedema can involve a single lower extremity, multiple limbs, genitalia, or face.

Page 6

1326
Option D: The familial version of congenital lymphedema is known as Meig’s disease
• A familial version of lymphedema praecox is known as Meig’s disease.
• Congenital lymphedema is typically present at birth.
• A familial version of congenital lymphedema is known as Milroy Disease.

Solution for Question 4:


Option C: Stemmer’s sign
• This is a case of right heart failure leading to systemic venous congestion.
• The sign shown in the image is the Stemmer’s sign.
• Stemmer's sign is the inability to pinch the skin of the dorsum of the foot or hand.
• Stemmer's positive finding is associated with lymphedema.
Signs of Chronic Lymphedema
Buffalo hump
• Contour of the ankle is lost through infilling of the sub malleolar depressions, and a 'buffalo hump'
forms on the dorsum of the foot
Square toes
• Toes appear 'Square because of the confinement of footwear
Stemmer’s sign
• Skin on the dorsum of the toes cannot be pinched because of subcutaneous fibrosis.

Page 7

1327
Option A: Ballance's sign
• Ballance's sign is the presence of persistent dullness to percussion in the left upper quadrant of the
abdomen, with shifting dullness in the right flank.
• Dullness in the left flank is due to coagulated blood, which may be seen in a patient with a splenic
injury.
• It is observed during a traumatic assessment of the abdomen,
Option B: Dahl's sign
• Dahl's is a clinical sign in patients with severe chronic respiratory disorders.
• Areas of thickened and darkened skin are seen on the lower thighs and elbows.
• Patients with COPD tend to sit forward with their arms resting on their thighs, leading to chronic skin
erythema at contact points.
Option D: Ladin's sign

Page 8

1328
• Ladin's sign is a clinical sign in which there is softening in the midline of the uterus anteriorly at the
junction of the uterus and cervix.
• It is seen in pregnancy at about six weeks gestation.
• It is detectable with manual examination.

Solution for Question 5:


Option D: Lymphangiosarcoma is associated with a relatively poor prognosis
• This is a case of lymphangiosarcoma in which the patient presents to the physician complaining of
swelling of the left proximal arm for three years.
• Lymphangiosarcoma is a malignant tumor of lymphatic vessels that can develop in patients with
long-standing chronic lymphedema.
Lymphangioma
Lymphangiosarcoma/ Angiosarcoma
• Located subcutaneously in the head, neck & axilla
• Cavernous lymphangioma/Cystic hygroma
• Surgical excision
• Neck or axilla
• Due to long-standing lymphedema >10yrs
• E.g: STEWART-TREVES SYNDROME
• Appearance or development of angiosarcoma in the patients of CA Breast after receiving radiotherapy
• The prognosis is poor.
Option A: Surgical excision is avoided for treatment of cavernous lymphangioma
• Lymphangioma is located subcutaneously in the head, neck, and axilla.
• Surgical excision is the preferred treatment for cavernous lymphangioma.
Option B: Cystic hygroma is a type of lymphangiosarcoma
• Cystic hygroma is a type of lymphangioma, a benign tumor of lymphatic vessels.
• Surgical excision is the treatment of choice in this pathology.
Option C: Cavernous lymphangiomas are mainly in the neck or axilla
• These are located subcutaneously in the head, neck, and axilla.
• While lymphangiosarcoma is confined to the neck or axilla.

Solution for Question 6:


Option C: Used in patients with lymphangiectasia and lymphatic fistulas
• According to the clinical presentation, this is a case of intestinal lymphangiectasia.

Page 9

1329
• Lymphangiography: Lymphangiography is used to visualise lymphatics. The lymphatic channels and
nodes are then seen with radiographs. Lymphangiography is used in patients with lymphangiectasia
and lymphatic fistulas and patients considered for microvascular reconstruction. The procedure is
performed under fluoroscopy; a blue indicator dye is injected between toes, resulting in thin bluish lines
on each foot's top. These are your lymphatic vessels. Under local anesthesia, the interventional
radiologist will cut into one of the larger blue lines of each foot and insert a needle or catheter into a
vessel. Contrast dye will then be injected into the vessel, making the vessels more visible under
imaging The interventional radiologist will use a fluoroscope, which projects the images on a TV
monitor, to observe the dye spreading through your lymphatic system, up your legs, groin, and
abdominal cavity. The X-rays will be taken to record any abnormalities and there will be further X-rays
the following day. It is used in patients of: Lymphangiectasia Lymphatic fistulas Patients being
considered for microvascular reconstruction.
• Lymphangiography is used to visualise lymphatics. The lymphatic channels and nodes are then seen
with radiographs.
• Lymphangiography is used in patients with lymphangiectasia and lymphatic fistulas and patients
considered for microvascular reconstruction.
• The procedure is performed under fluoroscopy; a blue indicator dye is injected between toes, resulting
in thin bluish lines on each foot's top. These are your lymphatic vessels. Under local anesthesia, the
interventional radiologist will cut into one of the larger blue lines of each foot and insert a needle or
catheter into a vessel. Contrast dye will then be injected into the vessel, making the vessels more
visible under imaging
• The interventional radiologist will use a fluoroscope, which projects the images on a TV monitor, to
observe the dye spreading through your lymphatic system, up your legs, groin, and abdominal cavity.
The X-rays will be taken to record any abnormalities and there will be further X-rays the following day.
• It is used in patients of: Lymphangiectasia Lymphatic fistulas Patients being considered for
microvascular reconstruction.
• Lymphangiectasia
• Lymphatic fistulas
• Patients being considered for microvascular reconstruction.
• Lymphangiography is used to visualise lymphatics. The lymphatic channels and nodes are then seen
with radiographs.
• Lymphangiography is used in patients with lymphangiectasia and lymphatic fistulas and patients
considered for microvascular reconstruction.
• The procedure is performed under fluoroscopy; a blue indicator dye is injected between toes, resulting
in thin bluish lines on each foot's top. These are your lymphatic vessels. Under local anesthesia, the
interventional radiologist will cut into one of the larger blue lines of each foot and insert a needle or
catheter into a vessel. Contrast dye will then be injected into the vessel, making the vessels more
visible under imaging
• The interventional radiologist will use a fluoroscope, which projects the images on a TV monitor, to
observe the dye spreading through your lymphatic system, up your legs, groin, and abdominal cavity.
The X-rays will be taken to record any abnormalities and there will be further X-rays the following day.
• It is used in patients of: Lymphangiectasia Lymphatic fistulas Patients being considered for
microvascular reconstruction.
• Lymphangiectasia
• Lymphatic fistulas

Page 10

1330
• Patients being considered for microvascular reconstruction.
• Lymphangiectasia
• Lymphatic fistulas
• Patients being considered for microvascular reconstruction.

Option A: The visualized vein is exposed through a small incision and cannulated with a
16- to 22-G needle
• The lymphatic segment is exposed through a small incision, cannulated with a 27- to 30-G needle.
Option B: Sodium diatrizoate injected retrogradely under pressure into a
small vein on the dorsum of the foot
• For Lymphangiography, an oil-based dye is injected slowly into the lymphatics.
• A colored dye is injected into the dorsum of the hand or foot to visualize lymphatics.
Option D: Lymphatic channels and nodes are then seen with ultrasonography
• In lymphangiography, the lymphatic channels and nodes are then seen with radiographs.
• It is used in patients with lymphangiectasia and lymphatic fistulas.

Solution for Question 7:


Option B: VATS (Video Assisted Thoracoscopic) Guided Thoracic Duct Ligation:
• Surgical intervention via VATS for thoracic duct ligation is indicated if: Despite at least 5 days of
conservative management, chyle loss exceeds 1500 ml within a 24-hour period. If the chyle leak
persists beyond 2 weeks despite attempts at conservative management, surgery is warranted to
address the underlying cause.
• Despite at least 5 days of conservative management, chyle loss exceeds 1500 ml within a 24-hour
period.

Page 11

1331
• If the chyle leak persists beyond 2 weeks despite attempts at conservative management, surgery is
warranted to address the underlying cause.
• VATS is a minimally invasive surgical technique that allows for visualization and intervention within
the thoracic cavity through small incisions using a thoracoscope.
• Thoracic duct ligation aims to stop chyle leakage into the pleural space by identifying and ligating the
thoracic duct, the primary conduit for lymphatic drainage from the lower half of the body.
• VATS provides excellent visualization of the thoracic duct and surrounding structures with minimal
trauma compared to traditional open thoracotomy.
• It offers quicker recovery, reduced postoperative pain, and shorter hospital stays than open surgical
approaches.
• Despite at least 5 days of conservative management, chyle loss exceeds 1500 ml within a 24-hour
period.
• If the chyle leak persists beyond 2 weeks despite attempts at conservative management, surgery is
warranted to address the underlying cause.
Option A: Neck exploration and ligation of the thoracic duct
• While the thoracic duct can be accessed through the neck, this approach is more invasive than VATS.
It is usually reserved for cases where thoracic duct ligation via thoracoscopy or thoracotomy is not
feasible.
• It involves a larger incision in the neck and carries a higher risk of complications such as damage to
nearby structures like nerves and blood vessels.
Option C: Thoracotomy and repair of the thoracic duct
• Repairing the thoracic duct is not typically performed in cases of chylothorax because it is challenging
due to the delicate nature of the duct and its complex anatomy.
• Thoracotomy, while effective in gaining access to the thoracic cavity, is more invasive than VATS and
associated with higher morbidity and longer recovery times.
Option D: Thoracotomy and ligation of the thoracic duct
• This option involves an open surgical approach, which is associated with more significant
postoperative pain, more extended hospital stays, and increased risk of complications compared to
VATS.
• While effective, it is considered less preferable than VATS due to its invasiveness and associated
morbidity.

Solution for Question 8:


Option A: Lymphatic filariasis
• The patient complains of swelling in his right leg and fever for two weeks.
• On examination of the lower limbs, the patient has unilateral pedal edema. A blood smear of the
patient was done, which revealed microfilariae. The patient's serology shows elevated anti-filarial IgG4
and raised IgE. All these findings are suggestive of lymphatic filariasis.
• It is caused by Wuchereria bancrofti, which is a nematode,transmitted by female mosquito.
• Its incubation period is 9-12 months.

Page 12

1332
• Filariasis is the most common cause of unilateral pedal edema in India.
Option B: Post-traumatic lymphedema
• Post-traumatic lymphedema is commonly caused by a road traffic accident or by a sports injury.
• It results in blockage in the lymphatic system.
• It does not show serological features or fever given in the question.
Option C: Post-irradiation lymphedema
• Post-irradiation lymphedema is commonly caused after radiation surgery.
• It results from damage to the lymphatic system during surgery.
• It does not show serological features or fever given in the question.
Option D: Milroy’s disease
• Milroy’s disease is a congenital abnormality of the lymphatic system.
• It causes lymphedema commonly in the lower limb.
• It does not show serological features or fever given in the question.

Page 13

1333
Thorax and Mediastinum
1. A 42-year-old woman presents to the emergency department with sudden-onset shortness of breath,
chest pain, and cough. She recently underwent surgery for a hip replacement. On examination, she
appears anxious, tachypneic, and has decreased breath sounds at the lung bases. A
ventilation-perfusion mismatch is suspected. Which of the following mechanisms contributes to this
mismatch?
(or)
Which of the following mechanism contributes to ventilation-perfusion mismatch?
A. Reduced alveolar ventilation
B. Elevated alveolar carbon dioxide levels
C. Diminished perfusion to ventilated alveoli
D. Right-to-left shunting in the pulmonary circulation
----------------------------------------
2. A 55-year-old man presents to the emergency department with sudden-onset dyspnea, pleuritic
chest pain, and cough. He has a history of smoking and hypertension. On examination, he appears
anxious, has tachypnea, and his oxygen saturation is 89% on room air. There is no swelling or redness
in his legs. Which of the following components of the Modified Wells Criteria is most relevant for
assessing the probability of pulmonary embolism in this patient?
(or)
Which of the following components of the Modified Wells Criteria is most relevant for assessing the
probability of pulmonary embolism?
A. Clinical signs of deep vein thrombosis (DVT)
B. An alternative diagnosis less likely than pulmonary embolism
C. Heart rate >100 bpm
D. Immobilization for more than 3 days
----------------------------------------
3. A 65-year-old man with a history of cirrhosis and ascites presents with increasing shortness of breath
and pleuritic chest pain. Physical examination reveals dullness to percussion over the right lung base,
decreased breath sounds on the right side, and new-onset confusion. A chest X-ray confirms the
presence of a large right-sided pleural effusion. The patient's blood pressure is 90/60 mmHg, heart rate
is 110 bpm, respiratory rate is 30/min, and oxygen saturation is 88% on room air. Initial management
done with supplemental oxygen, fluid resuscitation, and appropriate antibiotics. Which of the following
interventions should be prioritized next?
(or)
A 65-year-old man with cirrhosis presents with increasing shortness of breath, pleuritic chest pain, and
a right-sided pleural effusion. Vital signs show hypotension, tachycardia, tachypnea, and hypoxia. Initial
management includes oxygen, fluids, and antibiotics. What's the treatment of choice for to manage
pelural effusion in ths patient?
A. Thoracocentesis
B. ICD insertion
C. High-dose diuretic therapy

1334
D. Transfusion of packed red blood cells
----------------------------------------
4. What is the best initial investigation to confirm the diagnosis of pulmonary embolism?
(or)
What is the best initial test to confirm a pulmonary embolism?
A. Pulmonary angiography
B. Electrocardiogram (ECG)
C. Multi-detector CT
D. D-dimer assay
----------------------------------------
5. A 62-year-old man with a history of congestive heart failure and atrial fibrillation is admitted to the
hospital for a surgical procedure. He will be undergoing major abdominal surgery due to a
gastrointestinal malignancy. Considering his medical history and the upcoming surgery, what is the
most appropriate prophylactic measure to prevent the development of pulmonary embolism in this
patient?
(or)
For a 62-year-old man with congestive heart failure, atrial fibrillation, and scheduled major abdominal
surgery for gastrointestinal malignancy, what is the best prophylactic measure to prevent pulmonary
embolism?
A. Placement of an inferior vena cava (IVC) filter
B. Initiation of anticoagulant therapy with low-molecular-weight heparin
C. Administration of oxygen therapy during the surgery
D. Utilization of intermittent pneumatic compression devices
----------------------------------------
6. A 2-year-old child presents to the emergency department with sudden onset coughing and choking
while eating peanuts. The parents report that the coughing has persisted for 30 minutes and has not
improved. On physical examination, the child's respiratory rate increases, and a wheezing sound is
heard on the auscultation of the right lung. Which of the following complications is most likely
associated with the delayed diagnosis of this condition?
(or)
Which complication is most likely due to delayed diagnosis in a 2-year-old child who presented to the
emergency department with sudden onset coughing and choking while eating peanuts, with persistent
coughing for 30 minutes and wheezing on auscultation of the right lung?
A. Hypertension
B. Pneumothorax
C. Atelectasis
D. Hepatitis
----------------------------------------
7. A 47-year-old woman presents to her primary care physician with a history of recurrent episodes of
hemoptysis over the past few months. She describes these episodes as intermittent and often

Page 2

1335
associated with bouts of coughing. The patient also reports a persistent, non-productive cough and
occasional difficulty breathing. She denies any history of smoking. Physical examination is
unremarkable. A bronchoscopy with biopsy is performed for further evaluation. The biopsy reveals a
highly vascular tumor obstructing the main bronchi. What is the most likely diagnosis for this patient's
condition?
(or)
What diagnosis is most likely for a 47-year-old woman with recurrent episodes of hemoptysis,
intermittent coughing, non-productive cough, occasional difficulty breathing, and a highly vascular
tumor obstructing the main bronchi found on bronchoscopy biopsy, with no history of smoking?
A. Lung Cancer
B. Tuberculosis
C. Bronchial Adenoma
D. Pulmonary Embolism
----------------------------------------
8. A 50-year-old woman presents with persistent cough, fatigue, and generalized muscle weakness. On
examination, she has proximal muscle weakness and diminished reflexes. Laboratory tests show
elevated levels of serum calcium. Imaging reveals a lung mass, and biopsy confirms Small Cell Lung
Carcinoma. The patient's symptoms and findings are consistent with a paraneoplastic syndrome
affecting the neuromuscular system. Which of the following paraneoplastic syndromes is most likely
associated with this patient's presentation?
(or)
Which paraneoplastic syndrome is likely associated with Small Cell Lung Carcinoma?
A. Acanthosis Nigricans
B. SIADH
C. Lambert-Eaton Syndrome
D. Cushing Syndrome
----------------------------------------
9. A 58-year-old man presents with insidious onset of pain in the upper arm, sensory disturbances in
the medial aspect of the forearm and hand, weakness and atrophy of intrinsic hand muscles, and
ipsilateral Horner's syndrome. Imaging reveals a tumor located at the lung apex. Biopsy confirms the
diagnosis of Pancoast Tumor. Which of the following nerve root values is most likely involved in this
patient's condition?
(or)
Which nerve root value is likely involved in a 58-year-old man presenting with upper arm pain, sensory
disturbances in the forearm and hand, weakness, atrophy of hand muscles, and ipsilateral Horner's
syndrome, with a Pancoast Tumor located at the lung apex?
A. C4-C5
B. C6-C7
C. C8-T1
D. T1-T2
----------------------------------------

Page 3

1336
10. A 40-year-old woman presents with exertional dyspnea, palpitations, and occasional chest pain.
Physical examination reveals a diastolic rumble heard best at the apex. Imaging shows a mass
attached to the interatrial septum prolapsing into the left atrium during diastole. Biopsy confirms the
diagnosis. Which type of cardiac tumor is most likely responsible for these findings?
A. Myxoma
B. Angiosarcoma
C. Rhabdomyoma
D. Rhabdomyosarcoma
----------------------------------------
11. A 55-year-old man presents to the medicine outpatient department with his wife with a 4-week
history of progressively worsening right shoulder pain radiating down his right arm and into his upper
back. He has also reported blurred vision in his right eye and felt his right eyelid was 'drooping'. His only
other history, of note, is being an ex-smoker with a 25-pack-year history. He also reported that he had
recently been to the gym and noticed only the left side of his face was sweating. Examination reveals
partial ptosis of the right eye, a constricted right pupil, and reduced sensation in the right T1
dermatome. A chest radiograph is performed and shows the following finding (image). What will be the
most probable diagnosis?
(or)
What is the most probable diagnosis in a 55-year-old man presenting with right shoulder pain radiating
down his right arm, along with partial ptosis, miosis, anhidrosis of right side, ex-smoker with a
25-pack-year history. and a chest radiograph showing below finding?

A. Horner's syndrome
B. Pancoast tumour
C. Larynx CA
D. Superior vena cava syndrome
----------------------------------------
12. A 59-year-old woman arrives at the emergency room complaining of chest tightness for the past 6
hours. She also has a history of hypertension without any treatment for five years. Vital signs on arrival
indicate blood pressure of 78/50 mmHg. Her 24-hour urinary norepinephrine and catecholamine levels
are elevated. A 12-lead electrocardiogram indicates ST-segment depression in leads II, III, aVF, and
V3–V6 and QT prolongation. Contrast-enhanced computed tomography demonstrates an
inhomogeneous mass in the thoracic region (2.5 × 3.0 cm). Which is the most common location of the
above discussed disorder?

Page 4

1337
(or)
What is the common location of the inhomogeneous mass in the thorax seen in a 59-year-old woman
presenting with chest tightness and elevated urinary norepinephrine and catecholamine levels?
A. Anterior mediastinum
B. Posterior mediastinum
C. Middle mediastinum
D. Superior mediastinum
----------------------------------------
13. What is the probable diagnosis in a 66-year-old lady, whose pleural biopsy shows metastatic
deposits of dyscohesive malignant cells of epithelioid variety that are positive for calretinin and WT-1 on
immunohistochemistry?
(or)
A 66-year-old woman arrives at the pulmonology ward complaining of a dry cough, nonspecific chest
pain, and dyspnea on exertion for the past two years. Given persistent symptoms, she is evaluated with
chest x-rays, which show bilateral lower zone nodular opacities with calcification and a right pleural
effusion. Her CBC reveals an elevated platelet count of 650,000/mm3. The pleural fluid cytological and
biochemical examination shows an exudative lymphocyte-predominant pleural effusion with normal
levels of adenosine deaminase. Pleural fluid malignant cytology shows a few atypical cells. The
histopathological examination of the pleural biopsy suggests metastatic deposits of dyscohesive
malignant cells of epithelioid variety, which are favorable for calretinin and WT-1 on
immunohistochemistry. What will be the probable diagnosis in this patient?
A. Small cell lung cancer
B. Thoracic Pheochromocytoma
C. Malignant Mesothelioma
D. Carcinoid tumour
----------------------------------------
14. A 20-year-old man presents to a primary care physician complaining of chest discomfort for the last
ten days. His primary care physician provides symptomatic treatment, which gives an initial
improvement in symptoms. Afterwards, his chest discomfort started again, and he developed
progressive dyspnea with exertion. A CT chest reveals a round, well-demarcated mass in the middle
mediastinal compartment. Which of the following is rare enough to be included in the middle
mediastinal masses found in the patient discussed above?
(or)
What is a rare finding among middle mediastinal masses in a 20-year-old man presenting with chest
discomfort and a round well-demarcated mass on middle mediastinal compartment on CT chest?
A. Bronchogenic cysts
B. Ascending aortic aneurysm
C. Pericardial cyst
D. Ganglioneuroma
----------------------------------------

Page 5

1338
15. A 33-year-old female presents with the chief complaint of shortness of breath on exertion for the
last 20 days. She has a medical history of acute bronchitis and was treated with bronchodilators,
empirical antibiotics, and a short course of oral steroids six months before. This management did not
improve her symptoms; which have gradually worsened over the past six months.. The chest X-ray of
the patient is given below. What is the most likely cause of the disorder mentioned?
(or)
What is the most likely cause of the patient's shortness of breath on exertion for the last 20 days and
chest X-ray findings as shown below?

A. Aspiration
B. Hematogenous spread from a distant site
C. Direct Contact
D. Lymphatic spread
----------------------------------------
16. An 84-year-old woman arrives at the emergency department complaining of difficulty breathing and
chest pain for the past month. She gave a surgical history of thoracotomy for tension pneumothorax two
months before the consultation. A chest X-ray shows a right-sided pleural effusion. Drainage of the
effusion yields a cloudy, off-white fluid that settles in layers in the drainage container. Pleural fluid
examination reveals a lymphocyte-rich transudate with high levels of cholesterol and
triglycerides.Which of the following is true for the case mentioned above?
(or)
Which of the following is true for the 84-year-old lady with difficulty breathing, chest discomfort, a
history of thoracotomy for tension pneumothorax, chest X-ray showing right-sided pleural effusion with
cloudy off-white fluid, and pleural fluid examination revealing a lymphocyte-rich transudate with high
levels of cholesterol and triglycerides?
A. The most common cause is trauma
B. Occurs when the thoracic duct remains intact
C. Immediate thoracotomy should be done
D. Patients with this condition should undergo prolonged tube thoracostomy
----------------------------------------
17. A 72-year-old woman presents to the pulmonology ward with shortness of breath, a productive
cough, chest pain, mild fatigue, weight loss, and a history of hypertension. She is a school teacher with
a 45-pack-year smoking history. She is still an active smoker. On physical examination, she has mild
dullness on percussion with decreased breath sounds. The chest x-ray shows a left hilar mass and a

Page 6

1339
5.4 cm left upper lobe mass. A CT scan reveals a hilar mass with bilateral mediastinal extension; no
distant metastatic disease is found. The interventional radiographic test and biopsy of the lung revealed
small cell lung cancer. Which of the following hormones was found to have normal levels in the case
mentioned above?
(or)
Which hormone levels remain normal in small-cell lung cancer?
A. ACTH
B. Growth hormone
C. ANF
D. AVP
----------------------------------------
18. What is the most probable diagnosis for an individual with a history of pulmonary tuberculosis,
drooping of the left eyelid, pupillary miosis, and a round opacification in the left upper apical lobe on
chest X-ray?
(or)
A 65-year-old mine worker arrives at the pulmonology OPD after six weeks of coughing with
blood-streaked sputum. He lost 7 kg weight within two months and was treated for pulmonary
tuberculosis ten years ago. He has had a droop in his left eyelid for the past month. On physical
examination, there is ptosis of the left eye and pupillary miosis. A chest X-ray revealed round
opacification in the left upper apical lobe. What is the most probable diagnosis of the patient?
A. Secondary tuberculosis
B. Adenocarcinoma
C. Squamous cell carcinoma
D. Asbestosis
----------------------------------------
19. What additional investigation is needed to reach the diagnosis in a 65-year-old man with persistent
cough, hemoptysis, fever, weight loss, decreased appetite, night sweats, crepitations on examination,
and a cavitatory lesion on chest X-ray?
(or)
A 65-year-old man presents to the pulmonology OPD with a persistent cough with blood for six weeks.
There is also a history of fever (over 100 F) and weight loss. He also reports decreased appetite and
tiredness. He also has night sweats. On examination, the pulmonologist finds crepitations in the right
apical region. CRP and ESR levels are elevated in laboratory tests. An X-ray shows a cavitary lesion in
the right upper lobe of the lung. What additional tests are needed to make a diagnosis of the patient?
A. Bronchoscopy, lavage and brushing
B. CT scan
C. X-Ray
D. FNAC
----------------------------------------
20. Contrast-enhanced computed tomography revealed worsening hemothorax with contrast
extravasation 4 hours after arrival at the hospital. Which of the following vessel is probably injured in

Page 7

1340
hemothorax?
(or)
An 83-year-old man was transported to the hospital three hours after falling out of bed. He underwent
fluid resuscitation and a tube thoracostomy but became hemodynamically unstable. Computed
tomography revealed a hemothorax and multiple rib fractures. Contrast-enhanced computed
tomography revealed a worsening hemothorax with contrast extravasation 4 hours after arrival at the
hospital. Which of the following vessels is probably injured in this hemothorax patient?
A. Pulmonary artery
B. Pulmonary vein
C. Bronchial artery
D. Intercostal arteries
----------------------------------------
21. What is a correct statement about the condition in a 9-year-old boy with complaints of back and
chest pain, a depression in the chest, and physical findings suggestive of Marfan's disease?
(or)
A 9-year-old boy is brought to a primary care clinic by his mother complaining of back and chest pain
that has persisted for the past 40 days. She is also worried about a depression in the child's chest, as
shown in the image below, which she discovered two weeks before the visit. On examination, his vital
signs are within normal limits. Other physical findings are suggestive of Marfan's disease. Which of the
following statements is correct regarding this condition?

A. Most common chest wall deformity


B. Arises from excessive growth of the upper costal cartilages, causing posterior sternal depression
C. Haller index is the ratio of the length of the chest wall to the depth of the sternum to the vertebral
body
D. Ravitch procedure is minimally invasive surgery for pectus excavatum
----------------------------------------
22. A 34-year-old man presents to the surgical emergency room with a high-speed road traffic injury
sustained 4 hours earlier. He has severe shortness of breath at the time of the presentation. On
examination, his blood pressure is 90/60 mmHg, and his heart rate is 100 beats per minute with an
oxygen saturation of 85% at room air. The breath sounds are reduced on the left side. His jugular
venous pulse (JVP) is also elevated. A chest x-ray is done, which is shown in the picture below. Which
of the following statement is correct about this condition?

Page 8

1341
(or)
What is the correct statement about the condition of a 34-year-old man with road traffic injury presents
with acute shortness of breath, reduced breath sounds on the left side, elevated jugular venous pulse
(JVP), and chest x-ray as shown below?

A. The trachea has deviated to the affected side of the chest


B. The best investigation for diagnosis is a non-contrast CT scan
C. The treatment of choice is the insertion of intercostal drainage in 2nd intercostal space
D. The best X-ray film for diagnosis is a chest X-ray lateral view
----------------------------------------
23. A 60-year-old male presents to the emergency department with cough and sputum production,
fever with chills, night sweats, and rigor for the last six days. On examination, his blood pressure is
105/65mmHg, his heart rate is 99 beats per minute, and his oxygen saturation is 91%. Crepitations
present and reduced breath sounds on the right side. The chest X-ray has been done, and the findings
are given below. Which of the following is an indication of the surgery?
(or)
What finding on the chest X-ray suggests an indication for surgery in a 60-year-old male with symptoms
of cough, sputum production, fever with chills and rigors, night sweats, decreased breath sounds on
right side and crepitations?

A. Lung abscess unresponsive to medical management


B. Asthma exacerbation
C. Acute pulmonary edema
D. Mild community-acquired pneumonia

Page 9

1342
----------------------------------------
24. Preemptive ligation of the thoracic duct (TD) is occasionally performed during cardiothoracic
procedures to prevent chylothorax. Which area is historically used for identifying this structure in the
thorax?
(or)
A 65-year-old man with a known case of esophageal cancer presented to the surgical outpatient
department for his elective esophagectomy. Preemptive ligation of the thoracic duct (TD) is
occasionally performed during cardiothoracic procedures to prevent chylothorax. Landmarks for
localizing this structure are thus crucial to the surgeon during such procedures. Which area is
historically used for identifying this structure in the thorax?
A. Calot's triangle
B. Hesselbach’s triangle
C. Passaro's Triangle
D. Poirier's Triangle
----------------------------------------
25. A 71-year-old female presented with a sudden onset of massive hemoptysis. She had a
40-pack-year history of smoking. Physical examination revealed sinus tachycardia at 108 beats per
minute and a blood pressure of 128/72 mm Hg. Chest and cardiovascular examinations were
essentially unremarkable. Chest radiography and a CT scan were done (image given). Based on the
above history and clinical findings, which of the following is the most likely diagnosis?
(or)
What is the most likely diagnosis in a 71-year-old female with sudden massive hemoptysis, a
40-pack-year smoking history, with no constitutional symptoms and findings on chest radiography and
CT given below?

A. Primary lung cancer


B. Pulmonary hamartoma
C. Infectious granulomas
D. Sarcoidosis
----------------------------------------
26. A 70-year-old man presented to the emergency department with the complaint of sudden tearing
pain in the anterior chest extending to the neck and jaw. He has been a known case of hypertension for
twenty-five years. On general physical examination, he looks confused and agitated. His blood

Page 10

1343
pressure was 190/110 mm Hg. Electrocardiography (ECG) shows signs of left ventricular hypertrophy
and non-specific changes. A chest x-ray reveals a widened mediastinum at the aortic knuckle. Based
on the above history and clinical findings, which of the following is the gold standard test to confirm the
diagnosis?
(or)
What is the gold standard test to confirm the diagnosis in a 70-year-old man with sudden tearing chest
pain in the anterior chest extending to the neck and jaw, known hypertensive, elevated blood pressure,
and a widened mediastinum at the level of the aortic knuckle on chest x-ray?
A. Chest x-ray
B. Echocardiography
C. Computed Tomography Angiography (CTA)
D. Aortography
----------------------------------------
27. A 25-year-old tall male presented to the emergency department with complaints of sudden stabbing
left-sided chest pain and shortness of breath. On examination, there was reduced breath sound,
hyper-resonant percussion, and decreased fremitus in the left lung. The chest x-ray of the patient is
given below. Based on the above history and clinical findings, which of the following is the most likely
diagnosis?
(or)
What is the most likely diagnosis of a patient, based on the chest x-ray as given below?

A. Primary spontaneous pneumothorax


B. Tension Pneumothorax
C. Secondary spontaneous pneumothorax
D. Traumatic pneumothorax
----------------------------------------
28. Which arteries supply intercostal spaces three to six in the absence of anterior intercostal arteries
due to ligation during coronary bypass graft surgery using the internal thoracic artery?
(or)
A 49-year-old obese diabetic female presents with the complaint of shortness of breath for 12 hours.
She has been sweating profusely and is complaining of severe chest pain. She has a history of
ischemic heart disease and is seeing a cardiologist for advanced coronary bypass surgery. In coronary
bypass graft surgery, the internal thoracic artery is used as the coronary artery bypass graft. The

Page 11

1344
anterior intercostal arteries in intercostal spaces three to six are ligated. Which of the following arteries
will be expected to supply these intercostal spaces?
A. Musculophrenic
B. Superior epigastric
C. Posterior intercostal
D. Lateral thoracic
----------------------------------------
29. What is the most preferred graft for coronary artery bypass graft (CABG) in a 58-year-old man
diagnosed with triple vessel disease?
(or)
A 58-year-old man arrives at the cardiology clinic complaining of mild chest pain after walking a few
steps for the past 15 days. He is a known case of hypertension and hypercholesterolemia. He also has
a history of non-ST elevation myocardial infarction, for which angioplasty was done ten years ago. A
coronary angiography is performed, which shows triple vessel disease. He is referred to the
cardiovascular surgeon, who has advised coronary artery bypass graft (CABG). What is the most
preferred graft for CABG for this patient?
A. Left internal mammary artery
B. Saphenous vein
C. Reversed saphenous vein
D. Dacron
----------------------------------------
30. What is a potential complication of first rib resection and clavicle osteotomy performed through the
supraclavicular approach in a 28-year-old man with dull shoulder pain, weakness, decreased grip
power, and hypoesthesia in the medial side of the hand?
(or)
A 28-year-old man presents to the OPD with a feeling of weakness and dull aching shoulder pain and
experiences decreased grip power and hypoesthesia in the medial side of the hand in the full-draw
position for the last 12 days. On CT angiography, the cross-sectional area of the subclavian artery in
the costoclavicular space decreases by 40% compared with that of the subclavian artery in a
non-compressed state. The patient underwent the first rib resection through the supraclavicular
approach with a clavicle osteotomy. Which of the following complications could occur during this
surgery?
A. Hydropneumothorax
B. Ansa cervicalis injury
C. Lymphocutaneous fistula
D. Long thoracic nerve injury
----------------------------------------

Correct Answers
Question Correct Answer

Page 12

1345
Question 1 3
Question 2 1
Question 3 2
Question 4 3
Question 5 2
Question 6 3
Question 7 3
Question 8 3
Question 9 3
Question 10 1
Question 11 2
Question 12 2
Question 13 3
Question 14 4
Question 15 1
Question 16 1
Question 17 2
Question 18 3
Question 19 1
Question 20 4
Question 21 1
Question 22 2
Question 23 1
Question 24 4
Question 25 2
Question 26 3
Question 27 1
Question 28 3
Question 29 1
Question 30 4

Solution for Question 1:


Correct Option C - Diminished perfusion to ventilated alveoli:
• Pulmonary embolism: blood clot (thrombus) lodged in an artery in the lung and blocks the blood flow
• Risk factors: DVT (m/c) Obesity Prolonged immobilization Pregnancy and puerperium High dose
estrogen therapy Long duration duration surgery Inherited hypercoagulable states Acquired

Page 13

1346
hypercoagulable states
• DVT (m/c)
• Obesity
• Prolonged immobilization
• Pregnancy and puerperium
• High dose estrogen therapy
• Long duration duration surgery
• Inherited hypercoagulable states
• Acquired hypercoagulable states
• Pathophysiology: when embolus obstructs pulmonary artery → normal ventilation but no perfusion/
decreased perfusion → Ventilation perfusion mismatch → increased anatomical dead space → arterial
hypoxemia → increased alveolar arterial O2 tension gradient → breathlessness
• Clinical features: Dyspnea (m/c symptom) Other symptoms: Chest pain, hemoptysis and cough
Tachypnea (m/c sign) Other signs: fever, U/L leg swelling, wheeze and pleural friction rub
• Dyspnea (m/c symptom)
• Other symptoms: Chest pain, hemoptysis and cough
• Tachypnea (m/c sign)
• Other signs: fever, U/L leg swelling, wheeze and pleural friction rub
• Modified Well's criteria: used for predicting pulmonary embolism. D-DIMER → excellent screening
test
• Investigation: Multidetector CT - IOC Pulmonary angiography - Gold standard
• Multidetector CT - IOC
• Pulmonary angiography - Gold standard
• Treatment: Anticoagulant + observation Fibrinolytic treatment Radiological guided catheter
embolectomy
• Anticoagulant + observation
• Fibrinolytic treatment
• Radiological guided catheter embolectomy
• In pulmonary embolism, an embolus (usually a blood clot) obstructs a pulmonary artery, leading to
regions of the lung where ventilation is occurring but blood perfusion is reduced or completely blocked.
This results in a ventilation-perfusion mismatch, as the ventilation of the alveoli (air sacs) is not
matched with adequate blood flow, leading to decreased oxygen exchange and ventilation without
perfusion in the affected lung regions.
• DVT (m/c)
• Obesity
• Prolonged immobilization
• Pregnancy and puerperium
• High dose estrogen therapy

Page 14

1347
• Long duration duration surgery
• Inherited hypercoagulable states
• Acquired hypercoagulable states
• Dyspnea (m/c symptom)
• Other symptoms: Chest pain, hemoptysis and cough
• Tachypnea (m/c sign)
• Other signs: fever, U/L leg swelling, wheeze and pleural friction rub
• Multidetector CT - IOC
• Pulmonary angiography - Gold standard
• Anticoagulant + observation
• Fibrinolytic treatment
• Radiological guided catheter embolectomy

Incorrect Options:
Option A - Reduced alveolar ventilation: Reduced alveolar ventilation would result in decreased oxygen
intake and increased carbon dioxide levels in the alveoli. While this could contribute to decreased oxy
gen exchange, it doesn't directly lead to a ventilation-perfusion mismatch as described in the case of a
pulmonary embolism.
Option B - Elevated alveolar carbon dioxide levels: Elevated alveolar carbon dioxide levels (hypercapni
a) would suggest inadequate removal of carbon dioxide, typically due to reduced alveolar ventilation.
While this could result in acidosis and respiratory distress, it's not the primary mechanism leading to th
e ventilation-perfusion mismatch seen in pulmonary embolism.
Option D - Right-to-left shunting in the pulmonary circulation: Right-to-left shunting typically involves bl
ood bypassing the lungs and entering the systemic circulation without being oxygenated. While this ca
n result in hypoxemia, it's more commonly associated with certain congenital heart defects and conditio
ns like intracardiac shunts, rather than pulmonary embolism.

Solution for Question 2:


Correct Option A- Clinical signs of deep vein thrombosis (DVT)
• The Modified Wells Criteria is a tool used to assess the probability of pulmonary embolism. It assigns
points based on various clinical factors. In this case, the patient has no swelling or redness in his legs,
indicating the absence of clinical signs of DVT, which would contribute to a lower score. The presence
of clinical signs of DVT would increase the Wells score, suggesting a higher probability of pulmonary
embolism.
Interpretation:
• PE likely > 4
• PE unlikely ≤ 4
Incorrect Options:

Page 15

1348
Option B- An alternative diagnosis less likely than pulmonary embolism: This component involves consi
dering other possible diagnoses that could explain the patient's symptoms. If an alternative diagnosis is
considered less likely than pulmonary embolism, it adds points to the Wells score. However, the scena
rio does not provide information about alternative diagnoses, so this factor is not directly relevant in this
case.
Option C- Heart rate >100 bpm: An elevated heart rate is one of the criteria in the Wells score. A
heart rate >100 bpm adds points to the score, as it can be a sign of increased cardiovascular stress du
e to conditions like pulmonary embolism. However, in the scenario provided, the patient's heart rate is
not mentioned.
Option D- Immobilization for more than 3 days: Prolonged immobilization is one of the factors that contr
ibute to the Wells score. Immobilization for more than 3 days, such as after surgery or during prolonge
d bed rest, increases the risk of developing blood clots and subsequently a pulmonary embolism. How
ever, the scenario does not provide information about the patient's recent mobility.

Solution for Question 3:


Correct option B:ICD insertion
• The patient presents with transudative pleural effusion with underlying cirrhosis.
• Investigations for transudative pleural effusion include: Ultrasound: Used for detection of minimal
pleural effusion, as it can detect as little as 1 ml of fluid. X-ray views: Additional imaging to assess the
extent and characteristics of the pleural effusion.
• Ultrasound: Used for detection of minimal pleural effusion, as it can detect as little as 1 ml of fluid.
• X-ray views: Additional imaging to assess the extent and characteristics of the pleural effusion.
• Management of transudative pleural effusion involves treating the underlying cause.
• In many cases, the treatment of choice is inserting an intercostal drain (ICD) in the triangle of safety,
typically in the 5th intercostal space in the anterior axillary line.
• Ultrasound: Used for detection of minimal pleural effusion, as it can detect as little as 1 ml of fluid.
• X-ray views: Additional imaging to assess the extent and characteristics of the pleural effusion.
Incorrect options: A,C, and D
Due to the above given reasons

Solution for Question 4:


Correct Options C- Multi-detector CT (MDCT)

• Pulmonary embolism: blood clot (thrombus) lodged in an artery in the lung and blocks the blood flow
• Risk factors: DVT (m/c) Obesity Prolonged immobilization Pregnancy and puerperium High dose
estrogen therapy Long duration duration surgery Inherited hypercoagulable states Acquired
hypercoagulable states

Page 16

1349
• DVT (m/c)
• Obesity
• Prolonged immobilization
• Pregnancy and puerperium
• High dose estrogen therapy
• Long duration duration surgery
• Inherited hypercoagulable states
• Acquired hypercoagulable states
• Pathophysiology: when embolus obstructs pulmonary artery → normal ventilation but no perfusion/
decreased perfusion → Ventilation perfusion mismatch → increased anatomical dead space → arterial
hypoxemia → increased alveolar arterial O2 tension gradient → breathlessness
• Clinical features: Dyspnea (m/c symptom) Other symptoms: Chest pain, hemoptysis and cough
Tachypnea (m/c sign) Other signs: fever, U/L leg swelling, wheeze and pleural friction rub
• Dyspnea (m/c symptom)
• Other symptoms: Chest pain, hemoptysis and cough
• Tachypnea (m/c sign)
• Other signs: fever, U/L leg swelling, wheeze and pleural friction rub
• Modified Well's criteria: used for predicting pulmonary embolism. D-DIMER → excellent screening test
• In patients with suspected pulmonary embolism, the most appropriate initial investigation to confirm
the diagnosis is a Multi-Detector CT (MDCT) scan. MDCT provides detailed images of the pulmonary
arteries and can help visualize any blood clots (emboli) that might be causing the symptoms. It's a
non-invasive and efficient way to assess the pulmonary vasculature and diagnose or rule out
pulmonary embolism. While other options, such as D-dimer assay and ECG, may contribute to the
diagnostic process, MDCT is the primary imaging modality used for confirmation.
• Treatment: Anticoagulant + observation Fibrinolytic treatment Radiological guided catheter
embolectomy
• Anticoagulant + observation
• Fibrinolytic treatment
• Radiological guided catheter embolectomy
• DVT (m/c)
• Obesity
• Prolonged immobilization
• Pregnancy and puerperium
• High dose estrogen therapy
• Long duration duration surgery
• Inherited hypercoagulable states
• Acquired hypercoagulable states
• Dyspnea (m/c symptom)

Page 17

1350
• Other symptoms: Chest pain, hemoptysis and cough
• Tachypnea (m/c sign)
• Other signs: fever, U/L leg swelling, wheeze and pleural friction rub
• Anticoagulant + observation
• Fibrinolytic treatment
• Radiological guided catheter embolectomy
Incorrect Options:
Option A- Pulmonary angiography: Pulmonary angiography involves injecting contrast dye into the pul
monary arteries to visualize blood flow and any potential blockages. While it is considered the gold sta
ndard for diagnosing pulmonary embolism, it is an invasive procedure and is not typically used as the i
nitial investigation. Non-invasive imaging techniques like CT scans are preferred as initial diagnostic st
eps.
Option B- Electrocardiogram (ECG): An ECG can provide valuable information about the heart's electri
cal activity. While it may show signs of right heart strain, such as an S1Q3T3 pattern, it is not a definitiv
e test for diagnosing pulmonary embolism. ECG findings, along with other clinical information, can cont
ribute to the diagnostic process, but they are not the primary investigation used for confirmation.
Option D- D-dimer assay: A D-dimer assay measures a breakdown product of blood clots. Elevated D-
dimer levels can indicate the presence of blood clots, including those associated with pulmonary embol
ism. However, D-dimer is a screening tool and not a confirmatory test. A
positive result might prompt further diagnostic imaging, such as MDCT, to confirm the diagnosis.

Solution for Question 5:


Correct Option-B Initiation of anticoagulant therapy with low-molecular-weight heparin
• The risk of developing venous thromboembolism (VTE), in this patient including pulmonary embolism,
is elevated due to a combination of factors such as immobilization, surgery, and underlying medical
conditions. The most appropriate prophylactic measure in this scenario is to initiate anticoagulant
therapy with low-molecular-weight heparin. This helps prevent the formation of blood clots and reduces
the risk of VTE. This approach is in line with guidelines for thromboprophylaxis in surgical patients at
higher risk of VTE.
• LMWH: MOA: activating antithrombin which accelerates the inactivation of coagulation enzymes
thrombin, Factor Xa and Factor IXa Side effects: Uncontrolled bleeding, injection site reactions, loss of
bone strength, elevated liver enzymes, heparin induced thrombocytopenia
• MOA: activating antithrombin which accelerates the inactivation of coagulation enzymes thrombin,
Factor Xa and Factor IXa
• Side effects: Uncontrolled bleeding, injection site reactions, loss of bone strength, elevated liver
enzymes, heparin induced thrombocytopenia
• Management options of pulmonary embolism Anticoagulant Therapy: Most patients with pulmonary
embolism are initially treated with anticoagulant medications (e.g., low-molecular-weight heparin,
fondaparinux, or direct oral anticoagulants). These medications prevent further clot formation and allow
the body to naturally dissolve existing clots. Contraindications: active bleeding, coagulopathy, recent
major surgeries, acute intracranial hemorrhage, major trauma Fibrinolytic Therapy: In severe cases,
such as massive pulmonary embolism causing hemodynamic instability, fibrinolytic therapy

Page 18

1351
(thrombolytics) may be considered. These drugs help dissolve blood clots rapidly and improve blood
flow. Observation: Stable patients with moderate or low-risk pulmonary embolism may be managed
with anticoagulation and close observation. This approach allows monitoring of the patient's condition
and the natural resolution of clots over time. Inferior Vena Cava (IVC) Filter: For patients with
contraindications to anticoagulants or those at high risk of bleeding, an IVC filter may be placed. This
device prevents large blood clots from traveling to the lungs by trapping them in the vena cava.
Embolectomy or Catheter-based Interventions: In cases of severe pulmonary embolism with
hemodynamic instability or failure of medical therapy, procedures like radiological-guided catheter
embolectomy or surgical embolectomy may be performed to physically remove the blood clot and
restore blood flow.
• Anticoagulant Therapy: Most patients with pulmonary embolism are initially treated with anticoagulant
medications (e.g., low-molecular-weight heparin, fondaparinux, or direct oral anticoagulants). These
medications prevent further clot formation and allow the body to naturally dissolve existing clots.
• Contraindications: active bleeding, coagulopathy, recent major surgeries, acute intracranial
hemorrhage, major trauma
• Fibrinolytic Therapy: In severe cases, such as massive pulmonary embolism causing hemodynamic
instability, fibrinolytic therapy (thrombolytics) may be considered. These drugs help dissolve blood clots
rapidly and improve blood flow.
• Observation: Stable patients with moderate or low-risk pulmonary embolism may be managed with
anticoagulation and close observation. This approach allows monitoring of the patient's condition and
the natural resolution of clots over time.
• Inferior Vena Cava (IVC) Filter: For patients with contraindications to anticoagulants or those at high
risk of bleeding, an IVC filter may be placed. This device prevents large blood clots from traveling to the
lungs by trapping them in the vena cava.
• Embolectomy or Catheter-based Interventions: In cases of severe pulmonary embolism with
hemodynamic instability or failure of medical therapy, procedures like radiological-guided catheter
embolectomy or surgical embolectomy may be performed to physically remove the blood clot and
restore blood flow.
• MOA: activating antithrombin which accelerates the inactivation of coagulation enzymes thrombin,
Factor Xa and Factor IXa
• Side effects: Uncontrolled bleeding, injection site reactions, loss of bone strength, elevated liver
enzymes, heparin induced thrombocytopenia
Management options of pulmonary embolism
• Anticoagulant Therapy: Most patients with pulmonary embolism are initially treated with anticoagulant
medications (e.g., low-molecular-weight heparin, fondaparinux, or direct oral anticoagulants). These
medications prevent further clot formation and allow the body to naturally dissolve existing clots.
• Contraindications: active bleeding, coagulopathy, recent major surgeries, acute intracranial
hemorrhage, major trauma
• Fibrinolytic Therapy: In severe cases, such as massive pulmonary embolism causing hemodynamic
instability, fibrinolytic therapy (thrombolytics) may be considered. These drugs help dissolve blood clots
rapidly and improve blood flow.
• Observation: Stable patients with moderate or low-risk pulmonary embolism may be managed with
anticoagulation and close observation. This approach allows monitoring of the patient's condition and
the natural resolution of clots over time.
• Inferior Vena Cava (IVC) Filter: For patients with contraindications to anticoagulants or those at high
risk of bleeding, an IVC filter may be placed. This device prevents large blood clots from traveling to the

Page 19

1352
lungs by trapping them in the vena cava.
• Embolectomy or Catheter-based Interventions: In cases of severe pulmonary embolism with
hemodynamic instability or failure of medical therapy, procedures like radiological-guided catheter
embolectomy or surgical embolectomy may be performed to physically remove the blood clot and
restore blood flow.
Incorrect Options:
Option A- Placement of an inferior vena cava (IVC) filter: IVC filters are used in patients at high risk of
developing deep vein thrombosis (DVT) or pulmonary embolism when anticoagulant therapy is contrain
dicated or ineffective. However, in this case, the patient's history of congestive heart failure and atrial fi
brillation makes them a candidate for anticoagulant therapy, which is a
more appropriate prophylactic measure.
Option C- Administration of oxygen therapy during the surgery: While oxygen therapy is important durin
g and after surgery to maintain adequate oxygen saturation, it is not a specific prophylactic measure for
preventing pulmonary embolism. Oxygen therapy helps maintain optimal oxygen levels in the blood an
d supports healing, but it does not directly target the prevention of blood clot formation.
Option D- Utilization of intermittent pneumatic compression devices: Intermittent pneumatic compressio
n (IPC) devices are commonly used to prevent deep vein thrombosis (DVT) in hospitalized patients, pa
rticularly those at risk due to immobilizationor surgery. While they can help prevent DVT, the patient's h
istory of atrial fibrillation and congestive heart failure suggests a
higher risk of atrial thrombi, making anticoagulant therapy a more appropriate choice.

Solution for Question 6:


Correct Option C- Atelectasis
• In the given scenario, the child's history of choking on peanuts and the presence of coughing,
wheezing, and localized wheezing on auscultation suggest a tracheo-bronchial foreign body aspiration.
Atelectasis is a common complication associated with a delayed diagnosis of tracheo-bronchial foreign
body.
• Atelectasis refers to the partial or complete collapse of lung tissue, which can occur when a foreign
object obstructs the airways, preventing proper ventilation of the affected lung segment. This can lead
to decreased oxygenation and respiratory distress. Prompt diagnosis and removal of the foreign body
through rigid bronchoscopy are crucial to prevent complications such as atelectasis and associated
infections.
• Tracheo-Bronchial Foreign body
• Cause: Aspiration of foreign objects into airways (trachea/bronchi).
• Common in: Toddlers, often due to aspirated peanuts.
• Clinical Features: Choking, coughing, wheezing, potential respiratory obstruction.
• Diagnosis: Radiopaque foreign body on CXR, confirmed by rigid bronchoscopy.
• Complications: Air trapping, atelectasis, infection; prompt diagnosis is crucial.
Tracheo-Bronchial Foreign body
Incorrect Options:

Page 20

1353
Option A- Hypertension: Hypertension refers to high blood pressure. it is not directly related to the com
plications of tracheo-bronchial foreign body aspiration. The symptoms described in the scenario (cough
ing, choking, wheezing) are more indicative of a respiratory issue, not a cardiovascular one.
Option B- Pneumothorax: Pneumothorax is the presence of air or gas in the pleural cavity, causing lun
g collapse. It can occur spontaneously or due to trauma, but it is not a typical complication of tracheo-b
ronchial foreign body aspiration. The symptoms described are more consistent with airway obstruction
rather than a pneumothorax.
Option D- Hepatitis: Hepatitis is inflammation of the liver, often caused by viral infections or other factor
s. It is not directly related to the respiratory system or the complications of foreign body aspiration in th
e airway

Solution for Question 7:


Correct Option C- Bronchial Adenoma
• This patient's history of recurrent hemoptysis, non-productive cough, and intermittent symptoms,
along with the findings of a highly vascular tumor obstructing the main bronchi, are consistent with
bronchial adenoma. Bronchial adenomas include various neoplasms of the tracheobronchial tree, such
as carcinoid tumors (most common), adenocystic tumors, and muco-epidermoid tumors. These tumors
can cause bleeding, leading to hemoptysis, and often obstruct the airways, causing cough and
breathing difficulties.
• Clinical Features: Recurrent hemoptysis, chronic cough, intermittent symptoms, airway obstruction.
• Diagnosis: Bronchoscopy with biopsy; highly vascular tumors may bleed during biopsy.
• Treatment: Surgical resection; high survival rate (>95%) post-surgery.
Incorrect Options:
Option A- Lung Cancer: While lung cancer can present with symptoms like cough, hemoptysis, and air
way obstruction, the patient's lack of smoking history and the characteristic features of a
highly vascular tumor obstructing the main bronchi make bronchial adenoma a more likely diagnosis.
Option B- Tuberculosis: Tuberculosis primarily presents with symptoms like cough, fever, night sweats,
and weight loss. The intermittent nature of symptoms and the presence of a
highly vascular obstructing tumor are not consistent with tuberculosis.
Option D- Pulmonary Embolism: Pulmonary embolism typically presents with sudden onset of chest pai
n, shortness of breath, and possibly hemoptysis. While hemoptysis can occur, the persistent non-produ
ctive cough and the presence of a vascular tumor are more indicative of a
structural lesion like bronchial adenoma.

Solution for Question 8:


Correct Option C- Lambert-Eaton Syndrome
• Lambert-Eaton Syndrome (LES) is commonly associated with Small Cell Lung Carcinoma (SCLC),
which is the type of lung cancer diagnosed in this patient.
• Symptoms of LES include proximal muscle weakness and diminished reflexes, which align with the
patient's clinical presentation.

Page 21

1354
• Elevated serum calcium levels can also occur in Lambert-Eaton Syndrome due to the release of
parathyroid hormone-related peptide by the tumor cells, leading to hypercalcemia.
• Therapy is directed at the primary tumor with resection, radiation, and/or chemotherapy.
Incorrect Choices:
Option A- Acanthosis Nigricans:
• Acanthosis nigricans is a skin condition characterized by thick, dark, and velvety skin along the body
folds and creases.
• It is commonly associated with Adenocarcinoma, not Small Cell Lung Carcinoma, making it less likely
in this case.
Option B- SIADH (Syndrome of Inappropriate Antidiuretic Hormone):
• SIADH results in excessive production of antidiuretic hormone, leading to water retention and
dilutional hyponatremia.
• While SIADH can occur in Small Cell Lung Carcinoma, the patient's symptoms of muscle weakness
and elevated calcium levels suggest a different paraneoplastic syndrome.
Option D- Cushing Syndrome:
• Cushing Syndrome results from excessive cortisol production, often due to adrenal tumors or
ACTH-secreting tumors.
• While cortisol levels can be elevated in lung cancer, the patient's muscle weakness and
hypercalcemia are more suggestive of another paraneoplastic syndrome.

Solution for Question 9:


Correct Option C- C8-T1
• Pancoast Tumor, also known as superior sulcus tumor, commonly involves the nerve root values of
C8-T1. This involvement can lead to the clinical manifestations described, including pain in the upper
arm, sensory disturbances in the forearm and hand, weakness of intrinsic hand muscles, and ipsilateral
Horner's syndrome.
• Pancoast tumor: The clinical manifestations are known as Pancoast syndrome. It results from local
extension of tumor growing in the apex of lung.
• Diagnosis: IOC - MRI
• Treatment: Pre-op radiotherapy followed by En-bloc resection of lung and chest wall; +/- Post op RT/
Brachytherapy
Incorrect Choices:
Option A- C4-C5:
• Nerve roots at the C4-C5 level primarily contribute to motor and sensory functions of the diaphragm,
shoulder muscles, and upper extremities. However, they are not typically associated with the clinical
manifestations seen in Pancoast Tumor.

Option B- C6-C7:

Page 22

1355
• Nerve roots at the C6-C7 level are responsible for motor and sensory functions of the upper
extremities, particularly the forearm muscles. While they can cause symptoms in the arm and hand,
they are not commonly involved in Pancoast Tumor-related symptoms.
Option D- T1-T2:
• Nerve roots at the T1-T2 level contribute to motor and sensory functions of the upper extremities,
particularly the upper arm and shoulder. While they can lead to symptoms in the upper arm, they are
not commonly associated with the specific symptoms described in Pancoast Tumor.

Solution for Question 10:


Correct Options A- Myxoma
• The clinical presentation, physical findings, and imaging findings described are characteristic of a
cardiac myxoma.
• Myxomas are the most common primary cardiac tumors.
• They often attach to the interatrial septum and can prolapse into the left atrium during diastole, leading
to symptoms such as exertional dyspnea, palpitations, and chest pain.
Atrial myxoma
• M/c primary cardiac tumor, m/c site - left atrium (tumor arises from inter-atrial septum)
• Inheritance: Sporadic-90%; Familial-10%

• Clinical features: Signs and symptoms related to: Mitral stenosis, Mitral regurgitation, mitral valve
obstruction Systemic embolization- neurological deficit Constitutional symptoms - fever and weight loss
• Signs and symptoms related to: Mitral stenosis, Mitral regurgitation, mitral valve obstruction
• Systemic embolization- neurological deficit
• Constitutional symptoms - fever and weight loss
• Diagnosis: IOC - 2D transthoracic/ Transesophageal echocardiography

Page 23

1356
• Treatment: Surgical excision with cardiopulmonary bypass Causes of recurrence: Multifocal lesions
(familial cases), Inadequate resection (Sporadic cases)
• Surgical excision with cardiopulmonary bypass
• Causes of recurrence: Multifocal lesions (familial cases), Inadequate resection (Sporadic cases)
• Signs and symptoms related to: Mitral stenosis, Mitral regurgitation, mitral valve obstruction
• Systemic embolization- neurological deficit
• Constitutional symptoms - fever and weight loss
• Surgical excision with cardiopulmonary bypass
• Causes of recurrence: Multifocal lesions (familial cases), Inadequate resection (Sporadic cases)
Incorrect Options:
Option B- Angiosarcoma:
• Angiosarcoma is the most common malignant tumor of the heart in adults.
• It often affects the right atrium and is associated with a poor prognosis.
• Angiosarcomas can cause symptoms like fatigue, arrhythmias, and heart failure due to obstruction of
blood flow.
Option C- Rhabdomyoma:
• Rhabdomyoma is the most common benign tumor of the heart in children.
• It's typically seen in infants and young children.
• These tumors are usually found within the ventricles and can lead to heart failure and arrhythmias.
Option D- Rhabdomyosarcoma:
• Rhabdomyosarcoma is the most common malignant tumor of the heart in children.
• It's a type of soft tissue sarcoma that can occur in various parts of the body, including the heart.
• Symptoms and manifestations depend on the location and extent of the tumor.

Solution for Question 11:


Correct Option B - Pancoast tumour:

Page 24

1357
• The given Chest X-ray shows a tumour in the lung apices
• Clinical signs-- progressively worsening right shoulder pain radiating down the right arm and into the
upper back, sweating on the face, and ptosis; all these indicate the Pancoast tumour of the lung.
• A Pancoast tumour is an apical tumour of lungs typically found in conjunction with a history of smoking
tobacco.
• The clinical signs and symptoms may be mistaken for neurovascular compromise at the level of the
superior thoracic aperture.
• The patient's smoking history, rapid onset of clinical signs and symptoms, and pleuritic pain suggest
an apical tumour.
• A Pancoast tumour can give rise to both Pancoast syndrome and Horner's syndrome.
• When the brachial plexus roots are involved, it will produce Pancoast syndrome; involvement of
sympathetic fibres as they exit the cord at the T1 level and ascend to the superior cervical ganglion will
produce Horner's syndrome
Option A: Horner's syndrome
• A constricting pupil (miosis), drooping of the upper eyelid (ptosis), an absence of facial sweat
(anhidrosis), and sinking of the eyeball into the bone chamber that covers the eye are all signs of
Horner syndrome (enophthalmos).
Option C: Larynx CA
• Laryngopharyngeal cancer symptoms: A modification in voice, such as a raspy tone Difficult in
swallowing A growth or lump in the neck A persistent cough or shortness of breath A prolonged
headache or sore throat A loud wheezing sound was made during breathing In extreme circumstances,
respiratory problems
• A modification in voice, such as a raspy tone
• Difficult in swallowing
• A growth or lump in the neck
• A persistent cough or shortness of breath
• A prolonged headache or sore throat

Page 25

1358
• A loud wheezing sound was made during breathing
• In extreme circumstances, respiratory problems
• A modification in voice, such as a raspy tone
• Difficult in swallowing
• A growth or lump in the neck
• A persistent cough or shortness of breath
• A prolonged headache or sore throat
• A loud wheezing sound was made during breathing
• In extreme circumstances, respiratory problems
Option D: Superior vena cava syndrome
• When blood flow via the superior vena cava (SVC) is impeded,it leads to several issues , known as
superior vena cava syndrome (SVCs).
• A sizable vein called the SVC carries blood from the head, neck, arms, and upper chest to the heart.
The majority of patients with SVCs have cancer.

Solution for Question 12:


Correct Option B - Posterior mediastinum:
• Clinical signs-- previous history of hypertension, mass in the mediastinum and elevated
catecholamine levels suggest the pheochromocytoma in the thorax region.
• Pheochromocytoma is a rare tumour of the adrenal medulla composed of chromaffin cells, also known
as pheochromocytes.
• The posterior mediastinum is the usual site of intrathoracic pheochromocytoma. They have also rarely
been reported to occur in the middle mediastinum, involving the left atrial wall or interatrial septum and
aortic arch.

Page 26

1359
• The above radiography shows the posterior mediastinal mass, which is probably a
pheochromocytoma.
• Risk factors: Sporadic/ familial Syndromic: MEN2A/ VHL syndrome/ NGF-1/ Sturge Weber syndrome
• Sporadic/ familial
• Syndromic: MEN2A/ VHL syndrome/ NGF-1/ Sturge Weber syndrome
• Clinical features: Classical triad: Palpitations, Headache, Diaphoresis
• Classical triad: Palpitations, Headache, Diaphoresis
• Investigation: 24 hour urine catecholamines and VMA level (most sensitive screening test) Plasma
free metanephrine (best for diagnosis) MRI - radiologic IOC FDG-PET / PET scan: gold standard
• 24 hour urine catecholamines and VMA level (most sensitive screening test)
• Plasma free metanephrine (best for diagnosis)
• MRI - radiologic IOC
• FDG-PET / PET scan: gold standard
• Treatment: Adrenalectomy - TOC
• Sporadic/ familial
• Syndromic: MEN2A/ VHL syndrome/ NGF-1/ Sturge Weber syndrome
• Classical triad: Palpitations, Headache, Diaphoresis
• 24 hour urine catecholamines and VMA level (most sensitive screening test)
• Plasma free metanephrine (best for diagnosis)
• MRI - radiologic IOC
• FDG-PET / PET scan: gold standard
Incorrect Options:
Option A - Anterior mediastinum
• Pheochromocytomas are neuroendocrine tumours that produce catecholamines and typically develop
in the sympathetic paraganglia or adrenal medulla.
• Posterior mediastinum is the most common site for intrathoracic pheochromocytoma..
Option C - Middle mediastinum:
• Most adult mediastinal tumours are malignant lymphomas or thymomas typically developing in the
anterior mediastinum.
Option D - Superior mediastinum:
• The superior mediastinum does not have any pheochromocyte mass; otherwise, the neck and
surrounding area masses could extend into the superior mediastinum.

Solution for Question 13:


Correct Option C - Malignant Mesothelioma:

Page 27

1360
• These clinical signs are suggestive of malignant mesothelioma which is positive for calretinin and WT
1 markers.
• Malignant mesothelioma is the most common tumor of the pleura.

• In 20% of malignant mesotheliomas, the tumor arises from the peritoneum.


• Asbestos exposure is the major known risk factor. More typical in males, most common after 40 years
of age.
• There are three types: Epithelial Sarcomatous Biphasic
• Epithelial
• Sarcomatous
• Biphasic
• Hemorrhagic pleural effusion is usually seen with malignant mesothelioma.
• The mesothelium that lines the lung, chest wall and abdomen is affected by malignant mesothelioma.
• Diagnosis of mesothelioma is confirmed with a pleural biopsy.
• Treatment options are surgery, chemotherapy, radiotherapy, immunotherapy, and hyperthermic
intrathoracic chemotherapy.
• Epithelial
• Sarcomatous
• Biphasic
Incorrect Options:
Option A - Small cell lung cancer:
• Small-cell lung cancer is the development of malignant cells in the lung tissues.
• The primary risk factor for small cell lung cancer is smoking.
• Coughing and shortness of breath are symptoms of small-cell lung cancer.
• Marker: Neuron specific enolase

Page 28

1361
Option B - Thoracic Pheochromocytoma:
• The increased release of catecholamines causes elevated blood pressure and other
pheochromocytoma-specific symptoms.
• Less frequent symptoms may also include pain in the chest or abdomen, nausea, vomiting, diarrhea,
constipation, pallor, weakness, and weight loss.
• Marker: Plasma metanephrine
Option D - Carcinoid tumor:
• A particular variety of neuroendocrine tumors that develop from neuroendocrine cells is the carcinoid
tumor.
• The symptoms of carcinoid tumor are: Diarrhea Wheezing Flushing Abdominal cramping Peripheral
edema
• Diarrhea
• Wheezing
• Flushing
• Abdominal cramping
• Peripheral edema
• Marker: 5-hydroxy-indole-acetic acid and Chromogranin A
• Diarrhea
• Wheezing
• Flushing
• Abdominal cramping
• Peripheral edema

Solution for Question 14:


Option D: Ganglioneuroma
• Young age, progressive dyspnea on exertion and radiological evaluation showing round,
well-demarcated mass in middle mediastinum all suggest the differentials of the bronchogenic cyst,
ascending aortic aneurysm, pericardial cyst, ganglioneuroma.
• Ganglioneuromas are usually rare and asymptomatic; some possible symptoms are usually caused
by the compression of surrounding structures or hormone secretions (e.g., norepinephrine) and
associated effects of them (e.g., diarrhea, hypertension).
• Ganglioneuromas most often start to develop in autonomic nerve cells and are considered rare
tumours. These are usually noncancerous (benign) tumours.
• The intestines are the most common location of ganglioneuromas. They have a slow growth rate and
secrete certain hormones.
• No known risk factors are there. However, they are associated with genetic disorders such as
neurofibromatosis type 1.
• Diagnosis of the ganglioneuroma is usually done with the help of the CT imaging.

Page 29

1362
• Surgery is the only effective treatment.
Option A: Bronchogenic cysts
• Bronchogenic cysts form in the mediastinum and are abnormal growths of tissue.
• They are small in size,benign tumours and typically central in location.
• The most common symptoms include fever, cough, pain, dyspnea, pulmonary infections, and
tracheobronchial compression.
Option B: Ascending aortic aneurysm
• An ascending aortic aneurysm is a focal dilatation on the upper portion of the aorta to more than 1.5
times its normal diameter. They may cause the blood vessel wall to tear or break open as the
aneurysms bulge outward.
• The condition is commonly asymptomatic, but some patients may experience back pain, chest pain,
difficulty swallowing, coughing or wheezing, hoarseness, and shortness of breath.
• A chest X-ray, chest CT scan, and MR angiography are required to diagnose the condition.
Option C: Pericardial cyst
• The pericardial cyst causes a rare and benign mediastinal mass.
• They are most commonly asymptomatic, and incidental findings usually help diagnose the condition
on imaging.
• Less common symptoms may include persistent cough, dysphagia, dyspnea, and atypical chest pain.

Solution for Question 15:


Option A: Aspiration
• The given clinical signs and Chest X-ray suggest a lung abscess (Thick-walled right upper
lobe cavitary lung lesion with air-fluid level).
• A necrotizing lung infection, usually characterized by a pus-filled cavity lesion, is a condition known as
a lung abscess.
• The most common cause is patients with impaired consciousness aspiration of oral secretions.
• The most common cause of primary lung abscess is anaerobic bacteria.
• Symptoms include a persistent cough, weight loss, fever, and sweats.
• Primarily, a chest x-ray is required for the diagnosis. It is considered a common diagnostic tool for
monitoring aspiration pneumonia.
• Chest X-ray of a patient with anaerobic infection due to aspiration classically represents consolidation
with a single cavity that contains air-fluid levels in portions of the lung. This depends on when the
patient is recumbent.
• It could only be treated with aspiration of the lung abscess as it is non-responsive to medical
treatment.
Option B: Hematogenous spread from a distant site

Page 30

1363
• Necrotizing pneumonia is the less common cause of lung abscess. It usually develops from
hematogenous seeding of the lungs that is due to suppurative thromboembolism (e.g., septic embolism
because of IV drug use or Lemierre syndrome) or may be due to right-sided endocarditis.
• Unlike aspiration and obstruction, these conditions typically cause multiple abscesses instead of
isolated lung abscesses.
Option C: Direct Contact
• Aspiration of oral secretions by impaired-conscious patients often causes lung abscesses.
• The spread may occur from direct or indirect contact through the air or with an infected individual.
• Its spread occurs through contact with respiratory secretions, like mucus and saliva.
Option D: Lymphatic spread
• Spontaneous rupture of lung abscess into the airways mainly aids in clearing the infection.
• Also, it might spread the infection to other parts of the lung but not the body's lymphatic system.

Solution for Question 16:


Option A: The most common cause is trauma
• Clinical signs: Chest discomfort and history of thoracotomy for tension pneumothorax after that pleural
effusion and drainage of effusion showing cloudy, off-white and lymphocytic-rich exudates suggest the
post-traumatic chylothorax.
• Chylothorax also occurs after non-surgical trauma, mostly due to penetrating or blunt injuries to the
chest or neck area and is usually a rare event.
• Other than blunt injuries, the thoracic duct can be lacerated by penetrating wounds more often to the
vertebral bodies.

• Diagnosis of chylothorax: Pleural fluid triglyceride more than 110 mg/dl

Page 31

1364
• Conservative Rx: temporary diet including medium chain triglycerides, complete parenteral nutrition,
somatostatin/ octreotide, thoracocentesis / drainage
• Surgical Rx: Thoracic duct ligation
Option B: Occurs when the thoracic duct remains intact
• The leakage of lymph from the disrupted thoracic duct into the pleural space causes a chylothorax.
• The largest lymphatic duct is the thoracic duct, which typically transports lymph from the body to
veins.
Option C: Immediate thoracotomy should be done
• Thoracotomy includes immediate risks such as infection, persistent air leakage from the lungs,
bleeding and pain.
• After thoracotomy, intercostal nerve damage and subsequent dysfunction are the most commonly
encountered complications.
Option D: Patients with this condition should undergo prolonged tube thoracostomy
• Chest tube insertion and administration of octreotide and medium-chain triglycerides are the known
treatments of choice.
• Prolonged tube thoracostomy with chest tube drainage should not be done to the patient because this
will lead to malnutrition and immunologic incompetence.

Solution for Question 17:


Option B: Growth hormone
• Clinical signs: Known smoker has a history of hypertension and presents with shortness of breath on
examination, dullness on percussion, and reduced breath sound on further evaluation of bilateral hilar
lymph mass, of course, suggestive of small cell carcinoma, which is not associated with growth
hormone, so it remains same as normal and has no change at all.
• Tobacco smoking is the most prevalent cause of small-cell lung cancer.
• All lung cancer deaths (about 80%) result from smoking; even for small cell lung cancer (SCLC), this
number is probably higher.
• Small cell lung cancer is a tumour that characterizes differentiation of the neuroendocrine system and
various types of hormones, including adrenocorticotropic hormone (ACTH), antidiuretic hormone
(ADH), atrial natriuretic peptide (ANP), arginine vasopressin (AVP), calcitonin, and serotonin.
• These increased secretions of hormones lead to paraneoplastic syndrome development, and some
symptoms include consciousness, weakness or disturbance that precedes other clinical features of
cancer.
• A growth hormone is a central regulator for developing tissues and organs. It has anabolic as well as
catabolic effects in a tissue-dependent manner.
• Fluctuations in growth hormone levels are not associated with small cell lung cancer (SCLC)
compared to other hormones.
Central location

Page 32

1365
• ACTH
• Calcitonin
• Gastrin Releasing peptide
• AVP
• ANF
Superior response
• Markers of SCLC: NCAM/CD56, Chromogranin, Synaptophysin, INSM1
Option A: ACTH
• Small cell lung cancer is the most common cancer that secretes ACTH.
• The patient's pituitary gland with small cell lung cancer releases a large amount of ACTH.
• A large amount of ACTH stimulates the release and production of cortisol (a stress hormone).
Option C: ANF
• Atrial natriuretic factor (ANF) is a hormone secreted by cardiac atria.
• In small cell lung cancer, the levels of ANF are elevated because ANF inhibits the adhesion of tumour
cells to pulmonary arterial and microvascular endothelial cells; ANF does it by suppressing the
expression of E-selectin. ANF prevents cancer metastasis and inhibits the adhesion of tumour cells to
inflamed endothelial cells.
Option D: AVP
• Small-cell lung cancer (SCLC) produces various mitogenic neuropeptides, usually not found in normal
respiratory epithelium. Arginine vasopressin (AVP) is detected in up to two-thirds of SCLC tumours.
However, normal physiological expression of AVP is essentially restricted to the hypothalamus.
• In patients with small-cell lung cancer, hyponatremia can be caused by arginine vasopressin (AVP)
tumour production. This results in the syndrome of inappropriate antidiuretic hormone.

Solution for Question 18:


Option C: Squamous cell carcinoma
• Chest X-ray revealed round opacification in the left upper apical lobe, thus confirming the diagnosis of
squamous cell carcinoma.
• Squamous cell carcinoma (SCC) of the lung is a type of non-small cell lung cancer (NSCLC).
• Squamous cell lung carcinoma often occurs in the central part of the lung or the main airway, such as
the left or right bronchus.
• MC in smokers, MC type in India
• Central in distribution and prone to undergo central necrosis and cavitation.
• MC variety associated with hypercalcemia (produces PTHrP)
• Pancoast tumour is histologically SCC . Associated with the best prognosis
• Associated with the best prognosis

Page 33

1366
• Diagnostic tests: X-rays chest CT scans Sputum cytology Bronchoscopy Mediastinoscopy, in which
an incision made in your neck and tissue samples from lymph nodes are collected. Needle biopsy, in
which your doctor inserts a needle through your chest wall and into your lungs to take cell samples
Biopsy (tissue sample)
• X-rays chest
• CT scans
• Sputum cytology
• Bronchoscopy
• Mediastinoscopy, in which an incision made in your neck and tissue samples from lymph nodes are
collected.
• Needle biopsy, in which your doctor inserts a needle through your chest wall and into your lungs to
take cell samples
• Biopsy (tissue sample)
• Associated with the best prognosis
• X-rays chest
• CT scans
• Sputum cytology
• Bronchoscopy
• Mediastinoscopy, in which an incision made in your neck and tissue samples from lymph nodes are
collected.
• Needle biopsy, in which your doctor inserts a needle through your chest wall and into your lungs to
take cell samples
• Biopsy (tissue sample)

• Histology: polygonal cells with intercellular bridges, crisp eosinophilic cytoplasm.


• Tumor markers: p40, p63, cytokeratin 5/6, desmocollin-3

Page 34

1367
• Treatment: surgical resection is the first line of therapy
Option A: Secondary tuberculosis
• Secondary tuberculosis usually occurs because of the reactivation of latent tuberculosis infection.
• The lesions of secondary tuberculosis are present in the lung apices.
• A smaller proportion of people who develop secondary tuberculosis do so after getting infected a
second time (re-infection).
• The fate of secondary lung TB: Fibrosis Calcification Tuberculous pneumonia Miliary tuberculosis
Empyema
• Fibrosis
• Calcification
• Tuberculous pneumonia
• Miliary tuberculosis
• Empyema
• Fibrosis
• Calcification
• Tuberculous pneumonia
• Miliary tuberculosis
• Empyema
Option B: Adenocarcinoma
• Adenocarcinoma starts in the cells that secrete mucus.
• This type of lung cancer occurs mainly in chronic smokers.
• Adenocarcinoma is usually found in the outer parts of the lung and is more likely to be found before it
has spread.
Option D: Asbestosis
• Asbestosis is a chronic lung disease caused by inhaling asbestos fibres.
• Prolonged exposure to these fibres can cause lung tissue scarring and shortness of breath.
• Asbestosis symptoms can range from mild to severe and usually don't appear until many years after
initial exposure.
• Signs and symptoms include: Shortness of breath Persistent dry cough Chest tightness or pain Dry
and crackling sounds in your lungs during inhalation Clubbing
• Shortness of breath
• Persistent dry cough
• Chest tightness or pain
• Dry and crackling sounds in your lungs during inhalation
• Clubbing
• Shortness of breath
• Persistent dry cough

Page 35

1368
• Chest tightness or pain
• Dry and crackling sounds in your lungs during inhalation
• Clubbing

Solution for Question 19:


Option A: Bronchoscopy, lavage, and brushing
• According to the clinical scenario, the diagnosis is pulmonary tuberculosis, and the investigation
would include bronchoscopy, lavage, and brushing.
• Pulmonary tuberculosis is a severe infection caused by the bacterium.
• Mycobacterium tuberculosis (MTB) involves the lungs but may spread to other organs.
• TB is a contagious disease that can infect anyone exposed to MTB.
• Signs and symptoms of tuberculosis infection: Night sweats Persistent cough for more than two
weeks Bloody sputum Weight loss Fever Fatigue Anorexia Chest pain Difficulty breathing
• Night sweats
• Persistent cough for more than two weeks
• Bloody sputum
• Weight loss
• Fever
• Fatigue
• Anorexia
• Chest pain
• Difficulty breathing
• Bronchoscopy is a procedure that lets doctors look at your lungs and air passages.
• During bronchoscopy, a thin tube (bronchoscope) is passed through your nose or mouth, throat, and
lungs.
• Night sweats
• Persistent cough for more than two weeks
• Bloody sputum
• Weight loss
• Fever
• Fatigue
• Anorexia
• Chest pain
• Difficulty breathing

Page 36

1369
Confirmation of disease
• Carcinoma of the bronchus
• Inflammatory and infective process
• Before lung resection
• Before esophageal resection
• Persistent hemoptysis
• Removal of secretions
• Removal of foreign bodies
• Stent placement, endobronchial resection

Option B: CT scan
• A computerized tomography (CT) scan combines a series of X-ray images from different angles
around your body. It uses computer processing to create cross-sectional images (slices) of the bones,
blood vessels, and soft tissues inside your body.
• CT scan images provide more detailed information than plain X-rays .
Options C: X-Ray
• X-rays are a form of electromagnetic radiation similar to visible light.
• Unlike light, however, x-rays have higher energy and can pass through most objects, including the
body.
• Medical X-rays generate images of tissues and structures inside the body.
Option D: FNAC
• Fine needle aspiration cytology (FNAC) is a simple, quick, and inexpensive method to sample
superficial masses or lesions in various sites.
• The technique involves a small needle inserted into the lesion, drawing several cells. These cells are
then smeared on glass slides, stained, and examined under the microscope to determine the diagnosis.
• It requires 15-20 minutes.
• It causes minimal trauma to the patient and carries virtually no risk.

Solution for Question 20:


Option D: Intercostal arteries
• CT contrast revealed worsening hemothorax, and vessels involved in hemothorax are intercostal
arteries.
• Hemothorax is a collection of blood in the pleural cavity.
• The goal of treatment is to get the person stable, stop the bleeding, and remove the blood and air in
the pleural space.
• A chest tube is inserted through the chest wall between the ribs to drain the blood and air.

Page 37

1370
• It is left in place and attached to suction for several days to re-expand the lung.
• Management of Hemothorax Tube thoracostomy drainage is the primary mode of treatment for
hemothorax. In adult patients, large–bore chest tubes, usually 36-42F, should be used to achieve
adequate drainage in adults. Most patients with hemothorax should be treated with tube thoracostomy
which allows continuous quantification of bleeding. If the bleeding emanates from the laceration of the
pleura, apposition of the two pleural surfaces is likely to stop bleeding. If pleural hemorrhage exceeds
200 ml/h, consideration should be given to thoracoscopy or thoracotomy.
• Tube thoracostomy drainage is the primary mode of treatment for hemothorax. In adult patients,
large–bore chest tubes, usually 36-42F, should be used to achieve adequate drainage in adults.
• Most patients with hemothorax should be treated with tube thoracostomy which allows continuous
quantification of bleeding.
• If the bleeding emanates from the laceration of the pleura, apposition of the two pleural surfaces is
likely to stop bleeding.
• If pleural hemorrhage exceeds 200 ml/h, consideration should be given to thoracoscopy or
thoracotomy.
• Tube thoracostomy drainage is the primary mode of treatment for hemothorax. In adult patients,
large–bore chest tubes, usually 36-42F, should be used to achieve adequate drainage in adults.
• Most patients with hemothorax should be treated with tube thoracostomy which allows continuous
quantification of bleeding.
• If the bleeding emanates from the laceration of the pleura, apposition of the two pleural surfaces is
likely to stop bleeding.
• If pleural hemorrhage exceeds 200 ml/h, consideration should be given to thoracoscopy or
thoracotomy.
Option A: Pulmonary artery
• The pulmonary arteries transport deoxygenated blood from the right side of the heart to the lungs for
oxygenation.
• These vessels serve as the conduit between the right side of the heart and the lungs.
Option B: Pulmonary vein
• Pulmonary veins are blood vessels that carry oxygen-rich blood from your lungs to your heart.
• Pulmonary veins are part of the body's pulmonary circuit.
• This system of blood vessels moves blood between your heart and your lungs. This circuit also
includes your pulmonary arteries.
Option C: Bronchial artery
• The bronchial arteries carry oxygenated blood to the lungs at a pressure six times that of the
pulmonary arteries.
• The bronchial arteries nourish the supporting structures of the lungs, including the pulmonary arteries,
but generally do not participate in gas exchange.

Solution for Question 21:

Page 38

1371
Correct Option A - Most common chest wall deformity:
• Based on the symptom, the most likely diagnosis is the pectus excavatum.
• Pectus excavatum is the most common congenital chest wall abnormality (90%), followed by pectus
carinatum.
• Pectus excavatum is a congenital chest wall deformity in which a few ribs and the sternum grow
abnormally, resulting in a concave appearance in the anterior chest wall. It is also called a sunken or
funnel chest deformity.
• Abnormalities of rib morphogenesis and growth are the most likely causes of pectus excavatum.
• Treatment Options: Early repair with best results between 2-8 years of age. Ravitch procedure: Open
invasive surgical procedure for pectus excavatum. Nuss Procedure: Minimal invasive surgery for
pectus excavatum.
• Early repair with best results between 2-8 years of age.
• Ravitch procedure: Open invasive surgical procedure for pectus excavatum.
• Nuss Procedure: Minimal invasive surgery for pectus excavatum.
• Early repair with best results between 2-8 years of age.
• Ravitch procedure: Open invasive surgical procedure for pectus excavatum.
• Nuss Procedure: Minimal invasive surgery for pectus excavatum.
Incorrect Options:
Option B
- Arises from excessive growth of the upper costal cartilages, causing posterior sternal depression:
• Pectus excavatum arises from the excessive growth of the lower costal cartilages, not the upper
cartilages.
• Thus, causing posterior sternal depression.
Option C - Haller index is the ratio of the length of the chest wall to the depth of the sternum to the vert
ebral body:
• Haller index is the ratio of the width of the chest wall to the depth of the sternum to the vertebral body,
not the ratio of length to depth.
• It is defined as the ratio of the transverse diameter (the horizontal distance of the inside of the ribcage)
and the anteroposterior diameter (the shortest distance between the vertebrae and sternum).

Page 39

1372
Option D - Ravitch procedure is minimally invasive surgery for pectus excavatum:
• Ravitch procedure is an open procedure for correction of pectus excavatum and is not minimally
invasive.
• In the Ravitch procedure, a larger incision is given across the mid-chest. The deformed cartilage
attaching the ribs to the lower breastbone is removed, thus allowing the sternum to move forward in a
more normal position. A temporary metal bar may need to be placed to keep the sternum oriented in
the best position.

Solution for Question 22:


Correct Option B - The best investigation for diagnosis is a non-contrast CT scan:
• Based on the history of road traffic accident, symptoms of shortness of breath with hypotension and
tachycardia associated with raised JVP, and the image shown, the most likely diagnosis is left-sided
tension pneumothorax.
• The gold standard investigation for its diagnosis is the non-contrast CT scan.
• Pneumothorax is characterised by the presence of air in the pleural cavity, which results in defective
oxygenation and ventilation.
• Tension pneumothorax is characterised by low blood pressure and low pO2.
• On examination, breath sounds are reduced or absent on the affected side of the chest, and the
trachea deviates towards the better side.
• The thorax may also be hyper-resonant on percussion, and jugular venous distention and tachycardia
may also be present.
• Symptoms may include chest pain, dyspnea, anxiety, fatigue
• Treatment includes: Oxygen supplementation Needle decompression followed by tube thoracostomy.
Treatment of tension pneumothorax is urgent needle decompression by inserting a large-bore needle
into the 2nd intercostal space in the midclavicular line, followed by chest drain insertion immediately
into the 5th intercostal space in the triangle of safety. Triangle of safety: anterior border of the

Page 40

1373
latissimus dorsi, the lateral border of the pectoralis major muscle, a line superior to the horizontal level
of the nipple
• Oxygen supplementation
• Needle decompression followed by tube thoracostomy.
• Treatment of tension pneumothorax is urgent needle decompression by inserting a large-bore needle
into the 2nd intercostal space in the midclavicular line, followed by chest drain insertion immediately
into the 5th intercostal space in the triangle of safety.
• Triangle of safety: anterior border of the latissimus dorsi, the lateral border of the pectoralis major
muscle, a line superior to the horizontal level of the nipple
• Oxygen supplementation
• Needle decompression followed by tube thoracostomy.
• Treatment of tension pneumothorax is urgent needle decompression by inserting a large-bore needle
into the 2nd intercostal space in the midclavicular line, followed by chest drain insertion immediately
into the 5th intercostal space in the triangle of safety.
• Triangle of safety: anterior border of the latissimus dorsi, the lateral border of the pectoralis major
muscle, a line superior to the horizontal level of the nipple
Incorrect Options:
Option A - The trachea has deviated to the affected side of the chest:
• As the patient is diagnosed with tension pneumothorax, the trachea deviates towards the better side
and opposite to the affected side.
• Therefore, this is not the correct option.
Option C - The treatment of choice is the insertion of intercostal drainage in 2nd intercostal space:
• In tension pneumothorax, the treatment of choice is inserting a chest drain in the 5th intercostal space
and not the 2nd intercostal space.
• Initially, needle decompression is done, followed by chest drain insertion.
Option D - The best X-ray film for diagnosis is a chest X-ray lateral view:
• The best X-ray film for diagnosis is chest X-ray AP view.

Solution for Question 23:


Correct Option A - Lung abscess unresponsive to medical management:
• Based on the symptoms, the most likely diagnosis is lung abscess.
• A lung abscess that is unresponsive to medical management is an indication for surgery.
• It is characterized by necrosis of the lung tissue and the formation of cavities containing necrotic
debris, caused mainly by microbial anaerobic infection.
• The formation of multiple small (less than 2 cm) abscesses is sometimes called necrotizing
pneumonia or lung gangrene.
• The usual symptoms are fever, cough with sputum production, night sweats, anorexia, and weight
loss.

Page 41

1374
• The expectorated sputum is characteristically foul-smelling and bad-tasting.
• Diagnostic Tests: A complete white blood cell count with differential may reveal leukocytosis and a left
shift. Obtain sputum for gram stain, culture, and sensitivity. If tuberculosis is suspected, an acid-fast
bacilli stain and mycobacterial culture should be requested. A blood culture may help establish the
etiology. Bronchoalveolar lavage
• A complete white blood cell count with differential may reveal leukocytosis and a left shift.
• Obtain sputum for gram stain, culture, and sensitivity.
• If tuberculosis is suspected, an acid-fast bacilli stain and mycobacterial culture should be requested.
• A blood culture may help establish the etiology.
• Bronchoalveolar lavage
• A complete white blood cell count with differential may reveal leukocytosis and a left shift.
• Obtain sputum for gram stain, culture, and sensitivity.
• If tuberculosis is suspected, an acid-fast bacilli stain and mycobacterial culture should be requested.
• A blood culture may help establish the etiology.
• Bronchoalveolar lavage
• Treatment: Medical: The standard treatment for an anaerobic lung abscess is clindamycin. An
antibiotic regimen should include coverage against S. aureus, Enterobacter, and Pseudomonas
species in hospitalized patients who have aspiration pneumonia and then develop a lung abscess.
When methicillin-resistant Staph aureus (MRSA) is suspected, vancomycin and linezolid should be
used. The antibiotics should generally be prescribed for 4-6 weeks. Surgery: The surgical procedure
done is either lobectomy or pneumonectomy.
• Medical: The standard treatment for an anaerobic lung abscess is clindamycin. An antibiotic regimen
should include coverage against S. aureus, Enterobacter, and Pseudomonas species in hospitalized
patients who have aspiration pneumonia and then develop a lung abscess. When
methicillin-resistant Staph aureus (MRSA) is suspected, vancomycin and linezolid should be used. The
antibiotics should generally be prescribed for 4-6 weeks.
• The standard treatment for an anaerobic lung abscess is clindamycin.
• An antibiotic regimen should include coverage against S. aureus, Enterobacter, and Pseudomonas
species in hospitalized patients who have aspiration pneumonia and then develop a lung abscess.
• When methicillin-resistant Staph aureus (MRSA) is suspected, vancomycin and linezolid should be
used.
• The antibiotics should generally be prescribed for 4-6 weeks.
• Surgery: The surgical procedure done is either lobectomy or pneumonectomy.
• The surgical procedure done is either lobectomy or pneumonectomy.
• Medical: The standard treatment for an anaerobic lung abscess is clindamycin. An antibiotic regimen
should include coverage against S. aureus, Enterobacter, and Pseudomonas species in hospitalized
patients who have aspiration pneumonia and then develop a lung abscess. When
methicillin-resistant Staph aureus (MRSA) is suspected, vancomycin and linezolid should be used. The
antibiotics should generally be prescribed for 4-6 weeks.
• The standard treatment for an anaerobic lung abscess is clindamycin.
• An antibiotic regimen should include coverage against S. aureus, Enterobacter, and Pseudomonas
species in hospitalized patients who have aspiration pneumonia and then develop a lung abscess.

Page 42

1375
• When methicillin-resistant Staph aureus (MRSA) is suspected, vancomycin and linezolid should be
used.
• The antibiotics should generally be prescribed for 4-6 weeks.
• Surgery: The surgical procedure done is either lobectomy or pneumonectomy.
• The surgical procedure done is either lobectomy or pneumonectomy.
• The standard treatment for an anaerobic lung abscess is clindamycin.
• An antibiotic regimen should include coverage against S. aureus, Enterobacter, and Pseudomonas
species in hospitalized patients who have aspiration pneumonia and then develop a lung abscess.
• When methicillin-resistant Staph aureus (MRSA) is suspected, vancomycin and linezolid should be
used.
• The antibiotics should generally be prescribed for 4-6 weeks.
• The surgical procedure done is either lobectomy or pneumonectomy.
• The usual indications for surgery are: Failure to respond to medical management Suspected
neoplasm Bronchopleural fistula Congenital lung malformation Sepsis syndrome (fever, tachycardia,
leukocytosis) Signs of progressive infection (enlarging cavity) Life-threatening ailments such as
respiratory failure and uncontrolled hemoptysis.
• Failure to respond to medical management
• Suspected neoplasm
• Bronchopleural fistula
• Congenital lung malformation
• Sepsis syndrome (fever, tachycardia, leukocytosis)
• Signs of progressive infection (enlarging cavity)
• Life-threatening ailments such as respiratory failure and uncontrolled hemoptysis.
• Failure to respond to medical management
• Suspected neoplasm
• Bronchopleural fistula
• Congenital lung malformation
• Sepsis syndrome (fever, tachycardia, leukocytosis)
• Signs of progressive infection (enlarging cavity)
• Life-threatening ailments such as respiratory failure and uncontrolled hemoptysis.
Incorrect Options:
Option B - Asthma exacerbation:
• The patient has a diagnosis of lung abscess, and there is no indication of surgery for asthma
exacerbation.
• The signs and symptoms of asthma exacerbations are as follows: Breathlessness Coughing
Wheezing Chest tightness Agitation Tachypnea Tachycardia
• Breathlessness
• Coughing

Page 43

1376
• Wheezing
• Chest tightness
• Agitation
• Tachypnea
• Tachycardia
• Breathlessness
• Coughing
• Wheezing
• Chest tightness
• Agitation
• Tachypnea
• Tachycardia
Option C- Acute pulmonary edema:
• Acute pulmonary edema is primarily due to increased capillary hydrostatic pressure secondary to
increased pulmonary venous pressure.
• The significant complications associated are respiratory fatigue and failure.
• Common symptoms are: Shortness of breath Diaphoresis Dyspnea on exertion Orthopnea
Paroxysmal nocturnal dyspnea Cough Pink and frothy sputum may be present in patients with severe
disease. Auscultation of the lungs usually reveals fine crepitations.
• Shortness of breath
• Diaphoresis
• Dyspnea on exertion
• Orthopnea
• Paroxysmal nocturnal dyspnea
• Cough
• Pink and frothy sputum may be present in patients with severe disease.
• Auscultation of the lungs usually reveals fine crepitations.
• Shortness of breath
• Diaphoresis
• Dyspnea on exertion
• Orthopnea
• Paroxysmal nocturnal dyspnea
• Cough
• Pink and frothy sputum may be present in patients with severe disease.
• Auscultation of the lungs usually reveals fine crepitations.
Option D- Mild Community-acquired pneumonia:

Page 44

1377
• Community-acquired pneumonia is an infection of the pulmonary parenchyma acquired outside the
hospital.
• Streptococcus pneumoniae and respiratory viruses are the most common pathogens in patients with
CAP.

Solution for Question 24:


Correct Option D - Poirier’s triangle
• The thoracic duct can easily be identified within Poirier's triangle.
• The borders of Poirier's triangle are: Inferior-Arch of Aorta Anterior- Left subclavian artery Posterior-
Vertebral column
• Inferior-Arch of Aorta
• Anterior- Left subclavian artery
• Posterior- Vertebral column
• The thoracic duct traverses the triangle on the esophagus that forms the floor of the triangle.
• Inferior-Arch of Aorta
• Anterior- Left subclavian artery
• Posterior- Vertebral column

Incorrect Options:
Option A - Calot's triangle:
• It is bounded: Medially by the hepatic duct Laterally by the cystic duct Superiorly by cystic artery
• Medially by the hepatic duct
• Laterally by the cystic duct
• Superiorly by cystic artery

Page 45

1378
• It is dissected during cholecystectomy.
• Medially by the hepatic duct
• Laterally by the cystic duct
• Superiorly by cystic artery
Option B - Hesselbach’s triangle:
• The borders of hesselbach's triangle are: Medial border: Lateral margin of the rectus sheath.
Superolateral border: Inferior epigastric vessels. Inferior border: Inguinal ligament
• Medial border: Lateral margin of the rectus sheath.
• Superolateral border: Inferior epigastric vessels.
• Inferior border: Inguinal ligament
• It is a potential area of weakness in the abdomen through which a hernia can protrude.
• Medial border: Lateral margin of the rectus sheath.
• Superolateral border: Inferior epigastric vessels.
• Inferior border: Inguinal ligament
Option C - Passaro's triangle:
• The borders of Passaro's triangle are: The confluence of the cystic and common bile duct superiorly
The second and third portions of the duodenum inferiorly The neck and body of the pancreas medially,
both dorsally and ventrally
• The confluence of the cystic and common bile duct superiorly
• The second and third portions of the duodenum inferiorly
• The neck and body of the pancreas medially, both dorsally and ventrally
• It is a source of origin for most gastrinomas.
• The confluence of the cystic and common bile duct superiorly
• The second and third portions of the duodenum inferiorly
• The neck and body of the pancreas medially, both dorsally and ventrally

Solution for Question 25:


Correct Option B - Pulmonary hamartoma:

Page 46

1379
• On the lateral chest radiograph, there is a round nodule seen at the lung base (white arrows) that
appears quite dense, suggesting calcification.
• CT scan of the left lower lobe demonstrates the typical "popcorn" calcification of a pulmonary
hamartoma (red arrow).
• PULMONARY HAMARTOMA: Most common benign tumour of the lung Most commonly, Hamartomas
are manifested by overgrowth of cartilage Clinical features Typically seen in 40-60 years of age, more
common in males Usually peripheral, slow growing tumor of lungs They are very rarely known to cause
hemoptysis and bronchial obstruction. Diagnosis Chest X-ray: Popcorn calcification is diagnostic Size
typically 1-3 cm. 90% are peripheral, and 10% are endobronchial CT scan: Coin lesion Treatment
Definitive treatment: Excision of lesion
• Most common benign tumour of the lung
• Most commonly, Hamartomas are manifested by overgrowth of cartilage
• Clinical features Typically seen in 40-60 years of age, more common in males Usually peripheral, slow
growing tumor of lungs They are very rarely known to cause hemoptysis and bronchial obstruction.
• Typically seen in 40-60 years of age, more common in males
• Usually peripheral, slow growing tumor of lungs
• They are very rarely known to cause hemoptysis and bronchial obstruction.
• Diagnosis Chest X-ray: Popcorn calcification is diagnostic Size typically 1-3 cm. 90% are peripheral,
and 10% are endobronchial CT scan: Coin lesion
• Chest X-ray: Popcorn calcification is diagnostic
• Size typically 1-3 cm.
• 90% are peripheral, and 10% are endobronchial
• CT scan: Coin lesion
• Treatment Definitive treatment: Excision of lesion
• Definitive treatment: Excision of lesion
• Most common benign tumour of the lung

Page 47

1380
• Most commonly, Hamartomas are manifested by overgrowth of cartilage
• Clinical features Typically seen in 40-60 years of age, more common in males Usually peripheral, slow
growing tumor of lungs They are very rarely known to cause hemoptysis and bronchial obstruction.
• Typically seen in 40-60 years of age, more common in males
• Usually peripheral, slow growing tumor of lungs
• They are very rarely known to cause hemoptysis and bronchial obstruction.
• Diagnosis Chest X-ray: Popcorn calcification is diagnostic Size typically 1-3 cm. 90% are peripheral,
and 10% are endobronchial CT scan: Coin lesion
• Chest X-ray: Popcorn calcification is diagnostic
• Size typically 1-3 cm.
• 90% are peripheral, and 10% are endobronchial
• CT scan: Coin lesion
• Treatment Definitive treatment: Excision of lesion
• Definitive treatment: Excision of lesion
• Typically seen in 40-60 years of age, more common in males
• Usually peripheral, slow growing tumor of lungs
• They are very rarely known to cause hemoptysis and bronchial obstruction.
• Chest X-ray: Popcorn calcification is diagnostic
• Size typically 1-3 cm.
• 90% are peripheral, and 10% are endobronchial
• CT scan: Coin lesion
• Definitive treatment: Excision of lesion

Incorrect Options:
Option A - Primary lung cancer:

Page 48

1381
• It presents with a single central or peripheral nodule.
• No calcification or irregular calcification
• Size is typically greater than 2cm.
Option C - Infectious granulomas:
• It presents with round well defined calcified nodules.
• 80% of benign pulmonary nodules are infectious.
Option D - Sarcoidosis:
• It presents with multiple bilateral cavitating nodular lesions.
• It is commonly associated with other systemic findings.

Solution for Question 26:


Correct Option C - Computed Tomography Angiography (CTA):
• In this case, a hypertensive patient presents with signs and symptoms of aortic dissection.
• Aortic dissection is a tear in the inner layer of the aorta.
• Risk factors: systemic hypertension, cystic medial necrosis
• Classification: DeBakey and Stanford classification

• It typically presents with severe chest pain radiating to the interscapular region, syncope, dyspnea,
weakness, hypotension, and wide pulse pressure.
• A widened mediastinum on a chest x-ray is characteristic of the diagnosis, but normal chest x-ray
findings do not rule out aortic dissection.
• CTA is a gold standard test for diagnosing aortic dissection with high sensitivity and specificity.

Page 49

1382
• Rx: emergency Dacron graft repair

Incorrect Options:
Option A - Chest x-ray:
• It is rapid and non-invasive.
• The widened mediastinum is a classic radiographic finding of aortic dissection.
• It is poorly sensitive, and normal findings do not rule out aortic dissection.
Option B - Echocardiography:
• It has minimal sensitivity and requires trained clinicians.
• It is indicated in unstable patients.
Option D - Aortography:
• It has very high accuracy and can be performed at the bedside.
• It is invasive and does not help in excluding the differential diagnosis.

Solution for Question 27:


Correct Option A - Primary spontaneous pneumothorax:
• In this case, a young male with a smoking history of three pack year presented with signs and
symptoms of primary spontaneous pneumothorax.
• Chest x-ray shows a large left pneumothorax with the whole lung collapsed towards the hilum.
• Primary spontaneous pneumothorax- spontaneous pneumothorax without underlying lung disease
• The rupture of subpleural apical blebs causes it.
• Risk factors include family history, male sex, young age, slim and tall stature, smoking, and
homocystinuria

Page 50

1383
• C/F: sudden onset and mild dyspnea
• Investigation: NCCT, CXR
• Treatment for low-risk patients is conservative, while for high-risk patients is chest tube placement.
Incorrect Options:
Option B - Tension Pneumothorax:
• The one-way valve mechanism is when air enters the pleural space but cannot exit on expiration.
• Immediate chest decompression is needed.
Option C - Secondary spontaneous pneumothorax:
• It occurs as a complication of underlying lung disease.
• COPD, pulmonary TB, and cystic fibrosis are the risk factors.
Option D - Traumatic pneumothorax:
• It is caused by blunt trauma, penetrating injury, or ventilator-associated.
• Most patients require immediate chest tube placement.

Solution for Question 28:


Option C: Posterior intercostal
• The patient is suffering from coronary heart disease. The cardiologist advises coronary bypass
surgery and uses the internal thoracic artery as the graft. The anterior intercostal arteries are ligated.
Hence the intercostal space will be supplied by posterior intercostals.
• The internal thoracic artery travels along the inner surface of the anterior chest wall. It divides into
branches, the musculophrenic and the superior epigastric arteries.
• Coronary heart disease is a type of heart disease where the heart's arteries cannot deliver enough
oxygen-rich blood to the heart.
• Coronary artery disease is often caused by cholesterol, which can partially or totally block blood flow
in the heart's large arteries.
• Coronary microvascular disease happens when the heart’s tiny blood vessels do not work normally.
• Treatment options include: Lifestyle modifications Smoking cessation Weight reduction Managing
stress Heart-healthy eating Medication ACE inhibitors and beta blockers Calcium channel blockers
Medicines to control blood sugar Metformin Nitrates Ranolazine Statins or non-statin therapies
Non-statin therapies Procedures: Percutaneous coronary intervention (PCI) Coronary artery bypass
grafting (CABG) Transmyocardial laser revascularization or coronary endarterectomy
• Lifestyle modifications Smoking cessation Weight reduction Managing stress Heart-healthy eating
• Smoking cessation
• Weight reduction
• Managing stress
• Heart-healthy eating

Page 51

1384
• Medication ACE inhibitors and beta blockers Calcium channel blockers Medicines to control blood
sugar Metformin Nitrates Ranolazine Statins or non-statin therapies Non-statin therapies
• ACE inhibitors and beta blockers
• Calcium channel blockers
• Medicines to control blood sugar
• Metformin
• Nitrates
• Ranolazine
• Statins or non-statin therapies
• Non-statin therapies
• Procedures: Percutaneous coronary intervention (PCI) Coronary artery bypass grafting (CABG)
Transmyocardial laser revascularization or coronary endarterectomy
• Percutaneous coronary intervention (PCI)
• Coronary artery bypass grafting (CABG)
• Transmyocardial laser revascularization or coronary endarterectomy
• Lifestyle modifications Smoking cessation Weight reduction Managing stress Heart-healthy eating
• Smoking cessation
• Weight reduction
• Managing stress
• Heart-healthy eating
• Medication ACE inhibitors and beta blockers Calcium channel blockers Medicines to control blood
sugar Metformin Nitrates Ranolazine Statins or non-statin therapies Non-statin therapies
• ACE inhibitors and beta blockers
• Calcium channel blockers
• Medicines to control blood sugar
• Metformin
• Nitrates
• Ranolazine
• Statins or non-statin therapies
• Non-statin therapies
• Procedures: Percutaneous coronary intervention (PCI) Coronary artery bypass grafting (CABG)
Transmyocardial laser revascularization or coronary endarterectomy
• Percutaneous coronary intervention (PCI)
• Coronary artery bypass grafting (CABG)
• Transmyocardial laser revascularization or coronary endarterectomy
• Smoking cessation
• Weight reduction

Page 52

1385
• Managing stress
• Heart-healthy eating
• ACE inhibitors and beta blockers
• Calcium channel blockers
• Medicines to control blood sugar
• Metformin
• Nitrates
• Ranolazine
• Statins or non-statin therapies
• Non-statin therapies
• Percutaneous coronary intervention (PCI)
• Coronary artery bypass grafting (CABG)
• Transmyocardial laser revascularization or coronary endarterectomy
Option A: Musculophrenic
• Branches of the musculophrenic artery supply the lower seventh, eighth, and ninth intercostal spaces.
• The musculophrenic artery function is to supply blood to the muscles of the thoracic wall.
Option B: Superior epigastric
• The superior epigastric artery passes into the rectus sheath of the anterior abdominal wall.
• The superior epigastric artery is a terminal branch of the internal thoracic artery that contributes to the
blood supply of the anterior abdominal wall.
• The superior epigastric artery branches into: Muscular branches that supply rectus abdominis
Cutaneous branches to the skin of the anterior abdominal wall.
• Muscular branches that supply rectus abdominis
• Cutaneous branches to the skin of the anterior abdominal wall.
• Muscular branches that supply rectus abdominis
• Cutaneous branches to the skin of the anterior abdominal wall.
Option D: Lateral thoracic
• The lateral thoracic artery arises from the second part of the axillary artery, and the thoracodorsal
artery is a branch of the subscapular artery, a branch of the third part of the axillary artery.

Solution for Question 29:


Option A: Left internal mammary artery
• The patient has triple vessel coronary disease (Right coronary artery, Left anterior descending artery,
and Left circumflex artery) diagnosed on coronary angiography, therefore, the cardiothoracic surgeon
has advised CABG. The left internal mammary artery is the most preferred graft for CABG.

Page 53

1386
• The CABG grafts are routinely the left internal mammary artery and the saphenous vein grafts out of
the lower extremities. The left internal mammary artery graft is the most preferred one.
• Coronary artery bypass grafting (CABG) is done for patients with coronary artery disease (CAD) to
increase the quality of life and reduce the mortality rate due to cardiac issues.
• Indications of CABG: Left main vessel disease is more than 50%. Triple vessel coronary artery
disease of more than 70% with or without proximal LAD involvement. Two-vessel disease: LAD and
one other major artery. One or more stenosis greater than 70% in a patient with significant symptoms of
angina despite maximal medications. One vessel disease, more than 70% of a survivor of sudden
cardiac death with ischemia-related ventricular tachycardia.
• Left main vessel disease is more than 50%.
• Triple vessel coronary artery disease of more than 70% with or without proximal LAD involvement.
• Two-vessel disease: LAD and one other major artery.
• One or more stenosis greater than 70% in a patient with significant symptoms of angina despite
maximal medications.
• One vessel disease, more than 70% of a survivor of sudden cardiac death with ischemia-related
ventricular tachycardia.
• Left main vessel disease is more than 50%.
• Triple vessel coronary artery disease of more than 70% with or without proximal LAD involvement.
• Two-vessel disease: LAD and one other major artery.
• One or more stenosis greater than 70% in a patient with significant symptoms of angina despite
maximal medications.
• One vessel disease, more than 70% of a survivor of sudden cardiac death with ischemia-related
ventricular tachycardia.
• Complications: Stroke Wound infection Graft failure Renal failure Postoperative atrial fibrillation Death
• Stroke
• Wound infection
• Graft failure
• Renal failure
• Postoperative atrial fibrillation
• Death
• Stroke
• Wound infection
• Graft failure
• Renal failure
• Postoperative atrial fibrillation
• Death
Option B: Saphenous vein
• Saphenous vein graft can also be used for CABG, but the most preferred one is the left internal
mammary artery.

Page 54

1387
• It is also the best vascular graft for infra-inguinal bypass.
Option C: Reversed saphenous vein
• The reversed saphenous vein is the best natural vascular graft.
Option D: Dacron
• Dacron is the best synthetic vascular graft. It is the best vascular graft for supra-inguinal bypass.
• It is the most preferred vascular graft for the aorta.

Solution for Question 30:


Option D: Long thoracic nerve injury
• These clinical signs are suggestive of the first rib compression, and the only cure for it is resection by
clavicle osteotomy.
• A clavicular osteotomy in complicated cases of revision TSA is an important surgical method that
decreases damage to the anterior deltoid muscle, improves glenoid access, increases the superior
exposure area, provides safer dissection, and allows for superior dislocation of the humeral component.
• Brachial plexus injuries (long thoracic nerve injury), vascular injuries, pleural effusion, winged scapula,
and infection are a complication that may arise secondary to first rib removal.
• Long thoracic nerve injury results in observable scapular winging at rest (usually inferior border only),
with overhead movements, or resisted wall push-ups/push-ups whereby the entire scapula
demonstrates winging.
• Brachial plexus injuries
• Winged scapula due to long thoracic nerve injury
• Horner's syndrome
• Vascular injuries (subclavian vessels)
• Pleural effusion
• Pneumothorax
• Air embolism
• Infection
Option A: Hydropneumothorax
• Iatrogenic hydrothorax is very much rare.
• Although pneumothorax in cervical osteotomy is common.
Option B: Ansa cervicalis injury
• Surgery for the parotid gland, excision of the deep cervical lymph nodes, thyroplasty, arytenoid
adduction, teflon injection, nerve-muscle pedicle implantation, and others may result in iatrogenic
damage to the ansa cervicalis.
Option C: Lymphocutaneous fistula
• After surgical intervention in lymphatic-rich locations, lymphocutaneous fistulas are a rare event that
might develop.

Page 55

1388
• A spontaneous lymphatic leak may happen when lymphedema is present or close to venous stasis
ulcers.

Page 56

1389
Previous Year Questions
1. What is the most common manifestation resulting from the rupture of a saccular aneurysm?
A. Subdural hemorrhage
B. Subarachnoid hemorrhage
C. Intracerebral hemorrhage
D. Hydrocephalus
----------------------------------------
2. What investigation was performed for a patient who presents with sudden onset pain in the right leg,
with the obtained image displayed below?

A. A digital subtraction angiography


B. Ultrasound doppler
C. MR angiography
D. Plethysmography
----------------------------------------
3. What is the next course of action for a patient who has a tender pulsatile mass, as depicted in the
provided image?

A. Ultrasound doppler
B. Needle aspiration
C. CT angiogram with percutaneous management

1390
D. Intravenous antibiotics for 7 days
----------------------------------------
4. A male patient presents with impotence and bilateral resting pain. The following finding is seen in the
legs. At which level has the pathology occurred?

A. Bilateral popliteal arteries


B. Bilateral internal iliac arteries
C. Aortoiliac bifurcation
D. Bilateral femoral arteries
----------------------------------------
5. Which of the options listed below offers persistent outcomes in a coronary artery bypass graft
(CABG)?
A. Long saphenous vein
B. Internal mammary artery
C. Poly tetra fluoro eythlene (PTFE)
D. Gastroepiploic artery
----------------------------------------
6. During the process of washing clothes with cold water, a female patient experiences alterations in
the color of her hands, transitioning from white to red. Which of the following medications would be
most appropriate for this patient?
A. ACE inhibitors
B. Calcium channel blockers
C. Thiazides
D. Alpha-blockers
----------------------------------------
7. An individual who has a long history of alcoholism arrives at the emergency department complaining
of sudden, severe abdominal pain in the upper middle region that spreads to the back. They also report
feeling nauseous, having a decreased appetite, and experiencing occasional episodes of vomiting.
Upon conducting investigations, it was observed that their amylase levels and total count were
elevated. The probable diagnosis in this case is:
A. a.Acute hepatitis

Page 2

1391
B. b.Acute pancreatitis
C. c.Acute Cholecystitis
D. d.Acute appendicitis
----------------------------------------
8. Which of the following is not a type of superior mediastinal tumor?
A. Parathyroid
B. Lymphoma
C. Thyroid
D. Thymus
----------------------------------------
9. Among the listed procedures, which procedure poses the greatest risk of duodenal ulcer recurrence?
A. Gastrectomy
B. Highly selective vagotomy
C. Truncal vagotomy
D. Gastro-jejunostomy
----------------------------------------
10. Which structures being involved result in the clinical presentation depicted in the image?

A. Superficial lymphatics
B. Deep lymphatics
C. Superficial veins
D. Skin
----------------------------------------
11. What is the probable diagnosis for a 17-year-old female, who does not have any other medical
conditions, presenting with finger numbness and tingling sensations? She experiences these symptoms
when she is stressed or exposed to cold temperatures, with no history of smoking or other illnesses.

Page 3

1392
A. Raynaud's phenomenon
B. Buerger's disease
C. Atherosclerosis
D. Peripheral arterial disease
----------------------------------------
12. A 70-year-old female patient presents with pain in the thigh during walking. Investigation reveals the
following finding as given below. What will be the best surgical management?
(or)
What would be the most effective surgical treatment for a 70-year-old female patient experiencing thigh
pain while walking, based on the finding of Percutaneous Transluminal Angioplasty during
investigation?

A. PTA with stenting


B. Femoroiliac bypass
C. Aortofemoral bypass
D. Aorto-iliac bypass
----------------------------------------
13. In accordance with the CEAP classification, what stage of would this patient be assigned as patient
present with same problem again and again?

Page 4

1393
A. C2
B. C1
C. C4a
D. C5
----------------------------------------
14. A 45-year-old chronic smoker presents with pain in his right leg. Upon examination, the pulse in the
bilateral popliteal fossa is absent, and he experiences pain in the both buttock . Which level block is
most likely present?
A. Femoral artery
B. Aortoiliac
C. Popliteal artery
D. Iliac artery
----------------------------------------
15. What is the probable diagnosis for a 50-year-old male, who is a long-term smoker, and experiences
pain in both thighs and buttocks while walking approximately 500 meters?
A. Arterial disease involving the superficial femoral artery
B. Arterial disease with aortoiliac involvement
C. Femoral venous insufficiency
D. Arterial disease involving the profunda femoris artery
----------------------------------------
16. Which of the following is not a component of Thoracoscore?
(or)
Which of the following is not included as a component of Thoracoscore?
A. Performance status
B. Complication of surgery
C. ASA grading
D. Priority of surgery
----------------------------------------

Page 5

1394
17. According to the TNM staging, a patient with lung cancer of size 2.5 cm is staged as
(or)
In accordance with the TNM staging system, a patient diagnosed with lung cancer and a tumor
measuring 2.5 cm in size is categorized as
A. T1b
B. T1c
C. T2a
D. T1a
----------------------------------------
18. Which of the following statements regarding intermittent claudication is accurate?
A. Felt at rest
B. Most commonly felt in the calf muscles
C. Day to day distance is variable
D. Relived after getting out of bed
----------------------------------------
19. What is the management approach for a 50-year-old male patient who has a 40 mm abdominal
aortic aneurysm detected during his routine annual checkup?
A. Urgent endovascular aneurysm repair
B. Surgery is indicated if aneurysmal diameter is > 55mm
C. Urgently wheel the patient to OT for surgery
D. Surgery is indicated only if aneurysm is > 77mm
----------------------------------------
20. What is the most probable diagnosis for a 32-year-old male who is a chronic smoker and has been
experiencing finger pain for several months? Upon examination, gangrenous ulcers were observed on
the fingers. Blood tests revealed normal levels of blood glucose and lipid profile.

A. Buerger’s disease
B. Diabetic ulcers
C. Acute paronychia
D. Raynaud’s phenomenon

Page 6

1395
----------------------------------------
21. What is the probable medical condition for an 80-year-old male who experienced pain and swelling
in the calf muscle on the right side after being immobilized due to a fracture in the right
inter-trochanteric region?
A. Chronic osteomyelitis
B. Deep vein thrombosis
C. Compartment syndrome
D. Arterial claudication
----------------------------------------
22. What are the indications for surgical intervention in cases of abdominal aortic aneurysms?
A. Size more than 5.5 cm in males
B. Size more than 5 cm in females
C. Expansion of >0.5 cm within 6 months
D. All the above
----------------------------------------
23. A lady comes with a reddish firm nodule on her chest which hasn’t grown in size. Its
histopathological image is shown below. What is the diagnosis?

A. Lipoma
B. Fibroadenoma
C. Nevus
D. Hemangioma
----------------------------------------
24. What is the probable diagnosis for the lesion depicted in the image below?

Page 7

1396
A. Arterial insufficiency
B. Neuropathic ulcer
C. Venous ulcer
D. Tuberculous ulcer
----------------------------------------
25. In a 35-year-old individual with a history of chronic smoking, who is experiencing bluish
discoloration of the toes and intermittent claudication, what would be the most suitable investigation to
conduct next?
A. CT Angiography
B. MR Angiography
C. Digital subtraction angiography
D. Duplex scan
----------------------------------------
26. According to the CEAP classification system, the bus conductor's diagnosis would fall under
varicose veins, with accompanying skin eczema and lipodermatosclerosis.
A. C1
B. C2
C. C3
D. C4
----------------------------------------
27. What is the recommended approach to treating this patient who was involved in a road traffic
accident and is experiencing shortness of breath? The patient, a car driver, was brought to the Accident
and Emergency department. Upon examination, the patient has a heart rate of 112 beats per minute,
blood pressure of 100/70 mmHg, oxygen saturation level of 90%, and has significant bruising on the
chest.

Page 8

1397
A. Chest tube drainage
B. 24% oxygen
C. IPPV
D. Internal operative fixation of the fracture’s segments
----------------------------------------
28. What is the likely diagnosis when a patient who had surgery and was immobilized complains of leg
pain on the 5th day postoperatively, with a positive Homan's sign on examination?
A. Deep Venous thrombosis
B. Urinary sepsis
C. Peripheral vascular disease
D. Lymphangitis
----------------------------------------
29. Seat Belt injury leads to?
A. Splenic laceration
B. Splenic contusion
C. Mesenteric tear
D. Mesenteric adenitis
----------------------------------------
30. What is the complication observed following the Varicose vein stripping procedure?
A. Neuralgia
B. Deep vein thrombosis
C. Acrocyanosis
D. Telangiectasia
----------------------------------------
31. What is the appropriate course of action for managing a 25-year-old male patient who was brought
to the emergency room following a road traffic accident? The patient has a respiratory rate of 25
breaths per minute and blood pressure of 80/45 mmHg. During examination, hyper-resonance is
detected on the left side of the chest. Please refer to the provided chest X-ray for further evaluation.

Page 9

1398
A. Wide bore needle insertion in the 5th intercostal space (ICS)
B. Wide bore needle insertion in the 5th ICS followed by an intercostal drain in the 5th ICS
C. Aspiration of the pleural fluid
D. No intervention needed
----------------------------------------
32. What is the most frequently encountered tumor in the mediastinum?
A. Thymoma
B. Neurogenic
C. Lymphoma
D. Bronchial cyst
----------------------------------------
33. What is the likely reason behind the patient's tachypnea and hypotension on the fifth day after
coronary artery bypass graft surgery?
A. Sepsis
B. Acute kidney failure
C. Acute respiratory failure
D. Acute cardiac failure
----------------------------------------
34. Which artery is most frequently implicated in the formation of pseudoaneurysms?
A. Aorta
B. Femoral artery
C. Popliteal artery
D. None of the above
----------------------------------------
35. In cases of trauma, the leading cause of shock is most likely to be:
A. Injury to intra-abdominal solid organ
B. Head injury

Page 10

1399
C. Septicemia
D. Cardiac failure
----------------------------------------
36. What type of ulcer is most likely to be present in a 42-year-old barber who has a healed ulcer on the
lower medial aspect of his right leg and varicose veins on both legs?

A. Neuropathic ulcer
B. Arterial ulcer
C. Venous ulcer
D. Tubercular ulcer
----------------------------------------
37. What are the distinguishing features between cardiac tamponade and tension pneumothorax?
A. Raised JVP
B. Breath sounds
C. Raised heart rate
D. Not improving after initial fluid resuscitation
----------------------------------------
38. What is the most frequently observed complication in a 29-year-old man who visits the outpatient
department due to pain, heaviness, and swelling in his right leg? The patient has been dealing with
varicose veins since he was 12 years old. After undergoing below-knee stripping of the long saphenous
vein, both the symptoms and visible varicose veins disappeared.
A. Hemorrhage
B. Thromboembolism
C. Neuralgia
D. Infection
----------------------------------------
39. What is the quantity of blood loss in class III circulatory failure or haemorrhagic shock?
A. < 15%
B. 15-30%
C. 30-40%

Page 11

1400
D. >40%
----------------------------------------
40. Identify the lesion shown below?

A. Wet gangrene
B. Dry gengrene
C. Frost bite
D. Ainhum
----------------------------------------
41. The given below device is :

A. Pneumatic compression stocking to prevent the DVT


B. Varicose vein
C. Hypothemia
D. Cellulites
----------------------------------------
42. What graft is the least frequently utilized for coronary artery bypass graft (CABG) in cardiac
surgery?
A. Long saphenous vein
B. Internal mammary artery
C. Intercostal artery
D. Radial artery

Page 12

1401
----------------------------------------
43. What is the most probable diagnosis for a patient who experiences a sudden onset of chest pain
shooting to the neck and interscapular region, with a widened mediastinum on X-ray, and blood
pressure measuring 110/90 mmHg in the right upper limb and 160/100 mmHg in the left upper limb?
A. Acute coronary syndrome
B. Acute pulmonary embolism
C. Acute aortic dissection
D. Esophageal rupture
----------------------------------------
44. What is the most probable cause for the condition of white fingers in a construction worker who has
been involved in the cement and concrete industry for 20 years, operating heavy machinery drills, and
engaged in wood and furniture polishing?
A. Candidal infection of the fingers due to continuous exposure to water
B. Exposure to thinners and paints
C. Continuous exposure to cement and concrete
D. Continuous exposure to drills and machines
----------------------------------------
45. A patient with varicose veins complains of eczema in the same region. According to the CEAP
guidelines, what is the stage of the clinical classification?
A. C2
B. C3
C. C4a
D. C4b
----------------------------------------
46. By evaluating the respiratory component of the patient, which injury poses the greatest risk to their
life?
A. Cervical spine injury
B. Tension pneumothorax
C. Blunt cardiac injury
D. Laryngotracheal injury
----------------------------------------
47. Which of the following statements are true regarding central venous access and peripheral venous
access? Phlebitis is the most common complication in peripheral venous access. Isotonic saline and
heparin are flushed to retain patency If an arteriovenous fistula is present in a limb, avoid peripheral
venous access in the same limb. Vasopressors for septic shock are given in the central line to avoid
limb ischemia
A. 1, 3, 4
B. 2, 3, 4
C. 1, 2, 3, 4

Page 13

1402
D. 1, 2, 3
----------------------------------------
48. In a patient with grade 3 splenic trauma, who is being treated conservatively in the intensive care
unit (ICU), which of the following indicators on the second day would suggest the necessity of surgical
intervention?
A. Hemoglobin fall from 12mg/dL to 10mg/dL
B. Extraperitoneal bladder rupture
C. Pneumoperitoneum
D. Air in gallbladder
----------------------------------------
49. What is the most appropriate next course of action in managing a patient who has arrived with
multiple stab wounds to the chest, exhibiting reduced chest movement on the right side, tracheal
deviation to the left side, and distended neck veins?
A. Resuscitative thoracotomy
B. ICD placement
C. Needle thoracocentesis
D. Finger thoracotomy
----------------------------------------
50. A patient presents with dull aching pain and tortuous veins in both lower limbs. Which test would be
appropriate for evaluating superficial femoral vein (SFJ) competence?
A. Fegan's test
B. Homan’s test
C. Modified Perthes test
D. Brodie Trendelenburg test
----------------------------------------
51. A 62-year-old patient presents with pain in the calf muscles while walking. The pain subsides with
rest. Which of the following is not typically seen in intermittent claudication?
A. Cramping in the muscles during exercise
B. Pain starting on taking the first step
C. Most common cause of atheroma
D. Level of occlusion cannot be decided based on symptoms
----------------------------------------
52. What immediate steps would you take to manage a patient who has developed subcutaneous
emphysema following a road traffic accident, with absence of air entry on the right side during
auscultation and unstable vital signs?
A. Insertion of wide bore needle in the 5th intercostal space (ICS)
B. Wide bore needle decompression and IV fluids
C. Intubation and positive pressure ventilation

Page 14

1403
D. eFAST
----------------------------------------
53. A 28-year-old pregnant woman presents with a history of decreased fetal movements and
respiratory distress in her newborn. On examination, the neonate appears cyanotic with increased work
of breathing. Auscultation reveals bowel sounds in the left hemithorax. A chest x-ray is given in the
image. Which of the following congenital anomalies is most likely responsible for these findings?

A. Congenital diaphragmatic hernia


B. Tetralogy of Fallot
C. Transposition of the great vessels
D. Tracheoesophageal fistula
----------------------------------------

Correct Answers
Question Correct Answer

Question 1 2
Question 2 2
Question 3 1
Question 4 3
Question 5 2
Question 6 2
Question 7 2
Question 8 2
Question 9 2
Question 10 1
Question 11 1
Question 12 3
Question 13 1
Question 14 2

Page 15

1404
Question 15 2
Question 16 2
Question 17 2
Question 18 2
Question 19 2
Question 20 1
Question 21 2
Question 22 4
Question 23 4
Question 24 2
Question 25 4
Question 26 4
Question 27 1
Question 28 1
Question 29 3
Question 30 1
Question 31 2
Question 32 2
Question 33 3
Question 34 2
Question 35 1
Question 36 3
Question 37 2
Question 38 3
Question 39 3
Question 40 1
Question 41 1
Question 42 3
Question 43 3
Question 44 4
Question 45 3
Question 46 2
Question 47 3
Question 48 3
Question 49 3

Page 16

1405
Question 50 4
Question 51 2
Question 52 1
Question 53 1

Solution for Question 1:


Correct option B
• Subarachnoid hemorrhage is the most common type associated with the rupture of a saccular
aneurysm. It is caused by bleeding into the space between the arachnoid membrane and the pia mater,
two of the layers of tissue covering the brain. Symptoms of a subarachnoid hemorrhage include sudden
severe headache, nausea, vomiting, and altered mental status.
Incorrect options:
Option A. Subdural hemorrhage is caused by bleeding between the dura mater (the outermost layer of
the meninges) and the brain. It is typically associated with trauma rather than aneurysm rupture.
Option C. Intracerebral hemorrhage is bleeding that occurs directly into the brain tissue. It is typically c
aused by high blood pressure or other underlying conditions rather than aneurysm rupture.
Option D. Hydrocephalus is a condition in which cerebrospinal fluid accumulates within the brain, which
can increase pressure on the brain and cause damage. While various factors can cause hydrocephalu
s, it is not typically associated with aneurysm rupture.

Solution for Question 2:


Correct option B:
• Ultrasound Doppler is a non-invasive diagnostic test that uses high-frequency sound waves to
measure the amount of blood flow through the arteries and veins of the legs. It is commonly used to
diagnose deep vein thrombosis (DVT), where a blood clot forms in a deep vein, most often in the legs.
Incorrect options:
Option A. Digital subtraction angiography: This invasive imaging test involves injecting a
contrast dye into the blood vessels to visualize the blood flow. It is not typically used as a
first-line investigation for DVT.
Option C. MR angiography: This non-invasive imaging test uses magnetic fields and radio waves to cre
ate detailed images of blood vessels. While it can diagnose DVT, ultrasound Doppler is usually preferr
ed due to its lower cost and wider availability.
Option D. Plethysmography: This is a non-invasive test that measures changes in blood volume in the l
imbs to assess blood flow. It is not typically used as a first-line investigation for DVT.

Solution for Question 3:


Correct option A:

Page 17

1406
• The patient's presentation with a tender pulsatile mass suggests the possibility of an arterial
aneurysm, a localized dilation of an artery that can result from various causes such as atherosclerosis,
trauma, or infection. The most appropriate initial diagnostic test for an arterial aneurysm is an
ultrasound Doppler, which can accurately determine the size, location, and extent and assess the blood
flow within the aneurysm.
Incorrect options:
Option B. Needle aspiration: This may be considered in certain cases, such as when there is suspicion
of an abscess or fluid collection. However, it is not the initial step in management for a pulsatile mass.
Option C. CT angiogram with percutaneous management: This may be considered if there is suspicion
of an aneurysm or other vascular abnormality. However, it is not the initial step in management for a
pulsatile mass.
Option D. Intravenous antibiotics for 7 days: This may be considered if there is suspicion of an infection
, but it is not the initial step in management for a pulsatile mass.

Solution for Question 4:


Correct option C
• Impotence and Gluteal claudication are signs of Leriche syndrome.
• Which occurs due to Aortoiliac bifurcation
• Remember the pain of claudication is always one level below the level of block.
• So here pain will be at gluteal region
Incorrect options:
Option A. Bilateral popliteal arteries would not be the correct answer as the popliteal arteries are locate
d in the posterior aspect of the knee and supply blood to the lower leg and foot. Symptoms related to at
herosclerotic disease in this area typically include pain in the calf or foot during exercise (i.e., claudicati
on) and potential skin changes or ulceration in the lower leg.
Option B. Bilateral internal iliac arteries are a possible answer, as an atherosclerotic disease in these a
rteries can also result in reduced blood flow to the pelvic region. However, the aortoiliac bifurcation wo
uld be a more likely location for the pathology to occur, given the presence of bilateral symptoms and t
he fact that the internal iliac arteries arise from the common iliac arteries at this location.
Option D. Bilateral femoral arteries is unlikely as an atherosclerotic disease in these arteries typically re
sults in symptoms such as pain in the thigh or buttock during exercise (i.e., claudication) and potential
skin changes or ulceration in the lower leg. Symptoms related to reduced blood flow to the pelvic regio
n would not typically be associated with disease in the femoral arteries.

Solution for Question 5:


• The internal mammary artery is the vessel that provides long-lasting results in a coronary artery
bypass graft (CABG).
• Coronary artery bypass grafting is a surgical procedure that is used to improve blood flow to the heart
muscle by bypassing blocked or narrowed coronary arteries. During the procedure, a healthy blood

Page 18

1407
vessel from another part of the body is grafted onto the blocked or narrowed artery, creating a new
route for blood to flow to the heart.
• The internal mammary artery is located inside the chest, and it supplies blood to the chest wall and
the heart muscle. It is an ideal vessel for CABG because it is resistant to the development of
atherosclerosis (a buildup of plaque inside the artery), which can lead to re-narrowing or blockage of
the grafted vessel over time.
Incorrect Choices:
• Option a: The long saphenous vein is often used in CABG procedures, may develop atherosclerosis
and become blocked over time, leading to the need for repeat surgeries.
• Option c: PTFE is a synthetic material that can be used in CABG, but it may also develop
atherosclerosis over time.
• Option d: The gastroepiploic artery is another option, but it is not as commonly used as the internal
mammary artery.

Solution for Question 6:


• The symptoms described in the question suggest that the patient is experiencing Raynaud's
phenomenon.
• In this condition, the blood vessels in the fingers and toes spasm and temporarily restrict blood flow.
• This can cause the affected areas to turn white, blue, and red.
• Cold temperatures or emotional stress can trigger the spasms.
• The best-suited drug for this patient would be a calcium channel blocker.
• These drugs work by relaxing the smooth muscle in the blood vessel walls, which can help to prevent
or reduce the frequency of vasospasms.
• Calcium channel blockers are often used as first-line therapy for Raynaud's phenomenon.
Incorrect Choices:
• Option a. ACE inhibitors are a type of blood pressure medication that works by blocking the
production of angiotensin II, a hormone that can constrict blood vessels. However, they are ineffective
in treating vasospastic disorders like Raynaud's phenomenon.
• Option c. Thiazides are a type of diuretic used to treat high blood pressure and edema, but they do not
affect the blood vessels in a way that would be useful for treating Raynaud's phenomenon.
• Optiond. Alpha-blockers are another type of blood pressure medication that blocks the alpha
receptors on blood vessel walls, causing them to relax and dilate. However, alpha-blockers are
ineffective in treating Raynaud's phenomenon because they do not target the underlying vasospasms.

Solution for Question 7:


• Based on the symptoms and findings described, the likely diagnosis in this case is b. Acute
pancreatitis.

Page 19

1408
• Acute pancreatitis is characterized by inflammation of the pancreas, which can be caused by various
factors, including excessive alcohol consumption.
• The epigastric pain radiating to the back, along with nausea, anorexia, and vomiting, are typical
symptoms of acute pancreatitis.
• Elevated levels of amylase, are commonly seen in this condition.
• Elevated total white blood cell count is also a common finding in acute pancreatitis due to the
inflammatory response.
Incorrect Choices: Option a. Acute hepatitis: refers to inflammation of the liver, usually caused by a vira
l infection (such as hepatitis A, B, or C) or other factors like medications, toxins, or autoimmune disorde
rs. The key symptoms of acute hepatitis are jaundice (yellowing of the skin and eyes), fatigue, and elev
ated liver enzymes. Option c. Acute Cholecystitis: refers to inflammation of the gallbladder, often caus
ed by gallstones obstructing the cystic duct. The classic symptoms include right upper quadrant abdom
inal pain, tenderness, fever, and sometimes jaundice. While nausea, anorexia, and vomiting can also b
e present, the pain is usually localized to the right upper abdomen and does not radiate to the back as
described in the scenario. Option d. Acute appendicitis: is the inflammation of the appendix, a small po
uch-like structure located in the lower right abdomen. The typical symptoms of acute appendicitis inclu
de right lower quadrant abdominal pain, fever, rebound tenderness, and a possible shift of pain from th
e umbilical region to the right lower quadrant. The pain is usually not in the epigastric region and does
not radiate to the back.

Solution for Question 8:


• The correct answer is b. Lymphoma.
• Lymphoma is not considered a superior mediastinal tumor.
• Lymphomas are cancers that originate in the lymphatic system, which includes lymph nodes,
lymphatic vessels, and lymphoid tissues.
• While lymphomas can occur in various parts of the body, including the mediastinum, they are not
exclusive to the superior mediastinum.
Incorrect choices: Option a. Parathyroid tumors can develop in the parathyroid glands, which are locate
d in the neck, behind the thyroid gland. However, they are not typically classified as superior mediastin
al tumors. Option c. Thyroid tumors, specifically those that have grown and extended into the mediastin
um, can be considered superior mediastinal tumors. The primary location of the thyroid gland is in the
neck, but it can occasionally extend into the superior mediastinum. Option d. Thymomas are tumors th
at arise from the thymus gland, which is situated in the superior mediastinum. Therefore, thymomas ar
e indeed superior mediastinal tumors.

Solution for Question 9:


Correct Answer Option B:
• Highly selective vagotomy procedure carries a higher risk of causing the recurrence of duodenal
ulcers compared to the other options listed.
• Highly selective vagotomy involves the selective division of the branches of the vagus nerve that
innervate the stomach, aiming to reduce acid secretion while preserving the innervation to the rest of

Page 20

1409
the gastrointestinal tract. However, there is still a possibility of ulcers recurring after this procedure.
Incorrect Choices: Option a) Gastrectomy:. Gastrectomy is usually reserved for cases of stomach canc
er or other severe gastric conditions. Since it removes the stomach entirely or a significant portion of it,
the risk of duodenal ulcer recurrence is low because there is no stomach left to produce acid. Option c)
Truncal vagotomy: Truncal vagotomy involves the division or resection of the main trunk of the vagus
nerve, which supplies the stomach. This procedure reduces gastric acid secretion by disrupting the ner
ve signals that stimulate acid production. Although it carries some risk of ulcer recurrence, the risk is lo
wer compared to highly selective vagotomy (choice B).
Option d) Gastro-jejunostomy: Gastro-jejunostomy is a surgical procedure in which the connection is m
ade between the stomach and the jejunum. This procedure is typically performed to bypass an obstruct
ion in the stomach or to improve the passage of food in cases such as gastric outlet obstruction. While
it can alleviate symptoms of duodenal ulcers, it is not primarily intended to treat or prevent their recurre
nce. Gastro-jejunostomy does not directly address the overproduction of gastric acid, which is one of th
e main factors contributing to duodenal ulcers.

Solution for Question 10:


Lymphangitis is the inflammation of the lymphatic vessels, specifically the superficial lymphatics. It is ty
pically caused by an infection that enters the body through a break in the skin, such as a wound or an i
nsect bite. The infection can then spread to the lymphatic vessels, causing them to become inflamed a
nd visible as red streaks on the skin. Superficial lymphatics are a
network of lymphatic vessels that are close to the surface of the skin. They play a crucial role in carryin
g lymph fluid and immune cells from the tissues towards the lymph nodes for filtration and processing.
When an infection occurs in the surrounding tissues, the microorganisms can enter the lymphatic vess
els through the disrupted skin, leading to lymphangitis.
Incorrect options explanations:
Option b. Deep lymphatics: Deep lymphatics are located deeper within the body and are responsible fo
r draining lymph fluid from the organs and deeper tissues. While deep lymphatics play a crucial role in t
he lymphatic system, they are not directly involved in lymphangitis. Lymphangitis primarily affects the s
uperficial lymphatics, which are the lymphatic vessels closer to the skin's surface. Option c. Superficial
veins: Superficial veins are responsible for carrying deoxygenated blood back to the heart and are not
directly involved in the lymphatic system. They do not play a
role in lymphangitis, which is an inflammation of the lymphatic vessels rather than the veins. Option d.
Skin: While the skin can be affected by lymphangitis, it is not the structure directly responsible for causi
ng lymphangitis. Lymphangitis occurs when an infection enters the body through a break in the skin an
d then spreads to the superficial lymphatic vessels, leading to their inflammation. So, while the skin can
be a site of entry for the infection, it is the involvement of the superficial lymphatics that leads to lymph
angitis.

Solution for Question 11:


Raynaud's phenomenon is a condition characterized by episodic vasospasm of the small arteries and a
rterioles, typically affecting the fingers and toes.
It is commonly triggered by cold temperatures or emotional stress. The affected areas may turn white (
pallor) due to decreased blood flow, followed by blue discoloration (cyanosis) due to prolonged lack of
oxygen, and finally, redness (rubor) as blood flow is restored. Raynaud's phenomenon is more commo

Page 21

1410
n in women and often begins in adolescence or early adulthood. It is typically a benign condition and n
ot associated with underlying arterial disease. However, in some cases, it may be secondary to other c
onditions such as autoimmune disorders or connective tissue diseases.
Incorrect choices: Option b. Buerger's disease (thromboangiitis obliterans): This condition is primarily a
ssociated with heavy tobacco use, particularly smoking. It is characterized by inflammation and clotting
of small and medium-sized arteries and veins, leading to the blockage of blood flow. While Buerger's d
isease can cause symptoms similar to Raynaud's phenomenon, such as numbness and pain in the extr
emities, it is more commonly seen in individuals who have a history of smoking. Since the patient in qu
estion has no history of smoking, Buerger's disease is less likely.
Option c. Atherosclerosis: Atherosclerosis is a condition characterized by the buildup of fatty deposits (
plaques) within the arteries, leading to narrowing and hardening of the blood vessels. It is a progressiv
e disease that typically develops over time and is associated with risk factors such as older age, high bl
ood pressure, high cholesterol, and smoking. While atherosclerosis can cause symptoms similar to Ra
ynaud's phenomenon, such as numbness and pain in the extremities, it is less common in young indivi
duals without other comorbidities. Therefore, atherosclerosis is less likely in this case. Option d. Periph
eral arterial disease (PAD): PAD refers to the narrowing or blockage of arteries supplying blood to the
extremities, usually caused by atherosclerosis. Similar to atherosclerosis, PAD is more commonly seen
in older individuals with risk factors such as smoking, diabetes, high blood pressure, and high choleste
rol. While PAD can cause symptoms like numbness and pain in the extremities, it is less likely in a
17-year-old woman with no comorbidities or history of smoking.

Solution for Question 12:


Correct Option C:
• The given condition is of Aortofemoral block and best surgical management is Aortofemoral bypass .
Incorrect Options:
Option A. PTA with stenting : Also known as percutaneous transluminal angioplasty with stenting, is a
minimally invasive procedure used to treat narrowed or blocked blood vessels, particularly in the arterie
s. It is commonly performed to restore blood flow in peripheral arteries affected by atherosclerosis or ot
her conditions.
Option B. Femoroiliac bypass Femoroiliac bypass, also known as femoral-to-iliac bypass or femoral by
pass, is a surgical procedure used to treat peripheral artery disease (PAD) in the lower extremities. It in
volves creating a bypass graft to reroute blood flow around a
narrowed or blocked segment of the femoral or iliac artery.
Option D. Aorto-iliac bypass - An aorto-iliac bypass, also known as an aortobifemoral bypass, is a surgi
cal procedure used to treat severe peripheral artery disease (PAD) affecting the aorto-iliac arteries. It in
volves creating a bypass graft to restore blood flow from the aorta to the iliac arteries, bypassing the di
seased or blocked segments.

Solution for Question 13:


Correct Option A:
• As recurrent varicose veins are visible it is C2
Incorrect Options:

Page 22

1411
Option B - C1 is incorrect
Option C - C4a is incorrect as there is no pigmentation and eczema
Option D - C5 is incorrect as there is visible varicose vein

Solution for Question 14:


Correct option:
Option B: Aortoiliac
• The most likely level block in this case is an aortoiliac block.
• The most likely level block causing the absent pulse in the bilateral popliteal fossa and associated
symptoms is an aortoiliac blockage, affecting the blood flow to the lower extremity.
Incorrect options:
Option A: Femoral block: The femoral artery is located in the upper thigh region and does not explain b
uttock pain. Therefore, it is an incorrect option.
Option C: Popliteal block: This option is incorrect.
Option D: Iliac block: The iliac arteries are located in the lower abdomen and do not lead to pain in the
buttock. Therefore, it is an incorrect option.

Solution for Question 15:


Correct Option B.
Arterial disease with aortoiliac involvement: This option is correct.
• The muscle group affected in claudication is one anatomical level below the level of arterial disease.
• The patient's symptoms of intermittent claudication pain in both the thigh and buttock region on
walking are indicative of arterial disease with aortoiliac involvement.
• The aortoiliac arteries, including the abdominal aorta and iliac arteries, supply blood to the lower
extremities.
• Narrowing or blockage of these arteries due to atherosclerosis, often associated with chronic
smoking, can lead to reduced blood flow and cause symptoms of intermittent claudication.
Incorrect options
Option A. Arterial disease involving the superficial femoral artery: This option is incorrect. While arterial
disease involving the superficial femoral artery can cause claudication pain in the calf, it typically does
not cause buttock pain.
Option C. Femoral venous insufficiency: This option is incorrect. Femoral venous insufficiency refers to
the impaired function of the veins in the femoral region, leading to symptoms such as leg swelling, pai
n, and varicose veins. It is not the likely diagnosis in this case, as the symptoms described (intermittent
claudication) are characteristic of arterial disease rather than venous insufficiency.

Page 23

1412
Option D. Arterial disease involving the profunda femoris artery: This option is incorrect. While arterial
disease involving the profunda femoris artery can cause claudication pain in the calf,, it does not typical
ly cause buttock pain.

Solution for Question 16:


Correct Option B:
• Complication of surgery: This statement is incorrect. As it is not a component of thoracoscore
Incorrect option;
Option A. Performance status: Performance status is included in thoracoscore because it helps evaluat
e a patient's overall health and predicts their ability to tolerate surgical procedures.
Option C. ASA grading: It classifies patients into different categories based on their underlying medical
conditions and their potential to impact anesthesia and surgical outcomes. ASA grading is included in t
horacoscore because it provides valuable information about the patient's preoperative health status.
Option D. Priority of surgery: The priority of surgery, also known as surgical urgency or timing, can be a
n important consideration in certain scoring systems. It assesses the urgency of performing a surgical
procedure based on factors such as the severity of the condition, the potential for disease progression,
and the impact on the patient's overall health. The timing of surgery can influence the patient's prognos
is and guide decision-making. Therefore, the priority of surgery is typically included in scoring systems l
ike Thoracoscore.

Solution for Question 17:


Correct Option B:
• T1c: According to the TNM staging system, T1c is used for lung tumors larger than 2cm but not
exceeding 3 cm in greatest dimension.
• As the patient's lung cancer has a size of 2.5 cm, it does meet the criteria for T1c.
Incorrect Option;
Option A. T1b: This option is incorrect. In the TNM staging system for lung cancer, T1b refers to tumor
s larger than 1cm but not exceeding 2 cm in greatest dimension.
Option C. T2a: This option is incorrect. T2a in the TNM staging system is used for lung tumors larger th
an 4 cm but not exceeding 5 cm in greatest dimension. Since the patient's lung cancer has a
size of 2.5 cm, it does not qualify for T2a staging.
Option D. T1a: This option is incorrect. T1a is used for lung tumors measuring 1
cm or smaller in the greatest dimension.

Solution for Question 18:


Correct Option B.

Page 24

1413
• Intermittent claudication is a symptom of peripheral arterial disease and refers to the pain or
discomfort experienced in the muscles of the lower extremities, particularly the calf muscles, during
physical activity.
• The pain is typically described as a cramping, aching, or fatigue-like sensation and is caused by
inadequate blood supply to the muscles due to narrowed or blocked arteries.
• The calf muscles are commonly affected because they have higher oxygen demands during physical
exertion.
Incorrect options:
Option A. Intermittent claudication is not felt at rest. The pain is specifically associated with physical act
ivity or exercise and is relieved with rest.
Option C. Day-to-day distance is relatively consistent in individuals with intermittent claudication. The p
ain is typically reproducible at a similar distance or level of exertion and can serve as a
marker of disease severity.
Option D. Intermittent claudication is not relieved after getting out of bed. The pain is typically triggered
by physical activity and worsens with continued exertion. Resting and ceasing the activity help alleviate
the symptoms.

Solution for Question 19:


Correct Option B.
• Surgery is indicated if the aneurysmal diameter is > 55 mm: This statement is true.
• The management of abdominal artery aneurysm depends on the size of the aneurysm.
• Generally, surgical repair is recommended if the diameter exceeds 55 mm or if it shows rapid growth.
Incorrect options;
Option A. Urgent endovascular aneurysm repair: Urgent endovascular repair is typically reserved for p
atients with symptomatic abdominal artery aneurysm, rapid expansion, or signs of impending rupture. I
n this case, the patient does not exhibit any urgent indications for immediate intervention, such as sym
ptoms or signs of rupture.
Option C. Urgently wheel the patient to the operating theater for surgery: Urgent surgery is not indicate
d for an AAA measuring 40 mm. The decision for surgical intervention is based on the size of the aneur
ysm, growth rate, and presence of symptoms. Close monitoring and regular follow-up are typically reco
mmended in this situation.
Option D. Surgery is indicated only if the aneurysm is >
77 mm: This statement is incorrect. The threshold for surgical intervention in AAA is generally set at a
diameter of 55 mm. Aneurysms larger than 77 mm are considered very large and carry a
higher risk of rupture, but surgical management is typically recommended before reaching that size.

Solution for Question 20:


Correct Option A.

Page 25

1414
• Thromboangiitis obliterans/Buerger’s disease) is a condition that is characterized by occlusion of the
small and medium-sized arteries (plantars, tibialis, radial, etc.), and thrombophlebitis of superficial or
deep veins.
• It also causes Raynaud’s syndrome ( fingers and toes — to feel numb and cold) among male patients
in a young age group (usually under the age of 30 years).
• Gangrene of the toes and fingers is common and progressive.
• The treatment is total abstinence from smoking. While this will arrest the disease it will not reverse
established arterial occlusions.
Incorrect Options
Option B: Diabetic ulcers : Occurs more commonly among patients(both males and females) with diabe
tes. It is seen more commonly in the foot.
Option C: Acute paronychia : This is the most common infection of the hand. It is often caused due to c
areless nail trimming or picking the skin around the nail fold. The pus is trapped beside the nail.
Option D: Raynaud’s phenomenon : This phenomenon causes the blood vessels in the extremities to n
arrow and it restricts the blood flow. This usually affects the fingers and toes

Solution for Question 21:


Correct Option B.
The given scenario in which patient is immobilized and presents with symptoms like claf swelling and p
ain strongly suggests DVT
Deep vein thrombosis/DVT is the formation of a blood clot in a
deep vein. The clots usually develop in the lower leg, thigh, or pelvis.
Symptoms include pain, swelling, and tenderness in the calf or thigh, warm and red skin.
• The components of Virchow's Triad are: (increases the risk of DVT) Endothelial injury Stasis
Hypercoagulable state
• Endothelial injury
• Stasis
• Hypercoagulable state
• Endothelial injury
• Stasis
• Hypercoagulable state
Incorrect Options:
Option A: Chronic osteomyelitis:
• The intraosseous blood vessels thrombose and the bone infarcts. Pus lifts the periosteum. For
treating osteomyelitis, All dead tissue needs to be removed. If there is a fracture, it needs to be
stabilized.
Option C: Compartment syndrome:

Page 26

1415
• A compartment syndrome develops at any time during the first 48 hours after the accident or surgery.
The patient usually complains of severe pain with some numbness in the foot. The foot may be cold still
with pulses.
Option D: Arterial claudication:
• Claudication is usually a symptom of peripheral artery disease, in which the arteries that supply blood,
usually the legs, are narrowed.

Solution for Question 22:


Correct Option D: All the above
• All of the above are indications for surgical intervention in abdominal aortic aneurysms (AAAs).
Option A: Size more than 5.5 cm in males- This criterion is based on the risk of rupture. An AAA larger
than 5.5 cm in diameter has a higher risk of rupture, and surgical intervention is recommended to preve
nt rupture and its potentially life-threatening consequences.
Option B: Size more than 5 cm in females- Although the threshold for surgical intervention is slightly lo
wer in females compared to males, an AAA larger than 5 cm in diameter is still considered an indicatio
n for surgery in females. This is because females generally have a
smaller aorta, and smaller aneurysms in women may carry a higher risk of rupture compared to men.
Option C: Expansion of >0.5 cm within 6 months- Rapid expansion of an AAA indicates an increased ri
sk of rupture. If the aneurysm grows more than 0.5 cm within a
6-month period, surgical intervention is recommended to prevent rupture.

Solution for Question 23:


Correct Option D: Hemangioma
• Hemangioma, is the correct diagnosis based on the histopathological image shown below.
• Hemangiomas are benign tumors composed of blood vessels, and histologically they typically show
proliferation of endothelial cells forming vascular channels or spaces.
Incorrect Options:
Option A: Lipoma- Lipoma is a benign tumor composed of fat cells. Histologically, lipomas typically sho
w mature adipose tissue without any significant architectural or cellular abnormalities. The provided his
topathological image does not exhibit the characteristic features of a lipoma.
Option B: Fibroadenoma- Fibroadenoma is a
common benign tumor that typically occurs in the breast. Histologically, fibroadenomas show a combin
ation of stromal (connective tissue) and epithelial components. The provided histopathological image d
oes not resemble the features of a fibroadenoma.
Option C: Nevus- A nevus, also known as a mole, is a benign proliferation of melanocytes. Histologicall
y, nevi exhibit increased numbers of melanocytes within the skin or other tissues. The provided histopa
thological image does not show the characteristic features of a nevus.

Page 27

1416
Solution for Question 24:
Correct Option B: Neuropathic ulcer
• Neuropathic ulcers are a common type of ulcer that occurs in individuals with peripheral neuropathy,
which is nerve damage often associated with conditions like diabetes. Peripheral neuropathy can lead
to loss of sensation in the affected area, making the person unaware of injuries or excessive pressure.
Over time, this can result in the development of neuropathic ulcers. These ulcers often appear as
non-healing, painless, and shallow with a punched-out appearance. They are typically located over
weight-bearing areas of the feet or toes.
• This option could be the correct diagnosis if the image shows a characteristic neuropathic ulcer with
the aforementioned features.
Incorrect Options :
Option A: Arterial insufficiency
• Arterial insufficiency ulcers occur due to inadequate blood flow to the tissues, usually caused by
peripheral arterial disease (PAD). These ulcers commonly appear on the lower extremities, particularly
the feet or toes. They typically have well-defined, "punched-out" borders and are often accompanied by
other signs of poor arterial circulation, such as decreased or absent pulses, cool skin, and hair loss. If
the image does not show these features, arterial insufficiency ulcer is an unlikely diagnosis.
Option C: Venous ulcer
• Venous ulcers occur due to impaired venous return, usually associated with chronic venous
insufficiency (CVI). These ulcers are often found on the lower legs, near the ankles. They are typically
shallow, irregularly shaped, and may have a red or yellowish wound base. Venous ulcers are
commonly accompanied by edema, varicose veins, and skin changes such as discoloration or stasis
dermatitis. If the image does not display these characteristic features, venous ulcer is an unlikely
diagnosis.
Option D: Tuberculous ulcer
• Tuberculous ulcers are ulcers that develop as a result of cutaneous tuberculosis infection. These
ulcers are rare and occur in the context of tuberculosis infection, which usually affects the lungs but can
spread to the skin in some cases. Tuberculous ulcers typically have an undermined edge, a necrotic
base, and may show other signs of tuberculosis infection, such as scarring or lymphadenopathy. If the
image does not demonstrate these specific features, a tuberculous ulcer is an unlikely diagnosis.

Solution for Question 25:


Correct Option D- Duplex scan:
• Duplex scan, also known as duplex ultrasound, is a non-invasive imaging test that combines
conventional ultrasound with Doppler ultrasound. It is commonly used to evaluate peripheral arterial
disease (PAD) and assess blood flow in the arteries and veins. In this case, the patient's symptoms of
bluish toe discoloration and intermittent claudication are suggestive of reduced blood flow to the lower
extremities, which can be caused by PAD.
• A duplex scan can provide valuable information about the location and severity of arterial blockages or
narrowing, as well as the overall blood flow in the affected area. It can help identify any potential arterial
stenosis or occlusion that may be contributing to the patient's symptoms. Additionally, the duplex scan

Page 28

1417
can provide information about the extent of the disease and help guide further management decisions.
Incorrect Options:
Option A- CT Angiography: CT angiography is a cross-sectional imaging technique that uses X-rays an
d contrast dye to visualize blood vessels. While it can provide detailed images of the blood vessels, it is
not typically the initial investigation of choice for evaluating peripheral arterial disease. CT angiography
may be considered in certain cases where the duplex scan is inconclusive or when a
more detailed assessment is required.
Option B- MR Angiography: Magnetic resonance angiography (MRA) is a non-invasive imaging techniq
ue that uses magnetic fields and contrast dye to visualize blood vessels. Like CT angiography, it can pr
ovide detailed images of the blood vessels. However, similar to CT angiography, MR angiography is no
t usually the initial investigation for evaluating PAD. It may be used in specific cases or when other ima
ging modalities are not feasible or contraindicated.
Option C- Digital subtraction angiography: Digital subtraction angiography (DSA) is an invasive proced
ure that involves the injection of contrast dye into the blood vessels followed by X-ray imaging. It is con
sidered a more invasive and expensive option compared to the non-invasive duplex scan. DSA is typic
ally reserved for cases where other imaging modalities have not provided sufficient information or whe
n endovascular intervention (such as angioplasty) is planned.

Solution for Question 26:


Correct Option D:
• The CEAP classification system is used to classify and grade chronic venous disorders, including
varicose veins. It stands for Clinical, Etiological, Anatomical, and Pathophysiological categories.
In the CEAP classification system:
• C1 represents telangiectasias or spider veins.
• C2 represents varicose veins.
• C3 represents edema (swelling) without skin changes.
• C4 represents skin changes such as eczema, pigmentation, or lipodermatosclerosis.
• C5 represents healed venous ulcer.
• C6 represents active venous ulcer.
In the given scenario, the bus conductor has varicose veins along with skin eczema and lipodermatoscl
erosis. Skin eczema and lipodermatosclerosis are indicative of skin changes, which correspond to the
C4 category in the CEAP classification system. Therefore, the bus conductor would be classified as C4
.
Incorrect Options:
Option A: C1: This category represents telangiectasias or spider veins, which are not present in the giv
en scenario.
Option B: C2: This category represents varicose veins, but it does not include the additional skin chang
es observed in the bus conductor.
Option C: C3: This category represents edema (swelling) without skin changes, which is not consistent
with the skin eczema and lipodermatosclerosis observed in the bus conductor.

Page 29

1418
Solution for Question 27:
Correct Answer: A
The management of this patient would involve:
• Chest tube drainage. Based on the patient's presentation of breathlessness, low oxygen saturation
(SpO2), and extensive bruising on the chest, there is a concern for possible traumatic injury to the
chest, such as a pneumothorax or hemothorax. Chest tube drainage is the appropriate initial
management in this case. Chest tube drainage involves the insertion of a tube into the chest to remove
air or fluid that may be causing lung collapse (pneumothorax) or compromising breathing (hemothorax).
By draining the accumulated air or fluid, the lung can re-expand, and the patient's breathing can
improve.
• Based on the patient's presentation of breathlessness, low oxygen saturation (SpO2), and extensive
bruising on the chest, there is a concern for possible traumatic injury to the chest, such as a
pneumothorax or hemothorax. Chest tube drainage is the appropriate initial management in this case.
• Chest tube drainage involves the insertion of a tube into the chest to remove air or fluid that may be
causing lung collapse (pneumothorax) or compromising breathing (hemothorax). By draining the
accumulated air or fluid, the lung can re-expand, and the patient's breathing can improve.
• Based on the patient's presentation of breathlessness, low oxygen saturation (SpO2), and extensive
bruising on the chest, there is a concern for possible traumatic injury to the chest, such as a
pneumothorax or hemothorax. Chest tube drainage is the appropriate initial management in this case.
• Chest tube drainage involves the insertion of a tube into the chest to remove air or fluid that may be
causing lung collapse (pneumothorax) or compromising breathing (hemothorax). By draining the
accumulated air or fluid, the lung can re-expand, and the patient's breathing can improve.
Incorrect options:
Option B. 24% oxygen may be administered as supportive treatment to help improve oxygenation. How
ever, the primary intervention in this case is addressing the potential underlying chest injury by initiatin
g chest tube drainage.
Option C. IPPV (Intermittent Positive Pressure Ventilation), which refers to mechanical ventilation, may
be necessary if the patient's respiratory distress worsens and requires further respiratory support. How
ever, it is not the initial management step for this patient.
Option D. Internal operative fixation of the fracture's segments is not indicated in the initial managemen
t of a patient presenting with breathlessness and chest bruising. It is more relevant in cases where ther
e are evident fractures that require surgical stabilization.

Solution for Question 28:


Correct Answer : A
• Based on the information provided, the most probable diagnosis for the patient experiencing leg pain
on the 5th postoperative day with a positive Homan's sign is Deep Venous Thrombosis (DVT).
• Homan's sign refers to calf pain elicited by dorsiflexion of the foot, and it is often associated with DVT.
DVT is a condition in which a blood clot forms in a deep vein, commonly in the lower extremities.

Page 30

1419
Immobilization following surgery is a known risk factor for the development of DVT. Other risk factors
include a history of DVT, obesity, smoking, and certain medical conditions.
Incorrect options:
Option B. Urinary sepsis, peripheral vascular disease, and lymphangitis are less likely in this scenario.
Urinary sepsis typically presents with symptoms such as fever, urinary urgency, and dysuria (painful uri
nation), rather than leg pain.
Option C. Peripheral vascular disease refers to conditions that affect blood flow to the extremities and
may cause pain or cramping during physical activity, but it is less likely to be related to the immobilisati
on following surgery.
Option D. Lymphangitis refers to the inflammation of lymphatic vessels, which can cause red streaks o
n the skin and swollen lymph nodes but is less likely to be associated with leg pain in this context.

Solution for Question 29:


Correct Options C.
• Seat belt injuries are commonly seen in motor vehicle accidents and can result in various types of
injuries.
• One of the possible injuries associated with seat belt use is a mesenteric tear.
• The sudden deceleration force applied by the seat belt can cause shearing and tearing of the
mesentery, which is the tissue that attaches the intestines to the abdominal wall.
Incorrect Options:
Options A. Splenic laceration: While seat belt injuries can cause abdominal trauma, including injuries to
the spleen, it is not the most likely injury specifically associated with seat belt use.
Options B. Splenic contusion: Similar to the previous option, while splenic injuries can occur due to sea
t belt injuries, mesenteric tear is a more specific injury associated with seat belt use.
Options D. Mesenteric adenitis: Mesenteric adenitis refers to inflammation of the lymph nodes in the m
esentery and is not directly related to seat belt injuries.

Solution for Question 30:


Correct option A.
• Varicose vein stripping is a surgical procedure used to remove varicose veins, which are enlarged and
twisted veins usually found in the legs.
• While varicose vein stripping is generally safe, complications can occur. One of the complications
associated with this procedure is neuralgia, which refers to nerve pain or nerve damage.
• Neuralgia can manifest as persistent or intermittent pain along the distribution of the affected nerves.
Incorrect Options:

Page 31

1420
Options B. Deep vein thrombosis (DVT): Deep vein thrombosis is a condition characterized by the form
ation of blood clots in the deep veins, typically in the legs. While DVT is a possible complication of varic
ose veins themselves, it is not directly associated with the varicose vein stripping procedure.
Options C. Acrocyanosis: Acrocyanosis is a condition characterized by persistent bluish discoloration o
f the hands and feet, usually due to reduced blood flow to the peripheral vessels. It is not a
direct complication of varicose vein stripping.
Options D. Telangiectasia: Telangiectasia refers to the dilation of small blood vessels near the surface
of the skin, resulting in the appearance of fine red or purple lines. While telangiectasia may be associat
ed with varicose veins, it is not a specific complication of the varicose vein stripping procedure.

Solution for Question 31:


Correct Option B: Wide bore needle insertion in the 5th intercostal space (ICS) followed by an intercost
al drain in the 5th ICS.
• The chest X-ray shows evidence of tension pneumothorax, which is a life-threatening condition that
requires immediate intervention. Tension pneumothorax occurs when air accumulates in the pleural
space and cannot escape, causing increased pressure on the lung and mediastinal structures. This can
lead to compromised breathing and hemodynamic instability.
• The initial step in managing tension pneumothorax is to relieve the pressure and decompress the
pleural space. This is achieved by inserting a wide bore needle (such as a large-bore cannula or
angiocatheter) into the affected side's 5th intercostal space (ICS) in the mid-axillary line. This allows the
trapped air to escape, leading to immediate relief of symptoms and stabilization of the patient.
• However, simply inserting a needle may not be sufficient, especially in cases where there is ongoing
air leak or a large pneumothorax. Therefore, option B, which involves wide bore needle insertion in the
5th ICS followed by the placement of an intercostal drain (chest tube) in the same location, is the
appropriate management. The intercostal drain helps maintain continuous drainage of air and/or fluid
from the pleural space, allowing for lung re-expansion and preventing recurrence.
Incorrect options :
Option A. Wide bore needle insertion in the 5th ICS only.
• While needle decompression is the initial step, it may not be enough to fully manage the tension
pneumothorax. Inserting a wide bore needle alone without following it up with an intercostal drain may
not adequately address the ongoing problem, and the patient may require further intervention.
Option C. Aspiration of the pleural fluid.
• In the case of tension pneumothorax, the primary issue is the accumulation of air in the pleural space,
rather than fluid. Aspiration of pleural fluid alone would not address the underlying problem and relieve
the tension.
Option D. No intervention needed.
• This option is incorrect because the patient's presentation with signs of tension pneumothorax
(hyper-resonance on examination and characteristic chest X-ray findings) requires immediate
intervention to decompress the pleural space and stabilize the patient's condition. Delaying or not
performing an intervention in this scenario can be life-threatening.

Page 32

1421
Solution for Question 32:
Correct Option B. Neurogenic: Most common posterior mediastinal mass and also overall
most common mediastinal mass
Incorrect Options
Option A. Thymoma: Thymoma is a tumor that originates from the thymus gland, which is located in th
e mediastinum. They are most common anterior mediastinal mass
Option C. Lymphoma: Lymphomas are cancers that originate from lymphocytes, a type of white blood
cell. While lymphomas can involve the mediastinum, they are not as common as thymomas.
Option D. Bronchial cyst: Bronchial cysts are abnormal fluid-filled sacs that develop within the bronchial
walls. While they can occur in the mediastinum, they are not the most common mediastinal tumors.

Solution for Question 33:


Correct option:
Option C.
• Acute respiratory failure: This is the correct option. Acute respiratory failure refers to a condition in
which the lungs fail to provide adequate oxygenation to the body or remove carbon dioxide efficiently.
Tachypnea (rapid breathing) and hypotension can be signs of inadequate oxygenation and perfusion.
After CABG, potential causes of acute respiratory failure can include atelectasis (collapsed lung),
pneumonia, pulmonary embolism, or other complications affecting lung function. Prompt evaluation and
intervention are crucial to address respiratory compromise.
Incorrect Option:
Option A. Sepsis: Sepsis is a severe systemic infection that can cause various symptoms, including tac
hypnea and hypotension. However, without additional information, it is less likely to be the most probab
le cause on the 5th postoperative day after CABG. Other signs such as fever, elevated white blood cell
count, or evidence of infection would be important to consider for a definitive diagnosis of sepsis.
Option B. Acute kidney failure: Acute kidney failure, also known as acute renal failure, primarily affects
kidney function and may manifest as decreased urine output, fluid overload, electrolyte disturbances, o
r changes in laboratory parameters such as elevated creatinine levels. While it can cause hypotension,
tachypnea is not a typical symptom of acute kidney failure. Therefore, it is less likely to be the most pr
obable cause in this scenario.
Option D. Acute cardiac failure: Acute cardiac failure, also known as acute heart failure or congestive h
eart failure, typically presents with symptoms such as shortness of breath, fatigue, fluid retention, and s
igns of poor cardiac output. While acute cardiac failure can cause hypotension, tachypnea alone is less
commonly associated with this condition. Other cardiac symptoms, such as edema, jugular venous dis
tention, or abnormal heart sounds, would be important to consider for a
definitive diagnosis of acute cardiac failure.

Page 33

1422
Solution for Question 34:
Correct option:
Option B.
• Femoral artery: This is the correct option. The femoral artery is a common location for
pseudoaneurysm formation. It is a large artery located in the groin area and is frequently accessed for
arterial catheterization or invasive procedures such as angiography, cardiac catheterization, or arterial
line placement. Pseudoaneurysms can occur as a complication of these procedures due to the
puncture and subsequent incomplete closure of the artery, leading to the formation of a pulsating
hematoma.
Incorrect Option:
Option A. Aorta: While the aorta can develop aneurysms, a pseudoaneurysm, also known as a
false aneurysm, is less commonly associated with the aorta. Pseudoaneurysms typically involve a
breach in the arterial wall with the formation of a
pulsating hematoma contained by surrounding tissues. They often occur as a result of trauma or invasi
ve procedures, but they are not as commonly associated with the aorta as other arteries.
Option C. Popliteal artery: The popliteal artery is located behind the knee and is also susceptible to pse
udoaneurysm formation. However, compared to the femoral artery, pseudoaneurysms in the popliteal a
rtery are less common. Popliteal artery pseudoaneurysms can occur due to trauma, iatrogenic injury, o
r underlying vascular diseases affecting the artery.
Option D. None of the above: This option is incorrect as both the femoral artery (Option B) and the popl
iteal artery (Option C) can be involved in pseudoaneurysm formation, with the femoral artery being the
more commonly involved artery.

Solution for Question 35:


Correct Option A:
• Injury to intra-abdominal solid organ: This option is the correct answer. In traumatic cases, shock can
commonly occur due to injury to intra-abdominal solid organs such as the liver, spleen, or kidneys.
These organs are richly vascularized, and damage to them can result in significant internal bleeding,
leading to hypovolemic shock. Hypovolemic shock occurs when there is a rapid loss of blood or fluids,
causing a decrease in circulating blood volume and inadequate tissue perfusion.
Incorrect Options:
Option B: Head injury: While head injuries can be severe and life-threatening, they are not typically a di
rect cause of shock. Head injuries can result in intracranial bleeding or swelling, which may require im
mediate medical attention. However, shock in traumatic cases is more commonly associated with signif
icant blood loss or fluid loss rather than isolated head injuries.
Option C: Septicemia: Septicemia, also known as sepsis, is a
severe and potentially life-threatening condition characterized by a systemic inflammatory response to
an infection. While septicemia can cause shock, it is not typically associated with traumatic cases. Trau
matic shock is primarily caused by hemorrhagic shock resulting from acute blood loss rather than sepsi
s-related causes.
Option D: Cardiac failure: Cardiac failure, which refers to the inability of the heart to pump blood effecti
vely, can lead to shock. However, in traumatic cases, shock is more commonly attributed to other facto
rs such as blood loss or fluid loss. Traumatic shock is often classified as hypovolemic shock rather tha

Page 34

1423
n cardiogenic shock, which is caused by cardiac failure.

Solution for Question 36:


Correct Option C: Venous ulcer
• Venous ulcer: Venous ulcers are the most likely type of ulcer in this case.
• They develop due to chronic venous insufficiency, which leads to increased pressure in the veins and
impaired blood flow.
• Venous ulcers usually occur in the lower legs, particularly around the medial or lateral malleoli.
• The presence of varicose veins in both legs further supports the diagnosis of a venous ulcer.
Incorrect Options
Option A: Neuropathic ulcers or pressure ulcers are located over toe joints, under the heel, over the ma
lleoli, under the metatarsal head etc. Neuropathic ulcers can develop with any condition with peripheral
neuropathy, the most common being diabetes mellitus and B12 deficiency. Ulcer risk is further compo
unded by any foot deformity or concurrent peripheral vascular disease.
Option B: Arterial ulcer: It is located typically at the distal ends of the limbs. Typical features of Arterial
Ulcer are punched out, thin shining skin, absence of hair and brittle nails .
Etiology is associated with impaired blood flow.
• In Diabetes mellitus
• Microangiopathy
}Endothelial dysfunction
• Macroangiopathy ↓
• Impaired vascular supply lack of blood flow

Ischemia

Tissue death
Punched out ulcer
The Diagnosis is made clinically by using Doppler and is performed to assess vascular flow. Treatment
is revascularization procedures are done in the management and for infected ulcers, antibiotics and de
bridement.
Option D: Tubercular ulcer: An oral ulcer mimicking squamous cell carcinoma of buccal mucosa must b
e considered for being tubercular if biopsy of the lesion does not show definitive evidence of malignanc
y. TB is caused by the bacterium mycobacterium tuberculosis. Tuberculous lesions of the oral cavity m
ay be primary or secondary.

Page 35

1424
Solution for Question 37:
Correct Option B.
• Breath sounds: This option is correct. Differentiating between cardiac tamponade and tension
pneumothorax can be done by assessing breath sounds. In cardiac tamponade, breath sounds are
typically normal because the primary issue is compression of the heart. In tension pneumothorax, on
the other hand, there may be absent or decreased breath sounds on the affected side due to lung
collapse.
Incorrect Options:
Option A. Raised JVP (Jugular Venous Pressure): Both cardiac tamponade and tension pneumothorax
can result in an elevated JVP. However, this is not a
specific finding that can differentiate between the two conditions.
Option C. Raised heart rate: Both cardiac tamponade and tension pneumothorax can lead to an increa
sed heart rate. It is a
non-specific finding and cannot be used to differentiate between the two conditions.
Option D. Not improving after initial fluid resuscitation: This option is not specific to either cardiac tamp
onade or tension pneumothorax. It is more indicative of a
lack of response to initial fluid resuscitation and can occur in various conditions.

Solution for Question 38:


Correct Option is: C. Neuralgia
• Neuralgia refers to the condition characterized by sharp, shooting, or burning pain along the course of
a nerve. In the context of the patient's history of varicose veins and subsequent surgical intervention
(below-knee stripping of the long saphenous vein), neuralgia can occur as a common complication.
This can happen due to nerve damage during the surgical procedure or as a result of ongoing irritation
or inflammation of the nerves in the affected area.
Incorrect Options:
Options A. Hemorrhage: Hemorrhage, or excessive bleeding, is a potential complication of any surgical
procedure. However, in the given scenario, where the patient's symptoms resolved completely after th
e surgery, hemorrhage is less likely to be the most common complication.
Options B. Thromboembolism: Thromboembolism refers to the formation of a
blood clot (thrombus) that travels through the bloodstream and blocks a blood vessel. While varicose v
eins can increase the risk of blood clots, the patient's symptoms improved after the surgery, suggesting
that thromboembolism is not the most common complication in this case.
Options D. Infection: Infection is a possible complication of any surgical procedure. However, the patie
nt's symptoms resolved after the surgery, making infection less likely to be the most common complicat
ion.

Solution for Question 39:


Correct Option: C.

Page 36

1425
• In the classification of hemorrhagic shock, class III refers to moderate hypovolemia or circulatory
failure. It is characterized by a blood loss of approximately 30-40% of the total blood volume. This level
of blood loss can lead to significant cardiovascular compromise and compromised tissue perfusion.
Incorrect Option:
Option A. < 15%: This level of blood loss would typically correspond to class I
or mild hypovolemia. It may not result in significant hemodynamic instability or circulatory failure.
Option B. 15-30%: This range of blood loss would correspond to class II or moderate hypovolemia. Whi
le it can cause some hemodynamic instability, class III shock represents a
more severe stage of circulatory compromise.
Option D. > 40%: Blood loss exceeding 40% of the total blood volume would be classified as class IV o
r severe hypovolemia. This level of blood loss can result in life-threatening circulatory failure and requir
es immediate medical intervention.

Solution for Question 40:


Correct Option A:
• Wet gangrene is a type of gangrene characterized by tissue necrosis (death) that occurs in a moist or
wet environment. It is usually caused by a severe bacterial infection and impaired blood supply to the
affected area. The affected tissue appears swollen, discolored (often dark or black), and may have a
foul odor. It is commonly seen in the extremities, such as the feet.
• In the given image, the appearance of the foot with discoloration, swelling, and signs of tissue decay
suggests the presence of wet gangrene. The presence of a severe infection and impaired blood supply
are consistent with this diagnosis.
Incorrect Option:
Option B. Dry gangrene: Dry gangrene is characterized by tissue death in a dry environment without th
e presence of infection. It usually occurs due to reduced blood supply to the affected area, leading to ti
ssue necrosis. Dry gangrene often appears black, dry, and shriveled.
Option C. Frostbite: Frostbite refers to tissue injury caused by extreme cold exposure. It occurs when p
rolonged exposure to freezing temperatures leads to damage to the skin and underlying tissues. Frostb
ite can cause discoloration, numbness, and blistering of the affected area.
Option D. Ainhum: Ainhum is a rare condition characterized by the progressive constriction and autoa
mputation of the digits, usually the fifth toe. It typically affects individuals in certain geographical region
s and is associated with the formation of a fibrous band around the affected digit.

Solution for Question 41:


Correct Option A.
• Pneumatic compression stockings, also known as intermittent pneumatic compression (IPC) devices
or sequential compression devices (SCDs), are commonly used as a prophylactic measure to prevent
deep vein thrombosis (DVT) in various clinical settings. Here's an explanation of how pneumatic
compression stockings work to prevent DVT:

Page 37

1426
• Deep vein thrombosis (DVT) is the formation of a blood clot in the deep veins of the body, usually in
the legs. It can be a serious condition that may lead to complications such as pulmonary embolism if
the blood clot dislodges and travels to the lungs.
• Pneumatic compression stockings are designed to improve blood flow in the legs and reduce the risk
of blood clot formation. These stockings are made of inflatable chambers or sleeves that are connected
to a device that periodically inflates and deflates the chambers.
• When the pneumatic compression stockings are worn, the chambers sequentially inflate, applying
gentle pressure to the legs in a specific pattern. This pressure helps to promote blood flow in the veins
by compressing the superficial veins, thereby reducing venous stasis (slow blood flow) and enhancing
venous return (blood flow back to the heart).
• The sequential inflation and deflation of the chambers create a pumping effect, mimicking the natural
movement of the muscles during walking or exercising. This action helps prevent blood from pooling or
stagnating in the veins, reducing the likelihood of blood clot formation.
• Pneumatic compression stockings are often used in situations where individuals are at an increased
risk of developing DVT, such as during surgery, prolonged bed rest, or when mobility is limited. They
are also utilized for individuals with certain medical conditions or predisposing factors for DVT.
• It's important to note that pneumatic compression stockings are just one aspect of DVT prevention
and are often used in conjunction with other preventive measures. These may include early
mobilization, regular leg exercises, pharmacological prophylaxis (such as anticoagulant medications),
and maintaining adequate hydration.

Solution for Question 42:


• Among the options provided, the intercostal artery is the least commonly used graft for coronary artery
bypass graft (CABG).
• The intercostal arteries are small arteries that supply the muscles and tissues between the ribs. While
they can be used as grafts in certain situations, such as in cases where other graft options are not
available, they are not commonly used as a first-choice graft in CABG procedures.
Incorrect choices: Option a. Long saphenous vein: The long saphenous vein (also known as the great
saphenous vein) is one of the most commonly used grafts in coronary artery bypass graft (CABG) surg
ery. It is a large vein located in the leg and is frequently harvested to create bypass grafts. Its long leng
th and suitable diameter make it a favorable option for grafting coronary arteries. Option b. Internal ma
mmary artery: The internal mammary artery (also called the internal thoracic artery) is another common
ly used graft in CABG surgery. It is located in the chest wall and is known for its excellent long-term pat
ency rates. The internal mammary artery is often used to bypass the left anterior descending artery, on
e of the major coronary arteries. Option d. Radial artery: The radial artery is also a commonly used graf
t in CABG surgery. It is located in the forearm and can be harvested as an alternative to the saphenous
vein or internal mammary artery. The radial artery has shown comparable long-term patency rates to t
he internal mammary artery and is particularly useful for grafting smaller coronary arteries.

Solution for Question 43:

Page 38

1427
• The clinical picture is consistent with an acute aortic dissection. This is suggested by sudden onset
chest pain radiating to the neck and back, a widened mediastinum on X-ray, and differing blood
pressure measurements between the upper limbs. These are all classic signs of aortic dissection.
Incorrect Choices:
• Option a. Acute coronary syndrome: This typically presents with central chest pain radiating to the left
arm or jaw, accompanied by symptoms like sweating, nausea, and shortness of breath. The
discrepancy in blood pressure between the limbs is not a feature of acute coronary syndrome.
• Option b. Acute pulmonary embolism: This would present with sudden onset dyspnea, chest pain, and
possible hemoptysis. An ECG may show right heart strain or sinus tachycardia but not differing blood
pressures in the arms.
• Option d. Esophageal rupture: This would typically present with severe chest pain following a bout of
severe vomiting (Boerhaave's syndrome). The X-ray might show mediastinal air, not a widened
mediastinum.

Solution for Question 44:


• The most likely etiology of the worker's symptoms is continuous exposure to drills and machines. This
is suggestive of the vibration-induced white finger (VWF) or hand-arm vibration syndrome (HAVS), a
secondary form of Raynaud's syndrome. It is an industrial injury triggered by the continuous use of
vibrating hand-held machinery. This repeated exposure to vibration leads to damage in the nerves,
blood vessels, and joints of the hand, wrist, and arm.
Incorrect Choices:
• Option a. Candidal infection of the fingers due to continuous exposure to water: This would likely
present with symptoms such as redness, itching, and possibly pustules or blisters, rather than with
white fingers.
• Option b. Exposure to thinners and paints: Chronic exposure to certain chemicals can cause various
skin conditions, but the presentation of white fingers is not typical for this.
• Option c. Continuous exposure to cement and concrete: This could lead to irritant contact dermatitis,
but it would not cause white fingers.

Solution for Question 45:


• The correct answer is C4a. According to the CEAP (Clinical-Etiology-Anatomy-Pathophysiology)
classification for chronic venous disorders, C4a represents changes in skin and subcutaneous tissue
secondary to chronic venous disease, including eczema, pigmentation or purpura.

Page 39

1428
Incorrect Choices:
• Option a. C2 refers to varicose veins without any further complications.
• Option b. C3 signifies edema due to chronic venous disease.
• Option d. C4b represents lipodermatosclerosis or white atrophy, further progression of skin changes
due to chronic venous disease.

Solution for Question 46:


Correct Option B - Tension pneumothorax:
• Tension pneumothorax is a medical emergency that can rapidly lead to respiratory failure and cardiac
arrest. It occurs when air accumulates in the pleural space, causing lung collapse and displacement of
the mediastinum, which leads to compromised breathing. Prompt recognition and treatment, such as
needle decompression or chest tube insertion, are critical to relieve the pressure and restore lung
function.
Incorrect Options:
Option A - Cervical spine injury: While a cervical spine injury can be serious and lead to neurological c
omplications, it does not directly compromise the patient's breathing. Therefore, it is not the most life-th
reatening injury among the options.
Option C - Blunt cardiac injury: Blunt cardiac injury can occur due to trauma to the chest, resulting in da
mage to the heart or surrounding structures. While it can be severe and potentially life-threatening, it pr
imarily affects cardiac function rather than directly compromising breathing.
Option D - Laryngotracheal injury: Laryngotracheal injuries can occur in cases of severe neck trauma o
r direct airway injury. While they can lead to airway compromise and breathing difficulties, tension pneu
mothorax poses a more immediate threat to breathing and can rapidly progress to cardiorespiratory col
lapse if not promptly addressed.

Page 40

1429
Solution for Question 47:
Correct Option C - 1, 2, 3, 4:
• Since all options (1,2,3,4) are correct, C is the most appropriate answer.

Solution for Question 48:


Correct Option C - Pneumoperitoneum:
• In a patient with grade 3 splenic trauma who is managed conservatively in the ICU, the presence of
pneumoperitoneum on day 2 would indicate a need for surgery. Pneumoperitoneum refers to the
presence of air or gas in the peritoneal cavity, which suggests a perforation of an intra-abdominal organ
or structure.
• It is an important sign of an acute abdomen and typically requires surgical intervention to address the
underlying cause, such as a perforated viscus.
Incorrect Options:
Option A - Hemoglobin fall from 12mg/dL to 10mg/dL: It may not indicate a need for surgery in this cont
ext. It could be attributed to ongoing bleeding or hemolysis related to splenic trauma. The decision for s
urgery would depend on the overall clinical condition, ongoing bleeding, and hemodynamic stability of t
he patient.
Option B - Extraperitoneal bladder rupture: Although extraperitoneal bladder rupture is a
serious condition, it is not directly related to the grade 3 splenic trauma. It may require surgical repair o
r conservative management based on the extent of injury and associated complications. However, it do
es not indicate an immediate need for surgery in the context of splenic trauma.
Option D - Air in gallbladder: The presence of air in the gallbladder, known as pneumobilia, can occur d
ue to various reasons such as gallstone disease, infection, or prior surgery. It is not directly related to t
he splenic trauma and would not be a sign indicating the need for immediate surgery in this scenario.

Solution for Question 49:


Correct Option C - Needle thoracocentesis:
• Needle thoracocentesis involves inserting a large-bore needle into the pleural space to release
trapped air or fluid. In the case of a tension pneumothorax, this procedure can quickly decompress the
affected side of the chest, relieving the pressure and improving the patient's condition.
Incorrect Options:
Option A - Resuscitative thoracotomy: Resuscitative thoracotomy is a procedure performed in cases of
traumatic cardiac arrest. It involves opening the chest to gain direct access to the heart and major vess
els for resuscitative efforts. While this may be necessary in certain scenarios, such as when there is ca
rdiac arrest, it is not the immediate intervention for a tension pneumothorax.

Page 41

1430
Option B
- ICD placement: Intercostal drain cannot immediately relieve the symptoms of tension pneumothorax.
Option D - Finger thoracotomy: Finger thoracotomy is a procedure performed in cases of cardiac tamp
onade to relieve the pressure on the heart by manually opening the pericardial sac. It is not the appropr
iate intervention for a tension pneumothorax.

Solution for Question 50:


Correct Option D: Brodie Trendelenburg test
• The Brodie-Trendelenburg test is used to evaluate the competence of the superficial femoral vein
(SFJ) and to differentiate between perforator and great saphenous vein (GSV) incompetence. Here's
how the test is performed: Initially, the superficial lower extremity veins are drained by elevating the
lower limbs to 45 degrees and gently stroking the limb from the foot along the course of the major veins.
A tourniquet is then applied as close to the groin as possible, tight enough to prevent superficial vein
reflux. The patient is asked to stand, and the limb is examined. If the distal veins remain collapsed for
15 to 30 seconds after standing, the tourniquet is released. SFJ incompetence is diagnosed if the distal
veins fill rapidly upon release of the tourniquet. If the caudal veins fill rapidly when the patient stands
with the tourniquet in place, perforator incompetence is suggested. The location of the incompetent
perforator can then be determined by varying the position of the tourniquet. Rapid filling of the varices
with a tourniquet in different positions can identify the location of incompetent perforators.
• Initially, the superficial lower extremity veins are drained by elevating the lower limbs to 45 degrees
and gently stroking the limb from the foot along the course of the major veins.
• A tourniquet is then applied as close to the groin as possible, tight enough to prevent superficial vein
reflux.
• The patient is asked to stand, and the limb is examined.
• If the distal veins remain collapsed for 15 to 30 seconds after standing, the tourniquet is released.
• SFJ incompetence is diagnosed if the distal veins fill rapidly upon release of the tourniquet.
• If the caudal veins fill rapidly when the patient stands with the tourniquet in place, perforator
incompetence is suggested.
• The location of the incompetent perforator can then be determined by varying the position of the
tourniquet.
• Rapid filling of the varices with a tourniquet in different positions can identify the location of
incompetent perforators.
• Initially, the superficial lower extremity veins are drained by elevating the lower limbs to 45 degrees
and gently stroking the limb from the foot along the course of the major veins.
• A tourniquet is then applied as close to the groin as possible, tight enough to prevent superficial vein
reflux.
• The patient is asked to stand, and the limb is examined.
• If the distal veins remain collapsed for 15 to 30 seconds after standing, the tourniquet is released.
• SFJ incompetence is diagnosed if the distal veins fill rapidly upon release of the tourniquet.
• If the caudal veins fill rapidly when the patient stands with the tourniquet in place, perforator
incompetence is suggested.

Page 42

1431
• The location of the incompetent perforator can then be determined by varying the position of the
tourniquet.
• Rapid filling of the varices with a tourniquet in different positions can identify the location of
incompetent perforators.
Incorrect options:
Option A: Fegan's test:
• Fegan's test is used to assess the competency of perforator veins in the lower extremities. It involves
elevating the leg and applying pressure to the distal part of the varicose vein while simultaneously
compressing the proximal part. If there is retrograde filling of the varicose vein, it suggests perforator
incompetence.
Option B: Homan’s test:
• Homan’s test is used to assess for deep vein thrombosis (DVT) in the lower extremities. It involves
dorsiflexing the foot of the affected leg while the knee is extended, which may elicit pain in the calf
muscle if a DVT is present.
Option C: Modified Perthes test:
• The Modified Perthes test is used to assess for valve incompetence in the deep venous system,
particularly in the perforating veins. It involves applying a tourniquet to the upper thigh to occlude the
superficial veins, followed by the patient standing up. Retrograde filling of varicose veins suggests
valve incompetence.

Solution for Question 51:


Correct Option
Option B: Pain starting on taking the first step:
• Pain starting on taking the first step after a period of rest is a characteristic feature of neurogenic
claudication, which is commonly seen in spinal stenosis. It is not typically associated with intermittent
claudication.
Incorrect options:
Option A: Cramping in the muscles during exercise:
• Intermittent claudication is characterized by pain or cramping in the muscles during exercise, typically
in the lower extremities.
Option C: Most common cause of atheroma:
• Atheroma, or a buildup of fatty plaques in the arteries, is the most common cause of intermittent
claudication. The narrowing of the arteries due to atheroma restricts blood flow to the muscles during
exercise, leading to pain.
D: Level of occlusion cannot be decided based on symptoms
• The level of arterial occlusion cannot be determined solely based on symptoms in cases of
intermittent claudication. Additional diagnostic tests, such as ankle-brachial index (ABI) or angiography,
are necessary to identify the exact location and severity of arterial blockage.

Page 43

1432
Solution for Question 52:
Option A: Insertion of wide bore needle in the 5th intercostal space (ICS)
• This option involves immediate intervention to relieve tension pneumothorax, a potentially
life-threatening condition. By inserting a wide bore needle into the 5th intercostal space, slightly anterior
to the mid-axillary line, you create a pathway for trapped air to escape, thus alleviating the pressure
buildup in the thoracic cavity.
• This intervention is crucial for stabilizing the patient's condition and restoring normal respiratory
function. It allows the collapsed lung to re-expand and prevents further compromise of cardiovascular
function due to mediastinal shift.
• This procedure is supported by the ATLS (Advanced Trauma Life Support) guidelines and is
considered the standard of care for managing tension pneumothorax in adults.
Incorrect options:
Option B: Wide bore needle decompression and IV fluids
• While wide bore needle decompression is a correct intervention for tension pneumothorax, IV fluids
alone would not address the immediate threat to the patient's life. The priority should be to rapidly
decompress the tension pneumothorax to restore adequate ventilation and prevent cardiovascular
collapse.
• IV fluids may be necessary for resuscitation and stabilizing the patient's hemodynamic status, but they
should not delay the urgent intervention required for tension pneumothorax.
Option C: Intubation and positive pressure ventilation
• While intubation and positive pressure ventilation may be necessary for patients with respiratory
failure or impending respiratory arrest, it is not the immediate intervention for tension pneumothorax.
• Intubation and positive pressure ventilation may worsen the condition by further inflating the trapped
air in the thoracic cavity, exacerbating the mediastinal shift and compromising cardiovascular function.
• Intubation and positive pressure ventilation should be considered after initial decompression of
tension pneumothorax if the patient's respiratory status does not improve or deteriorates further.
Option D: eFAST
• eFAST (Extended Focused Assessment with Sonography for Trauma) is a valuable tool for rapid
assessment of trauma patients to detect free fluid in the abdomen or chest, pneumothorax, or
pericardial effusion.
• While eFAST can aid in diagnosing pneumothorax, it is not the immediate intervention required for
tension pneumothorax. Immediate decompression with a wide-bore needle is necessary to prevent
cardiovascular collapse and stabilize the patient's condition.
• eFAST may be performed as part of the secondary survey to further evaluate the patient's injuries
once the immediate life-threatening conditions, such as tension pneumothorax, have been addressed.

Solution for Question 53:


Correct Option A - Congenital diaphragmatic hernia:

Page 44

1433
• The above history along with the chest X-ray showing multiple airfluid levels above the diaphragm
indicates a diagnosis of congenital diaphragmatic hernia.
• The diagnosis of CDH is frequently made prenatally, as early as 15 weeks of gestation.
• The initial signs and symptoms include grunting respiration, chest retractions, dyspnea, and cyanosis.
• The chest X-ray demonstrates multiple bowel loops in the thoracic cavity along with a mediastinal
shift.
Incorrect Options:
Option B - Tetralogy of Fallot:
• Chest X-ray of Tetralogy of Fallot shows a boot-shaped heart.
Option C - Transposition of the great vessels:
• Chest X-ray findings of transposition of the great vessels show egg on a string appearance of the
heart
Option D - Tracheoesophageal fistula:
• Coiling of nasogastric tube in the upper esophageal pouch on chest X-ray indicates a diagnosis of
Tracheoesophageal fistula

Page 45

1434
Burns
1. A 30-year-old female is brought to the emergency department after being involved in a house fire.
She has sustained deep partial-thickness burns on her chest and both arms, involving 15% of her total
body surface area (TBSA). On examination, her burn wounds are blistered and pink with a few areas of
open skin. Laboratory tests reveals an elevated white blood cell count and increased levels of
inflammatory mediators. What underlying pathophysiological processes contribute to the observed
clinical findings in this patient?
(or)
Which of the following mechanism is seen in patients with Burns injury?
A. Decreased blood flow due to vasoconstriction.
B. Impaired immune response leading to reduced inflammation.
C. Loss of fluid through unaffected skin leading to dehydration.
D. Increased capillary permeability and extravasation of fluid.
----------------------------------------
2. A 25-year-old chef accidentally spilled a pot of boiling water on his hand and now experiences pain,
redness, and blister formation on the affected area. Upon examination, the skin appeared mottled and
moist. There were clear fluid-filled blisters surrounded by an area of redness. The patient reported
intense pain at the burn site. Based on the provided information, what is the most likely classification of
the burn injury?
(or)
What classification of burn injury is most likely with pain, redness, blister formation, mottled and moist
skin, clear fluid-filled blisters, and intense pain after spilling boiling water on the hand?
A. Superficial burn involving only the epidermis
B. Full-thickness burn extending through the dermis and into the subcutaneous tissue
C. Burn involving only the dermis without affecting deeper tissues
D. Partial-thickness burn affecting the epidermis and upper layers of the dermis
----------------------------------------
3. A patient presents with painless burn characterized by dry and leathery skin, extending from the wrist
to the elbow. What is the most appropriate initial management for this burn?
A. Application of a topical antibiotic ointment
B. Immediate surgical debridement and grafting
C. Administration of intravenous antibiotics
D. Daily dressing changes with sterile saline
----------------------------------------
4. A 28-year-old individual sustained severe burns involving over 50% of their body surface area due to
a gas explosion. Upon admission, the skin was charred and leathery, and the patient experienced
difficulty breathing. Despite supplemental oxygen, the patient's oxygen saturation remained low. What
is the most likely cause of death in this patient?
(or)

1435
What is the most likely cause of death after severe burns over 50% of body surface area, charred and
leathery skin, difficulty breathing, and low oxygen saturation despite supplemental oxygen?
A. Early hypovolemia
B. Inhalation injury causing asphyxia
C. Organ failure due to sepsis
D. Cardiac arrhythmias
----------------------------------------
5. A 10-year-old child was admitted to the emergency department with extensive burns sustained from
a house fire. The burns covered approximately 25% of the child's body surface area. Considering the
provided information, what is the most appropriate fluid resuscitation strategy for this pediatric burn
patient according to the modified ATLS version of the Parkland formula?
(or)
What is the most appropriate fluid resuscitation strategy for a 10-year-old child with extensive burns
covering approximately 25% of the body surface area?
A. Administer 2 ml of fluid per kg body weight × % BSA.
B. Administer 3 ml of fluid per kg body weight × % BSA.
C. Administer 4 ml of fluid per kg body weight × % BSA .
D. Calculate fluid resuscitation based solely on age and type of injury.
----------------------------------------
6. A 25-year-old laboratory technician accidentally spilled a corrosive chemical over his arm while
working. He immediately experiences pain and redness in the affected area. The chemical is identified
as phenol. The patient seeks medical attention promptly. What is the most appropriate course of action
for the management of the chemical burn caused by phenol exposure?
(or)
What is the most appropriate management for a chemical burn caused by phenol exposure?
A. Immediately neutralize the chemical with a base
B. Perform copious irrigation with water or saline for at least 20 minutes.
C. Apply a neutralizing agent directly to the burn site.
D. Remove contaminated clothing but keep jewelry on the affected area.
----------------------------------------
7. A 28-year-old electrician was working on a power distribution unit when he accidentally came in
contact with a live electrical wire. He experienced a sudden jolt and was immediately rushed to the
hospital. On examination, it was noted that the entry and exit points of the electrical current were on his
fingers. There were small, deep burns at these sites. What type of electrical current is likely responsible
for the injuries observed in this electrician?
(or)
What type of electrical current is likely responsible for the injuries observed in an electrician who
experienced a sudden jolt and had small, deep burns at the entry and exit points on his fingers while
working on a power distribution unit?
A. Direct current (DC)

Page 2

1436
B. High-voltage alternating current (AC)
C. Low-voltage direct current (DC)
D. Low-voltage alternating current (AC)
----------------------------------------
8. In a training workshop on treating burn patients, the instructor discusses different types of burns and
their management. He describes different formulas to calculate burn surface area and the indications
for referral to a burn unit. All the patients have to be referred to the burn centre except?
(or)
Which patient discussed in a training workshop on treating burn patients and calculating burn surface
area does not require referral to a burn center?
A. An 81-year-old patient suffering from Alzheimer's disease sustained a burn on her abdomen and
chest wall due to leakage of gastric content from her gastrostomy feeding tube
B. A patient with high-voltage electrical burns in a bathtub
C. A patient sustaining partial thickness burns over 6% of total body surface area
D. A patient who spilled boiling water on herself sustaining full thickness burns over 5% of total body
surface area
----------------------------------------
9. A 32-year-old male presents to the emergency department with a burn injury 2 hours ago.
Examination reveals second-degree burns covering 20% of the total body surface area. The patient is
immediately resuscitated with fluids. Which of the following fluids is most commonly used in the
patient?
(or)
In the immediate resuscitation of second-degree burns covering 20% of the total body surface area, the
most commonly used fluid is?
A. Dextrose 5%
B. Normal saline
C. Ringer lactate
D. Albumin 5%
----------------------------------------
10. What is the best method to assess burns caused by boiling water in a 15-month-old boy with burn
injuries on his bilateral lower extremities, lower torso, and abdomen, which initially appeared not severe
but later developed blisters?
(or)
A 15-month-old boy is brought to the hospital for a burn injury. Baby fell into a tub of hot water, father
pulled him out and poured water at room temparture over affected areas of lower extremities, abdomen
and torso. The burns were initially described as not too bad but later developed into blisters. The best
method to assess burns caused by boiling water in a child is?
A. Palm method
B. Rule of 9
C. Modified Lund and Browder chart

Page 3

1437
D. Parkland formula
----------------------------------------
11. A 25-year-old man comes to the emergency department after he sustained a burn injury to his right
arm when his hand was trapped in a steam-rolling machine for making carpets under high
temperatures. He has severe burn in his right arm. On examination, a third-degree circumferential burn
wound is noted in the right arm and forearm. Which of the following has to be closely monitored in this
patient?
(or)
In a third-degree circumferential burn on the right arm and forearm, what aspect needs to be closely
monitored?
A. Blood gases
B. Carboxyhemoglobin level
C. Myoglobinuria
D. Peripheral pulse and circulation
----------------------------------------
12. A 19-year-old woman is brought to the emergency department due to burns to the chest, arms and
legs. Her blood pressure is 110/70 mmHg, her pulse is 80/min, and her respiratory rate 18/minute. On
physical examination, the patient appears in pain with full awareness. She has deep second degree
burns to the chest, arms, and forearms. When can we expect the patient to have re-epithelialization of
the burn wound?
(or)
When can we expect re-epithelialization of the burn wound with deep second-degree burns to the
chest, arms, and forearms?
A. 1 week
B. 4 weeks
C. 10 weeks
D. 12 weeks
----------------------------------------
13. All of the following are causes of death in burns patients except?
A. ARDS
B. Shock
C. Sepsis
D. Hyperkalemia
----------------------------------------
14. In a patient with second and third-degree burns involving 5% of his total body surface area (TBSA),
what common electrolyte abnormality can be expected?
(or)
A 31-year-old male patient presents to the emergency department after a burn on his left arm. He
works in the glass manufacturing industry. He has a Hydrofluoric (HF) acid burn caused by the
explosion of a 5-liter acid container. On physical examination, second and third-degree burns involving

Page 4

1438
5% of his TBSA are noted, as shown in the image below. The lesions induced by the HF burn on the
left forearm and the trunk are immediately cleaned with plenty of water; then, his clothes are removed.
The common electrolyte abnormality that can be expected in this patient would be?

A. Hyponatremia
B. Hypocalcemia
C. Hypokalemia
D. None of the above
----------------------------------------
15. A 30-year-old woman weighing 50 kg comes to the Burn Institute 1.5 hours after a flame burns her
head, trunk, and limbs. The total burnt area is 50% TBSA (total body surface). How much lactate
Ringer's solution have to be infused for fluid resuscitation in this patient in the first 8 hours of injury?
A. 5 litres
B. 2.5 litres
C. 7 litres
D. 8 litres
----------------------------------------
16. What is the next best step in managing burns across the anterior portion of his chest and belly, with
dry, non-tender, circumferential burns over both arms, whose urine output is low despite fluid
resuscitation?
(or)
A 45-year-old man presents to the emergency department complaining of boiling oil splash on his
exposed skin. His vitals are stable. The patient has burns across the anterior portion of his chest and
belly with dry, non-tender, circumferential burns over both arms. After the primary survey, the patient is
administered a bolus of 1L of normal saline and the Foley's catheter drains 10 mL of urine after one
hour. Which of the following is the next best step in managing this patient?
A. Place moist dressings and discharge the patient
B. Plan Escharotomy
C. Keep patient under observation
D. Give the patient IV fluids and admit him to the intensive care unit
----------------------------------------

Page 5

1439
17. A 40-year-old woman with no notable medical history presents with a 1% deep second-degree burn
over the upper outer quadrant of her right breast after an accidental splatter of hot oil during cooking.
On examination, the patient is vitally stable with blister formation. Blister is deroofed, and the wound is
dressed. Which of the following is the characteristic feature of deep second-degree burns?
(or)
What is the characteristic feature of deep second-degree burns with a 1% burn over the upper outer
quadrant of the breast?
A. Injury is localised to the epidermis
B. Injury is localised to the epidermis and superficial dermis
C. Injury goes through the skin and subcutaneous fat into underlying muscle or bone
D. Injury to the epidermis and deep reticular dermis
----------------------------------------
18. Which of the following is false regarding silver sulfadiazine?
(or)
Which of the following is incorrect regarding silver sulfadiazine?
A. Used as prophylaxis against burn wound infections with a wide range of antimicrobial activity
B. It can penetrate an eschar.
C. Has limited systemic absorption
D. May inhibit epithelial migration in partial thickness wound healing
----------------------------------------
19. Which topical antimicrobial agent can be used for a large, purulent, and erythematous burn wound
with a separating eschar on the lateral aspect of the left foot?
(or)
A 55-year old man, who is a diabetic for 15 years on irregular treatment came to hospital with a wound
on the left foot for 2 weeks. He used a hot water bottle to warm his feet, next day he noticed the wound.
On examination the burn wound has a thick leathery eschar with purulent discharge. Which of the
following topical antibiotics is used in this patient?
A. Mafenide acetate
B. Neomycin
C. Silver nitrate
D. Silver sulfadiazine
----------------------------------------
20. What is the most common ignition source in an operating room that may cause thermal injuries, as
explored by the hospital administration after a 60-year-old man develops a burn mark on his left arm
after surgery?
(or)
What's the most common ignition source in an operating room leading to thermal injuries during
surgery?
A. Argon beam coagulation

Page 6

1440
B. Lasers
C. Fibre optic illumination
D. Electrosurgical equipment
----------------------------------------

Correct Answers
Question Correct Answer

Question 1 4
Question 2 4
Question 3 2
Question 4 2
Question 5 2
Question 6 2
Question 7 1
Question 8 3
Question 9 3
Question 10 3
Question 11 4
Question 12 2
Question 13 4
Question 14 2
Question 15 2
Question 16 4
Question 17 4
Question 18 2
Question 19 1
Question 20 4

Solution for Question 1:


Correct Option D- Increased capillary permeability and extravasation of fluid:
• Inflammatory Mediator Release: Burn injuries trigger the release of inflammatory mediators like
cytokines, histamine, and prostaglandins due to tissue damage and stress.
• Vasodilation and Increased Permeability: These mediators cause vasodilation and increased
permeability of capillaries, leading to increased blood flow, redness, and leakage of fluid, proteins, and
immune cells into the interstitial spaces.
• Formation of Edema and Blisters: The increased capillary permeability results in the accumulation of
fluid in the interstitial spaces, leading to edema and the formation of blisters at the burn site.

Page 7

1441
• Systemic Inflammatory Response: The release of inflammatory mediators and the migration of
immune cells contribute to a systemic inflammatory response, characterized by elevated white blood
cell count and increased levels of inflammatory markers.
• Hypermetabolic State: Burn injuries lead to a hypermetabolic state due to the release of stress
hormones like catecholamines, glucagon, and corticosteroids, resulting in increased energy
expenditure and heat production.
Incorrect Options:
Option A - Decreased blood flow due to vasoconstriction: In burn injuries, vasoconstriction is not the pri
mary response. Instead, there is vasodilation due to the release of inflammatory mediators like histami
ne, prostaglandins, and cytokines. This vasodilation leads to increased blood flow to the burn area, cau
sing redness, warmth, and edema.
Option B - Impaired immune response leading to reduced inflammation: Burn injuries typically result in
an exaggerated immune response characterized by the release of inflammatory mediators and recruit
ment of immune cells to the burn site. This immune response contributes to the observed inflammation,
edema, and increased white blood cell count. Thus, impaired immune response is not the underlying
mechanism.
Option C - Loss of fluid through unaffected skin leading to dehydration: While fluid loss is a concern in
burn injuries, the primary mechanism is the increased capillary permeability in the burn area, causing fl
uid to leak into the interstitial spaces and forming blisters. Loss of fluid through unaffected skin is not a
significant contributor to the observed fluid imbalance.

Solution for Question 2:


Correct Option D- Partial-thickness burns affecting the epidermis and upper layers of the dermis:
• This choice accurately describes a second-degree burn. It involves damage to both the epidermis and
the upper layers of the dermis.
• Second-degree burns can result in blister formation, redness, pain, and swelling.
• They are classified into two types: superficial partial-thickness (involving the upper layers of the
dermis) and deep partial-thickness (involving deeper layers of the dermis).
Characteristic
Superficial Partial-Thickness Burn
Deep Partial-Thickness Burn
Depth of Burn
Involves upper layers of the dermis
Involves deeper layers of the dermis
Appearance
Blisters with clear fluid formation
Blisters might be absent or fluid may be turbid
Color of Skin
Pink to red

Page 8

1442
Pale pink to white
Sensation
Extremely painful
Less painful than superficial burns, but still sensitive
Healing Time
Generally heals within 2-3 weeks with minimal scarring
Healing may take longer, and scarring is more likely
Skin Functionality
Typically intact
Skin grafting may be required to restore functionality
Risk of Infection
Relatively lower
Higher due to the depth and exposure
Long-term Consequences
Minimal scarring
Higher risk of scarring and contractures
Treatment
Generally requires minimal medical intervention
May require medical intervention and skin grafting
Incorrect Options:
Option A- Superficial burn involving only the epidermis: This choice describes a first-degree burn, whic
h is characterized by damage to the outermost layer of skin (epidermis). It typically presents with redne
ss, pain, and mild swelling but does not result in blister formation.
Option B- Full-thickness burn extending through the dermis and into the subcutaneous tissue: This choi
ce describes a third-degree burn, where all layers of the skin (epidermis, dermis, and sometimes subcu
taneous tissue) are damaged. This type of burn is often painless and appears charred or white in color.
Option C-Burn involving only the dermis without affecting deeper tissues: This choice doesn't accuratel
y describe any common burn classification. Burns that involve only the dermis would still affect deeper l
ayers of the skin and possibly lead to blister formation.

Solution for Question 3:


Correct Option B- Immediate surgical debridement and grafting:
• Burns characterized by dry and leathery skin, known as full-thickness (third-degree) burns, indicate
significant damage to the skin layers, including nerve endings.
• Painlessness suggests nerve injury and deep tissue damage.
• Immediate surgical debridement and grafting are crucial to remove dead tissue (debridement) and
replace it with healthy skin (grafting) to promote wound healing and prevent complications such as

Page 9

1443
infection.
• This approach helps in restoring function and aesthetics and reduces the risk of complications
associated with full-thickness burns.
Incorrect Options:
Option A- Application of a topical antibiotic ointment:
• Topical antibiotic ointments are not suitable for full-thickness burns as they cannot penetrate the deep
tissue layers affected by the burn.
• These burns require surgical intervention rather than topical treatment alone.
Option C- Administration of intravenous antibiotics:
• Intravenous antibiotics are not the primary treatment for burns, especially full-thickness burns, unless
there are signs of infection.
• Initial management should focus on addressing tissue damage and preventing infection through
surgical intervention.
Option D- Daily dressing changes with sterile saline:
• Daily dressing changes with sterile saline may be part of the wound care regimen after surgical
debridement and grafting.
• However, it is not the most appropriate initial management for a full-thickness burn extending from the
wrist to the elbow.

Solution for Question 4:


Correct Option B- Inhalation injury causing asphyxia
• Inhalation injury occurs when a burn victim breathes in hot air, smoke, or toxic gas particles.
• This can lead to damage in the respiratory tract, airway constriction, and inadequate oxygen intake,
causing asphyxia and respiratory distress.
• Inhalation injuries can rapidly compromise the airway and lead to death at the accident site.
• Early intubation is thus advisable in cases of burns with possible inhalational injury.
Incorrect Options:
Option A- Early hypovolemia: Early hypovolemia refers to the rapid loss of fluids and blood volume due
to the burn injury. While hypovolemic shock is a significant concern in burn patients, it usually develop
s shortly after the injury and may not be the immediate cause of death at the site of the burn.
Option C- Organ failure due to sepsis: Sepsis is a severe infection that can develop as a
complication of burn injuries. While sepsis is a common cause of death in burn patients, it typically occ
urs later in the course of treatment rather than immediately at the site of the burn.
Option D- Cardiac arrhythmias: Cardiac arrhythmias (irregular heartbeats) can result from the stress an
d physiological changes caused by severe burn injuries. While arrhythmias can contribute to complicati
ons in burn patients, they are unlikely to be the immediate cause of death at the site of the burn.

Page 10

1444
Solution for Question 5:
Correct Option B- Administer 3 ml of fluid per kg body weight × % BSA.
• According to the modified ATLS Parkland formula for children <14 years and ≤30 kg weight, the fluid
resuscitation rate is 3 ml × body weight × % BSA. Since this child is 10 years old and falls within the
specified weight range, the appropriate fluid resuscitation strategy involves calculating fluid volume
based on this formula.
Burn resuscitation fluid rates and Target Urine output by Burn type and Age
Category of Burn
Age and Weight
Adjusted Fluid rates
Urine Output
Flame or Scald
Adult and older 14 children (>14 years old)
2 ml LR x kg x % TBSA
0.5 ml / kg/hr 30-50 ml/kg/hr
Children (<14 years old)
3 ml LR x Kg x % TBSA
1 ml / kg / hr
Infant and young children (£ 30 kg)
3 ml LR x kg x % TBSA
Plus a sugar - containing solution at maintenance rate
1 ml/kg/hr
Electrical injury
All ages
4 ml LR x Kg x % TBSA until urine clears
1-1.5 ml/ kg / hr until urine clears
LR: Lactated Ringer’s solution
TBSA: Total Body Surface Area
Half of the fluid is given over the course of 8 hours and the remaining half is provided over a
span of 16 hours. The rate of fluid administration should be titrated to effect using a
target urine output of 0.5 ml/kg/hr in adults or 1
ml/kg/hr in children who are hemodynamically normal. Boluses are reserved for unstable patients.
Incorrect Choices:
Option A- Administer 2 ml of fluid per kg body weight × % BSA: This choice suggests a fluid resuscitati
on rate that is lower than the appropriate formula. For pediatric patients, the modified ATLS Parkland fo
rmula involves a higher fluid volume per body weight and percentage of burned surface area.
Option C- Administer 4 ml of fluid per kg body weight × % BSA : This choice suggests a fluid resuscitati
on rate that is higher than the appropriate formula. For pediatric patients, the modified ATLS Parkland f

Page 11

1445
ormula is used.
Option D- Calculate fluid resuscitation based solely on age and type of injury. Fluid resuscitation strate
gies should consider both age and weight along with the type and extent of burns. The modified ATLS
Parkland formula takes into account these factors to provide a
comprehensive approach to fluid resuscitation.

Solution for Question 6:


Correct Option B- Perform copious irrigation with water or saline for at least 20 minutes.
• In cases of chemical burns, copious irrigation with water or saline is the recommended initial step to
dilute and remove the chemical from the affected area.
• Immediately flush away the chemical with large amounts of warmed water, for at least 20 to 30
minutes, using a shower or hose. Alkali burns require longer irrigation. Alkali burns to the eye require
continuous irrigation during the first 8 hours after the burn.
• Low molecular weight polyethylene glycol (PEG) is the currently accepted decontamination fluid for
phenol burns. When LMW PEG is not available, flushing the exposed area with copious amounts of
water for 20-30 minutes may be effective.
• For phenol burns, which can lead to deeper tissue infiltration, proper and thorough irrigation is
especially important. While the instinct might be to neutralize the chemical, this is not advised as it can
lead to exothermic reactions and worsen the injury.
Incorrect Options:
Option A-Immediately neutralize the chemical with water or saline: This choice is not recommended for
chemical burns. Attempting to neutralize the chemical immediately can cause exothermic reactions, w
hich can lead to further tissue damage and injury.
Option C-Apply a neutralizing agent directly to the burn site: Applying a neutralizing agent directly to th
e burn site is not advised. It can lead to exothermic reactions and further injury. Copious irrigation with
water or saline is the preferred method to remove the chemical.
Option D-Remove contaminated clothing but keep jewelry on the affected area: Contaminated clothing
should be promptly removed from the burn area to prevent further chemical exposure. Jewelry should
also be removed to allow thorough irrigation and assessment of the burn.

Solution for Question 7:


Correct Option: A- Direct current (DC)
• The injuries described, including small, deep burns at the entry and exit points of electrical contact,
are characteristic of electrical injuries caused by direct current (DC).
• Direct Current (DC) Injuries: Low-tension situations with limited energy. Small, deep burns at entry
and exit points due to increased resistance. Often associated with localized tissue damage, especially
at fingers and toes. Tendon and nerve damage can occur.
• Low-tension situations with limited energy.
• Small, deep burns at entry and exit points due to increased resistance.

Page 12

1446
• Often associated with localized tissue damage, especially at fingers and toes.
• Tendon and nerve damage can occur.
• Alternate Current (AC) Injuries: May create tetany in muscles, making detachment from the power
source difficult, causing further damage. Main concern is interference with cardiac pacing, not causing
significant myocardial damage. ECG monitoring is performed.
• May create tetany in muscles, making detachment from the power source difficult, causing further
damage.
• Main concern is interference with cardiac pacing, not causing significant myocardial damage.
• ECG monitoring is performed.
• Low-tension situations with limited energy.
• Small, deep burns at entry and exit points due to increased resistance.
• Often associated with localized tissue damage, especially at fingers and toes.
• Tendon and nerve damage can occur.
• May create tetany in muscles, making detachment from the power source difficult, causing further
damage.
• Main concern is interference with cardiac pacing, not causing significant myocardial damage.
• ECG monitoring is performed.
Incorrect Options:
Option B- High-voltage alternating current (AC): This choice does not align with the information provide
d. High-voltage AC injuries are associated with more extensive tissue damage and can cause deep bur
ns, but the scenario describes low-tension injuries with small, deep burns.
Option C- Low-voltage direct current (DC): High-voltage current (greater than 500 V
to 1000 V) typically will result in deep burns, while low-voltage current (110 V
to 120 V) is more likely to result in tetany. Deep burns do not fit with low-voltage DC injuries.
Option D- Low-voltage alternating current (AC): While low-voltage AC injuries can cause burns and tiss
ue damage, the scenario specifically mentions direct current (DC) injuries with entry and exit point burn
s. AC injuries, even at low voltage, are more likely to produce surface burns and can also cause tetany
and interference with cardiac pacing.

Solution for Question 8:


Option C: A patient sustaining partial thickness burns over 6% of total body surface area
• A patient with a partial thickness burn involving 6% of body surface area need not be referred to the
burn centre.
PATIENTS WITH THE FOLLOWING CRITERIA ARE REFERRED TO DESIGNATED BURN CENTER
• Partial-thickness burns >10% of total body surface area (TBSA)
• Burns involving the face, hands, feet, genitalia, perineum or major joints
• Any full-thickness burn.
• Electrical burns, including lightning injury

Page 13

1447
• Chemical burns
• Inhalation injury
• Burns in patients with preexisting medical disorders that could complicate management, prolong
recovery, or affect the outcome
• Any patient with burns and concomitant trauma (e.g., fractures) in which the burn injury poses a
greater immediate risk for morbidity and mortality
• Burned children in hospitals without qualified personnel or equipment to care for children
• Burns in patients who will require social, emotional, or long-term rehabilitative intervention
Option A: An 81-year-old patient suffering from Alzheimer's disease sustained a burn on her abdomen
and chest wall due to leakage of gastric content from her gastrostomy feeding tube
• This 81-year-old woman has co-existent diseases has a risk for complications considering her
condition. Therefore, it is necessary to refer her to the burn centre.
Option B: A patient with high-voltage electrical burns in a bathtub
• The patient with electrical burns should be referred to a burn centre.
• Electrical burns differ from thermal or chemical burns in that they cause much more subdermal
damage
Option D: A patient who spilled boiling water on herself sustaining full thickness burns over 5% of total
body surface area
• The patient has full-thickness and partial-thickness burns covering 5 % of TBSA.
• The full-thickness burns in this patient make it necessary for her to be referred to a burn centre

Solution for Question 9:


Option C: Ringer lactate
• Ringer Lactate is the most commonly used fluid in burn patients.
• Hartmann’s solution or Ringer’s lactate is the most commonly used crystalloid as it most closely
replicates the osmolality of plasma. It is considerably less expensive than colloid and can maintain
intravascular volume.
• There are three main fuids used in the resuscitation stage: crystalloid (by far the most common),
colloid and, in some centres, hypertonic saline.
• Intravenous resuscitation is appropriate for any child with a burn greater than 10% of total body
surface area (TBSA) and 15% TBSA for adults
• Rule of 9 is used to calculate the body surface area with a burn.
• The most widely used and simplest formula to calculate fluid requirement is the Parkland formula in
adults Volume = total percentage body surface area x weight (kg) x 2 Half this volume is given in the
first 8 hours, and the second half is given in the subsequent 16 hours
• Volume = total percentage body surface area x weight (kg) x 2
• Half this volume is given in the first 8 hours, and the second half is given in the subsequent 16 hours
• Volume = total percentage body surface area x weight (kg) x 2

Page 14

1448
• Half this volume is given in the first 8 hours, and the second half is given in the subsequent 16 hours
Option A: Dextrose 5%
• Dextrose 5 % is a hypotonic fluid.
• It is used to cover the free water deficit. It is not the best choice for burn patients.
Option B: Normal saline
• Normal saline is not used in burn patients as it contains chloride in an increased amount, and it can
lead to hyperchloremic metabolic acidosis.
• Normal saline also increases the risk for hypercoagulability.
Option D: Albumin 5%
• Albumin 5% is a colloid. It is used in patients with hepatorenal syndrome and spontaneous bacterial
peritonitis.
• Although it can be used in burn patients, it is not the most commonly used fluids.

Solution for Question 10:


Option C: Modified Lund and Browder chart
MODIFIEDLUND AND BROWDER CHART
• Takes into account different proportional body surface areas in children according to age.
• Different percentages are used as the ratio of head and neck surface to the body surface is greater in
children than in adults.

Option A: Palm method


• In the case of smaller burns or patches of burn
• The palmar surface of the hand of the person who is burned (not the fingers or wrist area) accounts
for 1 % of total body surface area (TBSA) and is useful for calculating the percentage of burns

Page 15

1449
Option B: Rule of 9
• It is not the most accurate method for the calculation of TBSA.
• The ‘rule of nines’ is adequate for a first approximation only.
• It is a crude but quick and effective method of estimating burn size.

Option D: Parkland formula


• Parkland formula is used to calculate the amount of fluid needed for resuscitation.

Solution for Question 11:


Option D: Peripheral pulse and circulation
• This patient has circumferential burns to his right arm and forearm, which can lead to compartment
syndrome. Thus, circulation to the peripheries should be assessed.
• Full-thickness burns with a rigid eschar can form a tourniquet effect as oedema progresses, leading to
compromised venous outflow and, eventually, arterial inflow.
• Most common in circumferential extremity burns.
• Warning signs of impending compartment syndrome may include Paresthesias Pain Decreased
capillary refill (pallor) Progression to loss of distal pulses Paralysis
• Paresthesias
• Pain
• Decreased capillary refill (pallor)
• Progression to loss of distal pulses
• Paralysis
• Fasciotomy is done to relieve pressure.
• Paresthesias

Page 16

1450
• Pain
• Decreased capillary refill (pallor)
• Progression to loss of distal pulses
• Paralysis
Option A: Blood gases
• Blood gases can detect inhalational injury. However, this patient’s circumferential burn makes
monitoring of peripheral pulses necessary.
Option B: Carboxyhemoglobin level
• Carboxyhemoglobin level is measured in carbon monoxide poisoning as oxygen saturation may
falsely appear normal on pulse oximetry.
Option C: Myoglobinuria
• Myoglobinuria is a sign of rhabdomyolysis, commonly occurring in electrical burns and crush injuries.
• In thermal burns, rhabdomyolysis is not common.

Solution for Question 12:


Option B: 4 weeks
Second-degree (partial thickness) burns:
• This patient has deep second-degree burns in which the re-epithelialization occurs in 4 weeks.
• The burn site appears red, blistered and may be swollen and painful.
• Classified into superficial and deep
Superficial partial thickness burn
Deep partial thickness burn
Involvement of epidermis + Papillary dermis
• Involvement of epidermis + Reticular dermis
• Burn surface area is mottled having whitish and pinkish area
Blisters present/ Painful
No blisters/ no pain
Blanch to touch
Pink prick sensation is preserved
Healing occurs without scarring (within 7-14 days)
Healing occur with scarring (3-9 weeks)
• Early wound closure after burns is important in terms of survival and long-term outcomes.
• It reduces the risk of infection and fluid loss.
Option A: 1 week

Page 17

1451
• Superficial second-degree burns can epithelialize in 1 week. However, this patient has a deep
second-degree burn that does not epithelialize in 1 week.
Option C: 10 weeks
• The deep second-degree burn does not heal in 10 weeks; delayed healing can occur due to poor
wound care.
Option D: 12 weeks
• Second-degree burns re-epithelialize in 2-5 weeks, not in 12 weeks.

Solution for Question 13:


Option D: Hyperkalemia
• Hyperkalemia can occur in burn patients as cell lysis and tissue necrosis occurs.
• Rhabdomyolysis, especially in electric burn patients, leads to hyperkalemia.
• However, hyperkalemia is not the common cause of death in burn patients.
• All other listed options are the causes of death in burn patients
Option A: ARDS
• Acute respiratory distress syndrome (ARDS) is the leading cause of death in burn patients.
• It can occur due to inhalation injury or due to systemic inflammatory response to burns
Option B: Shock
• Burn patients with greater than 20% of TBSA have disruption of normal hemostasis and can lead to
intravascular hypovolemia.
Option C: Sepsis
• Burn wound infection is one of the most important causes of sepsis. It is associated with high fatality
rates in patients with burn injury.

Solution for Question 14:


Option B: Hypocalcemia
• At higher concentrations, hydrofluoric acid can penetrate deep tissues and cause liquefaction
necrosis.
• It is an extremely toxic and damaging acid.
• Hydrofluoric acid causes hypocalcemia
• Insoluble calcium fluoride is the cause of hypocalcemia and pain associated with tissue toxicity.
• Calcium-based therapies are the mainstay for treating Hydrofluoric acid burns.
• Topical application of calcium gluconate over wounds and IV administration of calcium gluconate for
systemic symptoms relief

Page 18

1452
• Intra-arterial calcium gluconate infusion provides effective treatment of progressive tissue injury and
intense pain
Option A: Hyponatremia
• Burn patients have water loss and can cause hypernatremia, not hyponatremia.
• This commonly occurs in severely burned patients due to insensible water loss.
Option C: Hypokalemia
• Hyperkalemia commonly occurs in hydrofluoric acid burns.
Option D: None of the above
• Absorbed fluoride can result in life-threatening hypocalcemia by binding fluoride ions to calcium.

Solution for Question 15:


Option B: 2.5 litres

From the given question:


% BSA= 50 %
Body wt. (kg) = 50 kg
• Total fluid volume= 2ml x kg body weight x %BSA
Therefore, 2 mL x 50
kg x 50 = 5000 mL or 5L
• 50% is given in first 8 hours and next 50% in next 16 hours. Thus, 2.5L should be given in the first 8
hours and 2.5 litres in the next 16 hours

Solution for Question 16:


Option D: Give the patient IV fluids and admit him to the intensive care unit
• According to the rule of nines, this patient's circumferential burns, which cover 36% or more of his
body (arms, chest, and abdomen), little urine production, and the need for vigorous IV fluid rehydration
make this patient a candidate for an escharotomy and ICU admission.
• Burns typically require active management because they increase the patient's risk of infection and
dehydration. The best first step in managing a patient who has had a burn is to examine the airway,
check for oral burns, and intubate if necessary. Patients will then require two large bore IVs and IV
fluids to titrate urine output to 0.5ml/kg/hr. Prophylactic antibiotics should be administered.
Option A: Place moist dressings and discharge the patient
• Only if the patient has a very minor/localised and superficial burn would moist dressings and
discharge be appropriate.
Option B: Plan Escharotomy

Page 19

1453
• An escharotomy is unquestionably required; however, given this patient's low urine production and
need for IV fluids, ICU admission seems to be the next management step.
Option C: Keep patient under observation
• It is inappropriate to keep this patient under constant supervision in the emergency room because
they require extra fluids, antibiotics, and an escharotomy.

Solution for Question 17:


Option D: Injury to the epidermis and deep reticular dermis
• Injury to the epidermis and deep reticular dermis is a characteristic feature of a deep second-degree
burn.
• Burns of the second degree are skin wounds brought on by heat, radiation, electricity, chemicals, or
friction. The epidermis, the top layer of skin, and the tissue beneath the skin are both damaged by a
deep second-degree burn (dermis). Another name for this kind of Burn is a deep partial-thickness burn.
• This kind of Burn typically results in blisters after an injury and may hurt when applied pressure. The
skin with this Burn is spotted, remains white when touched, may appear waxy in some spots, and is
either dry or slightly damp. With severe burns, infection risk is a significant worry.
Option A: Injury is localised to the epidermis
• It is a characteristic feature of First Degree Burn. First-degree burns only impact the epidermis or
skin's surface layer. The burn site is painful, dry, and red; no blisters are present. One example is a
minor sunburn. Rare cases of long-lasting tissue injury typically involve changes in skin tone.
Option B: Injury is localised to the epidermis and superficial dermis
• It is a characteristic feature of superficial Second Degree burns. According to their depth,
second-degree burns fall into two categories: The first and second layers of skin are damaged by
superficial partial-thickness burns (upto papillary dermis is involved), frequently brought on by hot water
or hot objects. When touched, the skin around the Burn blanches (turns white).
Option C: Injury goes through the skin and subcutaneous fat into underlying muscle or bone
• It is a characteristic feature of fourth-degree Burns. The most serious type of burns, fourth-degree
burns, may endanger life. All layers of the skin, muscles, tendons, and bones are affected, making
them the most serious and profound damage.

Solution for Question 18:


Option B: It can penetrate an eschar.
• Silver sulfadiazine cannot penetrate an eschar while mafenide acetate can.
• It has a broad antibacterial spectrum and is easily applied, inexpensive, and soothing.
• Application is painless.
• It does not cause significant systemic absorption or metabolic side effects.
• There may be mild inhibition of epithelialization, and it can leave black tattoos from silver ions.

Page 20

1454
• Transient leukopenia may develop, resolving spontaneously.
• True allergic reactions to the sulfa component are rare, but caution is needed in patients with
sulfonamide hypersensitivity or hepatic impairment.
Option A: Used as prophylaxis against burn wound infections with a wide range of antimicrobial activity
• Silver sulfadiazine is primarily used as prophylaxis against burn wound infections rather than
treatment of existing infections, hence this statement is correct
Option C: Has limited systemic absorption
• It has limited systemic absorption.
• It should be avoided in patients with increased sensitivity to it.
Option D: May inhibit epithelial migration in partial thickness wound healing
• Silver sulfadiazine inhibits epithelial migration in partial-thickness wound.
• It can delay healing.

Solution for Question 19:


Option A: Mafenide acetate
• Mafenide acetate - It is a Broad-spectrum antibacterial agent with an excellent eschar penetration
Topical antimicrobials:
It is the most commonly used topical antibiotic
Broad antibacterial spectrum
Painless on application
Fair to poor eschar penetration
No metabolic side effects
Mild inhibition of epithelialization
May leave black tattoos from silver ion
Transient leukopenia may develop in 3 to 5 days and resolve spontaneously.
Destroys skin grafts and is contraindicated on burns or donor sites in proximity to newly grafted areas
Painful on application
Broad-spectrum antibacterial
Particularly useful against resistant Pseudomonas and Enterococcus species
Reserved for small full-thickness injuries
It is an excellent antimicrobial for fresh skin grafts
Excellent eschar penetration
Broad-spectrum antibacterial and antifungal
Poor eschar penetration
Stains contacted areas

Page 21

1455
Leeches sodium from wounds
Option B: Neomycin
• It is a petroleum-based antimicrobial ointment
• It is commonly used for the treatment of facial burns, graft sites, healing donor sites, and small
partial-thickness burns
• Easy to apply
• Painless
• Has an antimicrobial spectrum not as wide as silver sulfadiazine
• However, Mafenide acetate is preferable for eschars due to its good penetration.
Option C: Silver nitrate
• Silver nitrate can be used for burns as an anti-infective agent. However, it is not as good for eschars
as mafenide acetate is.
Option D: Silver sulfadiazine
• Silver sulfadiazine is used for burns to prevent bacterial infections.
• Silver sulfadiazine has a wide spectrum of bactericidal activity against both gram-positive and
gram-negative organisms.
• However, it is not the best choice for eschars

Solution for Question 20:


Option D: Electrosurgical equipment
• This patient most likely sustained a thermal injury due to electrosurgery.
• Fire ignition is a relatively rarely encountered complication of the surgery. However, fire ignition can
seriously damage the patients, medical staff, and surgical instruments and can result even in casualties
in severe cases.
• The three components of an operating room can give rise to fire ignition.
• These components, which are present in virtually all surgical procedures, include: An oxidizer
(oxygen, nitrous oxide) An ignition source Fuel.
• An oxidizer (oxygen, nitrous oxide)
• An ignition source
• Fuel.
• The Emergency Care Research Institute (ECRI) performed an analysis of case reports about the fire
ignition events in the operating room, which showed that the most common ignition sources were
electrosurgical instruments (68%) & lasers (13%).
• Further, the most common fire location is the airway (35%), head or face (28%), & elsewhere on or
inside the patient (38%). An oxygen-enriched atmosphere contributed to (74%) of all cases.
• Surgical fires are preventable by adopting the following preventive measures:
• Practizing the fire safety measures via a team approach

Page 22

1456
• Minimizing or eliminating enriched oxygen delivery is fundamental
• Involving a comprehensive fire safety program
• An oxidizer (oxygen, nitrous oxide)
• An ignition source
• Fuel.
Option A: Argon beam coagulation
• ECRI's analysis of case reports shows that the most common ignition source is electrosurgical
instruments, responsible for 68% of surgical fires, not argon beam coagulation.
Option B: Lasers
• Lasers are not the most common reason for surgical fires.
Option C: Fiber optic illumination
• Fiber optic illumination is not the most common reason for surgical fires.

Page 23

1457
Plastic Surgery and Skin Lesions
1. A 50-year-old patient with a history of a deep burn injury on the forearm undergoes a split-thickness
skin grafting procedure for wound closure. During the recovery period, the patient develops localized
swelling, discomfort, and a collection of fluid underneath the graft site. The surgeon suspects a
common complication related to the grafting procedure. What is the most likely complication in this
case?
(or)
What is the likely issue of a forearm burn develops swelling, discomfort, and fluid accumulation
post-skin grafting?
A. Infection
B. Mechanical shear stress
C. Hematoma formation
D. Graft immobilization
----------------------------------------
2. A 30-year-old patient is admitted to the burn unit with extensive burn injuries to the chest and upper
limbs. The treating team decides to use a specific type of skin grafting technique that allows for
resurfacing a larger area and promotes wound healing. Tiny splits are made in the skin graft to stretch it
over a wider surface. The fluid egresses from these splits, and the graft adheres well to the wound bed
while the tiny splits eventually re-epithelialize from the underlying skin. What is the name of this grafting
technique?
(or)
What's the name of the skin grafting technique involving tiny splits in the graft to cover a larger area and
promote wound healing, where fluid egresses and the graft adheres well before the splits
re-epithelialize?
A. Full thickness skin grafting
B. Composite skin grafting
C. Meshed skin grafting
D. Microvascular free flap
----------------------------------------
3. What complication is important to consider when discussing the TRAM flap technique for breast
reconstruction?
(or)
What complication is important to consider when discussing the TRAM flap technique for breast
reconstruction?
A. Infection at the graft site
B. Graft rejection
C. Hernia in the abdominal wall
D. Necrosis of the breast tissue
----------------------------------------

1458
4. A 45-year-old patient presents with a non-healing ulcer on the lateral aspect of his lower leg. The
ulcer has punched-out edges and is surrounded by an area of induration. The patient reports
intermittent claudication and rest pain in the affected limb. On examination, the pedal pulses are weak,
and the skin appears cool. Which type of ulcer is most likely responsible for the presentation?
(or)
What type of ulcer is likely causing a non-healing wound with punched-out edges, surrounded by
induration, experiencing intermittent claudication and weak pedal pulses with cool skin on examination?
A. Healing ulcer
B. Ischemic ulcer
C. Varicose ulcer
D. Tuberculosis ulcer
----------------------------------------
5. A 40-year-old patient presents with a non-healing ulcer on the forearm. The ulcer developed over a
scar from a previous traumatic injury that occurred over a decade ago. The patient mentions that the
ulcer has been growing very slowly and is painless. What treatment option is typically recommended for
this condition?
(or)
What's the usual treatment for a painless, slow-growing ulcer on the forearm which developed within a
scar from a previous injury?
A. Topical antibiotics
B. Biopsy only
C. Surgical excision followed by flap cover
D. Aggressive debridement
----------------------------------------
6. A 60-year-old patient presents with a slowly growing, ulcerated lesion on their lower lip. The lesion
has everted edges and an indurated base. The patient mentions a history of chronic sun exposure due
to outdoor activities. Biopsy confirms the diagnosis of squamous cell carcinoma (SCC). What is the
most common causative factor for the development of SCC in this patient?
(or)
What's the primary cause of squamous cell carcinoma with a slowly growing, ulcerated lesion on the
lower lip, everted edges, and a history of chronic sun exposure?
A. Tobacco use
B. Chronic alcohol consumption
C. HPV infection
D. Sunlight exposure
----------------------------------------
7. A 24-year-old male patient presented to the dermatology department with a history of rapidly growing
skin lesions over the scalp. The lesion started around three months ago and rapidly increased to the
present size of 5 cm in diameter. A punch biopsy of the lesion showed features suggestive of Kaposi's
sarcoma with advice given for excision biopsy. The lesion was excised with the inclusion of a 1-cm
margin of clinically normal tissue. After excision, a circular defect of around 7 cm diameter was formed

Page 2

1459
on the scalp. Which technique should be used to close this defect in this patient?
(or)
What technique should be used to close a 7 cm diameter defect on the scalp of a patient after excising
a lesion suspected to be Kaposi's sarcoma?
A. Primary simple closure
B. Split skin grafting
C. Secondary closure
D. Local flaps
----------------------------------------
8. What is the most appropriate treatment option for managing the skin lesion of a 2-year-old boy with a
purple-pink patch extending from the right cheek to the right eyelid, with a firm right eyeball, and
cupping of the right optic disc on fundoscopy?
(or)
A 2-year-old boy was brought to the emergency department because of a 5-minute episode of
repetitive, involuntary, twitching movements of his left arm 1 hour ago. His parents say that he began to
stand with support at 18 months and has recently started to walk with support. He speaks in syllables.
Examination shows a purple-pink patch over the right cheek that extends to the right eyelid. The right
eyeball is firm. Fundoscopy showed cupping of the right optic disc. Which of the following is the most
appropriate treatment option for managing the skin lesion of this patient?
A. Radiotherapy
B. Tattooing
C. Excision with skin grafting
D. Pulsed dye laser
----------------------------------------
9. A 36-year-old male patient complains of prolonged bleeding from an ulcer over the right lower limb.
The daily dressings with non-adherent paraffin gas patients have been performed since then, but the
ulcer did not heal. Hence, he undergoes debridement and Thiersch grafting of the right leg ulcer under
spinal anesthesia. Which of the following is the advantage of the type of graft used in this patient?
(or)
What is the advantage of using Thiersch grafting for the debridement and grafting?
A. Large-size grafts can be taken
B. Consist of the epidermis only
C. The graft can not be taken from the upper arm, flexor aspect of the forearm, and abdominal wall
D. The is not affected by the hematoma
----------------------------------------
10. A 32-year-old woman is brought to the emergency department to evaluate for burn injuries. There
are two tender, erythemas without blisters over a 5 x 6 cm area of the left abdomen and a 3 x 2 cm area
of the left anterior thigh. There is also an area of white, leathery skin and tissue necrosis encircling the
right upper extremity just proximal to the elbow. The physician recommends skin grafting to prevent
future complications of the burn. What is the most likely disadvantage of using a mesh skin graft?

Page 3

1460
(or)
What is the most likely disadvantage of using a mesh skin graft?
A. They do not permit coverage of large areas
B. They do not allow regression of fluid collections under the graft
C. They result in poor cosmetic appearance and high secondary contraction
D. The fenestrations in meshed graft do not re-epithelialize by secondary intention from the surrounding
graft skin
----------------------------------------
11. Which of the following process is responsible for the survival of the skin graft within 48 hours of the
grafting?
A. Amount of saline in graft
B. Plasma imbibition
C. Inosculation
D. Revascularization
----------------------------------------
12. A 48-year-old male comes to the physician 3 months after noticing skin changes on his scalp. When
he scrapes off the crust of the lesion, it reappears after a few days. Occasionally, his scalp itches or he
notices a burning sensation. He had a mole removed on his right forearm 5 years ago. He is a retired
postman. Examination shows a single rough patch on his scalp. A photograph is shown. Which of the
following is the most likely diagnosis?
(or)
What is the most likely diagnosis of scalp skin changes that reappear after scraping off the crust of the
lesion?

A. Seborrheic keratosis
B. Keratoacanthoma
C. Actinic keratosis
D. Bowen disease
----------------------------------------
13. What is the most appropriate next step in management of a 1-cm (0.4-in), flesh-colored, nodular,
non-tender lesion with rolled borders after confirming the diagnosis through a shave biopsy?

Page 4

1461
(or)
A 73-year-old male patient presents to the physician because of a lesion on his eyelid for six months.
The lesion is not painful or pruritic. He initially dismissed it as a "skin tag," but the lesion has increased
in size over the past three months. He retired three years ago from his job as a construction contractor.
Examination shows a 1-cm (0.4-in) flesh-colored, nodular, non-tender lesion with rolled borders. There
is no lymphadenopathy. Which of the following is the most appropriate next step in management?
A. Wide local excision
B. Cryotherapy
C. Localised radiotherapy
D. Mohs micrographic surgery
----------------------------------------
14. A 50-year-old male presents to the physician because of a skin lesion on his nose. The lesion has
gradually increased since it first appeared 11 months ago. He is a farmer and lives together with his
wife. Examination of the skin shows a nontender lesion at the right medial canthus. An image of the
lesion is shown. Which of the following is the most likely diagnosis in this patient?
(or)
What is the most likely diagnosis for a nontender lesion at the right medial canthus, as depicted in the
image?

A. Basal cell carcinoma


B. Squamous cell carcinoma
C. Actinic keratosis
D. Keratoacanthoma
----------------------------------------
15. What characteristic feature would be most likely found on histopathological examination of a linear
basal cell carcinoma?
(or)
What characteristic feature would be most likely found on histopathological examination of a
79-year-old woman with a diagnosis of linear basal cell carcinoma?

Page 5

1462
A. Keratin pearls
B. Foam cells
C. Nuclear palisading
D. Psammoma bodies
----------------------------------------
16. A 5-month-old girl is brought to the physician because of a red lesion on her scalp that was first
noticed 2 months ago. The lesion has been slowly increasing in size. It is not associated with pain or
pruritus. Examination shows a solitary, soft lesion on the vertex of the scalp that blanches with
pressure. A photograph of the lesion is shown in the image: Which of the following is the most
appropriate next step in management?

A. Laser ablation
B. Intralesional bevacizumab
C. Topical clobetasol
D. Reassurance and follow-up
----------------------------------------
17. A 43-year-old male patient presents to the doctor with the complaint of a slow-growing mass on his
back. The mass is mobile and firm. It is painless and not associated with any discharge. There is no
crusting on the overlying skin. On examination, a dark-coloured punctum may be seen, as shown in the
image below: The rest of the review of the systems is unremarkable. Which of the following is the most
likely diagnosis in this patient?
(or)

Page 6

1463
What is the most likely diagnosis for a slow-growing mass on his back, showing a dark-colored
punctum, as seen in the image?

A. Basal Cell carcinoma


B. Sebaceous cyst
C. Lipoma
D. Squamous cell carcinoma
----------------------------------------
18. What is the most likely advantage of using modified extended Karapandzic flaps to rebuild the lower
lip and tongue mucosa flap in squamous cell carcinoma of the full lower lip?
(or)
A 65-year-old female presented with squamous cell carcinoma of her lower lip. The lesion involved the
full lower lip and partly skin of the chin. Under general anesthesia, an extended peripheral resection is
done. Then, modified extended Karapandzic flaps were used to rebuild the lower lip and tongue
mucosa flap to rebuild the vermilion. Which of the following is the most likely advantageous point of the
Karapandzic flap?
A. Two-stage procedure
B. It can cause Macrostomia
C. It is a neuromuscular preserving flap
D. All of the above
----------------------------------------
19. A 72-year-old man comes to the physician for evaluation of multiple red spots on his trunk. He first
noticed these several months ago, and some appear to have increased in size. One day ago, he
scratched one of these spots, which bled for several minutes. A photograph of the lesions is shown in
the image: Among the following clinical entities, which is this patient's most likely possible diagnosis?
(or)
What is the likely diagnosis in a patient with multiple red spots on his trunk which bleeds on scratching?

Page 7

1464
A. Bacillary angiomatosis
B. Cherry Hemangioma
C. Strawberry angioma
D. Amelanotic melanoma
----------------------------------------
20. Which of the following locations of lipoma is most likely associated with malignancy?
(or)
Which of the following locations of lipoma is most likely associated with malignancy?
A. Subcutaneous
B. Sub-aponeurotic
C. Retroperitoneal
D. Intermuscular
----------------------------------------
21. What is the most appropriate term to describe the following traumatic injury?
(or)
A 36-year-old man is brought to the emergency department one hour after sustaining a traumatic injury
during a high-speed motor vehicle collision. There are multiple bruises over the face, trunk, and right
upper and lower extremities. There is swelling of the right elbow and wrist. The right lower extremity is
shorter than the left lower extremity. There are multiple traumatic lacerations on both lower limbs, as
shown in the image below: Which of the following is the most appropriate term to describe the traumatic
injury of this patient shown in the picture?

Page 8

1465
A. Separation of skin
B. Separation of skin and subcutaneous tissue
C. Separation of fascia exposing tendons
D. Separation of tendon exposing the bone
----------------------------------------
22. A 64-year-old woman comes to the physician because of a 3-day history of fever, redness, and
swelling of her right leg. She says that over the past 24 hours, her leg has become increasingly painful,
and the redness has increased in size. Her temperature is 39.3°C (102.7°F), her pulse is 103/min, and
her blood pressure is 138/90 mm Hg. Examination shows an area of diffuse erythema and swelling over
the anterior right lower leg; it is warm and tender to the touch. The examination findings are shown in
the image below: There is swelling of the right inguinal lymph nodes. Pedal pulses are palpable
bilaterally. Which of the following is the most likely diagnosis in this patient?
(or)
What is the likely diagnosis in the following patient with an area of diffuse erythema and swelling over
the anterior right lower leg and swollen right inguinal lymph nodes?

A. Wet gangrene
B. Dry gangrene
C. Frost bite
D. Ainhum
----------------------------------------
23. What is the antibiotic of choice for cellulitis?
(or)
A 69-year-old man comes to the physician because of a 2-day history of redness, swelling, and pain in
the right leg with fever, chills, and nausea. He reports seeing liquid oozing from the affected area on his
right leg. His temperature is 38.7°C (102°F), his pulse is 106/min, and his blood pressure is 152/94 mm
Hg. Examination of the right lower leg shows a large area of erythema with poorly-demarcated borders
and purulent drainage. The area is fluctuant, warm, and tender to the touch. In addition to incision,
drainage, and taking cultures from the wound, which of the following is the most appropriate next step
in management?
A. Oral clindamycin
B. Intravenous vancomycin
C. Surgical debridement

Page 9

1466
D. Oral dicloxacillin
----------------------------------------
24. A 7-year-old girl is brought to the physician by her mother because of skin bumps on her scalp. Her
mother took her to the doctor when she first noticed them 2 years ago. Examining the scalp shows
hard, freely mobile, and slow-growing mass. A photograph of the lesions is shown below: Which of the
following is the most appropriate clinical diagnosis for this patient?
(or)
Which of the following is the most appropriate clinical diagnosis for this patient with a hard, freely
mobile, and slow-growing mass on the scalp. A photograph of the lesions is shown below.

A. Cock's Peculiar Tumor


B. Pott's Puffy Tumor
C. Pilomatrixoma
D. Cylindroma
----------------------------------------
25. A 65-year-old female patient presents to the outpatient department of surgery with complaints of
painless swelling below her tongue with ulceration. On examination, a 2*3 cm painless growth is
present around the floor of the mouth. The biopsy and the histopathology report reveal a squamous cell
carcinoma of the oral cavity. The physician recommends a surgical excision for the removal of the
mass. After the surgery, a tissue defect is found in her neck at the site of the excision of the mass, so
the surgeon decides to repair it using a flap, as shown in the image below: Which of the following type
of flap is used in the reconstructive surgery of this patient?
(or)
Which type of flap is used in reconstructive surgery of the oral cavity?

Page 10

1467
A. Pectoralis myocutaneous flap
B. TRAM flap
C. Deltopectoral flap
D. Latissimus Dorsi flap
----------------------------------------
26. Why is this instrument used for wound reconstruction used?
(or)
A 66-year-old man comes to the clinic because of a 5-week history of a rapidly growing skin lesion near
the right ear. A cutaneous squamous cell carcinoma diagnosis is established, and the patient
undergoes surgery for the excision of the tumour. After the surgery, a defect of a diameter of 5 cm is
seen. The surgeon decides to reconstruct the wound by using a tissue flap. During this, he uses an
instrument, as shown in the image below: Which of the following is the most likely purpose of this
instrument?

A. Used for separating periosteum from under the surface of the rib
B. Used for getting split-thickness graft & Full-thickness skin graft
C. It is a rib spreader utilized in cardiothoracic procedures
D. Used to dissect superior thyroid pedicle
----------------------------------------
27. What is a characteristic feature of an axial flap used for breast tissue reconstruction in a 47-year-old
nulliparous woman undergoing a modified radical mastectomy?
(or)
An axial flap is characterized by which of the following?
A. It carries its vessels within it
B. It is kept in the limb
C. It is a transverse flap
D. It carries its nerve in it
----------------------------------------
28. Which of the tissues is included in a myocutaneous flap?
(or)

Page 11

1468
Which of the tissues is included in a myocutaneous flap?
A. Muscle only
B. Muscle and vascular pedicle
C. Muscle and skin
D. Skin, muscle and vascular pedicle
----------------------------------------
29. A 31-year-old woman comes to the physician because of white lesions in her mouth that she first
noticed five days ago. The lesions are not painful or itchy. She has asthma treated with theophylline,
inhaled β-adrenergic agonists, and corticosteroids. She smokes half a pack of cigarettes daily. Oral
examination shows white plaques on the buccal mucosa and hard palate that are firmly adherent and
cannot be scraped off. She has no cervical or axillary lymphadenopathy. Which of the following is the
most appropriate next step in management?
(or)
What is the most appropriate next step in managing a white plaques on the buccal mucosa and hard
palate that is firmly adherent and cannot be scraped off and has no cervical or axillary
lymphadenopathy?
A. Topical nystatin
B. Biopsy of the lesions
C. Topical corticosteroids
D. Surgical excision of the lesions
----------------------------------------
30. A 65-year-old Asian woman comes to hospital complaining of occasional night sweats during the
past two months.There is a nontender skin lesion near the right large toenail. A photograph of the
lesion is shown below: The examination shows no other abnormalities. Which of the following is the
most likely diagnosis?
(or)
What is the most likely diagnosis for a nontender skin lesion near the right large toenail in an Asian
woman, as shown in the image?

A. Squamous cell carcinoma


B. Traumatic subungual hemorrhage
C. Malignant melanoma

Page 12

1469
D. Onychomycosis
----------------------------------------
31. A 40-year-old woman comes to the physician because she complained of a "weird-looking" dark
skin lesion present overher hand 3 months ago. She has not seen any changes in the lesion since then.
The lesion is not painful or pruritic. There is an elevated, symmetrical skin lesion with uniform borders
on the medial aspect of the patient's left hand. The lesion measures 5 mm in diameter and is dark
brown. Dermoscopy shows a brown pigment network with scattered grey and reddish spots. Which of
the following is the strongest indicator for performing an excisional biopsy?
(or)
What is the strongest indicator for performing an excisional biopsy in an elevated, symmetrical skin
lesion with uniform borders on the left hand, showing a brown pigment network with scattered grey and
reddish spots on dermoscopy?
A. Multiple colours of the lesion
B. 5 mm diameter of the lesion
C. No change in the lesion over the past 3 months
D. Uniform borders of the lesion
----------------------------------------
32. Which of the following is a characteristic feature of Marjolin's ulcer?
(or)
Which of the following is a characteristic feature of Marjolin's ulcer?
A. Marjolin's ulcer is a slow-growing ulcer and is not associated with secondary deposits in regional
lymph node
B. It is a high-grade Squamous cell carcinoma
C. It is a very painful ulcer
D. It is an extremely radiosensitive ulcer
----------------------------------------
33. A 32-year-old woman is posted for reconstruction surgery after being diagnosed and treated with a
malignant lesion on her nose. The surgeon takes the tissue from an adjacent area and applies it to the
area of the defect, as shown in the image below: Which of the following types of flaps is used?
(or)
Which type of flap is typically used when a surgeon takes tissue from an adjacent area and applies it to
the defect area during reconstruction surgery as shown below?

Page 13

1470
A. Bilobed flap
B. Advancement flap
C. Rhomboid flap
D. Z-plasty
----------------------------------------
34. A 28-year-old male complained of off-and-on pain, mild swelling, and pus discharge at the natal
cleft for the last 3 months. He also has a history of episodic fever and repeated abscesses that had
burst spontaneously in the recent past. By occupation, he is a jeep driver. On physical examination, a
small indurated swelling was found at the top of the buttock in the midline overlying the coccyx. Which
flap-based procedure is used in the surgery done for the management of this disease?
(or)
What flap is used in a jeep driver with pus discharge, small indurated swelling was found at the top of
the buttock in the midline overlying the coccyx?
A. Rhomboid flap
B. Circular flap
C. Free flap
D. Rotational flap
----------------------------------------
35. A 55-year-old female patient diagnosed with invasive intraductal carcinoma is scheduled to undergo
an elective mastectomy. Considering this reconstructive procedure which of the following local flap is
most probably used?
(or)
Which local flap is most likely to be used in reconstructive surgery following an elective mastectomy
after invasive intraductal carcinoma?
A. Rotation flap
B. Bilobed flap
C. Z-plasty
D. Rhomboid flap
----------------------------------------

Page 14

1471
36. A 45-year-old male patient has a segmental fracture of the right fibular shaft. After stabilization and
wound debridement, the patient is taken to the theatre, where vacuum-assisted negative pressure is
applied. Which of the following is associated with this procedure?
(or)
Which of the following is associated with negative pressure wound therapy?
A. Due to negative pressure, bacterial counts decrease, and cell proliferation increases in the wound
B. Continuous pressure of +125 mm hg is applied.
C. It causes interstitial edema, thereby increasing blood flow and better healing.
D. Suitable to be used over malignancy if present over wound.
----------------------------------------
37. A 48-year-old man with a history of diabetes mellitus presents to the emergency department with
severe pain and swelling in his right lower limb. Examination findings are shown in the image. There is
leukocytosis and elevated creatine kinase. What is the most appropriate treatment?
(or)
What is the most appropriate treatment for a condition causing leukocytosis, elevated creatine kinase,
and the following findings in a patient with diabetes?

A. Immediate limb amputation


B. Hyperbaric oxygen therapy
C. Intravenous penicillin and debridement of affected tissue
D. Intravenous fluoroquinolones
----------------------------------------
38. A 51-year-old woman presents with malaise and a painful swelling on her back. On examination,
there is an abscess with multiple draining sinuses and an infected mass lying beneath. The lesion
appears red with a whitish-yellow center. What is the causative agent?
(or)
Which of the following causes a painful abscess that has a whitish-yellow center and multiple draining
sinuses?
A. Streptococcus pyogenes
B. Pseudomonas aeruginosa
C. Candida albicans

Page 15

1472
D. Staphylococcus aureus
----------------------------------------
39. A 55-year-old man presents with severe pain in the left lower limb. On local examination, the wound
is edematous with a woody hard texture of subcutaneous tissue. Laboratory investigations show
leukocytosis with a white blood cell count of 17,000 cells/mm³. What is the best next step in treatment?
(or)
What is the treatment for an infection that causes an edematous wound with a woody hard texture of
subcutaneous tissue in the lower limb?
A. IV fluids and debridement
B. IV fluids + debridement + IV antibiotics + supportive care
C. Hyperbaric oxygen therapy + supportive therapy
D. IV fluids + IV antibiotics
----------------------------------------

Correct Answers
Question Correct Answer

Question 1 3
Question 2 3
Question 3 3
Question 4 2
Question 5 3
Question 6 4
Question 7 4
Question 8 4
Question 9 1
Question 10 3
Question 11 2
Question 12 4
Question 13 4
Question 14 1
Question 15 3
Question 16 4
Question 17 2
Question 18 3
Question 19 2
Question 20 3

Page 16

1473
Question 21 2
Question 22 1
Question 23 2
Question 24 3
Question 25 3
Question 26 2
Question 27 1
Question 28 4
Question 29 2
Question 30 3
Question 31 1
Question 32 1
Question 33 1
Question 34 1
Question 35 1
Question 36 1
Question 37 3
Question 38 4
Question 39 2

Solution for Question 1:


Correct Option C: Hematoma formation
• In this case, the patient's symptoms of localized swelling, discomfort, and a collection of fluid
underneath the graft site are indicative of hematoma formation, which is a common complication
related to the skin grafting procedure.
• Hematoma occurs when there is an accumulation of blood within the tissue, leading to a raised, tense
area. This can disrupt the healing process and compromise graft survival. Hematoma formation often
presents as a bluish discoloration due to the presence of trapped blood.
• Complications of Skin Grafting: Infection Mechanical/Shear stress Hematoma/Seroma formation
• Infection
• Mechanical/Shear stress
• Hematoma/Seroma formation
• Absolute Contraindication for Skin Grafting: Presence of Beta-hemolytic streptococci infection
(produces hyaluronidase, destroys split thickness skin grafts completely)
• Presence of Beta-hemolytic streptococci infection (produces hyaluronidase, destroys split thickness
skin grafts completely)
• Graft Immobilization Techniques: Bolster dressing Light compression wraps Vacuum-assisted closure
devices

Page 17

1474
• Bolster dressing
• Light compression wraps
• Vacuum-assisted closure devices
• Preventing Hematoma/Seroma Formation: Pie crusting technique (Multiple stab incisions in graft)
Small outlets created for fluid drainage to prevent complications
• Pie crusting technique (Multiple stab incisions in graft)
• Small outlets created for fluid drainage to prevent complications
Complications of Skin Grafting:
• Infection
• Mechanical/Shear stress
• Hematoma/Seroma formation
Absolute Contraindication for Skin Grafting:
• Presence of Beta-hemolytic streptococci infection (produces hyaluronidase, destroys split thickness
skin grafts completely)
Graft Immobilization Techniques:
• Bolster dressing
• Light compression wraps
• Vacuum-assisted closure devices
Preventing Hematoma/Seroma Formation:
• Pie crusting technique (Multiple stab incisions in graft)
• Small outlets created for fluid drainage to prevent complications
Incorrect Options:
Option A - Infection: While infection is a potential complication of any surgical procedure, the symptom
s described in the case (localized swelling, discomfort, fluid collection) are more consistent with hemat
oma formation. Infections usually involve redness, warmth, tenderness, and potentially fever.
Option B - Mechanical shear stress: Mechanical shear stress typically refers to forces applied to the gr
aft site during movement or dressing changes. While it can contribute to graft failure, it wouldn't likely c
ause localized swelling and fluid collection as described. Moreover, shear stress-related issues are usu
ally more chronic rather than occurring in the immediate recovery period.
Choice D) Graft immobilization: Graft immobilization refers to the need to prevent movement or displac
ement of the graft during the initial healing period. While graft immobilization is important to ensure graf
t adherence, it's not the cause of the symptoms described in the case, such as localized swelling and fl
uid collection.

Solution for Question 2:


Correct Option C- Meshed skin grafting
• In this case, the described technique involves making tiny splits in the skin graft to expand its surface
area, facilitating its application over a larger wound bed. This technique, known as meshed skin

Page 18

1475
grafting, is commonly used in cases of extensive burn injuries and significant burns.
• The advantages of meshed skin grafting include its ability to cover larger areas, promote fluid egress,
and provide contour matching of the wound bed.
Incorrect Options:
Option A – Full-thickness skin grafting: This technique involves harvesting the entire thickness of the sk
in, including both the epidermis and the dermis, from a donor site and grafting it onto the recipient site.
The description doesn't match the process of making tiny splits to expand the graft's surface area, as s
een in meshed skin grafting.
Option B – Composite skin grafting: Composite skin grafting involves grafting the skin and other tissue l
ayers, such as fat and muscle. This technique, which focuses on making tiny splits to expand the graft'
s surface area, is not mentioned in the case.
Option D – Microvascular free flap: A microvascular free flap involves transferring a section of tissue, in
cluding blood vessels, from one body area to another using microsurgery techniques. This technique is
more complex and requires tissue transplantation with its blood supply, which is not described in the c
ase.

Solution for Question 3:


Correct Option C - Hernia in the abdominal wall
• The TRAM flap (Transverse Rectus Abdominis Myocutaneous flap) technique, which is commonly
used for breast reconstruction, involves the transverse abdominal muscle flap based on the epigastric
vessels. It is either superior pedicle based on the superior epigastric vessels or inferior pedicle based
on the inferior epigastric vessels
• However, it's essential to consider that the TRAM flap, can lead to complications such as hernia in the
abdominal wall. This is due to the disruption of the abdominal wall musculature during flap harvesting,
which can weaken the abdominal wall and potentially lead to herniation.
• The Supercharged TRAM Flap, which combines superior and inferior epigastric vessels, doesn't
eliminate this concern.
Incorrect Options:
Option A – Infection at the graft site: While infection is a potential complication of surgical procedures, t
he TRAM flap technique's primary concern is not infection. The focus is more on the possible complicat
ions associated with the abdominal tissue transfer and the effect on the abdominal wall.
Option B – Graft rejection: Graft rejection is not a significant concern in autologous tissue transfer proc
edures like the TRAM flap, as the tissue is taken from the patient's body, minimizing the risk of rejectio
n.
Option D - Necrosis of the breast tissue: Necrosis of the breast tissue is not a primary concern with the
TRAM flap procedure. However, flap-related issues like poor blood supply to the flap can potentially lea
d to tissue necrosis if not appropriately managed.

Solution for Question 4:


Correct Option B - Ischemic ulcer

Page 19

1476
• An ischemic ulcer is the most likely responsible for the presentation described.
• Characteristics: Punched-out edges Surrounding induration Weak pedal pulses Cool skin Symptoms
of intermittent claudication and rest pain indicate inadequate blood supply to the affected limb, leading
to tissue ischemia and ulcer formation.
• Punched-out edges
• Surrounding induration
• Weak pedal pulses
• Cool skin
• Symptoms of intermittent claudication and rest pain indicate inadequate blood supply to the affected
limb, leading to tissue ischemia and ulcer formation.
• Punched-out edges
• Surrounding induration
• Weak pedal pulses
• Cool skin
• Symptoms of intermittent claudication and rest pain indicate inadequate blood supply to the affected
limb, leading to tissue ischemia and ulcer formation.
Type of Ulcer
Description
Characteristics
Healing Ulcer
Ulcer showing signs of progression towards wound closure
Shows improvement in size and appearance
Arterial / Ischemic Ulcer
Develops due to poor arterial circulation, inadequate blood supply
Punched-out edges, surrounding induration
Varicose Ulcer
Result from venous insufficiency, typically around lower legs
Purulent discharge, granulation tissue, sloping edges
Tuberculosis Ulcer
Rare type, can present with undermined edges
May be associated with systemic symptoms
Squamous Cell Carcinoma Ulcer
Associated with malignancy, irregular edges, induration
More commonly seen in sun-exposed
Incorrect Option:
Option A – Healing ulcer: This choice doesn't align with the description of a non-healing ulcer with punc
hed-out edges and surrounding induration. Healing ulcers would typically show signs of progress towar

Page 20

1477
d closure, not present with the described features.
Option C - Varicose ulcer: Varicose ulcers are more often associated with purulent discharge, granulati
on tissue, and sloping edges, and they typically occur in the lower legs due to venous insufficiency. Th
e described features in the case are not consistent with varicose ulcers.
Option D - Tuberculosis ulcer: Tuberculosis ulcers are uncommon and typically have undermined edge
s. Additionally, the case does not mention any systemic symptoms that would suggest tuberculosis as t
he cause.

Solution for Question 5:


Correct Option C - Surgical excision followed by flap cover
• The presentation of a non-healing ulcer within a scar from a previous traumatic injury, growing
slowly and being painless, is consistent with Marjolin's ulcer.
• Marjolin's ulcer is a low-grade squamous cell carcinoma arising from long-standing scar tissue.
• The recommended treatment for Marjolin's ulcer is surgical excision followed by a flap cover to
remove the malignant tissue and ensure proper wound closure.
• This approach helps prevent recurrence and addresses the potential complications associated with
untreated malignancy.
Incorrect options:
Option A – Topical antibiotics: Topical antibiotics are not the primary treatment for malignant lesions lik
e Marjolin's ulcers. Surgical excision with flap cover is required to address the underlying malignancy.
Option B – Biopsy only: While a biopsy may be necessary to confirm the diagnosis, biopsy alone is not
the recommended treatment for Marjolin's ulcer. Complete removal of the malignant tissue through sur
gical excision is crucial.
Option D – Aggressive debridement: While debridement may be a part of the surgical procedure, aggre
ssive debridement alone is not the primary treatment for Marjolin's ulcer. Surgical excision with a
flap cover is the standard for managing this condition.

Solution for Question 6:


Correct answer D - Sunlight exposure
• The clinical features described, including a slowly growing ulcerated lesion with everted edges and an
indurated base, along with a history of chronic sun exposure, are indicative of squamous cell carcinoma
(SCC) of the lower lip.
• The most common causative factor for the development of SCC is sunlight exposure, particularly in
areas of the skin that are exposed to sunlight.
• SCC is the 2nd most common skin malignancy, often arising from the prickle cell layer of the
epidermis due to factors like chronic sunlight exposure.
• It typically presents as nodules or ulcers with everted edges and indurated bases, commonly in areas
exposed to sunlight.

Page 21

1478
• Chronic sun exposure is the most common causative factor, particularly for SCC on the lower lip,
ears, cheeks, and back of hands.
• Treatment options include wide local excision with 2 cm margin for small or non-invasive SCC, and
Moh's micrographic surgery for larger, aggressive, or recurrent tumors involving vital or cosmetic areas.
Incorrect Choices:
Option A – Tobacco use: While tobacco use, particularly smoking, is a major risk factor for various type
s of cancers, including lung and oral cavity cancers, it is not the primary causative factor for SCC of the
lower lip. SCC of the lower lip is more strongly associated with chronic sun exposure, especially in the
areas exposed to sunlight.
Option B – Chronic alcohol consumption: Chronic alcohol consumption is also a risk factor for certain ty
pes of cancers, particularly those affecting the upper aerodigestive tract. However, it is not the primary
causative factor for SCC of the lower lip. The primary risk factor for SCC of the lower lip remains chroni
c sun exposure.
Option C - HPV infection: Human papillomavirus (HPV) infection is a well-known risk factor for certain t
ypes of cancers, particularly those affecting the genital and oropharyngeal regions. While HPV can con
tribute to some cases of SCC, it is not the primary causative factor for SCC of the lower lip. Chronic su
n exposure remains the primary risk factor in this context.

Solution for Question 7:


Option D: Local flap
• This patient is suffering from Kaposi sarcoma. He needs closure of the defect after the surgical
excision of the tumor. A circular defect of around 7 cm in diameter on the scalp should be closed by a
local flap in this patient.
• A flap is characterized by a tissue transferred from one particular site (donor site) to another (recipient
site).
• During the process of transferring the tissue, its blood supply is maintained. The types of tissue flaps
are: Local flaps Regional flaps Distant flaps
• Local flaps
• Regional flaps
• Distant flaps
• Local flaps are transferred from an area adjacent to the defect.
• Absolute indications for flaps include: Exposed bone Radiated vessel Exposed brain Open joint or
non-biological implant materials Pressure sores at bony prominences This patient with a defect of 7 cm
diameter can be optimally treated with a local flap.
• Exposed bone
• Radiated vessel
• Exposed brain
• Open joint or non-biological implant materials
• Pressure sores at bony prominences
• This patient with a defect of 7 cm diameter can be optimally treated with a local flap.

Page 22

1479
• Local flaps
• Regional flaps
• Distant flaps
• Exposed bone
• Radiated vessel
• Exposed brain
• Open joint or non-biological implant materials
• Pressure sores at bony prominences
• This patient with a defect of 7 cm diameter can be optimally treated with a local flap.
Option A: Primary simple closure
• Primary simple closure is characterized by the type of tissue repair in which the skin is closed at the
end of the surgery. However, it requires adequate soft tissue to avoid excessive wound tension. A
wound closed under absurd tension results in wound dehiscence and an unacceptably wide scar. So, a
wound with larger tissue defect cannot be used via a simple primary closure.
Option B: Split skin grafting
• A split-thickness skin graft (STSG) contains the epidermis and a portion of the dermis and consists
only of the epidermis and dermis.
• Hence, it should not be used in this patient.
Option C: Secondary closure
• The secondary closure is characterized by the wound being left open at the end of surgery and heals
by granulation and contraction. The indications of the secondary closure include the following: Infected
wounds Wounds with implanted foreign bodies Bite wounds that do not meet the criteria for primary
closure Large wounds with irregular edges that cannot be approximated Secondary closure results in
scar formation, so it should be avoided in this patient.
• Infected wounds
• Wounds with implanted foreign bodies
• Bite wounds that do not meet the criteria for primary closure
• Large wounds with irregular edges that cannot be approximated
• Secondary closure results in scar formation, so it should be avoided in this patient.
• Infected wounds
• Wounds with implanted foreign bodies
• Bite wounds that do not meet the criteria for primary closure
• Large wounds with irregular edges that cannot be approximated
• Secondary closure results in scar formation, so it should be avoided in this patient.

Solution for Question 8:


Option D: Pulsed dye laser

Page 23

1480
• This patient is most likely suffering from Sturge-Weber syndrome, a rare congenital vascular disorder.
• Somatic mutations in the GNAQ gene cause Sturge-Weber syndrome. This results in capillary-venous
malformations.
• Capillary-venous malformations affect the face, brain, and eyes, manifesting as the characteristic "port
wine" stain on the forehead and upper eyelid, as shown in the image below:

• The other features are seizures, intellectual and behavioral problems, and visual defects.
• This patient has several characteristics of Sturge-Weber syndrome. Most patients with the disorder
are born with a naevus flammeus (port-wine stain), usually over the trigeminal nerve region.
• The left-sided focal seizures, hypotonia, and absent reflexes are likely due to a right-sided
leptomeningeal angioma, which classically occurs on the same side as the naevus flammeus.
• Vascular malformations within the eye lead to increased intraocular pressure and early-onset
glaucoma.
• Patients with Sturge-Weber syndrome require anticonvulsants for seizure prevention and regular
ophthalmological screening. The naevus flammeus may be treated with laser therapy.
• Excellent results have been obtained with careful and time-consuming treatment with a 585-nm
flashlamp-pumped pulsed dye laser. Treatment sessions can begin in babies, and anesthesia is not
always necessary.
Option A: Radiotherapy
• Radiotherapy is NOT part of standard therapy for the naevus flammeus.
Option B: Tattooing
• Tattooing is NOT part of standard therapy for the naevus flammeus.
Option C: Excision with skin grafting
• Excision with skin grafting is NOT part of standard therapy for the naevus flammeus.

Page 24

1481
Solution for Question 9:
Option A: Large-size graft can be taken
• This patient with a chronic non-healing ulcer underwent debridement and Thiersch grafting.
• Skin reconstruction can be performed using flaps and grafts. A graft is a tissue that contains just the
skin without a blood supply.
• The Thiersch graft is characterized by a partial-thickness skin graft used to re-epithelialize portions of
the affected skin.
• This highly effective technique allows a significantly shorter postoperative recovery time.
• The advantage of Thiersch grafting is that large-size grafts can be taken.
• The following table summarizes the types of various skin grafts and their advantages and
disadvantages:
Type of Graft
Advantages
Disadvantages
Thin Split
Thickness
• Best Survival
• Heals Rapidly
• Least resembles original skin.
• Least resistance to trauma.
• Poor Sensation
• Maximal Secondary Contraction
Thick Split

• More qualifies of normal skin.


• Less Contraction
• Looks better
• Fair Sensation
• Lower graft survival
• Slower healing.
Full Thickness
• Most resembles normal skin.
• Minimal Secondary contraction
• Resistant to trauma
• Good Sensation
• Aesthetically pleasing

Page 25

1482
• Poorest survival.
• The donor site must be closed surgically.
• Donor sites are limited.
Option B: Consists of the epidermis only.
• This statement is incorrect, as a Thiersch graft consists of the epidermis and a variable portion of the
dermis.
Option C: The graft cannot be taken from the upper arm, flexor aspect of the forearm, and abdominal w
all
• This statement is incorrect as a Thiersch graft can be taken from the upper arm, flexor aspect of the
forearm, and abdominal wall
Option D: The is not affected by the hematoma
• This statement is incorrect, as hematoma (or seroma), infection, and movement (shear) are among
the most common causes of skin graft failure.

Solution for Question 10:


Option C: They result in poor cosmetic appearance and high secondary contraction
• This patient with a skin burn needs skin grafting to prevent complications because
deep second degree and full-thickness burns require skin graft surgery. This skin grafting ensures
quick healing and minimal scarring.
• Mesh skin grafts are commonly used after the burn, as seen in this patient. Mesh skin grafts are
characterized by split-thickness or full-thickness skin grafts in which parallel rows of staggered slits
have been cut.
• The mesh skin grafts are advantageous as mesh incisions expand the graft, making it possible to
cover significant defects.
• The mesh incisions in this skin graft provide a route for the drainage of blood or serum. It also
increases the flexibility of the graft.
• However, the disadvantage of the mesh skin graft includes a poor cosmetic appearance and high
secondary contraction after the grafting.
Option A: They do not permit coverage of large areas
• This statement is incorrect, as mesh skin graft permits coverage of large areas.
Option B: They do not allow regression of fluid collections under the graft
• This statement is incorrect, as mesh skin grafts allow regression of fluid collections under the graft
Option D: The fenestrations in the meshed graft do not re-epithelialize by secondary intention from the
surrounding graft skin.
• This statement is incorrect as fenestrations in meshed graft re-epithelialize by secondary intention
from the surrounding graft skin.

Page 26

1483
Solution for Question 11:
Option B: Plasma imbibition
• A skin graft is characterized by a patch of skin surgically removed from one area of the body (donor
site) and transplanted to another location where there is tissue loss (recipient site).
• Skin grafts help repair a wound in patients with severe burns, infections, or wounds
• It is also used in those patients who have had skin removed because of skin cancer.
• The skin grafts cover up wounds and facilitate a faster healing process.
• The stages of skin graft healing are mentioned in the following: Imbibition Inosculation
Revascularization
• Imbibition
• Inosculation
• Revascularization
• Imbibition: Thisis the first stage of the skin graft healing process. It occurs 24 to 48 hours after the
procedure.
• The imbibition stage is characterized by forming a thin film of fibrin and plasma. This film separates
the skin graft from the underlying wound bed.
• This film is essential and provides a moist environment. This thin film of fibrin and plasma is crucial for
the survival of the graft.
• Inosculation: This is the second stage of the process of skin graft healing and occurs around 48 hours
after the procedure when an exemplary vascular network begins to form within the fibrin layer.
• The new capillaries border the surface of the dermis, allowing the transfer of nutrients and oxygen.
• Revascularization is the third and last stage of the skin graft healing process. During this process, new
blood vessels invade the skin graft and restore the skin to its natural appearance.
• The process of skin graft healing is completed in a period of up to one week and is illustrated in the
image below:
• Imbibition
• Inosculation
• Revascularization

Page 27

1484
Option A: Amount of saline in graft
• This is an incorrect option, as the amount of saline is not primarily responsible for the survival of the
skin graft.
Option C: Inosculation
• This is an incorrect option, as inosculation occurs around 48 hours after the procedure when an
acceptable vascular network forms within the fibrin layer.
Option D: Revascularization
• This is an incorrect option, as revascularization is the third and last stage of the skin graft healing
process and does not occur within the initial 48 hours of grafting.

Solution for Question 12:


ANSWER
Option D: Bowen disease
• This patient is most likely suffering from Bowen's disease.
• Bowen disease is characterized by squamous cell carcinoma in situ (SCCIS) of the skin.
• Bowen disease is commonly associated with sun exposure and may appear similar to this patient's
lesion.
• It usually manifests as a solitary lesion rather than the multiple ill-defined, confluent lesions seen on
this patient's scalp, as shown in the image below:

Page 28

1485
• Bowen disease is considered a precancerous skin lesion. It may progress to invasive squamous cell
carcinoma.
• The diagnostic work-up includes clinical examination and biopsy of the lesion.
• The treatment includes pharmacotherapy (5-FU, imiquimod) and surgical excision.
Other
Option A: Seborrheic keratosis
• Seborrheic keratosis is the most common benign skin tumor in older people.
• However, in contrast to this patient's rough, patchy lesions, it manifests as darkly pigmented papules
or plaques that are soft, sharply demarcated, and appear greasy, wax-like, and 'stuck-on' appearance
as shown in the image below:

• These lesions involve the trunk, back of the hands, face and neck, and forearms rather than the scalp.
Option B: Keratoacanthoma

Page 29

1486
• Keratoacanthomas tend to occur in elderly patients with chronic sun exposure. These lesions manifest
as a round erythematous nodule with a central crater that proliferates, usually within weeks to months,
as shown in the image below:

• However, the multiple erythematous lesions with scales on the scalp suggest a different diagnosis in
this patient.
Option C: Actinic keratosis
• A precancerous lesion on sun-exposed skin such as the scalp, face, ears, and dorsal aspects of the
hands or arms characterizes actinic keratosis.
• It predominantly affects light-skinned individuals over 50 years of age.

Page 30

1487
Solution for Question 13:
Option D: Mohs micrographic surgery
• This patient with a nodular, non-tender lesion with rolled borders on the eyelid and occupational sun
exposure is most likely suffering from basal cell carcinoma (BCC).
• It is the most common type of skin cancer.
• Typically, basal cell carcinoma develops in sun-exposed areas as a pearly, indurated, nodular,
non-tender lesion with a rolled border and central ulceration, as illustrated in the image below:

• The subtypes of basal cell carcinoma include nodular basal cell carcinoma and superficial basal cell
carcinoma.
• Nodular basal cell carcinoma mainly involvesthe face, especially the nose.
• The flat eczematous plaque characterizes superficial basal cell carcinoma with a pearly border lesion.
• The superficial basal cell carcinoma variant mainly involves the trunk.

Page 31

1488
• The patient with basal cell carcinoma (BCC) should undergo a full-thickness biopsy of the edge of the
lesion for histopathological confirmation of BCC.
• The standard treatment options include surgical excision with a safety margin and Mohs micrographic
surgery.
• Mohs micrographic surgery is a procedure in which the tumor is removed layer by layer, and each
layer is examined for tumor cells. It is the standard treatment for BCCs not located on the trunk or
extremities.
• It is especially advantageous for functionally or cosmetically sensitive tumor locations such as the
eyelid because it ensures complete tumor removal while sparing unaffected skin.
Option A: Wide local excision
• The wide local excision is a surgical procedure in which the tumor and a small margin of normal tissue
are removed after establishing the diagnosis.
• This technique is mainly used for the treatment of early-stage melanoma.
• Although this procedure may also be used to treat BCCs located on the trunk or extremities, the
location of this patient's BCC requires a different technique that minimizes the removal of normal tissue.
Option B: Cryotherapy
• Cryotherapy can be used for the treatment of superficial BCC.
• However, it is not widely accepted as a first-line therapy because the recurrence rates after
cryotherapy are often higher than in other therapies.
• Further, cryotherapy does not involve microscopic tissue observation, so assessing the extent of
tumor removal is impossible.
Option C: Localised radiotherapy
• Radiotherapy is an alternative treatment for BCC, usually indicated in the case of large tumors or
tumors located in hard-to-reach locations.
• It can also be used if the prior treatment has failed.
• However, it is not the first-line treatment option.

Solution for Question 14:


Option A: Basal cell carcinoma
• This patient is most likely suffering from basal cell carcinoma. Basal cell carcinoma is characterized by
a slow-growing, locally invasive malignant neoplasm of the skin that arises from the basal epithelial
cells.
• It is the most common type of skin cancer.
• Typically, basal cell carcinoma develops in sun-exposed areas as a pearly, indurated, nodular,
non-tender lesion with a rolled border and central ulceration, as illustrated in the image below:

Page 32

1489
• The subtypes of basal cell carcinoma include nodular basal cell carcinoma and superficial basal cell
carcinoma.
• Nodular basal cell carcinoma mainly involves the face, especially the nose.
• Meanwhile, superficial basal cell carcinoma is characterized by a flat eczematous (scaly) plaque with
a pearly border lesion.
• The superficial basal cell carcinoma variant mainly involves the trunk. This variant is illustrated in the
image below:

• The patient with basal cell carcinoma (BCC) should undergo a full-thickness biopsy of the edge of the
lesion for histopathological confirmation of BCC.
• The standard treatment options include surgical excision with a safety margin and Mohs micrographic
surgery.
Option B: Squamous cell carcinoma

Page 33

1490
• Cutaneous squamous cell carcinoma (cSCC) is the second most common type of skin cancer and is
commonly located on the face or neck (the lower lip is a specific location). Sun exposure and smoking
present in this patient are risk factors for SCC. Although this patient also has a lesion involving his face,
SCC has a different appearance: it is typically a hyperkeratotic nodule or ulcer with crusting that does
not heal and bleeds easily. A pearly induration with a rolled border suggests another diagnosis.
Option C: Actinic keratosis
• While actinic keratosis typically affects patients over 50 and is associated with sun exposure, it does
not manifest as pearly nodules surrounding a central ulcerative depression.
• Instead, actinic keratosis presents with dry and scaly patches of skin.
Option D: Keratoacanthoma
• While keratoacanthomas may appear as an erythematous nodule with a central crater, which would
be consistent with this patient's lesion, they do not have a pearly appearance or a rolled border. In
contrast to this patient's lesion, keratoacanthomas typically proliferate over 2–3 months and usually
heal without treatment.

Solution for Question 15:


Option C: Nuclear palisading
• This patient most likely suffers from basal cell carcinoma, a slow-growing, locally invasive malignant
neoplasm of the skin that arises from basal epithelial cells. It is the most common type of skin cancer.
• This tumor typically develops in sun-exposed areas as a pearly, indurated, nodular, nontender lesion
with a rolled border and central ulceration.
• The characteristic features of basal cell carcinoma are:
• Low-grade malignancy.
• More common in fair & dry-skinned people.
• NUCLEAR PALISADING on histology is shown in the image below:

Page 34

1491
Option A: Keratin pearls
• Keratin pearls are found in squamous cell carcinoma.
Option B: Foam cells
• Foam cells are characterized by a type of macrophage that localize to fatty deposits on blood vessel
walls. These are not seen in basal cell carcinoma.
Option D: Psammoma bodies
• Psammoma bodies are characterized by the concentric, intracellular calcium collections found in
certain neoplasms, such as papillary carcinoma of the thyroid, papillary serous endometrial carcinoma,
and meningioma. However, Psammoma bodies are not seen in basal cell carcinoma.

Solution for Question 16:


Option D: Reassurance and follow-up
• This patient is most likely suffering from strawberry hemangiomas.
• Strawberry hemangiomas are common benign vascular tumors in up to 10% of infants and, more
commonly, in girls.
• These lesions usually appear within the first days to months of life. Most of these tumors resolve
spontaneously by age 5–8 without treatment.
• It contains immature vasoformative tissues. There will be eventually intravascular thrombosis, fibrosis
and mast cell infiltration.
• Management comprises active nonintervention, which includes regular monitoring to evaluate the
clinical course of the hemangioma. Active intervention is necessary for complicated hemangiomas,
such as those with a problematic location (e.g., periorbital region, airway). Other indications for active
intervention include ulcerated lesions and tumors that cause significant disfigurement or functional
impairment. In this patient, no further management besides reassurance and follow-up is necessary.
Option A: Laser ablation
• Laser ablation is a treatment option for strawberry hemangiomas that grow rapidly, ulcerate, or
manifest in problematic areas (e.g., periorbital region, airway) and do not respond to the recommended
first-line treatment. Since this patient has a slowly growing strawberry hemangioma that does not cause
significant functional impairment or disfigurement, laser ablation is not indicated at this point.
Option B: Intralesional bevacizumab
• Bevacizumab is a monoclonal antibody that inhibits angiogenesis and could, theoretically, aid in
treating a strawberry hemangioma. However, the intended use of bevacizumab is not for strawberry
hemangioma, and this patient's lesion does not cause significant enough functional impairment or
disfigurement to warrant treatment with bevacizumab.
Option C: Topical clobetasol
• Topical corticosteroids such as clobetasol have previously been used for small superficial strawberry
hemangiomas. However, they are no longer routinely used because topical corticosteroids are
associated with considerable adverse effects, including skin atrophy, hypertrichosis, and
hypopigmentation.
• Best cosmetic results obtained by Laser ablation.

Page 35

1492
Solution for Question 17:
Option B: Sebaceous cyst
• This patient is most likely suffering from a sebaceous cyst, a benign, sebum-containing lesion caused
by a blockage of a sebaceous gland.
• Usually, it is asymptomatic but can become inflamed.
• The clinical manifestations include a slow-growing, mobile, firm, painless nodule on the face, head,
neck, back, or genitals.
• They may get infected, resulting in pain and an erythematous mass that may exude pus.
• There is a high probability of recurrence.
• A dark-colored punctum may be seen.
• Treatment options include intralesional steroids and surgical excision.
Option A: Basal cell carcinoma
• A slow-growing, locally invasive malignant neoplasm of the skin arises from basal epithelial cells. A
most common type of skin cancer. Typically, it develops in sun-exposed areas as a pearly, indurated,
nodular, non-tender lesion with a rolled border and central ulceration.
• These features are not seen in this patient, so we can rule them out.
Option C: Lipoma
• A benign tumor of adipocytes presents as a painless, mobile, rubbery nodule not seen in the clinical
vignette. A dark-colored punctum in a sebaceous cyst is also not seen in lipoma.
Option D: Squamous cell carcinoma
• A cutaneous Squamous cell carcinoma manifests as ulcers, typically red with everted edges. The floor
of the ulcer resembles granulation tissue and bleeds easily. The skin around the ulcer is inflamed and
indurated. These features are not seen in the vignette.

Solution for Question 18:


Option C: Neuromuscular preserving flap
• This patient with squamous cell carcinoma of the lower lip needs reconstructive surgery using tissue
flaps to fill the gap by excision of the tumor.
• This wound can be repaired using a Karapandzic flap, a sensate axial musculocutaneous flap.
• The Karapandzic flap is based on the superior and inferior labial arteries. It provides good aesthetic
effects and is useful for closing one-half to two-thirds of defects of the upper lip and defects up to
three-quarters of the lower lip. The main advantageous point of the Karapandzic flap is that it is a
neuromuscular preserving flap.
• The modalities for the reconstruction of the lip are summarized in the following table:
• Karapandzic flap: Uses a sensate, neuromuscular flap based on the labial artery.

Page 36

1493
• Webster-Bernard repair: Uses lateral nasolabial flap with buccal advancement
Option A: 2-stage procedure
• This is not the advantageous point of the Karapandzic flap.
Option B: It can cause Macrostomia
• This is not the advantageous point of the Karapandzic flap.
Option D: All of the above
• This is an incorrect option.

Solution for Question 19:


Option B: Cherry hemangiomas
• This patient most likely suffers from Cherry hemangiomas, benign vascular proliferations commonly
developing in adult patients.
• The diagnosis is typically based on the clinical appearance of multiple bright red, dome-shaped
papules on the trunk and upper extremities that bleed with trauma and usually blanch with pressure.
• Cherry hemangiomas manifest more frequently with age and do not regress.
• On histological examination, dilated interconnecting capillaries within the papillary dermis are seen.
• The treatment is necessary unless lesions, such as frequent bleeding, become bothersome or are
cosmetically unappealing.
• The treatment options for removal include electrocauterization, laser therapy, and excision.
Option A: Bacillary angiomatosis
• The Bartonella henselae infections cause bacillary angiomatosis.
• Bacillary angiomatosis manifests with vascular proliferations that bleed with trauma, like the lesions
seen in this patient.
• Although the lesions of bacillary angiomatosis can manifest with a papular appearance, they also
commonly manifest as nodular, pedunculated, or verrucous lesions. In addition, this condition usually
develops in patients infected with HIV, of which there is no indication here.
Option C: Strawberry angiomas
• Strawberry angioma is characterized by benign, vascular proliferations that may bleed with trauma,
like the lesions in this patient.
• However, strawberry angiomas usually develop in the first weeks of life on the head or neck and then
involute by 5–8 years of age.
Option D: Amelanotic melanoma
• Amelanotic melanoma is characterized by a rare variant of melanoma that manifests as a skin-colored
or red papule or nodule.
• Amelanotic melanoma may also bleed with trauma. However, this condition typically manifests as a
solitary lesion, unlike multiple lesions.

Page 37

1494
Solution for Question 20:
Option C: Retroperitoneal
• Retroperitoneal lipoma is associated with malignancy.
• Lipoma is made up of mature fat cells.
• The clinical features include: Slow-growing round, soft, rubbery tumor Usually, it is non-tender
• Slow-growing round, soft, rubbery tumor
• Usually, it is non-tender
• The diagnostic work-up includes a clinical diagnosis. USG can be performed if there is an inconclusive
diagnosis.
• The treatment is usually by observation and reassurance. However, surgical excision can be
considered in the following cases: If the tumor causes pain or for cosmetic reasons.
• Slow-growing round, soft, rubbery tumor
• Usually, it is non-tender
Option A: Subcutaneous
• This location is not associated with malignancy.
Option B: Sub-aponeurotic
• This location is not associated with malignancy
Option D: Intermuscular
• This location is not associated with malignancy

Solution for Question 21:


Option B: Separation of skin and subcutaneous tissue
• This patient most likely suffers from a traumatic degloving injury due to trauma sustained during the
high-speed motor vehicle accident.
• A degloving injury is characterized by an injury in which the skin & subcutaneous fat are stripped by
avulsion from the underlying fascia, leaving neurovascular structures, tendons, or bone exposed, such
as bilateral degloving injury of lower limbs due to a truck trampling.
• The diagnosis is by clinical examination and detailed history.
• Managing the degloving injury involves surgical debridement followed by immediate full-thickness skin
grafting.
Option A: Separation of skin
• This option does not describe the degloving injury.
Option C: Separation of fascia exposing tendons
• This option does not describe the degloving injury.
Option D: Separation of tendon exposing the bone
• This option does not describe the degloving injury.

Page 38

1495
Solution for Question 22:
Option A: Wet gangrene
• This patient most likely suffers from wet gangrene, characterized by gangrenous necrosis caused by
superinfection resulting in coagulative and liquefactive necrosis on histopathologic examination. The
clinical features include; Oedema Blistering Discharge or a moist appearance as shown in the image:
• Oedema
• Blistering
• Discharge or a moist appearance as shown in the image:
• Oedema
• Blistering
• Discharge or a moist appearance as shown in the image:

• The rapid spread of infection


• The diagnostic work-up is based on clinical features and thorough physical examination.
• The management of these cases comprises analgesia, surgical debridement, drainage, and
amputation of infected and necrotic tissue.
• Broad-spectrum antibiotics are also administered.
• In case of an unsuccessful response to treatment, surgical amputation is indicated.
Option B: Dry gangrene
• Dry gangrene is a subtype of gangrenous necrosis caused by ischemia, not infection. Furthermore,
the gross appearance would be different from the wet gangrene, as illustrated in the image below:

Page 39

1496
• The difference between wet and dry gangrene is summarized in the following table:
Option C: Frostbite
• Frostbite is a severe localized tissue injury due to freezing. There is no history of freezing in the
vignette so that we can rule out this option.
Option D: Ainhum
• Ainhum is a band-like constriction of the soft tissue of a digit

Solution for Question 23:


Option B: Intravenous vancomycin
• This patient is most likely suffering from purulent cellulitis, a rapidly spreading local infection of the
deep dermis and subcutaneous tissue. Cellulitis is caused most commonly by Streptococcus pyogenes
and Staphylococcus aureus.
• It commonly appears in areas with broken skin (e.g., due to trauma or another infection). The
diagnosis is usually clinical.
• Intravenous vancomycin is effective against gram-positive bacteria, including MRSA, making it the
empirical treatment of choice for patients with purulent cellulitis and systemic signs of infection (e.g.,
tachycardia, fever).
• Since infection by MRSA is becoming increasingly common, an antibiotic that covers MRSA is also
required for empirical therapy in patients with a history of recent hospitalization, recent antibiotic use,
penetrating trauma, injection drug use, or evidence of MRSA infection or colonization elsewhere.
• Once culture and sensitivity results are available, a more targeted approach to antibiotic therapy
should be pursued.
Option A: Oral clindamycin
• Clindamycin is effective against gram-positive bacteria, including MRSA. While clindamycin is one of
the recommended empiric antibiotics for non-purulent cellulitis in patients who do not require parenteral

Page 40

1497
therapy, it is not recommended for purulent cellulitis. Oral clindamycin is inappropriate for this patient
with systemic signs of infection (tachycardia, fever).
Option C: Surgical debridement
• For patients with cellulitis, surgical debridement is indicated in the case of recurrent infection and
necrotizing fasciitis and if cellulitis continues to spread despite appropriate antibiotic therapy.
• This patient has no evidence of necrotizing fasciitis, such as pain out of proportion to the area of
erythema, skin necrosis, or crepitus.
Option D: Oral dicloxacillin
• Dicloxacillin is a type of anti-staphylococcal penicillin that is indicated for nonpurulent cellulitis.
However, it is not indicated for empiric treatment of purulent cellulitis because it is ineffective against
MRSA.

Solution for Question 24:


Answer
Option C: Pilomatrixoma
This patient is most likely suffering from pilomatrixoma.
• A benign skin lesion of the hair follicle characterizes Pilomatrixoma.
• Usually, pilomatrixoma develops in the head and neck area.
• The clinical features include a rugged, freely mobile, and slow-growing mass.
• Pilomatrixoma is usually asymptomatic, but in some cases, patients complain of pain or discharge
from the lesions.
• Calcium deposits are present in well over half the lesions identified. Thus, the skin lesion is also
described as a calcifying epithelioma.
• Rarely, pilomatrixoma can become cancerous, known as pilomatrix carcinoma.
• The treatment of choice is surgical excision.
Other Options
Option A: Cock's Peculiar Tumor
• It is characterized by an infected or ulcerated sebaceous cyst of the scalp, which is not consistent with
the features of the vignette.
Option B: Pott's Puffy Tumor
• It is characterized by osteomyelitis of the frontal bone of the skull, which is not consistent with the
features of the vignette.
Option D: Cylindroma
• Cylindroma is a malignant epithelial tumor characterized by solitary or multiple lesions.
• These lesions manifest as numerous small papules and large dome-shaped nodules are present on
the scalp like a turban, as shown in the image below:

Page 41

1498
• These features are not consistent with the features of the vignette.

Solution for Question 25:


Option C: Deltopectoral flap
• This patient, who is most likely suffering from squamous cell carcinoma of the oral cavity, needs
closure of the defect after the surgical excision of the tumour.
• A defect present around the site of excision of the mass should be closed by a local flap in this patient.
• Modern plastic surgery has enabled healthcare professionals to reconstruct the tissue after the
tumour excision to restore the body's normal morphology and function.
• A flap is characterized by a tissue that is transferred from one particular site (donor site) to another
(recipient site).
• In this patient, a deltopectoral flap is used for the repair of the tissue defect, as shown in the image
below:

Page 42

1499
• Bakamjian developed this type of flap in 1960. It is an axial-pattern skin flap derived from a pectoral
area for reconstruction in the head and neck region.
• The blood supply of this flap depends on intercostal perforating branches from the internal mammary
artery.
• Further, some branches from the thoracoacromial artery also perfuse this flap.

Incorrect Options:
Option A: Pectoral myocutaneous flap
• A pectoral myocutaneous flap is based on pectoral branches of the thoracoacromial artery. It is also
used for the reconstruction of head and neck defects. However, anatomically it is present at a different
location than a deltopectoral flap.
• This type of flap is illustrated in the image below:

Page 43

1500
Option B: TRAM flap
• A TRAM flap is based on epigastric vessels, the most common flap used for breast reconstruction.
This type of flap is illustrated in the image below:

Option D: Latissimus Dorsi flap


• Latissimus Dorsi flap is based on thoracodorsal vessels; used for breast reconstruction. This type of
flap is illustrated in the image below:

Solution for Question 26:


Option B: Used for getting split-thickness graft & Full-thickness skin graft
• This patient is most likely suffering from cutaneous squamous cell carcinoma. He needs closure of the
defect after the surgical excision of the tumor. A circular defect of around 5 cm diameter present around

Page 44

1501
his ear should be closed by a local flap in this patient.
• A flap is characterized by a tissue transferred from one particular site (donor site) to another (recipient
site).
• This Humby–Braithwaite Dermatome knife (skin graft knife) is used to collect a flap of healthy skin
from the donor site to another (recipient site) for performing a skin graft.
• This process fills the tissue gap on a site with tissue loss.

Incorrect Options:
Option A: Used for separating periosteum from under the surface of the rib
• The Humby – Braithwaite Dermatome knife is not used for separating the periosteum from under the
surface of the rib.
Option C: It is a rib Spreader utilized in cardiothoracic procedures
• The Humby – Braithwaite Dermatome knife is not used as a rib spreader in cardiothoracic procedures.
Option D: Used to dissect superior thyroid pedicle
• The Humby – Braithwaite Dermatome knife is not used to dissect the superior thyroid pedicle.

Solution for Question 27:


Option A: It carries its vessels within it
• This patient requires complete tumor excision and reconstruction of the wound using flaps. The
high-yield types of the various flaps are summarized in the table below:

• The tissue flaps can be classified based on circulation as follows: Axial pattern flap Random pattern
flap

Page 45

1502
• Axial pattern flap
• Random pattern flap
• Axial pattern flap is characterized by a flap that contains at least one direct cutaneous branch blood
supply along its longitudinal axis, as shown in the image below:
• Axial pattern flap
• Random pattern flap

Option B: It is kept in the limb


• This option is incorrect, as an axial flap is not kept in the limb.
Option C: It is a transverse flap
• This option is incorrect as the TRAM flap is a type of axial flap. However, an axial flap is not always
characterized by a transverse flap.
Option D: It carries its nerve in it
• This option is incorrect

Solution for Question 28:


Option D: Skin, muscle and vascular pedicle
• The tissue included in the myocutaneous flap is skin, subcutaneous tissue, fascia, and the underlying
muscle. So, option D is the correct answer.
• A myocutaneous flap (also known as an Axial flap) is based on a named vessel.
• It provides a reproducible & stable skin or skin-muscle flap, including skin, muscle and vascular
pedicle.
• It can be used to provide much-needed length & bulk for reconstructive surgery.
• An axial flap that remains attached to its proximal blood supply & transposed to a defect is known as a
pedicled flap

Page 46

1503
Option A: Muscle only
• The myocutaneous flap does not include muscle only.
• In addition to the muscle, they also contain skin, subcutaneous tissue, and fascia.
Option B: Muscle and vascular pedicle
• The statement describes a vascular pedicle and not the myocutaneous flap.
Option C: Muscle and skin
• The myocutaneous flaps do not include muscle and skin only.
• In addition to muscle and skin, they also contain subcutaneous tissue and fascia.

Solution for Question 29:


Option B: Biopsy of the lesions
• This patient is most likely suffering from leukoplakia.
• It is precancerous hyperkeratosis and dysplasia of the epithelium and mucous membranes most
commonly located in the oral cavity.
• The risk factors include tobacco use and alcohol consumption.
• Leukoplakia clinically manifests as persistent white plaques that usually cannot be scraped off, as
shown in the image below:

Page 47

1504
• While leukoplakia is a benign lesion, it predisposes to squamous cell carcinoma.
• Biopsy of the lesions is the next best step in managing leukoplakia, as up to 20% of leukoplakia
lesions progress to carcinoma within ten years.
• Because of this, it is essential to biopsy leukoplakia lesions to check for signs of dysplasia.
• The treatment consists of smoking cessation, cryotherapy, laser ablation, or surgical excision if the
lesion persists.
Option A: Topical nystatin
• Topical nystatin is the first-line therapy for oropharyngeal candidiasis.
• This patient uses inhaled corticosteroids, a known risk factor for developing oral candidiasis.
• However, candidiasis lesions can be scraped off unlike leukoplakia.
• Moreover, this patient lacks other common symptoms of candidiasis, including pain while eating, loss
of taste, and a cottony feeling in the mouth.
Option C: Topical corticosteroids
• Topical corticosteroids would be helpful in the management of aphthous stomatitis.
• The aphthous stomatitis or "canker sores" presents as a round to oval, crater-like appearance on a
yellowish-gray base and erythematous margins, which is not seen in this patient.
Option D: Surgical excision of the lesions
• Surgical excision may be needed if the lesions persist following more conservative treatment.
• It would not be the first-line therapy or diagnostic tool.

Solution for Question 30:


Option C: Malignant melanoma

Page 48

1505
• This patient's clinical presentation, such as a painless, irregularly shaped, brown-black pigmented
macule involving the toe and nail bed, is consistent with acral lentiginous melanoma.
• This tumor mainly affects dark-skinned and Asian populations.
• One subtype of acral lentiginous melanoma is subungual melanoma.
• Subungual melanoma usually develops within the nail matrix and manifests as a longitudinal black or
brown band.
• Moreover, periungual pigmentation of the proximal or lateral nail folds can also develop, which is the
Hutchinson sign of the nail. This sign is illustrated in the image below:

Option A: Squamous cell carcinoma (SCC)


• Squamous cell carcinoma can rarely involve the nail bed.
• However, SCCs are usually small, red tumours that may result in paronychia, pain, and bleeding, as
shown in the image below:

Page 49

1506
Option B: Traumatic subungual hemorrhage
• Subungual hematomas may manifest as dark-coloured macules. However, this diagnosis is unlikely
because this patient has no history of trauma to the foot and the absence of pain and swelling of the
affected nail.
Option D: Onychomycosis
• This patient presents with a single dark-coloured subungual macule. However, onychomycosis
typically involves the whole nail and manifests as discoloured, such as white, gray, or yellow. In
addition, brittle nails are also seen in the case of onychomycosis.

Solution for Question 31:


Option A: Multiple colors of the lesion
• This patient is most likely suffering from malignant melanoma.
• The melanocytic skin lesions that meet at least one ABCDE criterion should raise suspicion for
malignant transformation and consider full-thickness excisional biopsy with histological examination.
• The ABCDE criteria are widely used to identify signs of malignant melanoma. The ABCDE criteria
include the following: Asymmetry Border irregularity Color variation Diameter >6mm Evolving
• Asymmetry
• Border irregularity
• Color variation
• Diameter >6mm
• Evolving
• The mixture of colors in this patient's skin lesion meets criterion C (color variegation, such as different
shades of brown, black, red, white, and blue).
• Tumour thickness, as determined from the Breslow thickness (measured in mm), is the most
important prognostic factor in melanoma. However, it should be > 6mm to fulfill the ABCDE criteria.
• Types of malignant melanoma (MM) Superficial spreading: the most common type of MM; the most
common site is the torso. Nodular is the second most common type and the most malignant variant. Its
most common site is the head and neck Lentigo maligna: The least malignant, most common site is the
face Acral lentiginous: Least common, worst prognosis. Most common sites are sole and under the
great toenail
• Superficial spreading: the most common type of MM; the most common site is the torso.
• Nodular is the second most common type and the most malignant variant. Its most common site is the
head and neck
• Lentigo maligna: The least malignant, most common site is the face
• Acral lentiginous: Least common, worst prognosis. Most common sites are sole and under the great
toenail
• The diagnostic workup includes a full-thickness excisional biopsy.
• The management of melanoma is by surgical excision (full-thickness excision with appropriate safety
margins).

Page 50

1507
• The only FDA-approved adjuvant therapy for malignant melanoma is Interferon alpha 2b.
• Asymmetry
• Border irregularity
• Color variation
• Diameter >6mm
• Evolving
• Superficial spreading: the most common type of MM; the most common site is the torso.
• Nodular is the second most common type and the most malignant variant. Its most common site is the
head and neck
• Lentigo maligna: The least malignant, most common site is the face
• Acral lentiginous: Least common, worst prognosis. Most common sites are sole and under the great
toenail
The only FDA-approved adjuvant therapy for malignant melanoma is Interferon alpha 2b.
Option B: 5 mm diameter of the lesion
• According to criterion D (diameter) of the ABCDE melanoma criteria, skin lesions measuring more
than 6 mm should prompt further diagnostic steps.
• The diameter of this patient's lesion is 5 mm; therefore, it is not an indication for performing an
excisional biopsy.
According to criterion D
(diameter) of the ABCDE melanoma criteria, skin lesions measuring more than 6
mm should prompt further diagnostic steps.
Option C: No change in the lesion over the past three months
• According to criterion E (evolution) of the ABCDE criteria of melanoma, skin lesions that change in
size, shape, or color should prompt further diagnostic steps.
• However, in this case, the skin lesion that has not changed noticeably over time is not an indication for
performing an excisional biopsy.
Option D: Uniform borders of the lesion
• According to criterion B (borders) of the ABCDE melanoma criteria, skin lesions with irregular
borders should prompt further diagnostic steps.
• However, the uniform borders, as seen in this vignette, are not an indication for performing an
excisional biopsy.

Solution for Question 32:


Option A: Marjolin's ulcer is a
slow-growing ulcer and is not associated with secondary deposits in regional lymph node
• A Marjolin ulcer is a type of cutaneous malignancy that develops slowly in the presence of old
wounds, persistent scars, and previously damaged skin.

Page 51

1508
• Local lymphatic arteries are destroyed, there is inadequate vascularization, and there is decreased
Langerhans cell activity, which makes it possible for lesions to grow without being screened by the
immune system.
• No secondary deposits in regional lymph nodes, as there are no lymphatic vessels in scar tissue as
the vessels and lymphatics are destroyed.
• In the past, Marjolin ulcers were first referred to as dense villi ulcerations. Squamous cell carcinoma is
the most prevalent form of cancer that has been identified, while some other cell types have also been
identified (i.e., basal cell carcinoma). Lesions have a poor prognosis and a high rate of recurrence.
Preventive measures, such as proper wound care care, early detection of malignant conversion, and, if
practical, surgical excision, is crucial.
Option B: It is a high-grade Squamous cell carcinoma
• Marjolin's ulcer is a low-grade squamous cell carcinoma that develops on a chronic benign ulcer or a
long-standing scar tissue.
• Squamous cell carcinoma is the most prevalent form of cancer that has been identified, while other
cell types have been identified (i.e., basal cell carcinoma). Lesions have a poor prognosis and a high
rate of recurrence.
Option C: It is a very painful ulcer
• Marjolin's ulcer typically appears as a sore that won't heal.
• It is painless as there are no nerves in the scar tissue
Option D: It is an extremely radiosensitive ulcer
• Marjolin's ulcers are radio-resistant due to avascularity.
• The mainstay of treatment is wide excision with grafting or amputation, depending on the size and
stage of the ulcer.

Solution for Question 33:


Option A: Bilobed flap
• The flap shown in the image is a bilobed flap.
Local flaps
• Flap transferred from an area adjacent to the defect is known as a local flap.
• A local flap is raised next to a tissue defect to reconstruct it.
• Bilobed flap- For convex surfaces, especially the nose.
• Bilobed flap is a type of local flap
Distant flaps
• A flap transferred from a different anatomic site (i.e., from a different part of the body) is referred to as
a distant flap.
Other options
Option B: Advancement flap

Page 52

1509
• They are incised flaps that generate unidirectional movement to cover an existing defect created by
trauma.
• This flap is often used on the scalp to repair small mucosal lining defects adjacent to the nasal rim or
for septal perforation repair.

Option C: Rhomboid flap


• Rhomboid flaps are full-thickness cutaneous local flaps that typically rely on dermal–subdermal plexus
blood supply
• It is rhomboid in shape also known as Limberg flap.

Option D: Z-plasty
• Z-plasty is a plastic surgery technique used to improve the functional and cosmetic appearance of
scars.
• The image shown is not of z-plasty

Page 53

1510
Solution for Question 34:
Option A: Rhomboid flap
• The history and clinical examination point towards the diagnosis of pilonidal sinus.
• Pilonidal sinus usually presents with chronic episodic pain and swelling with discharge in the natal
cleft overlying the rhomboids.The rhomboid flap is used for repair surgery.
• It is more common in men aged 20 to 30, dark-haired individuals.
• Spontaneous regression is possible with rest, strict hygiene, and broad-spectrum antibiotics.
• Surgery is done in recurrent, symptomatic or complicated cases.
• Pilonidal sinus surgery includes the excision of the pilonidal sinus and the repair of the wound by a
flap.
• To prevent a recurrence, ensure the entire tract, along with all their openings, are excised
• All flaps are sutured and designed in a way to prevent midline wounds.
• Flaps used:
1. Z-plasty
2. Karydakis flap
3. Limberg Rhomboid flap
4. Bascom procedure
• Postoperative care mostly includes continuous hair removal and hygiene maintenance.
• Recurrence may be due to a part of the tract still being left inside.

Page 54

1511
Option B: Circular flap
• Such flap type is not usually performed at any time of surgery.
• Difficult to obtain and operate
Option C: Free flap
• It is a kind of lap that is disconnected from its original blood supply.
• It has a higher failure rate.
Option D: Rotational flap
• The semi-circular flap revolves in an arc.
• It does not usually fit on a circular surface.

Solution for Question 35:


Option A: Rotation flap
• This type of flap involves rotating adjacent tissue to cover the defect left after mastectomy. It is
commonly used in reconstructive surgeries due to its ability to provide good coverage and maintain
blood supply from the surrounding tissue.
• A flap is any tissue with an intrinsic blood supply that can be transferred from a donor to a recipient
site.
• Flap surgery is a reconstructive technique used to close a defect that cannot be completed by primary
or secondary intention.
• Some of the basic local flaps and their uses are:
Basic Patterns of Local Flaps
Transposition Flap
Most basic design, leaving a graftable donor site.

Page 55

1512
Z-plasty
For lengthening scars or tissues.
Rhomboid Flap
For cheek, temple, back, and flat surface defects.
Rotation Flap
For convex surfaces like breast
Advancement flap
Flexor surfaces; may need triangles excised at the base to make them work (Commonly called Burrow'
s triangles).
V-to-Y advancement
Commonly used for fingertips and extremities.
Bilobed flap
For convex surfaces, especially the nose.
Bipedicle flap
For eyelids, rarely elsewhere.
Option B: Bilobed flap
• It is a type of local flap.
• It is commonly used for reconstruction options for nasal tip.
Option C: Z-plasty
• It is also a type of local flap.
• It is designed to increase the length of the scar
Option D: Rhomboid flap
• Used for cheek and defects involving flat surfaces
• E.g. Basal cell carcinoma, Pilonidal sinus.

Solution for Question 36:


Option A: Due to negative pressure, bacterial counts decrease, and cell proliferation increases in the w
ound.
• Vacuum-assisted wound closure is a type of therapy that helps wounds healing.
• It is an alternative to conventional methods of wound management, like surgery. Also called negative
pressure wound therapy (NPWT)
• The use of negative pressure optimizes the wound for spontaneous healing or by lesser
reconstructive methods.
• It is used in diabetic ulcers, severe burns, crush injuries, bed sores, etc.

Page 56

1513
• The vacuum closure system consists of a sterilized foam covering, which is covered with a
transparent adhesive cover and an attached suction pump.
• The pump applies intermittent or continuous pressure
• NPWT promotes wound healing by applying a vacuum through a special sealed dressing.
• The continued vacuum draws out the fluid (interstitial fluid) from the wound and increases blood
flow to the area.
• As a result, bacterial counts decrease, and cell proliferation increases, creating a suitable bed for graft
or flap cover.
• The intermittent negative pressure of -125 mmHg appears to hasten debridement and granulation
tissue formation in chronic wounds and ulcers.
• Contraindications for NPWT use: Malignancy in the wounds Untreated osteomyelitis Non-enteric and
unexplored fistula Necrotic tissue with eschar
• Malignancy in the wounds
• Untreated osteomyelitis
• Non-enteric and unexplored fistula
• Necrotic tissue with eschar
• Malignancy in the wounds
• Untreated osteomyelitis
• Non-enteric and unexplored fistula
• Necrotic tissue with eschar

Option B: Continuous pressure of +125 mmHg is applied.


• The attached pump applies a continuous or intermittent negative pressure on the foam cover via a
non-bended discharge tube.
• The pressure is kept between 50 and 125 mmHg.
Option C: It causes interstitial edema, thereby increasing blood flow and better healing.

Page 57

1514
• The purpose of vacuum-assisted closure is to remove edema fluid from the wound by applying
negative pressure.
• It relieves pressure and reduces swelling.
• Removal of excess fluid allows for good perfusion, increasing blood flow.
Option D: Suitable to be used over malignancy if present in the wound.
• The presence of malignancy is considered a contraindication to vacuum-assisted closure.
• It may promote tumourigenesis and expedite metastasis.

Solution for Question 37:


Correct Option C- Intravenous penicillin and debridement of affected tissue:
• The most likely diagnosis in this patient, based on the presentation and examination findings, is gas
gangrene.
• Gas gangrene is a lethal soft tissue infection caused most commonly by Clostridium perfringens.
• The mainstay of treatment includes debridement (removal of dead and foreign matter and collection of
blood from the wound) and large doses of intravenous penicillin.
• Clindamycin should also be added because it inhibits the synthesis of clostridial exotoxins and
lessens their systemic effects.
• Because clindamycin is bacteriostatic and not bactericidal, it should be used in conjunction with
penicillin.
Incorrect Options:
Option A- Immediate limb amputation: The most common causes leading to amputation are diabetes m
ellitus, peripheral vascular disease, neuropathy, and trauma. Amputation is not required for managing
patients with gas gangrene because the limb is still salvageable.
Option B- Hyperbaric oxygen therapy: Early IV antibiotics with early surgical debridement followed by h
yperbaric oxygen therapy can salvage patients. It is used as an adjunctive therapy and is not considere
d the primary treatment on its own.
Option D- Intravenous fluoroquinolones: While fluoroquinolones have broad-spectrum activity and are
effective against many bacterial infections, they are not indicated for treating gas gangrene. Penicillin o
r other antibiotics with activity against anaerobic bacteria are preferred.

Solution for Question 38:


Correct Option D- Staphylococcus aureus:
• A carbuncle is an infection of the hair follicles that extends into the surrounding skin and underlying
subcutaneous tissue.

Page 58

1515
• They typically present as an erythematous, tender, inflamed, fluctuant nodule with multiple draining
sinus tracts or pustules on the surface.
• Bacterial infection within the hair follicle causes carbuncles.
• The most common causative organism is the gram-positive bacterium Staphylococcus aureus, which
frequently involves methicillin-resistant S aureus.
Incorrect Options: Option A- Streptococcus pyogenes: Streptococcus pyogenes is a
gram-positive organism responsible for skin infections such as erysipelas (characterized by a
well-demarcated and raised area of erythema) and cellulitis (characterized by a
poorly demarcated, warm, erythematous area with associated edema and tenderness).

Page 59

1516
Option B- Pseudomonas aeruginosa: It predominantly causes erythema gangrenosum, which is charac
terized by erythematous, ulcerated, purple, or black skin lesions in the axillary, inguinal, or anogenital a
reas.
Option C- Candida albicans: Intertrigo is a superficial inflammatory skin condition of the skin's flexural s
urfaces, prompted or irritated by warm temperatures, friction, moisture, maceration, and poor ventilatio
n. Candida albicans secondarily infects intertrigo.

Solution for Question 39:


Correct Option B- IV fluids + debridement + IV antibiotics + supportive care:
• The most likely diagnosis in this patient with an edematous wound with a woody hard texture of
subcutaneous tissue is necrotizing fasciitis. It is a rapidly progressive bacterial infection.
• It is characterized by skin necrosis, subcutaneous tissue, and fascia. It is a rapidly progressing soft
tissue infection that requires urgent intervention.
• Management includes: Immediate resuscitation with intravenous fluids because the sepsis causes
refractory hypotension and diffuse capillary leak. Early initiation of appropriate empiric antibacterial
coverage (broad spectrum). Adequate control of infection sources, such as aggressive surgical
intervention for abscess drainage and debridement of tissues. Supportive care includes monitoring and
management of systemic complications such as septic shock and organ dysfunction.
• Immediate resuscitation with intravenous fluids because the sepsis causes refractory hypotension and
diffuse capillary leak.
• Early initiation of appropriate empiric antibacterial coverage (broad spectrum).
• Adequate control of infection sources, such as aggressive surgical intervention for abscess drainage
and debridement of tissues.
• Supportive care includes monitoring and management of systemic complications such as septic shock
and organ dysfunction.

Page 60

1517
• Immediate resuscitation with intravenous fluids because the sepsis causes refractory hypotension and
diffuse capillary leak.
• Early initiation of appropriate empiric antibacterial coverage (broad spectrum).
• Adequate control of infection sources, such as aggressive surgical intervention for abscess drainage
and debridement of tissues.
• Supportive care includes monitoring and management of systemic complications such as septic shock
and organ dysfunction.
Incorrect Options:
Option A- IV fluids and debridement: These measures alone are inadequate. Antibiotics are essential i
n controlling the infection and should be a part of the treatment plan.
Option C- Hyperbaric oxygen therapy + supportive therapy: Hyperbaric oxygen therapy helps in wound
healing. However, it does not replace the need for surgical debridement and antibiotics.
Option D- IV fluids + IV antibiotics: Without surgical debridement, the mortality is 100%.

Page 61

1518
Wound Healing, Tissue Repair and Scar
1. Which of the following is the least reliable predictor for the viability of a muscle?
A. Colour of muscle tissue
B. Circulation
C. Contractility of muscle
D. Consistency of muscle
----------------------------------------
2. A first-year medical student is brought to the emergency department after sustaining a cut on his
right forearm while dissecting a cadaver in his morning class. He didn't inform anyone for fear of getting
scolded and was brought to the hospital only after 8 hours of injury by his friends. On examination, a
laceration of 6x2x1cm is noted in his forearm. The wound is noted to be an open, clean wound. What is
the next best step in the management of this patient?
(or)
A first-year medical student has a cut on his right forearm while dissecting a cadaver in his morning
class. The wound is noted to be an open, clean wound. What is the next best step in the management
of this patient?
A. Suturing
B. Debridement and suture
C. Secondary suturing
D. Heal by granulation
----------------------------------------
3. A 27-year-old, 38-week pregnant woman is posted for an emergency cesarean section due to fetal
distress. The classical skin incision was given for cesarean section. After surgery, she was monitored
closely for 12 hours in the postoperative room, which was uneventful. In this patient's sutured surgical
wound, when can you expect the epithelialization process to be completed?
(or)
A 27-year-old, 38-week pregnant woman with an emergency cesarean section. When can you expect
the epithelialization process to be completed in this patient's sutured surgical wound?
A. 24 hours
B. Begins from 3rd day and lasts for 3–6 weeks
C. Begins from 2nd day and lasts for 1–2 weeks
D. Begins after 1 month
----------------------------------------
4. Which drug can be used as an intralesional injection in keloid?
(or)
A 26-year-old female was diagnosed with Keloid and was advised for excision. However, the patient
deferred surgery and opted for medical treatment. Which drug can be used as an intralesional injection
in this patient?
A. Betamethasone

1519
B. Triamcinolone
C. Androgen
D. Hydrocortisone
----------------------------------------
5. A 7-year-old girl presents to the emergency department 30 minutes after she sustained a left shin
laceration after she fell off her bicycle. On examination, there is a 1 cm laceration on her left shin medial
to her tibia with slight pain and bleeding. In the process of wound healing, which of the following would
be responsible for wound contraction?
(or)
A 7-year-old girl has a 1 cm laceration on her left shin medial to her tibia with slight pain and bleeding.
Which of the following would be responsible for wound contraction in wound healing?
A. Collagen
B. Myofibroblast
C. Epithelial tissue
D. Elastin
----------------------------------------
6. A 47-year-old male patient is brought to the clinic complaining of a large erythematous skin pocket
on his sacrum and left hip. He has a history of systemic lupus erythematosus, paraplegia, and
steroid-induced diabetes, diagnosed two years ago. Examination reveals large 3-4 cm pressure sores,
as shown in the image below. Which would be the most appropriate treatment for this patient’s
condition?
(or)
A 47-year-old male patient has 3-4 cm pressure sores, as shown in the image below. Which would be
the most appropriate treatment for this patient’s condition?

A. Surgery is the only modality of the treatment


B. A fasciocutaneous flap should be used
C. Herbal remedies with or without insulin are mostly useful than other remedies.
D. Stage 2 and 3 ulcers may be left to heal secondarily after debridement.
----------------------------------------
7. A 40-year-old male presents to the clinic for a follow-up visit. A week ago, this patient presented to
the emergency department with a lacerated wound on his right foot when he accidentally fell from his

Page 2

1520
bike. The wound was sutured, and he was discharged. On his follow-up visit today, the surgeon graded
the wound as type IIIc according to Southampton grading. What does this indicate?
(or)
The surgeon graded the wound as type IIIc according to Southampton grading. What does this
indicate?
A. Erythema along sutures
B. A Large volume of haemoserous discharge
C. Prolonged haemoserous discharge
D. Pus at one point
----------------------------------------
8. A 32-year-old woman developed complete wound dehiscence with evisceration after a C-section and
developed the burst abdomen on the sixth day. What is the best next step in management?
(or)
What's the best next step in managing a 32-year-old woman with complete wound dehiscence,
evisceration post-C-section, and burst abdomen?
A. The wound and bowel should be covered with a moist towel
B. The wound will heal by secondary intention
C. Surgical mesh is placed over the eviscerated bowel and then returned to the abdomen.
D. Previous sutures should be repaired immediately.
----------------------------------------
9. The maggots that can be used for wound therapy are?
(or)
An 82-year-old male patient with chronic diabetes mellitus has infected wounds on the lower part of the
leg. The patient and his family opted for self-treatment with maggots. The maggots that can be used for
wound therapy are?
A. Black soldier fly larvae
B. Housefly larvae
C. Green bottle fly larvae
D. Bluebottle fly larvae
----------------------------------------
10. A 62-year-old woman presents to the emergency department with the complaint of an ulcer on her
lower back. She has a history of a recent fall where she fractured her right femur and has been on bed
rest ever since. On examination, a bed sore is noted on her lower back, as shown in the image below.
What is the stage of the following image of bed sore?
(or)
A 62-year-old woman has a bed sore on her lower back, as shown in the image below. What is the
stage of the following image of bed sore?

Page 3

1521
A. Stage 1
B. Stage 2
C. Stage 3
D. Stage 4
----------------------------------------
11. A 17-year-old male, a high school football player, fell after being tackled, injuring his right index
finger because of contact with the ground. On the field, a coach was reported to have applied a
commercially available cooling gel pack, wrapping it around the athlete's finger directly on the skin with
an elastic wrap. The patient did not recall being instructed to remove it and believed that leaving it on
could only help. After 02 hours, he removed the cooling pack and noticed that his index finger and the
radial half of his uninjured middle finger were red and swollen (image). He was brought to the hospital
and diagnosed with a case of frostbite. Which of the following would be the proper treatment for this
patient now?
(or)
A 17-year-old male is diagnosed with a case of frostbite. Which of the following would be the proper
treatment for this patient now?

A. Debridement of the affected part followed by silver sulfadiazine dressings


B. Administration of Corticosteroids
C. Administration of vasodilators
D. Immersion of the affected part in water at 40°–44°C (104°–111.2°F)
----------------------------------------
12. Which one of the following cases is considered a clean-contaminated wound?

Page 4

1522
A. Open cholecystectomy for cholelithiasis
B. Herniorrhaphy with mesh repair
C. Lumpectomy with axillary node dissection
D. Appendectomy with walled-off abscess
----------------------------------------

Correct Answers
Question Correct Answer

Question 1 1
Question 2 2
Question 3 2
Question 4 2
Question 5 2
Question 6 4
Question 7 2
Question 8 1
Question 9 3
Question 10 4
Question 11 4
Question 12 1

Solution for Question 1:


Answer
Option A: Colour
• It is the least reliable sign for muscle viability.
• Surface tissue may be discolored due to contusion or local vasoconstriction.
• '4C' are used to detect Muscle viability : Colour: Dead muscle has dark unhealthy colour and has lost
its sheen. Contractility: Dead muscles do not twitch when held by forceps. Consistency: Dead muscle
has lost its turgor & is mushy in consistency. Capillary bleeding: Dead muscle does not bleed at cut
ends.
• Colour: Dead muscle has dark unhealthy colour and has lost its sheen.
• Contractility: Dead muscles do not twitch when held by forceps.
• Consistency: Dead muscle has lost its turgor & is mushy in consistency.
• Capillary bleeding: Dead muscle does not bleed at cut ends.
• Colour: Dead muscle has dark unhealthy colour and has lost its sheen.
• Contractility: Dead muscles do not twitch when held by forceps.

Page 5

1523
• Consistency: Dead muscle has lost its turgor & is mushy in consistency.
• Capillary bleeding: Dead muscle does not bleed at cut ends.
Other options
Option B: Circulation
• The 4 C's (colour, consistency, contraction, and circulation) can identify nonviable muscles. Still,
bleeding during debridement is the best predictor of the viability of a muscle after a crushing injury, as
dead tissue will not bleed, and it will guide the surgeon about dead tissue during debridement.
Option C: Contractility
• A nonviable dead muscle has lost its turgor and is mushy in texture. As compared to the healthy one
Option D: Consistency
• When forceps are used to hold muscles, they do not twitch if the muscle is not viable and dead.
Healthy and viable muscle twitch after touching with forceps.

Solution for Question 2:


Option B: Debridement and suture
• Wound healing is when the body restores and replaces the function in damaged tissue.
• The basic principles in wound healing management include hemostasis, cleaning the wound,
analgesics, skin closure, dressing and follow-up.
• Wound cleaning is important for reducing infection. It has five aspects: disinfect, decontaminate,
debride, irrigation and antibiotics.
• If the blood supply to the wound is adequate and bacterial invasion is absent, the wound can be safely
closed anytime following proper debridement and irrigation, as in this case.
• If there is an established infection and the tissue is of doubtful viability and has been left in – situ, then
the wound is left open and re-explored after 48 hours.
• If there is an infection, and the doubtful viable tissue is now healthy, the wound is closed after
appropriate management.
• If, however, there is further necrosis and infection, the wound is again debrided and left open.
Option A: Suturing
• Suturing is used for lacerations greater than 5cm, deep dermal wounds, or locations that are prone to
flexion, tension or wetting.
• Suturing can only be done after cleaning, exploration and satisfactory excision of the wound.
• Immediate suturing can have serious complications if precautions to prevent infections and promote
healing are not taken.
Option C: Secondary suturing
• Secondary suturing is also called secondary closure or healing by secondary intention.
• It is done in cases in which wounds cannot be approximated by primary closure.
• It requires a granulation tissue to be built to fill the wound.

Page 6

1524
Option D: Healing by granulation
• Granulation tissue fills the wounds that heal by secondary intention.
• It is an important component in the wound-healing process.
• But, if the blood supply to the wound is adequate and bacterial invasion is absent, the wound can be
safely closed anytime following proper debridement and irrigation, as in this case.

Solution for Question 3:


Option B: Begins from 3rd day and lasts for 3–6 weeks
• Epithelialization, the process of new skin forming over a wound, typically begins around the 3rd day
after surgery.
• It can last for 3 to 6 weeks until the wound is fully healed.
• In the case of a cesarean section, where the surgical wound is sutured, epithelialization plays a crucial
role in the healing process.
• Proliferative phase (Collagen/fibroblastic phase) It begins on 3rd day and lasts for 3–6 weeks. There
will be the formation of granulation tissue and repair of the wound. Granulation tissue contains
fibroblasts, neocapillaries, collagen, fibronectin, and hyaluronic acid. (1) Initial angiogenesis (growth of
new blood vessels) occurs by the release of vascular endothelial cell growth factor (VEGF) by
keratinocytes; and by the release of TNF-α, TGF–β, PDGF, and FGF by macrophages. (2) Eventual
fibroplasia develops by fibroblast activity with the formation of the collagen and ground substance/
glycosaminoglycans. Type III collagen is deposited initially in a random fashion. (3) Later
re-epithelialization of the wound surface occurs by migration of the basal layer of the retained epidermis
which proliferates, differentiates, and stratifies to form wound closure.
• It begins on 3rd day and lasts for 3–6 weeks. There will be the formation of granulation tissue and
repair of the wound. Granulation tissue contains fibroblasts, neocapillaries, collagen, fibronectin, and
hyaluronic acid.
• (1) Initial angiogenesis (growth of new blood vessels) occurs by the release of vascular endothelial
cell growth factor (VEGF) by keratinocytes; and by the release of TNF-α, TGF–β, PDGF, and FGF by
macrophages.
• (2) Eventual fibroplasia develops by fibroblast activity with the formation of the collagen and ground
substance/ glycosaminoglycans. Type III collagen is deposited initially in a random fashion.
• (3) Later re-epithelialization of the wound surface occurs by migration of the basal layer of the retained
epidermis which proliferates, differentiates, and stratifies to form wound closure.
Proliferative phase (Collagen/fibroblastic phase)
• It begins on 3rd day and lasts for 3–6 weeks. There will be the formation of granulation tissue and
repair of the wound. Granulation tissue contains fibroblasts, neocapillaries, collagen, fibronectin, and
hyaluronic acid.
• (1) Initial angiogenesis (growth of new blood vessels) occurs by the release of vascular endothelial
cell growth factor (VEGF) by keratinocytes; and by the release of TNF-α, TGF–β, PDGF, and FGF by
macrophages.
• (2) Eventual fibroplasia develops by fibroblast activity with the formation of the collagen and ground
substance/ glycosaminoglycans. Type III collagen is deposited initially in a random fashion.

Page 7

1525
• (3) Later re-epithelialization of the wound surface occurs by migration of the basal layer of the retained
epidermis which proliferates, differentiates, and stratifies to form wound closure.
Option A, C, and D are incorrect.

Solution for Question 4:


Option B: Triamcinolone
• Keloids are firm, rubbery lesions that occur due to the abnormal proliferation of scar tissue at the site
of cutaneous injury.
• It grows beyond the original margins of the scar.
• A hypertrophic scar is a thick, raised scar resulting from an abnormal response to wound healing.
• Hypertrophic scars, which are raised scars, do not grow beyond the boundaries.
• Both of these lesions occur due to an overgrowth of granulation collagen type 3 at the site of a healed
skin injury. Collagen type 3 is then slowly replaced by collagen type 1.
• Treatment of Hypertrophic and Keloid scars Pressure – Local molds or elasticated garments Silicone
gel sheeting Intralesional steroid injection (triamcinolone). The benefit of this procedure is that the
injection attains a high drug concentration at the disease site with minimal systemic absorption. It is the
initial step taken for the management of keloids. Excision and steroid injections (for keloids only)
Excision and postoperative radiation (external beam or brachytherapy) Intralesional excision (keloids
only) Laser – to reduce redness (which may resolve in any event) Vitamin E or palm oil massage
• Pressure – Local molds or elasticated garments
• Silicone gel sheeting
• Intralesional steroid injection (triamcinolone). The benefit of this procedure is that the injection attains
a high drug concentration at the disease site with minimal systemic absorption. It is the initial step taken
for the management of keloids.
• Excision and steroid injections (for keloids only)
• Excision and postoperative radiation (external beam or brachytherapy)
• Intralesional excision (keloids only)
• Laser – to reduce redness (which may resolve in any event)
• Vitamin E or palm oil massage
• Pressure – Local molds or elasticated garments
• Silicone gel sheeting
• Intralesional steroid injection (triamcinolone). The benefit of this procedure is that the injection attains
a high drug concentration at the disease site with minimal systemic absorption. It is the initial step taken
for the management of keloids.
• Excision and steroid injections (for keloids only)
• Excision and postoperative radiation (external beam or brachytherapy)
• Intralesional excision (keloids only)
• Laser – to reduce redness (which may resolve in any event)

Page 8

1526
• Vitamin E or palm oil massage
Option A: Betamethasone
• Betamethasone is a corticosteroid and can be used to treat keloids and other forms of abnormal
scars.
• However, triamcinolone acetonide is the most common steroid that is used intralesionally.
Option C: Androgen
• Anti-androgen medicine is often recommended for keloid patients with hormonal disorder symptoms.
• However, it has yet to be officially included in keloid treatment regimens.
Option D: Hydrocortisone
• 0.5% hydrocortisone can be applied topically over keloids and hypertrophic scars.
• It is an over-the-counter treatment for keloids and hypertrophic scars.

Solution for Question 5:


Option B: Myofibroblast
• The healing of clean, uninfected wounds is called healing by primary intention.
• The wound's edges are well approximated, allowing for more rapid healing and less granulation
tissue.
• Within 0-24 hours, the inflammatory phase begins, involving vasodilation of capillaries and migration
of plasma leukocytes and macrophages into the wound space,
• Within 48 hours, the process of epithelialization begins. The macrophages remove the debris and
bacteria, stimulate the fibroblast, and promote angiogenesis.
• The fibroblasts are converted into myofibroblast, which has a contractile function and therefore helps
in the contraction of the wound. It helps in decreasing the size of the wound.
• The stage of remodelling, which occurs towards the end, is characterized by abundant collagen
accumulation.
• Type III collagen is replaced by type I collagen.
• This collagen maturation increases tensile strength in the wound, which is maximal at the 12th-week
post-surgery and represents 80 % of the uninjured skin strength.
• But the scar will never be as strong as its undivided tissue.
• The image below demonstrates the stages of wound healing.

Page 9

1527
Option A: Collagen
• Collagen increases the tensile strength of the scar; as remodelling increases, the strength of the
wound increases.
• However, collagen is not responsible for wound contraction.
Option C: Epithelial tissue
• The epithelialization process occurs within 48 hours because the cell undergoes rapid mitotic
divisions.
• It results in the migration of new cells to the wound site until the wound bed is completely covered.
Option D: Elastin
• Elastin provides mechanical elasticity.
• It reduces wound contraction and improves regeneration of the dermis.

Solution for Question 6:


Option 4: Stage 2 and 3 ulcers may be left to heal secondarily after debridement.
• Stage 2 and 3 pressure ulcers, as seen in the image, typically involve partial thickness skin loss or full
thickness tissue loss, respectively.
• Treatment often involves debridement to remove necrotic tissue and promote healing.
• Leaving the ulcers to heal secondarily after debridement allows for natural wound healing processes
to occur without the need for surgical intervention.
• This approach is suitable, particularly considering the patient's comorbidities such as systemic lupus
erythematosus, paraplegia, and steroid-induced diabetes, which may increase surgical risks.
Incorrect options:
Option 1: Surgery is the only modality of treatment.

Page 10

1528
• While surgery may be necessary in some cases of pressure ulcers, it is not the only treatment
modality, especially for stage 2 and 3 ulcers.
• Non-surgical interventions such as debridement and wound care are often utilized.
Option 2: A fasciocutaneous flap should be used.
• While flap surgery may be considered in severe cases of pressure ulcers, it is not always the most
appropriate initial treatment, especially for stage 2 and 3 ulcers.
• Flap surgery is typically reserved for more advanced or non-responsive cases.
Option 3: Herbal remedies with or without insulin are mostly useful than other remedies.
• Herbal remedies and insulin are not the primary treatments for pressure ulcers.
• Wound care, debridement, and addressing underlying factors such as pressure relief are essential in
managing pressure ulcers.

Solution for Question 7:


Option B: Large volume of haemoserous discharge
• In the Southampton grading system, Type IIIc wounds refer to wounds with a large volume of
haemoserous discharge. This indicates that there is significant fluid drainage from the wound, which
can delay healing and increase the risk of infection.
Southampton Wound grading system
0: Normal healing

I: Normal healing with mild bruising or erythema


• This category includes an uninfected clean operative wound with no inflammation.
Ia. Some bruising
Ib. Consideration bruising
Ic. Mild erythema
II: Erythema plus other signs of inflammation
• This category also includes a clean wound.
• Operations involving the biliary tract, appendix, vagina, and oropharynx are included in this category.
IIa. At one point
IIb. Around sutures
IIc: Along wound
IId. Around wound
III. Clear or haemoserous discharge
• This category includes a contaminated wound
• Open fresh accidental wounds, gross spillage from the gastrointestinal tract and incisions with
non-purulent discharge are included.

Page 11

1529
• Also includes open, fresh, accidental wounds and surgeries with major lacunas in sterile technique.
IIIa: At one point only (<2cm)
IIIb: Along wound (>2cm)
IIIc: Large volume
IIId: Prolonged (>3 days)
IV: Pus
• It includes a contaminated wound.
• Open fresh accidental wounds, gross spillage from the gastrointestinal tract and incisions with
non-purulent discharge are included.
IVa: At one point only (<2 cm)
IVb: Along wound (>2 cm)
V: Deep wound infection with or without tissue breakdown
Option A: Erythema along sutures
• Erythema along sutures comes under the type IIc category.
• It is considered a clean wound.
• Operations involving the biliary tract, appendix, vagina, and oropharynx are included in this category.
Option C: Prolonged haemoserous discharge
• Prolonged haemoserous discharge comes under the IIId category.
• It is considered a contaminated wound.
• Open fresh accidental wounds, gross spillage from the gastrointestinal tract and incisions with
non-purulent discharge are included.
Option D: Pus at one point
• Pus, at one point, comes under the IVa category.
• It is considered a contaminated wound.
• Emergency surgery for peritonitis and delayed presentation of traumatic wounds are included in this
category.

Solution for Question 8:


Option A: The wound and bowel should be covered with a moist towel
• Wound dehiscence is a surgical complication in which the edges of a surgical wound separate.
• There can be partial or total separation of a previously approximated wound suture. Partial
dehiscence: edges of an incision have been separated at one or more places. Complete dehiscence:
the entire incision reopens through all the layers of the skin and muscle.
• Partial dehiscence: edges of an incision have been separated at one or more places.
• Complete dehiscence: the entire incision reopens through all the layers of the skin and muscle.
• It occurs due to poor wound healing.

Page 12

1530
• This condition typically occurs 5 to 8 days post-surgery.
• Pain, redness, swelling, bleeding, and fluid drainage are signs of dehiscence.
• Serous or serosanguinous discharge from the wound is the first sign of dehiscence.
• The management of superficial dehiscence includes washing the wound with normal saline and
simple wound care (packing the wound with ribbon gauze). The wound is left to heal by secondary
intention. Vacuum-assisted closure is sometimes used to speed up the healing process.
• The management of complete dehiscence includes analgesia and broad-spectrum antibiotics,
covering the wound in saline-soaked gauze, and returning for immediate closure.
• If not treated promptly, wound dehiscence may lead to evisceration (burst abdomen), which is the
extrusion of peritoneal contents through the wound.
• Evisceration is a medical emergency, and it is essential to keep the wound and the peritoneal organs
moist by placing a damp, clean towel or bed sheet.
• Sterile saline should be used; bottled or tap water can be used if unavailable.
• The eviscerated organs are returned to the abdominal cavity under general anesthesia.
• Partial dehiscence: edges of an incision have been separated at one or more places.
• Complete dehiscence: the entire incision reopens through all the layers of the skin and muscle.
Option B: The wound will heal by secondary intention
• Superficial dehiscence heals by secondary intention, which may take several weeks.
• Before this, the wound is washed with saline water.
Option C: Surgical mesh is placed over the eviscerated bowel and then returned to the abdomen.
• The eviscerated bowel is rinsed with ringer lactate solution containing antibiotics and is then returned
to the abdomen.
• Mesh helps to close the wound.
Option D: Previous sutures should be immediately repaired
• The old sutures are removed.
• Damaged, infected, and dead tissue is removed.
• The wound is then closed with new sutures.

Solution for Question 9:


Option C: Green bottle fly larvae
• Maggot debridement therapy, also known as larvae debridement, treats chronic wounds.
• Sterile maggots of the green bottle fly are used.
• The flies used most often for maggot therapy are blow files of Calliphoridae.
• The blow fly species used most commonly is Lucilia sericata, the common green bottle fly.
• Maggots can be directly placed over the wound with a dressing covering them to keep them moist and
prevent them from escaping.

Page 13

1531
• Maggots have the following actions: Debridement: helps in the removal of necrotic and infected tissue.
It can debride the wound in a day or two. Disinfection: maggots release secretions that have
antimicrobial activity and thus can inhibit or kill infecting bacteria Acceleration of wound healing:
granulation tissue is stimulated after the wound has been debrided.
• Debridement: helps in the removal of necrotic and infected tissue. It can debride the wound in a day or
two.
• Disinfection: maggots release secretions that have antimicrobial activity and thus can inhibit or kill
infecting bacteria
• Acceleration of wound healing: granulation tissue is stimulated after the wound has been debrided.
• This therapy's primary advantage is that maggots separate the necrotic dead tissue from the healthy
tissue, making surgical debridement easier.
• Debridement: helps in the removal of necrotic and infected tissue. It can debride the wound in a day or
two.
• Disinfection: maggots release secretions that have antimicrobial activity and thus can inhibit or kill
infecting bacteria
• Acceleration of wound healing: granulation tissue is stimulated after the wound has been debrided.
Option A: Black soldier fly larvae
• The black soldier fly larvae can be used as food for fish, poultry, and pigs.
• It converts organic waste into high-quality nutrient food.
• In humans, it is an excellent source of protein.
Option B: Housefly larvae
• Housefly larvae are used to biodegrade manure.
• It helps break down and recycle organic matter.
Option D: Bluebottle fly larvae
• It is essential in aiding the breakdown of dead animal carcasses.

Solution for Question 10:


Correct Option D - Stage 4:
• The image given above is a stage 4 bed sore.
• Bed sores are also known as pressure sores, pressure ulcers, trophic ulcers, or decubitus ulcers.
• It results from injuries to the skin and underlying tissue from prolonged pressure on the skin.
• The constant pressure on the skin causes tissue ischemia and cessation of nutrients and oxygen
supply to the tissue, which eventually results in tissue necrosis.
• It is more common in individuals with decreased immobility.
• Staging of pressure sore: Stage 1- Early superficial ulcer: Non-blanchable erythema of skin without a
breach in the epidermis Stage 2- Late superficial ulcer: Partial thickness loss, including the epidermis
and the dermis. Stage 3- Early deep ulcer: Full-thickness skin loss involving the subcutaneous tissue
but not through the underlying fascia. Stage 4- Late deep ulcer: Full-thickness skin loss involving the

Page 14

1532
subcutaneous tissue, muscle, bone, tendon or joint.
• Stage 1- Early superficial ulcer: Non-blanchable erythema of skin without a breach in the epidermis
• Stage 2- Late superficial ulcer: Partial thickness loss, including the epidermis and the dermis.
• Stage 3- Early deep ulcer: Full-thickness skin loss involving the subcutaneous tissue but not through
the underlying fascia.
• Stage 4- Late deep ulcer: Full-thickness skin loss involving the subcutaneous tissue, muscle, bone,
tendon or joint.
• Prevention is the best treatment for pressure sores.
• Stage 1- Early superficial ulcer: Non-blanchable erythema of skin without a breach in the epidermis
• Stage 2- Late superficial ulcer: Partial thickness loss, including the epidermis and the dermis.
• Stage 3- Early deep ulcer: Full-thickness skin loss involving the subcutaneous tissue but not through
the underlying fascia.
• Stage 4- Late deep ulcer: Full-thickness skin loss involving the subcutaneous tissue, muscle, bone,
tendon or joint.
Incorrect options:
Option A: Stage 1
• It is an early superficial ulcer.
• Intact skin with non-blanchable erythema of a localized area.
Option B: Stage 2
• It is a late superficial ulcer.
• Partial thickness loss of the dermis presents as a shallow open ulcer.
Option C: Stage 3
• It is an early deep ulcer.
• Full-thickness tissue loss. Subcutaneous fat may be exposed, but the bone, tendon, and muscle are
not exposed.

Solution for Question 11:


Option D: Immersion of the affected part in water at 40°–44°C (104°–111.2°F)
• Immersion in warm water or covering the affected region is recommended treatment for frostbite. The
most efficient therapy is rapid warming by immersion in water just above body temperature (40°–44°C);
nevertheless, because the frostbitten area is numb and particularly susceptible, it should be protected
from trauma or excessive heat during treatment.
• Frostbite develops over the frozen tissue. The disease is one of morbidity rather than mortality. It most
frequently affects troops, those who work in the cold, the homeless, those who work outside in the
winter, mountaineers, and other cold-weather enthusiasts exposed to extreme cold. Serious morbidity
could result from frostbite.
• Frostbite therapy and diagnosis must be made correctly, and comorbidities and other illnesses must
also be addressed immediately.

Page 15

1533
• Many methods of treating frostbite have been tried throughout the years. These include Massage
Warm-water immersion or covering the affected area. Rapid warming by immersion in water slightly
above normal body temperature (40–44°C) is the most effective method; however, because the
frostbitten region is numb and especially vulnerable, it should be protected from trauma or excessive
heat during treatment. Elevation to minimize oedema. Administration of antibiotics and tetanus toxoid
Debridement of necrotic skin as needed.
• Massage
• Warm-water immersion or covering the affected area. Rapid warming by immersion in water slightly
above normal body temperature (40–44°C) is the most effective method; however, because the
frostbitten region is numb and especially vulnerable, it should be protected from trauma or excessive
heat during treatment.
• Elevation to minimize oedema.
• Administration of antibiotics and tetanus toxoid
• Debridement of necrotic skin as needed.
• Massage
• Warm-water immersion or covering the affected area. Rapid warming by immersion in water slightly
above normal body temperature (40–44°C) is the most effective method; however, because the
frostbitten region is numb and especially vulnerable, it should be protected from trauma or excessive
heat during treatment.
• Elevation to minimize oedema.
• Administration of antibiotics and tetanus toxoid
• Debridement of necrotic skin as needed.
Option A: Debridement of the affected part followed by silver sulfadiazine dressings
• Silver sulfadiazine dressing, usually used for burn patients, is better to avoid in patients with frostbite
because side effects like Pain, itching, or burning of the treated skin may happen. Skin and mucous
membranes (such as the gums) may become blue/grey, worsening it.
Option B: Administration of Corticosteroids
• Corticosteroids, which work as anti-inflammatory agents, may reduce inflammation and damage in
some cases, but it's still not a proven remedy.
Option C: Administration of vasodilators
• Vasodilator has a theoretical advantage for frostbite and may improve ischemia, and a potent
vasodilator Iloprost has been used as a potential treatment to prevent ischemia in frostbite, but it's not
easily available and has limited usage.

Solution for Question 12:


Option A: Open cholecystectomy for cholelithiasis
• It is an example of a clean-contaminated wound. These wounds lack unusual contamination. Clean
contaminated wounds enter the genital, alimentary, respiratory, or urinary tracts.
• Surgical wounds categorized based on bacterial contamination:

Page 16

1534
Clean wounds are those in which no part of the respiratory, gastrointestinal, or genitourinary tract is ent
ered. Examples include Herniorrhaphy and breast surgery.
Clean-contaminated wounds encompass those cases in which the above systems are entered but with
out evidence of active infection or gross spillage. Examples include elective cholecystectomy or electiv
e colon resection with adequate bowel preparation.
Contaminated wounds include active infection (perforated appendicitis with abscess) or gross spillage (
gunshot wounds with large or small bowel injuries).
Dirty wounds - post op infection already present in the wound before operation ,
associated with severe inflammation. For example- fecal peritonitis, necrotizing soft tissue infection.
Option B: Herniorrhaphy with mesh repair
• It is an example of a clean wound. Clean wounds are mostly closed, uninfected, and exhibit no
inflammation. Herniorrhaphy is usually a clean and closed wound, so this wound is in this category.
Option C: Lumpectomy with axillary node dissection
• It is also an example of a clean wound. These wounds do not enter alimentary, genital, urinary, or
respiratory tracts.
Option D: Appendectomy with walled-off abscess
• It is an example of contaminated wounds. They manifest in devitalized tissue and are frequently
caused by bacteria in the surgical field or perforated viscera.

Page 17

1535
Previous Year Questions
1. What is the diagnosis of a patient who presented with a 0.3 cm nodule on the left nasolabial fold,
which was subsequently excised and subjected to pathological examination?

A. Basal cell carcinoma


B. Melanoma
C. Squamous cell carcinoma
D. Nevus
----------------------------------------
2. Please determine the extent of burn injuries in a preschool-aged child portrayed in the attached
image.

A. 15-20%
B. 25-30%
C. 35-40%
D. 10-15%
----------------------------------------
3. Split-thickness skin graft for wound closure on the thigh includes:
(or)
What does the inclusion of a split-thickness skin graft in wound closure on the thigh entail?
A. Epidermis and full dermis

1536
B. Epidermis and variable part of the dermis
C. Epidermis, dermis, and fat
D. Epidermis only
----------------------------------------
4. A patient presents to the clinic with the lesion given in the image. He had a traumatic injury to the
chest one year ago and the scar has grown beyond the boundary of the initial injury. What is the most
likely diagnosis?

A. Hemangioma
B. Hypertrophic scar
C. Keloid
D. Neurofibroma
----------------------------------------
5. Which of the following statements is not true regarding the provided medical condition?

A. Repeated radical surgery every 24 hours may be required to adequately manage this condition
B. This is a case of Meleney's gangrene
C. The condition is caused by β hemolytic streptococcus and may sometimes be polymicrobial
D. Hyperbaric oxygen has no role in the treatment of this condition
----------------------------------------
6. Please determine the present condition.

Page 2

1537
A. Venous ulcer
B. Malignant ulcer
C. Arterial ulcer
D. Trophic ulcer
----------------------------------------
7. Which of the following statements is accurate concerning split-thickness and full-thickness skin
grafts? Primary contractures are more in split skin grafts Secondary contractures are more in split skin
grafts Primary contractures are more in full-thickness grafts Secondary contractures are more in
full-thickness grafts
A. 2,3
B. 2,4
C. 1,3
D. 1,4
----------------------------------------
8. Match the following: A. Marjolin ulcer over the dorsum of foot measuring 3cm x 2cm 1. Excision
biopsy B. 1.5 cm cervical lymph node in the neck 2. Core needle biopsy C. Soft tissue sarcoma over
mid -thigh 3. USG guided FNAC D. Solitary thyroid nodule 4. Wedge biopsy
A.Marjolin ulcer over the dorsum of foot measuring 3cm x 2cm 1.Excision biopsy
B.1.5 cm cervical lymph node in the neck 2.Core needle biopsy
C.Soft tissue sarcoma over mid -thigh 3.USG guided FNAC
D.Solitary thyroid nodule 4.Wedge biopsy

A. A-1, B-2, C-3, D-4


B. A-4, B-1, C-2, D-3
C. A-3, B-2, C-1, D-4
D. A-4, B-3, C-2, D-1
----------------------------------------
9. What is the classification of the burn in a female patient who has a 50% total body surface area
(TBSA) burn with dermis and subcutaneous tissue involvement and has presented to the emergency
department?

Page 3

1538
A. 1st
B. 2nd superficial
C. 2nd deep
D. 3rd degree
----------------------------------------
10. Rodent ulcer is
A. Squamous cell cancer
B. Basal cell cancer
C. Rhinophyma
D. Adenocarcinoma
----------------------------------------
11. What is the preferred management of angular dermoid cyst?
(or)
What is the recommended approach for managing the angular dermoid cyst?
A. Surgical excision
B. Radiotherapy
C. Antibiotics
D. Intralesional steroids
----------------------------------------
12. Which statements regarding a keloid scar are accurate?
A. The extant does not cross the wound margins
B. Wide excision of the keloid scar prevents recurrence
C. Histopathology of a keloid scar shows reduced collagen and increased vascularity
D. A keloid scar forms due to increase in the level of growth factors
----------------------------------------
13. Which of the following statements is false concerning Marjolin's ulcer?
A. It is seen in sites of chronic wounds or scars
B. It rapidly progresses to malignancy
C. It can progress to squamous cell carcinoma
D. It is radioresistant due to avascularity
----------------------------------------
14. What is the probable diagnosis for a 50-year-old woman who has a pigmented lesion measuring 1.5
cm on her face, which is rapidly increasing in size and causing itching?

Page 4

1539
A. Dysplastic nevus
B. Congenital melanocytic nevus
C. Acquired melanocytic nevus
D. Malignant melanoma
----------------------------------------
15. In a 40-year-old male, there is a lesion observed on the left ear. It has been gradually increasing in
size over the last three months. The patient has a history of chronic smoking, equivalent to 20 pack
years. Vital signs are normal, and the physical examination does not reveal any other notable findings.
The histopathologic analysis reveals the presence of keratin pearls. What is the probable diagnosis for
this patient?

A. Nodular melanoma
B. Basal cell carcinoma
C. Squamous cell carcinoma
D. Malignant melanoma
----------------------------------------
16. Which organism is responsible for causing erythrasma?
A. Corynebacterium diphtheria
B. Corynebacterium minutissimum
C. Corynebacterium tenuis
D. Diphtheriods

Page 5

1540
----------------------------------------
17. A 50-year-old male with a known case of the varicose veins in the lower limbs has developed
itching and ulceration around the ankle and calf region for a few days. Venous ulcers of the lower limb
usually develop in the area of the calf and ankle. Which ulcer is likely to develop in a long-standing
chronic venous ulcer?
A. Marjolin's ulcer
B. Aphthous ulcer
C. Pressure sores
D. Necrotizing fasciitis
----------------------------------------
18. A plastic surgeon is visited by a 36-year-old African-American male who has a recurring keloid
lesion in his left earlobe due to previous injuries. Despite undergoing surgery to remove the lesion, it
consistently returns after the procedure. During the examination, the patient has a painless swelling
measuring 2 cm in front of his tragus. What is the most suitable approach to manage this recurrent
keloid?
A. Excisional surgery
B. Intramarginal excision followed by radiation
C. Cryosurgery
D. Silicone gel sheeting
----------------------------------------
19. False regarding deep second-degree burns
(or)
False regarding deep second-degree burns
A. Heal by scar deposition
B. Less blanching
C. Damage to the deeper dermis
D. Painless
----------------------------------------
20. A 23-year-old man presented with a burn injury. He is covered with burns on his head & neck, both
upper limbs, front and back of the chest. Calculate the percentage of burns.
A. 27%
B. 36%
C. 45%
D. 18%
----------------------------------------
21. What is the primary concern in managing hand injuries?
A. Repair to tendons
B. Repair of skin cover

Page 6

1541
C. Repair of nerves
D. All
----------------------------------------
22. What is the probable diagnosis based on the provided image of the anus? It typically arises from
the obstruction and infection of an anal gland, leading to the accumulation of pus and fluid,
consequently causing a highly distressing abscess in the vicinity of the anus.

A. Hemorrhoids
B. Pelvirectal abscess
C. Perianal abscess
D. Pilonidal sinus
----------------------------------------
23. What is the specific instrument utilized for extracting the graft from the healthy region during split
thickness skin grafting?

A. Dermatome
B. Silver’s knife
C. Catlin amputating knife
D. Humby knife
----------------------------------------

Correct Answers

Page 7

1542
Question Correct Answer

Question 1 1
Question 2 2
Question 3 2
Question 4 3
Question 5 4
Question 6 4
Question 7 1
Question 8 2
Question 9 4
Question 10 2
Question 11 1
Question 12 4
Question 13 2
Question 14 4
Question 15 3
Question 16 2
Question 17 1
Question 18 2
Question 19 4
Question 20 3
Question 21 1
Question 22 3
Question 23 4

Solution for Question 1:


Correct option A:
• Basal cell carcinoma is a skin cancer most commonly found in sun-exposed body areas, such as the
face. It often presents as a small, shiny, pearly nodule that may bleed easily. The nodule, in this case,
could be a basal cell carcinoma, but further information is needed to confirm the diagnosis.
Incorrect options:
Option B. Melanoma is a
type of skin cancer that arises from the pigment-producing cells in the skin. It often presents as a dark
or multicolored mole that may be irregular in shape or size. Based on the size and location of the nodul
e described in this case, it is less likely to be melanoma.
Option C. Squamous cell carcinoma is a
skin cancer often associated with chronic sun exposure. It typically presents as a scaly or crusty growt

Page 8

1543
h that may bleed easily. Like basal cell carcinoma, it is possible that the nodule, in this case, could be a
squamous cell carcinoma, but further information would be needed to confirm the diagnosis.
Option D. A nevus, also known as a mole, is a
benign skin growth composed of pigment-producing cells. While the nodule, in this case, could be a
nevus, further information is needed to confirm the diagnosis.

Solution for Question 2:


Correct option B.
• 25-30% is the correct answer based on the Rule of Nines estimation method.
• Back fully burn 18% + some upper and lower chest 9% approx so in total approx 25 to 30%
Incorrect options:
Option A. 15-20%: This option is incorrect as the burns appear to involve a
larger percentage of the body surface area than this range.
Option C. 35-40%: This option is incorrect as the burns involve a
smaller percentage of the body surface area than this range.
Option D. 10-15%: This option is incorrect as the burns involve a
larger percentage of the body surface area than this range.

Solution for Question 3:


Answer Option B
• Split thickness skin graft (STSG) is a surgical technique used to repair skin defects by transferring a
thin layer of skin from a donor site to the recipient site. The graft includes the epidermis and a variable
thickness of the underlying dermis.
• The answer to the given question is option b, which states that STSG for wound closure on the thigh
includes the epidermis and variable parts of the dermis.
• The thickness of the dermis included in the graft can vary based on the location of the donor site and
the recipient site. The thickness of the graft is typically measured in millimeters, and the amount of
dermis included is referred to as the "split thickness." STSG is usually used for wound closure when the
underlying tissue is healthy, and the wound bed can support the graft.
• The donor site for STSG can be taken from the same patient (autograft), or from a different individual
(allograft or xenograft). Autografts are preferred because they have a lower risk of rejection and a
higher chance of successful integration. However, the size of the donor site is limited, and multiple sites
may be required for larger wounds.
Incorrect Choices:
• Option a states that the split-thickness skin graft includes the epidermis and full dermis. This is
incorrect, as split-thickness skin grafts do not include the full thickness of the dermis. Instead, only a
portion of the dermis is harvested along with the epidermis.
• Option c states that the split-thickness skin graft includes the epidermis, dermis, and fat. This is
incorrect, as split-thickness skin grafts do not include the subcutaneous fat layer.

Page 9

1544
• Option d states that the split-thickness skin graft includes the epidermis only. This is also incorrect, as
split-thickness skin grafts include a portion of the dermis as well.

Solution for Question 4:


Correct Option C - Keloid:
Based on the provided history of a traumatic injury to the chest a year ago and the appearance of the l
esion that is growing beyond the boundary of the initial injury, the most likely diagnosis is a
keloid. Keloids are benign overgrowths of scar tissue that often form at the site of skin injury.
Incorrect Choices:
Option A - Hemangiomas: They are benign tumors of blood vessels. They are typically present from bir
th or appear in early childhood and do not form in response to trauma.
Option B - Hypertrophic scar: It is like a keloid and is a result of an overactive healing process. Howeve
r, unlike keloids, they do not extend beyond the original wound boundaries.
Option D - Neurofibromas: They are benign nerve sheath tumors. They are associated with genetic con
ditions such as neurofibromatosis rather than trauma.

Solution for Question 5:


• The statement "Hyperbaric oxygen has no role in the treatment of this condition" is incorrect.
Hyperbaric oxygen therapy (HBOT) can actually play a critical role in the management of necrotizing
soft tissue infections, such as Fournier's gangrene or Meleney's gangrene, by enhancing the killing of
bacteria and promoting wound healing. The given condition is a non-necrotizing soft tissue infection
and thus, HBOT cannot be used to manage this condition.
Incorrect Choices:
• Option a. Repeated radical surgery every 24 hours may be required to adequately manage this
condition: This statement is true. Necrotizing soft tissue infections often require aggressive
debridement and multiple surgeries to control the infection.
• Option b. This is a case of Meleney's gangrene: Meleney's gangrene is a type of necrotizing infection
with extremely painful lesions formed after trauma or surgery and causes ulcers in lesions surfaced by
black scar, so this statement is wrong.
• Option c.The condition is caused by beta hemolytic streptococcus and may sometimes be
polymicrobial: This statement is true. Necrotizing soft tissue infections can be caused by a variety of
organisms, including beta-hemolytic streptococci, and are often polymicrobial.

Solution for Question 6:


• The correct answer is Trophic ulcer. Trophic ulcers, also known as neuropathic ulcers, develop due to
decreased sensation and a reduced ability to protect the skin, often due to underlying conditions such

Page 10

1545
as peripheral neuropathy or poor circulation. They typically present with a well-defined border and a
pale or pink wound bed, often seen in weight-bearing areas where pressure or friction commonly
occurs.
Incorrect Choices:
• Option a. Venous ulcer: These ulcers often occur due to venous insufficiency, typically found on the
lower leg, just above the ankle. Irregular borders and a red or pink wound bed characterize them.
• Option b. Malignant ulcer refers to ulcers associated with malignancy, such as squamous cell
carcinoma or melanoma. These ulcers often have irregular, raised, and indurated margins.
• Option c. Arterial ulcer: Arterial or ischemic ulcers result from poor blood circulation, especially in the
arteries. They typically occur on the feet, toes, or ankles and are characterized by well-defined borders,
a yellow or black wound bed, and often pain.

Solution for Question 7:


Correct Option A - 2, 3:
• Primary contraction is the immediate recoil of the skin graft that occurs more markedly in full-thickness
skin grafts.
• Secondary contraction occurs after the skin graft healing process and is more common in
split-thickness skin grafts.
Incorrect Options: Option B, C and D are incorrect.

Solution for Question 8:


Correct Option B - A-4, B-1, C-2, D-3:
A - Marjolin ulcer over the dorsum of foot measuring 3cm x 2cm: The Marjolin ulcer is a
type of malignant transformation that can occur in a chronic non-healing wound or scar. a
wedge biopsy is commonly performed. This technique involves removing a
small wedge-shaped tissue sample from the nodule for pathological examination.
B - 1.5 cm cervical lymph node in the neck: To evaluate a
lymph node, an excisional biopsy is commonly used.
C - Soft tissue sarcoma over mid-thigh: To obtain a tissue sample from a
soft tissue sarcoma for diagnosis, the technique involves obtaining a
small cylindrical core of tissue from the lymph node for pathological examination.
D - Solitary thyroid nodule: For the evaluation of a solitary thyroid nodule, an ultrasound-guided fine-ne
edle aspiration cytology (FNAC) is often performed. This technique involves using ultrasound guidance
to guide the needle into the tumor and aspirating cells for cytological examination.
In summary, the correct matching is:
• A - 4 (Wedge biopsy)
• B - 1 (Excision biopsy)
• C - 2 (Core needle biopsy)

Page 11

1546
• D -3 (USG guided FNAC)

Solution for Question 9:


Correct option:
Option D: 3rd degree
Option d, "3rd degree," is the correct answer for the given scenario.
A 3rd degree burn, also known as a full-thickness burn, involves the complete destruction of the epider
mis, dermis, and extends into the subcutaneous tissue. It typically appears as a white or charred area
and is often painless due to the destruction of nerve endings. In the given scenario, with 50% total bod
y surface area (TBSA) involvement and the inclusion of dermis and subcutaneous tissue, it is classified
as a 3rd degree burn.
Incorrect options:
Option A: 1st degree burn: A 1st degree burn, also known as a
superficial burn, only affects the outermost layer of the skin, known as the epidermis.
Option B: 2nd degree superficial burn: A 2nd degree superficial burn, also called a
partial-thickness burn, involves both the epidermis and a portion of the underlying dermis.
Option C: 2nd degree deep burn: A 2nd degree deep burn, also a partial-thickness burn, affects the epi
dermis and extends deeper into the dermis, involving the deeper layers of the skin. It often presents wit
h redness, pain, blistering, and severe swelling.

Solution for Question 10:


Correct option:
Option B:
• "Rodent ulcer" is used to describe basal cell cancer, which is a common type of skin cancer that
arises from basal cells in the skin.
• It is important to note that the term "Rodent ulcer" is not a medically recognized term, and the
condition is more accurately referred to as basal cell carcinoma.

Incorrect options:
Option A. Squamous cell cancer: Squamous cell cancer, also known as squamous cell carcinoma, is a
type of skin cancer that arises from squamous cells. It typically presents as a firm, red nodule or a
flat lesion with a scaly or crusty surface. While squamous cell carcinoma is a
common type of skin cancer, it is not specifically associated with the term "Rodent ulcer."
Option C. Rhinophyma: Rhinophyma is a
condition characterized by the thickening and enlargement of the nose, often resulting in a bulbous or ir
regular appearance. It is typically associated with severe forms of rosacea and is not related to "Roden
t ulcer," which specifically refers to a type of skin cancer.

Page 12

1547
Option D. Adenocarcinoma: Adenocarcinoma is a type of cancer that originates in glandular cells. It ca
n occur in various organs, such as the lung, colon, breast, and prostate. However, it is not associated
with the term "Rodent ulcer," which specifically refers to a type of skin cancer.

Solution for Question 11:


Correct Option A.
• The above condition is an angular dermoid cyst.
• An angular dermoid occurs at the outer angle of the eye, beneath or in close proximity to the eyebrow.
These are common peri-orbital tumors in children.
• Characteristically benign and slow growing, they are tumors of embryonic origin.
• External angular dermoid cysts can be excised using a minimally invasive subcutaneoscopic
technique. The technique that involves placing incisions above the hairline to avoid scarring on the
face.
Incorrect Options
Option B and C: Radiotherapy and antibiotics : Angular dermoid is a benign lesion and surgical excisio
n is the option because of the complications depending on their size and location.
Option D: Intralesional steroids : ILS/Intralesional steroids are indicated as standard treatment for skin
diseases like keloids, alopecia areata, and circumscribed plaques of dermatitis.

Solution for Question 12:


Correct Option D: A keloid scar forms due to increase in the level of growth factors
• Keloids result from an overactive healing response to an injury.
Incorrect Options;
Option A: The extent of a keloid scar can cross the wound margins- Keloid scars are characterized by t
heir tendency to extend beyond the original wound boundaries. Unlike normal scars, which typically sta
y within the limits of the wound, keloids can grow and spread beyond the initial injury site.
Option B: Wide excision of a keloid scar does not necessarily prevent recurrence- Keloid scars have a
high recurrence rate even after surgical removal. While excision may temporarily remove the keloid, it
does not guarantee that it won't reappear. Keloids have a complex and poorly understood formation pr
ocess involving various factors, and surgical removal alone may not address all underlying causes.
Option C: Histopathology of a keloid scar does not typically show reduced collagen and increased vasc
ularity- Keloid scars are characterized by excessive collagen deposition, leading to the formation of rais
ed, firm, and thickened scars. The histopathological examination of keloids typically reveals an abunda
nce of collagen fibers in irregular arrangements. Increased vascularity is not a
consistent finding in keloids.

Solution for Question 13:

Page 13

1548
Correct choice: A
• A marjolin’s ulcer usually occurs on long-standing chronic wound scars. It presents as a non-healing
ulcer with elevated margins that grow rapidly. A marjolin’s ulcer can become malignant in some cases
and can progress to squamous cell carcinoma. It is usually treated by surgical excision using the Mohs
surgery where removal of the lesion along with a 1-2 cm margin is done.
Incorrect choices:
Option B. It is seen in sited of chronic wounds or scars: This statement regarding marjolin’s ulcer is tru
e. Hence the choice is incorrect.
Option C. It can progress to squamous cell carcinoma: It has a greater chance of progressing to squam
ous cell carcinoma rather than basal cell carcinoma. Hence this statement is true.
Option D. It is radioresistant due to avascularity: This statement is also true as marjolin’s ulcer has less
vascularity and can be radioresistant.

Solution for Question 14:


Correct Option: D
Based on the description provided, the likely diagnosis for the 1.5 cm pigmented lesion on the face that
is growing rapidly in size and associated with itching is Malignant Melanoma.
Explanation of each option:
Option A: Dysplastic nevus: Dysplastic nevus, also known as atypical mole, is a benign pigmented lesi
on. It may have irregular borders, variation in color, and larger size compared to common acquired mel
anocytic nevi. However, dysplastic nevi typically do not exhibit rapid growth or associated symptoms su
ch as itching.
Option B: Congenital melanocytic nevus: Congenital melanocytic nevi are pigmented lesions present at
birth or shortly after. They can vary in size and may grow over time, but they are typically stable and d
o not exhibit rapid growth or associated itching as described in the case.
Option C: Acquired melanocytic nevus: Acquired melanocytic nevi, commonly known as moles, are be
nign pigmented lesions that develop after birth. They can vary in size, shape, and color. While some ac
quired nevi may undergo changes over time, such as slight growth or color variation, rapid growth and
associated itching are not typical features.
Option D: Malignant melanoma: Malignant melanoma is a
type of skin cancer that arises from melanocytes, the pigment-producing cells. It can present as a pigm
ented lesion that exhibits rapid growth, change in size, shape, or color, and associated symptoms like it
ching or bleeding. Given the description of the lesion in the case, malignant melanoma is the most likel
y diagnosis and should be evaluated further.

Solution for Question 15:


Correct Option: C
• Based on the information provided, the most likely diagnosis for the lesion over the left ear in the
40-year-old chronic smoker with histopathologic evaluation showing keratin pearls is Squamous Cell
Carcinoma.

Page 14

1549
• Squamous cell carcinoma: Squamous cell carcinoma is a type of skin cancer that arises from the
squamous cells of the epidermis. It commonly occurs in sun-exposed areas and can be associated with
chronic sun exposure, smoking, and other risk factors. It typically presents as a scaly, thickened, or
ulcerated lesion. Histopathologically, the presence of keratin pearls, which are concentric collections of
keratin within tumor cells, is characteristic of squamous cell carcinoma.
Incorrect options:
Option A: Nodular melanoma: Nodular melanoma is a type of melanoma, which is a
malignant skin cancer originating from melanocytes. It typically appears as a darkly pigmented, elevate
d nodule with irregular borders. It is not associated with the presence of keratin pearls, as mentioned in
the case.
Option B: Basal cell carcinoma: Basal cell carcinoma is the most common type of skin cancer. It often
presents as a slowly growing, non-healing ulcer or a pearly papule with telangiectasias (tiny blood vess
els). It is not associated with the presence of keratin pearls seen in the histopathologic evaluation.
Option D: Malignant melanoma: Malignant melanoma is a type of skin cancer originating from melanoc
ytes. It usually presents as an asymmetrical, pigmented lesion with irregular borders and may exhibit v
arious colors within the lesion. The presence of keratin pearls is not a
characteristic feature of malignant melanoma.
Based on the given description and histopathologic findings of keratin pearls, the most likely diagnosis i
s Squamous Cell Carcinoma.

Solution for Question 16:


Correct Answer : B.
• The organism that commonly causes erythrasma is Corynebacterium minutissimum. Erythrasma is a
bacterial skin infection characterized by red or brown patches on the skin, typically occurring in skin
folds such as the groin, armpits, and between the toes. Corynebacterium minutissimum is a
Gram-positive bacterium that colonizes the skin and can lead to the development of erythrasma. It is
important to note that other factors, such as warm and humid environments and poor hygiene, can
contribute to the development of erythrasma as well.
Incorrect options:
Option A. Corynebacterium diphtheria is the wrong answer
Option C. Corynebacterium tenuis is the wrong answer
Option D. Diphtheriods is the wrong answer

Solution for Question 17:


Correct Option: A.
• Marjolin's ulcer refers to a malignant transformation (squamous cell carcinoma) that can occur in a
long-standing chronic wound or scar.

Page 15

1550
• In the case of a long-standing chronic venous ulcer, there is a risk of developing Marjolin's ulcer over
time.
• Venous ulcers typically occur in the lower limbs, particularly in the calf and ankle region, due to
chronic venous insufficiency.
Incorrect Options:
Option B. Aphthous ulcer: Aphthous ulcers, also known as canker sores, are shallow and painful ulcers
that occur in the oral cavity, not in the lower limbs.
Option C. Pressure sores: Pressure sores, also known as bedsores or decubitus ulcers, develop over b
ony prominences due to prolonged pressure and lack of blood flow. They are not specific to the calf an
d ankle region.
Option D. Necrotizing fasciitis: Necrotizing fasciitis is a severe bacterial infection that affects the deeper
layers of the skin and subcutaneous tissue. It is not specific to the calf and ankle region and is not dire
ctly associated with long-standing venous ulcers

Solution for Question 18:


Correct Option: B.
• Keloids are abnormal scars that result from an exaggerated healing response. They tend to recur after
surgical excision.
• Intramarginal excision, which involves removing the keloid while keeping the wound edges within the
boundaries of the original scar, followed by radiation therapy, has shown good results in preventing
keloid recurrence.
• Radiation helps to prevent excessive scar formation.
Incorrect Options:
Option A. Excisional surgery: Simple excision of the keloid without additional measures is not likely to p
revent recurrence, as keloids have a high tendency to come back after surgery.
Option C. Cryosurgery: Cryosurgery involves using extreme cold to destroy abnormal tissue. While it m
ay be useful in some cases, it is generally not considered the primary treatment for recurrent keloids.
Option D. Silicone gel sheeting: Silicone gel sheeting is a non-invasive treatment option that can be us
ed to manage scars, including keloids. However, in the case of a recurrent keloid that has not respond
ed to previous treatments, more aggressive approaches like intramarginal excision and radiation are ty
pically required.

Solution for Question 19:


Correct Option D: Painless
This statement is false. Deep second-degree burns are characterized by damage to the nerve endings
in the dermis, which can result in varying degrees of pain. The affected area may be sensitive, and pai

Page 16

1551
n can be experienced during wound care, dressing changes, and during the healing process.
Incorrect options
Option A: Heal by scar deposition
This statement is true for deep second-degree burns. Deep second-degree burns, also known as deep
partial-thickness burns, extend deeper into the dermis than superficial second-degree burns. These bur
ns typically result in the destruction of the epidermis and damage to the deeper dermis. Due to the exte
nt of tissue damage, deep second-degree burns have a
higher likelihood of healing by scar deposition, as they often require a
more prolonged and complex healing process compared to superficial burns.
Option B: Less blanching
This statement is true. Blanching refers to the temporary whitening of the skin when pressure is applie
d and blood flow is momentarily interrupted. Deep second-degree burns typically result in damage to th
e blood vessels, which can lead to an impaired blanching response. As a
result, deep second-degree burns often exhibit less blanching compared to superficial burns.
Option C: Damage to the deeper dermis
This statement is true for deep second-degree burns. These burns extend beyond the superficial layers
of the skin and involve damage to the deeper dermis. This can include damage to blood vessels, swea
t glands, hair follicles, and connective tissue, leading to a
more significant injury compared to superficial burns.

Solution for Question 20:


Correct Option C:
The Rule of Nines is commonly used to estimate the percentage of burns in adults. According to this ru
le:
• The head and neck account for 9% of the total body surface area (TBSA).
• Each upper limb (including the front and back) accounts for 9% (9% for the right arm and 9% for the
left arm).
• The front and back of the chest together account for 18%.
• The total percentage of burns is the sum of the percentages assigned to each affected area.
Calculating the percentage of burns based on the given scenario:
• Head and neck: 9%
• Both upper limbs: 9% + 9% = 18%
• Front and back of the chest: 18%
Adding these percentages together, we have: 9% + 18% + 18% = 45%
Therefore, the estimated percentage of burns for the 23-year-old man in the given scenario is 45%.

Page 17

1552
Incorrect Options:
Option A. 27%: This option does not accurately represent the extent of burns described in the scenario.
It underestimates the total percentage of burns.
Option B. 36%: This option also underestimates the total percentage of burns and does not reflect the
extent of burns described in the scenario.
Option D. 18%: This option is incorrect because it represents only the percentage of burns on the front
and back of the chest. It does not consider the burns on the head, neck, and upper limbs mentioned in
the scenario

Solution for Question 21:


Correct Option A:
• Repair of Tendons: Tendons play a crucial role in the movement of the fingers and hand.
• If there is a significant tendon injury, such as a complete tendon laceration or avulsion, restoring the
continuity and function of the tendons becomes a top priority.
• Tendon repair is important to regain proper hand function and dexterity. In some cases, delayed or
inadequate tendon repair can result in long-term impairment of hand movement.
Incorrect Options:
Option B: Repair of skin cover: Skin cover refers to the integrity and closure of the skin over the injured
area. Skin closure is important to prevent infection, promote wound healing, and provide a protective b
arrier. Depending on the severity of the hand injury, the need for skin repair or closure may vary. Super
ficial lacerations or abrasions may require simple wound cleaning and closure, while more complex inju
ries may necessitate complex wound reconstruction or grafting.
Option C: Repair of nerves: Nerve injuries in the hand can result in sensory loss, motor deficits, and im
paired hand function. Nerve repair or reconstruction is crucial in cases of significant nerve injury, such
as complete transection or severe compression. The restoration of nerve function can help regain sens
ation, muscle control, and coordination in the hand. However, the prioritization of nerve repair may dep

Page 18

1553
end on the overall severity and complexity of the hand injury.

Solution for Question 22:


Correct Option: C: Perianal abscess
• A perianal abscess is an anorectal abscess that occurs only in the perianal area. Inflammatory bowel
illnesses, such as Crohn's disease, as well as trauma or malignant origins, can also be caused.
Patients who have recurring or complicated abscesses should be tested for Crohn's disease.
• A perianal abscess is a pus-filled boil that develops around the anus, rectum, or perineal area (the
space between your genitals and anus). It occurs when one of your anus glands becomes clogged and
infected. Pus and fluid accumulate in the anal gland, resulting in an excruciatingly painful abscess
around your anus (the hole in your buttocks).
Incorrect Options:
Option A: Hemorrhoids, unlike abscesses, are not infections. Hemorrhoids are bulging veins that can b
e treated with over-the-counter medications. Abscesses are typically more painful and larger than hem
orrhoids. Hemorrhoids are firmer to the touch, whereas abscesses are warm and painful. If left untreate
d, an abscess can cause complications such as fever, chills, and other symptoms. A
hemorrhoid is typically more of a bother.
Option B: A pelvirectal abscess develops when the intersphincteric abscess extends cranially past the l
evator muscles. The infection progresses semicircularly and primarily posteriorly, resulting in the creati
on of a horseshoe abscess or fistula. Such a condition does not represent the case here.
Option D: Pilonidal sinus describes a condition found in the natal cleft overlying the coccyx consisting o
f one or more, usually non-infected, midline openings that communicate with a fibrous track lined by gr
anulation tissue and containing hair lying loosely within the lumen. Pilonidal sinus is not indicative of th
e case presented here.

Solution for Question 23:


Correct Option D - Humby knife:
• The Humby knife, also known as a dermatome knife, is a specialized instrument used for skin grafting.
It is specifically designed to harvest thin slices of skin, making it suitable for split-thickness skin grafting
procedures. The Humby knife allows controlled removal of the epidermis and a portion of the dermis
from the donor site.
Incorrect Options:
Option A - Dermatome: A
dermatome is the correct instrument used for harvesting split-thickness skin grafts. It is a surgical instru
ment specifically designed for this purpose. The dermatome allows controlled and precise removal of t
he epidermis and a variable thickness of the dermis from the healthy donor area. The harvested graft is
then transferred to the recipient site, where it is secured and allowed to heal.

Page 19

1554
Option B - Silver's knife: Silver's knife, also known as a razor or scalpel, is a general surgical instrumen
t used for various purposes, including making incisions and excisions. While a scalpel may be used dur
ing certain steps of the split-thickness skin grafting procedure, it is not the primary instrument for graft h
arvesting.

Option C - Catlin amputating knife: Catlin amputating knife is a type of surgical instrument used for am
putations or limb removal. It is not typically used for harvesting split-thickness skin grafts, as its design
and purpose are different.

Page 20

1555
Page 21

1556
Cerebrovascular Diseases
1. A 4-month-old infant has undergone surgical treatment for meningomyelo encephalocele. At birth, an
operation was carried out in the posterior cranial fossa to partially replace cerebellar brain contents to
an intracranial position, leading to the development of hydrocephalus. On investigating progressive
hydrocephalus, it is noted that the cerebellar tonsils are herniated through the foramen magnum. The
neurosurgeon has diagnosed the baby with a syndrome that may include which of the following?
(or)
Which of the following is a part of Arnold-Chiari syndrome?
A. Fusion of the frontal lobes
B. Fusion of the temporal, parietal, and occipital lobes
C. Abnormal elongation of the medulla and lower cranial nerves
D. Partial or complete absence of the pituitary gland
----------------------------------------
2. A 1-month-old baby presents to the pediatric clinic with a fluctuating midline mass in the lower back
region. The swelling transilluminates along the vertebral column. The swelling is well-covered with skin.
The patient is thoroughly examined using plain X-rays, ultrasonography, and MRI to determine the
extent of swelling and neural tissue involvement. The swelling is shown in the image below. What is the
most common site of the given condition?
(or)
What is the most common location of a meningocele?

A. Lumbosacral
B. Occipital
C. Frontal
D. Thoracic
----------------------------------------
3. A 54-year-old male came to the emergency department with an altered state of consciousness for
the the last 6 hours. The patient complained of severe headache associated with vomiting earlier before
getting tired and confused. He has a history of hypertension. On examination, his blood pressure is
180/110 mmHg, and his heart rate is 90 beats per minute. His GCS score is 10/15. The neurologist
orders a plain CT brain and a CT angiogram of the brain, which reveals a ruptured cerebral aneurysm.
Which of the following is the most likely diagnosis?

1557
(or)
What is the diagnosis in the following CT image with a ruptured cerebral aneurysm?

A. Hemorrhagic Stroke
B. Ischemic Stroke
C. Subarachnoid Hemorrhage
D. Subdural Hematoma
----------------------------------------
4. Which one of the following is a component of the Cushing's triad?
A. Tachycardia
B. Hypotension
C. Pupillary dilatation
D. Respiratory irregularity
----------------------------------------
5. A 65-year-old male presents to the emergency department with seizures associated with high-grade
fever for the last ten days. The patient is a known case of diabetes mellitus type 2. On examination, the
patient appears confused and disoriented to time and place. There is no history of trauma. The
contrast-enhanced computed tomography of the brain, showed a walled, well-defined, ring-enhancing
mass.. Which of the following statements is true regarding central nervous system infections?
(or)
Which of the following is true regarding central nervous system infections?

A. Cranial osteomyelitis most frequently arises from the spread of bacteria through the bloodstream

Page 2

1558
B. Subdural empyema is ordinarily treated by administration of antibiotics without the need for surgical
drainage
C. Most common site of brain abscess due to hematogenous spread is the temporal lobe.
D. A bacterial brain abscess is accompanied by systemic signs of infection, such as fever or
leukocytosis
----------------------------------------
6. A 40-year-old male presents to the neurology department with complaints of severe headache
associated with blurred vision and vomiting for the last 1 hour. He is afebrile, has a blood pressure of
160/90 mmHg, a pulse rate of 47 beats per minute, and a respiratory rate of 27 breaths per minute. His
GCS is 12/15, and he has classical features of Cushing's Triad. His MRI brain reveals a posterior fossa
lesion leading to raised intracranial pressure. What would be the management of the raised ICP in this
case?
(or)
Which of the following is the management for raised ICP?
A. Hyperthermia
B. Hypercapnia
C. Decompressive craniectomy
D. Opioids
----------------------------------------
7. A 40-year-old male presents to the Neurology department with complaints of ongoing headache,
confusion, and drowsiness associated with vomiting for the last five weeks. He also reports slurred
speech and vision changes. He had a fall a few weeks back. He is afebrile, has a blood pressure of
130/90 mmHg, a pulse rate of 69 beats per minute, and a respiratory rate of 18 breaths per minute. His
examination suggests progressive neurological deterioration. What would be the diagnosis in this
case?
A. Acute subdural hematoma
B. Extradural hemorrhage
C. Chronic subdural hemorrhage
D. Skull fracture
----------------------------------------
8. A 45-year-old male presents to the surgical emergency department with a history of loss of
consciousness and slurred speech for the last 12 hours. The patient is a known smoker with a
20-pack-year history. The patient is normal and reports that he suddenly fell while walking in his house.
He lost consciousness for 5 minutes, after which the patient developed slurred speech. On
examination, the patient had unilateral hemiparesis, which later progressed to unilateral hemiplegia.
Which of the following is the most common cause of neurological deterioration in the case mentioned
above?
(or)
What is the cause of neurological deterioration in an-old male with loss of consciousness, slurred
speech for 12 hours, a 20-pack-year smoking history, and unilateral hemiparesis progressing to
hemiplegia ?
A. Rebleeding

Page 3

1559
B. Vasospasm
C. Embolism
D. Hydrocephalus
----------------------------------------
9. A 43-year-old mother gives birth to a newborn female infant through cesarean section after a full
term pregnancy due to a large occipital encephalocele diagnosed antenatally.. The pregnancy is
unplanned, and the mother did not take folic acid before conception, but she was on regular folic acid
supplements during the first trimester of pregnancy. The physical examination of the baby reveals two
large, soft blue-colored pouches; one is over the occiput, and the other is located over the nape of the
neck. Which condition could lead to a large head in this neonate?
(or)
Which conditions could contribute to a large head in a neonate diagnosed with two large, soft
blue-colored pouches over the occiput and nape of the neck?
A. Arnold Chiari malformation
B. Injury to the porous surface
C. Central canal injury
D. Arachnoidal block
----------------------------------------
10. Which of the following is true regarding Dandy-Walker syndrome?
(or)
A 1-year-old boy is brought in by his parents to the surgery department with complaints of a progressive
increase in head circumference for the last six months. They also report a delay in speech and difficulty
in coordinating movements. His examination findings reveal developmental delay, low tone (hypotonia),
and poor coordination. His MRI brain reveals hydrocephalus. A diagnosis of Dandy-Walker syndrome is
made. Which of the following statements about the syndrome is true?
A. Most common anterior fossa malformation
B. Consists of a cystic expansion of the 4th ventricle in the posterior fossa and midline cerebellar
hypoplasia
C. Managed by removal of the cystic cavity
D. Most common manifestation is microcephaly
----------------------------------------
11. What is the cause of spina bifida?
(or)
A 35-year-old woman, gravida 2 para 0, presents to the gynecology unit in the 10th week of her
pregnancy. She had a stillbirth three years back. A first-trimester screen, including an α-fetoprotein test
and chorionic villus sampling for chromosome analysis to determine the karyotype, is offered, but the
mother declines this even after understanding the risks involved. She has taken folic acid as advised
during the first trimester. A prenatal USG performed at 18 weeks reveals a swelling in the base of the
spine, in which meninges herniates through a bony defect. No other structural fetal defects are
identified. What is the possible cause of the condition mentioned above?
A. Defect in pedicle

Page 4

1560
B. Defect in the vertebral body
C. Defect in the fusion of vertebral arches
D. Defect in the transverse process
----------------------------------------
12. A 30-year-old female is brought to the emergency department by her parents in an unconscious
state following a head injury in a road traffic accident. She is afebrile, has a blood pressure of 160/90
mmHg, a pulse rate of 47 beats per minute, and a respiratory rate of 6 breaths per minute. Her GCS is
6/15. Non-contrast CT scan (NCCT) is performed, and the findings are given below. Which of the
following is true about this condition?
(or)
Which of the following is true regarding the condition in the following image?

A. Arterial bleeding
B. Lucid interval is negative
C. Full recovery is rare even after treatment
D. Conservative management should be done
----------------------------------------
13. A 77-year-old woman presents to the surgical outpatient department for evaluation of her lower
back pain for the last three weeks. It is exacerbated by walking and prolonged standing; occasionally, it
feels better by bending over. The physical examination reveals a thin, old woman who walks with a
cane with her back moderately flexed. Deep tendon reflexes are normal in her upper extremities but
absent in both lower extremities. She is has been referred for lumbosacral spine MRI. What will be the
most likely finding of this study?
(or)
Which is the most likely MRI finding in lumbar spinal stenosis?
A. Lumbar spinal stenosis
B. A fracture of the odontoid process
C. A herniated L3-L4 disc causing unilateral compression of the L4 root
D. Spinal cord compression at the level of L1 vertebra level
----------------------------------------

Correct Answers

Page 5

1561
Question Correct Answer

Question 1 3
Question 2 1
Question 3 3
Question 4 4
Question 5 4
Question 6 3
Question 7 3
Question 8 2
Question 9 1
Question 10 2
Question 11 3
Question 12 1
Question 13 1

Solution for Question 1:


Option C: Abnormal elongation of the medulla and lower cranial nerves
• Based on the surgical history of meningomyelo encephalocele, development of progressive
hydrocephalus, and herniation of the cerebellar tonsils through the foramen magnum, the most likely
diagnosis is Arnold-Chiari syndrome.
• In Arnold-Chiari syndrome, there is an abnormal elongation of the medulla and lower cranial nerves.
• There is the displacement of the cerebellar tonsils, medulla, and the fourth ventricle into the spinal
canal, sometimes with accompanying meningomyelocele. In such circumstances, the cord cavity
connects with and may merely represent a dilated central canal.
• Additional features include the fusion of the corpora quadrigemina, leading to a "beaked tectum''; the
partial or complete absence of the corpus callosum; and microgyria.
• MRI is the most useful imaging study for diagnosing Chiari malformation.
• Patients with Chiari malformations with minimal or equivocal symptoms without syringomyelia can be
treated conservatively. Mild neck pain and headaches can be treated with analgesics, muscle
relaxants, and the occasional use of a soft collar. The symptomatic patients should be offered surgical
treatment.
• The goals of surgical treatment are decompression of the cervicomedullary junction and restoration of
normal CSF flow in the region of the foramen magnum.

Page 6

1562
Severity

Page 7

1563
Chiari Spectrum
Dandy-Walker Spectrum
Type I - Tonsillar herniation
Mega Cisterna Magna
Type II - Brainstem and vermis herniation, lumbar myelomeningocele
Dandy-Walker variant-Mild enlargement of 4th ventricle
Type III - cerebellar herniation into cervical myelomeningocele
Dandy-Walker Malformation - Massive enlargement of 4th ventricle with large posterior fossa
Option A: Fusion of the frontal lobes
• Based on the diagnosis of Arnold-Chiari syndrome, a fusion of frontal lobes is unlikely.
• Fusion of the frontal lobes is seen in the holoprosencephaly group of structural disorders.
Option B: Fusion of the temporal, parietal, and occipital lobes
• Based on the diagnosis of Arnold-Chiari syndrome, no fusion of the temporal, parietal, and occipital
lobes will be seen.
• In Arnold-Chiari syndrome, there is neither fusion of the frontal lobes nor the fusion of the temporal,
parietal and occipital lobes.
Option D: Partial or complete absence of the pituitary gland
• There is no pituitary anomaly in Arnold-Chiari syndrome.

Solution for Question 2:


Correct Option A - Lumbosacral:
• Based on the baby's age, complaint of a midline mass, positive transillumination, and the swelling
shown in the image, the most likely diagnosis is a meningocele.

Page 8

1564
• The commonest site for meningocele is the lumbosacral region.
• A simple meningocele consists of meninges and CSF protruding into the subcutaneous tissue through
a spinal defect. The skin over the meningocele is usually intact.
• A complex meningocele is associated with other spinal anomalies.
• Meningocele is typically asymptomatic and not associated with acute neurologic conditions.
• All patients must be assessed radiologically after birth.
• Magnetic resonance imaging (MRI) is the best diagnostic test to study the extent of neural tissue
involvement and assess any associated hydrocephalus.
• Ultrasonography to have a quick assessment of hydrocephalus can be helpful.
• Management is mainly a multidisciplinary approach. Surgery is recommended as soon as it can be
done for the large meningoceles. In suspected cases of meningitis, treatment is started with
prophylactic antibiotics and anticonvulsants at once.
Incorrect Options:
Option B,C,D

Solution for Question 3:


Option C: Subarachnoid Hemorrhage
• Based on the history of severe headaches associated with vomiting and altered state of
consciousness and the finding of a ruptured cerebral aneurysm in the CT angiogram, the most likely
diagnosis is a subarachnoid hemorrhage (SAH).
• Subarachnoid hemorrhage is an extravasation of blood into the subarachnoid space between the pia
and arachnoid membrane.
• The most common cause of SAH is a head injury (traumatic SAH).
• In the absence of trauma, SAH is termed spontaneous SAH.
• Causes of spontaneous SAH include: Intracranial aneurysms rupture (saccular 'berry' aneurysms)
account for around 85% of cases. Conditions associated with berry aneurysms include adult polycystic
kidney disease, Ehler-Danlos syndrome, and coarctation of the aorta. Arteriovenous malformation
Pituitary apoplexy Arterial dissection Mycotic (infective) aneurysms Perimesencephalic (an idiopathic
venous bleed)
• Intracranial aneurysms rupture (saccular 'berry' aneurysms) account for around 85% of
cases. Conditions associated with berry aneurysms include adult polycystic kidney disease,
Ehler-Danlos syndrome, and coarctation of the aorta.
• Arteriovenous malformation
• Pituitary apoplexy
• Arterial dissection
• Mycotic (infective) aneurysms
• Perimesencephalic (an idiopathic venous bleed)
• SAH on CT scan is shown below:

Page 9

1565
• Intracranial aneurysms rupture (saccular 'berry' aneurysms) account for around 85% of
cases. Conditions associated with berry aneurysms include adult polycystic kidney disease,
Ehler-Danlos syndrome, and coarctation of the aorta.
• Arteriovenous malformation
• Pituitary apoplexy
• Arterial dissection
• Mycotic (infective) aneurysms
• Perimesencephalic (an idiopathic venous bleed)

• Signs and symptoms occur before the rupture of the cerebral aneurysm. The most common
symptoms and signs are as follows: Headache- Worst headache of life or thunderclap headache
Dizziness Orbital pain Diplopia Visual loss Seizures
• Headache- Worst headache of life or thunderclap headache
• Dizziness
• Orbital pain
• Diplopia
• Visual loss
• Seizures
• A negative non-contrast CT scan followed by CT angiography of the brain rules out SAH with great
sensitivity.
• The treatment of subarachnoid hemorrhage from a ruptured cerebral aneurysm includes strict blood
pressure control, restriction of fluids, and anti-hypertensive medications.
• Rebleeding is the most common early complication of SAH.
• Surgical treatment to prevent rebleeding is by clipping the ruptured berry aneurysm.
• Oral nimodipine is a calcium channel blocker to prevent vasospasm after SAH.
• An early CSF drainage via a ventricular drain may lessen the chance of vasospasm.

Page 10

1566
• Treatment for symptomatic vasospasm involves the induction of hypertension, hypervolemia, and
hemodilution (triple H therapy). This therapy should be reserved for patients with aneurysms secured
by surgical clipping or endovascular techniques to lessen the chances of rebleeding.
• Headache- Worst headache of life or thunderclap headache
• Dizziness
• Orbital pain
• Diplopia
• Visual loss
• Seizures
Option A: Hemorrhagic Stroke
• In hemorrhagic stroke, bleeding occurs directly into the brain parenchyma.
• Patients with hemorrhagic stroke may present with focal neurologic deficits similar to those of
ischemic stroke but are usually more ill.
• Non-contrast computed tomography scan or magnetic resonance imaging (MRI) brain is the
investigation of choice.
Option B: Ischemic Stroke
• Acute ischemic stroke occurs due to thrombotic or embolic occlusion of a cerebral artery and is more
frequent than hemorrhagic stroke.
• A non-contrast CT scan is most commonly used in the acute evaluation of patients with suspected
acute stroke. Magnetic resonance angiography (MRA) can also be used, but it is not readily available in
emergencies.
Option D: Subdural Hematoma
• A subdural hematoma is a collection of blood underneath the inner layer of the dura but outside the
brain and arachnoid membrane.
• Computed tomography (CT) scan of the head is generally performed when a patient presents with
suspected acute subdural hematoma.

Solution for Question 4:


Option D: Respiratory irregularity
• Cushing's Triad is the classic presentation of raised intracranial pressure, which consists of the
following: Hypertension Bradycardia Respiratory irregularity
• Hypertension
• Bradycardia
• Respiratory irregularity
• During an increase in ICP, the sympathetic and the parasympathetic nervous systems are activated.
• In the first stage of the reflex, the sympathetic response activates alpha-1 adrenergic receptors,
causing constriction of the arteries in the body.
• This constriction raises the total blood flow resistance, elevating blood pressure.

Page 11

1567
• Baroreceptors in the aortic arch detect increased blood pressure and trigger a parasympathetic
response through the vagus nerve, inducing bradycardia.
• Raised ICP causes increased pressure on the brain stem, which controls involuntary breathing, and
changes in its homeostasis result in irregular respiratory patterns.
• Hypertension
• Bradycardia
• Respiratory irregularity
Option A: Tachycardia
• Tachycardia is not a feature of the triad.
Option B: Hypotension
• Hypotension is not a feature of Cushing's triad.
Option C: Pupillary dilatation
• Cushing's Triad includes hypertension, bradycardia, and irregular respiration.
• It does not have any association with pupillary dilatation.

Solution for Question 5:


Option D: A
bacterial brain abscess is accompanied by systemic signs of infection, such as fever or leukocytosis
• A brain abscess typically presents with systemic signs of infection, such as fever and leukocytosis,
consistent with the patient's clinical presentation of high-grade fever and disorientation.
• The presence of fits (seizures) and fever suggests an acute inflammatory process within the brain,
characteristic of a brain abscess.
• In diabetic patients, infections can be more severe and may lead to complications such as brain
abscesses.
Incorrect options:
Option A: Cranial osteomyelitis usually arises from direct extension of infection from adjacent structure
s, such as sinusitis or otitis, rather than spread through the bloodstream.
Option B: Subdural empyema, a collection of pus between the dura and arachnoid mater, typically requ
ires surgical drainage and antibiotic therapy for resolution.
Option C: Hematogenous abscesses show a predilection for the territory of the middle cerebral artery (i
.e., posterior frontal or parietal lobes), not the temporal lobe.

Solution for Question 6:


Option C: Decompressive craniectomy
• Based on the case discussed above, the patient has classical symptoms (nausea, vomiting, and
severe headache) of raised intracranial pressure (ICP), and the management should be a

Page 12

1568
decompressive craniectomy.
• Stepwise approach to the treatment of elevated intracranial pressure: Insert ICP monitor -
ventriculostomy or parenchymal device. General goals: Maintain ICP < 20 mmHg and CPP(Cerebral
perfusion pressure) at 60 mmHg. For ICP > 20-25 mmHg for > 5 min: Elevate head of the bed, midline
head position Osmotherapy - mannitol Sedation (e.gMorphine, Propofol, or Midazolam); add
neuromuscular blockers , if necessary (patient will require endotracheal intubation and mechanical
ventilation at this point if not done before) . Hyperventilation to maintain PaCO2 between 30-35 mmHg
Pressor therapy - Phenylephrine, Dopamine, or Norepinephrine to maintain adequate MAP to ensure
CPP of 60 mmHg.
• Insert ICP monitor - ventriculostomy or parenchymal device.
• General goals: Maintain ICP < 20 mmHg and CPP(Cerebral perfusion pressure) at 60 mmHg.
• For ICP > 20-25 mmHg for > 5 min: Elevate head of the bed, midline head position Osmotherapy -
mannitol Sedation (e.gMorphine, Propofol, or Midazolam); add neuromuscular blockers , if necessary
(patient will require endotracheal intubation and mechanical ventilation at this point if not done before) .
Hyperventilation to maintain PaCO2 between 30-35 mmHg Pressor therapy - Phenylephrine,
Dopamine, or Norepinephrine to maintain adequate MAP to ensure CPP of 60 mmHg.
• Elevate head of the bed, midline head position
• Osmotherapy - mannitol
• Sedation (e.gMorphine, Propofol, or Midazolam); add neuromuscular blockers , if necessary (patient
will require endotracheal intubation and mechanical ventilation at this point if not done before) .
• Hyperventilation to maintain PaCO2 between 30-35 mmHg
• Pressor therapy - Phenylephrine, Dopamine, or Norepinephrine to maintain adequate MAP to ensure
CPP of 60 mmHg.
• Consider second-tier therapies for refractory elevated ICP: High-dose barbiturate therapy
("Pentobarbital coma”) Aggressive hyperventilation to maintain PaCO2 < 30 mmHg Hypothermia
Hemicraniectomy (decompressive craniectomy)
• High-dose barbiturate therapy ("Pentobarbital coma”)
• Aggressive hyperventilation to maintain PaCO2 < 30 mmHg
• Hypothermia
• Hemicraniectomy (decompressive craniectomy)
• Insert ICP monitor - ventriculostomy or parenchymal device.
• General goals: Maintain ICP < 20 mmHg and CPP(Cerebral perfusion pressure) at 60 mmHg.
• For ICP > 20-25 mmHg for > 5 min: Elevate head of the bed, midline head position Osmotherapy -
mannitol Sedation (e.gMorphine, Propofol, or Midazolam); add neuromuscular blockers , if necessary
(patient will require endotracheal intubation and mechanical ventilation at this point if not done before) .
Hyperventilation to maintain PaCO2 between 30-35 mmHg Pressor therapy - Phenylephrine,
Dopamine, or Norepinephrine to maintain adequate MAP to ensure CPP of 60 mmHg.
• Elevate head of the bed, midline head position
• Osmotherapy - mannitol
• Sedation (e.gMorphine, Propofol, or Midazolam); add neuromuscular blockers , if necessary (patient
will require endotracheal intubation and mechanical ventilation at this point if not done before) .
• Hyperventilation to maintain PaCO2 between 30-35 mmHg

Page 13

1569
• Pressor therapy - Phenylephrine, Dopamine, or Norepinephrine to maintain adequate MAP to ensure
CPP of 60 mmHg.
• Elevate head of the bed, midline head position
• Osmotherapy - mannitol
• Sedation (e.gMorphine, Propofol, or Midazolam); add neuromuscular blockers , if necessary (patient
will require endotracheal intubation and mechanical ventilation at this point if not done before) .
• Hyperventilation to maintain PaCO2 between 30-35 mmHg
• Pressor therapy - Phenylephrine, Dopamine, or Norepinephrine to maintain adequate MAP to ensure
CPP of 60 mmHg.
• High-dose barbiturate therapy ("Pentobarbital coma”)
• Aggressive hyperventilation to maintain PaCO2 < 30 mmHg
• Hypothermia
• Hemicraniectomy (decompressive craniectomy)
Option A: Hyperthermia
• Management of raised ICP includes hypothermia, not hyperthermia.
• Hypothermia reduces cerebral metabolism and cerebral blood flow, decreasing intracranial pressure.
Option B: Hypercapnia
• ICP is lowered by inducing hypocapnia, not by hypercapnia.
• Inducing hypocapnia via hyperventilation reduces the partial pressure of arterial carbon dioxide
(PaCO2), inciting vasoconstriction in the cerebral resistance arterioles.
• This constriction decreases cerebral blood flow, reduces cerebral blood volume, and decreases ICP.
Option D: Opioids
• High-dose barbiturate therapy ("Pentobarb coma") is given in cases with increased intracranial
pressure.
• Barbiturates reduce ICP by suppressing cerebral metabolism, leading to reduced cerebral blood
volume.

Solution for Question 7:


Option C: Chronic subdural hemorrhage
• In the case mentioned above, the patient has an ongoing headache and progressive neurological
deposits weeks after a fall. This is highly suggestive of chronic subdural hemorrhage.
• Subdural hematoma is a collection of blood deep to the dural layer of the meninges. The blood is not
within the brain's substance and is therefore called an 'extra-axial' or 'extrinsic lesion.'
• Chronic subdural hematoma (Chronic SDH) is a collection of blood within the subdural space that has
been present for weeks to months.
• Rupture of the small bridging veins within the subdural space causes slow bleeding. Elderly and
alcoholic patients are particularly at risk of subdural hematomas.

Page 14

1570
• Presentation is typically several weeks of progressive confusion, impaired consciousness, or
neurological deficit.
• Infants also have fragile bridging veins and can rupture in shaken baby syndrome.
• On CT imaging, they are crescentic in shape, not restricted by suture lines, and compress the brain
(‘mass effect’). Chronic SDH: a blood breakdown product collection at least three weeks old. Chronic
SDHs often occur in patients without a clear history of head trauma, as they may arise from minor
trauma. A chronic SDH >1 cm or any symptomatic SDH should be surgically drained
• Chronic SDH: a blood breakdown product collection at least three weeks old.
• Chronic SDHs often occur in patients without a clear history of head trauma, as they may arise from
minor trauma.
• A chronic SDH >1 cm or any symptomatic SDH should be surgically drained
• MX: Clinical observation coupled with serial imaging is a reasonable approach to patients with few
symptoms and small chronic subdural collections that do not cause mass effect. Treatment with
surgical evacuation through burr holes is usually successful, if a cranial drain is used postoperatively.
The fibrous membranes that grow from the dura and encapsulate the collection may require removal
with a craniotomy to prevent recurrent fluid accumulation.
• Chronic SDH: a blood breakdown product collection at least three weeks old.
• Chronic SDHs often occur in patients without a clear history of head trauma, as they may arise from
minor trauma.
• A chronic SDH >1 cm or any symptomatic SDH should be surgically drained
Option A: Acute subdural hematoma
• An acute subdural hematoma is a collection of fresh blood within the subdural space most commonly
caused by high-impact trauma.
• Since it is associated with high-impact injuries, there are often other underlying brain injuries. Acute
SDHs often occur in patients with a clear history of head trauma.
• Acute hematomas are bright white (hyperdense) on CT scan for approximately 3 days, after which
they fade to isodensity with brain, and then to hypodensity after 2 to 3 weeks
Option B: Extradural hemorrhage
• Extradural hemorrhage is bleeding into the space between the dura mater and the skull resulting from
acceleration-deceleration trauma or a blow to the side of the head.
• Clinical features manifest immediately after injury. Most epidural hematomas occur in the temporal
region due to rupture of the middle meningeal artery with features of raised intracranial pressure.
Option D: Skull fracture
• A skull fracture is a head injury with a break in the skull bone.
• Severe fractures can lead to complications, including bleeding, brain damage, leaking of
cerebrospinal fluid, infection, and seizures leading to the immediate presentation of the symptoms.

Solution for Question 8:


Option B: Vasospasm

Page 15

1571
• Clinical signs: Loss of consciousness, slurred speech, unilateral hemiparesis, and unilateral
hemiplegia suggest a cerebrovascular accident (CVA). This is most probably because of vasospasm.
• Types of CVA: Ischemic CVA- thrombotic or embolic Hemorrhagic CVA
• Ischemic CVA- thrombotic or embolic
• Hemorrhagic CVA
• Vasospasm Narrowing of the arteries at the base of the brain following subarachnoid hemorrhage
(SAH) occurs regularly. This vasospasm causes symptomatic ischemia and infarction in approximately
30% of patients and is the primary cause of delayed morbidity or death. Cigarette smoking is
associated with aneurysmal subarachnoid hemorrhage (SAH) and subsequent vasospasm.
• Narrowing of the arteries at the base of the brain following subarachnoid hemorrhage (SAH) occurs
regularly.
• This vasospasm causes symptomatic ischemia and infarction in approximately 30% of patients and is
the primary cause of delayed morbidity or death.
• Cigarette smoking is associated with aneurysmal subarachnoid hemorrhage (SAH) and subsequent
vasospasm.
• Ischemic CVA- thrombotic or embolic
• Hemorrhagic CVA
• Narrowing of the arteries at the base of the brain following subarachnoid hemorrhage (SAH) occurs
regularly.
• This vasospasm causes symptomatic ischemia and infarction in approximately 30% of patients and is
the primary cause of delayed morbidity or death.
• Cigarette smoking is associated with aneurysmal subarachnoid hemorrhage (SAH) and subsequent
vasospasm.

Option A: Rebleeding
• Recurrent cerebrovascular accidents are common, but one must have a history of previous CVA or
any other ischemic disease.
Option C: Embolism
• An embolus from another body part typically enters the bloodstream and travels to the brain to form
an embolic stroke.
• Cardiac illness or cardiac surgery are common causes of embolic strokes, which happen suddenly
without prior signs and symptoms.
Option D: Hydrocephalus
• Injuries or strokes can induce brain damage, which leads to hydrocephalus ex-vacuo. The ventricles
are more prominent than usual in these circumstances because the surrounding brain structures shrink.
• This disease is more of a "hydrocephalus look-alike" than a true hydrocephalus.

Solution for Question 9:


Option A: Arnold Chiari malformation

Page 16

1572
• The clinical signs are suggestive of myelomeningocele.
• Myelomeningocele is a type of spina bifida. Spina bifida occurs when the spine of a neonate does not
form normally during pregnancy.
• Etiologies associated with myelomeningocele: Arnold Chiari malformation (Type 2) Lack of folic acid
Exposure to viruses Exposure to radiation Genetics
• Arnold Chiari malformation (Type 2)
• Lack of folic acid
• Exposure to viruses
• Exposure to radiation
• Genetics
• Type II Arnold Chiari malformation is characterized by progressive hydrocephalus with
myelomeningocele.
• Arnold Chiari malformation (Type 2)
• Lack of folic acid
• Exposure to viruses
• Exposure to radiation
• Genetics

Option B: Injury to the porous surface


• Injury to the porous surface will not lead to a large sized head in a neonate.
• It is not associated with myelomeningocele.
Option C: Central canal injury
• Symptoms may include loss of fine control of movements in the upper limb, with relatively less
impairment of movements in the lower limbs.
• Sensory loss below the level of the injury and loss of bladder control may also occur, as well as
painful sensations such as burning, tinging, and dull pain.

Page 17

1573
• It is not associated with myelomeningocele.
Option D: Arachnoidal block
• The clinical features are unrelated to the defect caused by the arachnoidal block.

Solution for Question 10:


Option B: Consists of a
cystic expansion of the 4th ventricle in the posterior fossa and midline cerebellar hypoplasia
• Dandy Walker syndrome: Characterized by developmental failure of the roof of the 4th ventricle during
embryogenesis. It consists of a cystic expansion of the fourth ventricle in the posterior fossa and
midline cerebellar hypoplasia. Prevalence: 1 per 30,000 live births. The triad of Dandy-Walker
malformation. Hypoplasia of vermis Cephalad rotation of the vermian remnant and cystic dilatation of
the 4th ventricle extending posteriorly Enlarged posterior fossa with circular-lambdoid inversion. Clinical
features include: Macrocephaly (80% cases); associated with hydrocephalus (90% cases). Infants
present with a rapid increase in head size and prominent occiput. Transillumination of the skull may be
positive. Most children have evidence of long-tract signs (spasticity), cerebellar ataxia, and delayed
motor and cognitive milestones, probably due to the associated structural anomalies. Associations:
CNS abnormalities are present in 70% of cases (i.e. agenesis of the posterior cerebellar vermis
and corpus callosum) Cortical dysplasia Polymicrogyria MRI Brain is the investigation of choice.
Treatment: Managed by shunting the cystic cavity.
• Characterized by developmental failure of the roof of the 4th ventricle during embryogenesis.
• It consists of a cystic expansion of the fourth ventricle in the posterior fossa and midline cerebellar
hypoplasia.
• Prevalence: 1 per 30,000 live births.
• The triad of Dandy-Walker malformation. Hypoplasia of vermis Cephalad rotation of the vermian
remnant and cystic dilatation of the 4th ventricle extending posteriorly Enlarged posterior fossa with
circular-lambdoid inversion.
• Hypoplasia of vermis
• Cephalad rotation of the vermian remnant and cystic dilatation of the 4th ventricle extending
posteriorly
• Enlarged posterior fossa with circular-lambdoid inversion.
• Clinical features include: Macrocephaly (80% cases); associated with hydrocephalus (90% cases).
Infants present with a rapid increase in head size and prominent occiput. Transillumination of the skull
may be positive. Most children have evidence of long-tract signs (spasticity), cerebellar ataxia, and
delayed motor and cognitive milestones, probably due to the associated structural anomalies.
• Macrocephaly (80% cases); associated with hydrocephalus (90% cases).
• Infants present with a rapid increase in head size and prominent occiput.
• Transillumination of the skull may be positive.
• Most children have evidence of long-tract signs (spasticity), cerebellar ataxia, and delayed motor and
cognitive milestones, probably due to the associated structural anomalies.
• Associations: CNS abnormalities are present in 70% of cases (i.e. agenesis of the posterior cerebellar
vermis and corpus callosum) Cortical dysplasia Polymicrogyria

Page 18

1574
• CNS abnormalities are present in 70% of cases (i.e. agenesis of the posterior cerebellar vermis
and corpus callosum)
• Cortical dysplasia
• Polymicrogyria
• MRI Brain is the investigation of choice.
• Treatment: Managed by shunting the cystic cavity.
• Characterized by developmental failure of the roof of the 4th ventricle during embryogenesis.
• It consists of a cystic expansion of the fourth ventricle in the posterior fossa and midline cerebellar
hypoplasia.
• Prevalence: 1 per 30,000 live births.
• The triad of Dandy-Walker malformation. Hypoplasia of vermis Cephalad rotation of the vermian
remnant and cystic dilatation of the 4th ventricle extending posteriorly Enlarged posterior fossa with
circular-lambdoid inversion.
• Hypoplasia of vermis
• Cephalad rotation of the vermian remnant and cystic dilatation of the 4th ventricle extending
posteriorly
• Enlarged posterior fossa with circular-lambdoid inversion.
• Clinical features include: Macrocephaly (80% cases); associated with hydrocephalus (90% cases).
Infants present with a rapid increase in head size and prominent occiput. Transillumination of the skull
may be positive. Most children have evidence of long-tract signs (spasticity), cerebellar ataxia, and
delayed motor and cognitive milestones, probably due to the associated structural anomalies.
• Macrocephaly (80% cases); associated with hydrocephalus (90% cases).
• Infants present with a rapid increase in head size and prominent occiput.
• Transillumination of the skull may be positive.
• Most children have evidence of long-tract signs (spasticity), cerebellar ataxia, and delayed motor and
cognitive milestones, probably due to the associated structural anomalies.
• Associations: CNS abnormalities are present in 70% of cases (i.e. agenesis of the posterior cerebellar
vermis and corpus callosum) Cortical dysplasia Polymicrogyria
• CNS abnormalities are present in 70% of cases (i.e. agenesis of the posterior cerebellar vermis
and corpus callosum)
• Cortical dysplasia
• Polymicrogyria
• MRI Brain is the investigation of choice.
• Treatment: Managed by shunting the cystic cavity.
• Hypoplasia of vermis
• Cephalad rotation of the vermian remnant and cystic dilatation of the 4th ventricle extending
posteriorly
• Enlarged posterior fossa with circular-lambdoid inversion.
• Macrocephaly (80% cases); associated with hydrocephalus (90% cases).

Page 19

1575
• Infants present with a rapid increase in head size and prominent occiput.
• Transillumination of the skull may be positive.
• Most children have evidence of long-tract signs (spasticity), cerebellar ataxia, and delayed motor and
cognitive milestones, probably due to the associated structural anomalies.
• CNS abnormalities are present in 70% of cases (i.e. agenesis of the posterior cerebellar vermis
and corpus callosum)
• Cortical dysplasia
• Polymicrogyria
Option A: Most common anterior fossa malformation
• Dandy-Walker malformation is the most common posterior fossa malformation.
Option C: Managed by removal of the cystic cavity
• The shunting of the cystic cavity manages Dandy-Walker syndrome.
Option D: Most common manifestation is microcephaly
• Most common manifestation of Dandy-Walker syndrome is macrocephaly.

Solution for Question 11:


Option C: Defect in the fusion of vertebral arches
• Clinical sign: Swelling at the base of the spine where meninges herniate through bony defects.
• Pathophysiology: The maldevelopment of the ectodermal, mesodermal, and neuroectodermal tissues
causes this congenital pathology. Spina bifda is caused by a failure of fusion of the vertebral arches
and possibly the underlying neural tube.
• The maldevelopment of the ectodermal, mesodermal, and neuroectodermal tissues causes this
congenital pathology. Spina bifda is caused by a failure of fusion of the vertebral arches and possibly
the underlying neural tube.
• Spina bifida develops in the early weeks of pregnancy, frequently before a woman is aware that she is
pregnant.
• Taking folic acid preconception can help lower a woman's risk of having a baby with spina bifida.
• The maldevelopment of the ectodermal, mesodermal, and neuroectodermal tissues causes this
congenital pathology. Spina bifda is caused by a failure of fusion of the vertebral arches and possibly
the underlying neural tube.

Page 20

1576
• Causes are genetic, nutritional, and environmental risk factors, such as a family history of neural tube
defects and folate (vitamin B9) deficiency.
• Different treatment options for spina bifida: Initial surgery to repair the spine Treat hydrocephalus
Physiotherapy
• Initial surgery to repair the spine
• Treat hydrocephalus
• Physiotherapy
• Initial surgery to repair the spine
• Treat hydrocephalus
• Physiotherapy
Option A: Defect in pedicle
• Cervical spondylolysis and congenitally missing cervical pedicles are two uncommon abnormalities of
the posterior arch typically discovered by chance on neck radiographs.
Option B: Defect in the vertebral body
• Defects in the body cause scoliosis, kyphosis, and lordosis, typical adult spinal abnormalities.
• Kyphosis may take the shape of a hump. Lordosis is another term for swayback.
Option D: Defect in the transverse process
• Spina bifida is due to a defective fusion of the vertebral arches, not the transverse process.

Solution for Question 12:


Option A: Arterial bleeding
• In the case mentioned above, the patient's clinical features and NCCT image showing a
biconvex/lentiform hyperdensity is characteristic of an Epidural Hematoma (EDH).

Page 21

1577
• Epidural Hematoma: It is bleeding into the space between the dura mater and the skull. Often results
from acceleration-deceleration trauma or a blow to the side of the head. Most epidural hematomas
occur in the temporal region, where skull fractures cause a rupture of the middle meningeal artery. C/F:
EDH has a classic, three-stage clinical presentation that is probably seen in only 20% of cases. The
patient is initially unconscious from the concussive aspect of the head trauma. The patient then
awakens and has a “lucid interval,” while the hematoma subclinically expands. As the volume of the
hematoma grows, the decompensated region of the pressure-volume curve is reached, ICP increases,
and the patient rapidly becomes lethargic and herniates. Uncal herniation from an EDH classically
causes ipsilateral third nerve palsy and contralateral hemiparesis.
• It is bleeding into the space between the dura mater and the skull.
• Often results from acceleration-deceleration trauma or a blow to the side of the head.
• Most epidural hematomas occur in the temporal region, where skull fractures cause a rupture of the
middle meningeal artery.
• C/F: EDH has a classic, three-stage clinical presentation that is probably seen in only 20% of cases.
The patient is initially unconscious from the concussive aspect of the head trauma. The patient then
awakens and has a “lucid interval,” while the hematoma subclinically expands. As the volume of the
hematoma grows, the decompensated region of the pressure-volume curve is reached, ICP increases,
and the patient rapidly becomes lethargic and herniates. Uncal herniation from an EDH classically
causes ipsilateral third nerve palsy and contralateral hemiparesis.
• It is bleeding into the space between the dura mater and the skull.
• Often results from acceleration-deceleration trauma or a blow to the side of the head.
• Most epidural hematomas occur in the temporal region, where skull fractures cause a rupture of the
middle meningeal artery.
• C/F: EDH has a classic, three-stage clinical presentation that is probably seen in only 20% of cases.
The patient is initially unconscious from the concussive aspect of the head trauma. The patient then
awakens and has a “lucid interval,” while the hematoma subclinically expands. As the volume of the
hematoma grows, the decompensated region of the pressure-volume curve is reached, ICP increases,
and the patient rapidly becomes lethargic and herniates. Uncal herniation from an EDH classically
causes ipsilateral third nerve palsy and contralateral hemiparesis.
Option B: Lucid interval is negative
• A lucid interval (LI) is the period between regaining consciousness after a short period of
unconsciousness resulting from a head injury.
• Epidural hematoma has a positive lucid interval.
Option C: Full recovery is rare even after treatment
• Immediate surgical evacuation should be done in EDH and full recovery is expected after treatment
with a good prognosis.
Option D: Conservative management should be done
• Epidural hematoma is an intracranial bleed.
• Burr hole is made for immediate surgical evacuation.
• Open craniectomy for evacuation of the congealed clot and hemostasis is generally indicated for
EDH. In some cases, EDH can be caused by bony venous bleeding that is self-limited and may not
require surgical intervention. Generally, patients who meet all of the following criteria may be managed
conservatively: clot volume <30 cm3, maximum thickness <1.5 cm, and GCS score >8.

Page 22

1578
Solution for Question 13:
Option A: Lumbar spinal stenosis
• The clinical presentation of lower back pain associated with impaired sensation to light touch and
vibration and the absence of deep tendon reflexes in the lower extremities indicates a lower motor
neuron lesion. The clinical diagnosis is neurogenic claudication secondary to lumbar spinal stenosis.
• Lumbar spinal stenosis is the most likely finding in the MRI study. MRI is the diagnostic test of choice
for lumbar stenosis and typically shows an hourglass appearance on T2- weighted sagittal sequence.
• Lumbar spinal stenosis is commonly seen in elderly persons in whom (as a consequence of wear and
tear over the years) bony structures of the lumbar spine undergo hypertrophy and develop osteophytes.
These bony changes, in turn, lead to stenosis of the spinal canal and intervertebral foramina. Thus, the
result is bilateral compression and multiple lumbosacral nerve root dysfunction.
• LSS is most often acquired (75%) but can also be congenital or due to a mixture of both etiologies.
Congenital factors (achondroplasia and idiopathic), Acquired factors include degenerative diseases
(spondylosis, spondylolisthesis, and scoliosis), trauma, acromegaly, renal osteodystrophy,
hypoparathyroidism, and Paget’s disease.
• Bending over opens the lumbar canal and relieves the stenosis.
• - NSAIDs, analgesics, epidural steroid injections, and physical therapy are the mainstays of
nonsurgical management. - Surgical decompression iswarranted in patients with recurrent and
disabling pain that limits their daily activity.

Other
Option B:A fracture of the odontoid process
• Odontoid process fracture occurs when the fracture occurs through the odontoid process of the C2
vertebra.
Option C: A herniated L3–L4 disc causing unilateral compression of the L4 root

Page 23

1579
• Herniated lumbar disc displaces its nucleus pulposus or annulus fibrosus beyond the intervertebral
disc space. The radiological examination generally confirms the clinical diagnosis.
• The patient exhibits bilateral signs ruling out a unilateral compression.
Option D: Spinal cord compression at the level of L1 vertebra level
• Based on the clinical diagnosis of neurogenic claudication, the most likely feature that will be seen in
the MRI, in this case, is lumbar spinal stenosis, not the spinal cord compression at the L1
level. Clinically, there is no evidence of compression at the level of L1 vertebra.
• Spinal cord compression can occur anywhere from the cervical to the lumbar spine.
• Symptoms may include numbness, pain, and weakness.
• Depending on the cause of the compression, symptoms may develop suddenly or gradually, requiring
anything from supportive care to emergency surgery

Page 24

1580
CNS Tumours
1. A 4-year-old girl with a history of seizures comes to the emergency department because of
headaches and more frequently occurring seizures. Physical examination shows patches of
leathery-appearing and hypopigmented skin lesions scattered over her body. A subungual nodule is
found on her left thumb. CT scan of the brain shows subependymal nodules, cortical foci with loss of
the gray-white matter distinction, and a nodule partially obstructing the cerebral aqueduct. Which of the
following is the most likely cause of the patient’s symptoms?
(or)
What is the diagnosis in a school going girl with a history of seizures, headaches, worsening seizures,
examination findings of leathery and hypopigmented skin patches, and CT brain showing
subependymal nodules with obstruction of the cerebral aqueduct?
A. Craniopharyngioma
B. Ependymoma
C. Giant-cell astrocytoma
D. Medulloblastoma
----------------------------------------
2. What is the most likely diagnosis in a male presenting with loss of erections, low blood testosterone,
and high prolactin levels, with CT brain image provided below?
(or)
A 30-year-old male complains of loss of erection; he has low testosterone levels and high prolactin
levels in his blood. A brain CT scan was recommended, which is given below. What is the most likely
diagnosis?

A. Pituitary adenoma
B. Testicular failure
C. Craniopharyngioma
D. Cushing’s syndrome
----------------------------------------
3. A 2 year old child is presented to the hospital with symptoms of seizures and mental retardation with
a red port wine birthmark on his face. An MRI is done, which shows a leptomeningeal angioma. What
could be the possible diagnosis?
(or)

1581
What is the diagnosis in a child presenting with seizures, developmental delay, a port wine stain on the
face, and an MRI showing leptomeningeal angioma?
A. Craniopharyngioma
B. Glioblastoma
C. Neurofibromatosis 1
D. Encephalotrigeminal Syndrome
----------------------------------------
4. A 6-year-old boy has been complaining of headaches, ignoring objects on the sides for four months.
His grades at school are good.On examination, his visual acuity is diminished in both eyes. Visual
charting showed significant field defects. A CT scan of the head showed a suprasellar mass with
calcification. Which of the following is the most probable diagnosis?
(or)
What is the probable cause of a suprasellar mass with calcification in a boy presenting with headaches,
decreased visual acuity, and visual field defects ?
A. Astrocytoma
B. Craniopharyngioma
C. Pituitary adenoma
D. Meningioma
----------------------------------------
5. A 45-year-old patient complained of abnormal gait and difficulty walking in the dark. He was
diagnosed with a primary spinal tumor. What is the most common site of a primary spinal tumor?
(or)
What is the most common site of a primary spinal tumor?
A. Intramedullary; Extradural
B. Intramedullary
C. Intradural; Extramedullary
D. Intra-axial
----------------------------------------
6. Stereotactic radiosurgery is used in which of the following conditions?
A. Bell's palsy
B. Schizophrenia
C. Parkinson's disease
D. Metastatic brain tumours
----------------------------------------
7. Which of the following is true regarding brain metastases?
(or)
A 55-year-old patient presents with a headache and left-sided upper extremity weakness. He also
complains of difficulty in speech and loss of balance. The doctor advised for an MRI and he was

Page 2

1582
diagnosed with brain metastases. Which of the following is true about brain metastases?
A. Less than 55% of brain metastases are supratentorial
B. Lesions are located in white matter
C. Most common primary site is breast cancer
D. Stereotactic radiosurgery is recommended where surgery is not feasible
----------------------------------------
8. A 17-year-old man came to the emergency department because of a headache and diplopia for the
past one month. A lumbar puncture is performed, and cerebrospinal fluid analysis shows elevated
levels of human chorionic gonadotropin and plasma alpha fetoprotein . Which of the following is the
most likely diagnosis?
(or)
What is the diagnosis in an adolescent male presenting with diplopia for one week and elevated levels
of CSF human chorionic gonadotropin and plasma alpha fetoprotein ?
A. Ependymoma
B. Pineal germinoma
C. Pituitary adenoma
D. Meningioma
----------------------------------------
9. A 53-year-old woman comes to the neurosurgery clinic because of the discovery of a visual field
defect. The defect was identified during a routine follow-up. T1 contrast-enhanced MRI reveals a large
tumor along the base of the anterior cranial fossa. Which of the following is also likely to be present in
this patient?
(or)
Which of the following is likely to be present in a patient with a meningioma along the base of the
anterior cranial fossa?
A. Hemiparesis
B. Peripheral facial palsy
C. Diplopia and facial numbness
D. Loss of sense of smell
----------------------------------------
10. Which sinus is affected in Pott’s puffy tumor?
(or)
A patient presented with frontal scalp and periorbital swelling, fever, purulent rhinorrhoea, photophobia,
frontal sinus tenderness, and vomiting. On examination, signs of meningitis and encephalitis were
present. Which sinus among the following is affected in Pott’s puffy tumor?
A. Abscess of frontal sinus
B. Abscess of maxillary sinus
C. Abscess of the ethmoid sinus
D. Abscess of the sphenoid sinus

Page 3

1583
----------------------------------------
11. A 64-year-old man is suffering from a headache and left-sided upper extremity weakness. He also
complains of difficulty in speech. The doctor suggests an MRI brain which is given below. What does
this MRI exhibit?
(or)
What does the following MRI brain show?

A. The lesion is regular in shape


B. Well-demarcated from surrounding brain tissue
C. Shows a ring pattern of enhancement with intravenous contrast and has a non-enhancing necrotic
center
D. Shows an absence of surrounding white-matter edema
----------------------------------------
12. A 4-year-old boy is brought to the emergency department because of worsening headaches,
nausea, and vomiting for the past month. In the past three days, they have noticed he has been
"wobbly" when walking, losing his balance, and falling frequently. Examination shows a broad-based
gait and difficulty with heel-to-toe walking. Magnetic resonance imaging (MRI) reveals the following..
Which of the following is the most likely diagnosis?
(or)
What is the diagnosis in a toddler presenting with worsening headaches, nausea, vomiting for one
month, loss of balance for 2 weeks with the following MRI?

A. Medulloblastoma
B. Pilocytic astrocytoma

Page 4

1584
C. Brainstem glioma
D. Atypical teratoid
----------------------------------------
13. A 60-year-old otherwise healthy woman presents to her physician with a 3-week history of severe
headaches. A contrast CT scan reveals a small, circular, hypodense lesion with a ring-like contrast
enhancement. Which of the following is the most likely diagnosis?
(or)
An elderly female with history of severe headaches revealed a small, circular, hypodense lesion with a
ring-like contrast enhancement on contrast CT scan. Which of the following is the most likely
diagnosis?
A. High-grade astrocytoma
B. Metastatic lesion
C. Parenchymal Hemorrhage
D. Cerebral venous sinus thrombosis
----------------------------------------
14. An MRI done on a 28-year-old male suffering from neurofibromatosis type 2 showed an ice cream
cone appearance. On further evaluation, he revealed that it began with a ringing sensation in the ears,
which progressed to balance problems and hearing loss. Which of the following is the most likely
diagnosis?
(or)
The MRI scan of a male with NF-2 showing an ice cream cone appearance with ringing sensation in the
ears, balance problems, and hearing loss is likely to have which of the following?
A. Meningioma
B. Ependymoma
C. Schwannoma
D. Ganglioneuroma
----------------------------------------
15. A male patient of age 58 years presented to the OPD with the symptoms of changes in vision,
headaches, hearing loss, and memory loss. An MRI is done which shows the following image. What
could be the possible diagnosis?
(or)
What is the diagnosis in a male with complaints of changes in vision, headaches, hearing loss, memory
problems for 6 months, and an MRI brain showing the following image?

Page 5

1585
A. Germ cell tumour
B. Medulloblastoma
C. CNS lymphoma
D. Meningioma
----------------------------------------

Correct Answers
Question Correct Answer

Question 1 3
Question 2 1
Question 3 4
Question 4 2
Question 5 3
Question 6 4
Question 7 4
Question 8 2
Question 9 4
Question 10 1
Question 11 3
Question 12 1
Question 13 2
Question 14 3
Question 15 4

Solution for Question 1:


Option C: Giant-cell astrocytoma
• Tuberous sclerosis (TS) is an autosomal dominant disorder with genetic heterogeneity.

Page 6

1586
• The brain tumors that are most commonly found in tuberous sclerosis are cortical tubers,
subependymal nodules, or giant cell astrocytomas (SEGA).

Image shows cortical tubers

• Giant-cell astrocytomas are low-grade neoplasms that appear to develop from the hamartomatous
nodules in the brain.
• Ash-leaf spots, angiofibroma, and Shagreen patches are the most commonly seen dermatologic
features. Though these tumors are benign, they often interfere with function causing seizures and other
symptoms.
Option A: Craniopharyngioma
• Craniopharyngioma is a type of brain tumor derived from pituitary gland embryonic tissue in the
suprasellar region that occurs most commonly in children but also in adults, ruling out this option.

Page 7

1587
• Initial symptoms may be bitemporal inferior quadrantanopia leading to bitemporal hemianopsia, as the
tumor may compress the optic chiasm.
Option B: Ependymoma
• Ependymomas are tumours that usually arise in the ventricular system of the brain, including the
central canal of the spinal cord. In the first two decades of life, they typically occur near the fourth
ventricle, ruling out this option.
• Posterior fossa ependymomas manifest with hydrocephalus secondary to obstruction of the fourth
ventricle.
Option D: Medulloblastoma
• This malignant embryonal tumor occurs most commonly in children.
• Medulloblastomas are typically located in the midline of the cerebellum. This tumor is often
well-circumscribed and rapid growth may occlude the flow of CSF, leading to hydrocephalus. This
option is thus ruled out.

Solution for Question 2:


Correct Option A - Pituitary adenoma:
• Pituitary adenomas arise from the anterior pituitary gland. They may be microadenomas <10 mm in
size or macroadenomas >10 mm in size. They may be functional or non-functional.
• The most common endocrine syndromes produced by functional tumors include Cushing’s
disease(ACTH), Forbes Albright syndrome (prolactin) and acromegaly (growth hormone).
• Thirty percent of adenomas are prolactinomas; 20% are non-functioning, 15% secrete growth
hormone, and 10% secrete ACTH.
• Clinical features in the initial assessment include: History of galactorrhoea in females and impotence
in males, infertility in both (suggestive of prolactinoma) Cushingoid or Acromegalic features point to
growth hormone-secreting tumors.
• History of galactorrhoea in females and impotence in males, infertility in both (suggestive of
prolactinoma)
• Cushingoid or Acromegalic features point to growth hormone-secreting tumors.
• History of galactorrhoea in females and impotence in males, infertility in both (suggestive of
prolactinoma)
• Cushingoid or Acromegalic features point to growth hormone-secreting tumors.
Incorrect Options:
Option B - Testicular failure:
• When the testes cannot generate sperm or male hormones like testosterone, it is called testicular
failure.
• This would not show the lesion as shown in the question on CT brain, ruling out the option.
• Testicular failure is uncommon. Causes include: Certain medicines, including glucocorticoids,
ketoconazole, chemotherapy, and opioids Diseases that affect the testes, including hemochromatosis,
mumps, orchitis, testicular cancer, testicular torsion, and varicocele Injury or trauma to the testicles
Obesity

Page 8

1588
• Certain medicines, including glucocorticoids, ketoconazole, chemotherapy, and opioids
• Diseases that affect the testes, including hemochromatosis, mumps, orchitis, testicular cancer,
testicular torsion, and varicocele
• Injury or trauma to the testicles
• Obesity
• Symptoms depend on the age at which testicular failure develops, either before or after puberty.
• Symptoms may include: Decrease in height Enlarged breasts (gynecomastia)
• Decrease in height
• Enlarged breasts (gynecomastia)
• Treatment includes testosterone replacement therapy.
• Certain medicines, including glucocorticoids, ketoconazole, chemotherapy, and opioids
• Diseases that affect the testes, including hemochromatosis, mumps, orchitis, testicular cancer,
testicular torsion, and varicocele
• Injury or trauma to the testicles
• Obesity
• Decrease in height
• Enlarged breasts (gynecomastia)
Option C - Craniopharyngioma:
• Craniopharyngiomas are benign cystic tumors in the sellar region, most frequently in children. Around
50 years of age is when the second peak of incidence occurs.
• The majority of these tumors develop around the pituitary gland and do not metastasize.
• Compression of nearby structures, particularly the optic chiasm, causes visual symptoms.
Hydrocephalus or pituitary or hypothalamic dysfunction could manifest .
• CT shows calcification of the lesion.
• Surgery is the primary form of treatment.
• The hormonal symptoms mentioned in the patient are not suggestive of a craniopharyngioma.
Option D - Cushing’s syndrome:
• Syndrome associated with clinical features of high levels of cortisol in the body.
• Clinical features: Extreme tiredness Muscle sluggishness Moodiness, anxiety, and depression.
Emotional instability Cushing syndrome can be fatal if left untreated.
• Extreme tiredness
• Muscle sluggishness
• Moodiness, anxiety, and depression.
• Emotional instability
• Cushing syndrome can be fatal if left untreated.
• The clinical features mentioned in the patient are not suggestive of Cushing’s syndrome and he does
not seem to have any Cushingoid features.

Page 9

1589
• Extreme tiredness
• Muscle sluggishness
• Moodiness, anxiety, and depression.
• Emotional instability
• Cushing syndrome can be fatal if left untreated.

Solution for Question 3:


Option D: Encephalotrigeminal Syndrome
• Encephalotrigeminal Syndrome aka Sturge-Weber Syndrome
• Large unilateral cutaneous angioma (port–wine stain); a special form of nevus flammeus.
• These lesions are due to capillary malformations, which tend to grow during childhood, thicken the
skin surface and do not fade with time.
• They can be found where the trigeminal nerve's maxillary and ophthalmic branches meet.
• Clinical features: Ipsilateral venous angiomas in the cortical leptomeninges Mental retardation
Seizures Glaucoma Hemiplegia
• Ipsilateral venous angiomas in the cortical leptomeninges
• Mental retardation
• Seizures
• Glaucoma
• Hemiplegia
• CT Brain: tram track appearance A large facial telangiectasia in a child with mental deficiencies may
indicate the presence of additional vascular malformations.
• The above features present in the patient makes it the right diagnosis.
• Ipsilateral venous angiomas in the cortical leptomeninges
• Mental retardation
• Seizures
• Glaucoma
• Hemiplegia
Option A: Craniopharyngioma
• Craniopharyngiomas are benign cystic lesions in the sellar region, most frequently in children. The
second peak of incidence exists around 50 years of age.
• Most of these tumors develop around the pituitary gland and do not metastasize.
• Compression of nearby structures, particularly the optic chiasm, causes visual
symptoms. Hydrocephalus or pituitary or hypothalamic dysfunction could manifest.
• The absence of the features mentioned above rules out the diagnosis.
• The main treatment is surgery.

Page 10

1590
Option B: Glioblastoma
• The malignant glioblastoma can develop in the brain or spinal cord. The astrocytes that support nerve
cells are the source of glioblastoma.
• Glioblastoma can strike at any age but tends to strike older people more frequently. Seizures, nausea,
vomiting, and increasing headaches are all the clinical features
• The absence of the above features rules out the diagnosis.
• It is the most prevalent parenchymal brain tumor and almost always fatal, accounting for 16% of all
primary brain tumors.
Option C: Neurofibromatosis 1
• Neurofibromatosis type 1 is characterized by s nerve sheath tumors that grow along nerves in the
skin, brain, and other body parts.
• Autosomal dominant inheritance, NF 1 gene located on chromosome 17.
• People impacted by this ailment experience a wide range of indications and symptoms: Flat, light
brown macules on the skin (cafe-au-lait spots) Freckling in the armpits or groin area Tiny bumps on the
iris of the eyes (Lisch nodules) Soft, pea-sized bumps on or under the skin (neurofibromas)
• Flat, light brown macules on the skin (cafe-au-lait spots)
• Freckling in the armpits or groin area
• Tiny bumps on the iris of the eyes (Lisch nodules)
• Soft, pea-sized bumps on or under the skin (neurofibromas)
• The absence of the above features rules out the diagnosis.
• Flat, light brown macules on the skin (cafe-au-lait spots)
• Freckling in the armpits or groin area
• Tiny bumps on the iris of the eyes (Lisch nodules)
• Soft, pea-sized bumps on or under the skin (neurofibromas)

Solution for Question 4:


Option B: Craniopharyngioma
• The given scenario suggests the diagnosis of craniopharyngioma.
• CRANIOPHARYNGIOMA Derived from Rathke’s pouch They have a bimodal distribution occurring
predominantly in children and in adults between 55 and 65 years of age. Presents with symptoms of
increased intracranial pressure, including headache, vomiting, papilledema, and hydrocephalus. Visual
symptoms result from compression of adjacent structures, especially the optic chiasm. Compression of
the pituitary can lead to pituitary deficiencies. Calcification occurs in all pediatric and roughly half of
adult craniopharyngiomas. The cysts maybe multiloculated and contain liquid ranging from 'machinery
oil' to to shimmering cholesterol-laden fluid Treatment involves transcranial or transsphenoidal surgical
resection followed by postoperative radiation of the residual tumor. The most frequent postoperative
complications include diabetes insipidus and hypothalamic injury, with 5% to 10% mortality. Visual loss
and also cognitive impairment from basal frontal injury are common complications. Treatment requires
medical optimization before surgical resection due to adrenal cortical insufficiency; hydrocortisone
coverage should be given first perioperatively. Craniopharyngiomas tend to recur after resection.

Page 11

1591
• Derived from Rathke’s pouch
• They have a bimodal distribution occurring predominantly in children and in adults between 55 and 65
years of age.
• Presents with symptoms of increased intracranial pressure, including headache, vomiting,
papilledema, and hydrocephalus. Visual symptoms result from compression of adjacent structures,
especially the optic chiasm. Compression of the pituitary can lead to pituitary deficiencies.
• Calcification occurs in all pediatric and roughly half of adult craniopharyngiomas.
• The cysts maybe multiloculated and contain liquid ranging from 'machinery oil' to to shimmering
cholesterol-laden fluid
• Treatment involves transcranial or transsphenoidal surgical resection followed by postoperative
radiation of the residual tumor.
• The most frequent postoperative complications include diabetes insipidus and hypothalamic injury,
with 5% to 10% mortality. Visual loss and also cognitive impairment from basal frontal injury are
common complications.
• Treatment requires medical optimization before surgical resection due to adrenal cortical insufficiency;
hydrocortisone coverage should be given first perioperatively.
• Craniopharyngiomas tend to recur after resection.
• Derived from Rathke’s pouch
• They have a bimodal distribution occurring predominantly in children and in adults between 55 and 65
years of age.
• Presents with symptoms of increased intracranial pressure, including headache, vomiting,
papilledema, and hydrocephalus. Visual symptoms result from compression of adjacent structures,
especially the optic chiasm. Compression of the pituitary can lead to pituitary deficiencies.
• Calcification occurs in all pediatric and roughly half of adult craniopharyngiomas.
• The cysts maybe multiloculated and contain liquid ranging from 'machinery oil' to to shimmering
cholesterol-laden fluid
• Treatment involves transcranial or transsphenoidal surgical resection followed by postoperative
radiation of the residual tumor.
• The most frequent postoperative complications include diabetes insipidus and hypothalamic injury,
with 5% to 10% mortality. Visual loss and also cognitive impairment from basal frontal injury are
common complications.
• Treatment requires medical optimization before surgical resection due to adrenal cortical insufficiency;
hydrocortisone coverage should be given first perioperatively.
• Craniopharyngiomas tend to recur after resection.
Option A: Astrocytoma
• Astrocytoma is cancer that can develop in the spinal cord or brain.
• The astrocytes that sustain nerve cells are the sites of astrocytoma where it develops.
• Signs and symptoms of astrocytoma are dependent on the location of the tumour, which include
nausea, headaches, and seizures
• But, the most common cause of a suprasellar mass with calcification in a child is craniophyaringioma
Option C: Pituitary adenoma

Page 12

1592
• Pituitary adenomas are the tumors of the pituitary gland that are benign.
• In these circumstances, headache is often characterized by constant, unilateral (ipsilateral to the
tumor) or bifrontal aching. In some cases, the midface is the specific location of pain (either because of
involvement of the second division of the trigeminal nerve or secondary to sinusitis).
• A pituitary adenoma will occur in roughly 1 in 10 people during their lives.
• Some pituitary adenomas secrete an excessive amount of one or more hormones.
• The most frequent signs are double vision or vision loss, headaches, and hormonal dysfunction, in
which impotence, abnormal menstrual cycles, weight gain and fatigue are included.
• The patient has a suprasellar mass unlike in pituitary adenoma and the most common cause of a
suprasellar mass with calcification in a child is craniophyaringioma, ruling out this option.
Option D: Meningioma
• A meningioma is a tumor arising from the meninges
• Symptoms: Changes in the vision, such as blurriness or seeing double Headaches that worsen with
time Hearing loss or ringing in the ears Memory loss Loss of smell
• Changes in the vision, such as blurriness or seeing double
• Headaches that worsen with time
• Hearing loss or ringing in the ears
• Memory loss
• Loss of smell
• Changes in the vision, such as blurriness or seeing double
• Headaches that worsen with time
• Hearing loss or ringing in the ears
• Memory loss
• Loss of smell

Solution for Question 5:


Option C: Intradural; Extramedullary
• Most common site of primary spinal tumors – Intradural, Extra medullary
• Intradural extramedullary tumors constitute approximately 40% of spinal tumors and arise from the
meninges or nerve root elements.
• The most common intradural extramedullary tumors are typically benign (the commonest are
meningioma and neurofibromas), slow, growing and well-circumscribed.
• Most common spinal tumor - Metastasis
• The most common malignancies that metastasize to the spine are Breast > Lung > Prostate.
• Most common intramedullary tumor – Astrocytoma
• Primary bone tumors of the spine account for only 2% of all spinal tumors

Page 13

1593
• They may compress the spinal cord, causing myelopathy or the nerve roots, causing radiculopathy.
Option A: Intramedullary; Extradural
• Intramedullary tumors are growths that form in the spinal cord's supporting (glial) cells.
• A spinal tumor is a growth that develops within the spine or spinal canal bones.
• Intramedullary spinal tumors comprise 5-10% of all spinal tumors and are the most common spinal
tumors in children.
• Extradural tumors account for approximately 55% of spinal tumors.
Option B: Intramedullary
• Intramedullary tumors constitute approximately 5% of spinal tumors. They arise within the
parenchyma of the spinal cord.
• Ependymomas are the most common intramedullary tumors in adults.
• Astrocytomas are the most common intramedullary tumours in children, although they also occur in
adults.
• They most often occur in the cervical spinal cord.
• The tumor may interfere with the central canal of the spinal cord containing CSF, leading to a dilated
central canal, referred to as syringomyelia.
Option D: Intra-axial
• Brain tumors rooted in the brain parenchyma are referred to as intra-axial.
• If the origin of the tumor is outside of the brain (or it is due to metastasis), it is called extra-axial.
• Gliomas are intra-axial tumors.
• Gliomas are a group of tumors in the brain that start within the glial cells, which support the functions
of the primary brain cells called neurons. Gliomas can occur commonly in the brain's largest, outermost
regions but also in the brain stem.

Solution for Question 6:


Correct Option D - Metastatic brain tumors:
• The two most common devices used for conformal SRS for intracranial lesions are the LINAC (linear
accelerator) and the gamma knife.
• Stereotactic radiosurgery (SRS) is used in:
• Arteriovenous malformations - SRS is an effective stand-alone therapy for AVMs up to 3 cm in
diameter. It is best for lesions that are difficult to access surgically due to the high likelihood of
postoperative neurologic deficit.
• Effective obliteration and elimination of the risk of hemorrhage takes 2 to 3 years.
• Vestibular schwannomas - SRS has been introduced as a therapeutic alternative to microsurgical
resection for vestibular schwannomas up to 2.5 cm in maximum diameter.
• Intracranial metastasis - Patients with solitary or multiple intracranial metastases may be treated
primarily with SRS.

Page 14

1594
• Trigeminal neuralgia - It is also known as tic douloureux and is characterized by repetitive, unilateral,
sharp, and lancinating pains in the distribution of typically the second, but sometimes the third branch of
the trigeminal nerve. The patient may describe a trigger point, an area on the face that elicits pain when
touched.
• Side effects include difficulty swallowing and hair loss in the treatment area.
Incorrect Options:
Option A - Bell's palsy:
• Bell's palsy is a disorder that causes the muscles on one side of the face to weaken suddenly.
• The weakness often subsides over a few weeks and is only temporary.
• The weakening makes the lower portion of the face look sagging to one side. The affected eye also
resists closing.
• Corticosteroids, such as prednisone, are frequently prescribed to treat Bell's palsy.
• These anti-inflammatory medications work effectively, SRS has no role.
Option B: Schizophrenia
• Hallucinations, delusions and severely aberrant thinking and behavior may all be symptoms of
schizophrenia and can make it difficult to go about daily activities.
• People with schizophrenia require lifelong treatment.
• Schizophrenia is usually treated with a tailored combination of psychotherapy and drugs. SRS has no
role.
Option C - Parkinson's disease:
• Parkinson's disease is a brain condition that results in unintentional or uncontrollable movements like
trembling, stiffness, and issues with balance and coordination.
• Typically, symptoms start mildly and get worse over time. People could experience difficulties
speaking and walking as the illness worsens.
• First-line treatment for Parkinson's disease (PD) is typically either L-dopa or a non-ergot dopamine
agonist (DA); SRS has no role.

Solution for Question 7:


Correct Option D - Stereotactic radiosurgery is recommended where surgery is not feasible:
• Cerebral metastases is the most common type of intracranial tumor.
• The sources in most cerebral metastases are (in decreasing frequency): lung, breast, kidney, GI tract
and melanoma.
• MRI pre- and post-contrast administration is the study of choice for evaluation.
• Total surgical excision should be attempted when a solitary metastasis is encountered, followed by
whole-brain radiotherapy.
• SRS is typically advised if surgery is not an option.
• When multiple metastases are encountered, consideration should be given to the excision of the
symptomatic lesion or multiple lesions followed by brain therapy or radiotherapy alone. SRS will usually

Page 15

1595
be considered if surgery is not feasible.
Incorrect Option:
Option A - Less than 55% of brain metastases are supratentorial:
• The cerebrum, lateral ventricle, third ventricle , choroid plexus, pineal gland, hypothalamus, pituitary
gland, and optic nerve are all located in the supratentorial area (the upper part of the brain).
• Supratentorial tumors generally present with localizing neurological symptoms which may develop
over extended intervals.
• Seizures are the most common symptom in hemispheric cerebral lesions, especially with tumors
arising in the temporal lobe.
• Brain tumors in the lobes are called supratentorial, and tumors in the cerebellum or brainstem are
called infratentorial.
• Most brain metastases (85%) are supratentorial, ruling out the option
Option B - Lesions are located in white matter:
• Metastatic cells usually travel to the brain hematogenously and frequently seed the gray-white
junction due to the characteristic change in blood vessel caliber.
• Other common locations are the cerebellum and meninges.
• White matter lesions (WMLs) are areas of abnormal myelination in the brain, ruling out the option.
• Magnetic resonance imaging best visualizes these lesions as hyperintensities on T2 weighted and
FLAIR (Fluid-attenuated inversion recovery) sequences. They are considered a marker of small vessel
disease.
Option C - Most common primary site is breast cancer:
• The distribution of metastases in the brain approximates the proportion of blood flow such that 85% of
all metastases are supratentorial, and 15% occur in the posterior fossa.
• Melanoma has the greatest propensity to metastasise to the brain, found in 80% of patients at
autopsy.
• Meninges involvement may result in carcinomatous meningitis, also known as leptomeningeal
carcinomatosis. Most common primary for leptomeningeal metastases is breast cancer.
• Lung cancer > breast cancer is the most common primary site for metastatic brain tumors, ruling out
the option.

Solution for Question 8:


Option B: Pineal germinoma
• Germinomas are seen mainly in pediatric populations, the presentations of which depend on the
location of the tumor.
• Alpha-fetoprotein and beta-human chorionic gonadotropin levels are typically elevated.
• Elevations in these tumor markers, and neuroimaging showing a mass in the pineal region, are often
sufficient to make the diagnosis.
Pineal germinoma

Page 16

1596
• Intracranial germinomas are germ-cell tumors mainly seen in the pediatric age group. They tend to
occur in the midline (common sites: at the pineal region or along the floor of the third ventricle or
suprasellar region).
• They are the most common pinealomas.
• Presentation depends on the site, and pineal tumors typically cause compression of the tectal plate
causing obstructive hydrocephalus and neuro-ophthalmologic manifestations affecting vertical gaze
and convergence (Parinaud syndrome ). Diabetes insipidus, hypopituitarism, optic chiasm
compression, or signs of intracranial hypertension are caused by involvement of the pituitary
infundibulum. When the thalamus and basal ganglia are involved, the presentation is often late with a
large tumor.
• Alpha-fetoprotein (AFP) and beta-human chorionic gonadotrophin (β-hCG) levels are raised in the
serum and cerebrospinal fluid (CSF ). CSF tumor marker levels are usually higher than serum levels.
Detection of elevated tumor markers may be sufficient for diagnosis in patients who cannot undergo
endoscopic biopsy.
Other options
Option A: Ependymoma
• They constitute 5% of all intracranial gliomas. In the pediatric population, they constitute up to 70% of
all intracranial gliomas, and the peak age at presentation is between 10 and 15 years.
• In children, ependymoma will typically be present on the floor of the fourth ventricle. They are
slow-growing posterior fossa masses that may cause obstruction of CSF flow leading to hydrocephalus.
Treatment consists of the maximum possible resection followed by fractionated radiation.
• hCG and AFP levels are not raised.
Option C: Pituitary adenoma
• Pituitary adenomas are benign tumors of the anterior pituitary gland.
• Most patients with these adenomas present with symptoms of hormone hypersecretion, such as
hyperprolactinemia, growth hormone excess, hypercortisolism, or hyperthyroidism. Although some
"silent" tumors may not produce hormones and may present with visual symptoms and headaches.
• hCG and AFP levels are not raised.
Option D: Meningioma
• Meningiomas are derived from arachnoid cap cells of the arachnoid mater. They appear to arise from
the dura mater are referred to as dural-based tumors.
• Most common intracranial and most common primary brain tumors.
• They are more common in females (2:1)
• The most common intracranial locations are along the falx, the convexities(i.e., over the cerebral
hemispheres), and specifically adjacent to the sagittal sinus and the sphenoid wing. Less common
locations include the foramen magnum, olfactory groove, and inside the lateral ventricle.
• They are slow growing encapsulated benign tumors. Prior cranial irradiation increases the risk for
meningiomas. Approximately 10% of patients have multiple meningiomas.
• Meningiomas are classified by the WHO into three histologic grades of increasing aggressiveness:
grade I (benign), grade II (atypical), and grade III(malignant)
• hCG and AFP levels are not raised ruling out this diagnosis.

Page 17

1597
Solution for Question 9:
Option D: Loss of sense of smell
• Tumors at the base of the anterior cranial fossa may compress the olfactory bulbs and nerves, leading
to the symptom of anosmia. Due to the slow progression of a tumor, this may not be the presenting
complaint.
• Foster-Kennedy syndrome describes the symptoms of tumors on the inferior surface of the frontal
lobe, such as an olfactory groove meningioma. Symptoms consist of:
1.Anosmia
2.Ipsilateral optic atrophy
3.Ipsilateral scotoma
4.Contralateral papilledema
• Visual field deficits may present as bitemporal hemianopia that begins inferiorly if the optic chiasm is
compressed downward by the meningioma.
• Other symptoms include headache, mental disturbances, and, less commonly, seizure.
Option A: Hemiparesis
• Parasagittal meningiomas may result in monoparesis of the contralateral leg, and meningiomas
around the foramen magnum may result in paraparesis.
Option B: Peripheral facial palsy
• Meningiomas in the cerebellopontine angle may cause hearing loss and facial nerve deficits. Facial
symptoms may present as weakness or numbness.
Option C: Diplopia and facial numbness
• Meningiomas in the cavernous sinus can cause multiple cranial nerve deficits and lead to visual loss,
diplopia, and facial numbness.

Solution for Question 10:


Option A: Abscess of frontal sinus
• Subdural empyema associated with osteomyelitis of the frontal bone and associated scalp swelling,
which pits on pressure, is Pott's puffy tumor. Since Pott's puffy tumor involves osteomyelitis of the
frontal bone, an underlying abscess of the frontal sinus may be associated.
Pott’s puffy tumor:
• Pott’s puffy tumor is one of the most dangerous complications of frontal sinusitis and is characterized
by osteomyelitis of the frontal bone with an associated subperiosteal abscess.
• Symptoms include frontal scalp and periorbital swelling, fever, purulent rhinorrhoea, photophobia,
frontal sinus tenderness, vomiting, and signs of meningitis or encephalitis.
• Pott’s Puffy Tumor can lead to life-threatening complications.

Page 18

1598
• In 85% of patients, Pott’s puffy tumor is a predictor of intracranial complications such as meningitis,
cavernous sinus and dural venous sinus thrombosis, orbital cellulitis with or without intraorbital abscess
(if the inferior wall of the frontal sinus is involved) and epidural, subdural or intraparenchymal
abscesses.
• The spread of the infection can occur either due to direct extension by erosion of frontal sinus walls or
by migration of septic thrombi through the diploic veins.
Option B: Abscess of maxillary sinus
• Pott's puffy tumor involves osteomyelitis of the frontal bone. An underlying abscess of the frontal sinus
may be associated.
• There are two large maxillary sinuses, one in each of the maxillary bones, in the cheek area next to
the nose. The maxillary sinuses are lined with cells that make mucus to keep the nose from drying out.
• The clinical features of the question are not suggestive of maxillary sinus abscess.
Option C: Abscess of the ethmoid sinus
• Pott’s puffy tumor involves osteomyelitis of the frontal bone, and an underlying abscess of the frontal
sinus may be associated.
• The ethmoid sinuses are hollow spaces in the bones around the nose. They have a lining of mucus to
help prevent the nose from drying out. Inflammation of the ethmoid sinuses can lead to pressure and
pain around the nose and between the eyes.
• The clinical features of the question are not suggestive of ethmoid sinus abscess.
Option D: Abscess of the sphenoid sinus
• Pott’s puffy tumor involves osteomyelitis of the frontal bone, and an underlying abscess of the frontal
sinus may be associated.
• Sphenoid sinus: Paranasal sinus (a hollow space in the bones around the nose). Two large sphenoid
sinuses are in the sphenoid bone, which is behind the nose between the eyes. The sphenoid sinuses
are lined with cells that make mucus to keep the nose from drying out.
• The clinical features of the question are not suggestive of sphenoid sinus abscess.

Solution for Question 11:


Option C: Shows a ring pattern of enhancement with intravenous contrast and has a
non-enhancing necrotic center
• MRI shown above shows Glioblastoma multiforme, which crosses the midline, and the image also
shows a ring-enhancing lesion with central necrosis.
• Glioblastoma multiforme Most common primary malignant brain tumor in adults. It is a WHO grade IV
astrocytoma that accounts for most high-grade astrocytomas with poor prognosis and survival rate.
Known as a butterfly tumor as it crosses the midline of the brain. They usually present in the 6th and
7th decade of life with headaches, seizures, focal deficits, and personality changes.
• Most common primary malignant brain tumor in adults.
• It is a WHO grade IV astrocytoma that accounts for most high-grade astrocytomas with poor
prognosis and survival rate.
• Known as a butterfly tumor as it crosses the midline of the brain.

Page 19

1599
• They usually present in the 6th and 7th decade of life with headaches, seizures, focal deficits, and
personality changes.
• Treatment involves gross total resection with postoperative chemotherapy and radiation therapy.
Temozolomide is a new drug used in newly diagnosed GBM and recurrent GBM.
• Most common primary malignant brain tumor in adults.
• It is a WHO grade IV astrocytoma that accounts for most high-grade astrocytomas with poor
prognosis and survival rate.
• Known as a butterfly tumor as it crosses the midline of the brain.
• They usually present in the 6th and 7th decade of life with headaches, seizures, focal deficits, and
personality changes.
Option A: Lesion is regular in shape
• Imaging demonstrates a ring-enhancing lesion with central necrosis, surrounding edema, and mass
effect.
• These are irregular lesions in shape and highly infiltrative tumors.
• The areas of increased T2/FLAIR signal surrounding the main tumor mass contain invading tumor
cells
Option B: Well-demarcated from surrounding brain tissue
• The lesion is irregular in shape.
• It is highly infiltrative and hence cannot be well demarcated from surrounding tissue.
• Lesions in the brain include abscess, sclerosis, and cerebral infarction.
Option D: Shows an absence of surrounding white-matter edema
• Surrounding edema is present.
• In cerebral edema, the additional fluid is accumulated within the brain's white matter.
• It is usually the response of the brain to an insult. It is of two major types: cytotoxic (gray and white
matter) and vasogenic (white matter) edema.
• Treatment includes hyperventilation, osmotherapy, diuretics, corticosteroids, and surgical
decompression.

Solution for Question 12:


Option A: Medulloblastoma
• This patient has a medulloblastoma. This is the most common malignant brain tumor of childhood and
is exclusively located within the cerebellum. It accounts for 20% of primary tumors of the central
nervous system among children under 19 years of age, and incidence peaks between the ages of 5
and 9 years.
• Patients with medulloblastoma present with signs and symptoms of increased intracranial pressure
(ICP ), such as morning headaches, nausea, vomiting, altered mental statuses, and cerebellar
dysfunction, such as gait ataxia, truncal instability, and limb incoordination, depending on the tumor
location.

Page 20

1600
• Symptoms typically develop over weeks to months and begin with those associated with raised ICP
before those due to growth and invasion of the tumor within the cerebellum. Diplopia and dizziness may
also result from the involvement of the cerebellum, brainstem, or cranial nerves.
• Histology shows small round undifferentiated round cells with Homer Wright rosettes.
Option B: Pilocytic astrocytoma
• Pilocytic astrocytomas are the most common primary brain tumor in childhood. They are
well-circumscribed cystic tumors, often arising within the cerebellar hemispheres. Due to their
slow-growing nature, symptoms usually develop over the years. When cysts are present in
medulloblastomas, they are typically small and multiple rather than large solitary cysts.
Option C: Brainstem glioma
• Brainstem gliomas arise in the midbrain, pons, or medulla and account for 10-20% of central nervous
system tumors in children. Most patients have a relatively long history of minor symptoms, which may
have been present for over a decade.
Option D: Atypical teratoid
• Atypical teratoid, also known as rhabdoid tumors, are rare, highly malignant tumors primarily occurring
in children less than three years of age. Two-thirds occur within the cerebellum, symptoms are
location-dependent, and the prognosis is poor.

Solution for Question 13:


Option B: Metastatic lesion
• The most likely cause is a metastatic lesion.
• Brain metastases occur when cancer cells spread from their original site to the brain. Most common
cancers causing brain metastases are of the lung, breast, colon, kidney, and melanoma.
• Brain metastases may be single or multiple. As the metastatic brain tumors grow, they compress
surrounding brain tissue. This causes signs and symptoms such as headaches, personality changes,
memory loss, and seizures.
• Treatment for people whose cancer has spread to the brain may include surgery, radiation therapy,
chemotherapy, immunotherapy, or a combination of treatments. Other treatments might be
recommended in certain situations. Treatment is often focused on reducing pain and symptoms.
Option A: High-grade astrocytoma
• Astrocytoma is one of the most common types of brain tumor that can develop in children. It arises
from a specific type of cell in the brain called an astrocyte. Astrocytomas are divided into subgroups
based on their grade and location in the brain.
• Brain metastases are more common than primary tumors of the brain.
• Astrocytoma is the most common primary CNS neoplasm. Glioma is often used to refer to
astrocytomas, specifically excluding other glial tumors.
• Astrocytomas are graded I to IV. Grade I to II are referred to as low-grade astrocytoma or low-grade
glioma, grade III as anaplastic astrocytoma and grade IV as glioblastoma multiforme (GBM)
• High-grade astrocytomas are either grade 3 or grade 4 brain tumors. Both are malignant, but grade 4
tumors are more aggressive than grade 3 tumors. These tumors most often occur in the cerebral
hemispheres but can also occur in the thalamus or, less commonly, in the posterior fossa.

Page 21

1601
• Treatment involves gross total resection with postoperative chemotherapy and radiation therapy.
Option C: Parenchymal Hemorrhage
• A parenchymal hemorrhage, or an intraparenchymal hemorrhage (IPH), is a bleed within the brain
parenchyma, the functional tissue in the brain consisting of neurons and glial cells. A hemorrhage, or
the profuse release of blood from a ruptured blood vessel in the brain, disrupts normal blood flow and
deprives the brain of oxygen. It does not present as a ring enhancing lesion.
• As a result, the brain cells can die, which damages the affected nerves and the related functions they
control as well. Spontaneous IPH accounts for less than 20% of stroke cases, but it is associated with
the highest mortality rates of all forms of stroke. It can be classified as primary or secondary, depending
on the cause.
• Primary IPH accounts for about 80% of cases, occurring when a spontaneous rupture of damaged
arteries occurs. Meanwhile, secondary IPH is due to an underlying condition that increases the
likelihood of hemorrhage.
Option D: Cerebral venous sinus thrombosis
• Cerebral venous sinus thrombosis can cause headaches and vomiting but usually presents with
different imaging findings, such as filling defects or lack of flow in the cerebral venous sinuses on
imaging modalities like MRI or CT venography.
• The multiple small, circular, hypodense lesions with ring-like contrast enhancement described in the
scenario are not characteristic of venous sinus thrombosis.

Solution for Question 14:


Option C: Schwannoma
• Schwannoma is a rare type of tumor that grows from Schwann cells. Schwann cells protect and
support the nerve cells of the nervous system.
• NF2 is associated with bilateral vestibular schwannomas and hearing loss
• Benign tumors usually originating from perineural fibroblasts.
• Intracranial schwannomas originate from the vestibular branch of the eighth cranial nerve.
• Approximately 90% of cases occur sporadically; genetic disorders such- as Carney complex and
neurofibromatosis 2 (NF2) can cause schwannomas.
• Clinical features: SNHL (M/C presentation) Tinnitus Vertigo/Dizziness
• SNHL (M/C presentation)
• Tinnitus
• Vertigo/Dizziness
• MRI shows ice cream cone appearance – depicting bilateral vestibular schwannomas
• Histology: spindle-shaped cells with Antoni A and B type pattern)
• Malignant schwannomas are known as neurofibrosarcomas or malignant peripheral nerve sheath
tumors (MPNSTs). They could occur anywhere but sciatic nerve is the most common site. They are
treated with radiation therapy if curative resection is not possible.
• SNHL (M/C presentation)

Page 22

1602
• Tinnitus
• Vertigo/Dizziness
Option A: Meningioma
• Gradual progression of motor weakness in the setting of a known intracranial mass is consistent with
a slow-growing tumor like a meningioma. Meningiomas can occur in the skull or the spinal canal.
Presenting symptoms may include focal neurological deficits, symptoms of raised intracranial pressure,
and seizures. Many meningiomas are asymptomatic. MRI is the best form of imaging for further
assessment.
• They do not show the ice cream cone appearance on MRI as mentioned in this patient.
Option B: Ependymoma
• Ependymomas are tumors that usually arise in the ventricular system of the brain, including the
central canal of the spinal cord. In the first two decades of life, they typically occur near the fourth
ventricle.
• Posterior fossa ependymomas manifest with hydrocephalus secondary to obstruction of the fourth
ventricle.
• It is not specifically associated with NF2 and does not show the ice cream cone appearance on MRI
scan.
Option D: Ganglioneuroma
• Ganglioneuroma is a tumor of the autonomic nervous system.
• Autonomic nerves manage body functions such as blood pressure, heart rate, sweating, bowel and
bladder emptying, and digestion. These tumors are usually non cancerous.
• Ganglioneuromas usually occur in people over 10 years of age. They grow slowly and may release
certain chemicals or hormones.
• There are no known risk factors. However, the tumors may be associated with some genetic
problems, such as neurofibromatosis type I.
• It does not show ice cream cone appearance on MRI.

Solution for Question 15:


Option D: Meningioma
• The given clinical scenario and MRI are suggestive of Meningioma.
• Meningioma: A primary central nervous system (CNS) tumor Meningiomas are the most common
primary brain tumor overall. Higher-grade meningiomas are quite uncommon. Meningiomas, which are
extra-axial tumours, are the most frequent meningeal tumours. They are non-glial neoplasms which
develop from meningo-mesothelial cells of arachnoid.
• A primary central nervous system (CNS) tumor
• Meningiomas are the most common primary brain tumor overall. Higher-grade meningiomas are quite
uncommon.
• Meningiomas, which are extra-axial tumours, are the most frequent meningeal tumours. They are
non-glial neoplasms which develop from meningo-mesothelial cells of arachnoid.

Page 23

1603
• Symptoms: Changes in vision, such as blurriness or seeing double images Headaches are
associated with vomiting, worse in the morning, and seizures Hearing loss or ringing in the ears
Memory loss Loss of smell Parasagittal meningiomas (which are common) exhibhit the same features
as an ACA stroke - urinary incontinence and lower limb UMN
• Changes in vision, such as blurriness or seeing double images
• Headaches are associated with vomiting, worse in the morning, and seizures
• Hearing loss or ringing in the ears
• Memory loss
• Loss of smell
• Parasagittal meningiomas (which are common) exhibhit the same features as an ACA stroke - urinary
incontinence and lower limb UMN
• Histopathology exhibits Psammoma bodies due to dystrophic calcification.
• CT Brain: Dural tail sign and calcifications are very common
• Treatment for meningioma is surgery. When the neurosurgeon is unable to remove the meningioma
completely, radiation therapy can be used after surgery.
• A primary central nervous system (CNS) tumor
• Meningiomas are the most common primary brain tumor overall. Higher-grade meningiomas are quite
uncommon.
• Meningiomas, which are extra-axial tumours, are the most frequent meningeal tumours. They are
non-glial neoplasms which develop from meningo-mesothelial cells of arachnoid.
• Changes in vision, such as blurriness or seeing double images
• Headaches are associated with vomiting, worse in the morning, and seizures
• Hearing loss or ringing in the ears
• Memory loss
• Loss of smell
• Parasagittal meningiomas (which are common) exhibhit the same features as an ACA stroke - urinary
incontinence and lower limb UMN
Option A: Germ cell tumor
• Germ cell tumors are growths of cells that arise from reproductive cells. The tumors may be
cancerous or non cancerous. Most germ cell tumors occur in the testes or the ovaries.
• Some germ cell tumors occur in other areas of the body, such as the abdomen, brain and chest,
though it is unclear why.
• Several inherited defects have also been associated with an increased risk of developing germ cell
tumors, including the central nervous system, genitourinary tract malformations, and major
malformations of the lower spine.
• The most common type of germ cell tumor is a teratoma. These tumors are benign but can become
malignant. Teratomas are the most common germ cell tumor to develop in extragonadal areas.
• The symptoms given in the patient do not align with the above mentioned features ruling out this
option.
Option B: Medulloblastoma

Page 24

1604
• A primary central nervous system (CNS) tumour
• Many of the symptoms are general and non-specific. Some are similar to less serious childhood
illnesses.
• Symptoms include: Headaches in the morning Double vision Finding it hard to sit or stand
unsupported
• Headaches in the morning
• Double vision
• Finding it hard to sit or stand unsupported
• The tumor is distinctive on T1- and T2-weighted MRI with heterogeneous enhancement and a specific
location adjacent to and extending into the fourth ventricle, which is different from the findings in the
question, ruling out the option.
• Histologically, the tumor is solid, pink-gray in color, and well-circumscribed. The tumor is very cellular,
with high mitotic activity, little cytoplasm, and a tendency to form clusters and Homer Wright rosettes.
• Headaches in the morning
• Double vision
• Finding it hard to sit or stand unsupported
Option C: CNS lymphoma
• Primary diffuse large B-cell lymphoma of the central nervous system (DLBCL-CNS), commonly known
as primary central nervous system lymphoma (PCNSL), is a primary intracranial tumors appearing
mostly in patients with severe immunodeficiency (typical patients with AIDS) unlike this patient.
• A primary CNS lymphoma typically exhibits specific neurological impairments indicative of a mass
effect, such as seizures, headaches, cranial nerve abnormalities, altered mental status, and others.
Fever, night sweats, or weight loss are systemic symptoms. Other symptoms include: Dysphagia
Diplopia Monocular vision loss Vertigo
• Dysphagia
• Diplopia
• Monocular vision loss
• Vertigo
• The current diagnostic standard typically includes positive CSF cytology, vitreous biopsy, or
brain/leptomeningeal biopsy. Histopathological confirmation is essential for a definitive diagnosis.
• Dysphagia
• Diplopia
• Monocular vision loss
• Vertigo

Page 25

1605
Previous Year Questions
1. What should be the subsequent course of action for an elderly patient who has undergone full
removal of glioblastoma?
A. Chemotherapy with Bevacizumab
B. No further management
C. Chemoradiation
D. Radiotherapy
----------------------------------------
2. What is the diagnosis of a patient, aged 50, who had a trauma one week ago and now presents with
confusion and ataxia? The patient has a history of alcohol dependence and has not consumed alcohol
for the past two weeks. The CT scan shows a specific finding.

A. Extradural hemorrhage
B. Subdural hemorrhage
C. Intra-parenchymal hemorrhage
D. Subarachnoid hemorrhage
----------------------------------------
3. What is the most appropriate course of action for the management of a patient who presents to the
emergency department with a severe headache, and has a blood pressure reading of 160/100 mmHg,
along with a CT scan showing evidence of a subarachnoid hemorrhage?
A. Nimodipine
B. Angiography
C. Surgery
D. Fibrinolytic therapy
----------------------------------------
4. A 34-year-old woman was brought to the AIIMS emergency following a road traffic accident. She was
hemodynamically stable and her GCS score was 3. She was intubated and sent for non-contrast CT
which was found to be normal. What is the most likely diagnosis?
A. Subdural hemorrhage
B. Extradural hemorrhage

1606
C. Diffuse axonal injury
D. Cerebral contusion
----------------------------------------
5. What is your diagnosis for an elderly woman who experienced a fall and subsequently developed a
headache along with neurological symptoms after a few days?
A. Multi-infarct dementia
B. Frontal lobe meningioma
C. Korsakoff disease
D. Subdural hematoma
----------------------------------------
6. What is the probable diagnosis for a 15-year-old boy who has a painless, gradually enlarging, soft,
and elastic swelling on the lateral aspect of his neck, as depicted in the provided image? Additionally,
the examination reveals the presence of freckles and skin patches on his face and trunk.

A. Cystic hygroma
B. Chemodectoma
C. Neural tumor
D. Lipoma
----------------------------------------

Correct Answers
Question Correct Answer

Question 1 4
Question 2 2
Question 3 2
Question 4 3
Question 5 4
Question 6 3

Page 2

1607
Solution for Question 1:
• Glioblastoma is a type of brain tumor that arises from astrocytes, the supportive cells in the brain. The
treatment of glioblastoma usually involves surgery, radiation therapy, and chemotherapy. After surgical
excision of the glioblastoma, the next step in management depends on several factors such as the
extent of the surgery, the age and overall health of the patient, and the tumor's aggressiveness.
• Radiotherapy is the next step in management in this case after complete excision of glioblastoma. It
involves the use of high-energy X-rays or other types of radiation to kill the remaining cancer cells and
prevent the tumor from recurring. Radiotherapy is usually given over several weeks and may be
accompanied by chemotherapy.
Incorrect Choices:
• Option a. Chemotherapy with Bevacizumab is an option for recurrent glioblastoma or for patients who
are not able to undergo radiation therapy due to various reasons. It may also be used as an adjuvant
therapy in combination with radiation therapy and temozolomide. However, it is not the next step in the
management of an old patient who has undergone complete excision of glioblastoma.
• Option b. No further management is not an appropriate option for glioblastoma patients who have
undergone complete excision of the tumor. These patients require additional treatment to prevent the
tumor from recurring.
• Option c. Chemoradiation is usually the next step in management after surgery for patients with
glioblastoma. It involves the use of chemotherapy and radiation therapy in combination to treat the
remaining cancer cells and prevent recurrence. However, this option is usually reserved for patients
with residual tumor after surgery.

Solution for Question 2:


• The patient's presentation of confusion and ataxia, coupled with a history of trauma and the given CT
scan findings, points towards a subdural hemorrhage, making option b the correct answer. Subdural
hematomas are usually due to a tear in the bridging veins that run between the dura mater and the
brain surface.
Incorrect Choices:
• Option a. Extradural hemorrhage: This would typically present acutely after a head injury and show a
lens-shaped (or biconvex) hematoma on a CT scan. It is due to a rupture of the middle meningeal
artery usually.
• Option c. Intra-parenchymal hemorrhage: This would result in bleeding within the brain tissue itself,
often associated with hypertension, cerebral amyloid angiopathy, or a traumatic brain injury.
• Option d. Subarachnoid hemorrhage: This would typically present with a "thunderclap" headache and
is due to bleeding into the subarachnoid space, often from a ruptured aneurysm.

Solution for Question 3:


Option B
• The correct answer is Angiography.

Page 3

1608
• Following a confirmed diagnosis of subarachnoid hemorrhage (SAH) via CT scan, the most
appropriate next step is to perform a cerebral angiography.
• This diagnostic procedure provides a detailed visualization of the blood vessels in the brain and helps
to determine the cause of the SAH, which is often a ruptured aneurysm.
Incorrect Choices:
• Option A. Nimodipine is a calcium channel blocker often used in SAH management to prevent
vasospasm, a common complication of SAH. However, it is not typically the immediate next step
following diagnosis.
• Option C. Surgery may be required to treat the cause of the SAH, such as a ruptured aneurysm.
However, angiography is typically performed first to confirm the location and extent of the aneurysm.
• Option D. Fibrinolytic therapy is not typically used in the acute management of SAH. It dissolves
dangerous intravascular clots to prevent ischemic damage by improving blood flow.

Solution for Question 4:


Correct Option C - Diffuse axonal injury:
• In a patient who presents following a road traffic accident with a low GCS score, the most likely
diagnosis is diffuse axonal injury (DAI).
• Diffuse axonal injury (DAI) is one of the most common and important pathologic features of traumatic
brain injury (TBI) which occurs due to widespread shearing and stretching of axonal fibers in the brain,
resulting from rapid acceleration and deceleration forces during the accident.
• It often occurs without any visible abnormalities on imaging studies such as CT scans.
Incorrect Options:
Option A - Subdural hemorrhage: Subdural hemorrhage is characterized by bleeding between the dura
mater and the arachnoid mater. It is typically associated with head trauma and can cause neurological
deficits. However, in this case, since the CT scan is normal, subdural hemorrhage is less likely.
Option B - Extradural hemorrhage: Extradural hemorrhage, also known as epidural hematoma, refers t
o bleeding between the skull and the dura mater. It is often caused by trauma and can result in rapid d
eterioration of consciousness. However, since the CT scan is normal in this case, extradural hemorrha
ge is less likely.
Option D - Cerebral contusion: Cerebral contusion refers to bruising of the brain tissue caused by direc
t trauma to the head. It can result in focal neurological deficits and may be visible on imaging studies a
s areas of hemorrhage or edema. Since the CT scan is reported as normal in this case, cerebral contu
sion is less likely.

Solution for Question 5:


Correct Option D - Subdural hematoma:
• In this scenario, the history of a fall followed by the development of a headache and neurological
symptoms suggests the possibility of a subdural hematoma.

Page 4

1609
• A subdural hematoma can occur due to the tearing of bridging veins between the brain and the dura
mater following trauma.
• The symptoms may include headache, altered mental status, focal neurological deficits, or seizures,
depending on the size and location of the hematoma.
Incorrect Options:
Option A - Multi-infarct dementia: This option is unlikely in this case because multi-infarct dementia is a
form of cognitive decline that occurs as a result of multiple small strokes (infarcts) in the brain. The sym
ptoms usually develop gradually over time rather than suddenly following a fall.
Option B - Frontal lobe meningioma: This option is also unlikely because a meningioma is a slow-growi
ng benign tumor that arises from the meninges. It typically presents with symptoms that are specific to t
he location of the tumor, such as seizures, focal neurological deficits, or behavioral changes. Headach
e and neurological symptoms that develop shortly after a fall are not typical of a meningioma.
Option C - Korsakoff disease: This option is unlikely because Korsakoff disease is a chronic neurologic
al condition caused by thiamine (vitamin B1) deficiency, often associated with chronic alcoholism. It pri
marily affects memory and is characterized by severe memory impairment, confabulation (filling in me
mory gaps with false information), and other cognitive deficits. Headache and acute neurological sympt
oms are not characteristic of Korsakoff disease.

Solution for Question 6:


Correct Option C:
• Based on the given information, the most likely diagnosis is a neural tumor. Neural tumors arise from
the cells of the nervous system and can present as slow-growing, rubbery, non-tender swellings. The
presence of freckles and skin patches over the face and trunk further supports the suspicion of a neural
tumor.
• Neural tumors can include various types, such as neurofibromas, schwannomas, or ganglioneuromas.
These tumors can occur in different locations along the peripheral nerves, including the neck.
Incorrect Options:
Option A: Cystic hygroma: Cystic hygromas are benign masses that typically occur in the neck and are
characterized by large, fluid-filled cysts. They are usually present at birth or appear in early childhood.
Unlike the given scenario, cystic hygromas are not associated with freckles or skin patches over the fa
ce and trunk.
Option B: Chemodectoma: Chemodectomas, also known as paragangliomas, are rare tumors that aris
e from specialized cells called paraganglia. They commonly occur in the head and neck region, includi
ng the carotid body and jugulotympanic region. However, the presence of freckles and skin patches ov
er the face and trunk suggests a different diagnosis than chemodectoma.
Option D: Lipoma: Lipomas are benign tumors composed of fatty tissue. They are usually soft, mobile,
and non-tender. While lipomas can occur in various locations, they are less likely to present with associ
ated freckles and skin patches over the face and trunk, as described in the scenario.

Page 5

1610
Oral Cavity
1. A 73-year-old man comes to his primary care physician for an annual well-visit. He has no
complaints and feels well. He has smoked one pack of cigarettes every day for the past 45 years. Oral
examination reveals a thin, white patch with well-defined margins on the left upper palate. The
physician attempts to scrape it off with a tongue depressor but cannot do so. What is the diagnosis?
(or)
An elderly man, a chronic smoker, presented with a thin, white patch with well-defined margins on the
left upper palate that cannot be scraped off. What is the likely diagnosis?
A. Squamous cell carcinoma
B. Thrush
C. Gingivitis
D. Leukoplakia
----------------------------------------
2. What is the TNM stage of a cheek cancer tumor of 2.5 cm and involving the lower alveolus, with a
palpable single mobile ipsilateral lymph node of 6 cm?
(or)
A patient with cheek cancer has a tumor of 2.5 cm located close to and involving the lower alveolus. A
single mobile ipsilateral lymph node measuring 6 cm is palpable. What is the TNM stage of this tumor?
A. T1N1M0
B. T2N2M0
C. T2N1M0
D. T4N2M0
----------------------------------------
3. Which of the following is correct regarding Carcinoma of the lip?
(or)
A 40-year-old female came to the outpatient department complaining of sensations of tingling, pain,
and numbness of the lips and the skin around the mouth. She also had a flat or slightly raised whitish
discoloration of the lip and a sore on the lip that did not heal. Which of the following is the correct
regarding carcinoma of the lips?
A. Non-healing ulcers or growth is the most common presentation
B. The most common site is vermillion of the upper lip
C. If 1/3rd or less of the lip is involved, the Abbe-Estlander flap is used for reconstruction
D. Abbe-Estlander flap is based on the inferior labial artery
----------------------------------------
4. A 51-year-old woman comes to her physician due to a painless lump on the right side of her face.
She noticed the mass growing over the past several months, and one week ago she noticed a
right-sided facial droop. Physical examination shows a right-sided, non-tender, mobile, preauricular
mass extending to the angle of the mandible. A biopsy of the mass is performed, and pathological
examination shows the proliferation of ductal myoepithelial and epithelial cells with a marked increase

1611
in stromal components. Staining with S100 is negative. Which of the following is the most likely
diagnosis?
(or)
Which is the most likely diagnosis for a patient presenting with a painless lump on the right side of her
face, histopathological examination showing proliferation of ductal myoepithelial and epithelial cells with
a marked increase in stromal components, and negative staining with S100?
A. Mucoepidermoid carcinoma
B. Pleomorphic adenoma
C. Facial nerve schwannoma
D. Sialadenosis
----------------------------------------
5. What is the diagnosis for a patient with history of megaloblastic anemia and small, oval multiple
ulcers in the oral cavity with red erythematous margins, tingling sensations in the ulcers?
(or)
A 45-year-old female patient has multiple small, oval ulcers with red erythematous margins in the oral
cavity. She has tingling sensations in the ulcers. She also has a history of megaloblastic anemia. What
is the diagnosis?
A. Carcinoma
B. Tubercular ulcer
C. Aphthous ulcer
D. Syphilitic ulcer
----------------------------------------
6. A 55-year-old male presents to the surgical outpatient department for evaluation of a lesion on his
lower lip that developed in the past year. He is employed as a landscaper and reports infrequent use of
sun protection. Physical examination reveals a 5 mm erythematous, ulcerative nodule within the
vermilion border of the right lateral aspect of the lower lip. A biopsy confirms invasive squamous cell
carcinoma, and the doctor refers the patient for Mohs micrographic surgical excision. Which lymph
nodes would be most critical to evaluate for metastasis before the surgery?
(or)
Which lymph nodes would be most critical to evaluate for metastasis before Mohs micrographic surgical
excision in a patient with invasive squamous cell carcinoma of the right lateral lip??
A. Submental lymph nodes
B. Submandibular lymph nodes
C. Posterior cervical chain
D. Jugulodigastric lymph node
----------------------------------------
7. A 56-year-old man comes to the physician because of throat pain and increasing difficulty swallowing
food for the past 5 months. He has also had episodic left ear pain for the past 3 months. Further history
reveals hypertension, chronic smoking, and alcohol. Family history is positive for laryngeal cancer.
Examination shows left cervical lymphadenopathy. Oropharyngeal examination shows a 3 cm
ulcerating mass infiltrating the floor of the mouth. The senior physician recommends the commando

Page 2

1612
operation. Which of the following statements best describes the commando operation?
(or)
Which of the following statements best describes the commando operation?
A. Abdominoperineal resection of the rectum for carcinoma
B. Resection of all level 1 to 4 neck lymph nodes
C. Extended radical mastectomy
D. Excision of carcinoma of the jaw and lymph nodes
----------------------------------------
8. A 50-year-old woman presents to the emergency department with a history of facial swelling, ear
pain, dysphagia, and cervical lymphadenopathy. She is diagnosed with carcinoma of the oral cavity.
Which of the following is a precancerous lesion of the oral cavity?
(or)
Which of the following is a precancerous lesion of the oral cavity?
A. Acute tonsillitis
B. Speckled erythroplakia
C. Discoid lupus erythematosus
D. All of the above
----------------------------------------
9. What is the most likely diagnosis for a middle-aged man with a history of chronic smoking,
alcoholism, and a painless growth in the mouth, white patches on the oral mucosa along the gums,
cannot be scraped with gauze, marked increase in the thickness of the stratum corneum and the
presence of nucleated keratinocytes on biopsy?
(or)
A 58-year-old man comes to the physician because of a 3-week history of a painless growth in his
mouth. He is a chronic smoker and alcoholic.. Physical examination shows white patches on the oral
mucosa along the gums that cannot be scraped with gauze. The lesions have a granular texture. A
combination HIV-1/2 immunoassay is negative. A lesion biopsy shows a marked increase in the
thickness of the stratum corneum and the presence of nucleated keratinocytes in this layer. Which of
the following is the most likely diagnosis?
A. Oral candidiasis
B. Squamous cell carcinoma
C. Lichen planus
D. Leukoplakia
----------------------------------------
10. What treatment is indicated for a 1 x 1.5 cm growth on the lateral border of the tongue?
(or)
An abnormal growth measuring 1 x 1.5 cm was discovered on the lateral margin of the tongue of a
35-year-old male patient. His vital signs appear to be within normal ranges. The recommended course
of treatment is?

Page 3

1613
A. Laser ablation
B. Interstitial brachytherapy
C. External beam radiotherapy
D. Chemotherapy
----------------------------------------
11. A 32-year-old female from a rural background presented with a history of chronic tobacco chewing
since age 14. Now she has difficulty opening her mouth. On oral examination, no ulcers are seen.
Which of the following is the most probable diagnosis?
(or)
What is the most probable diagnosis in a middle-aged female with a history of chronic tobacco chewing
since the age of 14 and difficulty in mouth opening?
A. Oral submucous fibrosis
B. Oral hairy leukoplakia
C. Erythroleukoplakia
D. Trigeminal neuralgia
----------------------------------------
12. What is the treatment of choice for an elderly edentulous patient with a midline tumor of the lower
jaw involving the alveolar margin?
(or)
An 80-year-old patient presents with a midline tumor of the lower jaw involving the alveolar margin. He
is edentulous. What is the treatment of choice?
A. Hemimandibulectomy
B. Commando operation
C. Segmental mandibulectomy
D. Marginal mandibulectomy
----------------------------------------
13. A 56-year-old man comes to the physician because of throat pain and increasing difficulty
swallowing food for the past 5 months. He has also had episodic left ear pain for the past 3 months.
Further history reveals hypertension, chronic smoking, and alcohol. Family history is positive for
laryngeal cancer.. Examination shows left cervical lymphadenopathy. Oropharyngeal examination
shows a 3 cm ulcerating left tonsillar mass. Examination of both auditory canals is unremarkable.
Which of the following is the most appropriate initial step in management?
(or)
What is the initial step in management for a 56-year-old man with throat pain, difficulty swallowing, left
cervical lymphadenopathy, and a 3-cm ulcerating left tonsillar mass?
A. Esophagoscopy
B. Barium swallow
C. Nasopharyngoscopy
D. Panendoscopy

Page 4

1614
----------------------------------------
14. A 24-year-old woman presents to you with impacted wisdom teeth and associated pain. Impacted
wisdom teeth may produce referred pain via which of the following nerves?
(or)
Impacted wisdom teeth may produce referred pain via which of the following nerves?
A. Lingual nerve
B. Facial nerve
C. Branch of the Auriculotemporal nerve
D. Chorda tympani nerve
----------------------------------------
15. A 6-year-old girl is brought to the physician by her parents because of a 1-month history of pain and
increasing swelling over her scalp. There is no history of trauma, and vitals are stable. Examination
shows a 3 cm solitary, tender mass over the right parietal bone. An x-ray of the skull shows a solitary
osteolytic lesion. The radiographic finding of floating teeth can be seen. Laboratory studies show:
Haemoglobin 10.9 g/dL Leukocyte count 7300/mm3 Serum Na+ 136 mEq/L K+ 3.7 mEq/L Cl- 103
mEq/L Ca2+ 9.1 mg/dL Glucose 71 mg/dL Which of the following is the most likely diagnosis?
(or)
What is the most likely diagnosis in a child with a 1-month history of pain and swelling over the scalp, a
solitary tender mass over the right parietal bone, an x-ray showing a solitary osteolytic lesion, floating
teeth and Laboratory studies show: Haemoglobin 10.9 g/dL Leukocyte count 7300/mm3 Serum Na+
136 mEq/L K+ 3.7 mEq/L Cl- 103 mEq/L Ca2+ 9.1 mg/dL Glucose 71 mg/dL
Haemoglobin 10.9 g/dL
Leukocyte count 7300/mm3
Serum
Na+ 136 mEq/L
K+ 3.7 mEq/L
Cl- 103 mEq/L
Ca2+ 9.1 mg/dL
Glucose 71 mg/dL

A. Fibromuscular dysplasia
B. Cleidocranial dysplasia
C. Osteopetrosis
D. Histiocytosis – X
----------------------------------------
16. A 55-year-old woman comes to the physician with a 2-week history of painful swelling on the right
side of her face. The pain worsens when she eats. Examination of the face shows a right-sided, firm
swelling that is tender to palpation. An oral examination reveals no abnormalities. Ultrasonography
shows a stone in a duct that runs anterior to the masseter muscle and passes through the buccinator
muscle. A sial endoscopy is performed to remove the stone. At which of the following sites is the
endoscope most likely to be inserted during the procedure?

Page 5

1615
(or)
At which site endoscope is inserted during a sial endoscopy procedure in a patient with sialolithiasis of
parotid gland duct?
A. Lateral to the second upper molar tooth
B. Lateral to the lingual frenulum
C. Into the floor of the mouth
D. Into the mandibular foramen
----------------------------------------
17. A 25-year-old man presented to the emergency department. He has flat white patches in his oral
cavity and unusual bleeding in the mouth. A biopsy is done, and end-stage oral cavity cancer is
diagnosed. Which of the following is one of the biggest risk factors for developing oral cavity cancer?
(or)
Which of the following is one of the most important risk factors for developing oral cavity cancer?
A. Age
B. High blood cholesterol
C. Human papillomavirus (HPV) infection
D. Lack of sleep
----------------------------------------
18. A 55-year-old male presented to the emergency department with a complaint of a mass on the floor
of the oral cavity, as shown in the picture. Following the completion of the history and examination, He
has been diagnosed with Ranula. In the case ofmanagement of recurrent Ranula, which procedure is
used?
(or)
Which of the following procedures is done in case of management recurrent Ranula?

A. Incision and drainage


B. Aspiration
C. Excision
D. Sclerosant injection
----------------------------------------

Page 6

1616
19. A patient has multiple small, oval ulcers with red erythematous margins in the oral cavity. She has
tingling sensations in the ulcers. Later on, she was diagnosed with oral cancer. Which stain can be
used to identify areas of carcinoma of the oral mucosa?
(or)
Which of the following stains is used to identify areas of carcinoma of the oral mucosa?
A. 1% zinc chloride
B. 2% silver nitrate
C. Gentian violet
D. 2% toluidine blue
----------------------------------------
20. What is the most probable treatment for a male tobacco chewer with a maxillary malignancy staged
as T3N1 carcinoma?
(or)
A 41-year-old male tobacco chewer came to the hospital with a maxillary malignancy. Staging revealed
T3N1 cancer. Which of the following is the most probable treatment?
A. Radiation therapy only
B. Chemotherapy only
C. Surgery and radiation
D. Chemotherapy and radiation
----------------------------------------
21. A 40-year-old female from a rural area presented with a history of chronic tobacco chewing and
consumption of spicy foods since the age of 14. Now she has difficulty opening her mouth. She also
complains of trismus, dysphagia, and a severe burning sensation in the mouth after ingesting spicy
foods. The image is given below. Which of the following are the treatment options for this condition?
(or)
What are the treatment options for a female presenting with difficulty opening her mouth, trismus,
dysphagia, and a severe burning sensation in the mouth after ingesting spicy foods, as shown in the
image?

A. Intralesional steroids
B. Surgical excision with skin grafting

Page 7

1617
C. Radiotherapy
D. Chemotherapy
----------------------------------------
22. A 50-year-old man presents to the surgical outpatient department due to tongue tenderness, pain
with eating, and difficulty in tongue movement for 1 week. On examination, there is a mass on the
ventral aspect of the tongue that is tender to palpation. A biopsy of the lesion confirmed squamous cell
carcinoma of the tongue. Which of the following is an etiological factor for oral cavity malignancies?
(or)
What is a common etiological factor for the development of oral cavity malignancies?
A. Vitamin B12 deficiency
B. HPV infection
C. Vitamin C deficiency
D. Candida infection
----------------------------------------
23. A 41-year-old male tobacco chewing patient came to the hospital with a diagnosis of maxillary
carcinoma. Which of the following lymph nodes is least affected in maxillary carcinoma?
(or)
Which of the following lymph nodes is least affected in maxillary carcinoma?
A. Superior deep cervical nodes
B. Jugulodigastric nodes
C. Submandibular nodes
D. Subdigastric nodes
----------------------------------------
24. A 55-year-old woman presented to the OPD with a history of difficulty swallowing and mouth pain.
An examination revealed a mouth sore on the right side of the tongue and a whitish patch on the
surface of the tongue. She described a long history of smoking tobacco. The attending physician
diagnosed him with carcinoma of the oral cavity and informed her of treatment options. What is the
best-known treatment option for treating this disease?
(or)
What is the best-known treatment option for oral cavity carcinoma in a patient presenting with difficulty
in swallowing and mouth sore?
A. The most common gene mutated in carcinoma oral cavity is PTEN and can be treated with Limberg
Flap
B. The most common site of oral carcinoma cavity is the lip, which does not require treatment
C. Bilateral hematogenous spread is common in the lower lip, supraglottis, and soft palate and is
treated by radiotherapy
D. The most common flap used for the reconstruction of head and neck malignancies is the PMMC flap
----------------------------------------

Page 8

1618
25. A 45-year-old male patient presents to the surgical outpatient department due to mild swelling and
pain in the left zygomatic region for two weeks. It is associated with an intraoral ulcer. He has no history
of any medical disorder and takes multivitamins only. He has a smoking history for past 15 years.
General physical examination reveals a palpable, tender, and fixed left submandibular lymph node and
left cervical lymph node. Intraoral examination reveals an ulcerative growth measuring 5 cm. A biopsy
confirms the diagnosis of maxillary carcinoma. The surgeon uses an incision (shown below) for broad
access to the maxilla: Which of the following best represents the incision shown above?
(or)
Which of the following best represents the incision shown below used by a surgeon for broad access to
the maxilla in a patient diagnosed with maxillary carcinoma?

A. Lazy' S' incision


B. Modified Blair's incision
C. Weber–Ferguson incision
D. Sistrunk incision
----------------------------------------
26. A 46-year-old man develops a lesion in the vestibule of his mouth that on histological examination is
revealed to be verrucous carcinoma of the upper aerodigestive tract. Which of the following statements
is true of this lesion?
(or)
Which of the following statements best describes verrucous carcinoma of the upper aerodigestive
tract?
A. Most commonly seen in the inner part of the cheek
B. It is linked to a high metastatic rate
C. Gross appearance is ulcerative
D. Best treatment is radiation
----------------------------------------
27. A 45-year-old farmer presents to the surgical outpatient department due to a lesion on his lower lip.
He has no associated symptoms. He has smoked one pack a day for the past 15 years. Physical
examination reveals: Which of the following is the most likely diagnosis in this case?
(or)
What is the most likely diagnosis for a farmer who presents with a lesion on lower lip, with history of one
pack of cigarettes for the past 15 years. Physical examination reveals:

Page 9

1619
A. Basal cell carcinoma
B. Squamous cell carcinoma
C. Malignant melanoma
D. Verrucous carcinoma
----------------------------------------
28. A 45-year-old man presents to the emergency department with a painful sore in his mouth,
dysphagia, and hoarseness of voice. He is a chronic smoker. He is diagnosed with T4N0M0 stage head
and neck carcinoma. What is the treatment modality for this patient?
(or)
What is the treatment for T4N0M0 head and neck carcinoma?
A. Surgery alone
B. Radiotherapy alone
C. Chemoradiation
D. Surgery and radiotherapy
----------------------------------------

Correct Answers
Question Correct Answer

Question 1 4
Question 2 4
Question 3 1
Question 4 2
Question 5 3
Question 6 2
Question 7 4
Question 8 2
Question 9 4

Page 10

1620
Question 10 2
Question 11 1
Question 12 3
Question 13 4
Question 14 3
Question 15 4
Question 16 1
Question 17 3
Question 18 3
Question 19 4
Question 20 3
Question 21 1
Question 22 2
Question 23 3
Question 24 4
Question 25 3
Question 26 1
Question 27 2
Question 28 4

Solution for Question 1:


Option D: Leukoplakia
• Oral leukoplakia is a premalignant lesion that presents as a white patch on the oral mucosa. It cannot
be scraped off.
• Risk factors for developing oral leukoplakia are similar to those predisposing to squamous cell
carcinoma (SCC ), including alcohol and tobacco use (smoked and especially smokeless).
Option A: Squamous cell carcinoma
• Oral squamous cell carcinoma is a malignant tumor that may occur anywhere within the oral cavity.
• It is locally invasive, infrequently metastasizes to ipsilateral regional lymph nodes, and rarely spreads
to distant sites.
• Given the local invasiveness, the prognosis for this fast-growing tumour is poor, so it is vital to identify
and treat the condition early. The average reported duration of survival after diagnosis of this tumor was
reported to be as two m
Option B: Thrush
• Oral thrush also called oral candidiasis is a condition in which the fungus Candida albicans
accumulates on the lining of your mouth. Candida is a normal organism in your mouth, but sometimes it
can overgrow and cause symptoms.

Page 11

1621
• Oral thrush causes creamy white lesions, usually on your tongue or inner cheeks. Sometimes oral
thrush may spread to the roof of your mouth, your gums or tonsils, or the back of your throat.
• It is more likely to occur in immunocompromised individuals and in people who take certain
medications like steroids.
• In oral thrush, candida can be removed using a cotton swab where as in leukoplakia the patch can't
be scrapped off.
Option C: Gingivitis
• Gingivitis is an early form of periodontal disease. Periodontal disease is inflammation and infection
that destroys the tissues that support the teeth. This can include the gums, periodontal ligaments, and
bone.
• Gingivitis is due to the short-term effects of plaque deposits on your teeth. Plaque is a sticky material
made up of bacteria, mucus, and food debris that builds up on the teeth both above and below the
gums. It is also a major cause of tooth decay.

Solution for Question 2:


Option D: T4N2M0
• Cheek cancer (2.5 cm) located close to and involving the lower alveolus (T4a); A single mobile
ipsilateral lymph node measuring 6 cm is palpable (N2a); TNM stage is T4a N2a M0. T4a (lip)→ Tumor
invades through cortical bone, inferior Alveolar nerve, the floor of the mouth, and skin of the chin or
nose. T4a(oral cavity)→ Tumor invades the cortical bone, mandible, maxillary sinus, or skin of the face.
T4b → Tumors are involved through masticator space, pterygoid plate skull base, and/or encases
internal carotid artery. N2a → Metastasis to single ipsilateral LN up to >3-6 cm dimensions N2b →
Metastasis to multiple ipsilateral LN none >6 cm in greatest dimension M0 → No Metastasis
• T4a (lip)→ Tumor invades through cortical bone, inferior Alveolar nerve, the floor of the mouth, and
skin of the chin or nose.
• T4a(oral cavity)→ Tumor invades the cortical bone, mandible, maxillary sinus, or skin of the face.
• T4b → Tumors are involved through masticator space, pterygoid plate skull base, and/or encases
internal carotid artery.
• N2a → Metastasis to single ipsilateral LN up to >3-6 cm dimensions
• N2b → Metastasis to multiple ipsilateral LN none >6 cm in greatest dimension
• M0 → No Metastasis
• T4a (lip)→ Tumor invades through cortical bone, inferior Alveolar nerve, the floor of the mouth, and
skin of the chin or nose.
• T4a(oral cavity)→ Tumor invades the cortical bone, mandible, maxillary sinus, or skin of the face.
• T4b → Tumors are involved through masticator space, pterygoid plate skull base, and/or encases
internal carotid artery.
• N2a → Metastasis to single ipsilateral LN up to >3-6 cm dimensions
• N2b → Metastasis to multiple ipsilateral LN none >6 cm in greatest dimension
• M0 → No Metastasis
Option A: T1N1M0

Page 12

1622
• T1 → Tumor up to 2 cm in size; up to 5 mm depth of invasion
• N1 → Metastasis to single ipsilateral LN up to 3 cm in greatest dimension
• M0 → No Metastasis
Option B: T2N2M0
• T2 → Up to 2 cm in size with >5 -10 mm depth of invasion → >2 - 4 cm, depth of invasion up to 10 mm
• N2a → Metastasis to single ipsilateral LN up to >3 -6 cm dimensions
• N2b → Metastasis to multiple ipsilateral LN none >6 cm in greatest dimension
• M0 → No Metastasis
Option C: T2N1M0
• T2 → Up to 2 cm in size with >5 -10 mm depth of invasion → >2 - 4 cm, depth of invasion up to 10 mm
• N1 → Metastasis to single ipsilateral LN up to 3 cm in greatest dimension
• M0 → No Metastasis

Solution for Question 3:


Option A: Non-healing ulcers or growth is the most common presentation
• Lip cancer often looks like a mouth sore that won't heal. In people with light skin, this sore may appear
reddish. In people with darker skin, it may appear dark brown or grey.
• Most common in males
Option B: The most common site is vermillion of the upper lip
• The most common site is a vermillion of the lower lip.
• Lip cancer can occur anywhere along the upper or lower lip but is most common on the lower lip. Lip
cancer is considered a type of mouth (oral) cancer. Most lip cancers are squamous cell carcinomas,
which begin in the thin, flat cells in the middle and outer layers of the skin called squamous cells.
• The lower lip is affected far more frequently (80% to 95%) than the upper lip (2% to 12%) or
commissure (1% to 15%).
Option C: If 1/3rd or less of the lip is involved, the Abbe-Estlander flap is used for reconstruction
• If 1/3rd or less of the lip is involved: 'V' or 'W' shaped full-thickness excision with a lateral margin of 5
mm + primary closure is the treatment of choice.
• Full-thickness lip defects involving less than 1/3rd of lip width are most commonly surgically repaired
using primary closure.
• Different excision patterns like simple “V" excision, “W” or “M” plasty, and V–Y closure can be
performed to achieve primary closure of a lip defect.
• For larger defects, Abbe-Estlander flaps are used.
Option D: Abbe-Estlander flap is based on the inferior labial artery
• The Abbe–Estlander flap, fed by the opposite superior labial artery, is a full-thickness
triangular-shaped upper lip flap that creates the oral commissure and the lateral part of the lower lip.
• This flap begins from the medial part of the oral commissure and is approximately 1.5 × 3 cm in size.

Page 13

1623
Solution for Question 4:
Option B: Pleomorphic adenoma
• Pleomorphic adenomas are the most common salivary gland tumours and are usually benign. They
are composed of chondromyxoid stroma and epithelium derived from both epithelial and mesenchymal
cells.
• Pleomorphic adenomas typically present as slow-growing, mobile, painless masses.
• Symptoms and signs depend on the size, location, and potential to undergo malignant transformation.
In the parotid gland, symptoms of facial nerve weakness can occur when the tumour is large or
undergoes malignant change invading adjacent structures.

Option A: Mucoepidermoid carcinoma


• Mucoepidermoid carcinomas have variable amounts of squamous cells, mucus-secreting cells, and
intermediate cells.
• They are the most common primary malignant tumour of the salivary glands and occur mainly in the
parotid glands. This patient's biopsy shows epithelial and mesenchymal differentiation, thus
differentiating from the pleomorphic adenoma.
Option C: Facial nerve schwannoma
• Facial nerve schwannoma are benign peripheral nerve sheath tumours that commonly present with
facial paresis or hearing loss.
• Schwannomas demonstrate S100 expression; therefore, this patient's pathological and
immunohistochemical staining suggest a different diagnosis.
Option D: Sialadenosis
• Sialadenosis typically presents as bilateral, non-tender swelling of the parotid glands; the condition
occurs in several systemic conditions (e.g., nutritional disorders, hypothyroidism, cirrhosis ).
• Pathological examination typically reveals atrophy of parenchymal tissue and a compensatory
increase in adipose tissue.

Solution for Question 5:


Option C: Aphthous ulcer
• One of the causes of Aphthous ulcers is Vitamin B12 deficiency(megaloblastic anemia).
• Most canker sores are round or oval with a white or yellow centre and a red border. They form inside
the mouth — on or under the tongue, inside the cheeks or lips, at the base of the gums, or on the soft
palate.

Option A: Carcinoma

Page 14

1624
• Almost all of the cancers in the oral cavity and oropharynx are squamous cell carcinomas. These
cancers start in squamous cells, which are flat, thin cells that form the lining of the mouth and throat.
• The earliest form of squamous cell cancer is called carcinoma in situ. This means that the cancer cells
are only in the layer of cells called the epithelium (the top layer of cells lining the oral cavity and
oropharynx). This is different from invasive squamous cell cancer, where the cancer cells have grown
past the epithelium into the deeper layers of the oral cavity or oropharynx.
Option B: Tubercular ulcer
• Firm, non-tender erythematous nodules that soften, ulcerate, and form sinuses.
• When oral TB occurs as a primary lesion, an ulcer is the most common manifestation, usually
developing along the lateral margins of the tongue, which rest against rough, sharp, or broken teeth or
at the site of other irritants.
• Patients with oral tubercular lesions often have a history of pre-existing trauma.
Option D: Syphilitic ulcer
• Firm, painless, non-itchy skin ulceration
• Secondary syphilis presents in the oral cavity in many forms, including multiple white mucous
patches, condyloma lata, and split papules.
• Oral ulcers are a relatively common clinical presentation due to a wide variety of causes, which makes
recognition of oral syphilis challenging.

Solution for Question 6:


Option B: Submandibular lymph nodes

• Carcinoma of the lip most commonly involves the vermilion border of the lower lip away from the line
of contact with the upper lip.

Page 15

1625
• 15 % of lip carcinomas arise from the central third and commissure regions, and 5 % arise from the
upper lip.
• The tumours spread laterally rather than through deep infiltration. Uncontrolled tumours can spread
into the anterior triangle of the neck and can invade the mandible.
• Lymph node metastases occurs late. Therefore, surgery and radiotherapy are frequently employed
and are highly effective methods of treatment, each giving cure rates of about 90 percent.
• Lymphatic drainage from the upper lip and lateral aspects of the lower lip is directed to the ipsilateral
submandibular lymph nodes.
• The lip region has a significant crossover potential in lymphatic drainage patterns.
Option A: Submental lymph nodes
• Only the central portion of the lower lip drains into submental lymph nodes.
• This patient had a lesion involving the right lateral lip that drains into submandibular lymph nodes.
Option C: Posterior cervical chain
• The thyroid, parathyroid, and cervical part of the oesophagus and larynx drain into the posterior
cervical chain.
• These lymph nodes are not involved in lower lip carcinoma.
Option D: Jugulodigastric lymph node
• The jugulodigastric lymph nodes, also known as subdigastric lymph nodes, are the deep cervical
nodes that receive lymphatic drainage from the pharynx, tonsils, oral cavity, and face.
• They do not receive drainage from the lower lip. Hence, they are not involved in the carcinoma of the
lip.

Solution for Question 7:


Option D: Excision of carcinoma of the jaw and lymph nodes
Oral cavity cancer:

Page 16

1626
• Oral cavity cancers refer to malignant tumors of the oral mucosa, tonsils, and salivary glands.
• Predisposing factors - smoking, oral tobacco consumption, long-term alcohol use, and human
papillomavirus infection.
• Usually present in males aged 55–60 years
• Clinical features - pain, dysphagia, or a nonhealing ulcer on the tonsils, tongue, or oral mucosa.
• Clinically suspected cases are confirmed via histopathological examination of a biopsy specimen.
• Imaging and pan endoscopy help determine the extent of the tumor and rule out spread.
• Treatment depends upon the stage of the tumor and the extent of its spread and may include surgical
resection (usually with neck dissection), radiation therapy, and chemotherapy.
• Commando’s operation: Combined mandibulectomy and neck dissection operation Total
glossectomy + hemi-mandibulectomy + removal of floor of mouth + Radical lymph node dissection
Indication - Carcinoma fixed to mandible with infiltration to floor of the mouth
• Total glossectomy + hemi-mandibulectomy + removal of floor of mouth + Radical lymph node
dissection
• Indication - Carcinoma fixed to mandible with infiltration to floor of the mouth
• Total glossectomy: This is the removal of the entire tongue, including the base of the tongue.
• Mandibulectomy is a surgical procedure wherein the mandible (jaw), a crucial bone in the face that
contributes significantly to mastication or chewing, is removed or resected.
• Total glossectomy + hemi-mandibulectomy + removal of floor of mouth + Radical lymph node
dissection
• Indication - Carcinoma fixed to mandible with infiltration to floor of the mouth
Option A: Abdominoperineal resection of the rectum for carcinoma
• The patient has cancer of the oral cavity infiltrating the floor of the mouth. Its treatment includes
surgical resection of the tongue, mandible, floor of the mouth, and the infiltrating lymph nodes.
• Abdominoperineal resection (APR) is a type of surgery in which the anus, rectum, and sigmoid colon
are removed through small cuts in the belly. It is a surgical procedure done primarily to remove cancer

Page 17

1627
from the rectum.
Option B:Resection of all level 1 to 4 neck lymph nodes.
• Commando operation involves total glossectomy, hemimandibulectomy, resection of floor of the
mouth and radical neck dissection.
Option C: Extended radical mastectomy
• A mastectomy is a surgery to remove a breast. There are several types of mastectomy, including
total, modified radical, and radical.
• A radical mastectomy is the most extensive type. During the procedure, the surgeon removes the
entire breast, the underarm (axillary) lymph nodes, and the chest wall muscles.

Solution for Question 8:


Option B: Speckled erythroplakia
• Erythroplakia
• Speckled Erythroplakia
• Chronic hyperplastic candidiasis
• Oral submucous fibrosis
• Syphilitic glossitis
• Sideropenic dysphagia (Paterson – Kelly syndrome)
• Oral lichen planus
• Discoid lupus erythematosus
• Discoid keratosis congenita
• Speckled erythroplakia also known as Mixed leukoplakia/erythroplakia
• It is a nonhomogeneous oral leukoplakia characterized by white plaques with red speckles.
• It has a higher risk of malignant transformation than homogeneous leukoplakia.
Option A: Acute tonsillitis
• Inflammation of the pharyngeal tonsils. It frequently arises in combination with inflammation of the
pharynx; for this reason, the terms tonsillitis and pharyngitis are often used interchangeably or in
combination (tonsillopharyngitis).
• It is commonly caused by viruses and group A beta-hemolytic streptococci.
• Clinical presentation include fever, dysphagia, and tender anterior cervical lymphadenopathy
• The pharyngeal tonsils are enlarged, erythematous, edematous, and have streaks of mucopurulent
discharge bilaterally.
Option C: Discoid lupus erythematosus
• DLE is the most common form of chronic cutaneous lupus erythematosus.
• DLE has a low risk of malignant transformation

Page 18

1628
• Clinical features include : Erythematous, inflammatory scaly plaques that are painful to remove. The
plaques heal but cause scarring alopecia, atrophy, peripheral hyperpigmentation, and central
depigmentation. Typically affects the face, neck, and head (triggered by exposure to UV light)
• Erythematous, inflammatory scaly plaques that are painful to remove.
• The plaques heal but cause scarring alopecia, atrophy, peripheral hyperpigmentation, and central
depigmentation.
• Typically affects the face, neck, and head (triggered by exposure to UV light)
• Erythematous, inflammatory scaly plaques that are painful to remove.
• The plaques heal but cause scarring alopecia, atrophy, peripheral hyperpigmentation, and central
depigmentation.
• Typically affects the face, neck, and head (triggered by exposure to UV light)
Option D: All of the above
• Speckled erythroplakia has a higher risk of malignant transformation

Solution for Question 9:


Option D: Leukoplakia
• Precancerous hyperkeratosis and dysplasia of the epithelium and mucous membranes are most
commonly located in the oral cavity.
• This patient has oral leukoplakia, a relatively common precancerous condition of the oral epithelium.
• Smoking and alcohol consumption are important risk factors for this condition, manifesting as single
or multiple asymptomatic white plaques that cannot be scraped off.
• Any part of the oral cavity can be affected. If untreated, leukoplakia can undergo malignant
transformation into squamous cell carcinoma.
• The risk of malignant transformation is higher if the white patches are nonhomogenous in appearance
and texture (e.g., erythroplakia, verrucous leukoplakia).

Page 19

1629
Option A: Oral candidiasis
• The pseudomembranous (common) and hyperplastic (rare) forms of oral candidiasis manifest as
white lesions in the oral cavity.
• Chronic smoking, which can cause xerostomia and alter the balance of oral bacterial flora, is also a
minor risk factor for oral candidiasis.
• However, the lesions of pseudomembranous candidiasis can be scraped off, unlike this patient's
lesions.
• Hyperplastic candidiasis lesions cannot be scraped off, but they typically appear near the labial
commissure, unlike the gingival lesions in this patient. Moreover, a biopsy of oral candidiasis lesions
would show hyphal structures and an infiltration of inflammatory cells.

Option B: Squamous cell carcinoma


• A malignant epithelial cancer that arises from squamous cells. Common sites of origin include the
lung, head and neck, skin, and esophagus.

Page 20

1630
• In-situ oral squamous cell carcinoma (SCC) can appear as a white patch that cannot be scraped off.
This patient also has several risk factors for oral SCC (e.g., 50–70 years of age, male sex,
smoking, alcohol use).
• However, a biopsy in the case of carcinoma in situ would show evidence of dysplastic changes (e.g.,
hyperchromatism, increased nuclear-cytoplasmic ratio) extending to the basement membrane of the
oral epithelium.
• This patient has a premalignant condition that can develop into oral SCC, but there is no histological
evidence of SCC at this time.
Option C: Lichen Planus
• A chronic inflammatory disease characterized by skin and mucosal lesions. Skin lesions are
characterized by the 5 P's (planar, polygonal, pruritic, purple papules, and plaques). Involvement of the
oral mucosa manifests as reticular white lines (Wickham striae).
• Lichen planus typically occurs in middle-aged individuals, manifests as asymptomatic white lesions of
the oral mucosa, and can show hyperkeratosis and parakeratosis on biopsy.
• Moreover, lichen planus is characterized by hypergranulosis, degeneration of the stratum basale, and
saw-toothed rete ridges with lymphocytic infiltration, none of which are seen here.

Solution for Question 10:


Option B: Interstitial brachytherapy
• Brachytherapy or Internal radiation is used in the treatment of cancer where radioactive material is
injected internally.
• Compared to the traditional kind of radiation therapy (external beam radiation), which projects
radiation from a machine outside the body, brachytherapy enables doctors to administer larger doses of
radiation to more precise locations of the body.

Page 21

1631
• Interstitial brachytherapy involves implanting radioactive seeds into or near a cancer tumor. The
seeds may be left in place permanently or temporarily. Brachytherapy is used to destroy cancer cells by
using an internal radiation source.
Option A: Laser ablation
• Laser ablation is a procedure that uses a laser of a specific wavelength to heat and destroy abnormal
tissue.
• Often used in ophthalmologic, surgical, and dermatologic procedures.
Option C: External beam radiotherapy
• It is a treatment method where a machine delivers carefully targeted radiation beams from outside the
body to destroy cancer cells, halt their growth, or offer palliative therapy for locally advanced
unresectable or metastatic disease.
• It is the most commonly used form of radiation therapy.
Option D: Chemotherapy
• Chemotherapy is a course of action that uses medication to kill or prevent the division of cancer cells
from halting the spread of the disease.
• Depending on the type and stage of the cancer being treated, chemotherapy may be given orally,
intravenously, intramuscularly, topically, or by any combination of these methods.

Solution for Question 11:


Option A: Oral submucous fibrosis
• The most probable diagnosis of the given clinical history is Oral submucous fibrosis in which difficulty
in opening her mouth with no ulcers seen.
• ORAL SUBMUCOUS FIBROSIS: Progressive disease in which fibrous bands form beneath the oral
mucosa. Almost confined to the Asian population. Risk factors: Pan masala areca nut, with or without
concurrent alcohol use. Tobacco smoking alone is not associated with oral submucous fibrosis.
Pathology: Characterized by epithelial fibrosis with associated atrophy & hyperplasia of the overlying
epithelium. Epithelium shows changes in epithelial dysplasia. Clinical features: Scarring produces
contracture resulting in limited mouth opening and restricted tongue movement Treatment: Restricted
mouth opening can be treated with either Intralesional steroids or Surgical excision and Skin grafts.
• Progressive disease in which fibrous bands form beneath the oral mucosa.
• Almost confined to the Asian population.
• Risk factors: Pan masala areca nut, with or without concurrent alcohol use. Tobacco smoking alone is
not associated with oral submucous fibrosis.
• Pan masala areca nut, with or without concurrent alcohol use.
• Tobacco smoking alone is not associated with oral submucous fibrosis.
• Pathology: Characterized by epithelial fibrosis with associated atrophy & hyperplasia of the overlying
epithelium. Epithelium shows changes in epithelial dysplasia.
• Characterized by epithelial fibrosis with associated atrophy & hyperplasia of the overlying epithelium.
• Epithelium shows changes in epithelial dysplasia.

Page 22

1632
• Clinical features: Scarring produces contracture resulting in limited mouth opening and restricted
tongue movement
• Treatment: Restricted mouth opening can be treated with either Intralesional steroids or Surgical
excision and Skin grafts.
• Progressive disease in which fibrous bands form beneath the oral mucosa.
• Almost confined to the Asian population.
• Risk factors: Pan masala areca nut, with or without concurrent alcohol use. Tobacco smoking alone is
not associated with oral submucous fibrosis.
• Pan masala areca nut, with or without concurrent alcohol use.
• Tobacco smoking alone is not associated with oral submucous fibrosis.
• Pathology: Characterized by epithelial fibrosis with associated atrophy & hyperplasia of the overlying
epithelium. Epithelium shows changes in epithelial dysplasia.
• Characterized by epithelial fibrosis with associated atrophy & hyperplasia of the overlying epithelium.
• Epithelium shows changes in epithelial dysplasia.
• Clinical features: Scarring produces contracture resulting in limited mouth opening and restricted
tongue movement
• Treatment: Restricted mouth opening can be treated with either Intralesional steroids or Surgical
excision and Skin grafts.
• Pan masala areca nut, with or without concurrent alcohol use.
• Tobacco smoking alone is not associated with oral submucous fibrosis.
• Characterized by epithelial fibrosis with associated atrophy & hyperplasia of the overlying epithelium.
• Epithelium shows changes in epithelial dysplasia.

Option B: Oral hairy leukoplakia


• Oral hairy Leukoplakia is a benign condition characterized by painless, white plaques with a feathery
or hairy appearance on the lateral tongue (or, less commonly, the floor of the mouth, palate, or buccal
mucosa) that cannot be scraped off.
• It is caused by Epstein-Barr virus infection but typically only occurs in patients who are also
immunocompromised.
Option C: Erythroleukoplakia
• Erythroleukoplakia is a type of nonhomogeneous oral leukoplakia characterized by white plaques with
red speckles.
• It has a higher risk of malignant transformation than homogeneous.
Option D: Trigeminal neuralgia
• Trigeminal neuralgia, or Tic douloureux, is a condition characterized by attacks of facial pain in the
area of one or more branches of the trigeminal nerve.
• The pain is typically very intense, sharp, and stabbing and lasts for several seconds.
• Attacks can occur without provocation but are sometimes triggered by innocuous stimuli like chewing.

Page 23

1633
Solution for Question 12:
Option C: Segmental mandibulectomy
Segmental Mandibulectomy
• The entire segment of the mandible is resected.
• This leads to discontinuity in the lower jaw.
• Requires major reconstructive surgery for aesthetic and functional reasons.
• Indications Invasion of the medullary canal of the lower jaw Fixation of the tumor on the occlusal
surface of the lower jaw in an edentulous patient Tumour invasion in the lower jaw, the mandibular
foramen, or mental foramen The tumor attaches to the lower jaw
• Invasion of the medullary canal of the lower jaw
• Fixation of the tumor on the occlusal surface of the lower jaw in an edentulous patient
• Tumour invasion in the lower jaw, the mandibular foramen, or mental foramen
• The tumor attaches to the lower jaw
• Invasion of the medullary canal of the lower jaw
• Fixation of the tumor on the occlusal surface of the lower jaw in an edentulous patient
• Tumour invasion in the lower jaw, the mandibular foramen, or mental foramen
• The tumor attaches to the lower jaw

Option A: Hemimandibulectomy
• Hemimandibulectomy is the surgical removal of one lateral half of the mandible.
• It is usually performed on malignant growths involving either the alveolar ridge and the mandible, or
adjacent structures, including the floor of the mouth.
• It is also indicated for benign masses of the jaw, especially those that are large and have already
destroyed the jawbone's integrity.
Option B: Commando operation
• Commando operation is a complicated procedure for tongue cancer of the first degree (COMbined
MAndibulectomy and Neck Dissection Operation)
• It includes block dissection of the cervical nodes, hemi mandibulectomy, and glossectomy (complete
tongue removal). The operation's extensive scope gave rise to its name.
Option D: Marginal mandibulectomy
• It is a conservative mandibulectomy
• Partial excision of the upper part of the lower jaw in the vertical phase.
• The inner cortex and part of the medullary cavity below are removed, and the continuity of the lower
jaw is preserved.
• Indicated when the tumor is 1 cm from the lower jaw or is adjacent to the periosteum with no evidence
of direct bony invasion

Page 24

1634
Solution for Question 13:
Option D: Panendoscopy
• A procedure in which a camera-equipped blade or an endoscope (either flexible or rigid) is inserted
via the mouth or nose to visualize the complete upper airway and digestive tract, including the pharynx,
larynx, upper trachea, and esophagus.
• This is useful for evaluating the pathologies of these structures, particularly in cases of suspected
pharyngeal carcinoma.
• Panendoscopy allows for comprehensive evaluation of the entire upper airway and digestive tract and
is indicated in all patients with suspected oropharyngeal cancer and a high risk for second primary
tumors (e.g., due to heavy smoking and family history of oral cancer).
• Biopsies of any masses would also be taken during or following the panendoscopy for histological
evaluation.
Option A: Esophagoscopy
• A diagnostic test in which a flexible endoscope is inserted through the mouth into the esophagus to
visualize the esophageal mucosa down to the gastroesophageal junction.
• Esophagoscopy would be used to assess for any concurrent masses in the esophagus.
• However, it would not help further evaluate this patient's tonsillar mass. Other potential primary sites
of SCC should also be identified, but these may not be limited to the esophagus.
Option B: Barium swallow
• An esophageal barium swallow is a diagnostic test in which serial x-rays of the chest are taken while a
patient swallows liquid barium, a contrast medium.
• Barium coats the walls of the esophagus and can be used to identify anatomical defects in the
esophagus (e.g., strictures, rings, dilation). Performed to evaluate symptoms such as painful and
difficult swallowing.
• A barium swallow is used to evaluate gastroesophageal reflux, assess a fistula or hiatal hernia, or
identify the cause of persistent vomiting or dysphagia.
• However, the cause of this patient's dysphagia has already been identified (a tonsillar mass).
• As this patient has cervical lymphadenopathy and known risk factors for oropharyngeal SCC, further
workup of the tonsillar mass and assessment for other masses in the head and neck region are
indicated.
Option C: Nasopharyngoscopy
• Nasopharyngoscopy is a procedure in which a camera-equipped blade or a fiberoptic endoscope
(either flexible or rigid) is inserted via the mouth or nose to visualize the nasopharynx.
• Used to evaluate foreign bodies, masses, or polyps or to identify the cause of epistaxis.
• Nasopharyngoscopy would help further evaluate this patient's tonsillar mass and assess for
other nasopharyngeal masses. However, it would not help identify other potential primary cancer sites
outside of the nasopharynx.

Page 25

1635
Solution for Question 14:
Option C: Branch of the Auriculotemporal nerve
• The auriculotemporal nerve is a sensory branch of the mandibular division of the trigeminal nerve.
• It has a cutaneous branch that supplies sensory innervation to the skin over the tragus, temple, and
tympanic membrane.
• It also has small articular branches to the temporomandibular joint and a parasympathetic branch to
the parotid gland.
• Thus, it carries the referred pain sensation in cases of impacted wisdom teeth.

Option A: Lingual nerve


• The lingual nerve is a sensory branch of the mandibular division of the trigeminal nerve.
• It supplies somatic sensation from the anterior two-thirds of the tongue.
• It does not carry the referred pain sensation in impacted wisdom tooth cases.
Option B: Facial nerve
• The facial nerve is a cranial nerve that provides motor innervation to the muscles of facial expression
and parasympathetic innervation to the lacrimal, submandibular, and sublingual glands.
• It carries afferents from taste receptors of the anterior 2/3 of the tongue and somatic receptors from
the anterior auditory canal and pinna.
• It does not carry the referred pain sensation in impacted wisdom tooth cases.
Option D: Chorda tympani nerve
• Chorda tympani nerve carries the sensation of taste from the anterior portion of the tongue to the
brain through the middle ear.
• This nerve also transports efferent secretomotor innervation to both the submandibular and sublingual
glands.

Page 26

1636
Solution for Question 15:
Option D: Histiocytosis – X
• Floating Tooth Sign: Radiology of the mandible shows erosion of the bony alveolus around the teeth
so that they look as if floating in space, as seen in histiocytosis-X
• Langerhans cell histiocytosis Also known as Histiocytosis-X A disease caused by clonal proliferation
of Langerhans cells (dendritic cells that arise from bone marrow and migrate to the skin and lymphoid
tissue). Typically occurs in children. It can cause lytic bone lesions, skin rash, fever,
hepatosplenomegaly, and lymphadenopathy. Secondary anemia may develop due to bone
marrow involvement. Affected individuals typically have normal serum calcium levels.
• Also known as Histiocytosis-X
• A disease caused by clonal proliferation of Langerhans cells (dendritic cells that arise from bone
marrow and migrate to the skin and lymphoid tissue). Typically occurs in children.
• It can cause lytic bone lesions, skin rash, fever, hepatosplenomegaly, and lymphadenopathy.
• Secondary anemia may develop due to bone marrow involvement.
• Affected individuals typically have normal serum calcium levels.
• Also known as Histiocytosis-X
• A disease caused by clonal proliferation of Langerhans cells (dendritic cells that arise from bone
marrow and migrate to the skin and lymphoid tissue). Typically occurs in children.
• It can cause lytic bone lesions, skin rash, fever, hepatosplenomegaly, and lymphadenopathy.
• Secondary anemia may develop due to bone marrow involvement.
• Affected individuals typically have normal serum calcium levels.

Other Options
Option A: Fibromuscular dysplasia
• Fibromuscular dysplasia (FMD), a disease that primarily affects young to middle-aged women

Page 27

1637
• It is characterized by the proliferation of connective tissues and muscle fibers within the arterial
vessel walls.
• The renal, internal carotid, and vertebral arteries are predominantly involved Bruits at the
costovertebral angle and the carotid region are characteristic renal and carotid artery involvement
findings, respectively.
• Bruits at the costovertebral angle and the carotid region are characteristic renal and carotid
artery involvement findings, respectively.
• In rare cases, patients may present with mesenteric ischemia and peripheral artery disease due to
splanchnic or peripheral arterial involvement.
• The "string of beads" sign, a characteristic finding on angiography, distinguishes FMD from other
causes of arterial occlusion.
• A floating tooth sign is not a radiographic finding in this disease.
• Bruits at the costovertebral angle and the carotid region are characteristic renal and carotid
artery involvement findings, respectively.

Option B: Cleidocranial dysplasia


• Cleidocranial dysplasia is an autosomal dominant condition characterized by delayed closure of the
cranial sutures, craniofacial dysmorphism (e.g., frontal bossing, prominent chin, maxillary hypoplasia),
dental abnormalities, and hypoplastic clavicles.
• It is caused by a mutation in a transcription factor involved in osteoblast development.
Option C: Osteopetrosis
• An inherited, diffuse bone disease that results in sclerotic thickening of the skeleton on radiological
examination
• Clinical features Recurring pathological fractures (osteoporotic bone tissue is very dense but brittle)
Cranial nerve disorders (e.g., palsies) due to hyperostosis and stenosis of the cranial nerve foramina
Pancytopenia due to reduced marrow space Hepatosplenomegaly due to extramedullary
hematopoiesis.
• Recurring pathological fractures (osteoporotic bone tissue is very dense but brittle)
• Cranial nerve disorders (e.g., palsies) due to hyperostosis and stenosis of the cranial nerve foramina

Page 28

1638
• Pancytopenia due to reduced marrow space
• Hepatosplenomegaly due to extramedullary hematopoiesis.
• Diagnostics X-ray: symmetrical, homogenous, sclerotic thickening of both cortical and trabecular
bone (stone bone).
• X-ray: symmetrical, homogenous, sclerotic thickening of both cortical and trabecular bone (stone
bone).
• Recurring pathological fractures (osteoporotic bone tissue is very dense but brittle)
• Cranial nerve disorders (e.g., palsies) due to hyperostosis and stenosis of the cranial nerve foramina
• Pancytopenia due to reduced marrow space
• Hepatosplenomegaly due to extramedullary hematopoiesis.
• X-ray: symmetrical, homogenous, sclerotic thickening of both cortical and trabecular bone (stone
bone).

Solution for Question 16:


Option A: Lateral to the second upper molar tooth
• Sialolithiasis is salivary gland stone composed of calcium/ magnesium phosphate/ carbonate
• The parotid duct emerges from the anterior edge of the parotid gland, turns medially at the
anterior edge of the masseter muscle and, after piercing through the buccinator muscle enters the oral
cavity lateral to the second upper molar.
• Sialolithiasis, which affects the parotid gland or duct in approx. 20% of cases typically manifest with
acute pain while eating and tender swelling of the affected gland, both of which are seen here.
• Risk factors include dehydration due to certain medications (e.g., anticholinergics), and trauma.

Option B: Lateral to the lingual frenulum


• The submandibular duct arises from the deep part of the submandibular gland and opens into the oral
cavity lateral to the lingual frenulum.
• Although ∼ 80% of salivary stones are located in the submandibular gland or duct, this patient's
ultrasonography findings suggest that the stone is located in the parotid duct.
Option C: Into the floor of the mouth
• The sublingual gland is drained by numerous ducts, the largest of which joins the submandibular
duct, while the remaining small ducts drain into the floor of the mouth.
• Although salivary stones are sometimes located in the sublingual gland or ducts, this patient's
ultrasonography findings suggest that the stone is located in the parotid duct.
Option D: Into the mandibular foramen
• The mandibular foramen is an opening on the internal surface of the ramus of the mandible.
• Vessels and nerves pass through this opening; however, no salivary gland drains into the mandibular
foramen.

Page 29

1639
Solution for Question 17:
Option C: Human papillomavirus (HPV) infection
• A group of >150 viruses commonly cause skin diseases, including anogenital warts (e.g.,
condylomata acuminata), common warts (verruca vulgaris), and oral warts.
• Also implicated in the pathogenesis of some types of cancer (e.g., cervical cancer and oropharyngeal
cancer).
• HPV infection, particularly HPV 16, 18, 31, and 33, is an important risk factor for developing oral cavity
cancer. However, HPV-positive tumors have a good prognosis since they respond better to chemo- and
radiotherapy.
• Risk factors for the development of oral cavity cancer include: Oral tobacco consumption (e.g., snuff,
pan/betel quid), smoking. Long-term alcohol consumption. Poor oral hygiene, and chronic mechanical
irritation (e.g., badly positioned dentures). Human papillomavirus, particularly HPV 16, 18, 31, and 33.
Presence of precancerous lesions: leukoplakia, erythroplakia, erythroleukoplakia.
• Oral tobacco consumption (e.g., snuff, pan/betel quid), smoking.
• Long-term alcohol consumption.
• Poor oral hygiene, and chronic mechanical irritation (e.g., badly positioned dentures).
• Human papillomavirus, particularly HPV 16, 18, 31, and 33.
• Presence of precancerous lesions: leukoplakia, erythroplakia, erythroleukoplakia.
• Oral tobacco consumption (e.g., snuff, pan/betel quid), smoking.
• Long-term alcohol consumption.
• Poor oral hygiene, and chronic mechanical irritation (e.g., badly positioned dentures).
• Human papillomavirus, particularly HPV 16, 18, 31, and 33.
• Presence of precancerous lesions: leukoplakia, erythroplakia, erythroleukoplakia.
A. Option A: Age
• Age is not a risk factor for the development of oral cavity cancer.
Age is not a risk factor for the development of oral cavity cancer.
B. Option B:High blood cholesterol
• High blood cholesterol is not a risk factor for the development of oral cavity cancer. It leads to the
development of other cardiovascular disorders.
High blood cholesterol is not a risk factor for the development of oral cavity cancer. It leads to the devel
opment of other cardiovascular disorders.
D. Option D: Lack of sleep
• Lack of sleep does not lead to the development of oral cavity cancer. However, it leads to the
development of mood disorders and increased risk of suicide, workplace injuries, and reduced quality
of life.
Lack of sleep does not lead to the development of oral cavity cancer. However, it leads to the develop
ment of mood disorders and increased risk of suicide, workplace injuries, and reduced quality of life.

Page 30

1640
Solution for Question 18:
Option C: Excision
• It is a rare condition (∼ 3% of salivary cysts)
• Most common between 10–30 years of age
• It has been reported in patients aged 3–61 years
• Occurs equally in males and females.
• Clinical features include: Translucent blue swelling below the tongue. It can cause problems with
swallowing and speaking.
• Translucent blue swelling below the tongue.
• It can cause problems with swallowing and speaking.
• Treatment includes The first line of treatment is the Extirpation of the Ranula, including the sublingual
gland. Marsupialization: The surgical procedure involves incising a closed cavity(e.g., cyst) and
suturing its walls to adjacent tissue to convert the closed cavity into an open one.
• The first line of treatment is the Extirpation of the Ranula, including the sublingual gland.
• Marsupialization: The surgical procedure involves incising a closed cavity(e.g., cyst) and suturing its
walls to adjacent tissue to convert the closed cavity into an open one.
• Translucent blue swelling below the tongue.
• It can cause problems with swallowing and speaking.
• The first line of treatment is the Extirpation of the Ranula, including the sublingual gland.
• Marsupialization: The surgical procedure involves incising a closed cavity(e.g., cyst) and suturing its
walls to adjacent tissue to convert the closed cavity into an open one.

Option A: Incision and drainage


• Surgical incision and drainage are recommended in the cases of abscesses to drain all the pus and
fluid filled in the abscess; however, Ranula is not an abscess caused by bacterial infection. It is a
sublingual duct swelling caused by a retention cyst, and cysts cannot be drained. They are removed
surgically.
Option B: Aspiration
• Aspiration can also refer to a medical procedure during which a healthcare professional uses a
suction tube or needle to remove harmful fluid from the lungs, joints, abscesses, or other organs or
tissues.
Option D: Sclerosant injection
• Sclerotherapy is a medical procedure whereby a chemical, the sclerosant, is injected into a vein to
obliterate it.
• The sclerosant damages the innermost lining of the vessel (the endothelium), resulting in a clot that
blocks blood circulation in the vein beyond.

Page 31

1641
• Sclerosant is a medication typically used for the treatment of varicose veins. More recently, it has also
been used in the treatment of tendon problems (tendinosis) and surrounding tendon inflammation,
known as tenosynovitis.

Solution for Question 19:


Option D: 2% toluidine blue
• Toluidine blue is a basic thiazine metachromatic dye with a high affinity for acidic tissue components,
thereby staining tissues rich in DNA and RNA. It has found wide applications both as vital staining in
living tissues and as a special stain owing to its metachromatic property.
• Toluidine blue has been used in vivo to identify dysplasia and carcinoma of the oral cavity.
• The use of toluidine blue in tissue sections is done with the aim of highlighting components, such as
mast cell granules, mucins, and cartilage.
Other options
Option A: 1% zinc chloride
• In low concentrations, it is astringent and mildly antibacterial activity. It is used in vaginal douches to
suppress trichomonas and hemophilus infections.
• Although it is used in mouthwashes, contact time is too short, and only astringent and not antibacterial
action results.
Option B: 2% silver nitrate
• Silver nitrate forms insoluble silver phosphate with phosphate ions; this method is known as the Von
Kossa Stain. When subjected to a reducing agent, usually hydroquinone, it forms black elementary
silver.
• This is used for the study of the formation of calcium phosphate particles during bone growth.
Option C: Gentian violet
• Gentian violet is an antiseptic dye used to treat fungal infections of the skin (such as ringworm and
athlete's foot).
• It also has weak antibacterial effects and may be used on minor cuts and scrapes to prevent infection.

Solution for Question 20:


Option C: Surgery and radiation
• Surgery is the main treatment for stage 3 and 4A maxillary sinus cancer.
• The type of surgery done is a maxillectomy.
• If cancer has spread to the lymph nodes in the neck (called cervical lymph nodes), these lymph nodes
are also removed. This is called a neck dissection. Radiation therapy may be offered before or after
surgery.
• Radiation therapy may be offered before or after surgery. Radiation therapy may be given before
surgery: If it is likely that cancer cells will be in the tissue around the tumor that will be removed If

Page 32

1642
cancer has spread to the lymph nodes that can't be removed by surgery To shrink the tumor and make
it easier to remove through surgery Radiation therapy may be given after surgery to help prevent
cancer from recurrence. It may also be given after surgery if: Cancer cells are in the tissue around the
removed tumor Cancer cells are around or near a nerve Cancer has spread to the lymph nodes in the
neck
• Radiation therapy may be given before surgery: If it is likely that cancer cells will be in the tissue
around the tumor that will be removed If cancer has spread to the lymph nodes that can't be removed
by surgery To shrink the tumor and make it easier to remove through surgery
• If it is likely that cancer cells will be in the tissue around the tumor that will be removed
• If cancer has spread to the lymph nodes that can't be removed by surgery
• To shrink the tumor and make it easier to remove through surgery
• Radiation therapy may be given after surgery to help prevent cancer from recurrence. It may also be
given after surgery if: Cancer cells are in the tissue around the removed tumor Cancer cells are around
or near a nerve Cancer has spread to the lymph nodes in the neck
• Cancer cells are in the tissue around the removed tumor
• Cancer cells are around or near a nerve
• Cancer has spread to the lymph nodes in the neck
• Radiation therapy may be given before surgery: If it is likely that cancer cells will be in the tissue
around the tumor that will be removed If cancer has spread to the lymph nodes that can't be removed
by surgery To shrink the tumor and make it easier to remove through surgery
• If it is likely that cancer cells will be in the tissue around the tumor that will be removed
• If cancer has spread to the lymph nodes that can't be removed by surgery
• To shrink the tumor and make it easier to remove through surgery
• Radiation therapy may be given after surgery to help prevent cancer from recurrence. It may also be
given after surgery if: Cancer cells are in the tissue around the removed tumor Cancer cells are around
or near a nerve Cancer has spread to the lymph nodes in the neck
• Cancer cells are in the tissue around the removed tumor
• Cancer cells are around or near a nerve
• Cancer has spread to the lymph nodes in the neck
• If it is likely that cancer cells will be in the tissue around the tumor that will be removed
• If cancer has spread to the lymph nodes that can't be removed by surgery
• To shrink the tumor and make it easier to remove through surgery
• Cancer cells are in the tissue around the removed tumor
• Cancer cells are around or near a nerve
• Cancer has spread to the lymph nodes in the neck
Option A: Radiation therapy only
• Radiation therapy may be offered before or after surgery.
• In some cases, you may be offered radiation therapy instead of surgery as the main treatment if you
can't have surgery because of other health conditions.
Option B: Chemotherapy only

Page 33

1643
• Chemotherapy is the main treatment for stage 4C maxillary sinus cancer.
• The chemotherapy drug most often used is cisplatin. It is also given to relieve pain or control
symptoms (called palliative chemotherapy). It may be used alone or in combination with other
chemotherapy drugs.
Option D: Chemotherapy and radiation
• Chemotherapy may be used with or without radiation therapy to treat recurrences that are not
controlled by surgery or radiation therapy.
• Chemotherapy may also be used to treat cancer that has come back in other organs or to relieve pain
or control symptoms.

Solution for Question 21:


Option A: Intralesional steroids
• Chronic tobacco chewing, restriction in mouth opening and whitish appearance of the oral cavity are
classically seen in Submucosal fibrosis
• Treatment for SMF: At early stages, stopping the habit and nutritional supplements are done. At
moderate stages, conservative treatment such as intralesional injections and medical treatment is
provided. Intralesional corticosteroid injection is one of the most widely implemented interventions for
OSF, particularly for those patients with palpable fibrous bands. In non-responding patients Surgical
excision with skin, grafting is done.
• At early stages, stopping the habit and nutritional supplements are done. At moderate stages,
conservative treatment such as intralesional injections and medical treatment is provided.
• Intralesional corticosteroid injection is one of the most widely implemented interventions for OSF,
particularly for those patients with palpable fibrous bands.
• In non-responding patients Surgical excision with skin, grafting is done.
• At early stages, stopping the habit and nutritional supplements are done. At moderate stages,
conservative treatment such as intralesional injections and medical treatment is provided.
• Intralesional corticosteroid injection is one of the most widely implemented interventions for OSF,
particularly for those patients with palpable fibrous bands.
• In non-responding patients Surgical excision with skin, grafting is done.
Option B: Surgical excision with skin grafting
• Preferred in advanced stages of oral submucosal fibrosis and in patients not responding to
intralesional steroids.
• Various surgical approaches with full-thickness skin grafts have been used to reconstruct the surgical
defects following the excision of fibrous bands.
Option C: Radiotherapy
• Chemotherapy and radiation therapy may cause changes in the lining of the mouth and the salivary
glands, which make saliva. This can upset the healthy balance of bacteria. These changes may lead to
mouth sores, infections, and tooth decay.
• That can make it very hard to eat and drink. This can lead to weight loss and poor nutrition.
• Radiotherapy is not used for the treatment of oral submucosal fibrosis.

Page 34

1644
Option D: Chemotherapy
• Chemotherapy is indicated in case of malignancies, either localized or metastatic; however oral
submucosal fibrosis is not treated with chemotherapy as it is not a malignant condition.

Solution for Question 22:


Option B: HPV infection
• Oropharyngeal malignancy is India's most common malignant tumour that accounts for 40 % of all
malignancies.
• Oropharyngeal cancer is almost entirely a preventable disease.
• In Asia, the use of pan and tobacco smoking is one of the significant etiological risk factors. In
addition, combined use of tobacco with alcohol increases the risk of oral cancer.
• The risk factors include: Tobacco Alcohol Areca nut/pan masala Sharp or jagged tooth Ill-fitted
dentures Syphilitic glossitis Human papilloma virus (HPV) Epstein-Barr virus (EBV) Plummer-Vinson
syndrome Poor nutrition (Vitamin A deficiency)
• Tobacco
• Alcohol
• Areca nut/pan masala
• Sharp or jagged tooth
• Ill-fitted dentures
• Syphilitic glossitis
• Human papilloma virus (HPV)
• Epstein-Barr virus (EBV)
• Plummer-Vinson syndrome
• Poor nutrition (Vitamin A deficiency)
• Tobacco
• Alcohol
• Areca nut/pan masala
• Sharp or jagged tooth
• Ill-fitted dentures
• Syphilitic glossitis
• Human papilloma virus (HPV)
• Epstein-Barr virus (EBV)
• Plummer-Vinson syndrome
• Poor nutrition (Vitamin A deficiency)

Option A: Vitamin B12 deficiency

Page 35

1645
• Vitamin A deficiency due to poor nutrition is one of the risk factors for the development of cancer of
the oral cavity.
• However, Vitamin B12 is not a risk factor for developing cancer in the oral cavity.
• It is related to megaloblastic anemia and gastric cancers.
• This may cause aphthous ulcers.
Option C: Vitamin C deficiency
• Vitamin C deficiency is associated with a lower incidence of bladder, breast, cervical, endometrial,
esophageal, gastric, lung, and pancreatic cancers.
• It does not lead to an increased incidence of oropharyngeal carcinomas.
Option D: Candida infection
• Candida is associated with leukoplakia of the oral cavity.
• Although unclear, Candida may have a role in the genesis and progression of squamous cell
carcinoma.
• HPV is a stronger risk factor for oral squamous cell carcinoma than Candida.

Solution for Question 23:


Option C: Submandibular nodes
• The submandibular nodes are small, usually measuring approximately 1 centimeter in a healthy adult.
The submandibular duct, which brings lymph fluid to the node, is approximately 5 to 6 centimeters long
in the average adult. The wall of the duct is thin and flexible.
• As the duct runs forward, it passes between the sublingual gland and genioglossus (the primary
muscle of the tongue) to create an opening in the floor of the mouth. As the duct travels through the
deepest part of the submandibular gland, it connects with tributaries draining into the lobe.

Option A: Superior deep cervical nodes

Page 36

1646
• The superior deep cervical lymph nodes lie under the sternocleidomastoid muscle in close relation
with the accessory nerve and the internal jugular vein.
• Some of the glands lie in front of and others behind the vessel. Lymph from these deep nodes passes
to the jugular lymphatic trunk, which joins the thoracic duct on the left side and the brachiocephalic vein
on the right side.
Option B: Jugulodigastric nodes
• The jugulodigastric lymph nodes are found in the proximity of where the posterior belly of the digastric
muscle crosses the internal jugular vein. Nodes are typically around 15 mm in length in adults and
decrease in size during old age. They tend to be some of the largest lymph nodes in the cervical chain
due to their significant lymphatic drainage.
Option D: Subdigastric nodes
• Also known as Jugulodigastric nodes.
• A prominent lymph node in the deep lateral cervical group lies below the digastric muscle and anterior
to the internal jugular vein; it receives lymphatic drainage from the pharynx, palatine tonsil, and tongue.

Solution for Question 24:


Option D: The most common flap used for the reconstruction of head and neck malignancies is the PM
MC flap
• The pectoralis major myocutaneous (PMMC) flap has been used as a versatile and reliable flap since
its first description by Ariyan in 1979. In India, head and neck cancer patients usually present in the
advanced stage, making PMMC flap a viable option for reconstruction.
• The pectoralis major flap comprises the pectoralis major muscle, with or without overlying skin, and
may include the underlying ribs. It has an axial blood supply and is based superiorly on the pectoral
branch of the thoracoacromial artery.
Option A: The most common gene mutated in carcinoma oral cavity is PTEN and can be treated with Li
mberg Flap
• TP53 is the most frequently mutated gene in Oral Squamous Cell Carcinoma tumours.
• The p53 protein is a transcription factor that functions as a suppressor of tumour formation. The TP53
gene is the most commonly mutated in a wide variety of human cancers, and the functions of the
wild-type p53 protein are frequently compromised in many types of cancers.
• The Limberg Flap technique is used to treat pilonidal sinus, not oral squamous cell carcinoma.
Option B:Most common site of oral carcinoma cavity is the lip, which does not require treatment
• Most common site of CA oral cavity tongue > lip
• The most common locations for cancer in the oral cavity are Tongue, Tonsils, and Oropharynx.
• Carcinoma lip needs to be treated with surgery and radiotherapy.
Option C:Bilateral hematogenous spread is common in the lower lip, supraglottis, and soft palate and is
treated by radiotherapy
• Bilateral lymphatic spread is common in the Lower lip, supraglottis, and soft palate.
• Cervical lymph node metastases are common and occur in nearly half of patients who present
with oral cavity SCCs.

Page 37

1647
Solution for Question 25:
Option C: Weber-Ferguson incision

• The above image depicts Weber–Ferguson incision.


• Weber- Ferguson incision is used for tumours involving the maxilla that extends superiorly to the
infraorbital nerve or the orbit.
• It provides broad access to all areas of the maxilla.
• The incision line is drawn through the vermillion border, along the philtrum of the lip, extending around
the base of the nose, and along the facial nasal groove. It then extends to the infraorbital region, 3-4
mm below the cilium, to the lateral canthus.

Option A: Lazy 'S' incision

Page 38

1648
• A lazy 'S' incision is used to access the parotid gland in pleomorphic adenoma. It has no role in
maxillary carcinoma.
Option B: Modified Blair incision
• Modified Blair incision is the most common incision used for parotidectomy.
• It is also known as a lazy S incision.
• It extends below the ear lobe and into a crease in the neck below the jawline.
• It is not used in maxillary cancer.
Option D: Sistrunk incision

• Sistrunk incision is used in the management of thyroglossal cysts. It has no role in maxillary
carcinoma.

Page 39

1649
Solution for Question 26:
Option A: It is most commonly seen in the inner part of the cheek
• Verrucous carcinoma is a low-grade squamous cell carcinoma most common in the oral cavity,
particularly the lower gums and buccal lining. It usually has an indolent growth pattern and is often
associated with chronic snuff or chewing tobacco use.
• common in the southern United States
• It presents as dry, white, and exophytic growth
• No lymphatic or hematogenous spread.
• Surgery is the treatment-wide excision
• No radiotherapy
• It is a curable malignancy

Option B: It is linked to a high metastatic rate


• Often, verrucous carcinoma does not spread beyond the tumor site. Once healthcare providers
remove the tumor, most people recover.
• Thus, it is not linked to having a high metastatic rate.
Option C: Its gross appearance is ulcerative
• Verrucous carcinoma does not have an ulcerative appearance.
• It presents as slow-growing, well-circumscribed, thick, broad-based firm masses with papillary or
warty appearance.
Option D: Its best treatment is radiation
• Standard excision with permanent conventional sections is a highly effective treatment for many
verrucous carcinomas.
• The depth of the excision should include the subcutaneous fat because even small verrucous
carcinomas may extend into the subcutaneous fat.

Page 40

1650
Solution for Question 27:
Option B: Squamous cell carcinoma
• The image in question shows squamous cell carcinoma of the lower lip.
• Squamous cell carcinoma (SCC) of the lip is the most common malignant tumor of the lip.
• Most are present on parts exposed to sunlight, including the ears, cheeks, lower lip, and back of
hands.
• The significant risk factors for developing SCC of the lip include sunlight exposure and tobacco use.
• SCC of the lip usually presents with a raised lesion with whitish discoloration of the lip.
• Symptoms include tingling, numbness, or pain in the lips or the skin around the mouth.
• A biopsy is required to confirm the diagnosis.
• CT, MRI, or PET scans are used to locate spread beyond the lip.
• Treatment includes: Wide local excision for small or non-invasive SCC.
Mohs micrographic surgery for large, aggressive tumors involving vital areas/ cosmetic areas and
recurrent tumors.
• Wide local excision for small or non-invasive SCC.
• Mohs micrographic surgery for large, aggressive tumors involving vital areas/ cosmetic areas and
recurrent tumors.
• Wide local excision for small or non-invasive SCC.
• Mohs micrographic surgery for large, aggressive tumors involving vital areas/ cosmetic areas and
recurrent tumors.

Option A: Basal cell carcinoma


• Basal cell carcinomas present as slow-growing pearly nodules with telangiectatic vessels on the
surface and classical signs of central depression with umbilication.

Page 41

1651
• It affects the upper lip more often than the lower lip.
• Treatment is the same as that of SCC.

Option C: Malignant melanoma


• Malignant melanoma presents more frequently on the upper lip.
• A few important risk factors are UV-light exposure, family history, and pale complexion.
• Treatment includes surgical excision with sentinel lymph node biopsy.
• Malignant melanoma is a radio-resistant tumor.

Option D: Verrucous carcinoma


• Verrucous carcinoma (VC) is a rare subtype of squamous cell carcinoma that commonly involves the
oral cavity.
• It is associated with human papillomavirus infection

Page 42

1652
Solution for Question 28:
Option D: Surgery and radiotherapy
• Surgery and radiotherapy treat the T4N0M0 stage of head and neck carcinoma.
• Therapeutic options for patients with head and neck SCCs (HNSCCs) include surgery, radiation
therapy, chemotherapy, and combination regimens.
• Early-stage disease (stage I or II) generally is treated by surgery or radiation therapy.
• Late-stage disease (stage III or IV) is best treated by a combination of surgery and radiation therapy,
chemotherapy and radiation therapy, or all three modalities, depending on the primary site.
Option A : Surgery alone and Option B : Radiotherapy alone
• Surgery or radiation therapy treats early-stage disease (stage I or II).
Option C: Chemoradiation
• Advanced stages (unresectable tumors) are treated with radiation therapy and chemotherapy.

Page 43

1653
Salivary Glands
1. Which of the following statements is correct regarding stones in the submandibular gland?
A. 10% of stones occur in the submandibular gland
B. The majorityof submandibular stones are radiolucent
C. The most commonly injured nerve during submandibular gland excision is the facial nerve
D. Pain is referred to the tongue due to lingual nerve irritation
----------------------------------------
2. What is the most likely diagnosis in a 32-year-old woman presenting with cough, bilateral parotid
gland enlargement, multiple violaceous tender nodules on bilateral shins, and hypopyon in the anterior
chamber of the eye?
(or)
A 32-year-old woman presented with complaints of a cough and bilateral leg pain for the past 2 months.
She has occasional eye pain. Examination reveals bilateral parotid gland enlargement. There are
multiple violaceous tender nodules on bilateral shins. An eye examination shows hypopyon in the
anterior chamber. Which of the following is the most likely diagnosis?
A. Wegener’s granulomatosis
B. Sjogren’s syndrome
C. Kimura’s disease
D. Sarcoidosis
----------------------------------------
3. Which of the following statements is correct regarding pleomorphic adenoma?
A. It is the most common malignant tumor of the salivary glands.
B. It is the most common malignant tumor of the salivary glands in children.
C. It is the most common neoplasm of the salivary gland in children.
D. It is the most common malignant tumour of the minor salivary glands
E. It is the most common neoplasm of the salivary glands.
----------------------------------------
4. Which of the following is correct regarding benign mixed parotid tumors?
A. Slow-growing and lobular
B. Firm and unencapsulated
C. 10% of the parotid tumors
D. Composed of epithelium and endodermal components
----------------------------------------
5. Which benign parotid gland disease is most likely in a 60-year-old male with left-sided facial swelling,
a history of 40-pack-year smoking, and a marble-sized firm and non-tender mass beneath his left ear?
(or)

1654
A 60-year-old chronic smoker male presented complaints of left-sided facial swelling for the past 4
years. On examination, a marble-sized firm and non-tender mass is noticed beneath his left ear. A
diagnosis is made by biopsy, and the mass is eventually excised. Which of the following is this patient's
most likely benign parotid gland disease?
A. Glandular hypertrophy, secondary to vitamin A deficiency
B. Cystic dilation
C. Mikulicz’s disease
D. Papillary cystadenoma lymphomatosum
----------------------------------------
6. Which statement is correct regarding the Stafne bone cyst?
A. It is a minor salivary gland tissue
B. It is located above the inferior dental neurovascular bundle
C. No treatment is required
D. It increases the risk of malignancy
----------------------------------------
7. A computed tomography (CT) scan reveals an enhancing mass in the deep lobe of the right parotid
gland. A parotid gland tumor is suspected. Which of the following is the most appropriate surgical
technique for treating this tumor?
A. Total parotidectomy with facial nerve preservation
B. Total parotidectomy with facial nerve sacrifice
C. Deep lobe parotidectomy with facial nerve preservation
D. Deep lobe parotidectomy with facial nerve sacrifice
----------------------------------------
8. What is the correct shape of incision for superficial parotidectomy in a tumor localized to the
superficial lobe of the parotid gland?
(or)
A 60-year-old female is admitted with complaints of a slowly progressive, painless, right-sided facial
swelling for 6 months. It is firm in consistency with no skin or facial nerve involvement. Further workup
of the patient revealed that the tumor is localized to the superficial parotid gland. Which of the following
is the correct shape of incision for a superficial parotidectomy?
A. L-shaped
B. Y-shaped
C. S-shaped
D. Z-shaped
----------------------------------------
9. Which of the following is the most common site for a minor salivary gland tumor?
A. Cheeks
B. Palate

Page 2

1655
C. Sublingual glands
D. Tongue
----------------------------------------
10. Which nerve is typically involved in a patient experiencing sweating, flushing, and a burning
sensation on the right cheek after eating certain foods, with localized hyperhidrosis over the right
zygomatic arch?
A. Facial nerve
B. Trigeminal nerve
C. Auriculotemporal nerve
D. Glossopharyngeal nerve
----------------------------------------
11. Which of the following is an appropriate management strategy for Frey's syndrome?
A. Botulinum toxin
B. Temporalis fascia flap
C. Sternocleidomastoid muscle flap
D. Superficial parotidectomy
----------------------------------------
12. Which of the following factor increased the risk of mucoepidermoid carcinoma?
A. Radiotherapy to head and neck
B. Exposure to silica dust
C. EBV infection
D. Smoking
----------------------------------------
13. Which of the following is true about mucoepidermoid carcinoma?
A. The low-grade subtype has predominantly squamous cells.
B. It is the most common malignant salivary gland tumor in children
C. It usually causes facial paralysis
D. It is well-differentiated in adults
----------------------------------------
14. Which of the following is the most common site of origin of acinic cell carcinoma?
A. Parotid gland
B. Minor salivary glands
C. Submandibular glands
D. Sublingual gland
----------------------------------------
15. Which of the following statements is accurate regarding adenoid cystic carcinoma?

Page 3

1656
A. Most common malignant tumor of minor salivary glands
B. Most common site is the sinonasal tract
C. The most commonly involved nerve is trigeminalnerve
D. Most common site of metastasis is the liver
----------------------------------------
16. A 25-year-old male with a history of knife laceration to the left side of his face in front of the ear one
month ago complained of clear, watery discharge from the wound in spite of suturing. Laboratory
analysis of the fluid shows increased amylase levels. Which is the most appropriate treatment for this
patient's condition?
(or)
What is the most appropriate treatment for a 25-year-old male with a small opening above the angle of
the left mandible, continuous dribbling of clear serous fluid, and increased amylase levels in the fluid
analysis?
A. External beam radiotherapy
B. Newman and Seabrook’s operation
C. Intravenous antibiotics
D. Radical neck dissection
----------------------------------------
17. A 24-year-old female presented to the clinic with complaints of a submandibular mass for the past 2
months. The mass is soft and nontender. Examination of the oral cavity shows a unilateral translucent
swelling in the floor of the mouth, lateral to the midline. Which of the following is true regarding this
patient’s diagnosis?
(or)
What is true regarding the diagnosis of a 24-year-old female with a submandibular mass, and a
unilateral translucent swelling in the floor of the mouth, lateral to the midline?
A. It is a mucus extravasation cyst arising from the sublingual gland
B. It is treated by chemotherapy
C. It is the most common type of minor salivary gland tumor
D. It is diagnosed by biopsy
----------------------------------------
18. A 48-year-old male chronic smoker presented with a swelling on the right side of the face for the
past one and a half years. The onset was insidious, and the swelling gradually increased in size. The
skin over the swelling is normal. Examination showed a firm right preauricular mass. A mass biopsy
showed numerous papillations covered by monomorphic oncotic cells and germinal centers. Which of
the following is the most likely diagnosis?
(or)
What is the most likely diagnosis for a patient with a firm preauricular mass showing numerous
papillations covered by monomorphic oncotic cells and germinal centers on biopsy?
A. Pleomorphic adenoma
B. Adenoid cystic carcinoma

Page 4

1657
C. Warthin's tumor
D. Mucoepidermoid carcinoma
----------------------------------------

Correct Answers
Question Correct Answer

Question 1 4
Question 2 4
Question 3 5
Question 4 1
Question 5 4
Question 6 3
Question 7 3
Question 8 3
Question 9 2
Question 10 3
Question 11 1
Question 12 1
Question 13 2
Question 14 1
Question 15 1
Question 16 2
Question 17 1
Question 18 3

Solution for Question 1:


Option D: Pain is referred to the tongue due to lingual nerve irritation
• Sialolithiasis refers to the stones in the salivary gland. Calculi may form in the ducts or parenchyma of
submandibular or parotid glands. These calculi are formed by depositing calcium phosphate on the
organic mucin matrix or cellular debris. Magnesium phosphate or carbonate stones are also seen.
About 10% of stones are found in the parotid gland, and 80% in the submandibular gland. Around 7%
of salivary gland stones are seen in the sublingual glands, and the remaining 3% are in the minor
salivary glands.
• These calculi are formed by depositing calcium phosphate on the organic mucin matrix or cellular
debris. Magnesium phosphate or carbonate stones are also seen.
• About 10% of stones are found in the parotid gland, and 80% in the submandibular gland. Around 7%
of salivary gland stones are seen in the sublingual glands, and the remaining 3% are in the minor
salivary glands.

Page 5

1658
• Stones in the submandibular gland are the most common. These stones can cause partial or
complete obstruction of the submandibular duct. The patient presents with intermittent swelling of the
involved gland and intermittent pain due to obstruction or outflow of saliva. Pain is precipitated by
eating and resolves 1-2 hours after a meal, which is also called 'meal time' syndrome. Pain may be
referred to the tongue due to the impacted stone's irritation of the lingual nerve The clinical examination
may reveal an enlarged, firm submandibular gland that is tender on bimanual examination. Sometimes,
the stone is visible at the site of the duct opening or can be palpated. About 80% of stones are
radiopaque and can be seen on X-rays or CT scans. Non-contrast CT scan is the investigation of
choice for sialolithiasis. Sialography is required for radiolucent stones. Ultrasonography can also be
done. Stones in the peripheral part of the gland can be removed intraorally. Stones inside the
parenchyma may require surgical excision of the gland. An incision removes the stones in the duct to
the duct. Stone removal can be done by Dormia baskets or lithotripsy. The lingual nerve is the most
commonly injured nerve during the excision of the submandibular gland. The hypoglossal nerve is less
commonly injured.
• The patient presents with intermittent swelling of the involved gland and intermittent pain due to
obstruction or outflow of saliva. Pain is precipitated by eating and resolves 1-2 hours after a meal,
which is also called 'meal time' syndrome.
• Pain may be referred to the tongue due to the impacted stone's irritation of the lingual nerve
• The clinical examination may reveal an enlarged, firm submandibular gland that is tender on bimanual
examination.
• Sometimes, the stone is visible at the site of the duct opening or can be palpated.
• About 80% of stones are radiopaque and can be seen on X-rays or CT scans. Non-contrast CT scan
is the investigation of choice for sialolithiasis.
• Sialography is required for radiolucent stones. Ultrasonography can also be done.
• Stones in the peripheral part of the gland can be removed intraorally. Stones inside the parenchyma
may require surgical excision of the gland. An incision removes the stones in the duct to the duct. Stone
removal can be done by Dormia baskets or lithotripsy.
• The lingual nerve is the most commonly injured nerve during the excision of the submandibular gland.
The hypoglossal nerve is less commonly injured.
• These calculi are formed by depositing calcium phosphate on the organic mucin matrix or cellular
debris. Magnesium phosphate or carbonate stones are also seen.
• About 10% of stones are found in the parotid gland, and 80% in the submandibular gland. Around 7%
of salivary gland stones are seen in the sublingual glands, and the remaining 3% are in the minor
salivary glands.
• The patient presents with intermittent swelling of the involved gland and intermittent pain due to
obstruction or outflow of saliva. Pain is precipitated by eating and resolves 1-2 hours after a meal,
which is also called 'meal time' syndrome.
• Pain may be referred to the tongue due to the impacted stone's irritation of the lingual nerve
• The clinical examination may reveal an enlarged, firm submandibular gland that is tender on bimanual
examination.
• Sometimes, the stone is visible at the site of the duct opening or can be palpated.
• About 80% of stones are radiopaque and can be seen on X-rays or CT scans. Non-contrast CT scan
is the investigation of choice for sialolithiasis.
• Sialography is required for radiolucent stones. Ultrasonography can also be done.

Page 6

1659
• Stones in the peripheral part of the gland can be removed intraorally. Stones inside the parenchyma
may require surgical excision of the gland. An incision removes the stones in the duct to the duct. Stone
removal can be done by Dormia baskets or lithotripsy.
• The lingual nerve is the most commonly injured nerve during the excision of the submandibular gland.
The hypoglossal nerve is less commonly injured.
Option A: 10% of stones occur in the submandibular gland
• About 80% of the stones occur in the submandibular gland. Ten percent of stones occur in the parotid
gland, 7% in the sublingual, and 3% in minor salivary glands.
Option B: The majority of submandibular stones are radiolucent
• Around 80% of stones of the salivary glands are radiopaque and can be seen on X-rays or CT scans.
Option C: The most common injured nerve during submandibular gland excision is the facial nerve
• The lingual nerve is the most commonly injured nerve during submandibular gland excision.
• The facial nerve can be injured during parotidectomy.

Solution for Question 2:


Option D: Sarcoidosis
• This woman presenting with a cough, bilateral parotid enlargement, and violaceous tender nodules
(erythema nodosum) is most consistent with a diagnosis of sarcoidosis.
• It is an autoimmune disorder that commonly presents in middle-aged African-American women.
• It can also present with eye pain due to anterior uveitis (hypopyon in the eye's anterior chamber).
• Sarcoidosis causes non-caseating granulomas in the lungs, along with bilateral hilar
lymphadenopathy that produces a chronic cough.
• Other features of sarcoidosis include Bell's palsy, lupus pernio (skin lesions on the face resembling
lupus), interstitial fibrosis, arthropathy, and bilateral parotid gland enlargement.
• There are many causes of bilateral parotid gland enlargement, including viral infections like mumps,
EBV, CMV, and HIV. Metabolic causes include Diabetes mellitus, chronic pancreatitis, and hepatic
cirrhosis.
• Endocrine causes include acromegaly and hypogonadism. Other important causes include
amyloidosis, Wegener’s granulomatosis, Kimura’s disease, and Sjogren’s syndrome.
Option A: Wegener’s granulomatosis
• Wegener’s granulomatosis can present with a cough and bilateral parotid enlargement. However,
sinusitis and kidney dysfunction are also seen. Hemoptysis is common. It does not present with eye
pain or erythema nodosum.
Option B: Sjogren’s syndrome
• Sjogren’s syndrome can present with bilateral parotid enlargement. It can also present with eye pain
due to keratoconjunctivitis sicca. The examination does not show signs of anterior uveitis.
Option C: Kimura’s disease
• Kimura's disease is a chronic inflammatory condition with bilateral parotid enlargement. However,
uveitis and erythema nodosum is not seen.

Page 7

1660
Solution for Question 3:
Option E: It is the most common neoplasm of the salivary glands.
• A benign mixed parotid tumor (pleomorphic adenoma) is a slow-growing, lobular, firm, and
encapsulated tumor. It comprises approximately 50% of parotid tumors. It is a benign salivary gland
tumor and the most common tumor of major salivary glands. It is also known as a mixed tumor with
epithelial and mesenchymal components. The mesodermal portion arises from the myoepithelial cells,
contractile tissue surrounding the tubules. Clinical features include a slow-growing swelling at the angle
of the mandible for many years. It usually does not involve the facial nerve but can cause facial
asymmetry and eversion of the ear lobule. Swelling can not be moved over the zygomatic bone as the
deep fascia is attached above the zygomatic bone and does not allow the swelling to be mobile. A
long-standing pleomorphic adenoma may convert into malignancy. The most common site of a
pleomorphic adenoma is the tail of the parotid gland, including the superficial lobe. Pleomorphic
adenoma is unicentric, but its recurrences are multicentric. Diagnosis is based on the fine-needle
aspiration of the tumor. It is very helpful in diagnosing benign from malignant. Treatment involves
Patey’s operation or superficial parotidectomy. In superficial parotidectomy, the part that is superficial to
the facial nerve is removed. Enucleation is not done due to recurrence.
• It comprises approximately 50% of parotid tumors.
• It is a benign salivary gland tumor and the most common tumor of major salivary glands.
• It is also known as a mixed tumor with epithelial and mesenchymal components. The mesodermal
portion arises from the myoepithelial cells, contractile tissue surrounding the tubules.
• Clinical features include a slow-growing swelling at the angle of the mandible for many years. It
usually does not involve the facial nerve but can cause facial asymmetry and eversion of the ear lobule.
Swelling can not be moved over the zygomatic bone as the deep fascia is attached above the
zygomatic bone and does not allow the swelling to be mobile. A long-standing pleomorphic adenoma
may convert into malignancy.
• It usually does not involve the facial nerve but can cause facial asymmetry and eversion of the ear
lobule.
• Swelling can not be moved over the zygomatic bone as the deep fascia is attached above the
zygomatic bone and does not allow the swelling to be mobile.
• A long-standing pleomorphic adenoma may convert into malignancy.
• The most common site of a pleomorphic adenoma is the tail of the parotid gland, including the
superficial lobe. Pleomorphic adenoma is unicentric, but its recurrences are multicentric.
• Diagnosis is based on the fine-needle aspiration of the tumor. It is very helpful in diagnosing benign
from malignant.
• Treatment involves Patey’s operation or superficial parotidectomy. In superficial parotidectomy, the
part that is superficial to the facial nerve is removed. Enucleation is not done due to recurrence.
• It comprises approximately 50% of parotid tumors.
• It is a benign salivary gland tumor and the most common tumor of major salivary glands.
• It is also known as a mixed tumor with epithelial and mesenchymal components. The mesodermal
portion arises from the myoepithelial cells, contractile tissue surrounding the tubules.

Page 8

1661
• Clinical features include a slow-growing swelling at the angle of the mandible for many years. It
usually does not involve the facial nerve but can cause facial asymmetry and eversion of the ear lobule.
Swelling can not be moved over the zygomatic bone as the deep fascia is attached above the
zygomatic bone and does not allow the swelling to be mobile. A long-standing pleomorphic adenoma
may convert into malignancy.
• It usually does not involve the facial nerve but can cause facial asymmetry and eversion of the ear
lobule.
• Swelling can not be moved over the zygomatic bone as the deep fascia is attached above the
zygomatic bone and does not allow the swelling to be mobile.
• A long-standing pleomorphic adenoma may convert into malignancy.
• The most common site of a pleomorphic adenoma is the tail of the parotid gland, including the
superficial lobe. Pleomorphic adenoma is unicentric, but its recurrences are multicentric.
• Diagnosis is based on the fine-needle aspiration of the tumor. It is very helpful in diagnosing benign
from malignant.
• Treatment involves Patey’s operation or superficial parotidectomy. In superficial parotidectomy, the
part that is superficial to the facial nerve is removed. Enucleation is not done due to recurrence.
• It usually does not involve the facial nerve but can cause facial asymmetry and eversion of the ear
lobule.
• Swelling can not be moved over the zygomatic bone as the deep fascia is attached above the
zygomatic bone and does not allow the swelling to be mobile.
• A long-standing pleomorphic adenoma may convert into malignancy.
Option A: It is the most common malignant tumor of the salivary glands
• Mucoepidermoid carcinoma is the most common malignant tumor of the salivary glands, not
pleomorphic adenoma.
Option B: It is the most common malignant tumor of the salivary glands in children.
• Mucoepidermoid carcinoma is the most common malignant tumor of the salivary glands in children.
Option C: It is the most common neoplasm of the salivary gland in children
• Hemangioma is the most common neoplasm of the salivary gland in children.
Option D: It is the most common malignant tumor of the minor salivary glands.
• Adenoid cystic carcinoma is the most common malignant tumor of the minor salivary glands.

Solution for Question 4:


Option A: Slow-growing and lobular
• The parotid gland is one of 3 major salivary glands, the other 2 being sublingual and submandibular
glands. It is closely related to the ear canal posteriorly, zygoma superiorly, and sternocleidomastoid
inferiorly. The facial nerve crosses the parotid gland dividing the gland into superficial and deep parts
• A benign mixed parotid tumor (pleomorphic adenoma) is a slow-growing, lobular, firm, and
encapsulated tumor. It comprises approximately 50% of parotid tumors. It is a benign salivary gland

Page 9

1662
tumor and the most common tumor of major salivary glands. It is also known as a mixed tumor as it has
both epithelial and mesenchymal components. The mesodermal portion arises from the myoepithelial
cells, which are contractive tissues surrounding the tubules. Clinical features include a slow-growing
swelling at the angle of the mandible for many years. It usually does not involve the facial nerve but can
cause facial asymmetry and eversion of the ear lobule. Swelling can not be moved over the zygomatic
bone as the deep fascia is attached above the zygomatic bone and does not allow the swelling to
move over the bone. A long-standing pleomorphic adenoma may convert into malignancy. The most
common site of a pleomorphic adenoma is the tail of the parotid gland, including the superficial lobe.
Pleomorphic adenoma is unicentric, but its recurrences are multicentric. It is encapsulated but sends
pseudopodia (finger-like projections) into surrounding glands. Diagnosis is based on the fine-needle
aspiration of the tumor. It is very helpful in diagnosing benign from malignant. Other investigations that
can be done are Parotid gland ultrasound. Sialography. Trucut biopsy. Bone scan. CT scan and MRI to
look for the extent of the tumor. Treatment involves Patey’s operation or superficial parotidectomy. In
superficial parotidectomy, the part that is superficial to the facial nerve is removed. Enucleation is not
done due to recurrence.
• It comprises approximately 50% of parotid tumors.
• It is a benign salivary gland tumor and the most common tumor of major salivary glands.
• It is also known as a mixed tumor as it has both epithelial and mesenchymal components. The
mesodermal portion arises from the myoepithelial cells, which are contractive tissues surrounding the
tubules.
• Clinical features include a slow-growing swelling at the angle of the mandible for many years. It
usually does not involve the facial nerve but can cause facial asymmetry and eversion of the ear lobule.
Swelling can not be moved over the zygomatic bone as the deep fascia is attached above the
zygomatic bone and does not allow the swelling to move over the bone. A long-standing pleomorphic
adenoma may convert into malignancy.
• It usually does not involve the facial nerve but can cause facial asymmetry and eversion of the ear
lobule.
• Swelling can not be moved over the zygomatic bone as the deep fascia is attached above the
zygomatic bone and does not allow the swelling to move over the bone.
• A long-standing pleomorphic adenoma may convert into malignancy.
• The most common site of a pleomorphic adenoma is the tail of the parotid gland, including the
superficial lobe. Pleomorphic adenoma is unicentric, but its recurrences are multicentric.
• It is encapsulated but sends pseudopodia (finger-like projections) into surrounding glands.
• Diagnosis is based on the fine-needle aspiration of the tumor.

• It is very helpful in diagnosing benign from malignant. Other investigations that can be done are
Parotid gland ultrasound. Sialography. Trucut biopsy. Bone scan. CT scan and MRI to look for the
extent of the tumor.
• Parotid gland ultrasound.
• Sialography.
• Trucut biopsy.
• Bone scan.
• CT scan and MRI to look for the extent of the tumor.

Page 10

1663
• Treatment involves Patey’s operation or superficial parotidectomy. In superficial parotidectomy, the
part that is superficial to the facial nerve is removed. Enucleation is not done due to recurrence.
• It comprises approximately 50% of parotid tumors.
• It is a benign salivary gland tumor and the most common tumor of major salivary glands.
• It is also known as a mixed tumor as it has both epithelial and mesenchymal components. The
mesodermal portion arises from the myoepithelial cells, which are contractive tissues surrounding the
tubules.
• Clinical features include a slow-growing swelling at the angle of the mandible for many years. It
usually does not involve the facial nerve but can cause facial asymmetry and eversion of the ear lobule.
Swelling can not be moved over the zygomatic bone as the deep fascia is attached above the
zygomatic bone and does not allow the swelling to move over the bone. A long-standing pleomorphic
adenoma may convert into malignancy.
• It usually does not involve the facial nerve but can cause facial asymmetry and eversion of the ear
lobule.
• Swelling can not be moved over the zygomatic bone as the deep fascia is attached above the
zygomatic bone and does not allow the swelling to move over the bone.
• A long-standing pleomorphic adenoma may convert into malignancy.
• The most common site of a pleomorphic adenoma is the tail of the parotid gland, including the
superficial lobe. Pleomorphic adenoma is unicentric, but its recurrences are multicentric.
• It is encapsulated but sends pseudopodia (finger-like projections) into surrounding glands.
• Diagnosis is based on the fine-needle aspiration of the tumor.

• It is very helpful in diagnosing benign from malignant. Other investigations that can be done are
Parotid gland ultrasound. Sialography. Trucut biopsy. Bone scan. CT scan and MRI to look for the
extent of the tumor.
• Parotid gland ultrasound.
• Sialography.
• Trucut biopsy.
• Bone scan.
• CT scan and MRI to look for the extent of the tumor.
• Treatment involves Patey’s operation or superficial parotidectomy. In superficial parotidectomy, the
part that is superficial to the facial nerve is removed. Enucleation is not done due to recurrence.
• It usually does not involve the facial nerve but can cause facial asymmetry and eversion of the ear
lobule.
• Swelling can not be moved over the zygomatic bone as the deep fascia is attached above the
zygomatic bone and does not allow the swelling to move over the bone.
• A long-standing pleomorphic adenoma may convert into malignancy.

Page 11

1664
• Parotid gland ultrasound.
• Sialography.
• Trucut biopsy.
• Bone scan.
• CT scan and MRI to look for the extent of the tumor.
Option B: Firm and unencapsulated
• Pleomorphic adenomas are firm in consistency and are encapsulated, not unencapsulated.
Option C: 10% of the parotid tumor
• Pleomorphic adenomas comprise about 50% of parotid gland tumors.
Option D: Composed of epithelium and endodermal components
• The benign mixed parotid tumor has chondromyxoid stroma with epithelial and mesodermal
components.

Solution for Question 5:


Option D: Papillary cystadenoma lymphomatosum
• The above clinical scenario points towards the diagnosis of papillary cystadenoma lymphomatosum,
also known as Warthin's tumor or adenolymphoma.
• They are commonly seen between the fifth and seventh decades of life with male predominance.
• Histologically, it is composed of an oncotic epithelium and a lymphoid stroma.
• It consists of an encapsulated mass containing numerous irregular cystic spaces lined by bilayered
oncotic epithelium growing in the cystic spaces as papillary projections.
• The underlying stroma separating the oncocytic epithelium is typically a large lymphoid matrix with
germinal centers.

Page 12

1665
• Warthin's tumor and oncocytoma are the two tumor that show a 'hot spot' on the Tc-99m
pertechnetate scan. This is because of the high mitochondrial content within the oncocytes.
• Treatment is superficial parotidectomy, although they can be enucleated without danger of
recurrence.
Option A: Glandular hypertrophy, secondary to vitamin A deficiency
• Parotid gland hypertrophy can occur during malnutrition due to vitamin A deficiency. It is a reversible
condition that returns to normal after adequate nutrient intake. There is no history of malnutrition in this
patient.
Option B: Cystic dilation
• Sialectasis is the cystic dilatation of the parotid duct. It is associated with ascending infections and
gland destruction.
Option C: Mikulicz’s disease
• It is an autoimmune disorder of bilateral salivary and lacrimal glands, resulting in infiltration of the
glands with round cells.

Solution for Question 6:


Option C: No treatment is required

• Stafne bone cyst is the most common aberrant or ectopic salivary gland tissue.
• The origin of this ectopic salivary tissue is uncertain. However, it is thought to be caused by an ectopic
portion of the submandibular salivary gland, which causes the bone of the lingual cortical plate to
remodel.
• According to another theory, the cavity in the lingual plate houses part of the submandibular gland or
ectopic salivary tissue. Still, the origin of the cavity in this theory has been attributed to the pulse
pressure from the facial artery.
• It is usually asymptomatic and found incidentally during routine dental radiography.

Page 13

1666
• It is seen as a demarcated radiolucent lesion at the angle of the mandible, as shown in the radiograph
above (pointed by the white arrow). It is present below the inferior dental neurovascular bundle.
• As this condition is benign and asymptomatic, no treatment is required for a Stafne bone cyst.
Option A: It is a minor salivary gland tissue
• Stafne bone cyst is an aberrant or ectopic salivary gland tissue, the origin of which is uncertain. It
does not arise from a minor salivary gland.
Option B: It is located above the inferior dental neurovascular bundle
• It is located below (not above) the inferior dental neurovascular bundle.
Option D: It increases the risk of malignancy
• It is mostly asymptomatic and does not increase the risk of malignancy.

Solution for Question 7:


Option C: Deep lobe parotidectomy with facial nerve preservation.
• Features of malignancy in a parotid gland tumor involve an increase in size, skin involvement, cervical
lymphadenopathy or facial nerve palsy.
• Treatment of salivary gland tumors involves: Surgical excision alone or in combination with
radiotherapy. For parotid gland tumors, depending on the involvement of the parotid lobe or the facial
nerve, the following surgeries can be done: Superficial parotidectomy is done for benign and malignant
tumors involving the superficial lobe of the parotid. Salivary tissue lateral to the facial nerve is removed.
Deep lobe parotidectomy is done for benign and malignant tumors involving the deep lobe of the
parotid gland. Glandular tissue that is deep to the facial nerve is removed. Total parotidectomy is done
for the benign and malignant tumors of the parotid gland that involve the whole gland. The whole of the
glandular tissue is removed and the facial nerve is preserved. Radical parotidectomy is performed
when both lobes of the gland and facial nerve are involved by the tumors. All salivary tissue, along with
the facial nerve, is removed. In extended radical parotidectomy, the whole of the salivary gland, facial
nerve, and the adjacent structures (mandible, temporal bone, muscles) are removed. Other salivary
gland tumors can be removed by en-bloc excision of the tumors and the involved gland. Neck
dissection is performed in high-grade malignancies and when clinically palpable nodes are present.
Radiotherapy can be done in: High-grade tumors. Large primary tumors. Perineural invasion. Invasion
of lymph nodes. Positive margins. Cervical lymph node metastasis. Chemotherapy has no role in
salivary gland tumors due to incomplete and short-lived response of chemotherapy. It also has no
survival benefits.
• Surgical excision alone or in combination with radiotherapy. For parotid gland tumors, depending on
the involvement of the parotid lobe or the facial nerve, the following surgeries can be done: Superficial
parotidectomy is done for benign and malignant tumors involving the superficial lobe of the parotid.
Salivary tissue lateral to the facial nerve is removed. Deep lobe parotidectomy is done for benign and
malignant tumors involving the deep lobe of the parotid gland. Glandular tissue that is deep to the facial
nerve is removed. Total parotidectomy is done for the benign and malignant tumors of the parotid gland
that involve the whole gland. The whole of the glandular tissue is removed and the facial nerve is
preserved. Radical parotidectomy is performed when both lobes of the gland and facial nerve are
involved by the tumors. All salivary tissue, along with the facial nerve, is removed. In extended radical
parotidectomy, the whole of the salivary gland, facial nerve, and the adjacent structures (mandible,
temporal bone, muscles) are removed. Other salivary gland tumors can be removed by en-bloc

Page 14

1667
excision of the tumors and the involved gland.
• For parotid gland tumors, depending on the involvement of the parotid lobe or the facial nerve, the
following surgeries can be done: Superficial parotidectomy is done for benign and malignant
tumors involving the superficial lobe of the parotid. Salivary tissue lateral to the facial nerve is removed.
Deep lobe parotidectomy is done for benign and malignant tumors involving the deep lobe of the
parotid gland. Glandular tissue that is deep to the facial nerve is removed. Total parotidectomy is done
for the benign and malignant tumors of the parotid gland that involve the whole gland. The whole of the
glandular tissue is removed and the facial nerve is preserved. Radical parotidectomy is performed
when both lobes of the gland and facial nerve are involved by the tumors. All salivary tissue, along with
the facial nerve, is removed. In extended radical parotidectomy, the whole of the salivary gland, facial
nerve, and the adjacent structures (mandible, temporal bone, muscles) are removed.
• Superficial parotidectomy is done for benign and malignant tumors involving the superficial lobe of the
parotid. Salivary tissue lateral to the facial nerve is removed.
• Deep lobe parotidectomy is done for benign and malignant tumors involving the deep lobe of the
parotid gland. Glandular tissue that is deep to the facial nerve is removed.
• Total parotidectomy is done for the benign and malignant tumors of the parotid gland that involve the
whole gland. The whole of the glandular tissue is removed and the facial nerve is preserved.
• Radical parotidectomy is performed when both lobes of the gland and facial nerve are involved by the
tumors. All salivary tissue, along with the facial nerve, is removed.
• In extended radical parotidectomy, the whole of the salivary gland, facial nerve, and the adjacent
structures (mandible, temporal bone, muscles) are removed.
• Other salivary gland tumors can be removed by en-bloc excision of the tumors and the involved gland.
• Neck dissection is performed in high-grade malignancies and when clinically palpable nodes are
present.
• Radiotherapy can be done in: High-grade tumors. Large primary tumors. Perineural invasion. Invasion
of lymph nodes. Positive margins. Cervical lymph node metastasis.
• High-grade tumors.
• Large primary tumors.
• Perineural invasion.
• Invasion of lymph nodes.
• Positive margins.
• Cervical lymph node metastasis.
• Chemotherapy has no role in salivary gland tumors due to incomplete and short-lived response of
chemotherapy. It also has no survival benefits.
• Surgical excision alone or in combination with radiotherapy. For parotid gland tumors, depending on
the involvement of the parotid lobe or the facial nerve, the following surgeries can be done: Superficial
parotidectomy is done for benign and malignant tumors involving the superficial lobe of the parotid.
Salivary tissue lateral to the facial nerve is removed. Deep lobe parotidectomy is done for benign and
malignant tumors involving the deep lobe of the parotid gland. Glandular tissue that is deep to the facial
nerve is removed. Total parotidectomy is done for the benign and malignant tumors of the parotid gland
that involve the whole gland. The whole of the glandular tissue is removed and the facial nerve is
preserved. Radical parotidectomy is performed when both lobes of the gland and facial nerve are
involved by the tumors. All salivary tissue, along with the facial nerve, is removed. In extended radical
parotidectomy, the whole of the salivary gland, facial nerve, and the adjacent structures (mandible,

Page 15

1668
temporal bone, muscles) are removed. Other salivary gland tumors can be removed by en-bloc
excision of the tumors and the involved gland.
• For parotid gland tumors, depending on the involvement of the parotid lobe or the facial nerve, the
following surgeries can be done: Superficial parotidectomy is done for benign and malignant
tumors involving the superficial lobe of the parotid. Salivary tissue lateral to the facial nerve is removed.
Deep lobe parotidectomy is done for benign and malignant tumors involving the deep lobe of the
parotid gland. Glandular tissue that is deep to the facial nerve is removed. Total parotidectomy is done
for the benign and malignant tumors of the parotid gland that involve the whole gland. The whole of the
glandular tissue is removed and the facial nerve is preserved. Radical parotidectomy is performed
when both lobes of the gland and facial nerve are involved by the tumors. All salivary tissue, along with
the facial nerve, is removed. In extended radical parotidectomy, the whole of the salivary gland, facial
nerve, and the adjacent structures (mandible, temporal bone, muscles) are removed.
• Superficial parotidectomy is done for benign and malignant tumors involving the superficial lobe of the
parotid. Salivary tissue lateral to the facial nerve is removed.
• Deep lobe parotidectomy is done for benign and malignant tumors involving the deep lobe of the
parotid gland. Glandular tissue that is deep to the facial nerve is removed.
• Total parotidectomy is done for the benign and malignant tumors of the parotid gland that involve the
whole gland. The whole of the glandular tissue is removed and the facial nerve is preserved.
• Radical parotidectomy is performed when both lobes of the gland and facial nerve are involved by the
tumors. All salivary tissue, along with the facial nerve, is removed.
• In extended radical parotidectomy, the whole of the salivary gland, facial nerve, and the adjacent
structures (mandible, temporal bone, muscles) are removed.
• Other salivary gland tumors can be removed by en-bloc excision of the tumors and the involved gland.
• Neck dissection is performed in high-grade malignancies and when clinically palpable nodes are
present.
• Radiotherapy can be done in: High-grade tumors. Large primary tumors. Perineural invasion. Invasion
of lymph nodes. Positive margins. Cervical lymph node metastasis.
• High-grade tumors.
• Large primary tumors.
• Perineural invasion.
• Invasion of lymph nodes.
• Positive margins.
• Cervical lymph node metastasis.
• Chemotherapy has no role in salivary gland tumors due to incomplete and short-lived response of
chemotherapy. It also has no survival benefits.
• For parotid gland tumors, depending on the involvement of the parotid lobe or the facial nerve, the
following surgeries can be done: Superficial parotidectomy is done for benign and malignant
tumors involving the superficial lobe of the parotid. Salivary tissue lateral to the facial nerve is removed.
Deep lobe parotidectomy is done for benign and malignant tumors involving the deep lobe of the
parotid gland. Glandular tissue that is deep to the facial nerve is removed. Total parotidectomy is done
for the benign and malignant tumors of the parotid gland that involve the whole gland. The whole of the
glandular tissue is removed and the facial nerve is preserved. Radical parotidectomy is performed
when both lobes of the gland and facial nerve are involved by the tumors. All salivary tissue, along with
the facial nerve, is removed. In extended radical parotidectomy, the whole of the salivary gland, facial

Page 16

1669
nerve, and the adjacent structures (mandible, temporal bone, muscles) are removed.
• Superficial parotidectomy is done for benign and malignant tumors involving the superficial lobe of the
parotid. Salivary tissue lateral to the facial nerve is removed.
• Deep lobe parotidectomy is done for benign and malignant tumors involving the deep lobe of the
parotid gland. Glandular tissue that is deep to the facial nerve is removed.
• Total parotidectomy is done for the benign and malignant tumors of the parotid gland that involve the
whole gland. The whole of the glandular tissue is removed and the facial nerve is preserved.
• Radical parotidectomy is performed when both lobes of the gland and facial nerve are involved by the
tumors. All salivary tissue, along with the facial nerve, is removed.
• In extended radical parotidectomy, the whole of the salivary gland, facial nerve, and the adjacent
structures (mandible, temporal bone, muscles) are removed.
• Other salivary gland tumors can be removed by en-bloc excision of the tumors and the involved gland.
• Superficial parotidectomy is done for benign and malignant tumors involving the superficial lobe of the
parotid. Salivary tissue lateral to the facial nerve is removed.
• Deep lobe parotidectomy is done for benign and malignant tumors involving the deep lobe of the
parotid gland. Glandular tissue that is deep to the facial nerve is removed.
• Total parotidectomy is done for the benign and malignant tumors of the parotid gland that involve the
whole gland. The whole of the glandular tissue is removed and the facial nerve is preserved.
• Radical parotidectomy is performed when both lobes of the gland and facial nerve are involved by the
tumors. All salivary tissue, along with the facial nerve, is removed.
• In extended radical parotidectomy, the whole of the salivary gland, facial nerve, and the adjacent
structures (mandible, temporal bone, muscles) are removed.
• High-grade tumors.
• Large primary tumors.
• Perineural invasion.
• Invasion of lymph nodes.
• Positive margins.
• Cervical lymph node metastasis.

Option A: Total parotidectomy with facial nerve preservation


• Total parotidectomy with preservation of the facial nerve is done for the benign and malignant tumors
of the parotid gland that involve the whole gland, but the facial nerve is spared.
Option B: Total parotidectomy with facial nerve sacrifice
• Total parotidectomy with facial nerve removal is done for the tumors involving superficial and deep
lobes of the parotid gland along with the facial nerve.
Option D: Deep lobe parotidectomy with facial nerve sacrifice
• When a tumor involves the deep lobe of the parotid gland along with the facial nerve, then deep lobe
parotidectomy with facial nerve removal can be done.

Page 17

1670
Solution for Question 8:
Option C: S-shaped
• Superficial parotidectomy is done for parotid gland tumors that are localized to the superficial lobe of
the parotid gland, and the facial nerve is not involved. The steps of superficial parotidectomy are as
follows: The incision is made in the relaxed preauricular skin crease. This incision is S-shaped.
• The steps of superficial parotidectomy are as follows: The incision is made in the relaxed preauricular
skin crease. This incision is S-shaped.
• The incision is made in the relaxed preauricular skin crease. This incision is S-shaped.
• The incision is made around the lobule towards the mastoid tip and then anteriorly along a natural skin
crease and curves along the anterior border of the sternocleidomastoid. A skin flap is formed in this
way.
• Subplatysmal flaps are raised and the gland is mobilized. Greater auricular nerve is identified and the
main trunk is preserved. Fascia is incised along the sternocleidomastoid's anterior border, exposing the
muscle to the mastoid process level. The facial nerve's main trunk is visible below the external auditory
canal's membranous portion. The canal is overlapped by the deep styloid process, and the canal's
superficial posterior belly of the digastric. The superficial lobe is dissected and the deep lobe is visible
beneath the branches of the facial nerve. Complications of superficial parotidectomy includes:
Hematoma. Seroma. Infection. Temporary facial nerve weakness. Permanent facial nerve weakness.
Numbness of the ear lobe is associated with great auricular nerve transection. Frey’s syndrome
Sialocele. Deformity leads to an unsightly scar and retromandibular hollowing. Facial numbness.
• Subplatysmal flaps are raised and the gland is mobilized.
• Greater auricular nerve is identified and the main trunk is preserved.
• Fascia is incised along the sternocleidomastoid's anterior border, exposing the muscle to the mastoid
process level.
• The facial nerve's main trunk is visible below the external auditory canal's membranous portion. The
canal is overlapped by the deep styloid process, and the canal's superficial posterior belly of the
digastric.
• The superficial lobe is dissected and the deep lobe is visible beneath the branches of the facial nerve.
• Complications of superficial parotidectomy includes: Hematoma. Seroma. Infection. Temporary facial
nerve weakness. Permanent facial nerve weakness. Numbness of the ear lobe is associated with great
auricular nerve transection. Frey’s syndrome Sialocele. Deformity leads to an unsightly scar and
retromandibular hollowing. Facial numbness.
• Hematoma.
• Seroma.
• Infection.
• Temporary facial nerve weakness.
• Permanent facial nerve weakness.
• Numbness of the ear lobe is associated with great auricular nerve transection.
• Frey’s syndrome
• Sialocele.
• Deformity leads to an unsightly scar and retromandibular hollowing.

Page 18

1671
• Facial numbness.
• The steps of superficial parotidectomy are as follows: The incision is made in the relaxed preauricular
skin crease. This incision is S-shaped.
• The incision is made in the relaxed preauricular skin crease. This incision is S-shaped.
• The incision is made in the relaxed preauricular skin crease. This incision is S-shaped.
• Subplatysmal flaps are raised and the gland is mobilized.
• Greater auricular nerve is identified and the main trunk is preserved.
• Fascia is incised along the sternocleidomastoid's anterior border, exposing the muscle to the mastoid
process level.
• The facial nerve's main trunk is visible below the external auditory canal's membranous portion. The
canal is overlapped by the deep styloid process, and the canal's superficial posterior belly of the
digastric.
• The superficial lobe is dissected and the deep lobe is visible beneath the branches of the facial nerve.
• Complications of superficial parotidectomy includes: Hematoma. Seroma. Infection. Temporary facial
nerve weakness. Permanent facial nerve weakness. Numbness of the ear lobe is associated with great
auricular nerve transection. Frey’s syndrome Sialocele. Deformity leads to an unsightly scar and
retromandibular hollowing. Facial numbness.
• Hematoma.
• Seroma.
• Infection.
• Temporary facial nerve weakness.
• Permanent facial nerve weakness.
• Numbness of the ear lobe is associated with great auricular nerve transection.
• Frey’s syndrome
• Sialocele.
• Deformity leads to an unsightly scar and retromandibular hollowing.
• Facial numbness.
• Hematoma.
• Seroma.
• Infection.
• Temporary facial nerve weakness.
• Permanent facial nerve weakness.
• Numbness of the ear lobe is associated with great auricular nerve transection.
• Frey’s syndrome
• Sialocele.
• Deformity leads to an unsightly scar and retromandibular hollowing.
• Facial numbness.
Option A: L-shaped

Page 19

1672
• The L-shaped incision can be used in surgeries of the abdomen, thorax and plastic surgeries.
• The incision of superficial parotidectomy is an S-shaped incision.
Option B: Y-shaped
• A Y-shaped incision is given in postmortem examination or autopsies.
• This incision starts from bilateral shoulders to the midline of the chest and moves downwards towards
the pubis.
Option D: Z-shaped
• Z-shaped incisions are given in plastic surgeries to improve the cosmetic appearance of scars.

Solution for Question 9:


Option B: Palate
• Salivary glands categorized as major (parotid, sublingual, submandibular) and minor (about 450
distributed throughout the oral cavity submucosa).
• Submandibular gland produces 70%, parotid 25%, and sublingual/minor glands 5% of saliva.
• Saliva functions in initial food digestion, lubrication, and oral cavity protection.
• Minor salivary gland disorders: cysts (common in lower lip due to trauma, treated with surgical
excision) and tumors.
• Tumors treated with wide local excision with neck dissection.
• Most common site for minor salivary gland tumors: palate (hard palate), followed by upper lip,
retromolar regions, and sinonasal tract.
• Types of tumors: Benign: Pleomorphic adenoma, Canalicular adenoma, Papillary cystadenoma,
Ductal cystadenoma, Sialadenoma papilliferum. Malignant: Adenoid cystic carcinoma (50% of cases in
palatine glands, most common), Mucoepidermoid carcinoma, Polymorphous low-grade
adenocarcinoma, Acinic cell adenocarcinoma, Non-specified adenocarcinoma.
• Benign: Pleomorphic adenoma, Canalicular adenoma, Papillary cystadenoma, Ductal cystadenoma,
Sialadenoma papilliferum.
• Malignant: Adenoid cystic carcinoma (50% of cases in palatine glands, most common),
Mucoepidermoid carcinoma, Polymorphous low-grade adenocarcinoma, Acinic cell adenocarcinoma,
Non-specified adenocarcinoma.
• Tumors arise from submucosal glands throughout the upper aerodigestive system.
• Salivary gland tumor characterized by well-defined rubbery lump, usually painless, firm, slow-growing
with rare ulceration.
• Benign: Pleomorphic adenoma, Canalicular adenoma, Papillary cystadenoma, Ductal cystadenoma,
Sialadenoma papilliferum.
• Malignant: Adenoid cystic carcinoma (50% of cases in palatine glands, most common),
Mucoepidermoid carcinoma, Polymorphous low-grade adenocarcinoma, Acinic cell adenocarcinoma,
Non-specified adenocarcinoma.
Option A: Cheeks

Page 20

1673
• The buccal mucosa of the oral cavity contains minor salivary glands. However, it is not the most
common site of minor salivary gland tumors.
Option C: Sublingual glands
• Sublingual glands are major (not minor) salivary glands. Disorders of the sublingual glands include
cysts (e.g. ranula and plunging ranula) and tumors.
Option D: Tongue
• The tongue is not the most common site for minor salivary gland tumors. Squamous cell carcinoma of
the oral cavity can involve the lateral border of the tongue.

Solution for Question 10:


Option C: Auriculotemporal nerve
• Frey’s syndrome (gustatory sweating) occurs due to aberrant misdirection of fibers from the salivary
glands to the sweat glands, this misdirection occurs as a result of nerve regeneration following damage
during surgery or trauma. These fibers come from the auriculotemporal nerve. Parasympathetic fibers
of the auriculotemporal nerve supply the salivary glands and increase salivary secretions; these
fibers can regenerate inappropriately and can stimulate sweat glands of the preauricular skin. The
causes of Frey’s syndrome include: It can occur due to damage to the auriculotemporal nerve during
dissection in parotidectomy. Damage to the auriculotemporal nerve can also occur during the open
reduction of fractures of the mandibular condyle. Aberrant cross innervations between secretomotor
parasympathetic fibers of the parotid gland and sympathetic fibers supplying the sweat glands. Clinical
features involve: Sweating, burning sensation and erythema over the region of the parotid glands as a
consequence of autonomic stimulation of salivation by smell or taste of food. Diagnosis can be made
using Minor’s starch iodine test, this test involves applying iodine to the affected area after stimulating
the salivary glands, which causes the starch to turn blue-black in the presence of sweat. Gustatory
hyperhidrosis develops over the zygomatic arch. Treatment can be done by: Antiperspirants such as
aluminum chloride application. Botulinum toxin treatment improves the symptoms of Frey’s syndrome.
It is the most effective treatment. Surgical interruption of secretory fibers by tympanic neurectomy can
be done in nonresponding cases. While antiperspirants and botulinum toxin injections are effective
treatments, it's important to note that the effects of botulinum toxin are temporary and may require
repeated injections. Surgical options like tympanic neurectomy are considered more invasive and are
typically reserved for cases that do not respond to conservative treatments. Frey’s syndrome can be
prevented by: Sternocleidomastoid muscle flap between the skin and parotid bed. Temporalis fascial
flap. Insertion of an artificial membrane between the skin and the parotid gland.
• The causes of Frey’s syndrome include: It can occur due to damage to the auriculotemporal nerve
during dissection in parotidectomy. Damage to the auriculotemporal nerve can also occur during the
open reduction of fractures of the mandibular condyle. Aberrant cross innervations between
secretomotor parasympathetic fibers of the parotid gland and sympathetic fibers supplying the sweat
glands.
• It can occur due to damage to the auriculotemporal nerve during dissection in parotidectomy.
• Damage to the auriculotemporal nerve can also occur during the open reduction of fractures of the
mandibular condyle.
• Aberrant cross innervations between secretomotor parasympathetic fibers of the parotid gland and
sympathetic fibers supplying the sweat glands.

Page 21

1674
• Clinical features involve: Sweating, burning sensation and erythema over the region of the parotid
glands as a consequence of autonomic stimulation of salivation by smell or taste of food.
• Sweating, burning sensation and erythema over the region of the parotid glands as a consequence of
autonomic stimulation of salivation by smell or taste of food.
• Diagnosis can be made using Minor’s starch iodine test, this test involves applying iodine to the
affected area after stimulating the salivary glands, which causes the starch to turn blue-black in the
presence of sweat. Gustatory hyperhidrosis develops over the zygomatic arch.
• Treatment can be done by: Antiperspirants such as aluminum chloride application. Botulinum toxin
treatment improves the symptoms of Frey’s syndrome. It is the most effective treatment. Surgical
interruption of secretory fibers by tympanic neurectomy can be done in nonresponding cases.
• Antiperspirants such as aluminum chloride application.
• Botulinum toxin treatment improves the symptoms of Frey’s syndrome. It is the most effective
treatment.
• Surgical interruption of secretory fibers by tympanic neurectomy can be done in nonresponding cases.
• While antiperspirants and botulinum toxin injections are effective treatments, it's important to note that
the effects of botulinum toxin are temporary and may require repeated injections. Surgical options like
tympanic neurectomy are considered more invasive and are typically reserved for cases that do not
respond to conservative treatments.
• Frey’s syndrome can be prevented by: Sternocleidomastoid muscle flap between the skin and parotid
bed. Temporalis fascial flap. Insertion of an artificial membrane between the skin and the parotid gland.
• Sternocleidomastoid muscle flap between the skin and parotid bed.
• Temporalis fascial flap.
• Insertion of an artificial membrane between the skin and the parotid gland.
• The causes of Frey’s syndrome include: It can occur due to damage to the auriculotemporal nerve
during dissection in parotidectomy. Damage to the auriculotemporal nerve can also occur during the
open reduction of fractures of the mandibular condyle. Aberrant cross innervations between
secretomotor parasympathetic fibers of the parotid gland and sympathetic fibers supplying the sweat
glands.
• It can occur due to damage to the auriculotemporal nerve during dissection in parotidectomy.
• Damage to the auriculotemporal nerve can also occur during the open reduction of fractures of the
mandibular condyle.
• Aberrant cross innervations between secretomotor parasympathetic fibers of the parotid gland and
sympathetic fibers supplying the sweat glands.
• Clinical features involve: Sweating, burning sensation and erythema over the region of the parotid
glands as a consequence of autonomic stimulation of salivation by smell or taste of food.
• Sweating, burning sensation and erythema over the region of the parotid glands as a consequence of
autonomic stimulation of salivation by smell or taste of food.
• Diagnosis can be made using Minor’s starch iodine test, this test involves applying iodine to the
affected area after stimulating the salivary glands, which causes the starch to turn blue-black in the
presence of sweat. Gustatory hyperhidrosis develops over the zygomatic arch.
• Treatment can be done by: Antiperspirants such as aluminum chloride application. Botulinum toxin
treatment improves the symptoms of Frey’s syndrome. It is the most effective treatment. Surgical
interruption of secretory fibers by tympanic neurectomy can be done in nonresponding cases.

Page 22

1675
• Antiperspirants such as aluminum chloride application.
• Botulinum toxin treatment improves the symptoms of Frey’s syndrome. It is the most effective
treatment.
• Surgical interruption of secretory fibers by tympanic neurectomy can be done in nonresponding cases.
• While antiperspirants and botulinum toxin injections are effective treatments, it's important to note that
the effects of botulinum toxin are temporary and may require repeated injections. Surgical options like
tympanic neurectomy are considered more invasive and are typically reserved for cases that do not
respond to conservative treatments.
• Frey’s syndrome can be prevented by: Sternocleidomastoid muscle flap between the skin and parotid
bed. Temporalis fascial flap. Insertion of an artificial membrane between the skin and the parotid gland.
• Sternocleidomastoid muscle flap between the skin and parotid bed.
• Temporalis fascial flap.
• Insertion of an artificial membrane between the skin and the parotid gland.
• It can occur due to damage to the auriculotemporal nerve during dissection in parotidectomy.
• Damage to the auriculotemporal nerve can also occur during the open reduction of fractures of the
mandibular condyle.
• Aberrant cross innervations between secretomotor parasympathetic fibers of the parotid gland and
sympathetic fibers supplying the sweat glands.
• Sweating, burning sensation and erythema over the region of the parotid glands as a consequence of
autonomic stimulation of salivation by smell or taste of food.
• Antiperspirants such as aluminum chloride application.
• Botulinum toxin treatment improves the symptoms of Frey’s syndrome. It is the most effective
treatment.
• Surgical interruption of secretory fibers by tympanic neurectomy can be done in nonresponding cases.
While antiperspirants and botulinum toxin injections are effective treatments, it's important to note that t
he effects of botulinum toxin are temporary and may require repeated injections. Surgical options like t
ympanic neurectomy are considered more invasive and are typically reserved for cases that do not res
pond to conservative treatments.
• Sternocleidomastoid muscle flap between the skin and parotid bed.
• Temporalis fascial flap.
• Insertion of an artificial membrane between the skin and the parotid gland.

Option A: Facial nerve


• The Facial nerve supplies the muscles of the face, taste from the anterior 2/3rd of the tongue,
lacrimation, eye closing, and auditory volume modulation.
• The facial nerve supplies secretory parasympathetic fibers to submandibular and sublingual glands.
• The parotid gland receives parasympathetic supply via the auriculotemporal nerve.
Option B: Trigeminal nerve
• Trigeminal nerve functions in the muscle of mastication, facial sensation, somatosensation from
anterior 2/3rd of the tongue and dampening of loud noises.

Page 23

1676
• It does not supply the parotid gland.
Option D: Glossopharyngeal nerve
• The glossopharyngeal nerve carries taste and sensation from the posterior 1/3rd of the tongue,
swallowing, salivation (parotid gland via auriculotemporal nerve), carotid body and sinus chemo- and
baroreceptors, and elevation of pharynx and larynx.
• Frey's syndrome is caused by the transection of the auriculotemporal branch of the glossopharyngeal
nerve.

Solution for Question 11:


Option A: Botulinum toxin
• Frey's syndrome (gustatory sweating) occurs due to aberrant misdirection of fibers from the salivary
glands to the sweat glands. These fibers comes from the auriculotemporal nerve. Parasympathetic
fibers of the auriculotemporal nerve supply the salivary glands and increase salivary secretions. These
fibers can regenerate inappropriately and stimulate the preauricular skin's sweat glands.
• The causes of Frey's syndrome include: It can occur due to damage to the auriculotemporal nerve
during dissection in parotidectomy. Damage to the auriculotemporal nerve can also occur during the
open reduction of mandibular condyle fractures.
• It can occur due to damage to the auriculotemporal nerve during dissection in parotidectomy.
• Damage to the auriculotemporal nerve can also occur during the open reduction of mandibular
condyle fractures.
• Treatment can be done by: Antiperspirants such as aluminum chloride application. Botulinum toxin
treatment improves the symptoms of Frey's syndrome. It is the most effective treatment and can be
performed as an outpatient procedure. Surgeons can do surgical interruption of secretory fibers by
tympanic neurectomy in nonresponding cases.
• Antiperspirants such as aluminum chloride application.
• Botulinum toxin treatment improves the symptoms of Frey's syndrome. It is the most effective
treatment and can be performed as an outpatient procedure.
• Surgeons can do surgical interruption of secretory fibers by tympanic neurectomy in nonresponding
cases.
• It can occur due to damage to the auriculotemporal nerve during dissection in parotidectomy.
• Damage to the auriculotemporal nerve can also occur during the open reduction of mandibular
condyle fractures.
• Antiperspirants such as aluminum chloride application.
• Botulinum toxin treatment improves the symptoms of Frey's syndrome. It is the most effective
treatment and can be performed as an outpatient procedure.
• Surgeons can do surgical interruption of secretory fibers by tympanic neurectomy in nonresponding
cases.
Option B: Temporalis fascia flap
• Temporalis fascia flaps can be made during parotidectomy to minimize the regeneration of autonomic
nerve fibers.

Page 24

1677
• It is used to prevent Frey’s syndrome, not to manage it.
Option C: Sternocleidomastoid muscle flap
• The sternocleidomastoid muscle flap replaces the barrier between the skin and the parotid bed to
minimize the inappropriate regeneration of autonomic nerve fibers.
• This procedure can be used to prevent Frey's syndrome, not to treat Frey's syndrome.
Option D: Superficial parotidectomy
• Superficial and deep parotidectomy can cause damage to the auriculotemporal nerve that supplies
the parotid gland. This nerve regenerates inappropriately and stimulates the sweat glands.
• Parotidectomy is a cause of Frey's syndrome. It is not used for the management of this syndrome.

Solution for Question 12:


Option A: Radiotherapy to head and neck
• This patient with a history of radiotherapy to the neck and head region for treatment of his oral
squamous cell carcinoma presented with a retromolar mass that is firm, tender and bleeds on
palpation. This presentation and biopsy findings are consistent with a salivary gland tumor.
• Salivary gland tumors are abnormal cells growing in the salivary gland tissue or ducts. Salivary gland
tumors can be benign or malignant.
• Mucoepidermoid carcinoma is the second most common minor salivary gland tumor (the most
common is adenoid cystic carcinoma). It occurs during the fourth and fifth decades of life and is found
most commonly in the parotid gland, which causes a low-grade tumor.
• However, mucoepidermoid carcinoma of the minor salivary glands causes a much more aggressive
high-grade tumor. It presents as a firm mass that can bleed on palpation. It usually occurs on the
palate, sinonasal tract, and retromolar area.
• The most common risk factor for mucoepidermoid carcinoma is radiotherapy to the head and neck
region (as in this patient for his oral squamous cell carcinoma).
Option B: Exposure to silica dust
• Exposure to silica dust and nitrosamines increases the risk for certain salivary gland tumors. Silica
dust exposure occurs in patients working in glass manufacturing companies and those exposed to
sandblasting. This patient, however, works as a plumber, which does not involve exposure to silica
dust.
Option C: EBV infection
• EBV infection is linked with lymphoepithelial carcinoma, whereas this patient is diagnosed with
mucoepidermoid carcinoma.
Option D: Smoking
• Smoking increases the risk for benign salivary gland tumors such as Adenolymphoma (Warthin
tumor). It does not increase the risk of malignant tumor of the salivary gland.

Page 25

1678
Solution for Question 13:
Option B: It is the most common malignant salivary gland tumor in children
• Mucoepidermoid carcinoma is the second most common minor salivary gland tumor in adults, but
most common malignant salivary gland tumor in children.
• The most common in adults is adenoid cystic carcinoma.
• It most commonly occurs during the fourth and fifth decades of life. However, it can also be found in
children. It can be present in the parotid gland, where it causes a low-grade tumor.
• Mucoepidermoid carcinoma of the minor salivary glands usually causes a much more aggressive
high-grade tumor. It presents as a firm mass that can bleed on palpation. It usually occurs on the
palate, sinonasal tract, and retromolar area.
• The most common risk factor for mucoepidermoid carcinoma is radiotherapy to the head and neck
region.
• Mucoepidermoid carcinoma is the most common malignant salivary gland tumor in children.
• It can be a low-grade tumor having a good prognosis or a high-grade tumour with a fair or poor
prognosis.
• Mucin-producing cells predominate
• It is well-differentiated
• It is present mostly in children
• Treatment of choice is superficial or total parotidectomy.
• Squamous cells predominate
• It is poorly differentiated
• It is less common in children.
• Treatment includes total parotidectomy +/- radical neck dissection.

Option A: The low-grade subtype has predominantly squamous cells.


• A low-grade subtype of mucoepidermoid carcinoma has predominantly mucin-producing cells that
produce mucinous material. The high-grade subtype has squamous cells.
Option C: It usually causes facial paralysis
• Mucoepidermoid carcinoma usually does not involve the facial nerve leading to facial paralysis.
Adenoid cystic carcinoma invades perineural space and lymphatics and causes pain and facial nerve
paralysis.
Option D: It is well-differentiated in adults
• High-grade mucoepidermoid carcinoma is usually present in adults and is poorly differentiated.

Solution for Question 14:


Option A: Parotid gland

Page 26

1679
• Acinic cell carcinoma is a low-grade tumor that resembles a benign mixed tumor. It is a small, firm,
movable, slow-growing, and encapsulated tumor. It is often bilateral. The parotid gland is the most
common site of origin of this carcinoma. Women are more commonly affected than men. Previous
radiation exposure or family history of an acinic cell carcinoma are risk factors for this tumor. Diagnosis
can be made by fine needle aspiration biopsy of the mass. Basophilic small cells like acinar cells are
arranged in a cystic or follicular pattern on biopsy. Metastasis is usually rare. Acinic cell carcinoma can
be treated with either superficial or total parotidectomy.Radiotherapy may also be indicated in some
cases. Radiotherapy is indicated when: Margins are positive. Incomplete resection of the tumor.
Extension of tumor beyond the parotid gland. Lymph node positivity. This carcinoma can recur after
treatment and spread to cervical lymph nodes and lungs. Long-term follow-up is important, as acinic
cell carcinoma can recur and metastasize, but the prognosis is generally favorable, especially for
low-grade tumors.
• It is a small, firm, movable, slow-growing, and encapsulated tumor.
• It is often bilateral.
• The parotid gland is the most common site of origin of this carcinoma.
• Women are more commonly affected than men.
• Previous radiation exposure or family history of an acinic cell carcinoma are risk factors for this tumor.
• Diagnosis can be made by fine needle aspiration biopsy of the mass. Basophilic small cells like acinar
cells are arranged in a cystic or follicular pattern on biopsy.
• Metastasis is usually rare.
• Acinic cell carcinoma can be treated with either superficial or total parotidectomy.Radiotherapy may
also be indicated in some cases.
• Radiotherapy is indicated when: Margins are positive. Incomplete resection of the tumor. Extension of
tumor beyond the parotid gland. Lymph node positivity.
• Margins are positive.
• Incomplete resection of the tumor.
• Extension of tumor beyond the parotid gland.
• Lymph node positivity.
• This carcinoma can recur after treatment and spread to cervical lymph nodes and lungs.
• Long-term follow-up is important, as acinic cell carcinoma can recur and metastasize, but the
prognosis is generally favorable, especially for low-grade tumors.
• It is a small, firm, movable, slow-growing, and encapsulated tumor.
• It is often bilateral.
• The parotid gland is the most common site of origin of this carcinoma.
• Women are more commonly affected than men.
• Previous radiation exposure or family history of an acinic cell carcinoma are risk factors for this tumor.
• Diagnosis can be made by fine needle aspiration biopsy of the mass. Basophilic small cells like acinar
cells are arranged in a cystic or follicular pattern on biopsy.
• Metastasis is usually rare.
• Acinic cell carcinoma can be treated with either superficial or total parotidectomy.Radiotherapy may
also be indicated in some cases.

Page 27

1680
• Radiotherapy is indicated when: Margins are positive. Incomplete resection of the tumor. Extension of
tumor beyond the parotid gland. Lymph node positivity.
• Margins are positive.
• Incomplete resection of the tumor.
• Extension of tumor beyond the parotid gland.
• Lymph node positivity.
• This carcinoma can recur after treatment and spread to cervical lymph nodes and lungs.
• Long-term follow-up is important, as acinic cell carcinoma can recur and metastasize, but the
prognosis is generally favorable, especially for low-grade tumors.
• Margins are positive.
• Incomplete resection of the tumor.
• Extension of tumor beyond the parotid gland.
• Lymph node positivity.
Option B: Minor salivary glands
• Acinic cell carcinoma can arise from minor salivary glands in a minority of cases. The major site of
origin of acinic cell carcinoma is the parotid gland.
Option C: Submandibular glands
• Ninety percent of cases of acinic cell carcinoma arise from the parotid gland. Only 2% of cases arise
from the rest of the salivary glands.
Option D: Sublingual glands
• Sublingual glands are an uncommon site of origin of acinic cell carcinoma. The most common site is
the parotid glands.

Solution for Question 15:


Option A: Most common malignant tumor in minor salivary glands
• Adenoid cystic carcinoma is the most common malignant tumor of the sublingual, submandibular, and
minor salivary glands. It is often called the ‘treacherous tumor’ as it appears benign but is malignant.
• It is a slow-growing tumor that widely infiltrates the tissue planes and muscles.
• It can also involve the perineural spaces and lymphatics and cause pain. Because of this tumor's
neurotrophic characteristics, the facial nerve is most commonly involved. Trigeminal nerves are less
commonly affected.
• The most common site of origin of this tumor is the hard palate. It is less commonly found in the
sinonasal tracts.
• Biopsy of tumor specimens may reveal the following subtypes: The cribriform pattern is like ‘Swiss
cheese’ in appearance. Solid pattern is associated with a more aggressive disease course. Tubular
pattern.
• The cribriform pattern is like ‘Swiss cheese’ in appearance.

Page 28

1681
• Solid pattern is associated with a more aggressive disease course.
• Tubular pattern.
• It can metastasize to adjacent lymph nodes.
• Local recurrences are common after surgical excision and can occur about 10-20 years after surgery.
• Distant metastasis can go to the lung, brain, or bone. The lungs are the most common site of distant
metastasis of this tumor. The tumor can involve the Haversian canals of long bones.
• Treatment is surgical excision with the largest cuff of normal tissue around the tumor’s boundaries.
• Radical neck dissection can be done if nodal metastasis is present.
• Postoperative radiation can be given if the margins of the resected tumor are not free of tumor. This
radiotherapy can also prevent the recurrence of the disease.
• The cribriform pattern is like ‘Swiss cheese’ in appearance.
• Solid pattern is associated with a more aggressive disease course.
• Tubular pattern.
Option B: The most common site is the sinonasal tract
• Adenoid cystic carcinoma is most commonly found in the hard palate. It is less commonly seen in the
sinonasal tract.
Option C: The most commonly involved nerve is trigeminal nerve
• An adenoid cystic carcinoma most commonly involves the facial nerve. Involvement of trigeminal
nerves is less common.
Option D: The most common site of metastasis is the liver
• The most common site of distant metastasis of an adenoid cystic carcinoma is the lungs. Long bones
and brain metastasis are also common.
• The liver is less likely to be involved in adenoid cystic carcinoma.

Solution for Question 16:


Option B: Newman and Seabrook’s operation
• This patient, who presented with an opening above the angle of the mandible after suffering a knife
laceration in the same area, has a fistulous tract draining clear fluid. Increased amylase levels in the
fluid show that the fistula drains secretions from the parotid gland. Hence the diagnosis is a parotid
fistula.
• The parotid fistula drains a clear serous fluid that increases in quantity during eating meals and
chewing.
• Parotid fistula can be internal or external. The internal fistula opens inside the mouth and is usually
asymptomatic.
• The external fistula opens towards the skin and drains parotid secretions. It could be connected to the
parotid duct or the parotid gland.

Page 29

1682
• The external opening is pinpointed when the external fistula is connected to the gland. Serous
discharge can be present for several months but usually closes spontaneously.
• When the external fistula is connected with the major duct, parotid secretions are continuously
outpouring onto the cheek during meals with excoriation and scarring of the surrounding skin.
• There are multiple causes of the formation of a parotid fistula. The most common cause of a parotid
fistula is penetrating trauma to the gland. Other causes include rupture of a parotid gland abscess,
inadvertent incision during drainage of a parotid abscess, or as a complication after superficial
parotidectomy.
• A parotid fistula can be diagnosed through CT sialography or a sialogram.
• Newman and Seabrook’s operation is used to repair the fistulous tract. It reconstructs the main parotid
duct when the fistula is connected to the main duct.
Option A: External beam radiotherapy
• Radiotherapy can be used in the treatment of tumors such as those of the lip and tongue. It is not
used to treat parotid fistula.
Option C: Intravenous antibiotics
• Antibiotics are used in the treatment of acute suppurative parotitis. It is usually caused by bacterial
infection, most commonly Staphylococcus aureus, necessitating antibiotics.
Option D: Radical neck dissection
• Radical neck dissection treats tumors that have metastasized to the cervical lymph nodes. It can treat
squamous cell carcinoma of the oral cavity with cervical lymph node involvement.

Solution for Question 17:


Option A: It is a mucous extravasation cyst arising from the sublingual gland
• This patient presented with a submandibular swelling that is soft and nontender. Examination showed
a translucent swelling on the floor of the mouth. This presentation is consistent with a diagnosis of
ranula.
• The term ranula is applied to a mucous extravasation cyst that arises from the sublingual gland.
• It is mostly caused due to local trauma and duct rupture (i.e. during tongue piercing), followed by
mucin spillage into the surrounding soft tissues. It can also rarely be caused by obstruction due to a
sialolith or a mucus plug.
• Ranula is present on the floor of the mouth. It is usually unilateral and lateral to the midline. It is a
smooth, dome-shaped, fluctuating, and painless swelling.
• The size of a ranula varies from a few to several centimeters in diameter. Very large lesions may
occupy the entire floor of the mouth.
• Histopathological features include mucin accumulation surrounded by granulation and fibrous tissue
(mucous extravasation phenomenon) or a cyst filled with mucin and lined by the ductal epithelium
(mucus retention cyst)
• Diagnosis is usually made clinically, but MRI can be used in atypical cases. Treatment involves
surgical removal of the cyst or marsupialization of the cyst.

Page 30

1683
• Removal of the ranula can be tricky as there is a high risk of damaging the submandibular duct during
ranula surgery.
Option B: It is treated with chemotherapy
• Chemotherapy can be used in some cases of carcinomas of the oral cavity. A tumor presents as a
hard, fixed mass, unlike this patient, who presented with a soft, non-tender mass.
Option C: It is the most common type of minor salivary gland tumor
• Adenoid cystic carcinoma is the most common minor salivary gland tumor and is present most
commonly on the palate. This patient's presentation is not consistent with a minor salivary gland tumor.
Option D: It is diagnosed by biopsy
• A ranula is diagnosed clinically. MRI can be used in patients with atypical presentations. A biopsy is
not required.

Solution for Question 18:


Option C: Warthin's tumor
• Warthin's tumor, also called papillary cystadenoma lymphomatosum, is the second most common
benign salivary gland tumor (after pleomorphic adenoma). It represents 10-15% of the total salivary
gland neoplasms.
• They are commonly seen between the fifth and seventh decade of life, and there is an apparent male
predilection for their occurrence. They are typically found in smokers.
• They almost exclusively occur in the parotid glands and can be bilateral in 10% of cases.
• Histologically, it is composed of an oncotic epithelium and a lymphoid stroma.
• It consists of an encapsulated mass containing numerous irregular cystic spaces lined by bilayered
oncotic epithelium growing in the cystic spaces as papillary projections.
• The underlying stroma separating the oncotic epithelium is typically a large lymphoid matrix with
germinal centers.
• Treatment is superficial parotidectomy, although they can be enucleated without anydanger of
recurrence.
Option A: Pleomorphic adenoma
• Pleomorphic adenoma is the most common parotid gland neoplasm. It is composed of a
chondromyxoid stroma and epithelium. There are no germinal centers, and the tumor is pleomorphic
rather than monomorphic, as in this patient.
Option B: Adenoid cystic carcinoma
• It is the most common tumor of the minor salivary glands. It occurs most commonly in the palate,
retromolar area, and sinonasal tract.
• It does not occur in the parotid gland. It commonly involves the facial nerve.
Option D: Mucoepidermoid carcinoma
• It is the second most common salivary gland tumor. It has mucinous and squamous components.
Germinal centers are absent.

Page 31

1684
Page 32

1685
Neck
1. Which structure is most likely responsible for the copious dark blood encountered during a bedside
tracheostomy in a patient with head trauma?
A. Anterior jugular vein
B. External jugular vein
C. Internal jugular vein
D. Middle thyroid vein
----------------------------------------
2. A 28-year-old male patient comes complaining of swelling on the lateral aspect of the neck. Gentle
pressure on the swelling results in a non-bloody discharge. The physical examination of the neck
shows a draining sinus located on the anterior border of the sternocleidomastoid. Which is the most
likely correct description of the lining of this sinus?
(or)
What is the most likely lining of a draining sinus located on the anterior border of the
sternocleidomastoid?
A. Columnar epithelium
B. Cuboidal epithelium
C. Squamous epithelium
D. Ciliated columnar epithelium
----------------------------------------
3. Which of the following structures is not preserved during modified radical neck dissection?
A. Submandibular gland
B. Spinal accessory Nerve
C. Sternocleidomastoid
D. Internal jugular vein
----------------------------------------
4. A 4-month-old boy presents with a cystic, compressible swelling that is translucent and
trans-illuminates in the posterior triangle of the neck. Which of the following is the most likely etiology in
this patient?

1686
A. Cystic hygroma
B. Branchial cyst
C. Thyroglossal cyst
D. Dermoid cyst
----------------------------------------
5. What is the likely diagnosis in a 38-year-old woman with a progressive neck swelling that showed a
large infiltrative nasopharyngeal growth on endoscopy and an elevated antibody titer to EBV capsid?
A. Angiofibroma
B. Nasopharyngeal carcinoma
C. Laryngeal carcinoma
D. Squamous cell carcinoma of the tongue
----------------------------------------
6. A 55-year-old patient presents with complaints of painless neck swelling. A CT showed a 3.5cm wide
carotid body tumor without vessel encasement. Which of the following is the best next step in treating
this patient?
A. Excision of tumor
B. Radiotherapy
C. Chemotherapy
D. Carotid artery ligation, both proximal and distal to the tumor
----------------------------------------
7. A 3-year-old male child was brought with a painless, firm midline cystic mass near the hyoid bone
that elevates with swallowing and tongue protrusion. The examination findings of the neck are
illustrated in the image below: Which of the following best describes the formation of this neck mass?

A. Presence of ectopic tissue


B. Injury
C. Failure of involution of the thyroglossal duct
D. Inflammatory disorder
----------------------------------------
8. Level V cervical nodes include which of the following?

Page 2

1687
A. Upper jugular nodes
B. Middle jugular nodes
C. Lower jugular nodes
D. Posterior triangle nodes
----------------------------------------
9. A 6-day-old male child is brought with complaints of sucking difficulties and nasal regurgitation of
milk. The patient is also choking, coughing during feeds, and gasping for air. On examination, a cleft
palate is observed. Which of the following is not the primary purpose of the surgical correction of the
cleft palate?
(or)
Which of the following is NOT the primary purpose of the surgical correction of the cleft palate in a
6-days-old male?
A. Control of regurgitation
B. To promote normal dentition and facial growth
C. To reduce the frequency of paranasal sinuses infections
D. Normal appearance of lips, nose, and face
----------------------------------------
10. In a scenario of laryngeal carcinoma with nodal staging of N-3, what is the most likely size of the
lymph node draining the tumor?
A. Metastasis in lymph nodes more than 2 cm
B. Metastasis in lymph nodes more than 5cm
C. Metastasis in a lymph nodes more than 6 cm
D. No regional lymph node involvement
----------------------------------------
11. In a patient who underwent surgery for adenoid cystic carcinoma and experiences sweating and
flushing of cheeks while eating, which nerve is likely injured?
A. Facial nerve
B. Trigeminal nerve
C. Auriculotemporal nerve
D. Glossopharyngeal nerve
----------------------------------------
12. A 30-year-old female patient presents to the surgical outpatient department complaining of swelling
in the front of the neck. Initially, it was smaller in size, but it grew over time. On examination, the patient
appears sick but is vitally stable with a pulse of 63/min, respiratory rate 15/min, and blood pressure of
102/70 mm Hg. On palpation of the neck, a cystic mass of the neck is noted, which moves with
swallowing but does not move with tongue protrusion. The findings of the neck examination are
illustrated in the image below: Which of the following is the most likely diagnosis in this patient?
(or)

Page 3

1688
What is the most likely diagnosis in a 30-year-old female patient with a cystic mass in front of the neck
that moves with swallowing but not with tongue protrusion? Which of the following is the most likely
diagnosis in this patient?

A. Dermoid cyst
B. Sebaceous cyst
C. Branchial cyst
D. Metastatic tumours in the neck
----------------------------------------
13. A 63-year-old man with a history of anterior myocardial infarction, type 2 diabetes mellitus, and
hypertension visits the doctor after experiencing shortness of breath, low-grade fever, excessive night
sweats, and significant weight loss. His vital signs include a temperature of 38.0°C (100.4°F), a pulse of
104/min. Examination shows a painless fluctuant, non-transilluminate swelling on the neck with a thin
watery discharge, as shown in the image below: Chest x-ray shows bilateral pleural effusion and patchy
infiltrates in the right middle lobe of the lung. Which of the following is the most likely clinical diagnosis
for this patient?
(or)
What is the most likely diagnosis in a 63-year-old man with shortness of breath, night sweats, weight
loss, and a painless fluctuant, non-trans-illuminating neck swelling with thin watery discharge, bilateral
pleural effusion and patchy infiltrates in the right middle lobe of the lung on chest x-ray?

A. Branchial cyst
B. Metastatic tumorin the neck
C. Tuberculous lymphadenitis
D. Lymphoma

Page 4

1689
----------------------------------------
14. A 2-month-old female baby comes complaining of swelling on the posterior aspect of the neck. The
patient is stable. On examination of the posterior triangle of the neck, a soft, compressible, and painless
swelling that trans-illuminates is found. Which of the following clinical conditions may be associated
with the presentation of this patient?
(or)
Which clinical condition is associated with the presentation of a 2-month-old female child with a
painless, soft, compressible, translucent swelling on the posterior triangle of the neck that does not
result in discharge upon gentle pressure?
A. Turner's syndrome
B. Klinefelter's syndrome
C. Down's syndrome
D. All of the above
----------------------------------------
15. Which of the following classification systems is used to classify the carotid body tumor?
A. Shamblin classification
B. DeBakey classification
C. Stanford classification
D. Montreal classification
----------------------------------------
16. Which of the following is the most frequent site of branchial cyst?
A. The upper third of the posterior border of the sternocleidomastoid
B. The lower third of the anterior border of the sternocleidomastoid
C. The upper third of the anteromedial border of the sternocleidomastoid
D. Supraclavicular fossa
----------------------------------------
17. A 65-year-old female patient presents to the outpatient department of surgery with complaints of
painless swelling below her right ear. Mucosal ulceration in the overlying area is also present. On
examination, a 2*3 cm painless growth is present around his right parotid region. The patient also has
right-sided facial nerve palsy. Bilaterally enlarged cervical lymph nodes are noted. The biopsy and the
histopathology report reveal mucoepidermoid carcinoma of the parotid gland. The physician
recommends a surgical excision for the removal of the mass. During the preoperative workup, the
surgeon informs the patient that he will need an extended supraomohyoid neck dissection. During an
extended supraomohyoid neck dissection, which of the following structure(s) is/are removed?
(or)
Which structures are removed during an extended supraomohyoid neck dissection in a 65-year-old
female ?
A. Removal of level I-III lymph nodes
B. Removal of level I-IV lymph nodes

Page 5

1690
C. Removal of level II-V lymph node groups + suboccipital lymph node groups + retro auricular lymph
nodes
D. Removal of level II-V lymph nodes + internal jugular vein
----------------------------------------

Correct Answers
Question Correct Answer

Question 1 1
Question 2 3
Question 3 1
Question 4 1
Question 5 2
Question 6 1
Question 7 3
Question 8 4
Question 9 4
Question 10 3
Question 11 3
Question 12 1
Question 13 3
Question 14 4
Question 15 1
Question 16 3
Question 17 2

Solution for Question 1:


Option A: Anterior jugular vein
• This patient who underwent tracheostomy and bleeding during the procedure most likely sustained an
injury or transection to the anterior jugular vein.
• Tracheostomy is the most common surgical procedure for critically ill patients who require extended
mechanical ventilation.
• Indications for tracheostomy in critically ill patients: Presence of pathological conditions, prediction of
prolonged mechanical intubation, inability to protect the airways or both High-risk airway and airway
edema after surgery and maxillofacial trauma High-risk airway due to cervical immobilization for
fracture fixation Need for airway surgery due to inability to intubate the patient
• Presence of pathological conditions, prediction of prolonged mechanical intubation, inability to protect
the airways or both
• High-risk airway and airway edema after surgery and maxillofacial trauma

Page 6

1691
• High-risk airway due to cervical immobilization for fracture fixation
• Need for airway surgery due to inability to intubate the patient
• Perioperative complications of percutaneous tracheostomy include Peristomal bleeding due to injury
to the anterior jugular veins or thyroid isthmus. Injury to the trachea or esophagus, or both, by cutting
the posterior wall of the trachea Extra luminal placement, creating an incorrect tract during tracheal
cannula placement
• Peristomal bleeding due to injury to the anterior jugular veins or thyroid isthmus.
• Injury to the trachea or esophagus, or both, by cutting the posterior wall of the trachea
• Extra luminal placement, creating an incorrect tract during tracheal cannula placement
• The anterior jugular vein runs down from beneath the chin, turns laterally, and drains into the external
jugular vein. It's found just above the jugular notch, and may be encountered during a surgical airway if
the operator drifts laterally.
• Presence of pathological conditions, prediction of prolonged mechanical intubation, inability to protect
the airways or both
• High-risk airway and airway edema after surgery and maxillofacial trauma
• High-risk airway due to cervical immobilization for fracture fixation
• Need for airway surgery due to inability to intubate the patient
• Peristomal bleeding due to injury to the anterior jugular veins or thyroid isthmus.
• Injury to the trachea or esophagus, or both, by cutting the posterior wall of the trachea
• Extra luminal placement, creating an incorrect tract during tracheal cannula placement
Other options
Option B: External jugular vein
• The external jugular vein collects blood from the exterior cranium and parts of the face.
• It begins at the parotid gland, extends downwards, and runs along the side of the neck.
• This vein is covered by the platysma and pierces through the superficial layer of the fascia of the
neck.
• The external jugular veindrains into subclavian vein. However, this vein is not susceptible to injury
during the tracheostomy.
Option C: Internal jugular vein
• It is a paired vein that collects blood from the brain, face, and thyroid gland from the sigmoid and
inferior petrosal sinus confluence and exits the skull through the jugular foramen. Afterwards, it
continues inside the carotid sheath with the carotid artery and the vagus nerve.
• However, this vein is not susceptible to injury during the tracheostomy.
Option D: Middle thyroid vein
• This vein is not susceptible to injury during the tracheostomy.

Solution for Question 2:

Page 7

1692
Option C: Squamous epithelium
• This patient is most likely suffering from a branchial cleft sinus.
• It is a lateral neck sinus caused by incomplete obliteration of the branchial clefts between branchial
arches.
• The most common site of the branchial cleft sinus is the anterior border of the sternocleidomastoid
(due to incomplete obliteration of the second branchial cleft).
• The image below illustrates the findings of the branchial cleft sinus.

• It is a fluctuant swelling that may transilluminate and is often soft in its early stages, so it may be
difficult to palpate.
• The branchial cyst is lined by squamous epithelium and contains thick, turbid fluid.
• The diagnosis is usually clinical; however, ultrasonography may be performed.
• Treatment requires surgical removal of the branchial cleft sinus.
Incorrect Options:
Option A, B and, D are incorrect.

Solution for Question 3:


Option A: Submandibular gland
• The submandibular gland is not preserved during modified radical neck dissection.
• Radical neck dissection refers to removing nodal levels I-V, the SCM, IJV, CN XI, and the
submandibular gland.
• The modified radical neck dissection removes levels I-V while preserving one or more of the following
structures: SCM, IJV, or CN XI.
• In selective neck dissection, one or more of the major lymph node groups is preserved along with the
sternocleidomastoid muscle, accessory nerve, and internal jugular vein.

Page 8

1693
• Finally, an extended neck dissection refers to any neck dissection that removes additional structures
of lymph nodes from areas not addressed in radical neck dissection.
Other Options
Option B, C, and D
are selectively preserved during modified radical neck dissection, as explained above.

Solution for Question 4:


Option A: Cystic hygroma
• This patient is most likely suffering from cystic hygroma.
• Cystic hygroma is a multiloculated cystic space lined by endothelial cells.
• It occurs due to the sequestration of a portion of the jugular lymph sac from the lymphatic system.
• Cysts are filled with clear lymph and are lined by endothelium.
• Turner's syndrome is associated with cystic hygroma.
• Most cystic hygromas involve the lymphatic jugular sacs
• The most common site is the posterior neck region (Posterior triangle of the neck), as shown in the
image below:

• Other common sites for cystic hygroma development include: Axilla Mediastinum Inguinal
and retroperitoneal regions
• Axilla
• Mediastinum
• Inguinal and retroperitoneal regions
• The clinical features of cystic hygroma include: A soft cystic mass on the neck's posterior aspect that
distorts the surrounding anatomy can result in acute airway obstruction. Usually manifests in neonates
or early infancy (50% present at birth). Cyst contains clear lymph and the swelling is brilliantly

Page 9

1694
transilluminant.
• A soft cystic mass on the neck's posterior aspect that distorts the surrounding anatomy can result in
acute airway obstruction.
• Usually manifests in neonates or early infancy (50% present at birth).
• Cyst contains clear lymph and the swelling is brilliantly transilluminant.
• Diagnostics work-up involves ultrasonography
• The treatment is not indicated in some cases, as some may regress spontaneously. However,
surgical excision is indicated occasionally.
• Axilla
• Mediastinum
• Inguinal and retroperitoneal regions
• A soft cystic mass on the neck's posterior aspect that distorts the surrounding anatomy can result in
acute airway obstruction.
• Usually manifests in neonates or early infancy (50% present at birth).
• Cyst contains clear lymph and the swelling is brilliantly transilluminant.
Option B: Branchial cyst
• A branchial cyst presents as a painless mass lateral to the midline, usually anterior to the
sternocleidomastoid muscle, which does not move with swallowing.
• However, the mass described in this vignette is found on the neck's posterior aspect, making this
diagnosis unlikely.
Option C: Thyroglossal cyst
• A Thyroglossal cyst is a remnant of the thyroglossal duct that forms during the embryonic
development of the thyroid gland.
• It typically presents as a painless midline neck mass that elevates with swallowing and tongue
protrusion. However, the mass described in this vignette is found on the neck's posterior aspect,
making this diagnosis unlikely.
Option D: Dermoid cyst
• It is a mature cystic teratoma on the head, neck, spine, cranium, and abdomen (the most common
ovarian tumour in females).
• However, the dermoid cyst is not translucent and does not typically trans-illuminate, making this
diagnosis unlikely.

Solution for Question 5:


Option B: Nasopharyngeal carcinoma
• This patient's most likely suffering from nasopharyngeal carcinoma. Nasopharyngeal carcinoma is a
malignancy of the nasopharynx.
• The clinical features include: Painless lymphadenopathy Obstruction of the Eustachian tube, which
causes recurrent otitis media and conductive hearing loss Nasal discharge Nose-bleeds Impaired nasal

Page 10

1695
breathing
• Painless lymphadenopathy
• Obstruction of the Eustachian tube, which causes recurrent otitis media and conductive hearing loss
• Nasal discharge
• Nose-bleeds
• Impaired nasal breathing
• Nasopharyngeal carcinoma is usually associated with Epstein-Barr virus infection.
• Trotter's triad is also seen in nasopharyngeal carcinoma.
• Trotter’s triad is characterized by the following: Conductive deafness (eustachian tube blockage)
Ipsilateral temporoparietal neuralgia (involvement of CN V) Palatal paralysis (CN X)
• Conductive deafness (eustachian tube blockage)
• Ipsilateral temporoparietal neuralgia (involvement of CN V)
• Palatal paralysis (CN X)
• The findings of Trotter’s triad in nasopharyngeal carcinomas are labeled in the following image
• Painless lymphadenopathy
• Obstruction of the Eustachian tube, which causes recurrent otitis media and conductive hearing loss
• Nasal discharge
• Nose-bleeds
• Impaired nasal breathing
• Conductive deafness (eustachian tube blockage)
• Ipsilateral temporoparietal neuralgia (involvement of CN V)
• Palatal paralysis (CN X)

• The diagnostic work-up includes a CT and MRI scan of the head and neck and a biopsy.

Page 11

1696
• The treatment is aimed at surgical excision of the tumor and concurrent platinum-based
chemoradiotherapy.
Option A: Angiofibroma
• Angiofibroma is a benign tumour composed of fibrous tissue, connective tissue, and blood vessels.
• The tumour involves the face and appears as reddish nodules in a butterfly around the nose and
cheeks (acne-like appearance). Associated with tuberous sclerosis.
• These features are not mentioned in the clinical vignette making this diagnosis less likely.
Option C: Laryngeal carcinoma
• Laryngeal carcinoma is a malignant tumour of the larynx. It occurs most commonly in older men.
• Risk factors include smoking and alcohol consumption.
• Furthermore, the vignette is about nasopharyngeal growth and not laryngeal growth.
Option D: Squamous cell carcinoma of the tongue
• This is not the likely diagnosis, as this vignette is about the growth arising from the nasopharynx
andnot the tongue.

Solution for Question 6:


Option A: Excision of tumor
• Carotid body tumors are benign, slow-growing neck swellings in the carotid triangle. They arise from
the chemoreceptor cells of the carotid body and are also known as carotid body paraganglioma.
• The Shamblin classification is widely used to classify carotid body tumors. The Shamblin classification
system is summarized as follows: Group I – Includes localized tumors not involving the carotid vessels.
Group II – Includes the tumors partially surrounding the carotid vessels. Group III – Includes the larger
tumors, which completely encase the carotid vessels, as illustrated in the image below:
• Group I – Includes localized tumors not involving the carotid vessels.
• Group II – Includes the tumors partially surrounding the carotid vessels.
• Group III – Includes the larger tumors, which completely encase the carotid vessels, as illustrated in
the image below:
• Group I – Includes localized tumors not involving the carotid vessels.
• Group II – Includes the tumors partially surrounding the carotid vessels.
• Group III – Includes the larger tumors, which completely encase the carotid vessels, as illustrated in
the image below:

Page 12

1697
• Carotid body tumors may be functional and secrete catecholamines.
• Typically seen after the age of 40 years the clinical features of carotid body tumors include: Pain
Hoarseness Dysphagia Horner syndrome Shoulder drop Tongue paralysis
• Pain
• Hoarseness
• Dysphagia
• Horner syndrome
• Shoulder drop
• Tongue paralysis
• The diagnostic work-up includes CT and MRI scans of the head and neck and a biopsy. Carotid body
tumors cause characteristic splaying of the internal and external carotid arteries, known as the 'Lyre
Sign'. It is illustrated in the image below:

• Surgical excision is the treatment of choice. Preoperative embolization of feeder vessels, assessed
via arteriography, may help reduce intraoperative blood loss. Pre-operative Embolization for Tumors >
3cm Excision with Carotid Artery Replacement for Tumors > 5cm
• Pre-operative Embolization for Tumors > 3cm
• Excision with Carotid Artery Replacement for Tumors > 5cm
• The larger the tumor, the higher the risk of operative complications.
• Pain
• Hoarseness
• Dysphagia
• Horner syndrome
• Shoulder drop
• Tongue paralysis

Page 13

1698
• Pre-operative Embolization for Tumors > 3cm
• Excision with Carotid Artery Replacement for Tumors > 5cm
Option B: Radiotherapy
• In patients with the risk of complications precluding surgery, radiotherapy may be considered.
However, the first-line treatment option is the surgical excision of the tumor.
• Radiotherapy will not cure the tumor but can prevent further growth.
Option C: Chemotherapy
• Chemotherapy is not typically used as a primary treatment for carotid body tumors due to their
relatively low chemosensitivity. However, it may be considered in cases of metastatic or recurrent
disease, particularly in conjunction with other treatment modalities.
Option D: Carotid artery ligation, both proximal and distal to the tumor
• Carotid artery ligation, both proximal and distal to the tumor, is a treatment option that may be
considered in select cases of carotid body tumors, particularly when complete surgical resection is not
feasible or carries significant risks. This approach involves ligating the carotid artery both upstream and
downstream to the tumor to interrupt blood flow to the tumor and induce tumor ischemia and shrinkage.

Solution for Question 7:


Option C: Failure of involution of the thyroglossal duct
• This patient is most likely suffering from a thyroglossal duct cyst.
• A thyroglossal duct cyst is an embryologic remnant that forms due to the failure of closure of the
thyroglossal duct extending from the foramen cecum in the tongue to the thyroid’s location in the neck.
• A thyroglossal duct cyst usually manifests as a swelling in the midline of the neck in the line of thyroid
descent, as illustrated in the image below:

Page 14

1699
• Thyroglossal cysts may become infected and rupture onto the skin of the neck, presenting as a
discharging sinus.
• The diagnostic work-up includes USG. The biopsy is usually not indicated.
• A draining fistula should be excised along with the thyroglossal tract up to the base of the tongue.
INCORRECT OPTIONS:
Options B, C and D are incorrect.

Solution for Question 8:


Option D: Posterior triangle nodes
• The level V lymph nodes include posterior triangular cervical lymph nodes.
• The image below illustrates the various classes of cervical lymph nodes:

Page 15

1700
Option A: Upper jugular nodes
• The level II lymph nodes include upper jugular cervical lymph nodes.
Option B: Middle jugular nodes
• The level III lymph nodes include middle jugular cervical lymph nodes
Option C: Lower jugular nodes
• The level IV lymph nodes include lower jugular nodes and cervical lymph nodes.

Solution for Question 9:


Option D: Normal appearance of lips, nose, and face
• This patient is suffering from a cleft palate. Cleft lip (CL) and cleft palate (CP) are the most common
congenital deformities of the head and neck region.
• Usually, it is caused by a combination of genetic factors and teratogens such as nicotine, alcohol, and
antiepileptic drugs.
• The pathogenesis involves the arrest of the fusion of the facial processes. Typically, failure of fusion of
the maxillary prominence with the medial nasal prominence causes cleft lip (CL). However, cleft palate
(CP) is caused by the failure of fusion of the palatine prominences.
• Cleft lip and palate can be classified as unilateral or bilateral and complete or incomplete.
• The clinical features include: Facial deformity. Feeding, hearing, and speech difficulties. Defects in the
dentition.
• Facial deformity.
• Feeding, hearing, and speech difficulties.
• Defects in the dentition.

Page 16

1701
• The management of these cases involves special feeding techniques such as feeding in an upright
position and special feeding bottles since they cannot effectively create negative sucking pressure.
• Treatment should be started as early as possible and may involve an interdisciplinary approach
• Nasoalveolar molding and lip taping are non-surgical techniques used to decrease the size of the
lip/palatal defects and should be initiated early (at 2 weeks of age).
• The surgical repair of the cleft lip is done at 3 months of age and of the cleft palate at 6 months of age
to optimize feeding and speech development without interrupting normal maxillofacial growth. The
surgical treatment aims to restore the normal appearance of lips, nose, and face.
• However, restoring the normal appearance of lips, nose, and face is not the main aim of the surgical
correction of the cleft palate.
• Facial deformity.
• Feeding, hearing, and speech difficulties.
• Defects in the dentition.
Option A: Control of regurgitation
• Control of regurgitation is the primary aim of the surgical correction of the cleft palate.
Option B: To promote normal dentition and facial growth
• This is the primary aim of the surgical correction of the cleft palate.
Option C: To reduce the frequency of paranasal sinuses infections
• This is the primary aim of the surgical correction of the cleft palate.

Solution for Question 10:


Option C: Metastasis in a lymph nodes more than 6 cm
• In N3 lymph nodes stage as per TNM staging of the nodes involved at least one has to be ≥6cm
• The N staging for laryngeal cancer, according to the American Joint Committee on Cancer (AJCC), is
summarized in the table below:
AJCC N Classification
Definition
N0
No regional LN metastasis
N1
Single ipsilateral node ≤3cm in greatest diameter
N2a
Single ipsilateral node >3cm but ≤6cm
N2b
Multiple ipsilateral nodes ≤6cm in greatest diameter
N2c

Page 17

1702
Bilateral or Contralateral nodes ≤6cm in greatest diameter
N3
Any positive nodes, at least one ≥6cm

• Treatment is aimed at the surgical excision of the tumour.


Option A: Metastasis in lymph nodes more than 2 cm
• This statement describes an N stage of N1 and not N3.
Option B: Metastasis in lymph nodes more than 5cm
• This statement describes an N stage of N2 and not N3.
Option D: No regional lymph node involvement
• This statement describes an N stage of N0 and not N3.

Solution for Question 11:


Option C: Auriculotemporal nerve
• This patient who developed sweating and flushing of the cheek while eating after the parotid surgery
is most likely suffering from Frey's syndrome.
• The clinical features include sweating and erythema in the parotid region when tasting food or the
smell of food. It can be caused by surgical injury and subsequent regeneration of parasympathetic
nerve fibers that transmit gustatory stimuli.
• The newly developed fibers attach to the sympathetic fibers responsible for innervating the sweat
glands and cutaneous blood vessels.
• The injured parasympathetic fibers arise from the auriculotemporal nerve, which is a branch of
mandibular nerve V3, as shown in the image below:

Page 18

1703
• The diagnostic workup involves a detailed clinical history, examination, and a minor starch-iodine test,
a confirmatory test.
• Management Involves: Anti-perspirant application Botox injection Tympanic neurectomy for
non-responding cases
• Anti-perspirant application
• Botox injection
• Tympanic neurectomy for non-responding cases
• Anti-perspirant application
• Botox injection
• Tympanic neurectomy for non-responding cases
Option A: Facial nerve
• The fibres responsible for this manifestation arise from the auriculotemporal nerve, a branch of
mandibular nerve V3, and not from the facial nerve.
Option B: Trigeminal nerve
• The fibres responsible for this manifestation arise from the auriculotemporal nerve, which is a branch
of mandibular nerve V3, and not from the trigeminal nerve
Option D: Glossopharyngeal nerve
• The fibres responsible for this manifestation arise from the auriculotemporal nerve, a branch of
mandibular nerve V3, and not from the glossopharyngeal nerve.

Solution for Question 12:


Option A: Dermoid cyst
• The patient most likely suffers from a mature cystic teratoma or dermoid cyst.

Page 19

1704
• A dermoid cyst usually occurs on the head, neck, spine, cranium, and abdomen.
• Further, mature cystic teratoma is the most common ovarian tumour in females.
• The dermoid cysts typically become symptomatic as they grow in size.
• It usually manifests as a small, painless lump in the midline of the neck.
• Ultrasound of the neck is helpful in the diagnostic workup, and the treatment is aimed at complete
surgical excision of the cyst.
Option B: Sebaceous cyst
• A sebaceous cyst is a benign, sebum-containing lesion caused by a blockage of a sebaceous gland.
• Typically asymptomatic, but sometimes it can get infected.
• Treatment options include intralesional steroids and surgical excision.
• In contrast to the dermoid cyst, the sebaceous cyst typically occurs in: Back Scalp → No punctum
Scrotum
• Back
• Scalp → No punctum
• Scrotum
• Back
• Scalp → No punctum
• Scrotum
Option C: Branchial cyst
• A branchial cyst usually occurs in the lateral aspect of the neck anterior to the sternocleidomastoid.
However, it does not occur in the midline of the neck.
Option D: Metastatic tumours in the neck
• The malignancies can metastasize to the lymph nodes in the cervical region, and the lymph nodes
appear enlarged. However, this patient's history and examination suggest no malignancy. So this
option can be ruled out.

Solution for Question 13:


Option C: Tuberculous lymphadenitis
• This patient is most likely suffering from tuberculous lymphadenitis, a granulomatous inflammation
that results in necrosis of lymph nodes.
• Tuberculous lymphadenitis is among the most frequent extrapulmonary tuberculosis (TB)
presentations. Tuberculous lymphadenitis in the cervical region is known as scrofula.
• The patient displays signs of chronic non-tender cervical lymphadenopathy and fever. During the
physical examination, a firm and discrete mass or matted nodes that are fixed to surrounding structures
are observed. The overlying skin may be indurated, and there may or may not be discharging sinus
tracts, as depicted in the image below:

Page 20

1705
• Tuberculous lymphadenitis commonly involves the cervical and supraclavicular lymph node groups.
• The diagnostic workup for tuberculous lymphadenitis involves fine-needle aspiration or excisional
(FNAC) and biopsy with the histopathologic examination and culture for mycobacteria.
• The treatment is antituberculosis therapy.
Option A: Branchial cyst
• A congenital neck mass arises if the embryological second branchial cleft or cervical sinus fails to
obliterate. A branchial cyst typically presents as a painless mass lateral to the midline, usually anterior
to the sternocleidomastoid muscle, which does not move with swallowing. These features are not
present in this clinical vignette.
Option B: Metastatic tumour in the neck
• The malignancies can metastasize to the lymph nodes in the cervical region, and the lymph nodes
appear enlarged. However, this patient's history and examination suggest no malignancy. Furthermore,
the pulmonary infiltrate also suggests tuberculosis, so this option can be ruled out.
Option D: Lymphoma
• Most patients with lymphoma present with asymptomatic lymphadenopathy or a mass on a chest
radiograph.
• Constitutional symptoms such as fever, night sweats, or unintended weight loss will be seen in
lymphoma.
• However, the presence of pulmonary infiltrates and draining cervical sinus suggest tuberculous
lymphadenitis as a likely aetiology.

Solution for Question 14:


Option D: All of the above
• The diagnostic swelling suggests cystic hygroma, which is associated with Turner's syndrome,
Down's, and Klinefelter's syndrome.

Page 21

1706
• Cystic hygroma is a multiloculated cystic space lined by endothelial cells.
• It occurs due to the sequestration of a portion of the jugular lymph sac from the lymphatic system.
• Cysts are filled with clear lymph and are lined by endothelium.
• The most common site is the posterior neck region (Posterior triangle of the neck), as shown in the
image below:

Other options
Options A, B and C are associated with Cystic hygroma.

Solution for Question 15:


Option A: Shamblin classification
• Carotid body tumor is also known as chemodectoma.
• The Shamblin classification is widely used to classify carotid body tumors. According to this
classification system, the carotid body tumors are classified with the carotid vessels. The Shamblin
classification system is summarized as follows: Group I – Includes localized tumors not involving the
carotid vessels. Group II – Includes the tumors partially surrounding the carotid vessels. Group III –
Includes the larger tumors, which completely encase the carotid vessels.
• Group I – Includes localized tumors not involving the carotid vessels.
• Group II – Includes the tumors partially surrounding the carotid vessels.
• Group III – Includes the larger tumors, which completely encase the carotid vessels.
• This classification system is illustrated in the image below:
• Group I – Includes localized tumors not involving the carotid vessels.

Page 22

1707
• Group II – Includes the tumors partially surrounding the carotid vessels.
• Group III – Includes the larger tumors, which completely encase the carotid vessels.

Option B: DeBakey classification


• It is a system of classification of thoracic aortic dissection and not for classifying carotid body tumors.
Option C: Stanford classification
• Stanford classification is also a classification system of thoracic aortic dissection.
Option D: Montreal classification
• It is a system of classifying Crohn's disease and not for classifying carotid body tumors.

Solution for Question 16:


Option C: The upper third of the anteromedial border of the sternocleidomastoid
• Branchial cyst is caused by remnants of the embryological second branchial cleft or cervical sinus and
incomplete obliteration of branchial clefts.
• The clinical manifestations include the presentation in late childhood or adulthood after a previously
undiagnosed cyst becomes infected.
• It usually presents with a painless, firm mass that does not move with swallowing.
• Most commonly, it is located lateral to the midline and presents on the upper neck in early to
mid-adulthood.
• It is located on its anterior border at the junction of the sternocleidomastoid muscle's upper third and
middle third.
• Fistula is present on the lower one-third on the anterior side of sternocleidomastoid muscle.
• A small draining opening is usually seen if a fistula is present, as shown in the image below:

Page 23

1708
• Diagnostics workup includes an ultrasound of the neck
• Treatment is aimed at the surgical excision of the cyst.
• If not managed properly, it can be complicated by abscess formation.
INCORRECT OPTIONS:
Options A, B, and D are incorrect.

Solution for Question 17:


Option B: Removal of level I-IV lymph nodes.
• This patient is most likely suffering from mucoepidermoid carcinoma of the right-sided parotid gland
with signs of invasion such as lymphadenopathy and facial nerve palsy.
• Treating malignant tumours of the parotid gland involves surgical excision of the tumour with cervical
dissection.
• The cervical dissection has the following subtypes: Selective neck dissection Modified radical neck
dissection Radical neck dissection
• Selective neck dissection
• Modified radical neck dissection
• Radical neck dissection
• Selective neck dissection is characterized by the excision of the cervical lymph nodes with
preservation of one or more of the lymph node groups routinely removed in the radical neck dissection.
• The difference between modified radical neck dissection and radical neck dissection is based on the
extent of tissue and the number of lymph nodes removed.
• Each type of neck dissection depends upon the extent of the spread of cancer.
• In the case of radical neck dissection, all the tissue on the side of the neck from the jawbone to the
collarbone is removed. The muscle, nerve, salivary gland, and major blood vessels in this area are all

Page 24

1709
removed.
• The modified radical neck dissection is the excision of all lymph nodes routinely removed in a radical
neck dissection but with preservation of one or more non-lymphatic structures such as the spinal
accessory nerve, sternocleidomastoid, and internal jugular vein.
• Selective neck dissection
• Modified radical neck dissection
• Radical neck dissection
Option A: Removal of level I-III lymph nodes
• This option describes the suprahyoid neck dissection and not the extended suprahyoid neck
dissection.
Option C: Removal of level II-V lymph node groups + suboccipital lymph node groups +
retro auricular lymph nodes
• This option describes the posterolateral neck dissection, not the extended suprahyoid neck dissection.
Option D: Removal of level II-V lymph nodes + internal jugular vein
• This option describes the lateral neck dissection, not the extended suprahyoid neck dissection.

Page 25

1710
Facial Injuries and Abnormalities
1. A 55-year-old man presents to the outpatient department with complaints of swelling on the left side
of his face for two years and pain while chewing food for three months. The patient reports that the
swelling was insidious at onset and gradually increased to the present size. On examination, there is a
solitary ill-defined diffuse swelling over the left middle and lower third of the face with a smooth surface,
overlying stretched skin, and no secondary changes. It is non-tender and hard to palpate.
Histopathology reveals islands of odontogenic epithelium in fibrous connective tissue. Based on the
history and findings, what is the most likely diagnosis for this patient?
(or)
What is the most likely diagnosis for a patient with swelling on the left side of his face for, pain while
chewing food, and histopathology revealing islands of odontogenic epithelium in fibrous connective
tissue?
A. Odontogenic Keratocyst
B. Central Giant Cell Granuloma
C. Ameloblastoma
D. Brown's Tumor of Hyperparathyroidism
----------------------------------------
2. What is the indication for adjuvant radiotherapy in a patient with squamous cell carcinoma with
extranodal involvement?
(or)
A 60-year-old man presents to the outpatient department with complaints of a burning sensation and
paralysis on the right side of the tongue for six months. He is a heavy smoker, a retired factory worker.
Magnetic resonance imaging (MRI) revealed a 37 ×23 mm mass under the right side of the floor of the
mouth and an enlargement of lymph nodes >3 cm on both sides. Subsequently, a biopsy confirmed the
presence of squamous cell carcinoma with extranodal involvement. After his tumor resection, adjuvant
radiation therapy (RT) is advised. Which of the following is the indication for adjuvant radiotherapy in
this patient?
A. Lymph node >3 cm
B. Extranodal Involvement
C. >5 mm of Subglottic Invasion
D. Perineural Invasion
----------------------------------------
3. A 4-month-old female infant presents to the surgical outpatient department with a bilateral cleft lip
deformity known before birth by 3D ultrasound. She is ready for her initial cleft repair as she is four
months old. Subsequently, complete cleft lip repair surgery has been scheduled under general
anaesthesia. Which surgical technique can be used for bilateral cleft lip repair?
(or)
Which surgical technique can be used for the repair of bilateral cleft lip deformity in a 4-month-old
female infant?
A. Bardach's Method
B. Langenbeck Method

1711
C. Millard Method
D. Wardill Method
----------------------------------------
4. What is the best treatment option for a fractured mandible with an edentulous jaw in a patient with
radiography showing atrophic mandible with a left body fracture?
(or)
A 66-year-old female came to the emergency department (ER) with pain in the left chin area and a
partially edentulous jaw after a fall in the bathroom. Her radiographs show an atrophic mandible with a
left body fracture. Which of the following is the best treatment option for a fractured mandible with an
edentulous jaw?
A. External fixator
B. Minerva-plaster
C. Interdental wiring
D. Intermaxillary elastic traction
----------------------------------------
5. A 35-year-old man arrives at the emergency department with a head injury after being hit by a car
traveling at 50 miles per hour while riding his motorcycle. On examination, the Glasgow Coma
Scale(GCS) is 8, and there is a clear discharge from his left ear. Moreover, there is significant bruising
on his left temporal area and behind the left ear. What could be the likely cause of these findings?
(or)
What is the likely cause of a clear discharge from the left ear, significant bruising on the left temporal
area and behind the left ear, and a Glasgow Coma Scale (GCS) score of 8 in a patient following a head
injury in a road traffic accident?
A. Fracture of the Base of the Skull
B. Fracture of Mandible
C. Fracture of Maxilla
D. Fracture of Zygomatic
----------------------------------------
6. A 66-year-old male presents to the outpatient department with complaints of a burning sensation on
the right side of his cheek for six months. He is a heavy smoker, a retired factory worker. On
examination, no abnormalities other than an induration on the right side of his cheek were detected.
Magnetic resonance imaging (MRI) revealed a mass, and a biopsy confirmed the presence of
squamous cell carcinoma of the buccal mucosa. After his tumor resection, from which of the following
structures will the flap be used for his cheek reconstruction?
(or)
From which structure will the flap be used for cheek reconstruction in squamous cell carcinoma of the
buccal mucosa after tumour resection?
A. Local tongue
B. Cervical
C. Forehead

Page 2

1712
D. Pectoralis major myocutaneous
----------------------------------------
7. A 20-year-old man presents to the outpatient department with a complaint of a change in facial
shape and difficulty in mouth opening after being hit in the right side of the face by another individual's
knee during college sports activities. Immediately after the injury, he noticed a depression in the right
cheek and mild difficulty opening his mouth. On examination, there is a depression in his right cheek.
Which of the following investigations is necessary to find out if there is a fracture of the zygomatic
bone?
(or)
Which investigation will be done to confirm the zygomatic bone fracture?
A. Computerized Tomography
B. Magnetic Resonance Imaging
C. Ultrasonography
D. X-ray Scan
----------------------------------------
8. What is the most likely diagnosis associated with micrognathia, "U" shaped cleft palate, and
glossoptosis in a 6-day-old male neonate with feeding difficulty and respiratory distress?
(or)
A 6-day-old male neonate presents to the paediatrics department with feeding difficulty and respiratory
distress. He is born to a 22-year-old mother at a gestational age of 36 weeks with a low birth weight(2.2
kg). The mother complained of hyperemesis gravidarum in the first trimester, along with
oligohydramnios. On examination, the neonate has micrognathia, a "U"-shaped cleft palate, and
glossoptosis. What is the most likely diagnosis associated with cleft palate in this patient?
A. Down Syndrome
B. Treacher-Collins' Syndrome
C. Pierre Robin Sequence
D. Spina Bifida
----------------------------------------
9. A 52-year-old female patient presents to the outpatient department with a chief complaint of pain on
the right-side temporomandibular joint (TMJ) for one week. The patient has a history of falling a month
ago and hitting an object in the right-side TMJ region during the fall. On examination, there is
tenderness of the right-side TMJ region and a slight restriction of mouth opening; however, no deviation
of the mandible is observed, and no other significant findings were observed in the oral cavity. Her
panoramic radiographic examination reveals a fracture of the mandible. What is the most common
location for a mandibular fracture?
(or)
What is the commonest site of mandibular fracture, in a patient with history of fall a month ago?
A. At the neck of the condyle
B. Through the angle
C. Through the canine fossa
D. At the middle

Page 3

1713
----------------------------------------
10. A 60-year-old male presents to the emergency department with complaints of nausea, vomiting,
and headache after a head injury due to accidental trauma caused by a hammer on his head while
working. On examination, there are forehead, nasal bridge, and upper lip injuries. However, his eye
movements and vision are normal bilaterally. His computed tomography (CT) scan of the facial bones
shows Le fort II fracture. What are the most likely structures in this patient that have been damaged?
(or)
What are the most likely structures damaged in Le Fort II fracture of the facial bones?
A. Fracture Line Runs Above and Parallel to the Palate
B. Fracture Line Runs High Through the Nasal Bridge, Septum, And Ethmoids
C. Root of Nose, Lacrimal Bone, Orbital Floor, And Pterygoid Plate
D. Complete Disjunction of The Facial Skeleton from The Skull Base
----------------------------------------
11. What is the optimal timing for repairing a unilateral cleft lip deformity in an infant?
(or)
A 10-day-old female infant presents to the surgical outpatient department with a right unilateral cleft lip
deformity known before birth by 3D ultrasound. Her parents are concerned about her cleft lip repair
surgery. What is the best time to repair the infant's unilateral cleft lip?
A. 4-5 months
B. 5-6 months
C. 6-9 months
D. 15-18 months
----------------------------------------

Correct Answers
Question Correct Answer

Question 1 3
Question 2 2
Question 3 3
Question 4 1
Question 5 1
Question 6 4
Question 7 1
Question 8 3
Question 9 1
Question 10 3
Question 11 2

Page 4

1714
Solution for Question 1:
Option C: Ameloblastoma
• Ameloblastoma is the most common odontogenic tumor.
• Ameloblastoma is a rare, noncancerous (benign) tumor that often develops in the jaw near the
molars.
• Most ameloblastomas occur in the mandible near the angle and its ramus
• Ameloblastoma begins in the cells that form the protective enamel lining the teeth. Rarely, it may start
in the gum tissue.
• The exact cause of the tumor is unclear, but several genetic changes (mutations) may be involved in
the development of ameloblastoma.
• Ameloblastoma often causes no symptoms, but signs and symptoms may include pain and a lump or
swelling in the jaw.
• If left untreated, the tumor can grow very large, distorting the shape of the lower face and jaw..
• Treatment may include surgery and radiation. In some cases, reconstruction may be necessary to
restore the teeth, jaw, and facial appearance.

Page 5

1715
• Axial Computerized Tomography Scan(CT) showing a multilocular radiolucent lesion involving the left
body and ramus of the mandible.
Option A: Odontogenic Keratocyst
• Odontogenic keratocysts are developmental odontogenic cysts of epithelial origin.
• General presentation is swelling, with or without pain.
• Radiographically, they present as a well-defined radiolucent lesion with smooth and usually corticated
margins unless they have been secondarily infected.
Option B: Central Giant Cell Granuloma
• Central giant cell granuloma is an intraosseous lesion (osteoclast-like giant cells within connective
tissue stroma) characterized by the following: Multiple foci of haemorrhage Aggregations of
multinucleated giant cells Trabeculae of woven bone
• Multiple foci of haemorrhage
• Aggregations of multinucleated giant cells
• Trabeculae of woven bone
• The etiology is unknown. However, it may be related to trauma, inflammatory foci, or genetic
predisposition.
• These include giant cell granulomas, which may be central if they lie within the jawbone or peripheral
if they lie within the soft tissue.
• The clinical presentation varies from a red or blue, soft, painless, asymptomatic swelling to a painful,
aggressive lesion-producing local osseous destruction, root resorption, tooth displacement, or cortical
plate perforation with a tendency for recurrence.
• Multiple foci of haemorrhage
• Aggregations of multinucleated giant cells
• Trabeculae of woven bone
Option D: Brown's Tumor of Hyperparathyroidism

Page 6

1716
• Brown tumors are giant cell focal lesions resulting from abnormal bone metabolism in patients with
hyperparathyroidism.
• The lesions localize in areas of extensive bone resorption, replaced by fibrovascular tissue and giant
cells with abundant hemorrhage and deposits of hemosiderin.
• Clinically, brown tumors may present as tiny, asymptomatic swelling in the jawbone or as a painful
exophytic mass.
• Histologically, brown tumors are characterized by vascular fibroblastic stroma and several
osteoclast-like multinucleated giant cells often interspersed with hemorrhagic infiltrates and
hemosiderin deposits.

Solution for Question 2:


Option B: Extranodal Involvement
• Indications for postoperative radiation therapy (adjuvant radiation therapy) include the following:
Positive margins Invasion of the soft tissues of the neck Extranodal involvement Perineural invasion,
and More than 5 mm of subglottic invasion.
• Positive margins
• Invasion of the soft tissues of the neck
• Extranodal involvement
• Perineural invasion, and
• More than 5 mm of subglottic invasion.
• Head and Neck cancers are complex with multiple subsites that require different oncologic
approaches for the cure and, therefore, require a multidisciplinary approach.
• The advent of definitive and post-operative RT has significantly improved the outcomes of patients
with head and neck cancers.
• Surgery and radiotherapy are equally effective for the treatment of oral cancers.
• Small tumors can be managed by surgery or radiotherapy, while large and advanced tumors need
both.
• Surgery should be preferred when the tumor involves the bone, as radiotherapy is less effective in
controlling the disease.
• Similarly, large bulky tumors also need surgery followed by radiotherapy. If the cancer recurs after
radiotherapy, it should be treated by surgery.
• Positive margins
• Invasion of the soft tissues of the neck
• Extranodal involvement
• Perineural invasion, and
• More than 5 mm of subglottic invasion.
Option A: Lymph node >3 cm
• Lymph node >3 cm is not an indication for adjuvant radiotherapy in head-neck cancers.

Page 7

1717
Option C: >5 mm of Subglottic Invasion
• It is an indication of adjuvant radiotherapy in head-neck cancers.
• The case discussed above has no subglottic invasion.
Option D: Perineural Invasion
• Perineural invasion (PNI) is the process of neoplastic invasion of nerves.
• It is an indication of adjuvant radiotherapy in head and neck cancers.
• The case discussed above has no perineural invasion.

Solution for Question 3:


Option C: Millard method
• A bilateral cleft lip is a common congenital disability in which a baby's lip does not form properly,
resulting in splits (clefts) on both sides of the lip.

• Bilateral complete cleft lip, previously treated with nasoalveolar molding, is repaired with the
Millard-Mulliken technique, which employs the orbicularis oris muscle reconstruction by advancing
bilateral muscular segments.
• The Millard bilateral cleft lip repair technique provides a philtrum of proper width, a lip of appropriate
height, a full-depth labial sulcus, and complete muscle continuity.
• Transposition of pro-labium is not required in the definitive lip repair into the nose floor and permits
subsequent columellar construction.
• The operation meets all the requirements of a good technique for lip repair and has become popular
for repairing primary clefts of the lip.
Option A: Bardach's Method
• This is a modification of the von Langenbeck technique in which the incision is made along the cleft
and alveolar margins.

Page 8

1718
• These are joined anteriorly to free the mucoperiosteal flaps.
• These flaps are based on the greater palatine vessels.
• The soft palate is repaired in a straight line.
• The levator palatini muscle dissection and reconstruction of the muscle sling are performed as in
intravelar veloplasty.
Option B: Langenbeck Method
• It is a palatoplasty method using mucoperiosteal flaps to repair the hard palate region.
Option D: Wardill Method
• Wardill's method is used in cleft palate surgery.
• Until a few years back, this procedure was the commonest palatoplasty technique.
• In this technique, the V-Y procedure is performed so that the whole mucoperiosteal flap and the soft
palate are retroposed and the palate is lengthened.
• However, it leaves an extensive raw area anteriorly and laterally along the alveolar margin with
exposed bare membranous bone.
• The raw area heals with secondary intention.
• This causes shortening of the palate and results in velopharyngeal incompetence.
• The raw area adjacent to the alveolar margin also results in alveolar arch deformity and dental
malalignment.

Solution for Question 4:


Option A: External fixator
• Most common site of mandibular fracture is condylar neck which is the weakest part of the mandible
• Fracture mandible with an edentulous jaw is best treated with an external fixator.
• The restoration of the tooth body occlusion forms the basis for managing the fracture.
• The medical treatment of mandibular fractures includes : Pure diet Interdental fixation lasting several
weeks 1% chlorhexidine mouthwash Antibiotics
• Pure diet
• Interdental fixation lasting several weeks
• 1% chlorhexidine mouthwash
• Antibiotics
• Pure diet
• Interdental fixation lasting several weeks
• 1% chlorhexidine mouthwash
• Antibiotics
Option B: Minerva-plaster

Page 9

1719
• It is indicated for the following: Jefferson's fracture without transverse ligament injury Light wedge
fractures following traction Occipital post-op fractures Post-trauma Arthritis Acute pain Spinal stenosis
• Jefferson's fracture without transverse ligament injury
• Light wedge fractures following traction
• Occipital post-op fractures
• Post-trauma
• Arthritis
• Acute pain
• Spinal stenosis
• Jefferson's fracture without transverse ligament injury
• Light wedge fractures following traction
• Occipital post-op fractures
• Post-trauma
• Arthritis
• Acute pain
• Spinal stenosis
Option C: Interdental wiring
• Typical indications for its use are the following: Conservative management of minimally displaced
fractures Maxillofacial fractures in deep bite cases Stabilization of fracture during open reduction and
internal fixation Orthognathic surgeries and tumor resection surgeries
• Conservative management of minimally displaced fractures
• Maxillofacial fractures in deep bite cases
• Stabilization of fracture during open reduction and internal fixation
• Orthognathic surgeries and tumor resection surgeries
• Conservative management of minimally displaced fractures
• Maxillofacial fractures in deep bite cases
• Stabilization of fracture during open reduction and internal fixation
• Orthognathic surgeries and tumor resection surgeries
Option D: Intermaxillary elastic traction
• Intermaxillary fixation is required before fracture exposure and plating.
• Condylar and subcondylar fractures of the mandible are usually treated with it.

Solution for Question 5:


Option A: Fracture of the Base of the Skull

Page 10

1720
• Fracture of the base of the skull typically involves the temporal bone, occipital bone, sphenoid bone,
and ethmoid bone.
• Basilar skull fractures can lead to various clinical features, such as altered mental status, nausea,
vomiting, oculomotor deficits, and cerebrospinal fluid (CSF) leak.
• In this case, the fracture involves the posterior fossa, which may also result in cervical spine injury,
vertebral artery injury, and damage to lower cranial nerves.
• It can present as the following signs: CSF otorrhea: Fluid may accumulate in the middle ear space
and dribble through a perforated eardrum or into the nasopharynx via the Eustachian tube. CSF
rhinorrhea: CSF may also drip from the nose in fractures of the anterior skull base.
• CSF otorrhea: Fluid may accumulate in the middle ear space and dribble through a perforated
eardrum or into the nasopharynx via the Eustachian tube.
• CSF rhinorrhea: CSF may also drip from the nose in fractures of the anterior skull base.
• CT scans obtained during trauma evaluation showed a basilar skull fracture of the occipital bone, as
indicated by the circle in both the axial and coronal views.This type of fracture involves the base of the
skull, often associated with significant trauma.
• CSF otorrhea: Fluid may accumulate in the middle ear space and dribble through a perforated
eardrum or into the nasopharynx via the Eustachian tube.
• CSF rhinorrhea: CSF may also drip from the nose in fractures of the anterior skull base.
Option B: Fracture of Mandible
• Jaw fracture symptoms include pain, swelling, redness, and increased heat in the jaw or ear area.
• Difficulties speaking, chewing, and breathing. Numbness or bruising of your face and neck.
Option C: Fracture of Maxilla
• Maxillary fractures, particularly Le-fort fractures, are also associated with significant visual problems,
diplopia, and epiphora.
• Other common complications include infection, trismus, facial nerve deficits, malunion, and
asymmetry.
Option D: Fracture of Zygomatic
• The main clinical features of complex zygomatic fractures include diplopia, enophthalmos,
subconjunctival ecchymosis, and extraocular muscle entrapment.
• Cosmetic deformity with depression of the malar eminence, facial widening, malocclusion, and
neurosensory disturbances of the infraorbital nerve may be seen.

Solution for Question 6:


Option D: Pectoralis major myocutaneous
• The most common histological type of oral cancer is squamous cell carcinoma.
• Oral cancer often presents at an advanced stage, requiring extensive resection and complex
reconstruction, including free tissue transfers.
• The pectoralis major myocutaneous (PMMC) flap is used for lining, and a second regional flap is used
for cover.

Page 11

1721
• It is a versatile flap with an excellent reach to the face, oral cavity, and neck region.
• Owing to its versatility, easy learning curve, and constant vascular pedicle, the PMMC flap is still one
of the most favored approaches for head and neck reconstruction with acceptable cosmetic and
functional outcomes.
• Illustration showing the flap design (A) and orientation of flap for inset (B and C). Distal skin paddle
(D) coloured in green and proximal skin paddle (P) in blue; areas de-epithelialised before inset coloured
grey with cross-hatching

• Immediate post-operative image of the patient with cheek reconstruction.


Option A: Local Tongue
• Local tongue flap is a technique to restore a fully functioning tongue following partial
hemiglossectomy.
Option B: Cervical

Page 12

1722
• The cervicofacial flap has a broad pedicle and can be employed to cover significant anterolateral
craniofacial defects.
• As a thin flap, additional bulk may be provided with a pectoralis major muscle flap placed deep into
the cervicofacial flap.
• Indications of cervicofacial flap include: Superficial defects All zones of the cheek, external ear
Temporo-frontal and brow defects Orbital exenteration defects To provide skin cover in combination
with muscle flaps, e.g., pectoralis major and temporalis flaps.
• Superficial defects
• All zones of the cheek, external ear
• Temporo-frontal and brow defects
• Orbital exenteration defects
• To provide skin cover in combination with muscle flaps, e.g., pectoralis major and temporalis flaps.
• Superficial defects
• All zones of the cheek, external ear
• Temporo-frontal and brow defects
• Orbital exenteration defects
• To provide skin cover in combination with muscle flaps, e.g., pectoralis major and temporalis flaps.
Option C: Forehead
• The forehead flap is one of the surgical techniques used for nasal reconstruction.
• It is a gold standard method for nasal soft tissue reconstruction.
• The forehead flap provides the surgeon with a potent pedicle and a large amount of tissue to
reconstruct almost any defect.

Solution for Question 7:


Option A: Computerised Tomography
• Zygomatic bone fractures account for 20% of all facial bone fractures.
• An assault commonly produces zygomatic complex or Zygomatic arch fractures.
• Physical examination in a complex zygomatic fracture may reveal the following: Periorbital oedema,
Ecchymosis, Hematoma, Conjunctival haemorrhage, Enophthalmos, Paresthesia in the distribution of
the trigeminal nerve.
• Periorbital oedema,
• Ecchymosis,
• Hematoma,
• Conjunctival haemorrhage,
• Enophthalmos,
• Paresthesia in the distribution of the trigeminal nerve.

Page 13

1723
• CT imaging is the best option to diagnose the zygomatic fracture because of: Speed (especially with
"light speed" scanners) Superior clarity This technique provides more accurate diagnostic information.
• Speed (especially with "light speed" scanners)
• Superior clarity
• This technique provides more accurate diagnostic information.
• Only displaced fractures require treatment and the treatment of choice is Open reduction and internal
fixation.
• Periorbital oedema,
• Ecchymosis,
• Hematoma,
• Conjunctival haemorrhage,
• Enophthalmos,
• Paresthesia in the distribution of the trigeminal nerve.
• Speed (especially with "light speed" scanners)
• Superior clarity
• This technique provides more accurate diagnostic information.
Option B: Magnetic Resonance Imaging
• MRI uses a magnetic field to create a detailed picture of internal body soft-tissue structures.
• MRI is a non-invasive imaging technique.
• It does not involve radiation exposure, unlike X-ray and CT scans.
Option C: Ultrasonography
• Ultrasound uses high-frequency sound waves to produce real-time pictures or videos of soft tissues,
internal organs and blood vessels.
• Ultrasound scan is not used for mandibular fractures.
Option D: X-ray Scan
• X-ray water's view is the radiographic image obtained by passing a beam of X-rays through the chin at
an angle and is used especially to acquire diagnostic information about the bony structures of the front
of the head.
• It can be used to diagnose the fracture of the zygomatic bone, but still, a CT scan is a better choice
than an x-ray.

Page 14

1724
• Axial CT showing an isolated fracture of the left zygomatic arch.

Solution for Question 8:


Option C: Pierre Robin Sequence
• Pierre Robin syndrome (PRS) is a congenital condition in which the infant has: Micrognathia (small
jaw) Glossoptosis (a tongue that falls back toward the throat) Upper airway obstruction.
• Micrognathia (small jaw)
• Glossoptosis (a tongue that falls back toward the throat)
• Upper airway obstruction.
• Infants with the condition also commonly have a cleft palate.

Page 15

1725
• Although most lip and palate clefts occur as an isolated deformity, Pierre Robin Sequence remains
the most common syndrome.
• The most common symptoms may include the following: An underdeveloped jaw A tongue that's
more posteriorly positioned. Snoring and airway obstruction A high-arched palate Cleft palate Natal
teeth Dysphagia
• An underdeveloped jaw
• A tongue that's more posteriorly positioned.
• Snoring and airway obstruction
• A high-arched palate
• Cleft palate
• Natal teeth
• Dysphagia
• Micrognathia (small jaw)
• Glossoptosis (a tongue that falls back toward the throat)
• Upper airway obstruction.
• An underdeveloped jaw
• A tongue that's more posteriorly positioned.
• Snoring and airway obstruction
• A high-arched palate
• Cleft palate
• Natal teeth
• Dysphagia

Option A: Down Syndrome


• Down syndrome is a chromosomal abnormality caused by the presence of trisomy of chromosome 21.

Page 16

1726
• A cleft palate can also be associated with infants born with down's syndrome.
• Patients with downs syndrome present with the following: Decreased muscle tone Short neck Flat
face Small head, ears, and mouth Upward slanting eyes Epicanthal folds Wide, short hands
• Decreased muscle tone
• Short neck
• Flat face
• Small head, ears, and mouth
• Upward slanting eyes
• Epicanthal folds
• Wide, short hands
• Decreased muscle tone
• Short neck
• Flat face
• Small head, ears, and mouth
• Upward slanting eyes
• Epicanthal folds
• Wide, short hands
Option B: Treacher-Collins' Syndrome
• Treacher-Collins syndrome is a condition that affects the development of: Facial Bones Tissues of the
face
• Facial Bones
• Tissues of the face
• It is characterised by the following: Underdeveloped facial bones Micrognathia Cleft palate Eyelid
coloboma Hearing loss is caused by the underdevelopment of the ear canal
• Underdeveloped facial bones
• Micrognathia
• Cleft palate
• Eyelid coloboma
• Hearing loss is caused by the underdevelopment of the ear canal
• Facial Bones
• Tissues of the face
• Underdeveloped facial bones
• Micrognathia
• Cleft palate
• Eyelid coloboma
• Hearing loss is caused by the underdevelopment of the ear canal

Page 17

1727
Option D: Spina Bifida
• Spina Bifida is a type of neural tube defect in which a part of the neural tube does not develop or close
properly, leading to a defect in the spinal cord or the vertebrae of the fetus.
• Folic acid deficiency is one of the known risk factors for neural tube defects.
• Depending upon the type of defect, there are a few types of spina bifida: Spina bifida occulta,
Meningocele Myelomeningocele.
• Spina bifida occulta,
• Meningocele
• Myelomeningocele.
• Spina bifida occulta is the most common and the mildest form of spina bifida in which there is a
malformation of one or more vertebrae, leading to a small gap in the spine.
• Meningocele is a serious type of spina bifida where the meninges covering the spinal cord are pushed
out of the vertebrae.
• Myelomeningocele is a severe type of spina bifida in which a portion of the spinal cord or nerves are
pushed out of the vertebrae, which may or may not be covered with meninges.
• Spina bifida occulta,
• Meningocele
• Myelomeningocele.

Solution for Question 9:


Option A: At the neck of the condyle
Fracture of the mandible
• The condylar neck is the weakest part of the mandible and the most common site of fracture.
• The mandible may fracture directly at the point of the blow.
• Indirectly, where the force from the blow is transmitted, and the mandible fractures at a point of
weakness distant from the original blow, known as a 'Guardsman' fracture. Caused by a fall on the
midpoint of the chin resulting in fracture of the symphysis as well as both condyles.
• Caused by a fall on the midpoint of the chin resulting in fracture of the symphysis as well as both
condyles.
• Recommended radiographic evaluation of a mandible fracture is: Panoramic radiograph and Towne's
view X-ray
• Caused by a fall on the midpoint of the chin resulting in fracture of the symphysis as well as both
condyles.
Option B: Through the angle
• The bone grains, which are oriented vertically in the ramus, change their course at the angle of the
mandible as they enter the body of the mandible.
• This abrupt change in the bone grains also makes the bone weak.

Page 18

1728
• All these factors make the mandible vulnerable to fracture at this site.
Option C: Through the canine fossa
• In the canine area, the mandible is weaker as the canine has the longest and the strongest root, and it
occupies a lot of space in the bone and undermines and weakens it.
• Thus, the mandible has a tendency to fracture at this site.
Option D: At the middle
• Fractures that occur in the midline of the mandible are classified as symphyseal.
• When teeth are present, the fracture line passes between the mandibular central incisors.

Solution for Question 10:


Option C: Root of Nose, Lacrimal Bone, Orbital Floor, And Pterygoid Plate
• Le Fort II fractures transect the nasal bones, medial-anterior orbital walls, orbital floor, inferior orbital
rims and finally transversely fracture the posterior maxilla and pterygoid plates.
• Fracture line runs above and parallel to palate
• Effectively separates alveolus and palate from the facial skeleton above
• Pyramidal in shape
• Passes through the root of nose, lacrimal bone, floor of orbit, upper part of maxillary sinus, and
pterygoid plate
• Orbital floor is always involved
• Complete disjunction of the facial skeleton from the skull base
• Fracture line runs high through the nasal bridge, septum, and ethmoids and through the bones of orbit
to the frontozygomatic suture

Page 19

1729
Option A: Fracture Line Runs Above and Parallel to the Palate
• Le Fort I level fractures are essentially a separation of the hard palate from the upper maxilla due to a
transverse fracture running through the maxilla and pterygoid plates at a level just above the floor of the
nose.
Option B: Fracture Line Runs High Through the Nasal Bridge, Septum, And Ethmoids
• Le Fort III fractures result in craniofacial disjunction. This is the highest level Le Fort fracture and
essentially separates the maxilla from the skull base.
Option D: Complete Disjunction of The Facial Skeleton from The Skull Base
• Le Fort III fractures result in craniofacial disjunction. This is the highest level Le Fort fracture and
essentially separates the maxilla from the skull base.

Page 20

1730
Solution for Question 11:
Option B: 5-6 months
• Unilateral cleft lip is present when there is a gap on one side of the lip. Nine out of ten patients (90%)
with a cleft of the lip have a unilateral cleft lip.
• Unilateral cleft lip can cause problems with feeding and difficulty in speaking.. It is also associated
with frequent ear infections.
• Researchers believe that most cases of cleft lip and cleft palate are caused by an interaction of
genetic and environmental factors.
• A unilateral cleft lip is best repaired at 5-6 months.

• A unilateral cleft lip before and after repair.


• Soft palate at 6 months.
• Hard palate at 15-18 months.
• Cleft lip and soft palate at 5-6 months..
• Hard palate and gum pad with or without lip revision at 15-18 months.
• Cleft lip and soft palate at 4-5 months.
• Hard palate and gum pad with or without lip revision at 15-18 months.
Option A: 4-5 months
• Bilateral cleft lip is best repaired at 4-5 months.
Option C: 6-9 months
• Cleft palate (soft palate only) is repaired at 6 months.
Option D: 15-18 months
• Unilateral and bilateral hard palate and gum pad with or without lip revision at 15-18 months.

Page 21

1731
Previous Year Questions
1. What is the probable medical condition for a 45-year-old patient experiencing pain on one side of the
neck, who avoids eating due to aggravation of the pain? The provided ultrasound image of the salivary
glands will help determine the most likely diagnosis.

A. Sialolithiasis
B. Foreign body
C. Osteoma of the floor of the mouth
D. Cervical lymphadenopathy
----------------------------------------
2. What is the most common condition that chronic smokers are prone to developing? A 50-year-old
male, who is a chronic smoker, presented to the outpatient department (OPD) with symptoms of cough,
dyspnea, difficulty swallowing, and a palpable lump in the neck.
A. Adenocarcinoma
B. Squamous cell carcinoma
C. Large cell carcinoma
D. Adenoid cystic carcinoma
----------------------------------------
3. What should be the next course of action in managing a patient, aged 32, who has been chewing
tobacco chronically and presents with a whitish lesion in the gingivobuccal sulcus for the past seven
months?

A. Local excision and biopsy

1732
B. Avoid smoking; wait and watch
C. Steroidal injection
D. Sclerotherapy
----------------------------------------
4. Identify the condition looking at the given image of swelling in the floor of the mouth?
(or)
What is the correct diagnosis for the observed swelling in the floor of the mouth based on the provided
image?

A. Cystic hygroma
B. Ranula
C. Dermoid
D. Branchial cyst
----------------------------------------
5. Which of the following statements accurately describes thymoma?
A. Thymoma is the most common neoplasia of the thymus.
B. Thymoma is primarily located in the posterior mediastinum.
C. Chest X-ray is the investigation of choice for the diagnosis of thymoma.
D. Thymoma is asymptomatic.
----------------------------------------
6. Which of the subsequent conditions is linked with perineural invasion?
A. Adenoid cystic carcinoma
B. Pleomorphic adenoma
C. Mucoepidermoid tumor
D. Pancreatic cancer
----------------------------------------
7. What is depicted in the provided barium swallow film?

Page 2

1733
A. Aortic aneurysm
B. Pharyngeal pouch
C. Carotid body tumor
D. Esophageal carcinoma
----------------------------------------
8. Which of the following statements about cystic masses of the neck is false?
A. Type 3 branchial fistula has its internal orifice on cricopharynxgeus
B. Cystic hygroma is filled with clear lymph
C. Thyroglossal cyst moves upwards on swallowing and tongue protrusion
D. Fluctuation is positive in branchial
----------------------------------------
9. What is the most frequently observed location for a branchial cyst?
A. Junction of upper and middle third of the sternomastoid muscle
B. Junction of middle and lower third of the sternomastoid muscle
C. Middle of the sternomastoid muscle
D. Between the two heads of the sternomastoid muscle
----------------------------------------
10. The probable diagnosis for a 65-year-old man with an itchy lesion on his face, accompanied by
bleeding, is most likely to be:

Page 3

1734
A. Dermatofibrosarcoma
B. Marjolin’s ulcer
C. Basal cell carcinoma
D. Naevus
----------------------------------------
11. True about ranula are all except:
A. Mucous extravasation cyst
B. Arises from submandibular salivary gland
C. Bluish, translucent, cystic swelling in the floor of mouth
D. Marsupialization or excision is the treatment
----------------------------------------
12. What procedure is typically employed for managing recurrent Ranula in a 55-year-old male who
presented with a mass on the oral cavity floor, as depicted in the image, after undergoing a thorough
history and examination by the emergency physician?

A. Incision and drainage


B. Aspiration
C. Excision
D. Sclerosant injection
----------------------------------------
13. Comment on the diagnosis?

Page 4

1735
A. Sebaceous cyst
B. Implantation dermoid
C. Angular dermoid
D. Lipoma
----------------------------------------
14. Comment on the diagnosis?

A. Lipoma
B. Encephalocele
C. Cystic hygroma
D. Lymphadenopathy
----------------------------------------
15. Which of the following structures is most likely to be injured, leading to lower lip paralysis in a
30-year-old teacher who underwent parotidectomy?
A. Temporal branch of facial nerve
B. Cervical branch of facial nerve
C. Parotid duct
D. Mandibular branch of facial nerve
----------------------------------------
16. What is the primary source of brain metastasis that is most commonly observed?
A. Thyroid
B. Lung
C. Liver
D. Breast
----------------------------------------
17. Commonest site of branchial cyst:-
A. Lower 1/3 sternomastoid on anterior border
B. Lower 1/3rd sternomastoid on posterior border
C. Upper 1/3rd sternomastoid on anterior border

Page 5

1736
D. Upper 1/3rd sternomastoid on posterior border
----------------------------------------
18. The patient presents with painless swelling on the face and experiences difficulty while swallowing.
A visual representation of the patient's face is provided. The most likely diagnosis is:

A. Acute parotitis
B. Cancer of parotid gland
C. Angioedema of face
D. Acute sialadenitis
----------------------------------------
19. What is the clinical diagnosis for a 35-year-old woman who has a painless, fluctuating swelling with
no transillumination and a thin watery discharge that has been present for 6 months?

A. Brachial cyst
B. Secondaries
C. TB
D. Lymphoma
----------------------------------------
20. What is the probable diagnosis for a chronic alcoholic patient, aged 32, who complains of oral
ulcers and a burning sensation? A visual representation of the oral cavity is provided for reference.

Page 6

1737
A. Leukoplakia
B. Erythroplakia
C. Submucosal fibrosis
D. Malakoplakia
----------------------------------------

Correct Answers
Question Correct Answer

Question 1 1
Question 2 2
Question 3 1
Question 4 2
Question 5 1
Question 6 1
Question 7 2
Question 8 1
Question 9 1
Question 10 3
Question 11 2
Question 12 3
Question 13 1
Question 14 2
Question 15 4
Question 16 2
Question 17 4
Question 18 2
Question 19 3

Page 7

1738
Question 20 1

Solution for Question 1:


Correct option A:
• Sialolithiasis is a condition in which a stone or calcified mass forms within a salivary gland or its duct,
leading to obstruction of saliva flow and subsequent pain and swelling of the affected gland. This
condition is more common in the submandibular gland than the parotid gland, consistent with the
ultrasound image showing a stone in the submandibular gland.
Incorrect options:
Option B. Foreign body -
A foreign body in the salivary gland or duct can cause obstruction and inflammation, but it is a
relatively rare condition. The ultrasound image does not show any evidence of a foreign body.
Option C. Osteoma of the floor of the mouth - An osteoma is a benign tumor of bone tissue. It can occu
r in the head and neck bones, including the mandible and maxilla. However, an osteoma of the mouth f
loor would not cause neck pain or obstruction of the salivary gland, so this is an unlikely diagnosis bas
ed on the patient's symptoms.
Option D. Cervical lymphadenopathy - Cervical lymphadenopathy refers to enlarged lymph nodes in th
e neck, which can be caused by various conditions such as infection or cancer. However, this condition
would not cause pain with eating or obstruction of the salivary gland. Additionally, the ultrasound imag
e shows a salivary gland stone, not enlarged lymph nodes. Therefore, cervical lymphadenopathy is an
unlikely diagnosis in this case.

Solution for Question 2:


• In this case, a palpable lump in the neck suggests the possibility of head and neck cancer, which is
strongly associated with smoking.
• Squamous cell carcinoma is the most common type of head and neck cancer,
• It arises from the squamous cells that line the mucosal surfaces of the head and neck, including the
mouth, throat, and larynx.
• Risk factors include smoking, heavy alcohol consumption, and infection with certain types of human
papillomavirus (HPV).
Incorrect Choices:
• a. Adenocarcinoma is another type of cancer that can occur in the head and neck, but it is less
common than squamous cell carcinoma. It arises from the glandular cells that produce mucus and is
often associated with tobacco use and exposure to certain chemicals.
• c. Large cell carcinoma is a type of lung cancer, not head and neck cancer, that accounts for about
10-15% of all lung cancers. It is characterized by large, abnormal cells that grow and divide
uncontrollably.
• d. Adenoid cystic carcinoma is a rare type of cancer that can occur in several areas of the body,
including the head and neck. It arises from the secretory cells that line certain glands, such as the
salivary glands. It is not strongly associated with smoking, but its exact cause is unknown.

Page 8

1739
Solution for Question 3:
Local excision and biopsy: This option is the most appropriate choice in this scenario. A
whitish lesion on the gingivobuccal sulcus in a
chronic tobacco chewer could potentially be an oral pre-cancerous or cancerous lesion. Performing a
local excision (removal) of the lesion and obtaining a biopsy sample for pathological analysis is essenti
al to determine the nature of the lesion and guide further management.
Incorrect option explanation:
Option b. Avoid smoking; wait and watch: This option is not the best choice in this case. While quitting t
obacco use is always beneficial, a whitish lesion that has been present for 7 months warrants further e
valuation and investigation. Waiting and watching without taking any action may delay the diagnosis an
d appropriate treatment if the lesion is cancerous or pre-cancerous.
Option c. Steroidal injection: Steroidal injections are typically used for certain oral inflammatory conditio
ns, such as oral lichen planus. However, in the case of a chronic tobacco chewer with a
whitish lesion on the gingivobuccal sulcus, a steroidal injection would not be the next best step. It is im
portant to determine the nature of the lesion through a
biopsy before considering specific treatment options.
Option d. Sclerotherapy: Sclerotherapy is a
technique used for the treatment of certain vascular conditions by injecting a
sclerosing agent to induce vessel closure. It is not the appropriate management option for a whitish lesi
on on the gingivobuccal sulcus. Again, the first step in the management of this lesion should be to perf
orm a biopsy for a definitive diagnosis.

Solution for Question 4:


Correct Option B: Ranula
• A ranula is a specific condition characterized by a mucus retention cyst that forms in the floor of the
mouth. It is typically caused by the obstruction or rupture of the sublingual or submandibular gland
ducts, leading to the accumulation of mucus and the formation of a cystic swelling.
• Ranulas are often painless and have a characteristic appearance. They appear as soft, fluctuant
swellings in the floor of the mouth, usually on one side. The swelling may be bluish or translucent due
to the presence of accumulated mucus within the cyst. The size of the ranula can vary, and it may
increase or decrease in size over time.
Incorrect options:
Option A: Cystic hygroma: Cystic hygroma is a
congenital malformation of the lymphatic system. It typically presents as a
soft, cystic swelling in the neck region. It is not specific to the floor of the mouth.
Option C: Dermoid: Dermoid cysts are developmental cysts that contain a variety of tissues, including s
kin, hair follicles, and sebaceous glands. They can occur in various locations, including the floor of the
mouth, but they are not specific to this area.
Option D: Branchial cyst: Branchial cysts are congenital cysts that arise from remnants of the branchial
apparatus. They are typically located in the neck region and are not specific to the floor of the mouth.

Page 9

1740
Solution for Question 5:
Correct Option A: Thymoma is the most common neoplasia of the thymus.
• This statement is true. Thymoma is the most common primary neoplasm of the thymus gland,
accounting for the majority of thymic tumors. It typically arises from the epithelial cells of the thymus
and can have various clinical presentations and histological subtypes. Thymoma is commonly
associated with autoimmune disorders, such as myasthenia gravis.
Incorrect Options:
Option B: Thymoma is primarily located in the posterior mediastinum.
• This statement is incorrect. Thymoma arises in the anterior mediastinum, which is its typical location.
Option C: Chest X-ray is the investigation of choice for the diagnosis of thymoma.
• This statement is incorrect. Chest X-ray may show a mediastinal mass, but it is not the investigation of
choice for the diagnosis of thymoma. Computed tomography (CT) scan and magnetic resonance
imaging (MRI) are the preferred imaging modalities for the evaluation and diagnosis of thymoma.
Option D: Thymoma is asymptomatic.
This statement is incorrect. Due to the mass thymoma presents with symptoms like-
o Compression of Trachea: Dyspnea o Compression of Esophagus: Dysphagia
o Compression of recurrent laryngeal nerve: Hoarseness of voice (dysphonia)
o Compression of Superior vena Cava: Superior vena cava syndrome.

Solution for Question 6:


Correct Option A: Adenoid cystic carcinoma
• Perineural invasion refers to the spread of tumor cells along the nerves. It is commonly seen in certain
malignancies, and the correct option is (a) Adenoid cystic carcinoma.
• Adenoid cystic carcinoma is a malignant tumor that commonly involves salivary glands, particularly
the submandibular and minor salivary glands. It is characterized by the infiltration of tumor cells into the
surrounding nerves, leading to perineural invasion. This aggressive behavior of the tumor can result in
local recurrence and distant metastasis.
Incorrect options:
Option B: Pleomorphic adenoma: Pleomorphic adenoma is a most common benign tumorof the salivar
y glands. It typically does not exhibit perineural invasion unless it transforms into a malignant tumor. Ev
en if malignant change occurs, vascular invasion is more likely than perineural invasion
Option C: Mucoepidermoid tumor: Mucoepidermoid tumors are the most common malignant salivary gl
and tumors. While they can invade surrounding tissues, including nerves, perineural invasion is less co
mmon compared to adenoid cystic carcinoma.
Option D: Pancreatic cancer: Pancreatic cancer is a highly aggressive malignancy. It can invade nearb
y structures and spread to distant sites, but perineural invasion is not a
characteristic feature commonly associated with pancreatic cancer.

Page 10

1741
Solution for Question 7:
Correct Option B.
• The given barium swallow film shows a characteristic radiological finding known as a pharyngeal
pouch or Zenker's diverticulum.
• A pharyngeal pouch is an outpouching of the mucosa and submucosa through a weakness in the
posterior pharyngeal wall, just above the upper esophageal sphincter.
• It typically occurs in older individuals and is more common in males.
Incorrect options
Option A. Aortic aneurysm: A barium swallow film is not typically used to diagnose or visualize an aorti
c aneurysm. Imaging techniques such as ultrasound, CT scan, or MRI are preferred for diagnosing aort
ic aneurysms.
Option C. Carotid body tumor: A barium swallow film is not useful for visualizing or diagnosing carotid b
ody tumors. Imaging techniques such as ultrasound, CT scan, or MRI are more appropriate for evaluati
ng these tumors.
Option D. Esophageal carcinoma: Esophageal carcinoma may present with abnormal findings on a bari
um swallow, such as strictures or filling defects. However, the given barium swallow film does not show
typical features of esophageal carcinoma, such as irregular narrowing or focal lesions.

Solution for Question 8:


Correct Option A.
• A branchial fistula could be located unilateral or bilateral and assumed to represent a persistent
second branchial cleft.
• The external orifice is nearly always situated in the lower third of the neck near the anterior border of
the sternocleidomastoid, whilst the internal orifice is located on the anterior aspect of the posterior
faucial pillar just behind the tonsil.
• Type III– The tract courses between internal and external carotid arteries and may extend to the
lateral wall of the pharynx or skull base
Incorrect Options
Option B: Cystic hygroma is filled with clear lymph : Cystic hygroma /lymphangioma is a
birth defect with a large fluid-filled sac around the head and neck. The cyst comprises lymphatic tissue
s and white blood cells.
Option C: Thyroglossal cyst moves upwards on swallowing and tongue protrusion
• Thyroglossal duct cysts could be found anywhere in or adjacent to the midline from the tongue base to
the thyroid isthmus.s. The cysts almost always arise in the midline . Classically, the cyst moves
upwards on swallowing and notably with tongue protrusion but this may occur with other midline cysts.
Option D: Fluctuation is positive in branchial

Page 11

1742
• Branchial cyst is a fluctuant swelling that may transilluminate and is usually soft at its early stages
which make it difficult to palpate.

Solution for Question 9:


Correct Ans. A: Branchial cysts are congenital abnormalities that occur due to incomplete closure or ab
normal development of the branchial clefts, which are embryonic structures in the neck. The cysts typic
ally develop along the anterior border of the sternocleidomastoid muscle.
Incorrect Options
Option B : Junction of middle and lower third of the sternomastoid muscle: This option is not the most c
ommon site for a branchial cyst. While branchial cysts can occur along the anterior border of the sterno
cleidomastoid muscle, the junction of the middle and lower third is not the primary location for their dev
elopment.
Option C: Middle of the sternomastoid muscle: Similarly, the middle of the sternocleidomastoid muscle
is not the most common site for a branchial cyst. Branchial cysts typically occur at specific locations alo
ng the anterior border of the muscle, rather than within the middle portion.
Option D: Between the two heads of the sternomastoid muscle: This option is also incorrect. The two h
eads of the sternocleidomastoid muscle (sternomastoid muscle) are the sternal head and the clavicular
head. Branchial cysts do not typically develop between these two heads but rather along the anterior b
order of the muscle.

Solution for Question 10:


Correct choice: C
• Basal cell carcinoma is a Slow growing, Locally invasive, Malignant tumour. It is composed of
pluripotent cells and it affects the pilosebaceous skin. BCC can be divided into localised and
generalised. Nodular and nodulocystic variants are 90% of Basal cell carcinomas. Presented as a pink,
pearly papule with overlying telangiectasias, rolled borders and central crusty ulcerations.
Incorrect choices:
Option A. Dermatofibrosarcoma: It is a soft tissue sarcoma that presents as a
reddish-brown plaque that does not have rolled borders as seen in the image.
Option B. Marjolin’s ulcer: Marjolin’s ulcer can develop into squamous cell carcinoma. It occurs on a
previously long-standing scar. The image shown is not marjolin’s ulcer.
Option D. Nevus: It is a
benign skin growth that occurs due to the accumulation of melanocytes. The image shown is not a
nevus.

Page 12

1743
Solution for Question 11:
Correct Option B: Arises from submandibular salivary gland.
• Arises from the submandibular salivary gland: This statement is incorrect.
• Ranulas typically arise from the sublingual gland and submaxillary or there ducts, not from the
submandibular salivary gland. The sublingual gland is located in the floor of the mouth.
Incorrect Options:
Option A: Mucous extravasation cyst: This statement is true. A ranula is a type of mucous extravasatio
n cyst that occurs due to the rupture or blockage of the salivary gland duct, leading to the accumulation
of saliva in the surrounding tissue.
Option C: Bluish, translucent, cystic swelling in the floor of mouth: This statement is true. A
ranula presents as a
bluish, translucent, cystic swelling in the floor of the mouth due to the accumulation of saliva.
Option D: Marsupialization or excision is the treatment: This statement is true. The treatment options fo
r a ranula include marsupialization, which involves creating an opening to drain the cyst, or surgical exc
ision to remove the cyst and associated glandular tissue.
In summary, the incorrect statement about ranula is that it arises from the submandibular salivary glan
d. Ranulas actually arise from the sublingual gland or its duct.

Solution for Question 12:


Correct Option C:
• Explanation: When dealing with recurrent Ranula, the preferred procedure is excision.
• Ranula is a type of mucocele that occurs on the floor of the mouth due to the obstruction of the
salivary gland duct.
• Conservative management approaches like incision and drainage or aspiration may provide
temporary relief but have a higher chance of recurrence.
• Excision, which involves complete removal of the cyst and associated gland, is the most effective
approach to prevent recurrence.
Incorrect Options:
Option A: Incision and drainage: Incision and drainage may be used for an acute or infected Ranula bu
t are not the preferred management for recurrent cases as they have a higher risk of recurrence.
Option B. Aspiration: Aspiration involves removing the fluid from the cyst using a needle and syringe.
While it can provide temporary relief, it is not the ideal management for recurrent Ranula as it does not
address the underlying obstruction and has a higher chance of recurrence.
Option D. Sclerosant injection: Sclerosant injection involves injecting a sclerosing agent into the cyst to
induce scarring and subsequent closure. While this may be used in certain cases, it is not the primary
procedure for recurrent Ranula. Excision is preferred for long-term management.

Page 13

1744
Solution for Question 13:
Correct Option A
• A sebaceous cyst is a common benign cyst that develops beneath the skin. It forms when the
sebaceous glands, which produce oil for the hair and skin, become blocked or damaged.
• This blockage leads to the accumulation of sebum (oil) within a sac, resulting in the formation of a
cyst.
• Sebaceous cysts are typically painless, slow-growing, and often occur on the face, neck, or trunk.
Incorrect Options:
Option B - Implantation dermoid: An implantation dermoid is a rare condition that occurs when fragmen
ts of ectodermal tissue (skin cells) get implanted into the body during embryonic development or after tr
auma. This condition is not consistent with the image showing a sebaceous cyst.
Option C - Angular dermoid: Angular dermoid refers to a
type of dermoid cyst located near the medial or lateral canthus of the eye. It presents as a
lump or swelling near the corner of the eye and is not applicable to the provided image.
Option D - Lipoma: A lipoma is a benign tumor composed of fat tissue. It typically appears as a soft, m
ovable lump under the skin. Lipomas are different from sebaceous cysts and would not be an appropri
ate diagnosis based on the provided image.

Solution for Question 14:


Correct Option B:
• Encephalocele: Encephalocele is a congenital condition where a portion of the brain tissue protrudes
through an opening in the skull.
• This can result in a sac-like protrusion on the scalp or face. It is caused by a failure of the neural tube
to close during fetal development.
Incorrect option
Option A - Lipoma: A lipoma is a benign tumor composed of fat tissue. It typically presents as a
soft, movable lump under the skin. Lipomas are usually painless and harmless.
Option C - Cystic hygroma: Cystic hygroma is a fluid-filled sac that forms due to an abnormality in the l
ymphatic system. It is typically present at birth and most commonly affects the neck and head area. Cy
stic hygromas can vary in size and may cause swelling or a mass-like appearance.
Option D - Lymphadenopathy: Lymphadenopathy refers to the enlargement or swelling of lymph nodes
. It can be caused by various factors, including infection, inflammation, or malignancy. Lymphadenopat
hy is a general term and not a specific diagnosis. It would be important to identify the underlying cause
of lymphadenopathy through further evaluation and testing.

Solution for Question 15:


Correct Option D:

Page 14

1745
• During a parotidectomy, which is the surgical removal of the parotid gland, there is a risk of injuring
branches of the facial nerve that pass through or near the gland. The facial nerve is responsible for
controlling the muscles of facial expression, including those involved in lip movement.
• The mandibular branch of the facial nerve, also known as the marginal mandibular branch, innervates
the lower lip muscles. It travels in a superficial position along the mandible (lower jaw) and is
susceptible to injury during surgery. Damage to this branch of the facial nerve can result in lower lip
paralysis or weakness.
Incorrect Options:
Option A: Temporal branch of the facial nerve: The temporal branch of the facial nerve supplies the mu
scles responsible for forehead and eyebrow movement. Injury to this branch would typically result in ey
ebrow and forehead paralysis, not lower lip paralysis.
Option B: Cervical branch of the facial nerve: The cervical branch of the facial nerve supplies the platys
ma muscle in the neck. Damage to this branch would typically lead to weakness or paralysis of the nec
k muscles, not lower lip paralysis.
Option C: Parotid duct: The parotid duct, also known as Stensen's duct, carries saliva from the parotid
gland to the oral cavity. Injuries to the parotid duct may result in issues with saliva drainage or leakage,
but they would not directly cause lower lip paralysis.

Solution for Question 16:


Correct Option B: Lung
• Lung: Lung cancer is the most common primary source of metastasis to the brain. The lungs have a
rich blood supply and are frequently involved in metastatic spread. Lung cancer cells can travel through
the bloodstream or lymphatic system to reach the brain and form secondary tumors.
Incorrect Options:
Option A: Thyroid: While thyroid cancer can metastasize to the brain, it is not as common as other prim
ary sources. Thyroid cancer more commonly spreads to lymph nodes in the neck and mediastinum bef
ore reaching distant sites.
Option C: Liver: Liver cancer can also metastasize to the brain, but it is not as common as lung cancer.
Liver cancer often spreads to other organs such as the lungs, bones, or peritoneum.
Option D: Breast: Breast cancer is another primary source of metastasis to the brain. Breast cancer cel
ls can spread to the brain through the bloodstream or lymphatic system. Brain metastases are more co
mmonly seen in advanced stages of breast cancer.

Solution for Question 17:


Correct Option D:
• Upper 1/3 sternomastoid on posterior border: This option refers to the upper one-third of the
sternocleidomastoid muscle on its posterior border. The commonest site of branchial cysts is indeed in
this location. Branchial cysts are often found in the upper one-third of the sternocleidomastoid muscle,
near its attachment to the mastoid process.

Page 15

1746
Incorrect Options:
Option A: Lower 1/3 sternomastoid on the anterior border: This option refers to the lower one-third of th
e sternocleidomastoid muscle on its anterior border. Branchial cysts do not commonly occur in this loca
tion.
Option B: Lower 1/3 sternomastoid on posterior border: This option refers to the lower one-third of the
sternocleidomastoid muscle on its posterior border. Branchial cysts do not commonly occur in this locat
ion either.
Option C: Upper 1/3 sternomastoid on anterior border: This option refers to the upper one-third of the s
ternocleidomastoid muscle on its anterior border. Branchial cysts are not typically found in this specific
location.

Solution for Question 18:


Correct Option B:
• The probable diagnosis based on the provided information and the appearance of the face is "Cancer
of parotid gland."
• Cancer of the parotid gland refers to the development of malignant tumors in the parotid gland, which
is the largest salivary gland located in the face. The symptoms described by the patient, including
painless swelling over the face and difficulty in swallowing, are characteristic features of parotid gland
tumors. The image shown could indicate an enlarged parotid gland or a visible mass in the area.
Incorrect Options:
Option A: Acute parotitis: Acute parotitis refers to inflammation of the parotid gland, usually caused by
a bacterial infection. It typically presents with symptoms such as pain, swelling, and tenderness over th
e affected gland. However, in the given scenario, the painless nature of the swelling and the possibility
of a visible mass suggest a different diagnosis.
Option C: Angioedema of face: Angioedema refers to the rapid swelling of the deeper layers of the skin
, often caused by an allergic reaction or other factors. While angioedema can affect the face, it is usuall
y accompanied by other symptoms such as itching, hives, and a history of allergic reactions. The painl
ess swelling and difficulty in swallowing described by the patient are not typical features of angioedema
.
Option D: Acute sialadenitis: Acute sialadenitis refers to inflammation of the salivary glands, often caus
ed by a bacterial infection or blockage of the salivary ducts. It can result in pain, swelling, and tenderne
ss in the affected gland. However, the absence of pain in the described case makes acute sialadenitis l
ess likely.

Solution for Question 19:


Correct Option C:
• Clinical diagnosis is TB (Tuberculosis): Tuberculosis is a bacterial infection caused by Mycobacterium
tuberculosis. It can affect various organs, including the lymph nodes. Tuberculous lymphadenitis is
characterized by painless, fluctuant swelling with a thin watery discharge.
Incorrect Options:

Page 16

1747
Option A:Brachial cyst: A branchial cyst is a
congenital abnormality that arises from remnants of the branchial arches. It typically presents as a pain
less, cystic swelling in the neck, usually in the lateral aspect. It is not associated with watery discharge.
Option B:Secondaries: The term "secondaries" refers to metastatic cancer, which occurs when cancer
cells from the primary tumor spread to other parts of the body. The given clinical presentation with a
painless fluctuant swelling and watery discharge is not indicative of metastatic cancer.
Option D: Lymphoma: Lymphoma is a type of cancer that originates from lymphocytes, a type of white
blood cell. It can affect lymph nodes and other lymphoid tissues. However, lymphoma typically present
s with painless, firm lymph node enlargement, and watery discharge is not a characteristic feature.

Solution for Question 20:


Correct option A:
• Leukoplakia is a precancerous condition that results in white patches or plaques in the oral cavity. It is
typically caused by chronic irritation and inflammation, such as smoking or alcohol use. While alcohol
use can contribute to the development of leukoplakia, it does not typically present with oral ulcers or a
burning sensation.
Incorrect options:
Option B: Erythroplakia is a potentially malignant condition that presents as red patches in the oral cavi
ty. It is often associated with tobacco use and is less commonly associated with alcohol use. While eryt
hroplakia can present with oral ulcers, it is less likely to present with a burning sensation.
Option C: Submucosal fibrosis is characterized by the buildup of fibrous tissue beneath the mucosal lini
ng of the oral cavity. It is most commonly associated with betel nut use and is not typically associated
with alcohol use. While submucosal fibrosis can cause a
burning sensation in the oral cavity, it does not typically present with oral ulcers.
Option D: Malakoplakia is a rare inflammatory condition typically affecting the urinary tract or other org
ans. It is not typically associated with the oral cavity and would not be a likely diagnosis for a
patient with oral ulcers and a burning sensation.

Page 17

1748
Oncology
1. Smoking increases the risk of which of the following cancers?
A. Thyroid
B. Urinary bladder
C. Blood
D. Gallbladder
----------------------------------------
2. What is the likely diagnosis in a 13-year-old boy with progressive left leg pain, marked tenderness
along the left mid-femur, leukocytosis, elevated erythrocyte sedimentation rate, multiple lytic lesions in
the middle third of the femur on x-ray, new bone formation covering the cortex, and a biopsy showing
small round blue cells, with suspicion of a highly radiosensitive tumor?
(or)
What is the most likely diagnosis in a 13-year-old boy with multiple lytic lesions in the middle third of the
femur, new bone formation covering the surrounding cortex, a biopsy showing small round blue cells,
and a very high radio-sensitivity tumor?
A. Ewing’s sarcoma
B. Hodgkin’s disease
C. Multiple myeloma
D. Malignant fibrous histiocytoma
----------------------------------------
3. An 11-year-old boy with Burkitt lymphoma is brought to the emergency department because of
nausea, vomiting, flank pain, and dark urine for 1 day. Two days ago, he began induction
chemotherapy with cyclophosphamide, vincristine, prednisolone, and doxorubicin. Urinalysis shows 3+
blood and abundant amber-coloured rhomboid crystals. Which of the following is most likely to be
effective in preventing this patient’s symptoms?
(or)
An 11-year-old boy with Burkitt lymphoma began induction chemotherapy with cyclophosphamide,
vincristine, prednisolone, and doxorubicin. Urinalysis shows 3+ blood. Which of the following is most
likely effective in preventing this patient's symptoms?
A. Hydration with normal saline before and after chemotherapy
B. Allopurinol administration before chemotherapy
C. Prophylactic use of antiemetic medications before chemotherapy
D. Calcium supplementation during chemotherapy
----------------------------------------
4. A 45-year-old male came to the hospital with a sudden fracture of femur bone. The patient was
diagnosed with a metastatic bone tumour. Which of the following is most common primary metastatic
bone tumour in males?
(or)
What is the most common primary metastatic bone tumor in a 45-year-old male?

1749
A. Lung
B. Liver
C. Bone
D. Brain
----------------------------------------
5. What is the most appropriate next step in management for superficial spreading melanoma and a
lesion thickness of 1.1 mm?

A. Surgical excision with 0.5-1 cm safety margins only


B. Sentinel lymph node biopsy only
C. Surgical excision with 1-2 cm safety margins and sentinel lymph node biopsy
D. Surgical excision with 1 cm safety margins only
----------------------------------------
6. A woman brings her 3-year-old girl to the emergency department with a complaint of a clear,
grape-like, polypoid mass emerging from the vagina. Her vital signs are within normal limits. What is the
most likely diagnosis?
(or)
What is the most likely diagnosis in a 3-year-old girl presenting with a clear, grape-like polypoid mass
from the vagina with normal vital signs?
A. Sarcoma botryoides
B. Squamous cell carcinoma
C. Melanoma
D. Adenocarcinoma
----------------------------------------
7. What feature is necessary to determine the grade of a large right-sided renal mass in a 70-year-old
man presenting with right-sided back pain, hematuria, and weight loss for the last 4 months, and a
biopsy showing polygonal clear cells filled with lipids?
(or)
A 70-year-old man comes to the physician for right-sided back pain, red coloured urine, and weight loss
for the last 4 months. He had smoked one pack of cigarettes daily for 40 years. A CT scan of the
abdomen shows a large right-sided renal mass. A biopsy of the mass shows polygonal clear cells filled

Page 2

1750
with lipids. Which of the following features is necessary to determine the tumour grade in this patient?
A. Size of malignant proliferation
B. Invasion of surrounding structures
C. Degree of mitotic activity
D. Response to chemotherapy
----------------------------------------
8. A thirty-nine year old female presents to your clinic with a complaint of swelling on her scalp. The
swelling is associated with discharge. She previously had swelling over the same area, which she got
removed 10 months back. She now has developed a much larger lesion. The patient is found to have
Cock's peculiar tumour. What is Cock's peculiar tumour?
(or)
What is Cock's peculiar tumor?
A. Basal cell carcinoma
B. Squamous cell carcinoma
C. Infected sebaceous cyst
D. Cylindroma
----------------------------------------
9. What is the most common symptom of bronchogenic carcinoma ?
A. Hemoptysis
B. Dyspnoea
C. Cough
D. Wheezing
----------------------------------------
10. What are the components of the Currarino triad?
A. Pre-sacral meningocele + Sacral defect + Tethered cord
B. Ectopia vesicae + Anorectal malformation + Sacrococcygeal osseous defect
C. Anorectal malformations + Sacrococcygeal osseous defect + Presacral mass
D. Tethered cord + Anorectal malformations + Ectopia vesicae
----------------------------------------
11. What cellular changes are most likely to occur in an 84-year-old man with prostate cancer and
vertebral lesions treated with dexamethasone and external beam radiation?
A. Formation of pyrimidine dimers
B. Generation of hydroxyl radicals
C. Disruption of microtubule assembly
D. Formation of DNA crosslinks
----------------------------------------

Page 3

1751
12. A 53-year-old patient has SVC obstruction. Chest x-ray shows mediastinal enlargement. What is
the next step in establishing a diagnosis for this patient?
(or)
A 53-year-old patient presents with complaints of dyspnea. He is a lifetime smoker. His pulse is 99/min,
and his blood pressure is 95/66 mmHg. Examination shows swelling and redness of the face and neck.
There is distention of the superficial veins of his neck and upper thorax. SVC obstruction is suspected.
Chest x-ray shows mediastinal enlargement. What is the next step in establishing a diagnosis for this
patient?
A. Total blood count with peripheral smear
B. CT thorax
C. Start cyclophosphamide
D. Urgent referral to treatment
----------------------------------------
13. Which of the following statements about rhabdomyosarcoma (RMS) is correct?
A. The most common histological type of RMS in adults is alveolar RMS.
B. The most common histological type of RMS is the spindle cell variant.
C. The most common route of spread for RMS is hematogenous.
D. The most common site of metastasis for RMS is bone.
----------------------------------------
14. Which of the following statements regarding dermatofibrosarcoma protuberans (DFSP) is incorrect?
A. DFSP typically involves the dermis and subcutis.
B. Most DFSP cases exhibit a ring chromosome, composed of a translocated portion of chromosomes
17 and 22.
C. The most common site for DFSP is the trunk, followed by the extremities.
D. The initial investigation of choice for diagnosing DFSP is excisional biopsy.
----------------------------------------
15. A 56-year-old male with a history of advanced lung cancer presents to the emergency department
with confusion and fatigue. Laboratory investigations reveal hypercalcemia. Which of the following
factors is primarily responsible for hypercalcemia of malignancy in this patient?
(or)
In a 56-year-old male with advanced lung cancer presenting with confusion and fatigue, which factor
primarily causes hypercalcemia of malignancy?
A. Increased release of phosphate from bone
B. Decreased calcium reabsorption from the distal convoluted tubule
C. Increased secretion of parathyroid hormone-related protein
D. Enhanced activation of vitamin D in the kidney
----------------------------------------
16. A neutropenic patient who recently underwent chemotherapy presents with fever, right lower
quadrant tenderness, and bloody diarrhea. Imaging reveals a thickened cecal wall. What is the most

Page 4

1752
likely diagnosis, and when is surgical intervention indicated?
(or)
What is the most likely diagnosis and surgical treatment needed in a patient who underwent
chemotherapy presenting with fever, right lower quadrant tenderness, and bloody diarrhea, imaging
reveals a thickened cecal wall?
A. Diverticulitis; indicated if perforation occurs
B. Typhlitis; indicated if no improvement within 24 hours of antibiotics
C. Crohn's disease; indicated if perforation leads to peritonitis
D. Ischemic colitis; indicated if imaging shows bowel necrosis
----------------------------------------

Correct Answers
Question Correct Answer

Question 1 2
Question 2 1
Question 3 1
Question 4 1
Question 5 3
Question 6 1
Question 7 3
Question 8 3
Question 9 3
Question 10 3
Question 11 2
Question 12 2
Question 13 3
Question 14 4
Question 15 3
Question 16 2

Solution for Question 1:


Option B: Urinary bladder
• Smoking is an important risk factor for: Transitional cell carcinoma of the bladder Squamous cell
carcinoma (cervix, oropharynx, esophagus, larynx, lung) Small cell lung cancer Pancreatic
adenocarcinoma Adenocarcinoma of the esophagus Renal cell carcinoma
• Transitional cell carcinoma of the bladder
• Squamous cell carcinoma (cervix, oropharynx, esophagus, larynx, lung)

Page 5

1753
• Small cell lung cancer
• Pancreatic adenocarcinoma
• Adenocarcinoma of the esophagus
• Renal cell carcinoma
• Cigarette smoking increases the risk of many cancers. It includes: First-hand smoke: smoke inhaled
by the smoker Second-hand smoke: exhaled smoke that is inhaled by others in the vicinity of the
smoker Third-hand smoke: smoke particles that adhere to surfaces in the surroundings of the smoker
All three have been proven to be carcinogenic
• First-hand smoke: smoke inhaled by the smoker
• Second-hand smoke: exhaled smoke that is inhaled by others in the vicinity of the smoker
• Third-hand smoke: smoke particles that adhere to surfaces in the surroundings of the smoker All three
have been proven to be carcinogenic
• All three have been proven to be carcinogenic
• Transitional cell carcinoma of the bladder
• Squamous cell carcinoma (cervix, oropharynx, esophagus, larynx, lung)
• Small cell lung cancer
• Pancreatic adenocarcinoma
• Adenocarcinoma of the esophagus
• Renal cell carcinoma
• First-hand smoke: smoke inhaled by the smoker
• Second-hand smoke: exhaled smoke that is inhaled by others in the vicinity of the smoker
• Third-hand smoke: smoke particles that adhere to surfaces in the surroundings of the smoker All three
have been proven to be carcinogenic
• All three have been proven to be carcinogenic
• All three have been proven to be carcinogenic
Option A: Thyroid
• Smoking is not a risk factor for thyroid cancer.
Option C: Blood
• Cigarette smoke does not increase the risk of blood cancer.
Option D: Gallbladder
• Smoking is not a risk factor for carcinoma gallbladder.

Solution for Question 2:


Option A: Ewing’s sarcoma
• Ewing sarcoma is a malignant bone tumor that arises from neuroectodermal tissue. Peak incidence is
between 10 and 15 years of age.

Page 6

1754
• Typically occurs in flat bones (e.g., ilium, scapula) or the diaphysis of long bones. An X-ray usually
reveals a tumor with multiple lytic lesions that may extend into the surrounding soft tissue and an
aggressive lamellated periosteal reaction, giving rise to the classic onion skin appearance on X-ray.
• A biopsy reveals small, round, undifferentiated cells that appear dark blue on an H&E; stain because
these cells have a large nuclear-cytoplasmic ratio, and the nucleus stains blue with hematoxylin.
• Patients also often have systemic features such as fever, increased ESR, and leukocytosis.
• Among the options, the most radiosensitive tumor is Ewing's sarcoma.

Option B: Hodgkin’s disease


• Hodgkin lymphoma (HL) is a malignant disease typical of B-cell origin.
• HL typically presents painless cervical lymphadenopathy, fever, night sweats, and involuntary weight
loss.
Option C: Multiple myeloma
• Multiple myeloma is a clonal plasma cell proliferative disorder characterized by the abnormal increase
of monoclonal paraprotein.
• The given scenario is not suggestive of multiple myeloma.
Option D: Malignant fibrous histiocytoma
• A malignant fibrous histiocytoma is a cancerous tumor typically developing in soft tissue like muscles
and tendons. However, it can sporadically happen in bones.
• Malignant fibrous histiocytoma replaces healthy bone tissue with cancer cells. These cancer cells
make the bone weaker and increase the risk of fractures.
• Other signs and symptoms include: Pain in the muscle, tendon, or another tumor site. Swelling over a
bone or joint A lump that can be felt under the skin
• Pain in the muscle, tendon, or another tumor site.
• Swelling over a bone or joint
• A lump that can be felt under the skin

Page 7

1755
• Pain in the muscle, tendon, or another tumor site.
• Swelling over a bone or joint
• A lump that can be felt under the skin

Solution for Question 3:


Option A: Hydration with normal saline before and after chemotherapy.
• Hydration with normal saline before and after chemotherapy is effective in preventing tumor lysis
syndrome (TLS) in patients undergoing chemotherapy, particularly with drugs like cyclophosphamide,
vincristine, prednisolone, and doxorubicin.
• TLS is a potentially life-threatening complication characterized by metabolic disturbances such as
hyperkalemia, hyperuricemia, hyperphosphatemia, and hypocalcemia, resulting from the rapid release
of intracellular contents into the bloodstream due to chemotherapy-induced tumor cell lysis.
• Normal saline hydration helps maintain adequate renal perfusion and urinary flow, promoting the
excretion of uric acid and other metabolic byproducts, thus preventing the development of TLS and its
associated symptoms like nausea, vomiting, flank pain, and dark urine.

Incorrect Options:
Option B: Allopurinol administration before chemotherapy.
• Allopurinol is a xanthine oxidase inhibitor used to prevent and treat hyperuricemia by reducing the
production of uric acid.
• While allopurinol is commonly used to prevent TLS, especially in high-risk patients, hydration with
normal saline is the primary intervention to prevent TLS. Allopurinol alone may not be sufficient to
prevent TLS, particularly in patients undergoing aggressive chemotherapy regimens.
• Additionally, allopurinol does not directly address the hydration needs of the patient, which are crucial
in preventing TLS.
Option C: Prophylactic use of antiemetic medications before chemotherapy.
• Antiemetic medications are used to prevent or relieve nausea and vomiting associated with
chemotherapy.
• While nausea and vomiting are common side effects of chemotherapy, the patient's symptoms of
nausea, vomiting, flank pain, and dark urine in this scenario are more indicative of TLS rather than
chemotherapy-induced nausea and vomiting.
• Administering antiemetic medications alone would not adequately address the underlying
pathophysiology of TLS.
Option D: Calcium supplementation during chemotherapy.
• Calcium supplementation is not typically indicated as a preventive measure for TLS.
• TLS primarily involves metabolic disturbances such as hyperuricemia, hyperkalemia,
hyperphosphatemia, and hypocalcemia. Hydration with normal saline is the mainstay of prevention as it
promotes renal excretion of uric acid and other metabolic byproducts, thus preventing TLS and
associated electrolyte imbalances.

Page 8

1756
Solution for Question 4:
Correct Option
Option A: Lung
• Among the given options, The most common metastatic bone tumor in a male is Carcinoma Lung.
• BONE METASTASIS Bone metastasis occurs due to secondary bone tumors due
to metastasis (predominantly hematogenous) of primary malignancies of other organs. The primary
malignancies include lung, breast, and prostate cancer. These are more common than primary bone
tumors. Bones are the third most common site of metastases, after the lung and the liver. Classification
of bone metastasis is based on radiological findings Osteoblastic metastasis: New bone formation
by osteoblasts outweighs osteolytic processes → increase in radiographic density. Examples: prostate
cancer, small cell lung cancer Osteolytic metastasis: Osteolytic processes outweigh new
bone formation → decrease in radiographic density. Examples: multiple myeloma, thyroid
cancer, kidney cancer, melanoma, non-small cell lung cancer Mixed metastasis: e.g., breast cancer,
gastrointestinal cancer Clinical features include: Local pain and swelling Pathologic fractures Spinal
cord compression, and radicular symptoms
• Bone metastasis occurs due to secondary bone tumors due to metastasis (predominantly
hematogenous) of primary malignancies of other organs.
• The primary malignancies include lung, breast, and prostate cancer.
• These are more common than primary bone tumors.
• Bones are the third most common site of metastases, after the lung and the liver.
• Classification of bone metastasis is based on radiological findings Osteoblastic metastasis: New
bone formation by osteoblasts outweighs osteolytic processes → increase in radiographic
density. Examples: prostate cancer, small cell lung cancer Osteolytic metastasis: Osteolytic processes
outweigh new bone formation → decrease in radiographic density. Examples: multiple
myeloma, thyroid cancer, kidney cancer, melanoma, non-small cell lung cancer Mixed metastasis:
e.g., breast cancer, gastrointestinal cancer
• Osteoblastic metastasis: New bone formation by osteoblasts outweighs osteolytic processes →
increase in radiographic density. Examples: prostate cancer, small cell lung cancer
• Osteolytic metastasis: Osteolytic processes outweigh new bone formation → decrease in radiographic
density. Examples: multiple myeloma, thyroid cancer, kidney cancer, melanoma, non-small cell lung
cancer
• Mixed metastasis: e.g., breast cancer, gastrointestinal cancer
• Clinical features include: Local pain and swelling Pathologic fractures Spinal cord compression, and
radicular symptoms
• Local pain and swelling
• Pathologic fractures
• Spinal cord compression, and radicular symptoms
• Bone metastasis occurs due to secondary bone tumors due to metastasis (predominantly
hematogenous) of primary malignancies of other organs.
• The primary malignancies include lung, breast, and prostate cancer.

Page 9

1757
• These are more common than primary bone tumors.
• Bones are the third most common site of metastases, after the lung and the liver.
• Classification of bone metastasis is based on radiological findings Osteoblastic metastasis: New
bone formation by osteoblasts outweighs osteolytic processes → increase in radiographic
density. Examples: prostate cancer, small cell lung cancer Osteolytic metastasis: Osteolytic processes
outweigh new bone formation → decrease in radiographic density. Examples: multiple
myeloma, thyroid cancer, kidney cancer, melanoma, non-small cell lung cancer Mixed metastasis:
e.g., breast cancer, gastrointestinal cancer
• Osteoblastic metastasis: New bone formation by osteoblasts outweighs osteolytic processes →
increase in radiographic density. Examples: prostate cancer, small cell lung cancer
• Osteolytic metastasis: Osteolytic processes outweigh new bone formation → decrease in radiographic
density. Examples: multiple myeloma, thyroid cancer, kidney cancer, melanoma, non-small cell lung
cancer
• Mixed metastasis: e.g., breast cancer, gastrointestinal cancer
• Clinical features include: Local pain and swelling Pathologic fractures Spinal cord compression, and
radicular symptoms
• Local pain and swelling
• Pathologic fractures
• Spinal cord compression, and radicular symptoms
• Osteoblastic metastasis: New bone formation by osteoblasts outweighs osteolytic processes →
increase in radiographic density. Examples: prostate cancer, small cell lung cancer
• Osteolytic metastasis: Osteolytic processes outweigh new bone formation → decrease in radiographic
density. Examples: multiple myeloma, thyroid cancer, kidney cancer, melanoma, non-small cell lung
cancer
• Mixed metastasis: e.g., breast cancer, gastrointestinal cancer
• Local pain and swelling
• Pathologic fractures
• Spinal cord compression, and radicular symptoms
Other options
Option B: Liver
• Metastasis of hepatocellular carcinoma occurs hematogenously but is rare and usually only occurs in
advanced stages.
• The most common locations for metastatic spread are the lungs, abdomen, and bones
• Metastatic liver disease:
• Metastatic liver disease is the most common malignant liver lesion Typical
primary tumour sites: gastrointestinal tract (Colon >> Stomach > Pancreas), lung, and breast
• Metastatic liver disease is the most common malignant liver lesion
• Typical primary tumour sites: gastrointestinal tract (Colon >> Stomach > Pancreas), lung, and breast
• Metastatic liver disease is the most common malignant liver lesion
• Typical primary tumour sites: gastrointestinal tract (Colon >> Stomach > Pancreas), lung, and breast

Page 10

1758
Option C: Bone
• Bone cancer includes both benign and malignant cancers.
• The malignant bone tumors metastasize into the lungs, skeletal system, and bone marrow.
Option D: Brain
• Primary CNS tumors do not metastasize to organs outside the CNS.
• Drop metastases (intradural extramedullary spinal metastases ) and leptomeningeal metastases may
occur.
• Typically manifest as nodules along the spine and cauda equina that can cause back pain with
neurologic symptoms (e.g., limb weakness)
• Brain metastases: However, 50% of brain tumors occur due to metastases. The most common sites of
primary tumors that metastasize into the brain include: Lung > breast > melanoma, colon, kidney
• However, 50% of brain tumors occur due to metastases.
• The most common sites of primary tumors that metastasize into the brain include: Lung > breast >
melanoma, colon, kidney
• Lung > breast > melanoma, colon, kidney
• However, 50% of brain tumors occur due to metastases.
• The most common sites of primary tumors that metastasize into the brain include: Lung > breast >
melanoma, colon, kidney
• Lung > breast > melanoma, colon, kidney
• Lung > breast > melanoma, colon, kidney

Solution for Question 5:


Correct Option C: Surgical excision with 1-2 cm safety margins and sentinel lymph node biopsy.
• Superficial spreading melanoma is the most common type of melanoma, and its management
typically involves surgical excision with appropriate safety margins.
• The recommended safety margins for surgical excision of superficial spreading melanoma with a
lesion thickness of 1.1 mm range from 1 to 2 cm, depending on various factors such as the location of
the lesion, the presence of ulceration, and the patient's overall health status.
• Sentinel lymph node biopsy is also indicated in cases of superficial spreading melanoma with a
thickness greater than 1 mm to assess whether the melanoma has spread to nearby lymph nodes.
• Therefore, the most appropriate next step in management for this patient would be surgical excision
with 1-2 cm safety margins to ensure complete removal of the primary tumor, along with sentinel lymph
node biopsy to evaluate for regional lymph node involvement.
Incorrect options:
Option A: Surgical excision with 0.5-1 cm safety margins only.
• Safety margins of 0.5-1 cm may be appropriate for thinner melanomas, but for a lesion thickness of
1.1 mm, wider safety margins of 1-2 cm are generally recommended to reduce the risk of local

Page 11

1759
recurrence.
• Additionally, sentinel lymph node biopsy is indicated for melanomas with a thickness greater than 1
mm to assess lymph node involvement, so this option does not address the need for nodal evaluation.
Option B: Sentinel lymph node biopsy only.
• While sentinel lymph node biopsy is an important component of the management of melanoma,
surgical excision with appropriate safety margins is also necessary to ensure complete removal of the
primary tumor.
• This option neglects the importance of surgical excision with adequate safety margins, which is
essential for reducing the risk of local recurrence.
Option D: Surgical excision with 1 cm safety margins only.
• Safety margins of 1 cm may be appropriate for thinner melanomas, but for a lesion thickness of 1.1
mm, wider safety margins of 1-2 cm are generally recommended to reduce the risk of local recurrence.
• Additionally, this option does not address the need for sentinel lymph node biopsy, which is indicated
for melanomas with a thickness greater than 1 mm to assess lymph node involvement.

Solution for Question 6:


Option A: Sarcoma botryoides
• Sarcoma botryoides, also known as Embryonal rhabdomyosarcoma
• It is a rare, highly malignant rhabdomyosarcoma.
• Most commonly arises in the genitourinary system as a polypoid mass that resembles a bunch of
grapes (botryoid in Greek) protruding through the vagina.
• Peak incidence is before the age of 8.
• Pathology Gross: clear, polypoid masses that resemble a bunch of grapes protruding through
the vagina Microscopy: pleomorphic spindle-shaped cells
• Gross: clear, polypoid masses that resemble a bunch of grapes protruding through the vagina
• Microscopy: pleomorphic spindle-shaped cells
• Gross: clear, polypoid masses that resemble a bunch of grapes protruding through the vagina
• Microscopy: pleomorphic spindle-shaped cells
Immunohistochemical staining: desmin positive

Page 12

1760
Other options
Option B: Squamous cell carcinoma
• Squamous cell carcinoma is the most common type of vaginal carcinoma (∼ 75% of cases). Most
cases are caused by infection with HPV type 16 or 18.
• Other symptoms of vaginal carcinoma include vaginal ulceration and a malodorous discharge.
Option C: Melanoma
• Vaginal melanomas are seen in women 50–70 years of age and may manifest with postcoital
bleeding.
• However, this tumor is extremely rare (< 10% of all vaginal carcinomas) and typically occurs in the
lower third of the vagina.
Option D: Adenocarcinoma
• Epithelial cancer arises from glandular tissue. Common types include lung, gastrointestinal tract,
breast, and prostate adenocarcinomas.
• Vaginal adenocarcinomas are the primary vaginal cancer in women younger than 20 and the most
likely histology in patients with a history of intrauterine exposure to diethylstilbestrol.

Solution for Question 7:


Option C: Degree of mitotic activity
• Tumor grade: A histopathologic assessment of neoplastic cells. Cells with relatively normal
morphology and organization are well-differentiated (low grade); abnormal cells are poorly
differentiated or undifferentiated/anaplastic (high grade).
• Specific criteria for low- and high-grade classifications vary according to the type of malignancy.
• Tumor grading determines the degree of cellular differentiation based on histological findings.
Components of tumor grading systems include mitotic count, the degree of nuclear pleomorphism, and
cellular organization (tubular/glandular structures).

Page 13

1761
• Components of tumor grading systems include mitotic count, the degree of nuclear pleomorphism,
and cellular organization (tubular/glandular structures).
• Components of tumor grading systems include mitotic count, the degree of nuclear pleomorphism,
and cellular organization (tubular/glandular structures).
Option A: Size of malignant proliferation
• Proliferation is the rapid division and increased number of cells.
• The size of malignant proliferation is considered for tumor staging. The larger the tumor, the higher
the probability of metastases.
• Therefore, size correlates with a more advanced tumor stage. Tumor grading evaluates the
morphologic characteristics of tumors and is not based on tumor size.
Option B: Invasion of surrounding structures
• The involvement of surrounding structures is determined in tumor staging, not tumor grading, which
assesses the morphologic characteristics of tumor cells.
• Staging is an assessment of the disease stage of cancer.
• Factors evaluated include the size and extent of the primary tumor (T), the number of nearby lymph
nodes involved (N), and metastasis (M). Used to evaluate prognosis and guide management.
Option D: Response to chemotherapy
• Chemotherapy uses antineoplastic agents with cytotoxic properties to inhibit the proliferation of rapidly
growing cells (as in the treatment of cancer and other proliferative disorders).
• Classified according to the mechanism of action, including alkylating agents, antimetabolites,
topoisomerase inhibitors, and mitotic inhibitors. Associated with a range of adverse effects (e.g.,
nausea, vomiting, increased risk of infection, and impaired growth of healthy cells).
• Response to chemotherapy is not considered part of the tumor grading or staging. However, response
to chemotherapy has important implications for future treatment planning and prognosis.

Solution for Question 8:


Option C: Infected sebaceous cyst
• Cock’s peculiar tumour is an ulcerating cyst which is a complication of the sebaceous cyst.
• It resembles a squamous cell carcinoma.
• These are usually solitary but may be multiple in some cases.
• They are usually found in hairy areas such as the scalp, where the trichilemmal cyst proliferates to a
large size and ulcerates.
• Chronic inflammation causes the cyst to take the form of a granuloma.

Page 14

1762
Option A: Basal cell carcinoma
• Cock's peculiar tumors are infected sebaceous cysts of the scalp.
• It is not a basal cell carcinoma.
Option B: Squamous cell carcinoma
• Cock's peculiar tumours may resemble squamous cell carcinoma.
• They are infected sebaceous cysts.
Option D: Cylindroma
• Cylindromas are benign skin appendage tumors.
• They may be present on the scalp.
• Cock's peculiar tumors are infected sebaceous cysts of the scalp.

Solution for Question 9:


Option C: Cough
• Bronchogenic carcinoma is a malignant cancer of the lungs.
• It arises from the epithelium of the bronchi or bronchioles.
• Cough is the most common symptom, followed by dyspnea, chest pain and hemoptysis.
• Smoking is the most common cause; it is the cause of 90% of all lung cancer cases.
• Symptoms include: Persistent or worsening cough Wheezing Hemoptysis Chest pain Dyspnoea
Hoarseness Weakness, fatigue Frequent attacks of bronchitis or pneumonia
• Persistent or worsening cough
• Wheezing
• Hemoptysis

Page 15

1763
• Chest pain
• Dyspnoea
• Hoarseness
• Weakness, fatigue
• Frequent attacks of bronchitis or pneumonia
• Persistent or worsening cough
• Wheezing
• Hemoptysis
• Chest pain
• Dyspnoea
• Hoarseness
• Weakness, fatigue
• Frequent attacks of bronchitis or pneumonia

Option A: Hemoptysis
• Hemoptysis is one of the symptoms of bronchogenic carcinoma.
• It is not the most common symptom.
• The artery that is usually responsible for causing hemoptysis is the bronchial artery.
Option B: Dyspnoea
• Dyspnoea is one of the symptoms of bronchogenic carcinoma.
• It is not the most common symptom.
• The most common symptom is cough.
Option D: Wheezing
• Wheezing is one of the symptoms of bronchogenic carcinoma.
• It is not the most common symptom.
• The most common symptom is cough.

Solution for Question 10:


Option C: Anorectal malformations + Sacrococcygeal osseous defect + Presacral mass
• The Currarino triad or ASP triad is an inherited disorder characterized by anomalies of the sacrum,
anorectal, and presacral soft tissues.
• The characteristic findings of the Currarino triad are: Anorectal malformation or congenital anorectal
stenosis Sacrococcygeal osseous defect (always present) Classically, hemi sacrum with intact first
sacral vertebra ("sickle-shaped sacrum") Mild (hypoplasia) to severe (agenesis) of sacrum and coccyx
Presacral mass (various types) Anterior sacral meningocele Tumour, e.g., mature teratoma
Dermoid/epidermoid cyst

Page 16

1764
• Anorectal malformation or congenital anorectal stenosis
• Sacrococcygeal osseous defect (always present) Classically, hemi sacrum with intact first sacral
vertebra ("sickle-shaped sacrum") Mild (hypoplasia) to severe (agenesis) of sacrum and coccyx
• Classically, hemi sacrum with intact first sacral vertebra ("sickle-shaped sacrum")
• Mild (hypoplasia) to severe (agenesis) of sacrum and coccyx
• Presacral mass (various types) Anterior sacral meningocele Tumour, e.g., mature teratoma
Dermoid/epidermoid cyst
• Anterior sacral meningocele
• Tumour, e.g., mature teratoma
• Dermoid/epidermoid cyst
• Anorectal malformation or congenital anorectal stenosis
• Sacrococcygeal osseous defect (always present) Classically, hemi sacrum with intact first sacral
vertebra ("sickle-shaped sacrum") Mild (hypoplasia) to severe (agenesis) of sacrum and coccyx
• Classically, hemi sacrum with intact first sacral vertebra ("sickle-shaped sacrum")
• Mild (hypoplasia) to severe (agenesis) of sacrum and coccyx
• Presacral mass (various types) Anterior sacral meningocele Tumour, e.g., mature teratoma
Dermoid/epidermoid cyst
• Anterior sacral meningocele
• Tumour, e.g., mature teratoma
• Dermoid/epidermoid cyst
• Classically, hemi sacrum with intact first sacral vertebra ("sickle-shaped sacrum")
• Mild (hypoplasia) to severe (agenesis) of sacrum and coccyx
• Anterior sacral meningocele
• Tumour, e.g., mature teratoma
• Dermoid/epidermoid cyst

Page 17

1765
Option A: Pre-sacral meningocele + Sacral defect + Tethered cord
• Tethered spinal cord syndrome is the abnormal stretching of the spinal cord.
• This syndrome is associated with the following: Spina bifida Spinal surgery
• Spina bifida
• Spinal surgery
• The tethered spinal cord is not associated with the Currarino triad.
• Spina bifida
• Spinal surgery
Option B: Ectopia vesicae + Anorectal malformation + Sacrococcygeal osseous defect
• Ectopia vesicae is a congenital anomaly causing exstrophy of the bladder.
• It is associated with the exstrophy-epispadias complex.
• Ectopia vesicae are not associated with the Currarino triad.
Option D: Tethered cord + Anorectal malformations + Ectopia vesicae
• Tethered spinal cord and ectopia vesicae are not associated with the Currarino triad.

Solution for Question 11:


Option B: Generation of hydroxyl radicals
• Ionising radiation is a form of radiation that contains enough energy to liberate electrons from atomic
orbit, which ionizes them.
• Light in the X-ray spectrum or higher has sufficient energy to ionize atoms, which can create free
radicals which damages the cells of body.
• Ionising radiation (e.g., gamma irradiation, x-ray) causes ionization of the water component of cells
and the resultant generation of hydroxyl radicals. Hydroxyl radicals are reactive oxygen species that

Page 18

1766
attack the DNA backbone and bases.
• These reactions cause DNA single-strand breaks and, most deleteriously, double-strand breaks,
which are very difficult for the cell to repair, leading to cell death.

• The radiosensitivity of cells varies with the cell cycle phase. The image here shows the phases in
which radiation results in chromosome aberration (yellow), apoptosis (blue), or mitotic arrest (green).
• Cells in the G2 and M phases demonstrate the highest radiosensitivity, whereas cells in the G1 and S
phases generally show radioresistance.
Option A: Formation of pyrimidine dimers
• Non-ionising radiation (e.g., UV radiation) causes covalent binding between adjacent pyrimidines,
leading to dimer formation (most commonly thymine dimers).
• These dimers create bulky helix distortions, impairing regular base pairing during replication, which
increases the risk of developing mutations. Non-ionizing radiation is not used for external beam
radiation therapy for cancer; instead, it is the causative agent for skin cancers.
Option C: Disruption of microtubule assembly
• Vinca alkaloids (e.g., vincristine, vinblastine) are chemotherapeutic agents that disrupt
microtubule assembly by binding to β-tubulin.
• This disruption prevents the formation of the mitotic spindle, causing cell cycle arrest. Vinca
alkaloids are most commonly used to treat solid tumors, leukemias, and lymphomas.
Option D: Formation of DNA crosslinks
• The formation of DNA crosslinks is the mechanism by which alkylating chemotherapeutic agents (e.g.,
cyclophosphamide, busulfan) destroy malignant cells.
• The presence of DNA crosslinks results in impaired DNA replication of the malignant cells. Alkylating
agents are used for various malignancies, including brain tumors, leukemias, and lymphomas.

Solution for Question 12:

Page 19

1767
Option B: CT thorax
• Computed tomography (CT) of the thorax is the preferred imaging modality for evaluating suspected
SVC obstruction. It provides detailed visualization of the mediastinal structures, including the superior
vena cava, and any potential obstruction causes, such as tumors or thrombosis.
• CT imaging can help identify the exact location and extent of the obstruction and any associated
findings, such as mediastinal enlargement or compression of surrounding structures.
• Superior Vena Cava (SVC) Syndrome: Caused by obstruction of blood flow in the superior vena cava,
typically by a thrombus or external compression. Most commonly due to lung cancer (small cell &
squamous cell carcinoma), lymphoma, or metastatic tumors. Clinical features include head fullness,
dyspnea, upper extremity edema, and distended superficial veins in the chest, face, and upper
extremities. Hemodynamic symptoms include facial plethora, jugular venous distension, orthostatic
hypotension, syncope, and renal failure. Symptoms of neck congestion include dyspnea, cough,
hoarseness, stridor, and dysphagia. Neurological symptoms may include headache, dizziness,
confusion, and visual impairment.
• Caused by obstruction of blood flow in the superior vena cava, typically by a thrombus or external
compression.
• Most commonly due to lung cancer (small cell & squamous cell carcinoma), lymphoma, or metastatic
tumors.
• Clinical features include head fullness, dyspnea, upper extremity edema, and distended superficial
veins in the chest, face, and upper extremities.
• Hemodynamic symptoms include facial plethora, jugular venous distension, orthostatic hypotension,
syncope, and renal failure.
• Symptoms of neck congestion include dyspnea, cough, hoarseness, stridor, and dysphagia.
• Neurological symptoms may include headache, dizziness, confusion, and visual impairment.
• Diagnosis: In unstable or severely symptomatic patients, invasive venography with or without stent
placement is the gold standard. In stable patients, CT chest with CT venography is the modality of
choice for most cases. Doppler ultrasound may be used to investigate suspected catheter-associated
thrombosis.
• In unstable or severely symptomatic patients, invasive venography with or without stent placement is
the gold standard.
• In stable patients, CT chest with CT venography is the modality of choice for most cases.
• Doppler ultrasound may be used to investigate suspected catheter-associated thrombosis.
Superior Vena Cava (SVC) Syndrome:
• Caused by obstruction of blood flow in the superior vena cava, typically by a thrombus or external
compression.
• Most commonly due to lung cancer (small cell & squamous cell carcinoma), lymphoma, or metastatic
tumors.
• Clinical features include head fullness, dyspnea, upper extremity edema, and distended superficial
veins in the chest, face, and upper extremities.
• Hemodynamic symptoms include facial plethora, jugular venous distension, orthostatic hypotension,
syncope, and renal failure.
• Symptoms of neck congestion include dyspnea, cough, hoarseness, stridor, and dysphagia.
• Neurological symptoms may include headache, dizziness, confusion, and visual impairment.

Page 20

1768
Diagnosis:
• In unstable or severely symptomatic patients, invasive venography with or without stent placement is
the gold standard.
• In stable patients, CT chest with CT venography is the modality of choice for most cases.
• Doppler ultrasound may be used to investigate suspected catheter-associated thrombosis.

Option A: Total blood count with peripheral smear


• While laboratory tests such as a total blood count with peripheral smear may provide some
information about the patient's overall health status, they are not specific to diagnosing SVC
obstruction. These tests may show nonspecific findings such as anemia or leukocytosis but are unlikely
to confirm or exclude SVC obstruction directly.
Option C: Start cyclophosphamide
• Cyclophosphamide is used in the treatment of the following malignancies: Solid tumors (e.g., breast
cancer, ovarian cancer, small cell lung cancer) Leukemias Lymphomas Multiple myeloma
• Solid tumors (e.g., breast cancer, ovarian cancer, small cell lung cancer)
• Leukemias
• Lymphomas
• Multiple myeloma
• However, in the case of SVC syndrome, a chest CT scan is necessary to rule out the cause of
obstruction causing SVC syndrome.
• Solid tumors (e.g., breast cancer, ovarian cancer, small cell lung cancer)
• Leukemias
• Lymphomas
• Multiple myeloma
Option D: Urgent referral to treatment

Page 21

1769
• In stable patients of SVC syndrome, the investigation of choice is a chest CT scan used to diagnose
the cause of obstruction, and after the cause of SVC obstruction is known, treatment begins.

Solution for Question 13:


Correct Option C - The most common route of spread for RMS is hematogenous:
• Rhabdomyosarcoma (RMS) most commonly spreads hematogenously, meaning it spreads through
the bloodstream to distant sites in the body.
• Hematogenous spread is a well-documented characteristic of RMS and often leads to metastasis in
distant organs, particularly the lungs.
Incorrect Options:
Option A - The most common histological type of RMS in adults is alveolar RMS:
• This statement is incorrect. While alveolar RMS is a subtype of RMS, it is not the most common
histological type in adults. Pleomorphic variant is the most common histological type of RMS in adults.
Option B - The most common histological type of RMS is the spindle cell variant:
• This statement is incorrect. The spindle cell variant is a subtype of RMS, but it is not the most
common histological type overall. Embryonal RMS is the most common histological type of RMS.
Option D - The most common site of metastasis for RMS is bone:
• This statement is incorrect. While bone metastasis can occur in RMS, the most common site of
metastasis for RMS is the lungs. RMS tends to spread hematogenously to distant organs, with the
lungs being the most frequent site of metastasis.

Solution for Question 14:


Correct Option D - The initial investigation of choice for diagnosing DFSP is excisional biopsy:
• The correct initial investigation for diagnosing DFSP is an incisional biopsy, not excisional biopsy. An
incisional biopsy involves taking a small sample of tissue from the lesion for examination under a
microscope. This allows for accurate diagnosis and histologic assessment without compromising
subsequent surgical planning.
• Excisional biopsy, on the other hand, involves removing the entire lesion, which may not be practical
or advisable for DFSP due to its infiltrative nature and potential involvement of surrounding structures.
Incorrect Options:
Option A - DFSP typically involves the dermis and subcutis:
• This statement is correct. DFSP typically arises from the dermis and extends into the subcutaneous
tissue. It is characterized by infiltrative growth and often involves surrounding structures.
Option B - Most DFSP cases exhibit a ring chromosome, composed of a
translocated portion of chromosomes 17 and 22:
• This statement is correct. Most DFSP cases are associated with a characteristic chromosomal
abnormality involving a fusion of the COL1A1 and PDGFB genes, forming a ring chromosome
composed of portions of chromosomes 17 and 22.

Page 22

1770
Option C - The most common site for DFSP is the trunk, followed by the extremities:
• This statement is correct. The most common site is the trunk, followed by the extremities and then
head and neck region.

Solution for Question 15:


Correct Option C - Increased secretion of parathyroid hormone-related protein:
• The primary factor responsible for hypercalcemia of malignancy is increased secretion of parathyroid
hormone-related protein (PTH-rp) by tumor cells.
• PTH-rp mimics the action of parathyroid hormone (PTH), leading to increased calcium release from
bone and enhanced calcium reabsorption from the distal convoluted tubule (DCT).
Incorrect Options:
Option A - Increased release of phosphate from bone:
• Hypercalcemia of malignancy is primarily caused by increased calcium release from bone, not
phosphate.
Option B - Decreased calcium reabsorption from the distal convoluted tubule:
• Hypercalcemia of malignancy is characterized by increased calcium reabsorption from the DCT, not
decreased reabsorption.
Option D - Enhanced activation of vitamin D in the kidney:
• While vitamin D plays a role in calcium homeostasis, hypercalcemia of malignancy is primarily driven
by factors such as increased PTH-rp secretion rather than enhanced vitamin D activation.

Solution for Question 16:


Correct Option B - Typhlitis; indicated if no improvement within 24 hours of antibiotics:
• The clinical presentation of fever, right lower quadrant tenderness, bloody diarrhea, and a thickened
cecal wall on imaging is highly suggestive of typhlitis, also known as neutropenic colitis.
• In typhlitis, medical management with antibiotics is the mainstay of treatment. Surgical intervention is
warranted if there is no improvement within 24 hours of antibiotic therapy or if perforation occurs, which
can lead to peritonitis.
Incorrect Options:
Option A - Diverticulitis; indicated if perforation occurs:
• Diverticulitis: Diverticulitis typically presents with left lower quadrant pain and is not specifically
associated with neutropenia or chemotherapy. Surgical intervention for diverticulitis is indicated if
perforation occurs but is not solely based on the clinical scenario described.
Option C - Crohn's disease; indicated if perforation leads to peritonitis:
• Crohn's disease: Crohn's disease presents with various symptoms including abdominal pain, diarrhea,
and occasionally fever, but it is not specifically associated with neutropenia or chemotherapy. Surgical
intervention in Crohn's disease is typically based on complications such as strictures or fistulas, rather

Page 23

1771
than acute perforation.
Option D - Ischemic colitis; indicated if imaging shows bowel necrosis:
• Ischemic colitis: Ischemic colitis presents with abdominal pain and bloody diarrhea but is not
specifically associated with neutropenia or chemotherapy. Surgical intervention in ischemic colitis may
be indicated if there is evidence of bowel necrosis, but this is not mentioned in the clinical scenario
provided.

Page 24

1772
Previous Year Questions
1. Which drug is the preferred choice for reducing uric acid levels to prevent kidney damage in tumor
lysis syndrome?
A. Furosemide
B. Rasburicase
C. Corticosteroids
D. Febuxostat
----------------------------------------
2. A 55-year-old alcoholic patient presents with sudden and unintentional weight loss. Lab
investigations revealed serum ALP 240 IU/L, alpha-fetoprotein 600 ng/ml, and normal AST and ALT
serum levels. What is the most likely diagnosis?
A. Hepatic adenoma
B. Cholangiocarcinoma
C. Hepatocellular carcinoma
D. Alcoholic hepatitis
----------------------------------------
3. What is the meaning of neoadjuvant chemotherapy in the context of a 54-year-old woman diagnosed
with stage 2A cervical cancer?
A. Chemotherapy is given along with radiation.
B. Chemotherapy is given during surgery.
C. Chemotherapy is given before radical surgery to reduce the bulk of the tumor
D. Chemotherapy is given after radical surgery for micrometastases
----------------------------------------
4. What is the most probable diagnosis for a 40-year-old male patient who has a swelling that has been
gradually increasing in size over the last two years, with a variable consistency and complete mobility
upon examination?

A. Dermoid cyst
B. Parotid tumor
C. Sebaceous cyst

1773
D. Cervical lymph node
----------------------------------------
5. ABCDE rule applies for:
A. Malignant melanoma
B. BCC
C. SCC
D. Actinic keratosis
----------------------------------------
6. Paget's disease of the nipple is:-
A. Infection
B. Dermatitis
C. Neoplasia
D. Hypopigmentation
----------------------------------------
7. Cock's peculiar tumor is:-
A. Papilloma
B. Infected sebaceous cyst of scalp
C. Cylindroma
D. Squamous cell carcinoma
----------------------------------------
8. Please provide the relevant information and context for the diagnosis of this 60-year-old man.

A. Basal cell carcinoma


B. Plunging Ranula
C. Epulis
D. SCC of lip
----------------------------------------
9. Most common subtype of Rodent ulcer is:

Page 2

1774
A. Superficial
B. Cystic
C. Nodular
D. Pigmented
----------------------------------------

Correct Answers
Question Correct Answer

Question 1 2
Question 2 3
Question 3 3
Question 4 2
Question 5 1
Question 6 3
Question 7 2
Question 8 4
Question 9 3

Solution for Question 1:


Answer Option B - Rasburicase
• Rasburicase is preferred for lowering uric acid to prevent renal damage in tumor lysis syndrome.
Tumor lysis syndrome (TLS) is a potentially life-threatening complication of cancer treatment that
occurs when cancer cells release their contents into the bloodstream. This can lead to a buildup of uric
acid in the blood, which can cause acute kidney injury and other complications.
• Rasburicase is a recombinant urate oxidase enzyme that catalyzes the conversion of uric acid to
allantoin, a more soluble and less toxic compound that the kidneys can excrete. It is the preferred drug
for preventing and treating hyperuricemia in tumor lysis syndrome, as it rapidly lowers uric acid levels
and reduces the risk of renal damage.
Incorrect Choices:
• Option a. Furosemide is a loop diuretic used to treat fluid retention and edema caused by various
medical conditions such as heart failure, liver cirrhosis, and kidney disease. It does not lower uric acid
levels and is not indicated for the prevention of renal damage in tumor lysis syndrome.
• Option c. Corticosteroids - It is incorrect as corticosteroids have no renal protective role in tumor lysis
syndrome.
• Option d. Febuxostat is another xanthine oxidase inhibitor that is used to lower uric acid levels in
patients with gout. It is similar to allopurinol in its mechanism of action and is not the preferred drug for
preventing renal damage in tumor lysis syndrome.

Page 3

1775
Solution for Question 2:
Correct Option C - Hepatocellular carcinoma:
• An old alcoholic patient with sudden and unintentional weight loss, along with raised serum levels of
alpha-fetoprotein, i.e., 600 ng/ml, is a tumor marker and is suggestive of hepatocellular Carcinoma
(HCC).
Incorrect Choices:
Option A - Hepatic Adenoma: This benign liver tumor is usually associated with the use of oral contrac
eptives or anabolic steroids. The Alpha-Fetoprotein (AFP) level is typically normal in hepatic adenoma.
Option B - Cholangiocarcinoma: This malignancy arises from the bile ducts, not the liver parenchyma.
Tumor marker for chalangiocarcinoma is CA 19-9. AFP is not elevated in cholangiocarcinoma.
Option D - Alcoholic Hepatitis: While this condition can present with abnormal liver function tests, the A
FP level is usually not elevated in alcoholic hepatitis unless there is concurrent HCC.

Solution for Question 3:


• Neoadjuvant chemotherapy refers to the administration of therapeutic agents before a main treatment.
• The purpose of neoadjuvant chemotherapy is to reduce the size of the tumor, making it easier to
remove surgically and increasing the chances of complete removal. It can also potentially make surgery
less extensive and decrease the chance of cancer recurrence.
Incorrect Choices:
• Option a. Concomitant chemotherapy (not neoadjuvant) is often given along with radiation therapy to
increase its effectiveness. This is known as chemoradiation.
• Option b. Intraoperative chemotherapy, not neoadjuvant, is given during surgery directly into the area
where the tumor was to kill any remaining cancer cells.
• Option d. Adjuvant chemotherapy is given after primary treatments like surgery to kill any undetected
cancer cells that may be left in the body.

Solution for Question 4:


• The described clinical scenario of a slowly growing, variable consistency, and fully mobile swelling as
in the image in a 40-year-old male is most suggestive of a parotid tumor.
• The parotid gland is the most common site of salivary gland tumors, and these tumors often present
as painless, slowly enlarging masses in the cheek or jaw area.

Incorrect Choices:
Option a. Dermoid cyst: These are slow-growing benign cysts that are typically present at birth. They u
sually have a uniform consistency, unlike the variable consistency noted in this case.
• External Angular Dermoid : (Most common)

Page 4

1776
• Internal Angular Dermoid Present is relation to fronto – zygomatic suture Painless swelling Soft,
smooth, non – tender CT is done to rule out dural attachment
• Present is relation to fronto – zygomatic suture
• Painless swelling
• Soft, smooth, non – tender
• CT is done to rule out dural attachment
• Treatment · Investigation of choice: surgical excision
• Present is relation to fronto – zygomatic suture
• Painless swelling
• Soft, smooth, non – tender
• CT is done to rule out dural attachment
Option c. Sebaceous cyst: These are typically located on the skin surface and are filled with keratin, a
type of protein found in skin cells. They are usually small and have a uniform consistency.
• Proliferation of epidermal cells
• Content: White creamy material – desquamated epithelial cells & keratin
• Black colored punctum is seen in cyst except cysts present in scrap & scrotum
• Misnomer because not of sebaceous origin and content is not sebum.

Option d. Cervical lymph node: While swollen lymph nodes can cause lumps, they usually occur due to
an infection or a malignancy.

Solution for Question 5:


Correct Option A.
A. Malignant melanoma: The ABCDE rule is commonly used to assess and identify potential signs of m
alignant melanoma, a type of skin cancer. The rule helps individuals and healthcare professionals reco
gnize certain characteristics of moles or skin lesions that may indicate the presence of melanoma. The
ABCDE stands for:
• A: Asymmetry - Melanomas often have an irregular shape or asymmetrical appearance.
• B: Border irregularity - The borders of melanomas may be uneven, jagged, or poorly defined.
• C: Color variation - Melanomas may display various colors, including shades of brown, black, blue,
red, or white.
• D: Diameter - Melanomas are generally larger than 6 millimeters in diameter, but they can be smaller.
• E: Evolution - Any changes in size, shape, color, or symptoms over time are important to note.
Incorrect Options:
Option B. BCC (Basal cell carcinoma): The ABCDE rule is not typically applied to basal cell carcinoma,
the most common type of skin cancer. Basal cell carcinomas often have different characteristics comp
ared to melanoma, such as a pearly or translucent appearance, rolled edges, and ulceration or bleedin

Page 5

1777
g. The ABCDE rule is more specific to identifying potential signs of melanoma.
Option C. SCC (Squamous cell carcinoma): Similar to basal cell carcinoma, the ABCDE rule is not com
monly used for squamous cell carcinoma. Squamous cell carcinomas may have different features, incl
uding a scaly, crusted appearance, and may be more associated with chronic sun damage and precan
cerous lesions.
Option D. Actinic keratosis: Actinic keratosis refers to precancerous skin lesions that are caused by lon
g-term sun exposure. These lesions typically do not follow the ABCDE rule. Actinic keratosis is charact
erized by rough, scaly patches on the skin, which may be red, pink, or brown in color.

Solution for Question 6:


Correct Option C:
• Neoplasia: The correct answer is neoplasia. Paget's disease of the nipple is a rare form of breast
cancer that primarily affects the skin of the nipple and areola. It typically presents as an eczema-like
rash that affects the nipple and surrounding areas. The underlying cause is the presence of malignant
cells in the skin of the nipple, which can extend from an underlying breast ductal carcinoma.
Incorrect Options:
Option A: Infection: Paget's disease of the nipple is not caused by an infection. It is a
condition that involves neoplastic (cancerous) changes in the cells of the nipple and areola.
Option B: Dermatitis: Paget's disease of the nipple can sometimes mimic dermatitis or eczema due to it
s characteristic appearance. It can cause redness, scaling, itching, and crusting of the nipple and areol
a. However, unlike dermatitis, Paget's disease is a neoplastic condition involving cancerous cells
Option D: Hypopigmentation: Hypopigmentation refers to a loss of skin color or decreased pigmentatio
n. Paget's disease of the nipple does not typically present with hypopigmentation. Instead, it commonly
causes erythematous (red) changes in the nipple and areola.

Solution for Question 7:


Correct Option B:
• Infected sebaceous cyst of the scalp: A sebaceous cyst is a closed sac beneath the skin that is filled
with a cheese-like or oily material called sebum. When a sebaceous cyst becomes infected, it can
cause localized pain, redness, swelling, and tenderness. The infection can be treated with antibiotics,
and in some cases, surgical drainage may be required.
Incorrect Options:
Option A: Papilloma: A papilloma is a
benign tumor that consists of overgrown epithelial cells. It typically forms a small, wart-like growth on th
e skin or mucous membranes. Papillomas are usually harmless and can occur in various locations in th
e body.
Option C: Cylindroma: A cylindroma is a
benign tumor that arises from the sweat glands. It usually appears as a slow-growing, firm, and someti
mes painful nodule on the skin, particularly in the head and neck region. Cylindromas are typically not

Page 6

1778
cancerous but can cause cosmetic concerns or discomfort.
Option D: Squamous cell carcinoma: Squamous cell carcinoma is a type of skin cancer that arises from
the squamous cells in the outermost layer of the skin. It often presents as a scaly, red, or ulcerated les
ion that may bleed or crust. Squamous cell carcinoma can be locally invasive and has the potential to s
pread to other parts of the body if left untreated.

Solution for Question 8:


Correct Option D:
• The diagnosis for this 60-year-old man is SCC (Squamous Cell Carcinoma) of the lip, as indicated by
the provided image.
• Based on the image showing SCC of the lip, it is evident that the patient has a malignant tumor
involving the lip. Squamous cell carcinoma is a type of skin cancer that commonly occurs on
sun-exposed areas, including the lips. It typically presents as a non-healing ulcer or a raised, crusted
lesion with irregular borders.
Incorrect Option:
Option A: Basal cell carcinoma is another common type of skin cancer, but it typically presents with diff
erent clinical features and is less likely to be associated with lip involvement.
Option B: Plunging ranula refers to a
cystic lesion that occurs in the floor of the mouth and is not related to lip carcinoma.
Option C: Epulis refers to a benign tumor or overgrowth of tissue, which is different from the malignant
nature of the lesion seen in the image.

Solution for Question 9:


Correct Option C:
• The most common subtype of Rodent ulcer is Nodular.
• Rodent ulcer, also known as basal cell carcinoma (BCC), is a common type of skin cancer that
typically occurs on areas of the skin exposed to the sun, such as the face, neck, and hands. BCC is the
most common form of skin cancer and is characterized by slow-growing, locally invasive tumors.
Incorrect Option:
Option A: Superficial BCC: This subtype is characterized by a
red, scaly patch or plaque on the skin. It tends to grow slowly and can be mistaken for eczema or a
patch of psoriasis.
Option B: Cystic BCC: This subtype appears as a
translucent or pearly nodule with central cystic areas. It may have a
smooth, shiny surface and can resemble a benign cyst.
Option D: Pigmented BCC: Is rare variant

Page 7

1779
Trauma Part 1
1. A 30-year-old male presents to the emergency department with a history of abdominal gunshot
injury. The pulse is 130/min, blood pressure is 80/60 mmHg, and respiratory rate is 35/min. On
abdominal examination, generalized abdominal tenderness with guarding is present. Which of the
following compensatory mechanisms is currently acting in this patient?
(or)
A 30-year-old male has generalized abdominal tenderness with guarding is present with pulse
-130/min, blood pressure- 80/60 mmHg, and respiratory rate - 35/min.Which of the following
compensatory mechanisms is currently acting in this patient?
A. The juxtaglomerular apparatus in the kidney produces renin
B. There is afferent arteriolar constriction and efferent arteriolar dilatation
C. Increased excretion of sodium by the kidneys
D. Increased potassium reabsorption by the kidneys
----------------------------------------
2. Choose the option which correctly matches patients with their triage color codes in disaster
management.
(or)
Twenty patients were rushed into the emergency department of the hospital after a landslide occurred
in the area.Which of the following color codes and patients are correctly matched as per the triage used
in disaster management?
A. Red - Pulseless patient with no breathing
B. Black - patient with abrasions over hands and legs
C. Yellow- patient with an amputated leg
D. Green- patient with severe respiratory distress
----------------------------------------
3. What is the management of grade I splenic injury in a child?
(or)
An 8-year-old boy presents to the emergency department with a complaint of pain over the left flank
and tip of the left shoulder after a motor vehicle accident one hour ago. Vital signs are within normal
limits and examination is unremarkable except for left flank tenderness. A plain CT scan of the thorax
and abdomen reveals splenic injury with a mild peri-splenic collection. Which of the following is the
most appropriate line of management for this child?
A. Conservative management
B. Explorative laparotomy
C. Interventional radiology followed by angiography and embolization
D. Splenectomy
----------------------------------------
4. A 29-year-old man presents to the emergency department 45 minutes after sustaining a blow to the
right side of his head when his motorcycle slid on gravel and flipped on the side of the road. After the

1780
accident, the patient noted burning pain in his neck. On examination, complete quadriplegia is noted
with a sensory level at C4.The patient has not passed urine since then, and tenderness is elicited in the
cervical region. Which of the following is the next step in the management of this patient?
(or)
What is the next step in the management of a patient with complete quadriplegia and sensory level
noted at C4 with cervical tenderness after a road traffic accident?
A. The doctor should order a cervical x-ray and shift the patient from the trolley by himself.
B. The patient should not be shifted and a portable x-ray machine should be used after neck
stabilization.
C. The doctor will instruct the radiographer to take cervical and chest x-ray.
D. The doctor will instruct the radiographer to take cervical x-ray-AP and lateral view without any
cervical support
----------------------------------------
5. A 29-year-old blind patient presents to the emergency department after sustaining a fall from a height
of 5 m over a standing barrier in a stadium during a cricket match. The pulse is 104/min, blood pressure
is 100/60 mmHg, respiratory rate is 38/min, and the GCS is 3/15. On systemic examination, diminished
breath sounds are noted on the right side. Which of the following ratios of RBCs, fresh frozen plasma,
and platelets should be given to this patient for transfusion?
(or)
Which of the following ratios of RBCs, fresh frozen plasma, and platelets should be given to a trauma
patient for transfusion?
A. 1:1:3
B. 1:1:1
C. 1:1:2
D. 1:1:4
----------------------------------------
6. Match the following: 1. Most common injured organ in blunt trauma abdomen A. Liver 2. Most
common injured organ in penetrating trauma abdomen B. Spleen 3. Most common injured organ in
gunshot wound C. Duodenojejunal junction 4. Most common injured structure in seat belt injury D.
Small intestine 5. Most common injured site in deceleration injury E. FAST 6. The First investigation
was done in blunt trauma abdomen F. CECT 7. Gold standard investigation in stable patients of blunt
trauma abdomen G. Mesentery
(or)
Match the following: 1. Most common injured organ in blunt trauma abdomen A. Liver 2. Most common
injured organ in penetrating trauma abdomen B. Spleen 3. Most common injured organ in gunshot
wound C. Duodenojejunal junction 4. Most common injured structure in seat belt injury D. Small
intestine 5. Most common injured site in deceleration injury E. FAST 6. The First investigation was done
in blunt trauma abdomen F. CECT 7. Gold standard investigation in stable patients of blunt trauma
abdomen G. Mesentery
1. Most common injured organ in blunt trauma abdomen A. Liver
2. Most common injured organ in penetrating trauma abdomen B. Spleen
3. Most common injured organ in gunshot wound C. Duodenojejunal junction

Page 2

1781
4. Most common injured structure in seat belt injury D. Small intestine
5. Most common injured site in deceleration injury E. FAST
6. The First investigation was done in blunt trauma abdomen F. CECT
7. Gold standard investigation in stable patients of blunt trauma abdomen G. Mesentery

A. 1-A; 2-B; 3-D; 4-G; 5-C; 6-E; 7-F


B. 1-A; 2-D; 3-B; 4-G; 5-C; 6-E; 7-F
C. 1-B; 2-A; 3-D; 4-G; 5-C; 6-E; 7-F
D. 1-B; 2-A; 3-D; 4-C; 5-G; 6-E; 7-F
----------------------------------------
7. A 29-year-old man without significant medical history and weighing around 100 kg presents to the
emergency department after sustaining a severe motorbike collision. He was initially admitted to a
general hospital after being transported by a fire rescue team. The blood pressure is 80/60 mmHg, the
pulse is 120 beats/min, and the respiratory rate is 32/min. Which of the following guidelines should be
followed regarding the imaging in the primary survey of this patient?
(or)
Which of the following should be done in the primary survey of an unstable trauma patient?
A. A cervical X-ray is not mandatory
B. Chest X-ray and pelvic X-ray should not be done as a part of the primary survey
C. Hemodynamically unstable patients should also be sent for a CT scan
D. All patients should have chest X-ray PA view only
----------------------------------------
8. A 28-year-old patient presents to the emergency department after sustaining multiple injuries during
a road traffic accident 30 minutes back. The pulse is 107/min, blood pressure is 75/55 mmHg,
respiratory rate is 38/min, and the GCS is 4/15. On systemic examination, diminished breath sounds
and hyper resonance on percussion are noted on the left side of the chest. Which of the following is the
next best step in the management of this patient?
(or)
What is the next best step in the management of tension pneumothorax in a trauma patient?
A. Needle thoracostomy
B. Conservative management
C. Application of partially occlusive dressings
D. Finger thoracostomy
----------------------------------------
9. A 25-year-old male had a motorcycle collision and the patient has mild tenderness in the left
hypochondrium. A focused assessment with sonography in trauma is shown. Which of the following
does it suggest?
(or)
A 25-year-old male presents to the emergency department with a history of road traffic accidents due to
a motorcycle collision with a heavy vehicle. Vitals are within normal limits. On abdominal examination,

Page 3

1782
no guarding or rigidity is found, although the patient has mild tenderness in the left hypochondrium. A
focused assessment with sonography in trauma is done, and the findings are shown in the image
below: Which of the following does it suggest?

A. Positive FAST
B. Negative FAST
C. Equivocal FAST
D. Inconclusive FAST
----------------------------------------
10. A 44-year-old man presents to the emergency department after a vehicle’s transmission system fell
on his chest, squeezing his torso between the heavy item and the ground. The pulse is 130 beats per
minute, blood pressure is 80/40mmHg, and respiratory rate is 33/min. Which of the following conditions,
if present in this patient, is an indication for emergency thoracotomy?
(or)
Which of the following is an indication for emergency thoracotomy?
A. Flail chest
B. Pulmonary edema
C. Tension pneumothorax
D. Massive hemothorax
----------------------------------------
11. A 27-year-old male presented to the emergency room after a motorcycle accident. Vital signs were
within normal limits, and abdominal pain and two incised wounds on the right side of the abdomen were
found on physical examination. CT scan showed multiple lacerations on the right renal parenchyma
with mild urinoma. With the result of the CT and the patient’s clinically stable status, a conservative
approach was decided. Now on follow-up after 12 days, the urinoma has persisted, but the patient is
stable, and there is no fever. Which of the following is the best next step in the management of this
patient?
(or)
Which is the next best step in the management of a stable patient with persisting urinoma after 12 days
of an RTA?
A. Percutaneous exploration and repair
B. Wait and watch

Page 4

1783
C. J–shaped urinary stent
D. Percutaneous nephrectomy
----------------------------------------
12. A 45-year-old male presents to the emergency department after an accidental shotgun trauma to
the abdomen. The entry wound is at the umbilical region. The pulse is 122 bpm, the systolic blood
pressure is 95 mmHg, and the respiratory rate is 32/min. Which of the following measures should be
taken in this patient in accordance with the CRASH-2 Trial?
(or)
Which of the following measures should be taken in a patient with shotgun trauma to the abdomen in
accordance with the CRASH-2 Trial?
A. The dose of tranexamic acid to be administered is 5 gm IV over 10 minutes followed by 5 gm over 8
hours
B. Alteplase should be administered within 3 hours of injury
C. Tranexamic acid should be administered within 6 hours of injury
D. Tranexamic acid should be given to all trauma patients suspected to have a significant hemorrhage,
including SBP < 110 mm Hg or PR > 110/min
----------------------------------------
13. A 17-year-old male presents to the emergency department with abdominal gunshot injuries after a
failed suicide attempt at home. On examination, vital signs are stable. There is a 1 cm inlet wound 2 cm
above the level of the umbilicus on the left midclavicular line. There is tenderness all over the abdomen
with guarding, and bowel sounds are sluggish. The exit wound is noted at the left renal angle. Labs
show mild leukocytosis. Initial X-rays of the abdomen and chest are inconclusive. What is the next step
in the management of this patient?
A. FAST
B. Exploratory laparotomy
C. Local wound exploration and suturing
D. Serial Physical examination and erect abdominal X-ray to note bowel perforation
----------------------------------------
14. A 30-year-old male presents to the emergency department after a roadside accident. On admission,
his pulse rate is 120/minute, and his BP is 100/60 mmHg. USG examination reveals a laceration of the
lower pole of the spleen with hemoperitoneum. He is resuscitated with blood and fluid. Two hours later,
his pulse is 84/minute, and his BP is 120/70 mm Hg. What is the most appropriate course of
management in this case?
(or)
What is the most appropriate course of management in a stable patient with splenic laceration?
A. Exploring the patient followed by splenectomy
B. Exploring the patient followed by excision of the lower pole of the spleen
C. Splenorrhaphy
D. Continuation of conservative treatment under close monitoring and subsequent surgery if condition
deteriorates
----------------------------------------

Page 5

1784
15. A 17-year-old boy after a road traffic accident. Abdominal examination is normal. After adequate
resuscitation, pulse rate is 80/min, and BP is 110/70 mm Hg. Abdominal CT reveals a 1 cm deep
laceration in the left lobe of the liver extending from the dome more than halfway through the
parenchyma. What is the appropriate management at this time?
(or)
A 17-year-old boy presents to the hospital after a road traffic accident. Abdominal examination is
normal. After adequate resuscitation, his pulse rate is 80/min, and his BP is 110/70 mm Hg. Abdominal
CT reveals a 1 cm deep laceration in the left lobe of the liver extending from the dome more than
halfway through the parenchyma. What is the appropriate management at this time?
A. Conservative treatment
B. Abdominal exploration and packing of hepatic wounds
C. Abdominal exploration and ligation of left hepatic artery
D. Left hepatectomy
----------------------------------------
16. A 36-year-old man has a bilateral pulmonary contusion with small hemothorax, a renal contusion.
Laboratory analysis confirms coagulopathy associated with moderate metabolic acidosis. What is the
next step in the management?
(or)
A 36-year-old man presents to the hospital after sustaining a severe motorbike collision. On
examination, the patient is hemodynamically stable and alert. X-rays show multiple fractures. A
whole-body CT shows a bilateral pulmonary contusion with small hemothorax, a renal contusion, and
multiple pelvic fractures. Progressively, the patient becomes hemodynamically unstable. Three units of
packed erythrocytes are given together with 1 g of tranexamic acid and 1000 ml hydroxyethyl starch.
Laboratory parameter analysis from the blood sample drawn at admission confirms coagulopathy
associated with moderate metabolic acidosis. What is the next step in the management of this patient?
A. The patient should be prepared for damage control surgery
B. The patient should be hemodynamically stable for any operative procedure; hence aggressive
resuscitation is recommended
C. Coagulopathy has to be corrected as a part of resuscitation, and then operative control of
hemorrhage & contamination is performed
D. If hemodynamic stability is achieved, conservative management is enough for this patient
----------------------------------------
17. Which of the following is a feature of the zone 2 in the neck?
(or)
An 11-year-old girl presents to the hospital with a penetrating neck injury. During break time at her
school, she fell and lodged the sharpened end of a pencil into the left side of her neck. She is urgently
transported by an ambulance with the pencil kept in place. On examination, a pencil protrudes from the
left side of the neck without any external bleeding or hematoma from the puncture site. The injury
involves zone 2 of the neck. Her initial vital signs are normal, and there is no evidence of respiratory
distress or stridor. Which of the following is a feature of the zone involved in this injury?
A. Distal carotid vessels are located in this zone
B. It is the most surgically accessible zone

Page 6

1785
C. Has the highest mortality rate if injured
D. Thoracic duct is located in this zone
----------------------------------------
18. A 37-year-old male presents to the hospital after a roadside accident. He has an injury to the front
of the head. On examination, white transparent fluid is coming out of the nose. The patient tries to sniff
the fluid back but cannot do so. Which of the following bone fractures result in this patient’s condition?
(or)
Which fracture results in CSF rhinorrhoea?
A. Anterior cranial fossa fracture
B. Middle cranial fossa fracture
C. Posterior cranial fossa fracture
D. Occipital bone fracture
----------------------------------------

Correct Answers
Question Correct Answer

Question 1 1
Question 2 3
Question 3 1
Question 4 2
Question 5 2
Question 6 3
Question 7 1
Question 8 T,F,F,F,F
Question 9 1
Question 10 4
Question 11 3
Question 12 4
Question 13 2
Question 14 4
Question 15 1
Question 16 1
Question 17 2
Question 18 1

Solution for Question 1:

Page 7

1786
Option A: The juxtaglomerular apparatus in the kidney produces renin
• This patient with generalized abdominal tenderness with guarding, a pulse rate of 130/min and blood
pressure of 80/60 mmHg following a gunshot wound is in a state of hemorrhagic shock.
• In response to vasoconstriction and a decrease in blood flow, the juxtaglomerular apparatus in the
kidney produces the enzyme renin, which generates angiotensin I.
• In the lungs, the angiotensin-converting enzyme converts angiotensin I to angiotensin II, which
stimulates an increased sympathetic drive and causes the adrenal medulla to release catecholamines.
The catecholamines are responsible for the elevated glucose levels seen in hemorrhagic shock.
• In the adrenal cortex, the zona glomerulosa produces aldosterone in response to angiotensin II, which
causes increased sodium reabsorption and potassium excretion. Angiotensin II also has a direct effect
on renal tubules, reabsorbing sodium.
• Antidiuretic hormone is released from the pituitary, causing free water retention at the kidney's level.
Option B: There is afferent arteriolar constriction and efferent arteriolar constriction
• GFR is maintained by prostaglandin-mediated dilatation of afferent arterioles and angiotensin
II-mediated constriction of efferent arterioles.
• Once the mean arterial pressure falls below 80mmHg, renal autoregulation fails, and GFR declines
steeply.
Option C: Increased excretion of sodium by the kidneys
• As a compensatory mechanism in hypovolemic shock, increased aldosterone concentration occurs
secondary to angiotensin II, resulting in increased potassium excretion.
Option D: Increased potassium reabsorption by the kidneys
• Increased angiotensin II concentration in hypovolemic or hemorrhagic shock results in an increased
aldosterone concentration. Aldosterone causes increased absorption of sodium in the kidneys.

Solution for Question 2:


Option C: Yellow- patient with an amputated leg
• Triage means to "sort" and is specially used for mass-casualties.
• Involves prioritizing victims based on: severity of injury, likelihood of survival, & urgency of care.
• The yellow color in triage indicates stable patients who require observation in the hospital.
• A stable patient with an amputated leg will survive if treatment is delayed by a few hours and thus
labeled yellow.
Option A: Red - pulseless patient with no breathing
• The red color in triage indicates critical patients who are likely to survive only if treatment is given
immediately.
• A pulseless patient with no breathing is likely to be unsalvageable with medical help and should be
labeled black.
Option B: Black -patient with abrasions over hands and legs
• The black color in triage indicates pulseless patients with no breathing who are so severely injured
that no medical care is likely to help.

Page 8

1787
• A patient with abrasions over hands and legs only will survive even if treatment is delayed for days
and should be labeled green.
Option D: Green- patient with severe respiratory distress
• The green color in triage indicates stable patients who are likely to survive even if treatment is delayed
for hours to days.
• A patient with severe respiratory distress can have major injuries like tension pneumothorax, which
could be life threatening without immediate treatment and should be labeled red.

Solution for Question 3:


Option A: Conservative management
• The child has suffered a mild splenic injury from the road traffic accident resulting in pain over the left
flank and tip of the left shoulder (referred pain via phrenic nerve)
• Isolated splenic /hepatic injury in a child is most commonly managed by conservative management.
• The intra-abdominal solid organs are particularly vulnerable to blunt trauma in children.
• Splenic injuries are relatively common in pediatric trauma. Splenic injuries are managed
conservatively unless there is evidence of hemodynamic instability.
• Those who fail to respond to nonoperative management usually do so within the first 12 hours.
Option B: Explorative laparotomy
• It is the most appropriate line of management for blunt trauma with signs of peritonitis or patients with
hemodynamic instability.
Option C:Interventional radiology followed by angiography and embolization
• Nonoperative management is the standard therapy for hemodynamically stable children with blunt
solid organ injury.
Option D: Splenectomy and liver packing
• Splenectomy is only done for severe splenic injuries in hemodynamically unstable patients when
conservative measures fail.

Solution for Question 4:


Option B: The patient should not be shifted, and a
portable x-ray machine should be used after neck stabilization
• In a trauma patient, cervical stabilization is the first step in management along with the airway.
• A cross-table lateral cervical spine x-ray is considered in the same trolley itself without shifting the
patient.
• A surgeon should accompany the patient if needed, so that patient can be transferred immediately to
OT if any hemodynamic instability occurs.
• Blunt trauma causes cord injury through direct impingement or indirect compression due to vertebral
fracture or dislocation.

Page 9

1788
• Penetrating trauma causes direct cord laceration or indirectly through ischemia or fracture.
Option A: The doctor should order a cervical x-ray and shift the patient from the trolley by himself
• The first step in the management of any trauma patient is neck stabilization.
• The spine should always be immobilized, assuming a spine injury until proven otherwise.
Option C: The doctor will instruct the radiographer to take cervical and chest X-rays
• Portable cervical X-rays after initial neck stabilization are the immediate best step in the management
of such trauma patients.
Option D: The doctor will instruct the radiographer to take a
cervical x-ray AP and lateral view without any cervical support
• It should not be practiced as it may lead to further spinal injury and worsen the outcomes.
• All interventions should be preceded by neck stabilization in trauma patients.

Solution for Question 5:


Option B:1:1:1
• In a trauma-related transfusion, the ratio of RBCs, FFP, and platelets should be 1:1:1.
• Most patients do not require more than 6 units of PRBCs and do not receive FFP.
• Patients who require 7 to 12 units of PRBCs should be delivered along with 6 units of FFP and 1 unit
of apheresis platelets.
• Preferably, the FFP and platelets should be transfused before the next 6 units of PRBCs are
transfused.
Option A, B, C:
• The ratio of RBCs, FFP, and platelets should be 1:1:1 for transfusion in trauma-related patients.

Solution for Question 6:


Option C: 1-B; 2-A; 3-D; 4-G; 5-C; 6-E; 7-F
• Most common organ injured in blunt trauma abdomen: Spleen > liver
• The most common organ injured in penetrating trauma of the abdomen (ATLS 2018 update): liver
(26%) > stomach (17%) > small intestine (12.9%)
• The most common organ injured in gunshot wounds i: Small intestine
• The most common bowel injured in blunt trauma abdomen : jejunum
• Most common structure injured in seal belt injury: Mesentery
• Most common site injured in deceleration injury: e duodeno jejunal junction
• The first investigation done in blunt trauma abdomen: FAST
• Gold standard investigation in stable patients of blunt trauma abdomen: CECT

Page 10

1789
Option A, B, D: It is not correctly matched as illustrated above

Solution for Question 7:


Option A:Cervical X-ray is not mandatory
• In the primary survey of blunt trauma patients, a cervical X-ray is not mandatory, as imaging of the
cervical spine can be done later after a complete evaluation of spinal cord injuries.
Option B:Chest X-ray and pelvic X-ray are not taken as a part of the primary survey
• Both chest X-rays and pelvic X-rays are always done as a vital part of primary survey in the
management of any trauma patient.
Option C:Hemodynamically unstable patients should also be sent for a CT scan
• CT scan is avoided in hemodynamically unstable patients.
Option D: All patients should have chest X-ray PA view only
• Patients with blunt trauma are always sent for plain radiographs of the chest and pelvis with
anteroposterior views as the patient is in the supine position.

Solution for Question 8:


Correct Option A - Needle thoracostomy:
• This patient, with unstable vital signs, diminished breath sounds, hyper resonance on percussion on
the left side of the chest, and a history of a road traffic accident, most likely has a left-sided tension
pneumothorax.
• In tension pneumothorax, positive pressure builds up in the hemithorax as air accumulates in the
pleural space through a breach in the visceral pleura, acting like a valve allowing only unidirectional
airflow. The high intrapleural pressure results in compression of the ipsilateral lung, flattening of the
hemidiaphragm, mediastinal distortion and shift, and impairment of venous return to the heart, reducing
cardiac output.
• The most appropriate immediate step in the management of tension pneumothorax is chest
decompression via needle thoracostomy by putting a needle in the 5th intercostal space anterior to the
mid axillary line in adults.
Option B:Conservative management
• It is done in hemodynamically stable patients suffering from a small-sized spontaneous
pneumothorax.
Option C:Application of partially occlusive dressings
• It is the most immediate step in the management of open pneumothorax.
• A partially occlusive dressing comprises packing 3 out of 4 sides of the lesion.
Option D: Finger thoracostomy
• It is indicated in cases of tension pneumothorax only if needle thoracostomy fails.

Page 11

1790
Solution for Question 9:
Option A: Positive FAST
• In the given image there is Collection of fluid in the peri-splenic region is elicited in USG→ Positive
FAST
• FAST (Focused Assessment with Sonography for Trauma)

Subxiphoid transverse view


Assess pericardial sac
Right upper quadrant (RUQ) longitudinal view
Assess the perihepatic region
Left upper quadrant (LUQ) longitudinal view
Assess perisplenic region
Suprapubic longitudinal & transverse view
Assess pelvis
• e-FAST (extended FAST) has two additional views, right & left thoracic views, to rule out
pneumothorax or hemothorax
Other options
Option B: Negative FAST
• FAST is said to be negative when it does not illustrate any fluid collection
Option C: Equivocal FAST
• It means that the result of the FAST is non-concordant, which is not true in this patient's case, as the
FAST is significant for the presence of fluid.

Page 12

1791
Option D: Inconclusive FAST
• If the FAST does not lead to any definite finding, it is said to be inconclusive.

Solution for Question 10:


Option D: Massive Hemothorax
Following are the indications for emergency thoracotomy related to massive hemothorax:
• Initial tube thoracostomy drainage of > 1000 ml (penetrating injury) or >1500 ml (blunt injury).
• Ongoing tube thoracostomy drainage of >200 ml/hr for 3 consecutive hours in non-coagulopathic
patients.
• Caked hemothorax despite the placement of two chest tubes
• Tracheo-bronchial injury.

Incorrect Options:
Option A:Flail chest
• It should be managed conservatively at first, with the main focus on pain management.
Option B:Pulmonary oedema
• It is managed primarily with diuretics such as furosemide to drain the excess fluid out of the body.
Option C:Tension pneumothorax
• The initial management in cases of tension pneumothorax comprises needle thoracostomy.

Solution for Question 11:


Option C: J–shaped urinary stent
• Minor renal injuries from blunt trauma account for 85% cases and do not usually require surgery.
• The bleeding stops spontaneously with bed rest and hydration.
• In this patient, the urinoma has persisted even after 12 days, so a J-shaped urinary stent is required.

Incorrect Options:
Option A: Percutaneous exploration and repair
• Indications: Persistent retroperitoneal bleeding Urinary extravasation Evidence of nonviable renal
parenchyma Renal pedicle injuries
• Persistent retroperitoneal bleeding
• Urinary extravasation
• Evidence of nonviable renal parenchyma

Page 13

1792
• Renal pedicle injuries
• Persistent retroperitoneal bleeding
• Urinary extravasation
• Evidence of nonviable renal parenchyma
• Renal pedicle injuries
Option B: Wait and watch
• The wait-and-watch approach is useful when dealing with minor renal injuries, but in this case, the
urinoma has persisted even after 12 days, which warrants interventions.
Option D: Percutaneous nephrectomy
• It is the procedure of choice in cases of large renal stones or when other initial steps have failed.

Solution for Question 12:


Option D: Tranexamic acid should be given to all trauma patients suspected to have a
significant hemorrhage, including systolic blood pressure <110 mm Hg or pulse rate > 110/min
• This patient has a pulse rate 0 of 122 bpm and systolic blood pressure of 95 mmHg and should be
administered tranexamic acid in accordance with the CRASH-2 Trial.

Incorrect Options:
Option A: The dose of tranexamic acid to be administered is 5 gm IV over 10 minutes followed by 5
gm over 8 hours
• According to the CRASH-2 Trial, the dose of tranexamic acid to be administered to this patient is as
follows:
• 1 gm IV over 10 minutes followed by 1 gm over 8 hours
Option B: Alteplase should be administered within 3 hours of injury
• Alteplase is fibrinolytic, and its administration will further worsen this patient’s hemodynamic status.
Option C: Tranexamic acid should be administered within 6 hours of injury
• According to the CRASH-2 Trial, tranexamic acid should be administered within 3 hours of injury.

Solution for Question 13:


Option B: Exploratory laparotomy
• This young patient with self-inflicted gunshot wounds has a penetrating abdominal injury.
• His intestines and visceral organs are likely injured, as evidenced by tenderness, guarding and
sluggish bowel sounds.
• The next step in management is exploratory laparotomy to ascertain the degree and site of injury and
repair the defect.

Page 14

1793
• Chest and abdominal X-rays help help determine an intestinal perforation. In the given patient,
however, the X-rays are inconclusive.
Option A: FAST
• FAST scan is used to determine intra-abdominal bleeding in cases of blunt abdominal trauma.
• In blunt trauma, there is a risk of injury to the spleen and the liver.
• Positive FAST scan is an indication of exploratory laparotomy.
• FAST is not indicated in penetrating abdominal trauma where intra-abdominal bleeding always
occurs.
Option C: Local wound exploration and suturing
• The patient has abdominal tenderness, guarding and sluggish bowel sounds.
• He also has mild leukocytosis.
• Exploratory laparotomy is indicated due to signs of peritonitis after bowel injury. Local wound
exploration and suturing are done in cases of superficial injuries with no signs of peritonitis.
Option D: Serial Physical examination and erect abdominal X-ray to look for bowel perforation
• The abdominal X-rays of this patient are already inconclusive.
• There is no indication to wait and observe by serial physical examination.
• Exploratory laparotomy should be undertaken immediately as abdominal tenderness and guarding are
present.

Solution for Question 14:


Option D: Continuation of conservative treatment under close monitoring and subsequent surgery if co
ndition deteriorates
• The patient has a laceration of the lower pole of the spleen and a positive USG scan for
hemoperitoneum.
• His vitals are normal after resuscitation with fluid and blood.
• The patient likely has a grade 1 splenic injury.
• The ideal management course is to observe the patient conservatively and repeat the FAST scan
after 72 hours.
• If intra-abdominal bleeding still occurs and vitals are deranged, then surgical treatment is indicated.
• In most cases, grade 1 and 2 splenic injuries are managed conservatively, and surgery is not
necessary.
Option A:Exploring the patient followed by splenectomy
• Exploratory laparotomy and splenectomy are indicated in high-grade splenic injuries.
• Indications: There is continuous intra-abdominal bleeding or deranged vitals. Conservative
management fails to stop the bleeding.
• There is continuous intra-abdominal bleeding or deranged vitals.
• Conservative management fails to stop the bleeding.

Page 15

1794
• There is continuous intra-abdominal bleeding or deranged vitals.
• Conservative management fails to stop the bleeding.
Option B:Exploring the patient followed by excision of the lower pole of the spleen
• High-grade splenic injuries are an indication of surgical intervention.
• The whole of the spleen is removed in a splenectomy.
• The excision of the lacerated part alone fails to stop bleeding in most cases and is not recommended.
• Surgical intervention is not yet indicated in the patient, as he can be managed conservatively.
Option C: Splenorrhaphy
• In this surgical procedure, the injured part of the spleen is repaired.
• There is a high risk of rebleeding after splenorrhaphy.
• It is done in cases where there is a minor splenic injury.
• Surgical intervention is not yet indicated in the patient, as he can be managed conservatively.

Solution for Question 15:


Option A: Conservative treatment
• This patient has a hepatic laceration after a roadside accident.
• Liver injury can occur secondary to blunt abdominal trauma.
• The laceration is 1 cm deep. It is a low-grade hepatic injury with stable vitals.
• The ideal management at this time should be conservative treatment with close observation.

Incorrect options:
Option B: Abdominal exploration and packing of hepatic wounds
• Exploratory laparotomy is done for high-grade hepatic injuries.
• The patient has continuous intra-abdominal bleeding, unstable vitals, and a positive FAST scan.
• This patient has a low-grade hepatic injury with stable vitals. He should be managed conservatively at
this stage.
Option C: Abdominal exploration and ligation of left hepatic artery
• Hepatic artery ligation is done in cases of severe hepatic bleeding.
• Liver packing fails to stop bleeding in such cases.
• This patient has a low-grade hepatic injury with stable vitals. He should be managed conservatively at
this stage.
Option D: Left hepatectomy
• Hepatectomy is done in severe hepatic trauma where the liver is crushed and non-viable.
• This patient has a low-grade hepatic injury.
• He should be managed conservatively.

Page 16

1795
Solution for Question 16:
Option A: The patient should be prepared for damage control surgery
• The patient had a severe motorbike collision. He has pulmonary and renal contusions, hemothorax
and multiple fractures.
• Labs confirm coagulopathy and metabolic acidosis.
• As the patient progressively deteriorates, he should be immediately prepared for damage control
surgery.
• This life-saving surgery is needed to control hemorrhage. Definitive surgery can be performed later.
Option B: The patient should be hemodynamically stable for any operative procedure; hence aggressiv
e resuscitation is recommended
• The patient is progressively deteriorating.
• Immediate surgical intervention is needed to control bleeding and contamination.
• Aggressive resuscitation will fail as the patient has already developed coagulopathy and metabolic
acidosis.
• This patient is a candidate for damage control surgery.
Option C: Coagulopathy has to be corrected as a
part of resuscitation, and then operative control of hemorrhage & contamination is performed
• Consumptive coagulopathy leads to death in severely injured patients.
• Coagulopathy occurs as a part of the lethal triad of death which will not resolve without surgical
intervention and control of the bleed.
• Waiting for surgical intervention increases the risk of mortality.
Option D: If hemodynamic stability is achieved, conservative management is enough for this patient
• The patient is progressively becoming hemodynamically unstable.
• Conservative management is not indicated at this stage.
• The patient will not survive unless damage control surgery is performed.

Solution for Question 17:


Option B: It is the most surgically accessible zone
• This patient has a penetrating neck injury in zone 2 of the neck.
• The 2nd zone is the most surgically accessible zone of the neck.
• The carotid vessels are deep in this zone.
• There is a low risk of injury to major vessels during surgical exploration.
Option A: Distal carotid vessels are located in this zone
• Distal carotid vessels are located in zone 3 of the neck.

Page 17

1796
• This zone is near the base of the skull.
• Distal carotid vessels are not present in zone 2.
Option C: Has the highest mortality rate if injured
• Highest mortality is associated with penetrating injury to zone 1 of the neck due to the presence of
vital structures.
• Carotid vessels are superficial in zone 1.
• Vessels are located deep in zone 2. There is a comparatively lower risk of injury and mortality in zone
2 penetrating injury.
Option D: Thoracic duct is located in this zone
• Thoracic duct is located in zone 1 of the neck.
• It drains at the junction of the left internal jugular and left subclavian vein.
• Thoracic duct is not located in zone 2 of the neck.

Solution for Question 18:


Option A: Anterior cranial fossa fracture
• This patient, with clear fluid coming out of the nose after sustaining a head injury, has CSF rhinorrhea.
• Cerebrospinal fluid circulates in the subarachnoid space around the brain and the spinal cord.
• Anterior cranial fossa fracture leads to CSF leakage into the nasal cavity.
• The patient cannot sniff back this fluid which confirms the diagnosis.
Option B: Middle cranial fossa fracture
• Middle cranial fossa fracture leads to CSF leakage into the ear.
• This is known as CSF otorrhea.
• CSF leakage into the nasal cavity occurs secondary to anterior cranial fossa fracture, not middle
cranial fossa fracture.
Option C: Posterior cranial fossa fracture
• Posterior cranial fossa fracture leads to injury to cranial nerves and the cerebellum.
• CSF rhinorrhea is due to an anterior cranial fossa fracture, not a posterior cranial fossa fracture.
Option D: Occipital bone fracture
• Fracture of the occipital bone leads to injury to the visual cortex and occipital lobe.
• Occipital bone fracture does not lead to CSF rhinorrhea.

Page 18

1797
Trauma Part 2
1. A 55-year-old woman was evaluated 3 days after ingesting bleach. She was lethargic, had diffuse
abdominal pain and tenderness and had an SBP of 80 mmHg. Pneumoperitoneum was present on a
CXR. Endotracheal intubation was performed, and massive resuscitation with 14 liters of IV crystalloid
was instituted in preparation for exploratory laparotomy. Abdominal exploration revealed multiple
perforations along the greater curvature and fundus of the stomach. Intra-op esophagogastroscopy
revealed denuded gastric mucosa and hyperemic esophageal mucosa. She underwent total
gastrectomy with stapled closure of the distal esophagus and duodenal stump. Postoperatively, the
patient was hypotensive and had persistent severe metabolic acidosis. Her SBP was consistently
below 90 mmHg for 10 hours, in spite of high-dose dopamine and norepinephrine infusions. Her
abdomen was palpably tight, and the IAP measured via a urinary bladder catheter was 21 cmH2O. Her
peak inspiratory pressure & central venous pressure have also risen. What is the next step in the
management of this patient?
(or)
What is the next step in the management of abdominal compartment syndrome?
A. Emergency Re-exploration
B. Immediate decompression
C. Hypovolemic resuscitation
D. Normovolemic resuscitation
----------------------------------------
2. A 50-year-old man comes to the emergency department because of injuries after falling off a ladder.
He describes falling about 5 feet off the ground and suffering blunt trauma to his perineum and penis,
resulting in immediate pain lasting for 5 minutes. Following the injury, the patient experienced burning
with urination and noted large amounts of hematuria. He denies pain with ambulation and does not
recall any trauma to the pelvis during the fall. Genitourinary examination reveals blood at the urethral
meatus. Which of the following is the most appropriate initial step in management?
A. Antibiotic therapy
B. Endoscopic transurethral realignment
C. Observation and admission to the hospital
D. Placement of a suprapubic catheter
----------------------------------------
3. A 30-year-old man is brought to the hospital after being involved in a motor vehicle collision. He was
an unrestrained passenger in a vehicle that was hit when another vehicle ran a red light. He is
unconscious and was intubated on the scene. He presents to the hospital with a pulse of 120/min and a
blood pressure of 85/40 mm Hg. Focused assessment with sonography for trauma (FAST) is positive
for blood within the peritoneum. He is rushed to the operating room for an exploratory laparotomy. Prior
to the procedure, the anesthesiologist places a central venous catheter to assist with rapid fluid and
blood product administration. Which of the following is the best technique to place a subclavian central
venous catheter?
(or)
Which of the following is the best technique to place a subclavian central venous catheter?
A. Insert the needle lateral to the carotid artery pulsation, pointing the needle towards the contralateral
nipple

1798
B. Insert the needle inferior to the midpoint of the clavicle, pointing the needle laterally
C. Insert the needle inferior to the midpoint of the clavicle, pointing the needle medially
D. Insert the needle medial to the sternocleidomastoid, pointing the needle laterally
----------------------------------------
4. A 24-year-old male was referred to your hospital with a history of a head-on collision between two
motorbikes 2 days prior to admission. He had sustained multiple fractures - a compound fracture of the
left proximal humerus, a fracture of the left femur, and a comminuted fracture of the left tibia and fibula.
He was conscious, oriented, with normal breathing, and hemodynamically stable. He was treated in the
intensive care unit. About fifteen hours later, he developed respiratory distress and died after a few
days. CT brain showed petechial hemorrhage. What is the most likely diagnosis?
A. Fat embolism
B. Stroke
C. Hemorrhage
D. Hypoxic ischemic encephalopathy
----------------------------------------
5. A 50-year-old man comes to the emergency department because of a leg injury following a motor
vehicle accident. His past medical history is significant for hypertension and hyperlipidemia. Physical
examination shows minor contusions and a tibia-fibula compound fracture, requiring surgery.
Post-operatively, the patient experiences severe abdominal pain and the passing of an abundance of
bright red blood and mucus from his rectum. An emergent abdominal X-ray shows thumbprinting in the
colon and dilation of the small intestine. Which of the following is the most likely structure affected?
A. Ascending colon
B. Rectum
C. Spleen
D. Splenic flexure
----------------------------------------
6. A 63-year-older man presents to the emergency department with complaints of left lateral chest pain
following a fall on his chest at home 2 hours back. He is hemodynamically stable. An urgent chest X-ray
is requested, which shows evidence of left-sided pneumothorax. His CT scan of the chest reveals
pneumothorax of more than 30% of the left hemithorax with multiple left rib fractures. Thoracostomy
with a 28-French chest tube is performed via a standard approach. To check the patency of the
intra-coastal drainage tube (ICD) tube, which of the following can be done?
(or)
To check the patency of the Intra-coastal drainage tube (ICD) tube, which of the following can be done?
A. By observing for continuous air bubbles coming out of the underwater drain
B. By observing the movement of the air-water column in the tube during respiration
C. By taking chest X-rays repeatedly
D. By auscultation
----------------------------------------
7. Which of the following is the component of the revised trauma score:

Page 2

1799
(or)
A 28-year-old athlete presented to the emergency with an open wound bleeding profusely. His blood
pressure is 100/70 mmHg and his pulse is irregular and feeble. On taking the history, the physician
learns about a traumatic sports injury. Which of the following is the component of the revised trauma
score?
A. Blood pressure
B. Heart rate
C. ISS
D. Bleeding
----------------------------------------
8. A patient is presented to the hospital after a road traffic accident. He has contusions on his chest. He
presented with respiratory distress, tachypnea and hypotension. What is the next step in the
investigation of this patient?
A. FAST
B. E-FAST
C. MRI
D. Ultrasound
----------------------------------------
9. A 17-year-old boy was referred to the emergency room with decreased consciousness after a road
traffic accident that occurred 6 hours ago . He also had nausea and a generalized seizure, which had
resolved with intravenous anticonvulsants. He does not speak but moans now and then; his eyes are
closed but open in response to pain; the right limb is not moving, but the left limb shows movement in
response to pain. Both legs are in extended posture. What is the GCS score?
A. 5
B. 7
C. 8
D. 11
----------------------------------------
10. A 20-year-old farmer was admitted 6 h after a branch of a tree struck him on the head during
tree-felling. He lost consciousness immediately. His Glasgow coma score was 8 with a 7 mm diameter
sluggishly-reacting pupil on the left eye. You suspect an epidural hematoma in this patient but a mass
casualty due to a bus accident makes getting a CT to confirm the diagnosis difficult. If you want to
perform a Burr hole in this patient, what would be the ideal site?
A. Right temporal region
B. Left temporal region
C. Right parietal region
D. Left parietal region
----------------------------------------
11. Which of the following is a cause of secondary brain injury?
(or)

Page 3

1800
Which of the following is a cause of secondary brain injury?
A. Hypercapnia
B. Hypoglycemia
C. Hyperoxia
D. Hypertension
----------------------------------------
12. Which of the following would be the most appropriate next step in the management of main
pancreatic duct transection in the middle of the pancreatic body?
(or)
A 19-year-old man fell off his skateboard, reporting a blunt injury to his upper abdomen. Abdominal CT
and magnetic resonance cholangiopancreatography (MRCP) confirmed he suffered a transection of the
main pancreatic duct in the middle of the pancreatic body. Which of the following would be the most
appropriate next step in management?
A. Nonoperative management
B. Endoscopic retrograde cholangiopancreatography (ERCP) with stenting of pancreatic duct
C. Distal pancreatectomy with splenic preservation
D. Primary repair of the pancreatic duct with closed suction drainage
----------------------------------------
13. Following a sudden impact in an accident, the 34-year-old race car driver becomes unconscious. A
CT scan shows a right space-occupying lesion. What is the most likely diagnosis?
(or)
Following a sudden impact in an accident, the 34-year-old race car driver becomes unconscious and is
admitted to the hospital. A CT scan is performed, and a right space-occupying lesion is noted as shown
below. What is the most likely diagnosis?

A. Corpus callosum injury


B. Pituitary apoplexy
C. Acute subdural hematoma
D. Acute epidural hematoma
----------------------------------------

Page 4

1801
14. After being involved in a car accident, a patient is hospitalized. His Glasgow Coma Scale score is
8/15, and he has major blunt injuries to his head, chest, and abdomen. His oxygen saturations are
subpar, at 89%, using a rebreathing mask and 15 L of oxygen. You observe bruises around his eyes
and blood-stained fluid oozing from his left ear, which creates concentric circles when spilt on a white
sheet. In order to promote oxygenation, you want to assist his airway. The first choice of airway adjunct
would be
(or)
What is the airway of choice in a patient with skull base fracture?
A. Oropharyngeal airway
B. Nasopharyngeal tube
C. Laryngeal mask
D. Intubation
----------------------------------------
15. A 31-year-old man was admitted to the emergency department after a high-speed vehicle crash. the
patient was unrestrained and thrown from the vehicle, sustaining substantial facial and chest trauma.
The patient was conscious at admission and with a BP of 80/54 mmHg. He complained of severe
right-sided shoulder and chest pain. A supine chest radiograph obtained on a portable unit showed a
left-sided hemothorax, mild widening of the superior mediastinum, and absence of a definable
aortic–pulmonary window with a right-sided deviation of the trachea. CT thorax was performed and
image is given below. What is the probable diagnosis?

A. Ruptured esophagus
B. Cardiac tamponade
C. Right lobe collapse
D. Aortic rupture
----------------------------------------
16. A 31-year-old man comes to the emergency department because of blunt trauma to his perineum
and penis from falling off a ladder. He fell approximately 5 feet and landed on concrete, resulting in
immediate pain lasting several minutes. Since the injury, the patient has experienced burning with
urination and notes a large amount of blood mixed with urine. He denies pain with ambulation and does
not recall any trauma to the pelvis during the fall. Genitourinary examination shows blood at the urethral
meatus. Which of the following is the most appropriate study to order?
A. Radionuclide cystogram
B. Retrograde urethrogram

Page 5

1802
C. Voiding cystogram
D. X-ray of the pelvis
----------------------------------------
17. A patient has symptoms of Abdominal swelling, feeling of tightness in chest, and abdominal
discomfort. His intra-abdominal pressure, when measured, was 25 mmHg after an accident. What
could be the most likely diagnosis?
(or)
A patient is presented to the ER after a roadside accident with the symptoms of abdominal swelling and
discomfort, and a feeling of tightness in chest. His intra-abdominal pressure, when measured, was 25
mmHg. What could be the most likely diagnosis?
A. Abdominal compartment syndrome
B. Hemothorax
C. Portal hypertension
D. Aortic stenosis
----------------------------------------

Correct Answers
Question Correct Answer

Question 1 2
Question 2 4
Question 3 3
Question 4 1
Question 5 4
Question 6 2
Question 7 1
Question 8 2
Question 9 3
Question 10 2
Question 11 1
Question 12 3
Question 13 4
Question 14 1
Question 15 4
Question 16 2
Question 17 1

Solution for Question 1:

Page 6

1803
Correct Option B: Immediate decompression
Abdominal compartment syndrome of Grade 3 is mentioned in the question.
Symptoms
• Anuria, increased ventilation pressure.
Treatment:
• Immediate decompression

Correct Options:
Option A: Emergency Re-exploration
Exploratory laparotomy can be used when you:
• Have serious or long-term abdominal symptoms that defy diagnosis.
• Have had major abdominal trauma, and there's no time for another test.
• Are not good candidates for laparoscopic surgery.
Option C: Hypovolemic resuscitation
• Hemorrhage is the leading cause of preventable deaths in trauma patients. Massively bleeding
patients in hypovolemic shock should be treated with damage control resuscitation principles, including
limited crystalloid, whole blood or balanced blood component transfusion allowing permissive
hypotension, preventing hypothermia, and stopping bleeding as quickly as possible.
Option D: Normovolemic resuscitation
• The practice of maintaining normovolemia and normotension during intracranial surgery is now
well-established. Hyperglycemia, which worsens the consequences of cerebral ischemia, and
hypo-osmolality (target osmolality, 290 to 320 mOsm/kg), which can increase brain oedema, should be
avoided. Colloid oncotic pressure plays an unclear role in brain oedema.
• Glucose-containing or hypo-osmolar solutions (e.g., lactated Ringer’s solution, 254 mOsm/kg) should
be avoided. Suitable choices for infusion liquids during intracranial surgery include 0.9% NaCl and
modern 6% hydroxyethyl starch 130/0.4 solutions (304 mOsm/kg) with no deleterious effect on
coagulation. The hematocrit should be kept above 28%. Fluids should be warmed at the end of the
procedure to ensure normothermia for emergence from anesthesia.

Solution for Question 2:


Correct Option D: Placement of a suprapubic catheter
• Suprapubic catheterization is the best initial step in management of patients suffering from urethral
injury following blunt pelvic trauma. Surgical intervention is delayed days or weeks after the initial injury.
• Patients who present following urethral trauma should be initially managed in the context of other
injuries. The best initial approach involves the placement of a suprapubic catheter for urinary drainage,
which does not require urethral manipulation or invasion into the pelvic hematoma if one happens to be
present. Ideally, surgical repair of posterior urethral injury is performed 6-12 weeks after injury. after
stabilization of other injuries and resolution of pelvic hematoma have occurred.

Page 7

1804
Incorrect Options:
Option A: Antibiotic therapy
• Although antibiotic therapy is often required, it is important to first take measures to preserve organ
function. It is important to establish bladder drainage, so suprapubic catheterization should take
precedence.
Option B: Endoscopic transurethral realignment
• Early endoscopic realignment using a transurethral, percutaneous transvesical, or combined
approach may be attempted, but this should be done 5-7 days post-injury. By this time, the patient's
overall condition has usually improved, and sepsis is less of a concern.
Option C: Observation and admission to the hospital
• Observation with admission to the hospital is not adequate for the treatment of urethral injury and
places the patient at risk for severe sequelae.

Solution for Question 3:


Correct Option C: Insert the needle inferior to the midpoint of the clavicle, pointing the needle medially
• When placing a subclavian central venous catheter, the needle should be inserted inferior to the
midpoint of the clavicle and pointed towards the suprasternal notch.
• When canalizing the subclavian vein, the most common approach is to insert the needle at the
midpoint of the inferior surface of the clavicle. The physician will typically insert the needle 2-3 cm
inferior to the clavicle. The needle should be pointed deep and medially towards the suprasternal notch.
• In order to avoid vascular structures and nerves during a subclavian central venous access attempt,
the needle should be inserted 2-3 cm inferior to the clavicle, pointing medially towards the suprasternal
notch.

Incorrect Options:
Option A: Insert the needle lateral to the carotid artery pulsation, pointing the needle towards the contra
lateral nipple
• The internal jugular vein is located lateral to the carotid artery. When canalizing the internal jugular
vein, it is important to palpate the carotid artery to make sure that the needle enters the vein, not the
artery. The needle should be directed towards the ipsilateral nipple in this approach. This technique is
not used for placing a central venous catheter into the subclavian vein.
Option B: Insert the needle inferior to the midpoint of the clavicle, pointing the needle laterally
• For a subclavian central line, the needle should be pointed medially towards the suprasternal notch,
not laterally.
Option D: Insert the needle medial to the sternocleidomastoid, pointing the needle laterally
• In the supraclavicular subclavian approach, the patient lies in the supine position with the head turned
away from the side of canalization. In this approach, the needle is inserted at the insertion of the
sternocleidomastoid at the clavicle. The needle should be inserted lateral to the sternocleidomastoid
rather than medial to this muscle. Insertion of the needle medial to the sternocleidomastoid would insert
the needle within the internal jugular vein, not the subclavian vein. The needle should be pointed
towards the contralateral nipple (medially) at a 10 to 15-degree angle with the plane of the body.

Page 8

1805
Solution for Question 4:
Correct Option A: Fat embolism
• RTA with long bone fractures + respiratory distress +petechiae / hemorrhage in the brain suggests fat
embolism syndrome
• Symptoms generally begin within a day. This may include a petechial rash, decreased level of
consciousness, and shortness of breath. Other symptoms may include fever and decreased urine
output. The risk of death is about 10%.
• Fat embolism most commonly occurs as a result of fractures of bones such as the femur or pelvis.
Other potential causes include pancreatitis, orthopedic surgery, bone marrow transplant, and
liposuction. The underlying mechanism involves widespread inflammation. Diagnosis is based on
symptoms.
• Treatment is supportive care. This may involve oxygen therapy, intravenous fluids, albumin, and
mechanical ventilation. While small amounts of fat commonly occur in the blood after a bone fracture,
fat embolism syndrome is rare.

Incorrect Options:
Option B: Stroke
• A stroke, sometimes called a brain attack, occurs when something blocks the blood supply to part of
the brain or when a blood vessel in the brain bursts causing focal neurological deficits.
• It is unlikely in this young male with traumatic injuries.
Option C: Hemorrhage
• Clinical symptoms, CT imaging of the brain and the RTA history with multiple long bone fractures
suggests fat embolism.
• There is no evidence suggestive of a new haemorrhage.
Option D: Hypoxic ischemic encephalopathy
• Hypoxic ischemic encephalopathy (HIE) is a type of brain dysfunction that occurs when the brain
doesn't receive enough oxygen or blood flow for a period of time. Hypoxic means not enough oxygen,
ischemic means not enough blood flow, and encephalopathy means brain disorder.

Solution for Question 5:


Correct Option D: Splenic flexure
• Ischemic colitis presents with abdominal pain and passage of blood and mucus in stools.
• The splenic flexure is a watershed area where two mesenteric arteries overlap with a few collateral
vessels and is the area most vulnerable to ischemia in periods of hemodynamic instability.
• As ischemia persists, an acute abdomen could develop with generalized abdominal pain, rigidity, and
ileus. Finally, shock can develop, leading to acidosis and hypotension, and the patients are often
critically ill. Radiographic findings of ischemic colitis may include thumbprinting and dilation of the distal

Page 9

1806
small bowel.

Incorrect Options:
Option A: Ascending colon
• The ascending colon is predominantly supplied by the superior mesenteric artery and does not lie
within the watershed region. Hypotension or other causes of decreased perfusion does not easily
cause ischemia in this segment of the colon.
Option B: Rectum
• The rectum is perfused by the internal iliac artery as well as the inferior mesenteric artery. Because of
its dual blood supply, the rectum is very infrequently involved in colonic ischemia.
Option C: Spleen
• The spleen is very commonly injured by blunt abdominal trauma, such as during a motor vehicle
accident.
• It is possible that with a high-grade splenic laceration and devascularization of segmental or hilar
vessels that supply the spleen, the abdomen could fill with blood leading to an acute abdomen.
• But, radiographic signs and symptoms are suggestive of ischemic colitis.

Solution for Question 6:


Option B: By observing the movement of the air-water column in the tube during respiration
• Chest drainage system:

• Follow-up: The patency of the chest tube is assessed by observing the oscillations in the water seal
chamber with respiratory movements. The position of the chest tube & resolution of intrapleural air or
liquid is checked by x-ray (AP & cross-table lateral). The tube should be pulled back if it crosses the
mediastinum.

Page 10

1807
• The patency of the chest tube is assessed by observing the oscillations in the water seal chamber
with respiratory movements.
• The position of the chest tube & resolution of intrapleural air or liquid is checked by x-ray (AP &
cross-table lateral). The tube should be pulled back if it crosses the mediastinum.
• Removal: Chest tubes are generally removed when there has been air or fluid drainage of <200 ml in
24 hours for >24 hours.
• Chest tubes are generally removed when there has been air or fluid drainage of <200 ml in 24 hours
for >24 hours.
• The patency of the chest tube is assessed by observing the oscillations in the water seal chamber
with respiratory movements.
• The position of the chest tube & resolution of intrapleural air or liquid is checked by x-ray (AP &
cross-table lateral). The tube should be pulled back if it crosses the mediastinum.
• Chest tubes are generally removed when there has been air or fluid drainage of <200 ml in 24 hours
for >24 hours.

Incorrect Options:
Option A: Observe continuous air bubbles coming out of the underwater drain.
• The patency of the chest tube is assessed by observing the oscillations in the water seal chamber
with respiratory movements, not byobserving continuous air bubbles from the underwater drain.
Option C: By taking an X-ray chest repeatedly
• The position of the chest tube & resolution of intrapleural air or liquid is checked by x-ray (AP &
cross-table lateral). The tube should be pulled back if it crosses the mediastinum.
• The patency of the chest tube is assessed by observing the oscillations in the water seal chamber
with respiratory movements.
Option D: By auscultation
• The presence of pneumothorax can be assessed by auscultation.
• But patency of the chest tube is assessed by observing the oscillations in the water seal chamber with
respiratory movements.

Solution for Question 7:


Option A: Blood pressure
The revised trauma score (RTS) is a
physiological scoring system used to assess the severity of trauma in a patient. It helps in predicting pa
tient outcomes and guiding initial management decisions. The RTS is based on three components: Gla
sgow Coma Scale (GCS) score, systolic blood pressure, and respiratory rate.
The correct option among the given choices is:Blood pressure
Explanation of options:
Blood pressure: The systolic blood pressure is one of the components of the revised trauma score. It is
used to assess the severity of shock or hypoperfusion in trauma patients. A

Page 11

1808
lower systolic blood pressure may indicate a
more severe injury and potentially life-threatening condition.
Heart rate: The heart rate is not a component of the revised trauma score. However, it is an essential vi
tal sign that provides valuable information about the patient's hemodynamic status and response to the
injury. An irregular and feeble pulse, as mentioned in the case, can indicate cardiovascular compromis
e and the need for immediate resuscitation.
ISS (Injury Severity Score): The ISS is a different scoring system used to assess the severity of multipl
e injuries in trauma patients. It is based on the anatomical severity of injuries to specific body regions.
While the ISS is an important tool for trauma assessment, it is not part of the revised trauma score.
Bleeding: Bleeding is not a component of the revised trauma score. However, it is a critical aspect of tr
auma assessment, and controlling bleeding is an essential part of initial trauma management. Profuse
bleeding may contribute to hypovolemic shock, which can adversely affect the patient's outcome.

Solution for Question 8:


Correct Option B: E-FAST
• A trauma patient is screened for peritoneal fluid, pericardial fluid, pneumothorax, and hemothorax
using the bedside ultrasonographic approach known as E-FAST (Extended Focused Assessment with
Sonography in Trauma).
Technique
• In detecting free intraperitoneal and intrathoracic fluid, this protocol has a high specificity and
sensitivity, over 90%.
• The extended FAST includes further testing of the anterior and lateral pleural areas to look for
pneumothorax or pleural fluid, which, unless proven otherwise in the context of trauma , is thought to be
a hemothorax. It is considered positive if any free fluid is seen during a scan.

Incorrect Options:
Option A: FAST
• The FAST (Focused Assessment with Sonography in Trauma) test visualizes 10 structures or spaces
in four locations and searches for the presence of fluid, which is considered to represent blood in
clinical scenario of trauma: Pericardial Perihepatic Perisplenic Pelvic
• Pericardial
• Perihepatic
• Perisplenic
• Pelvic
• Over the past three decades, focused assessment with sonography in trauma (FAST) has been
extensively used and researched in blunt and penetrating trauma. Before FAST, invasive procedures
such as diagnostic peritoneal lavage and exploratory laparotomy were commonly used to diagnose an
intra abdominal injury.
• The right upper quadrant (RUQ) visualizes the hepatorenal recess, also known as Morrison's pouch,
the right paracolic gutter, the hepato-diaphragmatic area, and the caudal edge of the left liver lobe.

Page 12

1809
• Since FAST does not evaluate the pleural spaces adequately, eFAST is a better alternative.
• Pericardial
• Perihepatic
• Perisplenic
• Pelvic
Option C: MRI
• MRI cannot be done in emergency conditions due to longer duration of imaging and is mostly done for
brain and spinal cord-related disorders.
• A magnetic field and radio waves produced by a computer are used in magnetic resonance imaging
(MRI) to provide precise images of thebody's organs and tissues.
Option D: Ultrasound
• An ultrasound imaging scan employs sound waves to produce a sonogram, or image, of the organs,
tissues, and other internal body components. Ultrasound does not use radiation as x-rays do.
• E FAST is a specific form of USG scan used in trauma patients to screen potential sites of fluid
accumulation after trauma.

Solution for Question 9:


Correct Option C: 8
Glasgow Coma Scale (GCS)
Eye Opening
Score
Verbal response
Motor Response
Spontaneous
4
Oriented
5
Obeys commands
6
To loud voice
3
Confused, disoriented
Localizes pain
To pain
2

Page 13

1810
Inappropriate words
Flexion (withdrawal) response to pain
No response
1
Incomprehensible sounds
Abnormal flexion posturing

Extension posturing
• Classification of acute traumatic brain injury: Severe: GCS 3 to 8. Moderate: GCS 9 to 12. Mild: GCS
13 to 15.
• Severe: GCS 3 to 8.
• Moderate: GCS 9 to 12.
• Mild: GCS 13 to 15.
• The total score thus has values between three and 15, three being the worst and 15 being the highest.
The score is the sum of the scores and the individual elements.
• An initial score of less than 5 is linked to an 80% likelihood of passing away or being in a permanent
vegetative state. A 90% chance of recovery is linked to an initial score of more than 11.
• Patients are deemed comatose if their Glasgow ComaScale score is 7 or lower. Patients are deemed
to have experienced a severe head injury if they receive a Glasgow Coma Scale score of 8 or less.
• Accommodations are made for intubated patients and those with gross facial oedema or swelling.
• In trauma, a Glasgow Coma Scale score (GCS) of 8 or less indicates a need for endotracheal
intubation. Some advocate a similar approach for other causes of decreased consciousness. However,
the loss of airway reflexes and risk of aspiration cannot be reliably predicted using the GCS alone.
• Observations should be performed and recorded half-hourly until GCS equal to 15 has been
achieved.
• Severe: GCS 3 to 8.
• Moderate: GCS 9 to 12.
• Mild: GCS 13 to 15.

Solution for Question 10:


Option B: Left temporal region
Burr hole on the side of pupillary dilatation, i.e. left temporal.
• The temporal lobes sit behind the ears and are the second largest lobe. They are most commonly
associated with auditory processing information and with the encoding of memory. The temporal lobes
are also believed to play an important role in processing affect/emotions, language, and certain aspects
of visual perception.
• The dominant temporal lobe, which is the left side in most people, is involved in understanding
language and learning and remembering verbal information. The non-dominant lobe, which is typically

Page 14

1811
the right temporal lobe, is involved in learning and remembering non-verbal information (e.g.
visuospatial material and music).
Damage to the temporal lobes can result in
• Difficulty in understanding spoken words (Receptive Aphasia)
• Disturbance with selective attention to what we see and hear
• Difficulty with identification and categorization of objects
• Difficulty learning and retaining new information
• Impaired factual and long-term memory
• Persistent talking
• Difficulty in recognizing faces (Prosopagnosia)
• Increased or decreased interest in sexual behavior
• Emotional disturbance (e.g. Aggressive behavior)

Incorrect Options:
Option A: Right temporal region
• Burr hole on the side of the pupillary dilatation or dominant hemisphere, i.e. left temporal.
Option C: Right parietal region
• The right parietal-temporal lobe is concerned with non-verbal memory.
• Right parietal-temporal lesions can produce significant changes in personality.
• Some common tests for parietal lobe function are Kimura Box Test (apraxia) and the Two-Point
Discrimination Test (somatosensory).
• Burr hole is made on the temporal lobe on the side of pupillary dilatation.
Option D: Left parietal region
• The parietal lobes take up premises in both the right and left hemispheres of the brain.
• This lobe is particularly important in integrating information from the body’s senses to allow us to build
a coherent picture of the world around us.
• The parietal lobes allow us to coordinate our movements in response to the objects in our
environment through the use of visual pathways – allowing us to process what and where things are.
• Burr hole is made on the temporal lobe on the side of pupillary dilatation.

Solution for Question 11:


Option A: Hypercapnia
Secondary brain injury is caused by the following factors:
• Hypoxia
• Hypotension; Low cerebral perfusion pressure

Page 15

1812
• ICP >20 mmHg
• Hypercapnia
• Metabolic disturbance (hyperglycemia)

Incorrect Options:
Option B: Hypoglycemia
• Hyperglycemia has been associated with secondary brain injury, not hypoglycemia.
Option C: Hyperoxia
• Hypoxia has been associated with secondary brain injury, not hyperoxia.
• Hyperoxia is a state of excess supply of O2 in tissues and organs. Oxygen toxicity occurs when the
partial pressure of alveolar O2 (PAO2) exceeds that which is breathed under normal conditions. With
continuous exposure to supraphysiologic concentrations of O2, a state of hyperoxia develops.
Option D: Hypertension
• Hypotension has been associated with secondary brain injury not hypertension.

Solution for Question 12:


Correct Option C: Distal pancreatectomy with splenic preservation
• Optimal management of pancreatic trauma is based on where the parenchymal damage is located
and whether the intrapancreatic common bile duct and pancreatic duct remain intact.
• In contrast, distal pancreatic injuries, defined as those that lie to the left of superior mesenteric
vessels, are managed based on ductal integrity.
• Patients with distal ductal disruption undergo distal pancreatectomy, preferably with splenic
preservation. In stable patients, the spleen is preserved.
• If the patient is physiologically compromised, then distal pancreatectomy with splenectomy is the
preferred approach.
• The pancreatic duct in the proximal pancreas should be individually ligated or occluded with a stapler.
Treatment of injury to the pancreatic head
• Common bile duct disruption involves division of the common bile duct superior to the first part of the
duodenum with ligation of the distal duct and reconstruction with a Roux-en-Y choledochojejunostomy.
• If there is no common bile duct disruption, central pancreatectomy with posterior wall
pancreaticojejunostomy or a Roux-en-Y pancreaticojejunostomy is done.
• Pancreaticoduodenectomy is reserved for patients with catastrophic injury to the pancreatic head with
bile duct and pancreatic duct disruption, combined injuries of pancreas and duodenum, avulsion of the
papilla of Ampulla of Vater from the duodenum and destruction of the entire second portion of the
duodenum.

Incorrect Options:
Option A: Nonoperative management

Page 16

1813
• Patients with pancreatic contusions (injuries that leave the ductal system intact) can be treated
non-operatively or with closed suction drainage if undergoing laparotomy for other indications.
• Nonoperative management of pancreatic contusion and transection diagnosed radiologically is
effective and safe.
• Pseudocysts may form after pancreatic injury, which can be managed by percutaneous drainage, if
large or symptomatic.
Option B: Endoscopic retrograde cholangiopancreatography (ERCP) with stenting of pancreatic duct
• Endoscopic retrograde cholangiopancreatography, or ERCP, identifies and treats conditions affecting
the pancreas, liver, gallbladder, and bile ducts.
• It makes use of an endoscope, which is a long, flexible, lit tube.
• Several gastrointestinal problems can be diagnosed and treated using an ERCP, a minimally invasive
interventional technique.
• The ERCP procedure uses a video camera, whereas endoscopic ultrasound creates a virtual image
using high-frequency sound waves. These two methods are frequently used to examine organs such as
the pancreas, gallbladder, and liver.
Option D: Primary repair of the pancreatic duct with closed suction drainage
• Patients with proximal pancreatic injuries, defined as those that lie to the right of the superior
mesenteric vessels, are also managed with closed suction drainage.
• Patients with pancreatic contusions (injuries that leave the ductal system intact) can be treated
non-operatively or with closed suction drainage if undergoing laparotomy for other indications.
• A closed suction drain removes fluids that build up in body areas after surgery or when there is an
infection.
• Although there is more than one brand of closed suction drain, this drain often used is called a
Jackson-Pratt, or JP, drain.

Solution for Question 13:


Option D: Acute epidural hematoma
• The above features & CT findings with a biconvex mass are suggestive of an acute epidural
hematoma.
• Acute EDH is associated with high-energy trauma, underlying fracture, and lucid interval- brief
duration of consciousness before the patient deteriorates again
• Acute subdural hematomas occur most commonly due to a head trauma tearing the middle meningeal
artery and blood collecting over the dura mater.
• They generally imply a much more severe injury to the brain itself than in the case of their subdural
counterpart.
• For this reason, they are associated with cerebral contusions in over 30% of cases.
Option A: Corpus callosum injury
• Corpus callosum injury predominantly occurs in the posterior half of the corpus callosum.

Page 17

1814
• Closed head injury due to acceleration or deceleration may cause damage to the brain parenchyma,
which is believed to result from shear strain.
• Closed-head traumas frequently damage the corpus callosum, and the resulting deficits of
interhemispheric communication may vary according to the specific position of the lesion within the
corpus callosum.
• Left intentional movement apraxia, agraphia, left tactile anomia, dyslexia, alien hand syndrome,
decreased left visual recognition, and left auditory hearing loss are all symptoms of corpus callosum
injury, referred to as "corpus callosum syndrome."
Option B: Pituitary apoplexy
• Apoplexy refers to bleeding into an organ or a reduction in organ blood flow.
• Pituitary apoplexy is commonly caused by bleeding inside a benign pituitary tumor.
• These tumors are very common and are often not diagnosed.
• The pituitary is damaged when the tumor suddenly enlarges.
Option C: Acute subdural hematoma
• A dangerous disorder called subdural hematoma occurs when violent acceleration or deceleration
injuries of the head cause tearing of the bridging veins and blood builds up between the dura and
arachnoid mater. One of the signs of a subdural hemorrhage is a headache that does not go away,
being unwell and feeling sick.
• CT scan of the brain shows a crescent shaped mass as shown below

• Severe head injuries, such as those sustained in vehicle accidents, falls, or violent assaults,
frequently result in subdural hemorrhages. In rare instances, minor head injuries can also result in
subdural hemorrhages.

Solution for Question 14:


Option A: Oropharyngeal airway

Page 18

1815
• This patient has sustained a significant head injury which complicates the choice of airway.
• A patient with a GCS score of 8 may still gag on the Guedel airway, although with a patient thus
obtunded, it may be tolerated without sedation.
• An endotracheal tube may be required later in the resuscitation because this patient has a severe
head injury; however, endotracheal intubation requires significant time.
• Guedel’s airway placement should still be considered the first choice adjunct as it is quick, allowing
further assessment of the patient to continue and acting as a bridge to intubation.
• With nasopharyngeal tube placement contraindicated in skull base fractures, the first option in this
patient would be Guedel’s oropharyngeal airway placement.

Incorrect Options:
Option B: Nasopharyngeal tube
• A nasopharyngeal airway device (NPA) is a hollow tube made of plastic or soft rubber that a
healthcare professional can use to help patients breathe more easily when using a bag and mask
ventilation system.
• Any suspicion of a basal skull fracture is an absolute contraindication to nasopharyngeal airway and
nasogastric tube insertion.
• This is because of the risk of penetrating through the damaged cribriform plate and damaging the
brain.
• Basal skull fractures has the following clinical features: Periorbital ecchymosis (panda eyes/raccoon
eyes), Retroauricular ecchymosis (battle sign), CSF leakage from the nose or ears
• Periorbital ecchymosis (panda eyes/raccoon eyes),
• Retroauricular ecchymosis (battle sign),
• CSF leakage from the nose or ears
• Situations when oropharyngeal airways cannot be used, such as oral injuries or trismus (restriction of
mouth opening including spasm of muscles of mastication), nasopharyngeal airways can be used .
Additionally, nasal airways might make bag-valve-mask ventilation easier.
• Periorbital ecchymosis (panda eyes/raccoon eyes),
• Retroauricular ecchymosis (battle sign),
• CSF leakage from the nose or ears
Option C: Laryngeal mask
• Laryngeal mask airways (LMA) are supraglottic airway devices.
• They may be used as a quick method to maintain an open airway during the administration of
anesthesia or as an immediate life-saving measure in a difficult or failed airway patient.
• Indications for Laryngeal Mask Airway: apnea, severe respiratory failure, or impending respiratory
arrest in which endotracheal intubation cannot be accomplished.
• The uses of the laryngeal mask airway include anesthetic management: induction, maintenance, and
emergence.
• The placement of the LMA can be accomplished without muscle relaxants and laryngoscopy.
Option D: Intubation

Page 19

1816
• Intubation occurs when a medical professional inserts an endotracheal tube into a patient's trachea
through the mouth or nose.
• The tube maintains the tracheal opening, allowing air to pass through.
• The tube may be attached to a device that disperses air or oxygen.
• When the airway is clogged or injured, or a person cannot breathe independently, intubation is
required.
• Airway obstruction is one of the frequent conditions that might result in intubation (something caught
in the airway, blocking the flow of air). Cardiac arrest is also an indication for intubation.

Solution for Question 15:


Option D: Aortic rupture
The probable diagnosis in this case is: aortic rupture
Explanation of options:

Solution for Question 16:


Correct Option Option B: Retrograde urethrogram
• A retrograde urethrogram involves imaging of the urethra after the instillation of a contrast via a
Foley’s catheter. The retrograde urethrogram is the best imaging study for diagnosing urethral injury.
Patients with urethral injury typically present with pain, hematuria or meatal bleeding, dysuria, and
urinary retention. Physical signs may include blood at the urethral meatus, a high-riding prostate, or
evidence of additional trauma (e.g., hematoma, external laceration, or pelvic fracture ). During blunt
trauma to the pelvis, the injury most commonly occurs to the posterior urethra .

Incorrect Options:
Option A: Radionuclide cystogram
• A radionuclide cystogram is excellent for the evaluation of vesicoureteral reflux and frequent UTIs.
However, it is of no help in the work-up of urethral injury.
Option C: Voiding cystogram
• Voiding cystography is a type of X-ray performed during urination and is useful in diagnosing tumors
or other defects in the bladder wall, vesicoureteral reflux, as well as renal calculi.
Option D: X-ray of the pelvis
• Since the patient is able to ambulate without noticeable pain and there is no recall of blunt trauma to
the pelvis, plain films of the pelvis are not necessary at this time.

Solution for Question 17:

Page 20

1817
Correct Option A: Abdominal compartment syndrome
ABDOMINAL COMPARTMENT SYNDROME
• ACS is defined as increased intra-abdominal pressure (IAP > 20 mm Hg) resulting in compression of
abdominal structures, producing fatal complications due to pulmonary failure and mesenteric vascular
compromise.
• Normal IAP = 5-7 mm Hg: Intra-abdominal hypertension IAP ≥ 12 mm Hg
• ACS occurs predominantly in: Patients in profound shock Patients requiring large amounts of
resuscitation fluids & blood Those with major visceral or vascular abdominal injuries
• Patients in profound shock
• Patients requiring large amounts of resuscitation fluids & blood
• Those with major visceral or vascular abdominal injuries
• IVC compressed →Decreased Venous return→ Decrease Cardiac Output → Decrease systolic blood
pressure → Decrease renal blood flow & vascular blood flow → Decrease GFR → Decrease urine
output.
• Patients in profound shock
• Patients requiring large amounts of resuscitation fluids & blood
• Those with major visceral or vascular abdominal injuries

Incorrect Options:
Option B: Hemothorax
• Hemothorax is a frequent consequence of traumatic thoracic injuries. It is a blood collection in the
pleural space, a potential space between the visceral and parietal pleura.
• The most important treatment for hemothorax is draining the blood out of the chest cavity using a
chest tube, known as thoracentesis or thoracostomy.
Option C: Portal hypertension
• Portal hypertension is an increased blood pressure within the portal venous system.
• The esophagus, stomach, rectum, or umbilical region may develop varices due to this elevated portal
vein pressure
Option D: Aortic stenosis
• Aortic stenosis is obstruction of blood flow across the aortic valve due to aortic valve fibrosis and
calcification. The presentation includes fatigue; exertional symptoms like shortness of breath, angina,
or syncope; and heart failure.
• The blood flow from the left ventricle to the aorta is restricted by aortic stenosis, which may alter the
pressure in the left atrium.

Page 21

1818
Instruments
1. Which instrument is depicted in the image?

A. Moynihan's Tetra-Towel clip


B. Doyen's Towel Clip
C. Mayo's Towel Clip
D. Rampley's Sponge Holding Forceps
----------------------------------------
2. Which instrument is depicted in the image?

A. Adson Forceps
B. Metzenbaum Scissors
C. Bard Parker Handle
D. Backhaus Towel Clamp
----------------------------------------
3. For which procedure is the No. 11 blade primarily used?
A. Open skin/skin incision
B. Arteriotomy
C. Minor surgical procedure, plastic & pediatric surgery
D. Abdominal incision
----------------------------------------

1819
4. Which instrument is depicted in the image?

A. Kelly Forceps
B. Mosquito Forceps
C. Adson Forceps
D. Needle Holder Forceps
----------------------------------------
5. For which procedure is the given instrument primarily used?

A. Arthroscopy
B. Laparoscopic Cholecystectomy
C. Harvesting split-thickness skin grafts & full thickness skin grafts
D. Tonsillectomy
----------------------------------------
6. Which instrument is depicted in the image?

Page 2

1820
A. Rib Retractor
B. Kerrison Rongeur
C. Bone Cutter
D. Rib Shear
----------------------------------------
7. For which disease is the shown instrument the investigation of choice?

A. Rectal polyp
B. Rectal Ulcer
C. Hemorrhoids
D. CA Anal canal
----------------------------------------
8. For which procedure is the shown instrument primarily used?

Page 3

1821
A. Tonsillectomy
B. Appendectomy
C. Varicose vein surgery
D. Cataract surgery
----------------------------------------
9. For which surgical procedure is the shown instrument primarily used?

A. Hernia repair
B. Craniotomy
C. Tonsillectomy
D. Amputation
----------------------------------------
10. Which instrument is depicted in the image?

Page 4

1822
A. Kelly Forceps
B. Mosquito Hemostatic Forceps
C. Adson Forceps
D. Allis Forceps
----------------------------------------
11. Which instrument is depicted in the image?

A. Allis Forceps
B. Plain Dissecting Forceps
C. Mosquito Hemostatic Forceps
D. Adson Forceps
----------------------------------------
12. Which instrument is depicted in the image?

Page 5

1823
A. Beckman Weitlaner Retractor
B. Scapula Retractor
C. Balfour Self-Retaining Abdominal Wall Retractor
D. Allison Lung Retractor
----------------------------------------
13. For which surgical procedure is the given instrument primarily used?

A. Dissection of internal tissues


B. Dissection of subcutaneous tissue
C. Dissection of fat layer
D. Dissection of delicate tissues
----------------------------------------

Correct Answers
Question Correct Answer

Question 1 3
Question 2 3
Question 3 2
Question 4 4
Question 5 3

Page 6

1824
Question 6 4
Question 7 3
Question 8 3
Question 9 4
Question 10 2
Question 11 2
Question 12 2
Question 13 4

Solution for Question 1:


Correct Option C - Mayo's Towel Clip:
• Mayo's Towel Clip, also known as a Backhaus Towel Clamp, is a surgical instrument used to secure
surgical drapes or towels to the patient's skin during surgical procedures.
• It typically consists of two prongs with a ratcheted mechanism that allows for adjustable tension to
securely hold the drape or towel in place. The prongs have pointed tips to grip the material effectively.
• Mayo's Towel Clip is an essential tool in maintaining a sterile surgical field by preventing the drapes
from slipping or shifting during the operation, thus reducing the risk of contamination.

• In the provided image, Mayo's Towel Clip would be recognizable by its characteristic design with two
prongs and a ratcheted mechanism. Its purpose is distinct from other surgical instruments such as
clamps or forceps, as it is specifically intended for securing drapes and towels rather than tissue
manipulation or hemostasis.
Incorrect Options:
• Option A - Moynihan's Tetra-Towel clip: Moynihan's Tetra-Towel clip is a specialized instrument used
for holding four towels in position during surgery. It is not the instrument depicted in the image, which
specifically shows Mayo's Towel Clip.

Page 7

1825
• Option B - Doyen's Towel Clip: Doyen's Towel Clip is a surgical instrument similar to Mayo's Towel
Clip, used for securing surgical drapes or towels. However, it is not the instrument depicted in the
image, which specifically shows Mayo's Towel Clip.

• Option D - Rampley's Sponge Holding Forceps: Rampley's Sponge Holding Forceps, as the name
suggests, are forceps used for holding sponges during surgery. They are not designed for securing
surgical drapes or towels like Mayo's Towel Clip, and thus, they are not the instrument depicted in the
image.

Page 8

1826
Solution for Question 2:
Correct Option C - Bard Parker Handle:
• The Bard Parker Handle is a surgical instrument used for holding and attaching various types of
surgical blades.
• It typically features a slender, cylindrical design with a screw mechanism or a clip at one end to secure
the blade in place. The handle provides a comfortable grip for surgeons during surgical procedures,
allowing for precise control and maneuverability when making incisions.
• In the provided image, the Bard Parker Handle can be identified by its distinctive shape and the
presence of a mechanism for attaching surgical blades.

Incorrect Options:

Page 9

1827
Option A - Adson Forceps:
• Adson Forceps are a type of surgical forceps commonly used for grasping delicate tissues or sutures
during surgical procedures.
• It has fine, toothed tips for a secure grip.
• However, Adson Forceps differ in appearance and function from the Bard Parker Handle, which is
specifically designed for holding surgical blades.
Option B - Metzenbaum Scissors:
• Metzenbaum Scissors are surgical scissors commonly used for dissecting and cutting delicate
tissues.
• It has long, thin blades with curved or straight tips. While Metzenbaum Scissors are essential surgical
instruments, they are not the instrument depicted in the image, which shows the Bard Parker Handle
used for attaching surgical blades.
Option D - Backhaus Towel Clamp:
• Backhaus Towel Clamp, also known as Mayo's Towel Clamp, is a surgical instrument used for
securing surgical drapes or towels to the patient's skin during surgical procedures.
• It features pointed prongs and a ratcheted mechanism for secure attachment.
• Although both the Backhaus Towel Clamp and Bard Parker Handle are essential surgical instruments,
they serve different purposes

Solution for Question 3:


Correct Option B - Arteriotomy:
• The No. 11 blade is primarily used for arteriotomy procedures.
• Arteriotomy involves making an incision into an artery, typically for procedures such as vascular
surgeries or accessing blood vessels for various medical interventions.
• The No. 11 blade's fine, pointed tip and sharp edge make it suitable for precise incisions required in
arteriotomy procedures.
Incorrect Options:
Option A - Open skin/skin incision: This procedure is typically performed using a
No. 10 blade, which is specifically designed for making open skin incisions. The No. 10 blade has a
larger, more rounded blade compared to the No. 11 blade.
Option C - Minor surgical procedure, plastic & pediatric surgery: The No. 15 blade is commonly used fo
r minor surgical procedures, plastic surgery, and pediatric surgery. It has a smaller, more delicate blad
e compared to the No. 11 blade, making it suitable for intricate procedures on delicate tissues.
Option D - Abdominal incision: Abdominal incisions are typically made using larger blades such as the
No. 22 blade, which is specifically designed for this purpose. The No. 22 blade has a larger size and str
onger construction compared to the No. 11 blade, making it more suitable for cutting through thicker tis
sues encountered in abdominal surgeries.

Page 10

1828
Solution for Question 4:
Correct Option D - Needle Holder Forceps:
• The instrument depicted in the image is a Needle Holder Forceps. Needle Holder Forceps, also
known as needle drivers, are surgical instruments used for holding and manipulating needles during
suturing procedures.
• They typically have a ratcheted mechanism that allows for secure gripping of the needle, along with
serrated jaws or smooth jaws depending on the type of needle being used (e.g., curved or straight).
• Needle Holder Forceps provide surgeons with precise control and grip when suturing tissues,
ensuring accurate placement of sutures and optimal wound closure.

Incorrect Options:
Option A - Kelly Forceps:
• Kelly Forceps are a type of surgical forceps commonly used for grasping and holding tissues or
vessels during surgical procedures.
• They have interlocking teeth along the full length of the jaws and are available in various sizes. Kelly
Forceps differ from Needle Holder Forceps in both appearance and function, as they are not specifically
designed for holding needles during suturing.
Option B - Mosquito Forceps:
• Mosquito Forceps are fine, delicate surgical forceps commonly used for holding small blood vessels
or for precise dissection in surgical procedures. They have small, straight jaws with fine tips and are
typically used in procedures where delicate handling is required.
• However, Mosquito Forceps are not the instrument depicted in the image, which shows a Needle
Holder Forceps used for holding needles during suturing.
Option C - Adson Forceps:
• Adson Forceps, also known as Adson tissue forceps, are a type of surgical forceps with fine, serrated
tips commonly used for grasping delicate tissues during surgical procedures.
• They have small, toothed jaws that provide a firm grip on tissues without causing excessive trauma.

Page 11

1829
• Adson Forceps are not designed for holding needles during suturing, making them different from
Needle Holder Forceps.

Solution for Question 5:


Correct Option C - Harvesting split-thickness skin grafts & full thickness skin grafts:

• Humby's Knife is primarily used for harvesting split-thickness skin grafts and full-thickness skin grafts.
• This surgical instrument features a sharp, thin blade specifically designed for precise cutting of skin
grafts. Split-thickness skin grafts involve harvesting the epidermis and a portion of the dermis, while
full-thickness skin grafts involve harvesting the entire epidermis and dermis.
• Humby's Knife allows surgeons to perform these procedures with accuracy, facilitating the
transplantation of skin grafts for wound coverage and reconstruction.
Incorrect Options:
Option A - Arthroscopy:
• Arthroscopy involves the visualization and treatment of joint structures using a specialized camera
called an arthroscope.
• Humby's Knife is not used in arthroscopic procedures, which typically involve instruments specific to
joint surgery.
Option B - Laparoscopic Cholecystectomy:
• Laparoscopic cholecystectomy is a minimally invasive surgical procedure for removing the gallbladder
using a laparoscope and specialized instruments inserted through small incisions in the abdomen.
• Humby's Knife is not used in laparoscopic cholecystectomy, as it is primarily used for skin grafting
procedures.
Option D - Tonsillectomy:
• Tonsillectomy is a surgical procedure for removing the tonsils.

Page 12

1830
• Instruments such as tonsil dissectors, tonsil clamps, and electrocautery devices are commonly used
in tonsillectomy procedures, but Humby's Knife is not utilized for this purpose.

Solution for Question 6:


Correct Option D - Rib Shear:
• The instrument depicted in the image is a Rib Shear. Rib Shears are surgical instruments designed
specifically for cutting ribs during thoracic surgeries, such as thoracotomies or rib resections.
• They have a robust, scissor-like design with short blades that are angled and curved to facilitate
cutting through bone structures like ribs. Only the parietal blade is sharp, the visceral blade is blunt.
• Rib Shears are essential tools in procedures requiring access to the thoracic cavity or for removing
sections of ribs to address conditions such as trauma, tumors, or deformities.
Incorrect Options:
Option A - Rib Retractor:
• Rib retractors are surgical instruments used to hold back the edges of an incision, providing better
visualization and access to the surgical site.
• They are not designed for cutting ribs like Rib Shears.
Option B - Kerrison Rongeur:
• Kerrison Rongeurs are surgical instruments commonly used in neurosurgery and orthopedic
procedures for removing bone or tissue.
• They have a ratchet mechanism and sharp, angled jaws for precise bone removal.
• While they are used for bone resection, they are not specifically designed for cutting ribs like Rib
Shears.
Option C - Bone Cutter:
• The instrument depicted in the image has a design characteristic of Rib Shears, which are specifically
tailored for cutting ribs during thoracic surgeries.
• Therefore, "Bone Cutter" is not the most precise identification for the instrument shown in the image.

Solution for Question 7:


Correct Option C - Hemorrhoids:
• The instrument shown in the image is Kelly's Rectal Speculum (Protoscope). It is primarily used as the
investigation of choice for diagnosing hemorrhoids.
• Hemorrhoids are swollen veins in the rectum or anus, and Kelly's Rectal Speculum allows for direct
visualization of the rectal canal, enabling clinicians to identify and assess hemorrhoids accurately.
• It is particularly useful for detecting non-palpable hemorrhoids and assessing their severity.

Page 13

1831
Incorrect Options:
Option A - Rectal polyp:
• While Kelly's Rectal Speculum can aid in the visualization of rectal polyps during examination, it is not
specifically considered the investigation of choice for diagnosing rectal polyps.
• Other imaging modalities such as colonoscopy or sigmoidoscopy may be preferred for evaluating and
diagnosing rectal polyps.
Option B - Rectal Ulcer:
• Kelly's Rectal Speculum is not typically used as the investigation of choice for diagnosing rectal
ulcers.
• Rectal ulcers are often diagnosed through endoscopic examination, such as colonoscopy or
sigmoidoscopy, which allows for direct visualization and biopsy of the ulcerated area.
Option D - CA Anal canal:
• While Kelly's Rectal Speculum may aid in the examination of the anal canal during assessment for
conditions such as anal carcinoma, it is not considered the investigation of choice for diagnosing anal
canal carcinoma.
• Imaging studies, biopsies, and other diagnostic modalities such as anoscopy or sigmoidoscopy are
typically used for evaluating anal canal carcinoma.

Solution for Question 8:


Correct Option C - Varicose vein surgery:
• Mayo's Vein Stripper is primarily used in varicose vein surgery. Varicose veins are enlarged, twisted
veins that often appear blue or dark purple and usually occur in the legs.
• Mayo's Vein Stripper is a surgical instrument used to remove these varicose veins through vein
stripping. During vein stripping, the surgeon makes small incisions in the skin near the affected veins,
inserts the Vein Stripper into the vein, and then removes or "strips" the vein from the body.

Page 14

1832
• This procedure helps alleviate symptoms associated with varicose veins and improves the
appearance of the legs.

Incorrect Options:
Option A - Tonsillectomy:
• Mayo's Vein Stripper is not used in tonsillectomy procedures. Tonsillectomy involves the surgical
removal of the tonsils, which are located in the throat, while Mayo's Vein Stripper is specifically
designed for varicose vein surgery.
Option B - Appendectomy:
• Mayo's Vein Stripper is not used in appendectomy procedures. Appendectomy involves the surgical
removal of the appendix, typically due to appendicitis, a condition characterized by inflammation of the
appendix. Mayo's Vein Stripper is not involved in the treatment of appendicitis or appendectomy.
Option D - Cataract surgery:
• Mayo's Vein Stripper is not used in cataract surgery. Cataract surgery involves removing the cloudy
lens of the eye and replacing it with an artificial lens to restore vision. Mayo's Vein Stripper is unrelated
to this procedure, as it is specifically used for varicose vein surgery in the legs.

Solution for Question 9:


Correct Option D - Amputation:
• The Gigli Saw is primarily used in cutting bone during amputation procedures. Amputation involves
the surgical removal of a limb or part of a limb due to injury, disease, or infection.
• The Gigli Saw consists of a flexible wire with cutting teeth that is threaded through the bone to be cut.
The ends of the wire are anchored with hooks on the handle, allowing the surgeon to apply tension and
create a clean cut through the bone.
• This instrument provides a controlled and precise method for bone cutting during amputation
procedures.

Page 15

1833
Incorrect Options:
Option A - Hernia repair:
• The Gigli Saw is not typically used in hernia repair procedures.
• Hernia repair involves repairing a weakness or tear in the abdominal wall, typically using surgical
mesh or sutures to reinforce the area.
Option B - Craniotomy:
• A craniotomy is a surgical procedure involving the removal of a portion of the skull to access the brain.
• The Gigli Saw is not typically used in craniotomy procedures, as specialized instruments such as bone
rongeurs or pneumatic drills are commonly used for cutting the skull bone.
Option C - Tonsillectomy:
• The Gigli Saw is not used in tonsillectomy procedures.
• Tonsillectomy involves the surgical removal of the tonsils located in the throat, which does not require
bone cutting. Instead, tonsillectomy typically involves the use of surgical instruments such as scissors
or electrocautery.

Solution for Question 10:


Correct Option B - Mosquito Hemostatic Forceps:
• The instrument depicted in the image is the Mosquito Hemostatic Forceps.
• These forceps are commonly used in surgery for clamping small blood vessels or tissue to control
bleeding, particularly in delicate procedures or in areas where precision is required.
• Mosquito Hemostatic Forceps have fine, serrated tips that provide a firm grip on the tissue without
causing excessive trauma. They are essential tools for achieving hemostasis during surgical
procedures.
Incorrect Options:
Option A - Kelly Forceps:
• Kelly Forceps are a type of surgical forceps commonly used for grasping and holding tissues or
vessels during surgical procedures.
• They have interlocking teeth along the full length of the jaws. While similar in appearance to Mosquito
Hemostatic Forceps, they are not specifically designed for hemostasis.
Option C - Adson Forceps:
• Adson Forceps, also known as Adson tissue forceps, are a type of surgical forceps with fine, serrated
tips commonly used for grasping delicate tissues during surgical procedures.
• They have small, toothed jaws that provide a firm grip on tissues without causing excessive trauma.
Adson Forceps are not designed for hemostasis like Mosquito Hemostatic Forceps.
Option D - Allis Forceps:
• Allis Forceps are a type of surgical forceps commonly used for grasping and holding tough tissues or
skin during surgical procedures.

Page 16

1834
• They have sharp, interlocking teeth that provide a firm grip on the tissue. Allis Forceps are not
designed for hemostasis and are typically used for tissue manipulation or retraction.

Solution for Question 11:


Correct Option B - Plain Dissecting Forceps:
• The instrument depicted in the image is the Plain Dissecting Forceps.
• These forceps are commonly used in surgery for grasping and holding tissues or delicate structures
during dissection procedures.
• Plain Dissecting Forceps have smooth, non-toothed tips that provide a gentle grip on the tissue
without causing damage.
• They are versatile instruments used in various surgical specialties for tissue manipulation and
dissection.
Incorrect Options:
Option A - Allis Forceps:
• Allis Forceps are a type of surgical forceps commonly used for grasping and holding tough tissues or
skin during surgical procedures.
• They have sharp, interlocking teeth that provide a firm grip on the tissue.
• Allis Forceps are not the instrument depicted in the image, which shows Plain Dissecting Forceps with
smooth, non-toothed tips.
Option C - Mosquito Hemostatic Forceps:
• Mosquito Hemostatic Forceps are commonly used in surgery for clamping small blood vessels or
tissue to control bleeding. They have fine, serrated tips that provide a firm grip on the tissue.
• Mosquito Hemostatic Forceps are not the instrument depicted in the image, which shows Plain
Dissecting Forceps with smooth, non-toothed tips.
Option D - Adson Forceps:
• Adson Forceps, also known as Adson tissue forceps, are a type of surgical forceps with fine, serrated
tips commonly used for grasping delicate tissues during surgical procedures.
• They have small, toothed jaws that provide a firm grip on tissues without causing excessive trauma.
• Adson Forceps are not the instrument depicted in the image, which shows Plain Dissecting Forceps
with smooth, non-toothed tips.

Solution for Question 12:


Correct Option B - Scapula Retractor:
• The instrument depicted in the image is the Scapula Retractor.
• Scapula Retractors are surgical instruments designed to retract the scapula (shoulder blade) during
surgical procedures, allowing better access to the underlying tissues or organs.

Page 17

1835
• They typically have a curved or angled blade that fits under the scapula and a handle for maneuvering
and maintaining retraction.

Incorrect Options:
Option A - Beckman Weitlaner Retractor:

• Beckman Weitlaner Retractors are self-retaining retractors commonly used in orthopedic and general
surgeries to hold back soft tissues and maintain exposure of the surgical site.
• They have sharp, pointed prongs that can be adjusted to control the degree of retraction. However,
they are not designed specifically for retracting the scapula like the Scapula Retractor.
Option C - Balfour Self-Retaining Abdominal Wall Retractor:

Page 18

1836
• The Balfour Self-Retaining Abdominal Wall Retractor is a self-retaining retractor used in abdominal
surgeries to hold back the edges of an incision, providing better exposure of the abdominal cavity.
• It is not designed for retracting the scapula and is not the instrument depicted in the image.
Option D - Allison Lung Retractor:

• Allison Lung Retractors are surgical instruments used to retract lung tissue during thoracic surgeries,
providing access to the thoracic cavity.
• They are not used for retracting the scapula and are not the instrument depicted in the image.

Solution for Question 13:


Correct Option D - Dissection of delicate tissues:

Page 19

1837
• Metzenbaum scissors are primarily used for the dissection of delicate tissues.
• These scissors have long, thin blades that are curved and tapered, making them ideal for precise
cutting and dissection of delicate tissues such as fascia, vessels, and nerves.
• The design of Metzenbaum scissors allows surgeons to perform intricate dissections while minimizing
trauma to surrounding structures.
Incorrect Options:
Option A - Dissection of internal tissues:
• While Metzenbaum scissors can be used for dissecting internal tissues, their primary use is for
delicate tissues rather than all internal tissues.
• Internal tissues can vary significantly in texture and density, and other surgical instruments may be
more appropriate for specific types of internal tissue dissection.
Option B - Dissection of subcutaneous tissue:
• Subcutaneous tissue refers to the layer of tissue beneath the skin.
• While Metzenbaum scissors can be used for dissection in the subcutaneous tissue layer, their primary
use is for delicate tissue dissection rather than specifically subcutaneous tissue dissection.
Option C - Dissection of fat layer:
• Metzenbaum scissors can be used for dissecting the fat layer, particularly in procedures such as
liposuction or plastic surgery.
• However, their primary use is for delicate tissue dissection rather than specifically dissecting the fat
layer.
• Other surgical instruments, such as liposuction cannulas, may be more commonly used for fat layer
dissection in certain procedures.

Page 20

1838
General Surgery.
1. Identify the correct sequence of stages of tuberculous lymphadenitis.
A. Sinus formation → collar stud abscess → matting → lymphadentis→ cold abscess
B. Cold abscess → matting → sinus formation → lymphadentitis → collar stud abscess
C. Matting → collar stud abscess → lymphadenitis → sinus formation → cold abscess
D. Lymphadenitis → matting → cold abscess → collar stud abscess→ sinus formation
----------------------------------------
2. A 57-year-old man is brought to the ER with abdominal pain. He is hemodynamically stable. Further
evaluation reveals appendicitis, and a laparoscopic appendectomy is planned. Which of the following is
not a part of the prevention of wound infection in surgical management in general?
(or)
Which of the following is not a measure for preventing wound infections in surgeries?
A. Adequate hand washing
B. Hair removal before surgery
C. Prophylactic antibiotics a few minutes before the surgery
D. Cleaning of parts
----------------------------------------
3. A 41-year-old woman is scheduled for an elective laparoscopic cholecystectomy. During a
pre-operative evaluation, the surgical team discusses various measures to prevent surgical site
infections. Which of the following is an ideal OT parameter?
(or)
Which of the following is an ideal OT parameter?
A. Laminar airflow
B. Relative humidity of 25%
C. Negative pressure ventilation
D. Room temperature of 12-16°C
----------------------------------------
4. A 47-year-old patient presents with chronic abdominal pain and recurrent jaundice. After a thorough
evaluation, it is determined that he requires surgical intervention for a suspected pancreatic mass. The
patient is scheduled for surgery in the operating theater. Identify the OT zone that correctly matches its
description.
(or)
Which of the following OT zones correctly matches its description?
A. Zone 1 - Protective zone that includes reception/waiting area, changing rooms for staff
B. Zone 2 - Disposal zone that includes dirty utility, disposal corridor
C. Zone 3 - Clean zone that includes pre-operative and recovery rooms
D. Zone 4 - Sterile zone that includes scrub rooms

1839
----------------------------------------
5. A young patient presents with fever and abdominal pain. On examination, rose spots on the trunk
and splenomegaly are evident. The physician suspects a foodborne illness, and blood cultures confirm
the diagnosis. Which of the following is not a complication of this illness?
(or)
What is not a complication of a foodborne illness that produces rose spots on the trunk and
splenomegaly?
A. Paralytic ileus
B. Intestinal hemorrhage
C. Ileal perforation
D. Primary sclerosing cholangitis
----------------------------------------

Correct Answers
Question Correct Answer

Question 1 4
Question 2 3
Question 3 1
Question 4 1
Question 5 4

Solution for Question 1:


Correct Option D- Lymphadenitis → matting → cold abscess → collar stud abscess→ sinus formation:
• Tuberculous lymphadenitis is among the most frequent presentations of extrapulmonary tuberculosis
(TB).
• Cervical lymphadenopathy is the most common presentation of peripheral tuberculous
lymphadenopathy.
• The following image shows the sequence of events in the pathogenesis of TB lymphadenitis.

Page 2

1840
Incorrect Options:
Options A, B, and C (Sinus formation → collar stud abscess → matting →
lymphadentis→ cold abscess, Cold abscess → matting → sinus formation → lymphadentitis →
collar stud abscess & Matting → collar stud abscess → lymphadenitis → sinus formation →
cold abscess): The stages are not appropriately arranged in sequence in these options.

Solution for Question 2:


Correct Option C- Prophylactic antibiotics a few minutes before the surgery:
• Prophylactic antibiotics are administered before surgery to prevent surgical site infections. However,
the timing of antibiotic administration is important.
• Antibiotics should ideally be given 30 minutes to 1 hour before surgical incision to ensure adequate
tissue levels during the procedure.
• In prolonged surgery, a repeat dose is given after 4 hours.
• Administering antibiotics a few minutes before surgery may not allow sufficient time for the drug to
reach therapeutic levels in the tissues, thus compromising its efficacy in preventing infections.
Incorrect Options:
Option A- Adequate hand washing: Hand washing is the most important factor in preventing wound infe
ctions during surgery. It helps reduce the microbial load on the hands of surgical personnel, thereby mi
nimizing the risk of introducing pathogens into the surgical site.
Option B- Hair removal before surgery: Hair removal from the surgical site is important to reduce the ris
k of surgical site infections caused by bacterial colonization of hair follicles. It facilitates better visualizat
ion of the surgical field and reduces the likelihood of contamination during the procedure. It is best don
e by on-table clipping of hair.
Option D- Cleaning of parts: It helps remove dirt, debris, and transient microbes from the skin, reducing
the risk of contamination during the procedure.

Page 3

1841
• In abdominal surgery, nipple to mid-thigh should be cleaned with the swab moving from medial to
lateral while cleaning the abdomen and lateral to medial while cleaning the thighs.

Solution for Question 3:


Correct Option A- Laminar airflow:
• Ultra-clean laminar airflow is an ideal OT parameter for the prevention of wound infections.
• In a laminar airflow system, air flows in parallel layers with minimal turbulence, creating a clean and
sterile environment around the surgical site.
• This reduces the risk of airborne contamination and promotes optimal conditions for aseptic surgical
techniques.
• Additionally, inside the OT, 15-25 air exchanges/hour should be done.
Incorrect Options:
Option B- Relative humidity of 25%: Relative humidity refers to the amount of moisture present in the ai
r relative to the maximum amount the air can hold at a given temperature. A
humidity level of 50-60% is recommended. 25% is quite low and can lead to dryness in the OT.
Option C- Negative pressure ventilation: Inside the OT, positive pressure ventilation of 2
mmHg above atmospheric pressure is ideal.
Option D- Room temperature of 12-16°C: The recommended range is 18-25°C. The range of 12-16°C i
s excessively cold. Such low temperatures can cause discomfort and increase the risk of hypothermia f
or the patient and staff.

Solution for Question 4:


Correct Option A- Zone 1
- Protective zone that includes reception/waiting area, changing rooms for staff:
• This option accurately describes the purpose of OT zone 1. It is also called the protective zone.
• It includes: Reception/waiting area Trolley bags Changing rooms for staff
• Reception/waiting area
• Trolley bags
• Changing rooms for staff
• Reception/waiting area
• Trolley bags
• Changing rooms for staff
Incorrect Options:
Option B - Zone 2 - Disposal zone that includes dirty utility, disposal corridor: Zone 2, also called the cl
ean zone, includes pre-operative rooms, recovery rooms, and plaster/storerooms.

Page 4

1842
Option C - Zone 3 - Clean zone that includes pre-operative and recovery rooms: Zone 3
is called the sterile zone. It includes operating rooms and scrub rooms.
Option D - Zone 4 - Sterile zone that includes scrub rooms: Zone 4
is also the disposal zone. It includes dirty utility and disposal corridor.

Solution for Question 5:


Correct Option D- Primary sclerosing cholangitis:
• Based on the clinical presentation, the most likely diagnosis in this patient is typhoid (enteric fever).
• Patients with typhoid are prone to develop cholecystitis, not primary sclerosing cholangitis.
• Primary sclerosing cholangitis is a chronic and progressive cholestatic liver disorder of unknown
etiology. PSC is characterized by inflammation, fibrosis, and stricturing of intrahepatic and/or
extrahepatic biliary ducts.
• It is frequently associated with ulcerative colitis, an inflammatory bowel disease.
Incorrect Options:
Option A- Paralytic ileus: Paralytic ileus is a
functional obstruction of the intestine due to reduced or absent peristalsis. It can occur as a complicatio
n of typhoid fever due to inflammation and edema of the intestinal wall. Symptoms include abdominal d
istension, absence of bowel sounds, and constipation.
Option B- Intestinal hemorrhage: Intestinal bleeding can occur in typhoid fever due to ulceration of Pey
er's patches in the intestine. Patients present with melena or hematochezia, along with signs of anemia
.
Option C- Ileal perforation: Typhoid fever, especially if untreated, is the most common cause of ileal pe
rforation in tropical countries. It results from necrosis and weakening of the intestinal wall, leading to a
breach and subsequent leakage of intestinal contents into the peritoneal cavity. This can cause peritoni
tis, sepsis, and systemic complications.

Page 5

1843
Paediatric Surgery
1. Which of the following is true regarding a sacrococcygeal teratoma?
(or)
A 2-hour-old boy is evaluated in neonatal intensive care for a large-sized mass present at the
sacrococcygeal region of the newborn. The infant is born to a 27-year-old primigravida at 36 weeks
gestation by cesarean section. The mother did not seek prenatal medical care. On examination, his
temperature is 37.2°C (98.9°F), blood pressure is 90/70 mmHg, heart rate is 118/min, and respiratory
rate is 24/min. Pulse oximetry shows an oxygen saturation of 95%. Examination reveals a large-sized
solid mass at the sacrococcygeal with flaccid lower extremities. A CT scan reveals a sacrococcygeal
mass with a suggestion for the intraspinal extension to level L1. Subsequent MRI confirms this finding
with extension to the upper lumbar spine with mass effect on ureters. The condition of the newborn is
shown below in the image. Which of the following is true regarding the condition of this newborn?

A. Notochord
B. Primitive streaks remnants
C. Neural tube
D. Allantois
----------------------------------------
2. A 30-hour-old boy is evaluated in the emergency department due to vomiting. The infant was born to
a 25-year-old primigravida via normal vaginal delivery. According to the mother, he vomited
green-coloured vomitus when she was attempting to feed him. The vomiting recurred on the
subsequent attempt of breastfeeding. The newborn has not had a bowel movement yet. On
examination, his temperature is 37°C (98.6°F), his blood pressure is 96/70 mmHg, his pulse is 110/min,
and his respiratory rate is 20/min. Pulse oximetry shows an oxygen saturation of 95%. Chest
examination is unremarkable. Abdominal examination reveals a moderately distended abdomen. A
digital rectal examination shows a normal rectal tone with no stool in the rectal vault. An abdominal
x-ray is given below. A CT scan of the abdomen shows dilated loops of the small intestine with no air in
the rectum. Which of the following is the most likely cause of this patient’s presentation?
(or)
A 30-hour-old boy with green-coloured vomitus has a moderately distended abdomen. CT scan shows
dilated loops of the small intestine with no air in the rectum. An abdominal x-ray is given below. Which
of the following is the most likely cause of this patient’s presentation?

1844
A. Meconium ileus
B. Hirschsprung's disease
C. Ileocaecal tuberculosis
D. Pseudomyxoma Peritonei
----------------------------------------
3. A 2-year-old boy presented with recurrent episodes of abdominal pain with distension, bilious
vomiting, and fever for 3 weeks. He weighed 8 kg with a history of failure to thrive since early infancy.
The abdomen was mildly distended with non-tender hepatomegaly. The chest radiograph was normal.
USG suggested dilated large bowel loops that were peristaltic with moderate free fluid. The child was
kept nil per oral and the fluid and electrolyte imbalance was corrected. Due to persistent bilious
aspirates, an upper gastrointestinal (GI) contrast study was performed that suggested malrotation with
volvulus. The patient was scheduled for surgery. Which of the following is a component of the
procedure ?
(or)
A 2-year-old boy has malrotation with sigmoid volvulus. The patient is scheduled for surgery. Which of
the following is true regarding volvulus decompression surgery?
A. Reduction of volvulus by counter-clockwise rotation
B. Ladd's band divided near the medial aspect of the duodenum
C. Cholecystectomy
D. Part of the jejunum is resected and anastomosis performed
----------------------------------------
4. A 22-day-old girl is brought to the outpatient department due to persistent jaundice after birth. 10
days ago, her parents noticed yellow discoloration of her eyes and skin that has been progressively
increasing for the last 3 days. She was born to a 24-year-old primigravida woman via normal vaginal
delivery. The pregnancy was unremarkable. Further query reveals that her stools are pale in color. The
patient is breastfed exclusively and has been feeding well. On examination, her temperature is 37.2°C
(98.9°F), blood pressure is 90/70 mmHg, heart rate is 120/min, and respiratory rate is 26/min. Capillary
refill is <2 seconds. Examination reveals a diffusely jaundiced infant. Abdominal examination reveals a
mildly distended abdomen with hepatomegaly. The metabolic screen is normal, ultrasound
demonstrates an absent gallbladder, and a technetium-99m iminodiacetic acid scan shows radionuclide
that is concentrated in the liver but not excreted into the intestine. Which of the following is true
regarding the presentation of this patient?
(or)

Page 2

1845
Which of the following is true regarding biliary atresia?
A. This condition is usually managed non-operatively.
B. Surgery should be performed within 60 days of life.
C. Cystoenterostomy provides adequate biliary drainage.
D. Most of these patients will not require transplantation if left untreated.
----------------------------------------
5. A 4-week-old boy is brought to the emergency department with the complaint of vomiting for 2
weeks. According to the mother, vomitus consists of milk and is non bilious without blood. He has had
small spit-ups since birth, but for the last few days, he has been vomiting forcefully after every feed and
his condition is deteriorating . His appetite is normal and he always seems hungry. The infant was born
to a 27-year-old primigravida via normal vaginal delivery who did not seek prenatal care. On
examination, his temperature is 37.2°C (98.9°F), blood pressure is 90/70 mmHg, heart rate is 120/min,
and respiratory rate is 26/min. The baby appears dehydrated and abdominal examination reveals
visible gastric peristaltic waves and a palpable “olive-shaped” mass in the right upper quadrant of the
abdomen. Which of the following is the surgery of choice for this patient’s presentation?
(or)
What is the surgery of choice for hypertrophic pyloric stenosis?
A. Duodenostomy
B. Billroth Type-I procedure
C. KASAI operation
D. Ramstedt’s pyloromyotomy
----------------------------------------
6. A 4-hour-old boy is brought to the emergency department with complaints of difficulty in feeding and
cough. He was born full term at home via normal vaginal delivery to a 25-year-old primigravida, who did
not seek prenatal care. When his mother tried to breastfeed him, he coughed and vomited. Subsequent
attempts at feeding failed. On examination, his temperature is 37.2°C (98.9°F), blood pressure is 88/70
mmHg, heart rate is 116/min, and respiratory rate is 30/min. His weight and height are at the 25th and
50th percentile, respectively. Abdominal examination is unremarkable. Chest examination reveals
coarse breath sounds bilaterally along with intercostal retractions. Neurologic examination is
insignificant. What is the best initial investigation for this patient?
(or)
What is the best initial investigation in esophageal atresia?
A. Bronchoscopy with an injection of methylene blue
B. NG tube insertion and CXR to check the position of the tube
C. CT scan of the chest
D. Endoscopy
----------------------------------------
7. A 2-hour-old boy has intestinal loops covered in a whitish-grey membrane without overlying skin.
Which of the following is the most likely diagnosis of this newborn’s presentation?
(or)

Page 3

1846
A 2-hour-old boy is evaluated in the neonatal intensive care unit due to uncovered bowel loops. The
infant was born to a 26-year-old primigravida via normal vaginal delivery who did not seek prenatal
care. The mother does not use tobacco, alcohol, or any other illicit drug and did not take any
medication during pregnancy except for multivitamins. On examination, his temperature is 37.2°C
(98.9°F), blood pressure is 90/70 mm Hg, heart rate is 120/min, and respiratory rate is 26/min. Chest
examination reveals normal breath sounds bilaterally without any added sound. Neurologic examination
is unremarkable. Abdominal examination reveals intestinal loops covered in a whitish-gray membrane
without overlying skin. Which of the following is the most likely diagnosis of this newborn’s
presentation?
A. Gastroschisis
B. Omphalocele
C. Umbilical hernia
D. Epigastric hernia
----------------------------------------
8. A newborn boy is evaluated in the labour room due to respiratory distress. The newborn was born full
term 2 hours ago to a 25-year-old primigravida, who did not seek prenatal care. On examination, his
temperature is 37.2°C (98.9F), heart rate is 140/min, and respiratory rate is 36/min. The pulse oximeter
shows an oxygen saturation of 83%. Physical examination reveals a diffusely cyanosed newborn with
grunting and subcostal retractions. The abdomen appears scaphoid, while the chest appears rounded
in shape. Auscultation reveals absent breath sound on the left side of the chest, along with a gurgling
sound. Right-sided chest auscultation is normal. A cardiac examination reveals clear heart sounds,
best appreciated on the right chest. Chest x-ray reveals displaced cardiac silhouette and air-fluid levels.
Which of the following is the most likely diagnosis of this patient?
A. Umbilical hernia
B. Epigastric hernia
C. Pneumonia
D. Diaphragmatic hernia
----------------------------------------
9. What is the preferred treatment for a low imperforate anus?
(or)
A newborn male infant is admitted to the hospital with an imperforate anus. The newborn was born by
normal delivery at 40 weeks to a 28-year-old second gravida who did not seek prenatal care. His birth
weight is 3350 g with an Apgar score of 7 and 9 at 1 and 5 minutes respectively. On examination, his
temperature is 37.2°C (98.9°F), blood pressure is 89/70 mmHg, heart rate is 120/min, and respiratory
rate is 26/min. Chest examination reveals normal breath sounds bilaterally without any added sound.
Abdominal examination reveals a distended abdomen without rigidity. An imperforate anus with an
obvious anal dimple is noted on digital rectal examination. However, no fistula is present in the
perineum or scrotum. An invertogram at the 16th hour after birth shows rectal gas below the coccyx
level. Which of the following is the preferred treatment for this newborn’s presentation?
A. Observation
B. Anoplasty
C. Posterior sagittal anorectoplasty (PSARP)
D. KASAI procedure

Page 4

1847
----------------------------------------
10. A 7-year-old girl is brought to the outpatient department with a history of constipation for the past 6
months. Constipation was not relieved with laxatives and other conservative management. Her mother
denies altered bowel habits. She is a known case of vestibular fistula, and all three stages of surgery
were completed at the age of one year. The postoperative period was uneventful, with normal bladder
and bowel habits. Routine blood investigations were within normal limits. Radiographs of the pelvis,
including the sacrum, showed a sacral defect as a scimitar sacrum. A contrast-enhanced CT scan of
the abdomen showed a presacral cystic lesion, likely a sacral meningocele. Which of the following is
the most likely diagnosis of this patient’s presentation?
(or)
A 7-year-old girl has a sacral defect called a scimitar sacrum. A contrast-enhanced CT scan of the
abdomen showed a presacral cystic lesion, likely a sacral meningocele. Which of the following is the
most likely diagnosis of this patient’s presentation?
A. Trotter's syndrome
B. Borchardt's syndrome
C. Currarino syndrome
D. Whipple syndrome
----------------------------------------

Correct Answers
Question Correct Answer

Question 1 2
Question 2 1
Question 3 1
Question 4 2
Question 5 4
Question 6 2
Question 7 2
Question 8 4
Question 9 2
Question 10 3

Solution for Question 1:


Option B: Primitive streak remnants
The condition shown in the above image is a
sacrococcygeal teratoma. It originates from the primitive streak remnants.
Sacrococcygeal teratoma:
• Sacrococcygeal teratoma usually presents as a large mass extending from the sacrum and coccyx in
the newborn. They are believed to originate from the pluripotent cells in the Hensen node of the

Page 5

1848
primitive streak.
• Although most of these tumours are non-cancerous (benign), they may grow quite large and, once
diagnosed, always require surgical removal. It is likely that all sacrococcygeal teratomas are present at
birth (congenital) and most are discovered before birth by a routine prenatal ultrasound examination or
an examination indicated for a uterus too large for dates. In rare cases, sacrococcygeal teratomas may
be cancerous (malignant) at birth and many will become malignant if surgical resection is not
performed.
• The size of the mass may be as small as a few centimetres in diameter to as massive as the size of
the infant.
• The tumor is further classified based on the location and degree of intrapelvic extension. Lesions that
grow predominantly into the presacral region are often present later in childhood. The differential
diagnoses include neural tumors, lipoma, and myelomeningoceles.
• Diagnosis may be established by prenatal Ultrasonography. Ultrasound performed in the first trimester
has a high rate of detection of sacrococcygeal teratoma

This image shows sacrococcygeal teratoma


Treatment:
• Complete surgical excision through a chevron-shaped buttock incision. Complete resection of the
tumor as early as possible is essential for life. The rectum and genital structures are often distorted by
the tumor but can be preserved in the course of resection. There is an excellent cure rate if the tumor is
excised completely.
Option A: Notochord
• The notochord is a rod-like cartilaginous skeleton that supports the body. Notochord comes from the
cells called epiblast and hypoblast during the time of gastrulation. These cells develop the central
nervous system. Sacrococcygeal teratoma does not originate from the notochord.
Option C: Neural tube
• The neural tube is the primordium of the brain and spinal cord, and the process of its formation is
called neurulation. Neural tube defects are birth defects of the brain, spine, or spinal cord that usually
happen in the first month of pregnancy. The two most common neural tube defects are spina bifida and
anencephaly. Sacrococcygeal teratoma does not originate from the neural tube.

Page 6

1849
Option D: Allantois
• The allantois is a hollow sac-like structure that is filled with transparent fluid, which is involved in the
concept of developing an amniotic membrane. It is present in all embryonic and extraembryonic
tissues. Sacrococcygeal teratoma does not originate from the allantois.

Solution for Question 2:


Option A: Meconium ileus
• This patient's presentation, including bilious vomiting, delayed passage of meconium, and distended
abdomen, is consistent with intestinal obstruction. The most common cause in this patient with normal
rectal tone and no stool in the rectal vault, along with radiograph findings of no air in the rectum, is
consistent with meconium ileus.
Meconium ileus:
• Meconium ileus is a bowel obstruction that occurs when the meconium in a child's intestine is thicker
and stickier than normal meconium, creating a block in the ileum. Meconium ileus is most commonly
associated with cystic fibrosis.
• Meconium ileus accounts for up to 33% of neonatal small-bowel obstructions. Symptoms include
emesis that may be bilious, abdominal distention, and failure to pass meconium in the first several days
of life. Diagnosis is based on clinical presentation and x-rays. Treatment is enema with dilute contrast
under fluoroscopy and surgery if enema fails.
Meconium peritonitis in meconium ileus:
• Meconium peritonitis refers to sterile chemical peritonitis due to intrauterine bowel perforation and
spillage of fetal meconium into the fetal peritoneal cavity. It is a common cause of peritoneal
calcification.
• Plain X-ray of the abdomen post-natally shows specks of calcification mainly at the flanks. On
ultrasonography, specks of high echogenic areas are seen throughout the abdomen, which has been
described as a “snow-storm sign”.
Diagnosis of meconium ileus:
• Diagnosis of meconium ileus is suspected in a neonate with signs of intestinal obstruction, particularly
if a family history of cystic fibrosis exists. Patients should undergo abdominal x-rays, which will show
dilated intestinal loops; however, fluid levels may be absent. A “soap bubble” or “ground glass”
appearance due to small air bubbles mixed with the meconium is diagnostic of meconium ileus. If
meconium peritonitis is present, calcified meconium flecks may line the peritoneal surfaces and even
the scrotum. A water-soluble contrast enema reveals a microcolon with an obstruction in the terminal
ileum.

Page 7

1850
Ultrasound
• May show highly echogenic linear or clumped foci, which represent calcification - can also give a
snowstorm appearance
• Differentiated from other causes of intra-uterine calcification by its peritoneal distribution
• May show fetal ascites (the most common antenatal sonographic finding) and/or polyhydramnios
Option B: Hirschsprung's disease
• Hirschsprung’s disease is a developmental disorder characterized by the absence of ganglia in the
distal colon, resulting in a functional obstruction. It is characterized by dilated large bowel (unlike small
bowel as in this patient) and increased rectal tone (unlike normal rectal tone in this patient). Therefore,
it is an incorrect option.
Option C: Ileocaecal tuberculosis
• Ileocecal tuberculosis is the most common site of gastrointestinal tuberculosis, which in turn is the
third most common site of extrapulmonary tuberculosis. The most common presenting feature of
abdominal tuberculosis is abdominal pain, ascites and intestinal obstruction. Other clinical features are
fever, altered bowel habits, loss of weight and a feeling of a lump in the abdomen. Night sweats,
nausea, loss of appetite, constipation, diarrhea, blood in stool and perforation of the bowel are some of
the rare symptoms of abdominal tuberculosis. Also, tuberculosis is a chronic disease.
Option D: Pseudomyxoma peritonei
• Pseudomyxoma peritonei is a rare malignant growth characterized by the progressive accumulation of
mucus-secreting (mucinous) tumor cells within the abdomen and pelvis. The disorder develops after a
small growth (polyp) located within the appendix bursts through the wall of the appendix and spreads
mucus-producing tumor cells throughout the surrounding surfaces (e.g., the membrane that lines the
abdominal cavity (peritoneum).
• Clinical features include abdominal or pelvic pain, gradual increase in waist size, bloating, changes in
bowel habits, hernia, and loss of appetite.

Page 8

1851
Solution for Question 3:
Option A: Reduction of volvulus by counterclockwise wise rotation
• Volvulus usually twists in a clockwise direction, and reduction is accomplished by twisting in a
counterclockwise direction.
Volvulus:
• A volvulus is the twisting of a loop of the intestine around its mesenteric attachment, resulting in
closed-loop bowel obstruction. The affected bowel can become ischaemic due to a compromised blood
supply, rapidly leading to bowel necrosis and perforation.
• Most volvuli occur at the sigmoid colon, while the caecum is the second most common site. It is a
common cause of large bowel obstruction in many countries (after malignancy and diverticular
disease). They can also occur in the stomach, small intestine and transverse colon but are much rarer.
• It is a rotation of the bowel on its axis.

• Malrotation can be asymptomatic or may be manifested as chronic, vague abdominal pain, with or
without intermittent bilious emesis, and failure to thrive.
• The acute onset of bilious vomiting in a neonate is considered a sign of malrotation. It demands
immediate radiologic evaluation.
• Clinical features include abdominal distension, pain, tenderness over the abdomen and vomiting
followed by perforation/gangrene.
Upper Gastrointestinal Contrast Study
• Malrotation is diagnosed by an abnormal position of the ligament of Treitz. The normal location is
typically to the left of the vertebral column and posterior to the stomach.

Page 9

1852
Management:
Sigmoid Volvulus:
• Gangrenous sigmoid volvulus is best managed with Hartmann’s procedure. Non-gangrenous sigmoid
volvulus is best managed with resection and anastomosis unless there are risk factors for anastomotic
leakage, in which case the surgeon should consider mesosigmoidopexy with a non-absorbable suture.
• All patients admitted with suspected sigmoid volvulus should be managed initially like any patient with
bowel obstruction. In particular, they should be examined for any signs of ischaemia and given fluid
resuscitation.
• Conservative management: Most patients with sigmoid volvulus are treated conservatively initially
with decompression by sigmoidoscope and insertion of a flatus tube. In sigmoidoscopy decompression,
the patient is placed in the left lateral position and a lubricated sigmoidoscope is gently guided into the
rectum. It is maneuvered to locate the twisted bowel and once the sigmoidoscope is in the correct
position, there will be a rush of air and liquid feces as the obstruction is relieved. A flatus tube is often
left in situ for a period (up to 24 hours) after initial decompression to allow for the continued passage of
contents and aid recovery of the affected area. In 25-50% of patients, this approach is unsuccessful
and a formal decompression with a flexible sigmoidoscope is required.
• Surgical management: The indications for surgery (which is usually a laparotomy for a Hartmann’s
procedure) are:
1. Colonic ischaemia or perforation
2. Repeated failed attempts at decompression
3. Necrotic bowel noted at endoscopy
• The decision on which operation to perform will depend on the patient’s nutritional status, adequacy of
blood supply, haemodynamic stability, and the presence of any perforation or peritonitis. Patients with
recurrent volvulus who are otherwise healthy may choose to have an elective procedure (most common
: sigmoidectomy with primary anastomosis) to prevent further recurrence.

Page 10

1853
Caecal volvulus:
• vascular compromise is more common in cecal volvulus compared to sigmoid volvulus. The plain
x-rays are usually adequate for diagnosis. The options for treatment include endoscopic
decompression, caecopexy or a right hemicolectomy.
Option B: Ladd's band is divided near the medial aspect of the duodenum
• Ladd's band is divided near the lateral aspect of the duodenum. Therefore, it is an incorrect option.
Option C: Cholecystectomy
• Appendectomy can also be done during the procedure, not cholecystectomy.
Option D: Part of the jejunum is resected and anastomosis is performed
• Part of the jejunum should not be resected and anastomosed during the procedure.

Solution for Question 4:


Option B: Surgery should be performed within 60 days of life.
• This patient’s presentation which includes persistent jaundice, yellow discolouration of her eyes and
skin, mildly distended abdomen with hepatomegaly, an absent gallbladder, and a technetium-99m
iminodiacetic acid scan shows radionuclide that is concentrated in the liver but not excreted into the
intestine, is consistent with biliary atresia.
Biliary atresia:
• biliary atresia is a rare disease characterized by fibro-proliferative obliteration of the extrahepatic bile
duct, which progresses toward hepatic fibrosis, cirrhosis, and end-stage liver failure.
• Clinical features: Jaundice (yellowing of the skin) and pale stools that start in the first 4-8 weeks of life
are the main signs of the disease. Other features include direct hyperbilirubinemia, dark urine, and
hepatomegaly.
• Diagnosis:

Page 11

1854
1. An ultrasound scan is the initial investigation which may show an absent or abnormal gallbladder an
d/or common bile duct.
2. Liver biopsy may indicate intrahepatic bile duct proliferation along with portal tract fibrosis.
3. Intraoperative cholangiography is the gold standard test which may show biliary obstruction.
• Management:

This image shows the Kasai procedure.


Option A: This condition is usually managed non-operatively
• Biliary atresia must be treated surgically with the Kasai procedure. Untreated cases may need liver
transplantation; therefore, it is an incorrect option.
Option C: Cystoenterostomy provides adequate biliary drainage.
• Cystoenterostomy is the backup treatment for managing pancreatic pseudocysts after all other less
invasive maneuvers fail. The first-line therapy for symptomatic acute pancreatic pseudocysts should be

Page 12

1855
endoscopic intervention using transenteric or transpapillary drainage.
• It is not a treatment option for biliary atresia.
Option D: Most of these patients will not require transplantation if left untreated.
• Early surgical intervention to treat biliary atresia is critical to prevent irreversible liver damage. Once
the liver fails, a liver transplant is mostly required in untreated patients.

Solution for Question 5:


Option D: Ramstedt’s pyloromyotomy
This patient's presentation, including forceful vomiting, feeling hungry despite vomiting, visible gastric p
eristaltic waves, and a palpable "olive-sized" mass in the right upper quadrant of the abdomen, is consi
stent with hypertrophic pyloric stenosis.
Hypertrophic pyloric stenosis:
• Pyloric stenosis is a thickening of the pylorus (the muscle between the stomach and the intestines)
that causes severe and forceful vomiting in the first few months of life. It is also called infantile
hypertrophic pyloric stenosis.
• Clinical features: Vomiting after feeding at 4-8 weeks of age. The baby may vomit forcefully, ejecting
breast milk or formula up to several feet away (forceful projectile vomiting). Vomitus is usually
nonbloody and nonbilious. Persistent hunger: babies who have pyloric stenosis often want to eat soon
after vomiting Dehydration Changes in bowel movements Poor weight gain
• Vomiting after feeding at 4-8 weeks of age. The baby may vomit forcefully, ejecting breast milk or
formula up to several feet away (forceful projectile vomiting). Vomitus is usually nonbloody and
nonbilious.
• Persistent hunger: babies who have pyloric stenosis often want to eat soon after vomiting
• Dehydration
• Changes in bowel movements
• Poor weight gain
• Vomiting after feeding at 4-8 weeks of age. The baby may vomit forcefully, ejecting breast milk or
formula up to several feet away (forceful projectile vomiting). Vomitus is usually nonbloody and
nonbilious.
• Persistent hunger: babies who have pyloric stenosis often want to eat soon after vomiting
• Dehydration
• Changes in bowel movements
• Poor weight gain

Page 13

1856
Causes:
• The causes of pyloric stenosis are unknown, but genetic and environmental factors might play a role.
Risk factors:
• Risk factors for pyloric stenosis include: Sex: more often in boys, especially firstborn boys , than in
girls. Race: more common in whites of Northern European ancestry, less common in black people and
rare in Asians. Premature birth: more common in babies born prematurely than in full-term babies.
Family history: Pyloric stenosis develops in about 20% of male descendants and 10% of female
descendants of mothers who had the condition. Smoking during pregnancy: doubles the risk of pyloric
stenosis. Early antibiotic use: Babies given certain antibiotics like erythromycin in the first weeks of life
have an increased risk of pyloric stenosis. In addition, babies born to mothers who took certain
antibiotics (like macrolides) in late pregnancy may have an increased risk of pyloric stenosis.
Bottle-feeding: Some studies suggest that bottle-feeding rather than breastfeeding increases the risk of
pyloric stenosis. Most of the people who participated in these studies used formula feeds rather than
breast milk, so it isn't clear whether the increased risk is related to formula feeds or the mechanism of
bottle feeding.
• Sex: more often in boys, especially firstborn boys , than in girls.
• Race: more common in whites of Northern European ancestry, less common in black people and rare
in Asians.
• Premature birth: more common in babies born prematurely than in full-term babies.
• Family history: Pyloric stenosis develops in about 20% of male descendants and 10% of female
descendants of mothers who had the condition.
• Smoking during pregnancy: doubles the risk of pyloric stenosis.
• Early antibiotic use: Babies given certain antibiotics like erythromycin in the first weeks of life have an
increased risk of pyloric stenosis. In addition, babies born to mothers who took certain antibiotics (like
macrolides) in late pregnancy may have an increased risk of pyloric stenosis.
• Bottle-feeding: Some studies suggest that bottle-feeding rather than breastfeeding increases the risk
of pyloric stenosis. Most of the people who participated in these studies used formula feeds rather than
breast milk, so it isn't clear whether the increased risk is related to formula feeds or the mechanism of
bottle feeding.

Page 14

1857
• Electrolyte abnormality seen: hypochloremic hypokalemic metabolic alkalosis with paradoxical
aciduria in case of severe dehydration.
• Sex: more often in boys, especially firstborn boys , than in girls.
• Race: more common in whites of Northern European ancestry, less common in black people and rare
in Asians.
• Premature birth: more common in babies born prematurely than in full-term babies.
• Family history: Pyloric stenosis develops in about 20% of male descendants and 10% of female
descendants of mothers who had the condition.
• Smoking during pregnancy: doubles the risk of pyloric stenosis.
• Early antibiotic use: Babies given certain antibiotics like erythromycin in the first weeks of life have an
increased risk of pyloric stenosis. In addition, babies born to mothers who took certain antibiotics (like
macrolides) in late pregnancy may have an increased risk of pyloric stenosis.
• Bottle-feeding: Some studies suggest that bottle-feeding rather than breastfeeding increases the risk
of pyloric stenosis. Most of the people who participated in these studies used formula feeds rather than
breast milk, so it isn't clear whether the increased risk is related to formula feeds or the mechanism of
bottle feeding.
(Large amounts of vomitus lead to loss of HCl and K+ with the kidney in turn compensating by reabsor
bing more Na+ and HCO3- in exchange of H+ to maintain the intravascular volume and pH causing the
above mentioned abnormality.)
Diagnosis:
• Abdominal ultrasound: It is the gold standard for diagnosing infantile hypertrophic pyloric stenosis.
Management:
• Resuscitation by I.V fluids along with correction of electrolyte levels, particularly potassium
• Fluid of choice: Normal Saline
• The treatment of choice after resuscitation is surgery. Surgery is needed to treat pyloric stenosis. The
procedure (Ramstedt’s pyloromyotomy) is often scheduled on the same day as the diagnosis. If the
baby is dehydrated or has an electrolyte imbalance, he or she will have a fluid replacement before
surgery.
• A pyloromyotomy, or Ramstedt's procedure, is a procedure where the muscle of the pylorus of the
stomach is divided to allow normal stomach emptying.

Page 15

1858
Option A: Duodenostomy:
• Tube duodenostomy involves the insertion of a small feeding tube through the duodenal stump to
encourage the formation of a controlled duodeno-cutaneous fistula. It is done for the management of
difficult duodenal stumps in certain cases of partial gastrectomy or duodenal ulcers.
Option B: Billroth Type-I procedure
• The Billroth I operation is a type of reconstruction after a partial gastrectomy in which the pylorus of
the stomach is anastomosed to the duodenum. It is most commonly performed in the management of
peptic ulcers and gastric carcinoma.
Option C: KASAI operation
• The Kasai procedure involves removing the blocked bile ducts and gallbladder and replacing them
with a segment of the child's own small intestine. This segment of the intestine is sewn to the liver and
functions as a new extrahepatic biliary system. It is performed in biliary atresia.

Solution for Question 6:


Option B: NG tube insertion and CXR to check the position of the tube
This patient's presentation, including difficulty feeding and cough, coarse breath sounds bilaterally alon
g with intercostal retractions on chest examination, coughing fit along with vomiting on feeding attempt,
is suggestive of esophageal atresia with tracheoesophageal fistula.
Esophageal atresia with tracheoesophageal fistula:
• Defective division of the foregut into the esophagus and trachea resulting in a proximal esophageal
blind pouch and a fistula between the distal esophagus and trachea.
• Clinical features: The initial signs of esophageal atresia are usually seen soon after birth : : frothy
white bubbles in the baby's mouth coughing or choking when feeding cyanosis, especially when baby is
feeding difficulty breathing
• frothy white bubbles in the baby's mouth

Page 16

1859
• coughing or choking when feeding
• cyanosis, especially when baby is feeding
• difficulty breathing
• EA and TEF are also often found in babies born with VACTERL (vertebral defects, anal atresia,
cardiac defects, trachea-oesophagal fistula, renal anomalies and limb abnormalities) syndrome. Not all
babies born with VACTERL syndrome have abnormalities in all of these areas.
• Diagnosis: The initial investigation includes the inability to pass the nasogastric (NG) tube into the
stomach. It is later confirmed by an x-ray, which shows an NG tube coiled in the proximal esophagus.
The inability to pass an NG into the stomach is a cardinal feature for the diagnosis of esophageal
atresia If gas is present in the GIT below the diaphragm, an associated tracheoesophageal fistula (TEF)
is confirmed Inability to pass an NG tube with absent radiographic evidence for gastrointestinal gas is
virtually diagnostic of an isolated EA without TEF For H-type: tracheobronchoscopy + endoscopy
should be performed
• The initial investigation includes the inability to pass the nasogastric (NG) tube into the stomach. It is
later confirmed by an x-ray, which shows an NG tube coiled in the proximal esophagus. The inability to
pass an NG into the stomach is a cardinal feature for the diagnosis of esophageal atresia
• If gas is present in the GIT below the diaphragm, an associated tracheoesophageal fistula (TEF) is
confirmed
• Inability to pass an NG tube with absent radiographic evidence for gastrointestinal gas is virtually
diagnostic of an isolated EA without TEF
• For H-type: tracheobronchoscopy + endoscopy should be performed
• Management: Surgical correction
• frothy white bubbles in the baby's mouth
• coughing or choking when feeding
• cyanosis, especially when baby is feeding
• difficulty breathing
• The initial investigation includes the inability to pass the nasogastric (NG) tube into the stomach. It is
later confirmed by an x-ray, which shows an NG tube coiled in the proximal esophagus. The inability to
pass an NG into the stomach is a cardinal feature for the diagnosis of esophageal atresia
• If gas is present in the GIT below the diaphragm, an associated tracheoesophageal fistula (TEF) is
confirmed
• Inability to pass an NG tube with absent radiographic evidence for gastrointestinal gas is virtually
diagnostic of an isolated EA without TEF
• For H-type: tracheobronchoscopy + endoscopy should be performed
Option A: Bronchoscopy with an injection of methylene blue
• It is used to diagnose bronchopleural fistula. A bronchopleural fistula (BPF) is a communication
between the pleural space and the bronchial tree. BPF should be suspected in the postoperative lung
resection patient who presents with sudden onset of dyspnea, chest pain, hemodynamic instability, and
subcutaneous emphysema.
Option C: CT scan of the chest
• CT scan of the chest has no role in the diagnosis of EA with TEF.
Option D: Endoscopy

Page 17

1860
• It is used as a subsequent investigation in type E TEF to confirm the diagnosis, not as the first
investigation. H

Solution for Question 7:


Option B: Omphalocele
Omphalocele:
• Omphalocele, also called exomphalos, is a rare abdominal wall defect. At the 6th week of
development, rapid elongation of the gut and increased liver size reduces intra-abdominal space, which
pushes intestinal loops out of the abdominal cavity. Around the 10th week, the intestine returns to the
abdominal cavity and the process is completed by the 12th week. Persistence of the intestine or the
presence of other abdominal viscera (e.g. stomach, liver) at the umbilicus results in an omphalocele.
• 30% of infants with an omphalocele have other congenital abnormalities including heart defects.
• Related conditions: Omphalocele is commonly associated with Edward's syndrome (trisomy 18),
Patau syndrome (trisomy 13), Beckwith–Wiedemann syndrome, and Shprintzen-Goldberg syndrome.
• Features: The primary symptom of an omphalocele is the protrusion of the abdominal organs through
the abdominal muscle wall. The organs are covered in a thin, nearly transparent sac, unlike
gastroschisis
• The primary symptom of an omphalocele is the protrusion of the abdominal organs through the
abdominal muscle wall.
• The organs are covered in a thin, nearly transparent sac, unlike gastroschisis
• Diagnosis: The diagnosis of omphalocele is usually made by prenatal ultrasound during the second
trimester of pregnancy (around 20 weeks). An amniocentesis is recommended to evaluate
chromosomal abnormalities or genetic syndromes.
• Management: Surgical correction
• The primary symptom of an omphalocele is the protrusion of the abdominal organs through the
abdominal muscle wall.
• The organs are covered in a thin, nearly transparent sac, unlike gastroschisis

Page 18

1861
This image shows intestinal contents covered in a sac, i.e. omphalocele.
Option A: Gastroschisis:
• Gastroschisis is a birth defect in which an infant's intestines are outside the abdominal cavity because
of a defect in the abdominal wall. This condition resembles omphalocele; however, in gastroschisis, the
defect occurs on the right side of the umbilicus, and intestinal contents are not covered in the sac,
unlike omphalocele.

Option C: Umbilical hernia


• A congenital umbilical hernia occurs due to incomplete closure of abdominal muscles. Physical
examination reveals a soft, non-tender bulge at the umbilicus, covered by skin and protrudes with
increased intraabdominal pressure such as coughing, crying, straining etc.

Page 19

1862
This image shows an umbilical hernia covered by overlying skin.
Option D: Epigastric hernia
• An epigastric hernia is a type of hernia that causes fat to push through a weakened area in the walls
of the abdomen. It presents as a bulge in the epigastric area.

Solution for Question 8:


Option D: Diaphragmatic hernia
• Congenital diaphragmatic hernia (CDH) occurs when the diaphragm, the muscle that separates the
chest from the abdomen, fails to close during prenatal development due to incomplete fusion of
pleuroperitoneal folds.
• Clinical features include cyanosis, respiratory distress, absent breath sounds on the side ipsilateral to
the defect.
• The defect is left-sided in most cases.
• It can result in herniation of the bowel into the chest cavity, which can lead to pulmonary hypoplasia.
Herniation into the chest causes a concave (flat) shaped abdomen with a barrel-shaped chest.
• Chest x-ray shows intrathoracic bowel loops with a displaced cardiac silhouette.

Page 20

1863
• Management:
• Endotracheal intubation is the initial step of management.
• Gastric decompression by nasogastric tube.
• Bag-and-mask ventilation is contraindicated because it pumps air into the gastrointestinal tract and
can further deteriorate the condition.
• Definitive management is surgical correction.
Option A: Umbilical hernia
• A congenital umbilical hernia occurs due to incomplete closure of abdominal muscles. Physical
examination reveals a soft, non-tender bulge at the umbilicus, covered by skin and protrudes with
increased intraabdominal pressure such as coughing, crying, straining etc.

Option B: Epigastric hernia

Page 21

1864
• An epigastric hernia is a type of hernia that causes fat to push through a weakened area in the walls
of the abdomen. It presents as a bulge in the epigastric area.
Option C: Pneumonia
• Pneumonia presents with fever, productive cough, and crepitations on lung auscultation. Chest x-ray
typically shows consolidation.

Solution for Question 9:


Option B: Anoplasty
This patient's presentation is consistent with an imperforate anus.
Imperforate anus:
• Imperforate anus is a rare inborn abnormality characterized by the absence or abnormal localisation
of the anus. The rectum or the colon may be connected to the vagina or the bladder by a tunnel
(fistula).
• Symptoms: No passage of first stool within 24 to 48 hours after birth Distended abdomen Stool
passes out of the vagina, base of penis, scrotum or urethra in case of associated fistula
• No passage of first stool within 24 to 48 hours after birth
• Distended abdomen
• Stool passes out of the vagina, base of penis, scrotum or urethra in case of associated fistula
• Cause: Imperforate anus is a birth defect that usually appears to occur sporadically. Less commonly,
the condition may be familial, suggesting autosomal dominant, autosomal recessive, or X-linked
recessive inheritance. In addition, some researchers suggest that the condition may be caused by
changes (mutations) of one or more genes, possibly in association with particular environmental factors
(multifactorial inheritance).
• Diagnosis: In some cases, an ultrasound before birth may suggest an imperforate anus. The
diagnosis of an imperforate anus is almost always made after birth by doing a physical examination and
radiologic studies.
• Treatment: Depending on the invertogram, For high lesions- protective colostomy followed by
Posterior Sagittal Anorectoplasty (PSARP) is done. For Low lesions- Perineal anoplasty is performed.
• No passage of first stool within 24 to 48 hours after birth
• Distended abdomen
• Stool passes out of the vagina, base of penis, scrotum or urethra in case of associated fistula
Option A: Observation
• The imperforate anus should be treated surgically.
Option C: PSARP
• PSARP is the treatment of choice in imperforate anus with a high lesion.
Option D: Kasai procedure
• The Kasai procedure involves removing the blocked bile ducts and gallbladder and replacing them
with a segment of the child's own small intestine. This segment of the intestine is sewn to the liver and
functions as a new extrahepatic biliary system. It is performed in biliary atresia.

Page 22

1865
Solution for Question 10:
Option C: Currarino syndrome
• Currarino syndrome was described as a triad by Guido Currarino, an American radiologist, in 1981.
The triad consists of:
• This syndrome belongs to the group of persistent neurenteric malformations.
• Currarino syndrome is a multiple congenital anomalies syndrome characterized by partial agenesis of
the sacrum in association with pelvic malformation. Anal atresia and the presence of a presacral mass
(teratoma and/or anterior meningocele) make up the so-called Currarino triad. Other malformations,
such as renal (35%) and gynecological (19%), are common. Hirschsprung’s disease has also been
recorded. Other rarer complications include pelvic abscess, malignant degeneration of a presacral
teratoma, E.Coli ascending meningitis and spinal cord tethering. A recurrence of a benign teratoma has
been recorded. Urinary incontinence, dysmenorrhoea, dyspareunia, poor sphincter control, sacral
anesthesia and headaches precipitated by coughing or straining have been reported.
• Inheritance: Autosomal Dominant. De novo mutations have been recorded and account for
approximately 15% of cases.
Other options
Option A: Trotter's syndrome
• Trotter's syndrome is a clinical triad of unilateral deafness, neuralgia affecting branches of the
trigeminal nerve, and defective mobility of the soft palate, which is caused by malignant tumors
involving the lateral pharyngeal recess (Rosenmüller's fossa).
Option B: Borchardt’s syndrome
• Borchardt's syndrome consists of a triad consisting of acute gastric volvulus and consists of
unproductive retching, epigastric pain and distention, and the inability to pass a nasogastric tube. Acute
volvulus can cause gastric infarction leading to GI hemorrhage, acute cardiopulmonary distress, or
shock.
Option D: Whipple syndrome
• Whipple disease is a systemic infectious disease caused by the bacterium Tropheryma whipplei.
• The most common symptoms are diarrhea, abdominal pain, weight loss, and joint pains. The joint
pains may be due to migratory non-deforming arthritis, which may occur many years before any
digestive-tract symptoms develop; they tend to involve the large joints but can occur in any pattern and
tend not to damage the joint surface to the point that the joint becomes deformed. Fever and chills
occur in a small proportion of people.

Page 23

1866
Fluid,Electrolyte,Nutrition
1. A 34-year-old male presents to the emergency department with shortness of breath and multiple
injuries. The patient got injured when his house collapsed, and the roof fell on him. Chest X-ray shows
massive hemothorax on the left side. ICD insertion is done on the left side, and the patient is shifted to
the operation theater. Which of the following defines a massive blood transfusion?
(or)
Which of the following defines a massive blood transfusion?
A. Whole blood volume in 24 hours
B. Half blood volume in 12 hours
C. 40% blood volume in 24 hours
D. 60% blood volume in 24 hours
----------------------------------------
2. A 56-year-old male presents to the emergency department after sustaining a road traffic accident. On
examination, he has a right femur fracture. His blood pressure is 70/40 mmHg, and his pulse is 110 per
minute. A decision is made to transfuse RBCs and proceed with emergency surgery for the femur
fracture. At which of the following temperatures are packed RBCs stored?
(or)
At which of the following temperatures are packed RBCs stored?
A. −40 to −50°C
B. −30°C
C. 2-6°C
D. 20-24°C
----------------------------------------
3. A previously healthy 11-year-old boy presents to the emergency room with a 3-day history of nausea,
anorexia, weakness, abdominal pain, and six episodes of vomiting associated with loose stools. He has
no history of fever or respiratory or urinary symptoms. On examination, his blood pressure is 100/60
mmHg, and his heart rate is 120 beats per minute. The physical examination reveals a thin-built boy
with sunken eyes and slightly dry mucous membranes. Fluid resuscitation has been started for the
patient. Which of the following is the best guide for managing fluid resuscitation?
(or)
Which of the following is the best guide for managing fluid resuscitation?
A. Central venous pressure
B. Urine output
C. Blood pressure
D. Saturation of Oxygen
----------------------------------------
4. A 76-year-old male has been admitted to the ward in an unconscious state secondary to a brainstem
hemorrhage. The tube inserted has been shown below. Which of the following is the correct position of
the neck for ideal insertion of the tube?

1867
(or)
Which of the following neck positions is correct for an ideal insertion of the tube given below?

A. Supine with flexed neck


B. Supine with extended neck
C. Sitting with flexed neck
D. Sitting with extended neck
----------------------------------------
5. What is the least likely characteristic change that follows a major surgery?
(or)
A 75-year-old male is admitted to the hospital with a 3-week history of jaundice, pruritus, pale stools,
and dark urine. He has a history of alcohol abuse and stable angina and is on medication for
hypertension. After evaluation, the diagnosis of carcinoma of the head of pancreas is made. The patient
undergoes Whipple's procedure. What is the least likely characteristic change that follows a major
surgery?
A. Hypermetabolism
B. Positive nitrogen balance
C. Tachypnea
D. Fever
----------------------------------------
6. A 40-year-old woman is admitted to the surgical ward for postoperative care after undergoing surgery
for intestinal obstruction. She has been receiving total parenteral nutrition. After a week, a change is
seen in her electrolyte levels during the laboratory analysis. If the patient has hypophosphatemia, which
of the following could be attributed to this low phosphate level?
(or)
Which of the following could be attributed to hypophosphatemia in total parenteral nutrition?
A. Increased cardiac output
B. Diarrhea
C. Increased energy production
D. Rhabdomyolysis
----------------------------------------

Page 2

1868
7. Citrate-phosphate-dextrose solution with adenine is an anticoagulant and preservative for blood
storage. It can prolong red blood cell storage life. It maintains platelet viability. Usage of CPDA
improves post-transfusion visibility and blood glucose and ATP levels; how long can the blood be
stored with CPDA?
(or)
How long can blood be stored with Citrate-phosphate-dextrose solution with adenine(CPDA) ?
A. 12 days
B. 21 days
C. 35 days
D. 42 days
----------------------------------------

Correct Answers
Question Correct Answer

Question 1 1
Question 2 3
Question 3 2
Question 4 3
Question 5 2
Question 6 4
Question 7 3

Solution for Question 1:


Option A: Whole blood volume in 24 hours
• Massive transfusion is defined, in adults, as the replacement of whole blood volume in 24 hours or
>50% of total blood volume in 3 hours.
Massive blood transfusion:
• Massive transfusion has been defined as transfusion of ≥10 units of whole blood (WB) or packed red
blood cells (pRBCs) in 24 hours, ≥4 units of pRBCs in one hour, recognizing that blood loss is a
continuum, and these are arbitrary cutoffs.
• It identifies patients who require ongoing considerations of complex physiological relationships related
to cardiac output, oxygen-carrying capacity, and hemostasis.
• Common indications for massive transfusion include trauma, cardiac surgery, obstetric bleeding, and
liver disease. Cardiac surgery is the most common; trauma is the most extensively studied.
Approximately 3% of the most severely injured individuals will require massive transfusion.
• Crystalloid volume expanders generally work well to correct a volume deficit for most mildly and
moderately ill or injured patients. For more severely injured patients, the practice has shifted to
blood-component resuscitation.

Page 3

1869
• Attention must be paid to hemoglobin, platelet count, hemostasis, and metabolic status. This includes
complete blood count (CBC) with platelet count after every 5 units, along with coagulation testing.
Standard tests of coagulation include the prothrombin time (PT), activated partial thromboplastin time
(aPTT), and fibrinogen concentration; if available and clinicians are familiar with use, a viscoelastic test
such as thromboelastography (TEG) can be used instead. Ionized calcium and other metabolic
parameters should be monitored and treated systematically.
• Complications:
• Hemostatic – Plasma (2 to 8 units) is transfused if the PT or aPTT exceeds 1.5 times the control value
due to dilutional coagulopathy. Platelets (1 apheresis unit or 6 units of WB-derived platelets) are
transfused if the platelet count decreases to <50,000/microL. Cryoprecipitate for fibrinogen concentrate
can be used for fibrinogen <100 mg/dL (<200 for obstetric bleeding)
• Metabolic – Hypocalcemia can be caused by citrate toxicity; symptomatic hypocalcemia is treated with
intravenous calcium chloride or calcium gluconate (2 to 5 mL of a 10 percent calcium chloride solution
per unit of WB or pRBCs plus plasma; maximum 10 mL). Acid-base abnormalities and hyperkalemia
should be treated if present.
• Hypothermia – Hypothermia can exacerbate bleeding, cause arrhythmias, and lead to other
complications. A high-capacity commercial blood warmer should be used to warm blood components
toward body temperature when >2 units are transfused.
Option B: Half blood volume in 12 hours
• A whole blood transfusion in 24 hours or > 50% of total blood volume transfusion in 3 hours defines
massive blood transfusion.
Option C: 40% blood volume in 24 hours
• A massive blood transfusion is a 100% blood volume transfusion in 24 hours.
Option D: 60% blood volume in 24 hours
• It is the 100%, not the 60%, blood volume that is transferred in 24 hours for massive blood
transfusion.

Solution for Question 2:


Option C: 2-6°C
Blood products and transfusion:
• Although the incidence of severe transfusion reactions and infections is now very low, in recent years,
it has become apparent that there is an immunological price to be paid for the transfusion of
heterologous blood, leading to increased morbidity and decreased survival in certain population groups
(trauma, malignancy).
• Supplies are also limited, and therefore the use of blood and blood products must always be
reasonable and justifiable for clinical need.
• Whole blood is now rarely available in civilian practice as it is an inefficient use of limited resources.
However, whole blood transfusion has significant advantages over packed cells as it is coagulation
factor rich and, if fresh, more metabolically active than stored blood.
• Packed red blood cells are spun-down and concentrated packs of red blood cells. Each unit is
approximately 330 mL and has a hematocrit of 50–70 per cent.

Page 4

1870
• Packed cells are stored in a SAG-M solution (saline–adenine–glucose–mannitol) to increase shelf life
to 5 weeks at 2–6°C. (Older storage regimens included storage in CPD – citrate–phosphate–dextrose
solutions with a shelf life of 2–3 weeks.)
• Indications for blood transfusion: Acute blood loss, to replace circulating volume and maintain oxygen
delivery Perioperative anemia to ensure adequate oxygen delivery during the perioperative phase
Symptomatic chronic anemia without hemorrhage or impending surgery
• Acute blood loss, to replace circulating volume and maintain oxygen delivery
• Perioperative anemia to ensure adequate oxygen delivery during the perioperative phase
• Symptomatic chronic anemia without hemorrhage or impending surgery
• Acute blood loss, to replace circulating volume and maintain oxygen delivery
• Perioperative anemia to ensure adequate oxygen delivery during the perioperative phase
• Symptomatic chronic anemia without hemorrhage or impending surgery
Option A: −40 to −50°C
• Fresh-frozen plasma (FFP) is rich in coagulation factors and is removed from fresh blood and stored
at −40 to −50°C with a two-year shelf life.
• It is the first-line therapy for coagulopathic bleeding. Rhesus D-positive FFP may be given to a rhesus
D-negative woman.
• However, seroconversion can occur with large volumes due to red cell fragments, and rhesus D
immunization should be considered.
Option B: −30°C
• Cryoprecipitate is a supernatant precipitate of FFP and is rich in factor VIII and fibrinogen. It is stored
at −30°C with a two-year shelf life. It is given in low fibrinogen states or factor VIII deficiency.
Option D: 20-24°C
• Platelets are supplied as a pooled platelet concentrate and contain about 250 × 109 /L. Platelets are
stored on a special agitator at 20–24°C and have a shelf life of only 5 days. Platelet transfusions are
given to patients with thrombocytopenia or platelet dysfunction who are bleeding or undergoing
surgery.

Solution for Question 3:


Option B: Urine output
• Based on the symptoms of nausea, vomiting, anorexia and loose stools, the most likely diagnosis is
acute gastroenteritis associated with moderate dehydration.
• Urine output is the best measure of organ perfusion and adequacy of shock therapy.
• Minimum adequate urine output in adults – 1ml/kg/min
• Minimum adequate urine output in children – 0.5 ml/kg/min
• The signs of dehydration are given below.

Mild dehydration

Page 5

1871
Moderate dehydration
Severe Dehydration
Heart Rate
Normal
Slight increase
Significant tachycardia
Capillary refill
Around 2 seconds
>3 seconds
Peripheral pulses
Slightly decreased
Difficult to palpate
Urine output
Decreased
Little or none
Fontanelle
Flat
Soft
Sunken
Eyes
Mucus membranes
Dry
Extremely dry
Tear production
Diminished
No tears
Option A: Central Venous Pressure (CVP)
• Based on the symptoms of nausea, vomiting, anorexia and loose stools, the most likely diagnosis is
acute gastroenteritis associated with moderate dehydration.
• The best guide for fluid resuscitation is urine output, not central venous pressure.
• Central venous pressure is an estimate of right atrial pressure used to assess cardiac preload and
volume status in critically ill patients, diagnose right-sided heart failure, and guide fluid resuscitation.
Option C: Blood Pressure
• The best guide for managing fluid resuscitation is urine output, not blood pressure.
• Blood pressure is the force of circulating blood on the walls of the arteries.
Option D: Saturation of Oxygen

Page 6

1872
• The best guide for managing fluid resuscitation is urine output, not oxygen saturation.

Solution for Question 4:


Option C: Sitting with flexed neck
• The ideal position for Ryles tube insertion is sitting with the neck flexed if the patient is conscious
• In comatose patients, the ideal position is supine with the neck flexed
• To measure the required length of the tube, measure from the tip of the patient's nose to the ear and
then down to the xiphisternum.
Markings of Ryles tube
• At 40 cm: Indicates the level of GE junction
• 50 cm: Indicates the level of the body of the stomach
• 60 cm: Indicates the level of the pylorus
• 65 cm: indicates the level of the duodenum
Indications for NG tube insertion:
Nasogastric tubes are indicated for the following reasons:
• Treatment of ileus or bowel obstruction – Gastrointestinal decompression using nasogastric tubes is
important for treating patients with bowel obstruction or prolonged ileus. Nasogastric decompression
improves patient comfort, minimizes or prevents recurrent vomiting, and serves as a means to monitor
the progress or resolution of these conditions.
• Administration of medications – A nasogastric tube may be needed to administer medications, or oral
contrast for computed tomography, to patients who cannot swallow or are neurologically impaired.
• Enteral nutrition – Nasogastric and gastroenteric tubes deliver enteral nutrition into the stomach
(gastric feeding) or the small intestine (postpyloric).
• Stomach lavage may be needed to remove blood or clots to facilitate endoscopy.
Option A: Supine with flexed neck
• The ideal position for the insertion of the NG tube is sitting, not supine
Option B: Supine with extended neck
• The extended neck position is not favorable during the insertion of the NG tube as it increases the
chances of the tube entering into the respiratory passage.
Option D: Sitting with extended neck
• Although a sitting position is best for the NG tube insertion, a partially flexed neck rather than an
extended neck is recommended.

Solution for Question 5:


Correct Option B: Positive nitrogen balance

Page 7

1873
• There is a negative nitrogen balance following major surgery or trauma.
• The more severe the injury, the greater the response.
• Following elective surgery of intermediate severity, there may be a transient and modest rise in
temperature, heart rate, respiratory rate, energy expenditure, metabolism and peripheral white cell
count.
• The neuroendocrine response to severe injury is characterized by the secretion of cortisol, glucagon
and adrenaline
Physiological response to injury
• The natural response to injury includes: Immobility/rest Anorexia Catabolism
• Immobility/rest
• Anorexia
• Catabolism
• Following surgery or trauma, postoperative hyperglycemia develops due to increased glucose
production combined with decreased glucose uptake in peripheral tissues(due to increased insulin
resistance transiently induced within the stressed patient). There is also increased proteolysis causing
a negative nitrogen balance (catabolism).
• Immobility/rest
• Anorexia
• Catabolism
Option A: Hypermetabolism
• An acceleration of energy-dependent metabolic cycles mainly cause hypermetabolism following injury
or major surgery
• Hypermetabolism is limited in modern practice on account of elements of routine critical care
Option C: Tachypnea
• Tachypnea can occur due to inadequate respiration.
• This is true for abdominal surgeries where it is not possible to inhale to the full depth due to pain.
Option D: Fever
• Because of the release of inflammatory cytokines, the patient might experience an elevation in the
body temperature.

Solution for Question 6:


Correct Option D: Rhabdomyolysis
• Rhabdomyolysis is a condition that can be attributed to hypophosphatemia.
• The most common cause of mortality in long-term TPN use is electrolyte imbalance.
• Features are hypophosphatemia (MC), hypomagnesemia, and hypocalcemia.
• Cardiac features are arrhythmia and sudden cardiac death.

Page 8

1874
• Pulmonary features are effusion /ARDS
• Management includes correction of electrolytes and slow titration of nutrition.
Hypophosphatemia:
• Hypophosphatemia can cause rhabdomyolysis via ATP depletion and the consequent inability of
muscle cells to maintain membrane integrity.
• It results in decreased synthesis of phosphorylated intermediate metabolites such as adenosine
triphosphate (ATP), 2, 3-DPG, and cyclic adenosine monophosphate (cAMP).
• Deficiency of these metabolites can result in erythrocyte membrane instability, weakening of
respiratory muscles, WBC dysfunction, platelet dysfunction, congestive heart failure, arrhythmias,
hemolysis, and rhabdomyolysis.
Option A: Increased cardiac output
• Increased cardiac output is unlikely to occur due to low phosphate.
• low ATP levels will decline heart function because phosphate is not readily available for ATP
production
Option B: Diarrhea
• Diarrhea can occur due to hypocalcemia.
• However, low phosphate does not cause diarrhea.
Option C: Increased energy production
• Energy production is decreased in the case of hypophosphatemia.
• This is due to the low availability of phosphate for ATP production.

Solution for Question 7:


Correct Option C: 35 days
The storage life of blood is 35 days when CPD adenine is used as a preservative.
Anticoagulants and preservatives in stored blood:
• A breakthrough in preserving blood came with the development of the first anticoagulant-preservative
solution, acid citrate dextrose, in the early 1940s.
• This allowed blood to be stored for up to 21 days.
• Before this, red cells could only be stored in a citrate-glucose solution for brief periods.
• During the ensuing years, significant progress was made in the development of other A-P solutions:
• Citrate phosphate dextrose (CPD) – 21-day (three-week) storage
• CPD-adenine – 35-day (five-week) storage
• Current generation additive solutions – 42-day (six-week) storage
• These additive solutions improve the RBC shelf-life by maintaining pH and other essential
biochemical parameters.
• A benefit of the extended storage duration is that fewer collections are lost through out-dating.

Page 9

1875
• However, the average storage duration for RBCs is less than three weeks.
• The volume of the additive solution is smaller than the volume of plasma removed, resulting in a
smaller volume and higher hematocrit than in the original unit of whole blood.
• In modern practice, whole blood is collected into an anticoagulant solution. Subsequently, the additive
(preservative) solution is added to the manufactured RBC units through an integral bag system shortly
after collection and component preparation.
• This or a similar procedure is used for apheresis-derived RBC units, although the details may vary
slightly depending on the apheresis system used.
• When CPD-adenine (CPD-A1) is used as the A-P solution, the final volume of the unit is
approximately 225 to 350 mL, and the hematocrit is approximately 65 to 80 per cent.
• When one of the AS systems is used, virtually all the plasma is removed and replaced with
approximately 100 mL of storage solution.
• This results in a final unit volume of approximately 300 to 400 mL and a hematocrit of approximately
55 to 65 per cent.
• The final hematocrit of apheresis-derived units is approximately 55 to 60 per cent, and the volume is
targeted to be approximately 175 mL but may be higher.
• Units that require prolonged storage (e.g., due to extremely rare blood types) can be frozen.
• There have been (and continue to be) attempts to further improve the quality of RBC units during and
at the end of their 42-day storage period.
Option A: 12 days
• The CPD with adenine has a storage life of 35 days, not 12 days
Option B: 21 days
• A 21 days storage life is of CPD without adenine
Option D: 42 days
• This is the storage life when the SAGM is used as an anticoagulant.

Page 10

1876
Transplantation
1. A 68-year-old man is brought to the hospital from a skilled nursing care facility because of an ulcer
on his back that has increased in size over the past 3 weeks. He has been paraplegic and bedridden
following a spinal cord injury 1 year ago. Since then, he has also had fecal and urinary incontinence.
Vital signs are within normal limits. Skin examination shows a 5 × 3-cm full-thickness skin ulcer over the
sacrum; necrotic tissue and subcutaneous fat are visualized at the base of the ulcer. Laboratory studies
are within the reference range. The patient is scheduled for surgical debridement and a
fasciocutaneous graft placement. Which of the following is not correct?
(or)
Which of the following is not correct regarding the following types of grafts?
A. Heart valve transplant from Sus scrofa domesticus (pig) to Homo Sapiens is a type of Xenograft
B. Heterotopic graft - Graft placed in a site different from its normal anatomical location
C. Most preferred graft for CABG is the left internal mammary artery
D. Graft placed in a site different from its normal anatomical location is known as an orthotopic graft
----------------------------------------
2. A 68-year-old woman comes to the physician because of pain during urination associated with fever,
muscle pain, and weakness. Ten months ago, she underwent a pre-emptive renal allograft transplant.
Her medication includes thymoglobulin, tacrolimus, mycophenolate mofetil (MMF), and
methylprednisolone. Her temperature is 37.8ºC(100ºF), her pulse is 90/min, and her blood pressure is
163/85mm Hg. Examination shows a well-healed incision over the right lower abdomen. On laboratory
testing, a positive urinalysis for decoy cells was found. It preceded a rise in creatinine to 2 mg/dL.
Which of the following is the causal viral agent?
(or)
Which of the following causes post renal transplant infection with renal failure and decoy cells in the
urine?
A. Polyomavirus BK
B. Human herpes virus type 6
C. Cytomegalovirus
D. Human papillomavirus , high risk types
----------------------------------------
3. Which of the following scoring systems cannot be used to assess whether a patient is a suitable
candidate for liver transplantation?
(or)
A 45-year-old man comes to the physician because of vomiting blood, abdominal distension , and
yellowing of eyes. He has a ten-year history of drinking 1-3 bottles of wine daily. Temperature is 37.0°C
(98.6°F), pulse is 90/min, and blood pressure is 129/78 mm Hg. Examination shows jaundice, palmar
erythema, spider nevi on his chest, dilated veins on the anterior abdominal wall, and 2+ oedema of the
lower extremities. The abdomen is soft and diffusely tender; there is shifting dullness to percussion. His
albumin is 1.3 g/dL, bilirubin is 54.8mg/dL, and prothrombin time is 32 seconds (INR = 3.3). Hepatitis
serology is negative. Which scoring system cannot be used to assess whether the patient is a suitable
candidate for liver transplantation?
A. Child-Turcotte-Pugh (CTP)

1877
B. Monoethylglycinexylidide (MEGX)
C. Model for End-Stage Liver Disease (MELD)
D. Estimated Post Transplant Survival (EPTS) score
----------------------------------------
4. A 50-year-old male patient is brought to the physician by his wife for abdominal distention and
swelling of the legs. He drinks 12 to 13 alcoholic beverages daily. He is 178 cm (5 ft 10 in) tall and
weighs 68 kg (150 lb); his BMI is 22 kg/m2. Vital signs are within normal limits. Physical examination
shows pallor, jaundice, hepatomegaly, gynecomastia, and a protuberant abdomen with a fluid wave
and shifting dullness. Laboratory studies show: Serum Albumin 2.7 g/dL Bilirubin Total 1.9 mg/dL Direct
0.2 mg/dL Alkaline phosphatase 45 U/L AST 68 U/L ALT 55 U/L The patient was further evaluated and
was found to have cirrhosis. The patient was advised for liver transplantation. Which of the following is
a contraindication for liver transplantation?
(or)
Which of the following is a contraindication for liver transplantation?
Serum
Albumin 2.7 g/dL
Bilirubin
Total 1.9 mg/dL
Direct 0.2 mg/dL
Alkaline phosphatase 45 U/L
AST 68 U/L
ALT 55 U/L

A. Fulminant hepatic failure


B. Active alcohol abuse
C. Wilson’s disease
D. Primary hyperoxaluria
----------------------------------------
5. A 39-years-old female who is a known case of SLE is referred to the Nephrology Department. She
has a history of crescentic focal segmental glomerulosclerosis found on a biopsy performed due to
significant proteinuria and microscopic hematuria. She was initially managed with a combined
therapeutic regimen of cyclophosphamide, corticosteroids, and azathioprine. However, kidney function
gradually deteriorated, and the patient progressed to end stage renal disease about 4 years after the
initiation of immunosuppressants. Her temperature is 37.5°C (99.5°F), her pulse is 78/min, and her
blood pressure is 162/98 mm Hg. She now needs a renal transplant. A suitable donor for the patient is
found, and Human leukocyte antigen (HLA) typing needs to be done for the donor and the recipient.
Which HLA is the most important for organ transplantation and tissue typing?
(or)
Which HLA is the most important for organ transplantation and tissue typing?
A. HLA-A
B. HLA-B

Page 2

1878
C. HLA-C
D. HLA-DR
----------------------------------------
6. A 54-year-old male came to the OPD complaining of puffiness of the eyes, abdominal distension,
and pedal oedema. He is a non-compliant diabetic. His temperature is 36.0°C (96.8°F), his pulse is
130/min, his respiration is 26/min, and his blood pressure is 145/90 mmHg. Labs show very high blood
levels of urea and creatinine. DTPA renal scan showed poor functioning of bilateral kidneys. Ultrasound
shows bilateral thin paper cortex. The patient was diagnosed with end-stage chronic kidney disease
and was suggested to undergo renal transplantation while being started on dialysis. Which of the
following is not an absolute contraindication for renal transplantation?
(or)
Which of the following is not an absolute contraindication for renal transplantation?
A. Active infection
B. Active malignancy
C. Active drug abuse
D. Reduced life expectancy
----------------------------------------
7. A 25-year-old male diagnosed with systemic hypertension for four years and possible familial
nephropathy underwent a preemptive renal transplant procedure when his creatinine clearance
reached 11 ml/min/1.73 m2. Except for a prior tonsillectomy and a procedure to repair a deviated nasal
septum, the patient had no comorbidities. The patient had never had a blood transfusion; his
panel-reactive antibody (PRA) was 0, and his serologic tests for hepatitis B, hepatitis C, and HIV were
negative. The donor was his father, a 49-year-old man diagnosed with systemic hypertension for two
years. Which of the following is done during renal transplantation in this patient?
(or)
Which of the following is done during renal transplantation?
A. Renal graft is placed in the posterior abdominal wall in retroperitoneal position, leaving the native
kidney in-situ
B. Donor renal vein is anastomosed to internal iliac vein
C. Donor renal artery on Carrel’s patch (small portion of the surrounding aorta) of the donor aorta is
anastomosed to the internal iliac artery
D. Ureter is kept reasonably short to avoid distal ischemia & anastomosed to the bladder by
Lich-Gregoir Technique
----------------------------------------
8. A 48-year-old woman with cystic fibrosis is planned for a single lung transplant. Twenty years ago,
she was diagnosed with cystic fibrosis. She underwent a left pneumonectomy due to a huge
aspergilloma in the upper lobe of the left lung five years ago. She now suffers from progressive
dyspnea and recurrent respiratory infections in the single lung. The doctor decides to proceed with right
lung transplantation. The operating surgeon would prefer which order of anastomosis of the structures?
(or)
What is the preferred order of anastomosis in a lung transplant?
A. Pulmonary artery, pulmonary vein, bronchus

Page 3

1879
B. Pulmonary vein, bronchus, pulmonary artery
C. Pulmonary vein, pulmonary artery, bronchus
D. Pulmonary artery, bronchus, pulmonary vein
----------------------------------------
9. Who performed the first liver transplantation?
(or)
On 28th July, World hepatitis day, the professor of gastroenterology gave a speech and recalled the
history of liver transplantation. Which of the following surgeons had the honor of performing the first
liver transplantation?
A. Christian Barnard
B. Starzl
C. Kelly & Lillehei
D. Reitz & Shumway
----------------------------------------
10. The necessary procedures for organ donation after circulatory death for therapeutic transplantation
starts only after ascertainment of death. This comes under which category of Maastricht classification
for donation after circulatory death (DCD) Donors?
(or)
A 30-year-old male was admitted to the emergency room for a penetrating self-inflicted left chest stab
wound, extensive (80% TBSA) full-thickness burn injury, and carbonization. The patient received
continuous support in the intensive care unit. Due to hemodynamic instability, veno-arterial
extracorporeal membrane oxygenation (V-A ECMO) was initiated. This procedure allowed organ
preservation until appropriate family consultation and palliative care planning was done. The necessary
procedures for donation after circulatory death for therapeutic transplantation started only after the
ascertainment of death with cardio-circulatory criteria and family approval. This comes under which
category of Maastricht classification for donation after circulatory death (DCD) donors?
A. 2
B. 3
C. 4
D. 5
----------------------------------------
11. A 36-year-old male patient presents to the Gambat transplant center for the sake of liver
transplantation. He has a history of chronic hepatitis with recurrent ascites. He has been an alcoholic
for fifteen years. On examination, the stigmata of chronic liver disease are evident. Which of the
following is most relevant about liver transplantation?
(or)
Which of the following is most relevant about liver transplantation?
A. Most common indication of liver transplant is hepatitis B-induced cirrhosis
B. Three recipients benefit from split liver transplantation
C. Choledochojejunostomy is preferred over choledocho-choledochostomy

Page 4

1880
D. Sequence of anastomosis: 1. Supra hepatic IVC 2. Infra hepatic IVC 3. Portal vein 4. Hepatic artery
5. Bile duct
----------------------------------------
12. Which of the following is true regarding transplantation?
A. In islet cell transplantation, islet cells are injected into the splenic vein
B. Sequence of anastomosis in heart transplantation is Right Atrium → Left Atrium → Pulmonary artery
→ Aorta
C. Most common cause of death in intestinal transplant is rupture
D. In pancreas transplantation, enteric drainage is most commonly done.
----------------------------------------
13. A 69-year-old male patient underwent a Whipple’s procedure due to pancreatic head
adenocarcinoma. On admission, blood tests showed HCT: 41%, HGB: 13.7 g/dL, PLT: 221 K/µl, INR:
0.87. On the 1st postoperative day, there was an episode of melena. He was hemodynamically stable,
but a drop in hemoglobin necessitated two units of blood transfusion. Upper GI endoscopy revealed
blood oozing from the afferent limb of the gastrojejunal anastomosis. What type of bleeding is it?
A. Primary hemorrhage
B. Reactionary hemorrhage
C. Secondary hemorrhage
D. Tertiary hemorrhage
----------------------------------------
14. A 32-year-old man was assaulted with a sharp sword, resulting in a total amputation of the lower
extremity at the level of the tibiotalar joint, with a clean-cut wound (image). He presented 4 hours after
the injury in a stable hemodynamic condition with no other associated injuries. The patient was
immediately transferred to the operating theater. During the reconstruction of the amputated limb in this
patient, which of the following is the sequential arrangement?
(or)
During the reconstruction of an amputated limb, which of the following is the sequential arrangement?

A. Bone - Flexor tendon - Extensor tendon - Artery - Vein - Nerve - skin


B. Bone - Artery - Vein - nerve - Flexor tendon - Extensor tendon - skin
C. Bone - Extensor tendon - Flexor tendon - Artery - Nerve - Vein - Skin
D. Bone - Extensor tendon - Flexor tendon - Artery - Vein - Nerve - Skin

Page 5

1881
----------------------------------------
15. Auxiliary partial orthotopic liver transplant (APOLT) is a specialized technique where either the right
or the left lobe of the diseased native liver is removed and replaced with the healthy donor liver. It
provides temporary support until the native liver recovers and immunosuppression can be withdrawn.
Which of the following is an indication for auxiliary partial orthotopic liver transplantation?
(or)
Which of the following is an indication for auxiliary partial orthotopic liver transplantation?
A. Drug-induced hepatic failure
B. As a standby procedure till a suitable donor is found
C. All irreversible causes of fulminant liver failure
D. Metabolic liver disease
----------------------------------------
16. A 24-year-old female with a history of intravenous drug use and recent suicidal ideation was
reported missing by her family in late winter. She was missing for a total of seven days before she was
located by the police. During this time, the outdoor temperature ranged from −10.4 to +1.2°C. She was
found outdoors in a remote, wooded area and was transported in an ambulance to a tertiary care
center. Upon the paramedic’s arrival at the scene, the patient was unresponsive with a Glasgow Coma
Scale (GCS) of 4. Her pupils were 7 mm and non-responsive. She was severely hypothermic. Which of
the following organs is the first (sensitive) to get affected by hypothermia in this patient?
(or)
Which of the following organs is the first (sensitive) to get affected by hypothermia?
A. Liver
B. Kidney
C. Heart
D. Brain
----------------------------------------
17. Which of the following is a clinical implication regarding intestinal transplantation?
(or)
A 45-year-old female was referred to the hospital for short bowel syndrome secondary to acute superior
mesenteric thrombosis. Four months earlier, she underwent a right hemicolectomy plus massive small
bowel resection except for the proximal 15 cm of the jejunum and had been on total parenteral nutrition
(TPN) since then. She was soon considered as a bowel transplant candidate both because of the
availability of her twin sister as donor and her inability to obtain TPN in her hometown. Which of the
following is a clinical implication regarding intestinal transplantation?
A. Most dangerous complication is sepsis.
B. It is widely used due to successful transplantation rates.
C. Most preferred grafts are multi-visceral grafts.
D. Most common indication is carcinoma
----------------------------------------
18. Which of the following is not a typical feature of hyperacute rejection?

Page 6

1882
(or)
Two hours after undergoing allogeneic kidney transplantation for polycystic kidney disease, a
14-year-old girl has lower abdominal pain. Examination shows tenderness to palpation in the area the
donor's kidney was placed. Ultrasound of the donor's kidney shows diffuse tissue oedema. Serum
creatinine begins to increase and dialysis is initiated. Diagnosis of hyperacute rejection is made. Which
of the following is not a typical feature of hyperacute rejection?
A. ABO incompatibility is a contraindication for kidney transplantation
B. Can be found in all male patients older than 20 years
C. Positive cross-match
D. Occurs during pregnancy and in cases with prior history of blood transfusion.
----------------------------------------
19. A 50-year-old woman comes to the physician for the evaluation of excessive hair growth on her chin
over the past 2 weeks. She also reports progressive enlargement of her gums. Three months ago, she
underwent liver transplantation. Following the procedure, the patient was started on transplant rejection
prophylaxis. She has a history of poorly-controlled type 2 diabetes mellitus. Temperature is 37.0°C
(98.6°F), pulse is 80/min, respirations are 22/min, and blood pressure is 150/80 mm Hg. Physical
examination shows dark-pigmented, coarse hair on the chin, upper lip, and chest. The gingiva and the
labial mucosa are swollen. There is a well-healed scar on her right lower abdomen. Which of the
following drugs is the most likely cause of this patient's findings?
(or)
Which immunosuppressant causes dark-pigmented, coarse hair on the chin, upper lip, and chest with
swollen gingiva and labial mucosa?
A. Cyclosporine
B. Tacrolimus
C. Monoclonal antibody (OKT3)
D. Anti-lymphocyte globulin
----------------------------------------
20. What is the rationale for storing transplant organs at low temperature?
(or)
A 68-year-old man comes to the physician for a follow-up examination. He has type 2 diabetes mellitus,
hypertension, and chronic kidney disease. Medications include insulin, metoprolol, and atorvastatin.
Physical examination shows pitting oedema of both ankles. Serum creatinine concentration is 4.5
mg/dL. Donor kidney transplantation is planned by the physician. The kidney to be transplanted has to
be stored at a temperature between 0-5° C and flushed with University of Wisconsin solution. What is
the rationale behind storing kidneys at low temperatures?
A. Oxygen is more soluble in cold solutions and provides a continuous supply for energy metabolism
B. There is no way to suppress microbial growth except by cooling and slowing the growth rate
C. Hypothermia diminishes energy requirements and allows the limited energy reserve to keep the
organ alive
D. It slows down metabolism and the enzymatic processes that would destroy the cell
----------------------------------------

Page 7

1883
21. Which of the following is the most likely change to occur in cardiopulmonary function after lung
transplantation?
(or)
A 57-year-old man with chronic obstructive pulmonary disease comes to the emergency department
because of leg swelling for 2 weeks. He has smoked 2 packs of cigarettes daily for the past 20 years.
His vital signs are within normal limits. Physical examination shows jugular venous distention,
hepatomegaly, and pitting oedema of both lower extremities. A cardiac examination shows a regular
heart rate and normal heart sounds. Auscultation of the lungs shows scattered wheeze without
crackles. The physician plans for a lung transplantation. Which of the following is the most likely change
to occur in cardiopulmonary function after transplantation?
A. In patients with pulmonary hypertension, changes in right ventricular function and pulmonary artery
pressure take weeks to months to resolve
B. In single-lung transplantation, changes in pulmonary function are seen almost immediately following
transplantation
C. Patients with double lung transplants have both better pulmonary function studies as well as better
exercise capabilities
D. After a single-lung transplant, ventilation-perfusion mismatch persists and carbon dioxide retention is
seen
----------------------------------------
22. A 45-year-old patient with Type I diabetes mellitus came to the OPD with complaints of abdominal
distension and puffiness of the face. He was found to have elevated urea and creatinine levels. On
further evaluation, the patient was found to have end-stage renal disease. The patient was advised for
simultaneous kidney-pancreas transplantation (SKP). A brain-dead donor is found. Which of the
following is a contraindication to being a pancreas donor?
(or)
Which of the following is a contraindication to being a pancreas donor?
A. History of diabetes mellitus in the donor
B. No indication for liver transplant in the recipient
C. Technical complication rates are higher for pancreatic transplants than other solid organ transplants
D. Complications are less when compared to other solid organs
----------------------------------------

Correct Answers
Question Correct Answer

Question 1 4
Question 2 1
Question 3 4
Question 4 2
Question 5 4
Question 6 4

Page 8

1884
Question 7 4
Question 8 2
Question 9 2
Question 10 2
Question 11 4
Question 12 4
Question 13 2
Question 14 3
Question 15 4
Question 16 3
Question 17 1
Question 18 2
Question 19 1
Question 20 4
Question 21 2
Question 22 1

Solution for Question 1:


Option D: A graft placed in a
site different from its normal anatomical location is known as an orthotopic graft
• When grafts or organs are transplanted to the same anatomical location, the procedure is said to be
orthotopic, and the graft placed is called an orthotopic graft.

Option A: Heart valve transplant from Sus scrofa domesticus (pig) to Homo Sapiens is a
type of Xenograft

Page 9

1885
• The term xenograft refers to a tissue or organ derived from a species different from the recipient of the
specimen.
Xenograft: Graft from another species
Concordant Xenograft
Discordant Xenograft
• Transplant between closely related species
• Example: For humans, old-world monkeys & apes
• Advantage: hyperacute rejections are not a threat
• Disadvantages: zoonotic transfer of diseases (particularly retroviral transmission)
• Transplant between distant related or divergent species
• Example: For humans, new- world monkeys & other mammals
• For physiological concerns (organ size & availability), pigs are the preferred animal donor
• Disadvantages: High risk of hyperacute rejection

Option B: Heterotopic graft - Graft placed in a site different from its normal anatomical location
• When grafts or organs are transplanted to a different anatomical location, the procedure is said to be
heterotopic, and the graft placed is called a heterotopic graft.
Option C: Most preferred graft for CABG is the left internal mammary artery
• Best natural vascular graft: Reversed long saphenous vein
• Best synthetic vascular graft: Dacron
• Best vascular graft for suprainguinal bypass: Dacron
• Best vascular graft for a femoropopliteal bypass: Great Saphenous Vein
• Best vascular graft for infrainguinal bypass: Great saphenous vein
• Best vascular graft for aorta: Dacron
• Most preferred graft for coronary artery bypass grafting(CABG): LIMA (Left internal mammary artery /
internal thoracic artery > reversed long saphenous vein)
• Most commonly used graft for CABG: Reversed long saphenous vein

Solution for Question 2:


Option A: Polyomavirus BK
• A non-enveloped, double-stranded circular DNA virus of the polyomavirus family that infects the
urothelium.
• BK virus infection is widespread but asymptomatic in the general population.
• Reactivation of the virus in immunosuppressed patients (especially transplant recipients) is associated
with developing ureteric strictures, Polyomavirus-associated nephropathy, and generalized
vasculopathy.

Page 10

1886
• Renal biopsy, quantifying plasma or urinary BKV DNA, and urinary decoy cells in urine cytology are
necessary for diagnosis.
• Timely detection and early reduction of immunosuppression are critical and can reduce rates of graft
loss related to polyomavirus-associated nephropathy from 90% to 10-30%.

Most common opportunistic infections in renal transplant recipients:


Peri-transplant (<1 month )
• Wound infections
• Herpes virus
• Oral candidiasis
• Urinary tract infection
Early ( 1-6 months)
• Pneumocystis carinii
• Cytomegalovirus
• Legionella
• Listeria
• Hepatitis B
• Hepatitis C
Late (>6 months)
• Aspergillus
• Nocardia
• BK virus (polyoma)
• Herpes zoster
• Hepatitis B
• Hepatitis C
Option B: Human herpes virus type 6
• Human herpes virus type 6 is a herpes virus that causes roseola infantum in infants and children.
• Roseola infantum is characterized by high fever, which ends abruptly after three to five days, followed
by the sudden appearance of a maculopapular rash. Roseola infantum is a self-limiting
condition. Febrile seizures are a possible complication of infection.
Option C: Cytomegalovirus
• Cytomegalovirus is generally asymptomatic in an immunocompetent host. Immunocompromised
individuals (e.gAIDS, post-transplantation) are especially at risk of illness following reactivation or initial
infection.
• Any infection in a renal transplant patient is usually caused by Cytomegalovirus(CMV). It is the most
important pathogen in clinical transplantation. It has been recognized as a major factor for graft loss
and increased incidence of acute rejection.
• The infection usually occurs after 30-50 days following transplantation.

Page 11

1887
Clinical features
• CMV infections include fever, malaise, myalgia, arthralgia, and leukopenia.
• Organ involvement leads to pneumonitis, ulceration, and bleeding in the stomach, duodenum, or
colon; hepatitis; esophagitis; retinitis; encephalitis; or pancreatitis.
Diagnsis:
• Invasive CMV infection with histologic evidence or a positive CMV culture from deep tissue specimens
is confirmatory
• A rise in CMV viral load and IgM antibodies to CMV is also diagnostic for this infection
• Chest X-ray: Bilateral diffuse interstitial pneumonia
Treatment:
• IV ganciclovir is the mainstay of treatment and is safe and effective for prevention & treatment.
• The presence of decoy cells in the urine of the patient confirms the diagnosis of BK virus rather than
CMV.
Option D: Human papillomavirus, high-risk types
• High-risk human papillomavirus is associated with an increased risk of cervical, anogenital, oral, and
oropharyngeal squamous cell carcinomas.

Solution for Question 3:


Option D: Estimated Post Transplant Survival (EPTS) score
EPTS score is a numerical score given to all adult candidates on the kidney waiting list to estimate post
transplant survival.
Option A: Child-Turcotte-Pugh (CTP)
• Child-Turcotte-Pugh (CTP) score is used to assess the severity of cirrhosis.
• It is based on five variables:
• Bilirubin level
• Albumin level
• INR
• Encephalopathy
• Ascites
Option B: Monoethylglycinexylidide (MEGX)
• Monoethylglycinexylidide (MEGX) score is a rapid and simple test that can assess the function of the
liver and is widely used for liver transplantation in critical care medicine.
• It measures MEGX formation after the administration of lidocaine.
• It is approximately 80% sensitive and specific in diagnosing cirrhosis.
Option C: Model for End-Stage Liver Disease (MELD)

Page 12

1888
• Model for End-Stage Liver Disease (MELD) score is based on bilirubin, creatinine levels, and the
international normalized ratio (INR).
• It predicts survival in cirrhosis patients, especially those waiting for a liver transplant.
• A minimum MELD score of 18 is necessary to have a survival benefit post-transplant.
• Pediatric donor livers are distributed to pediatric patients preferentially. The Pediatric End-Stage Liver
Disease (PELD) score is used in pediatric patients.

Solution for Question 4:


Option B: Active alcohol abuse
• Active alcohol abuse is a contraindication for liver transplantation.
INDICATIONS FOR LIVER TRANSPLANTATION (LT):
• The indications for liver transplantation fall into four groups: Cirrhosis Acute fulminant liver failure
Metabolic liver disease Primary hepatic malignancy
• Cirrhosis
• Acute fulminant liver failure
• Metabolic liver disease
• Primary hepatic malignancy
• Most common indications for LT: HCV induced cirrhosis > Alcoholic liver disease
• Most common indication for pediatric LT: Biliary atresia
• Cirrhosis
• Acute fulminant liver failure
• Metabolic liver disease
• Primary hepatic malignancy
ADULTS
%
Noncholestatic cirrhosis
• Viral hepatitis B and C
• Alcoholic
• Cryptogenic
65
Cholestatic cirrhosis
• Primary biliary cirrhosis
• Primary sclerosing cholangitis
14
Autoimmune

Page 13

1889
5
Malignancy
• Malignant neoplasm
• (Hepatocellular carcinoma, Neuroendocrine tumors metastatic to liver)
2
Miscellaneous (Fulminant hepatic failure)
CHILDREN
Biliary atresia
58
Inborn errors of metabolism
α1Antitrypsin deficiency, cystic fibrosis, hemochromatosis, primary hyperoxaluria, tyrosinemia, Wilson’
s disease
11
Cholestatic
• Primary sclerosing cholangitis
• Alagille syndrome
9
4
Viral hepatitis
Miscellaneous
16
• Other indications include cryptogenic cirrhosis, Budd-Chiari syndrome, polycystic liver disease, and
amyloidosis.
• Contraindications for LT

Absolute
Relative
• Advanced cardiopulmonary disease
• Active untreated sepsis
• Metastatic malignancy to liver
• Active alcohol abuse
• Severe intractable depression
• Extensive venous thrombosis
• Age (>70 Years)- varies among transplant centers
• Obesity (BMI > 40 kg/m2 )
• Cholangiocarcinoma

Page 14

1890
• Chronic or refractory active infection
• Ongoing tobacco use or illegal drug use
Option A: Fulminant hepatic failure
• It is characterized by rapidly progressive liver injury, hepatic encephalopathy, and impaired synthetic
function, which results in coagulopathy.
• The most common causes are infections (e.g., viral hepatitis)and drug toxicity
(e.g., acetaminophen ingestion).
• The diagnosis is confirmed by identifying an elevation of hepatic enzymes and an altered coagulation
panel in patients with encephalopathy.
• The prognosis is poor, and most patients require urgent liver transplantation as definitive treatment.
Option C: Wilson's Disease
• Wilson disease (hepatolenticular degeneration) is an autosomal recessive metabolic disorder in which
impaired copper excretion causes copper to accumulate in the body.
• In its initial stages, Wilson's disease leads to copper deposits in the liver.
• As the disease progresses, copper accumulates in other organs, most importantly in the brain and
cornea.
• The disease often goes undiagnosed until the typical combination of hepatitis (or even cirrhosis),
dementia, and parkinsonism.
• Kayser-Fleischer rings are brownish copper deposits visible around the iris, further indicating
Wilson's disease.
Option D: Primary Hyperoxaluria
• A group of genetic disorders that impair the metabolism of glyoxylate and oxalate, resulting in an
accumulation of oxalate in the kidneys and other tissues.
• Primary hyperoxaluria typically manifests with recurrent urolithiasis and nephrocalcinosis.
• If the disease progresses to renal failure, systemic oxalosis occurs, and oxalate deposits may
develop in the bones, myocardium, soft tissues, retinas, and peripheral nervous system.

Solution for Question 5:


Option D: HLA-DR
• The most important HLA for organ transplantation and tissue typing is HLA-DR.
• HLA matching has a relatively small but definite beneficial effect on renal allograft survival (HLA-DR >
HLA-B > HLA- A)
• HLA-DR, A, and B are the most important antigens to consider when matching donor and recipient to
reduce the risk of graft rejection.
HLA MATCHING
• HLA molecules are encoded by the major histocompatibility complex (MHC: a cluster of genes
situated on the short arm of chromosome 6).
• HLA class I antigens encoded by MHC-I comprise HLA- A, B, C.

Page 15

1891
• HLA class II antigens encoded by MHC-II comprise HLA - DR, DP, and DQ.
• HLA-DR antigens are the most common cause of graft rejection.
• Anti-HLA antibodies may cause hyperacute rejection.
• In the case of liver transplants, HLA matching does not confer an advantage.
Option A: HLA-A
• HLA-A is encoded by MHC-1, an important antigen for matching during transplantation. It is, however,
not the most important for transplant and tissue typing.
Option B: HLA-B
• HLA-B is an important antigen for matching during transplantation.
• It belongs to MHC-1
Option C: HLA-C
• HLA-C is encoded by MHC-1 and is preferred but not always required for matching

Solution for Question 6:


Option D: Reduced life expectancy
• Reduced life expectancy is a relative contraindication, not an absolute contraindication for renal
transplantation.
• With an ever-increasing demand for kidneys and a limited supply pool, it is essential to understand the
balance between utility and equity in transplant access.
• A criterion is followed based on the current condition of any patient who needs a transplant. Patients
are classified based on absolute or relative contraindications. Patients with any absolute
contraindication are managed through dialysis and medical management. A transplant is reserved for a
patient with no absolute contraindication.
Contraindication for renal transplant is divided as
ABSOLUTE:
• These include any condition that will lead to severe complications in post-transplant patients resulting
in imminent rejection or demise of the patient. Any patient with such a condition is not placed on the
waiting list and is managed through dialysis and medical management.
• Absolute contraindications for kidney transplantation are the inability to tolerate surgery due to severe
cardiac or pulmonary disease, active malignancy, active infection, active drug abuse, or uncontrolled
psychiatric disease.
RELATIVE:
• These include conditions that put the patient at some risk or those in whom a kidney transplant might
not be that useful if, for example, the patient is so old that he won't even survive five years
post-transplant. But if a donor's kidney is available and no other patient is available until the donor's
kidney remains viable, they are preferred, and the kidney is donated to them.
• This patient is not young and has presented with the signs and symptoms of renal failure. This
patient's lab suggests end stage renal disease, and he requires a kidney transplant immediately. He
shows no signs of absolute contraindication and can be placed on the waiting list.

Page 16

1892
Option A: Active infection
• Any patient with active infection depicted by signs and symptoms such as fever, raised total leukocyte
count( TLC), and pus cannot be taken up for transplant unless the infection is completely treated.
• It is an absolute contraindication to renal transplant.
Option B: Active malignancy
• Active malignancy in any body part is an absolute contraindication to transplant.
• The patient's tumor cells can be invasive and spread to a newly transplanted kidney, making it
cancerous.
Option C: Active drug abuse
• People with active drug abuse are at risk of developing serious life-threatening infections, and added
immunosuppressants after transplant will weaken the immunity, further predisposing them to
life-threatening infections.
• It is, therefore, an absolute contraindication

Solution for Question 7:


Option D: The ureter is kept reasonably short to avoid distal ischemia &
anastomosed to the bladder by Lich-Gregoir Technique
• The ureter is kept reasonably short to avoid distal ischemia & anastomosed to the bladder by:
• Lich-Gregoir Technique (preferred): Direct implantation of the ureter into the dome of the bladder with
mucosal to mucosal anastomosis followed by closure over the ureter to create a short tunnel.
• Lead-Better Politano technique.
• After the transplant, we have 2 functioning kidneys- one that is transplanted and the other normal one
+ 1 diseased kidney in-situ (total-3).
Option A: Renal graft is placed in the posterior abdominal wall in retroperitoneal position, leaving the n
ative kidney in-situ
• The renal graft is placed in the iliac fossa in the retroperitoneal position, leaving the native kidney
in-situ.
• Therefore this option is incorrect.
Option B: Donor renal vein is anastomosed to internal iliac vein
• Donor renal vein is anastomosed to the external iliac vein.
• Therefore, this option is incorrect.
Option C: Donor renal artery on Carrel’s patch (A small portion of the surrounding aorta) of donor aorta
is anastomosed to the internal iliac artery
• The donor renal artery on Carrel’s patch (A small portion of the surrounding aorta) of the donor aorta
is anastomosed to the external iliac artery.
• If the donor renal artery lacks an aortic patch (in cases of living donor renal transplantation), the renal
graft artery is anastomosed to the internal iliac artery.

Page 17

1893
Solution for Question 8:
Option B: Pulmonary vein, bronchus, pulmonary artery
• The order of anastomosis in lung transplant is a pulmonary vein, bronchus, and pulmonary artery.
LUNG TRANSPLANTATION
• The most common indications for lung transplantation are Interstitial lung disease (ILD), Advanced
chronic obstructive pulmonary disease (COPD), Cystic fibrosis (CF), Emphysema due to alpha-1
antitrypsin deficiency Pulmonary arterial hypertension (PAH) Bronchiolitis obliterans
• Interstitial lung disease (ILD),
• Advanced chronic obstructive pulmonary disease (COPD),
• Cystic fibrosis (CF),
• Emphysema due to alpha-1 antitrypsin deficiency
• Pulmonary arterial hypertension (PAH)
• Bronchiolitis obliterans
• Interstitial lung disease (ILD),
• Advanced chronic obstructive pulmonary disease (COPD),
• Cystic fibrosis (CF),
• Emphysema due to alpha-1 antitrypsin deficiency
• Pulmonary arterial hypertension (PAH)
• Bronchiolitis obliterans
PROCEDURE: It is of two types:
• Single Lung Transplantation: It is performed through posterolateral thoracotomy. It is done for patients
with a disease affecting one lung way more than the other, or the person has one lung only

Page 18

1894
• It is performed through posterolateral thoracotomy.
• It is done for patients with a disease affecting one lung way more than the other, or the person has
one lung only
• Double Lung Transplantation: It is performed through bilateral thoracotomy or median sternotomy
• After the initial skin incision and separation of all thoracic layers, the lung tissue is exposed. The lung
is removed and replaced by a donor's lung. The structures are anastomosed in the following order.
• Pulmonary vein → Bronchial Anastomosis → Pulmonary Artery
• This allows for blood to flow without pooling in the tissue. As soon as the artery is connected, blood
starts flowing through the system, and if the veins have already been anastomosed, blood will flow
without causing hemorrhagic ischemia.
• Also, anastomosing the bronchus gives enough time for the venous system to play when arteries are
joined.
• It is performed through posterolateral thoracotomy.
• It is done for patients with a disease affecting one lung way more than the other, or the person has
one lung only
COMPLICATIONS OF LUNG TRANSPLANTATION:
• Bleeding.
• Infection.
• Blockage of the blood vessels to the new lung(s)
• Blockage of the airways.
• Severe pulmonary oedema (fluid in the lung)
• Blood clots
• Dehiscence of airway anastomosis
• Late airway stenosis at bronchial anastomosis due to ischemia: treated by dilatation
• Chronic rejection
Option A: Pulmonary artery, pulmonary vein, bronchus
• Anastomosing the artery before the veins will lead to increased blood flow through the lung tissue.
• It will lead to blood pooling and oedema.
Option C: Pulmonary vein, pulmonary artery, bronchus
• Bronchus needs to be joined before anastomosing arteries to give time to veins to start acting.
• If not, it will result in rapid blood flow through veins before the anastomosis is functional.
Option D: Pulmonary artery, bronchus, pulmonary vein
• Anastomosing the artery before the veins will lead to increased blood flow through the lung tissue
• It will lead to blood pooling and oedema.

Solution for Question 9:

Page 19

1895
Option B: Starzl
• Starzl had the honor of performing the first liver transplantation
Option A: Christian Barnard
• Christian Barnard had the honor of performing the first heart transplantation
Option C: Kelly & Lillehei
• Kelly & Lillehei had the honor of performing the first pancreas transplantation.
Option D: Reitz & Shumway
• Reitz & Shumway had the honor of performing the first heart & lung transplantation.

Solution for Question 10:


Option B: 3
• According to Maastricht classification, category 3 donation after circulatory death is a patient awaiting
cardiac arrest after withdrawal of life support.
• It is the most common condition for donation after circulatory death.
Option A: 2
• According to Maastricht classification, category 2 donation after circulatory death is a patient with
resuscitation attempted without success.
Option C: 4
• According to Maastricht classification, category 4 donation after circulatory death is a patient with
cardiac arrest while brain dead.
Option D: 5
• According to Maastricht classification, category 5 donation after circulatory death is a patient with
cardiac arrest & unsuccessful resuscitation in the hospital.

Solution for Question 11:


Option D: Sequence of anastomosis: 1. Supra hepatic IVC 2. Infra hepatic IVC 3. Portal vein 4. Hepatic
artery 5. Bile duct
• The sequence of anastomosis in liver transplant is supra hepatic IVC, infra hepatic IVC, hepatic
artery, portal vein and bile duct anastomosis.
Option A: Most common indication for liver transplant is hepatitis B-induced liver cirrhosis
The indications for liver transplantation fall into four groups:
• Cirrhosis
• Acute fulminant liver failure
• Metabolic liver disease
• Primary hepatic malignancy

Page 20

1896
• The most common indication for liver transplant : HCV-induced cirrhosis > alcoholic liver disease
• The most common indication for pediatric liver transplant is biliary atresia.
Adults
%
Non cholestatic cirrhosis
Viral hepatitis B and C
Alcoholic
Cryptogenic
65
Cholestatic
Primary biliary cirrhosis
Primary sclerosing cholangitis
14
Autoimmune
5
Malignant neoplasm
(Hepatocellular carcinoma, Neuroendocrine tumors metastatic to liver)
2
Miscellaneous (Fulminant hepatic failure)
Children
%
Biliary atresia
58
Inborn errors of metabolism
(α1Antitrypsin deficiency, cystic fibrosis, hemochromatosis, primary hyperoxaluria, tyrosinemia, Wilson’
s disease)
11
• Primary sclerosing cholangitis
• Alagille syndrome
9
4
Viral hepatitis
Miscellaneous
16
• Other indications include Cryptogenic cirrhosis, Budd-Chiari syndrome, Polycystic liver disease, and
Amyloidosis.

Page 21

1897
Option B: Three recipients are benefitted from split liver transplantation
Split Liver transplantation:
• The identification of separate units within the liver with unique blood supply, venous drainage and
biliary drainage makes using a single deceased donor liver applicable for two recipients.
• From brain dead cadaver, transplant the right lobe for the adult and the left lobe for the child. i.e. two
recipients benefited.
• The right lobe most commonly includes the donor vena cava and the right hepatic artery.
• The pediatric graft is based on the celiac trunk providing the left hepatic artery, left portal vein and left
hepatic vein.
Option C: Choledochojejunostomy is preferred over choledocho-choledochostomy
• Donor’s bile duct is anastomosed with recipient’s bile duct by choledochocholedochostomy (preferred)
• If the recipient’s bile duct is diseased or cannot be used, the donor bile duct is attached to the jejunum
(choledochojejunostomy).

Solution for Question 12:


Option D: In pancreas transplantation, enteric drainage most commonly done
• In pancreas transplantation, enteric drainage is most commonly done.
• For bladder drainage, the graft should be oriented with the duodenum inferiorly
• For enteric drainage, the duodenum can be superior or inferior.
• Enteric drainage is preferred over bladder drainage.
• Advantages of bladder drainage: Allows monitoring of urinary amylase level as an early sign of graft
rejection.
• The donor vessels of the pancreas graft are anastomosed to the recipient iliac vessels.
Other options
Option A: In islet cell transplantation, islet cells are injected into the splenic vein
• Islet cells are obtained by mechanically disrupting the pancreas after injecting collagenase into the
pancreatic duct.
• They are then purified and delivered to the recipient's liver by injecting into the portal vein.
Option B: The sequence of anastomosis in heart transplantation is Right Atrium → Left Atrium →
Pulmonary artery → Aorta The sequence of anastomosis in heart transplantation.
→ Left Atrium → Right atrium → Pulmonary artery → Aorta.

Page 22

1898
Option C: Most common cause of death in intestinal transplant is rupture
• Most common cause of mortality after small bowel transplantation is sepsis and multiorgan failure.

Solution for Question 13:


Option B: Reactionary hemorrhage
• Since bleeding has occurred within 24 hours, it is reactionary hemorrhage.
• It is caused by dislodgement of the clot due to excess resuscitation, normalization of blood pressure
and vasodilation.
• It can also result from a technical failure, such as slippage of a ligature.
Option A: Primary hemorrhage
• Primary hemorrhage occurs immediately as a result of an injury or surgery.
Option C: Secondary hemorrhage
• Secondary hemorrhage is caused by sloughing of the wall of a vessel.
• It occurs 7-14 days after injury.
• Factors such as infection, pressure necrosis or malignancy may precipitate it.
Option D: Tertiary hemorrhage
• Tertiary hemorrhage is usually due to infection.

Solution for Question 14:


Option C: Bone - Extensor tendon - Flexor tendon - Artery - Nerve - Vein – Skin

Page 23

1899
• While reconstructing the amputated limb, the correct sequence of reconstruction is bone, extensor
tendon, flexor tendon, artery, nerve, vein, and skin.
• Therefore, this option is correct.
Option A, B, D: While reconstructing the amputated limb, the correct sequence of reconstruction is bon
e, extensor tendon, flexor tendon, artery, vein, nerve and skin.Therefore, these options are incorrect.

Solution for Question 15:


Option D: Metabolic liver disease
• Auxiliary partial orthotopic liver transplantation- Left lobe of the recipient is excised & donor’s liver
occupies vacated space
• Indications of auxiliary partial orthotopic liver transplantation: Acute fulminant reversible hepatic failure
Non-cirrhotic metabolic liver disease Small for size graft ABO incompatibility
• Acute fulminant reversible hepatic failure
• Non-cirrhotic metabolic liver disease
• Small for size graft
• ABO incompatibility
• Acute fulminant reversible hepatic failure
• Non-cirrhotic metabolic liver disease
• Small for size graft
• ABO incompatibility
Option A: Drug-induced hepatic failure
• Drug-induced hepatic failure is not an indication for auxiliary partial orthotopic liver transplantation.
Option B: As a standby procedure till a suitable donor is found
• Auxiliary partial orthotopic liver transplantation is not indicated as a standby procedure till a suitable
donor is found.
Option C: All irreversible causes of fulminant liver failure
• Auxiliary partial orthotopic liver transplantation is not indicated for irreversible causes of fulminant liver
failure.
• However, it is indicated for acute fulminant reversible hepatic failure.

Solution for Question 16:


Option C: Heart
• The organ which is most sensitive to hypothermia is the heart.
• Therefore, hypothermia may present as ventricular fibrillation or asystole, especially when the core
temperature is less than 24 °C.

Page 24

1900
Other complications are as follows:
• Coagulopathies
• Disseminated Intravascular Coagulation (DIC)
• Pancreatitis
• Acute renal failure
Option A: Liver
• The heart is the most sensitive organ to be first affected by hypothermia.
• Therefore, this option is incorrect.
Option B: Kidney
• Hypothermia can precipitate acute renal failure.
• However, it is not the first organ to be affected by hypothermia.
Option D: Brain
• The brain is not the first organ to be affected by hypothermia.

Solution for Question 17:


Option A: Most dangerous complication is sepsis.
• The most dangerous complication is sepsis
• Most of the mortality after small bowel transplantation is due to sepsis and multiorgan failure
Option B: It is widely used due to successful transplantation rates.
• It is not widely used due to vigorous graft rejection because the small intestine contains a very large
amount of lymphoid tissue.
Option C: Most preferred grafts are multi-visceral grafts.
• Multivisceral or cluster transplants may be necessary in case of large dermoid tumors when excision
of both the small bowel and adjacent organ is required when there has been extensive thrombosis of
the splanchnic vessels and for generalized disorders of gastrointestinal motility.
Option D: Most common indication is carcinoma
• Isolated small bowel transplantation is undertaken when possible since patient survival is higher
• The most common indication is ischemia
• It is a treatment option for patients with intestinal failure requiring long-term parenteral nutrition.
• Intestinal failure may result from short bowel syndrome after resection of the intestine or from
intestinal dysfunction.

Solution for Question 18:


Option B: Can be found in all male patients older than 20 years

Page 25

1901
• Hyperacute rejection is a form of rejection that manifests itself in minutes to hours following
transplantation. It occurs due to the presence of pre-existing antibodies in the recipient that recognises
antigens in the donor organ.
• These antigens are located on the endothelial lining of blood vessels within the transplanted organ
and once antibodies bind, it will lead to the rapid activation of the complement system. Irreversible
damage due to thrombosis and subsequent graft necrosis is to be expected.
• Tissue left implanted will fail to work and could lead to high fever and malaise as the immune system
acts against foreign tissue. There is also tenderness at the location of the transplanted organ.
• These preformed antibodies may be the result of prior transplants, prior blood transfusions, or
pregnancy. Hyperacute rejection is most commonly from antibodies to ABO blood group antigens and
may also occur due to HLA mismatch. Consequently, transplants between individuals with different
ABO blood types are generally avoided.
• There is no gender or age specificity for the presence of antibodies against the donor recipient. Graft
failure secondary to hyperacute rejection has significantly decreased as a result of improved
pre-transplant screening for antibodies to donor tissues.
Option A: ABO incompatibility is a contraindication to kidney transplantation
• ABO incompatibility is an absolute contraindication to transplantation.
• It results in hyperacute rejection of graft immediately after transplant due to preformed antibodies
against incompatible donor antigens.
Option C: Positive cross-match
• A positive cross-match means that a donor and recipient are not compatible.
• Positive cross-match results in the recipient's antibodies attacking the donor, which results in
hyperacute graft rejection.
Option D: Occurs during pregnancy and in cases with prior history blood transfusion.
• Prior transplant, pregnancy and prior transfusion from incompatible donors results in the formation of
antibodies against that blood group.
• When that incompatible blood group organ is transplanted into a recipient in which antibodies have
already been produced, it results in hyperacute rejection of the graft.

Solution for Question 19:


Option A: Cyclosporine
• Cyclosporine is an immunosuppressant drug that is commonly used for transplant rejection
prophylaxis. It binds cyclophilin to form a complex that inhibits calcineurin. Inhibition of calcineurin
blocks activation of the NFAT transcription factors, which prevents interleukin-2 transcription and thus
decreases T cell activation.

Page 26

1902
• Common side effects of cyclosporine are: Hypertension Gingival hyperplasia Hirsutism
• Hypertension
• Gingival hyperplasia
• Hirsutism
• Other common side effects include Nephrotoxicity Hyperlipidemia Hyperglycemia Tremor Increased
risk of malignancies (esp. cutaneous squamous cell carcinomas) Infectious diseases (esp. upper
respiratory and urinary tract infections)
• Nephrotoxicity
• Hyperlipidemia
• Hyperglycemia
• Tremor
• Increased risk of malignancies (esp. cutaneous squamous cell carcinomas)
• Infectious diseases (esp. upper respiratory and urinary tract infections)
• Hypertension
• Gingival hyperplasia
• Hirsutism
• Nephrotoxicity
• Hyperlipidemia
• Hyperglycemia
• Tremor
• Increased risk of malignancies (esp. cutaneous squamous cell carcinomas)
• Infectious diseases (esp. upper respiratory and urinary tract infections)
Option B: Tacrolimus
• Tacrolimus is also a drug indicated for transplantation rejection prophylaxis

Page 27

1903
• Its most common side effects include hypertension, which this patient has, as well as hyperglycemia
and nephrotoxicity, which are not reported here
• Gingival hyperplasia and hirsutism are not typical side effects of this drug
Option C: Monoclonal antibody (OKT3)
• The general side effect of Muromonab- CD3 is Cytokine Release Syndrome.
• CRS causes the release of cytokines by activated T cells and is characterized by a "flu-like" illness,
including fever, chills, rigors, headache, nausea, vomiting, diarrhea, malaise, arthralgias, myalgias, and
weakness.
• More serious manifestations include pulmonary oedema, hemodynamic instability, shock, respiratory
arrest, and cardiac arrest.
Option D: Anti-lymphocyte globulin
• The most common adverse reactions and laboratory abnormalities (incidence >5% higher than
comparator) are urinary tract infections, abdominal pain, hypertension, nausea, shortness of breath,
fever, headache, anxiety, chills,hyperkalemia, thrombocytopenia, and leukopenia.

Solution for Question 20:


Option D: It slows down metabolism and the enzymatic processes that would destroy the cell
• Organ preservation is the supply line for organ transplantation. Currently, solid organs like the liver,
pancreas, and kidney can be successfully preserved for up to two days by flushing the organs with the
University of Wisconsin (UW) organ preservation solution and storing them at 0–5° C (hypothermia).
• At the cellular level, the basis of hypothermic protection is the effect of temperature on reaction rates
which are generally slowed by a reduction in temperature (typically a 50% reduction for each 10-degree
drop in temperature for most reactions). The processes of deterioration associated with ischemia and
anoxia are mediated by chemical reactions, and these reactions are slowed down by hypothermia.
These deleterious changes begin with the depletion of high energy reserves (adenosine triphosphate
and creatine phosphate) and membrane depolarization.
• Temperature is known to influence: - Reaction rates - Energy metabolism - Active ion transport and
ion homeostasis - Membrane fluidity - Function and secretion of hormones and neurotoxins
• Cooling has proven to be the first line of defense against hypoxic injury and is necessary to reduce
cellular metabolism and the requirements for oxygen to prevent tissue injury. Most organs are stored by
static cold storage in an icebox at 4 degree Celsius.
• Cold storage solutions like the UW solution mitigate the adverse effects of hypothermia. UW solution
is effective because it uses a number of impermeant cell agents (lactobionic acid, raffinose,
hydroxyethyl starch) that prevent the cells from swelling during cold ischemic storage.
• UW solution also contains glutathione and adenosine, agents that may stimulate recovery of normal
metabolism upon reperfusion by augmenting the antioxidant capacity and membrane integrity of the
organs (glutathione) or by stimulating high-energy phosphate generation ATP (adenosine) upon
reperfusion.
Organ
Optimum(hours)
Safe maximum (hours)

Page 28

1904
Kidney
<18
36
Liver
<12
18
Pancreas
<10
Small intestine
<4
6
Heart
<3
Lung
8
Other Options
Option A: Oxygen is more soluble in cold solutions and provides a
continual supply of energy metabolism
• As a fact, oxygen solubility increases with a decrease in temperature. But oxygen is not the sole
contributor to keeping the organ alive. Enzymatic functions and metabolic processes are more
important to be in a proper and balanced state to keep the organ alive.
Option B: There is no way to suppress microbial growth except by cooling and slowing the growth rate
• Pharmacological inhibition methods are used to inhibit the growth of microorganisms on preserved
organs rather than hypothermia.
Option C: Hypothermia diminishes energy requirements and allows the limited energy reserve to keep t
he organ alive
• Slowing down the energy requirement of the organ is not much helpful in preserving an organ.
Enzymatic processes damaging a cell are more important to deal with than those achieved with
hypothermia.

Solution for Question 21:


Option B: In single-lung transplantation, changes in pulmonary function are seen almost immediately fo
llowing transplantation
• This patient's right-sided heart failure is most likely related to chronic hypoxic pulmonary
vasoconstriction due to his COPD (e.g., cor pulmonale).

Page 29

1905
• COPD is characterized by pulmonary inflammation resulting in fibrosis, airway obstruction, and
capillary loss. Subsequent hypoxia and hypercapnia cause compensatory pulmonary vasoconstriction.
• The combination of hypoxic vasoconstriction and chronic inflammation triggers vascular remodeling,
which in turn causes elevated pulmonary capillary pressure and pulmonary hypertension.
• Chronic, untreated pulmonary hypertension puts a strain on the right ventricle, causing concentric
hypertrophy and progressive decline in right heart function, eventually leading to symptoms of
decompensated right heart failure, as seen in this patient.
• After the failure of medical treatment, a definitive treatment plan is the transplantation of the lungs.
Changes in the cardiopulmonary function that occur after transplantation are as follow:
• IMMEDIATE CHANGES:
• Reversal of right ventricular dysfunction (decreased dilation and increase in contractility) measured by
transesophageal echocardiography
• Pulmonary artery pressure decreases
• Improvement in pulmonary function measured by FEV1
All of these changes have occurred by the time the patient leaves the operation theater.
• LATE CHANGES:
• Ventilation/perfusion mismatch improvement within three months
• Marked improvement in FEV1 and gets tripled within two weeks
Improvement is better if bilateral lung transplantation is carried out.

Option A: In patients with pulmonary hypertension, changes in right ventricular function and pulmonary
artery pressure take weeks to months to resolve
• The right ventricular changes are assessed by transesophageal echocardiography. The
interventricular septum previously bulging into the left ventricle immediately assumes the normal
position. An increase in contractility of the right ventricle occurs with a significant decrease in dilatation.
• The pulmonary artery pressure immediately decreases and is essentially normal by the time the
patient leaves the operating room.
Option C: Patients with double lung transplants have both better pulmonary function studies as well as
better exercise capabilities
• Patients with double lung transplantation have better pulmonary function studies as compared to
single lung transplantation. On table assessment of pulmonary function studies, which is measured by
FEV1, shows more improvement.
• On the other hand, exercise capabilities are not significantly improved as compared to single lung
transplantation.
Option D: After a
single-lung transplant, ventilation-perfusion mismatch persists, and carbon dioxide retention is seen
• Significant ventilation-perfusion mismatch occurs with ventilation to the native lung occurring
preferentially due to its higher compliance. Conversely, perfusion should preferentially go to the newly
transplanted lung because of lower pulmonary vascular resistance.
• Despite this, patients post surgery do well from a functional standpoint and do not show carbon
dioxide retention.

Page 30

1906
• By three months after transplantation, the ventilation/perfusion mismatch narrows.

Solution for Question 22:


Option A: History of diabetes
• A kidney-pancreas transplant is a surgery to place both the kidneys and pancreas, at the same time,
into someone who has kidney failure related to type 1 diabetes mellitus.
• Both transplanted organs may come from one deceased donor. However, it is also possible for the
kidney to come from a living donor (a family member or friend) and the pancreas from a deceased
donor.
• Pancreatic transplants are performed in three recipient categories: Simultaneous pancreas and kidney
(SPK) transplant in diabetic and uremic patients Pancreas after kidney (PAK) transplant in diabetic and
post uremic patients Pancreas transplant alone (PTA) in non uremic patients with brittle diabetes
• Simultaneous pancreas and kidney (SPK) transplant in diabetic and uremic patients
• Pancreas after kidney (PAK) transplant in diabetic and post uremic patients
• Pancreas transplant alone (PTA) in non uremic patients with brittle diabetes
• Simultaneous pancreas and kidney (SPK) transplant in diabetic and uremic patients
• Pancreas after kidney (PAK) transplant in diabetic and post uremic patients
• Pancreas transplant alone (PTA) in non uremic patients with brittle diabetes
Contraindications to be a donor of the pancreas:
• Type I diabetes mellitus in the donor is an absolute contraindication.
Relative contraindications to be a pancreas donor:
Relative contraindications are
• Previous pancreatic procedures
• Pancreatic disorders such as chronic pancreatitis and intraductal papillary mucinous neoplasm.
• Hyperglycemia in itself is not a contraindication.
Exclusion Criteria
Inclusion Criteria
• Type 1 or 2 diabetes (A1c>6.5%)
• Malignancies other than primary brain tumor
• Septicemia
• Viral hepatitis
• Acquired immunodeficiency syndrome
• Human immunodeficiency virus
• Syphilis
• Viral encephalitis
• Creutzfeldt-Jakob disease

Page 31

1907
• Rabies
• Tuberculosis
• Periods of relevant hypotension
• Elevated serum creatinine (>2 times normal range)
• Abnormal liver function tests ( >3 times normal range)
• Elevation in serum amylase
• Elevation in serum lipase
• History of promiscuous sexual behavior
• History of IV drug use
• Multiorgan donors
• Either sex 15-70 years of age
• Warm ischemia time (WIT) <10 min
• Cold ischemia time (CIT) <12 h in the absence of TLM
Option B: No indication for liver transplant in the recipient.

All three abdominal organs (pancreas, liver and intestine) can be procured at the same time and transp
lanted into the same or different recipients.
Option C: Replaced hepatic artery arising from the superior mesenteric artery (SMA) in the donor
• If a replaced or aberrant right hepatic artery, the first branch of the superior mesenteric artery (SMA),
is present, it is carefully dissected out from the posterior surface of the pancreas.
• A replaced or aberrant right hepatic artery does not transverse the pancreas and is not a
contraindication to the combined pancreas and liver procurement.
Option D: Technical complication rates are higher for pancreatic transplants than other solid organ tran
splants.
The technical complication rate for pancreatic transplants is higher than for any other solid organ trans
plant.
• The most common surgical complications are thrombosis, intra-abdominal abscesses, and bleeding.
• The most common non-surgical complication post-transplant is rejection.

Page 32

1908
Robotics, Laparoscopy and Bariatric Surgery
1. Which of the following sites is the entry site for NOTES surgery?
(or)
A 50-year-old man is admitted to the hospital for a surgical procedure. The surgeon wants to perform
the surgery through the NOTES procedure. The device shown in the image is a Master and Slave
Robotic System used to perform NOTES surgery. In this procedure, the entry point is through which of
the following sites?

A. Ear
B. Umbilicus
C. Mouth
D. Nose
----------------------------------------
2. Which of the following is not an advantage of minimal access surgery?
(or)
A 23-year-old female presents with a cystic lesion in the distal body of the pancreas. On abdominal
examination, no mass is palpable. Serological examination revealed serum carcinoembryonic antigen
(CEA) to be 2.5 ng/dL, and serum carbohydrate antigen (CA) 19.9 to be 6.7 IU/dL. Triphasic CT
abdomen shows a 4.5 × 3.2 cm cystic lesion in the body of the pancreas with calcifications. The
diagnosis of a mucinous cystic neoplasm is made, and the patient is admitted for laparoscopic distal
pancreatectomy. Which of the following is not an advantage of advising this patient a minimal access
surgery?
A. Better tactile sensation
B. Better hemostasis control
C. Improved visual field
D. Less wound pain
----------------------------------------
3. Balu is a 15-year-old male adolescent referred to the weight clinic with a body mass index (BMI) of
50. He is referred based on his history of severe obesity and nonalcoholic fatty liver disease and for a
discussion of weight loss surgery. The patient has tried a low-carbohydrate and low-fat diet, reduced
his portion sizes, worked with a nutritionist, and increased his level of physical activity, but could not
achieve or maintain a healthy body weight. After evaluating the patient, bariatric surgery, specifically a

1909
laparoscopic sleeve gastrectomy (LSG), is Balu’s best option to treat his severe obesity. Which of the
following is a feature of bariatric surgery?
(or)
Which of the following is a feature of bariatric surgery?
A. Indication for bariatric surgery: patients with a BMI of 25 Kg/m2 or more with comorbidities.
B. The most commonly performed bariatric procedure nowadays is Roux-en-Y gastric bypass.
C. Laparoscopic adjustable gastric banding is irreversible
D. The most common cause of death within 30 days of bariatric surgery is peritonitis due to an
anastomotic leak
----------------------------------------
4. Marimuthu is a 65-year-old man with type 2 diabetes. His primary care physician refers him to you for
an evaluation of his obesity and treatment options, including weight-loss surgery. He weighs 156 kg
and is 182cm tall, with a BMI of 47.1 kg/m2. Roux-en-Y Gastric Bypass (RYGB) surgery is
recommended. The patient’s blood glucose level on post-op day 2 is 156 mg/dl. Two months
post-surgery, the patient is recovering well, and his fasting and pre-dinner blood glucose levels are
consistently < 120 mg/dl, without oral hypoglycemic agents. In addition to the effects of weight loss, the
resolution of type 2 diabetes mellitus after the RYGB is thought to be contributed by which of the
following?
(or)
The resolution of type 2 diabetes mellitus after the RYGB is thought to be contributed by which of the
following in addition to weight loss?
A. Increased ghrelin production
B. Decreased secretion of glucagon-like peptide-1 (GLP-1)
C. Increased activity of beta cells of the pancreas
D. Decreased level of peptide YY
----------------------------------------
5. A 36-year-old woman comes to the hospital for a hysterectomy. The surgeon advises her about the
advantages of using robotic devices for surgery. What is the name of the device shown in the image?
(or)
What is the name of the device shown in the image?

A. PUMA 560

Page 2

1910
B. DaVinci robot
C. PROBOT
D. ROBODOC
----------------------------------------
6. A 45-year-old woman with morbid obesity presents to the hospital for weight loss surgery. Her Body
Mass Index (BMI) is 42 kg/m2. She has a history of hypertension and type-2 diabetes mellitus. All her
previous attempts to lose weight via dietary modifications, lifestyle changes and exercise have failed.
The surgeon discusses the surgical procedure he wants to perform with the patient. A pictorial diagram
of the process is shown below. Which of the following is a feature of this procedure?
(or)
Which of the following is a feature of the procedure given below?

A. Moderate weight loss


B. Mildly malabsorptive & largely restrictive
C. The side effect is 2- 4 malodorous bowel movements per day
D. No effect on Type 2 diabetes mellitus
----------------------------------------
7. A 50-year-old man is admitted to the hospital for colorectal surgery. He has a history of bleeding per
rectum and recurrent diverticulitis. His blood pressure on admission is 110/70 mm Hg, and his heart
rate is 80 beats per minute. He has no history of hypertension or diabetes mellitus. The surgeon
decides to remove the involved segment of the large intestine and wants to do a fast track surgery.
Which of the following is a component of fast track surgery?
(or)
Which of the following is a component of fast track surgery?
A. Fasting of 12 hours for solids & liquids
B. Use of Romo vac drain
C. Delayed post operative mobilization
D. Prevention of gastrointestinal atony and postoperative nausea and vomiting
----------------------------------------

Correct Answers

Page 3

1911
Question Correct Answer

Question 1 3
Question 2 1
Question 3 2
Question 4 3
Question 5 2
Question 6 3
Question 7 4

Solution for Question 1:


Option C: Mouth
• In NOTES surgery, the peritoneal cavity is accessed via a natural orifice. So the entry point from the
given options is the mouth.
Option A: Ear
• The ear cannot be used to access the peritoneal cavity.
Option B: Umbilicus
• There is no natural opening through the umbilicus, so the device cannot be used.
Option D: Nose
• The nose cannot be used to access the peritoneal cavity.

Solution for Question 2:


Option A: Better tactile sensation
• Tactile sensation is reduced for the surgeon in minimal access surgeries due to the use of
instruments.
Option B: Better hemostasis control
• Laparoscopic surgery is considered to have a better hemostasis control
Option C: Improved visual field.
• By inserting a scope into the body, the surgeon gets a better visual field to operate.
Option D: Less wound pain.
• Postoperative pain is reduced due to smaller incisions and less tissue handling.

Solution for Question 3:


Option B: Most commonly performed procedure nowadays is Roux-en-Y gastric bypass (RYGB).

Page 4

1912
• Bariatric surgery is a procedure to help reduce weight in cases where dietary and medical
interventions have failed.
• RYGB is a bariatric procedure resulting in a restrictive and mildly malabsorptive condition.
• The upper part of the stomach is surgically attached directly to the small intestine.
• It is the most commonly performed procedure now.
Option A: Indications for Bariatric surgery: patients with a BMI of 25 Kg/m2 or more with comorbidities.
• Bariatric surgery is performed in patients with a BMI of 40 kg/m2 or more with or without
comorbidities, and in patients with a BMI of 35 kg/m2 or more with comorbidities .
Option C: Laparoscopic adjustable gastric banding is irreversible.
• Laparoscopic gastric banding is a reversible procedure.
Option D: The most common cause of death within 30 days of bariatric surgery is peritonitis due to an
anastomotic leak.
• One of the complications of bariatric surgery is peritonitis due to an anastomotic leak. However, this is
an immediate postoperative complication.
• The most common cause of death after bariatric surgery within 30 days is pulmonary embolism (36%),
followed by cardiac complications (24%) and anastomotic leaks (20%).

Solution for Question 4:


Option C: Increased activity of beta cells of the pancreas
• After RYGB, there is a marked increase in the activity of beta cells in the pancreas. This is thought to
be due to increased secretion of GLP-1 (Glucagon like peptide- 1). Hence type-2 Diabetes Mellitus
improves after the procedure.
Option A: Increased ghrelin production
• Ghrelin is a hormone that stimulates hunger and increases the drive to eat.
• Gastrectomy removes much of the ghrelin-producing portion of the stomach, leading to weight loss
after both sleeve gastrectomy and RYGB procedures. So there is a decrease in ghrelin production.
Option B: Decreased secretion of glucagon-like peptide-1 (GLP-1)
• GLP-1 is dramatically increased after RYGB and sleeve gastrectomy.
• GLP-1 helps stimulate beta cell proliferation and upregulation of genes involved in insulin secretion.
• It also inhibits glucagon secretion and reduces appetite and gastric motility.
Option D: Decreased level of peptide YY
• There is an increase in peptide YY (PYY) levels after RYGB. Peptide YY is thought to contribute
towards appetite suppression and therefore, helps with weight loss and improvement in diabetes
mellitus type 2.

Solution for Question 5:

Page 5

1913
Option B: DA-Vinci Robot
• It is the most commonly used robotic device worldwide.
• It is based on the Master-Slave principle
• During the procedure, the surgeon inserts his hand into the master and translates his hands into
robotic arms-like instruments on the Da-Vinci robot.
Indications for using DA-Vinci Robot:
• Prostatectomy
• Cardiac valve repair
• Gynaecological surgeries
Option A: PUMA 560
• PUMA (Programmable Universal Machine for Assembly) is a robotic device used in industries for
assembly of electronics.
Option C: PROBOT
• The PROBOT is used in prostate surgery for resection of the prostate.
Option D: ROBODOC
• The Robodoc is a surgical system used in orthopedic surgery.

Solution for Question 6:


Option C: The side effect is 2- 4
malodorous bowel movements per day The procedure shown in the picture is biliopancreatic diversion.
• It is the most effective type of bariatric surgery
• It is associated with maximum weight loss
• The main side effect is 2- 4 bowel malodorous movements per day (due to fat malabsorption)
• Components of this procedure include: Distal gastrectomy Gastro-ileostomy Jejunoileostomy
Cholecystectomy Appendectomy
• Distal gastrectomy
• Gastro-ileostomy
• Jejunoileostomy
• Cholecystectomy
• Appendectomy
• Length of the common channel created by this procedure is 50 cm. The food is allowed to mix with the
bile and pancreatic enzymes only in the distal 50 cm of the ileum, causing decreased digestion and fat
absorption.
• It is largely malabsorptive and mildly restrictive
• Maximum remission of type 2 diabetes mellitus occurs with this surgery
• Distal gastrectomy

Page 6

1914
• Gastro-ileostomy
• Jejunoileostomy
• Cholecystectomy
• Appendectomy
Disadvantage
• There is a high risk of ulcer at the site of anastomosis of gastroileostomy
Option A: Moderate weight loss
• Biliopancreatic diversion is considered to have the maximum weight loss in all bariatric surgeries.
Option B: Mildly malabsorptive and largely restrictive
• This procedure results in a largely malabsorptive and mildly restrictive state.
Option D: No effect on type-2 diabetes mellitus
• In a biliopancreatic diversion, maximum remission is seen for type-2 diabetes mellitus; therefore,
patients with type-2 diabetes benefit greatly from this surgery.

Solution for Question 7:


Option D: Prevention of gastrointestinal atony and postoperative nausea and vomiting.
• Fast-track (FT) surgery is a multimodal concept aiming to reduce postoperative pain and
stress-induced organ dysfunction.
• A major component of Fast track surgery (FT) is the prevention of gastrointestinal atony and
postoperative nausea and vomiting.
• This helps reduce morbidity post-surgery and results in better surgical outcomes.
Option A: Fasting 12 hours for solids and liquids.
• FT requires a fasting of 6 hours for solids and 2 hours for liquids, unlike conventional surgery, which
requires 10- 12 hours of fasting.
Option B: Use of Romo vac drain
• Fast-track surgery is an atraumatic technique that avoids usage of drains.
• A Romo vac drain is used in closed wound drainage but not in fast track surgery.
Option C: Delayed postoperative mobilization.
• The cornerstone of FT surgery is early mobilization post operatively, within the first 6 hours.

Page 7

1915
Sutures and Anastomosis
1. A 22-year-old otherwise healthy male presented to the ER following a single gunshot wound to the
left groin. On examination, the patient was hemodynamically stable but had no palpable lower extremity
pulses on the injured side. The ankle-brachial index confirmed an arterial injury (<0.9). On immediate
exploration, a transected superficial left femoral artery was identified. Which of the following is the
suture with maximum tensile strength and minimum tissue reaction that can be used in this patient?
(or)
Which of the following sutures have maximum tensile strength and minimum tissue reaction?
A. Poliglecaprone
B. Polypropylene
C. Polyglactin
D. Polydioxanone
----------------------------------------
2. An intern has his first day of rotation in the operation room; he asks a resident about the type of
sutures and their preferred uses. Which of the following is true regarding sutures?
(or)
Which of the following is true regarding sutures?
A. Catgut is a natural absorbable suture
B. Monocryl is used for sternotomy closure
C. Vicryl is the preferred suture for vascular anastomosis
D. Prolene suture is preferred for bile duct and urinary bladder
----------------------------------------
3. The tissue adhering glue contains which of the following materials?
(or)
A 3-year-old came to the hospital with a simple cheek laceration (image). A tissue adhesive was used
to repair the laceration. This tissue suturing glue contains which of the following materials?

A. Cyanoacrylate
B. Ethanolamine oleate
C. Methacrylate

1916
D. Polychloroprene
----------------------------------------
4. While assisting the hernia repair, the resident asks the intern to open the suture. When the intern
was opening the suture, he felt that the suture was preserved in some material. The suture material
shown in the image is preserved in which of the following?
(or)
The suture material shown in the image below is preserved in which of the following?

A. Glutaraldehyde
B. Isopropyl alcohol
C. Iodine
D. Cetrimide
----------------------------------------
5. A 74-year-old woman was hospitalized because of microcytic anemia, weight loss, anorexia, and
vomiting. Upper gastro-endoscopy with biopsy revealed an ulcer 4 cm x 3 cm at the gastric antrum.
Histopathological examination found a poorly differentiated ulcerative adenocarcinoma (ADC). The
patient was scheduled for an open D2 total gastrectomy with an end-to-side esophagojejunal
anastomosis with a 21 mm EEA stapler and Roux-en-Y reconstruction. In this intestinal anastomosis,
strength is mainly provided by which of the following?
(or)
In intestinal anastomoses, strength is mainly provided by which of the following layers?
A. Mucosa
B. Submucosa
C. Serosa
D. Muscularis mucosa
----------------------------------------
6. A 22-month-old female child was reported to the hospital with a chief complaint of swelling on the left
cheek region. There was a history of traumatic injury while eating sugarcane twenty-four hours back.
On examination, extraoral swelling was present in the left cheek region. A clinical diagnosis of
herniation of the buccal fat pad was made. The necrotic mass was surgically excised under local
anesthesia, and healthy tissue was repositioned, followed by primary closure using a 4-0 vicryl suture,
as shown in the image. Which of the following is related to this suture material?

Page 2

1917
(or)
Which of the following is related to vicryl suture material?

A. Tensile strength is maintained for 8-10 days


B. Copolymer of glycolide and caprolactone
C. Delayed absorbable suture
D. Used in Whipple's procedure
----------------------------------------
7. Which of the following statements is true?
A. Kocher's two-layer continuous type anastomosis is preferred for bowel anastomosis.
B. 8-o prolene sutures are used for popliteal to distal arteries.
C. Tissue glue is based upon a solution of n-butyl-2-cyanoacrylate monomer
D. Lightweight mesh weight is <80 gm/m2
----------------------------------------
8. Which of the following is the exact skin suturing principle?
(or)
A 34-year-old male sustained a laceration on the forearm while he was cutting coconuts. The wound
was thoroughly washed and sutured in layers. Which of the following is the exact suturing principle?
A. Each suture should be separated by a gap that is twice the thickness of the skin
B. Suture length to wound length ratio is 6:1
C. Length to-width ratio for elliptical incisions is 2:1
D. Distance from the entry point to the edge of the wound should be twice the depth of tissue being
sutured.
----------------------------------------
9. Match the following suture types with their composition. 1. Vicryl a. Glycolide + Caprolactone 2.
Polydioxanone b. Polyglycolic acid 3. Dexon c. Polyester 4. Monocryl d. Glycolide + Lactide
1. Vicryl a. Glycolide + Caprolactone
2. Polydioxanone b. Polyglycolic acid
3. Dexon c. Polyester

Page 3

1918
4. Monocryl d. Glycolide + Lactide

A. 1-a, 2-d, 3-b, 4-c


B. 1-a, 2-b, 3-c, 4-d
C. 1-d, 2-c, 3-b, 4-a
D. 1-c, 2-a, 3-d, 4-b
----------------------------------------
10. Below are images of different types of surgical knots. Match them with their corresponding names.
1. a. Granny knot 2. b. Square knot 3. c. Surgeon's knot

1. a. Granny knot

2. b. Square knot

3. c. Surgeon's knot

A. 1-c, 2-a, 3-b


B. 1- c, 2-b, 3-a

Page 4

1919
C. 1-b, 2-c, 3-a
D. 1-a, 2-b, 3-c
----------------------------------------
11. A general surgeon uses mattress sutures for wound edge eversion. Which of the following is not a
characteristic of horizontal suture?
A. Corner stitch is a variant
B. It is effective in holding fragile skin together
C. Used in wounds in the posterior neck or on a concave surface
D. Effective hemostatic sutures on vascular tissues such as the scalp
----------------------------------------
12. Identify all the statements that are true about the uses of linear staplers. Create side-to-side
anastomosis Create end-to-end anastomosis Ends are closed by a transverse stapler Low anterior
resection and stapled hemorrhoidopexy
A. 1,2
B. 1 only
C. 1, 3, 4
D. 1 and 3 only
----------------------------------------

Correct Answers
Question Correct Answer

Question 1 2
Question 2 1
Question 3 1
Question 4 2
Question 5 2
Question 6 3
Question 7 3
Question 8 1
Question 9 3
Question 10 1
Question 11 3
Question 12 4

Solution for Question 1:


Option B: Polypropylene

Page 5

1920
• Polypropylene (non-absorbable) has maximum tensile strength and minimum tissue reaction.
• Polypropylene suture: Polypropylene suture is a monofilament (non-absorbable) surgical suture. It
comprises an isotactic crystalline stereoisomer of polypropylene, a synthetic linear polyolefin. It is
pigmented in blue color to enhance visibility. Polypropylene sutures elicit maximum tensile strength and
minimal tissue reaction which does not cause tissue rupture. This suture is indicated for use in general
soft tissue approximation and ligation, including use in cardiovascular, plastic, neurological surgeries,
ophthalmic, micro procedures, and hernia repair. It is the preferred suture material for vascular
anastomosis. It is neither absorbed nor subjected to weakening by tissue enzymes, non-adhering, and
biologically inert. Thus, it is used for permanent fixtures.
• Polypropylene suture is a monofilament (non-absorbable) surgical suture. It comprises an isotactic
crystalline stereoisomer of polypropylene, a synthetic linear polyolefin. It is pigmented in blue color to
enhance visibility.
• Polypropylene sutures elicit maximum tensile strength and minimal tissue reaction which does not
cause tissue rupture.
• This suture is indicated for use in general soft tissue approximation and ligation, including use in
cardiovascular, plastic, neurological surgeries, ophthalmic, micro procedures, and hernia repair. It is the
preferred suture material for vascular anastomosis. It is neither absorbed nor subjected to weakening
by tissue enzymes, non-adhering, and biologically inert. Thus, it is used for permanent fixtures.
• Polypropylene suture is a monofilament (non-absorbable) surgical suture. It comprises an isotactic
crystalline stereoisomer of polypropylene, a synthetic linear polyolefin. It is pigmented in blue color to
enhance visibility.
• Polypropylene sutures elicit maximum tensile strength and minimal tissue reaction which does not
cause tissue rupture.
• This suture is indicated for use in general soft tissue approximation and ligation, including use in
cardiovascular, plastic, neurological surgeries, ophthalmic, micro procedures, and hernia repair. It is the
preferred suture material for vascular anastomosis. It is neither absorbed nor subjected to weakening
by tissue enzymes, non-adhering, and biologically inert. Thus, it is used for permanent fixtures.

Page 6

1921
Option A: Poliglecaprone
• Poliglecaprone is a synthetic absorbable sterile surgical suture composed of polymers made from
100% homopolymer of polyglycolide and caprolactone.
• It has been found non-antigenic and non-pyrogenic and exhibits slight tissue reaction during
absorption. It is available in undyed and violet colors.
• It is an ideal approximation suture for subcuticular closure. It retains 50%-60% of its tensile
strength for 7 days, 20%-30% for 14 days, provides wound support for 21-28 days and is absorbed
through simple hydrolysis, and completely gets absorbed within 90-120 days which is predictable.
Option C: Polyglactin
• Polyglactin is a synthetic absorbable sterile surgical suture. It is a co-polymer of 90% glycolide and
10% L-lactide.
• It retains tensile strength of about 70% till 14 days after implantation, it completely absorbs within
56-70 days by a simple hydrolytic mechanism, and its absorption is predictable.
Option D: Polydioxanone
• Polydioxanone is a synthetic absorbable sterile surgical suture composed of polyester
(poly-dioxanone).
• It is suitable for cases where extended wound support and strength are required.
• It retains about 70% of the initial strength for 2 weeks, 50% for 4 weeks, and 25% for 6 weeks and is
absorbed through simple hydrolysis and completely gets absorbed within 180-210 days.
• The absorption of this suture is reliable and predictable.

Solution for Question 2:


Option A: Catgut is a natural absorbable suture
• This statement is correct; catgut is a natural absorbable suture

Page 7

1922
Option B: Monocryl is used for sternotomy closure
• Stainless steel is used for sternotomy closure.
• Therefore, this statement is incorrect.
Option C: Vicryl is the preferred suture for vascular anastomosis
• Preferred suture for vascular anastomosis is prolene.
Option D: Prolene suture is preferred for bile duct and urinary bladder
• Vicryl is the preferred suture for bile duct and urinary bladder.

Solution for Question 3:


Correct Option A: Cyanoacrylate
• Tissue suturing glue contains Cyanoacrylate.
Tissue glue
• Tissue glue or adhesives can be used to approximate wounds that do not require deep-layer
closure and do not have significant tension on the edges of the wound.
• It provides an alternative method for wound closure that is painless, fast, water resistant and does not
require a follow-up visit for removal.
• Tissue glue is based on an n-butyl-2-cyanoacrylate monomer.
• When it is applied to a wound, it polymerizes to form a firm adhesive bond which causes the closure of
the wound.
• The wound does need to be clean and dry, with near-perfect hemostasis, and under no tension for
effective closure.
• Relatively expensive, it is quick to use, does not delay wound healing, and is associated with an
allegedly low infection rate.
Option B: Ethanolamine oleate
• It is a sclerosing agent used in sclerotherapy and is not present in tissue glue.
Option C: Methacrylate
• Methacrylate adhesives are used to bond metals, plastics, and composites. It is not used as tissue
glue.
Option D: Polychloroprene
• Polychloroprene adhesives are used for gaskets, cable jackets, tubing, seals, O-rings, tire sidewalls,
gasoline hoses, and weather-resistant products such as wet suits and orthopedic braces.
• It is also used as a base resin in electrical insulation, adhesives, and coatings. It is not used as tissue
glue.

Solution for Question 4:

Page 8

1923
Option B: Isopropyl alcohol
CATGUT
• Catgut is preserved in isopropyl alcohol, to prevent it from drying.
• Plain catgut loses 50% tensile strength in 3 days and all tensile strength in 15 days and is absorbed in
60 days.
• Isopropyl alcohol is used as a preservative for packing catgut sutures.
Option A: Glutaraldehyde
• Glutaraldehyde is a colorless glass-like crystal that is usually used in a concentration of 2% to 50% in
a water solution.
• It is used for cold sterilization of dental and medical equipment and as a preservative, biocide,
hardener, and tanning agent.
Option C: Iodine
• Because of their antimicrobial properties, iodine compounds are used as effective antiseptics.
• They are commonly used by medical professionals as a skin preparation agent to reduce the
incidence of infection after surgery.
Option D: Cetrimide
• Cetrimide is used as a common preservative in eye drops, eardrops, nasal drops or sprays,
antiseptics, abrasions, insect bites, minor burns, wounds and other conditions.

Solution for Question 5:


Option B: Submucosa
• In intestinal anastomosis, strength is mainly provided by the submucosa.
• It has a high collagen content and is the most stable suture layer in all sections of the gastrointestinal
tract
Option A: Mucosa
• The mucosa consists of epithelium, lamina propria and muscularis mucosae.
• Mucosa does not have enough collagen content.
• Therefore, it does not have enough strength for anastomosis.
Option C: Serosa
• Serosa is formed by a serous membrane; it does not have enough collagen content.
• Therefore, it does not have enough strength for anastomosis.
Option D: Muscularis mucosa
• Muscularis mucosa is a muscular layer of mucosa; it does not have enough collagen content.
• Therefore, it does not have enough strength for anastomosis.

Page 9

1924
Solution for Question 6:
Option C: Delayed absorbable suture
• Vicryl (polyglactin) is also known as the work-horse suture of a general surgeon.
• Vicryl is a synthetic absorbable sterile surgical suture composed of a co-polymer made from 90%
glycolide and 10% L-lactide.
• It retains tensile strength of about 70% and is maintained for 28-30 days, it completely absorbs within
56-70 days (delayed absorbable suture) by a simple hydrolytic mechanism, and its absorption is
predictable.
• It is violet in color and used in organs with an increased risk of stone formation, such as the common
bile duct, ureter, and urinary bladder.
Option A: Tensile strength is maintained for 8-10 days
• The tensile strength of Vicryl is maintained for 56-70 days, not for 8-10 days.
Option B: Copolymer of glycolide and caprolactone
• Vicryl is a copolymer of glycolide and L-lactide, not caprolactone.
Option D: Used in Whipple's procedure
• Vicryl is not used in Whipple's procedure.

Solution for Question 7:


Option C: Tissue glue is based upon a solution of n-butyl-2-cyanoacrylate monomer
Tissue glue
• It is an alternative to suture techniques.
• It is based on a solution of n-butyl-2-cyanoacrylate monomer.
• When applied to a wound, it polymerises to form a firm adhesive bond, but the wound needs to be
clean and dry, with near-perfect hemostasis and under no tension.
Option A: Kocher's two-layer anastomosis is preferred for bowel anastomosis
• Kocher’s two-layered anastomosis- first a continuous all-layer suture using catgut, then an inverting
continuous (or interrupted) seromuscular layer suture with silk is placed.
• Halsted and Matheson are one layered extra mucosal closure - least tissue necrosis or luminal
narrowing. This technique has now become widely accepted.
Option B: 8-o prolene sutures are used for popliteal to distal arteries
Vascular anastomoses
• Vascular anastomoses require more precision than bowel anastomosis as they must immediately be
watertight at the end of the surgery when the clamps are removed.
• Suture size depends on the vessel caliber: 2/0 for aorta 4/0 for femoral artery 6/0 for popliteal to
distal arteries
• 2/0 for aorta
• 4/0 for femoral artery

Page 10

1925
• 6/0 for popliteal to distal arteries
• Microvascular anastomosis is made using a loupe and an interrupted suture down to 10/0 size.
• Polypropylene-like sutures with indefinite integrity give the best results.
• The intimal suture line must be smooth.
• Knots must be secure.
• The needle must pass from within outwards.
• 2/0 for aorta
• 4/0 for femoral artery
• 6/0 for popliteal to distal arteries
Option D: Lightweight mesh weight is <80 gm/m2
• Lightweight mesh weight is <40 gm/m2 ; Heavyweight mesh weight is >80 gm/m2

Solution for Question 8:


Option A: Each suture should be separated by a gap that is twice the thickness of the skin
• The distance from the entry point to the edge of the wound should be approximately the same as the
depth of the tissue being sutured, and each successive suture should be placed twice this distance
apart.
JENKIN’S RULE:
• Suture length: wound length ratio= 4:1, i.e. if would length is X; the suture length is 4X
• Elliptical Incision: Length: Width is 3X (If the width of the incision is X, the length is 3X) If the thickness
of the skin is X, the distance between the sutures should be 2X
• Length: Width is 3X (If the width of the incision is X, the length is 3X)
• If the thickness of the skin is X, the distance between the sutures should be 2X
• Length: Width is 3X (If the width of the incision is X, the length is 3X)
• If the thickness of the skin is X, the distance between the sutures should be 2X
Option B: Suture length to wound length ratio of 6:1
• Suture length: wound length ratio= 4:1, i.e. if would length is X; the suture length is 4X
Option C: Length to width ratio for elliptical incisions is 2:1
• For elliptical incisions, the suture length to wound width ratio is 3:1
• If the width of the incision is X, the length is 3X.
Option D: The distance from the entry point to the edge of the wound should be twice the depth of tissu
e being sutured.
• The distance from the entry point to the edge of the wound should be approximately the same as the
depth of the tissue being sutured.

Page 11

1926
Solution for Question 9:
Correct Option C- 1-d, 2-c, 3-b, 4-a:
• Vicryl suture: It is a smooth, synthetic, absorbable suture that passes through the tissue readily with
minimal drag. It facilitates ease of handling, smooth tie-down, and good knot security. It is a copolymer
of glycolide and lactide.
• Polydioxanone: It is a synthetic, absorbable, monofilament suture made from a polymer of polyester. It
has greater initial tensile strength than polyglycolic acid and polyglactin 910 but has poor knot security.
• Dexon: It is also an absorbable suture and is a polymer of polyglycolic acid (polyfilament suture).
• Monocryl: It is also known as polyglycaprone and is a copolymer of glycolide and caprolactone.

Solution for Question 10:


Correct Option A- 1-c, 2-a, 3-b:
• Surgeon’s knot: It consists of two throws. There are two wraps in the first throw and one in the second,
and crossing occurs in each throw.
• Granny knot: This knot, also called a pseudo square knot, involves two throws taken without crossing.
One end is above, and the other is below. The granny knot is more prone to slip and is relatively
unstable.
• Square knot: This knot, also known as the reef knot, consists of two throws, and crossing is done in
each throw. It is relatively secure.

Solution for Question 11:


Correct Option C- Used in wounds in the posterior neck or on a concave surface:
• The interrupted horizontal and vertical mattress suture techniques are two of the most commonly used
skin closure methods. These suture techniques promote skin edge eversion.
• Eversion produced by the mattress sutures is valuable when closing sites with edges that tend to roll
under, such as on the posterior neck or in the groin.
• The vertical mattress suture is commonly used in body sites where the wound edges tend to invert,
such as the posterior neck or wounds that occur on a concave surface.
• Therefore, this option best fits the characteristics of a vertical mattress suture rather than a horizontal
mattress suture.

Page 12

1927
Horizontal Mattress
Vertical Mattress
Incorrect Options:
Option A- Corner stitch is a variant: A variation of the horizontal mattress suture, the half-buried horizo
ntal mattress suture, or corner stitch, is commonly used in closures performed in the office.
Option B- It is effective in holding fragile skin together: The horizontal mattress suture is an everting sut
ure technique that spreads tension along a wound edge. This suture effectively holds fragile skin toget
her, such as the skin of elderly patients or those receiving chronic steroid therapy.
Option D- Effective hemostatic sutures on vascular tissues such as the scalp: Because of the tissue co
ntained within the passage of the suture thread, horizontal mattress sutures are effective hemostatic su
tures on vascular tissues.

Page 13

1928
Solution for Question 12:
Correct Option D- 1 and 3 only:
• In the gastrointestinal tract, stapling devices tend to apply two rows of staples, offset in relation to
each other, to produce a sound anastomosis.
• A linear stapler is a stapling instrument used to create side-to-side anastomosis.
• The ends are closed by a transverse stapler.

Incorrect Options:
Options A, B, and C (1,2, 1 only & 1, 3, 4): Statements 2 and 4 represent the uses of circular staplers.
Circular stapling devices allow tubes to be joined together, and such instruments are commonly used i
n the esophagus and lower rectum. Low anterior resection and stapled hemorrhoidopexy are other use
s of circular staplers.

Page 14

1929
Page 15

1930
Shock
1. A 32-year-old female patient was admitted to the hospital with severe hemorrhagic shock due to
polytrauma with a hip fracture; she developed acidosis, coagulopathy, and hypothermia. Which of the
following is the first line of therapy for this patient of trauma?
(or)
Which of the following is the first line of therapy for traumatic hypovolemic shock?
A. Crystalloids
B. Colloids
C. Inotropes
D. Blood transfusion
----------------------------------------
2. Which of the following correctly represents the formula of the Modified Shock Index (MSI)?
(or)
A 26-year-old male presents to the emergency room after he sustained a road traffic accident. His
blood pressure is 88/76 mm Hg, and his pulse rate is 126 per minute. On examination, he appears pale
and somnolent. Which of the following correctly represents the formula of the Modified Shock Index
(MSI)?
A. Heart rate / systolic BP
B. Heart rate / diastolic BP
C. Heart rate / Mean arterial pressure
D. Pulse rate / Systolic BP
----------------------------------------
3. A 56-year-old male is admitted to the surgical ward after a Whipple’s procedure. On examination, the
patient is jaundiced; his Glasgow coma scale (GCS) is 5. Laboratory investigations revealed serum
bilirubin of 13 mg/dl and serum creatinine of 6 mg/dl. Which of the following is the exact 2016 criteria for
sepsis?
(or)
Which of the following is the exact 2016 criteria for sepsis?
A. Suspected (or documented) infection and an acute increase in ≥2 sepsis-related organ failure
assessment (SOFA) points
B. Suspected (or documented) infection plus vasopressor therapy needed to maintain mean arterial
pressure at ≥ 65 mm Hg & serum lactate > 2.0 mmol/L despite adequate fluid resuscitation
C. A life-threatening organ dysfunction caused by a dysregulated host response to infection
D. A subset of sepsis in which underlying circulatory and cellular metabolic abnormalities lead to
substantially increased mortality risk
----------------------------------------
4. An 85-year-old woman was brought to the emergency room by her son due to a penetrating stab
injury on the anterior neck. A kitchen knife was deeply stuck in the midline of the anterior neck, but
there was no active bleeding. The patient was semiconscious; bradycardia and hypotension were

1931
noted. Which of the following is the most likely reason?
(or)
Which type of shock exhibits bradycardia?
A. Hypovolemic shock
B. Distributive cshock
C. Hypovolemic + septicemic shock
D. Neurogenic shock
----------------------------------------
5. A 26-yr-old man without significant medical history and weighing around 100 kg sustained a severe
motorbike collision and was brought to the hospital in an ambulance. After complete evaluation and
examination, it was found that he lost approximately 30% of his total blood volume & blood pressure
was normal. He will be classified under which class of hypovolemic shock?
(or)
A 26-yr-old man who lost approximately 30% of his total blood volume after an accident with normal
blood pressure will be classified under which class of hypovolemic shock?
A. Class I
B. Class II
C. Class III
D. Class IV
----------------------------------------
6. During the treatment of septic shock, a 28-year-old male remains hypotensive despite adequate
volume replacement. When dopamine is started, ventricular tachycardia develops, and this is
unresponsive to lidocaine. The V-tach converts back to sinus rhythm once the dopamine is stopped. At
this point, which of the following treatment options is most appropriate for this hypotensive patient?
A. Amrinone
B. Dobutamine
C. Epinephrine
D. Phenylephrine
----------------------------------------

Correct Answers
Question Correct Answer

Question 1 1
Question 2 3
Question 3 1
Question 4 4
Question 5 2

Page 2

1932
Question 6 4

Solution for Question 1:


Correct Option A: Crystalloids
• The first line of therapy in shock in the patient of trauma is crystalloids.
• Most patients of shock, regardless of the cause, benefit from a trial of fluid resuscitation given in the
form of a crystalloid bolus.
• Resuscitation following blood loss in trauma patients begins with the administration of 1 liter of
isotonic crystalloid immediately to restore blood pressure and peripheral circulation.
• Lactated ringer (RL) solution is generally preferred over 0.9% NaCl (normal saline) as it is a balanced
salt solution designed to mimic extracellular fluid.
Option B: Colloids
• Resuscitation with colloids is no more effective than crystalloids but is also more expensive.
Option C: Inotropes
• Vasopressors and inotropes are drugs used to create vasoconstriction or increase cardiac
contractility, respectively, in patients for shock or any other reason for extremely low blood pressure.
• It is not used as the first line for management of hypovolemic shock.
Option D: Blood transfusion
• For actively bleeding patients, blood and blood products should remain the best therapy in
resuscitating bleeding patients, once it is arranged.

Solution for Question 2:


Correct Option C: Heart rate / Mean arterial pressure
• Modified shock index is heart rate divided by mean arterial pressure.
• High MSI indicates a value of stroke volume and low systemic vascular resistance, a sign of
hypodynamic circulation.
• Low MSI indicates hyperdynamic circulation.
• MSI is a better marker than stroke index for mortality rate prediction
Option A: Heart rate / systolic BP
• The heart rate divided by systolic BP is called the shock index.
• It is a better marker for assessing the severity of shock than heart rate & blood pressure alone.
Option B: Heart rate / diastolic BP
• Modified Stroke Index is heart rate divided by mean arterial pressure.
• Therefore, this option is incorrect.
Option D: Pulse rate / Systolic BP
• Modified Stroke Index is heart rate divided by mean arterial pressure.

Page 3

1933
• Therefore, this option is incorrect.

Solution for Question 3:


Option A: Suspected (or documented) infection and an acute increase in ≥2 sepsis-related organ failur
e assessment (SOFA) points.
• Sepsis-related organ failure assessment (SOFA) score is a 24-point measure of organ dysfunction
that uses six organ systems (renal, cardiovascular, pulmonary, hepatic, neurologic, hematologic) where
0-4 points are assigned per organ system.
• According to SOFA guidelines, suspected (or documented) infection and an acute increase in ≥ 2
sepsis-related organ failure assessment (SOFA) points are the criteria for sepsis.
Option B: Suspected (or documented) infection plus vasopressor therapy needed to maintain mean art
erial pressure at ≥ 65 mm Hg & serum lactate > 2.0 mmol/L despite adequate fluid resuscitation
• According to SOFA guidelines, suspected (or documented) infections plus vasopressor therapy
needed to maintain mean arterial pressure at ≥ 65 mm Hg & serum lactate > 2.0 mmol/L despite
adequate fluid resuscitation is the criteria for septic shock.
Option C: A life-threatening organ dysfunction caused by a dysregulated host response to infection
• Sepsis is a life-threatening organ dysfunction caused by a dysregulated host response to infection
• However, this is the definition of sepsis, not a criterion.
Option D: A subset of sepsis in which underlying circulatory and cellular metabolic abnormalities lead t
o substantially increased mortality risk.
• Septic shock is a subset of sepsis in which underlying circulatory and cellular metabolic abnormalities
lead to substantially increased mortality risk.
• However, it is the definition of septic shock, not a criterion for sepsis.

Solution for Question 4:


Correct Option D: Neurogenic shock
• Results from loss or suppression of sympathetic tone causing massive vasodilation in the venous
vasculature leading to decreased venous return and cardiac output, which in turn results in impaired
tissue perfusion & cellular metabolism.
• Clinical features Paralysis below the level of the lesion; hypotension Bradycardia due to loss of
sympathetic tone → Arterial & venous vasodilation → Warm & dry skin Hypothermia
• Paralysis below the level of the lesion; hypotension
• Bradycardia due to loss of sympathetic tone → Arterial & venous vasodilation → Warm & dry skin
• Hypothermia
• Paralysis below the level of the lesion; hypotension
• Bradycardia due to loss of sympathetic tone → Arterial & venous vasodilation → Warm & dry skin
• Hypothermia

Page 4

1934
Option A: Hypovolemic shock
• Hypovolemic shock results in tachycardia to compensate for cardiac output.
• Therefore, this option is incorrect.
Option B: Distributive
• Distributive shock is also called septic shock.
• In septic shock, there is a decrease in peripheral vascular resistance, which results in tachycardia and
increased cardiac output to maintain peripheral perfusion.
Option C: Hypovolemic + septicemic shock
• Hypovolemic & septicemia shock both result in tachycardia.
• Therefore, this option is incorrect.

Solution for Question 5:


Correct Option B: Class II
• The patient’s blood pressure is normal, and he has lost 30% of his total blood volume.
• According to ATLS (Advanced trauma life support) criteria, the patient is categorized as class II.
Option A: Class I
• According to ATLS criteria, in class I up to 15% of blood is lost, and blood pressure is normal.
Option C: Class III
• According to ATLS criteria, in class III up to 30%-40% of blood is lost, and blood pressure is
decreased.
Option D: Class IV
• According to ATLS criteria, in class IV >40% of blood is lost, and blood pressure is decreased.

Solution for Question 6:


Correct Option D: Phenylephrine
• Catecholamines are the vasopressors used most often for the treatment of septic shock, with
norepinephrine being the first line agent, followed by epinephrine.
• In this case, dopamine has caused ventricular tachycardia due to its action on the β1 receptor.
• The patient is hypotensive, and hence amrinone and dobutamine cannot be given as they both
decrease peripheral vascular resistance and cause a further decrease in blood pressure.
• Similar to dopamine, epinephrine is associated with severe arrhythmias, and this is due to increased
myocardial workload and oxygen demand; hence this also cannot be given.
• Phenylephrine is a selective α1agonist and hence is a potent vasoconstrictor. Phenylephrine hence
can be used in the treatment of septic shock in this case.

Page 5

1935
• Vasodilatory or septic shock is the result of dysfunction of the endothelium and the vasculature
secondary to circulating inflammatory mediators and cells or as a response to prolonged and severe
hypoperfusion. It is characterized by peripheral vasodilation with resultant hypotension and is resistant
to treatment with vasopressors.
• Dopamine stimulates α1 receptors (vasoconstriction), β1receptors (cardiac stimulation) and β2
receptors (vasodilation), with its effects on β receptors predominating at lower doses. Thus at normally
employed doses, it raises cardiac output and systolic blood pressure with little effect on diastolic blood
pressure. It may cause ventricular tachycardia, arrhythmia and increased peripheral resistance.
Other options
Option A: Amrinone
• Phosphodiesterase inhibitors like amrinone and milrinone are not adrenergic agonists, but they
behave similar to dobutamine and increase contractility but also promote vasodilation resulting in
hypotension and significant arrhythmia.
Option B: Dobutamine
• Dobutamine primarily stimulates cardiac β1 receptors to increase cardiac output but may also
vasodilate peripheral vascular beds, lower total peripheral resistance and lower systemic blood
pressure through effects on β2 receptors. Hence adequate preload and intravascular volume are
essential before administering dobutamine.
Option C: Epinephrine
• Similar to dopamine, epinephrine is associated with severe arrhythmias, and this is due to increased
myocardial workload and oxygen demand; hence this also cannot be given.

Page 6

1936
Tubes ,Catheters, Drains
1. A 64-year-old male is undergoing a hernia operation. On the table, the surgeon asks the nurse to
hand over an instrument to incise the skin. The nurse attaches the device to the power. What is the
name of the surgical instrument given below?

A. Harmonic scalpel
B. LigaSure
C. Monopolar diathermy
D. Bipolar diathermy
----------------------------------------
2. What is the name of this instrument shown in the image?

A. Ryle’s tube
B. Red rubber catheter
C. Sengstaken-Blakemore tube
D. Nelaton’s catheter
----------------------------------------
3. A 45-year-old female has undergone surgery. Before the closure, the surgeon implants a tube in the
abdomen to drain the fluid. The picture of the tube is given below. Which of the following procedure was
most likely performed on this patient?
(or)

1937
After the surgery of a female patient, the surgeon implants a tube in the abdomen to drain the fluid. A
picture of the tube is given. What procedure is most likely performed?

A. Gastric decompression
B. Ascitic Tap
C. Post choledochotomy
D. Tension pneumothorax
----------------------------------------
4. During a hernia operation of a 68-year-old male, the surgeon asked the nurse to hand over the
diathermy for the procedure, which shows the following waveform. What option best correlates with this
waveform?
(or)
A 68-year-old male is undergoing a hernia operation. On the table, the surgeon asked the nurse to
hand over the diathermy for the procedure. The machine attached to the diathermy shows the
waveform given in the picture below. Which of the following option best correlates with this waveform?

A. Fulguration
B. Cutting
C. Coagulation
D. Blended waveform
----------------------------------------
5. Doctors performed intercostal drainage using the instrument shown below. What is the name of the
catheter used?

Page 2

1938
A. Red rubber catheter
B. Foley's catheter
C. Malecot Catheter
D. Fogarty Catheter
----------------------------------------
6. A 46-year-old male is diagnosed with multinodular goiter and is scheduled for thyroidectomy. The
surgeon uses the following instrument during the procedure to hold and ligate the superior thyroid
pedicle. What is the name of the given instrument?
(or)
The following instrument is used to hold and ligate the superior thyroid pedicle. What is the name of the
given instrument?

A. Kocher’s thyroid dissector


B. Doyen’s Rib Raspatory
C. Joll’s thyroid Retractor
D. Deaver’s Retractor
----------------------------------------
7. A 5-year-old boy presents with a lesion in his oral cavity, which looks like a frog’s belly. The surgeon
employs the use of the following instrument. What is the name of this instrument?
(or)
A 5-year-old boy presents to the clinic complaining of a lesion in his oral cavity, which looks like a frog’s
belly. He is scheduled for elective surgery. During the surgery, the surgeon employs the following

Page 3

1939
instrument. What is the name of this instrument?

A. Joll’s thyroid retractor


B. Morris retractor
C. Doyen’s mouth gag
D. Beckman Weitlaner retractor
----------------------------------------
8. A 26-year-old male patient presents to the radiology department for contrast-enhanced computed
tomography. The patient has an instrument attached to his body which is shown below. What is the
name of this instrument?

A. Dialysis port
B. Chemoport
C. Electroconvulsive therapy lead
D. Gamma camera
----------------------------------------
9. The following retractor is used during surgery to push the liver away from the surgical field. What is
the name of this retractor?

Page 4

1940
A. Deaver retractor
B. Morris retractor
C. Langenbeck retractor
D. Doyen retractor
----------------------------------------
10. What is the name of the instrument shown?
(or)
A 36-year-old female is scheduled for laparoscopic cholecystectomy. During the procedure, which
instrument is used to create a pneumoperitoneum?

A. Hasson’s cannula
B. GB trocar
C. Veress needle
D. Central venous catheter
----------------------------------------
11. Which of the following statements regarding the tube shown in the picture is most appropriate?
(or)
Which of the following statements regarding the tube shown in the picture is most appropriate?

Page 5

1941
A. Contraindicated in TEF
B. Contraindicated in upper GI bleed
C. Indicated in perforation peritonitis
D. Used for parenteral nutrition
----------------------------------------
12. A female patient is scheduled for cholecystectomy. The surgeon uses an instrument to hold skin
and subcutaneous tissue for retraction. What is the name of this instrument?

A. Allis tissue forceps


B. Kocher’s forceps
C. Babcock’s tissue forceps
D. Lahey’s forceps
----------------------------------------
13. Match the following images with their names appropriately. 1. a. Chemoport 2. b. Nasogastric tube
3. c. Malecot catheter 4. d. Foley catheter

Page 6

1942
1. a. Chemoport

2. b. Nasogastric tube

3. c. Malecot catheter

4. d. Foley catheter

A. 1- D, 2-A, 3-B, 4-C


B. 1-C, 2-D, 3-B, 4-A

Page 7

1943
C. 1-A, 2-B, 3-C, 4-D
D. 1-B, 2-D, 3-C, 4-A
----------------------------------------

Correct Answers
Question Correct Answer

Question 1 3
Question 2 3
Question 3 3
Question 4 2
Question 5 3
Question 6 1
Question 7 3
Question 8 2
Question 9 1
Question 10 3
Question 11 3
Question 12 1
Question 13 2

Solution for Question 1:


Correct Option C - Monopolar diathermy:
• Diathermy uses an alternate high-frequency polarity radio-wave electrical current to cut or coagulate
tissue during surgery.
• It is now utilized in almost all surgical specialties because it creates precise incisions with little blood
loss. Electrical current at extremely high frequencies (about 0.5–3 MHz) is used in diathermy.
• To facilitate cutting and coagulation, the radio frequencies produced by diathermy heat the tissue by
causing intracellular oscillation of molecules within the cells.
• Various outcomes depend on the temperature attained: at 60º C, cell death (fulguration); between 60
and 99º C, tissue coagulation; and at about 100º C, tissue vaporization (cutting).
• The configuration of the diathermy devices may be monopolar or bipolar; both operations necessitate
the completion of the electrical circuit but do so in different ways.
• In a monopolar action, the electrical current oscillates between the surgeon's electrode and the
patient's body before it finally connects with the "grounding plate" (usually located beneath the patient's
leg) to complete the circuit.
• Different electrosurgical waveforms with various tissue effects are available with this electrosurgery.
• A pencil-like electrode is used in a monopolar electrosurgical operation to cut the target tissue and
stop bleeding.

Page 8

1944
Incorrect Options:
Option A - Harmonic scalpel:
• A harmonic scalpel is a surgical tool that cuts and cauterizes tissue simultaneously. It is the most
recent gadget to be released in the previous ten or so years.
• The Harmonic Scalpel is well known for using ultrasonic energy, which is transformed into mechanical
energy at the active blade.
• The active blade, which generates high-grade frictional force, is the primary mechanism, whereas the
inactive upper arm maintains tissue in apposition.

Option B - LigaSure:
• A bipolar electrosurgical tool powered by radiofrequency is called LigaSure (bipolar vascular sealing)
technology.

Page 9

1945
Option D - Bipolar diathermy:
• The two electrodes in bipolar diathermy are located on the device itself. A dispersive electrode is not
required in a bipolar setup; a pair of comparable sizes is employed in tandem. After that, the current is
moved between the electrodes.
• The most frequent uses of bipolar are in microsurgery, procedures on the digits (to prevent monopolar
current from being focussed over a smaller area), and patients who have pacemakers (to avoid
electrical interference with the pacemaker).

Solution for Question 2:


Correct Option C - Sengstaken-Blakemore tube:
• The image shows the Sengstaken-Blakemore (SB) tube.
• The Sengstaken-Blakemore (SB) tube is used to halt or delay esophageal and stomach bleeding.
• The bleeding is usually caused by stomach or esophageal varices and enlarged veins produced by
restricted blood flow.
• The SB tube includes three ports at one end, each with a different function: esophageal balloon port,
which inflates a tiny balloon in the esophagus; gastric aspiration port, which takes fluid and air from the
stomach; gastric balloon port, which inflates a balloon in the stomach.
• Two balloons are attached to the other end of the SB tube. These balloons are inflated, applying
pressure to the bleeding sites to stop bleeding
• To reach the stomach, the tube is usually placed by the mouth, but it can also be inserted through the
nose. Once the bleeding has stopped, it can be removed.
• The use of the SB tube comes with some dangers. The treatment may cause discomfort, especially if
the tube is put in through the mouth, which could cause a sore throat.
• The SB tube's placement could impair the ability to breathe.

Page 10

1946
Incorrect Options:
Option A - Ryle’s tube:
• Ryle’s (Nasogastric) tube is a single-use polyvinyl chloride tube that can be used for treatment and
diagnosis.
• It is used to aspirate gastric contents in hematemesis, intestinal obstruction, post-surgery, and gastric
acid aspiration for gastric function tests.
• It is used for barium examinations in the radiology division. For giardiasis diagnosis, it may pass in the
duodenum.

Page 11

1947
Option B - Red rubber catheter:
• Red rubber catheters are a form of intermittent catheter used to treat urine retention.
• They work similarly to straight catheters, except they are constructed of red rubber latex rather than
plastic.
• It makes the red rubber catheter more flexible, which some individuals prefers.

Option D - Nelaton’s catheter:


• Nelaton catheters are straight tube-like catheters with one hole at the tip and a connector at the other
end for drainage.
• The proximal end has a connector in the shape of a funnel for a secure connection to the urine bag.
• These single-use male Nelaton catheters are designed for men who prefer the straight Nelaton tip and
are intended for intermittent catheterization.

Page 12

1948
Solution for Question 3:
Correct Option C - Post choledochotomy:
• A Kehr’s tube is a flexible latex tube with vertical and short horizontal limbs.
• T-tube is a drainage tube put in the common bile duct following CBD exploration with supra-duodenal
choledochotomy.
• It allows for the external outflow of bile into a regulated channel while the choledochotomy healing
process matures and the initial disease resolves.
• The term T-tub refers to the form of the CBD drainage tube.
• T-tubes can also treat chronic duodenal fistulas, pancreaticoduodenectomies, and
hepaticojejunostomies after a liver transplant.
• T-tubes are not the same as tracheal tubes or tympanoplasty tubes.
• It permits bile to leak into the duodenum until the sphincter of Oddi spasm is alleviated.
• It is retained for 7-10 days, after which there is a free flow of bile, lowering the risk of biliary peritonitis.
Incorrect Options:
Option A - Gastric decompression:
• Gastric decompression is performed using a nasogastric tube in patients with known or suspected
stomach distension.
• The gastroenteric-decompression tube is introduced through the nose and advanced past the
stomach into the intestine.
• It is used to aspirate the intestine's contents for analysis and treat intestinal blockage.
• The tube may also prevent nausea, vomiting, and abdominal distention following GI surgery.
Option B - Ascitic Tap:

Page 13

1949
• An ascitic tap is a medical treatment that uses a needle to drain fluid from an internal bodily cavity,
most often the abdomen.
• A 20 cc or 60 cc syringe to take a fluid sample and a 14 or 16 gauge needle or IV catheter for fluid
aspiration are required.
• Obese individuals may also need a spinal needle.
Option D - Tension Pneumothorax:
• Needle decompression is a treatment method for people who suffer from tension pneumothorax.
When air from the lungs leaks into the pleural space(the area between the lungs and the chest), this is
known as tension pneumothorax.
• The optimal insertion site is the second intercostal space along the mid-clavicular line in the affected
hemithorax.

Solution for Question 4:


Correct Option B - Cutting:
• Diathermy uses a high-frequency alternating polarity radio-wave electrical current during surgery to
cut or coagulate tissue.
• It enables accurate incisions with minimal blood loss and is utilized in practically all surgical fields.
• The radio-frequencies created by diathermy heat the tissue by causing internal oscillation of
molecules within the cells, allowing for cutting and coagulation.
• Depending on the temperature, several outcomes occur: at 60º C, cell death (fulguration) happens;
between 60-99º C, dehydration occurs, and tissues coagulate; and at 100º C, tissues vaporize (cutting).
• Diathermy has two primary settings: cutting and coagulation.
• Cutting uses a low-voltage continuous waveform. In cutting mode, the electrode achieves a high
enough power level to vaporize the water content. As a result, it can make a clean cut but is less
effective in coagulating.
• The cutting mode focuses heat at the surgical site, with sparks being the more focused means to
disperse heat; consequently, the cutting mode should be utilized with the tip away from the tissue.
Incorrect Options:
Option A - Fulguration:
• Fulguration is an electrosurgical procedure. Healthcare personnel employ electrosurgery equipment
to guide high-frequency electrical currents from an electrosurgical generator to electrodes on the skin.
• In fulguration, healthcare personnel ignites the instrument and the electrodes on the skin.
• Fistulas and persistent cysts can be cauterized, and small hemorrhages can be halted using
fulguration.
Option C - Coagulation:
• Coagulation can also be accomplished using a pulsed waveform with a high voltage. The waveform
has a lower average power in coagulation, creating enough heat for thermal coagulation but not enough
for explosive vaporization.

Page 14

1950
• Although the tip should be held somewhat away from the tissue, the sparks are dispersed over a
larger region, producing charring rather than cutting.
Option D - Blended waveform:
• The 'Blend' approach combines short-wave diathermy and galvanic current into a single therapy.
• This short wave diathermy generates heat, while the galvanic generates a chemical reaction at the
hair's root due to water, salts, and a low current mixing.
• It has both cutting and coagulation advantages.

Solution for Question 5:


Option C: Malecot Catheter

Page 15

1951
• It is a reusable, self-retaining, radio-opaque India rubber catheter with a flower at the tip.
• The flower aids in anchoring the catheter and guards against unintentional removal.
• It is cleaned in cold running water with detergent before being sterilized by boiling, autoclaving,
chemical disinfection, or gas.
Option A: Red rubber Catheter
• Red rubber catheters are a form of intermittent catheter used to treat urine retention.
• They work similarly to straight catheters, except they are constructed of red rubber latex rather than
plastic.
• It makes the red rubber catheter more flexible.

Option B: Foley's catheter


• A Foley catheter is a type of indwelling catheter that is commonly used.
• A soft, plastic, or rubber tube is placed into the bladder to drain the urine.
• The majority of males utilize 14 Fr catheters.
• Adult women often utilize catheter sizes ranging from 10 Fr to 12 Fr.

Page 16

1952
Option D: Fogarty Catheter
• The Fogarty embolectomy catheter has been demonstrated to reduce blood loss during big surgical
operations
• Its value stems from its capacity to achieve temporary intraluminal blockage of the common iliac artery
during the proposed surgical operation.

Page 17

1953
Solution for Question 6:
Correct Option A - Kocher’s thyroid dissector:

• The above-given image shows Kocher’s thyroid dissector.


• During a thyroidectomy, Kocher's thyroid dissector is used to secure and ligate the superior pedicle of
the thyroid, which contains the superior thyroid artery.
• Its blunt tip, which has an eye, is inserted beneath the superior thyroid pedicle, and a suture, similar to
an aneurysm forceps, is threaded through the eye.
• Three sutures are then ligated close to the gland.
Incorrect Options:

Page 18

1954
Option B - Doyen’s Rib Raspatory:

• The Doyen’s rib raspatory is a specialist cardiothoracic device surgeons use to scrape bone and other
hard tissues of the ribs to smooth the edges of the chosen bone and expand the surgical approach.
Option C - Joll’s thyroid Retractor:

• The Joll’s self-retaining retractor is a specialty surgical tool for challenging incision sites.
• It is commonly used for thyroid and vaginal procedures due to its unusual form, which allows it to
access these anatomical locations more effectively.
Option D - Deaver’s Retractor:

Page 19

1955
• The Deaver Retractor is a large, portable retractor frequently used to hold the abdominal wall back
during abdominal or thoracic surgeries.
• It can also transfer or restrain organs from the surgical site.

Solution for Question 7:


Option C: Doyen’s mouth gag

• The patient has an intra-oral ranula and is being planned for marsupialization.
• The above-given image shows a Doyen’s mouth gag.
• It has blades that are semi-circular and curved at the tip.

Page 20

1956
• It is used to keep the mouth open during intra-oral surgeries like Glossectomy Cleft palate Intra-oral
ranula To acquire a biopsy from a suspicious lesion
• Glossectomy
• Cleft palate
• Intra-oral ranula
• To acquire a biopsy from a suspicious lesion
• Glossectomy
• Cleft palate
• Intra-oral ranula
• To acquire a biopsy from a suspicious lesion
To acquire a biopsy from a suspicious lesion
Option A: Joll’s thyroid retractor

• The Joll’s self-retaining retractor is a speciality surgical tool used for challenging incision sites.
• It is commonly used for thyroid and vaginal procedures due to its unusual form, which allows it to
access these anatomical locations more effectively.
Option B: Morris retractor
• The Morris Retractor is used to retract the abdominal wall, loin, and subcostal incisions.
• It can fit a wide range of wounds at any depth.
• The handle has an oval ring that provides an improved grip for an assistant's fingertips.
Option D: Beckman Weitlaner retractor

Page 21

1957
• The Beckman Weitlaner retractor is a surgical instrument used in veterinary surgery to retract tissues
and skin.
• It is most typically utilized in spinal and intracranial procedures.
• The instrument is equipped with finger ring grips and a self-locking mechanism.

Solution for Question 8:


Correct Option B - Chemo port:

Page 22

1958
• The instrument shown above is a chemo port.
• It is a small, implantable reservoir with a thin silicone tube attached to a vein.
• Its advantages include: It eliminates the need for needle sticks. It reduces the risk of medications
coming into contact with the skin It can be used for several days
• It eliminates the need for needle sticks.
• It reduces the risk of medications coming into contact with the skin
• It can be used for several days
• It eliminates the need for needle sticks.
• It reduces the risk of medications coming into contact with the skin
• It can be used for several days
Incorrect Options:
Option A - Dialysis port:

Page 23

1959
• A dialysis port is a catheter to exchange blood between a hemodialysis machine and a patient.
• The dialysis catheter has two lumens, one venous and one arterial. Even though both lumens are in
the vein, the "arterial" lumen, like natural arteries, transports blood away from the heart, while the
"venous" lumen returns blood to the heart.
• The arterial lumen (usually red) collects blood from the patient. It transports it to the dialysis machine,
whereas the venous lumen (typically blue) returns blood to the patient (from the dialysis machine).
Dialysis catheter flow rates range between 200 and 500 ml/min.
Option C - Electroconvulsive therapy lead:

• The instrument shown above is an electroconvulsive therapy lead.


• Electroconvulsive therapy (ECT) induces a widespread brain seizure in a patient under intravenous
sedation or general anesthesia.

Page 24

1960
• Although it is generally used to treat severe depression, it may also aid individuals with schizophrenia,
schizoaffective disorder, catatonia, neuroleptic malignant syndrome, and bipolar disorder.
• However, the procedure has a negative connotation due to misconceptions about procedural
methods.
Option D - Gamma camera:

• A gamma camera, also known as a scintillation camera or Anger camera, is used to picture gamma
radiation-generating radioisotopes using the scintigraphy technique.
• Early drug development and nuclear medical imaging use scintigraphy to see and analyze human
body pictures or the distribution of medically injected, inhaled, or ingested radionuclides generating
gamma rays.

Solution for Question 9:


Option A: Deaver retractor
• The instrument shown above is a Deaver’s retractor.
• It is specifically designed to hold the liver up out of the way during a cholecystectomy.
• It needs to be used carefully to avoid damaging the liver.
• Some surgeons protect the liver with an abdominal pack before placing the blade on top.
Option B: Morris retractor
• Subcostal wounds, abdominal wall incisions, and loin incisions can all be retracted using the Morris
Retractor.
• It can be used on wounds of any depth and a wide variety.
• An oval ring fenestrated in the handle allows for easier finger traction by the assistant.
Option C: Langenbeck retractor

Page 25

1961
• During various general surgeries, the Langenbeck Retractor is a common surgical tool that enables
surgeons to pull back soft tissues, incisions, or wound edges.
• It has a flexible L-shaped tip for pulling back wide tissue slices and a curve that descends at the end
to safeguard nearby structures.
Option D: Doyen retractor
• The Doyen Retractor is a handheld retractor primarily used in obstetric procedures such as abdominal
hysterectomies, caesarean section deliveries, and ectopic pregnancy procedures.
• The blade on this retractor is 90 degrees angled, blunt, and 1-3/4" wide.

Solution for Question 10:


Correct Option C - Veress Needle:

• A Veress needle, or Veres needle, is a spring-loaded needle used in laparoscopic surgery to create
pneumoperitoneum.
• The Veress needle technique is the oldest and most traditional of the three general approaches to
laparoscopic access.
• Modern needles range in length from 12 to 15 cm and have an external diameter of 2 mm.
• The outer cannula has a beveled needlepoint for cutting through abdominal wall tissues. Within the
outer cannula is a spring-loaded inner stylet. The blunt tip of this inner stylet protects any viscera from
injury by the sharp outer cannula. The direct pressure on the tip, such as when piercing tissue, forces
the blunt stylet into the shaft of the outer cannula. The blunt inner stylet springs forward when the
needle tip enters a space, such as the peritoneal cavity. The Veress needle is then used to inflate the
space, resulting in a pneumoperitoneum.
Incorrect Options:
Option A - Hasson’s Cannula:

Page 26

1962
• Hasson’s cannula was a blunt (open) mini-laparotomy access to reduce the incidence of injuries
associated with the blind access of the peritoneal cavity with the Veress needle and the initial trocar.
• It is a reusable device that looks like a standard cannula but has an olive-shaped cone attached to it
(sleeve).
• Several disposable open-access devices have been released.
• They are similar to Hasson's reusable system, except the stay sutures are attached directly to the
olive, allowing cannula depth manipulation without detaching the stay sutures.
• However, the primary method of peritoneal access has stayed the same.

Option B: GB trocar
• The Ochsner gallbladder trocar is a 100 mm long instrument in various circumferences.
• The primary purpose of this trocar is to drain a fluid-filled cavity using a sharp point.
• For example, if there is pus or mucus in the gallbladder, this instrument can remove it.
Option D - Central venous Catheter:
• A central venous catheter is a thin, flexible tube inserted into a vein, usually below the right
collarbone, and guided (threaded) into the superior vena cava, a large vein above the right side of the
heart.
• It is employed in administering intravenous fluids, blood transfusions, chemotherapy, and other drugs.

Solution for Question 11:


Option C: Indicated in perforation peritonitis

• The tube above is a Ryle's or nasogastric (NG) tube.

Page 27

1963
• It is indicated in the following conditions: Diagnostic Indications Saline load test in gastric outlet
obstruction To access free and total acid in peptic ulcer patient To diagnose trachea-esophageal fistula
(TEF) Hollander’s test to check for completion of vagotomy. Therapeutic indications
Decompression: Intestinal or gastric outlet obstruction Perforation peritonitis Upper gastrointestinal
bleeding Upper abdominal surgery Nutrition: Enteral Nutrition
• Diagnostic Indications Saline load test in gastric outlet obstruction To access free and total acid in
peptic ulcer patient To diagnose trachea-esophageal fistula (TEF) Hollander’s test to check for
completion of vagotomy.
• Saline load test in gastric outlet obstruction
• To access free and total acid in peptic ulcer patient
• To diagnose trachea-esophageal fistula (TEF)
• Hollander’s test to check for completion of vagotomy.
• Therapeutic indications Decompression: Intestinal or gastric outlet obstruction Perforation peritonitis
Upper gastrointestinal bleeding Upper abdominal surgery Nutrition: Enteral Nutrition
• Decompression: Intestinal or gastric outlet obstruction
• Perforation peritonitis
• Upper gastrointestinal bleeding
• Upper abdominal surgery
• Nutrition: Enteral Nutrition
• Diagnostic Indications Saline load test in gastric outlet obstruction To access free and total acid in
peptic ulcer patient To diagnose trachea-esophageal fistula (TEF) Hollander’s test to check for
completion of vagotomy.
• Saline load test in gastric outlet obstruction
• To access free and total acid in peptic ulcer patient
• To diagnose trachea-esophageal fistula (TEF)
• Hollander’s test to check for completion of vagotomy.
• Therapeutic indications Decompression: Intestinal or gastric outlet obstruction Perforation peritonitis
Upper gastrointestinal bleeding Upper abdominal surgery Nutrition: Enteral Nutrition
• Decompression: Intestinal or gastric outlet obstruction
• Perforation peritonitis
• Upper gastrointestinal bleeding
• Upper abdominal surgery
• Nutrition: Enteral Nutrition
• Saline load test in gastric outlet obstruction
• To access free and total acid in peptic ulcer patient
• To diagnose trachea-esophageal fistula (TEF)
• Hollander’s test to check for completion of vagotomy.
• Decompression: Intestinal or gastric outlet obstruction
• Perforation peritonitis

Page 28

1964
• Upper gastrointestinal bleeding
• Upper abdominal surgery
• Nutrition: Enteral Nutrition
Option A: Contraindicated in TEF
• It is used for the diagnosis of a trachea-esophageal fistula (TEF)
Option B: Contraindicated in upper GI bleed
• It is a therapeutic intervention to decompress and perform gastric lavage in case of upper
gastrointestinal bleeding.
Option D: Used for parenteral nutrition
• It is used for enteral nutrition.
• Examples of parenteral nutrition include partial parenteral nutrition (PPN) and total parenteral nutrition
(TPN)

Solution for Question 12:


Option D: Allis tissue forceps

• Allis tissue forceps have a tip which has alternate teeth or grooves
• Its uses include Holding skin and subcutaneous tissue flap for retraction Holding tough tissue Holding
skin during suturing Hold cut margins of the skin
• Holding skin and subcutaneous tissue flap for retraction
• Holding tough tissue
• Holding skin during suturing
• Hold cut margins of the skin
• Holding skin and subcutaneous tissue flap for retraction

Page 29

1965
• Holding tough tissue
• Holding skin during suturing
• Hold cut margins of the skin
Option A: Lahey’s forceps

• Lahey’s forceps are also known as right-angled forceps and mixter artery forceps.
• The terminal part of the blade has a right angle, and it has transverse serrations.
• It is used while dissecting the pedicle of solid organs and dissecting and passing ligature to the cystic
duct and cystic artery.
Option B: Kocher’s forceps

• Kocher’s forceps have teeth and a groove at the tip.


• They are used to Crush at the base of the appendix Hold perforating vessels in mastectomy Catch the
bleeder in surgeries of the palm and sole

Page 30

1966
• Crush at the base of the appendix
• Hold perforating vessels in mastectomy
• Catch the bleeder in surgeries of the palm and sole
Kocher’s forceps have teeth and a groove at the tip.
They are used to
• Crush at the base of the appendix
• Hold perforating vessels in mastectomy
• Catch the bleeder in surgeries of the palm and sole
Crush at the base of the appendix
Hold perforating vessels in mastectomy
Catch the bleeder in surgeries of the palm and sole
Option C: Babcock’s tissue forceps

• Babcock’s tissue forceps have curved tips with fenestrations.


• They are used in the following Appendectomy Gastrectomy Fallopian tube Ureter
• Appendectomy
• Gastrectomy
• Fallopian tube
• Ureter
• Appendectomy
• Gastrectomy
• Fallopian tube
• Ureter

Page 31

1967
Solution for Question 13:
Correct Option B 1-C, 2-D, 3-B, 4-A:
• Malecot catheter: It is a reusable, self-retaining, radio-opaque catheter made of India rubber with a
flower at the tip. The flower helps keep the catheter in place and prevents accidental removal. Uses:
Drainage of urine from bladder and kidneys, drainage of intraperitoneal collections, and as an
intercostal drain.
• Uses: Drainage of urine from bladder and kidneys, drainage of intraperitoneal collections, and as an
intercostal drain.
• Foley catheter: It is a common type of indwelling catheter made of latex. It is a self-retaining balloon
catheter. Uses: To measure urinary output, relieve urinary retention, and administer intravesicle
chemotherapy.
• Uses: To measure urinary output, relieve urinary retention, and administer intravesicle chemotherapy.
• Nasogastric tube: NG tubes are part of the standard of care in treating intestinal obstruction and can
also be used to provide nutritional support. They are most common in surgical patients but are useful in
any patient population where gastric decompression or nutritional support is necessary.
• Chemoport: It is a totally implantable venous access device used for chemotherapy administration.
• Uses: Drainage of urine from bladder and kidneys, drainage of intraperitoneal collections, and as an
intercostal drain.
• Uses: To measure urinary output, relieve urinary retention, and administer intravesicle chemotherapy.

Page 32

1968
Previous Year Questions
1. A patient presented with blunt trauma to the abdomen. On evaluation, liver injury was noted, for
which primary repair was done. Coagulation function was monitored intraoperatively, using the method
shown below. What is the method used?

A. Thromboeslastography
B. Plethysmography
C. Sonography
D. Elastography
----------------------------------------
2. A child with a prior incident of being stabbed in the front part of the abdomen has provided a visual
representation of the injury. The child's vital signs are stable. What will be the subsequent step in
managing this case?
A. Emergency laparotomy
B. Observation
C. Intravenous hydration
D. Wait and watch
----------------------------------------
3. What is the probable location of urethral damage in a young male patient who has a past medical
record of being involved in a motor vehicle accident and is currently experiencing difficulty in urinating,
along with the presence of blood at the opening of the urethra?

A. Bulbar urethra

1969
B. Spongy urethra
C. Membranous urethra
D. Penile urethra
----------------------------------------
4. Please determine the composition of the fluid displayed below: - Sodium (Na+): 131 millimoles per
liter - Chloride (Cl-): 111 millimoles per liter - Lactate: 29 millimoles per liter - Potassium (K+): 5
millimoles per liter - Calcium (Ca2+): 2 millimoles per liter - Total osmolarity: 279 milliosmoles per liter
A. Haemaccel
B. Ringer lactate
C. Isolyte
D. Isolyte M
----------------------------------------
5. The image below shows a pressure sore. Which stage does this belong to?

A. Stage 1
B. Stage 2
C. Stage 3
D. Stage 4
----------------------------------------
6. According to triage, which of the following categories of patients comes under green?
(or)
Based on triage, which group of patients is classified as green?
A. Ambulatory patients
B. Medium risk patients
C. High-risk patients
D. Dead patients
----------------------------------------
7. What would be the subsequent course of action for this injury depicted in the image after a three-day
period following a Road Traffic Accident (RTA)?

Page 2

1970
A. Debridement of necrotic tissue with primary closure
B. Debridement of necrotic tissue and hyperbaric oxygen
C. Amputation
D. Clean and close with rotation flap
----------------------------------------
8. What is the likely reason behind the occurrence of surgical emphysema in the patient following an
emergency tracheostomy performed outside of the hospital?
A. Puncture of posterior wall
B. High tracheostomy
C. Blockage of the tube
D. Tight suture
----------------------------------------
9. What is not a component of splenic trauma based on AAST grading?
(or)
Which of the following is not included in the AAST grading system for splenic trauma?
A. Parenchymal destruction
B. Vascular involvement
C. Adjacent organ involvement
D. Hematoma
----------------------------------------
10. What guidance would you provide to the female patient prior to undergoing laparoscopic surgery?
A. Eat full before surgery.
B. Drink plenty of water before surgery.
C. Enema on the night before surgery
D. NPO for specific hours before surgery
----------------------------------------
11. Which of the following tests is not conducted before a blood transfusion for a person with RTA,
when a family member is volunteering for blood donation?

Page 3

1971
A. Hepatitis A
B. Hepatitis B
C. Hepatitis C
D. HIV 1 & 2
----------------------------------------
12. What would be the most appropriate next course of action in managing a 60-year-old patient who is
experiencing painless blood in urine and has been diagnosed with bladder cancer that has affected the
muscle layer?
A. Intravesical administration of BCG
B. Radiotherapy
C. Neoadjuvant chemotherapy with Mitomycin C
D. Radical cystectomy
----------------------------------------
13. Which of the following is seen in all kinds of shock?
(or)
In all types of shock, which of the following is observed?
A. Reduced tissue perfusion
B. Increased peripheral resistance
C. Decreased respiratory rate
D. Cold clammy skin
----------------------------------------
14. Please determine the instrument provided.

A. Linear stapler
B. Skin stapler
C. Circular stapler
D. Curved stapler
----------------------------------------

Page 4

1972
15. What type of graft is used when a patient undergoes skin grafting following a road traffic accident in
which the graft is harvested from the patient's own body?

A. Autograft
B. Xenograft
C. Allograft
D. Isograft
----------------------------------------
16. A male patient presented with a sudden onset tearing type of chest pain radiating to the back,
shortness of breath, and nausea. CT chest image is given. What is the most appropriate next step in
the management of this patient?
(or)
What is the most suitable course of action to be taken in the management of a male patient who
reported sudden chest pain that feels like tearing, which extends to the back, accompanied by difficulty
breathing and nausea, after reviewing the CT scan of his chest?

A. Labetalol and urgent surgery


B. Urgent surgery
C. Wait and watch
D. Blood transfusion
----------------------------------------
17. What is the probable diagnosis for a 30-year-old woman who experienced nausea and vomiting
after being involved in a road traffic accident, and was brought to the emergency room unconscious,
based on the findings of the NCCT brain?

Page 5

1973
A. Extradural hemorrhage
B. Subdural hemorrhage
C. Intra-parenchymal hemorrhage
D. Subarachnoid hemorrhage
----------------------------------------
18. A soldier arrives at a referral hospital 24 hours after experiencing a shrapnel injury to his right thigh
caused by a high-speed projectile. He expresses significant discomfort at the site of the wound. Upon
examination, there is evidence of crepitus. Which of the following microorganisms is the probable
culprit behind his current state?
A. Clostridium septicum
B. Clostridium difficile
C. Clostridium novyi
D. Clostridium perfringens
----------------------------------------
19. Among the options provided, which pairing correctly matches the trauma patients brought to the
casualty?
A. Extradural hemorrhage (EDH)- Pin point pupil
B. Penetrating injury to eustachian tube- CSF otorrhea
C. Penetrating injury to eye- Battle sign
D. Gaze palseis-Mid brain lesion
----------------------------------------
20. Which of the following is not included in the qSOFA score components?
A. Respiratory rate >22
B. LDH-Lactate dehydrogenase
C. Altered mental status
D. Systolic blood pressure < 100 mmHg O
----------------------------------------
21. Identify the procedure being performed in the image given below.

Page 6

1974
A. Intraosseous cannula for pain relief
B. Bone marrow aspiration
C. Intraosseous route access for giving IV fluids
D. Bone marrow biopsy
----------------------------------------
22. What is the Glasgow Coma Scale (GCS) score of a patient who was admitted to the emergency
room after being involved in a road traffic accident, and on examination, opens his eyes in response to
a painful stimulus, speaks inappropriately, and withdraws his limbs in response to a painful stimulus?
A. E2V2M3
B. E3V3M3
C. E2V3M4
D. E3V2M2
----------------------------------------
23. A young patient is admitted to the hospital due to a 5-day history of fever. The patient's blood
pressure is 90/60 mmHg, heart rate is 120 beats per minute, respiratory rate is 24 breaths per minute,
and their Glasgow Coma Scale (GCS) score is 10. Laboratory tests reveal increased white blood cell
count with an elevated proportion of neutrophils, as well as a serum creatinine level of 2.6 mg/dL. An
intern doctor intends to calculate the quick Sequential Organ Failure Assessment (qSOFA) score to
predict the patient's prognosis and duration of hospitalization. Which of the following components are
included in the qSOFA score?
A. BP, RR and CBC
B. Creatinine, PR, BP
C. RR, Body temperature, PR
D. SBP, RR, GCS score
----------------------------------------
24. What are the associations of neurogenic shock?
A. Hypertension + tachycardia
B. Hypotension + tachycardia
C. Hypotension + bradycardia
D. Hypertension + bradycardia

Page 7

1975
----------------------------------------
25. Which of the following is NOT a criteria for SIRS?
A. Systolic blood pressure <90 mmHg
B. Temperature >38 degree Celsius or <36 degree Celsius
C. Respiratory rate >20 bpm
D. Heart rate >90/ min
----------------------------------------
26. Hand rub can be utilized in accordance with the standard hand hygiene protocol in all locations
listed below, with the exception of...
A. If the hands are visibly soiled
B. While moving from a contaminated site to a clean site during patient care
C. Before donning on gloves
D. During direct patient contact
----------------------------------------
27. In the case of a patient arriving at the emergency department with a road traffic accident (RTA),
what is the appropriate order of steps in their management? Cervical spine stabilization Intubation IV
cannulation CECT
A. 1,3,2,4
B. 1,2,3,4
C. 2,1,3,4
D. 2,1,4,3
----------------------------------------
28. Which of the following options does not represent one of the boundaries of the triangle of safety for
the insertion of an ICD in a patient with a chest x-ray showing pneumothorax?
A. Lateral edge of pectoralis major
B. Mid – clavicular line
C. Base of axilla
D. Lateral border of latissimus dorsi
----------------------------------------
29. A 30-year-old male patient presents with a pelvic fracture following blunt abdominal trauma. No
blood is seen at the meatus. Catheterization does not produce significant urine output. The radiological
image is given below. What is the next step in management ?

Page 8

1976
A. Suprapubic cystostomy
B. Per Urethral catheterization
C. Laparotomy
D. Pelvic packing along with B/L percutaneous nephrostomy
----------------------------------------
30. Which one of the options listed below does not belong to the category of damage control surgery?
A. Control of hemorrhage
B. Resuscitation
C. Decontamination
D. Vascular anastomosis
----------------------------------------
31. Which of the following is true regarding shock?
A. Anaerobic respiration produces carbon dioxide
B. Hypovolaemic shock—most common type
C. Stage of compensatory shock is also called stage 2 of shock
D. Anaphylactic shock—is due to Type 2 hypersensitivity reaction
----------------------------------------
32. Which of the following is a complication usually associated with the procedure shown in the image
below?

Page 9

1977
A. Aspiration
B. Torticollis
C. Pneumothorax
D. Recurrentlaryngeal nerve paralysis
----------------------------------------
33. Among the options provided, which one is NOT acknowledged as a potential complication resulting
from a significant blood transfusion?
A. Hypocalcemia
B. Hypokalemia
C. Hypothermia
D. Hypernatremia
----------------------------------------
34. Identify the given image of surgical instrument?
(or)
Please identify the surgical instrument depicted in the provided image.

A. Mixter
B. Rampley
C. Kocher
D. Adson
----------------------------------------
35. What is the minimum platelet count necessary for surgical procedures?
A. 20,000 (109/L)
B. 30,000 (109/L)
C. 40,000 (109/L)
D. 50,000 (109/L)
----------------------------------------
36. A patient after a road traffic accident presented with pain abdomen. The resident examines the child
and found that vitals were stable and tenderness was present in the left lumbar region. Which is the

Page 10

1978
best investigation of choice?
A. Contrast enhanced CT scan
B. Retrograde urethrogram
C. Wait and watch
D. Emergency laparotomy
----------------------------------------
37. A patient presents following a motorcycle road traffic accident. There was no pain or injury.
Examination shows blood at the tip of the urethral meatus. What will be your next step in management?
(or)
A patient arrives after being involved in a motorcycle road traffic accident. The patient does not report
any pain or injury. Upon examination, there is evidence of blood at the urethral meatus. What would be
your subsequent course of action in terms of managing this patient?
A. Foley's catheterization
B. Wait and watch
C. Nephrogram
D. Suprapubic cystostomy
----------------------------------------
38. In the case of a man who arrived at the emergency department with a head injury resulting from a
vehicle accident, the preferred diagnostic test is
A. CECT (Contrast-enhanced CT)
B. MRI (Magnetic Resonance Imaging)
C. NCCT (Non-Contrast CT)
D. X-ray
----------------------------------------
39. What could be the likely diagnosis for a 40-year-old patient presenting with a femur fracture
accompanied by pulmonary infiltration and respiratory distress?
A. Fat embolism
B. Pulmonary embolism
C. Air embolism
D. Obstruction
----------------------------------------
40. In the event of an accident, prompt action should be taken in the initial management following the
arrival of the medical team. Excluding
A. Stabilization of cervical vertebrae
B. Check BP
C. Check respiration
D. GCS score

Page 11

1979
----------------------------------------
41. Identify the condition shown below

A. Strawberry stain
B. Erythema multiforme
C. Erythema subitem
D. Port wine stain
----------------------------------------
42. Type of necrosis seen in brain is
A. Coagulative
B. Liquefactive
C. Fat
D. Fibrinoid
----------------------------------------
43. A 38-year-old man was brought to the emergency department after sustaining an RTA and injury to
the lower chest. On presentation, the patient is stable, and vitals are normal. Chest radiography
showed clear lung fields. What is the next step in management?
A. eFAST
B. Tube thoracostomy
C. CECT chest
D. CECT abdomen
----------------------------------------
44. Which of the following is not taken into consideration in ASEPSIS wound scoring?
A. Wound swab from the site
B. Serous discharge
C. Erythema
D. Induration
----------------------------------------

Page 12

1980
45. What is the most probable diagnosis for a 41-year-old female patient who presented with a twisted
swelling on the scalp and face, accompanied by a pulsating sensation and a bruit heard during
auscultation?

A. Neurofibromatosis
B. Liposarcoma
C. Cirsoid Aneurysm
D. Varicocele
----------------------------------------
46. Which one of the following is excluded from the primary survey?
A. Measuring blood pressure
B. Removing clothes to inspect for wounds
C. Taking CT scan of the patient
D. Checking airway patency
----------------------------------------
47. Long term diabetic patient with blisters, walked barefoot for a few miles on hot sand. He is
presented with this clinical condition. What is the probable diagnosis

A. Diabetic foot
B. Burn
C. Necrotizing fasciitis
D. Cellulitis

Page 13

1981
----------------------------------------
48. What is the next course of action for managing a 24-year-old male who was struck by a motorbike
while crossing the road, resulting in abdominal injuries and bruising? His blood pressure is 88/60, and
IV fluids have been initiated. A positive FAST examination was performed. FAST: (focused assessment
with sonography for trauma)
A. Abdominal CT scan
B. Abdominal MRI scan
C. Exploratory laparotomy
D. Diagnostic peritoneal lavage
----------------------------------------
49. In a patient with liver cirrhosis, what is the primary cause of mortality when esophageal varices are
present?
A. Hypovolemic shock
B. Cardiogenic shock
C. Obstructive shock
D. Distributive shock
----------------------------------------
50. What is the size of the cannula gauge used in this case?

A. 14
B. 16
C. 18
D. 20
----------------------------------------
51. A man presented with a history of stab injury over the arm. On examination, there was a hematoma,
and radial and ulnar pulses were not felt. A 3.5cm longitudinal tear is noted in the brachial artery. Which
among the following is the best treatment option?
A. Primary repair with end-to-end anastomosis
B. Repair with great saphenous vein graft
C. Repair with prosthetic graft

Page 14

1982
D. Repair with saphenous vein patch
----------------------------------------
52. A 24 year old male patient was brought to the emergency department following a road traffic
accident. His heart rate was 100/min, respiratory rate was 24/min, and his chest X ray is given below.
Which of the following is contraindicated in him?

A. Nasogastric tube insertion


B. Chest tube insertion
C. Log roll
D. Epidural anaesthesia
----------------------------------------
53. A patient was received in the casualty following a road traffic accident. His BP was 90/60 mmHg
and his pulse rate was 120/min. The emergency room physician asked you to replace the fluid loss in
this patient. What is the flow rate of fluid with the IV cannula given in the image?

A. 86ml/min
B. 96ml/min
C. 90 ml/min
D. 63ml/min
----------------------------------------
54. What is triage for?
A. To prepare for a disaster
B. To assess the impact of a disaster

Page 15

1983
C. To classify the priority of treatment
D. To rehabilitate following a disaster
----------------------------------------
55. A 32-year-old male patient admitted after a road traffic accident is put on mechanical ventilation. He
opens his eyes on verbal command and moves all four limbs spontaneously according to command.
Calculate his GCS.
(or)
A 32-year-old male patient was admitted to the emergency department following a road traffic accident.
He sustained head trauma and is currently receiving mechanical ventilation. The patient responds to
verbal commands by opening his eyes and exhibits spontaneous movement in all four limbs. What is
the Glasgow Coma Scale (GCS) score for this patient?
A. Eyes-2, Verbal -1, Motor -5
B. Eyes-2, Verbal -NT, Motor -5
C. Eyes -3, Verbal-1, Motor -6
D. Eyes-3, Verbal - 1, Motor-6
----------------------------------------
56. What is the needle placement angle when using the interrupted suturing technique to suture a
laceration in the ER?
A. 60 degrees
B. 70 degrees
C. 80 degrees
D. 90 degrees
----------------------------------------
57. What could be the likely diagnosis for a 26-year-old male patient who presented to the emergency
department with abdominal pain and difficulty in passing stool for the past three days? The patient
reports being gored by a bull in the abdomen three days ago. Please refer to the provided X-ray for
further evaluation.

A. Hemothorax
B. Hollow viscus perforation
C. Pneumothorax

Page 16

1984
D. Intestinal obstruction
----------------------------------------
58. Which of the following methods is accurate for measuring the nasogastric tube?
A. Tip of nose to ear lobule to xiphisternum
B. Angle of mouth to tragus to xiphisternum
C. Ala of nose to angle of mandible to xiphisternum
D. Angle of mouth to ear lobule to umbilicus
----------------------------------------
59. A patient with a history of a road traffic accident 2 hours ago was brought to the emergency
department with an altered sensorium. His GCS was 10 and the CT scan is given below. What is the
most likely diagnosis?
(or)
What is the probable diagnosis for a patient who was involved in a road traffic accident 2 hours ago,
brought to the emergency department, and exhibiting an altered level of consciousness with a Glasgow
Coma Scale (GCS) score of 10? Please refer to the provided CT scan.

A. Subdural hematoma
B. Intracerebral haemorrhage
C. Extradural hematoma
D. Hemorrhagic contusion
----------------------------------------
60. What would be the correct course of action for managing a 25-year-old male patient who was
brought to the emergency room after a road traffic accident? The patient has a respiratory rate of 25
breaths per minute and a blood pressure of 80/45mmHg. During examination, hyper-resonance is
detected on the left side of the chest. The provided image shows the chest x-ray.

Page 17

1985
A. Wide bore needle insertion in the 2ndICS
B. Wide bore needle insertion in the 5th ICS followed by an intercostal drain in the 5th ICS
C. Aspiration of the pleural fluid
D. No intervention needed
----------------------------------------
61. What is the Glasgow Coma Score of a patient who is making Incomprehensible sounds, opens their
eyes in response to painful stimuli and flexes one limb in response to painful stimuli.
A. 9
B. 15
C. 10
D. 8
----------------------------------------
62. During the surgical procedure for an inguinal hernia in a child, the surgeon observes that the bowel
is necrotic. Regarding the resection and anastomosis of the bowel, which statement is accurate?
A. The surgeon must use chromic catgut
B. The surgeon must not include the submucosa in the sutures
C. The surgeon must use the single-layer, extramucosal suture technique
D. The surgeon must use the single-layer seromuscular suture technique
----------------------------------------
63. What is the most suitable graft for performing a femoropopliteal bypass procedure?
A. Dacron
B. Great saphenous vein
C. Polytetrafluoroethylene
D. Short saphenous vein
----------------------------------------
64. What is a valid statement regarding chromic catgut?
A. It is made from cat gut
B. It is made from rabbit gut

Page 18

1986
C. It is absorbed by enzymatic degradation and macrophage phagocytosis
D. It is tanned with nickel salts to improve handling and to resist degradation in tissue
----------------------------------------
65. Please identify the instrument shown in the image provided:

A. Daever’s retractor
B. Morris retractor
C. Czerney’s retractor
D. Double hook retractor
----------------------------------------
66. During the procedure of performing a fasciotomy to treat compartment syndrome, the incision is
made through the following layers:
A. Skin and Subcutaneous tissue
B. Skin, Subcutaneous tissue, Superficial fascia
C. Skin, Subcutaneous tissue, Superficial fascia and Deep fascia
D. Skin, Subcutaneous tissue, Superficial fascia, Deep fascia and a few muscular fibres
----------------------------------------
67. Which of the options accurately represents a valid statement concerning retroperitoneal trauma?
A. All zone 1 injury to be treated conservatively
B. Zone 3 injury whether blunt or penetrative should always be explored
C. Zone 2 injury with expanding hematoma should be treated surgically
D. Mattox maneuver is when the ascending colon is mobilized to the right to visualize the Aorta
----------------------------------------
68. Which of the following conditions is the least effectively diagnosed by eFAST?
A. Pericardial Effusion
B. Pneumothorax
C. Retroperitoneal hematoma
D. Renal injury
----------------------------------------

Page 19

1987
69. Which of the following adverse effects is not observed following a massive blood transfusion?
A. Hypocalcemia
B. Hypercalcemia
C. Hypokalemia
D. Hyperkalemia
----------------------------------------
70. Apart from the abdomen, which body part is included in e-FAST?
A. Pelvic Cavity
B. Peripheral vessels
C. Thoracic cavity
D. Dural and subdural spaces
----------------------------------------
71. What is the calculation method for determining the modified shock index?
(or)
What is the calculation method for determining the modified shock index?
A. HR/SBP
B. HR/DBP
C. HR/MAP
D. PR/SBP
----------------------------------------
72. Which of the following steps was done incorrectly by the intern when placing an intercostal drainage
tube in the patient?
(or)
Which of the following steps was done incorrectly by the intern when placing an intercostal drainage
tube in the patient?
A. Identified 5th intercostal space anterior to mid axillary line to place the tube
B. Inserted above the upper border of the rib
C. Insertion of the needle, which will be between your ribs on your back
D. Incised and digitally explored
----------------------------------------
73. What is the probable diagnosis for a patient who is taking steroids for psoriasis and is experiencing
a low-grade fever only at night, weight loss, and has an ulcer on her neck with undermined edges, as
depicted in the provided image?

Page 20

1988
A. Psoriasis flare up
B. Tubercular lymphadenitis
C. Lymphosarcoma
D. Syphilis
----------------------------------------
74. What is the diagnosis for the unconscious patient brought to the casualty department following a
road accident, as depicted in the provided CT scan image?

A. Extradural hematoma (EDH)


B. Subdural hematoma (SDH)
C. Intraparenchymal bleed
D. Subarachnoid hemorrhage (SAH)
----------------------------------------
75. What is the diagnosis of a female patient who presents with a mass in the inguinal area that can be
reduced from the deep inguinal ring accompanied by a gurgling sound?
A. Direct inguinal hernia
B. Indirect inguinal hernia
C. Femoral hernia
D. Pantaloon hernia
----------------------------------------
76. Which of the following statements accurately describes Tania solium?

Page 21

1989
A. eggs are not infective to man
B. hooks are absent
C. The eggs do not float in saturated salt solution
D. intermediate host is cow
----------------------------------------
77. How is empyema thoracis treated?
A. Antibiotics alone.
B. Chest tube drainage and antibiotics
C. ICD alone
D. Wait and watch
----------------------------------------
78. Most common radio-isotope in pet scan :
A. 13C
B. 14C
C. 18F
D. ALL OF THE ABOVE
----------------------------------------
79. What would be your course of action in the ER when a young man, who has suffered a road traffic
accident, arrives with significant blood loss and only one IV line available, necessitating the acquisition
of 2 units of packed red blood cells (PRBC) and 4 platelets from the blood bank?
A. Start PRBC 1st and store platelet at room temperature
B. Start platelet and store PRBC at room temperature
C. Only transfuse PRBC
D. Transfuse PRBC and store platelet at 2-6 degrees
----------------------------------------
80. What would be the immediate next course of action for a conscious patient with multiple rib
fractures who presented to the emergency department after a road traffic accident? The patient is
speaking only single words and upon examination, has a respiratory rate of 40 breaths per minute and
a blood pressure of 90/40 mm Hg.
A. Urgent IV fluid administration
B. Intubate the patient
C. Chest X-ray
D. Insert needle in 2nd intercostal space
----------------------------------------
81. What is the accurate technique for the insertion of a Nasogastric tube?
A. Supine with neck flexed
B. Supine with neck extended

Page 22

1990
C. Sitting with neck flexed
D. Sitting with neck extended
----------------------------------------
82. What is the diagnosis for a 7-day-old infant who has bilious vomiting, significant abdominal
distention, absent bowel sounds, and an abdominal X-ray indicating the presence of multiple gas-filled
loops?
A. Hirschsprung disease
B. Congenital Hypertrophic pyloric stenosis
C. Duodenal atresia
D. Malrotation of gut
----------------------------------------
83. What is the initial treatment for a newborn baby who has abdominal distension, bilious vomiting,
failure to pass meconium after 24 hours, and an X-ray showing dilated small bowel loops with no
air-fluid levels and a soap bubble appearance in the lower right quadrant?
A. Paul mikulicz ileostomy
B. Bishop ileostomy
C. Contrast enema
D. Barium enema
----------------------------------------
84. What is the initial investigation that should be performed in the case of a 2.0 Kg male baby, born at
home as a full-term delivery, who is experiencing excessive frothing shortly after birth and displaying
mild respiratory distress, with an unsupervised antenatal period and uncomplicated delivery?
A. Bronchoscopy with injection of methylene blue
B. NG Tube insertion and CXR to check position of tube
C. CT chest
D. Endoscopy
----------------------------------------
85. Which of the subsequent options is not examined by FAST ultrasonography (USG)?
A. Pericardium
B. Pleural cavity
C. Spleen
D. Liver
----------------------------------------
86. Determine the Glasgow Coma Scale (GCS) score for a patient who displays response to pain by
opening their eyes, is conscious but confused, unable to perceive time, and demonstrates flexion in
response to painful stimuli applied to the arm.
A. 8
B. 9

Page 23

1991
C. 10
D. 11
----------------------------------------
87. Could the occurrence of surgical site infection (SSI) have been reduced in this patient if prophylactic
antibiotics were administered prior to the surgery?
A. 60 minutes before skin incision
B. 1-3 hours before skin incision
C. At time of surgical incision
D. Night before surgery for peaking of effect
----------------------------------------
88. Which Instrument is shown below?

A. Artery forceps
B. Kocher forceps
C. Allis forceps
D. Babcock forceps
----------------------------------------
89. Which organ has the highest chances of Graft rejection response?
A. Cornea
B. Gut
C. Liver
D. Skin
----------------------------------------
90. For the purpose of assessing under-perfusion in hypovolemic shock, which particular organ should
be evaluated?
A. Kidney
B. Heart
C. Lung
D. Liver

Page 24

1992
----------------------------------------
91. What is the best position for a conscious patient to be in during the insertion of a Ryle's tube?
A. Supine with neck flexion
B. Supine with neck extension
C. Sitting with neck flexion
D. Sitting with neck extension
----------------------------------------
92. Which of the following colored cannula will you use to achieve maximum flow rate, in a patient who
needs a large amount of fluids?
(or)
Which of the following colored cannula will you use to achieve maximum flow rate, in a patient who
needs a large amount of fluids?
A. Grey
B. Green
C. Blue
D. Pink
----------------------------------------
93. A patient was brought to the casualty after an RTA. He has chest trauma, respiratory rate of 44 and
BP is 90/40 mm/Hg. He is conscious and speaks single words. There is a hyper-resonance on the
involved side of his chest. What will be your next, immediate step?
A. Intubate
B. Insert needle in appropriate intercostal space
C. Start intravenous fluids urgently
D. Take a chest X-ray
----------------------------------------
94. Device shown in the image is used for which purpose?

A. Prevent viral infections


B. Prevent transfusion related reactions
C. Prevent infections

Page 25

1993
D. All of the above
----------------------------------------
95. The surgical site infection rate expected in a patient with a clean-contaminated wound that is
operated upon after antibiotics prophylaxis is:
A. 1-2%
B. 3%
C. 13-20%
D. 20-30%
----------------------------------------
96. Which of the following is the principle behind the wound-healing procedure shown in the image
below?

A. Negative pressure wound therapy


B. Positive pressure wound therapy
C. Foam therapy
D. Sponge therapy
----------------------------------------
97. The components of qSOFA scoring include: A. Mental status B. Blood pressure C. Respiratory rate
D. Heart rate
A. A & B
B. C & D
C. A, B&C;
D. A, B, C & D
----------------------------------------
98. Match the following: A. Urine Bag B. Nasogastric Tube C. Underwater seal bag D. Suction cannula

Page 26

1994
A. A-3, B-4, C-2, D-1
B. A-3, B-1, C-2, D-4
C. A-2, B-4, C-3, D-1
D. A-2, B-1, C-3, D-4
----------------------------------------
99. Who would be the most suitable candidate to receive organs from a 32-year-old woman who has
been declared brain dead and whose family has chosen to donate her organs?
A. 14-year-old child with multi-organ dysfunction syndrome
B. 50-year alcoholic man with liver cirrhosis with 1-month abstinence
C. 70-year-old woman with end-stage renal disease
D. 32-year-old woman with renal failure
----------------------------------------
100. What is the cannula's size shown in the image below?

A. 16 G
B. 18 G
C. 20 G
D. 22 G
----------------------------------------
101. FAST stands for

Page 27

1995
A. First assessment with sonography in trauma
B. Focussed assessment with sonography in trauma
C. First abdominal sonography in trauma
D. Focussed abdominal sonography in trauma
----------------------------------------
102. Calculate the Glasgow Coma Scale (GCS) of a patient admitted after a road traffic accident who is
currently on mechanical ventilation, but is able to open his eyes in response to a verbal command and
spontaneously moves all four limbs.
A. Eyes-2, Verbal-1, Motor-5
B. Eyes-2, Verbal-NT, Motor-5
C. Eyes-3, Verbal-1, Motor-6
D. Eyes-3, Verbal-NT, Motor-6
----------------------------------------
103. The position of the patient in the image shown below is known as:

A. Lloyd Davis position


B. Jack knife position
C. Trendelenburg position
D. Reverse Trendelenburg position
----------------------------------------
104. What is the tool utilized for the evaluation of wound infection severity?
A. ASA classification
B. SIRS criteria
C. Southampton grading
D. APACHE Il score
----------------------------------------
105. What is the most effective approach for treating the wound depicted in the provided image?

Page 28

1996
A. Cleaning and dressing
B. Debridement and dressing
C. Cleaning and skin grafting
D. Cleaning and use of allograft
----------------------------------------
106. Which of the options provided is not considered a part of qSOFA?
A. Respiratory rate ≥ 22/min
B. Systolic BP ≤ 100 mm Hg
C. Altered mental status
D. Bilateral pupillary reflex
----------------------------------------
107. Which of the following is not a feature of SIRS?
A. Temperature > 38 degrees Celsius
B. Respiratory Rate > 20/minute and PaCO2 < 35 mmHg
C. Pulse rate > 90/minute
D. WBC > 12000/mm3 or < 4000/mm3
----------------------------------------
108. Which nerve is typically affected when patients experience loss of sensation while shaving after
parotidectomy?
A. Auriculotemporal nerve
B. Great auricular nerve
C. Facial nerve
D. Mandibular nerve
----------------------------------------
109. Identify the following instrument?

Page 29

1997
A. Unipolar cautery
B. Bipolar cautery
C. Harmonic scalpel
D. Ligasure
----------------------------------------
110. What is the name of the instrument depicted below, which is utilized for the creation of a
pneumoperitoneum?

A. Veress needle
B. Morris' needle
C. Yankauer suction tip
D. Von Graefe needle
----------------------------------------
111. Please assess the severity of the bedsore depicted in the provided image.

Page 30

1998
A. 4
B. 3
C. 2
D. 1
----------------------------------------
112. Which intravenous cannula is recommended for a patient who arrived at the emergency room
following a road traffic accident and needs a significant volume of fluids to be administered?
A. Green
B. Blue
C. Pink
D. Grey
----------------------------------------
113. The instrument given in the image is:

A. Hemovac
B. Jackson-Pratt drain
C. Minivac
D. Corrugated drain
----------------------------------------
114. What is the likely diagnosis for a patient who is immunocompromised, comes from a lower-income
household, and exhibits mild fever only at night along with weight loss? The patient also has a soft,

Page 31

1999
non-tender swelling on their neck, as observed in the provided image, with a positive fluctuation test.

A. Branchial cyst
B. Tubercular lymphadenitis
C. Thyroglossal cyst
D. Lipoma
----------------------------------------
115. What would be the appropriate course of action for managing a 25-year-old male patient who was
brought to the emergency department after a road traffic accident (RTA)? The patient has a respiratory
rate of 25 breaths per minute and a blood pressure of 80/45 mmHg. During examination,
hyper-resonance is detected over the left side of the chest. The provided chest x-ray is shown below.

A. Wide bore needle insertion in the right 5th ICS


B. Wide bore needle insertion in the left 5th ICS followed by an intercostal drain
C. Aspiration of the pleural fluid
D. No intervention needed
----------------------------------------
116. In a patient experiencing excessive bleeding, with limbs displaying low temperature and reduced
central venous pressure (CVP), what type of shock is observed?
A. Cardiogenic shock
B. Hypovolemic shock
C. Obstructive shock
D. Hypo adrenal shock

Page 32

2000
----------------------------------------
117. What is the diagnosis based on the image below?

A. Botryomycosis
B. Erysipelas
C. Ecthyma
D. Carbuncle
----------------------------------------
118. What is the most likely diagnosis for a cricket player who was initially hit on the head with a cricket
ball, appeared fine, but suddenly collapsed? He later regained consciousness and played the match
exceptionally well, but was later found unconscious in the dressing room. The provided CT scan result
is displayed below.

A. Extradural hemorrhage
B. Artifact
C. Subdural hemorrhage
D. Subarachnoid hemorrhage
----------------------------------------
119. Sebaceous cyst is not seen in:
A. Back
B. Soles
C. Scalp

Page 33

2001
D. Scrotum
----------------------------------------
120. Which of the subsequent options does not constitute a Hospital-Acquired infection?
A. Surgical site infection
B. STD
C. UTI
D. Pneumonia
----------------------------------------
121. In a hypotensive patient with a head injury, which of the subsequent fluids should be avoided?
A. 0.9% NS
B. 5% Dextrose
C. Ringer lactate
D. 3% saline
----------------------------------------
122. What is the probable diagnosis for the lesion developed by a patient after 6 months of undergoing
heart surgery, as depicted below?

A. Hypertrophic scar
B. Atrophic scar
C. Keloid scar
D. Contracture
----------------------------------------

Correct Answers
Question Correct Answer

Question 1 1
Question 2 1
Question 3 3

Page 34

2002
Question 4 2
Question 5 4
Question 6 1
Question 7 2
Question 8 4
Question 9 3
Question 10 4
Question 11 1
Question 12 4
Question 13 1
Question 14 1
Question 15 1
Question 16 1
Question 17 1
Question 18 4
Question 19 4
Question 20 2
Question 21 3
Question 22 3
Question 23 4
Question 24 3
Question 25 1
Question 26 1
Question 27 2
Question 28 2
Question 29 3
Question 30 4
Question 31 2
Question 32 3
Question 33 4
Question 34 1
Question 35 4
Question 36 1
Question 37 2
Question 38 3

Page 35

2003
Question 39 1
Question 40 2
Question 41 4
Question 42 2
Question 43 1
Question 44 4
Question 45 3
Question 46 3
Question 47 3
Question 48 3
Question 49 1
Question 50 3
Question 51 2
Question 52 2
Question 53 3
Question 54 3
Question 55 4
Question 56 4
Question 57 2
Question 58 1
Question 59 3
Question 60 2
Question 61 4
Question 62 3
Question 63 2
Question 64 3
Question 65 2
Question 66 3
Question 67 3
Question 68 3
Question 69 2
Question 70 3
Question 71 3
Question 72 3
Question 73 2

Page 36

2004
Question 74 2
Question 75 2
Question 76 3
Question 77 2
Question 78 3
Question 79 1
Question 80 4
Question 81 3
Question 82 4
Question 83 3
Question 84 2
Question 85 2
Question 86 3
Question 87 1
Question 88 1
Question 89 2
Question 90 1
Question 91 3
Question 92 1
Question 93 2
Question 94 2
Question 95 2
Question 96 1
Question 97 3
Question 98 2
Question 99 4
Question 100 3
Question 101 2
Question 102 4
Question 103 4
Question 104 3
Question 105 1
Question 106 4
Question 107 2
Question 108 2

Page 37

2005
Question 109 1
Question 110 1
Question 111 1
Question 112 4
Question 113 2
Question 114 2
Question 115 2
Question 116 2
Question 117 4
Question 118 1
Question 119 2
Question 120 2
Question 121 2
Question 122 3

Solution for Question 1:


Correct option A:
• Thromboelastography (TEG) is a method used to monitor coagulation function intraoperatively. It
involves measuring the viscoelastic properties of whole blood as it clots, providing a comprehensive
assessment of the clotting system. TEG can detect coagulation abnormalities that may not be apparent
with standard laboratory tests, allowing for timely intervention to prevent excessive bleeding or
thrombosis.
Incorrect options:
Option B. Plethysmography: Plethysmography is a method used to measure changes in volume in an o
rgan or body part. It is often used to evaluate blood flow, such as in cases of deep vein thrombosis or p
eripheral arterial disease.
Option C. Sonography: Sonography, or ultrasound, is a diagnostic imaging technique that uses high-fre
quency sound waves to create images of organs and tissues inside the body. It is often used to evaluat
e soft tissue structures like the liver or gallbladder.
Option D. Elastography: Elastography is a newer imaging technique that uses ultrasound or other imag
ing modalities to evaluate the stiffness or elasticity of tissues. It is often used to evaluate liver fibrosis, f
or example, in patients with chronic liver disease.

Solution for Question 2:


Correct option A:
• Emergency laparotomy: This option involves performing an immediate surgical exploration of the
abdomen to identify and treat any internal injuries that may have occurred from the stab injury. While
this may be necessary in some cases, it may not be the best course of action if the child is stable and

Page 38

2006
there is no evidence of internal injury or bleeding.
Incorrect options:
Option B. Observation: This option involves closely monitoring the child's condition and looking for any
signs or symptoms of internal injury or bleeding. This may involve repeated physical exams, imaging st
udies, and laboratory tests. This may be a
reasonable course of action if the child remains stable without evidence of internal injury.
Option C. Intravenous hydration: This option involves providing fluids through an intravenous (IV) line t
o help maintain the child's blood pressure and hydration status. This may be necessary if the child has
lost fluids or blood from the injury, but it does not address any potential internal injuries.
Option D. Wait and watch: This option is similar to observation but involves less frequent monitoring an
d less aggressive intervention. This may be appropriate if the child is stable and there is no evidence of
internal injury, but it carries the risk of missing a significant injury.

Solution for Question 3:


Correct option C:
• Injuries to the male urethra can occur due to trauma, such as a motor vehicle accident.
• The urethra is divided into four parts: The prostatic urethra, the membranous urethra, Then Bulbous
and the Pendulous urethra.
• Proximal injuries are secondary to pelvic fracture.
• In the picture they have shown retrograde urethrogram to check the extravasation
• In the picture it can be noted that the leak is proximal to bulbar urethra which is membranous urethra.
Incorrect options:
Option A: The bulbar urethra is the part of the urethra that passes through the bulb of the penis. Injurie
s to the bulbar urethra usually result in blood in the urine and difficulty voiding, but not necessarily bloo
d at the meatus.
Option B: The spongy (penile) urethra is the longest part of the male urethra and runs through the lengt
h of the penis. Injuries to the spongy urethra can occur due to penile fractures or direct trauma to the p
enis. However, blood at the meatus suggests an injury near the external urethral opening, which is not i
n the penile shaft.
Option D: The penile urethra refers to the distal part of the spongy urethra that runs through the penis.
As mentioned above, blood at the meatus suggests an injury near the external urethral opening, which
is not located in the penile shaft.

Solution for Question 4:


Correct option B:
• The given image is of the Ringer lactate solution.
• Also known as Hartmann’s solution
• The given compositions are of Ringer lactate solution as

Page 39

2007
• It is the only solution containing Lactate and high Calcium
Incorrect options:
Option A. Haemacel solution
• Its compositions are
• Na+ 145 mmol/L
• Cl- 145 mmol/L
• Lactate absent
• K+ 5.1 mmol/L
• Ca2+ <1 mmol/L
Option C: Isolyte having a pH of 7.4, provides water for hydration and electrolytes. It can induce diuresi
s. Sodium element is the major cation of isolyte.
Option D: It provides electrolytes and calories, acts as a
water source for hydration, and can induce diuresis.

Solution for Question 5:


Correct option D:
• Pressure sores, also known as pressure ulcers, are a type of injury that occurs when prolonged
pressure is applied to a particular area of the skin, mainly over a bony prominence, thereby causing
damage to underlying tissues.
• The image is of stage 4 pressure sores, which are severe and involved in extensive tissue damage.
Damage is extended to the skin, muscles, bone, and supporting structures as the given wound has a
large, deep wound with a substantial loss of skin. And it is covered with necrotic tissue. These are the
characteristics of stage 4 pressure sores.
Incorrect option:
Option A. The image has extensive wound instead of redness, hence not stage 1.
Option B. Stage 2 involves the formation of blisters or ulcers, whereas the image is of having damage t
o underlying structures. Hence not stage 2
Option C: Stage 3 involves major tissue damage, whereas the image shows damage to underlying stru
ctures and muscles along with major tissue damage.

Solution for Question 6:


Correct option A:
• Ambulatory patients comes under the category of Green
• The green category includes patients with minor injuries or illnesses who can walk and do not require
immediate medical attention.

Page 40

2008
• These patients are also known as ambulatory patients. They may require medical attention, but their
condition is not life-threatening or severe.
Incorrect options:
Option B. Medium-risk patients: There is no "medium risk" category in triage. Triage categories are typi
cally divided into four or five levels based on the severity of a
patient's condition, such as "immediate," "urgent," "delayed," "minimal," or "expectant."
Option C. High-risk patients: This term is too vague to be used as a
triage category. Triage categories are based on specific criteria for a
patient's vital signs, level of consciousness, and other clinical findings.
Option D. Dead patients: Deceased Patients do not undergo triage, as triage is intended to prioritize pa
tients for treatment based on their clinical status. In the case of a deceased patient, the focus shifts to
other aspects of care, such as body preservation, family notification, and legal procedures.

Solution for Question 7:


• The injury in question involves tissue necrosis, possibly due to a traumatic event such as a road traffic
accident (RTA). Out of the four given options, the next management for this type of injury after 3 days of
RTA is most likely to be "Debridement of necrotic tissue and hyperbaric oxygen".
• Debridement of necrotic tissue involves the removal of dead or damaged tissue to prevent infection
and promote healing. This procedure is typically performed in cases of tissue necrosis, which can occur
due to various reasons such as trauma, infection, or ischemia.
• Hyperbaric oxygen therapy (HBOT) is a medical treatment that involves breathing pure oxygen in a
pressurized chamber. It treats various conditions, including tissue necrosis, by increasing the amount of
oxygen that reaches the damaged tissues. The high pressure helps dissolve more oxygen in the blood,
stimulating the growth of new blood vessels and promoting tissue regeneration.
• The combination of debridement and hyperbaric oxygen therapy can be particularly effective in tissue
necrosis caused by ischemia, as it helps restore blood flow and oxygenation to the affected area. This
can help prevent further tissue damage and promote healing.
Incorrect Choices:
• Option a. Debridement of necrotic tissue with primary closure", may not be appropriate in cases of
tissue necrosis, as it involves closing the wound immediately after debridement. This can trap bacteria
and other debris inside the wound, increasing the risk of infection and delaying healing.
• Option c. Amputation may be necessary in severe cases of tissue necrosis that cannot be treated by
other means. However, amputation is a last resort and is only considered when all other options have
been exhausted.
• Optiond. Clean and close with a rotation flap may be appropriate in certain cases of tissue necrosis,
particularly when the affected area is small, and the surrounding tissue is healthy. However, this option
is unlikely to be suitable in cases of extensive tissue damage, where debridement and HBOT are more
appropriate.

Solution for Question 8:

Page 41

2009
• Surgical emphysema is when air gets trapped in the soft tissues around the surgical site, causing
swelling and discomfort. It can occur as a complication of various surgical procedures, including
tracheostomy, and is most commonly caused by air escape from the surgical site.
• In the case of tracheostomy, tight suture is one of the most common causes of surgical emphysema.
This occurs when the sutures used to close the surgical site are too tight, which can cause the
surrounding tissues to tear and allow air to escape. This air can then get trapped in the soft tissues
around the tracheostomy site, leading to surgical emphysema.
Incorrect Choices:
• a. Puncture of the posterior wall is another possible cause of surgical emphysema, but it is less likely
in this case. Puncture of the posterior wall of the trachea during tracheostomy can cause air to leak into
the surrounding tissues, leading to surgical emphysema. However, this is a less common complication
and is usually associated with improper tracheostomy tube placement.
• b. High tracheostomy is not a likely cause of surgical emphysema in this case. A high tracheostomy is
one in which the tracheostomy tube is placed above the second or third tracheal ring. While this can
increase the risk of certain complications, such as bleeding and damage to the recurrent laryngeal
nerve, it is not typically associated with surgical emphysema.
• c. Blockage of the tube may cause respiratory distress, but it is not a likely cause of surgical
emphysema. Blockage of the tracheostomy tube can cause air to leak out of the surgical site, leading to
respiratory distress. However, this is not typically associated with surgical emphysema.

Solution for Question 9:


Answer option C-
• Adjacent organ involvement is not part of AAST grading.
• The American Association for the Surgery of Trauma (AAST) grading system is commonly used to
categorize splenic injury based on the severity of the trauma.
• The AAST grading system includes five grades of splenic injury, ranging from Grade I (minimal injury)
to Grade V (complete disruption of the spleen). The AAST grading system is based on four components
of splenic trauma, which are: Parenchymal destruction: This refers to the degree of damage to the
splenic tissue itself. The AAST grading system considers the size and depth of the laceration, as well
as the presence of subcapsular hematoma. Vascular involvement refers to damage to the blood
vessels that supply the spleen, including the splenic artery and vein. The AAST grading system takes
into account the degree of vascular injury, as well as the amount of blood loss. Hematoma: This refers
to the presence of a collection of blood within the spleen or surrounding tissue. The AAST grading
system considers the hematoma's size and location.
• Parenchymal destruction: This refers to the degree of damage to the splenic tissue itself. The AAST
grading system considers the size and depth of the laceration, as well as the presence of subcapsular
hematoma.
• Vascular involvement refers to damage to the blood vessels that supply the spleen, including the
splenic artery and vein. The AAST grading system takes into account the degree of vascular injury, as
well as the amount of blood loss.
• Hematoma: This refers to the presence of a collection of blood within the spleen or surrounding tissue.
The AAST grading system considers the hematoma's size and location.

Page 42

2010
• Adjacent organ involvement is not a component of splenic trauma based on AAST grading, as it is not
specific to the spleen itself. However, adjacent organ involvement may be considered in the overall
assessment of the patient's condition and management plan, as it can impact the severity and
complexity of the injury.
• Parenchymal destruction: This refers to the degree of damage to the splenic tissue itself. The AAST
grading system considers the size and depth of the laceration, as well as the presence of subcapsular
hematoma.
• Vascular involvement refers to damage to the blood vessels that supply the spleen, including the
splenic artery and vein. The AAST grading system takes into account the degree of vascular injury, as
well as the amount of blood loss.
• Hematoma: This refers to the presence of a collection of blood within the spleen or surrounding tissue.
The AAST grading system considers the hematoma's size and location.

Other options are incorrect

Solution for Question 10:


• Laparoscopic surgery is a minimally invasive surgical procedure that involves making small incisions
in the abdomen and inserting a laparoscope (a thin tube with a camera and light source) to view the
internal organs.
• During laparoscopic surgery, the patient is under general anesthesia, which can cause nausea and
vomiting if the stomach is not empty.
• To reduce the risk of aspiration (inhaling stomach contents into the lungs), it is recommended that
patients undergoing laparoscopic surgery follow NPO guidelines. The specific NPO guidelines may
vary depending on the type of surgery and the individual patient's medical history, but generally,
patients are advised not to eat or drink anything for several hours before the surgery.
Incorrect Choices:
• Option a. Eat full before surgery is incorrect advice that should not be given to the patient. Eating a full
meal before surgery is not recommended as it can increase the risk of aspiration and may cause delays
in the surgery.
• Option b. Drink plenty of water before surgery", is also incorrect advice that should not be given to the
patient. Drinking water before surgery can increase the risk of aspiration and delay the procedure.
Patients are typically allowed to drink clear liquids (such as water or apple juice) up to two hours before
the surgery.
• Optionc. Enema in the night before surgery", may be recommended for certain types of surgeries or
patients, but it is not a standard recommendation for laparoscopic surgery. An enema can help empty
the bowel and reduce the risk of complications during surgery, but this will be determined by the
surgical team based on the individual patient's needs.

Solution for Question 11:

Page 43

2011
• Blood transfusion is a medical procedure that involves the transfer of blood or blood products from
one person (the donor) to another person (the recipient). Before a blood transfusion can occur, the
blood from the donor must be carefully screened and tested to ensure it is safe for the recipient.
• Hep A virus is usually transmitted through contaminated food or water and is not typically transmitted
through blood transfusions. Therefore, testing for Hepatitis A is not routinely performed before blood
transfusion.
Incorrect Choices:
• Option b. Hepatitis B virus can be transmitted through blood and other bodily fluids, and is a routine
test performed before blood transfusion. If a donor tests positive for Hepatitis B, their blood is not used
for transfusion.
• Option c. Hepatitis C virus can also be transmitted through blood and other bodily fluids, and is a
routine test performed before blood transfusion. If a donor tests positive for Hepatitis C, their blood is
not used for transfusion.
• Optiond. HIV is a virus that attacks the immune system and can be transmitted through blood and
other bodily fluids. Testing for HIV 1 & 2 is a routine test performed before blood transfusion. If a donor
tests positive for HIV, their blood is not used for transfusion.

Solution for Question 12:


The next best step in the treatment of a
60-year-old patient with bladder cancer involving the muscle layer would be d. Radical cystectomy. Bla
dder cancer involving the muscle layer is considered invasive, and radical cystectomy is the standard tr
eatment for muscle-invasive bladder cancer. Radical cystectomy involves the surgical removal of the b
ladder, surrounding lymph nodes, and adjacent organs if necessary.
Incorrect choices:
Option a. Intravesical administration of BCG: Intravesical administration of Bacillus Calmette-Guérin (B
CG) is a treatment option for non-muscle invasive bladder cancer (NMIBC). NMIBC refers to bladder c
ancer that has not invaded the muscle layer. In these cases, BCG immunotherapy is commonly used t
o reduce the risk of tumor recurrence and progression. However, in the scenario provided where the pa
tient is diagnosed with bladder cancer involving the muscle layer (muscle-invasive bladder cancer), BC
G alone is not considered the next best step in treatment.
Option b. Radiotherapy: Radiotherapy may be used in the treatment of bladder cancer, especially whe
n radical cystectomy (complete removal of the bladder) is not feasible or when bladder-preserving strat
egies are pursued. However, it is typically not the first-line treatment for muscle-invasive bladder cance
r. The primary treatment for muscle-invasive bladder cancer is radical cystectomy, as it offers the best
chance for cure by removing the cancerous tissue and associated lymph nodes. Radiotherapy may be
used as part of multimodal therapy in some cases, but radical cystectomy is generally considered the tr
eatment of choice for muscle-invasive disease.
Option c. Neoadjuvant chemotherapy with Mitomycin C: Neoadjuvant chemotherapy is the administrati
on of chemotherapy drugs before surgery, with the goal of shrinking the tumor and improving the surgic
al outcome. While neoadjuvant chemotherapy is commonly used in the treatment of muscle-invasive bl
adder cancer, Mitomycin C is not typically the chemotherapy agent of choice. Platinum-based chemoth
erapy regimens, such as cisplatin-based combinations, are commonly used in neoadjuvant or adjuvant
settings for muscle-invasive bladder cancer.

Page 44

2012
Solution for Question 13:
Answer Option A. Reduced tissue perfusion
• Shock is supposed to be a life-threatening event in which circulatory failure occurs.
• All shocks have been reported with inadequate reduced tissue perfusion and tissue hypoxia.
• Hence, cellular as well as tissue hypoxia has been reported in the case of circulatory shock.
• This can lead to cellular death as well as vital organ dysfunction.
Incorrect Choices: Option b. Peripheral or systemic vascular resistance, SVR is considered resistance
in the circulatory system that creates blood pressure and blood flow and is also a
part of cardiac function. SVR decreases when the blood vessels are constricted. If such a type of resist
ance occurs within the pulmonary vasculature, it is called pulmonary vascular resistance (PVR). Option
c. As anybody goes in shock, blood circulation of the body starts to reduce, causing reduction in pulse
rate, breathing rate, and blood pressure to drop abruptly. Hence, tissues and cells of the body feels hy
poxia, i.e, lack of oxygen, and body is considered in shock as body feels less blood circulation in the bo
dy. Option d. Rapid heart rate, i.e, tachycardia, and low blood pressure are key signs of shock. All type
s of shock can have similar symptoms, including shallow breathing, clammy skin, weak pulse, fainting
or dizziness.

Solution for Question 14:


A linear stapler is a surgical instrument used to create secure and consistent staple lines in surgical pro
cedures. It is commonly used in gastrointestinal, thoracic, and gynecological surgeries. The linear stapl
er is designed to cut and staple tissues simultaneously, allowing for faster and more efficient closure of
incisions or anastomoses. A linear stapler typically consists of the following components: 1.
Handle: The handle of a linear stapler is the part held by the surgeon or operator. It is ergonomically d
esigned for easy grip and control during the surgical procedure. 2. Shaft: The shaft is a
long, slender part of the stapler that connects the handle to the stapling and cutting components. 3.
Anvil: The anvil is the stationary part of the stapler that holds the tissue in place during stapling. It is typ
ically curved or flat, depending on the specific application. 4. Stapling Cartridge: The stapling cartridg
e is the part of the linear stapler that contains the staples. It is attached to the shaft of the stapler and p
ositioned opposite the anvil. The cartridge includes rows of pre-loaded staples and a mechanism to dri
ve and deploy them into the tissue. The number and size of the staples can vary depending on the spe
cific stapler model and surgical procedure. 5. Cutting Blade: Some linear staplers include a
cutting blade within the stapling cartridge.
Incorrect choices: Option b. A skin stapler is a medical device used to close skin wounds or surgical inc
isions. It is primarily used in situations where the wound edges can be easily approximated, such as in
lacerations or surgical incisions. The skin stapler consists of a handle and a
cartridge containing small metal staples. Option c. A circular stapler is a specialized surgical instrumen
t used in procedures that require the joining or resectioning of tubular structures, such as the intestines
or blood vessels. It is commonly used in gastrointestinal surgeries, particularly for creating anastomos
es or connections between different segments of the intestine. The circular stapler is designed to creat
e a circular staple line and simultaneously cut and suture the tissue. Option d. A
curved stapler refers to a stapling device with a curved shaft or body. It is typically used in specific surg
ical procedures where access to the target area is limited or where a

Page 45

2013
curved configuration is required to reach certain anatomical structures.

Solution for Question 15:


The type of graft described, where the graft is taken from the same person, is called an autograft. In an
autograft, the donor and recipient are the same individuals. This type of graft is commonly used in cas
es where the patient's own skin is harvested from one area of the body (known as the donor site) and t
ransplanted to another area (known as the recipient site) to promote wound healing or to reconstruct d
amaged tissue. Autografts have a higher success rate compared to other types of grafts because they
are immunologically compatible with the recipient's body, reducing the risk of rejection.
Incorrect option explanations: Option b. Xenograft: Xenograft involves the transplantation of tissues or
organs between different species. For example, if the graft in the given scenario had been taken from a
n animal (such as pig skin) and transplanted onto the patient, it would be considered a xenograft. How
ever, in the given scenario, the graft was taken from the same person, so xenograft is not the correct a
nswer. Option c. Allograft: Allograft refers to the transplantation of tissues or organs between individual
s of the same species but with different genetic makeup. In other words, it involves using grafts from a
genetically different individual, such as a human donor. Since the graft in the given scenario was taken
from the same person, it does not fit the definition of an allograft. Option d. Isograft: Isograft involves th
e transplantation of tissues or organs between genetically identical individuals. This typically occurs bet
ween identical twins or clones where there is a perfect genetic match. In the given scenario, the graft w
as taken from the same person, so technically it could be considered an isograft. However, the term "is
ograft" is not commonly used in cases where the graft is taken from the same person. The more appro
priate term for this situation is autograft.

Solution for Question 16:


Answer Option A -
• These symptoms are highly suggestive of an aortic dissection, which is a life-threatening condition
that requires immediate surgical intervention.
• Aortic dissection involves a tear in the inner lining of the aorta, the large artery that carries blood from
the heart to the rest of the body.
• Urgent surgery is the standard treatment for aortic dissection to repair the tear and prevent further
complications, such as rupture or organ damage.
• Labetalol is a medication that can be used to manage hypertension or high blood pressure. Thus, the
doctor may advise the male to stop taking esmolol for 24 hours followed by urgent surgery as this can
lower your blood pressure too much when it's combined with some anesthetics.
Incorrect option explanation:
Option b. Although it is crucial to seek immediate medical attention like urgent surgery and transfer the
patient to a facility equipped to handle such emergencies, without anesthesia and lowering of blood pre
ssure, urgent surgery cannot be operated.
Option c. Aortic dissection is a medical emergency that requires immediate intervention. "Wait and wat
ch" is not an appropriate approach in this situation because delaying treatment can lead to life-threaten
ing complications such as rupture or organ damage. Timely surgical intervention is crucial to prevent th
ese complications.

Page 46

2014
Option d. Blood transfusion: While blood transfusion may be required in certain situations, such as sev
ere blood loss or anemia, it is not the primary management for an aortic dissection. The key interventio
n in aortic dissection is surgical repair to address the tear in the aorta.

Solution for Question 17:


Extradural hemorrhage (Epidural hemorrhage): This choice is characterized by bleeding between the in
ner surface of the skull and the outer layer of the dura mater. It is typically caused by trauma that result
s in the rupture of the middle meningeal artery. On CT scan, extradural hemorrhage often appears as a
lens-shaped or biconvex hematoma. Based on the symptoms of nausea, vomiting, and unconsciousne
ss, along with the findings on the NCCT brain, option a. Extradural hemorrhage is the most likely diagn
osis.
Incorrect choices:
Option b. Subdural hemorrhage: Subdural hemorrhage refers to bleeding between the dura mater and t
he arachnoid membrane. It is commonly caused by trauma and is associated with tearing of the bridgin
g veins that connect the brain to the dura. On CT scan, subdural hemorrhage usually appears as a
crescent-shaped or concave hematoma.
Option c. Intra-parenchymal hemorrhage: Intra-parenchymal hemorrhage involves bleeding within the b
rain tissue itself. It can be caused by trauma, hypertension, vascular malformations, or other underlying
conditions. On CT scan, intra-parenchymal hemorrhage appears as a
region of high-density blood within the brain tissue.
Option d. Subarachnoid hemorrhage: Subarachnoid hemorrhage occurs when there is bleeding into th
e subarachnoid space, which is the area between the arachnoid membrane and the pia mater surround
ing the brain. It is often caused by the rupture of a cerebral aneurysm. On CT scan, subarachnoid hem
orrhage is typically visualized as areas of high-density blood in the subarachnoid space.

Solution for Question 18:


Answer
Option D - Wounds caused by C. perfringens may occur as inflamed and painful. Sudden onset of swel
ling and pain, fever, delirium, crackles and hypotension can be characterized by the presence of gangr
ene gas.
Incorrect Choices: Option a. Clostridium septicum or perfringens causes the production of gangrene ga
s. It is anaerobic, spore forming, gram positive bacteria that make numerous exotoxins. Gangrene gas i
s most frequently seen after the closure of wound, or wound contaminated with soil or bowel contents.
Option b. Clostridioides difficile is a
type of germ or bacteria that originates from diarrhea that might be life-threatening. It occurs as a side
effect of antibiotics intake. Such infections may occur in the older aged people who use medical care a
nd antibiotics. Option c. Clostridium novyi is a
motile, anaerobic, gram-positive, and spore forming bacteria that is a
causative agent of bighead and black disease. C. novyi Type D
i.e, C. hemolyticum causes bacillary hemoglobinuria.

Page 47

2015
Solution for Question 19:
Gaze palsy refers to an abnormality of eye movement, where there is a
difficulty in directing the gaze towards one or both sides. This condition can indeed be due to a
lesion in the midbrain. In the midbrain, there are several structures involved in the control of eye move
ments, including the third cranial nerve nucleus and the paramedian pontine reticular formation. Dama
ge to these areas can result in gaze palsy.
Incorrect Choices: Option a: Extra dural hemorrhage (EDH) commonly presents with a lucid interval foll
owed by rapid neurological deterioration rather than pinpoint pupils. Pinpoint pupils are often associate
d with opioid overdose or pontine lesions. Option b: Penetrating injury to the eustachian tube may caus
e various complications but not typically CSF otorrhea. The eustachian tube connects the middle ear to
the nasopharynx, helping in maintaining equal pressure on either side of the eardrum. Penetrating trau
ma to the eustachian tube may result in hearing loss or tinnitus, not CSF otorrhea. CSF otorrhea is ofte
n associated with base of skull fractures. Option c: Battle sign, ecchymosis over the mastoid bone, is ty
pically seen in basilar skull fractures, not penetrating eye injuries. Penetrating eye injuries may cause v
isual disturbances, eye pain, or even loss of vision.

Solution for Question 20:


The qSOFA (quick Sequential Organ Failure Assessment) score is a
bedside clinical tool used to identify patients who are at risk of developing sepsis.
It consists of three criteria, and if a patient meets two or more of these criteria, it suggests a
higher risk of poor outcomes. T he components of the qSOFA score are: Lactate dehydrogenase (LDH
) is not included in the qSOFA score. LDH is an enzyme present in many tissues and is often used as a
marker of tissue damage or certain medical conditions. However, it is not part of the qSOFA scoring cri
teria.
Incorrect Choices:
Option a: Respiratory rate >22: This option is incorrect because an elevated respiratory rate greater th
an 22 breaths per minute is actually one of the components of the qSOFA score. An increased respirat
ory rate can be an indicator of impaired breathing and can be associated with sepsis or other severe co
nditions.
Option c: Altered mental status: Altered mental status is indeed one of the components of the qSOFA s
core. It refers to a change in a person's level of consciousness, cognition, or behavior. It can include co
nfusion, disorientation, difficulty staying awake, or other alterations in mental function.
Option d: Systolic blood pressure < 100 mmHg: This option is incorrect because a systolic blood press
ure lower than 100 mmHg is one of the components of the qSOFA score. Hypotension (low blood pres
sure) can be a sign of poor perfusion to vital organs and is considered a red flag for potential sepsis.

Solution for Question 21:


Correct Option C - Intraosseous route access for giving IV fluids:
• The image likely shows a procedure for establishing an intraosseous route for administering IV fluids.
This method is often used when IV access is difficult or impossible. The technique of injecting drugs,
fluids, or blood products straight into medullary cavity of the bone is known as an intraosseous infusion
(IO); this creates a non-collapsible entry point into the systemic venous system.

Page 48

2016
Incorrect Choices:
Option A
- Intraosseous cannula for pain relief: An intraosseous cannula is used for IV infusion of the drugs.
Option B - Bone marrow aspiration: Bone marrow aspiration would typically involve a
salah needle, and the most common sites of bone marrow aspiration are the iliac crest and sternum.

Option D - Bone marrow biopsy: Bone marrow biopsy is done by Jamshidi needle, and the most comm
on sites are the iliac crest and sternum.

Solution for Question 22:


Correct Option C - E2V3M4:

Page 49

2017
The Glasgow Coma Scale (GCS) is used to assess the level of consciousness in a person following a
traumatic brain injury.
Eye opening
Verbal response
(Mnemonics: "ONE CONFUSED WORD SOUNDS NOWHERE)
Motor response (Best)
(Mnemonics: "OBEY LOCALITIES WITH FLEXION AND EXTENSION")
• Opens his eyes to a painful stimulus - E2,
• Speaks inappropriately - V3,
• Withdraws his limbs to a painful stimulus - M4.
Adding these scores together gives a GCS score of 9 (E2V3M4), which corresponds to option c.
Incorrect Options:
Options A, B, and C are incorrect. Refer to Option D for an explanation.

Solution for Question 23:


• The quick Sequential Organ Failure Assessment (qSOFA) score is used to assess the severity of
illness in potentially septic patients outside the intensive care unit. The qSOFA score consists of three
components: systolic blood pressure (SBP), respiratory rate (RR), and Glasgow Coma Scale (GCS)
score. Each component is scored as 1 or 0, with 1 indicating a higher risk. A qSOFA score of 2 or
greater suggests a high risk of poor outcomes, including longer hospital stay and higher in-hospital
mortality.
Incorrect Choices:
• Option a. While blood pressure and respiratory rate are part of the qSOFA score, complete blood
count (CBC) is not. Although leucocytosis with neutrophilia may suggest infection, it is not a specific
marker of sepsis severity.
• Option b. Creatinine and pulse rate (PR) are not part of the qSOFA score, even though renal function
(indicated by creatinine) and heart rate (indicated by pulse rate) can be affected in sepsis.
• Option c. Body temperature is not part of the qSOFA score. Although fever may suggest infection, it is
not a specific marker of sepsis severity.

Solution for Question 24:


Correct Option C - Hypotension + bradycardia:
• Hypotension + bradycardia. This option is the correct answer.
• Neurogenic shock often presents with hypotension due to systemic vasodilation. Additionally,
bradycardia (slow heart rate) can be observed as a result of the unopposed parasympathetic (vagal)
tone in the absence of sympathetic activity.

Page 50

2018
• The combination of hypotension and bradycardia is a hallmark of neurogenic shock.
• Neurogenic shock is a type of distributive shock that occurs as a result of a spinal cord injury or
severe damage to the autonomic nervous system.
• It is characterized by a loss of sympathetic nervous system activity, leading to vasodilation and a
decrease in systemic vascular resistance.
Incorrect Options:
Option A - Hypertension + tachycardia: This option is incorrect. Hypertension (high blood pressure) is n
ot typically associated with neurogenic shock. Instead, due to the loss of sympathetic tone and vasodil
ation, blood pressure tends to decrease.
Option B - Hypotension + tachycardia: This option is also incorrect. While tachycardia (rapid heart rate)
can occur in response to the body's compensatory mechanisms trying to maintain cardiac output, hypo
tension (low blood pressure) is the characteristic feature of neurogenic shock. The loss of sympathetic
tone results in vasodilation, which reduces systemic vascular resistance and leads to hypotension.
Option D - Hypertension + bradycardia: This option is incorrect. Hypertension is not typically seen in ne
urogenic shock. While bradycardia can occur, it is usually associated with other conditions such as incr
eased intracranial pressure or certain drug effects, rather than neurogenic shock.

Solution for Question 25:


Correct Option A - Systolic blood pressure <90 mmHg:
• Systolic blood pressure <90 mmHg, is not a specific criterion for SIRS. Instead, it is used to define
hypotension, which is an indicator of severe sepsis or septic shock.
Incorrect Options:
Option B - Temperature >38 degrees Celsius or <36 degrees Celsius: This criterion indicates abnormal
body temperature and suggests an inflammatory response. Fever (temperature >38 degrees Celsius)
or hypothermia (temperature <36 degrees Celsius) can be seen in patients with SIRS.
Option C - Respiratory rate >20 breaths per minute: An increased respiratory rate is a sign of respirator
y distress and can be an indicator of the body's response to an insult. Tachypnea (respiratory rate >20
breaths per minute) is commonly observed in patients with SIRS.
Option D - Heart rate >90 beats per minute: Tachycardia (heart rate >90 beats per minute) is a commo
n response to various stressors and is frequently seen in patients with SIRS. It reflects an increased sy
mpathetic response and an attempt to maintain cardiac output.

Solution for Question 26:


Correct Option A - If the hands are visibly soiled:
• If the hands are visibly soiled: This option is incorrect because hand rub is not suitable for visibly
soiled hands. In this case, handwashing with soap and water is necessary to remove visible dirt and
contaminants.
Incorrect Options:

Page 51

2019
Option B - While moving from a contaminated site to a clean site during patient care: This option is corr
ect. Hand rub during patient care is essential, especially when moving from a contaminated site to a
clean site. Hand rub helps to maintain hand hygiene and reduce the risk of transmitting pathogens.
Option C - Before donning gloves: This option is correct. Using hand rub before donning gloves is reco
mmended as an additional measure to maintain hand hygiene. This helps prevent contamination of glo
ves and reduces the risk of transmission.
Option D - During direct patient contact: This option is correct. Hand rub is commonly used before and
after direct patient contact to ensure hand hygiene and prevent the spread of pathogens.

Solution for Question 27:


Correct Option B - 1,2,3,4:
The correct sequence of management in a
patient who presents to the casualty with an RTA (road traffic accident) is as follows:
• Cervical spine stabilization: This is important to prevent further injury to the spinal cord and should be
prioritized in patients with suspected or potential cervical spine injuries.
• Intubation: If the patient is unconscious or showing signs of compromised airway or respiratory
distress, intubation should be performed to secure the airway and ensure adequate ventilation.
• IV cannulation: Intravenous access should be established to administer fluids, medications, and blood
products if necessary. It allows for rapid resuscitation and stabilization of the patient.
• CECT (Contrast-enhanced computed tomography): Once the patient is stabilized, a computed
tomography scan with contrast may be performed to assess the extent of injuries, identify any internal
bleeding or organ damage, and guide further management.

Solution for Question 28:


Correct Option B - Mid – clavicular line:
• Mid-clavicular line is not a boundary of the triangle of safety. The mid-clavicular line is a vertical line
passing through the midpoint of the clavicle (collarbone) and does not correspond to the boundaries of
the triangle.
Incorrect Options:
Option A - Lateral edge of pectoralis major: Correspond to the actual boundaries of the triangle of safet
y for ICD insertion.
Option C
- Base of axilla: Correspond to the actual boundaries of the triangle of safety for ICD insertion.
Option D - Lateral border of latissimus dorsi: Correspond to the actual boundaries of the triangle of saf
ety for ICD insertion.

Page 52

2020
Solution for Question 29:
Correct Option C - Laparotomy:
• In a patient with a pelvic fracture, the presence of a bladder rupture seen on a CT scan requires
immediate surgical intervention, which is accomplished through a laparotomy.
• Bladder rupture can occur as a result of the pelvic fracture, and it is important to address this
complication promptly to prevent further complications such as peritonitis, sepsis, and urinary ascites.
Incorrect Options:
Options A, B, and D are incorrect.

Solution for Question 30:


Correct Option D - Vascular anastomosis:
• Vascular anastomosis, which refers to the surgical joining of blood vessels, is not typically performed
as a part of damage control surgery. In this acute and unstable setting, the focus is on initial
resuscitation, control of hemorrhage, and stabilization of the patient. Definitive vascular repairs or
anastomoses are usually performed during subsequent surgeries, once the patient is in a more stable
condition.
Incorrect Options:
Option A - Control of hemorrhage: This is an essential part of damage control surgery. The initial priorit
y is to control any active bleeding to prevent further blood loss and stabilize the patient's condition. Thi
s may involve techniques such as direct pressure, ligation, or the use of hemostatic agents.
Option B - Resuscitation: Resuscitation is a critical component of damage control surgery. It involves th
e rapid assessment and stabilization of the patient's vital signs, oxygenation, and circulation. Intraveno
us fluids, blood products, and other supportive measures are administered to restore and maintain ade
quate perfusion to vital organs.
Option C - Decontamination: Decontamination refers to the removal of contaminants or foreign material
from the surgical site, particularly in cases involving traumatic injuries or contaminated wounds. It is an
important step to minimize the risk of infection and complications. Decontamination may involve irrigati
on, cleaning, and removal of foreign bodies.

Solution for Question 31:


Correct Option B - Hypovolaemic shock—most common type:
• This statement is correct.
• Hemorrhage may be due to injury to the liver, spleen, bone fractures, haemothorax, vascular injury, or
severe bleeding on the table during surgeries of the thyroid, liver, portal vein or major vessels.
• Vomiting, diarrhea due to any cause.
• Burns.
Incorrect Options:

Page 53

2021
Option A - Anaerobic respiration produces carbon dioxide: This statement is incorrect. Anaerobic respir
ation occurs in the absence of oxygen and results in the production of lactic acid, not carbon dioxide. D
uring shock, inadequate oxygen delivery to tissues can lead to anaerobic metabolism and the accumul
ation of lactic acid, contributing to metabolic acidosis.
Option C - Stage of compensatory shock is also called stage 2 of shock : This statement is incorrect. St
age of compensatory shock by neuroendocrine response to maintain the perfusion of the vital organs li
ke brain, heart, kidney, liver.
Option D - Anaphylactic shock is due to Type 2
hypersensitivity reaction: This statement is incorrect. Anaphylactic shock—is due to Type 1
hypersensitivity reaction

Solution for Question 32:


Correct Option C. Pneumothorax:
• The procedure shown in the image is central line placement. Pneumothorax is a potential
complication associated with central line placement. It occurs when air enters the pleural space,
causing partial or complete collapse of the lung. This can happen during the insertion of the central line
needle or catheter into the subclavian, jugular, or femoral vein. Pneumothorax can lead to respiratory
distress and requires immediate intervention, such as chest tube insertion, to remove the trapped air
and re-expand the lung.
Incorrect options:
Option A: Aspiration
• Aspiration refers to the inhalation of stomach contents or foreign material into the lungs. While
aspiration can occur in various clinical settings, it is not typically associated with the placement of a
central line.
• Complications related to central line placement are more directly related to the procedure itself and
the anatomical structures involved.
Option B. Torticollis
• Torticollis, also known as wry neck, is a condition characterized by involuntary contraction of neck
muscles, leading to the neck being twisted to one side. It is not a complication directly related to central
line placement.
Option D. Recurrent laryngeal nerve paralysis
• Recurrent laryngeal nerve paralysis is a complication associated with surgeries or procedures
involving the neck or chest, such as thyroid surgery or thoracic surgery. It results from damage to the
recurrent laryngeal nerve, leading to vocal cord paralysis and potential airway obstruction. While it can
occur as a complication of procedures near the neck or chest, it is not commonly associated with
central line placement.

Solution for Question 33:


Correct Option D: Hypernatremia

Page 54

2022
• Hypernatremia: Hypernatremia, which is an elevated serum sodium level, is not a recognized
complication of massive blood transfusion. It is more commonly associated with conditions such as
dehydration or excessive sodium intake.
• Massive blood transfusion refers to the transfusion of a large volume of blood, usually more than one
blood volume within 24 hours. It is associated with several complications due to various factors involved
in the transfusion process.
Incorrect choices:
Option A. Hypocalcemia: Massive blood transfusion can lead to hypocalcemia due to the citrate antico
agulant present in the stored blood. Citrate binds to calcium, resulting in a
decrease in ionized calcium levels.
Option B. Hypokalemia: Due to transfusion of citrated cells potassium level decreases but initially hype
rkalemia occurs due to lysis of red blood cells.
Option C. Hypothermia: Large volumes of cold blood products can cause hypothermia in the recipient.
This occurs due to the temperature difference between the transfused blood and the patient's body tem
perature.

Solution for Question 34:


Correct Option A: Mixter
• The correct option, Mixter forceps, is the accurate identification of the surgical instrument depicted in
the image. Mixter forceps are commonly used in surgical procedures for grasping and manipulating
delicate tissues. Mixter forceps are commonly used for hemostasis during surgical procedures.They
have a distinct curved shape and serrated jaws with cross-grooves, which provide a secure grip on
tissues during surgical manipulation.
Incorrect options:
Option B: Rampley forceps: Rampley forceps, also called dressing forceps, are used for holding and m
anipulating dressings, sutures, and other surgical materials. They have a
straight design with cross-serrated tips that provide a secure grip.

Page 55

2023
Option C: Kocher forceps: Kocher forceps, also known as artery forceps or hemostats, are used for cla
mping blood vessels or tissue during surgical procedures. They have a
straight design with longitudinal serrations that provide a strong grip.

Option D: Adson forceps: Adson forceps, also called thumb forceps, are commonly used in surgical pro
cedures. They have a
delicate design with fine, serrated tips that allow for precise grasping and manipulation of tissues.

Solution for Question 35:


Correct Option D: 50,000 (109/L)
• Platelets are essential for normal blood clotting, and maintaining an adequate platelet count is crucial
during surgical procedures to prevent excessive bleeding and promote proper wound healing. While the
normal reference range for platelet count is typically between 150,000 and 400,000 (10^9/L), a

Page 56

2024
threshold of 50,000 (10^9/L) is generally accepted as the minimum platelet count required for most
surgical interventions.
Incorrect options:
Option A: 20,000 (10^9/L): This platelet count is significantly lower than the baseline requirement for su
rgery, and it would generally be considered inadequate. Patients with platelet counts this low are at a
high risk of bleeding complications during surgery.
Option B: 30,000 (10^9/L): This platelet count is still below the recommended threshold of 50,000 (10^
9/L) for surgery. While it may be sufficient for some minor procedures, it may not be ideal for more exte
nsive surgeries or in patients with additional bleeding risks.
Option C: 40,000 (10^9/L): This platelet count is closer to the baseline requirement, but it is still below t
he generally accepted threshold of 50,000 (10^9/L) for surgery. The decision to proceed with surgery a
t this platelet count may depend on various factors, including the individual patient's clinical condition a
nd the type of surgery being performed.

Solution for Question 36:


Correct Option A:
• Based on the information provided, the best investigation of choice in this scenario would be A.
Contrast-enhanced CT scan.
• A CT scan is a non-invasive imaging modality that can provide detailed information about the
abdominal organs, including the presence of any injuries or abnormalities. In the case of a patient with
pain abdomen after a road traffic accident, a CT scan can help identify any intra-abdominal injuries,
such as organ damage, bleeding, or fractures.
• Contrast-enhanced CT scan is particularly useful as it allows for better visualization of blood vessels
and can help detect subtle injuries that may not be evident on a regular CT scan. It can provide
important information to guide further management decisions, including the need for surgical
intervention.
Incorrect Options:
Option B: Retrograde urethrogram is a specialized imaging study that is typically performed when there
is suspicion of urethral injury, such as in cases of pelvic fractures. It may not be the most appropriate i
nitial investigation in a patient with pain in the abdomen and left lumbar tenderness.
Option C: Wait and watch (C) may be an option in certain cases where the patient is stable and there is
no immediate concern for significant intra-abdominal injury. However, given the presentation of pain a
bdomen and the potential for underlying injuries after a road traffic accident, it is important to perform a
n appropriate investigation to rule out any serious pathology.
Option D: Emergency laparotomy (D) would be an invasive surgical intervention and should be reserve
d for cases where there is strong evidence of a life-threatening condition that requires immediate surgic
al intervention, such as active bleeding or peritonitis. It is not the first-line investigation in this scenario.

Solution for Question 37:


Correct Option B: Wait and watch

Page 57

2025
• In the case of blood at the tip of the urethral meatus following a road traffic accident, the management
will depend on the specific circumstances and the severity of the injury. So Wait and watch. Here are
some general considerations:
• Assess the patient's stability: Evaluate the patient's vital signs and overall clinical condition. If the
patient is unstable or has signs of significant trauma or associated injuries, immediate resuscitation and
transfer to a trauma center may be necessary.
• Control bleeding: If there is active bleeding from the urethral meatus, gentle compression with sterile
gauze or a clean cloth can be applied to help control the bleeding. Avoid inserting anything into the
urethra, as it may worsen the injury or cause further bleeding.
Incorrect Options:
Option A: Foley's catheterization - Urinary catheterization: Catheterization should be approached with c
aution in cases of suspected urethral injury. If the patient has signs of urinary obstruction (inability to pa
ss urine, significant urinary retention), difficulty urinating, or inability to tolerate bladder filling, consider
consulting a urologist for evaluation and guidance on catheterization.
Option C: Nephrogram - It is not the next best step of management. A nephrogram is a
radiographic imaging study that focuses on visualizing the kidneys. It involves the injection of a contras
t agent into the bloodstream, which helps to highlight the renal vasculature and renal parenchyma.
Option D: Suprapubic cystostomy -
It is not the next best step of management. Suprapubic cystostomy is a
surgical procedure that involves creating a small opening in the lower abdomen to access the urinary bl
adder. This procedure is typically performed when there is an obstruction or injury to the urethra, makin
g it difficult or impossible to pass urine through the natural pathway.

Solution for Question 38:


Correct option:
Option C: NCCT (Non-Contrast CT)
• NCCT: Non-Contrast Computed Tomography scans are widely used in the emergency department to
evaluate head injuries.
• CT scans provide detailed images of the skull, brain, and surrounding structures, allowing for the
detection of skull fractures, intracranial bleeding (hemorrhage), and other traumatic brain injuries.
• CT scans are fast, readily available, and can quickly identify urgent or life-threatening conditions.
Incorrect Options:
Option A: CECT: Contrast-Enhanced Computed Tomography (CECT) involves injecting a
contrast agent into the patient's bloodstream before performing a CT scan. While CECT can be helpful
in certain situations, such as evaluating vascular abnormalities or tumors, it is not the initial investigatio
n of choice for a head injury.
Option B: MRI: Magnetic Resonance Imaging (MRI) uses magnetic fields and radio waves to produce d
etailed images of the brain. MRI is particularly useful in evaluating soft tissue injuries, such as brain co
ntusions or diffuse axonal injury. However, MRI scans take longer to perform and may not be readily av
ailable in emergency settings. In the acute phase of a
head injury, an NCCT scan is usually the preferred initial investigation.

Page 58

2026
Option D: X-ray: While X-ray can be useful in certain scenarios, it is not the investigation of choice for e
valuating head injuries. X-ray imaging primarily visualizes bony structures and may not provide sufficie
nt information about the soft tissues and intracranial structures affected by head trauma.

Solution for Question 39:


Correct option:
Option A: Fat embolism
• In the given scenario the probable diagnosis is fat embolism.
• Fat embolism occurs when fat globules enter the bloodstream and block small blood vessels in
various organs, including the lungs.
• It commonly occurs after long bone fractures, such as femur fractures.
• The fat globules can travel to the lungs and cause respiratory distress, pulmonary infiltration (seen on
imaging), and other systemic manifestations.
Incorrect options:
Option B: Pulmonary embolism: Pulmonary embolism refers to the blockage of the pulmonary arteries
by blood clots that usually originate from the deep veins of the legs (deep vein thrombosis). While pulm
onary embolism can cause respiratory distress, it is not directly associated with femur fractures unless
there is concurrent deep vein thrombosis.
Option C: Air embolism: Air embolism occurs when air bubbles enter the bloodstream and obstruct blo
od flow. It is usually associated with invasive medical procedures, such as central venous catheter plac
ement or certain surgical procedures. While air embolism can cause respiratory distress, it is less likely
to be associated with a femur fracture.
Option D: Obstruction: The term "obstruction" is broad and does not specify a particular condition. With
out further information, it is challenging to determine the specific diagnosis related to obstruction in this
scenario.

Solution for Question 40:


Correct Option:
Option B: Check BP is the correct option.
• In an accident case, the early management after the medical team's arrival involves various critical
steps to ensure the patient's stabilization and assessment
• While monitoring blood pressure is crucial in the overall management of trauma patients, it is not
typically considered an immediate priority in the early stages of patient care.
• Other vital assessments and interventions take precedence during the initial management.

Incorrect options:
Option A: Stabilization of cervical vertebrae is an essential step in managing trauma patients, especiall
y those involved in accidents. It involves immobilizing the neck and spine to prevent further injury and p

Page 59

2027
otential damage to the spinal cord.
Option C: Check respiration is an important step in the initial management of trauma patients. Assessin
g the patient's breathing pattern, rate, and adequacy is crucial to identify any potential airway comprom
ise or respiratory distress.
Option D: GCS score refers to the Glasgow Coma Scale, which is used to assess the level of consciou
sness and neurological status of the patient. Evaluating the patient's GCS score helps determine the s
everity of brain injury and guides subsequent management decisions.

Solution for Question 41:


Correct option:
Option D: Port wine stain
• The condition shown in the image is a port wine stain.
• Port wine stain, also known as capillary malformation or nevus flammeus, is a type of vascular
birthmark that appears as a pink, red, or purple discoloration on the skin.
• It is typically present at birth and persists throughout life.
• Port wine stains are caused by abnormal development of blood vessels near the surface of the skin.
• They can occur anywhere on the body but are most commonly found on the face, neck, or upper
extremities.
Incorrect Options:
Option A: Strawberry stain: A strawberry stain, also known as a
strawberry hemangioma, is another type of vascular birthmark. It appears as a raised, red, or purplish l
esion on the skin and is typically present at birth or appears within the first few weeks of life. Unlike a p
ort wine stain, strawberry stains tend to grow rapidly in the first months of life and then gradually fade o
ver time.
Option B: Erythema multiforme: Erythema multiforme is a skin condition characterized by the presence
of distinctive target-like lesions on the skin. These lesions have a central red spot surrounded by a pale
ring and an outer ring of redness. Erythema multiforme can be triggered by various factors, including i
nfections, medications, and allergic reactions.

Page 60

2028
Option C: Erythema subitum, also known as roseola infantum or sixth disease, is a
viral infection commonly seen in infants and young children. It is characterized by a
sudden onset of high fever followed by the appearance of a rash.

Solution for Question 42:


Correct option:
Option B: Liquefactive
• In the context of brain tissue, the type of necrosis seen is typically liquefactive necrosis.
• Liquefactive necrosis is the predominant type of necrosis observed in the brain due to the presence of
abundant water content and the breakdown of tissue by enzymes, resulting in the formation of cystic
spaces filled with liquid debris.
• This pattern of necrosis is often seen in conditions such as cerebral infarction or brain abscesses.
Incorrect options:
Option A: Coagulative necrosis: Coagulative necrosis is characterized by the preservation of tissue arc
hitecture but with loss of cellular details. It is typically seen in organs such as the heart, kidney, and live
r. However, in the brain, the high water content and the presence of enzymes lead to liquefactive necro
sis instead of coagulative necrosis.
Option C: Fat necrosis: Fat necrosis occurs when there is damage to adipose tissue, resulting in the rel
ease of fatty acids and subsequent inflammation. It is commonly seen in conditions like acute pancreati
tis or trauma to adipose-rich areas. Fat necrosis is not a primary type of necrosis seen in brain tissue.
Option D: Fibrinoid necrosis: Fibrinoid necrosis is characterized by the deposition of fibrin-like material i
n the walls of blood vessels. It is typically associated with immune-mediated reactions, such as in certa
in types of vasculitis. Fibrinoid necrosis is not commonly observed in brain tissue.

Page 61

2029
Solution for Question 43:
Correct option A - eFAST:
• eFAST (extended Focused Assessment with Sonography for Trauma) is a rapid bedside ultrasound
examination that assesses the chest and abdomen to identify potential life-threatening injuries in
trauma patients.
• It involves the evaluation of the pericardium, pleural spaces and abdominal cavity for the presence of
air or fluid.
• In this case, the patient has clear lung fields on chest radiography, suggesting that a pneumothorax or
hemothorax may not be the immediate cause of the hypotension.
• Therefore, performing an eFAST examination can help identify other possible sources of bleeding or
injury, such as intraperitoneal bleeding, which may guide further management.
Incorrect options:
Option B - Tube thoracostomy:
• Tube thoracostomy, the insertion of a chest tube into the pleural space, is commonly performed to
treat a pneumothorax or hemothorax.
• However, since the chest radiography showed clear lung fields, indicating no visible abnormalities in
the pleural space, a tube thoracostomy may not be the immediate next step in this case.
Option C and D
• CECT chest and abdomen is a valuable imaging modality to evaluate the chest in trauma patients.
• However, an immediate imaging study like CECT may cause delays in identifying and addressing the
underlying cause.
• It can be done later on for a detailed view .

Solution for Question 44:


Correct Option D.
• ASEPSIS wound scoring is a tool used to assess the severity of wound infections and monitor wound
healing progress.
• The parameters considered in ASEPSIS wound scoring does not include; induration
• Induration refers to the hardening or thickening of the skin around the wound site. It is often
associated with inflammation and can be a sign of infection or tissue damage.
• However, ASEPSIS wound scoring does not specifically include induration as a parameter.
Incorrect options;
Option A. Wound swab from the site: This parameter assesses the presence of wound infection by coll
ecting a swab from the wound site and identifying the causative microorganisms.
Option B. Serous discharge: Serous discharge refers to the clear, watery fluid that may be present in a
wound. Its presence can indicate a well-healing wound or an early sign of infection.

Page 62

2030
Option C. Erythema: Erythema refers to redness or inflammation around the wound site. It is a
common sign of inflammation and can indicate the presence of infection or poor wound healing.

Solution for Question 45:


Correct Option C.
• Based on the pulsatile nature, tortuous swelling, and presence of a bruit, the most likely diagnosis for
the patient's condition is a cirsoid aneurysm.
• A cirsoid aneurysm is a congenital or acquired arteriovenous malformation characterized by an
abnormal tangle of dilated blood vessels.
• It often presents as a pulsatile, tortuous swelling with a bruit due to the high blood flow within the
abnormal vessels.
• The image and clinical features described in the case are consistent with a cirsoid aneurysm.
Incorrect options:
Option A. Neurofibromatosis: Neurofibromatosis is a genetic disorder that causes the growth of benign
tumors on nerves. It typically presents as multiple soft nodules or skin-colored bumps on or under the s
kin. It is not associated with a pulsatile tortuous swelling with a
bruit like the patient in the case presented.
Option B. Liposarcoma: Liposarcoma is a
malignant tumor that arises from fat cells. It usually presents as a
painless, slowly growing mass. It is not typically associated with a pulsatile tortuous swelling with a
bruit, as seen in the patient.
Option D. Varicocele: A varicocele is a condition characterized by the enlargement of veins within the s
crotum, typically occurring on the left side. It is not relevant to the case presented, as the description m
entions a tortuous swelling over the left side of the scalp and face, not the scrotum.

Solution for Question 46:


Correct Option C.
• Taking a CT scan of the patient is not a part of the primary survey.
• The primary survey prioritizes immediate life-saving interventions and assessments, such as
maintaining a patent airway, ensuring breathing, assessing circulation, and identifying visible injuries
• Diagnostic imaging studies like CT scans are typically performed at a later stage to further evaluate
specific conditions or injuries.
Incorrect options;
Option A. Measuring blood pressure: Measuring blood pressure is an essential component of the prima
ry survey. It helps assess the patient's circulatory status and identify signs of shock or hypoperfusion.
Option B. Removing clothes to inspect for wounds: This step is an important part of the primary survey.
It allows healthcare providers to visually examine the patient's body for any visible injuries, wounds, or
bleeding that require immediate attention.

Page 63

2031
Option D. Checking airway patency: Checking airway patency is a critical step in the primary survey. It i
nvolves assessing the patient's ability to breathe and identifying any obstructions or compromises to th
e airway. Ensuring a patent airway is crucial for adequate oxygenation and ventilation.

Solution for Question 47:


Correct option:
Option C: Necrotizing fasciitis
• The probable diagnosis based on the given clinical condition where patient is diabetic and the image
is suggestive of necrotizing fasciitis
• The image suggestive of necrotizing fasciitis supports this diagnosis due to the presence of extensive
tissue necrosis and involvement.
• Necrotizing fasciitis: Necrotizing fasciitis is a rapidly progressive soft tissue infection that involves the
deep fascial layers, causing necrosis of the affected tissues.
• It is characterized by severe pain, erythema (redness), swelling, blistering, and signs of systemic
toxicity.
Incorrect options:
Option A: Diabetic foot: Diabetic foot refers to a range of foot complications that can occur in individual
s with diabetes. While diabetic foot ulcers and infections are common, the given clinical scenario and th
e image provided are more indicative of a
severe soft tissue infection like necrotizing fasciitis rather than a typical diabetic foot ulcer.
Option B: Burn: Burns are caused by thermal, chemical, electrical, or radiation injury. While walking bar
efoot on hot sand can result in burns, the clinical presentation and the image showing extensive tissue
involvement, necrosis, and signs of infection are not consistent with a simple burn injury.
Option D: Cellulitis: Cellulitis is a common bacterial skin infection characterized by redness, warmth, s
welling, and tenderness of the affected area. While cellulitis is a possibility in a diabetic patient who wal
ked barefoot, the image provided with extensive tissue necrosis is more indicative of necrotizing fasciiti
s, which is a more severe and rapidly progressing infection.

Solution for Question 48:


Correct Option C.
• Exploratory laparotomy indicated in patients with a positive FAST assessment to avoid further
bleeding in the abdomen.
• BP is 88/60 mm hg suggesting that the patient is not hemodynamically stable, therefore the next best
step in management will be exploratory laparotomy.
Blunt trauma management flowchart
• (BTA- Blunt Trauma Abdomen)

Page 64

2032
Incorrect Options
Option A: Abdominal CT scan: As the patient is not hemodynamically stable so abdominal CT will not b
e the first choice.
Option B: Abdominal MRI scan: MRIs are usually preferred for investigation of Soft tissue pathology, b
one pathology, and head trauma.
Option D: Diagnostic peritoneal lavage is indicated to detect covert intra-abdominal injury.

Solution for Question 49:


Correct Option A.
• Hypovolemic shock is circulatory failure due to loss of intravascular volume by variceal bleed.
• This loss of intravascular volume loss will lead to tissue hypoperfusion and tissue hypoxia.
• In cirrhosis, the blood flow to the liver is cut down and leads to more flow of blood through the veins of
the esophagus leading to hemorrhage.
Incorrect Options
Option B: Cardiogenic shock: ·
Characterized by impaired ability of heart to pump blood (especially by the left ventricle)
Option C: Obstructive shock: Condition wherein blood and oxygen are failed to reach the organs. Exam
ples include blood clots, collapsed lungs, tension pneumothorax, and pericardial tamponade.
Option D: Distributive shock: This leads to poor tissue perfusion due to excessive vasodilation and poo
r distribution of blood flow to the heart, brain, and kidneys.

Solution for Question 50:

Page 65

2033
Correct Option C.
The gauge size of the cannula shown in the image is 18.
Incorrect Options:
Option A. "14," is incorrect because the image shows a cannula with a larger gauge size.
Option B. "16," is incorrect because the image shows a cannula with a larger gauge size.
Option D. "20," is incorrect because the image shows a cannula with a smaller gauge size.

Solution for Question 51:


Correct Option B.
• In this case, the patient has a significant arterial injury with a longitudinal tear in the brachial artery
and the absence of radial and ulnar pulses.
• This indicates compromised blood flow to the distal part of the arm. Prompt surgical intervention is
necessary to restore blood flow and prevent ischemic complications.
• Repair with a great saphenous vein graft is a suitable treatment option for arterial repair in this
scenario.
• The great saphenous vein can be harvested from the patient's lower extremity and used as a graft to
bridge the defect in the brachial artery. This technique allows for reestablishment of blood flow and
preservation of the distal circulation.
Incorrect Options:
Option A. Primary repair with end-to-end anastomosis: Primary repair with end-to-end anastomosis inv
olves directly suturing the ends of the torn artery together to restore continuity. However, in this case, t
he longitudinal tear in the brachial artery is 3.5cm, which is a significant defect. Primary repair alone m
ay not be feasible or successful in achieving adequate vessel approximation and blood flow restoration
. Therefore, option A is not the best treatment option.
Option C. Repair with prosthetic graft: Repair with a prosthetic graft involves using a synthetic graft mat
erial to bridge the defect in the artery. While prosthetic grafts can be used for arterial repairs, they are t
ypically reserved for larger vessels or when autologous vein grafts are not available or suitable. In this
case, with a 3.5cm tear in the brachial artery, a prosthetic graft may not be the optimal choice, and a
vein graft would be a better option.
Option D. Repair with saphenous vein patch: Repair with a saphenous vein patch involves using a
segment of the saphenous vein to create a patch to cover the arterial defect. However, in this case, the
tear in the brachial artery is longitudinal and 3.5cm in length, which may require a
more substantial vascular reconstruction with a graft rather than a patch. Therefore, option D
is not the best treatment option.

Solution for Question 52:


Correct Option B.
• Chest tube insertion is generally contraindicated in patients with isolated rib fractures without
associated pneumothorax (collapsed lung) or hemothorax (blood in the pleural cavity).

Page 66

2034
• Given chest xray is suggestive of diaphragmatic hernia following blunt injury due to trauma, therefore
chest tube insertion becomes a contraindication in this case
• Chest tube insertion is primarily performed to drain air or fluid from the pleural space when there is a
pneumothorax or hemothorax.
Incorrect Options:
Option A. Nasogastric tube insertion: Nasogastric tube insertion is not directly contraindicated in a
patient with diaphragmatic hernia. Nasogastric tube insertion is a common procedure performed in em
ergency departments to decompress the stomach and remove gastric contents..
Option C. Log roll: A log roll is a
manoeuvre used to maintain spinal immobilisation when turning or repositioning a
trauma patient. It involves rolling the patient as a unit while keeping the spine in a neutral position. Log
roll precautions are generally recommended in trauma patients to minimise movement of the spine and
reduce the risk of spinal cord injury.
Option D. Epidural anaesthesia: Epidural anesthesia is a regional anesthesia technique commonly use
d for pain management during surgical procedures. Diaphragmatic hernia is not directly contraindicatio
n for epidural anesthesia.

Solution for Question 53:


Correct Option C.
• To calculate the maximum flow rate of fluid with a specific IV cannula, we need to consider the size of
the cannula, which is given as 18G (green).
• For an 18g cannula, the flow rate is 90 ml/hr.
• Among the given answer options, the closest value to the generally recommended maximum flow rate
for an 18g cannula is c. 90ml/min.

Incorrect Options
Option A. 86 ml/min: This option suggests a flow rate of 86 ml per minute. While this value is within a
reasonable range for IV fluid administration, it is lower than the commonly flow rate for an 18g cannula.
Option B. 96 ml/min: This option suggests a
flow rate of 96 ml per minute. Similar to option A, this value is lower than the generally flow rate for an
18g cannula. It is important to ensure an adequate flow rate to meet the patient's fluid needs, especiall
y in cases of significant fluid loss.
Option D. 63 ml/min: This option suggests a flow rate of 63 ml per minute. This value is significantly lo
wer than the generally recommended maximum flow rate for an 18g cannula. It may not provide suffici
ent fluid replacement in a patient with a significant fluid deficit.

Solution for Question 54:


• Triage is the process of classifying patients based on the severity of their condition and the priority of
treatment required.

Page 67

2035
• It is commonly used in emergencies or mass casualty incidents with limited resources.
• Triage helps in efficiently allocating resources by prioritising patients based on the urgency of their
condition.
Incorrect Choices:
• Option a. "To prepare for a disaster," is incorrect because the purpose of triage is not to prepare for a
disaster but rather to assess and prioritise patients during a disaster or emergency situation.
• Option b. "To assess the impact of a disaster," is incorrect because while triage is a part of disaster
response, its main objective is not to assess the overall impact of a disaster but to categorise patients
based on the severity of their injuries or illnesses.
• Option d. "To rehabilitate following a disaster," is incorrect because triage is not involved in the
rehabilitation process after a disaster. Triage focuses on initial assessment and prioritisation of patients
during the acute phase of a disaster

Solution for Question 55:


Answer option D -
• The Glasgow Coma Scale (GCS) is a neurological assessment tool used to evaluate a person's level
of consciousness and neurological functioning after a head injury or other neurological conditions.
• The GCS is commonly used in emergency departments, intensive care units, and other medical
settings. The Glasgow Coma Scale (GCS) score for the patient described is Eyes-3, Verbal-1, and
Motor-6.

Incorrect Choices:
Option a. "Eyes-2, Verbal-1, Motor-5," is incorrect
Option b. "Eyes-2, Verbal-NT, Motor-5," is incorrect
Option c. "Eyes-3, Verbal-1, Motor-6," is incorrect

Page 68

2036
Solution for Question 56:
• When using the interrupted suturing technique, the needle should be placed perpendicular to the skin
surface, creating a 90-degree angle.
• This angle allows for proper penetration and closure of the wound edges.
• By inserting the needle at a right angle to the skin, the sutures can adequately hold the tissues
together and provide optimal wound closure and healing.
Incorrect Choices:
• Option a. 60 degrees: A 60-degree angle would be too shallow for proper needle placement during
interrupted suturing. Such a low angle might result in inadequate tissue apposition and compromised
wound closure.
• Option b. 70 degrees: A 70-degree angle is also too shallow for interrupted suturing. Again, this angle
would not provide sufficient tissue penetration and secure wound closure. It is important to aim for a
perpendicular angle to achieve optimal wound healing.
• Option c. 80 degrees: An 80-degree angle is closer to the correct angle of 90 degrees but is still not
ideal for interrupted suturing. Although it is slightly better than the previous options, it may not provide
the same level of tissue apposition and secure wound closure as a 90-degree angle would

Solution for Question 57:


• Hollow viscus perforation refers to the rupture or perforation of a hollow organ within the abdominal
cavity.
• In this case, the patient's symptoms of abdominal pain and obstipation, along with a history of
abdominal trauma from a bull gore, raise suspicion for a hollow viscus injury.
• The X-ray suggestive of perforation, such as free air under the diaphragm
• Hollow viscus perforation can occur in various organs within the abdomen, including the stomach,
small intestine, large intestine, or other hollow structures.
• The severity of symptoms can vary depending on the location and extent of the perforation.
• It is considered a surgical emergency and requires immediate intervention to repair the perforation
and address any associated complications.
Incorrect Choices:
• Option a. Hemothorax: Hemothorax refers to the accumulation of blood in the pleural cavity, typically
caused by trauma or injury to the chest or lung. While the patient experienced trauma from a bull gore,
the symptoms described (abdominal pain and obstipation) are not consistent with hemothorax.
Hemothorax would typically present with respiratory symptoms such as chest pain, difficulty breathing,
and decreased breath sounds on examination.

Page 69

2037
• Option c. Pneumothorax: Pneumothorax is the presence of air in the pleural cavity, resulting in lung
collapse. While trauma can cause pneumothorax, the patient's symptoms of abdominal pain and
obstipation do not correlate with this diagnosis. Pneumothorax would typically present with respiratory
symptoms, such as sudden-onset chest pain, dyspnea, decreased breath sounds, and possible signs
of respiratory distress.
• Option d. Intestinal obstruction: Intestinal obstruction refers to the blockage of the normal flow of
contents through the intestines. While the patient presents with abdominal pain and obstipation, which
can be seen in intestinal obstruction, the history of bull gore and the X-ray findings of perforation
suggest a more specific diagnosis of hollow viscus perforation. Intestinal obstruction would not typically
be associated with a history of trauma and evidence of perforation on imaging.

Solution for Question 58:


• The correct method for measuring a nasogastric tube involves measuring the distance from the tip of
the patient's nose to the ear lobule and then down to the xiphisternum, which is the lower end of the
sternum (breastbone). This measurement helps determine the appropriate length of the nasogastric
tube that needs to be inserted.
Incorrect Choices:
• Option b. Angle of mouth to tragus to xiphisternum: It refers to measuring from the angle of the mouth
(corner of the mouth) to the tragus (a small pointed projection in front of the external ear) and down to
the xiphisternum.
• Option c. Ala of nose to angle of mandible to xiphisternum: It suggests measuring from the ala of the
nose (wing of the nose) to the angle of the mandible (corner of the lower jaw) and down to the
xiphisternum.
• Option d. Angle of mouth to ear lobule to umbilicus: This option is incorrect because it involves
measuring from the angle of the mouth to the ear lobule and extending the measurement all the way
down to the umbilicus (belly button).

Solution for Question 59:


Correct C: Extradural hematoma
• The common radiological finding in the case of an extradural hematoma is a characteristic biconvex or
lens-shaped hyperdense lesion in the temporal region. This description is similar to the findings given in
the CT scan image.
Incorrect Choices:
Option A. Subdural hematoma: commonly seen due to injury in cortical bridging veins. In the CT scan t
he lesion is seen as a
concave convex lesion in the subdural space. This is not seen in the given CT image.
Option B. Intracerebral haemorrhage: Commonly seen due to rupture of an aneurysm inside the brain.
The CT scan usually shows a well-defined hyperdensity and is usually surrounded by hypodense oede

Page 70

2038
ma. This is not seen in the given CT image.
Option D. Hemorrhagic contusion: Usually seen as small punctate hyperdense foci in the CT. This is n
ot seen in the given image.

Solution for Question 60:


Correct choice: B
• The above condition indicates a tension pneumothorax. It is a life-threatening condition and should be
immediately attended to. The appropriate management of a tension pneumothorax includes inserting a
Wide bore needle insertion in the 5th ICS along axillary line followed by an intercostal drain in the 5th
ICS to achieve decompression. Hence option B will be the correct management for the case given.
Incorrect choices:
Option A. Wide bore needle insertion in the 2nd ICS: The wide bore needle is to be inserted into the
ICS 5th at the mid-clavicular line. Hence this choice is incorrect.
Option C. Aspiration of the Pleural fluid: Aspiration of the pleural fluid is not done in case of a
tension pneumothorax. A Wide bore needle insertion in the 2nd ICS followed by a
chest tube at the 5th ICS to relieve decompression is the correct management.
Option D. No intervention needed: A tension pneumothorax is a
highly life-threatening condition and needs immediate intervention.

Solution for Question 61:


Correct Option D: 8
• Eye opening response: The patient opens his eyes in response to painful stimuli. This corresponds to
a score of 2 on the GCS.
• Best motor response which is scored as 4, as he can localise pain..
• Verbal response: is 2 , as he is making Incomprehensible sounds
• To calculate the GCS, we sum up the scores = 8
Incorrect Options
Option A: GCS of 9- This option is incorrect
Option B: GCS of 15- This option would be incorrect
Option C: GCS of 10- This option would also be incorrect

Solution for Question 62:


Correct Option C: The surgeon must use the single-layer, extramucosal suture technique
• The surgeon must use the single-layer, extramucosal suture technique for resection and anastomosis
of the necrotic bowel.

Page 71

2039
• The single-layer, extramucosal suture technique is the preferred method for anastomosis in cases of
necrotic bowel.
• This technique involves suturing only the outermost layers of the bowel (seromuscular layer) without
including the inner mucosal layer.
• By avoiding suturing the submucosa, the risk of leakage from compromised or ischemic mucosa is
reduced.
Incorrect Options:
Option A: The surgeon must use chromic catgut- Chromic catgut is a
type of absorbable suture material.
Option B: The surgeon must not include the submucosa in the sutures- The single-layer, extramucosal
suture technique involves suturing the outer seromuscular layer without including the inner mucosal lay
er. The submucosa is typically not included in the sutures to minimize the risk of leakage and promote
healing.
Option D: The surgeon must use the single-layer seromuscular suture technique: This statement is inc
orrect because the single-layer seromuscular suture technique does not specifically address the extra
mucosal aspect. The correct technique for anastomosis in cases of necrotic bowel is the single-layer, e
xtramucosal suture technique.

Solution for Question 63:


Correct Option B: Great saphenous vein
• The great saphenous vein, is considered the best graft for a femoropopliteal bypass due to its superior
long-term patency rates and overall success in these procedures.
Incorrect Options:
Option A: Dacron, also known as polyethylene terephthalate, is a synthetic graft material commonly us
ed in vascular surgery. While Dacron grafts can be used for femoropopliteal bypass, they tend to have l
ower patency rates compared to the great saphenous vein. Therefore, they are generally considered a
second-line option when autologous vein grafts are unavailable or unsuitable.
Option C: Polytetrafluoroethylene (PTFE)- Polytetrafluoroethylene, commonly known as PTFE or Teflo
n, is another synthetic graft material used in vascular surgery. PTFE grafts have been utilized for femor
opopliteal bypass when autologous vein grafts are not available. However, they have generally shown i
nferior long-term patency rates compared to the great saphenous vein graft.
Option D: Short saphenous vein- The short saphenous vein is a
vein located in the leg, but it is not typically used as a primary choice for femoropopliteal bypass. It is m
ore commonly used for other purposes, such as in the treatment of varicose veins. The great saphenou
s vein is preferred over the short saphenous vein for femoropopliteal bypass due to its larger diameter,
ease of harvesting, and better long-term outcomes.

Solution for Question 64:


Correct Option C: It is absorbed by enzymatic degradation and macrophage phagocytosis

Page 72

2040
• Chromic catgut is an absorbable suture material that is absorbed by the body over time. It is treated
with chromium salts to prolong its absorption rate.
• Once implanted, the sutures undergo enzymatic degradation by the body's enzymes, such as
proteases, and are subsequently phagocytosed (engulfed and broken down) by macrophages.
Incorrect Options:
Option A: It is made from catgut- Chromic catgut is not made from catgut. The term "catgut" is a
historical misnomer, as it does not actually come from cats. It is a type of absorbable suture material th
at was traditionally made from the intestines of sheep or other animals, not specifically cats.
Option B: It is made from rabbit gut- Chromic catgut is not made from rabbit gut. As mentioned earlier,
catgut is typically sourced from the intestines of sheep or other animals, not rabbits.
Option D: It is tanned with nickel salts to improve handling and to resist degradation in tissue- Chromic
catgut is not tanned with nickel salts. The term "chromic" in chromic catgut refers to the use of chromiu
m salts during the manufacturing process to delay the absorption of the suture material, not nickel salts
. The purpose of the chromic treatment is to slow down the absorption process, allowing the suture to
maintain its tensile strength for a longer period while still eventually being absorbed by the body.

Solution for Question 65:


Correct Option B: Morris retractor
• The Morris retractor is the correct identification for the instrument shown in the image.
• It is a self-retaining retractor commonly used in abdominal surgery.
• The Morris retractor consists of multiple, adjustable blades that can be positioned and locked in place
to hold back tissue and provide exposure during the surgical procedure.
Incorrect Options
Option A: Daever's retractor- It is not the correct identification for the instrument in the image.
Option C: Czerney's retractor- Czerney's retractor is another type of abdominal retractor. However, it h
as a different design and appearance compared to the instrument in the image. Czerney's retractor typi
cally consists of two curved blades with teeth or hooks at the ends that are used to hold back and retra
ct tissue.
Option D: Double hook retractor- Double hook retractors are surgical instruments with two curved hook
s at the end. They are used for tissue retraction and exposure during various surgical procedures. How
ever, the instrument in the image is not a double hook retractor.

Solution for Question 66:


Correct Option C: Skin, Subcutaneous tissue, Superficial fascia and Deep fascia
Fasciotomy is a surgical procedure performed to relieve this increased pressure by creating incisions in
the affected compartment's fascial layers.
The layers incised during a fasciotomy for compartment syndrome are as follows:
Incorrect Options:

Page 73

2041
Option A: Skin and Subcutaneous tissue- This option is incorrect because it only involves incising the s
kin and subcutaneous tissue. it is not adequate for a fasciotomy for compartment syndrome. In compar
tment syndrome, the increased pressure is located within the deep fascial compartment, and releasing
the pressure requires incising through the deep fascia as well.
Option B: Skin, Subcutaneous tissue, Superficial fascia- This option is also incorrect because it stops a
t the superficial fascia and does not include incising the deep fascia. As mentioned earlier, the deep fas
cia encapsulates the affected compartment and needs to be incised to relieve the pressure in compart
ment syndrome. Therefore, incising only the skin, subcutaneous tissue, and superficial fascia would no
t provide adequate decompression.
Option D: Skin, Subcutaneous tissue, Superficial fascia, Deep fascia, and a few muscular fibers- This o
ption is not entirely accurate, although it suggests going deeper by including a
few muscular fibers. While it is possible that during a fasciotomy, the incision may extend through a
few muscular fibers if necessary, it is not a standard component of the incision. The primary focus is on
reaching and incising the deep fascia to release the pressure within the affected compartment. The inv
olvement of muscular fibers would depend on the specific case and the surgeon's judgment.

Solution for Question 67:


Correct Option C: Zone 2
injury with an expanding hematoma should be treated surgically: This statement is correct. Zone 2
injuries involve the lateral retroperitoneum. If a patient with a zone 2 injury presents with an expanding
hematoma, it suggests active bleeding or ongoing hemorrhage. In such cases, surgical intervention is
generally required to control the bleeding and repair the injured structures.
Retroperitoneal trauma refers to injuries that occur in the retroperitoneal space, which is located behind
the peritoneum (the membrane that lines the abdominal cavity). Management of retroperitoneal traum
a depends on several factors, including the specific location and severity of the injury.
Incorrect Options:
Option A: All zone 1 injury to be treated conservatively: This statement is incorrect. Zone 1 injuries, whi
ch involve the central retroperitoneum, may require surgical intervention depending on the severity of t
he injury and associated complications. Conservative treatment alone may not be sufficient for all zone
1 injuries.
Option B:
• Zone 3 injury, whether blunt or penetrative, should always be explored: This statement is incorrect.
Zone 3 injuries involve the pelvic retroperitoneum. While some cases of zone 3 injuries may require
exploration, not all cases automatically necessitate surgical exploration. The decision to explore or not
is based on the specific clinical presentation, imaging findings, and associated injuries.
Option D: Mattox maneuver is when the ascending colon is mobilized to the right to visualize the Aorta:
This statement is incorrect. The Mattox maneuver is a surgical technique used to gain access to the di
stal aorta and iliac vessels during specific procedures. It involves mobilizing the left colon (sigmoid colo
n) to the right to expose the aorta. It does not involve mobilization of the ascending colon.

Solution for Question 68:


Correct Option C:

Page 74

2042
• eFAST is a focused ultrasound examination performed in trauma patients to assess for specific
conditions, including pericardial effusion, pneumothorax, and intra-abdominal free fluid. It is particularly
useful in the initial evaluation of trauma patients due to its rapidity and ability to provide real-time
information at the bedside.
• Retroperitoneal hematoma: The least helpful condition to diagnose using eFAST is a retroperitoneal
hematoma. eFAST primarily focuses on assessing intrathoracic and intra-abdominal conditions. While it
can detect free fluid in the abdomen, the retroperitoneal space, which is behind the abdominal cavity, is
not directly assessed. Therefore, eFAST may not be as helpful in diagnosing a retroperitoneal
hematoma compared to other imaging modalities such as CT scans.
Incorrect Options:
Option A: Pericardial effusion: eFAST is helpful in diagnosing pericardial effusion. It involves the asses
sment of the pericardial space for the presence of fluid, which can be indicative of a
pericardial effusion. The presence of a
pericardial effusion may require further evaluation and management to prevent cardiac tamponade.
Option B: Pneumothorax: eFAST is also useful in diagnosing pneumothorax. It involves the evaluation
of both lung fields for the presence of air between the lung and the chest wall, which can indicate a
pneumothorax. This information is valuable in the initial assessment of trauma patients, as a
pneumothorax may require prompt intervention.
Option D: Renal injury: eFAST can be helpful in diagnosing renal injuries. It involves assessing the pre
sence of free fluid in the perirenal space, which can indicate renal injury or hemorrhage. This informatio
n aids in the initial evaluation and management of trauma patients.

Solution for Question 69:


Correct Option B:
• Massive blood transfusion refers to the transfusion of a large volume of blood products over a
relatively short period. While blood transfusions can be life-saving, they can also be associated with
certain adverse effects. However, hypercalcemia is not typically one of them.
Incorrect Options:
Option A: Hypocalcemia: Hypocalcemia, or low levels of calcium in the blood, can be an adverse effect
of a massive blood transfusion. Blood products, such as citrate-anticoagulated blood, can bind to calci
um and decrease its levels in the recipient's blood.
Option C: Hypokalemia: Hypokalemia, or low levels of potassium in the blood, can occur after a massiv
e blood transfusion. Blood products may contain lower levels of potassium compared to the recipient's
blood, leading to a decrease in potassium levels.
Option D: Hyperkalemia: Hyperkalemia, or high levels of potassium in the blood, can be an adverse eff
ect of a massive blood transfusion. Blood products, especially older stored blood, can release potassiu
m into the recipient's blood, potentially causing hyperkalemia.

Solution for Question 70:


Correct Option C:

Page 75

2043
e-FAST is a focused ultrasound examination used in the assessment of trauma patients. It involves the
use of ultrasound to evaluate potential injuries in specific body regions. The traditional FAST examinati
on primarily focuses on the abdomen, but the extended version, e-FAST, includes additional areas.
The e-FAST examination consists of the following components:
Incorrect Options:
Option A: Pelvic Cavity: Although the pelvic cavity is an important region to assess in trauma patients, i
t is not specifically included in the e-FAST examination. The focused assessment of the pelvis is usuall
y performed separately using different imaging techniques, such as X-rays or CT scans.
Option B: Peripheral vessels: The e-FAST examination does not focus on assessing peripheral vessels
. Its primary purpose is to identify potential traumatic injuries within the abdomen and thoracic cavity, a
s mentioned earlier.
Option D: Dural and subdural spaces: The e-FAST examination does not include the assessment of th
e dural and subdural spaces. These spaces are related to the central nervous system and are typically
evaluated using imaging modalities like CT or MRI scans.

Solution for Question 71:


Correct Option C: HR/MAP
• HR/MAP (Heart Rate/Mean Arterial Pressure): This is the correct calculation for the modified shock
index.
• The modified shock index is calculated by dividing the heart rate by the mean arterial pressure.
• The mean arterial pressure is a measure of the average pressure within the arteries during one
cardiac cycle and is a more reliable indicator of tissue perfusion compared to the systolic blood
pressure alone.
Incorrect Option:
Option A: HR/SBP (Heart Rate/Systolic Blood Pressure)- This is not the correct calculation for the modi
fied shock index.
Option B: HR/DBP (Heart Rate/Diastolic Blood Pressure)- This is not the correct calculation for the mo
dified shock index.
Option D: PR/SBP (Pulse Rate/Systolic Blood Pressure)- This is not the correct calculation for the modi
fied shock index.

Solution for Question 72:


Option C: Insertion of the needle, which will be between your ribs on your back
• This step is incorrect. When placing an intercostal drainage tube, the needle should be inserted
between the ribs on the patient's side, not on the back. This ensures access to the pleural space for
proper drainage.
• This method is not correct as it may affect drainage.

Page 76

2044
Other options
Option A: Identified 5th intercostal space anterior to mid axillary line to place the tube
• This step is generally correct. The 5th intercostal space, anterior to the mid-axillary line, is a common
location for placing an intercostal drainage tube.
Option B: Inserted above the upper border of the rib
• This step is correct. When inserting an intercostal drainage tube, it should be inserted below the upper
border of the rib. Placing it above the upper border could lead to improper positioning or potential injury.
Option D: Incised and digitally explored
• This step is not typically part of the procedure for placing an intercostal drainage tube. The primary
goal is to insert the tube into the appropriate intercostal space and position it correctly for drainage,
without the need for extensive incisions or digital exploration.

Solution for Question 73:


Correct Option: B
• Based on the information provided, the most likely diagnosis in this patient is Tubercular
lymphadenitis.
• Tubercular lymphadenitis: Tubercular lymphadenitis, also known as tuberculous lymphadenopathy or
scrofula, is a form of tuberculosis that affects the lymph nodes. It commonly presents with painless,
enlarged lymph nodes that may be accompanied by systemic symptoms such as fever, night sweats,
and weight loss. The presence of a non-healing ulcer with undermined edges is a characteristic feature
of tubercular lymphadenitis. Given that the patient is on steroids, which can suppress the immune
system, the risk of tuberculosis reactivation is increased. Therefore, this is the most likely diagnosis in
this case.
Incorrect options:
Option A: Psoriasis flare up: Psoriasis is a chronic autoimmune skin condition characterized by red, itc
hy, and scaly patches on the skin. It does not typically present with a fever, weight loss, or ulceration wi
th undermined edges. Therefore, this is an unlikely diagnosis in this case.
Option C: Lymphosarcoma: Lymphosarcoma, also known as lymphoma, is a cancer that originates fro
m lymphocytes. It can present with enlarged lymph nodes, fever, weight loss, and other systemic symp
toms. However, the presence of a non-healing ulcer with undermined edges is not typically associated
with lymphosarcoma. Therefore, this is an unlikely diagnosis in this case.
Option D: Syphilis: Syphilis is a
sexually transmitted infection caused by the bacterium Treponema pallidum. It can present with a varie
ty of symptoms, including skin manifestations such as ulcers. However, the presence of undermined ed
ges on the ulcer is not a characteristic feature of syphilis. Additionally, the patient's symptoms of fever
and weight loss are not specific to syphilis. Therefore, this is an unlikely diagnosis in this case.
In summary, based on the given information, the most likely diagnosis in this patient is Tubercular lymp
hadenitis, as it aligns with the presence of a
non-healing ulcer with undermined edges, along with symptoms of fever and weight loss.

Page 77

2045
Solution for Question 74:
Correct Option B: Subdural hematoma
Subdural Hematoma (SDH):
• Subdural hematoma is another type of traumatic brain injury characterized by bleeding between the
dura mater and the arachnoid mater (the layer beneath the dura).
• On a CT head, a subdural hematoma appears as a crescent-shaped or concave collection of blood
adjacent to the brain tissue.
• It is typically seen in the subdural space overlying the cerebral hemispheres.
• Subdural hematomas can result from both acute and chronic injuries, and they often present with
symptoms such as headache, confusion, and neurological deficits.
• Acute subdural hematomas can be life-threatening and may require surgical intervention, while
chronic subdural hematomas may be managed conservatively or surgically depending on the size and
symptoms.
Incorrect options:
Option A: Extradural Hematoma (EDH):
• Extradural hematoma is a type of traumatic brain injury characterized by bleeding between the dura
mater (outermost layer of the meninges) and the inner surface of the skull.
• On a CT head, an extradural hematoma appears as a well-defined, lens-shaped collection of blood
that is typically located in the temporoparietal region of the skull.
• It is often associated with a history of head trauma, and the characteristic finding is a biconvex or
lens-shaped hematoma that does not cross suture lines.
• Extradural hematomas can cause a rapid increase in intracranial pressure and may require surgical
intervention to evacuate the hematoma and relieve the pressure.
Option C: Intraparenchymal Bleed:
• Intraparenchymal bleed refers to bleeding within the brain tissue itself.
• On a CT head, an intraparenchymal bleed appears as an irregular area of increased density or a
hemorrhagic lesion within the brain parenchyma.
• These bleeds are often associated with trauma, hypertension, vascular malformations, or other
underlying conditions.
• The location and extent of the bleed can vary depending on the underlying cause.
• Intraparenchymal bleeds can cause focal neurological deficits and may require specific management
based on the underlying cause and severity of the bleed.
Option D: Subarachnoid Hemorrhage (SAH):
• Subarachnoid hemorrhage refers to bleeding into the subarachnoid space, which is the space
between the arachnoid mater and the pia mater (innermost layer of the meninges).
• On a CT head, a subarachnoid hemorrhage appears as diffuse or localized high-density areas in the
subarachnoid space surrounding the brain.
• It is commonly caused by the rupture of an intracranial aneurysm or vascular malformation.
• Subarachnoid hemorrhage is often associated with a sudden severe headache, neck stiffness, and
altered consciousness.

Page 78

2046
• It is a medical emergency, and further evaluation with angiography or other imaging modalities may
be necessary to determine the source of the bleeding and guide treatment options.

Solution for Question 75:


Correct Answer B :
• Indirect inguinal hernia: Indirect inguinal hernias are the most common type of hernia and occur when
abdominal contents, such as intestine or fat, protrude through the deep inguinal ring and descend into
the inguinal canal. This type of hernia may present as a mass or bulge in the inguinal region, and it can
often be manually reduced, causing a gurgling sound due to the passage of intestinal contents. Indirect
inguinal hernias are more common in males and may be present from birth (congenital) or develop later
in life.
Incorrect options:
Option A. Direct inguinal hernia :
Direct inguinal hernias occur when abdominal contents, such as intestine or fat, protrude through a we
ak point in the abdominal wall in the inguinal region. Unlike indirect inguinal hernias, direct hernias do n
ot pass through the deep inguinal ring. They typically occur in older individuals and are more common i
n men. Direct hernias may cause a bulge in the inguinal region but do not typically exhibit a
gurgling sound upon reduction.
Option C. Femoral hernia: Femoral hernias occur when abdominal contents protrude through the femor
al canal, located just below the inguinal ligament. This type of hernia is more common in women and m
ay present as a mass or bulge in the upper thigh or groin area, rather than the inguinal region. Femoral
hernias typically do not reduce with a gurgling sound, as they have a
narrower passage compared to inguinal hernias.
Option D. Pantaloon hernia: A pantaloon hernia is a specific type of inguinal hernia that involves a
combination of direct and indirect hernias. It refers to a
hernia that protrudes both above and below the inguinal ligament. It can present with a
mass or bulge on both sides of the inguinal region and may exhibit a gurgling sound upon reduction.

Solution for Question 76:


Correct Answer C.
• Among the statements you provided, the true statement regarding Taenia solium is:
• The eggs do not float in the saturated salt solution.
Among the statements you provided, the true statement regarding Taenia solium is:
Incorrect Options:
Option A. Eggs are not infective to man: This statement is false. Taenia solium is a parasitic tapeworm
that can infect humans. The primary mode of infection occurs when humans ingest the eggs of Taenia
solium, typically through the consumption of undercooked pork containing cysticerci (larval stage of the
tapeworm). Once ingested, the eggs hatch in the intestine, and the larvae migrate to various tissues a
nd organs, leading to the development of cysticercosis.
Option B.

Page 79

2047
Hooks are absent: This statement is false. Taenia solium possesses hooks on its scolex (head) that it u
ses to attach itself to the intestinal lining of its host. These hooks aid in anchoring the tapeworm and en
suring its survival within the host's digestive system.
Option D. Intermediate host is cow: This statement is incorrect. While Taenia solium has an intermediat
e host in its lifecycle, it is not a cow. The intermediate host for Taenia solium is typically a pig. Pigs bec
ome infected by ingesting the eggs of Taenia solium, and the larvae develop into cysticerci within the pi
g's muscles. If humans consume undercooked pork containing these cysticerci, they can become the d
efinitive host and develop the adult tapeworm in their intestines.

Solution for Question 77:


CORRECT ANSWER: B. Chest tube drainage and antibiotics.
• This approach allows for the drainage of the infected fluid from the pleural space through the insertion
of a chest tube. The chest tube helps to drain the pus and debris, relieving pressure and facilitating lung
re-expansion. Antibiotics are prescribed to target the underlying infection and prevent further
complications.
Incorrect options:
Option A. Antibiotics alone is not sufficient for this condition.
Option C. ICD alone is not sufficient for this condition.
Option D. Wait and watch is incorrect.

Solution for Question 78:


CORRECT ANSWER : C.
• The most common radio-isotope used in PET (Positron Emission Tomography) scans is 18F
(Fluorine-18). Therefore, the correct option is 18F.
Incorrect options:
Option A. 13 C is incorrect
Option B. 14C is incorrect
Option D. all of the above is not correct

Solution for Question 79:


Correct Option: A
• Start PRBC 1st and store platelet at room temperature: This option is the correct answer. When faced
with limited IV access, the priority is to initiate the transfusion of packed red blood cells (PRBCs) since
the patient is experiencing massive blood loss. PRBCs should be transfused immediately to restore
oxygen-carrying capacity and address acute anemia. Platelets, on the other hand, can be stored at
room temperature for a short duration without significant loss of function.

Page 80

2048
Incorrect Options:
Option B. Start platelet and store PRBC at room temperature: This option is not the ideal choice. In a si
tuation of massive blood loss, addressing acute anemia by transfusing PRBCs takes precedence over
platelet transfusion. Delaying PRBC transfusion could lead to further complications related to inadequa
te oxygen delivery to vital organs.
Option C. Only transfuse PRBC: This option is not suitable in this scenario. While PRBC transfusion is
necessary for addressing the acute anemia, platelet transfusion should also be considered since the p
atient received 4 platelets from the blood bank. Platelets are vital for promoting clotting and preventing
bleeding complications, particularly in patients with significant blood loss.
Option D. Transfuse PRBC and store platelet at 2-6 degrees: This option is not the best choice in this s
ituation. Storing platelets at 2-6 degrees Celsius is the recommended temperature for long-term storag
e, but it is not necessary when there is limited IV access and a need for immediate transfusion. Additio
nally, as mentioned earlier, PRBC transfusion takes priority over platelet transfusion in the context of m
assive blood loss.

Solution for Question 80:


Correct
Option D.
• The immediate next step in this scenario would be to insert a needle in the 2nd intercostal space.
• The patient's clinical presentation suggests tension pneumothorax, which is a life-threatening
condition.
• Tension pneumothorax is a clinical diagnosis and treatment should never be delayed by waiting for
radiological confirmation
• Inserting a needle in the 2nd intercostal space (in the midclavicular line) can decompress the tension
pneumothorax and provide immediate relief by allowing air to escape from the pleural space.
Tension pneumothorax is a
clinical diagnosis and treatment should never be delayed by waiting for radiological confirmation
Incorrect Options:
Option A. Urgent IV fluid administration: While IV fluid administration may be necessary in trauma case
s, it is not the immediate next step in this scenario. The patient's clinical signs and symptoms suggest a
potentially life-threatening condition that requires immediate intervention.
Option B. Intubate the patient: Intubation may be required if the patient's respiratory status worsens or i
f other indications for intubation are present. However, the immediate concern in this scenario is addre
ssing the potential tension pneumothorax.
Option C. Chest X-ray: Chest X-ray may provide additional diagnostic information, but it should not del
ay the immediate intervention required for a potential tension pneumothorax.

Page 81

2049
Solution for Question 81:
Correct Option: C.
• The correct procedure for inserting a Nasogastric (NG) tube is to have the patient sitting with the neck
flexed.
• This position helps straighten the nasal passage and facilitate the passage of the NG tube into the
stomach.
Incorrect Options:
Options A. Supine with neck flexed: The supine position with neck flexed may cause difficulty in the pa
ssage of the NG tube and increase the risk of complications.
Options B. Supine with neck extended: The supine position with neck extended may also cause difficult
y in the passage of the NG tube and increase the risk of complications.
Options D. Sitting with neck extended: The sitting position with neck extended may make the insertion
of the NG tube more challenging and increase the risk of complications.

Solution for Question 82:


Correct Option D.
• The clinical presentation of bilious vomiting, gross abdominal distention, absent bowel sounds, and an
X-ray abdomen showing multiple gas-filled loops is consistent with malrotation of the gut.
• Malrotation is a congenital condition where the intestines do not rotate and fixate properly during fetal
development, leading to abnormal positioning and potential complications such as volvulus.
Incorrect Options:
Option A. Hirschsprung disease: Hirschsprung disease is characterized by a lack of ganglion cells in th
e distal bowel, leading to obstruction and failure to pass meconium. It does not typically present with bil
ious vomiting and gas-filled loops on an X-ray.
Option B. Congenital Hypertrophic pyloric stenosis: Congenital Hypertrophic pyloric stenosis is a condit
ion characterized by hypertrophy and narrowing of the pyloric sphincter, causing projectile nonbilious v
omiting. It does not typically present with gas-filled loops on an X-ray.
Option C. Duodenal atresia: Duodenal atresia is a congenital condition where there is complete or parti
al obstruction of the duodenum. It typically presents with bilious vomiting, but the X-ray findings may sh
ow a "double bubble" sign rather than gas-filled loops throughout the abdomen

Solution for Question 83:


Correct Option: C.
• The initial treatment for a newborn infant presenting with abdominal distension, bilious vomiting,
failure to pass meconium, and an X-ray showing dilated small bowel loops with a soap bubble

Page 82

2050
appearance in the right lower quadrant is a contrast enema.
• This presentation is suggestive of meconium ileus, which is commonly associated with cystic fibrosis.
• A contrast enema can be both diagnostic and therapeutic, as it helps dislodge the meconium plug
causing the obstruction.
Incorrect Options:
Options A. Paul mikulicz ileostomy: Paul mikulicz ileostomy involves creating a surgical opening (stom
a) in the ileum to divert the fecal stream. It is not the initial treatment for meconium ileus.
Options B. Bishop ileostomy: Bishop ileostomy is another surgical procedure that involves creating an il
eostomy to divert the fecal stream. It is not the initial treatment for meconium ileus.
Options D. Barium enema: Barium enema involves the use of barium contrast for imaging the lower ga
strointestinal tract. In meconium ileus, a contrast enema is preferred over a
barium enema due to the risk of barium impaction and increased bowel perforation.

Solution for Question 84:


Correct Option: B.
• In a newborn presenting with excessive frothing and mild respiratory distress, the first investigation to
be done is NG (Nasogastric) tube insertion and a chest X-ray (CXR) to check the position of the tube.
• This helps assess whether the frothing is due to tracheoesophageal fistula (TEF) or esophageal
atresia (EA). The NG tube can be inserted and advanced into the stomach, and a CXR can confirm its
position.
Incorrect Options:
Options A. Bronchoscopy with injection of methylene blue: Bronchoscopy with injection of methylene bl
ue is not the initial investigation in this scenario. It may be performed later if TEF or EA is suspected bu
t not confirmed by the NG tube insertion and CXR.
Options C. CT chest: CT chest is not the initial investigation for assessing TEF or EA in a newborn. It in
volves higher radiation exposure and is typically reserved for more complex cases or when other invest
igations are inconclusive.
Options D. Endoscopy: Endoscopy is not the initial investigation for assessing TEF or EA in a
newborn. It may be performed later if needed for further evaluation or management.

Solution for Question 85:


Correct option B.
• Pleural cavity: The pleural cavity is the space between the lungs and the chest wall. Although
ultrasound can be used to evaluate the pleural cavity for other purposes, such as pneumothorax, it is
not typically included in the FAST examination.
Incorrect Options:

Page 83

2051
Option A. Pericardium: The pericardium is the protective sac surrounding the heart. FAST can assess f
or the presence of fluid or blood within the pericardial sac, indicating cardiac tamponade.
Option C. Spleen: The spleen is an organ located in the upper left abdomen. FAST can detect the pres
ence of free fluid around the spleen, which may indicate splenic injury or bleeding.
Option D. Liver: The liver is the largest organ in the abdomen. FAST can identify the presence of free fl
uid in the space around the liver, suggesting liver injury or bleeding.

Solution for Question 86:


Correct Option: C. 10
• The Glasgow Coma Scale (GCS) is used to assess the level of consciousness in a patient. It consists
of three components: eye opening response, verbal response, and motor response. In this case, the
patient exhibits:
• Eye Opening: In response to pain stimuli
• Verbal Response: Confused and unable to tell time
• Motor Response: Flexion of the arm in response to painful stimuli
• According to the GCS, eye opening on pain scores 2 points, confused verbal response scores 4
points, and flexion to painful stimuli scores 4 points. Therefore, the total GCS score is 2 + 4 + 4 = 10.
Incorrect Options:
Options A. 8: This option does not account for the verbal response and flexion of the arm, which would
contribute to a higher GCS score.
Options B. 9: This option does not account for the verbal response, which would contribute to a
higher GCS score.
Options D. 11: This option overestimates the GCS score by including a
higher motor response component, as flexion to painful stimuli scores 4 points, not 5.

Solution for Question 87:


Correct Option A:
• Administering prophylactic antibiotics 60 minutes before the surgical incision allows enough time for
the antibiotics to reach effective levels in the patient's tissues and bloodstream.
• This timing ensures that the concentration of antibiotics is optimal at the time of surgery, providing
protection against potential bacteria introduced during the procedure.
Incorrect options:
Option B. "1-3 hours before skin incision":Administering prophylactic antibiotics 1-3 hours before the sk
in incision is not the recommended timing. Waiting too long between antibiotic administration and surge
ry can result in suboptimal antibiotic concentrations during the procedure, increasing the risk of surgical

Page 84

2052
site infections (SSI). The goal is to have adequate antibiotic levels at the time of surgery to effectively
prevent infections.
Option C. "At the time of surgical incision": Administering antibiotics at the time of surgical incision may
be too late for optimal prophylaxis. By the time the incision is made, bacteria may have already been i
ntroduced into the surgical site, making it more challenging for antibiotics to prevent infection effectivel
y. Administering antibiotics before the incision allows the medication to reach adequate levels in the tis
sues and bloodstream prior to potential bacterial exposure.
Option D. "Night before surgery for peaking of effect": Administering antibiotics the night before surgery
with the expectation of peaking effect is not the recommended timing for prophylactic antibiotics. While
it is true that some antibiotics may have a time-dependent effect and require a certain duration to reac
h peak concentration, administering them the night before surgery may result in suboptimal antibiotic le
vels during the actual procedure. Waiting too long between antibiotic administration and surgery can co
mpromise their effectiveness in preventing SSIs.

Solution for Question 88:


Correct Option A:
• Artery forceps, also known as hemostats or surgical forceps, are a type of surgical instrument
commonly used to grasp and manipulate tissues, including blood vessels. They have a locking
mechanism that allows them to securely hold tissues or control bleeding during surgical procedures.
Incorrect Options:
Option B. Kocher forceps: Kocher forceps, also known as Kocher clamps, are a
type of surgical forceps with serrated jaws and a ratchet locking mechanism. They are typically used to
grasp and manipulate heavy tissue or structures during surgical procedures.

Option C. Allis forceps: Allis forceps, also called Allis clamps, have interlocking teeth or tines on the ja
ws to provide a firm grip on tissues. They are commonly used to grasp tissue or organs during surgery,
such as during wound closure or tissue manipulation.

Page 85

2053
Option D. Babcock forceps: Babcock forceps are a type of non-traumatic surgical instrument used for g
rasping delicate tissues, such as intestines or fallopian tubes, during surgical procedures. They have a
smooth, atraumatic jaw design with a spring-like action that allows for gentle tissue manipulation and a
secure grip.

Solution for Question 89:


Correct Option B:
• Among the options provided, the gut (intestines) has the highest chances of graft rejection response.
organ transplants due to the large amount of lymphoid tissue present in the gut-associated lymphoid
system.
Incorrect options

Page 86

2054
Option A - Cornea: This option is incorrect. Although corneal transplantation can have graft rejection, th
e gut has a higher likelihood of graft rejection compared to the cornea.
Option C
- Liver: This option is incorrect. While liver transplantation can have graft rejection, the gut has a
higher likelihood of graft rejection compared to the liver.
Option D - Skin: This option is incorrect. Although skin grafts can undergo rejection, the gut has a
higher likelihood of graft rejection compared to the skin.

Solution for Question 90:


Correct Option A
• In hypovolemic shock, the organ that should be assessed for determining under-perfusion is the
kidney.
• The kidneys are highly sensitive to changes in blood volume and perfusion.
• In hypovolemic shock, decreased blood flow to the kidneys can lead to renal dysfunction and
subsequent impaired urine production.
Incorrect Options
Option B - Heart: This option is incorrect. While the heart can be affected by hypovolemic shock, the or
gan that is specifically assessed for under-perfusion in this condition is the kidney.
Option C - Lung: This option is incorrect. Although the lungs can be affected by hypovolemic shock, the
organ that is specifically assessed for under-perfusion in this condition is the kidney.
Option D - Liver: This option is incorrect. Although the liver can be affected by hypovolemic shock, the
organ that is specifically assessed for under-perfusion in this condition is the kidney.

Solution for Question 91:


Correct Option C.
Sitting with neck flexion: This is the correct position for inserting a Ryle's tube. Sitting position allows th
e patient's head to be slightly flexed forward, which helps to align the nasal passage for easier insertio
n of the tube. The flexed position helps straighten the path from the nostril to the throat, facilitating the
passage of the tube through the nasal passage and into the stomach.
Incorrect Option:
Option A. Supine with neck flexion: This position is not ideal for the insertion of a Ryle's tube. Neck flexi
on may cause difficulty in visualizing and accessing the patient's nasal passage, making the procedure
more challenging.
Option B. Supine with neck extension: This position is incorrect for the insertion of a Ryle's tube. Neck
extension can obstruct the airway and may cause discomfort or difficulty in passing the tube through th
e nasal passage.

Page 87

2055
Option D. Sitting with neck extension: This position is not recommended for the insertion of a Ryle's tub
e. Neck extension can obstruct the airway and may cause discomfort to the patient. It can also make it
more challenging to insert the tube properly.

Solution for Question 92:


Correct Option A.
This is the correct option for achieving the maximum flow rate. Grey cannulas typically have the largest
diameter (16G) among the options listed, allowing for a higher flow rate of fluids.
Incorrect Option:
Option B. Green cannula: Green cannulas are smaller in diameter (18G) compared to the Grey cannul
a. While they can still provide a
decent flow rate, they may not be as optimal as the Grey cannula for delivering a
large volume of fluids at the maximum rate.
Option C. Blue cannula: Blue cannulas are smaller in diameter (22G) than both Grey and Green cannul
as. They are suitable for patients who require a
moderate flow rate. However, they may not provide the maximum flow rate required for a
patient needing a large amount of fluids.
Option D. Pink cannula: Pink cannulas are the smallest in diameter (20G) among the options listed. Th
ey are commonly used for patients who require a low flow rate, such as for pediatric patients or those
with fragile veins. They are not ideal for achieving the maximum flow rate necessary for patients needin
g a large amount of fluids.

Solution for Question 93:


Correct Option B.
Insert needle in appropriate intercostal space: This is the correct option. The presence of hyper-resona
nce on the involved side of the chest, along with the patient's clinical findings (chest trauma, respiratory
distress, hypotension), suggests a potential tension pneumothorax. Inserting a needle in the appropria
te intercostal space (needle thoracostomy) is an emergency procedure used to decompress the accum
ulated air in the chest and relieve the tension. This intervention takes precedence in this situation.
Incorrect Option:
Option A. Intubate: Intubation may be necessary in certain cases, particularly if the patient's airway is c
ompromised or if there are signs of respiratory distress. However, in this specific scenario, the patient's
respiratory rate of 44 suggests that he is still maintaining his airway and breathing adequately. Therefo
re, intubation is not the immediate priority.
Option C. Start intravenous fluids urgently: While administering intravenous fluids may be necessary fo
r the management of hypotension, in this case, the primary concern is the potential tension pneumotho
rax. Addressing the tension pneumothorax should be the immediate priority before initiating intravenou
s fluid therapy.
Option D. Take a chest X-ray: Chest X-ray is an important diagnostic tool to confirm the presence of a
pneumothorax and assess its severity. However, in this critical scenario with signs of tension pneumoth
orax, waiting for a chest X-ray could cause significant delay in appropriate management. Immediate int

Page 88

2056
ervention to decompress the chest is necessary before further diagnostic confirmation.

Solution for Question 94:


Correct Option B.
Prevent transfusion-related reactions: This is the correct option. Leukoreduction filters are designed to
remove white blood cells from donated blood products, such as red blood cells or platelets. White bloo
d cells can contribute to transfusion-related reactions, including febrile non-hemolytic reactions, allergic
reactions, and some cases of transfusion-related acute lung injury (TRALI). By reducing the number of
white blood cells in the transfused blood, leukoreduction filters help minimize the occurrence of these r
eactions.
Incorrect Options:
Option A. Prevent viral infections: While leukoreduction filters can help reduce the risk of certain viral in
fections to some extent, their primary purpose is not specifically focused on preventing viral infections.
Leukoreduction filters are more effective in removing white blood cells and certain cellular components
from the blood to prevent transfusion-related reactions.
Option C. Prevent infections: While leukoreduction filters can help reduce the risk of bacterial and fung
al contamination to some extent, their primary purpose is not specifically focused on preventing infectio
ns. The primary goal of leukoreduction filters is to remove white blood cells, which can contribute to tra
nsfusion-related reactions, as mentioned earlier.
Option D. All of the above: This option is incorrect because leukoreduction filters primarily target transf
usion-related reactions by removing white blood cells. Although they may have some additional benefit
s in reducing the risk of certain viral infections and bacterial/fungal contamination, these are secondary
effects rather than the primary purpose of leukoreduction filters.

Solution for Question 95:


Correct Option B.
3%: This is the correct option. Clean-contaminated wounds, which have a controlled entry into a norma
lly sterile area (such as the gastrointestinal tract or genitourinary tract) with minimal contamination, carr
y a higher risk of surgical site infection compared to clean wounds. With the administration of appropria
te antibiotic prophylaxis, the expected surgical site infection rate for clean-contaminated wounds is aro
und 3%.
Incorrect Options:
Option A. 1-2%: This option is incorrect. A surgical site infection rate of 1-2% is more commonly associ
ated with clean wounds that are operated upon with appropriate aseptic techniques, rather than clean-
contaminated wounds. Clean-contaminated wounds have a slightly higher risk of infection due to poten
tial contamination from the surrounding environment or internal sources.
Option C. 13-20%: This option is incorrect. A surgical site infection rate of 13-20% is significantly highe
r and is more commonly associated with contaminated or dirty wounds, where there is significant micro
bial contamination or active infection at the surgical site. Clean-contaminated wounds, although carryin
g a higher risk compared to clean wounds, have a lower expected infection rate.

Page 89

2057
Option D. 20-30%: This option is incorrect. A surgical site infection rate of 20-30% is extremely high an
d is usually associated with heavily contaminated or dirty wounds, such as those involving fecal conta
mination or extensive tissue necrosis. Clean-contaminated wounds have a
lower expected infection rate compared to this range.

Solution for Question 96:


Correct Option A.
• Negative pressure wound therapy: This is the correct option. NPWT creates a controlled negative
pressure environment around the wound, which helps in wound healing through several mechanisms. It
promotes wound contraction, reduces edema, enhances blood flow and oxygenation, removes excess
fluid and wound exudate, and facilitates the formation of granulation tissue.
Incorrect Options:
Option B. Positive pressure wound therapy: This option is incorrect. Positive pressure wound therapy i
nvolves the application of positive pressure to the wound, which is different from the negative pressure
used in NPWT. Positive pressure wound therapy is not commonly used for wound healing and does no
t provide the same benefits as NPWT.
Option C. Foam therapy: This option is incorrect. Foam therapy refers to the use of specialized foam dr
essings in wound management. While foam dressings can be used in conjunction with NPWT, they are
not the principle behind the procedure shown in the image.
Option D. Sponge therapy: This option is incorrect. Sponge therapy is a
vague term and does not specifically describe a
recognized wound-healing technique. It is not synonymous with NPWT.

Solution for Question 97:


Correct option C
• The components of qSOFA (quick Sequential Organ Failure Assessment) scoring include:
• A. Mental status: qSOFA includes altered mental status as one of its components. It assesses the
level of consciousness or mental status of the patient, such as confusion, disorientation, or altered
arousal.
• B. Blood pressure: qSOFA includes blood pressure as one of its components. It assesses the systolic
blood pressure, and a value of less than or equal to 100 mm Hg is considered abnormal.
• C. Respiratory rate: qSOFA includes respiratory rate as one of its components. It assesses the
number of breaths per minute, and a value of 22 or higher is considered abnormal.
• D. Heart rate: qSOFA does not include heart rate as one of its components. It focuses on mental
status, blood pressure, and respiratory rate as indicators of organ dysfunction and increased risk of
mortality in patients with suspected infection.
Incorrect options:
Options A. A & B: This option is incorrect because it only includes mental status and blood pressure, b
ut it leaves out respiratory rate, which is an essential component of qSOFA scoring.

Page 90

2058
Options B. C & D: This option is incorrect because it includes respiratory rate and heart rate, but it leav
es out mental status and blood pressure, which are both part of the qSOFA scoring criteria.
Options D. A, B, C & D: This option is incorrect because it includes all the listed components, which is
not accurate. Heart rate is not included in qSOFA scoring.

Solution for Question 98:


Correct Option B
• A. Urine Bag - 3 B. Nasogastric Tube - 1 C. Underwater seal bag - 2 D. Suction cannula - 4
• Therefore, the correct option is "B. A-3, B-1, C-2, D-4."
Incorrect options:
Option A. A-3, B-4, C-2, D-1: This option is incorrect because it incorrectly matches the items.
Optoin C. A-2, B-4, C-3, D-1: This option is incorrect because it incorrectly matches the items.
Option D. A-2, B-1, C-3, D-4: This option is incorrect because it incorrectly matches the items.

Solution for Question 99:


Correct Option D:
Organ transplantation aims to provide the best possible outcomes for both the donor and the recipient.
When selecting an organ recipient, several factors are taken into consideration to ensure the highest c
hances of success and compatibility.
32-year-old woman with renal failure: The 32-year-old woman with renal failure would be the most suita
ble recipient in this scenario. The age match between the donor and recipient is optimal, which increas
es the chances of compatibility and successful transplantation. Kidney transplantation is a
well-established treatment for renal failure, and this recipient's age suggests that she may have a
longer life expectancy and a better chance of benefiting from the donated organ.
Incorrect Options:
Option A. 14-year-old child with multi-organ dysfunction syndrome: While organ transplantation could p
otentially benefit the 14-year-old child with multi-organ dysfunction syndrome, the age difference betwe
en the donor (32-year-old woman) and the recipient may present compatibility challenges. The optimal
scenario for organ transplantation usually involves donors and recipients within a
similar age range to minimize the risk of graft rejection and improve long-term outcomes.
Option B. 50-year alcoholic man with liver cirrhosis with 1-month abstinence: This option does not spec
ify the type of organ needed for transplantation. However, given the patient's history of alcoholism and l
iver cirrhosis, it is crucial to evaluate the recipient's suitability for organ transplantation. A thorough ass
essment of the patient's liver function, abstinence duration, and overall health status would be necessa
ry to determine if the recipient is an appropriate candidate for a liver transplant.

Page 91

2059
Option C. 70-year-old woman with end-stage renal disease: While the 70-year-old woman with end-sta
ge renal disease could potentially benefit from a kidney transplant, advanced age can be a significant f
actor in determining suitability. The age of the recipient and the associated comorbidities should be car
efully evaluated to assess the potential risks and benefits of transplantation.

Solution for Question 100:


Correct option C
• A 20 G cannula refers to a cannula with a gauge size of 20. The gauge size represents the diameter
of the cannula, with smaller gauge numbers indicating larger diameters.
Incorrect options:
Option A. 16 G: This option represents a larger cannula size compared to a 20 G cannula. A 16 G
cannula would have a wider diameter.
Option B. 18 G: This option represents a slightly smaller cannula size compared to a 20 G
cannula. An 18 G cannula would also have a wider diameter.
Option D. 22 G: This option represents a smaller cannula size compared to a 20 G cannula. A 22 G
cannula would have a narrower diameter.

Solution for Question 101:


Correct Option B:
FAST, which stands for Focused Assessment with Sonography in Trauma, is a rapid bedside ultrasoun
d examination performed in trauma patients. It is used to evaluate for the presence of free fluid or bloo
d in the abdomen or thorax, which can indicate internal bleeding or organ injury.
During a FAST examination, a handheld ultrasound probe is used to assess specific areas of the body,
including the abdomen (including the pericardium) and both sides of the chest. The aim is to quickly id
entify significant injuries that may require immediate intervention, such as surgery or other intervention
s.
Incorrect Options:
Option A. First assessment with sonography in trauma: This option is similar to the correct option, but it
does not specify the focus or purpose of the ultrasound examination. The term "focused" in FAST refer
s to the specific evaluation of free fluid or blood in trauma patients.
Option C. First abdominal sonography in trauma: This option is not entirely accurate. While FAST does
involve assessing the abdomen, it also includes the evaluation of the thorax for the presence of free fl
uid or blood. The term "focused" in FAST indicates that the examination is not comprehensive or compl
ete abdominal sonography but rather a targeted assessment for specific findings.
Option D. Focused abdominal sonography in trauma: This option is a close variation of the correct ans
wer. However, it does not encompass the evaluation of the thorax, which is an essential component of
the FAST examination. Therefore, the term "focused abdominal sonography" alone is not as comprehe
nsive or accurate as the term "focused assessment with sonography."

Page 92

2060
Solution for Question 102:
Correct Option:
Option D.
• Eyes-3, Verbal-NT, Motor-6: This is the correct option. The patient's score of 3 for Eyes indicates they
open their eyes in response to verbal command. The prompt does not provide information on the
patient's verbal response, so it is considered Not Testable (NT). Lastly, a score of 6 for Motor
accurately reflects the patient moving all four limbs spontaneously.
Incorrect Options :
Option A. Eyes-2, Verbal-1, Motor-5: According to the Glasgow Coma Scale (GCS), which assesses th
e level of consciousness in patients, a score of 2
for Eyes indicates the patient is opening their eyes in response to pain. A score of 1
for Verbal indicates an incomprehensible or inappropriate response. A score of 5 for Motor indicates lo
calized response or withdrawal to pain. This option does not accurately reflect the patient's response of
opening eyes on verbal command and moving all four limbs spontaneously.
Option B. Eyes-2, Verbal-NT, Motor-5: This option indicates a score of 2 for Eyes, suggesting the patie
nt opens their eyes in response to pain. However, it specifies Verbal as NT (Not Testable), implying tha
t the patient's verbal response cannot be assessed. Lastly, a score of 5 for Motor indicates localized re
sponse or withdrawal to pain. Since the verbal response is not testable, this option does not accurately
reflect the patient's GCS.
Option C. Eyes-3, Verbal-1, Motor-6: According to this option, the patient scores 3
for Eyes, indicating they open their eyes in response to verbal command. A score of 1
for Verbal suggests an incomprehensible or inappropriate response. Lastly, a score of 6 for Motor impli
es the patient obeys commands. However, the prompt states that the patient moves all four limbs spon
taneously, which indicates a higher level of motor function than just obeying commands. Therefore, this
option does not accurately represent the patient's GCS.

Solution for Question 103:


Correct Option D - Reverse Trendelenburg position:
• The position of the patient in the image is the Reverse Trendelenburg position. This position is
characterized by raising the head of the bed while keeping the feet lower than the head. In this position,
the patient's body is tilted with the head elevated above the level of the heart, and the legs are
positioned lower.
Incorrect Options:
Option A - Lloyd Davis position: The Lloyd Davis position refers to the lithotomy position, which is com
monly used for gynecological procedures. In the lithotomy position, the patient lies on their back with th
eir hips flexed and knees bent, and their feet are positioned in stirrups.
Option B - Jack knife position: The Jack knife position is a surgical position commonly used in procedur
es involving the rectum or lower spine. In this position, the patient is placed on their abdomen with the
hips flexed and the torso and legs raised, resembling the shape of a folding jack knife.

Page 93

2061
Option C - Trendelenburg position: The Trendelenburg position involves tilting the entire bed with the h
ead down and the feet elevated higher than the head. This position is often used in surgical procedures
or to improve blood flow in certain medical conditions.

Solution for Question 104:


Correct Option:
Option C.
• Southampton grading: This is the correct option. The Southampton grading system is a commonly
used system for grading wound infections. It classifies wound infections into four grades based on
clinical signs such as pain, erythema (redness), edema (swelling), discharge, and systemic symptoms.
The grading system helps categorize wound infections into different levels of severity and guides
appropriate management and treatment decisions.
Incorrect option :
Option A. ASA classification: The ASA (American Society of Anesthesiologists) classification is a syste
m used to assess the physical status of patients before surgery. It categorizes patients into different cla
sses based on their overall health and the presence of any comorbidities. While the ASA classification i
s useful for preoperative risk assessment, it is not specifically designed for grading wound infections.
Option B. SIRS criteria: The SIRS (Systemic Inflammatory Response Syndrome) criteria are used to id
entify patients who may have a systemic inflammatory response. SIRS criteria include abnormal body t
emperature (fever or hypothermia), increased heart rate, increased respiratory rate, and abnormal whit
e blood cell count. While these criteria are useful for identifying systemic inflammation, they do not spe
cifically grade wound infections.
Option D. APACHE II score: The APACHE II (Acute Physiology and Chronic Health Evaluation II) scor
e is a scoring system used to assess the severity of illness in critically ill patients. It takes into account
physiological parameters such as vital signs, laboratory values, and age. While the APACHE II score p
rovides valuable information about the overall severity of illness, it is not specifically designed for gradi
ng wound infections.

Solution for Question 105:


Correct option:
Option A.
• Cleaning and dressing: This is the correct option. Abrasions are superficial skin injuries that typically
involve the removal of the epidermal layer. The initial management of an abrasion involves cleaning the
wound thoroughly with a mild antiseptic solution or sterile saline to reduce the risk of infection. After
cleaning, applying a sterile dressing helps protect the wound from contamination and provides a moist
environment that promotes healing.
Incorrect Option:
Option B. Debridement and dressing: Debridement is the removal of dead, damaged, or contaminated t
issue from a wound. While debridement may be necessary for certain types of wounds, such as deep o

Page 94

2062
r contaminated wounds, it is not typically required for simple abrasions. Debridement is more commonl
y performed on wounds with significant tissue damage or signs of infection.
Option C. Cleaning and skin grafting: Skin grafting is a surgical procedure where healthy skin is taken f
rom one area of the body (donor site) and transplanted to cover a wound (recipient site). Skin grafting i
s generally not indicated for simple abrasions. Skin grafting is commonly used for larger wounds, burns
, or wounds with exposed underlying structures that require coverage to facilitate healing.
Option D. Cleaning and use of allograft: An allograft refers to the use of tissue or graft material from a
donor, typically obtained from a tissue bank. While allografts can be used in certain wound manageme
nt scenarios, such as deep or chronic wounds, they are not commonly utilized for simple abrasions. All
ografts are often reserved for more complex wounds that require additional support for healing.

Solution for Question 106:


Correct option:
Option D.
• Bilateral pupillary reflex: This is not a component of the qSOFA score. The qSOFA score focuses on
clinical signs that are easily and quickly assessable at the bedside to identify patients at risk of poor
outcomes in the context of suspected infection. Bilateral pupillary reflex is not directly included in the
qSOFA criteria.
Incorrect Option:
Option A. Respiratory rate ≥ 22/min: This is a
component of the qSOFA (quick Sequential Organ Failure Assessment) score. qSOFA is a bedside cli
nical tool used to identify patients with suspected infection who are at higher risk of poor outcomes. It c
onsists of three criteria: respiratory rate ≥ 22/min, altered mental status, and systolic blood pressure ≤
100 mm Hg. If a patient meets two or more of these criteria, it suggests a higher risk of poor outcomes.
Option B. Systolic BP ≤ 100 mm Hg: This is also a component of the qSOFA score. A systolic blood pr
essure of 100 mm Hg or lower is considered abnormal and indicates potential hemodynamic instability.
Meeting this criterion contributes to a higher qSOFA score.
Option C. Altered mental status: Altered mental status is another component of the qSOFA score. It ref
ers to any deviation from normal mental function, including confusion, disorientation, lethargy, or coma.
Presence of altered mental status contributes to the qSOFA score.

Solution for Question 107:


Correct option:
Option B.
• Respiratory Rate > 20/minute and PaCO2 < 35 mmHg: A respiratory rate greater than 20/minute and
a low PaCO2 level (more than 32 mmHg) are features of SIRS.
Incorrect Option:
Option A. Temperature > 38 degrees Celsius: This is a
feature of SIRS. Elevated body temperature (fever) is one of the criteria for diagnosing SIRS. A temper
ature higher than 38 degrees Celsius (or 100.4 degrees Fahrenheit) is considered abnormal and indica

Page 95

2063
tive of an inflammatory response.
Option C. Pulse rate > 90/minute: This is a feature of SIRS. An increased heart rate (pulse rate) greate
r than 90/minute is one of the criteria for diagnosing SIRS. It reflects the body's response to the system
ic inflammatory process.
Option D. WBC > 12,000/mm3 or < 4,000/mm3: This is a
feature of SIRS. Abnormal white blood cell (WBC) count is another criterion for diagnosing SIRS. A WB
C count greater than 12,000/mm3 (leukocytosis) or less than 4,000/mm3 (leukopenia) indicates an infla
mmatory response in the body.

Solution for Question 108:


Correct Option:
Option B.
• Great auricular nerve: This is the correct option. The great auricular nerve is a sensory branch of the
cervical plexus (specifically, from the C2 and C3 spinal nerves). It provides sensory innervation to the
skin over the parotid region, including the area below the ear and the angle of the mandible. During
parotidectomy, the great auricular nerve can be affected or injured, leading to loss of sensation in the
corresponding area, which can manifest as decreased sensation while shaving.
Incorrect Options:
Option A. Auriculotemporal nerve: This option is incorrect. The auriculotemporal nerve is a branch of th
e trigeminal nerve (mandibular division) that provides sensory innervation to the skin of the temporal re
gion and the external ear. It is not directly involved in sensation loss during shaving after parotidectomy
.
Option C. Facial nerve: This option is incorrect. The facial nerve is primarily responsible for motor inner
vation to the muscles of facial expression. While parotidectomy involves surgical manipulation of the fa
cial nerve, loss of sensation while shaving is not typically associated with facial nerve involvement. Sen
sory deficits are more commonly related to the involvement of other nerves, such as the great auricular
nerve.
Option D. Mandibular nerve: This option is incorrect. The mandibular nerve is one of the divisions of th
e trigeminal nerve (specifically, the third division). It is responsible for providing sensory innervation to t
he lower jaw, teeth, and gums, as well as motor innervation to the muscles of mastication. Loss of sens
ation while shaving after parotidectomy is not directly related to the mandibular nerve.

Solution for Question 109:


Correct option:
Option A.
• Unipolar cautery: The image shows a unipolar cautery, which is an electrical surgical instrument used
for cutting or coagulating tissue during surgical procedures. It consists of a handheld handle with an
attached electrode that delivers electrical current to the target tissue.
Incorrect Option:

Page 96

2064
Option B. Bipolar cautery: Bipolar cautery is another type of electrical surgical instrument used for coag
ulation and hemostasis. Unlike unipolar cautery, which requires a separate grounding pad, bipolar caut
ery uses forceps-like instruments with two electrodes to pass current directly between them. This allow
s for more precise control and localized tissue sealing.
Option C. Harmonic scalpel: The harmonic scalpel is a surgical instrument that uses ultrasonic vibratio
ns to simultaneously cut and coagulate tissue. It operates by converting electrical energy into mechani
cal vibrations, which effectively cut through tissue while sealing blood vessels.
Option D. Ligasure: Ligasure is a brand name for a vessel sealing system that utilizes bipolar energy to
seal blood vessels and tissue during surgery. It is a combination of an instrument with a generator that
delivers energy and specialized disposable tips that provide both cutting and sealing capabilities.

Solution for Question 110:


Correct option:
Option A.
• Veress needle: The Veress needle is a surgical instrument used to create a pneumoperitoneum,
which is the insufflation of carbon dioxide gas into the peritoneal cavity to create a working space
during laparoscopic procedures. The Veress needle is inserted through a small incision or through the
umbilicus, and its tip is equipped with a spring-loaded mechanism that allows for controlled penetration
of the abdominal wall. Once inserted, carbon dioxide gas is introduced through the needle to inflate the
peritoneal cavity.
Incorrect Option:
Option B. Morris' needle: Morris' needle is not the instrument used to create a
pneumoperitoneum. It is a specialized needle used for spermatic cord ligation during a
vasectomy procedure. It is not related to the creation of a pneumoperitoneum.
Option C. Yankauer suction tip: The Yankauer suction tip is a
suctioning instrument commonly used in various surgical procedures. It is a rigid, curved tube with mult
iple openings at the tip and is used for suctioning fluids and debris from the surgical field. It is not used
to create a pneumoperitoneum.
Option D. Von Graefe needle: Von Graefe needle is an ophthalmic instrument primarily used in ophthal
mic surgery. It has a curved sharp tip and is used for delicate procedures in ophthalmology, such as ca
taract surgery and corneal transplantations. It is not used for creating a pneumoperitoneum

Solution for Question 111:


Correct option:
Option A.
• Stage 4: In this stage, the bed sore (also known as a pressure ulcer or decubitus ulcer) exhibits
full-thickness skin loss with extensive destruction, tissue necrosis, or damage to muscle, bone, or
supporting structures. The wound may have slough or eschar, and it can extend beyond the visible
area. In stage 4, there is deep tissue involvement, and the ulcer is typically characterized by a large,
open wound with significant tissue loss.

Page 97

2065
Incorrect Options:
Option B. Stage 3: Stage 3 bed sores involve full-thickness skin loss, but the damage does not extend t
o the underlying tissues. In stage 3, the ulcer may present as a deep crater or have undermining of adj
acent tissue, but there is no direct involvement of muscle, bone, or supporting structures. The wound m
ay contain granulation tissue or slough, but there is no exposed muscle or bone.
Option C. Stage 2: Stage 2 bed sores involve partial-thickness skin loss, typically presenting as a shall
ow, open ulcer or blister. The wound extends through the epidermis and may involve the dermis. In sta
ge 2, the ulcer may appear as an abrasion, blister, or shallow crater. However, it does not extend to de
eper tissue layers.
Option D. Stage 1: Stage 1 bed sores are the earliest stage and involve intact skin with non-blanchable
redness. In this stage, the affected area may be painful, firm, soft, warmer, or cooler compared to the
surrounding skin. Stage 1 ulcers do not exhibit open wounds or visible skin loss.

Solution for Question 112:


Correct Option:
Option D.
• Grey: The grey-colored IV cannula is specifically designed for high-flow rates. It has a larger gauge
size, allowing for a faster and more efficient delivery of fluids. The grey cannula is preferred in
situations where a large amount of fluids needs to be transfused, such as in trauma cases or during
resuscitation efforts.
Incorrect Option :
Option A. Green: The green-colored IV cannula typically has a
smaller gauge size and is suitable for moderate-flow rates. It may not be the ideal choice when a large
amount of fluids needs to be transfused as it may result in slower infusion rates and potential discomfor
t for the patient.
Option B. Blue: The blue-colored IV cannula is commonly used for pediatric patients and for individuals
with smaller veins. It is also suitable for moderate-flow rates. Similar to the green cannula, it may not b
e the optimal choice when a
large amount of fluids needs to be administered due to its limited flow capacity.
Option C. Pink: The pink-colored IV cannula is typically used for neonatal patients and individuals with
extremely small veins. It is designed for low-flow rates and is not suitable for the rapid infusion of a
large volume of fluids.

Solution for Question 113:


Correct Option:
Option B.
• Jackson-Pratt drain: The Jackson-Pratt drain, also known as a JP drain, is a closed suction drainage
system used to remove fluid, such as blood or serous fluid, from a surgical site. It consists of a silicone
or latex drain connected to a collection bulb. The drain is typically placed in the surgical site, and the

Page 98

2066
bulb is compressed to create negative pressure, allowing for the continuous or intermittent drainage of
fluid. The JP drain is commonly used in various surgical procedures to reduce the risk of fluid
accumulation and promote healing.
Incorrect Option:
Option A. Hemovac: The Hemovac drain is a
type of closed suction drain used to remove fluid or blood from a surgical site. It consists of a
plastic reservoir connected to a flexible tube with a compressible bulb at the end. The bulb is compress
ed to create suction and facilitate drainage. While the Hemovac drain serves a
similar purpose as the Jackson-Pratt drain, the two differ in their design and mechanism of operation.

Option C. Minivac: There is no commonly recognized surgical drain known as the "Minivac." It is not a
recognized instrument for drainage purposes.

Option D. Corrugated drain: A corrugated drain, also known as a corrugated rubber drain, is a
flexible drainage tube with a ribbed or corrugated surface. It is typically used to provide drainage in vari
ous surgical or medical procedures, such as in the management of abscesses or wounds. However, th
e corrugated drain is different from the Jackson-Pratt drain in terms of its structure and mechanism of o

Page 99

2067
peration.

Solution for Question 114:


Correct Option B: Tubercular lymphadenitis
• Tubercular lymphadenitis: It Occurs in children & young adults. The Most common affected lymph
nodes are Deep cervical lymph nodes . TB bacilli gain entrance through tonsils.
Incorrect Options
Option A: Branchial cyst: Branchial cleft cysts are embryological anomalies and are defined by the inter
nal opening of the branchial sinuses arising from incomplete obliteration in embryogenesis. It is formed
from the Vestigial remnants of the 2nd branchial cleft. It is Lined by squamous epithelium. It consists o
f Thick, turbid & full of cholesterol crystals. Most common in early and middle adulthood. Located at the
junction of upper and middle 1/3rd of Sternocleidomastoid. On palpation, it is soft, fluctuant and transill
umination positive.
FNAC & Ultrasound helps in diagnosis. Treatment is complete excision of cyst.

Option C: Thyroglossal cyst: A thyroglossal duct cyst is a


mass or lump in the front part of the neck that is filled with fluid. While a baby is developing in the wom
b, the thyroid gland begins at the base of the tongue. Before birth the thyroid gland moves in the neck t
o its usual position below the thyroid cartilage and above the sternum. A thyroglossal duct cyst is an e
mbryologic remnant that forms due to the failure of closure of the thyroglossal duct extending from the f
oramen cecum in the tongue to the thyroid's location in the neck. The thyroid begins to develop in the t
hird week of gestation as a median outgrowth from the primitive pharynx. The treatment for a
thyroglossal duct cyst is surgical removal.
Option D: Lipoma: It is a fatty tumor located just below the skin. It is a Universal Tumor It is the most co
mmon subcutaneous neoplasm and the most common site is Trunk. On palpation is Soft, fleshy, slip a
nd sign is +ve. Dercum’s disease / Adiposa Dolorosa is the multiple painful lipoma seen (neuropathic p
ain). Bracket calcification is seen in lipoma of corpus callosum. Maximum risk of malignant is retroperit
oneal lipoma. Myxomatous degeneration is seen in retroperitoneal Lipoma. Treatment is surgical Excisi

Page 100

2068
on .

Solution for Question 115:


Correct Option B: Wide bore needle insertion in the left 5th ICS followed by an intercostal drain
The given clinical scenario is suggestive of tension pneumothorax so appropriate management in this c
ase would be wide bore needle insertion in the left 5th ICS followed by an intercostal drain
Tension pneumothorax
• It is a clinical Dx on the basis of suspicion.
• Wide bore is needle inserted in 2nd I/C space in the Mid clavicular line (In children) on the basis of
suspicion.
• TOC: ICD insertion in the 5th intercostal space

Incorrect Options
Option A: Wide bore needle insertion in the right 5th ICS: This option is incorrect
Option C: Aspiration of the pleural fluid: A pleural aspiration is a procedure where a small needle or tub
e is inserted into the space between the lung and chest wall to remove fluid that has accumulated arou
nd the lung. This space is called the pleural space. This may be for diagnostic purposes (usually remov
ing 20–50 ml fluid) or therapeutic to relieve symptoms.
Option D: No intervention needed; This option is incorrect

Solution for Question 116:


Correct Option B: Hypovolemic shock

Page 101

2069
• The type of shock seen in a patient with profuse bleeding, cold extremities, and decreased central
venous pressure (CVP) is "Hypovolemic shock"
• Hypovolemic shock occurs when there is a significant loss of blood volume or fluid, resulting in
inadequate circulating volume. Profuse bleeding can cause hypovolemic shock, leading to decreased
blood pressure, cold extremities, and decreased CVP.
Incorrect Options
Option A: Cardiogenic shock: Characterized by impaired ability of heart to pump blood (especially by th
e left ventricle). The causes are Systolic dysfunction (MI and Myocardial depressants - CCB, B-Blocker
s, Anti-arrhythmic), Diastolic dysfunction (Ventricular hypertrophy, Restrictive cardiomyopathy and Car
diac Tamponade), Valvular Structural abnormalities (Papillary muscle rupture and AR &
MR) and Arrhythmias (Ventricular Tachyarrhythmia).
Option C: Obstructive shock: It is a
condition that prevents blood and oxygen from getting to your organs.
• Obstructive Shock are Examples include tension pneumothorax, pericardial tamponade, restrictive
cardiomyopathy, constrictive pericarditis.
• Cardiac Tamponade: BECK’S TRIAD (Muffled heart sound, Distended neck veins, Hypotension) and
Pulsus Paradoxu.
Option D: Hypo adrenal shock: In normal individuals, stress, illness, surgery and trauma causes increa
sed amounts of cortisol release (from adrenal). But unrecognized adrenal insufficiency, complicates the
host response to Stress / illness / Surgery / Trauma which is leading to Hypo adrenal shock. The Caus
es are Chronic administration of high doses of exogenous Glucocorticoids (most common cause), Adre
nal Insufficiency due to (Mnemonics: “MA BETI”):
• Metastatic disease
• Amyloidosis
• Bilateral Adrenal hemorrhage
• Etomidate for intubation
• TB
• Idiopathic atrophy

Solution for Question 117:


Correct Option D: Carbuncle
Carbuncle: It is an infection of the hair follicle(s) that extends into the surrounding skin and deep underl
ying subcutaneous tissue. Abscess with one or more openings and pus draining onto the skin. The Mo
st common organism is Staphylococcus aureus (S aureus) and the Most common site is Nape of neck.
Etiology is more common in Diabetics & Immunocompromised patients. The triggering factors are Folli
culitis, Poor hygiene and Accidental pulling of hair during showing/Friction.
The Clinical features are
• Infected mass filled with Pus, Dead tissue, Fluid.
• It has Red & white with a yellow center.
For the Treatment, Incision & drainage and antibiotics given, also a cruciate incision is given too.

Page 102

2070
Incorrect Option:
Option A: Botryomycosis (or bacterial pseudo mycosis or pyoderma vegetans) is a rare chronic bacteri
al granulomatous disease that usually involves skin and rarely viscera. Common organisms causing bo
tryomycosis include Staphylococcus aureus, Pseudomonas aeruginosa, coagulase-negative staphyloc
occi, Streptococcus spp, Escherichia coli and Proteus spp. Risk factors include alcoholism, diabetes m
ellitus, HIV infection, cystic fibrosis, chronic granulomatous disease, trauma and surgery.
• Botryomycosis is treated with Long-term antibiotics, depending on the bacteria identified, including
trimethoprim-sulfamethoxazole, oral clindamycin, cephalexin, doxycycline and erythromycin. Surgical
drainage and debridement.

Option B: Erysipelas is an acute, sometimes recurrent disease caused by a bacterial infection, charact
erized by large raised red patches on the skin. The patches are well- defined margins. The most comm
on microorganism is streptococcus pyogenes. It is Associated with bulla or Vesicle formation. There is I
nvolvement of upper Subcutaneous tissue of lymphatics. The condition may affect both children and ad
ults. The most Common site is Face with bridge of nose & cheeks.
• Penicillin administered orally or intramuscularly is sufficient for most cases of classic erysipelas and
should be given for 5 days, but if the infection has not improved, treatment duration should be
extended. A first-generation cephalosporin may be used if the patient has an allergy to penicillin.
Option C: Ecthyma is a skin infection. It is similar to impetigo, but occurs deep inside the skin. For this r
eason, ecthyma is often called deep impetigo. Impetigo is a superficial skin infection, caused by Staphy
lococcus aureus, Streptococcus pyogenes, or both, that leads to the formation of scabby, yellow-cruste
d sores and, sometimes, small blisters filled with yellow fluid. Ecthyma is a form of impetigo that cause
s sores deeper in the skin. Oral penicillin is the standard of care for documented streptococcal ecthyma
.

Solution for Question 118:


Correct Option A: Extradural hemorrhage
• Based on the CT image and the given clinical history, the most likely diagnosis is an extradural
hemorrhage
• Extradural hemorrhage, also known as epidural hematoma, is a type of traumatic brain injury where
bleeding occurs between the dura mater (outermost layer of the meninges) and the inner surface of the
skull.
• It is commonly caused by a head injury, such as a direct blow to the head.
• The bleeding typically arises from an injury to the middle meningeal artery.
Incorrect Options:
Option B: Artifact: An artifact refers to any unwanted or unintended image distortion or abnormality that
can occur during imaging. In this case, the CT image is not likely to be an artifact since the clinical hist
ory indicates a head injury and subsequent loss of consciousness.
Option C: Subdural hemorrhage: Subdural hemorrhage is another type of traumatic brain injury where
bleeding occurs between the dura mater and the arachnoid membrane (one of the meninges). It is usu
ally caused by tears in the veins bridging the brain to the dural sinuses. However, the CT image does n
ot show the characteristic crescent-shaped bleeding that is typically seen in subdural hemorrhage.

Page 103

2071
Option D: Subarachnoid hemorrhage: Subarachnoid hemorrhage refers to bleeding that occurs within t
he subarachnoid space, which is the space between the arachnoid membrane and the pia mater (anot
her layer of the meninges). It is commonly caused by the rupture of an intracranial aneurysm or head tr
auma. However, the CT image does not show evidence of blood in the subarachnoid space.

Solution for Question 119:


Correct Option: B.
• Sebaceous cysts can occur in various parts of the body, including the back, scalp, and scrotum.
However, they are not commonly seen on the soles of the feet. Sebaceous cysts are typically found in
areas where there are hair follicles and sebaceous (oil) glands.
Incorrect Options:
Options A. Back: Sebaceous cysts can occur on the back. The back is a
common location for these cysts, along with other areas such as the face, neck, and trunk.
Options C. Scalp: Sebaceous cysts are frequently seen on the scalp. They can develop when the hair f
ollicles or oil glands on the scalp become blocked or damaged.
Options D. Scrotum: Sebaceous cysts can also occur on the scrotum. The scrotum has hair follicles an
d sebaceous glands, making it a potential site for the development of these cysts.

Solution for Question 120:


Correct Option: B.
• Hospital-Acquired Infections (HAIs) are infections that are acquired during a hospital stay or
healthcare facility visit. They are also known as nosocomial infections. These infections are typically
associated with medical procedures, invasive devices, or exposure to healthcare personnel.
Incorrect Options:
Options A. Surgical site infection: Surgical site infections are a common type of Hospital-Acquired Infec
tion. They occur after surgery and can affect the incision site or deeper tissues. Factors such as improp
er surgical technique, contamination, or compromised immune system can contribute to the developme
nt of surgical site infections.
Options C. UTI (Urinary Tract Infection): UTIs can be both community-acquired and hospital-acquired.
However, UTIs are one of the most common types of Hospital-Acquired Infections. They often result fro
m the use of urinary catheters, which can introduce bacteria into the urinary tract.
Options D. Pneumonia: Hospital-Acquired Pneumonia (HAP) is a
type of pneumonia that develops during a hospital stay. It is typically associated with mechanical ventil
ation, aspiration, or exposure to multidrug-resistant organisms in the hospital environment.

Solution for Question 121:


Correct Option B: 5% Dextrose

Page 104

2072
• Our goal in management of head injury is to minimise the cerebral injury and preserve penumbral
tissue. Traumatic brain injury causes interstitial cerebral oedema. When caring for a patient with a
neurological injury, hypotonic fluids and dextrose solutions in free water should be avoided as they
promote cerebral edema.
Incorrect Option:
Option A. 0.9% saline is the most commonly used fluid for resuscitation in patients with acute brain inju
ry as it is the prototypical “isotonic” solution relative to plasma.
Option C. Ringer lactate is an isotonic solution mainly used in aggressive volume resuscitation from blo
od loss or burn injuries; its hypo-osmolarity (273 mmol. L-1) could increase intracellular space volume l
eading to an increase in intracranial pressure in brain-injured trauma patients.
Option D. 3% saline is a hypertonic solution. Hypertonic saline solutions also decrease brain water and
ICP while temporarily increasing systolic blood pressure and cardiac output. They create an osmolar g
radient, which allows cerebrospinal fluid to move from the cranial space, leading to a decrease in ICP.

Solution for Question 122:


Correct Option C: Keloid scar
• The given image and clinical scenario suggests diagnosis of keloid scar.
Incorrect Option:
Option A: Hypertrophic Scar: Has excessive scar tissue. The scar doesn’t extend beyond the boundary
of original wound/incision. It results from prolonged inflammatory phase and unfavourable scar sightin
g (across tension lines).
Option B: Atrophic Scar: Pale, flat and stretched in appearance. Common Site:is Back (in areas of tens
ion). Easily traumatized (due to thin epidermis and dermis). Treatment is Excision &
Resuturing rarely improves it.
Option D: Contractures: An abnormal occurrence that happens when a
large area of skin is damaged and lost, resulting in a
scar. The scar formation pulls the edges of the skin together, causing a tight area of skin. For some pe
ople, scar tissue may cause pain, tightness, itching, or difficulty moving. Due to the way that scar tissue
matures over time, these symptoms may occur years after an injury. Helping scar tissue mature and h
eal at home may improve these symptoms. Best treatment for contracture is physical therapy. Casts an
d splints to keep the joint extended. Medicine to relax the joints. Nerve blocks to numb the affected join
t.

Page 105

2073
Previous Year Questions
1. What is the primary cause of severe malnutrition among the options provided below?
A. Pancreatic fistula
B. Duodenal fistula
C. Distal ileal fistula
D. Colonic fistula
----------------------------------------
2. Which of the following nerves would be involved if a patient experiences a loss of sensations over
the root of the penis after undergoing open hernia surgery?
A. Iliohypogastric nerve
B. Ilioinguinal nerve
C. Genitofemoral nerve
D. Medial cutaneous nerve to thigh
----------------------------------------
3. Which one of the following is absent in cases of massive blood transfusion?
A. DIC
B. Febrile transfusion reaction
C. Hypercalcemia
D. Thrombocytopenia
----------------------------------------
4. Which structures are preserved during radical neck dissection?
A. Digastric muscle
B. Sternocleidomastoid muscle
C. Spinal accessory nerve
D. Ipsilateral cervical lymph nodes
----------------------------------------
5. Most common site of peripheral aneurysm?
A. Brachial artery
B. Popliteal artery
C. Radial artery
D. Femoral artery
----------------------------------------
6. According to GCS, a verbal score of 1 indicates:
A. No Verbal response
B. Inappropriate words

2074
C. Incomprehensible sounds
D. Oriented response
----------------------------------------

Correct Answers
Question Correct Answer

Question 1 2
Question 2 2
Question 3 3
Question 4 1
Question 5 2
Question 6 1

Solution for Question 1:


Correct Option B - Duodenal fistula:
• Duodenal fistulas can lead to significant malnutrition because the duodenum is responsible for
receiving partially digested food from the stomach and plays a critical role in the absorption of nutrients,
including proteins, fats, carbohydrates, vitamins, and minerals.
• When a duodenal fistula is present, there is a direct connection between the duodenum and an
abnormal opening, leading to significant nutrient loss and malnutrition.
Incorrect Options: Severe malnutrition is more commonly seen with duodenal fistula, hence options A,
C and D are incorrect.

Solution for Question 2:


Correct Option B - Ilioinguinal nerve:
• The ilioinguinal nerve arises from the first lumbar nerve and travels through the inguinal canal,
providing sensory innervation to the upper medial thigh, the root of the penis, and the mons pubis in
males.
• During open hernia surgery, the ilioinguinal nerve can be injured or irritated, leading to the loss of
sensation over the root of the penis.
Incorrect Options:
Options A, C, and D are incorrect.

Solution for Question 3:


Correct Option C: Hypercalcemia

Page 2

2075
• Hypercalcemia: This option is the correct answer. Hypercalcemia, which is an elevated level of
calcium in the blood, is not typically associated with massive blood transfusion. Hypercalcemia can be
caused by various factors, such as certain medical conditions, medications, or hormonal imbalances,
but it is not a direct consequence of blood transfusion.
Incorrect options
Option A: DIC (Disseminated Intravascular Coagulation): DIC is a
serious condition characterized by abnormal blood clotting and excessive bleeding. It can occur as a c
omplication of various medical conditions, including massive blood transfusion. When large amounts of
blood are transfused rapidly, it can lead to the release of procoagulant substances, triggering DIC. The
refore, DIC can be seen in cases of massive blood transfusion.
Option B: Febrile transfusion reaction: Febrile transfusion reaction is very common after transfusion
Option D: Thrombocytopenia: Thrombocytopenia, which refers to a
low platelet count, can occur during or after a massive blood transfusion. Platelets, which play a crucial
role in blood clotting, can be depleted or damaged during the storage or transfusion process. This can
result in decreased platelet count and function, leading to thrombocytopenia. Therefore, thrombocytope
nia is a potential complication of massive blood transfusion.

Solution for Question 4:


Correct Option A.
Digastric muscle: This is the correct option. The digastric muscle, which is responsible for opening and
closing the jaw, is typically spared during a radical neck dissection. Preserving the digastric muscle hel
ps maintain normal jaw function and prevents complications related to swallowing and speech.
Incorrect Options:
Option B. Sternocleidomastoid muscle: This option is incorrect. The sternocleidomastoid muscle, locat
ed on the side of the neck, is often partially or completely removed during a radical neck dissection. Its
removal allows access to deeper structures and lymph nodes in the neck, especially in cases where ca
ncer has spread.
Option C. Spinal accessory nerve: This option is incorrect. The spinal accessory nerve (also known as
cranial nerve XI) is commonly sacrificed during a radical neck dissection. This nerve innervates the ster
nocleidomastoid muscle and the trapezius muscle. Sacrificing the spinal accessory nerve can result in
shoulder weakness or dysfunction.
Option D. Ipsilateral cervical lymph nodes: This option is incorrect. Ipsilateral cervical lymph nodes, ref
erring to lymph nodes on the same side as the affected neck, are typically removed during a radical ne
ck dissection. The procedure aims to remove the lymph nodes that may contain cancer cells or be at ri
sk of involvement.

Solution for Question 5:


Correct Option B: Popliteal artery
• Popliteal artery: This option is the correct answer. The popliteal artery is located behind the knee and
is a common site for peripheral aneurysm. Popliteal artery aneurysms are often associated with
degenerative changes in the arterial wall, such as atherosclerosis. These aneurysms can lead to the

Page 3

2076
development of a pulsatile mass behind the knee and may increase the risk of complications such as
thrombosis or embolism.
Incorrect Options:
Option A: Brachial artery: The brachial artery is located in the upper arm and is responsible for supplyin
g blood to the arm. While aneurysms can occur in the brachial artery, they are not as common as in oth
er peripheral arteries.
Option C: Radial artery: The radial artery is located in the forearm and is a major artery responsible for
supplying blood to the hand. While aneurysms can occur in the radial artery, they are less common co
mpared to other sites.
Option D: Femoral artery: The femoral artery is located in the groin region and is a major artery that su
pplies blood to the lower extremities. Aneurysms can develop in the femoral artery, but they are not as
common as in the popliteal artery.

Solution for Question 6:


Correct Option A: No verbal response
• No response: A verbal score of 1 on the Glasgow Coma Scale indicates "no response." This means
that the patient does not make any verbal sounds or utter any words, even in response to stimuli or
commands. It suggests a complete absence of verbal communication.
Incorrect Options:
Option B: Inappropriate words: In the Glasgow Coma Scale, an inappropriate verbal response is given
a score of 3. This means that the patient's verbalizations do not relate to the situation or are not approp
riate. For example, they may use words or phrases that are unrelated to the questions or commands gi
ven.
Option C: Incomprehensible sounds: In the Glasgow Coma Scale, incomprehensible sounds are assig
ned a verbal score of 2. This refers to sounds made by the patient that do not form coherent words or p
hrases. It may include moaning, groaning, or other unintelligible vocalizations.
Option D: Oriented response: Oriented response is given a verbal score of 5
on the Glasgow Coma Scale.

Page 4

2077
Previous Year Questions
1. What is the ideal time and method of removing hair before surgery to avoid surgical site infection?
(or)
For the purpose of preventing surgical site infection, what is the recommended technique and optimal
timing for hair removal prior to surgery?
A. On the OT table with electric hair clippers
B. With a razor 24 hours before surgery
C. With a hair removal cream on the previous day
D. On the OT table with a razor
----------------------------------------
2. A patient arrives at the emergency department after being involved in a car accident. The patient is
experiencing difficulty breathing and their neck veins appear dilated. The x-ray reveals a significant
amount of air on the left side. What is the most appropriate course of action for treatment?

A. Wide bore needle insertion in the 5th intercostal space


B. Endotracheal intubation
C. Tracheostomy
D. Pericardiocentesis
----------------------------------------
3. A patient sustained a crush injury on their right arm. He was taken to the hospital and on
examination, his arm was pulseless, swollen, and dark red. What would be the best way to manage this
case?
A. Intraosseous administration of fluids
B. Acidification of urine
C. Fasciotomy
D. Hemodialysis
----------------------------------------
4. What should be the next course of action in the management of a 30-year-old breastfeeding mother
who is experiencing pain and fever in her left breast? The left breast appears abnormal as depicted in
the provided image. There is slight swelling of the lymph nodes in the armpit on the affected side, while

2078
the opposite breast appears normal. Despite multiple attempts at draining the fluid with the use of
antibiotics, it was unsuccessful. An ultrasound examination revealed the presence of fluid collection.

A. Incision an drainage
B. Broad spectrum antibiotics only
C. Breast conservation surgery
D. Punch biopsy
----------------------------------------
5. In the case of a woman who has suffered third and fourth degree burns on her back, what would be
the appropriate sequence of steps in her treatment?
A. Resuscitation – reconstruction – debridement
B. Resuscitation – debridement – allograft or autograft
C. Debridement – resuscitation – autograft
D. Debridement – reconstruction – resuscitation
----------------------------------------
6. Which of the following carries the lowest risk of developing invasive ductal carcinoma?
A. Florid epithelial hyperplasia
B. Papilloma with fibrovascular core
C. Atypical hyperplasia
D. Squamous metaplasia
----------------------------------------
7. What is the probable diagnosis for a 30-year-old woman who complains of a painless mass in her left
breast that can be felt? The mass is highly movable, measures 3 x 3 cm, and has a rubbery texture.
A. Breast abscess
B. Phyllodes tumor
C. Fibroadenoma
D. Invasive ductal carcinoma
----------------------------------------
8. A 50-year-old diabetic man presents with a progressive increase in pain in his calf. He now prefers to
sleep in a seated position by hanging his leg to alleviate pain.Pain is more in the distal part like toes

Page 2

2079
and feet. It gets aggravated with movements and pressure. The pain is increased in lying down and
elevation of foot; it may be reduced on hanging the foot down. Which of the following is the most likely
cause?
A. Rest pain
B. Sciatica
C. Neurological claudication
D. Venous claudication
----------------------------------------
9. What is the classification of the head injury of a patient involved in a road traffic accident, based on a
GCS score of 10?
A. Moderate traumatic brain injury
B. Severe traumatic brain injury
C. Minor traumatic brain injury
D. Mild traumatic brain injury
----------------------------------------
10. In a patient requiring a significant volume of fluids, which colored cannula would be most
appropriate to achieve the highest flow rate?
A. Gray
B. Green
C. Blue
D. Pink
----------------------------------------
11. Calculate the Glasgow Coma Scale (GCS) of a patient who has been admitted after a road traffic
accident and is currently receiving mechanical ventilation. The patient demonstrates a response by
opening their eyes upon verbal command and exhibits spontaneous movement in all four limbs.
A. Eyes-2, Verbal-1, Motor-5
B. Eyes-2, Verbal-NT, Motor-5
C. Eyes-3, Verbal-1, Motor-6
D. Eyes-3, Verbal-NT, Motor-6
----------------------------------------
12. After a road traffic accident, a patient was brought to the emergency department with chest injuries.
The patient's respiratory rate is elevated at 44 breaths per minute, and their blood pressure is 90/40
mm/Hg. Although conscious, the patient can only speak single words. On examination,
hyper-resonance is noted on the affected side of the chest. What would be your next immediate course
of action?
A. Intubate
B. Insert needle in appropriate intercostal space
C. Start intravenous fluids urgently
D. Take a chest X-ray

Page 3

2080
----------------------------------------
13. What is the likely diagnosis for a 40-year-old woman who comes to the emergency room after a
road traffic accident? She has a Glasgow Coma Scale score of 15/15, a heart rate of 110 beats per
minute, a blood pressure of 90/60 mmHg, and an elevated jugular venous pressure. Percussion reveals
hyper-resonance and decreased breath sounds in the right chest, while the left chest appears normal.
However, the patient is experiencing difficulty breathing and there is bruising on the chest wall. The
abdomen feels soft.
A. Cardiac tamponade
B. Tension pneumothorax
C. Pericardial effusion
D. Massive hemothorax
----------------------------------------
14. During an emergency laparotomy performed for a patient with intestinal obstruction, which organ
would you initially observe to determine whether it is a case of small bowel or large bowel obstruction?
A. Ileum
B. Caecum
C. Jejunum
D. Sigmoid colon
----------------------------------------
15. What is the hue of the 16F Foley's catheter?
A. Orange
B. Blue
C. Yellow
D. Black
----------------------------------------
16. Which fluid is administered most commonly during intraoperative blood loss?
A. Isolyte P
B. Crystalloids
C. Blood
D. Colloids
----------------------------------------
17. A 45-year-old male presents with abdominal pain in the emergency room. His CT abdomen appears
normal, but the patient is hypotensive. On examination, there is blackish discoloration involving the
perineal skin. What is the most likely diagnosis for this presentation?

Page 4

2081
A. Acute pancreatitis
B. Necrotizing fasciitis
C. Chronic pancreatitis
D. Traumatic fat necrosis
----------------------------------------
18. A 56-year-old patient presented with varicose veins. What is the investigation of choice for varicose
veins?
A. Duplex scan
B. Plethysmography
C. MRI
D. CT
----------------------------------------
19. Which of the following is not correct for the device depicted below:

A. Based on infrared emitted from RBCs


B. Localizes calcified plaque
C. Checks blood flow
D. Checks thickness of artery and vein
----------------------------------------

Correct Answers

Page 5

2082
Question Correct Answer

Question 1 1
Question 2 1
Question 3 3
Question 4 1
Question 5 2
Question 6 4
Question 7 3
Question 8 1
Question 9 1
Question 10 1
Question 11 4
Question 12 2
Question 13 2
Question 14 2
Question 15 1
Question 16 2
Question 17 1
Question 18 1
Question 19 1

Solution for Question 1:


Correct Option A: On the OT table with electric hair clippers
• The ideal time and method of removing hair before surgery to avoid surgical site infection are on the
OT table with electric hair clippers. This method is preferred over the other options because it is
considered to be the safest and most effective method for hair removal.
• Electric clippers are less likely to cause nicks, cuts, or skin irritation than a razor or hair removal
cream. Moreover, electric clippers are designed to cut the hair close to the skin without causing any
damage to the skin itself.
Incorrect Options:
Option B: Removing hair with a razor 24 hours before surgery is not recommended because it can cau
se microscopic cuts on the skin, increasing the risk of infection.
Option C: Hair removal creams can cause skin irritation, which can also increase the risk of infection.
Option D: Removing hair on the OT table with a razor is not a recommended method because it increa
ses the risk of infection due to the fact that it is more difficult to maintain a
sterile environment in the operating room when using a razor.

Page 6

2083
Solution for Question 2:
Correct Option A:
• This scenario's next best management step is the wide bore needle insertion in the 5th intercostal
space. This is indicative of a tension pneumothorax, which is a medical emergency that requires
immediate treatment.
• A tension pneumothorax occurs when air enters the pleural space (between the lungs and the chest
wall) and becomes trapped. As more air accumulates, it can cause a shift in the position of the
mediastinum (the area in the chest between the lungs that contains the heart, great vessels, and other
structures), leading to compression of the lungs and blood vessels. This can result in respiratory
distress, hypoxemia (low oxygen levels in the blood), and cardiovascular collapse.
• Treating of a tension pneumothorax involves relieving the pressure in the pleural space by inserting a
needle or catheter into the chest cavity to allow the trapped air to escape. The insertion site is typically
in the 5th intercostal space in the midclavicular line (i.e., the space between the fifth and sixth ribs, in
line with the middle of the collarbone). This procedure is known as needle decompression or
thoracostomy.
Incorrect Options:
Option B: Endotracheal intubation involves placing a tube through the mouth or nose into the trachea t
o provide mechanical ventilation and oxygenation. While this may be necessary in some cases of respi
ratory distress, it does not address the underlying problem of a tension pneumothorax.
Option C: Tracheostomy is a surgical procedure that involves creating a
small opening in the neck and inserting a tube directly into the trachea. This is typically done in upper a
irway obstruction or long-term mechanical ventilation cases. Again, this does not address the underlyin
g problem of a tension pneumothorax.
Option D: Pericardiocentesis involves inserting a needle into the pericardial sac (the membrane surrou
nding the heart) to drain fluid or blood that may be compressing the heart. While this can be a life-savin
g procedure in cases of cardiac tamponade (heart compression due to fluid or blood in the pericardial s
ac), it is not indicated in a tension pneumothorax.

Solution for Question 3:


• The best way to manage a patient with a crush injury to the arm that presents with a pulseless,
swollen, and dark red arm is fasciotomy.
• A crush injury can result in a significant amount of tissue damage, which can lead to the development
of compartment syndrome. Compartment syndrome occurs when there is increased pressure within a
muscle compartment, which can result in tissue damage, nerve injury, and even loss of limb function. In
this case, the patient's pulseless, swollen, and dark red arm is likely indicative of compartment
syndrome, which is a medical emergency that requires immediate intervention to prevent irreversible
tissue damage.
• Fasciotomy is a surgical procedure that involves making an incision into the affected compartment to
relieve pressure and improve blood flow to the affected tissues. It is the recommended treatment for
compartment syndrome in cases where conservative measures, such as elevation and pain
management, are ineffective.

Page 7

2084
Incorrect Choices:
• Option a: Intraosseous administration of fluids is a method of delivering fluids and medications directly
into the bone marrow, bypassing the need for venous access. While this may be useful in certain
situations, such as when venous access is difficult to obtain, it is not the appropriate treatment for
compartment syndrome.
• Option b: Acidification of urine may be used in the management of certain types of poisoning, such as
aspirin overdose, but it is not indicated in the treatment of a crush injury.
• Option d: Hemodialysis is a treatment for kidney failure and is not indicated in the treatment of a crush
injury.

Solution for Question 4:


Correct choice :
• Option a :The clinical scenario described suggests that the patient may have developed a breast
abscess. Breast abscesses are typically caused by bacterial infection and are more common in
lactating women due to the increased risk of bacterial colonization of the breast ducts. The presence of
fever, pain, and fluid collection seen on ultrasound are also consistent with a breast abscess.
• Repeated aspirations under antibiotic cover have been unsuccessful, suggesting that the abscess is
not resolving with conservative management alone. Therefore, the next best step in the management of
this patient would be incision and drainage of the abscess.
• Incision and drainage is the most effective way to treat a breast abscess, as it allows for the complete
removal of the infected material and helps to prevent recurrence.
Incorrect Choices:
• Option b: Broad-spectrum antibiotics may be used in conjunction with incision and drainage to treat
the infection, but antibiotics alone are not sufficient for the management of a breast abscess.
• Option c: Breast conservation surgery typically involves the removal of a breast lump or tumor. Thus,
breast conservation surgery is not appropriate management option for a breast abscess.
• Option d: Punch biopsy is a diagnostic procedure used to obtain a tissue sample for analysis.
• Thus, punch biopsy is not appropriate management option for a breast abscess.

Solution for Question 5:


• Third-degree burns affect the full thickness of the skin and can cause damage to underlying tissue,
while fourth-degree burns extend into the muscle, bone, or internal organs. Both types of burns require
prompt and aggressive treatment to prevent complications such as infection, hypovolemia, and shock.
• The most appropriate order of sequence in the management of a woman who has sustained third and
fourth-degree burns on her back is as follows:
• Resuscitation: The first step in the management of severe burns is resuscitation. This involves
assessing and stabilizing the patient's airway, breathing, and circulation. Intravenous fluids and
electrolytes are administered to prevent hypovolemia and maintain hemodynamic stability.

Page 8

2085
• Debridement: Once the patient is stabilized, the next step is debridement. Debridement involves
removing all necrotic tissue and debris from the wound to prevent infection and promote healing.
Surgical or non-surgical methods can do debridement.
• Allograft or autograft: After debridement, the wound is covered with either an allograft or autograft.
Allografts are skin grafts taken from another person, while autografts are skin grafts taken from the
patient's own body. Both types of grafts can provide temporary coverage to the wound and promote
wound healing.
• Therefore, the correct option would be Resuscitation – Debridement – Allograft or Autograft.
Incorrect Choices:
• Option a. Resuscitation – reconstruction – debridement is incorrect because Ressuscitation –
Debridement – Allograft or Autograft is the correct sequence.
• Option c: Debridement – resuscitation – autograft is incorrect because Resuscitation – Debridement –
Allograft or Autograft is the correct sequence.
• Option d: Reconstruction is not an appropriate step in the initial management of third and
fourth-degree burns, as the focus should be on stabilizing the patient and preventing complications.
Reconstruction is typically done later in the treatment process once the wound has healed. Thus,
Resuscitation – Debridement – Allograft or Autograft is the correct sequence.

Solution for Question 6:


Correct choice :
• Option 4 :Invasive ductal carcinoma (IDC) is the most common type of breast cancer, accounting for
around 70-80% of all cases. It develops from the cells that line the milk ducts and can invade
surrounding tissues.
• Squamous metaplasia is a benign breast condition that is characterized by a change in the normal
breast tissue cells to a squamous cell type. Squamous metaplasia is not considered a risk factor for
developing breast cancer.
Incorrect Choices:
• Option a: Florid epithelial hyperplasia is a benign breast condition that is characterized by an
overgrowth of cells in the ducts and lobules of the breast. While florid epithelial hyperplasia is not
cancerous, it can be a risk factor for developing breast cancer.
• Option b: Papilloma with fibrovascular core is a benign growth that develops in the ducts of the breast.
It is usually small and does not cause any symptoms, but in some cases, it can cause nipple discharge.
Like florid epithelial hyperplasia, a papilloma can be a risk factor for developing breast cancer.
• Option c: atypical hyperplasia is a benign breast condition that is characterized by abnormal growth of
cells in the ducts and lobules of the breast. Atypical hyperplasia is considered a high-risk lesion and is
associated with an increased risk of developing breast cancer.

Solution for Question 7:


• The most likely diagnosis, in this case, is fibroadenoma.

Page 9

2086
• Fibroadenomas are benign (non-cancerous) breast tumors that are common in women between the
ages of 20 and 40. They are made up of glandular and connective tissue and are often
hormone-responsive, meaning they can change in size with hormonal fluctuations.
• On examination, fibroadenomas typically present as a well-circumscribed, mobile, and painless mass
in the breast. They are usually rubbery or firm in consistency and can vary in size.
Incorrect Choices:
• Option a: Breast abscesses, on the other hand, are typically associated with redness, warmth, and
tenderness in the breast, as well as fever and other signs of infection.
• Option b: Phyllodes tumors are another type of breast tumor that can be mistaken for a fibroadenoma,
but they tend to be larger in size and have a more irregular shape.
• Option d: : IDC often presents as a breast lump, which may feel hard or have an irregular shape, but it
is typically not as mobile or well-circumscribed as a fibroadenoma. IDC may cause the nipple to
become inverted or to produce discharge. Invasive ductal carcinoma is not typically associated with
redness, warmth, or tenderness in the breast

Solution for Question 8:


Correct option :
• Option a: The most likely cause of the patient's symptoms, based on the provided information, is rest
pain.
• Rest pain
• It is continuous aching in calf or feet and toes or in the region even at rest depending on site of
obstruction.
• It is a ‘cry of dying nerves’ due to ischemia of the somatic nerves. It signifies severe decompensated
ischemia.
• Pain gets aggravated by elevation and is relieved in dependent position of the limb
• Pain is more in the distal part like toes and feet. It gets aggravated with movements and pressure.
• Rest pain is increased in lying down and elevation of foot; it may be reduced on hanging the foot
down.
It is continuous aching in calf or feet and toes or in the region even at rest depending on site of obstruct
ion.
It is a ‘cry of dying nerves’ due to ischemia of the somatic nerves. It signifies severe decompensated is
chemia.
Pain gets aggravated by elevation and is relieved in dependent position of the limb
Pain is more in the distal part like toes and feet. It gets aggravated with movements and pressure.
Rest pain is increased in lying down and elevation of foot; it may be reduced on hanging the foot down.
Incorrect Choices:
• Option b: Sciatica is a condition that causes pain, numbness, or weakness in the lower back, buttocks,
and legs due to compression or irritation of the sciatic nerve. However, the patient's symptoms of
worsening pain with walking and relief by hanging the leg are inconsistent with sciatica.

Page 10

2087
• Option c: Neurogenic claudication is another possible cause of the patient's symptoms, which occurs
due to spinal stenosis or nerve root compression, causing pain, numbness, and weakness in the lower
back, buttocks, and legs. However, the patient's symptoms of worsening pain with walking and relief by
hanging the leg are not typical of neurogenic claudication.
• Option d: Venous claudication is caused by chronic venous insufficiency, leading to decreased blood
flow and oxygen supply to the affected tissues. However, the patient's symptoms of worsening pain
with walking and relief by hanging the leg are not typical of venous claudication.

Solution for Question 9:


Answer option A
• The GCS (Glasgow Coma Scale) scoring system is used to assess the level of consciousness in
patients with traumatic brain injury. It is a scale that ranges from 3 to 15, with lower scores indicating
more severe injury and higher scores indicating a better level of consciousness.
• In this case, the patient's GCS score is 10, which falls in the range of 9-12. According to the GCS
scoring system, this indicates moderate traumatic brain injury.
• The GCS scoring system classifies traumatic brain injury as follows:
• Severe traumatic brain injury: GCS score of 8 or less
• Moderate traumatic brain injury: GCS score of 9-12
• Minor traumatic brain injury: GCS score of 13-15 with loss of consciousness
• Mild traumatic brain injury: GCS score of 13-15 without loss of consciousness
• Therefore, in this case, the patient's head injury is classified as moderate traumatic brain injury based
on the GCS score of 10. This indicates that the patient has significant brain injury and may require
close monitoring and treatment.
Incorrect Choices:
• Option b. Severe traumatic brain injury is incorrect because GCS score of the patient is 10.
• Option c. Minor traumatic brain injury is incorrect because GCS score of the patient is 10.
• Option d. Mild traumatic brain injury is incorrect because GCS score of the patient is 10.

Solution for Question 10:


Correct Option A.
• Grey cannula: The grey color is typically associated with the largest gauge size,16G. It allows for a
high flow rate and is suitable for patients who require a large amount of fluids or rapid fluid
administration.
Incorrect Options:
Option B. Green cannula: The green color is usually associated with a
medium gauge size, 18G. It provides a
moderate flow rate and is commonly used for general fluid administration or blood transfusions.

Page 11

2088
Option C. Blue cannula: The blue color is typically associated with a smaller gauge size, 22G .
It allows for a lower flow rate and is commonly used for patients who require slower or controlled fluid a
dministration, such as in pediatric or geriatric populations.
Option D. Pink cannula: The pink color is typically associated with an even smaller gauge size, 20G. It
allows for a very low flow rate and is commonly used for delicate procedures, such as administering m
edications or fluids through small veins.

Solution for Question 11:


Correct Option D.
• Eyes-3, Verbal-NT, Motor-6: This option indicates that the patient opens eyes spontaneously, but the
verbal response is not testable (NT), and the patient exhibits purposeful motor response. Since the
verbal response is not provided, the total GCS score cannot be determined.
Incorrect Options:
Option A. Eyes-2, Verbal-1, Motor-5: This option suggests that the patient opens their eyes in response
to pain, gives an inappropriate verbal response, and exhibits localized motor response. The total GCS
score would be 8.
Option B. Eyes-2, Verbal-NT, Motor-5: This option indicates that the patient opens their eyes in respon
se to pain, but the verbal response is not testable (NT), and the patient exhibits a localized motor respo
nse. Since the verbal response is not provided, the total GCS score cannot be determined.
Option C. Eyes-3, Verbal-1, Motor-6: This option suggests that the patient opens their eyes spontaneo
usly, gives an inappropriate verbal response, and exhibits purposeful motor response. The total GCS s
core would be 10.

Solution for Question 12:


Correct Option B.
• Needle : In some cases, a small pneumothorax can be managed by needle aspiration. A healthcare
professional inserts a needle or catheter into the chest to remove the trapped air and allow the lung to
re-expand. This procedure may be performed at the bedside or in a medical facility, and it is typically
guided by imaging, such as ultrasound or chest X-ray.
• Put a large bore needle into the 2ndI/C space in the Midclavicular line
• Definitive Rx/ TOC- ICD insertion in Triangle of safety (5th I/C space in Anterior-axillary line)

Page 12

2089
Incorrect Options:
Option A, intubation, may be necessary if the patient's respiratory status deteriorates further, but it is n
ot the immediate step to take.
Option C, starting intravenous fluids urgently, is important for stabilizing the patient's blood pressure, b
ut it may not address the underlying issue of tension pneumothorax causing hyper-resonance and resp
iratory distress.
Option D, taking a chest X-ray, can help confirm the diagnosis and guide further management, but in a
critically unstable patient with signs of tension pneumothorax, immediate intervention is required before
obtaining diagnostic imaging. Therefore, the most appropriate immediate step to relieve the tension pn
eumothorax and improve the patient's respiratory status would be to insert a needle in the appropriate i
ntercostal space, allowing air to escape and reducing the pressure in the pleural cavity.

Solution for Question 13:


Correct Option: B
• The probable diagnosis, in this case, is B. Tension pneumothorax.
• Tension pneumothorax is a medical emergency characterized by the accumulation of air in the pleural
space, leading to increased pressure and compression of the lung. It typically occurs as a result of
trauma or injury to the chest, such as a rib fracture or lung laceration.
• In this scenario, the patient's history (RTA) suggests the possibility of chest trauma. The findings on
examination provide clues to the diagnosis: Hyperresonant percussion and decreased breath sounds
on the right side: This indicates the presence of air in the pleural space, resulting in a collapsed lung.
Dyspnea: The patient's difficulty in breathing is likely due to the collapsed lung and impaired gas
exchange. Elevated jugular venous pressure (JVP): The increased JVP may be observed in tension
pneumothorax due to impaired venous return to the heart as a result of increased intrathoracic
pressure. Ecchymosis over the chest wall: This bruising may suggest underlying trauma or injury to the
chest.

Page 13

2090
• Hyperresonant percussion and decreased breath sounds on the right side: This indicates the
presence of air in the pleural space, resulting in a collapsed lung.
• Dyspnea: The patient's difficulty in breathing is likely due to the collapsed lung and impaired gas
exchange.
• Elevated jugular venous pressure (JVP): The increased JVP may be observed in tension
pneumothorax due to impaired venous return to the heart as a result of increased intrathoracic
pressure.
• Ecchymosis over the chest wall: This bruising may suggest underlying trauma or injury to the chest.
• Hyperresonant percussion and decreased breath sounds on the right side: This indicates the
presence of air in the pleural space, resulting in a collapsed lung.
• Dyspnea: The patient's difficulty in breathing is likely due to the collapsed lung and impaired gas
exchange.
• Elevated jugular venous pressure (JVP): The increased JVP may be observed in tension
pneumothorax due to impaired venous return to the heart as a result of increased intrathoracic
pressure.
• Ecchymosis over the chest wall: This bruising may suggest underlying trauma or injury to the chest.
Incorrect Options:
The other options can be ruled out based on the clinical presentation:
Option A. Cardiac tamponade typically presents with signs of hemodynamic compromises, such as low
blood pressure, narrowed pulse pressure, and distant heart sounds. The absence of these findings ma
kes it less likely.
Option C. Pericardial effusion may cause symptoms related to cardiac compressions, such as muffled
heart sounds, jugular venous distention, and signs of heart failure. These findings are not present in thi
s case.
Option D. Massive hemothorax involves the accumulation of blood in the pleural space, leading to simil
ar symptoms as tension pneumothorax. However, in this scenario, the hyper resonant percussion and
decreased breath sounds suggest the presence of air rather than blood.
Prompt recognition and management of tension pneumothorax are essential. Immediate intervention, s
uch as needle decompression or chest tube insertion, is necessary to relieve the pressure, re-expand t
he lung, and restore normal breathing.

Solution for Question 14:


Correct Option: B
• Caecum: The caecum is the first part of the large intestine, situated in the lower right abdomen. It
receives the contents from the ileum and is involved in the absorption of water and electrolytes.
Visualizing the caecum during an emergency laparotomy can help determine if the obstruction is in the
large bowel.
Incorrect Options:
Option A. Ileum: The ileum is the final part of the small intestine, located between the jejunum and the l
arge intestine (cecum). It is responsible for the absorption of nutrients. In cases of small bowel obstruct

Page 14

2091
ion, the ileum can become dilated and distended.
Option C. Jejunum: The jejunum is the middle part of the small intestine, located between the duodenu
m and the ileum. It plays a crucial role in nutrient absorption. In cases of small bowel obstruction, the je
junum can also become distended.
Option D. Sigmoid colon: The sigmoid colon is the S-shaped portion of the large intestine that connects
the descending colon to the rectum. It is the final part of the colon before the rectum. Sigmoid colon ob
struction typically presents as a
distinct entity and is not commonly confused with small bowel obstruction.

Solution for Question 15:


Correct Option A:
• Foley's catheters are commonly used for urinary catheterization. The color-coding of Foley's catheters
may vary between manufacturers and regions. However, I can provide you with a general overview of
the color-coding commonly used for Foley's catheters.
• Orange: In some color-coding systems, an orange color may be used for a specific size or type of
Foley's catheter. However, it is important to note that color-coding can vary, so it's essential to refer to
the manufacturer's guidelines or specific institutional protocols for accurate information.
Incorrect Options:
Option B. Blue: Blue-colored Foley's catheters are also used in some color-coding systems. Again, it is
crucial to refer to specific guidelines or protocols to determine the intended meaning of the color.
Option C. Yellow: Yellow is another color that may be used in certain systems to designate a particular
size or type of Foley catheter. However, as mentioned earlier, color-coding can differ, so it's important t
o consult the appropriate guidelines.
Option D. Black: Black is not commonly used as a
color for Foley catheters in standard color-coding systems. It is not a
typical option for identifying the size or type of catheter.

Solution for Question 16:


Correct option B - Crystalloids:
• Crystalloids are the most commonly administered fluid for intraoperative fluid administration,
especially in minor surgical procedures.
• Crystalloids are cost-effective, readily available, and generally safe.
Incorrect options: Option A, C and D are incorrect.

Solution for Question 17:

Page 15

2092
Option A - Acute Pancreatitis: Acute pancreatitis is characterized by inflammation of the pancreas and
can cause severe abdominal pain. However, it typically doesn't lead to blackish discoloration of the peri
neal skin. Hypotension may occur in severe cases due to systemic inflammation and fluid shifts, but th
e perineal discoloration is not a common feature of acute pancreatitis.
• Presents with cullens sign

Incorrect Options:
Option B - Necrotizing Fasciitis:
• Deeper tissue injury, usually from anaerobic bacteria or S pyogenes. Pain may be out of proportion to
exam findings.
• Results in crepitus from methane and CO2 production.
• “Flesh-eating bacteria.” Causes bullae and skin necrosis causing violaceous color of bullae,
surrounding skin .
• Surgical emergency
Option C - Chronic Pancreatitis: Chronic pancreatitis is a long-standing inflammation of the pancreas, o
ften due to alcohol abuse or other underlying factors. It leads to persistent abdominal pain, digestive is
sues, and potentially complications over time. However, it is unlikely to cause the rapid onset of perine
al skin discoloration and hypotension as seen in the given presentation.
Option D - Traumatic Fat Necrosis: Traumatic fat necrosis can occur as a result of physical trauma to f
atty tissues, leading to localized inflammation and tissue damage. While it can cause skin discoloration
, it typically presents as a localized issue rather than involving the perineal skin. It's also less likely to c
ause the severe systemic symptoms, including hypotension, seen in this case.

Solution for Question 18:


Correct option A - Duplex scan:
• Duplex scan is the investigation of choice for varicose veins.

Page 16

2093
• It is a non-invasive imaging technique that combines conventional ultrasound with Doppler ultrasound.
• It allows visualization of blood flow in the veins and can identify abnormalities, such as reflux
(backward flow of blood) in the veins, which is a common finding in varicose veins.
• It provides valuable information about the anatomy and function of the veins, helping to guide
treatment decisions.
Incorrect options:
Option B - Plethysmography:
• Plethysmography is a technique used to measure changes in the volume of an organ or limb.
• While it can provide information about blood volume changes, it is not as specific or accurate in
diagnosing the underlying causes of varicose veins as duplex ultrasound.
Option C - MRI (Magnetic Resonance Imaging):
• While MRI can provide detailed images of soft tissues, it is usually reserved for more complex cases
or when other imaging methods are inconclusive.
• Duplex ultrasound is preferred due to its cost-effectiveness and accuracy in diagnosing varicose
veins.
Option D - CT (Computed Tomography):
• CT scans involve ionizing radiation and are generally used for specific indications.
• Duplex ultrasound is safer, non-invasive and more suitable for evaluating venous abnormalities like
varicose veins.

Solution for Question 19:


Correct option A - Based on infrared emitted from RBCs:
• The device depicted is a Doppler ultrasound machine, which uses sound waves and not infrared
emitted from RBCs, to create images of blood flow.
• Doppler ultrasound works by measuring the frequency shift of sound waves reflected off moving red
blood cells.
• This shift in frequency, known as the Doppler effect, helps in visualizing blood flow direction and
velocity.
Incorrect options:
Option B - Localizes calcified plaque:
• Doppler ultrasound can identify areas of calcified plaque within blood vessels.
• However, it might not provide as detailed information about the plaque compared to other imaging
techniques like CT scans or angiography.
Option C - Checks blood flow :
Doppler ultrasound is primarily used for assessing blood flow and vascular conditions.
Option D - Checks thickness of artery and vein:
• Doppler ultrasound can be used to measure the thickness of arteries and veins, as well as assess
their overall structure and condition.

Page 17

2094
• It is particularly useful for evaluating conditions like atherosclerosis, where arteries might become
thicker due to plaque buildup.

Page 18

2095
Previous Year Questions
1. The type of knot seen in the image is?

A. Half hitch knot


B. Reef knot
C. Surgeon’s knot
D. Crossed half hitch knot
----------------------------------------
2. What is the size of the given blade used for incision and drainage of a superficial abscess?

A. 10
B. 11
C. 22
D. 15
----------------------------------------
3. In a blast injury, which of the following organs is least vulnerable to the blast wave?
(or)
Which organ is the least susceptible to the blast wave in a blast injury?
A. Gl tract
B. Lungs
C. Liver

2096
D. Ear drum
----------------------------------------
4. A male patient comes with a gradually increasing enlargement for the past one and a half years.
Upon examination, the swelling demonstrates inconsistency in texture and matches the appearance
depicted in the provided image. What is the likely underlying reason for this?

A. Jaw Tumour
B. Dermoid cyst
C. Pleomorphic adenoma
D. Sebaceous cyst
----------------------------------------
5. What is shown in the given image?

A. Dermoid cyst
B. Sebaceous cyst
C. Meningioma
D. Post auricular fistula
----------------------------------------
6. Which specific nerve is affected in Frey's syndrome?
A. Facial Nerve
B. Oculomotor nerve
C. Auriculotemporal nerve

Page 2

2097
D. Glossopharyngeal nerve
----------------------------------------
7. A 56-year-old patient suffering from alcoholic cirrhosis receives a healthy liver from his biological
son. What is this type of transplant known as?
(or)
A 56-year-old patient suffering from alcoholic cirrhosis receives a healthy liver from his biological son.
What is this type of transplant known as?
A. Autograft
B. Allograft
C. Xenograft
D. Isograft
----------------------------------------
8. Which one of the options below is not included in the revised trauma score (RTS)?
A. Glasgow coma scale
B. Systolic blood pressure
C. Pulse rate
D. Respiratory rate
----------------------------------------

Correct Answers
Question Correct Answer

Question 1 2
Question 2 2
Question 3 3
Question 4 3
Question 5 2
Question 6 3
Question 7 2
Question 8 3

Solution for Question 1:


Correct Option:
Option B:
The reef knot that is commonly used in surgical procedures. It is a
secure knot that is used to secure and hold sutures in place.
The reef knot, also known as the square knot, is a
type of knot that is commonly used for joining two ends of a

Page 3

2098
single line together. It is formed by interlocking the ends of the line in opposite directions, creating a
flat, symmetrical knot.
To tie a reef knot, you can follow these steps:

Incorrect Option:
Option A:
Half hitch knot:
The half hitch knot is a simple knot that is created by passing the working end of a
rope over and around a post or another rope and then through the loop created. It is commonly used fo
r temporary or light-duty applications, such as securing a line to a pole or creating a
simple loop. The half hitch knot involves a
single turn around an object, while the reef knot involves interlocking turns of two ends.
Option C:
Surgeon's knot:
The surgeon's knot is a
knot commonly used in surgical procedures and also in various other applications that require a secure
knot. It is an extension of the reef knot, designed to be more secure. The surgeon's knot involves tying
an additional half hitch before completing the knot, which adds more friction and stability. It is often us
ed to secure sutures or tie ligatures.
Option D:
Crossed half hitch knot:
The crossed half hitch knot, also known as the clove hitch knot, is a
knot that is made by crossing the working end of a
rope over the standing part and then passing it under and over itself to form a hitch. It is commonly use
d for securing objects to posts or poles and can be easily adjusted or untied. While it may have similarit
ies in terms of crossed elements with the reef knot, the crossed half hitch knot is fundamentally a
different knot.

Solution for Question 2:


Correct Option:
Option B 11:
The size of the blade commonly used for incision and drainage of a superficial abscess is 11.

Incorrect Options:
Option A 10:
This blade size is larger and may be used for larger incisions or other surgical procedures, but it is not
commonly used for superficial abscess drainage.
Option C. 22:

Page 4

2099
This blade size is typically used for ophthalmic procedures and is not commonly used for superficial ab
scess incision and drainage.
Option D. 15:
This blade size is often used for making small surgical incisions or for procedures requiring precision, b
ut it is not commonly used for superficial abscess drainage.

Solution for Question 3:


Correct Option:
Option A. Gastrointestinal (GI) tract:
The GI tract, including the stomach and intestines, is relatively vulnerable to blast waves. The rapid ch
anges in pressure caused by the blast wave can lead to injury and perforation of the GI tract.

Incorrect Options:
Option B. Lungs:
The lungs are highly vulnerable to blast waves. The rapid changes in pressure can cause lung injuries,
including pneumothorax, pulmonary contusion, and blast lung injury.
Option C. Liver:
The liver is the least vulnerable organ in this context. While it can be affected by the blast wave, it is rel
atively more resistant compared to other organs The shock wave generated by the blast can result in li
ver injury, including contusions, lacerations, and hemorrhage.
Option D. Ear drum:
The ear drum, also known as the tympanic membrane. The ear drum can experience rupture or damag
e due to the sudden changes in air pressure caused by the blast wave.

Solution for Question 4:


Correct Option:
Option C
Pleomorphic adenoma:
Pleomorphic adenoma is a common benign tumor that can occur in the salivary glands, including the p
arotid gland. It often presents as a
slowly growing, painless swelling. The consistency of the tumor can vary depending on its composition.

Option B
Dermoid cyst:
Dermoid cysts are benign cystic masses that can occur in various parts of the body. They are typically
present since birth and can cause a

Page 5

2100
slowly enlarging swelling. Dermoid cysts can have variable consistency and contain a
mixture of different types of tissue.
Option D
Sebaceous cyst:
Sebaceous cysts are benign cystic lesions that arise from the sebaceous glands in the skin. They can
present as slowly growing swellings with a variable consistency. The cysts contain a
thick material called sebum.
Option A
Jaw Tumor:
Jaw tumors can present as slowly progressive swellings in the jaw area. There are various types of tu
mors that can occur in the jaw, including both benign (non-cancerous) and malignant (cancerous) tumo
rs.

Solution for Question 5:


Correct Option:
Option B.
Sebaceous cyst.
Sebaceous cysts are common benign cystic lesions that arise from the sebaceous glands in the skin. T
hey often appear as slow-growing, painless swellings beneath the skin. Sebaceous cysts contain a thic
k, cheesy material called sebum and can occur in various areas of the body, including the face, neck, s
calp, or trunk.

Incorrect Options:
Option A.
Dermoid cyst:
A dermoid cyst is a type of benign cystic lesion that typically contains a
variety of different types of tissues, such as hair, skin, and sometimes even teeth.
These cysts are usually present since birth and can occur in various parts of the body.
Black-colored punctum is seen in cyts except cysts present in scalp &
scrotum as shown in the image in the question
Option C
Meningioma:
Meningiomas are typically slow-growing tumors that arise from the meninges, the protective layers surr
ounding the brain and spinal cord. They are usually found within the cranial cavity and are less commo
nly located near the postauricular area. Meningiomas are not typically associated with the specific app
earance of the image provided.
Option D.
Post auricular fistula:

Page 6

2101
A postauricular fistula is a small abnormal tract or passage that connects the skin behind the ear to the
underlying structures. It can present as a
small opening with drainage or discharge. The image provided does not depict a
postauricular fistula, as it shows a cystic lesion rather than a fistulous opening.

Solution for Question 6:


Correct Option:
Option C
Auriculotemporal nerve
• Frey's syndrome is caused by damage or injury to the auriculotemporal nerve, which is a branch of the
trigeminal nerve (cranial nerve V). This nerve is responsible for providing sensory innervation to the
skin of the temple, external ear, and the parotid gland. When there is damage to this nerve, it can lead
to aberrant reinnervation of sweat glands in the affected area, resulting in gustatory sweating during
eating or other gustatory stimuli.

Incorrect Options:
Option B
Oculomotor nerve:
• The oculomotor nerve (cranial nerve III) is responsible for the motor control of several eye muscles,
including those that control eye movements, pupil constriction, and eyelid elevation. It is not involved in
Frey's syndrome.
Option D
Glossopharyngeal nerve:
• The glossopharyngeal nerve (cranial nerve IX) is responsible for various functions related to the
throat, including the sensation and taste from the posterior third of the tongue, the motor control of
some throat muscles, and the regulation of blood pressure. While it does play a role in taste sensation,
it is not directly involved in Frey's syndrome.
Option A
Facial Nerve:
• The facial nerve (cranial nerve VII) is a mixed nerve responsible for controlling the muscles of facial
expression, transmitting taste sensations from the anterior two-thirds of the tongue, and providing
parasympathetic innervation to the lacrimal and salivary glands.
• In the case of Frey's syndrome, the facial nerve is not directly involved in the syndrome itself.
However, it is associated with the condition due to the proximity and interconnections between the
facial nerve and the auriculotemporal nerve.

Solution for Question 7:


Correct Option:

Page 7

2102
Option B
Allograft
In the given scenario, the patient with alcoholic cirrhosis receives a
liver transplant from his biological son. This type of transplant is known as an allograft.
An allograft refers to the transplantation of an organ or tissue from one individual (the donor) to another
individual (the recipient) who is not genetically identical to the donor. In this case, the recipient receive
d a liver from his biological son, who is not an identical twin. Allografts are the most common type of tra
nsplant performed.

Incorrect Option
Option A.
Autograft:
An autograft involves the transplantation of tissue from one site to another within the same individual. It
refers to a situation where the recipient is also the donor. This is not the case in the given scenario, as
the liver is being transplanted from the biological son.
Option C.
Xenograft:
A xenograft involves the transplantation of tissue or organs between different species. For example, tra
nsplantation of pig organs into humans. This is not the case in the given scenario, as the liver transplan
t is occurring between two humans.
Option D.
Isograft:
An isograft refers to the transplantation of organs or tissues between genetically identical individuals. T
his can occur in the case of identical twins. However, in the given scenario, the recipient and donor are
not genetically identical.

Solution for Question 8:


correct Option:
Option C.
Pulse rate
The Revised Trauma Score (RTS) is a scoring system used to assess the severity of trauma in patient
s. It consists of three components: Glasgow Coma Scale (GCS), Systolic Blood Pressure (SBP), and R
espiratory Rate (RR). However, Pulse Rate is not a part of the RTS.

Incorrect Options:
Option A.
Glasgow Coma Scale (GCS):

Page 8

2103
The GCS assesses the patient's level of consciousness based on eye-opening, verbal response, and
motor response. It is an essential component of the RTS.
Option B.
Systolic Blood Pressure (SBP):
SBP, which measures the pressure exerted on the arterial walls during the contraction phase of the he
art, is another component of the RTS. It helps evaluate the patient's circulatory status.
Option D.
Respiratory rate
Respiratory rate, which refers to the number of breaths a person takes per minute, is indeed a part of t
he Revised Trauma Score (RTS). It is one of the vital signs used to assess the severity of trauma in pa
tients.

Page 9

2104

You might also like